diff --git a/README.md b/README.md index a712d48439270ed2c96a01c9d5dcf3016fef2d6a..475d1a16b16279913abaf409700fb86cc8959805 100644 --- a/README.md +++ b/README.md @@ -1,3 +1,102 @@ ---- -license: cc-by-nd-4.0 ---- +# BlueMO + +## BlueMO: A Comprehensive Collection of Challenging Mathematical Olympiad Problems from the Little Blue Book Series + +**BlueMO** is a comprehensive and challenging dataset comprising mathematical olympiad problems paired with detailed solutions, meticulously curated from the esteemed "Little Blue Book" (小蓝书) series (Second Edition)—a vital resource for Chinese students training for national and international olympiad math competitions. + +Designed to advance and assess sophisticated reasoning in LLMs, this dataset serves as a benchmark or training resource for high-level problem-solving in AI. + +## Introduction for "Little Blue Book" (小蓝书) + +The "Little Blue Book" (小蓝书) series, published by East China Normal University Press, is a cornerstone resource for students striving to master mathematical olympiads. Renowned for its depth, challenging problems, and elegant solutions, the series spans critical domains—Sets, Trigonometric, Geometry, Number Theory, Graph Theory, Extremal Combinatorics—providing rigorous training for olympiad competitions. + +## About Dataset + +BlueMO encompasses a total of 14 volumes from the third edition of the "Little Blue Book" series, covering a wide range of mathematical topics for both middle and high school levels. + +The dataset is structured as follows: + +**High School Collection:** + +* 小蓝书高中卷1 集合 - Little Blue Book High School Vol.1: Sets + +* 小蓝书高中卷2 函数与函数方程 - Little Blue Book High School Vol.2: Functions & Functional Equations + +* 小蓝书高中卷3 三角函数 - Little Blue Book High School Vol.3: Trigonometric Functions + +* 小蓝书高中卷4 平均值不等式与柯西不等式 - Little Blue Book High School Vol.4: Mean Value & Cauchy Inequalities + +* 小蓝书高中卷5 不等式的解题方法与技巧 - Little Blue Book High School Vol.5: Methods & Techniques for Solving Inequalities + +* 小蓝书高中卷6 数列与数学归纳法 - Little Blue Book High School Vol.6: Sequences & Mathematical Induction + +* 小蓝书高中卷7 平面几何 - Little Blue Book High School Vol.7: Plane Geometry + +* 小蓝书高中卷8 复数与向量 - Little Blue Book High School Vol.8: Complex Numbers & Vectors + +* 小蓝书高中卷9 几何不等式 - Little Blue Book High School Vol.9: Geometric Inequalities + +* 小蓝书高中卷10 数论 - Little Blue Book High School Vol.10: Number Theory + +* 小蓝书高中卷11 组合数学 - Little Blue Book High School Vol.11: Combinatorics + +* 小蓝书高中卷12 图论 - Little Blue Book High School Vol.12: Graph Theory + +* 小蓝书高中卷13 组合极值 - Little Blue Book High School Vol.13: Extremal Combinatorics + +* 小蓝书高中卷14 高中数学竞赛中的解题方法与策略 - Little Blue Book High School Vol.14: Problem-Solving Methods & Strategies for Math Competitions + +## Potential Usages + +This dataset is a resource for AI researchers and developers, with key applications including: + +* Training & Fine-Tuning – Enhancing large language models’ capabilities in advanced mathematical reasoning. + +* AI Evaluation – Benchmarking the problem-solving proficiency and logical rigor of AI systems. + +* Formal Verification – Formalizing problems into mathematical languages (e.g., LEAN) to evaluate AI's reasoning capability with formal methods. + +* Comparative Analysis – Systematically assessing reasoning skills across different models and methodologies. + +## How to Use + +We provide the raw data (*.tex) and the processed dataset, including calculation, proof, text and images they referred to. + +A case in `calculation`. + +```json +{ + "source_file": "./raw_volume-zh/volume1/chapter1.tex", + "problem_type": "calculation", + "problem": "例1. 设集合 $M=\\left\\{x |{\\frac{a x-5}{x^{2}-a}}<0,\\,x\\in\\mathbb{R}\\right\\}$ \n(1)当 $a=4$ 时,化简集合 $M$ ;\n(2)若 $3\\in M,$ ,且 $5\\notin M,$ 求实数a的取值范围.", + "solution": "分析: 化简集合 $M$, 实际上就是解不等式 ${\\frac{a x-5}{x^{2}-a}}<0.$ \n解: (1) 当 $a=4$ 时,有\n$$\n{\\frac{4x-5}{x^{2}-4}}<0\\,, \n$$\n即\n$$\n\\left(x-\\frac{5}{4}\\right)(x+2)(x-2)<0. \n$$\n$x<-2$ 或 ${\\frac{5}{4}}9. \n$$\n由 $5\\notin M$ 得, ${\\frac{5a-5}{5^{2}-a}}\\geqslant0$ 或 $5^{2}-a=0$ ,所以\n$$\n1\\leq a\\leq25. \n$$\n可得 $x\\in\\left[1,{\\frac{5}{3}}\\right)\\cup\\left(9,25\\right]$.\n说明: $5\\notin M$ 隐含了条件 $5^{2}-a=$ 0,这是容易被忽视的.\n由概括原则我们知道,判断一个对象 $x$ 是否为集合 $S$ 的元素,等价于判断 $x$ 是否具有性质 $P$.", + "remark": "", + "figures": [] +} +``` + +`source_file`: Path to the original .tex source file containing this problem. + +`problem_type`: Problem category (e.g., "calculation", "proof", etc.). + +`problem`: Complete problem statement in LaTeX format, including sub-questions. + +`solution`: Step-by-step solution with mathematical derivations in LaTeX. + +`remark`: Additional notes or comments about the problem (empty if none). + +`figures`: Array containing any associated diagram files (empty if none). + + +## Citation + + +If you use the BlueMO dataset in your research, please consider citing it as follows: + +```bibtex +@misc{bluemo2024, + title={BlueMO: A comprehensive collection of challenging mathematical olympiad problems from the little blue book series}, + author={Chen, Yizhou and Luo, Yifan and Zhang, Yifan and Yuan, Yang}, + year={2024}, +} +``` diff --git a/processed_dataset/calculation/0001.json b/processed_dataset/calculation/0001.json new file mode 100644 index 0000000000000000000000000000000000000000..882258b5395cfe1ae58a54b5afa0031e580d8bb5 --- /dev/null +++ b/processed_dataset/calculation/0001.json @@ -0,0 +1,8 @@ +{ + "source_file": "./raw_volume-zh/volume1/chapter1.tex", + "problem_type": "calculation", + "problem": "例1. 设集合 $M=\\left\\{x |{\\frac{a x-5}{x^{2}-a}}<0,\\,x\\in\\mathbb{R}\\right\\}$ \n(1)当 $a=4$ 时,化简集合 $M$ ;\n(2)若 $3\\in M,$ ,且 $5\\notin M,$ 求实数a的取值范围.", + "solution": "分析: 化简集合 $M$, 实际上就是解不等式 ${\\frac{a x-5}{x^{2}-a}}<0.$ \n解: (1) 当 $a=4$ 时,有\n$$\n{\\frac{4x-5}{x^{2}-4}}<0\\,, \n$$\n即\n$$\n\\left(x-\\frac{5}{4}\\right)(x+2)(x-2)<0. \n$$\n$x<-2$ 或 ${\\frac{5}{4}}9. \n$$\n由 $5\\notin M$ 得, ${\\frac{5a-5}{5^{2}-a}}\\geqslant0$ 或 $5^{2}-a=0$ ,所以\n$$\n1\\leq a\\leq25. \n$$\n可得 $x\\in\\left[1,{\\frac{5}{3}}\\right)\\cup\\left(9,25\\right]$.\n说明: $5\\notin M$ 隐含了条件 $5^{2}-a=$ 0,这是容易被忽视的.\n由概括原则我们知道,判断一个对象 $x$ 是否为集合 $S$ 的元素,等价于判断 $x$ 是否具有性质 $P$.", + "remark": "", + "figures": [] +} \ No newline at end of file diff --git a/processed_dataset/calculation/0002.json b/processed_dataset/calculation/0002.json new file mode 100644 index 0000000000000000000000000000000000000000..42f7ed9bc0d163bdceaee17cc505582b51fb7f8a --- /dev/null +++ b/processed_dataset/calculation/0002.json @@ -0,0 +1,8 @@ +{ + "source_file": "./raw_volume-zh/volume1/chapter1.tex", + "problem_type": "calculation", + "problem": "例4. 设关于 $x$ 的不等式 $\\left|x-{\\frac{(a+1)^{2}}{2}}\\right|\\leq{\\frac{(a-1)^{2}}{2}}$ 和 $x^{2}-3(a+1)x+2(3a+1)\\leq0\\ (a\\in\\mathbb{R})$ 的解集依次为 $A$、$B$,求使 $A\\,\\subseteq\\,B$ 的实数a 的取值范围.", + "solution": "分析: 要由 $A\\subseteq B$ 求出a的范围,必须先求出$A$和 $B$.\n解: 由 $\\left|x-{\\frac{(a+1)^{2}}{2}}\\right|\\leqslant{\\frac{(a-1)^{2}}{2}}$, 得\n$$\n-\\frac{(a-1)^{2}}{2}\\leq x-\\frac{(a+1)^{2}}{2}\\leq\\frac{(a-1)^{2}}{2}, \n$$\n解之,得 $2a\\leq x\\leq a^{2}+1.$ 所以 $,A=\\{x\\mid2a\\leq x\\leq a^{2}+1\\}$ \n由 $x^{2}-3(a+1)x+2(3a+1)\\leq0$,得\n$$\n(x-2)[x-(3a+1)]\\leq0. \n$$\n当 $a\\geq{\\frac{1}{3}}$ 时, $B = \\{ x \\mid 2 \\leq x \\leq 3a+1 \\}$ ;当 $a<{\\frac{1}{3}}$时,$B=\\{x \\mid 3a+1 \\leq x \\leq 2 \\}.$ \n因为 $A\\subseteq B$, 所以\n$$\n\\begin{align*}\n\\left\\{\n\\begin{aligned}\n a \\geq \\frac{1}{3}, \\\\\n 2a \\geq 2,\\\\\n a^2+1 \\leq 3a+1,\n\\end{aligned}\n\\right.\n\\end{align*}\n$$ \n或\n$$\n\\begin{align*}\n\\left\\{\n\\begin{aligned}\n a < \\frac{1}{3}, \\\\\n 2a \\geq 3a+1,\\\\\n a^2+1 \\leq 2.\n\\end{aligned}\n\\right.\n\\end{align*}\n$$ \n解之,得 $1\\leq a\\leq3$ 或 $a=-1.$ \n所以,a 的取值范围是[1, 3]U ${\\{-1}\\}.$ \n说明: 上述解答是通过对参数 $a$ 的分类讨论完成的,其实还有更直接的解法.\n方程的角度看 $A\\subseteq B$ 等价于方程 $x^{2}-3(a+1)x+2(3a+1)=0$ 在区间$(-\\infty,2a]$ 和 $[a^{2}+1,\\ +\\infty)$ 内各有一个实根.\n $f(x)\\,=\\,x^{2} - 3(a+1)x+2(3a+1)$ ,由 $A\\subseteq B$, 得\n$$\n\\begin{align*}\n\\left\\{\n\\begin{aligned}\n f(2a) \\leq 0, \\\\\n f(a^2+1) \\leq 0,\\\\\n\\end{aligned}\n\\right.\n\\end{align*}\n\\longrightarrow 1\\leq a \\leq 3 \\text{或} a=-1.$$", + "remark": "", + "figures": [] +} \ No newline at end of file diff --git a/processed_dataset/calculation/0003.json b/processed_dataset/calculation/0003.json new file mode 100644 index 0000000000000000000000000000000000000000..20f987e1b4c5d7f2ce89239768a9c81adda27116 --- /dev/null +++ b/processed_dataset/calculation/0003.json @@ -0,0 +1,8 @@ +{ + "source_file": "./raw_volume-zh/volume1/chapter1.tex", + "problem_type": "calculation", + "problem": "例5. 设实数$a-1 、 q<-2$.", + "remark": "", + "figures": [] +} \ No newline at end of file diff --git a/processed_dataset/calculation/0006.json b/processed_dataset/calculation/0006.json new file mode 100644 index 0000000000000000000000000000000000000000..08ef6e46578f6834147a0f6c8f0716073178ffa3 --- /dev/null +++ b/processed_dataset/calculation/0006.json @@ -0,0 +1,10 @@ +{ + "source_file": "./raw_volume-zh/volume1/chapter2.tex", + "problem_type": "calculation", + "problem": "例3. 设 $A 、 B$ 都是不超过 9 的正整数组成的全集 $U$ 的子集, $A \\cap B= \\{2\\},\\left(\\complement_U A\\right) \\cap\\left(\\complement_U B\\right)=\\{1,9\\},\\left(\\complement_U A\\right) \\cap B=\\{4,6,8\\}$, 求 $A \\backslash B$.", + "solution": "分析:直接进行集合间的运算和推理似乎较难人手, 但我们可从维恩图()中得到解题思路的提示.\n解因为 $\\complement_U(A \\cup B)=\\left(\\complement_U A\\right) \\cap\\left(\\complement_U B\\right)= \\{1,9\\}$, 所以\n$$\nA \\cup B=\\{2,3,4,5,6,7,8\\} .\n$$\n又\n$$\n\\begin{gathered}\nA \\cap B=\\{2\\}, \\\\\n\\left(\\complement_U A\\right) \\cap B=\\{4,6,8\\}, \\\\\nB=U \\cap B=\\left(A \\cup \\complement_U A\\right) \\cap B \\\\\n=(A \\cap B) \\cup\\left(\\left(\\complement_U A\\right) \\cap B\\right) \\\\\n=\\{2,4,6,8\\} .\n\\end{gathered}\n$$\n所以\n$$\n\\begin{aligned}\nB & =U \\cap B=\\left(A \\cup \\complement_U A\\right) \\cap B \\\\\n& =(A \\cap B) \\cup\\left(\\left(\\complement_U A\\right) \\cap B\\right) \\\\\n& =\\{2,4,6,8\\} .\n\\end{aligned}\n$$\n所以, $A \\backslash B=(A \\cup B) \\backslash B=\\{3,5,7\\}$.", + "remark": "", + "figures": [ + "./images/volume1/figures/fig-c2e3.png" + ] +} \ No newline at end of file diff --git a/processed_dataset/calculation/0007.json b/processed_dataset/calculation/0007.json new file mode 100644 index 0000000000000000000000000000000000000000..c88a8f9f24dd0b79759b9b59d7ac5deef62da76a --- /dev/null +++ b/processed_dataset/calculation/0007.json @@ -0,0 +1,8 @@ +{ + "source_file": "./raw_volume-zh/volume1/chapter2.tex", + "problem_type": "calculation", + "problem": "例4. 已知集合 $A=\\{(x, y) \\mid a x+y=1\\}, B=\\{(x, y) \\mid x+a y=1\\}$, $C=\\left\\{(x, y) \\mid x^2+y^2=1\\right\\}$. 问:\n(1) 当 $a$ 取何值时, $(A \\cup B) \\cap C$ 为含有两个元素的集合?\n(2) 当 $a$ 取何值时, $(A \\cup B) \\cap C$ 为含有三个元素的集合?", + "solution": "分析:因为 $(A \\cup B) \\cap C=(A \\cap C) \\cup(B \\cap C)$, 故可从解 $A \\cap C$ 及 $B \\cap C$ 对应的方程组人手.\n解: $(A \\cup B) \\cap C=(A \\cap C) \\cup(B \\cap C), A \\cap C$ 与 $B \\cap C$ 分别为方程组\n(i) $\\left\\{\\begin{array}{l}a x+y=1, \\\\ x^2+y^2=1,\\end{array}\\right.$\n(ii) $\\left\\{\\begin{array}{l}x+a y=1, \\\\ x^2+y^2=1\\end{array}\\right.$\n的解集.\n由 (i) 解得 $(x, y)=(0,1),\\left(\\frac{2 a}{1+a^2}, \\frac{1-a^2}{1+a^2}\\right)$;\n由(ii) 解得 $(x, y)=(1,0),\\left(\\frac{1-a^2}{1+a^2}, \\frac{2 a}{1+a^2}\\right)$.\n(1) 使 $(A \\cup B) \\cap C$ 恰有两个元素的情况只有两种可能:\n<1> $\\left\\{\\begin{array}{l}\\frac{2 a}{1+a^2}=0, \\\\ \\frac{1-a^2}{1+a^2}=1 ;\\end{array}\\right.$\n<2> $\\left\\{\\begin{array}{l}\\frac{2 a}{1+a^2}=1, \\\\ \\frac{1-a^2}{1+a^2}=0 .\\end{array}\\right.$\n由<1>得 $a=0$; 由<2>得 $a=1$.\n故当 $a=0$ 或 1 时, $(A \\cup B) \\cap C$ 恰有两个元素.\n(2) 使 $(A \\cup B) \\cap C$ 恰有三个元素的情况是\n$$\n\\frac{2 a}{1+a^2}=\\frac{1-a^2}{1+a^2}\n$$\n解得 $a=-1 \\pm \\sqrt{2}$.\n故当 $a=-1 \\pm \\sqrt{2}$ 时, $(A \\cup B) \\cap C$ 恰有三个元素.", + "remark": "", + "figures": [] +} \ No newline at end of file diff --git a/processed_dataset/calculation/0008.json b/processed_dataset/calculation/0008.json new file mode 100644 index 0000000000000000000000000000000000000000..0e2de24b7f59061f16c2045d0faaa22ae4bf0a85 --- /dev/null +++ b/processed_dataset/calculation/0008.json @@ -0,0 +1,8 @@ +{ + "source_file": "./raw_volume-zh/volume1/chapter2.tex", + "problem_type": "calculation", + "problem": "例5. 已知集合 $A=\\left\\{(x, y) \\mid \\frac{y-3}{x-2}=a+1\\right\\}, B=\\{(x, y) \\mid(a^2- 1) x+(a-1) y=15\\}$, 且 $A \\cap B=\\varnothing$, 求 $a$ 的值.", + "solution": "分析:当 $a=1$ 时, $B=\\varnothing$, 这时 $A \\cap B=\\varnothing$; 当 $a \\neq 1$ 时, $A \\cap B=\\varnothing$, 即 $A 、 B$ 对应的直线无公共点.\n解由 $\\frac{y-3}{x-2}=a+1$, 得\n$$\n(a+1) x-y-2 a+1=0, \\text { 且 } x \\neq 2 .\n$$\n这表明集合 $A$ 表示一条缺一个点的直线.\n而\n$$\n\\left(a^2-1\\right) x+(a-1) y=15,\n$$\n当 $a \\neq 1$ 时,表示一条直线; 当 $a=1$ 时,满足等式的点 $(x, y)$ 不存在.\n因此,当且仅当以下三种情况之一发生时, $A \\cap B=\\varnothing$.\n(1)当 $a=1$ 时, $B=\\varnothing$, 显然有 $A \\cap B=\\varnothing$.\n(2)当 $a=-1$ 时, $A$ 表示直线 $y=3(x \\neq 2), B$ 表示直线 $y=-\\frac{15}{2}$, 它们互相平行.\n所以, $A \\cap B=\\varnothing$.\n(3)当 $a \\neq \\pm 1$ 时, 直线 (1) 与 (2) 相交.\n但直线 (1) 上缺一点 $(2,3)$, 令 $(2, 3) \\in B$, 得\n$$\n\\left(a^2-1\\right) \\cdot 2+(a-1) \\cdot 3=15,\n$$\n解得 $a=-4$ 或 $a=\\frac{5}{2}$.\n综上所述, $a \\in\\left\\{-4,-1,1, \\frac{5}{2}\\right\\}$.\n说明 $a \\neq 1$ 时, $A \\cap B=\\varnothing$, 并不表明直线 (1) 与 (2) 必须平行, 由于直线 (1) 上缺了一个点 $(2,3)$, 当直线 (2) 穿过点 $(2,3)$ 时, 同样有 $A \\cap B=\\varnothing$.", + "remark": "", + "figures": [] +} \ No newline at end of file diff --git a/processed_dataset/calculation/0009.json b/processed_dataset/calculation/0009.json new file mode 100644 index 0000000000000000000000000000000000000000..b5517bf4ca4676bf8d04f6a89e5ae02fcc9522e3 --- /dev/null +++ b/processed_dataset/calculation/0009.json @@ -0,0 +1,10 @@ +{ + "source_file": "./raw_volume-zh/volume1/chapter2.tex", + "problem_type": "calculation", + "problem": "例7. 已知集合 $A 、 B 、 C$ (不必相异)的并集\n$$\nA \\cup B \\cup C=\\{1,2, \\cdots, 2005\\},\n$$\n求满足条件的有序三元组 $(A, B, C)$ 的个数.", + "solution": "解: 由图()可知,表示集合 $A 、 B 、 C$ 的 3 个圆交出了 7 个区域.\n这表明,在求 $A \\cup B \\cup C$ 时, 1 , $2, \\cdots, 2005$ 中每一个数都有 7 种选择.\n所以,所求的有序三元组的个数为 $7^{2005}$.", + "remark": "", + "figures": [ + "./images/volume1/figures/fig-c2e7.png" + ] +} \ No newline at end of file diff --git a/processed_dataset/calculation/0010.json b/processed_dataset/calculation/0010.json new file mode 100644 index 0000000000000000000000000000000000000000..afb508c89165865da91017f399debd39aeae75e4 --- /dev/null +++ b/processed_dataset/calculation/0010.json @@ -0,0 +1,8 @@ +{ + "source_file": "./raw_volume-zh/volume1/chapter3.tex", + "problem_type": "calculation", + "problem": "例1. 设集合 $A=\\left\\{(x, y, z) \\mid \\log _{\\frac{1}{4}}\\left(x^4+y^4+z^4+1\\right) \\geqslant \\log _4 \\frac{1}{x}+\\log _4 \\frac{1}{y}+\\log _4 \\frac{1}{z}-1\\right\\}$. 求 $|A|$.", + "solution": "分析:无疑应从考察 $(x, y, z)$ 满足的条件人手.\n解由 $\\log _{\\frac{1}{4}}\\left(x^4+y^4+z^4+1\\right) \\geqslant \\log _4 \\frac{1}{x}+\\log _4 \\frac{1}{y}+\\log _4 \\frac{1}{z}-1$ 得\n$$\nx^4+y^4+z^4+1 \\leqslant 4 x y z, x, y, z>0 .\n$$\n又由算术几何平均不等式, 得\n$$\nx^4+y^4+z^4+1 \\geqslant 4 x y z,\n$$\n其中等号当且仅当 $x=y=z=1$ 时成立.\n于是\n$$\n\\begin{gathered}\nx^4+y^4+z^4+1=4 x y z . \\\\\nx=y=z=1 .\n\\end{gathered}\n$$\n从而所以, $|A|=1$.", + "remark": "", + "figures": [] +} \ No newline at end of file diff --git a/processed_dataset/calculation/0011.json b/processed_dataset/calculation/0011.json new file mode 100644 index 0000000000000000000000000000000000000000..7e0856bd0ed503ed756e293d8bf4b303a8e29a9a --- /dev/null +++ b/processed_dataset/calculation/0011.json @@ -0,0 +1,8 @@ +{ + "source_file": "./raw_volume-zh/volume1/chapter3.tex", + "problem_type": "calculation", + "problem": "例2. 设集合 $A=\\{a \\mid 1 \\leqslant a \\leqslant 2000, a=4 k+1, k \\in \\mathbf{Z}\\}$, 集合 $B= \\{b \\mid 1 \\leqslant b \\leqslant 3000, b=3 k-1, k \\in \\mathbf{Z}\\}$. 求 $|A \\cap B|$.", + "solution": "分析:令 $4 k+1=3 m-1$, 得 $m=\\frac{4 k+2}{3}=k+1+\\frac{k-1}{3}$. 因 $m \\in \\mathbf{Z}$, 所以 $3 \\mid k-1$. 令 $k-1=3 r, r \\in \\mathbf{Z}$, 得 $m=4 r+2$. 这时 $b=12 r+5$, 故 $A \\cap B$的元素是形如 $12 r+5$ 的整数.\n解形如 $4 k+1$ 的数可分为 3 类:\n$$\n12 l+1,12 l+5,12 l+9(l \\in \\mathbf{Z}),\n$$\n其中只有形如 $12 l+5$ 的数是形如 $3 k-1$ 的数.\n令\n$$\n1 \\leqslant 12 l+5 \\leqslant 2000(l \\in \\mathbf{Z}),\n$$\n得 $0 \\leqslant l \\leqslant 166$. 所以, $A \\cap B=\\{5,17, \\cdots, 1997\\}$.\n所以, $|A \\cap B|=167$.\n以上两例, 我们都是采用列举出集合的全部元素的办法来求其元素的数目.\n对于一些较为复杂的集合, 这种方法是很难奏效的, 这时必须另辟蹊径.", + "remark": "", + "figures": [] +} \ No newline at end of file diff --git a/processed_dataset/calculation/0012.json b/processed_dataset/calculation/0012.json new file mode 100644 index 0000000000000000000000000000000000000000..5a2451dcdc23beb4ed29553b04c100686048c02d --- /dev/null +++ b/processed_dataset/calculation/0012.json @@ -0,0 +1,8 @@ +{ + "source_file": "./raw_volume-zh/volume1/chapter3.tex", + "problem_type": "calculation", + "problem": "例3. 设 $\\left(a_1, a_2, \\cdots, a_n\\right)$ 是集合 $\\{1,2, \\cdots, n\\}$ 中 $n$ 个元素的一个排列, 记所有满足\n$$\nk \\mid 2\\left(a_1+a_2+\\cdots+a_k\\right), k=1,2, \\cdots, n\n$$\n的排列 $\\left(a_1, a_2, \\cdots, a_n\\right)$ 的集合为 $A_n$. 求 $\\left|A_n\\right|$ 的值.", + "solution": "分析:显然 $1\\left|2 a_1, n\\right| 2\\left(a_1+a_2+\\cdots+a_n\\right)$, 我们需要研究当 $2 \\leqslant k \\leqslant n-1$ 时, $k \\mid 2\\left(a_1+a_2+\\cdots+a_k\\right)$ 应满足的条件.\n对于一般的 $k$, 我们没有更好的办法来表示 $a_1+a_2+\\cdots+a_k$, 但当 $k=n-1$ 时, 显然有 $a_1+a_2+\\cdots+ a_{n-1}=1+2+\\cdots+n-a_n {=} \\frac{n(n+1)}{2}-a_n$, 于是 $n-1 \\mid 2\\left(a_1+a_2+\\cdots+a_{n-1}\\right)$ 等价于 $n-1 \\mid n(n+1)-2 a_n$, 问题转化为对 $a_n$ 的研究.\n解设 $F_n=\\left|A_n\\right|$. 容易算出 $F_1=1, F_2=2, F_3=6$.\n当 $n>3$ 时,对于任意 $\\left(a_1, a_2, \\cdots, a_n\\right) \\in A_n$, 有\n$$\n\\begin{aligned}\n& 2\\left(a_1+a_2+\\cdots+a_{n-1}\\right) \\\\\n= & n(n+1)-2 a_n \\equiv 2-2 a_n(\\bmod (n-1)) .\n\\end{aligned}\n$$\n由 $A_n$ 的定义, 必有\n$$\nn-1 \\mid 2-2 a_n \\text {. }\n$$\n故 $a_n=1$,或 $a_n=n$, 或 $a_n=\\frac{n+1}{2}$.\n(1) 若 $a_n=\\frac{n+1}{2}$, 则\n$$\n\\begin{aligned}\n2\\left(a_1+a_2+\\cdots+a_{n-2}\\right) & =n(n+1)-2 a_{n-1}-(n+1) \\\\\n& =n^2-1-2 a_{n-1} \\\\\n& \\equiv 3-2 a_{n-1}(\\bmod (n-2)) .\n\\end{aligned}\n$$\n从而有\n$$\nn-2 \\mid 3-2 a_{n-1} .\n$$\n解得 $a_{n-1}=\\frac{n+1}{2}$. 于是 $a_{n-1}=a_n$, 矛盾.\n(2) 若 $a_n=n$, 则 $\\left(a_1, a_2, \\cdots, a_{n-1}, n\\right)$ 与 $A_{n-1}$ 的元素 $\\left(a_1, a_2, \\cdots, a_{n-1}\\right)$ 形成一一对应关系.\n所以, 这样的排列共有 $F_{n-1}$ 种.\n(3) 若 $a_n=1$, 则 $\\left(a_1-1, a_2-1, \\cdots, a_{n-1}-1\\right)$ 是集合 $\\{1,2, \\cdots, n-1\\}$ 中 $n-1$ 个元素的一个排列.\n由\n$$\n\\begin{aligned}\n& 2\\left[\\left(a_1-1\\right)+\\left(a_2-1\\right)+\\cdots+\\left(a_k-1\\right)\\right] \\\\\n= & 2\\left(a_1+a_2+\\cdots+a_k\\right)-2 k \\\\\n\\equiv & 0(\\bmod k) \\\\\n\\Leftrightarrow & \\left(a_1-1, a_2-1, \\cdots, a_{n-1}-1\\right) \\in A_{n-1}\n\\end{aligned}\n$$\n知 $\\left(a_1, a_2, \\cdots, a_{n-1}, 1\\right)$ 与 $A_{n-1}$ 的元素 $\\left(a_1-1, a_2-1, \\cdots, a_{n-1}-1\\right)$ 之间也形成一一对应关系.\n故这样的排列也有 $F_{n-1}$ 种.\n由(2)、(3), 可建立递推关系\n$$\nF_n=2 F_{n-1}, n>3 .\n$$\n由 $F_3=6$, 得 $F_n=3 \\cdot 2^{n-2}(n \\geqslant 3)$.\n综上, 当 $n=1$ 时, $F_1=1$; 当 $n=2$ 时, $F_2=2$; 当 $n \\geqslant 3$ 时, $F_n=3 \\cdot 2^{n-2}$.\n说明这里, 我们通过建立 $F_n$ 与 $F_{n-1}$ 之间的联系 (递推关系) 来达到求解的目的。", + "remark": "", + "figures": [] +} \ No newline at end of file diff --git a/processed_dataset/calculation/0013.json b/processed_dataset/calculation/0013.json new file mode 100644 index 0000000000000000000000000000000000000000..1c6117bee5068293b0afd01c9df4d3252c352691 --- /dev/null +++ b/processed_dataset/calculation/0013.json @@ -0,0 +1,8 @@ +{ + "source_file": "./raw_volume-zh/volume1/chapter3.tex", + "problem_type": "calculation", + "problem": "例8. 设 $S$ 是一个由正整数组成的集合, 具有如下性质: 对任意 $x \\in S$, 在 $S$ 中去掉 $x$ 后, 剩下的数的算术平均值都是正整数, 并且 $1 \\in S, 2002$ 是 $S$ 中的最大元.\n求 $|S|$ 的最大值.", + "solution": "分析:显然 1 是 $S$ 中的最小元.\n设 $S$ 的元素为 $1=x_1a>b$, 且 $a-b \\geqslant 14$. 故 $b1+\\frac{14}{42}=\\frac{4}{3} .\n$$\n所以, $m \\geqslant 56$.\n后同前解.", + "remark": "", + "figures": [] +} \ No newline at end of file diff --git a/processed_dataset/calculation/0018.json b/processed_dataset/calculation/0018.json new file mode 100644 index 0000000000000000000000000000000000000000..385d96419318ec45ef7aff3be475ec68e1827928 --- /dev/null +++ b/processed_dataset/calculation/0018.json @@ -0,0 +1,8 @@ +{ + "source_file": "./raw_volume-zh/volume1/chapter4.tex", + "problem_type": "calculation", + "problem": "例10. 设 $A=\\{1,2, \\cdots, 2002\\}, M=\\{1001,2003,3005\\}$. 对 $A$ 的任一非空子集 $B$, 当 $B$ 中任意两数之和不属于 $M$ 时, 称 $B$ 为 $M$-自由集.\n如果 $A= A_1 \\cup A_2, A_1 \\cap A_2=\\varnothing$, 且 $A_1 、 A_2$ 均为 $M$-自由集, 那么称有序对 $\\left(A_1, A_2\\right)$ 为 $A$ 的一个 $M$-划分.\n试求 $A$ 的所有 $M$-划分的个数.", + "solution": "解:对 $m, n \\in A$, 若 $m+n=1001$ 或 2003 或 3005 , 则称 $m$ 与 $n$ “有关”.\n易知, 与 1 有关的数仅有 1000 和 2002 ,与 1000 和 2002 有关的都是 1 和 1003 , 与 1003 有关的为 1000 和 2002 .\n将 $1,1003,1000,2002$ 分为两组 $\\{1,1003\\},\\{1000,2002\\}$, 其中一组中的数仅与另一组中的数有关, 我们将这样的两组叫做一个 “组对”. 同样可划分其他各组对 $\\{2,1004\\},\\{999,2001\\} ;\\{3,1005\\},\\{998,2000\\} ; \\cdots ;\\{500,1502\\},\\{501,1503\\} ;\\{1001\\},\\{1002\\}$.\n这样 $A$ 中的 2002 个数被分划成 501 个组对,共 1002 组.\n由于任意数与且只与对应另一组有关, 所以, 若一组对中一组在 $A_1$ 中, 另一组必在 $A_2$ 中.\n反之亦然, 且 $A_1$ 与 $A_2$ 中不再有有关的数.\n故 $A$ 的 $M$ 一划分的个数为 $2^{501}$.", + "remark": "", + "figures": [] +} \ No newline at end of file diff --git a/processed_dataset/calculation/0019.json b/processed_dataset/calculation/0019.json new file mode 100644 index 0000000000000000000000000000000000000000..e81366e28edb96802a97f2693845db1a82ac8def --- /dev/null +++ b/processed_dataset/calculation/0019.json @@ -0,0 +1,8 @@ +{ + "source_file": "./raw_volume-zh/volume1/chapter5.tex", + "problem_type": "calculation", + "problem": "例4. 已知集合 $A=\\{1,2, \\cdots, 10\\}$. 求集合 $A$ 的具有下列性质的子集个数: 每个子集至少含有 2 个元素, 且每个子集中任何两个元素的差的绝对值大于 1 .", + "solution": "分析:集合 $A$ 有 $2^{10}-1$ 个非空子集,逐一考察的工作只有交给计算机.\n像例 1 一样, 我们先来看看比 $A$ 的元素少一些的集合的情形.\n记集合 $A_i$ 符合条件的子集族为 $A_i^*,\\left|A_i^*\\right|=a_i$.\n$$\n\\begin{aligned}\nA_1 & =\\{1\\}, A_1^*=\\varnothing, a_1=0 ; \\\\\nA_2 & =\\{1,2\\}, A_2^*=\\varnothing, a_2=0 ; \\\\\nA_3 & =\\{1,2,3\\}, A_3^*=\\{\\{1,3\\}\\}, a_3=1 ; \\\\\nA_4 & =\\{1,2,3,4\\}, A_4^*=\\{\\{1,3\\},\\{1,4\\},\\{2,4\\}\\}, a_4=3 ; \\\\\nA_5 & =\\{1,2,3,4,5\\}, A_5^*=\\{\\{1,3\\},\\{1,4\\},\\{2,4\\},\\{1,3,5\\}, \\{1,5\\},\\{2,5\\},\\{3,5\\}\\}, a_5=7 . &\n\\end{aligned}\n$$\n我们来考察写出 $A_5^*$ 的过程, 这可以分作两步: 第一步写出 $A_4^*$ 的全部元素, 它们都不含元素 5 ; 第二步写出含 5 的子集, 它们是在 $A_3^*$ 的元素中添 5 所成, 或者是含 5 的二元子集, 即 $a_5=a_4+a_3+3$. 其实对 $A_4^* 、 A_3^*$ 有类似的结论: $a_4=a_3+a_2+2, a_3=a_2+a_1+1$. 我们可以将这个作法推广到一般。\n解设 $a_n$ 是集合 $\\{1,2, \\cdots, n\\}$ 的具有题设性质的子集个数.\n对于集合 $\\{1,2, \\cdots, n, n+1, n+2\\}$, 具有题设性质的子集可分为两类: 第一类子集不包含 $n+2$, 它们是集合 $\\{1,2, \\cdots, n, n+1\\}$ 的全部具有题设性质的子集, 共有 $a_{n-1}$ 个; 第二类子集包含 $n+2$, 它们是集合 $\\{1,2, \\cdots, n\\}$ 的每个具有题设性质的子集与 $\\{n+2\\}$ 的并集, 以及二元子集 $\\{1, n+2\\},\\{2$, $n+2\\}, \\cdots,\\{n, n+2\\}$, 共有 $a_n+n$ 个.\n于是, 我们有\n$$\na_{n+2}=a_{n+1}+a_n+n .\n$$\n易知, $a_1=a_2=0$, 因此 $a_3=1, a_4=3, a_5=7, a_6=14, a_7=26$, $a_8=46, a_9=79, a_{10}=133$.\n所以,所求子集的个数为 133 .\n说明上述解法的特点是将问题一般化,一般问题解决了, 特殊问题当然就解决了.\n这里用到了递推方法, 递推也是解决组合问题的常用方法之一.\n与上例相反,我们来看一个已知子集族求恰好包含这些子集的集合的阶的问题.", + "remark": "", + "figures": [] +} \ No newline at end of file diff --git a/processed_dataset/calculation/0020.json b/processed_dataset/calculation/0020.json new file mode 100644 index 0000000000000000000000000000000000000000..45a06ff128c13a78509e6e894ecd3e201761be6e --- /dev/null +++ b/processed_dataset/calculation/0020.json @@ -0,0 +1,8 @@ +{ + "source_file": "./raw_volume-zh/volume1/chapter5.tex", + "problem_type": "calculation", + "problem": "例8. 设 $A \\subseteq\\{0,1,2, \\cdots, 29\\}$ 满足:对任何整数 $k$ 及 $A$ 中任意数 $a 、 b$ ( $a 、 b$ 可以相同), $a+b+30 k$ 均不是两个相邻整数之积.\n试定出所含元素个数最多的 $A$.", + "solution": "分析:因为当 $b=a$ 时, $2 a+30 k$ 均不是两个相邻整数之积, 故我们只需考察 $2 a$ 被 30 除的余数.\n解所求 $A$ 为 $\\{3 l+2 \\mid 0 \\leqslant l \\leqslant 9\\}$.\n设 $A$ 满足题中条件且 $|A|$ 最大.\n因为两个相邻整数之积被 30 除, 余数为 $0,2,6,12,20,26$. 则对任一 $a \\in A$, 有 $2 a \\neq 0,2,6,12,20,26(\\bmod 30)$ ,\n即 $a \\neq 0,1,3,6,10,13,15,16,18,21,25,28$, 因此, $A \\subseteq\\{2,4,5,7,8,9,11,12,14,17,19,20,22,23,24,26,27,29\\}$, 后一集合可分拆成下列 10 个子集的并, 其中每一个子集至多有一个元素包含在 $A$ 中: $\\{2,4\\}, \\{5,7\\},\\{8,12\\},\\{9,11\\},\\{14,22\\},\\{17,19\\},\\{20\\},\\{23,27\\},\\{24,26\\},\\{29\\}$, 故 $|A| \\leqslant 10$.\n若 $|A|=10$, 则每个子集恰好有一个元素包含在 $A$ 中, 因此, $20 \\in A, 29 \\in A$.\n由 $20 \\in A$ 知 $12 \\notin A$, 从而 $8 \\in A$, 这样 $4 \\notin A, 22 \\notin A, 24 \\notin A$. 因此 $2 \\in A, 14 \\in A, 26 \\in A$.\n由 $29 \\in A$ 知 $7 \\notin A, 27 \\notin A$, 从而 $5 \\in A, 23 \\in A$, 这样 $9 \\notin A, 19 \\notin A$, 因此 $11 \\in A, 17 \\in A$.\n综上所述,有 $A=\\{2,5,8,11,14,17,20,23,26,29\\}$, 此集合 $A$ 确实满足要求.", + "remark": "", + "figures": [] +} \ No newline at end of file diff --git a/processed_dataset/calculation/0021.json b/processed_dataset/calculation/0021.json new file mode 100644 index 0000000000000000000000000000000000000000..8f31c6584a6e5368850122f8c2c316d441824235 --- /dev/null +++ b/processed_dataset/calculation/0021.json @@ -0,0 +1,8 @@ +{ + "source_file": "./raw_volume-zh/volume1/chapter7.tex", + "problem_type": "calculation", + "problem": "例1. 已知 $y=\\frac{x^2}{10}-\\frac{x}{10}+\\frac{9}{5}$, 且 $y \\leqslant|x|$, 求 $x$ 的取值范围.", + "solution": "分析:为了去掉 $y \\leqslant|x|$ 中 $|x|$ 的绝对值符号, 自然要对 $x$ 进行分类: 当 $x \\geqslant 0$ 时, $y \\leqslant x$; 当 $x<0$ 时, $y \\leqslant-x$. 由此知, 本题应分两种情况讨论.\n解当 $x \\geqslant 0$ 时, 有 $y \\leqslant x$. 即亦即\n$$\n\\begin{aligned}\n& \\frac{x^2}{10}-\\frac{x}{10}+\\frac{9}{5} \\leqslant x, \\\\\n& (x-2)(x-9) \\leqslant 0 .\n\\end{aligned}\n$$\n解得 $2 \\leqslant x \\leqslant 9$.\n当 $x<0$ 时, 有 $y \\leqslant-x$. 即\n$$\n\\begin{aligned}\n& \\frac{x^2}{10}-\\frac{x}{10}+\\frac{9}{5} \\leqslant-x, \\\\\n& (x+3)(x+6) \\leqslant 0 .\n\\end{aligned}\n$$\n解得 $-6 \\leqslant x \\leqslant-3$.\n综上,所求 $x$ 的取值范围为 $[-6,-3] \\cup[2,9]$.\n说明以上解答是以绝对值的定义为标准进行分类的.\n注意不要漏掉了 $x=0$ 的情形,这里我们是将 $x=0$ 与 $x>0$ 并在一起考虑的,但并非任何时候都可以这么做!", + "remark": "", + "figures": [] +} \ No newline at end of file diff --git a/processed_dataset/calculation/0022.json b/processed_dataset/calculation/0022.json new file mode 100644 index 0000000000000000000000000000000000000000..8deb6a5b7c83cd028143fac31cb1db24bdab5679 --- /dev/null +++ b/processed_dataset/calculation/0022.json @@ -0,0 +1,8 @@ +{ + "source_file": "./raw_volume-zh/volume1/chapter7.tex", + "problem_type": "calculation", + "problem": "例2. 已知 $a>0, a \\neq 1$, 解关于 $x$ 的不等式:\n$$\n2 \\log _a(x-1)>\\log _a[1+a(x-2)] .\n$$", + "solution": "分析:解对数不等式必然要考虑对数函数的单调性.\n于是, 将底数 $a$ 分为 $01$ 两种情形讨论.\n解 (1) 当 $00, \\\\\n1+a(x-2)>0, \\\\\n(x-1)^2<1+a x-2 a,\n\\end{array}\\right. \\\\\n& \\left\\{\\begin{array}{l}\nx>1, \\\\\nx>2-\\frac{1}{a}, \\\\\na2-\\frac{1}{a}$. 所以此时原不等式的解为 $11$ 时,原不等式等价于\n$$\n\\begin{aligned}\n& \\left\\{\\begin{array}{l}\nx-1>0, \\\\\n1+a(x-2)>0, \\\\\n(x-1)^2>1+a x-2 a,\n\\end{array}\\right. \\\\\n& \\left\\{\\begin{array}{l}\nx>2-\\frac{1}{a}, \\\\\n(x-2)(x-a)>0 .\n\\end{array}\\right.\n\\end{aligned}\n$$\ni) 当 $12$.\n因为 $a>1$, 所以 $a-\\left(2-\\frac{1}{a}\\right)=a+\\frac{1}{a}-2>2 \\sqrt{a \\cdot \\frac{1}{a}}-2=0$, 所以 $a>2-\\frac{1}{a}$.\n所以, 此时原不等式的解为 $2-\\frac{1}{a}2$.\nii) 当 $a \\geqslant 2$ 时,由 (2) 得 $x<2$ 或 $x>a$.\n因为 $a \\geqslant 2$, 所以 $2>2-\\frac{1}{a}$.\n所以, 此时原不等式的解为 $2-\\frac{1}{a}a$.\n综上, 当 $01$; 第二级的分类是为了比较不等式 (2) 对应的方程 $(x-2)(x-a)=0$ 的两根的大小, 我们将 $a>1$ 又分成两小类: i) $12003$ 时, 必有两个不同的子集的元素和关于模 2003 同余.\n设这两个子集为 $A 、 B$, 且 $A \\cap B=C$. 则集合 $A \\backslash C$ 与 $B \\backslash C$ 的元素和关于模 2003 仍同余.\n这时, 艾夫只要在集合 $A \\backslash C$ 的元素前加“十” 号,在 $B \\backslash C$ 的元素前加“一”号, 而将其他元素全删除, 即可获胜.\n取 $n \\geqslant 11$, 便有 $2^n-1>2003$.\n那么, 当 $n \\leqslant 10$ 时有什么结果呢? 这时只要安写下整数 $1,2, \\cdots, 2^{n-1} (n \\leqslant 10)$ 中的若干个, 则已立于不败之地.\n因为艾夫无论怎么做, 所得的和都只能在一 1023 与 1023 之间,且不等于 0 .\n解当 $n \\leqslant 10$ 时, 安有必胜策略.\n为此, 他可写出整数 $1,2, \\cdots, 2^{n-1}$. 因为 $1+2+\\cdots+2^{n-1}=2^n-1 \\leqslant 2^{10}-1=1023$, 所以, 艾夫可能得到的和只能在一 1023 与 1023 之间.\n由二进制数的表示的惟一性及添加正、负号的办法知, 艾夫得到的和也不可能为 0 . 所以,艾夫必败无疑.\n当 $n \\geqslant 11$ 时, 艾夫有必胜的策略.\n设安写出的整数所成之集为 $M$. 因为 $2^n-1 \\geqslant 2^{11}-1>2003$, 所以 $M$ 的非空子集数大于 2003 . 因而, 一定存在 $M$ 的两个不同子集, 例如 $A$ 和 $B$, 使得 $A$ 中数的和与 $B$ 中数的和关于模 2003 同余.\n如果艾夫将“+”号放在集合 $A \\backslash B$ 中的数的前面, 将“-”号放在集合 $B \\backslash A$ 中的数的前面, 并删除 $M$ 中所有其余的数,则艾夫必胜.", + "remark": "", + "figures": [] +} \ No newline at end of file diff --git a/processed_dataset/calculation/0024.json b/processed_dataset/calculation/0024.json new file mode 100644 index 0000000000000000000000000000000000000000..5dc4c718da58d23625c892403f9fe45375823868 --- /dev/null +++ b/processed_dataset/calculation/0024.json @@ -0,0 +1,8 @@ +{ + "source_file": "./raw_volume-zh/volume1/chapter7.tex", + "problem_type": "calculation", + "problem": "例4。 彼得有 25 名同班同学(他自己未计人数目 25 之内). 已知这 25 名同学在班内的朋友数目各不相同, 试问彼得在该班有多少名朋友?", + "solution": "分析:因为彼得也可能是同班同学的朋友, 所以彼得的 25 名同学的朋友数分别只能是 $0,1,2, \\cdots, 24,25$ 之一, 且互不相同.\n如果在彼得的同学中存在孤独者 (无朋友者), 则另 24 个同学的朋友数分别为 $1,2, \\cdots, 24$; 否则, 彼得的 25 个同学的朋友数分别为 $1,2, \\cdots, 25$. 看来我们得分如上两种情形讨论.\n解分两种情形讨论.\n第一种情形假定某位同学在班上的朋友数为 0 . 则除了这位孤独者和彼得以外,其他同学每人在班上的朋友数不多于 24 . 因为这些同学总共 24 人, 每人在班上的朋友数不同, 所以他们的朋友数依次为 $1,2, \\cdots, 24$.\n约定将朋友数为 $1,2, \\cdots, 12$ 的同学编为 $A$ 组, 将朋友数为 $13,14, \\cdots$, 24 的同学编为 $B$ 组.\n将各组同学的朋友数求和, 分别得到\n$$\n\\begin{aligned}\n& S(A)=1+2+\\cdots+12=78, \\\\\n& S(B)=13+14+\\cdots+24=222 .\n\\end{aligned}\n$$\n设 $A$ 组中有 $k$ 名同学是彼得的朋友.\n则 $A$ 组同学在 $B$ 组中的朋友数总和不多于 $S(A)-k$, 另外 $B$ 组同学在本组中的朋友数总和不超过 $12 \\times 11$. 因此, 彼得在 $B$ 组中的朋友数不少于\n$$\nS(B)-12 \\times 11-(S(A)-k)=12+k,\n$$\n但 $B$ 组总共只有 12 人, 所以只能是 $k=0 . A$ 组中没有彼得的朋友 $(A$ 组同学也没有在本组中的朋友), $B$ 组的每位同学都是彼得的朋友.\n对此情形, 彼得在班上有 12 名朋友.\n第二种情形设班上没有孤独者, 每个人都有朋友.\n朋友数各不相同, 最多可达 25 人.\n约定将朋友数为 $1,2, \\cdots, 12$ 的同学编人 $A$ 组; 将朋友数为 $13,14, \\cdots, 25$ 的同学编人 $B$ 组.\n将各组同学的朋友数求和, 分别得到\n$$\n\\begin{aligned}\n& S(A)=1+2+\\cdots+12=78, \\\\\n& S(B)=13+14+\\cdots+25=247 .\n\\end{aligned}\n$$\n设 $A$ 组中有 $k$ 名同学是彼得的朋友.\n则 $A$ 组同学在 $B$ 组中的朋友数总和不多于 $S(A)-k$. 另外, $B$ 组同学在本组中的朋友数总共不超过 $13 \\times 12$. 于是, 彼得在 $B$ 组中的朋友数不少于\n$$\nS(B)-13 \\times 12-(S(A)-k)=13+k,\n$$\n但 $B$ 组总共只有 13 人, 所以 $k=0 . A$ 组中无彼得的朋友, $B$ 组中的每位同学都是彼得的朋友.\n对此情形, 彼得在班上有 13 名朋友.", + "remark": "", + "figures": [] +} \ No newline at end of file diff --git a/processed_dataset/calculation/0025.json b/processed_dataset/calculation/0025.json new file mode 100644 index 0000000000000000000000000000000000000000..ab91e3a2e910ba0c77eeb832a6685fb4a3094e19 --- /dev/null +++ b/processed_dataset/calculation/0025.json @@ -0,0 +1,8 @@ +{ + "source_file": "./raw_volume-zh/volume1/chapter7.tex", + "problem_type": "calculation", + "problem": "例6. 对任意 $n, k \\in \\mathbf{N}^*$, 令 $S=1^n+2^n+3^n+\\cdots+k^n$. 求 $S$ 被 3 除所得的余数.", + "solution": "分析:因为 $(3 m)^n \\equiv 0(\\bmod 3),(3 m+1)^n \\equiv 1(\\bmod 3)$, $(3 m+2)^{2 r} \\equiv 1(\\bmod 3),(3 m+2)^{2 r+1} \\equiv 2(\\bmod 3)$, 所以对 $n$ 按奇偶性分类是自然的.\n解 (1) 当 $n$ 为奇数时, 不妨设 $n=2 l-1, l \\in \\mathbf{N}^*$. 对 $m \\in \\mathbf{N}^*$, 如果 $3 \\times m$, 则 $m^2 \\equiv 1(\\bmod 3) \\Rightarrow m^{2 l} \\equiv 1(\\bmod 3) \\Rightarrow m^{2 l-1} \\equiv m^{2(l-1)+1} \\equiv m(\\bmod 3)$; 如果 $3 \\mid m$, 则 $m^{2 l-1} \\equiv 0 \\equiv m(\\bmod 3)$. 于是, 当 $n$ 为奇数时, 对 $m \\in \\mathbf{N}$, 总有 $m^n \\equiv m(\\bmod 3)$. 从而\n$$\n\\begin{aligned}\nS & \\equiv 1+2+3+\\cdots+k \\\\\n& \\equiv(1+2+3)+(4+5+6)+\\cdots(\\bmod 3) .\n\\end{aligned}\n$$\n当 $k=3 t+3$ 或 $k=3 t+2$ 时, 就有\n$$\nS \\equiv 0(\\bmod 3)(t \\in \\mathbf{N})\n$$\n当 $k=3 t+1$ 时, 就有\n$$\n\\begin{aligned}\nS \\equiv & (1+2+3)+(4+5+6)+\\cdots \\\\\n& +[(k-3)+(k-2)+(k-1)]+k \\\\\n\\equiv & 1(\\bmod 3)(t \\in \\mathbf{N}) .\n\\end{aligned}\n$$\n(2) 当 $n$ 为偶数时, 对 $m \\in \\mathbf{N}$, 由 (1) 知, $3 \\times m \\Rightarrow m^n \\equiv 1(\\bmod 3), 3 \\mid m \\Rightarrow m^n \\equiv 0(\\bmod 3)$. 于是\n$$\nS \\equiv(1+1+0)+(1+1+0)+\\cdots(\\bmod 3) .\n$$\n当 $k=3 t+3(t \\in \\mathbf{N})$ 时, $(1+1+0)$ 共有 $t+1$ 组, 故 $S \\equiv(t+1)(1+1+0) \\equiv 2 t+2(\\bmod 3)$;\n当 $k=3 t+2(t \\in \\mathbf{N})$ 时, $(1+1+0)$ 共有 $t$ 组, 且 $(k-1)^n \\equiv k^n \\equiv 1(\\bmod 3)$, 故 $S \\equiv 2 t+1+1 \\equiv 2 t+2(\\bmod 3)$;\n当 $k=3 t+1(t \\in \\mathbf{N})$ 时, $(1+1+0)$ 共有 $t$ 组, 且 $k^n \\equiv 1(\\bmod 3)$, 故 $S \\equiv 2 t+1(\\bmod 3)$.\n综合 (1)、(2) 可知:\n当 $n$ 为奇正整数时,有当 $n$ 为偶正整数时,有\n$$\nS \\equiv\\left\\{\\begin{array}{l}\n0, k=9 t+4 \\text { 或 } 9 t+8 \\text { 或 } 9 t+9, \\\\\n1, k=9 t+1 \\text { 或 } 9 t+5 \\text { 或 } 9 t+6,(\\bmod 3)(t \\in \\mathbf{N}) . \\\\\n2, k=9 t+2 \\text { 或 } 9 t+3 \\text { 或 } 9 t+7\n\\end{array}\\right.\n$$\n说明这是一个两级分类的例子.\n首先是对 $n$ 按奇偶性(模 2 的剩余类) 分成两大类,然后又将每一大类对 $k$ 按模 3 的剩余类分成三个小类.", + "remark": "", + "figures": [] +} \ No newline at end of file diff --git a/processed_dataset/calculation/0026.json b/processed_dataset/calculation/0026.json new file mode 100644 index 0000000000000000000000000000000000000000..5e8c06b7f197ba069af152a5a4e49bdf8872fe4c --- /dev/null +++ b/processed_dataset/calculation/0026.json @@ -0,0 +1,8 @@ +{ + "source_file": "./raw_volume-zh/volume1/chapter7.tex", + "problem_type": "calculation", + "problem": "例7. 求集合 $B 、 C$, 使得 $B \\cup C=\\{1,2, \\cdots, 10\\}$, 并且 $C$ 的元素乘积等于 $B$ 的元素和.", + "solution": "分析:这实际上是求特殊条件下集合方程的解.\n注意到集合 $B$ 的元素和 $\\leqslant 1+2+\\cdots+10=55$, 而 $1 \\cdot 2 \\cdot 3 \\cdot 4 \\cdot 5=120$, 故知集合 $C$ 至多有 4 个元素.\n这样我们可以按 $|C|$ 的可能值分 4 类来讨论.\n解因为 $1+2+\\cdots+10=55<120=1 \\cdot 2 \\cdot 3 \\cdot 4 \\cdot 5$, 所以集合 $C$ 至多有 4 个元素.\n下面对 $|C|$ 分 4 种情况讨论.\n(1) $C$ 由一个元素构成.\n因为 $C$ 的元素乘积不超过 $10, B$ 的元素和至少为 $55-10=45$. 故此情况不成立.\n(2) $C$ 由两个元素 $x 、 y$ 构成.\n设 $x55-x-y-z$. 无解.\n(4) $C$ 由四个元素 $x55$. 这时\n$$\ny z t=54-y-z-t, 2 \\leqslant y0, b>0$, 求 $\\min \\left\\{\\max \\left\\{\\frac{1}{a}, \\frac{1}{b}, a^2+b^2\\right\\}\\right\\}$ 的值.", + "solution": "分析:为了求出 $\\max \\left\\{\\frac{1}{a}, \\frac{1}{b}, a^2+b^2\\right\\}$, 我们来比较 $\\frac{1}{a} 、 \\frac{1}{b} 、 a^2+b^2$ 的大小.\n令 $\\frac{1}{a}=\\frac{1}{b}=a^2+b^2$, 得 $a=b=\\sqrt[3]{\\frac{1}{2}}$. 如设 $a \\geqslant b>0$, 则 $a 、 b 、 \\sqrt[3]{\\frac{1}{2}}$ 有三种顺序关系: $a \\geqslant b \\geqslant \\sqrt[3]{\\frac{1}{2}}, \\sqrt[3]{\\frac{1}{2}} \\geqslant a \\geqslant b, a \\geqslant \\sqrt[3]{\\frac{1}{2}} \\geqslant b$. 我们就以此分类.\n解不失一般性, 不妨设 $a \\geqslant b>0$, 则 $0<\\frac{1}{a} \\leqslant \\frac{1}{b}$. 令 $\\frac{1}{a}=\\frac{1}{b}=a^2+b^2$, 则 $a=b=\\sqrt[3]{\\frac{1}{2}}$.\n(1) 若 $a \\geqslant b \\geqslant \\sqrt[3]{\\frac{1}{2}}$, 则\n$$\n\\frac{1}{a} \\leqslant \\frac{1}{b} \\leqslant \\sqrt[3]{2}, a^2+b^2 \\geqslant 2 b^2 \\geqslant \\sqrt[3]{2} .\n$$\n所以 $\\max \\left\\{\\frac{1}{a}, \\frac{1}{b}, a^2+b^2\\right\\}=a^2+b^2 \\geqslant \\sqrt[3]{2}$. 从而当且仅当 $a=b=\\sqrt[3]{\\frac{1}{2}}$ 时,\n$$\n\\min \\left\\{\\max \\left\\{\\frac{1}{a}, \\frac{1}{b}, a^2+b^2\\right\\}\\right\\}=\\min \\left\\{a^2+b^2\\right\\}=\\sqrt[3]{2} ;\n$$\n(2) 若 $\\sqrt[3]{\\frac{1}{2}} \\geqslant a \\geqslant b>0$, 则\n$$\n\\frac{1}{b} \\geqslant \\frac{1}{a} \\geqslant \\sqrt[3]{2}, a^2+b^2 \\leqslant 2 a^2 \\leqslant \\sqrt[3]{2}\n$$\n所以 $\\max \\left\\{\\frac{1}{a}, \\frac{1}{b}, a^2+b^2\\right\\}=\\frac{1}{b} \\geqslant \\sqrt[3]{2}$. 从而当且仅当 $a=b=\\sqrt[3]{\\frac{1}{2}}$ 时,\n$$\n\\min \\left\\{\\max \\left\\{\\frac{1}{a}, \\frac{1}{b}, a^2+b^2\\right\\}\\right\\}=\\min \\left\\{\\frac{1}{b}\\right\\}=\\sqrt[3]{2} .\n$$\n(3)若 $a \\geqslant \\sqrt[3]{\\frac{1}{2}} \\geqslant b>0$, 则 $\\frac{1}{b} \\geqslant \\sqrt[3]{2} \\geqslant \\frac{1}{a}>0$.\n此时若 $\\frac{1}{b} \\geqslant a^2+b^2$, 则\n$$\n\\max \\left\\{\\frac{1}{a}, \\frac{1}{b}, a^2+b^2\\right\\}=\\frac{1}{b} \\geqslant \\sqrt[3]{2}\n$$\n若 $\\frac{1}{b} \\leqslant a^2+b^2$, 则\n$$\n\\max \\left\\{\\frac{1}{a}, \\frac{1}{b}, a^2+b^2\\right\\}=a^2+b^2 \\geqslant \\frac{1}{b} \\geqslant \\sqrt[3]{2} ;\n$$\n所以当且仅当 $a=b=\\sqrt[3]{\\frac{1}{2}}$ 时,\n$$\n\\min \\left\\{\\max \\left\\{\\frac{1}{a}, \\frac{1}{b}, a^2+b^2\\right\\}\\right\\}=\\sqrt[3]{2}\n$$\n综上所述: 当且仅当 $a=b=\\sqrt[3]{\\frac{1}{2}}$ 时,\n$$\n\\min \\left\\{\\max \\left\\{\\frac{1}{a}, \\frac{1}{b}, a^2+b^2\\right\\}\\right\\}=\\sqrt[3]{2} .\n$$", + "remark": "", + "figures": [] +} \ No newline at end of file diff --git a/processed_dataset/calculation/0028.json b/processed_dataset/calculation/0028.json new file mode 100644 index 0000000000000000000000000000000000000000..b8a1076905a49f938be69762fdd56e82a7b4dd64 --- /dev/null +++ b/processed_dataset/calculation/0028.json @@ -0,0 +1,8 @@ +{ + "source_file": "./raw_volume-zh/volume1/chapter9.tex", + "problem_type": "calculation", + "problem": "例1. 设正整数 $a 、 b 、 c$ 为三角形三边长, $a+b=n, n \\in \\mathbf{N}^*, 1 \\leqslant c \\leqslant n-1$. 求这样的三角形的个数.", + "solution": "分析:设 $\\triangle A B C$ 的角 $A 、 B 、 C$ 的对应边分别为 $a 、 b 、 c$. 例 1 就是要计算有限集 $M=\\left\\{\\triangle A B C \\mid a+b=n, a, b \\in \\mathbf{N}^*, 1 \\leqslant c \\leqslant n-1\\right\\}$ 的阶.\n也就是要计算同时满足 $a+b>c, b+c>a, c+a>b$ 的三元正整数组 $\\{a, b, c\\}$ 的个数.\n解不妨设 $b \\geqslant a$, 则 $1 \\leqslant a \\leqslant\\left[\\frac{n}{2}\\right]$. 满足题设条件的三角形可分为两类:\n第一类: $c$ 为最大边.\n令 $a=i$, 则 $b=n-i, n-i \\leqslant c \\leqslant n-1$. 这样的三角形有 $(n-1)-(n-i)+1=i$ 个.\n第二类: $c$ 不为最大边.\n则 $b>c, c+a>b$, 故 $b-a=n-2 i, n-2 i< c1990$ 时,将 $a_i+k$ 理解为 $a_i+k-1990$. 这种 $A_0$ 与 $A_k$ 间的对应是一一的.\n所以有\n$$\n\\left|A_0\\right|=\\left|A_1\\right|=\\left|A_2\\right|=\\left|A_3\\right|=\\left|A_4\\right|,\n$$\n于是\n$$\n\\begin{aligned}\n\\left|A_0\\right| & =\\frac{1}{5}\\left(\\left|A_0\\right|+\\left|A_1\\right|+\\left|A_2\\right|+\\left|A_3\\right|+\\left|A_4\\right|\\right) \\\\\n& =\\frac{1}{5}|M|=\\frac{1}{5} C_{1990}^{31} .\n\\end{aligned}\n$$\n说明在这里,我们的目的并不是求 $|M|$, 而是由于 $|M|$ 易于计算, 我们反过来利用这一点来达到计算 $\\left|A_0\\right|$ 的目的.", + "remark": "", + "figures": [] +} \ No newline at end of file diff --git a/processed_dataset/calculation/0030.json b/processed_dataset/calculation/0030.json new file mode 100644 index 0000000000000000000000000000000000000000..905b62e4221d6ee0d0700dc69bc557625528ba8d --- /dev/null +++ b/processed_dataset/calculation/0030.json @@ -0,0 +1,8 @@ +{ + "source_file": "./raw_volume-zh/volume1/chapter9.tex", + "problem_type": "calculation", + "problem": "例3. 设集合 $S=\\{1,2, \\cdots, 1000\\}, A$ 是 $S$ 的子集,且 $A$ 的元素或是 3 的倍数, 或是 7 的倍数.\n试求 $A$ 的元素个数的最大值.", + "solution": "解:设 $A_1=\\{x \\mid x \\in S$, 且 $3 \\mid x\\}, A_2=\\{x \\mid x \\in S$, 且 $7 \\mid x\\}$, 则 $|A|_{\\text {max }}=\\left|A_1 \\cup A_2\\right|$. 显然有\n$$\n\\begin{gathered}\n\\left|A_1\\right|=\\left[\\frac{1000}{3}\\right]=333, \\\\\n\\left|A_2\\right|=\\left[\\frac{1000}{7}\\right]=142, \\\\\n\\left|A_1 \\cap A_2\\right|=\\left[\\frac{1000}{3 \\cdot 7}\\right]=47 .\n\\end{gathered}\n$$\n所以\n$$\n\\begin{aligned}\n\\left|A_1 \\cup A_2\\right| & =\\left|A_1\\right|+\\left|A_2\\right|-\\left|A_1 \\cap A_2\\right| \\\\\n& =333+142-47=428 .\n\\end{aligned}\n$$\n所以 $A$ 的元素个数的最大值为 428 .\n说明利用容斥原理的关键是构造所要计数的集合的一个合适的覆盖.\n上例解答中的覆盖是: $A_1, A_2$.", + "remark": "", + "figures": [] +} \ No newline at end of file diff --git a/processed_dataset/calculation/0031.json b/processed_dataset/calculation/0031.json new file mode 100644 index 0000000000000000000000000000000000000000..a5e755623407b807c192a3f62fb4fc821a167501 --- /dev/null +++ b/processed_dataset/calculation/0031.json @@ -0,0 +1,8 @@ +{ + "source_file": "./raw_volume-zh/volume1/chapter9.tex", + "problem_type": "calculation", + "problem": "例5. 设 $S$ 是有理数 $r$ 的集合, 其中 $0),集合 $A$ 为正方形 $O A B C$, 集合 $B$ 为 Rt $\\triangle D E F . O D 、 A E 、 B F 、 C F$ 、 $C D$ 的中点依次为 $M(3,1) 、 N(6,1)$ 、 $P(6,4) 、 Q(5,4) 、 R(3,2)$. 所成图形面积 $S_{M N P Q R}=7$.", + "remark": "", + "figures": [ + "./images/volume1/figures/fig-c1p4.png" + ] +} \ No newline at end of file diff --git a/processed_dataset/calculation/0038.json b/processed_dataset/calculation/0038.json new file mode 100644 index 0000000000000000000000000000000000000000..fe616509c1d2e159add3080a8485fda82f6e5c25 --- /dev/null +++ b/processed_dataset/calculation/0038.json @@ -0,0 +1,8 @@ +{ + "source_file": "./raw_volume-zh/volume1/exercise1.tex", + "problem_type": "calculation", + "problem": "问题5: 已知非空数集 $M \\subseteq\\{1,2,3,4,5\\}$, 则满足条件“若 $x \\in M$, 则 $6-x \\in M$ ” 的集合 $M$ 的个数是", + "solution": "解: 7. 因为 $1+5=2+4=3+3$, 故 $M$可以是 $\\{3\\},\\{1,5\\},\\{2,4\\},\\{1,3,5\\},\\{2,3,4\\},\\{1,2,4,5\\},\\{1,2,3$, $4,5\\}$.", + "remark": "", + "figures": [] +} \ No newline at end of file diff --git a/processed_dataset/calculation/0039.json b/processed_dataset/calculation/0039.json new file mode 100644 index 0000000000000000000000000000000000000000..6b39e9098a86547848d7c2f492683de5a702e5ea --- /dev/null +++ b/processed_dataset/calculation/0039.json @@ -0,0 +1,8 @@ +{ + "source_file": "./raw_volume-zh/volume1/exercise1.tex", + "problem_type": "calculation", + "problem": "问题6: 设 $a \\in \\mathbf{R}^{+}, A=\\left\\{(x, y) \\mid(x-1)^2+(y-2)^2 \\leqslant \\frac{4}{5}\\right\\}$ 与 $B=\\{(x, y) \\mid |x-1|+2|y-2| \\leqslant a\\}$ 是直角坐标平面 $x O y$ 内的点集.\n则 $A \\subseteq B$ 的充要条件是", + "solution": "解: $a \\geqslant 2$. 集合 $A$ 为以 $(1,2)$ 为圆心 $、 \\frac{2}{\\sqrt{5}}$ 为半径的圆面.\n集合 $B$ 为以 $(1,2)$ 为对角线交点的菱形, 且平行于 $x$ 轴的对角线长为 $2 a$, 平行于 $y$ 轴的对角线长为 $a$. 由 $A \\subseteq B$ 知, 当 $a \\cdot \\frac{a}{2}=\\frac{2}{\\sqrt{5}} \\cdot \\sqrt{a^2+\\left(\\frac{a}{2}\\right)^2}$ 时 $a$ 值最小, 所以 $a_{\\min }=2$.", + "remark": "", + "figures": [] +} \ No newline at end of file diff --git a/processed_dataset/calculation/0040.json b/processed_dataset/calculation/0040.json new file mode 100644 index 0000000000000000000000000000000000000000..45f806d4818864ff7429ade44379c2c5c64f7a2f --- /dev/null +++ b/processed_dataset/calculation/0040.json @@ -0,0 +1,8 @@ +{ + "source_file": "./raw_volume-zh/volume1/exercise1.tex", + "problem_type": "calculation", + "problem": "问题7: 集合 $\\left\\{x \\mid-1 \\leqslant \\log _{\\frac{1}{x}} 10<-\\frac{1}{2}, x>1\\right.$ 且 $\\left.x \\in \\mathbf{N}\\right\\}$ 的真子集的个数是", + "solution": "解: $2^{90}-1$.", + "remark": "", + "figures": [] +} \ No newline at end of file diff --git a/processed_dataset/calculation/0041.json b/processed_dataset/calculation/0041.json new file mode 100644 index 0000000000000000000000000000000000000000..f3cb0e318aa55d70bc5ab73006fc63302e42b2e5 --- /dev/null +++ b/processed_dataset/calculation/0041.json @@ -0,0 +1,8 @@ +{ + "source_file": "./raw_volume-zh/volume1/exercise1.tex", + "problem_type": "calculation", + "problem": "问题8: 已知 $A=\\left\\{x \\mid x^2-4 x+3<0, x \\in \\mathbf{R}\\right\\}, B=\\left\\{x \\mid 2^{1-x}+a \\leqslant 0, x^2-\\right. 2(a+7) x+5 \\leqslant 0, x \\in \\mathbf{R}\\}$. 若 $A \\subseteq B$, 则实数 $a$ 的取值范围是", + "solution": "解: $-4 \\leqslant a \\leqslant-1$. 易知 $A=(1,3)$. 记 $f(x)=2^{1-x}+a, g(x)=x^2-2(a+7) x+5 . A \\subseteq B$ 表明, 当 $10\\} \\cap A=\\varnothing$, 则实数 $p$ 的取值范围是", + "solution": "$(-4,+\\infty)$. 依题意, $x^2+(p+2) x+1=0$ 无解或有两个小于 0 的解.\n所以, $\\Delta<0$ 或 $\\Delta \\geqslant 0$ 且 $-(p+2)<0$.", + "remark": "", + "figures": [] +} \ No newline at end of file diff --git a/processed_dataset/calculation/0052.json b/processed_dataset/calculation/0052.json new file mode 100644 index 0000000000000000000000000000000000000000..22d3c0bacbb6fe65b04ba1bfa6f09e82b292b23a --- /dev/null +++ b/processed_dataset/calculation/0052.json @@ -0,0 +1,8 @@ +{ + "source_file": "./raw_volume-zh/volume1/exercise2.tex", + "problem_type": "calculation", + "problem": "问题5 若 $M=\\left\\{(x, y)|| \\tan \\pi y \\mid+\\sin ^2 \\pi x=0\\right\\}, N=\\left\\{(x, y) \\mid x^2+y^2 \\leqslant 2\\right\\}$, 则 $M \\cap N$ 的元素个数是", + "solution": "9. $M=\\{(x, y) \\mid x=k, y=l, k, l \\in \\mathbf{Z}\\}$.", + "remark": "", + "figures": [] +} \ No newline at end of file diff --git a/processed_dataset/calculation/0053.json b/processed_dataset/calculation/0053.json new file mode 100644 index 0000000000000000000000000000000000000000..f43f1fc83e5153dda0175dbba384508f14d91689 --- /dev/null +++ b/processed_dataset/calculation/0053.json @@ -0,0 +1,8 @@ +{ + "source_file": "./raw_volume-zh/volume1/exercise2.tex", + "problem_type": "calculation", + "problem": "问题6 已知两个复数集合 $M=\\left\\{z \\mid z=\\cos \\alpha+\\left(4-\\cos ^2 \\alpha\\right) \\mathrm{i}, \\alpha \\in \\mathbf{R}\\right\\}, N= \\{z \\mid z=\\cos \\beta+(\\lambda+\\sin \\beta) i, \\beta \\in \\mathbf{R}\\}$. 当 $M \\cap N \\neq \\varnothing$ 时, 实数 $\\lambda$ 的取值范围是", + "solution": "$\\left[\\frac{11}{4}, 5\\right]$. 设 $\\cos \\alpha+\\left(4-\\cos ^2 \\alpha\\right) \\mathrm{i}=\\cos \\beta+(\\lambda+\\sin \\beta) \\mathrm{i}$, 则有 $\\cos \\alpha= \\cos \\beta, 4-\\cos ^2 \\alpha=\\lambda+\\sin \\beta$. 消去 $\\alpha$, 得 $\\lambda=\\frac{11}{4}+\\left(\\sin \\beta-\\frac{1}{2}\\right)^2$, 所以 $\\frac{11}{4} \\leqslant \\lambda \\leqslant 5$.", + "remark": "", + "figures": [] +} \ No newline at end of file diff --git a/processed_dataset/calculation/0054.json b/processed_dataset/calculation/0054.json new file mode 100644 index 0000000000000000000000000000000000000000..b8e2e32e2592b9d0ab917ba2218e8340ae4bfed2 --- /dev/null +++ b/processed_dataset/calculation/0054.json @@ -0,0 +1,8 @@ +{ + "source_file": "./raw_volume-zh/volume1/exercise2.tex", + "problem_type": "calculation", + "problem": "问题7 若集合 $A=\\{x \\mid-2 \\leqslant x \\leqslant 5\\}, B=\\{x \\mid m+1 \\leqslant x \\leqslant 2 m-1\\}$, 且 $A \\cap B=B$, 则实数 $m$ 的取值范围是", + "solution": "$m \\leqslant 3$. 当 $B=\\varnothing$ 时, $m+1>2 m-1$, 得 $m<2$; 当 $B \\neq \\varnothing$ 时, 须 $-2 \\leqslant m+1$, $2 m-1 \\leqslant 5, m+1 \\leqslant 2 m-1$ 同时满足, 解得 $2 \\leqslant m \\leqslant 3$.", + "remark": "", + "figures": [] +} \ No newline at end of file diff --git a/processed_dataset/calculation/0055.json b/processed_dataset/calculation/0055.json new file mode 100644 index 0000000000000000000000000000000000000000..7082661decc29e8f4ea509d68942aa683ec81ce4 --- /dev/null +++ b/processed_dataset/calculation/0055.json @@ -0,0 +1,10 @@ +{ + "source_file": "./raw_volume-zh/volume1/exercise2.tex", + "problem_type": "calculation", + "problem": "问题8 设全集 $U=\\left\\{x \\mid x=2 n-1, n \\in \\mathbf{N}^*, n \\leqslant 7\\right\\}, A \\cap\\left(\\complement_U B\\right)=\\{3,7\\}$, $\\left(\\complement_U A\\right) \\cap B=\\{9,13\\},\\left(\\complement_U A\\right) \\cap\\left(\\complement_U B\\right)=\\{1,11\\}$. 则 $A=? B=?$", + "solution": "如图()所示,\n$A=\\{3,5,7\\}, B=\\{5,9,13\\}$.", + "remark": "", + "figures": [ + "./images/volume1/figures/fig-c2p8.png" + ] +} \ No newline at end of file diff --git a/processed_dataset/calculation/0056.json b/processed_dataset/calculation/0056.json new file mode 100644 index 0000000000000000000000000000000000000000..bab8c1e52e7f6e301a801bfc2e155e1cf104b8c9 --- /dev/null +++ b/processed_dataset/calculation/0056.json @@ -0,0 +1,8 @@ +{ + "source_file": "./raw_volume-zh/volume1/exercise2.tex", + "problem_type": "calculation", + "problem": "问题9 设集合 $A=\\left\\{x \\mid x^2-a x+a^2-19=0\\right\\}, B=\\left\\{x \\mid x^2-5 x+6=0\\right\\}$, $C=\\left\\{x \\mid x^2+2 x-8=0\\right\\}$, 且 $A \\cap B \\neq \\varnothing, A \\cap C=\\varnothing$. 则实数 $a=?$", + "solution": "$a=-2 . B=\\{2,3\\}, C=\\{2,-4\\}$. 由第 8 题图题设知 $3 \\in A, 2 \\notin A,-4 \\notin A$. 将 3 代入方程 $x^2-a x+a^2-19=0$, 得 $a=-2$ 或 5 . 然后逐一检验.", + "remark": "", + "figures": [] +} \ No newline at end of file diff --git a/processed_dataset/calculation/0057.json b/processed_dataset/calculation/0057.json new file mode 100644 index 0000000000000000000000000000000000000000..2dd2551398de51dc1525753f3b96a7f6337bfd11 --- /dev/null +++ b/processed_dataset/calculation/0057.json @@ -0,0 +1,10 @@ +{ + "source_file": "./raw_volume-zh/volume1/exercise2.tex", + "problem_type": "calculation", + "problem": "问题10 集合 $A=\\left\\{(x, y) \\mid \\sin (3 x+5 y)>0\\right.$, 且 $\\left.x^2+y^2 \\leqslant \\pi^2\\right\\}$ 所构成的平面图形的面积是", + "solution": "$\\frac{1}{2} \\pi^3 . A=\\{(x, y) \\mid 2 k \\pi<3 x+ 5 y<2 k \\pi+\\pi, k \\in \\mathbf{Z}, \\text{且} x^2+y^2 \\leqslant \\pi^2\\}, A$ 所成图形为图()中阴影部分.", + "remark": "", + "figures": [ + "./images/volume1/figures/fig-c2p10.png" + ] +} \ No newline at end of file diff --git a/processed_dataset/calculation/0058.json b/processed_dataset/calculation/0058.json new file mode 100644 index 0000000000000000000000000000000000000000..6817b3fe90aa6f094b243a00e06819bc1e98ac9e --- /dev/null +++ b/processed_dataset/calculation/0058.json @@ -0,0 +1,8 @@ +{ + "source_file": "./raw_volume-zh/volume1/exercise2.tex", + "problem_type": "calculation", + "problem": "问题11 设 $m, n \\in \\mathbf{N}^*$, 且 $m>n$, 集合 $A=\\{1,2, \\cdots, m\\}, B=\\{1,2, \\cdots, n\\}$, 又 $C \\subset A, B \\cap C \\neq \\varnothing$. 则这样的集合 $C$ 的个数是", + "solution": "$2^{m-n}\\left(2^n-1\\right)$. 由于 $A$ 的子集中只有由自然数 $n+1, n+2, \\cdots, m$ 中任取若干个数组成的集合 $C^{\\prime}$,才能使 $B \\cap C^{\\prime}=\\varnothing$, 而这样的集合 $C^{\\prime}$ 有 $2^{m-n}$ 个.\n所以满足 $B \\cap C \\neq \\varnothing$ 的集合 $C$ 的个数是 $2^m-2^{m-n}$.", + "remark": "", + "figures": [] +} \ No newline at end of file diff --git a/processed_dataset/calculation/0059.json b/processed_dataset/calculation/0059.json new file mode 100644 index 0000000000000000000000000000000000000000..62a4986def3f3a9366e875b41a87cfdba4d96834 --- /dev/null +++ b/processed_dataset/calculation/0059.json @@ -0,0 +1,8 @@ +{ + "source_file": "./raw_volume-zh/volume1/exercise2.tex", + "problem_type": "calculation", + "problem": "问题13 已知 $A=\\{(x, y) \\mid x=n, y=n a+b, n \\in \\mathbf{Z}\\}, B=\\{(x, y) \\mid x=m, y=3 m^2+15, m \\in \\mathbf{Z} \\}, C=\\left\\{(x, y) \\mid x^2+y^2 \\leqslant 144\\right\\}$ 是坐标平面内的三个点集.\n试问, 是否存在实数 $a 、 b$, 使得 $A \\cap B \\neq \\varnothing$ 、点 $(a, b) \\in C$ 同时成立? 若存在, 请求出 $a 、 b$ 的值; 若不存在, 则说明理由.", + "solution": "假设存在实数 $a 、 b$, 同时满足题中的两个条件, 则必存在整数 $n$, 使 $3 n^2-a n+(15-b)=0$, 于是它的判别式 $\\Delta=(-a)^2-12(15-b) \\geqslant 0$, 即 $a^2 \\geqslant 12(15-b)$. 又由 $a^2+b^2 \\leqslant 144$ 得 $a^2 \\leqslant 144-b^2$, 由此便得 $12(15- b) \\leqslant 144-b^2$, 即 $(b-6)^2 \\leqslant 0$, 故 $b=6$. 将 $b=6$ 代入上述的 $a^2 \\geqslant 12(15-b)$ 及 $a^2 \\leqslant 144-b^2$ 得 $a^2=108$, 所以 $a= \\pm 6 \\sqrt{3}$. 将 $a= \\pm 6 \\sqrt{3} 、 b=6$ 代入方程 $3 n^2-a n+(15-b)=0$, 求得 $n= \\pm \\sqrt{3} \\notin \\mathbf{Z}$. 这说明满足已知两个条件的实数 $a 、 b$ 是不存在的.", + "remark": "", + "figures": [] +} \ No newline at end of file diff --git a/processed_dataset/calculation/0060.json b/processed_dataset/calculation/0060.json new file mode 100644 index 0000000000000000000000000000000000000000..64ed4184b9b6f55f40609303d42eefea57293159 --- /dev/null +++ b/processed_dataset/calculation/0060.json @@ -0,0 +1,8 @@ +{ + "source_file": "./raw_volume-zh/volume1/exercise2.tex", + "problem_type": "calculation", + "problem": "问题14 给定自然数 $a \\geqslant 2$, 集合 $A=\\left\\{y \\mid y=a^x, x \\in \\mathbf{N}\\right\\}, B=\\{y \\mid y=(a+$ 1) $x+b, x \\in \\mathbf{N}\\}$. 在区间 $[1, a]$ 上是否存在 $b$, 使得 $C=A \\cap B \\neq \\varnothing$ ? 如果存在, 试求 $b$ 的一切可能值及相应的集合 $C$; 如果不存在, 试说明理由.", + "solution": "因 $a \\geqslant 2, a \\in \\mathbf{N}, x \\in \\mathbf{N}$, 所以 $a^x \\in \\mathbf{N}$, 且 $(a+1) x \\in \\mathbf{N}$. 又因为 $A \\cap B \\neq \\varnothing$, 所以 $b \\in \\mathbf{N}$. 只需求 $b \\in \\mathbf{N}$ 的 $b$ 值, 使得满足 $(a+1) x_2+b=a^{x_1}$, 即 $x_2=\\frac{a^{x_1}-b}{a+1}, x_1, x_2 \\in \\mathbf{N}$. 当 $x_1$ 为正偶数时, $x_2=-\\frac{1-a^{x_1}}{1+a}-\\frac{b-1}{1+a}$. 因为 $-\\frac{1-a^{x_1}}{1+a} \\in \\mathbf{N}, 1 \\leqslant b \\leqslant a$, 所以 $0 \\leqslant \\frac{b-1}{1+a}<1$. 因 $x_2 \\in \\mathbf{N}$, 故 $b=1$. 当 $x_1$ 为正奇数时, $x_2=\\frac{a^{x_1}+1}{a+1}-\\frac{b+1}{a+1}$. 因为 $\\frac{a^{x_1}+1}{a+1}$ 是大于 1 的自然数, $0 \\leqslant \\frac{b+1}{a+1} \\leqslant 1, x_2 \\in \\mathbf{N}$, 所以 $b=a$. 综上知, 在 $[1, a]$ 上存在 $b=1$ 或 $b=a$, 使得 $A \\cap B \\neq \\varnothing$. 当 $b=1$ 时, $A \\cap B=\\left\\{y \\mid y=a^{2 x}, x \\in \\mathbf{N}\\right\\}$; 当 $b=a$ 时, $A \\cap B=\\{y \\mid y=a^{2 x+1}, x \\in \\mathbf{N}\\}$.", + "remark": "", + "figures": [] +} \ No newline at end of file diff --git a/processed_dataset/calculation/0061.json b/processed_dataset/calculation/0061.json new file mode 100644 index 0000000000000000000000000000000000000000..0b1d7fd6f5aaf3bf37cbc3439129dfe33cb3e472 --- /dev/null +++ b/processed_dataset/calculation/0061.json @@ -0,0 +1,8 @@ +{ + "source_file": "./raw_volume-zh/volume1/exercise3.tex", + "problem_type": "calculation", + "problem": "问题1 设 $f(x)=x^2+x-2, A=\\{n \\mid 1 \\leqslant n \\leqslant 100, n \\in \\mathbf{Z}\\}, B=\\{y \\mid y=f(n), n \\in A\\}$,则集合 $B \\bigcap\\{2 m \\mid m \\in \\mathbf{Z}\\}$ 的元素的个数是", + "solution": "$100$. 当 $n \\in A$ 时, $y=f(n)=n^2+n-2$ 恒为偶数.", + "remark": "", + "figures": [] +} \ No newline at end of file diff --git a/processed_dataset/calculation/0062.json b/processed_dataset/calculation/0062.json new file mode 100644 index 0000000000000000000000000000000000000000..e2f0793e112114fb3bec2c5f0e79cc78c58ac0dc --- /dev/null +++ b/processed_dataset/calculation/0062.json @@ -0,0 +1,8 @@ +{ + "source_file": "./raw_volume-zh/volume1/exercise3.tex", + "problem_type": "calculation", + "problem": "问题2 设 $M=\\{1,2, \\cdots, 1995\\}, A$ 是 $M$ 的子集, 且满足条件: 当 $x \\in A$ 时, $15 x \\notin A$. 则 $A$ 中元素个数最多是", + "solution": "$1870$ . $k$ 与 $15 k(k=9,10, \\cdots, 133)$ 不能同在 $A$ 中, 又 $133<15 \\times 9$, 所以 $|A| \\leqslant 1995-(133-9+1)=1870$. 另一方面, 设 $B=\\{1,2, \\cdots, 8\\}$, $C=\\{134,135, \\cdots, 1995\\}$, 取 $A=B \\cup C$, 则 $|A|=1870$.", + "remark": "", + "figures": [] +} \ No newline at end of file diff --git a/processed_dataset/calculation/0063.json b/processed_dataset/calculation/0063.json new file mode 100644 index 0000000000000000000000000000000000000000..876faf6d2b855ef46e91bb209b065c7d7284cd9b --- /dev/null +++ b/processed_dataset/calculation/0063.json @@ -0,0 +1,8 @@ +{ + "source_file": "./raw_volume-zh/volume1/exercise3.tex", + "problem_type": "calculation", + "problem": "问题3 把集合 $\\left\\{1,2, \\cdots, 10^6\\right\\}$ 划分成两个不交的子集,一个是所有可以表示为一个完全平方数与一个完全立方数之和的数所成的子集, 另一个是集合中所有其余的数所成的子集.\n问哪一个子集元素较多? 说明理由.", + "solution": "将前一个子集记为 $A$, 依题设对任何 $n \\in A$, 都存在 $k, m \\in \\mathbf{N}^*$, 使得 $n=k^2+m^3$. 由于 $n \\leqslant 10^6$, 所以 $k \\leqslant 10^3, m \\leqslant 10^2$, 因而数对 $(k, m)$ 的个数不超过 $10^5$ 个, 从而 $n \\in A$ 的个数也不超过 $10^5$ 个.\n可见,第二个子集的元素多.", + "remark": "", + "figures": [] +} \ No newline at end of file diff --git a/processed_dataset/calculation/0064.json b/processed_dataset/calculation/0064.json new file mode 100644 index 0000000000000000000000000000000000000000..a1b6c0939732926a6320657c546f86657e22cdb5 --- /dev/null +++ b/processed_dataset/calculation/0064.json @@ -0,0 +1,8 @@ +{ + "source_file": "./raw_volume-zh/volume1/exercise3.tex", + "problem_type": "calculation", + "problem": "问题4 集合 $\\{00,01, \\cdots, 98,99\\}$ 的子集 $X$ 满足: 在任一无穷的数字序列中均有 2 个相邻数字构成 $X$ 的元素.\n$X$ 最少应含多少个元素?", + "solution": "对任意的 $i, j \\in\\{0,1, \\cdots, 9\\}, X$ 应包含 $\\overline{i j}$ 或 $\\overline{j i}$ 之一.\n这种无序对 $(i$, $j$ ) 共有 $10+\\frac{1}{2} \\cdot 10 \\cdot 9=55$ 个, 故 $|X| \\geqslant 55$. 又如取 $X=\\{\\overline{i j} \\mid 0 \\leqslant i \\leqslant j \\leqslant 9\\}$, 则 $|X|=55$, 且对任一无穷序列, 设 $i$ 为它所含的最小数字, $j$ 为 $i$ 的后一项,则 $\\overline{i j} \\in X$. 故 $X$ 最少含 55 个元素.", + "remark": "", + "figures": [] +} \ No newline at end of file diff --git a/processed_dataset/calculation/0065.json b/processed_dataset/calculation/0065.json new file mode 100644 index 0000000000000000000000000000000000000000..45cb531c1c7ef8dd869c9dc9651b43aad45ca62b --- /dev/null +++ b/processed_dataset/calculation/0065.json @@ -0,0 +1,8 @@ +{ + "source_file": "./raw_volume-zh/volume1/exercise3.tex", + "problem_type": "calculation", + "problem": "问题5 设 $S$ 是 $n$ 个不同实数的集合, $A_s$ 是由 $S$ 中所有互不相同的两元素的平均值所组成的集合.\n对给定 $n \\geqslant 2, A_s$ 最少可能有多少个元素?", + "solution": "设 $S=\\left\\{x_1, x_2, \\cdots, x_n\\right\\}$, 且 $x_1a_2>a_3>\\cdots>a_m$ 且 $a_4>0$ (因为若把每个数都乘以 -1 , 不会改变我们的性质). 于是, $a_1+a_2>a_1+a_3>a_1+$$a_4>a_1$, 从而和数 $a_1+a_2, a_1+a_3, a_1+a_4$ 都不属于集合 $A$. 并且和数 $a_2+a_3$ 与 $a_2+a_4$ 不可能同时属于集合 $A$, 此因 $a_2+a_3>a_2, a_2+a_4>a_2$, 且 $a_2+ a_3 \\neq a_2+a_4$. 这样一来, 对数组 $\\left(a_1, a_2, a_3\\right)$ 和 $\\left(a_1, a_2, a_4\\right)$, 至少有一个组中任何两个数的和都不是 $A$ 中的元素.", + "remark": "", + "figures": [] +} \ No newline at end of file diff --git a/processed_dataset/calculation/0069.json b/processed_dataset/calculation/0069.json new file mode 100644 index 0000000000000000000000000000000000000000..d94d35b6968ab00f3bbaae1a56bf917b09f281c1 --- /dev/null +++ b/processed_dataset/calculation/0069.json @@ -0,0 +1,8 @@ +{ + "source_file": "./raw_volume-zh/volume1/exercise3.tex", + "problem_type": "calculation", + "problem": "问题11 求最大正整数 $n$,使得 $n$ 元集合 $S$ 同时满足:\n(1) $S$ 中的每个数均为不超过 2002 的正整数;\n(2) 对于 $S$ 的两个数 $a$ 和 $b$ (可以相同), 它们的乘积 $a b$ 不属于 $S$.", + "solution": "设集合 $A=\\{1\\}, B=\\left\\{2^1, 2^2, \\cdots, 2^{10}\\right\\}, C=\\left\\{3^1, 3^2, \\cdots, 3^6\\right\\}$, $D=\\left\\{5^1, 5^2, 5^3, 5^4\\right\\}, E=\\left\\{6^1, 6^2, 6^3, 6^4\\right\\}, X_i=\\left\\{i, i^2\\right\\}$, 其中 $i$ 不是 $2 、 3 、 4 、 5 、 6$ 的幂, 且满足 $7 \\leqslant i \\leqslant 44$. 于是, 集合 $S$ 中: 至少不包含 $A$ 中的 1 个元素; 至少不包含 $B$ 中的 5 个元素; 至少不包含 $C$ 中的 3 个元素; 至少不包含 $D$ 中的 2 个元素; 至少不包含 $E$ 中的 2 个元素; 至少不刨含 31 个 $X_i$ 中每个集合中的 1 个元素.\n所以, $S$ 中最多有 $2002-(1+5+3+2+2+31)=1958$ 个元素.\n例如, $S=\\{45,46,47, \\cdots, 2002\\}$ 即为满足条件且有 1958 个元素的集合.", + "remark": "", + "figures": [] +} \ No newline at end of file diff --git a/processed_dataset/calculation/0070.json b/processed_dataset/calculation/0070.json new file mode 100644 index 0000000000000000000000000000000000000000..9000d626041ffa06409905d5c81f55d8fb67bc26 --- /dev/null +++ b/processed_dataset/calculation/0070.json @@ -0,0 +1,8 @@ +{ + "source_file": "./raw_volume-zh/volume1/exercise3.tex", + "problem_type": "calculation", + "problem": "问题13 已知集合 $M=\\{A \\mid A$ 是各位数字互不相同的十位正整数, 且 $11111 \\mid A\\}$. 求 $|M|$.", + "solution": "因为 $A$ 的各位数字互不相同, 所以 $A \\equiv 0+1+\\cdots+9 \\equiv 0(\\bmod 9)$, 即 $9 \\mid A$. 又 $11111 \\mid A$, 而 $(9,11111)=1$, 故 $99999 \\mid A$. 设 $A=99999 A_0, A_0 \\in \\mathbf{Z}^{+}$. 因为 $10^9 \\frac{10^9}{10^5}=10^4, 10^5+1<\\frac{10^{10}}{10^5-1}<10^5+2$, 所以 $10^40, a_i \\in\\{0,1, \\cdots, 9\\} i=0,1,2,3,4$, 则 $A=A_0\\left(10^5-1\\right)=\\overline{a_0 a_1 a_2 a_3 a_4 00000}-\\overline{a_0 a_1 a_2 a_3} \\overline{a_4}$. 记 $A= c_4=a_4-1, c_0^{\\prime}=9-a_0, c_1^{\\prime}=9-a_1, c_2^{\\prime}=9-a_2, c_3^{\\prime}=9-a_3, c_4^{\\prime}=10- a_4$; 若 $a_4=0$, 则 $a_3 \\neq 0$ (否则 $A$ 的数位中有两个 0 ). 于是, $c_0=a_0, c_1=a_1, c_2=a_2, c_3=a_3-1, c_4=9, c_0^{\\prime}=9-a_0, c_1^{\\prime}=9-a_1, c_2^{\\prime}=9-a_2, c_3^{\\prime}= 10-a_3, c_4^{\\prime}=0$. 所以 $c_i+c_i^{\\prime}=9(0 \\leqslant i \\leqslant 4$, 且 $i \\in \\mathbf{N})$. (1)\n因此 $\\left\\{\\left(c_i, c_i^{\\prime}\\right) \\mid 0 \\leqslant i \\leqslant 4, i \\in \\mathbf{N}\\right\\}=\\{(0,9),(1,8),(2,7),(3,6)$, $(4,5),(9,0),(8,1),(7,2),(6,3),(5,4)\\}$. 因为 $c_0 \\neq 0$, 所以满足条件 (1) 的 $A$ 的个数为 $5 ! \\times 2^5-4 ! \\times 2^4=3456$. 反之, 任何满足条件 (1) 的数都可表示为: $\\overline{c_0 c_1 c_2 c_3 c_4 00000}+\\overline{c_0^{\\prime} c_1^{\\prime} c_2{ }^{\\prime} c_3^{\\prime} c_4^{\\prime}}=\\overline{c_0 c_1 c_2 c_3 c_4} \\cdot\\left(10^5-1\\right)+99999$, 必可被 11111 整除.\n因此, $|M|=3456$.", + "remark": "", + "figures": [] +} \ No newline at end of file diff --git a/processed_dataset/calculation/0071.json b/processed_dataset/calculation/0071.json new file mode 100644 index 0000000000000000000000000000000000000000..47281da368bab0ddf13305480c768366cd4a16af --- /dev/null +++ b/processed_dataset/calculation/0071.json @@ -0,0 +1,8 @@ +{ + "source_file": "./raw_volume-zh/volume1/exercise3.tex", + "problem_type": "calculation", + "problem": "问题14 设 $F$ 是所有有序 $n$ 元组 $\\left(A_1, A_2, \\cdots, A_n\\right)$ 构成的集合, 其中 $A_i(1 \\leqslant i \\leqslant n$ ) 都是集合 $\\{1,2,3, \\cdots, 2002\\}$ 的子集, 设 $\\mid A$ | 表示集合 $A$ 的元素的数目, 对 $F$ 中的所有元素 $\\left(A_1, A_2, \\cdots, A_n\\right)$, 求 $\\left|A_1 \\cup A_2 \\cup \\cdots \\cup A_n\\right|$ 的总和, 即\n$$\n\\sum_{\\left(A_1, A_2, \\cdots, A_n\\right) \\in F}\\left|A_1 \\cup A_2 \\cup \\cdots \\cup A_n\\right| .\n$$", + "solution": "$\\sum_{\\left(A_1, A_2, \\cdots, A_n\\right) \\in F}\\left|A_1 \\cup A_2 \\cup \\cdots \\cup A_n\\right|$ 的值, 即是 $\\{1,2, \\cdots, 2002\\}$ 中元素出现的次数之和.\n对每一个 $k \\in\\{1,2,3, \\cdots, 2002\\}$, 因为 $\\{1,2, \\cdots$, $2002\\}$ 共有 $2^{2002}$ 个子集,这些子集的全体记作集合 $M$. 其中不含有元素 $k$ 的子集共有 $2^{2001}$ 个, 记这些子集全体为 $N$. 当 $n$ 元组 $\\left(A_1, A_2, \\cdots, A_n\\right)$ 中 $A_i(i=1$, $2, \\cdots, n)$ 都取遍 $M$ 时, 共有 $2^{2002 n}$ 个这样的 $n$ 元组.\n但其中当 $A_i(i=1,2, \\cdots$, $n)$ 都不含有 $k$ 即 $A_i \\in N(i=1,2 \\cdots, n)$ 时, 这样的 $n$ 元组 $\\left(A_1, A_2, \\cdots, A_n\\right)$ 共有 $2^{2001 n}$ 个.\n所以 $\\sum_{\\left(A_1, A_2, \\cdots, A_n\\right) \\in F}\\left|A_1 \\cup A_2 \\cup \\cdots \\cup A_n\\right|$ 中 $k$ 出现 $2^{2002 n}-2^{2001 n}$次, 当 $k=1,2, \\cdots, 2002$ 时, 即 $\\sum_{\\left(A_1, A_2, \\cdots, A_n\\right) \\in F}\\left|A_1 \\cup A_2 \\cup \\cdots \\cup A_n\\right|=2002 \\cdot(2^{2002 n}-2^{2001 n} )$.", + "remark": "", + "figures": [] +} \ No newline at end of file diff --git a/processed_dataset/calculation/0072.json b/processed_dataset/calculation/0072.json new file mode 100644 index 0000000000000000000000000000000000000000..2b7bdaf281165a9926bb3e86111ad399bc3d8c32 --- /dev/null +++ b/processed_dataset/calculation/0072.json @@ -0,0 +1,8 @@ +{ + "source_file": "./raw_volume-zh/volume1/exercise3.tex", + "problem_type": "calculation", + "problem": "问题16 设 $S$ 为十进制中至多有 $n$ 位数字的所有非负整数所成的集合, $S_k$ 由 $S$ 中那些数字之和小于 $k$ 的元素组成.\n对于怎样的 $n$, 有 $k$ 存在, 使得 $|S|= 2\\left|S_k\\right|$ ?", + "solution": "对于任一个 $n$ 位数 $A=\\overline{a_1 a_2 \\cdots a_n}\\left(0 \\leqslant a_i \\leqslant 9, i=1,2, \\cdots, n\\right)$, 对应 $A \\rightarrow B=\\overline{b_1 b_2 \\cdots b_n}$ 是位数不超过 $n$ 的所有非负整数的集合到它自身的一个双射, 其中 $b_i=9-a_i, i=1,2, \\cdots, n$. 若记 $d(A)=a_1+a_2+\\cdots+a_n$, 则 $d(A)+d(B)=9 n$. 由此可见, 对于任意 $09 n-k$. 因而有 $\\left|\\left\\{A \\mid d(A)<\\frac{9 n}{2}\\right\\}\\right|=\\left|\\left\\{A \\mid d(A)>\\frac{9 n}{2}\\right\\}\\right|$.\n当 $n$ 为奇数时, $\\frac{9 n}{2}$ 不是整数, 故 (1) 中左右两端的集合之并集为 $S$, 所以当 $k=\\left[\\frac{9 n}{2}\\right]+1$ 时, $|S|=2\\left|S_k\\right|$. 当 $n$ 为偶数时, $\\frac{9 n}{2}$ 是整数, 当 $k==\\frac{9 n}{2}$ 时, $|S|> 2\\left|S_k\\right|,|S|<2\\left|S_{k+1}\\right|$, 这时满足要求的 $k$ 不存在.", + "remark": "", + "figures": [] +} \ No newline at end of file diff --git a/processed_dataset/calculation/0073.json b/processed_dataset/calculation/0073.json new file mode 100644 index 0000000000000000000000000000000000000000..750fc58e214dc0e955c5a574da372fa895edafed --- /dev/null +++ b/processed_dataset/calculation/0073.json @@ -0,0 +1,8 @@ +{ + "source_file": "./raw_volume-zh/volume1/exercise3.tex", + "problem_type": "calculation", + "problem": "问题17 $n, m$ 为正整数, $A=\\{1,2, \\cdots, n\\}, B_n^m=\\{\\left(a_1, a_2, \\cdots, a_m\\right) \\mid a_i \\in A, i=1,2, \\cdots, m\\}$ 满足:\n(1) $\\left|a_i-a_{i+1}\\right| \\neq n-1, i=1,2, \\cdots, m-1$;\n(2) $a_1, a_2, \\cdots, a_m(m \\geqslant 3)$ 中至少有三个不同.\n求 $B_n^m$ 和 $B_6^3$ 的元素的个数.", + "solution": "由题意, 若 $B_n^m$ 非空, 则 $n, m \\geqslant 3$. 计算仅满足条件 (1) 的 $\\left(a_1, a_2, \\cdots\\right.$, $\\left.a_m\\right)$ 的个数, 这时 1 与 $n$ 不相邻.\n记这样的 $\\left(a_1, a_2, \\cdots, a_m\\right)$ 有 $S_m$ 个, 其中 $x_m$ 个以 1 开头, $y_m$ 个以 $2, \\cdots, n-1$ 开头, $z_m$ 个以 $n$ 开头, 则 $S_m=x_m+y_m+z_m$, 那么有递推式 $x_{m+1}=x_m+y_m, y_{m+1}=(n-2)\\left(x_m+y_m+z_m\\right), z_{m+1}=y_m+z_m$. 以上三式相加得 $S_{m+1}=(n-1) S_m+(n-2) S_{m-1}$. (*)\n又易知 $S_1=n, S_2=n^2-2$, 而 (*) 的特征方程是 $t^2-(n-1) t-(n-2)=0$, 其特征根为 $t_{1,2}=\\frac{n-1 \\pm \\sqrt{n^2+2 n-7}}{2}$, 故 $S_m=A \\cdot\\left(\\frac{n-1+\\sqrt{n^2+2 n-7}}{2}\\right)^m+ B \\cdot\\left(\\frac{n-1-\\sqrt{n^2+2 n-7}}{2}\\right)^m$, 其中 $A 、 B$ 由 $S_1=A \\cdot\\left(\\frac{n-1+\\sqrt{n^2+2 n-7}}{2}\\right)+ B \\cdot\\left(\\frac{n-1-\\sqrt{n^2+2 n-7}}{2}\\right)=n$ 及 $S_2=A \\cdot\\left(\\frac{n-1+\\sqrt{n^2+2 n-7}}{2}\\right)^2+B \\cdot$\n$$\n\\begin{aligned}\n& \\left(\\frac{n-1-\\sqrt{n^2+2 n-7}}{2}\\right)^2=n^2-2 \\text { 确定, 解得 } A=\\frac{1+\\frac{n+1}{\\sqrt{n^2+2 n-7}}}{2}, \\\\\n& B=\\frac{1-\\frac{n+1}{\\sqrt{n^2+2 n-7}}}{2} . \\\\\n& \\text { 故 } S_m=\\left(\\frac{1+\\frac{n+1}{\\sqrt{n^2+2 n-7}}}{2}\\right)\\left(\\frac{n-1+\\sqrt{n^2+2 n-7}}{2}\\right)^m+ \\\\\n& \\left(\\frac{1-\\frac{n+1}{\\sqrt{n^2+2 n-7}}}{2}\\right)\\left(\\frac{n-1-\\sqrt{n^2+2 n-7}}{2}\\right)^m .\n\\end{aligned}\n$$\n下面再减去满足 (1) 而不满足 (2) 的.\n若 $a_1, a_2, \\cdots, a_m$ 全相同, 即 $a_1= \\cdots=a_m=k \\in\\{1,2, \\cdots, n\\}$, 这样的 $\\left(a_1, a_2, \\cdots, a_m\\right)$ 有 $n$ 个 $(n \\geqslant 3)$; 若 $a_1$, $a_2, \\cdots, a_m$ 中恰含两个不同的数, 选出这两个数有 $\\mathrm{C}_n^2$ 种方法, 但不能选 $\\{1, n\\}$, 故有 $\\mathrm{C}_n^2-1$ 种选法, 这样的 $\\left(a_1, a_2, \\cdots, a_m\\right)$ 有 $\\left(\\mathrm{C}_n^2-1\\right)\\left(2^m-2\\right)$ 种.\n综上所述,\n$$\n\\begin{aligned}\n& \\left|B_n^m\\right|=S_m-n-\\left(\\mathrm{C}_n^2-1\\right)\\left(2^m-2\\right)=\\left(\\frac{1+\\frac{n+1}{\\sqrt{n^2+2 n-7}}}{2}\\right) \\\\\n& {\\left[\\frac{n-1+\\sqrt{n^2+2 n-7}}{2}\\right)^m+\\left(\\frac{1-\\frac{n+1}{\\sqrt{n^2+2 n-7}}}{2}\\right)\\left[\\left(\\frac{n-1-\\sqrt{n^2+2 n-7}}{2}\\right)^m-\\right.} \\\\\n& n-\\left(\\mathrm{C}_n^2-1\\right)\\left(2^m-2\\right)(n, m \\geqslant 3) .\n\\end{aligned}\n$$\n特别地, 当 $n=6, m=3$ 时, $S_3=5 S_2+4 S_1=5 \\times 34+4 \\times 6=194$, 所以 $\\left|B_6^3\\right|=194-6-\\left(C_6^2-1\\right)\\left(2^3-2\\right)=104$.", + "remark": "", + "figures": [] +} \ No newline at end of file diff --git a/processed_dataset/calculation/0074.json b/processed_dataset/calculation/0074.json new file mode 100644 index 0000000000000000000000000000000000000000..aa0d37b963510258b76964017ba01e1ec2cf3d21 --- /dev/null +++ b/processed_dataset/calculation/0074.json @@ -0,0 +1,8 @@ +{ + "source_file": "./raw_volume-zh/volume1/exercise4.tex", + "problem_type": "calculation", + "problem": "问题2 设 $S$ 是一个有 6 个元素的集合,选取 $S$ 的两个子集(可以相同), 使得这两个子集的并集是 $S$, 选取的次序无关紧要, 例如, 一对子集 $\\{a, c\\},\\{b, c$, $d, e, f\\}$ 与一对子集 $\\{b, c, d, e, f\\},\\{a, c\\}$ 表示同一种取法.\n这样的取法有?种。", + "solution": "$435$. 设 $S=A \\cup B,|A| \\leqslant|B|$. 若 $|A|=0$, 则 $|B|=6$, 有 1 种取法; 若 $|A|=1$, 则 $|B|=6,5$, 有 $\\mathrm{C}_6^1 \\mathrm{C}_6^6+\\mathrm{C}_6^1 \\mathrm{C}_5^5=12$ 种取法; 类似地, 可分别计算出 $|A|=2,3,4,5,6$ 时的取法数.", + "remark": "", + "figures": [] +} \ No newline at end of file diff --git a/processed_dataset/calculation/0075.json b/processed_dataset/calculation/0075.json new file mode 100644 index 0000000000000000000000000000000000000000..28677c70e2a31d25a3c899bb8243a83891002706 --- /dev/null +++ b/processed_dataset/calculation/0075.json @@ -0,0 +1,8 @@ +{ + "source_file": "./raw_volume-zh/volume1/exercise4.tex", + "problem_type": "calculation", + "problem": "问题4 已知集合 $M$ 是 $\\{1,2, \\cdots, 2003\\}$ 的子集,且 $M$ 中任意两个元素之和均不能被 3 整除.\n求集合 $M$ 中元素个数的最大值.", + "solution": "考虑集合 $A=\\{3,6,9, \\cdots, 2001\\}, B=\\{1,4,7, \\cdots, 2002\\}, C= \\{2,5,8, \\cdots, 2003\\}$. 由 $A$ 中至多选一个元素与集合 $B$ 或 $C$ 构成一个新的集合 $M$. 因为 $|B|=|C|=668$, 所以 $M$ 中最多有 669 个元素.", + "remark": "", + "figures": [] +} \ No newline at end of file diff --git a/processed_dataset/calculation/0076.json b/processed_dataset/calculation/0076.json new file mode 100644 index 0000000000000000000000000000000000000000..ad24ae57b62c72dce8e910ed43b029fca3b26f56 --- /dev/null +++ b/processed_dataset/calculation/0076.json @@ -0,0 +1,8 @@ +{ + "source_file": "./raw_volume-zh/volume1/exercise4.tex", + "problem_type": "calculation", + "problem": "问题7 $X=\\{1,2, \\cdots, n\\}, A 、 B 、 C$ 是 $X$ 的分划, 即 $A \\cup B \\cup C=X$, 并且 $A 、 B$ 、 $C$ 两两的交集都是空集.\n如果从 $A 、 B 、 C$ 中各取一个元素, 那么每两个的和都不等于第三个.\n求\n$$\n\\max \\{\\min (|A|,|B|,|C|)\\} .\n$$", + "solution": "考虑奇偶性.\n如果 $A$ 由 $X$ 中的奇数组成, $B \\cup C$ 由 $X$ 中的偶数组成, 那么它们合乎题设要求.\n这时 $\\min (|A|,|B|,|C|)=\\left[\\frac{n}{4}\\right]$. 由此, 猜测 $\\max \\{\\min (|A|,|B|,|C|)\\}=\\left[\\frac{n}{4}\\right]$, 也就是恒有 $\\min (|A|,|B|$, $|C|) \\leqslant \\frac{n}{4}$.", + "remark": "", + "figures": [] +} \ No newline at end of file diff --git a/processed_dataset/calculation/0077.json b/processed_dataset/calculation/0077.json new file mode 100644 index 0000000000000000000000000000000000000000..a4f2e1b628863fbb03e25f82c3c6e1b642324e34 --- /dev/null +++ b/processed_dataset/calculation/0077.json @@ -0,0 +1,8 @@ +{ + "source_file": "./raw_volume-zh/volume1/exercise4.tex", + "problem_type": "calculation", + "problem": "问题10 设 $k$ 为正整数, $M_k$ 是 $2 k^2+k$ 与 $2 k^2+3 k$ 之间 (包括这两个数在内) 的所有整数组成的集合.\n能否将 $M_k$ 拆分为两个不相交的子集 $A 、 B$, 使得\n$$\n\\sum_{x \\in A} x^2=\\sum_{x \\in B} x^2 ?\n$$", + "solution": "当 $k=1$ 时, $3^2+4^2=5^2$; 当 $k=2$ 时, $10^2+11^2+12^2=13^2+14^2$. 猜想: $M_k$ 中前 $k+1$ 个数的平方和与后 $k$ 个数的平方和相等.", + "remark": "", + "figures": [] +} \ No newline at end of file diff --git a/processed_dataset/calculation/0078.json b/processed_dataset/calculation/0078.json new file mode 100644 index 0000000000000000000000000000000000000000..8f825d8815c3b048a966b1c9d8608827f7502da4 --- /dev/null +++ b/processed_dataset/calculation/0078.json @@ -0,0 +1,8 @@ +{ + "source_file": "./raw_volume-zh/volume1/exercise4.tex", + "problem_type": "calculation", + "problem": "问题11 给定正整数 $n \\geqslant 3$, 求具有下列性质的正整数 $m$ 的最小值: 把集合 $S= \\{1,2, \\cdots, m\\}$ 任意分成两个不相交的非空子集的并集, 其中必有一个子集内含有 $n$ 个数(不要求它们互不相同): $x_1, x_2, \\cdots, x_n$, 使 $x_1+x_2+\\cdots+x_{n-1}=x_n$.", + "solution": "若 $S$ 不具有题设性质,则存在 $S$ 的两个非空不相交的子集 $A$ 和 $B$ 使 $S=A \\cup B$, 并且 $A$ (或 $B$ ) 中任意 $n-1$ 个数 (不要求互不相同) 的和都不在 $A$ (或 $B$ ) 内.\n不妨设 $1 \\in A$, 则 $\\underbrace{1+1+\\cdots+1}_{n-1 \\text { 个 }}=n-1 \\in B$ (只要 $m \\geqslant n-1$ ), 从而 $\\underbrace{(n-1)+(n-1)+\\cdots+(n-1)}_{n-1 \\text { 个 }}=(n-1)^2 \\in A\\left(\\right.$ 只要 $\\left.m \\geqslant(n-1)^2\\right)$. 若 $n \\in A$, 则 $(n-1)^2=\\underbrace{n+n+\\cdots+n}_{n-2 \\uparrow}+1 \\in B$, 矛盾.\n若 $n \\in B$, 则 $\\underbrace{n+n+\\cdots+n}+ (n-1)=n^2-n-1 \\in A\\left(\\right.$ 只要 $\\left.m \\geqslant n^2-n-1\\right)$, 但 $\\underbrace{1+1+\\cdots+1}_{n-2 \\text { 个 }}+(n-1)^2= n^2-n-1 \\in B$, 矛盾.\n可见, 当 $m \\geqslant n^2-n-1$ 时, 集合 $S=\\{1,2, \\cdots, m\\}$ 具有题设性质.\n其次,对于集合 $S=\\left\\{1,2,3, \\cdots, n^2-n-2\\right\\}$, 令 $A=\\left\\{1,2, \\cdots, n-2, (n-1)^2,(n-1)^2+1, \\cdots, n^2-n-2\\right\\}, B=\\left\\{(n-1), n, \\cdots,(n-1)^2-1\\right\\}$, 则 $A \\cap B=\\varnothing, S=A \\cup B$. 若 $x_1, x_2, \\cdots, x_{n-1} \\in\\{1,2, \\cdots, n-2\\} \\subset A$, 则 $n-1 \\leqslant x_1+x_2+\\cdots+x_{n-1} \\leqslant(n-1)(n-2)<(n-1)^2$, 故 $x_1+x_2+\\cdots+ x_{n-1} \\notin A$. 若 $x_1, x_2, \\cdots, x_{n-1}$ 中至少有一个 $\\in\\left\\{(n-1)^2,(n-1)^2+1, \\cdots\\right.$, $\\left.n^2-n-2\\right\\}$, 则 $x_1+x_2+\\cdots+x_{n-1} \\geqslant \\underbrace{1+1+\\cdots+1}_{n-2 \\text { 个 }}+(n-1)^2=n^2-n-$ 1,故 $x_1+x_2+\\cdots+x_{n-1} \\notin A$. 若 $x_1, x_2, \\cdots, x_{n-1} \\in B$, 则 $x_1+x_2+\\cdots+ x_{n-1} \\geqslant(n-1)^2$,故 $x_1+x_2+\\cdots+x_{n-1} \\notin B$. 可见, $m=n^2-n-2$ 时, 集合 $S=\\{1,2, \\cdots, m\\}$ 不具有题设条件.\n所求 $m$ 的最小值为 $n^2-n-1$.", + "remark": "", + "figures": [] +} \ No newline at end of file diff --git a/processed_dataset/calculation/0079.json b/processed_dataset/calculation/0079.json new file mode 100644 index 0000000000000000000000000000000000000000..ab396eb27e2c8e3825220ef101f34c881a1ac4fa --- /dev/null +++ b/processed_dataset/calculation/0079.json @@ -0,0 +1,8 @@ +{ + "source_file": "./raw_volume-zh/volume1/exercise4.tex", + "problem_type": "calculation", + "problem": "问题12 设 $n$ 是大于 3 的自然数,且具有下列性质: 把集合 $S_n=\\{1,2, \\cdots, n\\}$ 任意分为两个不相交的子集, 总有某个子集, 它含有三个数 $a 、 b 、 c$ (允许 $a=b)$, 使得 $a b=c$. 求这样的 $n$ 的最小值.", + "solution": "若 $n \\geqslant 3^5$, 则 $3,3^2, 3^4, 3^5 \\in S_n$. 设集合 $S_n$ 分为两个不相交的子集 $A$ 和 $B, 3 \\in A$, 且 $A$ 和 $B$ 不满足题中条件.\n于是 $3^2 \\in B, 3^4 \\in A, 3^3 \\in B, 3^5 \\in B$. 这时 $B$ 中三个元素 $3^2, 3^3, 3^5$ 满足 $a b=c$ 矛盾.\n这表明 $n \\geqslant 3^5$ 时, 把集合 $S_n$ 任意分为两组, 总有某个组, 具有题中性质, 即所求最小值不超过 $3^5=243$.\n另一方面, 取 $n=242$, 且设 $A=\\{k \\mid 9 \\leqslant k \\leqslant 80\\}, B=\\{k \\mid 3 \\leqslant k \\leqslant 8$,或 $81 \\leqslant k \\leqslant 242\\}$, 则 $S_{242}=A \\cup B$, 而且 $A$ 和 $B$ 都不具有题中性质.\n当 $n<$ 242 时, 将 $S_n$ 分为两组 $A \\cap S_n$ 和 $B \\cap S_n$, 则 $A \\cap S_n$ 和 $B \\cap S_n$ 也都不具有题中性质.\n故 $n$ 的最小值为 243 .", + "remark": "", + "figures": [] +} \ No newline at end of file diff --git a/processed_dataset/calculation/0080.json b/processed_dataset/calculation/0080.json new file mode 100644 index 0000000000000000000000000000000000000000..10964e47d8972ab1fce29a8fb5ccca266c230af5 --- /dev/null +++ b/processed_dataset/calculation/0080.json @@ -0,0 +1,8 @@ +{ + "source_file": "./raw_volume-zh/volume1/exercise4.tex", + "problem_type": "calculation", + "problem": "问题14 试求所有正整数 $k$, 使得集合 $M=\\{1990,1991, \\cdots, 1990+k\\}$ 可以分解为两个不相交的子集 $A$ 与 $B$, 且使两集合中的元素之和相等.", + "solution": "由 $\\sum_{n=0}^k(1990+n)=1990(k+1)+\\frac{1}{2} k(k+1)$ 为偶数, 知 $k(k+1) \\equiv 0(\\bmod 4)$. 由此可知 $k \\equiv 0(\\bmod 4)$ 或 $k \\equiv 3(\\bmod 4)$. 换句话说, $k=4 m+1$ 与 $k=4 m+2(m=0,1,2, \\cdots)$ 都不满足本题要求.\n设 $k \\equiv 3(\\bmod 4)$, 则 $4 \\mid(k+1)$. 令 $A=\\{1990+j \\mid j=4 m, 4 m+3,m=0,1, \\cdots,\\left[\\frac{k}{4}\\right]\\}, B=\\{1990+j \\mid j=4 m+1,4 m+2, m=0,1, \\cdots,\\left[\\frac{k}{4}\\right] \\}$, 易见,这样的 $A$ 和 $B$ 满足要求.\n设 $k \\equiv 0(\\bmod 4)$, 于是 $k=4 m, m \\in \\mathbf{N}^*$. 因为 $|M|$ 为奇数, 故有 $|A| \\neq|B|$. 不妨设 $|A|>|B|$, 于是 $|A| \\geqslant 2 m+1,|B| \\leqslant 2 m$. 因而, $A$ 中元素之和不小于 $\\sum_{n=0}^{2 m}(1990+n), B$ 中元素之和不大于 $\\sum_{n=2 m+1}^{4 m}(1990+n)$, 故得 $\\sum_{n=0}^{2 m}(1990+n) \\leqslant \\sum_{n=2 m+1}^{4 m}(1990+n)=(2 m)^2+\\sum_{n=1}^{2 m}(1990+n)$. 解得 $m \\geqslant 23$, $k \\geqslant 92$. 当 $k=92$ 时, 令 $A_1=\\{1990,1991, \\cdots, 1990+46\\}, B_1=\\{1990+47$, $1990+48, \\cdots, 1990+92\\}$. 两集合中元素之和分别为 $S_{A_1}=1990 \\times 47+1081$, $S_{B_1}=1990 \\times 46+1081+2116, S_{B_1}-S_{A_1}=126$. 再令 $A=A_1 \\cup\\{2053\\}- \\{1990\\}, B=B_1 \\cup\\{1990\\}-\\{2053\\}$, 则集合 $A$ 和 $B$ 即为满足题中要求的分解.\n当 $k>92$ 时, 我们将前 93 个数分组如上, 而将后面的 $k-92=4 m$ 个数中的每相邻 4 数按前面 $k \\equiv 3(\\bmod 4)$ 的办法处理即可得到所需要的分解.\n综上可知, 所求的所有 $k$ 的集合为 $\\{k \\mid k=4 m+3, m=0,1, \\cdots ; k= 4 m, m=23,24, \\cdots\\}$.", + "remark": "", + "figures": [] +} \ No newline at end of file diff --git a/processed_dataset/calculation/0081.json b/processed_dataset/calculation/0081.json new file mode 100644 index 0000000000000000000000000000000000000000..4633f89dcde3b5e8fa9813349f4843785d1822df --- /dev/null +++ b/processed_dataset/calculation/0081.json @@ -0,0 +1,8 @@ +{ + "source_file": "./raw_volume-zh/volume1/exercise5.tex", + "problem_type": "calculation", + "problem": "问题1 在集合 $M=\\{1,2, \\cdots, 10\\}$ 的所有子集中,有这样一族不同的子集,它们两两的交集都不是空集,那么这族子集最多有?个.", + "solution": "$2^9$. 显然不含空集.\n按所含元素的多少把这族子集分为 10 类, 设含 $i$ 个元素的子集为 $A_i$, 其个数为 $a_i$. 易知 $a_1+a_2+\\cdots+a_{10} \\leqslant \\mathrm{C}_9^0+\\mathrm{C}_9^1+\\cdots+\\mathrm{C}_9^9=2^9$.", + "remark": "", + "figures": [] +} \ No newline at end of file diff --git a/processed_dataset/calculation/0082.json b/processed_dataset/calculation/0082.json new file mode 100644 index 0000000000000000000000000000000000000000..08128535eea6bc5b1ae353841806f372372a4142 --- /dev/null +++ b/processed_dataset/calculation/0082.json @@ -0,0 +1,8 @@ +{ + "source_file": "./raw_volume-zh/volume1/exercise5.tex", + "problem_type": "calculation", + "problem": "问题2 设集合 $M=\\{1,2, \\cdots, 1000\\}$, 现对 $M$ 的任一非空子集 $X$, 令 $\\alpha_X$ 表示 $X$ 中最大数与最小数之和.\n则所有这样的 $\\alpha_X$ 的算术平均值为", + "solution": "1001 . 构造子集 $X^{\\prime}=\\{1001-x \\mid x \\in X\\}$, 则所有非空子集分成两类 $X^{\\prime}=X$ 和 $X^{\\prime} \\neq X$. 当 $X^{\\prime}=X$ 时, 必有 $X^{\\prime}=X=M$, 于是 $\\alpha_X=1001$. 当 $X^{\\prime} \\neq X$ 时, 设 $x 、 y$ 分别是 $X$ 中的最大数与最小数, 则 $1001-x 、 1001-y$ 分别是 $X^{\\prime}$ 中的最小数与最大数.\n于是, $\\alpha_X=x+y, \\alpha_{X^{\\prime}}=2002-x-y$. 从而, $\\frac{\\alpha_X+\\alpha_{X^{\\prime}}}{2}=1001$. 因此, 所求的 $\\alpha_X$ 的算术平均值为 1001 .", + "remark": "", + "figures": [] +} \ No newline at end of file diff --git a/processed_dataset/calculation/0083.json b/processed_dataset/calculation/0083.json new file mode 100644 index 0000000000000000000000000000000000000000..9a1e974c6c8c8a6c7903218e4643bca8a1c69beb --- /dev/null +++ b/processed_dataset/calculation/0083.json @@ -0,0 +1,8 @@ +{ + "source_file": "./raw_volume-zh/volume1/exercise5.tex", + "problem_type": "calculation", + "problem": "问题3 对于 $\\{1,2, \\cdots, n\\}$ 和它的每个非空子集, 我们定义“交替和”如下:把子集中的数按从大到小的顺序排列, 然后从最大的数开始交替地加减各数 (例如, $\\{1,2,4,6,9\\}$ 的交替和是 $9-6+4-2+1=6$, 而 $\\{5\\}$ 的交替和就是 5). 对于 $n=7$; 求所有这些交替和的总和.", + "solution": "集合 $\\{1,2,3,4,5,6,7\\}$ 中每个元素在子集中均出现 $2^6=64$ 次.\n可计算 $1,2,3,4,5,6$ 在子集中按从大到小的顺序排列时各有 32 次在奇数位, 32 次在偶数位,因此子集中这些数的交替和的总和为 0 ; 而 7 也出现 64 次, 且均取正值, 所以所有子集的交替和的总和为 $7 \\times 64=448$.", + "remark": "", + "figures": [] +} \ No newline at end of file diff --git a/processed_dataset/calculation/0084.json b/processed_dataset/calculation/0084.json new file mode 100644 index 0000000000000000000000000000000000000000..a8d606651481ac65edf692b1f25551cf93c0c0ee --- /dev/null +++ b/processed_dataset/calculation/0084.json @@ -0,0 +1,8 @@ +{ + "source_file": "./raw_volume-zh/volume1/exercise5.tex", + "problem_type": "calculation", + "problem": "问题4 $X=\\{1,2, \\cdots, 2 n+1\\} . A$ 是 $X$ 的子集, 且具有性质: $A$ 中任意两个数的和不在 $A$ 中, 求 $\\max |A|$.", + "solution": "取 $A=\\{1,3,5, \\cdots, 2 n+1\\}$ 合乎要求.\n故 $\\max |A| \\geqslant n+1$. 另一方面可设合乎要求的 $A$ 中有 $k(k \\leqslant n+1)$ 个奇数: $a_1>a_2>\\cdots>a_k$. 眇然,偶数 $a_1-a_2b$, 故知这样的勾股数共有下列 11 组: $(10,8,6),(26,24,10),(15,12,9),(17,15,8),(39,36,15),(25,24,7),(40,32,24),(41,40,9),(45,36,27),(25,20,15),(50,40,30)$. 注意到前 9 组勾股数中每组都有 $8 、 9 、 24 、 36$ 这 4 个数之一, 可知集合 $M=S-\\{5,8,9,20,24,30,35,36,50\\}$ 中任何 3 个数都不是一组勾股数.\n所以,所求的最小正整数 $n \\geqslant 42$.\n另一方面, 在下列 9 组勾股数 $(3,4,5),(7,24,25),(8,15,17),(9, 40,41),(12,35,37),(14,48,50),(16,30,34),(20,21,29),(27, 36,45)$ 中出现的 27 个数互不相同.\n故对 $S$ 的任一个 42 元子集 $M$, 不在 $M$ 中的 8 个数至多属于其中的 8 组, 从而至少有 1 组勾股数全在 $M$ 中.\n故所求的最小正整数 $n=42$.", + "remark": "", + "figures": [] +} \ No newline at end of file diff --git a/processed_dataset/calculation/0088.json b/processed_dataset/calculation/0088.json new file mode 100644 index 0000000000000000000000000000000000000000..1347536dc01b36d9d1898aa537caa7f81d2d463c --- /dev/null +++ b/processed_dataset/calculation/0088.json @@ -0,0 +1,8 @@ +{ + "source_file": "./raw_volume-zh/volume1/exercise5.tex", + "problem_type": "calculation", + "problem": "问题10 设 $P$ 是一个奇质数, 考虑集合 $\\{1,2, \\cdots, 2 p\\}$ 满足以下两个条件的子集 $A$ :\n(i) $A$ 恰有 $p$ 个元素;\n(ii) $A$ 中所有元素之和可被 $p$ 整除.\n试求所有这样的子集 $A$ 的个数.", + "solution": "记 $U=\\{1,2, \\cdots, p\\}, V=\\{p+1, p+2, \\cdots, 2 p\\}, W=\\{1,2, \\cdots, 2 p\\}$, 除去 $U$ 和 $V$ 而外, $W$ 的所有其他的 $p$ 元子集 $E$ 都使得 $E \\cap U \\neq \\varnothing$, $E \\cap V \\neq \\varnothing$. 若 $W$ 的两个这样的 $p$ 元子集 $S$ 和 $T$ 同时满足: $S \\cap V=T \\cap V$; 只要编号适当, $S \\cap U$ 的元素 $s_1, s_2, \\cdots, s_m$ 和 $T \\cap U$ 的元素 $t_1, t_2, \\cdots, t_m$ 对适当的 $k \\in\\{0,1, \\cdots, p-1\\}$ 满足同余式组 $s_i-t_i \\equiv k(\\bmod p), i=1,2, \\cdots, m$. 就约定将这两个子集 $S$ 和 $T$ 归人同一类.\n对于同一类中的不同子集 $S$ 和 $T$, 显然有 $k \\neq 0$, 因而 $\\sum_{i=1}^p s_i-\\sum_{i=1}^p t_i \\equiv m k \\not \\neq 0(\\bmod p)$. 对于同一类中的不同子集, 它们各自元素的和模 $p$ 的余数不相同.\n因而每一类恰含 $p$ 个子集, 其中仅一个适合题目的条件(ii).\n综上所述, 在 $W=\\{1,2, \\cdots, 2 p\\}$ 的 $\\mathrm{C}_{2 p}^p$ 个 $p$ 元子集当中, 除去 $U$ 和 $V$ 这两个特定子集外, 每 $p$ 个子集分成一类, 每类恰有一个子集满足题目的条件 (ii). 据此很容易算出, $W=\\{1,2, \\cdots, 2 p\\}$ 的适合条件(i) 和(ii) 的子集的总数为 $\\frac{1}{p}\\left(\\mathrm{C}_{2 p}^p-2\\right)+2$.", + "remark": "", + "figures": [] +} \ No newline at end of file diff --git a/processed_dataset/calculation/0089.json b/processed_dataset/calculation/0089.json new file mode 100644 index 0000000000000000000000000000000000000000..a27ecea5af9b38d7b70c47f564361172cd66a3a6 --- /dev/null +++ b/processed_dataset/calculation/0089.json @@ -0,0 +1,8 @@ +{ + "source_file": "./raw_volume-zh/volume1/exercise5.tex", + "problem_type": "calculation", + "problem": "问题12 设 $S=\\{1,2, \\cdots, 50\\}$. 求最小自然数 $k$, 使 $S$ 的任一 $k$ 元子集中都存在两个不同的数 $a$ 和 $b$, 满足 $(a+b) \\mid a b$.", + "solution": "设有 $a 、 b \\in S$ 满足 $(a+b) \\mid a b$. 记 $c=(a, b)$, 于是 $a=c a_1, b=c b_1$, 其中 $a_1 、 b_1 \\in \\mathbf{N}^*$ 且有 $\\left(a_1, b_1\\right)=1, a_1 \\neq b_1$, 不妨设 $a_1>b_1$. 由于 $a+b= c\\left(a_1+b_1\\right), a b=c^2 a_1 b_1$, 因此 $\\left(a_1+b_1\\right) \\mid c a_1 b_1$. 又由于 $\\left(a_1+b_1, a_1\\right)=1$, $\\left(a_1+b_1, b_1\\right)=1$, 因此 $\\left(a_1+b_1\\right) \\mid c$. 而 $a+b \\leqslant 99$, 即 $c\\left(a_1+b_1\\right) \\leqslant 99$, 所以 $3 \\leqslant a_1+b_1 \\leqslant 9$. 由此可知, $S$ 中满足 $(a+b) \\mid a b$ 的不同数对 $(a, b)$ 共有 23 对: $a_1+b_1=3$ 时, 有 $(6,3) ,(12,6) ,(18,9) ,(24,12) ,(30,15) ,(36,18)$ , $(42,21),(48,24) ; a_1+b_1=4$ 时, 有 $(12,4),(24,8),(36,12), (48, 16) ; a_1+b_1=5$ 时, 有 $(20,5), (40,10),(15,10),(30,20),(45,30)$; $a_1+b_1=6$ 时, 有 $(30,6) ; a_1+b_1=7$ 时, 有 $(42,7),(35,14) ,(28,21)$; $a_1+b_1=8$ 时,有 $(40,24) ; a_1+b_1=9$ 时,有 $(45,36)$.\n令 $M=\\{6,12,15,18,20,21,24,35,40,42,45,48\\}$. 则上述 23 个数对中的每一个数对都至少包含 $M$ 中的 1 个元素.\n令 $T=S-M$. 则 $T$ 中任何两数都不能成为满足要求的数对 $(a, b)$. 因为 $|T|=38$, 所以所求最小自然数 $K \\geqslant 39$.\n另一方面,下列 12 个满足题中要求的数对互不相交: $(6,3),(12,4)$, $(20,5),(42,7),(24,8),(18,9),(40,10),(35,14),(30,15),(48, 16),(28,21),(45,36)$. 对于 $S$ 中任一 39 元子集 $R$, 它只比 $S$ 少 11 个元素, 而这 11 个元素至多属于上述 12 个数对中的 11 个, 因此, 必有 12 对中的 1 对属于 $R$. 故所求的最小自然数 $K=39$.", + "remark": "", + "figures": [] +} \ No newline at end of file diff --git a/processed_dataset/calculation/0090.json b/processed_dataset/calculation/0090.json new file mode 100644 index 0000000000000000000000000000000000000000..5ca5a83b3cce2202100c4c9517348940a12d0867 --- /dev/null +++ b/processed_dataset/calculation/0090.json @@ -0,0 +1,8 @@ +{ + "source_file": "./raw_volume-zh/volume1/exercise5.tex", + "problem_type": "calculation", + "problem": "问题13 集合 $Z$ 由 $n$ 个元素组成, $Z$ 中最多有多少个这样的 3 元子集, 使得其中任意两个 3 元子集都恰好有一个公共元.", + "solution": "用 $k_n$ 表示所求的数.\n设从集合 $Z$ 中取出 $k_n$ 个 3 元子集, 其中任意两个都恰好有一个公共元, 分三种可能情况:\n(1) 集合 $Z$ 中的每个元素都至多出现在两个 3 元子集中.\n设 $\\{a, b, c\\}$ 是其中一个 3 元子集, 则其他任何一个 3 元子集都与 $\\{a, b, c\\}$ 相交, 而且所有其他子集中至多有一个含元素 $a$, 至多有一个含元素 $b$, 至多有一个含元素 $c$. 因此, 子集最多有 $1+3 \\times 1=4$ 个, 即 $k_n \\leqslant 4$.\n(2) 集合 $Z$ 中有一个元素出现在三个 3 元子集中,但集合 $Z$ 的每一个元素至多出现在三个 3 元子集中, 则设 $\\{a, b, c\\}$ 是其中一个 3 元子集, 于是其他任意一个子集都与它相交, 而且所有其他子集中至多有两个集合包含元素 $a$,至多有两个集合包含元素 $b$, 至多有两个集合包含元素 $c$. 因此, 所有子集至多有 $1+3 \\times 2=7$ 个, 即 $k_n \\leqslant 7$.\n(3) 集合 $Z$ 中含有元素 $a$, 它至少属于 4 个 3 元子集, 则这 4 子集也应包含元素 $a$. 否则, 这样的子集与 4 个 3 元子集中每一个恰有一个公共元素, 所以它至少含有 4 个元素.\n于是在此情形下, 有 $1+2 k_n \\leqslant n$, 即 $k_n \\leqslant\\left[\\frac{n-1}{2}\\right]$.\n当 $n=1,2,3,4,5$ 时, 显然有 $k_1=k_2=0, k_3=k_4=1, k_5=2$. 当 $n=6$ 时, 集合 $Z$ 的每个元素至多属于 2 个 3 元子集, 否则 3 元子集的并将含有 7 个元素, 因此情形 (1)成立, 即 $k_6 \\leqslant 4$. 另一方面, 设 $Z=\\{a, b, c, d, e, f\\}$, 则 3 元子集取为 $\\{a, b, c\\},\\{c, d, e\\},\\{e, f, a\\},\\{b, d, f\\}$.于是 $k_6=4$.\n设 $a \\in\\{7,8,9, \\cdots, 16\\}$, 则当出现情形 (3) 时, 3 元子集的个数至多为 $\\left[\\frac{16-1}{2}\\right]=7$, 当出现情形 (1)、(2) 时, 3 元子集的个数也至多为 7 . 另一方面, 如果集合 $Z$ 的元素中的 7 个为 $a, b, c, d, e, f, g$, 则有 7 个 3 元子集: $\\{a, b, c\\},\\{c, d, e\\},\\{e, f, a\\},\\{b, d, f\\},\\{a, g, d\\},\\{b, g, e\\},\\{c, g, f\\}$.于是 $n=7,8,9, \\cdots, 16$ 时, $k_n=7$.\n最后, 当 $n \\geqslant 17$ 时, 则不论哪种情形总有 $k_n \\leqslant\\left[\\frac{n-1}{2}\\right]$. 而且如下选取3元子集时达到上界: 集合 $Z$ 中取一个元素为所有 3 元子集的公共元,并将所有其他元素配成对 (当 $n$ 为偶数时需去掉一个), 且与公共元素一起组成 3 元子集,故当 $n \\geqslant 17$ 时, $k_n=\\left[\\frac{n-1}{2}\\right]$.", + "remark": "", + "figures": [] +} \ No newline at end of file diff --git a/processed_dataset/calculation/0091.json b/processed_dataset/calculation/0091.json new file mode 100644 index 0000000000000000000000000000000000000000..b14395d7b79249acd07911ea770f50c0bb024822 --- /dev/null +++ b/processed_dataset/calculation/0091.json @@ -0,0 +1,8 @@ +{ + "source_file": "./raw_volume-zh/volume1/exercise5.tex", + "problem_type": "calculation", + "problem": "问题14 设 $S=\\{1,2, \\cdots, 15\\}$, 从 $S$ 中取出 $n$ 个子集 $A_1, A_2, \\cdots, A_n$, 满足下列条件:\n(i) $\\left|A_i\\right|=7, i=1,2, \\cdots, n$;\n(ii) $\\left|A_i \\cap A_j\\right| \\leqslant 3,1 \\leqslant i90$, 无法使 (iii) 成立.\n所以, 为使条件 (i) 一 (iii) 成立, $S$ 中的每个数都至少属于 7 个子集.\n这样一来, 必有 $n \\geqslant 15$.\n用字典排列法可以写出满足题中要求的 15 个 7 元子集: $\\{1,2,3,4,5, 6,7\\},\\{1,2,3,8,9,10,11\\},\\{1,2,3,12,13,14,15\\},\\{1,4,5,8,9, 12,13\\},\\{1,4,5,10,11,14,15\\},\\{1,6,7,8,9,14,15\\},\\{1,6,7,10,11,12,13\\},\\{2,4,6,8,10,12,14\\},\\{2,4,6,9,11,13,15\\},\\{2,5,7,8,10,13,15\\},\\{2,5,7,9,11,12,14\\},\\{3,4,7,8,11,12,15\\},\\{3,4,7,9,10,13,14\\},\\{3,5,6,8,11,13,14\\},\\{3,5,6,9,10,12,15\\}$.", + "remark": "", + "figures": [] +} \ No newline at end of file diff --git a/processed_dataset/calculation/0092.json b/processed_dataset/calculation/0092.json new file mode 100644 index 0000000000000000000000000000000000000000..c9f968af24c4ec96cbefd7b558b3653b89cebc7b --- /dev/null +++ b/processed_dataset/calculation/0092.json @@ -0,0 +1,8 @@ +{ + "source_file": "./raw_volume-zh/volume1/exercise5.tex", + "problem_type": "calculation", + "problem": "问题15 设 $S \\subseteq\\{1,2, \\cdots, 2002\\}$. 对任意 $a, b \\in S$ ( $a 、 b$ 可以相同), 总有 $a b \\notin S$, 求 $|S|$ 的最大值.", + "solution": "首先, $1 \\notin S$. 其次, 若 $a \\in\\{2,3,4,5,6\\}$ 且 $a \\in S$, 则以下 45 对数对中, 每对的两个数不能同时属于 $S:(1, a),(2,2 a), \\cdots,\\left(a-1,(a-1) a\\right),(a+1,(a+1) a),(a+2,(a+2) a), \\cdots,(2 a-1,(2 a-1) a), \\cdots(44 a+1, (44 a+1) a),(44 a+2,(44 a+2) a), \\cdots,(45 a-1,(45 a-1) a)$. 由于 $(45 a- 1) a \\leqslant(45 \\times 6-1) \\times 6<2002$, 所以以上 90 个小于 2002 的数中至少有一半不属于 $S$. 从而 $|S| \\leqslant 2002-45=1957$.\n再次, 若 $2,3,4,5,6 \\notin S$, 考虑以下 38 个数对: $\\left(7,7^2\\right),\\left(8,8^2\\right), \\cdots, \\left(k, k^2\\right), \\cdots,\\left(44,44^2\\right)$, 若有某一对中的两个数 $k_0, k_0^2 \\in S$, 则令 $a=b=k_0$, 有 $a, b, a b \\in S$, 与题设矛盾! 因此这里至少有 38 个数不属于 $S$, 再减去 1 , $2, \\cdots, 6$, 有 $|S| \\leqslant 2002-38-6=1958$. 又 $S=\\{45,46,47, \\cdots, 2002\\}$ 满足要求.\n对任意 $a, b \\in S, a b \\geqslant 45^2=2025>2002$, 即 $a b \\notin S$, 此时 $|S|=1958$.", + "remark": "", + "figures": [] +} \ No newline at end of file diff --git a/processed_dataset/calculation/0093.json b/processed_dataset/calculation/0093.json new file mode 100644 index 0000000000000000000000000000000000000000..824166304ceb7fdd7b2ba439de429395c35b0261 --- /dev/null +++ b/processed_dataset/calculation/0093.json @@ -0,0 +1,8 @@ +{ + "source_file": "./raw_volume-zh/volume1/exercise5.tex", + "problem_type": "calculation", + "problem": "问题16 称子集 $A \\subseteq M=\\{1,2, \\cdots, 11\\}$ 是好的,如果它有下述性质: “如果 $2 k \\in A$, 则 $2 k-1 \\in A$, 且 $2 k+1 \\in A$ ” (空集和 $M$ 都是好的). $M$ 有多少个好子集?", + "solution": "设 $n(A)$ 为属于 $A$ 的偶数的个数.\n情形 $0: n(A)=0$. 只须确定 $A$ 中的奇数.\n有 $2^6$ 个好子集.\n情形 $1: n(A)=1$. 对偶数的选取有 5 种可能性.\n有 $5 \\times 2^4$ 个好集合 $A$.\n情形 $2: n(A)=2$. (I ) 在好子集中的偶数是相邻的.\n有 $4 \\times 2^3$ 个好子集.\n(II) $A$ 中的两个偶数不相邻.\n有 $6 \\times 2^2$ 个好子集.\n共有 56 个好子集.\n情形 $3: n(A)=3$. (I ) A 中的偶数是相邻的.\n有 $3 \\times 2^2$ 个好子集.\nII) $A$ 中的任意两个偶数都不相邻.\n$A=\\{1,2,3,5,6,7,9,10,11\\}$ 是惟一的选择.\n(III) $A$ 的 3 个偶数中恰好两个是相邻的.\n有 $6 \\times 2=12$ 个好子集.\n共有 25 个好子集.\n情形 $4: n(A)=4$. ( I ) $2 \\notin A$ 或 $10 \\notin A$. 有 4 个好子集.\n(II ) $2 \\in A$ 且 $10 \\in A$. 有 3 个好子集.\n共有 7 个好子集.\n情形 $5: n(A)=5$, 则 $A=M, 1$ 种可能性.\n最后, 好子集的总数是 $2^6+5 \\times 2^4+56+25+7+1=233$.", + "remark": "", + "figures": [] +} \ No newline at end of file diff --git a/processed_dataset/calculation/0094.json b/processed_dataset/calculation/0094.json new file mode 100644 index 0000000000000000000000000000000000000000..e4eebe8178a37cdc1588484bdfbda5fb82b57a6e --- /dev/null +++ b/processed_dataset/calculation/0094.json @@ -0,0 +1,8 @@ +{ + "source_file": "./raw_volume-zh/volume1/exercise5.tex", + "problem_type": "calculation", + "problem": "问题17 设 $n$ 为给定的正整数, $D_n$ 为 $2^n 3^n 5^n$ 的所有正因数所成的集合, $S \\subseteq D_n$, 且 $S$ 中任一数都不能整除 $S$ 中另一数.\n求 $|S|$ 的最大值.", + "solution": "显然 $D_n$ 中的每一个数都有形式 $2^\\alpha 3^\\beta 5^\\gamma$, 其中 $0 \\leqslant \\alpha, \\beta, \\gamma \\leqslant n$, 下面用数组 $(\\alpha, \\beta, \\gamma)$ 表示数 $2^\\alpha 3^\\beta 5^\\gamma$. 考察如下集合: $A_{i, j}=\\{(i, j, \\alpha), 0 \\leqslant \\alpha \\leqslant n- j\\} \\cup\\{(i, \\beta, n-j), j \\leqslant \\beta \\leqslant n\\}, i==0,1,2, \\cdots, n, j=0,1,2, \\cdots,\\left[\\frac{n}{2}\\right]$; $B_{i, j}=\\{(k, i, j), k=0,1,2, \\cdots, n\\}, i=\\left[\\frac{n}{2}\\right]+1, \\cdots, n, j=0,1,2, \\cdots,n-\\left[-\\frac{n}{2}\\right]-1$. 共有 $\\left(\\left[\\frac{n}{2}\\right]+1\\right) \\cdot(n+1)+\\left(n-\\left[\\frac{n}{2}\\right]\\right)^2=\\frac{3(n+1)^2+1}{4}(n$ 为偶数), 或 $\\left(\\left[\\frac{n}{2}\\right]+1\\right)(n+1)+\\left(n-\\left[\\frac{n}{2}\\right]\\right)^2=\\frac{3(n+1)^2}{4}(n$ 为奇数), 即 $\\left[\\frac{3(n+1)^2+1}{4}\\right]$ 个集合.\n又因为每个集合中的数至多有一个属于 $S$, 可得以上集合互不相交且包含所有的数, 即 $\\cup A_{i, j} \\cup B_{i, j}=D_n$. 因而可得\n$$\n|S| \\leqslant\\left[\\frac{3(n+1)^2+1}{4}\\right]\n$$\n另一方面, 考察满足 $\\alpha+\\beta+\\gamma=n+t$ (其中 $t=\\left[\\frac{n}{2}\\right]$) 的数组 $(\\alpha, \\beta, \\gamma)$ 的个数.\n由条件 $0 \\leqslant \\alpha 、 \\beta 、 \\gamma \\leqslant n$, 可得 $(n+t+1)+(n+t)+\\cdots+1-3[t+ (t-1)+\\cdots+1]=\\left[\\frac{3(n+1)^2+1}{4}\\right]$. (其中 $t=\\left[\\frac{n}{2}\\right]$ 当 $\\alpha, \\beta, \\gamma$ 中有一个大于 $n$ 时, 共有 $3(t+(t-1)+\\cdots+1)$ 组解, 故应除去) 取 $S= \\left\\{2^\\alpha 3^\\beta 5^\\gamma \\mid \\alpha+\\beta+\\gamma=n+\\left[\\frac{n}{2}\\right], 0 \\leqslant \\alpha, \\beta, \\gamma \\leqslant n\\right\\}$, 可得 $|S|=\\left[\\frac{3(n+1)^2+1}{4}\\right]$, 且 $S$ 中任意两数不具备倍数关系, 否则, 不妨设 $2^{\\alpha_1} 3^{\\beta_1} 5^{\\gamma_1} \\mid 2^{\\alpha_2} 3^{\\beta_2} 5^{\\gamma_2}$, 则 $\\alpha_1 \\leqslant \\alpha_2, \\beta_1 \\leqslant \\beta_2, \\gamma_1 \\leqslant \\gamma_2, \\alpha_1+\\beta_1+\\gamma_1=\\alpha_2+\\beta_2+\\gamma_2$, 因此 $\\alpha_1=\\alpha_2, \\beta_1=\\beta_2, \\gamma_1=\\gamma_2$, 产生矛盾! 综上可知, $|S|_{\\text {max }}=\\left[\\frac{3(n+1)^2+1}{4}\\right]$.", + "remark": "", + "figures": [] +} \ No newline at end of file diff --git a/processed_dataset/calculation/0095.json b/processed_dataset/calculation/0095.json new file mode 100644 index 0000000000000000000000000000000000000000..394272725459b78b62c490b08c5d75471ddbcf51 --- /dev/null +++ b/processed_dataset/calculation/0095.json @@ -0,0 +1,8 @@ +{ + "source_file": "./raw_volume-zh/volume1/exercise6.tex", + "problem_type": "calculation", + "problem": "问题5 求所有的由不同正整数 (至少 2 个)组成的集合,使其中各数之和等于它们的积.", + "solution": "显然, 1 在集合中起着保持积不动而增大和的作用, 而且它是具有这种性质的惟一正整数.\n先设集合中的 $n$ 个数中不含 1 且 $1j, j=1,2, \\cdots, n$. 因 $n \\geqslant 2$, 故有 $a_n a_{n-1}-a_n-a_{n-1}= (a_n - 1) \\left(a_{n-1}-1\\right)-1 \\geqslant 1, a_n a_{n-1} \\geqslant a_n+a_{n-1}+1$. 其中等号成立当且仅当 $a_n=3$, $a_{n-1}=2$. 从而当 $n \\geqslant 3$ 时, $a_n a_{n-1} a_{n-2} \\geqslant 2 a_n a_{n-1}>a_n+a_{n-1}+a_{n-2}+1$. 依此类推, 便得 $a_n a_{n-1} \\cdots a_1>a_n+a_{n-1}+\\cdots+a_1+1$. 这表明, 凡不含 1 的集合都不满足要求,而且当 $n \\geqslant 3$ 时, 即使将 1 添人集合中也不能满足要求.\n当 $n=2$ 时, 只有 $a_0=1, a_1=2, a_2=3$ 才满足题中要求.\n又当 $n=1$ 时, $a_1 \\cdot 16$. 所以, $k \\geqslant 7$.\n当 $k=7$ 时, 在 99 个数中任取 7 个数, 由抽屟原理知, 必有 2 个数属于同一个 $A_i(1 \\leqslant i \\leqslant 6)$, 这 2 个数的比值在闭区间 $\\left[\\frac{1}{2}, 2\\right]$ 中.", + "remark": "", + "figures": [] +} \ No newline at end of file diff --git a/processed_dataset/calculation/0099.json b/processed_dataset/calculation/0099.json new file mode 100644 index 0000000000000000000000000000000000000000..7e052f973d3d36fcd521813265b2b3dcd4c544ca --- /dev/null +++ b/processed_dataset/calculation/0099.json @@ -0,0 +1,8 @@ +{ + "source_file": "./raw_volume-zh/volume1/exercise7.tex", + "problem_type": "calculation", + "problem": "问题3 设变量 $x$ 满足 $x^2+b x \\leqslant-x(b<-1)$, 且 $f(x)=x^2+b x$ 的最小值是 $-\\frac{1}{2}$, 则 $b$ 等于", + "solution": "$b=-\\frac{3}{2}$.", + "remark": "", + "figures": [] +} \ No newline at end of file diff --git a/processed_dataset/calculation/0100.json b/processed_dataset/calculation/0100.json new file mode 100644 index 0000000000000000000000000000000000000000..dea0a6a4fdfb4239f4f3d79ae4577bde0f422d2c --- /dev/null +++ b/processed_dataset/calculation/0100.json @@ -0,0 +1,8 @@ +{ + "source_file": "./raw_volume-zh/volume1/exercise7.tex", + "problem_type": "calculation", + "problem": "问题4 若关于 $x$ 的不等式 $k x^2-2|x-1|+6 k<0$ 的解集为空集, 则 $k$ 的取值范围是", + "solution": "$k \\geqslant \\frac{1+\\sqrt{7}}{6}$. 对 $x \\in \\mathbf{R}$, 恒有 $k \\geqslant f(x)=\\frac{2|x-1|}{x^2+6}$. 所以, $k \\geqslant \\max f(x), x \\in \\mathbf{R}$. 当 $x \\geqslant 1$ 时, 令 $t=x-1, t \\geqslant 0$, 则 $f(x)=\\frac{2 t}{\\left(t^2+7\\right)+2 t} \\leqslant \\frac{2 t}{2 \\sqrt{7} t+2 t}=\\frac{\\sqrt{7}-1}{6}$. 当 $x<1$ 时, 令 $t=1-x, t>0$, 则 $f(x)=\\frac{2 t}{\\left(t^2+7\\right)-2 t} \\leqslant \\frac{2 t}{2 \\sqrt{7} t-2 t}=\\frac{\\sqrt{7}+1}{6}$, 当 $t=\\sqrt{7}$ 时, 等号成立.", + "remark": "", + "figures": [] +} \ No newline at end of file diff --git a/processed_dataset/calculation/0101.json b/processed_dataset/calculation/0101.json new file mode 100644 index 0000000000000000000000000000000000000000..0f9129bf76672ea3a0700f6bcb6bf2d325f90728 --- /dev/null +++ b/processed_dataset/calculation/0101.json @@ -0,0 +1,8 @@ +{ + "source_file": "./raw_volume-zh/volume1/exercise7.tex", + "problem_type": "calculation", + "problem": "问题8 求满足 $w !=x !+y !+z !$ 的所有正整数组 $w, x, y, z$.", + "solution": "不妨设 $w>x \\geqslant y \\geqslant z$. 若 $y>z$, 则以 $z$ ! 除等式两边得 $w(w-1) \\cdots (z+1)=x \\cdots(z+1)+y \\cdots(z+1)+1$, 其中 $z+1>1$ 能整除上式左边, 而不能整除其右边, 矛盾.\n若 $x>y=z$, 则可得 $w(w-1) \\cdots(z+1)=x \\cdots(z+ 1)+2$, 应有 $(z+1) \\mid 2$, 从而 $z+1=2$, 上式又可约简为 $w(w-1) \\cdots 3=x \\cdots 3+1$, 显然也不成立.\n于是, 必有 $x=y=z$, 此时原式为 $w !=3 \\cdot x !$. 所以 $x=y=z=2, w=3$.", + "remark": "", + "figures": [] +} \ No newline at end of file diff --git a/processed_dataset/calculation/0102.json b/processed_dataset/calculation/0102.json new file mode 100644 index 0000000000000000000000000000000000000000..df81d53161fa2417bd7530a145c4312d72b8c0d3 --- /dev/null +++ b/processed_dataset/calculation/0102.json @@ -0,0 +1,8 @@ +{ + "source_file": "./raw_volume-zh/volume1/exercise7.tex", + "problem_type": "calculation", + "problem": "问题10 若 $x 、 y$ 是任意正实数, 求 $\\max \\left\\{\\min \\left\\{x, \\frac{1}{y}, y+\\frac{1}{x}\\right\\}\\right\\}$ 的值.", + "solution": "不妨设 $x \\geqslant \\frac{1}{y}>0$, 令 $x=\\frac{1}{y}=y+\\frac{1}{x}$, 则 $x=\\sqrt{2}, y=\\frac{\\sqrt{2}}{2}$. 所以\n(1) 若 $x \\geqslant \\frac{1}{y} \\geqslant \\sqrt{2}$, 则 $y+\\frac{1}{x} \\leqslant \\frac{\\sqrt{2}}{2}+\\frac{\\sqrt{2}}{2}=\\sqrt{2}$, 所以 $\\min \\left\\{x, \\frac{1}{y}, y+\\frac{1}{x}\\right\\}= y+\\frac{1}{x} \\leqslant \\sqrt{2}$, 从而当且仅当 $x=\\frac{1}{y}=\\sqrt{2}$ 时, $\\max \\left\\{\\min \\left\\{x, \\frac{1}{y}, y+\\frac{1}{x}\\right\\}\\right\\}= \\max \\left\\{y+\\frac{1}{x}\\right\\}=\\sqrt{2}$;\n(2) 若 $\\sqrt{2} \\geqslant x \\geqslant \\frac{1}{y}>0$, 则 $y+\\frac{1}{x} \\geqslant \\frac{\\sqrt{2}}{2}+\\frac{\\sqrt{2}}{2}=\\sqrt{2}$. 所以 $\\min \\left\\{x, \\frac{1}{y}, y+ \\frac{1}{x}\\right\\}=\\frac{1}{y} \\leqslant \\sqrt{2}$, 从而当且仅当 $x=\\frac{1}{y}=\\sqrt{2}$ 时, $\\max \\left\\{\\min \\left\\{x, \\frac{1}{y}, y+\\frac{1}{x}\\right\\}\\right\\}= \\max \\left\\{\\frac{1}{y}\\right\\}=\\sqrt{2}$\n(3) 若 $x \\geqslant \\sqrt{2} \\geqslant \\frac{1}{y}>0$, 则 $y+\\frac{1}{x} \\geqslant 2 \\cdot \\frac{1}{x} \\leqslant 2 \\cdot \\frac{\\sqrt{2}}{2}=\\sqrt{2}, y+\\frac{1}{x} \\leqslant 2 \\cdot y \\geqslant 2 \\cdot \\frac{\\sqrt{2}}{2}=\\sqrt{2}$, 此时若 $\\frac{1}{y} \\geqslant y+\\frac{1}{x}$, 则 $\\min \\left\\{x, \\frac{1}{y}, y+\\frac{1}{x}\\right\\}=y+\\frac{1}{x} \\leqslant \\sqrt{2}$ ; 若 $\\frac{1}{y} \\leqslant y+\\frac{1}{x}$, 则 $\\min \\left\\{x, \\frac{1}{y}, y+\\frac{1}{x}\\right\\}=\\frac{1}{y} \\leqslant \\sqrt{2}$, 所以当且仅当 $x=\\frac{1}{y}=\\sqrt{2}$ 时, $\\max \\left\\{\\min \\left\\{x, \\frac{1}{y}, y+\\frac{1}{x}\\right\\}\\right\\}=\\sqrt{2}$.", + "remark": "", + "figures": [] +} \ No newline at end of file diff --git a/processed_dataset/calculation/0103.json b/processed_dataset/calculation/0103.json new file mode 100644 index 0000000000000000000000000000000000000000..2ea8eaacdb4cfa8504fb43f859b1c13e2cb50042 --- /dev/null +++ b/processed_dataset/calculation/0103.json @@ -0,0 +1,8 @@ +{ + "source_file": "./raw_volume-zh/volume1/exercise7.tex", + "problem_type": "calculation", + "problem": "问题19 如果一个正整数的所有正约数之和为其两倍, 则称该数为一个完全数.\n求所有的正整数 $n$, 使得 $n-1$ 和 $\\frac{n(n+1)}{2}$ 都是完全数.", + "solution": "这里需要用到 Euler 的一个结论: $n$ 为偶完全数 $\\Leftrightarrow$ 存在质数 $p$, 使得 $2^p-1$ 为质数, 且 $n=2^{p-1}\\left(2^p-1\\right)$. 下面以此来解本题.\n情形一: $n$ 为奇数, 则 $n-1$ 为偶完全数, 于是, 可写 $n-1=2^{p-1}\\left(2^p-1\\right)$, 其中 $p$ 与 $2^p-1$ 都为质数, 这时 $\\frac{n(n+1)}{2}=\\frac{1}{2}\\left(2^{p-1}\\left(2^p-1\\right)+1\\right)\\left(2^{p-1}\\left(2^p-1\\right)+ 2)= (2^{p-1}\\left(2^p-1\\right)+1\\right)\\left(2^{p-2}\\left(2^p-1\\right)+1\\right)$. 当 $p=2$ 时, $n=7, \\frac{n(n+1)}{2}= 28$ , 此时 $n-1$ 与 $\\frac{n(n+1)}{2}$ 都是完全数.\n当 $p \\geqslant 3$ 时, 记 $N=\\frac{n(n+1)}{2}$, 则 $N$ 为奇数, 且 $\\frac{n+1}{2}=4^{p-1}-2^{p-2}+1=(3+1)^{p-1}-(3-1)^{p-2}+1$, 由二项式定理可知 $\\frac{n+1}{2} \\equiv 3 \\times(p-1)-(p-2) \\times 3+1+1+1 \\equiv 6(\\bmod 9)$. 从而 $3 \\mid N$,但 $3^2 \\nmid N$, 可设 $N=3 k, 3 \\nmid k$, 此时, $\\sigma(N)=\\sigma(3), \\sigma(k)=4 \\sigma(k)$, 但是 $2 N \\equiv 2(\\bmod 4)$, 故 $\\sigma(N) \\neq 2 N$, 从而此时 $\\frac{n(n+1)}{2}$ 不是完全数.\n情形二: $n$ 为偶数, 如果 $4 \\mid n$, 则 $n-1 \\equiv-1(\\bmod 4) \\Rightarrow n-1$ 不是完全平方数, 此时对任意 $d \\mid n-1$, 由 $d \\times \\frac{n-1}{d}=n-1 \\equiv-1(\\bmod 4)$, 可知 $d$ 与 $\\frac{n-1}{d}$ 中一个 $\\bmod 4$ 余 -1 , 另一个 $\\bmod 4$ 余 1 , 导致 $d+\\frac{n-1}{d} \\equiv 0(\\bmod 4)$, 从而 $4 \\mid \\sigma(n-1)$, 但 $2(n-1) \\equiv 2(\\bmod 4)$, 故 $n-1$ 不是完全数.\n所以, $4 \\nmid n$,于是, 可设 $n=4 k+2$, 此时 $N=\\frac{n(n+1)}{2}=(2 k+1)(4 k+3)$ 为奇数.\n由于 $(2 k+ 1,4 k+3)=1$, 故 $\\sigma(N)=\\sigma(2 k+1) \\sigma(4 k+3)$. 同上可知 $4 \\mid \\sigma(4 k+3)$, 故若 $\\sigma(N)=2 N$, 则 $4|2 N \\Rightarrow 2| N$, 这是一个矛盾.\n综上可知, 满足条件的 $n$ 只有一个, 即 $n=7$.", + "remark": "", + "figures": [] +} \ No newline at end of file diff --git a/processed_dataset/calculation/0104.json b/processed_dataset/calculation/0104.json new file mode 100644 index 0000000000000000000000000000000000000000..ee9a4037b4ba494976c593bb9e399828bc106f69 --- /dev/null +++ b/processed_dataset/calculation/0104.json @@ -0,0 +1,8 @@ +{ + "source_file": "./raw_volume-zh/volume1/exercise8.tex", + "problem_type": "calculation", + "problem": "问题10 求所有的非空有限的正整数集 $S$, 使得对任意 $i, j \\in S$, 数 $\\frac{i+j}{(i, j)} \\in S$, 这里 $(i, j)$ 表示 $i$ 与 $j$ 的最大公约数.", + "solution": "设 $S$ 为满足条件的集合, 并设 $a \\in S$, 则 $\\frac{a+a}{(a, a)}=2 \\in S$, 如果 $1 \\in S$, 则 $\\frac{1+2}{(1,2)}=3 \\in S$. 一般地, 设 $n \\in S$, 则 $\\frac{n+1}{(n, 1)}=n+1 \\in S$. 这表明 $S=\\mathbf{N}^*$, 与 $S$ 为有限集矛盾.\n另一方面, 若 $S$ 中有大于 2 的元素, 取这些元素中的最小元素, 设为 $n$, 则 $\\frac{n+2}{(2, n)} \\in S$. 若 $(2, n)=2$, 则 $2<\\frac{n+2}{2})). 易知, 坚向相邻对与横向相邻对各有 $n(n-1)$ 个; 斜向相邻对有 $2(n-1)^2$ 个.\n故共有相邻的小方格 $2 n(n-1)+2(n-1)^2=2(n-1)(2 n-1)$ 个.", + "remark": "", + "figures": [ + "./images/volume1/figures/fig-c9p1.png" + ] +} \ No newline at end of file diff --git a/processed_dataset/calculation/0107.json b/processed_dataset/calculation/0107.json new file mode 100644 index 0000000000000000000000000000000000000000..c878a4a94567f7341764e3190187fa615c198aab --- /dev/null +++ b/processed_dataset/calculation/0107.json @@ -0,0 +1,8 @@ +{ + "source_file": "./raw_volume-zh/volume1/exercise9.tex", + "problem_type": "calculation", + "problem": "问题2 在所有四位数的号码 (从 0000 到 9999) 中, 有多少个号码的前两位数字之和同末两位数字之和相等?", + "solution": "它们的前两位数字之和与末两位数字之和等于固定的 $k=0,1, \\cdots$, 18 , 共有这样的四位数 $1^2+2^2+\\cdots+19^2=2470$ 个.", + "remark": "", + "figures": [] +} \ No newline at end of file diff --git a/processed_dataset/calculation/0108.json b/processed_dataset/calculation/0108.json new file mode 100644 index 0000000000000000000000000000000000000000..e1facfa2b10e80691a270940e65eb051182bbc0b --- /dev/null +++ b/processed_dataset/calculation/0108.json @@ -0,0 +1,8 @@ +{ + "source_file": "./raw_volume-zh/volume1/exercise9.tex", + "problem_type": "calculation", + "problem": "问题3 用 $2 、 4 、 6$ 三个数字来构造六位数, 但是不允许有两个连着的 2 出现在六位数中 (例如 626442 是允许的, 226 426 就不允许), 问这样的六位数共有多少个?", + "solution": "六位数中不可能出现 4 个或 4 个以上的 2 . 符合要求的六位数中, 不含 2 的有 $2^6$ 个,恰含 1 个 2 的有 $6 \\cdot 2^5$ 个, 恰含 2 个 2 的有 $2^4 \\cdot \\mathrm{C}_5^2$ 个, 恰含 3 个 2的有 $2^3 \\cdot \\mathrm{C}_4^3$ 个.\n共有 448 个.", + "remark": "", + "figures": [] +} \ No newline at end of file diff --git a/processed_dataset/calculation/0109.json b/processed_dataset/calculation/0109.json new file mode 100644 index 0000000000000000000000000000000000000000..bdb7041fb2c030641e3783dd5097c423b07bcc4b --- /dev/null +++ b/processed_dataset/calculation/0109.json @@ -0,0 +1,8 @@ +{ + "source_file": "./raw_volume-zh/volume1/exercise9.tex", + "problem_type": "calculation", + "problem": "问题4 某个国王的 25 位骑士围坐在他们的圆桌旁, 他们中间的 3 位被选派去杀一条恶龙.\n问被挑到的 3 位骑士中至少有两位是邻座的选派方法有多少种?", + "solution": "一种情况是, 3 位骑士依次相邻, 有 25 种选法; 另一种情况是, 两位骑士是邻座, 此时第三位骑士就不选在已经邻座的两位骑士的两旁, 也就是说第三位只能在 25-4 位中任选一位, 这样有 25(25-4) 种选法.\n因此, 共有选法 $25+25(25-4)=550$ 种.", + "remark": "", + "figures": [] +} \ No newline at end of file diff --git a/processed_dataset/calculation/0110.json b/processed_dataset/calculation/0110.json new file mode 100644 index 0000000000000000000000000000000000000000..c2438e70dd01fe125b3cf3d37f7dd56c64994ed8 --- /dev/null +++ b/processed_dataset/calculation/0110.json @@ -0,0 +1,8 @@ +{ + "source_file": "./raw_volume-zh/volume1/exercise9.tex", + "problem_type": "calculation", + "problem": "问题5 三边长为互不相等的自然数的三角形中, 最大边长恰为 $n$ 的共有 600 个.\n求 $n$ 的值.", + "solution": "设三角形三边的长是 $x 、 y 、 n$, 且 $x10^6$, 则由容斥原理得 $N\\left(x^y\\right)=N\\left(x^2\\right)+N\\left(x^3\\right)+ N\\left(x^5\\right)+N\\left(x^7\\right)+N\\left(x^{11}\\right)+N\\left(x^{13}\\right)+N\\left(x^{17}\\right)+N\\left(x^{19}\\right)-N\\left(x^6\\right)-N\\left(x^{10}\\right)- N\\left(x^{14}\\right)-N\\left(x^{15}\\right)$. 由于大于 1 且不大于 $10^6$ 的平方数有 $10^3-1$ 个, 所以 $N\\left(x^2\\right)=999$. 大于 1 且不大于 $10^6$ 的立方数有 $10^2-1$ 个, 即 $N\\left(x^3\\right)=99$ 个.\n因为 $15^5=819375<10^6$, 所以大于 1 不大于 $10^6$ 的 5 次方数有 $15-1$ 个, 即 $N\\left(x^5\\right)=14$. 以此类推可得, $11$ 且 $y=0$. 于是满足题意的整数 $n$ 有 1111 个.", + "remark": "", + "figures": [] +} \ No newline at end of file diff --git a/processed_dataset/calculation/0117.json b/processed_dataset/calculation/0117.json new file mode 100644 index 0000000000000000000000000000000000000000..a1787878e9c1e9662dd228305661d10cd0379944 --- /dev/null +++ b/processed_dataset/calculation/0117.json @@ -0,0 +1,8 @@ +{ + "source_file": "./raw_volume-zh/volume1/exercise9.tex", + "problem_type": "calculation", + "problem": "问题13 对于 $0 \\leqslant x \\leqslant 100$, 求函数 $f(x)=[x]+[2 x]+\\left[\\frac{5 x}{3}\\right]+[3 x]+[4 x]$ 所取的不同整数值的个数.", + "solution": "以 $A_1, A_2, A_3, A_4, A_5$ 来分别表示函数 $[x],[2 x],[3 x],[4 x]$ 和 $\\left[\\frac{5 x}{3}\\right]$ 的所有间断点的集合.\n则易知 $A_1 \\subset A_2 \\subset A_4$, 且 $A_3=\\left\\{\\frac{n}{3} \\mid n=1\\right.$, $2, \\cdots, 300\\}, A_4=\\left\\{\\frac{n}{4} \\mid n=1,2, \\cdots, 400\\right\\}, A_5=\\left\\{\\frac{3 n}{5} \\mid n=1,2, \\cdots, 166\\right\\}$. 由此可得 $A_3 \\cap A_4=\\{n \\mid n=1,2, \\cdots, 100\\}, A_3 \\cap A_5=A_4 \\cap A_5=A_3 \\cap A_4 \\cap A_5==\\{3 n \\mid n=1,2, \\cdots, 33\\}$. 由容斥原理知 $f(x)$ 的间断点的个数为 $\\left|A_3\\right|+\\left|A_4\\right|+\\left|A_5\\right|-\\left|A_3 \\cap A_4\\right|-\\left|A_3 \\cap A_5\\right|-\\left|A_4 \\cap A_5\\right|+\\mid A_3 \\cap A_4 \\cap A_5 \\mid=300+400+166-100-33-33+33=733$. 故知 $f(x)$ 所取的不同整数值的个数为 734 .", + "remark": "", + "figures": [] +} \ No newline at end of file diff --git a/processed_dataset/calculation/0118.json b/processed_dataset/calculation/0118.json new file mode 100644 index 0000000000000000000000000000000000000000..5c44dab38da43ac2dec266d349f9f68c12ee088e --- /dev/null +++ b/processed_dataset/calculation/0118.json @@ -0,0 +1,8 @@ +{ + "source_file": "./raw_volume-zh/volume1/exercise9.tex", + "problem_type": "calculation", + "problem": "问题15 已知 $N=1990^{1990}$. 求满足条件 $1 \\leqslant n \\leqslant N$, 且 $\\left(n^2-1, N\\right)=1$ 的整数 $n$ 的个数.", + "solution": "因为 $1990=2 \\times 5 \\times 199$, 所以 $\\left(n^2-1, N\\right)=1 \\Leftrightarrow n^2-1 \\not \\neq 0(\\bmod 2,5,199)$, 即 $n \\neq 11(\\bmod 2)$ 且 $n \\neq 1,4(\\bmod 5)$ 且 $n \\neq 1,198(\\bmod 199)$.\n令全集 $I=\\{1,2, \\cdots, N\\}, A=\\{n \\mid n \\equiv 1(\\bmod 2), n \\in I\\}, B= \\{n \\mid n \\equiv 1,4(\\bmod 5), n \\in I\\}, C=\\{n \\mid n \\equiv 1,198(\\bmod 199), n \\in I\\}$. 则 $A \\cap B=\\{n \\mid n \\equiv 1,9(\\bmod 10), n \\in I\\}, B \\cap C=\\{n \\mid n \\equiv 1,994(\\bmod 995)$, $n \\in I\\}, C \\cap A=\\{n \\mid n \\equiv 1,397(\\bmod 398), n \\in I\\}, A \\cap B \\cap C=\\{n \\mid n \\equiv 1$, $1989(\\bmod 1990), n \\in I\\} .|\\bar{A} \\cap \\bar{B} \\cap \\vec{C}|==N-\\left(\\frac{N}{2}+\\frac{2 N}{5}+\\frac{2 N}{199}\\right)+ \\left(\\frac{2 N}{10}+\\frac{2 N}{995}+\\frac{2 N}{398}\\right)-\\frac{2 N}{1990}=\\frac{589}{1990} N$. 故满足条件的整数 $n$ 的个数为 $\\frac{589}{1990} N$.", + "remark": "", + "figures": [] +} \ No newline at end of file diff --git a/processed_dataset/calculation/0119.json b/processed_dataset/calculation/0119.json new file mode 100644 index 0000000000000000000000000000000000000000..e677e943d6520295ad6f310e58248f6b1ceb3321 --- /dev/null +++ b/processed_dataset/calculation/0119.json @@ -0,0 +1,8 @@ +{ + "source_file": "./raw_volume-zh/volume1/exercise9.tex", + "problem_type": "calculation", + "problem": "问题17 上届获得前 $n$ 名的 $n$ 个球队参加本届争夺前 $n$ 名的比赛.\n如果不设并列名次, 问: 没有一个队取得的名次恰好紧接在上届比他高一个名次的球队之后的比赛结果有多少种可能?", + "solution": "以 $n$ 元有序数组 $\\left(a_1, a_2, \\cdots, a_n\\right)$ 表示本届的比赛结果: 上届第 $i$ 名的球队在本届获得第 $a_i$ 名 $(i=1,2, \\cdots, n)$. 以 $S$ 表示比赛可能结果的全体, 则 $|S|=n !$. 设 $A_i=\\{i+1$ 号队获得的名次比 $i$ 号队低一名次的比赛结果 $\\}, i=1,2, \\cdots, n$. 易知 $\\left|A_i\\right|=(n-1) !(1 \\leqslant i \\leqslant n),\\left|A_i \\cap A_j\\right|= (n-2) ! \\quad(1 \\leqslant i0$, 故方程左边 $\\geqslant(2 y-1)^2$.\n因此由原方程产生\n$$\n(2 y-1)^2 \\leqslant 1+16 y,\n$$\n故有 $0 \\leqslant y \\leqslant 5$. 逐一检验可求出全部整数解为 $\\left(x_1 y\\right)=( \\pm 1,0),( \\pm 4,3)$, $( \\pm 4,5)$.", + "remark": "", + "figures": [] +} \ No newline at end of file diff --git a/processed_dataset/calculation/0124.json b/processed_dataset/calculation/0124.json new file mode 100644 index 0000000000000000000000000000000000000000..4a92b5574d6104f154260f0f7ddc41db68cefe2c --- /dev/null +++ b/processed_dataset/calculation/0124.json @@ -0,0 +1,8 @@ +{ + "source_file": "./raw_volume-zh/volume10/chapter5.tex", + "problem_type": "calculation", + "problem": "例4. 求出不定方程\n$$\n(n-1) !=n^k-1 . \\label{eq1}\n$$\n的全部正整数解.", + "solution": "解:当 $n=2$ 时, 由式\\ref{eq1}得解 $(n, k)=(2,1)$. 当 $n>2$ 时, 式\\ref{eq1} 的左边是偶数, 故其右边也是偶数, 从而 $n$ 是奇数.\n当 $n=3,5$ 时, 由式\\ref{eq1}解出 $(n, k)=(3,1), (5,2)$.\n以下设 $n>5$ 且 $n$ 为奇数.\n此时 $\\frac{n-1}{2}$ 是整数且 $\\frac{n-1}{2}(n-1) ! .\n$$\n这表明, 当 $n>5$ 时方程式\\ref{eq1}没有正整数解, 即式\\ref{eq1}的全部正整数解为 $(n, k)= (2,1),(3,1),(5,2)$.", + "remark": "注1 上面解法的关键是在 $n>5$ 时,利用整除给出 $k$ 的下界: $k \\geqslant n-1$, 进而 (利用不等式)证明式\\ref{eq1}无解.\n论证的第一步, 是对奇数 $n>5$ 证明 $(n-1)^2 \\mid (n-1)$ !. 这个事实是下面结果的一个特别情形设 $m$ 是大于 4 的整数, 且不是素数, 则 $m \\mid(m-1) !$. \n注2 论证的第二步, 是用 $(n-1)^2$ 除 $n^k-1$, 这其实不必应用二项式定理, 只需注意: $(x+1)^k-1$ 的展开式, 是一个关于 $x$ 的整系数多项式, 其中常数项为零, 而一次项系数为 $k$.\n若应用下一单元讲的同余, 则可更为直接地证明 $(n-1) \\mid k$ :\n因为 $n^k-1=(n-1)\\left(n^{k-1}+n^{k-2}+\\cdots+n+1\\right)$, 而 $n^i \\equiv 1(\\bmod n-1)$, $i=1, \\cdots, k-1$, 故\n$$\nn^{k-1}+n^{k-2}+\\cdots+n+1 \\equiv \\underbrace{1+1+\\cdots+1}_{k \\text { 个 }}=k(\\bmod n-1) .\n$$\n从而 $n^k-1 \\equiv k(n-1)\\left(\\bmod (n-1)^2\\right)$, 于是由 $(n-1)^2 \\mid n^k-1$, 得出 $(n-1)^2 \\mid k(n-1)$, 即 $(n-1) \\mid k$.", + "figures": [] +} \ No newline at end of file diff --git a/processed_dataset/calculation/0125.json b/processed_dataset/calculation/0125.json new file mode 100644 index 0000000000000000000000000000000000000000..8172f9396e23f93cc399c5a98d808c8e592b3ae8 --- /dev/null +++ b/processed_dataset/calculation/0125.json @@ -0,0 +1,8 @@ +{ + "source_file": "./raw_volume-zh/volume10/chapter5.tex", + "problem_type": "calculation", + "problem": "例5. 求出不定方程:\n$$\nx^3+x^2 y+x y^2+y^3=8\\left(x^2+x y+y^2+1\\right)\n$$\n的全部整数解.", + "solution": "解:法一原方程左端是关于 $x 、 y$ 的三次多项式,右边是二次多项式.\n而对于整数 $x 、 y$, 三次式的值的绝对值一般应大于二次式的值的绝对值, 因此本题有希望用估计法解决.\n现将方程分解为\n$$\n\\left(x^2+y^2\\right)(x+y-8)=8(x y+1) . \\label{eq1}\n$$\n若 $x+y-8 \\geqslant 6$, 则 $x+y \\geqslant 14$, 从而\n$$\nx^2+y^2 \\geqslant \\frac{(x+y)^2}{2}>4 \\text {. }\n$$\n这时式\\ref{eq1}的左端\n$$\n\\begin{aligned}\n& \\geqslant 6\\left(x^2+y^2\\right)=4\\left(x^2+y^2\\right)+2\\left(x^2+y^2\\right) \\\\\n& \\geqslant 8 x y+2\\left(x^2+y^2\\right)>8(x y+1),\n\\end{aligned}\n$$\n故此时方程无整数解.\n若 $x+y-8 \\leqslant-4$, 则 $x+y \\leqslant 4$, 这时式\\ref{eq1}的左端\n$$\n\\leqslant-4\\left(x^2+y^2\\right) \\leqslant-4 \\times 2|x y| \\leqslant 8 x y<8(x y+1),\n$$\n此时方程亦无整数解.\n因此, 方程的整数解 $(x, y)$ 应满足\n$$\n-3 \\leqslant x+y-8 \\leqslant 5 .\n$$\n另一方面, 式\\ref{eq1}的左端应是偶数,这推出 $x, y$ 的奇偶性必须相同, 从而 $x+ y-8$ 是偶数, 故它只能是 $-2 、 0 、 2 、 4$. 结合 式\\ref{eq1}, 通过检验不难得知, 所求的解为 $(x, y)=(2,8),(8,2)$.", + "remark": "", + "figures": [] +} \ No newline at end of file diff --git a/processed_dataset/calculation/0126.json b/processed_dataset/calculation/0126.json new file mode 100644 index 0000000000000000000000000000000000000000..f17fa7e964accf5dd6399fcfe95a545cc319e4ee --- /dev/null +++ b/processed_dataset/calculation/0126.json @@ -0,0 +1,8 @@ +{ + "source_file": "./raw_volume-zh/volume10/chapter5.tex", + "problem_type": "calculation", + "problem": "例5. 求出不定方程:\n$$\nx^3+x^2 y+x y^2+y^3=8\\left(x^2+x y+y^2+1\\right)\n$$\n的全部整数解.", + "solution": "解法二记 $u=x+y, v=x y$, 则原方程可变形为\n$$\nu\\left(u^2-2 v\\right)=8\\left(u^2-v+1\\right), \\label{eq2}\n$$\n即\n$$\nu^3-2 u v=8 u^2-8 v+8,\n$$\n由此可见 $u$ 是偶数,设 $u=2 w$, 则\n$$\n2 w^3-w w=8 w^2-2 v+2 . \\label{eq3}\n$$\n我们解出 $v$, 得到\n$$\nv=\\frac{2 w^3-8 w^2-2}{w-2}=2 w^2-4 w-8-\\frac{18}{w-2} . \\label{eq4}\n$$\n因此 $w-2$ 是 18 的约数, 即是 $\\pm 1, \\pm 2, \\pm 3, \\pm 6, \\pm 9, \\pm 18$. 对于 $w$ 的每一个可能值, 结合式\\ref{eq4}可确定 $v$, 进而求得相应的整数解 $(x, y)$ 只有 $(2,8)$ 及 $(8$, 2 ). (注意, 求得一组 $w, v$ 的值, 则相应的 $x, y$ 为整数等价于 $w^2-v$ 为完全平方数.)", + "remark": "注:原方程的左、右两边均是关于 $x 、 y$ 的二元对称多项式,因此必能表示为关于 $u=x+y, v=x y$ 的多项式 (见 式\\ref{eq2}). 对本题而言, 这一表示的优点在于, 导出的方程式\\ref{eq3}关于 $v$ 是一次方程, 从而可解出 $v$ (用 $w$ 表示).", + "figures": [] +} \ No newline at end of file diff --git a/processed_dataset/calculation/0127.json b/processed_dataset/calculation/0127.json new file mode 100644 index 0000000000000000000000000000000000000000..02aaec7bd404a3edd475d4b5e530be50624e3eb0 --- /dev/null +++ b/processed_dataset/calculation/0127.json @@ -0,0 +1,8 @@ +{ + "source_file": "./raw_volume-zh/volume10/exercise1.tex", + "problem_type": "calculation", + "problem": "问题1. 设 $n$ 和 $k$ 都是正整数,则 $1,2, \\cdots, n$ 中恰有 $\\left[\\frac{n}{k}\\right]$ 个数被 $k$ 整除.", + "solution": "在 $1,2, \\cdots, n$ 中, 被 $k$ 整除的数为 $k, 2 k, \\cdots, d k$, 其中正整数 $d$ 满足 $d k \\leqslant n$ 但 $(d+1) k>n$, 从而 $\\frac{n}{k}-111$, 故 $n$ 只能是 9. 因此, 女孩比男孩多.", + "remark": "", + "figures": [] +} \ No newline at end of file diff --git a/processed_dataset/calculation/0129.json b/processed_dataset/calculation/0129.json new file mode 100644 index 0000000000000000000000000000000000000000..e4ca5ecb6921d6b023857f6fde3bffce46801425 --- /dev/null +++ b/processed_dataset/calculation/0129.json @@ -0,0 +1,8 @@ +{ + "source_file": "./raw_volume-zh/volume10/exercise4.tex", + "problem_type": "calculation", + "problem": "问题2. 求出所有可以表示为两个整数平方差的整数.", + "solution": "设整数 $n$ 可表示为两个整数的平方差: $n=x^2-y^2$, 即 $n=(x-y) (x+y)$. 由于 $x-y$ 与 $x+y$ 的奇偶相同,故或者 $n$ 是奇数,或者 $n$ 被 4 整除.\n反过来, 若 $n$ 为奇数, 可取 $x-y=1, x+y=n$, 即 $x=\\frac{n+1}{2}, y=\\frac{n-1}{2}$; 若\n$4 \\mid n$, 可取 $x-y=2, x+y=\\frac{n}{2}$, 即 $x=\\frac{n}{4}+1, y=\\frac{n}{4}-1$, 则 $x^2-y^2=n$.", + "remark": "", + "figures": [] +} \ No newline at end of file diff --git a/processed_dataset/calculation/0130.json b/processed_dataset/calculation/0130.json new file mode 100644 index 0000000000000000000000000000000000000000..66e04e01c5b393e2c6a5c5db455dfa61222417f8 --- /dev/null +++ b/processed_dataset/calculation/0130.json @@ -0,0 +1,8 @@ +{ + "source_file": "./raw_volume-zh/volume10/exercise4.tex", + "problem_type": "calculation", + "problem": "问题3. 求不定方程组\n$$\n\\left\\{\\begin{array}{l}\nx+y+z=3 \\\\\nx^3+y^3+z^3=3\n\\end{array}\\right.\n$$\n的全部整数解.", + "solution": "从方程组中消去 $z$, 得到\n$$\n8-9 x-9 z+3 x^2+6 x y+3 y^2-x^2 y-x y^2=0,\n$$\n变形为\n$$\n8-3 x(3-x)-3 y(3-x)+x y(3-x)+y^2(3-x)=0,\n$$\n即 $(3-x)\\left(3 x+3 y-x y-y^2\\right)=8$. 故 $(3-x) \\mid 8$, 从而 $3-x= \\pm 1, \\pm 2$, $\\pm 4, \\pm 8$, 即 $x=-5,-1,1,2,4,5,7,11$. 逐一代入原方程组检验, 可求出全部整数解为 $(x, y, z)=(1,1,1),(-5,4,4),(4,-5,4),(4,4,-5)$.", + "remark": "", + "figures": [] +} \ No newline at end of file diff --git a/processed_dataset/calculation/0131.json b/processed_dataset/calculation/0131.json new file mode 100644 index 0000000000000000000000000000000000000000..e221ff72f57da5d7079903569c176dc8d1dcdf6a --- /dev/null +++ b/processed_dataset/calculation/0131.json @@ -0,0 +1,8 @@ +{ + "source_file": "./raw_volume-zh/volume10/exercise4.tex", + "problem_type": "calculation", + "problem": "问题4. 求 $x^3=y^3+2 y^2+1$ 的全部整数解.", + "solution": "首先注意, 若 $y^2+3 y>0$, 则由原方程推出 $(y+1)^3>x^3>y^3$, 即 $x^3$ 介于两个相邻的完全立方之间, 这不可能.\n故必有 $y^2+3 y \\leqslant 0$, 得整数 $y= -3,-2,-1,0$. 代入原方程检验, 可求得全部整数解为 $(x, y)=(1,0)$, $(1,-2),(-2,-3)$.", + "remark": "", + "figures": [] +} \ No newline at end of file diff --git a/processed_dataset/calculation/0132.json b/processed_dataset/calculation/0132.json new file mode 100644 index 0000000000000000000000000000000000000000..20b17a5a2650c0d515befbf42e49016a160c6d37 --- /dev/null +++ b/processed_dataset/calculation/0132.json @@ -0,0 +1,8 @@ +{ + "source_file": "./raw_volume-zh/volume10/exercise4.tex", + "problem_type": "calculation", + "problem": "问题5. 求所有正整数 $x 、 y$, 使 $x^2+3 y, y^2+3 x$ 均是完全平方数.", + "solution": "设 $\\left\\{\\begin{array}{l}x^2+3 y=u^2, \\\\ y^2+3 x=v^2,\\end{array}\\right.$ 由于 $x, y$ 为正整数,故 $u>x, v>y$. 我们设 $u= x+a, v=y+b$, 这里 $a 、 b$ 为正整数.\n由\n$$\n\\left\\{\\begin{array}{l}\nx^2+3 y=(x+a)^2, \\\\\ny^2+3 x=(y+b)^2\n\\end{array}\\right.\n$$\n可化为\n$$\n\\left\\{\\begin{array}{l}\n3 y=2 a x+a^2, \\\\\n3 x=2 b y+b^2 .\n\\end{array}\\right.\n$$\n故 $9-4ab>0$, 因 $a 、 b$ 为正整数, 故 $a b=1$ 或 2 , 即 $(a, b)=(1,1),(1,2)$, $(2,1)$. 相应地求得 $(x, y)=(1,1),(16,11),(11,16)$.", + "remark": "", + "figures": [] +} \ No newline at end of file diff --git a/processed_dataset/calculation/0133.json b/processed_dataset/calculation/0133.json new file mode 100644 index 0000000000000000000000000000000000000000..595d2241446d275f98c6a6eda2a2f11f2ccf8eda --- /dev/null +++ b/processed_dataset/calculation/0133.json @@ -0,0 +1,8 @@ +{ + "source_file": "./raw_volume-zh/volume10/exercise8.tex", + "problem_type": "calculation", + "problem": "问题2. (1) 设 $m, n$ 是互素的正整数, $m, n>1 . a$ 是一个与 $m n$ 互素的整数.\n设 $a$ 模 $m$ 及模 $n$ 的阶分别为 $d_1 、 d_2$, 则 $a$ 模 $m n$ 的阶为 $\\left[d_1, d_2\\right]$;\n(2) 求出 3 模 $10^4$ 的阶.", + "solution": "(1) 设 $a$ 模 $m n$ 的阶为 $r$. 由 $a^r \\equiv 1(\\bmod m n)$ 可得 $a^r \\equiv 1(\\bmod m)$ 及 $a^r \\equiv 1(\\bmod n)$. 故 $d_1 \\mid r$ 及 $d_2 \\mid r$, 从而 $\\left[d_1, d_2\\right] \\mid r$. 另一方面, 由 $a^{d_1} \\equiv 1(\\bmod m)$ 及 $a^{d_2} \\equiv 1(\\bmod n)$, 推出 $a^{\\left[d_1, d_2\\right]} \\equiv 1(\\bmod m)$, 及 $a^{\\left[d_1, d_2\\right]} \\equiv 1(\\bmod n)$. 因 $(m, n)=1$, 故 $a^{\\left[d_1, d_2\\right]} \\equiv 1(\\bmod m m)$, 于是 $r \\mid\\left[d_1, d_2\\right]$. 综合两方面的结果即知 $r=\\left[d_1 ; d_2\\right]$.\n(2)直接验算可知 3 模 $2^4$ 的阶为 4 . 又易知 3 模 5 的阶为 4 ,故由例 5 中 (1) 可知, 3 模 $5^4$ 的阶为 $4 \\times 5^3$. 因此由本题的 (1) 推出, 3 模 $10^4$ 的阶为 $[4,4 \\times 5^3]=500$.", + "remark": "", + "figures": [] +} \ No newline at end of file diff --git a/processed_dataset/calculation/0134.json b/processed_dataset/calculation/0134.json new file mode 100644 index 0000000000000000000000000000000000000000..807d055fe9d33c518b4f2601acc6f57c1addb601 --- /dev/null +++ b/processed_dataset/calculation/0134.json @@ -0,0 +1,8 @@ +{ + "source_file": "./raw_volume-zh/volume10/exercise9.tex", + "problem_type": "calculation", + "problem": "问题3. 求所有素数 $p$, 使得 $2^p+3^p$ 为一个整数的 $k$ 次幂 (这里的 $k \\geqslant 2$ ).", + "solution": "$p=2,5$ 均不合要求.\n设素数 $p>2$ 且 $p \\neq 5$. 由二项式定理易知\n$$\n\\begin{aligned}\n2^p+3^p & =2^p+(5-2)^p=5^p-\\mathrm{C}_p^1 5^{p-1} \\times 2+\\cdots+5 \\mathrm{C}_p^{p-1} 2^{p-1} \\\\\n& =5^2 u+5 p \\times 2^{p-1}, u \\text { 为一个整数.\n}\n\\end{aligned}\n$$\n故 $5 \\|\\left(2^p+3^p\\right)$, 从而 $2^p+3^p$ 不能是整数的 $k$ 次幂 $(k>1)$.", + "remark": "", + "figures": [] +} \ No newline at end of file diff --git a/processed_dataset/calculation/0135.json b/processed_dataset/calculation/0135.json new file mode 100644 index 0000000000000000000000000000000000000000..b853f3b812112419a1b0cf76bce5dbaec3b017f1 --- /dev/null +++ b/processed_dataset/calculation/0135.json @@ -0,0 +1,8 @@ +{ + "source_file": "./raw_volume-zh/volume10/exercise9.tex", + "problem_type": "calculation", + "problem": "问题6. 设 $p$ 是给定的奇素数, 求 $p^x-y^p=1$ 的全部正整数解 $x 、 y$.", + "solution": "由 $p^x=y^p+1=(y+1)\\left(y^{p-1}-y^{p-2}+\\cdots-y+1\\right)$ 可知, $y+1=p^n$, $n$ 为一 个整数.\n因 $y>0$, 故 $n>0$. 因此\n$$\n\\begin{aligned}\np^x & =\\left(p^n-1\\right)^p+1 \\\\\n& =p^{n p}-p \\cdot p^{n(p-1)}+\\mathrm{C}_p^2 p^{n(p-2)}-\\cdots-\\mathrm{C}_p^{p-2} p^{2 n}+p \\cdot p^n . \\label{eq1}\n\\end{aligned}\n$$\n易知, 上式右边除最后一项外,均被 $p^{n+2}$ 整除 (注意, 因 $p$ 是素数,故所有 $\\mathrm{C}_p^i$ 对 $i=1, \\cdots, p-2$ 均被 $p$ 整除), 因此 $p^{n+1}$ 是 式\\ref{eq1} 的右边的 $p$ 的最高次幂, 故必须 $x=n+1$, 此时式\\ref{eq1}化为\n$$\np^{n p}-p \\cdot p^{n(p-1)}+\\mathrm{C}_p^2 p^{n(p-2)}-\\cdots-\\mathrm{C}_p^{p-2} p^{2 n}=0 . \\label{eq2}\n$$\n当 $p=3$ 时, 式\\ref{eq2} 即为 $3^{3 n}-3 \\cdot 3^{2 n}=0$, 得 $n=1$, 故 $x=y=2$. 若 $p \\geqslant 5$, 注意到 $\\mathrm{C}_p^{p-2}$ 不被 $p^2$ 整除,易知 式\\ref{eq2} 的左边除最后一项外,均被 $p^{2 n+2}$ 整除, 但最后一项不能被 $p^{2 n+2}$ 整除, 这表明 式\\ref{eq2} 不能成立.\n因此, 本题仅在 $p=3$ 时有解 $x=y=2$.", + "remark": "", + "figures": [] +} \ No newline at end of file diff --git a/processed_dataset/calculation/0136.json b/processed_dataset/calculation/0136.json new file mode 100644 index 0000000000000000000000000000000000000000..992aad6348d38f722f8ecf46ba1e4ebe8b04439b --- /dev/null +++ b/processed_dataset/calculation/0136.json @@ -0,0 +1,8 @@ +{ + "source_file": "./raw_volume-zh/volume11/chapter1.tex", + "problem_type": "calculation", + "problem": "例1. 设 $S=\\{1,2,3, \\cdots, 499,500\\}$, 从 $S$ 中任取 4 个不同的数,按照从小到大的顺序排列成一个公比为正整数的等比数列, 求这样的等比数列的个数.", + "solution": "解:设所求等比数列为 $a_1, a_1 q, a_1 q^2, a_1 q^3\\left(a_1, q \\in \\mathbf{N}_{+}, q \\geqslant 2\\right)$, 则 $a_1 q^3 \\leqslant 500, q \\leqslant \\sqrt[3]{\\frac{500}{a_1}} \\leqslant \\sqrt[3]{500}$, 所以 $2 \\leqslant q \\leqslant 7$, 且 $1 \\leqslant a_1 \\leqslant\\left[\\frac{500}{q^3}\\right]$, 即公比为 $q$ 的等比数列有 $\\left[\\frac{500}{q^3}\\right]$ 个.\n由加法原理得满足条件的等比数列共有 $\\sum_{q=2}^7\\left[\\frac{500}{q^3}\\right]=62+18+7+4+ 2+1=94$ (个).", + "remark": "", + "figures": [] +} \ No newline at end of file diff --git a/processed_dataset/calculation/0137.json b/processed_dataset/calculation/0137.json new file mode 100644 index 0000000000000000000000000000000000000000..9baec8d5b48fd1482a8a14c26cb631fe622fb111 --- /dev/null +++ b/processed_dataset/calculation/0137.json @@ -0,0 +1,8 @@ +{ + "source_file": "./raw_volume-zh/volume11/chapter1.tex", + "problem_type": "calculation", + "problem": "例2. 已知集合 $A=\\{x \\mid 5 x-a \\leqslant 0\\}, B=\\{x \\mid 6 x-b>0\\}, a, b \\in \\mathbf{N}$, 且 $A \\cap B \\cap \\mathbf{N}=\\{2,3,4\\}$, 则整数对 $(a, b)$ 的个数为\n(A) 20\n(B) 25\n(C) 30\n(D) 42", + "solution": "解:$5 x-a \\leqslant 0 \\Rightarrow x \\leqslant \\frac{a}{5} ; 6 x-b>0 \\Rightarrow x>\\frac{b}{6}$. 要使 $A \\cap B \\cap \\mathbf{N}=\\{2$,\n$3,4\\}$, 其充要条件是 $\\left\\{\\begin{array}{l}1 \\leqslant \\frac{b}{6}<2 \\\\ 4 \\leqslant \\frac{a}{5}<5\\end{array}\\right.$, 即 $\\left\\{\\begin{array}{l}6 \\leqslant b<12 \\\\ 20 \\leqslant a<25\\end{array}\\right.$, 故 $b$ 有 $\\mathrm{C}_6^1$ 种取法, $a$ 有 $\\mathrm{C}_5^1$ 种取法, 由乘法原理得数对 $(a, b)$ 的个数为 $\\mathrm{C}_5^1 \\mathrm{C}_6^1=30$ 个, 所以选 $\\mathrm{C}$.", + "remark": "", + "figures": [] +} \ No newline at end of file diff --git a/processed_dataset/calculation/0138.json b/processed_dataset/calculation/0138.json new file mode 100644 index 0000000000000000000000000000000000000000..d0cb83550870c158587a3baf4b05f8c1450665b8 --- /dev/null +++ b/processed_dataset/calculation/0138.json @@ -0,0 +1,8 @@ +{ + "source_file": "./raw_volume-zh/volume11/chapter1.tex", + "problem_type": "calculation", + "problem": "例3. 由 $1,2,3,4,5$ 可以组成多少个没有重复数字, 并且大于 21300 的正整数?", + "solution": "解:法 1 由 $1,2,3,4,5$ 组成的没有重复数字, 并且大于 21300 的正整数可分为 3 类:\n万位数字为 $3,4,5$ 的有 $\\mathrm{A}_3^1 \\cdot \\mathrm{A}_4^4$ 个;\n万位数字为 2 ,千位数字为 $3,4,5$ 的有 $\\mathrm{A}_3^1 \\cdot \\mathrm{A}_3^3$ 个;\n万位数字为 2 ,千位数字为 1 的有 $\\mathrm{A}_3^3$ 个.\n由加法原理, 符合条件的正整数的个数是\n$$\n\\mathrm{A}_3^1 \\cdot \\mathrm{A}_4^4+\\mathrm{A}_3^1 \\cdot \\mathrm{A}_3^3+\\mathrm{A}_3^3=96 .\n$$\n解法 2 由 $1,2,3,4,5$ 组成的没有重复数字的 5 位数共有 $\\mathrm{A}_5^5$ 个, 其中只有万位数字为 1 的数不大于 21300 ,这样的 5 位数有 $\\mathrm{A}_4^4$ 个,故符合条件的正整数的个数是\n$$\n\\mathrm{A}_5^5-\\mathrm{A}_4^4=5 !-4 !=96 .\n$$", + "remark": "", + "figures": [] +} \ No newline at end of file diff --git a/processed_dataset/calculation/0139.json b/processed_dataset/calculation/0139.json new file mode 100644 index 0000000000000000000000000000000000000000..7fb8c74de63fc3af40dd30c8c9cbe3175cb099bf --- /dev/null +++ b/processed_dataset/calculation/0139.json @@ -0,0 +1,8 @@ +{ + "source_file": "./raw_volume-zh/volume11/chapter1.tex", + "problem_type": "calculation", + "problem": "例4. 从银行中取出伍角、壹元、式元、伍元、拾元、伍拾元、壹百元的纸币共 10 张,共有多少种不同的取法?", + "solution": "解:本题为从 7 种不同的纸币中取 10 种纸币可重复的组合数, 依可重复的组合数公式得不同的取法数目为\n$$\n\\mathrm{C}_{7+10-1}^{10}=\\mathrm{C}_{16}^6=\\frac{16 \\times 15 \\times 14 \\times 13 \\times 12 \\times 11}{1 \\times 2 \\times 3 \\times 4 \\times 5 \\times 6}=8008 .\n$$", + "remark": "", + "figures": [] +} \ No newline at end of file diff --git a/processed_dataset/calculation/0140.json b/processed_dataset/calculation/0140.json new file mode 100644 index 0000000000000000000000000000000000000000..8352b40326d0636e56e263a022e3f02d77eb7f0c --- /dev/null +++ b/processed_dataset/calculation/0140.json @@ -0,0 +1,8 @@ +{ + "source_file": "./raw_volume-zh/volume11/chapter1.tex", + "problem_type": "calculation", + "problem": "例5. 将 3 面红旗、4 面蓝旗、2 面黄旗依次悬挂在旗杆上,问可以组成多少种不同的标志?", + "solution": "解:由不全相异元素的全排列公式得所求标志数目为\n$$\n\\left(\\begin{array}{lll} \n& 9 & \\\\\n3 & 4 & 2\n\\end{array}\\right)=\\frac{9 !}{3 ! 4 ! 2 !}=1260 \\text {. }\n$$", + "remark": "", + "figures": [] +} \ No newline at end of file diff --git a/processed_dataset/calculation/0141.json b/processed_dataset/calculation/0141.json new file mode 100644 index 0000000000000000000000000000000000000000..99da56c2b0948c7f75e621b95d2499d25ae412eb --- /dev/null +++ b/processed_dataset/calculation/0141.json @@ -0,0 +1,8 @@ +{ + "source_file": "./raw_volume-zh/volume11/chapter1.tex", + "problem_type": "calculation", + "problem": "例6. 从 $n(n \\geqslant 6)$ 名乒乓球选手中选拔出 3 对选手准备参加双打比赛, 问共有多少种不同的方法?", + "solution": "解:法 1 从 $n$ 名选手中选出 6 名选手有 $\\mathrm{C}_n^6$ 种方法, 将这 6 名选手分成 3 个不同的组,每组 2 名有 $\\left(\\begin{array}{lll} & 6 & \\\\ 2 & 2 & 2\\end{array}\\right)$ 种方法, 但 3 对选手是不计顺序的,故所求的方法数应为\n解法 2 从 $n$ 名选手中选出 6 人有 $\\mathrm{C}_n^6$ 种方法, 选出的 6 人中选出 2 人配成一对有 $\\mathrm{C}_6^2$ 种方法, 剩下 4 人中选 2 人配成一对有 $\\mathrm{C}_4^2$ 种方法, 最后剩下的 2 人配成一对有 $\\mathrm{C}_2^2$ 种方法.\n但因选出的 3 对是不计顺序的, 故所求方法数为\n$$\n\\frac{\\mathrm{C}_n^6 \\mathrm{C}_6^2 \\mathrm{C}_4^2 \\mathrm{C}_2^2}{3 !}=\\frac{n !}{48(n-6) !} .\n$$\n注意本题若改为从 $n$ 名选手中选出 3 对选手分别列为第 $1 、 2 、 3$ 号种子选手,则其不同的选法数目为\n$$\n\\mathrm{C}_n^6 \\mathrm{C}_6^2 \\mathrm{C}_4^2 \\mathrm{C}_2^2=\\frac{n !}{8(n-6) !} .\n$$\n这是因为选出的 3 对选手是排了序的,故不要除以 $3 !$.", + "remark": "", + "figures": [] +} \ No newline at end of file diff --git a/processed_dataset/calculation/0142.json b/processed_dataset/calculation/0142.json new file mode 100644 index 0000000000000000000000000000000000000000..fe944977ab12073779b98cf649460b68a1b3b699 --- /dev/null +++ b/processed_dataset/calculation/0142.json @@ -0,0 +1,8 @@ +{ + "source_file": "./raw_volume-zh/volume11/chapter1.tex", + "problem_type": "calculation", + "problem": "例7. 6 位女同学和 15 位男同学围成一圈跳集体舞,要求每两名女同学之间至少有两名男同学,那么共有多少种不同的围圈跳舞的方法?", + "solution": "解法一,首先让每位女同学选择两名男同学作为她的舞伴, 一人排在她左侧, 另一人排在她右侧.\n由于 6 位女同学互不相同,故第 1 名女同学有 $15 \\times 14$ 种选法,第 2 名有 $13 \\times 12$ 种选法……共有 $\\mathrm{A}_{15}^{12}$ 种\"配对\"方法.\n将每名女同学和她的舞伴看成一组,剩下 $15-12=3$ 名男同学每人看成一组,一共有 9 个组, 把这 9 个组排成一圈, 共有 $(9-1) !=8$ ! 种排法.\n由乘法原理得满足条件的排列数为\n$$\n\\mathrm{A}_{15}^{12} \\cdot 8 !=\\frac{15 ! \\cdot 8 !}{3 !}\n$$", + "remark": "", + "figures": [] +} \ No newline at end of file diff --git a/processed_dataset/calculation/0143.json b/processed_dataset/calculation/0143.json new file mode 100644 index 0000000000000000000000000000000000000000..9a961a2fee2a81b54f56156164a6e0c759da7991 --- /dev/null +++ b/processed_dataset/calculation/0143.json @@ -0,0 +1,8 @@ +{ + "source_file": "./raw_volume-zh/volume11/chapter1.tex", + "problem_type": "calculation", + "problem": "例7. 6 位女同学和 15 位男同学围成一圈跳集体舞,要求每两名女同学之间至少有两名男同学,那么共有多少种不同的围圈跳舞的方法?", + "solution": "解法二,以女同学为组长, 15 位男同学分人 6 个组每组至少有两位男同学, 且记各组内男同学数分别为 $x_1, x_2, \\cdots, x_6$, 则分组的方法数等于不定方程\n$$\nx_1+x_2+\\cdots+x_6=15, x_i \\geqslant 2(i=1,2, \\cdots, 6). \\label{eq1}\n$$\n的整数解的个数, 令 $y_i=x_i-2(i=1,2, \\cdots, 6)$ 则\n$$\ny_1+y_2+\\cdots+y_6=3, y_i \\geqslant 0(i=1,2, \\cdots, 6) . \\label{eq2}\n$$\n故式\\ref{eq1}的整数解的个数等于式\\ref{eq2}的非负整数解的个数 $\\mathrm{C}_{3+6-1}^{6-1}=\\mathrm{C}_8^5=\\mathrm{C}_8^3$, 即 15 位男同学分人 6 个组, 每组至少 2 人的方法数为 $\\mathrm{C}_8^3 .6$ 个组排成一个圆圈有 5 ! 种方法 (这时女同学排在每组的首位, 她的位置已排定), 又 15 个男同学站人的方法数为 $\\mathrm{A}_{15}^{15}$, 故满足条件的排列数为 $\\mathrm{C}_8^3 \\cdot 5 ! \\cdot \\mathrm{A}_{15}^{15}=\\frac{8 ! 15 !}{3 !}$.", + "remark": "", + "figures": [] +} \ No newline at end of file diff --git a/processed_dataset/calculation/0144.json b/processed_dataset/calculation/0144.json new file mode 100644 index 0000000000000000000000000000000000000000..891d626b92002a6ad4d1465cca66cac6931dfeff --- /dev/null +++ b/processed_dataset/calculation/0144.json @@ -0,0 +1,8 @@ +{ + "source_file": "./raw_volume-zh/volume11/chapter1.tex", + "problem_type": "calculation", + "problem": "例8. 将 24 个志愿者名额分配给 3 个学校,则每校至少有一个名额且各校名额互不相同的分配方法共有?种.", + "solution": "解:设分配给 3 个学校的名额数分别为 $x_1, x_2, x_3$, 则每个学校至少有一个名额的分配方法数为不定方程 $x_1+x_2+x_3=24$ 的正整数解的个数,即 $\\mathrm{C}_{24-1}^{3-1}=\\mathrm{C}_{23}^2=\\frac{23 \\times 22}{2}=253$. 但上述分配方法中\"至少有两个学校名额数相同\"的分配方法有下列 31 种:\n$$\n\\begin{aligned}\n& (i, i, 24-2 i) 、(i, 24-2 i, i) 、(24-2 i, i, i) \\\\\n& (i=1,2,3,4,5,6,7,9,10,11) \\text { 及 }(8,8,8) .\n\\end{aligned}\n$$\n故满足条件的分配方法共有 $253-31=222$ (种).", + "remark": "", + "figures": [] +} \ No newline at end of file diff --git a/processed_dataset/calculation/0145.json b/processed_dataset/calculation/0145.json new file mode 100644 index 0000000000000000000000000000000000000000..377b699b9e4dc31ece430cc004e91f7a4730b2ea --- /dev/null +++ b/processed_dataset/calculation/0145.json @@ -0,0 +1,8 @@ +{ + "source_file": "./raw_volume-zh/volume11/chapter1.tex", + "problem_type": "calculation", + "problem": "例9. 方程 $x+y+z=2010$ 满足 $x \\leqslant y \\leqslant z$ 的正整数解 $(x, y, z)$ 的个数是?", + "solution": "解:首先易知 $x+y+z=2010$ 的正整数解的个数为 $\\mathrm{C}_{2009}^2=2009 \\times 1004$; 其次,把 $x+y+z=2010$ 满足 $x \\leqslant y \\leqslant z$ 的正整数解分为三类:\n(1) $x=y=z$ 的正整数解只有 1 个: $(670,670,670)$;\n(2) $x, y, z$ 中恰有 2 个相等的正整数解有下列 1003 个: $(x, x, 2010- 2 x) 、(x=1,2, \\cdots, 669)$ 以及 $(2010-2 y, y, y)(y=671,672, \\cdots, 1004)$;\n(3) 设 $x, y, z$ 两两不等的正整数解有 $k$ 个,于是\n$$\n1+1003 \\times \\mathrm{C}_3^1+k \\times 3 !=\\mathrm{C}_{2009}^2=2009 \\times 1004,\n$$\n所以\n$$\n\\begin{aligned}\nk & =\\frac{1}{6}(2009 \\times 1004-3 \\times 1003-1) \\\\\n& =\\frac{1}{6}(2009 \\times 1005-2009-3 \\times 1005+3 \\times 2-1) \\\\\n& =\\frac{1}{6}(2006 \\times 1005-2004)=1003 \\times 335-334\n\\end{aligned}\n$$\n$$\n=335671 \\text {. }\n$$\n故满足 $x \\leqslant y \\leqslant z$ 的正整数解 $(x, y, z)$ 的个数为\n$$\n1+1003+335671=336675 .\n$$", + "remark": "", + "figures": [] +} \ No newline at end of file diff --git a/processed_dataset/calculation/0146.json b/processed_dataset/calculation/0146.json new file mode 100644 index 0000000000000000000000000000000000000000..f6e2989028fe0ce208b7361d46d9759c3ee880e4 --- /dev/null +++ b/processed_dataset/calculation/0146.json @@ -0,0 +1,8 @@ +{ + "source_file": "./raw_volume-zh/volume11/chapter1.tex", + "problem_type": "calculation", + "problem": "例10. 有 $n$ 封不同的信和 $n$ 个配套的写有收信人地址的信封, 现将 $n$ 封信一对一地套人到 $n$ 个信封中去, 结果发现没有一封信套对 (即每封信都没有按地址套人其应套人的信封), 问有多少种不同的套法?", + "solution": "解:设 $S$ 是所有套法组成的集合,则显然有 $|S|=n$ !. 我们把每封信和对应的信封都分别用 $1,2,3, \\cdots, n$ 进行编号, 并记 $A_i(i=1,2, \\cdots, n)$ 为第 $i$ 封信恰套人第 $i$ 个信封 (即套正确) 的所有套法构成的集合, 故所求的方法数即为 $\\left|\\complement_S A_1 \\cap \\complement_S A_2 \\cap \\cdots \\cap \\complement_S A_\\eta\\right|$, 而易知\n$$\n\\begin{aligned}\n& \\left|A_i\\right|=(n-1) !(1 \\leqslant i \\leqslant n), \\\\\n& \\left|A_i \\cap A_j\\right|=(n-2) !(1 \\leqslant in-\\frac{n}{3}-\\frac{n}{4}+\\left(\\frac{n}{3 \\times 4}-1\\right)+\\left(\\frac{n}{3 \\times 5}-1\\right)+\\left(\\frac{n}{4 \\times 5}-1\\right)- \\\\\n\\frac{n}{3 \\times 4 \\times 5} & =\\frac{3}{5} n-3 . \\label{eq3}\n\\end{aligned}\n$$\n由 式\\ref{eq2} 和 \\ref{eq3} 联立解得\n$$\n3346 \\frac{2}{3}2(n+1), m \\geqslant 3 .\n$$\n即 $A$ 至少平 3 场, 故可找到一个队, 它和 $A$ 踢成平局, 这个队得分至少为\n$2(n+1)+1$ 分, 从而有\n$$\n2 n+m>2(n+1)+1, m \\geqslant 4 .\n$$\n设共有 $S$ 个队参加比赛, 则 $A$ 队至少胜一场, 否则 $A$ 队的得分不超过 $S-1$, 而任何其他队的得分都严格少于 $S-1$, 这样所有参赛队的得分严格少于 $S(S-1)$ 这与 $S$ 个队参赛所得的总分为 $2 \\mathrm{C}_S^2=S(S-1)$ 矛盾.\n于是 $m \\geqslant 4, n \\geqslant 1$, 即 $A$ 队至少要比赛 5 场, 所以参加比赛的队不少于 6 个.\n\\begin{tabular}{|c|c|c|c|c|c|c|c|}\n\\hline & $A$ & $B$ & $C$ & $D$ & $E$ & $F$ & 得分 \\\\\n\\hline$A$ & & 1 & 1 & 1 & 1 & 2 & 6 \\\\\n\\hline$B$ & 1 & & 2 & 0 & 0 & 2 & 5 \\\\\n\\hline$C$ & 1 & 0 & & 0 & 2 & 2 & 5 \\\\\n\\hline$D$ & 1 & 2 & 2 & & 0 & 0 & 5 \\\\\n\\hline$E$ & 1 & 2 & 0 & 2 & & 0 & 5 \\\\\n\\hline$F$ & 0 & 0 & 0 & 2 & 2 & & 4 \\\\\n\\hline\n\\end{tabular}\n另一方面, 如表中所示 $A 、 B 、 C 、 D 、 E 、 F$ 六个队满足题目要求: $A$ 队胜的场次最少,而得分最多.\n综上可知,最少应有 6 个队参加比赛.", + "remark": "", + "figures": [] +} \ No newline at end of file diff --git a/processed_dataset/calculation/0150.json b/processed_dataset/calculation/0150.json new file mode 100644 index 0000000000000000000000000000000000000000..76f979f9b317b4236700d7e3319f49d2571a3987 --- /dev/null +++ b/processed_dataset/calculation/0150.json @@ -0,0 +1,8 @@ +{ + "source_file": "./raw_volume-zh/volume11/chapter11.tex", + "problem_type": "calculation", + "problem": "例2. 试将 1000 分成若干个互不相等的正整数之和,并且使得这些正整数的乘积为最大, 求这个最大值.", + "solution": "解:因分法个数有限, 故乘积最大的分法 $S$ 必存在, 设将 1000 分为不相等的正整数 $a_1, a_2, \\cdots, a_k$ 之和时, 其乘积为最大.\n用 $S=\\left\\{a_1, a_2, \\cdots, a_k\\right\\}$ 表示分法, $\\sum S$ 表示 $S$ 中各数之和.\n$\\prod S$ 表示 $S$ 中各数之积.\n$S$ 中最小数为 $a_1$, 最大数为 $a_k$ 则 $S$ 具有下列性质:\n(1) $S$ 中不属于 $\\left[a_1, a_k\\right]$ 的正整数至多只有一个.\n事实上, 若 $a, b \\notin S (a1, \\prod S^{\\prime}>\\prod S .\n$$\n这与 $\\prod S$ 最大矛盾.\n(2) $a_1 \\neq 1$, 因为若 $a_1=1$, 则令 $S^{\\prime}=\\left(S \\backslash\\left\\{1, a_k\\right\\}\\right) \\cup\\left\\{1+a_k\\right\\}$, 于是 $\\sum S^{\\prime}=\\sum S=1000$, 且\n$$\n\\frac{\\prod S^{\\prime}}{\\prod S}=\\frac{1+a_k}{1 \\cdot a_k}>1\n$$\n这与 $I S$ 最大矛盾.\n(3) $a_1=2$ 或 3 .\n(i) 若 $a_1=4$ 且 $5 \\in S$, 则令 $S^{\\prime}=(S \\backslash\\{5\\}) \\cup\\{2,3\\}$, 于是 $\\sum S^{\\prime}= \\sum S=1000$ 且 $\\frac{\\prod S^{\\prime}}{\\prod S}=\\frac{2 \\times 3}{5}>1$, 矛盾.\n(ii) 若 $a_1=4$, 且 $j \\notin S(j=5,6, \\cdots, k-1)$, 但 $k \\in S(k \\geqslant 6)$, 则令 $S^{\\prime}=(S \\backslash\\{4, k\\}) \\cup\\{2,3, k-1\\}$, 于是 $\\sum S^{\\prime}=\\sum S=1000$, 而 $\\frac{\\prod S^{\\prime}}{\\prod S}= \\frac{2 \\times 3 \\times(k-1)}{4 k}=\\frac{4 k+2(k-3)}{4 k}>1$, 矛盾.\n(iii) 若 $a_1 \\geqslant 5$, 则令 $S^{\\prime}=\\left(S \\backslash\\left\\{a_1\\right\\}\\right) \\cup\\left\\{2, a_1-2\\right\\}$, 于是 $\\sum S^{\\prime}= \\sum S=1000$, 而 $\\frac{\\prod S}{\\prod S}=\\frac{2\\left(a_1-2\\right)}{a_1}=\\frac{a_1+\\left(a_1-4\\right)}{a_1}>1$, 矛盾.\n由上述 (1), (2), (3) 知, 若 $a_1=3$, 则必有 $3+4+5+\\cdots+n-k=1000$, 即 $(n-2)(n+3)=2000+2 k$, 于是 $n=45, k=32$, 即 $S=\\{3,4, \\cdots, 30$,\n$31,33,34, \\cdots, 44,45\\}$. 令 $\\left.S^{\\prime}=(S \\backslash\\{34\\}) \\cup\\{2,32\\}\\right)$, 则 $\\sum S^{\\prime}=\\sum S=$ 1000 , 而 $\\frac{\\prod S^{\\prime}}{\\prod S}=\\frac{2 \\times 32}{34}>1$, 矛盾.\n故只有 $a_1=2$, 这时 $S=\\{2,3, \\cdots, 33$, $35,36, \\cdots, 45\\}$, 使乘积 $\\prod S=\\frac{45 !}{34}$ 最大.", + "remark": "", + "figures": [] +} \ No newline at end of file diff --git a/processed_dataset/calculation/0151.json b/processed_dataset/calculation/0151.json new file mode 100644 index 0000000000000000000000000000000000000000..92dd3dc8fd0ed9a4fe7529df9073c9d394490adb --- /dev/null +++ b/processed_dataset/calculation/0151.json @@ -0,0 +1,8 @@ +{ + "source_file": "./raw_volume-zh/volume11/chapter11.tex", + "problem_type": "calculation", + "problem": "例4. 设圆周上放若干堆小球,每堆中的小球数都是 3 的倍数,但各堆中的球数不必相等.\n按下列规则调整各堆中的球数; 把各堆球三等分, 本堆留一份, 其余两份分别放人左、右两堆中, 如果某堆球数不是 3 的倍数,则可从布袋中取出一球或两球补人, 使该堆球数是 3 的倍数,然后按上述方法继续调整, 问能否经过有限次调整使各堆的球数相等?", + "solution": "解:设某次调整前, 球数最多的堆有 $3 m$ 个球,最少的有 $3 n$ 个球且 $m> n$, 那么\n(1)经过调整后,各堆中的球数仍在 $3 m$ 与 $3 n$ 之间(包括 $3 n$ 和 $3 m$ 在内、 下同).\n事实上,设某堆有 $3 l$ 个球,它的左、右两堆的球数分别为 $3 k$ 和 $3 h(3 m \\geqslant 3 l \\geqslant 3 n, 3 m \\geqslant 3 k \\geqslant 3 n, 3 m \\geqslant 3 h \\geqslant 3 n)$ 于是调整后该堆球数为 $k+l+h$, 满足 $3 m \\geqslant k+l+h \\geqslant 3 n$, 若这堆球数不是 3 的倍数 (即 $3 m>k+l+h>3 n$ ), 则补充一个或两个球成为 3 的倍数,其球数仍在 $3 n$ 与 $3 m$ 之间.\n(2) 原来球数大于 $3 n$ 的堆,调整后的球数仍大于 $3 n$.\n事实上同 (1) 知道若 $3 l>3 n, 3 k \\geqslant 3 n, 3 h \\geqslant 3 n$, 则 $k+l+h>3 n$.\n(3) 原来球数为 $3 n$ 的堆中, 经过调整, 至少有一堆的球数大于 $3 n$.\n事实上, 原来球数为 $3 n$ 的堆中, 至少有一堆, 它的左或右堆中的球数大于 $3 n$, 同(1)记号不妨设 $3 k>3 n, 3 l=3 n, 3 h \\geqslant 3 n$, 则 $k+l+h>3 n$.\n于是, 每调整一次, 球数为 $3 n$ 的堆至少减少一堆.\n故经过有限次调整, 可使每堆球数都大于 $3 n$. 而含球数最多的堆中的球数不会增大, 于是含球数最多的堆与含球数最少的堆所含球数之差 $f$ 将严格地减少.\n故经过有限步, 这个差数 $f$ 必为零, 即各堆球数相等.", + "remark": "注:在解单人操作能否达到预定目标的问题时, 常常根据题目条件建立一个取非负整数值的目标函数 $f$, 若没有达到目标, 则证明经过适当调整后可使 $f$ 严格减少一个正整数.\n于是由 $f$ 恒取非负整数值知, 这种调整过程不可能无限次继续下去, 这就证明了可经过有限次操作达到预定目标, 在本题中目标函数 $f=3 m-3 n$.", + "figures": [] +} \ No newline at end of file diff --git a/processed_dataset/calculation/0152.json b/processed_dataset/calculation/0152.json new file mode 100644 index 0000000000000000000000000000000000000000..f54d69a1441c2dd3930422edfe68f4d12bb951b7 --- /dev/null +++ b/processed_dataset/calculation/0152.json @@ -0,0 +1,8 @@ +{ + "source_file": "./raw_volume-zh/volume11/chapter13.tex", + "problem_type": "calculation", + "problem": "例1. 有 6 个红球, 3 个蓝球, 3 个黄球.\n将这些球放在一条直线上.\n假设同色球没有区别,试问: 有多少种不同的放法使得同色球不相邻?", + "solution": "解:因为有 6 个红球和 6 个非红球, 且任意两个红球之间至少有一个非红球,所以一个蓝球与一个黄球相邻最多出现一次.\n若蓝球与黄球不相邻, 则红球和非红球交替放着, 这时红球的放法只有 2 种 (即最左端为红球或非红球), 蓝球有 $\\mathrm{C}_6^3$ 种放法, 黄球有 $\\mathrm{C}_3^3$ 种放法), 不同的放法有 $2 \\mathrm{C}_6^3 \\mathrm{C}_3^3=40$ 种.\n若有一个蓝球和一个黄球相邻, 可将其合并看成一个 \"花球\", 则非红球由 2 个蓝球、 2 个黄球和一个花球组成, 于是任意两个相邻红球之间将有一个非红球, 这样的放法有 $\\frac{5 !}{2 ! 2 ! 1 !}=30$ (种). 又因花球包含 2 种不同的放法, 所以有一个蓝球与一个黄球相邻的放法有 $2 \\times 30=60$ 种.\n综上,满足题目条件的放法共有 $40+60=100$ 种.", + "remark": "", + "figures": [] +} \ No newline at end of file diff --git a/processed_dataset/calculation/0153.json b/processed_dataset/calculation/0153.json new file mode 100644 index 0000000000000000000000000000000000000000..3333670553693bc42ddb92ef85c1296a2a6a5393 --- /dev/null +++ b/processed_dataset/calculation/0153.json @@ -0,0 +1,8 @@ +{ + "source_file": "./raw_volume-zh/volume11/chapter13.tex", + "problem_type": "calculation", + "problem": "例2. 从给定的 6 种不同颜色中选用若干种颜色, 将一个正方体的 6 个面染色,每面恰染一色, 具有公共棱的两个面不同色, 则不同的染色方案有种.\n(约定经过翻滚或旋转可以变相同的染色方案是相同的染色方案)", + "solution": "解:因有公共顶点的三个面互不同色,故至少要用 3 色,下分 4 种情形.\n(1)6种颜色都用时,先将染某种固定颜色的面朝上, 从剩下 5 色中取 1 色染下底面有 $\\mathrm{C}_5^1$ 种方法,余下 4 色染余下的 4 个侧面 (应是 4 种颜色的圆排列) 有 $(4-1)$ ! 种染法, 所以 6 种颜色都用时, 染色方案有 $\\mathrm{C}_5^1 \\cdot(4-1)$ ! = 30 种.\n(2)只用 5 种颜色时, 从 6 色中取 5 色有 $\\mathrm{C}_6^5$ 种方法, 这时必有一组对面同色, 从 5 色中取 1 色染一组对面, 并将它们朝上和朝下, 有 $\\mathrm{C}_5^1$ 种方法, 余下 4 色染余下的 4 个侧面 (应是 4 粒不同颜色珠子的项链), 有 $\\frac{1}{2} \\cdot(4-1)$ ! 种方法, 所以只用 5 种颜色时, 染色方案有 $\\mathrm{C}_6^5 \\cdot \\mathrm{C}_5^1 \\cdot \\frac{1}{2} \\cdot(4-1) !=90$ 种.\n(3)只用 4 种颜色时, 从 6 色中取 4 色有 $\\mathrm{C}_6^4$ 种方法, 这时必有 2 组对面同色, 另一组对面不同色, 从 4 色中取 2 色染一组对面, 并将它们朝上和朝下 (这时上、下底面无区别)有 $\\mathrm{C}_4^2$ 种方法, 其余 2 色染余下 4 个侧面且使 2 组对面同色 (应是 2 粒不同颜色珠子的项链) 只有 1 种染法, 故只用 4 色时, 染色方案有 $\\mathrm{C}_6^4 \\mathrm{C}_4^2 \\cdot 1=90$ 种.\n(4)只用 3 色时, 从 6 色中取 3 色有 $\\mathrm{C}_6^3$ 种方法, 这时 3 组对面同色, 用 3 种颜色去给它们染色只有 1 种染法, 所以只用 3 色时, 染色方案有 $\\mathrm{C}_6^3 \\cdot 1=20$ 种.\n综上可知,不同的染色方案共有 $30+90+90+20=230$ 种.", + "remark": "", + "figures": [] +} \ No newline at end of file diff --git a/processed_dataset/calculation/0154.json b/processed_dataset/calculation/0154.json new file mode 100644 index 0000000000000000000000000000000000000000..1b596f994abcc28b1ed74c8574db32d3251d7434 --- /dev/null +++ b/processed_dataset/calculation/0154.json @@ -0,0 +1,8 @@ +{ + "source_file": "./raw_volume-zh/volume11/chapter13.tex", + "problem_type": "calculation", + "problem": "例3. 凸 $n$ 边形的任意 3 条对角线不交于形内一点, 求这些对角线将凸 $n$ 边形分成的区域的个数.", + "solution": "解法一设凸 $n$ 边形被对角线分成的区域中边数最多的为 $m$ 边形, 其中三角形有 $n_3$ 个, 四边形有 $n_4$ 个, $\\cdots, m$ 边形有 $n_m$ 个, 于是凸 $n$ 边形被分成的区域个数为\n$$\nS=n_3+n_4+\\cdots+n_m .\n$$\n一方面各区域的顶点数的总和为 $3 n_3+4 n_4+\\cdots+m m_m$, 另一方面, 对角线在形内有 $\\mathrm{C}_n^4$ 个交点,每个交点是 4 个区域的公共顶点 (因无 3 条对角线交于形内一点), 又凸 $n$ 边形有 $n$ 个顶点, 每个顶点是 $n-2$ 个区域的公共顶点, 所以\n$$\n3 n_3+4 n_4+\\cdots+m m_m=4 \\mathrm{C}_n^4+n(n-2) . \\label{eq1}\n$$\n其次, 各区域的内角总和为 $n_3 \\cdot 180^{\\circ}+n_4 \\cdot 360^{\\circ}+\\cdots+n_m \\cdot(m-2) \\cdot 180^{\\circ}$,\n另一方面, 原凸 $n$ 边形的 $n$ 个顶点处的内角和为 $(n-2) \\cdot 180^{\\circ}$, 又 $\\mathrm{C}_n^4$ 个对角线交点处的内角和为 $\\mathrm{C}_n^4 \\cdot 360^{\\circ}$, 所以\n$$\n\\begin{aligned}\n& n_3 \\cdot 180^{\\circ}+n_4 \\cdot 360^{\\circ}+\\cdots+n_m \\cdot(m-2) \\cdot 180^{\\circ} \\\\\n= & (n-2) \\cdot 180^{\\circ}+\\mathrm{C}_n^4 \\cdot 360^{\\circ},\n\\end{aligned}\n$$\n即\n$$\nn_3+2 n_4+\\cdots+(m-2) n_m=2 \\mathrm{C}_n^4+(n-2), \\label{eq2}\n$$\n$[式\\ref{eq1} - \\ref{eq2}] \\div 2$, 得\n$$\n\\begin{aligned}\nS & =n_3+n_4+\\cdots+n_m=\\mathrm{C}_n^4+\\frac{1}{2}(n-1)(n-2) \\\\\n& =\\frac{1}{24}(n-1)(n-2)\\left(n^2-3 n+12\\right) .\n\\end{aligned}\n$$", + "remark": "", + "figures": [] +} \ No newline at end of file diff --git a/processed_dataset/calculation/0155.json b/processed_dataset/calculation/0155.json new file mode 100644 index 0000000000000000000000000000000000000000..32943ee5c590fccd9ef984127d7b57d3b4a6bc79 --- /dev/null +++ b/processed_dataset/calculation/0155.json @@ -0,0 +1,8 @@ +{ + "source_file": "./raw_volume-zh/volume11/chapter13.tex", + "problem_type": "calculation", + "problem": "例3. 凸 $n$ 边形的任意 3 条对角线不交于形内一点, 求这些对角线将凸 $n$ 边形分成的区域的个数.", + "solution": "解法二 每去掉一条对角线, 则区域减少的个数为 $a_i+1$, 这是 $a_i$ 是该对角线与还没有去掉的对角线在形内的交点数, 逐步将 $\\mathrm{C}_n^2-n$ 条对角线去掉, 最后, 剩下一个区域,故所求区域数为 $S=1+\\sum_{i=1}^{C_n^2-n}\\left(a_i+1\\right)$, 而 $\\sum_{i=1}^{C_n^2 \\rightarrow n} a_i$ 恰等于对角线在形内的交点数 $\\mathrm{C}_n^4$, 所以\n$$\nS=1+\\mathrm{C}_n^4+\\mathrm{C}_n^2-n=\\frac{1}{24}(n-1)(n-2)\\left(n^2-3 n+12\\right) .\n$$", + "remark": "", + "figures": [] +} \ No newline at end of file diff --git a/processed_dataset/calculation/0156.json b/processed_dataset/calculation/0156.json new file mode 100644 index 0000000000000000000000000000000000000000..d045119ecf13ee4c6033112b4074f3226b2bef88 --- /dev/null +++ b/processed_dataset/calculation/0156.json @@ -0,0 +1,8 @@ +{ + "source_file": "./raw_volume-zh/volume11/chapter13.tex", + "problem_type": "calculation", + "problem": "例3. 凸 $n$ 边形的任意 3 条对角线不交于形内一点, 求这些对角线将凸 $n$ 边形分成的区域的个数.", + "solution": "解法三 设凸 $n$ 边形被对角线分成的区域数为 $S$,加上凸 $n$ 边形外的区域, 共有 $F=S+1$ 个区域, 相当一个凸 $F$ 面体, 它的顶点数为 $V=\\mathrm{C}_n^4+n$, 又设它的边数为 $E$, 则从凸 $n$ 边形内每一交点处出发有 4 条边, 共 $4 \\mathrm{C}_n^4$ 条边, 另外, 从凸 $n$ 边形每一顶点出发有 $n-3$ 条对角线上的线段, 故从 $n$ 个顶点出发共有 $n(n-3)$ 条线段,但以上计算每条线段计算了 2 次, 故对角线被交点分成的线段数为\n$$\n\\frac{1}{2}\\left[4 \\mathrm{C}_n^4+n(n-3)\\right]=2 \\mathrm{C}_n^4+\\frac{1}{2} n(n-3),\n$$\n加上凸 $n$ 边形的 $n$ 条边得\n$$\n\\begin{aligned}\nE & =2 \\mathrm{C}_n^4+\\frac{1}{2} n(n-3)+n \\\\\n& =2 \\mathrm{C}_n^4+\\mathrm{C}_n^2 .\n\\end{aligned}\n$$\n代入欧拉(Euler)公式 $V+F-E=2$ 得\n$$\n\\begin{aligned}\nS=F-1 & =E-V+1 \\\\\n& =2 \\mathrm{C}_n^4+\\mathrm{C}_n^2-\\mathrm{C}_n^4-n+1\n\\end{aligned}\n$$\n$$\n=\\frac{1}{24}(n-1)(n-2)\\left(n^2-3 n+12\\right) .\n$$", + "remark": "", + "figures": [] +} \ No newline at end of file diff --git a/processed_dataset/calculation/0157.json b/processed_dataset/calculation/0157.json new file mode 100644 index 0000000000000000000000000000000000000000..6b32ced1b04b5a3f825398e9dd8dad5b7b2c4386 --- /dev/null +++ b/processed_dataset/calculation/0157.json @@ -0,0 +1,8 @@ +{ + "source_file": "./raw_volume-zh/volume11/chapter13.tex", + "problem_type": "calculation", + "problem": "例3. 凸 $n$ 边形的任意 3 条对角线不交于形内一点, 求这些对角线将凸 $n$ 边形分成的区域的个数.", + "solution": "解法四设凸 $n$ 边形被对角线分成的区域数为 $a_n$,于是 $a_3=1, a_4=4$. 在凸 $n-1$ 边形 $P_1 P_2 \\cdots P_{n-1}$ 基础上增加一个顶点 $P_n$ 得凸 $n$ 边形 $P_1 P_2 \\cdots P_n$, 则增加 1 个区域 $\\triangle P_1 P_{n-1} P_n$, 再增加从 $P_n$ 出发的 $n-3$ 条对角线, 则增加的区域数应为这 $n-3$ 条对角线上的交点数 $\\mathrm{C}_n^4-\\mathrm{C}_{n-1}^4$ 加上 $n-3$. 故得\n$$\n\\begin{aligned}\na_n & =a_{n-1}+\\left(\\mathrm{C}_n^4-\\mathrm{C}_{n-1}^4\\right)+(n-3)+1 \\\\\n& =a_{n-1}+\\left(\\mathrm{C}_n^4-\\mathrm{C}_{n-1}^4\\right)+n-2(n \\geqslant 5) .\n\\end{aligned}\n$$\n如果约定 $a_2=0, \\mathrm{C}_2^4=\\mathrm{C}_3^4=0$, 则上式当 $n=3,4$ 时也成立.\n所以\n$$\n\\begin{aligned}\na_n & =a_2+\\sum_{k=3}^n\\left(a_k-a_{k-1}\\right) \\\\\n& =0+\\sum_{k=3}^n\\left[\\left(\\mathrm{C}_k^4-\\mathrm{C}_{k-1}^4\\right)+(k-2)\\right] \\\\\n& =\\mathrm{C}_n^4+\\frac{1}{2}(n-1)(n-2) \\\\\n& =\\frac{1}{24}(n-1)(n-2)\\left(n^2-3 n+12\\right) .\n\\end{aligned}\n$$", + "remark": "", + "figures": [] +} \ No newline at end of file diff --git a/processed_dataset/calculation/0158.json b/processed_dataset/calculation/0158.json new file mode 100644 index 0000000000000000000000000000000000000000..3c66c62898e00cbd997a5dc56dcae4d213aa99d8 --- /dev/null +++ b/processed_dataset/calculation/0158.json @@ -0,0 +1,8 @@ +{ + "source_file": "./raw_volume-zh/volume11/chapter13.tex", + "problem_type": "calculation", + "problem": "例4. 设 $a_1 a_2 a_3 a_4 a_5$ 是 $1,2,3,4,5$ 的排列, 满足: 当 $i=1,2,3,4$ 时, $a_1 a_2 \\cdots a_i$ 都不是 $1,2, \\cdots, i$ 的排列, 求这种排列的个数.", + "solution": "解法一显然 $a_1 \\neq 1$.\n(1) $a_1=5$ 时, $a_2 a_3 a_4 a_5$ 是 $1,2,3,4$ 的任何排列均满足题目要求, 这时排列有 4 ! 个;\n(2) $a_1=4$ 时,一共有 4 ! 个排列, 其中形如 $4 \\times \\times \\times 5$ 的 3 ! 个排列不满足要求, 故这时满足要求的排列有 $4 !-3$ !个;\n(3) $a_1=3$ 时,一共有 4 ! 个排列, 其中形如 $3 \\times \\times \\times 5$ 和 $3 \\times \\times 54$ 的排列不满足要求,故这时满足要求的排列个数为 $4 !-3 !-2 !$;\n(4) $a_1=2$ 时,一共有 4 ! 个排列, 其中形如 $215 \\times \\times, 21453,2 \\times \\times 54,2 \\times \\times \\times 5$ 的排列不满足要求, 故这时满足要求的排列有 $4 !-2 !-1- 2 !-3 !$ 个.\n综上可得, 满足要求的排列共有\n$$\n\\begin{aligned}\n& 4 !+(4 !-3 !)+(4 !-3 !-2 !)+(4 !-2 !-1-2 !-3 !) \\\\\n= & 24+18+16+13=71 \\text { (个). }\n\\end{aligned}\n$$", + "remark": "", + "figures": [] +} \ No newline at end of file diff --git a/processed_dataset/calculation/0159.json b/processed_dataset/calculation/0159.json new file mode 100644 index 0000000000000000000000000000000000000000..7b2278c97841ecf403b78897e6441bd82a11b19f --- /dev/null +++ b/processed_dataset/calculation/0159.json @@ -0,0 +1,8 @@ +{ + "source_file": "./raw_volume-zh/volume11/chapter13.tex", + "problem_type": "calculation", + "problem": "例4. 设 $a_1 a_2 a_3 a_4 a_5$ 是 $1,2,3,4,5$ 的排列, 满足: 当 $i=1,2,3,4$ 时, $a_1 a_2 \\cdots a_i$ 都不是 $1,2, \\cdots, i$ 的排列, 求这种排列的个数.", + "solution": "解法二设 $1,2,3,4,5$ 的所有排列组成的集合为 $S$, 令\n$$\n\\begin{aligned}\n& A_i=\\left\\{\\left(a_1, a_2, a_3, a_4, a_5\\right) \\mid\\left(a_1, a_2, a_3, a_4, a_5\\right) \\in S,\\right. \\\\\n& \\text { 且 } \\left.\\left(a_1, a_2, \\cdots, a_i\\right) \\text { 是 }(1,2, \\cdots, i) \\text { 的排列 }\\right\\}(1 \\leqslant i \\leqslant 4)\n\\end{aligned}\n$$\n于是所求排列个数为 $\\left|\\complement_S A_1 \\cap \\complement_S A_2 \\cap \\complement_S A_3 \\cap \\complement_S A_4\\right|$, 易知\n$$\n\\begin{aligned}\n& |S|=5 !,\\left|A_1\\right|=4 !,\\left|A_2\\right|=2 ! \\cdot 3 !,\\left|A_3\\right|=3 ! \\cdot 2 !, \\\\\n& \\left|A_4\\right|=4 !,\\left|A_1 \\cap A_2\\right|=3 !,\\left|A_1 \\cap A_3\\right|=2 ! 2 !,\\left|A_1 \\cap A_4\\right|=3 !, \\\\\n& \\left|A_2 \\cap A_3\\right|=2 ! \\cdot 2 !,\\left|A_2 \\cap A_4\\right|=2 ! \\cdot 2 !,\\left|A_3 \\cap A_4\\right|=3 !, \\\\\n& \\left|A_1 \\cap A_2 \\cap A_3\\right|=2 !,\\left|A_1 \\cap A_2 \\cap A_4\\right|=2 !,\\left|A_1 \\cap A_3 \\cap A_4\\right|=2 !, \\\\\n& \\left|A_2 \\cap A_3 \\cap A_4\\right|=2 !,\\left|A_1 \\cap A_2 \\cap A_3 \\cap A_4\\right|==1 .\n\\end{aligned}\n$$\n故由容斥原理可得符合条件的排列个数为\n$$\n\\begin{aligned}\n& \\left|\\complement_S A_1 \\cap \\complement_S A_2 \\cap \\complement_S A_3 \\cap \\complement_S A_4\\right| \\\\\n= & |S|-\\sum_{i=1}^4\\left|A_i\\right|+\\sum_{1 \\leqslant i3$, 考虑 $\\{1,2, \\cdots, n\\}$ 的一个好的排列 $\\left(a_1, a_2, \\cdots a_n\\right)$, 由好的排列的定义知 $n-1$ 应是\n$$\n\\begin{aligned}\n2\\left(a_1+a_2+\\cdots+a_{n-1}\\right) & =2\\left(1+2+\\cdots+n-a_n\\right) \\\\\n& =n(n+1)-2 a_n=(n-1)(n+2)-\\left(2 a_n-2\\right)\n\\end{aligned}\n$$\n的因数, 因此, $2 a_n-2$ 可以被 $n-1$ 整除.\n而 $0 \\leqslant 2 a_n-2 \\leqslant 2(n-1)$, 于是, $2 a_n -2=0$ 或 $n-1$,或 $2(n-1)$. 即 $a_n=1$ 或 $\\frac{n+1}{2}$ 或 $n$.\n(1) 若 $a_n=\\frac{n+1}{2}$, 则 $n-2$ 应是\n$$\n\\begin{aligned}\n2\\left(a_1+a_2+\\cdots+a_{n-2}\\right) & =2\\left[(1+2+\\cdots+n)-a_n-a_{n-1}\\right] \\\\\n& =n(n+1)-(n+1)-2 a_{n-1} \\\\\n& =(n-2)(n+2)-\\left(2 a_{n-1}-3\\right)\n\\end{aligned}\n$$\n的因数,因此, $2 a_{n-1}-3$ 可以被 $n-2$ 整除, 而 $-1 \\leqslant 2 a_n-3 \\leqslant 2 n-3$,于是 $2 a_n-3=0$ 或 $n-2$ 或 $2 n-4$. 由于 $2 a_n-3$ 是奇数,故它不可能等于 0 或 $2 n-4$, 所以 $2 a_n-3=n-2$, 即 $a_{n-1}=\\frac{n+1}{2}=a_n$ 矛盾;\n(2) 若 $a_n=n$, 则 $\\left(a_1, a_2, \\cdots, a_{n-1}\\right)$ 是 $\\{1,2, \\cdots, n-1\\}$ 的好的排列, 这样的好的排列有 $x_{n-1}$ 个, 每一个这样的好排列的后面加上 $n$, 使得到 $\\{1$, $2, \\cdots, n\\}$ 的一个满足 $a_n=n$ 的好的排列 $\\left\\{a_1, a_2, \\cdots, a_n\\right\\}$, 这种对应是一一对应, 故满足 $a_n=n$ 的好的排列 $\\left\\{a_1, a_2, \\cdots, a_n\\right\\}$ 有 $x_{n-1}$ 个;\n(3) 若 $a_n=1$, 则 $\\left\\{a_1-1, a_2-1, \\cdots, a_{n-1}-1\\right\\}$ 是 $\\{1,2, \\cdots, n-1\\}$ 的好的排列, 这是因为对任意 $k \\leqslant n-1$, 有\n$$\n2\\left[\\left(a_1-1\\right)+\\left(a_2-1\\right)+\\cdots+\\left(a_k-1\\right)\\right]=2\\left(a_1+\\cdots+a_k\\right)-2 k\n$$\n被 $k$ 整除.\n因此, $\\{1,2, \\cdots, n-1\\}$ 的 $x_{n-1}$ 个好的排列中任意一个 $\\left(b_1, b_2, \\cdots\\right.$, $\\left.b_{n-1}\\right)$ 对应于 $\\{1,2, \\cdots, n\\}$ 的一个好排列 $\\left\\{b_1+1, b_2+1, \\cdots, b_{n-1}+1,1\\right\\}$, 这种对应是一一对应,故满足 $a_n=1$ 的好的排列有 $x_{n-1}$ 个.\n综上知 $x_n=2 x_{n-1}(n \\geqslant 4)$. 由 $x_3=6$, 得当 $n \\geqslant 4$ 时, $x_n=3 \\times 2^{n-2}$, 故满足题目条件的排列个数为 $x_n= \\begin{cases}n & (n=1,2), \\\\ 3 \\times 2^{n-2} & (n \\geqslant 3) .\\end{cases}$", + "remark": "", + "figures": [] +} \ No newline at end of file diff --git a/processed_dataset/calculation/0161.json b/processed_dataset/calculation/0161.json new file mode 100644 index 0000000000000000000000000000000000000000..d8d9c76975d603ebb971e999c37522a66cf694ae --- /dev/null +++ b/processed_dataset/calculation/0161.json @@ -0,0 +1,10 @@ +{ + "source_file": "./raw_volume-zh/volume11/chapter13.tex", + "problem_type": "calculation", + "problem": "例6. 设 $M$ 为平面上坐标为 $(p \\times 1994,7 p \\times 1994)$ 的点, 其中 $p$ 为素数, 求满足下列条件的直角三角形的个数:\n(1)三角形的顶点都是整点, $M$ 是直角顶点;\n(2) 三角形的内心是坐标原点.", + "solution": "分析:如图(), 直角三角形 $M A B$ 满足条件 (1),(2), 其内切圆半径为 $r$, 过 $O$ 分别作 $M A$, $M B$ 的垂线, 垂足分别为 $C, D$, 则易知 $O C= O D=M C=M D=r=\\frac{1}{\\sqrt{2}} O M$, 故 Rt $\\triangle M A B$ 由 $B D=u$ 和 $A C=v$ 唯一确定.\n为此, 我们只要根据条件 (1),(2) 建立 $u, v$ 满足的不定方程 (组), 再通过不定方程 (组) 的解数的计算就可得出所求直角三角形的个数.\n这样就通过建立对应关系, 将一个复杂的不熟悉的问题转化为一个熟知的较易的问题来求解.\n解法一为了计算方便, 将坐标原点平移到 $M$, 建立新的直角坐标系 $x^{\\prime} M y^{\\prime}$, 于是在新的坐标系中, $M$ 的坐标为 $(0,0)$, 内心 $O$ 的坐标为 $(-p \\times 1994,-7 p \\times 1994)$. 所以\n$$\n\\begin{aligned}\n|O M| & =\\sqrt{(p \\times 1994)^2+(7 p \\times 1994)^2} \\\\\n& =p \\times 1994 \\times 5 \\sqrt{2} .\n\\end{aligned}\n$$\n设 $\\triangle M A B$ 符合条件,其内切圆半径为 $r$,过 $O$ 分别作 $M A$ 和 $M B$ 的垂线, 垂足分别为 $C$ 和 $D$, 设 $A M$ 的斜率为 $k$, 易知 $O M$ 的斜率为\n$$\nk^{\\prime}=\\frac{7 p \\times 1994}{p \\times 1994}=7,\n$$\n注意到 $O$ 为 Rt $\\triangle M A B$ 的内心, $\\angle O M A=45^{\\circ}$, 故由两直线夹角公式得\n$$\n1=\\tan 45^{\\circ}=\\frac{k^{\\prime}-k}{1+k k^{\\prime}}=\\frac{7-k}{1+7 k},\n$$\n所以 $k=\\frac{3}{4}$, 而 $M B \\perp M A$, 故 $M B$ 的斜率为 $k_1=-\\frac{4}{3}$. 由此, 可设 $A, B$ 的坐标为 $A(-4 t,-3 t), B\\left(3 t^{\\prime},-4 t^{\\prime}\\right)$, 因 $(3,4)=1$, 且 $A, B$ 为整点, 故 $t$, $t^{\\prime}$ 皆为正整数, 于是 $M A=5 t, M B=5 t^{\\prime}$, 并且 $M C=M D=r=M O \\cos 45^{\\circ}= p \\times 1994 \\times 5$.\n$$\n\\text { 记 } B D=u, A C=v \\text {, 则 } u=M B-M D=5\\left(t^{\\prime}-p \\times 1994\\right), v=\n$$\n$M A-M C=5(t-p \\times 1994)$. 记 $\\angle O B D=\\alpha, \\angle O A C=\\beta$, 易知 $\\alpha+\\beta= \\frac{1}{2}(\\angle M A B+\\angle M B A)=45^{\\circ}, \\tan \\alpha=\\frac{r}{u}, \\tan \\beta=\\frac{r}{v}$ 且 $\\frac{r}{u}=\\tan \\alpha= \\tan \\left(45^{\\circ}-\\beta\\right)=\\frac{1-\\tan \\beta}{1+\\tan \\beta}=\\frac{v-r}{v+r}$, 所以 $u=\\frac{r(v+r)}{v-r}$ 记 $v-r=m, \\frac{2 r^2}{m}=n$, 则 $u=\\frac{r(m+2 r)}{m}=r+n$. 可见 $m$ 及 $n$ 均为 5 的正整数倍, 且\n$$\nm n=2 r^2=2(p \\times 1994 \\times 5)^2=2^3 \\times 5^2 \\times 997^2 \\times p^2 .\n$$\n因为对于 $m, n$ 的一组正整数解 $(m, n)$, 可求出唯一一组 $u, v$ 的正整数解 $(u, v)=(n+r, m+r)$, 从而唯一确定一个符合条件的 Rt $\\triangle M A B$, 反之亦真.\n故符合条件的 Rt $\\triangle M A B$ 的个数 $S$ 等于不定方程 (1)的满足条件 $5 \\mid m$ 和 $5 \\mid n$ 的正整数解组 $(m, n)$ 的组数.\n1. 若 $p \\neq 2, p \\neq 997$, 则 $\\frac{m}{5} \\cdot \\frac{n}{5}=2^3 \\times p^2 \\times 997^2$ 的一切解为\n$$\n\\left\\{\\begin{array}{l}\n\\frac{m}{5}=2^i \\times p^j \\times 997^k \\\\\n\\frac{n}{5}=2^{3-i} \\times p^{2-j} \\times 997^{2-k}\n\\end{array} \\quad\\left(\\begin{array}{l}\ni=0,1,2,3 \\\\\nj=0,1,2 \\\\\nk=0,1,2\n\\end{array}\\right),\\right.\n$$\n共有 $4 \\times 3 \\times 3=36$ 组解.\n2. 若 $p=997$, 则 $\\frac{m}{5} \\cdot \\frac{n}{5}=2^3 \\times 997^4$ 的一切解为\n$$\n\\left\\{\\begin{array}{ll}\n\\frac{m}{5} & =2^i \\times 997^j \\\\\n\\frac{n}{5} & =2^{3-i} \\times 997^{4-j}\n\\end{array} \\quad\\left(\\begin{array}{l}\ni=0,1,2,3 \\\\\nj=0,1,2,3,4\n\\end{array}\\right),\\right.\n$$\n共有 $4 \\times 5=20$ 组解.\n3. 若 $p=2$, 则 $\\frac{m}{5} \\cdot \\frac{n}{5}=2^5 \\times 997^2$ 的一切解为\n$$\n\\left\\{\\begin{array}{ll}\n\\frac{m}{5}=2^i \\times 997^j \\\\\n\\frac{n}{5}=2^{5-i} \\times 997^{2-j}\n\\end{array} \\quad\\left(\\begin{array}{l}\ni=0,1,2,3,4,5 \\\\\nj=0,1,2\n\\end{array}\\right),\\right.\n$$\n共有 $6 \\times 3=18$ 组解.\n故符合条件的三角形个数为\n$$\nS=\\left\\{\\begin{array}{l}\n36, p \\neq 2 \\text { 且 } p \\neq 997 \\text { 时, } \\\\\n20, p=997 \\text { 时, } \\\\\n18, p=2 \\text { 时.\n}\n\\end{array}\\right.\n$$", + "remark": "", + "figures": [ + "./images/volume11/figures/fig-c13i1.png" + ] +} \ No newline at end of file diff --git a/processed_dataset/calculation/0162.json b/processed_dataset/calculation/0162.json new file mode 100644 index 0000000000000000000000000000000000000000..19bf201ac4b83462889e47728ba060762d1a37b5 --- /dev/null +++ b/processed_dataset/calculation/0162.json @@ -0,0 +1,8 @@ +{ + "source_file": "./raw_volume-zh/volume11/chapter13.tex", + "problem_type": "calculation", + "problem": "例6. 设 $M$ 为平面上坐标为 $(p \\times 1994,7 p \\times 1994)$ 的点, 其中 $p$ 为素数, 求满足下列条件的直角三角形的个数:\n(1)三角形的顶点都是整点, $M$ 是直角顶点;\n(2) 三角形的内心是坐标原点.", + "solution": "解法二记号同解法一, 并且同解法一可得 $M A=5 t, M B=5 t^{\\prime}\\left(t, t^{\\prime}\\right.$ 为正整数), 且 $r=p \\times 1994 \\times 5$, 记 $r_0=\\frac{r}{5}=2 \\times 997 \\times p(2,997, p$ 皆为素数 $)$ 于是 $A B=\\sqrt{M A^2+M B^2}=5 \\sqrt{t^2+t^{\\prime 2}}$. 由平面几何知识, 易知 $A B=M A+ M B-2 r$, 即 $5 \\sqrt{t^2+t^{\\prime 2}}=5 t+5 t^{\\prime}-10 r_0$, 两边除以 5 后, 再平方并经过整理可得\n$$\n\\left(t-2 r_0\\right)\\left(t^{\\prime}-2 r_0\\right)=2 r_0{ }^2=2^3 \\times 997^2 \\times p^2 .\n$$\n记 $m_0=t-2 r_0, n_0=t^{\\prime}-2 r_0$, 则\n$$\nm_0 n_0=2^3 \\times 997^2 \\times p^2 .\n$$\n易知所求 Rt $\\triangle M A B$ 的个数 $S$ 等于(2) 的正整数解组 $\\left(m_0, n_0\\right)$ 的个数, 同解法一可求得\n$$\nS=\\left\\{\\begin{array}{l}\n36, p \\neq 2 \\text { 且 } p \\neq 997, \\\\\n20, p=997, \\\\\n18, p=2 .\n\\end{array}\\right.\n$$", + "remark": "", + "figures": [] +} \ No newline at end of file diff --git a/processed_dataset/calculation/0163.json b/processed_dataset/calculation/0163.json new file mode 100644 index 0000000000000000000000000000000000000000..171f4af2102f44d1e6342ba2899d4454183ed667 --- /dev/null +++ b/processed_dataset/calculation/0163.json @@ -0,0 +1,8 @@ +{ + "source_file": "./raw_volume-zh/volume11/chapter15.tex", + "problem_type": "calculation", + "problem": "例1. 设 $M=\\{1,2, \\cdots, 1995\\}, A$ 是 $M$ 的子集且满足条件: 当 $x \\in A$ 时 $15 x \\notin A$, 则 $A$ 中元素的个数最多是 .", + "solution": "解:我们尽可能构造出一个满足条件且含元素最多的子集 $A$, 因为要使当 $x \\in A$ 时 $15 x \\notin A$, 只要 $15 x>1995$, 即 $x>133$, 可见 $A$ 包含 $\\{134,135$, $\\cdots, 1995\\}$, 并且要使当 $x \\in A$ 时, $15 x \\notin A$, 只要 $15 x<134$, 即 $x<9$, 可见 $A$ 又包含 $\\{1,2,3, \\cdots, 8\\}$, 于是, 我们取 $M$ 的子集 $A=\\{1,2, \\cdots, 8\\} \\cup \\{134,135, \\cdots, 1995\\}$, 它满足题目条件且 $|A|=8+(1995-133)=1870$.\n另一方面, 任取 $M$ 的一个满足条件的子集 $A$. 因为 $x$ 与 $15 x(x=9,10$, $11, \\cdots, 133)$ 中至少有一个不属于 $A$, 故 $|A| \\leqslant 1995-(133-8)=1870$.\n综上知 $A$ 中元素最多有 1870 个.", + "remark": "", + "figures": [] +} \ No newline at end of file diff --git a/processed_dataset/calculation/0164.json b/processed_dataset/calculation/0164.json new file mode 100644 index 0000000000000000000000000000000000000000..c3ed06e09865a5567d99d861c767d4823b64b534 --- /dev/null +++ b/processed_dataset/calculation/0164.json @@ -0,0 +1,8 @@ +{ + "source_file": "./raw_volume-zh/volume11/chapter15.tex", + "problem_type": "calculation", + "problem": "例3. 求最大正整数 $n$, 使存在 $n$ 个不同实数 $x_1, x_2, \\cdots, x_n$ 满足: 对任意 $1 \\leqslant i\\frac{n-1}{n} \\pi=\\pi-\\frac{\\pi}{n}$ 时, 由 $\\theta_n-\\theta_1<\\pi$ 可得 $\\tan ^2\\left(\\theta_n-\\theta_1\\right)< \\tan ^2 \\frac{\\pi}{n}$; 当 $\\theta_n-\\theta_1 \\leqslant \\frac{n-1}{n} \\pi$ 时, 由 $\\theta_n-\\theta_1=\\left(\\theta_n-\\theta_{n-1}\\right)+\\left(\\theta_{n-1}-\\theta_{n-2}\\right)+\\cdots+\\left(\\theta_2-\\right. \\left.\\theta_1\\right) \\leqslant \\frac{n-1}{n} \\pi$ 知, 存在 $i(1 \\leqslant i \\leqslant n-1)$ 使 $0<\\theta_{i+1}-\\theta_i \\leqslant \\frac{\\pi}{n}$, 从而有 $\\tan ^2\\left(\\theta_{i+1}-\\theta_i\\right) \\leqslant \\tan ^2 \\frac{\\pi}{n}$. 由上可知, 总存在 $1 \\leqslant i1-\\left(\\frac{1}{10}\\right)^2=0.99 .\n\\end{aligned}\n$$\n即当 $n \\geqslant 32$ 时, 对任意 $n$ 个实数 $x_1, x_2, \\cdots, x_n$, 其中必存在两个实数 $x_i, x_j$ 使\n$$\n\\left(1+x_i x_j\\right)^2>0.99\\left(1+x_i^2\\right)\\left(1+x_j^2\\right),\n$$\n故 $n \\geqslant 32$ 时, 不存在 $n$ 个实数 $x_1, x_2, \\cdots, x_n$ 满足题目条件.\n另一方面, 取 31 个实数 $x_i=\\tan (i \\theta)(i=1,2, \\cdots, 31)$, 其中 $\\theta= \\arctan \\frac{1}{\\sqrt{99}}$, 则 $\\tan \\theta=\\frac{1}{\\sqrt{99}}=\\frac{\\sqrt{9} \\overline{9}}{99}<\\frac{10}{99}<\\frac{\\pi}{31}$, 故 $0<\\theta<\\frac{\\pi}{31}$, 所以, 当 $1 \\leqslant ib_2>b_3$. 并且对任意 $1 \\leqslant i), 存在含 6 个点的集合 $A=\\left\\{A_1, A_2, A_3, A_4, A_5, A_6\\right\\}$ 满足题目条件,其中 $A_1, A_4$ 为红色, $A_2, A_5$ 为蓝色, $A_3, A_6$ 为黄色.\n其次,如果存在点数不少于 7 的点集 $A$ 满足题目的条件, 那么由抽㞕原理知其中必有 $\\left[\\frac{7-1}{3}\\right]+1=3$ 个点同色, 并且它们不共线.\n(否则,不妨设 $A_1, A_2, A_3$ 同色且依次在一条直线上, 则由已知条件知 $A_1$ 与 $A_2$ 之间有异色点 $B, A_2$ 与 $A_3$ 之间有异色点 $C$, 于是 $A_1$ 与 $A_3$ 之间有 2 个异色点 $B$ 和 $C$, 这与已知条件矛盾!). 从而存在一个 3 个顶点同色的三角形.\n考察所有以 $A$ 中点为顶点且 3 个顶点同色的三角形, 因其个数有限, 故其中必有一个 $\\triangle A_1 A_2 A_3$, 它的 3 个顶点同色 (不妨设为红色, 用\"・\"表示) 且它的面积最小.\n由已知条件知 $\\triangle A_1 A_2 A_3$ 的每条边上必有一个不同于红色的点, 若这 3 点同色, 则以它们为顶点的三角形面积小于 $\\triangle A_1 A_2 A_3$ 的面积, 这与假设矛盾.\n故这 3 点不能全同色, 不如设 $B_1, B_2$ 为蓝色 (用\".\"表示), $B_3$ 为黄色 (用 \"×\" 表示), 如图() 所示.\n若线段 $B_1 B_2$ 上的异色点 $T$ 为红色, 则 $S_{\\triangle T A_2 A_3}b$, 有 $b1+\\frac{r}{m_0-r},\n$$\n故当 $m_0=n r+r$ 时, 有 $\\frac{a}{b}>1+\\frac{1}{n}$, 即 $a>\\frac{n+1}{n} b$, 不满足题目要求.\n另一方面, 若 $m \\geqslant m_0=n r+r$, 将 $S$ 任意剖分为 $r$ 个两两不相交的集合 $A_1, A_2, \\cdots, A_r$ 之后, 取 $S$ 中 $r+1$ 个数 $n r, n r+1, n r+2, \\cdots, n r+r$, 则由抽庶原理知其中必有 2 个数 $a, b(a>b)$ 属于同一个子集 $A_i(1 \\leqslant i \\leqslant r)$, 且 $a-b \\leqslant r, b \\geqslant n r$, 于是\n$$\n1<\\frac{a}{b}=1+\\frac{a-b}{b} \\leqslant 1+\\frac{r}{n r}=1+\\frac{1}{n},\n$$\n即 $b0$, 故有 $b_{n+1}=\\frac{1}{2} b_n+\\frac{3}{2}$, 即\n$$\nb_{n+1}-3=\\frac{1}{2}\\left(b_n-3\\right),\n$$\n又 $b_1=\\sqrt{1+24 a_1}=5$, 由等比数列通项公式得\n$$\nb_n-3=\\left(b_1-3\\right)\\left(\\frac{1}{2}\\right)^{n-1}=2^{2-n} \\text {. }\n$$\n所以\n$$\n\\begin{aligned}\na_n & =\\frac{1}{24}\\left(b_n^2-1\\right)=\\frac{1}{24}\\left[\\left(2^{2-n}+3\\right)^2-1\\right] \\\\\n& =\\frac{1}{3}\\left(2^{1-2 n}+3 \\cdot 2^{-n}+1\\right) .\n\\end{aligned}\n$$", + "remark": "注:本例中由 $b_{n+1}=\\frac{1}{2} b_n+\\frac{3}{2}$ 变为 $b_{n+1}-3=\\frac{1}{2}\\left(b_n-3\\right)$ 时, 其中 3 是方程 $x=\\frac{1}{2} x+\\frac{3}{2}$ 的根.\n也称为函数 $f(x)=\\frac{1}{2} x+\\frac{3}{2}$ 的不动点.\n一般说来, 对于一阶常系数线性递推数列 $b_{n+1}=p b_n+q(p, q$ 为常数, $p \\neq 1)$, 通过解方程 $x=p x+q$, 解出 $x=-\\frac{q}{p-1}$, 便可化为 $b_{n+1}+\\frac{q}{p-1}=p\\left(b_n+\\frac{q}{p-1}\\right)$.", + "figures": [] +} \ No newline at end of file diff --git a/processed_dataset/calculation/0175.json b/processed_dataset/calculation/0175.json new file mode 100644 index 0000000000000000000000000000000000000000..683bf265aef997e6dc82bfe0489bb18c8a5d558d --- /dev/null +++ b/processed_dataset/calculation/0175.json @@ -0,0 +1,8 @@ +{ + "source_file": "./raw_volume-zh/volume11/chapter4.tex", + "problem_type": "calculation", + "problem": "例6. (斐波那契数列) 假设开始时有雌雄各一的一对小兔, 一个月后长成大兔, 再一个月后生了一对雌雄各一的小兔, 而这对小兔经过一个月后就长成大兔, 此后, 每对大兔每月生一对雌雄各一的小兔, 每对小兔经过一个月又长成大兔,问经过 $n$ 个月一共有多少对兔?", + "solution": "解:设经过 $n$ 个月有小兔 $a_n$ 对, 大兔 $b_n$ 对, 大、小兔共 $F_n$ 对.\n于是, $b_n$ 由两部分组成, 第一部分是上月已有的大兔数, 即 $b_{n-1}$, 另一部分是上月的小兔长成的大兔数, 即 $a_{n-1}$, 故得 $\\quad b_n=b_{n-1}+a_{n-1}=F_{n-1}, \\label{eq1}$, \n而 $a_n$ 是上一月大兔生出来的,故应等于 $b_{n-1}$. 于是有 $a_n=b_{n-1}, \\label{eq2}$, \n由式\\ref{eq1}及\\ref{eq2}可得\n$$\nF_n=a_n+b_n=b_{n-1}+b_n=F_{n-2}+F_{n-1}, \\label{eq3}\n$$\n并且显然 $F_0=1$ (开始时只有一对小兔), $F_1=1$ (经过一个月后只有一对大兔). 式\\ref{eq3}的特征方程是 $r^2=r+1$. 特征根为 $r_1=\\frac{1+\\sqrt{5}}{2}, r_2=\\frac{1-\\sqrt{5}}{2}$, 故\n$$\n\\begin{array}{r}\nF_n=c_1\\left(\\frac{1+\\sqrt{5}}{2}\\right)^n+c_2\\left(\\frac{1-\\sqrt{5}}{2}\\right)^n, \\\\\n\\text { 由 } F_0=1, F_1=1 \\text { 得 }\\left\\{\\begin{array}{l}\nc_1+c_2=1, \\\\\nc_1\\left(\\frac{1+\\sqrt{5}}{2}\\right)+c_2\\left(\\frac{1-\\sqrt{5}}{2}\\right)=1 .\n\\end{array}\\right.\n\\end{array}\n$$\n解得 $c_1=\\frac{1+\\sqrt{5}}{2 \\sqrt{5}}, c_2=-\\frac{1-\\sqrt{5}}{2 \\sqrt{5}}$, 故得\n$$\nF_n=\\frac{1}{\\sqrt{5}}\\left[\\left(\\frac{1+\\sqrt{5}}{2}\\right)^{n+1}-\\left(\\frac{1-\\sqrt{5}}{2}\\right)^{n+1}\\right], n=0,1,2, \\cdots\n$$\n满足 $F_n=F_{n-1}+F_{n-2}(n=2,3, \\cdots), F_0=F_1=1$ 的数列, 叫做斐波那契 (Fibonacci) 数列.\n它的前面 10 项是\n$$\n1,1,2,3,5,8,13,21,34,55, \\cdots\n$$\n许多组合计数问题和数学竞赛试题都与斐波那契数列有关.", + "remark": "", + "figures": [] +} \ No newline at end of file diff --git a/processed_dataset/calculation/0176.json b/processed_dataset/calculation/0176.json new file mode 100644 index 0000000000000000000000000000000000000000..36a6684a5f1ec0aacaaf1582ea6da3b64374f50a --- /dev/null +++ b/processed_dataset/calculation/0176.json @@ -0,0 +1,8 @@ +{ + "source_file": "./raw_volume-zh/volume11/chapter4.tex", + "problem_type": "calculation", + "problem": "例7. 用 $1,2,3$ 组成 $n$ 位数, 如果要求没有 2 个 1 相邻, 问这样的 $n$ 位数共有多少个?", + "solution": "解:用 $a_n$ 表示所求 $n$ 位数的个数,显然, $a_1=3, a_2=8$. (因满足要求的 2 位数只有 $12,13,21,22,23,31,32,33$ 共 8 个). 当 $n \\geqslant 3$ 时, 如果第一个数字为 1 , 那么第 2 个数字只能是 2 或 3 , 余下的数字有 $a_{n-2}$ 种不同的取法; 如果第 1 位数字是 2 或 3 , 那么余下的数字有 $a_{n-1}$ 种不同的取法, 由加法法则和乘法法则得 $a_n=2 a_{n-1}+2 a_{n-2}(n \\geqslant 3)$. 特征方程为 $r^2-2 r-2=0$, 特征根为 $r_1=1+\\sqrt{3}, r_2=1-\\sqrt{3}$, 故得\n$$\na_n=c_1(1+\\sqrt{3})^n+c_2(1-\\sqrt{3})^n,\n$$\n补充定义 $a_0$ 满足 $a_2=2 a_1+2 a_0$, 即 $a_0=\\frac{1}{2}\\left(a_2-2 a_1\\right)=1$. 由 $a_0=1, a_1=$ 3 , 得\n$$\n\\left\\{\\begin{array}{l}\nc_1+c_2=1, \\\\\nc_1(1+\\sqrt{3})+c_2(1-\\sqrt{3})=3 .\n\\end{array}\\right.\n$$\n解得 $c_1=\\frac{2+\\sqrt{3}}{2 \\sqrt{3}}=\\frac{(1+\\sqrt{3})^2}{4 \\sqrt{3}}, c_2=-\\frac{2-\\sqrt{3}}{2 \\sqrt{3}}=-\\frac{(1-\\sqrt{3})^2}{4 \\sqrt{3}}$, 所以\n$$\na_n=\\frac{1}{4 \\sqrt{3}}\\left[(1+\\sqrt{3})^{n+2}-(1-\\sqrt{3})^{n+2}\\right] \\text {. }\n$$", + "remark": "", + "figures": [] +} \ No newline at end of file diff --git a/processed_dataset/calculation/0177.json b/processed_dataset/calculation/0177.json new file mode 100644 index 0000000000000000000000000000000000000000..4b147a76adaa2f1c56bc886b929594aeba3e0916 --- /dev/null +++ b/processed_dataset/calculation/0177.json @@ -0,0 +1,8 @@ +{ + "source_file": "./raw_volume-zh/volume11/chapter4.tex", + "problem_type": "calculation", + "problem": "例10. 试确定 $(\\sqrt{2}+\\sqrt{3})^{2004}$ 小数点前一位数字和小数点后一位数字.", + "solution": "解:记 $N=(\\sqrt{2}+\\sqrt{3})^{2004}=(5+2 \\sqrt{6})^{1002}$, 令 $x_n=(5+2 \\sqrt{6})^n+(5- 2 \\sqrt{6})^n$, 则数列 $\\left\\{x_n\\right\\}$ 对应的特征方程是\n$$\n[r-(5+2 \\sqrt{6})][r-(5-2 \\sqrt{6})]=0,\n$$\n即 $r^2-10 r+1=0$, 所以数列 $\\left\\{x_n\\right\\}$ 满足的递推关系是\n$$\nx_n=10 x_{n-1}-x_{n-2}(n \\geqslant 3), \\label{eq1}\n$$\n其中 $x_1=(5+2 \\sqrt{6})+(5-2 \\sqrt{6})=10, x_2=(5+2 \\sqrt{6})^2+(5-2 \\sqrt{6})^2=$ 98 皆为整数.\n若 $x_{n-2}, x_{n-1}$ 为整数,则由 式\\ref{eq1}知 $x_n$ 也为整数,故对一切 $n \\in \\mathbf{N}_{+}$, $x_n$ 为整数, 且由 式\\ref{eq1}得\n$$\nx_n=10 x_{n-1}-\\left(10 x_{n-3}-x_{n-4}\\right)=10\\left(x_{n-1}-x_{n-3}\\right)+x_{n-4} .\n$$\n故 $x_n \\equiv x_{n-4}(\\bmod 10)$. 特别 $x_{1002} \\equiv x_2(\\bmod 10)$, 即知 $x_{1002}$ 的个位数字是 8 . 又因为 $0<5-2 \\sqrt{6}<0.2$, 于是\n$$\n0<(5-2 \\sqrt{6})^{1002}<0.2^{1002}=0.008^{334}<0.01^{334}=\\underbrace{0.00 \\cdots 0}_{668 \\uparrow 0} 1,\n$$\n即\n$$\nx_{1002}=N+(5-2 \\sqrt{6})^{1002}=N+0 . \\underbrace{00 \\cdots 0} * * * \\cdots 668 个 0\n$$\n因 $x_{1002}$ 的个位数字是 8 , 所以 $N$ 的小数点前一位数字是 7 ,小数点后一位数字是 9 .", + "remark": "", + "figures": [] +} \ No newline at end of file diff --git a/processed_dataset/calculation/0178.json b/processed_dataset/calculation/0178.json new file mode 100644 index 0000000000000000000000000000000000000000..f7588268662605c858d8f8747a0fb67aa9f7ea89 --- /dev/null +++ b/processed_dataset/calculation/0178.json @@ -0,0 +1,8 @@ +{ + "source_file": "./raw_volume-zh/volume11/chapter5.tex", + "problem_type": "calculation", + "problem": "例1. 将 9 个 1,9 个 2,9 个 $3, \\cdots, 9$ 个 1000 共 9000 个数填人一个 9 行、 1000 列的表格内 (每格内填人一个数), 使得同一列中任何两数之差的绝对值不超过 3. 设这个表格中每列中各数之和 (共 1000 个列和) 的最小值为 $M$, 试求 $M$ 的最大值.", + "solution": "解:我们依据 9 个 1 的分布的列数的不同情形来分别求列和的最小值 $M$.\n如果 9 个 1 分布在同一列,那么 $M=9$.\n如果 9 个 1 分布在两列中,那么这两列中各数之和不小于 $2 M$, 同时由已知条件知两列中出现的最大数至多只能为 4 , 故这两列数之和 $\\leqslant 9 \\times 1+9 \\times 4=45$, 即 $2 M \\leqslant 45$, 所以 $M \\leqslant 22$.\n如果 9 个 1 分布在三列中,那么同上讨论可得 $3 M \\leqslant 9 \\times 1+9 \\times 4+9 \\times 3=72$, 所以 $M \\leqslant 24$.\n如果 9 个 1 分布在四列中,那么类似可得 $4 M \\leqslant 9 \\times 1+9 \\times 4+9 \\times 3+ 9 \\times 2=90$, 所以 $M \\leqslant 22$.\n如果 9 个 1 分布的列数大于 4 , 那么其中某一列必有一个数大于 4 (因为\n$2,3,4$ 一共 27 个数, 不足以填满所有出现 1 的列), 这与已知条件中任何一列中任意两个数之差的绝对值不大于 3 矛盾, 所以这种情形不出现.\n综上可知, 列和的最小值 $M \\leqslant 24$.\n所以,所求列和的最小值的最大值为 24 .\n对于较复杂的问题, 有时一次分类还不够, 还要进行第二次分类, 两次分类可以互相独立,也可能第二次是将第一次的一个子类再进行分类.\n总之, 可以根据问题的需要多层次地进行分类.", + "remark": "", + "figures": [] +} \ No newline at end of file diff --git a/processed_dataset/calculation/0179.json b/processed_dataset/calculation/0179.json new file mode 100644 index 0000000000000000000000000000000000000000..4f1efdc3ea065e0dba28e25c3343f2325b2177a0 --- /dev/null +++ b/processed_dataset/calculation/0179.json @@ -0,0 +1,8 @@ +{ + "source_file": "./raw_volume-zh/volume11/chapter6.tex", + "problem_type": "calculation", + "problem": "例1. 设集合 $M=\\{1,2, \\cdots, 1000\\}$, 对 $M$ 的任意非空子集 $X$, 令 $\\alpha_X$ 表示 $X$ 中最大数与最小数之和, 那么所有 $\\alpha_X$ 的算术平均值为 ?", + "solution": "解:将 $M$ 的所有非空子集配对, 对 $M$ 的任意非空子集 $X$, 令\n$$\nX^{\\prime}=\\{1001-x \\mid x \\in X\\},\n$$\n则 $X^{\\prime}$ 也是 $M$ 的非空子集, 且当 $X \\neq X_1$ 时, $X^{\\prime} \\neq X_1{ }^{\\prime}$.\n于是, $M$ 的所有非空子集分为两类:\n(1) $X \\neq X^{\\prime}$;\n(2) $X=X^{\\prime}$.\n对于 (2) 类中的 $X$, 若 $x \\in X$, 则 $1001-x \\in X$, 当 $x_0$ 为 $X$ 中最小数时, $1001-x_0$ 为 $X$ 中最大数, 这时 $\\alpha_X=x_0+\\left(1001-x_0\\right)=1001$.\n对于(1) 中一对 $X$ 和 $X^{\\prime}$, 若 $X$ 中最小数和最大数分别为 $x_0, y_0$, 则 $X^{\\prime}$ 中的最大数和最小数分别为 $1001-x_0$ 和 $1001-y_0$, 这时\n$$\n\\alpha_X+\\alpha_{X^{\\prime}}=x_0+y_0+\\left(1001-x_0\\right)+\\left(1001-y_0\\right)=2002 .\n$$\n综上可得,所有 $\\alpha_X$ 的平均值等于 1001 .", + "remark": "", + "figures": [] +} \ No newline at end of file diff --git a/processed_dataset/calculation/0180.json b/processed_dataset/calculation/0180.json new file mode 100644 index 0000000000000000000000000000000000000000..a1964fc9b520087eeb0c9d25d106f49b05a609c2 --- /dev/null +++ b/processed_dataset/calculation/0180.json @@ -0,0 +1,8 @@ +{ + "source_file": "./raw_volume-zh/volume11/chapter6.tex", + "problem_type": "calculation", + "problem": "例4. 某班共 30 名学生, 每名学生在班的内部都有同样多的朋友, 期末考试后, 任何两人的成绩都可以分出优劣, 没有并列者.\n比自己的多半朋友的成绩都好的学生称之为好学生, 问好学生最多有几名?", + "solution": "解法一设每人有 $k$ 个朋友, 全班有 $x$ 个好学生.\n若学生 $a$ 比他的朋友 $b$ 的成绩好,则将 $a$ 与 $b$ 配成一对.\n设这种对子共有 $n$ 对.\n一方面, 最好的学生比他的 $k$ 个朋友的成绩都好, 可配成 $k$ 对, 其余 $x^{-1}$ 名好学生至少比他的 $\\left[\\frac{k}{2}\\right]+1 \\geqslant \\frac{k+1}{2}$ 个朋友的成绩都好, 每人至少配成 $\\frac{k+1}{2}$ 对,所以\n$$\nn \\geqslant k+(x-1)\\left(\\frac{k+1}{2}\\right) .\n$$\n另一方面, 30 名学生, 每人恰有 $k$ 个朋友, 至多共形成 $\\frac{30 k}{2}=15 k$ 对, 即 $n \\leqslant 15 k$, 所以\n$$\nk+(x-1)\\left(\\frac{k+1}{2}\\right) \\leqslant 15 k \\text {, 即 } x \\leqslant \\frac{28 k}{k+1}+1=29-\\frac{28}{k+1} . \\label{eq1}\n$$\n其次, 设 $c$ 是好学生中最差的 1 名, 于是 $c$ 至多比 $30-x$ 名学生的成绩要好, 从而 $c$ 至多生成 $30-x$ 对, 另一方面 $c$ 至少比他的 $\\left[\\frac{k}{2}\\right]+1 \\geqslant \\frac{k+1}{2}$ 名朋友的成绩好.\n故 $c$ 至少可生成 $\\frac{k+1}{2}$ 对, 所以\n$$\n30-x \\geqslant \\frac{k+1}{2} \\text {, 即 } k \\leqslant 59-2 x . \\label{eq2}\n$$\n式\\ref{eq2}代入\\ref{eq1}得 $x \\leqslant 29-\\frac{28}{60-2 x}=29-\\frac{14}{30-x}$, 即\n$$\nx^2-59 x+856 \\geqslant 0 \\text {, }\n$$\n解得 $x \\leqslant \\frac{59-\\sqrt{57}}{2}<26$ 或 $x \\geqslant \\frac{59+\\sqrt{57}}{2}>30$ (舍去), 所以 $x \\leqslant 25$.\n下面例子表明好学生可以是 25 人(由式\\ref{eq2}中等号成立知这时 $k=9$ )\n用 $1,2, \\cdots, 30$ 这 30 个号码分别表示第 1 名,第 2 名, $\\cdots$,第 30 名学生, 并将这些号码填人右侧 $6 \\times 5$ 的表格中.\n(1) 第 1 行每个学生的朋友是同行以及下一行不同列的其他 8 人以及第 6 行中同列的那个人 (例如 3 号学生的朋友的编号是 $1,2,4,5,6,7,9,10,28)$;\n(2) 第 2 行至第 5 行中每个学生的朋友是相邻\n\\begin{tabular}{|c|c|c|c|c|}\n\\hline 1 & 2 & 3 & 4 & 5 \\\\\n\\hline 6 & 7 & 8 & 9 & 10 \\\\\n\\hline 11 & 12 & 13 & 14 & 15 \\\\\n\\hline 16 & 17 & 18 & 19 & 20 \\\\\n\\hline 21 & 22 & 23 & 24 & 25 \\\\\n\\hline 26 & 27 & 28 & 29 & 30 \\\\\n\\hline\n\\end{tabular}\n上、下 2 行中与他不同列的其他 8 人以及第 6 行中与他同列的那个人(例如 17 号学生的朋友的编号是 $11,13,14,15,21,23,24,25,27)$;\n(3) 第 6 行每个学生的朋友是上一行以及同列的其他 9 人 (例如 29 号学生的朋友的编号是 $21,22,23,24,25,4,9,14,19)$.\n于是, 每人恰有 9 位朋友,并且编号 1 至 25 的学生都是好学生.", + "remark": "", + "figures": [] +} \ No newline at end of file diff --git a/processed_dataset/calculation/0181.json b/processed_dataset/calculation/0181.json new file mode 100644 index 0000000000000000000000000000000000000000..9aa695b864d11b7abcac5c55da701027dd5f0e12 --- /dev/null +++ b/processed_dataset/calculation/0181.json @@ -0,0 +1,8 @@ +{ + "source_file": "./raw_volume-zh/volume11/chapter6.tex", + "problem_type": "calculation", + "problem": "例4. 某班共 30 名学生, 每名学生在班的内部都有同样多的朋友, 期末考试后, 任何两人的成绩都可以分出优劣, 没有并列者.\n比自己的多半朋友的成绩都好的学生称之为好学生, 问好学生最多有几名?", + "solution": "解法二若每位学生有 $2 k-1$ 个朋友, 那么 30 人中所有朋友对的数目为 $\\frac{1}{2} \\times 30 \\times(2 k-1)=30 k-15$. 每对朋友中给成绩较优的发一张奖状, 这样共发出了 $30 k-15$ 张奖状.\n第 1 名总是拿 $2 k-1$ 张奖状,第 2 名至少拿 $2 k-2$ 张奖状, $\\cdots$,第 $k$ 名至少拿 $k$ 张奖状,于是这 $k$ 名好学生至少一共拿了 $(2 k-1)+(2 k- 2)+\\cdots+k=\\frac{1}{2} k(3 k-1)$ 张奖状, 至多还剩 $30 k-15-\\frac{1}{2} k(3 k-1)$ 张奖状, 因为每个好学生至少得 $k$ 张奖状, 故至多还有 $\\frac{1}{k}\\left[30 k-15-\\frac{1}{2} k(3 k-1)\\right]$ 个好学生.\n所以,好学生的人数 $n$ 至多为\n$$\n\\begin{gathered}\nk+\\frac{1}{k}\\left[30 k-15-\\frac{1}{2} k(3 k-1)\\right]=30 \\frac{1}{2}-\\left(\\frac{15}{k}+\\frac{k}{2}\\right) \\\\\n\\leqslant 30 \\frac{1}{2}-2 \\sqrt{\\frac{15}{k} \\times \\frac{k}{2}}=30 \\frac{1}{2}-\\sqrt{30}<30 \\frac{1}{2}-5=25 \\frac{1}{2},\n\\end{gathered}\n$$\n因此, $n \\leqslant 25$.\n若每位学生有 $2 k$ 个朋友, 则完全类似地可发出 $30 k$ 张奖状, 至少得 $k+1$ 张奖状的为好学生.\n第 1 名至第 $k$ 名至少共得了 $2 k+(2 k-1)+\\cdots+(k+ 1)=\\frac{1}{2} k(3 k+1)$ 张奖状, 故好学生人数 $n$ 至多为\n$$\n\\begin{aligned}\n& k+\\frac{1}{k+1}\\left[30 k-\\frac{1}{2} k(3 k+1)\\right] \\\\\n= & \\frac{1}{k+1}\\left[30 k-\\frac{1}{2} k(k-1)\\right] \\\\\n= & \\frac{1}{k+1}\\left[30(k+1)-30-\\frac{1}{2} k(k-1)\\right]=30-\\left(\\frac{31}{k+1}+\\frac{k-2}{2}\\right) \\\\\n= & 31 \\frac{1}{2}-\\left(\\frac{31}{k+1}+\\frac{k+1}{2}\\right) \\leqslant 31 \\frac{1}{2}-2 \\sqrt{\\frac{31}{k+1} \\cdot \\frac{k+1}{2}} \\\\\n= & 31 \\frac{1}{2}-\\sqrt{62}<25 .\n\\end{aligned}\n$$\n综合两种情形知,好学生至多有 25 人,下同解法一.", + "remark": "", + "figures": [] +} \ No newline at end of file diff --git a/processed_dataset/calculation/0182.json b/processed_dataset/calculation/0182.json new file mode 100644 index 0000000000000000000000000000000000000000..eb23d4489091747dd7a134504dd3b72f744eac44 --- /dev/null +++ b/processed_dataset/calculation/0182.json @@ -0,0 +1,10 @@ +{ + "source_file": "./raw_volume-zh/volume11/chapter6.tex", + "problem_type": "calculation", + "problem": "例9. 设凸 $n$ 边形的任意 3 条对角线不交于形内同一点,求它的对角线在形内的交点的个数.", + "solution": "解:依题意,一个交点 $P$ 由两条对角线 $l$ 和 $m$ 相交而得, 反之,两条相交对角线 $l$ 和 $m$, 确定一个交点 $P$, 从而 $P$ 与 $(l, m)$ 可建立一一对应.\n又因两条相交对角线 $l, m$ 分别由凸 $n$ 边形中两对顶点 $A 、 B$ 和 $C 、 D$ 连接而成, 而且对于凸 $n$ 边形的任意 4 个顶点,有且只有一对对角线相交于多边形内 (如图()), 故 $(l, m)$ 又可与 4 顶点组 $(A, C, B, D)$ 建立一一对应, 即有 $P \\leftrightarrow(l, m) \\leftrightarrow(A, C, B, D)$. 因此, 形内对角线的交点总数等于凸 $n$ 边形的 4 顶点组数 $\\mathrm{C}_n^4$.", + "remark": "注:本题中结论是组合几何中一个重要结论, 今后可用它去解决组合几何中较为复杂的计数问题.", + "figures": [ + "./images/volume11/figures/fig-c6i3.png" + ] +} \ No newline at end of file diff --git a/processed_dataset/calculation/0183.json b/processed_dataset/calculation/0183.json new file mode 100644 index 0000000000000000000000000000000000000000..d82cf7eb15d10c89a54c3b9a6d27afe8ed5e4a35 --- /dev/null +++ b/processed_dataset/calculation/0183.json @@ -0,0 +1,10 @@ +{ + "source_file": "./raw_volume-zh/volume11/chapter6.tex", + "problem_type": "calculation", + "problem": "例10. 把正三角形 $A B C$ 各边 $n$ 等分, 过各分点在三角形内作边的平行线段将 $\\triangle A B C$ 完全分割成边长为 $\\frac{1}{n} B C$ 的小正三角形.\n求其中边长为 $\\frac{1}{n} B C$ 的小菱形个数.", + "solution": "如图(),解首先考虑边不平行 $B C$ 的小菱形,延长每个菱形的边顺次与 $B C$ 相交于 4 个分点(特殊情形下,第 2 个交点与第 3 个交点重合于菱形的一个顶点) 为了便于处理, 可延长 $A B$ 到 $B^{\\prime}$ 使 $B B^{\\prime}= \\frac{1}{n} A B$, 延长 $A C$ 到 $C^{\\prime}$ 使 $C C^{\\prime}=\\frac{1}{n} A C$, 并延长各平行线交线段 $B^{\\prime} C^{\\prime}$ 于 $n+2$ 个等分点, 记为 0,1 , $2, \\cdots, n+1$ (包括 $B^{\\prime}, C^{\\prime}$ 两个端点), 于是每边不平行 $B C$ 的小菱形的两组对边延长后交 $B^{\\prime} C^{\\prime}$ 于 4 个不同分点 $i, i+1, k, k+1$. 反之,任给这样 4 个分点必对应一个边不平行 $B C$ 的小菱形, 二者具有一一对应关系.\n由于有序数组 $(i, i+1, k, k+1)(0 \\leqslant i0, x_1 \\cdot x_2=S(B)>0$, 从而 $0) 圆周上任取 4 点 $A_1, A_2, A_3, A_4$ 两两相连的线段,确定了 4 个第二类三角形: $\\triangle A_1 O A_2, \\triangle A_2 O A_3, \\triangle A_3 O A_4, \\triangle A_4 O A_1$, 反之每 4 个这样有公共内顶点的第二类三角形对应了圆周上的一个 4 点组, 于是 $I_2=4 \\mathrm{C}_n^4$.\n类似地, 如图(), 圆周上任取 5 点 $A_1, A_2, A_3, A_4, A_5$, 两两连一线段,确定了 5 个第三类三角形: $\\triangle A_1 B_1 B_2, \\triangle A_2 B_2 B_3, \\triangle A_3 B_3 B_4, \\triangle A_4 B_4 B_5$, $\\triangle A_5 B_5 B_1$, 于是可得 $I_3=5 \\mathrm{C}_n^5$.\n最后, 如图(), 圆周上任取 6 点 $A_1, A_2, A_3, A_4, A_5, A_6$ 对应于 1 个第四类三角形,所以 $I_4=\\mathrm{C}_n^6$.\n综上所述,得所确定的三角形共有 $\\mathrm{C}_n^3+4 \\mathrm{C}_n^4+5 \\mathrm{C}_n^5+\\mathrm{C}_n^6$ 个.", + "remark": "", + "figures": [ + "./images/volume11/figures/fig-c6i5-1.png", + "./images/volume11/figures/fig-c6i5-2.png", + "./images/volume11/figures/fig-c6i5-3.png" + ] +} \ No newline at end of file diff --git a/processed_dataset/calculation/0186.json b/processed_dataset/calculation/0186.json new file mode 100644 index 0000000000000000000000000000000000000000..f343d0e6d353f59c75f3b0293e64482a7cf50154 --- /dev/null +++ b/processed_dataset/calculation/0186.json @@ -0,0 +1,8 @@ +{ + "source_file": "./raw_volume-zh/volume11/chapter7.tex", + "problem_type": "calculation", + "problem": "例1. 一张正方形纸片内有 1000 个点, 这些点及正方形的顶点中任意 3 点不共线,然后在这些点及正方形顶点之间连一些线段, 将正方形全部分成小三角形 (以所连线段及正方形的边为边, 且所连线段除端点外, 两两无公共点), 问一共连有多少条线段? 一共得到多少个三角形?", + "solution": "解:设一共连有 $l$ 条线段,一共得到 $k$ 个三角形.\n一方面, 所得 $k$ 个三角形的内角总和为 $k \\cdot 180^{\\circ}$, 另一方面, 所得 $k$ 个三角形中, 以 1000 个内点为顶点的所有内角之和为 $1000 \\times 360^{\\circ}$, 以正方形的顶点为顶点的所有内角之和为 $4 \\times 90^{\\circ}$, 所以\n$$\nk \\cdot 180^{\\circ}=1000 \\times 360^{\\circ}+4 \\times 90^{\\circ},\n$$\n解得 $k=2002$.\n其次,每个三角形有 3 条边, $k$ 个三角形一共有 $3 k$ 条边, 另一方面,所连每条线段是两个三角形的公共边, 而正方形的每条边都是一个三角形的一边, 于是 $3 k=2 l+4$, 解得 $l=\\frac{3}{2} k-2=3001$.", + "remark": "", + "figures": [] +} \ No newline at end of file diff --git a/processed_dataset/calculation/0187.json b/processed_dataset/calculation/0187.json new file mode 100644 index 0000000000000000000000000000000000000000..603edd1cd7414e4d7442eae2fb4468eb28ab0586 --- /dev/null +++ b/processed_dataset/calculation/0187.json @@ -0,0 +1,8 @@ +{ + "source_file": "./raw_volume-zh/volume11/chapter7.tex", + "problem_type": "calculation", + "problem": "例2. 8 位歌手参加艺术节, 准备为他们安排 $m$ 次演出, 每次由其中 4 位登台表演, 并且 8 位歌手中任意两位同台演出的次数一样多, 请设计一种方案, 使他们一共演出的次数 $m$ 最少.", + "solution": "解:设任意一对歌手同台演出的次数都为 $r$. 若某对歌手 $a_i, a_j$ 同在第 $k$ 场演出, 则将 $\\left(a_i, a_j, k\\right)$ 组成一个三元组, 设这种三元组共有 $S$ 个, 一方面, 因为 8 位歌手可形成 $\\mathrm{C}_8^2=28$ 对,每对歌手同台演出 $r$ 次,所以 $S=28 r$.\n另一方面, 每场有 4 位歌手参加, 可形成 $\\mathrm{C}_4^2$ 个三元组, $m$ 场演出一共可形成 $m \\mathrm{C}_4^2$ 个三元组,所以 $S=m \\mathrm{C}_4^2=6 m$. 于是 $6 m=28 r, 3 m=14 r$.\n因为 $(3,14)=1$, 故 $14 \\mid m$, 从而 $m \\geqslant 14$.\n下面实例说明 $m=14$ (从而 $r=3$ ) 是可以实现的(数字 1 至 8 代表 8 位歌手,每个括号内的 4 个数字代表同台演出的 4 位歌手):\n$$\n\\begin{gathered}\n\\{1,2,3,4\\},\\{1,2,5,6\\},\\{1,2,7,8\\},\\{1,3,5,7\\},\\{1,3,6,8\\}, \\\\\n\\{1,4,5,8\\},\\{1,4,6,7\\},\\{2,3,5,8\\},\\{2,3,6,7\\},\\{2,4,5,7\\}, \\\\\n\\{2,4,6,8\\},\\{3,4,5,6\\},\\{3,4,7,8\\},\\{5,6,7,8\\} .\n\\end{gathered}\n$$\n综上可知,所求 $m$ 的最小值为 14 .", + "remark": "注:进一步,我们可以求出当 $m$ 取最小值时 (必有 $r=3$ ), 每位歌手参加演出的场数.\n设第 $i$ 位歌手 $a_i$ 参加了 $n_i$ 场演出 $(i=1,2, \\cdots, 8)$. 考虑含 $a_1$ 的三元组 $\\left(a_1, a_i, k\\right)(2 \\leqslant i \\leqslant 8)$ 的个数 $S_1$, 一方面对任意 $a_i(2 \\leqslant i \\leqslant 8), a_1$ 与 $a_i$ 同台演出了 $r=3$ 场, 故含 $a_1, a_i$ 的三元组有 3 个, 又 $a_i$ 有 7 种取法, 所以 $S_1=3 \\times 7=21$. 另一方面 $a_1$ 共参加 $n_1$ 场演出, 每场演出时, $a_1$ 与其他 3 位歌手同台,故 $S_1=3 n_1$. 于是 $3 n_1=21$, 由此得 $n_1=7$. 同理可得 $n_2=n_3= \\cdots=n_8=7$. 即每位歌手都参加了 7 场演出.", + "figures": [] +} \ No newline at end of file diff --git a/processed_dataset/calculation/0188.json b/processed_dataset/calculation/0188.json new file mode 100644 index 0000000000000000000000000000000000000000..937007db67858dbcf49edea49f935566261e5f7d --- /dev/null +++ b/processed_dataset/calculation/0188.json @@ -0,0 +1,8 @@ +{ + "source_file": "./raw_volume-zh/volume11/chapter8.tex", + "problem_type": "calculation", + "problem": "例1. 整数 $1,2, \\cdots, n$ 的排列满足: 每个数或者大于它前面的所有的数, 或者小于它前面的所有的数, 试问有多少个这样的排列?", + "solution": "解:记所求排列的个数为 $a_n$.\n$n=1$ 时, 只有数 1 , 显然 $a_1=1 . n=2$ 时, 显然只有 2 个排列 1,2 和 2 , 1 , 所以 $a_2=2$.\n对于 $n>2$, 考虑 $n$ 在第 $i$ 个位置的排列, 这时 $n$ 之后的 $n-i$ 个数的位置唯一确定, 只能是 $n-i, n-i-1, \\cdots, 2,1$, 而它前面的 $i-1$ 个数有 $a_{i-1}$ 种排法 (约定 $a_0=1$ ), 又 $i=1,2, \\cdots, n$, 故必有\n$$\na_n=1+a_1+\\cdots+a_{n-1},\n$$\n从而\n$$\na_{n-1}=1+a_1+\\cdots+a_{n-2}(n \\geqslant 3),\n$$\n两式相减得 $a_n-a_{n-1}=a_{n-1}$, 即 $a_n=2 a_{n-1}(n \\geqslant 3)$, 所以 $a_n=a_2 \\cdot 2^{n-2}=2^{n-1}$, 经检验 $a_1=1, a_2=2$ 满足 $a_n=2^{n-1}$, 所以 $a_n=2^{n-1}(n \\geqslant 1)$, 即所求的排列个数为 $2^{n-1}$.", + "remark": "", + "figures": [] +} \ No newline at end of file diff --git a/processed_dataset/calculation/0189.json b/processed_dataset/calculation/0189.json new file mode 100644 index 0000000000000000000000000000000000000000..015a8c4db2c6e34e8399f4c50e61e58ba9a353f9 --- /dev/null +++ b/processed_dataset/calculation/0189.json @@ -0,0 +1,8 @@ +{ + "source_file": "./raw_volume-zh/volume11/chapter8.tex", + "problem_type": "calculation", + "problem": "例3. 一个由若干行数字组成的数表, 从第二行起每行中的数字均等于其肩上的两个数的和, 最后一行仅一个数.\n第一行是前 100 个正整数按从小到大排成的行, 则最后一行的数是 ?(可以用指数表示).", + "solution": "解:易知:\n(1) 该数表共有 100 行;\n(2)每一行构成一个等差数列,且公差依次是:\n$$\nd_1=1, d_2=2, d_3=2^2, \\cdots, d_{99}=2^{98} ;\n$$\n(3)设第 $n$ 行中第一个数为 $a_n$, 则 $a_{100}$ 即为所求.\n依题意\n$$\na_n=a_{n-1}+\\left(a_{n-1}+2^{n-2}\\right)=2 a_n+2^{n-2},\n$$\n即 $\\frac{a_n}{2^n}=\\frac{a_{n-1}}{2^{n-1}}+\\frac{1}{4}$. 于是 $\\left\\{\\frac{a_n}{2^n}\\right\\}$ 是首项为 $\\frac{a_1}{2}=\\frac{1}{2}$, 公差为 $\\frac{1}{4}$ 的等差数列, 所以\n$$\n\\frac{a_n}{2^n}=\\frac{1}{2}+\\frac{1}{4}(n-1)=\\frac{1}{4}(n+1),\n$$\n故 $a_n=(n+1) \\cdot 2^{n-2}$, 特别, $a_{100}=101 \\times 2^{98}$. 也就是表中最后一行的数是 $101 \\times 2^{98}$.", + "remark": "", + "figures": [] +} \ No newline at end of file diff --git a/processed_dataset/calculation/0190.json b/processed_dataset/calculation/0190.json new file mode 100644 index 0000000000000000000000000000000000000000..745c02a003aab82459d54f8dcdb6a249818c6274 --- /dev/null +++ b/processed_dataset/calculation/0190.json @@ -0,0 +1,8 @@ +{ + "source_file": "./raw_volume-zh/volume11/chapter8.tex", + "problem_type": "calculation", + "problem": "例4. 一种密码锁的密码设置是在正 $n$ 边形 $A_1 A_2 \\cdots A_n$ 的每个顶点处赋值 0 和 1 两个数中的一个, 同时, 在每个顶点处染红、蓝两种颜色之一, 使得相邻两个顶点的数字和颜色至少有一个相同,问该种密码锁共有多少种不同的密码设置?", + "solution": "解:设共有 $a_n$ 种不同的密码设置, 且不妨设一个点为正 1 边形,一条线段为正 2 边形,于是 $a_1=4, a_2=4 \\times 3=12$.\n依次将 $A_1, A_2, \\cdots, A_n$ 赋值和染色,使得 $A_{i+1}$ 与 $A_i$ 的数字与颜色中至少有一种相同, 于是 $A_1$ 有 4 种设置, 当 $A_i$ 取定后, $A_{i+1}$ 有 3 种设置 $(i=1$, $2, \\cdots, n-1)$, 故共有 $4 \\times 3^{n-1}$ 种设置.\n设其中 $A_n$ 与 $A_1$ 的数字与颜色都不相同的有 $b_n$ 种, $A_n$ 与 $A_1$ 的数字与颜色完全相同的有 $c_n$ 种, $A_n$ 与 $A_1$ 的数字与颜色恰有一种相同的有 $d_n$ 种, 则\n$$\n\\begin{gathered}\na_n+b_n=4 \\times 3^{n-1}, \\label{eq1}\\\\\nc_n=a_{n-1}, \\label{eq2}\\\\\na_n=c_n+d_n, \\label{eq4}\\\\\nb_n=b_{n-1}+d_{n-1},\\label{eq4}\n\\end{gathered}\n$$\n由式\\ref{eq1}得\n$$\na_{n-1}+b_{n-1}=4 \\times 3^{n-2} . \\label{eq5}\n$$\n式\\ref{eq1}-\\ref{eq5}得\n$$\n\\left(a_n-a_{n-1}\\right)+\\left(b_n-b_{n-1}\\right)=4 \\times\\left(3^{n-1}-3^{n-2}\\right)=8 \\times 3^{n-2} . \\label{eq6}\n$$\n而由式\\ref{eq4}, \\ref{eq3}, 式\\ref{eq2}有\n$$\nb_n-b_{n-1}=d_{n-1}=a_{n-1}-c_{n-1}=a_{n-1}-a_{n-2}, \\label{eq7}\n$$\n式\\ref{eq7}代入\\ref{eq6}得\n$$\n\\left(a_n-a_{n-1}\\right)+\\left(a_{n-1}-a_{n-2}\\right)=8 \\times 3^{n-2},\n$$\n所以\n$$\na_n-a_{n-2}=8 \\times 3^{n-2}(n \\geqslant 3) .\n$$\n于是, 当 $n=2 k$ 时,\n$$\n\\begin{aligned}\na_n=a_{2 k} & =a_2+\\left(a_4-a_2\\right)+\\left(a_6-a_4\\right)+\\cdots+\\left(a_{2 k}-a_{2 k-2}\\right) \\\\\n& =12+8 \\times 3^2+8 \\times 3^4+\\cdots+8 \\times 3^{2 k-2} \\\\\n& =12+\\frac{8 \\times 3^2 \\times\\left(3^{2 k-2}-1\\right)}{3^2-1}=12+3^2 \\times\\left(3^{2 k-2}-1\\right) \\\\\n& =3^{2 k}+3=3^n+2+(-1)^n .\n\\end{aligned}\n$$\n当 $n=2 k+1$ 时,\n$$\n\\begin{aligned}\na_n=a_{2 k+1} & =a_1+\\left(a_3-a_1\\right)+\\left(a_5-a_3\\right)+\\cdots+\\left(a_{2 k+1}-a_{2 k-1}\\right) \\\\\n& =4+8 \\times 3+8 \\times 3^3+\\cdots+8 \\times 3^{2 k-1} \\\\\n& =4+\\frac{8 \\times 3 \\times\\left(3^{2 k}-1\\right)}{3^2-1}=4+3 \\times\\left(3^{2 k}-1\\right) \\\\\n& =3^{2 k+1}+1=3^n+2+(-1)^n,\n\\end{aligned}\n$$\n故总有 $a_n=3^n+2+(-1)^n$.\n综上得不同的密码设置共有 $3^n+2+(-1)^n$ 种.", + "remark": "", + "figures": [] +} \ No newline at end of file diff --git a/processed_dataset/calculation/0191.json b/processed_dataset/calculation/0191.json new file mode 100644 index 0000000000000000000000000000000000000000..03a6fbbf49a04a6802e75bd067be15dbaec3739e --- /dev/null +++ b/processed_dataset/calculation/0191.json @@ -0,0 +1,8 @@ +{ + "source_file": "./raw_volume-zh/volume11/chapter8.tex", + "problem_type": "calculation", + "problem": "例5. 将周长为 24 的圆周等分为 24 段, 从 24 个分点中选取 8 个分点使得其中任意两点所夹的弧长不等于 3 和 8 , 问满足要求的 8 点组的不同取法有多少种?", + "solution": "解:设 24 个分点依次为 $1,2, \\cdots, 24$. 将这 24 个数列成下表:\n\\begin{tabular}{|c|c|c|c|c|c|c|c|}\n\\hline 1 & 4 & 7 & 10 & 13 & 16 & 19 & 22 \\\\\n\\hline 9 & 12 & 15 & 18 & 21 & 24 & 3 & 6 \\\\\n\\hline 17 & 20 & 23 & 2 & 5 & 8 & 11 & 14 \\\\\n\\hline\n\\end{tabular}\n表中每行相邻两数所代表的点所夹的弧长等于 3(认为同一行首尾两数也相邻), 每列相邻两数所代表的点所夹的弧长等于 8 (认为同一列首尾两数也相邻), 故每列中至多只能取出 1 个数, 8 列至多取出 8 个数, 但一共要取出 8 个数, 故每列恰取出一个数且相邻两列所取的数不同行.\n(认为首尾两列也相邻)\n仿照例 4 ,若将每列看成正 $n$ 边形的一个顶点 (本例中 $n=8$ ), 每列中第一、三、三行看成 3 种不同的颜色,则这个问题等价于下列问题中 $n=8$ 的情形: 将正 $n$ 边形的 $n$ 个顶点染色,每个顶点任意染成红、蓝、黄三种颜色之一, 使任意相邻两顶点不同色, 则不同的染色方法共有多少种?(约定线段叫做正二边形).\n假设共有 $a_n$ 种不同的染色方法, 则类似于例 4 可得 $a_2=3 \\times 2, a_n+a_{n-1} =3 \\times 2^{n-1}(n \\geqslant 3)$ 于是\n$$\n\\begin{aligned}\na_8= & \\left(a_8+a_7\\right)-\\left(a_7+a_6\\right)+\\left(a_6+a_5\\right)-\\left(a_5+a_4\\right)+ \\\\\n& \\left(a_4+a_3\\right)-\\left(a_3+a_2\\right)+a_2 \\\\\n= & 3 \\times 2^7-3 \\times 2^6+3 \\times 2^5-3 \\times 2^4+3 \\times 2^3-3 \\times 2^2+3 \\times 2 \\\\\n= & \\frac{3 \\times 2^7\\left[1-\\left(-\\frac{1}{2}\\right)^7\\right]}{1-\\left(-\\frac{1}{2}\\right)}=2^8+2=258 \\text { (种). }\n\\end{aligned}\n$$\n故共有 258 种不同的取法.", + "remark": "注1 本题的解答也可由习题 8 中第 5 题的解答内给出的公式令 $n=8$, $m=3$ 而得到.\n注2 本题可推广为下列一般性结论:\n设 $m, n$ 为大于 1 的正整数, 又 $d=(m, n)$ 表示 $m, n$ 的最大公约数且 $n>d$. 将周长为 $m n$ 的圆周等分为 $m n$ 段, 从 $m n$ 个分点中取 $n$ 个点,使其中任意两点所夹的弧长不等于 $m$ 和 $k n\\left(k=1,2, \\cdots,\\left[\\frac{m}{2}\\right]\\right)$, 记满足要求的 $n$ 点组的不同取法总数为 $A_n(m)$, 则 $A_n(m)=\\left[(m-1)^{\\frac{n}{d}}+(-1)^{\\frac{n}{d}}\\right]^d$.", + "figures": [] +} \ No newline at end of file diff --git a/processed_dataset/calculation/0192.json b/processed_dataset/calculation/0192.json new file mode 100644 index 0000000000000000000000000000000000000000..fcfce880b9be2a8a4e2f22d55013ea919c1dcb8a --- /dev/null +++ b/processed_dataset/calculation/0192.json @@ -0,0 +1,11 @@ +{ + "source_file": "./raw_volume-zh/volume11/chapter9.tex", + "problem_type": "calculation", + "problem": "例1. 用 $1 \\times 1,2 \\times 2,3 \\times 3$ 的瓷砖铺满 $23 \\times 23$ 的地(不允许重叠,也不留空隙), 问最少要用几块 $1 \\times 1$ 的瓷砖?(假设每块瓷砖不允许分割成小的瓷砖)", + "solution": "解:将 $23 \\times 23$ 的方格地面 (已画成 $1 \\times 1$ 的小方格) 中第 $1,4,7, \\cdots$, 19,22 列中小方格染成黑色,其余各列中小方格染成白色,则每块 $2 \\times 2$ 的瓷砖或盖住了 2 个白色方格和 2 个黑色方格或盖住了 4 个白色方格,而每块 $3 \\times$ 3 的瓷砖盖住了 3 个黑色方格和 6 个白色方格, 如果不用 $1 \\times 1$ 的瓷砖, 无论用多少块 $2 \\times 2$ 和 $3 \\times 3$ 的瓷砖,盖住的白色方格数总是一个偶数,但一共有 $15 \\times$ 23 个白色方格, 而 $15 \\times 23$ 是一个奇数,矛盾, 故不用 $1 \\times 1$ 的瓷砖不可能将 $23 \\times 23$ 的正方形铺满.\n其次,如图() 表明用 $2 \\times 2$ 和 $3 \\times 3$ 的瓷砖可铺满 $12 \\times 11$ 的矩形,如图() 表明只要用 1 块 $1 \\times 1$ 的瓷砖可将 $23 \\times 23$ 的正方形铺满.\n综上可知,最少要用 1 块 $1 \\times 1$ 的瓷砖.", + "remark": "", + "figures": [ + "./images/volume11/figures/fig-c9i1.png", + "./images/volume11/figures/fig-c9i2.png" + ] +} \ No newline at end of file diff --git a/processed_dataset/calculation/0193.json b/processed_dataset/calculation/0193.json new file mode 100644 index 0000000000000000000000000000000000000000..5d48b6fa1c057da4b6ee2cef6423c24fedbde467 --- /dev/null +++ b/processed_dataset/calculation/0193.json @@ -0,0 +1,19 @@ +{ + "source_file": "./raw_volume-zh/volume11/chapter9.tex", + "problem_type": "calculation", + "problem": "例2. 给定边长为 10 的正三角形, 用平行于其边的直线将它全部剖分为边长为 1 的小正三角形.\n现有 $m$ 个如图() 所示的三角形块且有 25- $m$ 个如图() 所示的四边形块, 问\n(1) 若 $m=10$, 能否用它们拼出原三角形?\n(2) 求能拼出原三角形的所有 $m$.", + "solution": "解:如图() 把小正三角形块染上黑白相间的两种颜色.\n设 $m$ 个三角形块能覆盖如图() 中 3 个白色小正三角形的有 $x$ 个, 则如图() 中白色小正三角形的个数为\n$$\n3 x+(m-x)+2(25-m)=2 x+50-m .\n$$\n而如图() 中共有 55 个白色小正三角形,若 $m$ 个三角形块和 25- $m$ 个四边形块可以覆盖如图(), 则\n$$\n2 x+50-m=55 \\text {, 即 } 2 x=m+5 .\n$$\n由此可知 $m$ 为奇数.\n(1) 若 $m=10$, 则 $m$ 为偶数,故不能用 10 个三角形块及 15 个四边形块拼出原三角形;\n(2)显然 $m \\geqslant x$, 于是 $2 m \\geqslant 2 x=m+5$, 即 $m \\geqslant 5$, 又 $25-m \\geqslant 0$, 所以 $m \\leqslant 25$. 故 $m \\in M=\\left\\{n \\mid 5 \\leqslant n \\leqslant 25, n \\in \\mathbf{N}_{+}, n\\right.$ 为奇数 $\\}$. 另一方面, 对任意 $m=2 k-1 \\in M(3 \\leqslant k \\leqslant 13)$. 如图(), 我们先用 5 个正三角形块覆盖原三角形右侧的 5 个边长为 2 的正三角形 (如图() 中画有阴影的正三角形). 再用 $m-5=2(k-3)$ 个如图() 的正三角形块覆盖图中 $k-3$ 个 $2 \\times 2$ 的菱形.\n最后余下的 $10-(k-3)=13-k$ 个 $2 \\times 2$ 的菱形可用 $25-m=2(13-k)$ 个平行四边形覆盖.\n综上可知, 所求的一切 $m$ 组成的集合为 $M$.", + "remark": "", + "figures": [ + "./images/volume11/figures/fig-c9i3.png", + "./images/volume11/figures/fig-c9i4.png", + "./images/volume11/figures/fig-c9i5.png", + "./images/volume11/figures/fig-c9i5.png", + "./images/volume11/figures/fig-c9i5.png", + "./images/volume11/figures/fig-c9i5.png", + "./images/volume11/figures/fig-c9i5.png", + "./images/volume11/figures/fig-c9i6.png", + "./images/volume11/figures/fig-c9i6.png", + "./images/volume11/figures/fig-c9i3.png" + ] +} \ No newline at end of file diff --git a/processed_dataset/calculation/0194.json b/processed_dataset/calculation/0194.json new file mode 100644 index 0000000000000000000000000000000000000000..d3504490c641536c2e6ae84bfca3fb316adfb2c2 --- /dev/null +++ b/processed_dataset/calculation/0194.json @@ -0,0 +1,8 @@ +{ + "source_file": "./raw_volume-zh/volume11/chapter9.tex", + "problem_type": "calculation", + "problem": "例3. 已知某议会共有 30 位议员, 其中每两人或者是朋友, 或者是政敌, 每位议员恰有 6 个政敌.\n每 3 个人组成一个 3 人委员会, 如果一个委员会里 3 个人两两都是朋友或者两两都是政敌,则称之为好委员会.\n求所有好委员会的个数.", + "solution": "解:用 30 个点代表 30 个委员 (其中任意 4 点不共面), 若两位议员是朋友, 则对应两点连一红色线段, 否则连一蓝色线段.\n显然好委员的个数就是三边同色的三角形 (简称同色三角形) 的个数.\n每个非同色三角形内有 2 个异色角 (从一点出发的两条不同色线段组成的角) 图中每点出发有 6 条蓝色线段, 23 条红色线段可组成以该点为顶点的 $23 \\times 6=138$ 个异色角, 图中共有 $138 \\times 30=4140$ 个异色角.\n所以非同色三角形有 $\\frac{1}{2} \\times 4140=2070$ 个.\n故同色三角形 (即好委员会) 的总数为 $\\mathrm{C}_{30}^3-2070=1990$ 个.", + "remark": "", + "figures": [] +} \ No newline at end of file diff --git a/processed_dataset/calculation/0195.json b/processed_dataset/calculation/0195.json new file mode 100644 index 0000000000000000000000000000000000000000..ded1f100a000fa9cb14997241bf9a43e8eda9dc2 --- /dev/null +++ b/processed_dataset/calculation/0195.json @@ -0,0 +1,14 @@ +{ + "source_file": "./raw_volume-zh/volume11/chapter9.tex", + "problem_type": "calculation", + "problem": "例4. 已知 8 个人中既不存在三个人互相认识, 也不存在四个人两两互相不认识, 问这 8 个人中最少有几对人互相认识? 最少有几对人互相不认识? 说明理由.", + "solution": "解:用 8 个点表示 8 个人, 若两人互相认识, 则对应点连线染红色, 否则对应点连线染蓝色, 得到一个 2 色完全图 $K_8$, 由已知条件知其中既不存在红色三角形又不存在蓝色完全图 $K_4$.\n若从某顶点 $A$ 出发至少有 6 条蓝边, 那么由.\nRamsey 定理知这 6 条蓝边另一端为顶点的 2 色 $K_6$ 中或者有红色三角形或者有蓝色三角形, 后者又导致存在蓝色 $K_4$, 都与已知矛盾.\n由此可知, 这个 $K_8$ 中从每个顶点出发至少有 2 条红边, 故图中至少有 $\\frac{1}{2} \\times 2 \\times 8=8$ 条红边.\n若从某点 $B$ 出发至少有 4 条红边, 则以这 4 条红边的另一端为顶点的 $K_4$ 中或者有一条红边, 从而导致存在红色三角形, 或者它本身就是一个蓝色 $K_4$, 这都与已知矛盾, 故知图中每点出发至多有 3 条红边, 从而图中一共至多有 $\\frac{1}{2} \\times 3 \\times 8=12$ 条红边.\n(1)若 $K_8$ 中恰有 8 条红边, 则每点恰引出两条红边, 从而 8 条红边必构成一个或两个圈 (闭折线). 因为每点都只引出 2 条红边, 故当有两个圈时, 这两个圈无公共顶点, 又图中不存在三角形, 故当有两个圈时, 必然各有 4 条边.\n当 8 条红边构成两个圈 $A_1 A_2 A_3 A_4$ 和 $A_5 A_6 A_7 A_8$ 时, $A_1 A_3 A_5 A_7$ 为蓝色 $K_4$, 当 8 条红边构成一个圈 $A_1 A_2 A_3 A_4 A_5 A_6 A_7 A_8$ 时, $A_1 A_3 A_5 A_7$ 也构成蓝色 $K_4$, 均与已知矛盾.\n可见, $K_8$ 中至少有 9 条红边.\n(2) 设 $K_8$ 中恰有 9 条红边时, 由于其中没有红色三角形且每点至多引出 3 条红边, 至少 2 条红边.\n这样一来, $K_8$ 只有下列如图()和()两种可能(图中实线表红边,蓝边没有画出), 其中如图()中虚线表示可将边 $A_2 A_7$ 去掉而代之以 $A_2 A_6$. 容易看出, (a) 和 (b) 中都有蓝色 $K_4: A_1 A_3 A_5 A_7$, 矛盾, 故知 $K_8$ 中至少有 10 条红线.\n如图(),()所示, 中都既无红色三角形又无蓝色 $K_4$, 综上可知, 2 色 $K_8$ 中最少有 10 条红边, 最多有 12 条红边, 即 8 个人中最少有 10 对人互相认识, 最多有 12 对人互相认识, 从而最少有 $\\mathrm{C}_8^2-12=16$ 对人互相不认识.", + "remark": "", + "figures": [ + "./images/volume11/figures/fig-c9i7-1.png", + "./images/volume11/figures/fig-c9i7-2.png", + "./images/volume11/figures/fig-c9i7-1.png", + "./images/volume11/figures/fig-c9i8-1.png", + "./images/volume11/figures/fig-c9i8-2.png" + ] +} \ No newline at end of file diff --git a/processed_dataset/calculation/0196.json b/processed_dataset/calculation/0196.json new file mode 100644 index 0000000000000000000000000000000000000000..bf33e7601f5683e7e9da9d091dfc306df3dab43c --- /dev/null +++ b/processed_dataset/calculation/0196.json @@ -0,0 +1,10 @@ +{ + "source_file": "./raw_volume-zh/volume11/chapter9.tex", + "problem_type": "calculation", + "problem": "例5. 由 $2 \\times 2$ 的方格纸去掉一个方格余下的图形称为拐形.\n用这种拐形去覆盖 $5 \\times 7$ 的方格板, 每个拐形恰覆盖 3 个方格, 可以重叠但不能超出方格板的边界.\n问能否使方格板上每个方格被覆盖的层数都相同? 说明理由.", + "solution": "解:将 $5 \\times 7$ 方格板的每一个小方格内填写数 -2 和 1 如图() 所示.\n易见每个拐形覆盖的 3 个数之和非负.\n因而无论用多少个拐形覆盖多少次, 盖住的所有数字之和 (一个数被覆盖了几层就计算几次)都是非负的.\n另一方面, 方格板上数字的总和为 $12 \\times(-2)+23 \\times 1=-1$, 当被覆盖 $k$ 层时, 盖住的数字之和等于 $-k$, 这表明不存在\n\\begin{tabular}{|c|c|c|c|c|c|c|}\n\\hline-2 & 1 & -2 & 1 & -2 & 1 & -2 \\\\\n\\hline 1 & 1 & 1 & 1 & 1 & 1 & 1 \\\\\n\\hline-2 & 1 & -2 & 1 & -2 & 1 & -2 \\\\\n\\hline 1 & 1 & 1 & 1 & 1 & 1 & 1 \\\\\n\\hline-2 & 1 & -2 & 1 & -2 & 1 & -2 \\\\\n\\hline\n\\end{tabular}\n满足题中要求的覆盖.", + "remark": "", + "figures": [ + "./images/volume11/figures/fig-c9i9.png" + ] +} \ No newline at end of file diff --git a/processed_dataset/calculation/0197.json b/processed_dataset/calculation/0197.json new file mode 100644 index 0000000000000000000000000000000000000000..f701bbc4e542f9eceffa22b6811ac6a2eac870c1 --- /dev/null +++ b/processed_dataset/calculation/0197.json @@ -0,0 +1,8 @@ +{ + "source_file": "./raw_volume-zh/volume11/chapter9.tex", + "problem_type": "calculation", + "problem": "例7. 将 $m \\times n$ 棋盘(由 $m$ 行 $n$ 列方格组成, $m \\geqslant 3, n \\geqslant 3$ ) 的所有小方格都染上红、蓝二色之一.\n如果两个相邻 (有公共边) 的小方格异色, 则称这两个小方格为一个\"标准对\". 设棋盘中 \"标准对\" 的个数为 $S$. 试问: $S$ 是奇数还是偶数? 由哪些方格的颜色确定? 什么情况下 $S$ 为奇数? 什么情况下 $S$ 为偶数? 说明理由.", + "solution": "解:把所有方格分为 3 类.\n第一类方格位于棋盘的四个角上,第二类方格位于棋盘的边界 (不包括四个角)上,其余方格为第三类.\n将所有红色方格填上数 1 , 所有蓝色方格填上数 -1 . 记第一类方格的填数分别为 $a, b, c, d$. 第二类方格的填数分别为 $x_1, x_2, \\cdots, x_{2 m+2 n-8}$. 第三类方格的填数为 $y_1, y_2, \\cdots, y_{(n-2)(m-2)}$. 对任何两个相邻的方格, 在它们的公共边上标上这两个方格内标数的乘积.\n设所有公共边上的标数的积为 $H$.\n对每个第一类方格, 它有两个邻格, 所以它的标数在 $H$ 中出现两次, 对每个第二类方格, 它有 3 个邻格, 所以它的标数在 $H$ 中出现 3 次, 对于第三类方格, 它有 4 个邻格, 它的标数在 $H$ 中出现 4 次,于是\n$$\n\\begin{aligned}\nH & =(a b c d)^2\\left(x_1 x_2 \\cdots x_{2 m+2 n-8}\\right)^3\\left(y_1 y_2 \\cdots y_{(n-2)(m-2)}\\right)^4 \\\\\n& =x_1 x_2 \\cdots x_{2 m+2 n-8} .\n\\end{aligned}\n$$\n当 $x_1 x_2 \\cdots x_{2 m+2 n-8}=1$ 时, $H=1$, 此时有偶数个标准对, 当 $x_1 x_2 \\cdots x_{2 m+2 n-8}=-1$ 时, $H=-1$, 此时有奇数个标准对, 这表明: $S$ 的奇偶性由第二类格的颜色确定.\n当第二类格中有奇数个蓝色格时, $S$ 为奇数, 当第二类格中有偶数个蓝色格时, $S$ 为偶数.", + "remark": "", + "figures": [] +} \ No newline at end of file diff --git a/processed_dataset/calculation/0198.json b/processed_dataset/calculation/0198.json new file mode 100644 index 0000000000000000000000000000000000000000..64d6dd68ea5064e9fc57270b7118e26f4810f027 --- /dev/null +++ b/processed_dataset/calculation/0198.json @@ -0,0 +1,8 @@ +{ + "source_file": "./raw_volume-zh/volume11/exercise1.tex", + "problem_type": "calculation", + "problem": "问题1. 将 $n+1$ 件不同奖品全部发给 $n$ 个同学, 每人至少一件, 则发放的方法数为", + "solution": "由题意 $n$ 个同学中恰有 1 人得 2 件, 其余 $n-1$ 个人每人得 1 件, 故发放奖品的方法数为 $\\mathrm{C}_{n+1}^2 \\mathrm{~A}_n^n=\\frac{(n+1) \\cdot n \\cdot n !}{2}$.", + "remark": "", + "figures": [] +} \ No newline at end of file diff --git a/processed_dataset/calculation/0199.json b/processed_dataset/calculation/0199.json new file mode 100644 index 0000000000000000000000000000000000000000..d103bf38a1ff0a9e0f59b2882eaf4e4608c0553f --- /dev/null +++ b/processed_dataset/calculation/0199.json @@ -0,0 +1,8 @@ +{ + "source_file": "./raw_volume-zh/volume11/exercise1.tex", + "problem_type": "calculation", + "problem": "问题2. 从 5 位男同学和 4 位女同学中选出 4 人组成一个代表团参加全校辩论比赛.\n若要求男同学和女同学都至少一人,则不同的选择种数为", + "solution": "不同的选法种数为 $\\mathrm{C}_5^1 \\mathrm{C}_4^3+\\mathrm{C}_5^2 \\mathrm{C}_4^2+\\mathrm{C}_5^3 \\mathrm{C}_4^1=120$.", + "remark": "", + "figures": [] +} \ No newline at end of file diff --git a/processed_dataset/calculation/0200.json b/processed_dataset/calculation/0200.json new file mode 100644 index 0000000000000000000000000000000000000000..1a50561f8355d08a4688158c76f011fbf1a376a5 --- /dev/null +++ b/processed_dataset/calculation/0200.json @@ -0,0 +1,8 @@ +{ + "source_file": "./raw_volume-zh/volume11/exercise1.tex", + "problem_type": "calculation", + "problem": "问题3. 如果自然数 $a$ 的各位数字之和等于 7 , 那么称 $a$ 为 \"吉祥数\". 将所有 \"吉祥数\" 从小到大排成一列 $a_1, a_2, a_3, \\cdots$, 若 $a_n=2005$, 则 $a_{5 n}=$ .", + "solution": "设 $k$ 位 \"吉祥数\" 的各位数字从高位到低位依次为 $x_1, x_2, \\cdots, x_k$, 则 $x_1+x_2+\\cdots+x_k=7$ 其中 $x_1 \\geqslant 1, x_i \\geqslant 0(2 \\leqslant i \\leqslant k)$, 令 $y_1=x_1-1, y_i= x_i(2 \\leqslant i \\leqslant k)$, 则 $y_1+y_2+\\cdots+y_k=6$ 其中 $y_i \\geqslant 0(1 \\leqslant i \\leqslant k)$ (1), 故 $k$ 位\"吉祥数\" 的个数 $p(k)$ 等于不定方程 (1) 的非负整数解的个数, 即 $p(k)=\\mathrm{C}_{k+5}^6$. 而 2005 是形如 $\\overline{2 a b c}$ 的 \"吉祥数\" 中最小的一个, 且 $p(1)=\\mathrm{C}_6^6=1, p(2)=\\mathrm{C}_7^6=7$, $p(3)=\\mathrm{C}_8^6=28$, 以及形如 $\\overline{1 a b c}$ 的 \"吉祥数\" 的个数等于不定方程 $a+b+c=6$ 的非负整数解的个数, 即 $\\mathrm{C}_{6+3-1}^6=28$ 个.\n故 2005 是第 $1+7+28+28+1=65$ 个 \"吉祥数\", 即 $a_{65}=2005$, 从而 $n=65,5 n=325$. 又 $p(4)=\\mathrm{C}_9^6=84, p(5)= \\mathrm{C}_{10}^6=210, \\sum_{k=1}^5 p(k)=330$ 即 $a_{330}=70000$, 从而 $a_{329}=61000, a_{328}=60100$, $a_{327}=60010, a_{326}=60001, a_{325}=52000$, 即 $a_{5 n}=a_{325}=52000$.", + "remark": "", + "figures": [] +} \ No newline at end of file diff --git a/processed_dataset/calculation/0201.json b/processed_dataset/calculation/0201.json new file mode 100644 index 0000000000000000000000000000000000000000..60538590c11f26c354e0bb61166a526cd31a3ea7 --- /dev/null +++ b/processed_dataset/calculation/0201.json @@ -0,0 +1,8 @@ +{ + "source_file": "./raw_volume-zh/volume11/exercise1.tex", + "problem_type": "calculation", + "problem": "问题4. 设三位数 $n=\\overline{a b c}$, 若以 $a, b, c$ 为 3 条边的长可以构成一个等腰(含等边) 三角形,则这样的三位数 $n$ 有个.", + "solution": "显然 $a, b, c \\in\\{1,2,3, \\cdots, 9\\}$. (1) 若构成等边三角形, 则这样的三位数的个数为 $n_1=\\mathrm{C}_9^1=9$ 个; (2) 若构成等腰 (非等边) 三角形, 设这样的三位数有 $n_2$ 个.\n当小数为底边长时, 设小数为 $i$, 则大数可以为 $i+1, i+ 2, \\cdots, 9$, 有 $9-i(1 \\leqslant i \\leqslant 8)$ 个.\n这时三角形的个数为 $\\sum_{i=1}^8(9-i)=\\frac{1}{2}(1+$ 8) $\\bullet 8=36$ 个; 当大数为底边时, 可能构成二角形的数码如下表, 共 16 种情况,\n\\begin{tabular}{|c|c|c|c|c|c|c|c|}\n\\hline 小数 & 2 & 3 & 4 & 5 & 6 & 7 & 8 \\\\\n\\hline 大数 & 3 & 4,5 & $5,6,7$ & $6,7,8,9$ & $7,8,9$ & 8,9 & 9 \\\\\n\\hline\n\\end{tabular}\n故等腰(非等边)三角形共有 $36+16=52$ 个, 对应的三位数的个数为 $n_2= \\mathrm{C}_3^2 \\times 52=156$. 综上知满足题目条件的三位数 $n$ 共有 $n_1+n_2=9+156=165$ 个.", + "remark": "", + "figures": [] +} \ No newline at end of file diff --git a/processed_dataset/calculation/0202.json b/processed_dataset/calculation/0202.json new file mode 100644 index 0000000000000000000000000000000000000000..3d85fc7e9343a5a577a0784113b6763c7ee530cd --- /dev/null +++ b/processed_dataset/calculation/0202.json @@ -0,0 +1,8 @@ +{ + "source_file": "./raw_volume-zh/volume11/exercise1.tex", + "problem_type": "calculation", + "problem": "问题5. 从 $1,2,3,4,5,7,9$ 这 7 个数字中任取两个数字分别做成对数的底数和真数, 则可构成不相等的对数值的数目是", + "solution": "由 $1,2,3,4,5,7,9$ 这 7 个数字中任取两个不同数字做成对数的底数和真数有 $A_7^2$ 种方法, 但 1 不能做底数,故应减去 $A_6^1$, 又以 $2,3,4,5,7,9$ 中任何一个做底数, 1 做真数时, 得到的对数值都等于 0 , 故又要减去 $\\mathrm{A}_6^1-1$ 个, 此外 $\\log _2 4=\\log _3 9, \\log _4 2=\\log _9 3, \\log _3 2=\\log _9 4, \\log _2 3=\\log _4 9$, 故还应减去 4 个, 因此, 不同的对数值共有 $\\mathrm{A}_7^2-\\mathrm{A}_6^1-\\left(\\mathrm{A}_6^1-1\\right)-4=27$ 个.", + "remark": "", + "figures": [] +} \ No newline at end of file diff --git a/processed_dataset/calculation/0203.json b/processed_dataset/calculation/0203.json new file mode 100644 index 0000000000000000000000000000000000000000..0a84681f0da09b647bf5da8f5217bd1f1d4efcef --- /dev/null +++ b/processed_dataset/calculation/0203.json @@ -0,0 +1,8 @@ +{ + "source_file": "./raw_volume-zh/volume11/exercise1.tex", + "problem_type": "calculation", + "problem": "问题6. 某次兵乓球单打比赛, 原计划每两名选手比赛一场, 但有 3 名选手各比赛了 2 场后退出了比赛, 这样全部比赛一共进行了 50 场, 那么上述 3 名选手之间比赛的场次数是 .", + "solution": "设共有 $n$ 名选手,该 3 名选手之间比赛的场数为 $r$, 则 $50=\\mathrm{C}_{n-3}^2+(3 \\times 2-r)$, 即 $(n-3)(n-4)=88+2 r$, 经检验仅当 $r=1$ 时, $n=13$ 为正整数, 故 3 名选手之间比赛了 1 场.", + "remark": "", + "figures": [] +} \ No newline at end of file diff --git a/processed_dataset/calculation/0204.json b/processed_dataset/calculation/0204.json new file mode 100644 index 0000000000000000000000000000000000000000..44ee0a9eba7b063629232f3462516da6a8a2b0d9 --- /dev/null +++ b/processed_dataset/calculation/0204.json @@ -0,0 +1,8 @@ +{ + "source_file": "./raw_volume-zh/volume11/exercise1.tex", + "problem_type": "calculation", + "problem": "问题7. 已知直线 $a x+b y+c=0$ 中, $a, b, c$ 取自集合 $\\{-3,-2,-1,0,1,2$, $3\\}$ 中三个不同元素, 并没该直线的倾斜角为锐角, 那么这样直线的条数是 .", + "solution": "设倾斜角为 $\\theta$, 则 $\\tan \\theta=-\\frac{a}{b}>0$, 由 $a, b$ 取值集合的对称性,不失一般性可设 $a>0, b<0$. (1) 当 $c=0$ 时, $a$ 有 $\\mathrm{C}_3^1$ 种取法, $b$ 有 $\\mathrm{C}_3^1$ 种取法, 排除 2 个重复 (因 $x-y=0,2 x-2 y=0,3 x-3 y=0$ 表示同一直线), 故这样的直线有 $\\mathrm{C}_3^1 \\mathrm{C}_3^1-2=7$ 条; (2) 当 $c \\neq 0$ 时, $a$ 有 $\\mathrm{C}_3^1$ 种取法, $b$ 有 $\\mathrm{C}_3^1$ 种取法, $c$ 有 $\\mathrm{C}_4^1$ 种取法, 故这样的直线有 $\\mathrm{C}_3^1 \\mathrm{C}_3^1 \\cdot \\mathrm{C}_4^1=36$ 条, 从而符合条件的直线共有 $7+36=43$ 条.", + "remark": "", + "figures": [] +} \ No newline at end of file diff --git a/processed_dataset/calculation/0205.json b/processed_dataset/calculation/0205.json new file mode 100644 index 0000000000000000000000000000000000000000..044056aae205779876fc153ec3bb168eeb9a0cc0 --- /dev/null +++ b/processed_dataset/calculation/0205.json @@ -0,0 +1,10 @@ +{ + "source_file": "./raw_volume-zh/volume11/exercise1.tex", + "problem_type": "calculation", + "problem": "问题8. $2 \\times 3$ 的矩形花坛被分成 6 个 $1 \\times 1$ 的小正方形区域: $A, B, C, D, E, F$, 在每个区域内栽种一种植物, 相邻两个区域内栽种的植物不同, 今有 6 种植物可供选择,则共有种不同的栽种方法.", + "solution": "如图(), $A$ 与 $B$ 内栽种植物的方法有 $\\mathrm{A}_6^2$ 种.\n若 $C$ 与 $B$ 内栽种同一种植物, 则 $C$ 和 $D$ 内栽种植物的方法有 $\\mathrm{C}_5^1$ 种; 若 $C$ 与 $B$ 内栽种不同的植物, 则 $C$ 与 $D$ 内栽种植物的方法有 $\\mathrm{C}_4^1 \\mathrm{C}_4^1$ 种,故 $C$ 与 $D$ 内栽种植物的方法有 $\\mathrm{C}_5^1+\\mathrm{C}_4^1 \\mathrm{C}_4^1=21$ 种, 同理 $E$ 与 $F$ 内栽种植物的方法也有 21 种, 故符合条件的栽种方法共有 $\\mathrm{A}_6^2 \\times 21^2=13230$ 种.", + "remark": "", + "figures": [ + "./images/volume11/figures/fig-c1a8.png" + ] +} \ No newline at end of file diff --git a/processed_dataset/calculation/0206.json b/processed_dataset/calculation/0206.json new file mode 100644 index 0000000000000000000000000000000000000000..b9682c1f61ceb3df42595f94e6a17536688a1a24 --- /dev/null +++ b/processed_dataset/calculation/0206.json @@ -0,0 +1,8 @@ +{ + "source_file": "./raw_volume-zh/volume11/exercise1.tex", + "problem_type": "calculation", + "problem": "问题9. 甲、乙两队各抽出 7 名队员按事先排好的顺序出场参加围棋擂台赛, 双方先由 1 号队员比赛, 负者被淘汰, 胜者再与负方 2 号队员比赛, $\\cdots \\cdots \\cdot$, 直到一方队员全部被淘汰为止, 另一方获得胜利, 形成一个比赛过程, 那么所有可能出现的比赛过程的种数为 .", + "solution": "先考虑甲获胜的比赛过程的种数,设甲队第 $i$ 号队员胜了 $x_i$ 场 $(i= 1,2, \\cdots, 7)$, 于是 $x_1+x_2+\\cdots+x_7=7$, 且甲队获胜的比赛过程同不定方程 $x_1+x_2+\\cdots+x_7=7$ 的非负整数解组 $\\left(x_1, x_2, \\cdots, x_7\\right)$ 成一一对应, 故甲队不同的比赛过程的数目等于不定方程 $x_1+x_2+\\cdots+x_7=7$ 的非负整数解组的数目为 $\\mathrm{C}_{13}^6$. 同理, 乙获胜的不同比赛过程的数目也为 $\\mathrm{C}_{13}^6$, 故不同的比赛过程共有 $2 \\mathrm{C}_{13}^6=3432$ 种.", + "remark": "", + "figures": [] +} \ No newline at end of file diff --git a/processed_dataset/calculation/0207.json b/processed_dataset/calculation/0207.json new file mode 100644 index 0000000000000000000000000000000000000000..805696dc5c1ca9bbedc84ad5a5aaed3f950a04ab --- /dev/null +++ b/processed_dataset/calculation/0207.json @@ -0,0 +1,10 @@ +{ + "source_file": "./raw_volume-zh/volume11/exercise1.tex", + "problem_type": "calculation", + "problem": "问题10. 在一次射击比赛中, 有 8 个泥制的靶子挂成如图()所示的三列 (其中两列 3 个,一列 2 个),一位神枪手每一枪按下面规则打中靶子:\n(1) 选择一列;\n(2)打中所选一列的最下面未打过的靶子.\n问打中这 8 个靶子共有多少种不同的顺序?", + "solution": "随意射击 8 个靶子有 8 ! 种方法, 由于每列靶子的顺序已经确定, 故现在的射击方法共有 $\\frac{8 !}{3 ! 2 ! 3 !}=560$ 种不同的顺序.", + "remark": "", + "figures": [ + "./images/volume11/figures/fig-c1p10.png" + ] +} \ No newline at end of file diff --git a/processed_dataset/calculation/0208.json b/processed_dataset/calculation/0208.json new file mode 100644 index 0000000000000000000000000000000000000000..5da3c72be3da532fd0de91a845c7005bdf167dbd --- /dev/null +++ b/processed_dataset/calculation/0208.json @@ -0,0 +1,8 @@ +{ + "source_file": "./raw_volume-zh/volume11/exercise1.tex", + "problem_type": "calculation", + "problem": "问题11. 将正四棱雉的顶点染色, 要求同一条棱的两个端点不同色.\n如果只有 5 种颜色可供使用,那么不同的染色方法共有多少种?(假设经过绕对称轴旋转后可以变相同的染色方法是同一种染色方法)", + "solution": "因侧面三角形的三个顶点互不同色,故最少要用 3 种颜色.\n(1)使用 5 种颜色时, 从 5 种颜色中取 1 种颜色染上顶点有 $\\mathrm{C}_5^1$ 种方法, 其余 4 色染底面 4 个顶点(4 个元素的圆排列)有 3 ! 种方法.\n于是, 这时不同的染色方法有 $\\mathrm{C}_5^1 \\cdot 3 !=30$ 种; (2) 使用 4 种颜色时, 从 5 色中取出 4 色有 $\\mathrm{C}_5^4$ 种方法, 从取出的 4 色中取一种颜色染上顶点有 $\\mathrm{C}_4^1$ 种方法,其余 3 色染下底面的 4 个顶点,其中必有一对顶点同色, 从 3 色中取 1 色染一对顶点有 $\\mathrm{C}_3^1$ 种方法, 其余 2 色染剩下 2 个顶点 (2 个元素的圆排列)有 1 ! 种方法, 这时不同的染色方法有 $\\mathrm{C}_5^4 \\mathrm{C}_4^1 \\mathrm{C}_3^1 \\cdot 1 !=60$ 种; (3)使用 3 种颜色时, 从 5 色中取出 3 色有 $\\mathrm{C}_5^3$ 种方法, 从取出的 3 色中取出 1 色染上顶点有 $\\mathrm{C}_3^1$ 种方法, 其余 2 色染下底面的 4 个顶点, 其中两对顶点分别同色, 用 2 色染这两对顶点 (2个元素的圆排列) 只有 1 ! 种方法, 这时不同的染色方法有 $\\mathrm{C}_5^3 \\mathrm{C}_3^1 \\cdot 1 !=30$ 种.\n综上知满足题目要求的不同染色方法共有 $30+60+30=120$ 种.", + "remark": "", + "figures": [] +} \ No newline at end of file diff --git a/processed_dataset/calculation/0209.json b/processed_dataset/calculation/0209.json new file mode 100644 index 0000000000000000000000000000000000000000..7859df1cdf3a3524f0f45e88f7dddaec3d23cf17 --- /dev/null +++ b/processed_dataset/calculation/0209.json @@ -0,0 +1,8 @@ +{ + "source_file": "./raw_volume-zh/volume11/exercise1.tex", + "problem_type": "calculation", + "problem": "问题12. 已知两个实数集合 $A=\\left\\{a_1, a_2, \\cdots, a_{100}\\right\\}$ 与 $B=\\left\\{b_1, b_2, \\cdots, b_{50}\\right\\}$, 若从 $A$ 到 $B$ 的映射 $f$ 使得 $B$ 中每个元素都有原象,且 $f\\left(a_1\\right) \\leqslant f\\left(a_2\\right) \\leqslant \\cdots \\leqslant f\\left(a_{100}\\right)$, 则这样的映射的总个数为 .", + "solution": "不妨设 $b_1n-\\left(\\frac{n}{3}+\\frac{n}{5}+\\frac{n}{7}\\right)+\\left(\\frac{n}{3 \\times 5}-1+\\frac{n}{3 \\times 7}-1+\\frac{n}{5 \\times 7}-1\\right)- \\frac{n}{3 \\times 5 \\times 7}$ 和 $1000)), 而 $2005 \\times 2007=1998 \\times 2007+ 7 \\times 2007=(2 \\times 3) \\times(999 \\times 669)+(7 \\times 9) \\times 223$. 故知 $2005 \\times 2007$ 的矩形可完全剖分为若干个 $\\mathrm{L}$ 形.", + "remark": "", + "figures": [ + "./images/volume11/figures/fig-c12a3.png" + ] +} \ No newline at end of file diff --git a/processed_dataset/calculation/0217.json b/processed_dataset/calculation/0217.json new file mode 100644 index 0000000000000000000000000000000000000000..3ed9acbe1f44414353f6007112d85e4d47d55237 --- /dev/null +++ b/processed_dataset/calculation/0217.json @@ -0,0 +1,12 @@ +{ + "source_file": "./raw_volume-zh/volume11/exercise12.tex", + "problem_type": "calculation", + "problem": "问题4. 平面内 4 条直线, 其中任意 2 条不平行且任意 3 条不共点, 从而每条直线上有 3 个点, 在此条直线上截出两条线段.\n问所得 8 条线段的长度能否为\n(1) $1,2,3,4,5,6,7,8$ ?\n(2) 互不相等的正整数?", + "solution": "(1) 不能.\n事实上, 设如图() 所得 8 条线段的长分别为 $1,2,3,4,5,6$, 7,8 . 因为三角形中任意一边大于其余两边之差的绝对值, 所以边长为正整数的非等腰三角形中任何一边的长均大于 1 , 故图中长为 1 的线段只可能是 $A B$ 或 $A F$. 不妨设 $A B=1$, 由 $1=A B>|B E-A E|$, 得 $B E=A E$. 在 $\\triangle A B E$ 中由余弦定理得 $\\cos E=\\frac{B E^2+A E^2-A B^2}{2 B E \\cdot A E}=1-\\frac{1}{2 A E^2}$. 又在 $\\triangle D E F$ 中应用余弦定理得 $D F^2=D E^2+E F^2-2 D E \\cdot E F \\cos E=D E^2+E F^2-2 D E \\cdot E F \\cdot\\left(1-\\frac{1}{2 A E^2}\\right)= D E^2+E F^2-2 D E \\cdot E F+\\frac{E F}{A E} \\cdot \\frac{D E}{B E}$. 上式中 $\\frac{E F}{A E} \\cdot \\frac{D E}{B E}$ 为小于 1 的正分数, 其余各项为正整数,矛盾! 故不存在 4 条满足要求的直线.\n(2) 能.\n如图() 我们从边长为 $3,4,5$ 的直角三角形出发,适当找出正整数 $k>1$ 及 $m, n$ 使图形符合要求.\n从相似三角形性质得 $(5+4 k): m:(n+ 3)=(4+5 k): n:(m+3 k)=3: 4: 5$. 这要求 $5+4 k$ 及 $4+5 k$ 均为 3 的倍数, 不难得到 $k=4, m=28, n=32$ 时满足要求.\n即如图() 所示的图形满足题目要求.", + "remark": "", + "figures": [ + "./images/volume11/figures/fig-c12a4-1.png", + "./images/volume11/figures/fig-c12a4-2.png", + "./images/volume11/figures/fig-c12a4-3.png" + ] +} \ No newline at end of file diff --git a/processed_dataset/calculation/0218.json b/processed_dataset/calculation/0218.json new file mode 100644 index 0000000000000000000000000000000000000000..6fdfe98a4638be11536afd954f493571b926065c --- /dev/null +++ b/processed_dataset/calculation/0218.json @@ -0,0 +1,8 @@ +{ + "source_file": "./raw_volume-zh/volume11/exercise13.tex", + "problem_type": "calculation", + "problem": "问题1. 以一个正 $6 n\\left(n \\in \\mathbf{N}_{+}\\right)$边形的顶点为顶点的互不全等的三角形有多少个?", + "solution": "将正 $6 n$ 边形的顶点沿顺时针方向标号为 $1,2, \\cdots, 6 n$. 对以 $i, j, k$ 为顶点的三角形, 定义 $i, j$ 之间的距离为 $i, j$ 之间 (不含 $k$ 的一侧) 正 $6 n$ 边形的边数.\n显然,一个三角形顶点间的 3 个距离之和为 $6 n$, 其中最小距离不大于 $2 n$. (1) 当最小距离为偶数 $2 k(k==1,2, \\cdots, n)$ 时.\n中间距离不小于 $2 k$ 且不大于 $\\frac{1}{2}(6 n-2 k)=3 n-k$. 这时共有 $(3 n-k)-(2 k-1)=3 n-3 k+1$ 个互不全等的三角形; (2) 当最小距离为奇数 $2 k-1(k=1,2, \\cdots, n)$ 时, 中间距离不小于 $2 k-1$ 且不大于 $\\left[\\frac{1}{2}(6 n-(2 k-1))\\right]=3 n-k$, 这时共有 $3 n-k- (2 k-2)=3 n-3 k+2$ 个互不全等的三角形.\n由 (1), (2) 得互不全等的三角形个数为 $m=\\sum_{k=1}^n(3 n-3 k+1+3 n-3 k+2)=\\sum_{k=1}^n(6 n-6 k+3)=6 n^2-6$. $\\frac{n(n+1)}{2}+3 n=3 n^2$ (个).", + "remark": "", + "figures": [] +} \ No newline at end of file diff --git a/processed_dataset/calculation/0219.json b/processed_dataset/calculation/0219.json new file mode 100644 index 0000000000000000000000000000000000000000..0f3c092915e8c69004a9f12da8319e0a349c73ad --- /dev/null +++ b/processed_dataset/calculation/0219.json @@ -0,0 +1,8 @@ +{ + "source_file": "./raw_volume-zh/volume11/exercise13.tex", + "problem_type": "calculation", + "problem": "问题2. 将正三角形每边 $n$ 等分, 过分点在正三角形内作边的平行线将正三角形剖分为小正三角形, 问 (1) 其中有多少个正三角形 (包括原来的正三角形在内)?(2)有多少个菱形?", + "solution": "(1) 边长为 $k$ 且 \"头朝上\" 的正三角形个数为 $x_k=1+2+\\cdots+[n- (k-1)]=\\frac{1}{2}(n-k+1)(n-k+2)=\\frac{1}{6}[(n-k+1)(n-k+2)(n-k+ 3)-(n-k)(n-k+1)(n-k+2)]$, 故 \"头朝上\" 的正三角形个数为 $S_1= \\sum_{k=1}^n x_k=\\frac{1}{6} \\sum_{k=1}^n[(n-k+1)(n-k+2)(n-k+3)-(n-k)(n-k+1)(n- k+2)]=\\frac{1}{6} n(n+1)(n+2)$. 边长为 $l$ 且 \"头朝下\" 的正三角形个数为 $y_i= 1+2+3+\\cdots+(n-2 l+1)=\\frac{1}{2}(n-2 l+1)(n-2 l+2)\\left(1 \\leqslant l \\leqslant\\left[\\frac{n}{2}\\right]\\right)$.\n当 $n=2 m$ 为偶数时, \"头朝下\"的正三角形个数为 $S_2=\\sum_{l=1}^{\\left[\\frac{n}{2}\\right]} y_l=\\frac{1}{2} \\sum_{l=1}^m[(2 m- \\left.2 l+2)^2-(2 m-2 l+2)\\right]=\\sum_{l=1}^m\\left[2(m-l+1)^2-(m-l+1)\\right]=2 \\sum_{l=1}^m l^2- \\sum_{l=1}^m l=2 \\times \\frac{1}{6} m(m+1)(2 m+1)-\\frac{1}{2} m(m+1)=\\frac{1}{6} m(m+1)(4 m-1)= \\frac{1}{24} n(n+2)(2 n-1)$, 当 $n=2 m-1$ 时, 类似可得 $S_2=\\sum_{l=1}^{\\left[\\frac{n}{2}\\right]} y_l=\\frac{1}{2} \\sum_{l=1}^{m-1}(2 m- 2 l)(2 m-2 l+1)=2 \\sum_{l=1}^{m-1}(m-l)^2+\\sum_{l=1}^{m-1}(m-l)=\\frac{1}{3}(m-1) m(2 m-1)+ \\frac{1}{2}(m-1) m=\\frac{1}{6}(m-1) m(4 m+1)=\\frac{1}{24}(n-1)(n+1)(2 n+3)$, 故当 $n$ 为偶数时正三角形总个数为 $S=S_1+S_2=\\frac{1}{6} n(n+1)(n+2)+\\frac{1}{24} n(n+2) (2 n-1)=\\frac{1}{8} n(n+2)(2 n+1)$; 当 $n$ 为奇数时, 正三角形总个数为 $S= \\frac{1}{6} n(n+1)(n+2)+\\frac{1}{24}(n-1)(n+1)(2 n+3)=\\frac{1}{8}(n+1)\\left(2 n^2+3 n-1\\right)$.\n(2)因边不平行 $B C$ 的菱形的下半部分正好是一个 \"头朝下\" 的正三角形,这种对应是一一对应.\n故边不平行 $B C$ 的菱形个数等于 \"头朝下\"的正三角形个数 $S_2$. 于是由 (1) 知当 $n$ 为偶数时, 菱形的个数为 $3 S_2=\\frac{3}{8} n(n+2)(2 n- 1), n$ 为奇数时,菱形的个数为 $3 S_2=\\frac{3}{8}(n-1)(n+1)(2 n+3)$.", + "remark": "", + "figures": [] +} \ No newline at end of file diff --git a/processed_dataset/calculation/0220.json b/processed_dataset/calculation/0220.json new file mode 100644 index 0000000000000000000000000000000000000000..c801245909fb19120ec249e5c3d3a6a013d95413 --- /dev/null +++ b/processed_dataset/calculation/0220.json @@ -0,0 +1,8 @@ +{ + "source_file": "./raw_volume-zh/volume11/exercise13.tex", + "problem_type": "calculation", + "problem": "问题3. 某种比赛中每个队恰好与其他队各赛一场, 每场比赛中胜者得 2 分, 负者得 0 分, 平局各得 1 分, 比赛结束后发现, 每队所得分数中恰有一半是该队同十个得分最低的队的比赛中得到的(十个得分最低的队所得分数中一半是他们彼此比赛中得到的), 问共有几个队参加比赛?", + "solution": "设有 $n$ 个队, 则 $n$ 个队共得了 $2 \\mathrm{C}_n^2=n(n-1)$ 分.\n而 10 个得分最低的队彼此之间对局共得 $2 \\mathrm{C}_{10}^2=90$ 分.\n因为这是他们得分的一半, 故这 10 个队共得 180 分; 其余 $n-10$ 个队彼此之间比赛共得 $2 \\mathrm{C}_{n-10}^2=(n-10)(n-11)$ 分, 这也是他们得分的一半, 所以他们共得 $2(n-10)(n-11)$ 分.\n于是 $n(n- 1)=180+2(n-10)(n-11)$, 即 $(n-16)(n-25)=0$, 解得 $n_1=25, n_2=$ 16 (舍去, 因必须 $2(n-10)(n-11) \\div(n-10)>180 \\div-10)$.", + "remark": "", + "figures": [] +} \ No newline at end of file diff --git a/processed_dataset/calculation/0221.json b/processed_dataset/calculation/0221.json new file mode 100644 index 0000000000000000000000000000000000000000..cb839689892575db069b04deb96b2b182706506f --- /dev/null +++ b/processed_dataset/calculation/0221.json @@ -0,0 +1,8 @@ +{ + "source_file": "./raw_volume-zh/volume11/exercise13.tex", + "problem_type": "calculation", + "problem": "问题4. 平面内有 18 个点, 其中任意 3 点不共线, 每两点连一线段,这些线段用红蓝两色染色, 每条线段恰染一色, 其中从某点 $A$ 出发的红色线段为奇数条, 而从其他 17 个点出发的红色线段数互不相同, 求以已知点为顶点各边为红色的三角形个数以及有两边为红色一边为蓝色的三角形个数.", + "solution": "设所有三角形中三边为红色、两边红色一边蓝色、两边蓝色一边红色、 三边为蓝色的三角形个数分别为 $m, n, p, q$. 因除 $A$ 外, 从其余各点出发的红色线段数互不相同, 它们只可能是 $0,1,2, \\cdots, 16$ 或 $1,2, \\cdots, 17$. \n若为前者, 设从 $A$ 出发有 $2 k-1$ 条红线, 则图中红线总数为 $\\frac{1}{2}(0+1+2+\\cdots+16+ 2 k-1)=17 \\times 4+k-\\frac{1}{2}$ 矛盾, 故只能为后者情形.\n设除 $A$ 外, 其余 17 个点为 $B_1, B_2, \\cdots, B_{17}$, 从 $B_i$ 出发有 $i$ 条红线 $(i=1,2, \\cdots, 17)$. \n于是 $B_{17}$ 与其余 17 点连有红线, $B_1$ 仅与 $B_{17}$ 连有红线, 进一步不难得出对 $i==1,2, \\cdots, 8$, $B_{17-i}$ 仅与除 $B_1, B_2, \\cdots, B_i$ 外的 $17-i$ 个点连有红线, 而 $B_i$ 仅与 $B_{17}, B_{16}$, $\\cdots, B_{18-i}$ 这 $i$ 个点连有红线.\n从而 $A$ 仅与 $B_{17}, B_{16}, \\cdots, B_9$ 这 9 个点连有红线, 其余所连线段均为蓝线.\n设从一点出发的两条红色线段叫做红色角.\n从一点出发的两条蓝色线段叫做蓝色角, 于是红色角的总数为 $3 m+n=\\sum_{i=2}^{17} \\mathrm{C}_i^2+ \\mathrm{C}_9^2=\\mathrm{C}_3^3+\\sum_{i=3}^{17}\\left(\\mathrm{C}_{i+1}^3-\\mathrm{C}_i^3\\right)+\\mathrm{C}_9^2=\\mathrm{C}_{18}^3+\\mathrm{C}_9^2=852 \\cdots$ (1), \n蓝色角的总数为 $p+ 3 q=\\sum_{k=1}^{15} \\mathrm{C}_{17-k}^2+\\mathrm{C}_8^2=\\mathrm{C}_{17}^3+\\mathrm{C}_8^2=708 \\cdots$ (2), \n图中红色线段数为 $\\frac{1}{16}(3 m+2 n+ p)=\\frac{1}{2}(1+2+\\cdots+17+9)=81 \\cdots$ (3), \n蓝色线段数为 $\\frac{1}{16}(n+2 p+3 q)= \\frac{1}{2}(16+15+\\cdots+1+8)=72 \\cdots$ (4). \n联立(1), (2), (3), (4)解得 $m=204, n= 240, p=204, q=168$. 即三边为红色的三角形有 204 个, 两边红一边蓝的三角形有 240 个.", + "remark": "", + "figures": [] +} \ No newline at end of file diff --git a/processed_dataset/calculation/0222.json b/processed_dataset/calculation/0222.json new file mode 100644 index 0000000000000000000000000000000000000000..0407eff9badd0d47f25c3c6a65f9a0d73dd22177 --- /dev/null +++ b/processed_dataset/calculation/0222.json @@ -0,0 +1,8 @@ +{ + "source_file": "./raw_volume-zh/volume11/exercise13.tex", + "problem_type": "calculation", + "problem": "问题5. 已知有三个委员会, 对任意两个来自不同委员会的工作人员, 第三个委员会中恰有 10 人都认识他们, 也恰有 10 人都不认识他们, 试求这三个委员会中工作人员的总数.", + "solution": "将每个工作人员对应平面上一个点 (任何三点不共线), 将三个委员会的人员所对应的点集分别记为 $A 、 B 、 C$. 在任何不属于同一点集的两点之间连一条线段: 若两人互相认识, 就连红线; 若不认识, 就连蓝线, 这样所得到的图称为三部图.\n设三部图 $A-B-C$ 的顶点数为 $(a, b, c)$. 根据题意知 $A-B$ 之间的每条红色线段上恰有 10 个红色三角形, 每条蓝色线段上恰有 10 个蓝色三角形.\n由于每个同色三角形恰有一边为 $A-B$ 之间的一条线段, 故全图中同色三角形个数为 $10 a b$. 同理, 它也等于 $10 b c, 10 c a$. 因此 $a=b=c$. 这样一来, 即知同色三角形个数为 $10 a^2$. 于是异色三角形个数为 $a^3-10 a^2$. 由于每个异色三角形恰有一个同色角, 而每个同色三角形有三个同色角.\n故同色角的总数为 $3 \\times 10 a^2+a^3-10 a^2=a^3+20 a^2$. 另一方面, 每条线段上张有 20 个同色角, 故同色角的总数又等于 $20 \\times 3 a^2=60 a^2$. 故 $a^3+20 a^2=60 a^2$, 解得 $a=$ 40 , 因此, 总人数为 $3 a=120$.", + "remark": "", + "figures": [] +} \ No newline at end of file diff --git a/processed_dataset/calculation/0223.json b/processed_dataset/calculation/0223.json new file mode 100644 index 0000000000000000000000000000000000000000..04806b78ac22488053c59cc1e81947ee46dcd83e --- /dev/null +++ b/processed_dataset/calculation/0223.json @@ -0,0 +1,8 @@ +{ + "source_file": "./raw_volume-zh/volume11/exercise15.tex", + "problem_type": "calculation", + "problem": "问题1. 对于有限集 $A$, 存在函数 $f: \\mathbf{N}_{+} \\rightarrow A$, 具有以下性质: 若 $i, j \\in \\mathbf{N}_{+}$, 且 $|i-j|$ 为素数,则 $f(i) \\neq f(j)$, 问集合 $A$ 中最少有几个元素?", + "solution": "因 1,3,6,8 中任意两个之差的绝对值为素数, 由题意知 $f(1)$, $f(3), f(6), f(8)$ 为 $A$ 中两两不同的数, 从而 $|A| \\geqslant 4$, 另一方面, 假设 $A= \\{0,1,2,3\\}$ 且对任意 $x \\in \\mathbf{N}_{+}, x=4 k+r(k \\in \\mathbf{N}, r=0,1,2,3)$, 定义 $f(x)=r$. 于是, 对任意 $x, y \\in \\mathbf{N}_{+}$, 若当 $|x-y|$ 为素数时有 $f(x)=f(y)$, 则 4||$x-y \\mid$, 这与 $|x-y|$ 为素数矛盾.\n故所作 $f: \\mathbf{N}_{+} \\rightarrow A$ 满足题目条件且 $|A|=4$, 所以 $A$ 中最少有 4 个元素.", + "remark": "", + "figures": [] +} \ No newline at end of file diff --git a/processed_dataset/calculation/0224.json b/processed_dataset/calculation/0224.json new file mode 100644 index 0000000000000000000000000000000000000000..7464a8b5084c148c784008086f558edd64df0fdc --- /dev/null +++ b/processed_dataset/calculation/0224.json @@ -0,0 +1,8 @@ +{ + "source_file": "./raw_volume-zh/volume11/exercise15.tex", + "problem_type": "calculation", + "problem": "问题3. 设 $n$ 为给定的正整数, 求最大正整数 $k$, 使得存在三个由非负整数组成的集合 $A=\\left\\{x_1, x_2, \\cdots, x_k\\right\\}, B=\\left\\{y_1, y_2, \\cdots, y_k\\right\\}, C=\\left\\{z_1, z_2, \\cdots, z_k\\right\\}$ 满足: 对任意 $1 \\leqslant j \\leqslant k$, 都有 $x_j+y_j+z_j=n$.", + "solution": "由已知条件可知 $k n=\\sum_{i=1}^k\\left(x_i+y_i+z_i\\right) \\geqslant 3 \\sum_{i=1}^k(i-1)=\\frac{3 k(k-1)}{2}$, 因此, $k \\leqslant\\left[\\frac{2 n}{3}\\right]+1$. 下面给出一个 $k=\\left[\\frac{2 n}{3}\\right]+1$ 的例子: 令 $m \\in \\mathbf{N}_{+}$, 若 $n= 3 m$, 则 $k=2 m+1$, 对 $1 \\leqslant j \\leqslant m+1$, 令 $x_j=j-1, y_j=m+j-1, z_j= 2 m-2 j+2$; 对 $m+2 \\leqslant j \\leqslant 2 m+1$, 令 $x_j=j-1, y_j=j-m-2, z_j=4 m- 2 j+3$ 即可; 若 $n=3 m+1$, 则 $k=2 m+1$, 对 $1 \\leqslant j \\leqslant m$, 令 $x_j=j-1, y_j= m+j, z_j=2 m-2 j+2$; 对 $m+1 \\leqslant j \\leqslant 2 m$, 令 $x_j=j+1, y_j=j-m-1$, $\\boldsymbol{z}_j=4 m+1-2 j$; 而 $x_{2 m+1}=m, y_{2 m+1}=2 m+1, z_{2 m+1}=0$; 当 $n=3 m+2$ 时, $k=2 m+2$, 对 $1 \\leqslant j \\leqslant m+1$ 令 $x_j=j-1, y_j=m+j, z_j=2 m-2 j+3$ ; 对 $m+2 \\leqslant j \\leqslant 2 m+1$, 令 $x_j=j, y_j=j-m-2, z_j=4 m-2 j+4$, 而 $x_{2 m+2}=2 m+2, y_{2 m+2}=m, z_{2 m+2}=0$ 即可.", + "remark": "", + "figures": [] +} \ No newline at end of file diff --git a/processed_dataset/calculation/0225.json b/processed_dataset/calculation/0225.json new file mode 100644 index 0000000000000000000000000000000000000000..48655b94aad6e311e932ba02a44e9b03a7e814b8 --- /dev/null +++ b/processed_dataset/calculation/0225.json @@ -0,0 +1,8 @@ +{ + "source_file": "./raw_volume-zh/volume11/exercise15.tex", + "problem_type": "calculation", + "problem": "问题4. 在一次由 $n$ 个是非题构成的竞赛中, 有 8 名选手参加.\n已知对任意一对是非题 $(A, B)$ 而言, (称 ( $A, B)$ 为有序对), 恰有两人的答案为 (对, 对); 恰有两人的答案为 (对, 错); 恰有两人的答案为 (错, 对); 恰有两人的答案为 (错、错), 求 $n$ 的最大值,并说明理由.", + "solution": "设 8 名选手为 $p_1, p_2, \\cdots, p_8, n$ 道是非题为 $A_1, A_2, \\cdots, A_n$. 作 $8 \\times n$ 表格,其中第 $i$ 行第 $j$ 列处的数为 $x_{i j}=\\left\\{\\begin{array}{l}1, p_i \\text { 对 } A_j \\text { 的答案是\"对\", } \\\\ 0, p_i \\text { 对 } A_j \\text { 的答案是\"错\" }\\end{array}(i=1, 2, \\cdots, 8 ; j=1,2, \\cdots, n)\\right.$ 于是, 第 $i$ 行各数之和为 $a_i=\\sum_{i=1}^n x_{i j}$ 表示 $p_i$ 对 $A_1, A_2, \\cdots, A_n$ 的答案为 \"对\" 的个数.\n又依题意, 每列中恰有 4 个 1 和 4 个零,所以 $\\sum_{i=1}^8 x_{i j}=4$. 从而 $\\sum_{i=1}^8 a_i=\\sum_{i=1}^8 \\sum_{j=1}^n x_{i j}=\\sum_{j=1}^n \\sum_{i=1}^8 x_{i j}=4 n \\cdots$ (1). \n注意到, 由已知条件知, 对表中任何一列, 将其中 1 全部换为 0 , 并且将 0 全部换为 1 后, 表中各数仍具有题设性质.\n故不失一般性, 可设表中第一行的数全等于 1 , 从而 $a_1=n, \\sum_{i=2}^8 a_i=3 n$. 如果 $p_i$ 对题目 $\\left(A_j, A_k\\right)(j \\neq k)$ 的答案为 (对, 对), 则将 $\\left(p_i, A_j, A_k\\right)$ 组成三元组.\n依题目条件可得这种三元组的个数既为 $\\sum_{i=1}^8 \\mathrm{C}_{a_i}^2$, 又为 $2 \\mathrm{C}_n^2=n(n-1)$. \n于是(并利用(1)和柯西不等式) $n(n-1)=\\sum_{i=1}^8 \\mathrm{C}_{a_i}^2=\\mathrm{C}_{a_1}^2+\\sum_{i=2}^8 \\mathrm{C}_{a_i}^2=\\mathrm{C}_n^2+\\frac{1}{2}\\left(\\sum_{i=2}^8 a_i^2-\\sum_{i=2}^8 a_i\\right) \\geqslant \\frac{1}{2} n(n-1)+\\frac{1}{2}\\left[\\frac{1}{7}\\left(\\sum_{i=2}^8 a_i\\right)^2-\\sum_{i=2}^8 a_i\\right]=\\frac{1}{2} n(n-1)+\\frac{1}{14}\\left[(3 n)^2-7(3 n)\\right]=\\frac{2}{7} n(4 n-7)$, 解得 $n \\leqslant 7$. \n如表中所示例子表明 $n$ 可以等于 7 . 故所求 $n$ 的最值为 7 .\n\\begin{tabular}{|c|c|c|c|c|c|c|c|}\n\\hline & $A_1$ & $A_2$ & $A_3$ & $A_4$ & $A_5$ & $A_6$ & $A_7$ \\\\\n\\hline$p_1$ & 1 & 1 & 1 & 1 & 1 & 1 & 1 \\\\\n\\hline$p_2$ & 1 & 0 & 0 & 0 & 0 & 1 & 1 \\\\\n\\hline$p_3$ & 1 & 0 & 0 & 1 & 1 & 0 & 0 \\\\\n\\hline$p_4$ & 1 & 1 & 1 & 0 & 0 & 0 & 0 \\\\\n\\hline$p_5$ & 0 & 1 & 0 & 1 & 0 & 1 & 0 \\\\\n\\hline$p_6$ & 0 & 1 & 0 & 0 & 1 & 0 & 1 \\\\\n\\hline$p_7$ & 0 & 0 & 1 & 1 & 0 & 0 & 1 \\\\\n\\hline$p_8$ & 0 & 0 & 1 & 0 & 1 & 1 & 0 \\\\\n\\hline\n\\end{tabular}", + "remark": "", + "figures": [] +} \ No newline at end of file diff --git a/processed_dataset/calculation/0226.json b/processed_dataset/calculation/0226.json new file mode 100644 index 0000000000000000000000000000000000000000..5643d521f7826619a13dae6f13813bdc03054aa4 --- /dev/null +++ b/processed_dataset/calculation/0226.json @@ -0,0 +1,8 @@ +{ + "source_file": "./raw_volume-zh/volume11/exercise15.tex", + "problem_type": "calculation", + "problem": "问题5. 在一个圆周上给定 12 个红点, 求 $n$ 的最小值, 使得存在以红点为顶点的 $n$ 个三角形满足: 以红点为端点的每条弦, 都是其中某个三点均为红点的三角形的一条边.", + "solution": "设红点集为 $A=\\left\\{A_1, A_2, \\cdots, A_{12}\\right\\}$, 过 $A_1$ 的弦有 11 条, 而任意含 $A_1$ 的三角形恰含过 $A_1$ 的两条弦,故这 11 条过 $A_1$ 的弦至少要分布于 6 个含顶点 $A_1$ 的三角形中, 同理知, 过点 $A_i(i=2,3, \\cdots, 12)$ 的弦, 也各要分布在 6 个含顶点 $A_i$ 的三角形中, 这样就需要 $12 \\times 6=72$ (个) 三角形, 而每个三角形含有三个顶点, 故都被重复计算了三次.\n因此, 至少需要 $\\frac{72}{3}=24$ 个不同的三角形.\n另一方面, 下面实例表明 $n=24$ 可以被取到.\n不失一般性, 考虑周长为 12 的圆周, 其十二等分点为红点, 以红点为端点的弦共有 $\\mathrm{C}_{12}^2=66$ (条). 如果弦所对的劣弧长为 $k$, 就称该弦的刻度为 $k$, 于是红端点的弦的刻度的弦只有 6 种, 其中刻度为 $1,2,3,4,5$ 的弦各有 12 条, 刻度为 6 的弦有 6 条.\n如果刻度为 $a$, $b, c$ 的三条弦构成一个三角形的三条边, 则必须满足下列两个条件之一:或者 $a+b=c$, 或者 $a+b+c==12$. 下面是刻度组的一种搭配: 以 $(1,2,3),(1,5$, $6),(2,3,5)$ 型各 6 个, $(4,4,4)$ 型 4 个, 这时恰好得 66 条弦, 其中含刻度为 $1,2,3,4,5$ 的弦各 12 条, 刻度为 6 的弦恰有 6 条.\n今构造如下: 先作刻度为 $(1,2,3) ,(1,5,6) ,(2,3,5)$ 型的三角形各 6 个, $(4,4,4)$ 型的三角形三个,再用三个 $(2,4,6)$ 型的三角形来补充.\n刻度为 $(1,2,3)$ 型的 6 个, 其顶点标号为 $\\{2,3,5\\},\\{4,5,7\\},\\{6,7,9\\},\\{8,9,11\\},\\{10,11,1\\},\\{12,1,3\\}$ ; 刻度为 $(1,5,6)$ 型的 6 个, 其顶点标号为 $\\{1,2,7\\},\\{3,4,9\\},\\{5,6,11\\},\\{7$, $8,1\\},\\{9,10,3\\},\\{11,12,5\\}$; 刻度为 $(2,3,5)$ 型的 6 个, 其顶点标号为 $\\{2,4$, $11\\},\\{4,6,1\\},\\{6,8,3\\},\\{8,10,5\\},\\{10,12,7\\},\\{12,2,9\\}$; 刻度为 $(4,4,4)$型的 3 个,其顶点标号为 $\\{1,5,9\\},\\{2,6,10\\},\\{3,7,11\\}$; 刻度为 $(2,4,6)$ 型的 3 个, 其顶点标号为 $\\{4,6,12\\},\\{8,10,4\\},\\{12,2,8\\}$. (注意, 每种情况下的其余三角形都可由前一个三角形绕圆心适当旋转而得到). 这样共得 24 个三角形,且满足题目条件.\n综上可知,所求 $n$ 的最小值为 24 .", + "remark": "", + "figures": [] +} \ No newline at end of file diff --git a/processed_dataset/calculation/0227.json b/processed_dataset/calculation/0227.json new file mode 100644 index 0000000000000000000000000000000000000000..7347dce3844eb4e98ac102939ab44e41bb41de60 --- /dev/null +++ b/processed_dataset/calculation/0227.json @@ -0,0 +1,8 @@ +{ + "source_file": "./raw_volume-zh/volume11/exercise2.tex", + "problem_type": "calculation", + "problem": "问题2. 是否存在 (1) 4 个; (2) 5 个不同的正整数,它们中任意 3 个数之和是素数?", + "solution": "(1) $1,3,7,9$ 满足要求; (2) 考虑任意 5 个数被 3 除的余数,若 0,1 ,\n2 这三种余数都出现, 则这 3 个数之和被 3 整除, 不是素数; 若 $0,1,2$ 这三种余数中至多出现 2 种, 则由抽屈原理知至少有 $\\left[\\frac{5-1}{2}\\right]+1=3$ 个数被 3 除的余数相等, 从而这三个数之和也是 3 的倍数, 不是素数.\n故不存在 5 个不同的正整数,它们中任意三个数之和为素数.", + "remark": "", + "figures": [] +} \ No newline at end of file diff --git a/processed_dataset/calculation/0228.json b/processed_dataset/calculation/0228.json new file mode 100644 index 0000000000000000000000000000000000000000..b44dfeec33dec16872e3f45ed1b6474b5731d60d --- /dev/null +++ b/processed_dataset/calculation/0228.json @@ -0,0 +1,8 @@ +{ + "source_file": "./raw_volume-zh/volume11/exercise2.tex", + "problem_type": "calculation", + "problem": "问题4. 设 $S=\\{1,2,3, \\cdots, 2011\\}$, 问从 $S$ 中最多能选出多少个数,使得其中任何两数之和都不能被它们的差整除?", + "solution": "令 $M=\\{3 k+1 \\mid k=0,1,2, \\cdots, 670\\}$, 则 $M \\varsubsetneqq S$ 且 $|M|=671$, $M$ 中任意两数之差均是 3 的倍数但两数之和则不是 3 的倍数,故 $M$ 中任意两数之和都不被它们的差整除, 所以, 所求的最大值不小于 671 . 其次, 将 $S$ 中的数分为下列 671 个子集: $\\{1,2,3\\},\\{4,5,6\\},\\{7,8,9\\}, \\cdots,\\{2008,2009$, $2010\\},\\{2011\\}$,从 $M$ 中任取 672 个数,其中必有 $\\left[\\frac{672-1}{671}\\right]+1=2$ 个数属于同一组, 这两个数之差不大于 2 , 设这两个数为 $a$ 和 $b$ 且 $a>b$. 若 $a-b=1$, 则显然有 $a+b$ 被 $a-b$ 整除; 若 $a-b=2$, 则 $a$ 与 $b$ 同为奇数或同为偶数,即 $a+b$ 必为偶数, 从而 $a+b$ 也被 $a-b$ 整除.\n综上可知从 $S$ 中最多可选出 671 个数,使其中任意两数之和都不能被它们的差整除.", + "remark": "", + "figures": [] +} \ No newline at end of file diff --git a/processed_dataset/calculation/0229.json b/processed_dataset/calculation/0229.json new file mode 100644 index 0000000000000000000000000000000000000000..e693e13c0491b9366927a39d1679a80acde29fe8 --- /dev/null +++ b/processed_dataset/calculation/0229.json @@ -0,0 +1,8 @@ +{ + "source_file": "./raw_volume-zh/volume11/exercise2.tex", + "problem_type": "calculation", + "problem": "问题6. 有 $n(n>12)$ 个人参加某次数学邀请赛, 试卷由 15 个填空题组成, 每答对 1 题得 1 分, 不答或答错得 0 分.\n分析每一种可能的得分情况, 发现: 只要其中任意 12 个人得分之和不少于 36 分, 则这 $n$ 个人中至少有 3 个人答对了至少 3 个同样的题,求 $n$ 的最小可能值.", + "solution": "注意到 $\\mathrm{C}_{15}^3=455$. 若有 $2 \\mathrm{C}_{15}^3=910$ 名学生参赛, 将这 910 名学生分为 455 组,每组 2 人,并且每组恰答对同样的 3 个题并且不同的组答对的 3 个题不全相同, 此时不满足题设条件,故所求 $n$ 的最小值不小于 911 . 另一方面若至少有 911 名学生参赛, 则只有下列两种情形: (1) 当每个学生至少答对 3 个题时, 而每个学生答对 3 个题的不同情况只有 $\\mathrm{C}_{15}^3=455$ 种, 故由抽屉原理知至少有 $\\left[\\frac{911-1}{455}\\right]+1=3$ 名学生答对了同样的 3 个问题; (2) 当有一名学生 $A$ 答对的题数不多于 2 时, 其他学生中答对不超过 3 个题的学生人数不超过 10 人 (否则他们中 11 人与第一个学生 $A$ 共 12 人的得分总和不多于 $2 \\times 1+3 \\times 11=35<36$, 这与已知条件矛盾). 故至少有 $911-11=900$ 名学生, 他们中每一个至少答对了 4 个问题, 由于 $\\mathrm{C}_4^3=4$, 故由抽居原理知这时至少有 $\\left[\\frac{4 \\times 900-1}{455}\\right]+1=8$ 名学生答对了同样的 3 个问题.\n总之, 当参赛学生人数不少于 911 人中, 其中至少有 3 人答对了至少 3 个同样的问题.\n综上可得所求 $n$ 的最小值为 911 .", + "remark": "", + "figures": [] +} \ No newline at end of file diff --git a/processed_dataset/calculation/0230.json b/processed_dataset/calculation/0230.json new file mode 100644 index 0000000000000000000000000000000000000000..c2e45c786e92a0fba5fc4acd076de6125952adc6 --- /dev/null +++ b/processed_dataset/calculation/0230.json @@ -0,0 +1,8 @@ +{ + "source_file": "./raw_volume-zh/volume11/exercise3.tex", + "problem_type": "calculation", + "problem": "问题1. 试用母函数方法求下列数列的通项 $a_n$.\n(1) $a_0=2, a_1=5, a_{n+2}=3 a_{n+1}-2 a_n(n=0,1,2, \\cdots)$;\n(2) $a_1=3, a_2=15, a_{n+2}=a_{n+1}+6 a_n-12(n=1,2, \\cdots)$.", + "solution": "(1) 设 $f(x)=a_0+a_1 x+a_2 x^2+\\cdots+a_n x^n+\\cdots$, 则\n$$\n\\begin{array}{rr}\n-3 x f(x)= & -3 a_0 x-3 a_1 x^2-\\cdots-3 a_{n-1} x^n-\\cdots, \\\\\n2 x^2 f(x)= & 2 a_0 x^2+\\cdots+2 a_{n-1} x^n+\\cdots .\n\\end{array}\n$$\n三式相加, 并利用 $a_0=2, a_1=5, a_n-3 a_{n-1}+2 a_{n-2}=0(n=2,3, \\cdots)$\n得 $\\left(1-3 x+2 x^2\\right) f(x)=2-x$, 所以 $f(x)=\\frac{2-x}{1-3 x+2 x^2}= \\frac{3(1-x)-(1-2 x)}{(1-x)(1-2 x)}=\\frac{3}{1-2 x}-\\frac{1}{1-x}=3 \\sum_{n=0}^{\\infty}(2 x)^n-\\sum_{n=0}^{\\infty} x^n=\\sum_{n=0}^{\\infty}\\left(3 \\cdot 2^n\\right. -1) x^n$, 所以 $a_n=3 \\cdot 2^n-1$.\n(2) 设 $f(x)=a_0+a_1 x+a_2 x^2+\\cdots+a_n x^n+\\cdots$, 则\n$$\n\\begin{array}{rlrl}\n-x f(x) & = & -a_0 x-a_1 x^2-\\cdots-a_{n-1} x^n-\\cdots, \\\\\n-6 x^2 f(x) & = & -6 a_0 x^2-\\cdots-6 a_{n-2} x^n-\\cdots, \\\\\n\\frac{12}{1-x} & =12+12 x+12 x^2+\\cdots+12 x^n+\\cdots .\n\\end{array}\n$$\n四式相加, 并利用 $a_0=\\frac{1}{6}\\left(a_2-a_1+12\\right)=4, a_1=3, a_n-a_{n-1}-6 a_{n-2}+ 12=0(n=2,3, \\cdots)$ 得 $\\left(1-x-6 x^2\\right) f(x)+\\frac{12}{1-x}=16+11 x, f(x)= \\frac{4-5 x-11 x^2}{\\left(1-x-6 x^2\\right)(1-x)}=\\frac{4-5 x-11 x^2}{(1-x)(1+2 x)(1-3 x)}=\\frac{A}{1-x}+\\frac{B}{1+2 x}+ \\frac{C}{1-3 x}$, 于是 $A=\\left.\\frac{4-5 x-11 x^2}{(1+2 x)(1-3 x)}\\right|_{x=1}=2, B=\\left.\\frac{4-5 x-11 x^2}{(1-x)(1-3 x)}\\right|_{x=-\\frac{1}{2}} =1, C=\\left.\\frac{4-5 x-11 x^2}{(1-x)(1+2 x)}\\right|_{x=\\frac{1}{3}}=1$, 所以 $f(x)=\\frac{2}{1-x}+\\frac{1}{1+2 x}+\\frac{1}{1-3 x}$. $=2 \\sum_{n=0}^{\\infty} x^n+\\sum_{n=0}^{\\infty}(-2 x)^n+\\sum_{n=0}^{\\infty}(3 x)^n=\\sum_{n=0}^{\\infty}\\left(3^n+(-2)^n+2\\right) x^n$, 故得 $a_n=3^n +(-2)^n+2$.", + "remark": "", + "figures": [] +} \ No newline at end of file diff --git a/processed_dataset/calculation/0231.json b/processed_dataset/calculation/0231.json new file mode 100644 index 0000000000000000000000000000000000000000..063c60b722af8d76b1967a8f17fcdd369a4bec7d --- /dev/null +++ b/processed_dataset/calculation/0231.json @@ -0,0 +1,8 @@ +{ + "source_file": "./raw_volume-zh/volume11/exercise3.tex", + "problem_type": "calculation", + "problem": "问题3. 用母函数方法求下列问题的解.\n(1) 求各位数字之和等于 17 的三位正整数的个数;\n(2) 将一张 $n$ 元的纸币全部兑换为 1 元和 2 元的纸币, 问有多少种不同的兑换方法?", + "solution": "(1) 设各位数字之和等于 $n$ 的三位数有 $a_n$ 个, 则 $a_n=\\sum_{a+b+c=n} 1(a, b, c$ 均为整数且 $1 \\leqslant a \\leqslant 9,0 \\leqslant b \\leqslant 9,0 \\leqslant c \\leqslant 9) . a_n$ 的母函数为 $f(x)=\\sum_{n=0}^{\\infty} a_n x^n= \\sum_{n=0}^{\\infty}\\left(\\sum_{a+b+c=n} 1\\right) x^n=\\sum_{a=1}^9 \\sum_{b=0}^9 \\sum_{c=0}^9 x^{a+b+c}=\\left(\\sum_{a=1}^9 x^a\\right)\\left(\\sum_{b=0}^9 x^b\\right)\\left(\\sum_{c=0}^g x^c\\right)= \\frac{x\\left(1-x^9\\right)\\left(1-x^{10}\\right)^2}{(1-x)^3}=\\left(x-x^{10}-2 x^{11}+2 x^{20}+x^{21}-x^{30}\\right)\\left(\\sum_{i=0}^{\\infty} \\mathrm{C}_{i+2}^2 x^i\\right)$. 其中 $x^{17}$ 的系数为 $a_{17}=\\mathrm{C}_{18}^2-\\mathrm{C}_9^2-2 \\mathrm{C}_8^2=61$, 即各位数字之和等于 17 的三位数有 61 个; (2) 类似于 (1) 可知所求方法数为下列多项式中 $x^n$ 的系数 $a_n: f(x)=\\sum_{n=0}^{\\infty} a_n x^n=\\left(\\sum_{a=0}^{\\infty} x^a\\right)\\left(\\sum_{b=0}^{\\infty} x^{2 b}\\right)=\\left(\\frac{1}{1-x}\\right)\\left(\\frac{1}{1-x^2}\\right)= \\frac{1}{(1+x)(1-x)^2}=\\frac{1}{4(1+x)}+\\frac{1}{2(1-x)^2}+\\frac{1}{4(1-x)}=\\frac{1}{4} \\sum_{n=0}^{\\infty}(-1)^n x^n+ \\frac{1}{2} \\sum_{n=0}^{\\infty} \\mathrm{C}_{n+1}^1 x^n+\\frac{1}{4} \\sum_{n=0}^{\\infty} x^n=\\sum_{n=0}^{\\infty}\\left(\\frac{n+1}{2}+\\frac{1+(-1)^n}{4}\\right) x^n$, 故所求兑换方法数为 $a_n=\\frac{n+1}{2}+\\frac{1+(-1)^n}{4}=\\left[\\frac{n+2}{2}\\right]$.", + "remark": "", + "figures": [] +} \ No newline at end of file diff --git a/processed_dataset/calculation/0232.json b/processed_dataset/calculation/0232.json new file mode 100644 index 0000000000000000000000000000000000000000..803f017d818133170f65f4ab9108337acb741941 --- /dev/null +++ b/processed_dataset/calculation/0232.json @@ -0,0 +1,8 @@ +{ + "source_file": "./raw_volume-zh/volume11/exercise4.tex", + "problem_type": "calculation", + "problem": "问题1. 设正数列 $a_0, a_1, \\cdots, a_n, \\cdots$ 满足\n(1) $\\sqrt{a_n a_{n-2}}-\\sqrt{a_{n-1} a_{n-2}}=2 a_{n-1} ;(n \\geqslant 2)$\n(2) $a_0=a_1=1$.\n求 $\\left\\{a_n\\right\\}$ 的通项.", + "solution": "(1) 式两边除以 $\\sqrt{a_{n-1} a_{n-2}}$ 得 $\\sqrt{\\frac{a_n}{a_{n-1}}}=1+2 \\sqrt{\\frac{a_{n-1}}{a_{n-2}}}$, 令 $b_n= \\sqrt{\\frac{a_n}{a_{n-1}}}(n \\geqslant 1)$, 则 $b_n=1+2 b_{n-1}$, 即 $b_n+1=2\\left(b_{n-1}+1\\right)$. 从而 $b_n+1=\\left(b_1+\\right.$ 1) $\\cdot 2^{n-1}=2^n$, 即 $\\frac{a_n}{a_{n-1}}=\\left(2^n-1\\right)^2$. 所以 $a_n=\\frac{a_n}{a_{n-1}} \\cdot \\frac{a_{n-1}}{a_{n-2}} \\cdots \\frac{a_2}{a_1} \\cdot a_1=\\left(2^n-\\right. 1)^2\\left(2^{n-1}-1\\right)^2 \\cdots\\left(2^2-1\\right)^2 \\cdot\\left(2^1-1\\right)$.", + "remark": "", + "figures": [] +} \ No newline at end of file diff --git a/processed_dataset/calculation/0233.json b/processed_dataset/calculation/0233.json new file mode 100644 index 0000000000000000000000000000000000000000..215ac44b93bfe68a4a211753249bfa1785a87cd2 --- /dev/null +++ b/processed_dataset/calculation/0233.json @@ -0,0 +1,8 @@ +{ + "source_file": "./raw_volume-zh/volume11/exercise4.tex", + "problem_type": "calculation", + "problem": "问题2. 试确定实数 $a_0$, 使得由递推关系 $a_{n+1}=-3 a_n+2^n(n=0,1,2, \\cdots)$ 决定的数列 $\\left\\{a_n\\right\\}$ 严格递增, 即对 $n \\geqslant 0, a_{n+1}>a_n$.", + "solution": "两边除以 $2^{n+1}$ 得 $\\frac{a_{n+1}}{2^{n+1}}=-\\frac{3}{2} \\frac{a_n}{2^n}+\\frac{1}{2}$, 令 $b_n=\\frac{a_n}{2^n}$, 则 $b_{n+1}=-\\frac{3}{2} b_n+\\frac{1}{2}$, 即 $b_{n+1}-\\frac{1}{5}=-\\frac{3}{2}\\left(b_n-\\frac{1}{5}\\right)$ (注意, 这是 $\\frac{1}{5}$ 是 $f(x)=-\\frac{3}{2} x+\\frac{1}{2}$ 的不动点, 即方程 $f(x)=x$ 的根). 于是 $b_n-\\frac{1}{5}=\\left(b_0-\\frac{1}{5}\\right)\\left(-\\frac{3}{2}\\right)^n=\\left(a_0-\\frac{1}{5}\\right)\\left(-\\frac{3}{2}\\right)^n$, 所以 $a_n=2^n b_n=2^n\\left[\\left(a_0-\\frac{1}{5}\\right)\\left(-\\frac{3}{2}\\right)^n+\\frac{1}{5}\\right]==(-3)^n\\left[\\left(a_0-\\frac{1}{5}\\right)+\\frac{1}{5}\\left(-\\frac{2}{3}\\right)^n\\right]$. \n因为当 $n$ 足够大时, $\\left(\\frac{2}{3}\\right)^n$ 趋于 0 ,于是, 若 $a_0-\\frac{1}{5} \\neq 0$, 则上式中括号内的数在 $n$ 足够大时与 $a_0-\\frac{1}{5}$ 同号, 但 $(-3)^n(n=0,1,2, \\cdots)$ 轮流为正、负数, 从而不可能严格递增, 因此, 当且仅当 $a_0-\\frac{1}{5}=0$, 即 $a_0=\\frac{1}{5}$ 时, $a_n=\\frac{2^n}{5}$ 严格递增.", + "remark": "", + "figures": [] +} \ No newline at end of file diff --git a/processed_dataset/calculation/0234.json b/processed_dataset/calculation/0234.json new file mode 100644 index 0000000000000000000000000000000000000000..4e073f0469555b8a3d76e681ce2b22590b032960 --- /dev/null +++ b/processed_dataset/calculation/0234.json @@ -0,0 +1,8 @@ +{ + "source_file": "./raw_volume-zh/volume11/exercise4.tex", + "problem_type": "calculation", + "problem": "问题3. 设数列 $a_1, a_2, \\cdots, a_n, \\cdots$ 满足 $a_1=\\frac{1}{2}, a_1+a_2+\\cdots+a_n=n^2 a_n(n \\geqslant 1)$, 确定 $a_n(n \\geqslant 1)$ 的值.", + "solution": "令 $S_n=a_1+a_2+\\cdots+a_n$, 则 $a_n=S_n-S_{n-1}=n^2 a_n-(n-1)^2 a_{n-1}$, 解得 $a_n=\\frac{n-1}{n+1} a_{n-1}$. 所以 $a_n=\\frac{a_n}{a_{n-1}} \\cdot \\frac{a_{n-1}}{a_{n-2}} \\cdots \\frac{a_2}{a_1} \\cdot a_1=\\frac{n-1}{n+1} \\cdot \\frac{n-2}{n} \\cdot \\frac{n-3}{n-1} \\cdots \\frac{1}{3} \\cdot \\frac{1}{2}= \\frac{1}{n(n+1)}$.", + "remark": "", + "figures": [] +} \ No newline at end of file diff --git a/processed_dataset/calculation/0235.json b/processed_dataset/calculation/0235.json new file mode 100644 index 0000000000000000000000000000000000000000..d08bb25fa5202e9615fd5ec3a24857b11f918fd7 --- /dev/null +++ b/processed_dataset/calculation/0235.json @@ -0,0 +1,8 @@ +{ + "source_file": "./raw_volume-zh/volume11/exercise4.tex", + "problem_type": "calculation", + "problem": "问题5. 用 $n$ 块 $1 \\times 2$ 的纸片覆盖一个 $2 \\times n$ 的棋盘(没有重叠也没有空隙)共有多少种不同的方法?", + "solution": "设共有 $a_n$ 种不同的覆盖方法, 显然 $a_1=1, a_2=2$. 对于 $2 \\times n$ 的棋盘, 若一开始用一张 $1 \\times 2$ 的纸片坚方向盖住最左边一列的 2 格, 则剩下的 $2 \\times (n-1)$ 个方格有 $a_{n-1}$ 种覆盖方法, 若一开始用 2 张 $1 \\times 2$ 的纸片横方向盖住最左边的 $2 \\times 2$ 的方格, 则剩下 $2 \\times(n-2)$ 个方格有 $a_{n-2}$ 种覆盖方法.\n所以 $a_n= a_{n-1}+a_{n-2}(n \\geqslant 3)$. 可见 $\\left\\{a_n\\right\\}$ 为斐波那契数列, 由例 6 得 $a_n= \\frac{1}{\\sqrt{5}}\\left[\\left(\\frac{1+\\sqrt{5}}{2}\\right)^{n+1}-\\left(\\frac{1-\\sqrt{5}}{2}\\right)^{n+1}\\right](n \\geqslant 1)$.", + "remark": "", + "figures": [] +} \ No newline at end of file diff --git a/processed_dataset/calculation/0236.json b/processed_dataset/calculation/0236.json new file mode 100644 index 0000000000000000000000000000000000000000..7b21d920d962312564d55377827a4d5952ca48fb --- /dev/null +++ b/processed_dataset/calculation/0236.json @@ -0,0 +1,8 @@ +{ + "source_file": "./raw_volume-zh/volume11/exercise4.tex", + "problem_type": "calculation", + "problem": "问题6. 球面上有 $n$ 个大圆, 其中没有 3 个大圆通过同一点,求这 $n$ 个大圆将球面分成的区域的个数 $a_n$.", + "solution": "显然 $a_1=2$, 球面上 $n-1$ 个大圆将球面分成 $a_{n-1}$ 个区域,再加上第 $n$ 个大圆, 它同前 $n-1$ 个大圆无三圆交于一点, 故有 $2(n-1)$ 个不同的交点, 将第 $n$ 个圆分成 $2(n-1)$ 段弧, 每段弧将原有区域一分为二, 故增加了 $2(n-1)$ 个区域, 故 $a_n=a_{n-1}+2(n-1)$, 所以 $a_n=a_1+\\sum_{k=2}^n\\left(a_k-a_{k-1}\\right)=2+ \\sum_{k=2}^n 2(k-1)=2+2 \\cdot \\frac{n(n-1)}{2}=n^2-n+2$.", + "remark": "", + "figures": [] +} \ No newline at end of file diff --git a/processed_dataset/calculation/0237.json b/processed_dataset/calculation/0237.json new file mode 100644 index 0000000000000000000000000000000000000000..5c416e6eb3d36f6fc75397bbb3393bc0f41da8d7 --- /dev/null +++ b/processed_dataset/calculation/0237.json @@ -0,0 +1,8 @@ +{ + "source_file": "./raw_volume-zh/volume11/exercise4.tex", + "problem_type": "calculation", + "problem": "问题7. 用 $1,2,3,4$ 可组成多少个含偶数个 1 的 $n$ 位数?", + "solution": "首位数字为 1 时, 余下 $n-1$ 位数是含奇数个 1 的 $n-1$ 位数有 $4^{n-1}- a_{n-1}$ 个; 首位数字不为 1 时,首位数字有 3 种不同取法余下 $n-1$ 余数字有 $a_{n-1}$ 种取法, 这样的 $n$ 位数有 $3 a_{n-1}$ 个, 故 $a_n=4^{n-1}-a_{n-1}+3 a_{n-1}=2 a_{n-1}+4^{n-1}$ 并且显然 $a_1=3$. 于是 $\\frac{a_n}{4^n}=\\frac{1}{2} \\frac{a_{n-1}}{4^{n-1}}+\\frac{1}{4}$, 令 $b_n=\\frac{a_n}{4^n}$ 则 $b_n=\\frac{1}{2} b_{n-1}+\\frac{1}{4}$, 即 $b_n- \\frac{1}{2}=\\frac{1}{2}\\left(b_{n-1}-\\frac{1}{2}\\right)$, 所以 $b_n-\\frac{1}{2}=\\left(b_1-\\frac{1}{2}\\right)\\left(\\frac{1}{2}\\right)^{n-1}=\\left(\\frac{3}{4}-\\frac{1}{2}\\right)\\left(\\frac{1}{2}\\right)^{n-1}= \\frac{1}{2^{n+1}}$, 故 $a_n=4^n b_n=4^n\\left(\\frac{1}{2^{n+1}}+\\frac{1}{2}\\right)=\\frac{1}{2}\\left(2^n+4^n\\right)$.", + "remark": "", + "figures": [] +} \ No newline at end of file diff --git a/processed_dataset/calculation/0238.json b/processed_dataset/calculation/0238.json new file mode 100644 index 0000000000000000000000000000000000000000..5b009a076f2b2e1241480c597d2242b61701b5cc --- /dev/null +++ b/processed_dataset/calculation/0238.json @@ -0,0 +1,8 @@ +{ + "source_file": "./raw_volume-zh/volume11/exercise5.tex", + "problem_type": "calculation", + "problem": "问题1. 如果 (1) $a, b, c, d$ 都属于 $\\{1,2,3,4\\} ;$; (2) $a \\neq b, b \\neq c, c \\neq d, d \\neq a$; (3) $a$ 是 $a, b, c, d$ 中最小值, 那么可组成不同的四位数 $\\overline{a b c d}$ 的个数是 .", + "solution": "$\\overline{a b c d}$ 中恰有 2 个不同数字时, 从 4 个数字中取 2 个数字有 $\\mathrm{C}_4^2$ 种方法, 其中较小的数字放在第 1,3 位, 较大的数字放在第 2,4 位, 只能组成一个四位数, 故这时不同的四位数有 $\\mathrm{C}_4^2=6$ 个; $\\overline{a b c d}$ 中恰有 3 个不同数字时, 从 4 个数字中取 3 个数字有 $\\mathrm{C}_4^3$ 种取法.\n组成第 1,3 位数字相同的四位数有 $\\mathrm{A}_2^2$ 个,组成第 2,4 位数字相同的四位数也有 $\\mathrm{A}_2^2$ 个, 故这时不同的四位数有 $\\mathrm{C}_4^3\\left(\\mathrm{~A}_2^2+\\mathrm{A}_2^2\\right)=16$ 个; $\\overline{a b c d}$ 中恰有 4 个数字不同时, 取最小的数字放在第 1 位,其余 3 个数字任意排列, 这时不同的四位数有 $\\mathrm{A}_3^3=6$ 个.\n综上可得不同的四位数的个数为 $6+16+6=28$ 个.", + "remark": "", + "figures": [] +} \ No newline at end of file diff --git a/processed_dataset/calculation/0239.json b/processed_dataset/calculation/0239.json new file mode 100644 index 0000000000000000000000000000000000000000..28ab56802f1ea4d599b7c594d12585917c4bcf76 --- /dev/null +++ b/processed_dataset/calculation/0239.json @@ -0,0 +1,8 @@ +{ + "source_file": "./raw_volume-zh/volume11/exercise5.tex", + "problem_type": "calculation", + "problem": "问题2. 由 $1,2,3,4,5,6$ 组成的, 至少有三个数位上的数码不同的 5 位数中, 有多少个数使得数码 1 和 6 不相邻?", + "solution": "设 $S$ 为由 $1,2,3,4,5,6$ 组成的 5 位数集合, $A$ 是其中至多由两个不同数码组成的 5 位数构成的集合, $B$ 为其中 1,6 相邻的 5 位数构成的集合.\n于是所求 5 位数的个数为 $m=\\left|\\complement_S A \\cap \\complement_S B\\right|=|S|-|A|-|B|+|A \\cap B|$, 其中 $|S|=6^5,|A|=6+\\mathrm{C}_6^2\\left(\\mathrm{C}_5^1+\\mathrm{C}_5^2+\\mathrm{C}_5^3+\\mathrm{C}_5^4\\right)=456,|A \\cap B|=\\mathrm{C}_5^1+ \\mathrm{C}_5^2+\\mathrm{C}_5^3+\\mathrm{C}_5^4=30$, 设 $b_n$ 为 1,6 相邻的 $n$ 位数的个数, $c_n$ 为首位为 1 或 6 且 1,6 相邻的 $n$ 位数个数, 则 $b_n=c_n+4 b_{n-1}$. 再根据前二位为 16 或 61 ; 前二位为 11 或 66 ; 第一位为 1 或 6 第二位不为 1,6 , 三种情形得 $c_n=2 \\times 6^{n-2}+ c_{n-1}+8 b_{n-2}$, 代入上式得 $b_n=2 \\times 6^{n-2}+4 b_{n-2}+5 b_{n-1}$, 结合 $b_1=0, b_2=2$. 可算出 $|B|=b_5=1470$, 故所求 5 位数的个数为 $m=6^5-456-1470+30=$ 5880 个.", + "remark": "", + "figures": [] +} \ No newline at end of file diff --git a/processed_dataset/calculation/0240.json b/processed_dataset/calculation/0240.json new file mode 100644 index 0000000000000000000000000000000000000000..ae578821170f0045f917b7f96b8b514488bd841b --- /dev/null +++ b/processed_dataset/calculation/0240.json @@ -0,0 +1,8 @@ +{ + "source_file": "./raw_volume-zh/volume11/exercise5.tex", + "problem_type": "calculation", + "problem": "问题3. 将 2 个 $a$ 和 2 个 $b$ 共 4 个字母填在如图所示的 16 个方格内, 每个小方格内至多填 1 个字母,若使相同字母既不同行也不同列, 则不同的填法共有?种.\n(用数字作答)", + "solution": "使 2 个 $a$ 既不同行也不同列的填法有 $\\mathrm{C}_4^2 \\mathrm{~A}_4^2=72$ 种, 同理,使 2 个 $b$ 既不同行也不同列的填法也有 $\\mathrm{C}_4^2 \\mathrm{~A}_4^2=72$ 种, 故由乘法原理, 这样的填法共有 $72^2$ 种, 其中不符合要求的填法有两种情况: 2 个 $a$ 所在方格内都填有 $b$ 的情况有 72 种; 2 个 $a$ 所在方格内仅有 1 个方格内填有 $b$ 的情况有 $\\mathrm{C}_{16}^1 \\cdot \\mathrm{A}_9^2=16 \\times 72$ 种.\n所以符合题设条件的填法共有 $72^2-72-16 \\times 72=3960$ 种.", + "remark": "", + "figures": [] +} \ No newline at end of file diff --git a/processed_dataset/calculation/0241.json b/processed_dataset/calculation/0241.json new file mode 100644 index 0000000000000000000000000000000000000000..0abcb73e5f38aecdd85c47d3a29b08d960356b47 --- /dev/null +++ b/processed_dataset/calculation/0241.json @@ -0,0 +1,8 @@ +{ + "source_file": "./raw_volume-zh/volume11/exercise5.tex", + "problem_type": "calculation", + "problem": "问题5. 从集合 $S=\\{1,2, \\cdots, n\\}\\left(n \\in \\mathbf{N}_{+}\\right)$中先取出子集 $X$, 再取出子集 $Y$, 使 $X$ 不是 $Y$ 的子集, 且 $Y$ 也不是 $X$ 的子集, 问这种有序选取有多少种不同的方法?", + "solution": "由于 $S$ 中有 $2^n$ 个子集,故当 $X \\neq Y$ 时, 从 $S$ 中有序选取两个不同子集 $X$ 和 $Y$ 有 $\\mathrm{A}_{2^n}^2=2^n\\left(2^n-1\\right)$ 种方法.\n从这总的选法中减去 $X \\varsubsetneqq Y$ 和 $Y \\varsubsetneqq X$ 的情况, 即为所求的数 $m$. 当 $X \\varsubsetneqq Y$ 时, 设 $|Y|=i(1 \\leqslant i \\leqslant n)$, 则 $Y$ 有 $\\mathrm{C}_n^i$ 种取法,而 $X$ 是 $Y$ 的真子集, $X$ 有 $\\left(2^i-1\\right)$ 种取法, 故 $X \\varsubsetneqq Y$ 的取法种数为 $\\sum_{i=1}^n \\mathrm{C}_n^i\\left(2^i-1\\right)=\\sum_{i=0}^n \\mathrm{C}_n^i\\left(2^i-1\\right)=\\sum_{i=0}^n \\mathrm{C}_n^i 2^i-\\sum_{i=0}^n \\mathrm{C}_n^i=3^n-2^n$, 由对称性, $Y \\varsubsetneqq X$ 的取法也有 $3^n-2^n$ 种, 故得 $m=2^n\\left(2^n-1\\right)-2\\left(3^n-2^n\\right)=2^{2 n}-2 \\cdot 3^n+2^n$.", + "remark": "", + "figures": [] +} \ No newline at end of file diff --git a/processed_dataset/calculation/0242.json b/processed_dataset/calculation/0242.json new file mode 100644 index 0000000000000000000000000000000000000000..1507e740c25de2bb5cb2b72166858e320cbc4432 --- /dev/null +++ b/processed_dataset/calculation/0242.json @@ -0,0 +1,10 @@ +{ + "source_file": "./raw_volume-zh/volume11/exercise5.tex", + "problem_type": "calculation", + "problem": "问题7. 某协会共有 $n$ 个人,已知其中任意 3 人中必有两人互相认识, 求最小正整数 $n$ 使得其中必存在 4 人互相都认识.", + "solution": "如图()所示, 用 $n$ 个点(其中任意 4 点不共面)表示 $n$ 个人, 若两人互相不认识, 则对应两点连实线, 否则连虚线, 得到图 $G$. 于是图中不存在三边为实线的三角形,要求最小正整数 $n$ 使图中存在 4 点, 每两点间连有虚线.\n首先, 如图 8 个点, 每两点间连有实线或虚线, 图中既不存在实线三角形, 也不存在 4 点, 每两点间连有虚线.\n若从中去掉一些点以及从该点出发的所有实线和 虚线, 图中更不存在实线三角形, 也不存在 4 点, 每两点间连有虚线, 故所求最小正整数 $n \\geqslant 9$. 当 $n=9$ 时, 分为下列 3 种情形:\n(1) 存在一点 $A_1$, 从 $A_1$ 出发至少有 4 条实线, 不妨设 $A_1 A_2, A_1 A_3$, $A_1 A_4, A_1 A_5$ 为实线.\n由已知条件知 $A_2, A_3, A_4, A_5$ 任意两点间的连线不能为实线而只能为虚线, 结论成立; \n(2) 存在一点 $A_1$, 从 $A_1$ 出发至多有 2 条实线, 从而从 $A_1$ 出发至少有 6 条虚线.\n考虑以这 6 条虚线另一端为顶点的图, 由 Ramsey 定理(第二章例 1) 知其中必存在一个三边同为实线或同为虚线的三角形, 而由已知条件知不存在实线三角形,故必存在虚线三角形, 设为 $\\triangle A_2 A_3 A_4$. 于是存在 4 点 $A_1, A_2, A_3, A_4$ 使得其中每两点间的连线为虚线,结论也成立; (3) 从每点出发都恰有 3 条实线, 于是图中的实线数为 $\\frac{1}{2} \\times 9 \\times 3=\\frac{27}{2}$, 这不是整数,矛盾, 即情形 (3)不可能出现.\n综上得所求 $n$ 的最小值为 9 .", + "remark": "", + "figures": [ + "./images/volume11/figures/fig-c5a7.png" + ] +} \ No newline at end of file diff --git a/processed_dataset/calculation/0243.json b/processed_dataset/calculation/0243.json new file mode 100644 index 0000000000000000000000000000000000000000..5993a36486ade0daacff6edb8fe69026e7448026 --- /dev/null +++ b/processed_dataset/calculation/0243.json @@ -0,0 +1,8 @@ +{ + "source_file": "./raw_volume-zh/volume11/exercise6.tex", + "problem_type": "calculation", + "problem": "问题1. 从 $\\{1,2, \\cdots, 100\\}$ 中任取 55 个不同的数, 问其中是否必有两个数, 使得这两个数之差等于: (a)9; (b)11?", + "solution": "考察下列 91 个数对: $\\{i, i+9\\}, i=1,2,3, \\cdots, 91 \\cdots$ (1). 易见, 1,2 , $\\cdots, 9$ 和 $92,93, \\cdots, 100$ 这 18 个数中每个数恰属于上述数对中之一对, 而其余 82 个数中每个数恰属于上述数对中两对.\n因此 55 个数至少属于(1)中 $18+$ (55-18) $\\times 2=92$ 个数对, 由抽庶原理知其中必有两个数属于(1)中同一数对, 即两数之差等于 9 . 而当所取 55 个数为 $1,2, \\cdots, 11 ; 23,24, \\cdots, 33 ; 45,46$, $\\cdots, 55 ; 67,68, \\cdots, 77 ; 89,90, \\cdots, 99$ 时,其中任何两数之差不等于 11.", + "remark": "", + "figures": [] +} \ No newline at end of file diff --git a/processed_dataset/calculation/0244.json b/processed_dataset/calculation/0244.json new file mode 100644 index 0000000000000000000000000000000000000000..c74f196a3e466a32c6ce91d1019169c56568275c --- /dev/null +++ b/processed_dataset/calculation/0244.json @@ -0,0 +1,8 @@ +{ + "source_file": "./raw_volume-zh/volume11/exercise6.tex", + "problem_type": "calculation", + "problem": "问题2. 设 $n \\equiv 1(\\bmod 4)$ 且 $n>1, P=\\left\\{a_1, a_2, \\cdots, a_n\\right\\}$ 是 $\\{1,2,3, \\cdots, n\\}$ 的任意排列, $k_P$ 表示使下列不等式成立的最大下标 $k$ :\n$$\na_1+a_2+\\cdots+a_ka_{k_{\\mathrm{p}}+2}+a_{k_{\\mathrm{P}}+3}+\\cdots+a_n \\cdots$ (3). \n于是对于 $P$ 的反序排列 $P^{\\prime}=\\left\\{a_n, a_{n-1}\\right.$, $\\left.\\cdots, a_2, a_1\\right\\}$. \n由(1)及(3)得 $k_{P^{\\prime}}=n-\\left(k_P+1\\right)$, 即 $k_P+k_{P^{\\prime}}=n-1$. \n将 $\\{1,2, \\cdots$, $n\\}$ 的 $n$ ! 个排列两两配对, 每对中两个排列 $P$ 与 $P^{\\prime}$ 互为反序, 它们对应的 $k_P$ 与 $k_{P^{\\prime}}$ 之和为 $n-1$, 又一共可配成 $\\frac{n !}{2}$ 对, \n故对一切不同的排列 $P$, 所对应的 $k_P$ 之和为 $\\frac{1}{2}(n-1)(n !)$.", + "remark": "", + "figures": [] +} \ No newline at end of file diff --git a/processed_dataset/calculation/0245.json b/processed_dataset/calculation/0245.json new file mode 100644 index 0000000000000000000000000000000000000000..db3f529432e1c0c4aa166858e9c036fbf764453c --- /dev/null +++ b/processed_dataset/calculation/0245.json @@ -0,0 +1,8 @@ +{ + "source_file": "./raw_volume-zh/volume11/exercise6.tex", + "problem_type": "calculation", + "problem": "问题5. 在 $m \\times n$ 棋盘的一个格子中放一枚棋子, 两人轮流走, 每步可将棋子从一格走到与它有公共边的邻格中, 但已经走过的格子不能第二次进人, 谁最后没处可走谁输.\n(1)若开始棋子放在左下角格子中,则谁有必胜策略?\n(2)若开始的棋子放在左下角格子的邻格中,则谁有必胜策略?", + "solution": "(1) 明显的配对方法是将相邻两格配对, 即用 $1 \\times 2$ 的骨牌铺砌棋盘.\n保持每步可在同一骨牌内部走的人必胜.\n当 $2 \\mid m n$ 时, 骨牌可铺满棋盘, 先走的甲第一步可在同一块骨牌内部走,故甲必胜.\n当 $2 \\times m n$ 时, 可用骨牌铺满除左下角格子外的其余棋盘, 故先走的甲第一步只能走人相邻的一块骨牌内, 乙总可以在甲进人的这块骨牌内再走一格,故乙有必胜策略;\n(2)无论 $m, n$ 的奇偶性如何, 总是先走的甲有必胜策略.\n当 $2 \\mid m n$ 时, 理由同 (1); 当 $2 \\times m n$ 时, 仍用骨牌铺满除左下角格子以外的其余棋盘, 并且将所有棋盘用黑白两色染色, 使任何相邻两格不同色, 且不妨设左下角格子是白色, 它没有同其他格子配对, 先走的甲每一步总可以在同一骨牌内从黑格走人白格, 而乙只能从白格走人黑格而进人一块的新的骨牌, 左下角的方格始终没有棋子进人,形同虚设,故甲有必胜策略.", + "remark": "", + "figures": [] +} \ No newline at end of file diff --git a/processed_dataset/calculation/0246.json b/processed_dataset/calculation/0246.json new file mode 100644 index 0000000000000000000000000000000000000000..b311bdb525f4ac3b4c5887ed74c6d9d86fb18ee9 --- /dev/null +++ b/processed_dataset/calculation/0246.json @@ -0,0 +1,8 @@ +{ + "source_file": "./raw_volume-zh/volume11/exercise6.tex", + "problem_type": "calculation", + "problem": "问题6. 以凸 $n$ 边形的顶点为顶点, 对角线为边的凸 $k$ 边形共有多少个?", + "solution": "设凸 $n$ 边形为 $A_1 A_2 \\cdots A_n$, 符合条件的一个凸 $k$ 边形为 $A_{i_1} A_{i_2} \\cdots A_{i_k}$, 则只有下列两种可能 $(1) i_1=1,3 \\leqslant i_2b>d$, 由 $a+c=b+d$ 得 $d>c$. 考虑从 $1,2, \\cdots, n$ 中选 3 个数 $a>b>c$ 且满足 $a+c-b \\neq b$ 的选法: 从 $n$ 个数中选三个数 $a> b>c$ 有 $\\mathrm{C}_n^3$ 种选法, 其中满足 $a+c-b=b$, 即 $a+c=2 b$ 的, $a$ 与 $c$ 同奇偶的选法有 $2 \\mathrm{C}_{\\frac{\\pi}{2}}^2$ 种.\n故三元数组 $B=\\{(a, b, c) \\mid n \\geqslant a>b>c \\geqslant 1, a+c-b \\neq b\\}$ 的个数为 $|B|=\\mathrm{C}_n^3-2 \\mathrm{C}_{\\frac{n}{2}}^2=\\frac{1}{12} n(n-2)(2 n-5)$. 对任意三元数组 $(a, b$, $c) \\in B$, 令 $a+c-b=d$, 则四元组 $(a, b, c, d)$ 满足题目条件.\n但每个四元数组 $(a, b, c, d)$ 对应两个三元数组 $(a, b, c)$ 与 $(a, d, c)$ (因为 $d>c$ ). 故所求的选取方法有 $\\frac{1}{2}|B|=\\frac{1}{24} n(n-2)(2 n-5)$ 种.", + "remark": "", + "figures": [] +} \ No newline at end of file diff --git a/processed_dataset/calculation/0249.json b/processed_dataset/calculation/0249.json new file mode 100644 index 0000000000000000000000000000000000000000..e93edd6e84cf33d20a841ddf9c92df6291ea0353 --- /dev/null +++ b/processed_dataset/calculation/0249.json @@ -0,0 +1,8 @@ +{ + "source_file": "./raw_volume-zh/volume11/exercise7.tex", + "problem_type": "calculation", + "problem": "问题1. 两个高一的学生被允许参加高二学生的象棋比赛, 参加比赛的每个选手都同其他选手比赛一次,胜得一分, 和得半分, 负得零分.\n两个高一的学生一共得 8 分, 每个高二的学生都和他的同年级同学得分相同.\n问有几个高二的学生参加比赛? 他们每人得几分?", + "solution": "设高二的学生有 $n$ 人, 每人得 $k$ 分, 于是所有学生得分的总和为 $8+ n k$, 另一方面, 一共比赛了 $\\mathrm{C}_{n+2}^2$ 场, 每场队员得分的总和为 1 分, 所有学生得分的总和为 $\\mathrm{C}_{n+2}^2$, 所以 $\\mathrm{C}_{n+2}^2=8+n k$, 即 $n^2-(2 k-3) n-14=0$, 因此, 14 应被 $n$ 整除, 于是 $n=1,2,7$ 或 14 . 但 $n=1,2$ 时, $k<0$, 不可能; $n=$ 7 时, $k=4, n=14$ 时, $k=8$ 均可实现.\n故参赛的高二学生有 7 人, 每人得 4 分或有 14 人, 每人得 8 分.", + "remark": "", + "figures": [] +} \ No newline at end of file diff --git a/processed_dataset/calculation/0250.json b/processed_dataset/calculation/0250.json new file mode 100644 index 0000000000000000000000000000000000000000..117edd6d261e9823f82d96d4cd948bbe46976611 --- /dev/null +++ b/processed_dataset/calculation/0250.json @@ -0,0 +1,8 @@ +{ + "source_file": "./raw_volume-zh/volume11/exercise8.tex", + "problem_type": "calculation", + "problem": "问题2. 设 $n \\geqslant 2$ 为正整数, 问存在多少个 $\\{1,2, \\cdots, n\\}$ 的排列 $\\left\\{a_1, a_2, \\cdots, a_n\\right\\}$ 满足 $i$ 整除 $a_i-a_{i+1}(i=1,2, \\cdots, n-1)$, 并求出一切这种排列.", + "solution": "将符合题目条件的排列 $\\left\\{a_1, a_2, \\cdots, a_n\\right\\}$ 称为长度等于 $n$ 的好排列, 并设其个数为 $S_n$. 因为 $n-1 \\mid a_{n-1}-a_n$ 又 $\\left|a_{n-1}-a_n\\right| \\leqslant n-1$, 故只有 $\\left|a_{n-1}-a_n\\right|=n-1$,\n所以 $\\left(a_{n-1}, a_n\\right)$ 只可能为 $(1, n)$ 或 $(n, 1)$. 当 $\\left(a_{n-1}, a_n\\right)=(1, n)$ 时, $\\left(a_1, a_2, \\cdots\\right.$, $\\left.a_{n-1}\\right)$ 是长度等于 $n-1$ 的好排列; 当 $\\left(a_{n-1}, a_n\\right)=(n, 1)$ 时, $\\left(a_1-1, a_2-1, \\cdots\\right.$, $\\left.a_{n-1}-1\\right)$ 是长度为 $n-1$ 的好排列, 可见每一个长度为 $n$ 的好排列对应于一个长度为 $n-1$ 的好排列, 反之亦然, 上述对应是一一对应, 所以 $S_n=S_{n-1}$. 而 $S_2=2$ (因 $\\{1,2\\}$ 的好排列有 2 个: $(1,2)$ 和 $(2,1))$, 故对一切 $n \\in \\mathbf{N}_{+}, n \\geqslant 2$ 符合条件的排列个数都为 $S_n=2$. 当 $n=2 m$ 时长为 $2 m$ 的好排列有下列 2 个: $(m, m+1, m-1$, $m+2, m-2, m+3, \\cdots, 2,2 m-1,1,2 m)$ 与 $(m+1, m, m+2, m-1, m+3$, $m-2, \\cdots, 2 m-1,2,2 m, 1)$. 当 $n=2 m+1$ 时长为 $2 m+1$ 的好排列有下列 2 个: $(m+1, m, m+2, m-1, m+3, m-2, \\cdots, 2,2 m, 1,2 m+1)$ 与 $(m+1, m+ 2, m, m+3, m-1, m+4, \\cdots, 2 m, 2,2 m+1,1)$.", + "remark": "", + "figures": [] +} \ No newline at end of file diff --git a/processed_dataset/calculation/0251.json b/processed_dataset/calculation/0251.json new file mode 100644 index 0000000000000000000000000000000000000000..1b9e6707e9ea7228bda503b3923cf61601ba56ee --- /dev/null +++ b/processed_dataset/calculation/0251.json @@ -0,0 +1,8 @@ +{ + "source_file": "./raw_volume-zh/volume11/exercise8.tex", + "problem_type": "calculation", + "problem": "问题3. 各项的值都等于 0 或 1 的数列称为 0,1 数列, 设 $A$ 是一个有限的 0,1 数列, $f(A)$ 表示在 $A$ 中把每个 1 都改为 0,1 , 每个 0 都改为 1,0 所得到的 0,1 数列, 例如\n$$\nf((1,0,0,1))=(0,1,1,0,1,0,0,1)\n$$\n试问 $f^{(n)}((1))$ 中连续两项是 0,0 的数对有多少个? 这里 $f^{(1)}(A)= f(A), f^{(k)}(A)=f\\left(f^{(k-1)}(A)\\right), k=2,3, \\cdots$.", + "solution": "记 $f^{(n)}((1))$ 为 $A_n$, 且 $A_n$ 中连续两项是 0,0 的数对个数为 $b_n$, 连续两项是 0,1 的数对个数记为 $c_n$. 依题意, $A_n$ 中的 0,0 数对仅能由 $A_{n-1}$ 中的 0,1 数对经变换 $f$ 而得到, 故 $b_n=c_{n-1}$. 又 $A_{n-1}$ 中的 0,1 数对, 必须由 $A_{n-2}$ 中的 1 或 0 , 0 数对经变换 $f$ 而得到.\n因为 $A_{n-2}$ 中共 $2^{n-2}$ 项, 其中恰有一半的项是 1 , 并且恰有 $b_{n-2}$ 个 0,0 数对, 所以 $b_n=c_{n-1}=2^{n-3}+b_{n-2}$, 逐步递推得 $b_n=2^{n-3}+ 2^{n-5}+\\cdots+\\left\\{\\begin{array}{l}2^0+b_1, 2 \\times n, \\\\ 2^1+b_2, 2 \\mid n \\text {. }\\end{array}\\right.$ 其中 $b_1=0, b_2=1$. 由此可得 $b_n=\\frac{1}{3}\\left[2^{n-1}+(-1)^n\\right]$, 即 $f^{(n)}((1))$ 中连续两项为 0,0 的数对的个数为 $b_n=\\frac{1}{3}\\left[2^{n-1}+(-1)^n\\right]$.", + "remark": "", + "figures": [] +} \ No newline at end of file diff --git a/processed_dataset/calculation/0252.json b/processed_dataset/calculation/0252.json new file mode 100644 index 0000000000000000000000000000000000000000..dd220607edbed4f7dd3dd48a22411c24b5a0b060 --- /dev/null +++ b/processed_dataset/calculation/0252.json @@ -0,0 +1,8 @@ +{ + "source_file": "./raw_volume-zh/volume11/exercise8.tex", + "problem_type": "calculation", + "problem": "问题4. 设有 2009 个人站成一排, 从第 1 名开始 1 至 3 报数, 全部报完数后, 凡是报到 3 的倍数的人就退出队伍, 其余的人向前靠拢站成新的一排, 再按此规则继续进行, 直到经过 $p$ 轮报数后只剩下 3 人为止, 试问最后剩下的 3 人最初在什么位置?", + "solution": "$p$ 轮报数后只剩下的 3 人中前两人最初的位置显然是原来队伍中的第 1 和第 2 位置.\n设第 3 人的最初位置是第 $a_{p+1}$ 个位置,则第 1 次报数后他站到第 $a_p$ 个位置上, $\\cdots \\cdots$, 第 $p$ 次报数后他站在第 $a_1$ 个位置上, 显然 $a_1=3$. 由于 $a_2, a_3, \\cdots, a_{p+1}$ 都没有被淘汰, 可见这些数都不是 3 的倍数,且由 $a_{p+1}$ 到 $a_p$ 变动的数目 $a_{p+1}-a_p$ 恰等于从 1 到 $a_{p+1}$ 内 3 的倍数的个数, 即 $a_{p+1}-a_p= \\frac{1}{3}\\left(a_{p+1}-r\\right)(r$ 等于 1 或 2$)$, 故 $a_{p+1}=\\frac{3 a_p-r}{2}$, 因 $a_p, a_{p+1}$ 均为正整数,所以当 $a_p$ 为奇数时 $r=1$, 当 $a_p$ 为偶数时 $r=2$. 由此可得 $a_1=3, a_{p+1}=\\left[\\frac{3 a_p-1}{2}\\right] (p=1,2, \\cdots) \\cdots$ (1), 并且 $a_{p+1} \\leqslant 20092009 . \\text { 综上可知,最后剩下 } 3 \\text { 人 }\n\\end{aligned}\n$$\n的最初位置是第 1 ,第 2 和第 1600 个位置.", + "remark": "", + "figures": [] +} \ No newline at end of file diff --git a/processed_dataset/calculation/0253.json b/processed_dataset/calculation/0253.json new file mode 100644 index 0000000000000000000000000000000000000000..f1171744b9ea3beb1ec8b925855f602a4875f3da --- /dev/null +++ b/processed_dataset/calculation/0253.json @@ -0,0 +1,8 @@ +{ + "source_file": "./raw_volume-zh/volume11/exercise8.tex", + "problem_type": "calculation", + "problem": "问题6. 在小于 $10^4$ 的正整数中, 有多少个正整数 $n$, 使 $2^n-n^2$ 被 7 整除.", + "solution": "设 $b_n=2^n, c_n=n^2, a_n=b_n-c_n=2^n-n^2$. 因为 $b_{n \\vdash 3}=2^{n+3}=8$ ・ $2^n \\equiv 2^n=b_n(\\bmod 7), c_{n+7}=(n+7)^2 \\equiv n^2=c_n(\\bmod 7)$, 所以 $a_{n+21}=b_{n+21}- c_{n+21} \\equiv b_n-c_n=a_n(\\bmod 7)$, 而 $a_1, a_2, \\cdots, a_{21}$ 中只有 6 项 $a_2=0, a_4=0$, $a_5=7, a_6=28, a_{10}=924, a_{15}=32543$ 被 7 整除.\n又 $9999=476 \\times 21+3$\n且 $a_1, a_2, a_3$ 中只有 $a_2$ 被 7 整除.\n故小于 $10^4$ 的正整数中使 $2^n-n^2$ 被 7 整除的正整数 $n$ 的个数为 $476 \\times 6+1=2857$ 个.", + "remark": "", + "figures": [] +} \ No newline at end of file diff --git a/processed_dataset/calculation/0254.json b/processed_dataset/calculation/0254.json new file mode 100644 index 0000000000000000000000000000000000000000..b09a7edb803cd4e217cdc3a695b364a82ce55c6f --- /dev/null +++ b/processed_dataset/calculation/0254.json @@ -0,0 +1,8 @@ +{ + "source_file": "./raw_volume-zh/volume11/exercise8.tex", + "problem_type": "calculation", + "problem": "问题9. 是否存在无穷多个 $\\triangle A B C$, 使 $A B, B C, C A$ 的长是成等差数列的互素的正整数 $(A Bd\\right) . \\triangle A B C$ 的面积为 $S, B C$ 边上的高为 $h_a$, 则 $S=\\sqrt{\\frac{3 a}{2} \\cdot\\left(\\frac{a}{2}+d\\right) \\cdot \\frac{a}{2} \\cdot\\left(\\frac{a}{2}-d\\right)}= \\frac{1}{2} a \\sqrt{3\\left[\\left(\\frac{a}{2}\\right)^2-d^2\\right]}$. 要使 $S$ 为正整数, $a$ 必须为偶数.\n令 $a=2 x\\left(x \\in \\mathbf{N}_{+}\\right)$, 则 $S=x \\sqrt{3\\left(x^2-d^2\\right)}, h_a=\\frac{2 S}{a}=\\sqrt{3\\left(x^2-d^2\\right)}, h_a^2=3\\left(x^2-d^2\\right)$, 故 $h_a$ 必须为 3 的倍数.\n设 $h_a=3 y\\left(y \\in \\mathbf{N}_{+}\\right)$, 则 $x^2-3 y^2=d^2$. 为简单起见, 取 $d=$ 1 , 则 $x^2-3 y^2=1 \\cdots$ (1), 于是 $(x, y)=(2,1)$ 为(1)的一个解.\n令 $(2+\\sqrt{3})^n= x_n+y_n \\sqrt{3}\\left(x_n, y_n\\right.$ 为正整数, $\\left.n \\in \\mathbf{N}_{+}\\right)$, 则 $(2-\\sqrt{3})^n=x_n-y_n \\sqrt{3}$, 于是 $x_n^2- 3 y_n^2=1$, 即 $(x, y)=\\left(x_n, y_n\\right)\\left(n \\in \\mathbf{N}_{+}\\right)$都为(1)的正整数解.\n由 $x_{n+1}+y_{n+1} \\sqrt{3}=(2+\\sqrt{3})^{n+1}=(2+\\sqrt{3})\\left(x_n+y_n \\sqrt{3}\\right)=\\left(2 x_n+3 y_n\\right)+\\left(x_n+2 y_n\\right) \\sqrt{3}$ 得 $\\left\\{\\begin{array}{l}x_{n+1}=2 x_n+3 y_n, \\\\ y_{n+1}=x_n+2 y_n .\\end{array}\\right.$ 且 $\\left\\{\\begin{array}{l}x_1=2, \\\\ y_1=1 .\\end{array}\\right.$ 消去 $y_n$ 可得 $x_{n+2}=4 x_{n+1}-x_n, x_1=2$, $x_2=7$. 于是可取 $A B=2 x_n-1, B C=2 x_n, C A=2 x_n+1,(n=1,2, \\cdots)$, 则 $A B, B C, C A$ 是成等差数列的互素的正整数 ( $A B) 所示的阵势变为如图() 所示的阵势?(\"马\"按照国际象棋规则跳动)\n\\begin{tabular}{|l|l|l|}\n\\hline 1 & 4 & 7 \\\\\n\\hline 2 & 5 & 8 \\\\\n\\hline 3 & 6 & 9 \\\\\n\\hline\n\\end{tabular}", + "solution": "解:如图() 所示, 将九个方格编号.\n再把每个方格对应为平面上一点.\n若马能从一个方格跳往另一个方格, 则在相应两点之间连一条边, 如图().\n于是如图() 所示的开始的阵势以及如图() 所示的要求变成的阵势分别变成了如图()、如图() 中的两个图形.\n显然,马在一个圆上的前后跟随顺序是不变的,所以不能按要求改变阵势.", + "remark": "", + "figures": [ + "./images/volume12/figures/fig-c1i6.png", + "./images/volume12/figures/fig-c1i7.png", + "./images/volume12/figures/fig-c1i8.png", + "./images/volume12/figures/fig-c1i9.png", + "./images/volume12/figures/fig-c1i6.png", + "./images/volume12/figures/fig-c1i7.png", + "./images/volume12/figures/fig-c1i10.png", + "./images/volume12/figures/fig-c1i11.png" + ] +} \ No newline at end of file diff --git a/processed_dataset/calculation/0256.json b/processed_dataset/calculation/0256.json new file mode 100644 index 0000000000000000000000000000000000000000..ad6737e0c72d16c06d5b60ba7ea6f8dde875f2af --- /dev/null +++ b/processed_dataset/calculation/0256.json @@ -0,0 +1,8 @@ +{ + "source_file": "./raw_volume-zh/volume12/chapter1.tex", + "problem_type": "calculation", + "problem": "例3. 有 $n(n>3)$ 个人,他们之间有些人互相认识, 有些人互相不认识,而且至少有一个人没有与其他人都认识.\n问: 与其他人都认识的人数的最大值是多少?", + "solution": "解:作图 $G$ : 用 $n$ 个点表示这 $n$ 个人,两顶点相邻当且仅当相应的两个人互相认识.\n由于至少有一个人没有与其他人都认识, 所以图 $G$ 中至少有两点不相邻, 设 $v_1, v_2$ 之间没有边 $e=\\left(v_1, v_2\\right)$. 则图 $G$ 的边数最多时的图形为 $K_n- e$, 即从完全图 $K_n$ 中去掉边 $e$ 后所得的图.\n从而与其他顶点都相邻的顶点个数的最大值为 $n-2$. 故与其他人都认识的人数的最大值是 $n-2$.", + "remark": "", + "figures": [] +} \ No newline at end of file diff --git a/processed_dataset/calculation/0257.json b/processed_dataset/calculation/0257.json new file mode 100644 index 0000000000000000000000000000000000000000..5935296a4bc4a1a4a281949fef25b712b17d36d2 --- /dev/null +++ b/processed_dataset/calculation/0257.json @@ -0,0 +1,8 @@ +{ + "source_file": "./raw_volume-zh/volume12/chapter1.tex", + "problem_type": "calculation", + "problem": "例6. 有 $n$ 个人,已知他们中的任意两人至多通电话一次, 他们中的任意 $n-2$ 个人之间通电话的总次数相等, 都是 $3^m$ 次, 其中 $m$ 是自然数.\n求 $n$ 的所有可能值.", + "solution": "解:显然 $n \\geqslant 5$. 记 $n$ 个人为几个点 $A_1, A_2, \\cdots, A_n$. 若 $A_i, A_j$ 之间通电话, 则连 $\\left(A_i, A_j\\right)$. 因此这 $n$ 个点中必有边相连, 不妨设为 $\\left(A_1, A_2\\right)$.\n倘若设 $A_1$ 与 $A_3$ 之间无边,分别考虑 $n-2$ 个点 $A_1, A_4, A_5, \\cdots, A_n$ ; $A_2, A_4, A_5, \\cdots, A_n$; 及 $A_3, A_4, A_5, \\cdots, A_n$. 由题意知 $A_1, A_2, A_3$ 分别与 $A_4, A_5, \\cdots, A_n$ 之间所连边的总数相等, 记为 $k$.\n将 $A_2$ 加人到 $A_1, A_4, A_5, \\cdots, A_n$ 中, 则这 $n-1$ 个点之间边的总数 $S= 3^m+k+1$. 从这 $n-1$ 点中任意去掉一点剩下的 $n-2$ 个点所连边数都是 $3^m$, 故每个点都与其余 $n-2$ 个点连 $k+1$ 条边.\n从而\n$$\nS=\\frac{1}{2}(n-1)(k+1) \\text {. }\n$$\n同理, 考虑 $A_3$ 加人 $A_1, A_4, A_5, \\cdots, A_n$ 中所得的 $n-1$ 个点的情况可知边的总数为 $t=3^m+k=\\frac{1}{2}(n-1) k$.\n因为 $S=t+1$, 得\n$$\n\\frac{1}{2}(n-1)(k+1)=\\frac{1}{2}(n-1) k+1,\n$$\n即 $n=3$, 矛盾.\n所以 $A_1 、 A_3$ 之间有边.\n同理 $A_2, A_3$ 之间也有边.\n进而 $A_1, A_2$ 与所有 $A_i(i=3,4, \\cdots, n)$ 之间有边.\n对于 $A_i, A_j(i \\neq j)$, 因为 $A_i$ 与 $A_1$ 之间有边, 可知 $A_i$ 与 $A_j$ 之间有边.\n因此这 $n$ 个点构成一个完全图.\n所以\n$$\n3^m=\\frac{1}{2}(n-2)(n-3) .\n$$\n故 $n=5$.", + "remark": "", + "figures": [] +} \ No newline at end of file diff --git a/processed_dataset/calculation/0258.json b/processed_dataset/calculation/0258.json new file mode 100644 index 0000000000000000000000000000000000000000..b07bb43396ace0dee9bfed51546f2daac7c0c3c6 --- /dev/null +++ b/processed_dataset/calculation/0258.json @@ -0,0 +1,8 @@ +{ + "source_file": "./raw_volume-zh/volume12/chapter1.tex", + "problem_type": "calculation", + "problem": "例7. 设 $n$ 为一正整数,且 $A_1, A_2, \\cdots, A_{2 n+1}$ 是某个集合 $B$ 的子集.\n设\n(1) 每一个 $A_i$ 恰含有 $2 n$ 个元素;\n(2) 每一个 $A_i \\cap A_j(1 \\leqslant i)列出 21 条航线的实例,其中用了不同线条来表示不同航空公司的航线.", + "remark": "", + "figures": [ + "./images/volume12/figures/fig-c4i5.png" + ] +} \ No newline at end of file diff --git a/processed_dataset/calculation/0263.json b/processed_dataset/calculation/0263.json new file mode 100644 index 0000000000000000000000000000000000000000..404b0d656a12d24ade5e1c9a49df38a5b8db0dad --- /dev/null +++ b/processed_dataset/calculation/0263.json @@ -0,0 +1,11 @@ +{ + "source_file": "./raw_volume-zh/volume12/chapter5.tex", + "problem_type": "calculation", + "problem": "例1. 如图() 所示是一幢房子的平面图形, 前门进人是一个客厅, 由客厅可通向四个房间.\n如果现在你由前门进去, 能否通过所有的门走遍所有的房间和客厅, 然后从后门走出, 而且要求每扇门只能进出一次?", + "solution": "解:答案是否定的.\n把 5 个房间以及前门外面和后门外面作为顶点, 两个地方有门相通就在相应的顶点之间连一条边, 得到图 $G$, 如图() 所示.\n在图 $G$ 中, 奇顶点的个数是 4 , 故 $G$ 不是一条链.\n所以问题的答案是否定的.", + "remark": "", + "figures": [ + "./images/volume12/figures/fig-c5i3.png", + "./images/volume12/figures/fig-c5i4.png" + ] +} \ No newline at end of file diff --git a/processed_dataset/calculation/0264.json b/processed_dataset/calculation/0264.json new file mode 100644 index 0000000000000000000000000000000000000000..80330ee461d358667a961cbe6ce18ac3c79b64a9 --- /dev/null +++ b/processed_dataset/calculation/0264.json @@ -0,0 +1,11 @@ +{ + "source_file": "./raw_volume-zh/volume12/chapter5.tex", + "problem_type": "calculation", + "problem": "例3. 如图() 所示,在 $8 \\times 8$ 黑白方格的棋盘上跳动一只马, 不论跳动方向如何, 要使这只马跳遍棋盘的每一格且每格只经过一次, 问这是否可能? (一只马跳动一次是指从 $2 \\times 3$ 黑白方格组成的长方形的一个对角跳到另一个对角上)", + "solution": "解:如图() 中给出这个问题的一个解答.\n\\begin{tabular}{|l|l|l|l|l|l|l|l|}\n\\hline 56 & 41 & 58 & 35 & 50 & 39 & 60 & 33 \\\\\n\\hline 47 & 44 & 55 & 40 & 59 & 34 & 51 & 38 \\\\\n\\hline 42 & 57 & 46 & 49 & 36 & 53 & 32 & 61 \\\\\n\\hline 45 & 48 & 43 & 54 & 31 & 62 & 37 & 52 \\\\\n\\hline 20 & 5 & 30 & 63 & 22 & 11 & 16 & 13 \\\\\n\\hline 29 & 64 & 21 & 4 & 17 & 14 & 25 & 10 \\\\\n\\hline 6 & 19 & 2 & 27 & 8 & 23 & 12 & 15 \\\\\n\\hline 1 & 28 & 7 & 18 & 3 & 26 & 9 & 24 \\\\\n\\hline\n\\end{tabular}\n解决这类问题, 常常用以下 4 种方法尝试:\n1. 每次将马放到使它能走到的 (尚未走过的)方格为最少的位置, 即先走\"出路\"少的方格,后走\"出路\"多的方格.\n2. 将棋盘分为几个部分, 在每个部分中找一条哈密顿链 (见第六节), 然后把它们连接起来.\n3. 在棋盘上找几个圈,然后将这些圈连接起来.\n4. 将一个较小的棋盘镶上边, 产生一个大棋盘上的哈密顿链.", + "remark": "", + "figures": [ + "./images/volume12/figures/fig-c5i7.png", + "./images/volume12/figures/fig-c5i8.png" + ] +} \ No newline at end of file diff --git a/processed_dataset/calculation/0265.json b/processed_dataset/calculation/0265.json new file mode 100644 index 0000000000000000000000000000000000000000..e36347d2a45334944491ccfaca7e793d21f55324 --- /dev/null +++ b/processed_dataset/calculation/0265.json @@ -0,0 +1,10 @@ +{ + "source_file": "./raw_volume-zh/volume12/chapter5.tex", + "problem_type": "calculation", + "problem": "例4. 如图() 中,甲、乙两只蚂蚁分别处在 $A$ 、 $B$ 两点.\n甲蚂蚁向乙蚂蚁提出: \"咱俩比赛,看谁先把这个图中的九条边都爬遍后到达 $E$ 点.\n乙蚂蚁同意.\n假定两只蚂蚁爬的速度相同且同时开始.\n问: 究竟哪只蚂蚁最先到达 $E$ 点?", + "solution": "解:把 $A 、 B 、 C 、 D 、 E$ 作为顶点, 原来的九条线段作为 9 条边, 得图 $G$. 则 $G$ 是连通图, 且奇顶点个数为 2 , 根据定理一, 它是一条链.\n由于点 $B$ 是奇顶点, $E$ 也是奇顶点,所以从 $B$ 到 $E$ 存在着一条链,例如 $B C D A C E A B D E$,\n对乙蚂蚁来说, 它可以从 $B$ 点出发, 沿着这条链到达 $E$ 点.\n但顶点 $A$ 是偶顶点, 从 $A$ 出发到达 $E$ 不可能不重复地走遍图 $G$ 中的所有边, 它至少要重复经过某一条边.\n所以蚂蚁乙可以选择一条正确路线比蚂蚁甲先爬到 $E$ 点.", + "remark": "", + "figures": [ + "./images/volume12/figures/fig-c5i9.png" + ] +} \ No newline at end of file diff --git a/processed_dataset/calculation/0266.json b/processed_dataset/calculation/0266.json new file mode 100644 index 0000000000000000000000000000000000000000..5516985718f9aedc23df7f760f8fa56606924cd1 --- /dev/null +++ b/processed_dataset/calculation/0266.json @@ -0,0 +1,11 @@ +{ + "source_file": "./raw_volume-zh/volume12/chapter6.tex", + "problem_type": "calculation", + "problem": "例1. 如图() 有无哈密顿链或哈密顿圈?", + "solution": "解:按照如图() 中所给的编号, 可以看出这样的一个圈是存在的.\n这里是采用\"直接求解\"的方法来解决环游世界的问题, 即从图的某一个顶点出发, 采用一步步试探的方法, 来找出图的哈密顿链 (圈). 如果找到了一条解就出来了, 如果找不到, 就可能没有解.\n这种方法一般只用在比较简单的图上, 而且多用在肯定有哈密顿链 (圈) 存在的情况.", + "remark": "", + "figures": [ + "./images/volume12/figures/fig-c6i1.png", + "./images/volume12/figures/fig-c6i1.png" + ] +} \ No newline at end of file diff --git a/processed_dataset/calculation/0267.json b/processed_dataset/calculation/0267.json new file mode 100644 index 0000000000000000000000000000000000000000..3acfc47cd1ee18bd31acf5cb3abad7ba265a0c4e --- /dev/null +++ b/processed_dataset/calculation/0267.json @@ -0,0 +1,13 @@ +{ + "source_file": "./raw_volume-zh/volume12/chapter6.tex", + "problem_type": "calculation", + "problem": "例2. 在一次国际数学家大会上, 7 位来自不同国家的数学家会话能力如下:\n$A$ :英语.\n$B$ :英语和汉语.\n$C$ :英语、意大利语和西班牙语.\n$D$ : 汉语和日语.\n$E$ : 德语和意大利语.\n$F$ : 法语、日语和西班牙语.\n$G$ : 法语和德语.\n问怎样安排这 7 名数学家围着一个圆桌坐下, 使得每个人都能和他身边的两个人交谈?", + "solution": "解:设 7 个顶点 $A 、 B 、 C 、 D 、 E 、 F 、 G$ 对应这 7 名数学家, 其中会用同一种语言的人对应的顶点之间连一条边,这样就得到了一个图 $G_1$, 如图() 所示.\n于是原来的排座问题就变成了如图() 中找一条哈密顿圈的问题了.\n按圈上顶点的顺序来排座位,那么每个人和他相邻的两个人都能交谈.\n如图() 中用粗线画出的一个圈, 就是我们所求的解.\n也就是说, 如果按照 $A 、 B 、 D 、 F 、 G 、 E 、 C$ 的顺序排座位, 每个人就都可以和他的两个邻座交谈,所采用的语言种类标明如图() 中的对应边上.", + "remark": "", + "figures": [ + "./images/volume12/figures/fig-c6i2.png", + "./images/volume12/figures/fig-c6i2.png", + "./images/volume12/figures/fig-c6i2.png", + "./images/volume12/figures/fig-c6i3.png" + ] +} \ No newline at end of file diff --git a/processed_dataset/calculation/0268.json b/processed_dataset/calculation/0268.json new file mode 100644 index 0000000000000000000000000000000000000000..9b6de03dce59981bc08ff545efc23093459524da --- /dev/null +++ b/processed_dataset/calculation/0268.json @@ -0,0 +1,11 @@ +{ + "source_file": "./raw_volume-zh/volume12/chapter6.tex", + "problem_type": "calculation", + "problem": "例4. 如图() 是半个国际象棋盘,一匹马在右下角,试问: 马能否连续地把棋盘上所有的格都跳到一次并且仅仅一次? 如果去掉了棋盘对角上的两个黑色方格, 又将怎样?", + "solution": "解:我们考虑这样的图:将棋盘方格对应于图的顶点,如果马从棋盘上的一个方格跳一次后能到另一个方格, 就在这两个方格所对应的顶点之间连上一条边.\n于是问题就转化为判断棋盘所对应的这个图是否有一条哈密顿链.\n在图中, 顶点是否相邻是由马跳的方式决定的, 也就是说每个顶点只能跟和它组成一个\"日\"字的对角线上的顶点相邻.\n棋盘上组成\"日\"字对角线的方格所着的颜色正好是相反的,让图上每个顶点涂上它所对应的方格颜色.\n这样, 图上每条边所相邻的两个顶点的颜色都是一黑 一白,并且黑、白顶点的个数是相等的, 这就可能有哈密顿链.\n用试探的方法可以找到一条链.\n如图() 所示的就是一个答案.\n\\begin{tabular}{|c|c|c|c|c|c|c|c|}\n\\hline 15 & 18 & 7 & 22 & 11 & 28 & 5 & 24 \\\\\n\\hline 8 & 21 & 16 & 27 & 6 & 23 & 2 & 29 \\\\\n\\hline 17 & 14 & 19 & 10 & 31 & 12 & 25 & 4 \\\\\n\\hline 20 & 9 & 32 & 13 & 26 & 3 & 30 & 1 \\\\\n\\hline\n\\end{tabular}\n现在来看问题的第二部分.\n仍采用涂色法将问题转化为求对应的图是否存在哈密顿链的问题.\n由于黑顶点个数是 14 , 白顶点个数是 16 , 根据定理一, 这个图中没有哈密顿链.\n即对于去掉两个黑色方格的半个棋盘来说, 马是无法连续地把每个方格都跳到一次并且仅仅一次.", + "remark": "", + "figures": [ + "./images/volume12/figures/fig-c6i6.png", + "./images/volume12/figures/fig-c6i7.png" + ] +} \ No newline at end of file diff --git a/processed_dataset/calculation/0269.json b/processed_dataset/calculation/0269.json new file mode 100644 index 0000000000000000000000000000000000000000..239c49c8e4f39c343d110c66490cb32ae41d57e0 --- /dev/null +++ b/processed_dataset/calculation/0269.json @@ -0,0 +1,13 @@ +{ + "source_file": "./raw_volume-zh/volume12/chapter7.tex", + "problem_type": "calculation", + "problem": "例3. 如图()、如图() 是平面图吗?", + "solution": "解:如图() 包含了一个 $K_5$ 图, 如图() 包含了一个 $K_{3,3}$ 图, 根据库拉托夫斯基定理,这两个图都是非平面图.", + "remark": "", + "figures": [ + "./images/volume12/figures/fig-c7i5.png", + "./images/volume12/figures/fig-c7i6.png", + "./images/volume12/figures/fig-c7i5.png", + "./images/volume12/figures/fig-c7i6.png" + ] +} \ No newline at end of file diff --git a/processed_dataset/calculation/0270.json b/processed_dataset/calculation/0270.json new file mode 100644 index 0000000000000000000000000000000000000000..1a845617a8dd95a0d01725d7dfa890f71a56c587 --- /dev/null +++ b/processed_dataset/calculation/0270.json @@ -0,0 +1,10 @@ +{ + "source_file": "./raw_volume-zh/volume12/exercise1.tex", + "problem_type": "calculation", + "problem": "问题1. 设图 $G=(V, E), V=\\left\\{v_1, v_2, \\cdots, v_5\\right\\}, E=\\left\\{\\left(v_1, v_2\\right),\\left(v_2, v_4\\right),\\left(v_3\\right.\\right.$, $\\left.\\left.v_4\\right),\\left(v_4, v_5\\right),\\left(v_1, v_3\\right)\\right\\}$. 画出 $G$ 的图形.", + "solution": "$G$ 的图形如图()所示.", + "remark": "", + "figures": [ + "./images/volume12/figures/fig-c1a1.png" + ] +} \ No newline at end of file diff --git a/processed_dataset/calculation/0271.json b/processed_dataset/calculation/0271.json new file mode 100644 index 0000000000000000000000000000000000000000..9c63db70aa59e4b15fac0b0c35598d4276c5e845 --- /dev/null +++ b/processed_dataset/calculation/0271.json @@ -0,0 +1,13 @@ +{ + "source_file": "./raw_volume-zh/volume12/exercise1.tex", + "problem_type": "calculation", + "problem": "问题3. 说明下面两个图()和() 是同构的.", + "solution": "如图()和如图()所示建立点的对应关系为: $v_1 \\longleftrightarrow u_1, v_2 \\longleftrightarrow u_2, v_3 \\longleftrightarrow u_3, v_4 \\longleftrightarrow u_4, v_5 \\leftrightarrow u_5$, 边的对应关系为 $e_1 \\longleftrightarrow e_1^{\\prime}, e_2 \\longleftrightarrow e_2^{\\prime}, e_3 \\leftrightarrow e_3^{\\prime}, \\cdots, e_8 \\leftrightarrow e_8^{\\prime}$. 这两个图的顶点数相等, 边数相等, 且顶点和边相互对应, 因此这两图是同构的.", + "remark": "", + "figures": [ + "./images/volume12/figures/fig-c1p3-1.png", + "./images/volume12/figures/fig-c1p3-2.png", + "./images/volume12/figures/fig-c1a3-1.png", + "./images/volume12/figures/fig-c1a3-2.png" + ] +} \ No newline at end of file diff --git a/processed_dataset/calculation/0272.json b/processed_dataset/calculation/0272.json new file mode 100644 index 0000000000000000000000000000000000000000..5fb52ed4f549d3b0f068a6a808cee42508d39c4f --- /dev/null +++ b/processed_dataset/calculation/0272.json @@ -0,0 +1,8 @@ +{ + "source_file": "./raw_volume-zh/volume12/exercise1.tex", + "problem_type": "calculation", + "problem": "问题4. 有 $n$ 个药箱,每两个药箱里有一种相同的药,每种药恰好在两个药箱里出现, 问有多少种药?", + "solution": "按下法作一个图: 每一个顶点表示一个药箱, 每一条边 $\\left(v_i, v_j\\right)$ 表示两个药箱 $v_i$ 与 $v_j$ 所共有的那一种药.\n据题设, 此图是 $n$ 个顶点的完全图 $K_n$. 药的种数即 $K_n$ 的边数 $\\frac{1}{2} n(n-1)$.", + "remark": "", + "figures": [] +} \ No newline at end of file diff --git a/processed_dataset/calculation/0273.json b/processed_dataset/calculation/0273.json new file mode 100644 index 0000000000000000000000000000000000000000..a9b1792d8956372a72ad16119aba63ff025b4519 --- /dev/null +++ b/processed_dataset/calculation/0273.json @@ -0,0 +1,8 @@ +{ + "source_file": "./raw_volume-zh/volume12/exercise2.tex", + "problem_type": "calculation", + "problem": "问题5. 小城共有 15 部电话, 能否用电线连接它们, 使得每部电话恰好与 5 部别的电话相连?", + "solution": "不能.\n由定理二即得.", + "remark": "", + "figures": [] +} \ No newline at end of file diff --git a/processed_dataset/calculation/0274.json b/processed_dataset/calculation/0274.json new file mode 100644 index 0000000000000000000000000000000000000000..c4d35a72882eba30fa31d6da97626122791945a8 --- /dev/null +++ b/processed_dataset/calculation/0274.json @@ -0,0 +1,8 @@ +{ + "source_file": "./raw_volume-zh/volume12/exercise2.tex", + "problem_type": "calculation", + "problem": "问题12. 棱长为 $n$ (自然数) 的立方体被平行于它的侧面的平面切成 $n^3$ 个单位立方体,其中有多少对公共顶点不多于 2 的单位立方体?", + "solution": "以单位立方体为顶点, 当且仅当二单位立方体有公共侧面时, 在此二顶点间加一边, 构成一个图 $G, G$ 的补图 $\\bar{G}$ 的边数即为所求.\n易知 $G$ 的边数为 $3 n^2(n-1), K_{n^2}$ 的边数为 $\\frac{1}{2} n^3\\left(n^3-1\\right)$, 故 $\\bar{G}$ 的边数为 $\\frac{1}{2} n^3\\left(n^3-1\\right)-3 n^2(n-1)=\\frac{1}{2} n^6-\\frac{7}{2} n^3+3 n^2$. 即公共顶点不多于 2 的单位立方体共有 $\\frac{1}{2} n^6-\\frac{7}{2} n^3+3 n^2$ 对.", + "remark": "", + "figures": [] +} \ No newline at end of file diff --git a/processed_dataset/calculation/0275.json b/processed_dataset/calculation/0275.json new file mode 100644 index 0000000000000000000000000000000000000000..0a19d3baac0fda7f4e71c2917491d587559163d7 --- /dev/null +++ b/processed_dataset/calculation/0275.json @@ -0,0 +1,8 @@ +{ + "source_file": "./raw_volume-zh/volume12/exercise3.tex", + "problem_type": "calculation", + "problem": "问题2. 作一个不含三角形的, 有 20 个顶点, 100 条边的简单图.", + "solution": "作完全偶图 $K_{10,10}$.", + "remark": "", + "figures": [] +} \ No newline at end of file diff --git a/processed_dataset/calculation/0276.json b/processed_dataset/calculation/0276.json new file mode 100644 index 0000000000000000000000000000000000000000..37315a9d0196e05893326bc47ddece216a7060ec --- /dev/null +++ b/processed_dataset/calculation/0276.json @@ -0,0 +1,8 @@ +{ + "source_file": "./raw_volume-zh/volume12/exercise3.tex", + "problem_type": "calculation", + "problem": "问题4. 设图 $G$ 有 $n(n>5)$ 个顶点, 则在 $G$ 和 $G$ 的补图 $\\bar{G}$ 中总共含有至少 $\\frac{1}{24} n(n-$ 1) $(n-5)$ 个三角形.", + "solution": "设 $G=(V, E)$, 有 $d(x)\\{n-1-d(x)\\}$ 个三点组 $\\{x, y, z\\}$, 它们在 $G$ 或 $\\bar{G}$ 中都不构成三角形, 且在 $G$ 中有唯一的一条边以 $x \\in V$ 为端点.\n在 $G$ 或 $\\bar{G}$ 中不构成三角形的每一个三点组 $\\{x, y, z\\}$ 含有 $G$ 的一条边或两条边.\n设 $(x$, $y)$ 是 $G$ 的一条边, $(x, z) 、(y, z)$ 是 $\\bar{G}$ 的两条边, 在总和 $\\sum_{x \\in V} d(x)\\{n-1- d(x)\\}$ 中, 三点组 $\\{x, y, z\\}$ 被计算两次: 一次关于 $x$, 一次关于 $y$. 而如果 $(x$, $y) 、(y, z)$ 是 $G$ 的边, $(x, z)$ 是 $\\bar{G}$ 的边, 则上述和数中三点组 $\\{x, y, z\\}$ 也是被计算两次:一次关于 $x$, 另一次关于 $z$. 因此在 $G$ 和 $\\bar{G}$ 中三角形的总数为\n$$\n\\begin{aligned}\n\\mathrm{C}_n^3-\\frac{1}{2} \\sum_{x \\in V} d(x)\\{n-1-d(x)\\} & \\geqslant \\mathrm{C}_n^3-\\frac{n}{2}\\left(\\frac{n-1}{2}\\right)^2 \\\\\n& =\\frac{1}{24} n(n-1)(n-5) .\n\\end{aligned}\n$$", + "remark": "", + "figures": [] +} \ No newline at end of file diff --git a/processed_dataset/calculation/0277.json b/processed_dataset/calculation/0277.json new file mode 100644 index 0000000000000000000000000000000000000000..bd7249c30a9e401c27d10af4054135a8c9f53e9a --- /dev/null +++ b/processed_dataset/calculation/0277.json @@ -0,0 +1,8 @@ +{ + "source_file": "./raw_volume-zh/volume12/exercise4.tex", + "problem_type": "calculation", + "problem": "问题1. 如果连通图 $G$ 的顶点数 $\\geqslant 2$, 则 $G$ 中至少存在两个顶点, 将这两个顶点及其关联的边去掉后,图仍然连通 (没有顶点的\"图\"也看作是连通图).", + "solution": "$G$ 的生成树有两个悬挂点, 去掉这两点后, 图仍然连通.", + "remark": "", + "figures": [] +} \ No newline at end of file diff --git a/processed_dataset/calculation/0278.json b/processed_dataset/calculation/0278.json new file mode 100644 index 0000000000000000000000000000000000000000..2450dcc5405fff5bfaa6c83d35bdd4ce33dc9aff --- /dev/null +++ b/processed_dataset/calculation/0278.json @@ -0,0 +1,8 @@ +{ + "source_file": "./raw_volume-zh/volume12/exercise4.tex", + "problem_type": "calculation", + "problem": "问题2. 坐标纸上的 11 条纵线和 11 条横线构成一个图,图的顶点是纵、横线的交点 (格点), 边是格点间的纵、横线段, 问至少应当去掉多少条边才能使每点的度 $<4$ ? 至多可以去掉多少条边还能使图保持连通?", + "solution": "其中有 $9 \\times 9=81$ 个度为 4 的顶点,故至少要去掉 $\\left[\\frac{81}{2}\\right]+1=41$ 条边, 才能使每点的度 $<4$. 至多可以去掉 $2 \\times 11 \\times 10-120=100$ 条边, 还可以使图保持连通.", + "remark": "", + "figures": [] +} \ No newline at end of file diff --git a/processed_dataset/calculation/0279.json b/processed_dataset/calculation/0279.json new file mode 100644 index 0000000000000000000000000000000000000000..692294697e05873d2d5fbd5ce7e1baf87d7d51a7 --- /dev/null +++ b/processed_dataset/calculation/0279.json @@ -0,0 +1,8 @@ +{ + "source_file": "./raw_volume-zh/volume12/exercise4.tex", + "problem_type": "calculation", + "problem": "问题3. 若图 $G$ 有 $n$ 个顶点, $n-1$ 条边, 则 $G$ 为树.\n这个命题正确吗? 为什么?", + "solution": "命题不正确, 取 $K_3$ 与一个孤立点 (和图中其他点都不相邻的点) 组成图 $G$. 则 $G$ 有 4 个顶点 3 条边, 它不连通, 显然不是树.", + "remark": "", + "figures": [] +} \ No newline at end of file diff --git a/processed_dataset/calculation/0280.json b/processed_dataset/calculation/0280.json new file mode 100644 index 0000000000000000000000000000000000000000..66e02d3c54f26fb0ca744e6a1028b1a5ad91cd1b --- /dev/null +++ b/processed_dataset/calculation/0280.json @@ -0,0 +1,10 @@ +{ + "source_file": "./raw_volume-zh/volume12/exercise4.tex", + "problem_type": "calculation", + "problem": "问题4. 已知一棵树 $T$ 有3个3度顶点,一个2度定点, 其余的都是悬挂点.\n1) $T$中有几个悬挂点? (2)试画出两棵满足上述度数要求的不同构的树.", + "solution": "(1) 设 $T$ 中有 $x$ 个悬挂点, 则 $T$ 的顶点数 $n=3+1+x$, 边数 $e=n- 1=x+3$. $\\sum_{i=1}^n d\\left(v_i\\right)=3 \\times 3+2 \\times 1+1 \\times x=11+x$, 故 $11+x=2(x+3)$, $x=5$.\n(2) 如图()所示的两棵树均满足要求,但它们不同构.", + "remark": "", + "figures": [ + "./images/volume12/figures/fig-c4a4.png" + ] +} \ No newline at end of file diff --git a/processed_dataset/calculation/0281.json b/processed_dataset/calculation/0281.json new file mode 100644 index 0000000000000000000000000000000000000000..ce9bcc93ef2779f5c6ab5b5e6de3fbee9d000c16 --- /dev/null +++ b/processed_dataset/calculation/0281.json @@ -0,0 +1,8 @@ +{ + "source_file": "./raw_volume-zh/volume12/exercise4.tex", + "problem_type": "calculation", + "problem": "问题5. 一棵树有 $n_i$ 个顶点的度数为 $i, i=1,2, \\cdots, k, n_2, \\cdots, n_k$ 均为已知数, 问 $n_1$ 应为多少? 若 $n_r(3 \\leqslant r \\leqslant k)$ 末知, $n_j(j \\neq r)$ 均已知, 问 $n_r$ 应为多少?", + "solution": "设 $T$ 有 $n$ 个顶点, $e$ 条边, 则 $n=\\sum_{i=1}^k n_i, e=n-1, \\sum_{i=1}^n d\\left(v_i\\right)=\\sum_{i=1}^k \\dot{m}_i= 2 e=2 n-2=2 \\sum_{i=1}^k n_i-2$, 所以, $n_1=\\sum_{i=2}^k(i-2) n_i+2$.\n对于 $r \\geqslant 3$, 由上面的式子可知 $n_r=\\frac{1}{r-2}\\left[\\sum_{i \\neq r}^k(2-i) n_i-2\\right]$.", + "remark": "", + "figures": [] +} \ No newline at end of file diff --git a/processed_dataset/calculation/0282.json b/processed_dataset/calculation/0282.json new file mode 100644 index 0000000000000000000000000000000000000000..a325ad7347f519c6236d4ece8f7971d6d3b51472 --- /dev/null +++ b/processed_dataset/calculation/0282.json @@ -0,0 +1,8 @@ +{ + "source_file": "./raw_volume-zh/volume12/exercise4.tex", + "problem_type": "calculation", + "problem": "问题7. 平面上有 $n(n \\geqslant 3)$ 条线段, 其中任意 3 条都有公共端点, 则这 $n$ 条线段有一个公共端点.", + "solution": "作图 $G, n$ 条线段的端点为 $G$ 的顶点, 线段为 $G$ 的边, 依题意, $G$ 连通无圈, 故 $G$ 是树, 且最长的链长度为 2 , 故 $G$ 只有一个顶点不是悬挂点, 这点即为 $n$ 条线段的公共端点.", + "remark": "", + "figures": [] +} \ No newline at end of file diff --git a/processed_dataset/calculation/0283.json b/processed_dataset/calculation/0283.json new file mode 100644 index 0000000000000000000000000000000000000000..958aba14fca3b2dba76011202e2438a5549f9ef7 --- /dev/null +++ b/processed_dataset/calculation/0283.json @@ -0,0 +1,8 @@ +{ + "source_file": "./raw_volume-zh/volume12/exercise5.tex", + "problem_type": "calculation", + "problem": "问题1. $n$ 为何值时,完全图 $K_n$ 是圈? $n$ 为何值时, $K_n$ 是一条链? 当 $m, n$ 为何值时,完全偶图 $K_{m, n}$ 是圈?", + "solution": "$n(n \\geqslant 2)$ 为奇数时, $K_n$ 是圈; $n=2$ 时, $K_2$ 是链.\n当 $m, n$ 均为偶数时, $K_{m, n}$ 是圈.", + "remark": "", + "figures": [] +} \ No newline at end of file diff --git a/processed_dataset/calculation/0284.json b/processed_dataset/calculation/0284.json new file mode 100644 index 0000000000000000000000000000000000000000..72dd16d6d1e6059bcd70d9290c2333c7e7ad4454 --- /dev/null +++ b/processed_dataset/calculation/0284.json @@ -0,0 +1,8 @@ +{ + "source_file": "./raw_volume-zh/volume12/exercise5.tex", + "problem_type": "calculation", + "problem": "问题2. 已知图 $G$ 至少要 $k$ 笔才能画成, 删去一边后得到图 $G^{\\prime}$, 问 $G^{\\prime}$ 至少需要几笔才能画成?", + "solution": "设 $G$ 至少有 $2 k$ 个奇顶点, 删去一边得 $G^{\\prime}$, 有三种情况: (1) $G^{\\prime}$ 的奇顶点数少 2 个, 则 $G^{\\prime}$ 至少需 $k-1$ 笔画成; (2) $G^{\\prime}$ 的奇顶点数多 2 个, $G^{\\prime}$ 至少需 $k+1$ 笔画成; (3) $G^{\\prime}$ 的奇顶点数不变, $G^{\\prime}$ 至少需 $k$ 笔画成.", + "remark": "", + "figures": [] +} \ No newline at end of file diff --git a/processed_dataset/calculation/0285.json b/processed_dataset/calculation/0285.json new file mode 100644 index 0000000000000000000000000000000000000000..a2c12789c615b921d1bb0c82bd327c1d6f71a6d2 --- /dev/null +++ b/processed_dataset/calculation/0285.json @@ -0,0 +1,10 @@ +{ + "source_file": "./raw_volume-zh/volume12/exercise5.tex", + "problem_type": "calculation", + "problem": "问题3. 判断如图()中的两个图形是否能一笔画.", + "solution": "这两个图都能一笔画.", + "remark": "", + "figures": [ + "./images/volume12/figures/fig-c5p3.png" + ] +} \ No newline at end of file diff --git a/processed_dataset/calculation/0286.json b/processed_dataset/calculation/0286.json new file mode 100644 index 0000000000000000000000000000000000000000..0f3337a47f8e1feb25b8f9e5fef421f46996c59d --- /dev/null +++ b/processed_dataset/calculation/0286.json @@ -0,0 +1,8 @@ +{ + "source_file": "./raw_volume-zh/volume12/exercise5.tex", + "problem_type": "calculation", + "problem": "问题4. 在圆上任取 $n(n>2)$ 个点, 把每个点用线段与其余各点相连接, 能否一笔画出所有这些线段,使它们首尾相接,最后回到出发点?", + "solution": "当 $n$ 是奇数时可以画成; 当 $n$ 是偶数时不能画成.", + "remark": "", + "figures": [] +} \ No newline at end of file diff --git a/processed_dataset/calculation/0287.json b/processed_dataset/calculation/0287.json new file mode 100644 index 0000000000000000000000000000000000000000..f83b9000405cb78ef641662b3ccdcb3b28fc89ce --- /dev/null +++ b/processed_dataset/calculation/0287.json @@ -0,0 +1,11 @@ +{ + "source_file": "./raw_volume-zh/volume12/exercise5.tex", + "problem_type": "calculation", + "problem": "问题7. 如图(), 在上面由 25 个小正方形组成的图形中,试设计一条从 $A$ 点出发的路径, 走过所有小正方形的边, 最后回到 $A$ 点, 并且使得路径最短.", + "solution": "如图(), 所给的图有 16 个奇顶点 $B_i, C_i(i=1,2, \\cdots, 8)$. 如果要使它是一个圈, 至少要加 8 条边.\n如图所添加的 8 条边 $B_i C_i(i=1,2, \\cdots, 8)$ 后, 能使得该图是一个圈, 图中所示的圈使得路径最短.", + "remark": "", + "figures": [ + "./images/volume12/figures/fig-c5p7.png", + "./images/volume12/figures/fig-c5a7.png" + ] +} \ No newline at end of file diff --git a/processed_dataset/calculation/0288.json b/processed_dataset/calculation/0288.json new file mode 100644 index 0000000000000000000000000000000000000000..a9d867d1115e0da800389304d9cb2c51f8445e2e --- /dev/null +++ b/processed_dataset/calculation/0288.json @@ -0,0 +1,8 @@ +{ + "source_file": "./raw_volume-zh/volume12/exercise6.tex", + "problem_type": "calculation", + "problem": "问题1. $n$ 为何值时, 完全图 $K_n$ 是哈密顿图? 对什么样的 $m, n$, 完全偶图 $K_{m, n}$ 是哈密顿图?", + "solution": "当 $n \\geqslant 3$ 时, $K_n$ 为哈密顿图.\n当 $m=n \\geqslant 2$ 时, 完全偶图 $K_{m, n}$ 是哈密顿图.", + "remark": "", + "figures": [] +} \ No newline at end of file diff --git a/processed_dataset/calculation/0289.json b/processed_dataset/calculation/0289.json new file mode 100644 index 0000000000000000000000000000000000000000..04ea0a889b5d513b733f8aa5a7fe2cdc722cd99b --- /dev/null +++ b/processed_dataset/calculation/0289.json @@ -0,0 +1,10 @@ +{ + "source_file": "./raw_volume-zh/volume12/exercise6.tex", + "problem_type": "calculation", + "problem": "问题3. 正二十面体用纸制成, 能否把它剪成两部分, 使每个面也剪成两部分, 而截痕不通过二十面体的顶点? 个 $1 \\times 1 \\times 1$ 子立方体.\n如果它在一个角上开始, 然后依次走向未吃的立方体, 问它吃完时能否恰在立方体的中心?", + "solution": "如图(), 正二十面体是由 20 个全等正三角形围成的.\n在每个正三角形的中心设一个顶点, 仅当两个正三角形有公共边时, 在相应的两顶点间连一边, 则构成由 12 个正五边形围成的正十二面体.\n从哈密顿周游世界的游戏中已知可从正十二面体上找到一个哈密顿圈, 用剪刀沿此哈密顿圈剪开, 即可把正二十面体剪成两块,且把每个正三角形也剪成两块,截痕不过二十面体顶点.", + "remark": "", + "figures": [ + "./images/volume12/figures/fig-c6a3.png" + ] +} \ No newline at end of file diff --git a/processed_dataset/calculation/0290.json b/processed_dataset/calculation/0290.json new file mode 100644 index 0000000000000000000000000000000000000000..abc24f9d0afe3e46c011aaeea3a115f74473f15a --- /dev/null +++ b/processed_dataset/calculation/0290.json @@ -0,0 +1,8 @@ +{ + "source_file": "./raw_volume-zh/volume12/exercise6.tex", + "problem_type": "calculation", + "problem": "问题4. 一只老鼠吃 $3\\times 3\\times 3$ 立方体的乳酪,其方法是借助于打洞通过所有的 27 个$1\\times 1\\times 1$ 子立方体.\n果它在一个角上开始;然后依次走向未吃的立方体,问它吃完时能否恰在立方体的中心?", + "solution": "作图 $G$ : 顶点表示 $1 \\times 1 \\times 1$ 的立方体, 当且仅当两小立方体有公共面时, 对应的两顶点以边相连.\n易知 $G$ 是偶图, 设 $G=(X, Y ; E)$, 若角上一小立方体对应点属于 $X$, 则中心的小立方体对应的点属于 $Y$, 又因为 $|X|=14$, $|Y|=13$. 故 $G$ 无哈密顿链.", + "remark": "", + "figures": [] +} \ No newline at end of file diff --git a/processed_dataset/calculation/0291.json b/processed_dataset/calculation/0291.json new file mode 100644 index 0000000000000000000000000000000000000000..76a80df4d890ab6e21f65484fb814b333128aa53 --- /dev/null +++ b/processed_dataset/calculation/0291.json @@ -0,0 +1,8 @@ +{ + "source_file": "./raw_volume-zh/volume12/exercise6.tex", + "problem_type": "calculation", + "problem": "问题5. 今要将 6 人分成 3 组 (每组 2 个人)去完成 3 项任务.\n已知每个人至少与其余 5 个人中的 3 个人能相互合作.\n(1) 能否使得每组的 2 个人都能相互合作?(2)你能给出几种不同的分组方案?", + "solution": "(1)用 6 个点 $v_1, v_2, \\cdots, v_6$ 代表六个人, 若 $v_i, v_j$ 能相互合作, 则 $v_i$ 与 $v_j$ 相邻.\n由已知条件知, $d\\left(v_i\\right) \\geqslant 3, i=1,2, \\cdots, 6$. 根据定理四, $G$ 中有一个哈密顿圈 $C=v_{i_1} v_{i_2} \\cdots v_{i_3} v_{i_1}$, 在圈中, 相邻的两点代表的两人是能相互合作的.\n(2) 将 $v_{i_1}, v_{i_2}$ 分在一组, $v_{i_3}, v_{i_4}$ 分在一组, $v_{i_5}, v_{i_6}$ 分在一组, 也可将 $v_{i_6}$, $v_{i_1}$ 分在一组, $v_{i_2}, v_{i_3}$ 分在一组, $v_{i_4}, v_{i_5}$ 分在一组, 这是两种不同的分组方案.", + "remark": "", + "figures": [] +} \ No newline at end of file diff --git a/processed_dataset/calculation/0292.json b/processed_dataset/calculation/0292.json new file mode 100644 index 0000000000000000000000000000000000000000..b4226cac9cc3039ca39d1663b5c2d6ea538f8691 --- /dev/null +++ b/processed_dataset/calculation/0292.json @@ -0,0 +1,8 @@ +{ + "source_file": "./raw_volume-zh/volume12/exercise6.tex", + "problem_type": "calculation", + "problem": "问题6. 某国王有 $2n$个大臣,其中某些大臣互相有怨仇,但每个大臣的仇人(限于大臣内部)不超过 $n-1$ 人 (互为仇人), 问能否让他们围圆桌而坐, 使仇人不相邻?", + "solution": "用 $2 n$ 个顶点表示 $2 n$ 个大臣, 若两人不是仇人, 则相应的顶点相邻, 得图 $G$. 在 $G$ 中, 每个顶点 $v$ 的度 $d(v) \\geqslant(2 n-1)-(n-1)=n$, 根据定理四, $G$ 中有哈密顿圈, 按圈上顶点的顺序安排位置即可.", + "remark": "", + "figures": [] +} \ No newline at end of file diff --git a/processed_dataset/calculation/0293.json b/processed_dataset/calculation/0293.json new file mode 100644 index 0000000000000000000000000000000000000000..ee8143fa2abdc2dc29daf6d84492cb863ccc2fed --- /dev/null +++ b/processed_dataset/calculation/0293.json @@ -0,0 +1,8 @@ +{ + "source_file": "./raw_volume-zh/volume12/exercise6.tex", + "problem_type": "calculation", + "problem": "问题7. 已知在 9 个小孩中,每个小孩至少认识其他 4 个小孩,能否让这 9 个小孩排成一行, 使得每个小孩和与他相邻的小孩都认识?", + "solution": "作图 $G: 9$ 个顶点表示 9 名小孩, 两小孩认识, 则相应的顶点相邻.\n在 $G$ 中, 任意两个顶点 $v$ 与 $v^{\\prime}$, 有 $d(v)+d\\left(v^{\\prime}\\right) \\geqslant 8$, 根据定理二, $G$ 有哈密顿链.\n按链上顶点顺序把 9 名小孩排成一行即可.", + "remark": "", + "figures": [] +} \ No newline at end of file diff --git a/processed_dataset/calculation/0294.json b/processed_dataset/calculation/0294.json new file mode 100644 index 0000000000000000000000000000000000000000..4dd891017e04b739babe9c5ce1dbeeff0848d1c9 --- /dev/null +++ b/processed_dataset/calculation/0294.json @@ -0,0 +1,8 @@ +{ + "source_file": "./raw_volume-zh/volume12/exercise6.tex", + "problem_type": "calculation", + "problem": "问题8. 一位厨师用8种原料做菜,每种菜都用2种原料搭配.\n已知每种原料都至少用在4种菜里,问:能否从这位厨师做的菜中选出4种,恰好包括了 8 种不同的原料?", + "solution": "作图 $G$ : 顶点代表原料, 每种菜对应于一条边.\n在 $G$ 中, 每个顶点的度 $\\geqslant 4$, 根据定理四, $G$ 中有一个哈密顿圈.", + "remark": "", + "figures": [] +} \ No newline at end of file diff --git a/processed_dataset/calculation/0295.json b/processed_dataset/calculation/0295.json new file mode 100644 index 0000000000000000000000000000000000000000..1d064345d40492dbef2bcd9f42bcc82c3a5ba624 --- /dev/null +++ b/processed_dataset/calculation/0295.json @@ -0,0 +1,8 @@ +{ + "source_file": "./raw_volume-zh/volume12/exercise7.tex", + "problem_type": "calculation", + "problem": "问题5. 将平面分成 $f$ 个区域,每两个区域都相邻, 问 $f$ 最大为多少?", + "solution": "考虑对偶图, 由于 $K_5$ 不是平面图, 所以 $f \\leqslant 4$.", + "remark": "", + "figures": [] +} \ No newline at end of file diff --git a/processed_dataset/calculation/0296.json b/processed_dataset/calculation/0296.json new file mode 100644 index 0000000000000000000000000000000000000000..5ebcdfdbe384ac353d03acb39313888c9d8fd491 --- /dev/null +++ b/processed_dataset/calculation/0296.json @@ -0,0 +1,8 @@ +{ + "source_file": "./raw_volume-zh/volume12/exercise7.tex", + "problem_type": "calculation", + "problem": "问题8. 对哪些 $n$, 存在 $n$ 条棱的多面体?", + "solution": "以多面体的顶点为图的顶点, 以多面体的棱为边, 构成一个连通平面图.\n则 $v \\geqslant 4, f \\geqslant 4$. 由欧拉公式 $e=v+f-2 \\geqslant 6$, 即没有棱数少于 6 的多面体.\n若有 $e=7$ 的图, 则 $3 f \\leqslant 2 \\times 7$, 即 $f=4$, 但四个面的多面体只有 6 条棱, 故无 7 条棱的多面体.\n考虑 $k \\geqslant 4$, 以 $k$ 边形为底的棱雉为 $2 k$ 条棱的多面体,而把 $k-1$ 边形为底的棱雉底角处的一个三面角\"锯掉一个小尖儿\", 得 $2 k+1$ 条棱的多面体.\n综上, $n \\geqslant 6, n \\neq 7$ 时, 有 $n$ 条棱的多面体.", + "remark": "", + "figures": [] +} \ No newline at end of file diff --git a/processed_dataset/calculation/0297.json b/processed_dataset/calculation/0297.json new file mode 100644 index 0000000000000000000000000000000000000000..9098e79b12d7670f59826a424977421806d1384e --- /dev/null +++ b/processed_dataset/calculation/0297.json @@ -0,0 +1,8 @@ +{ + "source_file": "./raw_volume-zh/volume12/exercise9.tex", + "problem_type": "calculation", + "problem": "问题4. 在排球单循环赛中, 若 $A$ 队胜 $B$ 队, 或 $A$ 队胜 $C$ 队而 $C$ 队胜 $B$ 队, 则称 $A$ 队优于 $B$ 队.\n称优于所有对手的队为冠军.\n试问.\n按此规定, 是否会出现恰好两个冠军?", + "solution": "参见例 4 .", + "remark": "", + "figures": [] +} \ No newline at end of file diff --git a/processed_dataset/calculation/0298.json b/processed_dataset/calculation/0298.json new file mode 100644 index 0000000000000000000000000000000000000000..66bdbacdc875f726d4af7a905655f1866231cbc4 --- /dev/null +++ b/processed_dataset/calculation/0298.json @@ -0,0 +1,8 @@ +{ + "source_file": "./raw_volume-zh/volume12/exercise9.tex", + "problem_type": "calculation", + "problem": "问题7. 甲、乙、丙、丁四个人比赛乒乓球, 每两个人都要赛一场, 结果甲胜了丁, 且甲、乙、丙三人胜的场数相同,问乙胜几场?", + "solution": "乙胜了两场.", + "remark": "", + "figures": [] +} \ No newline at end of file diff --git a/processed_dataset/calculation/0299.json b/processed_dataset/calculation/0299.json new file mode 100644 index 0000000000000000000000000000000000000000..4893ffe113c0bf2a7d2694b04f0eb289836a015f --- /dev/null +++ b/processed_dataset/calculation/0299.json @@ -0,0 +1,8 @@ +{ + "source_file": "./raw_volume-zh/volume13/chapter1.tex", + "problem_type": "calculation", + "problem": "例1. 设 $m$ 个互异的正偶数与 $n$ 个互异的正奇数的和为 1987 , 求 $3 m+ 4 n$ 的最大值.", + "solution": "分析:解本题的难点在于约束条件较复杂, 可先利用不等式将其化简,进而将其放缩到出现目标函数式.\n设题给的 $m$ 个正偶数为 $a_1, a_2, \\cdots, a_m, n$ 个正奇数为 $b_1, b_2, \\cdots, b_n$, 则\n$$\n\\left(a_1+a_2+\\cdots+a_m\\right)+\\left(b_1+b_2+\\cdots+b_n\\right)=1987 . \\label{eq1}\n$$\n注意到极值函数是关于 $m 、 n$ 的函数,而在约束条件中, $m 、 n$ 仅作为各变量的下标.\n于是, 应将 式\\ref{eq1} 中对 $a_1, a_2, \\cdots, a_m$ 及 $b_1, b_2, \\cdots, b_n$ 的约束转化为对 $m 、 n$ 的约束.\n因为 $a_1, a_2, \\cdots, a_m$ 与 $b_1, b_2, \\cdots, b_n$ 是互异的正偶数与正奇数,所以\n$$\n\\begin{aligned}\n1987 & =\\left(a_1+a_2+\\cdots+a_m\\right)+\\left(b_1+b_2+\\cdots+b_n\\right) \\\\\n& \\geqslant(2+4+6+\\cdots+2 m)+(1+3+\\cdots+2 n-1) \\\\\n& =m^2+n^2+m . \\label{eq2}\n\\end{aligned}\n$$\n注意到我们的目标是: $3 m+4 n \\leqslant A$ (常数) 的形式, 呈现 Cauchy 不等式结构, 所以应将 式\\ref{eq2} 的右边配方, 化为\"平方和\". 从而\n$$\n1987+\\frac{1}{4} \\geqslant\\left(m+\\frac{1}{2}\\right)^2+n^2,\n$$\n$$\n\\begin{aligned}\n\\left(1987+\\frac{1}{4}\\right)\\left(3^2+4^2\\right) & \\geqslant\\left(3^2+4^2\\right)\\left[\\left(m+\\frac{1}{2}\\right)^2+n^2\\right] \\\\\n& \\geqslant\\left(3\\left(m+\\frac{1}{2}\\right)+4 n\\right)^2,\n\\end{aligned}\n$$\n所以 $3 m+\\frac{3}{2}+4 n \\leqslant 5 \\sqrt{1987+\\frac{1}{4}}$, 所以 $3 m+4 n \\leqslant\\left[5 \\sqrt{1987+\\frac{1}{4}}-\\frac{3}{2}\\right]=$ 221.\n下面构造一组数, 使不等式成立等号.\n先找 $(m, n)$, 使 $3 m+4 n=221$.\n此不定方程有多个解, 但为了使 $(m, n)$ 满足 式\\ref{eq2}, 应使相应的偶数和奇数都尽可能小, 这就要求 $m$ 与 $n$ 充分接近.\n通过试验, 得到 $m=27, n=35$ 时, $3 m+4 n=221$, 且 $m^2+n^2+m=1981<1987$, 满足 式\\ref{eq2}.\n取最小的 27 个正偶数为 $a_1=2, a_2=4, \\cdots, a_{27}=54$, 最小的 35 个正奇数为 $b_1=1, b_2=3, \\cdots, b_{34}=67, b_{35}=69$, 则\n$$\n\\left(a_1+a_2+\\cdots+a_{27}\\right)+\\left(b_1+b_2+\\cdots+b_{35}\\right)=1987-6,\n$$\n再将 $b_{35}$ 修改为: $69+6=75$, 得\n$$\n\\left(a_1+a_2+\\cdots+a_{27}\\right)+\\left(b_1+b_2+\\cdots+b_{35}\\right)=1987 .\n$$\n综上所述, $3 m+4 n$ 的最大值为 221 .", + "remark": "注:本例解题的关键, 是将 \\ref{eq1} 式化为 \\ref{eq2} 式, 而后面利用 Cauchy 不等式则不是本质的.\n实际上, 得到 \\ref{eq2} 式后, 求 $3 m+4 n$ 的极值也可用三角代换:\n由 式\\ref{eq2}, 可令 $r=\\sqrt{\\left(m+\\frac{1}{2}\\right)^2+n^2}, m=-\\frac{1}{2}+r \\cos \\theta, n=r \\sin \\theta$,\n则 $3 m+4 n=3 r \\cos \\theta+4 r \\sin \\theta-\\frac{3}{2}=5 r \\sin (\\theta+t)-\\frac{3}{2} \\leqslant 5 r-\\frac{3}{2} \\leqslant 5 \\sqrt{1987+\\frac{1}{4}}-\\frac{3}{2}$ (下同).", + "figures": [] +} \ No newline at end of file diff --git a/processed_dataset/calculation/0300.json b/processed_dataset/calculation/0300.json new file mode 100644 index 0000000000000000000000000000000000000000..dcc834075eb6199db40ccb60b680e01e23dbb06e --- /dev/null +++ b/processed_dataset/calculation/0300.json @@ -0,0 +1,8 @@ +{ + "source_file": "./raw_volume-zh/volume13/chapter1.tex", + "problem_type": "calculation", + "problem": "例2. 设 $x_1, x_2, \\cdots, x_n \\in \\mathbf{R}^{+}, \\sum_{i=1}^n \\frac{1}{x_i}=A$ (常数), 对给定的正整数 $k$, 求 $\\sum \\frac{1}{x_{i_1}+x_{i_2}+\\cdots+x_{i_k}}$ 的最大值.\n其中求和对 $1,2, \\cdots, n$ 中的所有 $k-$ 元数组 $\\left(i_1, i_2, \\cdots, i_k\\right)$ 进行.", + "solution": "分析:解本题的难点在于目标函数较复杂, 期望利用不等式将其化简.\n由目标函数的结构特征, 想到将 $\\frac{1}{x_{i_1}+x_{i_2}+\\cdots+x_{i_k}}$ 化为 $\\frac{1}{x_{i_1}}+\\frac{1}{x_{i_2}}+\\cdots+ \\frac{1}{x_{i_k}}$ 以利用条件 $\\sum_{i=1}^n \\frac{1}{x_i}=A$. 这恰好符合\"倒数型不等式\": $\\frac{1}{a_1}+\\frac{1}{a_2}+\\cdots+\\frac{1}{a_k} \\geqslant\\frac{k^2}{a_1+a_2+\\cdots+a_k}$ 的特征.\n于是, 利用\"倒数型不等式\", 有\n$$\n\\frac{1}{a_1+a_2+\\cdots+a_k} \\leqslant \\frac{\\frac{1}{a_1}+\\frac{1}{a_2}+\\cdots+\\frac{1}{a_k}}{k^2} .\n$$\n所以 $\\sum \\frac{1}{x_{i_1}+x_{i_2}+\\cdots+x_{i_k}} \\leqslant \\sum \\frac{\\frac{1}{x_{i_1}}+\\frac{1}{x_{i_2}}+\\cdots+\\frac{1}{x_{i_k}}}{k^2}$\n$$\n=\\frac{1}{k^2} \\sum\\left(\\frac{1}{x_{i_1}}+\\frac{1}{x_{i_2}}+\\cdots+\\frac{1}{x_{i_k}}\\right) .\n$$\n考察上式右边 \"和式\"中每个项 $\\frac{1}{x_{i_j}}(j=1,2, \\cdots, k)$ 出现的次数.\n显然, $\\frac{1}{x_{i_j}}$ 出现一次, 等价于出现一个 $\\left(\\frac{1}{x_1}, \\frac{1}{x_2}, \\cdots, \\frac{1}{x_k}\\right)$ 的含 $\\frac{1}{x_{i_j}}$ 的 $k$-组合.\n因为含有 $\\frac{1}{x_{i_j}}$ 的 $k$-组合有 $\\mathrm{C}_{n-1}^{k-1}$ 个, 所以 $\\frac{1}{x_{i_j}}$ 在\"和式\" 中共出现 $\\mathrm{C}_{n-1}^{k-1}$ 次, 所以\n$$\n\\frac{1}{k^2} \\sum\\left(\\frac{1}{x_{i_1}}+\\frac{1}{x_{i_2}}+\\cdots+\\frac{1}{x_{i_k}}\\right)=\\frac{1}{k^2} \\mathrm{C}_{n-1}^{k-1} \\sum_{i=1}^n \\frac{1}{x_i}=\\frac{A}{k^2} \\mathrm{C}_{n-1}^{k-1} \\text {. }\n$$\n其中等式在 $x_1=x_2=\\cdots=x_n=\\frac{n}{A}$ 时成立.\n故 $\\sum \\frac{1}{x_{i_1}+x_{i_2}+\\cdots+x_{i_k}}$ 的最大值为 $\\frac{A}{k^2} \\mathrm{C}_{n-1}^{k-1}$.", + "remark": "", + "figures": [] +} \ No newline at end of file diff --git a/processed_dataset/calculation/0301.json b/processed_dataset/calculation/0301.json new file mode 100644 index 0000000000000000000000000000000000000000..33f9c3231a409f8a0d5cc61d2a060d90860be024 --- /dev/null +++ b/processed_dataset/calculation/0301.json @@ -0,0 +1,8 @@ +{ + "source_file": "./raw_volume-zh/volume13/chapter1.tex", + "problem_type": "calculation", + "problem": "例3. 设 $P$ 是体积为 1 的正四面体 $T$ 内 (包括边界) 的一个点, 过 $P$ 作 4 个平面平行 $T$ 的 4 个面, 将 $T$ 分成 14 块, $f(P)$ 是那些既不是四面体也不是平行六面体的几何体的体积之和, 求 $f(P)$ 的取值范围.", + "solution": "解: $P$ 到正四面体 $A B C D$ 的四个面的距离为 $d_1 、 d_2 、 d_3 、 d_4$.\n令 $x_i=\\frac{d_i}{h}, h$ 为正四面体的高.\n则 $\\sum_{i=1}^4 x_i=1$.\n由于 $T$ 分成的 14 块中, 显然有 4 个体积分别为 $x_i^3$ 的四面体.\n此外, 还有 4 个体积分别为 $6 \\prod_{\\substack{j \\neq i \\\\ 1 \\leqslant j \\leqslant 4}} x_j$ 的平行六面体 $(i=1,2,3,4)$. 比如, 以 $A$ 出发的 3 条棱为 3 度方向可得一个平行六面体, 由对称性可作出 4 个平行六面体.\n于是,\n$$\nf(P)=1-\\sum_{i=1}^4 x_i^3-6 \\sum_{1 \\leqslant i3)$ 名演员, 他们编排了一些节目, 每个节目都由 3 个演员同台表演.\n在一次演出中, 他们发现: 能适当安排若干个节目, 使团中每 2 个演员都恰有一次在这次演出中同台表演, 求 $n$ 的最小值.", + "solution": "解: $n$ 个点表示 $n$ 个演员, 若某 2 个演员有一次同台表演则将对应的点连边, 那么, 本题的条件等价于: 能将 $n$ 阶完全图 $K_n$ 分割为若干个 3 阶完全图 $K_3$, 使每一条边都恰属于一个 $K_3$.\n显然, $\\mathrm{C}_3^2 \\mid \\mathrm{C}_n^2$, 即 $6 \\mid n(n-1)$, 所以 $3 \\mid n$, 或 $3 \\mid n-1$.\n其次, (研究 $n$ 的另外的性质, 缩小包围圈), 考察含点 $A$ (以为顶点) 的边,\n共有 $n-1$ 条, 每条边都恰属于一个 $K_3$, 从而共有 $n-1$ 个含点 $A$ (以为顶点) 的 $K_3$. 但每个含点 $A$ 的 $K_3$ 都有 2 条含点 $A$ 的边, 从而每个 $K_3$ 都被计算 2 次, 于是 $2 \\mid n-1$, 所以 $n$ 为奇数.\n由上可知, $3 \\mid n$ ( $n$ 为奇数), 或 $6 \\mid n-1$, 即 $n=6 k+3$, 或 $6 k+1\\left(k \\in \\mathbf{N}_{+}\\right)$, 于是 $n \\geqslant 7$.\n当 $n=7$ 时,将 7 个点用 $0,1,2,3,4,5,6$ 表示, 对 $m=0,1,2,3,4$ , $5,6,7$, 令 $m 、 m+1 、 m+3$ 组成一个 $K_3$ (下标模 7 理解, 也就是将 $0 、 1 、 3$ 构成的 $K_3$ 依次旋转 6 次), 则 7 个 $K_3$ 是合乎条件的分割.\n综上所述, $n$ 的最小值为 7 .", + "remark": "", + "figures": [] +} \ No newline at end of file diff --git a/processed_dataset/calculation/0311.json b/processed_dataset/calculation/0311.json new file mode 100644 index 0000000000000000000000000000000000000000..ad4028de5d92c9150afc586d1072a3529cb07168 --- /dev/null +++ b/processed_dataset/calculation/0311.json @@ -0,0 +1,8 @@ +{ + "source_file": "./raw_volume-zh/volume13/chapter13.tex", + "problem_type": "calculation", + "problem": "例8. 正 2006 边形 $P$ 的一条对角线称为好的, 如果它的两端点将 $P$ 的边界分成的两部分各含 $P$ 的奇数条边.\n特别地, 称 $P$ 的边也是好的.\n设 $P$ 被不在 $P$ 的内部相交的 2003 条对角线剖分为三角形,试求这种剖分图中有两条边为好的等腰三角形个数的最大值.", + "solution": "解:两条边为好的等腰三角形为好三角形, 先考察特例.\n对于正方形,本质上只有一种剖分, 此时好三角形个数为 2 ;\n对于正 6 边形,本质上只有 3 种剖分, 此时好三角形个数最大值为 3 , 而且我们发现达到最大值时,好三角形的腰都是 $P$ 的边.\n对一般情况, 不难发现, 正 $2 n$ 边形的剖分中好三角形个数最大值为 $n$.\n实际上,对于剖分图中的任一三角形 $A B C, P$ 的边界被 $A 、 B 、 C$ 分为 3 段,将 $A-B$ 段 (按逆时钟方向从 $A$ 到 $B$ ) 所含 $P$ 的边数记作 $m(A B)$, 以此类推.\n由于 $m(A B)+m(B C)+m(C A)=2006$, 故等腰三角形若有两条好边, 则恰有两条好边,且两条好边是两腰 (否则 3 条边都是好边,矛盾).\n考虑任一好三角形 $A B C$, 其中 $A B=A C$, 若 $A-B$ 段上有别的好三角形, 则将其两腰所截下的 $P$ 的边全部去掉, 则去掉的 $P$ 的边数为偶数,如此下去, 直至 $A-B$ 段上没有好三角形, 由于 $A-B$ 段上共有奇数条边, 至少有一条边 $\\alpha$ 没有去掉(如果 $A B$ 本身是 $P$ 的一条边,则 $\\alpha=A B$ ), $\\alpha$ 不属于比 $A B$ 小的腰段.\n同理, $A-C$ 段上也去掉若干个好三角形后有 $P$ 的一边 $\\beta$ 不属于比 $A C$ 小的腰段,令 $\\triangle A B C$ 对应于 2 元集 $\\{\\alpha, \\beta\\}$.\n对于同一剖分中的两个不同的好三角形 $\\triangle A B C 、 \\triangle A_1 B_1 C_1$, 它们对应的 2 元集分别为 $\\{\\alpha, \\beta\\} 、\\left\\{\\alpha_1, \\beta_1\\right\\}$. 如果 $\\triangle A_1 B_1 C_1$ 不位于 $\\triangle A B C$ 的腰段, 则 $\\triangle A_1 B_1 C_1$ 位于 $\\triangle A B C$ 的 $B-C$ 段, 此时, $\\{\\alpha, \\beta\\}$ 中的边在 $\\triangle A B C$ 的腰段上, $\\left\\{\\alpha_1, \\beta_1\\right\\}$ 中的边在 $\\triangle A B C$ 的 $B-C$ 段上, 所以 $\\{\\alpha, \\beta\\}$ 与 $\\left\\{\\alpha_1, \\beta_1\\right\\}$ 没有公共的边; 如果 $\\triangle A_1 B_1 C_1$ 位于 $\\triangle A B C$ 的腰段上, 设在 $A-B$ 段上, 则 2 元集 $\\{\\alpha, \\beta\\}$ 中的边属于去掉 $\\triangle A_1 B_1 C_1$ 的腰段上的边, 而 2 元集 $\\left\\{\\alpha_1, \\beta_1\\right\\}$ 中的边是 $\\triangle A_1 B_1 C_1$ 的腰段上的边,从而两个 2 元集没有公共的边.\n注意到 2006 条边最多有 $\\frac{2006}{2}=1003$ 个两两无公共元的 2 元子集, 所以好三角形不多于 1003 个.\n最后, 设 $P=A_1 A_2 \\cdots A_{2006}$, 用对角线 $A_1 A_{2 k+1}(1 \\leqslant k \\leqslant 1002)$ 及 $A_{2 k+1} A_{2 k+3} (1 \\leqslant k \\leqslant 1001)$ 所作的剖分图恰有 1003 个好三角形.\n因此,好三角形个数的最大值是 1003 .", + "remark": "", + "figures": [] +} \ No newline at end of file diff --git a/processed_dataset/calculation/0312.json b/processed_dataset/calculation/0312.json new file mode 100644 index 0000000000000000000000000000000000000000..a0164efe46e4534641ee167f950b4d43db8c8038 --- /dev/null +++ b/processed_dataset/calculation/0312.json @@ -0,0 +1,8 @@ +{ + "source_file": "./raw_volume-zh/volume13/chapter2.tex", + "problem_type": "calculation", + "problem": "例1. 设 $x 、 y 、 z$ 为非负实数, $x+y+z=1$, 求 $F=2 x^2+y+3 z^2$ 的最值.", + "solution": "解:先, 采用代入消元, 有 $y=1-x-z$, 所以\n$$\n\\begin{aligned}\nF & =2 x^2+1-x-z+3 z^2 \\\\\n& =2\\left(x-\\frac{1}{4}\\right)^2+3\\left(z-\\frac{1}{6}\\right)^2+\\frac{19}{24} \\\\\n& \\geqslant \\frac{19}{24} .\n\\end{aligned}\n$$\n又 $F\\left(\\frac{1}{4}, \\frac{7}{12}, \\frac{1}{6}\\right)=\\frac{19}{24}$, 所以 $F$ 的最小值为 $\\frac{19}{24}$.\n下面用求累次极值的方法求 $F$ 的最大值.\n固定变量 $z$, 则 $x+y=1-z$ (常数).\n对 $F=2 x^2+y+3 z^2$, 因为 $z$ 为常数, 所以只须求 $2 x^2+y=A$ 的最大值, 其中 $x+y=1-z$. 为叙述问题方便, 令 $1-z=t$, 则 $x+y=t, 0 \\leqslant x, y \\leqslant t \\leqslant 1, t$ 为常数.\n因为 $A=2 x^2+y=2 x^2+t-x$ (代入消元), 注意到 $0 \\leqslant x \\leqslant t \\leqslant 1$, 而二次函数的开口向上, 顶点处不是最大值, 所以 $A$ 只能在 $x=0$ 或 $x=t$ 处取最大值.\n所以,\n$$\ng(z)=A_{\\max }=\\max \\left\\{t, 2 t^2\\right\\}= \\begin{cases}t & \\left(0 \\leqslant t \\leqslant \\frac{1}{2}\\right), \\\\ 2 t^2 & \\left(\\frac{1}{2} \\leqslant t \\leqslant 1\\right) .\\end{cases}\n$$\n还原成原变量, 有\n$$\ng(z)=A_{\\max }=\\max \\left\\{1-z, 2(1-z)^2\\right\\}= \\begin{cases}1-z & \\left(\\frac{1}{2} \\leqslant z \\leqslant 1\\right) \\\\ 2(1-z)^2 & \\left(0 \\leqslant z \\leqslant \\frac{1}{2}\\right)\\end{cases}\n$$\n再对 $g(z)$ 求最大值.\n$$\n\\begin{aligned}\n& \\text { 当 } 0 \\leqslant z \\leqslant \\frac{1}{2} \\text { 时, } F \\leqslant g(z)+3 z^2=2(1-z)^2+3 z^2=5 z^2-4 z+2 \\leqslant 2 . \\\\\n& \\text { 当 } \\frac{1}{2} \\leqslant z \\leqslant 1 \\text { 时, } F \\leqslant g(z)+3 z^2=(1-z)+3 z^2 \\leqslant 3 .\n\\end{aligned}\n$$\n由此可见, 对一切 $x 、 y 、 z$, 恒有 $F \\leqslant 3$, 其中等式在 $x=y=0, z=1$ 时成立.\n所以 $F$ 的最大值为 3 .\n综上所述, $F$ 的最小值为 $\\frac{19}{24}$, 最大值为 3 .", + "remark": "注:本题求 $F$ 的最大值时, 若对次数与系数进行放缩, 则解答异常简单.\n实际上, $2 x^2+y+3 z^2 \\leqslant 2 x+y+3 z \\leqslant 3 x+3 y+3 z \\leqslant 3$.", + "figures": [] +} \ No newline at end of file diff --git a/processed_dataset/calculation/0313.json b/processed_dataset/calculation/0313.json new file mode 100644 index 0000000000000000000000000000000000000000..ccd973a1457bf62d92940a6fbc909877f8d15eb3 --- /dev/null +++ b/processed_dataset/calculation/0313.json @@ -0,0 +1,8 @@ +{ + "source_file": "./raw_volume-zh/volume13/chapter2.tex", + "problem_type": "calculation", + "problem": "例2. 有 1988 个单位立方体, 用它们(全部或一部分)拼成高为 1 , 底边长为 $a 、 b 、 c(a1\\left(x_i1$. 此时, 自然的想法是: 将 $x_i$ 减小 1 , 将 $y_i$ 增大 1 . 也就是将点 $P_i\\left(x_i, y_i\\right)$ 调整为 $P_i^{\\prime}\\left(x_i-\\right. \\left.1, y_i+1\\right)$, 其余点不变.\n但调整后的路线是否仍合乎条件? 显然, 要使调整后的路线仍合乎条件, 则 $P_i$ 应该满足: $P_{i-1} P_i$ 是横向边且 $P_i P_{i+1}$ 是纵向边.\n但 $P_i$ 末必满足这样的条件.\n此时, 观察路径, 发现一定有一个点 $P_t\\left(x_t, y_t\\right)$ 满足这样的条件, 即路径中存在这样连续三点 $P_{t-1}\\left(x_{t-1}, y_{t-1}\\right) 、 P_t\\left(x_t, y_t\\right)$ 、 $P_{t+1}\\left(x_{t+1}, y_{t+1}\\right)$, 使得 $P_{t-1} P_t$ 是横向边且 $P_t P_{t+1}$ 是纵向边, 且 $x_t-y_t>1$.\n实际上, 若 $P_{i-1} P_i$ 是纵向边, 则考察横坐标为 $x_i$ 且纵坐标最小的点.\n设其为 $P_t\\left(x_t, y_t\\right)$, 其中 $x_t=x_i, y_tx_i-y_i>1$. 又因为 $x_t>1+y_t>1$, 所以到达 $P_t\\left(x_t, y_t\\right)$ 之前一定有横向边.\n于是由 $y_t$ 的最小性可知, $P_{t-1} P_t$ 是横向边, $P_t P_{t+1}$ 是纵向边.\n若 $P_{i-1} P_i$ 是横向边, 则考察纵坐标为 $y_i$ 且横坐标最大的点, 设为 $P_t\\left(x_t, y_t\\right)$, 其中 $x_t>x_i, y_t=y_i$. 此时 $x_t-y_t=x_t-y_i>x_i-y_i>1$. 又因为 $y_i1$ 时, 则一定存在这样的连续三点 $P_{t-1}\\left(x_{t-1}, y_{t-1}\\right) 、 P_t\\left(x_t, y_t\\right)$ 、 $P_{t+1}\\left(x_{t+1}, y_{t+1}\\right)$, 使得 $P_{t-1} P_t$ 是横向边且 $P_t P_{t+1}$ 是纵向边, 且 $x_t-y_t>1$. 于是, 用 $P_t^{\\prime}\\left(x_t-1, y_t+1\\right)$ 代替 $P_t\\left(x_t, y_t\\right)$, 得到的路径仍合乎要求.\n但 $\\left(x_t-1\\right) \\left(y_t+1\\right)=x_t y_t+x_t-y_t-1>x_t y_t$. 所以调整后 $S$ 的值增加, 矛盾.\n由上可知, 对路径中的任何一个点 $P_i\\left(x_i, y_i\\right)$, 若 $x_i \\neq y_i$, 则从 $P_i\\left(x_i, y_i\\right)$ 出发的边是唯一的,下一个点是将 $P_i\\left(x_i, y_i\\right)$ 的坐标中较小的一个增加 1 . 而 $x_i=y_i$ 时, 则从 $P_i\\left(x_i, y_i\\right)$ 出发的边有两种选择,下一个点是将 $P_i\\left(x_i, y_i\\right)$ 的横坐标或纵坐标增加 1 . 于是, 当 $m \\geqslant n$ 时, 其路径为:\n$$\n\\begin{aligned}\n& A(1,1) \\rightarrow P_2(2,1) \\text { 或 } P_2(1,2) \\rightarrow P_3(2,2) \\rightarrow P_4(2,3) \\text { 或 } P_4(3,2) \\rightarrow \\\\\n& P_5(3,3) \\rightarrow \\cdots \\rightarrow P_{2 n-1}(n, n) \\rightarrow P_{2 n}(n+1, n) \\rightarrow P_{2 n+1}(n+2, n) \\rightarrow \\cdots \\rightarrow \\\\\n& P_{m+n-1}(m, n) . \\\\\n& \\quad \\text { 此时, } \\quad S_{\\max }=\\sum_{i=1}^n i^2+\\sum_{i=1}^{n-1} i(i+1)+n \\sum_{i=1}^{m-n}(n+i) \\\\\n& =\\frac{1}{6} n\\left(3 m^2+n^2+3 m-1\\right) .\n\\end{aligned}\n$$\n当 $m0$, 所以 $f(x)$ 是凸函数, 所以当 $f(x)\\left(-\\frac{1}{\\sqrt{3}} \\leqslant x \\leqslant \\sqrt{3}\\right)$ 达到最大时, 必有 $x \\in\\left\\{-\\frac{1}{\\sqrt{3}}, \\sqrt{3}\\right\\}$. 由对称性, 对任何两个变量 $x_i 、 x_j(1 \\leqslant i1$ 时,若固定设 $x_2, x_3, \\cdots, x_n$, 则\n$$\n\\begin{aligned}\nf & =y-x^2=\\frac{1}{n} \\sum_{j=1}^n x_j^2-\\left(\\frac{1}{n} \\sum_{j=1}^n x_j\\right)^2 \\\\\n& =\\frac{n-1}{n^2} x_1^2-\\left(\\frac{2}{n^2} \\sum_{j=2}^n x_j\\right) x_1+\\frac{1}{n} \\sum_{j=2}^n x_j^2-\\left(\\frac{1}{n} \\sum_{j=2}^n x_j\\right)^2,\n\\end{aligned}\n$$\n由于 $f$ 是 $x_1$ 在区间 $[a, a+1]$ 上的二次函数, 且二次项系数 $\\frac{n-1}{n^2}>0$, 所以\n$$\nf \\leqslant \\max \\left\\{f\\left(a, x_2, x_3, \\cdots, x_n\\right), f\\left(a+1, x_2, x_3, \\cdots, x_n\\right)\\right\\} .\n$$\n这表明, 当 $f$ 达到最大值时, 必有 $x_1 \\in\\{a, a+1\\}$, 由对称性, 当 $f$ 达到最大值时, 必有 $x_1, x_2, \\cdots, x_n \\in\\{a, a+1\\}$, 即\n$$\nf \\leqslant \\max _{x_i \\in\\{a, a+1\\}, 1 \\leqslant i \\leqslant n}\\left\\{f\\left(x_1, x_2, x_3, \\cdots, x_n\\right)\\right\\} .\n$$\n设 $f$ 达到最大值时, $x_1, x_2, \\cdots, x_n$ 中有 $s$ 个为 $a$, 另 $n-s$ 个为 $a+1$, 则\n$$\n\\begin{aligned}\n& \\max _{x_i \\in\\{a, a+1\\}, 1 \\leqslant i \\leqslant n}\\left\\{f\\left(x_1, x_2, x_3, \\cdots, x_n\\right)\\right\\} \\\\\n= & \\frac{1}{n}\\left[s a^2+(n-s)(a+1)^2\\right]-\\frac{1}{n^2}[s a+(n-s)(a+1)]^2 \\\\\n= & \\frac{1}{n^2} s(n-s) \\leqslant\\left\\{\\begin{array}{cl}\n\\frac{n^2-1}{4 n^2} & (n \\text { 为奇 }) ; \\\\\n\\frac{1}{4} & (n \\text { 为偶 }) .\n\\end{array}\\right.\n\\end{aligned}\n$$\n当 $x_1, x_2, \\cdots, x_n$ 中有 $\\left[\\frac{n+1}{2}\\right]$ 个为 $a$, 另 $n-\\left[\\frac{n+1}{2}\\right]$ 个为 $a+1$ 时, 上式等号成立, 故\n$$\nf_{\\max }=\\left\\{\\begin{array}{cl}\n\\frac{n^2-1}{4 n^2} & (n \\text { 为奇 }) ; \\\\\n\\frac{1}{4} & (n \\text { 为偶 }) .\n\\end{array}\\right.\n$$", + "remark": "", + "figures": [] +} \ No newline at end of file diff --git a/processed_dataset/calculation/0318.json b/processed_dataset/calculation/0318.json new file mode 100644 index 0000000000000000000000000000000000000000..156bfc41a7b8fd2c4f8e4dd458eac8c5d2995170 --- /dev/null +++ b/processed_dataset/calculation/0318.json @@ -0,0 +1,8 @@ +{ + "source_file": "./raw_volume-zh/volume13/chapter5.tex", + "problem_type": "calculation", + "problem": "例1. 给定 $2 n$ 个实数: $a_1 \\leqslant a_2 \\leqslant \\cdots \\leqslant a_n, b_1 \\leqslant b_2 \\leqslant \\cdots \\leqslant b_n$. 令\n$$\nF=a_1 b_{i_1}+a_2 b_{i_2}+\\cdots+a_n b_{i_n}\n$$\n其中 $b_{i_1}, b_{i_2}, \\cdots, b_{i_n}$ 是 $b_1, b_2, \\cdots, b_n$ 的一个排列.\n求 $F$ 的最大值与最小值(排序不等式).", + "solution": "分析:直观, 想到 $a_i$ 中最小的与 $b_i$ 中最小的相乘、 $a_i$ 中最大的与 $b_i$ 中最大的相乘时, 其 $F$ 的值最大.\n由此想到通过磨光变换构造 $a_1 b_1$, $a_2 b_2, \\cdots, a_n b_n$.\n解若 $F$ 的表达式中无 $a_1 b_1$ 这一项,则考察分别含有 $a_1 、 b_1$ 的两个项: $a_1 b_{i_1}$ 和 $a_j b_1$, 将它们分别调整为 $a_1 b_1 、 a_j b_{i_1}$, 那么,\n$a_1 b_1+a_j b_{i_1}-\\left(a_1 b_{i_1}+a_j b_1\\right)=\\left(a_1-a_j\\right)\\left(b_1-b_{i_1}\\right) \\geqslant 0$ (磨光工具).\n由此可见,调整出 $a_1 b_1$ 这个项后, $F$ 的值不减.\n固定这个积 $a_1 b_1$ 不动, 再考察余下的 $n-1$ 个积, 如此下去, 最多调整 $n-1$ 次, 即可出现 $a_1 b_1, a_2 b_2, \\cdots, a_n b_n, F$ 的值恒保持不减, 于是 $F_1=a_1 b_1+ a_2 b_2+\\cdots+a_n b_n$ 是 $F$ 的最大值.\n同样,当出现 $a_1 b_n, a_2 b_{n-1}, \\cdots, a_n b_1$, 则 $F$ 达到最小值.", + "remark": "", + "figures": [] +} \ No newline at end of file diff --git a/processed_dataset/calculation/0319.json b/processed_dataset/calculation/0319.json new file mode 100644 index 0000000000000000000000000000000000000000..834c66544301e3c284eb53aea210fbc933d87fcc --- /dev/null +++ b/processed_dataset/calculation/0319.json @@ -0,0 +1,8 @@ +{ + "source_file": "./raw_volume-zh/volume13/chapter5.tex", + "problem_type": "calculation", + "problem": "例4. 设 $f(x)=a x^2+b x+c$ 的所有系数都是正的, 且 $a+b+c=1$. 对所有满足: $x_1 x_2 \\cdots x_n=1$ 的正数组 $x_1, x_2, \\cdots, x_n$, 求 $f\\left(x_1\\right) f\\left(x_2\\right) \\cdots f\\left(x_n\\right)$ 的最小值.", + "solution": "解:$f(1)=a+b+c=1$. 若 $x_1=x_2=\\cdots=x_n=1$, 则 $f\\left(x_1\\right) f\\left(x_2\\right) \\cdots f\\left(x_n\\right)=1$.\n若 $x_1, x_2, \\cdots, x_n$ 不全为 1 , 则由 $x_1 x_2 \\cdots x_n=1$ 知, 其中必有一个小于 1 , 也必有一个大于 1 . 不妨设 $x_1>1, x_2<1$, 将 $x_1 、 x_2$ 用 $1 、 x_1 x_2$ 代替, 考察其变化:\n$$\n\\begin{gathered}\nf\\left(x_1\\right) f\\left(x_2\\right)=\\left(a x_1^2+b x_1+c\\right)\\left(a x_2^2+b x_2+c\\right) \\\\\n=a^2 x_1^2 x_2^2+b^2 x_1 x_2+c^2+a b\\left(x_1^2 x_2+x_1 x_2^2\\right) \\\\\n+a c\\left(x_1^2+x_2^2\\right)+b c\\left(x_1+x_2\\right) \\\\\nf(1) f\\left(x_1 x_2\\right)=(a+b+c)\\left(a x_1^2 x_2^2+b x_1 x_2+c\\right) \\\\\n=a^2 x_1^2 x_2^2+b^2 x_1 x_2+c^2+a b\\left(x_1^2 x_2^2+x_1 x_2\\right) \\\\\n+a c\\left(x_1^2 x_2^2+1\\right)+b c\\left(x_1 x_2+1\\right), \\\\\nf\\left(x_1\\right) f\\left(x_2\\right)-f(1) f\\left(x_1 x_2\\right) \\\\\n=a b x_1 x_2\\left(x_1+x_2-x_1 x_2-1\\right)+a c\\left(x_1^2+x_2^2-x_1^2 x_2^2-1\\right) \\\\\n+b c\\left(x_1+x_2-x_1 x_2-1\\right) \\\\\n=-a b x_1 x_2\\left(x_1-1\\right)\\left(x_2-1\\right)-a c\\left(x_1^2-1\\right)\\left(x_2^2-1\\right) \\\\\n-b c\\left(x_1-1\\right)\\left(x_2-1\\right)>0 .\n\\end{gathered}\n$$\n反复进行上述变换, 得 $f\\left(x_1\\right) f\\left(x_2\\right) \\cdots f\\left(x_n\\right) \\geqslant f(1) f(1) \\cdots f(1)=1$. 故\n$f\\left(x_1\\right) f\\left(x_2\\right) \\cdots f\\left(x_n\\right)$ 的最小值为 1 .", + "remark": "", + "figures": [] +} \ No newline at end of file diff --git a/processed_dataset/calculation/0320.json b/processed_dataset/calculation/0320.json new file mode 100644 index 0000000000000000000000000000000000000000..5b390e768e0deba45c1363822f0737d50ca435c2 --- /dev/null +++ b/processed_dataset/calculation/0320.json @@ -0,0 +1,8 @@ +{ + "source_file": "./raw_volume-zh/volume13/chapter5.tex", + "problem_type": "calculation", + "problem": "例5. 对于满足条件 $x_1+x_2+\\cdots+x_n=1$ 的非负实数 $x_1, x_2, \\cdots, x_n$, 求 $S=\\sum_{j=1}^n\\left(x_j^4-x_j^5\\right)$ 的最大值.", + "solution": "分析:考虑 $n=2 、 3$ 的情形, 可发现 $\\sum_{j=1}^n\\left(x_j^4-x_j^5\\right)$ 达到最大时, $x_1, x_2$, $\\cdots, x_n$ 中最多有 2 个不为零.\n考虑这样的磨光工具 $(x, y) \\rightarrow(x+y, 0)$, 希望有 $(x+y)^4-(x+y)^5+0^4-0^5>x^4-x^5+y^4-y^5$. 此不等式等价于 $4 x^2+ 4 y^2+6 x y>5 x^3+5 y^3+10 x^2 y+10 x y^2$. 此式左边 $=\\frac{7}{2}\\left(x^2+y^2\\right)+\\frac{1}{2}\\left(x^2+\\right. \\left.y^2\\right)+6 x y \\geqslant \\frac{7}{2}\\left(x^2+y^2\\right)+x y+6 x y=\\frac{7}{2}(x+y)^2$, 而右边 $\\leqslant 5 x^3+5 y^3+ 15 x^2 y+15 x y^2=5(x+y)^3$. 于是, 上式成立的一个充分条件是 $\\frac{7}{2}(x+y)^2> 5(x+y)^3$, 即 $x+y<\\frac{7}{10}$. 这样, 我们得到如下的引理: 如果 $x+y<\\frac{7}{10}$, 则 $(x+y)^4-(x+y)^5>x^4-x^5+y^4-y^5$.\n解设 $x_1, x_2, \\cdots, x_n$ 中非零数的个数为 $k$, 不妨设 $x_1 \\geqslant x_2 \\geqslant \\cdots \\geqslant x_k>0, x_{k+1}=x_{k+2}=\\cdots=x_n=0$. 如果 $k \\geqslant 3$, 则令 $x_i^{\\prime}=x_i(i=1,2, \\cdots$, $k-2), x_{k-1}^{\\prime}=x_{k-1}+x_k, x_k^{\\prime}=x_{k+1}^{\\prime}=\\cdots=x_n^{\\prime}=0$, 因为 $x_{k-1}+x_k \\leqslant \\frac{2}{n} \\leqslant \\frac{2}{3}<\\frac{7}{10}$, 由引理, 有 $\\sum_{j=1}^n\\left(x_j^{\\prime 4}-x_j^{\\prime 5}\\right)>\\sum_{j=1}^n\\left(x_j^4-x_j^5\\right)$. 只要非零变量个数不小于 3 , 上述调整就可进行.\n最多经过 $n-2$ 次调整, 可以将 $x_3, \\cdots, x_n$ 调为 0 , 而 $S$ 不减.\n记此时的 $x_1 、 x_2$ 为 $a 、 b$, 则 $a+b=1$, 且 $S=a^4(1-a)+b^4(1-b)=a^4 b+ a b^4=a b\\left(a^3+b^3\\right)=a b(a+b)\\left(a^2-a b+b^2\\right)=a b\\left[(a+b)^2-3 a b\\right]=a b(1- 3 a b)=\\frac{1}{3}(3 a b)(1-3 a b) \\leqslant \\frac{1}{3} \\times \\frac{1}{4}=\\frac{1}{12}$.\n又当 $x_1=\\frac{3+\\sqrt{3}}{6}, x_2=\\frac{3-\\sqrt{3}}{6}, x_3=\\cdots=x_n=0$ 时 $S=\\frac{1}{12}$, 故 $S_{\\max }=\\frac{1}{12}$.", + "remark": "", + "figures": [] +} \ No newline at end of file diff --git a/processed_dataset/calculation/0321.json b/processed_dataset/calculation/0321.json new file mode 100644 index 0000000000000000000000000000000000000000..2efaf1deebce86253937174a248fb441ba53b9d5 --- /dev/null +++ b/processed_dataset/calculation/0321.json @@ -0,0 +1,8 @@ +{ + "source_file": "./raw_volume-zh/volume13/chapter5.tex", + "problem_type": "calculation", + "problem": "例6. 设 $x 、 y 、 z$ 为非负实数, 满足: $x+y+z=1$, 求 $Q=\\sqrt{2-x}+ \\sqrt{2-y}+\\sqrt{2-z}$ 的最小值.", + "solution": "解:对称性, 不妨设 $x \\leqslant y \\leqslant z$, 令 $x^{\\prime}=0, y^{\\prime}=y, z^{\\prime}=z+x-x^{\\prime}$, 则 $x^{\\prime} \\geqslant 0, y^{\\prime} \\geqslant 0, z^{\\prime} \\geqslant 0$, 且 $z^{\\prime}-z=x-x^{\\prime}, y^{\\prime}+z^{\\prime}=x+y+z-x^{\\prime}=x+y+z=1$, 于是\n$$\n\\begin{aligned}\n& Q-\\left(\\sqrt{2-x^{\\prime}}+\\sqrt{2-y^{\\prime}}+\\sqrt{2-z^{\\prime}}\\right) \\\\\n& =\\sqrt{2-x}+\\sqrt{2-y}+\\sqrt{2-z}-\\left(\\sqrt{2-x^{\\prime}}+\\sqrt{2-y^{\\prime}}+\\sqrt{2-z^{\\prime}}\\right) \\\\\n& =\\left(\\sqrt{2-x}-\\sqrt{2-x^{\\prime}}\\right)+\\left(\\sqrt{2-z}-\\sqrt{2-z^{\\prime}}\\right) \\\\\n& =\\frac{x^{\\prime}-x}{\\sqrt{2-x}+\\sqrt{2-x^{\\prime}}}+\\frac{z^{\\prime}-z}{\\sqrt{2-z}+\\sqrt{2-z^{\\prime}}} \\\\\n& =\\left(x-x^{\\prime}\\right)\\left(\\frac{-1}{\\sqrt{2-x}+\\sqrt{2-x^{\\prime}}}+\\frac{1}{\\sqrt{2-z}+\\sqrt{2-z^{\\prime}}}\\right) \\\\\n& =x \\cdot \\frac{\\left(\\sqrt{2-x}+\\sqrt{2-x^{\\prime}}\\right)-\\left(\\sqrt{2-z}+\\sqrt{2-z^{\\prime}}\\right)}{\\left(\\sqrt{2-x}+\\sqrt{2-x^{\\prime}}\\right)\\left(\\sqrt{2-z}+\\sqrt{2-z^{\\prime}}\\right)} \\\\\n& =x \\cdot \\frac{(\\sqrt{2-x}-\\sqrt{2-z})+\\left(\\sqrt{2-x^{\\prime}}-\\sqrt{2-z^{\\prime}}\\right)}{\\left(\\sqrt{2-x}+\\sqrt{2-x^{\\prime}}\\right)\\left(\\sqrt{2-z}+\\sqrt{2-z^{\\prime}}\\right)} \\\\\n& =x \\cdot \\frac{\\frac{z-x}{\\sqrt{2-x}+\\sqrt{2-z}}+\\frac{z^{\\prime}-x^{\\prime}}{\\sqrt{2-x^{\\prime}}-\\sqrt{2-z^{\\prime}}}}{\\left(\\sqrt{2-x}+\\sqrt{2-x^{\\prime}}\\right)\\left(\\sqrt{2-z}+\\sqrt{2-z^{\\prime}}\\right)} \\\\\n& =x \\cdot \\frac{\\frac{z-x}{\\sqrt{2-x}+\\sqrt{2-z}}+\\frac{z^{\\prime}}{\\sqrt{2-x^{\\prime}}-\\sqrt{2-z^{\\prime}}}}{\\left(\\sqrt{2-x}+\\sqrt{2-x^{\\prime}}\\right)\\left(\\sqrt{2-z}+\\sqrt{2-z^{\\prime}}\\right)} \\geqslant 0 . \\\\\n&\n\\end{aligned}\n$$\n所以,\n$$\nQ \\geqslant \\sqrt{2-x^{\\prime}}+\\sqrt{2-y^{\\prime}}+\\sqrt{2-z^{\\prime}}=\\sqrt{2}+\\sqrt{2-y^{\\prime}}+\\sqrt{2-z^{\\prime}} . \\label{eq1}\n$$\n因为 $\\left(\\sqrt{2-y^{\\prime}}+\\sqrt{2-z^{\\prime}}\\right)^2=\\left(2-y^{\\prime}\\right)+\\left(2-z^{\\prime}\\right)+2 \\sqrt{2-y^{\\prime}} \\sqrt{2-z^{\\prime}}$\n$$\n\\begin{aligned}\n& \\left.=4-\\left(y^{\\prime}+z^{\\prime}\\right)+2 \\sqrt{4-2\\left(y^{\\prime}\\right.}+z^{\\prime}\\right)+y^{\\prime} z^{\\prime} \\\\\n& =4-1+2 \\sqrt{4-2 \\cdot 1+y^{\\prime} z^{\\prime}} \\\\\n& =3+2 \\sqrt{2+y^{\\prime} z^{\\prime}} \\\\\n& \\geqslant 3+2 \\sqrt{2}=(1+\\sqrt{2})^2,\n\\end{aligned}\n$$\n所以\n$$\n\\sqrt{2-y^{\\prime}}+\\sqrt{2-z^{\\prime}} \\geqslant 1+\\sqrt{2} \\text {. } \\label{eq2}\n$$\n由 式\\ref{eq1} \\ref{eq2}, 得 $Q \\geqslant 1+2 \\sqrt{2}$, 当 $x=y=0, z=1$ 时, $Q=1+2 \\sqrt{2}$, 因此 $Q$ 的最小值是 $1+2 \\sqrt{2}$.", + "remark": "", + "figures": [] +} \ No newline at end of file diff --git a/processed_dataset/calculation/0322.json b/processed_dataset/calculation/0322.json new file mode 100644 index 0000000000000000000000000000000000000000..68f720acbb5a1bad842420f40d1d4da8b6cde941 --- /dev/null +++ b/processed_dataset/calculation/0322.json @@ -0,0 +1,8 @@ +{ + "source_file": "./raw_volume-zh/volume13/chapter6.tex", + "problem_type": "calculation", + "problem": "例1. 设 $M=\\{1,2, \\cdots, 2005\\}, A$ 是 $M$ 的子集, 若对任何 $a_i, a_j \\in A$, $a_i \\neq a_j$, 都能以 $a_i 、 a_j$ 为边长唯一地确定一个等腰三角形, 求 $|A|$ 的最大值.", + "solution": "分析:考虑在什么条件下, 两个数 $a 、 b(ay$.\n解将 $M$ 划分为 11 个子集: $A_1=\\{1\\}, A_2=\\{2,3\\}, A_3=\\left\\{2^2, 2^2+\\right. \\left.1, \\cdots, 2^3-1\\right\\}, \\cdots, A_{11}=\\left\\{2^{10}, 2^{10}+1, \\cdots, 2005\\right\\}$, 因为对每个集合 $A_i$ 中的任何元素 $x, y(xy$, 从而 $\\left|A \\cap A_i\\right| \\leqslant 1(i=1,2,3, \\cdots, 11)$, 所以 $|A| \\leqslant 11$. 又 $A=\\left\\{1,2,2^2, \\cdots, 2^{10}\\right\\}$ 合乎条件, 故 $|A|$ 的最大值为 11 .", + "remark": "", + "figures": [] +} \ No newline at end of file diff --git a/processed_dataset/calculation/0326.json b/processed_dataset/calculation/0326.json new file mode 100644 index 0000000000000000000000000000000000000000..fb0bb90bb5d7fad004a171064c45e42834be37a3 --- /dev/null +++ b/processed_dataset/calculation/0326.json @@ -0,0 +1,8 @@ +{ + "source_file": "./raw_volume-zh/volume13/chapter7.tex", + "problem_type": "calculation", + "problem": "例2. 设 $A$ 是正整数集合 $\\mathbf{N}^*$ 的子集,对任何 $x, y \\in A, x \\neq y$, 有 $\\mid x- y \\mid \\geqslant \\frac{x y}{25}$. 求 $|A|$ 的最大值.", + "solution": "分析:题在前一节中用\"间距估计\"给出过解答, 现考虑用划块估计求解.\n其基本想法是: 将 $\\mathbf{N}^*$ 划分为若干块, 使 $A$ 在每一块中至多含有 1 个元素.\n注意到 $A$ 满足的条件是: 对任何 $a_i25$, 所以 $y-x),用 4 条直线将棋盘划分为 $A 、B 、 C 、 D 、 E 、 F 、 G 、 H 、 O$ 共 9 个区域.\n由条件, $A \\cup G$ 中至少 2 个空, $E \\cup O$ 中至少 3 个空, $C \\cup H$ 中至少 2 个空, $D \\cup F$ 中至少 3 个空, 从而 $k \\geqslant 2+3+2+3=10$.\n如果 $k=10$, 则区域 $D$ 中没有空, 由对称性, 区域 $A 、 B 、 C 、 D$ 中都没有空.\n进而, 因为 $A \\cup G$ 中至少 2 个空,而 $A$ 中没有空, 从而 $G$ 中至少 2 个空, 同理, $H$ 中至少 2 个空, 因为 $A \\cup E$ 中至少 3 个空, 而 $A$ 中没有空, 从而 $E$\n中至少 3 个空, 同理, $F$ 中至少 3 个空, 于是, $E 、 F 、 G 、 H$ 中共至少 10 个空, 又棋盘中恰有 10 个空, 从而 $E 、 F$ 中各有 3 个空, $G 、 H$ 中各有 2 个空, 区域 $O$ 中没有空.\n因为 $A \\cup E$ 中每列至少 1 个空, 而 $A$ 中没有空, 从而 $E$ 中每列至少 1 个空.\n又 $E$ 中只有 3 个空, 所以 $E$ 中每列恰有 1 个空, 于是格 $1 、 2$ 中至少有一个\n不是空, 于是, 图中 2 条直线中有一条是 5 子相连,矛盾, 所以 $k \\geqslant 11$.\n当 $k=11$ 时, 如图()\n\\begin{tabular}{|l|l|l|l|l|l|l|l|}\n\\hline 1 & & & & & 6 & & \\\\\n\\hline & & & 5 & & & & \\\\\n\\hline & 2 & & & & & 9 & \\\\\n\\hline & & & & 8 & & & \\\\\n\\hline & & 3 & & & & & 11 \\\\\n\\hline 7 & & & & & 10 & & \\\\\n\\hline & & & 4 & & & & \\\\\n\\hline\n\\end{tabular}\n(比\"评分标准\"中的构造更自然), 我们采用 \"马步\" 布子, 则棋盘中没有同一直线上的 5 子相连.\n综上所述, $k_{min}=11$.", + "remark": "", + "figures": [ + "./images/volume13/figures/fig-c7i6.png", + "./images/volume13/figures/fig-c7i7.png" + ] +} \ No newline at end of file diff --git a/processed_dataset/calculation/0328.json b/processed_dataset/calculation/0328.json new file mode 100644 index 0000000000000000000000000000000000000000..46eb4213e7a00c30092ae5b0327cb191a9a868a3 --- /dev/null +++ b/processed_dataset/calculation/0328.json @@ -0,0 +1,8 @@ +{ + "source_file": "./raw_volume-zh/volume13/chapter7.tex", + "problem_type": "calculation", + "problem": "例10. 设集合 $S=\\{1,2, \\cdots, 50\\}, X$ 是 $S$ 的任意子集, $|X|=n$. 求最小正整数 $n$, 使得集合 $X$ 中必有三个数为直角三角形的三条边长.", + "solution": "解:直角三角形三边长分别为 $x, y, z$, 有 $x^2+y^2=z^2$, 其正整数解可表示为\n$$\nx=k\\left(a^2-b^2\\right), y=2 k a b, z=k\\left(a^2+b^2\\right), \\label{eq1}\n$$\n其中 $k, a, b \\in \\mathbf{N}^*$ 且 $(a, b)=1, a>b$.\n首先, $x, y, z$ 中必有一个为 5 的倍数.\n否则, 若 $a, b, c$ 均不是 5 的倍数, 则 $a, b, c$ 都是形如 $5 m \\pm 1,5 m \\pm 2$ 的数 $(m \\in \\mathbf{N})$, 则 $a^2 \\equiv \\pm 1(\\bmod 5)$, $b^2 \\equiv \\pm(\\bmod 5), c^2 \\equiv \\pm 1(\\bmod 5)$, 而 $c^2=a^2+b^2 \\equiv 0$ 或士2,矛盾!\n令集 $A=\\{S$ 中所有与 5 互质的数 $\\}$, 则 $\\operatorname{Card} A=40$. 若以 10, 15, 25, 40,45 分别作直角三角形的某边长, 则由 式\\ref{eq1} 知可在 $A$ 中找到相应的边构成如下直角三角形: $(10,8,6),(26,24,10),(15,12,9),(17,15,8)$, (39, $36,15),(25,24,7),(40,32,24),(41,40,9),(42,27,36)$, 此外, $A$ 中再没有能与 $10,15,25,40,45$ 构成直角三角形三边的数.\n令 $M=A \\cup\\{10,15,25,40,45\\} \\backslash\\{8,9,24,36\\}$, 则 $\\operatorname{Card} M=41$.\n由以上知, $A$ 中三数不能组成直角三角形, 由于 $M$ 中不含 $8,9,24,36$, 所以 $10,15,25,40,45$ 在 $M$ 中找不到可搭配成直角三角形三边的数, 即 $M$ 中任三数均不构成直角三角形三边,故 $n \\geqslant 42$.\n另外, 由 式\\ref{eq1} 的整数解可作集合: $B=\\{3,4,5,17,15,8,29,21,20$, $25,24,7,34,16,30,37,35,12,50,48,14,41,40,9,45,36,27\\}$, 其中横线上三数可作直角三角形三边, Card $B=27$.\n$S \\backslash B$ 中元素的个数为 $50-27=23$, 在 $S$ 中任取 42 个数, 因 $42-23=19$, 于是, 取的 42 个数中必含有 $B$ 中的 19 个数, 因此 $B$ 中至少有一条横线上的三个数在所选的 42 个数中, 即任取 42 个数, 其中至少有三数可作直角三角形三边, 因此, $n$ 的最小值为 42 .", + "remark": "", + "figures": [] +} \ No newline at end of file diff --git a/processed_dataset/calculation/0329.json b/processed_dataset/calculation/0329.json new file mode 100644 index 0000000000000000000000000000000000000000..54203fa2fd9eb93eb9e7b8d94454b7585f38c3ba --- /dev/null +++ b/processed_dataset/calculation/0329.json @@ -0,0 +1,8 @@ +{ + "source_file": "./raw_volume-zh/volume13/chapter8.tex", + "problem_type": "calculation", + "problem": "例2. 设 $X=\\{1,2, \\cdots, 100\\}, A$ 是 $X$ 的子集, 若对 $A$ 中任何两个元素 $x$ 、 $y(x76$, 则 $A$ 必含有某个集合 $A_k$ 中的 2 个数, 其中较大的数是其较小的数的 3 倍,矛盾.\n综上所述, $|A|$ 的最大值是 76 .", + "remark": "", + "figures": [] +} \ No newline at end of file diff --git a/processed_dataset/calculation/0330.json b/processed_dataset/calculation/0330.json new file mode 100644 index 0000000000000000000000000000000000000000..3a085cb8283dd8441a7a92f8683daa3462d4fa95 --- /dev/null +++ b/processed_dataset/calculation/0330.json @@ -0,0 +1,8 @@ +{ + "source_file": "./raw_volume-zh/volume13/chapter8.tex", + "problem_type": "calculation", + "problem": "例5. 有 18 支球队进行单循环赛, 即每轮将 18 支球队分成 9 组, 每组的 2 个队比赛一场.\n下一轮重新分组比赛, 共赛 17 轮, 使得每队都与另 17 支队各赛一场.\n按任意可行的程序比赛了 $n$ 轮以后, 总存在 4 支球队, 他们之间总共只赛了 1 场.\n求 $n$ 的最大可能值.", + "solution": "解:察如下的比赛程序:\n1. $(1,2)(3,4)(5,6)(7,8)(9,18)(10,11)(12,13)(14,15) (16,17)$;\n2. $(1,3)(2,4)(5,7)(6,9)(8,17)(10,12)(11,13)(14,16) (15,18)$;\n3. $(1,4)(2,5)(3,6)(8,9)(7,16)(10,13)(11,14)(12,15) (17,18)$;\n4. $(1,5)(2,7)(3,8)(4,9)(6,15)(10,14)(11,16)(12,17) (13,18)$;\n5. $(1,6)(2,8)(3,9)(4,7)(5,14)(10,15)(11,17)(12,18) (13,16)$;\n6. $(1,7)(2,9)(3,5)(6,8)(4,13)(10,16)(11,18)(12,14) (15,17)$\n7. $(1,8)(2,6)(4,5)(7,9)(3,12)(10,17)(11,15)(13,14) (16,18)$;\n8. $(1,9)(3,7)(4,6)(5,8)(2,11)(10,18)(12,16)(13,15) (14,17)$\n9. $(1,10)(2,3)(4,8)(5,9)(6,7)(11,12)(13,17)(14,18) (15,16)$\n10. $(1,11)(2,12)(3,13)(4,14)(5,15)(6,16)(7,17)(8,18) (9,10)$;\n11. $(1,12)(2,13)(3,14)(4,15)(5,16)(6,17)(7,18)(8,10) (9,11)$;\n12. $(1,13)(2,14)(3,15)(4,16)(5,17)(6,18)(7,10)(8,11) (9,12)$;\n$\\cdots$\n17. $(1,18)(2,10)(3,11)(4,12)(5,13)(6,14)(7,15)(8,16) (9,17)$.\n将前 9 队称为 $A$ 组, 后 9 队称为 $B$ 组, 易见 9 轮之后, 凡同组两队均已赛过.\n所以, 任何 4 队之间至少已赛过 2 场, 不满足题目要求.\n如果把上述程序颠倒过来, 然后按照新序比赛, 则 8 轮过后, 同组任何 2 队均未赛过, 每个队都是与另一组中 9 支队中的 8 个队各赛一场, 这时同组 4 支队之间一场未赛, 而不全同组的 4 支队之间至少已赛 2 场, 不满足题目要求.\n于是 $n \\leqslant 7$. 当 $n=7$ 时, 反设任何 4 队都不满足题目要求, 选取已赛过的 2 队 $A_1 、 A_2$, 则每队都与另外 6 队比赛过, 2 个队至多与另外 12 支队赛过, 于是至少存在 4 个队 $B_1 、 B_2 、 B_3$ 、 $B_4$, 它们与 $A_1 、 A_2$ 两队均未赛过.\n考察 4 个队 $A_1 、 A_2 、 B_i 、 B_j(1 \\leqslant i2 k \\geqslant z$, 从而存在一个以 $x$ 、 $y 、 z$ 为三边的三角形, 且 $|A|=k+1=\\frac{n}{2}+1=\\frac{n+2}{2}=\\left[\\frac{n+2}{2}\\right]$.\n若 $n=2 k+1$, 则令 $A=\\{k+1, k+2, \\cdots, 2 k+1\\}$, 此时, 对任何 $x2 k+1 \\geqslant z$, 从而存在一个以 $x 、 y 、 z$ 为三边的三角形, 且 $|A|=k+1=\\frac{n-1}{2}+1=\\frac{n+1}{2}=\\left[\\frac{n+2}{2}\\right]$.\n(注:上述构造可合并为 $A=\\left\\{\\left[\\frac{n+1}{2}\\right],\\left[\\frac{n+1}{2}\\right]+1,\\left[\\frac{n+1}{2}\\right]+2, \\cdots, n\\right\\}$ ) 故 $|A|$ 的最大值为 $\\left[\\frac{n+2}{2}\\right]$.", + "remark": "", + "figures": [] +} \ No newline at end of file diff --git a/processed_dataset/calculation/0333.json b/processed_dataset/calculation/0333.json new file mode 100644 index 0000000000000000000000000000000000000000..59ca28a143ff4db8f9e75b36f58ae3606c0bfae4 --- /dev/null +++ b/processed_dataset/calculation/0333.json @@ -0,0 +1,8 @@ +{ + "source_file": "./raw_volume-zh/volume13/exercise1.tex", + "problem_type": "calculation", + "problem": "问题1. 设 $a 、 b 、 c 、 a+b-c 、 b+c-a 、 c+a-b 、 a+b+c$ 是 7 个两两不同的质数,且 $a 、 b 、 c$ 中有两个数的和是 800 . 设 $d$ 是这 7 个质数中最大数与最小数的差,求 $d$ 的最大可能值.", + "solution": "不妨设 $a0$, 所以 $c80$, 使对应的 $x$ 是唯一的且 $S$ 是最小的.\n因为由 $S$ 可猜想 $(x, y)$, 也可猜想 $(x-1, y+5)$, 还可猜想 $(x+1, y-5)$, 要使猜想的结果只能是 $(x$, $y)$, 必须 $(x-1, y+5) 、(x+1, y-5)$ 都不存在.\n因为 $(x-1, y+5)$ 都不存在, 所以 $x-1<0$ 或 $(x-1)+(y+5)>30$. 但 $x \\geqslant 1$, 所以只能是 $(x-1)+(y+5)>30$. 于是, $x+y \\geqslant 27$. 因为 $(x+1, y-5)$ 不存在, 所以 $y-5<0$ 或 $(x+1)+(y-5)>30$. 但 $x+y+z=30$ 得 $x+y \\leqslant 30$, 所以只能是 $y- 5<0$. 于是, $y \\leqslant 4$. 所以 $S=5 x+y=5(x+y)-4 y \\geqslant 5 \\times 27-4 y \\geqslant 5 \\times 27-4 \\times 4=119$. 等式在 $x=23, y=4$ 时成立.\n故 $S_{\\min }=119$, 即此次考试甲得了 119 分.", + "remark": "", + "figures": [] +} \ No newline at end of file diff --git a/processed_dataset/calculation/0339.json b/processed_dataset/calculation/0339.json new file mode 100644 index 0000000000000000000000000000000000000000..e5d5068966b8b5bfeaafa0948fee2822688e4a56 --- /dev/null +++ b/processed_dataset/calculation/0339.json @@ -0,0 +1,8 @@ +{ + "source_file": "./raw_volume-zh/volume13/exercise10.tex", + "problem_type": "calculation", + "problem": "问题2. 某班有 47 个学生, 所用教室有 6 排, 每排有 8 个座位, 用 $(i, j)$ 表示位于第 $i$ 排第 $j$ 列的座位.\n新学期准备调整座位, 设一个学生原来的座位为 $(i$, $j)$, 如果调整后的座位为 $(m, n)$, 则称该生作了移动 $[a, b]=[i-m, j- n]$, 并称 $a+b$ 为该生的位置数, 所有学生的位置数之和记为 $S$. 求 $S$ 的最大可能值与最小可能值之差.", + "solution": "称 $i+j$ 为格 $(i, j)$ 的特征值, 则学生的位置数即是他前后位置的特征值之差.\n记所有格的特征值之和为 $M$. 引人参数: 设最初空格的位置为 $(x$, $y)$, 调整后为 $(p, q)$, 那么最初所有学生的特征值之和为 $M-(x+y)$, 调整后所有学生的特征值之和为 $M-(p+q)$, 于是 $S=M-(x+y)-[M-(p+ q)]=p+q-(x+y) . S_{\\max }=6+8-(x+y), S_{\\min }=1+1-(x+y)$, $S_{\\max }-S_{\\min }=[6+8-(x+y)]-[1+1-(x+y)]=14-2=12$.", + "remark": "", + "figures": [] +} \ No newline at end of file diff --git a/processed_dataset/calculation/0340.json b/processed_dataset/calculation/0340.json new file mode 100644 index 0000000000000000000000000000000000000000..714987891a1bb9d16df5a45d0bdf975cc363955d --- /dev/null +++ b/processed_dataset/calculation/0340.json @@ -0,0 +1,8 @@ +{ + "source_file": "./raw_volume-zh/volume13/exercise10.tex", + "problem_type": "calculation", + "problem": "问题3. 已知共有 12 个剧团参加为期 7 天的演出, 要求每个剧团都能看到其他所有剧团的演出, 而只能是当天没演出的在台下观看, 问最少共要演出多少场?", + "solution": "设 $A=\\{1,2, \\cdots, 7\\}$ 为演出日期的集合, $A_i$ 为第 $i$ 个剧团演出的日期的集合 $(i=1,2, \\cdots, 12)$. 易知, $A_1, A_2, \\cdots, A_{12}$ 互不包含.\n否则对 $i \\neq j$, 设 $A_i$ 包含于 $A_j$ 内, 则第 $j$ 团看不到第 $i$ 团的演出.\n考察演出场数 $t=\\sum_{i=1}^{12}\\left|A_i\\right|$, 若有某个 $\\left|A_i\\right|=0$, 则其他团都看不到第 $i$ 团的演出, 矛盾.\n因此, 对任何 $i$, 有 $\\left|A_i\\right| \\geqslant 1$. 引人参数: 设其中恰有 $k$ 个集合是单元集,而其他集合中的元素个数至少是 2 , 于是 $t=\\sum_{i=1}^{12}\\left|A_i\\right| \\geqslant \\underbrace{1+1+\\cdots+1}_{k \\uparrow 1}+\\underbrace{2+2+\\cdots+2}_{12-k \\uparrow 2}= 24-k$. 不妨设 $\\left\\{a_1\\right\\}=A_1,\\left\\{a_2\\right\\}=A_2, \\cdots,\\left\\{a_k\\right\\}=A_k$. 因为 $A_1, A_2, \\cdots, A_{12}$ 互不包含, 从而 $a_1, a_2, \\cdots, a_k$ 都不在 $A_{k+1}, A_{k+2}, \\cdots, A_{12}$ 中, 所以 $A_{k+1}$, $A_{k+2}, \\cdots, A_7$ 都是 $\\left\\{a_{k+1}, a_{k+2}, \\cdots, a_7\\right\\}$ 的子集, 即 $\\left\\{A_{k+1}, A_{k+2}, \\cdots, A_{12}\\right\\}$ 是 $A \\backslash\\left\\{a_1, a_2, \\cdots, a_k\\right\\}$ 的互不包含子集族, 所以 $\\mathrm{C}_{7-k}^{\\left[\\frac{7-k}{2}\\right]} \\geqslant 12-k$. 此式在 $7-k= 1,2,3,4$ 时不成立, 所以 $7-k \\geqslant 5$, 即 $k \\leqslant 2$. 所以 $t \\geqslant 24-k \\geqslant 22$. 最后, 当 $t=$ 22 时, 令 $A_1=\\{1\\} 、 A_2=\\{2\\}$, 而 $A_3, A_4, \\cdots, A_{12}$ 取 $\\{3,4,5,6,7\\}$ 中的 10 个互异二元集即可.\n比如 $A_3=\\{3,4\\} 、 A_4=\\{3,5\\} 、 A_5=\\{3,6\\} 、 A_6=\\{3$ , 7\\}、 $A_7=\\{4,5\\} 、 A_8=\\{4,6\\} 、 A_9=\\{4,7\\} 、 A_{10}=\\{5,6\\} 、 A_{11}=\\{5,7\\} 、 A_{12}=\\{6,7\\}$. 所以, 演出场数的最小值为 22 .", + "remark": "", + "figures": [] +} \ No newline at end of file diff --git a/processed_dataset/calculation/0341.json b/processed_dataset/calculation/0341.json new file mode 100644 index 0000000000000000000000000000000000000000..aaf159dcc0ef1dedd71ee3d5a6a1b567e70bd512 --- /dev/null +++ b/processed_dataset/calculation/0341.json @@ -0,0 +1,8 @@ +{ + "source_file": "./raw_volume-zh/volume13/exercise11.tex", + "problem_type": "calculation", + "problem": "问题3. 有 $n$ 名选手参加比赛, 历时 $k$ 天, 其中任何一天 $n$ 名选手的得分都恰好是 . $1,2,3, \\cdots, n$ 的一个排列.\n如果在第 $k$ 天末, 每个选手的总分都是 26 . 求 $(n, k)$ 的所有可能取值.", + "solution": "由条件\"每个选手的总分都是 26 \", 想到计算第 $k$ 天后 $n$ 名选手得分之和 $S$. 一方面, 每天的得分为 $1+2+3+\\cdots+n$, 所以, $S=k(1+2+\\cdots+n)$. 另一方面, 每个选手得 26 分, 从而 $S=26 n$. 所以, $k(n+1)=52$. 所以 $(n, k)= (51,1) 、(25,2) 、(12,4) 、(3,13)$. 其中 $(n, k)=(51,1)$ 时, 各选手的总分互异, 矛盾, 故舍去.\n由下面的构造可知, 其他 3 种情况都是可能的.\n当 $(n$, $k)=(25,2)$ 时, 第 $i$ 号选手的名次集合为 $A_i=\\{i, 26-i\\}(i=1,2, \\cdots$, $25)$. 当 $(n, k)=(12,4)$ 时, 第 $i$ 号选手的名次集合为 $A_i=\\{i, 13-i, i, 13-i\\}(i=1,2, \\cdots, 12)$. 当 $(n, k)=(3,13)$ 时, 各选手的名次集合为 $A_1= \\{2,3,1\\} \\cup\\{1,3,1,3, \\cdots, 1,3\\} 、 A_2=\\{3,1,2\\} \\cup\\{2,2,2,2, \\cdots, 2$, 2\\}、 $A_3=\\{1,2,3\\} \\cup\\{3,1,3,1, \\cdots, 3,1\\}$.", + "remark": "", + "figures": [] +} \ No newline at end of file diff --git a/processed_dataset/calculation/0342.json b/processed_dataset/calculation/0342.json new file mode 100644 index 0000000000000000000000000000000000000000..f8c644c92f4165c55594e798401adc6a9fa78f62 --- /dev/null +++ b/processed_dataset/calculation/0342.json @@ -0,0 +1,10 @@ +{ + "source_file": "./raw_volume-zh/volume13/exercise11.tex", + "problem_type": "calculation", + "problem": "问题4. 某班有 30 个学生, 年龄互不相同, 每个学生在同班中有相同个数的朋友.\n对某个学生 $A$, 若 $A$ 的年龄比 $A$ 的一半以上 (不包括一半) 朋友大, 则称 $A$ 为大龄的.\n问大龄的学生最多有多少个?", + "solution": "设有 $t$ 个大龄学生, 每个学生都有 $k$ 个朋友.\n为叙述问题方便, 用 30 个点表示 30 个学生.\n对任何两个点 $A 、 B$, 如果 $A 、 B$ 是朋友, 且 $A$ 的年龄大于 $B$ 的年龄, 则连一条指向 $B$ 的有向边, 得到一个竟赛图.\n称大龄学生对应的点为 \"大点\", 设所有的大点为 $A_1, A_2, \\cdots, A_t$. 依题意, 有 $d^{+}\\left(A_i\\right)>d\\left(A_i\\right)$, $d\\left(A_i\\right)=d^{+}\\left(A_i\\right)+d^{-}\\left(A_i\\right)=k$, 于是 $d^{+}\\left(A_i\\right) \\geqslant \\frac{k+1}{2}$. \n不妨设 $A_1$ 的年龄 $\\leqslant A_2$ 的年龄 $\\leqslant \\cdots \\leqslant A_t$ 的年龄, 则 $d^{+}\\left(A_1\\right) \\leqslant 30-t\\left(A_1\\right.$ 最多向 $A_1, A_2, \\cdots, A_t$ 外的 $30-t$ 个点引出边 $), d^{+}\\left(A_t\\right)=k$. 计算所有 \"大点\" 的出度的和 $S$. \n一方面, $S=d^{+}\\left(A_1\\right)+d^{+}\\left(A_2\\right)+\\cdots+d^{+}\\left(A_{t-1}\\right)+k \\geqslant \\frac{k+1}{2}(t-1)+k$. 另一方面, $S \\leqslant\\|G\\|=15 k$, 所以 $15 k=\\|G\\| \\geqslant S \\geqslant \\frac{k+1}{2}(t-1)+k$, 所以 $t \\leqslant \\frac{28 k}{k+1}+1 . \\label{eq1}$. \n此外, $\\frac{k+1}{2} \\leqslant d^{+}\\left(A_1\\right) \\leqslant 30-t$, 所以 $k \\leqslant 59-2 t . \\label{eq2}$. 由式\\ref{eq1},\\ref{eq2}消去 $k$ (利用(1)右边关于 $k$ 的函数的单调性), 有 $t \\leqslant \\frac{28(59-2 t)}{60-2 t}+1$, 即 $t^2- 59 t+856 \\geqslant 0 . \\label{eq3}$ . \n但 $t \\leqslant 30$, 使 式\\ref{eq3}成立的最大整数是 $t=25$. 即大龄学生不多于 25 . 最后, $t=25$ 是可能的.\n实际上, 当 $t=25$ 时, 以上不等式成立等号, 代入 式\\ref{eq2} , 解得 $k=9$.\n将 $1,2, \\cdots, 30$ 排成 6 行 (如图()),\n规定 $i 、 j$ 是一对朋友, 当且仅当 $i 、 j$ 满足下列 3 个条件之一:\n(1) $i 、 j$ 在相邻的行中但不同列.\n(2) $i 、 j$ 同列,但其中一个在最后一行.\n(3) $i 、 j$ 都在第一行中.\n此时, 每人有 9 个朋友.\n比如 1 的 9 个朋友为 $2 、 3 、 4 、 5 、 7 、 8 、 9 、 10$ 、 26 . 故 $t$ 的最大值为 25 .", + "remark": "", + "figures": [ + "./images/volume13/figures/fig-c11a4.png" + ] +} \ No newline at end of file diff --git a/processed_dataset/calculation/0343.json b/processed_dataset/calculation/0343.json new file mode 100644 index 0000000000000000000000000000000000000000..21484e5d20270b85de1c203da9b223fc6427cc13 --- /dev/null +++ b/processed_dataset/calculation/0343.json @@ -0,0 +1,8 @@ +{ + "source_file": "./raw_volume-zh/volume13/exercise13.tex", + "problem_type": "calculation", + "problem": "问题1. 某次考试有 5 道选择题, 每题都有 4 个不同的答案供选择, 每人每题恰选一个答案.\n在 2000 份答卷中发现存在一个数 $n$, 使得任何 $n$ 份答卷中都存在 4 份, 其中每两份答卷选择的答案都至多有 3 题相同.\n求 $n$ 的最小可能值.", + "solution": "$n$ 的最小可能值是 25 . 将每道题的 4 种答案分别记为 $1 、 2 、 3 、 4$, 每份试卷上的答案记为 $(g, h, i, j, k)$, 其中 $g, h, i, j, k \\in\\{1,2,3,4\\}$. 对于所有 2000 份答案 $(g, h, i, j, k)$, 将后 4 个分量完全相同的看作一类, 则共有 $4^4=256$ 类.\n因为 $2000=256 \\times 7+208$, 于是必有 8 份试卷属于同一个类 $A$.取出这 8 份试卷,剩下 1992 份试卷中仍有 8 份试卷属于同一个类 $B$. 再取出这 8 份试卷, 剩下 1984 份试卷中仍有 8 份试卷属于同一个类 $C$. 取出的 24 份试卷共属于 3 个类 $A 、 B 、 C$, 于是, 当 $n \\leqslant 24$ 时, 从这 24 份试卷中任取 $n$ 份,\n则 $n$ 份中的任何 4 份试卷必有 2 份属于同一个类, 不满足题目要求, 于是 $n \\geqslant 25$.\n下面构造这样的 2000 份答卷, 它们共有 250 种不同答案, 同一种答案的试卷各有 8 份.\n这 250 种答案是满足 $g+h+i+j+k \\equiv 0(\\bmod 4)$ 的所有 $4^4=256$ 种答案 $(g, h, i, j, k)$ 中的任意 250 种.\n显然, 对于任何 2 份不同的答案, 它们至多有 3 个分量相同, 否则, 若有 4 个分量相同, 则由同余式可知, 第 5 个分量也相同, 矛盾.\n在这样的 2000 份答卷中任取 25 份, 由于相同的答卷至多有 8 份, 从而至少有 4 份试卷是两两不同的, 它们至多有 3 个分量相同,故 $n=25$ 合乎题目要求.", + "remark": "", + "figures": [] +} \ No newline at end of file diff --git a/processed_dataset/calculation/0344.json b/processed_dataset/calculation/0344.json new file mode 100644 index 0000000000000000000000000000000000000000..bd1710905fa2a6c7018b7a4ec954d5a5ec7c3792 --- /dev/null +++ b/processed_dataset/calculation/0344.json @@ -0,0 +1,8 @@ +{ + "source_file": "./raw_volume-zh/volume13/exercise2.tex", + "problem_type": "calculation", + "problem": "问题1. 设 $x 、 y 、 z$ 为非负实数, $x+y+z=a(a \\geqslant 1)$, 求 $F=2 x^2+y+3 z^2$ 的最大值.", + "solution": "由 $x+y+z=a$, 得 $\\frac{x}{a}+\\frac{y}{a}+\\frac{z}{a}=1$, 令 $x=a u, y=a v, z=a w$, 则 $0 \\leqslant u, v, w \\leqslant 1, u+v+w=1, F=2 x^2+y+3 z^2=2 a^2 u^2+a v+3 a^2 w^2$. 因为 $a \\geqslant 1$ 时, $a^2 \\geqslant a \\geqslant 1$, 于是, $F=2 a^2 u^2+a v+3 a^2 w^2 \\leqslant 2 a^2 u^2+a^2 v+ 3 a^2 w^2 \\leqslant 3 a^2 u^2+3 a^2 v+3 a^2 w^2 \\leqslant 3 a^2 u+3 a^2 v+3 a^2 w=3 a^2$. 等号在 $u=v=0$ 、 $w=1$, 即 $x=y=0 、 z=a$ 时成立.\n所以 $2 x^2+y+3 z^2$ 的最大值为 $3 a^2$.", + "remark": "", + "figures": [] +} \ No newline at end of file diff --git a/processed_dataset/calculation/0345.json b/processed_dataset/calculation/0345.json new file mode 100644 index 0000000000000000000000000000000000000000..460937d4c4a43210e845a72b2a43cc13bfccb546 --- /dev/null +++ b/processed_dataset/calculation/0345.json @@ -0,0 +1,8 @@ +{ + "source_file": "./raw_volume-zh/volume13/exercise2.tex", + "problem_type": "calculation", + "problem": "问题2. 求一个十进制 3 位数, 使它与其各位数字之和的比最小.", + "solution": "设所求的三位数为 $100 x+20 y+z$, 考察 $F=\\frac{100 x+10 y+z}{x+y+z}=1+ \\frac{99 x+9 y}{x+y+a}$, 注意到上式右边, $z$ 仅在分母中出现, 从而 $F$ 是关于 $z$ 的单调函数.\n固定 $x 、 y$, 则由 $z \\leqslant 9$, 得 $F \\geqslant 1+\\frac{99 x+9 y}{x+y+9}=10+\\frac{90 x-81}{x+y+9}$. 注意到上式右边, $y$ 仅在分母中出现, 从而右边是关于 $y$ 的单调函数.\n再固定 $x$, 则由 $y \\leqslant 9$, 得 $F \\geqslant 10+\\frac{90 x-81}{x+9+9}=100-\\frac{1701}{18+x} \\geqslant 100-\\frac{1701}{19}=\\frac{199}{99}$. 其中等式在 $x=1 、 y=z=9$ 时成立.\n故所求的三位数为 199 .", + "remark": "", + "figures": [] +} \ No newline at end of file diff --git a/processed_dataset/calculation/0346.json b/processed_dataset/calculation/0346.json new file mode 100644 index 0000000000000000000000000000000000000000..9a7bbf4156d4c1caaf65a5fb8685ee153974ef70 --- /dev/null +++ b/processed_dataset/calculation/0346.json @@ -0,0 +1,8 @@ +{ + "source_file": "./raw_volume-zh/volume13/exercise2.tex", + "problem_type": "calculation", + "problem": "问题4. 设 $n$ 是给定的整数 $(n>1)$, 正整数 $a 、 b 、 c 、 d$ 满足 $\\frac{b}{a}+\\frac{d}{c}<1, b+ d \\leqslant n$, 求 $\\frac{b}{a}+\\frac{d}{c}$ 的最大值.", + "solution": "记 $\\frac{b}{a}+\\frac{d}{c}$ 的最大值为 $f(n)$, 不妨设 $a \\leqslant c$. 如果 $a \\geqslant n+1$, 则 $\\frac{b}{a}+\\frac{d}{c} \\leqslant \\frac{b}{a}+\\frac{d}{a}=\\frac{b+d}{a} \\leqslant \\frac{n}{n+1}$. 如果 $a \\leqslant n$, 则固定 $a$, 记 $x=a(n-a+1)+1$. 若 $c \\leqslant x$, 则由 $\\frac{b}{a}+\\frac{d}{c}<1$, 得 $b c+a dx$, 则由 $\\frac{b}{a}+\\frac{d}{c}<1$, 得 $\\frac{b}{a}<1$, 于是 $a \\geqslant b+1$. 所以 $\\frac{b}{a}+\\frac{d}{c}-\\left(\\frac{a-1}{a}+\\frac{b+d-a+1}{c}\\right)=(b+1-$ a) $\\left(\\frac{1}{a}-\\frac{1}{c}\\right) \\leqslant 0$, 所以 $\\frac{b}{a}+\\frac{d}{c} \\leqslant \\frac{a-1}{a}+\\frac{b+d-a+1}{c} \\leqslant \\frac{a-1}{a}+\\frac{n-a+1}{c} \\leqslant \\frac{a-1}{a}+\\frac{n-a+1}{x}=1-\\frac{1}{a x}$. 所以, $a \\leqslant n$ 时, 恒有 $\\frac{b}{a}+\\frac{d}{c} \\leqslant 1-\\frac{1}{a x}$, 且等式能够成立, 于是问题转化为求函数 $g(a)=1-\\frac{1}{a x}=1-\\frac{1}{a[a(n-a+1)+1]} \\left(2 \\leqslant a \\leqslant n, a \\in \\mathbf{N}^*\\right)$ 的最大值, 也就是求 $h(a)=a[a(n-a+1)+1](2 \\leqslant \\left.a \\leqslant n, a \\in \\mathbf{N}^*\\right)$ 的最大值.\n因为 $h^{\\prime}(a)=-3 a^2+(2 n+2) a+1$, 所以 $h^{\\prime}(a)=0$ 的正根为 $a_0=\\frac{n+1+\\sqrt{(n+1)^2+3}}{3}$. 而 $\\frac{2 n+2}{3}a_i$, 所以 $a_{i+1} \\geqslant a_i+1$. 但 $a_i\\frac{\\pi}{4}>x_2$, 则固定 $x_3, x_4$, 对 $x_1 、 x_2$ 作磨光变换: 即令 $x_1^{\\prime}=\\frac{\\pi}{4}$, $x_2^{\\prime}=x_1+x_2-\\frac{\\pi}{4}, x_3^{\\prime}=x_3, x_4^{\\prime}=x_4$, 则 $x_1^{\\prime}+x_2^{\\prime}=x_1+x_2,\\left|x_1^{\\prime}-x_2^{\\prime}\\right|<\\left|x_1-x_2\\right|$, 于是, 由上述磨光工具, 有\n$\\sin x_1 \\sin x_2<\\sin x_1^{\\prime} \\sin x^{\\prime}{ }_2$, 所以 $\\sin ^2 x_1 \\sin ^2 x_2<\\sin ^2 x_1^{\\prime} \\sin ^2 x^{\\prime}{ }_2$.\n记 $f(x, y)=\\left(2 \\sin ^2 x+\\frac{1}{\\sin ^2 x}\\right)\\left(2 \\sin ^2 y+\\frac{1}{\\sin ^2 y}\\right)$, 则\n$$\n\\begin{aligned}\nf\\left(x_1, x_2\\right) & =\\left(2 \\sin ^2 x_1+\\frac{1}{\\sin ^2 x_1}\\right)\\left(2 \\sin ^2 x_2+\\frac{1}{\\sin ^2 x_2}\\right) \\\\\n& =2\\left(2 \\sin ^2 x_1 \\sin ^2 x_2+\\frac{1}{2 \\sin ^2 x_1 \\sin ^2 x_2}\\right)+2\\left(\\frac{\\sin ^2 x_1}{\\sin ^2 x_2}+\\frac{\\sin ^2 x_2}{\\sin ^2 x_1}\\right)\n\\end{aligned}\n$$\n$$\n\\begin{aligned}\n& =2\\left(2 \\sin ^2 x_1 \\sin ^2 x_2+\\frac{1}{2 \\sin ^2 x_1 \\sin ^2 x_2}\\right)+2\\left(\\frac{\\sin ^4 x_1+\\sin ^4 x_2}{\\sin ^2 x_1 \\sin ^2 x_2}\\right) \\\\\n& =2 g\\left(2 \\sin ^2 x_1 \\sin ^2 x_2\\right)+2\\left(\\frac{\\sin ^4 x_1+\\sin ^4 x_2}{\\sin ^2 x_1 \\sin ^2 x_2}\\right) \\text {, 其中 } g(x)=x+\\frac{1}{x} \\text {. } \\\\\n&\n\\end{aligned}\n$$\n注意到 $g(x)$ 在 $(0,1)$ 上递减, 而 $2 \\sin ^2 x_1 \\sin ^2 x_2 \\leqslant 2 \\sin ^2 x_2<2 \\sin ^2 \\frac{\\pi}{4}=1$, 所以\n$$\ng\\left(2 \\sin ^2 x_1 \\sin ^2 x_2\\right)>g\\left(2 \\sin ^2 x_1^{\\prime} \\sin ^2 x_2^{\\prime}\\right) .\n$$\n所以, $f\\left(x_1, x_2\\right)=2 g\\left(2 \\sin ^2 x_1 \\sin ^2 x_2\\right)+2\\left(\\frac{\\sin ^2 x_1}{\\sin ^2 x_2}+\\frac{\\sin ^2 x_2}{\\sin ^2 x_1}\\right)$\n$$\n\\begin{aligned}\n& >2 g\\left(2 \\sin ^2 x_1^{\\prime} \\sin ^2 x_2^{\\prime}\\right)+2\\left(\\frac{\\sin ^4 x_1^{\\prime}+\\sin ^4 x_2^{\\prime}}{\\sin ^2 x_1^{\\prime} \\sin ^2 x_2^{\\prime}}\\right)=f\\left(x_1^{\\prime}, x_2^{\\prime}\\right), \\\\\n& A\\left(x_1, x_2, x_3, x_4\\right)=f\\left(x_1, x_2\\right) f\\left(x_3, x_4\\right)>f\\left(x_1^{\\prime}, x_2^{\\prime}\\right) f\\left(x_3^{\\prime}, x_4^{\\prime}\\right)=\n\\end{aligned}\n$$\n$A\\left(x_1^{\\prime}, x_2^{\\prime}, x_3^{\\prime}, x_4^{\\prime}\\right)$.\n这样, 我们将 $x_1$ 磨光到 $\\frac{\\pi}{4}$, 函值减小.\n如果 $x_2^{\\prime} 、 x_3^{\\prime} 、 x_4^{\\prime}$ 中仍有不等于 $-\\frac{\\pi}{4}$ 者, 则继续上述变换, 由此可见, $A$ 在 $x_1=x_2=x_3=x_4=\\frac{\\pi}{4}$ 时达到最小.\n综上所述, $A$ 的最小值为 $\\left[2 \\sin ^2 \\frac{\\pi}{4}+\\frac{1}{\\sin ^2 \\frac{\\pi}{4}}\\right)^4=81$.", + "remark": "", + "figures": [] +} \ No newline at end of file diff --git a/processed_dataset/calculation/0354.json b/processed_dataset/calculation/0354.json new file mode 100644 index 0000000000000000000000000000000000000000..7d93f0886d5592ab1de00a1844feb5a1af4a3297 --- /dev/null +++ b/processed_dataset/calculation/0354.json @@ -0,0 +1,8 @@ +{ + "source_file": "./raw_volume-zh/volume13/exercise6.tex", + "problem_type": "calculation", + "problem": "问题1. 设 $X$ 是 $\\mathbf{N}^*$ 的子集, $X$ 的最小元为 1 ,最大元为 100 ,对 $X$ 中任何一个大于 1 的数,都可表成 $X$ 中两个数 (可以相同) 的和, 求 $|X|$ 的最小值.", + "solution": "$|X|_{\\min }=9$. 设 $X=\\left\\{a_1, a_2, \\cdots, a_n\\right\\}$, 且 $1=a_11)$, 都存在 $a_i 、 a_j$, 使 $a_k=a_i+a_j$, 其中必有 $a_i1)$, 都存在 $a_i 、 a_j$, 使 $a_k=a_i+a_j$, 其中必有 $a_i1870$, 则 $A$ 必含有某个集合 $A_k$ 中的 2 个数, 其中较大的数是其较小的数的 15 倍, 矛盾.\n故 $|A|$ 的最大值是 1870 .", + "remark": "", + "figures": [] +} \ No newline at end of file diff --git a/processed_dataset/calculation/0364.json b/processed_dataset/calculation/0364.json new file mode 100644 index 0000000000000000000000000000000000000000..50d1e18bad70f93094ebb88c390b6a5505346657 --- /dev/null +++ b/processed_dataset/calculation/0364.json @@ -0,0 +1,8 @@ +{ + "source_file": "./raw_volume-zh/volume13/exercise9.tex", + "problem_type": "calculation", + "problem": "问题2. 设 $A_i$ 为 $M=\\{1,2, \\cdots, 10\\}$ 的子集, 且 $\\left|A_i\\right|=5(i=1,2, \\cdots, k)$, $\\left|A_i \\cap A_j\\right| \\leqslant 2(1 \\leqslant i0$, 这又等价于关于 $x$ 的一元二次方程\n$$\n(c+1) x^2-x+(c-1)=0 \\label{eq2}\n$$\n存在正数解.\n(1) 若 $c \\leqslant-1$, 因为 $(c+1) x^2 \\leqslant 0, c-1<0$, 故方程 式\\ref{eq2} 显然没有正数解;\n(2) 若 $c>-1$, 由于方程 式\\ref{eq2} 中二次项系数大于 0 ,一次项系数小于 0 , 根据根与系数的关系知: 式\\ref{eq2} 存在正数解当且仅当判别式 $\\Delta==1-4(c+1)(c-1) \\geqslant$ 0 , 即 $4 c^2 \\leqslant 5$, 解得 $-1) 所示, 那么选 6个人是可以的, 例如, 选 1, $3,5,7,9,11$ 号这 6 位小朋友, 他们是不相邻的.\n现在来说明至多可选 6 名.\n先任意选定 1 个, 不妨设为 1 号, 这时候与他相邻的 2 号与 13 号不能选了.\n把剩下的 10 位小朋友配成 5 对: $(3,4) 、(5,6) 、(7,8)$ 、 $(9,10) 、(11,12)$. 在这 5 对中,每一对中至多只能选出 1 个, 连同 1 号在内, 至多可选出 6 个人, 他们互不相邻.\n综上所述, 从圈上至多能选出 6 个人, 他们互不相邻.\n(2) 我们把这题\"化归\"为题 (1).\n我们把 $1,2, \\cdots, 13$ 按如下规则排成一个圆圈: 先排 1 , 在 1 的旁边放 9 (与 1 的差为 8), 在 9 的旁边放 4 (与 9 的差为 5 ), ……这样继续放下去, 每个数旁边的数与它相差 8 或 5 ,最后得到如图() 所示的一个圈.\n圈上的数满足:\n(1) 每两个相邻的数的差或是 8 , 或是 5 ;\n(2) 两个不相邻的数的差既不等于 5 , 也不等于 8 .\n于是问题 (2) 就转化为: 在这个圈上至多能选几个数, 使每两个数在圈上不相邻? 由 (1) 的结论知, 答案是 6 . 例如, 选 $1,4,7,10,13,3$.", + "remark": "注从题目上看, (1), (2) 两个小题除了 13 这个数字外, 没有任何相同的地方, 如果直接解 (2), 是比较困难的, 通过转化, 把 (2) 化归为 (1), 问题就解决了.", + "figures": [ + "./images/volume14/figures/fig-c1i1.png", + "./images/volume14/figures/fig-c1i2.png" + ] +} \ No newline at end of file diff --git a/processed_dataset/calculation/0368.json b/processed_dataset/calculation/0368.json new file mode 100644 index 0000000000000000000000000000000000000000..d481d60255938a985a930e510789e435063faa23 --- /dev/null +++ b/processed_dataset/calculation/0368.json @@ -0,0 +1,10 @@ +{ + "source_file": "./raw_volume-zh/volume14/chapter1.tex", + "problem_type": "calculation", + "problem": "例8. 20 个方块分别标有 $1,2,3, \\cdots, 20$, 排成一个圈, 每四个连续的方块可以颠倒次序 (如 $20,1,2,3$ 可以变为 $3,2,1,20$ ). 如果原来的方块依照数的大小顺序排列, 问: 能否通过多次颠倒次序, 将它的次序变为:\n(1) $5,1,2,3,4,6,7, \\cdots, 20$;\n(2) $6,1,2,3,4,5,7, \\cdots, 20$.", + "solution": "解: (1) 的答案是肯定的.\n下面是一个具体的操作方式:\n$$\n1 \\underline{2345} \\rightarrow \\underline{15432} \\rightarrow 3 \\underline{4512} \\rightarrow \\underline{32154} \\rightarrow 51234\n$$\n我们就是把 $1,2,3,4,5$ 这 5 个数字进行 4 次颠倒次序, 其余数字的位置不动, 就变成了 $5,1,2,3,4,6,7, \\cdots, 20$.\n题 (2) 的答案也是肯定的.\n我们也是利用 (1) 所得的结论: 每一方块可经过 4 次颠倒次序前移四位,而其他方块顺序不变.\n如图() 所示, 把数字 6 依次移到 1,2 之间, 17, 18 之间, 14,13 之间, 10 , 9 之间, 4, 5 之间, 20, 1 之间, 此时, 其他的数字的位置不动, 从而, 经过这些颠倒次序后, 就变成了 $6,1,2,3,4,5,7, \\cdots, 20$.", + "remark": "", + "figures": [ + "./images/volume14/figures/fig-c1i3.png" + ] +} \ No newline at end of file diff --git a/processed_dataset/calculation/0369.json b/processed_dataset/calculation/0369.json new file mode 100644 index 0000000000000000000000000000000000000000..a3447c2482e8b59686923f05144a8010aa254b50 --- /dev/null +++ b/processed_dataset/calculation/0369.json @@ -0,0 +1,8 @@ +{ + "source_file": "./raw_volume-zh/volume14/chapter10.tex", + "problem_type": "calculation", + "problem": "例3. 集合 $S=\\{1,2, \\cdots, 3000\\}$ 中是否包含一个具有 2000 个元素的子集 $A$, 它满足下述性质: 当 $x \\in A$ 时, $2 x \\notin A$ ?", + "solution": "解:答案是否定的.\n把每个正整数都表成 $2^s t$ 的形式,其中 $s$ 是非负整数, $t$ 是奇数.\n如果集合 $A \\subseteq S$ 具有性质: 当 $x \\in A$ 时, $2 x \\notin A$, 那么在 $2^s t \\in A$ 时, $2^{s+1} t \\notin A$. 因此, 对每个奇数 $t$, 有\n$$\n\\left|A \\cap\\left\\{t, 2 t, 2^2 t, \\cdots\\right\\}\\right| \\leqslant\\left|S \\cap\\left\\{t, 2^2 t, 2^4 t, \\cdots\\right\\}\\right|,\n$$\n其中 $|X|$ 表示有限集合 $X$ 的元素个数.\n从而, $|A|$ 不大于下述集合的元素个数:\n$$\n\\begin{gathered}\n\\left\\{1,3, \\cdots, 2999,1 \\times 2^2, 3 \\times 2^2, \\cdots, 749 \\times 2^2, 1 \\times 2^4, 3 \\times 2^4, \\cdots, 187 \\times 2^4,\\right. \\\\\n\\left.1 \\times 2^6, 3 \\times 2^6, \\cdots, 45 \\times 2^6, 1 \\times 2^8, 3 \\times 2^8, \\cdots, 11 \\times 2^8, 1 \\times 2^{10}\\right\\}\n\\end{gathered}\n$$\n即\n$$\n|A| \\leqslant 15000+375+94+23+6+1=1999<2000 .\n$$\n从而不存在含 2000 个元素的集合 $A$ 满足题述性质.", + "remark": "注:本例中的条件涉及 $x$ 及其两倍 $2 x$ 关于集合 $A$ 的从属关系,故而把正整数表成 $2^s t$ ( $t$ 为奇数) 的形式有助于讨论, 使问题迎刃而解.", + "figures": [] +} \ No newline at end of file diff --git a/processed_dataset/calculation/0370.json b/processed_dataset/calculation/0370.json new file mode 100644 index 0000000000000000000000000000000000000000..44a7953920647ff1671fb18f2ca62f4ffb852274 --- /dev/null +++ b/processed_dataset/calculation/0370.json @@ -0,0 +1,8 @@ +{ + "source_file": "./raw_volume-zh/volume14/chapter10.tex", + "problem_type": "calculation", + "problem": "例6. 求集合 $S=\\{1,2, \\cdots, n\\}$ 的不含两个相邻整数的非空子集的个数.", + "solution": "解:据正整数 Fibonacci 表示的性质可知, $S$ 的每个不含两个相邻整数的非空子集 $A$ 恰好对应一个小于 $\\left({\\overline{1}(\\underbrace{00 \\cdots 0}_{n \\uparrow 0})_F}^{0 \\text { 个 }}=F_{n+1}\\right.$ 的正整数 $m= \\left(\\overline{a_n a_{n-1} \\cdots a_1}\\right)_F$, 其中只需规定\n$$\na_i= \\begin{cases}1, & i \\in A \\\\ 0, & i \\notin A .\\end{cases}\n$$\n从而满足条件的非空子集个数为 $F_{n+1}-1$, 即 $\\frac{1}{\\sqrt{5}}\\left(\\left(\\frac{1+\\sqrt{5}}{2}\\right)^{n+2}- \\left(\\frac{1-\\sqrt{5}}{2}\\right)^{n+2}\\right)-1$", + "remark": "", + "figures": [] +} \ No newline at end of file diff --git a/processed_dataset/calculation/0371.json b/processed_dataset/calculation/0371.json new file mode 100644 index 0000000000000000000000000000000000000000..c1ab5ff6d48ade3a6f85062898544b4a07116ea7 --- /dev/null +++ b/processed_dataset/calculation/0371.json @@ -0,0 +1,10 @@ +{ + "source_file": "./raw_volume-zh/volume14/chapter11.tex", + "problem_type": "calculation", + "problem": "例1. 设实系数一元二次方程 $x^2+a x+2 b-2=0$ 有两个相异实根, 其中一根在 $(0,1)$ 内,另一根在 $(1,2)$ 内, 求 $\\frac{b-4}{a-1}$ 的取值范围.", + "solution": "解:令 $f(x)=x^2+a x+2 b-2$, 则已知条件等价于 $f(0)>0, f(1)< 0, f(2)>0$, 化简即\n$$\n\\left\\{\\begin{array}{l}\nb>1, \\\\\na+2 b<1, \\\\\na+b>-1 .\n\\end{array}\\right.\n$$\n如图(),在直角坐标平面 $a O b$ 内画出满足这个不等式组的区域,并观察该区域中每个点 $(a, b)$ 与 $P (1,4)$ 的连线的斜率 $\\frac{b-4}{a-1}$, 可得 $\\frac{b-4}{a-1} \\in\\left(\\frac{1}{2}, \\frac{3}{2}\\right)$.", + "remark": "注:本题有 $a, b$ 两个参数,约束条件较多且不直观, 故先等价转化为 $f(0)>0, f(1)<0$, $f(2)>0$, 再把这些抽象的数量关系对应为直观的几何图形位置关系, 每一步都保持了命题的等价性, 同时使问题简单化.\n现另有一种解法同样可以求得结果, 具体如下:\n设两个相异实根为 $x_1, x_2$, 且 $01$. 求 $1,2, \\cdots, n$ 的满足下列性质的排列 $\\left(a_1\\right.$, $\\left.a_2, \\cdots, a_n\\right)$ 的个数: 仅存在一个 $i \\in\\{1,2, \\cdots, n-1\\}$, 使得 $a_i>a_{i+1}$.", + "solution": "解: $p_n$ 表示具有题设性质的排列的个数, $n \\geqslant 2$. 易知, $p_2=1$.\n对于 $n \\geqslant 3$, 若 $a_n=n$, 则这样的排列个数有 $p_{n-1}$ 个;\n若 $a_i=n, 1 \\leqslant i \\leqslant n-1$, 考虑所有这样的排列, 可以从 $n-1$ 个数 1 , $2, \\cdots, n-1$ 中选 $i-1$ 个数按从小到大的顺序排列成 $a_1, a_2, \\cdots, a_{i-1}$, 其余的按从小到大的顺序排列在剩下的位置, 于是有 $\\mathrm{C}_{n-1}^{i-1}$ 种排法, 所以\n$$\np_n=p_{n-1}+\\sum_{i=1}^{n-1} \\mathrm{C}_{n-1}^{i-1}=p_{n-1}+2^{n-1}-1 .\n$$\n即\n$$\n\\begin{gathered}\np_n-p_{n-1}=2^{n-1}-1, \\\\\np_{n-1}-p_{n-2}=2^{n-2}-1, \\\\\n\\cdots \\cdots \\\\\np_2-p_1=2-1 .\n\\end{gathered}\n$$\n把上面这些式子相加, 得\n$$\np_n=\\left(2^{n-1}-1\\right)+\\left(2^{n-2}-1\\right)+\\cdots+(2-1)=2^n-n-1 .\n$$", + "remark": "注:在解决一些计数问题时, 往往题目并不给出明显表达式, 需通过观察、分析、归纳、猜想、论证等来确定递推关系.\n本题中, 将 $p_n$ 分类计数,一类与 $n-1$ 个数的情形相联系, 另一类可直接计数, 从而得到了递推关系.", + "figures": [] +} \ No newline at end of file diff --git a/processed_dataset/calculation/0380.json b/processed_dataset/calculation/0380.json new file mode 100644 index 0000000000000000000000000000000000000000..d073a90cd9b6f586a2267b162fa6144a03805130 --- /dev/null +++ b/processed_dataset/calculation/0380.json @@ -0,0 +1,8 @@ +{ + "source_file": "./raw_volume-zh/volume14/chapter13.tex", + "problem_type": "calculation", + "problem": "例2. 对一个边长互不相等的凸 $n(n \\geqslant 3)$ 边形的边染色, 每条边可以染红、黄、蓝三种颜色中的一种, 但是不允许相邻的边有相同的颜色.\n问: 共有多少种不同的染色方法?", + "solution": "解:设不同的染色法有 $p_n$ 种.\n易知 $p_3=6$.\n当 $n \\geqslant 4$ 时,首先, 对于边 $a_1$, 有 3 种不同的染法, 由于边 $a_2$ 的颜色与边 $a_1$ 的颜色不同, 所以,对边 $a_2$ 有 2 种不同的染法, 类似地, 对边 $a_3, \\cdots$, 边 $a_{n-1}$ 均有 2 种染法.\n对于边 $a_n$, 用与边 $a_{n-1}$ 不同的 2 种颜色染色, 但是, 这样也包括了它与边 $a_1$ 颜色相同的情况, 而边 $a_1$ 与边 $a_n$ 颜色相同的不同染色方法数就是凸 $n-1$ 边形的不同染色方法数的种数 $p_{n-1}$, 于是可得图 $13-1$\n$$\n\\begin{gathered}\np_n=3 \\times 2^{n-1}-p_{n-1}, \\\\\np_n-2^n=-\\left(p_{n-1}-2^{n-1}\\right) .\n\\end{gathered}\n$$\n于是\n$$\n\\begin{gathered}\np_n-2^n=(-1)^{n-3}\\left(p_3-2^3\\right)=(-1)^{n-2} \\cdot 2, \\\\\np_n=2^n+(-1)^n \\cdot 2 .\n\\end{gathered}\n$$\n综上所述, 不同的染色方法数为 $p_n=2^n+(-1)^n \\cdot 2, n \\geqslant 3$.", + "remark": "注:本例与前一例略有不同, 是将计算 $p_n$ 时需扣除的量与 $p_{n-1}$ 建立联系.", + "figures": [] +} \ No newline at end of file diff --git a/processed_dataset/calculation/0381.json b/processed_dataset/calculation/0381.json new file mode 100644 index 0000000000000000000000000000000000000000..9066c942367760bbd85e66ee093c8810e58d11bf --- /dev/null +++ b/processed_dataset/calculation/0381.json @@ -0,0 +1,8 @@ +{ + "source_file": "./raw_volume-zh/volume14/chapter13.tex", + "problem_type": "calculation", + "problem": "例4. 在 $(1,2, \\cdots, n)$ 的一个排列 $\\left(a_1, a_2, \\cdots, a_n\\right)$ 中, 如果 $a_i \\neq i(i= 1,2, \\cdots, n$ ), 则称这种排列为一个错位排列 (也称更列). 求错位排列的个数 $D_n$.", + "solution": "解:知 $D_1=0, D_2=1$.\n对于 $n \\geqslant 3$ 及 $(1,2, \\cdots, n)$ 的任意一个错位排列 $\\left(a_1, a_2, \\cdots, a_n\\right)$ 中, $a_1$ 可取除 1 以外的任一其他 $n-1$ 个数,设 $a_1=k(k \\neq 1)$, 于是\n(1) 如果 $a_k=1$, 这种错位排列数等于 $n-2$ 个元素的错位排列数 $D_{n-2}$;\n(2) 如果 $a_k \\neq 1$, 则这种错位排列就是元素 $1,2, \\cdots, k-1, k+1, \\cdots, n$ 在第 2 到第 $n$ 这 $n-1$ 个位置上的一个排列, 其中 1 不在第 $k$ 个位置, 其他元素都不在它自身所标记的位置上, 这种排列相当于 $2,3, \\cdots, n$ 这 $n-1$ 个元素的一个错位排列, 所以共有 $D_{n-1}$ 个.\n考虑到 $k=2,3, \\cdots, n$ 共 $n-1$ 种这样的情况, 我们得到\n$$\nD_n=(n-1)\\left(D_{n-1}+D_{n-2}\\right)(n \\geqslant 3) . \\label{eq1}\n$$\n令 $E_n=\\frac{D_n}{n !}$, 则 式\\ref{eq1} 可变形为 $E_n=\\left(1-\\frac{1}{n}\\right) E_{n-1}+\\frac{1}{n} E_{n-2}$, 所以\n$$\nE_n-E_{n-1}=-\\frac{1}{n} \\cdot\\left(E_{n-1}-E_{n-2}\\right) . \\label{eq2}\n$$\n反复利用 式\\ref{eq2}, 并注意到 $E_1=0, E_2=\\frac{1}{2}$, 对 $n \\geqslant 2$, 有\n$$\n\\begin{aligned}\nE_n-E_{n-1} & =\\left(-\\frac{1}{n}\\right) \\cdot\\left(-\\frac{1}{n-1}\\right) \\cdot\\left(E_{n-2}-E_{n-3}\\right)=\\cdots \\\\\n& =\\left(-\\frac{1}{n}\\right) \\cdot\\left(-\\frac{1}{n-1}\\right) \\cdots\\left(-\\frac{1}{3}\\right) \\cdot\\left(E_2-E_1\\right)=\\frac{(-1)^n}{n !} .\n\\end{aligned}\n$$\n由此可得\n$$\nE_n=\\frac{(-1)^n}{n !}+\\frac{(-1)^{n-1}}{(n-1) !}+\\cdots+\\frac{(-1)^2}{2 !},\n$$\n所以\n$$\nD_n=n ! \\cdot\\left(1-\\frac{1}{1 !}+\\frac{1}{2 !}-\\cdots+(-1)^n \\frac{1}{n !}\\right) .\n$$", + "remark": "注:这是著名的\"错位排列\"计数问题的一种递推解法.\n在建立递推关系时, 对应原理是必不可少的, 但有时候需要变通地找出对应关系, 例如在上述解法讨论 $a_1=k, a_k \\neq 1$ 的情况时, 把元素 1 暂时视作 $k$, 即对应到 $2,3, \\cdots, n$ 这 $n-1$ 个元素的一个错位排列.", + "figures": [] +} \ No newline at end of file diff --git a/processed_dataset/calculation/0382.json b/processed_dataset/calculation/0382.json new file mode 100644 index 0000000000000000000000000000000000000000..1182915bb2e9533b0b8b7e7c4298395cdccbc1e1 --- /dev/null +++ b/processed_dataset/calculation/0382.json @@ -0,0 +1,8 @@ +{ + "source_file": "./raw_volume-zh/volume14/chapter13.tex", + "problem_type": "calculation", + "problem": "例5. 一种密码锁的密码设置是在正 $n$ 边形 $A_1 A_2 \\cdots A_n$ 的每个顶点处赋值 0 和 1 两个数中的一个, 同时在每个顶点处染红、蓝两种颜色之一, 使得任意相邻的两个顶点的数字或颜色中至少有一个相同.\n问: 该种密码锁共有多少种不同的密码设置方案?", + "solution": "解:设满足条件的密码设置方案共 $a_n$ 种, 其中, 顶点 $A_1, A_n$ 赋值与染色全同的方案有 $b_n$ 种, 其全体构成集合 $B_n ; A_1, A_n$ 赋值与染色恰有一项相同的方案有 $c_n$ 种, 其全体构成集合 $C_n$, 则\n$$\na_n=b_n+c_n . \\label{eq1}\n$$\n此外, 若不考虑相邻顶点 $A_1, A_n$ 间的赋值与染色是否兼容, 首先设置 $A_1$ 位置, 有 4 种方式, 再依次设置 $A_2, \\cdots, A_n$, 各有 3 种方式, 根据乘法原理, 共 $4 \\times 3^{n-1}$ 种方式, 因此, 若将 $A_1, A_n$ 赋值与染色完全不同, 但其余任意两个相邻顶点赋值与染色至少有一项相同的设置方式数记为 $d_n$, 则\n$$\na_n+d_n=4 \\times 3^{n-1} . \\label{eq2}\n$$\n对每种密码设置方案, 考虑顶点 $A_1, A_{n-1}$ 的赋值与染色情况.\n(1)若赋值与染色情况全同, 这样的情况数为 $b_{n-1}$, 每种情况恰可对应 $B_n$ 中的一个元素及 $C_n$ 中的两个元素 (例如, 若 $A_1, A_{n-1}$ 均为 \"0 红\", 则 $A_n$ 可为\" 0 红\"、\" 0 蓝\"或\" 1 红\", 前者对应 $B_n$ 中的方案, 后两者对应 $C_n$ 中的方案).\n(2)若赋值与染色恰有一项相同, 这样的情况数为 $c_{n-1}$, 每种情况恰可对应 $B_n$ 中的一个元素及 $C_n$ 中的一个元素 (例如, 若 $A_1$ 为 \"0 红\", $A_{n-1}$ 为 \"1 红\", 则 $A_n$ 可为\" 0 红\"、\" 1 红\", 前者对应 $B_n$ 中的方案, 后者对应 $C_n$ 中的方案).\n(3)若赋值与染色均不同, 这样的情况数为 $d_{n-1}$, 每种情况恰可对应 $C_n$\n中两个元素 (例如, 若 $A_1$ 为\" 0 红\", $A_{n-1}$ 为\" 1 蓝\", 则 $A_n$ 可为\" 0 蓝\"、\" 1 红\").\n由此,可建立递推关系\n$$\n\\begin{gathered}\nb_n=b_{n-1}+c_{n-1} ; \\label{eq3} \\\\\nc_n=2 b_{n-1}+c_{n-1}+2 d_{n-1} . \\label{eq4}\n\\end{gathered}\n$$\n由 式\\ref{eq1} 和 \\ref{eq3} 知 $b_n=a_{n-1}, c_n=a_n-a_{n-1}$, 结合 式\\ref{eq2} 与 \\ref{eq4} 可整理得 $a_n=a_{n-2}+ 8 \\times 3^{n-2}$, 又枚举得 $a_2=12, a_3=28$, 所以\n$$\na_n=\\left\\{\\begin{array}{l}\n3^n+1, n=3,5,7, \\cdots \\\\\n3^n+3, n=4,6,8, \\cdots .\n\\end{array}\\right.\n$$", + "remark": "注:本题为 2010 年全国高中数学联赛加试最后一题, 标准解法是直接计数, 其中涉及组合式的化简.\n上述解法为递推方法, 虽不算很简洁, 但体现了递推的思维特点: 为了计算 $a_n$, 可根据解题的实际需要引人一系列辅助量 $b_n$, $c_n, d_n$, 通过计数原理清楚地列出它们之间的等量关系 (如果是 $k$ 个辅助量, 等量关系通常应列出 $k+1$ 个), 最后消去辅助量便可得到 $\\left\\{a_n\\right\\}$ 的递推关系.\n下面的一些例子说明递推思想的运用不纯然得借助于递推式,而是可以有灵活多样的形式.", + "figures": [] +} \ No newline at end of file diff --git a/processed_dataset/calculation/0383.json b/processed_dataset/calculation/0383.json new file mode 100644 index 0000000000000000000000000000000000000000..0eb65ca27c7d0c641091094ae4227c128d6f7a88 --- /dev/null +++ b/processed_dataset/calculation/0383.json @@ -0,0 +1,10 @@ +{ + "source_file": "./raw_volume-zh/volume14/chapter14.tex", + "problem_type": "calculation", + "problem": "例2. 一只内空尺寸是 $6 \\times 6 \\times 6$ 的木箱最多能装多少件尺寸为 $1 \\times 2 \\times 4$ 的长方体货物?", + "solution": "解:如图(),将 $6 \\times 6 \\times 6$ 的木箱分隔成 27 个 $2 \\times 2 \\times 2$ 的\"中正方体\",并进行黑白相间染色, 这样共有 1.4 个黑色的、13 个白色的中正方体.\n再将每个中正方体分隔成 8 个小正方体,共 216 个小正方体,其中黑色小正方体 112 个,白色小正方体 104 个.\n设最多能放 $k$ 件货物.\n显然 $k \\leqslant \\frac{216}{8}=27$.\n假定 $k=27$, 则木箱必被填满而不留空隙, 此时每件货物恰好占据 4 个黑色小正方体和 4 个白色小正方体的空间, 但黑、白小正方体的总个数不同, 故得矛盾.\n因此 $k \\leqslant 26$.\n另一方面又可以放人 26 件货物, 只需注意到图中 27 个中正方体可以配成 13 个两两相邻的对(剩下一个黑色的中正方体),每对提供了 $2 \\times 2 \\times 4$ 的空间,可容下两件货物.\n故 $k=26$.", + "remark": "注:本题本身与染色无关, 但若就 $1 \\times 2 \\times 4$ 长方体本身的形状作讨论, 则难以取得进展.\n我们通过染色将木箱的空间进行分类, 说明了有一类空间不可能被填满, 从而找到了解决问题的捷径.", + "figures": [ + "./images/volume14/figures/fig-c14i1.png" + ] +} \ No newline at end of file diff --git a/processed_dataset/calculation/0384.json b/processed_dataset/calculation/0384.json new file mode 100644 index 0000000000000000000000000000000000000000..c52e737ed56014458dc82a2725c9d8b6f5b919af --- /dev/null +++ b/processed_dataset/calculation/0384.json @@ -0,0 +1,10 @@ +{ + "source_file": "./raw_volume-zh/volume14/chapter14.tex", + "problem_type": "calculation", + "problem": "例6. 平面上有一无穷大的方格棋盘, 每格只允许放一枚棋子.\n两枚棋子称为 \"相邻\", 是指它们所在的格子有至少一个公共的顶点.\n一开始棋子正好摆成一个 $9 \\times 9$ 的正方形.\n每一步可选择下述两种规则之一进行操作:\n(1)如果一枚棋子 $X$ 在水平或垂直方向有相邻的棋子 $Y$, 并且 $X$ 关于 $Y$ 对称的格子是空的, 则可在那个空格放一枚棋子, 同时取出 $X$ 和 $Y$ 这两枚棋子;\n(2)如果一枚棋子 $X$ 没有相邻的棋子, 那么可将它取出, 并在所有相邻的 8 个格子中同时放上棋子.\n问 : 能否通过有限步操作使棋盘上只剩一枚棋子?", + "solution": "解:结论是否定的.\n将棋盘按如图()中方式分别染上 $A, B, C$ 三种颜色.\n对于每步操作, 若是按规则 (1) 操作, 则两种颜色的方格所含的棋子数减少 1 , 另一种颜色的方格所含的棋子数增加 1 ; 若是按规则 (2) 操作, 不妨设这枚取出的棋子所在格为 $A$ 色, 由于 $A$ 色格周围 8 格中恰有 2 个 $A$ 色格, 3 个 $B$ 色格, 3 个 $C$ 色格, 因此操作后 $A$ 色格所含的棋子数增加 $1, B$ 色格与 $C$ 色格所含的棋子数各增加 3. 可见无论哪种操作规则, 每步操作后每种颜色的方格所含棋子数的奇偶性都同时改变.\n开始时在 $A, B, C$ 三种颜色的方格内各有 27 枚棋子, 其奇偶性相同.\n假如若干步后棋盘上只剩一枚棋子, 则三种颜色方格内的棋子数肯定分别为 $1,0,0$, 其奇偶性不同, 这不可能.\n因此无法通过有限步操作使棋盘上只剩一枚棋子.", + "remark": "注:本题中运用\"三染色\"方法是解题的关键, 这样便使操作的不变规律 (即三类方格内棋子数总是同时改变奇偶性) 直观地显示出来, 使分析和处理问题变得明朗.\n下面是关于本题的两点补充说明:\n(1)当题目中 $9 \\times 9$ 改成一般的 $n \\times n$ 时,可以证明当且仅当 $n$ 不是 3 的倍数时, 可通过有限步操作使棋盘上只剩一枚棋子(相应的操作步骤可以归纳构造).\n(2)从所采用的染色方法中不难发现,即便放宽规则 (1), 允许被操作的棋子 $X$ 与 $Y$ 之间互为\"左上一一右下\"的相邻关系, 上述证明同样奏效.\n又根据对称性,若规则 (1)放宽为允许互为 \"左下一一右上\"的相邻关系, 那么可将染色方法改为镜像,使证明仍有效.\n但若将规则 (1) 同时作上述两种放宽, 那么本题结论将变为肯定的,请读者自行验证.", + "figures": [ + "./images/volume14/figures/fig-c14i8.png" + ] +} \ No newline at end of file diff --git a/processed_dataset/calculation/0385.json b/processed_dataset/calculation/0385.json new file mode 100644 index 0000000000000000000000000000000000000000..1a0234b8d066f23ec56cff97c43575ecaa24a14e --- /dev/null +++ b/processed_dataset/calculation/0385.json @@ -0,0 +1,8 @@ +{ + "source_file": "./raw_volume-zh/volume14/chapter15.tex", + "problem_type": "calculation", + "problem": "例3. 把 $1,2,3, \\cdots, 2004$ 这 2004 个正整数随意放置在一个圆周上,统计所有相邻三个数的奇偶性得知: 三个数全是奇数的有 600 组, 恰好两个奇数的有 500 组, 问: 恰好一个奇数的有几组? 全部不是奇数的有几组?", + "solution": "解:设恰好 1 个奇数的有 $x$ 组, 则全部不是奇数的有\n$$\n2004-600-500-x=904-x .\n$$\n将圆周上的数从某个数开始, 依次计为 $x_1, x_2, \\cdots, x_{2004}$, 令\n$$\ny_i= \\begin{cases}-1, & \\text { 当 } x_i \\text { 为奇数时, } \\\\ 1, & \\text { 当 } x_i \\text { 为偶数, }\\end{cases}\n$$\n则 $y_1+y_2+\\cdots+y_{2004}=0$, 再令\n$$\nA_i=y_i+y_{i+1}+y_{i+2}= \\begin{cases}-3, & \\text { 当 } x_i, x_{i+1}, x_{i+2} \\text { 全为奇数时, } \\\\ -1, & \\text { 当 } x_i, x_{i+1}, x_{i+2} \\text { 恰好 } 2 \\text { 个奇数时, } \\\\ 1, & \\text { 当 } x_i, x_{i+1}, x_{i+2} \\text { 恰好一个奇数时, } \\\\ 3, & \\text { 当 } x_i, x_{i+1}, x_{i+2} \\text { 全为偶数时, }\\end{cases}\n$$\n其中约定 $x_{2004+i}=x_i(i=1,2)$. 于是\n$$\n\\begin{aligned}\n0 & =3\\left(y_1+y_2+\\cdots+y_{2004}\\right)=A_1+A_2+\\cdots+A_{2004} \\\\\n& =-3 \\times 600-500+x+3(904-x),\n\\end{aligned}\n$$\n解得 $x=206$.\n恰好一个奇数的有 206 组, 全部不是奇数的有 $904-206=698$ 组.", + "remark": "", + "figures": [] +} \ No newline at end of file diff --git a/processed_dataset/calculation/0386.json b/processed_dataset/calculation/0386.json new file mode 100644 index 0000000000000000000000000000000000000000..0f5e0d20b3f0f7cb3846763f76e5838ba1e213a5 --- /dev/null +++ b/processed_dataset/calculation/0386.json @@ -0,0 +1,12 @@ +{ + "source_file": "./raw_volume-zh/volume14/chapter15.tex", + "problem_type": "calculation", + "problem": "例4. 将 $5 \\times 7$ 棋盘用某种规格的若干张纸片覆盖, 纸片不许超出棋盘, 但可彼此交叠.\n考虑下述各种情形, 能否找到一种覆盖方式, 使得棋盘的每个小方格被覆盖的层数相同?(注 : 纸片可翻折使用,但纸片的边必须平行于棋盘的边.)\n(1) 纸片为 $1 \\times 3$ 的规格;\n(2)纸片为 \"特里米诺\", 即 $2 \\times 2$ 方格纸去掉一个方格后所余下的图形;\n(3)纸片为 \"刀把五\", 即 $2 \\times 3$ 方格纸去掉一个角上方格后所余下的图形.", + "solution": "解:1) 按下表,即如图() 对棋盘的每个方格赋值, 易见每张 $1 \\times 3$ 纸片所覆盖的三数之和为 0 ,因而无论用多少张纸片,盖住的所有数之和恒等于 0 (一个数被盖了几层就计算几次). 但棋盘上的数字总和为 1 , 假如被恰好覆盖 $k$ 层, 则盖住的数字之和应为 $k$, 因此不存在满足条件的覆盖方式.\n\\begin{tabular}{|c|c|c|c|c|c|c|}\n\\hline 2 & -1 & -1 & 2 & -1 & -1 & 2 \\\\\n\\hline-1 & -1 & 2 & -1 & -1 & 2 & -1 \\\\\n\\hline-1 & 2 & -1 & -1 & 2 & -1 & -1 \\\\\n\\hline 2 & -1 & -1 & 2 & -1 & -1 & 2 \\\\\n\\hline-1 & -1 & 2 & -1 & -1 & 2 & -1 \\\\\n\\hline\n\\end{tabular}\n(2)按下表,即如图() 对棋盘的每个方格赋值,易见每张\"特里米诺\"纸片所覆盖的三数之和不大于 0 , 因而无论用多少张纸片, 盖住的所有数之和是个非正数.\n但棋盘上的数字总和为 1 , 覆盖 $k$ 层时盖住的数字之和等于正数 $k$,因此不存在满足条件的覆盖方式.\n\\begin{tabular}{|c|c|c|c|c|c|c|}\n\\hline 2 & -1 & 2 & -1 & 2 & -1 & 2 \\\\\n\\hline-1 & -1 & -1 & -1 & -1 & -1 & -1 \\\\\n\\hline 2 & -1 & 2 & -1 & 2 & -1 & 2 \\\\\n\\hline-1 & -1 & -1 & -1 & -1 & -1 & -1 \\\\\n\\hline 2 & -1 & 2 & -1 & 2 & -1 & 2 \\\\\n\\hline\n\\end{tabular}\n(3)按下表,即如图() 对棋盘的每个方格赋值,易见每张\"刀把五\"纸片所覆盖的五数之和不小于 0 , 因而无论用多少张纸片, 盖住的所有数之和是个非负数.\n但棋盘上的数字总和为 -5 , 覆盖 $k$ 层时盖住的数字之和小于 0 , 因此不存在满足条件的覆盖方式.\n\\begin{tabular}{|c|c|c|c|c|c|c|}\n\\hline-1 & -1 & -1 & -1 & -1 & -1 & -1 \\\\\n\\hline-1 & 4 & -1 & 4 & -1 & 4 & -1 \\\\\n\\hline-1 & -1 & -1 & -1 & -1 & -1 & -1 \\\\\n\\hline-1 & 4 & -1 & 4 & -1 & 4 & -1 \\\\\n\\hline-1 & -1 & -1 & -1 & -1 & -1 & -1 \\\\\n\\hline\n\\end{tabular}", + "remark": "注:本题的 3 个小题都运用赋值法来求解,但由于每种规格的纸片特征不同, 所以具体做法也不同.\n本题中, 棋盘的特性也是重要的.\n特别地, 在解 (2)、(3)两小题时, 都是赋以两种不同的值对方格分类,从局部看,这两种分类方式一样 (其中的 \" 2 \" 和 \" 4 \" 都是隔行、隔列出现),但整体上来看, $5 \\times 7$ 棋盘边界的特性就影响到赋值后解决问题的可行性.\n读者不妨自行试探, 仔细体会上述每种赋值方式对具体问题的\"针对性\" 效果.\n另外, 如果在纸片规格和棋盘规格等方面进行推广, 仍有大量问题可以研究.", + "figures": [ + "./images/volume14/figures/fig-c15i2.png", + "./images/volume14/figures/fig-c15i3.png", + "./images/volume14/figures/fig-c15i4.png" + ] +} \ No newline at end of file diff --git a/processed_dataset/calculation/0387.json b/processed_dataset/calculation/0387.json new file mode 100644 index 0000000000000000000000000000000000000000..8fc5509854126d2d0f1fdbe91dc2efe65d784e0b --- /dev/null +++ b/processed_dataset/calculation/0387.json @@ -0,0 +1,8 @@ +{ + "source_file": "./raw_volume-zh/volume14/chapter16.tex", + "problem_type": "calculation", + "problem": "例4. 设 $n(n \\geqslant 3)$ 是给定的正整数,对于 $n$ 个给定的实数 $a_1, a_2, \\cdots$, $a_n$, 记 $\\left|a_i-a_j\\right|(1 \\leqslant i1$, 则 $x_1+x_2=\\left(x_1-1\\right)+\\left(x_2+1\\right)$, 且\n$$\n\\left(x_1-1\\right)^2+\\left(x_2+1\\right)^2=x_1^2+x_2^2+2\\left(x_2-x_1\\right)+2>x_1^2+x_2^2 .\n$$\n所以,当 $x_1>1$ 时, 可以把 $x_1$ 逐步调整到 1 , 这时, $x_1^2+x_2^2+\\cdots+x_{40}^2$ 将增大; 同样地, 可以把 $x_2, x_3, \\cdots, x_{39}$ 逐步调整到 1 , 这时 $x_1^2+x_2^2+\\cdots+x_{40}^2$ 将增大.\n于是, 当 $x_1, x_2, \\cdots, x_{39}$ 均为 $1, x_{40}=19$ 时, $x_1^2+x_2^2+\\cdots+x_{40}^2$ 取得最大值, 即\n$$\nA=\\underbrace{1^2+1^2+\\cdots+1^2}_{39 \\uparrow}+19^2=400 .\n$$\n若存在两个数 $x_i, x_j$, 使得 $x_j-x_i \\geqslant 2(1 \\leqslant i0$, 故 $F_{q-p}$ 是 10000 的倍数, 且 $1 \\leqslant q-p \\leqslant 10^8$, 说明 $F_{q-p}$ 的确是前一亿项中的一项.\n结论成立.", + "remark": "注:本例若直接递推或构造出某项是 10000 的倍数固然直截了当, 但显然困难重重, 而作为一个存在性命题, 抽屉原理也许是解决的办法之一.\n本例中用的是通过剩余类制造抽屉的方法, 然而若单纯地证明存在两项关于模 10000 同余并无多大用处, 关键在于如何使同余关系不断\"前置\", 最终得到某项与 $F_0=0$ 同余.\n考虑到有 $F_{n-1}=F_{n+1}-F_n$, 我们只需说明存在两个数组 $\\left(F_n, F_{n+1}\\right)$, 它们每个分量对应同余即可, 而这点仍可用抽屉原理给出证明.", + "figures": [] +} \ No newline at end of file diff --git a/processed_dataset/calculation/0397.json b/processed_dataset/calculation/0397.json new file mode 100644 index 0000000000000000000000000000000000000000..be4a869f41fe4511633cea789b4fa4c4fbea0d5f --- /dev/null +++ b/processed_dataset/calculation/0397.json @@ -0,0 +1,8 @@ +{ + "source_file": "./raw_volume-zh/volume14/chapter4.tex", + "problem_type": "calculation", + "problem": "例11. 某次考试有 5 道选择题, 每题都有 4 个不同的答案供选择, 每人每题恰选 1 个答案.\n在 2000 份答卷中发现存在一个 $n$, 使得任何 $n$ 份答卷中都存在 4 份, 其中每两份的答案都至多有 3 道题相同.\n求 $n$ 的最小可能值.", + "solution": "解:将每题的四个选项依次记为 $1,2,3,4$, 每份答案记为 $(a, b, c, d$, $e)$, 其中 $a, b, c, d, e \\in\\{1,2,3,4\\}$.\n对给定数组 $(a, b, c, d), a, b, c, d \\in\\{1,2,3,4\\}$, 将 2000 份答卷中答案为 $(a, b, c, d, 1),(a, b, c, d, 2),(a, b, c, d, 3),(a, b, c, d, 4)$ 的答卷归为一类, 这样共 $4^4=256$ 类.\n由抽屉原理, 至少有 $\\left\\lceil\\frac{2000}{256}\\right\\rceil=8$ 份答卷属于一类.\n不妨取其中 8 份答卷作为 $A$ 组答卷; 在剩下的答卷中, 同理至少可取出 $\\left\\lceil\\frac{1992}{256}\\right\\rceil=8$ 份, 作为 $B$ 组答卷; 剩下的答卷中仍至少可取出 $\\left\\lceil\\frac{1984}{256}\\right\\rceil=8$ 份, 作为 $C$ 组答卷.\n在 $A, B, C$ 组这 24 份答卷中, 根据抽屉原理, 任意 4 份答卷必有两份在一组, 则它们至少有 4 题答案相同.\n所以 $n \\geqslant 25$.\n另一方面,当 $n=25$ 时,构造如下 2000 份答卷:\n取 250 组不同的数组 $(a, b, c, d), a, b, c, d \\in\\{1,2,3,4\\}$, 对其中每个数组, 取 $e \\in\\{1,2,3,4\\}$ 使得 $a+b+c+d+e$ 是 4 的倍数, 对这样一组 ( $a$, $b, c, d, e)$, 令 8 份答卷写有这样的答案, 共 2000 份答卷.\n任取其中 25 份答卷, 必有 4 份答案两两不同, 再根据数组 $(a, b, c, d, e)$ 的取法可知, 其中任何两份答卷至多只有 3 题答案相同.\n综上可知, $n$ 的最小可能值为 25 .", + "remark": "注:本题的表述比较复杂, 应先仔细分析题目要求.\n方面我们要构造 2000 份答卷, 尽可能使比较小的 $n$ 满足题意; 另一方面又要找到尽可能大的 $k$, 使得任意 2000 份答卷都不满足题意, 即存在 $k$ 份答卷, 使得其中任何 4 份答卷中, 总有两份的答案至少有 4 题一致.\n在上述两方面的讨论中, \"任何 $n$ 份答卷\"、\"任意 2000 份答卷\"、\"任何 4 份答卷\"这些表述均涉及到答卷的任意性, 而要处理的又是存在性问题, 于是抽屉原理便有了用武之地(注意在解答最后一步验证\"25 份答卷中必有 4 份答案两两不同\"时,也用了抽屉原理: 假设 25 份答卷中只出现 3 种不同的答案, 那么必有一种答案出现超过 8 次).\n抽屉原理是解决存在性问题的有力工具.\n抽屉原理本身并不难,难在如何运用它去解决问题.\n在具体问题中如何制造抽屉, 希望本节的例题对读者能有所启示.", + "figures": [] +} \ No newline at end of file diff --git a/processed_dataset/calculation/0398.json b/processed_dataset/calculation/0398.json new file mode 100644 index 0000000000000000000000000000000000000000..ce89ca48f88f5e7c7466b21387258630f73da59e --- /dev/null +++ b/processed_dataset/calculation/0398.json @@ -0,0 +1,8 @@ +{ + "source_file": "./raw_volume-zh/volume14/chapter5.tex", + "problem_type": "calculation", + "problem": "例1. 在不大于 1000 的正整数中, 有多少个数既不被 5 整除又不被 7 整除? 这些正整数的和是多少?", + "solution": "解:设不大于 1000 的正整数组成集合 $I$, 对 $k=5,7,35$, 设 $I$ 中所有 $k$ 的倍数组成集合 $A_k$, 其中 $A_{35}=A_5 \\cap A_7$.\n由容厉原理得, 所求正整数的个数为\n$$\n\\begin{aligned}\n& \\left|\\overline{A_5} \\cap \\overline{A_7}\\right|=|I|-\\left|A_5\\right|-\\left|A_7\\right|+\\left|A_{35}\\right| \\\\\n= & 1000-\\left[\\frac{1000}{5}\\right]-\\left[\\frac{1000}{7}\\right]+\\left[\\frac{1000}{35}\\right] \\\\\n= & 1000-200-142+28=686 .\n\\end{aligned}\n$$\n这些正整数的和为\n$$\n\\begin{aligned}\n& \\sum_{i=1}^{1000} i-\\sum_{i \\in A_5} i-\\sum_{i \\in A_7} i+\\sum_{i \\in A_{35}} i=\\sum_{i=1}^{1000} i-\\sum_{i=1}^{200} 5 i-\\sum_{i=1}^{142} 7 i+\\sum_{i=1}^{28} 35 i \\\\\n= & \\frac{1000 \\times 1001}{2}-\\frac{5 \\times 200 \\times 201}{2}-\\frac{7 \\times 142 \\times 143}{2}+\\frac{35 \\times 28 \\times 29}{2} \\\\\n= & 343139 .\n\\end{aligned}\n$$", + "remark": "注:在求元素个数时, 我们直截了当地应用容斥原理; 在求元素之和时, 则采用了与容斥原理同样的思想.", + "figures": [] +} \ No newline at end of file diff --git a/processed_dataset/calculation/0399.json b/processed_dataset/calculation/0399.json new file mode 100644 index 0000000000000000000000000000000000000000..fd7d96bc85cf851b142da77acb220ebb6a0d49d7 --- /dev/null +++ b/processed_dataset/calculation/0399.json @@ -0,0 +1,8 @@ +{ + "source_file": "./raw_volume-zh/volume14/chapter5.tex", + "problem_type": "calculation", + "problem": "例3. 在 $(1,2, \\cdots, n)$ 的一个排列 $\\left(a_1, a_2, \\cdots, a_n\\right)$ 中, 如果 $a_i \\neq i(i= 1,2, \\cdots, n$ ), 则称这种排列为一个错位排列 (也称更列). 求错位排列的个数 $D_n$.", + "solution": "解:设 $(1,2, \\cdots, n)$ 的所有排列组成集合 $I$, 并将 $I$ 中满足条件 $a_i=i$ 的排列全体记为 $A_i$. 显然 $D_n=\\left|\\overline{A_1} \\cap \\overline{A_2} \\cap \\cdots \\cap \\overline{A_n}\\right|$. 易知\n$$\n|I|=\\mathrm{P}_n^n=n !,\\left|A_i\\right|=\\mathrm{P}_{n-1}^{n-1}=(n-1) !(1 \\leqslant i \\leqslant n),\n$$\n同理, 对 $1 \\leqslant i_10$.\n又 $\\frac{b+1}{a^{\\prime}}+\\frac{a^{\\prime}+1}{b}=k$, 由 $b$ 的假定知 $a \\geqslant b, a^{\\prime} \\geqslant b$, 因此 $a, a^{\\prime}$ 必为 $b$, $b+1$ 的一个排列.\n这样就有 $k=\\frac{a+a^{\\prime}+1}{b}=2+\\frac{2}{b}$.\n所以 $b=1,2$, 从而 $k=3,4$.\n取 $a=b=1$ 知 $k=4$ 可取到,取 $a=b=2$ 知 $k=3$ 可取到.\n所以 $k=3,4$.", + "remark": "", + "figures": [] +} \ No newline at end of file diff --git a/processed_dataset/calculation/0402.json b/processed_dataset/calculation/0402.json new file mode 100644 index 0000000000000000000000000000000000000000..693f27bb95155edbcaead50640b25007e58c1354 --- /dev/null +++ b/processed_dataset/calculation/0402.json @@ -0,0 +1,8 @@ +{ + "source_file": "./raw_volume-zh/volume14/chapter7.tex", + "problem_type": "calculation", + "problem": "例6. 设 $n \\in \\mathbf{N}^*$, 且使得 $37.5^n+26.5^n$ 为正整数, 求 $n$ 的值.", + "solution": "解:易知 $37.5^n+26.5^n=\\frac{1}{2^n}\\left(75^n+53^n\\right)$.\n当 $n$ 为正偶数时,\n$$\n75^n+53^n \\equiv(-1)^n+1^n \\equiv 2(\\bmod 4),\n$$\n即 $75^n+53^n=4 m+2$, 这里 $m \\in \\mathbf{N}^*$. 故 $37.5^n+26.5^n=\\frac{1}{2^{n-1}}(2 m+1)$ 不是正整数.\n当 $n$ 为正奇数时,\n$$\n\\begin{aligned}\n75^n+53^n & =(75+53)\\left(75^{n-1}-75^{n-2} \\cdot 53+\\cdots-75 \\cdot 53^{n-2}+53^{n-1}\\right) \\\\\n& =2^7 \\cdot\\left(75^{n-1}-75^{n-2} \\cdot 53+\\cdots-75 \\cdot 53^{n-2}+53^{n-1}\\right) .\n\\end{aligned}\n$$\n上式括号内有 $n$ 项,每一项都是奇数, 因而和为奇数.\n由此可见, 只有当 $n=1,3,5,7$ 时, $37.5^n+26.5^n$ 为正整数.", + "remark": "注:我们常常会遇到需对奇偶作分类讨论的情况, 而分类讨论的本质是 \"加条件解题\", 是求解数学问题的一种基本思想.\n本题采用了对 $n$ 的奇偶性分类讨论的解题策略, 这与题目本身的特点有极大关系.\n就 $a^n+b^n$ ( $a, b$ 为整数,例如本题中 $a=75, b=53$ ) 的结构而言, 当 $n$ 为奇数时便于因式分解, 当 $n$为偶数时则便于从模 4 的剩余类考虑问题.\n在两种情形的讨论中, 人为创设的前提条件 \" $n$ 为奇数\" 或是 \" $n$ 为偶数\" 均对进一步分析问题起到了重要的作用.", + "figures": [] +} \ No newline at end of file diff --git a/processed_dataset/calculation/0403.json b/processed_dataset/calculation/0403.json new file mode 100644 index 0000000000000000000000000000000000000000..59d94fd40051c75dce2525672e233a6a5ad9232e --- /dev/null +++ b/processed_dataset/calculation/0403.json @@ -0,0 +1,8 @@ +{ + "source_file": "./raw_volume-zh/volume14/chapter7.tex", + "problem_type": "calculation", + "problem": "例8. 设有一个正 $2 n+1$ 边形 $(n>1)$. 两人按如下法则做游戏: 轮流在该正多边形内画对角线; 每人每次画一条新的 (以前没有画过的)对角线,而它恰好与已画出的偶数条对角线相交 (交点在正多边形内); 凡无法按照要求画出对角线者即为负方.\n问: 谁有取胜策略?", + "solution": "解:将先开始的人称为甲, 后开始的人称为乙.\n我们断言: 如果 $n$ 为奇数,则乙必胜; 如果 $n$ 为偶数,则甲必胜.\n对正 $2 n+1$ 边形的任何一条对角线来说, 它两侧的顶点个数和为奇数, 必有一侧有偶数个顶点.\n因此每条对角线与偶数条其他对角线相交.\n假设到某个时刻游戏无法继续, 那么此时每条未画出的对角线都与奇数条已画的对角线相交, 也与奇数条未画的对角线相交.\n这样的情况只能出现在未画的对角线条数为偶数的时刻 (事实上, 假设此时未画的对角线 $d_i$ 共奇数条, 由于每个 $d_i$ 上共有奇数个它们相互之间的交点, 因而从所有 $d_i$ 上数得的交点总数为奇数, 但每个交点恰被计数两次, 数出的交点数理应为偶数, 矛盾). 由此可知, 甲能取胜当且仅当该正 $2 n+1$ 边形的对角线总数为奇数.\n在正 $2 n+1$ 边形中, 对角线共有 $\\frac{(2 n+1)(2 n-2)}{2}=(n-1)(2 n+1)$ 条,\n所以当 $n$ 为奇数时, 对角线有偶数条, 乙必胜; 当 $n$ 为偶数时, 则甲必胜.\n而且任何一方取胜不需要制定特别的策略.", + "remark": "注:本题不妨先对 $n=2,3$ 等较小情况予以探索, 发现 $n=2$ 时甲必胜, $n=3$ 时乙必胜, 同时也发现当 $n=3$ 时情况已经变得相当复杂, 很难真正为乙设计一种合适的取胜策略, 但另一方面又能发现, 乙获胜似乎是自然而然的, 无需特别的策略.\n于是我们再回到题目条件,充分利用\"正 $2 n+1$ 边形\" 及\"恰好与已画出的偶数条对角线相交\" 这些涉及奇偶性的信息来作分析, 并结合了\"算两次\" 的技巧,最终获知游戏必停止于 \"偶数条对角线未画\" 的时刻.\n从而, 一旦确定正 $2 n+1$ 边形对角线条数的奇偶性, 就能确定获胜方.", + "figures": [] +} \ No newline at end of file diff --git a/processed_dataset/calculation/0404.json b/processed_dataset/calculation/0404.json new file mode 100644 index 0000000000000000000000000000000000000000..cd39fb80fcf9b789ffaddd5dbf23a5cdb516f775 --- /dev/null +++ b/processed_dataset/calculation/0404.json @@ -0,0 +1,8 @@ +{ + "source_file": "./raw_volume-zh/volume14/chapter9.tex", + "problem_type": "calculation", + "problem": "例1. 设数列 $\\left\\{a_n\\right\\}$ 与 $\\left\\{b_n\\right\\}$ 满足 $a_1=1, b_1=3$, 且\n$$\n\\left\\{\\begin{array}{l}\na_{n+1}=a_n+b_n-\\sqrt{a_n^2-a_n b_n+b_n^2}, \\\\\nb_{n+1}=b_n+a_n+\\sqrt{b_n^2-b_n a_n+a_n^2},\n\\end{array} n=1,2, \\cdots .\\right.\n$$\n求数列 $\\left\\{a_n\\right\\}$ 与 $\\left\\{b_n\\right\\}$ 的通项公式.", + "solution": "解:已知得:\n$$\n\\begin{aligned}\n& a_{n+1}+b_{n+1}=2\\left(a_n+b_n\\right), \\\\\n& a_{n+1} b_{n+1}=\\left(a_n+b_n\\right)^2-\\left(a_n^2-a_n b_n+b_n^2\\right)=3 a_n b_n .\n\\end{aligned}\n$$\n从而\n$$\n\\begin{aligned}\n& a_n+b_n=2^{n-1}\\left(a_1+b_1\\right)=2^{n+1}, \\\\\n& a_n b_n=3^{n-1} a_1 b_1=3^n,\n\\end{aligned}\n$$\n故 $a_n, b_n$ 是方程 $x^2-2^{n+1} x+3^n=0$ 的两个实根, 且根据条件得 $a_n0$, 但两侧的数字也减小了, 是否会产生新的负数未尝可知.\n如此讨论下去无济于事.\n上述解答中找到了\"五个数的平方和再加上每相邻两数和的平方\"这一整体结构, 它比\"5 个顶点上的数字之和\"优越的地方在于: 前者在操作中具有递减性(当然这依赖于后者在操作中的不变性), 因此有助于判定操作的有限性.\n本题中,也可借助下面的整体结构 $f$ 来证明操作的有限性:\n将 5 个整数依次写为 $u_1, u_2, u_3, u_4, u_5$, 其中 $u_1, u_5$ 也相邻.\n令\n$$\n\\begin{aligned}\nf\\left(u_1, u_2, u_3, u_4, u_5\\right)= & \\sum_{i=1}^5\\left|u_i\\right|+\\sum_{i=1}^5\\left|u_i+u_{i+1}\\right|+\\sum_{i=1}^5 \\mid u_i+u_{i+1}+ \\\\\n& u_{i+2}\\left|+\\sum_{i=1}^5\\right| u_i+u_{i+1}+u_{i+2}+u_{i+3} \\mid,\n\\end{aligned}\n$$\n其中 $u_{5+i}=u_i, i=1,2,3$. 可证明每次操作使 $f$ 的值严格递减.\n纵观两个方法, 关键在于找到一个恰当的整体化的结构, 这种结构往往具有一定的对称美.", + "figures": [] +} \ No newline at end of file diff --git a/processed_dataset/calculation/0407.json b/processed_dataset/calculation/0407.json new file mode 100644 index 0000000000000000000000000000000000000000..e16af3364ea9782aaecc1b58dc8c44fdf76a5658 --- /dev/null +++ b/processed_dataset/calculation/0407.json @@ -0,0 +1,8 @@ +{ + "source_file": "./raw_volume-zh/volume14/exercise1.tex", + "problem_type": "calculation", + "problem": "问题1. 求 $x \\sqrt{1-y^2}+y \\sqrt{1-x^2}$ 的最大值.", + "solution": "解:法一:令 $x=\\sin \\alpha, y=\\sin \\beta$, 其中 $\\alpha, \\beta \\in\\left[-\\frac{\\pi}{2}, \\frac{\\pi}{2}\\right]$, 则\n$$\nx \\sqrt{1-\\overline{y^2}}+y \\sqrt{1-x^2}=\\sin \\alpha \\cos \\beta+\\sin \\beta \\cos \\alpha=\\sin (\\alpha+\\beta) \\leqslant 1,\n$$\n等号可在 $\\alpha+\\beta=\\frac{\\pi}{2}$ 时取到.", + "remark": "", + "figures": [] +} \ No newline at end of file diff --git a/processed_dataset/calculation/0408.json b/processed_dataset/calculation/0408.json new file mode 100644 index 0000000000000000000000000000000000000000..26c8cdda70357cb5b5ceb7feea249b20179f7db4 --- /dev/null +++ b/processed_dataset/calculation/0408.json @@ -0,0 +1,8 @@ +{ + "source_file": "./raw_volume-zh/volume14/exercise1.tex", + "problem_type": "calculation", + "problem": "问题3. 5 这个数, 可以写成 3 个正整数之和, 如果计入不同的顺序, 则有 6 种方式, 即\n$5=1+1+3=1+3+1=3+1+1=1+2+2=2+1+2=2+2+1$.\n设 $m 、 n$ 都是正整数, 且 $m \\leqslant n$, 问 $n$ 可以用多少种方式写为 $m$ 个正整数之和 (计入顺序)?", + "solution": "把 $n$ 写成 $n$ 个 1 的和:\n$$\nn=1+1+\\cdots+1 \\text {. }\n$$\n我们要求的方式数就转化为从上式中的 $n-1$ 个加号中选 $m-1$ 个的选法数, 即 $\\mathrm{C}_{n^{-1}}^{m-1}$.", + "remark": "注:计数问题常可用一个集合中的元素与另一其元素更易计数的集合 中的元素\"对应\"的办法来\"化简\".", + "figures": [] +} \ No newline at end of file diff --git a/processed_dataset/calculation/0409.json b/processed_dataset/calculation/0409.json new file mode 100644 index 0000000000000000000000000000000000000000..16a8e1b20c37412fa6d8ac41330f1c40e09b4f64 --- /dev/null +++ b/processed_dataset/calculation/0409.json @@ -0,0 +1,8 @@ +{ + "source_file": "./raw_volume-zh/volume14/exercise1.tex", + "problem_type": "calculation", + "problem": "问题6. 设 $P$ 是三角形 $A B C$ 内部的一个点, $D 、 E 、 F$ 分别是由 $P$ 向线段 $B C$ 、 $C A 、 A B$ 作垂线所得的垂足,求使\n$$\n\\frac{B C}{P D}+\\frac{C A}{P E}+\\frac{A B}{P F}\n$$\n达到最小时点 $P$ 的位置.", + "solution": "设 $B C 、 C A 、 A B$ 的长度分别为 $a 、 b 、 c ; P D 、 P E 、 P F$ 的长度分别为 $p 、 q 、 r$. 我们要在 $\\triangle A B C$ 的内部找一点 $P$, 使 $\\frac{a}{p}+\\frac{b}{q}+\\frac{c}{r}$ 达到最小.\n由于\n$$\n\\begin{aligned}\nS_{\\triangle A B C} & =S_{\\triangle P B C}+S_{\\triangle P C A}+S_{\\triangle P A B} \\\\\n& =\\frac{1}{2} a p+\\frac{1}{2} b q+\\frac{1}{2} c r,\n\\end{aligned}\n$$\n所以, $a p+b q+c r=2 S_{\\triangle A B C}$ 是一个与 $P$ 点的位置无关的常数, 因此, 我们可以用 $(a p+b q+c r)\\left(\\frac{a}{p}+\\frac{b}{q}+\\frac{c}{r}\\right)$ 取最小值来代替使 $\\frac{a}{p}+\\frac{b}{q}+\\frac{c}{r}$ 取最小值(问题已被转化).\n由柯西不等式\n$$\n(a p+b q+c r)\\left(\\frac{a}{p}+\\frac{b}{q}+\\frac{c}{r}\\right) \\geqslant(a+b+c)^2,\n$$\n并且当 $a p \\times \\frac{p}{a}=b q \\times \\frac{q}{b}=c r \\times \\frac{r}{c}$, 即 $p=q=r$ 时上述不等式取等号, 从而当 $p=q=r$ 时, $(a p+b q+c r)\\left(\\frac{a}{p}+\\frac{b}{q}+\\frac{c}{r}\\right)$ 取最小值, 也就是说, 当 $P$ 为 $\\triangle A B C$ 的内心时, $\\frac{a}{p}+\\frac{b}{q}+\\frac{c}{r}$ 取到最小值.", + "remark": "", + "figures": [] +} \ No newline at end of file diff --git a/processed_dataset/calculation/0410.json b/processed_dataset/calculation/0410.json new file mode 100644 index 0000000000000000000000000000000000000000..15c82a7710a14dadd6d127d9dc296f2bc4ecdad7 --- /dev/null +++ b/processed_dataset/calculation/0410.json @@ -0,0 +1,8 @@ +{ + "source_file": "./raw_volume-zh/volume14/exercise1.tex", + "problem_type": "calculation", + "problem": "问题7. 已知 $t$ 为一元二次方程 $x^2-3 x+1=0$ 的根.\n(1) 对任一给定的有理数 $a$, 求有理数 $b, c$, 使得 $(t+a)(b t+c)=1$ 成立;\n(2) 将 $\\frac{1}{t^2+2}$ 表示成 $d t+e$ 的形式, 其中 $d, e$ 为有理数.", + "solution": "(1) 解方程得 $t=\\frac{3 \\pm \\sqrt{5}}{2}$ 是无理数, 由 $(t+a)(b t+c)=1$ 得\n$$\nb t^2+(a b+c) t+a c-1=0 .\n$$\n因为 $t^2-3 t+1=0$, 所以 $t^2=3 t-1$, 于是上式可化为\n$$\n(3 b+a b+c) t-b+a c-1=0,\n$$\n由于 $t$ 是无理数,所以\n$$\n\\left\\{\\begin{array}{l}\n3 b+a b+c=0 \\\\\n-b+a c-1=0\n\\end{array}\\right.\n$$\n因为 $a, b$ 是有理数,所以 $a^2+3 a+1 \\neq 0$, 由上面方程组解得\n$$\nb=-\\frac{1}{a^2+3 a+1}, c=-\\frac{a+3}{a^2+3 a+1} \\text {. }\n$$\n(2)因为 $t^2+2=(3 t-1)+2=3 t+1=3\\left(t+\\frac{1}{3}\\right)$, 由(1)知, 对 $a=\\frac{1}{3}$, 有\n$$\nb=-\\frac{1}{a^2+3 a+1}=-\\frac{9}{19}, c=\\frac{a+3}{a^2+3 a+1}=\\frac{30}{19},\n$$\n使得\n$$\n\\left(t+\\frac{1}{3}\\right)\\left(-\\frac{9}{19} t+\\frac{30}{19}\\right)=1,\n$$\n所以\n$$\n\\frac{1}{t^2+2}=\\frac{1}{3}\\left(-\\frac{9}{19} t+\\frac{30}{19}\\right)=-\\frac{3}{19} t+\\frac{10}{19} \\text {. }\n$$", + "remark": "注:本题的第 (2) 小题, 我们就利用了 (1) 的结论, 从而把陌生问题化归为熟悉问题.", + "figures": [] +} \ No newline at end of file diff --git a/processed_dataset/calculation/0411.json b/processed_dataset/calculation/0411.json new file mode 100644 index 0000000000000000000000000000000000000000..98c85271b3df2f3722adcf4aea2c5ada8bfe6f74 --- /dev/null +++ b/processed_dataset/calculation/0411.json @@ -0,0 +1,8 @@ +{ + "source_file": "./raw_volume-zh/volume14/exercise1.tex", + "problem_type": "calculation", + "problem": "问题8. 设 $x_1, x_2, \\cdots, x_n$ 是整数, 并且满足:\n(1) $-1 \\leqslant x_i \\leqslant 2, i=1,2, \\cdots, n$;\n(2) $x_1+x_2+\\cdots+x_n=19$;\n(3) $x_1^2+x_2^2+\\cdots+x_n^2=99$.\n求 $x_1^3+x_2^3+\\cdots+x_n^3$ 的最大值和最小值.", + "solution": "设 $x_1, x_2, \\cdots, x_n$ 中有 $r$ 个- $1, s$ 个 $1, t$ 个 2 , 由题设得\n$$\n\\left\\{\\begin{array}{l}\n-r+s+2 t=19 \\\\\nr+s+4 t=99\n\\end{array}\\right.\n$$\n可得\n$$\n\\left\\{\\begin{array}{l}\nr=40-t \\\\\ns=59-3 t\n\\end{array}\\right.\n$$\n所以\n$$\n\\left\\{\\begin{array}{l}\nr=40-t \\geqslant 0, \\\\\ns=59-3 t \\geqslant 0, \\\\\nt \\geqslant 0\n\\end{array}\\right.\n$$\n故\n$$\n0 \\leqslant t \\leqslant 19 \\text {. }\n$$\n$$\nx_1^3+x_2^3+\\cdots+x_n^3=--r+s+8 t=(19-2 t)+8 t=6 t+19,\n$$\n所以\n$$\n19 \\leqslant x_1^3+x_2^3+\\cdots+x_n^3 \\leqslant 133 .\n$$\n又当 $r=40, s=59, t=0$ 时, $x_1^3+x_2^3+\\cdots+x_n^3=19$; 当 $r=21, s= 2, t=19$ 时, $x_1^3+x_2^3+\\cdots+x_n^3=133$, 所以 $x_1^3+x_2^3+\\cdots+x_n^3$ 的最小值为 19 , 最大值为 133 .", + "remark": "", + "figures": [] +} \ No newline at end of file diff --git a/processed_dataset/calculation/0412.json b/processed_dataset/calculation/0412.json new file mode 100644 index 0000000000000000000000000000000000000000..c5950276766fa16d7ff56fe5c5c7099952da735f --- /dev/null +++ b/processed_dataset/calculation/0412.json @@ -0,0 +1,8 @@ +{ + "source_file": "./raw_volume-zh/volume14/exercise1.tex", + "problem_type": "calculation", + "problem": "问题9. 设 $x, y$ 为非负整数, 使得 $x+2 y$ 是 5 的倍数, $x+y$ 是 3 的倍数, 且 $2 x+ y \\geqslant 99$, 求 $7 x+5 y$ 的最小值.", + "solution": "设 $x+2 y=5 a, x+y=3 b$, 则 $x=6 b-5 a, y=5 a-3 b$. 于是 $2 x+ y=9 b-5 a, 7 x+5 y=27 b-10 a$. 故\n$$\n\\left\\{\\begin{array}{l}\n6 b-5 a \\geqslant 0, \\\\\n5 a-3 b \\geqslant 0, \\\\\n9 b-5 a \\geqslant 99,\n\\end{array}\\right.\n$$\n所以 $9 b \\geqslant 5 a+99 \\geqslant 3 b+99$, 可得 $b \\geqslant 17$, 于是 $5 a \\geqslant 3 b \\geqslant 51$, 可得 $a \\geqslant 11$, 进而 $9 b \\geqslant 5 a+99 \\geqslant 3 \\cdot 11+99$, 可得 $b \\geqslant 18$.\n若 $b=18$, 则 $5 a \\leqslant 9 b-99=63, a \\leqslant 12$, 从而\n$$\n7 x+5 y=27 b-10 a \\geqslant 27 \\times 18-10 \\times 12=366 .\n$$\n若 $b>18$, 则 $7 x+5 y=27 b-10 a=9 b+2(9 b-5 a) \\geqslant 9 \\times 19+2 \\times 99=369$.\n综上所述, $7 x+5 y$ 的最小值为 366 .", + "remark": "", + "figures": [] +} \ No newline at end of file diff --git a/processed_dataset/calculation/0413.json b/processed_dataset/calculation/0413.json new file mode 100644 index 0000000000000000000000000000000000000000..bcf12530cec6eb784a05d47d43b789bfa5d3421d --- /dev/null +++ b/processed_dataset/calculation/0413.json @@ -0,0 +1,8 @@ +{ + "source_file": "./raw_volume-zh/volume14/exercise1.tex", + "problem_type": "calculation", + "problem": "问题10. 设 $x 、 y 、 z$ 是正实数, 且满足 $x y z+x+z=y$, 求\n$$\np=\\frac{2}{x^2+1}-\\frac{2}{y^2+1}+\\frac{3}{z^2+1}\n$$\n的最大值.", + "solution": "由已知条件得 $x+z=(1-x z) y$, 显然, $1-x z \\neq 0$, 所以 $y= \\frac{x+z}{1-x z}$. 由此联想到正切和公式, 于是令\n$$\n\\alpha=\\arctan x, \\beta=\\arctan y, \\gamma=\\arctan z, \\alpha, \\beta, \\gamma \\in\\left(0, \\frac{\\pi}{2}\\right),\n$$\n则\n$$\n\\tan \\beta=\\frac{\\tan \\alpha+\\tan \\gamma}{1-\\tan \\alpha \\tan \\gamma}=\\tan (\\alpha+\\gamma) .\n$$\n由于 $\\beta, \\alpha+\\beta \\in(0, \\pi)$, 所以 $\\beta=\\alpha+\\gamma$. 于是\n$$\n\\begin{aligned}\np & =\\frac{2}{\\tan ^2 \\alpha+1}-\\frac{2}{\\tan ^2 \\beta+1}+\\frac{3}{\\tan ^2 \\gamma+1} \\\\\n& =2 \\cos ^2 \\alpha-2 \\cos ^2(\\alpha+\\gamma)+3 \\cos ^2 \\gamma \\\\\n& =(\\cos 2 \\alpha+1)-[\\cos (2 \\alpha+2 \\gamma)+1]+3 \\cos ^2 \\gamma \\\\\n& =2 \\sin \\gamma \\cdot \\sin (2 \\alpha+\\gamma)+3 \\cos ^2 \\gamma \\\\\n& \\leqslant 2 \\sin \\gamma+3\\left(1-\\sin ^2 \\gamma\\right) \\\\\n& =-3\\left(\\sin \\gamma-\\frac{1}{3}\\right)^2+\\frac{10}{3} \\leqslant \\frac{10}{3}\n\\end{aligned}\n$$\n等号在 $2 \\alpha+\\gamma==\\frac{\\pi}{2}, \\sin \\gamma=\\frac{1}{3}$, 即 $a=\\frac{\\sqrt{2}}{2}, b=\\sqrt{2}, c=\\frac{\\sqrt{2}}{4}$ 时成立, 故欲求的最大值为 $\\frac{10}{3}$.", + "remark": "", + "figures": [] +} \ No newline at end of file diff --git a/processed_dataset/calculation/0414.json b/processed_dataset/calculation/0414.json new file mode 100644 index 0000000000000000000000000000000000000000..5dfdcc7055ed617b7f9c82659f1f0281dc09d86f --- /dev/null +++ b/processed_dataset/calculation/0414.json @@ -0,0 +1,8 @@ +{ + "source_file": "./raw_volume-zh/volume14/exercise10.tex", + "problem_type": "calculation", + "problem": "问题1. 小明家电话号码原为六位数,第一次升位是在首位号码和第二位号码之间加上数字 8 , 成为一个七位数的电话号码; 第二次升位是在首位号码前加上数字 2 , 成为一个八位数的电话号码.\n小明发现, 他家两次升位后的电话号码的八位数, 恰是原来电话号码的六位数的 81 倍, 问: 小明家原来的电话号码是多少?", + "solution": "设原来电话号码的六位数为 $\\overline{a b c} \\overline{d e f}$, 则经过两次升位后电话号码的八位数为 $\\overline{2 a 8 b c d e f}$. 根据题意, 有 $81 \\times \\overline{a b c d e f}=\\overline{2 a 8 b c d e f}$.\n记 $x=b \\times 10^4+c \\times 10^3+d \\times 10^2+e \\times 10+f$, 于是\n$$\n81 \\times a \\times 10^5+81 x=208 \\times 10^5+a \\times 10^6+x,\n$$\n解得 $x=1250 \\times(208-71 a)$.\n因为 $0 \\leqslant x \\leqslant 10^5$, 所以 $0 \\leqslant 1250 \\times(208-71 a)<10^5$, 故 $\\frac{128}{71}), $p(5,5)$ 等于从 $A$ 到 $B$ 不能穿过对角线的路径数, 即 $p(5,5)=42$.\n故所求的概率为 $\\frac{42}{252}=\\frac{1}{6}$.", + "remark": "", + "figures": [ + "./images/volume14/figures/fig-c11a2.png" + ] +} \ No newline at end of file diff --git a/processed_dataset/calculation/0419.json b/processed_dataset/calculation/0419.json new file mode 100644 index 0000000000000000000000000000000000000000..dfc9f82f9fcfdb13fa011a15e081a64b54e691ac --- /dev/null +++ b/processed_dataset/calculation/0419.json @@ -0,0 +1,8 @@ +{ + "source_file": "./raw_volume-zh/volume14/exercise11.tex", + "problem_type": "calculation", + "problem": "问题3. 在关于 $x$ 的二次方程 $x^2+z_1 x+z_2+m=0$ 中, $z_1, z_2, m$ 均是复数, 且 $z_1^2-4 z_2=16+20 \\mathrm{i}$. 设这个方程的两根 $\\alpha, \\beta$ 满足 $|\\alpha-\\beta|=2 \\sqrt{7}$, 求 $|m|$ 的最大值和最小值.", + "solution": "由韦达定理有 $\\left\\{\\begin{array}{l}\\alpha+\\beta=-z_1, \\\\ \\alpha \\beta=z_2+m,\\end{array}\\right.$ 故\n$$\n(\\alpha-\\beta)^2=(\\alpha+\\beta)^2-4 \\alpha \\beta=z_1^2-4 z_2-4 m,\n$$\n所以 $\\left|(\\alpha-\\beta)^2\\right|=\\left|z_1^2-4 z_2-4 m\\right|=|16+20 \\mathrm{i}-4 m|=4|m-(4+5 \\mathrm{i})|$.\n另一方面, $\\left|(\\alpha-\\beta)^2\\right|=|\\alpha-\\beta|^2=28$, 故 $|m-(4+5 \\mathrm{i})|=7$, 即 $m$ 在以 $A(4,5)$ 为圆心, 以 7 为半径的圆上.\n因为 $|O A|=\\sqrt{4^2+5^2}=\\sqrt{41}<7$, 故原点 $O$ 在上述圆内, 连接 $O A$ 延长交上述圆于 $B$, 延长 $A O$ 交上述圆于 $C$, 则\n$$\n|m|_{\\max }=|O B|=\\sqrt{41}+7,|m|_{\\min }=|O C|=7-\\sqrt{41} .\n$$", + "remark": "", + "figures": [] +} \ No newline at end of file diff --git a/processed_dataset/calculation/0420.json b/processed_dataset/calculation/0420.json new file mode 100644 index 0000000000000000000000000000000000000000..ada4383443b6ff1ff6c94ff08aa61d7948fd964a --- /dev/null +++ b/processed_dataset/calculation/0420.json @@ -0,0 +1,8 @@ +{ + "source_file": "./raw_volume-zh/volume14/exercise11.tex", + "problem_type": "calculation", + "problem": "问题5. 正实数 $x, y, z$ 满足\n$$\n\\left\\{\\begin{array}{l}\n\\frac{1}{3} y^2+z^2=9, \\label{eq1}\\\\\nx^2+x z+z^2=16, \\label{eq2}\\\\\nx^2+x y+\\frac{1}{3} y^2=25, \\label{eq3}\n\\end{array}\\right.\n$$\n试求代数式 $x y+2 y z+3 z x$ 的值.", + "solution": "构造 $\\triangle A B C$, 使 $A B=5, A C=4, B C=3$, 显然 $\\angle A C B=90^{\\circ}$.\n以 $A C$ 为一边向 $\\triangle A B C$ 外作正三角形, 再作该三角形的外接圆, 与以 $B C$ 为直径的圆交于 $C, O$ 两点, 连接 $O A, O B, O C$, 则由平面几何知识得\n$$\n\\angle B O C=90^{\\circ}, \\angle A O C=120^{\\circ}, \\angle A O B=150^{\\circ} .\n$$\n设 $A O=x, B O=\\frac{y}{\\sqrt{3}}, C O=z$, 由 $S_{\\triangle A O B}+S_{\\triangle B O C}+S_{\\triangle C O A}=S_{\\triangle A B C}$ 得 $\\frac{1}{2} x \\cdot \\frac{y}{\\sqrt{3}} \\sin 150^{\\circ}+\\frac{1}{2} z \\cdot \\frac{y}{\\sqrt{3}}+\\frac{1}{2} x \\cdot z \\cdot \\sin 120^{\\circ}=\\frac{1}{2} \\times 3 \\times 4$, 化简得\n$$\nx y+2 y z+3 x z=24 \\sqrt{3} .\n$$", + "remark": "", + "figures": [] +} \ No newline at end of file diff --git a/processed_dataset/calculation/0421.json b/processed_dataset/calculation/0421.json new file mode 100644 index 0000000000000000000000000000000000000000..3e7eaf1e06fc2e145982b053cfa0e88f43c67b90 --- /dev/null +++ b/processed_dataset/calculation/0421.json @@ -0,0 +1,10 @@ +{ + "source_file": "./raw_volume-zh/volume14/exercise11.tex", + "problem_type": "calculation", + "problem": "问题6. 给定实数 $\\alpha$, 求最小实数 $\\lambda=\\lambda(\\alpha)$, 使得对任意复数 $z_1, z_2$ 和实数 $x \\in[0$, $1]$, 若 $\\left|z_1\\right| \\leqslant \\alpha\\left|z_1-z_2\\right|$, 则 $\\left|z_1-x z_2\\right| \\leqslant \\lambda\\left|z_1-z_2\\right|$.", + "solution": "如图(), 在复平面内, 点 $A, B, C$ 对应的复数分别为 $z_1, z_2, x z_2$. 显然, 点 $C$ 在线段 $O B$ 上.\n向量 $\\overrightarrow{B A}$ 对应的复数为 $z_1-z_2$. 向量 $\\overrightarrow{C A}$ 对应的复数为 $z_1-x z_2$. 由 $\\left|z_1\\right| \\leqslant \\alpha\\left|z_1-z_2\\right|$, 得 $|\\overrightarrow{O A}| \\leqslant \\alpha|\\overrightarrow{B A}|$. 于是,\n$$\n\\begin{aligned}\n\\left|z_1-x z_2\\right|_{\\max } & =|\\overrightarrow{A C}|_{\\max }=\\max \\{|\\overrightarrow{O A}|,|\\overrightarrow{B A}|\\} \\\\\n& =\\max \\left\\{\\left|z_1\\right|,\\left|z_1-z_2\\right|\\right\\} \\\\\n& =\\max \\left\\{\\alpha\\left|z_1-z_2\\right|,\\left|z_1-z_2\\right|\\right\\},\n\\end{aligned}\n$$\n故 $\\lambda(\\alpha)=\\max \\{\\alpha, 1\\}$.", + "remark": "", + "figures": [ + "./images/volume14/figures/fig-c11a6.png" + ] +} \ No newline at end of file diff --git a/processed_dataset/calculation/0422.json b/processed_dataset/calculation/0422.json new file mode 100644 index 0000000000000000000000000000000000000000..809aec9ead12fdd2b805d5656c2556d0775cb630 --- /dev/null +++ b/processed_dataset/calculation/0422.json @@ -0,0 +1,8 @@ +{ + "source_file": "./raw_volume-zh/volume14/exercise12.tex", + "problem_type": "calculation", + "problem": "问题1. 在一个圆周上如此选定 $n$ 个点,使得它们两两之间连以弦之后, 任何三条弦之间除端点外不交于同一个点, 问: 这时圆内一共有多少个交点?", + "solution": "圆上每 4 个点构成一个凸四边形, 它的两条对角线 (弦) 交于一点, 因此每 4 点组成的集合对应一个交点, 由于没有三条弦交于一点,所以不同的 4 个点对应于不同的交点.\n反之, 设点 $P$ 是弦 $A_1 A_3$ 与 $A_2 A_4$ 的交点, 那么 $P$ 是 4 点集 $\\left\\{A_1, A_2, A_3, A_4\\right\\}$ 对应的点.\n从而交点个数就是这 $n$ 个点中取 4 点的取法总数 $\\mathrm{C}_n^4$.", + "remark": "", + "figures": [] +} \ No newline at end of file diff --git a/processed_dataset/calculation/0423.json b/processed_dataset/calculation/0423.json new file mode 100644 index 0000000000000000000000000000000000000000..615f3657ea6003238ff4df94fe7d724337d5d5e7 --- /dev/null +++ b/processed_dataset/calculation/0423.json @@ -0,0 +1,8 @@ +{ + "source_file": "./raw_volume-zh/volume14/exercise12.tex", + "problem_type": "calculation", + "problem": "问题2. 对于集合 $\\{1,2, \\cdots, n\\}\\left(n \\in \\mathbf{N}^*\\right)$ 和它的每个非空子集,我们定义\"交替和\" 如下: 把集合中的数按从大到小顺序排列, 然后从最大的数开始交替地加减各数(例如 $\\{1,2,4,6,9\\}$ 的交替和是 $9-6+4-2+1=6$, 而 $\\{5\\}$ 的交替和就是 5$)$. 求所有这些交替和的总和.", + "solution": "记集合 $A \\subseteq\\{1,2, \\cdots, n\\}$ 的交替和为 $f(A)$. 约定 $f(\\varnothing)=0$, 不影响结果.\n对每个 $X \\subseteq\\{1,2, \\cdots, n-1\\}$, 设 $Y=X \\cup\\{n\\}=\\left\\{a_1, a_2, \\cdots, a_k\\right\\}$ 是 $\\{1,2, \\cdots, n\\}$ 中含有元素 $n$ 的子集, 其中 $n=a_1>a_2>\\cdots>a_k \\geqslant 1, k \\in \\mathbf{N}^*$. 则对集组 $(X, Y)$ 有\n$$\n\\begin{aligned}\nf(Y)+f(X) & =\\sum_{i=1}^k(-1)^{i-1} a_i+\\sum_{j=2}^k(-1)^j a_j \\\\\n& =n+\\sum_{i=2}^k(-1)^{i-1} a_i-\\sum_{j=2}^k(-1)^{j-1} a_j=n .\n\\end{aligned}\n$$\n由于 $\\{1,2, \\cdots, n\\}$ 的所有子集可以分成 $2^{n-1}$ 个这样的集组 $(X, Y)$, 故所有这些交替和的总和为 $n \\cdot 2^{n-1}$.", + "remark": "", + "figures": [] +} \ No newline at end of file diff --git a/processed_dataset/calculation/0424.json b/processed_dataset/calculation/0424.json new file mode 100644 index 0000000000000000000000000000000000000000..2807df111aeb491f6488ecd6fa2d99dfdcbe3f17 --- /dev/null +++ b/processed_dataset/calculation/0424.json @@ -0,0 +1,8 @@ +{ + "source_file": "./raw_volume-zh/volume14/exercise12.tex", + "problem_type": "calculation", + "problem": "问题3. 已知正整数 $r 、 n$ 满足.\n$1 \\leqslant r \\leqslant n$, 求 $\\{1,2, \\cdots, n\\}$ 的一切 $r$ 元子集中最小数的算术平均值 $f(r, n)$.", + "solution": "$\\{1,2, \\cdots, n\\}$ 共 $\\mathrm{C}_n^r$ 个 $r$ 元子集, 下求这 $\\mathrm{C}_n^r$ 个子集中最小数的总和.\n对于 $P=\\{1,2, \\cdots, n\\}$ 的 $r$ 元子集 $A$, 设其最小数为 $a$, 作 $n+1$ 元集 $Q= \\{0,1, \\cdots, n\\}$ 的 $r+1$ 元子集 $A \\cup\\{a-1\\}, A \\cup\\{a-2\\}, \\cdots, A \\cup\\{0\\}$ 与之对应, 这样的 $r+1$ 元子集的个数恰好等于 $A$ 的最小数 $a$. 又显然不同的 $A$ 对应 $Q$ 中不同的子集, 因此, 所有 $P$ 的 $r$ 元子集的最小数之和即为 $Q$ 的 $r+1$ 元子集的总个数 $\\mathrm{C}_{n+1}^{+1}$.\n所以, $f(r, n)=\\frac{\\mathrm{C}_{n+1}^{r+1}}{\\mathrm{C}_n^r}=\\frac{n+1}{r+1}$.", + "remark": "", + "figures": [] +} \ No newline at end of file diff --git a/processed_dataset/calculation/0425.json b/processed_dataset/calculation/0425.json new file mode 100644 index 0000000000000000000000000000000000000000..afe401b7d4ebce20c4b15b93c755a15ea50d9dc1 --- /dev/null +++ b/processed_dataset/calculation/0425.json @@ -0,0 +1,8 @@ +{ + "source_file": "./raw_volume-zh/volume14/exercise12.tex", + "problem_type": "calculation", + "problem": "问题4. 设 $A$ 是 $X$ 的子集.\n若 $A$ 中所有数的和为奇数, 则称 $A$ 为 $X$ 的奇子集.\n若 $A$ 中所有数的和为偶数,则称 $A$ 为 $X$ 的偶子集.\n(1) 对 $n \\in \\mathbf{N}^*$ 求 $\\{1,2, \\cdots, n\\}$ 的奇子集的个数与偶子集的个数;\n(2) 对 $n \\in \\mathbf{N}^*, n \\geqslant 3$, 求 $\\{1,2, \\cdots, n\\}$ 的所有奇子集的元素和的总和.", + "solution": "(1) 设 $A$ 是 $\\{1,2, \\cdots, n\\}$ 的奇子集.\n考虑映射 $f$ :\n$$\n\\left\\{\\begin{array}{l}\nA \\mapsto A-\\{1\\}, \\text { 若 } 1 \\in A, \\\\\nA \\mapsto A \\cup\\{1\\}, \\text { 若 } 1 \\notin A .\n\\end{array}\\right.\n$$\n显然 $f$ 是将奇子集映为偶子集的映射.\n$f$ 是单射, 即对不同的 $A, f(A)$ 不同.\n$f$ 是满射, 即对每一个偶子集 $B$, 都有一个 $A$, 满足 $f(A)=B$. 事实上, 当 $1 \\in B$ 时, 令 $A=B-\\{1\\}$; 当 $1 \\notin B$ 时, 令 $A=B \\cup\\{1\\}$, 则 $f(A)=B$.\n于是 $f$ 是从奇子集族到偶子集族的一一对应.\n从而 $\\{1,2, \\cdots, n\\}$ 的奇子集与偶子集个数相等, 都等于 $\\frac{1}{2} \\times 2^n=2^{n-1}$.\n(2) 若集合族 $P$ 含有有限个数集, 且每个数集的元素个数有限, 则将所有这些数集的元素和的总和 $\\sum_{A \\in P} \\sum_{x \\in A} x$ 称为\"相应于 $P$ 的和\", 记为 $m(P)$.\n对集合 $X=\\{1,2, \\cdots, n\\}(n \\geqslant 3)$, 将其含有元素 $n, n-1$ 的奇、偶子集全体分别记为 $S_1 、 T_1$; 含有 $n$ 但不含 $n-1$ 的奇、偶子集全体分别记为 $S_2 、 T_2$; 不含 $n$ 但含有 $n-1$ 的奇、偶子集全体分别记为 $S_3 、 T_3$; 不含 $n, n-1$ 的奇、偶子集全体分别记为 $S_4 、 T_4$. 作为 (1) 的推论, $S_i$ 与 $T_i$ 中的集合个数都等于 $2^{(n-2)-1}=2^{n-3}, i=1,2,3,4$.\n我们可将 $S_1$ 中每个集合去掉元素 $n, n-1$, 对应于 $T_4$ 中的一个集合, 因此考虑相应于 $S_1$ 的和及相应于 $T_4$ 的和, 有\n$$\nm\\left(S_1\\right)=m\\left(T_4\\right)+2^{n-3}(2 n-1),\n$$\n同理可得\n$$\n\\begin{gathered}\nm\\left(S_4\\right)=m\\left(T_1\\right)-2^{n-3}(2 n-1), \\\\\nm\\left(S_2\\right)=m\\left(T_3\\right)+2^{n-3}, \\\\\nm\\left(S_3\\right)=m\\left(T_2\\right)-2^{n-3} .\n\\end{gathered}\n$$\n上述 4 式相加可知, 相应于 $X$ 的所有奇子集的和 $m\\left(S_1 \\cup S_2 \\cup S_3 \\cup S_4\\right)$ 与相应于 $X$ 的所有偶子集的和 $m\\left(T_1 \\cup T_2 \\cup T_3 \\cup T_4\\right)$ 相等.\n注意到\n$$\nm(X)=2^{n-1} \\times(1+2+\\cdots+n)=2^{n-2} n(n+1)\n$$\n(因为每个 $i \\in X$ 均在 $2^{n-1}$ 个子集中出现), 故所求奇子集元素和的总和为 $m(X)$ 的一半, 即 $2^{n-3} n(n+1)$.", + "remark": "", + "figures": [] +} \ No newline at end of file diff --git a/processed_dataset/calculation/0426.json b/processed_dataset/calculation/0426.json new file mode 100644 index 0000000000000000000000000000000000000000..dab434dac7c787ba0e6a60fb43c5b0ee7955d5f1 --- /dev/null +++ b/processed_dataset/calculation/0426.json @@ -0,0 +1,8 @@ +{ + "source_file": "./raw_volume-zh/volume14/exercise12.tex", + "problem_type": "calculation", + "problem": "问题5. 设 $n \\equiv 1(\\bmod 4)$ 且 $n>1, P=\\left(a_1, a_2, \\cdots, a_n\\right)$ 是 $(1,2, \\cdots, n)$ 的任意排列, $k_P$ 表示使不等式 $a_1+a_2+\\cdots+a_k)中给出的是一个 $4 \\times 5$ 的矩形, 且没有搁浅的黑格.\n求没有搁浅的黑格的 $2 \\times n$ 的矩形的数目.", + "solution": "记有搁浅黑格的 $2 \\times n$ 矩形的数目为 $a_n$, 无搁浅黑格的 $2 \\times n$ 矩形的数目为 $b_n$, 则 $b_n=2^{2 n}-a_n$, 且 $a_1=0$.\n由题意知, 搁浅的黑格只可能位于第 2 行.\n(1) 若第 2 行前 $n-1$ 个方格中已有搁浅的黑格, 则第 $n$ 列无论怎样染色, 均使 $2 \\times n$ 矩形中存在搁浅的黑格, 此时共有 $2^2 a_{n-1}=4 a_{n-1}$ 种情况.\n(2) 若第 2 行前 $n-1$ 个方格中无搁浅的黑格,则第 2 行第 $n$ 列的方格必为 $2 \\times n$ 矩形中唯一的挌浅的黑格,所以它上面的方格确定为白格, 且第 2 行前 $n-1$ 个方格中存在白格, 而在第 2 行前 $n-1$ 个方格均为黑格的情况下,第一行任意一种染色方法均可使前 $n-1$ 列无搁浅的黑格, 所以这里 $2^{n-1}$ 种情况需要排除,故共有 $b_{n-1}-2^{n-1}=2^{2(n-1)}-a_{n-1}-2^{n-1}$ 种情况.\n$$\n\\text { 综合 (1)、(2) 知 } a_n=4 a_{n-1}+2^{2(n-1)}-a_{n-1}-2^{n-1}=3 a_{n-1}+4^{n-1}-2^{n-1} \\text {. }\n$$\n即\n$$\n\\frac{a_n}{3^{n-1}}=\\frac{a_{n-1}}{3^{n-2}}+\\left(\\frac{4}{3}\\right)^{n-1}-\\left(\\frac{2}{3}\\right)^{n-1}\n$$\n将该式中的 $n$ 分别代换成 $2,3, \\cdots, n$ 并累加, 注意 $a_1=0$, 有\n$$\n\\begin{aligned}\n\\frac{a_n}{3^{n-1}} & =\\sum_{k=2}^n\\left(\\frac{4}{3}\\right)^{n-1}-\\sum_{k=2}^n\\left(\\frac{2}{3}\\right)^{n-1} \\\\\n& =4 \\times\\left(\\left(\\frac{4}{3}\\right)^{n-1}-1\\right)-2 \\times\\left(1-\\left(\\frac{2}{3}\\right)^{n-1}\\right)=\\frac{4^n+2^n}{3^{n-1}}-6,\n\\end{aligned}\n$$\n即 $a_n=4^n+2^n-6 \\times 3^{n-1}=4^n+2^n-2 \\times 3^n$.\n从而 $b_n=2^{2 n}-a_n=2 \\times 3^n-2^n$.", + "remark": "", + "figures": [ + "./images/volume14/figures/fig-c13p7.png" + ] +} \ No newline at end of file diff --git a/processed_dataset/calculation/0432.json b/processed_dataset/calculation/0432.json new file mode 100644 index 0000000000000000000000000000000000000000..ef1e1d16644e269c3e822bf797cca2e9eb0c5850 --- /dev/null +++ b/processed_dataset/calculation/0432.json @@ -0,0 +1,8 @@ +{ + "source_file": "./raw_volume-zh/volume14/exercise14.tex", + "problem_type": "calculation", + "problem": "问题1. 一张 $8 \\times 8$ 的方格表切去对角两个方格, 这个缺角的方格表能否用 31 张 $2 \\times 1$ 的骨牌覆盖?", + "solution": "答案是不能把这个残缺的方格表盖住.\n事实上, 把 $8 \\times 8$ 的方格表按国际象棋棋盘黑白染色, 每张骨牌盖住一个黑格和一个白格.\n若有能够盖住棋盘的方法, 他们将盖住 31 个黑格和 31 个白格.\n但缺角方格表上 30 个格子是一种颜色的, 32 个格子是另一种颜色的.", + "remark": "", + "figures": [] +} \ No newline at end of file diff --git a/processed_dataset/calculation/0433.json b/processed_dataset/calculation/0433.json new file mode 100644 index 0000000000000000000000000000000000000000..1618955a6c6e7707ac738ffc1517609fe21b099e --- /dev/null +++ b/processed_dataset/calculation/0433.json @@ -0,0 +1,11 @@ +{ + "source_file": "./raw_volume-zh/volume14/exercise14.tex", + "problem_type": "calculation", + "problem": "问题2. 如图(),把正方体形的房子分割成 27 个相等的小房间,每相邻两个房间 (即有公共面的两个小正方体) 都有门相通, 在中心的那个小正方体中有一只甲虫, 甲虫能从每个小房间走到与它相邻的任何一个小房间去.\n如果要求甲虫只能走到每个小房间一次,那么甲虫能走遍所有的小房间吗?", + "solution": "甲虫不能走遍所有的小房间.\n我们如图(), 将正方体分割成 27 个小正方体 (每个小正方体表示一间房间), 涂上黑白相间的两种颜色, 使得中心的小正方体染成白色,再使两个相邻的小正方体染上不同的颜色.\n显然, 在 27 个小正方体中, 14 个是黑的, 13 个是白的.\n甲虫从中间的白色小正方体出发, 每走一步, 方格就改变一种颜色.\n故它走 27 步, 应该经过 14 个白色的小正方体、13 个黑色的小正方体.\n因此在 27 步中至少有一个小正方体, 甲虫进去过两次.\n由此可见, 如果要求甲虫到每一个小房间只去一次, 那么甲虫不能走遍所有的小房间.", + "remark": "", + "figures": [ + "./images/volume14/figures/fig-c14p2.png", + "./images/volume14/figures/fig-c14a2.png" + ] +} \ No newline at end of file diff --git a/processed_dataset/calculation/0434.json b/processed_dataset/calculation/0434.json new file mode 100644 index 0000000000000000000000000000000000000000..02248ec4ba4c344f66cd190c3f7f29a71b0bc1c1 --- /dev/null +++ b/processed_dataset/calculation/0434.json @@ -0,0 +1,11 @@ +{ + "source_file": "./raw_volume-zh/volume14/exercise14.tex", + "problem_type": "calculation", + "problem": "问题8. 在 $8 \\times 8$ 的棋盘的每个方格中任写一个正整数,然后施行以下操作: 任取一个 $3 \\times 3$ 或 $4 \\times 4$ 的正方形 \"子棋盘\", 将其中每个数都加 1 . 能否经过有限次操作,使棋盘中每个数字都是 10 的倍数?", + "solution": "如图(), 的方法将棋盘中的 24 个方格涂上颜色.\n假如一开始染色方格上的所有数之和 $S$ 为奇数 (这很容易做到), 那么由于每个 $3 \\times 3$ 或 $4 \\times 4$ \"子棋盘\" 恰好覆盖这 24 个方格中的偶数个, 因此每次操作必使 $S$ 增加一个偶数, 从而 $S$ 总为奇数,不可能使所有方格内都出现 10 的倍数.", + "remark": "注:本题的染色方法有很多种, 例如按如图(), 的染色方法可以证明更强的命题: 即使允许对任意一个 $2 \\times 2$ 的 \"子棋盘\"进行每个数都加 1 的操作,仍不能保证棋盘中的数都变成偶数.", + "figures": [ + "./images/volume14/figures/fig-c14a8-1.png", + "./images/volume14/figures/fig-c14a8-2.png" + ] +} \ No newline at end of file diff --git a/processed_dataset/calculation/0435.json b/processed_dataset/calculation/0435.json new file mode 100644 index 0000000000000000000000000000000000000000..216534de281fc7e6e6d8d266f21e207e091e7a61 --- /dev/null +++ b/processed_dataset/calculation/0435.json @@ -0,0 +1,11 @@ +{ + "source_file": "./raw_volume-zh/volume14/exercise14.tex", + "problem_type": "calculation", + "problem": "问题9. 如图(), $8 \\times 9$ 的方格表上已经放置了 6 块 $1 \\times 2$ 的多米诺骨牌.\n问方格表内最多还能放置多少块两两不重叠的多米诺骨牌?", + "solution": "如图(), 设方格表中已被多米诺骨牌占据的 12 个小方格的集合为 $C$; 右上角、 左下角的小方格分别为 $B 、 D ; B \\cup C \\cup D$ 左上方的小方格集合为 $A ; B \\cup C \\cup D$ 右下方的小方格集合为 $E$.\n将方格表黑白相间染色, 使右上角 $B$ 为白格, 则推算知: $D$ 为黑格; $A$ 中含有 16 个白格、13 个黑格; $E$ 中含有 16 个黑格、13 个白格.\n再放人一块多米诺骨牌必须全属于 $A \\cup B \\cup D$ 或全属于 $E \\cup B \\cup D$, 每一集合至多包含 14 块多米诺骨牌 (因为每块骨牌占据一个黑格和一个白格), 因此最多还能两两不重叠地放置不超过 28 块多米诺骨牌.\n易知可以如图所示放 28 块骨牌, 因此本题所求结果为 28 .", + "remark": "", + "figures": [ + "./images/volume14/figures/fig-c14p9.png", + "./images/volume14/figures/fig-c14a9.png" + ] +} \ No newline at end of file diff --git a/processed_dataset/calculation/0436.json b/processed_dataset/calculation/0436.json new file mode 100644 index 0000000000000000000000000000000000000000..0a2d3c1f6ddc5ba289d6e30f73282e25c6a0d3e8 --- /dev/null +++ b/processed_dataset/calculation/0436.json @@ -0,0 +1,8 @@ +{ + "source_file": "./raw_volume-zh/volume14/exercise15.tex", + "problem_type": "calculation", + "problem": "问题2. 已知 $\\triangle A B C$ 内有 $m$ 个点, 连同 $A, B, C$ 三点一共 $m+3$ 个点.\n以这些点为顶点将 $\\triangle A B C$ 分成若干个互不重叠的小三角形.\n将 $A, B, C$ 三点分别染成红色、黄色、蓝色.\n而三角形内的 $m$ 个点, 每个点任意染成红色、黄色、蓝色三色之一.\n问: 三个顶点颜色都不同的小三角形的个数是奇数还是偶数?", + "solution": "将每个小三角形的各条边按如下规则赋值: 若边的两端点同色, 赋值为 0 ; 若边的两端点异色, 赋值为 1 . 这样每个小三角形各边赋值之和, 有如下三种情况:\n(1) 三个顶点都不同色的小三角形,赋值和为 3 ;\n(2)只有两个顶点同色的小三角形,赋值和为 2;\n(3) 三个顶点都同色的小三角形,赋值和为 0 .\n另设所有小三角形的边赋值总和为 $S$, 上述 (1)、(2)、(3) 三类小三角形的个数分别为 $x, y, z$,于是有\n$$\nS=3 x+2 y+0 z=3 x+2 y . \\label{eq1}\n$$\n在计算所有小三角形边的赋值总和 $S$ 时, 除了 $\\triangle A B C$ 三边 $A B, B C$, $C A$ 外, 其余各边都被重复计算了两次, 设这些边的赋值和为 $k$, 则 $S=3+2 k$, 显然 $S$ 是一个奇数, 由 \\ref{eq1} 式知 $x$ 是一个奇数, 即三个顶点颜色都不同的三角形的个数是一个奇数.", + "remark": "", + "figures": [] +} \ No newline at end of file diff --git a/processed_dataset/calculation/0437.json b/processed_dataset/calculation/0437.json new file mode 100644 index 0000000000000000000000000000000000000000..4a59cec1d7dcbaf9e1cf1c818d7e3ee79a9d4d28 --- /dev/null +++ b/processed_dataset/calculation/0437.json @@ -0,0 +1,10 @@ +{ + "source_file": "./raw_volume-zh/volume14/exercise15.tex", + "problem_type": "calculation", + "problem": "问题4. 如图(), 是一个向右和向下无限的表格.\n一开始在左上角 $A$ 格内放一枚棋子, 此后每一步下棋规则如下: 若某格 $P$ 放有棋子, 且它的右边相邻两格 $Q$ 和 $R$ 都没有棋子, 则可将 $P$ 中的棋子去掉, 在 $Q 、 R$ 两格中各放一枚棋子; 同样若 $P$ 的下边相邻两格 $S$ 和 $T$ 都没有棋子, 则可将 $P$ 中的棋子去掉, 在 $S 、 T$ 两格中各放一枚棋子.\n问: 能否在有限步后让所有棋子都不出现在前 4 列中?", + "solution": "我们对第 $i$ 行第 $j$ 列的格子赋值 $\\lambda^{i+j}, i, j \\in \\mathbf{N}^*$, 其中 $\\lambda=\\frac{\\sqrt{5}-1}{2}$.\n由于 $\\lambda^{i+(j+1)}+\\lambda^{i+(j+2)}=\\lambda^{(i+1)+j}+\\lambda^{(i+2)+j}=\\lambda^{i+j}$, 故每下一步棋不改变所有棋子所在格的赋值之和, 记这个和为 $S$, 其中初始情况下的 $S=\\lambda^2$.\n假设下棋过程中某一时刻所有棋子都不出现在前 4 列, 那么此时\n$$\nS \\leqslant \\sum_{i=1}^{\\infty} \\sum_{j=5}^{\\infty} \\lambda^{i+j}=\\lambda^6\\left(\\sum_{i=0}^{\\infty} \\lambda^i\\right)^2=\\lambda^6\\left(\\frac{1}{1-\\lambda}\\right)^2=\\lambda^6\\left(\\frac{1}{\\lambda^2}\\right)^2=\\lambda^2,\n$$\n说明从第 5 列开始的所有格子都已被摆满,这是不可能的.\n因此,有限步后无法让所有棋子都不出现在前 4 列中.", + "remark": "", + "figures": [ + "./images/volume14/figures/fig-c15p4.png" + ] +} \ No newline at end of file diff --git a/processed_dataset/calculation/0438.json b/processed_dataset/calculation/0438.json new file mode 100644 index 0000000000000000000000000000000000000000..ebfa688252a1485730f6d4fb748b93b22c8ff6e1 --- /dev/null +++ b/processed_dataset/calculation/0438.json @@ -0,0 +1,10 @@ +{ + "source_file": "./raw_volume-zh/volume14/exercise15.tex", + "problem_type": "calculation", + "problem": "问题5. 如图(), 平面上由边长为 1 的正三角形构成一个 (无穷的)三角形网格.\n三角形的顶点称为格点, 距离为 1 的格点称为相邻格点.\n$A 、 B$ 两只青蛙进行跳跃游戏.\n\"一次跳跃\" 是指青蛙从所在的格点跳至相邻的格点.\n\" $A 、 B$ 的一轮跳跃\" 是指它们按下列规则进行的先 $A$ 后 $B$ 的跳跃:\n规则(1): $A$ 任意跳一次,则 $B$ 沿与 $A$ 相同的跳跃方向跳跃一次,或沿与之相反的方向跳跃两次.\n规则 (2) : 当 $A 、 B$ 两所在的格点相邻时,它们可执行规则 (1)完成一轮跳跃, 也可以由 $A$ 连跳两次, 每次跳跃均保持与 $B$ 相邻, 而 $B$ 则留在原地不动.\n若 $A 、 B$ 的起始位置为两个相邻格点, 问: 能否经过有限轮跳跃, 使 $A$ 、 $B$ 恰好位于对方的起始位置上?", + "solution": "不可能.\n不妨设 $B$ 的起始位置在 $A$ 右方相邻格点上.\n现为每个格点赋值如下:\n先将 $A$ 初始所在格点赋值 1 , 以后赋值规则如下: 任意一个格点赋值为它左边相邻格点处乘以 $\\omega$ (其中 $\\omega=\\frac{-1+\\sqrt{3} i}{2}$ ), 又是它斜左下方与之相邻格点处值乘以 $\\omega^2$.\n开始时 $A=1, B=\\omega$, 而由规则知, 任意一轮跳跃不改变 $A 、 B$ 所在格点的值的比值.\n若最终 $A 、 B$ 能换位, 则 $\\frac{1}{\\omega}=\\frac{\\omega}{1}$, 矛盾!所以不能做到.", + "remark": "", + "figures": [ + "./images/volume14/figures/fig-c15p5.png" + ] +} \ No newline at end of file diff --git a/processed_dataset/calculation/0439.json b/processed_dataset/calculation/0439.json new file mode 100644 index 0000000000000000000000000000000000000000..bd563b6fafde5d9026182c7ccf401940b0a77477 --- /dev/null +++ b/processed_dataset/calculation/0439.json @@ -0,0 +1,8 @@ +{ + "source_file": "./raw_volume-zh/volume14/exercise17.tex", + "problem_type": "calculation", + "problem": "问题1. 设非负实数 $x_1,x_2,\\cdots,x_n$ 满足 $x_{1}+x_{2}+\\cdots+x_{n}\\leq{\\frac{1}{2}}$ ,求$(1-x_1)(1-x_2)\\cdots(1-x_{n})$ 的最小值", + "solution": "先固定 $x_1, x_2, \\cdots, x_{n-2}$, 注意到 $\\left(1-x_{n-1}\\right)\\left(1-x_n\\right)=1-x_{n-1}-x_n+ 222 x_{n-1} x_n$, 其中 $x_{n-1}+x_n$ 为定值, 所以 $x_{n-1} x_n$ 的值越小则原式值越小, 故令\n$$\nx_i^{\\prime}=x_i, i==1,2, \\cdots, n-2, x_{n-1}^{\\prime}=x_{n-1}+x_n, x_n^{\\prime}=0,\n$$\n此时 $x_{n-1}^{\\prime} x_n^{\\prime}=0 \\leqslant x_{n-1} x_n$, 所以\n$$\n\\left(1-x_1^{\\prime}\\right)\\left(1-x_2^{\\prime}\\right) \\cdots\\left(1-x_{n-1}^{\\prime}\\right) \\leqslant\\left(1-x_1\\right)\\left(1-x_2\\right) \\cdots\\left(1-x_n\\right),\n$$\n其中 $x_1^{\\prime}+x_2^{\\prime}+\\cdots+x_{n-1}^{\\prime}=x_1+x_2+\\cdots+x_n \\leqslant \\frac{1}{2}$.\n再进行 $n-2$ 次类似的调整过程可知\n$$\n\\left(1-x_1\\right)\\left(1-x_2\\right) \\cdots\\left(1-x_n\\right) \\geqslant 1-\\left(x_1+x_2+\\cdots+x_n\\right) \\geqslant \\frac{1}{2},\n$$\n等号当 $x_1=\\frac{1}{2}, x_2=x_3=\\cdots=x_n=0$ 时可取到, 所以所求最小值为 $\\frac{1}{2}$.", + "remark": "", + "figures": [] +} \ No newline at end of file diff --git a/processed_dataset/calculation/0440.json b/processed_dataset/calculation/0440.json new file mode 100644 index 0000000000000000000000000000000000000000..9b8543b0b6bea6ba45f872cea7663ca3a92823ea --- /dev/null +++ b/processed_dataset/calculation/0440.json @@ -0,0 +1,8 @@ +{ + "source_file": "./raw_volume-zh/volume14/exercise17.tex", + "problem_type": "calculation", + "problem": "问题2. 设 $x_1, x_2, \\cdots, x_{10}$ 是正整数,且满足\n$$\nx_1+x_2+\\cdots+x_{10}=49,\n$$\n求 $x_1^2+x_2^2+\\cdots+x_{10}^2$ 的最大值和最小值.", + "solution": "由于把 49 写成 10 个正整数的和, 写法只有有限种, 所以一定有一种使得 $x_1^2+x_2^2+\\cdots+x_{10}^2$ 达到最大值, 也一定有一种使得 $x_1^2+x_2^2+\\cdots+x_{10}^2$ 达到最小值.\n假设 $x_1 \\leqslant x_2 \\leqslant \\cdots \\leqslant x_{10}$ 满足 $x_1+x_2+\\cdots+x_{10}=49$, 且使得 $x_1^2+x_2^2+\\cdots +x_{10}^2$ 达到最大值, 若 $x_1>1$, 取 $y_1=x_1-1, y_2=x_2+1, y_k=x_k, k=3,4$,\n$\\cdots, 10$, 则\n$$\ny_1+y_2+\\cdots+y_{10}=49,\n$$\n且 $y_1^2+y_2^2=\\left(x_1-1\\right)^2+\\left(x_2+1\\right)^2=x_1^2+x_2^2+2\\left(x_2-x_1\\right)+2>x_1^2+x_2^2$, 从而 $y_1^2+y_2^2+\\cdots+y_{10}^2>x_1^2+x_2^2+\\cdots+x_{10}^2$, 矛盾.\n所以 $x_1=1$, 进而 $x_2=1, \\cdots, x_9=1$, 所以 $x_{10}=40$. 于是 $x_1^2+ x_2^2+\\cdots+x_{10}^2$ 的最大值为 $9+40^2=1609$.\n假设 $x_1 \\leqslant x_2 \\leqslant \\cdots \\leqslant x_{10}$ 是满足 $x_1+x_2+\\cdots+x_{10}=49$, 且使得 $x_1^2+ x_2^2+\\cdots+x_{10}^2$ 达到最小值, 则 $x_1, x_2, \\cdots, x_{10}$ 中任意两个数的差的绝对值不超过 1 .\n事实上, 若存在 $x_i, x_j, 1 \\leqslant i \\leqslant j \\leqslant 10$, 有 $x_j-x_i \\geqslant 2$, 令\n$$\ny_i=x_i+1, y_j=x_j-1, x_k=y_k, k \\neq i, j\n$$\n则 $y_i^2+y_j^2=\\left(x_i+1\\right)^2+\\left(x_j-1\\right)^2=x_i^2+x_j^2-2\\left(x_j-x_i\\right)+20\\right)$. 这时\n$$\n\\begin{gathered}\n2+3+4+\\cdots+63=2015, \\\\\n2015-2006=9 .\n\\end{gathered}\n$$\n所以,把 2006 分成\n$$\n(2+3+\\cdots+8)+(10+11+\\cdots+63),\n$$\n这一形式时, 这些数的乘积最大, 其积为\n$$\n2 \\times 3 \\times \\cdots \\times 8 \\times 10 \\times \\cdots \\times 63=\\frac{63 !}{9} .\n$$", + "remark": "", + "figures": [] +} \ No newline at end of file diff --git a/processed_dataset/calculation/0442.json b/processed_dataset/calculation/0442.json new file mode 100644 index 0000000000000000000000000000000000000000..a7f8750b0538d5d78f3ad16b653805262aa0bea2 --- /dev/null +++ b/processed_dataset/calculation/0442.json @@ -0,0 +1,8 @@ +{ + "source_file": "./raw_volume-zh/volume14/exercise17.tex", + "problem_type": "calculation", + "problem": "问题6. 空间中有 1989 个点, 其中任意三点不共线, 把它们分成点数互不相同的 30 组,在任何三个不同的组中各取一点为顶点作三角形,要使这种三角形的总数最大,各组的点数应为多少?", + "solution": "设这 1989 个点分成的 30 组中, 每组的点数分别为 $n_1, n_2, \\cdots, n_{30}$, 且不妨设 $n_1n_1 n_2,\n$$\n所以用 $n_1^{\\prime}, n_2^{\\prime}$ 代替 $n_1, n_2$ 时, $S$ 中的第一项 $n_1 n_2 \\sum_{k=3}^{30} n_k$ 变大, 而后面两项不变, 从而 $S$ 的值增大,矛盾.\n所以, $n_{i+1}-n_i \\leqslant 2(i=1,2, \\cdots, 29)$.\n(2) 欲使 $S$ 最大, $n_{i+1}-n_i=2$ 的 $i$ 值至多只有一个, 若不然, 设 $n_{i+1}- n_i=2, n_{j_0+1}-n_{j_0}=2$. 那么作如下调整: 用 $n_{i_0}^{\\prime}=n_{i_0}+1$ 及 $n_{j_0+1}^{\\prime}=n_{j_0+1}-1$ 代替 $n_{i_0} 、 n_{j_0+1}, S$ 便会增大.\n所以使得 $n_{i+1}-n_i=2$ 的下标 $i$ 最多只有一个.\n(3) 如果 $n_1, n_2, \\cdots, n_{30}$ 组成公差为 1 的等差数列, 那么\n$$\nn_1+n_2+\\cdots+n_{30}=15 \\times\\left(2 n_1+29\\right)=1989,\n$$\n而 15 不整除 1989 , 故上式不成立, 因此 $n_1, n_2, \\cdots, n_{30}$ 中, 必有两项的差为 2 . 现设各组点数为\n$$\nn_1, n_1+1, \\cdots, n_1+i_0-1, n_1+i_0+1, \\cdots, n_1+30 .\n$$\n其中 $1 \\leqslant i_0 \\leqslant 29$, 它们的总和为 $1989=\\left(2 n_1+30\\right) \\times 31-\\left(n_1+i_0\\right)$, 所以\n$$\n30 n_1-i_0=1524 \\text {. }\n$$\n易知 $n_1=51, i_0=6$. 所以, 当 30 组点数依次为 $51,52, \\cdots, 56,58$, $59, \\cdots, 81$ 时, $S$ 最大.", + "remark": "", + "figures": [] +} \ No newline at end of file diff --git a/processed_dataset/calculation/0443.json b/processed_dataset/calculation/0443.json new file mode 100644 index 0000000000000000000000000000000000000000..8e2fbaa0282538529df41ee9307dd5a8976175fb --- /dev/null +++ b/processed_dataset/calculation/0443.json @@ -0,0 +1,8 @@ +{ + "source_file": "./raw_volume-zh/volume14/exercise18.tex", + "problem_type": "calculation", + "problem": "问题2. 求方程组\n$$\n\\left\\{\\begin{array}{l}\nx^3+y^3+z^3=x+y+z, \\\\\nx^2+y^2+z^2=x y z\n\\end{array}\\right.\n$$\n的所有正实数解.", + "solution": "构造方程 $f(x, y, z)=\\left(x^3-x\\right)+\\left(y^3-y\\right)+\\left(z^3-z\\right)$, 则第一个方程等价于 $f(x, y, z)=0$.\n若 $x, y, z \\geqslant 1$, 则 $f(x, y, z) \\geqslant 0$ 当且仅当 $x=y=z=1$ 时等号成立.\n但若 $x=y=z=1$, 则不满足第二个方程.\n所以, 如果假设此方程组解存在, 则任意一组解中至少有一个未知数小于 1 , 不妨设 $x<1$, 则 $x^2+y^2+z^2>y^2+z^2 \\geqslant 2 y z>x y z$ 与已知矛盾, 因此原方程组没有正实数解.", + "remark": "", + "figures": [] +} \ No newline at end of file diff --git a/processed_dataset/calculation/0444.json b/processed_dataset/calculation/0444.json new file mode 100644 index 0000000000000000000000000000000000000000..452f64704bce69a0020dfdc4d099ae210aeadb31 --- /dev/null +++ b/processed_dataset/calculation/0444.json @@ -0,0 +1,10 @@ +{ + "source_file": "./raw_volume-zh/volume14/exercise18.tex", + "problem_type": "calculation", + "problem": "问题3. 设 $x \\in \\mathbf{R}$, 求函数 $f(x)=\\sqrt{x^2+1}+\\sqrt{(x-12)^2+16}$ 的最小值.", + "solution": "如图(), 取 $A$ 为数轴原点, $A B=12$, 再作 $A B$ 垂线 $A C, B D$,使 $A C=1, B D=4$,在数轴上取点 $P$,使 $A P=x$, 则 $f(x)=|C P|+|D P|$, 当 $C, P, D$ 共线时, $f$ 值最小, 此时 $f_{\\text {min }}=|C D|= |A E|=\\sqrt{12^2+5^2}=13$.", + "remark": "", + "figures": [ + "./images/volume14/figures/fig-c18a3.png" + ] +} \ No newline at end of file diff --git a/processed_dataset/calculation/0445.json b/processed_dataset/calculation/0445.json new file mode 100644 index 0000000000000000000000000000000000000000..2500ff4de2ce7d197a8651bdecfe056931457714 --- /dev/null +++ b/processed_dataset/calculation/0445.json @@ -0,0 +1,8 @@ +{ + "source_file": "./raw_volume-zh/volume14/exercise18.tex", + "problem_type": "calculation", + "problem": "问题4. 已知 $x, y \\in\\left[-\\frac{\\pi}{4}, \\frac{\\pi}{4}\\right], a \\in \\mathbf{R}$, 且满足\n$$\n\\left\\{\\begin{array}{l}\nx^3+\\sin x-2 a=0, \\\\\n4 y^3+\\frac{1}{2} \\sin 2 y+a=0,\n\\end{array}\\right.\n$$\n求 $\\cos (x+2 y)$ 的值.", + "solution": "由 $\\left\\{\\begin{array}{l}x^3+\\sin x-2 a=0, \\\\ 4 y^3+\\frac{1}{2} \\sin 2 y+a=0\\end{array}\\right.$ 可得 $\\left\\{\\begin{array}{l}x^3+\\sin x=2 a, \\\\ 8 y^3+\\sin 2 y=-2 a .\\end{array}\\right.$\n构造函数 $f(t)=t^3+\\sin t$, 则 $f(t)$ 为单调增函数, 因为 $f(x)= -f(2 y)=f(-2 y)$, 所以, $x+2 y=0$, 故 $\\cos (x+2 y)=1$.", + "remark": "", + "figures": [] +} \ No newline at end of file diff --git a/processed_dataset/calculation/0446.json b/processed_dataset/calculation/0446.json new file mode 100644 index 0000000000000000000000000000000000000000..f386a524d488db45339a7932fbbb28b38ac909ef --- /dev/null +++ b/processed_dataset/calculation/0446.json @@ -0,0 +1,8 @@ +{ + "source_file": "./raw_volume-zh/volume14/exercise18.tex", + "problem_type": "calculation", + "problem": "问题5. 实数 $\\alpha$ 与 $\\beta$ 满足 $\\alpha^3-3 \\alpha^2+5 \\alpha=1, \\beta^3-3 \\beta^2+5 \\beta=5$, 求 $\\alpha+\\beta$ 的值.", + "solution": "因为 $x^3-3 x^2+5 x-3=\\left(x^3-3 x^2+3 x-1\\right)+2(x-1)=(x- 1)^3+2(x-1)$, 构造函数 $f(t)=t^3+2 t$, 显然 $t$ 为奇函数.\n$$\n\\text { 由 }\\left\\{\\begin{array} { l } \n{ \\alpha ^ { 3 } - 3 \\alpha ^ { 2 } + 5 \\alpha = 1 } \\\\\n{ \\beta ^ { 3 } - 3 \\beta ^ { 2 } + 5 \\beta = 5 }\n\\end{array} \\Rightarrow \\left\\{\\begin{array}{l}\n\\alpha^3-3 \\alpha^2+5 \\alpha-3=-2, \\\\\n\\beta^3-3 \\beta^2+5 \\beta-3=2,\n\\end{array}\\right.\\right.\n$$\n所以\n$$\nf(\\alpha-1)=-f(\\beta-1)=f(1-\\beta) .\n$$\n由于 $f(x)$ 在 $\\mathbf{R}$ 上为单调增函数, 所以 $\\alpha-1=1-\\beta$, 故 $\\alpha+\\beta=2$.", + "remark": "", + "figures": [] +} \ No newline at end of file diff --git a/processed_dataset/calculation/0447.json b/processed_dataset/calculation/0447.json new file mode 100644 index 0000000000000000000000000000000000000000..a4e34309f27f93930fcb52416183319eb9c9af62 --- /dev/null +++ b/processed_dataset/calculation/0447.json @@ -0,0 +1,8 @@ +{ + "source_file": "./raw_volume-zh/volume14/exercise18.tex", + "problem_type": "calculation", + "problem": "问题6. 解方程组 $\\sum_{i=1}^n x_i^k=n(k=1,2, \\cdots, n)$.", + "solution": "当 $n=1$ 时,就一个方程,显然解为 $x_1=1$.\n以下不妨设 $n \\geqslant 2$. 此时构造函数\n$$\nf(t)=\\left(t-x_1\\right)\\left(t-x_2\\right) \\cdots\\left(t-x_n\\right)=t^n+a_1 t^{n-1}+a_2 t^{n-2}+\\cdots+a_{n-1} t+a_n .\n$$\n则 $f\\left(x_1\\right)=f\\left(x_2\\right)=\\cdots=f\\left(x_n\\right)=0$. 而另一方面,结合原方程组可得\n$$\n\\begin{aligned}\n\\sum_{i=1}^n f\\left(x_i\\right) & =\\sum_{i=1}^n x_i^n+a_1 \\sum_{i=1}^n x_i^{n-1}+a_2 \\sum_{i=1}^n x_i^{n-2}+\\cdots+a_{n-1} \\sum_{i=1}^n x_i+a_n \\sum_{i=1}^n 1 \\\\\n& =n+a_1 n+a_2 n+\\cdots+a_{n-1} n+a_n n=n f(1),\n\\end{aligned}\n$$\n对照可得 $f(1)=0$, 这说明 $x_1, x_2, \\cdots, x_n$ 中有一个为 1 , 不妨设 $x_n=1$, 则剩下的未知数 $x_1, x_2, \\cdots, x_{n-1}$ 满足方程组 $\\sum_{i=1}^{n-1} x_i^k=n-1(k=1,2, \\cdots, n)$. 以此类推可得所有的 $x_i=1$. 从而原方程组的解为 $x_1=x_2=\\cdots=x_n=1$.", + "remark": "", + "figures": [] +} \ No newline at end of file diff --git a/processed_dataset/calculation/0448.json b/processed_dataset/calculation/0448.json new file mode 100644 index 0000000000000000000000000000000000000000..dbe1bceb8482929778bdfbcb219018da92448be9 --- /dev/null +++ b/processed_dataset/calculation/0448.json @@ -0,0 +1,8 @@ +{ + "source_file": "./raw_volume-zh/volume14/exercise18.tex", + "problem_type": "calculation", + "problem": "问题7. (1) 能否将集合 $\\{1,2, \\cdots, 96\\}$ 表示为它的 32 个三元子集的并集,且三元子集的元素之和都相等?\n(2)能否将集合 $\\{1,2, \\cdots, 99\\}$ 表示为它的 33 个三元子集的并集,且三元子集的元素之和都相等?", + "solution": "(1) 不能.\n因为 $1+2+\\cdots+96=\\frac{96 \\times(96+1)}{2}=48 \\times 97$ 不被 32 整除.\n(2) 能.\n每个三元集的元素和为 $\\frac{1+2+\\cdots+99}{33}=\\frac{99 \\times(99+1)}{33 \\times 2}=150$. 将 $1,2,3, \\cdots, 66$ 每两个一组, 分成 33 个组, 每组两数之和可以排成一个公差为 1 的等差数列:\n$$\n1+50,3+49, \\cdots, 33+34,2+66,4+65, \\cdots, 32+51 .\n$$\n故如下 33 组数, 每组三个数之和均相等:\n$$\n\\begin{aligned}\n& \\{1,50,99\\},\\{3,49,98\\}, \\cdots,\\{33,34,83\\}, \\\\\n& \\{2,66,82\\},\\{4,65,81\\}, \\cdots,\\{32,51,67\\} .\n\\end{aligned}\n$$", + "remark": "注:此题的一般情况是\n对哪些正整数 $n$, 能将集合 $M=\\{1,2,3, \\cdots, 3 n\\}$ 表示为它的 $n$ 个三元子集的并集,且这几个三元子集的元素之和都相等?\n解首先, 要求 $n \\mid 1+2+3+\\cdots+3 n$, 即\n$$\nn\\left|\\frac{3 n(3 n+1)}{2} \\Rightarrow 2\\right| 3 n+1 \\text {. }\n$$\n所以, $n$ 为奇数.\n当 $n$ 为奇数时, 可将 $1,2,3, \\cdots, 2 n$ 每两个一组, 分成 $n$ 个组, 每组两数之和可以排成一个公差为 1 的等差数列:\n$$\n\\begin{aligned}\n& 1+\\left(n+\\frac{n+1}{2}\\right), 3+\\left(n+\\frac{n-1}{2}\\right), \\cdots, n+(n+1) ; \\\\\n& 2+2 n, 4+(2 n-1), \\cdots,(n-1)+\\left(n+\\frac{n+3}{2}\\right) .\n\\end{aligned}\n$$\n其通项公式为\n$$\na_k=\\left\\{\\begin{array}{l}\n2 k-1+\\left(n+\\frac{n+1}{2}+1-k\\right), 1 \\leqslant k \\leqslant \\frac{n+1}{2}, \\\\\n{[1-n+2(k-1)]+\\left[2 n+\\frac{n+1}{2}-(k-1)\\right], \\frac{n+3}{2} \\leqslant k \\leqslant n .}\n\\end{array}\\right.\n$$\n易知 $a_k+3 n+1-k=\\frac{9 n+3}{2}$ 为一常数, 故如下 $n$ 组数每组三个数之和均相等:\n$$\n\\begin{gathered}\n\\left\\{1, n+\\frac{n+1}{2}, 3 n\\right\\},\\left\\{3, n+\\frac{n--1}{2}, 3 n-1\\right\\}, \\cdots,\\left\\{n, n+1,3 n+1-\\frac{n+1}{2}\\right\\} ; \\\\\n\\left\\{2,2 n, 3 n+1-\\frac{n+3}{2}\\right\\}, \\cdots,\\left\\{n-1, n+\\frac{n+3}{2}, 2 n+1\\right\\} .\n\\end{gathered}\n$$\n当 $n$ 为奇数时, 依次取上述数组为 $A_1, A_2, \\cdots, A_n$, 则其为满足题设的三元子集族.\n故 $n$ 为所有的奇数.", + "figures": [] +} \ No newline at end of file diff --git a/processed_dataset/calculation/0449.json b/processed_dataset/calculation/0449.json new file mode 100644 index 0000000000000000000000000000000000000000..9d996f36d1aca5a485ccf2891627284d9a0adc1c --- /dev/null +++ b/processed_dataset/calculation/0449.json @@ -0,0 +1,8 @@ +{ + "source_file": "./raw_volume-zh/volume14/exercise18.tex", + "problem_type": "calculation", + "problem": "问题8. 能否用 2009 种颜色将所有正整数如下染色:\n(1) 每种颜色的数都有无穷多个;\n(2)不存在三个两两不同色的正整数 $a, b, c$, 满足 $a=b c$ ?", + "solution": "能.\n取 2008 个素数 $p_1)所示, 图中所标数字为相邻两点之间的弧长, 圆剖分序列为 $T_{13}=(1,3,2,7)$ 或 $(1,2,6,4)$.\n求 $P_{21}$ 和 $P_{31}$, 并各给出一个相应的圆剖分序列.", + "solution": "由于 $k$ 个点中, 每两个点间可得一段优弧和一段劣弧, 故至多可得\n$k(k-1)$ 个弧长值.\n当 $k(k-1) \\geqslant 20$ 时, 则 $k \\geqslant 5$;\n而当 $k(k-1) \\geqslant 30$ 时, 则 $k \\geqslant 6$.\n另一方面, 在 $k=5$ 时, 可以给出剖分图如图()\n所以, $P_{21}=5, T_{21}=(1,3,10,2,5)$.\n对于 $n=31$, 在 $k=6$ 时,类似可给出剖分图如图()\n所以, $P_{31}=6, T_{31}=(1,2,7,4,12,5),(1,2,5,4,6,13)$, (1, 3, $2,7,8,10),(1,3,6,2,5,14)$ 或 $(1,7,3,2,4,14)$ 等.", + "remark": "", + "figures": [ + "./images/volume14/figures/fig-c18p10.png", + "./images/volume14/figures/fig-c18a10-1.png", + "./images/volume14/figures/fig-c18a10-2.png" + ] +} \ No newline at end of file diff --git a/processed_dataset/calculation/0451.json b/processed_dataset/calculation/0451.json new file mode 100644 index 0000000000000000000000000000000000000000..a12b9379799da9947842e0417e2434a8806fc976 --- /dev/null +++ b/processed_dataset/calculation/0451.json @@ -0,0 +1,8 @@ +{ + "source_file": "./raw_volume-zh/volume14/exercise19.tex", + "problem_type": "calculation", + "problem": "问题1. 已知三个数 $5,12,18$. 每一次操作是从这三个数中任意选出两个 $a, b$, 并用 $\\frac{\\sqrt{2}}{2}(a+b), \\frac{\\sqrt{2}}{2}(a-b)$ 代替它们.\n问: 是否能进行有限次操作, 使得得到的三个数为 $3,13,20$ ?", + "solution": "因为\n$$\n\\left[\\frac{\\sqrt{2}}{2}(a+b)\\right]^2+\\left[\\frac{\\sqrt{2}}{2}(a-b)\\right]^2=a^2+b^2,\n$$\n所以, 每一次操作后, 这三个数的平方和保持不变 (即三个数的平方和是不变量). 而 $5^2+12^2+18^2=493,3^2+13^2+20^2=578$, 于是, 无论如何操作, 数 $5,12,18$ 不会变为数 $3,13,20$.", + "remark": "", + "figures": [] +} \ No newline at end of file diff --git a/processed_dataset/calculation/0452.json b/processed_dataset/calculation/0452.json new file mode 100644 index 0000000000000000000000000000000000000000..3e999a3da074652fc116f762988c4d1399e90234 --- /dev/null +++ b/processed_dataset/calculation/0452.json @@ -0,0 +1,8 @@ +{ + "source_file": "./raw_volume-zh/volume14/exercise19.tex", + "problem_type": "calculation", + "problem": "问题2. 对于黑板上的 100 个数 $1, \\frac{1}{2}, \\cdots, \\frac{1}{100}$, 每次任意选定两数 $a, b$ 擦去,同时补上 $a+b+a b$, 共操作 99 次, 求最后剩下的那个数.", + "solution": "考虑到 $(1+a)(1+b)=1+a+b+a b$, 因此操作前后 $\\prod_{i=1}^n\\left(1+a_i\\right)$ 保持不变, 其中 $n$ 为黑板上数的个数, $a_1, a_2, \\cdots, a_n$ 为这 $n$ 个数的值.\n设最后剩下的数为 $x$, 则\n$$\nx+1=\\prod_{i=1}^{100}\\left(1+\\frac{1}{i}\\right)=\\prod_{i=1}^{100} \\frac{i+1}{i}=101,\n$$\n所以 $x=100$.", + "remark": "", + "figures": [] +} \ No newline at end of file diff --git a/processed_dataset/calculation/0453.json b/processed_dataset/calculation/0453.json new file mode 100644 index 0000000000000000000000000000000000000000..af76aba8f6a7a09fb7f7a9573081654c616867d7 --- /dev/null +++ b/processed_dataset/calculation/0453.json @@ -0,0 +1,10 @@ +{ + "source_file": "./raw_volume-zh/volume14/exercise19.tex", + "problem_type": "calculation", + "problem": "问题3. 在平面上画一个 $4 \\times 4$ 的方格表, 在这些小方格的每一个中都任意地填人 1 或 -1 . 下面的一种改变填人数字的方式称为一次操作: 对任意一个小方格, 凡是与此小方格有一条公共边的所有小方格(不包括此小方格本身) 中的数作连乘积, 于是每取一格, 就算出了一个数.\n在所有小方格都取遍后, 再将这些算出的数放人相应的小方格中.\n问: 对任意的一种初始填表方式, 是否能经过有限次操作后, 使得所有小方格中的数都变为 1 ?", + "solution": "答案是否定的.\n对于如图()所示的一个 $4 \\times 4$ 的数表, 每经过一次操作, 仍然是它本身, 即这个数表在上述操作中是不变量.\n所以, 这个数表不可能经过有限次操作后, 使得所有的小方格中的数都变为 1 .\n\\begin{tabular}{|c|c|c|c|}\n\\hline 1 & -1 & -1 & 1 \\\\\n\\hline-1 & 1 & 1 & -1 \\\\\n\\hline-1 & 1 & 1 & -1 \\\\\n\\hline 1 & -1 & -1 & 1 \\\\\n\\hline\n\\end{tabular}", + "remark": "", + "figures": [ + "./images/volume14/figures/fig-c19a3.png" + ] +} \ No newline at end of file diff --git a/processed_dataset/calculation/0454.json b/processed_dataset/calculation/0454.json new file mode 100644 index 0000000000000000000000000000000000000000..4784a4f120ba9ec25e6d895a81c07b15363c1c62 --- /dev/null +++ b/processed_dataset/calculation/0454.json @@ -0,0 +1,8 @@ +{ + "source_file": "./raw_volume-zh/volume14/exercise19.tex", + "problem_type": "calculation", + "problem": "问题4. 某岛上生活着 45 条变色龙, 其中有 13 条灰色的, 15 条褐色的, 17 条紫色的.\n每当两条颜色不同的变色龙相遇时, 它们就一起都变为第三种颜色.\n问:能否经过一段时间, 45 条变色龙全都变为同一颜色?", + "solution": "每一次变化, 都有两条不同颜色的变色龙消失, 而增加了两条第三种颜色的变色龙.\n用三元数组 $(a, b, c)$ 来表示变色龙的条数, 其中 $a, b, c$ 分别表示灰色,褐色,紫色的变色龙的条数.\n在一次变化后, $(a, b$, c) 变为 $(a-1, b-1, c+2)$, 或者变为 $(a-1, b+2, c-1)$, 或者变为 $(a+2$, $b-1, c-1)$.\n由于灰色和褐色变色龙的数目之差的变化只能是 $0,-3$ 和 3 , 也就是说, 这个差被 3 除所得的余数不变, 这是一个不变量! 开始时, $a-b=13-15=$ -2 , 如果变为同一颜色, 则有 $a-b \\equiv 0(\\bmod 3)$, 不可能.", + "remark": "", + "figures": [] +} \ No newline at end of file diff --git a/processed_dataset/calculation/0455.json b/processed_dataset/calculation/0455.json new file mode 100644 index 0000000000000000000000000000000000000000..bba5ba46575c23589259de5d457056ac6abe9a82 --- /dev/null +++ b/processed_dataset/calculation/0455.json @@ -0,0 +1,8 @@ +{ + "source_file": "./raw_volume-zh/volume14/exercise2.tex", + "problem_type": "calculation", + "problem": "问题3. (1) 是否存在正整数 $m 、 n$, 使得 $m(m+2)=n(n+1)$;\n(2) 设 $k(\\geqslant 3)$ 是给定的正整数, 是否存在正整数 $m 、 n$, 使得\n$$\nm(m+k)=n(n+1) .\n$$", + "solution": "(1) 答案是否定的.\n若存在正整数 $m, n$, 使得 $m(m+2)=n(n+1)$, 则\n$$\n(m+1)^2=n^2+n+1,\n$$\n显然 $n>1$, 于是\n$$\nn^22$, 故上式不可能成立.\n当 $k \\geqslant 4$ 时, 若 $k=2 t$ ( $t$ 是不小于 2 的整数) 为偶数, 取\n$$\nm=t^2-t, n=t^2-1,\n$$\n则\n$$\n\\begin{gathered}\nm(\\dot{m}+k)=\\left(t^2-t\\right)\\left(t^2+t\\right)=t^4-t^2, \\\\\nn(n+1)=\\left(t^2-1\\right) t^2=t^4-t^2,\n\\end{gathered}\n$$\n故这样的 $(m, n)$ 满足条件.\n若 $k=2 t+1$ ( $t$ 是不小于 2 的整数) 为奇数, 取\n$$\nm=\\frac{t^2-t}{2}, n=\\frac{t^2+t-2}{2},\n$$\n则\n$$\n\\begin{gathered}\nm(m+k)=\\frac{t^2-t}{2}\\left(\\frac{t^2-t}{2}+2 t+1\\right)=\\frac{1}{4}\\left(t^4+2 t^3-t^2-2 t\\right), \\\\\nn(n+1)=\\frac{t^2+t-2}{2} \\cdot \\frac{t^2+t}{2}=\\frac{1}{4}\\left(t^4+2 t^3-t^2-2 t\\right),\n\\end{gathered}\n$$\n故这样的 $(m, n)$ 满足条件.\n综上所述, 当 $k=3$ 时,答案是否定的; 当 $k \\geqslant 4$ 时, 答案是肯定的.", + "remark": "", + "figures": [] +} \ No newline at end of file diff --git a/processed_dataset/calculation/0456.json b/processed_dataset/calculation/0456.json new file mode 100644 index 0000000000000000000000000000000000000000..7d871bc689678bf4ddf559ac7074431c12a9a1da --- /dev/null +++ b/processed_dataset/calculation/0456.json @@ -0,0 +1,8 @@ +{ + "source_file": "./raw_volume-zh/volume14/exercise20.tex", + "problem_type": "calculation", + "problem": "问题3. 有一个团体会议,有 100 人参加.\n其中任意四个人都至少有一个人认识三人.\n问: 该团体中认识其他所有人的成员最少有多少?", + "solution": "把该团体的成员视为顶点,其顶点全体记做 $V$. 对于任意两个顶点 $u$, $v$ 所代表的成员, 当且仅当彼此认识, 则在 $u, v$ 之间连一条边, 得到一个含 100 个顶点的简单图 $G$. 已知条件是, 图 $G$ 中任意四个顶点中都至少有一顶点和其他三个顶点相邻.\n要求图 $G$ 中度为 99 的顶点个数的最小值 $m$.\n当图 $G$ 是完全图时, 每个顶点的度都是 99 , 所以有 100 个度为 99 的顶点.\n当图 $G$ 是非完全图时, 图 $G$ 中必有两个不相邻的顶点 $u$ 和 $v$. 如果除 $u$ 和 $v$ 外另有两个顶点 $x, y$ 不相邻, 则 $u, v, x$ 和 $y$ 中不存在和其他三个顶点都相邻的顶点, 与题意矛盾 (与图 $G$ 的性质矛盾). 因此 $G$ 中除 $u, v$ 外任意两个顶点相邻.\n此时, 如果 $G$ 中除 $u 、 v$ 外的任何 $x$ 都和 $u, v$ 相邻, 则 $d(x)=99$, 即 $G$ 中度为 99 的顶点个数为 98 . 设 $G$ 中除 $u$ 、 $v$ 外有个顶点 $x$ 和 $u 、 v$ 不都相邻, 则由 $G$ 的性质知, $G$ 中除 $u, v, x$ 外的任意顶点 $y$ 和 $u 、 v 、 x$ 都相邻.\n因此 $d(u) \\leqslant 98, d(v) \\leqslant 98, d(x) \\leqslant 98, d(y)=99$. 所以 $G$ 中度为 99 的顶点个数为 97 .\n这表明图 $G$ 中度为 99 的顶点个数的最小值为 97 .\n回到原问题,即得: 该团体中认识其他所有人的成员最少是 97 个.", + "remark": "注:例题中的成员数 100 改为任意的 $n(n \\geqslant 4)$, 其他条件不变,则结论为该团体至少有 $n-3$ 人认识其他所有人.", + "figures": [] +} \ No newline at end of file diff --git a/processed_dataset/calculation/0457.json b/processed_dataset/calculation/0457.json new file mode 100644 index 0000000000000000000000000000000000000000..57cdb9ed6250c3f93ef648a06751c665c2d4c938 --- /dev/null +++ b/processed_dataset/calculation/0457.json @@ -0,0 +1,8 @@ +{ + "source_file": "./raw_volume-zh/volume14/exercise20.tex", + "problem_type": "calculation", + "problem": "问题5. 一个给定圆周上有 13 个点.\n能否用数字 $1,2, \\cdots, 13$ 给它们编号, 使得相邻两点标上的数之差的绝对值至多是 5 , 至少是 3 ?", + "solution": "作一个图 $G$ : 将 $1,2, \\cdots, 13$ 看成 13 个顶点, 其中对满足 $3 \\leqslant|i-j| \\leqslant$ 5 的正整数 $i, j$, 令顶点 $i, j$ 相邻.\n如果可以按照题目要求进行编号, 则图 $G$ 必有一个哈密顿圈 $C$ (经过图中每个顶点恰好一次的圈).\n将 $G$ 的顶点分成如下两个集合\n$$\nA=\\{1,2,3,11,12,13\\}, B=\\{4,5,6,7,8,9,10\\} .\n$$\n由于每个顶点必是哈密顿圈 $C$ 中两条边的端点, 但 $A$ 中任意两个顶点不相邻, 因此 $A, B$ 之间共有 $C$ 中的 12 条边, 因此 $C$ 中恰有一条边是连接 $B$ 中两个顶点的, 不妨记为 $e$.\n对于 $B$ 的顶点 $4, A$ 中只有一个顶点 1 与之相邻, 所以 4 必是边 $e$ 的一个端点.\n同理 10 也是边 $e$ 的端点.\n但 4 与 10 不相邻.\n矛盾.\n所以满足题意的编号方式不存在.", + "remark": "", + "figures": [] +} \ No newline at end of file diff --git a/processed_dataset/calculation/0458.json b/processed_dataset/calculation/0458.json new file mode 100644 index 0000000000000000000000000000000000000000..abaa2c10d4b13ea807f141b8ea0454b728df9467 --- /dev/null +++ b/processed_dataset/calculation/0458.json @@ -0,0 +1,8 @@ +{ + "source_file": "./raw_volume-zh/volume14/exercise20.tex", + "problem_type": "calculation", + "problem": "问题6. 如果凸 $n$ 边形的任意 3 条对角线都不交于一点, 试问该凸 $n$ 边形被它的对角线分成多少部分?", + "solution": "以凸 $n$ 边形的顶点以及所有对角线的交点为顶点, 以顶点间已连有的线段为边构成一个平面图 $G$. 设 $G$ 的顶点数为 $V$, 边数为 $E$, 面数为 $F$ (其中, 凸多边形的外部也算作一个面,故所求的结果即为 $F-1$ ).\n由于凸 $n$ 边形的任意 4 个顶点唯一对应形内的一个交点, 从而形内的交点总数为 $\\mathrm{C}_n^4$, 故 $G$ 的顶点数 $V=n+\\mathrm{C}_n^4$.\n又由于 $G$ 中作为凸 $n$ 边形顶点的顶点具有度数 $n-1$, 而作为凸 $n$ 边形对角线交点的顶点具有度数 4 , 所以边数 $E=\\frac{1}{2}\\left(n(n-1)+4 \\mathrm{C}_n^4\\right)$.\n将以上两式代入平面图的欧拉公式 $V-E+F=2$ 得\n$$\n\\begin{aligned}\nF-1 & =E-V+1=\\frac{1}{2}\\left(n(n-1)+4 \\mathrm{C}_n^4\\right)-\\left(n+\\mathrm{C}_n^4\\right)+1 \\\\\n& =\\frac{1}{24}(n-1)(n-2)\\left(n^2-3 n+12\\right),\n\\end{aligned}\n$$\n即该凸 $n$ 边形被它的对角线分成 $\\frac{1}{24}(n-1)(n-2)\\left(n^2-3 n+12\\right)$ 个部分.", + "remark": "", + "figures": [] +} \ No newline at end of file diff --git a/processed_dataset/calculation/0459.json b/processed_dataset/calculation/0459.json new file mode 100644 index 0000000000000000000000000000000000000000..b39e6f031e033f8561490782c81f2534a915cf1d --- /dev/null +++ b/processed_dataset/calculation/0459.json @@ -0,0 +1,8 @@ +{ + "source_file": "./raw_volume-zh/volume14/exercise3.tex", + "problem_type": "calculation", + "problem": "问题8. 给定整数 $n>0$. 有一个天平和 $n$ 个重量分别为 $2^0, 2^1, \\cdots, 2^{n-1}$ 的砝码.\n现通过 $n$ 步操作逐个将所有砝码都放上天平, 使得在操作过程中, 右边的重量总不超过左边的重量.\n每一步操作是从尚未放上天平的砝码中选择一个砝码, 将其放到天平的左边或右边, 直至所有砝码都被放上天平.\n求整个操作过程的不同方法个数.", + "solution": "操作过程的不同方法个数为 $(2 n-1) ! !=1 \\times 3 \\times 5 \\times \\cdots \\times(2 n-1)$.\n下面我们对 $n$ 用数学归纳法.\n当 $n=1$ 时, 只有一个砝码, 只能放在天平的左边, 故只有 1 种方法.\n假设 $n=k$ 时, $k$ 个重量为 $2^0, 2^1, \\cdots, 2^{k-1}$ 按题设要求有 $(2 k-1) !$ ! 种方法.\n当 $n=k+1$ 时, 此时将所有砝码的重量都乘以 $\\frac{1}{2}$, 不影响问题的本质.\n此时 $k+1$ 个砝码的重量为 $\\frac{1}{2}, 1,2, \\cdots, 2^{k-1}$. 由于对任意正整数 $r$, 有\n$$\n2^r>2^{r-1}+2^{-2}+\\cdots+1+\\frac{1}{2} \\geqslant \\sum_{i=-1}^{r-1}\\left( \\pm 2^i\\right),\n$$\n所以当所有砝码都放上天平时, 天平的较重的一端只取决于天平上最重砝码的位置, 故最重砝码一定在左边.\n下面考虑重量为 $\\frac{1}{2}$ 的砝码在操作过程中的位置.\n(1) 若重量为 $\\frac{1}{2}$ 的砝码第 1 个放, 它只能放在左边, 然后剩下的 $k$ 个砝码有 $(2 k-1)$ !! 种放法.\n(2) 若重量为 $\\frac{1}{2}$ 的砝码在第 $t$ 次操作时放, $t=2,3, \\cdots, k+1$. 由于此时已经放在天平上的砝码重量均大于 $\\frac{1}{2}$, 所以重量为 $\\frac{1}{2}$ 的砝码不会成为最重的一个,无论它放左边还是右边都不会影响最重砝码的位置, 于是有 2 种放法, 而剩下的砝码的放法不受影响, 此时有 $2 \\times(2 k-1)$ !! 种放法.\n综上所述, 当 $n=k+1$ 时,共有\n$$\n(2 k-1) ! !+k \\times 2 \\times(2 k-1) ! !=(1+2 k)(2 k-1) ! !=(2 k+1) ! !\n$$\n种放法.\n所以, 由数学归纳法知, 对于任意整数 $n>0$, 整个操作过程的不同方法个数为 $(2 n-1) ! !$.", + "remark": "", + "figures": [] +} \ No newline at end of file diff --git a/processed_dataset/calculation/0460.json b/processed_dataset/calculation/0460.json new file mode 100644 index 0000000000000000000000000000000000000000..a4448eb1799263885e6c8e9629cc0497c3265555 --- /dev/null +++ b/processed_dataset/calculation/0460.json @@ -0,0 +1,8 @@ +{ + "source_file": "./raw_volume-zh/volume14/exercise4.tex", + "problem_type": "calculation", + "problem": "问题5. 已知集合 $A$ 与 $B$ 是 $\\{1,2, \\cdots, 100\\}$ 的两个子集,满足: $A$ 与 $B$ 的元素个数相同,且 $A \\cap B$ 为空集,且 $n \\in A$ 时,总有 $2 n+2 \\in B$. 求集合 $A \\cup B$ 的元素个数最大值.", + "solution": "对每个 $n \\in A$, 由于 $2 n+2 \\in B$, 故 $2 n+2 \\leqslant$ 100 , 即 $n \\leqslant 49$.\n$$\n\\begin{aligned}\n& \\{2 k-1,4 k\\}, k=1,2, \\cdots, 12 ; \\\\\n& \\{2 k-1\\}, k=13,14, \\cdots, 25 ; \\\\\n& \\{2,6\\},\\{10,22\\},\\{14,30\\},\\{18,38\\} ; \\\\\n& \\{26\\},\\{34\\},\\{42\\},\\{46\\} .\n\\end{aligned}\n$$\n若 $|A|>33$, 则根据抽屉原理, 上述 33 个集合中必有一个二元集包含于 $A$, 即存在 $n \\in A$ 使 $2 n+2 \\in A$, 故 $2 n+2 \\in A \\cap B$, 矛盾.\n从而 $|A| \\leqslant 33$, 故 $|A \\cup B| \\leqslant 66$.\n另一方面, 如取\n$$\n\\begin{aligned}\n& A=\\{1,3,5, \\cdots, 49,2,10,14,18,26,34,42,46\\}, \\\\\n& B=\\{2 n+2 \\mid n \\in A\\},\n\\end{aligned}\n$$\n则 $A, B$ 满足题设,且 $|A \\cup B|=66$.\n综上可知 $|A \\cup B|_{\\text {max }}=66$.", + "remark": "", + "figures": [] +} \ No newline at end of file diff --git a/processed_dataset/calculation/0461.json b/processed_dataset/calculation/0461.json new file mode 100644 index 0000000000000000000000000000000000000000..99caa4356de4a17f2b5dffeb7964af2e466f64f2 --- /dev/null +++ b/processed_dataset/calculation/0461.json @@ -0,0 +1,8 @@ +{ + "source_file": "./raw_volume-zh/volume14/exercise4.tex", + "problem_type": "calculation", + "problem": "问题6. 某年级 $n$ 位同学参加语文和数学两门课的考试, 每门课的考分从 0 到 100 分.\n假如考试的结果没有两位同学的成绩是完全相同的 (即至少有一门课的成绩不同). 另外, \"甲比乙好\" 是指同学甲的语文和数学的考分均分别高于同学乙的语文和数学的考分.\n试问: 当 $n$ 最小为何值时, 必存在三位同学 (设为甲、乙、丙), 有甲比乙好, 乙比丙好?", + "solution": "不妨将语文、数学分别考 $i$ 分、 $j$ 分的同学的成绩记为有序整数对 $(i$, $j)$, 其中 $A=\\{(i, j) \\mid 0 \\leqslant i \\leqslant 100,0 \\leqslant j \\leqslant 100, i, j \\in \\mathbf{N}\\}$ 包含所有可能的成绩.\n对 $k=0, \\pm 1, \\pm 2, \\cdots, \\pm 100$, 这 201 个集合 $A_k=\\{(i, j) \\mid(i, j) \\in A$, $j=i+k\\}$ 的并集为 $A$, 且两两交集为空.\n若有 401 位同学参加考试, 由于 $A_{100}=\\{(0,100)\\}, A_{-100}=\\{(100,0)\\}$ 为单元集, 故至少有 399 人的成绩属于某个 $A_k(-99 \\leqslant k \\leqslant 99)$, 根据抽屉原理, 至少有 3 人的成绩属于同一集合, 按照定义可将他们排列为甲、乙、丙, 使甲比乙好, 乙比丙好.\n另一方面,若 400 位同学的考试成绩构成集合\n$$\nA=\\{(i, j) \\mid(i, j) \\in A, \\max \\{i, j\\} \\geqslant 99\\},\n$$\n那么不存在三位同学甲、乙、丙, 使甲比乙好, 乙比丙好.\n综上所述, $n$ 的最小值为 401 .", + "remark": "", + "figures": [] +} \ No newline at end of file diff --git a/processed_dataset/calculation/0462.json b/processed_dataset/calculation/0462.json new file mode 100644 index 0000000000000000000000000000000000000000..938c23315379efe565c7b42431dce46923026b09 --- /dev/null +++ b/processed_dataset/calculation/0462.json @@ -0,0 +1,8 @@ +{ + "source_file": "./raw_volume-zh/volume14/exercise5.tex", + "problem_type": "calculation", + "problem": "问题1. 在所有 $n$ 位数中, 求数码 $1,2,3$ 都出现, 但其他数码均不出现的数的个数.", + "solution": "设不出现数码 $4,5, \\cdots, 9,0$ 的全体 $n$ 位数组成集合 $I$.\n对 $k=1,2,3$, 记 $I$ 中不含数码 $k$ 的 $n$ 位数全体组成集合 $A_k$. 显然, 满足条件的数的全体组成集合 $\\overline{A_1} \\cap \\overline{A_2} \\cap \\overline{A_3}$.\n由于\n$$\n\\begin{gathered}\n|I|=3^n,\\left|A_1\\right|=\\left|A_2\\right|=\\left|A_3\\right|=2^n, \\\\\n\\left|A_1 \\cap A_2\\right|=\\left|A_2 \\cap A_3\\right|=\\left|A_3 \\cap A_1\\right|=1,\\left|A_1 \\cap A_2 \\cap A_3\\right|=0,\n\\end{gathered}\n$$\n由容斥原理得\n$$\n\\begin{aligned}\n\\left|\\overline{A_1} \\cap \\overline{A_2} \\cap \\overline{A_3}\\right|= & |I|-\\left|A_1\\right|-\\left|A_2\\right|-\\left|A_3\\right|+\\left|A_1 \\cap A_2\\right| \\\\\n& +\\left|A_2 \\cap A_3\\right|+\\left|A_3 \\cap A_1\\right|-\\left|A_1 \\cap A_2 \\cap A_3\\right| \\\\\n= & 3^n-3 \\cdot 2^n+3 .\n\\end{aligned}\n$$\n故所求 $n$ 位数有 $3^n-3 \\cdot 2^n+3$ 个.", + "remark": "", + "figures": [] +} \ No newline at end of file diff --git a/processed_dataset/calculation/0463.json b/processed_dataset/calculation/0463.json new file mode 100644 index 0000000000000000000000000000000000000000..60473a672d2d90a9425e3b13d41536643369d0d8 --- /dev/null +++ b/processed_dataset/calculation/0463.json @@ -0,0 +1,8 @@ +{ + "source_file": "./raw_volume-zh/volume14/exercise5.tex", + "problem_type": "calculation", + "problem": "问题2. 全体正整数中凡是 3 的倍数或 4 的倍数都划去,但其中 5 的倍数都保留 (例如, $15,20,30,40,60, \\cdots$ 都保留). 将留下的数,按从小到大的顺序写成了一个数列 $\\left\\{a_n\\right\\}: a_1=1, a_2=2, a_3=5, \\cdots$. 求 $a_{2005}$.", + "solution": "设 $a_{2005}=n$. 记 $s=\\{1,2, \\cdots, n\\}, A_i=\\{k \\mid k \\in s$ 且 $k$ 被 $i$ 整除 $\\}$.\n$A_i$ 在 $s$ 中的补集为 $\\overline{A_i}(i=3,4,5)$.\n$$\n\\begin{aligned}\n2005= & \\left|\\left(\\bar{A}_3 \\cap \\bar{A}_4 \\cap \\bar{A}_5\\right) \\cup A_5\\right| \\\\\n= & \\left|\\bar{A}_3 \\cap \\bar{A}_4 \\cap \\bar{A}_5\\right|+\\left|A_5\\right| \\\\\n= & |s|-\\left|A_3\\right|-\\left|A_4\\right|-\\left|A_5\\right|+\\left|A_3 \\cap A_4\\right|+\\left|A_3 \\cap A_5\\right|+\\left|A_4 \\cap A_5\\right| \\\\\n& -\\left|A_3 \\cap A_4 \\cap A_5\\right|+\\left|A_5\\right| \\\\\n= & n-\\left[\\frac{n}{3}\\right]-\\left[\\frac{n}{4}\\right]-\\left[\\frac{n}{5}\\right]+\\left[\\frac{n}{12}\\right]+\\left[\\frac{n}{15}\\right]+\\left[\\frac{n}{20}\\right]-\\left[\\frac{n}{60}\\right]+\\left[\\frac{n}{5}\\right] . \\label{eq1}\n\\end{aligned}\n$$\n利用 $x-1<[x] \\leqslant x$, 由 式\\ref{eq1} 得\n$$\n\\left\\{\\begin{array}{l}\n2005n-\\frac{n}{3}-\\frac{n}{4}+\\left(\\frac{n}{12}-1\\right)+\\left(\\frac{n}{15}-1\\right)+\\left(\\frac{n}{20}-1\\right)-\\frac{n}{60}\n\\end{array}\\right.\n$$\n解得 $3336 \\frac{2}{3}4$ 成立的解的全体, 则\n$$\n\\left|\\bigcap_{k=1}^6 \\overline{A_k}\\right|=|I|-\\left|\\bigcup_{k=1}^6 A_k\\right|=|I|-\\sum_{k=1}^6\\left|A_k\\right|+\\sum_{1 \\leqslant j4$ 成立的解.\n又根据对应原理可知上式中 $|I|=\\mathrm{C}_{14}^5,\\left|A_k\\right|=\\mathrm{C}_{10}^5$, $\\left|A_j \\cap A_k\\right|=\\mathrm{C}_6^5$, 所以\n$$\n\\left|\\bigcap_{k=1}^6 \\overline{A_k}\\right|=\\mathrm{C}_{14}^5-6 \\mathrm{C}_{10}^5+\\mathrm{C}_6^2 \\mathrm{C}_6^5=2002-1512+90=580 .\n$$\n上面求得了 5 个岗位的选取方法数.\n考虑到 5 种新式武器在 5 个岗位的排列方法有 $5 !=120$ 种, 故总共配备新式武器的方案数为\n$$\n580 \\times 120=69600 .\n$$", + "remark": "", + "figures": [] +} \ No newline at end of file diff --git a/processed_dataset/calculation/0467.json b/processed_dataset/calculation/0467.json new file mode 100644 index 0000000000000000000000000000000000000000..82efd937d24c0c6ef3f0b3a63ac23b5571a1af03 --- /dev/null +++ b/processed_dataset/calculation/0467.json @@ -0,0 +1,8 @@ +{ + "source_file": "./raw_volume-zh/volume14/exercise7.tex", + "problem_type": "calculation", + "problem": "问题1. 把 $1,2,3,4, \\cdots, 80,81$ 这 81 个数任意排列为 $a_1, a_2, a_3, \\cdots, a_{81}$. 计算:\n$$\n\\left|a_1-a_2+a_3\\right|,\\left|a_4-a_5+a_6\\right|, \\cdots,\\left|a_{79}-a_{80}+a_{81}\\right|,\n$$\n再将这 27 个数任意排列为 $b_1, b_2, b_3, \\cdots, b_{27}$. 计算出:\n$$\n\\left|b_1-b_2+b_3\\right|,\\left|b_4-b_5+b_6\\right|, \\cdots,\\left|b_{25}-b_{26}+b_{27}\\right|,\n$$\n如此继续下去, 最后得到一个数 $x$, 问: $x$ 是奇数还是偶数?", + "solution": "因为\n$$\n\\begin{aligned}\n& b_1+b_2+\\cdots b_{27} \\\\\n= & \\left|a_1-a_2+a_3\\right|+\\left|a_4-a_5+a_6\\right|+\\cdots+\\left|a_{79}-a_{80}+a_{81}\\right| \\\\\n\\equiv & a_1-a_2+a_3+a_4-a_5+a_6+\\cdots+a_{79}-a_{80}+a_{81} \\\\\n\\equiv & a_1+a_2+a_3+\\cdots+a_{79}+a_{80}+a_{81}(\\bmod 2),\n\\end{aligned}\n$$\n所以,将 $a_1, a_2, a_3, \\cdots, a_{81}$ 变换为 $b_1, b_2, b_3, \\cdots, b_{27}$, 并不改变它们的和的奇偶性,因此经过多次变换后依然如此.\n所以\n$$\n\\begin{aligned}\nx & \\equiv a_1+a_2+a_3+\\cdots+a_{81}=1+2+3+\\cdots+81 \\\\\n& =41 \\times 81 \\equiv 1(\\bmod 2),\n\\end{aligned}\n$$\n即 $x$ 为奇数.", + "remark": "", + "figures": [] +} \ No newline at end of file diff --git a/processed_dataset/calculation/0468.json b/processed_dataset/calculation/0468.json new file mode 100644 index 0000000000000000000000000000000000000000..adb7b81807f7a013909933e823a67711e2aef0b7 --- /dev/null +++ b/processed_dataset/calculation/0468.json @@ -0,0 +1,8 @@ +{ + "source_file": "./raw_volume-zh/volume14/exercise7.tex", + "problem_type": "calculation", + "problem": "问题4. 今有两张 $3 \\times 3$ 方格表 A 与 B, 现将数 $1,2, \\cdots, 9$ 按某种顺序填人 A 表 (每格填写一个数), 然后依照如下规则填写 B 表: 使 B 表中第 $i$ 行、第 $j$ 列交叉处的方格内所填的数等于 $\\mathrm{A}$ 表中第 $i$ 行的各数和与第 $j$ 列的各数和之差的绝对值; 例如 B 表中的\n$$\nb_{12}=\\left|\\left(a_{11}+a_{12}+a_{13}\\right)-\\left(a_{12}+a_{22}+a_{32}\\right)\\right| .\n$$\n问: 能否在 $A$ 表适当填人 $1,2, \\cdots, 9$, 使得在 $B$ 表中也出现 $1,2, \\cdots, 9$\n这九个数字?\n\\begin{tabular}{|l|l|l|}\n\\hline$a_{11}$ & $a_{12}$ & $a_{13}$ \\\\\n\\hline$a_{21}$ & $a_{22}$ & $a_{23}$ \\\\\n\\hline$a_{31}$ & $a_{32}$ & $a_{33}$ \\\\\n\\hline\n\\end{tabular}\n(第 4 题表 $A$ )\n\\begin{tabular}{|l|l|l|}\n\\hline$b_{11}$ & $b_{12}$ & $b_{13}$ \\\\\n\\hline$b_{21}$ & $b_{22}$ & $b_{23}$ \\\\\n\\hline$b_{31}$ & $b_{32}$ & $b_{33}$ \\\\\n\\hline\n\\end{tabular}\n(第 4 题表 $B$)", + "solution": "不能作出这样的安排, 为此, 将 $\\mathrm{B}$ 表中的各数去掉绝对值符号, 所得到的表格记为表 $\\mathrm{C}$ :\n则\n$$\n\\begin{gathered}\nc_{11}=\\left(a_{11}+a_{12}+a_{13}\\right)-\\left(a_{11}+a_{21}+a_{31}\\right), \\\\\nc_{12}=\\left(a_{11}+a_{12}+a_{13}\\right)-\\left(a_{12}+a_{22}+a_{32}\\right), \\\\\n\\ldots \\ldots . \\\\\nc_{33}=\\left(a_{31}+a_{32}+a_{33}\\right)-\\left(a_{13}+a_{23}+a_{33}\\right),\n\\end{gathered}\n$$\n\\begin{tabular}{|l|l|l|}\n\\hline$c_{11}$ & $c_{12}$ & $c_{13}$ \\\\\n\\hline$c_{21}$ & $c_{22}$ & $c_{23}$ \\\\\n\\hline$c_{31}$ & $c_{32}$ & $c_{33}$ \\\\\n\\hline\n\\end{tabular}\n表 C\n易见, $c_{11}+c_{12}+\\cdots+c_{33}=0$, 故 C 表中有偶数个奇数, 因为 $b_{i j}==\\left|c_{i j}\\right|$, 故 $b_{i j}$ 与 $c_{i j}$ 同奇偶, 所以 B 表中也有偶数个奇数,但 $1,2, \\cdots, 9$ 中有奇数个奇数, 因此不能作出这样的安排.", + "remark": "", + "figures": [] +} \ No newline at end of file diff --git a/processed_dataset/calculation/0469.json b/processed_dataset/calculation/0469.json new file mode 100644 index 0000000000000000000000000000000000000000..4c5c638eacec6352046e81d1d5b6d3e002da33f7 --- /dev/null +++ b/processed_dataset/calculation/0469.json @@ -0,0 +1,8 @@ +{ + "source_file": "./raw_volume-zh/volume14/exercise7.tex", + "problem_type": "calculation", + "problem": "问题5. 设正整数 $n$ 的所有正约数从小到大依次为 $d_1|\\varepsilon S|$ 可知只能是 $\\varepsilon= \\pm 1$, 故易得 $S= \\pm \\frac{\\sqrt{2}}{2}$\n当 $S=\\frac{\\sqrt{2}}{2}$ 时, $x_1^2-\\frac{\\sqrt{2}}{2} x_1-1=0$, 解得 $x_1=\\sqrt{2}$ 或 $x_1=-\\frac{\\sqrt{2}}{2}$; 当 $S=-\\frac{\\sqrt{2}}{2}$ 时, 同理得 $x_1=-\\sqrt{2}$ 或 $x_1=\\frac{\\sqrt{2}}{2}$.\n所以 $x_1$ 的所有可能值为 $\\pm \\sqrt{2}, \\pm \\frac{\\sqrt{2}}{2}$.", + "remark": "注:条件中的 5 个方程具有明显的循环特征,这就提示我们从整体考虑问题.\n纵观全局, 联想到引人辅助量 $S=x_1+x_2+x_3+x_4+x_5$, 从而将原先具有 5 个变元的方程组转化为考虑 \" $x_1, x_2, x_3, x_4, x_5$ 可能取 $x^2-S x-1=0$ 两根各多少次\" 的简单问题.", + "figures": [] +} \ No newline at end of file diff --git a/processed_dataset/calculation/0473.json b/processed_dataset/calculation/0473.json new file mode 100644 index 0000000000000000000000000000000000000000..f1139bd64cde05529b72687a8ca2421f12c74a91 --- /dev/null +++ b/processed_dataset/calculation/0473.json @@ -0,0 +1,8 @@ +{ + "source_file": "./raw_volume-zh/volume14/exercise9.tex", + "problem_type": "calculation", + "problem": "问题5. 给定 $n(n>1)$ 个二次三项式 $x^2-a_i x+b_i(1 \\leqslant i \\leqslant n)$, 其中 $2 n$ 个实数 $a_i, b_i$ 互不相同.\n试问: 是否可能 $a_i, b_i(1 \\leqslant i \\leqslant n)$ 中的每个数都是其中某个多项式的根?", + "solution": "假定有这样的可能性.\n因二次三项式至多有两个实根, 而 $2 n$ 个实数 $a_i, b_i$ 互不相同, 所以每个二次三项式 $x^2-a_i x+b_i(1 \\leqslant i \\leqslant n)$ 必含有两个不相等的实根 $u_i, v_i$, 且 $\\left(a_1, a_2, \\cdots, a_n, b_1, b_2, \\cdots, b_n\\right)$ 构成 $\\left(u_1, u_2, \\cdots, u_n\\right.$, $\\left.v_1, v_2, \\cdots, v_n\\right)$ 的一个排列.\n对 $1 \\leqslant i \\leqslant n$, 由韦达定理知 $u_i+v_i=a_i, u_i v_i=b_i$. 所以,\n$$\n\\sum_{i=1}^n a_i=\\sum_{i=1}^n\\left(u_i+v_i\\right)=\\sum_{i=1}^n a_i+\\sum_{i=1}^n b_i,\n$$\n因此 $\\sum_{i=1}^n b_i=0$.\n另一方面,还有 $u_i^2+v_i^2=\\left(u_i+v_i\\right)^2-2 u_i v_i=a_i^2-2 b_i$, 从而,\n$$\n\\sum_{i=1}^n\\left(a_i^2+b_i^2\\right)=\\sum_{i=1}^n\\left(u_i^2+v_i^2\\right)=\\sum_{i=1}^n\\left(a_i^2-2 b_i\\right)=\\sum_{i=1}^n a_i^2 .\n$$\n这表明了 $\\sum_{i=1}^n b_i^2=0$, 于是所有 $b_i$ 全为 0 ,矛盾.\n因此不可能 $a_i, b_i(1 \\leqslant i \\leqslant n)$ 中的每个数都是其中某个多项式的根.", + "remark": "", + "figures": [] +} \ No newline at end of file diff --git a/processed_dataset/calculation/0474.json b/processed_dataset/calculation/0474.json new file mode 100644 index 0000000000000000000000000000000000000000..07127e0c8e9e8f6da95a412166cab9ea0f4cdcea --- /dev/null +++ b/processed_dataset/calculation/0474.json @@ -0,0 +1,10 @@ +{ + "source_file": "./raw_volume-zh/volume14/exercise9.tex", + "problem_type": "calculation", + "problem": "问题6. 沿着圆周放着一些数,如果有依次相连的 4 个数 $a, b, c, d$ 满足不等式 $(a-d)(b-c)>0$, 那么就可以交换 $b, c$ 的位置, 这称为一次操作.\n(1)若圆周上依次放着数 $1,2,3,4,5,6$, 问: 是否能经过有限次操作后, 对圆周上任意依次相连的 4 个数 $a, b, c, d$, 都有 $(a-d)(b-c) \\leqslant 0$ ? 请说明理由.\n(2)若圆周上从小到大按顺时针方向依次放着 2006 个正整数 $1,2, \\cdots$, 2006, 问: 是否能经过有限次操作后, 对圆周上任意依次相连的 4 个数 $a, b, c, d$, 都有 $(a-d)(b-c) \\leqslant 0$ ? 请说明理由.", + "solution": "(1) 答案是肯定的.\n具体操作如图()\n(2) 答案是肯定的.\n考虑这 2006 个数的相邻两数乘积之和为 $P$.\n开始时, $P_0=1 \\times 2+2 \\times 3+3 \\times 4+\\cdots+2005 \\times 2006+2006 \\times 1$, 经过 $k(k \\geqslant 0)$ 次操作后, 这 2006 个数的相邻两数乘积之和为 $P_k$, 此时若圆周上依次相连的 4 个数 $a, b, c, d$ 满足不等式 $(a-d)(b-c)>0$, 即 $a b+c d>a c+ b d$, 交换 $b, c$ 的位置后, 这 2006 个数的相邻两数乘积之和为 $P_{k+1}$, 有\n$$\nP_{k+1}-P_k=(a c+c b+b d)-(a b+b c+c d)=a c+b d-a b-c d<0 .\n$$\n所以 $P_{k+1}-P_k \\leqslant-1$, 即每一次操作, 相邻两数乘积的和至少减少 1 , 由于相邻两数乘积总大于 0 ,故经过有限次操作后, $P$ 无法变得更小, 此时对任意依次相连的 4 个数 $a, b, c, d$, 一定有 $(a-d)(b-c) \\leqslant 0$.", + "remark": "", + "figures": [ + "./images/volume14/figures/fig-c9a6.png" + ] +} \ No newline at end of file diff --git a/processed_dataset/calculation/0475.json b/processed_dataset/calculation/0475.json new file mode 100644 index 0000000000000000000000000000000000000000..b60bb1ce8ad25d8ee538ff1108c299cd0318d1f8 --- /dev/null +++ b/processed_dataset/calculation/0475.json @@ -0,0 +1,8 @@ +{ + "source_file": "./raw_volume-zh/volume2/chapter1.tex", + "problem_type": "calculation", + "problem": "例1. 设集合 $A=\\{a, b, c\\}, B=\\{x, y, z\\}$.\n判断以下三种对应:\n$$\n\\begin{aligned}\n& f: a \\mapsto y, b \\mapsto z, c \\mapsto x, \\\\\n& g: a \\mapsto y, c \\mapsto x, \\\\\n& h: a \\mapsto y, b \\mapsto z, c \\mapsto x, c \\mapsto z,\n\\end{aligned}\n$$\n是否是 $A$ 到 $B$ 的映射.", + "solution": "解:$$\nf: a \\mapsto y, b \\mapsto z, c \\mapsto x\n$$\n是 $A$ 到 $B$ 的一个映射.\n而\n$$\ng: a \\mapsto y, c \\longmapsto x\n$$\n不是 $A$ 到 $B$ 的映射, 因为 $b$ 在 $g$ 的作用下没有象.\n$$\nh: a \\mapsto y, b \\mapsto z, c \\longmapsto x, c \\mapsto z\n$$\n也不是 $A$ 到 $B$ 的映射.\n因为 $A$ 中元素 $c$ 有 $B$ 中两个元素 $x$ 和 $z$ 与它对应.", + "remark": "", + "figures": [] +} \ No newline at end of file diff --git a/processed_dataset/calculation/0476.json b/processed_dataset/calculation/0476.json new file mode 100644 index 0000000000000000000000000000000000000000..cb4b94ee90953e2ceb0d8c2f30e68d8af3d7dabb --- /dev/null +++ b/processed_dataset/calculation/0476.json @@ -0,0 +1,8 @@ +{ + "source_file": "./raw_volume-zh/volume2/chapter1.tex", + "problem_type": "calculation", + "problem": "例2 设 $A=\\left\\{a_1, a_2, a_3\\right\\}, B=\\{-1,0,1\\}$.\n(1) 问从 $A$ 到 $B$ 的不同的映射有多少个?\n(2) 从 $A$ 到 $B$ 的映射满足 $f\\left(a_1\\right)>f\\left(a_2\\right) \\geqslant f\\left(a_3\\right)$, 确定这样的映射 $f$ : $A \\rightarrow B$ 的个数.", + "solution": "解:(1) 确定 $a_1$ 的象,有 3 种方法; 确定 $a_2$ 的象, 也有 3 种方法; 确定 $a_3$ 的象,还是有 3 种方法.\n所以, 从 $A$ 到 $B$ 不同的映射共有\n$$\n3 \\times 3 \\times 3=27 \\text { (个). }\n$$\n(2) 由 $f\\left(a_1\\right)>f\\left(a_2\\right) \\geqslant f\\left(a_3\\right)$ 知, $f\\left(a_1\\right)=0 或 1$.\n若 $f\\left(a_1\\right)=0$, 则 $f\\left(a_2\\right)=f\\left(a_3\\right)=-1$.\n若 $f\\left(a_1\\right)=1$, 则 $f\\left(a_2\\right)=f\\left(a_3\\right)=0$, 或 $f\\left(a_2\\right)=f\\left(a_3\\right)=-1$, 或 $f\\left(a_2\\right)=0$, $f\\left(a_3\\right)=-1$.\n综上,共有 4 种满足题意的映射.", + "remark": "", + "figures": [] +} \ No newline at end of file diff --git a/processed_dataset/calculation/0477.json b/processed_dataset/calculation/0477.json new file mode 100644 index 0000000000000000000000000000000000000000..b18da7dd786bfbbecfb578419f0c799cd0f120da --- /dev/null +++ b/processed_dataset/calculation/0477.json @@ -0,0 +1,8 @@ +{ + "source_file": "./raw_volume-zh/volume2/chapter1.tex", + "problem_type": "calculation", + "problem": "例3 设 $A=\\left\\{a_1, a_2, a_3\\right\\}, B=\\left\\{b_1, b_2, b_3, b_4\\right\\}$.\n(1) 写出一个 $f: A \\rightarrow B$, 使得 $f$ 是单射, 并求 $A$ 到 $B$ 的单射个数;\n(2) 写出一个 $f: A \\rightarrow B$, 使得 $f$ 不是单射, 并求所有这种映射的个数;\n(3) $A$ 到 $B$ 的映射能否是满射?", + "solution": "解:(1) 映射\n$$\nf: a_1 \\mapsto b_1, a_2 \\mapsto b_2, a_3 \\mapsto b_3\n$$\n就是 $A$ 到 $B$ 的一个单射.\n这种映射的个数为 $\\mathrm{P}_4^3=24$ (个).\n(2) 映射\n$$\nf: a_1 \\mapsto b_1, a_2 \\mapsto b_1, a_3 \\mapsto b_1\n$$\n即为所求.\n这种映射的个数为 $4^3-\\mathrm{P}_4^3=40$ (个).\n(3) 答案是否定的.\n由于集合 $A$ 中的每一个元素恰与集合 $B$ 中的一个元素对应, 而 $|A|=3,|B|=4$ (用 $|A|$ 表示集合 $A$ 的元素个数), 所以集合 $B$ 中至少有一个元素,在集合 $A$ 中找不到与它对应的元素.\n因而 $A$ 到 $B$ 的满射不存在.\n一般地, 如果 $A$ 到 $B$ 有一个单射, 那么 $|A| \\leqslant|B|$; 如果 $A$ 到 $B$ 有一个满射, 那么 $|A| \\geqslant|B|$.", + "remark": "", + "figures": [] +} \ No newline at end of file diff --git a/processed_dataset/calculation/0478.json b/processed_dataset/calculation/0478.json new file mode 100644 index 0000000000000000000000000000000000000000..d857b802f04790e3e21edadafe1b26d936a3fb70 --- /dev/null +++ b/processed_dataset/calculation/0478.json @@ -0,0 +1,8 @@ +{ + "source_file": "./raw_volume-zh/volume2/chapter1.tex", + "problem_type": "calculation", + "problem": "例9 试作开区间 $(0,1)$ 与闭区间 $[0,1]$ 的一一对应.", + "solution": "分析:由于 $[0,1]$ 比 $(0,1)$ 多了两个点, 所以处理好这两个点是关键所在.\n解取开区间 $(0,1)$ 的子集\n$$\nA=\\left\\{\\frac{1}{2}, \\frac{1}{3}, \\cdots, \\frac{1}{n}, \\cdots\\right\\},\n$$\n在 $A$ 中加上两个数 0,1 后得到闭区间 $[0,1]$ 的子集\n$$\nB=\\left\\{0,1, \\frac{1}{2}, \\frac{1}{3}, \\cdots, \\frac{1}{n}, \\cdots\\right\\} .\n$$\n显然, $(0,1) \\backslash A=[0,1] \\backslash B$. 于是定义映射 $f$ 如下:\n$$\nf(x)= \\begin{cases}0, & \\text { 当 } x=\\frac{1}{2} \\text { 时, } \\\\ 1, & \\text { 当 } x=\\frac{1}{3} \\text { 时, } \\\\ \\frac{1}{n-2}, & \\text { 当 } x=\\frac{1}{n}, n=4,5, \\cdots \\text { 时, } \\\\ x, & \\text { 当 } x \\in(0,1) \\backslash A \\text { 时.\n}\\end{cases}\n$$\n易知 $f$ 为一一对应.\n说明集合 $A \\backslash B$ (或 $A-B$ ) 定义为\n$$\nA \\backslash B=\\{x \\mid x \\in A \\text {, 且 } x \\bar{\\in} B\\} .\n$$", + "remark": "", + "figures": [] +} \ No newline at end of file diff --git a/processed_dataset/calculation/0479.json b/processed_dataset/calculation/0479.json new file mode 100644 index 0000000000000000000000000000000000000000..13c53a93222f559d315cb12109fa2c7de773fb5e --- /dev/null +++ b/processed_dataset/calculation/0479.json @@ -0,0 +1,8 @@ +{ + "source_file": "./raw_volume-zh/volume2/chapter1.tex", + "problem_type": "calculation", + "problem": "例10 (1) 求函数 $y=\\frac{\\sqrt{x^2-4}}{\\log _2\\left(x^2+2 x-3\\right)}$ 的定义域;\n(2) 已知函数 $f(x)$ 的定义域为 $[-1,1]$. 求 $f(a x)+f\\left(\\frac{x}{a}\\right)$ 的定义域, 其中 $a>0$.", + "solution": "解:(1) 函数的定义域是满足下列条件的解集.\n$$\n\\left\\{\\begin{array}{l}\nx^2-4 \\geqslant 0, \\\\\nx^2+2 x-3>0, \\\\\nx^2+2 x-3 \\neq 1 .\n\\end{array}\\right.\n$$\n因此, 定义域为 $(-\\infty,-1-\\sqrt{5}) \\cup(-1-\\sqrt{5},-3) \\cup[2,+\\infty)$.\n(2) $f(a x)+f\\left(\\frac{x}{a}\\right)$ 的定义域是下列两个集合的交集:\n$$\n\\begin{aligned}\n& D_1=\\{x \\mid-1 \\leqslant a x \\leqslant 1\\}=\\left[-\\frac{1}{a}, \\frac{1}{a}\\right], \\\\\n& D_2=\\left\\{x \\mid-1 \\leqslant \\frac{x}{a} \\leqslant 1\\right\\}=[-a, a] .\n\\end{aligned}\n$$\n当 $a \\geqslant 1$ 时, $a \\geqslant \\frac{1}{a},-a \\leqslant-\\frac{1}{a}$, 故 $D_1 \\cap D_2=D_1$;\n当 $0a,-\\frac{1}{a}<-a$, 故 $D_1 \\cap D_2=D_2$.\n因此, $f(a x)+f\\left(\\frac{x}{a}\\right)$ 的定义域为\n$$\n[-a, a] \\text { (当 } 01 \\text { 时, }\\end{array} g(x)= \\begin{cases}2-x^2, & \\text { 当 }|x| \\leqslant 1 \\text { 时, } \\\\ 2, & \\text { 当 }|x|>1 \\text { 时.\n}\\end{cases}\\right.$\n求 $f[f(x)], f[g(x)], g[f(x)], g[g(x)]$.", + "solution": "解:$f[f(x)]=1, x \\in \\mathbf{R}$.\n$$\nf[g(x)]= \\begin{cases}0, & \\text { 当 } x \\neq \\pm 1 \\text { 时, } \\\\ 1, & \\text { 当 } x= \\pm 1 \\text { 时.\n}\\end{cases}\n$$\n$$\n\\begin{aligned}\n& g[f(x)]= \\begin{cases}1, & \\text { 当 }|x| \\leqslant 1 \\text { 时, } \\\\\n2, & \\text { 当 }|x|>1 \\text { 时.\n}\\end{cases} \\\\\n& g[g(x)]= \\begin{cases}2, & \\text { 当 } x \\neq \\pm 1 \\text { 时, } \\\\\n1, & \\text { 当 } x= \\pm 1 \\text { 时.\n}\\end{cases}\n\\end{aligned}\n$$\n从本题可以看出, $f[g(x)]$ 不一定与 $g[f(x)]$ 相等.", + "remark": "", + "figures": [] +} \ No newline at end of file diff --git a/processed_dataset/calculation/0481.json b/processed_dataset/calculation/0481.json new file mode 100644 index 0000000000000000000000000000000000000000..038b361f018ed31227e2c2cadb4a4bd5696329e5 --- /dev/null +++ b/processed_dataset/calculation/0481.json @@ -0,0 +1,10 @@ +{ + "source_file": "./raw_volume-zh/volume2/chapter1.tex", + "problem_type": "calculation", + "problem": "例13 作出函数 $y=\\left|x^2+x-6\\right|$ 的图象.", + "solution": "解:先作出 $y=x^2+x-6$ 的图象, 然后将此图象在 $x$ 轴下方的部分对称地翻折到 $x$ 轴的上方即可,如图()实线所示部分.", + "remark": "", + "figures": [ + "./images/volume2/figures/fig-c1e13.png" + ] +} \ No newline at end of file diff --git a/processed_dataset/calculation/0482.json b/processed_dataset/calculation/0482.json new file mode 100644 index 0000000000000000000000000000000000000000..fc859e9f61b18b738871ef2c62b13a07d8b4d026 --- /dev/null +++ b/processed_dataset/calculation/0482.json @@ -0,0 +1,8 @@ +{ + "source_file": "./raw_volume-zh/volume2/chapter1.tex", + "problem_type": "calculation", + "problem": "例14 (1) 设抛物线 $y=2 x^2$, 把它向右平移 $p$ 个单位, 或向下移 $q$ 个单位, 都能使得抛物线与直线 $y=x-4$ 恰好有一个交点, 求 $p, q$ 的值;\n(2) 把抛物线 $y=2 x^2$ 向左平移 $p$ 个单位, 向上平移 $q$ 个单位, 则得到的抛物线经过点 $(1,3)$ 与 $(4,9)$, 求 $p, q$ 的值.\n(3) 把抛物线 $y=a x^2+b x+c$ 向左平移 3 个单位,向下平移 2 个单位后, 所得图象是经过点 $\\left(-1,-\\frac{1}{2}\\right)$ 的抛物线 $y=a x^2$, 求原二次函数的解析式.", + "solution": "解:(1) 抛物线 $y=2 x^2$ 向右平移 $p$ 个单位后, 得到的抛物线为 $y= 2(x-p)^2$. 于是方程 $2(x-p)^2=x-4$ 有两个相同的根, 即方程\n$$\n2 x^2-(4 p+1) x+2 p^2+4=0\n$$\n根的判别式\n$$\n\\Delta=(4 p+1)^2-4 \\cdot 2 \\cdot\\left(2 p^2+4\\right)=0 .\n$$\n所以 $p=\\frac{31}{8}$. 这时的交点为 $\\left(\\frac{33}{8}, \\frac{1}{8}\\right)$.\n抛物线 $y=2 x^2$ 向下平移 $q$ 个单位, 得到抛物线 $y=2 x^2-q$. 于是方程 $2 x^2-q=x-4$ 有两个相同的根, 即\n$$\n\\Delta=1-4 \\cdot 2(4-q)=0 .\n$$\n所以 $q=\\frac{31}{8}$. 这时的交点为 $\\left(\\frac{1}{4},-\\frac{15}{4}\\right)$.\n(2) 把 $y=2 x^2$ 向左平移 $p$ 个单位, 向上平移 $q$ 个单位, 得到的抛物线为 $y=2(x+p)^2+q$. 于是, 由题设, 得\n$$\n\\left\\{\\begin{array}{l}\n3=2(1+p)^2+q, \\\\\n9=2(4+p)^2+q .\n\\end{array}\\right.\n$$\n解方程组, 得 $p=-2, q=1$, 即抛物线向右平移了 2 个单位, 向上平移了 1 个单位.\n(3)首先, 抛物线 $y=a x^2$ 经过点 $\\left(-1,-\\frac{1}{2}\\right)$, 可求得 $a=-\\frac{1}{2}$.\n设原来的二次函数为 $y=-\\frac{1}{2}(x-h)^2+k$, 由题设知\n$$\n\\left\\{\\begin{array}{l}\n-h+3=0 \\\\\nk-2=0\n\\end{array}\\right.\n$$\n解方程组, 得 $h=3, k=2$. 故原二次函数为\n$$\ny=-\\frac{1}{2}(x-3)^2+2 .\n$$", + "remark": "", + "figures": [] +} \ No newline at end of file diff --git a/processed_dataset/calculation/0483.json b/processed_dataset/calculation/0483.json new file mode 100644 index 0000000000000000000000000000000000000000..dba69d1c84b04982cc9421f239dcbf8c799f4060 --- /dev/null +++ b/processed_dataset/calculation/0483.json @@ -0,0 +1,13 @@ +{ + "source_file": "./raw_volume-zh/volume2/chapter1.tex", + "problem_type": "calculation", + "problem": "例15. 已知函数 $y=f(x)$ 的图象如图() 所示.\n(1) 写出 $y=f(x)$ 的解析式;\n(2) 求 $y=f(2 x)$ 的解析式,并作出 $y=f(2 x)$ 的图象;\n(3) 求 $y=f(2 x-1)$ 的解析式,并作出 $y=f(2 x-1)$ 的图象.", + "solution": "解:(1) 由图 (), 可知 $f(x)=|x|(-1 \\leqslant x \\leqslant 1)$.\n(2) 由第 (1) 小题, 知 $f(2 x)=|2 x|=2|x|$, 注意到 $-1 \\leqslant 2 x \\leqslant 1$, 所以\n$$\nf(2 x)=2|x|\\left(-\\frac{1}{2} \\leqslant x \\leqslant \\frac{1}{2}\\right) .\n$$\n其图象如图 ()所示.\n(3) 同上, 可得 $f(2 x-1)=|2 x-1|(0 \\leqslant x \\leqslant 1)$, 其图象如图()所示.", + "remark": "", + "figures": [ + "./images/volume2/figures/fig-c1e15-1.png", + "./images/volume2/figures/fig-c1e15-1.png", + "./images/volume2/figures/fig-c1e15-2.png", + "./images/volume2/figures/fig-c1e15-3.png" + ] +} \ No newline at end of file diff --git a/processed_dataset/calculation/0484.json b/processed_dataset/calculation/0484.json new file mode 100644 index 0000000000000000000000000000000000000000..7be43f3a9949b61df21716533385a5ae5b1dc4a0 --- /dev/null +++ b/processed_dataset/calculation/0484.json @@ -0,0 +1,8 @@ +{ + "source_file": "./raw_volume-zh/volume2/chapter1.tex", + "problem_type": "calculation", + "problem": "例16 设 $f(x)=x^2+a x+b \\cos x,\\{x \\mid f(x)=0, x \\in \\mathbf{R}\\}=\\{x \\mid f(f(x))=0, x \\in \\mathbf{R}\\} \\neq \\varnothing$, 求满足条件的所有实数 $a, b$ 的值.", + "solution": "解:设 $x_0 \\in\\{x \\mid f(x)=0, x \\in \\mathbf{R}\\}$, 则 $b=f(0)=f\\left(f\\left(x_0\\right)\\right)=0$.\n于是 $f(x)=x(x+a)$. 故\n$$\nf(f(x))=f(x)(f(x)+a)=x(x+a)\\left(x^2+a x+a\\right) .\n$$\n显然, $a=0$ 满足题意.\n若 $a \\neq 0$, 由于 $x^2+a x+a=0$ 的根不可能是 0 或者 $-a$, 故 $x^2+a x+a=0$ 没有实数根, 于是 $\\Delta=a^2-4 a<0$, 所以 $00$, 又因为 $x_1^2-x_2^2>0$, 故只要\n$$\nx_1^2+x_2^2+2-\\lambda>0 \\text {. }\n$$\n所以\n$$\n\\lambda3 \\text {. }\n$$\n因此只要 $\\lambda \\leqslant 3, F(x)$ 在 $\\left(-\\infty,-\\frac{\\sqrt{2}}{2}\\right)$ 内是减函数.\n同理, 当 $\\lambda \\geqslant 3$ 时, $F(x)$ 在 $\\left(-\\frac{\\sqrt{2}}{2}, 0\\right)$ 内是增函数.\n综上讨论,存在唯一的实数 $\\lambda=3$, 使得对应的 $F(x)$ 满足要求.", + "remark": "", + "figures": [] +} \ No newline at end of file diff --git a/processed_dataset/calculation/0489.json b/processed_dataset/calculation/0489.json new file mode 100644 index 0000000000000000000000000000000000000000..d22cf80968146ef1f6b95599d33ca235ba241afd --- /dev/null +++ b/processed_dataset/calculation/0489.json @@ -0,0 +1,8 @@ +{ + "source_file": "./raw_volume-zh/volume2/chapter2.tex", + "problem_type": "calculation", + "problem": "例6 已知 $x, y \\in\\left[-\\frac{\\pi}{4}, \\frac{\\pi}{4}\\right], a \\in \\mathbf{R}$, 且 $\\left\\{\\begin{array}{l}x^3+\\sin x-2 a=0, \\\\ 4 y^3+\\sin y \\cos y+a=0 .\\end{array}\\right.$ 求 $\\cos (x+2 y)$ 的值.", + "solution": "分析:此题的特点就是人口非常小, 所求的 $\\cos (x+2 y)$ 的值好像与题设条件没有什么关系.\n我们再对方程组中的三个变量 $x, y, a$ 的系数进行观察,\n大胆想像, 从中利用立方和公式、倍角公式、 $t^3+\\sin t$ 在 $\\left[-\\frac{\\pi}{2}, \\frac{\\pi}{2}\\right]$ 的单调性, 就能找到一条通向胜利之路.\n解由于\n$$\n\\left\\{\\begin{array}{l}\nx^3+\\sin x-2 a=0, \\\\\n4 y^3+\\sin y \\cos y+a=0 .\n\\end{array}\\right.\n$$\n将第二式乘以 2 与第一式相加并整理, 得\n$$\nx^3+\\sin x=(-2 y)^3+\\sin (-2 y) .\n$$\n已知 $x, y \\in\\left[-\\frac{\\pi}{4}, \\frac{\\pi}{4}\\right]$,所以 $x,-2 y \\in\\left[-\\frac{\\pi}{2}, \\frac{\\pi}{2}\\right]$.\n构造函数 $f(t)=t^3+\\sin t, t \\in\\left[-\\frac{\\pi}{2}, \\frac{\\pi}{2}\\right]$. 则由 $f(t)=t^3+\\sin t$ 的单调性可知 $x=-2 y$, 所以 $x+2 y=0$.\n于是 $\\cos (x+2 y)=1$.\n评注这是一道经典的好题.\n好在它既能考查学生基础知识的掌握程度, 又能考查学生对基础知识的灵活应用能力, 还能考查学生对各数学分支的基础知识的综合整合能力.\n其中涉及的知识点有代数公式, 方程变形, 三角公式, 函数单调性等等.", + "remark": "", + "figures": [] +} \ No newline at end of file diff --git a/processed_dataset/calculation/0490.json b/processed_dataset/calculation/0490.json new file mode 100644 index 0000000000000000000000000000000000000000..cf571961ac2ab0bb0972234b111400222eefa313 --- /dev/null +++ b/processed_dataset/calculation/0490.json @@ -0,0 +1,8 @@ +{ + "source_file": "./raw_volume-zh/volume2/chapter2.tex", + "problem_type": "calculation", + "problem": "例7 已知函数 $f(x)=\\frac{\\sin (\\pi x)-\\cos (\\pi x)+2}{\\sqrt{x}}\\left(\\frac{1}{4} \\leqslant x \\leqslant \\frac{5}{4}\\right)$, 求 $f(x)$ 的最小值.", + "solution": "解:因为 $f(x)=\\frac{\\sqrt{2} \\sin \\left(\\pi x-\\frac{\\pi}{4}\\right)+2}{\\sqrt{x}}\\left(\\frac{1}{4} \\leqslant x \\leqslant \\frac{5}{4}\\right)$, 故可设 $g(x)= \\sqrt{2} \\sin \\left(\\pi x-\\frac{\\pi}{4}\\right)\\left(\\frac{1}{4} \\leqslant x \\leqslant \\frac{5}{4}\\right)$, 则 $g(x) \\geqslant 0, g(x)$ 在 $\\left[\\frac{1}{4}, \\frac{3}{4}\\right]$ 上是增函数, 在 $\\left[\\frac{3}{4}, \\frac{5}{4}\\right]$ 上是减函数, 且 $y=g(x)$ 的图象关于直线 $x=\\frac{3}{4}$ 对称.\n那么对任意 $x_1 \\in\\left[-\\frac{1}{4}, \\frac{3}{4}\\right]$, 存在 $x_2 \\in\\left[\\frac{3}{4}, \\frac{5}{4}\\right]$, 使得 $g\\left(x_2\\right)=g\\left(x_1\\right)$. 于是\n$$\n\\begin{aligned}\nf\\left(x_1\\right) & =\\frac{g\\left(x_1\\right)+2}{\\sqrt{x_1}}=\\frac{g\\left(x_2\\right)+2}{\\sqrt{x_1}} \\\\\n& \\geqslant \\frac{g\\left(x_2\\right)+2}{\\sqrt{x_2}}=f\\left(x_2\\right),\n\\end{aligned}\n$$\n而 $f(x)$ 在 $\\left[\\frac{3}{4}, \\frac{5}{4}\\right]$ 上是减函数, 所以 $f(x) \\geqslant f\\left(\\frac{5}{4}\\right)=\\frac{4 \\sqrt{5}}{5}$, 即 $f(x)$ 在\n$\\left[\\frac{1}{4}, \\frac{5}{4}\\right]$ 上的最小值是 $\\frac{4 \\sqrt{5}}{5}$.", + "remark": "", + "figures": [] +} \ No newline at end of file diff --git a/processed_dataset/calculation/0491.json b/processed_dataset/calculation/0491.json new file mode 100644 index 0000000000000000000000000000000000000000..0bc5a000b81f5ea688ed7afa974c91914882ec3a --- /dev/null +++ b/processed_dataset/calculation/0491.json @@ -0,0 +1,8 @@ +{ + "source_file": "./raw_volume-zh/volume2/chapter2.tex", + "problem_type": "calculation", + "problem": "例14 在 $\\mathbf{R}$ 上是否存在非常数的周期函数, 它没有最小正周期?", + "solution": "解:答案是肯定的.\n定义函数\n$$\nD(x)= \\begin{cases}1, & \\text { 当 } x \\text { 是有理数时, } \\\\ 0, & \\text { 当 } x \\text { 是无理数时.\n}\\end{cases}\n$$\n显然, 任何正的有理数都是 $D(x)$ 的周期, 因此 $D(x)$ 没有最小正周期.\n函数 $D(x)$ 称为狄利克莱函数, 在高等数学中还会遇到它.", + "remark": "", + "figures": [] +} \ No newline at end of file diff --git a/processed_dataset/calculation/0492.json b/processed_dataset/calculation/0492.json new file mode 100644 index 0000000000000000000000000000000000000000..01330d3d311bfeeac1bd5280c1f95b1c0b2862f3 --- /dev/null +++ b/processed_dataset/calculation/0492.json @@ -0,0 +1,8 @@ +{ + "source_file": "./raw_volume-zh/volume2/chapter3.tex", + "problem_type": "calculation", + "problem": "例1 设二次函数 $f(x)$ 满足 $f(x-2)=f(-x-2), x \\in \\mathbf{R}$, 且 $f(x)$ 的图象在 $y$ 轴上的截距为 1 , 在 $x$ 轴上截得的线段长为 $2 \\sqrt{2}$, 求 $f(x)$ 的解析式.", + "solution": "分析:由 $f(x-2)=f(-x-2)$ 知, $f(x)$ 的图象关于 $x=-2$ 对称.\n解由题设知, $f(x)$ 的图象有对称轴 $x=-2$, 由 $f(x)$ 的图象在 $x$ 轴上截得的线段长为 $2 \\sqrt{2}$, 可得 $f(x)$ 的图象与 $x$ 轴的交点为 $(-2-\\sqrt{2}, 0),(-2+\\sqrt{2}, 0)$,于是可设\n$$\nf(x)=a(x+2+\\sqrt{2})(x+2-\\sqrt{2}) .\n$$\n又 $f(0)=1$, 所以\n$$\na(2+\\sqrt{2})(2-\\sqrt{2})=1 .\n$$\n解方程, 得 $a=\\frac{1}{2}$.\n所以\n$$\nf(x)=\\frac{1}{2}(x+2+\\sqrt{2})(x+2-\\sqrt{2}),\n$$\n即\n$$\nf(x)=\\frac{1}{2} x^2+2 x+1 .\n$$", + "remark": "", + "figures": [] +} \ No newline at end of file diff --git a/processed_dataset/calculation/0493.json b/processed_dataset/calculation/0493.json new file mode 100644 index 0000000000000000000000000000000000000000..9d4e99a7925b4308bcf000158ab8c52c460a499b --- /dev/null +++ b/processed_dataset/calculation/0493.json @@ -0,0 +1,8 @@ +{ + "source_file": "./raw_volume-zh/volume2/chapter3.tex", + "problem_type": "calculation", + "problem": "例2 若函数 $f(x)=-\\frac{1}{2} x^2+\\frac{13}{2}$ 在区间 $[a, b]$ 上的最小值为 $2 a$, 最大值为 $2 b$, 求 $[a, b]$.", + "solution": "解:分如下三种情形来讨论区间 $[a, b]$.\n(1) 当 $a0 .\n$$\n而 $a<0$, 故 $f(b) \\neq 2 a$, 所以 $f(a)=2 a$, 即\n$$\n-\\frac{1}{2} a^2+\\frac{13}{2}=2 a \\text {. }\n$$\n解方程, 得 $a=-2-\\sqrt{17}$. 此时 $[a, b]=\\left[-2-\\sqrt{17}, \\frac{13}{4}\\right]$.\n(3). 当 $0 \\leqslant a0)$, 方程 $f(x)=x$ 的两个根是 $x_1$ 和 $x_2$, 且 $x_1>0, x_2-x_1>\\frac{1}{a}$. 又若 $00, t-x_1<0$, 得 $f(t)-x_1>0$.\n所以,当 $0x_1$.", + "remark": "", + "figures": [] +} \ No newline at end of file diff --git a/processed_dataset/calculation/0495.json b/processed_dataset/calculation/0495.json new file mode 100644 index 0000000000000000000000000000000000000000..9a8aad5ff72521bdb21d875d184470a550433150 --- /dev/null +++ b/processed_dataset/calculation/0495.json @@ -0,0 +1,8 @@ +{ + "source_file": "./raw_volume-zh/volume2/chapter3.tex", + "problem_type": "calculation", + "problem": "例4 若关于 $x$ 的方程 $3 x^2-5 x+a=0$ 的一根大于 -2 而小于 0 , 另一根大于 1 小于 3 , 求 $a$ 的取值范围.", + "solution": "分析:此题由于抛物线开口向上, 故只要画出一个草图就能发现: 当 $f(-2)>0, f(0)<0, f(1)<0, f(3)>0$ 时就能使方程的根满足题设条件, 然而这些条件是必需的吗? 对这个问题的分析就要用到二次函数的对称轴了.\n此外本题也可采用更为自然的方法, 即利用求根公式将两根用关于 $a$ 的代数式表示出来, 再利用两根的分布条件及判别式非负转化为关于 $a$ 的不等式.\n解法一由于二次函数 $y=3 x^2-5 x+a=0$ 的开口向上, 而二次方程 $y=3 x^2-5 x+a=0$ 如有两个不相等的实根, 必位于二次函数 $y=3 x^2-5 x+a=0$ 的对称轴的两侧.\n对称轴左侧, 函数递减; 对称轴右侧, 函数递增.\n所以必须有\n$$\n\\left\\{\\begin{array}{l}\nf(-2)>0, \\\\\nf(0)<0, \\\\\nf(1)<0, \\\\\nf(3)>0 .\n\\end{array}\\right.\n$$\n反过来在这种情况下, 此方程必有两满足题设的相异实根.\n解此不等式组得 $-121)$, 使得存在 $t \\in \\mathbf{R}$, 只要 $x \\in[1, m]$, 就有 $f(x+t) \\leqslant x$.", + "solution": "分析:先根据题设条件 (1)、(2)、(3), 把 $f(x)$ 的解析式求出来, 进而再确定 $m$ 的最大值.\n解由 $f(x-4)=f(2-x), x \\in \\mathbf{R}$, 可知二次函数 $f(x)$ 的对称轴为 $x=-1$. 又由 (3) 知, 二次函数 $f(x)$ 的开口向上, 即 $a>0$, 故可设\n$$\nf(x)=a(x+1)^2,(a>0) .\n$$\n由 (1) 知, $f(1) \\geqslant 1$, 由 (2) 知, $f(1) \\leqslant\\left(\\frac{1+1}{2}\\right)^2=1$.\n所以 $f(1)=1$, 故\n$$\n1=a(1+1)^2, a=\\frac{1}{4} .\n$$\n所以\n$$\nf(x)=\\frac{1}{4}(x+1)^2 .\n$$\n因为 $f(x)=\\frac{1}{4}(x+1)^2$ 的图象开口向上, 而 $y=f(x+t)$ 的图象是由 $y=f(x)$ 的图象平移 $|t|$ 个单位得到.\n要在区间 $[1, m]$ 上, 使得 $y=f(x+t)$ 的图象在 $y=x$ 的图象的下方, 且 $m$ 最大, 则 1 和 $m$ 应当是关于 $x$ 的方程\n$$\n\\frac{1}{4}(x+t+1)^2=x\n$$\n的两个根.\n令 $x=1$ 代入方程(1), 得 $t=0$ 或 $t=-4$.\n当 $t=0$ 时,方程(1)的解为 $x_1=x_2=1$ (这与 $m>1$ 矛盾!);\n当 $t=-4$ 时,方程(1)的解为 $x_1=1, x_2=9$, 所以 $m=9$.\n又当 $t=-4$ 时,对任意 $x \\in[1,9]$, 恒有\n$$\n\\begin{aligned}\n& (x-1)(x-9) \\leqslant 0 \\\\\n\\Leftrightarrow & \\frac{1}{4}(x-4+1)^2 \\leqslant x,\n\\end{aligned}\n$$\n即 $f(x-4) \\leqslant x$.\n所以, $m$ 的最大值为 9 .\n说明我们由 $f(x-4)=f(2-x), x \\in \\mathbf{R}$ 导出 $f(x)$ 的图象关于 $x=$ -1 对称.\n一般地, 若 $f(x-a)=f(b-x), x \\in \\mathbf{R}$, 则\n$$\nf\\left(x+\\frac{b-a}{2}\\right)=f\\left(x+\\frac{b+a}{2}-a\\right)=f\\left(b-x-\\frac{b+a}{2}\\right)\n$$\n$$\n=f\\left(\\frac{b-a}{2}-x\\right)\n$$\n故 $f(x)$ 的图象关于 $x=\\frac{b-a}{2}$ 对称.\n这个性质在解题中常常用到.", + "remark": "", + "figures": [] +} \ No newline at end of file diff --git a/processed_dataset/calculation/0498.json b/processed_dataset/calculation/0498.json new file mode 100644 index 0000000000000000000000000000000000000000..ca23820537752e0e90806c63b78bc7f40f47179e --- /dev/null +++ b/processed_dataset/calculation/0498.json @@ -0,0 +1,8 @@ +{ + "source_file": "./raw_volume-zh/volume2/chapter3.tex", + "problem_type": "calculation", + "problem": "例7 函数 $f(x)=\\log _{\\frac{1}{2}}\\left(x^2-2 x-3\\right)$ 的单调递增区间是 $(\\quad)$.\nA. $(-\\infty,-1)$\nB. $(-\\infty, 1)$\nC. $(1,+\\infty)$\nD. $(3,+\\infty)$", + "solution": "解:由 $x^2-2 x-3>0$, 得函数的定义域为 $(-\\infty,-1) \\cup(3,+\\infty)$.\n而 $u=x^2-2 x-3=(x-1)^2-4$ 在 $(-\\infty,-1)$ 上单调递减, 在 $(3,+\\infty)$ 上单调递增, 所以, $f(x)=\\log _{\\frac{1}{2}}\\left(x^2-2 x-3\\right)$ 在 $(-\\infty,-1)$ 上单调递增, 在 $(3,+\\infty)$ 上单调递减, 故应选 $\\mathrm{A}$.", + "remark": "", + "figures": [] +} \ No newline at end of file diff --git a/processed_dataset/calculation/0499.json b/processed_dataset/calculation/0499.json new file mode 100644 index 0000000000000000000000000000000000000000..f30daaa54c5405fd350b66c8623d5596608e4673 --- /dev/null +++ b/processed_dataset/calculation/0499.json @@ -0,0 +1,8 @@ +{ + "source_file": "./raw_volume-zh/volume2/chapter3.tex", + "problem_type": "calculation", + "problem": "例9 若函数 $g(x)$ 的图象与函数 $f(x)=\\frac{1-2^x}{1+2^x}$ 的图象关于直线 $y=x$ 对称, 求 $g\\left(\\frac{3}{5}\\right)$ 的值.", + "solution": "解:法一先求出 $g(x)$ 的解析式.\n因为\n$$\n\\begin{gathered}\nf(x)=\\frac{1-2^x}{1+2^x}, \\\\\n2^x=\\frac{1-f(x)}{1+f(x)}, \\\\\nx=\\log _2 \\frac{1-f(x)}{1+f(x)} .\n\\end{gathered}\n$$\n所以\n$$\n2^x=\\frac{1-f(x)}{1+f(x)}\n$$\n因此 $g(x)=f^{-1}(x)=\\log _2 \\frac{1-x}{1+x},(-10$.\n所以 $\\left\\{\\begin{array}{l}a \\leqslant 2, \\\\ g(2)=4-4 a+5 a-1>0\\end{array}\\right.$ 或 $\\left\\{\\begin{array}{l}a \\geqslant 4, \\\\ g(4)=16-8 a+5 a-1>0 .\\end{array}\\right.$\n解得 $-32$, 则当 $t=2$ 时, $g[f(x)]$ 取得最小值 $a+3$.", + "remark": "", + "figures": [] +} \ No newline at end of file diff --git a/processed_dataset/calculation/0501.json b/processed_dataset/calculation/0501.json new file mode 100644 index 0000000000000000000000000000000000000000..e1745a84d985d7eefc518c230f8724c661d3451a --- /dev/null +++ b/processed_dataset/calculation/0501.json @@ -0,0 +1,8 @@ +{ + "source_file": "./raw_volume-zh/volume2/chapter3.tex", + "problem_type": "calculation", + "problem": "例12 设 $f(x)=\\lg \\frac{1+2^x+4^x a}{3}$, 其中 $a \\in \\mathbf{R}$. 当 $x \\in(-\\infty, 1]$ 时, $f(x)$有意义,求 $a$ 的取值范围.", + "solution": "分析:首先需理解题意, 当 $x \\in(-\\infty, 1]$ 时, $f(x)$ 有意义, 说明当 $x \\leqslant 1$ 时, 恒有 $\\frac{1+2^x+4^x a}{3}>0$, 即\n$$\na>-\\left[\\left(\\frac{1}{4}\\right)^x+\\left(\\frac{1}{2}\\right)^x\\right]\n$$\n对 $x \\leqslant 1$ 恒成立, 故 $a$ 就大于 $-\\left[\\left(\\frac{1}{4}\\right)^x+\\left(\\frac{1}{2}\\right)^x\\right](x \\in(-\\infty, 1])$ 的最大值.\n解由题意知, 当 $x \\leqslant 1$ 时, 恒有\n$$\n\\frac{1+2^x+4^x a}{3}>0 .\n$$\n故\n$$\na>-\\left[\\left(\\frac{1}{4}\\right)^x+\\left(\\frac{1}{2}\\right)^x\\right], x \\leqslant 1 .\n$$\n令 $u=-\\left[\\left(\\frac{1}{4}\\right)^x+\\left(\\frac{1}{2}\\right)^x\\right], x \\leqslant 1$, 则\n$$\nu=-\\left[\\left(\\frac{1}{2}\\right)^x+\\frac{1}{2}\\right]^2+\\frac{1}{4}\n$$\n在 $(-\\infty, 1]$ 上单调递增,所以,当 $x=1$ 时,有\n$$\nu_{\\max }=-\\frac{3}{4} \\text {. }\n$$\n所以 $a$ 的取值范围为 $\\left(-\\frac{3}{4},+\\infty\\right)$.", + "remark": "", + "figures": [] +} \ No newline at end of file diff --git a/processed_dataset/calculation/0502.json b/processed_dataset/calculation/0502.json new file mode 100644 index 0000000000000000000000000000000000000000..66911ce5e62461106564ba9dbbc8a3da446a7f56 --- /dev/null +++ b/processed_dataset/calculation/0502.json @@ -0,0 +1,8 @@ +{ + "source_file": "./raw_volume-zh/volume2/chapter3.tex", + "problem_type": "calculation", + "problem": "例13 求函数 $f(x)=\\frac{x-1}{x^2-2 x+5}, \\frac{3}{2} \\leqslant x \\leqslant 2$ 的最大值和最小值.", + "solution": "解:因为 $x \\neq 1$, 所以\n$$\nf(x)=\\frac{x-1}{(x-1)^2+4}=\\frac{1}{x-1+\\frac{4}{x-1}}, x \\in\\left[\\frac{3}{2}, 2\\right] .\n$$\n令 $g(t)=t+\\frac{4}{t}, t \\in\\left[\\frac{1}{2}, 1\\right]$, 则 $g(t)$ 在 $\\left[\\frac{1}{2}, 1\\right]$ 上是减函数, 所以\n$$\n\\begin{gathered}\ng_{\\min }(t)=g(1)=5, \\\\\ng_{\\max }(t)=g\\left(\\frac{1}{2}\\right)=\\frac{17}{2} .\n\\end{gathered}\n$$\n所以, $f(x)$ 的最大值为 $f\\left(\\frac{3}{2}\\right)=\\frac{1}{5}, f(x)$ 的最小值为 $f(2)=\\frac{2}{17}$.", + "remark": "", + "figures": [] +} \ No newline at end of file diff --git a/processed_dataset/calculation/0503.json b/processed_dataset/calculation/0503.json new file mode 100644 index 0000000000000000000000000000000000000000..9fcc0d69e5fa18d6229e84654a729a573d74e611 --- /dev/null +++ b/processed_dataset/calculation/0503.json @@ -0,0 +1,8 @@ +{ + "source_file": "./raw_volume-zh/volume2/chapter3.tex", + "problem_type": "calculation", + "problem": "例14 设 $x, y \\in \\mathbf{R}^{+}, x+y=c, c$ 为常数且 $c \\in(0,2]$, 求 $u=\\left(x+\\frac{1}{x}\\right)\\left(y+\\frac{1}{y}\\right)$ 的最小值.", + "solution": "解:$u=\\left(x+\\frac{1}{x}\\right)\\left(y+\\frac{1}{y}\\right)=x y+\\frac{1}{x y}+\\frac{x}{y}+\\frac{y}{x} \\geqslant x y+\\frac{1}{x y}+2$.\n令 $x y=t$, 则 $0g(\\theta)$ (或 $ag_{\\max }(\\theta)$ (或 $a0,\n\\end{gathered}\n$$\n$$\n2 x\\left(x+\\frac{2}{x}-a\\right)-3\\left(x+\\frac{2}{x}-a\\right)>0,\n$$\n即\n$$\n(2 x-3)\\left(x+\\frac{2}{x}-a\\right)>0 \\text {. }\n$$\n因为 $x \\in[1, \\sqrt{2}]$, 所以 $2 x-3<0$, 故\n$$\nx+\\frac{2}{x}-a<0 \\text {. }\n$$\n所以 $a>x+\\frac{2}{x}, x \\in[1, \\sqrt{2}]$ 恒成立.\n令 $f(x)=x+\\frac{2}{x}, x \\in[1, \\sqrt{2}]$, 由于 $f(x)$ 在 $[1, \\sqrt{2}]$ 上单调递减, 所以\n$$\n\\begin{gathered}\nf_{\\max }(x)=f(1)=3, \\\\\na>f_{\\max }(x)=3 .\n\\end{gathered}\n$$\n所以 $a$ 的取值范围为 $(3,+\\infty)$.", + "remark": "", + "figures": [] +} \ No newline at end of file diff --git a/processed_dataset/calculation/0506.json b/processed_dataset/calculation/0506.json new file mode 100644 index 0000000000000000000000000000000000000000..cfc49fe3356290b5a4e3185f38bf5e7bb684f54f --- /dev/null +++ b/processed_dataset/calculation/0506.json @@ -0,0 +1,8 @@ +{ + "source_file": "./raw_volume-zh/volume2/chapter4.tex", + "problem_type": "calculation", + "problem": "例1 设 $x, y \\in \\mathbf{R}$, 求 $u=x^2+x y+y^2-x-2 y+3$ 的最小值.", + "solution": "分析:把 $u$ 写成若干个非负数的和再加上一个常数, 从而 $u$ 就大于等于这个常数,再说明当 $x, y$ 取某值时, $u$ 可以取到这个常数即可.\n解 $u=x^2+(y-1) x+y^2-2 y+3$\n$$\n=\\left[x^2+(y-1) x+\\frac{(y-1)^2}{4}\\right]+y^2-2 y+3-\\frac{(y-1)^2}{4}\n$$\n$$\n\\begin{aligned}\n& =\\left(x+\\frac{y-1}{2}\\right)^2+\\frac{3}{4}\\left(y^2-2 y+1\\right)+2 \\\\\n& =\\left(x+\\frac{y-1}{2}\\right)^2+\\frac{3}{4}(y-1)^2+2 \\geqslant 2,\n\\end{aligned}\n$$\n又当 $x=0, y=1$ 时等号成立, 所以 $u$ 的最小值为 2 .\n说明在估计 $u$ 的下界时, 进行了配方, 需要注意的是这些非负项是可以同时达到 0 的.", + "remark": "", + "figures": [] +} \ No newline at end of file diff --git a/processed_dataset/calculation/0507.json b/processed_dataset/calculation/0507.json new file mode 100644 index 0000000000000000000000000000000000000000..527da23fea5c7ed687408790944c2370c0773095 --- /dev/null +++ b/processed_dataset/calculation/0507.json @@ -0,0 +1,8 @@ +{ + "source_file": "./raw_volume-zh/volume2/chapter4.tex", + "problem_type": "calculation", + "problem": "例2 设 $x \\in \\mathbf{R}^{+}$, 求函数 $f(x)=x^2-x+\\frac{1}{x}$ 的最小值.", + "solution": "分析:先估计 $f(x)$ 的下界, 再说明这个下界是可以达到的.\n解\n$$\n\\begin{aligned}\nf(x) & =\\left(x^2-2 x+1\\right)+\\left(x+\\frac{1}{x}-2\\right)+1 \\\\\n& =(x-1)^2+\\left(\\sqrt{x}-\\frac{1}{\\sqrt{x}}\\right)^2+1 \\geqslant 1,\n\\end{aligned}\n$$\n又当 $x=1$ 时, $f(x)=1$, 所以 $f(x)$ 的最小值为 1 .\n说明在求最大(小)值时, 估计了上 (下) 界后, 一定要举例说明这个界是可以取到的, 才能说这就是最大 (小) 值, 否则就不一定对了.\n例如, 本题我们也可以这样估计:\n$$\n\\begin{aligned}\nf(x) & =\\left(x^2-2 x+1\\right)+\\left(x+\\frac{1}{x}+2\\right)-3 \\\\\n& =(x-1)^2+\\left(\\sqrt{x}+\\frac{1}{\\sqrt{x}}\\right)^2-3 \\geqslant-3,\n\\end{aligned}\n$$\n但无论 $x$ 取何值时, $f(x) \\neq-3$, 即一 3 不能作为 $f(x)$ 的最小值.", + "remark": "", + "figures": [] +} \ No newline at end of file diff --git a/processed_dataset/calculation/0508.json b/processed_dataset/calculation/0508.json new file mode 100644 index 0000000000000000000000000000000000000000..b85ca0b0dfdbbc7cdd9b656b773c228cb5263185 --- /dev/null +++ b/processed_dataset/calculation/0508.json @@ -0,0 +1,11 @@ +{ + "source_file": "./raw_volume-zh/volume2/chapter4.tex", + "problem_type": "calculation", + "problem": "例3 试求函数 $f(x)=(x+1)(x+2)(x+3)(x+4)+5$ 在闭区间 $[-3,3]$ 上的最大值与最小值.", + "solution": "解:令 $t=x^2+5 x$, 则\n$$\n\\begin{aligned}\nf(x) & =\\left(x^2+5 x+4\\right)\\left(x^2+5 x+6\\right)+5 \\\\\n& =(t+4)(t+6)+5 \\\\\n& =t^2+10 t+29 .\n\\end{aligned}\n$$\n当 $x \\in[-3,3]$ 时, $t$ 的取值范围是 $\\left[-\\frac{25}{4}, 24\\right]$, 如图()所示.\n于是原题转化为在 $\\left[-\\frac{25}{4}, 24\\right]$ 内求二次函数 $f(t)=t^2+10 t+29$ 的最大值和最小值.\n因 $f(t)=(t+5)^2+4$, 又 $-5 \\in\\left[-\\frac{25}{4}, 24\\right]$, 故当 $t=-5$ 时, $f_{\\text {min }}=4$. 而由 $-5=x^2+5 x$ 解得 $x_{1,2}=\\frac{-5 \\pm \\sqrt{5}}{2}$, 但 $\\frac{-5-\\sqrt{5}}{2} \\notin[-3,3]$, 故当 $x= \\frac{-5+\\sqrt{5}}{2}$ 时, $f_{\\text {min }}=4$. 如图()所示.\n当 $t=-\\frac{25}{4}$ 时, $f(t)=5 \\frac{9}{16}$; 当 $t=24$ 时, $f(t)=845$. 而 $845>5 \\frac{9}{16}$, 所以当 $t=24$, 即 $x=3$ 时, $f_{\\text {max }}=845$.\n说明一个复杂的函数式, 如能写成二次函数型的复合函数, 即 $f(x)= a g^2(x)+b g(x)+c$ ( $a 、 b 、 c$ 为常数), 此时用配方法求函数的最值问题往往是行之有效的.", + "remark": "", + "figures": [ + "./images/volume2/figures/fig-c4e3-1.png", + "./images/volume2/figures/fig-c4e3-2.png" + ] +} \ No newline at end of file diff --git a/processed_dataset/calculation/0509.json b/processed_dataset/calculation/0509.json new file mode 100644 index 0000000000000000000000000000000000000000..8c943646d2ddef913de1325aeefcda8c6f28eaaf --- /dev/null +++ b/processed_dataset/calculation/0509.json @@ -0,0 +1,8 @@ +{ + "source_file": "./raw_volume-zh/volume2/chapter4.tex", + "problem_type": "calculation", + "problem": "例4 求函数 $y=\\frac{x^2-2 x-3}{2 x^2+2 x+1}$ 的最大值和最小值.", + "solution": "分析:把它写成关于 $x$ 的二次方程形式,利用 $\\Delta \\geqslant 0$ 来得到 $y$ 的范围.\n解去分母并整理, 得\n$$\n(2 y-1) x^2+2(y+1) x+(y+3)=0 .\n$$\n当 $y=\\frac{1}{2}$ 时, $x=-\\frac{7}{6}$.\n当 $y \\neq \\frac{1}{2}$ 时, 这是一个关于 $x$ 的二次方程, 由 $x \\in \\mathbf{R}$, 所以\n$$\n\\Delta=[2(y+1)]^2-4(2 y-1)(y+3) \\geqslant 0 .\n$$\n解方程, 得 $-4 \\leqslant y \\leqslant 1$.\n当 $y=-4$ 时, $x=-\\frac{1}{3}$; 当 $y=1$ 时, $x=-2$.\n由此即知, 当 $x=-\\frac{1}{3}$ 时, $y$ 取最小值 -4 ; 当 $x=-2$ 时, $y$ 取最大值 1 .\n说明在用判别式法求最值时, 应特别注意这个最值能否取到, 即是否有与最值相应的 $x$ 值.", + "remark": "", + "figures": [] +} \ No newline at end of file diff --git a/processed_dataset/calculation/0510.json b/processed_dataset/calculation/0510.json new file mode 100644 index 0000000000000000000000000000000000000000..3583031db8497ffa0e6c7ba29fd40135e8e06918 --- /dev/null +++ b/processed_dataset/calculation/0510.json @@ -0,0 +1,8 @@ +{ + "source_file": "./raw_volume-zh/volume2/chapter4.tex", + "problem_type": "calculation", + "problem": "例5 函数 $y=\\frac{a x^2+3 x+b}{x^2+1}$ 的最大值为 $5 \\frac{1}{2}$, 最小值为 $\\frac{1}{2}$. 求 $a, b$ 的值.", + "solution": "解:将原式化为\n$$\n\\begin{aligned}\n& (a-y) x^2+3 x+(b-y)=0 . \\\\\n& \\Delta=9-4(a-y)(b-y) \\geqslant 0, \\\\\n& 4 y^2-4(a+b) y+4 a b-9 \\leqslant 0 .\n\\end{aligned}\n$$\n即显然 $y$ 的值在此不等式所对应的二次方程的两根之间, 根据求根公式,有\n$$\n\\begin{gathered}\n\\left\\{\\begin{array}{l}\na+b+\\sqrt{(a-b)^2+9}=11, \\\\\na+b-\\sqrt{(a-b)^2+9}=1\n\\end{array}\\right. \\\\\n\\Rightarrow\\left\\{\\begin{array} { l } \n{ a + b = 6 , } \\\\\n{ a - b = \\pm 4 }\n\\end{array} \\Rightarrow \\left\\{\\begin{array} { l } \n{ a = 5 , } \\\\\n{ b = 1 }\n\\end{array} \\text { 或 } \\left\\{\\begin{array}{l}\na=1, \\\\\nb=5 .\n\\end{array}\\right.\\right.\\right.\n\\end{gathered}\n$$\n以上两组都是满足题设的解.\n说明本题为判别式法的一种经典应用, 希望读者能熟练掌握.\n从严格意义上来说,此处的判别式法在应用时还必须考虑当 $a-y=0$ 的特殊情形, 可以推出, 此时 $b=a$, 进一步推下去 (用函数 $f(x)=x+\\frac{1}{x}$ 的值域的结论) 可以发现, 这种情形无法满足题设.\n虽然如此, 但这一点也必须考虑到.", + "remark": "", + "figures": [] +} \ No newline at end of file diff --git a/processed_dataset/calculation/0511.json b/processed_dataset/calculation/0511.json new file mode 100644 index 0000000000000000000000000000000000000000..c75b85f1407ef59013c9be0148c0817e159f5b81 --- /dev/null +++ b/processed_dataset/calculation/0511.json @@ -0,0 +1,8 @@ +{ + "source_file": "./raw_volume-zh/volume2/chapter4.tex", + "problem_type": "calculation", + "problem": "例6 已知函数 $f(x)=\\log _2(x+1)$, 并且当点 $(x, y)$ 在 $y=f(x)$ 的图象上运动时, 点 $\\left(\\frac{x}{3}, \\frac{y}{2}\\right)$ 在 $y=g(x)$ 的图象上运动.\n求函数 $p(x)=g(x)-f(x)$ 的最大值.", + "solution": "解:因点 $(x, y)$ 在 $y=f(x)$ 的图象上, 故 $y=\\log _2(x+1)$. 又点 $\\left(\\frac{x}{3}, \\frac{y}{2}\\right)$ 在 $y=g(x)$ 的图象上,故 $\\frac{y}{2}=g\\left(\\frac{x}{3}\\right)$. 从而\n$$\ng\\left(\\frac{x}{3}\\right)=\\frac{1}{2} \\log _2(x+1)\n$$\n即 $g(x)=\\frac{1}{2} \\log _2(3 x+1)$, 那么\n$$\n\\begin{aligned}\np(x) & =g(x)-f(x)=\\frac{1}{2} \\log _2(3 x+1)-\\log _2(x+1) \\\\\n& =\\frac{1}{2} \\log _2 u,\n\\end{aligned}\n$$\n其中,\n$$\nu=\\frac{3 x+1}{(x+1)^2}=-\\frac{2}{(x+1)^2}+\\frac{3}{x+1}=-2\\left(\\frac{1}{x+1}-\\frac{3}{4}\\right)^2+\\frac{9}{8} .\n$$\n因此当 $\\frac{1}{x+1}=\\frac{3}{4}$, 即 $x=\\frac{1}{3}$ 时, $u_{\\text {max }}=\\frac{9}{8}$. 从而 $p(x)$ 的最大值为 $\\frac{1}{2} \\log _2 \\frac{9}{8}$.", + "remark": "", + "figures": [] +} \ No newline at end of file diff --git a/processed_dataset/calculation/0512.json b/processed_dataset/calculation/0512.json new file mode 100644 index 0000000000000000000000000000000000000000..6600c796dfa8ba9e741348f3d89dcb2b56dc5a24 --- /dev/null +++ b/processed_dataset/calculation/0512.json @@ -0,0 +1,8 @@ +{ + "source_file": "./raw_volume-zh/volume2/chapter4.tex", + "problem_type": "calculation", + "problem": "例8 求函数 $y=\\frac{2-\\cos x}{4+3 \\cos x}$ 的最大值和最小值.", + "solution": "分析:这类问题我们常可以利用 $|\\cos x| \\leqslant 1,|\\sin x| \\leqslant 1, \\mid a \\sin x+ b \\cos x\\left|=\\sqrt{a^2+b^2}\\right| \\sin (x+\\varphi) \\mid \\leqslant \\sqrt{a^2+b^2}$ 来处理.\n解由 $y=\\frac{2-\\cos x}{4+3 \\cos x}$, 得\n$$\n\\begin{gathered}\n(3 y+1) \\cos x=2-4 y, \\\\\n|3 y+1| \\cdot|\\cos x|=|2-4 y| .\n\\end{gathered}\n$$\n所以\n$$\n|3 y+1| \\geqslant|2-4 y| \\text {, }\n$$\n即\n$$\n9 y^2+6 y+1 \\geqslant 4-16 y+16 y^2 \\text {. }\n$$\n解不等式, 得 $\\frac{1}{7} \\leqslant y \\leqslant 3$.\n当 $\\cos x=1$. 即 $x=2 k \\pi(k \\in \\mathbf{Z})$ 时, $y=\\frac{1}{7}$;\n当 $\\cos x=-1$, 即 $x=(2 k+1) \\pi(k \\in \\mathbf{Z})$ 时, $y=3$.\n所以\n$$\ny_{\\min }=\\frac{1}{7}, y_{\\max }=3 \\text {. }\n$$", + "remark": "", + "figures": [] +} \ No newline at end of file diff --git a/processed_dataset/calculation/0513.json b/processed_dataset/calculation/0513.json new file mode 100644 index 0000000000000000000000000000000000000000..ffe460612e9e9307718c13e0533777fd9362d423 --- /dev/null +++ b/processed_dataset/calculation/0513.json @@ -0,0 +1,8 @@ +{ + "source_file": "./raw_volume-zh/volume2/chapter4.tex", + "problem_type": "calculation", + "problem": "例10 已知 $\\alpha \\in\\left[0, \\frac{\\pi}{2}\\right]$, 求 $y=\\sqrt{5-4 \\sin \\alpha}+\\sin \\alpha$ 的最小值和最大值.", + "solution": "分析:通过变量代换,把 $y$ 表示成二次函数的形式.\n解设 $x=\\sqrt{5-4 \\sin \\alpha}$, 因为 $0 \\leqslant \\sin \\alpha \\leqslant 1$, 所以 $1 \\leqslant x \\leqslant \\sqrt{5}$, 且\n$$\n\\begin{aligned}\n& \\text { i } \\sin \\alpha=\\frac{5-x^2}{4} \\text {, 于是 } \\\\\n& y=x+\\frac{5-x^2}{4}=-\\frac{1}{4}(x-2)^2+\\frac{9}{4}(1 \\leqslant x \\leqslant \\sqrt{5}) .\n\\end{aligned}\n$$\n故当 $x=2$ 时, $y$ 的最大值为 $\\frac{9}{4}$; 当 $x=1$ 时, $y$ 的最小值为 2 .\n说明通过换元, 常常可以把较复杂的形式转化为较简单的形式, 从而使问题得以解决.", + "remark": "", + "figures": [] +} \ No newline at end of file diff --git a/processed_dataset/calculation/0514.json b/processed_dataset/calculation/0514.json new file mode 100644 index 0000000000000000000000000000000000000000..067bfeef4a2e6979a50b10f7541e10401183342f --- /dev/null +++ b/processed_dataset/calculation/0514.json @@ -0,0 +1,8 @@ +{ + "source_file": "./raw_volume-zh/volume2/chapter4.tex", + "problem_type": "calculation", + "problem": "例 11 已知 $x^2+4 y^2=4 x$, 求下列各式的最大值与最小值.\n(1) $u=x^2+y^2$; (2) $v=x+y$.", + "solution": "解:条件式可化为\n$$\n\\frac{(x-2)^2}{4}+y^2=1 .\n$$\n设 $\\left\\{\\begin{array}{l}x=2+2 \\cos \\theta, \\\\ y=\\sin \\theta\\end{array}\\right.$ ( $\\theta$ 为参数), 则\n(1) $u=(2+2 \\cos \\theta)^2+\\sin ^2 \\theta=3\\left(\\cos \\theta+\\frac{4}{3}\\right)^2-\\frac{1}{3}$,\n故\n$$\nu_{\\min }=0, u_{\\max }=16 \\text {. }\n$$\n(2) $v=2+2 \\cos \\theta+\\sin \\theta=\\sqrt{5} \\sin (\\theta+\\varphi)+2$,\n所以\n$$\nv_{\\min }=2-\\sqrt{5}, v_{\\max }=2+\\sqrt{5} \\text {. }\n$$\n说明若 $u^2+v^2=1$, 则令 $u=\\sin \\alpha, v=\\cos \\alpha, \\alpha \\in[0,2 \\pi)$. 若 $u, v \\in \\mathbf{R}^{+}$, 且 $u+v=1$, 则令 $u=\\sin ^2 \\alpha, v=\\cos ^2 \\alpha, \\alpha \\in\\left(0, \\frac{\\pi}{2}\\right)$ 等等.\n这些代换能帮助我们简化问题, 从而解决问题.", + "remark": "", + "figures": [] +} \ No newline at end of file diff --git a/processed_dataset/calculation/0515.json b/processed_dataset/calculation/0515.json new file mode 100644 index 0000000000000000000000000000000000000000..a32d970957018337f83e0ffc7a83e3fff7bb64fa --- /dev/null +++ b/processed_dataset/calculation/0515.json @@ -0,0 +1,8 @@ +{ + "source_file": "./raw_volume-zh/volume2/chapter4.tex", + "problem_type": "calculation", + "problem": "例12 在约束条件 $x \\geqslant 0, y \\geqslant 0$ 及 $3 \\leqslant x+y \\leqslant 5$ 下, 求函数 $u=x^2- x y+y^2$ 的最大值和最小值.", + "solution": "解:令 $x=a \\sin ^2 \\theta, y=a \\cos ^2 \\theta, \\theta \\in[0,2 \\pi)$, 则 $3 \\leqslant a \\leqslant 5$. 于是\n$$\n\\begin{aligned}\n& \\begin{aligned}\nu & =a^2 \\sin ^4 \\theta-a^2 \\sin ^2 \\theta \\cos ^2 \\theta+a^2 \\cos ^4 \\theta \\\\\n& =a^2\\left[\\left(\\sin ^2 \\theta+\\cos ^2 \\theta\\right)^2-3 \\sin ^2 \\theta \\cos ^2 \\theta\\right] \\\\\n& =a^2\\left(1-\\frac{3}{4} \\sin ^2 2 \\theta\\right) .\n\\end{aligned} \\\\\n& \\leqslant 2 \\text {. 所以 } u_{\\text {min }}=\\frac{9}{4}, u_{\\text {max }}=25 .\n\\end{aligned}\n$$\n从而 $\\frac{9}{4} \\leqslant u \\leqslant 25$. 所以 $u_{\\text {min }}=\\frac{9}{4}, u_{\\text {max }}=25$.\n注意, $\\frac{9}{4}$ 和 25 都是能取到的.", + "remark": "", + "figures": [] +} \ No newline at end of file diff --git a/processed_dataset/calculation/0516.json b/processed_dataset/calculation/0516.json new file mode 100644 index 0000000000000000000000000000000000000000..a942a5b18e9812b281b514067fef31acdad63915 --- /dev/null +++ b/processed_dataset/calculation/0516.json @@ -0,0 +1,8 @@ +{ + "source_file": "./raw_volume-zh/volume2/chapter4.tex", + "problem_type": "calculation", + "problem": "例13 设 $f: \\mathbf{R} \\rightarrow \\mathbf{R}$, 满足 $f(\\cot x)=\\cos 2 x+\\sin 2 x$ 对所有 $0) 所示.\n而 $P\\left(x, x^2\\right)$ 在抛物线 $y=x^2$ 上,易知\n$$\n|P A|-|P B| \\leqslant|A B|\n$$\n取等号时, $P$ 在 $A B$ 的延长线上.\n$A B$ 的方程为\n$$\ny=\\frac{1}{3} x+1\n$$\n解方程组 $\\left\\{\\begin{array}{l}y=x^2, \\\\ y=\\frac{1}{3} x+1,\\end{array}\\right.$, 得\n$$\nx_1=\\frac{1-\\sqrt{37}}{6}, x_2=\\frac{1+\\sqrt{37}}{6} \\text {. }\n$$\n因为点 $P$ 在 $A B$ 的延长线上, 舍去 $x_2$, 所以 $x=\\frac{1-\\sqrt{37}}{6}$.\n最小距离为 $|A B|=\\sqrt{10}$, 这正是需求的函数的最大值.\n说明对于根号里的代数式, 我们常常通过配方, 把它看成平面上两个点之间的距离.", + "remark": "", + "figures": [ + "./images/volume2/figures/fig-c4e14.png" + ] +} \ No newline at end of file diff --git a/processed_dataset/calculation/0518.json b/processed_dataset/calculation/0518.json new file mode 100644 index 0000000000000000000000000000000000000000..a98400bad49d88694d041728babde99f43dbfb2a --- /dev/null +++ b/processed_dataset/calculation/0518.json @@ -0,0 +1,10 @@ +{ + "source_file": "./raw_volume-zh/volume2/chapter4.tex", + "problem_type": "calculation", + "problem": "例15 设 $x \\in\\left(0, \\frac{\\pi}{2}\\right), y \\in(0,+\\infty)$, 求\n$$\n(\\sqrt{2} \\cos x-y)^2+\\left(\\sqrt{2} \\sin x-\\frac{9}{y}\\right)^2\n$$\n的最小值.", + "solution": "分析:联想到两点间的距离公式, $(\\sqrt{2} \\cos x-y)^2+\\left(\\sqrt{2} \\sin x-\\frac{9}{y}\\right)^2$ 可视为平面直角坐标系中点 $A(\\sqrt{2} \\cos x, \\sqrt{2} \\sin x)$ 与 $B\\left(y, \\frac{9}{y}\\right)$ 的距离的平方, 而这两点又可看作参数 $x, y$ 所确定的曲线上的点, 问题即可转化为求两曲线上点之间距离的最小值问题.\n解如图()所示, 建立直角坐标系 $u O v$.\n设 $C_1:\\left\\{\\begin{array}{l}u=\\sqrt{2} \\cos x, \\\\ v=\\sqrt{2} \\sin x,\\end{array} x \\in\\left(0, \\frac{\\pi}{2}\\right)\\right.$\n及 $C_2:\\left\\{\\begin{array}{l}u=y, \\\\ v=\\frac{9}{y}, y \\in(0,+\\infty) .\\end{array}\\right.$\n则 $(\\sqrt{2} \\cos x-y)^2+\\left(\\sqrt{2} \\sin x-\\frac{9}{y}\\right)^2$ 表示曲线 $C_1$ 上点 $A(\\sqrt{2} \\cos x, \\sqrt{2} \\sin x)$ 与曲线 $C_2$ 上点 $B\\left(y, \\frac{9}{y}\\right)$ 之间的距离的平方.\n作出曲线 $C_1: u^2+v^2=2(u>0, v>0)$ 及 $C_2: u v=9(u>0, v>0)$ 的图象, 显然, 当 $|O B|$ 取得最小值而 $O 、 A 、 B$ 三点共线时, $|A B|^2$ 最小.\n因为\n$$\n|O B|=\\sqrt{y^2+\\left(\\frac{9}{y}\\right)^2} \\geqslant 3 \\sqrt{2} .\n$$\n当 $y=3$ 时,等号成立, 所以\n$$\n\\begin{gathered}\n|A B|=|O B|-|O A|=3 \\sqrt{2}-\\sqrt{2}=2 \\sqrt{2} . \\\\\n|A B|^2=8 .\n\\end{gathered}\n$$\n所以, $(\\sqrt{2} \\cos x-y)^2+\\left(\\sqrt{2} \\sin x-\\frac{9}{y}\\right)^2$ 的最小值为 8 .", + "remark": "", + "figures": [ + "./images/volume2/figures/fig-c4e15.png" + ] +} \ No newline at end of file diff --git a/processed_dataset/calculation/0519.json b/processed_dataset/calculation/0519.json new file mode 100644 index 0000000000000000000000000000000000000000..bf2f3fe6eb5fff4168ce3f016acc59f16fb3eb50 --- /dev/null +++ b/processed_dataset/calculation/0519.json @@ -0,0 +1,11 @@ +{ + "source_file": "./raw_volume-zh/volume2/chapter4.tex", + "problem_type": "calculation", + "problem": "例16 若实数 $x, y$ 满足关系\n$$\n\\left\\{\\begin{array}{l}\ny \\geqslant x^2, \\\\\n2 x^2+2 x y+y^2 \\leqslant 5,\n\\end{array}\\right.\n$$\n求函数 $w=2 x+y$ 的最大值和最小值.", + "solution": "解:令 $u=x, v=x+y$, 则原问题就转化为在约束条件\n$$\n\\left\\{\\begin{array}{l}\nu^2+u \\leqslant v, \\\\\nu^2+v^2 \\leqslant 5\n\\end{array}\\right.\n$$\n下,求函数 $w=u+v$ 的最大值和最小值.\n把约束条件在 $u v$ 平面上表示出来, 它就是如图 () 所示的阴影部分(包括边界).\n令 $u+v=m$, 即 $v=-u+m$, 它是倾角为 $\\frac{3}{4} \\pi$ 的直线系, $m$ 为它的纵截距.\n我们把问题又转化为在直线系: $v=-u+m$ 中确定这样的直线, 它通过图 4-5 所示的闭区域中的至少一个点,且使得纵截距取得最大值和最小值.\n从图 () 中可以看出, 直线系中与抛物线 $v= u^2+u$ 相切于点 $T$ 的直线 $l_1$ 所对应的 $m_1$ 最小, 过抛物线与圆的右边的一个交点 $P$ 的直线 $l_2$ 所对应的 $m_2$ 最大,下面求 $m_1$ 和 $m_2$. 由\n$$\n\\left\\{\\begin{array}{l}\nu+v=m_1, \\\\\nv=u^2+u\n\\end{array}\\right.\n$$\n中消去 $v$, 得 $u^2+2 u-m_1=0 . \\Delta=4+4 m_1=0$, 故 $m_1=-1$.\n解方程组\n$$\n\\left\\{\\begin{array}{l}\nv=u^2+u, \\\\\nu^2+v^2=5,\n\\end{array}\\right.\n$$\n得点 $P$ 的坐标为 $(1,2)$. 因 $l_2$ 经过点 $P$, 故 $m_2=1+2=3$.\n综上所述, 我们有 $w_{\\min }=-1, w_{\\max }=3$.\n说明在约束条件下, 求函数的最大值和最小值是数学中常见的问题, 利用函数图象的性质和曲线系来求解这类问题, 往往能化繁为简, 出奇制胜.", + "remark": "", + "figures": [ + "./images/volume2/figures/fig-c4e16.png", + "./images/volume2/figures/fig-c4e16.png" + ] +} \ No newline at end of file diff --git a/processed_dataset/calculation/0520.json b/processed_dataset/calculation/0520.json new file mode 100644 index 0000000000000000000000000000000000000000..67d27c338bfb648bde2fbe07780380bcde44752a --- /dev/null +++ b/processed_dataset/calculation/0520.json @@ -0,0 +1,8 @@ +{ + "source_file": "./raw_volume-zh/volume2/chapter4.tex", + "problem_type": "calculation", + "problem": "例17 求函数 $y=\\sqrt{2 x^2-3 x+1}+\\sqrt{x^2}-2 x$ 的最小值.", + "solution": "解:先求定义域,再研究函数的单调区间.\n易知定义域为 $(-\\infty, 0] \\cup[2,+\\infty)$.\n因为 $2 x^2-3 x+1$ 在 $(-\\infty, 0]$ 上递减, 在 $[2,+\\infty)$ 上递增, 所以 $\\sqrt{2 x^2-3 x+1}$ 在 $(-\\infty, 0]$ 上递减, 在 $[2,+\\infty)$ 上递增.\n同理, $\\sqrt{x^2-} \\overline{2 x}$ 在 $(-\\infty, 0]$ 上递减, 在 $[2,+\\infty)$ 上递增.\n所以 $y==\\sqrt{2 x^2-3 x+1}+\\sqrt{x^2-2 x}$ 在 $(-\\infty, 0]$ 上递减, 在 $[2,+\\infty)$ 上递增.\n所以 $y_{\\min }=\\min \\{f(0), f(2)\\}=\\min \\{\\sqrt{1}, \\sqrt{8-6+1}\\}=1$.\n所以 $y_{\\text {min }}=1$ (在 $x=0$ 时取到).\n说明本题的函数可看成两个函数的和, 而这两个函数在定义域内的单调性是一致的,利用“单调性一致的两个函数的和仍具有相同单调性”这一性质求出各个单调区间上的最小值, 再比较得出结论.", + "remark": "", + "figures": [] +} \ No newline at end of file diff --git a/processed_dataset/calculation/0521.json b/processed_dataset/calculation/0521.json new file mode 100644 index 0000000000000000000000000000000000000000..877f1feb7092a96181da51e3eb952c84eb8ac2ca --- /dev/null +++ b/processed_dataset/calculation/0521.json @@ -0,0 +1,8 @@ +{ + "source_file": "./raw_volume-zh/volume2/chapter4.tex", + "problem_type": "calculation", + "problem": "例18 设 $a>0, r(x)=\\frac{a x^2+1}{x}=a x+\\frac{1}{x}$. 试讨论函数 $r(x)$ 在 $(0$, $+\\infty)$ 中的单调性, 最小值与最大值.", + "solution": "解:设 $0\\left(x_2-x_1\\right)\\left(a-\\frac{1}{x_1^2}\\right) \\geqslant 0,\n$$\n所以在区间 $\\left[\\frac{1}{\\sqrt{a}},+\\infty\\right)$ 上, $r(x)$ 是单调递增的.\n因此, $r(x)$ 没有最大值, 当且仅当 $x=\\frac{1}{\\sqrt{a}}$ 时, $r(x)$ 取最小值, 最小值为 $r\\left(\\frac{1}{\\sqrt{a}}\\right)=2 \\sqrt{a}$.", + "remark": "", + "figures": [] +} \ No newline at end of file diff --git a/processed_dataset/calculation/0522.json b/processed_dataset/calculation/0522.json new file mode 100644 index 0000000000000000000000000000000000000000..98fcae317da8b57affb125f26efdf8f5bda88742 --- /dev/null +++ b/processed_dataset/calculation/0522.json @@ -0,0 +1,8 @@ +{ + "source_file": "./raw_volume-zh/volume2/chapter5.tex", + "problem_type": "calculation", + "problem": "例8 设 $f(x)$ 是一个 98 次的多项式,使得\n$$\nf(k)=\\frac{1}{k}, k=1,2, \\cdots, 99\n$$\n求 $f(100)$ 的值.", + "solution": "解:构造一个函数\n$$\ng(x)=x f(x)-1,\n$$\n则\n$$\ng(1)=g(2)=\\cdots=g(99)=0,\n$$\n并且 $g(x)$ 是 99 次多项式, 所以\n$$\ng(x)=a(x-1)(x-2) \\cdots(x-99) .\n$$\n其中 $g(x)$ 的首项系数 $a$ 是一个待定常数.\n由于\n$$\nf(x)=\\frac{g(x)+1}{x}=\\frac{a(x-1)(x-2) \\cdots(x-99)+1}{x}\n$$\n是一个 98 次多项式, 故 $a(x-1)(x-2) \\cdots(x-99)+1$ 的常数项必须为 0 , 即\n$$\n-99 ! a+1=0\n$$\n所以\n$$\na=\\frac{1}{99 !}\n$$\n因此 $f(x)=\\frac{\\frac{1}{99 !}(x-1)(x-2) \\cdots(x-99)+1}{x}$, 所以\n$$\nf(100)=\\frac{1+1}{100}=\\frac{1}{50} .\n$$\n说明对于二次三项式 $f(x)=a x^2+b x+c$, 若 $f(x)=0$ 有两个根 $x_1$ 、 $x_2$, 则 $f(x)=a\\left(x-x_1\\right)\\left(x-x_2\\right)$. 一般地, 一个 $n(\\geqslant 2)$ 次多项式 $f(x)= a_n x^n+a_{n-1} x^{n-1}+\\cdots+a_1 x+a_0$, 若它的 $n$ 个根为 $x_1, x_2, \\cdots, x_n$, 则 $f(x)= a_n\\left(x-x_1\\right)\\left(x-x_2\\right) \\cdots\\left(x-x_n\\right)$.", + "remark": "", + "figures": [] +} \ No newline at end of file diff --git a/processed_dataset/calculation/0523.json b/processed_dataset/calculation/0523.json new file mode 100644 index 0000000000000000000000000000000000000000..adeba2970ea5c5f1ebee3debcd4e9116467d7e6a --- /dev/null +++ b/processed_dataset/calculation/0523.json @@ -0,0 +1,8 @@ +{ + "source_file": "./raw_volume-zh/volume2/chapter5.tex", + "problem_type": "calculation", + "problem": "例10 正五边形的每个顶点对应一个整数, 使得这五个整数的和为正; 若其中三个相邻顶点对应的整数依次为 $x 、 y 、 z$, 而中间的 $y<0$, 则要进行如下的变换: 整数 $x 、 y 、 z$ 分别换为 $x+y 、-y 、 x+y$. 要是所得的五个整数中还有一个为负时, 这种变换就继续进行, 问: 这样的变换进行有限次后是否必定终止?", + "solution": "解:问题的答案是肯定的.\n也就是说, 这样的变换进行有限次后必定终止.\n为了方便起见,把五个数的环列写成横列\n$$\nv, w, x, y, z .\n$$\n这里的 $v 、 z$ 在环列中是相邻的.\n并且由题设知\n$$\nv+w+x+y+z>0 .\n$$\n不妨设 $y<0$, 经变换后得到的环列是\n$$\nv, w, x+y,-y, y+z .\n$$\n构造函数\n$$\n\\begin{aligned}\nf\\left(x_1, x_2, x_3, x_4, x_5\\right)= & x_1^2+x_2^2+x_3^2+x_4^2+x_5^2+\\left(x_1+x_2\\right)^2+ \\\\\n& \\left(x_2+x_3\\right)^2+\\left(x_3+x_4\\right)^2+ \\\\\n& \\left(x_4+x_5\\right)^2+\\left(x_5+x_1\\right)^2,\n\\end{aligned}\n$$\n那么, 变换前后函数值的差为\n$$\n\\begin{aligned}\n& f(v, w, x+y,-y, y+z)-f(v, w, x, y, z) \\\\\n= & {\\left[v^2+w^2+(x+y)^2+(-y)^2+(y+z)^2+(v+w)^2+(w+\\right.} \\\\\n& \\left.x+y)^2+x^2+z^2+(y+z+v)^2\\right]-\\left[v^2+w^2+x^2+y^2+z^2+\\right. \\\\\n& \\left.(v+w)^2+(w+x)^2+(x+y)^2+(y+z)^2+(z+v)^2\\right] \\\\\n= & 2 y(v+w+x+y+z)<0 .\n\\end{aligned}\n$$\n由于当变量取整数值时 $f$ 的函数值为非负整数,所以\n$$\nf(v, w, x+y,-y, y+z) \\leqslant f(v, w, x, y, z)-1,\n$$\n即每经一次变换, $f$ 的值至少减少 1. 所以经有限次变换后, 就不能再变换下去了.\n说明本题是第 27 届国际数学奥林匹克竞赛中的一道试题,是那届得分最低的一题.\n但美国选手约瑟夫 - 基内对这题的解法获得了特别奖.\n他构造的辅助函数是\n$$\n\\begin{aligned}\nf\\left(x_1, x_2, x_3, x_4, x_5\\right)= & \\sum_{i=1}^5\\left|x_i\\right|+\\sum_{i=1}^5\\left|x_i+x_{i+1}\\right|+ \\\\\n& \\sum_{i=1}^5\\left|x_i+x_{i+1}+x_{i+2}\\right|+ \\\\\n& \\sum_{i=1}^5\\left|x_i+x_{i+1}+x_{i+2}+x_{i+3}\\right|,\n\\end{aligned}\n$$\n其中, $x_6=x_1, x_7=x_2, x_8=x_3$.", + "remark": "", + "figures": [] +} \ No newline at end of file diff --git a/processed_dataset/calculation/0524.json b/processed_dataset/calculation/0524.json new file mode 100644 index 0000000000000000000000000000000000000000..680131e2da52b00c4ae8d20dc9d698903d6e3ebe --- /dev/null +++ b/processed_dataset/calculation/0524.json @@ -0,0 +1,8 @@ +{ + "source_file": "./raw_volume-zh/volume2/chapter6.tex", + "problem_type": "calculation", + "problem": "五只猴子, 分一堆桃子, 怎么也平分不了, 于是大家同意先去睡觉, 明天再说.\n夜里一个猴子偷偷起来, 把一个桃子吃掉后正好可以分成 5 份, 收藏起自己的一份后又去睡觉了.\n第二只猴子起来后, 像先前的那个猴子一样, 先吃掉一个,剩下的又刚好分成 5 份, 也把自己的一份收藏起来睡觉去了.\n第三、四、五只猴子也都是这样: 先吃掉一个,剩下的刚好分成 5 份, 问这堆桃子至少是多少个?", + "solution": "这个题目有好几种解法,下面介绍一种:\n设桃子的总数有 $x$ 个, 第 $i$ 个猴子吃掉一个并拿走一份后, 剩下的桃子数目为 $x_i$ 个, 则\n$$\nx_i=\\frac{4}{5}\\left(x_{i-1}-1\\right), i=1,2,3,4,5,\n$$\n且 $x_0=x$.\n设函数 $f(x)=\\frac{4}{5}(x-1)=\\frac{4}{5}(x+4)-4$, 于是\n$$\n\\begin{aligned}\n& x_1=f(x)=\\frac{4}{5}(x+4)-4, \\\\\n& x_2=f(f(x))=\\left(\\frac{4}{5}\\right)^2(x+4)-4, \\\\\n& x_3=f(f(f(x)))=\\left(\\frac{4}{5}\\right)^3(x+4)-4, \\\\\n& x_4=f(f(f(f(x))))=\\left(\\frac{4}{5}\\right)^4(x+4)-4, \\\\\n& x_5=f(f(f(f(f(x)))))=\\left(\\frac{4}{5}\\right)^5(x+4)-4 .\n\\end{aligned}\n$$\n由于剩下的桃子数都是整数, 所以 $5^5 \\mid(x+4)$, 因此, 最小的 $x$ 为 $x= 5^5-4=3121$.", + "remark": "", + "figures": [] +} \ No newline at end of file diff --git a/processed_dataset/calculation/0525.json b/processed_dataset/calculation/0525.json new file mode 100644 index 0000000000000000000000000000000000000000..46c15c12aea3a079f5209d0a21c36b7644abf739 --- /dev/null +++ b/processed_dataset/calculation/0525.json @@ -0,0 +1,8 @@ +{ + "source_file": "./raw_volume-zh/volume2/chapter6.tex", + "problem_type": "calculation", + "problem": "某男孩付一角钱进入第一家商店, 他在店里花了剩余的钱的一半, 走出商店时又付了一角钱.\n之后, 他又付一角钱进人第二家商店, 在这里他花了剩余的钱的一半, 走出商店时又付了一角钱.\n接着, 他又用同样的方式进出第三和第四家商店, 当他走出第四家商店后, 这时, 他身上只剩下一角钱, 问: 他进人第一家商店之前身上有多少钱?", + "solution": "设这个男孩进人第 $i$ 个商店之前身上的钱为 $x_i$ 角, $i=1,2,3,4$, 且设 $x_5=1$, 于是\n$$\n\\begin{aligned}\n& x_{i+1}=\\frac{1}{2}\\left(x_i-1\\right)-1, i=1,2,3,4 . \\\\\n& \\text { 令 } f(x)=\\frac{1}{2}(x-1)-1=\\frac{1}{2}(x+3)-3, \\text { 则 } \\\\\n& x_2=f\\left(x_1\\right)=\\frac{1}{2}\\left(x_1+3\\right)-3, \\\\\n& x_3=f\\left(f\\left(x_1\\right)\\right)=\\frac{1}{2^2}\\left(x_1+3\\right)-3, \\\\\n& x_4=f\\left(f\\left(f\\left(x_1\\right)\\right)\\right)=\\frac{1}{2^3}(x+3)-3, \\\\\n& x_5=f\\left(f\\left(f\\left(f\\left(x_1\\right)\\right)\\right)\\right)=\\frac{1}{2^4}(x+3)-3 .\n\\end{aligned}\n$$\n因为\n$$\n\\frac{1}{2^4}\\left(x_1+3\\right)-3=1,\n$$\n故\n$$\nx_1=61,\n$$\n即这个男孩进人第一家商店之前身上有 6.10 元.", + "remark": "", + "figures": [] +} \ No newline at end of file diff --git a/processed_dataset/calculation/0526.json b/processed_dataset/calculation/0526.json new file mode 100644 index 0000000000000000000000000000000000000000..8b858312a4edc8b4379f47e65fd44a9b4d3dad3f --- /dev/null +++ b/processed_dataset/calculation/0526.json @@ -0,0 +1,8 @@ +{ + "source_file": "./raw_volume-zh/volume2/chapter6.tex", + "problem_type": "calculation", + "problem": "例1 已知 $f(x)$ 是一次函数, 且\n$$\nf^{(10)}(x)=1024 x+1023,\n$$\n求 $f(x)$ 的解析式.", + "solution": "解:设 $f(x)=a x+b$, 则\n$$\nf^{(10)}(x)=a^{10}\\left(x-\\frac{b}{1-a}\\right)+\\frac{b}{1-a} .\n$$\n故 $\\quad a^{10}\\left(x-\\frac{b}{1-a}\\right)+\\frac{b}{1-a}=1024 x+1023$.\n比较上式两边系数, 得\n$$\n\\left\\{\\begin{array}{l}\na^{10}=1024, \\\\\n-\\frac{a^{10} b}{1-a}+\\frac{b}{1-a}=1023\n\\end{array}\\right.\n$$\n解方程组得 $a_1=2, b_1=1 ; a_2=-2, b_2=-3$.\n因此, 所求的一次函数为\n$$\nf(x)=2 x+1 \\text { 或 } f(x)=-2 x-3 .\n$$", + "remark": "", + "figures": [] +} \ No newline at end of file diff --git a/processed_dataset/calculation/0527.json b/processed_dataset/calculation/0527.json new file mode 100644 index 0000000000000000000000000000000000000000..c73780ad56b4166b74b57913f9bf98bf5c2202ba --- /dev/null +++ b/processed_dataset/calculation/0527.json @@ -0,0 +1,8 @@ +{ + "source_file": "./raw_volume-zh/volume2/chapter6.tex", + "problem_type": "calculation", + "problem": "例2 $f(n)$ 是定义在 $\\mathbf{N}_{+}$上的函数, 并且满足\n(1) $f(f(n))=4 n+9, n \\in \\mathbf{N}_{+}$;\n(2) $f\\left(2^k\\right)=2^{k+1}+3, k \\in \\mathbf{N}_{+} \\cup\\{0\\}$.\n求 $f(1789)$ 的值.", + "solution": "解:$f(4 n+9)=f^{(3)}(n)=f^{(2)}(f(n))=4 f(n)+9$, 而\n$$\n1789=4^4 \\times 2^2+4^3 \\times 9+4^2 \\times 9+4 \\times 9+9,\n$$\n所以\n$$\n\\begin{aligned}\nf(1789) & =f\\left(4^4 \\times 2^2+4^3 \\times 9+4^2 \\times 9+4 \\times 9+9\\right) \\\\\n& =4 f\\left(4^3 \\times 2^2+4^2 \\times 9+4 \\times 9+9\\right)+9 \\\\\n& =4^2 f\\left(4^2 \\times 2^2+4 \\times 9+9\\right)+4 \\times 9+9 \\\\\n& =4^3 f\\left(4 \\times 2^2+9\\right)+4^2 \\times 9+4 \\times 9+9 \\\\\n& =4^4 f\\left(2^2\\right)+4^3 \\times 9+4^2 \\times 9+4 \\times 9+9 \\\\\n& =4^4\\left(2^3+3\\right)+4^3 \\times 9+4^2 \\times 9+4 \\times 9+9 \\\\\n& =3581 .\n\\end{aligned}\n$$", + "remark": "", + "figures": [] +} \ No newline at end of file diff --git a/processed_dataset/calculation/0528.json b/processed_dataset/calculation/0528.json new file mode 100644 index 0000000000000000000000000000000000000000..bee759f4100219d41b1e7d4396d0c53347d39ba3 --- /dev/null +++ b/processed_dataset/calculation/0528.json @@ -0,0 +1,8 @@ +{ + "source_file": "./raw_volume-zh/volume2/chapter6.tex", + "problem_type": "calculation", + "problem": "例8 设 $f(x)=3 \\sqrt[3]{x}(\\sqrt[3]{x}+1)+x+1$, 求 $f^{(n)}(x)$.", + "solution": "解:$f(x)=(\\sqrt[3]{x}+1)^3$, 设 $a_0=x, a_n=f^{(n)}(x)$,\n则即\n$$\na_n=f\\left(a_{n-1}\\right)=\\left(\\sqrt[3]{a_{n-1}}+1\\right)^3,\n$$\n$$\n\\sqrt[3]{a_n}-\\sqrt[3]{a_{n-1}}=1 .\n$$\n从而\n$$\n\\sqrt[3]{a_n}=\\sqrt[3]{a_0}+n=\\sqrt[3]{x}+n\n$$\n故\n$$\na_n=(\\sqrt[3]{x}+n)^3,\n$$\n即\n$$\nf^{(n)}(x)=(\\sqrt[3]{x}+n)^3 .\n$$", + "remark": "", + "figures": [] +} \ No newline at end of file diff --git a/processed_dataset/calculation/0529.json b/processed_dataset/calculation/0529.json new file mode 100644 index 0000000000000000000000000000000000000000..5152bb626f56cdae0c90d6c5fcd332d7b3e7741c --- /dev/null +++ b/processed_dataset/calculation/0529.json @@ -0,0 +1,8 @@ +{ + "source_file": "./raw_volume-zh/volume2/chapter6.tex", + "problem_type": "calculation", + "problem": "例10 设 $f(x)=a x+b$, 用相似法求 $f^{(n)}(x)$.", + "solution": "解:取 $g(x)=a x, \\varphi(x)=x-\\frac{b}{1-a}$, 则\n$$\n\\varphi^{-1}(x)=x+\\frac{b}{1-a} \\text {. }\n$$\n于是\n$$\n\\begin{aligned}\n\\varphi^{-1}(g(\\varphi(x))) & =\\varphi^{-1}(a \\varphi(x)) \\\\\n& =\\varphi^{-1}\\left(a\\left(x-\\frac{b}{1-a}\\right)\\right) \\\\\n& =a\\left(x-\\frac{b}{1-a}\\right)+\\frac{b}{1-a} \\\\\n& =a x+b=f(x) .\n\\end{aligned}\n$$\n所以 $f \\sim g$. 而 $g^{(n)}(x)=a^n x$, 因此\n$$\n\\begin{aligned}\nf^{(n)}(x) & =\\varphi^{-1}\\left(g^{(n)}(\\varphi(x))\\right) \\\\\n& =\\varphi^{-1}\\left(a^n \\varphi(x)\\right) \\\\\n& =a^n\\left(x-\\frac{b}{1-a}\\right)+\\frac{b}{1-a} .\n\\end{aligned}\n$$", + "remark": "", + "figures": [] +} \ No newline at end of file diff --git a/processed_dataset/calculation/0530.json b/processed_dataset/calculation/0530.json new file mode 100644 index 0000000000000000000000000000000000000000..d8685f310ca0adf986203d553dcf0c82840fac87 --- /dev/null +++ b/processed_dataset/calculation/0530.json @@ -0,0 +1,8 @@ +{ + "source_file": "./raw_volume-zh/volume2/chapter6.tex", + "problem_type": "calculation", + "problem": "例12 设 $f(x)=2 x^2-1$, 求 $f^{(n)}(x)$.", + "solution": "解:令 $g(x)=2 x, \\varphi(x)=\\arccos x$, 则 $\\varphi^{-1}(x)=\\cos x$.\n$$\n\\begin{aligned}\nf(x) & =2 x^2-1 \\\\\n& =2 \\cos ^2(\\arccos x)-1 \\\\\n& =\\cos 2(\\arccos x) \\\\\n& =\\varphi^{-1}(g(\\varphi(x))) .\n\\end{aligned}\n$$\n所以 $f \\stackrel{\\varphi}{\\sim} g$. 而 $g^{(n)}(x)=2^n x$, 因此\n$$\n\\begin{aligned}\nf^{(n)}(x) & =\\varphi^{-1}\\left(g^{(n)}(\\varphi(x))\\right) \\\\\n& =\\cos \\left(2^n \\arccos x\\right) .\n\\end{aligned}\n$$\n这个迭代结果就是切比雪夫多项式.\n一般来说, 要找出桥函数 $\\varphi(x)$ 往往并不容易, 要对 $f(x)$ 进行观察、变形, 并利用经验来完成.", + "remark": "", + "figures": [] +} \ No newline at end of file diff --git a/processed_dataset/calculation/0531.json b/processed_dataset/calculation/0531.json new file mode 100644 index 0000000000000000000000000000000000000000..e04e9502c1c6ef724dd39a1008b772cd3ca472b4 --- /dev/null +++ b/processed_dataset/calculation/0531.json @@ -0,0 +1,8 @@ +{ + "source_file": "./raw_volume-zh/volume2/chapter6.tex", + "problem_type": "calculation", + "problem": "例13 试求一个函数 $p(x)$, 使 $p^{(8)}(x)=x^2+2 x$.", + "solution": "解:令 $f(x)=x^2+2 x, \\varphi(x)=x+1, g(x)=x^2$, 则 $\\varphi^{-1}(x)=x-1$.\n于是\n$$\nf(x)=\\varphi^{-1}(g(\\varphi(x))) .\n$$\n再令 $h(x)=x^{\\sqrt[8]{2}}$, 那么\n$$\nh^{(8)}(x)=x^2=g(x) .\n$$\n于是取 $p(x)=\\varphi^{-1}(h(\\varphi(x)))=(x+1)^{8 \\sqrt{2}}-1$, 那么\n$$\n\\begin{aligned}\np^{(8)}(x) & =\\varphi^{-1}\\left(h^{(8)}(\\varphi(x))\\right) \\\\\n& =\\varphi^{-1}(g(\\varphi(x))) \\\\\n& =f(x)=x^2+2 x .\n\\end{aligned}\n$$\n所以, $p(x)=(x+1)^{8 \\sqrt{2}}-1$, 即为所求.", + "remark": "", + "figures": [] +} \ No newline at end of file diff --git a/processed_dataset/calculation/0532.json b/processed_dataset/calculation/0532.json new file mode 100644 index 0000000000000000000000000000000000000000..6bc95ddd2f0e2074de9a2fb91e1bdff44f4eeddc --- /dev/null +++ b/processed_dataset/calculation/0532.json @@ -0,0 +1,8 @@ +{ + "source_file": "./raw_volume-zh/volume2/chapter6.tex", + "problem_type": "calculation", + "problem": "例15 用不动点法求解,已知 $f(x)$ 是一次函数, 且\n$$\nf^{(10)}(x)=1024 x+1023,\n$$\n求 $f(x)$ 的解析式.\n且设 $a \\neq 1$ ).", + "solution": "解:令 $f(x)=x$, 得 $f(x)$ 的唯一不动点 $x=\\frac{b}{1-a}$.\n故\n$$\nf(x)=a\\left(x-\\frac{b}{1-a}\\right)+\\frac{b}{1-a},\n$$\n$$\nf^{(2)}(x)=a^2\\left(x-\\frac{b}{1-a}\\right), 1-a,\n$$\n由归纳法, 得\n$$\nf^{(n)}(x)=a^n\\left(x-\\frac{b}{1-a}\\right)+\\frac{b}{1-a} .\n$$", + "remark": "", + "figures": [] +} \ No newline at end of file diff --git a/processed_dataset/calculation/0533.json b/processed_dataset/calculation/0533.json new file mode 100644 index 0000000000000000000000000000000000000000..ab5dab5986c7e08d1a65de97244a5e2cba3f3574 --- /dev/null +++ b/processed_dataset/calculation/0533.json @@ -0,0 +1,8 @@ +{ + "source_file": "./raw_volume-zh/volume2/chapter6.tex", + "problem_type": "calculation", + "problem": "例16 设 $f(x)==\\sqrt{19 x^2+93}$, 求 $f^{(n)}(x)$.", + "solution": "解:先求 $f(x)$ 的不动点.\n由 $\\sqrt{19 x^2+93}=x$, 得 $x^2=-\\frac{31}{6}$. 所以\n$$\n\\begin{aligned}\n& f(x)=\\sqrt{19\\left(x^2+\\frac{31}{6}\\right)-\\frac{31}{6}}, \\\\\n& f^{(2)}(x)=\\sqrt{19^2\\left(x^2+\\frac{31}{6}\\right)-\\frac{31}{6}}, \\\\\n& \\ldots . .\n\\end{aligned}\n$$\n由归纳法, 得\n$$\nf^{(n)}(x)=\\sqrt{19^n\\left(x^2+\\frac{31}{6}\\right)-\\frac{31}{6}} .\n$$\n下面介绍利用不动点寻找桥函数的方法.\n由不动点性质知, 桥函数 $\\varphi$ 具有下列性质: 它将 $f$ 的不动点 $x_0$ 映成 $g$ 的不动点 $\\varphi\\left(x_0\\right)$, 通常为了便于求 $g^{(n)}(x), g(x)$ 通常取为 $a x, x+a, a x^2, a x^3$ 等, 这时, $g(x)$ 的不动点为 0 或 $\\infty$, 此时, 若 $f(x)$ 只有唯一不动点 $\\alpha$ 时, 则可考虑取 $\\varphi(x)=x-\\alpha$ (或 $\\left.\\frac{1}{x-\\alpha}\\right)$, 这时 $\\varphi(\\alpha)=0$ (或 $\\infty)$; 若 $f(x)$ 有两个不动点 $\\alpha 、 \\beta(\\alpha \\neq \\beta)$, 则可考虑取 $\\varphi(x)=\\frac{x-\\alpha}{x-\\beta}$, 这里 $\\varphi(\\alpha)=0, \\varphi(\\beta)=\\infty$.", + "remark": "", + "figures": [] +} \ No newline at end of file diff --git a/processed_dataset/calculation/0534.json b/processed_dataset/calculation/0534.json new file mode 100644 index 0000000000000000000000000000000000000000..da2e44cb6c0275cbc59f1caa3e7d2fb7334bf965 --- /dev/null +++ b/processed_dataset/calculation/0534.json @@ -0,0 +1,8 @@ +{ + "source_file": "./raw_volume-zh/volume2/chapter6.tex", + "problem_type": "calculation", + "problem": "例17 设 $f(x)=\\frac{x^2}{2 x-1}$, 求 $f^{(n)}(x)$.", + "solution": "解:令 $f(x)=x$, 求得 $f(x)$ 的不动点为 $x_0=0$ 或 1 , 取 $\\varphi(x)$ 使满足 $\\varphi(0)=\\infty, \\varphi(1)=0$, 最简单的取法是 $\\varphi(x)=\\frac{x-1}{x}$, 则 $\\varphi^{-1}(x)=\\frac{1}{1-x}$, 算出 $g(x)=\\varphi\\left(f\\left(\\varphi^{-1}(x)\\right)\\right)=x^2$, 则 $f(x)=\\varphi^{-1}(g(\\varphi(x)))$.\n于是\n$$\n\\begin{aligned}\nf^{(n)}(x) & =\\varphi^{-1}\\left(g^{(n)}(\\varphi(x))\\right) \\\\\n& =\\frac{1}{1-\\left(1-\\frac{1}{x}\\right)^{2^n}}=\\frac{x^{2^n}}{x^{2^n}-(x-1)^{2^n}} .\n\\end{aligned}\n$$", + "remark": "", + "figures": [] +} \ No newline at end of file diff --git a/processed_dataset/calculation/0535.json b/processed_dataset/calculation/0535.json new file mode 100644 index 0000000000000000000000000000000000000000..2871cc9371ae3e87f56b76840b26f5fcf0305ba2 --- /dev/null +++ b/processed_dataset/calculation/0535.json @@ -0,0 +1,8 @@ +{ + "source_file": "./raw_volume-zh/volume2/chapter6.tex", + "problem_type": "calculation", + "problem": "例18 设 $f(x)=a x^2+b x+c(a \\neq 0)$, 求 $f^{(n)}(x)$.", + "solution": "解:对于二次函数, 并不是所有的情形都能很简单地得到 $f^{(n)}(x)$ 的表达式,下面只考虑当 $c=\\frac{b^2-2 b}{4 a}$ 时的情形.\n由\n$$\na x^2+b x+\\frac{b^2-2 b}{4 a}=x,\n$$\n得两个不动点 $-\\frac{b}{2 a}, \\frac{b+2}{2 a}$. 取其中一个 $x_0=-\\frac{b}{2 a}$.\n令 $g(x)=a x^2, \\varphi(x)=x-x_0$, 则 $\\varphi^{-1}(x)=x+x_0$, 于是\n$$\n\\begin{aligned}\n\\varphi^{-1}(g(\\varphi(x))) & =\\varphi^{-1}\\left(g\\left(x-x_0\\right)\\right) \\\\\n& =\\varphi^{-1}\\left(a\\left(x-x_0\\right)^2\\right) \\\\\n& =a\\left(x-x_0\\right)^2+x_0 \\\\\n& =a x^2+b x+c=f(x) .\n\\end{aligned}\n$$\n故\n$$\n\\begin{aligned}\nf^{(n)}(x) & =\\varphi^{-1}\\left(g^{(n)}(\\varphi(x))\\right) \\\\\n& =\\varphi^{-1}\\left(g^{(n)}\\left(x-x_0\\right)\\right) \\\\\n& =\\varphi^{-1}\\left(a^{2^n-1}\\left(x-x_0\\right)^{2^n}\\right) \\\\\n& =a^{2^n-1}\\left(x-x_0\\right)^{2^n}+x_0 \\\\\n& =a^{2^n-1}\\left(x+\\frac{b}{2 a}\\right)-\\frac{b}{2 a} .\n\\end{aligned}\n$$", + "remark": "", + "figures": [] +} \ No newline at end of file diff --git a/processed_dataset/calculation/0536.json b/processed_dataset/calculation/0536.json new file mode 100644 index 0000000000000000000000000000000000000000..3f69b9959d7377b2523c57b3fa9db1198c625f4a --- /dev/null +++ b/processed_dataset/calculation/0536.json @@ -0,0 +1,8 @@ +{ + "source_file": "./raw_volume-zh/volume2/chapter7.tex", + "problem_type": "calculation", + "problem": "例1 求解函数方程:\n$$\nf(\\sin x-1)=\\cos ^2 x+2(-\\infty2$ 时, 令 $x=2+t(t>0)$, 有\n$$\nf(t f(2)) f(2)=f(t+2)=f(x) .\n$$\n因为 $f(2)=0$, 所以 $f(x)=0(x \\geqslant 2)$.\n当 $0 \\leqslant x<2$ 时, 令 $x+t=2(t>0)$, 有\n$$\n0=f(2)=f(x+t)=f(t f(x)) f(x) .\n$$\n又因为 $f(t f(x))=0$, 故 $t f(x) \\geqslant 2$, 即\n$$\nf(x) \\geqslant \\frac{2}{t}=\\frac{2}{-x+2} .\n$$\n但 $f(x)>\\frac{2}{2-x}$ 在 $x \\in[0,2]$ 时不成立.\n若有 $x_1 \\in[0,2]$,且\n$$\n\\begin{gathered}\nf\\left(x_1\\right)>\\frac{2}{2-x_1}, \\\\\nf\\left(x_1\\right)\\left(2-x_1\\right)>2 .\n\\end{gathered}\n$$\n则可得这时总可找到 $y<2-x_1$, 使 $f\\left(x_1\\right) \\cdot y \\geqslant 2$, 做\n$$\nf\\left(y f\\left(x_1\\right)\\right)=0,\n$$\n也即\n$$\nf\\left(x_1+y\\right)=f\\left(y f\\left(x_1\\right)\\right) f\\left(x_1\\right)=0 .\n$$\n此式与 $x_1+y \\geqslant 2$ 矛盾, 即 $f(x)=\\frac{2}{2-x}$.\n从而 $f(x)=\\left\\{\\begin{array}{l}0, \\text { 当 } x \\geqslant 2 \\text { 时; } \\\\ \\frac{2}{2-x}, \\text { 当 } 0 \\leqslant x<2 \\text { 时.\n}\\end{array}\\right.$", + "remark": "", + "figures": [] +} \ No newline at end of file diff --git a/processed_dataset/calculation/0540.json b/processed_dataset/calculation/0540.json new file mode 100644 index 0000000000000000000000000000000000000000..7379107b82e45383e27eb37f2baa5360e429b38e --- /dev/null +++ b/processed_dataset/calculation/0540.json @@ -0,0 +1,8 @@ +{ + "source_file": "./raw_volume-zh/volume2/chapter7.tex", + "problem_type": "calculation", + "problem": "例7 求满足下列条件的多项式 $f(x, y)$ :\n(1) $f(x, y)$ 对于 $x 、 y$ 是齐次的, 即对任意实数 $t 、 x 、 y$, 若 $f$ 是 $n$ 次的, 则有\n$$\nf(t x, t y)==t^n f(x, y) ;\n$$\n(2) 对任意 $a 、 b 、 c \\in \\mathbf{R}$, 有\n$$\nf(a+b, c)+f(b+c, a)+f(c+a, b)=0 ;\n$$\n(3) $f(1,0)=1$.", + "solution": "解:在(2) 中令 $a=b=c=x$, 得\n$$\nf(2 x, x)=0 .\n$$\n根据多项式的因式定理,有\n$$\nf(x, y)=(x-2 y) g(x, y) . \\quad\\quad <1>\n$$\n这里 $g(x, y)$ 是 $n-1$ 次齐次多项式.\n再在 (2) 中令 $a=b=x, c=2 y$, 得\n$$\nf(2 x, 2 y)=-2 f(x+2 y, x) .\n$$\n由 $f$ 的齐次性,得\n$$\n2^n f(x, y)=-2 f(x+2 y, x) .\n$$\n将<1>式代入上式, 得\n$$\n2^{n-1} g(x, y)=g(x+2 y, x) . \\quad\\quad <2>\n$$\n由此利用条件(3), 在<1>中令 $x=1, y=0$, 得\n$$\ng(1,0)=1 \\text {. }\n$$\n由<2>式递推,得\n$$\n\\begin{gathered}\ng(1,1)=2^{n-1}, g(3,1)=4^{n-1}, \\\\\ng(5,3)=8^{n-1}, g(11,5)=16^{n-1}, \\cdots,\n\\end{gathered}\n$$\n即存在无限多对 $(x, y)$, 使 $g(x, y)=(x+y)^{n-1}$ 成立, 由多项式恒等定理, 知\n$$\ng(x, y)=(x+y)^{n-1} .\n$$\n所以 $f(x, y)=(x-2 y)(x+y)^{n-1}$.\n经检验, 可知上述的函数满足题中条件.", + "remark": "", + "figures": [] +} \ No newline at end of file diff --git a/processed_dataset/calculation/0541.json b/processed_dataset/calculation/0541.json new file mode 100644 index 0000000000000000000000000000000000000000..6f7b57ed0263239876be60505172d3dea5fd2f76 --- /dev/null +++ b/processed_dataset/calculation/0541.json @@ -0,0 +1,8 @@ +{ + "source_file": "./raw_volume-zh/volume2/chapter7.tex", + "problem_type": "calculation", + "problem": "例10 解函数方程: 对任意 $x, y \\in \\mathbf{R}$, 都有\n$$\nf(x+y)+f(x-y)=2 f(x) \\cdot \\cos y .\n$$", + "solution": "解:令 $x=0, y=t$, 得\n$$\nf(t)+f(-t)=2 f(0) \\cos t . \\quad\\quad (1)\n$$\n令 $x=\\frac{\\pi}{2}+t, y=\\frac{\\pi}{2}$, 得\n$$\nf(\\pi+t)+f(t)=0 .\\quad\\quad (2)\n$$\n令 $x=\\frac{\\pi}{2}, y=\\frac{\\pi}{2}+t$, 得\n$$\nf(\\pi+t)+f(-t)=-2 f\\left(\\frac{\\pi}{2}\\right) \\sin t .\\quad\\quad (3)\n$$\n由 $((1) +(2)-(3)) / 2$, 得\n$$\nf(t)=f(0) \\cos t+f\\left(\\frac{\\pi}{2}\\right) \\sin t .\n$$\n所以\n$$\nf(x)=a \\cos x+b \\sin x,\n$$\n其中 $a=f(0), b=f\\left(\\frac{\\pi}{2}\\right)$ 为常数.\n经检验, $f(x)=a \\cos x+b \\sin x$ 满足题设条件.", + "remark": "", + "figures": [] +} \ No newline at end of file diff --git a/processed_dataset/calculation/0542.json b/processed_dataset/calculation/0542.json new file mode 100644 index 0000000000000000000000000000000000000000..78fff58525b73fbf5c82ced9501f12115c0343f1 --- /dev/null +++ b/processed_dataset/calculation/0542.json @@ -0,0 +1,8 @@ +{ + "source_file": "./raw_volume-zh/volume2/chapter7.tex", + "problem_type": "calculation", + "problem": "例11 求所有满足下列条件的 $f: \\mathbf{N}_{+} \\rightarrow \\mathbf{R}$ :\n$$\nf(n+m)+f(n-m)=f(3 n), n, m \\in \\mathbf{N}_{+}, n \\geqslant m .\n$$", + "solution": "解:令 $m=0$, 得\n$$\n2 f(n)=f(3 n), n \\in \\mathbf{N}_{+} .\n$$\n令 $m=n=0$, 得 $f(0)=0$.\n令 $m=n$, 得\n$$\nf(2 n)+f(0)=f(3 n),\n$$\n即 $f(2 n)=f(3 n)$.\n于是, 对任意 $m \\in \\mathbf{N}_{+}$, 有\n$$\nf(4 m)=f(6 m)=f(9 m) . \\quad\\quad (1)\n$$\n另一方面, 在原恒等式中令 $n=3 m$, 得\n$$\nf(4 m)+f(2 m)=f(9 m) .\n$$\n因此, 对任意 $m \\in \\mathbf{N}_{+}$, 都有 $f(2 m)=0$. 于是, 对任意 $n \\in \\mathbf{N}_{+}$, 都有\n$$\nf(n)=\\frac{1}{2} f(3 n)=\\frac{1}{2} f(2 n)=0 .\n$$\n故所求的 $f(n) \\equiv 0$ 才能满足题意.", + "remark": "", + "figures": [] +} \ No newline at end of file diff --git a/processed_dataset/calculation/0543.json b/processed_dataset/calculation/0543.json new file mode 100644 index 0000000000000000000000000000000000000000..3c899513fb5e87988804f342dff3cb9a39a74dd0 --- /dev/null +++ b/processed_dataset/calculation/0543.json @@ -0,0 +1,8 @@ +{ + "source_file": "./raw_volume-zh/volume2/chapter7.tex", + "problem_type": "calculation", + "problem": "例12 函数 $f, g: \\mathbf{R} \\rightarrow \\mathbf{R}$ 均非常数, 且满足:\n$$\n\\left\\{\\begin{array}{l}\nf(x+y)=f(x) g(y)+g(x) f(y), \\quad\\quad (1)\\\\\ng(x+y)=g(x) g(y)-f(x) f(y) .\\quad\\quad (2)\n\\end{array}\\right.\n$$\n求 $f(0)$ 与 $g(0)$ 的所有可能值.", + "solution": "解:自然地,令 $x=y=0$, 代入(1)、(2), 得\n$$\n\\left\\{\\begin{array}{l}\nf(0)=2 f(0) g(0), \\quad\\quad (3)\\\\\ng(0)=g^2(0)-f^2(0) . \\quad\\quad (4)\n\\end{array}\\right.\n$$\n若 $f(0) \\neq 0$, 则由 (3), $g(0)=\\frac{1}{2}$. 由 (4), 有\n$$\nf^2(0)=g^2(0)-g(0)=-\\frac{1}{4}<0 \\text {,矛盾! }\n$$\n所以 $f(0)=0$, 因此\n$$\ng(0)=g^2(0) .\n$$\n若 $g(0)=0$, 在 (1) 中令 $y=0$, 得\n$$\nf(x) \\equiv 0 .\n$$\n与题设不符.\n所以 $g(0)=1$.\n综上所述, $f(0)=0, g(0)=1$.\n说明我们经常会遇到函数在某个点上取值可能不确定的情况, 这就需要我们去伪存真,并意识到题目可能会有多解.", + "remark": "", + "figures": [] +} \ No newline at end of file diff --git a/processed_dataset/calculation/0544.json b/processed_dataset/calculation/0544.json new file mode 100644 index 0000000000000000000000000000000000000000..88e08df15473f40eddd5d0b6f390153e2ea0fc17 --- /dev/null +++ b/processed_dataset/calculation/0544.json @@ -0,0 +1,8 @@ +{ + "source_file": "./raw_volume-zh/volume2/chapter7.tex", + "problem_type": "calculation", + "problem": "例13 求所有满足 $f(1)=2$ 和 $f(x y)=f(x) f(y)-f(x+y)+1, x$, $y \\in \\mathbf{Q}$ 的函数 $f: \\mathbf{Q} \\rightarrow \\mathbf{Q}$ ( $\\mathbf{Q}$ 为有理数集).", + "solution": "解:在原恒等式中令 $y=1$, 得\n$$\nf(x)=f(x) \\cdot f(1)-f(x+1)+1, x \\in \\mathbf{Q} .\n$$\n即\n$$\nf(x+1)=f(x)+1 .\n$$\n因此, $f(n)=f(1)+n-1=n+1$.\n另外, 在原恒等式中取 $x=\\frac{1}{n}, y=n, n \\in \\mathbf{Z}$, 有\n$$\n\\begin{gathered}\nf\\left(\\frac{1}{n} \\cdot n\\right)=f\\left(\\frac{1}{n}\\right) \\cdot f(n)-f\\left(\\frac{1}{n}+n\\right)+1, \\\\\n2=f\\left(\\frac{1}{n}\\right)(n+1)-f\\left(\\frac{1}{n}\\right)-n+1 .\n\\end{gathered}\n$$\n即\n$$\n2=f\\left(\\frac{1}{n}\\right)(n+1)-f\\left(\\frac{1}{n}\\right)-n+1\n$$\n所以\n$$\nf\\left(\\frac{1}{n}\\right)=1+\\frac{1}{n}\n$$\n最后, 我们取 $x=p, y=\\frac{1}{q}, p 、 q \\in \\mathbf{Z}, q \\neq 0$, 得\n$$\nf\\left(p, \\frac{1}{q}\\right)=f(p) f\\left(\\frac{1}{q}\\right)-f\\left(p+\\frac{1}{q}\\right)+1 .\n$$\n故 $\\quad f\\left(\\frac{p}{q}\\right)=(p+1)\\left(\\frac{1}{q}+1\\right)-\\frac{1}{q}-p=\\frac{p}{q}+1$.\n所以, 只有函数 $f(x)=x+1$ 满足条件.\n说明这种“爬坡式”的推理技巧称为柯西方法, 后面我们会专门讲述.", + "remark": "", + "figures": [] +} \ No newline at end of file diff --git a/processed_dataset/calculation/0545.json b/processed_dataset/calculation/0545.json new file mode 100644 index 0000000000000000000000000000000000000000..c88cbd3b1599f89ed4d90d36ad49495646493bbf --- /dev/null +++ b/processed_dataset/calculation/0545.json @@ -0,0 +1,8 @@ +{ + "source_file": "./raw_volume-zh/volume2/chapter7.tex", + "problem_type": "calculation", + "problem": "例14 设 $f: \\mathbf{R} \\rightarrow \\mathbf{R}$ 满足如下条件:\n(1)对任意实数 $x 、 y$, 有\n$$\nf(2 x)=f\\left(\\sin \\left(\\frac{\\pi x}{2}+\\frac{\\pi y}{2}\\right)\\right)+f\\left(\\sin \\left(\\frac{\\pi x}{2}-\\frac{\\pi y}{2}\\right)\\right) ;\n$$\n(2)对任意实数 $x 、 y$,有\n$$\nf\\left(x^2-y^2\\right)=(x+y) f(x-y)+(x-y) f(x+y) .\n$$\n求 $f(2012+\\sqrt{2012}+\\sqrt[3]{2012})$ 的值.", + "solution": "解:令 $u=x+y, v=x-y$, 则\n$$\n\\begin{gathered}\nf(u+v)=f\\left(\\sin \\frac{u \\pi}{2}\\right)+f\\left(\\sin \\frac{v \\pi}{2}\\right), \\quad\\quad <1>\\\\\nf(u v)=u f(v)+v f(u) . \\quad\\quad <2>\n\\end{gathered}\n$$\n在<2>中令 $u=0, v=2$, 得 $f(0)=0$.\n令 $u=0$, 代入 (1), 得 $f(v)=f\\left(\\sin \\frac{v \\pi}{2}\\right)$, 同理 $f(u)=f\\left(\\sin \\frac{u \\pi}{2}\\right)$, 所以\n$$\nf(u+v)=f(u)+f(v) . \\quad\\quad <3>\n$$\n在<2>中令 $u=v=1$, 得 $f(1)=0$.\n在<3>中令 $v=1$, 得 $f(u+1)=f(u)$, 从而 $f(2012)=0$.\n在<2>中令 $u=v$, 得\n$$\nf\\left(u^2\\right)=2 u f(u) . \\quad\\quad <4>\n$$\n所以, $f(2012)=2 \\cdot \\sqrt{2012} f(\\sqrt{2012})$, 故 $f(\\sqrt{2012})=0$.\n在<2>中令 $v=u^2$, 得\n$$\nf\\left(u^3\\right)=u f\\left(u^2\\right)+u^2 f(u)=3 u^2 f(u),\n$$\n所以, $f(2012)=3 \\cdot 2012^{\\frac{2}{3}} f(\\sqrt[3]{2012})$, 于是 $f(\\sqrt[3]{2012})=0$.\n所以\n$$\nf(2012+\\sqrt{2012}+\\sqrt[3]{2012})=f(2012)+f(\\sqrt{2012})+f(\\sqrt[3]{2012})=0 .\n$$", + "remark": "", + "figures": [] +} \ No newline at end of file diff --git a/processed_dataset/calculation/0546.json b/processed_dataset/calculation/0546.json new file mode 100644 index 0000000000000000000000000000000000000000..72abf03de75aa1b1ee62e00b62d514c764ed8c9b --- /dev/null +++ b/processed_dataset/calculation/0546.json @@ -0,0 +1,8 @@ +{ + "source_file": "./raw_volume-zh/volume2/chapter7.tex", + "problem_type": "calculation", + "problem": "例17 求所有的函数 $f: \\mathbf{R} \\rightarrow \\mathbf{R}$, 使得对任意实数 $x 、 y 、 z$, 有\n$$\n\\frac{1}{2} f(x y)+\\frac{1}{2} f(x z)-f(x) f(y z) \\geqslant \\frac{1}{4} . \\quad\\quad (1)\n$$", + "solution": "解:题设所给的是一个不等式, 而不是方程, 而且变元有三个, 即 $x 、 y$ 、 $z$. 我们设法通过取一些特殊值来寻求结果.\n令 $x=y=z=1$, 代入 (1), 得\n$$\nf(1)-(f(1))^2 \\geqslant \\frac{1}{4} .\n$$\n所以 $\\left(f(1)-\\frac{1}{2}\\right)^2 \\leqslant 0$.\n故\n$$\nf(1)=\\frac{1}{2} \\text {. } \\quad\\quad (2)\n$$\n令 $y=z=1$, 代入 (1) 并利用 (2), 得\n$$\nf(x)-\\frac{1}{2} f(x) \\geqslant \\frac{1}{4} .\n$$\n所以\n$$\nf(x) \\geqslant \\frac{1}{2} . \\quad\\quad (3)\n$$\n令 $x=y=z=0$, 代入 (1), 得\n$$\nf(0)-f^2(0) \\geqslant \\frac{1}{4} .\n$$\n所以\n$$\nf(0)=\\frac{1}{2} . \\quad\\quad (4)\n$$\n令 $x=0$, 代入 (1) 并利用 (4), 得\n$$\n\\frac{1}{2}-\\frac{1}{2} f(y z) \\geqslant \\frac{1}{4} \\text {. }\n$$\n故\n$$\n\\begin{aligned}\n& f(y z) \\leqslant \\frac{1}{2}, \\\\\n& f(x) \\leqslant \\frac{1}{2} . \\quad\\quad (5)\n\\end{aligned}\n$$\n综合(3)和(5), 即得 $f(x) \\equiv \\frac{1}{2}$.", + "remark": "", + "figures": [] +} \ No newline at end of file diff --git a/processed_dataset/calculation/0547.json b/processed_dataset/calculation/0547.json new file mode 100644 index 0000000000000000000000000000000000000000..9cff489fb3e769cb8b0a0cb3896d64ecbbe57aa7 --- /dev/null +++ b/processed_dataset/calculation/0547.json @@ -0,0 +1,8 @@ +{ + "source_file": "./raw_volume-zh/volume2/chapter7.tex", + "problem_type": "calculation", + "problem": "例18 求所有的函数 $f: \\mathbf{R} \\rightarrow \\mathbf{R}$, 使得等式\n$$\nf([x] y)=f(x)[f(y)] . \\quad\\quad <1>\n$$\n对所有 $x, y \\in \\mathbf{R}$ 成立.\n(这里 $[z]$ 表示不超过实数 $z$ 的最大整数)", + "solution": "解:答案是 $f(x)=C$ (常数), 这里 $C=0$ 或者 $1 \\leqslant C<2$.\n令 $x=0$ 代入 <1>, 得\n$$\nf(0)=f(0)[f(y)] . \\quad\\quad <2>\n$$\n对所有 $y \\in \\mathbf{R}$ 成立.\n于是有如下两种情形:\n(1) 当 $f(0) \\neq 0$ 时, 由 <2> 知, $[f(y)]=1$ 对所有 $y \\in \\mathbf{R}$ 成立.\n所以, <1> 式为 $f([x] y)=f(x)$. 令 $y=0$, 得 $f(x)=f(0)=C \\neq 0$.\n由 $[f(y)]=1=[C]$, 知 $1 \\leqslant C<2$.\n(2) 当 $f(0)=0$ 时, 若存在 $0<\\alpha<1$, 使得 $f(\\alpha) \\neq 0$, 令 $x=\\alpha$ 代入 <1> 式, 得\n$$\n0=f(0)=f(\\alpha)[f(y)]\n$$\n对所有 $y \\in \\mathbf{R}$ 成立, 所以 $[f(y)]=0$ 对所有 $y \\in \\mathbf{R}$ 成立.\n令 $x=1$ 代入 <1> 式, 得 $f(y)=0$ 对所有 $y \\in \\mathbf{R}$ 成立,这与 $f(\\alpha) \\neq 0$ 矛盾.\n所以, 我们有 $f(\\alpha)=0,0 \\leqslant \\alpha<1$. 对于任意实数 $z$, 存在整数 $N$, 使得 $\\alpha=\\frac{z}{N} \\in[0,1)$. 由 <1> 式,有\n$$\nf(z)=f([N] \\alpha)=f(N)[f(\\alpha)]=0\n$$\n对所有 $z \\in \\mathbf{R}$ 成立.\n经检验, $f(x)=C$ (常数), 这里 $C=0$ 或者 $1 \\leqslant C<2$ 满足题设.", + "remark": "", + "figures": [] +} \ No newline at end of file diff --git a/processed_dataset/calculation/0548.json b/processed_dataset/calculation/0548.json new file mode 100644 index 0000000000000000000000000000000000000000..e31e83f968a1f06755632d35c3c8b2b6f60f4897 --- /dev/null +++ b/processed_dataset/calculation/0548.json @@ -0,0 +1,8 @@ +{ + "source_file": "./raw_volume-zh/volume2/chapter7.tex", + "problem_type": "calculation", + "problem": "例21 设 $f(x)$ 满足柯西方程, 且在 $\\mathbf{R}$ 上连续, 求 $f(x)$.", + "solution": "解:利用 $\\circledast$, 用数学归纳法易得: 对任意正整数 $n$ 和实数 $x$,有\n$$\nf(n x)=n f(x) . \\quad\\quad (1)\n$$\n令 $x=1$, 有 $f(n)=n f(1)$.\n记 $a=f(1)$, 那么对一切正整数 $n$, 有 $f(n)=a n$.\n再令 $x=\\frac{m}{n}$ ( $m 、 n$ 是正整数 $)$ 为正有理数, 有\n$$\nn f\\left(\\frac{m}{n}\\right)=f\\left(n \\cdot \\frac{m}{n}\\right)=f(m)=f(m \\cdot 1)=m f(1)=a m,\n$$\n所以 $f\\left(\\frac{m}{n}\\right)=a \\cdot \\frac{m}{n}$.\n又 $\\quad f(0)=f(0+0)=f(0)+f(0)$,\n所以\n$$\nf(0)=0=a \\cdot 0 \\text {. }\n$$\n$$\nf(0)=f(x-x)=f(x)+f(-x),\n$$\n所以 $f(x)=-f(-x)$,\n即 $f(x)$ 是奇函数.\n于是从上面可知, 对一切有理数 $r$, 有 $f(r)=a r$.\n所以, $g(x)=f(x)-a x$ 在有理数集上处处等于 0 . 因 $f(x)$ 连续,所以 $g(x)$ 也连续,进而 $g(x)=0$.\n事实上, 若 $g\\left(x_0\\right) \\neq 0$, 由 $g$ 的连续性知, 存在 $x_0$ 的一个邻域 $u\\left(x_0, \\varepsilon\\right)$, 使 $g(x)$ 在此邻域内处处不等于 0 . 这与它在有理点为 0 矛盾.\n所以,对一切实数 $x$, 都有: $f(x)=a x$, 其中 $a=f(1)$.\n说明在本题中, 我们要求 $f(x)$ 连续, 其实这个要求太强了.\n我们只要 $f(x)$ 在某个区间内有界, 或 $f(x)$ 在某区间内单调均可导出 $f(x)=a x(a=f(1))$.", + "remark": "", + "figures": [] +} \ No newline at end of file diff --git a/processed_dataset/calculation/0549.json b/processed_dataset/calculation/0549.json new file mode 100644 index 0000000000000000000000000000000000000000..762b4b32ab0fb3fbb83c22dc72665470b6a4f7d3 --- /dev/null +++ b/processed_dataset/calculation/0549.json @@ -0,0 +1,8 @@ +{ + "source_file": "./raw_volume-zh/volume2/chapter7.tex", + "problem_type": "calculation", + "problem": "例23 设 $f: \\mathbf{Q}^{+} \\rightarrow \\mathbf{Q}^{+}$( $\\mathbf{Q}^{+}$为正有理数的全体) 满足\n$$\nf(x)=f(x f(y)) y . \\quad\\quad (1)\n$$\n试求其一个解.", + "solution": "解:在(1)中取 $x=y=1$, 得 $f(1)=f(f(1))$.\n在(1)中取 $y=f(1)$, 得\n$$\n\\begin{aligned}\nf(x) & =f(x f(f(1))) f(1)=f(x f(1)) f(1) \\\\\n& =f(x) f(1) .\n\\end{aligned}\n$$\n于是 $f(1)=1$. 因此, 再从 (1) 中取 $x=1$, 得\n$$\ny f(f(y))=f(1)=1 . \\quad\\quad (2)\n$$\n另外,从(1)中取 $y=f(t)$, 得\n$$\nf(x)=f(t) f(x f(f(t)))=f(t) f\\left(\\frac{x}{t}\\right) .\n$$\n继续取 $x=s t$, 得\n$$\nf(s t)=f(t) f(s) . \\quad\\quad (3)\n$$\n显然, 若 $f(x)$ 满足 (2) 和 (3), 则 $f(x)$ 也必满足(1). 事实上, $f(x f(y))=f(x) f(f(y))=\\frac{f(x)}{y}$. 我们利用这一充要条件 (2) 和 (3) 来构造解.\n首先, 当 $x$ 为素数时构造 $f(x)$. 记 $p_k$ 是从小到大的第 $k$ 个素数.\n令\n$$\nf\\left(p_k\\right)= \\begin{cases}p_{k+1}, & \\text { 当 } k=2 m+1, m=0,1,2, \\cdots \\text { 时; } \\\\ \\frac{1}{p_{k-1}}, & \\text { 当 } k=2 m, m=1,2, \\cdots \\text { 时.\n}\\end{cases}\n$$\n它显然满足(2).\n其次,对任意正整数 $x$ 定义 $f(x)$. 令 $x=p_1^{\\alpha_1} p_2^{\\alpha_2} \\cdots p_s^{\\alpha_s}$, 其中 $\\alpha_1, \\alpha_2, \\cdots, \\alpha_s$ 是非负整数.\n由(3), 得\n$$\nf(x)=\\left(f\\left(p_1\\right)\\right)^{\\alpha_1}\\left(f\\left(p_2\\right)\\right)^{a_2 \\cdots\\left(f\\left(p_s\\right)\\right)^{a_s} .} \n$$\n最后, 对任意正有理数 $x$, 即 $x=\\frac{n}{m} \\in \\mathbf{Q}^{+}$, 其中 $m, n \\in \\mathbf{N}_{+}$.\n由(3), 得\n$$\n\\begin{gathered}\nf(n)=f(m x)=f(m) f(x) . \\\\\nf(x)=\\frac{f(n)}{f(m)} .\n\\end{gathered}\n$$\n从而这样, 最终我们给出了原函数方程在 $\\mathbf{Q}^{+}$上的一个解.", + "remark": "", + "figures": [] +} \ No newline at end of file diff --git a/processed_dataset/calculation/0550.json b/processed_dataset/calculation/0550.json new file mode 100644 index 0000000000000000000000000000000000000000..c68a293875a9dbe15afc1ae6dce1f323e8c1a4f5 --- /dev/null +++ b/processed_dataset/calculation/0550.json @@ -0,0 +1,8 @@ +{ + "source_file": "./raw_volume-zh/volume2/chapter7.tex", + "problem_type": "calculation", + "problem": "例24 已知定义的正整数集 $\\mathbf{N}_{+}$上的函数 $f(n)$ 满足: $f(n+2)=f(n+1)+f(n)$, 且 $f(1)=f(2)=1$. 求 $f(n)$.", + "solution": "解:所给递归函数的特征方程为 $x^2=x+1$. 解方程, 得 $x_{1,2}=\\frac{1 \\pm \\sqrt{5}}{2}$.\n所以 $\\quad f(n)=c_1\\left(\\frac{1+\\sqrt{5}}{2}\\right)^n+c_2\\left(\\frac{1-\\sqrt{5}}{2}\\right)^n$.\n由初始条件 $f(1)=f(2)=1$, 得\n$$\n\\begin{gathered}\n\\frac{1+\\sqrt{5}}{2} c_1+\\frac{1-\\sqrt{5}}{2} c_2=1 \\\\\n\\left(\\frac{1+\\sqrt{5}}{2}\\right)^2 c_1+\\left(\\frac{1-\\sqrt{5}}{2}\\right)^2 c_2=1 .\n\\end{gathered}\n$$\n解方程组, 得 $c_1=\\frac{1}{\\sqrt{5}}, c_2=-\\frac{1}{\\sqrt{5}}$. 所以\n$$\nf(n)=\\frac{1}{\\sqrt{5}}\\left[\\left(\\frac{1+\\sqrt{5}}{2}\\right)^n-\\left(\\frac{1-\\sqrt{5}}{2}\\right)^n\\right] .\n$$\n这就是著名的斐波那契数列.", + "remark": "", + "figures": [] +} \ No newline at end of file diff --git a/processed_dataset/calculation/0551.json b/processed_dataset/calculation/0551.json new file mode 100644 index 0000000000000000000000000000000000000000..4c24e387162cd7c4cf57fd06ccfecb084de432a7 --- /dev/null +++ b/processed_dataset/calculation/0551.json @@ -0,0 +1,8 @@ +{ + "source_file": "./raw_volume-zh/volume2/chapter7.tex", + "problem_type": "calculation", + "problem": "例26 已知函数 $f$ 定义在正整数集 $\\mathbf{N}_{+}$上, $f(1)=\\frac{3}{2}$, 并且对任意正整数 $m 、 n$,均有\n$$\nf(m+n)=\\left(1+\\frac{n}{m+1}\\right) f(m)+\\left(1+\\frac{m}{n+1}\\right) f(n)+m^2 n+m n+m n^2 .\n$$\n求 $f$.", + "solution": "解:令 $n=1$, 得\n$$\nf(m+1)=\\left(1+\\frac{1}{m+1}\\right) f(m)+\\left(1+\\frac{m}{2}\\right) \\cdot \\frac{3}{2}+m^2+2 m .\n$$\n整理, 得\n$$\n\\frac{f(m+1)}{m+2}-\\frac{f(m)}{m+1}=m+\\frac{3}{4} .\n$$\n于是,利用累差求和的方法,有\n$$\n\\begin{aligned}\n\\sum_{k=1}^{m-1}\\left(\\frac{f(k+1)}{k+2}-\\frac{f(k)}{k+1}\\right) & =\\sum_{k=1}^{m-1}\\left(k+\\frac{3}{4}\\right) \\\\\n& =\\frac{(m-1) m}{2}+\\frac{3}{4}(m-1) .\n\\end{aligned}\n$$\n所以\n$$\n\\frac{f(m)}{m+1}-\\frac{f(1)}{2}=\\frac{1}{4}(m-1)(2 m+3),\n$$\n故\n$$\nf(m)=\\frac{1}{4} m(m+1)(2 m+1) .\n$$\n因此所求的函数即为\n$$\nf(m)=\\frac{1}{4} m(m+1)(2 m+1) .\n$$", + "remark": "", + "figures": [] +} \ No newline at end of file diff --git a/processed_dataset/calculation/0552.json b/processed_dataset/calculation/0552.json new file mode 100644 index 0000000000000000000000000000000000000000..5568b5c8bd76ccb07563cfd288693202e830c122 --- /dev/null +++ b/processed_dataset/calculation/0552.json @@ -0,0 +1,8 @@ +{ + "source_file": "./raw_volume-zh/volume2/chapter7.tex", + "problem_type": "calculation", + "problem": "例27 设 $f: \\mathbf{Q} \\rightarrow \\mathbf{Q}(\\mathbf{Q}$ 为有理数集 $)$ 且\n$$\nf(x+y)=f(x)+f(y)+4 x y \\text {, 对任意 } x, y \\in \\mathbf{Q} \\text {. } \\quad\\quad (1)\n$$\n如果 $f(-1) f(1) \\geqslant 4$, 求 $f(x)$.", + "solution": "解:取 $x=y=0$, 由 (1), 得 $f(0)=0$.\n在(1)中取 $x=1$ 和 $y=-1$, 得\n$$\nf(0)=f(1)+f(-1)-4,\n$$\n即 $f(1)+f(-1)=4$.\n因为 $f(-1) f(1) \\geqslant 4$, 所以 $f(1)$ 和 $f(-1)$ 皆为正数, 从而\n$$\n4=f(1)+f(-1) \\geqslant 2 \\sqrt{f(1) f(-1)} \\geqslant 4 .\n$$\n上式等号成立,所以 $f(1)=f(-1)=2$.\n再在(1)中取 $y=1$, 得\n$$\nf(x+1)=f(x)+4 x+2 . \\quad\\quad (2)\n$$\n对(2)递推,有\n$$\n\\begin{aligned}\n& f(2)=f(1+1)=f(1)+4 \\cdot 1+2, \\\\\n& f(3)=f(2+1)=f(2)+4 \\cdot 2+2 \\text {, } \\\\\n& f(n)=f(n-1+1)=f(n-1)+4(n-1)+2 . \\\\\n&\n\\end{aligned}\n$$\n因此\n$$\nf(n)=f(1)+4(1+2+\\cdots+(n-1))+2(n-1)=2 n^2 . \\quad\\quad (3)\n$$\n又因为 $\\quad f\\left(\\frac{k}{n}+\\frac{1}{n}\\right)=f\\left(\\frac{k}{n}\\right)+f\\left(\\frac{1}{n}\\right)+\\frac{4 k}{n^2}$,\n即\n$$\nf\\left(\\frac{k+1}{n}\\right)-f\\left(\\frac{k}{n}\\right)=f\\left(\\frac{1}{n}\\right)+\\frac{4 k}{n^2} .\n$$\n利用 $f(1)=2$, 对 $k$ 从 1 到 $n-1$ 求和,得\n$$\nf\\left(\\frac{1}{n}\\right)=\\frac{2}{n^2} . \\quad\\quad (4)\n$$\n同理对 $k$ 从 1 到 $m-1$ 求和,得\n$$\nf\\left(\\frac{m}{n}\\right)=2 \\frac{m^2}{n^2}\n$$\n最后, 令 $x=\\frac{m}{n}, y=-\\frac{m}{n}$, 由 (1), 有\n$$\nf(0)=f\\left(\\frac{m}{n}-\\frac{m}{n}\\right)=f\\left(\\frac{m}{n}\\right)+f\\left(-\\frac{m}{n}\\right)-\\frac{4 m^2}{n^2},\n$$\n故\n$$\nf\\left(-\\frac{m}{n}\\right)=2 \\cdot \\frac{m^2}{n^2}\n$$\n因此对所有 $x \\in \\mathbf{Q}$, 有 $f(x)=2 x^2$.", + "remark": "", + "figures": [] +} \ No newline at end of file diff --git a/processed_dataset/calculation/0553.json b/processed_dataset/calculation/0553.json new file mode 100644 index 0000000000000000000000000000000000000000..671a4e740516f4f1f53f0307240a35b617a463ec --- /dev/null +++ b/processed_dataset/calculation/0553.json @@ -0,0 +1,8 @@ +{ + "source_file": "./raw_volume-zh/volume2/chapter7.tex", + "problem_type": "calculation", + "problem": "例29 设 $f(x)$ 是定义在 $\\mathbf{R}$ 上的函数, 若 $f(0)=2008$, 且对任意 $x \\in \\mathbf{R}$, 满足\n$$\nf(x+2)-f(x) \\leqslant 3 \\cdot 2^x, f(x+6)-f(x) \\geqslant 63 \\cdot 2^x,\n$$\n求 $f(2008)$ 的值.", + "solution": "解:法一由题设条件知\n$$\n\\begin{aligned}\nf(x+2)-f(x)= & -(f(x+4)-f(x+2))-(f(x+6)-f(x+4))+ \\\\\n& (f(x+6)-f(x)) \\\\\n\\geqslant & -3 \\cdot 2^{x+2}-3 \\cdot 2^{x+4}+63 \\cdot 2^x=3 \\cdot 2^x,\n\\end{aligned}\n$$\n因此有 $f(x+2)-f(x)=3 \\cdot 2^x$, 故\n$$\n\\begin{aligned}\nf(2008)= & f(2008)-f(2006)+f(2006)-f(2004)+\\cdots+ \\\\\n& f(2)-f(0)+f(0) \\\\\n= & 3 \\cdot\\left(2^{2006}+2^{2004}+\\cdots+2^2+1\\right)+f(0) \\\\\n= & 3 \\cdot \\frac{4^{1003+1}-1}{4-1}+f(0) \\\\\n= & 2^{2008}+2007 .\n\\end{aligned}\n$$\n解法二令 $g(x)=f(x)-2^x$, 则\n$$\n\\begin{aligned}\n& g(x+2)-g(x)=f(x+2)-f(x)-2^{x+2}+2^x \\leqslant 3 \\cdot 2^x-3 \\cdot 2^x=0, \\\\\n& g(x+6)-g(x)=f(x+6)-f(x)-2^{x+6}+2^x \\geqslant 63 \\cdot 2^x-63 \\cdot 2^x=0,\n\\end{aligned}\n$$\n即\n$$\ng(x+2) \\leqslant g(x), g(x+6) \\geqslant g(x),\n$$\n故 $\\quad g(x) \\leqslant g(x+6) \\leqslant g(x+4) \\leqslant g(x+2) \\leqslant g(x)$,\n得 $g(x)$ 是周期为 2 的周期函数,所以\n$$\nf(2008)=g(2008)+2^{2008}=g(0)+2^{2008}=2^{2008}+2007 .\n$$", + "remark": "", + "figures": [] +} \ No newline at end of file diff --git a/processed_dataset/calculation/0554.json b/processed_dataset/calculation/0554.json new file mode 100644 index 0000000000000000000000000000000000000000..7b026ac782d1224bc68c3d3ddb0edc609f04c4e4 --- /dev/null +++ b/processed_dataset/calculation/0554.json @@ -0,0 +1,8 @@ +{ + "source_file": "./raw_volume-zh/volume2/chapter7.tex", + "problem_type": "calculation", + "problem": "例31 求所有的函数 $f:(0,+\\infty) \\rightarrow(0,+\\infty)$, 满足对所有的正实数 $w 、 x 、 y 、 z, w x=y z$, 都有\n$$\n\\frac{(f(w))^2+(f(x))^2}{f\\left(y^2\\right)+f\\left(z^2\\right)}=\\frac{w^2+x^2}{y^2+z^2} .\n$$", + "solution": "解:令 $w=x=y=z=1$, 得 $(f(1))^2=f(1)$, 所以 $f(1)=1$.\n对任意 $t>0$, 令 $w=t, x=1, y=z=\\sqrt{t}$, 得 $\\frac{(f(t))^2+1}{2 f(t)}=\\frac{t^2+1}{2 t}$, 去分母整理得 $(t f(t)-1)(f(t)-t)=0$, 所以, 对每个 $t>0$,\n$$\nf(t)=t \\text {, 或者 } f(t)=\\frac{1}{t} . \\quad\\quad \\circledast\n$$\n若存在 $b, c \\in(0,+\\infty)$, 使得 $f(b) \\neq b, f(c) \\neq \\frac{1}{c}$, 则由 $\\circledast$ 式知, $b 、 c$ 都不等于 1 , 且 $f(b)=\\frac{1}{b}, f(c)=c$. 令 $w=b, x=c, y=z=\\sqrt{b c}$, 则所以\n$$\n\\begin{aligned}\n& \\frac{\\frac{1}{b^2}+c^2}{2 f(b c)}=\\frac{b^2+c^2}{2 b c} \\\\\n& f(b c)=\\frac{c+b^2 c^3}{b\\left(b^2+c^2\\right)} .\n\\end{aligned}\n$$\n因为 $f(b c)=b c$, 或者 $f(b c)=\\frac{1}{b c}$. 若 $f(b c)=b c$, 则\n$$\nb c=\\frac{c+b^2 c^3}{b\\left(b^2+c^2\\right)}\n$$\n得 $b^4 c=c, b=1$, 矛盾! 若 $f(b c)=\\frac{1}{b c}$, 则\n$$\n\\frac{1}{b c}=\\frac{c+b^2 c^3}{b\\left(b^2+c^2\\right)}\n$$\n得 $b^2 c^4=b^2, c=1$, 矛盾!\n所以, 或者 $f(x)=x, x \\in(0,+\\infty)$, 或者 $f(x)=\\frac{1}{x}, x \\in(0,+\\infty)$.\n经检验, $f(x)==x, x \\in(0,+\\infty)$ 和 $f(x)=\\frac{1}{x}, x \\in(0,+\\infty)$ 都满足要求.", + "remark": "", + "figures": [] +} \ No newline at end of file diff --git a/processed_dataset/calculation/0555.json b/processed_dataset/calculation/0555.json new file mode 100644 index 0000000000000000000000000000000000000000..2b39c400a76373e6af05cb5a77e6a1e86783e1f2 --- /dev/null +++ b/processed_dataset/calculation/0555.json @@ -0,0 +1,8 @@ +{ + "source_file": "./raw_volume-zh/volume2/exercise1.tex", + "problem_type": "calculation", + "problem": "问题2 设集合\n$$\n\\begin{aligned}\n& M=\\{x \\mid 1 \\leqslant x \\leqslant 9, x \\in N\\}, \\\\\n& F=\\{(a, b, c, d) \\mid a, b, c, d \\in M\\} .\n\\end{aligned}\n$$\n定义 $F$ 到 $Z$ 的映射\n$$\nf:(a, b, c, d) \\mapsto a b-c d .\n$$\n若 $u 、 v 、 x 、 y$ 都是 $M$ 中的元素, 且满足\n$$\n\\begin{aligned}\n& f:(u, v, x, y) \\mapsto 39, \\\\\n& (u, y, x, v) \\mapsto 66 .\n\\end{aligned}\n$$\n求 $x 、 y 、 u 、 v$.", + "solution": "由题意, 得 $\\left\\{\\begin{array}{l}u v-x y=39, \\quad\\quad (1) \\\\ u y-x v=66 .\\quad\\quad (2) \\end{array}\\right.$\n(1) + (2), (2) - (1), 得 $(u-x)(v+ y)=3 \\times 5 \\times 7 \\cdots$, $(y-v)(u+x)=3 \\times 3 \\times 3 \\cdots \\quad (4)$. 因为 $|u-x|<9$, $0f(10)$, 故 $f(8) \\leqslant f(u) \\leqslant f(6)$, 即 $15 \\leqslant f(u) \\leqslant 15 \\frac{2}{3}$.", + "remark": "", + "figures": [] +} \ No newline at end of file diff --git a/processed_dataset/calculation/0562.json b/processed_dataset/calculation/0562.json new file mode 100644 index 0000000000000000000000000000000000000000..fc391e29a367be80d84dd84dd971225d530f36db --- /dev/null +++ b/processed_dataset/calculation/0562.json @@ -0,0 +1,8 @@ +{ + "source_file": "./raw_volume-zh/volume2/exercise1.tex", + "problem_type": "calculation", + "problem": "问题9 已知函数 $f(x)$ 对于任意实数 $x$, 都有\n$$\nf(x)=f(398-x)=f(2158-x)=f(3214-x),\n$$\n$\\therefore$ 问: 函数值列 $f(0), f(1), f(2), \\cdots, f(999)$ 中最多有多少个不同的值?", + "solution": "由 $f(398-x)=f(2158-x)$, 得 $f(x)=f(x+1760)$. 由 $f(2158- x)=f(3214-x)$, 得 $f(x)=f(x+1056)$. 故 $f(x)=f(x+1056)= f(x+2112)=f(x+352)$. 从而 $f(x)$ 为周期函数, 且它的一个周期为 352 . 又由于 $f(x)=f(398-x)$, 故 $f(x)$ 关于 $x=199$ 对称.\n于是当 $f$ 在 $x \\in[23,199]$ 定义后, 由 $f(x)$ 的对称性知 $x \\in[199,375]$ 时, $f$ 亦可定义; 再由 $f(x)$ 的周期性知对任意 $x \\in \\mathbf{R}, f$ 可定义.\n故在一个周期内, 每一个函数值至少对应两个不同的 $x$, 其中 $x \\neq 199+352 k(k \\in \\mathbf{Z})$. 因此, $f(0), f(1), \\cdots, f(999)$ 至多有 $\\frac{1}{2} \\cdot 352+1=177$ 个不同的值, 且当 $f(23), f(24), \\cdots, f(199)$ 两两不等时, 177 可以取到.\n因此, 所求的最大值为 177.", + "remark": "", + "figures": [] +} \ No newline at end of file diff --git a/processed_dataset/calculation/0563.json b/processed_dataset/calculation/0563.json new file mode 100644 index 0000000000000000000000000000000000000000..3ed95264d21764ad3205b170fefa98ae781b4aea --- /dev/null +++ b/processed_dataset/calculation/0563.json @@ -0,0 +1,8 @@ +{ + "source_file": "./raw_volume-zh/volume2/exercise1.tex", + "problem_type": "calculation", + "problem": "问题10 设 $f$ 为 $\\mathbf{R}^{+} \\rightarrow \\mathbf{R}^{+}$的函数, 对任意正实数 $x, f(3 x)=3 f(x)$, 且\n$$\nf(x)=1-|x-2|, 1 \\leqslant x \\leqslant 3 .\n$$\n求最小的实数 $x$, 使得 $f(x)=f(2004)$.", + "solution": "由已知条件得 $f(x)=\\left\\{\\begin{array}{l}x-1, 当1\\leqslant x \\leqslant 2时; \\\\ 3-x , 当2\\leqslant x \\leqslant 3时.\n\\end{array}\\right.$ \n当 $3\\leq x\\leq6$ 时, 令 $t=\\frac{x}{3}$, 则 $1 \\leqslant t \\leqslant 2$, 此时 $f(x)=f(3 t)=3 f(t)=3(t-1)=x-3$. 即得 $f(x)=|x-3|, 2 \\leqslant x \\leqslant 6$. 当 $6 \\leqslant x \\leqslant 18$ 时, 令 $t=\\frac{x}{3}$, 则 $2 \\leqslant t \\leqslant 6$,\n于是 $f(x)=f(3 t)=3 f(t)=3|t-3|=|x-9|$. 依次类推, 得\n$$\nf(x)= \\begin{cases}x-1, & \\text { 当 } 1 \\leqslant x \\leqslant 2 \\text { 时; } \\\\ |x-3|, & \\text { 当 } 2 \\leqslant x \\leqslant 6 \\text { 时; } \\\\ |x-9|, & \\text { 当 } 6 \\leqslant x \\leqslant 18 \\text { 时; } \\\\ |x-27|, & \\text { 当 } 18 \\leqslant x \\leqslant 54 \\text { 时; } \\\\ |x-243|, & \\text { 当 } 54 \\leqslant x \\leqslant 162 \\text { 时; } \\\\ |x-729|, & \\text { 当 } 162 \\leqslant x \\leqslant 486 \\text { 时; } \\\\ |x-2187|, & \\text { 当 } 1458 \\leqslant x \\leqslant 1458 \\text { 时; }\\end{cases}\n$$\n所以 $f(2004)=2187-2004=183$. 由于 $162-81<183,486-243>183$, 而 $243-162<183$, 所以, 最小的满足 $f(x)=f(2004)$ 的实数 $x=243+ 183=426$.", + "remark": "", + "figures": [] +} \ No newline at end of file diff --git a/processed_dataset/calculation/0564.json b/processed_dataset/calculation/0564.json new file mode 100644 index 0000000000000000000000000000000000000000..9ce759b5d42b05fce078f1c3f24660f978bf6ad0 --- /dev/null +++ b/processed_dataset/calculation/0564.json @@ -0,0 +1,8 @@ +{ + "source_file": "./raw_volume-zh/volume2/exercise2.tex", + "problem_type": "calculation", + "problem": "问题1 已知 $f(x)=x^5+a x^3+b x+c \\sin x+8$ (其中 $a 、 b 、 c$ 是实常数), 且 $f(-2)=10$, 求 $f(2)$ 的值.", + "solution": "设 $g(x)=f(x)-8$, 则 $g(x)$ 是奇函数, 且 $g(-2)=f(-2)-8=10- 8=2$. 所以 $g(2)=-g(-2)=-2$. 从而 $f(2)=g(2)+8=-2+8=6$.", + "remark": "", + "figures": [] +} \ No newline at end of file diff --git a/processed_dataset/calculation/0565.json b/processed_dataset/calculation/0565.json new file mode 100644 index 0000000000000000000000000000000000000000..168186ef73868ca8ec86527b4d8b4a27fcd84b2f --- /dev/null +++ b/processed_dataset/calculation/0565.json @@ -0,0 +1,8 @@ +{ + "source_file": "./raw_volume-zh/volume2/exercise2.tex", + "problem_type": "calculation", + "problem": "问题3 设 $f(x)=\\frac{2 x+3}{x-1}$, 函数 $g(x)$ 的图象与 $y=f^{-1}(x+1)$ 的图象关于 $y= x$ 对称, 求 $g(3)$ 的值.", + "solution": "设 $g(3)=a$, 则 $f^{-1}(a+1)=3$, 由此得 $f(3)=a+1$, 即 $a+1=\\frac{9}{2}$, 故 $a=\\frac{7}{2}$.", + "remark": "", + "figures": [] +} \ No newline at end of file diff --git a/processed_dataset/calculation/0566.json b/processed_dataset/calculation/0566.json new file mode 100644 index 0000000000000000000000000000000000000000..9fbe1bd2b997da1dcbc6dbca60c9d841481fd613 --- /dev/null +++ b/processed_dataset/calculation/0566.json @@ -0,0 +1,8 @@ +{ + "source_file": "./raw_volume-zh/volume2/exercise2.tex", + "problem_type": "calculation", + "problem": "问题5 已知 $f(x)$ 是定义在 $\\mathbf{R}$ 上的函数, $f(1)=1$, 且对任意 $x \\in \\mathbf{R}$, 都有\n$$\nf(x+5) \\geqslant f(x)+5, f(x+1) \\leqslant f(x)+1 .\n$$\n若 $g(x)==f(x)+1-x$, 求 $g(2002)$.", + "solution": "因为 $f(x)=g(x)+x-1$, 所以 $g(x+5)+(x+5)-1 \\geqslant g(x)+ (x-1)+5, g(x+1)+(x+1)-1 \\leqslant g(x)+(x-1)+1$, 即 $g(x+5) \\geqslant g(x), g(x+1) \\leqslant g(x)$. \n故 $g(x) \\leqslant g(x+5) \\leqslant g(x+4) \\leqslant g(x+3) \\leqslant g(x+2) \\leqslant g(x+1) \\leqslant g(x)$, 所以 $g(x+1)=g(x)$, 即 $g(x)$ 是周期为 1 的周期函数,故 $g(2002)=g(1)=1$.", + "remark": "", + "figures": [] +} \ No newline at end of file diff --git a/processed_dataset/calculation/0567.json b/processed_dataset/calculation/0567.json new file mode 100644 index 0000000000000000000000000000000000000000..ced005748b8c0247c36f034ad9e3c30b1087ba4f --- /dev/null +++ b/processed_dataset/calculation/0567.json @@ -0,0 +1,8 @@ +{ + "source_file": "./raw_volume-zh/volume2/exercise2.tex", + "problem_type": "calculation", + "problem": "问题6 设函数 $y=f(x)$ 对一切实数 $x$ 都满足\n$$\nf(3+x)=f(3-x),\n$$\n且方程 $f(x)=0$ 恰有 6 个不同的实根, 求这 6 个实根的和.", + "solution": "由题设知, 函数 $f(x)$ 的图象是关于直线 $x=3$ 对称的,因而每两个关于 $x=3$ 对称的根的和为 6 . 于是这 6 个根的和为 $3 \\times 6=18$. 其实, 下面的说法似乎更自然一些.\n若 $3+x_0$ 为 $f(x)=0$ 的根, 则由题设知 $f\\left(3-x_0\\right)=f\\left(3+x_0\\right)=0$, 所以 $3-x_0$ 也是 $f(x)=0$ 的根.\n于是可设这 6 个根为 $3 \\pm x_1, 3 \\pm x_2, 3 \\pm x_3$, 故它们的和为 18 .", + "remark": "", + "figures": [] +} \ No newline at end of file diff --git a/processed_dataset/calculation/0568.json b/processed_dataset/calculation/0568.json new file mode 100644 index 0000000000000000000000000000000000000000..4f7e3f81a296745f6c25adfc2877ae16acca9d01 --- /dev/null +++ b/processed_dataset/calculation/0568.json @@ -0,0 +1,8 @@ +{ + "source_file": "./raw_volume-zh/volume2/exercise2.tex", + "problem_type": "calculation", + "problem": "问题7 已知 $f(x)$ 是定义在 $\\mathbf{R}$ 上的奇函数, 当 $x \\geqslant 0$ 时, $f(x)=2 x-x^2$. 问 : 是否存在实数 $a 、 b(a \\neq b)$, 使 $f(x)$ 在 $[a, b]$ 上的值域为 $\\left[\\frac{1}{b}, \\frac{1}{a}\\right]$ ?", + "solution": "利用条件: $f(x)$ 在 $\\mathbf{R}$ 上为奇函数,且 $x \\geqslant 0$ 时, $f(x)=2 x-x^2$, 可以 $\\left\\{\\begin{array}{l}a-b<0, \\\\ \\frac{a-b}{a b}<0\\end{array} \\Leftrightarrow a b>0\\right.$, 故只能有 $00$ 有意义,且满足下列条件:\n(1) 对于 $x>0$, 有 $f(x) f\\left[f(x)+\\frac{1}{x}\\right]=1$;\n(2) $f(x)$ 在 $(0,+\\infty)$ 上递增.\n求 $f(1)$ 的值.", + "solution": "设 $f(1)=a$, 则当 $x=1$ 时, 由条件 (1) 得 $f(a+1)=\\frac{1}{a}$. 令 $x= a+1$, 由条件(1) 得 $f(a+1) f\\left[f(a+1)+\\frac{1}{a+1}\\right]=1$, 即 $f\\left(\\frac{1}{a}+\\frac{1}{a+1}\\right)= a=f(1)$. 由于 $f(x)$ 在 $(0,+\\infty)$ 上是递增的, 所以 $\\frac{1}{a}+\\frac{1}{a+1}=1$, 解方程得 $a=\\frac{1 \\pm \\sqrt{5}}{2}$. 若 $a=\\frac{1+\\sqrt{5}}{2}$, 则 $10$ 知, 对任意的 $k$ 值, 抛物线与 $x$ 轴总有两个交点.\n设抛物线与 $x$ 轴的两个交点的横坐标分别为 $x_1 、 x_2$, 那么 $|A B|=\\left|x_2-x_1\\right|=\\sqrt{\\left(x_2-x_1\\right)^2}= \\sqrt{\\left(x_1+x_2\\right)^2-4 x_1 x_2}=\\sqrt{k^2+2 k+5}$. 又抛物线的顶点坐标是$C\\left(\\frac{k-1}{2},-\\frac{k^2+2 k+5}{4}\\right)$, 所以, $S_{\\triangle A B C}=\\frac{1}{2} \\sqrt{k^2+2 k+5} \\cdot\\left|-\\frac{k^2+2 k+5}{4}\\right|= \\frac{1}{8} \\sqrt{\\left(k^2+2 k+5\\right)^3}$. 因为 $k^2+2 k+5=(k+1)^2+4 \\geqslant 4$, 当且仅当 $k=-1$ 时等号成立, 所以 $S_{\\triangle A B C} \\geqslant \\frac{1}{8} \\sqrt{4^3}=1$. 故 $\\triangle A B C$ 的面积的最小值为 1 .", + "remark": "", + "figures": [] +} \ No newline at end of file diff --git a/processed_dataset/calculation/0572.json b/processed_dataset/calculation/0572.json new file mode 100644 index 0000000000000000000000000000000000000000..5d9aa467afcda99657439b65d9b7678d20599543 --- /dev/null +++ b/processed_dataset/calculation/0572.json @@ -0,0 +1,8 @@ +{ + "source_file": "./raw_volume-zh/volume2/exercise3.tex", + "problem_type": "calculation", + "problem": "问题2 设二次函数 $f(x)=x^2+x+a(a>0)$ 满足 $f(m)<0$, 判断 $f(m+1)$ 的符号.", + "solution": "因为 $a>0, f(m)=m^2+m+a<0$, 所以 $m^2+m=m(m+1)<0$, 于是 $\\left\\{\\begin{array}{l}m<0, \\\\ m+1>0 .\\end{array}\\right.$ 所以 $f(m+1)=(m+1)^2+m+1+a>0$, 故 $f(m+1)$ 的符号为正.", + "remark": "", + "figures": [] +} \ No newline at end of file diff --git a/processed_dataset/calculation/0573.json b/processed_dataset/calculation/0573.json new file mode 100644 index 0000000000000000000000000000000000000000..5c917fefed9f92eb9a613e2054d6f3597d0b9063 --- /dev/null +++ b/processed_dataset/calculation/0573.json @@ -0,0 +1,8 @@ +{ + "source_file": "./raw_volume-zh/volume2/exercise3.tex", + "problem_type": "calculation", + "problem": "问题3 已知 $f(x)=x^2+2 x+11$ 在 $[t, t+1]$ 上的最小值为 $g(t)$, 求 $g(t)$.", + "solution": "$f(x)$ 的对称轴为 $x=-1$. \n(1) 当 $t+1<-1$ 时, 即 $t<-2$ 时, 有 $f_{\\text {min }}(x)=f(t+1)=t^2+4 t+14$. \n(2) 当 $t+1 \\geqslant-1$, 且 $t \\leqslant-1$ 时, 即 $-2 \\leqslant t \\leqslant-1$ 时, $f_{\\text {min }}(x)=10$. \n(3) 当 $t>-1$ 时, 有 $f_{\\text {min }}(x)=f(t)= t^2+2 t+11$. \n所以 $g(t)= \\begin{cases}t^2+4 t+14, & \\text { 当 } t<-2 \\text { 时; } \\\\ 10, & \\text { 当 }-2 \\leqslant t \\leqslant-1 \\text { 时; } \\\\ t^2+2 t+11, & \\text { 当 } t>-1 \\text { 时.\n}\\end{cases}$", + "remark": "", + "figures": [] +} \ No newline at end of file diff --git a/processed_dataset/calculation/0574.json b/processed_dataset/calculation/0574.json new file mode 100644 index 0000000000000000000000000000000000000000..0215b9ca39f4708f1d9f47d740bc574643f09bec --- /dev/null +++ b/processed_dataset/calculation/0574.json @@ -0,0 +1,8 @@ +{ + "source_file": "./raw_volume-zh/volume2/exercise3.tex", + "problem_type": "calculation", + "problem": "问题4 已知 $a$ 是正整数, 抛物线 $y==a x^2+b x+c$ 过点 $A(-1,4), B(2,1)$, 并且与 $x$ 轴有两个不同的交点, 求:\n(1) $a$ 的最小值;\n(2) $b+c$ 的最大值.", + "solution": "(1) 因为抛物线过点 $A(-1,4), B(2,1)$, 所以 $a-b+c=4 \\cdots(1)$ , $4 a+ 2 b+c=1 \\cdots(2)$ , 且 $\\Delta=b^2-4 a c>0 \\cdots(3)$ . 由(1)、(2)可得 $b=-1-a, c= 3-2 a$. 代入 (3), 得 $(-1-a)^2-4 a(3-2 a)>0,(9 a-1)(a-1)>0$. 由于 $a \\geqslant 1$, 故 $9 a-1>0$, 所以 $a>1$, 从而 $a \\geqslant 2$. 又当 $a=2, b=-3, c=-1$ 时, 满足题意.\n所以 $a$ 的最小值为 2 . \n(2) 由上题知, $b+c=2-3 a \\leqslant 2-3 \\times 2=-4$, 当 $a=2, b=-3, c=-1$ 时等号成立, 所以 $b+c$ 的最大值为 -4 .", + "remark": "", + "figures": [] +} \ No newline at end of file diff --git a/processed_dataset/calculation/0575.json b/processed_dataset/calculation/0575.json new file mode 100644 index 0000000000000000000000000000000000000000..0dee0bbf1ac8705c06d3f888bf62bc46b68bf7bd --- /dev/null +++ b/processed_dataset/calculation/0575.json @@ -0,0 +1,8 @@ +{ + "source_file": "./raw_volume-zh/volume2/exercise3.tex", + "problem_type": "calculation", + "problem": "问题5 设 $f(x)=x^2+a x+3-a$, 若 $f(x)$ 在闭区间 $[-2,2]$ 上恒为非负数, 求实数 $a$ 的取值范围.", + "solution": "$f(x)=\\left(x+\\frac{a}{2}\\right)^2+3-a-\\frac{a^2}{4} . f(x) \\geqslant 0$ 在 $x \\in[-2,2]$ 上恒成立, 等价于 $f(x)$ 在 $[-2,2]$ 上的最小值非负.\n(1) 当 $-\\frac{a}{2}<-2$, 即 $a>4$ 时, $f_{\\min }(x)=f(-2)=7-3 a \\geqslant 0$. 解不等式, 得 $a \\leqslant \\frac{7}{3}$, 与 $a>4$ 矛盾.\n(2) 当 $-2 \\leqslant-\\frac{a}{2} \\leqslant 2$, 即 $-4 \\leqslant a \\leqslant 4$ 时, 有 $f_{\\text {min }}(x)=f\\left(-\\frac{a}{2}\\right)=3-a-\\frac{a^2}{4} \\geqslant 0$, 解不等式得 $-6 \\leqslant a \\leqslant 2$. 故此时可得 $-4 \\leqslant a \\leqslant 2$. \n(3) 当 $-\\frac{a}{2}>2$, 即 $a<$ -4 时, 有 $f_{\\text {min }}(x)=f(2)=7+a \\geqslant 0$, 解不等式得 $a \\geqslant-7$. 故此时 $-7 \\leqslant a<-4$. 综上, $a$ 的范围为 $[-7,2]$.", + "remark": "", + "figures": [] +} \ No newline at end of file diff --git a/processed_dataset/calculation/0576.json b/processed_dataset/calculation/0576.json new file mode 100644 index 0000000000000000000000000000000000000000..645eee37c4176ea6f4a5b275702621866fcd5367 --- /dev/null +++ b/processed_dataset/calculation/0576.json @@ -0,0 +1,10 @@ +{ + "source_file": "./raw_volume-zh/volume2/exercise3.tex", + "problem_type": "calculation", + "problem": "问题6. 设 $a 、 b$ 分别是方程 $\\log _2 x+x-3=0$ 和 $2^x+x-3=0$ 的根, 求 $a+b$ 及 $\\log _2 a+2^b$ 的值.", + "solution": "在直角坐标系内分别作出函数 $y=2^x$ 和 $y=\\log _2 x$ 的图象, 再作直线 $y=x$ 和 $y=-x+3$, 如图()所示.\n由于 $y=2^x$ 和 $y=\\log _2 x$ 互为反函数, 故它们的图象关于直线 $y=x$ 对称.\n方程 $\\log _2 x+x-3=0$ 的根 $a$ 就是直线 $y=-x+3$ 与对数曲线 $y=\\log _2 x$ 的交点 $A$ 的横坐标, 方程 $2^x+x-3=0$ 的根 $b$ 就是直线 $y=-x+3$ 与指数曲线 $y=2^x$ 的交点 $B$ 的横坐标.\n设 $y=-x+3$ 与 $y=x$ 的交点为 $M$, 则点 $M$ 的坐标为 $\\left(\\frac{3}{2}, \\frac{3}{2}\\right)$. 所以 $a+b=2 x_M=3, \\log _2 a+2^b= 2 y_M=3$.", + "remark": "", + "figures": [ + "./images/volume2/figures/fig-c3p6.png" + ] +} \ No newline at end of file diff --git a/processed_dataset/calculation/0577.json b/processed_dataset/calculation/0577.json new file mode 100644 index 0000000000000000000000000000000000000000..3cb4cf9cc66ef728dd9bbc2dc7f72f8f20b12aa5 --- /dev/null +++ b/processed_dataset/calculation/0577.json @@ -0,0 +1,8 @@ +{ + "source_file": "./raw_volume-zh/volume2/exercise3.tex", + "problem_type": "calculation", + "problem": "问题7 (1) 已知函数 $y=f\\left(\\log _2 x\\right)$ 的定义域为 $\\left[\\frac{1}{2}, 2\\right]$, 求函数 $f\\left(\\left(\\frac{1}{2}\\right)^x-2\\right)$ 的定义域;\n(2) 设函数 $f(x)=\\log _{\\frac{1}{2}}\\left(x^2+2 x+2 a\\right)$ 的值域为 $\\mathbf{R}$, 求实数 $a$ 的取值范围.", + "solution": "(1) 由题意知, $\\frac{1}{2} \\leqslant x \\leqslant 2$, 所以, $\\log _2 \\frac{1}{2} \\leqslant \\log _2 x \\leqslant \\log _2 2$, 即 $-1 \\leqslant \\log _2 x \\leqslant 1$. 故 $y=f(x)$ 的定义域为 $[-1,1]$. 解不等式 $-1 \\leqslant\\left(\\frac{1}{2}\\right)^x-2 \\leqslant 1$, 得 $-\\log _2 3 \\leqslant x \\leqslant 0$, 所以, $f\\left(\\left(\\frac{1}{2}\\right)^x-2\\right)$ 的定义域为 $\\left[-\\log _2 3,0\\right]$.\n(2) 只需 $x^2+2 x+2 a$ 能取到一切正实数, 从而 $\\Delta=4-8 a<0$, 故 $a>\\frac{1}{2}$.", + "remark": "", + "figures": [] +} \ No newline at end of file diff --git a/processed_dataset/calculation/0578.json b/processed_dataset/calculation/0578.json new file mode 100644 index 0000000000000000000000000000000000000000..b9fa7fd2e2ff82943ad50467a280433b80759ab2 --- /dev/null +++ b/processed_dataset/calculation/0578.json @@ -0,0 +1,8 @@ +{ + "source_file": "./raw_volume-zh/volume2/exercise3.tex", + "problem_type": "calculation", + "problem": "问题8 函数 $f(x)=a^{2 x}+2 a^x-1(a>0$, 且 $a \\neq 1)$ 在区间 $[-1,1]$ 上的最大值为 14 , 求 $a$ 的值.", + "solution": "令 $t=a^x$, 则 $y=t^2+2 t-1$. 因为 $x \\in[-1,1]$, 所以有如下两种情形: \n(1) 当 $01$ 时, $\\frac{1}{a} \\leqslant t \\leqslant a$, 于是 $y=(t+1)^2-2$ 的对称轴 $t=-1< \\frac{1}{a}$, 故 $y=t^2+2 t-1$ 在 $\\left[\\frac{1}{a}, a\\right]$ 上为单调增函数,所以, 当 $t=a$ 时, $y$ 取最大值 14 , 即 $a^2+2 a-1=14$. \n解方程得 $a=3$ 或 $a=-5$ (舍去), 故 $a=3$. \n综上所述, $a$ 的值为 $\\frac{1}{3}$ 或 3 .", + "remark": "", + "figures": [] +} \ No newline at end of file diff --git a/processed_dataset/calculation/0579.json b/processed_dataset/calculation/0579.json new file mode 100644 index 0000000000000000000000000000000000000000..17e61d51ca8ad3753b5a04a031092c6f1721939b --- /dev/null +++ b/processed_dataset/calculation/0579.json @@ -0,0 +1,10 @@ +{ + "source_file": "./raw_volume-zh/volume2/exercise3.tex", + "problem_type": "calculation", + "problem": "问题9 设 $f(x)=\\min \\left\\{3+\\log _{\\frac{1}{4}} x, \\log _2 x\\right\\}$, 其中 $\\min \\{p, q\\}$ 表示 $p 、 q$ 中的较小者, 求 $f(x)$ 的最大值.", + "solution": "易知 $f(x)$ 的定义域是 $(0,+\\infty)$. \n因为 $y_1= 3+\\log _{\\frac{1}{4}} x$ 在 $(0,+\\infty)$ 上是减函数, $y_2=\\log _2 x$ 在 $(0$, $+\\infty)$ 上是增函数, 而当 $y_1=y_2$, 即 $3+\\log _{\\frac{1}{4}} x=\\log _2 x$ 时, $x=4$, 所以由 $y_1=3+\\log _{\\frac{1}{4}} x$ 和 $y_2=\\log _2 x$ 的图象()可知 $f(x)=\\left\\{\\begin{array}{ll}3+\\log _{\\frac{1}{4}} x, & \\text { 当 } x \\geqslant 4 \\text { 时; } \\\\ \\log _2 x, & \\text { 当 } 00\n$$\n恒成立, 其中 $0 \\leqslant \\theta \\leqslant 2 \\pi$, 求 $\\theta$ 的取值范围.", + "solution": "设 $f(x)=x^2 \\cos \\theta-x(1-x)+(1-x)^2 \\sin \\theta=(1+\\sin \\theta+\\cos \\theta) x^2- (1+2 \\sin \\theta) x+\\sin \\theta$. 因为, $f(0)=\\sin \\theta>0, f(1)=\\cos \\theta>0$, 所以 $\\theta \\in\\left(0, \\frac{\\pi}{2}\\right)$, 由于 $f(x)$ 的对称轴 $x=\\frac{1+2 \\sin \\theta}{2(1+\\sin \\theta+\\cos \\theta)}>0$, 且 $\\frac{1+2 \\sin \\theta}{2(1+\\sin \\theta+\\cos \\theta)}= \\frac{1+2 \\sin \\theta}{1+2 \\sin \\theta+1+2 \\cos \\theta}<1$, 所以 $f(x)$ 在 $[0,1]$ 上的最小值就是 $f(x)$ 在 $\\mathbf{R}$ 上的最小值, 它大于 0. 故 $\\frac{4 \\sin \\theta}{(1+\\sin \\theta+\\cos \\theta)-(1+2 \\sin \\theta)^2}->0$. 即 $4 \\sin \\theta \\cos \\theta-1>0, \\sin 2 \\theta>\\frac{1}{2}$. 所以 $\\frac{\\pi}{6}<2 \\theta<\\frac{5 \\pi}{6}, \\frac{\\pi}{12}<\\theta<\\frac{5 \\pi}{12}$. $\\theta$ 的取值范围是 $\\frac{\\pi}{12}<\\theta<\\frac{5 \\pi}{12}$.", + "remark": "", + "figures": [] +} \ No newline at end of file diff --git a/processed_dataset/calculation/0585.json b/processed_dataset/calculation/0585.json new file mode 100644 index 0000000000000000000000000000000000000000..5b75d97e910ada5a3bfc3ef81c3ccf5bae7bd832 --- /dev/null +++ b/processed_dataset/calculation/0585.json @@ -0,0 +1,8 @@ +{ + "source_file": "./raw_volume-zh/volume2/exercise3.tex", + "problem_type": "calculation", + "problem": "问题18 已知 $f(x)=a x^2+b x+c$ 在 $[0,1]$ 上的函数值的绝对值不超过 1 , 求 $|a|+|b|+|c|$ 的最大值.", + "solution": "由已知得 $\\left\\{\\begin{array}{l}a=2 f(1)+2 f(0)-4 f\\left(\\frac{1}{2}\\right), \\\\ b=4 f\\left(\\frac{1}{2}\\right)-f(1)-3 f(0), \\\\ c=f(0) .\\end{array}\\right.$ 因此 $|a|=|2 f(1)+ 2 f(0)-4 f\\left(\\frac{1}{2}\\right)|\\leqslant 2| f(1)|+2| f(0)|+4| f\\left(\\frac{1}{2}\\right)|\\leqslant 8,\\, | b|=| 4 f\\left(\\frac{1}{2}\\right)- f(1)-3 f(0)|\\leqslant 4| f\\left(\\frac{1}{2}\\right)|+| f(1)|+3| f(0)|\\leqslant 8,\\,|c|=| f(0)| \\leqslant 1$. $|a|+|b|+|c| \\leqslant 8+8+1=17$. 对于二次函数 $f(x)=8 x^2-8 x+1$, 当 $x \\in[0,1]$ 时, $|f(x)| \\leqslant 1$, 且 $|a|+|b|+|c|=17$, 所以 $|a|+|b|+|c|$ 的最大值为 17 .", + "remark": "", + "figures": [] +} \ No newline at end of file diff --git a/processed_dataset/calculation/0586.json b/processed_dataset/calculation/0586.json new file mode 100644 index 0000000000000000000000000000000000000000..fd4e77d7f0870a7e352db9500b7e17663bcb40b2 --- /dev/null +++ b/processed_dataset/calculation/0586.json @@ -0,0 +1,8 @@ +{ + "source_file": "./raw_volume-zh/volume2/exercise3.tex", + "problem_type": "calculation", + "problem": "问题19 设函数 $f(x)=a x^2+8 x+3(a<0)$, 对于给定的负数 $a$, 有一个最大的正数 $l(a)$, 使得在整个区间 $[0, l(a)]$ 上,不等式 $|f(x)| \\leqslant 5$ 都成立.\n问:\n$a$ 为何值时, $l(a)$ 最大? 求出这个最大的 $l(a)$.", + "solution": "$f(x)=a\\left(x+\\frac{4}{a}\\right)^2+3-\\frac{16}{a}$.\n (1) 当 $3-\\frac{16}{a}>5$, 即 $-8\\left(x+\\frac{b}{x}\\right)_{\\text {max }} \\quad\\quad(1)$\n且 $a<\\left(x-\\frac{b}{x}\\right)_{\\text {min }}$ (其中, $x \\in(0,1]$ ). $\\quad\\quad(2)$\n对式(1), $b<0$, 由耐克函数性质知, $g(x)=x+\\frac{b}{x}$ 在 $(0,1]$ 上单调递增, 故\n$$\na>\\left(x+\\frac{b}{x}\\right)_{\\max }=g(1)=1+b . \\quad\\quad(3)\n$$\n对式(2), 当 $-1 \\leqslant b<0$ 时, 由均值不等式 $x-\\frac{b}{x}=x+\\frac{-b}{x} \\geqslant 2 \\sqrt{-b}$, 当且仅当 $x=\\sqrt{-b} \\in(0,1]$ 时等号成立, 所以 $\\left(x-\\frac{b}{x}\\right)_{\\text {min }}=2 \\sqrt{-b}$, 故\n$$\na<2 \\sqrt{-b} . \\quad\\quad(4)\n$$\n由式(3)、(4)要使 $a$ 存在, 必须 $\\left\\{\\begin{array}{l}1+b<2 \\sqrt{-b}, \\\\ -1 \\leqslant b<2 \\sqrt{2}-3,\\end{array}\\right.$ 此时, 存在 $1+b<$ $a<2 \\sqrt{-b}$.\n当 $b<-1$ 时, $f(x)=x-\\frac{b}{x}$ 在 $(0,1]$ 上单调递减(耐克函数性质), 所以\n$$\n\\left(x-\\frac{b}{x}\\right)_{\\min }=f(1)=1-b .\n$$\n综上所述, 当 $-1 \\leqslant b<2 \\sqrt{2}-3$ 时, $a \\in(1+b, 2 \\sqrt{-b})$; 当 $b<-1$ 时, $a \\in(1+b, 1-b)$.\n说明: (1) 本题多次用到耐克函数性质;\n(2) 若 $f(x)$ 在区间 $D$ 上存在最大值或最小值, 则 $f(x) \\geqslant a(\\leqslant a), a$ 是常数, $\\forall x \\in D \\Leftrightarrow f(x)_{\\min } \\geqslant a\\left(f(x)_{\\max } \\leqslant a\\right)$.", + "remark": "", + "figures": [] +} \ No newline at end of file diff --git a/processed_dataset/calculation/0588.json b/processed_dataset/calculation/0588.json new file mode 100644 index 0000000000000000000000000000000000000000..259980d316e02af75773191e6ca6f565050c8cb9 --- /dev/null +++ b/processed_dataset/calculation/0588.json @@ -0,0 +1,8 @@ +{ + "source_file": "./raw_volume-zh/volume2/exercise4.tex", + "problem_type": "calculation", + "problem": "问题1 设 $x 、 y$ 是正整数, 求:\n$$\nf(x, y)=\\frac{x^4}{y^4}+\\frac{y^4}{x^4}-\\frac{x^2}{y^2}-\\frac{y^2}{x^2}+\\frac{x}{y}+\\frac{y}{x}\n$$\n的最小值.", + "solution": "$$\nf(x, y)=\\left(\\frac{x^4}{y^4}-2 \\cdot \\frac{x^2}{y^2}+1\\right)+\\left(\\frac{y^4}{x^4}-2 \\cdot \\frac{y^2}{x^2}+1\\right)+\\left(\\frac{x^2}{y^2}-2+\\frac{y^2}{x^2}\\right)+\\left(\\frac{x}{y}-2+\\frac{y}{x}\\right)+2=\\left(\\frac{x^2}{y^2}-1\\right)^2+\\left(\\frac{y^2}{x^2}-1\\right)^2+\\left(\\frac{x}{y}-\\frac{y}{x}\\right)^2+\\left(\\sqrt{\\frac{x}{y}}-\\sqrt{\\frac{y}{x}}\\right)^2+2 \\geqslant 2\n$$\n并且当且仅当 $x=y$ 时上述等号成立.\n故 $f_{\\text {min }}=2$.", + "remark": "", + "figures": [] +} \ No newline at end of file diff --git a/processed_dataset/calculation/0589.json b/processed_dataset/calculation/0589.json new file mode 100644 index 0000000000000000000000000000000000000000..44d1b78fbe31bf089daff59a97f2cf481019d7e2 --- /dev/null +++ b/processed_dataset/calculation/0589.json @@ -0,0 +1,8 @@ +{ + "source_file": "./raw_volume-zh/volume2/exercise4.tex", + "problem_type": "calculation", + "problem": "问题2 求函数 $y=\\frac{2 x}{x^2+x+1}$ 的最大值和最小值.", + "solution": "去分母, 并整理成关于 $x$ 的二次方程 $y x^2+(y-2) x+y=0$. 因 $x$ 是实数, 当 $y \\neq 0$ 时, 判别式需恒大于或等于 0 , 即 $\\Delta=(y-2)^2-4 y^2 \\geqslant 0$, 解不等式得 $-2 \\leqslant y \\leqslant \\frac{2}{3}$. 而当 $x=-1$ 时, $y=-2$; 当 $x=1$ 时, $y=\\frac{2}{3}$. 故 $y_{\\min }=-2, y_{\\max }=\\frac{2}{3}$.", + "remark": "", + "figures": [] +} \ No newline at end of file diff --git a/processed_dataset/calculation/0590.json b/processed_dataset/calculation/0590.json new file mode 100644 index 0000000000000000000000000000000000000000..2de8ed884cd84b3b1fcea294a12f4d67ace440ce --- /dev/null +++ b/processed_dataset/calculation/0590.json @@ -0,0 +1,8 @@ +{ + "source_file": "./raw_volume-zh/volume2/exercise4.tex", + "problem_type": "calculation", + "problem": "问题3 已知 $a, b, x, y \\in \\mathbf{R}^{+}$, 且 $\\frac{a}{x}+\\frac{b}{y}=1$, 求 $x+y$ 的最小值.", + "solution": "令 $\\left\\{\\begin{array}{l}\\frac{a}{x}=\\sin ^2 \\alpha, \\\\ \\frac{b}{y}=\\cos ^2 \\alpha,\\end{array} \\alpha \\in\\left(0, \\frac{\\pi}{2}\\right)\\right.$, 则 $x+y=\\frac{a}{\\sin ^2 \\alpha}+\\frac{b}{\\cos ^2 \\alpha}=a \\csc ^2 \\alpha+ b \\sec ^2 \\alpha=a+b+a \\cot ^2 \\alpha+b \\tan ^2 \\alpha \\geqslant a+b+2 \\sqrt{a b}$, 所以, $(x+y)_{\\min }=a+b+ 2 \\sqrt{a b}$.", + "remark": "", + "figures": [] +} \ No newline at end of file diff --git a/processed_dataset/calculation/0591.json b/processed_dataset/calculation/0591.json new file mode 100644 index 0000000000000000000000000000000000000000..5460138885c13aa3d819c1e23873f5fcfff349b8 --- /dev/null +++ b/processed_dataset/calculation/0591.json @@ -0,0 +1,8 @@ +{ + "source_file": "./raw_volume-zh/volume2/exercise4.tex", + "problem_type": "calculation", + "problem": "问题4 求函数 $f(x)=\\frac{\\sqrt{x^4+x^2+1}-\\sqrt{x^4+1}}{x}$ 的最大值.", + "solution": "显然当 $x<0$ 时, $f(x)<0$; 当 $x>0$ 时, $f(x)>0$, 因此其最大值应在 $x>0$ 时取得.\n当 $x>0$ 时, 由于 $\\frac{\\sqrt{x^4+x^2+1}-\\sqrt{x^4+1}}{x}= \\sqrt{x^2+\\frac{1}{x^2}+1}-\\sqrt{x^2+\\frac{1}{x^2}}=\\frac{1}{\\sqrt{x^2+\\frac{1}{x^2}+1}+\\sqrt{x^2+\\frac{1}{x^2}}}$, 又因为函数 $y=x+\\frac{1}{x}$ 的值域为 $(-\\infty,-2] \\cup[2,+\\infty)$, 且 $x^2>0(x \\neq 0)$, 所以 $f(x)=\\frac{\\sqrt{x^4+x^2+1}-\\sqrt{x^4+1}}{x} \\leqslant \\frac{1}{\\sqrt{3}+\\sqrt{2}}=\\sqrt{3}-\\sqrt{2}$. 并且可以看出, 当 $x=1$ 时, $f_{\\text {max }}(x)=\\sqrt{3}-\\sqrt{2}$.", + "remark": "", + "figures": [] +} \ No newline at end of file diff --git a/processed_dataset/calculation/0592.json b/processed_dataset/calculation/0592.json new file mode 100644 index 0000000000000000000000000000000000000000..bd31a56266699fbc12803ed3816de7dcb7ab134b --- /dev/null +++ b/processed_dataset/calculation/0592.json @@ -0,0 +1,8 @@ +{ + "source_file": "./raw_volume-zh/volume2/exercise4.tex", + "problem_type": "calculation", + "problem": "问题5 已知实数 $x 、 y$ 满足 $1 \\leqslant x^2+y^2 \\leqslant 4$, 求 $u=x^2+x y+y^2$ 的最大值和最小值.", + "solution": "利用不等式 $-\\frac{x^2+y^2}{2} \\leqslant x y \\leqslant \\frac{x^2+y^2}{2}$, 可得 $u=x^2+y^2+x y \\leqslant x^2+y^2+\\frac{x^2+y^2}{2}=\\frac{3}{2}\\left(x^2+y^2\\right) \\leqslant \\frac{3}{2} \\times 4=6$, 当 $x=y=\\sqrt{2}$ 时等号成立; $u=x^2+y^2+x y \\geqslant x^2+y^2-\\frac{x^2+y^2}{2}=\\frac{1}{2}\\left(x^2+y^2\\right) \\geqslant \\frac{1}{2} \\times 1=\\frac{1}{2}$, 当 $x= \\frac{\\sqrt{2}}{2}, y=-\\frac{\\sqrt{2}}{2}$ 时等号成立.\n所以, $u$ 的最大值为 6 , 最小值为 $\\frac{1}{2}$.", + "remark": "", + "figures": [] +} \ No newline at end of file diff --git a/processed_dataset/calculation/0593.json b/processed_dataset/calculation/0593.json new file mode 100644 index 0000000000000000000000000000000000000000..85d07f4724e3999939ff07a99d5f041d48ae6323 --- /dev/null +++ b/processed_dataset/calculation/0593.json @@ -0,0 +1,8 @@ +{ + "source_file": "./raw_volume-zh/volume2/exercise4.tex", + "problem_type": "calculation", + "problem": "问题6 已知 $x, y, z \\in \\mathbf{R}^{+}$, 且 $x y z(x+y+z)=1$, 求 $(x+y)(y+z)$ 的最小值.", + "solution": "由平均不等式 $(x+y)(y+z)=y(x+y+z)+x z \\geqslant 2 \\sqrt{y(x+y+z) \\cdot x z}=2$, 当 $x=z=1, y=\\sqrt{2}-1$ 时等号成立.\n故最小值为 2 .", + "remark": "", + "figures": [] +} \ No newline at end of file diff --git a/processed_dataset/calculation/0594.json b/processed_dataset/calculation/0594.json new file mode 100644 index 0000000000000000000000000000000000000000..9b9e1315f942f974742245bcfd938fdaee1f4e48 --- /dev/null +++ b/processed_dataset/calculation/0594.json @@ -0,0 +1,8 @@ +{ + "source_file": "./raw_volume-zh/volume2/exercise4.tex", + "problem_type": "calculation", + "problem": "问题7 已知函数 $y={\\frac{2+x}{{\\sqrt{1-x^{2}}}+1}}+{\\frac{1-{\\sqrt{1-x^{2}}}}{x}}\\,,\\,x\\in[-1\\,,\\,0)\\cup(0,\\,1]$, 求此函数的最大值和最小值", + "solution": "由 $|x| \\leqslant 1$ 且 $x \\neq 0$, 可令 $x=\\sin \\theta, \\theta \\in\\left[-\\frac{\\pi}{2}, 0\\right) \\cup\\left(0, \\frac{\\pi}{2}\\right] . y= \\frac{2(1+x)}{1+\\sqrt{1-x^2}}=\\frac{2(1+\\sin \\theta)}{1+\\sqrt{1-\\sin ^2 \\theta}}=\\frac{2(1+\\sin \\theta)}{1+\\cos \\theta}=\\frac{2\\left(\\sin \\frac{\\theta}{2}+\\cos \\frac{\\theta}{2}\\right)^2}{2 \\cos ^2 \\frac{\\theta}{2}}= \\left(\\tan \\frac{\\theta}{2}+1\\right)^2$, 当 $\\tan \\frac{\\theta}{2}=-1$ 时, $\\theta=-\\frac{\\pi}{2}$, 即 $x=-1$ 时, $y_{\\min }=0$; 当 $\\tan \\frac{\\theta}{2}=1$, 即 $x=1$ 时, $y_{\\text {max }}=4$.", + "remark": "", + "figures": [] +} \ No newline at end of file diff --git a/processed_dataset/calculation/0595.json b/processed_dataset/calculation/0595.json new file mode 100644 index 0000000000000000000000000000000000000000..5c2300b7bb2a18a5ad80b9b4b4acb12d42dea7bd --- /dev/null +++ b/processed_dataset/calculation/0595.json @@ -0,0 +1,8 @@ +{ + "source_file": "./raw_volume-zh/volume2/exercise4.tex", + "problem_type": "calculation", + "problem": "问题8. 设 $x_1, x_2, y_1, y_2 \\in \\mathbf{R}$, 求:\n$u=\\sqrt{\\left(1185-x_1-\\overline{\\left.x_2\\right)^2+y_2^2}\\right.}+\\sqrt{x_2^2+y_1^2}+\\sqrt{x_1^2+\\left(1580-y_1-y_2\\right)^2}$ 的最小值.", + "solution": "在直角坐标平面上, 取点 $A(1185,0), X\\left(x_1+x_2, y_2\\right), Y\\left(x_1, y_1+\\right. \\left.y_2\\right), B(0,1580)$. 则 $A X=\\sqrt{\\left(1185-x_1-x_2\\right)^2+y_2^2}, X Y=\\sqrt{x_2^2+y_1^2}$, $Y B=\\sqrt{x_1^2+\\left(1580-y_1-y_2\\right)^2}$. 所以 $u=A X+X Y+Y B \\geqslant A B= \\sqrt{1185^2+1580^2}=1925$. 故 $u$ 的最小值为 1925 .", + "remark": "", + "figures": [] +} \ No newline at end of file diff --git a/processed_dataset/calculation/0596.json b/processed_dataset/calculation/0596.json new file mode 100644 index 0000000000000000000000000000000000000000..141047fa4cf4f13d4c266c5c83cfb447251e518b --- /dev/null +++ b/processed_dataset/calculation/0596.json @@ -0,0 +1,8 @@ +{ + "source_file": "./raw_volume-zh/volume2/exercise4.tex", + "problem_type": "calculation", + "problem": "问题9 一幢 $k(>2)$ 层楼的公寓有一部电梯, 最多能容纳 $k-1$ 个人.\n现有 $k-1$ 个学生同时在第一层楼乘电梯, 他们中没有两人是住同一层楼的.\n电梯只能停一次,停在任意选择的一层.\n而对每一个学生而言, 自己往下走一层感到 1 分不满意, 而往上走一层感到 2 分不满意.\n问电梯停在哪一层, 可使不满意的总分达到最小?", + "solution": "设电梯停在第 $x$ 层, 则不满意的总分为 $S=(1+2+\\cdots+x-2)+ 2(1+2+\\cdots+k-x)=\\frac{1}{2}\\left[3 x^2-(4 k+5) x\\right]+k^2+k+1$. 所以当 $x= N\\left(\\frac{4 k+5}{6}\\right)$ 时, $S$ 最小, 其中 $N(a)$ 表示最接近于 $a$ 的整数.\n例如 $N(3)=3$, $N(3.6)=4, N(2.1)=2, N(2.5)=2或 3$ . 故当电梯停在 $N\\left(\\frac{4 k+5}{6}\\right)$ 层时, 不满意的总分最小.", + "remark": "", + "figures": [] +} \ No newline at end of file diff --git a/processed_dataset/calculation/0597.json b/processed_dataset/calculation/0597.json new file mode 100644 index 0000000000000000000000000000000000000000..9e5a67c00affac2a45082ad38bf107f52d3aba80 --- /dev/null +++ b/processed_dataset/calculation/0597.json @@ -0,0 +1,8 @@ +{ + "source_file": "./raw_volume-zh/volume2/exercise4.tex", + "problem_type": "calculation", + "problem": "问题10 设 $x 、 y 、 z$ 是 3 个不全为零的实数, 求 $\\frac{x y+2 y z}{x^2+y^2+z^2}$ 的最大值.", + "solution": "引人两个正参数 $\\alpha 、 \\beta$, 有 $\\alpha^2 x^2+y^2 \\geqslant 2 \\alpha x y, \\beta^2 y^2+z^2 \\geqslant 2 \\beta y z$. 所以 $x y \\leqslant \\frac{\\alpha}{2} x^2+\\frac{1}{2 \\alpha} y^2, 2 y z \\leqslant \\beta y^2+\\frac{1}{\\beta} z^2$. 因此 $x y+2 y z \\leqslant \\frac{\\alpha}{2} x^2+\\left(\\frac{1}{2 \\alpha}+\\beta\\right) y^2+ \\frac{1}{\\beta} z^2$. 令 $\\frac{\\alpha}{2}=\\frac{1}{2 \\alpha}+\\beta=\\frac{1}{\\beta}$, 解得 $\\alpha=\\sqrt{5}, \\beta=\\frac{2 \\sqrt{5}}{5}$. 故 $x y+2 y z \\leqslant \\frac{\\sqrt{5}}{2}\\left(x^2+\\right. \\left.y^2+z^2\\right)$, 即 $\\frac{x y+2 y z}{x^2+y^2+z^2} \\leqslant \\frac{\\sqrt{5}}{2}$. 解方程组 $\\left\\{\\begin{array}{l}\\frac{\\alpha}{2} x^2=\\frac{1}{2 \\alpha} y^2, \\\\ \\beta y^2=\\frac{1}{\\beta} z^2,\\end{array}\\right.$ 可得一组解 $(x, y, z)=(1, \\sqrt{5}, 2)$, 即当 $x=1, y=\\sqrt{5}, z=2$ 时, $\\frac{x y+2 y z}{x^2+y^2+z^2}$ 取 $\\frac{\\sqrt{5}}{2}$. 从而它的最大值为 $\\frac{\\sqrt{5}}{2}$.", + "remark": "", + "figures": [] +} \ No newline at end of file diff --git a/processed_dataset/calculation/0598.json b/processed_dataset/calculation/0598.json new file mode 100644 index 0000000000000000000000000000000000000000..02d8c6c2bf5348c4e79b194ad37f96161090a044 --- /dev/null +++ b/processed_dataset/calculation/0598.json @@ -0,0 +1,10 @@ +{ + "source_file": "./raw_volume-zh/volume2/exercise4.tex", + "problem_type": "calculation", + "problem": "问题11 已知 $x \\geqslant 1, y \\geqslant 1$, 且 $\\lg ^2 x+\\lg ^2 y=\\lg 10 x^2+\\lg 10 y^2$, 求 $\\lg x y$ 的最大值与最小值.", + "solution": "由题设 $\\lg ^2 x+\\lg ^2 y=\\lg 10 x^2+\\lg 10 y^2$, 得 $(\\lg x-1)^2+(\\lg y-1)^2= 4$. 由于 $\\lg x \\geqslant 0, \\lg y \\geqslant 0$, 所以 $(\\lg x, \\lg y)$ 在以点 $(1,1)$ 为圆心 2 为半径的圆弧上,如图()所示, 易知此圆弧的两个端点 $A(0,1+\\sqrt{3}), B(1+\\sqrt{3}, 0)$. 令 $u=\\lg x y=\\lg x+\\lg y$, 则 $u$ 是直线系: $\\lg y=-\\lg x+u$ 的纵截距.\n因此直线系需与圆弧有公共点, 故当它过 $A 、 B$ 两点时截距最小, 当它与圆弧相切时, 截距最大.\n所以 $u_{\\text {min }}= \\sqrt{3}+1, u_{\\max }=\\sqrt{2}(\\sqrt{2}+2)=2 \\sqrt{2}+2$.", + "remark": "", + "figures": [ + "./images/volume2/figures/fig-c4p11.png" + ] +} \ No newline at end of file diff --git a/processed_dataset/calculation/0599.json b/processed_dataset/calculation/0599.json new file mode 100644 index 0000000000000000000000000000000000000000..14ae14c2af22fd28f770f5bb1f0f705b0bb4818f --- /dev/null +++ b/processed_dataset/calculation/0599.json @@ -0,0 +1,8 @@ +{ + "source_file": "./raw_volume-zh/volume2/exercise4.tex", + "problem_type": "calculation", + "problem": "问题13 当 $s$ 和 $t$ 取遍所有实数时, 求 $M=(s+5-3|\\cos t|)^2+(s-2|\\sin t|)^2$ 的最小值.", + "solution": "设$l:\\left\\{\\begin{array}{l}x=s+5,\\\\y=s,\\end{array}\\right.$, $C:\\left\\{\\begin{array}{l}x=3|cos t|,\\\\y=2|sin t|,\\end{array}\\right.$, 则$M$表示直线$l$上的点与曲线 $C$ 上的点之间距离的平方.\n作出直线 $l: y=x-5$ 及椭圆 $C: \\frac{x^2}{9}+\\frac{y^2}{4}=1 (x \\geqslant 0, y \\geqslant 0)$. 则所求的 $M$ 的最小值为 $A(3,0)$ 到直线 $l$ 的距离的平方.\n即为 2 .", + "remark": "", + "figures": [] +} \ No newline at end of file diff --git a/processed_dataset/calculation/0600.json b/processed_dataset/calculation/0600.json new file mode 100644 index 0000000000000000000000000000000000000000..101e19cc80766b0595bdd85f9e8dbb279d685da9 --- /dev/null +++ b/processed_dataset/calculation/0600.json @@ -0,0 +1,8 @@ +{ + "source_file": "./raw_volume-zh/volume2/exercise4.tex", + "problem_type": "calculation", + "problem": "问题14 已知 $x, y \\in \\mathbf{R}, M=\\max \\{|x-2 y|,|1+x|,|2-2 y|\\}$, 求 $M$ 的最小值.", + "solution": "因为 $M$ 是 $|x-2 y|,|1+x|,|2-2 y|$ 中的最大者, 故 $M$ 不小于这三者的算术平均, 即 $M \\geqslant \\frac{1}{3}(|x-2 y|+|1+x|+|2-2 y|) \\geqslant \\frac{1}{3} \\mid(2 y-x)+ (x+1)+(2-2 y) \\mid=1$, 另一方面, 当 $x=0, y=\\frac{1}{2}$ 时, $|x-2 y|=\\mid 1+ x|=| 2-2 y \\mid=1$, 此时 $M=1$, 故 $M$ 的最小值为 1 .", + "remark": "", + "figures": [] +} \ No newline at end of file diff --git a/processed_dataset/calculation/0601.json b/processed_dataset/calculation/0601.json new file mode 100644 index 0000000000000000000000000000000000000000..78d6d8b3fd2526af077eef1d82a8f0f772fa959a --- /dev/null +++ b/processed_dataset/calculation/0601.json @@ -0,0 +1,8 @@ +{ + "source_file": "./raw_volume-zh/volume2/exercise4.tex", + "problem_type": "calculation", + "problem": "问题15 设 $f(x)=x^2+p x+q, p, q \\in \\mathbf{R}$. 若 $|f(x)|$ 在 $[-1,1]$ 上的最大值为 $M$, 求 $M$ 的最小值.", + "solution": "由于 $M=\\max _{-1 \\leqslant x \\leqslant 1}|f(x)|$, 所以 $M \\geqslant|f(1)|, M \\geqslant|f(0)|, M \\geqslant |f(-1)|$, 故 $4 M \\geqslant|1+p+q|+2|-q|+|1-p+q| \\geqslant \\mid 1+p+q- 2 q+1-p+q \\mid=2$. 所以 $M \\geqslant \\frac{1}{2}$, 而当 $f(x)=x^2-\\frac{1}{2}$ 时, $M=\\frac{1}{2}$. 所以 $M$ 的最小值为 $\\frac{1}{2}$.", + "remark": "", + "figures": [] +} \ No newline at end of file diff --git a/processed_dataset/calculation/0602.json b/processed_dataset/calculation/0602.json new file mode 100644 index 0000000000000000000000000000000000000000..0ee18aeff0bcf0dbd1f1b92234f6863b5c54af49 --- /dev/null +++ b/processed_dataset/calculation/0602.json @@ -0,0 +1,8 @@ +{ + "source_file": "./raw_volume-zh/volume2/exercise5.tex", + "problem_type": "calculation", + "problem": "问题2 已知实数 $x, y$ 满足\n$$\n(3 x+y)^5+x^5+4 x+y=0,\n$$\n求 $4 x+y$ 的值.", + "solution": "原方程可写成 $(3 x+y)^5+(3 x+y)=(-x)^5+(-x)$. 令 $f(t)= t^5+t$, 则 $f(t)$ 在 $\\mathbf{R}$ 上是单调增加的, 于是由 $f(3 x+y)=f(-x)$, 得 $3 x+ y=-x$, 故 $4 x+y=0$.", + "remark": "", + "figures": [] +} \ No newline at end of file diff --git a/processed_dataset/calculation/0603.json b/processed_dataset/calculation/0603.json new file mode 100644 index 0000000000000000000000000000000000000000..03c78d0f14ac686957fb4042ae9f196dca95c0cf --- /dev/null +++ b/processed_dataset/calculation/0603.json @@ -0,0 +1,8 @@ +{ + "source_file": "./raw_volume-zh/volume2/exercise5.tex", + "problem_type": "calculation", + "problem": "问题3 设 $x, y \\in \\mathbf{R}$, 且满足\n$$\n\\left\\{\\begin{array}{l}\n(x-1)^3+2005(x-1)=-1, \\\\\n(y-1)^3+2005(y-1)=1,\n\\end{array}\\right.\n$$\n求 $x+y$ 的值.", + "solution": "原方程组可变形为 $\\left\\{\\begin{array}{l}(x-1)^3+2005(x-1)=-1, \\\\ (1-y)^3+2005(1-y)=-1 .\\end{array}\\right.$ 所以 $(x-1)^3+ 2005(x-1)=(1-y)^3+2005(1-y)$. 令 $f(t)=t^3+2005 t$, 则 $f(t)$ 是 $\\mathbf{R}$ 上的增函数,故由 $f(x-1)=f(1-y)$, 得 $x-1=1-y$, 故 $x+y=2$.", + "remark": "", + "figures": [] +} \ No newline at end of file diff --git a/processed_dataset/calculation/0604.json b/processed_dataset/calculation/0604.json new file mode 100644 index 0000000000000000000000000000000000000000..804c18e1b374089ed2284c1d0e23f43ed0b5a4a7 --- /dev/null +++ b/processed_dataset/calculation/0604.json @@ -0,0 +1,8 @@ +{ + "source_file": "./raw_volume-zh/volume2/exercise5.tex", + "problem_type": "calculation", + "problem": "问题10 若抛物线 $y=x^2+a x+2$ 与连接两点 $M(0,1) 、 N(2,3)$ 的线段 (包括 $M 、 N$ 两点)有两个相异的交点, 求 $a$ 的取值范围.", + "solution": "易知过点 $(0,1),(2,3)$ 的直线方程为 $y=x+1$, 而抛物线 $y= x^2+a x+2$ 与线段 $M N$ 有两个交点就是方程 $x^2+a x+2=x+1$. 在区间 $[0$, $2]$ 上有两个不等实根.\n令 $f(x)=x^2+(a-1) x+1$, 则 $\\left\\{\\begin{array}{l}0<-\\frac{a-1}{2}<2, \\\\ \\Delta=(a-1)^2-4>0, \\text { 解不等式组, 得 } a \\text { 的范围是 }-\\frac{3}{2} \\leqslant a<-1 . \\\\ f(0)=1 \\geqslant 0, \\\\ f(2)=2 a+3 \\geqslant 0 .\\end{array}\\right.$", + "remark": "", + "figures": [] +} \ No newline at end of file diff --git a/processed_dataset/calculation/0605.json b/processed_dataset/calculation/0605.json new file mode 100644 index 0000000000000000000000000000000000000000..6900b3cc97abc586f34fd3e2ef5219b97a29fb16 --- /dev/null +++ b/processed_dataset/calculation/0605.json @@ -0,0 +1,8 @@ +{ + "source_file": "./raw_volume-zh/volume2/exercise6.tex", + "problem_type": "calculation", + "problem": "问题1 求下列函数的 $n$ 次迭代:\n(1) $f(x)=\\frac{x}{\\sqrt{1+x^2}}$;\n(2) $f(x)=\\frac{x+6}{x+2}$;\n(3) $f(x)=x+2 \\sqrt{x}+1$;\n(4) $f(x)=\\sqrt{7 x^2+2}$.", + "solution": "(1) 用数学归纳法, $f^{(n)}(x)=\\frac{x}{\\sqrt{1+n x^2}}$; \n(2) 用递归法, $f^{(n)}(x)= \\frac{\\left(2 \\cdot(-4)^n+3\\right) x+6\\left((-4)^n-1\\right)}{\\left((-4)^n-1\\right) x+\\left(3 \\cdot(-4)^n+2\\right)}$\n(3) 令 $\\varphi(x)=\\arcsin \\sqrt{x}, g(x)= 2 x$, 则 $f^{(n)}(x)=\\left(\\sin \\left(2^n \\arcsin \\sqrt{x}\\right)\\right)^2$;\n(4) 用不动点法, $f^{(n)}(x)= \\sqrt{7^n\\left(x^2+\\frac{1}{3}\\right)-\\frac{1}{3}}$.", + "remark": "", + "figures": [] +} \ No newline at end of file diff --git a/processed_dataset/calculation/0606.json b/processed_dataset/calculation/0606.json new file mode 100644 index 0000000000000000000000000000000000000000..67d90e2b0271787fe4d8e325f583b4d705dff4f6 --- /dev/null +++ b/processed_dataset/calculation/0606.json @@ -0,0 +1,8 @@ +{ + "source_file": "./raw_volume-zh/volume2/exercise6.tex", + "problem_type": "calculation", + "problem": "问题2. 已知 $f^{(3)}(x)=8 x+7$, 求一次函数 $f(x)$.", + "solution": "设 $f(x)=a x+b$, 代入求得 $f(x)=2 x+1$.", + "remark": "", + "figures": [] +} \ No newline at end of file diff --git a/processed_dataset/calculation/0607.json b/processed_dataset/calculation/0607.json new file mode 100644 index 0000000000000000000000000000000000000000..b58bc7078a75e7d7accacc9a2e23b7d379749692 --- /dev/null +++ b/processed_dataset/calculation/0607.json @@ -0,0 +1,8 @@ +{ + "source_file": "./raw_volume-zh/volume2/exercise6.tex", + "problem_type": "calculation", + "problem": "问题3. $n$ 为自然数, $f(n)$ 为 $n^2+1$ (十进制) 的数字之和, 求 $f^{(100)}(1990)$ 的值.", + "solution": "$11$.", + "remark": "", + "figures": [] +} \ No newline at end of file diff --git a/processed_dataset/calculation/0608.json b/processed_dataset/calculation/0608.json new file mode 100644 index 0000000000000000000000000000000000000000..956210fcdfba11678a0121105e40416e155ba1db --- /dev/null +++ b/processed_dataset/calculation/0608.json @@ -0,0 +1,8 @@ +{ + "source_file": "./raw_volume-zh/volume2/exercise6.tex", + "problem_type": "calculation", + "problem": "问题4. 若 $x_1=a>2, x_{n+1}=\\frac{x_n^2}{2\\left(x_n-1\\right)}(n=1,2, \\cdots)$, 求 $\\left\\{x_n\\right\\}$ 的通项.", + "solution": "设 $f(x)=\\frac{x^2}{2(x-1)}=\\frac{1}{1-\\left(1-\\frac{2}{x}\\right)^2}$. 取 $\\varphi(x)=1-\\frac{2}{x}, g(x)=x^2$, 则 $f(x)=\\varphi^{-1}(g(\\varphi(x)))$, 于是 $f^{(n)}(x)=\\frac{2}{1-\\left(1-\\frac{2}{x}\\right)^{2^n}}$. 取 $x_1=a$, 则 $x_2=f\\left(x_1\\right), \\cdots, x_{n+1}=f\\left(x_n\\right)=f^{(n)}\\left(x_1\\right)$, 故 $x_{n+1}=\\frac{2}{1-\\left(1-\\frac{2}{a}\\right)^{2^n}}$.", + "remark": "", + "figures": [] +} \ No newline at end of file diff --git a/processed_dataset/calculation/0609.json b/processed_dataset/calculation/0609.json new file mode 100644 index 0000000000000000000000000000000000000000..2be6cf6ffd86147d4128bb9212b885a3a3b80c06 --- /dev/null +++ b/processed_dataset/calculation/0609.json @@ -0,0 +1,10 @@ +{ + "source_file": "./raw_volume-zh/volume2/exercise6.tex", + "problem_type": "calculation", + "problem": "问题6 设一个长方形长为 1 , 宽为 $\\frac{\\sqrt{5}-1}{2}$, 按如图()所示放人直角坐标中, 然后按逆时针方向不断割去正方形,那么剩下的长方形将收缩至一点,求该点坐标.", + "solution": "$\\left(\\frac{5+\\sqrt{5}}{10}, \\frac{\\sqrt{5}}{5}\\right)$.", + "remark": "", + "figures": [ + "./images/volume2/figures/fig-c6p6.png" + ] +} \ No newline at end of file diff --git a/processed_dataset/calculation/0610.json b/processed_dataset/calculation/0610.json new file mode 100644 index 0000000000000000000000000000000000000000..0f5f8914e3558c6fdcdb30f17acc8d67cccc7e29 --- /dev/null +++ b/processed_dataset/calculation/0610.json @@ -0,0 +1,8 @@ +{ + "source_file": "./raw_volume-zh/volume2/exercise6.tex", + "problem_type": "calculation", + "problem": "问题7 已知 $f(x)=4 x(1-x), 0 \\leqslant x \\leqslant 1$,\n(1) 求 $f^{(n)}(x)$;\n(2) 设 $f^{(n)}(x)$ 取最大和最小的 $x$ 的个数 $a_n 、 b_n$. 试用 $n$ 表示 $a_n$ 和 $b_n$.", + "solution": "(1) 令 $\\varphi(x)=\\arcsin \\sqrt{x}, g(x)=2 x$, 则可求得 $f^{(n)}(x)=$ $\\left(\\sin \\left(2^n \\arcsin \\sqrt{x}\\right)\\right)^2 ; \\quad$ (2) $a_n=2^{n-1}, b_n=2^{n-1}+1$.", + "remark": "", + "figures": [] +} \ No newline at end of file diff --git a/processed_dataset/calculation/0611.json b/processed_dataset/calculation/0611.json new file mode 100644 index 0000000000000000000000000000000000000000..c7f5029062cdfd473343bc3a6420182975ae0702 --- /dev/null +++ b/processed_dataset/calculation/0611.json @@ -0,0 +1,8 @@ +{ + "source_file": "./raw_volume-zh/volume2/exercise6.tex", + "problem_type": "calculation", + "problem": "问题8 设 $D=\\{1,2, \\cdots, 10\\}, f: D \\rightarrow D$, 且 $f$ 是一一映射, 令 $f^{(1)}(x)=f(x)$, $f^{(n+1)}(x)=f\\left(f^{(n)}(x)\\right)$, 试求 $D$ 的某一个排列 $\\left\\{x_i\\right\\}_{i=1}^{10}$, 使 $\\sum_{i=1}^{10} x_i f^{(2520)}(i)=$ 220.", + "solution": "对 $i=1,2, \\cdots, 10$, 由 $\\left\\{i, f_1(i), \\cdots, f_{10}(i)\\right\\} \\subseteq D$, 据抽屈原则, 并注意到 $f$ 是一一映射, 知存在 $1 \\leqslant r_i \\leqslant 10$, 使 $f r_i(i)=i$. 又 $2520=2^3 \\cdot 3^2$ ・ $5 \\cdot 7$ 为 $1,2, \\cdots, 10$ 的最小公倍数, 故上述 $r_i \\mid 2520$, 于是 $f_{2520}(i)=i$ 对一切 $i \\in D$ 成立, 原式为 $\\sum_{i=1}^{10} x_i \\cdot i=220$. 又由排序不等式 $\\sum_{i=1}^{10} x_i \\cdot i \\geqslant 1 \\times 10+\\cdots+$ $10 \\times 1=220$. 从而所求排列为 $10,9,8, \\cdots, 1$.", + "remark": "", + "figures": [] +} \ No newline at end of file diff --git a/processed_dataset/calculation/0612.json b/processed_dataset/calculation/0612.json new file mode 100644 index 0000000000000000000000000000000000000000..c3dc453ce7b8982a787f121b060413689bfa48be --- /dev/null +++ b/processed_dataset/calculation/0612.json @@ -0,0 +1,8 @@ +{ + "source_file": "./raw_volume-zh/volume2/exercise7.tex", + "problem_type": "calculation", + "problem": "问题1 求函数方程:\n$$\n2 f(x)+x^2 f\\left(\\frac{1}{x}\\right)=\\sin x(x>0)\n$$\n在 $(0,+\\infty)$ 上的解.", + "solution": "将函数方程中的 $x$ 用 $\\frac{1}{x}$ 代换, 得 $2 f\\left(\\frac{1}{x}\\right)+\\frac{1}{x^2} f(x)=\\sin \\frac{1}{x}$, 即 $f(x)+ 2 x^2 f\\left(\\frac{1}{x}\\right)=x^2 \\sin \\frac{1}{x}$. 将上面方程与原方程联立消去 $f\\left(\\frac{1}{x}\\right)$, 得 $f(x)= \\frac{2}{3}\\left(\\sin x-\\frac{x^2}{2} \\sin \\frac{1}{x}\\right)$.", + "remark": "", + "figures": [] +} \ No newline at end of file diff --git a/processed_dataset/calculation/0613.json b/processed_dataset/calculation/0613.json new file mode 100644 index 0000000000000000000000000000000000000000..2534b30214654053b0df703df50eef32c431543a --- /dev/null +++ b/processed_dataset/calculation/0613.json @@ -0,0 +1,8 @@ +{ + "source_file": "./raw_volume-zh/volume2/exercise7.tex", + "problem_type": "calculation", + "problem": "问题2 已知 $f(x)$ 是偶函数, $g(x)$ 是奇函数, 并且\n$$\nf(x)+g(x)=1993 x \\sqrt{9-x^2}+x^{1994},\n$$\n求 $f(x)$ 和 $g(x)$.", + "solution": "由题设知, $f(x), g(x)$ 的定义域为 $[-3,3]$. 用 $-x$ 代换方程中的 $x$, 并将原方程与所得的新方程联立得 $\\left\\{\\begin{array}{l}f(x)+g(x)=1993 x \\sqrt{9-x^2}+x^{1994}, \\\\ f(x)-g(x)=-1993 x \\sqrt{9-x^2}+x^{1994} .\\end{array}\\right.$ 解方程组得 $f(x)=x^{1994}, g(x)=1993 x \\sqrt{9-x^2}(-3 \\leqslant x \\leqslant 3)$.", + "remark": "", + "figures": [] +} \ No newline at end of file diff --git a/processed_dataset/calculation/0614.json b/processed_dataset/calculation/0614.json new file mode 100644 index 0000000000000000000000000000000000000000..bc12627a2dc30cdba85f4f17dee14520dce0b57d --- /dev/null +++ b/processed_dataset/calculation/0614.json @@ -0,0 +1,8 @@ +{ + "source_file": "./raw_volume-zh/volume2/exercise7.tex", + "problem_type": "calculation", + "problem": "问题3 求所有满足方程:\n$$\nf(x)+f(y)+g(x)-g(y)=\\sin x+\\cos y, x, y \\in \\mathbf{R}\n$$\n的函数 $f(x)$ 与 $g(x)$.", + "solution": "令 $x=y$ 代入题设方程, 得 $f(x)=\\frac{\\sin x+\\cos x}{2}$. 于是 $\\frac{\\sin x+\\cos x}{2}+ \\frac{\\sin y+\\cos y}{2}+g(x)-g(y)=\\sin x+\\cos y$, 即 $g(x)-\\frac{1}{2} \\sin x+\\frac{1}{2} \\cos x= g(y)-\\frac{1}{2} \\sin y+\\frac{1}{2} \\cos y$. 令 $h(x)=g(x)-\\frac{1}{2} \\sin x+\\frac{1}{2} \\cos x$, 那么对任意的 $x, y \\in \\mathbf{R}$, 都有 $h(x) \\equiv h(y)$. 因此, $h(x)$ 恒等于一个常数.\n所以满足题设方程的函数是 $f(x)=\\frac{\\sin x+\\cos x}{2}, g(x)=\\frac{\\sin x-\\cos x}{2}+C$, 其中 $C$ 为常数.", + "remark": "", + "figures": [] +} \ No newline at end of file diff --git a/processed_dataset/calculation/0615.json b/processed_dataset/calculation/0615.json new file mode 100644 index 0000000000000000000000000000000000000000..dd7e63d799954ba4d8907150c73244811130ac8a --- /dev/null +++ b/processed_dataset/calculation/0615.json @@ -0,0 +1,8 @@ +{ + "source_file": "./raw_volume-zh/volume2/exercise7.tex", + "problem_type": "calculation", + "problem": "问题5 求出满足以下要求的所有 $\\mathbf{R} \\rightarrow \\mathbf{R}$ 的函数 $f 、 g 、 h$, 对任意的 $x, y \\in \\mathbf{R}$, 有\n$$\nf(x)-g(y)=(x-y) h(x+y) .\n$$", + "solution": "记原式为(1)式.\n在(1)中令 $x=y$, 得 $f(x)-g(x)=0$, 所以 $f(x)= g(x)$. 于是(1)式为 $f(x)-f(y)=(x-y) h(x+y) \\cdots$ (2). 在(2)中令 $y=0$, 并记 $f(0)=c$, 得 $f(x)=c+x h(x) \\cdots$ (3). 将 (3) 代入 (2) 中, 得 $x h(x)- y h(y)=(x-y) h(x+y) \\cdots$ (4). 在(4)中令 $y=-x$, 并记 $h(0)=b$, 则 $x(h(x)+ h(-x))=2 b x$. 从而 $h(x)+h(-x)=2 b \\cdots$ (5). 再在(4)式中用 $x+y$ 代换 $x$, $-y$ 代换 $y$, 得 $(x+y) h(x+y)+y h(-y)=(x+2 y) h(x) \\cdots$ (6), 在(6)式中将 $x, y$ 互换, 得 $(x+y) h(x+y)+x h(-x)=(2 x+y) h(y) \\cdots$ (7). (7)一(6), 得 $x h(-x)+x h(x)-y h(-y)-y h(y)=2 x h(y)-2 y h(x) \\cdots$ (8). 由(5)和(8), 便得 $2 b x-2 b y=2 x h(y)-2 y h(x)$. 即 $x h(y)-y h(x)=b(x-y), x(h(y)-b)=y(h(x)-b)$. 对 $x \\neq 0, y \\neq 0$, 有 $\\frac{h(x)-b}{x}=\\frac{h(y)-b}{y}$. 于是对任意的 $x(\\neq 0), \\frac{h(x)-b}{x}$ 为常数, 设为 $a$. 那么 $h(x)=a x+b, x \\neq 0 \\cdots$ (9), 易知(9)式对 $x=0$ 也成立.\n将(9)代入(3), 得 $f(x)=a x^2+b x+c$. 所以 $f(x)=g(x)= a x^2+b x+c, h(x)=a x+b$. 其中的 $a 、 b 、 c$ 为任意常数.\n经检验, $f(x)$, $g(x), h(x)$ 满足题设方程.", + "remark": "", + "figures": [] +} \ No newline at end of file diff --git a/processed_dataset/calculation/0616.json b/processed_dataset/calculation/0616.json new file mode 100644 index 0000000000000000000000000000000000000000..aaf7da176a1e37551600f35e9218f78c8bb65091 --- /dev/null +++ b/processed_dataset/calculation/0616.json @@ -0,0 +1,8 @@ +{ + "source_file": "./raw_volume-zh/volume2/exercise7.tex", + "problem_type": "calculation", + "problem": "问题9 求所有函数 $f, g: \\mathbf{R} \\rightarrow \\mathbf{R}$, 使得对任意 $x, y \\in \\mathbf{R}$,\n$$\nf(x+y g(x))=g(x)+x f(y) .\n$$", + "solution": "(1) $f(x) \\equiv 0, g(x) \\equiv 0$ 为此函数方程的平凡解.\n(2) 下面我们考虑非平凡解.\n令 $x=0$, 则 $f(y g(0))=g(0)$. 若 $g(0) \\neq 0$, 则 $f(x)=g(0)$, 即 $f(x)$ 为常数 $c$; 于是 $g(0)=g(x)+x g(0), g(x)=(1-x) g(0)$. 故 $f(x)=c$, $g(x)=(1-x) c$ 为解.\n(3) 若 $g(0)=0$, 则 $f(0)=0$. 令 $y=0$, 有 $f(x)=g(x)+x f(0)=g(x)$, 任意 $x \\in \\mathbf{R}$. \n故问题转化为求一个函数方程: $f(x+y f(x))=f(x)+x f(y)$ (对任意 $x, y \\in \\mathbf{R}$ ) 的解.\n令 $y=0$, 则 $f(x)=f(x)+x f(0)$, 故 $f(0)=0$; 若 $f(x)=0$, 则 $0=x f(y)$, 因而 $x=0$. 即 $f(x)=0 \\Leftrightarrow x=0 \\cdots \\circledast$. 令 $x=1$, 得 $f(1+y f(1))=f(1)+f(y) \\cdots$ (1). \n若 $f(1) \\neq 1$, 取 $y=\\frac{1}{1-f(1)}$, 代入 (1), 得 $f(1)=0$, 与 $\\circledast$ 矛盾.\n所以 $f(1)=1$, 故 $f(1+y)=1+f(y)$, 任意 $y \\in \\mathbf{R} \\cdots$ (2). \n特别地, $f(n)=n$, $n \\in \\mathbf{Z}$. 取 $x=n \\in \\mathbf{Z}, y=z-1$, 代入方程得 $f(n z)=f(n+f(z-1) f(n))= n+n f(z-1)=n f(z)$, 任意 $n \\in \\mathbf{Z}, z \\in \\mathbf{R}$. \n因此 $f(r z)=r f(z)$, 任意 $r \\in \\mathbf{Q}, z \\in \\mathbf{R} \\cdots$ (3). \n由(3), $f(a)+f(-a)=0=f(a-a)$. 当 $a+b \\neq 0$ 时, $f(a)+ f(b)=f\\left(\\frac{a+b}{2}+\\frac{\\frac{a-b}{2}}{f\\left(\\frac{a+b}{2}\\right)} \\cdot f\\left(\\frac{a+b}{2}\\right)\\right)+f\\left(\\frac{a+b}{2}+\\frac{\\frac{b-a}{2}}{f\\left(\\frac{a+b}{2}\\right)} \\cdot f\\left(\\frac{a+b}{2}\\right)\\right)=\\cdot f\\left(\\frac{a+b}{2}\\right)+\\frac{a+b}{2} \\cdot f\\left(\\frac{\\frac{a-b}{2}}{f\\left(\\frac{a+b}{2}\\right)}\\right)+f\\left(\\frac{a+b}{2}\\right)+\\frac{a+b}{2} f\\left(\\frac{\\frac{b-a}{2}}{f\\left(\\frac{a+b}{2}\\right)}\\right)= 2 f\\left(\\frac{a+b}{2}\\right)=f(a+b)$. \n所以 $f(x+y f(x))=f(x)+f(y f(x)), x+y f(x) \\neq$ 0. \n又 $f(x+y f(x))=f(x)+x f(y)$, 因此 $f(y f(x))=x f(y)$. \n令 $y=1$, 则 $f(f(x))=x$, 故 $f$ 为双射.\n将 $x$ 换成 $f(x)$, 有 $f(x y)=f(x) f(y)$, 任意 $x$, $y \\in \\mathbf{R}$. 令 $y=x$, 则$f\\left(x^2\\right)=f^2(x)>0$. \n令 $y=-x$, 则 $f\\left(-x^2\\right)=f(x) f(-x)$, 故 $f(x)>0 \\Leftrightarrow x>0$. \n又因为 $f(x-f(x))=f(x)-x=-(x- f(x)$ ), 所以 $f(x)-x=0, x \\in \\mathbf{R}$. 从而 $f(x)=g(x)=x$, 任意 $x \\in \\mathbf{R}$.", + "remark": "", + "figures": [] +} \ No newline at end of file diff --git a/processed_dataset/calculation/0617.json b/processed_dataset/calculation/0617.json new file mode 100644 index 0000000000000000000000000000000000000000..1616147a041ff26a026ed3e5a498b806d744bfba --- /dev/null +++ b/processed_dataset/calculation/0617.json @@ -0,0 +1,8 @@ +{ + "source_file": "./raw_volume-zh/volume2/exercise7.tex", + "problem_type": "calculation", + "problem": "问题10 求所有满足下面方程的函数 $f: \\mathbf{R} \\rightarrow \\mathbf{R}$ : 对任意实数 $x 、 y$, 有\n$$\nx f(y)+y f(x)=(x+y) f(x) f(y) .\n$$", + "solution": "取 $x=y=1$ 代入题设方程, 得 $2 f(1)=2(f(1))^2$, 所以 $f(1)=0$ 或 $f(1)=1$. (1) 若 $f(1)=0$. 取 $y=1$ 代入原方程, 得 $x f(1)+f(x)= (x+1) f(x) f(1)$, 所以 $f(x)=0$. (2) 若 $f(1)=1$, 取 $y=1$ 代入原方程, 得 $x+f(x)=(x+1) f(x), x(f(x)-1)=0$. 所以 $f(x)= \\begin{cases}1, \\text { 当 } x \\neq 0 \\text { 时; } \\\\ \\text { 任意实数, 当 } x=0 \\text { 时.\n}\\end{cases}$.", + "remark": "", + "figures": [] +} \ No newline at end of file diff --git a/processed_dataset/calculation/0618.json b/processed_dataset/calculation/0618.json new file mode 100644 index 0000000000000000000000000000000000000000..7d6df97d88d0583ca1645fa21d42743f7903be76 --- /dev/null +++ b/processed_dataset/calculation/0618.json @@ -0,0 +1,8 @@ +{ + "source_file": "./raw_volume-zh/volume2/exercise7.tex", + "problem_type": "calculation", + "problem": "问题11 设函数 $f: \\mathbf{R} \\rightarrow \\mathbf{R}$ 满足\n$$\nf(x y)=\\frac{f(x)+f(y)}{x+y}, \\quad x, y \\in \\mathbf{R}, x+y \\neq 0 .\n$$\n求 $f(x)$.", + "solution": "令 $y=1$, 得 $f(x)=\\frac{f(x)+f(1)}{x+1}, x \\neq-1$. 故 $x f(x)=f(1)$. 当 $x=0$ 时, $f(1)=0$. 从而当 $x \\neq 0, x \\neq-1$ 时, $f(x)=0$. 令 $x=2, y=0$, 得 $f(0)=\\frac{f(2)+f(0)}{2}$, 所以 $f(0)=f(2)=0$. 令 $x=-1, y=0$, 得 $f(0)=\\frac{f(-1)+f(0)}{2}$, 所以 $f(-1)=f(0)=0$. 综上所述, $f(x)=0$.", + "remark": "", + "figures": [] +} \ No newline at end of file diff --git a/processed_dataset/calculation/0619.json b/processed_dataset/calculation/0619.json new file mode 100644 index 0000000000000000000000000000000000000000..132ea4cdb33be9b0b9afc7a84169c565e7a38cdf --- /dev/null +++ b/processed_dataset/calculation/0619.json @@ -0,0 +1,8 @@ +{ + "source_file": "./raw_volume-zh/volume2/exercise7.tex", + "problem_type": "calculation", + "problem": "问题13 设 $f(n)$ 是定义在整数集 $\\mathbf{Z}$ 上的函数, 且满足:\n(1) $f(0)=1, f(1)=0$;\n(2) $f(m+n)+f(m-n)=2 f(m) f(n), m 、 n \\in \\mathbf{Z}$.\n求 $f(n)$.", + "solution": "在条件 (2) 中令 $n=1$, 得 $f(m+1)+f(m-1)=0$. 即 $f(m+1)= -f(m-1) \\cdots$ (1). \n在(1)式中用 $m+2$ 代换 $m$, 得 $f(m+3)=-f(m+1) \\cdots$ (2). \n由(1)、(2)式, 可得 $(m+3)=f(m-1)$. 即 $f(m+4)=f(m)$. \n因此, $f(n)$ 是周期为 4 的周期函数.\n取 $m=1$ 代入(1), 得 $f(2)=-f(0)=-1$. 取 $m=2$ 代入(1), 得 $f(3)=-f(1)=0$. \n所以 $f(n)= \\begin{cases}0, & \\text { 当 } n \\text { 为奇数时; } \\\\ 1, & \\text { 当 } n=4 k, k \\in \\mathbf{Z} \\text { 时; } \\\\ -1, & \\text { 当 } n=4 k+2, k \\in \\mathbf{Z} \\text { 时.\n}\\end{cases}$.", + "remark": "", + "figures": [] +} \ No newline at end of file diff --git a/processed_dataset/calculation/0620.json b/processed_dataset/calculation/0620.json new file mode 100644 index 0000000000000000000000000000000000000000..a07c11292c85a4bc9c2fb69a2aab1fb106b48a37 --- /dev/null +++ b/processed_dataset/calculation/0620.json @@ -0,0 +1,8 @@ +{ + "source_file": "./raw_volume-zh/volume2/exercise7.tex", + "problem_type": "calculation", + "problem": "问题15 设 $f: \\mathbf{R} \\rightarrow \\mathbf{R}$, 对任意 $x, y \\in \\mathbf{R} \\backslash\\{\\ln 2\\}$, 有\n$$\nf(x+y)+f(x-y)=\\mathrm{e}^{x+y}(f(x)+f(y)),\n$$\n若 $f(0)=1$, 求 $f(x)$.", + "solution": "在所给等式中令 $x=0$, 并将 $y$ 用 $x$ 代替, 得 $f(x)+f(-x)= \\mathrm{e}^x(f(x)+1)$. 即 $\\left(1-\\mathrm{e}^x\\right) f(x)+f(-x)=\\mathrm{e}^x$. 注意到当 $x, y$ 互换后所给等式的右边不变, 则有 $f(x-y)=f(y-x)$. 在上式中取 $y=0$, 则 $f(x)=f(-x)$, 故 $f(x)$ 为偶函数.\n因而 $\\left(2-\\mathrm{e}^x\\right) f(x)=\\mathrm{e}^x$. 于是 $f(x)=\\frac{\\mathrm{e}^x}{2-\\mathrm{e}^x}, x \\in \\mathbf{R} \\backslash\\{\\ln 2\\}$.", + "remark": "", + "figures": [] +} \ No newline at end of file diff --git a/processed_dataset/calculation/0621.json b/processed_dataset/calculation/0621.json new file mode 100644 index 0000000000000000000000000000000000000000..590ab27546f6107d1d8b62ac567f013d0a84a985 --- /dev/null +++ b/processed_dataset/calculation/0621.json @@ -0,0 +1,8 @@ +{ + "source_file": "./raw_volume-zh/volume2/exercise7.tex", + "problem_type": "calculation", + "problem": "问题16 $a$ 为已知实数, $00$. 对原恒等式两端取对数, 有 $\\ln f(x+y)=\\ln f(x)+ \\ln f(y)$. \n令 $g(x)=\\ln f(x)$, 则 $g(x)$ 满足柯西方程 $\\circledast$, 因此 $g(x)=a x$, 其中 $a=g(1)=\\ln f(1)$. \n从而 $f(x)=\\mathrm{e}^{a x}=c^x$. 其中 $c=f(1)$. \n综上所述, 原方程的连续解为 $f(x)=c^x, c=f(1)>0$ 或 $f(x) \\equiv 0, x \\in \\mathbf{R}$.", + "remark": "", + "figures": [] +} \ No newline at end of file diff --git a/processed_dataset/calculation/0626.json b/processed_dataset/calculation/0626.json new file mode 100644 index 0000000000000000000000000000000000000000..87d66967fac92e5033b97f5965844e91145e1eda --- /dev/null +++ b/processed_dataset/calculation/0626.json @@ -0,0 +1,8 @@ +{ + "source_file": "./raw_volume-zh/volume2/exercise7.tex", + "problem_type": "calculation", + "problem": "问题22 求函数方程 $f(x y)=f(x) f(y)$, 对任意 $x>0, y>0$ 都成立的连续解.", + "solution": "除去 $f(x) \\equiv 0, x>0$ 这样的平凡解外, 若 $f(x)$ 不恒为 0 , 则必有 $f(x)>0$, 对任意 $x>0$. 令 $u=\\ln x, v=\\ln y$. 原方程转化为 $f\\left(\\mathrm{e}^{u+0}\\right)= f\\left(\\mathrm{e}^u\\right) f\\left(\\mathrm{e}^v\\right)$, 任意 $u, v \\in \\mathbf{R} \\cdots$ (1). 令 $\\varphi(x)=f\\left(\\mathrm{e}^x\\right)$, 则(1)又成为 $\\varphi(u+v)= \\varphi(u) \\varphi(v)$, 任意 $a, u \\in \\mathbf{R} \\cdots$ (2). \n由上题知 (2) 的连续解只有 $\\varphi(x) \\equiv 0$ 或 $\\varphi(x)=[\\varphi(1)]^x$, 因此, 原方程的连续解只能为 $f(x) \\equiv 0$ 或 $f(x)=\\varphi(\\ln x)=(\\varphi(1))^{\\ln x}=x^a$, 其中 $a=\\ln \\varphi(1)=\\ln f(\\mathrm{e})$.", + "remark": "", + "figures": [] +} \ No newline at end of file diff --git a/processed_dataset/calculation/0627.json b/processed_dataset/calculation/0627.json new file mode 100644 index 0000000000000000000000000000000000000000..286f12e0f66cdbef78e8100f15510c8aa70402f2 --- /dev/null +++ b/processed_dataset/calculation/0627.json @@ -0,0 +1,8 @@ +{ + "source_file": "./raw_volume-zh/volume2/exercise7.tex", + "problem_type": "calculation", + "problem": "问题23 设 $f: \\mathbf{R} \\rightarrow \\mathbf{R}$ 是连续函数, 且满足:\n$$\n2 f\\left(\\frac{x+y}{2}\\right)=f(x)+f(y), x, y \\in \\mathbf{R} .\n$$\n求 $f(x)$.", + "solution": "设 $f(0)=b$. 由题给函数方程得 $f\\left(\\frac{x}{2}\\right)=f\\left(\\frac{x+0}{2}\\right)=\\frac{1}{2}[f(x)+ f(0)]=\\frac{1}{2}[f(x)+b]$. 另外 $\\frac{1}{2}[f(x)+f(y)]=f\\left(\\frac{x+y}{2}\\right)=\\frac{1}{2}[f(x+ y)+b]$. 所以 $f(x+y)=f(x)+f(y)-b$. \n令 $g(x)=f(x)-b$, 代入上式得 $g(x+y)=g(x)+g(y)$. 这正是柯西方程, 所以 $g(x)=a x$, 其中 $a= g(1)=f(1)-b$. 所以 $f(x)=a x+b$. 其中 $a=f(1)-f(0), b=f(0)$.", + "remark": "", + "figures": [] +} \ No newline at end of file diff --git a/processed_dataset/calculation/0628.json b/processed_dataset/calculation/0628.json new file mode 100644 index 0000000000000000000000000000000000000000..e63e1573b492cef16274af234987a0ac0b804c57 --- /dev/null +++ b/processed_dataset/calculation/0628.json @@ -0,0 +1,8 @@ +{ + "source_file": "./raw_volume-zh/volume2/exercise7.tex", + "problem_type": "calculation", + "problem": "问题26. 试解函数方程: $f(n+1)=2 f(n)+1$, 且 $f(1)=1, n \\in \\mathbf{N}_{+}$.", + "solution": "因为 $f(1)=1, f(n+1)=2 f(n)+1$. 所以 $f(1)+1=2, f(n+ 1)+1=2[f(n)+1]$. 即 $\\{f(n)+1\\}$ 是首次为 2 , 公比为 2 的等比数列.\n从而有 $f(n)+1=2 \\cdot 2^{n-1}=2^n$. 故 $f(n)=2^n-1$ 即为所求函数.", + "remark": "", + "figures": [] +} \ No newline at end of file diff --git a/processed_dataset/calculation/0629.json b/processed_dataset/calculation/0629.json new file mode 100644 index 0000000000000000000000000000000000000000..b4f3f74a7dd8fffc4e92e1c4dfbeefc5ee14bb81 --- /dev/null +++ b/processed_dataset/calculation/0629.json @@ -0,0 +1,8 @@ +{ + "source_file": "./raw_volume-zh/volume2/exercise7.tex", + "problem_type": "calculation", + "problem": "问题27 定义在正整数集 $\\mathbf{N}_{+}$上的函数 $f$ 满足 $f(1)=1$, 且对任意正整数 $m$ 、 $n$, 有\n$$\nf(m)+f(n)=f(m+n)-m n .\n$$\n求 $f$.", + "solution": "在(1)中令 $n=1$, 得 $f(m)+f(1)=f(m+1)-m$, 即 $f(m+1)- f(m)=m+1$. 于是有 $f(m)-f(m-1)=m, f(m-1)-f(m-2)=m- 1, \\cdots, f(2)-f(1)=2$. 将上面这 $m-1$ 个等式相加, 得 $f(m)-f(1)=2+ 3+\\cdots+m$, 所以 $f(m)=\\frac{m(m+1)}{2}$. 经检验, $f(m)=\\frac{m(m+1)}{2}$ 是原方程的解.", + "remark": "", + "figures": [] +} \ No newline at end of file diff --git a/processed_dataset/calculation/0630.json b/processed_dataset/calculation/0630.json new file mode 100644 index 0000000000000000000000000000000000000000..9edbe08927b17c77037e3bbfa810aae00c3777bd --- /dev/null +++ b/processed_dataset/calculation/0630.json @@ -0,0 +1,8 @@ +{ + "source_file": "./raw_volume-zh/volume2/exercise7.tex", + "problem_type": "calculation", + "problem": "问题28. 求函数方程:\n$$\nf(4 x)=f(2 x)+f(x)(x \\in(-\\infty,+\\infty))\n$$\n的所有解.", + "solution": "在原恒等式中取 $x=0$, 得 $f(0)=0$. 若任取 $x=a \\neq 0$, 设 $f(a)=b_1, f(2 a)=b_2$. \n由原恒等式可得 $\\left\\{\\begin{array}{l}f(4 a)=b_1+b_2, \\\\ f(8 a)=b_1+2 b_2, \\\\ f(18 a)=2 b_1+3 b_2, \\\\ \\ldots \\ldots\\end{array}\\right.$ \n将原恒等式改写为 $f(x)=f(4 x)-f(2 x)$. 于是 $f\\left(\\frac{x}{2}\\right)=f(2 x)-f(x)$. 从而 $\\left\\{\\begin{array}{l}f\\left(\\frac{a}{2}\\right)=b_2-b_1, \\\\ f\\left(\\frac{a}{4}\\right)=-b_2+2 b_1, \\\\ f\\left(\\frac{9}{8}\\right)=2 b_2-3 b_1, \\\\ \\ldots \\ldots\\end{array}\\right.$ 合 $\\left\\{2^n a \\mid n=0, \\pm 1, \\pm 2, \\cdots\\right\\}$ 上的取值就唯一确定了下来.\n因此, 只要任给了 $f(x)$ 在区间 $[1,2)$ 和 $[2,4)$ 上的值, $f$ 在 $(0,+\\infty)$ 上就完全确定了.\n同样, 再给定 $f$ 在 $[-4,-2]$ 和 $(-2,-1]$ 上的值, $f$ 在 $(-\\infty, 0)$ 上也唯一确定了.\n记 $t_n=f\\left(2^n x\\right)$. \n于是, 由函数方程 $f(4 x)=f(2 x)+f(x)$, 知 $t_n=t_{n-1}+t_{n-2}$. 对应的特征方程为 $x^2=x+1$. 其根 $x_{1,2}=\\frac{1 \\pm \\sqrt{5}}{2}$. 故 $t_n=\\alpha_1 x_1^n+\\alpha_2 x_2^n$. 由于 $\\left\\{\\begin{array}{l}\\alpha_1=\\frac{1}{\\sqrt{5}}\\left(f(2 x)-\\frac{1-\\sqrt{5}}{2} f(x)\\right), \\\\ \\alpha_2=-\\frac{1}{\\sqrt{5}}\\left(f(2 x)-\\frac{1+\\sqrt{5}}{2} f(x)\\right) .\\end{array}\\right.$\n因此, 函数方程的解可表达如下: $f(x)=\\left\\{\\begin{aligned} g(x), \\text { 当 } 1 \\leqslant x<4 \\text { 或 }-40 .\\quad\\quad (4)\n\\end{array}\\right.\n$$\n由(1)式, 得 $k \\pi+\\frac{\\pi}{2})\n(2) $y^4 \\geqslant \\sin ^2 x+\\cos ^2 x=1 \\Rightarrow y \\geqslant 1$,\n$(\\sin x+\\cos x)^2 \\leqslant 2\\left(\\sin ^2 x+\\cos ^2 x\\right)=2$, $\\sin x+\\cos x \\leqslant \\sqrt{2}$,\n$(\\sqrt{\\sin x}+\\sqrt{\\cos x})^2 \\leqslant 2(\\sin x+\\cos x)= 2 \\sqrt{2}, \\sqrt{\\sin x}+\\sqrt{\\cos x} \\leqslant 2^{\\frac{3}{4}}$.\n这时我们有 $y \\in\\left[1,2^{\\frac{3}{4}}\\right]$, 而假设 $\\sin ^2 x=a$ ,那么 $y=a^{\\frac{1}{4}}+(1-a)^{\\frac{1}{4}}$ 在 $a \\in[0$ , 1] 上是连续的, 因此 $y$ 可以取到 1 到 $2^{\\frac{3}{4}}$ 之间的任何一个值, 所以所求值域为 $\\left[1,2^{\\frac{3}{4}}\\right]$.\n评注这两道题都是考查的三角函数的基本性质,第二题里面反复运用了均值不等式, 不过难度都不高, 这种题往往需要学生细心谨慎, 不要算错或者写错了, 关于第一问如何看出 $\\frac{\\pi}{2}$ 是周期, 实际上最后可以用图象分析出来, 因为画图后发现是一样的.", + "remark": "", + "figures": [ + "./images/volume3/figures/fig-c1e2.png" + ] +} \ No newline at end of file diff --git a/processed_dataset/calculation/0634.json b/processed_dataset/calculation/0634.json new file mode 100644 index 0000000000000000000000000000000000000000..24de18bde0e01d40fa52219ed9f683433d27e5d7 --- /dev/null +++ b/processed_dataset/calculation/0634.json @@ -0,0 +1,8 @@ +{ + "source_file": "./raw_volume-zh/volume3/chapter1.tex", + "problem_type": "calculation", + "problem": "例3 已知 $f(x)=a x+\\sin x$ 表示的图象上有两条切线相互垂直, 求 $a$ 的值.", + "solution": "解:$f(x)=a x+\\sin x \\Rightarrow f^{\\prime}(x)=a+\\cos x$, 从而如果有两条切线垂直, 那么存在这样的 $x_1, x_2$ 使得 $\\left(a+\\cos x_1\\right)\\left(a+\\cos x_2\\right)=-1$, 从函数图象来看,一个二次函数的两个根都在 $(-1,1)$ 上, 首项系数为 1 , 并且开口朝上, 可以感觉到能取到的最小值只有在尽量的往下移动, 也就是在两个根分别是一 1,1 的时候最小值可以尽可能的小, 此时刚好等于 -1 , 因此可以感觉到这道题实际上卡得很死 (指的中间的放缩), 我们具体的操作如下:\n不妨设 $\\cos x_1 \\leqslant \\cos x_2,\\left(a+\\cos x_1\\right)\\left(a+\\cos x_2\\right)<0$, 从而 $a \\in\\left(-\\cos x_2\\right.$, $\\left.-\\cos x_1\\right)$, 此时可以得到 $0g\\left(\\frac{1}{2}\\right)$, 所以\n$$\nf_{\\min }=g\\left(\\frac{1-2 a}{2}\\right)=a, f_{\\max }=g(-1)=a^2-2 a+\\frac{9}{4} .\n$$\n当 $\\frac{1-2 a}{2}>\\frac{1}{2}$ 即 $a<0$ 时, $\\left[-1, \\frac{1}{2}\\right]$ 为 $g(t)$ 的单调减区间, 所以\n$$\nf_{\\min }=g\\left(\\frac{1}{2}\\right)=a^2+a, f_{\\max }=g(-1)=a^2-2 a+\\frac{9}{4} .\n$$\n评注求形如 $f(x)=A \\sin ^2 x+B \\sin x+C$ 形式的最值, 通常用换元法, 化成二次函数在区间的最值问题.", + "remark": "", + "figures": [] +} \ No newline at end of file diff --git a/processed_dataset/calculation/0636.json b/processed_dataset/calculation/0636.json new file mode 100644 index 0000000000000000000000000000000000000000..6a06ffa319ce8af0b13bfa9425bc085b256995b3 --- /dev/null +++ b/processed_dataset/calculation/0636.json @@ -0,0 +1,8 @@ +{ + "source_file": "./raw_volume-zh/volume3/chapter1.tex", + "problem_type": "calculation", + "problem": "例5 已知函数 $f(x)=\\frac{\\sqrt{2} \\sin x}{\\sqrt{1+\\cos 2 x}}$.\n(1) 求函数 $f(x)$ 的定义域、值域、最小正周期;\n(2) 判断函数 $f(x)$ 的奇偶性.", + "solution": "解:(1) $\\begin{aligned} f(x) & =\\frac{\\sqrt{2} \\sin x}{\\sqrt{1+\\cos 2 x}}=\\frac{\\sin x}{|\\cos x|} \\\\ & =\\left\\{\\begin{array}{l}\\tan x, x \\in\\left(2 k \\pi-\\frac{\\pi}{2}, 2 k \\pi+\\frac{\\pi}{2}\\right) \\\\ -\\tan x, x \\in\\left(2 k \\pi+\\frac{\\pi}{2}, 2 k \\pi+\\frac{3 \\pi}{2}\\right)\\end{array} k \\in \\mathbf{Z},\\right.\\end{aligned}$\n定义域: $\\left\\{x \\mid x \\neq k \\pi+\\frac{\\pi}{2}, k \\in \\mathbf{Z}\\right\\}$, 值域为: $\\mathbf{R}$, 最小正周期为 $T=2 \\pi$.\n(2) $f(-x)=\\frac{\\sin (-x)}{|\\cos (-x)|}=-\\frac{\\sin x}{|\\cos x|}=-f(x)$, 且定义域关于原点对称, 所以 $f(x)$ 为奇函数.\n评注判断函数周期性时, 一要恒等变形, 二要注意定义域的影响.", + "remark": "", + "figures": [] +} \ No newline at end of file diff --git a/processed_dataset/calculation/0637.json b/processed_dataset/calculation/0637.json new file mode 100644 index 0000000000000000000000000000000000000000..b540b6e34632db074700ede5f86b09cad52ef182 --- /dev/null +++ b/processed_dataset/calculation/0637.json @@ -0,0 +1,8 @@ +{ + "source_file": "./raw_volume-zh/volume3/chapter1.tex", + "problem_type": "calculation", + "problem": "例6 已知函数 $f(x)=\\frac{a-2 \\cos x}{3 \\sin x}$ 在区间 $\\left(0, \\frac{\\pi}{2}\\right)$ 内是增函数, 求 $a$ 的取值范围.", + "solution": "分析:根据增函数的定义, 列出不等式, 求 $a$ 的取值范围.\n解法一由条件得: 当 $0\\sin x_1>0$, 所以去分母得\n$$\na \\sin x_2-2 \\cos x_1 \\sin x_2-a \\sin x_1+2 \\cos x_2 \\sin x_1<0,\n$$\n整理得\n$$\na\\left(\\sin x_2-\\sin x_1\\right)-2 \\sin \\left(x_2-x_1\\right)<0,\n$$\n故 $a<\\frac{2 \\sin \\left(x_2-x_1\\right)}{\\sin x_2-\\sin x_1}=\\frac{4 \\sin \\frac{x_2-x_1}{2} \\cos \\frac{x_2-x_1}{2}}{2 \\cos \\frac{x_2+x_1}{2} \\sin \\frac{x_2-x_1}{2}}=\\frac{2 \\cos \\frac{x_2-x_1}{2}}{\\cos \\frac{x_2+x_1}{2}}$.\n由于\n$$\n\\begin{aligned}\n\\cos \\frac{x_2-x_1}{2} & =\\cos \\frac{x_2}{2} \\cos \\frac{x_1}{2}+\\sin \\frac{x_2}{2} \\sin \\frac{x_1}{2} \\\\\n& >\\cos \\frac{x_2}{2} \\cos \\frac{x_1}{2}-\\sin \\frac{x_2}{2} \\sin \\frac{x_1}{2} \\\\\n& =\\cos \\frac{x_1+x_2}{2}>0,\n\\end{aligned}\n$$\n所以 $\\frac{\\cos \\frac{x_2-x_1}{2}}{\\cos \\frac{x_1+x_2}{2}}>1$, 从而 $a \\leqslant 2$, 即 $a$ 的取值范围为 $(-\\infty, 2]$.\n解法二记 $\\tan \\frac{x}{2}=t$, 则 $\\sin x=\\frac{2 t}{1+t^2}, \\cos x=\\frac{1-t^2}{1+t^2}$, 且 $t \\in(0,1)$,\n所以\n$$\n\\begin{aligned}\ng(t) & =f(x)=\\frac{a-2 \\times \\frac{1-t^2}{1+t^2}}{3 \\times \\frac{2 t}{1+t^2}}=\\frac{(a-2)+(a+2) t^2}{6 t} \\\\\n& =\\frac{a-2}{6 t}+\\frac{a+2}{6} \\cdot t .\n\\end{aligned}\n$$\n设 $01$, 故 $a \\leqslant 2$, 即 $a$ 的取值范围为 $(-\\infty, 2]$.\n评注对于含参数不等式的问题, 如 $af(t)$, 则取 $f(t)$ 的最大值后得 $a$ 的取值范围, 如 $f(t)$ 无最值, 则取它的变化趋势的最值.", + "remark": "", + "figures": [] +} \ No newline at end of file diff --git a/processed_dataset/calculation/0638.json b/processed_dataset/calculation/0638.json new file mode 100644 index 0000000000000000000000000000000000000000..14e4e71a5367e9132b13b5a474a1eae874221591 --- /dev/null +++ b/processed_dataset/calculation/0638.json @@ -0,0 +1,8 @@ +{ + "source_file": "./raw_volume-zh/volume3/chapter1.tex", + "problem_type": "calculation", + "problem": "例8 已知函数 $f(x)=\\tan \\left(\\frac{\\pi}{\\sqrt{3}} \\sin x\\right)$.\n(1) 求 $f(x)$ 的定义域和值域;\n(2) 在 $(-\\pi, \\pi)$ 中, 求 $f(x)$ 的单调区间;\n(3) 判定方程 $f(x)=\\tan \\frac{\\sqrt{2}}{3} \\pi$ 在区间 $(-\\pi, \\pi)$ 上解的个数.", + "solution": "解:(1) 因为 $-1 \\leqslant \\sin x \\leqslant 1$, 所以 $-\\frac{\\pi}{\\sqrt{3}} \\leqslant \\frac{\\pi}{\\sqrt{3}} \\sin x \\leqslant \\frac{\\pi}{\\sqrt{3}}$.\n又函数 $y=\\tan x$ 在 $x=k \\pi+\\frac{\\pi}{2}(k \\in \\mathbf{Z})$ 处无定义,且\n$$\n\\left(-\\frac{\\pi}{2}, \\frac{\\pi}{2}\\right) \\varsubsetneqq\\left[-\\frac{\\pi}{\\sqrt{3}}, \\frac{\\pi}{\\sqrt{3}}\\right] \\varsubsetneqq(-\\pi, \\pi),\n$$\n所以令 $\\frac{\\pi}{\\sqrt{3}} \\sin x= \\pm \\frac{\\pi}{2}$, 则 $\\sin x= \\pm \\frac{\\sqrt{3}}{2}$, 解之得: $x=k \\pi \\pm \\frac{\\pi}{3}(k \\in \\mathbf{Z})$.\n所以 $f(x)$ 的定义域是 $A=\\left\\{x \\mid x \\in \\mathbf{R}\\right.$, 且 $\\left.x \\neq k \\pi \\pm \\frac{\\pi}{3}, k \\in \\mathbf{Z}\\right\\}$.\n因为 $\\tan x$ 在 $\\left(-\\frac{\\pi}{2}, \\frac{\\pi}{2}\\right)$ 内的值域为 $(-\\infty,+\\infty)$, 而当 $x \\in A$ 时, 函数 $y=\\tan \\left(\\frac{\\pi}{\\sqrt{3}} \\sin x\\right)$ 的值域 $B$ 满足 $\\left(-\\frac{\\pi}{2}, \\frac{\\pi}{2}\\right) \\varsubsetneqq B$, 所以 $f(x)$ 的值域是 $(-\\infty,+\\infty)$.\n(2) 由 $f(x)$ 的定义域知, $f(x)$ 在 $[0, \\pi]$ 中的 $x=\\frac{\\pi}{3}$ 和 $x=\\frac{2 \\pi}{3}$ 处无定义.\n设 $t=\\frac{\\pi}{\\sqrt{3}} \\sin x$, 则当 $x \\in\\left[0, \\frac{\\pi}{3}\\right) \\cup\\left(\\frac{\\pi}{3}, \\frac{2 \\pi}{3}\\right) \\cup\\left(\\frac{2 \\pi}{3}, \\pi\\right)$ 时, $t \\in \\left[0, \\frac{\\pi}{2}\\right) \\cup\\left(\\frac{\\pi}{2}, \\frac{\\pi}{\\sqrt{3}}\\right]$, 且以 $t$ 为自变量的函数 $y=\\tan t$ 在区间 $\\left(0, \\frac{\\pi}{2}\\right)$, $\\left(\\frac{\\pi}{2}, \\frac{\\pi}{\\sqrt{3}}\\right]$ 上分别单调递增.\n又因为当 $x \\in\\left[0, \\frac{\\pi}{3}\\right]$ 时, 函数 $t=\\frac{\\pi}{\\sqrt{3}} \\sin x$ 单调递增, 且 $t \\in\\left[0, \\frac{\\pi}{2}\\right)$; 当 $x \\in\\left(\\frac{\\pi}{3}, \\frac{\\pi}{2}\\right]$ 时, 函数 $t=\\frac{\\pi}{\\sqrt{3}} \\sin x$ 单调递增, 且 $t \\in\\left(\\frac{\\pi}{2}, \\frac{\\pi}{\\sqrt{3}}\\right]$;\n当 $x \\in\\left[\\frac{\\pi}{2}, \\frac{2 \\pi}{3}\\right)$ 时,函数 $t=\\frac{\\pi}{\\sqrt{3}} \\sin x$ 单调递减, 且 $t \\in\\left(\\frac{\\pi}{2}, \\frac{\\pi}{\\sqrt{3}}\\right]$;\n当 $x \\in\\left(\\frac{2 \\pi}{3}, \\pi\\right)$ 时,函数 $t=\\frac{\\pi}{\\sqrt{3}} \\sin x$ 单调递减, 且 $t \\in\\left(0, \\frac{\\pi}{2}\\right)$.\n所以 $f(x)=\\tan \\left(\\frac{\\pi}{\\sqrt{13}} \\sin x\\right)$ 在区间 $\\left[0, \\frac{\\pi}{3}\\right),\\left(\\frac{\\pi}{3}, \\frac{\\pi}{2}\\right]$ 上分别是单调递增函数; 在 $\\left[\\frac{\\pi}{2}, \\frac{2 \\pi}{3}\\right),\\left(\\frac{2 \\pi}{3}, \\pi\\right)$ 上是单调递减函数.\n又 $f(x)$ 是奇函数, 所以区间 $\\left(-\\frac{\\pi}{3}, 0\\right],\\left[-\\frac{\\pi}{2},-\\frac{\\pi}{3}\\right)$ 也是 $f(x)$ 的单调递增区间, $\\left[-\\pi,-\\frac{2 \\pi}{3}\\right),\\left(-\\frac{2 \\pi}{3},-\\frac{\\pi}{2}\\right]$ 是 $f(x)$ 的单调递减区间.\n故在区间 $(-\\pi, \\pi)$ 中, $f(x)$ 的单调递增区间为: $\\left[-\\frac{\\pi}{2},-\\frac{\\pi}{3}\\right),\\left(-\\frac{\\pi}{3}\\right.$, $\\left.\\frac{\\pi}{3}\\right),\\left(\\frac{\\pi}{3}, \\frac{\\pi}{2}\\right]$, 单调递减区间为: $\\left[-\\pi,-\\frac{2 \\pi}{3}\\right),\\left(-\\frac{2 \\pi}{3}, \\frac{2 \\pi}{3}\\right),\\left(\\frac{2 \\pi}{3}, \\pi\\right)$.\n(3) 由 $f(x)=\\tan \\frac{\\sqrt{2}}{3} \\pi$, 得 $\\tan \\left(\\frac{\\pi}{\\sqrt{3}} \\sin x\\right)=\\tan \\left(\\frac{\\sqrt{2}}{3} \\pi\\right)$\n$$\n\\Leftrightarrow \\frac{\\pi}{\\sqrt{3}} \\sin x=k \\pi+\\frac{\\sqrt{2}}{3} \\pi(k \\in \\mathbf{Z}) \\Leftrightarrow \\sin x=k \\sqrt{3}+\\frac{\\sqrt{6}}{3}(k \\in \\mathbf{Z}) .\n$$\n又因为 $-1 \\leqslant \\sin x \\leqslant 1, \\frac{-\\sqrt{3}-\\sqrt{2}}{3} \\leqslant k \\leqslant \\frac{\\sqrt{3}-\\sqrt{2}}{3}$, 所以 $k=0$ 或 $k=$ -1 .\n当 $k=0$ 时, 从(1)得方程 $\\sin x=\\frac{\\sqrt{6}}{3}$;\n当 $k=1$ 时, 从(1)得方程 $\\sin x=-\\sqrt{3}+\\frac{\\sqrt{6}}{3}$.\n显然方程 $\\sin x=\\frac{\\sqrt{6}}{3}, \\sin x=-\\sqrt{3}+\\frac{\\sqrt{6}}{3}$, 在 $(-\\pi, \\pi)$ 上各有两个解, 故 $f(x)=\\tan \\frac{\\sqrt{2}}{3} \\pi$ 在区间 $(-\\pi, \\pi)$ 上共有 4 个解.\n评注本题是正弦函数与正切函数的复合.\n(1) 求 $f(x)$ 的定义域和值域, 应当先搞清楚 $y=\\frac{\\pi}{\\sqrt{3}} \\sin x$ 的值域与 $y=\\tan x$ 的定义域的交集; (2) 求 $f(x)$ 的单调区间, 必须先搞清 $f(x)$ 的基本性质, 如奇偶性、周期性、复合函数单调性等.", + "remark": "", + "figures": [] +} \ No newline at end of file diff --git a/processed_dataset/calculation/0639.json b/processed_dataset/calculation/0639.json new file mode 100644 index 0000000000000000000000000000000000000000..39f08043e5b2a7d3961e90d2a7bbb0908dd0c2e1 --- /dev/null +++ b/processed_dataset/calculation/0639.json @@ -0,0 +1,8 @@ +{ + "source_file": "./raw_volume-zh/volume3/chapter1.tex", + "problem_type": "calculation", + "problem": "例9 求函数 $y=\\cos x(1+\\cos x) \\tan \\frac{x}{2}$ 的周期.", + "solution": "分析:利用半角的正切公式变形可将函数解析式化为 $y=\\cos x(1+ \\cos x) \\frac{\\sin x}{1+\\cos x}=\\frac{1}{2} \\sin 2 x$, 虽然最后所得函数形式的周期为 $\\pi$, 但由于在运用公式变形过程中 $x$ 的范围有了变化, 原函数中要求 $x \\neq 2 k \\pi$, 即根据等价变形的要求, 最后原函数等价于函数 $y=\\frac{1}{2} \\sin 2 x(x \\neq 2 k \\pi)$, 故原函数的周期为 $2 \\pi$.\n评注由于有些三角公式本身只是一般的恒等式, 而两边的范围并非是等价的, 因此运用这些公式需要附加一定的条件, 或者说可能会出现运用公式的前后范围不相同.", + "remark": "", + "figures": [] +} \ No newline at end of file diff --git a/processed_dataset/calculation/0640.json b/processed_dataset/calculation/0640.json new file mode 100644 index 0000000000000000000000000000000000000000..da4ebcf6ea8e0bd9e113b48afa1ed5811ae00984 --- /dev/null +++ b/processed_dataset/calculation/0640.json @@ -0,0 +1,13 @@ +{ + "source_file": "./raw_volume-zh/volume3/chapter1.tex", + "problem_type": "calculation", + "problem": "例11 若方程 $\\sin ^2 x+\\cos x+a=0$ 有解,求实数 $a$ 的取值范围.", + "solution": "分析:将原方程化归为一元二次方程后, 根据根的分布特征求 $a$ 的取值范围;也可将原方程化归为二次函数后, 根据值域求 $a$ 的取值范围.\n解法一原方程可变形为 $\\cos ^2 x-\\cos x-1-a=0$, 当 $\\Delta=(-1)^2- 4(-1-a)=5+4 a \\geqslant 0$, 即 $a \\geqslant-\\frac{5}{4}$ 时, $\\cos x=\\frac{1 \\pm \\sqrt{5+4 a}}{2}$. 但 $|\\cos x| \\leqslant$ 1 , 所以 $-1 \\leqslant \\frac{1+\\sqrt{5+4 a}}{2} \\leqslant 1$ 或 $-1 \\leqslant \\frac{1-\\sqrt{5+4 a}}{2} \\leqslant 1$, 解这两个不等式,得 $-\\frac{5}{4} \\leqslant a \\leqslant 1$.\n解法二设 $t=\\cos x$, 则 $f(t)=t^2-t-a-1$, 且 $t \\in[-1,1]$, 因原方程有解, 所以 $f(t)$ 的图象与横轴 $t$ 在 $[-1,1]$ 上有交点, 有下列三种情形:\n如图 () () , 满足 $f(1) f(-1) \\leqslant 0$;\n如图 () , 满足\n$$\n\\left\\{\\begin{array}{l}\nf(1) \\geqslant 0, \\\\\nf(-1) \\geqslant 0, \\\\\n\\Delta \\geqslant 0, \\\\\n-1<\\frac{1}{2}<1,\n\\end{array}\\right.\n$$\n即 $(1-a)(-1-a) \\leqslant 0$, 或 $\\left\\{\\begin{array}{l}1-a \\geqslant 0, \\\\ -1-a \\geqslant 0, \\text { 解之得 }-\\frac{5}{4} \\leqslant a \\leqslant 1 \\text {, 所以 } a \\text { 的取 } \\\\ 5+4 a \\geqslant 0 .\\end{array}\\right.$ 值范围为 $\\left[-\\frac{5}{4}, 1\\right]$.\n解法三原方程可变形为 $a=\\cos ^2 x-\\cos x-1=\\left(\\cos x-\\frac{1}{2}\\right)^2-\\frac{5}{4}$, 因为 $|\\cos x| \\leqslant 1$, 所以 $a_{\\max }=1, a_{\\min }=-\\frac{5}{4}$, 故 $a$ 的取值范围为 $\\left[-\\frac{5}{4}, 1\\right]$.\n评注有关 $\\sin x$ 和 $\\cos x$ 的三角方程有解的问题,转化为二次函数的方法最为简洁.\n另外, 在本题中因二次函数的对称轴为 $t==\\frac{1}{2}$, 所以图()实际上是不可能的, 可不考虑.", + "remark": "", + "figures": [ + "./images/volume3/figures/fig-c1e11-1.png", + "./images/volume3/figures/fig-c1e11-2.png", + "./images/volume3/figures/fig-c1e11-3.png", + "./images/volume3/figures/fig-c1e11-1.png" + ] +} \ No newline at end of file diff --git a/processed_dataset/calculation/0641.json b/processed_dataset/calculation/0641.json new file mode 100644 index 0000000000000000000000000000000000000000..6c8287854d6f809f5f57a718af88fe5bd24260b5 --- /dev/null +++ b/processed_dataset/calculation/0641.json @@ -0,0 +1,8 @@ +{ + "source_file": "./raw_volume-zh/volume3/chapter1.tex", + "problem_type": "calculation", + "problem": "例12 求函数 $y=(a+\\cos x)(a+\\sin x)$ 的值域.", + "solution": "分析:对于含参数的函数, 应对 $a$ 进行分类讨论.\n解 $y=a^2+a(\\sin x+\\cos x)+\\sin x \\cos x$.\n设 $\\sin x+\\cos x=t$, 则 $\\sin x \\cos x=\\frac{t^2-1}{2},-\\sqrt{2} \\leqslant t \\leqslant \\sqrt{2}$, 所以\n$$\ny=a^2+a t+\\frac{1}{2}\\left(t^2-1\\right)=\\frac{1}{2}(t+a)^2+\\frac{a^2-1}{2} .\n$$\n(1) 当 $a \\geqslant \\sqrt{2}$ 时, 当 $t=\\sqrt{2}$ 时, $y_{\\text {max }}=a^2+\\sqrt{2} a+\\frac{1}{2}=\\left(a+\\frac{\\sqrt{2}}{2}\\right)^2$;\n当 $t=-\\sqrt{2}$ 时, $y_{\\text {min }}=a^2-\\sqrt{2} a+\\frac{1}{2}=\\left(a-\\frac{\\sqrt{2}}{2}\\right)^2$.\n所以函数的值域为 $\\left[\\left(a-\\frac{\\sqrt{2}}{2}\\right)^2,\\left(a+\\frac{\\sqrt{2}}{2}\\right)^2\\right]$.\n(2) 当 $0 \\leqslant a \\leqslant \\sqrt{2}$ 时, 当 $t=\\sqrt{2}$ 时, $y_{\\text {max }}=\\left(a+\\frac{\\sqrt{2}}{2}\\right)^2$;\n当 $t=-a$ 时, $y_{\\text {min }}=\\frac{a^2-1}{2}$.\n所以函数的值域为 $\\left[\\frac{a^2-1}{2},\\left(a+\\frac{\\sqrt{2}}{2}\\right)^2\\right]$.\n(3) 当 $-\\sqrt{2} \\leqslant a \\leqslant 0$ 时, 当 $t=-\\sqrt{2}$ 时, $y_{\\text {max }}=\\left(a-\\frac{\\sqrt{2}}{2}\\right)^2$;\n当 $t=-a$ 时, $y_{\\text {min }}=\\frac{a^2-1}{2}$.\n所以函数的值域为 $\\left[\\frac{a^2-1}{2},\\left(a-\\frac{\\sqrt{2}}{2}\\right)^2\\right]$.\n(4) 当 $a<-\\sqrt{2}$ 时, 当 $t=-\\sqrt{2}$ 时, $y_{\\text {max }}=\\left(a-\\frac{\\sqrt{2}}{2}\\right)^2$;\n当 $t=\\sqrt{2}$ 时, $y_{\\text {min }}=\\left(a+\\frac{\\sqrt{2}}{2}\\right)^2$.\n所以函数的值域为 $\\left[\\left(a+\\frac{\\sqrt{2}}{2}\\right)^2,\\left(a-\\frac{\\sqrt{2}}{2}\\right)^2\\right]$.\n评注有关含 $\\sin x$ 和 $\\cos x$ 的二次函数值域问题, 必须注意隐含条件 $|\\sin x| \\leqslant 1$ 和 $|\\cos x| \\leqslant 1$.", + "remark": "", + "figures": [] +} \ No newline at end of file diff --git a/processed_dataset/calculation/0642.json b/processed_dataset/calculation/0642.json new file mode 100644 index 0000000000000000000000000000000000000000..822656f18c09002bb7006f37bb49309f28598e11 --- /dev/null +++ b/processed_dataset/calculation/0642.json @@ -0,0 +1,8 @@ +{ + "source_file": "./raw_volume-zh/volume3/chapter1.tex", + "problem_type": "calculation", + "problem": "例16 求实数 $a$ 的取值范围,使得对任意实数 $x$ 和任意 $\\theta \\in\\left[0, \\frac{\\pi}{2}\\right]$, 恒有\n$$\n(x+3+2 \\sin \\theta \\cos \\theta)^2+(x+a \\sin \\theta+a \\cos \\theta)^2 \\geqslant \\frac{1}{8} .\n$$", + "solution": "解:令 $\\sin \\theta+\\cos \\theta=u$, 则 $2 \\sin \\theta \\cos \\theta=u^2-1$, 当 $\\theta \\in\\left[0, \\frac{\\pi}{2}\\right]$ 时, $u \\in$ 016 [1, $[1]$.\n并记 $f(x)=(x+3+2 \\sin \\theta \\cos \\theta)^2+(x+a \\sin \\theta+a \\cos \\theta)^2$.\n所以 $f(x)=\\left(x+2+u^2\\right)^2+(x+a u)^2=2 x^2+2\\left(u^2+a u+2\\right) x+ \\left(u^2+2\\right)^2+(a u)^2=2\\left[x+\\frac{1}{2}\\left(u^2+a u+2\\right)\\right]^2+\\frac{1}{2}\\left(u^2-a u+2\\right)^2$.\n所以 $x=-\\frac{1}{2}\\left(u^2+a u+2\\right)$ 时, $f(x)$ 取得最小值 $\\frac{1}{2}\\left(u^2-a u+2\\right)^2$. 所以 $u^2-a u+2 \\geqslant \\frac{1}{2}$ 或 $u^2-a u+2 \\leqslant-\\frac{1}{2}$.\n所以 $a \\leqslant u+\\frac{3}{2 u}$ 或 $a \\geqslant u+\\frac{5}{2 u}$. 当 $u \\in[1, \\sqrt{2}]$ 时, $u+\\frac{3}{2 u} \\in\\left[\\sqrt{6}, \\frac{5}{2}\\right]$, $u+\\frac{5}{2 u} \\in\\left[\\frac{9}{4} \\sqrt{2}, \\frac{7}{2}\\right]$.\n所以 $a \\leqslant \\sqrt{6}$ 或 $a \\geqslant \\frac{7}{2}$.\n评注在三角函数式中,如果同时出现 $\\sin x \\pm \\cos x$ 及 $\\sin x \\cos x$ 的式子, 常用换元法, 通常将三角函数转变成二次函数问题来求解.", + "remark": "", + "figures": [] +} \ No newline at end of file diff --git a/processed_dataset/calculation/0643.json b/processed_dataset/calculation/0643.json new file mode 100644 index 0000000000000000000000000000000000000000..1bf7f491aa8f8983644bc62a904adfe4c8b2f41b --- /dev/null +++ b/processed_dataset/calculation/0643.json @@ -0,0 +1,8 @@ +{ + "source_file": "./raw_volume-zh/volume3/chapter1.tex", + "problem_type": "calculation", + "problem": "例17 设 $x \\geqslant y \\geqslant z \\geqslant \\frac{\\pi}{12}$, 且 $x+y+z=\\frac{\\pi}{2}$.\n求乘积 $\\cos x \\cdot \\sin y \\cdot \\cos z$ 的最大值及最小值.", + "solution": "解:由于 $x \\geqslant y \\geqslant z \\geqslant \\frac{\\pi}{12}$, 故\n$$\n\\frac{\\pi}{6} \\leqslant x \\leqslant \\frac{\\pi}{2}-\\frac{\\pi}{12} \\times 2=\\frac{\\pi}{3} .\n$$\n所以 $\\cos x \\sin y \\cos z=\\cos x \\times \\frac{1}{2}[\\sin (y+z)+\\sin (y-z)]=\\frac{1}{2} \\cos ^2 x+ \\frac{1}{2} \\cos x \\sin (y-z) \\geqslant \\frac{1}{2} \\cos ^2 \\frac{\\pi}{3}=\\frac{1}{8}$. 即为最小值 (由于 $\\frac{\\pi}{6} \\leqslant x \\leqslant \\frac{\\pi}{3}, y \\geqslant z$, 故 $\\cos x \\sin (y-z) \\geqslant 0)$, 当 $y=z=\\frac{\\pi}{12}, x=\\frac{\\pi}{3}$ 时, $\\cos x \\sin y \\cos z=\\frac{1}{8}$.\n因为\n$$\n\\begin{aligned}\n\\cos x \\sin y \\cos z & =\\cos z \\times \\frac{1}{2}[\\sin (x+y)-\\sin (x-y)] \\\\\n& =\\frac{1}{2} \\cos ^2 z-\\frac{1}{2} \\cos z \\sin (x-y),\n\\end{aligned}\n$$\n由于 $\\sin (x-y) \\geqslant 0, \\cos z>0$, 故\n$$\n\\cos x \\sin y \\cos z \\leqslant \\frac{1}{2} \\cos ^2 z==\\frac{1}{2} \\cos ^2 \\frac{\\pi}{12}=\\frac{1}{4}\\left(1+\\cos \\frac{\\pi}{6}\\right)=\\frac{2+\\sqrt{3}}{8} .\n$$\n当 $x=y=\\frac{5 \\pi}{12}, z=\\frac{\\pi}{12}$ 时取得最大值.\n所以最大值 $\\frac{2+\\sqrt{3}}{8}$, 最小值 $\\frac{1}{8}$.\n评注本题是 1997 年全国高中数学联赛试题.\n巧妙应用积化和差公式及放缩法是解本题的关键.", + "remark": "", + "figures": [] +} \ No newline at end of file diff --git a/processed_dataset/calculation/0644.json b/processed_dataset/calculation/0644.json new file mode 100644 index 0000000000000000000000000000000000000000..85a6e3d87b8f39ec85000a2e8f69945eb6c0f5f6 --- /dev/null +++ b/processed_dataset/calculation/0644.json @@ -0,0 +1,8 @@ +{ + "source_file": "./raw_volume-zh/volume3/chapter2.tex", + "problem_type": "calculation", + "problem": "例2 (1) 若 $\\mathrm{e}^{\\mathrm{i} \\theta}=\\cos \\theta+\\mathrm{i} \\sin \\theta$, 求 $\\left|2+2 \\mathrm{e}^{\\frac{2}{5} \\pi \\mathrm{i}}+\\mathrm{e}^{\\frac{6}{5} \\pi \\mathrm{i}}\\right|$\n(2) 已知 $\\sin 2(\\alpha+\\gamma)=n \\sin 2 \\beta$, 则 $\\frac{\\tan (\\alpha+\\beta+\\gamma)}{\\tan (\\alpha-\\beta+\\gamma)}$ 的值是\n(3) 求 $\\left(1+\\tan 1^{\\circ}\\right)\\left(1+\\tan 2^{\\circ}\\right) \\cdots\\left(1+\\tan 44^{\\circ}\\right)(1+ \\tan 45^{\\circ} )$.", + "solution": "解:(1) $2+2 \\mathrm{e}^{\\frac{2}{5} \\pi \\mathrm{i}}+\\mathrm{e}^{\\frac{6}{5} \\pi \\mathrm{i}}=2+2 \\cos \\frac{2}{5} \\pi+\\cos \\frac{6}{5} \\pi+\\mathrm{i}\\left(2 \\sin \\frac{2}{5} \\pi+\\sin \\frac{6}{5} \\pi\\right)$,\n$$\n\\begin{aligned}\n& \\left|2+2 \\mathrm{e}^{\\frac{2}{5} \\pi i}+\\mathrm{e}^{\\frac{6}{5} \\pi \\mathrm{i}}\\right| \\\\\n= & \\left(2+2 \\cos \\frac{2}{5} \\pi+\\cos \\frac{6}{5} \\pi\\right)^2+\\left(2 \\sin \\frac{2}{5} \\pi+\\sin \\frac{6}{5} \\pi\\right)^2 \\\\\n= & 4+4\\left(2 \\cos \\frac{2}{5} \\pi+\\cos \\frac{6}{5} \\pi\\right)+\\left(2 \\cos \\frac{2}{5} \\pi+\\cos \\frac{6}{5} \\pi\\right)^2+ \\left(2 \\sin \\frac{2}{5} \\pi+\\sin \\frac{6}{5} \\pi\\right)^2 \\\\\n= & 4+4\\left(2 \\cos \\frac{2}{5} \\pi+\\cos \\frac{6}{5} \\pi\\right)+4+1+4\\left(\\cos \\frac{2}{5} \\pi \\cos \\frac{6}{5} \\pi+\\sin \\frac{2}{5} \\pi \\sin \\frac{6}{5} \\pi\\right)\n= & 9+4\\left(2 \\cos \\frac{2}{5} \\pi+\\cos \\frac{6}{5} \\pi+\\cos \\left(\\frac{6}{5} \\pi-\\frac{2}{5} \\pi\\right)\\right) \\\\\n= & 9+8\\left(\\cos \\frac{2}{5} \\pi+\\cos \\frac{4}{5} \\pi\\right) .\n\\end{aligned}\n$$\n我们要求 $\\cos \\frac{2}{5} \\pi, \\cos \\frac{4}{5} \\pi$ 的大小,这点可以画图求,用传统的相似三角形做几何图进行求解, 但是我们只用三角函数也是可以做的:\n$$\n\\text { 设 } 2\\left(2 \\cos ^2 \\frac{1}{5} \\pi-1\\right)^2-1=2 \\cos ^2-\\frac{2}{5} \\pi-1=\\cos \\frac{4}{5} \\pi=-\\cos \\frac{1}{5} \\pi ,\n$$\n可以求得\n$$\n\\left(\\cos \\frac{1}{5} \\pi+1\\right)\\left(\\cos \\frac{1}{5} \\pi-\\frac{1}{2}\\right)\\left(\\cos \\frac{1}{5} \\pi-\\frac{1+\\sqrt{5}}{2}\\right)\\left(\\cos \\frac{1}{5} \\pi-\\frac{1-\\sqrt{5}}{2}\\right)=0,\n$$\n很容易分析得出 $\\cos \\frac{1}{5} \\pi=\\frac{1+\\sqrt{5}}{2}$.\n(2) 如果觉得看起来有点复杂,可以先分析里面的情况,我们发现 $\\alpha, \\gamma$ 绑在一起, 所以直接假设 $\\alpha+\\gamma=\\varphi$, 从而条件转化为 $\\sin 2 \\varphi=n \\sin 2 \\beta$, 所以 $\\frac{\\tan (\\alpha+\\beta+\\gamma)}{\\tan (\\alpha-\\beta+\\gamma)}=\\frac{\\tan (\\varphi+\\beta)}{\\tan (\\varphi-\\beta)}=\\frac{\\sin (\\varphi+\\beta) \\cos (\\varphi-\\beta)}{\\cos (\\varphi+\\beta) \\sin (\\varphi-\\beta)}$ 想到利用积化和差公式,因此\n$$\n\\begin{aligned}\n\\frac{\\tan (\\alpha+\\beta+\\gamma)}{\\tan (\\alpha-\\beta+\\gamma)} & =\\frac{\\sin (\\varphi+\\beta) \\cos (\\varphi-\\beta)}{\\cos (\\varphi+\\beta) \\sin (\\varphi-\\beta)} \\\\\n& =\\frac{\\sin ((\\varphi+\\beta)+(\\varphi-\\beta))+\\sin ((\\varphi+\\beta)-(\\varphi-\\beta))}{\\sin ((\\varphi+\\beta)+(\\varphi-\\beta))-\\sin ((\\varphi+\\beta)-(\\varphi-\\beta))} \\\\\n& =\\frac{\\sin 2 \\varphi+\\sin 2 \\beta}{\\sin 2 \\varphi-\\sin 2 \\beta}=\\frac{n+1}{n-1} .\n\\end{aligned}\n$$\n实际上本题最好还强调一下在 $n=1$ 并且在 $\\sin (\\varphi-\\beta)=0$, 即 $\\alpha-\\beta+ \\gamma=k \\pi, k=0,1,2 \\cdots$ 的时候表达式并无意义.\n(3) 一般这种情况可以考虑配对相乘:\n$$\n(1+\\tan \\alpha)\\left(1+\\tan \\left(\\frac{\\pi}{4}-\\alpha\\right)\\right)=(1+\\tan \\alpha)\\left(1+\\frac{\\tan \\frac{\\pi}{4}-\\tan \\alpha}{1+\\tan \\frac{\\pi}{4} \\tan \\alpha}\\right)=2\n$$\n因此, 原式 $=\\left[\\left(1+\\tan 1^{\\circ}\\right)\\left(1+\\tan 44^{\\circ}\\right)\\right]\\left[\\left(1+\\tan 2^{\\circ}\\right)\\left(1+\\tan 43^{\\circ}\\right)\\right] \\cdots \\left[\\left(1+\\tan 22^{\\circ}\\right)\\left(1+\\tan 23^{\\circ}\\right)\\right]\\left(1+\\tan 45^{\\circ}\\right)=2^{23}$.\n评注这些题目依旧考查三角函数的变形能力, 不过比第一问要稍难一点, 并且第二问第三问涉及到了技巧运算, 第二问一开始可以不讲最后无意义的情况, 如果学生自动提出, 可以适当鼓励, 第三问如果学生觉得配对相乘有点难以接受的话,可以考虑采用如下方法,直接考查单项进行变形:\n$$\n\\begin{aligned}\n1+\\tan \\alpha & =1+\\frac{\\sin \\alpha}{\\cos \\alpha}=\\frac{\\sin \\alpha+\\cos \\alpha}{\\cos \\alpha}=\\sqrt{2} \\frac{\\sin \\left(\\alpha+\\frac{\\pi}{4}\\right)}{\\cos \\alpha} \\\\\n& =\\sqrt{2} \\frac{\\sin \\left(\\alpha+\\frac{\\pi}{4}\\right)}{\\sin \\left(\\frac{\\pi}{2}-\\alpha\\right)}\n\\end{aligned}\n$$\n那么原式 $=(\\sqrt{2})^{44} \\cdot \\frac{\\sin 46^{\\circ}}{\\sin 89^{\\circ}} \\cdot \\frac{\\sin 47^{\\circ}}{\\sin 88^{\\circ}} \\cdots \\cdot \\frac{\\sin 89^{\\circ}}{\\sin 46^{\\circ}} \\cdot 2=2^{23}$.\n这种做法强调一种变形后分子分母尽量形式调整为一样的想法, 因为肯定能消掉,所以形式相同更容易看出来如何消掉.", + "remark": "", + "figures": [] +} \ No newline at end of file diff --git a/processed_dataset/calculation/0645.json b/processed_dataset/calculation/0645.json new file mode 100644 index 0000000000000000000000000000000000000000..6346e2e79683a81ea48f46add7bd552cb1b73524 --- /dev/null +++ b/processed_dataset/calculation/0645.json @@ -0,0 +1,8 @@ +{ + "source_file": "./raw_volume-zh/volume3/chapter2.tex", + "problem_type": "calculation", + "problem": "例9 计算下列各式的值:\n(1) 已知 $\\alpha 、 \\beta$ 为锐角, $\\cos \\alpha=\\frac{1}{7}, \\sin (\\alpha+\\beta)=\\frac{5 \\sqrt{3}}{14}$, 求 $\\cos \\beta$;\n(2) 若 $\\tan \\alpha 、 \\tan \\beta$ 是方程 $x^2+b x+c=0(b \\neq 0)$ 的两根, 求 $\\sin ^2(\\alpha+ \\beta)+b \\sin (\\alpha+\\beta) \\cos (\\alpha+\\beta)+c \\cos ^2(\\alpha+\\beta)$ 的值.", + "solution": "分析:充分挖掘隐含条件, 正确估算角的范围, 计算第 (1) 题.\n恰当应用公式,分类讨论求第 $(2)$ 题.\n解 (1) 因为 $\\alpha 、 \\beta$ 均为锐角, $\\sin (\\alpha+\\beta)=\\frac{5 \\sqrt{3}}{14}<\\frac{\\sqrt{3}}{2}$. 所以 $0<\\alpha+\\beta< \\frac{\\pi}{3}$ 或 $\\frac{2 \\pi}{3}<\\alpha+\\beta<\\pi$, 而 $\\cos \\alpha=\\frac{1}{7}<\\frac{1}{2}$, 所以 $\\frac{\\pi}{3}<\\alpha<\\frac{\\pi}{2}$, 从而 $\\alpha+\\beta$ 为钝角, $\\cos (\\alpha+\\beta)=-\\sqrt{1-\\sin ^2(\\alpha+\\beta)}=-\\sqrt{1-\\left(\\frac{5 \\sqrt{3}}{14}\\right)^2}=-\\frac{11}{14}, \\sin \\alpha=\\frac{4 \\sqrt{3}}{7}$.\n故 $\\cos \\beta=\\cos (\\alpha+\\beta-\\alpha)=\\cos (\\alpha+\\beta) \\cos \\alpha+\\sin (\\alpha+\\beta) \\sin \\alpha$\n$$\n=-\\frac{11}{14} \\times \\frac{1}{7}+\\frac{5 \\sqrt{3}}{14} \\times \\frac{4 \\sqrt{3}}{7}=\\frac{1}{2} .\n$$\n(2) 由已知条件,得 $\\tan \\alpha+\\tan \\beta=-b, \\tan \\alpha \\cdot \\tan \\beta=c$. 所以当 $c \\neq 1$ 时,\n$$\n\\begin{gathered}\n\\tan (\\alpha+\\beta)=\\frac{\\tan \\alpha+\\tan \\beta}{1-\\tan \\alpha \\cdot \\tan \\beta}=\\frac{b}{c-1} . \\\\\n\\sin ^2(\\alpha+\\beta)+b \\sin (\\alpha+\\beta) \\cos (\\alpha+\\beta)+c \\cos ^2(\\alpha+\\beta) \\\\\n=\\frac{\\sin ^2(\\alpha+\\beta)+b \\sin (\\alpha+\\beta) \\cos (\\alpha+\\beta)+c \\cos ^2(\\alpha+\\beta)}{\\sin ^2(\\alpha+\\beta)+\\cos ^2(\\alpha+\\beta)} \\\\\n=\\frac{\\tan ^2(\\alpha+\\beta)+b \\tan (\\alpha+\\beta)+c}{\\tan ^2(\\alpha+\\beta)+1}=\\frac{\\left(\\frac{b}{c-1}\\right)^2+\\frac{b^2}{c-1}+c}{1+\\left(\\frac{b}{c-1}\\right)^2}=c .\n\\end{gathered}\n$$\n当 $c=1$ 时, $\\tan \\alpha \\cdot \\tan \\beta=1$, 此时, $\\alpha+\\beta=k \\pi+\\frac{\\pi}{2}(k \\in \\mathbf{Z}), \\cos (\\alpha+ \\beta)=0, \\sin (\\alpha+\\beta)= \\pm 1$, 原式 $=1$.\n评注本题第 (1) 题如不能正确估计, 会出现 $\\cos (\\alpha+\\beta)= \\pm \\frac{11}{14}$, 从而 $\\cos \\beta=\\frac{1}{2}$ 或 $\\frac{71}{98}$, 其原因是未能就 $\\sin (\\alpha+\\beta)=\\frac{5 \\sqrt{3}}{14}$ 深人挖掘 $\\alpha+\\beta$ 的取值范围;\n第(2) 题容易忽略 $\\tan (\\alpha+\\beta)=\\frac{\\tan \\alpha+\\tan \\beta}{1-\\tan (\\alpha+\\beta)}$ 中 $\\tan \\alpha \\cdot \\tan \\beta \\neq 1$ 这一隐含的约束条件, 即 $\\tan (\\alpha+\\beta)$ 不存在的情形.", + "remark": "", + "figures": [] +} \ No newline at end of file diff --git a/processed_dataset/calculation/0646.json b/processed_dataset/calculation/0646.json new file mode 100644 index 0000000000000000000000000000000000000000..ea8f89734323d38cb7c70a2e95d3d764a9634ec8 --- /dev/null +++ b/processed_dataset/calculation/0646.json @@ -0,0 +1,8 @@ +{ + "source_file": "./raw_volume-zh/volume3/chapter2.tex", + "problem_type": "calculation", + "problem": "例10 已知 $\\sin \\alpha+\\sin \\beta=\\frac{3}{5}, \\cos \\alpha+\\cos \\beta=\\frac{4}{5}$, 试求 $\\cos (\\alpha-\\beta)$ 和 $\\sin (\\alpha+\\beta)$ 的值.", + "solution": "分析:注意所求式子中角与已知条件中角的关系, 将已知条件平方相加可出现结论中的角 $\\alpha-\\beta$, 将已知条件和差化积可出现结论中的角 $\\frac{\\alpha+\\beta}{2}$, 再用万能公式求 $\\sin (\\alpha+\\beta)$.\n解由\n$$\n\\begin{aligned}\n& \\sin \\alpha+\\sin \\beta=\\frac{3}{5}, \\quad\\quad(1) \\\\\n& \\cos \\alpha+\\cos \\beta=\\frac{4}{5}, \\quad\\quad(2) \n\\end{aligned}\n$$\n$(1)^2+(2)^2$ 得\n$$\n(\\sin \\alpha+\\sin \\beta)^2+(\\cos \\alpha+\\cos \\beta)^2=\\left(\\frac{3}{5}\\right)^2+\\left(\\frac{4}{5}\\right)^2,\n$$\n即\n$$\n2+2 \\sin \\alpha \\sin \\beta+2 \\cos \\alpha \\cos \\beta=1,\n$$\n所以\n$$\n\\cos (\\alpha-\\beta)=-\\frac{1}{2} .\n$$\n由(1)和差化积得\n$$\n2 \\sin \\frac{\\alpha+\\beta}{2} \\cos \\frac{\\alpha-\\beta}{2}=\\frac{3}{5} \\quad\\quad(3) \n$$\n由(2)和差化积得\n$$\n2 \\cos \\frac{\\alpha+\\beta}{2} \\cos \\frac{\\alpha-\\beta}{2}=\\frac{4}{5} . \\quad\\quad(4) \n$$\n于是由 $\\frac{(3)}{(4)}$ 得 $\\tan \\frac{(\\alpha+\\beta)}{2}=\\frac{3}{4}$. 故\n$$\n\\sin (\\alpha+\\beta)=\\frac{2 \\tan \\frac{\\alpha+\\beta}{2}}{1+\\tan ^2 \\frac{\\alpha+\\beta}{2}}=\\frac{2 \\times \\frac{3}{4}}{1+\\left(\\frac{3}{4}\\right)^2}=\\frac{24}{25} .\n$$\n评注形如 $\\sin \\alpha+\\sin \\beta=a, \\cos \\alpha+\\cos \\beta=b$ 的三角函数求值题, 主要是采用题中两种方法.\n当然, 还可作如下变换: 由 $(1 )^2+(2)^2$ 得 $\\cos (\\alpha-\\beta)= -\\frac{1}{2}$, 由 $(2)^2-(1)^2$ 得 $\\cos 2 \\alpha+\\cos 2 \\beta+2 \\cos (\\alpha+\\beta)=\\frac{7}{25}$.\n即 $2 \\cos (\\alpha+\\beta) \\cos (\\alpha-\\beta)+2 \\cos (\\alpha+\\beta)=\\frac{7}{25}$, 从而 $\\cos (\\alpha+\\beta)=\\frac{7}{25}$.\n由$(1) \\times(2)$得 $\\sin (\\alpha+\\beta)+\\frac{1}{2} \\sin 2 \\alpha+\\frac{1}{2} \\sin 2 \\beta=\\frac{12}{25}$, 从而 $\\sin (\\alpha+\\beta)+ \\sin (\\alpha+\\beta) \\cos (\\alpha-\\beta)=\\frac{12}{25}$, 可得 $\\sin (\\alpha+\\beta)=\\frac{24}{25}$.", + "remark": "", + "figures": [] +} \ No newline at end of file diff --git a/processed_dataset/calculation/0647.json b/processed_dataset/calculation/0647.json new file mode 100644 index 0000000000000000000000000000000000000000..94ec75edc5acc09eb361871004811cae5b7a8efa --- /dev/null +++ b/processed_dataset/calculation/0647.json @@ -0,0 +1,8 @@ +{ + "source_file": "./raw_volume-zh/volume3/chapter2.tex", + "problem_type": "calculation", + "problem": "例14 (1) 设 $n$ 是一个大于 3 的素数, 求 $\\left(1+2 \\cos \\frac{2 \\pi}{n}\\right)\\left(1+2 \\cos \\frac{4 \\pi}{n}\\right)\\left(1+ 2 \\cos \\frac{6 \\pi}{n}\\right) \\cdots\\left(1+2 \\cos \\frac{2 n \\pi}{n}\\right)$ 的值;\n(2) 设 $n$ 是一个大于 3 的自然数, 求 $\\left(1+2 \\cos \\frac{\\pi}{n}\\right)\\left(1+2 \\cos \\frac{2 \\pi}{n}\\right)\\left(1+ 2 \\cos \\frac{3 \\pi}{n}\\right) \\cdots\\left(1+2 \\cos \\frac{(n-1) \\pi}{n}\\right)$ 的值.", + "solution": "分析:由题中 $\\cos \\frac{2 k \\pi}{n}, \\cos \\frac{k \\pi}{n}$, 可联系到 $n$ 次单位根和 $2 n$ 次单位根.\n解 (1) 记 $w=\\mathrm{e}^{\\frac{2 \\pi i}{n}}$, 则 $w^n=1, w^{-\\frac{n}{2}}=\\mathrm{e}^{-\\pi i}=-1,2 \\cos \\frac{2 k \\pi}{n}=w^k+w^{-k}$.\n$$\n\\begin{aligned}\n\\prod_{k=1}^n\\left(1+2 \\cos \\frac{2 k \\pi}{n}\\right) & =\\prod_{k=1}^n\\left(1+w^k+w^{-k}\\right) \\\\\n& =\\prod_{k=1}^n w^{-k}\\left(w^k+w^{2 k}+1\\right) \\\\\n& =w^{-\\frac{n(n+1)}{2}} \\cdot 3 \\prod_{k=1}^{n-1} \\frac{1-w^{3 k}}{1-w^k} \\\\\n& =(-1)^{n+1} \\cdot 3 \\prod_{k=1}^{n-1} \\frac{1}{1-w^{3 k}} .\n\\end{aligned}\n$$\n因为 $n$ 为大于 3 的索数, 所以 $(-1)^{n+1}=1$, 且当 $k=1,2, \\cdots, n-1$ 时, $3 k$ 取遍模 $n$ 的剩余类, 从而\n$$\n\\prod_{k=1}^{n-1}\\left(1-w^{3 k}\\right)=\\prod_{k=1}^{n-1}\\left(1-w^k\\right),\n$$\n于是\n$$\n\\prod_{k=1}^n\\left(1+2 \\cos \\frac{2 k \\pi}{n}\\right)=3 .\n$$\n(2) $Z^{2 n}=1$ 的 $2 n$ 个根是 $\\pm 1$ 和 $Z_k=\\mathrm{e}^{\\frac{k \\pi \\pi j}{n}}(k=1,2, \\cdots, n-1)$, 所以\n$$\n\\begin{aligned}\nZ^{2 n}-1 & =\\left(Z^2-1\\right) \\prod_{k=1}^{n-1}\\left(Z-\\mathrm{e}^{\\frac{k \\pi i}{n}}\\right)\\left(Z-\\mathrm{e}^{-\\frac{k \\pi i}{n}}\\right) \\\\\n& =\\left(Z^2-1\\right) \\prod_{k=1}^{n-1}\\left(Z^2+1-2 Z \\cos \\frac{k \\pi}{n}\\right) .\n\\end{aligned}\n$$\n取 $Z=\\mathrm{e}^{\\frac{2 \\pi i}{3}}$, 则 $Z^2+1=-Z$, 于是有\n$$\n\\begin{aligned}\n& Z^{2 n}-1=\\left(Z^2-1\\right)(-Z)^{n-1} \\prod_{k=1}^{n-1}\\left(1+2 \\cos \\frac{k \\pi}{n}\\right) . \\\\\n= & \\prod_{k=1}^{n-1}\\left(1+2 \\cos \\frac{k \\pi}{n}\\right) \\\\\n= & \\frac{Z^{2 n}-1}{\\left(Z^2-1\\right)(-Z)^{n-1}} \\\\\n= & \\left\\{\\begin{array}{l}\n0, n=3 k \\\\\n\\frac{Z^2-1}{\\left(Z^2-1\\right)(-Z)^{3 k}}=(-1)^{3 k}=(-1)^{n-1}, n=3 k+1, \\\\\n\\frac{Z-1}{\\left(Z^2-1\\right)(-Z)^{3 k+1}}=\\frac{(-1)^{3 k+1}}{(Z+1) Z}=\\frac{(-1)^{3 k+1}}{-1}=(-1)^n, n=3 k+2,\n\\end{array}\\right.\n\\end{aligned}\n$$\n其中 $k$ 为自然数.", + "remark": "", + "figures": [] +} \ No newline at end of file diff --git a/processed_dataset/calculation/0648.json b/processed_dataset/calculation/0648.json new file mode 100644 index 0000000000000000000000000000000000000000..978422a8ba1a8ef5b86e4b22e28f0803e87a7078 --- /dev/null +++ b/processed_dataset/calculation/0648.json @@ -0,0 +1,8 @@ +{ + "source_file": "./raw_volume-zh/volume3/chapter2.tex", + "problem_type": "calculation", + "problem": "例15 求值: $\\cos \\frac{\\pi}{13}+\\cos \\frac{3 \\pi}{13}+\\cos \\frac{9 \\pi}{13}$.", + "solution": "解:设\n$$\n\\begin{aligned}\n& x=\\cos \\frac{\\pi}{13}+\\cos \\frac{3 \\pi}{13}+\\cos \\frac{9 \\pi}{13}, \\\\\n& y=\\cos \\frac{5 \\pi}{13}+\\cos \\frac{7 \\pi}{13}+\\cos \\frac{11 \\pi}{13},\n\\end{aligned}\n$$\n则\n$$\n\\begin{aligned}\nx+y & =\\cos \\frac{\\pi}{13}+\\cos \\frac{3 \\pi}{13}+\\cos \\frac{5 \\pi}{13}+\\cos \\frac{7 \\pi}{13}+\\cos \\frac{9 \\pi}{13}+\\cos \\frac{11 \\pi}{13} \\\\\n& =\\frac{1}{2 \\sin \\frac{\\pi}{13}}\\left[2 \\sin \\frac{\\pi}{13}\\left(\\cos \\frac{\\pi}{13}+\\cos \\frac{3 \\pi}{13}+\\cdots+\\cos \\frac{11 \\pi}{13}\\right)\\right] \\\\\n& =\\frac{1}{2 \\sin \\frac{\\pi}{13}}\\left[\\sin \\frac{2 \\pi}{13}+\\left(\\sin \\frac{4 \\pi}{13}-\\sin \\frac{2 \\pi}{13}\\right)+\\cdots+\\left(\\sin \\frac{12 \\pi}{13}-\\sin \\frac{10 \\pi}{13}\\right)\\right] \\\\\n& =\\frac{\\sin \\frac{12 \\pi}{13}}{2 \\sin \\frac{ \\pi}{13}}=\\frac{1}{2}, \\\\\nx \\cdot y & =\\left(\\cos \\frac{\\pi}{13}+\\cos \\frac{3 \\pi}{13}+\\cos \\frac{9 \\pi}{13}\\right)\\left(\\cos \\frac{5 \\pi}{13}+\\cos \\frac{7 \\pi}{13}+\\cos \\frac{11 \\pi}{13}\\right) \\\\\n&= -\\frac{3}{2}\\left(\\cos \\frac{\\pi}{13}-\\cos \\frac{2 \\pi}{13}+\\cos \\frac{3 \\pi}{13}-\\cos \\frac{4 \\pi}{13}+\\cos \\frac{5 \\pi}{13}-\\cos \\frac{6 \\pi}{13}\\right) \\\\\n& =-\\frac{3}{2}\\left(\\cos \\frac{\\pi}{13}+\\cos \\frac{3 \\pi}{13}+\\cos \\frac{5 \\pi}{13}+\\cos \\frac{7 \\pi}{13}+\\cos \\frac{9 \\pi}{13}+\\cos \\frac{11 \\pi}{13}\\right) \\\\\n& =-\\frac{3}{4} .\n\\end{aligned}\n$$\n故 $x 、 y$ 是方程 $t^2-\\frac{1}{2} t-\\frac{3}{4}=0$ 的两根, $t_{1,2}=\\frac{1 \\pm \\sqrt{13}}{4}$.\n又由于 $x>0$, 故 $x=\\frac{1+\\sqrt{13}}{4}$.\n即\n$$\n\\cos \\frac{\\pi}{13}+\\cos \\frac{3 \\pi}{13}+\\cos \\frac{5 \\pi}{13}=\\frac{1+\\sqrt{13}}{4} .\n$$\n评注此例中配对的式子与上例不同, 当然也可采用自配对方式.\n$$\n\\begin{aligned}\nx^2= & \\left(\\cos \\frac{\\pi}{13}+\\cos \\frac{3 \\pi}{13}+\\cos \\frac{9 \\pi}{13}\\right)^2 \\\\\n= & \\cos ^2 \\frac{\\pi}{13}+\\cos ^2 \\frac{3 \\pi}{13}+\\cos ^2 \\frac{9 \\pi}{13}+2 \\cos \\frac{\\pi}{13} \\cos \\frac{3 \\pi}{13} +2 \\cos \\frac{3 \\pi}{13} \\cos \\frac{9 \\pi}{13}+2 \\cos \\frac{9 \\pi}{13} \\cos \\frac{\\pi}{13} \\\\\n= & \\frac{1}{2}\\left(1+\\cos \\frac{2 \\pi}{13}\\right)+\\frac{1}{2}\\left(1+\\cos \\frac{6 \\pi}{13}\\right)+\\frac{1}{2}\\left(1+\\cos \\frac{18 \\pi}{13}\\right) +\\cos \\frac{4 \\pi}{13}+\\cos \\frac{2 \\pi}{13}+\\cos \\frac{12 \\pi}{13}+\\cos \\frac{6 \\pi}{13}+\\cos \\frac{10 \\pi}{13}+\\cos \\frac{8 \\pi}{13} \\\\\n= & \\frac{3}{2}-\\frac{1}{2}\\left(\\cos \\frac{11 \\pi}{13}+\\cos \\frac{7 \\pi}{13}+\\cos \\frac{5 \\pi}{13}\\right) -\\left(\\cos \\frac{11 \\pi}{13}+\\cos \\frac{9 \\pi}{13}+\\cos \\frac{7 \\pi}{13}+\\cos \\frac{\\pi}{13}+\\cos \\frac{5 \\pi}{13}+\\cos \\frac{3 \\pi}{13}\\right),\n\\end{aligned}\n$$\n又因为\n$$\n\\cos \\frac{11 \\pi}{13}+\\cos \\frac{7 \\pi}{13}+\\cos \\frac{5 \\pi}{13}=\\frac{1}{2}-\\left(\\cos \\frac{9 \\pi}{13}+\\cos \\frac{3 \\pi}{13}+\\cos \\frac{\\pi}{13}\\right),\n$$\n从而\n$$\nx^2=\\frac{3}{2}-\\frac{1}{2}\\left(\\frac{1}{2}-x\\right)-\\frac{1}{2}, x=\\frac{1 \\pm \\sqrt{13}}{4} .\n$$\n又因为 $x>0$, 所以 $x=\\frac{1+\\sqrt{13}}{4}$.", + "remark": "", + "figures": [] +} \ No newline at end of file diff --git a/processed_dataset/calculation/0649.json b/processed_dataset/calculation/0649.json new file mode 100644 index 0000000000000000000000000000000000000000..056ca0b12f2b2725a49898543a2299c0d3517134 --- /dev/null +++ b/processed_dataset/calculation/0649.json @@ -0,0 +1,11 @@ +{ + "source_file": "./raw_volume-zh/volume3/chapter3.tex", + "problem_type": "calculation", + "problem": "例1 如图(), 走廊宽为 $3 \\mathrm{~m}$, 夹角为 $120^{\\circ}$, 地面是水平的, 走廊两端足够长, 问: 保持水平位置通过走廊的木棒 (不计粗细) 的最大长度是多少?", + "solution": "分析:本题的关键是把棒的长度表示为某一个变量的函数, 然后求最值.\n解如图(), 过走廊转角内顶点 $P$ 任作水平直线与走廊外侧交于点 $A 、 B$. 则在水平位置通过走廊的木棒长度小于或等于 $A B$.\n设 $\\angle B A Q=\\alpha$, 则 $\\angle A B Q=60^{\\circ}-\\alpha$,\n$$\nA B=A P+P B=\\frac{3}{\\sin \\alpha}+\\frac{3}{\\sin \\left(60^{\\circ}-\\alpha\\right)} .\n$$\n当 $\\alpha$ 变化时, 上式的最小值即是在水平位置通过走廊的木棒的最大长度.\n由平均不等式及积化和差得:\n$$\n\\begin{aligned}\nA B & \\geqslant 6 \\sqrt{\\frac{1}{\\sin \\alpha \\cdot \\sin \\left(60^{\\circ}-\\alpha\\right)}}=6 \\sqrt{\\frac{1}{\\frac{1}{2}\\left[\\cos \\left(60^{\\circ}-2 \\alpha\\right)-\\cos 60^{\\circ}\\right]}} \\\\\n& \\geqslant 6 \\sqrt{\\frac{2}{1-\\frac{1}{2}}}=12 .\n\\end{aligned}\n$$\n当且仅当 $\\alpha=30^{\\circ}$ 时, $A B=12$.\n故在水平位置能通过走廊的木棒的最大长度为 $12 \\mathrm{~m}$.\n评注本题是求最小值中的最大值.", + "remark": "", + "figures": [ + "./images/volume3/figures/fig-c3e1-1.png", + "./images/volume3/figures/fig-c3e1-2.png" + ] +} \ No newline at end of file diff --git a/processed_dataset/calculation/0650.json b/processed_dataset/calculation/0650.json new file mode 100644 index 0000000000000000000000000000000000000000..fb3ba676f1a90b97dafb96b5145689f3243f1038 --- /dev/null +++ b/processed_dataset/calculation/0650.json @@ -0,0 +1,10 @@ +{ + "source_file": "./raw_volume-zh/volume3/chapter3.tex", + "problem_type": "calculation", + "problem": "例6 在 $\\triangle A B C$ 中, $A B=A C, \\angle C A B 、 \\angle A B C$ 的内角平分线分别与边 $B C 、 C A$ 交于点 $D 、 E$, 设 $K$ 是 $\\triangle A D C$ 的内心, 若 $\\angle B E K=45^{\\circ}$, 求 $\\angle C A B$ 所有可能的值.", + "solution": "分析:用三角法求解平面几何问题,可以先建立三角方程,再求解。\n解如图 (), 设 $A D$ 与 $B E$ 交于点 $I, \\triangle A B C$ 内切圆 $\\odot I$ 的半径为 $r$.\n$\\angle A B C=2 \\alpha\\left(0<\\alpha<45^{\\circ}\\right)$, 由 $A B=A C$, 知 $A D \\perp B C, \\angle A C B=2 \\alpha$, 故 $I D=r$.\n$$\nI E=\\frac{r}{\\sin \\angle B E C}=\\frac{r}{\\sin 3 \\alpha}, \\quad\\quad (1)\n$$\n连结 $D K$, 由 $K$ 是 $\\triangle A C D$ 的内心, 知 $I 、 K 、 C$ 三点共线, 且 $\\angle I C D= \\angle I C E=\\alpha$,\n$$\n\\angle C D K=\\angle I D K=45^{\\circ}=\\angle B E K,\n$$\n从而 $\\triangle I D K 、 \\triangle I E K$ 的外接圆半径相等, 即\n$$\n\\frac{I D}{\\sin \\angle I K D}=\\frac{I E}{\\sin \\angle I K E} . \\quad\\quad (2)\n$$\n又\n$$\n\\angle I K D=\\angle I C D+\\angle C D K=\\alpha+45^{\\circ},\n$$\n$$\n\\angle I K E=\\angle B I C-\\angle B E K=2\\left(90^{\\circ}-\\alpha\\right)-45^{\\circ}=135^{\\circ}-2 \\alpha .\n$$\n由(1)、(2)得, $\\sin \\left(\\alpha+45^{\\circ}\\right)=\\sin 3 \\alpha \\cdot \\sin \\left(135^{\\circ}-2 \\alpha\\right)$\n$$\n\\begin{aligned}\n& \\Leftrightarrow \\sin \\alpha+\\cos \\alpha=\\sin 3 \\alpha(\\sin 2 \\alpha+\\cos 2 \\alpha) \\\\\n& \\Leftrightarrow 2 \\sin \\alpha+2 \\cos \\alpha=\\cos \\alpha-\\cos 5 \\alpha+\\sin \\alpha+\\sin 5 \\alpha \\\\\n& \\Leftrightarrow \\sin \\alpha+\\cos \\alpha=\\sin 5 \\alpha-\\cos 5 \\alpha \\\\\n& \\Leftrightarrow \\sin \\left(\\alpha+45^{\\circ}\\right)=\\sin \\left(5 \\alpha-45^{\\circ}\\right) \\\\\n& \\Leftrightarrow 2 \\sin \\left(2 \\alpha-45^{\\circ}\\right) \\cos 3 \\alpha=0 \\\\\n& \\Leftrightarrow 3 \\alpha=90^{\\circ} \\text { 或 } 2 \\alpha=45^{\\circ} \\\\\n& \\Leftrightarrow \\angle B A C=60^{\\circ} \\text { 或 } 90^{\\circ} .\n\\end{aligned}\n$$\n评注本题用三角法求解的关键是: 选择变量, 建立方程.", + "remark": "", + "figures": [ + "./images/volume3/figures/fig-c3e6.png" + ] +} \ No newline at end of file diff --git a/processed_dataset/calculation/0651.json b/processed_dataset/calculation/0651.json new file mode 100644 index 0000000000000000000000000000000000000000..ca8f048af3f11a6bf9a1827a2a19e57bb5fe6f59 --- /dev/null +++ b/processed_dataset/calculation/0651.json @@ -0,0 +1,8 @@ +{ + "source_file": "./raw_volume-zh/volume3/chapter3.tex", + "problem_type": "calculation", + "problem": "例9 在 $\\triangle A B C$ 中, 若角 $A 、 B 、 C$ 成等差数列, 外接圆直径为 1 , 若三个角所对的边分别为 $a 、 b 、 c$, 求 $a^2+c^2$ 的取值范围.", + "solution": "解:法一由条件得 $2 B=A+C$, 故有 $B=60^{\\circ}, A+C=120^{\\circ}$.\n由正弦定理得 \n$$\n\\begin{aligned}\na^2+c^2 & =(2 R \\sin A)^2+(2 R \\sin C)^2 \\\\\n& =\\sin ^2 A+\\sin ^2 C=\\frac{1-\\cos 2 A+1-\\cos 2 C}{2} \\\\\n& =1-\\cos (A+C) \\cos (A-C) \\\\\n& =1+\\frac{1}{2} \\cos (A-C), \\\\\n\\end{aligned}\n$$\n因为 $-120^{\\circ}\\frac{3}{4}$, 故 $a^2+c^2 \\in\\left(\\frac{3}{4}, \\frac{3}{2}\\right]$.\n评注在三角形中, 三角成等差数列, 则 $B=60^{\\circ}, A+C=120^{\\circ}$, 由此结合其他条件可求三角形中的有关问题.", + "remark": "", + "figures": [] +} \ No newline at end of file diff --git a/processed_dataset/calculation/0652.json b/processed_dataset/calculation/0652.json new file mode 100644 index 0000000000000000000000000000000000000000..7c5814e0f4f183292eba42e4378627dfa862f032 --- /dev/null +++ b/processed_dataset/calculation/0652.json @@ -0,0 +1,8 @@ +{ + "source_file": "./raw_volume-zh/volume3/chapter3.tex", + "problem_type": "calculation", + "problem": "例11 根据条件,分别判定满足下列条件的 $\\triangle A B C$ 是怎样的三角形:\n(1) $a \\cos B=b \\cos A$;\n(2) $a \\cos A=b \\cos B$.", + "solution": "解:法一 (1)由正弦定理, 得 $2 R \\sin A \\cos B=2 R \\sin B \\cos A$, 即 $\\sin (A- B)=0$.\n又 $-\\pi), 设 $\\angle C A B$ 为 $\\triangle A B C$ 的最大角.\n由 $S_{\\triangle D E F}=\\frac{\\sqrt{3}}{4} E F^2$, 知 $E F$ 最大时, $S_{\\triangle D E F}$ 也最大.\n而 $E F$ 由 $\\angle B A F$ 所唯一确定, 故可选取 $\\angle B A F=\\theta$ 为自变量, 则 $\\angle A B F=120^{\\circ}-\\theta, \\angle A C E=(\\theta+\\angle C A B)-60^{\\circ}$. 由正弦定理, 得\n$$\n\\begin{aligned}\nE F & =A F+A E \\\\\n& =\\frac{c \\sin \\left(120^{\\circ}-\\theta\\right)}{\\sin 60^{\\circ}}+\\frac{b \\sin \\left(\\theta+\\frac{\\left.\\angle C A B-60^{\\circ}\\right)}{\\sin 60^{\\circ}}\\right.}{3} \\cdot \\sin (\\theta+\\varphi) .\n\\end{aligned}\n$$\n其中\n$$\n\\begin{aligned}\nk= & \\left\\{\\left[c \\cos 60^{\\circ}+b \\cos \\left(\\angle C A B-60^{\\circ}\\right)\\right]^2\\right. \\\\\n& \\left.+\\left[c \\sin 60^{\\circ}+b \\sin \\left(\\angle C A B-60^{\\circ}\\right)\\right]^2\\right\\}^{\\frac{1}{2}} \\\\\n= & \\sqrt{c^2+b^2+2 b c \\cos \\left(120^{\\circ}-\\angle C A B\\right),}\n\\end{aligned}\n$$\n$\\varphi=\\arccos \\frac{c \\cos 60^{\\circ}+b \\cos \\left(\\angle C A B-60^{\\circ}\\right)}{k}$ 为锐角.\n当 $\\theta=90^{\\circ}-\\varphi$ 时, $E F$ 有最大值 $\\frac{2 \\sqrt{3} k}{3}$, 此时 $\\theta$ 也是锐角, 故 $60^{\\circ}< \\angle C A B+\\theta<180^{\\circ}$, 相应的 $\\triangle D E F$ 确是 $\\triangle A B C$ 的外接正三角形.\n则 $S_{\\triangle D E F}$ 的最大值是 $\\frac{\\sqrt{3} k^2}{3}=\\frac{\\sqrt{3}}{3}\\left[b^2+c^2+2 b c \\cos \\left(120^{\\circ}-\\angle C A B\\right)\\right]$.", + "remark": "", + "figures": [ + "./images/volume3/figures/fig-c3e17.png" + ] +} \ No newline at end of file diff --git a/processed_dataset/calculation/0656.json b/processed_dataset/calculation/0656.json new file mode 100644 index 0000000000000000000000000000000000000000..b31389aef40dfe6309ef4941f4fca27baa6aaf68 --- /dev/null +++ b/processed_dataset/calculation/0656.json @@ -0,0 +1,8 @@ +{ + "source_file": "./raw_volume-zh/volume3/chapter3.tex", + "problem_type": "calculation", + "problem": "例18 已知 $\\triangle A B C$ 的三边 $a 、 b 、 c$ 和面积 $S$ 有如下关系: $S=a^2-(b- c)^2$, 且 $b+c=8$, 求 $\\triangle A B C$ 的面积 $S$ 的最大值.", + "solution": "分析:最值问题通常用函数性质来求.\n本题从条件出发, 利用余弦定理和三角形面积公式去求角 $A$ 的正弦函数值,再将面积 $S$ 转化为关于 $b$ 或 $c$ 的二次函数来求解.\n解法一由条件可得\n$$\nS=a^2-(b-c)^2=\\frac{1}{2} b c \\sin A,\n$$\n而\n$$\na^2-(b-c)^2=2 b c-2 b c \\cos A,\n$$\n所以\n$$\n2 b c-2 b c \\cos A=\\frac{1}{2} b c \\sin A .\n$$\n则\n$$\n\\cos A=1-\\frac{1}{4} \\sin A,\n$$\n故\n$$\n\\frac{1-\\cos A}{\\sin A}=\\frac{1}{4},\n$$\n即\n$$\n\\tan \\frac{A}{2}=\\frac{1}{4},\n$$\n所以\n$$\n\\sin A=\\frac{2 \\tan \\frac{A}{2}}{1+\\tan ^2 \\frac{A}{2}}=\\frac{8}{17}\n$$\n故\n$$\nS=\\frac{1}{2} b c \\sin A=\\frac{4}{17} b c=\\frac{4}{17} b(8-b)=\\frac{4}{17}\\left[-(b-4)^2+16\\right],\n$$\n当且仅当 $b=c=4$ 时, $S_{\\text {max }}=\\frac{64}{17}$.\n解法二利用海伦公式 $S=\\sqrt{p(p-a)(p-b)(p-c)}$ 和已知条件得\n$$\na^2-(b-c)^2=\\sqrt{\\frac{1}{16}(a+b+c)(b+c-a)(a+b-c)(a+c-b)},\n$$\n两边平方得 $a^2-(b-c)^2=\\frac{1}{16}(a+b+c)(b+c-a)$, 即\n$$\na^2=b^2+c^2-\\frac{30}{17} b c\n$$\n由此可得 $\\cos A=\\frac{15}{17}$, 故 $\\sin A=\\frac{8}{17}$.\n所以\n$$\nS=\\frac{1}{2} b c \\sin A=\\frac{4}{17} b c=\\frac{4}{17} b(8-b)=\\frac{4}{17}\\left[-(b-4)^2+16\\right],\n$$\n当且仅当 $b=c=4$ 时, $S_{\\text {max }}=\\frac{64}{17}$.", + "remark": "", + "figures": [] +} \ No newline at end of file diff --git a/processed_dataset/calculation/0657.json b/processed_dataset/calculation/0657.json new file mode 100644 index 0000000000000000000000000000000000000000..e50aa26cc96744ce52f137cfdcdc9d1497d1a8d8 --- /dev/null +++ b/processed_dataset/calculation/0657.json @@ -0,0 +1,10 @@ +{ + "source_file": "./raw_volume-zh/volume3/chapter3.tex", + "problem_type": "calculation", + "problem": "例19 如图 (), 平面上有四个点 $A 、 B 、 P 、 Q$, 其中 $A 、 B$ 为定点, 且 $A B=\\sqrt{3}, P 、 Q$ 为动点, 满足 $A P=P Q=Q B=1$, 又 $\\triangle A P B$ 和 $\\triangle P Q B$ 的面积分别为 $S$ 和 $T$, 求 $S^2+T^2$ 的最大值.", + "solution": "$$\n\\text { 解 } \\begin{aligned}\nS & =\\frac{1}{2} P A \\cdot A B \\cdot \\sin A=\\frac{\\sqrt{3}}{2} \\sin A, \\\\\nT & =\\frac{1}{2} P Q \\cdot Q B \\cdot \\sin Q=\\frac{1}{2} \\sin Q,\n\\end{aligned}\n$$\n所以\n$$\nS^2+T^2=\\frac{3}{4} \\sin ^2 A+\\frac{1}{4} \\sin ^2 Q .\n$$\n由余弦定理, 在 $\\triangle P A B$ 中,\n$$\nP B^2=P A^2+A B^2-2 \\cdot P A \\cdot A B \\cdot \\cos A=4-2 \\sqrt{3} \\cos A,\n$$\n在 $\\triangle P Q B$ 中,\n$$\nP B^2=P Q^2+Q B^2-2 P Q \\cdot Q B \\cos Q=2-2 \\cos Q,\n$$\n所以\n$$\n4-2 \\sqrt{3} \\cos A=2-2 \\cos Q,\n$$\n即 $\\cos Q=\\sqrt{3} \\cos A-1$.\n所以\n$$\n\\begin{aligned}\nS^2+T^2 & =-\\frac{3}{4}\\left(1-\\cos ^2 A\\right)+\\frac{1}{4}\\left(1-\\cos ^2 Q\\right) \\\\\n& =-\\frac{3}{2} \\cos ^2 A+\\frac{\\sqrt{3}}{2} \\cos A+\\frac{3}{4} \\\\\n& =-\\frac{3}{2}\\left(\\cos A-\\frac{\\sqrt{3}}{6}\\right)^2+\\frac{7}{8}\n\\end{aligned}\n$$\n当 $\\cos A=\\frac{\\sqrt{3}}{6}$ 时, $S^2+T^2$ 有最大值 $\\frac{7}{8}$.", + "remark": "", + "figures": [ + "./images/volume3/figures/fig-c3e19.png" + ] +} \ No newline at end of file diff --git a/processed_dataset/calculation/0658.json b/processed_dataset/calculation/0658.json new file mode 100644 index 0000000000000000000000000000000000000000..42d1c8e59f1b54b4b48b73d97aa708c3b2769a54 --- /dev/null +++ b/processed_dataset/calculation/0658.json @@ -0,0 +1,8 @@ +{ + "source_file": "./raw_volume-zh/volume3/chapter4.tex", + "problem_type": "calculation", + "problem": "例1 设 $f(x)=x^2-\\pi x, \\alpha=\\arcsin \\frac{1}{3}, \\beta=\\arctan \\frac{5}{4}, \\gamma=\\arccos \\left(-\\frac{1}{3}\\right)$, $\\varphi=\\operatorname{arccot}\\left(-\\frac{5}{4}\\right)$, 则 $(\\quad)$.\n(A) $f(\\alpha)>f(\\beta)>f(\\varphi)>f(\\gamma)$\n(B) $f(\\alpha)>f(\\varphi)>f(\\beta)>f(\\gamma)$\n(C) $f(\\varphi)>f(\\alpha)>f(\\beta)>f(\\gamma)$\n(D) $f(\\varphi)>f(\\alpha)>f(\\gamma)>f(\\beta)$", + "solution": "解:选 B. 由题意, $f(x)$ 的图象关于直线 $x=\\frac{\\pi}{2}$ 对称, 且在 $\\left(-\\infty, \\frac{\\pi}{2}\\right)$ 内单调递减, 在 $\\left(\\frac{\\pi}{2},+\\infty\\right)$ 内单调递增, 所以, 当 $\\left|x_1-\\frac{\\pi}{2}\\right|>\\left|x_2-\\frac{\\pi}{2}\\right|$ 时, 有$f\\left(x_1\\right)>f\\left(x_2\\right)$.\n又知 $0<\\alpha<\\frac{\\pi}{6}, \\frac{\\pi}{4}<\\beta<\\frac{\\pi}{3}, \\frac{\\pi}{2}<\\gamma<\\frac{2 \\pi}{3}, \\frac{3 \\pi}{4}<\\varphi<\\frac{5 \\pi}{6}$, 所以 $0< \\left|\\gamma-\\frac{\\pi}{2}\\right|<\\frac{\\pi}{6}<\\left|\\beta-\\frac{\\pi}{2}\\right|<\\frac{\\pi}{4}<\\left|\\varphi-\\frac{\\pi}{2}\\right|<\\frac{\\pi}{3}<\\left|\\alpha-\\frac{\\pi}{2}\\right|<\\frac{\\pi}{2}$, 故有 $f(\\alpha)>f(\\varphi)>f(\\beta)>f(\\gamma)$.\n评注正确估算角的范围是求解本题的关键.", + "remark": "", + "figures": [] +} \ No newline at end of file diff --git a/processed_dataset/calculation/0659.json b/processed_dataset/calculation/0659.json new file mode 100644 index 0000000000000000000000000000000000000000..8aee9a95d08007a335fbe51ea766e48cdbe51e30 --- /dev/null +++ b/processed_dataset/calculation/0659.json @@ -0,0 +1,8 @@ +{ + "source_file": "./raw_volume-zh/volume3/chapter4.tex", + "problem_type": "calculation", + "problem": "例6 求方程 $x \\sqrt{y-1}+y \\sqrt{x-1}=x y$ 的实数解.", + "solution": "解:由题意知 $x, y>1$, 可设 $x=\\sec ^2 \\alpha, y=\\csc ^2 \\beta$, 其中 $0<\\alpha 、 \\beta<\\frac{\\pi}{2}$, 代入原方程得,\n$$\n\\sec ^2 \\alpha \\sqrt{\\csc ^2 \\beta-1}+\\csc ^2 \\beta \\cdot \\sqrt{\\sec ^2 \\alpha-1}=\\sec ^2 \\alpha \\csc ^2 \\beta,\n$$\n即\n$$\n\\frac{1}{\\cos ^2 \\alpha} \\cdot \\frac{\\cos \\beta}{\\sin \\beta}+\\frac{1}{\\sin ^2 \\beta} \\cdot \\frac{\\sin \\alpha}{\\cos \\alpha}=\\frac{1}{\\cos ^2 \\alpha \\sin ^2 \\beta}\n$$\n$$\n\\begin{array}{lc}\n\\Rightarrow & \\sin \\beta \\cos \\beta+\\cos \\alpha \\sin \\alpha=1 \\\\\n\\Rightarrow & \\sin 2 \\alpha+\\sin 2 \\beta=2 \\text {, 又 } \\sin 2 \\alpha \\leqslant 1, \\sin ^2 \\beta \\leqslant 1,\n\\end{array}\n$$\n故 $\\sin 2 \\alpha=\\sin 2 \\beta=1$, 从而 $\\alpha=\\beta=\\frac{\\pi}{4}$.\n所以 $x=y=2$, 经检验知, $x=y=2$ 是原方程的解.\n评注恰当的三角换元,能够使复杂的代数问题转化为简单的三角问题,使问题得到解决.", + "remark": "", + "figures": [] +} \ No newline at end of file diff --git a/processed_dataset/calculation/0660.json b/processed_dataset/calculation/0660.json new file mode 100644 index 0000000000000000000000000000000000000000..b84411ee9b8efcd3c2f437ac91543650f4d1cd4c --- /dev/null +++ b/processed_dataset/calculation/0660.json @@ -0,0 +1,8 @@ +{ + "source_file": "./raw_volume-zh/volume3/chapter4.tex", + "problem_type": "calculation", + "problem": "例7 若 $[x]$ 表示不超过实数 $x$ 的最大整数,则方程 $[\\cot x]=2 \\cos ^2 x$ 的解集是", + "solution": "解:因为 $[\\cot x]=2 \\cos ^2 x$, 故 $2 \\cos ^2 x$ 取整数.\n又 $2 \\cos ^2 x \\in[0,2]$, 所以 $\\cos ^2 x=0, \\cos ^2 x=\\frac{1}{2}, \\cos ^2 x=1$.\n当 $\\cos x=0$ 时, $x=k \\pi+\\frac{\\pi}{2}, k \\in \\mathbf{Z}$.\n当 $\\cos x= \\pm \\frac{\\sqrt{2}}{2}$ 时, $\\cot x= \\pm 1$, 只有 $x=k \\pi+\\frac{\\pi}{4}, k \\in \\mathbf{Z}$.\n当 $\\cos x= \\pm 1$ 时, $\\sin x=0$, 无意义,舍去.\n故应填 $x=k \\pi+\\frac{\\pi}{2}$ 或 $x=k \\pi+\\frac{\\pi}{4}, k \\in \\mathbf{Z}$.", + "remark": "", + "figures": [] +} \ No newline at end of file diff --git a/processed_dataset/calculation/0661.json b/processed_dataset/calculation/0661.json new file mode 100644 index 0000000000000000000000000000000000000000..9b3587b1630c5fda11813a9a9e1974edad30aad1 --- /dev/null +++ b/processed_dataset/calculation/0661.json @@ -0,0 +1,8 @@ +{ + "source_file": "./raw_volume-zh/volume3/chapter4.tex", + "problem_type": "calculation", + "problem": "例11 若 $x_1 、 x_2$ 是方程 $x^2+6 x+7=0$ 的两根, 求 $\\arctan x_1+\\arctan x_2$ 值.", + "solution": "分析:由一元二次方程之根联想韦达定理, 利用结果求 $\\arctan x_1+ \\arctan x_2$ 的某个三角函数值, 再反求其角度.\n解由\n$$\nx_1+x_2=-6, x_1 \\cdot x_2=7 \\text {, }\n$$\n得\n$$\n\\tan \\left(\\arctan x_1+\\arctan x_2\\right)=1 \\text {, }\n$$\n且由\n$$\nx_1<0, x_2<0 \\text {, }\n$$\n可知\n$$\n\\arctan x_1, \\arctan x_2 \\in\\left(-\\frac{\\pi}{2}, 0\\right),\n$$\n得\n$$\n\\arctan x_1+\\arctan x_2 \\in(-\\pi, 0),\n$$\n从而\n$$\n\\arctan x_1+\\arctan x_2=-\\frac{3}{4} \\pi\n$$\n评注当所求角的正切值为 1 时, 学生误认为所求角一定为 $\\frac{\\pi}{4}$. 其实不然, 由 $x_1<0, x_2<0$, 可知 $\\arctan x_1, \\arctan x_2 \\in\\left(-\\frac{\\pi}{2}, 0\\right)$ 是本题之关键, 即估算所求角大致在什么范围, 是引起学生警惕的重要步骤.\n推广: 当 $x_1 、 x_2$ 是关于 $x$ 的方程 $x^2+m x+(m+1)=0$ 的两根, 且 $m>$ 0 , 则 $\\arctan x_1+\\arctan x_2=-\\frac{3}{4} \\pi$.", + "remark": "", + "figures": [] +} \ No newline at end of file diff --git a/processed_dataset/calculation/0662.json b/processed_dataset/calculation/0662.json new file mode 100644 index 0000000000000000000000000000000000000000..cc65a0a3b8942f2cf4cb13f2d3c7330a7a09ff7a --- /dev/null +++ b/processed_dataset/calculation/0662.json @@ -0,0 +1,8 @@ +{ + "source_file": "./raw_volume-zh/volume3/chapter4.tex", + "problem_type": "calculation", + "problem": "例12 若 $x_1 、 x_2$ 是方程 $x^2-x \\sin \\alpha+\\cos \\alpha=0$ 的两个根, 且 $0<\\alpha<\\pi$, 求 $\\arctan x_1+\\arctan x_2$ 的值.", + "solution": "解:由韦达定理, $x_1+x_2=\\sin \\alpha, x_1 \\cdot x_2=\\cos \\alpha$, 所以\n$$\n\\begin{aligned}\n\\tan \\left(\\arctan x_1+\\arctan x_2\\right) & =\\frac{x_1+x_2}{1-x_1 x_2}=\\frac{\\sin \\alpha}{1-\\cos \\alpha} \\\\\n& =\\cot \\frac{\\alpha}{2}=\\tan \\left(\\frac{\\pi}{2}-\\frac{\\alpha}{2}\\right) .\n\\end{aligned}\n$$\n因为 $-\\pi<\\arctan x_1+\\arctan x_2<\\pi$, 以及 $0<\\alpha<\\pi$, 故\n$$\n0<\\frac{\\pi}{2}-\\frac{\\alpha}{2}<\\frac{\\pi}{2}\n$$\n所以\n$$\n\\arctan x_1+\\arctan x_2=\\frac{\\pi}{2}-\\frac{\\alpha}{2}\n$$\n或\n$$\n\\arctan x_1+\\arctan x_2=-\\frac{\\pi}{2}-\\frac{\\alpha}{2} .\n$$", + "remark": "", + "figures": [] +} \ No newline at end of file diff --git a/processed_dataset/calculation/0663.json b/processed_dataset/calculation/0663.json new file mode 100644 index 0000000000000000000000000000000000000000..e7ffc1e0e0c73e9faf042df06f1a70ea762b0a19 --- /dev/null +++ b/processed_dataset/calculation/0663.json @@ -0,0 +1,8 @@ +{ + "source_file": "./raw_volume-zh/volume3/chapter4.tex", + "problem_type": "calculation", + "problem": "例13 求函数 $f(x)=2 \\arccos \\left(\\frac{x^2-x}{2}\\right)$ 的定义域、值域及单调区间.\n分析从定义出发, 根据反三角函数的图象与性质回答问题.", + "solution": "解:由 $-1 \\leqslant \\frac{x^2-x}{2} \\leqslant 1$, 得定义域是 $x \\in[-1,2]$;\n由 $\\frac{x^2-x}{2}=\\frac{1}{2}\\left[\\left(x-\\frac{1}{2}\\right)^2-\\frac{1}{4}\\right] \\in\\left[-\\frac{1}{8}, 1\\right]$,\n故值域是\n$$\ny \\in\\left[0,2 \\arccos \\left(-\\frac{1}{8}\\right)\\right]\n$$\n由上述两步可知 $x \\in\\left[-1, \\frac{1}{2}\\right]$ 时, $f(x)$ 是单调递增的, $x \\in\\left[\\frac{1}{2}, 2\\right]$ 时, $f(x)$ 是单调递减的.\n评注在求函数单调性时, 必须考虑函数定义域.\n有关复合函数的单调区间求法可根据“增增得增, 增减得减, 减减得增” 的记忆法则求之, 本题中设 $t=\\frac{1}{2}\\left(x^2-x\\right)$, 则 $y=2 \\arccos t$ 是复合函数.", + "remark": "", + "figures": [] +} \ No newline at end of file diff --git a/processed_dataset/calculation/0664.json b/processed_dataset/calculation/0664.json new file mode 100644 index 0000000000000000000000000000000000000000..cb2e0b35d0f2aa7d617d36caccbf582719c6f856 --- /dev/null +++ b/processed_dataset/calculation/0664.json @@ -0,0 +1,8 @@ +{ + "source_file": "./raw_volume-zh/volume3/chapter4.tex", + "problem_type": "calculation", + "problem": "例14 满足 $\\arccos (1-x) \\geqslant \\arccos x$ 的 $x$ 的取值范围是 ( ).\n(A) $\\left[-1,-\\frac{1}{2}\\right]$\n(B) $\\left[-\\frac{1}{2}, 0\\right]$\n(C) $\\left[0, \\frac{1}{2}\\right]$\n(D) $\\left[\\frac{1}{2}, 1\\right]$", + "solution": "分析:反余弦函数 $y=\\arccos x$ 在定义域 $[-1,1]$ 内是单调递减函数,所以有 $\\left\\{\\begin{array}{l}|1-x| \\leqslant 1, \\\\ |x| \\leqslant 1, \\\\ 1-x \\leqslant x,\\end{array}\\right.$ 解得 $x \\in\\left[\\frac{1}{2}, 1\\right]$.\n所以, 选 D.\n评注函数定义域是必须考虑的条件, 本题如仅考虑 $1-x \\leqslant x$, 在选项中最佳答案仍是 $\\mathrm{D}$, 但这仅仅是“偶尔恰恰”而已, 不能作一般方法.", + "remark": "", + "figures": [] +} \ No newline at end of file diff --git a/processed_dataset/calculation/0665.json b/processed_dataset/calculation/0665.json new file mode 100644 index 0000000000000000000000000000000000000000..e889b905261241022986f93c29c75e04f47e9567 --- /dev/null +++ b/processed_dataset/calculation/0665.json @@ -0,0 +1,8 @@ +{ + "source_file": "./raw_volume-zh/volume3/chapter4.tex", + "problem_type": "calculation", + "problem": "例15 方程 $\\sin 2 x=\\sin x$ 在区间 $(0,2 \\pi)$ 内解的个数是 ( ).\n(A) 1\n(B) 2\n(C) 3\n(D) 4", + "solution": "分析:对于选择题来说, 可利用直接求解法或通项求解法.\n解法一将原方程化为 $\\sin x(2 \\cos x-1)=0$, 在区间 $(0,2 \\pi)$ 内 $\\sin x=$ 0 有一解 $x=\\pi ; 2 \\cos x-1=0$ 有两解 $x=\\frac{\\pi}{3}$ 或 $\\frac{5 \\pi}{3}$; 故共有三解.\n解法二对于简单的三角方程 $\\sin x=\\sin \\alpha$, 其解是 $x=n \\pi+(-1)^n \\alpha$, $(n \\in \\mathbf{Z})$, 因此本题解法二, 由原方程得 $2 x=n \\pi+(-1)^n x$, 即 $x=2 k \\pi$ 或 $x= \\frac{2 k+1}{3} \\pi$,在区间 $(0,2 \\pi)$ 内, 取 $k=0 、 1 、 2$ 得三解.\n解法三遇到形如 $\\sin \\alpha \\pm \\sin \\beta$ 等形式的三角函数式也可联想到和差化积, 即原方程化为 $\\sin 2 x-\\sin x=0$, 得 $2 \\cos \\frac{3 x}{2} \\sin \\frac{x}{2}=0$, 从而在区间 $(0,2 \\pi)$ 内得三解.\n所以, 选 C.", + "remark": "", + "figures": [] +} \ No newline at end of file diff --git a/processed_dataset/calculation/0666.json b/processed_dataset/calculation/0666.json new file mode 100644 index 0000000000000000000000000000000000000000..775de525bceda9ca4cde730f55949fe6566a1d24 --- /dev/null +++ b/processed_dataset/calculation/0666.json @@ -0,0 +1,8 @@ +{ + "source_file": "./raw_volume-zh/volume3/chapter4.tex", + "problem_type": "calculation", + "problem": "例16 解方程 $\\sin ^2 x-3 \\sin x \\cos x+1=0$.", + "solution": "分析:通过“ “ ”的代换可将原方程化成齐次型,转化为关于 $\\tan x$ 的二次方程再解之; 亦可通过 “降次”, 转化为 $a \\sin 2 x+b \\cos 2 x=c$ 的方程解之.\n解法一由原方程得\n$$\n2 \\sin ^2 x-3 \\sin x \\cos x+\\cos ^2 x=0,\n$$\n显然 $\\cos x \\neq 0$, 则有\n$$\n2 \\tan ^2 x-3 \\tan x+1=0,\n$$\n得\n$$\n\\tan x=\\frac{1}{2} \\text { 或 } \\tan x=1 \\text {. }\n$$\n从而 $x=k \\pi+\\arctan \\frac{1}{2}, k \\in \\mathbf{Z}$ 或 $x=k \\pi+\\frac{\\pi}{4}, k \\in \\mathbf{Z}$.\n解法二由原方程得\n$$\n\\frac{1-\\cos 2 x}{2}-\\frac{3}{2} \\sin 2 x+1=0,\n$$\n即\n$$\n3 \\sin 2 x+\\cos 2 x=3 \\text {. }\n$$\n于是\n$$\n\\sin (2 x+\\varphi)=\\frac{3}{\\sqrt{10}},\n$$\n其中\n$$\n\\varphi=\\arctan \\frac{1}{3}\n$$\n所以\n$$\n2 x+\\varphi=k \\pi+(-1)^k \\arcsin \\frac{3}{\\sqrt{10}} .\n$$\n故 $x=\\frac{1}{2} k \\pi+\\frac{1}{2}(-1)^k \\arcsin \\frac{3}{\\sqrt{10}}-\\frac{1}{2} \\arctan \\frac{1}{3}, k \\in \\mathbf{Z}$.\n评注三角方程的解的表示法并不唯一, 在本题中角 $\\arcsin \\frac{3}{\\sqrt{10}}$ 与角 $\\arctan \\frac{1}{3}$ 互余,两种答案是等价的; 形如 $a \\sin x+b \\cos x=c$ 的三角方程通常在方程两边除以 $\\sqrt{a^2+b^2}$ 后化成 $\\sin (x+\\varphi)=\\frac{c}{\\sqrt{a^2+b^2}}$, 其中 $\\sin \\varphi= \\frac{b}{\\sqrt{a^2+b^2}}, \\cos \\varphi=\\frac{a}{\\sqrt{a^2+b^2}}$, 再求之; 而形如 $a \\sin x+b \\cos x=0$ 和\n$a \\sin ^2 x+b \\sin x \\cos x+c \\cos ^2 x=0$ 等齐次型, 可转化为关于 $\\tan x$ 的二次方程再求解.", + "remark": "", + "figures": [] +} \ No newline at end of file diff --git a/processed_dataset/calculation/0667.json b/processed_dataset/calculation/0667.json new file mode 100644 index 0000000000000000000000000000000000000000..3fbdf40a93de9a232850c1de87a620b4b1e068c8 --- /dev/null +++ b/processed_dataset/calculation/0667.json @@ -0,0 +1,10 @@ +{ + "source_file": "./raw_volume-zh/volume3/chapter4.tex", + "problem_type": "calculation", + "problem": "例17 关于 $x$ 的方程 $\\sin x+\\sqrt{3} \\cos x+a=0$ 在 $(0,2 \\pi)$ 内有两个相异的实数解 $\\alpha 、 \\beta$, 求实数 $a$ 的取值范围及 $\\alpha+\\beta$ 之值.", + "solution": "分析:将原方程化为 $2 \\sin \\left(x+\\frac{\\pi}{3}\\right)+a=0$, 再由值域求 $a$ 的取值范围.\n也可将方程之解看成两个函数图象的交点横坐标, 从图象观察出 $a$ 的取值范围, 由此解法同时可求 $\\alpha+\\beta$ 之值.\n解法一由原方程得 $2 \\sin \\left(x+\\frac{\\pi}{3}\\right)+a=0$, 结合题设条件得\n$$\n\\left\\{\\begin{array}{l}\n|-a|<2, \\\\\n-a \\neq \\sqrt{3} .\n\\end{array}\\right.\n$$\n即 $-2), 由此观察, 即得 $\\left\\{\\begin{array}{l}|-a|<2, \\\\ -a \\neq \\sqrt{3},\\end{array}\\right.$ 即 $-2), 由此观察, 得\n(1)当 $a-\\frac{3}{4}<0$ 或 $a-\\frac{3}{4}>\\frac{9}{4}$,\n即 $a<\\frac{3}{4}$ 或 $a>3$ 时, 方程无解;\n(2)当 $a-\\frac{3}{4}=\\frac{9}{4}$, 即 $a=3$ 时,方程有一解 $x=\\frac{3}{2} \\pi$;\n(3)当 $\\frac{1}{4}$ 0 恒成立, 试求 $\\theta$ 的取值范围.", + "solution": "分析:将原不等式看作关于 $x$ 的一元二次不等式,结合二次函数的图象,列出关于 $\\theta$ 的三角不等式,求出 $\\theta$ 的取值范围.\n解令\n$$\n\\begin{aligned}\nf(x) & =x^2 \\cos \\theta-x(1-x)+(1-x)^2 \\sin \\theta \\\\\n& =(\\cos \\theta+\\sin \\theta+1) x^2-(1+2 \\sin \\theta) x+\\sin \\theta .\n\\end{aligned}\n$$\n对称轴 $x=\\frac{1+2 \\sin \\theta}{2(\\cos \\theta+\\sin \\theta+1)}=\\frac{2 \\sin \\theta+1}{2 \\sin \\theta+2 \\cos \\theta+2}$.\n由条件知 $\\left\\{\\begin{array}{l}f(0)=\\sin \\theta>0, \\\\ f(1)=\\cos \\theta>0,\\end{array}\\right.$ 从而 $\\frac{2 \\sin \\theta+1}{2 \\sin \\theta+2 \\cos \\theta+2} \\in(0,1)$, 要使 $f(x)$ 在 $[0,1]$ 时 $f(x)>0$ 恒成立, 必须有\n$$\n\\Delta=(1+2 \\sin \\theta)^2-4 \\sin \\theta(\\cos \\theta+\\sin \\theta+1)<0,\n$$\n解得 $\\sin 2 \\theta>\\frac{1}{2}$, 又 $\\sin \\theta>0, \\cos \\theta>0$, 所以\n$$\n2 k \\pi+\\frac{\\pi}{12}<\\theta<2 k \\pi+\\frac{5 \\pi}{12}(k \\in \\mathbf{Z}) .\n$$\n评注本题中对称轴的范围是关键.", + "remark": "", + "figures": [] +} \ No newline at end of file diff --git a/processed_dataset/calculation/0671.json b/processed_dataset/calculation/0671.json new file mode 100644 index 0000000000000000000000000000000000000000..4999105c565d24405e316eedad1604248f30603a --- /dev/null +++ b/processed_dataset/calculation/0671.json @@ -0,0 +1,8 @@ +{ + "source_file": "./raw_volume-zh/volume3/chapter6.tex", + "problem_type": "calculation", + "problem": "例3. 设 $a 、 b 、 c$ 是正实数且 $a b c+a+c=b$, 求 $P=\\frac{2}{a^2+1}+\\frac{3}{c^2+1}- \\frac{2}{b^2+1}$ 的最大值.", + "solution": "解:由 $a b c+a+c=b \\Rightarrow b=\\frac{a+c}{1-a c}(a c \\neq 1)$, 令 $a=\\tan \\alpha, b=\\tan \\beta$, $c=\\tan \\gamma\\left(\\alpha, \\beta, \\gamma \\in\\left(0, \\frac{\\pi}{2}\\right)\\right)$, 则 $\\tan \\beta=\\frac{\\tan \\alpha+\\tan \\gamma}{1-\\tan \\alpha \\tan \\gamma}=\\tan (\\alpha+\\gamma)$, 故 $\\beta= \\alpha+\\gamma$,从而\n$$\n\\begin{aligned}\nP & =\\frac{2}{1+\\tan ^2 \\alpha}-\\frac{2}{1+\\tan ^2 \\beta}+\\frac{3}{1+\\tan ^2 \\gamma} \\\\\n& =2 \\cos ^2 \\alpha-2 \\cos ^2(\\alpha+\\gamma)+3 \\cos ^2 \\gamma \\\\\n& =1+\\cos 2 \\alpha-[1+\\cos 2(\\alpha+\\gamma)]+3 \\cos ^2 \\gamma \\\\\n& =2 \\sin \\gamma \\sin (2 \\alpha+\\gamma)+3 \\cos ^2 \\gamma \\leqslant 2 \\sin \\gamma+3-3 \\sin ^2 \\gamma \\\\\n& =-3\\left(\\sin \\gamma-\\frac{1}{3}\\right)^2+\\frac{10}{3} \\leqslant \\frac{10}{3} .\n\\end{aligned}\n$$\n当且仅当 $2 \\alpha+\\gamma=\\frac{\\pi}{2}$ 且 $\\sin \\gamma=\\frac{1}{3}$ 时, 等号成立.\n即 $a=\\frac{\\sqrt{2}}{2}, b=\\sqrt{2}, c=$\n$$\n\\frac{\\sqrt{2}}{4} \\text { 时, } P_{\\text {max }}=\\frac{10}{3} \\text {. }\n$$\n评注形如 $\\frac{2 x}{1 \\pm x}, \\frac{1-x^2}{1+x^2}, \\frac{x-y}{1+x y}, \\frac{x+y}{1-x y}, \\sqrt{a^2+x^2}$ 等代数式中的 $x, y$,可用正 (余) 切代换, 而 $\\sqrt{1-x^2}$ 之类的 $x$ 常可用正 (余) 弦代换, $\\sqrt{x^2-a^2}$ 之类用正(余) 割代换.", + "remark": "", + "figures": [] +} \ No newline at end of file diff --git a/processed_dataset/calculation/0672.json b/processed_dataset/calculation/0672.json new file mode 100644 index 0000000000000000000000000000000000000000..4a834b5fa88fe44d35a11262004606a654efaea8 --- /dev/null +++ b/processed_dataset/calculation/0672.json @@ -0,0 +1,10 @@ +{ + "source_file": "./raw_volume-zh/volume3/chapter6.tex", + "problem_type": "calculation", + "problem": "例4. 由沿河城市 $A$ 运货到城市 $B, B$ 离河岸最近点 $C$ 为 $30 \\mathrm{~km}, C$ 和 $A$ 的距离为 $40 \\mathrm{~km}$, 如果每公里运费水路比公路便宜一半, 如图() 所示,计划沿 $B D$ 修一条公路,则 $A 、 D$ 之间距离为多少时,运费最低?", + "solution": "分析:利用函数求最值问题时, 关键是参数的假定, 究竟以哪个量作为自变量呢? 本题设$\\angle C D B=\\theta$ 最佳.\n解设 $\\angle B D C=\\theta$, 则 $B D=\\frac{30}{\\sin \\theta}, A D=40-\\frac{30}{\\tan \\theta}$, 且 $\\arctan \\frac{3}{4}<\\theta<\\frac{\\pi}{2}$.\n设每公里运费水路和公路分别为 $a$ 和 $2 a$, 不妨设由 $A$ 到 $B$ 的单程运费为 $y$, 则\n$$\ny=a\\left(40-\\frac{30}{\\tan \\theta}\\right)+2 a \\cdot \\frac{30}{\\sin \\theta}=10 a \\cdot\\left(4+3 \\cdot \\frac{2-\\cos \\theta}{\\sin \\theta}\\right),\n$$\n令 $k=\\frac{2-\\cos \\theta}{\\sin \\theta}$, 则 $\\cos \\theta+k \\sin \\theta=2, \\sqrt{1+k^2} \\sin (\\theta+\\varphi)=2$.\n其中 $\\sin \\varphi=\\frac{1}{\\sqrt{1+k^2}}, \\cos \\varphi=\\frac{k}{\\sqrt{1+k^2}}$, 于是 $\\frac{2}{\\sqrt{1+k^2}} \\leqslant 1$, 解得\n$$\nk \\geqslant \\sqrt{3} \\text {, 即 } y \\geqslant 10(4+3 \\sqrt{3}) a \\text {. }\n$$\n所以, 当 $k=\\sqrt{3}$, 即 $\\cos \\theta+\\sqrt{3} \\sin \\theta=2, \\theta=60^{\\circ}$ 时, $y$ 有最小值 $10(4+3 \\sqrt{3}) a$, 此时 $A D=40-10 \\sqrt{3}(\\mathrm{~km})$.\n答: 当 $A 、 D$ 之间距离为 $40-10 \\sqrt{3} \\mathrm{~km}$ 时, 运费最低.\n评注本题如直接设 $A D=x \\mathrm{~km}$, 则 $C D=40-x, B D= \\sqrt{30^2+(40-x)^2}$, 从而 $y=a x+2 a \\sqrt{900+(40-x)^2}$, 这个函数的最值问题, 仍宜采用三角换元法来解.", + "remark": "", + "figures": [ + "./images/volume3/figures/fig-c6i2.png" + ] +} \ No newline at end of file diff --git a/processed_dataset/calculation/0673.json b/processed_dataset/calculation/0673.json new file mode 100644 index 0000000000000000000000000000000000000000..294a5f53fe948465806ff54320152821e0397be1 --- /dev/null +++ b/processed_dataset/calculation/0673.json @@ -0,0 +1,14 @@ +{ + "source_file": "./raw_volume-zh/volume3/chapter6.tex", + "problem_type": "calculation", + "problem": "例6. 要想在一块圆心角为 $\\alpha(0<\\alpha<\\pi)$ 、半径为 $R$ 的扇形铁板中截出一块面积最大的矩形, 应该怎样截取? 求出这个矩形的面积.", + "solution": "解:(1) 当 $0<\\alpha \\leqslant \\frac{\\pi}{2}$ 时,有两种截取的情形:\n情形 1: 如图(), 矩形的一条边落在半径上, 设 $A B=x, A D=y$, 在 Rt $\\triangle A O D$ 中, $O D=\\frac{y}{\\sin \\alpha}$, 在 $\\triangle O D C$ 中, $\\angle O D C=\\pi-\\alpha$, 由余弦定理得\n$$\nR^2=x^2+\\frac{y^2}{\\sin ^2 \\alpha}-2 \\cdot x \\cdot \\frac{y}{\\sin \\alpha} \\cos (\\pi-\\alpha) \\geqslant \\frac{2 x y}{\\sin \\alpha}+\\frac{2 x y \\cos \\alpha}{\\sin \\alpha},\n$$\n所以 $x y \\leqslant \\frac{R^2 \\sin \\alpha}{2(1+\\cos \\alpha)}=\\frac{1}{2} R^2 \\tan \\frac{\\alpha}{2}$. 当且仅当 $x=\\frac{y}{\\sin \\alpha}$ 时等号成立.\n结合 $x y=\\frac{1}{2} R^2 \\tan \\frac{\\alpha}{2}$, 易求 $y=R \\sin \\frac{\\alpha}{2}, O D=\\frac{R}{2 \\cos \\frac{\\alpha}{2}}$.\n情形 2 : 如图(), 矩形的两个顶点分别在两条半径上,另两个点在圆弧上, 如图所示, $E 、 F$ 分别是 $A B 、 C D$ 的中点, 则可化为情形 1 , 先求矩形 $E F C B$ 面积最大值, 最大面积为 $\\frac{R^2}{2} \\tan \\frac{\\alpha}{4}$. 根据对称性, 矩形 $A B C D$ 的最大面积为 $2 \\cdot \\frac{R^2}{2} \\tan \\frac{\\alpha}{4}=R^2 \\tan \\frac{\\alpha}{4}$. 此时 $O A=O B=\\frac{R}{2 \\cos \\frac{\\alpha}{4}}$.\n因为 $\\tan \\frac{\\alpha}{2}=\\frac{2 \\tan \\frac{\\alpha}{4}}{1-\\tan ^2 \\frac{\\alpha}{4}}, 0<\\alpha \\leqslant \\frac{\\pi}{2}$, 所以 $0<\\frac{\\alpha}{4}<\\frac{\\pi}{8}, 0<1- \\tan ^2 \\frac{\\alpha}{4}<1$. 从而 $\\tan \\frac{\\alpha}{2}>2 \\tan \\frac{\\alpha}{4}$, 即 $\\frac{R^2}{2} \\tan \\frac{\\alpha}{2}>R^2 \\tan \\frac{\\alpha}{4}$. 即 $0<\\alpha \\leqslant \\frac{\\pi}{2}$ 时, $S_{\\max }=\\frac{R^2}{2} \\tan \\frac{\\alpha}{2}$.\n (2)当 $\\frac{\\alpha}{2}<\\alpha<\\pi$ 时,也有两种截取的情形:\n情形 3: 如图(), 矩形的一条边在半径上, 设 $\\angle A O B=\\theta$, 则 $O A=R \\cos \\theta$, $A B=R \\sin \\theta$, 矩形 $O A B C$ 的面积\n$$\nS=O A \\cdot A B=R^2 \\sin \\theta \\cos \\theta=\\frac{1}{2} R^2 \\sin 2 \\theta .\n$$\n故 $\\theta=\\frac{\\pi}{4}$ 时, $S_{\\text {max }}=\\frac{1}{2} R^2$.\n情形 4: 如图(), 矩形的两个顶点分别在两条半径上, 另两个顶点在圆弧上, 同情形 2 , 可求\n$S_{\\text {max }}=R^2 \\tan \\frac{\\alpha}{4}$. 当 $-\\frac{1}{2} R^2 \\geqslant R^2 \\tan \\frac{\\alpha}{4}$, 即 $\\frac{\\pi}{2}<\\alpha \\leqslant 4 \\arctan \\frac{1}{2}$ 时, $S_{\\text {max }}=\\frac{R^2}{2}$ ;\n当 $\\frac{1}{2} R^2), 截取 $O A=\\frac{R}{2 \\cos \\frac{\\alpha}{4}}$. 且\n$$\nS_{\\max }=\\left\\{\\begin{array}{l}\n\\frac{1}{2} R^2 \\tan \\frac{\\alpha}{2}, 0<\\alpha \\leqslant \\frac{\\pi}{2}, \\\\\n\\frac{R^2}{2}, \\frac{\\pi}{2}<\\alpha \\leqslant 4 \\arctan \\frac{1}{2}, \\\\\nR^2 \\tan \\frac{\\alpha}{4}, 4 \\arctan \\frac{1}{2}<\\alpha<\\pi .\n\\end{array}\\right.\n$$", + "remark": "", + "figures": [ + "./images/volume3/figures/fig-c6i3-1.png", + "./images/volume3/figures/fig-c6i3-2.png", + "./images/volume3/figures/fig-c6i3-3.png", + "./images/volume3/figures/fig-c6i3-4.png", + "./images/volume3/figures/fig-c6i3-4.png" + ] +} \ No newline at end of file diff --git a/processed_dataset/calculation/0674.json b/processed_dataset/calculation/0674.json new file mode 100644 index 0000000000000000000000000000000000000000..b502ccedf2c57fa9d497f92dd5fa75bc926a4d3f --- /dev/null +++ b/processed_dataset/calculation/0674.json @@ -0,0 +1,8 @@ +{ + "source_file": "./raw_volume-zh/volume3/chapter6.tex", + "problem_type": "calculation", + "problem": "例7. 已知实数 $x, y$ 满足 $4 x^2-5 x y+4 y^2=5$, 求 $x^2+y^2$ 的最大值和最小值.", + "solution": "分析:从 $x^2+y^2$ 这一特征联想到 $\\sin ^2 \\alpha+\\cos ^2 \\alpha=1$, 所以利用三角换元法.\n解设 $x^2+y^2=p$, 则 $x=\\sqrt{p} \\cos \\varphi, y=\\sqrt{p} \\sin \\varphi$, 其中 $\\varphi \\in[0,2 \\pi)$,代入已知等式得 $4 p-5 p \\sin \\varphi \\cos \\varphi=5$, 得 $p=\\frac{5}{4-5 \\sin \\varphi \\cos \\varphi}=\\frac{10}{8-5 \\sin 2 \\varphi}$. 所以当 $\\sin 2 \\varphi=1$ 时, $p$ 有最大值 $\\frac{10}{3}$;\n当 $\\sin 2 \\varphi=-1$ 时, $p$ 有最小值 $\\frac{10}{13}$.\n评注当题中出现 $x^2+y^2=a^2$ 时, 可令 $x=a \\cos \\varphi, y=a \\sin \\varphi$; 若 $x^2- y^2=a^2$, 则可令 $x=a \\sec \\varphi, y=a \\tan \\varphi$; 若 $x^2+y^2 \\leqslant a^2(a>0)$ 时, 则可令 $x=k \\cos \\varphi, y=k \\sin \\varphi$, 其中 $|k| \\leqslant a$.", + "remark": "", + "figures": [] +} \ No newline at end of file diff --git a/processed_dataset/calculation/0675.json b/processed_dataset/calculation/0675.json new file mode 100644 index 0000000000000000000000000000000000000000..fe2dde6dd97ebbc68048ec1d7ef32d23dba530c7 --- /dev/null +++ b/processed_dataset/calculation/0675.json @@ -0,0 +1,8 @@ +{ + "source_file": "./raw_volume-zh/volume3/chapter6.tex", + "problem_type": "calculation", + "problem": "例10. 解方程 $2 \\sqrt{2} x^2+x-\\sqrt{1-x^2}-\\sqrt{2}=0$.", + "solution": "分析:由于 $|x| \\leqslant 1$, 可设 $x=\\sin \\theta, \\theta \\in\\left[-\\frac{\\pi}{2}, \\frac{\\pi}{2}\\right]$.\n解设 $x=\\sin \\theta, \\theta \\in\\left[-\\frac{\\pi}{2}, \\frac{\\pi}{2}\\right]$, 代入原方程, 得 $2 \\sqrt{2} \\sin ^2 \\theta+\\sin \\theta- \\cos \\theta-\\sqrt{2}=0$,\n$$\n\\begin{gathered}\n\\sqrt{2}\\left(\\sin ^2 \\theta-\\cos ^2 \\theta\\right)+\\sin \\theta-\\cos \\theta=0, \\\\\n\\sin \\theta-\\cos \\theta)[\\sqrt{2}(\\sin \\theta+\\cos \\theta)+1]=0, \\\\\n\\sin \\theta-\\cos \\theta)\\left[2 \\sin \\left(\\theta+\\frac{\\pi}{4}\\right)+1\\right]=0 .\n\\end{gathered}\n$$\n由 $\\sin \\theta-\\cos \\theta=0$ 及 $\\theta \\in\\left[-\\frac{\\pi}{2}, \\frac{\\pi}{2}\\right]$ 得 $\\theta=\\frac{\\pi}{4}, x=\\frac{\\sqrt{2}}{2}$.\n由 $\\sin \\left(\\theta+\\frac{\\pi}{4}\\right)=-\\frac{1}{2}$ 及 $\\theta \\in\\left[-\\frac{\\pi}{2}, \\frac{\\pi}{2}\\right]$ 得 $\\theta=-\\frac{5 \\pi}{12}, x=-\\frac{\\sqrt{6}+\\sqrt{2}}{4}$.\n所以原方程的解是 $x=\\frac{\\sqrt{2}}{2}$ 或 $x=-\\frac{\\sqrt{6}+\\sqrt{2}}{4}$.", + "remark": "", + "figures": [] +} \ No newline at end of file diff --git a/processed_dataset/calculation/0676.json b/processed_dataset/calculation/0676.json new file mode 100644 index 0000000000000000000000000000000000000000..fe843cf62bf7a7cd355898a8c848af40673f459b --- /dev/null +++ b/processed_dataset/calculation/0676.json @@ -0,0 +1,8 @@ +{ + "source_file": "./raw_volume-zh/volume3/chapter6.tex", + "problem_type": "calculation", + "problem": "例11. 解方程组\n$$\n\\left\\{\\begin{array}{l}\n\\left(x+\\frac{1}{x}\\right)=4\\left(y+\\frac{1}{y}\\right)=5\\left(z+\\frac{1}{z}\\right), \\quad\\quad(1)\\\\\nx y+y z+z x=1 . \\quad\\quad(2)\n\\end{array}\\right.\n$$", + "solution": "解:由(1)知 $x 、 y 、 z$ 同号, 不妨先考虑它们都是正数的情形.\n令 $x=\\tan \\frac{\\alpha}{2}, y=\\tan \\frac{\\beta}{2}, z=\\tan \\frac{\\gamma}{2}, \\alpha, \\beta, \\gamma \\in(0, \\pi)$.\n由于 $\\tan \\frac{\\theta}{2}+\\cot \\frac{\\theta}{2}=\\frac{\\sin \\frac{\\theta}{2}}{\\cos \\frac{\\theta}{2}}+\\frac{\\cos \\frac{\\theta}{2}}{\\sin \\frac{\\theta}{2}}=\\frac{1}{\\sin \\frac{\\theta}{2} \\cos \\frac{\\theta}{2}}=\\frac{2}{\\sin \\theta}$,\n所以(1)式就是\n$$\n\\frac{3}{\\sin \\alpha}=\\frac{4}{\\sin \\beta}=\\frac{5}{\\sin \\gamma} \\text {. }\n$$\n式就是 $\\tan \\frac{\\alpha}{2} \\tan \\frac{\\beta}{2}+\\tan \\frac{\\beta}{2} \\tan \\frac{\\gamma}{2}+\\tan \\frac{\\gamma}{2} \\tan \\frac{\\alpha}{2}=1$.\n变形为 $\\tan \\frac{\\gamma}{2}\\left(\\tan \\frac{\\alpha}{2}+\\tan \\frac{\\beta}{2}\\right)=1-\\tan \\frac{\\alpha}{2} \\tan \\frac{\\beta}{2}$,\n$$\n\\tan \\frac{\\gamma}{2}=\\frac{1-\\tan \\frac{\\alpha}{2} \\tan \\frac{\\beta}{2}}{\\tan \\frac{\\alpha}{2}+\\tan \\frac{\\beta}{2}}=\\cot \\left(\\frac{\\alpha}{2}+\\frac{\\beta}{2}\\right)=\\tan \\left(\\frac{\\pi}{2}-\\frac{\\alpha+\\beta}{2}\\right) .\n$$\n因为 $\\frac{\\gamma}{2} \\in\\left(0, \\frac{\\pi}{2}\\right),\\left(\\frac{\\pi}{2}-\\frac{\\alpha+\\beta}{2}\\right) \\in\\left(0, \\frac{\\pi}{2}\\right)$, 所以 $\\frac{\\gamma}{2}=\\frac{\\pi}{2}-\\frac{\\alpha+\\beta}{2}, \\alpha+ \\beta+\\gamma=\\pi$.\n$\\alpha 、 \\beta 、 \\gamma$ 是某个三角形的三个内角,且它的三边之比为 $3: 4: 5$, 由此可知 $\\sin \\gamma=1, \\sin \\alpha=\\frac{3}{5}, \\sin \\beta=\\frac{4}{5}$. 从而 $x=\\tan \\frac{\\alpha}{2}=\\frac{1}{3}, y=\\tan \\frac{\\beta}{2}=\\frac{1}{2}$, $z=\\tan \\frac{\\gamma}{2}=1$\n若 $x 、 y 、 z$ 都是负数时, 用同样的方法可求得 $x=-\\frac{1}{3}, y=-\\frac{1}{2}$, $z=-1$.\n即原方程组的解是\n$$\n\\left\\{\\begin{array}{l}\nx=\\frac{1}{3}, \\\\\ny=\\frac{1}{2}, \\text { 或 }\\left\\{\\begin{array}{l}\nx=-\\frac{1}{3}, \\\\\nz=1 .\n\\end{array}, \\frac{1}{2},\\right. \\\\\nz=-1 .\n\\end{array}\\right.\n$$\n评注形如 $x+\\frac{1}{x}$ 的代数式, 通过三角代换, 转化为 $x+\\frac{1}{x}=\\tan \\frac{\\theta}{2}+ \\cot \\frac{\\theta}{2}=\\frac{2}{\\sin \\theta}$, 是本题的重要技巧.", + "remark": "", + "figures": [] +} \ No newline at end of file diff --git a/processed_dataset/calculation/0677.json b/processed_dataset/calculation/0677.json new file mode 100644 index 0000000000000000000000000000000000000000..7f9aa4b47c01a3fb3ac5fe0cedd238889c3a62f9 --- /dev/null +++ b/processed_dataset/calculation/0677.json @@ -0,0 +1,8 @@ +{ + "source_file": "./raw_volume-zh/volume3/chapter6.tex", + "problem_type": "calculation", + "problem": "例13. 求 $y=\\frac{x-x^3}{1+2 x^2+x^4}$ 的最大、最小值.", + "solution": "分析:函数的解析式可变形为 $y=\\frac{x\\left(1-x^2\\right)}{\\left(1+x^2\\right)^2}=\\frac{x}{1+x^2} \\cdot \\frac{1-x^2}{1+x^2}$.\n解设 $x=\\tan \\theta, \\theta \\in\\left(-\\frac{\\pi}{2}, \\frac{\\pi}{2}\\right)$, 则 $y=\\frac{\\tan \\theta}{1+\\tan ^2 \\theta} \\cdot \\frac{1-\\tan ^2 \\theta}{1+\\tan ^2 \\theta}= \\frac{1}{2} \\sin 2 \\theta \\cdot \\cos 2 \\theta=\\frac{1}{4} \\sin 4 \\theta, \\sin 4 \\theta=1$ 时, $y_{\\text {max }}=\\frac{1}{4}$; $\\sin 4 \\theta=-1$ 时, $y_{\\text {min }}=-\\frac{1}{4}$.\n评注利用正切函数的和角公式是上述两题的重要方法.", + "remark": "", + "figures": [] +} \ No newline at end of file diff --git a/processed_dataset/calculation/0678.json b/processed_dataset/calculation/0678.json new file mode 100644 index 0000000000000000000000000000000000000000..132e021cd4ecf78e5c7f0bf3123f40254f3c5a51 --- /dev/null +++ b/processed_dataset/calculation/0678.json @@ -0,0 +1,8 @@ +{ + "source_file": "./raw_volume-zh/volume3/exercise1.tex", + "problem_type": "calculation", + "problem": "问题1 函数 $y=\\sin \\left(x+\\frac{3}{2} \\pi\\right)$ 的图象 ( ).\n(A) 关于 $x$ 轴对称\n(B) 关于 $y$ 轴对称\n(C) 关于原点对称\n(D) 关于直线 $x=-\\frac{3}{2} \\pi$ 对称", + "solution": "B. 因 $y=\\sin \\left(x+\\frac{3}{2} \\pi\\right)=-\\cos x$, 故选 B.", + "remark": "", + "figures": [] +} \ No newline at end of file diff --git a/processed_dataset/calculation/0679.json b/processed_dataset/calculation/0679.json new file mode 100644 index 0000000000000000000000000000000000000000..0c73d4d4dfc35f4c1f35bb1f2b52cfa97fa0e68c --- /dev/null +++ b/processed_dataset/calculation/0679.json @@ -0,0 +1,8 @@ +{ + "source_file": "./raw_volume-zh/volume3/exercise1.tex", + "problem_type": "calculation", + "problem": "问题2 営与正弦曲线 $y=\\sin x$ 关于直线 $x=\\frac{3 \\pi}{4}$ 对称的曲线是 ( ).\n(A) $y=\\sin x$\n(B) $y=\\cos x$\n(C) $y=-\\sin x$\n(D) $y=-\\cos x$", + "solution": "D. 若点 $(x, y)$ 在函数 $y=\\sin x$ 的图象上, 则它关于直线 $x=\\frac{3}{4} \\pi$ 对称的点 $\\left(\\frac{3}{2} \\pi-x, y\\right)$ 在函数 $y=-\\cos x$ 的图象上.", + "remark": "", + "figures": [] +} \ No newline at end of file diff --git a/processed_dataset/calculation/0680.json b/processed_dataset/calculation/0680.json new file mode 100644 index 0000000000000000000000000000000000000000..fa93df6b69ac61487f3519567c9d3c49447096d0 --- /dev/null +++ b/processed_dataset/calculation/0680.json @@ -0,0 +1,8 @@ +{ + "source_file": "./raw_volume-zh/volume3/exercise1.tex", + "problem_type": "calculation", + "problem": "问题3 函数 $y=-3 \\cos \\left(-2 x+\\frac{\\pi}{3}\\right)$ 的图象可由 $y=-3 \\cos (-2 x)$ 的图象( ).\n(A) 向左平移 $\\frac{\\pi}{3}$ 得到\n(B) 向右平移 $\\frac{\\pi}{3}$ 得到\n(C) 向左平移 $\\frac{\\pi}{6}$ 得到\n(D) 向右平移 $\\frac{\\pi}{6}$ 得到", + "solution": "D. 原函数可变形为 $y=-3 \\cos \\left[-2\\left(x-\\frac{\\pi}{6}\\right)\\right]$.", + "remark": "", + "figures": [] +} \ No newline at end of file diff --git a/processed_dataset/calculation/0681.json b/processed_dataset/calculation/0681.json new file mode 100644 index 0000000000000000000000000000000000000000..bfe1ec432b00d3bdd7772c9239a52aa79992ad98 --- /dev/null +++ b/processed_dataset/calculation/0681.json @@ -0,0 +1,8 @@ +{ + "source_file": "./raw_volume-zh/volume3/exercise1.tex", + "problem_type": "calculation", + "problem": "问题4 已知 $f(x)=\\sin \\left(x+\\frac{\\pi}{2}\\right), g(x)=\\cos \\left(x-\\frac{\\pi}{2}\\right)$, 则 $f(x)$ 的图象 ( ).\n(A) 与 $g(x)$ 的图象相同\n(B) 与 $g(x)$ 的图象关于 $x$ 轴对称\n(C) 是由 $g(x)$ 的图象向左平移 $\\frac{\\pi}{2}$ 个单位得到\n(D) 是由 $g(x)$ 的图象向右平移 $\\frac{\\pi}{2}$ 个单位得到", + "solution": "C. 原函数变形为 $f(x)=\\cos x, g(x)=\\sin x$.", + "remark": "", + "figures": [] +} \ No newline at end of file diff --git a/processed_dataset/calculation/0682.json b/processed_dataset/calculation/0682.json new file mode 100644 index 0000000000000000000000000000000000000000..0fdd52c7cec6e8ebb96f80f225d31a6d59cbacc2 --- /dev/null +++ b/processed_dataset/calculation/0682.json @@ -0,0 +1,8 @@ +{ + "source_file": "./raw_volume-zh/volume3/exercise1.tex", + "problem_type": "calculation", + "problem": "问题5 函数 $y=2^{-\\cos x}$ 的单调递增区间是 ( ).\n(A) $[2 k \\pi+\\pi, 2 k \\pi+2 \\pi], k \\in \\mathbf{Z}$\n(B) $[k \\pi+\\pi, k \\pi+2 \\pi], k \\in \\mathbf{Z}$\n(C) $\\left[2 k \\pi, 2 k \\pi+\\frac{\\pi}{2}\\right], k \\in \\mathbf{Z}$\n(D) $[2 k \\pi, 2 k \\pi+\\pi], k \\in \\mathbf{Z}$", + "solution": "D. 因函数 $y=2^u$ 在 $\\mathbf{R}$ 上单调递增, 而 $u=-\\cos x$ 在 $[2 k \\pi, 2 k \\pi+\\pi] (k \\in \\mathbf{Z})$ 上也单调递增.", + "remark": "", + "figures": [] +} \ No newline at end of file diff --git a/processed_dataset/calculation/0683.json b/processed_dataset/calculation/0683.json new file mode 100644 index 0000000000000000000000000000000000000000..e44cc45cf45f7f9a7bb8d3a59b39f44922f797fe --- /dev/null +++ b/processed_dataset/calculation/0683.json @@ -0,0 +1,8 @@ +{ + "source_file": "./raw_volume-zh/volume3/exercise1.tex", + "problem_type": "calculation", + "problem": "问题6 函数 $y=\\sqrt{\\cos (\\sin x)}$ 的定义域是 ( ).\n(A) $2 k \\pi-\\frac{\\pi}{2} \\leqslant x \\leqslant 2 k \\pi+\\frac{\\pi}{2}(k \\in \\mathbf{Z})$\n(B) $2 k \\pi \\leqslant x \\leqslant 2 k \\pi+\\frac{\\pi}{2}(k \\in \\mathbf{Z})$\n(C) $2 k \\pi-\\frac{\\pi}{2} \\leqslant x \\leqslant 2 k \\pi(k \\in$\n(D) $x \\in \\mathbf{R}$", + "solution": "D. 由原函数得 $\\cos (\\sin x) \\geqslant 0$, 因为 $|\\sin x| \\leqslant 1$, 即 $-1 \\leqslant \\sin x \\leqslant 1$, 所以 $\\cos (\\sin x) \\geqslant 0$ 恒成立.", + "remark": "", + "figures": [] +} \ No newline at end of file diff --git a/processed_dataset/calculation/0684.json b/processed_dataset/calculation/0684.json new file mode 100644 index 0000000000000000000000000000000000000000..fd60f5d102ab6cc181927982e0cc4c47320ce532 --- /dev/null +++ b/processed_dataset/calculation/0684.json @@ -0,0 +1,8 @@ +{ + "source_file": "./raw_volume-zh/volume3/exercise1.tex", + "problem_type": "calculation", + "problem": "问题7 存在 $x \\in[0,2 \\pi)$, 使 $\\sin x-\\sqrt{3} \\cos x=\\frac{4 m-6}{4-m}$ 成立, 则 $m$ 的取值范围是\n(A) $\\left[-1, \\frac{7}{3}\\right]$\n(B) $\\left(-\\infty, \\frac{7}{3}\\right]$\n(C) $[-1,+\\infty)$\n(D) $(-\\infty,-1) \\cup\\left(\\frac{7}{3},+\\infty\\right)$", + "solution": "A. 等式左边 $=2\\left(\\sin x \\cos \\frac{\\pi}{3}-\\cos x \\sin \\frac{\\pi}{3}\\right)=2 \\sin \\left(x-\\frac{\\pi}{3}\\right)$. 因原等式成立, 所以 $\\left|\\frac{4 m-6}{4-m}\\right|=\\left|2 \\sin \\left(x-\\frac{\\pi}{3}\\right)\\right| \\leqslant 2$, 即 $4 m^2-12 m+9 \\leqslant m^2- 8 m+16$, 整理得 $3 m^2-4 m-7 \\leqslant 0$, 解得 $-1 \\leqslant m \\leqslant \\frac{7}{3}$.", + "remark": "", + "figures": [] +} \ No newline at end of file diff --git a/processed_dataset/calculation/0685.json b/processed_dataset/calculation/0685.json new file mode 100644 index 0000000000000000000000000000000000000000..fee5538279045afd2360b7e28d904418aa796900 --- /dev/null +++ b/processed_dataset/calculation/0685.json @@ -0,0 +1,8 @@ +{ + "source_file": "./raw_volume-zh/volume3/exercise1.tex", + "problem_type": "calculation", + "problem": "问题8. $f(x)$ 是以 5 为周期的奇函数, $f(-1)=1$, 且 $\\tan \\alpha=3$, 则 $f(20 \\sin \\alpha \\cos \\alpha)$ 的值是 ( ).\n(A) 1\n(B) -1\n(C) 3\n(D) 8", + "solution": "B. 当 $\\tan \\alpha=3$ 时, $20 \\sin \\alpha \\cos \\alpha=10 \\sin 2 \\alpha=10 \\times \\frac{2 \\tan \\alpha}{1+\\tan ^2 \\alpha}=10 \\times \\frac{2 \\times 3}{1+3^2}=6$, 所以 $f(20 \\sin \\alpha \\cos \\alpha)=f(6)$, 由条件得 $f(6)=f(5+1)= f(1)=-f(-1)=-1$.", + "remark": "", + "figures": [] +} \ No newline at end of file diff --git a/processed_dataset/calculation/0686.json b/processed_dataset/calculation/0686.json new file mode 100644 index 0000000000000000000000000000000000000000..73f0bc526c302d485cfd50fe814e6fadfa9679b3 --- /dev/null +++ b/processed_dataset/calculation/0686.json @@ -0,0 +1,8 @@ +{ + "source_file": "./raw_volume-zh/volume3/exercise1.tex", + "problem_type": "calculation", + "problem": "问题9 函数 $f(x)=\\frac{1}{\\sec ^2 x}+\\sin x$ 在 $|x| \\leqslant \\frac{\\pi}{4}$ 上的最小值是 ( ).\n(A) $\\frac{\\sqrt{2}-1}{2}$\n(B) $-\\frac{1+\\sqrt{2}}{2}$\n(C) -1\n(D) $\\frac{1-\\sqrt{2}}{2}$", + "solution": "D. 因 $f(x)=\\frac{1}{\\sec ^2 x}+\\sin x=\\cos ^2 x+\\sin x=-\\sin ^2 x+\\sin x+1= -\\left(\\sin x-\\frac{1}{2}\\right)^2+\\frac{5}{4}$, 当 $|x| \\leqslant \\frac{\\pi}{4}$ 时, $f(x)_{\\text {max }}=f\\left(\\frac{\\pi}{6}\\right)=\\frac{5}{4}, f(x)_{min}=f\\left(-\\frac{\\pi}{4}\\right)=\\frac{1-\\sqrt{2}}{2}$", + "remark": "", + "figures": [] +} \ No newline at end of file diff --git a/processed_dataset/calculation/0687.json b/processed_dataset/calculation/0687.json new file mode 100644 index 0000000000000000000000000000000000000000..e467397ad3ac94917068b973ac862096cf57a879 --- /dev/null +++ b/processed_dataset/calculation/0687.json @@ -0,0 +1,8 @@ +{ + "source_file": "./raw_volume-zh/volume3/exercise1.tex", + "problem_type": "calculation", + "problem": "问题10 下列四个命题中, 正确的是 ( ).\n(A) 正切函数在整个定义域内是增函数\n(B) 周期函数一定有最小正周期\n(C) 函数 $y=3 \\tan \\sqrt{x^2}$ 的图象关于 $y$ 轴对称\n(D) 若 $x$ 是第一象限的角, 则 $\\sin x$ 是增函数, $\\cos x$ 是减函数", + "solution": "C. 正切函数在 $\\left(k \\pi-\\frac{\\pi}{2}, k \\pi+\\frac{\\pi}{2}\\right)(k \\in \\mathbf{Z})$ 上是增函数, 但不是在整个定义域上; 常数函数也是周期函数, 但无最小正周期; 若 $x$ 为第一象限角, 则在 $\\left(2 k \\pi, 2 k \\pi+\\frac{\\pi}{2}\\right)(k \\in \\mathbf{Z})$ 上 $\\sin x$ 是增函数,而 $\\cos x$ 是减函数,所以 $\\mathrm{A}$ 、 B、D均错误.\n又对 $f(x)=3 \\tan \\sqrt{x^2}$ 来说, $f(-x)=3 \\tan \\sqrt{(-x)^2}=f(x)$, 这是偶函数,所以其图象关于 $y$ 轴对称, 故选 C.", + "remark": "", + "figures": [] +} \ No newline at end of file diff --git a/processed_dataset/calculation/0688.json b/processed_dataset/calculation/0688.json new file mode 100644 index 0000000000000000000000000000000000000000..61e19a05713a95ae82a5c3f5401b44860a9df780 --- /dev/null +++ b/processed_dataset/calculation/0688.json @@ -0,0 +1,8 @@ +{ + "source_file": "./raw_volume-zh/volume3/exercise1.tex", + "problem_type": "calculation", + "problem": "问题11 将正弦曲线 $y=\\sin (-x)$ 的图象向右平移 $\\frac{\\pi}{3}$ 个单位, 所得到的函数图象的解析式是 ; 将余弦曲线 $y=\\cos (-2 x)$ 的图象向左平移 $\\frac{\\pi}{6}$ 个单位, 所得到的函数图象的解析式是", + "solution": "$y=\\sin \\left(\\frac{\\pi}{3}-x\\right), y=\\cos \\left(2 x+\\frac{\\pi}{3}\\right)$.", + "remark": "", + "figures": [] +} \ No newline at end of file diff --git a/processed_dataset/calculation/0689.json b/processed_dataset/calculation/0689.json new file mode 100644 index 0000000000000000000000000000000000000000..ad13ec36e2d55ded73647a99b6666ee405fb7bac --- /dev/null +++ b/processed_dataset/calculation/0689.json @@ -0,0 +1,8 @@ +{ + "source_file": "./raw_volume-zh/volume3/exercise1.tex", + "problem_type": "calculation", + "problem": "问题12 函数 $y=\\sqrt{\\cos x-2 \\cos ^2 x}$ 的定义域是 , 值域是", + "solution": "$\\left[2 k \\pi+\\frac{\\pi}{3}, 2 k \\pi+\\frac{\\pi}{2}\\right] \\cup\\left[2 k \\pi-\\frac{\\pi}{2}, 2 k \\pi-\\frac{\\pi}{3}\\right](k \\in \\mathbf{Z}),\\left[0, \\frac{\\sqrt{2}}{4}\\right]$.\n由条件得 $\\cos x-2 \\cos ^2 x \\geqslant 0$, 即 $0 \\leqslant \\cos x \\leqslant \\frac{1}{2}$, 所以定义域为 $\\left[2 k \\pi+\\frac{\\pi}{3}, 2 k \\pi+\\frac{\\pi}{2}\\right] \\cup\\left[2 k \\pi-\\frac{\\pi}{2}, 2 k \\pi-\\frac{\\pi}{3}\\right](k \\in \\mathbf{Z})$, 又 $y=\\sqrt{-2\\left(\\cos x-\\frac{1}{4}\\right)^2+\\frac{1}{8}}$, 故值域为 $\\left[0, \\frac{\\sqrt{2}}{4}\\right]$.", + "remark": "", + "figures": [] +} \ No newline at end of file diff --git a/processed_dataset/calculation/0690.json b/processed_dataset/calculation/0690.json new file mode 100644 index 0000000000000000000000000000000000000000..82c70225914e52b366893e4f50756f9ca3f8d834 --- /dev/null +++ b/processed_dataset/calculation/0690.json @@ -0,0 +1,8 @@ +{ + "source_file": "./raw_volume-zh/volume3/exercise1.tex", + "problem_type": "calculation", + "problem": "问题13 函数 $y=\\sin x\\left(1+\\tan \\frac{x}{2}\\right)$ 的最小正周期是", + "solution": "$2 \\pi$. 由原函数变换得 $y=\\sin x+\\sin x \\cdot \\frac{1-\\cos x}{\\sin x}=\\sin x+1- \\cos x=\\sqrt{2} \\sin \\left(x-\\frac{\\pi}{4}\\right)+1$, 故 $T=\\frac{2 \\pi}{\\omega}=2 \\pi$.", + "remark": "", + "figures": [] +} \ No newline at end of file diff --git a/processed_dataset/calculation/0691.json b/processed_dataset/calculation/0691.json new file mode 100644 index 0000000000000000000000000000000000000000..6d01dac1a3318d3d47f522a067cb6521f853b949 --- /dev/null +++ b/processed_dataset/calculation/0691.json @@ -0,0 +1,8 @@ +{ + "source_file": "./raw_volume-zh/volume3/exercise1.tex", + "problem_type": "calculation", + "problem": "问题14 已知 $f(x)=a \\sin ^3 x+b \\sqrt[3]{x} \\cdot \\cos ^3 x+4(a, b \\in \\mathbf{R})$, 且 $f\\left(\\sin 10^{\\circ}\\right)=5$, 则 $f\\left(\\cos 100^{\\circ}\\right)=$", + "solution": "3. 由题意得 $f(x)+f(-x)=8$, 而 $\\cos 100^{\\circ}=-\\sin 10^{\\circ}$, 所以 $f\\left(\\sin 10^{\\circ}\\right)+f\\left(-\\sin 10^{\\circ}\\right)=8$, 于是 $f\\left(\\cos 100^{\\circ}\\right)=3$.", + "remark": "", + "figures": [] +} \ No newline at end of file diff --git a/processed_dataset/calculation/0692.json b/processed_dataset/calculation/0692.json new file mode 100644 index 0000000000000000000000000000000000000000..fe6acf41ed2fb1963fbc67c83b7da6d8aca28747 --- /dev/null +++ b/processed_dataset/calculation/0692.json @@ -0,0 +1,8 @@ +{ + "source_file": "./raw_volume-zh/volume3/exercise1.tex", + "problem_type": "calculation", + "problem": "问题15 函数 $f(x)=\\sqrt{\\sin 2 x+\\sqrt{3} \\cos 2 x-1}$ 的定义域是", + "solution": "$\\left[k \\pi-\\frac{\\pi}{12}, k \\pi+\\frac{\\pi}{4}\\right](k \\in \\mathbf{Z})$. 由原函数得 $\\sin 2 x+\\sqrt{3} \\cos 2 x-1 \\geqslant 0$, 即 $2 \\sin \\left(2 x+\\frac{\\pi}{3}\\right) \\geqslant 1,2 k \\pi+\\frac{\\pi}{6} \\leqslant 2 x+\\frac{\\pi}{3} \\leqslant 2 k \\pi+\\frac{5 \\pi}{6}$, 解得 $k \\pi-\\frac{\\pi}{12} \\leqslant x \\leqslant k \\pi+\\frac{\\pi}{4}$, 所以定义域为 $\\left[k \\pi-\\frac{\\pi}{12}, k \\pi+\\frac{\\pi}{4}\\right](k \\in \\mathbf{Z})$.", + "remark": "", + "figures": [] +} \ No newline at end of file diff --git a/processed_dataset/calculation/0693.json b/processed_dataset/calculation/0693.json new file mode 100644 index 0000000000000000000000000000000000000000..e772c11d34517284cb29d18aeabf88bdec604759 --- /dev/null +++ b/processed_dataset/calculation/0693.json @@ -0,0 +1,8 @@ +{ + "source_file": "./raw_volume-zh/volume3/exercise1.tex", + "problem_type": "calculation", + "problem": "问题16 函数 $y=\\sin \\left(2 x+\\frac{\\pi}{3}\\right)$ 图象的对称轴是", + "solution": "$x=\\frac{k \\pi}{2}+\\frac{\\pi}{12}(k \\in \\mathbf{Z})$. 由 $2 x+\\frac{\\pi}{3}=k \\pi+\\frac{\\pi}{2}$, 得对称轴 $x=\\frac{k \\pi}{2}+\\frac{\\pi}{12}$.", + "remark": "", + "figures": [] +} \ No newline at end of file diff --git a/processed_dataset/calculation/0694.json b/processed_dataset/calculation/0694.json new file mode 100644 index 0000000000000000000000000000000000000000..fe21a0cefdef97e36496ca718fbd571794a08acf --- /dev/null +++ b/processed_dataset/calculation/0694.json @@ -0,0 +1,8 @@ +{ + "source_file": "./raw_volume-zh/volume3/exercise1.tex", + "problem_type": "calculation", + "problem": "问题17. 函数 $y=\\tan \\left(\\frac{1}{2} x+\\frac{\\pi}{6}\\right)$ 图象的对称中心是", + "solution": "$\\left(2 k \\pi-\\frac{\\pi}{3}, 0\\right)(k \\in \\mathbf{Z})$. 由 $\\frac{1}{2} x+\\frac{\\pi}{6}=k \\pi$, 得对称中心 $\\left(2 k \\pi-\\frac{\\pi}{3}, 0\\right)$.", + "remark": "", + "figures": [] +} \ No newline at end of file diff --git a/processed_dataset/calculation/0695.json b/processed_dataset/calculation/0695.json new file mode 100644 index 0000000000000000000000000000000000000000..1eb4de8bce631141fd4c1d285c2ea370076c8bb1 --- /dev/null +++ b/processed_dataset/calculation/0695.json @@ -0,0 +1,8 @@ +{ + "source_file": "./raw_volume-zh/volume3/exercise1.tex", + "problem_type": "calculation", + "problem": "问题18 设函数 $f(x)=\\sin (\\omega x+\\varphi)\\left(\\omega>0,|\\varphi|<\\frac{\\pi}{2}\\right)$, 给出四个判断:\n(1) 它的图象关于直线 $x=\\frac{\\pi}{12}$ 对称; (2) 它的图象关于点 $\\left(\\frac{\\pi}{3}, 0\\right)$ 对称;\n(3) 它的最小正周期是 $\\pi$; (4) 在区间 $\\left[-\\frac{\\pi}{6}, 0\\right]$ 上是增函数.\n以其中两个论断作为条件, 另两个论断作为结论, 你认为正确的两个命题是", + "solution": "(1)(3) $\\Rightarrow$ (2)(4) 及 (2)(3) $\\Rightarrow$ (1)(4). 若 (3) 成立, 则 $\\omega=2$, 得 $f(x)=\\sin (2 x+\\varphi)$, 又 (1) 成立, 则 $x=\\frac{\\pi}{12}$ 时 $f(x)$ 有最大值或最小值, 即 $\\sin \\left(\\frac{\\pi}{6}+\\varphi\\right)= \\pm 1$, 从而\n$\\varphi=\\frac{\\pi}{3}$, 所以 $f(x)=\\sin \\left(2 x+\\frac{\\pi}{3}\\right)$, 它关于 $\\left(\\frac{\\pi}{3}, 0\\right)$ 成中心对称, 且在 $\\left[-\\frac{\\pi}{6}, 0\\right]$ 上单调递增.\n若 (3) 成立, 则 $\\omega=2$, 得 $f(x)=\\sin (2 x+\\varphi)$, 又 (2) 成立, 则当 $x=\\frac{\\pi}{3}$ 时, $f(x)=0$, 即 $\\sin \\left(\\frac{2 \\pi}{3}+\\varphi\\right)=0$. 从而 $\\varphi=\\frac{\\pi}{3}$, 所以 $f(x)=\\sin \\left(2 x+\\frac{\\pi}{3}\\right)$, 得 (1)(4) 成立.", + "remark": "", + "figures": [] +} \ No newline at end of file diff --git a/processed_dataset/calculation/0696.json b/processed_dataset/calculation/0696.json new file mode 100644 index 0000000000000000000000000000000000000000..c1907b2257c9b8ec13618166c2f5fee223c77f4b --- /dev/null +++ b/processed_dataset/calculation/0696.json @@ -0,0 +1,8 @@ +{ + "source_file": "./raw_volume-zh/volume3/exercise1.tex", + "problem_type": "calculation", + "problem": "问题20. 方程 $\\sin x=\\frac{x}{100}$ 的实根个数有个.", + "solution": "63. 由 $y=\\sin x$ 和 $y=\\frac{x}{100}$ 都是奇函数知, 它的正负根个数相同,在 $x>0$ 时, 由 $\\frac{100}{\\pi}=31.8$, 知在 $(0,100]$ 上两函数的图象有 31 个交点, 同理在 $[-100,0)$ 上两函数的图象有 31 个交点, 又 $x=0$ 是方程的根,所以共有 63 个实数根.", + "remark": "", + "figures": [] +} \ No newline at end of file diff --git a/processed_dataset/calculation/0697.json b/processed_dataset/calculation/0697.json new file mode 100644 index 0000000000000000000000000000000000000000..9a8b2f9c4716c9199d50a8f41058213fc4428125 --- /dev/null +++ b/processed_dataset/calculation/0697.json @@ -0,0 +1,8 @@ +{ + "source_file": "./raw_volume-zh/volume3/exercise1.tex", + "problem_type": "calculation", + "problem": "问题21 作出函数 $y=\\sqrt{3} \\sin 2 x-\\cos 2 x-1$ 在一个周期上的图象,并指出它与 $y=\\sin x$ 的图象间的关系.", + "solution": "由 $y=2 \\sin \\left(2 x-\\frac{\\pi}{6}\\right)-1$ 知 $T=\\pi$, 列表由五点描图法作出其图象, 该图象可看作将函数 $y=\\sin x$ 的图象横坐标压缩为原来的一半得 $y=\\sin 2 x$, 再向右平移 $\\frac{\\pi}{12}$ 个单位长度, 得 $y=\\sin \\left(2 x-\\frac{\\pi}{6}\\right)$, 再将纵坐标扩大为原来的 2 倍, 得 $y=2 \\sin \\left(2 x-\\frac{\\pi}{6}\\right)$, 最后, 将整个图象向下平移 1 个单位, 即得函数 $y= 2 \\sin \\left(2 x-\\frac{\\pi}{6}\\right)-1$ 的图象.\n(图略)", + "remark": "", + "figures": [] +} \ No newline at end of file diff --git a/processed_dataset/calculation/0698.json b/processed_dataset/calculation/0698.json new file mode 100644 index 0000000000000000000000000000000000000000..4d9a3cc8888138b3c58273b171332c1526f0c89b --- /dev/null +++ b/processed_dataset/calculation/0698.json @@ -0,0 +1,8 @@ +{ + "source_file": "./raw_volume-zh/volume3/exercise1.tex", + "problem_type": "calculation", + "problem": "问题22 已知 $f(x)=2 \\sin \\left(x+\\frac{\\theta}{2}\\right) \\cos \\left(x+\\frac{\\theta}{2}\\right)+2 \\sqrt{3} \\cos ^2\\left(x+\\frac{\\theta}{2}\\right)-\\sqrt{3}$.\n(1) 化简 $f(x)$ 的解析式;\n(2) 若 $0 \\leqslant \\theta \\leqslant \\pi$, 求 $\\theta$ 值,使函数 $f(x)$ 为偶函数;\n(3) 在 (2) 的条件下,求满足 $f(x)=1$ 在 $[-\\pi, \\pi]$ 上的 $x$ 集合.", + "solution": "(1) $f(x)=\\sin (2 x+\\theta)+\\sqrt{3} \\cos (2 x+\\theta)=2 \\cos \\left(2 x+\\theta-\\frac{\\pi}{6}\\right)$.\n(2) 要使 $f(x)$ 为偶函数, 只要满足 $\\theta-\\frac{\\pi}{6}=k \\pi$, 又 $0 \\leqslant \\theta \\leqslant \\pi$, 所以 $\\theta=\\frac{\\pi}{6}$.\n(3) 由 $f(x)=2 \\cos 2 x=1$, 得 $2 x=2 k \\pi \\pm \\frac{\\pi}{3}$, 因为 $x \\in[-\\pi, \\pi]$, 所以解集为 $\\left\\{x \\mid x= \\pm \\frac{5 \\pi}{6}, \\pm \\frac{\\pi}{6}\\right\\}$.", + "remark": "", + "figures": [] +} \ No newline at end of file diff --git a/processed_dataset/calculation/0699.json b/processed_dataset/calculation/0699.json new file mode 100644 index 0000000000000000000000000000000000000000..b0c3cebe69fc7acff8f028297bde4b49c4f51d69 --- /dev/null +++ b/processed_dataset/calculation/0699.json @@ -0,0 +1,8 @@ +{ + "source_file": "./raw_volume-zh/volume3/exercise1.tex", + "problem_type": "calculation", + "problem": "问题23 已知当 $x \\in[0,1]$ 时,不等式 $x^2 \\cos \\theta-x(1-x)+(1-x)^2 \\sin \\theta>0$ 恒成立, 试求 $\\theta$ 的取值范围.", + "solution": "若对一切 $x \\in[0,1]$, 恒有 $f(x)=x^2 \\cos \\theta-x(1-x)+(1-x)^2 \\sin \\theta>0$, 则 $\\cos \\theta=f(1)>0, \\sin \\theta=f(0)>0 \\cdots$ (1). 取 $x_0=\\frac{\\sqrt{\\sin \\theta}}{\\sqrt{\\cos \\theta}+\\sqrt{\\sin \\theta}} \\in(0,1)$, 则 $\\sqrt{\\cos \\theta} x_0-\\sqrt{\\sin \\theta}\\left(1-x_0\\right)=0$. 由于 $f(x)=[\\sqrt{\\cos \\theta} x-\\sqrt{\\sin \\theta}(1- x)]^2+2\\left(-\\frac{1}{2}+\\sqrt{\\cos \\theta \\sin \\theta}\\right) x(1-x)$. 所以, $00 \\cdots$ (2). 反之, 当(1), (2)成立时, $f(0)=\\sin \\theta>0, f(1)=\\cos \\theta>0$, 且 $x \\in(0,1)$ 时, $f(x) \\geqslant 2\\left(-\\frac{1}{2}+\\right. \\sqrt{\\cos \\theta \\sin \\theta}) x(1-x)>0$. 先在 $[0,2 \\pi]$ 中解(1)与(2), 由 $\\cos \\theta>0, \\sin \\theta>0$, 可得 $0<\\theta<\\frac{\\pi}{2}$. 又因为 $-\\frac{1}{2}+\\sqrt{\\cos \\theta \\sin \\theta}>0, \\sqrt{\\cos \\theta \\sin \\theta}>\\frac{1}{2}, \\sin 2 \\theta> \\frac{1}{2}$, 注意到 $0<2 \\theta<\\pi$, 故有 $\\frac{\\pi}{6}<2 \\theta<\\frac{5 \\pi}{6}$. 所以, $\\frac{\\pi}{12}<\\theta<\\frac{5 \\pi}{12}$. 因此, 原题中 $\\theta$ 的取值范围是 $2 k \\pi+\\frac{\\pi}{12}<\\theta<2 k \\pi+\\frac{5 \\pi}{12}, k \\in \\mathbf{Z}$.", + "remark": "", + "figures": [] +} \ No newline at end of file diff --git a/processed_dataset/calculation/0700.json b/processed_dataset/calculation/0700.json new file mode 100644 index 0000000000000000000000000000000000000000..1738bdf41a13f98378a4110256711d82eff4f7ef --- /dev/null +++ b/processed_dataset/calculation/0700.json @@ -0,0 +1,8 @@ +{ + "source_file": "./raw_volume-zh/volume3/exercise1.tex", + "problem_type": "calculation", + "problem": "问题24. 已知函数 $f(x)=\\sin (\\omega x+\\varphi)(\\omega>0,0 \\leqslant \\varphi \\leqslant \\pi)$ 为 $\\mathbf{R}$ 上的偶函数, 其图象关于点 $M\\left(\\frac{3 \\pi}{4}, 0\\right)$ 对称, 且在区间 $\\left[0, \\frac{\\pi}{2}\\right]$ 上是单调函数, 求 $\\varphi$ 和 $\\omega$ 的值.", + "solution": "由 $f(x)$ 是偶函数, 得 $f(-x)=f(x)$, 即 $\\sin (-\\omega x+\\varphi)=\\sin (\\omega x+ \\varphi$ ), 所以 $-\\cos \\varphi \\sin \\omega x=\\cos \\varphi \\sin \\omega x$ 对任意 $x \\in \\mathbf{R}$ 都成立, 且 $\\omega>0$, 所以 $\\cos \\varphi=0$. 依题设 $0 \\leqslant \\varphi \\leqslant \\pi$, 所以解得 $\\varphi=\\frac{\\pi}{2}$. 由 $f(x)$ 的图象关于点 $M$ 对称, 得 $f\\left(\\frac{3 \\pi}{4}-x\\right)=f\\left(\\frac{3 \\pi}{4}+x\\right)$, 取 $x=0$, 得 $f\\left(\\frac{3 \\pi}{4}\\right)=\\sin \\left(\\frac{3 \\omega \\pi}{4}+\\frac{\\pi}{2}\\right)= \\cos \\frac{3 \\omega \\pi}{4}$, 所以 $f\\left(\\frac{3 \\pi}{4}\\right)=\\sin \\left(\\frac{3 \\omega \\pi}{4}+\\frac{\\pi}{2}\\right)=\\cos \\frac{3 \\omega \\pi}{4}$. 所以 $\\cos \\frac{3 \\omega \\pi}{4}=0$, 又 $\\omega=0$, 得 $\\frac{3 \\omega \\pi}{4}=\\frac{\\pi}{2}+k \\pi, k=1,2,3, \\cdots$. 所以 $\\omega=\\frac{2}{3}(2 k+1), k=0,1,2, \\cdots$. 当 $k=0$ 时, $\\omega=\\frac{2}{3}, f(x)=\\sin \\left(\\frac{2}{3} x+\\frac{\\pi}{2}\\right)$ 在 $\\left[0, \\frac{\\pi}{2}\\right]$ 上是减函数; 当 $k=1$ 时, $\\omega=2, f(x)=\\sin \\left(2 x+\\frac{\\pi}{2}\\right)$ 在 $\\left[0, \\frac{\\pi}{2}\\right]$ 上是减函数; 当 $k \\geqslant 2$ 时, $\\omega \\geqslant \\frac{10}{3}$, $f(x)=\\sin \\left(\\omega x+\\frac{\\pi}{2}\\right)$ 在 $\\left[0, \\frac{\\pi}{2}\\right]$ 上不是单调函数; 所以,综上得 $\\omega=\\frac{2}{3}$ 或 $\\omega=2$.", + "remark": "", + "figures": [] +} \ No newline at end of file diff --git a/processed_dataset/calculation/0701.json b/processed_dataset/calculation/0701.json new file mode 100644 index 0000000000000000000000000000000000000000..34cf2cd3049247a96132118b81ac9855b5fc18cb --- /dev/null +++ b/processed_dataset/calculation/0701.json @@ -0,0 +1,8 @@ +{ + "source_file": "./raw_volume-zh/volume3/exercise1.tex", + "problem_type": "calculation", + "problem": "问题26. (1) 求函数 $f(\\theta)=\\cos \\frac{\\theta}{2} \\sin \\theta, \\theta \\in\\left(0, \\frac{\\pi}{2}\\right)$ 的最大值;\n(2) 求函数 $g(\\theta)=\\sin \\frac{\\theta}{2} \\cos \\theta, \\theta \\in\\left(0, \\frac{\\pi}{2}\\right)$ 的最大值.", + "solution": "(1) $f(\\theta)=2 \\sin \\frac{\\theta}{2} \\cdot \\cos ^2 \\frac{\\theta}{2}=2 \\sqrt{\\sin ^2 \\frac{\\theta}{2} \\cdot \\cos ^4 \\frac{\\theta}{2}}=\\sqrt{2}\\sqrt{2 \\sin ^2 \\frac{\\theta}{2} \\cdot \\cos ^2 \\frac{\\theta}{2} \\cdot \\cos ^2 \\frac{\\theta}{2}} \\leqslant \\sqrt{2} \\cdot \\sqrt{\\left[\\frac{2 \\sin ^2 \\frac{\\theta}{2}+\\cos ^2 \\frac{\\theta}{2}+\\cos ^2 \\frac{\\theta}{2}}{3}\\right]^3}=\\sqrt{2} \\cdot \\left(\\frac{2}{3}\\right)^{\\frac{3}{2}}=\\frac{4 \\sqrt{3}}{9}$, 当且仅当 $\\theta=2 \\arctan \\frac{\\sqrt{2}}{2}$ 时取等号.\n所以, $f(\\theta)$ 的最大值为 $\\frac{4 \\sqrt{3}}{9}$.\n(2) 由于 $\\theta \\in\\left(0, \\frac{\\pi}{2}\\right)$, 故 $g(\\theta)>0, g^2(\\theta)=\\sin ^2 \\frac{\\theta}{2} \\cdot \\cos ^2 \\theta=\\frac{1-\\cos \\theta}{2}$. $\\cos ^2 \\theta=\\frac{2(1-\\cos \\theta) \\cdot \\cos \\theta \\cdot \\cos \\theta}{4} \\leqslant \\frac{1}{4} \\cdot\\left[\\frac{2(1-\\cos \\theta)+\\cos \\theta+\\cos \\theta}{3}\\right]^3= \\frac{2}{27}$, 当且仅当 $\\theta=\\arccos \\frac{2}{3}$ 时取等号.\n所以, $g(\\theta)$ 的最大值为 $\\frac{\\sqrt{6}}{9}$. 当然也可以如下这样, $g(\\theta)=\\sin \\frac{\\theta}{2} \\cdot\\left(1-2 \\sin ^2 \\frac{\\theta}{2}\\right)=\\frac{1}{2} \\sqrt{4 \\sin ^2 \\frac{\\theta}{2} \\cdot\\left(1-2 \\sin ^2 \\frac{\\theta}{2}\\right) \\cdot\\left(1-2 \\sin ^2 \\frac{\\theta}{2}\\right)} \\leqslant \\frac{1}{2} \\cdot \\sqrt{\\left[\\frac{4 \\sin ^2 \\frac{\\theta}{2}+\\left(1-2 \\sin ^2 \\frac{\\theta}{2}\\right)+\\left(1-2 \\sin ^2 \\frac{\\theta}{2}\\right)}{3}\\right]^3}=\\frac{\\sqrt{6}}{9}$. 当且仅当 $\\theta=2 \\arcsin \\frac{\\sqrt{6}}{6}$ 时取等号.\n故 $g(\\theta)$ 的最大值为 $\\frac{\\sqrt{6}}{9}$.", + "remark": "", + "figures": [] +} \ No newline at end of file diff --git a/processed_dataset/calculation/0702.json b/processed_dataset/calculation/0702.json new file mode 100644 index 0000000000000000000000000000000000000000..1c1deeb53226312691f46ca2de8660b2b0217c0b --- /dev/null +++ b/processed_dataset/calculation/0702.json @@ -0,0 +1,8 @@ +{ + "source_file": "./raw_volume-zh/volume3/exercise1.tex", + "problem_type": "calculation", + "problem": "问题27 教室的墙壁上挂着一块黑板, 它的上下边缘分别在学生的水平视线上方 $a$ 米和 $b$ 米, 问学生距墙壁多远时看黑板的视角最大?", + "solution": "设学生 $P$ 距墙壁 $x$ 米,黑板上、下边缘与学生 $P$ 的水平视线 $P H$ 夹角分别为 $\\angle A P H=\\alpha, \\angle B P H=\\beta$, 其中 $\\alpha>\\beta$, 有 $\\tan \\alpha=\\frac{a}{x}, \\tan \\beta=\\frac{b}{x}$, 则 $\\tan (\\alpha-\\beta)=\\frac{\\tan \\alpha-\\tan \\beta}{1+\\tan \\alpha \\tan \\beta}=\\frac{\\frac{a}{x}-\\frac{b}{x}}{1+\\frac{a b}{x^2}} \\leqslant \\frac{a-b}{2 \\sqrt{x \\cdot \\frac{a b}{x}}}=\\frac{a-b}{2 \\sqrt{a b}}$, 当且仅当 $x= \\frac{a b}{x}$ 即 $x=\\sqrt{a b}$ 时, $\\tan (\\alpha-\\beta)$ 取最大值 $\\frac{a-b}{2 \\sqrt{a b}}$, 由于 $\\alpha-\\beta$ 为锐角, 故此时 $\\alpha-\\beta$ 也最大, 即学生距墙壁 $\\sqrt{a b}$ 米时看黑板的视角最大, 最大视角为 $\\arctan \\frac{a-b}{2 \\sqrt{a b}}$.", + "remark": "", + "figures": [] +} \ No newline at end of file diff --git a/processed_dataset/calculation/0703.json b/processed_dataset/calculation/0703.json new file mode 100644 index 0000000000000000000000000000000000000000..d0c6c8eb2259c9ef1aba0fb001f22c1630c13bad --- /dev/null +++ b/processed_dataset/calculation/0703.json @@ -0,0 +1,8 @@ +{ + "source_file": "./raw_volume-zh/volume3/exercise1.tex", + "problem_type": "calculation", + "problem": "问题29 试求正整数 $k$, 使 $f(x)=\\sin k x \\cdot \\sin ^k x+\\cos k x \\cdot \\cos ^k x-\\cos ^k 2 x$ 的值不依赖于 $x$.", + "solution": "若 $f(x)$ 的值不依赖于 $x$, 即 $f(x)$ 为常数.\n由 $f(0)=0$, 得 $f(x)=0$, 取 $x=\\frac{\\pi}{k}$, 得 $f\\left(\\frac{\\pi}{k}\\right)=-\\cos ^k \\frac{\\pi}{k}-\\cos ^k \\frac{2 \\pi}{k}=0$, 若 $k$ 为正偶数, 则 $\\cos ^k \\frac{\\pi}{k} \\geqslant 0$, $\\cos ^k \\frac{2 \\pi}{k} \\geqslant 0$, 从而 $\\cos \\frac{\\pi}{k}=\\cos \\frac{2 \\pi}{k}=0$, 此时无解; 若 $k$ 为正奇数, 则由 $\\cos ^k \\frac{\\pi}{k}= -\\cos ^k \\frac{2 \\pi}{k}$, 得 $\\cos \\frac{\\pi}{k}=-\\cos \\frac{2 \\pi}{k}$, 所以 $\\frac{2 \\pi}{k}=2 n \\pi \\pm\\left(\\pi-\\frac{\\pi}{k}\\right)$,即 $\\frac{3}{k}=2 n \\neq 1$ 或 $\\frac{1}{k}=2 n-1$,故 $k=\\frac{3}{2 n+1}$ 或 $\\frac{1}{2 n-1}$, 从而 $k=1$ 或 3 , 但 $k=1$ 时, $f(x)$ 不为常数, $k=3$ 时, $f(x)$ 为常数, 所以 $k=3$.", + "remark": "", + "figures": [] +} \ No newline at end of file diff --git a/processed_dataset/calculation/0704.json b/processed_dataset/calculation/0704.json new file mode 100644 index 0000000000000000000000000000000000000000..a6b965349e312ad422c79f8af2d75f33d7c8b6e3 --- /dev/null +++ b/processed_dataset/calculation/0704.json @@ -0,0 +1,8 @@ +{ + "source_file": "./raw_volume-zh/volume3/exercise2.tex", + "problem_type": "calculation", + "problem": "问题1 已知 $\\frac{1+\\sin \\theta+\\cos \\theta}{1+\\sin \\theta-\\cos \\theta}=\\frac{1}{2}$, 则 $\\cos \\theta$ 的值等于 ( ).\n(A) $\\frac{3}{5}$\n(B) $-\\frac{3}{5}$\n(C) $-\\frac{\\sqrt{5}}{5}$\n(D) $\\frac{4}{5}$", + "solution": "B. 因 $\\frac{1+\\sin \\theta+\\cos \\theta}{1+\\sin \\theta-\\cos \\theta}=\\frac{1}{2}$, 由合分比性质得 $\\frac{2 \\cos \\theta}{2+2 \\sin \\theta}=\\frac{-1}{3}$, 故 $\\cos \\theta=\\frac{1-\\tan ^2 \\frac{\\theta}{2}}{1+\\tan ^2 \\frac{\\theta}{2}}=\\frac{1-2^2}{1+2^2}=-\\frac{3}{5}$.", + "remark": "", + "figures": [] +} \ No newline at end of file diff --git a/processed_dataset/calculation/0705.json b/processed_dataset/calculation/0705.json new file mode 100644 index 0000000000000000000000000000000000000000..3d0de92e30987c06514f8df0ddf263ea1d587992 --- /dev/null +++ b/processed_dataset/calculation/0705.json @@ -0,0 +1,8 @@ +{ + "source_file": "./raw_volume-zh/volume3/exercise2.tex", + "problem_type": "calculation", + "problem": "问题2. 化简: $2 \\sqrt{1+\\sin 8}+\\sqrt{2+2 \\cos 8}$ 的结果( ).\n(A) $2 \\sin 4$\n(B) $2 \\sin 4+4 \\cos 4$\n(C) $-2 \\sin 4-4 \\cos 4$\n(D) $2 \\sin 4-4 \\cos 4$", + "solution": "C. $2 \\sqrt{1+\\sin 8}+\\sqrt{2+2 \\cos 8}=2 \\sqrt{\\sin ^2 4+\\cos ^2 4+2 \\sin 4 \\cos 4}+ \\sqrt{2+2\\left(2 \\cos ^2 4-1\\right)}=2|\\sin 4+\\cos 4|+|2 \\cos 4|=-2 \\sin 4-2 \\cos 4- 2 \\cos 4=-2 \\sin 4-4 \\cos 4$.", + "remark": "", + "figures": [] +} \ No newline at end of file diff --git a/processed_dataset/calculation/0706.json b/processed_dataset/calculation/0706.json new file mode 100644 index 0000000000000000000000000000000000000000..a2d84d5a8b25cbbadb8033b8edfc74037e10530c --- /dev/null +++ b/processed_dataset/calculation/0706.json @@ -0,0 +1,8 @@ +{ + "source_file": "./raw_volume-zh/volume3/exercise2.tex", + "problem_type": "calculation", + "problem": "问题3 计算 $\\sin 10^{\\circ} \\sin 30^{\\circ} \\sin 50^{\\circ} \\sin 70^{\\circ}$ 的值是 ( ).\n(A) $\\frac{1}{2}$\n(B) $\\frac{1}{4}$\n(C) $\\frac{1}{8}$\n(D) $\\frac{1}{16}$", + "solution": "D. 参照例 4 可得原式 $=\\frac{1}{16}$.", + "remark": "", + "figures": [] +} \ No newline at end of file diff --git a/processed_dataset/calculation/0707.json b/processed_dataset/calculation/0707.json new file mode 100644 index 0000000000000000000000000000000000000000..f7874a123bb1dbb700393cc97509a5b1b21f2f65 --- /dev/null +++ b/processed_dataset/calculation/0707.json @@ -0,0 +1,8 @@ +{ + "source_file": "./raw_volume-zh/volume3/exercise2.tex", + "problem_type": "calculation", + "problem": "问题4 化简 $\\frac{2 \\cos ^2 \\alpha-1}{2 \\operatorname{sen}\\left(\\frac{\\pi}{4}-\\alpha\\right) \\sin ^2\\left(\\frac{\\pi}{4}+\\alpha\\right)}$ 等于( ).\n(A) 1\n(B) -1\n(C) $\\cos \\alpha$\n(D) $-\\sin \\alpha$", + "solution": "$\\mathrm{A}$ 原式 $=\\frac{\\cos 2 \\alpha}{2 \\cdot \\frac{1-\\tan \\alpha}{1+\\tan \\alpha} \\cdot \\frac{1-\\cos \\left(\\frac{\\pi}{2}+2 \\alpha\\right)}{2}}=\\frac{\\cos ^2 \\alpha-\\sin ^2 \\alpha}{\\frac{\\cos \\alpha-\\sin \\alpha}{\\cos \\alpha+\\sin \\alpha} \\cdot(1+\\sin 2 \\alpha)}= \\frac{(\\cos \\alpha+\\sin \\alpha)^2}{1+\\sin 2 \\alpha}=1$.", + "remark": "", + "figures": [] +} \ No newline at end of file diff --git a/processed_dataset/calculation/0708.json b/processed_dataset/calculation/0708.json new file mode 100644 index 0000000000000000000000000000000000000000..448283e85030c4dc3e4c9c2dd760581fd40f9f8a --- /dev/null +++ b/processed_dataset/calculation/0708.json @@ -0,0 +1,8 @@ +{ + "source_file": "./raw_volume-zh/volume3/exercise2.tex", + "problem_type": "calculation", + "problem": "问题5 设 $a=\\sin 14^{\\circ}+\\cos 14^{\\circ}, b=\\sin 16^{\\circ}+\\cos 16^{\\circ}, c=\\frac{\\sqrt{6}}{2}$, 则 $a 、 b 、 c$ 的大小关系是 ( ).\n(A) $a0$, 所以 $-\\frac{3}{5} \\sqrt{1-x^2}+\\frac{4}{5} x>0$. 即 $4 x>3 \\sqrt{1-x^2}$, 平方得 $25 x^2>9$, 所以 $x>\\frac{3}{5}$. 又 $x<1$, 故 $\\frac{3}{5}-1$, 从而 $\\cot \\alpha=-\\frac{1}{2}$, 所以 $\\tan \\alpha=-2$, $\\cos 2 \\alpha=\\frac{1-\\tan ^2 \\alpha}{1+\\tan ^2 \\alpha}=-\\frac{3}{5}$.", + "remark": "", + "figures": [] +} \ No newline at end of file diff --git a/processed_dataset/calculation/0719.json b/processed_dataset/calculation/0719.json new file mode 100644 index 0000000000000000000000000000000000000000..68b29de893745c46a0fb4871dfd7990fecbd1ee2 --- /dev/null +++ b/processed_dataset/calculation/0719.json @@ -0,0 +1,8 @@ +{ + "source_file": "./raw_volume-zh/volume3/exercise2.tex", + "problem_type": "calculation", + "problem": "问题16 已知 $\\sin \\frac{\\theta}{2}=\\sqrt{\\frac{x-1}{2 x}}$, 并且 $0<\\theta<\\frac{\\pi}{2}$, 则 $\\tan \\theta=?$, $ \\sin 2 \\theta=?$ , $\\cos 2 \\theta=?$", + "solution": "$\\sqrt{x^2-1}, \\frac{2 \\sqrt{x^2-1}}{x^2}, \\frac{2-x^2}{x^2}$.", + "remark": "", + "figures": [] +} \ No newline at end of file diff --git a/processed_dataset/calculation/0720.json b/processed_dataset/calculation/0720.json new file mode 100644 index 0000000000000000000000000000000000000000..96ec51dcd881231091025cc3b7f79c736e3c6c82 --- /dev/null +++ b/processed_dataset/calculation/0720.json @@ -0,0 +1,8 @@ +{ + "source_file": "./raw_volume-zh/volume3/exercise2.tex", + "problem_type": "calculation", + "problem": "问题17. 计算 $\\sin 6^{\\circ} \\sin 42^{\\circ} \\sin 66^{\\circ} \\sin 78^{\\circ}=$", + "solution": "$\\frac{1}{16}$. 参照例 4 可得原式 $=\\frac{1}{16}$.", + "remark": "", + "figures": [] +} \ No newline at end of file diff --git a/processed_dataset/calculation/0721.json b/processed_dataset/calculation/0721.json new file mode 100644 index 0000000000000000000000000000000000000000..bc1add7af9c7d2a90f9d2de463c0e62c5f3220f2 --- /dev/null +++ b/processed_dataset/calculation/0721.json @@ -0,0 +1,8 @@ +{ + "source_file": "./raw_volume-zh/volume3/exercise2.tex", + "problem_type": "calculation", + "problem": "问题18 计算 $\\tan 5^{\\circ}+\\cot 5^{\\circ}-\\frac{2}{\\cos 80^{\\circ}}=$", + "solution": "0 . 原式 $=\\frac{\\sin 5^{\\circ}}{\\cos 5^{\\circ}}+\\frac{\\cos 5^{\\circ}}{\\sin 5^{\\circ}}-\\frac{2}{\\cos 80^{\\circ}}=\\frac{\\sin ^2 5^{\\circ}+\\cos ^2 5^{\\circ}}{\\sin 5^{\\circ} \\cos 5^{\\circ}}-\\frac{2}{\\cos 80^{\\circ}}= \\frac{2}{\\sin 10^{\\circ}}-\\frac{2}{\\sin 10^{\\circ}}=0$.", + "remark": "", + "figures": [] +} \ No newline at end of file diff --git a/processed_dataset/calculation/0722.json b/processed_dataset/calculation/0722.json new file mode 100644 index 0000000000000000000000000000000000000000..048990adadb7e4e4d5e867cefac2a54f99dec454 --- /dev/null +++ b/processed_dataset/calculation/0722.json @@ -0,0 +1,8 @@ +{ + "source_file": "./raw_volume-zh/volume3/exercise2.tex", + "problem_type": "calculation", + "problem": "问题19 已知 $\\theta \\in\\left(\\frac{3 \\pi}{2}, 2 \\pi\\right)$, 则 $\\sqrt{1+\\sin \\theta}-\\sqrt{1-\\sin \\theta}=$", + "solution": "$-2 \\sin \\frac{\\theta}{2}$. 原式 $=\\left|\\sin \\frac{\\theta}{2}+\\cos \\frac{\\theta}{2}\\right|-\\left|\\sin \\frac{\\theta}{2}-\\cos \\frac{\\theta}{2}\\right|=-\\cos \\frac{\\theta}{2}- \\sin \\frac{\\theta}{2}-\\sin \\frac{\\theta}{2}+\\cos \\frac{\\theta}{2}=-2 \\sin \\frac{\\theta}{2}$.", + "remark": "", + "figures": [] +} \ No newline at end of file diff --git a/processed_dataset/calculation/0723.json b/processed_dataset/calculation/0723.json new file mode 100644 index 0000000000000000000000000000000000000000..6f5b0e2412731bdf78605a0ae2cdee24b109c674 --- /dev/null +++ b/processed_dataset/calculation/0723.json @@ -0,0 +1,8 @@ +{ + "source_file": "./raw_volume-zh/volume3/exercise2.tex", + "problem_type": "calculation", + "problem": "问题20 已知 $\\sin \\alpha+\\sin \\beta=\\frac{1}{2}, \\cos \\alpha+\\cos \\beta=\\frac{2}{3}$, 则 $\\cos (\\alpha-\\beta)=$", + "solution": "$-\\frac{47}{72}$. 两式平方和得 $2+2 \\cos (\\alpha-\\beta)=\\frac{25}{36}$, 所以原式 $=-\\frac{47}{72}$.", + "remark": "", + "figures": [] +} \ No newline at end of file diff --git a/processed_dataset/calculation/0724.json b/processed_dataset/calculation/0724.json new file mode 100644 index 0000000000000000000000000000000000000000..0bd602bb6ca5b486dabf9361cb44b966a5199b38 --- /dev/null +++ b/processed_dataset/calculation/0724.json @@ -0,0 +1,8 @@ +{ + "source_file": "./raw_volume-zh/volume3/exercise2.tex", + "problem_type": "calculation", + "problem": "问题21. 求下列各式的值:\n(1) $\\cos ^2 24^{\\circ}+\\sin ^2 6^{\\circ}+\\cos ^2 18^{\\circ}$;\n(2) $4 \\cos ^2 36^{\\circ}-\\sin 84^{\\circ}\\left(\\sqrt{3}-\\tan 6^{\\circ}\\right)$.", + "solution": "(1) $\\cos ^2 24^{\\circ}+\\sin ^2 6^{\\circ}+\\cos ^2 18^{\\circ}=\\frac{3}{2}+\\frac{1}{2}\\left(\\cos 48^{\\circ}-\\cos 12^{\\circ}+\\right. \\left.\\cos 36^{\\circ}\\right)=\\frac{3}{2}+\\frac{1}{2}\\left(-2 \\sin 30^{\\circ} \\sin 18^{\\circ}+\\cos 36^{\\circ}\\right)=\\frac{3}{2}+\\frac{1}{2}\\left(\\sin 54^{\\circ}-\\sin 18^{\\circ}\\right)= \\frac{3}{2}+\\cos 36^{\\circ} \\sin 18^{\\circ}=\\frac{3}{2}+\\frac{\\cos 36^{\\circ} \\sin 18^{\\circ} \\cos 18^{\\circ}}{\\cos 18^{\\circ}}=\\frac{3}{2}+\\frac{\\frac{1}{4} \\sin 72^{\\circ}}{\\cos 18^{\\circ}}=\\frac{7}{4}$.\n (2)参照例 3 的解法, 可得 $4 \\cos ^2 36^{\\circ}-\\sin 84^{\\circ}\\left(\\sqrt{3}-\\tan 6^{\\circ}\\right)=1$.", + "remark": "", + "figures": [] +} \ No newline at end of file diff --git a/processed_dataset/calculation/0725.json b/processed_dataset/calculation/0725.json new file mode 100644 index 0000000000000000000000000000000000000000..86bcf8fd5f64a99f34f6108a62568abf700d351d --- /dev/null +++ b/processed_dataset/calculation/0725.json @@ -0,0 +1,8 @@ +{ + "source_file": "./raw_volume-zh/volume3/exercise2.tex", + "problem_type": "calculation", + "problem": "问题22. 设 $\\cos \\left(\\alpha-\\frac{\\beta}{2}\\right)=-\\frac{1}{9}, \\sin \\left(\\frac{\\alpha}{2}-\\beta\\right)=\\frac{2}{3}$, 且 $\\frac{\\pi}{2}<\\alpha<\\pi, 0<\\beta<\\frac{\\pi}{2}$, 求 $\\sin \\frac{\\alpha+\\beta}{2}$ 和 $\\cos (\\alpha+\\beta)$ 的值.", + "solution": "由 $\\frac{\\pi}{2}<\\alpha<\\pi, 0<\\beta<\\frac{\\pi}{2}$, 得 $\\frac{\\pi}{4}<\\alpha-\\frac{\\beta}{2}<\\pi,-\\frac{\\pi}{4}<\\frac{\\alpha}{2}-\\beta<\\frac{\\pi}{2}$,\n故由 $\\cos \\left(\\alpha-\\frac{\\beta}{2}\\right)=-\\frac{1}{9}$, 得 $\\sin \\left(\\alpha-\\frac{\\beta}{2}\\right)=\\frac{4 \\sqrt{5}}{9}$, 由 $\\sin \\left(\\frac{\\alpha}{2}-\\beta\\right)=\\frac{2}{3}$ 得 $\\cos \\left(\\frac{\\alpha}{2}-\\beta\\right)=\\frac{\\sqrt{5}}{3}$, 所以 $\\sin \\frac{\\alpha+\\beta}{2}=\\sin \\left[\\left(\\alpha-\\frac{\\beta}{2}\\right)-\\left(\\frac{\\alpha}{2}-\\beta\\right)\\right]= \\sin \\left(\\alpha-\\frac{\\beta}{2}\\right) \\cos \\left(\\frac{\\alpha}{2}-\\beta\\right)-\\cos \\left(\\alpha-\\frac{\\beta}{2}\\right) \\sin \\left(\\frac{\\alpha}{2}-\\beta\\right)=\\frac{4 \\sqrt{5}}{9} \\times \\frac{\\sqrt{5}}{3}-\\left(-\\frac{1}{9}\\right) \\times \\frac{2}{3}=\\frac{22}{27}$. 从而 $\\cos (\\alpha+\\beta)=1-2 \\sin ^2 \\frac{\\alpha+\\beta}{2}=1-2 \\times\\left(\\frac{22}{27}\\right)^2=-\\frac{239}{729}$.", + "remark": "", + "figures": [] +} \ No newline at end of file diff --git a/processed_dataset/calculation/0726.json b/processed_dataset/calculation/0726.json new file mode 100644 index 0000000000000000000000000000000000000000..a0563c8fe914b12a2af32b9b7187146b5795598e --- /dev/null +++ b/processed_dataset/calculation/0726.json @@ -0,0 +1,8 @@ +{ + "source_file": "./raw_volume-zh/volume3/exercise2.tex", + "problem_type": "calculation", + "problem": "问题26 (1) 求值: $\\cos ^4 \\frac{\\pi}{16}+\\cos ^4 \\frac{3 \\pi}{16}+\\cos ^4 \\frac{5 \\pi}{16}+\\cdots+\\cos ^4 \\frac{15 \\pi}{16}$;\n(2) 求 $\\cos ^5 \\frac{\\pi}{9}+\\cos ^5 \\frac{5 \\pi}{9}+\\cos ^5 \\frac{7 \\pi}{9}$ 的值.", + "solution": "(1) 设 $x=\\cos ^4 \\frac{\\pi}{16}+\\cos ^4 \\frac{3 \\pi}{16}+\\cos ^4 \\frac{5 \\pi}{16}+\\cos ^4 \\frac{7 \\pi}{16}, y=\\sin ^4 \\frac{\\pi}{16}+\\sin ^4 \\frac{3 \\pi}{16}+ \\sin ^4 \\frac{5 \\pi}{16}+\\sin ^4 \\frac{7 \\pi}{16}$, 则 $x-y=\\left(\\cos ^2 \\frac{\\pi}{16}-\\sin ^2 \\frac{\\pi}{16}\\right)+\\left(\\cos ^2 \\frac{3 \\pi}{16}-\\sin ^2 \\frac{3 \\pi}{16}\\right)+\\left(\\cos ^2 \\frac{5 \\pi}{16}-\\right. \\left.\\sin ^2 \\frac{5 \\pi}{16}\\right)+\\left(\\cos ^2 \\frac{7 \\pi}{16}-\\sin ^2 \\frac{7 \\pi}{16}\\right)=\\cos \\frac{\\pi}{8}+\\cos \\frac{3 \\pi}{8}+\\cos \\frac{5 \\pi}{8}+\\cos \\frac{7 \\pi}{8}=0 ; x+ y=4-2\\left(\\cos ^2 \\frac{\\pi}{16} \\sin ^2 \\frac{\\pi}{16}+\\cos ^2 \\frac{3 \\pi}{16} \\sin ^2 \\frac{3 \\pi}{16}+\\cos ^2 \\frac{5 \\pi}{16} \\sin ^2 \\frac{5 \\pi}{16}+\\cos ^2 \\frac{7 \\pi}{16} \\sin ^2 \\frac{7 \\pi}{16}\\right)=4- \\frac{1}{2}\\left(\\sin ^2 \\frac{\\pi}{8}+\\sin ^2 \\frac{3 \\pi}{8}+\\sin ^2 \\frac{5 \\pi}{8}+\\sin ^2 \\frac{7 \\pi}{8}\\right)=4-\\left(\\sin ^2 \\frac{\\pi}{8}+\\sin ^2 \\frac{3 \\pi}{8}\\right)=4- \\left(\\frac{1-\\cos \\frac{\\pi}{4}}{2}+\\frac{1-\\cos \\frac{3 \\pi}{4}}{2}\\right)=3$. 故 $x=y=\\frac{3}{2}$. (2) 设 $x_1=\\cos \\frac{\\pi}{9}, x_2= \\cos \\frac{5 \\pi}{9}, x_3=\\cos \\frac{7 \\pi}{9}$, 则 $x_1+x_2+x_3=\\cos \\frac{\\pi}{9}+\\cos \\frac{5 \\pi}{9}+\\cos \\frac{7 \\pi}{9}=2 \\cos \\frac{4 \\pi}{9} \\cos \\frac{\\pi}{3}+ \\cos \\frac{5 \\pi}{9}=\\cos \\frac{4 \\pi}{9}+\\cos \\frac{5 \\pi}{9}=0 ; x_1 x_2+x_2 x_3+x_3 x_1=\\cos \\frac{\\pi}{9} \\cos \\frac{5 \\pi}{9}+\\cos \\frac{5 \\pi}{9}$\n$$\n\\begin{aligned}\n& \\cos \\frac{7 \\pi}{9}+\\cos \\frac{7 \\pi}{9} \\cos \\frac{\\pi}{9}=\\frac{1}{2}\\left(\\cos \\frac{2 \\pi}{3}+\\cos \\frac{4 \\pi}{9}+\\cos \\frac{4 \\pi}{3}+\\cos \\frac{2 \\pi}{9}+\\cos \\frac{8 \\pi}{9}+\\right. \\\\\n& \\left.\\cos \\frac{2 \\pi}{3}\\right)=\\frac{1}{2}\\left[-\\frac{3}{2}+\\cos \\frac{4 \\pi}{9}+\\left(\\cos \\frac{2 \\pi}{9}+\\cos \\frac{8 \\pi}{9}\\right)\\right]=-\\frac{3}{4}+\\frac{1}{2}\\left(\\cos \\frac{4 \\pi}{9}+\\right. \\\\\n& \\left.2 \\cos \\frac{5 \\pi}{9} \\cos \\frac{\\pi}{3}\\right)=-\\frac{3}{4}+\\frac{1}{2}\\left(\\cos \\frac{4 \\pi}{9}+\\cos \\frac{5 \\pi}{9}\\right)=-\\frac{3}{4} ; x_1 \\cdot x_2 \\cdot x_3=\\cos \\frac{\\pi}{9} \\cos \\frac{5 \\pi}{9} \\\\\n& \\cos \\frac{7 \\pi}{9}=\\cos \\frac{\\pi}{9} \\cos \\frac{4 \\pi}{9} \\cos \\frac{2 \\pi}{9}=\\frac{8 \\sin \\frac{\\pi}{9} \\cos \\frac{\\pi}{9} \\cos \\frac{2 \\pi}{9} \\cos \\frac{4 \\pi}{9}}{8 \\sin \\frac{\\pi}{9}}=\\frac{\\sin \\frac{8 \\pi}{9}}{8 \\sin \\frac{\\pi}{9}}=\\frac{1}{8}, \\text { 故 } \\\\\n& x_1 、 x_2 、 x_3 \\text { 是方程 } x^3-\\frac{3}{4} x-\\frac{1}{8}=0 \\text { 的根.\n所以 } x^5=\\frac{3}{4} x^3+\\frac{1}{8} x^2= \\\\\n& \\frac{3}{4}\\left(\\frac{3}{4} x+\\frac{1}{8}\\right)+\\frac{1}{8} x^2=\\frac{1}{8} x^2+\\frac{9}{16} x+\\frac{3}{32}, \\text { 从而 } \\cos ^5 \\frac{\\pi}{9}+\\cos ^5 \\frac{5 \\pi}{9}+\\cos ^5 \\frac{7 \\pi}{9}= \\\\\n& \\frac{1}{8}\\left(x_1^2+x_2^2+x_3^2\\right)+\\frac{9}{16}\\left(x_1+x_2+x_3\\right)+\\frac{3}{32} \\times 3=\\frac{1}{8}\\left[\\left(x_1+x_2+x_3\\right)^2-\\right. \\\\\n& \\left.2\\left(x_1 x_2+x_2 x_3+x_3 x_1\\right)\\right]+\\frac{9}{16}\\left(x_1+x_2+x_3\\right)+\\frac{9}{32}=\\frac{1}{8} \\times(-2) \\times\\left(-\\frac{3}{4}\\right)+ \\\\\n& \\frac{9}{32}=\\frac{15}{32} .\n\\end{aligned}\n$$\n$x_1 、 x_2 、 x_3$ 是方程 $x^3-\\frac{3}{4} x-\\frac{1}{8}=0$ 的根.\n所以 $x^5=\\frac{3}{4} x^3+\\frac{1}{8} x^2= \\frac{3}{4}\\left(\\frac{3}{4} x+\\frac{1}{8}\\right)+\\frac{1}{8} x^2=\\frac{1}{8} x^2+\\frac{9}{16} x+\\frac{3}{32}$, 从而 $\\cos ^5 \\frac{\\pi}{9}+\\cos ^5 \\frac{5 \\pi}{9}+\\cos ^5 \\frac{7 \\pi}{9}= \\frac{1}{8}\\left(x_1^2+x_2^2+x_3^2\\right)+\\frac{9}{16}\\left(x_1+x_2+x_3\\right)+\\frac{3}{32} \\times 3=\\frac{1}{8}\\left[\\left(x_1+x_2+x_3\\right)^2-\\right. \\left.2\\left(x_1 x_2+x_2 x_3+x_3 x_1\\right)\\right]+\\frac{9}{16}\\left(x_1+x_2+x_3\\right)+\\frac{9}{32}=\\frac{1}{8} \\times(-2) \\times\\left(-\\frac{3}{4}\\right)+ \\frac{9}{32}=\\frac{15}{32}$.", + "remark": "", + "figures": [] +} \ No newline at end of file diff --git a/processed_dataset/calculation/0727.json b/processed_dataset/calculation/0727.json new file mode 100644 index 0000000000000000000000000000000000000000..f3119ed63e0e671609c7446f0a094eaeaddd49ba --- /dev/null +++ b/processed_dataset/calculation/0727.json @@ -0,0 +1,8 @@ +{ + "source_file": "./raw_volume-zh/volume3/exercise2.tex", + "problem_type": "calculation", + "problem": "问题28. 已知 $\\cos \\alpha=\\tan \\beta, \\cos \\beta=\\tan \\gamma, \\cos \\gamma=\\tan \\alpha$, 则\n$$\n\\sin ^2 \\alpha=\\sin ^2 \\beta=\\sin ^2 \\gamma=\\cos ^4 \\alpha=\\cos ^4 \\beta=\\cos ^4 \\gamma=4 \\sin ^2 18^{\\circ} .\n$$", + "solution": "令 $x=\\cos \\alpha, y=\\cos \\beta, z=\\cos \\gamma$, 则 $x^2 y^2=\\tan ^2 \\beta \\cdot \\cos ^2 \\beta=\\sin ^2 \\beta= 1-y^2 \\cdots$ (1). $y^2 z^2=\\tan ^2 \\gamma \\cdot \\cos ^2 \\gamma=\\sin ^2 \\gamma=1-z^2 \\cdots$ (2). $z^2 x^2=\\tan ^2 \\alpha$. $\\cos ^2 \\alpha=\\sin ^2 \\alpha=1-x^2 \\cdots$ (3). 解之得, $x^2=y^2=z^2=\\frac{\\sqrt{5}-1}{2}=2 \\cdot \\sin 18^{\\circ}$. 所以 $\\cos ^4 \\alpha=\\cos ^4 \\beta=\\cos ^4 \\gamma=4 \\sin ^2 18^{\\circ}$, 又 $\\tan \\alpha=\\cos \\gamma$, 所以 $\\tan ^2 \\alpha=\\cos ^2 \\gamma= \\cos ^2 \\alpha$, 则 $\\sin ^2 \\alpha=\\cos ^4 \\alpha$, 同理, 可得 $\\sin ^2 \\alpha=\\sin ^2 \\beta=\\sin ^2 \\gamma=\\cos ^4 \\alpha=\\cos ^4 \\beta= \\cos ^4 \\gamma=4 \\sin ^2 18^{\\circ}$.", + "remark": "", + "figures": [] +} \ No newline at end of file diff --git a/processed_dataset/calculation/0728.json b/processed_dataset/calculation/0728.json new file mode 100644 index 0000000000000000000000000000000000000000..1f00abac62f5ea10c8d5f357a667991622c53272 --- /dev/null +++ b/processed_dataset/calculation/0728.json @@ -0,0 +1,8 @@ +{ + "source_file": "./raw_volume-zh/volume3/exercise3.tex", + "problem_type": "calculation", + "problem": "问题1 在 $\\triangle A B C$ 中, $a 、 b 、 c$ 三边成等差数列, 则 ( ).\n(A) $01$, 故无解.", + "remark": "", + "figures": [] +} \ No newline at end of file diff --git a/processed_dataset/calculation/0731.json b/processed_dataset/calculation/0731.json new file mode 100644 index 0000000000000000000000000000000000000000..a85483e467d04b695a49b1b26e98a7927fd715db --- /dev/null +++ b/processed_dataset/calculation/0731.json @@ -0,0 +1,8 @@ +{ + "source_file": "./raw_volume-zh/volume3/exercise3.tex", + "problem_type": "calculation", + "problem": "问题4 在 $\\triangle A B C$ 中, 若 $\\sin A-2 \\sin B \\cos C=0$, 则其形状为().\n(A) 直角三角形\n(B) 等腰三角形\n(C) 等腰直角三角形\n(D) 等边三角形", + "solution": "B. 由 $\\sin A=2 \\sin B \\cos C$, 得 $a=2 b \\cdot \\frac{a^2+b^2-c^2}{2 b a}$, 从而 $b=c, \\triangle A B C$ 是等腰三角形.", + "remark": "", + "figures": [] +} \ No newline at end of file diff --git a/processed_dataset/calculation/0732.json b/processed_dataset/calculation/0732.json new file mode 100644 index 0000000000000000000000000000000000000000..22d32100065342518a5882c59c2eef5cd7656d42 --- /dev/null +++ b/processed_dataset/calculation/0732.json @@ -0,0 +1,8 @@ +{ + "source_file": "./raw_volume-zh/volume3/exercise3.tex", + "problem_type": "calculation", + "problem": "问题5 在直角三角形中, $\\angle C=90^{\\circ}$, 则 $\\sin A \\cos ^2\\left(45^{\\circ}-\\frac{B}{2}\\right)-\\sin \\frac{A}{2} \\cos \\frac{A}{2}(\\quad)$.\n(A) 有最大值 $\\frac{1}{4}$ 和最小值 0\n(B) 有最大值 $\\frac{1}{4}$ 但无最小值\n(C) 既无最大值又无最小值\n(D) 有最大值 $\\frac{1}{2}$ 但无最小值", + "solution": "B. 原式 $=\\sin A \\cdot \\frac{1+\\cos \\left(90^{\\circ}-B\\right)}{2}-\\frac{1}{2} \\sin A=\\frac{1}{2} \\sin A \\cdot \\sin B= \\frac{1}{2} \\sin A \\cos A=\\frac{1}{4} \\sin 2 A \\leqslant \\frac{1}{4}$.", + "remark": "", + "figures": [] +} \ No newline at end of file diff --git a/processed_dataset/calculation/0733.json b/processed_dataset/calculation/0733.json new file mode 100644 index 0000000000000000000000000000000000000000..0b1a9466355282ebe3727787ddca93a97b474496 --- /dev/null +++ b/processed_dataset/calculation/0733.json @@ -0,0 +1,8 @@ +{ + "source_file": "./raw_volume-zh/volume3/exercise3.tex", + "problem_type": "calculation", + "problem": "问题6. 已知锐角 $\\triangle A B C$ 的边长分别为 $2 、 3 、 x$, 则第三边 $x$ 适合的条件是 ( ).\n(A) $1x^2$, 且 $2^2+x^2>3^2$, 且 $10$, 所以 $A+B<\\frac{\\pi}{2}, \\triangle A B C$ 为钝角三角形,其外心在三角形外部.", + "remark": "", + "figures": [] +} \ No newline at end of file diff --git a/processed_dataset/calculation/0735.json b/processed_dataset/calculation/0735.json new file mode 100644 index 0000000000000000000000000000000000000000..c385c7e7a7a96dbaf936b53918f805adff67c8e3 --- /dev/null +++ b/processed_dataset/calculation/0735.json @@ -0,0 +1,8 @@ +{ + "source_file": "./raw_volume-zh/volume3/exercise3.tex", + "problem_type": "calculation", + "problem": "问题8. 已知 $\\triangle A B C$ 的三边 $a, b, c$ 成等比数列.\n$a 、 b 、 c$ 所对的角依次为 $\\angle A$ 、 $\\angle B 、 \\angle C$, 则 $\\sin B+\\cos B$ 的取值范围是 ( ).\n(A) $\\left(1,1+\\frac{\\sqrt{3}}{2}\\right]$\n(B) $\\left[\\frac{1}{2}, 1+\\frac{\\sqrt{3}}{2}\\right]$\n(C) $(1, \\sqrt{2}]$\n(D) $\\left[\\frac{1}{2}, \\sqrt{2}\\right]$", + "solution": "C. 由 $a c=b^2=a^2+c^2-2 a c \\cos B \\geqslant 2 a c-2 a c \\cos B \\Rightarrow \\cos B \\geqslant \\frac{1}{2} \\Rightarrow 0b>c$, 在此三角形的两边上分别取点 $P 、 Q$, 使线段 $P Q$ 把 $\\triangle A B C$ 分成面积相等的两部分, 求使 $P Q$ 长度为最短的点 $P 、 Q$ 的位置.", + "solution": "若 $P 、 Q$ 分别取在 $B C 、 A C$ 上, 设 $C P=x, C Q=y$, 则 $\\frac{1}{2} x y \\sin C= \\frac{1}{2} \\cdot \\frac{1}{2} a b \\sin C, x y=\\frac{1}{2} a b$. 由余弦定理, 在 $\\triangle C P Q$ 中, $P Q^2=x^2+y^2- 2 x y \\cos C=(x-y)^2+2 x y(1-\\cos C)=(x-y)^2+a b(1-\\cos C)$. 所以, 当 $x=y$ 时, $P Q$ 有最小值, 这时 $P Q=\\sqrt{a b-a b \\cos C}=\\sqrt{a b-\\frac{1}{2}\\left(a^2+b^2-c^2\\right)}= \\sqrt{\\frac{1}{2}(c+a-b)(c-a+b)}=\\sqrt{2(p-a)(p-b)}$. 其中 $p=\\frac{1}{2}(a+b+c)$. 若 $P 、 Q$ 分别取在 $A C 、 A B$ 边上, 则当 $A P=A Q$ 时, $P Q$ 有最小值 $\\sqrt{2(p-b)(p-c)}$; 若 $P 、 Q$ 分别取在边 $A B 、 B C$ 上, 则当 $B P=B Q$ 时, $P Q$ 有最小值 $\\sqrt{2(p-c)(p-a)}$. 但 $a>b>c$, 因此 $\\sqrt{2(p-a)(p-b)}$ 最小, 即 $P$ 、 $Q$ 分别取在边 $B C 、 A C$ 上, 且 $C P=C Q$ 时, $P Q$ 长度最短.\n因为此时 $P Q= \\sqrt{a b(1-\\cos C)}=\\sqrt{2 a b} \\sin \\frac{C}{2}, \\triangle C P Q$ 是等腰三角形, 所以 $C P=C Q= \\frac{\\frac{1}{2} P Q}{\\sin \\frac{C}{2}}=\\frac{1}{2} \\sqrt{2 a b}$.", + "remark": "", + "figures": [] +} \ No newline at end of file diff --git a/processed_dataset/calculation/0749.json b/processed_dataset/calculation/0749.json new file mode 100644 index 0000000000000000000000000000000000000000..3bbca93d3081c1bb734fcf2c7f0f9e7fb5762e83 --- /dev/null +++ b/processed_dataset/calculation/0749.json @@ -0,0 +1,8 @@ +{ + "source_file": "./raw_volume-zh/volume3/exercise3.tex", + "problem_type": "calculation", + "problem": "问题30. $\\triangle A B C$ 中, 若 $\\frac{\\cos A}{\\sin B}+\\frac{\\cos B}{\\sin A}=2$, 且该三角形的周长为 12 , 求这个三角形面积的最大值.", + "solution": "由 $\\frac{\\cos A}{\\sin B}+\\frac{\\cos B}{\\sin A}=2$ 得 $\\sin A \\cos A+\\sin B \\cos B=2 \\sin A \\sin B$, 即 $\\sin A(\\cos A-\\sin B)+\\sin B(\\cos B-\\sin A)=0 \\cdots$ (1). 也可以变形为 $\\frac{1}{2}(\\sin 2 A+ \\sin 2 B)=2 \\sin A \\sin B, \\sin (A+B) \\cos (A-B)=\\cos (A-B)-\\cos (A+B)$, $\\cos (A+B)=\\cos (A-B)[1-\\sin (A+B)] \\cdots$ (2). 若 $A+B<\\frac{\\pi}{2}$, 则 $A$ 、 $B \\in\\left(0, \\frac{\\pi}{2}\\right), A<\\frac{\\pi}{2}-B, \\cos A>\\sin B, \\sin A<\\cos B$. 所以 (1) 不成立; 若 $A+B>\\frac{\\pi}{2}$, 则 $\\cos (A+B)<0$, 由 (2) 得 $\\cos (A-B)<0$, 且 $\\cos (A+B)> \\cos (A-B)$, 从而 $A+B<|A-B|$. 这不可能, 所以 (2) 不成立.\n综上所述, 可知 $A+B=\\frac{\\pi}{2}, \\triangle A B C$ 是直角三角形.\n由 $a+b+c=12$ 得 $c(1+\\sin A+ \\cos A)=12, c=\\frac{12}{1+\\sin A+\\cos A} . \\triangle A B C$ 的面 积 $S=\\frac{1}{2} a b= \\frac{36\\left(t^2-1\\right)}{(1+t)^2}=\\frac{36(t-1)}{t+1}=36\\left(1+\\frac{-2}{t+1}\\right)$, 且由 $t=\\sqrt{2} \\sin \\left(A+\\frac{\\pi}{4}\\right)$ 及 $0< A<\\frac{\\pi}{2}$ 得 $1 \\frac{\\pi}{4}$, 故选 D.", + "remark": "", + "figures": [] +} \ No newline at end of file diff --git a/processed_dataset/calculation/0754.json b/processed_dataset/calculation/0754.json new file mode 100644 index 0000000000000000000000000000000000000000..ac39147ddd2fe3a3e1a1f3f4351c6d6085945129 --- /dev/null +++ b/processed_dataset/calculation/0754.json @@ -0,0 +1,8 @@ +{ + "source_file": "./raw_volume-zh/volume3/exercise4.tex", + "problem_type": "calculation", + "problem": "问题5. 已知 $x \\in\\left(\\frac{\\pi}{4}, \\frac{5 \\pi}{4}\\right)$, 则 $\\arcsin \\frac{\\sin x+\\cos x}{\\sqrt{2}}$ 的值为 ( ).\n(A) $x+\\frac{\\pi}{4}$\n(B) $\\frac{\\pi}{2}-x$\n(C) $x-\\frac{3 \\pi}{4}$\n(D) $\\frac{3 \\pi}{4}-x$", + "solution": "D. $\\arcsin \\frac{\\sin x+\\cos x}{\\sqrt{2}}=\\arcsin \\left[\\sin \\left(\\frac{\\pi}{4}+x\\right)\\right]$, 因 $x \\in\\left(\\frac{\\pi}{4}, \\frac{5 \\pi}{4}\\right)$, 所以 $\\frac{\\pi}{4}+x \\in\\left(\\frac{\\pi}{2}, \\frac{3 \\pi}{2}\\right)$,故原式 $=\\pi-\\left(\\frac{\\pi}{4}+x\\right)=\\frac{3 \\pi}{4}-x$.", + "remark": "", + "figures": [] +} \ No newline at end of file diff --git a/processed_dataset/calculation/0755.json b/processed_dataset/calculation/0755.json new file mode 100644 index 0000000000000000000000000000000000000000..e90f6bfd44de67240811087cab31f65235a9e3f1 --- /dev/null +++ b/processed_dataset/calculation/0755.json @@ -0,0 +1,8 @@ +{ + "source_file": "./raw_volume-zh/volume3/exercise4.tex", + "problem_type": "calculation", + "problem": "问题6. 设 $M=\\{(x, y)|| x y \\mid=1, x>0\\}, N=\\{(x, y) \\mid \\arctan x+ \\operatorname{arccot} y=\\pi\\}$, 那么 ( ).\n(A) $M \\cup N=\\{(x, y)|| x y \\mid=1\\}$\n(B) $M \\cup N=M$\n(C) $M \\cup N=N \\cup\\{(0,0)\\}$\n(D) $M \\cup N=\\{(x, y)|| x y \\mid=1$, 且 $x 、 y$ 不同时为负数 $\\}$", + "solution": "B. 在集合 $M$ 中,由 $|x y|=1$ 得 $x y=1$ 或 $x y=-1$ 但 $x>0$,故表示反比例函数在 $\\mathrm{I} 、 \\mathrm{IN}$ 象限的两支, 在集合 $N$ 中, 由 $\\arctan x+\\operatorname{arccot} y= \\pi \\Rightarrow \\arctan x=\\pi-\\operatorname{arccot} y$, 所以 $x=\\tan (\\arctan x)=\\tan (\\pi-\\operatorname{arccot} y)= -\\frac{1}{\\cot (\\operatorname{arccot} y)}=-\\frac{1}{y}$, 即 $x y=-1$, 但当 $x<0$ 时, $-\\frac{\\pi}{2}<\\arctan x<0$, 此时 $y>0,0<\\operatorname{arccot} y<\\frac{\\pi}{2}$, 故 $-\\frac{\\pi}{2}<\\arctan x+\\operatorname{arccot} y<\\frac{\\pi}{2}$, 这与\n$\\arctan x+\\operatorname{arccot} y=\\pi$, 矛盾.\n所以 $N=\\{(x, y) \\mid x y=-1, x>0\\} \\varsubsetneqq\\{(x$, $y) \\mid x y=1$ 或 $x y=-1, x>0\\}=M$, 故 $M \\cup N=M$.", + "remark": "", + "figures": [] +} \ No newline at end of file diff --git a/processed_dataset/calculation/0756.json b/processed_dataset/calculation/0756.json new file mode 100644 index 0000000000000000000000000000000000000000..a101d027a604fa42a3326190e95e780f96917ab1 --- /dev/null +++ b/processed_dataset/calculation/0756.json @@ -0,0 +1,8 @@ +{ + "source_file": "./raw_volume-zh/volume3/exercise4.tex", + "problem_type": "calculation", + "problem": "问题7 已知方程 $\\arccos \\frac{4}{5}-\\arccos \\left(-\\frac{4}{5}\\right)=\\arcsin x$, 则 ( ).\n(A) $x=\\frac{24}{25}$\n(B) $x=-\\frac{24}{25}$\n(C) $x=0$\n(D) 无解", + "solution": "D. 因为 $\\arccos \\left(-\\frac{4}{5}\\right)=\\pi-\\arccos \\frac{4}{5}$, 原方程化为 $2 \\arccos \\frac{4}{5}- \\arcsin x=\\pi$, 因为 $\\arccos \\frac{4}{5}<\\arccos \\frac{\\sqrt{2}}{2}=\\frac{\\pi}{4}$, 又 $-\\arcsin x \\leqslant \\frac{\\pi}{2}$, 所以 $2 \\arccos \\frac{4}{5}-\\arcsin x<\\pi$, 故无解.", + "remark": "", + "figures": [] +} \ No newline at end of file diff --git a/processed_dataset/calculation/0757.json b/processed_dataset/calculation/0757.json new file mode 100644 index 0000000000000000000000000000000000000000..0d309042e715ddc6e099971386c915ed90f2fd9b --- /dev/null +++ b/processed_dataset/calculation/0757.json @@ -0,0 +1,8 @@ +{ + "source_file": "./raw_volume-zh/volume3/exercise4.tex", + "problem_type": "calculation", + "problem": "问题8. 方程 $\\frac{\\cos 2 x}{1+\\sin 2 x}=0$ 的解集是 ( ).\n(A) $\\left\\{x \\mid x=2 k \\pi \\pm \\frac{\\pi}{4}, k \\in \\mathbf{Z}\\right\\}$\n(B) $\\left\\{x \\mid x=k \\pi \\pm \\frac{\\pi}{4}, k \\in \\mathbf{Z}\\right\\}$\n(C) $\\left\\{x \\mid x=k \\pi+\\frac{\\pi}{4}, k \\in \\mathbf{Z}\\right\\}$\n(D) $\\left\\{x \\mid x=\\frac{1}{2} k \\pi+\\frac{\\pi}{4}, k \\in \\mathbf{Z}\\right\\}$", + "solution": "C. 由原方程得 $2 x=k \\pi+\\frac{\\pi}{2}$ 且 $2 x \\neq 2 k \\pi+\\frac{3 \\pi}{2}$, 即 $x=\\frac{1}{2} k \\pi+\\frac{\\pi}{4}$ 且 $x \\neq k \\pi+\\frac{3 \\pi}{4}$, 故选 C.", + "remark": "", + "figures": [] +} \ No newline at end of file diff --git a/processed_dataset/calculation/0758.json b/processed_dataset/calculation/0758.json new file mode 100644 index 0000000000000000000000000000000000000000..fb3f54248cf2f0774e3b8f7e0c1caa32b69841d8 --- /dev/null +++ b/processed_dataset/calculation/0758.json @@ -0,0 +1,8 @@ +{ + "source_file": "./raw_volume-zh/volume3/exercise4.tex", + "problem_type": "calculation", + "problem": "问题9. 方程 $\\lg x=\\cos 2 x$ 的解有( ).\n(A) 2 个\n(B) 5 个\n(C) 7 个\n(D) 无数个", + "solution": "B. 分别作出 $y=\\lg x$ 与 $y=\\cos 2 x$ 的图象而观察.", + "remark": "", + "figures": [] +} \ No newline at end of file diff --git a/processed_dataset/calculation/0759.json b/processed_dataset/calculation/0759.json new file mode 100644 index 0000000000000000000000000000000000000000..557eafe5b9bff0e7f77b1b8342c7b23500f4a358 --- /dev/null +++ b/processed_dataset/calculation/0759.json @@ -0,0 +1,8 @@ +{ + "source_file": "./raw_volume-zh/volume3/exercise4.tex", + "problem_type": "calculation", + "problem": "问题11 函数 $y=\\arccos \\left(-2 x^2+x\\right)$ 的值域是", + "solution": "$\\left[\\arccos \\frac{1}{8}, \\pi\\right]$. 因 $-2 x^2+x=-2\\left(x-\\frac{1}{4}\\right)^2+\\frac{1}{8} \\leqslant \\frac{1}{8}$, 所以其值域为 $\\left[\\arccos \\frac{1}{8}, \\pi\\right]$.", + "remark": "", + "figures": [] +} \ No newline at end of file diff --git a/processed_dataset/calculation/0760.json b/processed_dataset/calculation/0760.json new file mode 100644 index 0000000000000000000000000000000000000000..de5f6bcafd8ea9c9a263b1082b31852c67c90b26 --- /dev/null +++ b/processed_dataset/calculation/0760.json @@ -0,0 +1,8 @@ +{ + "source_file": "./raw_volume-zh/volume3/exercise4.tex", + "problem_type": "calculation", + "problem": "问题12 函数 $y=\\sin x, x \\in\\left[\\pi, \\frac{3}{2} \\pi\\right]$ 的反函数是", + "solution": "$f^{-1}(x)=\\pi-\\arcsin x(-1 \\leqslant x \\leqslant 0)$.", + "remark": "", + "figures": [] +} \ No newline at end of file diff --git a/processed_dataset/calculation/0761.json b/processed_dataset/calculation/0761.json new file mode 100644 index 0000000000000000000000000000000000000000..67440155101fec6678d0af35af29689d2e535e87 --- /dev/null +++ b/processed_dataset/calculation/0761.json @@ -0,0 +1,8 @@ +{ + "source_file": "./raw_volume-zh/volume3/exercise4.tex", + "problem_type": "calculation", + "problem": "问题13 不等式 $\\arccos x>\\arcsin x$ 的解集是", + "solution": "$\\left[-1, \\frac{\\sqrt{2}}{2}\\right)$. 显然 $-1 \\leqslant x \\leqslant 0$ 时, 原不等式恒成立; 当 $x>0$ 时, 由 $\\arccos x>\\arcsin x$, 得 $\\sqrt{1-x^2}>x$, 即 $x^2<\\frac{1}{2}$, 所以 $00, a<0, c<0$, 又 $\\tan (-1)=\\frac{\\sin (-1)}{\\cos (-1)}< \\sin (-1)$, 所以 $c0$, 所以 $\\frac{a^2+b^2}{2}>b$. 选 B.", + "remark": "", + "figures": [] +} \ No newline at end of file diff --git a/processed_dataset/calculation/0779.json b/processed_dataset/calculation/0779.json new file mode 100644 index 0000000000000000000000000000000000000000..eadca66cc45391e0c42e11903aa7458277581c60 --- /dev/null +++ b/processed_dataset/calculation/0779.json @@ -0,0 +1,8 @@ +{ + "source_file": "./raw_volume-zh/volume3/exercise5.tex", + "problem_type": "calculation", + "problem": "问题3. 若 $a=(\\tan x)^{\\cot x}, b=(\\cot x)^{\\tan x}, c=(\\tan x)^{\\cos x}$, 当 $0\\cos x$, 所以 $(\\tan x)^{\\cot x}<(\\tan x)^{\\cos x}$, 即 $a(\\tan x)^{\\cos x}$, 得 $b>c$, 故 $a\\sin \\beta$\n(D) 不能确定", + "solution": "B. $\\cos \\alpha-\\sin \\beta=\\sin \\left(\\frac{\\pi}{2}-\\alpha\\right)-\\sin \\beta=2 \\cos \\left(\\frac{\\pi}{4}-\\frac{\\alpha-\\beta}{2}\\right) \\sin \\left(\\frac{\\pi}{4}-\\right. \\left.\\frac{\\alpha+\\beta}{2}\\right), \\cos \\left(\\frac{\\pi}{4}-\\frac{\\alpha-\\beta}{2}\\right)>0, \\sin \\left(\\frac{\\pi}{4}-\\frac{\\alpha+\\beta}{2}\\right)<0$, 故 $\\cos \\alpha<\\sin \\beta$.", + "remark": "", + "figures": [] +} \ No newline at end of file diff --git a/processed_dataset/calculation/0782.json b/processed_dataset/calculation/0782.json new file mode 100644 index 0000000000000000000000000000000000000000..c8ddceace49f379edb12a21d4ad1f22c957d83ce --- /dev/null +++ b/processed_dataset/calculation/0782.json @@ -0,0 +1,8 @@ +{ + "source_file": "./raw_volume-zh/volume3/exercise5.tex", + "problem_type": "calculation", + "problem": "问题6. 设 $\\alpha \\in\\left(\\frac{\\pi}{4}, \\frac{\\pi}{2}\\right)$, 则 $(\\cos \\alpha)^{\\cos \\alpha}=a,(\\sin \\alpha)^{\\cos \\alpha}=b,(\\cos \\alpha)^{\\sin \\alpha}=c$ 的大小顺序是 ( ).\n(A) $a\\cos 2 y$ 成立的一个充分非必要条件是( ).\n(A) $|\\cos x|>|\\cos y|$\n(B) $|\\cos x|>\\cos y$\n(C) $\\cos x>|\\cos y|$\n(D) $\\cos x+\\cos y>0$", + "solution": "C. $\\cos 2 x>\\cos 2 y \\Leftrightarrow 2 \\cos ^2 x-1>2 \\cos ^2 y-1 \\Leftrightarrow \\cos ^2 x>\\cos ^2 y \\Leftrightarrow |\\cos x|>|\\cos y|$. 故选 C.", + "remark": "", + "figures": [] +} \ No newline at end of file diff --git a/processed_dataset/calculation/0786.json b/processed_dataset/calculation/0786.json new file mode 100644 index 0000000000000000000000000000000000000000..80e09ecf378f48469fe9be1315c57c91b6d2e624 --- /dev/null +++ b/processed_dataset/calculation/0786.json @@ -0,0 +1,8 @@ +{ + "source_file": "./raw_volume-zh/volume3/exercise5.tex", + "problem_type": "calculation", + "problem": "问题10. 记 $a=\\log _{\\sin 1} \\cos 1, b=\\log _{\\sin 1} \\tan 1, c=\\log _{\\cos 1} \\sin 1, d=\\log _{\\cos 1} \\tan 1$, 则四个数的大小关系是( ).\n(A) $a\\cos 1, \\tan 1>1$, 所以 $\\log _{\\tan 1} \\cos 1<\\log _{\\tan 1} \\sin 1<0$, 故 $\\log _{\\cos 1} \\tan 1>\\log _{\\sin 1} \\tan 1$, 即 $d>b$. 因为 $0<\\cos 1<1, \\sin 1<\\tan 1$, 所以 $\\log _{\\cos 1} \\sin 1>\\log _{\\cos 1} \\tan 1$, 即 $c>d$; 同理 $a=\\log _{\\sin 1} \\cos 1>\\log _{\\sin 1} \\sin 1= \\log _{\\cos 1} \\cos 1>\\log _{\\cos 1} \\sin 1$, 即 $a>c$, 综上所述 $a>c>d>b$.", + "remark": "", + "figures": [] +} \ No newline at end of file diff --git a/processed_dataset/calculation/0787.json b/processed_dataset/calculation/0787.json new file mode 100644 index 0000000000000000000000000000000000000000..8f450330e1d7064c6d68783ee049feb4ddec58e9 --- /dev/null +++ b/processed_dataset/calculation/0787.json @@ -0,0 +1,8 @@ +{ + "source_file": "./raw_volume-zh/volume3/exercise5.tex", + "problem_type": "calculation", + "problem": "问题11. 当 $02$ 的解集是", + "solution": "$\\left(\\frac{\\pi}{4}, \\frac{\\pi}{3}\\right) \\cup\\left(\\frac{\\pi}{2}, \\frac{3 \\pi}{4}\\right) \\cup\\left(\\frac{5 \\pi}{4}, \\frac{3 \\pi}{2}\\right) \\cup\\left(\\frac{5 \\pi}{3}, \\frac{7 \\pi}{4}\\right)$. 由原不等式得 $\\frac{\\cos 2 x-\\cos x+1}{\\cos 2 x}<0 \\Leftrightarrow \\frac{2 \\cos ^2 x-\\cos x}{\\cos 2 x}<0 \\Leftrightarrow\\left\\{\\begin{array}{l}2 \\cos ^2 x-\\cos x>0 \\\\ \\cos 2 x<0\\end{array}\\right.$ 或 $\\left\\{\\begin{array}{l}2 \\cos ^2 x-\\cos x<0 \\\\ \\cos 2 x>0\\end{array} \\Leftrightarrow x \\in\\left(\\frac{\\pi}{4}, \\frac{\\pi}{3}\\right) \\cup\\left(\\frac{\\pi}{2}, \\frac{3 \\pi}{4}\\right) \\cup\\left(\\frac{5 \\pi}{4}, \\frac{3 \\pi}{2}\\right) \\cup\\left(\\frac{5 \\pi}{3}, \\frac{7 \\pi}{4}\\right)\\right.$.", + "remark": "", + "figures": [] +} \ No newline at end of file diff --git a/processed_dataset/calculation/0788.json b/processed_dataset/calculation/0788.json new file mode 100644 index 0000000000000000000000000000000000000000..95ffb48ed020c15902c2c0dd781bec45f5f552d4 --- /dev/null +++ b/processed_dataset/calculation/0788.json @@ -0,0 +1,8 @@ +{ + "source_file": "./raw_volume-zh/volume3/exercise5.tex", + "problem_type": "calculation", + "problem": "问题12. 已知 $\\sin x<\\sin \\frac{5 \\pi}{4}$, 且 $x$ 为第三象限角, 则 $x$ 的取值范围为", + "solution": "$\\left(2 k \\pi+\\frac{5 \\pi}{4}, 2 k \\pi+\\frac{3 \\pi}{2}\\right),(k \\in \\mathbf{Z})$. 利用函数的单调性可得.", + "remark": "", + "figures": [] +} \ No newline at end of file diff --git a/processed_dataset/calculation/0789.json b/processed_dataset/calculation/0789.json new file mode 100644 index 0000000000000000000000000000000000000000..7aa37d0cef9bf5fc9e819405207555d92a08076d --- /dev/null +++ b/processed_dataset/calculation/0789.json @@ -0,0 +1,8 @@ +{ + "source_file": "./raw_volume-zh/volume3/exercise5.tex", + "problem_type": "calculation", + "problem": "问题13. 若 $0B$, 下列三个不等式 (1) $\\sin A>\\sin B$; (2) $\\cos A<\\cos B$; (3) $\\tan A>\\tan B$, 其中正确的有", + "solution": "(1)(2). 在 $\\triangle A B C$ 中, $A>B$ 则 $a>b$, 所以 $2 R \\sin A>2 R \\sin B$, 即 (1) 正确; 又若 $A$ 为针角或直角, 则 $\\cos A \\leqslant 0<\\cos B$; 若 $A$ 为锐角, 则 $\\cos A< \\cos B$, 所以 (2) 正确; 若 $A$ 为钝角, $\\tan A<0$, 所以 (3) 错误.\n综上所述, 正确的有 (1)(2).", + "remark": "", + "figures": [] +} \ No newline at end of file diff --git a/processed_dataset/calculation/0794.json b/processed_dataset/calculation/0794.json new file mode 100644 index 0000000000000000000000000000000000000000..b22d25ed25b6390ce7d960f6d966b617074e7433 --- /dev/null +++ b/processed_dataset/calculation/0794.json @@ -0,0 +1,8 @@ +{ + "source_file": "./raw_volume-zh/volume3/exercise5.tex", + "problem_type": "calculation", + "problem": "问题18. 若函数 $f(1+\\cos x)=\\cos ^2 x$, 则 $f(1+\\cos x)$ 与 $f(1-\\cos x)$ 的大小关系是", + "solution": "$f(1+\\cos x)=f(1-\\cos x)$. 设 $1+\\cos x=t$, 则 $\\cos x=t-1$, $f(t)=(t-1)^2$, 所以 $f(1-\\cos x)=(1-\\cos x-1)^2=\\cos ^2 x$. 故 $f(1+\\cos x)= f(1-\\cos x)$.", + "remark": "", + "figures": [] +} \ No newline at end of file diff --git a/processed_dataset/calculation/0795.json b/processed_dataset/calculation/0795.json new file mode 100644 index 0000000000000000000000000000000000000000..eeb84bc65617c2c7260a163bff5a35cc2a605992 --- /dev/null +++ b/processed_dataset/calculation/0795.json @@ -0,0 +1,8 @@ +{ + "source_file": "./raw_volume-zh/volume3/exercise5.tex", + "problem_type": "calculation", + "problem": "问题19. 当 $y=2 \\cos x-3 \\sin x$ 取得最大值时, $\\tan x=$", + "solution": "$\\frac{3}{2} \\cdot y=\\sqrt{13}\\left(\\frac{2}{\\sqrt{13}} \\cos x-\\frac{3}{\\sqrt{13}}-\\sin x\\right)=\\sqrt{13} \\sin (\\varphi-x)$, 其中 $\\sin \\varphi=\\frac{2}{\\sqrt{13}}, \\cos \\varphi=-\\frac{3}{\\sqrt{13}}$, 所以当 $\\sin (\\varphi-x)=1$ 时, $y$ 有最大值 $\\sqrt{13}$, 此时, $\\varphi-x=2 k \\pi+\\frac{\\pi}{2}, x=\\varphi-\\left(2 k \\pi+\\frac{\\pi}{2}\\right), \\tan x=\\tan \\left[\\varphi-\\left(2 k \\pi+\\frac{\\pi}{2}\\right)\\right]=$\n$$\n-\\cot \\varphi=-\\frac{\\cos \\varphi}{\\sin \\varphi}=\\frac{3}{2} .\n$$", + "remark": "", + "figures": [] +} \ No newline at end of file diff --git a/processed_dataset/calculation/0796.json b/processed_dataset/calculation/0796.json new file mode 100644 index 0000000000000000000000000000000000000000..3baf608fe7de27fa5133f48ac0d840651411876b --- /dev/null +++ b/processed_dataset/calculation/0796.json @@ -0,0 +1,8 @@ +{ + "source_file": "./raw_volume-zh/volume3/exercise5.tex", + "problem_type": "calculation", + "problem": "问题20. 知 $\\triangle A B C$ 是锐角三角形, $m=\\log _{\\cos A} \\frac{1}{\\sin B}, n=\\log _{\\cos A} \\frac{1}{\\cos C}$, 则 $m$ 与 $n$ 的大小关系为", + "solution": "$m>n . \\triangle A B C$ 为锐角三角形, 得 $0\\frac{\\pi}{2}-C$, 又 $B 、 \\frac{\\pi}{2}-C$ 均为锐角, 故 $\\sin B>\\cos C>0$, 从而 $\\frac{1}{\\sin B}<\\frac{1}{\\cos C}$, 又 $0<\\cos A<1$. 所以 $m>n$.", + "remark": "", + "figures": [] +} \ No newline at end of file diff --git a/processed_dataset/calculation/0797.json b/processed_dataset/calculation/0797.json new file mode 100644 index 0000000000000000000000000000000000000000..b547cf0d42abc001a7a9c37002a740cd15ee4abb --- /dev/null +++ b/processed_dataset/calculation/0797.json @@ -0,0 +1,8 @@ +{ + "source_file": "./raw_volume-zh/volume3/exercise5.tex", + "problem_type": "calculation", + "problem": "问题29. 求实数 $a$ 的取值范围, 使得对任意实数 $x$ 和任意 $\\theta \\in\\left[0, \\frac{\\pi}{2}\\right]$, 恒有 $(x+$\n$$\n3+2 \\sin \\theta \\cdot \\cos \\theta)^2+(x+a \\sin \\theta+a \\cos \\theta)^2 \\geqslant \\frac{1}{8} \\text {. }\n$$", + "solution": "设 $u=\\sin \\theta+\\cos \\theta=\\sqrt{2} \\sin \\left(\\theta+\\frac{\\pi}{4}\\right) \\in[1, \\sqrt{2}]$, 则 $2 \\sin \\theta \\cos \\theta= u^2-1$, 原不等式可化为 $\\left(x+u^2+2\\right)^2+(x+a u)^2 \\geqslant \\frac{1}{8}$, 即 $2 x^2+2\\left(u^2+a u+\\right.$ 2) $x+\\left(u^2+2\\right)^2+a^2 u^2-\\frac{1}{8} \\geqslant 0$, 所以 $\\Delta=4\\left(u^2+a u+2\\right)^2-8\\left(u^2+2\\right)^2- 8 a^2 u^2+1 \\leqslant 0$. 从而 $\\left(u^2-a u+2\\right)^2 \\geqslant \\frac{1}{4}$, 即 $u^2-a u+\\frac{3}{2} \\geqslant 0$ 或 $u^2-a u+ \\frac{5}{2} \\leqslant 0$, 故 $a \\geqslant u+\\frac{5}{2 u}$ 或 $a \\leqslant u+\\frac{3}{2 u}$. 当 $u \\in[1, \\sqrt{2}]$ 时, $f(u)=u+\\frac{5}{2 u}$ 在 $[1, \\sqrt{2}]$ 上为减函数, 故 $a \\geqslant 1+\\frac{5}{2}=\\frac{7}{2}$, 而 $g(u)=u+\\frac{3}{2 u} \\geqslant \\sqrt{6}$, 当且仅当 $u=\\sqrt{\\frac{3}{2}}$ 时等号成立, 故 $a \\leqslant \\sqrt{6}$, 综上所述, $a$ 的取值范围为 $a \\geqslant \\frac{7}{2}$ 或 $a \\leqslant \\sqrt{6}$.", + "remark": "", + "figures": [] +} \ No newline at end of file diff --git a/processed_dataset/calculation/0798.json b/processed_dataset/calculation/0798.json new file mode 100644 index 0000000000000000000000000000000000000000..48441d704f854b9685d145f873f0c2762f64407e --- /dev/null +++ b/processed_dataset/calculation/0798.json @@ -0,0 +1,8 @@ +{ + "source_file": "./raw_volume-zh/volume3/exercise6.tex", + "problem_type": "calculation", + "problem": "问题1. 解方程组 $\\left(1-x_k^2\\right) x_{k+1}=2 x_k$, 且 $x_{n+1}=x_1$, 其中 $k=1,2,3, \\cdots, n$.", + "solution": "令 $x_1=\\tan \\theta$, 其中 $\\theta \\in\\left(-\\frac{\\pi}{2}, \\frac{\\pi}{2}\\right)$, 则 $x_2=\\frac{2 x_1}{1-x_1^2}=\\tan 2 \\theta$, 所以 $x_k=\\tan 2^{k-1} \\theta(k=1,2, \\cdots, n)$, 又 $x_{n+1}==x_1$, 得 $\\tan 2^n \\theta=\\tan \\theta$, 故 $2^n \\theta= \\theta+m \\pi\\left[m \\in \\mathbf{Z}\\right.$, 且 $\\left.\\theta \\in\\left(-\\frac{\\pi}{2}, \\frac{\\pi}{2}\\right)\\right]$, 即 $\\theta=\\frac{m \\pi}{2^n-1}$. 从而原方程组的解为 $x_k=\\tan \\frac{2^{k-1} m \\pi}{2^n-1}(m \\in \\mathbf{Z}, k=1,2,3, \\cdots, n+1)$.", + "remark": "", + "figures": [] +} \ No newline at end of file diff --git a/processed_dataset/calculation/0799.json b/processed_dataset/calculation/0799.json new file mode 100644 index 0000000000000000000000000000000000000000..1b045ab995ed0f6afeba79324b0e37f62dd8152f --- /dev/null +++ b/processed_dataset/calculation/0799.json @@ -0,0 +1,8 @@ +{ + "source_file": "./raw_volume-zh/volume3/exercise6.tex", + "problem_type": "calculation", + "problem": "问题2. 解不等式 $\\frac{x}{\\sqrt{1+x^2}}+\\frac{1-x^2}{1+x^2}>0$.", + "solution": "设 $x=\\tan \\theta$, 其中 $\\theta \\in\\left(-\\frac{\\pi}{2}, \\frac{\\pi}{2}\\right)$, 则 $\\frac{x}{\\sqrt{1+x^2}}=\\sin \\theta, \\frac{1-x^2}{1+x^2}= \\cos 2 \\theta$, 原不等式可化为 $\\sin \\theta+\\cos 2 \\theta>0$, 即 $(2 \\sin \\theta+1)(\\sin \\theta-1)<0$. 所以 $\\sin \\theta>-\\frac{1}{2}$, 得 $-\\frac{\\pi}{6}<\\theta<\\frac{\\pi}{2}$. 即 $\\tan \\theta>\\tan \\left(-\\frac{\\pi}{6}\\right)=-\\frac{\\sqrt{3}}{3}$, 故原不等式的解集为 $\\left\\{x \\mid x>-\\frac{\\sqrt{3}}{3}\\right\\}$.", + "remark": "", + "figures": [] +} \ No newline at end of file diff --git a/processed_dataset/calculation/0800.json b/processed_dataset/calculation/0800.json new file mode 100644 index 0000000000000000000000000000000000000000..d08e0b5c1f3741fcaf60af7e24be33f5720c2a5e --- /dev/null +++ b/processed_dataset/calculation/0800.json @@ -0,0 +1,8 @@ +{ + "source_file": "./raw_volume-zh/volume3/exercise6.tex", + "problem_type": "calculation", + "problem": "问题7. 解方程 $x+\\frac{x}{\\sqrt{x^2-1}}=\\frac{35}{12}$.", + "solution": "因 $|x|>1$, 且 $x>0$, 所以设 $x=\\frac{1}{\\cos \\alpha}, \\alpha \\in\\left(0, \\frac{\\pi}{2}\\right)$, 代入原方程得 $\\frac{1}{\\cos \\alpha}+\\frac{1}{\\sin \\alpha}=\\frac{35}{12}$, 解得 $\\cos \\alpha=\\frac{3}{5}$ 或 $\\frac{4}{5}$, 所以 $x=\\frac{5}{3}$ 或 $\\frac{5}{4}$.", + "remark": "", + "figures": [] +} \ No newline at end of file diff --git a/processed_dataset/calculation/0801.json b/processed_dataset/calculation/0801.json new file mode 100644 index 0000000000000000000000000000000000000000..521af2c20c26a25183a15d71241a362088027ca1 --- /dev/null +++ b/processed_dataset/calculation/0801.json @@ -0,0 +1,8 @@ +{ + "source_file": "./raw_volume-zh/volume3/exercise6.tex", + "problem_type": "calculation", + "problem": "问题8. 解方程组 $\\left\\{\\begin{array}{l}\\sqrt{x(1-y)}+\\sqrt{y(1-x)}=1, \\\\ \\sqrt{x y}+\\sqrt{(1-x)(1-y)}=1 .\\end{array}\\right.$", + "solution": "从原方程组得 $0 \\leqslant x \\leqslant 1,0 \\leqslant y \\leqslant 1$, 故设 $x=\\cos ^2 \\alpha, y=\\cos ^2 \\beta$, 其 $\\left\\{\\begin{array}{l}\\sin (\\alpha+\\beta)=1, \\\\ \\cos (\\alpha-\\beta)=1 .\\end{array}\\right.$ 所以 $\\alpha=\\beta=\\frac{\\pi}{4}$, 得 $x=y=\\frac{1}{2}$.", + "remark": "", + "figures": [] +} \ No newline at end of file diff --git a/processed_dataset/calculation/0802.json b/processed_dataset/calculation/0802.json new file mode 100644 index 0000000000000000000000000000000000000000..4803c5949f62106503b4a7c71531e5da4836d228 --- /dev/null +++ b/processed_dataset/calculation/0802.json @@ -0,0 +1,8 @@ +{ + "source_file": "./raw_volume-zh/volume3/exercise6.tex", + "problem_type": "calculation", + "problem": "问题9. 求满足下列等式的实数 $x 、 y 、 z:\\left(1+x^2\\right)\\left(1+y^2\\right)\\left(1+z^2\\right)=4 x y\\left(1-z^2\\right)$.", + "solution": "原等式变形为 $\\frac{2 \\dot{x}}{1+x^2} \\cdot \\frac{2 y}{1+y^2} \\cdot \\frac{1-z^2}{1+z^2}=1$, 设 $x=\\tan \\alpha, y=\\tan \\beta$, $z=\\tan \\gamma$, 其中 $\\alpha, \\beta, \\gamma \\in\\left(-\\frac{\\pi}{2}, \\frac{\\pi}{2}\\right)$, 则 $\\sin 2 \\alpha \\cdot \\sin 2 \\beta \\cdot \\cos 2 \\gamma=1$. 但 $|\\sin 2 \\alpha| \\leqslant 1,|\\sin 2 \\beta| \\leqslant 1,|\\cos 2 \\gamma| \\leqslant 1$, 又 $2 \\alpha, 2 \\beta, 2 \\gamma \\in(-\\pi, \\pi)$, 所以 $\\sin 2 \\alpha=\\sin 2 \\beta=\\cos 2 \\gamma=1$ 或 $\\sin 2 \\alpha=\\sin 2 \\beta=-1, \\cos 2 \\gamma=1$, 即 $\\alpha=\\beta= \\frac{\\pi}{4}, \\gamma=0$ 或 $\\alpha=\\beta=-\\frac{\\pi}{4}, \\gamma=0$. 从而 $x=y=1$ 及 $z=0$ 或 $x=y=-1$ 及 $z=0$.", + "remark": "", + "figures": [] +} \ No newline at end of file diff --git a/processed_dataset/calculation/0803.json b/processed_dataset/calculation/0803.json new file mode 100644 index 0000000000000000000000000000000000000000..119db0f9373d5aa720517c4caeb06fa860bbb23f --- /dev/null +++ b/processed_dataset/calculation/0803.json @@ -0,0 +1,8 @@ +{ + "source_file": "./raw_volume-zh/volume3/exercise6.tex", + "problem_type": "calculation", + "problem": "问题12. 求函数 $y=x+4+\\sqrt{5-x^2}$ 的最小值和最大值.", + "solution": "设 $x=\\sqrt{5} \\cos \\theta, \\theta \\in[0, \\pi]$, 则 $y=\\sqrt{5} \\cos \\theta+4+\\sqrt{5} \\sin \\theta= \\sqrt{10} \\sin \\left(\\theta+\\frac{\\pi}{4}\\right)+4$, 由 $\\theta+\\frac{\\pi}{4} \\in\\left[\\frac{\\pi}{4}, \\frac{5 \\pi}{4}\\right]$, 得 $\\sin \\left(\\theta+\\frac{\\pi}{4}\\right) \\in\\left[-\\frac{\\sqrt{2}}{2}, 1\\right]$, 当 $\\theta=\\pi$ 时, $y_{\\text {min }}=4-\\sqrt{5}$; 当 $\\theta=\\frac{\\pi}{4}$ 时, $y_{\\text {max }}=4+\\sqrt{10}$.", + "remark": "", + "figures": [] +} \ No newline at end of file diff --git a/processed_dataset/calculation/0804.json b/processed_dataset/calculation/0804.json new file mode 100644 index 0000000000000000000000000000000000000000..7d181daa7b90b10461c1f41b3a984f469ed2d932 --- /dev/null +++ b/processed_dataset/calculation/0804.json @@ -0,0 +1,10 @@ +{ + "source_file": "./raw_volume-zh/volume3/exercise6.tex", + "problem_type": "calculation", + "problem": "问题15. 某体育馆拟用运动场的边角地建一个矩形的健身室,如图()所示, $A B C D$ 是一块边长为 50 米的正方形地皮, 扇形 $C E F$ 是运动场的一部分, 其半径为 40 米,矩形 $A G H M$ 就是拟建的健身室, 其中 $G$ 、 $M$ 分别在 $A B$ 和 $A D$ 上, $H$ 在 $\\overparen{E F}$ 上, 设矩形 $A G H M$ 的面积为 $S, \\angle H C F=\\theta$, 请将 $S$ 表示为 $\\theta$ 的函数,并找出点 $H$ 在 $\\overparen{E F}$ 的何处时,该健身室的面积最大,并求出最大面积.", + "solution": "延长 $G H$ 交 $C D$ 于 $P$, 则 $H P \\perp C D, H P=C H \\sin \\theta=40 \\sin \\theta$, $C P=C H \\cos \\theta=40 \\cos \\theta, H G=50-40 \\sin \\theta, H M=50-40 \\cos \\theta$, 所以 $S= H G \\cdot H M=(50-40 \\sin \\theta)(50-40 \\cos \\theta)=100[25-20(\\sin \\theta+\\cos \\theta)+ 16 \\sin \\theta \\cos \\theta]$, 其中 $0 \\leqslant \\theta \\leqslant \\frac{\\pi}{2}$, 设 $t=\\sin \\theta+\\cos \\theta$, 则 $2 \\sin \\theta \\cos \\theta=t^2-1,1 \\leqslant t \\leqslant \\sqrt{2}$, 故 $S=800\\left(t-\\frac{5}{4}\\right)^2+450$, 当 $t=1$ 时, $S_{\\text {max }}=500\\left(\\right.$ 米 $\\left.^2\\right)$. 此时 $\\theta=0$ 或 $\\frac{\\pi}{2}$, 即 $H$ 在 $\\overparen{E F}$ 的端点 $E$ 或 $F$ 处.", + "remark": "", + "figures": [ + "./images/volume3/figures/fig-c6p15.png" + ] +} \ No newline at end of file diff --git a/processed_dataset/calculation/0805.json b/processed_dataset/calculation/0805.json new file mode 100644 index 0000000000000000000000000000000000000000..710503596bd86e3e4da1a7177717f772dff975d8 --- /dev/null +++ b/processed_dataset/calculation/0805.json @@ -0,0 +1,10 @@ +{ + "source_file": "./raw_volume-zh/volume3/exercise6.tex", + "problem_type": "calculation", + "problem": "问题16. 某城市有一条公路从正西方向 $O A$ 通过市中心 $O$ 后转向东北方向 $O B$, 现要修建一条铁路 $L, L$ 在 $O A$ 上设一车站 $A$, 在 $O B$ 上设一车站 $B$, 铁路在 $A B$ 部分为直线段, 现要求市中心 $O$ 与 $A B$ 之间距离为 10 千米, 问把 $A B$分别设在公路上距中心 $O$ 多远处才能使 $|A B|$ 最短? 并求出最短距离.", + "solution": "如图(), 由已知条件得 $O H \\perp A B, O H=10, \\angle A O B=135^{\\circ}$, 设 $\\angle O A B=\\alpha, \\angle O B A=45^{\\circ}-\\alpha$, 则 $A H=10 \\cot \\alpha, B H=10 \\cot \\left(45^{\\circ}-\\alpha\\right)$,于是 $A B=10 \\cot \\alpha+ 10 \\cot \\left(45^{\\circ}-\\alpha\\right)=\\frac{10}{\\tan \\alpha}+\\frac{10(1+\\tan \\alpha)}{1-\\tan \\alpha}= \\frac{10\\left(1+\\tan ^2 \\alpha\\right)}{\\tan \\alpha(1-\\tan \\alpha)}=\\frac{10}{\\sin \\alpha(\\cos \\alpha-\\sin \\alpha)}=\\frac{20}{\\sin 2 \\alpha+\\cos 2 \\alpha-1}=\\frac{20}{\\sqrt{2} \\sin \\left(2 \\alpha+45^{\\circ}\\right)-1}$, 所以当 $\\alpha=22.5^{\\circ}$ 时, $(A B)_{\\text {min }}= 20(\\sqrt{2}+1)$ 千米.\n此时 $O A=O B=\\frac{10}{\\sin 22.5^{\\circ}}=10 \\sqrt{4+2 \\sqrt{2}}$ (千米).", + "remark": "", + "figures": [ + "./images/volume3/figures/fig-c6p16.png" + ] +} \ No newline at end of file diff --git a/processed_dataset/calculation/0806.json b/processed_dataset/calculation/0806.json new file mode 100644 index 0000000000000000000000000000000000000000..2521b7cde681922dba3ede416dd89f81cd64fc29 --- /dev/null +++ b/processed_dataset/calculation/0806.json @@ -0,0 +1,10 @@ +{ + "source_file": "./raw_volume-zh/volume3/exercise6.tex", + "problem_type": "calculation", + "problem": "问题17. 在体育比赛中,有一种\"铁人\"项目的比赛,运动员通过跑步、划船、骑自行车等项目的比赛, 以累计成绩决定胜负.\n在这类比赛中常遇到如下情况: 运动员从 $A$ 地出发跑步到河岸渡口 $B$ 处,然后划船到河对岸 $P$ 处.\n上岸后沿河岸骑自行车到达河岸边的终点 $C$ 处.\n如果某两名运动员的跑步、划船、骑自行车的速度均相同, 那么他们如何选择登岸地点 $P$ 的位置, 才能取得胜利呢?", + "solution": "如图(),设河水流速为 $v_0$, 人在静水中划船的速度为 $v_1$, 在岸上骑车的速度为 $v_2\\left(v_2>v_1+v_0\\right)$, 河宽为 $\\left|B B^{\\prime}\\right|=d,\\left|B^{\\prime} C\\right|=s$, 船速方向 $\\overrightarrow{B D}$ 与岸的夹角为 $\\theta$, 船实际行驶方向 $\\overrightarrow{B P}\\left(v_1\\right.$ 与 $v_0$ 的合速度方向与岸的夹角为 $\\alpha$, 从 $B$ 经 $P$ 到 $C$ 所用时间为 $T$, 过 $D$ 作 $D E / / B^{\\prime} C$ 交 $B P$ 于 $E$, 则 $\\frac{v_1}{\\sin \\alpha}=\\frac{v_0}{\\sin (\\theta-\\alpha)} \\cdots$ (1).\n$T=\\frac{d}{v_1 \\sin \\theta}+\\frac{s-d \\cot \\alpha}{v_2} \\cdots$ (2). 由 (1) 得 $\\cot \\alpha=\\frac{v_1 \\cos \\theta+v_0}{v_1 \\sin \\theta}$ 代入 (2) 得 $T= \\frac{d}{v_1 \\sin \\theta}+\\frac{s v_1 \\sin \\theta-d v_1 \\cos \\theta-d v_0}{v_1 v_2 \\sin \\theta}$, 所以 $T v_1 v_2 \\sin \\theta=d v_2+s v_1 \\sin \\theta- d v_1 \\cos \\theta-d v_0$. 整理得 $\\left(T v_1 v_2-s v_1\\right) \\sin \\theta+d v_1 \\cos \\theta=d\\left(v_2-v_0\\right)$. 从而 $v_1 \\sqrt{\\left(T v_2-s\\right)^2+d^2} \\sin (\\theta+\\varphi)=d\\left(v_1-v_b\\right)$, 于是 $\\left|\\frac{d\\left(v_2-v_b\\right)}{v_1 \\sqrt{\\left(T v_2-s\\right)^2+d^2}}\\right| \\leqslant 1$. \n解得 $T \\geqslant \\frac{s}{v_2}+\\frac{d \\sqrt{\\left(v_2-v_b\\right)^2-v_1^2}}{v_1 v_2}$. 等号当且仅当 $\\sin (\\theta+\\varphi)=1$ 时成立.\n从而 $\\cos \\theta= \\sin \\varphi=\\frac{d}{\\sqrt{\\left(T v_2-s\\right)^2+d^2}}=\\frac{v_1}{v_2-v_1}, \\sin \\theta=\\cos \\varphi=\\frac{T v_2-s}{\\sqrt{\\left(T v_2-s\\right)^2+d^2}}= \\frac{\\sqrt{\\left(v_2-v_1\\right)^2-v_1^2}}{v_2-v_1}$, 所以 $\\cot \\alpha=\\frac{v_1 \\cos \\theta+v_0}{v_1 \\sin \\theta}=\\frac{v_1^2+v_2 v_0-v_0^2}{v_1 \\sqrt{\\left(v_2-v_0\\right)^2-v_1^2}}$. 故登岸的最佳地点 $P$ 随 $B^{\\prime}$ 的距离为 $\\left|B^{\\prime} P\\right|=\\frac{\\left(v_1^2+v_2 r_0-v_0^2\\right) d}{v_1 \\sqrt{\\left(v_2-r_0\\right)^2-v_1^2}}$.", + "remark": "", + "figures": [ + "./images/volume3/figures/fig-c6p17.png" + ] +} \ No newline at end of file diff --git a/processed_dataset/calculation/0807.json b/processed_dataset/calculation/0807.json new file mode 100644 index 0000000000000000000000000000000000000000..eeb844fba416e1e698d509eee7e715eeae5dec1a --- /dev/null +++ b/processed_dataset/calculation/0807.json @@ -0,0 +1,10 @@ +{ + "source_file": "./raw_volume-zh/volume3/exercise6.tex", + "problem_type": "calculation", + "problem": "问题18. 某大型装载车的车轮直径为 3 米, 车轮外沿有一污点 $A$, 当装载车以 $\\pi$ 米/秒从最初的位置开始运行时, 点 $A$ 上升到最高点需要 1 秒钟.\n(1) 求点 $A$ 离地高度 $h$ (单位: 米) 与运行时间 $t$ (单位: 秒) 的函数关系式; (2) 若此车运行距离为 50 米, 试求污点距地面高度为 $\\frac{3}{4}$ 米时的 $t$ 之值.", + "solution": "(1) 不难发现, 点 $A$ 运动的路程就是装载车运行的路程, 点 $A$ 运动一周,装载车行距离为 $3 \\pi$ 米, 由于装载车运行速度为 $\\pi$ 米/秒,故车轮运行一周需要 3 秒钟.\n点 $A$ 上升到最高点需要 1 秒钟, 表明车轮转动 $\\frac{1}{3}$ 个圆周, 即 $A$ 运行一秒钟, 转过圆心角 $\\frac{2}{3} \\pi$. 不妨设点 $A$ 沿逆时针方向转动, 根据物理中相对运动原理, 如图()所示, 我们可以把装载车的向左运动看作是车轮中心 $O$ 静止, 道路带着车轮向右运动, 这样点 $A$ 就绕固定点 $O$ 作圆周运动, 设射线 $O X$ 方向向右, 根据三角函数中角的定义及三角函数的定义, 可得 $\\sin \\left(\\frac{2 \\pi}{3} t-\\frac{\\pi}{6}\\right)=\\frac{h-\\frac{3}{2}}{\\frac{3}{2}}$, 即 $h=\\frac{3}{2} \\sin \\left(\\frac{2 \\pi}{3} t-\\frac{\\pi}{6}\\right)+\\frac{3}{2}$.\n(2) 由 $\\frac{3}{4}=\\frac{3}{2}+\\frac{3}{2} \\sin \\left(\\frac{2 \\pi}{3} t-\\frac{\\pi}{6}\\right)$, 得 $\\sin \\left(\\frac{2 \\pi}{3} t-\\frac{\\pi}{6}\\right)=-\\frac{1}{2}, \\frac{2 \\pi}{3} t-\\frac{\\pi}{6}=2 k \\pi- \\frac{\\pi}{6}$ 或 $2 k \\pi+\\frac{7}{6} \\pi$, 得 $t=3 k$ 或 $t=3 k+2$, 又因为 $0 \\leqslant t \\leqslant \\frac{50}{\\pi} \\approx 15.9$, 所以 $t=0$ 、 $2 、 3 、 5 、 6 、 8 、 9 、 11 、 12 、 14 、 15($ 秒).", + "remark": "", + "figures": [ + "./images/volume3/figures/fig-c6p18.png" + ] +} \ No newline at end of file diff --git a/processed_dataset/calculation/0808.json b/processed_dataset/calculation/0808.json new file mode 100644 index 0000000000000000000000000000000000000000..9f8d52fff1febe0be837862d95ee4360b8f427b9 --- /dev/null +++ b/processed_dataset/calculation/0808.json @@ -0,0 +1,8 @@ +{ + "source_file": "./raw_volume-zh/volume3/exercise6.tex", + "problem_type": "calculation", + "problem": "问题21. 在 $\\triangle A B C$ 中, $A C=b, A B=c, \\angle B A C=\\alpha$, 试用 $b 、 c 、 \\alpha$ 表示角 $A$ 的平分线 $A T$ 之长.", + "solution": "由 $S_{\\triangle A B T}+S_{\\triangle A T C}=S_{\\triangle A B C}$, 得 $\\frac{1}{2} \\cdot c \\cdot A T \\cdot \\sin \\frac{\\alpha}{2}+\\frac{1}{2} b \\cdot A T \\cdot \\sin \\frac{\\alpha}{2}= \\frac{1}{2} b c \\sin \\alpha$, 故 $A T=\\frac{2 b c}{b+c} \\cos \\frac{\\alpha}{2}$.", + "remark": "", + "figures": [] +} \ No newline at end of file diff --git a/processed_dataset/calculation/0809.json b/processed_dataset/calculation/0809.json new file mode 100644 index 0000000000000000000000000000000000000000..4129643041459a0ab63b3b4a5e639b34097db97e --- /dev/null +++ b/processed_dataset/calculation/0809.json @@ -0,0 +1,8 @@ +{ + "source_file": "./raw_volume-zh/volume4/chapter2-1.tex", + "problem_type": "calculation", + "problem": "例25. 设 $x, y, z \\in(0,1)$, 满足:\n$$\n\\sqrt{\\frac{1-x}{y z}}+\\sqrt{\\frac{1-y}{z x}}+\\sqrt{\\frac{1-z}{x y}}=2,\n$$\n求 $x y z$ 的最大值.", + "solution": "解:记 $u=\\sqrt[6]{x y z}$, 则由条件及均值不等式可知\n$$\n\\begin{aligned}\n2 u^3= & 2 \\sqrt{x y z}=\\frac{1}{\\sqrt{3}} \\sum \\sqrt{x(3-3 x)} \\\\\n\\leqslant & \\frac{1}{\\sqrt{3}} \\sum \\frac{x+(3-3 x)}{2} \\\\\n& =\\frac{3 \\sqrt{3}}{2}-\\frac{1}{\\sqrt{3}}(x+y+z) \\\\\n& \\leqslant \\frac{3 \\sqrt{3}}{2}-\\sqrt{3} \\cdot \\sqrt[3]{x y z} \\\\\n& =\\frac{3 \\sqrt{3}}{2}-\\sqrt{3} u^2 .\n\\end{aligned}\n$$\n故\n$$\n4 u^3+2 \\sqrt{3} u^2-3 \\sqrt{3} \\leqslant 0,\n$$\n即\n$$\n(2 u-\\sqrt{3})\\left(2 u^2+2 \\sqrt{3} u+3\\right) \\leqslant 0 .\n$$\n所以, $u \\leqslant \\frac{\\sqrt{3}}{2}$. 依此可知, $x y z \\leqslant \\frac{27}{64}$, 等号在\n$$\nx=y=z=\\frac{3}{4}\n$$\n时取得.\n因此, 所求最大值为 $\\frac{27}{64}$.", + "remark": "", + "figures": [] +} \ No newline at end of file diff --git a/processed_dataset/calculation/0810.json b/processed_dataset/calculation/0810.json new file mode 100644 index 0000000000000000000000000000000000000000..6907fe0ad46d388678a54626764e3bdcd6c8775a --- /dev/null +++ b/processed_dataset/calculation/0810.json @@ -0,0 +1,8 @@ +{ + "source_file": "./raw_volume-zh/volume4/chapter2-2.tex", + "problem_type": "calculation", + "problem": "例1. 设 $a, b, c$ 为正实数,求\n$$\n\\frac{a+3 c}{a+2 b+c}+\\frac{4 b}{a+b+2 c}-\\frac{8 c}{a+b+3 c}\n$$\n的最小值.", + "solution": "解:法一令 $x=a+2 b+c, y=a+b+2 c, z=a+b+3 c$, 则有 $x- y=b-c, z-y=c$, 由此可得 $a+3 c=2 y-x, b=z+x-2 y, c=z- y$, 从而\n$$\n\\begin{aligned}\n& \\frac{a+3 c}{a+2 b+c}+\\frac{4 b}{a+b+2 c}-\\frac{8 c}{a+b+3 c} \\\\\n= & \\frac{2 y-x}{x}+\\frac{4(z+x-2 y)}{y}-\\frac{8(z-y)}{z} \\\\\n= & -17+2 \\frac{y}{x}+4 \\frac{x}{y}+4 \\frac{z}{y}+8 \\frac{y}{z} \\\\\n\\geqslant & -17+2 \\sqrt{8}+2 \\sqrt{32}=-17+12 \\sqrt{2} .\n\\end{aligned}\n$$\n取 $a=3-2 \\sqrt{2}, b=\\sqrt{2}-1, c=\\sqrt{2}$ 时,等号成立.\n故最小值为 $-17+12 \\sqrt{2}$.", + "remark": "", + "figures": [] +} \ No newline at end of file diff --git a/processed_dataset/calculation/0811.json b/processed_dataset/calculation/0811.json new file mode 100644 index 0000000000000000000000000000000000000000..a775c74707238071255c205e742e9dbcee98d18d --- /dev/null +++ b/processed_dataset/calculation/0811.json @@ -0,0 +1,8 @@ +{ + "source_file": "./raw_volume-zh/volume4/chapter2-2.tex", + "problem_type": "calculation", + "problem": "例1. 设 $a, b, c$ 为正实数,求\n$$\n\\frac{a+3 c}{a+2 b+c}+\\frac{4 b}{a+b+2 c}-\\frac{8 c}{a+b+3 c}\n$$\n的最小值.", + "solution": "解法二不妨设 $a+b+c=1$, 则\n$$\n\\begin{aligned}\n& \\frac{a+3 c}{a+2 b+c}+\\frac{4 b}{a+b+2 c}-\\frac{8 c}{a+b+3 c} \\\\\n= & \\frac{1+2 c-b}{1+b}+\\frac{4 b}{1+c}-\\frac{8 c}{1+2 c} \\\\\n= & -1+\\frac{2+2 c}{1+b}+\\frac{4 b+4}{1+c}-\\frac{4}{1+c}+\\frac{4}{1+2 c}-4 \\\\\n= & -5+2 \\frac{1+c}{1+b}+4 \\frac{1+b}{1+c}-\\frac{4 c}{(1+c)(1+2 c)} \\\\\n\\geqslant & -5+2 \\sqrt{8}-\\frac{4}{\\frac{1}{c}+3+2 c} \\geqslant-5+4 \\sqrt{2}-\\frac{4}{3+2 \\sqrt{2}} \\\\\n= & 12 \\sqrt{2}-17 .\n\\end{aligned}\n$$\n取 $a=3-2 \\sqrt{2}, b=\\sqrt{2}-1, c=\\sqrt{2}$ 时,等号成立.\n故最小值为 $-17+12 \\sqrt{2}$.", + "remark": "", + "figures": [] +} \ No newline at end of file diff --git a/processed_dataset/calculation/0812.json b/processed_dataset/calculation/0812.json new file mode 100644 index 0000000000000000000000000000000000000000..66fc10f033acf33fae226db3563213a19a9d78da --- /dev/null +++ b/processed_dataset/calculation/0812.json @@ -0,0 +1,8 @@ +{ + "source_file": "./raw_volume-zh/volume4/chapter2-2.tex", + "problem_type": "calculation", + "problem": "例2. 设非负实数 $a$ 和 $d$, 正数 $b$ 和 $c$, 满足条件 $b+c \\geqslant a+d$, 求 $\\frac{b}{c+d}+ \\frac{c}{a+b}$ 的最小值.", + "solution": "解:不妨设 $a+b \\geqslant c+d$. 因为 $\\frac{b}{c+d}+\\frac{c}{a+b}=\\frac{b+c}{c+d}- c\\left(\\frac{1}{c+d}-\\frac{1}{a+b}\\right)$, 注意到 $c \\leqslant c+d$ 及 $b+c \\geqslant a+d \\Leftrightarrow b+c \\geqslant \\frac{1}{2}(a+b+ c+d)$. 因此, 得\n$$\n\\begin{aligned}\n\\frac{b}{c+d}+\\frac{c}{a+b} & \\geqslant \\frac{1}{2} \\frac{a+b+c+d}{c+d}-(c+d)\\left(\\frac{1}{c+d}-\\frac{1}{a+b}\\right) \\\\\n& =\\frac{1}{2} \\frac{a+b}{c+d}+\\frac{c+d}{a+b}-\\frac{1}{2} \\geqslant 2 \\sqrt{\\frac{a+b}{2(c+d)} \\frac{c+d}{a+b}}-\\frac{1}{2} \\\\\n& =\\sqrt{2}-\\frac{1}{2} .\n\\end{aligned}\n$$\n等号成立当且仅当 $a=\\sqrt{2}+1, b=\\sqrt{2}-1, c=2, d=0$, 所以 $\\frac{b}{c+d}+ \\frac{c}{a+b}$ 的最小值为 $\\sqrt{2}-\\frac{1}{2}$.", + "remark": "", + "figures": [] +} \ No newline at end of file diff --git a/processed_dataset/calculation/0813.json b/processed_dataset/calculation/0813.json new file mode 100644 index 0000000000000000000000000000000000000000..c146312dafb4ac6df0b840ac4ccb81c832d59e74 --- /dev/null +++ b/processed_dataset/calculation/0813.json @@ -0,0 +1,8 @@ +{ + "source_file": "./raw_volume-zh/volume4/chapter2-2.tex", + "problem_type": "calculation", + "problem": "例3. 设 $2 x>3 y>0$, 求 $\\sqrt{2} x^3+\\frac{3}{2 x y-3 y^2}$ 的最小值.", + "solution": "解:因为 $2 x>3 y>0$, 所以 $2 x-3 y>0$. 由平均值不等式, 得\n$$\n\\begin{aligned}\n2 x y-3 y^2 & =y(2 x-3 y)=\\frac{1}{3} \\cdot 3 y(2 x-3 y) \\\\\n& \\leqslant \\frac{1}{3} \\cdot\\left[\\frac{3 y+(2 x-3 y)}{2}\\right]^2=\\frac{1}{3} x^2 .\n\\end{aligned}\n$$\n所以\n$$\n\\begin{aligned}\n& \\sqrt{2} x^3+\\frac{3}{2 x y-3 y^2} \\geqslant \\sqrt{2} x^3+\\frac{9}{x^2} \\\\\n= & \\frac{\\sqrt{2}}{2} x^3+\\frac{\\sqrt{2}}{2} x^3+\\frac{3}{x^2}+\\frac{3}{x^2}+\\frac{3}{x^2} \\\\\n\\geqslant & 5 \\sqrt[5]{\\left(\\frac{\\sqrt{2}}{2} x^3\\right)^2 \\cdot\\left(\\frac{3}{x^2}\\right)^3}=5 \\sqrt[5]{\\frac{27}{2}} .\n\\end{aligned}\n$$\n等号成立当且仅当 $3 y=2 x-3 y, \\frac{\\sqrt{2}}{2} x^3=\\frac{3}{x^2}$, 即 $x=18^{\\frac{1}{10}}, y=\\frac{1}{3} \\cdot 18^{\\frac{1}{10}}$ 时取到.\n因此, $\\sqrt{2} x^3+\\frac{3}{2 x y-3 y^2}$ 的最小值为 $5 \\sqrt[5]{\\frac{27}{2}}$.", + "remark": "", + "figures": [] +} \ No newline at end of file diff --git a/processed_dataset/calculation/0814.json b/processed_dataset/calculation/0814.json new file mode 100644 index 0000000000000000000000000000000000000000..86447b94faf187f5b47fec8b980df160feb35bf2 --- /dev/null +++ b/processed_dataset/calculation/0814.json @@ -0,0 +1,8 @@ +{ + "source_file": "./raw_volume-zh/volume4/chapter2-2.tex", + "problem_type": "calculation", + "problem": "例6. 已知两两不同的正整数 $a, b, c, d, e, f, g, h, n$ 满足\n$$\nn=a b+c d=e f+g h \\text {. }\n$$\n求 $n$ 的最小值.", + "solution": "解:若 $a, b, c, d, e, f, g, h$ 中没有一个等于 1 , 则\n$$\n\\begin{aligned}\n2 n & =a b+c d+e f+g h \\\\\n& \\geqslant 4 \\sqrt[4]{a b c d e f g h} \\\\\n& \\geqslant 4 \\sqrt[4]{2 \\times 3 \\times 4 \\times 5 \\times 6 \\times 7 \\times 8 \\times 9} \\\\\n& =4 \\sqrt[4]{2^7 \\times 3^4 \\times 5 \\times 7} \\\\\n& =4 \\times 4 \\times 3 \\times \\sqrt[4]{\\frac{35}{2}} \\\\\n& >4 \\times 4 \\times 3 \\times 2=96 .\n\\end{aligned}\n$$\n所以 $n \\geqslant 48$. 设 $a, b, c, d, e, f, g, h$ 中有一个等于 1 , 不妨设 $h=1$, 则 $2 n=a b+c d+e f+g$, 且存在最小值.\n此时 $g$ 一定是这些数中最大的一个.\n于是有\n$$\n\\begin{aligned}\n2 n & =a b+c d+e f+g \\\\\n& \\geqslant g+3 \\sqrt[3]{a b c d e f} \\\\\n& \\geqslant g+3 \\sqrt[3]{2 \\times 3 \\times 4 \\times 5 \\times 6 \\times 7} \\\\\n& =g+3 \\sqrt[3]{5040} \\\\\n& >g+3 \\sqrt[3]{4913}=g+51 .\n\\end{aligned}\n$$\n因为 $g \\geqslant 8$, 所以, $2 n \\geqslant 60, n \\geqslant 30$.\n如果 $g \\geqslant 9$, 则 $2 n \\geqslant 61, n \\geqslant 31$.\n假设 $n=30$, 则 $g=8$. 于是, $a, b, c, d, e, f, g, h$ 是集合 $\\{1,2,3,4$ , $5,6,7,8\\}$ 的一个排列, 特别地, 有一个数是 5 , 不妨设 $a=5$. 所以 $30=a b+ c d$, 即 $c d$ 可以被 5 整除.\n矛盾.\n因此 $n \\geqslant 31$. 又因为 $31=1 \\times 7+4 \\times 6=2 \\times 8+3 \\times 5$, 因此 $n$ 的最小值为 31 .", + "remark": "", + "figures": [] +} \ No newline at end of file diff --git a/processed_dataset/calculation/0815.json b/processed_dataset/calculation/0815.json new file mode 100644 index 0000000000000000000000000000000000000000..94678ac1bbc5d4fd50230c785960d08e6e83820c --- /dev/null +++ b/processed_dataset/calculation/0815.json @@ -0,0 +1,8 @@ +{ + "source_file": "./raw_volume-zh/volume4/chapter2-2.tex", + "problem_type": "calculation", + "problem": "例8. 设 $a, b, c \\in \\mathbf{R}^{+}$, 满足 $a+b+c=a b c$. 求 $a^7(b c-1)+ b^7(a c-1)+c^7(a b-1)$ 的最小值.", + "solution": "解:因为 $a, b, c>0$, 且 $a+b+c=a b c$, 所以 $c(a b-1)=a+b$.\n同理可得 $b(a c-1)=a+c, a(b c-1)=b+c$.\n由平均值不等式, 得\n$$\na b c=a+b+c \\geqslant 3 \\sqrt[3]{a b c},\n$$\n推出 $a b c \\geqslant 3 \\sqrt{3}$, 等号成立当且仅当 $a=b=c=\\sqrt{3}$, 所以\n$$\n\\begin{aligned}\n& a^7(b c-1)+b^7(a c-1)+c^7(a b-1) \\\\\n= & a^6(b+c)+b^6(a+c)+c^5(a+b) \\\\\n\\geqslant & 6 \\sqrt[6]{a^6 b a^6 c b^6 a b^6 c c^6 a c^6 b}=6 \\sqrt[6]{a^{14} b^{14} c^{14}}=6(a b c)^{\\frac{7}{3}} \\\\\n\\geqslant & 6(\\sqrt{3})^7=6 \\times 27 \\sqrt{3}=162 \\sqrt{3},\n\\end{aligned}\n$$\n等号成立的充要条件是 $a=b=c=\\sqrt{3}$, 故所求的最小值为 $162 \\sqrt{3}$.", + "remark": "", + "figures": [] +} \ No newline at end of file diff --git a/processed_dataset/calculation/0816.json b/processed_dataset/calculation/0816.json new file mode 100644 index 0000000000000000000000000000000000000000..2d08f98a0806f170a135bcc93f133f570a7c8dd0 --- /dev/null +++ b/processed_dataset/calculation/0816.json @@ -0,0 +1,8 @@ +{ + "source_file": "./raw_volume-zh/volume4/chapter2-2.tex", + "problem_type": "calculation", + "problem": "例11. 对 $a, b, c \\in \\mathbf{R}^{+}$, 求\n$$\n\\frac{(a+b)^2+(a+b+4 c)^2}{a b c}(a+b+c)\n$$\n的最小值.", + "solution": "解:由平均值不等式, 得\n$$\n\\begin{aligned}\n(a+b)^2+(a+b+4 c)^2 & =(a+b)^2+[(a+2 c)+(b+2 c)]^2 \\\\\n& \\geqslant(2 \\sqrt{a b})^2+(2 \\sqrt{2 a c}+2 \\sqrt{2 b c})^2 \\\\\n& =4 a b+8 a c+8 b c+16 c \\sqrt{a b},\n\\end{aligned}\n$$\n于是 $\\quad \\frac{(a+b)^2+(a+b+4 c)^2}{a b c} \\cdot(a+b+c)$\n$$\n\\begin{aligned}\n& \\geqslant \\frac{4 a b+8 a c+8 b c+16 c \\sqrt{a b}}{a b c} \\cdot(a+b+c) \\\\\n& =\\left(\\frac{4}{c}+\\frac{8}{b}+\\frac{8}{a}+\\frac{16}{\\sqrt{a b}}\\right)(a+b+c) \\\\\n& =8\\left(\\frac{1}{2 c}+\\frac{1}{b}+\\frac{1}{a}+\\frac{1}{\\sqrt{a b}}+\\frac{1}{\\sqrt{a b}}\\right)\\left(\\frac{a}{2}+\\frac{a}{2}+\\frac{b}{2}+\\frac{b}{2}+c\\right) \\\\\n& \\geqslant 8\\left(5 \\sqrt[5]{\\frac{1}{2 a^2 b^2 c}}\\right)\\left(5 \\sqrt[5]{\\frac{a^2 b^2 c}{2^4}}\\right)=100 .\n\\end{aligned}\n$$\n当 $a=b=2 c>0$ 时取等号.\n故所求最小值为 100 .", + "remark": "", + "figures": [] +} \ No newline at end of file diff --git a/processed_dataset/calculation/0817.json b/processed_dataset/calculation/0817.json new file mode 100644 index 0000000000000000000000000000000000000000..0348e480f4d8ee4ec63c08a635de1ccac2f9085d --- /dev/null +++ b/processed_dataset/calculation/0817.json @@ -0,0 +1,8 @@ +{ + "source_file": "./raw_volume-zh/volume4/chapter2-3.tex", + "problem_type": "calculation", + "problem": "例5. 设 $P$ 为 $\\triangle A B C$ 内一点, $D, E, F$ 分别为 $P$ 到 $B C, C A, A B$ 各边的垂足.\n试确定点 $P$, 使 $P D \\times P E \\times P F$ 最大.", + "solution": "解:记 $\\triangle A B C$ 的三个内角为 $A, B, C$, 其对边为 $a, b, c$. 记 $\\triangle A B C$ 的面积为 $S$ (以下各题记号均同, 不再注明).\n设 $P D=x, P E=y, P F=z$. 连结 $A P, B P, C P$. 易知 $S_1(\\triangle P B C$ 面积 $)=\\frac{1}{2} a x, S_2(\\triangle P C A$ 面积 $)=\\frac{1}{2} b y, S_3(\\triangle P A B$ 面积 $)=\\frac{1}{2} c z$.\n从而有 $a x+b y+c z=2\\left(S_1+S_2+S_3\\right)=2 S=$ 定值.\n由平均值不等式, 得\n$$\n\\begin{gathered}\na x \\cdot b y \\cdot c z \\leqslant\\left(\\frac{a x+b y+c z}{3}\\right)^3=\\left(\\frac{2 S}{3}\\right)^3, \\\\\nx y z \\leqslant \\frac{8 S^3}{27 a b c} .\n\\end{gathered}\n$$\n即上式等号当且仅当 $a x=b y=c z$ 时成立.\n这就是说, $S_1=S_2=S_3=\\frac{1}{3} S$ 使得 $x y z$ 取最大.\n这时 $P$ 为 $\\triangle A B C$ 的重心.", + "remark": "", + "figures": [] +} \ No newline at end of file diff --git a/processed_dataset/calculation/0818.json b/processed_dataset/calculation/0818.json new file mode 100644 index 0000000000000000000000000000000000000000..06f9ebb100ab48af482b58c034b5f7ac1bdc2324 --- /dev/null +++ b/processed_dataset/calculation/0818.json @@ -0,0 +1,8 @@ +{ + "source_file": "./raw_volume-zh/volume4/chapter2-4.tex", + "problem_type": "calculation", + "problem": "例7. 设 $a_i \\in \\mathbf{R}^{+}, i=1,2, \\cdots, n$, 且 $\\sum_{i=1}^n a_i=1$, 求\n$$\nM=\\sum_{i=1}^n \\frac{a_i}{1+\\sum_{j \\neq i, j=1}^n a_j}\n$$\n的最小值.", + "solution": "解:由 $M+n=\\left(\\frac{a_1}{2-a_1}+1\\right)+\\left(\\frac{a_2}{2-a_2}+1\\right)+\\cdots+\\left(\\frac{a_n}{2-a_n}+1\\right)$\n$$\n\\begin{aligned}\n& =\\frac{2}{2-a_1}+\\frac{2}{2-a_2}+\\cdots+\\frac{2}{2-a_n}\\left(\\text { 由 } H_n \\leqslant A_n\\right) \\\\\n& \\geqslant \\frac{n^2}{\\frac{1}{2}\\left(2-a_1\\right)+\\frac{1}{2}\\left(2-a_2\\right)+\\cdots+\\frac{1}{2}\\left(2-a_n\\right)} \\\\\n& =\\frac{n^2}{\\frac{1}{2}(2 n-1)}=\\frac{2 n^2}{2 n-1} .\n\\end{aligned}\n$$\n所以 $M \\geqslant \\frac{n}{2 n-1}$, 当且仅当 $a_1=a_2=\\cdots=a_n=\\frac{1}{n}$ 时, $M=\\frac{n}{2 n-1}$.\n于是 $M$ 的最小值为 $\\frac{n}{2 n-1}$.", + "remark": "", + "figures": [] +} \ No newline at end of file diff --git a/processed_dataset/calculation/0819.json b/processed_dataset/calculation/0819.json new file mode 100644 index 0000000000000000000000000000000000000000..611963cdf1d6df91dd4be8f4980797cd812e3773 --- /dev/null +++ b/processed_dataset/calculation/0819.json @@ -0,0 +1,8 @@ +{ + "source_file": "./raw_volume-zh/volume4/chapter2-4.tex", + "problem_type": "calculation", + "problem": "例9. 给定正整数 $k$, 当 $x^k+y^k+z^k=1$ 时, 求 $x^{k+1}+y^{k+1}+z^{k+1}$ 的最小值.", + "solution": "解:由假设和幕平均不等式, 得\n$$\n\\left(\\frac{x^{k+1}+y^{k+1}+z^{k+1}}{3}\\right)^{\\frac{1}{k+1}} \\geqslant\\left(\\frac{x^k+y^k+z^k}{3}\\right)^{\\frac{1}{k}}=\\left(\\frac{1}{3}\\right)^{\\frac{1}{k}},\n$$\n所以\n$$\nx^{k+1}+y^{k+1}+z^{k+1} \\geqslant 3\\left(\\frac{1}{3}\\right)^{\\frac{k+1}{k}}=3^{-\\frac{1}{k}} \\text {. }\n$$\n当 $x=y=z=3^{-\\frac{1}{k}}$ 时等号成立, 所以最小值为 $3^{-\\frac{1}{k}}$.", + "remark": "", + "figures": [] +} \ No newline at end of file diff --git a/processed_dataset/calculation/0820.json b/processed_dataset/calculation/0820.json new file mode 100644 index 0000000000000000000000000000000000000000..d02bf875d81e1cb5a7e4633474633006a6aceaae --- /dev/null +++ b/processed_dataset/calculation/0820.json @@ -0,0 +1,8 @@ +{ + "source_file": "./raw_volume-zh/volume4/chapter2-5.tex", + "problem_type": "calculation", + "problem": "例1. 设 $a, b, c, d$ 是不全为 0 的实数,求\n$$\nf=\\frac{a b+2 b c+c d}{a^2+b^2+c^2+d^2}\n$$\n的最大值.", + "solution": "解:如果假设 $f$ 的最大值为 $M$, 则\n$$\na b+2 b c+c d \\leqslant M\\left(a^2+b^2+c^2+d^2\\right),\n$$\n因此, 要建立一个上面形式的不等式, 并找一组 $a, b, c, d$ 的值, 使不等式等号成立.\n设 $\\alpha, \\beta, \\gamma>0$, 则\n$$\n\\frac{\\alpha}{2} a^2+\\frac{b^2}{2 \\alpha} \\geqslant a b, \\beta b^2+\\frac{c^2}{\\beta} \\geqslant 2 b c, \\frac{\\gamma}{2} c^2+\\frac{d^2}{2 \\gamma} \\geqslant c d .\n$$\n将上面三式相加, 得\n$$\n\\begin{gathered}\n\\frac{\\alpha}{2} a^2+\\left(\\frac{1}{2 \\alpha}+\\beta\\right) b^2+\\left(\\frac{1}{\\beta}+\\frac{\\gamma}{2}\\right) c^2+\\frac{d^2}{2 \\gamma} \\\\\n\\geqslant a b+2 b c+c d\n\\end{gathered}\n$$\n令 $\\frac{\\alpha}{2}=\\frac{1}{2 \\alpha}+\\beta=\\frac{1}{\\beta}+\\frac{\\gamma}{2}=\\frac{1}{2 \\gamma}$, 得 $\\gamma=\\frac{1}{\\alpha}, \\beta=\\frac{1}{2}\\left(\\alpha-\\frac{1}{\\alpha}\\right)$, 及 $\\alpha^4- 6 \\alpha^2+1=0$, 解方程得 $\\alpha=\\sqrt{2}+1$ (另外三个解不合要求), 于是\n$$\n\\begin{gathered}\n\\frac{\\sqrt{2}+1}{2}\\left(a^2+b^2+c^2+d^2\\right) \\geqslant a b+2 b c+c d, \\\\\nf \\leqslant \\frac{\\sqrt{2}+1}{2} .\n\\end{gathered}\n$$\n又当 $a=d=1, b=c=\\sqrt{2}+1$ 时, 不等式等号成立, 故 $f$ 的最大值为 $\\frac{\\sqrt{2}+1}{2}$.", + "remark": "", + "figures": [] +} \ No newline at end of file diff --git a/processed_dataset/calculation/0821.json b/processed_dataset/calculation/0821.json new file mode 100644 index 0000000000000000000000000000000000000000..2733aee85be511d298ea47e2826d00de016b5fef --- /dev/null +++ b/processed_dataset/calculation/0821.json @@ -0,0 +1,8 @@ +{ + "source_file": "./raw_volume-zh/volume4/chapter2-5.tex", + "problem_type": "calculation", + "problem": "例2. 求出最大的正数 $\\lambda$, 使得对于满足 $x^2+y^2+z^2=1$ 的任何实数 $x$, $y, z$ 成立不等式:\n$$\n|\\lambda x y+y z| \\leqslant \\frac{\\sqrt{5}}{2} .\n$$", + "solution": "解:由于\n$$\n\\begin{aligned}\n1 & =x^2+y^2+z^2=x^2+\\frac{\\lambda^2}{1+\\lambda^2} y^2+\\frac{1}{1+\\lambda^2} y^2+z^2 \\\\\n& \\geqslant \\frac{2}{\\sqrt{1+\\lambda^2}}(\\lambda|x y|+|y z|) \\geqslant \\frac{2}{\\sqrt{1+\\lambda^2}}(|\\lambda x y+y z|),\n\\end{aligned}\n$$\n且当 $y=\\frac{\\sqrt{2}}{2}, x=\\frac{\\sqrt{2} \\lambda}{2 \\sqrt{\\lambda^2+1}}, z=\\frac{\\sqrt{2}}{2 \\sqrt{\\lambda^2+1}}$ 时, 上述两个等号可同时取到.\n因此 $\\frac{\\sqrt{1+\\lambda^2}}{2}$ 是 $|\\lambda x y+y z|$ 的最大值.\n令 $\\frac{\\sqrt{1+\\lambda^2}}{2} \\leqslant \\frac{\\sqrt{5}}{2}$, 解得 $|\\lambda| \\leqslant 2$.\n故 $\\lambda$ 的最大值为 2 .", + "remark": "", + "figures": [] +} \ No newline at end of file diff --git a/processed_dataset/calculation/0822.json b/processed_dataset/calculation/0822.json new file mode 100644 index 0000000000000000000000000000000000000000..f9559a6c1067fd03bec654f2469e37b13bfb42e8 --- /dev/null +++ b/processed_dataset/calculation/0822.json @@ -0,0 +1,8 @@ +{ + "source_file": "./raw_volume-zh/volume4/chapter2-5.tex", + "problem_type": "calculation", + "problem": "例3. 已知 $\\alpha, \\beta, \\gamma>0$, 且\n$$\n\\frac{1}{\\alpha^2+1}+\\frac{1}{\\beta^2+1}+\\frac{1}{\\gamma^2+1}=1,\n$$\n求函数 $u=\\frac{\\alpha x y+\\beta y z+\\gamma z x}{x^2+y^2+z^2}$ 的最大值.", + "solution": "解:对于任意正实数 $a, b, c$, 有\n$$\n\\begin{aligned}\n& x^2+y^2+z^2 \\\\\n= & \\frac{b}{b+c} x^2+\\frac{c}{b+c} x^2+\\frac{a}{a+c} y^2+\\frac{c}{a+c} y^2+\\frac{a}{a+b} z^2+\\frac{b}{a+b} z^2 \\\\\n= & \\left(\\frac{b}{b+c} x^2+\\frac{a}{a+c} y^2\\right)+\\left(\\frac{c}{c+a} y^2+\\frac{b}{b+a} z^2\\right)+\\left(\\frac{a}{a+b} z^2+\\frac{c}{c+b} x^2\\right) \\\\\n\\geqslant & 2\\left(\\sqrt{\\frac{a b}{(a+c)(b+c)}} x y+\\sqrt{\\frac{b c}{(b+a)(c+a)}} y z+\\sqrt{\\frac{c a}{(c+b)(a+b)}} z x\\right) \\\\\n= & 2 \\sqrt{\\frac{a b c}{(a+b)(b+c)(c+a)}}\\left(\\sqrt{\\frac{a+b}{c}} x y+\\sqrt{\\frac{b+c}{a}} y z+\\sqrt{\\frac{c+a}{b}} z x\\right) \\\\\n& \\left\\{\\begin{array}{l}\n\\sqrt{\\frac{b}{b+c}} x=\\sqrt{\\frac{a}{a+c}} y, \\\\\n\\sqrt{\\frac{c}{c+a}} y=\\sqrt{\\frac{b}{b+a}} z, \\\\\n\\sqrt{\\frac{a}{a+b}} z=\\sqrt{\\frac{c}{c+b}} x,\n\\end{array}\\right.\n\\end{aligned}\n$$\n当且仅当\n$$\n\\left\\{\\begin{array}{l}\n\\sqrt{\\frac{b}{b+c} x}=\\sqrt{\\frac{a}{a+c} y,} \\\\\n\\sqrt{\\frac{c}{c+a} y}=\\sqrt{\\frac{b}{b+a}} z, \\\\\n\\sqrt{\\frac{a}{a+b} z}=\\sqrt{\\frac{c}{c+b}} x,\n\\end{array}\\right.\n$$\n亦即 $\\frac{x}{\\sqrt{a b+a c}}=\\frac{y}{\\sqrt{b a+b c}}=\\frac{z}{\\sqrt{c a+c b}}$ 时, 上式取等号.\n$$\n\\begin{array}{r}\n\\text { 令 } \\alpha=\\sqrt{\\frac{a+b}{c}}, \\beta=\\sqrt{\\frac{b+c}{a}}, \\gamma=\\sqrt{\\frac{c+a}{b}}, \\text { 则 } \\\\\n\\frac{1}{\\alpha^2+1}+\\frac{1}{\\beta^2+1}+\\frac{1}{\\gamma^2+1}=1,\n\\end{array}\n$$\n且\n$$\n\\begin{gathered}\nx^2+y^2+z^2 \\geqslant \\frac{2}{\\alpha \\beta \\gamma}(\\alpha x y+\\beta y z+\\gamma z x), \\\\\n\\frac{\\alpha x y+\\beta y z+\\gamma z x}{x^2+y^2+z^2} \\leqslant \\frac{\\alpha \\beta \\gamma}{2},\n\\end{gathered}\n$$\n所以, $u$ 的最大值为 $\\frac{\\alpha \\beta \\gamma}{2}$.", + "remark": "注:(1) 如果 $\\frac{1}{\\alpha^2+k}+\\frac{1}{\\beta^2+k}+\\frac{1}{\\gamma^2+k}=\\frac{1}{k}(\\alpha, \\beta, \\gamma, k>0)$, 那么 $u= \\frac{\\alpha x y+\\beta y z+\\gamma z x}{x^2+y^2+z^2}$ 有最大值 $\\frac{\\alpha \\beta y}{2 k}$.\n这是因为\n$$\n\\begin{gathered}\n\\frac{1}{\\left(\\frac{\\alpha}{\\sqrt{k}}\\right)^2+1}+\\frac{1}{\\left(\\frac{\\beta}{\\sqrt{k}}\\right)^2+1}+\\frac{1}{\\left(\\frac{\\gamma}{\\sqrt{k}}\\right)^2+1}=1, \\\\\n\\frac{\\frac{\\alpha}{\\sqrt{k}} x y+\\frac{\\beta}{\\sqrt{k}} y z+\\frac{\\gamma}{\\sqrt{k}} z x}{x^2+y^2+z^2} \\leqslant \\frac{\\frac{\\alpha}{\\sqrt{k}} \\cdot \\frac{\\beta}{\\sqrt{k}} \\cdot \\frac{\\gamma}{\\sqrt{k}}}{2}, \\\\\n\\frac{\\alpha x y+\\beta y z+\\gamma z x}{x^2+y^2+z^2} \\leqslant \\frac{\\alpha \\beta \\gamma}{2 k} .\n\\end{gathered}\n$$\n化简整理后 $\\quad \\frac{\\alpha x y+\\beta y z+\\gamma z x}{x^2+y^2+z^2} \\leqslant \\frac{\\alpha \\beta \\gamma}{2 k}$.\n(2) 若 $\\frac{k_1 k_2}{\\alpha^2+k_1 k_2 k}+\\frac{k_2 k_3}{\\beta^2+k_2 k_3 k}+\\frac{k_1 k_3}{\\gamma^2+k_1 k_3 k}=\\frac{1}{k}\\left(\\alpha, \\beta, \\gamma, k_1, k_2, k_3\\right.$, $k>0)$, 则函数 $\\frac{\\alpha x y+\\beta y z+\\gamma z x}{x^2+y^2+z^2}$ 有最大值 $\\frac{\\alpha \\beta \\gamma}{2 k_1 k_2 k_3 k}$.\n事实上, 只须令 $x^{\\prime}=\\sqrt{k_1} x, y^{\\prime}=\\sqrt{k_2} y, z^{\\prime}=\\sqrt{k_3} z, \\alpha^{\\prime}=\\frac{\\alpha}{\\sqrt{k_1 k_2}}$, $\\beta^{\\prime}=\\frac{\\beta}{\\sqrt{k_2 k_3}}, \\gamma^{\\prime}=\\frac{\\gamma}{\\sqrt{k_1 k_3}}$ 即可化归为 (1) 的情形.", + "figures": [] +} \ No newline at end of file diff --git a/processed_dataset/calculation/0823.json b/processed_dataset/calculation/0823.json new file mode 100644 index 0000000000000000000000000000000000000000..f994aed3c262eebe3ac052c79b65143ed171b9ca --- /dev/null +++ b/processed_dataset/calculation/0823.json @@ -0,0 +1,8 @@ +{ + "source_file": "./raw_volume-zh/volume4/chapter2-5.tex", + "problem_type": "calculation", + "problem": "例4. 设 $a$ 为实数, 求函数 $f(x)=|\\sin x(a+\\cos x)|(x \\in \\mathbf{R})$ 的最大值.", + "solution": "解:设 $\\alpha$ 为参数, 使得\n$$\n\\begin{aligned}\nf^2(x) & =\\frac{1}{\\alpha^2} \\sin ^2 x(a \\alpha+\\alpha \\cos x)^2 \\leqslant \\frac{1}{\\alpha^2} \\sin ^2 x\\left(\\alpha^2+\\cos ^2 x\\right)\\left(a^2+\\alpha^2\\right) \\\\\n& \\leqslant \\frac{1}{\\alpha^2}\\left(\\frac{\\sin ^2 x+\\alpha^2+\\cos ^2 x}{2}\\right)^2\\left(a^2+\\alpha^2\\right)=\\frac{1}{\\alpha^2}\\left(\\frac{\\alpha^2+1}{2}\\right)^2\\left(a^2+\\alpha^2\\right),\n\\end{aligned}\n$$\n当且仅当 $\\alpha^2=a \\cos x, \\sin ^2 x=\\alpha^2+\\cos ^2 x$ 时等号成立.\n消除 $x$, 得 $2 \\alpha^4+a^2 \\alpha^2-a^2=0$.\n解方程, 得 $\\alpha^2=\\frac{1}{4}\\left(\\sqrt{a^4+8 a^2}-a^2\\right)$, 从而 $\\cos x=\\frac{1}{4}\\left(\\sqrt{a^2+8}-a\\right)$.\n所以当 $x=2 k \\pi \\pm \\arccos \\left[\\frac{1}{4}\\left(\\sqrt{a^2+8}-a\\right)\\right](k \\in \\mathbf{Z})$ 时,\n$$\nf(x)_{\\max }=\\frac{\\sqrt{a^4+8 a^2}-a^2+4}{8} \\cdot \\sqrt{\\frac{\\sqrt{a^4+8 a^2}+a^2+2}{2}} .\n$$", + "remark": "", + "figures": [] +} \ No newline at end of file diff --git a/processed_dataset/calculation/0824.json b/processed_dataset/calculation/0824.json new file mode 100644 index 0000000000000000000000000000000000000000..277262334c1a63570a912027bb18ed0f632bd98d --- /dev/null +++ b/processed_dataset/calculation/0824.json @@ -0,0 +1,8 @@ +{ + "source_file": "./raw_volume-zh/volume4/chapter2-5.tex", + "problem_type": "calculation", + "problem": "例5. 设 $x, y, z \\in \\mathbf{R}^{+}$, 且 $x^4+y^4+z^4=1$, 求\n$$\nf(x, y, z)=\\frac{x^3}{1-x^8}+\\frac{y^3}{1-y^8}+\\frac{z^3}{1-z^8}\n$$\n的最小值.", + "solution": "解:将原式变形为\n$$\nf(x, y, z)=\\frac{x^4}{x\\left(1-x^8\\right)}+\\frac{y^4}{y\\left(1-y^8\\right)}+\\frac{z^4}{z\\left(1-z^8\\right)} .\n$$\n对于 $w \\in(0,1)$, 令 $\\phi(w)=w\\left(1-w^8\\right)$, 先求 $\\phi(w)$ 的最大值.\n选一参数 $a$, 并利用 $G_9 \\leqslant A_9$, 得\n$$\n\\begin{aligned}\na(\\phi(w))^8 & =a w^8\\left(1-w^8\\right)^8 \\\\\n& \\leqslant\\left[\\frac{1}{9}\\left(a w^8+8\\left(1-w^8\\right)\\right)\\right]^9 \\\\\n& =\\left[\\frac{1}{9}\\left(8+(a-8) w^8\\right)\\right]^9 .\n\\end{aligned}\n$$\n取 $a=8$, 得\n$$\n8(\\phi(w))^8 \\leqslant\\left(\\frac{8}{9}\\right)^9\n$$\n由于 $\\phi(w)>0$, 从而\n$$\n\\phi(w) \\leqslant \\frac{8}{\\sqrt[4]{3^9}}\n$$\n于是\n$$\nf(x, y, z) \\geqslant \\frac{x^4+y^4+z^4}{8} \\cdot \\sqrt[4]{3^9}=\\frac{9}{8} \\sqrt[4]{3},\n$$\n当 $x=y=z=\\frac{1}{\\sqrt[4]{3}}$ 时, 等号成立.\n故 $f(x, y, z)$ 的最小值为 $\\frac{9}{8} \\sqrt[4]{3}$.", + "remark": "注:这里选择 $a=8$, 是为了消除变量 $w^8$, 使得右边为常数.", + "figures": [] +} \ No newline at end of file diff --git a/processed_dataset/calculation/0825.json b/processed_dataset/calculation/0825.json new file mode 100644 index 0000000000000000000000000000000000000000..2fb5824784a78cabdf218cc48fa89ff8e9d0f05a --- /dev/null +++ b/processed_dataset/calculation/0825.json @@ -0,0 +1,8 @@ +{ + "source_file": "./raw_volume-zh/volume4/chapter4-2.tex", + "problem_type": "calculation", + "problem": "例1. 求方程组\n$$\n\\left\\{\\begin{array}{l}\na^2=\\frac{\\sqrt{b c} \\sqrt[3]{b c} \\overline{(b+c)(b+c+d)}} \\label{eq1}\\\\\n{b^2}=\\frac{\\sqrt{c d} \\sqrt[3]{c d a}}{(c+d)(c+d+a)} \\label{eq2}\\\\\nc^2=\\frac{\\sqrt{d a} \\sqrt[3]{d a b}}{(d+a)(d+a+b)} \\label{eq3}\\\\\nd^2=\\frac{\\sqrt{a b} \\sqrt[3]{a b c}}{(a+b)(a+b+c)} . \\label{eq4}\n\\end{array}\\right.\n$$\n的实数解.", + "solution": "解:首先, 注意到没有一个变量等于零.\n不失一般性, 假设 $b=0$, 由 式\\ref{eq1} 得 $a=0$, 由 式\\ref{eq4} 得 $d=0$, 由 式\\ref{eq3} 得 $c=0$, 这就意味着所有值为零, 但这是不可能的, 因为分母会为零.\n其次, 注意到 $b c, c d, d a, a b$ 的平方根一定都存在, 这就表明 $a, b, c, d$ 一定都是负数或都是正数.\n如果都是负数, 这些方程的右边是负的, 与它们是实数的平方相矛盾, 由此可得 4 个值一定都是正的.\n根据算术一几何平均值不等式, 有\n$$\n\\sqrt{b c} \\leqslant \\frac{b+c}{2} \\text {, 即 } \\frac{\\sqrt{b c}}{b+c} \\leqslant \\frac{1}{2} \\text { 和 } \\sqrt[3]{b c d} \\leqslant \\frac{b+c+d}{3},\n$$\n即\n$$\n\\frac{\\sqrt[3]{b c d}}{b+c+d} \\leqslant \\frac{1}{3}\n$$\n因此 $\\quad a^2=\\frac{\\sqrt{b c} \\sqrt[3]{b c d}}{(b+c)(b+c+d)} \\leqslant \\frac{1}{2} \\times \\frac{1}{3}=\\frac{1}{6}$.\n从而 $a \\leqslant \\frac{1}{\\sqrt{6}}$.\n类似地, 有 $b \\leqslant \\frac{1}{\\sqrt{6}}, c \\leqslant \\frac{1}{\\sqrt{6}}, d \\leqslant \\frac{1}{\\sqrt{6}}$.\n由此得 $(b+c)(b+c+d) \\leqslant \\frac{2}{\\sqrt{6}} \\times \\frac{3}{\\sqrt{6}}=1$.\n同样地,有\n$$\n\\begin{aligned}\n& (c+d)(c+d+a) \\leqslant 1, \\\\\n& (d+a)(d+a+b) \\leqslant 1, \\\\\n& (a+b)(a+b+c) \\leqslant 1 .\n\\end{aligned}\n$$\n由 $式\\ref{eq1} \\times 式\\ref{eq2} \\times式\\ref{eq3}\\times式\\ref{eq4}$, 可得\n$$\n1=(b+c)(b+c+d)(c+d)(c+d+a) \\cdot(d+a)(d+a+b)(a+b)(a+b+c) \\text {. }\n$$\n因为 4 个小于或等于 1 的表达式的积等于 1 , 那么, 这 4 个表达式一定都等于 1 .\n从而唯一的可能是每个变量取它的最大的可能值.\n因此 $a=b=c=d=\\frac{\\sqrt{6}}{6}$ 为给定方程组的唯一解.", + "remark": "", + "figures": [] +} \ No newline at end of file diff --git a/processed_dataset/calculation/0826.json b/processed_dataset/calculation/0826.json new file mode 100644 index 0000000000000000000000000000000000000000..f6b2d9eebe8dd3a2fa697a2968f5092242e82b1c --- /dev/null +++ b/processed_dataset/calculation/0826.json @@ -0,0 +1,8 @@ +{ + "source_file": "./raw_volume-zh/volume4/chapter4-2.tex", + "problem_type": "calculation", + "problem": "例2. 已知实数 $x 、 y 、 z>3$, 求方程\n$$\n\\frac{(x+2)^2}{y+z-2}+\\frac{(y+4)^2}{z+x-4}+\\frac{(z+6)^2}{x+y-6}=36\n$$\n的所有实数解 $(x, y, z)$.", + "solution": "解:由 $x 、 y 、 z>3$, 知\n$$\ny+z-2>0, z+x-4>0, x+y-6>0 .\n$$\n由柯西一施瓦兹不等式得\n$$\n\\begin{aligned}\n& {\\left[\\frac{(x+2)^2}{y+z-2}+\\frac{(y+4)^2}{x+z-4}+\\frac{(z+6)^2}{x+y-6}\\right] . } \\\\\n& {[(y+z-2)+(x+z-4)+(x+y-6)] } \\\\\n\\geqslant & (x+y+z+12)^2 \\\\\n\\Leftrightarrow & \\frac{(x+2)^2}{y+z-2}+\\frac{(y+4)^2}{x+z-4}+\\frac{(z+6)^2}{x+y-6} \\\\\n\\geqslant & \\frac{1}{2} \\cdot \\frac{(x+y+z+12)^2}{x+y+z-6} .\n\\end{aligned}\n$$\n结合题设等式得\n$$\n\\frac{(x+y+z+12)^2}{x+y+z-6} \\leqslant 72 . \\label{(35)}\n$$\n当 $\\frac{x+2}{y+z-2}=\\frac{y+4}{x+z-4}=\\frac{z+6}{x+y-6}=\\lambda$, 即\n$$\n\\left\\{\\begin{array}{l}\n\\lambda(y+z)-x=2(\\lambda+1), \\\\\n\\lambda(x+z)-y=4(\\lambda+1), \\\\\n\\lambda(x+y)-z=6(\\lambda+1) .\n\\end{array}\\right. \\label{(36)}\n$$\n时,式(35)等号成立.\n设 $w=x+y+z+12$. 则又\n$$\n\\begin{aligned}\n& \\frac{(x+y+z+12)^2}{x+y+z-6}=\\frac{w^2}{w-18} . \\\\\n& \\frac{w^2}{w-18} \\geqslant 4 \\times 18=72 \\\\\n\\Leftrightarrow & w^2-4 \\times 18 w+4 \\times 18^2 \\geqslant 0 \\\\\n\\Leftrightarrow & (w-36)^2 \\geqslant 0,\n\\end{aligned}\n$$\n则\n$$\n\\frac{(x+y+z+12)^2}{x+y+z-6} \\geqslant 72 . \\label{(37)}\n$$\n当 $\\quad w=x+y+z+12=36$\n$$\n\\Leftrightarrow x+y+z=24 . \\label{(38)}\n$$\n时, 式(37)等号成立.\n由式(35),(37)得\n$$\n\\frac{(x+y+z+12)^2}{x+y+z-6}=72 .\n$$\n由方程组 (36) 与式 (38) 得\n$$\n\\left\\{\\begin{array}{l}\n(2 \\lambda-1)(x+y+z)=12(\\lambda+1), \\\\\nx+y+z=24\n\\end{array} \\Rightarrow \\lambda=1 .\\right.\n$$\n将 $\\lambda=1$ 代入方程组 (36) 得\n$$\n\\left\\{\\begin{array}{l}\ny+z-x=4, \\\\\nx+z-y=8, \\\\\nx+y-z=12\n\\end{array} \\Rightarrow(x, y, z)=(10,8,6) .\\right.\n$$\n所以,所求唯一实数解为\n$$\n(x, y, z)=(10,8,6) .\n$$", + "remark": "", + "figures": [] +} \ No newline at end of file diff --git a/processed_dataset/calculation/0827.json b/processed_dataset/calculation/0827.json new file mode 100644 index 0000000000000000000000000000000000000000..92682a3e12e3c4ee66eee166e1c5ff09a0111b16 --- /dev/null +++ b/processed_dataset/calculation/0827.json @@ -0,0 +1,8 @@ +{ + "source_file": "./raw_volume-zh/volume4/chapter4-2.tex", + "problem_type": "calculation", + "problem": "例3. $n$ 是一个正整数, $a_1, a_2, \\cdots, a_n, b_1, b_2, \\cdots, b_n$ 是 $2 n$ 个正实数,满足 $a_1+a_2+\\cdots+a_n=1, b_1+b_2+\\cdots+b_n=1$, 求 $\\frac{a_1^2}{a_1+b_1}+\\frac{a_2^2}{a_2+b_2}+\\cdots+\\frac{a_n^2}{a_n+b_n}$ 的最小值.", + "solution": "解:由柯西不等式知\n$$\n\\begin{aligned}\n& \\left(a_1+a_2+\\cdots+a_n+b_1+b_2+\\cdots+b_n\\right)\\left(\\frac{a_1^2}{a_1+b_1}+\\frac{a_2^2}{a_2+b_2}+\\cdots+\\frac{a_n^2}{a_n+b_n}\\right) . \\\\\n\\geqslant & \\left(a_1+a_2+\\cdots+a_n\\right)^2=1,\n\\end{aligned}\n$$\n且\n$$\na_1+a_2+\\cdots+a_n+b_1+b_2+\\cdots+b_n=2,\n$$\n所以\n$$\n\\frac{a_1^2}{a_1+b_1}+\\frac{a_2^2}{a_2+b_2}+\\cdots+\\frac{a_n^2}{a_n+b_n} \\geqslant \\frac{1}{2}\n$$\n且当 $a_1=a_2=\\cdots=a_n=b_1=b_2=\\cdots=b_n=\\frac{1}{n}$ 时取到.\n所以 $\\frac{a_1^2}{a_1+b_1}+\\frac{a_2^2}{a_2+b_2}+\\cdots+\\frac{a_n^2}{a_n+b_n}$ 的最小值为 $\\frac{1}{2}$.", + "remark": "", + "figures": [] +} \ No newline at end of file diff --git a/processed_dataset/calculation/0828.json b/processed_dataset/calculation/0828.json new file mode 100644 index 0000000000000000000000000000000000000000..051e458f2b0324c4b2483fda1850261255a42dc7 --- /dev/null +++ b/processed_dataset/calculation/0828.json @@ -0,0 +1,8 @@ +{ + "source_file": "./raw_volume-zh/volume4/chapter4-2.tex", + "problem_type": "calculation", + "problem": "例5. 设 $a 、 b 、 c 、 x 、 y 、 z$ 为实数, 且\n$$\na^2+b^2+c^2=25, x^2+y^2+z^2=36, a x+b y+c z=30 .\n$$\n求 $\\frac{a+b+c}{x+y+z}$ 的值.", + "solution": "解:由柯西不等式, 得\n$$\n25 \\times 36=\\left(a^2+b^2+c^2\\right)\\left(x^2+y^2+z^2\\right) \\geqslant(a x+b y+c z)^2=30^2 .\n$$\n上述不等式等号成立, 得\n$$\n\\frac{a}{x}=\\frac{b}{y}=\\frac{c}{z}=k\n$$\n于是 $k^2\\left(x^2+y^2+z^2\\right)=25$, 所以 $k= \\pm \\frac{5}{6}$ (负的舍去). 从而\n$$\n\\frac{a+b+c}{x+y+z}=k=\\frac{5}{6} \\text {. }\n$$", + "remark": "", + "figures": [] +} \ No newline at end of file diff --git a/processed_dataset/calculation/0829.json b/processed_dataset/calculation/0829.json new file mode 100644 index 0000000000000000000000000000000000000000..d608dac230d1159f19b118102567a34fa52c7a13 --- /dev/null +++ b/processed_dataset/calculation/0829.json @@ -0,0 +1,8 @@ +{ + "source_file": "./raw_volume-zh/volume4/chapter4-2.tex", + "problem_type": "calculation", + "problem": "例6. 设实数 $a 、 b 、 c 、 d 、 e$ 满足\n$$\na+b+c+d+e=8, a^2+b^2+c^2+d^2+e^2=16,\n$$\n求 $e$ 的最大值.", + "solution": "解:将条件改写为\n$$\n8-e=a+b+c+d, 16-e^2=a^2+b^2+c^2+d^2,\n$$\n由此得到一个包含 $e$ 的不等式.\n由柯西不等式, 得\n$$\na+b+c+d \\leqslant(1+1+1+1)^{\\frac{1}{2}}\\left(a^2+b^2+c^2+d^2\\right)^{\\frac{1}{2}} .\n$$\n将条件代入并两边平方, 得\n$$\n\\begin{gathered}\n(8-e)^2 \\leqslant 4\\left(16-e^2\\right), \\\\\n64-16 e+e^2 \\leqslant 64-4 e^2, \\\\\n5 e^2-16 e \\leqslant 0, e(5-16 e) \\leqslant 0 .\n\\end{gathered}\n$$\n从此得到 $0 \\leqslant e \\leqslant \\frac{16}{5}$, 当 $a=b=c=d=\\frac{6}{5}$ 时达到最大值 $\\frac{16}{5}$.", + "remark": "注:用类似的方法可以证明下面的命题:\n设 $n(\\geqslant 3)$ 为正整数, $a 、 b$ 为给定的实数, 实数 $x_0, x_1, x_2, \\cdots, x_n$ 满足\n$$\n\\begin{aligned}\n& x_0+x_1+x_2+\\cdots+\\dot{x}_n=a, \\\\\n& x_0^2+x_1^2+x_2^2+\\cdots+x_n^2=b,\n\\end{aligned}\n$$\n则当 $b<\\frac{a^2}{n+1}$ 时, $x_0$ 不存在;\n当 $b=\\frac{a^2}{n+1}$ 时, $x_0=\\frac{a}{n+1}$;\n当 $b>\\frac{a^2}{n+1}$ 时, $x_0$ 满足\n$$\n\\frac{a-\\frac{1}{2} \\sqrt{\\delta}}{n+1} \\leqslant x_0 \\leqslant \\frac{a+\\frac{1}{2} \\sqrt{\\delta}}{n+1},\n$$\n其中 $\\delta$ 为二次方程 $(n+1) x_0^2-2 a x_0+a^2-n b=0$ 的判别式.", + "figures": [] +} \ No newline at end of file diff --git a/processed_dataset/calculation/0830.json b/processed_dataset/calculation/0830.json new file mode 100644 index 0000000000000000000000000000000000000000..57123c9e72119361bfafe262f47da9ace211455a --- /dev/null +++ b/processed_dataset/calculation/0830.json @@ -0,0 +1,8 @@ +{ + "source_file": "./raw_volume-zh/volume4/chapter4-2.tex", + "problem_type": "calculation", + "problem": "例7. 设 $x \\geqslant 0, y \\geqslant 0, z \\geqslant 0, a 、 b 、 c 、 l 、 m 、 n$ 是给定的正数, 并且 $a x+b y+c z=\\delta$ 为常数, 求\n$$\nw=\\frac{l}{x}+\\frac{m}{y}+\\frac{n}{z}\n$$\n的最小值.", + "solution": "解:由柯西不等式, 得\n$$\n\\begin{aligned}\n& w \\cdot \\delta= {\\left[\\left(\\sqrt{\\frac{l}{x}}\\right)^2+\\left(\\sqrt{\\frac{m}{y}}\\right)^2+\\left(\\sqrt{\\frac{n}{z}}\\right)^2\\right] } \\\\\n& \\cdot\\left[(\\sqrt{a x})^2+(\\sqrt{b y})^2+(\\sqrt{c z})^2\\right] \\\\\n& \\geqslant(\\sqrt{a l}+\\sqrt{b m}+\\sqrt{c n})^2, \\\\\n& w \\geqslant(\\sqrt{a l}+\\sqrt{b m}+\\sqrt{c n})^2 \\\\\n& \\delta\n\\end{aligned}\n$$\n所以\n$$\nw \\geqslant \\frac{(\\sqrt{a l}+\\sqrt{b m}+\\sqrt{c n})^2}{\\delta} .\n$$\n利用柯西等式成立的条件, 得 $x=k \\sqrt{\\frac{l}{a}}, y=k \\sqrt{\\frac{m}{b}}, z=k \\sqrt{\\frac{n}{c}}$, 其中 $k=\\frac{\\delta}{\\sqrt{a l}+\\sqrt{b m}+\\sqrt{c n}}$, 它们使得 $a x+b y+c z=\\delta$, 且 $w= \\frac{(\\sqrt{a l}+\\sqrt{b m}+\\sqrt{c n})^2}{\\delta}$, 所以\n$$\nw_{\\min }=\\frac{(\\sqrt{a l}+\\sqrt{b m}+\\sqrt{c n})^2}{\\delta} .\n$$", + "remark": "", + "figures": [] +} \ No newline at end of file diff --git a/processed_dataset/calculation/0831.json b/processed_dataset/calculation/0831.json new file mode 100644 index 0000000000000000000000000000000000000000..852e0ebd820bc88812ef96aa2048561e8243f20f --- /dev/null +++ b/processed_dataset/calculation/0831.json @@ -0,0 +1,8 @@ +{ + "source_file": "./raw_volume-zh/volume4/chapter4-2.tex", + "problem_type": "calculation", + "problem": "例9. 设 $n$ 和 $k$ 是给定的正整数 $(k-1$, 则 $\\frac{1+x^2}{1+y+z^2}, \\frac{1+y^2}{1+z+x^2}, \\frac{1+z^2}{1+x+y^2}$ 的分子、分母均为正, 所以\n$$\n\\begin{aligned}\n& \\frac{1+x^2}{1+y+z^2}+\\frac{1+y^2}{1+z+x^2}+\\frac{1+z^2}{1+x+y^2} \\\\\n\\geqslant & \\frac{1+x^2}{1+z^2+\\frac{1+y^2}{2}}+\\frac{1+y^2}{1+x^2+\\frac{1+z^2}{2}}+\\frac{1+z^2}{1+y^2+\\frac{1+x^2}{2}} \\\\\n= & \\frac{2 a}{2 c+b}+\\frac{2 b}{2 a+c}+\\frac{2 c}{2 b+a},\n\\end{aligned}\n$$\n其中 $a=\\frac{1+x^2}{2}, b=\\frac{1+y^2}{2}, c=\\frac{1+z^2}{2}$.\n由柯西不等式,得\n$$\n\\begin{aligned}\n& \\frac{a}{2 c+b}+\\frac{b}{2 a+c}+\\frac{c}{2 b+a} \\\\\n\\geqslant & \\frac{(a+b+c)^2}{a(b+2 c)+b(c+2 a)+c(a+2 b)} \\\\\n= & \\frac{3(a b+b c+a c)+\\frac{1}{2}\\left[(b-c)^2+(c-a)^2+(a-b)^2\\right]}{3(a b+b c+a c)} \\\\\n\\geqslant & 1 .\n\\end{aligned}\n$$\n且当 $a=b=c=1$ 时, 上式取到最小值.\n故所求的最小值为 2 .", + "remark": "", + "figures": [] +} \ No newline at end of file diff --git a/processed_dataset/calculation/0835.json b/processed_dataset/calculation/0835.json new file mode 100644 index 0000000000000000000000000000000000000000..9f49483ddd18b87ccc435c3c2930e3fecec34880 --- /dev/null +++ b/processed_dataset/calculation/0835.json @@ -0,0 +1,8 @@ +{ + "source_file": "./raw_volume-zh/volume4/chapter4-2.tex", + "problem_type": "calculation", + "problem": "例14. 设 $n>3$ 为给定的正整数, 实数 $x_1, x_2, \\cdots, x_{n+1}, x_{n+2}$ 满足 $0< x_10$, 得\n$$\nx_k=t_{k-1} t_{k-2} \\cdots t_1 a=a b^{k-1} c^{\\frac{(k-1)(k-2)}{2}}, 2 \\leqslant k \\leqslant n+2 .\n$$\n因为 $x_2>x_1$, 所以 $b=\\frac{x_2}{x_1}>1$.\n又因为 $t_j=b c^{j-1}>1,1 \\leqslant j \\leqslant n+1$, 所以\n$$\nc>\\sqrt[n]{\\frac{1}{b}}\\left(\\geqslant \\sqrt[n-1]{\\frac{1}{b}}, 1 \\leqslant j \\leqslant n+1\\right) .\n$$\n故最小值为 1 , 且当且仅当 $x_1=a, x_k=a b^{k-1} c^{\\frac{(k-1)(k-2)}{2}}(2 \\leqslant k \\leqslant n+2$, 其中 $\\left.a>0, b>1, c>\\sqrt[n]{\\frac{1}{b}}\\right)$ 时等号成立.", + "remark": "注:这个题目的表达形式看起来很复杂, 但通过变量代换后, 可以发现各项之间的关系, 借助于柯西不等式,估计出它的下界.", + "figures": [] +} \ No newline at end of file diff --git a/processed_dataset/calculation/0836.json b/processed_dataset/calculation/0836.json new file mode 100644 index 0000000000000000000000000000000000000000..9444790009d60aaedd755e526a25f617cc088e58 --- /dev/null +++ b/processed_dataset/calculation/0836.json @@ -0,0 +1,8 @@ +{ + "source_file": "./raw_volume-zh/volume4/chapter4-4.tex", + "problem_type": "calculation", + "problem": "例4. 在 $m \\times m$ 方格纸中, 至少要挑出多少个小方格,才能使得这些小方格中存在四个小方格, 它们的中心组成一个矩形的 4 个顶点, 而矩形的边平行于原正方形的边.", + "solution": "解:所求的最小值为 $\\left[\\frac{m}{2}(1+\\sqrt{4 m-3})-1\\right]+1$. 设最多能挑出 $k$ 个小方格, 使得这些小方格中不存在任何四个小方格, 它们的中点组成一个矩形的 4 个顶点 (矩形的边平行于原正方形的边). 并假设位于第 $i$ 行的有 $k_i(i= 1,2, \\cdots, m)$ 个, 则\n$$\n\\sum_{i=1}^n k_i=k\n$$\n设第 $i$ 行的 $k_i$ 个小方格位于这行的第 $j_1, j_2, \\cdots, j_{k_i}$ 列, $1 \\leqslant j_10$, 于是 $y_1 y_2 \\cdots y_n=1$, 我们有\n$$\n\\left.S-x_i=\\sum_{j \\neq i} y_j^n \\geqslant(n-1)\\left(\\frac{\\sum_{j \\neq i} y_j}{n-1}\\right)^n \\text { (幂平均不等式 }\\right)\n$$\n$$\n\\begin{aligned}\n& \\geqslant(n-1)\\left(\\frac{\\sum_{j \\neq i} y_j}{n-1}\\right) \\cdot \\prod_{j \\neq i} y_j(\\text { 算术平均 } \\geqslant \\text { 几何平均 }) \\\\\n& =\\frac{\\sum_{j \\neq i} y_j}{y_i} .\n\\end{aligned}\n$$\n于是\n$$\n\\sum_{i=1}^n \\frac{1}{a+S-x_i} \\leqslant \\sum_{i=1}^n \\frac{y_i}{a y_i+\\sum_{j \\neq i} y_j} . \\label{(46)}\n$$\n当 $a=1$ 时,\n$$\n\\sum_{i=1}^n \\frac{y_i}{a y_i+\\sum_{j \\neq i} y_j}=\\sum_{i=1}^n \\frac{y_i}{\\sum_{j=1}^n y_j}=1\n$$\n且当 $x_1=x_2=\\cdots=x_n=1$ 时, $\\sum_{i=1}^n \\frac{1}{a+S-x_i}=1$, 此时 $M=1$.\n下面假设 $a>1$. 令 $z_i=\\frac{y_i}{\\sum_{j=1}^n y_j}, i=1,2, \\cdots, n$, 有 $\\sum_{i=1}^n z_i=1$.\n$$\n\\begin{aligned}\n& \\frac{y_i}{a y_i+\\sum_{j \\neq i} y_j}=\\frac{y_i}{(a-1) y_i+\\sum_{j=1}^n y_j} \\\\\n& =\\frac{z_i}{(a-1) z_i+1} \\\\\n& =\\frac{1}{a-1}\\left[1-\\frac{1}{(a-1) z_i+1}\\right] .\n\\end{aligned} \\label{(47)}\n$$\n由柯西不等式\n$$\n\\left\\{\\sum_{i=1}^n\\left[(a-1) z_i+1\\right]\\right\\}\\left[\\sum_{i=1}^n \\frac{1}{(a-1) z_i+1}\\right] \\geqslant n^2 .\n$$\n而\n$$\n\\sum_{i=1}^n\\left[(a-1) z_i+1\\right]=a-1+n,\n$$\n故\n$$\n\\sum_{i=1}^n \\frac{1}{(a-1) z_i+1} \\geqslant \\frac{n^2}{a-1+n} . \\label{(48)}\n$$\n结合 (46)、(47)、(48), 我们有\n$$\n\\begin{aligned}\n\\sum_{i=1}^n \\frac{1}{a+S-x_i} & \\leqslant \\sum_{i=1}^n\\left[\\frac{1}{a-1}\\left(1-\\frac{1}{(a-1) z_i+1}\\right)\\right] \\\\\n& \\leqslant \\frac{n}{a-1}-\\frac{1}{a-1} \\cdot \\frac{n^2}{a-1+n} \\\\\n& =\\frac{n}{a-1+n} .\n\\end{aligned}\n$$\n当 $x_1=x_2=\\cdots=x_n=1$ 时,有\n$$\n\\sum_{i=1}^n \\frac{1}{a+S-x_i}=\\frac{n}{a-1+n}\n$$\n故 $M=\\frac{n}{a-1+n}$.\n下面考虑 $a<1$ 的情况: 对任何常数 $\\lambda>0$, 函数\n$$\nf(x)=\\frac{x}{x+\\lambda}=1-\\frac{\\lambda}{x+\\lambda}\n$$\n在区间 $(0,+\\infty)$ 上严格单调递增, 故 $f(a)0$ 且 $b+c \\geqslant a+d$. 求 $\\frac{b}{c+d}+\\frac{c}{a+b}$ 的最小值.", + "solution": "由已知, 得 $b+c \\geqslant \\frac{1}{2}(a+b+c+d)$. 不妨设 $a+b=c+d$, 则 $\\frac{b}{c+d}+\\frac{c}{a+b}=\\frac{b+c}{c+d}+c\\left(\\frac{1}{a+b}-\\frac{1}{c+d}\\right) \\geqslant \\frac{\\frac{1}{2}(a+b+c+d)}{c+d}+(c+$ d) $\\left(\\frac{1}{a+b}-\\frac{1}{c+d}\\right)=\\frac{a+b}{2(c+d)}+\\frac{c+d}{a+b}-\\frac{1}{2} \\geqslant \\sqrt{2}-\\frac{1}{2}$. 当 $a=\\sqrt{2}+1, b= \\sqrt{2}-1, c=2, d=0$ 时, 取等号.\n故所求最小值为 $\\sqrt{2}-\\frac{1}{2}$.", + "remark": "", + "figures": [] +} \ No newline at end of file diff --git a/processed_dataset/calculation/0843.json b/processed_dataset/calculation/0843.json new file mode 100644 index 0000000000000000000000000000000000000000..10ba83f6a949f8f1e6c265c03b3329b336543f4f --- /dev/null +++ b/processed_dataset/calculation/0843.json @@ -0,0 +1,8 @@ +{ + "source_file": "./raw_volume-zh/volume4/exercise2.tex", + "problem_type": "calculation", + "problem": "问题12. 对任意正数 $a_1, a_2, \\cdots, a_n, n \\geqslant 2$, 求 $\\sum_{i=1}^n \\frac{a_i}{S-a_i}$ 的最小值, 其中 $S= \\sum_{i=1}^n a_i$", + "solution": "令 $b_i=S-a_i$, 则 $\\sum_{i=1}^n b_i=(n-1) S$. 由平均值不等式, 得 $\\sum_{i=1}^n \\frac{a_i}{S-a_i} =\\sum_{i=1}^n\\left(\\frac{a_i}{b_i}+1\\right)-n=S \\sum_{i=1}^n \\frac{1}{b_i}-n \\geqslant \\frac{n S}{\\sqrt[n]{b_1 \\cdots b_n}}-n \\geqslant \\frac{n^2 S}{b_1+\\cdots+b_n}-n=\\frac{n}{n-1}$. 当 $a_1=a_2=\\cdots=a_n=1$ 时,等号成立, 故最小值为 $\\frac{n}{n-1}$.", + "remark": "", + "figures": [] +} \ No newline at end of file diff --git a/processed_dataset/calculation/0844.json b/processed_dataset/calculation/0844.json new file mode 100644 index 0000000000000000000000000000000000000000..4d268676b2bb1e8c81cf86e3f1689cff1b4f98bd --- /dev/null +++ b/processed_dataset/calculation/0844.json @@ -0,0 +1,8 @@ +{ + "source_file": "./raw_volume-zh/volume4/exercise2.tex", + "problem_type": "calculation", + "problem": "问题13. 求乘积 $x^2 y^2 z^2 u$ 在条件 $x, y, z, u \\geqslant 0$ 与 $2 x+x y+z+y z u=1$ 下的最大值.", + "solution": "由平均值不等式, 得 $\\sqrt[4]{2 x^2 y^2 z^2 u} \\leqslant \\frac{2 x+x y+z+z y u}{4}=\\frac{1}{4}$, 即\n$x^2 y^2 z^2 u \\leqslant \\frac{1}{512}$. 而且当 $2 x=x y=z=y z u=\\frac{1}{4}$, 即 $x=\\frac{1}{8}, y=2, z==\\frac{1}{4}$, $u=\\frac{1}{2}$ 时等式成立.\n于是, 所求的最大值为 $\\frac{1}{512}$.", + "remark": "", + "figures": [] +} \ No newline at end of file diff --git a/processed_dataset/calculation/0845.json b/processed_dataset/calculation/0845.json new file mode 100644 index 0000000000000000000000000000000000000000..03fcaf2fa88a9829d9ddf698ff09f43c59da96cd --- /dev/null +++ b/processed_dataset/calculation/0845.json @@ -0,0 +1,8 @@ +{ + "source_file": "./raw_volume-zh/volume4/exercise2.tex", + "problem_type": "calculation", + "problem": "问题15. 设正实数 $a_1, a_2, \\cdots, a_n(n \\geqslant 2)$ 满足 $a_1+a_2+\\cdots+a_n=1$, 求\n$$\n\\sum_{i=1}^n \\frac{a_i}{2-a_i}\n$$\n的最小值.", + "solution": "令 $b_i=2-a_i \\geqslant 0(i=1,2, \\cdots, n)$, 则 $\\sum_{i=1}^n b_i=2 n-1$. 由平均值不等式, 得 $\\sum_{i=1}^n \\frac{a_i}{b_i}=\\sum_{i=1}^n\\left(\\frac{a_i}{b_i}-1\\right)-n=2 \\sum_{i=1}^n \\frac{1}{b_i}-n \\geqslant \\frac{2 n}{\\sqrt[n]{b_1 \\cdots}}-n \\geqslant \\frac{2 n^2}{b_1+\\cdots+b_n}-n=\\frac{2 n^2}{2 n-1}-n=\\frac{n}{2 n-1}$. 当 $a_1=\\cdots=a_n=\\frac{1}{n}$ 时, $\\sum_{i=1}^n \\frac{a_i}{2-a_i}= \\frac{n}{2 n-1}$, 故最小值为 $\\frac{n}{2 n-1}$.", + "remark": "", + "figures": [] +} \ No newline at end of file diff --git a/processed_dataset/calculation/0846.json b/processed_dataset/calculation/0846.json new file mode 100644 index 0000000000000000000000000000000000000000..88bf3e212154a9daa3f894515f22c8a0d1d2176b --- /dev/null +++ b/processed_dataset/calculation/0846.json @@ -0,0 +1,8 @@ +{ + "source_file": "./raw_volume-zh/volume4/exercise2.tex", + "problem_type": "calculation", + "problem": "问题16. 设 $a>0, x_1, x_2, \\cdots, x_n \\in[0, a](n \\geqslant 2)$ 且满足\n$$\nx_1 x_2 \\cdots x_n=\\left(a-x_1\\right)^2\\left(a-x_2\\right)^2 \\cdots\\left(a-x_n\\right)^2 .\n$$\n求 $x_1 x_2 \\cdots x_n$ 的最大值.", + "solution": "由平均值不等式, 得 $\\left(x_1 x_2 \\cdots, x_n\\right)^{\\frac{1}{2 n}}=\\left[\\left(a-x_1\\right)\\left(a-x_2\\right) \\cdots(a-\\right. \\left.\\left.x_n\\right)\\right]^{\\frac{1}{n}} \\leqslant a-\\frac{x_1+\\cdots+x_n}{n} \\leqslant a-\\left(x_1 \\cdots x_n\\right)^{\\frac{1}{n}}$. 令 $y=\\left(x_1 x_2 \\cdots x_n\\right)^{\\frac{1}{2 n}} \\geqslant 0$, 则有 $y \\leqslant a-y^2$, 即 $y^2+y-a \\leqslant 0$. 解不等式得 $0 \\leqslant y \\leqslant \\frac{-1+\\sqrt{4 a+1}}{2}$, 故 $x_1 x_2 \\cdots x_n$ 的最大值为 $\\left(\\frac{-1+\\sqrt{4 a+1}}{2}\\right)^{2 n}$.", + "remark": "", + "figures": [] +} \ No newline at end of file diff --git a/processed_dataset/calculation/0847.json b/processed_dataset/calculation/0847.json new file mode 100644 index 0000000000000000000000000000000000000000..eda47ab39da265f5e15d8306f04b400686538bed --- /dev/null +++ b/processed_dataset/calculation/0847.json @@ -0,0 +1,8 @@ +{ + "source_file": "./raw_volume-zh/volume4/exercise2.tex", + "problem_type": "calculation", + "problem": "问题17. 设 $n \\geqslant 2, x_1, x_2, \\cdots, x_n$ 为实数, 且 $\\sum_{i=1}^n x_i^2+\\sum_{i=1}^{n-1} x_i x_{i+1}=1$, 对每个给定的正整数 $k, 1 \\leqslant k \\leqslant n$, 求 $\\left|x_k\\right|$ 的最大值.", + "solution": "由已知条件, 得 $2 \\sum_{i=1}^n x_i^2+2 \\sum_{i=1}^{n-1} x_i x_{i+1}=2$. 即 $x_1^2+\\left(x_1+x_2\\right)^2+ \\left(x_2+x_3\\right)^2+\\cdots+\\left(x_{n-2}+x_{n-1}\\right)^2+\\left(x_{n-1}+x_n\\right)^2+x_n^2=2$. 对给定的正整数 $k$, $1 \\leqslant k \\leqslant n$, 由平均值不等式, 得 $\\sqrt{\\frac{x_1^2+\\left(x_1+x_2\\right)^2+\\cdots+\\left(x_{k-1}+x_k\\right)^2}{k}} \\geqslant \\frac{\\left|x_1\\right|+\\left|x_1+x_2\\right|+\\cdots+\\left|x_{k-1}+x_k\\right|}{k} \\geqslant \\frac{\\left|x_1-\\left(x_1+x_2\\right)+\\cdots+(-1)^{k-1}\\left(x_{k-1}+x_k\\right)\\right|}{k}=\\frac{\\left|x_k\\right|}{k}$. 所以 $\\frac{x_1^2+\\left(x_1+x_2\\right)^2+\\cdots+\\left(x_{k-1}+x_k\\right)^2}{k} \\geqslant \\frac{x_k^2}{k^2}$, 即 $x_1^2+\\left(x_1+x_2\\right)^2 +\\cdots+\\left(x_{k-1}+x_k\\right)^2 \\geqslant \\frac{x_k^2}{k}$. 同理, 可得 $\\left(x_k+x_{k+1}\\right)^2+\\cdots+\\left(x_{n-1}+x_n\\right)^2+x_n^2 \\geqslant \\frac{x_k^2}{n-k+1}$. 将以上两式相加, 得 $\\left|x_k\\right| \\leqslant \\sqrt{\\frac{2 k(n+1-k)}{n+1}}(k=1,2, \\cdots, n)$, 当且仅当 $x_1=-\\left(x_1+x_2\\right)=\\left(x_2+x_3\\right)=\\cdots=(-1)^{k-1}\\left(x_{k-1}+x_k\\right)$ 及 $x_k+x_{k+1}=-\\left(x_{k+1}+x_{k+2}\\right)=\\cdots=(-1)^{n-k} x_n$ 时, 等号成立, 即当且仅当 $x_i= x_k(-1)^{i-k} \\frac{i}{k}(i=1,2, \\cdots, k-1)$ 且 $x_j=x_k(-1)^{j-k} \\frac{n+1--j}{n-k+1}(j=k+1$, $k+2, \\cdots, n)$ 时, $\\left|x_k\\right|=\\sqrt{\\frac{2 k(n+1-k)}{n+1}}$. 于是 $\\left|x_k\\right|_{\\text {max }}= \\sqrt{\\frac{2 k(n+1-k)}{n+1}}$.", + "remark": "", + "figures": [] +} \ No newline at end of file diff --git a/processed_dataset/calculation/0848.json b/processed_dataset/calculation/0848.json new file mode 100644 index 0000000000000000000000000000000000000000..ac0c0d7ff12b62373ebc879a4dff952e1f678087 --- /dev/null +++ b/processed_dataset/calculation/0848.json @@ -0,0 +1,10 @@ +{ + "source_file": "./raw_volume-zh/volume4/exercise2.tex", + "problem_type": "calculation", + "problem": "问题22. 设等腰梯形的最大边长为 13 , 周长为 28 .\n(1) 设梯形的面积为 27 , 求它的边长;\n(2) 这种梯形的面积能否等于 27.001 ?", + "solution": "如图()所示, 设 $A D$ 是较大的底边, $B H$ 是给定梯形 $A B C D$ 的高.\n如果\n$A B=C D=13$, 则 $A D+B C=2$, 且 $S_{\\text {梯形 } A B C D}=B H \\cdot \\frac{A D+B C}{2} \\leqslant 13 \\cdot \\frac{2}{2}= 13<27$, 不可能.\n因此, $A D=13$. 记 $A B=x$, 则 $B C=28-13-2 x=15-2 x$, $A H=x-1, B H=\\sqrt{2 x-1}$. 由平均值不等式, 得 $S_{\\text {梯形 } A B C D}=\\sqrt{2 x-1} \\cdot \\frac{28-2 x}{2} =\\sqrt{(2 x-1)(14-x)^2} \\leqslant \\sqrt{\\left[\\frac{(2 x-1)+(14-x)+(14-x)}{3}\\right]^3}=27$. 当且仅当 $2 x-1=14-x$, 即 $x=5$, 也是 $A B=B C=C D=5$ 时, $S_{\\text {梯形 } A B C D}=27$, 而等式 $S_{\\text {梯形 } A B C D}=27.001$ 是不可能成立的.", + "remark": "", + "figures": [ + "./images/volume4/figures/fig-c2p22.png" + ] +} \ No newline at end of file diff --git a/processed_dataset/calculation/0849.json b/processed_dataset/calculation/0849.json new file mode 100644 index 0000000000000000000000000000000000000000..6f5f2b150b91f9833d0d5e5a2541a039062ab7b8 --- /dev/null +++ b/processed_dataset/calculation/0849.json @@ -0,0 +1,8 @@ +{ + "source_file": "./raw_volume-zh/volume4/exercise2.tex", + "problem_type": "calculation", + "problem": "问题23. 在所有周长一定的三角形中,求内切圆半径最大的三角形.", + "solution": "设 $a 、 b 、 c$ 是半周长 $p$ 一定的三角形的边长, $S$ 与 $r$ 是它的面积与内切圆半径.\n则由平均值不等式, 得 $(r p)^2=S^2=p(p-a)(p-b)(p-c) \\leqslant p\\left[\\frac{(p-a)+(p-b)+(p-c)}{3}\\right]^3=\\frac{p^4}{27}$. 由此得到 $r \\leqslant \\frac{p}{\\sqrt{27}}$, 当且仅当 $p- a=p-b=p-c$, 即三角形为等边三角形时, $r$ 取到最大值.", + "remark": "", + "figures": [] +} \ No newline at end of file diff --git a/processed_dataset/calculation/0850.json b/processed_dataset/calculation/0850.json new file mode 100644 index 0000000000000000000000000000000000000000..182e88b85bfe5eb15b2811cc586ec05485065ff2 --- /dev/null +++ b/processed_dataset/calculation/0850.json @@ -0,0 +1,8 @@ +{ + "source_file": "./raw_volume-zh/volume4/exercise2.tex", + "problem_type": "calculation", + "problem": "问题25. 设 $n \\geqslant 2$, 求乘积 $x_1 x_2 \\cdots x_n$ 在条件 $x_i \\geqslant \\frac{1}{n}(i=1,2, \\cdots, n)$ 与 $x_1^2+x_2^2+\\cdots+x_n^2=1$ 下的最大值和最小值.", + "solution": "首先求最大值, 由平均值不等式, 得 $\\sqrt[n]{x_1^2 x_2^2 \\cdots x_n^2} \\leqslant \\frac{x_1^2+x_2^2+\\cdots+x_n^2}{n}=\\frac{1}{n}$. 当 $x_1=x_2=\\cdots=x_n=\\frac{1}{\\sqrt{n}}>\\frac{1}{n}(n \\geqslant 2)$ 时等号成立.\n所以最大值为 $n^{-\\frac{n}{2}}$. 再求最小值.\n令 $y_1=x_1, \\cdots, y_{n-2}=x_{n-2}, y_{n-1}= \\sqrt{x_{n-1}^2+x_n^2-\\frac{1}{n^2}}, y_n=\\frac{1}{n}$, 则 $y_i \\geqslant \\frac{1}{n}, i=1,2, \\cdots, n$, 且 $y_1^2+\\cdots+y_n^2= x_1^2+\\cdots+x_n^2=1$. 由于 $y_{n-1}^2 y_n^2-x_{n-1}^2 x_n^2=-\\left(x_{n-1}^2-\\frac{1}{n^2}\\right)\\left(x_n^2-\\frac{1}{n^2}\\right) \\leqslant 0$, 所以 $y_1 y_2 \\cdots y_{n-2} y_{n-1} y_n \\leqslant x_1 x_2 \\cdots x_{n-2} x_{n-1} x_n$. 重复这个过程 $n-1$ 次, 得 $x_1 x_2 \\cdots x_n \\geqslant\\left(\\frac{1}{n}\\right)^{n-1} \\sqrt{1-\\frac{n-1}{n^2}}=\\frac{\\sqrt{n^2-n+1}}{n^n}$. 当 $x_1=\\cdots=x_{n-1}=\\frac{1}{n}$, $x_n=\\frac{\\sqrt{n^2--} n+1}{n}$. 时, 等号成立.\n故最小值为 $\\frac{\\sqrt{n^2-n+1}}{n^n}$.", + "remark": "", + "figures": [] +} \ No newline at end of file diff --git a/processed_dataset/calculation/0851.json b/processed_dataset/calculation/0851.json new file mode 100644 index 0000000000000000000000000000000000000000..306b0ed4ac586e8ee0a74c8845ac5e0456acd6c0 --- /dev/null +++ b/processed_dataset/calculation/0851.json @@ -0,0 +1,8 @@ +{ + "source_file": "./raw_volume-zh/volume4/exercise2.tex", + "problem_type": "calculation", + "problem": "问题26. 求最小正数 $\\lambda$, 使得对于任一三角形的三边长 $a 、 b 、 c$, 只要 $a \\geqslant \\frac{b+c}{3}$, 就有 $a c+b c-c^2 \\leqslant \\lambda\\left(a^2+b^2+3 c^2+2 a b-4 b c\\right)$.", + "solution": "易知 $a^2+b^2+3 c^2+2 a b-4 b c=(a+b-c)^2+2 c^2+2 a c-2 b c=(a+b-c)^2+2 c(a+c-b)$. 令 $I=\\frac{(a+b-c)^2+2 c(a+c-b)}{2 c(a+b-c)}=\\frac{a+b-c}{2 c} +\\frac{a+c-b}{a+b-c}$, 由于 $a \\geqslant \\frac{1}{3}(b+c)$, 所以 $a \\geqslant \\frac{1}{4}(a+b-c)+\\frac{c}{2}$. 于是 $a+c- b=2 a-(a+b-c) \\geqslant-\\frac{1}{2}(a+b-c)+c$. 由此可知 $I \\geqslant \\frac{a+b-c}{2 c}- \\frac{\\frac{1}{2}(a+b-c)}{a+b-c}+\\frac{c}{a+b-c}=-\\frac{1}{2}+\\frac{a+b-c}{2 c}+\\frac{c}{a+b-c} \\geqslant-\\frac{1}{2}+2 \\sqrt{\\frac{1}{2}}= \\sqrt{2}-\\frac{1}{2}$. 即 $\\frac{a c+b c-c^2}{a^2+b^2+3 c^2+2 a b-4 b c}=\\frac{1}{2 I} \\leqslant \\frac{1}{2 \\sqrt{2}-1}=\\frac{2 \\sqrt{2}+1}{7}$. 所以, $\\lambda \\geqslant \\frac{2 \\sqrt{2}+1}{7}$. 另一方面, 当 $a=\\frac{\\sqrt{2}}{4}+\\frac{1}{2}, b=\\frac{3 \\sqrt{2}}{4}+\\frac{1}{2}, c=1$, 则 $a c+b c- c^2=\\sqrt{2}, a^2+b^2+3 c^2+2 a b-4 b c=4-\\sqrt{2}$, 所以 $\\frac{1}{2 I}=\\frac{\\sqrt{2}}{4-\\sqrt{2}}=\\frac{2 \\sqrt{2}+1}{7}$. 故 $\\lambda=\\frac{2 \\sqrt{2}+1}{7}$.", + "remark": "", + "figures": [] +} \ No newline at end of file diff --git a/processed_dataset/calculation/0852.json b/processed_dataset/calculation/0852.json new file mode 100644 index 0000000000000000000000000000000000000000..df43fb2ee78ccc2f441e9fe0de67d2528cb4e4b1 --- /dev/null +++ b/processed_dataset/calculation/0852.json @@ -0,0 +1,8 @@ +{ + "source_file": "./raw_volume-zh/volume4/exercise2.tex", + "problem_type": "calculation", + "problem": "问题33. 求最大的实数 $\\lambda$, 使得当实系数多项式 $f(x)=x^3+a x^2+c$ 的所有根都是非负实数时, 只要 $x \\geqslant 0$, 就有 $f(x) \\geqslant \\lambda(x-a)^3$, 并求等号成立的条件.", + "solution": "设 $f(x)$ 的三个根为 $\\alpha, \\beta, \\gamma$, 并设 $0 \\leqslant \\alpha \\leqslant \\beta \\leqslant \\gamma$, 则 $x-a=x+\\alpha+ \\beta+\\gamma, f(x)=(x-\\alpha)(x-\\beta)(x-\\gamma)$. (1) 当 $0 \\leqslant x \\leqslant \\alpha$ 时, 则有 $-f(x)= (\\alpha-x)(\\beta-x)(\\gamma-x) \\leqslant\\left(\\frac{\\alpha+\\beta+\\gamma-3 x}{3}\\right)^3 \\leqslant \\frac{1}{27}(x+\\alpha+\\beta+\\gamma)^3=\\frac{1}{27}(x- a)^3$, 所以 $f(x) \\geqslant-\\frac{1}{27}(x-a)^3$. 当 $x=0, \\alpha=\\beta=\\gamma$ 时, 等号成立; (2) 当 $\\alpha \\leqslant x \\leqslant \\beta$ 或 $x>\\gamma$ 时, $f(x)=(x-\\alpha)(x-\\beta)(x-\\gamma)>0>-\\frac{1}{27}(x-a)^3$.\n(3) 当 $\\beta \\leqslant x \\leqslant \\gamma$ 时, $-f(x)=(x-\\alpha)(x-\\beta)(\\gamma-x) \\leqslant\\left(\\frac{x+\\gamma-\\alpha-\\beta}{3}\\right)^3 \\leqslant \\frac{1}{27}(x+\\alpha+\\beta+\\gamma)^3=\\frac{1}{27}(x-a)^3$. 所以 $f(x) \\geqslant-\\frac{1}{27}(x-a)^3$. 当 $\\alpha=\\beta=0$, $\\gamma=2 x$ 时, 等号成立.\n综上所述, $\\lambda$ 的最大值 $-\\frac{1}{27}$.", + "remark": "", + "figures": [] +} \ No newline at end of file diff --git a/processed_dataset/calculation/0853.json b/processed_dataset/calculation/0853.json new file mode 100644 index 0000000000000000000000000000000000000000..6de53e604aee150245bdabbfb2653f7990c278bd --- /dev/null +++ b/processed_dataset/calculation/0853.json @@ -0,0 +1,8 @@ +{ + "source_file": "./raw_volume-zh/volume4/exercise4.tex", + "problem_type": "calculation", + "problem": "问题2. 设 $x, y, z \\in \\mathbf{R}^{+}$, 且 $x+y+z \\geqslant 6$. 求\n$$\nM=\\sum x^2+\\sum \\frac{x}{y^2+z+1}\n$$\n的最小值, 其中, \" $\\sum$ \" 表示轮换对称和.", + "solution": "由均值不等式有 ${\\frac{x^{2}}{14}}+{\\frac{x}{y^{2}+z+1}}+{\\frac{2}{49}}(y^{2}+z+1)\\geq3{\\sqrt{{\\frac{x^{3}}{7^{3}}}}}={\\frac{3}{7}}x$. 则 $\\frac{1}{14} \\sum x^2+\\sum \\frac{x}{y^2+z+1}+\\frac{2}{49} \\sum x^2+\\frac{2}{49} \\sum x+\\frac{6}{49} \\geqslant \\frac{3}{7} \\sum x$. 故 $\\frac{11}{98} \\sum x^2+\\sum \\frac{x}{y^2+z+1}+\\frac{6}{49} \\geqslant\\left(\\frac{3}{7}-\\frac{2}{49}\\right) \\sum x=\\frac{19}{49} \\sum x$. 又 $\\sum x^2 \\geqslant \\frac{1}{3}\\left(\\sum x\\right)^2 \\geqslant 12$, 故 $\\sum x^2+\\sum \\frac{x}{y^2+z+1}=\\frac{87}{98} \\sum x^2+\\frac{11}{98} \\sum x^2-\\frac{6}{49} \\geqslant \\frac{87}{98} \\sum x^2+\\frac{19}{49} \\sum x-\\frac{6}{49} \\geqslant \\frac{87 \\times 6+19 \\times 6-6}{49}=\\frac{90}{7}$. 从而, $M_{\\min }=\\frac{90}{7}$. 此时, $(x, y, z)=(2,2,2)$.", + "remark": "", + "figures": [] +} \ No newline at end of file diff --git a/processed_dataset/calculation/0854.json b/processed_dataset/calculation/0854.json new file mode 100644 index 0000000000000000000000000000000000000000..d1d0c68b06b9e1b1403fbaa895e6a63c3ba027d6 --- /dev/null +++ b/processed_dataset/calculation/0854.json @@ -0,0 +1,8 @@ +{ + "source_file": "./raw_volume-zh/volume4/exercise4.tex", + "problem_type": "calculation", + "problem": "问题9. 已知 $00, b_i>0, a_i b_i-c_i^2>0(i=1,2, \\cdots, n)$, 则\n$$\n\\frac{n^3}{\\left(\\sum_{i=1}^n a_i\\right)\\left(\\sum_{i=1}^n b_i\\right)-\\left(\\sum_{i=1}^n c_i\\right)^2} \\leqslant \\sum_{i=1}^n \\frac{1}{a_i b_i-c_i^2} .\n$$", + "solution": "令 $a_i b_i-c_i^2=d_i^2>0$, 则由柯西不等式, 得 $\\left(\\sum a_i\\right)\\left(\\sum b_i\\right) \\geqslant\\left(\\sum \\sqrt{a_i b_i}\\right)^2=\\sum_{i=1}^n \\sum_{j=1}^n \\sqrt{a_i} \\overline{b_i} \\sqrt{a_j b_j}=\\sum_{i=1}^n \\sum_{j=1}^n \\sqrt{c_i^2+d_i^2} \\sqrt{c_j^2+d_j^2} \\geqslant \\sum_{i=1}^n \\sum_{j=1}^n\\left(c_i c_j+d_i d_j\\right)=\\left(\\sum_{i=1}^n c_i\\right)^2+\\left(\\sum_{i=1}^n d_i\\right)^2$, 又因为 $\\left(\\sum_{i=1}^n d_i\\right)^2\\left(\\sum_{i=1}^n d_i^{-2}\\right) \\geqslant n^3$, 故左边 $\\leqslant \\frac{n^3}{\\left(\\sum d_i\\right)^2} \\leqslant \\sum d_i^{-2}$. 等号成立当且仅当 $a_1=a_2=\\cdots=a_n, b_1= b_2=\\cdots=b_n, c_1=c_2=\\cdots=c_n$.", + "remark": "", + "figures": [] +} \ No newline at end of file diff --git a/processed_dataset/calculation/0857.json b/processed_dataset/calculation/0857.json new file mode 100644 index 0000000000000000000000000000000000000000..761fafb8034d1d4f4aa67d9b94894c93899410ac --- /dev/null +++ b/processed_dataset/calculation/0857.json @@ -0,0 +1,8 @@ +{ + "source_file": "./raw_volume-zh/volume4/exercise4.tex", + "problem_type": "calculation", + "problem": "问题15. 已给自然数 $n \\geqslant 2$, 求最小正数 $\\lambda$, 使得对任意正数 $a_1, a_2, \\cdots, a_n$, 及 $\\left[0, \\frac{1}{2}\\right]$ 中任意 $n$ 个数 $b_1, b_2, \\cdots, b_n$, 只要\n$$\n\\begin{gathered}\n\\sum_{i=1}^n a_i=\\sum_{i=1}^n b_i=1, \\\\\n\\prod_{i=1}^n a_i \\leqslant \\lambda \\sum_{i=1}^n a_i b_i .\n\\end{gathered}\n$$\n就有\n$$\n\\prod_{i=1}^n a_i \\leqslant \\lambda \\sum_{i=1}^n a_i b_i\n$$", + "solution": "由柯西不等式, 得 $1=\\sum_{i=1}^n b_i \\leqslant\\left(\\sum_{i=1}^n \\frac{b_i}{a_i}\\right)^{\\frac{1}{2}}\\left(\\sum_{i=1}^n a_i b_i\\right)^{\\frac{1}{2}}$, 从而 $\\frac{1}{\\sum_{i=1}^n a_i b_i} \\leqslant \\sum_{i=1}^n \\frac{b_i}{a_i}$. 令 $M=\\prod_{i=1}^n a_i, A_i=\\frac{M}{a_i}, i=1,2, \\cdots, n$, 则 $\\frac{M}{\\sum_{i=1}^n a_i b_i} \\leqslant \\sum_{i=1}^n b_i A_i$. 不妨设 $b_1 \\geqslant b_2 \\geqslant \\cdots \\geqslant b_n, A_1 \\geqslant A_2 \\geqslant \\cdots \\geqslant A_n$, 由排序不等式, 得 $\\sum_{i=1}^n b_i A_i \\leqslant b_1 A_1+\\left(1-b_1\\right) A_2$. 由于 $0 \\leqslant b_1 \\leqslant \\frac{1}{2}, A_1 \\geqslant A_2$, 所以 $\\sum_{i=1}^n b_i A_i \\leqslant\\frac{1}{2}\\left(A_1+A_2\\right)=\\frac{1}{2}\\left(a_1+a_2\\right) a_3 \\cdots a_n$. 由平均值不等式, 得 $\\sum_{i=1}^n b_i A_i \\leqslant \\frac{1}{2}\\left(\\frac{1}{n-1}\\right)^{n-1}$, 所以 $\\lambda \\leqslant \\frac{1}{2}\\left(\\frac{1}{n-1}\\right)^{n-1}$. 另一方面, 当 $a_1=a_2=\\frac{1}{2(n-1)}$, $a_3=\\cdots=a_n=\\frac{1}{n-1}, b_1=b_2=\\frac{1}{2}, b_3=\\cdots=b_n=0$ 时, $\\prod_{i=1}^n a_i= \\frac{1}{2}\\left(\\frac{1}{n-1}\\right)^{n-1} \\sum_{i=1}^n a_i b_i$, 所以 $\\lambda \\geqslant \\frac{1}{2}\\left(\\frac{1}{n-1}\\right)^{n-1}$. 故 $\\lambda_{\\min }=\\frac{1}{2}\\left(\\frac{1}{n-1}\\right)^{n-1}$.", + "remark": "", + "figures": [] +} \ No newline at end of file diff --git a/processed_dataset/calculation/0858.json b/processed_dataset/calculation/0858.json new file mode 100644 index 0000000000000000000000000000000000000000..4cf23ac8b32533421a4fbe7cc5bfd70250c91ba9 --- /dev/null +++ b/processed_dataset/calculation/0858.json @@ -0,0 +1,8 @@ +{ + "source_file": "./raw_volume-zh/volume4/exercise4.tex", + "problem_type": "calculation", + "problem": "问题16. 已给两个大于 1 的自然数 $n$ 和 $m$, 求所有的自然数 $l$, 使得对任意正数 $a_1$, $a_2, \\cdots, a_n$, 都有\n$$\n\\sum_{k=1}^n \\frac{1}{S_k}\\left(l k+\\frac{1}{4} l^2\\right)2(m-1)$, 即 $l \\geqslant 2 m-1$ 时, 任意给定 $a_1>0$. 令 $a_k==\\frac{l+2}{2(k-1)} S_{k-1}, k=2,3, \\cdots, n$, 则 $\\sum_{k=1}^n \\frac{1}{S_k}\\left(l k+\\frac{1}{4} l^2\\right)= \\left(\\frac{l}{2}+1\\right)^2 \\sum_{i=1}^n \\frac{1}{a_i}-\\frac{n^2}{S_n}=\\left[\\left(\\frac{l}{2}+1\\right)^2-1\\right] \\sum_{k=1}^n \\frac{1}{a_k}+\\sum_{k=1}^n \\frac{1}{a_k}-\\frac{n^2}{S_n}$. 由 $l \\geqslant 2 m-$ 1 , 可推出 $\\left(\\frac{l}{2}+1\\right)^2-1 \\geqslant\\left(m+\\frac{1}{2}\\right)^2-1=m^2+m+\\frac{1}{4}-1>m^2$. 由柯西不等式, 得 $n^2 \\leqslant\\left(\\sum_{k=1}^n a_k\\right)\\left(\\sum_{k=1}^n \\frac{1}{a_k}\\right)=S_n \\sum_{k=1}^n \\frac{1}{a_k}$, 即 $\\sum_{k=1}^n \\frac{1}{a_k}-\\frac{n^2}{S_n} \\geqslant 0$. 从而 $\\sum_{k=1}^n \\frac{1}{S_k}\\left(l k+\\frac{1}{4} l^2\\right)>m^2 \\sum_{k=1}^n \\frac{1}{a_k}$, 于是 $1,2, \\cdots, 2(m-1)$ 是满足要求的所有自然数 $l$.", + "remark": "", + "figures": [] +} \ No newline at end of file diff --git a/processed_dataset/calculation/0859.json b/processed_dataset/calculation/0859.json new file mode 100644 index 0000000000000000000000000000000000000000..077c2fc3947cc5f95add6b191ddbf064e117b02f --- /dev/null +++ b/processed_dataset/calculation/0859.json @@ -0,0 +1,8 @@ +{ + "source_file": "./raw_volume-zh/volume5/chapter1.tex", + "problem_type": "calculation", + "problem": "例17. 对于 $\\frac{1}{2} \\leqslant x \\leqslant 1$, 求 $(1+x)^5(1-x)(1-2 x)^2$ 的最大值.", + "solution": "解:我们考虑 $[\\alpha(1+x)]^5[\\beta(1-x)][\\gamma(2 x-1)]^2$ 的最大值, 这里 $\\alpha 、 \\beta$ 、 $\\gamma$ 是正整数, 满足 $5 \\alpha-\\beta+4 \\gamma=0, \\alpha(1+x)=\\beta(1-x)=\\gamma(2 x-1)$. 后者即\n$$\n\\frac{\\beta-\\alpha}{\\beta+\\alpha}=\\frac{\\beta+\\gamma}{2 \\gamma+\\beta}\n$$\n代入 $\\beta=5 \\alpha+4 \\gamma$, 得\n$$\n0=2\\left(3 \\alpha \\gamma+5 \\alpha^2-2 \\gamma^2\\right)=2(5 \\alpha-2 \\gamma)(\\alpha+\\gamma),\n$$\n我们取 $(\\alpha, \\beta, \\gamma)=(2,30,5)$, 由平均不等式得,\n$$\n[2(1+x)]^5[30(1-x)][5(2 x-1)]^2 \\leqslant\\left(\\frac{15}{4}\\right)^8,\n$$\n此时 $x=\\frac{7}{8}$. 所以, 当 $x=\\frac{7}{8}$ 时, $(1+x)^5(1-x)(1-2 x)^2$ 的最大值为 $\\frac{3^7 \\cdot 5^5}{2^{22}}$.", + "remark": "", + "figures": [] +} \ No newline at end of file diff --git a/processed_dataset/calculation/0860.json b/processed_dataset/calculation/0860.json new file mode 100644 index 0000000000000000000000000000000000000000..738f5b878048ecb51c4e0cfcb7d9683b1efc2333 --- /dev/null +++ b/processed_dataset/calculation/0860.json @@ -0,0 +1,8 @@ +{ + "source_file": "./raw_volume-zh/volume5/chapter1.tex", + "problem_type": "calculation", + "problem": "例19. 求函数 $f_n\\left(x_1, x_2, \\cdots, x_n\\right)=\\frac{x_1}{\\left(1+x_1+\\cdots+x_n\\right)^2}+ \\frac{x_2}{\\left.1+x_2+\\cdots+x_n\\right)^2}+\\cdots+\\frac{x_n}{\\left(1+x_n\\right)^2}$ 的最大值 $m_n$ (其中 $x_i \\geqslant 0$ ). 用 $m_{n-1}$ 表宗 $m_n$, 并求 $\\lim _{n \\rightarrow \\infty} m_n$.", + "solution": "分析:$f_n$ 的每一项分母都很复杂, 自然应先作代换将其简化.\n解令 $a_i=\\frac{1}{1+x_i+\\cdots+x_n}, 1 \\leqslant i \\leqslant n$, 并约定 $a_{n+1}=1$. 则\n$$\n\\begin{gathered}\n1+x_i+x_{i+1}+\\cdots+x_n=\\frac{1}{a_i} . \\\\\n1+x_{i+1}+x_{i+2}+\\cdots+x_n=\\frac{1}{a_{i+1}}, \\\\\nx_i=\\frac{1}{a_i}-\\frac{1}{a_{i+1}} . \\\\\nf_n=\\sum_{i=1}^n a_i^2\\left(\\frac{1}{a_i}-\\frac{1}{a_{i+1}}\\right)=\\sum_{i=1}^n\\left(a_i-\\frac{a_i^2}{a_{i+1}}\\right) \\\\\n=\\left(a_1+a_2+\\cdots+a_n\\right)-\\left(\\frac{a_1^2}{a_2}+\\frac{a_2^2}{a_3}+\\cdots+\\frac{a_n^2}{1}\\right) .\n\\end{gathered}\n$$\n因此\n$$\n\\begin{aligned}\nf_n & =\\sum_{i=1}^n a_i^2\\left(\\frac{1}{a_i}-\\frac{1}{a_{i+1}}\\right)=\\sum_{i=1}^n\\left(a_i-\\frac{a_i^2}{a_{i+1}}\\right) \\\\\n& =\\left(a_1+a_2+\\cdots+a_n\\right)-\\left(\\frac{a_1^2}{a_2}+\\frac{a_2^2}{a_3}+\\cdots+\\frac{a_n^2}{1}\\right) .\n\\end{aligned}\n$$\n为求 $f_n$ 之最大值, 构造下列不等式:\n$$\n\\left\\{\\begin{array}{l}\n\\frac{a_1^2}{a_2}+\\lambda_1^2 a_2 \\geqslant 2 \\lambda_1 a_1, \\\\\n\\frac{a_2^2}{a_3}+\\lambda_2^2 a_3 \\geqslant 2 \\lambda_2 a_2, \\\\\n\\cdots \\cdots \\cdots \\cdots \\cdots \\cdots \\cdots \\cdots . . . \\cdots \\cdots \\cdots . . . \\\\\n\\frac{a_n^2}{1}+\\lambda_n^2 \\geqslant 2 \\lambda_n a_n .\n\\end{array}\\right. \\label{(1)}\n$$\n其中 $\\lambda_1, \\lambda_2, \\cdots, \\lambda_n$ 为参数, $\\lambda_i \\geqslant 0$.\n将(1)中 $n$ 个不等式相加, 只须使\n$$\n\\left\\{\\begin{array}{l}\n2 \\lambda_1=1 \\\\\n2 \\lambda_2=1+\\lambda_1^2 \\\\\n\\cdots \\cdots \\cdots \\cdots \\cdots \\cdots \\\\\n2 \\lambda_n=1+\\lambda_{n-1}^2\n\\end{array}\\right. \\label{(2)}\n$$\n即有 $f_n \\leqslant \\lambda_n^2$.\n注意到 $\\lambda_i \\geqslant \\lambda_{i-1}$, 且 $0 \\leqslant \\lambda_i \\leqslant 1$, 故 $\\lim _{n \\rightarrow \\infty} \\lambda_n$ 存在, 易见它的值为 1 .", + "remark": "", + "figures": [] +} \ No newline at end of file diff --git a/processed_dataset/calculation/0861.json b/processed_dataset/calculation/0861.json new file mode 100644 index 0000000000000000000000000000000000000000..2b7b5ccee8011466a391f51807234b73e77d7a6a --- /dev/null +++ b/processed_dataset/calculation/0861.json @@ -0,0 +1,8 @@ +{ + "source_file": "./raw_volume-zh/volume5/chapter1.tex", + "problem_type": "calculation", + "problem": "例28. 设 $a, b \\in \\mathbf{R}^{+}$.\n(1) 求 $S=\\frac{(a+1)^2}{b}+\\frac{(b+3)^2}{a}$ 的最小值;\n(2) 求 $T=\\frac{(a+1)^3}{b^2}+\\frac{(b+3)^3}{a^2}$ 的最小值.", + "solution": "解:(1) 由柯西不等式, 得所以\n$$\n\\begin{aligned}\n& S \\cdot(b+a) \\geqslant(a+1+b+3)^2, \\\\\n& S \\geqslant \\frac{(a+b+4)^2}{a+b}=(a+b)+\\frac{16}{a+b}+8 \\\\\n& \\geqslant 2 \\sqrt{16}+8=16,\n\\end{aligned}\n$$\n故 $S$ 的最小值为 16 .\n(2) 由(3) (Hölder 不等式),有\n$$\n\\left(\\frac{(a+1)^3}{b^2}+\\frac{(b+3)^3}{a^2}\\right)(b+a)(b+a) \\geqslant(a+1+b+3)^3,\n$$\n所以\n$$\n\\begin{aligned}\nT & \\geqslant \\frac{(a+b+4)^3}{(a+b)^2}=x+12+\\frac{48}{x}+\\frac{64}{x^2}(\\text { 记 } a+b=x) \\\\\n& =x+\\frac{64}{x}+\\left(\\frac{64}{x^2}-\\frac{16}{x}+1\\right)+11 \\\\\n& =\\left(x+\\frac{64}{x}\\right)+\\left(\\frac{8}{x}-1\\right)^2+11 \\geqslant 2 \\sqrt{64}+0+11 \\\\\n& =27 .\n\\end{aligned}\n$$\n当 $a=\\frac{22}{5}, b=\\frac{18}{5}$ 时等号成立.\n所以 $T$ 的最小值为 27 .", + "remark": "", + "figures": [] +} \ No newline at end of file diff --git a/processed_dataset/calculation/0862.json b/processed_dataset/calculation/0862.json new file mode 100644 index 0000000000000000000000000000000000000000..594bf8ad835bdd83c1bde3bb73bce56d73cfdd61 --- /dev/null +++ b/processed_dataset/calculation/0862.json @@ -0,0 +1,8 @@ +{ + "source_file": "./raw_volume-zh/volume5/chapter3.tex", + "problem_type": "calculation", + "problem": "例5. 设 $a 、 b 、 c$ 是正实数, 求\n$$\n\\frac{a+3 c}{a+2 b+c}+\\frac{4 b}{a+b+2 c}-\\frac{8 c}{a+b+3 c}\n$$\n的最小值.", + "solution": "解:令\n$$\n\\left\\{\\begin{array}{l}\nx=a+2 b+c, \\\\\ny=a+b+2 c, \\\\\nz=a+b+3 c,\n\\end{array}\\right.\n$$\n则有 $x-y=b-c, z-y=c$, 由此可得\n$$\n\\left\\{\\begin{array}{l}\na+3 c=2 y-x, \\\\\nb=z+x-2 y, \\\\\nc=z-y .\n\\end{array}\\right.\n$$\n从而\n$$\n\\begin{aligned}\n& \\frac{a+3 c}{a+2 b+c}+\\frac{4 b}{a+b+2 c}-\\frac{8 c}{a+b+3 c} \\\\\n= & \\frac{2 y-x}{x}+\\frac{4 \\cdot(z+x-2 y)}{y}-\\frac{8(z-y)}{z} \\\\\n= & -17+2 \\frac{y}{x}+4 \\frac{x}{y}+4 \\frac{z}{y}+8 \\frac{y}{z} \\\\\n\\geqslant & -17+2 \\sqrt{8}+2 \\sqrt{32}=-17+12 \\sqrt{2} .\n\\end{aligned}\n$$\n上式中的等号可以成立.\n事实上, 由上述推导过程知, 等号成立, 当且仅当平均不等式中的等号成立, 而这等价于\n$$\n\\left\\{\\begin{array} { l } \n{ 2 \\frac { y } { x } = 4 \\frac { x } { y } , } \\\\\n{ 4 \\frac { z } { y } = 8 \\frac { y } { z } , }\n\\end{array} \\text { 即 } \\left\\{\\begin{array} { l } \n{ y ^ { 2 } = 2 x ^ { 2 } , } \\\\\n{ z ^ { 2 } = 2 y ^ { 2 } , }\n\\end{array} \\text { 即 } \\left\\{\\begin{array}{l}\ny=\\sqrt{2} x, \\\\\nz=2 x,\n\\end{array}\\right.\\right.\\right.\n$$\n亦即\n$$\n\\left\\{\\begin{array}{l}\na+b+2 c=\\sqrt{2}(a+2 b+c), \\\\\na+b+3 c=2(a+2 b+c) .\n\\end{array}\\right.\n$$\n解该不定方程,得到\n$$\n\\left\\{\\begin{array}{l}\nb=(1+\\sqrt{2}) a, \\\\\nc=(4+3 \\sqrt{2}) a .\n\\end{array}\\right.\n$$\n不难算出, 对任何正实数 $a$, 只要 $b=(1+\\sqrt{2}) a, c=(4+3 \\sqrt{2}) a$, 就都有\n$$\n\\frac{a+3 c}{a+2 b+c}+\\frac{4 b}{a+b+2 c}-\\frac{8 c}{a+b+3 c}=-17+12 \\sqrt{2},\n$$\n所以所求的最小值为 $-17+12 \\sqrt{2}$.", + "remark": "", + "figures": [] +} \ No newline at end of file diff --git a/processed_dataset/calculation/0863.json b/processed_dataset/calculation/0863.json new file mode 100644 index 0000000000000000000000000000000000000000..edace0049f3592f6ae49b1a1ee0140e9b6b098aa --- /dev/null +++ b/processed_dataset/calculation/0863.json @@ -0,0 +1,8 @@ +{ + "source_file": "./raw_volume-zh/volume5/chapter3.tex", + "problem_type": "calculation", + "problem": "例11. 求最大的正整数 $n$, 使得存在 $n$ 个不同的实数 $x_1, x_2, \\cdots, x_n$, 满足: 对任意 $1 \\leqslant i\\pi$. 所以\n$$\n9 \\theta<\\pi<10 \\theta .\n$$\n又因为 $\\pi-\\theta \\geqslant \\alpha_n-\\alpha_1 \\geqslant(n-1) \\theta$, 则 $n \\leqslant 9$.\n当 $\\alpha_1=0, \\alpha_2=\\theta, \\alpha_3=2 \\theta, \\cdots, \\alpha_9=8 \\theta$ 时, 等号可以取到.\n故 $n$ 的最大值为 9 .", + "remark": "", + "figures": [] +} \ No newline at end of file diff --git a/processed_dataset/calculation/0864.json b/processed_dataset/calculation/0864.json new file mode 100644 index 0000000000000000000000000000000000000000..31b8ffb732ac9396970fdae4374b8130bac238cc --- /dev/null +++ b/processed_dataset/calculation/0864.json @@ -0,0 +1,8 @@ +{ + "source_file": "./raw_volume-zh/volume5/chapter5.tex", + "problem_type": "calculation", + "problem": "例16. 对于给定的大于 1 的正整数 $n$, 是否存在 $2 n$ 个两两不同的正整数 $a_1, a_2, \\cdots, a_n ; b_1, b_2, \\cdots, b_n$, 同时满足以下条件:\n(1) $a_1+a_2+\\cdots+a_n=b_1+b_2+\\cdots+b_n$;\n(2) $n-1>\\sum_{i=1}^n \\frac{a_i-b_i}{a_i+b_i}>n-1-\\frac{1}{2002}$.", + "solution": "解:答案是肯定的.\n取 $\\quad a_1=N+1, a_2=N+2, \\cdots, a_{n-1}=N+(n-1)$ ;\n$$\nb_1=1, b_2=2, \\cdots, b_{n-1}=n-1 .\n$$\n于是, 由(1)有 $b_n-a_n=N(n-1)$. 令 $a_n=N^2, b_n=N^2+N(n-1)$.\n因此\n$$\n\\begin{aligned}\n\\sum_{i=1}^n \\frac{a_i-b_i}{a_i+b_i} & =n-1-\\left(\\frac{2}{N+2}+\\cdots+\\frac{2(n-1)}{2(n-1)+N}+\\frac{n-1}{2 N+(n-1)}\\right) \\\\\n& 2002 n(n-1)$, 则\n$$\n\\sum_{i=1}^n \\frac{a_i-b_i}{a_i+b_i}>n-1-\\frac{2 n(n-1)}{N}>n-1-\\frac{1}{2002} .\n$$", + "remark": "", + "figures": [] +} \ No newline at end of file diff --git a/processed_dataset/calculation/0865.json b/processed_dataset/calculation/0865.json new file mode 100644 index 0000000000000000000000000000000000000000..52c1340c9f1a86145a54e098b0cd17b8361a3315 --- /dev/null +++ b/processed_dataset/calculation/0865.json @@ -0,0 +1,8 @@ +{ + "source_file": "./raw_volume-zh/volume5/chapter8.tex", + "problem_type": "calculation", + "problem": "例1. 求三位数 (十进制下) 与其各位数字之和的比的最小值.", + "solution": "解:设此三位数为 $100 x+10 y+z$, 其中 $x 、 y 、 z$ 为整数, 且 $1 \\leqslant x \\leqslant 9$, $0 \\leqslant y, z \\leqslant 9$. 于是, 比值为\n$$\nf(x, y, z)=\\frac{100 x+10 y+z}{x+y+z}=1+\\frac{99 x+9 y}{x+y+z} . \\label{(1)}\n$$\n在(1)右端表达式中, 只有分母含有 $z$, 故当 $x 、 y$ 固定时, $f(x, y, z)$ 当且仅当 $z=9$ 时取最小值.\n同理可得\n$$\n\\begin{aligned}\nf(x, y, z) & \\geqslant 1+\\frac{99 x+9 y}{9+x+y}=10+\\frac{90 x-81}{9+x+y} \\\\\n& \\geqslant 10+\\frac{90 x-81}{18+x}=100-\\frac{1701}{18+x}\n\\end{aligned}\n$$\n$$\n\\geqslant 100-\\frac{1701}{18+1}=\\frac{199}{19}\n$$\n故\n$$\nf_{\\min }(x, y, z)=f(1,9,9)=\\frac{199}{19} \\text {. }\n$$", + "remark": "", + "figures": [] +} \ No newline at end of file diff --git a/processed_dataset/calculation/0866.json b/processed_dataset/calculation/0866.json new file mode 100644 index 0000000000000000000000000000000000000000..191c40023dc294aea744a686c688b19e02efc59b --- /dev/null +++ b/processed_dataset/calculation/0866.json @@ -0,0 +1,8 @@ +{ + "source_file": "./raw_volume-zh/volume5/chapter8.tex", + "problem_type": "calculation", + "problem": "例3. 设 $A 、 B 、 C 、 D$ 为空间四点, 连结 $A B 、 A C 、 A D 、 B C 、 B D 、 C D$ 中至多有一条的长度大于 1 , 试求六条线段长度之和的最大值.", + "solution": "解:设 $A D$ 为六条线段中最长的一条.\n(1) 将其余五条线段长度固定,易见当 $A$ 和 $D$ 为平行四边形 $A B D C$ 两相对顶点时, $A D$ 取得最大值.\n(2) 固定 $B 、 C$ 的位置,则 $A 、 D$ 必落在分别以 $B 、 C$ 为圆心, 1 为半径的两个圆相交的区域内.\n此时具有最大可能距离的唯一一对点恰为两圆的两个交点,当 $A 、 D$ 取这两点时, $A B 、 B D 、 A C 、 C D$ 均达到各自的最大值 1 .\n(3) 剩下的问题是当 $B C$ 变小时, $A D$ 变大; 当 $B C$ 变大时, $A D$ 变小,故我们应在其余四边固定为 1 的情况下探求 $B C+A D$ 的最大值.\n记 $\\angle A B O=\\theta$, 由 $A B=1,04$, 由于 $2+\\left(x_j-2\\right)= x_j, 2\\left(x_j-2\\right)=x_j+\\left(x_j-4\\right)>x_j$, 可用 2 和 $x_j-2$ 代替 $x_j$ 使和不变而积增大,矛盾!\n(2) $x_i \\geqslant 2(i=1,2, \\cdots, n)$. 若不然, 有某个 $x_k=1$, 由 $x_k \\cdot x_i=x_i< x_i+x_k$, 可用 $x_k+x_i$ 代替 $x_k x_i$ 而使和不变且积变大,矛盾!\n(3) 因为 $4=2+2=2 \\times 2$, 故 $x_j=4$ 可换为两个 2 而使 $u$ 不变.\n(4)设 $u=2^\\alpha \\cdot 3^\\beta$. 若 $\\alpha>3$, 则 $u=2^3 \\cdot 2^{\\alpha-3} \\cdot 3^\\beta<3^2 \\cdot 2^{\\alpha-3} \\cdot 3^\\beta$, 且 $2+ 2+2=3+3$ 和不改变, $u$ 却变大了, 矛盾!\n综上, 因为 $2011=3 \\times 670+1$, 故 $u_{\\max }=2^2 \\times 3^{669}$.\n一般地, 若和为 $m$, 则\n$$\nu_{\\max }=\\left\\{\\begin{array}{l}\n3^s, m=3 s ; \\\\\n2^2 \\cdot 3^{s-1}, m=3 s+1 ; \\\\\n2 \\cdot 3^s, m=3 s+2 .\n\\end{array}\\right.\n$$", + "remark": "", + "figures": [] +} \ No newline at end of file diff --git a/processed_dataset/calculation/0871.json b/processed_dataset/calculation/0871.json new file mode 100644 index 0000000000000000000000000000000000000000..7ca6c774a443fa759298e1485522aed02021e3ec --- /dev/null +++ b/processed_dataset/calculation/0871.json @@ -0,0 +1,8 @@ +{ + "source_file": "./raw_volume-zh/volume5/chapter8.tex", + "problem_type": "calculation", + "problem": "例12. 设 $a_1, a_2, \\cdots, a_n, \\cdots$ 是一个不减的正整数序列, 对于 $m \\geqslant 1$, 定义 $b_m=\\min \\left\\{n, a_n \\geqslant m\\right\\}$, 即 $b_m$ 是使 $a_n \\geqslant m$ 的 $n$ 的最小值.\n若已知 $a_{19}=85$, 求\n$$\na_1+a_2+\\cdots+a_{19}+b_1+b_2+\\cdots+b_{85}, \\label{(1)}\n$$\n的最大值.", + "solution": "解:若存在 $i$, 使得 $a_iy>z$, 则 $z \\geqslant 1, y \\geqslant 2, x \\geqslant 3$.\n由第(1)小题,\n$$\n\\begin{aligned}\n(x+y+z)^3-27 x y z \\geqslant & \\frac{6}{2} \\cdot\\left[(x-y)^2+(y-z)^2+(z-x)^2\\right]+3 \\cdot[3(y \\\\\n& \\left.-z)^2+2(z-x)^2+(x-y)^2\\right] \\\\\n\\geqslant & 3 \\cdot\\left(1^2+1^2+2^2\\right)+3 \\cdot\\left(3 \\cdot 1^2+2 \\cdot 2^2+1^2\\right) \\\\\n= & 54 .\n\\end{aligned}\n$$\n因此 $(x+y+z)^3-27 x y z \\geqslant 54$, 且等号当 $(x, y, z)=(3,2,1)$ 时取到.", + "remark": "说明第(1) 小题等价于下列命题:\n对于不全相等的正整数 $a 、 b 、 c$, 有\n$$\n\\frac{a+b+c}{3} \\geqslant \\sqrt[3]{a b c+\\frac{10}{27}} . \\label{eq1}\n$$\n当然, 式\\ref{eq1}也可以用以下方法(增量代换)来证明:\n证明不妨设 $1 \\leqslant a \\leqslant b \\leqslant c, b=a+x, c=a+y$. 则 $x, y \\geqslant 0$, 且 $x$ 、 $y$ 不全为 $0(x \\leqslant y)$.\n此时, \\ref{eq1}式等价于证明:\n$$\n9 a\\left(x^2-x y+y^2\\right)+(x+y)^3 \\geqslant 10 .\n$$\n由于 $a \\geqslant 1, x^2-x y+y^2 \\geqslant 1, x+y \\geqslant 1$, 上式显然成立, 并且等号当 $a=1, x^2-x y+y^2=1, x+y=1$, 即 $(a, x, y)=(1,0,1)$ 时取到.", + "figures": [] +} \ No newline at end of file diff --git a/processed_dataset/calculation/0873.json b/processed_dataset/calculation/0873.json new file mode 100644 index 0000000000000000000000000000000000000000..d7f0f098870afebb35bb15498d8d90817f1feab3 --- /dev/null +++ b/processed_dataset/calculation/0873.json @@ -0,0 +1,8 @@ +{ + "source_file": "./raw_volume-zh/volume5/chapter9.tex", + "problem_type": "calculation", + "problem": "例10. 实数 $a$ 使得对于任意实数 $x_1 、 x_2 、 x_3 、 x_4 、 x_5$, 不等式\n$$\nx_1^2+x_2^2+x_3^2+x_4^2+x_5^2 \\geqslant a\\left(x_1 x_2+x_2 x_3+x_3 x_4+x_4 x_5\\right)\n$$\n都成立, 求 $a$ 的最大值.", + "solution": "解:$a$ 的最大值为 $\\frac{2 \\sqrt{3}}{3}$.\n因为当 $x_1=1, x_2=\\sqrt{3}, x_3=2, x_4=\\sqrt{3}, x_5=1$ 时, 得 $a \\leqslant \\frac{2}{\\sqrt{3}}$.\n又由于当 $a=\\frac{2}{\\sqrt{3}}$ 时,不等式恒成立.\n事实上\n$$\n\\begin{aligned}\nx_1^2+x_2^2+x_3^2+x_4^2+x_5^2 & =\\left(x_1^2+\\frac{x_2^2}{3}\\right)+\\left(\\frac{2 x_2^2}{3}+\\frac{x_3^2}{2}\\right)+\\left(\\frac{x_3^2}{2}+\\frac{2 x_4^2}{3}\\right)+\\left(\\frac{x_4^2}{3}+x_5^2\\right) \\\\\n& \\geqslant \\frac{2}{\\sqrt{3}} x_1 x_2+\\frac{2}{\\sqrt{3}} x_2 x_3+\\frac{2}{\\sqrt{3}} x_3 x_4+\\frac{2}{\\sqrt{3}} x_4 x_5\n\\end{aligned}\n$$\n所以, $a$ 的最大值为 $\\frac{2 \\sqrt{3}}{3}$.", + "remark": "", + "figures": [] +} \ No newline at end of file diff --git a/processed_dataset/calculation/0874.json b/processed_dataset/calculation/0874.json new file mode 100644 index 0000000000000000000000000000000000000000..a0260323f274f4fed5127dceba11f41bd242049e --- /dev/null +++ b/processed_dataset/calculation/0874.json @@ -0,0 +1,8 @@ +{ + "source_file": "./raw_volume-zh/volume5/chapter9.tex", + "problem_type": "calculation", + "problem": "例11. 已知 $a 、 b 、 c$ 为正数, $a+b+c=10$, 且 $a \\leqslant 2 b, b \\leqslant 2 c, c \\leqslant 2 a$, 求 $a b c$ 的最小值.", + "solution": "解:令 $x=2 b-a, y=2 c-b, z=2 a-c$,\n则\n$$\nx+y+z=10, x \\geqslant 0, y \\geqslant 0, z \\geqslant 0,\n$$\n且 $\\quad a=\\frac{x+2 y+4 z}{7}, b=\\frac{y+2 z+4 x}{7}, c=\\frac{z+2 x+4 y}{7}$,\n故 $\\quad a b c=\\frac{1}{343} \\cdot[(x+2 y+4 z)(y+2 z+4 x)(z+2 x+4 y)]$,\n而\n$$\n\\begin{aligned}\n& (x+2 y+4 z)(y+2 z+4 x)(z+2 x+4 y) \\\\\n= & (10+y+3 z)(10+z+3 x)(10+x+3 y) \\\\\n= & 1000+400(x+y+z)+130(x y+y z+z x)+30\\left(x^2+y^2+z^2\\right)+\\left(3 y^2 z\\right. \\\\\n& \\left.+3 x^2 y+9 x y^2+3 x z^2+9 y z^2+9 x^2 z+28 x y z\\right) \\\\\n= & 1000+4000+30(x+y+z)^2+\\left[70(x y+y z+z x)+3\\left(y^2 z+x^2 y+\\right.\\right. \\\\\n& \\left.\\left.x z^2\\right)+9\\left(x y^2+y z^2+x^2 z\\right)+28 x y z\\right] \\\\\n\\geqslant & 8000 .\n\\end{aligned}\n$$\n等号当且仅当 $x=y=0, z=10$ 时取到, 此时 $a=\\frac{40}{7}, b=\\frac{20}{7}, c=\\frac{10}{7}$.\n故 $a b c_{\\min }=\\frac{8000}{343}$.", + "remark": "", + "figures": [] +} \ No newline at end of file diff --git a/processed_dataset/calculation/0875.json b/processed_dataset/calculation/0875.json new file mode 100644 index 0000000000000000000000000000000000000000..1a400ef0ed2746c1b032c7a4a5994705505946db --- /dev/null +++ b/processed_dataset/calculation/0875.json @@ -0,0 +1,8 @@ +{ + "source_file": "./raw_volume-zh/volume5/chapter9.tex", + "problem_type": "calculation", + "problem": "例12. 设 $a, b, c, d \\in \\mathbf{R}^{+}, a b c d=1$, 令 $T=a(b+c+d)+b(c+d)+c d$.\n(1) 求 $a^2+b^2+T$ 的最小值;\n(2) 求 $a^2+b^2+c^2+T$ 的最小值.", + "solution": "分析:对 (1), 我们把 $a 、 b$ 放在一起, $c 、 d$ 放在一起考虑, 对 $T$ 作适当变形.\n对 (2),则应看到 $a 、 b 、 c$ 的地位是相同的.\n解 (1)\n$$\n\\begin{aligned}\na^2+b^2+T & =a^2+b^2+(a+b)(c+d)+a b+c d \\\\\n& \\geqslant 2 a b+2 \\sqrt{a b} \\cdot 2 \\sqrt{c d}+a b+c d \\\\\n& =4+3 a b+c d \\geqslant 4+2 \\cdot \\sqrt{3 a b c d} \\\\\n& =4+2 \\sqrt{3} .\n\\end{aligned}\n$$\n当 $a=b, c=d, 3 a b=c d$, 即 $a=b=\\left(\\frac{1}{3}\\right)^{\\frac{1}{4}}, c=d=3^{\\frac{1}{4}}$ 时等号成立.\n所以, $a^2+b^2+T$ 的最小值为 $4+2 \\sqrt{3}$.\n$$\n\\begin{aligned}\na^2+b^2+c^2+T & =a^2+b^2+c^2+(a+b+c) d+a b+b c+c a \\\\\n& \\geqslant 3 \\sqrt[3]{a^2 b^2 c^2}+3 \\sqrt[3]{a b c} \\cdot d+3 \\cdot \\sqrt[3]{(a b c)^2} \\\\\n& =3 \\cdot\\left[2 \\sqrt[3]{(a b c)^2}+\\sqrt[3]{a b c} \\cdot d\\right] \\\\\n& \\geqslant 3 \\cdot 2 \\cdot \\sqrt{2 \\sqrt[3]{(a b c)^2}} \\cdot \\sqrt[3]{a b c} \\cdot d \\\\\n& =6 \\sqrt{2}\n\\end{aligned}\n$$\n等号当 $a=b=c=\\left(\\frac{1}{2}\\right)^{\\frac{1}{4}}, d=2^{\\frac{3}{1}}$ 时取到.\n所以, $a^2+b^2+c^2+T$ 的最小值为 $6 \\sqrt{2}$.", + "remark": "", + "figures": [] +} \ No newline at end of file diff --git a/processed_dataset/calculation/0876.json b/processed_dataset/calculation/0876.json new file mode 100644 index 0000000000000000000000000000000000000000..fdc815d011b4d8a0b7a3da5482fd4258dd1823ed --- /dev/null +++ b/processed_dataset/calculation/0876.json @@ -0,0 +1,8 @@ +{ + "source_file": "./raw_volume-zh/volume5/exercise1.tex", + "problem_type": "calculation", + "problem": "问题5. 设实数 $a_1, a_2, \\cdots, a_{100}$ 满足:\n(1) $a_1 \\geqslant a_2 \\geqslant \\cdots \\geqslant a_{100} \\geqslant 0$;\n(2) $a_1+a_2 \\leqslant 100$;\n(3) $a_3+a_4+\\cdots+a_{100} \\leqslant 100$.\n求 $a_1^2+a_2^2+\\cdots+a_{100}^2$ 的最大值.", + "solution": "解:$$\n\\begin{aligned}\n& a_1^2+a_2^2+\\cdots+a_{100}^2 \\leqslant\\left(100-a_2\\right)^2+a_2^2+a_3^2+\\cdots+a_{100}^2 \\\\\n\\leqslant & 100^2-\\left(a_1+a_2+\\cdots+a_{100}\\right) a_2+2 a_2^2+a_3^2+\\cdots+a_{100}^2 \\\\\n= & 100^2-a_2\\left(a_1-a_2\\right)-a_3\\left(a_2-a_3\\right)-\\cdots-a_{100}\\left(a_2-a_{100}\\right) \\\\\n\\leqslant & 100^2 .\n\\end{aligned}\n$$\n等号成立, 当 $a_1=100, a_i=0(i>1)$ 或者 $a_1=a_2=a_3=a_4=50, a_j= 0(j>4)$ 时.", + "remark": "", + "figures": [] +} \ No newline at end of file diff --git a/processed_dataset/calculation/0877.json b/processed_dataset/calculation/0877.json new file mode 100644 index 0000000000000000000000000000000000000000..34d985efb197c50a3720b7ba880b592a8bd17c32 --- /dev/null +++ b/processed_dataset/calculation/0877.json @@ -0,0 +1,8 @@ +{ + "source_file": "./raw_volume-zh/volume5/exercise1.tex", + "problem_type": "calculation", + "problem": "问题10. 设 $a 、 b$ 是正常数, $\\theta \\in\\left(0, \\frac{\\pi}{2}\\right)$, 求 $y=a \\sqrt{\\sin \\theta}+b \\sqrt{\\cos \\theta}$ 的最大值.", + "solution": "设 $\\sqrt{u}=a \\cdot \\sqrt{\\sin \\theta}, \\sqrt{v}=b \\cdot \\sqrt{\\cos \\theta}$, 则条件转化为: $\\frac{u^2}{a^4}+\\frac{v^2}{b^4}=1, u$ 、 $v \\geqslant 0$, 利用 Cauchy 不等式,\n$$\n\\begin{aligned}\ny & =\\sqrt{u}+\\sqrt{v} \\leqslant \\sqrt{\\left(\\frac{u}{a^{\\frac{1}{3}}}+\\frac{v}{b^{\\frac{1}{3}}}\\right)\\left(a^{\\frac{4}{3}}+b^{\\frac{4}{3}}\\right)} \\\\\n& \\leqslant \\sqrt{\\sqrt{\\left(\\frac{u^2}{a^4}+\\frac{v^2}{b^4}\\right)\\left(a^{\\frac{1}{3}}+b^{\\frac{1}{3}}\\right)} \\cdot\\left(a^{\\frac{4}{3}}+b^{\\frac{4}{3}}\\right)} \\\\\n& =\\left(a^{\\frac{1}{3}}+b^{\\frac{1}{3}}\\right)^{\\frac{3}{4}},\n\\end{aligned}\n$$\n并且易求得等号成立的条件.", + "remark": "注:: 在求解过程中, 幂次都可以用待定系数法来确定.", + "figures": [] +} \ No newline at end of file diff --git a/processed_dataset/calculation/0878.json b/processed_dataset/calculation/0878.json new file mode 100644 index 0000000000000000000000000000000000000000..ad8276025ec196e5ca79cfed05c67e3c959d81c4 --- /dev/null +++ b/processed_dataset/calculation/0878.json @@ -0,0 +1,8 @@ +{ + "source_file": "./raw_volume-zh/volume5/exercise2.tex", + "problem_type": "calculation", + "problem": "问题2. 设 $0a_{k-1}$, 且 $a_k \\geqslant 2 . k=1,2, \\cdots, n-1$.\n由条件可得\n$$\n\\frac{a_{k-1}}{a_k} \\leqslant \\frac{a_{k-1}}{a_k-1}-\\frac{a_k}{a_{k+1}-1} .\n$$\n上面不等式对 $k=i+1, i+2, \\cdots, n$ 求和, 得\n$$\n\\frac{a_i}{a_{i+1}}+\\frac{a_{i+1}}{a_{i+2}}+\\cdots+\\frac{a_{n-1}}{a_n}<\\frac{a_i}{a_{i+1}-1} .\n$$\n当 $i=0$ 时, 由题设及上式, $\\frac{1}{a_1} \\leqslant \\frac{99}{100}<\\frac{1}{a_1-1}$. 则\n$$\n\\frac{100}{99} \\leqslant a_1<\\frac{100}{99}+1 \\text {, 故 } a_1=2 \\text {. }\n$$\n当 $i=1 、 2 、 3$ 时, 同样可求得 $a_2=5, a_3=56, a_4=78400$.\n所以 $\\frac{1}{a_5} \\leqslant \\frac{1}{a_4}\\left(\\frac{99}{100}-\\frac{1}{2}-\\frac{2}{5}-\\frac{5}{56}-\\frac{56}{25-56^2}\\right)=0$, 故没有 $a_5$, 因此, 所求的正整数 $a_1=2, a_2=5, a_3=56, a_4=78400$.", + "remark": "", + "figures": [] +} \ No newline at end of file diff --git a/processed_dataset/calculation/0881.json b/processed_dataset/calculation/0881.json new file mode 100644 index 0000000000000000000000000000000000000000..efb773d1d76a6aa40346b244cde387640e8c4f2b --- /dev/null +++ b/processed_dataset/calculation/0881.json @@ -0,0 +1,8 @@ +{ + "source_file": "./raw_volume-zh/volume5/exercise2.tex", + "problem_type": "calculation", + "problem": "问题11. (W. Janous 不等式推广形式)\n设 $a_1, a_2, \\cdots, a_n \\in \\mathbf{R}^{+}, p, q \\in \\mathbf{R}^{+}$. 记 $S=\\left(a_1^p+a_2^p+\\cdots+a_n^p\\right)^{\\frac{1}{p}}$, 则对于 $1,2, \\cdots, n$ 的任一排列 $i_1, i_2, \\cdots, i_n$, 有: $\\sum_{k=1}^n \\frac{a_k^q-a_{i_k}^q}{S^p-a_k^p} \\geqslant 0$.", + "solution": "由对称性, 不妨设 $a_1 \\geqslant a_2 \\geqslant \\cdots \\geqslant a_n$, 则\n$$\n\\sum_{j=1}^k a_j^q \\geqslant \\sum_{j=1}^k a_{i_j}^q(1 \\leqslant k \\leqslant n-1), \\sum_{j=1}^n a_j^q=\\sum_{j=1}^n a_{i_j}^q,\n$$\n且 $\\left(S^p-a_k^p\\right)^{-1} \\geqslant\\left(S^p-a_{k+1}^p\\right)^{-1}(1 \\leqslant k \\leqslant n-1)$.\n由分部求和公式, 有 $\\sum_{k=1}^n \\frac{a_k^q-a_{k_k}^q}{S^p-a_k^p}=\\left(S^p-a_n^p\\right)^{-1} \\cdot \\sum_{j=1}^n\\left(a_j^q-a_{i_j}^q\\right)+ \\sum_{k=1}^{n-1}\\left[\\sum_{j=1}^k\\left(a_j^q-a_{i_j}^q\\right)\\right] \\cdot\\left[\\left(S^p-a_k^p\\right)^{-1}-\\left(S^p-a_{k+1}^p\\right)^{-1}\\right] \\geqslant 0$, 故原不等式成立.", + "remark": "", + "figures": [] +} \ No newline at end of file diff --git a/processed_dataset/calculation/0882.json b/processed_dataset/calculation/0882.json new file mode 100644 index 0000000000000000000000000000000000000000..0816b42e9041faf11f19315148860738740cd7b7 --- /dev/null +++ b/processed_dataset/calculation/0882.json @@ -0,0 +1,8 @@ +{ + "source_file": "./raw_volume-zh/volume5/exercise3.tex", + "problem_type": "calculation", + "problem": "问题1. 已知 $a, b, c \\in \\mathbf{R}^{+}$. 求 $\\frac{a}{b+3 c}+\\frac{b}{8 c+4 a}+\\frac{9 c}{3 a+2 b}$ 的最小值.", + "solution": "令 $b+3 c=x, 8 c+4 a=y, 3 a+2 b=z$, 则原式 $=\\frac{1}{8}\\left(\\frac{y}{x}+\\frac{4 x}{y}\\right)+\\frac{1}{6}\\left(\\frac{z}{x}+\\frac{9 x}{z}\\right)+\\frac{1}{16}\\left(\\frac{4 z}{y}+\\frac{9 y}{z}\\right)-\\frac{61}{48} \\geqslant \\frac{47}{48}$.", + "remark": "", + "figures": [] +} \ No newline at end of file diff --git a/processed_dataset/calculation/0883.json b/processed_dataset/calculation/0883.json new file mode 100644 index 0000000000000000000000000000000000000000..717fdef9e5178061db03145847a41799ca295289 --- /dev/null +++ b/processed_dataset/calculation/0883.json @@ -0,0 +1,8 @@ +{ + "source_file": "./raw_volume-zh/volume5/exercise3.tex", + "problem_type": "calculation", + "problem": "问题2. 若椭圆 $\\frac{x^2}{m^2}+\\frac{y^2}{n^2}=1(m, n>0)$ 经过点 $p(a, b)(a b \\neq 0,|a| \\neq|b|)$, 求 $m+n$ 的最小值.", + "solution": "不妨设 $m, n \\in \\mathbf{R}^{+}, a, b \\in \\mathbf{R}^{+}$, 令 $a=m \\cos \\alpha, b=n \\sin \\alpha$, 其中 $\\alpha \\in$\n$$\n\\begin{aligned}\n& \\left(0, \\frac{\\pi}{2}\\right] \\text {, 则 }(m+n)^2=\\left(\\frac{a}{\\cos \\alpha}+\\frac{b}{\\sin \\alpha}\\right)^2=\\frac{a^2}{\\cos ^2 \\alpha}+\\frac{b^2}{\\sin ^2 \\alpha}+\\frac{2 a b}{\\sin \\alpha \\cos \\alpha}= \\\\\n& a^2\\left(1+\\tan ^2 \\alpha\\right)+b^2\\left(1+\\frac{1}{\\tan ^2 \\alpha}\\right)+\\frac{2 a b}{\\frac{\\left(1+\\tan ^2 \\alpha\\right.}{\\tan \\alpha}} .\n\\end{aligned}\n$$\n记 $\\tan \\alpha=t$, 则 $t \\in \\mathbf{R}^{+}$, 故\n$$\n\\begin{aligned}\n(m+n)^2 & =\\left(a^2+b^2\\right)+a^2 t^2+\\frac{b^2}{t^2}+\\frac{2 a b}{t}+2 a b t \\\\\n& =a\\left(a t^2+\\frac{b}{t}+\\frac{b}{t}\\right)+b\\left(\\frac{b}{t^2}+a t+a t\\right)+\\left(a^2+b^2\\right) \\\\\n& \\geqslant a \\cdot 3 \\sqrt[3]{a b^2}+b \\cdot 3 \\sqrt[3]{a^2 b}+\\left(a^2+b^2\\right) .\n\\end{aligned}\n$$\n因此 $(m+n)_{\\min }=\\left(a^{\\frac{2}{3}}+b^{\\frac{2}{3}}\\right)^{\\frac{3}{2}}$, 等号当 $t=\\sqrt[3]{\\frac{b}{a}}$ 时成立.", + "remark": "注::也可以用 Cauchy 不等式来解.\n由于 $\\left(a^{\\frac{1}{3}} \\cos \\alpha+b^{\\frac{1}{3}} \\sin \\alpha\\right)^2 \\leqslant\\left(a^{\\frac{2}{3}}+b^{\\frac{2}{3}}\\right)\\left(\\sin ^2 \\alpha+\\cos ^2 \\alpha\\right)$, 故 $\\left(a^{\\frac{2}{3}}+b^{\\frac{2}{3}}\\right)^{\\frac{1}{2}}\\left(\\frac{a}{\\cos \\alpha}+\\frac{b}{\\sin \\alpha}\\right) \\geqslant\\left(a^{\\frac{1}{3}} \\cos \\alpha+b^{\\frac{1}{3}} \\sin \\alpha\\right)\\left(\\frac{a}{\\cos \\alpha}+\\frac{b}{\\sin \\alpha}\\right) \\geqslant\\left(a^{\\frac{2}{3}}+\\right. \\left.b^{\\frac{2}{3}}\\right)^2$, 即 $m+n \\geqslant\\left(a^{\\frac{2}{3}}+b^{\\frac{2}{3}}\\right)^{\\frac{3}{2}}$, 且当 $\\alpha=\\arctan \\sqrt[3]{\\frac{a}{b}}$ 时等号成立.", + "figures": [] +} \ No newline at end of file diff --git a/processed_dataset/calculation/0884.json b/processed_dataset/calculation/0884.json new file mode 100644 index 0000000000000000000000000000000000000000..dd73045a76c22d4360cb7dd9a8096296cce28ce4 --- /dev/null +++ b/processed_dataset/calculation/0884.json @@ -0,0 +1,8 @@ +{ + "source_file": "./raw_volume-zh/volume5/exercise3.tex", + "problem_type": "calculation", + "problem": "问题10. 设 $x, y, z \\in \\mathbf{R}^{+}$, 且满足 $x y z+x+z=y$, 求 $p=\\frac{2}{x^2+1}-\\frac{2}{y^2+1}+\\frac{3}{z^2+1}$ 的最大值.", + "solution": "由已知, $x+(-y)+z=x \\cdot(-y) \\cdot z$.\n设 $\\dot{x}=\\tan \\alpha, y=-\\tan \\beta, z=\\tan \\gamma,(\\alpha+\\beta+\\gamma=k \\pi)$.\n故 $p=2 \\cos ^2 \\alpha-2 \\cos ^2 \\beta+3 \\cos ^2 \\gamma=2 \\cos ^2 \\alpha-2 \\cos ^2 \\beta+3 \\cos ^2(\\alpha+\\beta)= -2 \\sin (\\alpha+\\beta) \\sin (\\alpha-\\beta)+3 \\cos ^2 \\gamma \\leqslant 2 \\sin \\gamma+3-3 \\sin ^2 \\gamma=-3\\left(\\sin \\gamma-\\frac{1}{3}\\right)^2+ \\frac{10}{3} \\leqslant \\frac{10}{3}$, 因此 $p_{\\max }=\\frac{10}{3}$.", + "remark": "", + "figures": [] +} \ No newline at end of file diff --git a/processed_dataset/calculation/0885.json b/processed_dataset/calculation/0885.json new file mode 100644 index 0000000000000000000000000000000000000000..e1ae16a8487519e52e443ad3d63e661fc0db6adc --- /dev/null +++ b/processed_dataset/calculation/0885.json @@ -0,0 +1,8 @@ +{ + "source_file": "./raw_volume-zh/volume5/exercise5.tex", + "problem_type": "calculation", + "problem": "问题3. 设 $F(x)=|f(x) \\cdot g(x)|$, 其中 $f(x)=a x^2 \\dashv b x+c, x \\in[-1,1]$; $g(x)=c x^2+b x+a, x \\in[-1,1]$, 且对任意参数 $a 、 b 、 c$, 恒有 $|f(x)| \\leqslant 1$, 求 $F(x)$ 的最大值.", + "solution": "$|g(x)|=\\left|c x^2-c+b x+a+c\\right| \\leqslant|c| \\cdot\\left|x^2-1\\right|+|b x+a+c| \\leqslant 1+|b x+a+c|$.\n考察一次函数 $T(x)=|b x+a+c|,-1 \\leqslant x \\leqslant 1$.\n当 $x=-1$ 时, $T(-1)=|a-b+c| \\leqslant 1$; 当 $x=1$ 时, $T(1)=\\mid a+b+ c \\mid \\leqslant 1$. 故当 $|x| \\leqslant 1$ 时, $T(x) \\leqslant 1$. 则 $|g(x)| \\leqslant 1+T(x) \\leqslant 2$. 于是 $F(x) \\leqslant 2$. 且当 $x= \\pm 1$ 时取到此最值.", + "remark": "", + "figures": [] +} \ No newline at end of file diff --git a/processed_dataset/calculation/0886.json b/processed_dataset/calculation/0886.json new file mode 100644 index 0000000000000000000000000000000000000000..bd83d119afb67d6a6f5b12094d148eec77c73056 --- /dev/null +++ b/processed_dataset/calculation/0886.json @@ -0,0 +1,8 @@ +{ + "source_file": "./raw_volume-zh/volume5/exercise6.tex", + "problem_type": "calculation", + "problem": "问题10. (反向 Cauchy 不等式-Polya-Szego 不等式)\n设 $0\\frac{\\pi}{2}$ 时, 两角差的绝对值越小, 上式的值就越大.\n当 $n \\geqslant 4$ 时, 总有两角之和 $\\leqslant \\frac{\\pi}{2}$, 可以将这两个角调整成一个为 0 , 另一个为原来两角之和而使正弦平方和不减, 这样一来, 所求 $n$ 个正弦平方和化为三个角的情形.\n当 $n=3$ 时, 若 $\\theta_1 、 \\theta_2 、 \\theta_3$ 中有两个角为 $\\frac{\\pi}{2}$,一个角为 0 , 可以将三者改为 $\\frac{\\pi}{2} 、 \\frac{\\pi}{4} 、 \\frac{\\pi}{4}$. 设 $\\theta_1 \\leqslant \\theta_2 \\leqslant \\theta_3, \\theta_1<\\theta_3$. 则 $\\theta_1+\\theta_3>\\frac{\\pi}{2}, \\theta_1<\\frac{\\pi}{3}<\\theta_3$.\n令 $\\theta_1^{\\prime}=\\frac{\\pi}{3}, \\theta_2^{\\prime}=\\theta_2, \\theta_3^{\\prime}=\\theta_1+\\theta_3-\\theta_1^{\\prime}$, 则 $\\theta_1^{\\prime}+\\theta_3^{\\prime}=\\theta_1+\\theta_3,\\left|\\theta_1^{\\prime}-\\theta_3^{\\prime}\\right|< \\left|\\theta_1-\\theta_3\\right|$, 由前面讨论知 $\\sin ^2 \\theta_1+\\sin ^2 \\theta_2+\\sin ^2 \\theta_3<\\sin ^2 \\theta_1^{\\prime}+\\sin ^2 \\theta_2^{\\prime}+\\sin ^2 \\theta_3^{\\prime}$. 因为 $\\theta_2^{\\prime}+\\theta_3^{\\prime}=\\frac{2 \\pi}{3}$, 又可得 $\\sin ^2 \\theta_2^{\\prime}+\\sin ^2 \\theta_3^{\\prime} \\leqslant 2 \\sin ^2 \\frac{\\pi}{3}=\\frac{3}{2}$, 故 $\\sin ^2 \\theta_1+\\sin ^2 \\theta_2+ \\sin ^2 \\theta_3 \\leqslant \\frac{9}{4}$, 等号成立当且仅当 $\\theta_1=\\theta_2=\\theta_3=\\frac{\\pi}{3}$ 时, 因此当 $n \\geqslant 3$ 时所求最大值为 $\\frac{9}{4}$.\n当 $n=2$ 时, $\\theta_1+\\theta_2=\\pi$, 故 $\\sin ^2 \\theta_1+\\sin ^2 \\theta_2=2 \\sin ^2 \\theta_1 \\leqslant 2$. 即最大值为 2 .", + "remark": "", + "figures": [] +} \ No newline at end of file diff --git a/processed_dataset/calculation/0889.json b/processed_dataset/calculation/0889.json new file mode 100644 index 0000000000000000000000000000000000000000..152f35459fcb949ccfd51211dc6cc672f97de01c --- /dev/null +++ b/processed_dataset/calculation/0889.json @@ -0,0 +1,8 @@ +{ + "source_file": "./raw_volume-zh/volume5/exercise8.tex", + "problem_type": "calculation", + "problem": "问题6. 设 $x_1, x_2, \\cdots, x_n$ 是 $n$ 个非负实数 $\\left(n>2, n \\in \\mathbf{N}^*\\right)$, 且\n$$\n\\sum_{i=1}^n x_i=n, \\sum_{i=1}^n i x_i=2 n-2 .\n$$\n求 $x_1+4 x_2+\\cdots+n^2 x_n$ 的最大值.", + "solution": "令 $y_i=\\sum_{j=i}^n x_j$, 则 $y_1=n, \\sum_{i=1}^n y_i=2 n-2$, 故 $S=\\sum_{k=1}^n k^2 x_k=\\sum_{k=1}^n k^2 \\left(y_k-y_{k+1}\\right)+n^2 y_n=\\sum_{k=1}^n(2 k-1) y_k$.\n由于 $y_1=n, y_2 \\geqslant y_3 \\geqslant \\cdots \\geqslant y_n$, 若存在 $i \\in\\{2,3, \\cdots, n-1\\}$, 使得 $y_i>y_n$ (记 $i$ 为使 $y_i>y_n$ 成立的最大下标). 因为\n$$\n\\begin{aligned}\n& (2 i-1) y_i+(2 i+1+\\cdots+2 n-1) y_n \\\\\n< & (2 i-1+\\cdots+2 n-1) \\cdot \\frac{y_i+(n-i) y_n}{n-i+1}\\left(\\text { 上式等价于 } y_i>y_n\\right) .\n\\end{aligned}\n$$\n故在保持 $y_i+y_{i+1}+\\cdots+y_n$ 不变的前提下, 用它们的平均值来代替它们, $S$ 将增大.\n所以, 当 $y_2=y_3=\\cdots=y_n$ 时, $S_{\\text {max }}=n^2-2$.", + "remark": "", + "figures": [] +} \ No newline at end of file diff --git a/processed_dataset/calculation/0890.json b/processed_dataset/calculation/0890.json new file mode 100644 index 0000000000000000000000000000000000000000..7882d361fd44e6f92307511af9b582820632fa1c --- /dev/null +++ b/processed_dataset/calculation/0890.json @@ -0,0 +1,8 @@ +{ + "source_file": "./raw_volume-zh/volume5/exercise8.tex", + "problem_type": "calculation", + "problem": "问题7. 对于满足条件 $x_1+x_2+\\cdots+x_n=1$ 的非负实数 $x_i(i=1,2, \\cdots, n)$, 求 $\\sum_{j=1}^n\\left(x_j^4-x_j^5\\right)$ 的最大值.", + "solution": "当 $n=1$ 时, $\\sum_{j=1}^n\\left(x_j^4-x_j^5\\right)=0$.\n当 $n=2$ 时, $\\sum_{j=1}^n\\left(x_j^4-x_j^5\\right)=\\left(x_1^4+x_2^4\\right)-\\left(x_1+x_2\\right)\\left(x_1^4-x_1^3 x_2+x_1^2 x_2^2-\\right. \\left.x_1 x_2^3+x_2^4\\right)=x_1^3 x_2-x_1^2 x_2^2+x_1 x_2^3=x_1 x_2\\left(1-3 x_1 x_2\\right)$.\n又由于 $x_1 x_2 \\leqslant \\frac{1}{4}$, 故可知所求最大值为 $\\frac{1}{12}$.\n当 $n \\geqslant 3$ 时, 则\n$$\n\\begin{aligned}\n& -\\left(x^4+y^4-x^5-y^5\\right)+\\left[(x+y)^4-(x+y)^5\\right] \\\\\n= & {\\left[(x+y)^4-x^4-y^4\\right]-\\left[(x+y)^5-x^5-y^5\\right] }\n\\end{aligned}\n$$\n$$\n\\begin{aligned}\n& =x y\\left(4 x^2+4 y^2+6 x y\\right)-x y\\left(5 x^3+5 y^3+10 x y^2+10 x^2 y\\right) \\\\\n& \\geqslant x y\\left(4 x^2+4 y^2+6 x y\\right)-5 x y(x+y)^3 \\\\\n& =x y\\left(\\frac{7}{2} x^2+\\frac{7}{2} y^2+\\frac{x^2+y^2}{2}+6 x y\\right)-5 x y(x+y)^3 \\\\\n& \\geqslant x y\\left(\\frac{7}{2} x^2+\\frac{7}{2} y^2+7 x y\\right)-5 x y(x+y)^3>0\n\\end{aligned}\n$$\n等价于 $\\frac{7}{2} x y(x+y)^2 \\cdot[7-10(x+y)]>0$, 即 $x+y<\\frac{7}{10}$.\n而当 $n \\geqslant 3$ 时, 总有 2 个数之和 $<\\frac{2}{3}<\\frac{7}{10}$, 不断用 $x+y$ 替换 $x 、 y$, 最终必可化为 $n=2$ 时的情形.\n综上, $\\sum_{j=1}^n\\left(x_j^4-x_j^5\\right)$ 的最大值为 $\\frac{1}{12}$.", + "remark": "", + "figures": [] +} \ No newline at end of file diff --git a/processed_dataset/calculation/0891.json b/processed_dataset/calculation/0891.json new file mode 100644 index 0000000000000000000000000000000000000000..5ac1b629158a96b46a9bedecfeb8625f7dbc167a --- /dev/null +++ b/processed_dataset/calculation/0891.json @@ -0,0 +1,8 @@ +{ + "source_file": "./raw_volume-zh/volume5/exercise8.tex", + "problem_type": "calculation", + "problem": "问题12. 设 $x_1, x_2, \\cdots, x_n$ 均不小于 0 , 且 $\\sum_{i=1}^n x_i=1$, 求和式\n$$\n\\sum_{1 \\leqslant i2$, 不妨设 $x_1 x_2>2$, 则\n$$\nx_1+x_2+x_3 \\geqslant x_1+x_2 \\geqslant 2 \\sqrt{x_1 x_2}>2 \\sqrt{2}>\\sqrt{6} .\n$$\n当 $n \\geqslant 4$ 时,令 $x_1=x_2=x, x_3=x_4=\\cdots=x_n=0$.\n则 $2 x^2+x^4=\\frac{n(n+1)}{2}$, 故 $x= \\pm \\sqrt{\\frac{\\sqrt{2 n(n+1)+4}-2}{2}}$ (负的舍去).\n此时, $\\sum_{i=1}^n x_i=\\sqrt{2 \\sqrt{2 n(n+1)+4}-4}<\\sqrt{\\frac{1}{2} n(n+1)}$.\n综上所述, 所求的正整数 $n=1,2$ 或 3 .", + "remark": "", + "figures": [] +} \ No newline at end of file diff --git a/processed_dataset/calculation/0893.json b/processed_dataset/calculation/0893.json new file mode 100644 index 0000000000000000000000000000000000000000..9d8e73d065652a02a6e63b2f5d7ba96b03bcae95 --- /dev/null +++ b/processed_dataset/calculation/0893.json @@ -0,0 +1,8 @@ +{ + "source_file": "./raw_volume-zh/volume5/exercise9.tex", + "problem_type": "calculation", + "problem": "问题8. 有 5 个正数满足条件:\n(1) 其中一数为 $\\frac{1}{2}$;\n(2) 从这 5 个数中任取 2 个数,在剩下的 3 个数中必有一个数, 与前面取出的两数之和为 1 .\n求这 5 个数.", + "solution": "设这 5 个数为 $0y$, 这时关于 $x$ 的一元二次方程\n$$\nx^2-\\beta y \\cdot x+y^2+\\alpha=0 . \\label{eq2}\n$$\n还有一个实数解 $\\bar{x}$.\n由韦达定理及式\\ref{eq2}知 $\\bar{x}=\\beta y-x \\in \\mathbf{Z}$, 又 $x \\cdot \\bar{x}=y^2+\\alpha$, 即 $\\bar{x}=\\frac{y^2+\\alpha}{x}>$ 0 , 所以, $\\bar{x}$ 为正整数.\n此时, $(\\bar{x}, y)$ 也是满足式\\ref{eq1}的正整数对, 因此\n$$\n\\bar{x}+y=\\frac{y^2+\\alpha}{x}+y \\geqslant x+y,\n$$\n这里用到 $x+y$ 的最小性.\n于是, 我们有 $x^2 \\leqslant y^2+\\alpha$, 且 $x \\geqslant y+1$. 进而\n$$\n\\beta=\\frac{x^2+y^2+\\alpha}{x y} \\leqslant \\frac{2\\left(y^2+\\alpha\\right)}{x y} \\leqslant \\frac{2\\left(y^2+\\alpha\\right)}{y(y+1)}=\\frac{2 y^2}{y^2+y}+\\frac{2 \\alpha}{y(y+1)}<2+\\alpha .\n$$\n这表明 $\\beta_{\\max } \\leqslant 2+\\alpha$.\n综上可知, $\\beta$ 的最大值为 $\\alpha+2$.", + "remark": "", + "figures": [] +} \ No newline at end of file diff --git a/processed_dataset/calculation/0898.json b/processed_dataset/calculation/0898.json new file mode 100644 index 0000000000000000000000000000000000000000..41a9db4896602ec5fba7434bd5d887fd0f61d512 --- /dev/null +++ b/processed_dataset/calculation/0898.json @@ -0,0 +1,8 @@ +{ + "source_file": "./raw_volume-zh/volume6/chapter1-3.tex", + "problem_type": "calculation", + "problem": "例7. 求所有的整数 $n>1$, 使得它的任何大于 1 的因数可以表示为 $a^r+$ 1 的形式, 这里 $a 、 r \\in \\mathbf{N}^*, r \\geqslant 2$.", + "solution": "解:设 $S$ 是所有满足条件的正整数组成的集合, 则对任意 $n \\in S, n>1$,\n$n$ 的每个大于 1 的因数都具有 $a^r+1$ 的形式, 这里 $a 、 r \\in \\mathbf{N}^*, r>1$.\n由上可知, 对任意 $n \\in S(n>2)$, 存在 $a 、 r \\in \\mathbf{N}^*, a 、 r>1$, 使得 $n= a^r+1$. 我们设 $n$ 的这种表示中 $a$ 是最小的, 即不存在 $b 、 t \\in \\mathbf{N}^*, t>1$, 使得 $a=b^t$. 这时, $r$ 必为偶数 (若否, 设 $r$ 为奇数,则 $(a+1) \\mid n$,于是, $a+1$ 可表示为 $b^t+1$ 的形式,导致 $a=b^t$, 与 $a$ 的最小矛盾). 所以, $S$ 中的每个大于 1 的元素 $n$ 都可表示为 $n=x^2+1, x \\in \\mathbf{N}^*$ 的形式.\n下面来求 $S$ 的每个元素 $n$.\n如果 $n$ 为素数,那么 $n$ 是具有 $x^2+1$ 形式的素数.\n如果 $n$ 为合数, 分两种情况讨论:\n(1) 若 $n$ 为奇合数,则存在奇素数 $p 、 q$, 使得 $p 、 q 、 p q \\in S$, 此时, 应存在 $a 、 b 、 c \\in \\mathbf{N}^*$, 满足\n$$\np=4 a^2+1, q=4 b^2+1, p q=4 c^2+1 .\n$$\n这里还可设 $a \\leqslant b1$ & $y_0>1, y_1 \\leqslant 1$ & $y_0>1, y_1>1$ \\\\\n\\hline$y_2=$ & $\\frac{1}{y_0}$ & $\\frac{y_1}{y_0}$ & $\\frac{1}{y_0}$ & $\\frac{y_1}{y_0}$ \\\\\n$y_3=$ & $\\frac{1}{y_0}$ & $\\frac{1}{y_1}$ & $\\max \\left\\{\\frac{1}{y_0}, \\frac{1}{y_1}\\right\\}$ \\\\\n$y_4=$ & $\\frac{1}{y_1 y_1}$ & $\\frac{y_0}{y_1}$ & $\\frac{y_0}{y_1}$ \\\\\n$y_5=$ & $y_1$ & $y_0$ & $y_0$ \\\\\n$y_6=$ & $y_0$ & $y_1$ & $y_1$ & $y_1$ \\\\\n\\hline\n\\end{tabular}\n所以, $\\left\\{y_n\\right\\}$ 是一个以 5 为周期的纯周期数列, 对应地, $\\left\\{x_n\\right\\}$ 也是.\n故 $x_{2011}=x_1$.\n说明题中所给递推关系式是一种特殊形式的 Lyness 方程, 这里通过直接计算来确定周期的方法对付分式递推 (具有周期性的) 数列是直接而有效的手段.", + "remark": "", + "figures": [] +} \ No newline at end of file diff --git a/processed_dataset/calculation/0907.json b/processed_dataset/calculation/0907.json new file mode 100644 index 0000000000000000000000000000000000000000..80c5547ad6a5ec540229868d6b73d5bd90127d0a --- /dev/null +++ b/processed_dataset/calculation/0907.json @@ -0,0 +1,8 @@ +{ + "source_file": "./raw_volume-zh/volume6/chapter1-8.tex", + "problem_type": "calculation", + "problem": "例2. 已知 $0 \\leqslant x_0<1$, 数列 $\\left\\{x_n\\right\\}$ 满足\n$$\nx_{n+1}=\\left\\{\\begin{array}{l}\n2 x_n-1, \\text { 若 } \\frac{1}{2} \\leqslant x_n<1, \\\\\n2 x_n, \\quad \\text { 若 } 0 \\leqslant x_n<\\frac{1}{2} .\n\\end{array}(n=0,1,2, \\cdots)\\right.\n$$\n并且 $x_5=x_0$. 问 : 满足条件的数列有多少个?", + "solution": "解:注意到, 当 $x_0$ 确定后, 数列 $\\left\\{x_n\\right\\}$ 是唯一确定的, 故问题可转为求 $x_0$ 的不同取值情况的个数.\n利用二进制来处理, 将 $x_n$ 用二进制表示, 设 $x_n=\\left(0 . b_1 b_2 \\cdots\\right)_2$, 如果 $b_1=1$, 那么 $\\frac{1}{2} \\leqslant x_n<1$, 此时 $x_{n+1}=2 x_n-1=\\left(0 . b_2 b_3 \\cdots\\right)_2$; 如果 $b_1=0$, 那么 $0 \\leqslant x_n<\\frac{1}{2}$, 此时 $x_{n+1}=2 x_n=\\left(0 . b_2 b_3 \\cdots\\right)_2$. 这表明: 当 $x_n=\\left(0 . b_1 b_2 \\cdots\\right)_2$ 时, 总有 $x_{n+1}=\\left(0 . b_2 b_3 \\cdots\\right)_2$ (相当于将二进制表示下 $x_n$ 的小数点后第一位\"吃掉了\".\n现在, 设 $x_0=\\left(0 . a_1 a_2 \\cdots\\right)_2$, 那么由上述讨论可知 $x_5=\\left(0 . a_6 a_7 \\cdots\\right)_2$, 结合 $x_5=x_0$ 得 $x_0$ 是一个二进制下的循环小数, 即 $x_0=\\left(0 . \\dot{a}_1 a_2 \\cdots \\dot{a}_5\\right)_2= \\frac{\\left(a_1 \\cdots a_5\\right)_2}{2^5-1}$, 其中 $\\left(a_1 \\cdots a_5\\right)_2$ 是二进制表示下的一个非负整数 (注意 $a_1, \\cdots, a_5$ 不全为 1).\n综上可知, $x_0$ 共有 $2^5-1=31$ 种不同的可能取值 $\\left(a_1, \\cdots, a_5\\right.$ 每个数均可取 0 或 1 ,但不能全部取 1 ), 相应的不同数列共 31 个.\n说明这里利用二进制表示将递推式变为规律性更强的式子, 然后结合数列的周期性掌控数列的结构.\n本质上而言是做了一个对应.", + "remark": "", + "figures": [] +} \ No newline at end of file diff --git a/processed_dataset/calculation/0908.json b/processed_dataset/calculation/0908.json new file mode 100644 index 0000000000000000000000000000000000000000..d72faf7686cb95368b2a0ec4ff4ad53f8df6130a --- /dev/null +++ b/processed_dataset/calculation/0908.json @@ -0,0 +1,8 @@ +{ + "source_file": "./raw_volume-zh/volume6/chapter2-7.tex", + "problem_type": "calculation", + "problem": "例4. 是否存在一个由正整数组成的数列 $\\left\\{a_n\\right\\}$, 使得每一个正整数都在该数列中恰好出现一次, 并且对任意 $k \\in \\mathbf{N}^*$, 都有 $k \\mid\\left(a_1+\\cdots+a_k\\right)$ ?", + "solution": "解:存在这样的数列.\n我们采用递归方法来构造: 取 $a_1=1$, 现设 $a_1, a_2, \\cdots, a_m$ (两两不同) 已取定, 令 $t$ 为不在 $a_1, \\cdots, a_m$ 中出现的最小正整数.\n由于 $(m+1, m+2)=1$, 故利用中国剩余定理可知: 存在无穷多个正整数 $r$, 使得 (记 $s=a_1+\\cdots+a_m$ )\n$$\n\\left\\{\\begin{array}{l}\ns+r \\equiv 0(\\bmod m+1), \\\\\ns+r+t \\equiv 0(\\bmod m+2) .\n\\end{array}\\right.\n$$\n取这样的一个 $r$, 使得 $r>\\max \\left\\{a_1, \\cdots, a_m, t\\right\\}$, 令 $a_{m+1}=r, a_{m+2}=t$. 依此定义的数列即符合要求.\n说明利用递推方法来处理存在性问题本质上还是一种直接构造的技 巧.\n本题中定义的数列依次写出可以是 $1,3,2,10,4, \\cdots$, 每次增加两项的做法可确保不重复地遍经所有正整数.", + "remark": "", + "figures": [] +} \ No newline at end of file diff --git a/processed_dataset/calculation/0909.json b/processed_dataset/calculation/0909.json new file mode 100644 index 0000000000000000000000000000000000000000..e99666e917333188d6a06fa2e2801542dfeab5fd --- /dev/null +++ b/processed_dataset/calculation/0909.json @@ -0,0 +1,8 @@ +{ + "source_file": "./raw_volume-zh/volume6/exercise1.tex", + "problem_type": "calculation", + "problem": "问题22. 设 $\\left\\{a_n\\right\\}$ 为一个实数数列, 定义如下: $a_1=t, a_{n+1}=4 a_n\\left(1-a_n\\right), n=1$, $2, \\cdots$. 问:有多少个不同的实数 $t$, 使得 $a_{2011}=0$ ?", + "solution": "若 $t>1$, 则 $a_2<0$, 依此结合数学归纳法, 可知当 $n \\geqslant 2$ 时, 都有 $a_n<$ 0 , 从而 $a_{2011} \\neq 0$; 若 $t<0$, 同上可得 $n \\geqslant 1$ 时, 都有 $a_n<0$, 也不会有 $a_{2011}=$ 0 . 因此, 使 $a_{2011}=0$ 的 $t \\in[0,1]$.\n现可设 $t=\\sin ^2 \\alpha, 0 \\leqslant \\alpha \\leqslant \\frac{\\pi}{2}$, 则 $a_1=\\sin ^2 \\alpha$. 若 $a_n=\\sin ^2\\left(2^{n-1} \\alpha\\right)$, 则 $a_{n+1}=4 \\sin ^2\\left(2^{n-1} \\alpha\\right) \\cos ^2\\left(2^{n-1} \\alpha\\right)=\\sin ^2\\left(2^n \\alpha\\right)$. 于是, 由数学归纳法原理知对任意 $n$, 有 $a_n=\\sin ^2\\left(2^{n-1} \\alpha\\right)$. 因此, 由 $a_{2011}=0$, 得 $\\sin ^2\\left(2^{2010} \\alpha\\right)=0$, 从而 $2^{2010} \\alpha= k \\pi$, 即 $\\alpha=\\frac{k \\pi}{2^{2010}}, k \\in \\mathbf{Z}$. 结合 $0 \\leqslant \\alpha \\leqslant \\frac{\\pi}{2}$, 知 $0 \\leqslant k \\leqslant 2^{2009}$, 利用正弦函数在 $\\left[0, \\frac{\\pi}{2}\\right]$ 上是非负的, 且是单调递增的, 可得有 $2^{2009}+1$ 个不同的实数 $t$, 使得 $a_{2011}=0$.", + "remark": "", + "figures": [] +} \ No newline at end of file diff --git a/processed_dataset/calculation/0910.json b/processed_dataset/calculation/0910.json new file mode 100644 index 0000000000000000000000000000000000000000..ff72af213a38886e9041cd2d3b961e77385615ee --- /dev/null +++ b/processed_dataset/calculation/0910.json @@ -0,0 +1,8 @@ +{ + "source_file": "./raw_volume-zh/volume6/exercise1.tex", + "problem_type": "calculation", + "problem": "问题31. 设 $\\alpha$ 为有理数, $\\cos \\alpha \\pi$ 也是有理数.\n求 $\\cos \\alpha \\pi$ 的所有可能值.", + "solution": "记 $\\theta=\\alpha \\pi$, 由 $\\alpha$ 为有理数, 知存在 $n \\in \\mathbf{N}^*$, 使得 $n \\theta=2 k \\pi, k \\in \\mathbf{Z}$, 即 $\\cos n \\theta=1$. 由上题的结论知存在整系数多项式 $f(x)=x^n+a_{n-1} x^{n-1}+\\cdots+ a_0$, 使得 $2 \\cos n \\theta=f(2 \\cos \\theta)$, 从而有\n$$\n(2 \\cos \\theta)^n+a_{n-1}(2 \\cos \\theta)^{n-1}+\\cdots+a_1(2 \\cos \\theta)+a_0-2=0,\n$$\n这表明 $2 \\cos \\theta$ (注意 $\\cos \\alpha \\pi \\in \\mathbf{Q})$ 是方程(1)\n$$\nx^n+a_{n-1} x^{n-1}+\\cdots+a_1 x+a_0-2=0\n$$\n的有理根.\n然而(1)左边是一个首项系数为 1 的多项式, 故(1)的有理根都是整数根.\n所以 $2 \\cos \\theta$ 为整数, 结合 $|\\cos \\theta| \\leqslant 1$, 知 $2 \\cos \\theta \\in\\{-2,-1,0,1,2\\}$, 于是 $\\cos \\alpha \\pi \\in\\left\\{0, \\pm \\frac{1}{2}, \\pm 1\\right\\}$ (集合中的每个值都存在 $\\alpha$ 取到是显然的).", + "remark": "", + "figures": [] +} \ No newline at end of file diff --git a/processed_dataset/calculation/0911.json b/processed_dataset/calculation/0911.json new file mode 100644 index 0000000000000000000000000000000000000000..4e27455b99ea159d540ea22b6c381fe4a18614b4 --- /dev/null +++ b/processed_dataset/calculation/0911.json @@ -0,0 +1,8 @@ +{ + "source_file": "./raw_volume-zh/volume6/exercise1.tex", + "problem_type": "calculation", + "problem": "问题40. 求所有的正整数数列 $a_0, a_1, \\cdots, a_n$, 使得\n(1) $\\frac{a_0}{a_1}+\\frac{a_1}{a_2}+\\cdots+\\frac{a_{n-1}}{a_n}=\\frac{99}{100}$;\n(2) $a_0=1$, 且 $\\left(a_{k+1}-1\\right) a_{k-1} \\geqslant a_k^2\\left(a_k-1\\right), k=1,2, \\cdots, n-1$.", + "solution": "设 $a_1, \\cdots, a_n$ 是符合条件的正整数, 则由 (1) 可知对 $1 \\leqslant k \\leqslant n$, 都有 $a_k>a_{k-1}$ (否则左边 $\\geqslant 1$, 而右边 $<1$ ), 从而, 有 $a_k>1$, 于是, 由 (2) 知\n$$\n\\frac{a_{k-1}}{a_k\\left(a_k-1\\right)} \\geqslant \\frac{a_k}{a_{k+1}-1},\n$$\n即 $\\frac{a_{k-1}}{a_k-1}-\\frac{a_{k-1}}{a_k} \\geqslant \\frac{a_k}{a_{k+1}-1}$, 所以 $\\frac{a_{k-1}}{a_k} \\leqslant \\frac{a_{k-1}}{a_k-1}-\\frac{a_k}{a_{k+1}-1}$, 对 $k=i+1, \\cdots$,\n$n-1$ 求和, 就有\n$$\n\\frac{a_i}{a_{i+1}}+\\cdots+\\frac{a_{n-1}}{a_n} \\leqslant \\frac{a_i}{a_{i+1}-1}-\\frac{a_{n-1}}{a_n-1}+\\frac{a_{n-1}}{a_n}<\\frac{a_i}{a_{i+1}-1} . \\label{eq1}\n$$\n在式\\ref{eq1}中令 $i=0$, 利用 (1) 就有 $\\frac{1}{a_1} \\leqslant \\frac{99}{100}=\\sum_{i=0}^{n-1} \\frac{a_i}{a_{i+1}}<\\frac{1}{a_1-1}$, 故 $a_1=2$. 类似地, 在 式\\ref{eq1} 中取 $i=1$, 结合 (1) 就有\n$$\n\\frac{1}{a_2} \\leqslant \\frac{1}{a_1}\\left(\\frac{99}{100}-\\frac{1}{a_1}\\right)<\\frac{1}{a_2-1},\n$$\n可得 $\\frac{1}{a_2} \\leqslant \\frac{49}{200}<\\frac{1}{a_2-1}$, 知 $a_2=5$. 重复这样的讨论, 在 式\\ref{eq1} 中分别取 $i=2 、 3$, 可得 $a_3=56, a_4=25 \\times 56^2=78400$. 此时\n$$\n\\frac{1}{a_5} \\leqslant \\frac{1}{a_4}\\left(\\frac{99}{100}-\\frac{1}{2}-\\frac{2}{5}-\\frac{5}{56}-\\frac{56}{25 \\times 56^2}\\right)=0 .\n$$\n故 $a_5$ 不存在.\n综上可知, 仅当 $n=4$ 时, 这样的数列存在, 对应地 $a_1, a_2, a_3, a_4$ 为 2,5 , 56, 78400 .", + "remark": "", + "figures": [] +} \ No newline at end of file diff --git a/processed_dataset/calculation/0912.json b/processed_dataset/calculation/0912.json new file mode 100644 index 0000000000000000000000000000000000000000..43b40f12f8d6615f97489767ebb9b95948089500 --- /dev/null +++ b/processed_dataset/calculation/0912.json @@ -0,0 +1,8 @@ +{ + "source_file": "./raw_volume-zh/volume6/exercise2.tex", + "problem_type": "calculation", + "problem": "问题10. 求最小的正整数 $k$, 使得至少存在两个由正整数组成的数列 $\\left\\{a_n\\right\\}$ 满足下述条件:\n(1) 对任意正整数 $n$, 都有 $a_n \\leqslant a_{n+1}$;\n(2) 对任意正整数 $n$, 都有 $a_{n+2}=a_{n+1}+a_n$;\n(3) $a_9=k$.", + "solution": "利用 $a_{n+2}=a_{n+1}+a_n$ 可知 $a_9=a_8+a_7=2 a_7+a_6=\\cdots=21 a_2+13 a_1$, 依题中条件, 可知不定方程\n$$\n13 x+21 y=k . \\label{eq1}\n$$\n有至少两组正整数解 $(x, y)$, 使得 $x \\leqslant y$.\n注意到, 若 式\\ref{eq1} 有两组正整数解 $\\left(x_1, y_1\\right)$ 和 $\\left(x_2, y_2\\right)$, 使得 $x_1 \\leqslant y_1, x_2 \\leqslant y_2$, 则 $13 x_1+21 y_1=13 x_2+21 y_2=k$, 由对称性, 不妨设 $x_1 \\leqslant x_2$, 那么 $13\\left(x_2-\\right. \\left.x_1\\right)=21\\left(y_1-y_2\\right)$, 此时, 由 $x_1=x_2$ 可得 $y_1=y_2$, 导致 $\\left(x_1, y_1\\right)=\\left(x_2, y_2\\right)$ 矛盾, 故 $x_11$, 设点 $P$ 是 $\\triangle A B C$ 外接圆的弧 $B A C$ 上的一个动点, 在射线 $B P$ 和 $C P$ 上分别取定点 $U$ 和 $V$, 使得 $B U=\\gamma B A, C V=\\gamma C A$, 再在射线 $U V$ 上取点 $Q$, 使得 $U Q=\\gamma U V$. 求点 $Q$ 的轨迹.", + "solution": "解:如图(), 连结 $A U 、 A V 、 A Q$, 在 $B C$ 延长线上取点 $D$, 使 $B D=\\gamma B C$.\n连结 $A D 、 D Q$, 因为 $C V=\\gamma C A, B U=\\gamma B A$, $\\angle A C V=\\angle A B U$, 所以 $\\triangle A C V \\backsim \\triangle A B U$.\n所以 $\\frac{A U}{A V}=\\frac{A B}{A C}, \\angle V A C=\\angle U A B$.\n于是 $\\angle U A V=\\angle B A C$, 则 $\\triangle A U V \\backsim \\triangle A B C$, 故 $\\frac{U V}{B C}=\\frac{A U}{A B}, \\angle A U V=\\angle A B C$.\n又 $U Q=\\gamma U V, B D=\\gamma B C$, 所以 $\\frac{U Q}{B D}=\\frac{U V}{B C}= \\frac{A U}{A B}$, 故 $\\triangle A U Q \\backsim \\triangle A B D, \\triangle A V Q \\backsim \\triangle A C D, \\triangle A Q D \\backsim \\triangle A V C$.\n则 $\\frac{Q D}{V C}=\\frac{A D}{A C}$, 于是 $Q D=\\gamma A D$, 这表明 $Q$ 位于以点 $D$ 为圆心, $\\gamma A D$ 为半径的圆上.\n当 $P$ 运动到点 $B$ 和点 $C$ 时, 割线 $B P$ 和 $C P$ 分别变为过点 $B$ 和 $C$ 的切线, 这时得到的 $Q^{\\prime}, Q^{\\prime \\prime}$ 为轨迹弧的端点.", + "remark": "", + "figures": [ + "./images/volume7/figures/fig-c1i12.png" + ] +} \ No newline at end of file diff --git a/processed_dataset/calculation/0914.json b/processed_dataset/calculation/0914.json new file mode 100644 index 0000000000000000000000000000000000000000..01fd510873915d57ed3b4175aee08c351cdefc05 --- /dev/null +++ b/processed_dataset/calculation/0914.json @@ -0,0 +1,10 @@ +{ + "source_file": "./raw_volume-zh/volume7/chapter2.tex", + "problem_type": "calculation", + "problem": "例5. 如图(), 在四边形 $A B C D$ 中, $\\angle C A B=30^{\\circ}, \\angle A B D=26^{\\circ}, \\angle A C D=13^{\\circ}, \\angle D B C= 51^{\\circ}$. 求 $\\angle A D B$ 的度数.", + "solution": "解:设 $\\angle A D B=x$, 则由于 $\\angle C A B=30^{\\circ}$, $\\angle A B D=26^{\\circ}, \\angle D B C=51^{\\circ}, \\angle A C D=13^{\\circ}$, 则 $\\angle A C B=73^{\\circ}, \\angle B D C=43^{\\circ}$, 所以 $\\angle A D C=x+43^{\\circ}$.\n对 $\\triangle B C D$ 和点 $A$ 应用角元塞瓦定理有\n$$\n\\begin{aligned}\n1 & =\\frac{\\sin \\angle D C A}{\\sin \\angle A C B} \\cdot \\frac{\\sin \\angle C B A}{\\sin \\angle A B D} \\cdot \\frac{\\sin \\angle B D A}{\\sin \\angle A D C} \\\\\n& =\\frac{\\sin 13^{\\circ}}{\\sin 73^{\\circ}} \\cdot \\frac{\\sin 77^{\\circ}}{\\sin 26^{\\circ}} \\cdot \\frac{\\sin x}{\\sin \\left(x+43^{\\circ}\\right)} .\n\\end{aligned}\n$$\n则 $\\frac{\\sin \\left(x+43^{\\circ}\\right)}{\\sin x}=\\frac{\\sin 13^{\\circ} \\cdot \\sin 77^{\\circ}}{\\sin 73^{\\circ} \\cdot \\sin 26^{\\circ}}$\n$$\n\\begin{aligned}\n& =\\frac{\\sin 13^{\\circ} \\cdot \\cos 13^{\\circ}}{\\sin 73^{\\circ} \\cdot \\sin 26^{\\circ}}=\\frac{1}{2 \\sin 73^{\\circ}}=\\frac{\\sin 30^{\\circ}}{\\sin 73^{\\circ}} \\\\\n& =\\frac{\\sin 150^{\\circ}}{\\sin 107^{\\circ}}=\\frac{\\sin \\left(107^{\\circ}+43^{\\circ}\\right)}{\\sin 107^{\\circ}} .\n\\end{aligned}\n$$\n故 $\\cos 43^{\\circ}+\\cot x \\cdot \\sin 43^{\\circ}=\\cos 43^{\\circ}+\\cot 107^{\\circ}$. $\\sin 43^{\\circ}$.\n所以, $\\angle A D B=x=107^{\\circ}$.", + "remark": "", + "figures": [ + "./images/volume7/figures/fig-c2i10.png" + ] +} \ No newline at end of file diff --git a/processed_dataset/calculation/0915.json b/processed_dataset/calculation/0915.json new file mode 100644 index 0000000000000000000000000000000000000000..de3dec70e2b83de1768cde19f6a5c5f7ac91572f --- /dev/null +++ b/processed_dataset/calculation/0915.json @@ -0,0 +1,10 @@ +{ + "source_file": "./raw_volume-zh/volume7/exercise1.tex", + "problem_type": "calculation", + "problem": "问题10. 如图(), 设 $I$ 为 $\\triangle A B C$ 的内心, 过 $I$ 分别作 $A B$ 、 $B C 、 C A$ 的平行线 $A_1 B_2 、 B_1 C_2 、 C_1 A_2$. 求 $\\frac{A_1 B_2}{A B}+\\frac{B_1 C_2}{B C}+\\frac{C_1 A_2}{C A}$ 的值.", + "solution": "解: 设 $r$ 为 $\\triangle A B C$ 的内切圆半径, $h_a 、 h_b 、 h_c$ 分别为顶点 $A 、 B 、 C$ 对应的高.\n因为 $A_1 B_2 / / A B$, 所以, $\\triangle A_1 C B_2 \\backsim \\triangle A C B$. 从而, $\\frac{A_1 B_2}{A B}=\\frac{h_c-r}{h_c}$. 同理, $\\frac{B_1 C_2}{B C}=\\frac{h_a-r}{h_a}, \\frac{C_1 A_2}{C A}=\\frac{h_b-r}{h_b}$. 故 $S=\\frac{A_1 B_2}{A B}+\\frac{B_1 C_2}{B C}+\\frac{C_1 A_2}{C A}=\\frac{h_c-r}{h_c}+ \\frac{h_a-r}{h_a}+\\frac{h_b-r}{h_b}=3-r\\left(\\frac{1}{h_c}+\\frac{1}{h_a}+\\frac{1}{h_b}\\right)$. 设 $S$ 为 $\\triangle A B C$ 的面积.\n则 $2 S= a h_a=b h_b=c h_c=r(a+b+c)$. 于是, $S=3-r\\left(\\frac{a}{2 S}+\\frac{b}{2 S}+\\frac{c}{2 S}\\right)=2$.", + "remark": "注:: 事实上, $I$ 为 $\\triangle A B C$ 内任意一点时, 均有 $S=2$, 这是因为 $\\frac{B_1 C_2}{B C}= \\frac{A B_1}{A B}, \\frac{C_1 A_2}{C A}=\\frac{A_2 B}{A B}$. 故 $S=\\frac{A_1 B_2+A B_1+A_2 B}{A B}=\\frac{A_1 I+I B_2+A B_1+A_2 B}{A B}= \\frac{A A_2+B_1 B+A B_1+A_2 B}{A B}=2$.", + "figures": [ + "./images/volume7/figures/fig-c1p10.png" + ] +} \ No newline at end of file diff --git a/processed_dataset/calculation/0916.json b/processed_dataset/calculation/0916.json new file mode 100644 index 0000000000000000000000000000000000000000..4fa80922761beda93b2a896b4b68d70906501bb3 --- /dev/null +++ b/processed_dataset/calculation/0916.json @@ -0,0 +1,10 @@ +{ + "source_file": "./raw_volume-zh/volume7/exercise1.tex", + "problem_type": "calculation", + "problem": "问题11. 设 $D$ 是 $\\triangle A B C$ 的边 $B C$的中点, $\\triangle A B D$, $\\triangle A D C$ 的外心分别为 $E 、 F$, 直线 $B E 、 C F$ 交于点 $G$. 若 $B C=2 D G=2008, E F=1255$, 求 $\\triangle A E F$ 的面积.", + "solution": "解: : 如图(), 连结 $D E 、 D F 、 A C$. 因为 $B C= 2 D G$, 所以, $\\triangle B G C$ 是直角三角形.\n故 $\\angle G B C+ \\angle G C B=90^{\\circ}$. 又 $E 、 F$ 是外心, 则 $\\angle G B C=90^{\\circ}- \\angle B A D, \\angle G C B=90^{\\circ}-\\angle D A C$. 故 $90^{\\circ}=\\angle G B C+ \\angle G C B=180^{\\circ}-\\angle B A D-\\angle C A D=180^{\\circ}- \\angle B A C \\Rightarrow \\angle B A C=90^{\\circ} \\Rightarrow \\triangle B A C$ 是直角三角形.\n因此, $D A=\\frac{1}{2} B C=1004$. 又 $A E=D E, A F=D F$, 则 $\\triangle A E F \\cong \\triangle D E F$, 且 $E F$ 垂直平分 $A D$. 故 $S_{\\triangle A E F}=\\frac{1}{2} S_{\\text {四边形 } A E D F}=\\frac{1}{2} A D \\cdot E F=\\frac{1}{4} \\times 1004 \\times 1255=315005$.", + "remark": "", + "figures": [ + "./images/volume7/figures/fig-c1a11.png" + ] +} \ No newline at end of file diff --git a/processed_dataset/calculation/0917.json b/processed_dataset/calculation/0917.json new file mode 100644 index 0000000000000000000000000000000000000000..a8dc5b1176705338f5f72e0229a0c9e5b2f755de --- /dev/null +++ b/processed_dataset/calculation/0917.json @@ -0,0 +1,10 @@ +{ + "source_file": "./raw_volume-zh/volume7/exercise1.tex", + "problem_type": "calculation", + "problem": "问题14. 设 $D$ 是锐角 $\\triangle A B C$ 内部的一个点, 使得 $\\angle A D B=\\angle A C B+90^{\\circ}$, 并有 $A C \\cdot B D=A D \\cdot B C$. 计算比值 $\\frac{A B \\cdot C D}{A C \\cdot B D}$.", + "solution": "解:如图(), 分别作 $\\angle C B E=\\angle C A D, \\angle A C D= \\angle B C E$, 边 $B E 、 C E$ 相交于 $E$. 于是 $\\triangle A C D \\backsim \\triangle B C E$. 从而\n$$\n\\frac{A C}{B C}=\\frac{A D}{B E}=\\frac{C D}{C E} . \\label{eq1}\n$$\n所以 $A C \\cdot B E=B C \\cdot A D=A C \\cdot B D$, 即 $B E=B D$.\n又因为 $\\angle A D B=\\angle C B D+\\angle C A D+\\angle A C B=90^{\\circ}+\\angle A C B$, 所以 $\\angle B D E= \\angle C B D+\\angle C B E=\\angle C B D+\\angle C A D=90^{\\circ}$. 故连结 $D E, \\triangle D B E$ 是等腰直角三角形.\n由 式\\ref{eq1} 知 $\\frac{A C}{B C}=\\frac{C D}{C E}$, 且 $\\angle A C D=\\angle B C E$, 于是 $\\frac{C A}{C D}=\\frac{C B}{C E}, \\angle A C B= \\angle D C E$, 从而 $\\triangle C A B \\backsim \\triangle C D E$, 所以 $\\frac{D E}{A B}=\\frac{C D}{C A}, \\frac{A B \\cdot C D}{B D \\cdot C A}=\\frac{D E}{B D}=\\sqrt{2}$.", + "remark": "", + "figures": [ + "./images/volume7/figures/fig-c1a14.png" + ] +} \ No newline at end of file diff --git a/processed_dataset/calculation/0918.json b/processed_dataset/calculation/0918.json new file mode 100644 index 0000000000000000000000000000000000000000..042f298e6f14be54064761c290d93e452f8f4710 --- /dev/null +++ b/processed_dataset/calculation/0918.json @@ -0,0 +1,10 @@ +{ + "source_file": "./raw_volume-zh/volume7/exercise1.tex", + "problem_type": "calculation", + "problem": "问题15. 在一个圆中, 两条弦 $A B 、 C D$ 相交于 $E$ 点, $M$ 为弦 $A B$ 上严格在 $E 、 B$ 之间的点, 过 $D 、 E 、 M$ 的圆在 $E$ 点的切线分别交 $B C 、 A C$ 于 $F 、 G$. 已知 $\\frac{A M}{A B}=t$, 求 $\\frac{G E}{E F}$ (用 $t$ 表示).", + "solution": "解: 如图(), 连结 $A D, D M, D B$. 由题设, 有 $\\angle C E F=\\angle D E G=\\angle E M D, \\angle E C F=\\angle M A D$, 于是 $\\triangle C E F \\backsim \\triangle A M D$.\n从而 $C E \\cdot M D=A M \\cdot E F \\cdots$ (1).\n另一方面, 又有 $\\angle E C G=\\angle M B D$, 于是 $\\angle C G E=\\angle C E F-\\angle E C G=\\angle E M D-\\angle M B D= \\angle B D M$, 故 $\\triangle C G E \\backsim \\triangle B D M$,\n从而 $G E \\cdot B M=C E \\cdot D M \\cdots$ (2).\n由 (1), (2)得 $G E \\cdot B M=A M \\cdot E F$, 故\n$$\n\\frac{G E}{E F}=\\frac{A M}{B M}=\\frac{t \\cdot A B}{(1-t) \\cdot A B}=\\frac{t}{1-t} .\n$$", + "remark": "", + "figures": [ + "./images/volume7/figures/fig-c1a15.png" + ] +} \ No newline at end of file diff --git a/processed_dataset/calculation/0919.json b/processed_dataset/calculation/0919.json new file mode 100644 index 0000000000000000000000000000000000000000..ae4e3c654e4ad4b74532dcd3683f6df5f53d198c --- /dev/null +++ b/processed_dataset/calculation/0919.json @@ -0,0 +1,8 @@ +{ + "source_file": "./raw_volume-zh/volume7/exercise11.tex", + "problem_type": "calculation", + "problem": "问题1. 设 $A 、 B 、 C$ 为单位圆上的三个不同的点, $G 、 H$ 分别为 $\\triangle A B C$ 的重心、垂心.\n若 $F$ 为线段 $G H$ 的中点, 求 $|\\overrightarrow{A F}|^2+|\\overrightarrow{B F}|^2+|\\overrightarrow{C F}|^2$ 的值.", + "solution": "解:\n显然 $\\overrightarrow{O H}=\\overrightarrow{O A}+\\overrightarrow{O B}+\\overrightarrow{O C}, \\overrightarrow{O G}=\\frac{1}{3}(\\overrightarrow{O A}+\\overrightarrow{O B}+\\overrightarrow{O C})$.\n则 $\\overrightarrow{O F}=\\frac{\\overrightarrow{O G}+\\overrightarrow{O H}}{2}=\\frac{2}{3}(\\overrightarrow{O A}+\\overrightarrow{O B}+\\overrightarrow{O C})$.\n故 $|\\overrightarrow{A F}|^2+|\\overrightarrow{B F}|^2+|\\overrightarrow{C F}|^2=(\\overrightarrow{O A}-\\overrightarrow{O F}) \\cdot(\\overrightarrow{O A}-\\overrightarrow{O F})+(\\overrightarrow{O B}- \\overrightarrow{O F}) \\cdot(\\overrightarrow{O B}-\\overrightarrow{O F})+(\\overrightarrow{O C}-\\overrightarrow{O F}) \\cdot(\\overrightarrow{O C}-\\overrightarrow{O F})=|\\overrightarrow{O A}|^2+|\\overrightarrow{O B}|^2+ |\\overrightarrow{O C}|^2-2(\\overrightarrow{O A}+\\overrightarrow{O B}+\\overrightarrow{O C}) \\cdot \\overrightarrow{O F}+3 \\overrightarrow{O F} \\cdot \\overrightarrow{O F}=|\\overrightarrow{O A}|^2+|\\overrightarrow{O B}|^2+ |\\overrightarrow{O C}|^2-[2(\\overrightarrow{O A}+\\overrightarrow{O B}+\\overrightarrow{O C})-3 \\overrightarrow{O F}] \\cdot \\overrightarrow{O F}=|\\overrightarrow{O A}|^2+|\\overrightarrow{O B}|^2+|\\overrightarrow{O C}|^2=3$.", + "remark": "", + "figures": [] +} \ No newline at end of file diff --git a/processed_dataset/calculation/0920.json b/processed_dataset/calculation/0920.json new file mode 100644 index 0000000000000000000000000000000000000000..00825562a3e46f88115166dd75961dc739a46aee --- /dev/null +++ b/processed_dataset/calculation/0920.json @@ -0,0 +1,10 @@ +{ + "source_file": "./raw_volume-zh/volume7/exercise11.tex", + "problem_type": "calculation", + "problem": "问题8. 在凸四边形 $A B C D$ 的外部分别作正三角形 $A B Q$, 正三角形 $B C R$, 正三角形 $C D S$, 正三角形 $D A P$, 记四边形 $A B C D$ 的对角线之和为 $x$, 四边形 $P Q R S$ 的对边中点连线之和为 $y$, 求 $\\frac{y}{x}$ 的最大值.", + "solution": "解: : 如图(), 若四边形 $A B C D$ 是正方形时, 可得 $\\frac{y}{x}=\\frac{1+\\sqrt{3}}{2}$.\n$$\n\\begin{aligned}\n& D S_1=S_1 N=D N=E M, \\\\\n& D P_1=P_1 M=M D=E N,\n\\end{aligned}\n$$\n又\n$$\n\\begin{aligned}\n\\angle P_1 D S_1 & =360^{\\circ}-60^{\\circ}-60^{\\circ}-\\angle P D S \\\\\n& =240^{\\circ}-\\left(180^{\\circ}-\\angle E N D\\right) \\\\\n& =60^{\\circ}+\\angle E N D=\\angle E N S_1=\\angle E M P_1,\n\\end{aligned}\n$$\n所以\n$$\n\\triangle D P_1 S_1 \\cong \\triangle M P_1 E \\cong \\triangle N E S_1,\n$$\n从而, $\\triangle E P_1 S_1$ 是正三角形.\n同理可得, $\\triangle G Q_1 R_1$ 也是正三角形.\n设 $U 、 V$ 分别是 $P_1 S_1 、 Q_1 R_1$ 的中点, 于是有\n$$\n\\begin{aligned}\nE G & \\leqslant E U+U V+V G=\\frac{\\sqrt{3}}{2} P_1 S_1+P_1 Q_1+\\frac{\\sqrt{3}}{2} Q_1 R_1 \\\\\n& =P_1 Q_1+\\sqrt{3} P_1 S_1=\\frac{1}{2} B D+\\frac{\\sqrt{3}}{2} A C\n\\end{aligned}\n$$\n同理可得\n$$\nF H \\leqslant \\frac{1}{2} A C+\\frac{\\sqrt{3}}{2} B D,\n$$\n把上面两式相加, 得即\n$$\n\\begin{aligned}\n& y \\leqslant \\frac{1+\\sqrt{3}}{2} x, \\\\\n& \\frac{y}{x} \\leqslant \\frac{1+\\sqrt{3}}{2} .\n\\end{aligned}\n$$", + "remark": "", + "figures": [ + "./images/volume7/figures/fig-c11a8.png" + ] +} \ No newline at end of file diff --git a/processed_dataset/calculation/0921.json b/processed_dataset/calculation/0921.json new file mode 100644 index 0000000000000000000000000000000000000000..01f03169cd320e15c9bf30dd20a83e3360c06693 --- /dev/null +++ b/processed_dataset/calculation/0921.json @@ -0,0 +1,10 @@ +{ + "source_file": "./raw_volume-zh/volume7/exercise2.tex", + "problem_type": "calculation", + "problem": "问题15. 在 $\\triangle A B C$ 中, $A B=A C, \\angle A=20^{\\circ}$, 在边 $A B 、 A C$ 上分别取点 $D 、 E$, 使得 $\\angle E B C=60^{\\circ}, \\angle D C B=50^{\\circ}$. 求 $\\angle B E D$ 的度数.", + "solution": "解: 如图(), 设 $\\angle B E D=x$, 于是, $\\angle C E D=40^{\\circ}+x$. 对 $\\triangle B C E$ 和点 $D$ 应用角元塞瓦定理有\n$$\n\\begin{aligned}\n1 & =\\frac{\\sin \\angle B C D}{\\sin \\angle D C E} \\cdot \\frac{\\sin \\angle C E D}{\\sin \\angle D E B} \\cdot \\frac{\\sin \\angle E B D}{\\sin \\angle D B C} \\\\\n& =\\frac{\\sin 50^{\\circ}}{\\sin 30^{\\circ}} \\cdot \\frac{\\sin \\left(x+40^{\\circ}\\right)}{\\sin x} \\cdot \\frac{\\sin 20^{\\circ}}{\\sin 80^{\\circ}} .\n\\end{aligned}\n$$\n则 $\\frac{\\sin \\left(x+40^{\\circ}\\right)}{\\sin x}=\\frac{\\sin 80^{\\circ}}{2 \\cos 40^{\\circ} \\cdot \\sin 20^{\\circ}}=2 \\cos 20^{\\circ}=\\frac{\\sin 70^{\\circ}}{\\sin 30^{\\circ}}=\\frac{\\sin \\left(30^{\\circ}+40^{\\circ}\\right)}{\\sin 30^{\\circ}}$.\n因为 $\\frac{\\sin \\left(x+40^{\\circ}\\right)}{\\sin x}=\\cos 40^{\\circ}+\\cot x \\cdot \\sin 40^{\\circ}$,\n作为 $x$ 的函数在 $(0, \\pi)$ 上严格递减, 所以, $\\angle B E D=x=30^{\\circ}$.", + "remark": "", + "figures": [ + "./images/volume7/figures/fig-c2a15.png" + ] +} \ No newline at end of file diff --git a/processed_dataset/calculation/0922.json b/processed_dataset/calculation/0922.json new file mode 100644 index 0000000000000000000000000000000000000000..cff100ef6638ce35a44d1e2bd22298bd04029677 --- /dev/null +++ b/processed_dataset/calculation/0922.json @@ -0,0 +1,10 @@ +{ + "source_file": "./raw_volume-zh/volume7/exercise5.tex", + "problem_type": "calculation", + "problem": "问题17. 已知圆 $\\Gamma$ 和直线 $l$ 不相交, $P 、 Q 、 R 、 S$ 为圆 $\\Gamma$ 上的点, $P Q$ 与 $R S 、 P S$ 与 $Q R$ 分别交于点 $A 、 B, A 、 B$ 在直线 $l$ 上.\n试确定所有以 $A B$ 为直径的圆的公共点.", + "solution": "如图(), 设圆 $\\Gamma$ 的圆心为 $O$, 半径为 $r$. 由密克定理知, $\\triangle A P S$ 和 $\\triangle B R S$ 的外接圆交于点 $K$, 且 $K$ 在边 $A B$ 上.\n由圆幂定理得: $A O^2-r^2=A S \\cdot A R= A K \\cdot A B=A K^2+A K \\cdot K B, B O^2-r^2=B S \\cdot B P=B K \\cdot B A=B K^2+B K \\cdot K A$. 于是 $A O^2- A K^2=B O^2-B K^2$, 即 $O K$ 为 $A B$ 的垂线, 且 $A K K B=O K^2-r^2$.\n对任何一对满足条件的点 $\\{A, B\\}$, 因为 $O 、 K$ 、 $r$ 是固定的, 所以, 以 $A B$ 为直径的圆一定过直线\n$O K$ 上的两点, 其到直线 $l$ 距离为 $\\sqrt{O K^2-r^2}$.", + "remark": "", + "figures": [ + "./images/volume7/figures/fig-c5a17.png" + ] +} \ No newline at end of file diff --git a/processed_dataset/calculation/0923.json b/processed_dataset/calculation/0923.json new file mode 100644 index 0000000000000000000000000000000000000000..7048cf9d6ec095f7cc9b64c6e3d323712a7ee92e --- /dev/null +++ b/processed_dataset/calculation/0923.json @@ -0,0 +1,10 @@ +{ + "source_file": "./raw_volume-zh/volume7/exercise6.tex", + "problem_type": "calculation", + "problem": "问题6. 在圆内接四边形 $A B C D$ 中, 已知 $A B=B C, A D=3 D C, R$ 为对角线 $B D$ 上一点, 且满足 $D R=2 R B, Q$ 为线段 $A R$ 上一点, 且满足 $\\angle A D Q= \\angle B D Q$. 设 $P$ 为线段 $A B$ 与直线 $D Q$ 的交点.\n若 $\\angle A B Q+\\angle C B D= \\angle Q B D$, 求 $\\angle A P D$ 的度数.", + "solution": "解: 如图(), 以 $B$ 为旋转中心旋转 $\\triangle B C D$, 使\n$B C$ 与 $B A$ 重合, 点 $D$ 转到 $D^{\\prime}$.\n因为四边形 $A B C D$ 为圆内接四边形, 所以, $\\angle B A D^{\\prime}+\\angle D A B=\\angle B C D+\\angle D A B=180^{\\circ}$. 从而, $D^{\\prime} 、 A 、 D$ 三点共线.\n设 $H$ 为 $B Q$ 与 $A D$ 的交点.\n由 $\\angle D^{\\prime} B A=\\angle D B C$, 有 $\\angle D^{\\prime} B Q=\\angle D^{\\prime} B A+ \\angle A B Q=\\angle D B C+\\angle A B Q=\\angle Q B D$. 故 $\\triangle D^{\\prime} B H \\cong \\triangle D B H \\Rightarrow D^{\\prime} H=H D$. 因为 $A D=3 D C=3 D^{\\prime} A$, 所以, $D^{\\prime} H=\\frac{1}{2} D^{\\prime} D=2 D^{\\prime} A$. 于是, $\\frac{A H}{A D}=\\frac{1}{3}$. 视直线\n$R A$ 为 $\\triangle B H D$ 的截线, 由梅涅劳斯定理有 $\\frac{D R}{R B} \\cdot \\frac{B Q}{Q H} \\cdot \\frac{H A}{A D}=1$. 因此, $\\frac{B Q}{Q H}= \\frac{3}{2}$. 又 $\\angle A D Q=\\angle B D Q$, 则 $\\frac{B D}{D H}=\\frac{B Q}{Q H}=\\frac{3}{2}$. 因此, $B D=\\frac{3}{2} D H=3 A H=D A$. 于是, $D P \\perp A B \\Rightarrow \\angle A P D=90^{\\circ}$.", + "remark": "", + "figures": [ + "./images/volume7/figures/fig-c6a6.png" + ] +} \ No newline at end of file diff --git a/processed_dataset/calculation/0924.json b/processed_dataset/calculation/0924.json new file mode 100644 index 0000000000000000000000000000000000000000..5c8643c8278eb0a060a1109d21c9180c8c1a8c94 --- /dev/null +++ b/processed_dataset/calculation/0924.json @@ -0,0 +1,10 @@ +{ + "source_file": "./raw_volume-zh/volume7/exercise6.tex", + "problem_type": "calculation", + "problem": "问题9. 在 $\\triangle A B C$ 中, $A B=A C, \\angle A=20^{\\circ}$, 点 $D 、 E$ 分别在腰 $A B 、 A C$ 上, 且 $\\angle C B E=60^{\\circ}, \\angle D C B=50^{\\circ}$, 求 $\\angle D E B$.", + "solution": "解: 如图(), 注意到条件 $\\angle D C B=50^{\\circ}$ 和 $\\angle A=20^{\\circ}$, 得 $B D=B C$, 即 $\\triangle B C D$ 也为等腰三角形, 于是以 $\\triangle B C D$ 的对称轴为反射轴作轴反射变换, 则 $D \\rightarrow C$; 设直线 $B E$ 的像直线交 $A C$ 于 $F$, 则 $\\angle F B C=\\angle E B D=20^{\\circ}$, $\\angle B F C=80^{\\circ}=\\angle F C B$, 所以 $B F=B C=B D$. 又 $\\angle F B D=80^{\\circ}-20^{\\circ}=60^{\\circ}$, 因此 $\\triangle D B F$ 是一个正三角形, 所以 $F D=F B$, 又易知 $\\angle F B E= \\angle B E F\\left(=40^{\\circ}\\right)$, 从而 $F E=F B$, 于是 $F$ 为 $\\triangle B E D$ 的外心, 故 $\\angle D E B= \\frac{1}{2} \\angle D F B=30^{\\circ}$.", + "remark": "", + "figures": [ + "./images/volume7/figures/fig-c6a9.png" + ] +} \ No newline at end of file diff --git a/processed_dataset/calculation/0925.json b/processed_dataset/calculation/0925.json new file mode 100644 index 0000000000000000000000000000000000000000..ffcf637551528756fb582e9445fb7c126906bed8 --- /dev/null +++ b/processed_dataset/calculation/0925.json @@ -0,0 +1,8 @@ +{ + "source_file": "./raw_volume-zh/volume7/exercise7.tex", + "problem_type": "calculation", + "problem": "问题7. 在一个非钝角 $\\triangle A B C$ 中, $A B>A C, \\angle B=45^{\\circ}, O$ 和 $I$ 分别是 $\\triangle A B C$ 的外心和内心, 且 $\\sqrt{2} O I=A B-A C$. 求 $\\sin A$.", + "solution": "解:由已知条件及 Euler 公式得 $\\left(\\frac{c-b}{\\sqrt{2}}\\right)^2=O I^2=R^2-2 R r \\cdots$ (1). 再由熟知的几何关系得 $r=\\frac{c+a-b}{2} \\tan \\frac{B}{2}=\\frac{c+a-b}{2} \\tan \\frac{\\pi}{8}= \\frac{\\sqrt{2}-1}{2}(c+a-b) \\cdots(2)$.\n由(1)和(2)及正弦定理 $\\frac{a}{\\sin A}=\\frac{b}{\\sin B}=\\frac{c}{\\sin C}=2 R$ 得, $1-2(\\sin C-\\sin B)^2= 2(\\sin A+\\sin C-\\sin B)(\\sqrt{2}-1)$. 因为 $\\angle B=\\frac{\\pi}{4}, \\sin B=\\frac{\\sqrt{2}}{2}, \\sin C= \\sin \\left(\\frac{3 \\pi}{4}-\\angle A\\right)=\\frac{\\sqrt{2}}{2}(\\sin A+\\cos A)$. 所以, $2 \\sin A \\cos A-(2-\\sqrt{2}) \\sin A- \\sqrt{2} \\cos A+\\sqrt{2}-1=0,(\\sqrt{2} \\sin A-1)(\\sqrt{2} \\cos A-\\sqrt{2}+1)=0$. 于是, $\\sin A= \\frac{\\sqrt{2}}{2}$ 或 $\\cos A=1-\\frac{\\sqrt{2}}{2}$ (这时 $\\sin A=\\sqrt{1-\\cos ^2 A}=\\sqrt{1-\\left(1-\\frac{\\sqrt{2}}{2}\\right)^2}=\\sqrt{\\sqrt{2}-\\frac{1}{2}}$\n总之,或 $\\sin A=\\frac{\\sqrt{2}}{2}$ 或 $\\sin A=\\sqrt{\\sqrt{2}-\\frac{1}{2}}$.", + "remark": "", + "figures": [] +} \ No newline at end of file diff --git a/processed_dataset/calculation/0926.json b/processed_dataset/calculation/0926.json new file mode 100644 index 0000000000000000000000000000000000000000..e386d16f04ab9d309b75f11e079ff0ef06a69bdf --- /dev/null +++ b/processed_dataset/calculation/0926.json @@ -0,0 +1,8 @@ +{ + "source_file": "./raw_volume-zh/volume7/exercise7.tex", + "problem_type": "calculation", + "problem": "问题8. 设 $a 、 b 、 c$ 为 $\\triangle A B C$ 的三条边, $a \\leqslant b \\leqslant c, R$ 和 $r$ 分别为 $\\triangle A B C$ 的外接圆半径和内切圆半径.\n令 $f=a+b-2 R-2 r$, 试用角 $C$ 的大小来判定 $f$ 的符号.", + "solution": "解: 用 $A 、 B 、 C$ 分别表示 $\\triangle A B C$ 的三个内角.\n熟知 $a=2 R \\sin A, b= 2 R \\sin B, r=4 R \\sin \\frac{A}{2} \\cdot \\sin \\frac{B}{2} \\cdot \\sin \\frac{C}{2}$. 于是, $f=2 R(\\sin A+\\sin B-1- \\left.4 \\sin \\frac{A}{2} \\sin \\frac{B}{2} \\sin \\frac{C}{2}\\right)=2 R\\left[2 \\sin \\frac{B+A}{2} \\cdot \\cos \\frac{B-A}{2}-1+2\\left(\\cos \\frac{B+A}{2}-\\right.\\right. \\left.\\left.\\cos \\frac{B-A}{2}\\right) \\sin \\frac{C}{2}\\right]=4 R \\cos \\frac{B-A}{2}\\left(\\sin \\frac{B+A}{2}-\\sin \\frac{C}{2}\\right)-2 R+4 R \\cos \\frac{\\pi-C}{2} \\sin \\frac{C}{2}=4 R \\cos \\frac{B-A}{2}\\left(\\sin \\frac{\\pi-C}{2}-\\sin \\frac{C}{2}\\right)-2 R+4 R \\sin ^2 \\frac{C}{2}= 4 R \\cos \\frac{B-A}{2}\\left(\\cos \\frac{C}{2}-\\sin \\frac{C}{2}\\right)-2 R\\left(\\cos ^2 \\frac{C}{2}-\\sin ^2 \\frac{C}{2}\\right)=2 R\\left(\\cos \\frac{C}{2}-\\sin \\frac{C}{2}\\right) \\left(2 \\cos \\frac{B-A}{2}-\\cos \\frac{C}{2}-\\sin \\frac{C}{2}\\right)$.\n令 $A \\leqslant B \\leqslant C$, 所以 $0 \\leqslant B-A\\cos \\frac{C}{2} \\cdot \\cos \\frac{B-A}{2}>\\cos \\frac{B+A}{2}=\\cos \\frac{\\pi-C}{2}=\\sin \\frac{C}{2}$. 所以 $2 \\cos \\frac{B-A}{2}>\\cos \\frac{C}{2}+\\sin \\frac{C}{2}$. 则 $f>0 \\Leftrightarrow \\cos \\frac{C}{2}>\\sin \\frac{C}{2} \\Leftrightarrow C<\\frac{\\pi}{2} ; f=0 \\Leftrightarrow \\cos \\frac{C}{2}=\\sin \\frac{C}{2} \\Leftrightarrow C=\\frac{\\pi}{2} ; f<0 \\Leftrightarrow \\cos \\frac{C}{2}<\\sin \\frac{C}{2} \\Leftrightarrow C>\\frac{\\pi}{2}$.", + "remark": "", + "figures": [] +} \ No newline at end of file diff --git a/processed_dataset/calculation/0927.json b/processed_dataset/calculation/0927.json new file mode 100644 index 0000000000000000000000000000000000000000..3a10cd3807614815a4c0580977a6b91b6d9d2a9b --- /dev/null +++ b/processed_dataset/calculation/0927.json @@ -0,0 +1,11 @@ +{ + "source_file": "./raw_volume-zh/volume7/exercise9.tex", + "problem_type": "calculation", + "problem": "问题4. $G$ 是 $\\triangle A B C$ 的重心, $M 、 N$ 分别是 $A C 、 A B$ 的中点.\n设 $\\triangle A N C$ 和 $\\triangle A M B$ 的外接圆相交于 $A$ 和 $P, \\triangle A M N$ 的外接圆交 $A P$ 于 $T$. 求 $A T: A P$.", + "solution": "解: 如图(), 以 $A$ 为反演中心, 单位长度为反演幂,\n并记 $K$ 点的反象为 $K^{\\prime}$. 则由 $A 、 M 、 P 、 B$ 四点共圆及定理 5 知 $M^{\\prime} 、 P^{\\prime} 、 B^{\\prime}$ 三点共线.\n同理 $N^{\\prime} 、 P^{\\prime} 、 C^{\\prime}$ 也共线.\n又 $\\frac{A N^{\\prime}}{A B^{\\prime}}=\\frac{\\frac{1}{A N}}{\\frac{1}{A B}}=\\frac{A B}{A N}=2$. 所以 $B^{\\prime}$ 为 $A N^{\\prime}$ 的中点, 同理 $C^{\\prime}$ 为 $A M^{\\prime}$ 中点, 于是 $P^{\\prime}$ 为 $\\triangle A N^{\\prime} B^{\\prime}$ 的重心.\n反演后的图形成为图(), , 于是 $\\frac{A T}{A P}=\\frac{\\frac{1}{A T^{\\prime}}}{\\frac{1}{A P^{\\prime}}}=\\frac{A P^{\\prime}}{A T^{\\prime}}=\\frac{2}{3}$.", + "remark": "", + "figures": [ + "./images/volume7/figures/fig-c9a4-1.png", + "./images/volume7/figures/fig-c9a4-2.png" + ] +} \ No newline at end of file diff --git a/processed_dataset/calculation/0928.json b/processed_dataset/calculation/0928.json new file mode 100644 index 0000000000000000000000000000000000000000..4cd570848f584c5422a4cf04868fedaa62eb5b22 --- /dev/null +++ b/processed_dataset/calculation/0928.json @@ -0,0 +1,8 @@ +{ + "source_file": "./raw_volume-zh/volume8/chapter1.tex", + "problem_type": "calculation", + "problem": "例1. 已知复数 $z_1=(m-3)+(m-1) \\mathrm{i}, z_2=(2 m-5)+\\left(m^2+m-\\right. 2) \\mathrm{i}$, 且 $z_1>\\overline{z_2}$, 试求实数 $m$ 的值.", + "solution": "分析:与解由 $z_1>\\overline{z_2}$ 知, $z_1 、 \\overline{z_2}$ 均为实数, 即有\n$$\n\\left\\{\\begin{array}{l}\nm-1=0, \\\\\n-\\left(m^2+m-2\\right)=0,\n\\end{array}\\right.\n$$\n解得 $m=1$.\n因为 $z_1>\\overline{z_2}$, 所以 $m-3>2 m-5$, 即 $m<2$. 而 $m=1$ 适合 $m<2$. 故所求 $m=1$.", + "remark": "注:解题的突破口在于发现\" $z_1 、 \\overline{z_2}$ 均为实数\"这一隐含条件.", + "figures": [] +} \ No newline at end of file diff --git a/processed_dataset/calculation/0929.json b/processed_dataset/calculation/0929.json new file mode 100644 index 0000000000000000000000000000000000000000..630b81bcf2b52011e6f740ec90ad5c725d7763b0 --- /dev/null +++ b/processed_dataset/calculation/0929.json @@ -0,0 +1,8 @@ +{ + "source_file": "./raw_volume-zh/volume8/chapter1.tex", + "problem_type": "calculation", + "problem": "例2. 已知 $\\frac{z}{z-2}$ 是纯虚数, 求复数 $z$ 在复平面内对应点轨迹的方程.", + "solution": "分析:与解设 $z=x+y \\mathrm{i}(x, y \\in \\mathbf{R})$, 则\n$$\n\\frac{z}{z-2}=\\frac{x+y \\mathrm{i}}{(x-2)+y \\mathrm{i}}=\\frac{(x+y \\mathrm{i})[(x-2)-y \\mathrm{i}]}{(x-2)^2+y^2}\n$$\n$$\n\\begin{aligned}\n& =\\frac{x(x-2)+y^2+[y(x-2)-x y] \\mathrm{i}}{(x-2)^2+y^2} \\\\\n& =\\frac{x(x-2)+y^2-2 y \\mathrm{i}}{(x-2)^2+y^2} .\n\\end{aligned}\n$$\n因为 $\\frac{z}{z-2}$ 是纯虚数, 所以 $\\left\\{\\begin{array}{l}x(x-2)+y^2=0, \\\\ y \\neq 0,\\end{array}\\right.$ 即复数 $z$ 在复平面内对应点的轨迹是圆 (除去两点), 轨迹方程是\n$$\n(x-1)^2+y^2=1(y \\neq 0) .\n$$", + "remark": "注:初学复数的读者要千万留心: 纯虚数不仅是实部等于 0 , 还要求虚部不等于 0 .", + "figures": [] +} \ No newline at end of file diff --git a/processed_dataset/calculation/0930.json b/processed_dataset/calculation/0930.json new file mode 100644 index 0000000000000000000000000000000000000000..2309e8a9d9fc1e96c030df862ab2acf90514955b --- /dev/null +++ b/processed_dataset/calculation/0930.json @@ -0,0 +1,8 @@ +{ + "source_file": "./raw_volume-zh/volume8/chapter1.tex", + "problem_type": "calculation", + "problem": "例3. 已知非零复数 $a 、 b 、 c$ 满足 $\\frac{a}{b}=\\frac{b}{c}=\\frac{c}{a}$, 试求 $\\frac{a+b-c}{a-b+c}$ 的一切可能值.", + "solution": "分析:与解设 $\\frac{a}{b}=\\frac{b}{c}=\\frac{c}{a}=k$, 则 $a=b k, b=c k, c=a k$, 也就有 $c= a k, b=a k \\cdot k=a k^2, a=a k^2 \\cdot k=a k^3$. 因为 $a \\neq 0$, 所以有 $k^3=1$, 解得 $k=1$ 或 $k=-\\frac{1}{2} \\pm \\frac{\\sqrt{3}}{2} \\mathrm{i}$.\n所以 $\\frac{a+b-c}{a-b+c}=\\frac{a+a k^2-a k}{a-a k^2+a k}=\\frac{1+k^2-k}{1-k^2+k}$.\n若 $k=1$, 则原式 $=1$;\n若 $k=-\\frac{1}{2}+\\frac{\\sqrt{3}}{2} \\mathrm{i}$, 则原式 $=-\\frac{1}{2}-\\frac{\\sqrt{3}}{2} \\mathrm{i}$ ;\n若 $k=-\\frac{1}{2}-\\frac{\\sqrt{3}}{2} \\mathrm{i}$, 则原式 $=-\\frac{1}{2}+\\frac{\\sqrt{3}}{2} \\mathrm{i}$.\n综上所述, $\\frac{a+b-c}{a-b+c}$ 的一切可能值为 $1 、-\\frac{1}{2}-\\frac{\\sqrt{3}}{2} \\mathrm{i}$ 和 $-\\frac{1}{2}+\\frac{\\sqrt{3}}{2} \\mathrm{i}$.", + "remark": "注:连等式设 $k$ 是常用的解题技巧.", + "figures": [] +} \ No newline at end of file diff --git a/processed_dataset/calculation/0931.json b/processed_dataset/calculation/0931.json new file mode 100644 index 0000000000000000000000000000000000000000..c9e182f5f3778fc5cb4fb100e8902ffb5c9c1474 --- /dev/null +++ b/processed_dataset/calculation/0931.json @@ -0,0 +1,8 @@ +{ + "source_file": "./raw_volume-zh/volume8/chapter1.tex", + "problem_type": "calculation", + "problem": "例4. 已知 $z \\in \\mathbf{C}$, 关于 $x$ 的一元二次方程\n$$\nx^2-z x+4+3 \\mathrm{i}=0\n$$\n有实根,求使复数 $z$ 的模取得最小值的复数 $z$.", + "solution": "分析:与解设出复数 $z$ 的代数形式, 利用方程的实根将实部, 虚部分离.\n设已知方程的实根为 $x_0$, 并记 $z=a+b \\mathrm{i}(a 、 b \\in \\mathbf{R})$, 则有\n$$\nx_0^2-(a+b \\mathrm{i}) x_0+4+3 \\mathrm{i}=0,\n$$\n即\n$$\n\\left(x_0^2-a x_0+4\\right)+\\left(-b x_0+3\\right) \\mathrm{i}=0 .\n$$\n于是, 有\n$$\n\\begin{gathered}\nx_0^2-a x_0+4=0, \\label{eq1} \\\\\n-b x_0+3=0 . \\label{eq2}\n\\end{gathered}\n$$\n因为 $b=0$ 时,方程 式\\ref{eq2} 无解, 所以 $b \\neq 0$.\n由式\\ref{eq2}有 $x_0=\\frac{3}{b}$, 代入 \\ref{eq1} 式,得 $\\left(\\frac{3}{b}\\right)^2-\\left(\\frac{3}{b}\\right) a+4=0$, 解得 $a=\\frac{4 b^2+9}{3 b}, \\label{eq3}$.\n于是 $|z|^2=a^2+b^2=\\left(\\frac{4 b^2+9}{3 b}\\right)^2+b^2=\\frac{25}{9} b^2+\\frac{9}{b^2}+8 \\geqslant 2 \\sqrt{\\frac{25}{9} b^2 \\cdot \\frac{9}{b^2}}+ 8=18$.\n当且仅当 $\\frac{25}{9} b^2=\\frac{9}{b^2}$, 也即 $b^2=\\frac{9}{5}$ 时, 上式中的等号成立.\n此时, 对应的 $a^2=18-\\frac{9}{5}=\\frac{81}{5}$.\n由\\ref{eq3}式可知 $a 、 b$ 同号, 从而所求的复数 $z= \\pm\\left(\\frac{9 \\sqrt{5}}{5}+\\frac{3 \\sqrt{5}}{5} \\mathrm{i}\\right)$.", + "remark": "", + "figures": [] +} \ No newline at end of file diff --git a/processed_dataset/calculation/0932.json b/processed_dataset/calculation/0932.json new file mode 100644 index 0000000000000000000000000000000000000000..ea6b2e9b82d4bca6d0068944ced34fb082382654 --- /dev/null +++ b/processed_dataset/calculation/0932.json @@ -0,0 +1,8 @@ +{ + "source_file": "./raw_volume-zh/volume8/chapter2.tex", + "problem_type": "calculation", + "problem": "例1. 求下列各复数的辐角主值:\n(1) $z=\\mathrm{i} \\cos 100^{\\circ}$;\n(2) $z=a+b \\mathrm{i}\\left(a 、 b \\in \\mathbf{R}, a^2+b^2 \\neq 0\\right)$;\n(3) $z=\\sin 4(\\cos 4+\\operatorname{isin} 4)$;\n(4) $z=|\\cos \\theta|+\\mathrm{i}|\\sin \\theta|, \\theta \\in\\left(\\frac{5}{2} \\pi, 3 \\pi\\right)$.", + "solution": "分析:与解 (1) 因为 $i \\cos 100^{\\circ}$ 是纯虚数, 且 $\\cos 100^{\\circ}<0$, 所以\n$\\arg \\left(\\right.$ icos $\\left.100^{\\circ}\\right)=\\frac{3}{2} \\pi$.\n(2). 由于 $a 、 b$ 不确定,需讨论, 分六类:\n当 $a>0, b>0$ 时,复数对应的点位于第一象限,由辐角主值的定义, $\\tan \\theta=\\frac{b}{a}$, 由反正切函数的定义知, $\\arg z=\\arctan \\frac{b}{a}$.\n当 $a<0, b>0$ 时,复数对应的点位于第二象限,\n$$\n\\arg z=\\pi-\\arctan \\frac{b}{|a|}=\\pi+\\arctan \\frac{b}{a} .\n$$\n当 $a<0, b<0$ 时,复数对应的点位于第三象限, $\\arg z=\\pi+\\arctan \\frac{b}{a}$.\n当 $a>0, b<0$ 时, 复数对应的点位于第四象限,\n$$\n\\arg z=2 \\pi-\\arctan \\frac{|b|}{a}=2 \\pi+\\arctan \\frac{b}{a} .\n$$\n当 $a=0$ 时,如果 $b>0, \\arg z=\\frac{\\pi}{2}$; 如果 $b<0, \\arg z=\\frac{3}{2} \\pi$.\n当 $b=0$ 时,如果 $a>0, \\arg z=0$; 如果 $a<0, \\arg z=\\pi$.\n综上所述\n$$\n\\arg z= \\begin{cases}\\arctan \\frac{b}{a}, & (a>0, b \\geqslant 0) \\\\ \\pi+\\arctan \\frac{b}{a}, & (a<0) \\\\ 2 \\pi+\\arctan \\frac{b}{a}, & (a>0, b<0) \\\\ \\frac{\\pi}{2}, & (a=0, b>0) \\\\ \\frac{3 \\pi}{2} . & (a=0, b<0)\\end{cases}\n$$\n(3)不能错误地理解为 4 (因为 $4 \\in(0,2 \\pi)$ ) 就是该复数的辐角主值, 因为 $\\sin 4<0$,已知的复数表达形式不是三角形式, 事实上,\n$$\n\\begin{aligned}\nz & =\\sin 4(\\cos 4+\\mathrm{isin} 4) \\\\\n& =-\\sin 4[\\cos (\\pi+4)+\\mathrm{i} \\sin (\\pi+4)] .\n\\end{aligned}\n$$\n此时, 不能错误地理解为 $4+\\pi$ 是辐角主值 (这是因为 $4+\\pi>2 \\pi$ ). 该复数的辐角主值是:\n$$\n(4+\\pi)-2 \\pi=4-\\pi .\n$$\n(4)首先把绝对值符号去掉, 并\"改造\"成复数的三角形式:\n$$\n\\begin{aligned}\nz & =|\\cos \\theta|+\\mathrm{i}|\\sin \\theta|==-\\cos \\theta+\\mathrm{i} \\sin \\theta \\\\\n& =\\cos (\\pi-\\theta)+\\mathrm{i} \\sin (\\pi-\\theta), \\theta \\in\\left(\\frac{5 \\pi}{2}, 3 \\pi\\right)\n\\end{aligned}\n$$\n因为 $(\\pi-\\theta) \\in\\left(-2 \\pi,-\\frac{3}{2} \\pi\\right)$, 所以 $\\arg z=(\\pi-\\theta)+2 \\pi=3 \\pi-\\theta$.", + "remark": "", + "figures": [] +} \ No newline at end of file diff --git a/processed_dataset/calculation/0933.json b/processed_dataset/calculation/0933.json new file mode 100644 index 0000000000000000000000000000000000000000..ec3a92a9e8dceed6ce7202acc476652a23615e64 --- /dev/null +++ b/processed_dataset/calculation/0933.json @@ -0,0 +1,8 @@ +{ + "source_file": "./raw_volume-zh/volume8/chapter2.tex", + "problem_type": "calculation", + "problem": "例2. 化下列各复数为三角形式:\n(1) $z=\\sqrt{6}-\\sqrt{2} \\mathrm{i}$;\n(2) $z=r(\\cos \\theta-i \\sin \\theta)(r>0)$;\n(3) $z=1+\\mathrm{i} \\tan \\theta\\left(\\theta \\neq k \\pi+\\frac{\\pi}{2}, k \\in \\mathbf{Z}\\right)$;\n(4) $z=1+\\cos \\theta-i \\sin \\theta$.", + "solution": "分析:与解解决这类问题的关键是准确地掌握复数三角形式的四个外 部特征:\n模 $r \\geqslant 0$ 一一根据模的定义: 向量 $\\overrightarrow{O Z}$ 的长度有 $r \\geqslant 0$;\n角相同一这是因为两个角表示的是同一个复数的辐角;\n余弦在前, 正弦在后一一因为在 $a+b \\mathrm{i}$ 中, $a$ 是点 $(a, b)$ 的横坐标, 且 $a= r \\cos \\theta$, 所以\"余弦在前\";\n\"加\"相连一由 $a+b \\mathrm{i}$, 自然有\"加\"相连.\n(1) 因为 $|z|=2 \\sqrt{2}$, 并且\n$$\n\\left\\{\\begin{array}{l}\n\\sin \\theta=\\frac{-\\sqrt{2}}{2 \\sqrt{2}}=-\\frac{1}{2}, \\\\\n\\cos \\theta=\\frac{\\sqrt{6}}{2 \\sqrt{2}}=\\frac{\\sqrt{3}}{2} .\n\\end{array}\\right.\n$$\n解得 $\\theta=2 k \\pi+\\frac{11}{6} \\pi(k \\in \\mathbf{Z})$.\n所以 $z=\\sqrt{6}-\\sqrt{2} \\mathrm{i}$ 的三角形式是 $2 \\sqrt{2}\\left(\\cos \\frac{11}{6} \\pi+\\mathrm{i} \\sin \\frac{11}{6} \\pi\\right)$.\n(2) $z$\n$$\n\\begin{aligned}\nz & =r(\\cos \\theta-\\mathrm{i} \\sin \\theta) \\\\\n& =r[\\cos \\theta+\\mathrm{i} \\sin (-\\theta)] \\\\\n& =r[\\cos (-\\theta)+\\mathrm{i} \\sin (-\\theta)] .\n\\end{aligned}\n$$\n事实上, $r(\\cos \\theta+i \\sin \\theta)$ 与 $r(\\cos \\theta-i \\sin \\theta)$ 互为共轭复数; 它们在复平面上对应的向量关于实轴对称, 因此, 它们的模相等, 有一对辐角互为相反数, 即\n$$\nr(\\cos \\theta-\\mathrm{i} \\sin \\theta)=r[\\cos (-\\theta)+\\mathrm{i} \\sin (-\\theta)] .\n$$\n类似地有\n$$\n\\begin{aligned}\n& r(-\\cos \\theta+\\mathrm{i} \\sin \\theta)=r[\\cos (\\pi-\\theta)+\\mathrm{i} \\sin (\\pi-\\theta)], \\\\\n& -r(\\cos \\theta+\\mathrm{i} \\sin \\theta)=r[\\cos (\\pi+\\theta)+\\mathrm{i} \\sin (\\pi+\\theta)], \\\\\n& r(\\sin \\theta+i \\cos \\theta)=r\\left[\\cos \\left(\\frac{\\pi}{2}-\\theta\\right)+i \\sin \\left(\\frac{\\pi}{2}-\\theta\\right)\\right]\n\\end{aligned}\n$$\n等等.\n(3)利用三角函数公式\n$$\n\\begin{aligned}\nz & =1+\\mathrm{i} \\tan \\theta\\left(\\theta \\neq k \\pi+\\frac{\\pi}{2}, k \\in \\mathbf{Z}\\right) \\\\\n& =1+\\mathrm{i} \\frac{\\sin \\theta}{\\cos \\theta} \\\\\n& =\\frac{1}{\\cos \\theta}(\\cos \\theta+\\mathrm{i} \\sin \\theta) .\n\\end{aligned}\n$$\n至此, 不要以为它就是三角形式了, 由于 $\\theta$ 的取值范围不确定, $\\frac{1}{\\cos \\theta}$ 可正可负, 需讨论:\n当 $\\cos \\theta>0$ 时,复数的三角形式是 $\\sec \\theta(\\cos \\theta+i \\sin \\theta)$;\n当 $\\cos \\theta<0$ 时,复数的三角形式是 $-\\sec \\theta[\\cos (\\pi+\\theta)+i \\sin (\\pi+\\theta)]$.\n$$\n\\begin{aligned}\nz & =1+\\cos \\theta-i \\sin \\theta \\\\\n& =2 \\cos ^2 \\frac{\\theta}{2}-2 i \\sin \\frac{\\theta}{2} \\cos \\frac{\\theta}{2} \\\\\n& =2 \\cos \\frac{\\theta}{2}\\left(\\cos \\frac{\\theta}{2}-i \\sin \\frac{\\theta}{2}\\right) \\\\\n& =2 \\cos \\frac{\\theta}{2}\\left[\\cos \\left(-\\frac{\\theta}{2}\\right)+i \\sin \\left(-\\frac{\\theta}{2}\\right)\\right]\n\\end{aligned}\n$$\n当 $\\cos \\frac{\\theta}{2} \\geqslant 0$ 时,复数的三角形式是 $2 \\cos \\frac{\\theta}{2}\\left[\\cos \\left(-\\frac{\\theta}{2}\\right)+i \\sin \\left(-\\frac{\\theta}{2}\\right)\\right]$;\n当 $\\cos \\frac{\\theta}{2}<0$ 时,复数的三角形式是 $-2 \\cos \\frac{\\theta}{2}\\left[\\cos \\left(\\pi-\\frac{\\theta}{2}\\right)+i \\sin \\left(\\pi-\\frac{\\theta}{2}\\right)\\right]$.", + "remark": "", + "figures": [] +} \ No newline at end of file diff --git a/processed_dataset/calculation/0934.json b/processed_dataset/calculation/0934.json new file mode 100644 index 0000000000000000000000000000000000000000..fd723426abe5cfb2c17c52c99f8d13ae9f6c5b25 --- /dev/null +++ b/processed_dataset/calculation/0934.json @@ -0,0 +1,8 @@ +{ + "source_file": "./raw_volume-zh/volume8/chapter2.tex", + "problem_type": "calculation", + "problem": "例3. 已知复数 $z_1 、 z_2$ 满足 $\\left|z_1\\right|=1,\\left|z_2\\right|=r, \\arg \\left(z_1-\\sqrt{3}\\right)=\\frac{5 \\pi}{6}$,\n$\\quad \\arg \\left(\\frac{z_2}{z_1}\\right)=\\frac{\\pi}{4}$, 求 $z_1 、 z_2$.", + "solution": "分析:与解引人复数 $z_1$ 的三角形式, 设 $z_1=(\\cos \\theta+\\mathrm{i} \\sin \\theta)(0 \\leqslant \\theta<2 \\pi)$, 则\n$$\nz_1-\\sqrt{3}=(\\cos \\theta-\\sqrt{3})+\\mathrm{i} \\sin \\theta .\n$$\n因为\n$$\n\\arg \\left(z_1-\\sqrt{3}\\right)=\\frac{5 \\pi}{6},\n$$\n所以\n$$\n\\frac{\\sin \\theta}{\\cos \\theta-\\sqrt{3}}=\\tan \\frac{5 \\pi}{6}=-\\frac{\\sqrt{3}}{3},\n$$\n即\n$$\n\\begin{gathered}\n3 \\sin \\theta+\\sqrt{3} \\cos \\theta=3, \\\\\n\\sin \\left(\\theta+\\frac{\\pi}{6}\\right)=\\frac{\\sqrt{3}}{2} .\n\\end{gathered}\n$$\n因为\n$$\n0 \\leqslant \\theta<2 \\pi \\text {, }\n$$\n所以\n$$\n\\frac{\\pi}{6} \\leqslant \\theta+\\frac{\\pi}{6}<\\frac{13 \\pi}{6},\n$$\n于是,有\n$\\theta+\\frac{\\pi}{6}=\\frac{\\pi}{3}$ 或 $\\theta+\\frac{\\pi}{6}=\\frac{2 \\pi}{3}$,\n即\n$\\theta=\\frac{\\pi}{6}$ 或 $\\theta=\\frac{\\pi}{2}$.\n故\n$$\nz_1=\\frac{\\sqrt{3}}{2}+\\frac{1}{2} \\mathrm{i} \\text { 或 } z_1=\\mathrm{i} \\text {. }\n$$\n由 $\\arg \\left(\\frac{z_2}{z_1}\\right)=\\frac{\\pi}{4}$, 知\n$\\arg z_2=\\frac{\\pi}{4}+\\frac{\\pi}{6}=\\frac{5 \\pi}{12}$ 或 $\\arg z_2=\\frac{\\pi}{4}+\\frac{\\pi}{2}=\\frac{3 \\pi}{4}$,\n故知 $\\quad z_2=r\\left(\\cos \\frac{5 \\pi}{12}+\\mathrm{i} \\sin \\frac{5 \\pi}{12}\\right)=\\frac{r}{4}[\\sqrt{6}-\\sqrt{2}+\\mathrm{i}(\\sqrt{6}+\\sqrt{2})]$\n或\n$$\nz_2=r\\left(\\cos \\frac{3 \\pi}{4}+\\mathrm{i} \\sin \\frac{3 \\pi}{4}\\right)=\\frac{\\sqrt{2}}{2} r(-1+\\mathrm{i}) .\n$$\n综上可知, 所求复数为 $z_1=\\frac{\\sqrt{3}}{2}+\\frac{1}{2} \\mathrm{i}, z_2=\\frac{r}{4}[\\sqrt{6}-\\sqrt{2}+\\mathrm{i}(\\sqrt{6}+\\sqrt{2})]$ 或 $z_1=\\mathrm{i}, z_2=\\frac{\\sqrt{2}}{2} r(-1+\\mathrm{i})$.", + "remark": "", + "figures": [] +} \ No newline at end of file diff --git a/processed_dataset/calculation/0935.json b/processed_dataset/calculation/0935.json new file mode 100644 index 0000000000000000000000000000000000000000..bf188c2b32490e5a3bb05a308600b7c15de6dbf7 --- /dev/null +++ b/processed_dataset/calculation/0935.json @@ -0,0 +1,8 @@ +{ + "source_file": "./raw_volume-zh/volume8/chapter2.tex", + "problem_type": "calculation", + "problem": "例4. 已知复数 $z_1 、 z_2$ 满足 $\\left|z_1\\right|=2,\\left|z_2\\right|=3,3 z_1-2 z_2=\\frac{6}{5}(3+\\mathrm{i})$, 试求 $z_1 z_2$ 的值.", + "solution": "分析:与解由 $\\left|z_1\\right|==2,\\left|z_2\\right|=3$ 知, $z_1 \\overline{z_1}=4, z_2 \\overline{z_2}=9$.\n由 $3 z_1-2 z_2=\\frac{1}{3} z_2 \\overline{z_2} z_1-\\frac{1}{2} z_1 \\overline{z_1} z_2$\n$$\n\\begin{aligned}\n& =\\frac{1}{6} z_1 z_2\\left(2 \\overline{z_2}-3 \\overline{z_1}\\right) \\\\\n& =-\\frac{1}{6} z_1 z_2\\left(\\overline{3 z_1-2} \\overline{z_2}\\right),\n\\end{aligned}\n$$\n可知\n$$\n\\begin{aligned}\nz_1 z_2 & =-6 \\cdot \\frac{3 z_1-2 z_2}{3 z_1-2 z_2} \\\\\n& =-6 \\cdot \\frac{3+\\mathrm{i}}{3-\\mathrm{i}} \\\\\n& =-\\frac{24}{5}-\\frac{18}{5} \\mathrm{i} .\n\\end{aligned}\n$$", + "remark": "注:本题亦可设出 $z_1 、 z_2$ 的三角形式求解, 计算较为复杂, 读者不妨一试.", + "figures": [] +} \ No newline at end of file diff --git a/processed_dataset/calculation/0936.json b/processed_dataset/calculation/0936.json new file mode 100644 index 0000000000000000000000000000000000000000..815fa09a263dc7ec82a142edcc93c8b14797d8a6 --- /dev/null +++ b/processed_dataset/calculation/0936.json @@ -0,0 +1,8 @@ +{ + "source_file": "./raw_volume-zh/volume8/chapter2.tex", + "problem_type": "calculation", + "problem": "例7. 求 $\\tan 20^{\\circ}+4 \\sin 20^{\\circ}$ 的值.", + "solution": "分析:与解引人复数, 设 $z=\\cos 20^{\\circ}+\\mathrm{i} \\sin 20^{\\circ}$, 则 $\\bar{z}=\\frac{1}{z}=\\cos 20^{\\circ}-$ isin $20^{\\circ}$. 于是,有\n$$\n\\begin{gathered}\n\\cos 20^{\\circ}=\\frac{z+\\bar{z}}{2}=\\frac{z^2+1}{2 z}, \\\\\n\\sin 20^{\\circ}=\\frac{z-\\bar{z}}{2 \\mathrm{i}}=\\frac{z^2-1}{2 z \\mathrm{i}}, \\\\\nz^3=\\cos 60^{\\circ}+\\mathrm{i} \\sin 60^{\\circ}=\\frac{1}{2}+\\frac{\\sqrt{3}}{2} \\mathrm{i} .\n\\end{gathered}\n$$\n所以\n$$\n\\begin{aligned}\n\\tan 20^{\\circ}+4 \\sin 20^{\\circ} & =\\frac{z^2-1}{\\left(z^2+1\\right) \\mathrm{i}}+4 \\cdot \\frac{z^2-1}{2 z \\mathrm{i}} \\\\\n& =\\frac{2 z^4+z^3-z-2}{\\mathrm{i}\\left(z^3+z\\right)} \\\\\n& =\\frac{2\\left(\\frac{1}{2}+\\frac{\\sqrt{3}}{2} \\mathrm{i}\\right) z+\\left(\\frac{1}{2}+\\frac{\\sqrt{3}}{2} \\mathrm{i}\\right)-z-2}{\\mathrm{i}\\left(\\frac{1}{2}+\\frac{\\sqrt{3}}{2} \\mathrm{i}+z\\right)} \\\\\n& =\\frac{\\sqrt{3}\\left(z \\mathrm{i}+\\frac{1}{2} \\mathrm{i}-\\frac{\\sqrt{3}}{2}\\right)}{\\frac{1}{2} \\mathrm{i}+z \\mathrm{i}-\\frac{\\sqrt{3}}{2}} \\\\\n& =\\sqrt{3} .\n\\end{aligned}\n$$\n故 $\\tan 20^{\\circ}+4 \\sin 20^{\\circ}=\\sqrt{3}$.", + "remark": "注:(1) 以上两题阐明了复数与三角的联系.\n(2) 下面给出一组求值题, 有趣的是, 它们的值均为 $\\sqrt{3}$.\n$$\n\\begin{aligned}\n& \\cot 10^{\\circ}-4 \\cos 10^{\\circ} ; \\\\\n& \\cot 20^{\\circ}-\\sec 10^{\\circ} ; \\\\\n& \\csc 40^{\\circ}+\\tan 10^{\\circ} ; \\\\\n& 4 \\sin 40^{\\circ}-\\tan 40^{\\circ} \\text {. }\n\\end{aligned}\n$$", + "figures": [] +} \ No newline at end of file diff --git a/processed_dataset/calculation/0937.json b/processed_dataset/calculation/0937.json new file mode 100644 index 0000000000000000000000000000000000000000..ea3348dbfec7487b5d695094a1ce5c9bad67286c --- /dev/null +++ b/processed_dataset/calculation/0937.json @@ -0,0 +1,8 @@ +{ + "source_file": "./raw_volume-zh/volume8/chapter2.tex", + "problem_type": "calculation", + "problem": "例8. 已知数列 $\\left\\{a_n\\right\\} 、\\left\\{b_n\\right\\}$, 对大于 1 的整数 $n$,均成立\n$$\n\\begin{aligned}\n& a_n=a_{n-1} \\cos \\theta-b_{n-1} \\sin \\theta, \\\\\n& b_n=a_{n-1} \\sin \\theta+b_{n-1} \\cos \\theta,\n\\end{aligned}\n$$\n且 $a_1=1, b_1=\\tan \\theta$, 其中 $\\theta$ 为已知锐角, 试求数列 $\\left\\{a_n\\right\\} 、\\left\\{b_n\\right\\}$ 的通项公式.", + "solution": "分析:与解引人复数,构造等比数列.\n设 $z_n=a_n+b_n \\mathrm{i}\\left(a_n, b_n \\in \\mathbf{R}, n \\in \\mathbf{N}^*\\right)$, 则\n$$\n\\begin{aligned}\n\\frac{z_n}{z_{n-1}} & =\\frac{\\left(a_{n-1} \\cos \\theta-b_{n-1} \\sin \\theta\\right)+\\left(a_{n-1} \\sin \\theta+b_{n-1} \\cos \\theta\\right) \\mathrm{i}}{a_{n-1}+b_{n-1} \\mathrm{i}} \\\\\n& =\\frac{(\\cos \\theta+\\mathrm{i} \\sin \\theta)\\left(a_{n-1}+b_{n-1} \\mathrm{i}\\right)}{a_{n-1}+b_{n-1} \\mathrm{i}} \\\\\n& =\\cos \\theta+\\mathrm{i} \\sin \\theta,\n\\end{aligned}\n$$\n这说明复数列 $\\left\\{z_n\\right\\}$ 是以 $z_1=1+\\mathrm{i} \\tan \\theta$ 为首项, 以 $q=\\cos \\theta+i \\sin \\theta$ 为公比的等比数列, 于是, 有\n$$\n\\begin{aligned}\nz_n & =(1+i \\tan \\theta)(\\cos \\theta+i \\sin \\theta)^{n-1} \\\\\n& =\\sec \\theta \\cdot(\\cos \\theta+i \\sin \\theta)(\\cos \\theta+i \\sin \\theta)^{n-1} \\\\\n& =\\sec \\theta \\cdot(\\cos \\theta+i \\sin \\theta)^n \\\\\n& =(\\cos n \\theta+i \\sin n \\theta) \\sec \\theta,\n\\end{aligned}\n$$\n故 $a_n=\\sec \\theta \\cdot \\cos n \\theta, b_n=\\sec \\theta \\cdot \\sin n \\theta$.", + "remark": "注:这是复数与数列的结合.", + "figures": [] +} \ No newline at end of file diff --git a/processed_dataset/calculation/0938.json b/processed_dataset/calculation/0938.json new file mode 100644 index 0000000000000000000000000000000000000000..99300f9b5a6cd7fb681355e309ef84f5a6a6b04d --- /dev/null +++ b/processed_dataset/calculation/0938.json @@ -0,0 +1,8 @@ +{ + "source_file": "./raw_volume-zh/volume8/chapter3.tex", + "problem_type": "calculation", + "problem": "例1. 设 $z 、 \\omega 、 \\lambda$ 为复数, $|\\lambda| \\neq 1$, 解关于 $z$ 的方程: $\\bar{z}-\\lambda z=\\omega$.", + "solution": "分析:与解方程两边取共轭得, $\\overline{\\bar{z}-\\lambda z}=\\bar{\\omega}$, 即 $z-\\bar{\\lambda} \\bar{z}=\\bar{\\omega}$. 两边同乘 $\\lambda$ 得\n$$\n\\lambda z-|\\lambda|^2 \\bar{z}=\\lambda \\bar{\\omega}, \\label{eq1}\n$$\n又因为\n$$\n\\bar{z}-\\lambda z=\\omega . \\label{eq2}\n$$\n所以式\\ref{eq1}+\\ref{eq2}可得 $\\bar{z}\\left(1-|\\lambda|^2\\right)==\\omega+\\lambda \\bar{\\omega}$, 取共轭得 $z\\left(1-|\\lambda|^2\\right)=\\bar{\\omega}+\\bar{\\lambda} \\omega$. 因为 $|\\lambda| \\neq 1$, 所以 $z=\\frac{\\bar{\\lambda} \\omega+\\bar{\\omega}}{1-|\\lambda|^2}$.", + "remark": "注:在解复数方程的问题中, 适当地取模或者共轭往往会简化很多计算过程.", + "figures": [] +} \ No newline at end of file diff --git a/processed_dataset/calculation/0939.json b/processed_dataset/calculation/0939.json new file mode 100644 index 0000000000000000000000000000000000000000..b96df31c8d348abff9aac56c0a8ed3f4e8da9051 --- /dev/null +++ b/processed_dataset/calculation/0939.json @@ -0,0 +1,8 @@ +{ + "source_file": "./raw_volume-zh/volume8/chapter3.tex", + "problem_type": "calculation", + "problem": "例2. 已知关于 $z$ 的实系数方程 $z^2-2 p z+q=0(p \\neq 0)$ 的两虚根 $z_1 、 z_2$ 在复平面内的对应点为 $F_1 、 F_2$, 求以 $F_1 、 F_2$ 为两焦点, 且经过原点的椭圆的普通方程.", + "solution": "分析:与解由原方程有两个虚根可知: $\\Delta= 4 p^2-4 q<0$, 因此 $q>p^2>0$.\n设 $z_1=a+b \\mathrm{i}(a 、 b \\in \\mathbf{R})$, 则 $z_2=a-b \\mathrm{i}$.\n由韦达定理得, $\\left\\{\\begin{array}{l}z_1+z_2=2 a=2 p , \\\\ z_1 z_2=a^2+b^2=q ,\\end{array}\\right.$\n于是 $a=p,\\left|O F_1\\right|=\\left|O F_2\\right|=\\sqrt{a^2+b^2}= \\sqrt{q}$ (如图).\n显然, 椭圆的半短轴长 $=|O M|=|a|=|p|$, 半焦距 $=|b|$, 则半长轴长 $=\\sqrt{a^2+b^2}=\\sqrt{q}$, 而椭圆的中心为 $(a, 0)$, 即 $(p, 0)$.\n所以椭圆的普通方程为 $\\frac{(x-p)^2}{p^2}+\\frac{y^2}{q}=1$.", + "remark": "", + "figures": [] +} \ No newline at end of file diff --git a/processed_dataset/calculation/0940.json b/processed_dataset/calculation/0940.json new file mode 100644 index 0000000000000000000000000000000000000000..6196ee01888e078c0102c8612e543e904d19929a --- /dev/null +++ b/processed_dataset/calculation/0940.json @@ -0,0 +1,8 @@ +{ + "source_file": "./raw_volume-zh/volume8/chapter3.tex", + "problem_type": "calculation", + "problem": "例3. 已知实系数方程\n$$\nx^3+2(k-1) x^2+9 x+5(k-1)=0, \\label{eq1}\n$$\n有一个模为 $\\sqrt{5}$ 的虚根.\n求 $k$ 的值, 并解此方程.", + "solution": "分析:与解因为 $x^3+2(k-1) x^2+9 x+5(k-1)=0$, 由虚根成对原理, 可知 式\\ref{eq1} 有一个实根和两个模长为 $\\sqrt{5}$ 的虚根, 它们互为共轭, 设这三个根为 $a+b \\mathrm{i}, a-b \\mathrm{i}, c(a 、 b 、 c \\in \\mathbf{R})$, 则\n$$\na^2+b^2=5 . \\label{eq2}\n$$\n由韦达定理, 有 $\\left\\{\\begin{array}{l}(a+b \\mathrm{i})+(a-b \\mathrm{i})+c=-2(k-1), \\\\ (a+b \\mathrm{i})(a-b \\mathrm{i})+c(a+b \\mathrm{i})+c(a-b \\mathrm{i})=9, \\\\ (a+b \\mathrm{i})(a-b \\mathrm{i}) c=-5(k-1) .\\end{array}\\right.$\n结合(2)整理得\n$$\n\\begin{gathered}\n2 a+c=-2(k-1), \\label{eq3} \\\\\na c=2, \\label{eq4}\\\\\nc=-k+1 . \\label{eq5}\n\\end{gathered}\n$$\n由式\\ref{eq3}, \\ref{eq5}知 $c=1-k, a=\\frac{1}{2}(1-k)$, 并将其代入 式\\ref{eq2}, 可得 $k=-1$ 或 3 .\n再求解方程式\\ref{eq1}知: 当 $k=-1$ 时, 式\\ref{eq1}的解为 $1+2 \\mathrm{i}, 1-2 \\mathrm{i}, 2$; 当 $k=3$ 时, 式\\ref{eq1}的解为 $-2,-1+2 \\mathrm{i},-1-2 \\mathrm{i}$.", + "remark": "注:利用虚根成对是本题的关键.", + "figures": [] +} \ No newline at end of file diff --git a/processed_dataset/calculation/0941.json b/processed_dataset/calculation/0941.json new file mode 100644 index 0000000000000000000000000000000000000000..2a6468f9369106a49cf0c36f028d80d9908cd576 --- /dev/null +++ b/processed_dataset/calculation/0941.json @@ -0,0 +1,8 @@ +{ + "source_file": "./raw_volume-zh/volume8/chapter3.tex", + "problem_type": "calculation", + "problem": "例5. 设复平面上一个正方形的四个顶点对应的复数恰好是某个整系数一元四次方程 $x^4+p x^3+q x^2+r x+s=0$ 的四个根.\n求这个正方形面积的最小值.", + "solution": "分析:与解设正方形的中心 $A$ 对应的复数是 $a$, 该正方形的顶点均匀分布在一个圆周上, 它们对应的复数是方程 $(x-a)^4=b$ 的解, 其中的 $b$ 是某个复数.\n于是\n$$\n\\begin{aligned}\n& x^4+p x^3+q x^2+r x+s \\\\\n= & (x-a)^4-b \\\\\n= & x^4-4 a x^3+6 a^2 x^2-4 a^3 x+a^4-b .\n\\end{aligned}\n$$\n通过对比系数, 可知 $-a=\\frac{p}{4}$ 是有理数, 再结合 $-4 a^3=r$ 是整数, 便知 $a$ 是整数.\n于是, 由 $a^4-b=s$ 是整数, 可知 $b$ 亦是整数.\n以上的讨论表明,正方形顶点对应的复数是整系数方程 $(x-a)^4=b$ 的根, 其外接圆半径 $\\sqrt[4]{|b|}$ 不小于 1 . 于是, 正方形的面积不小于 $(\\sqrt{2})^2=2$. 而方程 $x^4=1$ 的四个根在复平面上对应于-一个正方形的顶点, 此正方形面积为 2. 故所求正方形面积的最小值是 2 .", + "remark": "", + "figures": [] +} \ No newline at end of file diff --git a/processed_dataset/calculation/0942.json b/processed_dataset/calculation/0942.json new file mode 100644 index 0000000000000000000000000000000000000000..13e3c26dd0484dc7bcdf058d4d9027629c3a2104 --- /dev/null +++ b/processed_dataset/calculation/0942.json @@ -0,0 +1,8 @@ +{ + "source_file": "./raw_volume-zh/volume8/chapter4.tex", + "problem_type": "calculation", + "problem": "例1. 已知 $\\vec{a}=\\{1,2\\}, \\vec{b}=\\{-3,2\\}$, 求实数 $k$ 使 $k \\vec{a}+\\vec{b}$ 与 $\\vec{a}-3 \\vec{b}$ 同方向或反方向.", + "solution": "分析:与解 $k \\vec{a}+\\vec{b}=\\{k-3,2 k+2\\}, \\vec{a}-3 \\vec{b}=\\{10,-4\\}$.\n由题意得 $k \\vec{a}+\\vec{b} / / \\vec{a}-3 \\vec{b}$, 所以 $\\frac{k-3}{10}=\\frac{2 k+2}{-4}$, 解得 $k=-\\frac{1}{3}$.", + "remark": "", + "figures": [] +} \ No newline at end of file diff --git a/processed_dataset/calculation/0943.json b/processed_dataset/calculation/0943.json new file mode 100644 index 0000000000000000000000000000000000000000..1b9f3f77a89504dddd3538966372941cef560c38 --- /dev/null +++ b/processed_dataset/calculation/0943.json @@ -0,0 +1,8 @@ +{ + "source_file": "./raw_volume-zh/volume8/chapter4.tex", + "problem_type": "calculation", + "problem": "例4. 设直线 $l: y=k x+m$ (其中 $k 、 m$ 为整数) 与椭圆 $\\frac{x^2}{16}+\\frac{y^2}{12}=1$ 交于不同两点 $A 、 B$, 与双曲线 $\\frac{x^2}{4}-\\frac{y^2}{12}=1$ 交于不同两点 $C 、 D$, 问是否存在直线 $l$, 使得向量 $\\overrightarrow{A C}+\\overrightarrow{B D}=\\overrightarrow{0}$, 若存在, 指出这样的直线有多少条? 若不存在, 请说明理由.", + "solution": "分析:与解由 $\\left\\{\\begin{array}{l}y=k x+m, \\\\ \\frac{x^2}{16}+\\frac{y^2}{12}=1,\\end{array}\\right.$ 消去 $y$ 化简整理得\n$$\n\\left(3+4 k^2\\right) x^2+8 k m x+4 m^2-48=0 . \n$$\n设 $A\\left(x_1, y_1\\right) 、 B\\left(x_2, y_2\\right)$, 则 $x_1+x_2=-\\frac{8 k m}{3+4 k^2}$.\n$$\n\\Delta_1=(8 k m)^2-4\\left(3+4 k^2\\right)\\left(4 m^2-48\\right)>0 . \\label{eq1}\n$$\n由 $\\left\\{\\begin{array}{l}y=k x+m, \\\\ \\frac{x^2}{4}-\\frac{y^2}{12}=1,\\end{array}\\right.$ 消去 $y$ 化简整理得\n$$\n\\left(3-k^2\\right) x^2-2 k m x-m^2-12=0 .\n$$\n设 $C\\left(x_3, y_3\\right) 、 D\\left(x_4, y_4\\right)$, 则 $x_3+x_4=\\frac{2 k m}{3-k^2}$.\n$$\n\\Delta_2=(-2 k m)^2+4\\left(3-k^2\\right)\\left(m^2+12\\right)>0 . \\label{eq2}\n$$\n因为 $\\overrightarrow{A C}+\\overrightarrow{B D}=\\overrightarrow{0}$, 所以 $\\left(x_4-x_2\\right)+\\left(x_3-x_1\\right)=0$, 此时 $\\left(y_4-y_2\\right)+ \\left(y_3-y_1\\right)==0$. 由 $x_1+x_2=x_3+x_4$ 得\n$$\n-\\frac{8 k m}{3+4 k^2}=\\frac{2 k m}{3-k^2} \\text {. }\n$$\n所以 $2 k m=0$ 或 $-\\frac{4}{3+4 k^2}=\\frac{1}{3-k^2}$. 由上式解得 $k=0$ 或 $m=0$. 当 $k=0$ 时, 由 式\\ref{eq1} 和 \\ref{eq2} 得 $-2 \\sqrt{3}) 建立空间直角坐标系, 则 $A(0,0,3), B(0,-3,0), D(0,3,0), C(2 \\sqrt{3}, 3$, $0), O(0,0,0)$.\n(1) $A C=\\sqrt{12+9+9}=\\sqrt{30}$.\n(2) $\\overrightarrow{A B}=\\{0,-3,-3\\}, \\overrightarrow{D C}=\\{2 \\sqrt{3}, 0,0\\}$, $\\overrightarrow{A D}=\\{0,3,-3\\}$.\n因为 $\\overrightarrow{A B} \\cdot \\overrightarrow{C D}=0, \\overrightarrow{D C} \\cdot \\overrightarrow{A D}=0$, 所以 $C D \\perp$ 平面 $A B D$, 故平面 $A C D \\perp$ 平面 $A B D$.\n所以二面角 $B-A D-C$ 的大小为 $90^{\\circ}$.\n(3) $\\overrightarrow{A C}=\\{2 \\sqrt{3}, 3,-3\\}, \\overrightarrow{B D}=\\{0,6,0\\}, \\cos \\theta=\\frac{\\overrightarrow{A C} \\cdot \\overrightarrow{B D}}{|\\overrightarrow{A C}| \\cdot|\\overrightarrow{B D}|}= \\frac{18}{6 \\cdot \\sqrt{30}}=\\frac{\\sqrt{30}}{10}$, 所以异面直线 $A C$ 和 $B D$ 所成的角为 $\\arccos \\frac{\\sqrt{30}}{10}$.", + "remark": "", + "figures": [ + "./images/volume8/figures/fig-c6i1.png" + ] +} \ No newline at end of file diff --git a/processed_dataset/calculation/0948.json b/processed_dataset/calculation/0948.json new file mode 100644 index 0000000000000000000000000000000000000000..b5ce6bb388a394c317538abef5a0cac6665de250 --- /dev/null +++ b/processed_dataset/calculation/0948.json @@ -0,0 +1,11 @@ +{ + "source_file": "./raw_volume-zh/volume8/chapter6.tex", + "problem_type": "calculation", + "problem": "例6. 如图(), 长方体 $A B C D-A_1 B_1 C_1 D_1$ 中, $A B= B C=4, A A_1=8, E$ 为 $C C_1$ 的中点, $O$ 为下底面正方形的中心.\n求:\n(1) 二面角 $C_1-A_1 B_1-O$ 的平面角 $\\alpha$ 的大小;\n(2) 异面直线 $A_1 B_1$ 和 $E O$ 所成角的大小;\n(3) 三棱雉 $O-A_1 B_1 E$ 的体积.", + "solution": "分析:与解 (1) 如图() 建立图间直角坐标系, 则 $O(2,2,0), C_1(0,4,8), A_1(4,0,8), B_1(4,4,8)$,\n$E(0,4,4), \\overrightarrow{A_1 B_1}=\\{0,4,0\\}, \\overrightarrow{A_1 O}=\\{-2,2,-8\\}$.\n设 $\\overrightarrow{n_0}=\\{x, y, z\\}$ 是平面 $A_1 B_1 O$ 的单位法向量, 则\n$$\n\\left\\{\\begin{array}{l}\n\\overrightarrow{n_0} \\cdot \\overrightarrow{A_1 B_1}=0, \\\\\n\\overrightarrow{n_0} \\cdot \\overrightarrow{A_1 O}=0, \\\\\n\\left|\\overrightarrow{n_0}\\right|=1 .\n\\end{array}\\right.\n$$\n所以\n$$\n\\left\\{\\begin{array}{l}\n4 y=0, \\\\\n-2 x+2 y-8 z=0, \\\\\nx^2+y^2+z^2=1\n\\end{array}\\right.\n$$\n由此解得\n$$\n\\left\\{\\begin{array} { l } \n{ x = - \\frac { 4 } { \\sqrt { 1 7 } } , } \\\\\n{ y = 0 , } \\\\\n{ z = - \\frac { 1 } { \\sqrt { 1 7 } } ; }\n\\end{array} \\text { 或 } \\left\\{\\begin{array}{l}\nx=-\\frac{4}{\\sqrt{17}}, \\\\\ny=0, \\\\\nz=\\frac{1}{\\sqrt{17}} .\n\\end{array}\\right.\\right.\n$$\n取 $\\overrightarrow{n_0}=\\left\\{\\frac{4}{\\sqrt{17}}, 0,-\\frac{1}{\\sqrt{17}}\\right\\}$.\n平面 $A_1 B_1 C_1$ 有一个单位法向量为 $\\overrightarrow{m_0}=\\{0,0,-1\\}$, 显然 $\\cos \\alpha>0$, 所以\n$$\n\\cos \\alpha=\\overrightarrow{m_0} \\cdot \\overrightarrow{n_0}=\\frac{1}{\\sqrt{17}}\n$$\n所以二面角 $C_1-A_1 B_1-O$ 的平面角的大小为 $\\arccos \\frac{1}{\\sqrt{17}}$.\n(2) $\\overrightarrow{A_1 B_1}=\\{0,4,0\\}, \\overrightarrow{E O}=\\{2,-2,-4\\}, \\cos \\beta=\\frac{\\overrightarrow{A_1 B_1} \\cdot \\overrightarrow{E O}}{\\left|\\overrightarrow{A_1 B_1}\\right| \\cdot|\\overrightarrow{E O}|}= \\frac{-8}{4 \\times 2 \\sqrt{6}}=-\\frac{\\sqrt{6}}{6}$\n所以异面直线 $A_1 B_1$ 和 $E O$ 所成的角为 $\\arccos \\frac{\\sqrt{6}}{6}$.\n(3) 因为 $\\overrightarrow{A_1 B_1}=\\{0,4,0\\}, \\overrightarrow{B_1 E}=\\{-4,0,-4\\}$, 所以 $\\overrightarrow{A_1 B_1} \\cdot \\overrightarrow{B_1 E}= 0,\\left|\\overrightarrow{A_1 B_1}\\right|=4,\\left|\\overrightarrow{B_1 E}\\right|=4 \\sqrt{2}$.\n所以 $S_{\\triangle A_1 B_1 E}=\\frac{1}{2} \\times 4 \\times 4 \\sqrt{2}=8 \\sqrt{2}$.\n设 $\\overrightarrow{p_0}$ 是平面 $A_1 B_1 O$ 的单位法向量, 记为 $\\overrightarrow{p_0}=\\{x, y, z\\}$, 则\n$$\n\\left\\{\\begin{array}{l}\n\\overrightarrow{p_0} \\cdot \\overrightarrow{A_1 B_1}=0, \\\\\n\\overrightarrow{p_0} \\cdot \\overrightarrow{B_1 E}=0, \\\\\n\\left|\\overrightarrow{p_0}\\right|=1 .\n\\end{array}\\right.\n$$\n所以\n$$\n\\left\\{\\begin{array}{l}\n4 y=0 \\\\\n-4 x-4 z=0, \\\\\nx^2+y^2+z^2=1\n\\end{array}\\right.\n$$\n由此解得\n$$\n\\left\\{\\begin{array} { l } \n{ x = \\frac { \\sqrt { 2 } } { 2 } , } \\\\\n{ y = 0 , } \\\\\n{ z = - \\frac { \\sqrt { 2 } } { 2 } ; }\n\\end{array} \\text { 或 } \\left\\{\\begin{array}{l}\nx=-\\frac{\\sqrt{2}}{2}, \\\\\ny=0, \\\\\nz=\\frac{\\sqrt{2}}{2} .\n\\end{array}\\right.\\right.\n$$\n取 $\\overrightarrow{p_0}=\\left\\{\\frac{\\sqrt{2}}{2}, 0,-\\frac{\\sqrt{2}}{2}\\right\\}, \\overrightarrow{O A_1}=\\{2,-2,8\\}$, 则 $h=\\left|\\overrightarrow{O A_1} \\cdot p_0\\right|= |\\sqrt{2}-4 \\sqrt{2}|=3 \\sqrt{2}$.\n所以 $V_{O A_1 B_1 E}=\\frac{1}{3} S_{\\triangle A_1 B_1 E} \\cdot h=\\frac{1}{3} \\times 8 \\sqrt{2} \\times 3 \\sqrt{2}=16$.", + "remark": "", + "figures": [ + "./images/volume8/figures/fig-c6i6.png", + "./images/volume8/figures/fig-c6i7.png" + ] +} \ No newline at end of file diff --git a/processed_dataset/calculation/0949.json b/processed_dataset/calculation/0949.json new file mode 100644 index 0000000000000000000000000000000000000000..a446d1c2d609bad617b42f8b445d422c1563d8e1 --- /dev/null +++ b/processed_dataset/calculation/0949.json @@ -0,0 +1,8 @@ +{ + "source_file": "./raw_volume-zh/volume8/chapter7.tex", + "problem_type": "calculation", + "problem": "例5. 试求一切有序正整数对 $(n, k)$, 使得 $x^n+x+1$ 被 $x^k+x+1$ 整除.", + "solution": "分析:与解显然, $n \\geqslant k$.\n当 $n>k$ 时, 设 $\\omega$ 是 $x^k+x+1=0$ 的一个根, 则 $\\omega \\neq 0, \\omega^n+\\omega+1=0$, 于是\n$$\n\\omega^n-\\omega^k=\\omega^k\\left(\\omega^{n-k}-1\\right)=0 .\n$$\n从而有\n$$\n\\omega^{n-k}=1 \\text {. }\n$$\n由 $|\\omega|^{n-k}=\\left|\\omega^{n-k}\\right|=1$, 知 $|\\omega|=1$.\n由 $1=|\\omega|^k=\\left|\\omega^k\\right|=|\\omega+1|$, 可知 $\\omega$ 的实部为 $-\\frac{1}{2}$, 则 $k \\geqslant 2$.\n$\\omega_1=\\frac{-1+\\sqrt{3} \\mathrm{i}}{2}$ 或 $\\omega_2=\\frac{-1-\\sqrt{3} \\mathrm{i}}{2}$ 是 $x^k+x+1=0$ 的所有根, 从而有\n$$\nx^k+x+1=\\left(x-\\omega_1\\right)^{k_1}\\left(x-\\omega_2\\right)^{k-k_1}, k_1 \\in \\mathbf{Z}, 0 \\leqslant k_1 \\leqslant k .\n$$\n若 $k>2$, 考虑上面等式两边含 $x^{k-1}$ 的项的系数, 便有 $k_1 \\omega_1+\\left(k-k_1\\right) \\omega_2=$ 0 ,考虑实部即有 $k=0$, 产生矛盾.\n若 $k=2$, 令 $n \\equiv l(\\bmod 3), 0 \\leqslant l<3$. 由 $\\omega^n+\\omega+1=\\omega^l+\\omega+1=0$, 得 $l=2, n \\equiv 2(\\bmod 3)$.\n故知 $(n, k)=(k, k)$ 或 $(3 m+2,2), m$ 是正整数.", + "remark": "", + "figures": [] +} \ No newline at end of file diff --git a/processed_dataset/calculation/0950.json b/processed_dataset/calculation/0950.json new file mode 100644 index 0000000000000000000000000000000000000000..4397b3e5847ebf28bbb19e21228e586595994b49 --- /dev/null +++ b/processed_dataset/calculation/0950.json @@ -0,0 +1,8 @@ +{ + "source_file": "./raw_volume-zh/volume8/chapter8.tex", + "problem_type": "calculation", + "problem": "例1. 对于给定的角 $\\alpha_1, \\alpha_2, \\cdots, \\alpha_n$, 试讨论方程\n$$\nx^n+x^{n-1} \\sin \\alpha_1+x^{n-2} \\sin \\alpha_2+\\cdots+x \\sin \\alpha_{n-1}+\\sin \\alpha_n=0\n$$\n是否有模大于 2 的复数根?", + "solution": "分析:与解答案是否定的.\n可以考虑从反面人手去解决.\n假定存在 $x_0$ 是原方程的复数解, 并且 $\\left|x_0\\right|>2$, 则有\n$$\nx_0^n=-x_0^{n-1} \\sin \\alpha_1-\\cdots-x_0 \\sin \\alpha_{n-1}-\\sin \\alpha_n,\n$$\n从而对上式两边取模, 并应用模的不等式, 得\n$$\n\\begin{aligned}\n\\left|x_0\\right|^n & \\leqslant\\left|x_0\\right|^{n-1}\\left|\\sin \\alpha_1\\right|+\\cdots+\\left|x_0\\right|\\left|\\sin \\alpha_{n-1}\\right|+\\left|\\sin \\alpha_n\\right| \\\\\n& \\leqslant\\left|x_0\\right|^{n-1}+\\left|x_0\\right|^{n-2}+\\cdots+\\left|x_0\\right|+1 \\\\\n& =\\frac{\\left|x_0\\right|^n-1}{\\left|x_0\\right|-1}<\\frac{\\left|x_0\\right|^n}{\\left|x_0\\right|-1}<\\frac{\\left|x_0\\right|^n}{2-1}=\\left|x_0\\right|^n .\n\\end{aligned}\n$$\n这显然产生矛盾, 由此说明原方程没有模大于 2 的复数根.", + "remark": "注:将一个等于 0 的式子中起主要作用的项移到 0 的那边, 再两边取模, 用不等式放缩, 是一个重要的技巧, 在之前第三章的例 7 , 例 8 中已有这样的手法, 在之后的例题中仍会出现, 望读者注意.", + "figures": [] +} \ No newline at end of file diff --git a/processed_dataset/calculation/0951.json b/processed_dataset/calculation/0951.json new file mode 100644 index 0000000000000000000000000000000000000000..430c8a61edd25cfb094a4277915de4df8893887b --- /dev/null +++ b/processed_dataset/calculation/0951.json @@ -0,0 +1,8 @@ +{ + "source_file": "./raw_volume-zh/volume8/chapter9.tex", + "problem_type": "calculation", + "problem": "例2. 已知 $\\triangle Z_1 Z_2 Z_3$ 三顶点对应复数为 $z_1 、 z_2 、 z_3$, 求其面积, 并以此求出过两复数 $z_1$ 与 $z_2$ 的直线方程.", + "solution": "分析:与解 $\\triangle Z_1 Z_2 Z_3$ 的面积 $S_{\\triangle Z_1 z_2 z_3}$\n$$\n\\begin{aligned}\n& =\\frac{1}{2} \\cdot Z_1 Z_2 \\cdot Z_1 Z_3 \\cdot \\sin \\angle Z_2 Z_1 Z_3 \\\\\n& =\\frac{1}{2} \\cdot\\left|z_2-z_1\\right| \\cdot\\left|z_3-z_1\\right| \\cdot \\operatorname{Im} \\frac{z_3-z_1}{z_2-z_1} \\cdot\\left|\\frac{z_2-z_1}{z_3-z_1}\\right| \\\\\n& =\\frac{1}{2} \\cdot \\operatorname{Im}\\left(\\bar{z}_1 z_2+\\bar{z}_2 z_3+\\bar{z}_3 z_1\\right),\\label{eq1} \\\\\n& =\\frac{\\mathrm{i}}{4}\\left|\\begin{array}{ccc}\n1 & 1 & 1 \\\\\nz_1 & z_2 & z_3 \\\\\n\\frac{z_1}{z_2} & \\overline{z_3}\n\\end{array}\\right| . \\label{eq2}\n\\end{aligned}\n$$\n\\ref{eq1}式和\\ref{eq2}式中给出的面积表达式可能是负的, 这是因为它们表示的是有向面积, 满足 $S_{\\triangle z_1 z_2 z_3}=-S_{\\triangle z_2 z_1 z_3}$. 如果是通常意义下的面积, 就取模, 为\n$\\frac{1}{2}\\left|\\operatorname{Im}\\left(\\overline{z_1} z_2+\\overline{z_2} z_3+\\overline{z_3} z_1\\right)\\right|=\\frac{1}{4}|| \\begin{array}{ccc}1 & 1 & 1 \\\\ z_1 & z_2 & z_3 \\\\ \\overline{z_1} & \\overline{z_2} & \\overline{z_3}\\end{array}||$. 而复平面上三点 $Z_1$ 、 $Z_2 、 Z_3$ 共线的充要条件是\n$$\n\\left|\\begin{array}{ccc}\n1 & 1 & 1 \\\\\nz_1 & z_2 & z_3 \\\\\n\\overline{z_1} & \\overline{z_2} & \\overline{z_3}\n\\end{array}\\right|=0\n$$\n即 $z_1 \\overline{z_2}+z_2 \\overline{z_3}+z_3 \\overline{z_1}=\\overline{z_1} z_2+\\overline{z_2} z_3+\\overline{z_3} z_1$, 于是直线 $Z_1 Z_2$ 的方程为:\n$$\n\\left(\\overline{z_1}-\\overline{z_2}\\right) z-\\left(z_1-z_2\\right) \\bar{z}+z_1 \\overline{z_2}-\\overline{z_1} z_2=0 .\n$$", + "remark": "", + "figures": [] +} \ No newline at end of file diff --git a/processed_dataset/calculation/0952.json b/processed_dataset/calculation/0952.json new file mode 100644 index 0000000000000000000000000000000000000000..2e9fc511453bef79b6979c1ed6e968eb9749d465 --- /dev/null +++ b/processed_dataset/calculation/0952.json @@ -0,0 +1,8 @@ +{ + "source_file": "./raw_volume-zh/volume8/chapter9.tex", + "problem_type": "calculation", + "problem": "例3. 试根据复数的几何意义推导出用复数表示点到直线距离的公式.", + "solution": "分析:与解设 $Z_1 、 Z_2$ 为直线 $l$ 上的两点, $Z_3$ 为直线 $l$ 外一点, 则 $\\overrightarrow{Z_1 Z_2}$ 方向上的单位向量为 $\\frac{z_2-z_1}{\\left|z_2-z_1\\right|}$, 而 $\\overrightarrow{Z_1 Z_3}$ 方向上的单位向量为 $\\frac{z_3-z_1}{\\left|z_3-z_1\\right|}$. 根据复数除法的几何意义, 由 $\\overrightarrow{Z_1 Z_2}$ 旋转到 $\\bar{Z}_1 \\vec{Z}_3$ 所转过的角 $\\varphi$ 由下式确定:\n$$\n\\begin{aligned}\n\\sin \\varphi & ==\\operatorname{Im}\\left(\\frac{\\left|z_2-z_1\\right|}{\\left|z_3-z_1\\right|} \\cdot \\frac{\\left(z_3-z_1\\right)\\left(\\overline{z_2}-\\overline{z_1}\\right)}{\\left(z_2-z_1\\right)\\left(\\overline{z_2}-\\overline{z_1}\\right)}\\right) \\\\\n& =\\frac{1}{\\left|z_3-z_1\\right| \\cdot\\left|\\overline{z_2}-z_1\\right|} \\operatorname{Im}\\left(-\\overline{z_1} \\overline{z_2}+\\overline{z_2} z_3-z_3 \\overline{z_1}+z_1 \\overline{z_1}\\right) .\n\\end{aligned}\n$$\n因为 $\\left|z_1^2\\right| \\in \\mathbf{R},--\\operatorname{Im} z_1 \\overline{z_2}=\\operatorname{Im} \\overline{z_1} z_2,-\\operatorname{Im} z_3 \\overline{z_1}=\\operatorname{Im} \\overline{z_3} z_1$, 由上式知 $Z_3$ 到直线 $Z_1 Z_2$ 的距离为 $d=\\frac{1}{\\left|z_2-z_1\\right|} \\cdot \\operatorname{Im}\\left(\\overline{z_1} z_2+\\overline{z_2} z_3+\\overline{z_3} z_1\\right)$. 约定距离 $d$ 非负,所以上式取绝对值.", + "remark": "注:由此可知复数 $z_1 、 z_2 、 z_3$ 为顶点的三角形面积为\n$$\nS_{\\triangle z_1 z_2 z_3}=\\frac{1}{2} \\cdot\\left|\\operatorname{Im}\\left(\\overline{z_1} z_2+\\overline{z_2} z_3+\\overline{z_3} z_1\\right)\\right|,\n$$\n当 $z_1 、 z_2 、 z_3$ 逆时针排列时,绝对值去掉; 当顺时针排列时,去掉绝对值后添负号.", + "figures": [] +} \ No newline at end of file diff --git a/processed_dataset/calculation/0953.json b/processed_dataset/calculation/0953.json new file mode 100644 index 0000000000000000000000000000000000000000..4efe04de92ac2313088b5a3712949e77ebda5130 --- /dev/null +++ b/processed_dataset/calculation/0953.json @@ -0,0 +1,8 @@ +{ + "source_file": "./raw_volume-zh/volume8/chapter9.tex", + "problem_type": "calculation", + "problem": "例5. 设 $\\triangle Z_1 Z_2 Z_3$ 与 $\\triangle Z_1^{\\prime} Z_2^{\\prime} Z_3^{\\prime}$ 顺相似, 求其顶点对应复数 (如 $Z_1$ 对应 $z_1$ 等)满足的充要条件,并由此推出中垂线的方程.", + "solution": "分析:与解由于 $z_1=z_2 \\Rightarrow\\left|z_1\\right|=\\left|z_2\\right|$ 且 $\\arg z_1=\\arg z_2$,于是 $\\triangle Z_1 Z_2 Z_3$ 与 $\\triangle Z^{\\prime}{ }_1 Z^{\\prime}{ }_2 Z^{\\prime}{ }_3$ 顺相似的充要条件即为 $\\frac{z_3-z_1}{z_2-z_1}=\\frac{z_3^{\\prime}-z_1^{\\prime}}{z_2^{\\prime}-z_1^{\\prime}}$, 这等价于 $\\left|\\begin{array}{ccc}1 & 1 & 1 \\\\ z_1 & z_2 & z_3 \\\\ z_1^{\\prime} & z_2^{\\prime} & z_3^{\\prime}\\end{array}\\right|=0$\n由此推知 $\\triangle Z_1 Z_2 Z_3$ 与 $\\triangle Z_1^{\\prime} \\triangle Z_2^{\\prime} \\triangle Z_3^{\\prime}$ 逆相似的充要条件为 $\\left|\\begin{array}{ccc}1 & 1 & 1 \\\\ z_1 & z_2 & z_3 \\\\ \\bar{z}_1^{\\prime} & \\bar{z}_2^{\\prime} & \\bar{z}_3^{\\prime}\\end{array}\\right|=0$, 而 $Z_1 、 Z_2$ 的中垂线方程为 $\\triangle Z Z_1 Z_2$ 逆相似于 $\\triangle Z Z_2 Z_1$ 所满足的方程, 即\n$$\n\\left|\\begin{array}{ccc}\n1 & 1 & 1 \\\\\nz & z_1 & z_2 \\\\\n\\bar{z} & \\overline{z_2} & \\overline{z_1}\n\\end{array}\\right|=0\n$$\n即 $\\frac{z}{z_1-z_2}+\\frac{\\bar{z}}{z_1-z_2}=\\frac{\\left|z_1\\right|^2-\\left|z_2\\right|^2}{\\left|z_1-z_2\\right|^2}$.", + "remark": "注:当 $z_2=0$ 时, 中垂线方程为 $\\frac{z}{z_1}+\\frac{\\bar{z}}{z_1}=1$.\n顺相似的依据是复数的三角形式, 这十分有用.\n例如我们还可以证明: $\\triangle Z_1 Z_2 Z_3$ 为正三角形的充要条件是 $z_1^2+z_2^2+z_3^2=z_1 z_2+z_2 z_3+z_3 z_1$ (当然 $z_1$ 、 $z_2 、 z_3$ 两两不等).", + "figures": [] +} \ No newline at end of file diff --git a/processed_dataset/calculation/0954.json b/processed_dataset/calculation/0954.json new file mode 100644 index 0000000000000000000000000000000000000000..b0ceaeb21a7ee4adc99ff0377dbc0526ab0011b1 --- /dev/null +++ b/processed_dataset/calculation/0954.json @@ -0,0 +1,8 @@ +{ + "source_file": "./raw_volume-zh/volume8/exercise1.tex", + "problem_type": "calculation", + "problem": "问题1. 已知 $f(x)=a x^2+b x$ ( $a 、 b$ 为非零实数), 存在一个虚数 $x_1$, 使 $f\\left(x_1\\right)$ 为实数 $-c$, 则 $b^2-4 a c$ 与 $\\left(2 a x_1+b\\right)^2$ 的关系是 $(\\quad)$.\n(A) 不能比较大小\n(B) $b^2-4 a c>\\left(2 a x_1+b\\right)^2$\n(C) $b^2-4 a c=\\left(2 a x_1+b\\right)^2$\n(D) $b^2-4 a c<\\left(2 a x_1+b\\right)^2$", + "solution": "C.\n事实上\n$$\n\\begin{aligned}\n\\left(2 a x_1+b\\right)^2 & =4 a^2 x_1^2+4 a b x_1+b^2 \\\\\n& =4 a\\left(a x_1^2+b x_1+c\\right)+b^2-4 a c \\\\\n& =b^2-4 a c .\n\\end{aligned}\n$$", + "remark": "", + "figures": [] +} \ No newline at end of file diff --git a/processed_dataset/calculation/0955.json b/processed_dataset/calculation/0955.json new file mode 100644 index 0000000000000000000000000000000000000000..2f881223c49f6c80d47b1fb056dec40b4aa93947 --- /dev/null +++ b/processed_dataset/calculation/0955.json @@ -0,0 +1,8 @@ +{ + "source_file": "./raw_volume-zh/volume8/exercise1.tex", + "problem_type": "calculation", + "problem": "问题2. 若 $z_1 、 z_2 、 z_3 \\in \\mathbf{C}$, 则 $z_1=z_2=z_3$ 是 $\\left(z_1-z_2\\right)^2+\\left(z_2-z_3\\right)^2=0$ 成立的 ( ).\n(A) 充分但不必要条件\n(B) 必要但不充分条件\n(C) 充分且必要条件\n(D) 既不充分也不必要条件", + "solution": "A.\n充分是明显的; 若取 $z_1=\\mathrm{i}, z_2=0, z_3=1$, 有 $\\left(z_1-z_2\\right)^2+\\left(z_2-z_3\\right)^2= -1+1=0$, 而 $\\mathrm{i} \\neq 0 \\neq 1$, 即说明条件是不必要的.", + "remark": "", + "figures": [] +} \ No newline at end of file diff --git a/processed_dataset/calculation/0956.json b/processed_dataset/calculation/0956.json new file mode 100644 index 0000000000000000000000000000000000000000..96e70103b013f7e6d4ac0b0b597aa6cd565b376b --- /dev/null +++ b/processed_dataset/calculation/0956.json @@ -0,0 +1,8 @@ +{ + "source_file": "./raw_volume-zh/volume8/exercise1.tex", + "problem_type": "calculation", + "problem": "问题3. 已知实数 $a 、 x 、 y$ 满足 $a^2+(4+\\mathrm{i}) a+2 x y+(x-y) \\mathrm{i}=0$, 则点 $(x, y)$ 的轨迹是 ( ).\n(A) 直线\n(B) 圆心在原点的圆\n(C) 圆心不在原点的圆\n(D) 椭圆", + "solution": "C.\n将题设之式整理得\n$$\na^2+4 a+2 x y+(a+x-y) \\mathrm{i}=0,\n$$\n则且\n$$\n\\begin{gathered}\na^2+4 a+2 x y=0, \\label{eq1}\\\\\na+x-y=0 . \\label{eq2}\n\\end{gathered}\n$$\n由式\\ref{eq2}, $a=y-x$, 代入式\\ref{eq1}得\n$$\n(y-x)^2+4(y-x)+2 x y=0,\n$$\n即\n$$\n\\begin{gathered}\nx^2+y^2-4 x+4 y=0, \\\\\n(x-2)^2+(y+2)^2=8 .\n\\end{gathered}\n$$\n故应选取 C", + "remark": "", + "figures": [] +} \ No newline at end of file diff --git a/processed_dataset/calculation/0957.json b/processed_dataset/calculation/0957.json new file mode 100644 index 0000000000000000000000000000000000000000..e708e96d0bfaaa21b5d88e32119dcd04603ec37d --- /dev/null +++ b/processed_dataset/calculation/0957.json @@ -0,0 +1,8 @@ +{ + "source_file": "./raw_volume-zh/volume8/exercise1.tex", + "problem_type": "calculation", + "problem": "问题5. 已知 $a \\in \\mathbf{R}$,试问:复数\n$$\nz=\\left(a^2-2 a+3\\right)-\\left(a^2-2 a+2\\right) \\mathrm{i}\n$$\n所对应的点在第几象限? 复数 $z$ 所对应点的轨迹是什么?", + "solution": "因为 $\\operatorname{Re}(z)=a^2-2 a+3=(a-1)^2+2 \\geqslant 2$,\n$$\n\\operatorname{Im}(z)=-\\left(a^2-2 a+2\\right)=-(a-1)^2-1 \\leqslant-1,\n$$\n所以 $\\operatorname{Re}(z)>0, \\operatorname{Im}(z)<0$, 故复数 $z$ 所对应的点在第四象限内.\n设 $z=x+y \\mathrm{i}(x, y \\in \\mathbf{R})$, 则 $\\left\\{\\begin{array}{l}x=a^2-2 a+3, \\\\ y=-\\left(a^2-2 a+2\\right),\\end{array}\\right.$ 消去 $a^2-2 a$, 得 $y= -x+1(x \\geqslant 2)$.\n所以, 复数 $z$ 所对应点的轨迹是以 $(2,-1)$ 为端点的一条射线 $y=-x+ 1(x \\geqslant 2)$.", + "remark": "", + "figures": [] +} \ No newline at end of file diff --git a/processed_dataset/calculation/0958.json b/processed_dataset/calculation/0958.json new file mode 100644 index 0000000000000000000000000000000000000000..cf122db472d8c7ccd4dd14bbcb325307f1befa38 --- /dev/null +++ b/processed_dataset/calculation/0958.json @@ -0,0 +1,8 @@ +{ + "source_file": "./raw_volume-zh/volume8/exercise1.tex", + "problem_type": "calculation", + "problem": "问题6. 设 $z_1 、 z_2 \\in \\mathbf{C}$, 且 $z_1 z_2 \\neq 0, A=z_1 \\overline{z_1}+z_2 \\overline{z_2}, B=z_1 \\overline{z_2}+\\overline{z_1} z_2$, 试问: $A$ 与 $B$ 能否比较大小关系? 若能, 请指明大小关系; 若不能, 请说明理由.", + "solution": "只有当 $A$ 与 $B$ 均为实数时, 二者之间才能比较大小.\n因为\n$$\nA=\\left|z_1\\right|^2+\\left|z_2\\right|^2,\n$$\n所以 $A$ 是实数.\n又因为\n$$\n\\bar{B}=\\overline{z_1 \\overline{z_2}+\\overline{z_1} z_2}=z_1 \\overline{z_2}+\\overline{z_1} z_2=B,\n$$\n所以 $B$ 也是实数.\n故 $A 、 B$ 二者之间可以比较大小.\n事实上\n$$\n\\begin{aligned}\nA-B & =z_1\\left(\\overline{z_1}-\\overline{z_2}\\right)-z_2\\left(\\overline{z_1}-\\overline{z_2}\\right) \\\\\n& =\\left(z_1-z_2\\right)\\left(\\overline{z_1}-\\overline{z_2}\\right) \\\\\n& =\\left|z_1-z_2\\right|^2 \\geqslant 0 .\n\\end{aligned}\n$$\n当 $z_1=z_2$ 时, $A=B$;\n当 $z_1 \\neq z_2$ 时, $A>B$.", + "remark": "注:(1) 在比较两实数的大小时, 对 \"不小于\"、\"不大于\"的情形, 要对其中的\"相等\"分而述之.\n(2) 以 $\\bar{z}=z$ 来说明 $z$ 为实数是复数问题中的常用做法.", + "figures": [] +} \ No newline at end of file diff --git a/processed_dataset/calculation/0959.json b/processed_dataset/calculation/0959.json new file mode 100644 index 0000000000000000000000000000000000000000..1d63f55d0a45c00f0e68c1cc48ca0dfc4bb549e6 --- /dev/null +++ b/processed_dataset/calculation/0959.json @@ -0,0 +1,8 @@ +{ + "source_file": "./raw_volume-zh/volume8/exercise2.tex", + "problem_type": "calculation", + "problem": "问题1. 设 $z$ 是模为 1 的复数, 则函数 $f(z)=z^2+\\frac{1}{z^2}$ 的最小值 ( ).\n(A) 是 0\n(B) 是 -2\n(C) 是 2\n(D) 不存在", + "solution": "B.\n由 $|z|=1$, 可令 $z=\\cos \\theta+i \\sin \\theta$, 则\n$$\n\\begin{aligned}\nf(z) & =z^2+\\frac{1}{z^2}=z^2+\\overline{z^2} \\\\\n& =(\\cos 2 \\theta+i \\sin 2 \\theta)+(\\cos 2 \\theta-i \\sin 2 \\theta) \\\\\n& =2 \\cos 2 \\theta .\n\\end{aligned}\n$$\n故当 $\\cos 2 \\theta=-1$ (或 $z=\\mathrm{i}$ )时,函数 $f(z)$ 有最小值 -2 .", + "remark": "", + "figures": [] +} \ No newline at end of file diff --git a/processed_dataset/calculation/0960.json b/processed_dataset/calculation/0960.json new file mode 100644 index 0000000000000000000000000000000000000000..dc6d0dc95a52bc04cce34cd1a46091a4cc46759e --- /dev/null +++ b/processed_dataset/calculation/0960.json @@ -0,0 +1,10 @@ +{ + "source_file": "./raw_volume-zh/volume8/exercise2.tex", + "problem_type": "calculation", + "problem": "问题2. 已知 $\\left|z_1\\right|=\\left|z_2\\right|=1, z_1+z_2=\\frac{1}{2}+\\frac{\\sqrt{3}}{2} \\mathrm{i}$, 求复数 $z_1, z_2$.", + "solution": "如图(), 设 $\\overrightarrow{O A}$ 对应于复数 $z_1, \\overrightarrow{O B}$ 对应于复数 $z_2, z_1+z_2$ 对应于向量 $\\overrightarrow{O C}$.\n由 $z_1+z_2=\\frac{1}{2}+\\frac{\\sqrt{3}}{2} \\mathrm{i}$ 知, $\\left|z_1+z_2\\right|=1$. 所以 $\\triangle A O C$ 和 $\\triangle B O C$ 都是等边三角形, 于是\n$$\n\\begin{aligned}\n& z_1=1, z_2=-\\frac{1}{2}+\\frac{\\sqrt{3}}{2} \\mathrm{i} \\text { 或 } \\\\\n& \\dot{z}_1=-\\frac{1}{2}+\\frac{\\sqrt{3}}{2} \\mathrm{i}, z_2=1 .\n\\end{aligned}\n$$", + "remark": "", + "figures": [ + "./images/volume8/figures/fig-c2a2.png" + ] +} \ No newline at end of file diff --git a/processed_dataset/calculation/0961.json b/processed_dataset/calculation/0961.json new file mode 100644 index 0000000000000000000000000000000000000000..b7e7937575b2abe7b7f465bbfe127ad75ad38f4b --- /dev/null +++ b/processed_dataset/calculation/0961.json @@ -0,0 +1,11 @@ +{ + "source_file": "./raw_volume-zh/volume8/exercise2.tex", + "problem_type": "calculation", + "problem": "问题3. 已知非零复数 $z$ 满足 $|z-\\mathrm{i}|=1$, 且 $\\arg z=\\theta$, 求\n(1) $\\theta$ 的取值范围; (2) 复数 $z$ 的模 (用 $\\theta$ 表示);(3)复数 $z^2-z i$ 的辐角.", + "solution": "(1) 因为 $|z-\\mathrm{i}|=1$, 故 $z$ 在复平面上的对应点 $P$ 在以 $(0,1)$ 为圆心, 半径为 1 的圆上 (去除 $(0,0)$ 点), 如图(), 所以 $\\theta$ 的取值范围是 $0< \\theta<\\pi$.\n(2) 如图(), 在 Rt $\\triangle A O P$ 中, 因为 $|O P|=2 \\sin \\theta$, 故 $|z|=2 \\sin \\theta$.\n(3) 由于 $|z-\\mathrm{i}|=1$, 故可令 $z-\\mathrm{i}=\\cos \\varphi+\\mathrm{i} \\sin \\varphi(\\varphi \\in \\mathbf{R})$, 于是\n$$\n\\begin{aligned}\nz^2-z \\mathrm{i} & =z(z-\\mathrm{i})=2 \\sin \\theta(\\cos \\theta+\\mathrm{i} \\sin \\theta) \\cdot(\\cos \\varphi+\\mathrm{i} \\sin \\varphi) \\\\\n& =2 \\sin \\theta[\\cos (\\theta+\\varphi)+\\mathrm{i} \\cdot \\sin (\\theta+\\varphi)] .\n\\end{aligned}\n$$\n又由 $\\cos \\varphi+\\mathrm{i} \\sin \\varphi=z-\\mathrm{i}$\n$$\n\\begin{aligned}\n& =2 \\sin \\theta(\\cos \\theta+i \\sin \\theta)-\\mathrm{i} \\\\\n& =2 \\sin \\theta \\cos \\theta+\\mathrm{i}\\left(2 \\sin ^2 \\theta-1\\right) .\n\\end{aligned}\n$$\n$$\n\\begin{aligned}\n& =\\sin 2 \\theta-i \\cos 2 \\theta \\\\\n& =\\cos \\left(2 \\theta-\\frac{\\pi}{2}\\right)+i \\sin \\left(2 \\theta-\\frac{\\pi}{2}\\right) .\n\\end{aligned}\n$$\n所以 $\\varphi=2 k \\pi+2 \\theta-\\frac{\\pi}{2}(k \\in \\mathbf{Z}), \\varphi+\\theta=2 k \\pi+3 \\theta-\\frac{\\pi}{2}(k \\in \\mathbf{Z})$.\n即 $\\operatorname{Arg}\\left(z^2-z \\mathrm{i}\\right)=2 k \\pi+3 \\theta-\\frac{\\pi}{2}(k \\in \\mathbf{Z})$.", + "remark": "注:对于已知 $|z|=r(r>0)$ 的有关问题, 可以从以下四个方面去思考:\n(1) 令 $z=r(\\cos \\theta+\\mathrm{i} \\sin \\theta)$;\n(2) 令 $z=a+b \\mathrm{i}(a, b \\in \\mathbf{R})$ 且 $a^2+b^2=r^2$;\n(3) 由 $|z|^2=r^2$ 得 $z \\bar{z}=r^2, z=\\frac{r^2}{\\bar{z}}, \\bar{z}=\\frac{r^2}{z}$;\n(4) $z$ 在复平面内的对应点在以原点为圆心 $r$ 为半径的圆上.", + "figures": [ + "./images/volume8/figures/fig-c2a3-1.png", + "./images/volume8/figures/fig-c2a3-2.png" + ] +} \ No newline at end of file diff --git a/processed_dataset/calculation/0962.json b/processed_dataset/calculation/0962.json new file mode 100644 index 0000000000000000000000000000000000000000..2b92021663ffd3b1cb893f1d57c575f874749a85 --- /dev/null +++ b/processed_dataset/calculation/0962.json @@ -0,0 +1,8 @@ +{ + "source_file": "./raw_volume-zh/volume8/exercise2.tex", + "problem_type": "calculation", + "problem": "问题4. 已知等边 $\\triangle A B C$ 的两个顶点坐标是 $A(2,1), B(3,2)$, 求顶点 $C$ 的对应坐标.", + "solution": "记 $A, B, C$ 的对应复数分别为 $z_A=2+\\mathrm{i}, z_B=3+2 \\mathrm{i}, z_C$, 则由\n$$\nz_C=z_A+\\left(z_B-z_A\\right)\\left(\\cos 60^{\\circ} \\pm \\operatorname{isin} 60^{\\circ}\\right),\n$$\n得\n$$\nz_C=(2+i)+(1+i)\\left(\\frac{1}{2} \\pm \\frac{\\sqrt{3}}{2} i\\right)=\\frac{5 \\mp \\sqrt{3}}{2}+\\frac{3 \\pm \\sqrt{3}}{2} i,\n$$\n即 $C$ 点坐标是 $C\\left(\\frac{5-\\sqrt{3}}{2}, \\frac{3+\\sqrt{3}}{2}\\right)$ 或 $C\\left(\\frac{5+\\sqrt{3}}{2}, \\frac{3-\\sqrt{3}}{2}\\right)$.", + "remark": "", + "figures": [] +} \ No newline at end of file diff --git a/processed_dataset/calculation/0963.json b/processed_dataset/calculation/0963.json new file mode 100644 index 0000000000000000000000000000000000000000..c98d52a5e84b27fc5902acd191bc25b046acfc81 --- /dev/null +++ b/processed_dataset/calculation/0963.json @@ -0,0 +1,8 @@ +{ + "source_file": "./raw_volume-zh/volume8/exercise2.tex", + "problem_type": "calculation", + "problem": "问题5. 已知 $z_1 、 z_2 \\in \\mathbf{C}$, 且 $\\left|z_1\\right|=2,\\left|z_2\\right|=9,\\left|5 z_1-z_2\\right|=9$, 试求 $\\mid 5 z_1+ z_2 \\mid$ 的值.", + "solution": "由模引人辐角, 设 $z_1=2(\\cos \\alpha+i \\sin \\alpha), z_2=9(\\cos \\beta+\\mathrm{i} \\sin \\beta)$, 代入 $\\left|5 z_1-z_2\\right|=9$, 便得\n$$\n\\sqrt{(10 \\cos \\alpha-9 \\cos \\beta)^2+(10 \\sin \\alpha-9 \\sin \\beta)^2}=9,\n$$\n即\n$$\n\\begin{gathered}\n181-180 \\cos (\\alpha-\\beta)=81, \\\\\n\\cos (\\alpha-\\beta)=\\frac{100}{180} .\n\\end{gathered}\n$$\n故 $\\quad\\left|5 z_1+z_2\\right|=\\sqrt{181+180 \\cos (\\alpha-\\beta)}=\\sqrt{281}$.", + "remark": "注:用如下恒等式, 可给出此题的另一种解法.\n$$\n\\left|z_1+z_2\\right|^2+\\left|z_1-z_2\\right|^2=2\\left(\\left|z_1\\right|^2+\\left|z_2\\right|^2\\right) \\text {. }\n$$", + "figures": [] +} \ No newline at end of file diff --git a/processed_dataset/calculation/0964.json b/processed_dataset/calculation/0964.json new file mode 100644 index 0000000000000000000000000000000000000000..5684c40d03d6568e348d6142c6b3f4e372b031c4 --- /dev/null +++ b/processed_dataset/calculation/0964.json @@ -0,0 +1,10 @@ +{ + "source_file": "./raw_volume-zh/volume8/exercise2.tex", + "problem_type": "calculation", + "problem": "问题7. 已知复数 $z_1 、 z_2$ 满足 $\\left|z_1\\right|==\\left|z_2\\right|=1$.\n(1) 若 $z_1-z_2=\\frac{\\sqrt{6}}{3}+\\frac{\\sqrt{3}}{3} \\mathrm{i}$, 求 $z_1 、 z_2$ 的值;\n(2) 若 $z_1+z_2=\\frac{12}{13}-\\frac{5}{13} \\mathrm{i}$, 求 $z_1 z_2$ 的值.", + "solution": "注意到 $\\left|z_1+z_2\\right| 、\\left|z_1-z_2\\right|$ 分别是以 $\\left|z_1\\right| 、\\left|z_2\\right|$ 为邻边的平行四边形的对角线.\n(1) 因为 $\\left|z_1-z_2\\right|=1=\\left|z_1\\right|=\\left|z_2\\right|$, 所以 $\\left|z_1\\right| 、\\left|z_2\\right| 、\\left|z_1-z_2\\right|$ 是正三角形的边.\n复数 $z_1 、 z_2$ 分别可以看作 $z_1-z_2$ 按顺时针或逆时针旋转 $\\frac{\\pi}{3}$ 、 $\\frac{2 \\pi}{3}$ 得到.\n于是\n$$\n\\begin{aligned}\n& z_1=\\left(\\frac{\\sqrt{6}}{3}+\\frac{\\sqrt{3}}{3} \\mathrm{i}\\right)\\left(\\frac{1}{2}-\\frac{\\sqrt{3}}{2} \\mathrm{i}\\right)=\\frac{1}{6}[(\\sqrt{6}+3)+(\\sqrt{3}-3 \\sqrt{2}) \\mathrm{i}], \\\\\n& z_2=\\left(\\frac{\\sqrt{6}}{3}+\\frac{\\sqrt{3}}{3} \\mathrm{i}\\right)\\left(-\\frac{1}{2}-\\frac{\\sqrt{3}}{2} \\mathrm{i}\\right)=\\frac{1}{6}[(-\\sqrt{6}+3)+(-\\sqrt{3}-3 \\sqrt{2}) \\mathrm{i}]\n\\end{aligned}\n$$\n或 $z_1=\\left(\\frac{\\sqrt{6}}{3}+\\frac{\\sqrt{3}}{3} \\mathrm{i}\\right)\\left(\\frac{1}{2}+\\frac{\\sqrt{3}}{2} \\mathrm{i}\\right)=\\frac{1}{6}[(\\sqrt{6}-3)+(\\sqrt{3}+3 \\sqrt{2}) \\mathrm{i}]$,\n$$\nz_2=\\left(\\frac{\\sqrt{6}}{3}+\\frac{\\sqrt{3}}{3} \\mathrm{i}\\right)\\left(-\\frac{1}{2}+\\frac{\\sqrt{3}}{2} \\mathrm{i}\\right)=\\frac{1}{6}[(-\\sqrt{6}-3)+(-\\sqrt{3}+3 \\sqrt{2}) \\mathrm{i}] .\n$$\n(2) 因为 $\\left|z_1+z_2\\right|=1=\\left|z_1\\right|=\\left|z_2\\right|$, 所以 $\\left|z_1\\right| 、\\left|z_2\\right| 、\\left|z_1-z_2\\right|$ 组成顶角为 $\\frac{2 \\pi}{3}$, 以 $\\left|z_1\\right| 、\\left|z_2\\right|$ 为腰长的等腰三角形.\n$z_1+z_2$ 按逆(顺) 时针旋转 $\\frac{\\pi}{3}$ 后, 就可得出 $z_1$ 或 $z_2$ (如图()所示).\n设 $z_0=\\cos 60^{\\circ}+\\mathrm{i} \\sin 60^{\\circ}$, 则当 $z_1=\\left(z_1+z_2\\right) z_0$ 时, $z_2= \\frac{z_1+z_2}{z_0}$; 当 $z_1=\\frac{z_1+z_2}{z_0}$ 时, $z_2=\\left(z_1+z_2\\right) z_0$.\n于是 $z_1 z_2=\\left(z_1+z_2\\right)^2=\\left(\\frac{12}{13}-\\frac{5}{13} \\mathrm{i}\\right)^2=\\frac{119}{169}-\\frac{120}{169} \\mathrm{i}$.", + "remark": "注:对于第(1)小题,应用复数减法的几何意义就可直接求出 $z_1 、 z_2$, 但应注意复数旋转的方向和具有相同的始点.", + "figures": [ + "./images/volume8/figures/fig-c2a7.png" + ] +} \ No newline at end of file diff --git a/processed_dataset/calculation/0965.json b/processed_dataset/calculation/0965.json new file mode 100644 index 0000000000000000000000000000000000000000..df3fb508bc4df6efe5c6196dacd83cf1b647ad8f --- /dev/null +++ b/processed_dataset/calculation/0965.json @@ -0,0 +1,8 @@ +{ + "source_file": "./raw_volume-zh/volume8/exercise3.tex", + "problem_type": "calculation", + "problem": "问题1. $p^2 \\geqslant 4 q$ 是关于 $x$ 的实系数方程 $x^4+p x^2+q=0$ 有实根的 ( ).\n(A) 充分而不必要条件\n(B) 必要而不充分条件\n(C) 充分且必要条件\n(D) 非充分又非必要条件", + "solution": "B.\n令 $x^2=t$, 则原方程变形为 $t^2+p t+q=0$.\n原方程有实根的充要条件是关于 $t$ 的方程 有非负实根, 从而可知 $p^2 \\geqslant 4 q$ 是该方程 有非负实根的必要而不充分条件.", + "remark": "", + "figures": [] +} \ No newline at end of file diff --git a/processed_dataset/calculation/0966.json b/processed_dataset/calculation/0966.json new file mode 100644 index 0000000000000000000000000000000000000000..b7f036767b3f44441248c164a5666435b304ef90 --- /dev/null +++ b/processed_dataset/calculation/0966.json @@ -0,0 +1,8 @@ +{ + "source_file": "./raw_volume-zh/volume8/exercise3.tex", + "problem_type": "calculation", + "problem": "问题2. 设方程 $x^2-2 \\sqrt{2} x+m=0$ 的两个虚根为 $\\alpha$ 、 $\\beta$, 且 $|\\alpha-\\beta|=1$, 则实数 $m$ 的值等于 ( ).\n(A) $-\\frac{1}{4}$\n(B) $\\frac{1}{4}$\n(C) $-\\frac{9}{4}$\n(D) $\\frac{9}{4}$", + "solution": "D.\n由 $\\alpha+\\beta=2 \\sqrt{2},|\\alpha-\\beta|=1$, 而 $\\alpha-\\beta$ 是纯虚数, 从而 $\\alpha-\\beta= \\pm \\mathrm{i}$.\n$$\n4 \\alpha \\beta=(\\alpha+\\beta)^2-(\\alpha-\\beta)^2=(2 \\sqrt{2})^2+1=9 \\text {, 所以 } m=\\alpha \\beta=\\frac{9}{4} \\text {. }\n$$", + "remark": "", + "figures": [] +} \ No newline at end of file diff --git a/processed_dataset/calculation/0967.json b/processed_dataset/calculation/0967.json new file mode 100644 index 0000000000000000000000000000000000000000..a7444130342cbdd3daa31c5843e0fe0f18aaa57e --- /dev/null +++ b/processed_dataset/calculation/0967.json @@ -0,0 +1,8 @@ +{ + "source_file": "./raw_volume-zh/volume8/exercise3.tex", + "problem_type": "calculation", + "problem": "问题3. 方程 $a x^2+b|x|+c=0(a 、 b 、 c \\in \\mathbf{R}, a \\neq 0)$ 在复数集内的根的个数是 $n$, 则 ( ).\n(A) $n$ 最大是 2\n(B) $n$ 最大是 4\n(C) $n$ 最大是 6\n(D) $n$ 最大是 8", + "solution": "C.\n不妨设 $a=1$.\n由 $b 、 c 、|x|$ 均是实数,可知 $x^2$ 必是实数,因此 $x$ 是实数或纯虚数.\n若 $x$ 是实数 $u$, 则 $u^2+b u+c=0(u \\geqslant 0)$ 或 $u^2-b u+c=0(u<0)$.\n若 $x$ 是纯虚数 $v \\mathrm{i}$, 则 $v^2-b v-c=0(v \\geqslant 0)$ 或 $v^2+b v-c=0(v<0)$.\n对 $b 、 c$ 异号时, 有 6 个根满足要求; 对其他的情况, 满足要求的根都不足 6 个.", + "remark": "", + "figures": [] +} \ No newline at end of file diff --git a/processed_dataset/calculation/0968.json b/processed_dataset/calculation/0968.json new file mode 100644 index 0000000000000000000000000000000000000000..16e1d73fd705549de5bbcfb147493a39920c64b7 --- /dev/null +++ b/processed_dataset/calculation/0968.json @@ -0,0 +1,8 @@ +{ + "source_file": "./raw_volume-zh/volume8/exercise3.tex", + "problem_type": "calculation", + "problem": "问题5. 设非零复数 $z_1 、 z_2$ 在复平面内的对应点分别为 $A 、 B$, 且满足 $\\left|z_2\\right|=2$, $z_1^2-2 z_1 z_2+4 z_2^2=0$.\n(1) 试判断 $\\triangle A O B$ ( $O$ 为原点) 的形状;\n(2) 求 $\\triangle A O B$ 的面积.", + "solution": "(1) 由 $z_1^2-2 z_1 z_2+4 z_2^2=0$ 得 $z_1=\\frac{2 z_2 \\pm 2 \\sqrt{3} i z_2}{2}$, 即 $z_1=(1 \\pm \\sqrt{3} i) z_2$, 亦即 $z_1=2\\left(\\cos \\frac{\\pi}{3} \\pm i \\sin \\frac{\\pi}{3}\\right) z_2$.\n由此得 $\\triangle A O B$ 是直角三角形, 且 $\\angle A O B=60^{\\circ}, \\angle A B O=90^{\\circ}$;\n(2) $S_{\\triangle A O B}=\\frac{1}{2}|A O| \\cdot|B O| \\sin \\frac{\\pi}{3}=\\frac{\\sqrt{3}}{4} \\cdot 2 \\cdot|B O|^2=2 \\sqrt{3}$.", + "remark": "", + "figures": [] +} \ No newline at end of file diff --git a/processed_dataset/calculation/0969.json b/processed_dataset/calculation/0969.json new file mode 100644 index 0000000000000000000000000000000000000000..b740aeaa9051d5bb80d7d22e111c9dc51233a2af --- /dev/null +++ b/processed_dataset/calculation/0969.json @@ -0,0 +1,8 @@ +{ + "source_file": "./raw_volume-zh/volume8/exercise3.tex", + "problem_type": "calculation", + "problem": "问题7. 已知关于 $x$ 的二次方程 $a(1+\\mathrm{i}) x^2+\\left(1+a^2 \\mathrm{i}\\right) x+a^2+\\mathrm{i}=0$ 有实根, 求实数 $a$ 的值.", + "solution": "设原方程有一实根 $x_0$, 则\n$$\na(1+\\mathrm{i}) x_0^2+\\left(1+a^2 \\mathrm{i}\\right) x_0+a^2+\\mathrm{i}=0,\n$$\n即\n$$\n\\left(a x_0^2+x_0+a^2\\right)+\\mathrm{i}\\left(a x_0^2+a^2 x_0+1\\right)=0 .\n$$\n根据复数相等的充要条件,有\n$$\n\\left\\{\\begin{array}{l}\na x_0^2+x_0+a^2=0, \\\\\na x_0^2+a^2 x_0+1=0 .\n\\end{array}\\right.\n$$\n解得 $x_0=1$ 或 $a= \\pm 1$.\n当 $x_0=1$ 时, 代入 $a x_0^2+x_0+a^2=0$ 得 $a^2+a+1=0$. 因为 $a \\in \\mathbf{R}, a^2+ a+1=\\left(a+\\frac{1}{2}\\right)^2+\\frac{3}{4}>0$, 所以不成立.\n同理,当 $a=1$ 时,也不成立.\n当 $a=-1$ 时,方程变为 $x_0^2-x_0-1=0$, 所以 $x_0=\\frac{1 \\pm \\sqrt{5}}{2}$ 满足题意, 故 $a=-1$.", + "remark": "", + "figures": [] +} \ No newline at end of file diff --git a/processed_dataset/calculation/0970.json b/processed_dataset/calculation/0970.json new file mode 100644 index 0000000000000000000000000000000000000000..9c603e9c90400219539a763c5b3c1269d29e1758 --- /dev/null +++ b/processed_dataset/calculation/0970.json @@ -0,0 +1,8 @@ +{ + "source_file": "./raw_volume-zh/volume8/exercise4.tex", + "problem_type": "calculation", + "problem": "问题1. 在四边形 $A B C D$ 中, $\\overrightarrow{A B}=\\vec{a}+\\vec{b}, \\overrightarrow{B C}=-4 \\vec{a}-\\vec{b}, \\overrightarrow{C D}=-5 \\vec{a}-2 \\vec{b}$, 四边形 $A B C D$ 的形状为 ( ).\n(A) 长方形\n(B) 平行四边形\n(C) 菱形\n(D) 梯形", + "solution": "D.\n$$\n\\overrightarrow{A D}=\\overrightarrow{A B}+\\overrightarrow{B C}+\\overrightarrow{C D}=-8 \\vec{a}-2 \\vec{b}=2 \\overrightarrow{B C}\n$$", + "remark": "", + "figures": [] +} \ No newline at end of file diff --git a/processed_dataset/calculation/0971.json b/processed_dataset/calculation/0971.json new file mode 100644 index 0000000000000000000000000000000000000000..e43c419daca745992ea4a536b5b72b460cdbe251 --- /dev/null +++ b/processed_dataset/calculation/0971.json @@ -0,0 +1,8 @@ +{ + "source_file": "./raw_volume-zh/volume8/exercise4.tex", + "problem_type": "calculation", + "problem": "问题2. 若 $\\overrightarrow{A B}=3 \\vec{e}, \\overrightarrow{C D}=4 \\vec{e}$, 且 $|\\overrightarrow{A D}|=|\\overrightarrow{C B}|$, 则四边形 $A B C D$ 是( ).\n(A) 平行四边形\n(B) 梯形\n(C) 等腰梯形\n(D) 菱形", + "solution": "C.\n$$\nA B / / C D, A D=C B \\text {. }\n$$", + "remark": "", + "figures": [] +} \ No newline at end of file diff --git a/processed_dataset/calculation/0972.json b/processed_dataset/calculation/0972.json new file mode 100644 index 0000000000000000000000000000000000000000..f154f10ccebcf0040ec0ae8779a374ffd75f65c9 --- /dev/null +++ b/processed_dataset/calculation/0972.json @@ -0,0 +1,8 @@ +{ + "source_file": "./raw_volume-zh/volume8/exercise4.tex", + "problem_type": "calculation", + "problem": "问题3. 平行四边形 $A B C D$ 中, 对角线 $A C$ 和 $B D$ 交于点 $O$. 若 $\\overrightarrow{A C}=\\vec{a}, \\overrightarrow{B D}=\\vec{b}$, 那么用 $\\vec{a}$ 和 $\\vec{b}$ 表示 $\\overrightarrow{A B}$ 为 $(\\quad)$.\n(A) $\\frac{1}{2}(\\vec{a}+\\vec{b})$\n(B) $\\frac{1}{2}(\\vec{a}-\\vec{b})$\n(C) $\\frac{1}{2}(\\vec{b}-\\vec{a})$\n(D) $\\vec{b}-\\vec{a}$", + "solution": "B.\n$$\n\\overrightarrow{A B}=\\overrightarrow{O B}-\\overrightarrow{O A}=\\left(-\\frac{\\vec{b}}{2}\\right)-\\left(-\\frac{\\vec{a}}{2}\\right)=\\frac{1}{2}(\\vec{a}-\\vec{b}) .\n$$", + "remark": "", + "figures": [] +} \ No newline at end of file diff --git a/processed_dataset/calculation/0973.json b/processed_dataset/calculation/0973.json new file mode 100644 index 0000000000000000000000000000000000000000..92db5bbb5577e9f06386e1ab0f9772c6ce347534 --- /dev/null +++ b/processed_dataset/calculation/0973.json @@ -0,0 +1,8 @@ +{ + "source_file": "./raw_volume-zh/volume8/exercise4.tex", + "problem_type": "calculation", + "problem": "问题4. 若 $\\overrightarrow{O A}=\\vec{a}+\\vec{b}, \\overrightarrow{A B}=3(\\vec{a}-\\vec{b}), \\overrightarrow{C B}=3 \\vec{a}+\\vec{b}$, 则 $\\overrightarrow{O C}=$", + "solution": "$\\vec{a}-3 \\vec{b}$.\n$$\n\\begin{aligned}\n\\overrightarrow{O C} & =\\overrightarrow{O A}+\\overrightarrow{A B}+\\overrightarrow{B C}=\\overrightarrow{O A}+\\overrightarrow{A B}-\\overrightarrow{C B} \\\\\n& =\\vec{a}+\\vec{b}+3(\\vec{a}-\\vec{b})-(3 \\vec{a}+\\vec{b})=\\vec{a}-3 \\vec{b}\n\\end{aligned}\n$$", + "remark": "", + "figures": [] +} \ No newline at end of file diff --git a/processed_dataset/calculation/0974.json b/processed_dataset/calculation/0974.json new file mode 100644 index 0000000000000000000000000000000000000000..44506ae86d5d798721abe3d6501bb4ffbba29677 --- /dev/null +++ b/processed_dataset/calculation/0974.json @@ -0,0 +1,8 @@ +{ + "source_file": "./raw_volume-zh/volume8/exercise4.tex", + "problem_type": "calculation", + "problem": "问题5. 已知梯形 $O A B C$ 中, $\\overrightarrow{C B} / / \\overrightarrow{O A}$, 且 $|\\overrightarrow{C B}|=\\frac{1}{3}|\\overrightarrow{O A}|$. 若 $\\overrightarrow{O A}=\\vec{a}, \\overrightarrow{O C}= \\vec{b}$, 则 $\\overrightarrow{A B}=$", + "solution": "$\\vec{b}-\\frac{2 \\vec{a}}{3}$.\n$$\n\\overrightarrow{A B}=\\overrightarrow{A O}+\\overrightarrow{O C}+\\overrightarrow{C B}=-\\vec{a}+\\vec{b}+\\frac{\\vec{a}}{3}=\\vec{b}-\\frac{2 \\vec{a}}{3} .\n$$", + "remark": "", + "figures": [] +} \ No newline at end of file diff --git a/processed_dataset/calculation/0975.json b/processed_dataset/calculation/0975.json new file mode 100644 index 0000000000000000000000000000000000000000..d36397ef4e8bb7a0d013914930bf2099e8b6f962 --- /dev/null +++ b/processed_dataset/calculation/0975.json @@ -0,0 +1,8 @@ +{ + "source_file": "./raw_volume-zh/volume8/exercise5.tex", + "problem_type": "calculation", + "problem": "问题1. 空间四点 $A, B, C, D$ 满足 $|\\overrightarrow{A B}|=3,|\\overrightarrow{B C}|=7,|\\overrightarrow{C D}|=11,|\\overrightarrow{D A}|=$ 9 , 则 $\\overrightarrow{A C} \\cdot \\overrightarrow{B D}$ 的取值 ( ).\n(A) 只有一个\n(B) 有两个\n(C) 有四个\n(D) 有无穷多个", + "solution": "A.\n因为 $\\overrightarrow{A B}^2+\\overrightarrow{C D}^2=3^2+11^2=130=7^2+9^2=\\overrightarrow{B C}^2+\\overrightarrow{D A}^2$, 由 $\\overrightarrow{A B}+\\overrightarrow{B C}+ \\overrightarrow{C D}+\\overrightarrow{D A}=\\overrightarrow{0}$, 得 $\\overrightarrow{A B}+\\overrightarrow{C D}=-(\\overrightarrow{B C}+\\overrightarrow{D A})$, 两边平方得 $\\overrightarrow{A B} \\cdot \\overrightarrow{C D}=\\overrightarrow{B C} \\cdot \\overrightarrow{D A}$, 故 $\\overrightarrow{A B} \\cdot \\overrightarrow{C D}=-\\overrightarrow{A D} \\cdot \\overrightarrow{B C}$, 于是\n$$\n\\begin{aligned}\n\\overrightarrow{A C} \\cdot \\overrightarrow{B D} & =(\\overrightarrow{A B}+\\overrightarrow{B C})(\\overrightarrow{B C}+\\overrightarrow{C D}) \\\\\n& =(\\overrightarrow{A B}+\\overrightarrow{B C}+\\overrightarrow{C D}) \\overrightarrow{B C}+\\overrightarrow{A B} \\cdot \\overrightarrow{C D} \\\\\n& =\\overrightarrow{A D} \\cdot \\overrightarrow{B C}+\\overrightarrow{A B} \\cdot \\overrightarrow{C D}=0 .\n\\end{aligned}\n$$\n$\\overrightarrow{A C} \\cdot \\overrightarrow{B D}$ 只有一个值 0. 故选 A.", + "remark": "", + "figures": [] +} \ No newline at end of file diff --git a/processed_dataset/calculation/0976.json b/processed_dataset/calculation/0976.json new file mode 100644 index 0000000000000000000000000000000000000000..6a57afc5404e5c4c96a46389f141d79be607dad9 --- /dev/null +++ b/processed_dataset/calculation/0976.json @@ -0,0 +1,8 @@ +{ + "source_file": "./raw_volume-zh/volume8/exercise5.tex", + "problem_type": "calculation", + "problem": "问题2. $|\\vec{a}|=5,|\\vec{b}|=3,|\\vec{a}-\\vec{b}|=6, \\vec{a} \\cdot \\vec{b}=(\\quad)$.", + "solution": "-1 .\n$$\n|\\vec{a}-\\vec{b}|^2=\\vec{a}^2-2 \\vec{a} \\cdot \\vec{b}+\\vec{b}^2 .\n$$", + "remark": "", + "figures": [] +} \ No newline at end of file diff --git a/processed_dataset/calculation/0977.json b/processed_dataset/calculation/0977.json new file mode 100644 index 0000000000000000000000000000000000000000..9e4e2f5a5263fed975063b1698ceb750822c51db --- /dev/null +++ b/processed_dataset/calculation/0977.json @@ -0,0 +1,8 @@ +{ + "source_file": "./raw_volume-zh/volume8/exercise5.tex", + "problem_type": "calculation", + "problem": "问题3. 已知向量 $\\vec{a}=\\{2,-3\\}$, 向量 $\\overrightarrow{A B}$ 与 $\\vec{a}$ 垂直, 且 $|\\overrightarrow{A B}|=3 \\sqrt{13}$, 点 $A$ 坐标为 $(-3,1)$, 求位置向量 $\\overrightarrow{O B}$ 的坐标.", + "solution": "设 $\\overrightarrow{O B}=\\{x, y\\}, \\overrightarrow{A B}=\\{x+3, y-1\\}$. 因为 $\\overrightarrow{A B}$ 与 $\\vec{a}$ 垂直, $|\\overrightarrow{A B}|= 3 \\sqrt{13}$, 所以\n$$\n\\left\\{\\begin{array}{l}\n2(x+3)-3(y-1)=0, \\\\\n(x+3)^2+(y-1)^2=(3 \\sqrt{13})^2,\n\\end{array}\\right.\n$$\n得\n$$\n\\left\\{\\begin{array} { l } \n{ x = 6 , } \\\\\n{ y = 7 }\n\\end{array} \\text { 或 } \\left\\{\\begin{array}{l}\nx=-12, \\\\\ny=-5 .\n\\end{array}\\right.\\right.\n$$", + "remark": "", + "figures": [] +} \ No newline at end of file diff --git a/processed_dataset/calculation/0978.json b/processed_dataset/calculation/0978.json new file mode 100644 index 0000000000000000000000000000000000000000..6f4ea7e922741edb20b0e2400f3b5a3438aa2dcb --- /dev/null +++ b/processed_dataset/calculation/0978.json @@ -0,0 +1,8 @@ +{ + "source_file": "./raw_volume-zh/volume8/exercise5.tex", + "problem_type": "calculation", + "problem": "问题5. 若 $\\vec{a}=\\{\\cos \\alpha, \\sin \\alpha\\}, \\vec{b}=\\{\\cos \\beta, \\sin \\beta\\}$, 且满足 $|k \\vec{a}+\\vec{b}|=\\sqrt{3}|\\vec{a}-k \\vec{b}|(k>0)$.\n(1)用 $\\alpha 、 \\beta$ 表示 $\\vec{a} \\cdot \\vec{b}$;\n(2) 用 $k$ 表示 $\\vec{a} \\cdot \\vec{b}$;\n(3) 求 $\\vec{a} \\cdot \\vec{b}$ 的最小值及此时 $\\vec{a}$ 与 $\\vec{b}$ 所成的角的大小 $(0 \\leqslant \\theta \\leqslant \\pi)$.", + "solution": "(1) $\\vec{a} \\cdot \\vec{b}=\\cos \\alpha \\cos \\beta+\\sin \\alpha \\sin \\beta=\\cos (\\alpha-\\beta)$.\n(2) $|k \\vec{a}+\\vec{b}|^2=3|\\vec{a}-k \\vec{b}|^2$, 即 $k^2 \\vec{a}^2+2 k \\vec{a} \\cdot \\vec{b}+\\vec{b}^2=3\\left(\\vec{a}^2-2 k \\vec{a} \\cdot \\vec{b}+k^2 \\vec{b}^2\\right)$\n所以\n$$\n\\vec{a} \\cdot \\vec{b}=\\frac{k^2+1}{4 k}(k>0) .\n$$\n(3) $\\vec{a} \\cdot \\vec{b}=\\frac{k^2+1}{4 k} \\geqslant \\frac{1}{2}$, 当且仅当 $k=1$ 时等号成立, 此时 $\\theta=\\frac{\\pi}{3}$.", + "remark": "", + "figures": [] +} \ No newline at end of file diff --git a/processed_dataset/calculation/0979.json b/processed_dataset/calculation/0979.json new file mode 100644 index 0000000000000000000000000000000000000000..b4d5aee724932fdcad9dd80e52b14eaa753a1a1d --- /dev/null +++ b/processed_dataset/calculation/0979.json @@ -0,0 +1,8 @@ +{ + "source_file": "./raw_volume-zh/volume8/exercise5.tex", + "problem_type": "calculation", + "problem": "问题6. 已知 $|\\vec{a}|=\\sqrt{2},|\\vec{b}|=3, \\vec{a}$ 与 $\\vec{b}$ 的夹角为 $45^{\\circ}$, 求使 $\\vec{a}+\\lambda \\vec{b}$ 与 $\\lambda \\vec{a}+\\vec{b}$ 的夹角为锐角时, $\\lambda$ 的取值范围.", + "solution": "由题意得 $(\\vec{a}+\\lambda \\vec{b}) \\cdot(\\lambda \\vec{a}+\\vec{b})>0$, 则\n$$\n\\lambda \\vec{a}^2+\\left(\\lambda^2+1\\right) \\vec{a} \\cdot \\vec{b}+\\lambda \\vec{b}^2>0 .\n$$\n又 $\\vec{a} \\cdot \\vec{b}=|\\vec{a}||\\vec{b}| \\cos 45^{\\circ}=3$, 于是\n$$\n3 \\lambda^2+11 \\lambda+3>0 .\n$$\n解得 $\\lambda<\\frac{-11-\\sqrt{85}}{6}$ 或 $\\lambda>\\frac{-11+\\sqrt{85}}{6}$.", + "remark": "", + "figures": [] +} \ No newline at end of file diff --git a/processed_dataset/calculation/0980.json b/processed_dataset/calculation/0980.json new file mode 100644 index 0000000000000000000000000000000000000000..1927e67f79ba8a9afdbd8b8d6beefafae8874bc6 --- /dev/null +++ b/processed_dataset/calculation/0980.json @@ -0,0 +1,8 @@ +{ + "source_file": "./raw_volume-zh/volume8/exercise5.tex", + "problem_type": "calculation", + "problem": "问题7. 已知点 $A(0,2,3) 、 B(-2,1,6) 、 C(1,-1,5)$.\n(1) 求以 $\\overrightarrow{A B} 、 \\overrightarrow{A C}$ 为边的平行四边形的面积;\n(2) 若 $|\\vec{a}|=\\sqrt{3}$, 且 $\\vec{a}$ 分别与 $\\overrightarrow{A B} 、 \\overrightarrow{A C}$ 垂直, 求向量 $\\vec{a}$ 的坐标.", + "solution": "(1) $\\overrightarrow{A B}=\\{-2,-1,3\\}, \\overrightarrow{A C}=\\{1,-3,2\\}$,\n$$\n\\cos \\theta=\\frac{\\overrightarrow{A B} \\cdot \\overrightarrow{A C}}{|\\overrightarrow{A B}| \\cdot|\\overrightarrow{A C}|}=\\frac{1}{2}, \\sin \\theta=\\frac{\\sqrt{3}}{2},\n$$\n$$\nS=|\\overrightarrow{A B}| \\cdot|\\overrightarrow{A C}| \\sin \\theta=\\sqrt{14} \\cdot \\sqrt{14} \\cdot \\frac{\\sqrt{3}}{2}=7 \\sqrt{3}\n$$\n(2) 设 $\\vec{a}=\\{x, y, z\\}$. 由 $\\overrightarrow{A B} \\cdot \\vec{a}=0, \\overrightarrow{A C} \\cdot \\vec{a}=0,|\\vec{a}|=\\sqrt{3}$ 得", + "remark": "", + "figures": [] +} \ No newline at end of file diff --git a/processed_dataset/calculation/0981.json b/processed_dataset/calculation/0981.json new file mode 100644 index 0000000000000000000000000000000000000000..dc6c2a7084ef8941c3c64352b39fc03cab0f5e00 --- /dev/null +++ b/processed_dataset/calculation/0981.json @@ -0,0 +1,10 @@ +{ + "source_file": "./raw_volume-zh/volume8/exercise6.tex", + "problem_type": "calculation", + "problem": "问题1. 在长方体 $O A B C-O_1 A_1 B_1 C_1$ 中, $|O A|=2,|A B|=3,\\left|A A_1\\right|=2, E$\n是 $B C$ 中点.\n(1) 求异面直线 $A O_1$ 和 $B_1 E$ 所成的角;\n(2) 作 $O_1 D \\perp A C$ 于点 $D$, 求向量 $\\overrightarrow{O_1 D}$.", + "solution": "如图(), 建立空间直角坐标系, 则 $A(2,0$, $0), O_1(0,0,2), B(2,3,0), C(0,3,0), B_1 (2,3,2), E(1,3,0)$.\n(1) $\\overrightarrow{A O_1}=\\{-2,0,2\\}, \\overrightarrow{B_1 E}=\\{-1,0$, $-2\\}$.\n$$\n\\cos \\theta=\\frac{\\overrightarrow{A O_1} \\cdot \\overrightarrow{B_1 E}}{\\left|\\overrightarrow{A O_1}\\right| \\cdot\\left|\\overrightarrow{B_1 E}\\right|}=\\frac{2-4}{2 \\sqrt{2} \\times \\sqrt{5}}=\n$$\n$-\\frac{\\sqrt{10}}{10}$, 所以异面直线 $A O_1$ 和 $B_1 E$ 所成的角为\n$\\arccos \\frac{\\sqrt{10}}{10}$\n(2) $\\overrightarrow{A C}=\\{-2,3,0\\}$, 在 $x O y$ 坐标系中, 直线 $A C$ 的方程为 $\\frac{x}{2}+\\frac{y}{3}=1$, 则 $D\\left(t, \\frac{6-3 t}{2}, 0\\right), t \\in \\mathbf{R}$, 所以 $\\overrightarrow{O_1 D}=\\left\\{t, \\frac{6-3 t}{2},-2\\right\\}$.\n又因为 $O_1 D \\perp A C$, 所以 $\\overrightarrow{O_1 D} \\cdot \\overrightarrow{A C}=0$, 故 $-2 t+\\frac{18-9 t}{2}=0$, 即 $t=\\frac{18}{13}$.\n所以 $\\overrightarrow{O_1 D}=\\left\\{\\frac{18}{13}, \\frac{12}{13},-2\\right\\}$.", + "remark": "", + "figures": [ + "./images/volume8/figures/fig-c6a1.png" + ] +} \ No newline at end of file diff --git a/processed_dataset/calculation/0982.json b/processed_dataset/calculation/0982.json new file mode 100644 index 0000000000000000000000000000000000000000..7d20b3c616272ed47fdb045f1dec3b7f75ce556d --- /dev/null +++ b/processed_dataset/calculation/0982.json @@ -0,0 +1,8 @@ +{ + "source_file": "./raw_volume-zh/volume8/exercise6.tex", + "problem_type": "calculation", + "problem": "问题2. 已知非零向量 $\\vec{a}$ 和 $\\vec{b}$ 不平行, 且 $\\vec{m}=3 \\vec{a}-2 \\vec{b}, \\vec{n}=2 \\vec{a}+k \\vec{b}$, 问是否存在实数 $k$, 使 $\\vec{m}$ 和 $\\vec{n}$ 互相平行? 若存在求出 $k$ 的值, 若不存在, 说明理由.", + "solution": "因为 $\\vec{a}$ 和 $\\vec{b}$ 不平行, 且 $|\\vec{a}| \\neq 0,|\\vec{b}| \\neq 0$, 所以 $\\vec{a}$ 和 $\\vec{b}$ 可以确定平面的一个坐标系 (有可能不是直角坐标系), 且分别以 $\\vec{a}$ 和 $\\vec{b}$ 的方向作为两坐标轴的正方向, 以 $|\\vec{a}|$ 和 $|\\vec{b}|$ 作为两坐标轴的单位长度, 则\n$$\n\\vec{m}=\\{3,-2\\}, \\vec{n}=\\{2, k\\} .\n$$\n如果 $\\vec{m}$ 和 $\\vec{n}$ 互相平行, 则 $\\frac{2}{3}=\\frac{k}{-2}$, 即 $k=-\\frac{4}{3}$.", + "remark": "", + "figures": [] +} \ No newline at end of file diff --git a/processed_dataset/calculation/0983.json b/processed_dataset/calculation/0983.json new file mode 100644 index 0000000000000000000000000000000000000000..da9fc8ee93c4810187bdcb405d00477de597ff53 --- /dev/null +++ b/processed_dataset/calculation/0983.json @@ -0,0 +1,11 @@ +{ + "source_file": "./raw_volume-zh/volume8/exercise6.tex", + "problem_type": "calculation", + "problem": "问题3. 如图(), 直四棱柱 $A B C D-A_1 B_1 C_1 D_1$ 中, $A D / / B C$, 且 $\\angle C B A=90^{\\circ}, B C=2, A D=6, A B= 2, A A_1=4$, 且 $C E=C_1 E, C G=D G, A F= \\frac{1}{2} F D$, 求:\n(1) $E F$ 和 $D_1 G$ 所成角的大小;\n(2) $E F$ 与平面 $A_1 C_1$ 所成角的大小;\n(3) 二面角 $E-F G-D_1$ 的平面角的大小.", + "solution": "如图(), 建立空间直角坐标系, 则 $E(2,2$, 2), $F(0,2,0), D_1(0,6,4), G(1,4,0)$.\n(1) $\\overrightarrow{E F}=\\{-2,0,-2\\}, \\overrightarrow{D_1 G}=\\{1,-2$, $-4\\}, \\cos \\alpha=\\frac{\\overrightarrow{E F} \\cdot \\overrightarrow{D_1 G}}{|\\overrightarrow{E F}| \\cdot\\left|\\overrightarrow{D_1 G}\\right|}=\\frac{-2+8}{2 \\sqrt{2} \\cdot \\sqrt{21}}= \\frac{\\sqrt{21}}{14}$\n所以 $E F$ 和 $D_1 G$ 所成的角为 $\\arccos \\frac{\\sqrt{21}}{14}$.\n(2)显然 $\\overrightarrow{p_0}=\\{0,0,-1\\}$ 是平面 $A_1 C_1$ 的一个单位法向量.\n所以 $\\cos \\beta=\\frac{\\overrightarrow{E F} \\cdot \\overrightarrow{p_0}}{|\\overrightarrow{E F}|}=\\frac{2}{2 \\sqrt{2}}=\\frac{\\sqrt{2}}{2}$, 故 $\\frac{\\pi}{2}-\\beta=\\frac{\\pi}{4}$.\n所以 $E F$ 与平面 $A_1 C_1$ 所成的角为 $\\frac{\\pi}{4}$.\n(3) 因为平面 $x O y$ 内 $F G$ 的直线方程为 $2 x-y+2=0$, 设 $P 、 Q$ 在直线 $F G$ 上, 且 $E P \\perp F G, D_1 Q \\perp F G$, 则可设 $P\\left(t_1, 2 t_1+2,0\\right), Q\\left(t_2, 2 t_2+2\\right.$, $0)\\left(t_1 、 t_2 \\in \\mathbf{R}\\right)$.\n因为 $\\left\\{\\begin{array}{l}\\overrightarrow{E P} \\cdot \\overrightarrow{F G}=0, \\\\ \\overrightarrow{D_1 Q} \\cdot \\overrightarrow{F G}=0,\\end{array}\\right.$ 所以 $\\left\\{\\begin{array}{l}\\left\\{t_1-2,2 t_1,-2\\right\\} \\cdot\\{1,2,0\\}=0, \\\\ \\left\\{t_2, 2 t_2-4,-4\\right\\} \\cdot\\{1,2,0\\}=0 .\\end{array}\\right.$\n即 $\\left\\{\\begin{array}{l}t_1-2+4 t_1=0, \\\\ t_2+4 t_2-8=0 .\\end{array}\\right.$ 解方程, 得 $\\left\\{\\begin{array}{l}t_1=\\frac{2}{5}, \\\\ t_2=\\frac{8}{5} .\\end{array}\\right.$\n所以 $\\overrightarrow{E P}=\\left\\{-\\frac{8}{5}, \\frac{4}{5},-2\\right\\}, \\overrightarrow{D_1 Q}=\\left\\{\\frac{8}{5},-\\frac{4}{5},-4\\right\\}$, 故\n$$\n\\cos \\theta=\\frac{\\overrightarrow{E P} \\cdot \\overrightarrow{D_1 Q}}{|\\overrightarrow{E P}| \\cdot\\left|\\overrightarrow{D_1 Q}\\right|}=\\frac{-\\frac{64}{25}-\\frac{16}{25}+8}{\\frac{3 \\sqrt{20}}{5} \\cdot \\frac{4 \\sqrt{30}}{5}}=\\frac{\\sqrt{6}}{6} \\text {. }\n$$\n所以二面角 $E-F G-D_1$ 的大小为 $\\arccos \\frac{\\sqrt{6}}{6}$.", + "remark": "", + "figures": [ + "./images/volume8/figures/fig-c6p3.png", + "./images/volume8/figures/fig-c6a3.png" + ] +} \ No newline at end of file diff --git a/processed_dataset/calculation/0984.json b/processed_dataset/calculation/0984.json new file mode 100644 index 0000000000000000000000000000000000000000..7738a6837596cdd5a148a517e963a23718582358 --- /dev/null +++ b/processed_dataset/calculation/0984.json @@ -0,0 +1,10 @@ +{ + "source_file": "./raw_volume-zh/volume8/exercise6.tex", + "problem_type": "calculation", + "problem": "问题4. 对于 $n$ 个向量 $\\overrightarrow{a_1}, \\overrightarrow{a_2}, \\cdots, \\overrightarrow{a_n}$, 如存在不全为零的 $n$ 个实数 $k_1, k_2, \\cdots, k_n$, 使 $k_1 \\overrightarrow{a_1}+k_2 \\overrightarrow{a_2}+\\cdots+k_n \\overrightarrow{a_n}=\\overrightarrow{0}$ 成立, 则称这 $n$ 个向量 $\\overrightarrow{a_1}, \\overrightarrow{a_2}, \\cdots, \\overrightarrow{a_n}$ 线性相关; 反之如果 $k_1 \\overrightarrow{a_1}+k_2 \\overrightarrow{a_2}+\\cdots+k_n \\overrightarrow{a_n}=\\overrightarrow{0}$ 当且仅当 $k_1=k_2=\\cdots= k_n=0$ 时成立,则称这 $n$ 个向量 $\\overrightarrow{a_1}, \\overrightarrow{a_2}, \\cdots, \\overrightarrow{a_n}$ 线性无关.\n在棱长为 1 的立方体 $A B C D-A_1 B_1 C_1 D_1$ 中, $N$ 为正方形 $A_1 B_1 C_1 D_1$ 的中心.\n(1) 判断 $\\overrightarrow{A A_1}$ 和 $\\overrightarrow{B C} ; \\overrightarrow{A_1 N}$ 和 $\\overrightarrow{C A} ; \\overrightarrow{A_1 N}$ 和 $\\overrightarrow{B_1 D_1}$ 是否线性相关?\n(2) 判断 $\\overrightarrow{A A_1} 、 \\overrightarrow{A B}$ 和 $\\overrightarrow{B C} ; \\overrightarrow{A_1 D_1} 、 \\overrightarrow{A_1 B_1}$ 和 $\\overrightarrow{A C}$ 这两组向量是否线性相关?\n(3) 说明 $\\vec{a} 、 \\vec{b}$ 线性相关和 $\\vec{a} 、 \\vec{b} 、 \\vec{c}$ 线性相关的几何意义.", + "solution": "如图(), 建立空间直角坐标系, 则 $A(0,0,0)$, $A_1(0,0,1), B(1,0,0), C(1,1,0), N\\left(\\frac{1}{2}, \\frac{1}{2}, 1\\right)$, $D(0,1,0), D_1(\\theta, 1,1), B_1(1,0,1)$.\n(1) $\\overrightarrow{A A_1}=\\{0,0,1\\}, \\overrightarrow{B C}=\\{0,1,0\\}$.\n设 $k_1 、 k_2 \\in \\mathbf{R}$, 有 $k_1 \\overrightarrow{A A_1}+k_2 \\overrightarrow{B C}=\\overrightarrow{0}$, 即 $\\left\\{0, k_2\\right.$, $\\left.k_1\\right\\}=\\overrightarrow{0}$.\n所以 $k_1=k_2=0$,所以 $\\overrightarrow{A A_1}$ 和 $\\overrightarrow{B C}$ 线性无关, $\\overrightarrow{A A_1}$\n和 $\\overrightarrow{B C}$ 不平行.\n$$\n\\overrightarrow{A_1} \\vec{N}=\\left\\{\\frac{1}{2}, \\frac{1}{2}, 0\\right\\}, \\overrightarrow{C A}=\\{-1,-1,0\\}\n$$\n设 $k_1 、 k_2 \\in \\mathbf{R}$, 有 $k_1 \\overrightarrow{A_1} \\vec{N}+k_2 \\overrightarrow{C A}=\\overrightarrow{0}$, 即 $\\left\\{\\frac{1}{2} k_1-k_2, \\frac{1}{2} k_1-k_2, 0\\right\\}=\\overrightarrow{0}$. 此时存在不全为零的实数 $k_1=2, k_2=1$ 有 $k_1 \\overrightarrow{A_1 N}+k_2 \\overrightarrow{C A}=\\overrightarrow{0}$, 所以 $\\overrightarrow{A_1 N}$ 和 $\\overrightarrow{C A}$ 线性相关, $\\overrightarrow{A_1 N}$ 和 $\\overrightarrow{C A}$ 平行.\n$$\n\\overrightarrow{A_1 N}=\\left\\{\\frac{1}{2}, \\frac{1}{2}, 0\\right\\}, \\overrightarrow{B_1 D_1}=\\{-1,1,0\\} .\n$$\n设 $k_1 、 k_2 \\in \\mathbf{R}$, 有 $k_1 \\overrightarrow{A_1 N}+k_2 \\overrightarrow{B_1 D_1}=\\overrightarrow{0}$, 即 $\\left\\{\\frac{k_1}{2}-k_2, \\frac{k_1}{2}+k_2, 0\\right\\}=\\overrightarrow{0}$.\n所以 $\\left\\{\\begin{array}{l}\\frac{k_1}{2}-k_2=0, \\\\ \\frac{k_1}{2}+k_2=0 .\\end{array}\\right.$ 即 $k_1=k_2=0$.\n所以 $\\overrightarrow{A_1 N}$ 和 $\\overrightarrow{B_1 D_1}$ 线性无关, $\\overrightarrow{A_1 N}$ 和 $\\overrightarrow{B_1 D_1}$ 不平行.\n(2) $\\overrightarrow{A A_1}=\\{0,0,1\\}, \\overrightarrow{A B}=\\{1,0,0\\}, \\overrightarrow{B C}=\\{0,1,0\\}$.\n设 $k_1 、 k_2 、 k_3 \\in \\mathbf{R}$, 有 $k_1 \\overrightarrow{A A_1}+k_2 \\overrightarrow{A B}+k_3 \\overrightarrow{B C}=\\overrightarrow{0}$, 即 $\\left\\{k_2, k_3, k_1\\right\\}=\\overrightarrow{0}$.\n所以 $k_1=k_2=k_3=0$, 所以 $\\overrightarrow{A A_1} 、 \\overrightarrow{A B} 、 \\overrightarrow{B C}$ 线性无关, $\\overrightarrow{A A_1} 、 \\overrightarrow{A B} 、 \\overrightarrow{B C}$ 不共面.\n$$\n\\overrightarrow{A_1 D_1}=\\{0,1,0\\}, \\overrightarrow{A_1 B_1}=\\{1,0,0\\}, \\overrightarrow{A C}=\\{1,1,0\\} .\n$$\n设 $k_1 、 k_2 、 k_3 \\in \\mathbf{R}$, 有 $k_1 \\overrightarrow{A_1 D_1}+k_2 \\overrightarrow{A_1 B_1}+k_3 \\overrightarrow{A C}=\\overrightarrow{0}$, 即 $\\left\\{k_2+k_3, k_1+\\right. \\left.k_3, 0\\right\\}=\\overrightarrow{0}$.\n所以 $\\left\\{\\begin{array}{l}k_2+k_3=0, \\\\ k_1+k_3=0 .\\end{array}\\right.$ 可以取 $k_1=k_2=1, k_3=-1$.\n所以存在不全为零的系数 $k_1=k_2=1, k_3=-1$ 有 $k_1 \\overrightarrow{A_1 D_1}+k_2 \\overrightarrow{A_1 B_1}+ k_3 \\overrightarrow{A C}=\\overrightarrow{0}$.\n所以 $\\overrightarrow{A_1 D_1} 、 \\overrightarrow{A_1 B_1} 、 \\overrightarrow{A C}$ 线性相关, $\\overrightarrow{A_1 D_1} 、 \\overrightarrow{A_1 B_1} 、 \\overrightarrow{A C}$ 可以共面.\n(3) $\\vec{a} 、 \\vec{b}$ 线性相关当且仅当 $\\vec{a}$ 和 $\\vec{b}$ 平行, $\\vec{a} 、 \\vec{b} 、 \\vec{c}$ 线性相关当且仅当 $\\vec{a} 、 \\vec{b} 、 \\vec{c}$ 共面.", + "remark": "", + "figures": [ + "./images/volume8/figures/fig-c6a4.png" + ] +} \ No newline at end of file diff --git a/processed_dataset/calculation/0985.json b/processed_dataset/calculation/0985.json new file mode 100644 index 0000000000000000000000000000000000000000..5200e9d0b367463f4610fb876c6e2529598ff46a --- /dev/null +++ b/processed_dataset/calculation/0985.json @@ -0,0 +1,8 @@ +{ + "source_file": "./raw_volume-zh/volume8/exercise7.tex", + "problem_type": "calculation", + "problem": "问题1. 设 $z=\\cos \\frac{\\pi}{4}+\\mathrm{i} \\sin \\frac{\\pi}{4}$, 若 $z^n=\\bar{z}\\left(n \\in \\mathbf{N}^*\\right)$, 则 $n$ 的最小值是 ( ).\n(A) 7\n(B) 8\n(C) 9\n(D) 10", + "solution": "A.\n由 $z^n=\\bar{z}$, 知\n$$\nz^{n+1}==z \\bar{z}=1, z^{n+1}=\\cos \\frac{(n+1) \\pi}{4}+\\mathrm{i} \\sin \\frac{(n+1) \\pi}{4}=1,\n$$\n所以得\n$$\n\\begin{aligned}\n& \\cos \\frac{(n+1) \\pi}{4}=1, \\text { 且 } i \\sin \\frac{(n+1) \\pi}{4}=0, \\\\\n& \\frac{n+1}{4} \\pi=2 k \\pi \\text { 且 } \\frac{n+1}{4} \\pi=k \\pi,(k \\in \\mathbf{Z})\n\\end{aligned}\n$$\n于是 $n=8 k-1$, 即 $n$ 的最小值是 7 .", + "remark": "", + "figures": [] +} \ No newline at end of file diff --git a/processed_dataset/calculation/0986.json b/processed_dataset/calculation/0986.json new file mode 100644 index 0000000000000000000000000000000000000000..7f24d1eb880acdb338f75b0e93c6ab92f342ec39 --- /dev/null +++ b/processed_dataset/calculation/0986.json @@ -0,0 +1,8 @@ +{ + "source_file": "./raw_volume-zh/volume8/exercise7.tex", + "problem_type": "calculation", + "problem": "问题2. 设 $\\omega=\\cos \\frac{\\pi}{5}+i \\sin \\frac{\\pi}{5}$, 则以 $\\omega 、 \\omega^3 、 \\omega^7 、 \\omega^9$ 为根的方程是 $(\\quad)$.\n(A) $x^4+x^3+x^2+x+1=0$\n(B) $x^4-x^3+x^2-x+1=0$\n(C) $x^4-x^3-x^2+x+1=0$\n(D) $x^4+x^3+x^2-x-1=0$", + "solution": "B.\n由 $\\omega=\\cos \\frac{\\pi}{5}+\\mathrm{i} \\sin \\frac{\\pi}{5}$ 知, $\\omega, \\omega^2, \\omega^3, \\cdots, \\omega^{10}$ 是 1 的 10 个 10 次方根.\n于是\n$$\n(x-\\omega)\\left(x-\\omega^2\\right)\\left(x-\\omega^3\\right) \\cdots\\left(x-\\omega^{10}\\right)=x^{10}-1 . \\label{eq1}\n$$\n因为 $\\omega^2 、 \\omega^4 、 \\omega^6 、 \\omega^8 、 \\omega^{10}$ 是 1 的 5 个 5 次方根, 所以\n$$\n\\left(x-\\omega^2\\right)\\left(x-\\omega^4\\right)\\left(x-\\omega^6\\right)\\left(x-\\omega^8\\right)\\left(x-\\omega^{10}\\right)=x^5-1 . \\label{eq2}\n$$\n由式\\ref{eq1} $\\div$ 式\\ref{eq2}, 便有\n$$\n(x-\\omega)\\left(x-\\omega^3\\right)\\left(x-\\omega^5\\right)\\left(x-\\omega^7\\right)\\left(x-\\omega^9\\right)=x^5+1 . \\label{eq3}\n$$\n对\\ref{eq3}式两边同除以 $x-\\omega^5$, 也就是 $x+1$, 得 $(x-\\omega)\\left(x-\\omega^3\\right)\\left(x-\\omega^5\\right)\\left(x-\\omega^7\\right)\\left(x-\\omega^9\\right)=x^4-x^3+x^2-x+1$.", + "remark": "", + "figures": [] +} \ No newline at end of file diff --git a/processed_dataset/calculation/0987.json b/processed_dataset/calculation/0987.json new file mode 100644 index 0000000000000000000000000000000000000000..a8a9081cf1a687b4d378c695bf6b0baeca1ec6a1 --- /dev/null +++ b/processed_dataset/calculation/0987.json @@ -0,0 +1,8 @@ +{ + "source_file": "./raw_volume-zh/volume8/exercise7.tex", + "problem_type": "calculation", + "problem": "问题4. 方程 $x^{10}+(13 x-1)^{10}=0$ 的 10 个复数根分别为 $r_1, \\overline{r_1}, r_2, \\overline{r_2}, r_3, \\overline{r_3}$, $r_4, \\overline{r_4}, r_5, \\overline{r_5}$. 求代数式 $\\frac{1}{r_1 \\overline{r_1}}+\\frac{1}{r_2 \\overline{r_2}}+\\cdots+\\frac{1}{r_5 \\overline{r_5}}$ 的值.", + "solution": "设 $\\varepsilon=\\cos \\frac{\\pi}{10}+\\mathrm{i} \\sin \\frac{\\pi}{10}$, 则 $\\varepsilon^{10}=-1$.\n由方程 $(13 x-1)^{10}=-x^{10}$, 我们可设 $13 r_k-1=r_k \\cdot \\varepsilon^{2 k-1}, k=1,2, \\cdots, 5$. 于是 $\\frac{1}{r_k}=13-\\varepsilon^{2 k-1}$. 所以, 有\n$$\n\\begin{aligned}\n\\sum_{k=1}^5 \\frac{1}{r_k \\overline{r_k}} & =\\sum_{k=1}^5\\left(13-\\varepsilon^{2 k-1}\\right)\\left(13-\\bar{\\varepsilon}^{2 k-1}\\right)=\\sum_{k=1}^5\\left[170-13\\left(\\varepsilon^{2 k-1}+\\bar{\\varepsilon}^{2 k-1}\\right)\\right] \\\\\n& =850-13 \\sum_{k=1}^5\\left(\\varepsilon^{2 k-1}+\\bar{\\varepsilon}^{2 k-1}\\right) \\\\\n& =850-26\\left(\\cos \\frac{\\pi}{10}+\\cos \\frac{3 \\pi}{10}+\\cos \\frac{5 \\pi}{10}+\\cos \\frac{7 \\pi}{10}+\\cos \\frac{9 \\pi}{10}\\right)=850 .\n\\end{aligned}\n$$\n于是, 所求代数式的值为 850 .", + "remark": "", + "figures": [] +} \ No newline at end of file diff --git a/processed_dataset/calculation/0988.json b/processed_dataset/calculation/0988.json new file mode 100644 index 0000000000000000000000000000000000000000..dbfc825e1d437e533c59f266b068cdf9d9984331 --- /dev/null +++ b/processed_dataset/calculation/0988.json @@ -0,0 +1,8 @@ +{ + "source_file": "./raw_volume-zh/volume8/exercise7.tex", + "problem_type": "calculation", + "problem": "问题5. 设 $f(x)=x^4+x^3+x^2+x+1$, 求 $f\\left(x^5\\right)$ 被 $f(x)$ 除得的余数.", + "solution": "设 $f\\left(x^5\\right)=f(x) q(x)+r(x)$, 这里 $r(x)=0$ 或 $\\operatorname{deg} r \\leqslant 3$.\n设 $\\zeta \\neq 1$ 是一个 5 次单位根, 则 $r(\\zeta)=r\\left(\\zeta^2\\right)=r\\left(\\zeta^3\\right)=r\\left(\\zeta^4\\right)=5$, 而 $\\operatorname{deg} r \\leqslant 3$, 故必须 $r(x)=5$, 即余式是常数 5 .", + "remark": "", + "figures": [] +} \ No newline at end of file diff --git a/processed_dataset/calculation/0989.json b/processed_dataset/calculation/0989.json new file mode 100644 index 0000000000000000000000000000000000000000..0bf94b0ada95789b1023f20bd1d19e22812936d5 --- /dev/null +++ b/processed_dataset/calculation/0989.json @@ -0,0 +1,8 @@ +{ + "source_file": "./raw_volume-zh/volume8/exercise8.tex", + "problem_type": "calculation", + "problem": "问题1. 已知复数 $Z_1 、 Z_2$ 满足 $\\left|Z_1\\right|=2,\\left|Z_2\\right|=3$. 若它们所对应向量的夹角为 $60^{\\circ}$, 则 $\\left|\\frac{Z_1+Z_2}{Z_1-Z_2}\\right|=$", + "solution": "由余弦定理得\n$$\n\\begin{gathered}\n\\left|Z_1+Z_2\\right|=\\sqrt{\\left|Z_1\\right|^2+\\left|Z_2\\right|^2-2\\left|Z_1\\right|\\left|Z_2\\right| \\cos 120^{\\circ}}=\\sqrt{19}, \\\\\n\\left|Z_1-Z_2\\right|=\\sqrt{\\left|Z_1\\right|^2+\\left|Z_2\\right|^2-2\\left|Z_1\\right|\\left|Z_2\\right| \\cos 60^{\\circ}}=\\sqrt{7},\n\\end{gathered}\n$$\n所以\n$$\n\\frac{\\left|Z_1+Z_2\\right|}{\\left|Z_1-Z_2\\right|}=\\frac{\\sqrt{19}}{\\sqrt{7}}=\\frac{\\sqrt{133}}{7} \\text {. }\n$$", + "remark": "", + "figures": [] +} \ No newline at end of file diff --git a/processed_dataset/calculation/0990.json b/processed_dataset/calculation/0990.json new file mode 100644 index 0000000000000000000000000000000000000000..59d333baa16d68e1d3d66fd39d6969c4b22a2192 --- /dev/null +++ b/processed_dataset/calculation/0990.json @@ -0,0 +1,8 @@ +{ + "source_file": "./raw_volume-zh/volume8/exercise9.tex", + "problem_type": "calculation", + "problem": "问题3. 设 $\\triangle A B C$ 内切圆圆心为原点, 半径为 $r$, 切 $B C 、 C A 、 A B$ 于 $D 、 E 、 F, D$ 、 $E 、 F$ 对应的复数分别为 $t_1 、 t_2 、 t_3$, 试用 $r 、 t_1 、 t_2 、 t_3$ 表示 $\\triangle A B C$ 的外接圆半径 $R$.", + "solution": "由例 6 可知 $A 、 B 、 C$ 所对应的复数分别是 $\\frac{2 t_2 t_3}{t_2+t_3}, \\frac{2 t_1 t_3}{t_1+t_3}, \\frac{2 t_1 t_2}{t_1+t_2}$, 由三角形面积公式, 知\n$$\nA B \\cdot B C \\cdot C A=4 R \\cdot S_{\\triangle A B C}, D E \\cdot E F \\cdot F D=4 r \\cdot S_{\\triangle D E F},\n$$\n于是\n$$\n\\frac{S_{\\triangle D E F}}{S_{\\triangle A B C}}=\\frac{R}{r} \\cdot \\frac{D E \\cdot E F \\cdot F D}{A B \\cdot B C \\cdot C A},\n$$\n又\n$$\n\\begin{aligned}\nA B & =\\left|\\frac{2 t_2 t_3}{t_2+t_3}-\\frac{2 t_1 t_3}{t_1+t_3}\\right|=2\\left|t_3\\right|^2\\left|\\frac{t_2-t_1}{\\left(t_1+t_3\\right)\\left(t_2+t_3\\right)}\\right| \\\\\n& =\\frac{2 r^2}{\\left|\\left(t_1+t_3\\right)\\left(t_2+t_3\\right)\\right|} \\cdot D E,\n\\end{aligned}\n$$\n于是 $\\frac{D E}{A B}=\\frac{\\left|\\left(t_1+t_3\\right)\\left(t_2+t_3\\right)\\right|}{2 r^2}$, 同理还有另外两个式子, 故\n$$\n\\frac{S_{\\triangle D E F}}{S_{\\triangle A B C}}=\\frac{R}{8 r^7}\\left|\\left(t_1+t_2\\right)\\left(t_2+t_3\\right)\\left(t_3+t_1\\right)\\right|^2 .\n$$\n$$\n\\begin{aligned}\n\\text { 又 } S_{\\triangle D E F} & =S_{\\triangle D I E}+S_{\\triangle E I F}+S_{\\triangle F I D}=\\frac{1}{2} r^2(\\sin A+\\sin B+\\sin C) \\\\\n& =\\frac{r^2(A B+B C+C A)}{4 R}=\\frac{r}{2 R} \\cdot S_{\\triangle A B C},\n\\end{aligned}\n$$\n故 $\\frac{R}{8 r^7}\\left|\\left(t_1+t_2\\right)\\left(t_2+t_3\\right)\\left(t_3+t_1\\right)\\right|^2=\\frac{r}{2 R}$. 于是 $R=\\frac{2 r^4}{\\left|\\left(t_1+t_2\\right)\\left(t_2+t_3\\right)\\left(t_3+t_1\\right)\\right|}$.", + "remark": "", + "figures": [] +} \ No newline at end of file diff --git a/processed_dataset/calculation/0991.json b/processed_dataset/calculation/0991.json new file mode 100644 index 0000000000000000000000000000000000000000..d7ef4050a6987287959bfd96903d82011fe3ac78 --- /dev/null +++ b/processed_dataset/calculation/0991.json @@ -0,0 +1,10 @@ +{ + "source_file": "./raw_volume-zh/volume8/exercise9.tex", + "problem_type": "calculation", + "problem": "问题4. 设 $D$ 是锐角 $\\triangle A B C$ 内部一点, 使 $\\angle A D B=\\angle A C B+90^{\\circ}$, 且 $A C \\cdot B D= A D \\cdot B C$, 求 $\\frac{A B \\cdot C D}{A C \\cdot B D}$ 的值.", + "solution": "如图(), 以 $C$ 为原点建立复平面, 设 $\\angle A C B= \\theta, C A=1$,\n$$\n\\begin{aligned}\n\\overrightarrow{D B} & =\\frac{|B D|}{|D A|} \\cdot \\overrightarrow{D A} \\cdot \\mathrm{e}^{\\mathrm{i}\\left(\\theta+90^{\\circ}\\right)} \\\\\n& =\\frac{|B C|}{|A C|} \\cdot\\left(1-z_D\\right) \\cdot \\mathrm{i} \\cdot \\mathrm{e}^{\\mathrm{i} \\theta} \\\\\n& =\\overrightarrow{C B} \\cdot\\left(1-z_D\\right) \\cdot \\mathrm{i} \\\\\n& =z_B\\left(1-z_D\\right) \\cdot \\mathrm{i} .\n\\end{aligned}\n$$\n而 $\\overrightarrow{D B}=z_B-z_D$, 所以 $z_D=\\frac{z_B(i-1)}{z_B i-1}$, 从而\n$$\n\\frac{|A B| \\cdot \\mid C D}{|A C| \\cdot|B D|}=\\frac{\\left|z_B-1\\right| \\frac{z_B \\mathrm{i}-z_B}{z_B \\mathrm{i}-1} \\mid}{\\left|z_B-\\frac{z_B \\mathrm{i}-z_B}{z_B \\mathrm{i}-1}\\right|}=\\frac{\\left|z_B\\right| \\cdot\\left|z_B-1\\right| \\cdot|\\mathrm{i}-1|}{\\left|z_B\\right| \\cdot\\left|z_B-1\\right|}=\\sqrt{2} .\n$$", + "remark": "注:这题可用纯几何方法做, 但需要构造一个适当的角 (类似于旋转, 并且此题可推广为: 设 $D$ 是 $\\triangle A B C$ 内一点, 使得 $\\angle B D C=\\angle B A C+\\alpha$, $\\angle C D A=\\angle C B A+\\beta, \\angle A D B=\\angle A C B+\\gamma$, 则 $(B C \\cdot A D):(C A \\cdot B D): (A B \\cdot C D)=\\sin \\alpha: \\sin \\beta: \\sin \\gamma$.", + "figures": [ + "./images/volume8/figures/fig-c9a4.png" + ] +} \ No newline at end of file diff --git a/processed_dataset/calculation/0992.json b/processed_dataset/calculation/0992.json new file mode 100644 index 0000000000000000000000000000000000000000..010587d5c81c1e8ddb34eb5aa5293f7d2f2c6bbc --- /dev/null +++ b/processed_dataset/calculation/0992.json @@ -0,0 +1,8 @@ +{ + "source_file": "./raw_volume-zh/volume8/exercise9.tex", + "problem_type": "calculation", + "problem": "问题9. 已知 $f(x)=\\frac{\\sqrt{3} x-1}{x+\\sqrt{3}}$, 求 $g(x)=\\underbrace{f\\{f \\cdots[f}_{1986 \\uparrow f}(x)]\\}$.", + "solution": "解:$$\nf(x)=\\frac{\\sqrt{3} x-1}{x+\\sqrt{3}}=\\frac{\\frac{\\sqrt{3}}{2} x-\\frac{1}{2}}{\\frac{1}{2} x+\\frac{\\sqrt{3}}{2}},\n$$\n故对应的复数为 $z=\\frac{\\sqrt{3}}{2}-\\frac{1}{2} \\mathrm{i}$.\n所以 $g(x)=\\underbrace{f\\{f \\cdots[f}_{1986}(x)]\\}$ 对应的复数为\n$$\n\\begin{aligned}\nz^{1986} & =\\left(\\frac{\\sqrt{3}}{2}-\\frac{1}{2} \\mathrm{i}\\right)^{1986} \\\\\n& =\\left[\\cos \\left(-\\frac{\\pi}{6}\\right)+\\mathrm{i} \\sin \\left(-\\frac{\\pi}{6}\\right)\\right]^{1986} \\\\\n& =\\cos \\left(-\\frac{1986 \\pi}{6}\\right)+i \\sin \\left(-\\frac{1986 \\pi}{6}\\right) \\\\\n& =-1 .\n\\end{aligned}\n$$\n因为复数 -1 对应的 $H$ 型函数为 $\\frac{-x-0}{0 \\cdot x-1}=x$, 所以\n$$\ng(x)=\\underbrace{f\\{f \\cdots[f}_{1986}(x)]\\}=x .\n$$", + "remark": "", + "figures": [] +} \ No newline at end of file diff --git a/processed_dataset/calculation/0993.json b/processed_dataset/calculation/0993.json new file mode 100644 index 0000000000000000000000000000000000000000..b8b425c3ec3aaa6127f55866a4a0d0eb89b395ab --- /dev/null +++ b/processed_dataset/calculation/0993.json @@ -0,0 +1,8 @@ +{ + "source_file": "./raw_volume-zh/volume8/exercise9.tex", + "problem_type": "calculation", + "problem": "问题10. 已知 $f(x)=\\frac{x \\cos \\frac{\\pi}{96}+\\sin \\frac{\\pi}{96}}{-x \\sin \\frac{\\pi}{96}+\\cos \\frac{\\pi}{96}}, g(x)=\\frac{x \\cos \\frac{\\pi}{48}+\\sin \\frac{\\pi}{48}}{-x \\sin \\frac{\\pi}{48}+\\cos \\frac{\\pi}{48}}$, 求\n$$\nH(x)=\\underbrace{f\\{g \\cdots f[g(x)]\\} .}_{1000 \\uparrow f, 1000 \\uparrow g}\n$$", + "solution": "因为 $f(x) 、 g(x)$ 对应的复数分别为\n$$\n\\cos \\frac{\\pi}{96}+i \\sin \\frac{\\pi}{96}, \\cos \\frac{\\pi}{48}+i \\sin \\frac{\\pi}{48} .\n$$\n所以 $f[g(x)]$ 所对应的复数为\n$$\n\\left(\\cos \\frac{\\pi}{96}+i \\sin \\frac{\\pi}{96}\\right)\\left(\\cos \\frac{\\pi}{48}+i \\sin \\frac{\\pi}{48}\\right)=\\cos \\frac{\\pi}{32}+i \\sin \\frac{\\pi}{32} .\n$$\n所以 $H(x)$ 对应的复数为\n$$\n\\left(\\cos \\frac{\\pi}{32}+\\mathrm{i} \\sin \\frac{\\pi}{32}\\right)^{1000}=\\cos \\frac{1000 \\pi}{32}+\\mathrm{i} \\sin \\frac{1000 \\pi}{32}=-\\frac{\\sqrt{2}}{2}-\\frac{\\sqrt{2}}{2} \\mathrm{i} .\n$$\n所以 $H(x)=\\frac{-\\frac{\\sqrt{2}}{2} x-\\frac{\\sqrt{2}}{2}}{\\frac{\\sqrt{2}}{2} x-\\frac{\\sqrt{2}}{2}}=\\frac{-x-1}{x-1}=-\\frac{x+1}{x-1}$.", + "remark": "", + "figures": [] +} \ No newline at end of file diff --git a/processed_dataset/calculation/0994.json b/processed_dataset/calculation/0994.json new file mode 100644 index 0000000000000000000000000000000000000000..1509d5af590655061f57ba40326bcf85b1386ba2 --- /dev/null +++ b/processed_dataset/calculation/0994.json @@ -0,0 +1,8 @@ +{ + "source_file": "./raw_volume-zh/volume9/exercise1.tex", + "problem_type": "calculation", + "problem": "问题4. 设 $E 、 F$ 分别是 $\\triangle A B C$ 中以 $A$ 为端点的射线 $A C 、 A B$ 上的点, 则\n$$\n|A B-A C|+|A E-A F| \\geqslant|B E-C F|,\n$$\n当且仅当 $A B=A C$ 且 $A E=A F$ 时等号成立.", + "solution": "这里仅就 $E 、 F$ 分别在边 $A C 、 A B$ 上的情况给出提示, 在其他情况下类似可得.\n不妨设 $A C \\geqslant A B$, 在 $A C$ 上取点 $D$ 使得 $A D=A B$, 又在 $A B$ (或 $A B$ 的延长线) 上取点 $G$, 使得 $A G=A E$, 则 $|A B-A C|+|A E-A F|=|C D|+ |F G| \\geqslant|C G-D G|+|C F-C G| \\geqslant|C G-D G+C F-C G|=|C F-D G|= |B E-C F|$.", + "remark": "", + "figures": [] +} \ No newline at end of file diff --git a/processed_dataset/calculation/0995.json b/processed_dataset/calculation/0995.json new file mode 100644 index 0000000000000000000000000000000000000000..38a890f668246fbe3f196e4a646e2ae63b00f44e --- /dev/null +++ b/processed_dataset/calculation/0995.json @@ -0,0 +1,8 @@ +{ + "source_file": "./raw_volume-zh/volume9/exercise1.tex", + "problem_type": "calculation", + "problem": "问题5. 在六边形 $A_1 A_2 A_3 A_4 A_5 A_6$ 内存在一点 $O$, 使得 $\\angle A_i O A_{i+1}=\\frac{\\pi}{3} ( i=1$, $2,3,4,5,6$ ) (约定 $A_7=A_1$ ). 如果 $O A_1>O A_3>O A_5, O A_2>O A_4> O A_6$, 则\n$$\nA_1 A_2+A_3 A_4+A_5 A_6P B_3> P B_5, P B_6

\\sqrt{\\frac{n}{3}}$.\n若 $m \\leqslant n-2$, 则 $m^2+m+n \\leqslant(n-2)(n-1)+np$, 这与 $m$ 是使 $m^2+m+n$ 为合数的最小正整数矛盾.\n(2) 若 $m \\leqslant p-1$, 则\n$$\n(p-1-m)^2+(p-1-m)+n=p^2-2 p(m+1)+m^2+m+n\n$$\n被 $p$ 整除, 且 $(p-1-m)^2+(p-1-m)+n$ 大于 $p$, 故为合数.\n根据 $m$ 的最小性可知 $p-1-m \\geqslant m$, 所以\n$$\n2 m+1 \\leqslant p \\leqslant \\sqrt{m^2+m+n},\n$$\n平方整理得 $3 m^2+3 m+1 \\leqslant n$, 但 $m>\\sqrt{\\frac{n}{3}}$, 故 $3 m^2+3 m+1>3 m^2>n$, 矛盾!\n所以对所有整数 $k, 0 \\leqslant k \\leqslant n-2, k^2+k+n$ 是素数.", + "remark": "注:在反证法假设下, 找到最小的 $m$. 和此时 $m^2+m+n$ 的最小素因子 $p$, 通过调整得到更小的,因而矛盾!", + "figures": [] +} \ No newline at end of file diff --git a/processed_dataset/proof/0839.json b/processed_dataset/proof/0839.json new file mode 100644 index 0000000000000000000000000000000000000000..09ea5b54bf4578661f22861cf12dbc5b8eca55a1 --- /dev/null +++ b/processed_dataset/proof/0839.json @@ -0,0 +1,8 @@ +{ + "source_file": "./raw_volume-zh/volume14/exercise7.tex", + "problem_type": "proof", + "problem": "问题2. 若整数 $a_1, a_2, \\cdots, a_n\\left(n \\in \\mathbf{N}^*\\right)$ 满足 $a_1 a_2 \\cdots a_n=n$ 且 $a_1+a_2+\\cdots+a_n=0$, 证明: $4 \\mid n$.", + "solution": "先证 $n$ 为偶数.\n用反证法.\n假设 $n$ 为奇数,则由 $a_1 a_2 \\cdots a_n=n$ 知 $a_1, a_2, \\cdots, a_n$ 均为奇数,故\n$$\na_1+a_2+\\cdots+a_n \\equiv n \\equiv 1(\\bmod 2),\n$$\n与 $a_1+a_2+\\cdots+a_n=0$ 矛盾.\n故 $n$ 为偶数.\n再证 $4 \\mid n$. 仍用反证法.\n如若不然, 则 $a_1, a_2, \\cdots, a_n$ 中有且仅一个偶数, 故\n$$\na_1+a_2+\\cdots+a_n \\equiv n-1 \\equiv 1(\\bmod 2),\n$$\n仍与 $a_1+a_2+\\cdots+a_n=0$ 矛盾.\n故 $4 \\mid n$.", + "remark": "", + "figures": [] +} \ No newline at end of file diff --git a/processed_dataset/proof/0840.json b/processed_dataset/proof/0840.json new file mode 100644 index 0000000000000000000000000000000000000000..3fdb8ce8aa4f150215c98588e23c67c5bd25eed8 --- /dev/null +++ b/processed_dataset/proof/0840.json @@ -0,0 +1,8 @@ +{ + "source_file": "./raw_volume-zh/volume14/exercise7.tex", + "problem_type": "proof", + "problem": "问题3. 代数式\n$$\nr v z-r w y-s u z+s w x+t u y-t v x . \\label{eq1}\n$$\n中, $r, s, t, u, v, w, x, y, z$ 可以分别取 +1 或 -1 .\n(1)证明:代数式的值都是偶数;\n(2) 求这个代数式所能取到的最大值.", + "solution": "(1) \\ref{eq1} 式中共有 6 项, 每项的值都是奇数 $(+1$ 或 -1$)$, 所以它们的代数和为偶数.\n(2) 显然, \\ref{eq1} 式的值 $\\leqslant 6$, 但它取不到 6 这个值, 事实上,在\n$rvz, - rwy, - suz, swx, tuy, - tvx$\n这六项中, 至少有一项是一 1 , 要证明这一点, 将上面这 6 项相乘, 积是\n$-(\\text { rstuvwxyz })^2=-1$.\n所以六项中, 至少一项是 -1 , 这样, 六项和至多是\n$$\n5-1=4 .\n$$\n在 $u, x, y$ 为 -1 , 其他字母为 1 时, \\ref{eq1} 式的值是 4, 所以 式\\ref{eq1} 的最大值为 4 .", + "remark": "", + "figures": [] +} \ No newline at end of file diff --git a/processed_dataset/proof/0841.json b/processed_dataset/proof/0841.json new file mode 100644 index 0000000000000000000000000000000000000000..4da30564e978da310fcc44780f4fe4c3b49f29ef --- /dev/null +++ b/processed_dataset/proof/0841.json @@ -0,0 +1,8 @@ +{ + "source_file": "./raw_volume-zh/volume14/exercise7.tex", + "problem_type": "proof", + "problem": "问题7. 数列 $\\left\\{a_n\\right\\}$ 满足 $a_1=2, a_{n+1}=\\left[\\frac{3 a_n}{2}\\right], n \\in \\mathbf{N}^*$ (这里 $[x]$ 表示不超过 $x$ 的最大整数). 证明: 数列 $\\left\\{a_n\\right\\}$ 中有无穷多项为奇数, 也有无穷多项为偶数.", + "solution": "由已知得: 数列 $\\left\\{a_n\\right\\}$ 中每项都是大于 1 的正整数.\n下用反证法证明结论:\n先假设 $\\left\\{a_n\\right\\}$ 中只有有限项为奇数, 则其中必有最后一项, 不妨设为 $a_m$. 于是对任意 $n \\in \\mathbf{N}^*$, 均有 $a_{m+n}$ 是偶数.\n设 $a_{m+1}=2^p \\cdot q$, 其中 $p \\in \\mathbf{N}^*, q$ 为奇数,则由递推式可知\n$$\n\\begin{gathered}\na_{m+2}=\\left[\\frac{3 a_{m+1}}{2}\\right]=3 \\cdot 2^{p-1} \\cdot q, \\\\\na_{m+3}=\\left[\\frac{3 a_{m+2}}{2}\\right]=3^2 \\cdot 2^{p-2} \\cdot q, \\\\\n\\cdots \\cdots \\\\\na_{m+p+1}=\\left[\\frac{3 a_{m+p}}{2}\\right]=3^p \\cdot q .\n\\end{gathered}\n$$\n这表明 $a_{m+p+1}$ 为奇数,与假设矛盾.\n再假设 $\\left\\{a_n\\right\\}$ 中只有有限项为偶数, 则其中必有最后一项, 不妨设为 $a_l$. 于是对任意 $n \\in \\mathbf{N}^*$, 均有 $a_{l+n}$ 是大于 1 的奇数.\n可设 $a_{l+1}-1=2^p \\cdot q$, 其中 $p \\in \\mathbf{N}^*, q$ 为奇数.\n由递推式可知\n$$\n\\begin{gathered}\na_{l+2}=\\left[\\frac{3 a_{l+1}}{2}\\right]=\\left[3 \\cdot 2^{p-1} \\cdot q+\\frac{3}{2}\\right]=3 \\cdot 2^{p-1} \\cdot q+1, \\\\\na_{l+3}=\\left[\\frac{3 a_{l+2}}{2}\\right]=3^2 \\cdot 2^{p-2} \\cdot q+1, \\\\\n\\cdots \\cdots \\\\\na_{l+p+1}=\\left[\\frac{3 a_{l+p}}{2}\\right]=3^p \\cdot q+1 .\n\\end{gathered}\n$$\n这表明 $a_{l+p+1}$ 为偶数, 与假设矛盾.\n综上所述, 命题成立.", + "remark": "", + "figures": [] +} \ No newline at end of file diff --git a/processed_dataset/proof/0842.json b/processed_dataset/proof/0842.json new file mode 100644 index 0000000000000000000000000000000000000000..c8a48ebc1098c0b3f3fd4838e9d190b5a7884eac --- /dev/null +++ b/processed_dataset/proof/0842.json @@ -0,0 +1,8 @@ +{ + "source_file": "./raw_volume-zh/volume14/exercise7.tex", + "problem_type": "proof", + "problem": "问题8. 设 $n$ 为正整数.\n证明: 若 $n$ 的所有正因子之和是 2 的整数次幂, 则这些正因子的个数也是 2 的整数次幂.", + "solution": "设 $n=p_1^{\\alpha_1} p_2^\\alpha \\cdots p_k^{\\alpha_k}$ 为 $n$ 的素因数分解,则 $n$ 的所有正因子之和可表示为 $\\left(1+p_1+\\cdots+p_1^{\\alpha_1}\\right)\\left(1+p_2+\\cdots+p_2^{\\alpha_2}\\right) \\cdots\\left(1+p_k+\\cdots+p_{k^k}^{\\alpha_k}\\right)$.\n若它是 2 的幂, 则每个因子 $f_i=1+p_i+\\cdots+p_i^{\\alpha_i}(i=1,2, \\cdots, k)$ 都是 2 的幂, 从而所有的 $p_i, \\alpha_i$ 均为奇数.\n若有某个 $\\alpha_i>1$, 则\n$$\nf_i=\\left(1+p_i\\right)\\left(1+p_i^2+p_i^4+\\cdots+p_i^{\\alpha_i-1}\\right),\n$$\n后一个因式必须为偶数, 从而 $\\frac{\\alpha_i-1}{2}$ 为奇数, 于是进一步可得: $1+p_i^2$ 也是 $f_i$ 的因子.\n由于 $1+p_i$ 与 $1+p_i^2$ 均为 2 的幕, 所以 $\\left(1+p_i\\right) \\mid\\left(1+p_i^2\\right)$, 但\n$$\n1+p_i^2=\\left(1+p_i\\right)\\left(p_i-1\\right)+2,\n$$\n故 $\\left(1+p_i\\right) \\mid 2$,矛盾.\n因此必有 $\\alpha_i=1(i=1,2, \\cdots, k)$.\n从而 $n$ 的正因子个数是 2 的幕.", + "remark": "", + "figures": [] +} \ No newline at end of file diff --git a/processed_dataset/proof/0843.json b/processed_dataset/proof/0843.json new file mode 100644 index 0000000000000000000000000000000000000000..29fe66245a8edf27804a25a4df4aef129f827bdd --- /dev/null +++ b/processed_dataset/proof/0843.json @@ -0,0 +1,8 @@ +{ + "source_file": "./raw_volume-zh/volume14/exercise7.tex", + "problem_type": "proof", + "problem": "问题9. 若干个球放在 $2 n+1$ 个袋中, 如果任意取走一个袋, 总可以把剩下的 $2 n$ 个袋分成两组,每组 $n$ 个袋, 并且这两组的球的个数相等.\n证明: 每个袋中的球的个数相等.", + "solution": "用数 $a_1, a_2, \\cdots, a_{2 n+1}$ 分别表示这 $2 n+1$ 个袋中的球的个数.\n显然, $a_1, a_2, \\cdots, a_{2 n+1}$ 是非负整数,不妨设 $a_1 \\leqslant a_2 \\leqslant \\cdots \\leqslant a_{2 n+1}$. 于是问题转化为: 有 $2 n+1$ 个非负整数,如果从中任意取走一个数,剩下的 $2 n$ 个数可以分成两组,每组 $n$ 个,和相等, 证明这 $2 n+1$ 个数全相等.\n令 $A=a_1+a_2+\\cdots+a_{2 n+1}$, 则对每个 $i(1 \\leqslant i \\leqslant 2 n+1), A-a_i$ 都是偶数 (否则剩下的数不能分成和数相等的两部分). 从而 $a_i$ 与 $A$ 有相同的奇偶性.\n$a_1, a_2, \\cdots, a_{2 n+1}$ 也具有相同的奇偶性.\n易知把 $a_1, a_2, \\cdots, a_{2 n+1}$ 中的每一个都减去 $a_1$ 后所得到的 $2 n+-1$ 个数\n$$\n0, a_2-a_1, a_3-a_1, \\cdots, a_{2 n+1}-a_1\n$$\n也满足题设性质 (即从中任意取走一数,剩下的能分成和数相等的两部分). 因为 $a_i-a_1(i=2,3, \\cdots, 2 n+1)$ 都是偶数, 从而\n$$\n0, \\frac{a_2-a_1}{2}, \\frac{a_3-a_1}{2}, \\cdots, \\frac{a_{2 n+1}-a_1}{2}\n$$\n这 $2 n+1$ 个数也满足题意, 故也都是偶数.\n把它们再都除以 2 , 这个过程不可能永远继续下去,除非\n$$\na_1=a_2=\\cdots=a_{2 n+1},\n$$\n所以, 每个袋中的球数相等.", + "remark": "", + "figures": [] +} \ No newline at end of file diff --git a/processed_dataset/proof/0844.json b/processed_dataset/proof/0844.json new file mode 100644 index 0000000000000000000000000000000000000000..66baa1a50d4e523d3dd60d8316179d3c8b1c21d2 --- /dev/null +++ b/processed_dataset/proof/0844.json @@ -0,0 +1,8 @@ +{ + "source_file": "./raw_volume-zh/volume14/exercise8.tex", + "problem_type": "proof", + "problem": "问题1. 已知 $\\triangle P_1 P_2 P_3$ 及三角形内任意一点 $P$, 直线 $P_1 P, P_2 P, P_3 P$ 分别交对边于 $Q_1, Q_2, Q_3$. 求证: 在 $\\frac{P_1 P}{P Q_1}, \\frac{P_2 P}{P Q_2}, \\frac{P_3 P}{P Q_3}$ 这三个比值中, 至少有一个不大于 2 ,也至少有一个不小于 2.", + "solution": "设 $S_{\\triangle P P_2 P_3}=S_1, S_{\\triangle P P_3 P_1}=S_2, S_{\\triangle P P_1 P_2}=S_3$.\n易知 $\\frac{P_1 P}{P Q_1}=\\frac{S_{\\triangle P_1 P_2 P}}{S_{\\triangle P P_2 Q_1}}=\\frac{S_{\\triangle P_1 P_3 P}}{S_{\\triangle P P_3 Q_1}}$, 故\n$$\n\\frac{P_1 P}{P Q_1}=\\frac{S_{\\triangle P_1 P_2 P}+S_{\\triangle P_1 P_3 P}}{S_{\\triangle P P_2 Q_1}+S_{\\triangle P P_3 Q_1}}=\\frac{S_2+S_3}{S_1} .\n$$\n同理可得\n$$\n\\frac{P_2 P}{P Q_2}=\\frac{S_3+S_1}{S_2}, \\frac{P_3 P}{P Q_3}=\\frac{S_1+S_2}{S_3} .\n$$\n由对称性, 不妨设 $S_1 \\geqslant S_2 \\geqslant S_3$, 则有 $\\frac{P_1 P}{P Q_1} \\leqslant 2, \\frac{P_3 P}{P Q_3} \\geqslant 2$.", + "remark": "", + "figures": [] +} \ No newline at end of file diff --git a/processed_dataset/proof/0845.json b/processed_dataset/proof/0845.json new file mode 100644 index 0000000000000000000000000000000000000000..63b4bef5257a73558696bd31de4dcbad1bce44d8 --- /dev/null +++ b/processed_dataset/proof/0845.json @@ -0,0 +1,10 @@ +{ + "source_file": "./raw_volume-zh/volume14/exercise8.tex", + "problem_type": "proof", + "problem": "问题2. 求证: 同时平分一个三角形的面积和周长的直线一定经过三角形的内心.", + "solution": "设直线 $l$ 平分 $\\triangle A B C$ 的周长.\n如果 $l$ 过一个顶点, 例如过顶点 $A$, 则不难知道必有 $A B=A C$, 从而 $l$ 过内心 $I$.\n如图(), 如果 $l$ 经过 $\\triangle A B C$ 的两条边, 设它与 $A B 、 A C$ 交于点 $E 、 D$. 我们证明 $S_{\\triangle D E}=0$.\n设 $\\triangle A B C$ 的内切圆半径为 $r, p$ 为半周长, 则\n$$\n\\begin{gathered}\nA E+A D=p, \\\\\nS_{\\triangle A E}+S_{\\triangle A I D}=\\frac{1}{2} r \\cdot A E+\\frac{1}{2} r \\cdot A D=\\frac{1}{2} r p=\\frac{1}{2} S_{\\triangle A B C},\n\\end{gathered}\n$$\n但 $S_{\\triangle A E D}=\\frac{1}{2} S_{\\triangle A B C}$, 从而 $S_{\\triangle D E}=0$.", + "remark": "", + "figures": [ + "./images/volume14/figures/fig-c8a2.png" + ] +} \ No newline at end of file diff --git a/processed_dataset/proof/0846.json b/processed_dataset/proof/0846.json new file mode 100644 index 0000000000000000000000000000000000000000..75271a8c8baf598dc18a4dba0b0ef8e542e838f9 --- /dev/null +++ b/processed_dataset/proof/0846.json @@ -0,0 +1,11 @@ +{ + "source_file": "./raw_volume-zh/volume14/exercise8.tex", + "problem_type": "proof", + "problem": "问题3. 如图(), 在四边形 $A B C D$ 中, $\\triangle A B D, \\triangle B C D$, $\\triangle A B C$ 的面积之比为 $3: 4: 1$. 点 $M, N$ 分别在 $A C, C D$ 上, 满足 $\\frac{A M}{A C}=\\frac{C N}{C D}$, 并且 $B, M, N$ 三点共线.\n求证: $M$ 与 $N$ 分别是 $A C$ 与 $C D$ 的中点.", + "solution": "如图(), 设 $\\frac{A M}{A C}=\\frac{C N}{C D}=r(0), 作边 $B C$ 上的中线 $A D$. 设 $\\triangle A B C$ 的面积为 1, $\\triangle A B_1 C_1$ 的面积为 $S$. 我们用两种方法计算 $S$.\n一方面, $S=\\frac{S}{1}=\\frac{A B_1 \\cdot A C_1}{A B \\cdot A C}=\\lambda \\mu$.\n另一方面, $S=S_{\\triangle A B_1 G}+S_{\\triangle A G C_1}$.\n与(1)类似地, $S_{\\triangle A B_1 G}=\\frac{\\lambda}{3}, S_{\\triangle A G C_1}=\\frac{\\mu}{3}$,\n所以\n$$\nS=\\frac{\\lambda+\\mu}{3} .\n$$\n综合两方面可得\n$$\n\\lambda \\mu=\\frac{\\lambda+\\mu}{3},\n$$\n两边同乘 $\\frac{3}{\\lambda \\mu}$, 即证.", + "remark": "", + "figures": [ + "./images/volume14/figures/fig-c8a4.png" + ] +} \ No newline at end of file diff --git a/processed_dataset/proof/0848.json b/processed_dataset/proof/0848.json new file mode 100644 index 0000000000000000000000000000000000000000..d7f87842a7f137cceb49a6287858e49336291eb6 --- /dev/null +++ b/processed_dataset/proof/0848.json @@ -0,0 +1,10 @@ +{ + "source_file": "./raw_volume-zh/volume14/exercise8.tex", + "problem_type": "proof", + "problem": "问题5. 凸六边形 $A B C D E F$ 中, $P, Q, R, S, T, U$ 分别为 $A B, B C, C D, D E$, $E F, F A$ 的中点.\n若 $P S, Q T, R U$ 均平分六边形 $A B C D E F$ 的面积, 证明: $P S, Q T, R U$ 三线共点.", + "solution": "如图(), 设 $Q T, R U$ 交于点 $X$. 连接 $X A, X B, X C, X D, X E, X F, X P, X S$. 由已知得\n$$\nS_{A B Q T F}=\\frac{1}{2} S_{A B C D E F}=S_{A B C R U},\n$$\n两边减去 $S_{A B Q X U}$, 得\n$$\nS_{X T F U}=S_{X Q C R} .\n$$\n又\n$$\n\\begin{aligned}\n& S_{X E F A}=S_{\\triangle X E F}+S_{\\triangle X F A}=2 S_{\\triangle X T F}+2 S_{\\triangle X F U}=2 S_{X T F U}, \\\\\n& S_{X B C D}=S_{\\triangle X B C}+S_{\\triangle X C D}=2 S_{\\triangle X Q C}+2 S_{\\triangle X C R}=2 S_{X Q C R},\n\\end{aligned}\n$$\n所以 $S_{X E F A}=S_{X B C D}$. 结合 $S_{\\triangle X A P}=S_{\\triangle X B P}, S_{\\triangle X E S}=S_{\\triangle X D S}$ 可知, 折线 $P X S$ 平分凸六边形 $A B C D E F$ 的面积.\n考虑到 $P S$ 也平分 $A B C D E F$ 的面积, 故 $P S$ 过点 $X$.\n从而 $P S, Q T, R U$ 三线共点.", + "remark": "", + "figures": [ + "./images/volume14/figures/fig-c85.png" + ] +} \ No newline at end of file diff --git a/processed_dataset/proof/0849.json b/processed_dataset/proof/0849.json new file mode 100644 index 0000000000000000000000000000000000000000..662ce1cf2426f5038ccf6c717d417dbc3170537f --- /dev/null +++ b/processed_dataset/proof/0849.json @@ -0,0 +1,8 @@ +{ + "source_file": "./raw_volume-zh/volume14/exercise8.tex", + "problem_type": "proof", + "problem": "问题6. 记凸四边形的面积为 $S$. 对它的每个顶点, 都作其关于不经过它的对角线的对称点.\n将所得到的四个像点组成的四边形的面积记作 $S^{\\prime}$. 证明: $\\frac{S^{\\prime}}{S}<$ 3.", + "solution": "记原凸四边形为 $A B C D$, 其中 $A, B, C, D$ 的像点分别为 $A^{\\prime}, B^{\\prime}, C^{\\prime}$, $D^{\\prime}, A C$ 与 $B D$ 交于点 $P$.\n设 $A C$ 与 $B D$ 的夹角为 $\\alpha, A^{\\prime} C^{\\prime}$ 与 $B^{\\prime} D^{\\prime}$ 的夹角为 $\\beta$. 根据对称性, 四边形 $A^{\\prime} B^{\\prime} C^{\\prime} D^{\\prime}$ 的对角线 $A^{\\prime} C^{\\prime}$ 与 $B^{\\prime} D^{\\prime}$ 过点 $P$, 且 $A^{\\prime} C^{\\prime}=A C, B^{\\prime} D^{\\prime}=B D$, 且 $\\beta=3 \\alpha$ 或 $\\pi-3 \\alpha$ 或 $3 \\alpha-\\pi$, 无论何种情形下均有 $\\sin \\beta=|\\sin 3 \\alpha|=\\sin \\alpha \\cdot\\left|3-4 \\sin ^2 \\alpha\\right|$.\n由四边形面积公式得\n$$\n\\begin{aligned}\nS & =\\frac{1}{2} A C \\cdot B D \\cdot \\sin \\alpha, S^{\\prime}=\\frac{1}{2} A^{\\prime} C^{\\prime} \\cdot B^{\\prime} D^{\\prime} \\cdot \\sin \\beta \\\\\n& =\\frac{1}{2} A C \\cdot B D \\cdot \\sin \\alpha \\cdot\\left|3-4 \\sin ^2 \\alpha\\right|,\n\\end{aligned}\n$$\n从而易知: $\\frac{S^{\\prime}}{S}=\\left|3-4 \\sin ^2 \\alpha\\right|<3$.", + "remark": "", + "figures": [] +} \ No newline at end of file diff --git a/processed_dataset/proof/0850.json b/processed_dataset/proof/0850.json new file mode 100644 index 0000000000000000000000000000000000000000..c016fb0d528fab0ab49ac7ebed08ce0cc993a3de --- /dev/null +++ b/processed_dataset/proof/0850.json @@ -0,0 +1,11 @@ +{ + "source_file": "./raw_volume-zh/volume14/exercise8.tex", + "problem_type": "proof", + "problem": "问题7. 如图(), 在 $\\triangle A B C$ 中, $M$ 为 $B C$ 中点, $N, P, Q$ 分别在线段 $A M, A B, A C$ 内,使 $A, P, N, Q$ 四点共圆.\n证明: $A P \\cdot A B+A Q \\cdot A C=2 A N \\cdot A M$.", + "solution": "如图(), 连接 $P Q, P N, N Q$. 记 $\\angle P A N=\\alpha, \\angle Q A N= \\beta$. 由于 $M$ 为 $B C$ 中点, 故 $S_{\\triangle A B C}=2 S_{\\triangle A B M}=2 S_{\\triangle A M C}$, 即\n$$\n\\begin{gathered}\n\\frac{1}{2} A B \\cdot A C \\cdot \\sin (\\alpha+\\beta)=A B \\cdot A M \\cdot \\sin \\alpha= \\\\\nA M \\cdot A C \\cdot \\sin \\beta,\n\\end{gathered}\n$$\n所以 $\\frac{\\sin \\alpha}{\\sin (\\alpha+\\beta)}=\\frac{A C}{2 A M}, \\frac{\\sin \\beta}{\\sin (\\alpha+\\beta)}=\\frac{A B}{2 A M}$. \\label{eq1}\n由正弦定理得\n$$\n\\frac{N P}{\\sin \\alpha}=\\frac{N Q}{\\sin \\beta}=\\frac{P Q}{\\sin (\\alpha+\\beta)} . \\label{eq2}\n$$\n因为 $A, P, N, Q$ 四点共圆, 由托勒密定理得\n$$\nA N \\cdot P Q=A P \\cdot N Q+A Q \\cdot N P,\n$$\n结合 式\\ref{eq1}, \\ref{eq2}可得\n$$\n\\begin{aligned}\nA N & =A P \\cdot \\frac{N Q}{P Q}+A Q \\cdot \\frac{N P}{P Q}=\\frac{A P \\cdot \\sin \\beta}{\\sin (\\alpha+\\beta)}+\\frac{A Q \\cdot \\sin \\alpha}{\\sin (\\alpha+\\beta)} \\\\\n& =\\frac{A P \\cdot A B}{2 A M}+\\frac{A Q \\cdot A C}{2 A M} .\n\\end{aligned}\n$$\n从而 $A P \\cdot A B+A Q \\cdot A C=2 A N \\cdot A M$.", + "remark": "", + "figures": [ + "./images/volume14/figures/fig-c8p7.png", + "./images/volume14/figures/fig-c8a7.png" + ] +} \ No newline at end of file diff --git a/processed_dataset/proof/0851.json b/processed_dataset/proof/0851.json new file mode 100644 index 0000000000000000000000000000000000000000..a69f61f1726354bda3ed937b312abd1b72514d23 --- /dev/null +++ b/processed_dataset/proof/0851.json @@ -0,0 +1,10 @@ +{ + "source_file": "./raw_volume-zh/volume14/exercise8.tex", + "problem_type": "proof", + "problem": "问题8. 在凸五边形 $A B C D E$ 中, $A D$ 与 $B E$ 相交于 $F, B E$ 与 $C A$ 相交于 $G, C A$ 与 $D B$ 相交于 $H, D B$ 与 $E C$ 相交于 $I, E C$ 与 $A D$ 相交于 $J$. 设 $A^{\\prime} 、 B^{\\prime} 、 C^{\\prime} 、 D^{\\prime} 、 E^{\\prime}$ 分别为 $A I$ 与 $B E 、 B J$ 与 $C A 、 C F$ 与 $D B 、 D G$ 与 $E C 、 E H$ 与 $A D$ 的交点, 求证:\n$$\n\\frac{A B^{\\prime}}{B^{\\prime} C} \\cdot \\frac{C D^{\\prime}}{D^{\\prime} E} \\cdot \\frac{E A^{\\prime}}{A^{\\prime} B} \\cdot \\frac{B C^{\\prime}}{C^{\\prime} D} \\cdot \\frac{D E^{\\prime}}{E^{\\prime} A}=1 .\n$$", + "solution": "如图(), 由共边定理, 有 $\\frac{A B^{\\prime}}{B^{\\prime} C}=\\frac{S_{\\triangle A B J}}{S_{\\triangle C B J}}$, 其他的比例线段用同样的方法转化, 即只须证明\n$$\n\\frac{S_{\\triangle A B J}}{S_{\\triangle C B J}} \\cdot \\frac{S_{\\triangle C D G}}{S_{\\triangle E D G}} \\cdot \\frac{S_{\\triangle E A I}}{S_{\\triangle B A I}} \\cdot \\frac{S_{\\triangle B C F}}{S_{\\triangle D C F}} \\cdot \\frac{S_{\\triangle D E H}}{S_{\\triangle A E H}}=1 .\n$$\n因为\n$$\n\\frac{S_{\\triangle A B J}}{S_{\\triangle B A I}}=\\frac{S_{\\triangle A B J}}{S_{\\triangle A B D}} \\cdot \\frac{S_{\\triangle A B D}}{S_{\\triangle B A I}}=\\frac{A J}{A D} \\cdot \\frac{B D}{B I},\n$$\n用同样方法转化面积比,并消去上下相同的线段,只须证明:\n$$\n\\frac{A J}{B I} \\cdot \\frac{B F}{C J} \\cdot \\frac{C G}{D F} \\cdot \\frac{D H}{E G} \\cdot \\frac{E I}{A H}=1,\n$$\n或\n$$\n\\frac{A J}{C J} \\cdot \\frac{B F}{D F} \\cdot \\frac{C G}{E G} \\cdot \\frac{D H}{A H} \\cdot \\frac{E I}{B I}=1 .\n$$\n利用正弦定理转换上式的线段比, 只须证明\n$$\n\\frac{\\sin \\angle E C A}{\\sin \\angle C A D} \\cdot \\frac{\\sin \\angle A D B}{\\sin \\angle D B E} \\cdot \\frac{\\sin \\angle B E C}{\\sin \\angle E C A} \\cdot \\frac{\\sin \\angle C A D}{\\sin \\angle A D B} \\cdot \\frac{\\sin \\angle D B E}{\\sin \\angle B E C}=1,\n$$\n这显然成立.\n证毕.", + "remark": "", + "figures": [ + "./images/volume14/figures/fig-c8a8.png" + ] +} \ No newline at end of file diff --git a/processed_dataset/proof/0852.json b/processed_dataset/proof/0852.json new file mode 100644 index 0000000000000000000000000000000000000000..a409444c6eb374f3c18268c5a59e4f9d84d0e0a0 --- /dev/null +++ b/processed_dataset/proof/0852.json @@ -0,0 +1,8 @@ +{ + "source_file": "./raw_volume-zh/volume14/exercise9.tex", + "problem_type": "proof", + "problem": "问题2. 设 $a_1, a_2, \\cdots, a_9$ 都是非零实数,证明:下面的 6 个数:\n$$\na_1 a_5 a_9, a_2 a_6 a_7, a_3 a_4 a_8,-a_3 a_5 a_7,-a_1 a_6 a_8,-a_2 a_4 a_9\n$$\n中至少有一个是负数.", + "solution": "我们考虑这 6 个数的乘积:\n$$\n\\begin{aligned}\n& \\left(a_1 a_5 a_9\\right)\\left(a_2 a_6 a_7\\right)\\left(a_3 a_4 a_8\\right)\\left(-a_3 a_5 a_7\\right)\\left(-a_1 a_6 a_8\\right)\\left(-a_2 a_4 a_9\\right) \\\\\n= & -\\left(a_1 a_2 a_3 a_4 a_5 a_6 a_7 a_8 a_9\\right)^2<0,\n\\end{aligned}\n$$\n所以,这 6 个数中至少有一个是负数.", + "remark": "", + "figures": [] +} \ No newline at end of file diff --git a/processed_dataset/proof/0853.json b/processed_dataset/proof/0853.json new file mode 100644 index 0000000000000000000000000000000000000000..d9c139a7da3108fc99bc7fa5b153adcb720a0a2c --- /dev/null +++ b/processed_dataset/proof/0853.json @@ -0,0 +1,8 @@ +{ + "source_file": "./raw_volume-zh/volume14/exercise9.tex", + "problem_type": "proof", + "problem": "问题3. 已知: $S=\\{1,2, \\cdots, 21\\}$, 有限集 $A \\subseteq \\mathbf{N}^*$, 使得 $S$ 中任一元素或者属于 $A$,或者等于 $A$ 中两个不同元素的和.\n求满足条件的集合 $A$ 元素个数的最小值.", + "solution": "设 $A=\\left\\{a_1, a_2, \\cdots, a_t\\right\\}$.\n$A$ 中元素个数为 $t$, 二元子集数为 $\\mathrm{C}_t^2$, 故至多有 $t+\\mathrm{C}_t^2=\\frac{t(t+1)}{2}$ 个正整数值出现在 $A$ 中-一个元素或两个元素的和所对应的值中.\n由题意得, $\\frac{t(t+1)}{2} \\geqslant|S|=21 \\Rightarrow t \\geqslant 6$.\n若 $t=6$, 则 $S$ 中每个元素恰能表示成 $a_i(i=1,2, \\cdots, 6)$ 或 $a_i+a_j(1 \\leqslant i0, c>0$. 我们先证明 $a=c$.\n若不然, 由对称性不妨设 $a>c$.\n当 $x=-\\frac{b}{a}$ 时, $f(x)=0$, 因此 $g\\left(-\\frac{b}{a}\\right)$ 是整数; 当 $x=-\\frac{b-1}{a}$ 时, $f(x)=1$, 因此 $g\\left(-\\frac{b-1}{a}\\right)$ 是整数.\n故\n$$\ng\\left(-\\frac{b}{a}\\right)-g\\left(-\\frac{b-1}{a}\\right)=\\left(c \\cdot\\left(-\\frac{b}{a}\\right)+d\\right)-\\left(c \\cdot\\left(-\\frac{b-1}{a}\\right)+d\\right)=-\\frac{c}{a}\n$$\n是一个整数, 但这与 $a>c>0$ 矛盾, 又当 $x=-\\frac{b}{a}$ 时, $f(x)=0$, 因此 $g\\left(-\\frac{b}{a}\\right)=d-b$ 是整数, 因此对任意的 $x \\in \\mathbf{R}, f(x)-g(x)=b-d$ 是整数, 从而命题得证.", + "remark": "", + "figures": [] +} \ No newline at end of file diff --git a/processed_dataset/proof/0862.json b/processed_dataset/proof/0862.json new file mode 100644 index 0000000000000000000000000000000000000000..d7bff4a4cba1beee6c3f44df0bc7d48f5425edf5 --- /dev/null +++ b/processed_dataset/proof/0862.json @@ -0,0 +1,8 @@ +{ + "source_file": "./raw_volume-zh/volume2/chapter2.tex", + "problem_type": "proof", + "problem": "例2 证明: 任何定义域关于原点对称的函数都可以表示为一个奇函数和一个偶函数的和.", + "solution": "证.\n设定义域关于原点对称的函数为 $f(x)$, 则 $f(x)$ 与 $f(-x)(x \\in D)$ 同时有意义.\n因为\n$$\nf(x)=\\frac{f(x)+f(-x)}{2}+\\frac{f(x)-f(-x)}{2} .\n$$\n所以, 令 $f_1(x)=\\frac{f(x)+f(-x)}{2}, f_2(x)=\\frac{f(x)-f(-x)}{2}$, 下面我们来验证 $f_1(x)$ 是偶函数, $f_2(x)$ 是奇函数.\n$$\nf_1(-x)=\\frac{f(-x)+f(x)}{2}=f_1(x),\n$$\n$$\nf_2(-x)=\\frac{f(-x)-f(x)}{2}=-\\frac{f(x)-f(-x)}{2}=-f_2(x) .\n$$\n故 $f_1(x)$ 为偶函数, $f_2(x)$ 为奇函数, 从而命题得证.\n说明证明中的 $f_1(x)$ 和 $f_2(x)$ 是如何想出来的呢? 其实, $f_1(x)$ 和 $f_2(x)$ 是可以 “解” 出来的.\n设 $f(x)=f_1(x)+f_2(x)$, 其中 $f_1(x)$ 是偶函数, $f_2(x)$ 是奇函数.\n则解方程组, 得\n$$\n\\begin{gathered}\nf(x)=f_1(x)+f_2(x), \\\\\nf(-x)=f_1(x)-f_2(x), \\\\\nf_1(x)=\\frac{f(x)+f(-x)}{2}, \\\\\nf_2(x)=\\frac{f(x)-f(-x)}{2} .\n\\end{gathered}\n$$", + "remark": "", + "figures": [] +} \ No newline at end of file diff --git a/processed_dataset/proof/0863.json b/processed_dataset/proof/0863.json new file mode 100644 index 0000000000000000000000000000000000000000..fac7b70155450f2b1531128bdf73f77cdcd149ef --- /dev/null +++ b/processed_dataset/proof/0863.json @@ -0,0 +1,8 @@ +{ + "source_file": "./raw_volume-zh/volume2/chapter2.tex", + "problem_type": "proof", + "problem": "例3 设函数 $f(x)$ 对所有的实数 $x$ 都满足 $f(x+2 \\pi)=f(x)$, 求证: 存在 4 个函数 $f_i(x)(i=1,2,3,4)$ 满足:\n(1) 对 $i=1,2,3,4, f_i(x)$ 是偶函数, 且对任意的实数 $x$, 有 $f_i(x+ \\pi)=f_i(x)$;\n(2) 对任意的实数 $x$, 有 $f(x)=f_1(x)+f_2(x) \\cos x+f_3(x) \\sin x+ f_4(x) \\sin 2 x$.", + "solution": "证记 $g(x)=\\frac{f(x)+f(-x)}{2}, h(x)=\\frac{f(x)-f(-x)}{2}$, 则 $f(x)= g(x)+h(x)$, 且 $g(x)$ 是偶函数, $h(x)$ 是奇函数, 对任意的 $x \\in \\mathbf{R}, g(x+2 \\pi)= g(x), h(x+2 \\pi)=h(x)$. 令\n$$\n\\begin{aligned}\n& f_1(x)=\\frac{g(x)+g(x+\\pi)}{2}, \\\\\n& f_2(x)= \\begin{cases}\\frac{g(x)-g(x+\\pi)}{2 \\cos x}, & x \\neq k \\pi+\\frac{\\pi}{2}, \\\\\n0, & x=k \\pi+\\frac{\\pi}{2},\\end{cases} \\\\\n& f_3(x)= \\begin{cases}\\frac{h(x)-h(x+\\pi)}{2 \\sin x}, & x \\neq k \\pi, \\\\\n0, & x=k \\pi,\\end{cases} \\\\\n& f_4(x)=\\left\\{\\begin{array}{ll}\n\\frac{h(x)+h(x+\\pi)}{2 \\sin 2 x}, & x \\neq \\frac{k \\pi}{2}, \\\\\n0, & x=\\frac{k \\pi}{2},\n\\end{array} \\text { 其中 } k\\right. \\text { 为任意整数.\n}\n\\end{aligned}\n$$\n容易验证 $f_i(x), i=1,2,3,4$ 是偶函数, 且对任意的 $x \\in \\mathbf{R}, f_i(x+\\pi)= f_i(x), i=1,2,3,4$.\n下证对任意的 $x \\in \\mathbf{R}$, 有 $f_1(x)+f_2(x) \\cos x=g(x)$.\n当 $x \\neq k \\pi+\\frac{\\pi}{2}$ 时, 显然成立; 当 $x=k \\pi+\\frac{\\pi}{2}$ 时, 因为\n$$\nf_1(x)+f_2(x) \\cos x=f_1(x)=\\frac{g(x)+g(x+\\pi)}{2}\n$$\n而\n$$\n\\begin{aligned}\ng(x+\\pi) & =g\\left(k \\pi+\\frac{3 \\pi}{2}\\right)=g\\left(k \\pi+\\frac{3 \\pi}{2}-2(k+1) \\pi\\right) \\\\\n& =g\\left(-k \\pi-\\frac{\\pi}{2}\\right)=g\\left(k \\pi+\\frac{\\pi}{2}\\right)=g(x),\n\\end{aligned}\n$$\n故对任意的 $x \\in \\mathbf{R}, f_1(x)+f_2(x) \\cos x=g(x)$.\n下证对任意的 $x \\in \\mathbf{R}$, 有 $f_3(x) \\sin x+f_4(x) \\sin 2 x=h(x)$.\n当 $x \\neq \\frac{k \\pi}{2}$ 时, 显然成立; 当 $x=k \\pi$ 时, $h(x)=h(k \\pi)=h(k \\pi-2 k \\pi)= h(-k \\pi)=-h(k \\pi)$, 所以 $h(x)=h(k \\pi)=0$, 而此时\n$$\nf_3(x) \\sin x+f_4(x) \\sin 2 x=0,\n$$\n故 $h(x)=f_3(x) \\sin x+f_4(x) \\sin 2 x$;\n当 $x=k \\pi+\\frac{\\pi}{2}$ 时,\n$$\n\\begin{aligned}\nh(x+\\pi) & =h\\left(k \\pi+\\frac{3 \\pi}{2}\\right)=h\\left(k \\pi+\\frac{3 \\pi}{2}-2(k+1) \\pi\\right) \\\\\n& =h\\left(-k \\pi-\\frac{\\pi}{2}\\right)=-h\\left(k \\pi+\\frac{\\pi}{2}\\right)=-h(x),\n\\end{aligned}\n$$\n故 $f_3(x) \\sin x=\\frac{h(x)-h(x+\\pi)}{2}=h(x)$, 又 $f_4(x) \\sin 2 x=0$, 从而有\n$$\nh(x)=f_3(x) \\sin x+f_4(x) \\sin 2 x .\n$$\n于是, 对任意的 $x \\in \\mathbf{R}$, 有 $f_3(x) \\sin x+f_4(x) \\sin 2 x=h(x)$. 综上所述,结论得证.", + "remark": "", + "figures": [] +} \ No newline at end of file diff --git a/processed_dataset/proof/0864.json b/processed_dataset/proof/0864.json new file mode 100644 index 0000000000000000000000000000000000000000..c0fe1268651056d0accaaa79798be0b4caf89546 --- /dev/null +++ b/processed_dataset/proof/0864.json @@ -0,0 +1,8 @@ +{ + "source_file": "./raw_volume-zh/volume2/chapter2.tex", + "problem_type": "proof", + "problem": "例8 证明: 函数 $3 x^2+x$ 可以表示为两个单调递增的多项式函数之差.", + "solution": "证因为有恒等式\n$$\n3 x^2+x \\equiv(x+1)^3-\\left(x^3+2 x+1\\right),\n$$\n而函数 $g(x)=(x+1)^3, h(x)=x^3+2 x+1$ 都是单调递增的多项式函数, 从而命题得证.\n说明一般地, 任意实系数多项式可表示为两个单调递增的多项式函数之差.", + "remark": "", + "figures": [] +} \ No newline at end of file diff --git a/processed_dataset/proof/0865.json b/processed_dataset/proof/0865.json new file mode 100644 index 0000000000000000000000000000000000000000..eb02381b5d9e64c0211db141388e33d8e587beb4 --- /dev/null +++ b/processed_dataset/proof/0865.json @@ -0,0 +1,8 @@ +{ + "source_file": "./raw_volume-zh/volume2/chapter2.tex", + "problem_type": "proof", + "problem": "例9 证明: 函数 $f(x)=\\sin x^2$ 不是周期函数.", + "solution": "证我们常用反证法来证明某个函数不是周期函数.\n假设 $f(x)=\\sin x^2$ 是周期函数, $T$ 是它的一个正周期, 那么对每个 $x \\in \\mathbf{R}$, 有\n$$\n\\sin (x+T)^2=\\sin x^2 .\n$$\n令 $x=0$, 得 $\\sin T^2=0$. 所以 $T^2=k \\pi, T=\\sqrt{k \\pi}$, 其中 $k$ 是某个正整数, 代入(1)式, 得\n$$\n\\sin (x+\\sqrt{k \\pi})^2=\\sin x^2 .\n$$\n在(2)中令 $x=\\sqrt{2} T(=\\sqrt{2 k \\pi})$, 得\n$$\n\\sin (\\sqrt{2} T+T)^2=\\sin (\\sqrt{2} T)^2,\n$$\n即\n$$\n\\sin \\left[(\\sqrt{2}+1)^2 k \\pi\\right]=\\sin 2 k \\pi=0 .\n$$\n所以\n$$\n\\begin{gathered}\n(\\sqrt{2}+1)^2 k \\pi=l \\pi(l \\in \\mathbf{N}), \\\\\n(\\sqrt{2}+1)^2=\\frac{l}{k} .\n\\end{gathered}\n$$\n由于 $(\\sqrt{2}+1)^2=3+2 \\sqrt{2}$ 是无理数, 而 $\\frac{l}{k}$ 是有理数, 矛盾.\n因此 $f(x)=\\sin x^2$ 不是周期函数.", + "remark": "", + "figures": [] +} \ No newline at end of file diff --git a/processed_dataset/proof/0866.json b/processed_dataset/proof/0866.json new file mode 100644 index 0000000000000000000000000000000000000000..9b5627e13d725e0edb5c6c07ffb85bb390585ad7 --- /dev/null +++ b/processed_dataset/proof/0866.json @@ -0,0 +1,8 @@ +{ + "source_file": "./raw_volume-zh/volume2/chapter2.tex", + "problem_type": "proof", + "problem": "例10 证明: 若函数 $y=f(x)$ 在 $\\mathbf{R}$ 上的图象关于点 $A\\left(a, y_0\\right)$ 和直线 $x=b(b>a)$ 皆对称, 则函数 $f(x)$ 是 $\\mathbf{R}$ 上的周期函数.", + "solution": "证已知函数 $y=f(x)$ 的图象关于点 $A\\left(a, y_0\\right)$ 与直线 $x=b$ 对称, 所以对任意 $x \\in \\mathbf{R}$, 分别有\n$$\n\\begin{aligned}\nf(a+x)-y_0 & =y_0-f(a-x), \\\\\nf(b+x) & =f(b-x) .\n\\end{aligned}\n$$\n下面证明 $4(b-a)$ 是 $f(x)$ 的周期.\n事实上, 对任意 $x \\in \\mathbf{R}$, 反复利用 (1), (2), 有\n$$\n\\begin{aligned}\nf[x+4(b-a)] & =f[b+(x+3 b-4 a)] \\\\\n& =f[b-(x+3 b-4 a)] \\\\\n& =f[a+(3 a-2 b-x)] \\\\\n& =2 y_0-f[a-(3 a-2 b-x)] \\\\\n& =2 y_0-f[b+(b-2 a+x)] \\\\\n& =2 y_0-f[b-(b-2 a+x)] \\\\\n& =2 y_0-f[a+(a-x)] \\\\\n& =2 y_0-2 y_0+f[a-(a-x)] \\\\\n& =f(x) .\n\\end{aligned}\n$$\n同理可证 $f[x-4(b-a)]=f(x)$.\n所以 $4(b-a)$ 是函数 $f(x)$ 在 $\\mathbf{R}$ 上的一个周期.\n故 $f(x)$ 是周期函数.\n用完全相同的方法可以证明, 若函数 $f(x)$ 在 $\\mathbf{R}$ 上的图象关于直线 $x=a$ 与 $x=b(b>a)$ 对称, 则函数 $f(x)$ 是周期函数 $(2(b-a)$ 是它的一个周期).", + "remark": "", + "figures": [] +} \ No newline at end of file diff --git a/processed_dataset/proof/0867.json b/processed_dataset/proof/0867.json new file mode 100644 index 0000000000000000000000000000000000000000..15b4b09ded6af17a1badbdd2b87c43ab1bb4d33e --- /dev/null +++ b/processed_dataset/proof/0867.json @@ -0,0 +1,8 @@ +{ + "source_file": "./raw_volume-zh/volume2/chapter2.tex", + "problem_type": "proof", + "problem": "例11 设 $f(x)$ 是定义在 $\\mathbf{R}$ 上的偶函数, 其图象关于直线 $x=1$ 对称, 对任意 $x_1, x_2 \\in\\left[0, \\frac{1}{2}\\right]$, 都有 $f\\left(x_1+x_2\\right)=f\\left(x_1\\right) f\\left(x_2\\right)$, 且 $f(1)=a>0$.\n(1) 求 $f\\left(\\frac{1}{2}\\right)$ 和 $f\\left(\\frac{1}{4}\\right)$;\n(2) 证明 $f(x)$ 是周期函数;\n(3) 记 $a_n=f\\left(2 n+\\frac{1}{2 n}\\right)$, 求 $\\lim _{n \\rightarrow \\infty}\\left(\\ln a_n\\right)$.", + "solution": "解:(1) 令 $x_1=x_2=\\frac{x}{2} \\in\\left[0, \\frac{1}{2}\\right]$, 即 $x \\in[0,1]$, 有\n$$\n\\begin{aligned}\n& f(x)=f\\left(\\frac{x}{2}\\right) f\\left(\\frac{x}{2}\\right) \\geqslant 0, x \\in[0,1] . \\\\\n& a=f(1)=f\\left(\\frac{1}{2}+\\frac{1}{2}\\right)=f\\left(\\frac{1}{2}\\right) f\\left(\\frac{1}{2}\\right),\n\\end{aligned}\n$$\n所以\n$$\nf\\left(\\frac{1}{2}\\right)=\\sqrt{a} \\text {. }\n$$\n$$\n\\begin{gathered}\n\\sqrt{a}=f\\left(\\frac{1}{2}\\right)=f\\left(\\frac{1}{4}+\\frac{1}{4}\\right)=f\\left(\\frac{1}{4}\\right) f\\left(\\frac{1}{4}\\right), \\\\\nf\\left(\\frac{1}{4}\\right)=\\sqrt[4]{a} .\n\\end{gathered}\n$$\n所以\n$$\nf\\left(\\frac{1}{4}\\right)=\\sqrt[4]{a} .\n$$\n(2) 因为 $y=f(x)$ 的图象关于直线 $x=1$ 对称,所以\n$$\nf(1+x)=f(1-x), x \\in \\mathbf{R},\n$$\n于是\n$$\nf(x)=f(2-x), x \\in \\mathbf{R} .\n$$\n又 $f(x)$ 是偶函数,所以 $f(-x)=f(x)$, 故\n$$\nf(-x)=f(2-x), x \\in \\mathbf{R} .\n$$\n即\n$$\nf(x)=f(x+2), x \\in \\mathbf{R} .\n$$\n这就表明 $f(x)$ 是 $\\mathbf{R}$ 上的周期函数, 2 是它的一个周期.\n(3) 由第 (1) 小题知, $f(x) \\geqslant 0, x \\in[0,1]$. 因为\n$$\n\\begin{aligned}\n\\sqrt{a} & =f\\left(\\frac{1}{2}\\right)=f\\left(n \\cdot \\frac{1}{2 n}\\right)=f\\left(\\frac{1}{2 n}+(n-1) \\frac{1}{2 n}\\right) \\\\\n& =f\\left(\\frac{1}{2 n}\\right) \\cdot f\\left((n-1) \\frac{1}{2 n}\\right) \\\\\n& =\\cdots \\\\\n& =f\\left(\\frac{1}{2 n}\\right) \\cdot f\\left(\\frac{1}{2 n}\\right) \\cdot \\cdots \\cdot f\\left(\\frac{1}{2 n}\\right) \\\\\n& =\\left(f\\left(\\frac{1}{2 n}\\right)\\right)^n,\n\\end{aligned}\n$$\n故\n$$\nf\\left(\\frac{1}{2 n}\\right)=a^{\\frac{1}{2 n}}\n$$\n又由第 (2) 小题知, $f(x)$ 是一个周期函数, 2 是它的一个周期, 所以\n$$\nf\\left(2 n+\\frac{1}{2 n}\\right)=f\\left(\\frac{1}{2 n}\\right) .\n$$\n故\n$$\na_n=a^{\\frac{1}{2 n}} \\text {. }\n$$\n所以\n$$\n\\lim _{n \\rightarrow \\infty}\\left(\\ln a_n\\right)=\\lim _{n \\rightarrow \\infty}\\left(\\frac{1}{2 n} \\ln a\\right)=0 .\n$$", + "remark": "", + "figures": [] +} \ No newline at end of file diff --git a/processed_dataset/proof/0868.json b/processed_dataset/proof/0868.json new file mode 100644 index 0000000000000000000000000000000000000000..568d7369caa2703a5f396452445338ad03ae4df7 --- /dev/null +++ b/processed_dataset/proof/0868.json @@ -0,0 +1,8 @@ +{ + "source_file": "./raw_volume-zh/volume2/chapter2.tex", + "problem_type": "proof", + "problem": "例12 求函数 $g(x)=|\\sin x|+|\\cos x|$ 的最小正周期.", + "solution": "解:对任意 $x \\in \\mathbf{R}$, 有\n$$\n\\begin{aligned}\ng\\left(x+\\frac{\\pi}{2}\\right) & =\\left|\\sin \\left(x+\\frac{\\pi}{2}\\right)\\right|+\\left|\\cos \\left(x+\\frac{\\pi}{2}\\right)\\right| \\\\\n& =|\\sin x|+|\\cos x| \\\\\n& =g(x) .\n\\end{aligned}\n$$\n因此, $g(x)$ 是周期函数, $\\frac{\\pi}{2}$ 是它的一个周期.\n下面证明 $\\frac{\\pi}{2}$ 是 $g(x)$ 的最小正周期.\n设函数 $g(x)$ 有小于 $\\frac{\\pi}{2}$ 的正周期 $T$, 则对 $x \\in \\mathbf{R}$, 有\n$$\n|\\sin (x+T)|+|\\cos (x+T)|=|\\sin x|+|\\cos x| .\n$$\n令 $x=0$ 代入(1), 得\n$$\n|\\sin T|+|\\cos T|=1 .\n$$\n两边平方, 得\n$$\n2|\\sin T||\\cos T|=0,\n$$\n即\n$$\n|\\sin 2 T|=0 \\text {. }\n$$\n从而 $\\sin 2 T=0$, 但 $0a_n> a_{n+1}>0(n=1,2, \\cdots)$, 且每个 $a_n(n=1,2, \\cdots)$ 都是 $f(x)$ 的周期.", + "solution": "证 (1) 若 $T$ 是有理数,则存在正整数 $m 、 n$ 使得 $T=\\frac{n}{m}$ 且 $(m, n)=1$, 从而存在整数 $a 、 b$, 使得 $m a+n b=1$. 于是\n$$\n\\frac{1}{m}=\\frac{m a+n \\underline{b}}{m}=a+b T=a \\cdot 1+b \\cdot T\n$$\n是 $f(x)$ 的周期.\n又因 $00$;\n(2) 设 $F(x)=f(x+t)-f(x-t)(t \\neq 0)$, 判断 $F(x)$ 的奇偶性.", + "solution": "分析:第(1) 小题的解决可通过尝试的办法, 根据 $f(x)$ 的解析式的结构特点, 计算 $f(x)+f(-x)$ 及 $f(x)-f(-x)$ 可能比直接判断 $f(x)$ 与 $f(-x)$ 的关系来得更加方便.\n解决第 (2) 小题的关键在于理解 $t$ 是一个与 $x$ 无关的常数.\n解 (1) 首先容易确定 $f(x)$ 的定义域为 $\\{x \\mid x \\in \\mathbf{R}$, 且 $x \\neq 0\\}$, 关于原点对称, 而\n$$\nf(x)-f(-x)=\\left(\\frac{1}{2^x-1}+\\frac{1}{2}\\right) x-\\left(\\frac{1}{2^{-x}-1}+\\frac{1}{2}\\right)(-x)=0 .\n$$\n于是 $f(x)=f(-x)$. 故 $f(x)$ 是偶函数.\n当 $x>0$ 时, $2^x-1>0 \\Rightarrow f(x)>0$, 而当 $x<0$ 时, 由于 $f(x)$ 是偶函数及 $f(x)>0$, 所以 $f(x)$ 在其定义域上恒大于 0 .\n(2) 由 $\\left\\{\\begin{array}{l}x+t \\neq 0, \\\\ x-t \\neq 0,\\end{array}\\right.$ 知 $F(x)$ 的定义域为 $\\{x \\mid x \\in \\mathbf{R}$, 且 $x \\neq \\pm t\\}$.\n因为\n$$\n\\begin{aligned}\nF(-x) & =f(-x+t)-f(-x-t) \\\\\n& =f(-(x-t))-f(-(x+t)) \\\\\n& =f(x-t)-f(x+t)=-F(x) .\n\\end{aligned}\n$$\n所以 $F(x)$ 为奇函数.\n说明本题在两次判断函数的奇偶性时采用了不同的方法.\n这是因为一般来说如果函数的解析式中含有分式、指数式、对数式时, 判断奇偶性用作差法或作商法可能更加简洁自然.", + "remark": "", + "figures": [] +} \ No newline at end of file diff --git a/processed_dataset/proof/0872.json b/processed_dataset/proof/0872.json new file mode 100644 index 0000000000000000000000000000000000000000..1830316295f2910d184b4274afc835d60c6255e8 --- /dev/null +++ b/processed_dataset/proof/0872.json @@ -0,0 +1,8 @@ +{ + "source_file": "./raw_volume-zh/volume2/chapter3.tex", + "problem_type": "proof", + "problem": "例10 已知 $00, a^y>0$, 由平均不等式\n$$\na^x+a^y \\geqslant 2 \\sqrt{a^x \\cdot a^y}=2 a^{\\frac{x+y}{2}} .\n$$\n故\n$$\n\\begin{aligned}\n\\log _a\\left(a^x+a^y\\right) & \\leqslant \\log _a\\left(2 a^{\\frac{x+y}{2}}\\right) \\\\\n& =\\log _a 2+\\frac{1}{2}(x+y) \\\\\n& =\\log _a 2+\\frac{1}{2}\\left(x-x^2\\right) \\\\\n& =\\log _a 2-\\frac{1}{2}\\left(x-\\frac{1}{2}\\right)^2+\\frac{1}{8} \\\\\n& \\leqslant \\log _a 2+\\frac{1}{8} .\n\\end{aligned}\n$$", + "remark": "", + "figures": [] +} \ No newline at end of file diff --git a/processed_dataset/proof/0873.json b/processed_dataset/proof/0873.json new file mode 100644 index 0000000000000000000000000000000000000000..d4904dc27d12662cadf08e046aef6038b74f3a68 --- /dev/null +++ b/processed_dataset/proof/0873.json @@ -0,0 +1,8 @@ +{ + "source_file": "./raw_volume-zh/volume2/chapter4.tex", + "problem_type": "proof", + "problem": "例7 已知 $x, y$ 是实数, 且满足 $x^2+x y+y^2=3$, 求 $u=x^2-x y+y^2$ 的最大值与最小值.", + "solution": "分析:这是一个有关最值的经典问题, 有多种解法, 这里介绍两种最常见的解法.\n由于已知条件都是二次的代数式, 故我们试图将其与判别式或基本不等式建立联系.\n解法一将题设中的两式相减再除以 2 , 得 $x y=\\frac{3-u}{2}$, 再用题设的第一式相加此式及用题设的第二式相减此式分别得\n$$\n(x+y)^2=\\frac{9-u}{2}\n$$\n和\n$$\n(x-y)^2=\\frac{3 u-3}{2},\n$$\n可得\n$$\n1 \\leqslant u \\leqslant 9\n$$\n而容易验证当 $x=\\sqrt{3}, y=-\\sqrt{3}$ 时, $u=9$;\n当 $x=y=1$ 时, $u=1$.\n所以 $u_{\\max }=9, u_{\\min }=1$.\n解法二因为\n$$\n\\begin{aligned}\nu & =x^2-x y+y^2 \\\\\n& =x^2+x y+y^2-2 x y \\\\\n& =3-2 x y,\n\\end{aligned}\n$$\n由于\n$$\nx^2+y^2 \\geqslant 2 x y \\text { 及 } x^2+y^2 \\geqslant-2 x y,\n$$\n将此两式分别代入 $x^2+x y+y^2=3$, 得 $x y \\leqslant 1$ 和 $x y \\geqslant-3$.\n所以\n$$\n1 \\leqslant u=3-2 x y \\leqslant 9 .\n$$\n再从推导的过程中可以看到:\n当 $x=\\sqrt{3}, y=-\\sqrt{3}$ 时, $u=9$ ;\n当 $x=y=1$ 时, $u=1$.\n所以\n$$\nu_{\\max }=9, u_{\\min }=1 \\text {. }\n$$", + "remark": "", + "figures": [] +} \ No newline at end of file diff --git a/processed_dataset/proof/0874.json b/processed_dataset/proof/0874.json new file mode 100644 index 0000000000000000000000000000000000000000..aae14c62ecada2af995bfafa1b0d318877c3320b --- /dev/null +++ b/processed_dataset/proof/0874.json @@ -0,0 +1,8 @@ +{ + "source_file": "./raw_volume-zh/volume2/chapter4.tex", + "problem_type": "proof", + "problem": "例9 设函数 $f:(0,1) \\rightarrow \\mathbf{R}$ 定义为\n$$\nf(x)= \\begin{cases}x, & \\text { 当 } x \\text { 是无理数时; } \\\\ \\frac{p+1}{q}, & \\text { 当 } x=\\frac{p}{q},(p, q)=1,0\\frac{8}{9}$, 只需证 $x$ 是有理数时, $f(x) \\leqslant \\frac{16}{17}$ 即可.\n解因为 $\\frac{7}{8}<\\frac{7+8}{8+9}<\\frac{8}{9}$, 即 $\\frac{7}{8}<\\frac{15}{17}<\\frac{8}{9}$.\n由定义知 $f\\left(\\frac{15}{17}\\right)=\\frac{16}{17}$. 下面证明: $f(x) \\leqslant \\frac{16}{17}, x \\in\\left(\\frac{7}{8}, \\frac{8}{9}\\right)$.\n(1) 若 $x \\in\\left(\\frac{7}{8}, \\frac{8}{9}\\right)$, 且 $x$ 是无理数,则 $f(x)=x<\\frac{8}{9}<\\frac{16}{17}$.\n(2) 若 $x \\in\\left(\\frac{7}{8}, \\frac{8}{9}\\right)$, 且 $x$ 是有理数, 设 $x=\\frac{p}{q}$, 其中 $(p, q)=1,0< py$, 则 $x \\geqslant y+1$. 这一结论非常有用.\n本题就是由 $\\left\\{\\begin{array}{l}7 q<8 p, \\\\ 9 p<8 q\\end{array}\\right.$ 推出 $\\left\\{\\begin{array}{l}7 q+1 \\leqslant 8 p, \\\\ 9 p+1 \\leqslant 8 q,\\end{array}\\right.$ 从而得出 $q \\geqslant 17$ 的.", + "remark": "", + "figures": [] +} \ No newline at end of file diff --git a/processed_dataset/proof/0875.json b/processed_dataset/proof/0875.json new file mode 100644 index 0000000000000000000000000000000000000000..cc6bdda5201bf1ef46faa1f641ad7f909fb47b95 --- /dev/null +++ b/processed_dataset/proof/0875.json @@ -0,0 +1,8 @@ +{ + "source_file": "./raw_volume-zh/volume2/chapter4.tex", + "problem_type": "proof", + "problem": "例19 已知 $\\alpha, \\beta$ 是方程 $4 x^2-4 t x-1=0(t \\in \\mathbf{R})$ 的两个不等实根, 函数 $f(x)=\\frac{2 x-t}{x^2+1}$ 的定义域为 $[\\alpha, \\beta]$.\n(1) 求 $g(t)=\\max f(x)-\\min f(x)$;\n(2) 证明: 对于 $u_i \\in\\left(0, \\frac{\\pi}{2}\\right)(i=1,2,3)$, 若 $\\sin u_1+\\sin u_2+\\sin u_3=$ 1 , 则\n$$\n\\frac{1}{g\\left(\\tan u_1\\right)}+\\frac{1}{g\\left(\\tan u_2\\right)}+\\frac{1}{g\\left(\\tan u_3\\right)}<\\frac{3}{4} \\sqrt{6}\n$$", + "solution": "解:(1) 由于 $\\alpha<\\beta$, 所以\n$$\n\\alpha=\\frac{t-\\sqrt{t^2+1}}{2}, \\beta=\\frac{t+\\sqrt{t^2+1}}{2} .\n$$\n令 $u=2 x-t$, 则 $x=\\frac{u+t}{2}$,\n$$\nf(x)=\\frac{u}{\\left(\\frac{u+t}{2}\\right)^2+1}=\\frac{4 u}{u^2+2 t u+t^2+4} .\n$$\n若 $u=0$, 即 $x=\\frac{t}{2}$, 此时 $f(x)==0$.\n若 $u \\neq 0$, 则\n$$\nf(x)=-\\frac{4}{u+\\frac{t^2+4}{u}+2 t}, u \\in\\left[-\\sqrt{t^2+1}, 0\\right) \\cup\\left(0, \\sqrt{t^2+1}\\right] .\n$$\n设 $h(u)=u+\\frac{t^2+4}{u}, u \\in\\left[-\\sqrt{t^2+1}, \\sqrt{t^2+1}\\right]$, 易知 $h(u)$ 在$\\left[-\\sqrt{t^2+1}, 0\\right)$ 上是递减的, 且 $h(u)<0, h(u)$ 在 $\\left(0, \\sqrt{t^2+1}\\right]$ 上也是递减的, 且 $h(u)>0$, 所以, $f(x)$ 在 $\\left[-\\sqrt{t^2+1}, 0\\right)$ 上是递增的, 在 $\\left(0, \\sqrt{t^2+1}\\right]$ 上也是递增的, 且当 $u \\in\\left[-\\sqrt{t^2+1}, 0\\right)$ 时, $f(u)<0$, 当 $u \\in\\left(0, \\sqrt{t^2+1}\\right]$ 时, $f(u)>0$. 于是 $f(x)$ 在 $[\\alpha, \\beta]$ 上是递增的.\n故\n$$\n\\begin{aligned}\ng(t) & =f(\\beta)-f(\\alpha) \\\\\n& =\\frac{(\\beta-\\alpha)[t(\\alpha+\\beta)-2 \\alpha \\beta+2]}{\\alpha^2 \\beta^2+\\alpha^2+\\beta^2+1} \\\\\n& =\\frac{8 \\sqrt{t^2+1}\\left(2 t^2+5\\right)}{16 t^2+25} .\n\\end{aligned}\n$$\n(2) 因为由 (1) 知\n$$\n\\begin{aligned}\ng\\left(\\tan u_i\\right) & =\\frac{8 \\cdot \\sec u_i\\left(2 \\sec ^2 u_i+3\\right)}{16 \\sec ^2 u_i+9} \\\\\n& =\\frac{24 \\cos ^2 u_i+16}{9 \\cos ^3 u_i+16 \\cos u_i}, i=1,2,3 .\n\\end{aligned}\n$$\n所以\n$$\n\\begin{aligned}\n\\frac{1}{g\\left(\\tan u_i\\right)} & =\\frac{9 \\cos ^3 u_i+16 \\cos u_i}{24 \\cos ^2 u_i+16} \\\\\n& =\\frac{3}{8} \\cos u_i+\\frac{5 \\cos u_i}{12 \\cos ^2 u_i+8} \\\\\n& \\leqslant \\frac{3}{8} \\cos u_i+\\frac{5 \\cos u_i}{2 \\sqrt{12 \\times 8} \\cos u_i} \\\\\n& =\\frac{3}{8} \\cos u_i+\\frac{5 \\sqrt{6}}{48}\n\\end{aligned}\n$$\n而由柯西不等式, 得\n$$\n\\begin{gathered}\n\\left(\\cos u_1+\\cos u_2+\\cos u_3\\right)^2 \\\\\n\\leqslant 3\\left(\\cos ^2 u_1+\\cos ^2 u_2+\\cos ^2 u_3\\right)\n\\end{gathered}\n$$\n$$\n\\begin{aligned}\n& =3\\left[3-\\left(\\sin ^2 u_1+\\sin ^2 u_2+\\sin ^2 u_3\\right)\\right] \\\\\n& =9-3\\left(\\sin ^2 u_1+\\sin ^2 u_2+\\sin ^2 u_3\\right) \\\\\n& \\leqslant 9-3 \\cdot \\frac{1}{3}\\left(\\sin u_1+\\sin u_2+\\sin u_3\\right)^2=8 .\n\\end{aligned}\n$$\n所以\n$$\n\\cos u_1+\\cos u_2+\\cos u_3 \\leqslant 2 \\sqrt{2} .\n$$\n于是\n$$\n\\begin{aligned}\n& \\frac{1}{g\\left(\\tan u_1\\right)}+\\frac{1}{g\\left(\\tan u_2\\right)}+\\frac{1}{g\\left(\\tan u_3\\right)} \\\\\n\\leqslant & \\frac{3}{8}\\left(\\cos u_1+\\cos u_2+\\cos u_3\\right)+\\frac{5 \\sqrt{6}}{16} \\\\\n\\leqslant & \\frac{3 \\sqrt{2}}{4}+\\frac{5 \\sqrt{6}}{16}<\\frac{7}{16} \\sqrt{6}+\\frac{5}{16} \\sqrt{6} \\\\\n= & \\frac{3}{4} \\sqrt{6}\n\\end{aligned}\n$$", + "remark": "", + "figures": [] +} \ No newline at end of file diff --git a/processed_dataset/proof/0876.json b/processed_dataset/proof/0876.json new file mode 100644 index 0000000000000000000000000000000000000000..7ed18871401ba4127745b5f38796b2c0747ed880 --- /dev/null +++ b/processed_dataset/proof/0876.json @@ -0,0 +1,8 @@ +{ + "source_file": "./raw_volume-zh/volume2/chapter5.tex", + "problem_type": "proof", + "problem": "例1 设 $a 、 b 、 c$ 是绝对值小于 1 的实数, 证明:\n$$\na b+b c+c a+1>0 .\n$$", + "solution": "证构造一次函数:\n$$\nf(x)=(b+c) x+b c+1,-10, \\\\\nf(1)=(b+c)+b c+1=(b+1)(c+1)>0,\n\\end{gathered}\n$$\n所以当 $-10$.\n说明利用一次函数的单调性来证明不等式是一种常用的方法.\n如何 “构造”好这个一次函数是解题的关键.", + "remark": "", + "figures": [] +} \ No newline at end of file diff --git a/processed_dataset/proof/0877.json b/processed_dataset/proof/0877.json new file mode 100644 index 0000000000000000000000000000000000000000..78d5c75054f1e75da833ad2dff05341cec3b718e --- /dev/null +++ b/processed_dataset/proof/0877.json @@ -0,0 +1,8 @@ +{ + "source_file": "./raw_volume-zh/volume2/chapter5.tex", + "problem_type": "proof", + "problem": "例2 证明柯西不等式: 设 $a_1, a_2, \\cdots, a_n, b_1, b_2, \\cdots, b_n$ 是实数, 则 $\\left(a_1^2+a_2^2+\\cdots+a_n^2\\right)\\left(b_1^2+b_2^2+\\cdots+b_n^2\\right) \\geqslant\\left(a_1 b_1+a_2 b_2+\\cdots+a_n b_n\\right)^2$.", + "solution": "证若 $a_1^2+a_2^2+\\cdots+a_n^2=0$, 则 $a_1=a_2=\\cdots=a_n=0$, 此时命题显然成立.\n若 $a_1^2+a_2^2+\\cdots+a_n^2 \\neq 0$, 构造一个二次函数\n$$\n\\begin{aligned}\nf(x) & =\\left(a_1^2+a_2^2+\\cdots+a_n^2\\right) x^2-2\\left(a_1 b_1+a_2 b_2+\\cdots+a_n b_n\\right) x+\\left(b_1^2+b_2^2+\\cdots+b_n^2\\right) \\\\\n& =\\left(a_1 x-b_1\\right)^2+\\left(a_2 x-b_2\\right)^2+\\cdots+\\left(a_n x-b_n\\right)^2 .\n\\end{aligned} .\n$$\n这是一条开口向上的抛物线, 而且 $f(x) \\geqslant 0$ 恒成立, 所以\n$$\n\\begin{aligned}\n\\Delta= & 4\\left(a_1 b_1+a_2 b_2+\\cdots+a_n b_n\\right)^2- \\\\\n& 4\\left(a_1^2+a_2^2+\\cdots+a_n^2\\right)\\left(b_1^2+b_2^2+\\cdots+b_n^2\\right) \\leqslant 0, \\\\\n& \\left(a_1^2+a_2^2+\\cdots+a_n^2\\right)\\left(b_1^2+b_2^2+\\cdots+b_n^2\\right) \\\\\n\\geqslant & \\left(a_1 b_1+a_2 b_2+\\cdots+a_n b_n\\right)^2 .\n\\end{aligned}\n$$\n即其中等号当且仅当 $a_i=k b_i, i=1,2, \\cdots, n, k$ 是某个常数时成立.\n说明对于要证明\n$$\nA C \\leqslant(\\text { 或 } \\geqslant) B^2,\n$$\n这类不等式,我们先把不等式变形为\n$$\n4 A C \\leqslant(\\text { 或 } \\geqslant)(2 B)^2,\n$$\n然后构造一个二次函数\n$$\nf(x)=A x^2-(2 B) x+C,\n$$\n再设法证明其判别式 $\\geqslant 0$ (或 $\\leqslant 0$ ).", + "remark": "", + "figures": [] +} \ No newline at end of file diff --git a/processed_dataset/proof/0878.json b/processed_dataset/proof/0878.json new file mode 100644 index 0000000000000000000000000000000000000000..ddbf1dda0c79a3ef077918ea0881f8b32b7c31de --- /dev/null +++ b/processed_dataset/proof/0878.json @@ -0,0 +1,8 @@ +{ + "source_file": "./raw_volume-zh/volume2/chapter5.tex", + "problem_type": "proof", + "problem": "例3 设 $x_1, x_2, \\cdots, x_n, y_1, y_2, \\cdots, y_n(n \\geqslant 2)$ 都是实数, 且满足\n$$\nx_1^2+x_2^2+\\cdots+x_n^2 \\leqslant 1 \\text {. }\n$$\n求证: $\\quad\\left(x_1 y_1+x_2 y_2+\\cdots+x_n y_n-1\\right)^2 \\geqslant\\left(x_1^2+x_2^2+\\cdots+x_n^2-1\\right)\\left(y_1^2+y_2^2+\\cdots+y_n^2-1\\right) .$", + "solution": "证当 $x_1^2+x_2^2+\\cdots+x_n^2=1$ 时, 原不等式显然成立.\n当 $x_1^2+x_2^2+\\cdots+x_n^2<1$ 时, 构造如下关于 $t$ 的二次函数\n$$\n\\begin{aligned}\nf(t)= & \\left(x_1^2+x_2^2+\\cdots+x_n^2-1\\right) t^2-2\\left(x_1 y_1+x_2 y_2+\\cdots+x_n y_n-1\\right) t \\\\\n& +\\left(y_1^2+y_2^2+\\cdots+y_n^2-1\\right) \\\\\n= & \\left(x_1 t-y_1\\right)^2+\\left(x_2 t-y_2\\right)^2+\\cdots+\\left(x_n t-y_n\\right)^2-(t-1)^2 .\n\\end{aligned}\n$$\n此二次函数的图象是一条开口向下的抛物线, 因为\n$$\nf(1)=\\left(x_1-y_1\\right)^2+\\left(x_2-y_2\\right)^2+\\cdots+\\left(x_n-y_n\\right)^2 \\geqslant 0,\n$$\n所以此抛物线一定与 $x$ 轴有交点, 从而\n$$\n\\Delta=4\\left(x_1 y_1+x_2 y_2+\\cdots+x_n y_n-1\\right)^2-\n$$\n$$\n\\begin{aligned}\n& 4\\left(x_1^2+x_2^2+\\cdots+x_n^2-1\\right)\\left(y_1^2+y_2^2+\\cdots+y_n^2-1\\right) \\geqslant 0, \\\\\n& \\left(x_1 y_1+x_2 y_2+\\cdots+x_n y_n-1\\right)^2 \\\\\n\\geqslant & \\left(x_1^2+x_2^2+\\cdots+x_n^2-1\\right)\\left(y_1^2+y_2^2+\\cdots+y_n^2-1\\right) .\n\\end{aligned}\n$$", + "remark": "", + "figures": [] +} \ No newline at end of file diff --git a/processed_dataset/proof/0879.json b/processed_dataset/proof/0879.json new file mode 100644 index 0000000000000000000000000000000000000000..554bb2165e5712a08a376b9121d41b3e740ff766 --- /dev/null +++ b/processed_dataset/proof/0879.json @@ -0,0 +1,8 @@ +{ + "source_file": "./raw_volume-zh/volume2/chapter5.tex", + "problem_type": "proof", + "problem": "例4 设 $x, y \\in \\mathbf{R}^{+}, x+y=c, c$ 为定值且 $01$. 证明: 方程的正根比 1 小, 负根比 -1 大.", + "solution": "证原方程整理后, 得 $2 a^2 x^2+2 a x+1-a^2=0$.\n令 $\\quad f(x)=2 a^2 x^2+2 a x+1-a^2$,\n则 $f(x)$ 是开口向上的抛物线, 且 $f(0)=1-a^2<0$. 故此二次函数 $f(x)=0$ 有一个正根, 一个负根.\n要证明正根比 1 小, 只需证 $f(1)>0$. 要证明负根比 -1 大, 只需证 $f(-1)>0$. 因为\n$$\n\\begin{gathered}\nf(1)=2 a^2+2 a+1-a^2=(a+1)^2>0, \\\\\nf(-1)=2 a^2-2 a+1-a^2=(a-1)^2>0,\n\\end{gathered}\n$$\n从而命题得证.", + "remark": "", + "figures": [] +} \ No newline at end of file diff --git a/processed_dataset/proof/0882.json b/processed_dataset/proof/0882.json new file mode 100644 index 0000000000000000000000000000000000000000..97eee12195c4c026aba99dc1591f44be742f5a78 --- /dev/null +++ b/processed_dataset/proof/0882.json @@ -0,0 +1,8 @@ +{ + "source_file": "./raw_volume-zh/volume2/chapter5.tex", + "problem_type": "proof", + "problem": "例7 求 $y=(3 x-1)\\left(\\sqrt{9 x^2-6 x+5}+1\\right)+(2 x-3) (\\sqrt{4 x^2-12 x+13}+ 1)$ 的图象与 $x$ 轴的交点的坐标.", + "solution": "分析:仔细观察所给式子的特点,发现\n$$\n\\begin{aligned}\ny= & (3 x-1)\\left(\\sqrt{(3 x-1)^2+4}+1\\right)+ \\\\\n& (2 x-3)\\left(\\sqrt{(2 x-3)^2+4}+1\\right),\n\\end{aligned}\n$$\n从而可以找到解题的途径.\n解因为\n$$\n\\begin{aligned}\ny= & (3 x-1)\\left(\\sqrt{(3 x-1)^2+4}+1\\right)+ \\\\\n& (2 x-3)\\left(\\sqrt{(2 x-3)^2+4}+1\\right) .\n\\end{aligned}\n$$\n构造函数\n$$\nf(t)=t\\left(\\sqrt{t^2+4}+1\\right) .\n$$\n因为 $f(-t)=-t\\left(\\sqrt{t^2+4}+1\\right)=-f(t)$, 所以 $f(t)$ 是奇函数.\n又因为当 $t \\geqslant 0$ 时, $f(t)$ 是递增的, 所以, 当 $t \\in \\mathbf{R}$ 时, $f(t)$ 也是递增的(易证), 而\n$$\ny=f(3 x-1)+f(2 x-3) .\n$$\n当 $y=0$ 时, 得\n$$\nf(3 x-1)=-f(2 x-3)=f(3-2 x),\n$$\n所以\n$$\n3 x-1=3-2 x .\n$$\n解方程, 得 $x=\\frac{4}{5}$.\n故图象与 $x$ 轴的交点坐标为 $\\left(\\frac{4}{5}, 0\\right)$.", + "remark": "", + "figures": [] +} \ No newline at end of file diff --git a/processed_dataset/proof/0883.json b/processed_dataset/proof/0883.json new file mode 100644 index 0000000000000000000000000000000000000000..6305cac9e5249ee221650e9c941b40df45d74044 --- /dev/null +++ b/processed_dataset/proof/0883.json @@ -0,0 +1,8 @@ +{ + "source_file": "./raw_volume-zh/volume2/chapter5.tex", + "problem_type": "proof", + "problem": "例9 整数 $a 、 b 、 c$ 使得 $\\frac{a}{b}+\\frac{b}{c}+\\frac{c}{a}$ 和 $\\frac{a}{c}+\\frac{c}{b}+\\frac{b}{a}$ 仍为整数, 求证: $|a|=|b|=|c|$.", + "solution": "证先证一个引理.\n引理设 $p(x)=a_n x^n+a_{n-1} x^{n-1}+\\cdots+a_1 x+a_0$ 是整系数多项式, 若\n$\\frac{p}{q}$ ( $p$ 和 $q$ 是互质的整数) 是它的一个有理根, 则 $p\\left|a_0, q\\right| a_n$.\n事实上,我们有\n$$\n\\begin{aligned}\n& a_n\\left(\\frac{p}{q}\\right)^n+a_{n-1}\\left(\\frac{p}{q}\\right)^{n-1}+\\cdots+a_1\\left(\\frac{p}{q}\\right)+a_0=0, \\\\\n& \\frac{a_n p^n}{q}+a_{n-1} p^{n-1}+\\cdots+a_1 p q^{n-2}+a_0 q^{n-1}=0 .\n\\end{aligned}\n$$\n所以 $q \\mid a_n p^n$,而 $\\left(q, p^n\\right)=1$,所以 $q \\mid a_n$.\n又因为\n$$\na_n p^n+a_{n-1} p^{n-1} q+\\cdots+a_1 p q^{n-1}+a_0 q^n=0 .\n$$\n所以 $p \\mid a_0 q^n$, 而 $\\left(p, q^n\\right)=1$, 从而 $p \\mid a_0$.\n下面我们来证明本题.\n构造一个三次函数\n$$\n\\begin{aligned}\nf(x) & =\\left(x-\\frac{a}{b}\\right)\\left(x-\\frac{b}{c}\\right)\\left(x-\\frac{c}{a}\\right) \\\\\n& =x^3-\\left(\\frac{a}{b}+\\frac{b}{c}+\\frac{c}{a}\\right) x^2+\\left(\\frac{a}{c}+\\frac{c}{b}+\\frac{b}{a}\\right) x-1,\n\\end{aligned}\n$$\n由题设知, $f(x)$ 是一个整系数多项式, 它的三个有理根为 $\\frac{a}{b}, \\frac{b}{c}, \\frac{c}{a}$, 而由引理, $f(x)$ 的有理根只能为 $\\pm 1$ , 从而\n$$\n\\left|\\frac{a}{b}\\right|=\\left|\\frac{b}{c}\\right|=\\left|\\frac{c}{a}\\right|=1,\n$$\n故\n$$\n|a|=|b|=|c| \\text {. }\n$$\n说明本题当然用数论的方法也能解决, 但是我们通过构造一个三次多项式,利用引理来解显得非常巧妙.", + "remark": "", + "figures": [] +} \ No newline at end of file diff --git a/processed_dataset/proof/0884.json b/processed_dataset/proof/0884.json new file mode 100644 index 0000000000000000000000000000000000000000..123f50aefb3b386542819ee04bf3bc218cca70e4 --- /dev/null +++ b/processed_dataset/proof/0884.json @@ -0,0 +1,8 @@ +{ + "source_file": "./raw_volume-zh/volume2/chapter5.tex", + "problem_type": "proof", + "problem": "例11 设 $a_n$ 为下述自然数 $N$ 的个数: $N$ 的各位数字之和为 $n$ 且每位数字只能取 1,3 或 4 . 求证 $a_{2 n}$ 是完全平方数, 这里 $n=1,2, \\cdots$.", + "solution": "证记集 $A$ 为数码仅有 $1,3,4$ 的数的全体, $A_n=\\{N \\in A \\mid N$ 的各位数码之和为 $n\\}$, 则 $\\left|A_n\\right|=a_n$. 欲证 $a_{2 n}$ 是完全平方数.\n再记集 $B$ 为数码仅有 1,2 的数的全体, $B_n=\\{N \\in B \\mid N$ 的各位数码之和为 $n\\}$, 令 $\\left|B_n\\right|=b_n$, 下证 $a_{2 n}=b_n^2$.\n作映射 $f: B \\rightarrow \\mathbf{N}_{+}$, 对于 $N \\in B, f(N)$ 是由 $N$ 按如下法则得到的一个数: 把 $N$ 的数码从左向右看, 凡见到 2 , 把它与后面的一个数相加, 用和代替, 再继续看下去, 直到不能做为止(例如 $f(1221212)=14$ 132, $f(21121221$ ) = $31341)$. 易知 $f$ 是单射.\n于是\n$$\nf\\left(B_{2 n}\\right)=A_{2 n} \\cup A_{2 n-2}^{\\prime} \\text {. }\n$$\n其中, $A_{2 n-2}^{\\prime}=\\left\\{10 k+2 \\mid k \\in A_{2 n-2}\\right\\}$. 所以\n$$\n\\begin{gathered}\nb_{2 n}=a_{2 n}+a_{2 n-2} . \\\\\nb_{2 n}=b_n^2+b_{n-1}^2,\n\\end{gathered}\n$$\n这是因为 $B_{2 n}$ 中的数或是两个 $B_n$ 中的数拼接而成, 或是两个 $B_{n-1}$ 中的数中间放 2 拼接而成.\n所以\n$$\na_{2 n}+a_{2 n-2}=b_n^2+b_{n-1}^2 \\quad(n \\geqslant 2) .\n$$\n因为 $a_2=b_1^2=1$, 由上式便知, 对-一切正整数 $n, a_{2 n}=b_n^2$, 即 $a_{2 n}$ 是完全平方数.", + "remark": "", + "figures": [] +} \ No newline at end of file diff --git a/processed_dataset/proof/0885.json b/processed_dataset/proof/0885.json new file mode 100644 index 0000000000000000000000000000000000000000..0c54c729135f5a47a8b3044e84e654122b5f4199 --- /dev/null +++ b/processed_dataset/proof/0885.json @@ -0,0 +1,8 @@ +{ + "source_file": "./raw_volume-zh/volume2/chapter6.tex", + "problem_type": "proof", + "problem": "例3 设 $f(x)=3 x+2$, 证明: 存在 $m \\in \\mathbf{N}_{+}$, 使 $f^{(100)}(m)$ 能被 2005 整除.", + "solution": "证由 $f(x)=3 x+2$, 知\n$$\nf^{(100)}(x)=3^{100} x+\\left(3^{99}+3^{98}+\\cdots+1\\right) \\times 2,\n$$\n故\n$$\nf^{(100)}(m)=3^{100} \\cdot m+2\\left(3^{99}+3^{98}+\\cdots+1\\right) .\n$$\n由于 $(3,2005)=1$, 故 $\\left(3^{100}, 2005\\right)=1$, 根据 Bezout 定理, 存在 $u 、 v \\in$\n081\n$\\mathbf{Z}$, 使\n$$\n2005 u-3^{100} v=1 .\n$$\n记 $n=2\\left(3^{99}+3^{98}+\\cdots+3+1\\right)$, 那么由 $2005 \\mid\\left(3^{100} v+1\\right)$,\n知\n$$\n2005 \\mid n\\left(3^{100} v+1\\right) \\text {. }\n$$\n此时, 取 $m=n v$, 那么\n$$\n\\begin{gathered}\n2005 \\mid\\left(3^{100} m+n\\right), \\\\\n2005 \\mid f^{(100)}(m)\n\\end{gathered}\n$$\n从而命题得证.\n说明 Bezout 定理是: 若 $x 、 y$ 是两个互质的正整数, 则存在整数 $u 、 v$, 使得 $u x-v y=1$.", + "remark": "", + "figures": [] +} \ No newline at end of file diff --git a/processed_dataset/proof/0886.json b/processed_dataset/proof/0886.json new file mode 100644 index 0000000000000000000000000000000000000000..46a4076631cd48a6f72f078b976676d904a771f7 --- /dev/null +++ b/processed_dataset/proof/0886.json @@ -0,0 +1,8 @@ +{ + "source_file": "./raw_volume-zh/volume2/chapter6.tex", + "problem_type": "proof", + "problem": "例4 设 $n$ 是不小于 3 的正整数, 以 $f(n)$ 表示不是 $n$ 的因数的最小正整数 (例如 $f(12)=5)$. 如果 $f(n) \\geqslant 3$, 又可作 $f(f(n))$. 类似地, 如果 $f(f(n)) \\geqslant 3$, 又可作 $f(f(f(n)))$ 等等.\n如果 $f^{(k)}(n)=2$, 就将 $k$ 称为 $n$ 的 “长度”, 记为 $l_n$. 试对任意 $n \\in \\mathbf{N}_{+}, n \\geqslant 3$, 求 $l_n$, 并证明你的结论.", + "solution": "解:首先证明任意 $n \\in \\mathbf{N}_{+}, n \\geqslant 3, f(n)$ 不会是两个大于等于 2 的互素的数的乘积.\n事实上, 若 $f(n)=p q$, 其中 $p \\geqslant 2, q \\geqslant 2 ,(p, q)=1$. 由 $f$ 的定义, $p$ 、 $q$ 均为 $n$ 的因数, 即 $p|n, q| n$. 由 $(p, q)=1$ 知, $p q \\mid n$, 这便与 $f(n)= p q$ 矛盾.\n由于 $f(n)$ 不可能是两个互素的 ( $\\geqslant 2)$ 数的积, 因而 $f(n)$ 必为 $p^k$ 形式的数,其中 $p$ 是素数, $k \\in \\mathbf{N}_{+}$.\n因此, 对 $n \\in \\mathbf{N}_{+}, n \\geqslant 3$, 或者 $f(n)=2$ (当且仅当 $2 \\nmid n$) 或者 $f(n)=2^k(k \\geqslant 2)$, 此时 $f(f(n))=3$, 即 $l_n=3$. 此外, $f(n)=p^k$ ( $p$ 是奇素数, $k \\geqslant 1$), 这时 $l_n=2$.\n综上所述,\n$$\nl_n= \\begin{cases}1, & \\text { 当 } n \\text { 是奇数时; } \\\\ 3, & \\text { 有大于 } 1 \\text { 的正整数 } k, \\text { 使得对任何 }<2^k \\text { 的素数 } p \\text { 及 } \\\\ & \\text { 满足 } p^j<2^k \\text { 的正整数 } j, p^j \\mid n, \\text { 且 } 2^k \\nmid n \\text { 时 ; } \\\\ 2, & \\text { 其他情况.\n}\\end{cases}\n$$", + "remark": "", + "figures": [] +} \ No newline at end of file diff --git a/processed_dataset/proof/0887.json b/processed_dataset/proof/0887.json new file mode 100644 index 0000000000000000000000000000000000000000..0aa96b9c9ca9ed63e286ab3d2408b5e45c8633dc --- /dev/null +++ b/processed_dataset/proof/0887.json @@ -0,0 +1,8 @@ +{ + "source_file": "./raw_volume-zh/volume2/chapter6.tex", + "problem_type": "proof", + "problem": "例5 对任意 $k \\in \\mathbf{N}_{+}$, 令 $f(k)$ 表示 $k$ 的各位数字的和的平方, 且对于 $n \\geqslant$ 2 , 令 $f^{(n)}(k)=f\\left(f^{(n-1)}(k)\\right)$, 求 $f^{(2005)}\\left(2^{2002}\\right)$ 的值.", + "solution": "解:设正整数 $a$ 的位数为 $m$, 则当 $a \\leqslant b$ 时, $m \\leqslant 1+\\lg b$, 因此\n$$\nf(a) \\leqslant 9^2 m^2 \\leqslant 81(1+\\lg b)^2<\\left(4 \\log _2 16 b\\right)^2 .\n$$\n由此得\n$$\n\\begin{gathered}\nf\\left(2^{2002}\\right)<\\left(4 \\log _2\\left(16 \\times 2^{2002}\\right)\\right)^2=2^4 \\times 2006^2<2^4 \\times 2048^2=2^{26}, \\\\\nf^{(2)}\\left(2^{2002}\\right)<\\left(4 \\log _2\\left(16 \\times 2^{26}\\right)\\right)^2=(4 \\times 30)^2=14400 . \\\\\n\\text { 而 } \\quad(9+9+9+9)^2<14400<(1+9+9+9+9)^2,\n\\end{gathered}\n$$\n故 $f^{(2)}\\left(2^{2002}\\right)$ 的各位数字之和 $\\leqslant 4 \\times 9$, 因此\n$$\n\\begin{gathered}\nf^{(3)}\\left(2^{2002}\\right)<36^2=1296, f^{(4)}\\left(2^{2002}\\right)<(9+9+9)^2=729, \\\\\nf^{(5)}\\left(2^{2002}\\right)<(6+9+9)^2=24^2 .\n\\end{gathered}\n$$\n另一方面, 因为 $f(k) \\equiv k^2(\\bmod 9)$, 故\n$$\n\\begin{gathered}\nf\\left(2^{2002}\\right) \\equiv\\left(2^{2002}\\right)^2 \\equiv\\left(2^4\\right)^2 \\times\\left(\\left(2^3\\right)^{666}\\right)^2 \\equiv\\left(2^4\\right)^2 \\equiv 4(\\bmod 9), \\\\\nf^{(2)}\\left(2^{2002}\\right) \\equiv 4^2 \\equiv-2(\\bmod 9), \\\\\nf^{(3)}\\left(2^{2002}\\right) \\equiv(-2)^2 \\equiv 4(\\bmod 9) .\n\\end{gathered}\n$$\n由数学归纳法容易证明\n$$\nf^{(n)}\\left(2^{2002}\\right) \\equiv \\begin{cases}4(\\bmod 9), & \\text { 当 } 2 \\nmid n \\text { 时; } \\\\ -2(\\bmod 9), & \\text { 当 } 2 \\mid n \\text { 时.\n}\\end{cases}\n$$\n因此, 由 $f^{(5)}\\left(2^{2002}\\right)<24^2, f^{(5)}\\left(2^{2002}\\right) \\equiv 4(\\bmod 9)$, 且 $f^{(5)}\\left(2^{2002}\\right)$ 应为完全平方数知, $f^{(5)}\\left(2^{2002}\\right) \\in\\{4,49,121,256,400\\}$, 所以 $f^{(6)}\\left(2^{2002}\\right) \\in\\{16$, $169\\}, f^{(7)}\\left(2^{2002}\\right) \\in\\{49,256\\}, f^{(8)}\\left(2^{2002}\\right)=169, f^{(9)}\\left(2^{2002}\\right)=256$, $f^{(10)}\\left(2^{2002}\\right)=169, \\cdots$, 即当 $n \\geqslant 8$ 时, 有\n$$\nf^{(n)}\\left(2^{2002}\\right)=\\left\\{\\begin{array}{l}\n169, \\text { 当 } n \\text { 为偶数时; } \\\\\n256, \\text { 当 } n \\text { 为奇数时.\n}\n\\end{array}\\right.\n$$\n因此, $f^{(2005)}\\left(2^{2002}\\right)=256$.", + "remark": "", + "figures": [] +} \ No newline at end of file diff --git a/processed_dataset/proof/0888.json b/processed_dataset/proof/0888.json new file mode 100644 index 0000000000000000000000000000000000000000..736b7e75db4d8ffe902b969ec3207416a5fa9e19 --- /dev/null +++ b/processed_dataset/proof/0888.json @@ -0,0 +1,8 @@ +{ + "source_file": "./raw_volume-zh/volume2/chapter6.tex", + "problem_type": "proof", + "problem": "例6 设 $f(x)=a x+b$, 求 $f^{(n)}(x)$.", + "solution": "解:$f(x)=a x+b$,\n$$\n\\begin{aligned}\nf^{(2)}(x) & =f(f(x))=a(a x+b)+b=a^2 x+a b+b, \\\\\nf^{(3)}(x) & =f\\left(f^{(2)}(x)\\right)=a\\left(a^2 x+a b+b\\right)+b \\\\\n& =a^3 x+a^2 b+a b+b,\n\\end{aligned}\n$$\n由此猜测\n$$\nf^{(n)}(x)=a^n x+a^{n-1} b+a^{n-2} b+\\cdots+a b+b .\n$$\n下面用数学归纳法证明.\n(1) 当 $n=1$ 时,命题成立.\n(2) 假设 $f^{(k)}(x)=a^k x+a^{k-1} b+\\cdots+a b+b$ 成立, 那么\n$$\n\\begin{aligned}\nf^{(k+1)}(x) & =f\\left(f^{(k)}(x)\\right)=a\\left(a^k x+a^{k-1} b+\\cdots+a b+b\\right)+b \\\\\n& =a^{k+1} x+a^k b+\\cdots+a b+b,\n\\end{aligned}\n$$\n即 $n=k+1$ 时,命题亦成立.\n由(1)、(2)就证得了\n$$\nf^{(n)}(x)=a^n x+a^{n-1} b+\\cdots+a b+b .\n$$", + "remark": "", + "figures": [] +} \ No newline at end of file diff --git a/processed_dataset/proof/0889.json b/processed_dataset/proof/0889.json new file mode 100644 index 0000000000000000000000000000000000000000..a0411b41dbc5f6198f672446e2bc97a30461c29a --- /dev/null +++ b/processed_dataset/proof/0889.json @@ -0,0 +1,8 @@ +{ + "source_file": "./raw_volume-zh/volume2/chapter6.tex", + "problem_type": "proof", + "problem": "例7 已知 $f(x)=\\frac{x}{a+b x}$, 求 $f^{(n)}(x)$.", + "solution": "解:$f(x)=\\frac{x}{a+b x}$,\n$$\n\\begin{aligned}\nf^{(2)}(x) & =f(f(x))=-\\frac{\\frac{x}{a+b x}}{a+b \\cdot \\frac{x}{a+b x}}=\\frac{x}{a^2+b x(1+a)}, \\\\\nf^{(3)}(x) & =f\\left(f^{(2)}(x)\\right)=\\frac{\\frac{x}{a^2+b x(1+a)}}{a+b \\cdot \\frac{x}{a^2+b x(1+a)}} \\\\\n& =\\frac{x}{a^3+b x\\left(1+a+a^2\\right)},\n\\end{aligned}\n$$\n因此猜测\n$$\nf^{(i)}(x)=\\frac{x}{a^n+b x\\left(1+a+\\cdots+a^{n-1}\\right)}=\\frac{x}{a^n+b x \\cdot \\frac{1-a^n}{1-a}} .\n$$\n用数学归纳法是容易证明的.\n事实上,\n$$\n\\begin{aligned}\nf^{(n+1)}(x) & =f\\left(f^{(n)}(x)\\right)=\\frac{\\frac{x}{a^n+b x \\cdot \\frac{1-a^n}{1-a}}}{a+b \\cdot \\frac{x}{a^n+b x \\cdot \\frac{1-a^n}{1-a}}} \\\\\n& =\\frac{x}{a^{n+1}+b x \\frac{1-a^{n+1}}{1-a}} .\n\\end{aligned}\n$$\n于是命题获证.", + "remark": "", + "figures": [] +} \ No newline at end of file diff --git a/processed_dataset/proof/0890.json b/processed_dataset/proof/0890.json new file mode 100644 index 0000000000000000000000000000000000000000..a5c97ec063c1e13152aedd61c2b30956b2fb6238 --- /dev/null +++ b/processed_dataset/proof/0890.json @@ -0,0 +1,8 @@ +{ + "source_file": "./raw_volume-zh/volume2/chapter6.tex", + "problem_type": "proof", + "problem": "例9 设 $f(x)=\\sqrt{2+x}$, 求 $f^{(n)}(x)$.", + "solution": "解:设 $a_0=x, a_n=f^{(n)}(x)$, 则 $a_n=\\sqrt{2+a_{n-1}}$.\n(1) 若 $|x| \\leqslant 2$, 则令 $x=2 \\cos \\theta$, 取 $\\theta=\\arccos \\frac{x}{2}$, 则\n$$\n\\begin{aligned}\n& a_0=2 \\cos \\theta, a_1=\\sqrt{2+2 \\cos \\theta}=2 \\cos \\frac{\\theta}{2}, a_2=2 \\cos \\frac{\\theta}{2^2}, \\cdots, \\\\\n& a_n=2 \\cos \\frac{\\theta}{2^n}=2 \\cos \\left(\\frac{1}{2^n} \\arccos \\frac{x}{2}\\right) \\text { (用数学归纳法易证). }\n\\end{aligned}\n$$\n因此 $f^{(n)}(x)=2 \\cos \\left(\\frac{1}{2^n} \\arccos \\frac{x}{2}\\right)(|x| \\leqslant 2)$.\n(2) 若 $|x|>2$, 则令 $x=t+\\frac{1}{t}$, 取 $t=\\frac{1}{2}\\left(x+\\sqrt{x^2-4}\\right)$, 则 $a_0=t+ \\frac{1}{t}, a_1=t^{\\frac{1}{2}}+t^{-\\frac{1}{2}}, a_2=t_{2^2}^{\\frac{1}{2}}+t^{-\\frac{1}{2^2}}$, 由数学归纳法易得\n$$\na_n=t^{\\frac{1}{n}}+t^{-\\frac{1}{2^n}}\n$$\n故 $\\quad f^{(n)}(x)=\\frac{1}{t 2^n}+t^{-\\frac{1}{2^n}}$\n$$\n=\\left(\\frac{x+\\sqrt{x^2-4}}{2}\\right)^{\\frac{1}{2^n}}+\\left(\\frac{x-\\sqrt{x^2-4}}{2}\\right)^{\\frac{1}{2^n}}(|x|>2) .\n$$\n综上所述,\n$$\nf^{(n)}(x)= \\begin{cases}2 \\cos \\left(\\frac{1}{2^n} \\arccos \\frac{x}{2}\\right), & \\text { 当 }|x| \\leqslant 2 \\text { 时; } \\\\ \\left(\\frac{x+\\sqrt{x^2-4}}{2}-\\right)^{\\frac{1}{2^n}}+\\left(\\frac{x-\\sqrt{x^2-4}}{2}\\right)^{\\frac{1}{2^n}}, & \\text { 当 }|x|>2 \\text { 时.\n}\\end{cases}\n$$", + "remark": "", + "figures": [] +} \ No newline at end of file diff --git a/processed_dataset/proof/0891.json b/processed_dataset/proof/0891.json new file mode 100644 index 0000000000000000000000000000000000000000..c81f5f88aef05e5e7b2d991d048c3c1a7d26fc31 --- /dev/null +++ b/processed_dataset/proof/0891.json @@ -0,0 +1,8 @@ +{ + "source_file": "./raw_volume-zh/volume2/chapter6.tex", + "problem_type": "proof", + "problem": "例11 设 $f(x)=\\frac{x}{1+a x}$, 求 $f^{(n)}(x)$.", + "solution": "解:令 $g(x)=x+a, \\varphi(x)=\\frac{1}{x}$, 则 $\\varphi^{-1}(x)=\\frac{1}{x}$. 容易验证 $f(x)= \\varphi^{-1}(g(\\varphi(x)))$, 即 $f \\stackrel{\\varphi}{\\sim} g$. 所以\n$$\n\\begin{aligned}\nf^{(n)}(x) & =\\varphi^{-1}\\left(g^{(n)}(\\varphi(x))\\right) \\\\\n& =\\varphi^{-1}(\\varphi(x)+n a) \\\\\n& =\\frac{1}{\\frac{1}{x}+n a}=\\frac{x}{1+n a x} .\n\\end{aligned}\n$$", + "remark": "", + "figures": [] +} \ No newline at end of file diff --git a/processed_dataset/proof/0892.json b/processed_dataset/proof/0892.json new file mode 100644 index 0000000000000000000000000000000000000000..8877a78f1ee9807f659e99ce4cbd34845bde583a --- /dev/null +++ b/processed_dataset/proof/0892.json @@ -0,0 +1,8 @@ +{ + "source_file": "./raw_volume-zh/volume2/chapter6.tex", + "problem_type": "proof", + "problem": "例14 设 $p(x)=x^2-2$, 试证对任意正整数 $n$,方程 $p^{(n)}(x)=x$ 的根全是相异实根.", + "solution": "证先看 $p^{(n)}(x)=x$ 且 $x \\in[-2,2]$ 时的情形.\n当 $x \\in[-2,2]$ 时, 令 $x=2 \\cos t, t=\\arccos \\frac{x}{2}$. 设 $g(x)=2 x, t=\\varphi(x)= \\arccos \\frac{x}{2}$, 则 $\\varphi^{-1}(x)=2 \\cos x$, 于是\n$$\n\\begin{aligned}\n\\varphi^{-1}(g(\\varphi(x))) & =\\varphi^{-1}\\left(g\\left(\\arccos \\frac{x}{2}\\right)\\right) \\\\\n& =\\varphi^{-1}\\left(2 \\arccos \\frac{x}{2}\\right) \\\\\n& =2 \\cos \\left(2 \\arccos \\frac{x}{2}\\right) \\\\\n& =2\\left(2 \\cdot\\left(\\frac{x}{2}\\right)^2-1\\right) \\\\\n& =x^2-2=p(x) .\n\\end{aligned}\n$$\n所以 $p(x) \\sim g(x)$. 于是\n$$\n\\begin{aligned}\np^{(n)}(x) & =\\varphi^{-1}\\left(g^{(n)}(\\varphi(x))\\right) \\\\\n& =2 \\cos \\left(2^n \\arccos \\frac{x}{2}\\right) .\n\\end{aligned}\n$$\n于是方程 $p^{(n)}(x)=x,(x \\in[-2,2])$ 变为\n$$\n2 \\cos \\left(2^n \\arccos \\frac{x}{2}\\right)=x,\n$$\n即\n$$\n2 \\cos \\left(2^n t\\right)=2 \\cos t(t \\in[0, \\pi]) .\n$$\n故\n$$\n\\cos \\left(2^n t\\right)=\\cos t\n$$\n解方程, 得 $t=\\frac{2 l \\pi}{2^n-1}$ 或 $t=\\frac{2 m \\pi}{2^n+1} \\quad(m, l \\in \\mathbf{Z})$.\n所以方程 $p^{(n)}(x)=x$ 在 $[-2,2]$ 中有 $2^n$ 个不同的实根 $x=2 \\cos \\frac{2 l \\pi}{2^n-1}$,\n$l=0,1,2, \\cdots, 2^{n-1}-1, x=2 \\cos \\frac{2 m \\pi}{2^n+1}, m=1,2, \\cdots, 2^{n-1}$.\n由于 $p^{(n)}(x)$ 是 $2^n$ 次多项式, $p^{(n)}(x)=x$ 至多有 $2^n$ 个实根, 故知方程 $p^{(n)}(x)=x$ 的所有根都是实数且各不相同.", + "remark": "", + "figures": [] +} \ No newline at end of file diff --git a/processed_dataset/proof/0893.json b/processed_dataset/proof/0893.json new file mode 100644 index 0000000000000000000000000000000000000000..407f6d30075ce5551f97c122f506ea4727e92c2f --- /dev/null +++ b/processed_dataset/proof/0893.json @@ -0,0 +1,8 @@ +{ + "source_file": "./raw_volume-zh/volume2/chapter6.tex", + "problem_type": "proof", + "problem": "例19 设 $f(x)=\\frac{a x+b}{c x+d}(c \\neq 0, a d \\neq b c)$, 求 $f^{(n)}(x)$.", + "solution": "解:分两种情况.\n(1) 若 $f(x)=x$ 有两个不相等的不动点 $x_1, x_2$, 则取 $g(x)=\\frac{a-c x_1}{a-c x_2} x$, $\\varphi(x)=\\frac{x-x_1}{x-x_2}$, 于是 $f(x)=\\varphi^{-1}(g(\\varphi(x)))$.\n(2) 若 $f(x)=x$ 有唯一的不动点 $x_0$, 则取\n$$\ng(x)=x+\\frac{2 c}{a+d}, \\varphi(x)=\\frac{1}{x-x_0} .\n$$\n同样有\n$$\nf(x)=\\varphi^{-i}(g(\\varphi(x))) .\n$$\n下面给出 (1)、(2) 的证明.\n对于 (1), 由 $\\frac{a x+b}{c x+d}=x$, 得\n$$\nc x^2+(d-a) x-b=0 .\n$$\n故\n$$\nx_1+x_2=\\frac{a-d}{c}, x_1 x_2=-\\frac{b}{c} .\n$$\n又 $\\varphi(x)=\\frac{x-x_1}{x-x_2}$, 所以 $\\varphi^{-1}(x)=\\frac{x_1-x_2 x}{1-x}$, 于是\n$$\n\\begin{aligned}\n\\varphi^{-1}(g(\\varphi(x))) & =\\varphi^{-1}\\left(g\\left(\\frac{x-x_1}{x-x_2}\\right)\\right) \\\\\n& =\\varphi^{-1}\\left(\\frac{a-c x_1}{a-c x_2} \\cdot \\frac{x-x_1}{x-x_2}\\right) \\\\\n& =\\frac{x_1-\\frac{a-c x_1}{a-c x_2} \\cdot \\frac{x-x_1}{x-x_2}}{1-\\frac{a-c x_1}{a-c x_2} \\cdot \\frac{x-x_1}{x-x_2}} \\cdot x_2 \\\\\n& =\\frac{a x-x_1 x_2 c}{c x+a-c\\left(x_1+x_2\\right)} \\\\\n& =\\frac{a x+b}{c x+d}=f(x) .\n\\end{aligned}\n$$\n因为\n$$\n2 x_0=\\frac{a-d}{c},\n$$\n所以\n$$\n\\begin{aligned}\n\\varphi^{-1}(g(\\varphi(x))) & =\\varphi^{-1}\\left(\\frac{1}{x-x_0}+\\frac{2 c}{a+d}\\right) \\\\\n& =\\frac{1}{\\frac{1}{x-x_0}+\\frac{2 c}{a+d}}+x_0 \\\\\n& =\\frac{\\left(a+d+2 c x_0\\right) x-2 c x_0^2}{2 c+\\left(a+d-2 c x_0\\right)}\n\\end{aligned}\n$$\n从而\n$$\n\\varphi^{-1}(g(\\varphi(x)))=\\frac{2 a x+2 b}{2 c x+2 d}=f(x) .\n$$\n下面来求 $f^{(n)}(x)$.\n$$\n\\begin{aligned}\n& \\text { 对于(1), }f^{(n)}(x)=\\varphi^{-1}\\left(g^{(n)}(\\varphi(x))\\right) \\\\\n& =\\frac{x_1-\\left(\\frac{a-c x_1}{a-c x_2}\\right)^n \\cdot \\frac{x-x_1}{x-x_2} \\cdot x_2}{1-\\left(\\frac{a-c x_1}{a-c x_2}\\right)^n \\cdot \\frac{x-x_1}{x-x_2}} \\\\\n& =\\frac{x_1\\left(a-c x_2\\right)^n\\left(x-x_2\\right)-x_2\\left(a-c x_1\\right)^n\\left(x-x_1\\right)}{\\left(a-c x_2\\right)^n\\left(x-x_2\\right)-\\left(a-c x_1\\right)^n\\left(x-x_1\\right)} \\text {. } \\\\\n\\end{aligned}\n$$\n$$\n\\begin{aligned}\n& \\text { 对于(2), } f^{(n)}(x)=\\varphi^{-1}\\left(g^{(n)}(\\varphi(x))\\right) \\\\\n& =\\frac{1}{\\frac{1}{x-x_0}+\\frac{2 n c}{a+d}}+x_0 \\\\\n& =\\frac{\\left(a+d+2 n c x_0\\right) x-2 n c x_0^2}{2 n c x+a+d-2 n c x_0} .\\\\\n\\end{aligned}\n$$", + "remark": "", + "figures": [] +} \ No newline at end of file diff --git a/processed_dataset/proof/0894.json b/processed_dataset/proof/0894.json new file mode 100644 index 0000000000000000000000000000000000000000..635c5ce3cfbd806fbb10be84e8dd82992fcfc4fb --- /dev/null +++ b/processed_dataset/proof/0894.json @@ -0,0 +1,8 @@ +{ + "source_file": "./raw_volume-zh/volume2/chapter6.tex", + "problem_type": "proof", + "problem": "例20 $M$ 是形如 $f(x)=a x+b(a, b \\in \\mathbf{R})$ 的实变量 $x$ 的非零函数集, 且 $M$ 具有下列性质:\n(1) 若 $f(x), g(x) \\in M$, 则 $g(f(x)) \\in M$;\n(2) 若 $f \\in M$, 且 $f(x)=a x+b$, 则反函数 $f^{-1} \\in M$, 这里 $f^{-1}(x)=\\frac{x-b}{a} (a \\neq 0)$;\n(3) 对 $M$ 中每一个 $f$, 存在一个 $x_j \\in \\mathbf{R}$, 使 $f\\left(x_j\\right)=x_j$.\n求证: 总存在一个 $k \\in \\mathbf{R}$, 对所有 $f \\in M$, 均有 $f(k)=k$.", + "solution": "证条件 (3) 表明, 对每一个 $f \\in M$, 都有一个不动点 $x_j$, 使 $f\\left(x_j\\right)=x_j$. 现要证集 $M$ 中所有函数 $f$, 必有一个公共不动点 $k$.\n设 $f(x)=a x+b$ 的不动点为 $x_j$, 即 $a x_j+b=x_j$.\n若 $a \\neq 1$, 则 $x_j=\\frac{-b}{a-1}$ 是唯一不动点;\n若 $a=1$ 且 $b=0$, 则任何实数均是 $f$ 的不动点;\n若 $a=1$ 且 $b \\neq 0$, 则 $f$ 无不动点, 此时 $f \\bar{\\in} M$.\n因此, 只需证明: 当 $f(x)=a x+b(a \\neq 1) \\in M$ 时, 必有 $\\frac{-b}{a-1}$ 是一个常数.\n这时, 取 $k=\\frac{-b}{a-1}$ 知命题获证.\n首先证明,若 $g_1(x)=a x+b_1 \\in M, g_2(x)=a x+b_2 \\in M$, 则 $b_1=b_2$. 事实上,由性质 (1)、(2)有\n$$\ng_2^{-1}\\left(g_1(x)\\right)=\\frac{\\left(a x+b_1\\right)-b_2}{a}=x+\\frac{b_1-b_2}{a} \\in M .\n$$\n由性质 (3) 知, $g_2^{-1}\\left(g_1(x)\\right)$ 存在不动点, 故 $b_1=b_2$.\n其次, 对形如 $h(x)=x+b$ 的函数, 当 $b \\neq 0$ 时, $h(x) \\bar{\\in} M$; 当 $b=0$ 时, 对任意 $k \\in \\mathbf{R}$ 有 $h(k)=k$. 故只需考虑 $M$ 中形如 $f(x)=a x+b(a \\neq 1)$ 的函数.\n设 $f_1(x)=a_1 x+b_1\\left(a_1 \\neq 1\\right) \\in M, f_2(x)=a_2 x+b_2\\left(a_2 \\neq 1\\right) \\in M$. 那么由性质 (1), 得\n$$\n\\begin{aligned}\nf_1\\left(f_2(x)\\right) & =a_1\\left(a_2 x+b_2\\right)+b_1 \\\\\n& =a_1 a_2 x+a_1 b_2+b_1 \\in M, \\\\\nf_2\\left(f_1(x)\\right) & =a_2\\left(a_1 x+b_1\\right)+b_2 \\\\\n& =a_1 a_2 x+a_2 b_1+b_2 \\in M .\n\\end{aligned}\n$$\n则\n$$\na_1 b_2+b_1=a_2 b_1+b_2 \\text {. }\n$$\n变形, 得\n$$\n\\frac{-b_1}{a_1-1}=\\frac{-b_2}{a_2-1} \\text {. }\n$$\n此式表明, 对任意 $f(x)=a x+b(a \\neq 1) \\in M, \\frac{-b}{a-1}$ 是常数.\n取 $k= \\frac{-b}{a-1}$, 则 $f(k)=f\\left(\\frac{-b}{a-1}\\right)=a\\left(\\frac{-b}{a-1}\\right)+b=\\frac{-b}{a-1}=k$, 即知题中结论成立.", + "remark": "", + "figures": [] +} \ No newline at end of file diff --git a/processed_dataset/proof/0895.json b/processed_dataset/proof/0895.json new file mode 100644 index 0000000000000000000000000000000000000000..3a3d30abf900e9cadd00c889e2e0b74dddae4273 --- /dev/null +++ b/processed_dataset/proof/0895.json @@ -0,0 +1,8 @@ +{ + "source_file": "./raw_volume-zh/volume2/chapter6.tex", + "problem_type": "proof", + "problem": "例21 设 $f: \\mathbf{N}_{+} \\rightarrow \\mathbf{N}_{+}$, 且对每个 $n \\in \\mathbf{N}_{+}$, 均有\n$$\nf(n+1)>f(f(n)) .\n$$\n求证: 每个正整数均为 $f$ 的不动点.", + "solution": "证结论其实就是要证对任意正整数 $n, f(n)=n$. 先证一个辅助命题: 对任意两个正整数 $m, n$, 只要 $m \\geqslant n$, 就有 $f(m) \\geqslant n$.\n对 $n$ 用数学归纳法.\n$n=1$ 时,显然 $f(m) \\geqslant 1$.\n设 $n=k$ 时,命题成立,当 $n=k+1$ 时,任取一个正整数 $m \\geqslant k+1$, 要证明的是 $f(m) \\geqslant k+1$.\n事实上, 由于 $m \\geqslant k+1$, 故 $m-1 \\geqslant k$, 由归纳假设, 知 $f(m-1) \\geqslant k$, 再用一次归纳假设, 又有\n$$\nf(f(m-1)) \\geqslant k .\n$$\n由题设\n$$\nf(m)>f(f(m-1)),\n$$\n所以\n$$\nf(m)>k,\n$$\n从而\n$$\nf(m) \\geqslant k+1 \\text {. }\n$$\n于是命题获证.\n在辅助命题中令 $m=n$, 就得 $f(n) \\geqslant n$. 再用一次辅助命题, 有\n$f(f(n)) \\geqslant f(n)$, 又有\n$$\nf(n+1)>f(f(n)) \\geqslant f(n),\n$$\n即 $f(n)$ 是严格递增的.\n于是由 $f(n+1)>f(f(n))$ 可得 $n+1>f(n)$, 综合两方面, 有\n$$\n\\begin{gathered}\nn \\leqslant f(n)0$. 又由 $F(0), 0 \\in M$ 及 $F$ 的单调增性, 有 $F(F(0)) \\geqslant F(0)$. 令 $D=\\{x \\in M \\mid x \\leqslant F(x)\\}$. 则 $F(0) \\in D$.\n由 $D \\subset M$, 而 $M$ 是有限集, 故 $D$ 是有限集.\n设 $p \\in D$ 为 $D$ 中的最大数,则 $p \\leqslant F(p)$.\n再由 $F$ 的单调增性, 有 $F(p) \\leqslant F(F(p))$, 从而 $F(p) \\in D$. 由 $p$ 是 $D$ 中的最大数, 有 $F(p)=g(f(p))=p$.", + "remark": "", + "figures": [] +} \ No newline at end of file diff --git a/processed_dataset/proof/0897.json b/processed_dataset/proof/0897.json new file mode 100644 index 0000000000000000000000000000000000000000..131b5e8457d67083300fa29b91762b491d47d111 --- /dev/null +++ b/processed_dataset/proof/0897.json @@ -0,0 +1,10 @@ +{ + "source_file": "./raw_volume-zh/volume2/chapter6.tex", + "problem_type": "proof", + "problem": "例23 将一张地图按比例缩小之后放人原地图中, 证明: 有且仅有一点代表了两张地图的同一位置 (不动点).", + "solution": "证如图 ()所示, 先把两张地图放人一个复平面上.\n记 $A, B$ 两点对应的复数为 $0,1, A^{\\prime}$, $B^{\\prime}$ 两点对应的复数为 $z_1, z_2$, 那么 $\\left|z_1-z_2\\right|<1$.\n在小地图中任取一点 $P$ (对应复数 $z$ ), 连结 $P A, P B$. 再在小地图中另找一点 $Q$ (对应复数 $f(z))$, 使 $\\triangle Q A^{\\prime} B^{\\prime} \\backsim \\triangle P A B$.\n若 $f(z)=z$, 则点 $P$ 就是不动点.\n否则, 再计算 $f^{(2)}(z), \\cdots$. 下面证明 $: \\lim _{n \\rightarrow \\infty} f^{(n)}(z)$ 一定存在.\n因为\n$$\n\\triangle Q A^{\\prime} B^{\\prime} \\backsim \\triangle P A B\n$$\n所以\n$$\n\\frac{f(z)-z_1}{z_2-z_1}=\\frac{z-O}{1-O}\n$$\n$$\n\\begin{aligned}\n& f(z)=\\left(z_2-z_1\\right) z+z_1, \\\\\n& f^{(2)}(z)=\\left(z_2-z_1\\right) f(z)+z_1=\\left(z_2-z_1\\right)^2 z+\\left(z_2-z_1\\right) z_1+z_1, \\\\\n& \\cdots \\cdots \\\\\n& f^{(n)}(z)=\\left(z_2-z_1\\right)^n z+\\frac{1-\\left(z_2-z_1\\right)^n}{1+z_1-z_2} z_1 .\n\\end{aligned}\n$$\n因为 $\\left|z_2-z_1\\right|<1$, 故 $\\lim _{n \\rightarrow \\infty} f^{(n)}(z)=\\frac{z_1}{1+z_1-z_2}$.\n因此, $\\frac{z_1}{1+z_1-z_2}$ 对应的点即为所求.\n而不可能有两个不动点, 否则两张地图一样大.\n证毕.", + "remark": "", + "figures": [ + "./images/volume2/figures/fig-c6e23.png" + ] +} \ No newline at end of file diff --git a/processed_dataset/proof/0898.json b/processed_dataset/proof/0898.json new file mode 100644 index 0000000000000000000000000000000000000000..fd6e42d3369ab4ca9e7d2eca00ed142ae6fbe154 --- /dev/null +++ b/processed_dataset/proof/0898.json @@ -0,0 +1,8 @@ +{ + "source_file": "./raw_volume-zh/volume2/chapter6.tex", + "problem_type": "proof", + "problem": "例24 设圆 $O$ 中有一个任意内接 $\\triangle A B C$, 取 $\\overparen{A B}, \\overparen{B C}, \\overparen{C A}$ 的中点分别记为 $C_1, A_1, B_1$, 得到一个新的内接 $\\triangle A_1 B_1 C_1$; 取 $\\overparen{A_1 B_1}, \\overparen{B_1 C_1}, \\overparen{C_1 A_1}$ 的中点分别记为 $C_2, A_2, B_2$, 又得一内接 $\\triangle A_2 B_2 C_2$; 如此继续, 得一组内接三角形 $A_n B_n C_n$, 求证:\n$$\n\\lim _{n \\rightarrow \\infty} \\angle A_n=\\lim _{n \\rightarrow \\infty} \\angle B_n=\\lim _{n \\rightarrow \\infty} \\angle C_n=\\frac{\\pi}{3} .\n$$", + "solution": "证显然, $\\angle A_1=\\frac{1}{2}(\\angle B+\\angle C)=\\frac{1}{2}(\\pi-\\angle A)$. 记 $f(x)=\\frac{1}{2}(\\pi-x)$, $x_0=\\angle A$, 则有\n$$\n\\begin{aligned}\n& \\angle A_1=f\\left(x_0\\right), \\\\\n& \\angle A_2=f\\left(f\\left(x_0\\right)\\right)=f^{(2)}\\left(x_0\\right), \\\\\n& \\cdots \\cdots \\\\\n& \\angle A_n=f^{(n)}\\left(x_0\\right) .\n\\end{aligned}\n$$\n而 $f(x)=\\frac{1}{2}(\\pi-x)$ 从而\n$$\nf^{(n)}\\left(x_0\\right)=\\left(-\\frac{1}{2}\\right)^n x_0+\\frac{1-\\left(-\\frac{1}{2}\\right)^n}{1-\\left(-\\frac{1}{2}\\right)} \\cdot \\frac{\\pi}{2} .\n$$\n故 $\\lim _{n \\rightarrow \\infty} \\angle A_n=\\lim _{n \\rightarrow \\infty}\\left(\\left(-\\frac{1}{2}\\right)^n \\angle A+\\frac{1-\\left(-\\frac{1}{2}\\right)^n}{1-\\left(-\\frac{1}{2}\\right)} \\cdot \\frac{\\pi}{2}\\right)=\\frac{\\pi}{3}$.\n同理\n$$\n\\lim _{n \\rightarrow \\infty} \\angle B_n=\\lim _{n \\rightarrow \\infty} \\angle C_n=\\frac{\\pi}{3} .\n$$", + "remark": "", + "figures": [] +} \ No newline at end of file diff --git a/processed_dataset/proof/0899.json b/processed_dataset/proof/0899.json new file mode 100644 index 0000000000000000000000000000000000000000..093775bf57d4fbdc7ecaf75ee4892048996d9973 --- /dev/null +++ b/processed_dataset/proof/0899.json @@ -0,0 +1,8 @@ +{ + "source_file": "./raw_volume-zh/volume2/chapter6.tex", + "problem_type": "proof", + "problem": "例25 设 $f(x)=\\sin x$, 对于 $x_0 \\in\\left(0, \\frac{\\pi}{2}\\right]$, 给出 $f^{(n)}\\left(x_0\\right)$ 的估计式.", + "solution": "解我们证明\n$$\n\\lim _{n \\rightarrow \\infty} \\frac{f^{(n)}\\left(x_0\\right)}{\\frac{1}{\\sqrt{n}}}=\\sqrt{3}\n$$\n这表明 $f^{(n)}\\left(x_0\\right)$ 与 $\\frac{1}{\\sqrt{n}}$ 是同价无穷小量, 并且不依赖于初始值 $x_0$.\n令 $\\varphi(x)=\\varphi(x, c)=\\frac{1}{\\sqrt{\\frac{1}{x^2}+\\frac{1}{3 c^2}}}$, 其中 $c>0$ 是参数.\n利用泰勒公式展开, 知\n$$\n\\begin{gathered}\n\\sin x=x-\\frac{x^3}{6}+o\\left(x^3\\right) \\\\\n\\varphi(x)=\\varphi(x, c)=x-\\frac{1}{6 c^2} x^3+o\\left(x^3\\right) .\n\\end{gathered}\n$$\n故 $\\forall \\varepsilon>0, \\exists \\delta>0$, 当 $0\\frac{1-\\varepsilon}{\\sqrt{\\frac{1}{x^2}+\\frac{n}{3}}} .\n\\end{aligned}\n$$\n同理,\n$$\n\\begin{aligned}\n\\varphi^{(n)}(x, 1+\\varepsilon) & =\\frac{1+\\varepsilon}{\\sqrt{\\frac{(1+\\varepsilon)^2}{x^2}+\\frac{n}{3}}} \\\\\n& <\\frac{1+\\varepsilon}{\\sqrt{\\frac{1}{x^2}+\\frac{n}{3}}} .\n\\end{aligned}\n$$\n所以 $\\quad\\left|\\sqrt{n} f^{(n)}(x)-\\frac{1}{\\sqrt{\\frac{1}{n x^2}+\\frac{1}{3}}}\\right|<\\frac{\\varepsilon}{\\sqrt{\\frac{1}{n x^2}+\\frac{1}{3}}}$.\n由于 $\\lim _{n \\rightarrow \\infty} f^{(n)}\\left(x_0\\right)=0$ (0 是 $\\sin x$ 的唯一不动点), 故 $\\forall \\varepsilon>0, \\exists m \\in \\mathbf{N}_{+}$, 使\n$$\n0m$, 令 $f^{(n)}\\left(x_0\\right)=y_0$, 于是有\n$$\n\\begin{aligned}\n& \\left|\\sqrt{n-m} f^{(n)}\\left(x_0\\right)-\\frac{1}{\\sqrt{\\frac{1}{(n-m) y_0^2}+\\frac{1}{3}}}\\right| \\\\\n& =\\left|\\sqrt{n-m} f^{(n-m)}\\left(y_0\\right)-\\frac{1}{\\sqrt{\\frac{1}{(n-m) y_0^2}+\\frac{1}{3}}}\\right| \\\\\n& <\\frac{\\varepsilon}{\\sqrt{\\frac{1}{(n-m) y_0^2}+\\frac{1}{3}}} \\text {. } \\\\\n&\n\\end{aligned}\n$$\n令 $n \\rightarrow \\infty$, 有\n$$\n\\left|\\lim _{n \\rightarrow \\infty} \\sqrt{n-m} f^{(n)}\\left(x_0\\right)-\\sqrt{3}\\right| \\leqslant \\sqrt{3} \\varepsilon .\n$$\n又 $\\left|\\sqrt{n} f^{(n)}\\left(x_0\\right)-\\sqrt{3}\\right|$\n$$\n\\begin{aligned}\n& =\\left|(\\sqrt{n}-\\sqrt{n-m}) f^{(n)}\\left(x_0\\right)+\\sqrt{n-m} f^{(n)}\\left(x_0\\right)-\\sqrt{3}\\right| \\\\\n& \\leqslant \\frac{m}{\\sqrt{n}+\\sqrt{n-m}} f^{(n)}\\left(x_0\\right)+\\left|\\sqrt{n-m} f^{(n)}\\left(x_0\\right)-\\sqrt{3}\\right| \\\\\n& \\leqslant \\frac{m f^{(n)}\\left(x_0\\right)}{2 \\sqrt{n--} m}+\\sqrt{3} \\varepsilon+o(0) .\n\\end{aligned}\n$$\n故 $\\left|\\lim _{n \\rightarrow \\infty} \\sqrt{n} f^{(n)}\\left(x_0\\right)-\\sqrt{3}\\right| \\leqslant \\sqrt{3} \\varepsilon$.\n由 $\\varepsilon$ 的任意性,得\n$$\n\\lim _{n \\rightarrow \\infty} \\sqrt{n} f^{(n)}\\left(x_0\\right)=\\sqrt{3} .\n$$\n上面这个例子表明,当 $f$ 的 $n$ 次迭代比较复杂时,我们可用较简单的函数去逼近它, 从而取得较好的结果.", + "remark": "", + "figures": [] +} \ No newline at end of file diff --git a/processed_dataset/proof/0900.json b/processed_dataset/proof/0900.json new file mode 100644 index 0000000000000000000000000000000000000000..3445337ccfcf7331b5bfe237ba0e5927f2693d62 --- /dev/null +++ b/processed_dataset/proof/0900.json @@ -0,0 +1,8 @@ +{ + "source_file": "./raw_volume-zh/volume2/chapter7.tex", + "problem_type": "proof", + "problem": "例3 证明: 恰有一个定义在所有非零实数上的函数 $f$, 满足条件:\n(1) 对所有非零实数 $x, f(x)=x f\\left(\\frac{1}{x}\\right)$;\n(2) 对所有 $x \\neq-y$ 的非零实数对 $(x, y)$, 有\n$$\nf(x)+f(y)=1+f(x+y) .\n$$", + "solution": "证 $f(x)=x+1$ 显然适合 (1)、(2).下证唯一性.\n即设 $f(x)$ 满足 (1)、 (2), 那么 $f(x)=x+1$.\n在(2)中, 令 $y=1$, 得\n$$\nf(x)+f(1)=1+f(x+1)(x \\neq-1, x \\neq 0) . \\quad\\quad <1>\n$$\n在 (2) 中, 以- $x$ 代换 $x, x+1$ 代换 $y$, 得\n$$\nf(-x)+f(x+1)=1+f(1)(x \\neq-1, x \\neq 0) . \\quad\\quad <2>\n$$\n综合<1>、<2>, 得\n$$\nf(x)+f(-x)=2(x \\neq-1, x \\neq 0) . \\quad\\quad <3>\n$$\n<3>在 $x=1$ 时成立,所以在 $x=-1$ 时也成立.\n由(1)及<3>, 当 $x \\neq 0$ 时,\n$$\n\\begin{aligned}\nf(x) & =x f\\left(\\frac{1}{x}\\right)=x\\left[2-f\\left(-\\frac{1}{x}\\right)\\right] \\\\\n& =2 x+\\left[-x f\\left(-\\frac{1}{x}\\right)\\right] \\\\\n& =2 x+f(-x) .\n\\end{aligned}\n$$\n所以\n$$\nf(x)-f(-x)=2 x . \\quad\\quad <4>\n$$\n从<3>、<4>中消去 $f(-x)$, 得 $f(x)=x+1$.", + "remark": "", + "figures": [] +} \ No newline at end of file diff --git a/processed_dataset/proof/0901.json b/processed_dataset/proof/0901.json new file mode 100644 index 0000000000000000000000000000000000000000..cc2b47362486422ee185f5813073db5f1079a448 --- /dev/null +++ b/processed_dataset/proof/0901.json @@ -0,0 +1,8 @@ +{ + "source_file": "./raw_volume-zh/volume2/chapter7.tex", + "problem_type": "proof", + "problem": "例6 求所有的函数 $f: \\mathbf{R} \\rightarrow \\mathbf{R}$ 使得\n$$\nf(f(x)+y)=f\\left(x^2-y\\right)+4 f(x) y . \\quad\\quad (1)\n$$\n对所有 $x, y \\in \\mathbf{R}$ 成立.", + "solution": "解:易见 $f(x) \\equiv 0$ 或 $f(x)=x^2$ 皆为上述方程 (1) 的解.\n我们来证明它们是唯一的解.\n设对某个 $a, f(a) \\neq a^2$.\n在(1)中令 $y=\\frac{x^2-f(x)}{2}$, 得\n$$\nf(x) \\cdot\\left(x^2-f(x)\\right)=0 .\\quad\\quad (2)\n$$\n由于 $f(a) \\neq a^2$, 故只能 $f(a)=0$, 并且可见 $a \\neq 0$ (否则 $a^2=0=f(a)$ 与 $a$ 的定义相违).\n于是我们得到, 对任何 $x$, 要么 $f(x)=0$, 要么 $f(x)=x^2$.\n在(2)中令 $x=0$, 有 $f(0)=0$.\n在(1)中令 $x=0$, 有\n$$\nf(y)=f(-y) .\n$$\n在(1)中令 $x=a$, 并用 $-y$ 替换 $y$, 得\n$$\nf\\left(a^2+y\\right)=f(-y)=f(y) .\n$$\n从上式可见 $f$ 以 $a^2$ 为周期, 进而我们有\n$$\nf(f(x))=f\\left(f(x)+a^2\\right)=f\\left(x^2-a^2\\right)+4 f(x) a^2 .\n$$\n在(1)中令 $y=0$, 有\n$$\nf(f(x))=f\\left(x^2\\right) .\n$$\n利用 $f(x)$ 的周期性, 得 $f(x) \\cdot a^2=0$.\n所以 $f(x)=0$ (因为 $a \\neq 0$ ).\n也就是说, 若 $f(x) \\neq x^2$, 则必有 $f(x) \\equiv 0$ 成立.\n因此结论成立.", + "remark": "", + "figures": [] +} \ No newline at end of file diff --git a/processed_dataset/proof/0902.json b/processed_dataset/proof/0902.json new file mode 100644 index 0000000000000000000000000000000000000000..5018342b7103400ac9df9f154de645656c2e92a6 --- /dev/null +++ b/processed_dataset/proof/0902.json @@ -0,0 +1,8 @@ +{ + "source_file": "./raw_volume-zh/volume2/chapter7.tex", + "problem_type": "proof", + "problem": "例8 求所有满足下列条件的函数 $f: \\mathbf{N}_{+} \\rightarrow \\mathbf{N}_{+}$, 使得\n(1) $f(2)=2$;\n(2) $f(m m)=f(m) \\cdot f(n)$ 对所有 $m, n \\in \\mathbf{N}_{+}$成立;\n(3) 若 $m\n$$\n以及 $f(9)\n由<1>和<2>两个不等式,有\n$$\nf(3) f(5)<2 f(9)<4 f(5),\n$$\n故 $f(3)<4$, 而 $2=f(2)0$, 这是不难证的.\n事实上,\n$$\n\\begin{aligned}\nf(x) & =f(|x|)=F\\left(\\sqrt{\\frac{x^2}{2}+\\frac{x^2}{2}}\\right) \\\\\n& =\\left(f\\left(\\frac{x}{\\sqrt{2}}\\right)\\right)^2 \\geqslant 0 .\n\\end{aligned}\n$$\n又 $f(x) \\neq 0$, 故 $f(x)>0$.", + "remark": "", + "figures": [] +} \ No newline at end of file diff --git a/processed_dataset/proof/0907.json b/processed_dataset/proof/0907.json new file mode 100644 index 0000000000000000000000000000000000000000..fc7ba83e3d423dd2af083ae23f6d1296388fa23d --- /dev/null +++ b/processed_dataset/proof/0907.json @@ -0,0 +1,8 @@ +{ + "source_file": "./raw_volume-zh/volume2/chapter7.tex", + "problem_type": "proof", + "problem": "例20 函数 $f: \\mathbf{R} \\rightarrow \\mathbf{R}$ 满足 $f(1)=1$, 且对任意 $a, b \\in \\mathbf{R}$, 有\n$$\nf(a+b)=f(a)+f(b) ; \\quad\\quad (1)\n$$\n对任意 $x \\neq 0$, 有\n$$\nf(x) f\\left(\\frac{1}{x}\\right)=1 . \\quad\\quad (2)\n$$\n求证: $f(x)=x$.", + "solution": "证由题设, 得\n$$\n1=f(1)=f(1+0)=f(1)+f(0),\n$$\n所以\n$$\nf(0)=0 \\text {. }\n$$\n又因为\n$$\nf(x)+f(-x)=f(0)=0,\n$$\n所以 $f(x)$ 是奇函数.\n下面只需证明: 当 $x>0$ 时, $f(x)=x$ 即可.\n由(1)式,用数学归纳法易证得: 对任意正整数 $n$ 及正实数 $x$, 有\n$$\nf(n x)=n f(x) .\n$$\n令 $x=1$, 便有\n$$\nf(n)=n .\n$$\n令 $x=\\frac{1}{n}$, 得\n$$\nf\\left(\\frac{1}{n}\\right)=\\frac{1}{n} f(1)=\\frac{1}{n}\n$$\n于是对于正有理数 $\\frac{m}{n}$, 有\n$$\nf\\left(\\frac{m}{n}\\right)=m f\\left(\\frac{1}{n}\\right)=\\frac{m}{n} .\n$$\n最后, 讨论 $x$ 是无理数的情况, 为此先证 $f(x)$ 在 $x=0$ 处连续.\n对于任意实数 $y>2$, 必存在正实数 $x$, 满足 $x+\\frac{1}{x}=y$, 于是\n$$\n|f(y)|=\\left|f(x)+f\\left(\\frac{1}{x}\\right)\\right| \\geqslant 2 \\sqrt{f(x) f\\left(\\frac{1}{x}\\right)} .\n$$\n因此, 当 $y<\\frac{1}{2}$ 时,\n$$\n|f(y)|=\\left|\\left(f\\left(\\frac{1}{y}\\right)\\right)^{-1}\\right| \\leqslant \\frac{1}{2} .\n$$\n任给 $\\varepsilon>0$, 总存在正整数 $N=\\left[\\frac{1}{2 \\varepsilon}\\right]+1$, 当 $x<\\frac{1}{2} \\frac{1}{N}$ 时, 有\n$$\n|f(x)|=\\frac{1}{N}\\left|f\\left(N_x\\right)\\right| \\leqslant \\frac{1}{2 N}<\\varepsilon .\n$$\n所以\n$$\n\\lim _{x \\rightarrow 0} f(x)=0=f(0) . \\quad\\quad (3)\n$$\n对于正无理数 $x$, 存在一有理数数列 $\\left\\{x_m\\right\\}, x_m \\rightarrow x(m \\rightarrow \\infty)$, 于是\n$$\nf(x)=f\\left(x_m\\right)+f\\left(x-x_m\\right)=x_m+f\\left(x-x_m\\right) .\n$$\n令 $m \\rightarrow \\infty$, 利用(3)式, 得\n$$\nf(x)=\\lim _{m \\rightarrow \\infty} x_m=x . \n$$", + "remark": "", + "figures": [] +} \ No newline at end of file diff --git a/processed_dataset/proof/0908.json b/processed_dataset/proof/0908.json new file mode 100644 index 0000000000000000000000000000000000000000..94d10717ee6023499484e9d4fd36b911e632ac97 --- /dev/null +++ b/processed_dataset/proof/0908.json @@ -0,0 +1,8 @@ +{ + "source_file": "./raw_volume-zh/volume2/chapter7.tex", + "problem_type": "proof", + "problem": "例22 求二元一次函数方程:\n$$\nf(x+y)=f(x)+f(y) . \\quad\\quad (1)\n$$\n在 $(-\\infty,+\\infty)$ 内的一切单调函数的解.", + "solution": "解:.\n同上题可证: 对一切有理数 $r$, 有 $f(r)=a r$, 其中 $a=f(1)$. 设 $\\lambda$ 是任意无理数,则必存在有理数 $r_1 、 r_2$, 使得 $r_1<\\lambdas_2\\right), f\\left(z+s_1\\right)=f\\left(z+s_2\\right)$ 成立, 即 $f$ 是以 $s_1-s_2$ 为周期的周期函数.\n由于 $S$ 至少含有两个元素, 故存在 $\\lambda>0$, 使得 $f$ 以 $\\lambda$ 为周期.\n于是\n$$\n\\left(f(y+\\lambda)+(y+\\lambda)^n\\right)-\\left(f(y)+y^n\\right)=(y+\\lambda)^n-y^n\n$$\n也是 $f$ 的周期.\n这是关于 $y$ 的 $n-1$ 次多项式, 因为首项系数 $n \\lambda>0$, 所以该多项式可以取到充分大的所有实数, 即充分大的实数均为 $f$ 的周期, 于是所有实数均为 $f$ 的周期, 这样 $f$ 是常数函数.\n由 (1) 式即知 $f(x)=0$, 易验证 $f(x)=$ 0 满足要求.\n综上所述, 所有满足要求的函数为 $f(x)=-x^n$ 和 $f(x)=0$.", + "remark": "", + "figures": [] +} \ No newline at end of file diff --git a/processed_dataset/proof/0913.json b/processed_dataset/proof/0913.json new file mode 100644 index 0000000000000000000000000000000000000000..1d17d28def36bb948315d376604389b38527e9a1 --- /dev/null +++ b/processed_dataset/proof/0913.json @@ -0,0 +1,8 @@ +{ + "source_file": "./raw_volume-zh/volume2/chapter7.tex", + "problem_type": "proof", + "problem": "例33 设 $f: \\mathbf{R} \\rightarrow \\mathbf{R}$ 是一个定义在实数集上的实值函数, 满足对所有实数 $x 、 y$, 都有\n$$\nf(x+y) \\leqslant y f(x)+f(f(x)), \\quad\\quad (1)\n$$\n证明: 对所有实数 $x \\leqslant 0$, 有 $f(x)=0$.", + "solution": "证令 $y=t-x$, 则 (1) 式可写为\n$$\nf(t) \\leqslant t f(x)-x f(x)+f(f(x)) . \\quad\\quad (2)\n$$\n在(2)式中分别令 $t=f(a), x=b$ 和 $t=f(b), x=a$, 可得\n$$\n\\begin{aligned}\n& f(f(a))-f(f(b)) \\leqslant f(a) f(b)-b f(b), \\\\\n& f(f(b))-f(f(a)) \\leqslant f(a) f(b)-a f(a) .\n\\end{aligned}\n$$\n把上面两式相加得\n$$\n2 f(a) f(b) \\geqslant a f(a)+b f(b) .\n$$\n令 $b=2 f(a)$, 得 $2 f(a) f(b) \\geqslant a f(a)+2 f(a) f(b)$, 即 $a f(a) \\leqslant 0$. 所以\n$$\nf(a) \\geqslant 0 \\text {, 当 } a<0 \\text { 时.\n} \\quad\\quad (3)\n$$\n假设存在某个实数 $x$, 使得 $f(x)>0$. 由 (2) 式, 对每个 $t<\\frac{x f(x)-f(f(x))}{f(x)}$, 有 $f(t)<0$, 这与 (3) 矛盾.\n所以, 对所有实数 $x$\n$$\nf(x) \\leqslant 0, \\quad\\quad (4)\n$$\n结合 (3)知, 对所有实数 $x<0$, 有 $f(x)=0$.\n在(2)中令 $t=x<0$, 得 $f(x) \\leqslant f(f(x))$, 故 $0 \\leqslant f(0)$, 结合 (4), 知 $f(0)=0$.\n综上可知, 对任意 $x \\leqslant 0$, 都有 $f(x)=0$.", + "remark": "", + "figures": [] +} \ No newline at end of file diff --git a/processed_dataset/proof/0914.json b/processed_dataset/proof/0914.json new file mode 100644 index 0000000000000000000000000000000000000000..7b5200508a1f7de865b618ca20df7c994e33e326 --- /dev/null +++ b/processed_dataset/proof/0914.json @@ -0,0 +1,8 @@ +{ + "source_file": "./raw_volume-zh/volume2/chapter7.tex", + "problem_type": "proof", + "problem": "例34 对每一个正整数 $n$, 求具有下述性质的最大常数 $C_n$ : 对任意 $n$ 个定义在闭区间 $[0,1]$ 上的实值函数 $f_1(x), f_2(x), \\cdots, f_n(x)$, 都存在实数 $x_1, x_2, \\cdots, x_n$, 满足 $0 \\leqslant x_i \\leqslant 1$, 且 $\\mid f_1\\left(x_1\\right)+f_2\\left(x_2\\right)+\\cdots+f_n\\left(x_n\\right)-x_1 x_2 \\cdots x_n \\mid \\geqslant C_n$.", + "solution": "解:所求的最大常数 $C_n=\\frac{n-1}{2 n}$.\n一方面, 取 $x_1=x_2=\\cdots=x_n=1$, 得题中不等式的左式 $=\\mid \\sum_{i=1}^n f_i(1)-1 \\mid$, 取 $x_1=x_2=\\cdots=x_n=0$, 得不等式左式 $=\\left|\\sum_{i=1}^n f_i(0)\\right|$, 取 $x_i=0, x_j=1(j \\neq i)$, 得不等式左式 $=\\left|\\sum_{j \\neq i} f_j(1)+f_i(0)\\right|$. 由三角形不等式可知\n$$\n\\begin{aligned}\n& (n-1)\\left|\\sum_{i=1}^n f_i(1)-1\\right|+\\sum_{i=1}^n\\left|\\sum_{j \\neq i} f_j(1)+f_i(0)\\right|+\\left|\\sum_{i=1}^n f_i(0)\\right| \\\\\n\\geqslant & \\left|(n-1)\\left(\\sum_{i=1}^n f_i(1)-1\\right)-\\sum_{i=1}^n\\left(\\sum_{j \\neq i} f_j(1)+f_i(0)\\right)+\\sum_{i=1}^n f_i(0)\\right|=n-1 .\n\\end{aligned}\n$$\n故 $\\left|\\sum_{i=1}^n f_i(1)-1\\right|,\\left|\\sum_{i=1}^n f_i(0)\\right|,\\left|\\sum_{j \\neq i} f_j(1)+f_i(0)\\right|(i=1,2, \\cdots, n)$ 中必有一个数不小于 $\\frac{n-1}{2 n}$, 从而, $C_n \\geqslant \\frac{n-1}{2 n}$.\n另一方面, 令 $f_i(x)=\\frac{x}{n}-\\frac{n-1}{2 n^2}, i=1,2, \\cdots, n$, 我们证明: 对任意实数 $x_1, x_2, \\cdots, x_n \\in[0,1]$, 都有 $\\mid f_1\\left(x_1\\right)+f_2\\left(x_2\\right)+\\cdots+f_n\\left(x_n\\right)-x_1 x_2 \\cdots x_n \\mid \\leqslant \\frac{n-1}{2 n}$.\n为此, 只需证明: $1-n \\leqslant n x_1 \\cdots x_n-\\sum_{i=1}^n x_i \\leqslant 0$.\n左边不等式等价于 $(n-1) x_1 \\cdots x_n+\\left(x_1-1\\right)\\left(x_2 \\cdots x_n-1\\right)+\\cdots+\\left(x_{n-1}-\\right.$ 1) $\\left(x_n-1\\right) \\geqslant 0$, 此式中每一个加项都不小于 0 , 故成立.\n右边不等式等价于 $\\sum_{i=1}^n x_i-n x_1 \\cdots x_n \\geqslant 0 \\Leftrightarrow \\sum_{i=1}^n x_i\\left(1-\\frac{x_1 \\cdots x_n}{x_i}\\right) \\geqslant 0$, 同上可知亦成立.\n所以, $C_n \\leqslant \\frac{n-1}{2 n}$.\n综上所述, 所求的最大常数 $C_n=\\frac{n-1}{2 n}$.", + "remark": "", + "figures": [] +} \ No newline at end of file diff --git a/processed_dataset/proof/0915.json b/processed_dataset/proof/0915.json new file mode 100644 index 0000000000000000000000000000000000000000..c8a201f1dc81f9631ce6a481aedb2baccc6dbb65 --- /dev/null +++ b/processed_dataset/proof/0915.json @@ -0,0 +1,8 @@ +{ + "source_file": "./raw_volume-zh/volume2/exercise1.tex", + "problem_type": "proof", + "problem": "问题1 设 $X$ 和 $Y$ 是两个集合, $B \\subset A \\subset X$. 举例说明, 存在着映射 $f: X \\rightarrow Y$, 使得\n$$\nf(A-B) \\neq f(A)-f(B) .\n$$\n并证明, 当 $f$ 是单射时, $f(A-B)=-f(A)--f(B)$.", + "solution": "取 $A=\\{1,2,3\\}, B=\\{1,2\\}, X=Y=\\mathbf{N}_{+}$. 则 $f: 1 \\mapsto 4,2 \\mapsto 5$, $3 \\mapsto 4$, 则 $f(A-B)=\\{4\\}, f(A)-f(B)=\\{4,5\\}-\\{4,5\\}=\\varnothing$, 故 $f(A- B) \\neq f(A)-f(B)$. 当 $f$ 是单射时, 若 $y \\in f(A)-f(B)$, 则 $y \\in f(A)$, 且 $y \\bar{\\epsilon} f(B)$, 故存在 $x \\in A$, 使得 $f(x)=y$, 由于 $f$ 是单射, 故 $x \\bar{\\epsilon} B$, 从而 $x \\in A-B$, 于是 $y \\in f(A-B)$, 所以 $f(A)-f(B) \\subseteq f(A-B)$; 若 $y \\in f(A- B)$, 则存在 $x \\in A-B$, 使得 $y=f(x)$, 所以 $y \\in f(A)$, 但 $x \\bar{\\in} B$, 且 $f$ 是单射, 故 $y \\bar{\\epsilon} f(B)$, 从而 $y \\in f(A)-f(B)$, 即 $f(A-B) \\subseteq f(A)-f(B)$. 故当 $f$ 为单射时,有 $f(A-B)=f(A)-f(B)$.", + "remark": "", + "figures": [] +} \ No newline at end of file diff --git a/processed_dataset/proof/0916.json b/processed_dataset/proof/0916.json new file mode 100644 index 0000000000000000000000000000000000000000..0ecee0033a4561ab01f88f662ab0feb835f00984 --- /dev/null +++ b/processed_dataset/proof/0916.json @@ -0,0 +1,8 @@ +{ + "source_file": "./raw_volume-zh/volume2/exercise2.tex", + "problem_type": "proof", + "problem": "问题2 已知 $f\\left(\\log _a x\\right)=\\frac{a\\left(x^2-1\\right)}{x\\left(a^2-1\\right)}(a>0, a \\neq 1, x>0)$, 判断 $f(x)$ 的单调性, 并证明你的结论.", + "solution": "设 $t=\\log _a x(x>0)$, 则 $f\\left(t_1\\right)-f\\left(t_2\\right)=\\frac{a\\left(x_1^2-1\\right)}{x_1\\left(a^2-1\\right)}-\\frac{a\\left(x_2^2-1\\right)}{x_2\\left(a^2-1\\right)}= \\frac{a\\left(x_1-x_2\\right)\\left(x_1 x_2+1\\right)}{x_1 x_2\\left(a^2-1\\right)}$.\n(1) 当 $a>1$ 时, 若 $t_1x_2>0, f\\left(t_1\\right))所示, 由图象可知 (也可通过函数计算), $f\\left(\\frac{2}{3}\\right)=\\frac{2}{3}$, 所以 $f_{15}\\left(\\frac{2}{3}\\right)=\\frac{2}{3}$. 故 $\\frac{2}{3} \\in B$. 设 $C=\\left\\{0, \\frac{1}{2}, 1\\right\\}$. 由于 $f(0)==\\frac{1}{2}$, $f\\left(\\frac{1}{2}\\right)=1, f(1)=0$, 则对于 $c \\in C$, 有 $f_3(c)=c$, 所以 $f_{15}(c)=c$. 故 $C \\subset B$. 综上, 得 $\\left\\{\\frac{2}{15}, \\frac{19}{30}, \\frac{11}{15}, \\frac{8}{15}, \\frac{14}{15}, \\frac{2}{3}, 0, \\frac{1}{2}, 1\\right\\} \\subset B$, 故 $B$ 中至少含有 9 个元素.", + "remark": "", + "figures": [ + "./images/volume2/figures/fig-c2p8.png" + ] +} \ No newline at end of file diff --git a/processed_dataset/proof/0919.json b/processed_dataset/proof/0919.json new file mode 100644 index 0000000000000000000000000000000000000000..ba1e5c1149f7449fa9f018f65a3001fe2a29df8b --- /dev/null +++ b/processed_dataset/proof/0919.json @@ -0,0 +1,8 @@ +{ + "source_file": "./raw_volume-zh/volume2/exercise2.tex", + "problem_type": "proof", + "problem": "问题9 设 $f(x)$ 是定义在 $\\mathbf{R}$ 上的偶函数, 且 $f(x)$ 在 $(-\\infty, 0]$ 上是增函数, $f (2 a^2+ a+1)0,3 a^2-2 a+1>0$, 由 $f\\left(2 a^2+a+1\\right)3 a^2-2 a+1$, 解不等式得 $00$.\n证明: (1) $f(x)$ 是奇函数;\n(2) $f(x)$ 是周期函数, 并求出它的一个周期;\n(3) $f(x)$ 在 $(0,4 a)$ 内为减函数.", + "solution": "(1) 对定义域中的 $x$, 由题设知在定义域中存在 $x_1 、 x_2$, 使 $x=x_1- x_2, f\\left(x_1\\right) \\neq f\\left(x_2\\right)$, 则 $f(x)=f\\left(x_1-x_2\\right)=\\frac{f\\left(x_1\\right) f\\left(x_2\\right)+1}{f\\left(x_2\\right)-f\\left(x_1\\right)}=-f\\left(x_2-\\right. \\left.x_1\\right)=-f(-x)$, 所以 $f(x)$ 为奇函数.\n(2) 因 $f(a)=1$, 所以 $f(-a)= -f(a)=-1$. 于是 $f(-2 a)=f(-a-a)=\\frac{f(-a) f(a)+1}{f(a)-f(-a)}=0$. 若 $f(x) \\neq 0$ , 则 $f(x+2 a)=f[x-(-2 a)]=\\frac{f(x) f(-2 a)+1}{f(-2 a)-f(x)}=\\frac{1}{-f(x)}, f(x+ 4 a)=f[x+2 a+(2 a)]=\\frac{1}{-f(x+2 a)}=f(x)$. 若 $f(x)=0$, 则 $f(x+a)= f[x-(-a)]=\\frac{f(x) f(-a)+1}{f(-a)-f(x)}=-1, f(x+3 a)=f[(x+a)+2 a]= \\frac{1}{-f(x+a)}=1, f(x+4 a)=f[(x+3 a)-(-a)]=\\frac{f(x+3 a) f(-a)+1}{f(-a)-f(x+3 a)}=0$ . 仍有 $f(x+4 a)=f(x)$. 所以, $f(x)$ 为周期函数, $4 a$ 是它的一个周期.\n(3) 先证在 $(0,2 a]$ 内 $f(x)$ 为减函数.\n事实上, 设 $00, f\\left(x_2\\right) \\geqslant 0$ (当 $x_2=2 a$ 时, $f\\left(x_2\\right)=-f(-2 a)= 0), \\frac{f\\left(x_2\\right) f\\left(x_1\\right)+1}{f\\left(x_1\\right)-f\\left(x_2\\right)}=f\\left(x_2-x_1\\right)>0$. 所以 $f\\left(x_1\\right)>f\\left(x_2\\right)$. 当 $2 af\\left(x_2-2 a\\right)>0$, 于是 $f(x)=f[(x-2 a)+2 a]=-\\frac{1}{f(x-2 a)}, f\\left(x_1\\right)-f\\left(x_2\\right)=-\\frac{1}{f\\left(x_1-2 a\\right)}+ \\frac{1}{f\\left(x_2-2 a\\right)}>0$, 即在 $(2 a, 4 a)$ 内, $f(x)$ 也是减函数.\n从而命题得证.", + "remark": "", + "figures": [] +} \ No newline at end of file diff --git a/processed_dataset/proof/0921.json b/processed_dataset/proof/0921.json new file mode 100644 index 0000000000000000000000000000000000000000..d112e1b454fca660dfe3a36dac9bc5790e942c45 --- /dev/null +++ b/processed_dataset/proof/0921.json @@ -0,0 +1,8 @@ +{ + "source_file": "./raw_volume-zh/volume2/exercise3.tex", + "problem_type": "proof", + "problem": "问题12 求证: $\\sin ^2 x+\\frac{4}{\\sin ^2 x} \\geqslant 5$.", + "solution": "令 $t=\\sin ^2 x$, 则 $00$. 求证: 方程 $a x^2+b x+c=0$ 有一根 $x_0$ 满足 $00$ 时, 令 $f(x)=a x^2+b x+c$, 则 $f\\left(\\frac{m}{m+1}\\right)=a\\left(\\frac{m}{m+1}\\right)^2 +b\\left(\\frac{m}{m+1}\\right)+c=a\\left(\\frac{m}{m+1}\\right)^2-\\frac{a m}{m+2}=-\\frac{a m}{(m+1)^2(m+2)}<0$. 若 $c>0$, 因 $f(0)=c>0$, 故必有一根 $x_0$ 满足 $00$, 故必有一根 $x_0$ 满足 $0< \\frac{m}{m+1}0$, 则对于所有实数 $x$, 有 $f(x) \\geqslant 0$.", + "solution": "用反证法.\n若 $\\Delta=a^2-4 b>0$, 则由 $a^2-4 b$ 是整数知, $a^2-4 b \\geqslant 1$. 设方程 $x^2+a x+b=0$ 的两实根为 $x_1 、 x_2$, 则 $\\left|x_1-x_2\\right|= \\sqrt{\\left(x_1+x_2\\right)^2-4 x_1 x_2}=\\sqrt{a^2-4 b} \\geqslant 1$, 从而在 $x_1$ 与 $x_2$ 之间一定存在一个整数 $x_0$, 对于这个整数 $x_0$, 有 $f\\left(x_0\\right) \\leqslant 0$, 与题设矛盾.\n所以 $\\Delta \\leqslant 0$, 从而对一切实数 $x$, 都有 $f(x) \\geqslant 0$.", + "remark": "", + "figures": [] +} \ No newline at end of file diff --git a/processed_dataset/proof/0924.json b/processed_dataset/proof/0924.json new file mode 100644 index 0000000000000000000000000000000000000000..30993ffa87e24af7d36bb89afded6c809fc30f22 --- /dev/null +++ b/processed_dataset/proof/0924.json @@ -0,0 +1,8 @@ +{ + "source_file": "./raw_volume-zh/volume2/exercise4.tex", + "problem_type": "proof", + "problem": "问题12 设 $n(\\geqslant 2)$ 是正整数, $x_i \\in[0,2], i=1,2, \\cdots, n$. 证明: 二次函数 $f(x)=n x^2-2\\left(\\sum_{i=1}^n x_i\\right) x+\\sum_{i=1}^n x_i^2$ 的最小值不超过 $n$.", + "solution": "由题设知 $f(x)=\\left(x-x_1\\right)^2+\\left(x-x_2\\right)^2+\\cdots+\\left(x-x_n\\right)^2$. 由于 $x_i \\in[0,2], i=1,2, \\cdots, n$, 所以 $0 \\leqslant\\left|1-x_i\\right| \\leqslant 1$, $0 \\leqslant\\left(1-x_i\\right)^2 \\leqslant 1$. 于是 $f(1)=\\left(1-x_1\\right)^2+\\left(1-x_2\\right)^2+\\cdots+\\left(1-x_n\\right)^2 \\leqslant n$. 所以, $f_{\\text {min }}(x) \\leqslant f(1) \\leqslant n$.", + "remark": "", + "figures": [] +} \ No newline at end of file diff --git a/processed_dataset/proof/0925.json b/processed_dataset/proof/0925.json new file mode 100644 index 0000000000000000000000000000000000000000..04285ed74b87bc6eff1705c226bee4e8f944f9bd --- /dev/null +++ b/processed_dataset/proof/0925.json @@ -0,0 +1,8 @@ +{ + "source_file": "./raw_volume-zh/volume2/exercise4.tex", + "problem_type": "proof", + "problem": "问题16 关于 $x$ 的一元二次方程 $2 x^2-t x-2=0$ 的两个实根为 $\\alpha 、 \\beta(\\alpha<\\beta)$.\n(1) 若 $x_1 、 x_2$ 为区间 $[\\alpha, \\beta]$ 上的两个不同的点,求证:\n$$\n4 x_1 x_2-t\\left(x_1+x_2\\right)-4<0 ;\n$$\n(2) 设 $f(x)=\\frac{4 x-t}{x^2+1}, f(x)$ 在区间 $[\\alpha, \\beta]$ 上的最大值和最小值分别记为 $f_{\\max }$ 和 $f_{\\min }, g(t)=f_{\\text {max }}-f_{\\text {min }}$, 求 $g(t)$ 的最小值.", + "solution": "(1) 不妨设 $\\alpha \\leqslant x_10$, 所以, $f(x)$ 在 $[\\alpha, \\beta]$ 上是增函数, 故 $g(t)=f(\\beta)-f(\\alpha)=\\frac{8}{\\sqrt{t^2+16}+t}+\\frac{8}{\\sqrt{t^2+16}-t}=\\sqrt{t^2+16} \\geqslant 4$, 当 $t=$ 0 时等号成立.\n故 $g(t)$ 的最小值为 4 .", + "remark": "", + "figures": [] +} \ No newline at end of file diff --git a/processed_dataset/proof/0926.json b/processed_dataset/proof/0926.json new file mode 100644 index 0000000000000000000000000000000000000000..09f2bb412e45755b62890de9fc5e04773e1206fa --- /dev/null +++ b/processed_dataset/proof/0926.json @@ -0,0 +1,8 @@ +{ + "source_file": "./raw_volume-zh/volume2/exercise5.tex", + "problem_type": "proof", + "problem": "问题1 设 $x, y, z \\in(0,1)$, 求证:\n$$\nx(1-y)+y(1-z)+z(1-x)<1 .\n$$", + "solution": "设 $f(x)=(1-y-z) x+y+z-y z-1$. 把 $y 、 z$ 看作常数, 则 $f(x)$ 是关于 $x$ 的一次函数.\n因为 $f(0)=y+z-y z-1=-(y-1)(z-1)<0$, $f(1)=(1-y-z)+y+z-y z-1=-y z<0$, 所以, 对于 $0a_2^2+\\cdots+a_n^2$, 求证:\n$$\n\\begin{aligned}\n& \\left(a_1^2-a_2^2-\\cdots-a_n^2\\right)\\left(b_1^2-b_2^2-\\cdots-b_n^2\\right) \\\\\n\\leqslant & \\left(a_1 b_1-a_2 b_2-\\cdots-a_n b_n\\right)^2 .\n\\end{aligned}\n$$", + "solution": "若 $\\frac{b_1}{a_1}, \\frac{b_2}{a_2}, \\cdots, \\frac{b_n}{a_n}$ 都相等, 记它们等于 $k$, 则 $b_i=k a_i, i=1,2, \\cdots, n$, 代入原不等式知等式成立.\n若这 $n$ 个比值不全相等, 不妨设 $\\frac{b_1}{a_1} \\neq \\frac{b_2}{a_2}$, 则 $\\frac{b_1}{a_1} a_2- b_2 \\neq 0$. 构造二次函数 $f(x)=\\left(a_1^2-a_2^2-\\cdots-a_n^2\\right) x^2-2\\left(a_1 b_1-a_2 b_2-\\cdots-\\right. \\left.a_n b_n\\right) x+\\left(b_1^2-b_2^2-\\cdots-b_n^2\\right)=\\left(a_1 x-b_1\\right)^2-\\left(a_2 x-b_2\\right)^2-\\cdots-\\left(a_n x-b_n\\right)^2$, $f(x)$ 的二次项系数为正, 且当 $x=\\frac{b_1}{a_1}$ 时, 有 $f\\left(\\frac{b_1}{a_1}\\right)<0$. 因此 $f(x)$ 与 $x$ 轴有交点, 其判别式 $\\Delta \\geqslant 0$, 故 $\\left(a_1^2-a_2^2-\\cdots-a_n^2\\right)\\left(b_1^2-b_2^2-\\cdots-b_n^2\\right) \\leqslant\\left(a_1 b_1-a_2 b_2-\\cdots-a_n b_n\\right)^2$.", + "remark": "", + "figures": [] +} \ No newline at end of file diff --git a/processed_dataset/proof/0929.json b/processed_dataset/proof/0929.json new file mode 100644 index 0000000000000000000000000000000000000000..6a8661a052973f46e6b77ee8a71c15c09edcb3bc --- /dev/null +++ b/processed_dataset/proof/0929.json @@ -0,0 +1,8 @@ +{ + "source_file": "./raw_volume-zh/volume2/exercise5.tex", + "problem_type": "proof", + "problem": "问题6. 已知 $a 、 b$ 为不全为 0 的实数, 求证: 方程\n$$\n3 a x^2+2 b x-(a+b)=0\n$$\n在 $(0,1)$ 内至少有一个实根.", + "solution": "若 $a=0$, 则 $b \\neq 0$, 此时方程的根为 $x=\\frac{1}{2}$, 满足题意.\n当 $a \\neq 0$ 时, 令 $f(x)=3 a x^2+2 b x-(a+b)$. \n(1) 若 $a(a+b)<0$, 则 $f(0) f\\left(\\frac{1}{2}\\right)= -(a+b) \\cdot\\left(-\\frac{1}{4} a\\right)=\\frac{1}{4} a(a+b)<0$, 所以 $f(x)$ 在 $\\left(0, \\frac{1}{2}\\right)$ 内有一实根.\n(2) 若 $a(a+b) \\geqslant 0$, 则 $f\\left(\\frac{1}{2}\\right) f(1)=-\\frac{1}{4} a(2 a+b)=-\\frac{1}{4} a^2-\\frac{1}{4} a(a+ b)<0$, 所以 $f(x)$ 在 $\\left(\\frac{1}{2}, 1\\right)$ 内有一实根.", + "remark": "", + "figures": [] +} \ No newline at end of file diff --git a/processed_dataset/proof/0930.json b/processed_dataset/proof/0930.json new file mode 100644 index 0000000000000000000000000000000000000000..2cdd38f7298f3d7bee99c661b492e43800ecc218 --- /dev/null +++ b/processed_dataset/proof/0930.json @@ -0,0 +1,8 @@ +{ + "source_file": "./raw_volume-zh/volume2/exercise5.tex", + "problem_type": "proof", + "problem": "问题7. $\\triangle A B C$ 的三边长分别为 $a 、 b 、 c$, 周长为 2 , 求证:\n$$\na^2+b^2+c^2+2 a b c<2 .\n$$", + "solution": "由 $a+b+c=2$, 得 $a^2+b^2+c^2=4-2(a b+b c+c a)$, 原不等式等价于 $a b+b c+c a-a b c>1$. 令 $f(x)=(x-a)(x-b)(x-c)=x^3-2 x^2+ (a b+b c+c a) x-a b c$, 则 $f(1)=1-2+(a b+b c+c a)-a b c=a b+b c+ c a-a b c-1$. 另一方面, 由 $a, b, c \\in(0,1)$, 得 $f(1)=(1-a)(1-b)(1-$ c) $>0$, 所以 $a b+b c+c a-a b c>1$.", + "remark": "", + "figures": [] +} \ No newline at end of file diff --git a/processed_dataset/proof/0931.json b/processed_dataset/proof/0931.json new file mode 100644 index 0000000000000000000000000000000000000000..e4e920d402297766b2c4da7698a2e5c2579bc83c --- /dev/null +++ b/processed_dataset/proof/0931.json @@ -0,0 +1,8 @@ +{ + "source_file": "./raw_volume-zh/volume2/exercise5.tex", + "problem_type": "proof", + "problem": "问题8 已知方程 $x^2+b x+c=0$ 有两个实数根 $s$ 、 $t$, 并且 $|s|<2,|t|<2$. 求证:\n(1) $|c|<4$\n(2) $|b|<4+c$.", + "solution": "(1) 由韦达定理知 $|c|=|s t|=|s||t|<4$. \n(2) 设 $f(x)=x^2+ b x+c$, 则 $y=f(x)$ 的图象是开口向上的抛物线, 且与 $x$ 轴的两交点在一 2 与 2 之间, 所以 $f( \\pm 2)>0$, 即 $4+2 b+c>0,4-2 b+c>0$, 所以 $\\pm 2 b<4+ c, 2|b|<4+c$, 故 $|b| \\leqslant 2|b|<4+c$.", + "remark": "", + "figures": [] +} \ No newline at end of file diff --git a/processed_dataset/proof/0932.json b/processed_dataset/proof/0932.json new file mode 100644 index 0000000000000000000000000000000000000000..3f80d664a7f2d466e60116d3dd5ec7b51b88ee7a --- /dev/null +++ b/processed_dataset/proof/0932.json @@ -0,0 +1,8 @@ +{ + "source_file": "./raw_volume-zh/volume2/exercise5.tex", + "problem_type": "proof", + "problem": "问题9 设 $a+b+c=1, a^2+b^2+c^2=1$, 且 $a>b>c$, 求证: $-\\frac{1}{3}b>c$, 故方程有均大于 $c$ 的两个不等实根.\n设 $f(x)=x^2- (1-c) x+c^2-c$, 则 $\\left\\{\\begin{array}{l}\\Delta>0, \\\\ \\frac{1-c}{2}>0, \\\\ f(c)>0,\\end{array}\\left\\{\\begin{array}{l}(1-c)^2+4\\left(c^2-c\\right)>0, \\\\ \\frac{1-c}{2}>c, \\\\ c^2-(1-c) c+c^2-c>0,\\end{array}\\right.\\right.$ 解不等式组, 得 $-\\frac{1}{3}a$, 从而 $\\Delta>0, f(x)$ 与 $x$ 轴有两个不同的交点.\n易知这两个交点为 $u=2 b-a-2 \\sqrt{b(b-a)}, v= 2 b-a+2 \\sqrt{b(b-a)}$. 下面证明 $x_1 \\in[u, v]$. 因 $a \\leqslant 3 x_1 \\leqslant 3 a$, 故 $x_1 \\in \\left[\\frac{a}{3}, a\\right]$, 只需证 $\\left[\\frac{a}{3}, a\\right] \\subset[u, v]$, 即 $u \\leqslant \\frac{a}{3}, a \\leqslant v$. 由于 $v=2 b-a+2 \\sqrt{b(b-a)}>2 b-a>a, u=2 b-a-2 \\sqrt{b(b-a)}=(\\sqrt{b}-\\sqrt{b-a})^2= \\left(\\frac{a}{\\sqrt{b}+\\sqrt{b-a}}\\right)^2=\\frac{a}{\\left(\\sqrt{\\frac{b}{a}}+\\sqrt{\\frac{b}{a}-1}\\right)^2} \\leqslant \\frac{a}{\\left(\\sqrt{\\frac{4}{3}}+\\sqrt{\\frac{1}{3}}\\right)^2}=\\frac{a}{3}$, 所以 $x_1 \\in[u, v]$, 从而必有 $f\\left(x_1\\right) \\leqslant 0$.", + "remark": "", + "figures": [] +} \ No newline at end of file diff --git a/processed_dataset/proof/0934.json b/processed_dataset/proof/0934.json new file mode 100644 index 0000000000000000000000000000000000000000..d0d57ce69b0608a7f56943ecd7307613a63019ff --- /dev/null +++ b/processed_dataset/proof/0934.json @@ -0,0 +1,8 @@ +{ + "source_file": "./raw_volume-zh/volume2/exercise5.tex", + "problem_type": "proof", + "problem": "问题12 证明: 存在两个函数 $f, g: \\mathbf{R} \\rightarrow \\mathbf{R}$, 使得函数 $f(g(x))$ 在 $\\mathbf{R}$ 上是严格递减的, 而 $g(f(x))$ 在 $\\mathbf{R}$ 上是严格递增的.", + "solution": "设 $A=\\bigcup_{k \\in \\mathbf{Z}}\\left([-2^{2 k+1},-2^{2 k}) \\bigcup(2^{2 k}, 2^{2 k+1}]\\right), B=\\bigcup_{k \\in \\mathbf{Z}}\\left(\\left[-2^{2 k},-2^{2 k-1}\\right) \\cup\\left(2^{2 k-1}, 2^{2 k}\\right]\\right)$, 则 $A=2 B, B=2 A, A=-A, B=-B, A \\cap B=\\varnothing$, 并且 $A \\cup B \\cup\\{0\\}=\\mathbf{R}$. 现在令\n$$\nf(x)=\\left\\{\\begin{array}{ll}\nx, & x \\in A, \\\\\n-x, & x \\in B, \\\\\n0, & x=0,\n\\end{array} \\text { 而 } g(x)=2 f(x),\\right.\n$$\n那么, $f(g(x))=f(2 f(x))=-2 x$, 而 $g(f(x))=2 f(f(x))=2 x$. 所以, 满足条件的函数存在.", + "remark": "", + "figures": [] +} \ No newline at end of file diff --git a/processed_dataset/proof/0935.json b/processed_dataset/proof/0935.json new file mode 100644 index 0000000000000000000000000000000000000000..133eaac2558a80ba0b224c7d5ed2a3af633049c5 --- /dev/null +++ b/processed_dataset/proof/0935.json @@ -0,0 +1,8 @@ +{ + "source_file": "./raw_volume-zh/volume2/exercise6.tex", + "problem_type": "proof", + "problem": "问题5 设 $f(x)=4\\left(x-\\frac{1}{2}\\right)^2, 0 \\leqslant x \\leqslant 1$. 求证: 对任意给定的 $n \\in \\mathbf{N}_{+}$, 必有 $x_0$ 使 $f^{(n)}\\left(x_0\\right)=x_0$, 但当 $kx$, 则上式左边为负, 而右边却为正, 矛盾! 同样, 若 $f(f(x))\\frac{1}{2}$, 那么 $f(x) \\leqslant 1<2 x$. 现在假设 $0< x \\leqslant \\frac{1}{2}$, 选取 $n \\geqslant 1$, 使得 $\\frac{1}{2^{n+1}}1$ 及 $u 、 v>0$,\n$$\nf\\left(x^u y^v\\right) \\leqslant f(x)^{\\frac{1}{4 u}} f(y)^{\\frac{1}{4 v}}\n$$\n都成立, 试确定所有这样的函数 $f$.", + "solution": "本题所给的是一个函数不等式, 而不是一个等式, 我们设法先把它变成一个等式.\n令 $x=y, u=v$, 代入(1), 得 $f\\left(x^{2 u}\\right) \\leqslant f(x)^{\\frac{1}{2 u}}$. 再将 $2 u$ 用 $u$ 代换得, 对所有 $x>1, u>0$, 均有 $f\\left(x^u\\right) \\leqslant f(x)^{\\frac{1}{u}} \\cdots (2)$. 令 $y=x^u, v=\\frac{1}{u}$, 则 $x=y^{\\frac{1}{u}}=y^v, u=\\frac{1}{v}$, 代入(2)式, 得 $f(y) \\leqslant f\\left(y^v\\right)^v$, 用 $x$ 代换 $y, u$ 代换 $v$, 则对所有 $x>1, u>0$, 又有 $f\\left(x^u\\right) \\geqslant f(x)^{\\frac{1}{u}} \\cdots (3)$. 由(2)、(3)便知 $f\\left(x^u\\right)= f(x)^{\\frac{1}{u}}\\cdots (4)$. 现在来求(4)的解.\n取 $x=\\mathrm{e}, t=\\mathrm{e}^u$ (则 $u=\\ln t$ ), 当 $u$ 从 0 变化到 $+\\infty$ 时, $t$ 从 1 变化到 $+\\infty$, 于是(4)式为 $f(t)=f(\\mathrm{e})^{\\frac{1}{\\ln } t}$ 、令 $f(\\mathrm{e})=a>1$, 用 $x$ 代换 $t$, 便得 $f(x)=a^{\\frac{1}{1 \\ln }}, a>1, \\cdots(5)$ . 下面验证(5)所给出的函数满足(1) $\\frac{1}{4 u \\ln x}+\\frac{1}{4 v \\ln y} \\geqslant \\frac{1}{u \\ln x+v \\ln y}$, 从而 $f\\left(x^u y^v\\right)=a^{\\frac{1}{u \\ln x+v \\ln y}} \\leqslant a^{\\frac{1}{4 u \\ln x}+\\frac{1}{4 v \\ln y}}= f(x)^{\\frac{1}{4 u}} f(y)^{\\frac{1}{4 x}}$. 这就证明了对所有 $a>1$, (5)式所给出的函数 $f(x)$ 即为所求.", + "remark": "", + "figures": [] +} \ No newline at end of file diff --git a/processed_dataset/proof/0940.json b/processed_dataset/proof/0940.json new file mode 100644 index 0000000000000000000000000000000000000000..85881adf9837857c2e1cece144860f766022d098 --- /dev/null +++ b/processed_dataset/proof/0940.json @@ -0,0 +1,8 @@ +{ + "source_file": "./raw_volume-zh/volume2/exercise7.tex", + "problem_type": "proof", + "problem": "问题8. 证明:如果函数 $f: \\mathbf{R} \\rightarrow \\mathbf{R}$ 满足下面两个恒等式中的一个, 则必满足另一个:\n$$\n\\begin{gathered}\nf(x+y) \\equiv f(x)+f(y), x, y \\in \\mathbf{R} ; \\\\\nf(x y+x+y) \\equiv f(x y)+f(x)+f(y), x, y \\in \\mathbf{R} .\n\\end{gathered}\n$$", + "solution": "如果函数 $f(x)$ 满足第一个恒等式, 那么 $f(x y+x+y) \\equiv f(x y)+ f(x+y) \\equiv f(x y)+f(x)+f(y), x, y \\in \\mathbf{R}$, 即 $f(x)$ 也满足第二个恒等式.\n现在设函数 $f(x)$ 满足第二个恒等式, 令 $y=u+v+u v$, 得 $f(x+u+v+u v+x u+x v+x u v) \\equiv f(x)+f(u+v+u v)+f(x u+x v+x u v)$. 又 $f(u+ v+u v) \\equiv f(u)+f(v)+f(u v)$. 故 $f(x+u+v+x u+x v+u v+x u v) \\equiv f(x)+f(u)+f(v)+f(u v)+f(x u+x v+x u v) \\cdots$ (1). 在(1)中变换变量 $x$ 和 $u$ 的位置, 又得 $f(x+u+v+x u+x v+u v+x u v) \\equiv f(x)+f(u)+f(v)+ f(x v)+f(x u+v v+x u v) \\cdots$ (2). 由(1)和(2), 有 $f(u v)+f(x u+x v+x u v) \\equiv f(x v)+f(x u+w v+x u v) \\cdots$ (3). 在(3)中令 $x=1$, 有 $f(u v)+f(u+v+u v) \\equiv f(v)+f(u+2 w v)$. 所以 $f(u v)+f(u)+f(v)+f(u v) \\equiv f(v)+f(u+2 u v)$, 即 $f(u)+2 f(u v) \\equiv f(u+2 u v) \\cdots$ (4). 在(4)中令 $u=0$, 易得 $f(0)=0 \\cdots$ (5). 在 (4)中令 $u=-1$, 有 $f(-u) \\equiv f(u)+2 f(-u)$. 从而 $f(-u)=-f(u)$. 在(4) 中令 $v=-\\frac{1}{2}$, 有 $f(0) \\equiv f(u)+2 f\\left(-\\frac{u}{2}\\right) \\cdots$ (6). 由(5)和 (6), 我们得到 $f(u) \\equiv 2 f\\left(\\frac{u}{2}\\right)$, 即 $f(2 u)=2 f(u) \\cdots$ (7). 由(4)和(7), 有 $f(u+2 u v) \\equiv f(u)+ f(2 u v)$. 在上式中再令 $2 v=t$, 得到 $f(u+u t) \\equiv f(u)+f(u t) \\cdots$ (8). 所以当 $x \\neq 0$ 时, 由 (8), 有 $f(x+y) \\equiv f\\left(x+\\frac{y}{x}\\right) \\equiv f(x)+f\\left(x \\cdot \\frac{y}{x}\\right) \\equiv f(x)+ f(y)$. 另一方面, 当 $x=0$ 时, 由(5), $f(x+y) \\equiv f(x)+f(y)$ 成立.\n因此, $f(x, y)$ 满足第一个恒等式.", + "remark": "", + "figures": [] +} \ No newline at end of file diff --git a/processed_dataset/proof/0941.json b/processed_dataset/proof/0941.json new file mode 100644 index 0000000000000000000000000000000000000000..c3ee4ecf3b404bbe3c472d7bd451cc3f219ea7a1 --- /dev/null +++ b/processed_dataset/proof/0941.json @@ -0,0 +1,8 @@ +{ + "source_file": "./raw_volume-zh/volume2/exercise7.tex", + "problem_type": "proof", + "problem": "问题12. 设 $M$ 是满足 $f(0) \\neq 0$ 与\n$$\nf(n) f(m) \\equiv f(n+m)+f(n-m), n, m \\in \\mathbf{Z}\n$$\n的函数 $f: \\mathbf{Z} \\rightarrow \\mathbf{R}$ 之集合, 试求:\n(1) 满足 $f(1)=\\frac{5}{2}$ 的所有函数 $f(n) \\in M$;\n(2) 满足 $f(1)=\\sqrt{3}$ 的所有函数 $f(n) \\in M$.", + "solution": "在题述的恒等式中, 取 $n=m=0$, 得 $(f(0))^2=2 f(0)$. 但是 $f(0) \\neq 0$, 因此 $f(0)=2$. 再在恒等式中取 $m=1$, 得 $f(n) f(1) \\equiv f(n+ 1) +f(n-1), n \\in \\mathbf{Z}$. 如果在 $n=0$ 与 $n=1$ 时函数值 $f(n)$ 确定, 那么由上面的恒等式可唯一确定 $f(2)$ 与 $f(-1)$ 的值, 继而可确定 $f(3)$ 与 $f(-2)$ 的值, 等等.\n这样一来对每个 $n \\in \\mathbf{Z}$ 皆可确定 $f(n)$ 的值.\n于是, 若已知 $f(0)=2$, 且 $f(1)=\\frac{5}{2}$ (在 (1)中)或 $f(1)=\\sqrt{3}$ (在 (2) 中), 则我们可唯一确定 $f(n)$. 下面证明: 函数 $f(n)=2^n+\\frac{1}{2^n}$ 与函数 $f(n)=2 \\cos \\frac{n \\pi}{6}$ 分别满足 (1) 和 (2). \n事实上, 有 (1) $f(0)=2^0+2^0, f(1)=2^1+\\frac{1}{2^1}=\\frac{5}{2}, f(n) f(m)= \\left(2^n+\\frac{1}{2^n}\\right)\\left(2^m+\\frac{1}{2^m}\\right)=\\left(2^{n+m}+\\frac{1}{2^{n+m}}\\right)+\\left(2^{n-m}+\\frac{1}{2^{n-m}}\\right)=f(n+m)+f(n-m), m, n \\in \\mathbf{Z}$.\n(2) $f(0)=2 \\cos 0, f(1)=2 \\cos \\frac{\\pi}{6}=\\sqrt{3}, f(n) f(m)= 4 \\cos \\frac{n \\pi}{6} \\cos \\frac{m \\pi}{6}=2 \\cos \\frac{(m+n) \\pi}{6}+2 \\cos \\frac{(n-m) \\pi}{6}=f(n+m)+f(n-m), m, n \\in \\mathbf{Z}$.", + "remark": "", + "figures": [] +} \ No newline at end of file diff --git a/processed_dataset/proof/0942.json b/processed_dataset/proof/0942.json new file mode 100644 index 0000000000000000000000000000000000000000..255e18f7415df7f74a77a141d04a7632aa93deb5 --- /dev/null +++ b/processed_dataset/proof/0942.json @@ -0,0 +1,8 @@ +{ + "source_file": "./raw_volume-zh/volume2/exercise7.tex", + "problem_type": "proof", + "problem": "问题14. 设 $f: \\mathbf{R} \\rightarrow \\mathbf{R}$ 满足: 对任意实数 $x 、 y$, 有\n$$\nf(2 x)+f(2 y)=f(x+y) f(x-y) .\n$$\n又 $f(\\pi)=0$, 但 $f(x)$ 不恒等于零.\n(1) 确定函数 $f(x)$ 的奇偶性;\n(2) 证明: $f(x)$ 是周期函数.", + "solution": "(1) 取 $x=y=\\frac{1}{2}$ 代入(1), 得 $2 f(t)=f(t) f(0)$. 因 $f(x)$ 不恒等于零, 所以 $f(0)=2$. 取 $x=\\frac{1}{2}, y=-\\frac{1}{2}$ 代入 (1), 得 $f(t)+f(-t)= f(0) f(t)$, 即 $f(-t)=f(t)$, 所以, $f(x)$ 是偶函数; \n(2) 令 $x=t+\\frac{\\pi}{2}, y= t-\\frac{\\pi}{2}$ 代入(1), 得 $f(2 t+\\pi)+f(2 t-\\pi)=f(2 t) f(\\pi)$, 所以 $f(2 t+\\pi)= -f(2 t-\\pi) \\cdots$ (2). 取 $x=2 t-\\pi$ 代入(2), 得 $f(x+2 \\pi)=-f(x)$. 故 $f(x+ 4 \\pi)=-f(x+2 \\pi)=f(x)$, 即 $f(x)$ 是以 $4 \\pi$ 为周期的周期函数.", + "remark": "", + "figures": [] +} \ No newline at end of file diff --git a/processed_dataset/proof/0943.json b/processed_dataset/proof/0943.json new file mode 100644 index 0000000000000000000000000000000000000000..5e299a6af26da60a429588b54cb895dc35d7bb64 --- /dev/null +++ b/processed_dataset/proof/0943.json @@ -0,0 +1,8 @@ +{ + "source_file": "./raw_volume-zh/volume2/exercise7.tex", + "problem_type": "proof", + "problem": "问题17 已知多项式 $f(x)$ 满足:\n$$\nf\\left(2 x^2+1\\right)-4 f(x)=4(f(x))^2+1 .\n$$\n且 $f(0)=0$, 求 $f(x)$.", + "solution": "原函数方程可化为 $f\\left(2 x^2+1\\right)=(2 f(x)+1)^2 \\cdots$ (1). \n取 $x=0$ 代入 (1), 得 $f(1)=(2 f(0)+1)^2=1$. \n取 $x=1$ 代入 (1), 并利用 $f(1)=1$, 得 $f(3)=(2 f(1)+1)^2=3^2$. \n取 $x=3$ 代入(1), 并利用 $f(3)=3^2$, 得 $f(19)= (2 f(3)+1)^2=19, \\cdots$. \n作数列 $\\left\\{a_n\\right\\}, a_1=0, a_{n+1}=2 a_n^2+1, n=1,2, \\cdots$. \n用数学归纳法易证 $f\\left(a_n\\right)=a_n^2, n \\in \\mathbf{N}_{+}$\n事实上, 假设 $f\\left(a_k\\right)=a_k^2$, 那么 $f\\left(a_{k+1}\\right)=f\\left(2 a_k^2+1\\right)=\\left(2 f\\left(a_k\\right)+1\\right)$ (利用(1) $)=\\left(2 a_k^2+1\\right)^2$ (利用归纳假设) $=a_{k+1}^2$. \n设 $f(x)-x^2$ 是 $m$ 次多项式, 由代数基本定理知它有 $m$ 个根.\n但上面已证 $f(x)-x^2$ 有无穷多个根, 从而 $f(x)-x^2 \\equiv 0$. 即 $f(x)=x^2$.", + "remark": "", + "figures": [] +} \ No newline at end of file diff --git a/processed_dataset/proof/0944.json b/processed_dataset/proof/0944.json new file mode 100644 index 0000000000000000000000000000000000000000..e3d6768062c43b0b84c87f9c5f51fe59e3cd6a51 --- /dev/null +++ b/processed_dataset/proof/0944.json @@ -0,0 +1,8 @@ +{ + "source_file": "./raw_volume-zh/volume2/exercise7.tex", + "problem_type": "proof", + "problem": "问题24. 设 $f(x)$ 满足柯西方程 $\\circledast$, 但存在某一个数 $\\alpha$, 使 $f(\\alpha) \\neq \\alpha f(1)$. 求证: 对任给的区间 $(u, v)$ 和 $(s, t)$, 必存在 $c \\in(u, v)$, 使 $f(c) \\in(s, t)$.", + "solution": "因为 $f(x)$ 满足柯西方程 $\\circledast$, 不难证明对所有的有理数 $x$, 有 $f(x)= f(1) x$. 因而 $\\alpha$ 必为无理解.\n令 $p=\\frac{u+v}{2}, q=\\frac{s+t}{2}$. 考虑下列方程组: $\\left\\{\\begin{array}{l}\\alpha x+y=p, \\\\ f(\\alpha) x+f(1) y=q .\\end{array}\\right. \\cdots$ (1), 由于此方程组的行列式 $\\left|\\begin{array}{cc}\\alpha & 1 \\\\ f(\\alpha) & f(1)\\end{array}\\right|=\\alpha f(1)- f(\\alpha) \\neq 0$, 故它必有唯一解 $(x, y)$, 但 $x, y$ 不一定是有理数.\n在 $x 、 y$ 附近取两个有理数 $x_0, y_0$, 使 $\\left|x_0-x\\right|$ 和 $\\left|y_0-y\\right|$ 非常小, 以至: $\\left\\{\\begin{array}{l}\\left|\\alpha\\left(x_0-x\\right)\\right|+\\left|y_0-y\\right|<\\frac{1}{4}|u-v|, \\\\ \\left|f(\\alpha)\\left(x_0-x\\right)\\right|+\\left|f(1)\\left(y_0-y_1\\right)\\right|<\\frac{1}{4}|s-t| .\\end{array}\\right.$\n所以, 由 $p, q$ 意义及 (1)知 $\\left\\{\\begin{array}{l}\\left(\\alpha x_0+y_0\\right) \\in(u, v), \\\\ \\left(f(\\alpha) x_0+f(1) y_0\\right) \\in(s, t) .\\end{array}\\right.$ \n取 $c=\\alpha x_0+y_0$, 则 $c \\in(u, v)$, 又 $x_0$, $y_0 \\in \\mathbf{Q}$, 故 $f(c)=f\\left(\\alpha x_0+y_0\\right)=f\\left(\\alpha x_0\\right)+f\\left(y_0\\right)=x_0 f(\\alpha)+y_0 f(1) \\in(s$, $t$ ). 因此结论成立.\n可见, 柯西方程的解 $f(x)$, 只要在某一点连续, 或只要在某一小区间上有上界或下界, 则必为 $f(1) x$.", + "remark": "", + "figures": [] +} \ No newline at end of file diff --git a/processed_dataset/proof/0945.json b/processed_dataset/proof/0945.json new file mode 100644 index 0000000000000000000000000000000000000000..62b8447aa2f903e0f342debdf311fd4ffc7c9acf --- /dev/null +++ b/processed_dataset/proof/0945.json @@ -0,0 +1,8 @@ +{ + "source_file": "./raw_volume-zh/volume2/exercise7.tex", + "problem_type": "proof", + "problem": "问题25 求满足函数方程:\n$$\n\\frac{f(x)+f(y)}{f(x)-f(y)}=f\\left(\\frac{x+y}{x-y}\\right)(x \\neq y)\n$$\n的所有连续函数 $f(x)$.", + "solution": "显然 $f(x)=x$ 是原方程的解, 下证只有这个解.\n在原恒等式中令 $y=k x$, 得 $\\frac{f(x)+f(k x)}{f(x)-f(k x)}=f\\left(\\frac{x+k x}{x-k x}\\right)=f\\left(\\frac{1+k}{1-k}\\right)=\\frac{f(1)+f(k)}{f(1)-f(k)}$. \n从而, $f(k x)=\\frac{f(x)}{f(1)} \\cdot f(k)$. 令 $k=0$, 有 $f(0) \\cdot\\left(1-\\frac{f(x)}{f(1)}\\right)=0$. \n由于 $f(x)$ 不恒等于 $f(1)$, 所以 $f(0)=0$. 在原恒等式中令 $y=0$, 得 $f(1)=1$. 于是 $f(k x)=f(k) f(x) \\cdots$ (1). \n我们的目标是去证对 $n \\in \\mathbf{N}_{+}$, 有 $f(n)=n$. 先证明 $f(2)=2$. \n由(1), $f(4)=f^2(2) \\cdots$ (2). 再由原恒等式及(1), $\\frac{f(2)+1}{f(2)-1}=f(3) \\cdots$ (3), $\\frac{f(4)+1}{f(4)-1}=f\\left(\\frac{5}{3}\\right)=\\frac{f(5)}{f(3)} \\cdots$ (4), $\\frac{f(3)+f(2)}{f(3)-f(2)}=f(5) \\cdots$ (5). \n在(2)、 (3)、 (4)、 (5)中消去 $f(3) 、 f(4) 、 f(5)$, 有 $\\frac{f^2(2)+1}{f^2(2)-1}=\\frac{\\left(f^2(2)+1\\right)(f(2)-1)}{(f(2)+1)\\left(1+2 f(2)-f^2(2)\\right)}$,\n即 $f^2(2)=2 f(2)$. 如果 $f(2)=0$, 那么 $f\\left(2 \\times \\frac{1}{2}\\right)=0$, 与 $f(1)=1$ 矛盾.\n所以 $f(2)=2$. 若 $f(n)=n$ 对 $n<2 m$ 成立, 则 $f(2 m)=f(2) f(m)=2 f(m)=2 m$, $f(2 m+1)=\\frac{f(m+1)+f(m)}{f(m+1)-f(m)}=\\frac{m+1+m}{m+1-m}=2 m+1$. \n故 $f(n)=n$ 对一切 $n \\in \\mathbf{N}_{+}$成立.\n在原恒等式中令 $y=-x$, 得 $f(x)+f(-x)=0$. 即 $f(-x)= -f(x)$. 故 $f(n)=n$ 对一切整数 $n$ 成立.\n再由 $f\\left(\\frac{p}{q}\\right)=\\frac{f(p)}{f(q)}=\\frac{p}{q}$, 所以 $f(n)=n$ 对一切有理数 $n=\\frac{p}{q}$ 成立.\n最后利用连续性, 即知 $f(x)=x$, 任意 $x \\in \\mathbf{R}$.", + "remark": "", + "figures": [] +} \ No newline at end of file diff --git a/processed_dataset/proof/0946.json b/processed_dataset/proof/0946.json new file mode 100644 index 0000000000000000000000000000000000000000..1c3f4455526a4b524466a1fa64959d1a3dae6abc --- /dev/null +++ b/processed_dataset/proof/0946.json @@ -0,0 +1,8 @@ +{ + "source_file": "./raw_volume-zh/volume2/exercise7.tex", + "problem_type": "proof", + "problem": "问题30 证明存在唯一的一个 $f: \\mathbf{R}^{+} \\rightarrow \\mathbf{R}^{+}$, 使得对任意 $x \\in \\mathbf{R}^{+}$, 都有\n$$\nf(f(x))=6 x-f(x) .\n$$", + "solution": "构造正数列 $\\left\\{a_n\\right\\}$ 如下: $a_1=6, a_{n+1}=\\frac{6}{1+a_{n-1}} \\cdots$ (1). 用数学归纳法容易证明 (证明留给读者) : $\\left\\{a_{2 n-1}\\right\\}$ 递减, $\\left\\{a_{2 n}\\right\\}$ 递增, 并且 $a_{2 n-1}>2>a_{2 n}$. 因此 $\\left\\{a_{2 n-1}\\right\\}$ 和 $\\left\\{a_{2 n}\\right\\}$ 均有极限, 分别设为 $\\alpha 、 \\beta$. 由(1)式知 $\\alpha=\\frac{6}{1+\\beta}, \\beta=\\frac{6}{1+\\alpha}$. 解得 $\\alpha=\\beta=2$. 对每个正实数 $x$, 显然有 $f(x)<6 x=a_1 x$. 若对每个 $x \\in \\mathbf{R}^{+}$, 有 $f(x)\\frac{6 x}{1+a_{n-1}}=a_n x$. 同样, 若对每个 $x \\in \\mathbf{R}^{+}$, 有 $f(x)>a_n x$, 则必有 $f(x)n$ 的值.\n考虑带余除法 $m=q n+r, 0 \\leqslant r \\leqslant n-1$. 若 $r=0$, 则有 $\\frac{f(n, n q)}{n q}=\\frac{f(n, n(q-1))}{n(q-1)}=\\cdots=\\frac{f(n, n)}{n}=1$, 所以 $f(n, n q)=n q$. \n若 $1 \\leqslant r \\leqslant n-1$, 则有 $\\frac{f(n, n q+r)}{n q+r}= \\frac{f(n, n(q-1)+r)}{n(q-1)+r}=\\cdots=\\frac{f(n, r)}{r}$, 故 $f(n, m)=\\frac{m}{r} f(n, r)$. \n综上所述, 满足条件的 $f(x, y)$ 可由上述递推式加以确定.\n故它存在且唯一.", + "remark": "", + "figures": [] +} \ No newline at end of file diff --git a/processed_dataset/proof/0948.json b/processed_dataset/proof/0948.json new file mode 100644 index 0000000000000000000000000000000000000000..64b050598f8bb7df359c3c359aaf30bb08762d05 --- /dev/null +++ b/processed_dataset/proof/0948.json @@ -0,0 +1,8 @@ +{ + "source_file": "./raw_volume-zh/volume3/chapter1.tex", + "problem_type": "proof", + "problem": "例7 设函数 $f(x), g(x)$ 对任意实数 $x$ 均有 $-\\frac{\\pi}{2} \\sin f(x)$, 并由此证明: 对任意实数 $x$ 均有 $\\cos (\\cos x)>\\sin (\\sin x)$.", + "solution": "证明:由条件可得 $-\\frac{\\pi}{2}\\sin (\\sin x)$.\n利用正、余弦函数的单调性,结合正、余弦函数的有界性以及上述结论, 我们还有如下的一些结论: $\\sin (\\cos x)<\\cos (\\sin x), \\sin (\\sin (\\sin x))< \\sin (\\cos (\\cos x))<\\cos (\\cos (\\cos x))$ 等.", + "remark": "", + "figures": [] +} \ No newline at end of file diff --git a/processed_dataset/proof/0949.json b/processed_dataset/proof/0949.json new file mode 100644 index 0000000000000000000000000000000000000000..a9f3e72fb2eb3b21a1c7a3ccb091df2e6fde91d7 --- /dev/null +++ b/processed_dataset/proof/0949.json @@ -0,0 +1,8 @@ +{ + "source_file": "./raw_volume-zh/volume3/chapter1.tex", + "problem_type": "proof", + "problem": "例10 证明函数 $g(x)=\\cos \\sqrt[3]{x}$ 不是周期函数.", + "solution": "分析:当结论出现否定的形式时,宜采用反证法.\n证明假设 $g(x)$ 是周期函数, 非零常数 $T$ 是它的一个周期, 则 $\\cos \\sqrt[3]{x+T}=\\cos \\sqrt[3]{x}$ 对一切实数 $x$ 都成立.\n取 $x=0$, 得 $\\cos \\sqrt[3]{T}=1$, 从而 $\\sqrt[3]{T}=2 k \\pi(k \\neq 0, k \\in \\mathbf{Z})$.\n取 $x=T$, 得 $\\cos \\sqrt[3]{2 T}=\\cos \\sqrt[3]{T}=1$, 有 $\\sqrt[3]{2 T}=2 e \\pi(e \\neq 0, e \\in \\mathbf{Z})$. 于是 $\\frac{\\sqrt[3]{2 T}}{\\sqrt[3]{T}}=\\frac{2 e \\pi}{2 k \\pi}=\\frac{e}{k}$, 即 $\\sqrt[3]{2}=\\frac{e}{k}$, 从而 $\\sqrt[3]{2}$ 是有理数, 这与 $\\sqrt[3]{2}$ 是无理数相矛盾,故函数 $g(x)=\\cos \\sqrt[3]{x}$ 不是周期函数.\n评注当结论是肯定或否定形式, 含有 “至多”、“至少”等字样时, 可利用反证法证明问题.\n又如: 求证函数 $y=|\\sin x|+|\\cos x|$ 的最小正周期是 $\\frac{\\pi}{2}$.\n易知 $\\frac{\\pi}{2}$ 是它的周期, 再证 $\\frac{\\pi}{2}$ 是它的最小正周期.\n假设 $01$, 故矛盾,所以 $T$ 不存在,原命题正确.", + "remark": "", + "figures": [] +} \ No newline at end of file diff --git a/processed_dataset/proof/0950.json b/processed_dataset/proof/0950.json new file mode 100644 index 0000000000000000000000000000000000000000..2167302ecab8277b693ec099c3c3f0673c354f25 --- /dev/null +++ b/processed_dataset/proof/0950.json @@ -0,0 +1,8 @@ +{ + "source_file": "./raw_volume-zh/volume3/chapter1.tex", + "problem_type": "proof", + "problem": "例13 设函数 $f(x)$ 的定义域为 $\\mathbf{R}$, 对任意实数 $\\alpha 、 \\beta$ 有\n$f(\\alpha)+f(\\beta)=2 f\\left(\\frac{\\alpha+\\beta}{2}\\right) f\\left(\\frac{\\alpha-\\beta}{2}\\right)$, 且 $f\\left(\\frac{\\pi}{3}\\right)=\\frac{1}{2}, f\\left(\\frac{\\pi}{2}\\right)=0$.\n(1) 求证: $f(-x)=f(x)=-f(\\pi-x)$;\n(2) 若 $0 \\leqslant x<\\frac{\\pi}{2}$ 时, $f(x)>0$, 求证: $f(x)$ 在 $[0, \\pi]$ 上单调递减;\n(3) 求 $f(x)$ 的最小周期并加以证明.", + "solution": "分析:正确理解所给等式, 通过赋值法、定义法解答本题.\n解 (1) 因为 $f\\left(\\frac{\\pi}{3}\\right)+f\\left(\\frac{\\pi}{3}\\right)=2 f\\left(\\frac{\\pi}{3}\\right) f(0)$, 且 $f\\left(\\frac{\\pi}{3}\\right)=\\frac{1}{2}$, 所以 $f(0)=1$.\n又 $f(x)+f(-x)=2 f(0) f(x)$, 故 $f(x)=f(-x)$.\n又由于 $f(x)+f(\\pi-x)=2 f\\left(\\frac{\\pi}{2}\\right) f\\left(x-\\frac{\\pi}{2}\\right)$, 且 $f\\left(\\frac{\\pi}{2}\\right)=0$, 故有\n$$\nf(x)=f(-x)=-f(\\pi-x) .\n$$\n(2) 由 $f(-x)=f(x)$ 且 $0 \\leqslant x<\\frac{\\pi}{2}$ 时, $f(x)>0$, 得当 $-\\frac{\\pi}{2}0$.\n设 $0 \\leqslant x_10, f\\left(\\frac{x_1+x_2-\\pi}{2}\\right)>0$. 从而 $f\\left(x_1\\right)>f\\left(x_2\\right)$, 即 $f(x)$ 在 $[0, \\pi]$ 上单调递减.\n(3) 由 (1) $f(-x)=-f(\\pi-x)$, 得 $f(x)=-f(\\pi+x), f(\\pi+x)= -f(2 \\pi+x)$.\n所以 $f(2 \\pi+x)=f(x)$, 说明 $2 \\pi$ 是原函数的一个周期.\n假设 $T_0$ 也是原函数的一个周期, 且 $T_0 \\in(0,2 \\pi)$, 则由 $f\\left(T_0+x\\right)= f(x)$, 得 $f(0)=f\\left(T_0\\right)$.\n但若 $T_0 \\in(0, \\pi]$ 时, 因原函数是单调递减函数, 所以 $f(0)>f\\left(T_0\\right)$, 两者矛盾;\n若 $T_0 \\in(\\pi, 2 \\pi)$ 时, $2 \\pi-T_0 \\in(0, \\pi)$, 从而 $f(0)>f\\left(2 \\pi-T_0\\right)= f\\left(-T_0\\right)=f\\left(T_0\\right)$, 两者矛盾, 所以 $T_0$ 不是原函数的一个周期, 即 $2 \\pi$ 是原函数的最小正周期.\n评注有关周期函数有下面几个结论: (1) 若 $f(x)$ 的图象有两条对称轴 $x=a$ 和 $x=b$, 则 $f(x)$ 是周期函数,且 $2|b-a|$ 是它的一个周期;\n(2) 若 $f(x)$ 的图象有两个对称中心 $(a, 0)$ 和 $(b, 0)$, 则 $f(x)$ 是周期函数, 且 $2|b-a|$ 是它的一个周期;\n(3) 若 $f(x)$ 的图象有一个对称中心 $(a, 0)$ 和一条对称轴 $x=b$, 则 $f(x)$ 是周期函数,且 $4|b-a|$ 是它的一个周期.\n上述结论中, 不妨证明结论 (1):\n因为\n$$\n\\begin{gathered}\nf(2 a-x)=f(x) \\Leftrightarrow f(a+x)=f(a-x), \\\\\nf(2 b-x)=f(x) \\Leftrightarrow f(b+x)=f(b-x), \\\\\nf[2 b-(2 a-x)]=f(2 a-x)=f(x) .\n\\end{gathered}\n$$\n即 $f[x+2(b-a)]=f(x)$, 所以 $f(x)$ 是以 $2|b-a|$ 为周期的周期函数.\n读者不妨对结论 (2) 和 (3) 加以证明.", + "remark": "", + "figures": [] +} \ No newline at end of file diff --git a/processed_dataset/proof/0951.json b/processed_dataset/proof/0951.json new file mode 100644 index 0000000000000000000000000000000000000000..12c019e968402880332a2d1a25f748a8e779d5f8 --- /dev/null +++ b/processed_dataset/proof/0951.json @@ -0,0 +1,8 @@ +{ + "source_file": "./raw_volume-zh/volume3/chapter1.tex", + "problem_type": "proof", + "problem": "例14 设函数 $f(x)=\\sin \\left(\\frac{11}{6} \\pi x+\\frac{\\pi}{3}\\right)$.\n(1) 求 $f(x)$ 的最小正周期;\n(2) 对于任意的正数 $\\alpha$, 是否总能找到不小于 $\\alpha$, 且不大于 $(\\alpha+1)$ 的两个数 $a$ 和 $b$,使 $f(a)=1$ 而 $f(b)=-1$ ? 请回答并论证;\n(3) 若 $\\alpha$ 限定为任意自然数,请重新回答和论证上述问题.", + "solution": "分析:本题的第 (2)、(3) 题实际上说的是能否找到一个长度为 1 的区间, 使在此区间上, $f(x)$ 既取得最大值, 又能取得最小值.\n解 (1) $f(x)$ 的最小正周期 $T=\\frac{2 \\pi}{\\frac{11}{6} \\pi}=\\frac{12}{11}$.\n(2) 由于 $T>1$, 因此在长为 1 的区间上, $f(x)$ 不能得出一段完整周期的图形.\n现任取一使 $f(x)$ 取最大值 1 的 $x$ 值为 $a$, 如取 $a=\\frac{1}{11}$, 则 $f\\left(\\frac{1}{11}\\right)=\\sin \\left(-\\frac{\\pi}{6}+\\frac{\\pi}{3}\\right)=1$, 令 $\\alpha=a-\\frac{5.5}{11}=-\\frac{9}{22}$, 则 $\\alpha+1=-\\frac{9}{22}+1=\\frac{13}{22}$, 则对于 $\\alpha=-\\frac{9}{22}$, 就不能在区间 $\\left(-\\frac{9}{22}, \\frac{13}{22}\\right)$ 上找到 $b$, 使 $f(b)=-1$.\n(3) 使 $f(x)$ 取最大值 1 的 $x$ 集合为 $\\left\\{x \\mid x=\\frac{12}{11} k+\\frac{1}{11}, k \\in \\mathbf{Z}\\right\\}$ (只须令 $\\frac{11}{6} \\pi x+\\frac{\\pi}{3}=2 k \\pi+\\frac{\\pi}{2}$ 即可解出这些值).\n使 $f(x)$ 取最小值 -1 的 $x$ 集合为 $\\left\\{x \\mid x=\\frac{12}{11} k+\\frac{7}{11}, k \\in \\mathbf{Z}\\right\\}$, 由于\n$$\n\\begin{gathered}\n\\left(\\frac{12}{11} k+\\frac{7}{11}\\right)-\\left(\\frac{12}{11} k+\\frac{1}{11}\\right)=\\frac{6}{11}, \\\\\n{\\left[\\frac{12}{11}(k+1)+\\frac{1}{11}\\right]-\\left(\\frac{12}{11} k+\\frac{1}{11}\\right)=\\frac{12}{11},}\n\\end{gathered}\n$$\n故若 $\\left[\\frac{12}{11} k+\\frac{1}{11}\\right]=n(n \\in \\mathbf{Z})$ ([ $[x]$ 表示不超过 $x$ 的最大整数), 则 $\\left[\\frac{12}{11} k+\\frac{7}{11}\\right]=n$ 或 $n+1$, 且 $\\left[\\frac{12}{11}(k+1)+\\frac{1}{11}\\right]=n+1$ 或 $n+2$, 而 $k=0$ 时, $\\left[\\frac{12}{11} k+\\frac{1}{11}\\right]=0$, 这说明对于任一自然数 $n$, 必存在 $k$, 使 $\\left[\\frac{12}{11} k+\\frac{1}{11}\\right]=n$ 或 $\\left[\\frac{12}{11} k+\\frac{7}{11}\\right]=n$\n若对某一自然数 $n$, 有 $\\left[\\frac{12}{11} k+\\frac{1}{11}\\right]=n$, 令 $\\alpha=\\left(\\frac{12}{11} k+\\frac{1}{11}\\right)-n$, 则当 $0 \\leqslant \\alpha \\leqslant \\frac{5}{11}$ 时, $\\frac{12}{11} k+\\frac{7}{11} \\in(n, n+1]$. 当 $\\frac{6}{11} \\leqslant \\alpha \\leqslant \\frac{10}{11}$ 时, $\\frac{12}{11}(k-1)+\\frac{7}{11} \\in[n$, $n+1)$, 总之, 在 $[n, n+1]$ 中, 存在二数 $a 、 b$, 使 $f(a)=1$ 且 $f(b)=-1$.\n若对某一自然数 $n$, 有 $\\left[\\frac{12}{11} k+\\frac{7}{11}\\right]=n$, 且 $\\left[\\frac{12}{11} k+\\frac{1}{11}\\right]=n-1$, 则令 $\\alpha^{\\prime}= \\left(\\frac{12}{11} k+\\frac{7}{11}\\right)-n$, 显然 $\\alpha^{\\prime}<\\frac{6}{11}$, 即 $0 \\leqslant \\alpha^{\\prime} \\leqslant \\frac{5}{11}$, 此时 $\\left[\\frac{12}{11}(k+1)+\\frac{1}{11}\\right] \\in(n$, $n+1]$, 即在 $[n, n+1]$ 中仍可找到二数 $a 、 b$, 使 $f(a)=1$ 且 $f(b)=-1$.\n综上所述,对于任意自然数 $n$, 总能找到不小于 $n$ 且不大于 $(n+1)$ 的两个数 $a 、 b$, 使 $f(a)=1$ 且 $f(b)=-1$.\n评注对于存在性问题的探索, 通常以举出反例来说明其不存在, 而必须通过严密论证来说明其存在.", + "remark": "", + "figures": [] +} \ No newline at end of file diff --git a/processed_dataset/proof/0952.json b/processed_dataset/proof/0952.json new file mode 100644 index 0000000000000000000000000000000000000000..9b957ec49b111d02848e331c2fa93202258ac70a --- /dev/null +++ b/processed_dataset/proof/0952.json @@ -0,0 +1,8 @@ +{ + "source_file": "./raw_volume-zh/volume3/chapter1.tex", + "problem_type": "proof", + "problem": "例15 是否存在实数 $x$, 使 $\\tan x+\\sqrt{3}$ 与 $\\cot x+\\sqrt{3}$ 是有理数?", + "solution": "分析:假设不存在实数 $x$,使 $\\tan x+\\sqrt{3}$ 与 $\\cot x+\\sqrt{3}$ 是有理数.\n证明若 $\\tan x+\\sqrt{3}$ 是有理数,则存在 $p, q \\in \\mathbf{Z}$ 且 $(p, q)=1$, 使得 $\\tan x+ \\sqrt{3}=\\frac{p}{q}$, 故 $\\tan x=\\frac{p}{q}-\\sqrt{3}$, 同理存在既约分数 $s, t$, 使得 $\\cot x=\\frac{s}{t}-\\sqrt{3}$, 所以 $\\left(\\frac{p}{q}-\\sqrt{3}\\right)\\left(\\frac{s}{t}-\\sqrt{3}\\right)=1$, 即 $\\sqrt{3}(p t+q s)=2 q t+p s$.\n有理数分析知 $p t+q s=0$ 且 $2 q t+p s=0$, 把前两式移项相乘得 $p^2=2 q^2$. 奇偶性分析知 $p, q$ 都为偶数,这与 $(p, q)=1$ 矛盾.\n评注 (1) 反证法是解决这类问题的常用方法; (2) 用有理数、奇偶性分析来找整数问题的矛盾.", + "remark": "", + "figures": [] +} \ No newline at end of file diff --git a/processed_dataset/proof/0953.json b/processed_dataset/proof/0953.json new file mode 100644 index 0000000000000000000000000000000000000000..ec4c167633e66c67c7a55065b588836d6b53cd40 --- /dev/null +++ b/processed_dataset/proof/0953.json @@ -0,0 +1,8 @@ +{ + "source_file": "./raw_volume-zh/volume3/chapter1.tex", + "problem_type": "proof", + "problem": "例18 函数 $F(x)=\\mid \\cos ^2 x+2 \\sin x \\cos x- \\sin ^2 x+A x+B \\mid$ 在 $0 \\leqslant x \\leqslant \\frac{3}{2} \\pi$ 上的最大值 $M$ 与参数 $A 、 B$ 有关, 问 $A 、 B$ 取什么值时, $M$ 为最小? 证明你的结论.", + "solution": "分析:$\\left|\\cos ^2 x+2 \\sin x \\cdot \\cos x-\\sin ^2 x+A x+B\\right|=\\mid \\sqrt{2} \\sin \\left(2 x+\\frac{\\pi}{4}\\right)+ A x+B \\mid$. 故 $F(x)$ 是一个三角函数与一个一次函数之和, 因为三角函数是一个周期函数, 而 $A x+B$ 是一个一次或零次函数, 所以不管怎样, 只要 $A 、 B$ 中有一个不为 $0, F(x)$ 最大值显然增大.\n故猜想 $M$ 的最小值在 $A=B=0$ 时取得.\n解 $F(x)=\\left|\\sqrt{2} \\sin \\left(2 x+\\frac{\\pi}{4}\\right)+A x+B\\right|$. 取 $g(x)=\\sqrt{2} \\sin \\left(2 x+\\frac{\\pi}{4}\\right)$, 则 $g\\left(\\frac{\\pi}{8}\\right)=g\\left(\\frac{9 \\pi}{8}\\right)=\\sqrt{2}, g\\left(\\frac{5 \\pi}{8}\\right)=-\\sqrt{2}$.\n取 $h(x)=A x+B$, 若 $A=0, B \\neq 0$, 则当 $B>0$ 时, $F\\left(\\frac{\\pi}{8}\\right)>\\sqrt{2}$, 当 $B<0$ 时, $F\\left(\\frac{5 \\pi}{8}\\right)<\\sqrt{2}$. 从而 $M>\\sqrt{2}$.\n若 $A \\neq 0$, 则当 $h\\left(\\frac{5 \\pi}{8}\\right)<0$ 时, $F\\left(\\frac{5 \\pi}{8}\\right)>\\sqrt{2}$, 当 $h\\left(\\frac{5 \\pi}{8}\\right) \\geqslant 0$ 时, 由于 $h(x)$ 是一次函数, 当 $A>0$ 时, $h(x)$ 递增, $h\\left(\\frac{9 \\pi}{8}\\right)>h\\left(\\frac{5 \\pi}{8}\\right)>0$, 此时 $F\\left(\\frac{9 \\pi}{8}\\right)>\\sqrt{2}$; 当 $A<0$ 时, $h(x)$ 递减, $h\\left(\\frac{\\pi}{8}\\right)>h\\left(\\frac{5 \\pi}{8}\\right)>0$, 此时 $F\\left(\\frac{\\pi}{8}\\right)>\\sqrt{2}$. 故此时 $M>\\sqrt{2}$. 若 $A=B=0$ ,显然有 $M=\\sqrt{2}$.\n从而 $M$ 的最小值为 $\\sqrt{2}$, 这个最小值在 $A=B=0$ 时取得.", + "remark": "", + "figures": [] +} \ No newline at end of file diff --git a/processed_dataset/proof/0954.json b/processed_dataset/proof/0954.json new file mode 100644 index 0000000000000000000000000000000000000000..9a0febda10627590f5fa35d0ca33f402eb4550a2 --- /dev/null +++ b/processed_dataset/proof/0954.json @@ -0,0 +1,8 @@ +{ + "source_file": "./raw_volume-zh/volume3/chapter1.tex", + "problem_type": "proof", + "problem": "例19 已知 $\\theta_1+\\theta_2+\\cdots+\\theta_n=\\pi, \\theta_i \\geqslant 0 (i=1,2, \\cdots, n)$, 求 $\\sin ^2 \\theta_1+\\sin ^2 \\theta_2+\\cdots+\\sin ^2 \\theta_n$ 的最大值.", + "solution": "解:因为 $\\sin ^2 \\theta_1+\\sin ^2 \\theta_2$\n$$\n\\begin{aligned}\n& =\\left(\\sin \\theta_1+\\sin \\theta_2\\right)^2-2 \\sin \\theta_1 \\cdot \\sin \\theta_2 \\\\\n& =4 \\sin ^2 \\frac{\\theta_1+\\theta_2}{2} \\cdot \\cos ^2 \\frac{\\theta_1-\\theta_2}{2}+\\cos \\left(\\theta_1+\\theta_2\\right)-\\cos \\left(\\theta_1-\\theta_2\\right) \\\\\n& =2 \\cos ^2 \\frac{\\theta_1-\\theta_2}{2}\\left(2 \\sin ^2 \\frac{\\theta_1+\\theta_2}{2}-1\\right)+1+\\cos \\left(\\theta_1+\\theta_2\\right),\n\\end{aligned}\n$$\n当 $\\theta_1+\\theta_2<\\frac{\\pi}{2}$ 时, $2 \\sin ^2 \\frac{\\theta_1+\\theta_2}{2}-1<0$;\n当 $\\theta_1+\\theta_2=\\frac{\\pi}{2}$ 时, $2 \\sin ^2 \\frac{\\theta_1+\\theta_2}{2}-1=0$;\n当 $\\theta_1+\\theta_2>\\frac{\\pi}{2}$ 时, $2 \\sin ^2 \\frac{\\theta_1+\\theta_2}{2}-1>0$.\n由此可得出, 当 $\\theta_1+\\theta_2<\\frac{\\pi}{2}$ 时, $\\theta_1$ 与 $\\theta_2$ 有一个为零时, $\\sin ^2 \\theta_1+\\sin ^2 \\theta_2$ 有最大值; 当 $\\theta_1+\\theta_2>\\frac{\\pi}{2}$, 且 $\\left|\\theta_1-\\theta_2\\right|$ 越小时, $\\sin ^2 \\theta_1+\\sin ^2 \\theta_2$ 值越大.\n当 $n=3$ 时, 即 $\\theta_1+\\theta_2+\\theta_3=\\pi$ 时, 容易证明\n$$\n\\sin ^2 \\theta_1+\\sin ^2 \\theta_2+\\sin ^2 \\theta_3 \\leqslant \\frac{9}{4} \\text {. }\n$$\n而当 $n \\geqslant 4$ 时, 可知 $\\theta_1, \\theta_2, \\theta_3, \\theta_4$ 中必有两个角不超过 $\\frac{\\pi}{2}$.\n由前面结论知, $\\theta_1+\\theta_2 \\leqslant \\frac{\\pi}{2}$ 时, 当 $\\theta_1$ 与 $\\theta_2$ 有一个为零时, $\\sin ^2 \\theta_1+\\sin ^2 \\theta_2$ 有最大值.\n于是所求的最大值可转化成三个角的和为 $\\pi$, 其正弦值的平方的最大值问题.\n另一方面 $n=2$ 时, $\\theta_1+\\theta_2=\\pi, \\sin ^2 \\theta_1+\\sin ^2 \\theta_2 \\leqslant 2$.\n综上所述,当 $n=2$ 时, $\\left(\\sin ^2 \\theta_1+\\sin ^2 \\theta_2\\right)_{\\text {max }}=2$.\n当 $n \\geqslant 3$ 时, $\\left(\\sin ^2 \\theta_1+\\sin ^2 \\theta_2+\\sin ^2 \\theta_3+\\cdots+\\sin ^2 \\theta_n\\right)_{\\text {max }}=\\frac{9}{4}$, 且当 $\\theta_1= \\theta_2=\\theta_3=\\frac{\\pi}{3}, \\theta_4=\\theta_5=\\cdots=\\theta_n=0$ 时,取等号.", + "remark": "", + "figures": [] +} \ No newline at end of file diff --git a/processed_dataset/proof/0955.json b/processed_dataset/proof/0955.json new file mode 100644 index 0000000000000000000000000000000000000000..75966e2d5a320a859e40cdd3eb7e912382d1330c --- /dev/null +++ b/processed_dataset/proof/0955.json @@ -0,0 +1,8 @@ +{ + "source_file": "./raw_volume-zh/volume3/chapter1.tex", + "problem_type": "proof", + "problem": "例20 求所有的实数 $\\alpha$ 的值, 使数列 $a_n= \\cos 2^n \\alpha(n=1,2, \\cdots)$ 中每一项都为负数.", + "solution": "证明:首先, 若 $\\alpha$ 是满足条件的实数, 则 $\\cos \\alpha \\leqslant-\\frac{1}{4}$.\n事实上: 若 $\\cos \\alpha \\in\\left(-\\frac{1}{4}, 0\\right)$, 则\n$$\n\\cos 2 \\alpha=2 \\cos ^2 \\alpha-1<-\\frac{7}{8} .\n$$\n$\\cos 4 \\alpha=2 \\cos ^2 2 \\alpha-1>0$, 矛盾.\n由上述推导可知: 对于任意 $n \\in \\mathrm{N}^*$, 均有 $\\cos 2^n \\alpha \\leqslant-\\frac{1}{4}$, 于是\n$$\n\\left|\\cos 2^n \\alpha-\\frac{1}{2}\\right| \\geqslant \\frac{3}{4},\n$$\n注意到 $\\left|\\cos 2 \\alpha+\\frac{1}{2}\\right|=\\left|\\cos ^2 \\alpha-\\frac{1}{2}\\right|=2\\left|\\cos \\alpha+\\frac{1}{2}\\right|\\left|\\cos \\alpha-\\frac{1}{2}\\right|$,\n有 $\\quad\\left|\\cos \\alpha+\\frac{1}{2}\\right| \\leqslant \\frac{2}{3}\\left|\\cos 2 \\alpha+\\frac{1}{2}\\right| \\leqslant\\left(\\frac{2}{3}\\right)^2\\left(\\cos 4 \\alpha+\\frac{1}{2}\\right) \\leqslant \\cdots$\n$$\n\\leqslant\\left(\\frac{2}{3}\\right)^n\\left|\\cos 2^n \\alpha+\\frac{1}{2}\\right| \\leqslant\\left(\\frac{2}{3}\\right)^n \\text {. }\n$$\n当 $n \\rightarrow \\infty$ 时, $\\left(\\frac{2}{3}\\right)^n \\rightarrow 0$, 故 $\\left(\\cos \\alpha+\\frac{1}{2}\\right)=0$.\n所以 $\\cos \\alpha=-\\frac{1}{2}$, 即 $\\alpha=2 k \\pi \\pm \\frac{\\pi}{3}, k \\in \\mathbf{Z}$.\n另一方面, 当 $\\alpha=2 k \\pi \\pm \\frac{2}{3} \\pi, k \\in \\mathbf{Z}$ 时, 对于任意 $n \\in \\mathbf{N}^*$, 均有 $\\cos 2^n \\alpha= -\\frac{1}{2}$ 满足条件.\n综上所述, $\\alpha=2 k \\pi \\pm \\frac{\\pi}{3}, k \\in \\mathbf{Z}$.", + "remark": "", + "figures": [] +} \ No newline at end of file diff --git a/processed_dataset/proof/0956.json b/processed_dataset/proof/0956.json new file mode 100644 index 0000000000000000000000000000000000000000..ffc9bb1a58ad2a78af48fe70cff66dafd3d53171 --- /dev/null +++ b/processed_dataset/proof/0956.json @@ -0,0 +1,8 @@ +{ + "source_file": "./raw_volume-zh/volume3/chapter2.tex", + "problem_type": "proof", + "problem": "例1 (1) 求 $\\sin ^4 10^{\\circ}+\\sin ^4 50^{\\circ}+\\sin ^4 70^{\\circ}$;\n(2) $\\sin ^2 \\alpha+\\sin ^2\\left(\\alpha+\\frac{\\pi}{3}\\right)+\\sin ^2\\left(\\alpha-\\frac{\\pi}{3}\\right)$;\n(3) 圆 $x^2+y^2=1$ 上有三点,坐标分别为 $\\left(x_1\\right.$, $\\left.y_1\\right),\\left(x_2, y_2\\right),\\left(x_3, y_3\\right)$, 且 $x_1+x_2+x_3=y_1+y_2+y_3=0$. 求证: $x_1^2+ x_2^2+x_3^2=y_1^2+y_2^2+y_3^2=\\frac{3}{2}$.", + "solution": "分析:第一小题和第二小题都可以采用二倍角公式进行计算.\n解 (1) \n$$\n\\begin{aligned}\n& \\sin ^4 10^{\\circ}+\\sin ^4 50^{\\circ}+\\sin ^4 70^{\\circ} \\\\\n& =\\left(\\frac{1-\\cos 20^{\\circ}}{2}\\right)^2+\\left(\\frac{1-\\cos 100^{\\circ}}{2}\\right)^2+\\left(\\frac{1-\\cos 140^{\\circ}}{2}\\right)^2 \\\\\n& =\\frac{3}{4}-\\frac{1}{2}\\left(\\cos 20^{\\circ}+\\cos 100^{\\circ}+\\cos 140^{\\circ}\\right)+\\frac{1}{4}\\left(\\cos ^2 20^{\\circ}+\\cos ^2 100^{\\circ}+\\cos ^2 140^{\\circ}\\right) \\\\\n& \\quad=\\frac{3}{4}-\\frac{1}{2}\\left(2 \\cos 60^{\\circ} \\cos 40^{\\circ}-\\cos 40^{\\circ}\\right)+\\frac{1}{4}\\left(\\frac{1+\\cos 40^{\\circ}}{2}+\\frac{1+\\cos 200^{\\circ}}{2}+\\frac{1+\\cos 280^{\\circ}}{2}\\right) \\\\\n& =\\frac{3}{4}+\\frac{3}{8}-0+\\frac{1}{8}\\left(\\cos 40^{\\circ}+\\cos 80^{\\circ}-\\cos 20^{\\circ}\\right) \\\\\n& =\\frac{3}{4}+\\frac{3}{8}+\\frac{1}{8}\\left(2 \\cos 60^{\\circ} \\cos 20^{\\circ}-\\cos 20^{\\circ}\\right)=\\frac{9}{8} .\n\\end{aligned}\n$$\n(2)\n$$\n\\begin{aligned}\n& \\sin ^2 \\alpha+\\sin ^2\\left(\\alpha+\\frac{\\pi}{3}\\right)+\\sin ^2\\left(\\alpha-\\frac{\\pi}{3}\\right) \\\\\n& =\\frac{1-\\cos 2 \\alpha}{2}+\\frac{1-\\cos 2\\left(\\alpha+\\frac{\\pi}{3}\\right)}{2}+\\frac{1-\\cos 2\\left(\\alpha-\\frac{\\pi}{3}\\right)}{2} \\\\\n& =\\frac{3}{2}-\\frac{1}{2}\\left(\\cos 2 \\alpha+2 \\cos 2 \\alpha \\cos \\frac{2 \\pi}{3}\\right)=\\frac{3}{2} .\n\\end{aligned}\n$$\n(3) 我们假设 $\\left(x_1, y_1\\right),\\left(x_2, y_2\\right),\\left(x_3, y_3\\right)$ 对应的三个点为 $A, B, C$, 原点为 $O$, 原题说的实际上就是 $\\overrightarrow{O A}+\\overrightarrow{O B}+\\overrightarrow{O C}=0$, 也就是说 $\\overrightarrow{O A}+\\overrightarrow{O B}=-\\overrightarrow{O C}$, \n画图马上可以得出 $\\angle A O B=\\frac{2 \\pi}{3}$, 同理可以得出 $\\angle B O C=\\angle C O A=\\frac{2 \\pi}{3}$.\n假设 $A(\\cos \\alpha, \\sin \\alpha)$, 那么有 $B\\left(\\cos \\left(\\alpha+\\frac{2 \\pi}{3}\\right), \\sin \\left(\\alpha+\\frac{2 \\pi}{3}\\right)\\right), C(\\cos (\\alpha-\\frac{2 \\pi}{3}), \\sin (\\alpha-\\frac{2 \\pi}{3}))$\n$$\n\\begin{aligned}\nx_1^2+x_2^2+x_3^2 & =\\cos ^2 \\alpha+\\cos ^2\\left(\\alpha+\\frac{2 \\pi}{3}\\right)+\\cos ^2\\left(\\alpha-\\frac{2 \\pi}{3}\\right) \\\\\n& =\\frac{1}{2}\\left(1+\\cos 2 \\alpha+1+\\cos 2\\left(\\alpha+\\frac{2 \\pi}{3}\\right)+1+\\cos 2\\left(\\alpha-\\frac{2 \\pi}{3}\\right)\\right) \\\\\n& =\\frac{1}{2}\\left(3+\\cos 2 \\alpha+2 \\cos 2 \\alpha \\cos \\frac{2 \\pi}{3}\\right)=\\frac{3}{2}\n\\end{aligned}\n$$\n$x_1^2+x_2^2+x_3^2+y_1^2+y_2^2+y_3^2=3$ 是显然的,所以 $y_1^2+y_2^2+y_3^2=\\frac{3}{2}$.\n评注前两道题都是基本的倍角公式十和差化积变形以及基本的计算, 注意在三角函数里面看到平方或者高次方降次数的方法就是倍角公式法, 能将平方化为一次的, 最后一道题实际上计算步骤和前两道一样, 但是如何得出三角形是一个等边三角形有很多种思路可循, 还有一种思路是, 该三角形的重心和外心重合, 都是原点,所以只能是等边三角形.", + "remark": "", + "figures": [] +} \ No newline at end of file diff --git a/processed_dataset/proof/0957.json b/processed_dataset/proof/0957.json new file mode 100644 index 0000000000000000000000000000000000000000..eedf32a888645b0591141988bd860471f6c651fb --- /dev/null +++ b/processed_dataset/proof/0957.json @@ -0,0 +1,8 @@ +{ + "source_file": "./raw_volume-zh/volume3/chapter2.tex", + "problem_type": "proof", + "problem": "例3 求下列各式的值:\n(1) $\\sin ^2 10^{\\circ}+\\cos ^2 40^{\\circ}+\\sin 10^{\\circ} \\cos 40^{\\circ}$;\n(2) $\\sin ^2 80^{\\circ}-\\sin ^2 40^{\\circ}+\\sqrt{3} \\sin 40^{\\circ} \\cos 80^{\\circ}$;\n(3) $\\sin ^2 20^{\\circ}-\\sin 5^{\\circ}\\left(\\sin 5^{\\circ}+\\frac{\\sqrt{6}-\\sqrt{2}}{2} \\cos 20^{\\circ}\\right)$.", + "solution": "分析:在第 (1)题中, 因 $40^{\\circ}-10^{\\circ}=30^{\\circ}$, 所以可转化到 $30^{\\circ}$ 与 $10^{\\circ}$ 这两角的关系式而求解; 同理,第 (2) 题中, 因 $80^{\\circ}+40^{\\circ}=120^{\\circ}$, 所以可转化到 $120^{\\circ}$ 与 $40^{\\circ}$ 这两角的关系而求解; 第 (3) 题中 $\\frac{\\sqrt{6}-\\sqrt{2}}{2}$ 虽非特殊值, 但若知道 $\\sin 15^{\\circ}= \\frac{\\sqrt{6}-\\sqrt{2}}{4}$, 则 $20^{\\circ}=15^{\\circ}+5^{\\circ}$, 可转化到 $15^{\\circ}$ 的三角函数值.\n(1) 解法一\n$$\n\\begin{aligned}\n& \\sin ^2 10^{\\circ}+\\cos ^2 40^{\\circ}+\\sin 10^{\\circ} \\cos 40^{\\circ} \\\\\n= & \\sin ^2 10^{\\circ}+\\cos ^2\\left(30^{\\circ}+10^{\\circ}\\right)+\\sin 10^{\\circ} \\cdot \\cos \\left(30^{\\circ}+10^{\\circ}\\right) \\\\\n= & \\sin ^2 10^{\\circ}+\\left(\\frac{\\sqrt{3}}{2} \\cos 10^{\\circ}-\\frac{1}{2} \\sin 10^{\\circ}\\right)^2 +\\sin 10^{\\circ} \\cdot\\left(\\frac{\\sqrt{3}}{2} \\cos 10^{\\circ}-\\frac{1}{2} \\sin 10^{\\circ}\\right) \\\\\n= & \\sin ^2 10^{\\circ}+\\frac{3}{4} \\cos ^2 10^{\\circ}-\\frac{\\sqrt{3}}{2} \\sin 10^{\\circ} \\cos 10^{\\circ}+\\frac{1}{4} \\sin ^2 10^{\\circ} +\\frac{\\sqrt{3}}{2} \\sin 10^{\\circ} \\cos 10^{\\circ}-\\frac{1}{2} \\sin ^2 10^{\\circ} \\\\\n= & \\frac{3}{4} \\sin ^2 10^{\\circ}+\\frac{3}{4} \\cos ^2 10^{\\circ}=\\frac{3}{4} .\n\\end{aligned}\n$$\n解法二利用先降次再和差化积与积化和差.\n$$\n\\begin{aligned}\n\\text { 原式 } & =\\frac{1-\\cos 20^{\\circ}}{2}+\\frac{1+\\cos 80^{\\circ}}{2}+\\frac{\\sin 50^{\\circ}-\\sin 30^{\\circ}}{2} \\\\\n& =\\frac{1}{2}\\left(\\frac{3}{2}+\\cos 80^{\\circ}-\\cos 20^{\\circ}+\\sin 50^{\\circ}\\right) \\\\\n& =\\frac{1}{2}\\left(\\frac{3}{2}-2 \\sin \\frac{80^{\\circ}+20^{\\circ}}{2} \\sin \\frac{80^{\\circ}-20^{\\circ}}{2}+\\sin 50^{\\circ}\\right)=\\frac{3}{4} .\n\\end{aligned}\n$$\n(2) 解法一\n$$\n\\begin{aligned}\n& \\sin ^2 80^{\\circ}-\\sin ^2 40^{\\circ}+\\sqrt{3} \\sin 40^{\\circ} \\cos 80^{\\circ} \\\\\n= & \\sin ^2\\left(120^{\\circ}-40^{\\circ}\\right)-\\sin ^2 40^{\\circ}+\\sqrt{3} \\sin 40^{\\circ} \\cdot \\cos \\left(120^{\\circ}-40^{\\circ}\\right) \\\\\n= & \\left(\\frac{\\sqrt{3}}{2} \\cos 40^{\\circ}+\\frac{1}{2} \\sin 40^{\\circ}\\right)^2-\\sin ^2 40^{\\circ} +\\sqrt{3} \\sin 40^{\\circ}\\left(-\\frac{1}{2} \\cos 40^{\\circ}+\\frac{\\sqrt{3}}{2} \\sin 40^{\\circ}\\right) \\\\\n= & \\frac{3}{4} \\cos ^2 40^{\\circ}+\\frac{\\sqrt{3}}{2} \\sin 40^{\\circ} \\cos 40^{\\circ}+\\frac{1}{4} \\sin ^2 40^{\\circ} -\\sin ^2 40^{\\circ}-\\frac{\\sqrt{3}}{2} \\sin 40^{\\circ} \\cos 40^{\\circ}+\\frac{3}{2} \\sin ^2 40^{\\circ} \\\\\n= & \\frac{3}{4} \\cos ^2 40^{\\circ}+\\frac{3}{4} \\sin ^2 40^{\\circ}=\\frac{3}{4} .\n\\end{aligned}\n$$\n解法二原式 $=\\frac{1-\\cos 160^{\\circ}}{2}-\\frac{1-\\cos 80^{\\circ}}{2}+\\frac{\\sqrt{3}}{2} \\cdot\\left(\\sin 120^{\\circ}-\\sin 40^{\\circ}\\right)$\n$$\n\\begin{aligned}\n& =\\frac{1}{2}\\left(\\cos 80^{\\circ}+\\cos 20^{\\circ}+\\frac{3}{2}-\\sqrt{3} \\sin 40^{\\circ}\\right) \\\\\n& =\\frac{1}{2}\\left(\\frac{3}{2}+2 \\cos 50^{\\circ} \\cos 30^{\\circ}-\\sqrt{3} \\sin 40^{\\circ}\\right)=\\frac{3}{4} .\n\\end{aligned}\n$$\n(3)\n$$\n\\begin{aligned}\n& \\sin ^2 20^{\\circ}-\\sin 5^{\\circ}\\left(\\sin 5^{\\circ}+\\frac{\\sqrt{6}-\\sqrt{2}}{2} \\cos 20^{\\circ}\\right) \\\\\n= & \\sin ^2 20^{\\circ}-\\sin ^2 5^{\\circ}-\\frac{\\sqrt{6}-\\sqrt{2}}{2} \\sin 5^{\\circ} \\cos 20^{\\circ} \\\\\n= & \\sin \\left(20^{\\circ}+5^{\\circ}\\right) \\sin \\left(20^{\\circ}-5^{\\circ}\\right)-\\frac{\\sqrt{6}-\\sqrt{2}}{2} \\cdot \\frac{1}{2}\\left[\\sin \\left(5^{\\circ}+20^{\\circ}\\right)+ \\sin \\left(5^{\\circ}-20^{\\circ}\\right)\\right] \\\\\n= & \\frac{\\sqrt{6}-\\sqrt{2}}{4} \\cdot \\sin 25^{\\circ}-\\frac{\\sqrt{6}-\\sqrt{2}}{4} \\sin 25^{\\circ}+\\frac{\\sqrt{6}-\\sqrt{2}}{4} \\sin 15^{\\circ} \\\\\n= & \\frac{2-\\sqrt{3}}{4} .\n\\end{aligned}\n$$\n评注仔细观察本题, 第(1) 题可改写成 $\\sin ^2 10^{\\circ}+\\cos ^2 40^{\\circ}+\\sin 10^{\\circ} \\cos 40^{\\circ}= \\sin ^2 10^{\\circ}+\\sin ^2 50^{\\circ}-2 \\sin 10^{\\circ} \\sin 50^{\\circ} \\cos 120^{\\circ}=\\sin ^2 120^{\\circ}$, 于是, 我们猜想: 若 $\\angle A+\\angle B+\\angle C=\\pi$, 则 $\\sin ^2 B+\\sin ^2 C-2 \\sin B \\sin C \\cos A=\\sin ^2 A$, 这与我们今后学到的三角形中正余弦定理十分类似, 读者不妨根据原题中的解答部分证明一下.\n事实上, 我们还可以证明: 等式 $\\sin ^2 B+\\sin ^2 C-2 \\sin B \\sin C \\cos A=\\sin ^2 A$ 成立的充要条件是 $A=2 k \\pi \\pm(B-C)$ 或 $A=(2 k+1) \\pi \\pm(B+C)$.\n略证充分性: 当 $A=2 k \\pi \\pm(B-C)$ 时,\n$$\n\\begin{aligned}\n\\text { 左边 } & =\\sin ^2 B+\\sin ^2 C-2 \\sin B \\sin C \\cos [2 k \\pi \\pm(B-C)] \\\\\n& =\\sin ^2 B+\\sin ^2 C-2 \\sin B \\sin C \\cos B \\cos C-2 \\sin ^2 B \\sin ^2 C \\\\\n& =\\sin ^2 B \\cos ^2 C-2 \\sin B \\sin C \\cos B \\cos C+\\sin ^2 C \\cos ^2 B \\\\\n& =(\\sin B \\cos C-\\cos B \\sin C)^2=\\sin ^2(B-C)=\\text { 右边.\n}\n\\end{aligned}\n$$\n同理当 $A=(2 k+1) \\pi \\pm(B+C)$ 时,等式成立.\n必要性: 因为 $\\sin ^2 B+\\sin ^2 C-2 \\sin B \\sin C \\cos A=\\sin ^2 A$,\n即整理得所以\n$$\n\\sin ^2 B+\\sin ^2 C-2 \\sin B \\sin C \\cos A=1-\\cos ^2 A,\n$$\n$$\n\\begin{gathered}\n(\\cos A-\\sin B \\sin C)^2=\\cos ^2 B \\cos ^2 C, \\\\\n\\cos A=\\sin B \\sin C \\pm \\cos B \\cos C .\n\\end{gathered}\n$$\n当 $\\cos A=\\sin B \\sin C+\\cos B \\cos C=\\cos (B-C)$ 时,\n$$\nA=2 k \\pi \\pm(B-C), k \\in \\mathbf{Z}\n$$\n当 $\\cos A=\\sin B \\sin C-\\cos B \\cos C=-\\cos (B+C)$ 时,\n$$\nA=(2 k+1) \\pi \\pm(B+C), k \\in \\mathbf{Z} .\n$$\n根据上述结论, 请读者利用公式计算本例中的第 (1)、(2)题.\n至于第 (3) 题是上述公式的变形: 即当 $C=(2 k+1) \\pi-(A-B)$ 或 $C=2 k \\pi-(A-B)$ 时, 有 $\\sin ^2 A-\\sin ^2 B-2 \\cos A \\sin B \\sin (A-B)=\\sin ^2(A-B)$. 于是, 第(3)题也能套用公式.", + "remark": "", + "figures": [] +} \ No newline at end of file diff --git a/processed_dataset/proof/0958.json b/processed_dataset/proof/0958.json new file mode 100644 index 0000000000000000000000000000000000000000..e51e25855dfef4f2514d4bde423c17c401bc0edd --- /dev/null +++ b/processed_dataset/proof/0958.json @@ -0,0 +1,8 @@ +{ + "source_file": "./raw_volume-zh/volume3/chapter2.tex", + "problem_type": "proof", + "problem": "例4 不查表, 求 $\\left(1+\\cos \\frac{\\pi}{5}\\right)\\left(1+\\cos \\frac{3 \\pi}{5}\\right)$ 的值.", + "solution": "分析:熟练应用和差化积与积化和差公式.\n解法一 \n$$\n\\begin{aligned}\n& \\left(1+\\cos \\frac{\\pi}{5}\\right)\\left(1+\\cos \\frac{3 \\pi}{5}\\right) \\\\\n& =1+\\cos \\frac{\\pi}{5}+\\cos \\frac{3 \\pi}{5}+\\cos \\frac{\\pi}{5} \\cos \\frac{3 \\pi}{5} \\\\\n& =1+\\cos \\frac{\\pi}{5}+\\cos \\frac{3 \\pi}{5}+\\frac{1}{2}\\left(\\cos \\frac{2 \\pi}{5}+\\cos \\frac{4 \\pi}{5}\\right) \\\\\n& =1+\\cos \\frac{\\pi}{5}-\\cos \\frac{2 \\pi}{5}+\\frac{1}{2} \\cos \\frac{2 \\pi}{5}-\\frac{1}{2} \\cos \\frac{\\pi}{5} \\\\\n& =1+\\frac{1}{2}\\left(\\cos \\frac{\\pi}{5}-\\cos \\frac{2 \\pi}{5}\\right) \\\\\n& =1+\\sin \\frac{\\pi}{10} \\sin \\frac{3 \\pi}{10} \\\\\n& =1+\\frac{2 \\cos \\frac{\\pi}{10} \\sin \\frac{\\pi}{10} \\sin \\frac{3 \\pi}{10}}{2 \\cos \\frac{\\pi}{10}} \\\\\n& =1+\\frac{\\sin \\frac{2 \\pi}{10} \\cos \\frac{2 \\pi}{10}}{2 \\cos \\frac{\\pi}{10}} \\\\\n& =1+\\frac{\\sin \\frac{4 \\pi}{10}}{4 \\cos \\frac{\\pi}{10}} \\\\\n& =1+\\frac{\\cos \\frac{ \\pi}{10}}{4 \\cos \\frac{\\pi}{10}} \\\\\n& =\\frac{5}{4} . \\\\\n\\end{aligned}\n$$\n解法二设原式 $=A=1+\\cos \\frac{\\pi}{5}+\\cos \\frac{3 \\pi}{5}+\\cos \\frac{\\pi}{5} \\cos \\frac{3 \\pi}{5}$, 令 $\\theta=\\frac{\\pi}{5}$, 则 $2 \\theta=\\pi-3 \\theta$, 于是 $\\cos 2 \\theta+\\cos 3 \\theta=0$, 即\n$$\n4 \\cos ^3 \\theta+2 \\cos ^2 \\theta-3 \\cos \\theta-1=0 .\n$$\n因式分解得 $(\\cos \\theta+1)\\left(4 \\cos ^2 \\theta-2 \\cos \\theta-1\\right)=0$.\n因为 $\\cos \\theta+1 \\neq 0$, 所以 $4 \\cos ^2 \\theta-2 \\cos \\theta-1=0$. 即 $\\cos \\frac{\\pi}{5}$ 是方程 $4 x^2- 2 x-1=0$ 的一根.\n同理可证 $\\cos \\frac{2 \\pi}{5}$ 也是方程 $4 x^2-2 x-1=0$ 的一根.\n由韦达定理得 $\\cos \\frac{\\pi}{5}+\\cos \\frac{3 \\pi}{5}=\\frac{1}{2}, \\cos \\frac{\\pi}{5} \\cos \\frac{3 \\pi}{5}=-\\frac{1}{4}$, 故 $A=\\frac{5}{4}$.\n评注本题解法一具有很强的技巧性, 在解题过程中, 出现了形如 $\\sin \\frac{\\pi}{10} \\sin \\frac{3 \\pi}{10}$ 的算式.\n当它乘以 $\\cos \\frac{\\pi}{10}$ 后, 则可连续应用二倍角正弦公式进行计算化简.\n与此类似的如:\n$$\n\\begin{aligned}\n\\cos \\frac{2 \\pi}{7} \\cos \\frac{4 \\pi}{7} \\cos \\frac{8 \\pi}{7} & =\\frac{8 \\sin \\frac{2 \\pi}{7} \\cos \\frac{2 \\pi}{7} \\cos \\frac{4 \\pi}{7} \\cos \\frac{8 \\pi}{7}}{8 \\sin \\frac{2 \\pi}{7}} \\\\\n& =\\frac{4 \\sin \\frac{4 \\pi}{7} \\cos \\frac{4 \\pi}{7} \\cos \\frac{8 \\pi}{7}}{8 \\sin \\frac{2 \\pi}{7}} \\\\\n& =\\frac{2 \\sin \\frac{8 \\pi}{7} \\cos \\frac{8 \\pi}{7}}{8 \\sin \\frac{2 \\pi}{7}}=\\frac{\\sin \\frac{16}{7} \\pi}{8 \\sin \\frac{2 \\pi}{7}} \\\\\n& =\\frac{\\sin \\frac{2 \\pi}{7}}{8 \\sin \\frac{2 \\pi}{7}}=\\frac{1}{8}, \\\\\n\\cos \\frac{2 \\pi}{7} \\cos \\frac{4 \\pi}{7} \\cos \\frac{6 \\pi}{7} & =\\cos \\frac{\\pi}{7} \\cos \\frac{2 \\pi}{7} \\cos \\frac{3 \\pi}{7} .\n\\end{aligned}\n$$\n而\n$$\n\\cos \\frac{2 \\pi}{7} \\cos \\frac{4 \\pi}{7} \\cos \\frac{6 \\pi}{7}=\\cos \\frac{\\pi}{7} \\cos \\frac{2 \\pi}{7} \\cos \\frac{3 \\pi}{7}\n$$\n我们再观察一下: $\\cos \\frac{\\pi}{3}=\\frac{1}{2}, \\cos \\frac{\\pi}{5} \\cos \\frac{2 \\pi}{5}=\\frac{1}{4}$, 于是, 归纳猜想\n$$\n\\cos \\frac{\\pi}{2 n+1} \\cos \\frac{2 \\pi}{2 n+1} \\cos \\frac{3 \\pi}{2 n+1} \\cdots \\cos \\frac{n \\pi}{2 n+1}=\\frac{1}{2^n} .\n$$\n证明如下: 令\n$$\n\\begin{aligned}\n& S=\\cos \\frac{\\pi}{2 n+1} \\cos \\frac{2 \\pi}{2 n+1} \\cos \\frac{3 \\pi}{2 n+1} \\cdots \\cos \\frac{n \\pi}{2 n+1}, \\\\\n& T=\\sin \\frac{\\pi}{2 n+1} \\sin \\frac{2 \\pi}{2 n+1} \\sin \\frac{3 \\pi}{2 n+1} \\cdots \\sin \\frac{n \\pi}{2 n+1},\n\\end{aligned}\n$$\n则\n$$\n\\begin{aligned}\nS T & =\\frac{1}{2^n} \\sin \\frac{2 \\pi}{2 n+1} \\sin \\frac{4 \\pi}{2 n+1} \\sin \\frac{6 \\pi}{2 n+1} \\cdots \\sin \\frac{2 n \\pi}{2 n+1} \\\\\n& =\\frac{1}{2^n} \\sin \\frac{2 \\pi}{2 n+1} \\sin \\frac{4 \\pi}{2 n+1} \\cdots \\sin \\frac{3 \\pi}{2 n+1} \\sin \\frac{\\pi}{2 n+1} \\\\\n& =\\frac{1}{2^n} T .\n\\end{aligned}\n$$\n从而 $S=\\frac{1}{2^n}$.", + "remark": "", + "figures": [] +} \ No newline at end of file diff --git a/processed_dataset/proof/0959.json b/processed_dataset/proof/0959.json new file mode 100644 index 0000000000000000000000000000000000000000..399be4d719ae8a8789659b24061967f1c0646a4e --- /dev/null +++ b/processed_dataset/proof/0959.json @@ -0,0 +1,8 @@ +{ + "source_file": "./raw_volume-zh/volume3/chapter2.tex", + "problem_type": "proof", + "problem": "例5 已知 $\\frac{\\sin ^4 x}{a}+\\frac{\\cos ^4 x}{b}=\\frac{1}{a+b}$, 求证: $\\frac{\\sin ^{4 n} x}{a^{2 n-1}}+\\frac{\\cos ^{4 n} x}{b^{2 n-1}}=\\frac{1}{(a+b)^{2 n-1}}$, $n \\in \\mathbf{N}^*$.", + "solution": "证明:一令 $\\left\\{\\begin{array}{l}u=\\sin ^2 x, \\\\ v=\\cos ^2 x,\\end{array}\\right.$ 则 $0 \\leqslant u, v \\leqslant 1$, 且 $u+v=1$.\n因为\n$$\n\\left\\{\\begin{array} { l } \n{ \\frac { u ^ { 2 } } { a } + \\frac { v ^ { 2 } } { b } = \\frac { 1 } { a + b } } \\\\\n{ u + v = 1 }\n\\end{array} \\Leftrightarrow \\left\\{\\begin{array}{l}\nu=\\frac{a}{a+b}, \\\\\nv=\\frac{b}{a+b},\n\\end{array}\\right.\\right.\n$$\n所以 $\\frac{\\sin ^{4 n} x}{a^{2 n-1}}+\\frac{\\cos ^{4 n} x}{b^{2 n-1}}=\\frac{u^{2 n}}{a^{2 n-1}}+\\frac{v^{2 n}}{b^{2 n-1}}=\\frac{\\left(\\frac{a}{a+b}\\right)^{2 n}}{a^{2 n-1}}+\\frac{\\left(\\frac{b}{a+b}\\right)^{2 n}}{b^{2 n-1}}$\n$$\n=\\frac{a}{(a+b)^{2 n}}+\\frac{b}{(a+b)^{2 n}}=\\frac{1}{(a+b)^{2 n-1}} \\text {. }\n$$\n证明二因为 $\\frac{\\sin ^4 x}{a}+\\frac{\\cos ^4 x}{b} \\geqslant \\frac{\\left(\\sin ^2 x+\\cos ^2 x\\right)^2}{a+b}=\\frac{1}{a+b}$,\n当且仅当 $\\frac{\\sin ^2 x}{a}=\\frac{\\cos ^2 x}{b}$ 即 $\\frac{\\sin ^2 x}{a}=\\frac{\\cos ^2 x}{b}=\\frac{\\sin ^2 x+\\cos ^2 x}{a+b}=\\frac{1}{a+b}$ 时,等号成立.\n所以 $\\sin ^2 x=\\frac{a}{a+b}, \\cos ^2 x=\\frac{b}{a+b}$, 代入即可.\n评注证明二中我们利用不等式的方式来证明等式, 有时是迫不得已, 有时是出奇制胜.", + "remark": "", + "figures": [] +} \ No newline at end of file diff --git a/processed_dataset/proof/0960.json b/processed_dataset/proof/0960.json new file mode 100644 index 0000000000000000000000000000000000000000..4992995d6fc627851ca8c0320dc3e5a20077bf49 --- /dev/null +++ b/processed_dataset/proof/0960.json @@ -0,0 +1,8 @@ +{ + "source_file": "./raw_volume-zh/volume3/chapter2.tex", + "problem_type": "proof", + "problem": "例6 求证: 对于任意 $\\alpha \\in(-\\infty,+\\infty),\\left|2 \\cos ^2 \\alpha+a \\cos \\alpha+b\\right| \\leqslant 1$ 成立的充要条件是 $2 \\cos ^2 \\alpha+a \\cos \\alpha+b==\\cos 2 \\alpha$.", + "solution": "证明:充分性: 当 $2 \\cos ^2 \\alpha+a \\cos \\alpha+b=\\cos 2 \\alpha$ 时, 显然有\n$$\n\\left|2 \\cos ^2 \\alpha+a \\cos \\alpha+b\\right| \\leqslant 1 \\text {. }\n$$\n必要性:若 $\\left|2 \\cos ^2 \\alpha+a \\cos \\alpha+b\\right| \\leqslant 1$ 对于任意 $\\alpha \\in(-\\infty,+\\infty)$ 均成立, 则 \n(1) 当 $b>-1$ 时,\n$$\n2 \\cos ^2 \\alpha+a \\cos \\alpha+b=2 \\cos ^2 \\alpha-1+a \\cos \\alpha+(b+1) .\n$$\n由于 $b+1>0$, 取 $\\alpha$, 使 $a \\cos \\alpha=|a|$, 则\n$$\n\\left|2 \\cos ^2 \\alpha+a \\cos \\alpha+b\\right|=1+|a|+(b+1)>1+|a| \\geqslant 1,\n$$\n矛盾.\n(2) 当 $b<-1$ 时, 令 $\\cos \\alpha=0$, 则\n$$\n\\left|2 \\cos ^2 \\alpha+a \\cos \\alpha+b\\right|=|b|>1 \\text {, }\n$$\n矛盾.\n由(1), (2) 可知: $b=-1$, 现取 $\\alpha$, 使 $a \\cos \\alpha=|a|$, 则\n$$\n\\left|2 \\cos ^2 \\alpha+a \\cos \\alpha+b\\right|=1+|a| \\geqslant 1,\n$$\n故 $|a|=0$, 即 $a=0$.\n这样 $2 \\cos ^2 \\alpha+a \\cos \\alpha+b=2 \\cos ^2 \\alpha-1=\\cos 2 \\alpha$.", + "remark": "", + "figures": [] +} \ No newline at end of file diff --git a/processed_dataset/proof/0961.json b/processed_dataset/proof/0961.json new file mode 100644 index 0000000000000000000000000000000000000000..209acd01fa942f099d2a5bf1ce6f0be239890c94 --- /dev/null +++ b/processed_dataset/proof/0961.json @@ -0,0 +1,8 @@ +{ + "source_file": "./raw_volume-zh/volume3/chapter2.tex", + "problem_type": "proof", + "problem": "例7 已知锐角 $\\alpha 、 \\beta$ 满足 $\\sin ^2 \\alpha+\\sin ^2 \\beta=\\sin (\\alpha+ \\beta)$, 求证: $\\alpha+\\beta=\\frac{\\pi}{2}$.", + "solution": "证明:一 \n$$\n\\begin{aligned}\n& \\sin ^2 \\alpha+\\sin ^2 \\beta=\\sin \\alpha \\cos \\beta+\\cos \\alpha \\sin \\beta \\\\\n\\Leftrightarrow & \\sin \\alpha(\\sin \\alpha-\\cos \\beta)=-\\sin \\beta(\\sin \\beta-\\cos \\alpha) \\\\\n\\Leftrightarrow & \\sin \\alpha\\left[\\sin \\alpha-\\sin \\left(90^{\\circ}-\\beta\\right)\\right]+\\sin \\beta\\left[\\sin \\beta-\\sin \\left(90^{\\circ}-\\alpha\\right)\\right]=0 \\\\\n\\Leftrightarrow & 2 \\sin \\alpha \\cos \\left(\\frac{\\alpha-\\beta}{2}+\\frac{\\pi}{4}\\right) \\sin \\left(\\frac{\\alpha+\\beta}{2}-\\frac{\\pi}{4}\\right)+2 \\sin \\beta \\cos \\left(\\frac{\\beta-\\alpha}{2}+\\frac{\\pi}{4}\\right) \\sin \\left(\\frac{\\alpha+\\beta}{2}-\\frac{\\pi}{4}\\right)=0 \\\\\n\\Leftrightarrow & \\sin \\left(\\frac{\\alpha+\\beta}{2}-\\frac{\\pi}{4}\\right)\\left[\\sin \\alpha \\cos \\left(\\frac{\\alpha-\\beta}{2}+\\frac{\\pi}{4}\\right)+\\sin \\beta \\cos \\left(\\frac{\\beta-\\alpha}{2}+\\frac{\\pi}{4}\\right)\\right]=0 .\n\\end{aligned}\n$$\n不妨设 $\\alpha \\geqslant \\beta$, 则 $\\frac{\\pi}{4}<\\frac{\\alpha-\\beta}{2}+\\frac{\\pi}{4}<\\frac{\\pi}{2},-\\frac{\\pi}{4}<\\frac{\\beta-\\alpha}{2}+\\frac{\\pi}{4}<\\frac{\\pi}{4}$.\n所以 $\\cos \\left(\\frac{\\alpha-\\beta}{2}+\\frac{\\pi}{4}\\right)>0, \\cos \\left(\\frac{\\beta-\\alpha}{2}+\\frac{\\pi}{4}\\right)>0$, 当然 $\\sin \\alpha>0, \\sin \\beta>0$.\n所以 $\\sin \\left(\\frac{\\alpha+\\beta}{2}-\\frac{\\pi}{4}\\right)=0$, 又因为 $-\\frac{\\pi}{4}<\\frac{\\alpha+\\beta}{2}-\\frac{\\pi}{4}<\\frac{\\pi}{4}$, 所以 $\\frac{\\alpha+\\beta}{2}- \\frac{\\pi}{4}=0$, 即 $\\alpha+\\beta=\\frac{\\pi}{2}$.\n证明二由(1)式知, $\\sin \\alpha-\\cos \\beta$ 与 $\\cos \\alpha-\\sin \\beta$ 同号或同为 0 .\n若 $\\sin \\alpha>\\cos \\beta$ 且 $\\cos \\alpha>\\sin \\beta$, 则 $1=\\sin ^2 \\alpha+\\cos ^2 \\alpha>\\cos ^2 \\beta+\\sin ^2 \\beta=1$, 矛盾.\n若 $\\sin \\alpha<\\cos \\beta$ 且 $\\cos \\alpha<\\sin \\beta$, 则 $1=\\sin ^2 \\alpha+\\cos ^2 \\alpha<\\cos ^2 \\beta+\\sin ^2 \\beta=$ 1 ,矛盾.\n故 $\\cos \\alpha=\\sin \\beta$ 且 $\\sin \\alpha=\\cos \\beta$, 又因为 $\\alpha, \\beta \\in\\left(0, \\frac{\\pi}{2}\\right)$, 故 $\\alpha+\\beta=\\frac{\\pi}{2}$.\n评注本题是俄罗斯数学竞赛试题,有时正面不好证明的情况下, 可考虑反证法.", + "remark": "", + "figures": [] +} \ No newline at end of file diff --git a/processed_dataset/proof/0962.json b/processed_dataset/proof/0962.json new file mode 100644 index 0000000000000000000000000000000000000000..caf40b8ac4fca5c8747e333c63d1f22958c52fcc --- /dev/null +++ b/processed_dataset/proof/0962.json @@ -0,0 +1,8 @@ +{ + "source_file": "./raw_volume-zh/volume3/chapter2.tex", + "problem_type": "proof", + "problem": "例8 已知 $\\alpha 、 \\beta$ 为锐角, 则 $\\alpha+\\beta=\\frac{\\pi}{2}$ 的充要条件为: $\\frac{\\sin ^4 \\alpha}{\\cos ^2 \\beta}+\\frac{\\cos ^4 \\alpha}{\\sin ^2 \\beta}=1$.", + "solution": "证明:必要性: 因为 $\\alpha+\\beta=\\frac{\\pi}{2}$, 则 $\\sin ^2 \\alpha=\\cos ^2 \\beta, \\cos ^2 \\alpha=\\sin ^2 \\beta$, 则左边 $=\\sin ^2 \\alpha+\\cos ^2 \\alpha==1=$ 右边.\n充分性:\n证法一由 Cauchy 不等式知\n$$\n\\frac{\\sin ^4 \\alpha}{\\cos ^2 \\beta}+\\frac{\\cos ^4 \\alpha}{\\sin ^2 \\beta} \\geqslant \\frac{\\left(\\sin ^2 \\alpha+\\cos ^2 \\alpha\\right)^2}{\\cos ^2 \\beta+\\sin ^2 \\beta}=\\frac{1^2}{1}=1 .\n$$\n当且仅当\n$$\n\\frac{\\sin ^2 \\alpha}{\\cos ^2 \\beta}=\\frac{\\cos ^2 \\alpha}{\\sin ^2 \\beta},\n$$\n即\n$$\n\\sin ^2 \\alpha \\sin ^2 \\beta-\\cos ^2 \\alpha \\cos ^2 \\beta=0,\n$$\n即 $(\\cos \\alpha \\cos \\beta-\\sin \\alpha \\sin \\beta)(\\cos \\alpha \\cos \\beta+\\sin \\alpha \\sin \\beta)=0$,\n即 $\\cos (\\alpha+\\beta) \\cos (\\alpha-\\beta)=0$ 时等号成立.\n又因为 $\\alpha 、 \\beta \\in\\left(0, \\frac{\\pi}{2}\\right)$, 则 $-\\frac{\\pi}{2}<\\alpha-\\beta<\\frac{\\pi}{2}, 0<\\alpha+\\beta<\\pi$, 所以 $\\alpha+\\beta= \\frac{\\pi}{2}$.\n证法二\n$$\n\\begin{aligned}\n& \\frac{\\sin ^4 \\alpha}{\\cos ^2 \\beta}+\\cos ^2 \\beta \\geqslant 2 \\sin ^2 \\alpha, \\\\\n& \\frac{\\cos ^4 \\alpha}{\\sin ^2 \\beta}+\\sin ^2 \\beta \\geqslant 2 \\cos ^2 \\alpha,\n\\end{aligned}\n$$\n相加得: $\\frac{\\sin ^4 \\alpha}{\\cos ^2 \\beta}+\\frac{\\cos ^4 \\alpha}{\\sin ^2 \\beta} \\geqslant 1$, 所以 $\\left\\{\\begin{array}{l}\\sin ^2 \\alpha=\\cos ^2 \\beta, \\\\ \\cos ^2 \\alpha=\\sin ^2 \\beta,\\end{array}\\right.$ 同上可得.", + "remark": "", + "figures": [] +} \ No newline at end of file diff --git a/processed_dataset/proof/0963.json b/processed_dataset/proof/0963.json new file mode 100644 index 0000000000000000000000000000000000000000..b9b5307417ea6c16433297f8058df6995797957d --- /dev/null +++ b/processed_dataset/proof/0963.json @@ -0,0 +1,8 @@ +{ + "source_file": "./raw_volume-zh/volume3/chapter2.tex", + "problem_type": "proof", + "problem": "推广1 已知 $\\alpha, \\beta \\in\\left(0, \\frac{\\pi}{2}\\right)$, 则证明 $\\alpha+\\beta=\\frac{\\pi}{2}$ 的充要条件为: $\\frac{\\sin ^3 \\alpha}{\\cos \\beta}+ \\frac{\\cos ^3 \\alpha}{\\sin \\beta}=1$.", + "solution": "证明:必要性显然.\n下证充分性:\n证法一 $\\frac{\\sin ^3 \\alpha}{\\cos \\beta}+\\frac{\\cos ^3 \\alpha}{\\sin \\beta}=\\frac{\\sin ^4 \\alpha}{\\cos \\beta \\sin \\alpha}+\\frac{\\cos ^4 \\alpha}{\\sin \\beta \\cos \\alpha} \\geqslant \\frac{\\left(\\sin ^2 \\alpha+\\cos ^2 \\alpha\\right)^2}{(\\cos \\beta \\sin \\alpha+\\sin \\beta \\cos \\alpha)} =\\frac{1}{\\sin (\\alpha+\\beta)} \\geqslant 1$, 当且仅当 $\\left\\{\\begin{array}{l}\\frac{\\sin ^2 \\alpha}{\\cos \\beta \\sin \\alpha}=\\frac{\\cos ^2 \\alpha}{\\sin \\beta \\cos \\alpha}, \\\\ \\alpha+\\beta=\\frac{\\pi}{2},\\end{array}\\right.$ 即 $\\alpha+\\beta=\\frac{\\pi}{2}$ 时取“=”. \n证法二 $\\frac{\\sin ^3 \\alpha}{\\cos \\beta}+\\frac{\\sin ^3 \\alpha}{\\cos \\beta}+\\cos ^2 \\beta \\geqslant 3 \\sqrt[3]{\\frac{\\sin ^3 \\alpha}{\\cos \\beta} \\cdot \\frac{\\sin ^3 \\alpha}{\\cos \\beta} \\cdot \\cos ^2 \\beta}=3 \\sin ^2 \\alpha$, 同理, $\\frac{\\cos ^3 \\alpha}{\\sin \\beta}+\\frac{\\cos ^3 \\alpha}{\\sin \\beta}+\\sin ^2 \\beta \\geqslant 3 \\cos ^2 \\alpha$, 两式相加即有: $\\frac{\\sin ^3 \\alpha}{\\cos \\beta}+\\frac{\\cos ^3 \\alpha}{\\sin \\beta} \\geqslant \\sin ^2 \\alpha+\\cos ^2 \\alpha=1$, 当且仅当 $\\left\\{\\begin{array}{l}\\sin ^2 \\alpha=\\cos ^2 \\beta, \\\\ \\cos ^2 \\alpha=\\sin ^2 \\beta,\\end{array}\\right.$ 即 $\\alpha+\\beta=\\frac{\\pi}{2}$ 时取“=”.", + "remark": "", + "figures": [] +} \ No newline at end of file diff --git a/processed_dataset/proof/0964.json b/processed_dataset/proof/0964.json new file mode 100644 index 0000000000000000000000000000000000000000..824158ee0d948cb4911d1ed13923d5436f15ae57 --- /dev/null +++ b/processed_dataset/proof/0964.json @@ -0,0 +1,8 @@ +{ + "source_file": "./raw_volume-zh/volume3/chapter2.tex", + "problem_type": "proof", + "problem": "推广2 已知 $\\alpha, \\beta \\in\\left(0, \\frac{\\pi}{2}\\right)$, 则证明 $\\alpha+\\beta=\\frac{\\pi}{2}$ 的充要条件为:\n$$\n\\frac{\\sin ^{k+2} \\alpha}{\\cos ^k \\beta}+\\frac{\\cos ^{k+2} \\alpha}{\\sin ^k \\beta}=1, k \\in \\mathbf{N}^* .\n$$", + "solution": "证明:必要性显然.\n下证充分性:\n证法一 (i)当 $k=2 m, m \\in \\mathbf{N}^*$ 时,\n$$\n\\begin{aligned}\n& \\frac{\\sin ^{2 m+2} \\alpha}{\\cos ^{2 m} \\beta}+\\underbrace{\\cos ^2 \\beta+\\cos ^2 \\beta+\\cdots+\\cos ^2 \\beta}_{m 个} \\\\\n\\geqslant & (m+1) \\cdot \\sqrt[m+1]{\\frac{\\sin ^{2 m+2} \\alpha}{\\cos ^{2 m} \\beta} \\cdot \\underbrace{\\cos ^2 \\beta \\cdots \\cos ^2 \\beta}_{m 个}} \\\\\n= & (m+1) \\sin ^2 \\alpha ; \\\\\n& \\frac{\\cos ^{2 m+2} \\alpha}{\\sin ^{2 m} \\beta}+\\underbrace{\\sin ^2 \\beta+\\sin ^2 \\beta+\\cdots+\\sin ^2 \\beta}_{m 个} \\\\\n\\geqslant & (m+1) \\cdot \\sqrt[m+1]{\\frac{\\cos ^{2 m+2} \\alpha}{\\sin ^{2 m} \\beta} \\cdot \\underbrace{\\sin ^2 \\beta \\cdot \\sin ^2 \\beta \\cdots \\sin ^2 \\beta}_{m 个}} \\\\\n= & (m+1) \\cos ^2 \\alpha,\n\\end{aligned}\n$$\n两式相加得: $\\frac{\\sin ^{2 m+2} \\alpha}{\\cos ^{2 m} \\beta}+\\frac{\\cos ^{2 m+2} \\alpha}{\\sin ^{2 m} \\beta} \\geqslant 1$.\n当且仅当 $\\alpha+\\beta=\\frac{\\pi}{2}$ 时取 “ $=$ ”.\n(ii) 当 $k=2 m-1, m \\in \\mathbf{N}^*$ 时,\n$$\n\\begin{aligned}\n& \\frac{\\sin ^{2 m+1} \\alpha}{\\cos ^{2 m-1} \\beta}+\\frac{\\sin ^{2 m+1} \\alpha}{\\cos ^{2 m-1} \\beta}+\\underbrace{\\cos ^2 \\beta+\\cdots+\\cos ^2 \\beta}_{2 m-1} \\\\\n\\geqslant & (2 m+1) \\cdot \\sqrt[2 m+1]{\\frac{\\sin ^{2 m+1} \\alpha}{\\cos ^{2 m-1} \\beta} \\cdot \\frac{\\sin ^{2 m+1} \\alpha}{\\cos ^{2 m-1} \\beta} \\cdot \\underbrace{\\cos ^2 \\beta \\cdots \\cos ^2 \\beta}_{2 m-1}} \\\\\n= & (2 m+1) \\cdot \\sin ^2 \\alpha ;\\\\\n& \\frac{\\cos ^{2 m+1} \\alpha}{\\sin ^{2 m-1} \\beta}+\\frac{\\cos ^{2 m+1} \\alpha}{\\sin ^{2 m-1} \\beta} +(\\underbrace{\\sin ^2 \\beta+\\cdots+\\sin ^2 \\beta}_{2 m-1} \\geqslant(2 m+1) \\cdot \\cos ^2 \\alpha,\n\\end{aligned}\n$$\n两式相加得: $\\frac{\\sin ^{2 m+1} \\alpha}{\\cos ^{2 m-1} \\beta}+\\frac{\\cos ^{2 m+1} \\alpha}{\\sin ^{2 m-1} \\beta} \\geqslant 1$.\n当且仅当 $\\alpha+\\beta=\\frac{\\pi}{2}$ 时取 “ $=$ ”.\n证法二 $\\frac{\\sin ^{k+2} \\alpha}{\\cos ^k \\beta}+\\underbrace{\\cos \\beta \\sin \\alpha+\\cdots+\\cos \\beta \\sin \\alpha}_{k \\uparrow} \\geqslant(k+1) \\sin ^2 \\alpha\\left(A_{k+1} \\geqslant G_{k+1}\\right)$;\n$$\n\\frac{\\cos ^{k+2} \\alpha}{\\sin ^k \\beta}+\\underbrace{\\cos \\alpha \\sin \\beta+\\cdots+\\cos \\alpha \\sin \\beta}_{k \\uparrow} \\geqslant(k+1) \\cos ^2 \\alpha\\left(A_{k+1} \\geqslant G_{k+1}\\right),\n$$\n两式相加得:\n$$\n\\frac{\\sin ^{k+2} \\alpha}{\\cos ^k \\beta}+\\frac{\\cos ^{k+2} \\alpha}{\\sin ^k \\beta} \\geqslant(k+1)-k \\sin (\\alpha+\\beta) \\geqslant 1 .\n$$\n当且仅当 $\\alpha+\\beta=\\frac{\\pi}{2}$ 时取 “ $=$”.", + "remark": "", + "figures": [] +} \ No newline at end of file diff --git a/processed_dataset/proof/0965.json b/processed_dataset/proof/0965.json new file mode 100644 index 0000000000000000000000000000000000000000..5921364d9ed0f4a3bc5f0af307ce2de54411dc05 --- /dev/null +++ b/processed_dataset/proof/0965.json @@ -0,0 +1,8 @@ +{ + "source_file": "./raw_volume-zh/volume3/chapter2.tex", + "problem_type": "proof", + "problem": "推广3 已知 $\\alpha, \\beta \\in\\left(0, \\frac{\\pi}{2}\\right)$, 求证: $\\alpha+\\beta=\\frac{\\pi}{2}$ 的充要条件为 $\\frac{\\sin ^4 \\alpha}{\\cos ^2 \\beta}+ \\frac{\\sin ^4 \\beta}{\\cos ^2 \\alpha}=1$.", + "solution": "证明:必要性显然.\n下证充分性:\n证法一令 $\\sin ^2 \\alpha=a, \\cos ^2 \\beta=b, a, b \\in(0,1)$, 则\n$$\n\\begin{aligned}\n1 & =\\frac{a^2}{b}+\\frac{(1-b)^2}{1-a} \\Leftrightarrow b(1-a)=a^2-a^3+b^3-2 b^2+b \\\\\n& \\Leftrightarrow\\left(b^3-a^3\\right)-\\left(b^2-a^2\\right)-\\left(b^2-a b\\right)=0 \\\\\n& \\Leftrightarrow(b-a)\\left(a^2+a b+b^2-2 b-a\\right)=0,\n\\end{aligned}\n$$\n因为 $a, b \\in(0,1)$, 所以 $a>a^2, b>b^2, b>a b$.\n因为\n$$\na^2+a b+b^2-2 b-a<0,\n$$\n故 $b=a$, 即 $\\sin ^2 \\alpha=\\cos ^2 \\beta$, 所以 $\\alpha+\\beta=\\frac{\\pi}{2}$.\n证法二 \n$$\n\\begin{aligned}\n0 & =\\frac{\\sin ^4 \\alpha}{\\cos ^2 \\beta}+\\frac{\\sin ^4 \\beta}{\\cos ^2 \\alpha}-1=\\frac{\\sin ^4 \\alpha}{\\cos ^2 \\beta}+\\frac{\\sin ^4 \\beta}{\\cos ^2 \\alpha}-\\left(\\cos ^2 \\beta+\\sin ^2 \\beta\\right) \\\\\n& =\\frac{\\sin ^4 \\alpha-\\cos ^4 \\beta}{\\cos ^2 \\beta}+\\frac{\\sin ^2 \\beta\\left(\\sin ^2 \\beta-\\cos ^2 \\alpha\\right)}{\\cos ^2 \\alpha}, \\quad \\quad (1)\n\\end{aligned}\n$$\n因为 $\\alpha, \\beta \\in\\left(0, \\frac{\\pi}{2}\\right)$, 所以 $\\alpha+\\beta \\in(0, \\pi)$.\n若 $0<\\alpha+\\beta<\\frac{\\pi}{2}$, 则\n$\\cos \\alpha>\\cos \\left(\\frac{\\pi}{2}-\\beta\\right)=\\sin \\beta>0$, 所以 $\\cos ^2 \\alpha>\\sin ^2 \\beta$;\n$\\cos \\beta>\\cos \\left(\\frac{\\pi}{2}-\\beta\\right)=\\sin \\beta>0$, 所以 $\\cos ^4 \\beta>\\sin ^4 \\alpha$.\n(1)式右边 $<0$ 矛盾.\n同理, 若 $\\frac{\\pi}{2}<\\alpha+\\beta<\\pi$, 则(1)式右边 $>0$, 矛盾.\n如证法二, 直接可将结论拓展到:\n$$\n\\frac{\\sin ^{k+2} \\alpha}{\\cos ^k \\beta}+\\frac{\\sin ^{k+2} \\beta}{\\cos ^k \\alpha}=1, k>0 .\n$$\n证法二也可证明例 8 的充分性:\n事实上: $\\frac{\\sin ^4 \\alpha}{\\cos ^2 \\beta}+\\frac{\\cos ^4 \\alpha}{\\sin ^2 \\beta}=1 \\Leftrightarrow \\frac{\\sin ^4 \\alpha}{\\cos ^2 \\beta}+\\frac{\\cos ^4 \\alpha}{\\sin ^2 \\beta}=\\sin ^2 \\alpha+\\cos ^2 \\alpha$\n$$\n\\Leftrightarrow \\frac{\\sin ^2 \\alpha\\left(\\sin ^2 \\alpha-\\cos ^2 \\beta\\right)}{\\cos ^2 \\beta}+\\frac{\\cos ^2 \\alpha\\left(\\cos ^2 \\alpha-\\sin ^2 \\beta\\right)}{\\sin ^2 \\beta}=0 . \\quad\\quad (*)\n$$\n因为 $\\alpha, \\beta \\in(0, \\pi)$, 若 $\\alpha+\\beta \\neq \\frac{\\pi}{2}$, 则 $0<\\alpha+\\beta<\\frac{\\pi}{2}$, 或 $\\frac{\\pi}{2}<\\alpha+\\beta<\\pi$.\n若 $0<\\alpha+\\beta<\\frac{\\pi}{2}$, 则 $\\cos \\alpha>\\sin \\beta>0, \\cos \\beta>\\sin \\alpha>0$, 所以\n$$\n\\cos ^2 \\alpha>\\sin ^2 \\beta>0, \\sin ^2 \\alpha<\\cos ^2 \\beta \\text {. }\n$$\n即\n$$\n\\frac{\\sin ^2 \\alpha}{\\cos ^2 \\beta}<1, \\frac{\\cos ^2 \\alpha}{\\sin \\beta}>1\n$$\n所以 $\\frac{\\sin ^2 \\alpha\\left(\\sin ^2 \\alpha-\\cos ^2 \\beta\\right)}{\\cos ^2 \\beta}>\\sin ^2 \\alpha-\\cos ^2 \\beta$,\n$$\n\\frac{\\cos ^2 \\alpha\\left(\\cos ^2 \\alpha-\\sin ^2 \\beta\\right)}{\\sin ^2 \\beta}>\\cos ^2 \\alpha-\\sin ^2 \\beta\n$$\n$\\sin ^2 \\beta=0$.\n这与 $(*)$ 矛盾.\n同理,若 $\\frac{\\pi}{2}<\\alpha+\\beta<\\pi,(*)$ 式左边大于 0 ,矛盾.", + "remark": "", + "figures": [] +} \ No newline at end of file diff --git a/processed_dataset/proof/0966.json b/processed_dataset/proof/0966.json new file mode 100644 index 0000000000000000000000000000000000000000..3d77d17ceb03461d35b5b4fe0394d39b5cb2e84d --- /dev/null +++ b/processed_dataset/proof/0966.json @@ -0,0 +1,8 @@ +{ + "source_file": "./raw_volume-zh/volume3/chapter2.tex", + "problem_type": "proof", + "problem": "例11 已知 $0<\\alpha<\\beta<\\gamma<2 \\pi$, 且 $\\sin \\alpha+\\sin \\beta+\\sin \\gamma=0, \\cos \\alpha+ \\cos \\beta+\\cos \\gamma=0$. (1) 求 $\\beta-\\alpha$ 的值; (2) 求证: $\\cos ^2 \\alpha+\\cos ^2 \\beta+\\cos ^2 \\gamma$ 为定值.", + "solution": "分析:通过消元法将 $\\gamma$ 消去, 得关于 $\\alpha 、 \\beta$ 的关系式, 从而求得 $\\beta-\\alpha$ 的某一三角函数值, 进而求出 $\\beta-\\alpha$ 的值; 根据轮换对称式的性质, 求得 $\\alpha 、 \\beta 、 \\gamma$ 三者关系,最后计算 $\\cos ^2 \\alpha+\\cos ^2 \\beta+\\cos ^2 \\gamma$ 之值.\n解 (1) 由条件得 $\\sin \\alpha+\\sin \\beta=-\\sin \\gamma, \\cos \\alpha+\\cos \\beta=-\\cos \\gamma$, 两式平方相加得 $2+2 \\cos (\\alpha-\\beta)=1$, 于是 $\\cos (\\alpha-\\beta)=-\\frac{1}{2}$. 因为 $0<\\alpha<\\beta<2 \\pi$, 所以 $\\beta-\\alpha=\\frac{2 \\pi}{3}$ 或 $\\frac{4 \\pi}{3}$.\n但是, 当 $\\beta-\\alpha=\\frac{4 \\pi}{3}$ 时, 根据题中等式, 同理可得 $\\gamma-\\beta=\\frac{2 \\pi}{3}$ 或 $\\frac{4 \\pi}{3}$. 从而 $\\gamma \\geqslant \\beta+\\frac{2 \\pi}{3}=\\alpha+\\frac{4 \\pi}{3}+\\frac{2 \\pi}{3}=\\alpha+2 \\pi>2 \\pi$, 这与题设矛盾, 所以 $\\beta-\\alpha=\\frac{2 \\pi}{3}$.\n(2) 由(1)可以推知: $\\gamma-\\beta=\\frac{2 \\pi}{3}, \\beta-\\alpha=\\frac{2 \\pi}{3}$, 所以\n$$\n\\begin{aligned}\n& \\cos ^2 \\alpha+\\cos ^2 \\beta+\\cos ^2 \\gamma=\\cos ^2 \\alpha+\\cos ^2\\left(\\frac{2 \\pi}{3}+\\alpha\\right)+\\cos ^2\\left(\\frac{4 \\pi}{3}+\\alpha\\right) \\\\\n= & \\cos ^2 \\alpha+\\left(\\cos \\frac{2 \\pi}{3} \\cos \\alpha-\\sin \\frac{2 \\pi}{3} \\sin \\alpha\\right)^2+\\left(\\cos \\frac{4 \\pi}{3} \\cos \\alpha-\\sin \\frac{4 \\pi}{3} \\sin \\alpha\\right)^2 \\\\\n= & \\cos ^2 \\alpha+\\left(\\frac{1}{2} \\cos \\alpha+\\frac{\\sqrt{3}}{2} \\sin \\alpha\\right)^2+\\left(-\\frac{1}{2} \\cos \\alpha+\\frac{\\sqrt{3}}{2} \\sin \\alpha\\right)^2 \\\\\n= & \\cos ^2 \\alpha+\\frac{1}{4} \\cos ^2 \\alpha+\\frac{\\sqrt{3}}{2} \\sin \\alpha \\cos \\alpha+\\frac{3}{4} \\sin ^2 \\alpha +\\frac{1}{4} \\cos ^2 \\alpha-\\frac{\\sqrt{3}}{2} \\sin \\alpha \\cos \\alpha+\\frac{3}{4} \\sin ^2 \\alpha \\\\\n= & \\frac{3}{2} \\cos ^2 \\alpha+\\frac{3}{2} \\sin ^2 \\alpha=\\frac{3}{2} .\n\\end{aligned}\n$$\n评注本题容易出现的错误是认为 $\\beta-\\alpha$ 可为 $\\frac{4 \\pi}{3}$. 其实通过估算, 可发现矛盾.\n当我们学习了平面向量以后, 还可通过构造法证明本题.\n即在平面直角坐标系中, 设三点 $A(\\cos \\alpha, \\sin \\alpha), B(\\cos \\beta, \\sin \\beta), C(\\cos \\gamma, \\sin \\gamma)$, 则 $\\triangle A B C$ 的三个顶点都在以 $O$ 为圆心的单位圆上, 又根据三角形重心坐标公式得 $x_G=\\frac{1}{3}(\\cos \\alpha+\\cos \\beta+\\cos \\gamma)=0, y_G=\\frac{1}{3}(\\sin \\alpha+\\sin \\beta+\\sin \\gamma)=0$, 即重心 $G$ 与外心 $O$ 重合, 于是 $\\triangle A B C$ 为单位圆的内接正三角形, 从而 $\\gamma-\\beta= \\beta-\\alpha=\\frac{2 \\pi}{3}$.", + "remark": "", + "figures": [] +} \ No newline at end of file diff --git a/processed_dataset/proof/0967.json b/processed_dataset/proof/0967.json new file mode 100644 index 0000000000000000000000000000000000000000..56c6c776f6a2cf9e74e812c9171b53948e6c0a4f --- /dev/null +++ b/processed_dataset/proof/0967.json @@ -0,0 +1,8 @@ +{ + "source_file": "./raw_volume-zh/volume3/chapter2.tex", + "problem_type": "proof", + "problem": "例12 证明对所有 $n \\geqslant 2$ 的自然数 $n$, 有等式\n$$\n\\prod_{i=1}^n \\tan \\left[\\frac{\\pi}{3}\\left(1+\\frac{3^i}{3^n-1}\\right)\\right]=\\prod_{i=1}^n \\cot \\left[\\frac{\\pi}{3}\\left(1-\\frac{3^i}{3^n-1}\\right)\\right]\n$$\n这里 $\\prod_{k=1}^n k=1 \\times 2 \\times \\cdots \\times n$.", + "solution": "证明:令 $A_i=\\tan \\left[\\frac{\\pi}{3}\\left(1+\\frac{3^i}{3^n-1}\\right)\\right], B_i=\\tan \\left[\\frac{\\pi}{3}\\left(1-\\frac{3^i}{3^n-1}\\right)\\right]$, 因为 $\\tan 3 \\theta=\\tan \\theta \\tan \\left(\\frac{\\pi}{3}-\\theta\\right) \\tan \\left(\\frac{\\pi}{3}+\\theta\\right)$, 取 $\\theta=\\frac{\\pi}{3}\\left(1+\\frac{3^i}{3^n-1}\\right)$, 得\n$$\n\\tan \\pi\\left(1+\\frac{3^i}{3^n-1}\\right)=\\tan \\frac{\\pi}{3}\\left(1+\\frac{3^i}{3^n-1}\\right) \\tan \\left[\\frac{\\pi}{3}-\\frac{\\pi}{3}\\left(1+\\frac{3^i}{3^n-1}\\right)\\right] \\tan \\left[\\frac{\\pi}{3}+\\frac{\\pi}{3}\\left(1+\\frac{3^i}{3^n-1}\\right)\\right],\n$$\n化简得 $\\tan \\left[\\frac{\\pi}{3}\\left(1+\\frac{3^i}{3^n-1}\\right)\\right] \\cdot \\tan \\left[\\frac{\\pi}{3}\\left(1-\\frac{3^i}{3^n-1}\\right)\\right]=1$, 于是 $\\prod_{i=1}^n A_i B_i=1$, 变形即得证.", + "remark": "", + "figures": [] +} \ No newline at end of file diff --git a/processed_dataset/proof/0968.json b/processed_dataset/proof/0968.json new file mode 100644 index 0000000000000000000000000000000000000000..2b3b49b3f512409bc5f9f2c3aadb95e64335c2d1 --- /dev/null +++ b/processed_dataset/proof/0968.json @@ -0,0 +1,8 @@ +{ + "source_file": "./raw_volume-zh/volume3/chapter2.tex", + "problem_type": "proof", + "problem": "例13 证明下列三角恒等式:\n(1) $\\frac{1}{\\sin 2 x}+\\frac{1}{\\sin 4 x}+\\frac{1}{\\sin 8 x}+\\cdots+\\frac{1}{\\sin 2^n x}=\\cot x-\\cot 2^n x$;\n(2) $\\frac{1}{2} \\tan \\frac{x}{2}+\\frac{1}{2^2} \\tan \\frac{x}{2^2}+\\cdots+\\frac{1}{2^n} \\tan \\frac{x}{2^n}=\\frac{1}{2^n} \\cot \\frac{x}{2^n}-\\cot x$.", + "solution": "分析:利用数列知识证明三角恒等式.\n证明 (1) 记 $a_n=\\cot x-\\cot 2^n x$, 则\n$$\n\\begin{gathered}\na_1=\\cot x-\\cot 2 x=\\frac{\\cos x}{\\sin x}-\\frac{\\cos 2 x}{\\sin 2 x}=\\frac{2 \\cos ^2 x-\\cos 2 x}{2 \\sin x \\cos x}=\\frac{1}{\\sin 2 x}, \\\\\n\\begin{aligned}\na_{k+1}-a_k= & \\left(\\cot x-\\cot 2^{k+1} x\\right)-\\left(\\cot x-\\cot 2^k x\\right) \\\\\n= & \\cot 2^k x-\\cot 2^{k+1} x=\\frac{\\cos 2^k x}{\\sin 2^k x}-\\frac{\\cos 2^{k+1} x}{\\sin 2^{k+1} x} \\\\\n= & \\frac{2\\left(\\cos 2^k x\\right)^2-\\cos 2^{k+1} x}{2 \\sin 2^k x \\cdot \\cos 2^k x}=\\frac{1}{\\sin 2^{k+1} x} .\n\\end{aligned}\n\\end{gathered}\n$$\n于是左边 $=a_1+\\left(a_2-a_1\\right)+\\left(a_3-a_2\\right)+\\cdots+\\left(a_n-a_{n-1}\\right)$\n$$\n=a_n=\\cot x-\\cot 2^n x \\text {. }\n$$\n(2) 记 $a_n=\\frac{1}{2^n} \\cot \\frac{x}{2^n}-\\cot x$, 则\n$$\n\\begin{aligned}\na_1=\\frac{1}{2} \\cot \\frac{x}{2}-\\cot x=\\frac{1}{2 \\tan \\frac{x}{2}}-\\frac{1-\\tan ^2 \\frac{x}{2}}{2 \\tan \\frac{x}{2}}=\\frac{1}{2} \\tan \\frac{x}{2}, \\\\\na_{k+1}-a_k=\\left(\\frac{1}{2^{k+1}} \\cot \\frac{x}{2^{k+1}}-\\cot x\\right)-\\left(\\frac{1}{2^k} \\cot \\frac{x}{2^k}-\\cot x\\right) \\\\\n=\\frac{1}{2^{k+1}}\\left(\\cot \\frac{x}{2^{k+1}}-2 \\cot \\frac{x}{2^k}\\right) \\\\\n=\\frac{1}{2^{k+1}} \\cdot\\left(\\frac{1}{\\tan \\frac{x}{2^{k+1}}}-2 \\times \\frac{1-\\tan ^2 \\frac{x}{2^{k+1}}}{2 \\cdot \\tan \\frac{x}{2^{k+1}}}\\right)=\\frac{1}{2^{k+1}} \\cdot \\tan \\frac{x}{2^{k+1}},\n\\end{aligned}\n$$\n所以左边 $=a_1+\\left(a_2-a_1\\right)+\\left(a_3-a_2\\right)+\\cdots+\\left(a_n-a_{n-1}\\right)$\n$$\n=a_n=\\text { 右边.\n}\n$$\n故原等式成立.\n评注本题的左边都是同名三角函数, 前面的系数以及角的度数是有一定规律的,并且与自然数有关,因此证明的思路是: 记 $a_n=$ 右边, 然后求出 $a_1$ 、 $a_{k+1}-a_k$ 或 $\\frac{a_{k+1}}{a_k}$, 即可找到等式左边的每一项与 $a_n$ 的关系, 最后把它们代入等式的左边,再进行计算, 就得到 $a_n$.", + "remark": "", + "figures": [] +} \ No newline at end of file diff --git a/processed_dataset/proof/0969.json b/processed_dataset/proof/0969.json new file mode 100644 index 0000000000000000000000000000000000000000..32489fe58332b0157a861381fe4bd6e96eb0a43a --- /dev/null +++ b/processed_dataset/proof/0969.json @@ -0,0 +1,8 @@ +{ + "source_file": "./raw_volume-zh/volume3/chapter2.tex", + "problem_type": "proof", + "problem": "例16 求下列各式的值:\n(1) $\\cos \\frac{\\pi}{13}+\\cos \\frac{3 \\pi}{13}+\\cos \\frac{5 \\pi}{13}+\\cos \\frac{7 \\pi}{13}+\\cos \\frac{9 \\pi}{13}+\\cos \\frac{11 \\pi}{13}$;\n(2) $\\cos \\frac{\\pi}{13}+\\cos \\frac{3 \\pi}{13}+\\cos \\frac{9 \\pi}{13}$.", + "solution": "分析:反复利用积化和差与和差化积公式计算原式, 当项数较多时, 其方法有两种: 配对或裂项, 本题采用裂项方法.\n解 (1)\n$$\n\\begin{aligned}\n& \\cos \\frac{\\pi}{13}+\\cos \\frac{3 \\pi}{13}+\\cos \\frac{5 \\pi}{13}+\\cos \\frac{7 \\pi}{13}+\\cos \\frac{9 \\pi}{13}+\\cos \\frac{11 \\pi}{13} \\\\\n= & \\frac{1}{2 \\sin \\frac{\\pi}{13}}\\left(2 \\sin \\frac{\\pi}{13} \\cos \\frac{\\pi}{13}+2 \\sin \\frac{\\pi}{13} \\cos \\frac{3 \\pi}{13}+2 \\sin \\frac{\\pi}{13} \\cos \\frac{5 \\pi}{13}\\right. \\\\\n+ & \\left.2 \\sin \\frac{\\pi}{13} \\cos \\frac{7 \\pi}{13}+2 \\sin \\frac{\\pi}{13} \\cos \\frac{9 \\pi}{13}+2 \\sin \\frac{\\pi}{13} \\cos \\frac{11 \\pi}{13}\\right) \\\\\n= & \\frac{1}{2 \\sin \\frac{\\pi}{13}}\\left[\\sin \\frac{2 \\pi}{13}+\\left(\\sin \\frac{4 \\pi}{13}-\\sin \\frac{2 \\pi}{13}\\right)+\\left(\\sin \\frac{6 \\pi}{13}-\\sin \\frac{4 \\pi}{13}\\right)\\right. \\\\\n& \\left.+\\left(\\sin \\frac{8 \\pi}{13}-\\sin \\frac{6 \\pi}{13}\\right)+\\left(\\sin \\frac{10 \\pi}{13}-\\sin \\frac{8 \\pi}{13}\\right) +\\left(\\sin \\frac{12 \\pi}{13}-\\sin \\frac{10 \\pi}{13}\\right)\\right] \\\\\n= & \\frac{1}{2 \\sin \\frac{\\pi}{13}} \\cdot \\sin \\frac{12 \\pi}{13}=\\frac{1}{2} .\n\\end{aligned}\n$$\n(2)\n$$\n\\begin{aligned}\n\\text { 令 } & \\cos \\frac{\\pi}{13}+\\cos \\frac{3 \\pi}{13}+\\cos \\frac{9 \\pi}{13}=x \\text {, 则 } \\\\\nx^2= & \\cos ^2 \\frac{\\pi}{13}+\\cos ^2 \\frac{3 \\pi}{13}+\\cos ^2 \\frac{9 \\pi}{13}+2 \\cos \\frac{\\pi}{13} \\cos \\frac{3 \\pi}{13} \\\\\n& +2 \\cos \\frac{3 \\pi}{13} \\cos \\frac{9 \\pi}{13}+2 \\cos \\frac{\\pi}{13} \\cos \\frac{9 \\pi}{13} \\\\\n= & \\frac{1}{2}\\left(1+\\cos \\frac{2 \\pi}{13}\\right)+\\frac{1}{2}\\left(1+\\cos \\frac{6 \\pi}{13}\\right)+\\frac{1}{2}\\left(1+\\cos \\frac{18 \\pi}{13}\\right) \\\\\n& +\\cos \\frac{2 \\pi}{13}+\\cos \\frac{4 \\pi}{13}+\\cos \\frac{6 \\pi}{13}+\\cos \\frac{12 \\pi}{13}+\\cos \\frac{8 \\pi}{13}+\\cos \\frac{10 \\pi}{13} \\\\\n= & \\frac{3}{2}-\\frac{1}{2}\\left(\\cos \\frac{11 \\pi}{13}+\\cos \\frac{7 \\pi}{13}+\\cos \\frac{5 \\pi}{13}\\right) \\\\\n& -\\left(\\cos \\frac{11 \\pi}{13}+\\cos \\frac{9 \\pi}{13}+\\cos \\frac{7 \\pi}{13}+\\cos \\frac{\\pi}{13}+\\cos \\frac{5 \\pi}{13}+\\cos \\frac{3 \\pi}{13}\\right),\n\\end{aligned}\n$$\n由(1)得\n$$\n\\cos \\frac{11 \\pi}{13}+\\cos \\frac{7 \\pi}{13}+\\cos \\frac{5 \\pi}{13}=\\frac{1}{2}-\\left(\\cos \\frac{9 \\pi}{13}+\\cos \\frac{3 \\pi}{13}+\\cos \\frac{\\pi}{13}\\right)\n$$\n从而\n$$\nx^2=\\frac{3}{2}-\\frac{1}{2}\\left(\\frac{1}{2}-x\\right)-\\frac{1}{2},\n$$\n解之得 $x=\\frac{1 \\pm \\sqrt{13}}{2}$ (负值舍去), 所以\n$$\n\\cos \\frac{\\pi}{13}+\\cos \\frac{3 \\pi}{13}+\\cos \\frac{9 \\pi}{13}=\\frac{1+\\sqrt{13}}{2} .\n$$\n评注由本题第 (1) 题的结论, 再结合算式 $\\cos \\frac{\\pi}{3}, \\cos \\frac{\\pi}{5}+\\cos \\frac{3 \\pi}{5}$, $\\cos \\frac{\\pi}{7}+\\cos \\frac{3 \\pi}{7}+\\cos \\frac{5 \\pi}{7}, \\cos \\frac{\\pi}{9}+\\cos \\frac{3 \\pi}{9}+\\cos \\frac{5 \\pi}{9}+\\cos \\frac{7 \\pi}{9}$ 的值, 我们可以归纳猜想:\n$$\n\\cos \\frac{\\pi}{2 n+1}+\\cos \\frac{3 \\pi}{2 n+1}+\\cos \\frac{5 \\pi}{2 n+1}+\\cdots+\\cos \\frac{2 n-1}{2 n+1} \\pi=\\frac{1}{2} .\n$$\n证明令 $S=\\cos \\frac{\\pi}{2 n+1}+\\cos \\frac{3 \\pi}{2 n+1}+\\cos \\frac{5 \\pi}{2 n+1}+\\cdots+\\cos \\frac{2 n-1}{2 n+1} \\pi$,\n则\n$$\n\\begin{aligned}\n2 \\sin \\frac{\\pi}{2 n+1} \\cdot S= & \\sin \\frac{2 \\pi}{2 n+1}+\\left(\\sin \\frac{4 \\pi}{2 n+1}-\\sin \\frac{2 \\pi}{2 n+1}\\right) \\\\\n& +\\left(\\sin \\frac{6 \\pi}{2 n+1}-\\sin \\frac{4 \\pi}{2 n+1}\\right)+\\cdots \\\\\n& +\\left(\\sin \\frac{2 n}{2 n+1} \\pi-\\sin \\frac{2 n-2}{2 n+1} \\pi\\right) \\\\\n= & \\sin \\frac{2 n}{2 n+1} \\pi .\n\\end{aligned}\n$$\n于是\n$$\nS=\\frac{1}{2} \\text {. }\n$$", + "remark": "", + "figures": [] +} \ No newline at end of file diff --git a/processed_dataset/proof/0970.json b/processed_dataset/proof/0970.json new file mode 100644 index 0000000000000000000000000000000000000000..6e90ffadbad796b93abac7881d09d377eff62da8 --- /dev/null +++ b/processed_dataset/proof/0970.json @@ -0,0 +1,10 @@ +{ + "source_file": "./raw_volume-zh/volume3/chapter3.tex", + "problem_type": "proof", + "problem": "例2 已知 $\\triangle A B C$ 中, $\\angle B A O=\\angle C A O=\\angle C B O=\\angle A C O$.\n求证: $\\triangle A B C$ 三边长为等比数列.", + "solution": "分析:边角关系互化,用三角函数处理也是较好的方法之一,本书多次用三角法解决平面儿何问题.\n证明如图 (), 设 $\\angle B A O=\\angle C B O= \\angle A C O=\\alpha$.\n在 $\\triangle O A B$ 中, 由正弦定理: $\\frac{A B}{\\sin \\angle A O B}=\\frac{O B}{\\sin \\angle B A O}$, 而 $\\sin \\angle A O B=\\sin (\\angle B A O+\\angle A B O)= \\sin (\\angle C B O+\\angle A B O)=\\sin B$.\n所以\n$$\n\\frac{A B}{\\sin B}=\\frac{O B}{\\sin \\alpha} \\quad\\quad (1),\n$$\n同理在 $\\triangle O B C$ 中, 有 $\\frac{B C}{\\sin C}=\\frac{O B}{\\sin (C-\\alpha)} \\quad\\quad (2)$ .\n$(1) \\div (2)$, 有\n$$\n\\frac{A B \\sin C}{B C \\sin B}=\\frac{\\sin (C-\\alpha)}{\\sin \\alpha} \\text {. } \n$$\n$$\n\\text { 即 } \\frac{\\sin ^2 C}{\\sin A \\sin B}=\\frac{\\sin (C-\\alpha)}{\\sin \\alpha} \\text {, 而 } \\frac{\\sin (C-\\alpha)}{\\sin \\alpha}=\\frac{\\sin C \\cos \\alpha-\\cos C \\sin \\alpha}{\\sin \\alpha}=\\sin C \\cot \\alpha-\\cos C\n$$\n所以 $\\frac{\\sin ^2 C}{\\sin A \\sin B}=\\sin C \\cdot \\cot \\alpha-\\cos C$, 则 \n$$\n\\cot \\alpha=\\frac{\\sin C}{\\sin A \\sin B}+\\cot C=\\frac{\\sin (A+B)}{\\sin A \\sin B}+\\cot C=\\frac{\\sin A \\cos B+\\cos A \\sin B}{\\sin A \\sin B}+\\cot C=\\cot A+\\cot B+\\cot C \\text {. }\n$$\n因为 $\\alpha=\\frac{A}{2}$, 所以 $\\cot \\frac{A}{2}=\\cot A+\\cot B+\\cot C$,\n而\n$$\n\\begin{gathered}\n\\cot \\frac{A}{2}-\\cot A=\\frac{\\cos \\frac{A}{2}}{\\sin \\frac{A}{2}}-\\frac{\\cos A}{\\sin A}=\\frac{\\sin \\frac{A}{2}}{\\sin \\frac{A}{2} \\sin A}=\\frac{1}{\\sin A} \\text {, 又 } \\\\\n\\cot B+\\cot C=\\frac{\\cos B}{\\sin B}+\\frac{\\cos C}{\\sin C}=\\frac{\\sin A}{\\sin B \\sin C},\n\\end{gathered}\n$$\n所以 $\\frac{1}{\\sin A}=\\frac{\\sin A}{\\sin B \\sin C}$, 即 $\\sin ^2 A=\\sin B \\sin C$, 即 $a^2=b c$, 得证.\n评注如果 $P$ 是 $\\triangle A B C$ 内一点, 且 $\\angle P A B=\\angle P B C=\\angle P C A=\\alpha$, 则称 $P$ 为 $\\triangle A B C$ 的布洛卡点.\n布洛卡点的一个基本性质是: $\\cot A+\\cot B+\\cot C= \\cot \\alpha$. 本题 $\\alpha=\\frac{A}{2}$. 另证: $\\frac{O A}{\\sin \\left(B-\\frac{A}{2}\\right)}=\\frac{O B}{\\sin \\frac{A}{2}}, \\frac{O C}{\\sin \\frac{A}{2}}=\\frac{O B}{\\sin \\left(C-\\frac{A}{2}\\right)}$, 又因为 $O A=O C$, 所以上两式相比得 $\\sin ^2 \\frac{A}{2}=\\sin \\left(B-\\frac{A}{2}\\right) \\cdot \\sin \\left(C-\\frac{A}{2}\\right) \\Rightarrow 1- \\cos A=\\cos (B-C)-\\cos (B+C-A)$, 因为 $B+C-A=\\pi-2 A$, 所以 $1 - \\cos 2 A=\\cos (B-C)+\\cos A$, 所以 $2 \\sin ^2 A=2 \\sin B \\sin C$, 即 $a^2=b c$, 得证.", + "remark": "", + "figures": [ + "./images/volume3/figures/fig-c3e2.png" + ] +} \ No newline at end of file diff --git a/processed_dataset/proof/0971.json b/processed_dataset/proof/0971.json new file mode 100644 index 0000000000000000000000000000000000000000..19882acdd14aa30226dcb4cc7ca5b9f60e047cb0 --- /dev/null +++ b/processed_dataset/proof/0971.json @@ -0,0 +1,10 @@ +{ + "source_file": "./raw_volume-zh/volume3/chapter3.tex", + "problem_type": "proof", + "problem": "例3 在四边形 $A B C D$ 中, 若 $A B=a, B C=b, C D=c, A D=d, A C=e, B D=f$, 则求证 $a^2 c^2+b^2 d^2=e^2 f^2+2 a b c d \\cos (A+C)$.", + "solution": "分析:由于要证等式的左边是 $a^2 c^2+b^2 d^2$, 联想到 $1=\\sin ^2 \\alpha+\\cos ^2 \\alpha$. 将右边的项 $e^2 f^2$ 分解成: $e^2 f^2=(e f \\sin \\alpha)^2+(e f \\cos \\alpha)^2$, (其中令 $\\alpha=\\angle A O B$ ), 而 ef $\\sin \\alpha=2 S$ ( $S$ 为四边形 $A B C D$ 的面积), $e=O A+O C, f=O B+O D$. 于是应用余弦定理可将 $(e f)^2$ 用 $S 、 a 、 b 、 c 、 d$ 表示,进而变形得解.\n证明如图(), 设 $A C$ 与 $B D$ 交于 $O, \\angle A O B= \\alpha$, 四边形 $A B C D$ 的面积为 $S$. 则 $e^2 f^2=(e f \\sin \\alpha)^2+ (e f \\cos \\alpha)^2$.\n而\n$e f \\sin \\alpha=2 S$,\n$$\n\\begin{aligned}\ne f \\cos \\alpha= & (O A+O C)(O B+O D) \\cos \\alpha \\\\\n= & O A \\cdot O B \\cos \\alpha+O B \\cdot O C \\cos \\alpha+O C \\cdot O D \\cos \\alpha +O A \\cdot O D \\cos \\alpha \\\\\n= & \\frac{O A^2+O B^2-A B^2}{2}-\\frac{O B^2+O C^2-B C^2}{2} +\\frac{O C^2+O D^2-C D^2}{2}-\\frac{O A^2+O D^2-A D^2}{2} \\\\\n= & \\frac{b^2+d^2-a^2-c^2}{2},\n\\end{aligned}\n$$\n故\n$$\n(e f)^2=4 S^2+\\frac{1}{4}\\left(b^2+d^2-a^2-c^2\\right)^2, \\quad\\quad (1)\n$$\n但\n$$\nf^2=a^2+d^2-2 a d \\cos A=b^2+c^2-2 b c \\cos C,\n$$\n所以\n$$\na d \\cos A-b c \\cos C=\\frac{a^2+d^2-b^2-c^2}{2}, \\quad\\quad (2)\n$$\n而\n$$\na d \\sin A+b c \\sin C=2 S \\text {. } \\quad\\quad (3)\n$$\n由 $(2)^2+(3)^2$, 得\n$$\na^2 d^2+b^2 c^2-2 a b c d \\cos (A+C)=4 S^2+\\frac{\\left(a^2+d^2-b^2-c^2\\right)^2}{4} . \\quad\\quad (4)\n$$\n由(1)、(4), 得 \n$$\n\\begin{aligned}\ne^2 f^2=& a^2 d^2+b^2 c^2-2 a b c d \\cos (A+C)-\\frac{1}{4}\\left(a^2+b^2-d^2- c^2\\right)^2+\\frac{1}{4}\\left(b^2+d^2-a^2-c^2\\right)^2 \\\\\n= & a^2 d^2+b^2 c^2-2 a b c d \\cos (A+C)+\\left(d^2-c^2\\right)\\left(b^2-a^2\\right) \\\\\n= & a^2 c^2+b^2 d^2-2 a b c d \\cos (A+C),\n\\end{aligned}\n$$\n故\n$$\na^2 c^2+b^2 d^2=e^2 f^2+2 a b c d \\cos (A+C) .\n$$\n评注本例可视为余弦定理对四边形的一个推广, 另外当四边形为圆内接四边形时本例成为托勒密定理.", + "remark": "", + "figures": [ + "./images/volume3/figures/fig-c3e3.png" + ] +} \ No newline at end of file diff --git a/processed_dataset/proof/0972.json b/processed_dataset/proof/0972.json new file mode 100644 index 0000000000000000000000000000000000000000..4893dad2d3d7d25c1c5a5183aca1ee113e5f9903 --- /dev/null +++ b/processed_dataset/proof/0972.json @@ -0,0 +1,10 @@ +{ + "source_file": "./raw_volume-zh/volume3/chapter3.tex", + "problem_type": "proof", + "problem": "例4 在圆内接四边形 $A B C D$ 中, 设 $A B=a, B C=b, C D=c, D A= d$, 四边形 $A B C D$ 面积为 $S$, 圆半径为 $R$, 且令 $p=\\frac{a+b+c+d}{2}$.\n求证: (1) $\\cos B=\\frac{a^2+b^2-c^2-d^2}{2(a b+c d)}$;\n(2) $R=\\frac{1}{4} \\sqrt{\\frac{(a b+c d)(a c+b d)(a d+b c)}{(p-a)(p-b)(p-c)(p-d)}}$.", + "solution": "分析:(1) 的右边类似于两个余弦定理的合成; (2) 的证明要综合运用正、余弦定理.\n证明如图(), 设 $A C=m$.\n(1) 在 $\\triangle A B C$ 中, $\\cos B=\\frac{a^2+b^2-m^2}{2 a b}$, 在 $\\triangle A D C$ 中, $\\cos D=\\frac{d^2+c^2-m^2}{2 d c}$.\n因为四边形 $A B C D$ 内接于圆, 所以 $\\cos B=-\\cos D$.\n故 $\\frac{a^2+b^2-m^2}{2 a b}=\\frac{m^2-d^2-c^2}{2 d c}=\\frac{a^2+b^2-d^2-c^2}{2 a b+2 d c}$ (合比定理),所以\n$$\n\\cos B=\\frac{a^2+b^2-d^2-c^2}{2(a b+c d)} .\n$$\n(2) 由于 $B$ 是 $\\triangle A B C$ 的内角,所以\n$$\n\\begin{aligned}\n\\sin B & =\\sqrt{1-\\cos ^2 B}=\\sqrt{1-\\left[\\frac{a^2+b^2-c^2-d^2}{2(a b+c d)}\\right]^2} \\\\\n& =\\frac{\\sqrt{4(a b+c d)^2-\\left(a^2+b^2-c^2-d^2\\right)^2}}{2(a b+c d)} \\\\\n& =\\frac{\\left[\\left(2 a b+2 c d+a^2+b^2-c^2-d^2\\right)\\left(2 a b+2 c d-a^2-b^2+c^2+d^2\\right)\\right]^{\\frac{1}{2}}}{2(a b+c d)} \\\\\n& =\\frac{\\left\\{\\left[(a+b)^2-(c-d)^2\\right]\\left[(c+d)^2-(a-b)^2\\right]\\right\\}^{\\frac{1}{2}}}{2(a b+c d)} \\\\\n& =\\frac{[(a+b+c-d)(a+b-c+d)(a-b+c+d)(-a+b+c+d)]^{\\frac{1}{2}}}{2(a b+c d)} \\\\\n& =\\frac{2 \\sqrt{(p-a)(p-b)(p-c)(p-d)}}{a b+c d} .\n\\end{aligned}\n$$\n在 $\\triangle A B C$ 中,\n$$\n\\begin{aligned}\nA C^2 & =a^2+b^2-2 a b \\cos B \\\\\n& =a^2+b^2-2 a b \\cdot \\frac{a^2+b^2-c^2-d^2}{2(a b+c d)} \\\\\n& =\\frac{c d\\left(a^2+b^2\\right)+a b\\left(c^2+d^2\\right)}{a b+c d} \\\\\n& =\\frac{(a c+b d)(a d+b c)}{a b+c d},\n\\end{aligned}\n$$\n所以\n$$\nA C=\\frac{\\sqrt{(a b+c d)(a c+b d)(a d+b c)}}{a b+c d} .\n$$\n由正弦定理得, $2 R=\\frac{A C}{\\sin B}, R=\\frac{A C}{2 \\sin B}$, 故\n$$\nR=\\frac{1}{4} \\sqrt{\\frac{(a b+c d)(a c+b d)(a d+b c)}{(p-a)(p-b)(p-c)(p-d)}} .\n$$\n评注本例(1)可以看作圆内接四边形的余弦定理, 另外由\n$$\n\\begin{aligned}\nS & =S_{\\triangle A B C}+S_{\\triangle A C D}=\\frac{1}{2} a b \\sin B+\\frac{1}{2} c d \\sin D \\\\\n& =\\frac{1}{2}(a b+c d) \\sin B \\\\\n& =\\sqrt{(p-a)(p-b)(p-c)(p-d)},\n\\end{aligned}\n$$\n可以看成圆内接四边形面积的海伦公式.", + "remark": "", + "figures": [ + "./images/volume3/figures/fig-c3e4.png" + ] +} \ No newline at end of file diff --git a/processed_dataset/proof/0973.json b/processed_dataset/proof/0973.json new file mode 100644 index 0000000000000000000000000000000000000000..7d7f8a1b8e666a7bbf6a92aef0b8d839e0ef1f4d --- /dev/null +++ b/processed_dataset/proof/0973.json @@ -0,0 +1,8 @@ +{ + "source_file": "./raw_volume-zh/volume3/chapter3.tex", + "problem_type": "proof", + "problem": "例5 设 $\\alpha 、 \\beta 、 \\gamma 、 \\varphi$ 为某四边形的四个内角, $n$ 为正偶数, 若 $\\alpha 、 \\beta 、 \\gamma 、 \\varphi$ 满足 $\\sin n \\alpha+\\sin n \\beta+\\sin n \\gamma+\\sin n \\varphi=0$.", + "solution": "求证: $\\alpha 、 \\beta 、 \\gamma 、 \\varphi$ 之中必有两个角之和在集合 $\\left\\{\\frac{2 \\pi}{n}, \\frac{4 \\pi}{n}, \\frac{6 \\pi}{n}, \\cdots, \\pi\\right\\}$ 中.\n分析由 $\\alpha+\\beta+\\gamma+\\varphi=2 \\pi$, 故两角和之间可以互换.\n证明因为 $\\alpha+\\beta+\\gamma+\\varphi=2 \\pi$, 所以 $\\frac{n \\alpha+n \\beta}{2}+\\frac{m \\gamma+n \\varphi}{2}=n \\pi$.\n又 $n$ 为正偶数,所以 $\\quad \\sin \\frac{n \\alpha+n \\beta}{2}=-\\sin \\frac{m \\gamma+n \\varphi}{2}$,\n$$\n\\begin{aligned}\n& \\sin n \\alpha+\\sin n \\beta+\\sin n \\gamma+\\sin n \\varphi=0 \\\\\n\\Leftrightarrow & 2 \\sin \\frac{n \\alpha+n \\beta}{2} \\cos \\frac{n \\alpha-n \\beta}{2}+2 \\sin \\frac{n \\gamma+n \\varphi}{2} \\cos \\frac{n \\gamma-n \\varphi}{2}=0 \\\\\n\\Leftrightarrow & 2 \\sin \\frac{n \\alpha+n \\beta}{2}\\left(\\cos \\frac{n \\alpha-n \\beta}{2}-\\cos \\frac{n \\gamma-n \\varphi}{2}\\right)=0 \\\\\n\\Leftrightarrow & \\sin \\frac{n \\alpha+n \\beta}{2}=0 \\text { 或 } \\cos \\frac{n \\alpha-n \\beta}{2}=\\cos \\frac{n \\gamma-n \\varphi}{2} .\n\\end{aligned}\n$$\n(1) 若 $\\sin \\frac{n \\alpha+n \\beta}{2}=0$, 则 $\\frac{n \\alpha+n \\beta}{2}=k \\pi(k \\in \\mathbf{Z})$, 即 $\\alpha+\\beta=\\frac{2 k \\pi}{n}(k \\in \\mathbf{Z})$, 又 $0<\\alpha+\\beta<2 \\pi$, 则 $0<\\frac{2 k \\pi}{n}<2 \\pi \\Rightarrow 0)和() 所示, 即射影定理.", + "remark": "", + "figures": [ + "./images/volume3/figures/fig-c3e14-1.png", + "./images/volume3/figures/fig-c3e14-2.png" + ] +} \ No newline at end of file diff --git a/processed_dataset/proof/0978.json b/processed_dataset/proof/0978.json new file mode 100644 index 0000000000000000000000000000000000000000..582321d4c4aa89367ce3642b16bc4d1efa076229 --- /dev/null +++ b/processed_dataset/proof/0978.json @@ -0,0 +1,8 @@ +{ + "source_file": "./raw_volume-zh/volume3/chapter3.tex", + "problem_type": "proof", + "problem": "例15 在锐角 $\\triangle A B C$ 中,\n(1) 求证: $\\tan A+\\tan B+\\tan C=\\tan A \\cdot \\tan B \\cdot \\tan C$;\n(2) 求证: $\\tan A+\\tan B+\\tan C \\geqslant 3 \\sqrt{3}$.", + "solution": "分析:三角恒等式的证明或从左到右、或从右到左、或由繁到简.\n本题隐含条件 $A+B+C=\\pi$, 可采用两角和的正切公式来证.\n第(2) 题利用基本不等式 $a+b+c \\geqslant 3 \\sqrt[3]{a b c}\\left(a, b, c \\in \\mathbf{R}^{+}\\right)$.\n证明 (1)由 $A+B=\\pi-C$ 得\n$$\n\\tan (A+B)=\\tan (\\pi-C),\n$$\n即\n$$\n\\frac{\\tan A+\\tan B}{1-\\tan A \\cdot \\tan B}=-\\tan C,\n$$\n去分母即得原等式成立.\n(2)因为 $\\tan A, \\tan B, \\tan C>0$, 所以\n$$\n\\tan A+\\tan B+\\tan C \\geqslant 3 \\sqrt[3]{\\tan A \\cdot \\tan B \\cdot \\tan C},\n$$\n即\n$$\n\\tan A \\cdot \\tan B \\cdot \\tan C \\geqslant 3 \\sqrt[3]{\\tan A \\cdot \\tan B \\cdot \\tan C}\n$$\n两边立方后再开方得 $\\tan A \\cdot \\tan B \\cdot \\tan C \\geqslant 3 \\sqrt{3}$, 即\n$$\n\\tan A+\\tan B+\\tan C \\geqslant 3 \\sqrt{3} .\n$$\n评注若将题设条件改成 $A+B+C=k \\pi, k \\in \\mathbf{Z}$, 且 $A+B+C \\neq k \\pi+ \\frac{\\pi}{2}, k \\in \\mathbf{Z}$, 结论仍成立.", + "remark": "", + "figures": [] +} \ No newline at end of file diff --git a/processed_dataset/proof/0979.json b/processed_dataset/proof/0979.json new file mode 100644 index 0000000000000000000000000000000000000000..9d82f7f2caf27c43eba8e8e7b66d0325e0ef2ea4 --- /dev/null +++ b/processed_dataset/proof/0979.json @@ -0,0 +1,8 @@ +{ + "source_file": "./raw_volume-zh/volume3/chapter3.tex", + "problem_type": "proof", + "problem": "例16 在 $\\triangle A B C$ 中, 若角 $A 、 B 、 C$ 的三角关系式是 $y=2+\\cos C \\cdot \\cos (A-B)-\\cos ^2 C$.\n(1) 若任意交换 $A 、 B 、 C$ 的位置, $y$ 的值是否会发生变化? 试证明你的结论;\n(2) 求 $y$ 的最大值.", + "solution": "分析:如此问题,应猜想 $y$ 是轮换对称式,其值才不变,故将原关系式进行恒等变形, 分析其特征, 从而判断之.\n至于最值求法则通过放缩而得.\n解 (1)\n$$\n\\begin{aligned}\ny & =2+\\cos C \\cdot \\cos (A-B)-\\cos ^2 C \\\\\n& =2-\\cos (A+B) \\cdot \\cos (A-B)-\\cos ^2 C \\\\\n& =2-\\cos ^2 A \\cdot \\cos ^2 B+\\sin ^2 A \\cdot \\sin ^2 B-\\cos ^2 C \\\\\n& =\\sin ^2 A+\\sin ^2 B+\\sin ^2 C,\n\\end{aligned}\n$$\n由此可知任意交换 $A 、 B 、 C$ 的位置, $y$ 的值不会发生变化.\n(2) 不妨设 $C$ 为锐角, 则 $\\cos C>0, \\cos (A-B) \\leqslant 1$, 从而\n$$\ny \\leqslant 2+\\cos C-\\cos ^2 C=-\\left(\\cos C-\\frac{1}{2}\\right)^2+\\frac{9}{4},\n$$\n当 $A=B=C=\\frac{\\pi}{3}$ 时, $y_{\\text {max }}=\\frac{9}{4}$.\n评注 “大胆猜想, 小心求证” 是数学发现的至理名言.\n题中问题具有探索性, 须经猜想, 方可确定目标.\n对于题 (2), 也许有的学生以为既然“ $y$ 的值不会发生变化”, 那为什么又要求最大值呢? 这是混淆了 “无论 $A 、 B 、 C$ 取何值, $y$ 的值不会发生变化”与 “任意交换 $A 、 B 、 C$ 的位置, $y$ 的值不会发生变化”的意义.", + "remark": "", + "figures": [] +} \ No newline at end of file diff --git a/processed_dataset/proof/0980.json b/processed_dataset/proof/0980.json new file mode 100644 index 0000000000000000000000000000000000000000..1034a31fa156c6a7242fae673b522a02d4495b09 --- /dev/null +++ b/processed_dataset/proof/0980.json @@ -0,0 +1,8 @@ +{ + "source_file": "./raw_volume-zh/volume3/chapter3.tex", + "problem_type": "proof", + "problem": "例20 已知 $\\triangle A B C$ 的三个内角 $A 、 B 、 C$ 满足 $A+C=2 B$, 设 $x= \\cos \\frac{A-C}{2}, f(x)==\\cos B \\cdot\\left(\\frac{1}{\\cos A}+\\frac{1}{\\cos C}\\right)$.\n(1) 试求函数 $f(x)$ 的解析式及其定义域;\n(2) 判断其单调性, 并加以证明;\n(3) 求这个函数的值域.", + "solution": "解:(1) 因为 $A+C=2 B$, 所以 $B=60^{\\circ}, A+C=120^{\\circ}$,\n$$\n\\begin{aligned}\nf(x) & =\\frac{1}{2} \\cdot \\frac{\\cos A+\\cos C}{\\cos A \\cdot \\cos C} \\\\\n& =\\frac{2 \\cos \\frac{A+C}{2} \\cdot \\cos \\frac{A-C}{2}}{\\cos (A+C)+\\cos (A-C)} \\\\\n& =\\frac{x}{-\\frac{1}{2}+2 x^2-1}=\\frac{2 x}{4 x^2-3} .\n\\end{aligned}\n$$\n由 $0^{\\circ} \\leqslant\\left|\\frac{A-C}{2}\\right|<60^{\\circ}$, 得 $x=\\cos \\frac{A-C}{2} \\in\\left(\\frac{1}{2}, 1\\right]$.\n又因为 $4 x^2-3 \\neq 0$, 所以 $x \\neq \\frac{\\sqrt{3}}{2}$, 故 $f(x)$ 的定义域为 $\\left(\\frac{1}{2}, \\frac{\\sqrt{3}}{2}\\right) \\cup \\left(\\frac{\\sqrt{3}}{2}, 1\\right]$.\n(2) 设 $x_12$ 时, $|\\cos x|^n+|\\sin x|^n<\\cos ^2 x+\\sin ^2 x=1$, 故当 $x \\neq k \\cdot \\frac{\\pi}{2}$ 时, 方程无解.\n当 $n=1$ 时, $|\\cos x|^n+|\\sin x|^n=|\\cos x|+|\\sin x|= \\sqrt{(|\\cos x|+|\\sin x|)^2}>\\sqrt{|\\cos x|^2+|\\sin x|^2}=1$, 此时也无解.\n对 $x=\\frac{k \\pi}{2}, k \\in \\mathbf{Z}$, 通过验证可知, $x=2 k \\pi, x=2 k \\pi-\\frac{\\pi}{2}(k \\in \\mathbf{Z})$ 是原方程的解.\n综上可知, 当 $n$ 为偶数时, 原方程的解为 $x=k \\pi(k \\in \\mathbf{Z})$; 当 $n$ 为奇数时, $x=2 k \\pi$ 或 $x=2 k \\pi-\\frac{\\pi}{2}(k \\in \\mathbf{Z})$.\n评注由 $\\cos ^2 x+\\sin ^2 x=1$,联想 $\\left|\\cos ^n x\\right|+\\left|\\sin ^n x\\right|$ 与 1 的大小关系, 同时结合 $|\\cos x| \\leqslant 1,|\\sin x| \\leqslant 1$, 对解的情形进行估算.", + "remark": "", + "figures": [] +} \ No newline at end of file diff --git a/processed_dataset/proof/0983.json b/processed_dataset/proof/0983.json new file mode 100644 index 0000000000000000000000000000000000000000..a8f63427e4c01dced26a20bcdc8dcf862d37fb6f --- /dev/null +++ b/processed_dataset/proof/0983.json @@ -0,0 +1,8 @@ +{ + "source_file": "./raw_volume-zh/volume3/chapter4.tex", + "problem_type": "proof", + "problem": "例4 解不等式\n$$\n2 \\cos x \\leqslant|\\sqrt{1+\\sin 2 x}-\\sqrt{1-\\sin 2 x}| \\leqslant \\sqrt{2}(0 \\leqslant x \\leqslant 2 \\pi) .\n$$", + "solution": "解:设 $|\\sqrt{1+\\sin 2 x}-\\sqrt{1-\\sin 2 x}|=2 y$, 则原不等式变形为 $\\cos x \\leqslant y \\leqslant \\frac{\\sqrt{2}}{2}$, 且有 $2 y=|| \\sin x+\\cos x|-| \\sin x-\\cos x||$.\n(1) 当 $0 \\leqslant x \\leqslant \\frac{\\pi}{4}$ 或 $\\frac{3 \\pi}{4} \\leqslant x \\leqslant \\frac{5 \\pi}{4}$ 或 $\\frac{7 \\pi}{4} \\leqslant x \\leqslant 2 \\pi$ 时, $y=|\\sin x|$, 原不等式变为 $\\cos x \\leqslant|\\sin x| \\leqslant \\frac{\\sqrt{2}}{2}$, 验证得 $\\frac{3 \\pi}{4} \\leqslant x \\leqslant \\frac{5 \\pi}{4}$.\n(2) 当 $\\frac{\\pi}{4} \\leqslant x \\leqslant \\frac{3 \\pi}{4}$ 或 $\\frac{5 \\pi}{4} \\leqslant x \\leqslant \\frac{7 \\pi}{4}$ 时, $y=|\\cos x|$, 原不等式变为 $\\cos x \\leqslant|\\cos x| \\leqslant \\frac{\\sqrt{2}}{2}$, 都成立.\n综上,解集为 $\\frac{\\pi}{4} \\leqslant x \\leqslant \\frac{7 \\pi}{4}$.\n评注本例的关键是根据范围去绝对值, 化简.", + "remark": "", + "figures": [] +} \ No newline at end of file diff --git a/processed_dataset/proof/0984.json b/processed_dataset/proof/0984.json new file mode 100644 index 0000000000000000000000000000000000000000..cbcdd60941cc6d082e4bc984650c3b412066a8e9 --- /dev/null +++ b/processed_dataset/proof/0984.json @@ -0,0 +1,8 @@ +{ + "source_file": "./raw_volume-zh/volume3/chapter4.tex", + "problem_type": "proof", + "problem": "例5 求所有的常数 $C$, 使得函数 $f(x)=\\arctan \\frac{2-2 x}{1+4 x}+C$, 在区间$\\left(-\\frac{1}{4}, \\frac{1}{4}\\right)$ 上为奇函数.", + "solution": "分析:因为 0 在定义域内, 所以可根据 $f(0)=0$ 得 $C$ 的必要条件, 再验证是否为奇函数.\n解如果 $f(x)$ 为 $\\left(-\\frac{1}{4}, \\frac{1}{4}\\right)$ 上的奇函数, 则 $f(0)=0$, 于是 $C=-\\arctan 2$.\n下面证明 $f(x)=\\arctan \\frac{2-2 x}{1+4 x}-\\arctan 2$ 是奇函数.\n当 $x \\in\\left(-\\frac{1}{4}, \\frac{1}{4}\\right)$ 时, 函数 $u(x)=\\frac{2-2 x}{1+4 x}=\\frac{1}{2}\\left(\\frac{5}{1+4 x}-1\\right)$, 故 $u(x) \\in\\left(\\frac{3}{4},+\\infty\\right)$, 所以 $f(x)=\\arctan u(x)-\\arctan 2 \\in\\left(\\arctan \\frac{3}{4}-\\arctan 2\\right.$, $\\left.\\frac{\\pi}{2}-\\arctan 2\\right) \\varsubsetneqq\\left(-\\frac{\\pi}{2}, \\frac{\\pi}{2}\\right)$.\n显然 $-f(-x) \\in\\left(-\\frac{\\pi}{2}, \\frac{\\pi}{2}\\right)$.\n因为 $y=\\tan x$ 在 $\\left(-\\frac{\\pi}{2}, \\frac{\\pi}{2}\\right)$ 上一一对应, 故要证 $f(-x)=-f(x)$, 只要证明 $\\tan [-f(-x)]=\\tan f(x)$,\n$$\n\\begin{aligned}\n\\tan f(x) & =\\tan [\\arctan u(x)-\\arctan 2] \\\\\n& =\\frac{u(x)-2}{1+2 u(x)}=\\frac{(2-2 x)-2(1+4 x)}{1+4 x+2(2-2 x)}=-2 x,\n\\end{aligned}\n$$\n$$\n\\text { 又 } \\begin{aligned}\n\\tan [-f(-x)] & =\\tan [\\arctan 2-\\arctan u(-x)] \\\\\n& =\\frac{2-u(-x)}{1+2 u(-x)}=\\frac{2(1-4 x)-(2+2 x)}{1-4 x+2(2+2 x)}=-2 x,\n\\end{aligned}\n$$\n所以, 当且仅当 $C=-\\arctan 2$ 时, 函数 $f(x)$ 为 $\\left(-\\frac{1}{4}, \\frac{1}{4}\\right)$ 上的奇函数.\n评注本例也可直接用反三角函数进行运算, 这里根据奇函数的反函数也是奇函数进行证明.", + "remark": "", + "figures": [] +} \ No newline at end of file diff --git a/processed_dataset/proof/0985.json b/processed_dataset/proof/0985.json new file mode 100644 index 0000000000000000000000000000000000000000..3a0254726e62030b22fdeb03ca4fc7acdcaca0cf --- /dev/null +++ b/processed_dataset/proof/0985.json @@ -0,0 +1,8 @@ +{ + "source_file": "./raw_volume-zh/volume3/chapter4.tex", + "problem_type": "proof", + "problem": "例8 求证: $\\arctan \\frac{1}{3}+\\arctan \\frac{1}{5}+\\arctan \\frac{1}{7}+\\arctan \\frac{1}{8}=\\frac{\\pi}{4}$.", + "solution": "分析:本题化归为求四角和的三角函数值, 将证明题改编为计算题.\n证明设则\n$$\n\\begin{gathered}\n\\alpha=\\arctan \\frac{1}{3}, \\beta=\\arctan \\frac{1}{5}, \\\\\n\\gamma=\\arctan \\frac{1}{7}, \\delta=\\arctan \\frac{1}{8}, \\\\\n\\tan \\alpha=\\frac{1}{3}, \\tan \\beta=\\frac{1}{5}, \\\\\n\\tan \\gamma=\\frac{1}{7}, \\tan \\delta=\\frac{1}{8}, \\\\\n\\alpha, \\beta, \\gamma, \\delta \\in\\left(0, \\frac{\\pi}{4}\\right) .\n\\end{gathered}\n$$\n$$\n\\begin{aligned}\n& \\tan \\alpha=\\frac{1}{3}, \\tan \\beta=\\frac{1}{5}, \\\\\n& \\tan \\gamma=\\frac{1}{7}, \\tan \\delta=\\frac{1}{8}, \\\\\n& \\alpha, \\beta, \\gamma, \\delta \\in\\left(0, \\frac{\\pi}{4}\\right) .\n\\end{aligned}\n$$\n于是\n$$\n\\begin{gathered}\n\\tan (\\alpha+\\beta)=\\frac{4}{7}, \\tan (\\gamma+\\delta)=\\frac{3}{11}, \\\\\n\\alpha+\\beta+\\gamma+\\delta \\in(0, \\pi) .\n\\end{gathered}\n$$\n所以\n$$\n\\tan (\\alpha+\\beta+\\gamma+\\delta)=1 .\n$$\n从而\n$$\n\\alpha+\\beta+\\gamma+\\delta=\\frac{\\pi}{4} .\n$$\n评注有关反三角恒等式的证明须掌握两点: 其一证明等式两边的角的同一个三角函数值相等; 其二证明等式两边都在所取三角函数的同一个一一对应的区间内.", + "remark": "", + "figures": [] +} \ No newline at end of file diff --git a/processed_dataset/proof/0986.json b/processed_dataset/proof/0986.json new file mode 100644 index 0000000000000000000000000000000000000000..c77d7500184a583dec2fa727ff3c4607a2a6514c --- /dev/null +++ b/processed_dataset/proof/0986.json @@ -0,0 +1,8 @@ +{ + "source_file": "./raw_volume-zh/volume3/chapter4.tex", + "problem_type": "proof", + "problem": "例9 求证: $\\arctan \\frac{1}{3}+\\arctan \\frac{1}{7}+\\arctan \\frac{1}{13}+\\cdots+\\arctan \\frac{1}{1+n+n^2}= \\arctan \\frac{n}{n+2}$.", + "solution": "分析:形如数列求和的三角函数式, 通常采用 “裂项”或 “配对”之法, 如何将 $\\arctan \\frac{1}{1+n+n^2}$ 一分为二呢, 通过构造等式来解之.\n证明设\n$$\nf(n)=\\frac{n}{n+2},\n$$\n则\n$$\n\\begin{aligned}\n\\frac{f(n)-f(n-1)}{1+f(n) f(n-1)} & =\\frac{\\frac{n}{n+2}-\\frac{n-1}{n+1}}{1+\\frac{n}{n+2} \\cdot \\frac{n-1}{n+1}} \\\\\n& =\\frac{n^2+n-\\left(n^2+n-2\\right)}{2 n^2+2 n+2} \\\\\n& =\\frac{1}{n^2+n+1} .\n\\end{aligned}\n$$\n即 $\\tan \\left(\\arctan \\frac{1}{1+n+n^2}\\right)=\\frac{\\tan [\\arctan f(n)]-\\tan [\\arctan f(n-1)]}{1+\\tan [\\arctan f(n)] \\cdot \\tan [\\arctan f(n-1)]}$\n$$\n=\\tan [\\arctan f(n)-\\arctan f(n-1)] \\text {, }\n$$\n从而\n$$\n\\begin{aligned}\n\\arctan \\frac{1}{1+n+n^2} & =\\arctan f(n)-\\arctan f(n-1) \\\\\n& =\\arctan \\frac{n}{n+2}-\\arctan \\frac{n-1}{n+1} .\n\\end{aligned}\n$$\n所以\n$$\n\\begin{aligned}\n\\text { 左式 }= & \\arctan \\frac{1}{3}+\\left(\\arctan \\frac{2}{4}-\\arctan \\frac{1}{3}\\right) \\\\\n& +\\left(\\arctan \\frac{3}{5}-\\arctan \\frac{2}{4}\\right)+\\cdots \\\\\n& +\\left(\\arctan \\frac{n}{n+2}-\\arctan \\frac{n-1}{n+1}\\right) \\\\\n= & \\arctan \\frac{n}{n+2} .\n\\end{aligned}\n$$\n评注本题又可改编为 $\\arctan \\frac{1}{3}+\\arctan \\frac{1}{7}+\\cdots+\\arctan \\frac{1}{1+n+n^2}+ \\arctan \\frac{1}{n+1}=\\frac{\\pi}{4}$.", + "remark": "", + "figures": [] +} \ No newline at end of file diff --git a/processed_dataset/proof/0987.json b/processed_dataset/proof/0987.json new file mode 100644 index 0000000000000000000000000000000000000000..328aaa2beffd6020d6697d8c596d91260663d1d4 --- /dev/null +++ b/processed_dataset/proof/0987.json @@ -0,0 +1,8 @@ +{ + "source_file": "./raw_volume-zh/volume3/chapter4.tex", + "problem_type": "proof", + "problem": "例10 求证: $\\arcsin x+\\arccos x=\\frac{\\pi}{2}$.", + "solution": "分析:证明反三角恒等式的基本步骤是: (1)证等式两边的角的同名三角函数值相等; (2)证角的唯一性.\n证明 $\\sin (\\arcsin x+\\arccos x)$\n$$\n\\begin{aligned}\n& =\\sin (\\arcsin x) \\cdot \\cos (\\arccos x)+\\cos (\\arcsin x) \\cdot \\sin (\\arccos x) \\\\\n& =x \\cdot x+\\sqrt{1-x^2} \\cdot \\sqrt{1-x^2}=1=\\sin \\frac{\\pi}{2} .\n\\end{aligned}\n$$\n又因为 $-\\frac{\\pi}{2} \\leqslant \\arcsin x \\leqslant \\frac{\\pi}{2}, 0 \\leqslant \\arccos x \\leqslant \\pi$, 所以\n$$\n-\\frac{\\pi}{2} \\leqslant \\arcsin x+\\arccos x \\leqslant \\frac{3 \\pi}{2}\n$$\n在 $\\left[-\\frac{\\pi}{2}, \\frac{3}{2} \\pi\\right]$ 上, $\\sin \\alpha=1$ 的角 $\\alpha$ 是唯一的, 只有 $\\frac{\\pi}{2}$, 即\n$$\n\\arcsin x+\\arccos x=\\frac{\\pi}{2}\n$$\n评注同理可证: $\\arcsin x+\\arcsin \\sqrt{1-x^2}=\\frac{\\pi}{2}$,\n$$\n\\arctan x+\\operatorname{arccot} x=\\frac{\\pi}{2} .\n$$", + "remark": "", + "figures": [] +} \ No newline at end of file diff --git a/processed_dataset/proof/0988.json b/processed_dataset/proof/0988.json new file mode 100644 index 0000000000000000000000000000000000000000..ae06dce03b7b0f09498f5fb6ce9524a3f269fad5 --- /dev/null +++ b/processed_dataset/proof/0988.json @@ -0,0 +1,8 @@ +{ + "source_file": "./raw_volume-zh/volume3/chapter4.tex", + "problem_type": "proof", + "problem": "例19 已知 $\\alpha$ 、 $\\beta$ 是方程 $a \\cos x+b \\sin x=c$ 的两个相异的根,且 $\\alpha \\pm \\beta \\neq k \\pi, k \\in \\mathbf{Z}, a b \\neq 0$, 求证: (1) $\\tan (\\alpha+\\beta)=\\frac{2 a b}{a^2-b^2}$; (2) $\\cos ^2 \\frac{\\alpha-\\beta}{2}=\\frac{c^2}{a^2+b^2}$.", + "solution": "证明:(1) 由条件得\n$$\n\\begin{aligned}\n& a \\cos \\alpha+b \\sin \\alpha=c, \\quad\\quad (1)\\\\\n& a \\cos \\beta+b \\sin \\beta=c, \\quad\\quad (2)\n\\end{aligned}\n$$\n由(1)-(2)得\n$$\na(\\cos \\alpha-\\cos \\beta)+b(\\sin \\alpha-\\sin \\beta)=0,\n$$\n和差化积得\n$$\n-2 a \\sin \\frac{\\alpha+\\beta}{2} \\sin \\frac{\\alpha-\\beta}{2}+2 b \\cos \\frac{\\alpha+\\beta}{2} \\sin \\frac{\\alpha-\\beta}{2}=0 .\n$$\n因为 $\\alpha \\pm \\beta \\neq k \\pi$, 所以 $\\sin \\frac{\\alpha-\\beta}{2} \\neq 0, \\cos \\frac{\\alpha+\\beta}{2} \\neq 0$. 故 $\\tan \\frac{\\alpha+\\beta}{2}=\\frac{b}{a}$,\n从而\n$$\n\\tan (\\alpha+\\beta)=\\frac{2 \\tan \\frac{\\alpha+\\beta}{2}}{1-\\tan ^2 \\frac{\\alpha+\\beta}{2}}=\\frac{2 \\cdot \\frac{b}{a}}{1-\\frac{b^2}{a^2}}=\\frac{2 a b}{a^2-b^2} .\n$$\n(2) 由(1) 可得\n$$\n\\begin{aligned}\n& \\sin (\\alpha+\\beta)=\\frac{2 \\times \\frac{b}{a}}{1+\\frac{b^2}{a^2}}=\\frac{2 a b}{a^2+b^2}, \\\\\n& \\cos (\\alpha+\\beta)=\\frac{a^2-b^2}{a^2+b^2},\n\\end{aligned}\n$$\n由(1) X(2)得\n$$\n\\begin{gathered}\na^2 \\cos \\alpha \\cos \\beta+a b(\\sin \\alpha \\cos \\beta+\\cos \\alpha \\sin \\beta)+b^2 \\sin \\alpha \\sin \\beta=c^2, \\\\\n\\frac{1}{2} a^2[\\cos (\\alpha+\\beta)+\\cos (\\alpha-\\beta)]+a b \\sin (\\alpha+\\beta) \\\\\n+\\frac{1}{2} b^2[\\cos (\\alpha-\\beta)-\\cos (\\alpha+\\beta)]=c^2 .\n\\end{gathered}\n$$\n即\n$$\n\\begin{gathered}\n\\frac{1}{2} a^2[\\cos (\\alpha+\\beta)+\\cos (\\alpha-\\beta)]+a b \\sin (\\alpha+\\beta) \\\\\n+\\frac{1}{2} b^2[\\cos (\\alpha-\\beta)-\\cos (\\alpha+\\beta)]=c^2\n\\end{gathered}\n$$\n所以\n$$\n\\left(\\frac{1}{2} a^2+\\frac{1}{2} b^2\\right) \\cos (\\alpha-\\beta)=c^2-\\left(\\frac{1}{2} a^2-\\frac{1}{2} b^2\\right) \\times \\frac{a^2-b^2}{a^2+b^2}-a b \\cdot \\frac{2 a b}{a^2+b^2} .\n$$\n即\n$$\n\\cos (\\alpha-\\beta)=\\frac{2 c^2}{a^2+b^2}-1\n$$\n从而\n$$\n\\cos ^2 \\frac{\\alpha-\\beta}{2}=\\frac{c^2}{a^2+b^2}\n$$", + "remark": "", + "figures": [] +} \ No newline at end of file diff --git a/processed_dataset/proof/0989.json b/processed_dataset/proof/0989.json new file mode 100644 index 0000000000000000000000000000000000000000..392d94c788727fdc694c8daf33c6b564102221a3 --- /dev/null +++ b/processed_dataset/proof/0989.json @@ -0,0 +1,8 @@ +{ + "source_file": "./raw_volume-zh/volume3/chapter5.tex", + "problem_type": "proof", + "problem": "例1. 已知 $0<\\alpha<\\beta<\\frac{\\pi}{2}$, 求证: $\\frac{\\cot \\beta}{\\cot \\alpha}<\\frac{\\cos \\beta}{\\cos \\alpha}<\\frac{\\beta}{\\alpha}$.", + "solution": "分析:可以构造函数, 用单调性来证明.\n证明设 $y=\\cos x, x \\in\\left(0, \\frac{\\pi}{2}\\right)$. 在其图象上取两点 $A(\\alpha, \\cos \\alpha), B(\\beta$, $\\cos \\beta)$, 由 $y=\\cos x$ 在 $\\left(0, \\frac{\\pi}{2}\\right)$ 上是减函数, 点 $A$ 在点 $B$ 的上方, 所以\n$$\n\\begin{gathered}\nk_{O A}>k_{O B} \\Rightarrow \\frac{\\cos \\alpha}{\\alpha}>\\frac{\\cos \\beta}{\\beta}, \\\\\n\\frac{\\cos \\beta}{\\cos \\alpha}<\\frac{\\beta}{\\alpha} .\n\\end{gathered}\n$$\n所以\n$$\n\\frac{\\cos \\beta}{\\cos \\alpha}<\\frac{\\beta}{\\alpha}\n$$\n再设 $y=\\cot x, x \\in\\left(0, \\frac{\\pi}{2}\\right)$, 在其图象上取点 $C(\\alpha, \\cot \\alpha), D(\\beta, \\cot \\beta)$, 同理 $k_{O C}>k_{O D}$, 所以 $\\frac{\\cot \\alpha}{\\alpha}>\\frac{\\cot \\beta}{\\beta}$, 所以\n$$\n\\frac{\\cot \\beta}{\\cot \\alpha}<\\frac{\\beta}{\\alpha} \\text {. }\n$$\n又因为 $\\frac{\\cos \\beta}{\\cos \\alpha}-\\frac{\\cot \\beta}{\\cot \\alpha}=\\frac{\\cos \\beta}{\\cos \\alpha}\\left(1-\\frac{\\sin \\alpha}{\\sin \\beta}\\right), 0<\\alpha<\\beta<\\frac{\\pi}{2}$,\n所以\n$$\n0<\\sin \\alpha<\\sin \\beta, \\frac{\\sin \\alpha}{\\sin \\beta}<1,\n$$\n所以\n$$\n\\frac{\\cos \\beta}{\\cos \\alpha}>\\frac{\\cot \\beta}{\\cot \\alpha}\n$$\n即\n$$\n\\frac{\\cot \\beta}{\\cot \\alpha}<\\frac{\\cos \\beta}{\\cos \\alpha}<\\frac{\\beta}{\\alpha}\n$$\n评注本例进一步可得如下结论, 若 $0<\\alpha<\\beta<\\frac{\\pi}{2}$, 则\n$$\n\\frac{\\cot \\beta}{\\cot \\alpha}<\\frac{\\cos \\beta}{\\cos \\alpha}<\\frac{\\sin \\beta}{\\sin \\alpha}<\\frac{\\beta}{\\alpha}<\\frac{\\tan \\beta}{\\tan \\alpha} .\n$$", + "remark": "", + "figures": [] +} \ No newline at end of file diff --git a/processed_dataset/proof/0990.json b/processed_dataset/proof/0990.json new file mode 100644 index 0000000000000000000000000000000000000000..541540fb2ec13f2db43c432da6b6bad75e2223bd --- /dev/null +++ b/processed_dataset/proof/0990.json @@ -0,0 +1,8 @@ +{ + "source_file": "./raw_volume-zh/volume3/chapter5.tex", + "problem_type": "proof", + "problem": "例2. 在 $\\triangle A B C$ 中, $\\angle C \\geqslant 60^{\\circ}$, 求证:\n$$\n(a+b)\\left(\\frac{1}{a}+\\frac{1}{b}+\\frac{1}{c}\\right) \\geqslant 4+\\frac{1}{\\sin \\frac{C}{2}} .\n$$", + "solution": "分析:由 $(a+b)\\left(\\frac{1}{a}+\\frac{1}{b}\\right) \\geqslant 4$, 所证不等式可化为求证 $(a+b) \\frac{1}{c} \\geqslant \\frac{1}{\\sin \\frac{C}{2}}$, 但取例以后发现此路不通, 原因是放缩过头了.\n证明\n$$\n\\begin{aligned}\n& (a+b)\\left(\\frac{1}{a}+\\frac{1}{b}+\\frac{1}{c}\\right) \\\\\n= & (\\sin A+\\sin B)\\left(\\frac{1}{\\sin A}+\\frac{1}{\\sin B}+\\frac{1}{\\sin C}\\right) \\\\\n= & 2+\\frac{\\sin A+\\sin B}{\\sin C}+\\frac{\\sin B}{\\sin A}+\\frac{\\sin A}{\\sin B} \\\\\n= & 4+\\frac{(\\sin A-\\sin B)^2}{\\sin A \\cdot \\sin B}+\\frac{\\sin B+\\sin A}{\\sin C} \\\\\n= & 4+\\frac{8 \\cos ^2 \\frac{A+B}{2} \\sin ^2 \\frac{A-B}{2}}{\\cos (A-B)-\\cos (A+B)}+\\frac{2 \\sin \\frac{A+B}{2} \\cos \\frac{A-B}{2}}{2 \\sin \\frac{C}{2} \\cos \\frac{C}{2}} \\\\\n= & 4+\\frac{8 \\cos ^2 \\frac{A+B}{2} \\sin ^2 \\frac{A-B}{2}}{\\cos (A-B)-\\cos (A+B)}+\\frac{1-2 \\sin ^2 \\frac{A-B}{4}}{\\sin ^2 \\frac{C}{2}}- \\\\\n= & 4+\\frac{1}{\\sin \\frac{C}{2}}+2 \\sin ^2 \\frac{A-B}{4} \\cdot\\left[\\frac{8 \\sin ^2 \\frac{C}{2} \\cos ^2 \\frac{A-B}{4}}{\\cos ^2 \\frac{A-B}{2}-\\sin ^2 \\frac{C}{2}}-\\frac{1}{\\sin \\frac{C}{2}}\\right], \\quad\\quad(1)\n\\end{aligned}\n$$\n因为\n$$\n0<\\left|\\frac{A-B}{2}\\right|<\\frac{A+B}{2}<\\frac{\\pi}{2}\n$$\n所以 $\\cos ^2 \\frac{A-B}{2}>\\cos ^2 \\frac{A+B}{2}=\\sin ^2 \\frac{C}{2} \\quad\\quad(2)$.\n又因为 $\\angle C \\geqslant 60^{\\circ}$, 所以 $8 \\sin ^3 \\frac{C}{2} \\geqslant 1$, 所以\n$$\n8 \\sin ^3 \\frac{C}{2} \\cos ^2 \\frac{A-B}{2} \\geqslant \\cos ^2 \\frac{A-B}{4} \\geqslant \\cos ^2 \\frac{A-B}{2} \\geqslant \\cos ^2 \\frac{A-B}{2}-\\sin ^2 \\frac{C}{2},\n$$\n再由 (2) 得代入(1), 得\n$$\n(a+b)\\left(\\frac{1}{a}+\\frac{1}{b}+\\frac{1}{c}\\right) \\geqslant 4+\\frac{1}{\\sin \\frac{C}{2}} .\n$$", + "remark": "", + "figures": [] +} \ No newline at end of file diff --git a/processed_dataset/proof/0991.json b/processed_dataset/proof/0991.json new file mode 100644 index 0000000000000000000000000000000000000000..a857f409a9ac78e9c2e695c440dd40a3f4fb0cbb --- /dev/null +++ b/processed_dataset/proof/0991.json @@ -0,0 +1,8 @@ +{ + "source_file": "./raw_volume-zh/volume3/chapter5.tex", + "problem_type": "proof", + "problem": "例3. 已知 $\\theta_i(i=1,2, \\cdots, n)$ 均为实数且满足 $\\sum_{i=1}^n\\left|\\cos \\theta_i\\right| \\leqslant \\frac{2}{n+1}$, ( $n>1$ 且 $\\left.n \\in \\mathbf{N}^*\\right)$, 求证:\n$$\n\\left|\\sum_{i=1}^n i \\cos \\theta_i\\right| \\leqslant\\left\\lceil\\frac{n^2}{4}\\right]+1 .\n$$", + "solution": "分析:通过适当的放缩去掉绝对值, 利用已知证明.\n证明 (1) 当 $n=2 k$ 时,\n$$\n\\begin{aligned}\n& {\\left[\\frac{n^2}{4}\\right]+1-\\left|\\sum_{i=1}^n i \\cos \\theta_i\\right|=k^2+1-\\left|\\sum_{i=1}^n i \\cos \\theta_i\\right| } \\\\\n\\geqslant & k^2+1-\\sum_{i=1}^n\\left|i \\cos \\theta_i\\right|=n+(n-1)+\\cdots+(k+1) \\\\\n& -k-(k-1)-\\cdots-1+1-\\left(\\left|\\cos \\theta_1\\right|+\\left|2 \\cos \\theta_2\\right|\\right. \\\\\n& \\left.+\\left|3 \\cos \\theta_3\\right|+\\cdots+\\left|n \\cos \\theta_n\\right|\\right) \\\\\n= & n\\left(1-\\left|\\cos \\theta_n\\right|\\right)+(n-1)\\left(1-\\left|\\cos \\theta_{n-1}\\right|\\right)+\\cdots+ \\\\\n& (k+1)\\left(1-\\left|\\cos \\theta_{k+1}\\right|\\right)-k\\left(1+\\left|\\cos \\theta_k\\right|\\right)-(k-1)(1+ \\\\\n& \\left.\\left|\\cos \\theta_{k-1}\\right|\\right)-\\cdots-1 \\cdot\\left(1+\\left|\\cos \\theta_1\\right|\\right)+1 \\\\\n\\geqslant & k\\left[\\left(1-\\left|\\cos \\theta_n\\right|\\right)+\\left(1-\\left|\\cos \\theta_{n-1}\\right|\\right)+\\cdots+\\left(1-\\left|\\cos \\theta_{k+1}\\right|\\right)-\\right. \\\\\n& \\left.\\left(1+\\left|\\cos \\theta_k\\right|\\right)-\\left(1+\\left|\\cos \\theta_{k-1}\\right|\\right)-\\cdots-\\left(1+\\left|\\cos \\theta_1\\right|\\right)\\right]+1 \\\\\n= & -k \\sum_{i=1}^n\\left|\\cos \\theta_i\\right|+1 \\geqslant-\\frac{2}{n+1} \\cdot k+1=\\frac{1}{n+1}>0 .\n\\end{aligned}\n$$\n(2) 当 $n=2 k+1$ 时同样有\n$$\n\\begin{aligned}\n& {\\left[\\frac{n^2}{4}\\right]+1-\\left|\\sum_{i=1}^n i \\cos \\theta_i\\right| } \\\\\n= & n+(n-1)+\\cdots+(k+2)+0-k-(k-1)-\\cdots-1-\\left|\\sum_{i=1}^n i \\cos \\theta_i\\right|+1 \\\\\n\\geqslant(k+1)\\left(k-\\left|\\cos \\theta_n\\right|-\\left|\\cos \\theta_{n-1}\\right|-\\cdots-\\left|\\cos \\theta_{k+2}\\right|\\right. & \\left.\\quad-\\left|\\cos \\theta_{k+1}\\right|-k-\\left|\\cos \\theta_k\\right|-\\left|\\cos \\theta_{k-1}\\right|-\\cdots-\\left|\\cos \\theta_1\\right|\\right)+1 \\\\\n= & -(k+1) \\sum_{i=1}^n\\left|\\cos \\theta_i\\right|+1 \\geqslant-(k+1) \\cdot \\frac{2}{n+1}+1=0 .\n\\end{aligned}\n$$\n综上, (1)、(2), 得证.\n评注本题只不过用三角函数做个样子, 本身并没有用到多少三角函数性质.", + "remark": "", + "figures": [] +} \ No newline at end of file diff --git a/processed_dataset/proof/0992.json b/processed_dataset/proof/0992.json new file mode 100644 index 0000000000000000000000000000000000000000..9cb05b52fa3bb60b11ed72e7faa26030128849af --- /dev/null +++ b/processed_dataset/proof/0992.json @@ -0,0 +1,8 @@ +{ + "source_file": "./raw_volume-zh/volume3/chapter5.tex", + "problem_type": "proof", + "problem": "例4. 求证: $\\left|\\sum_{k=1}^n \\frac{\\sin k x}{k}\\right| \\leqslant 2 \\sqrt{\\pi}$, 对一切实数 $x$ 成立.", + "solution": "分析:当 $\\theta \\in(0,+\\infty)$ 时, 有 $\\sin \\theta<\\theta$. 当 $\\theta \\in\\left(0, \\frac{\\pi}{2}\\right)$ 时, 有 $\\sin \\theta \\geqslant \\frac{2}{\\pi} \\theta$, 再结合函数的奇偶性, 周期性, 可得证明.\n证明首先考查 $x \\in(0, \\pi)$, 取 $m$ 为某个自然数,使 $m \\leqslant \\frac{\\sqrt{\\pi}}{x}\\frac{x}{\\pi}$, 又 $m+1>\\frac{\\sqrt{\\pi}}{x}$, 故\n$$\n\\left|\\sum_{k=m+1}^n \\frac{\\sin k x}{k}\\right| \\leqslant \\frac{1}{\\frac{x}{\\pi} \\cdot \\frac{\\sqrt{\\pi}}{x}}=\\sqrt{\\pi}, \\quad\\quad(2)\n$$\n由(1)、(2), 知当 $x \\in(0, \\pi)$ 时, 原式得证.\n当 $x \\in(-\\pi, 0)$ 时, 由于 $f(x)=\\left|\\sum_{k=1}^n \\frac{\\sin k x}{k}\\right|$ 为偶函数.\n所以原式也成立, 当 $x= \\pm \\pi, 0$ 时, 显然成立.\n又由于 $f(x)$ 是以 $2 \\pi$ 为周期的周期函数, 故原式对一切 $x \\in \\mathbf{R}$ 都成立, 得证.", + "remark": "", + "figures": [] +} \ No newline at end of file diff --git a/processed_dataset/proof/0993.json b/processed_dataset/proof/0993.json new file mode 100644 index 0000000000000000000000000000000000000000..7bb6bc00a85fc39bc47d2737d8ef113627143bc8 --- /dev/null +++ b/processed_dataset/proof/0993.json @@ -0,0 +1,11 @@ +{ + "source_file": "./raw_volume-zh/volume3/chapter5.tex", + "problem_type": "proof", + "problem": "例5. 三个数 $a 、 b 、 c$ 属干开区间 $\\left(0, \\frac{\\pi}{2}\\right)$, 且满足下列等式 $a=\\cos a$, $b=\\sin (\\cos b), c=\\cos (\\sin c)$, 试按从小到大的顺序排列这三个数.", + "solution": "分析:从题目条件来看, 直接计算 $a 、 b 、 c$ 的值是极为困难的, 所以考虑用图象法解之.\n解法一 $\\cos a=a$ 表示曲线 $y=\\cos x$ 与 $y=x$ 的交点的横坐标即为 $a$.\n同理, 曲线 $y=\\sin (\\cos x)$ 与 $y=x$ 的交点的横坐标即为 $b$. 而曲线 $y= \\cos (\\sin x)$ 与 $y=x$ 的交点的横坐标即为 $c$.\n在同一坐标系中作出曲线 $y=\\cos x$, $y=\\cos (\\sin x), y=\\sin (\\cos x)$ 及直线 $y=x$. (如图() 所示)\n由于当 $x \\in\\left(0, \\frac{\\pi}{2}\\right)$ 时, 有 $\\sin x\\cos x$. 即当 $x \\in\\left(0, \\frac{\\pi}{2}\\right)$ 时, $y=\\cos (\\sin x)$ 的图象在 $y=\\cos x$ 的上方.\n由于 $x \\in\\left(0, \\frac{\\pi}{2}\\right)$ 时, $\\cos x \\in(0,1)$. 在$\\sin x) 所示.\n由 $\\cos a=a$, 知 $a$ 是 $y=\\cos x$ 与 $y=x$ 交点的横坐标.\n而 $\\sin (\\cos b)=b$, 可看成 $\\cos b= \\arcsin b$. 即为 $y=\\cos x$ 与 $y=\\arcsin x$ 的交点的横坐标.\n注意到 $x \\in\\left(0, \\frac{\\pi}{2}\\right)$ 时, $0< \\sin x\\cos c$, 从而 $a<\\cos (\\sin c)$, 即 $a< c$, 两者矛盾, 所以假设错误, 得 $a\\sin (\\cos b)=b$, 两者也矛盾.\n所以 $a>b$, 综上所述 $b0,\n$$\n故 $f(x)=\\frac{1}{\\sin x}$ 是 $\\left(0, \\frac{\\pi}{2}\\right)$ 上的下凸函数.\n所以 $\\frac{1}{\\sin \\frac{A}{2}}+\\frac{1}{\\sin \\frac{B}{2}}+\\frac{1}{\\sin \\frac{C}{2}} \\geqslant 3 \\frac{1}{\\sin \\frac{A+B+C}{6}}=6$.", + "remark": "", + "figures": [] +} \ No newline at end of file diff --git a/processed_dataset/proof/0995.json b/processed_dataset/proof/0995.json new file mode 100644 index 0000000000000000000000000000000000000000..efe6dd165f47c6d3dd92ac8b95679aba4d3e61ee --- /dev/null +++ b/processed_dataset/proof/0995.json @@ -0,0 +1,8 @@ +{ + "source_file": "./raw_volume-zh/volume3/chapter5.tex", + "problem_type": "proof", + "problem": "例7. 设 $\\theta_i \\in\\left(-\\frac{\\pi}{2}, \\frac{\\pi}{2}\\right), i=1 、 2 、 3 、 4$, 证明: 存在 $\\theta \\in \\mathbf{R}$, 使得如下两个不等式:\n$$\n\\begin{aligned}\n& \\cos ^2 \\theta_1 \\cos ^2 \\theta_2-\\left(\\sin \\theta_1 \\sin \\theta_2-x\\right)^2 \\geqslant 0, \\quad\\quad(1)\\\\\n& \\cos ^2 \\theta_3 \\cos ^2 \\theta_4-\\left(\\sin \\theta_3 \\sin \\theta_4-x\\right)^2 \\geqslant 0, \\quad\\quad(2)\n\\end{aligned}\n$$\n同时成立的充分必要条件是\n$$\n\\sum_{i=1}^4 \\sin ^2 \\theta_i \\leqslant 2\\left(1+\\prod_{i=1}^4 \\sin \\theta_i+\\prod_{i=1}^4 \\cos \\theta_i\\right) .\\quad\\quad(3)\n$$", + "solution": "证明:式(1)、(2)分别等价于\n$$\n\\begin{aligned}\n& \\sin \\theta_1 \\sin \\theta_2-\\cos \\theta_1 \\cos \\theta_2 \\leqslant x \\leqslant \\sin \\theta_1 \\sin \\theta_2+\\cos \\theta_1 \\cos \\theta_2,\\quad\\quad(4)\\\\\n& \\sin \\theta_3 \\sin \\theta_4-\\cos \\theta_3 \\cos \\theta_4 \\leqslant x \\leqslant \\sin \\theta_3 \\sin \\theta_4+\\cos \\theta_3 \\cos \\theta_4,\\quad\\quad(5)\n\\end{aligned}\n$$\n易知存在 $x \\in \\mathbf{R}$, 使得(4)、(5)同时成立的充要条件是\n$$\n\\begin{aligned}\n& \\sin \\theta_1 \\sin \\theta_2+\\cos \\theta_1 \\cos \\theta_2-\\sin \\theta_3 \\sin \\theta_4+\\cos \\theta_3 \\cos \\theta_4 \\geqslant 0, \\quad\\quad(6)\\\\\n& \\sin \\theta_3 \\sin \\theta_4+\\cos \\theta_3 \\cos \\theta_4-\\sin \\theta_1 \\sin \\theta_2+\\cos \\theta_1 \\cos \\theta_2 \\geqslant 0, \\quad\\quad(7)\n\\end{aligned}\n$$\n另一方面,利用 $\\sin ^2 \\alpha=1-\\cos ^2 \\alpha$, 可将(3)式化为\n$$\n\\begin{aligned}\n& \\quad \\cos ^2 \\theta_1 \\cos ^2 \\theta_2+2 \\cos \\theta_1 \\cos \\theta_2 \\cos \\theta_3 \\cos \\theta_4+\\cos ^2 \\theta_3 \\cos ^2 \\theta_4-\\sin ^2 \\theta_1 \\sin ^2 \\theta_2+ \\\\\n& 2 \\sin \\theta_1 \\sin \\theta_2 \\sin \\theta_3 \\sin \\theta_4-\\sin ^2 \\theta_3 \\sin ^2 \\theta_4 \\geqslant 0 \\\\\n& \\Leftrightarrow\\left(\\cos \\theta_1 \\cos \\theta_2+\\cos \\theta_3 \\cos \\theta_4\\right)^2-\\left(\\sin \\theta_1 \\sin \\theta_2-\\sin \\theta_3 \\sin \\theta_4\\right)^2 \\geqslant 0 \\\\\n& \\quad \\Leftrightarrow\\left(\\sin \\theta_1 \\sin \\theta_2+\\cos \\theta_1 \\cos \\theta_2-\\sin \\theta_3 \\sin \\theta_4+\\cos \\theta_1 \\cos \\theta_4\\right)\\left(\\sin \\theta_3 \\sin \\theta_4+\\right. \\\\\n& \\left.\\quad \\cos \\theta_3 \\cos \\theta_4-\\sin \\theta_1 \\sin \\theta_2+\\cos \\theta_1 \\cos \\theta_2\\right) \\geqslant 0. \\quad\\quad(8)\n\\end{aligned}\n$$\n当存在 $x \\in \\mathbf{R}$, 使(1)(2)成立时 $\\Rightarrow$ (4)(5)成立 $\\Rightarrow$ (6)(7)成立 $\\Rightarrow$ (8)式成立 $\\Rightarrow$ (3)式成立.\n另一方面, 当(3)式也即(8)式成立, 但(6)(7)式不成立.\n则\n$$\n\\sin \\theta_1 \\sin \\theta_2+\\cos \\theta_1 \\cos \\theta_2-\\sin \\theta_3 \\sin \\theta_4+\\cos \\theta_3 \\cos \\theta_4<0,\n$$\n$$\n\\sin \\theta_3 \\sin \\theta_4+\\cos \\theta_3 \\cos \\theta_4-\\sin \\theta_1 \\sin \\theta_2+\\cos \\theta_1 \\cos \\theta_2<0,\n$$\n两式相加 $\\Rightarrow 2\\left(\\cos \\theta_1 \\cos \\theta_2+\\cos \\theta_3 \\cos \\theta_4\\right)<0$, 这与 $\\theta_i \\in\\left(-\\frac{\\pi}{2}, \\frac{\\pi}{2}\\right), i=1,2$, 3,4 矛盾.\n故必有(3) $\\Rightarrow$ (8) $\\Rightarrow$ (6) (7) 成立 $\\Rightarrow$ 存在 $x \\in \\mathbf{R}$, (4)(5)成立 $\\Rightarrow$ 存在 $x \\in \\mathbf{R}$, (1) (2) 成立.\n综上, 得证.", + "remark": "", + "figures": [] +} \ No newline at end of file diff --git a/processed_dataset/proof/0996.json b/processed_dataset/proof/0996.json new file mode 100644 index 0000000000000000000000000000000000000000..5c4e0e9fcedf32bceee78a24181cfcd14e9a4a2a --- /dev/null +++ b/processed_dataset/proof/0996.json @@ -0,0 +1,10 @@ +{ + "source_file": "./raw_volume-zh/volume3/chapter5.tex", + "problem_type": "proof", + "problem": "例8. 若 $0<\\beta<\\alpha<\\frac{\\pi}{2}$, 求证: $\\sin \\alpha-\\sin \\beta<\\alpha-\\beta<\\tan \\alpha-\\tan \\beta$.", + "solution": "分析:构造单位圆, 借助于三角函数线与三角函数式的关系, 把数的比较转化为几何图形面积的比较.\n证明如图(), 作单位圆, 记 $\\overparen{A P_1}=\\beta$, $\\overparen{A P_2}=\\alpha$. 则 $\\overparen{P_1 P_2}=\\alpha-\\beta, M_1 P_1=\\sin \\beta, M_2 P_2= \\sin \\alpha, A T_1=\\tan \\beta, A T_2=\\tan \\alpha, S_{\\triangle A P_2 O}=\\frac{1}{2} \\sin \\alpha$, $S_{\\triangle A P_1 O}=\\frac{1}{2} \\sin \\beta, S_{\\triangle A T_2 O}=\\frac{1}{2} \\tan \\alpha, S_{\\triangle A T_1 O}=\\frac{1}{2} \\tan \\beta$, $S_{\\text {扇形 } O A P_2}=\\frac{1}{2} \\alpha, S_{\\text {扇形 } O A P_1}=\\frac{1}{2} \\beta, S_{\\text {扇形 } O P_1 P_2}=\\frac{1}{2}(\\alpha- \\beta), S_{\\triangle O T_1 T_2}=\\frac{1}{2}(\\tan \\alpha-\\tan \\beta)$, 设 $\\triangle O P_2 C$ 的面积为 $S_1$, 则 $S_1\\frac{\\pi}{2}$, 所以对 $\\sin \\theta$ 值进行分类讨论.\n当 $\\sin \\theta \\in[0,1]$ 时, $\\frac{\\pi}{2}-\\sin \\theta \\in\\left[\\frac{\\pi}{2}-1, \\frac{\\pi}{2}\\right] \\varsubsetneqq\\left[-\\frac{\\pi}{2}, \\frac{\\pi}{2}\\right]$, $\\sin (\\cos \\theta)<\\cos (\\sin \\theta)$, 即 $\\sin (\\cos \\theta)<\\sin \\left(\\frac{\\pi}{2}-\\sin \\theta\\right)$.\n因 $y=\\sin x$ 在 $\\left[-\\frac{\\pi}{2}, \\frac{\\pi}{2}\\right]$ 上是单调递增函数,而\n$$\n\\cos \\theta-\\left(\\frac{\\pi}{2}-\\sin \\theta\\right)=\\cos \\theta+\\sin \\theta-\\frac{\\pi}{2}=\\sqrt{2} \\sin \\left(\\theta+\\frac{\\pi}{4}\\right)-\\frac{\\pi}{2}<0,\n$$\n所以 $\\cos \\theta<\\frac{\\pi}{2}-\\sin \\theta$, 从而 $\\sin (\\cos \\theta)<\\sin \\left(\\frac{\\pi}{2}-\\sin \\theta\\right)$ 成立;\n当 $\\sin \\theta \\in[-1,0)$ 时, $\\frac{\\pi}{2}+\\sin \\theta \\in\\left[\\frac{\\pi}{2}-1, \\frac{\\pi}{2}\\right] \\varsubsetneqq\\left[-\\frac{\\pi}{2}, \\frac{\\pi}{2}\\right]$, $\\sin (\\cos \\theta)<\\cos (\\sin \\theta)$, 即 $\\sin (\\cos \\theta)<\\sin \\left(\\frac{\\pi}{2}+\\sin \\theta\\right)$, 而\n$$\n\\cos \\theta-\\left(\\frac{\\pi}{2}+\\sin \\theta\\right)=\\sqrt{2} \\cos \\left(\\theta+\\frac{\\pi}{4}\\right)-\\frac{\\pi}{2}<0,\n$$\n所以 $\\cos \\theta<\\frac{\\pi}{2}+\\sin \\theta$, 得 $\\sin (\\cos \\theta)<\\sin \\left(\\frac{\\pi}{2}+\\sin \\theta\\right)$, 成立.\n综上所述, 原不等式成立.\n评注利用函数的单调性证明不等式是一种很好的方法.", + "remark": "", + "figures": [] +} \ No newline at end of file diff --git a/processed_dataset/proof/0999.json b/processed_dataset/proof/0999.json new file mode 100644 index 0000000000000000000000000000000000000000..62f2374045312f0cc6952ea6df98753e6c9ce15e --- /dev/null +++ b/processed_dataset/proof/0999.json @@ -0,0 +1,8 @@ +{ + "source_file": "./raw_volume-zh/volume3/chapter5.tex", + "problem_type": "proof", + "problem": "例11. 在 $\\triangle A B C$ 中, 求证: (1) $\\sin A+\\sin B+\\sin C \\leqslant \\frac{3 \\sqrt{3}}{2}$;\n(2) $\\sin \\frac{A}{2}+\\sin \\frac{B}{2}+\\sin \\frac{C}{2} \\leqslant \\frac{3}{2}$.", + "solution": "分析:利用函数凹凸性的有关性质证明.\n证法一先确定 $f(x)=\\sin x$ 在 $(0, \\pi)$ 内的凹凸性.\n设 $x_1, x_2 \\in(0, \\pi)$, 且 $x_1 \\neq x_2$, 则 $\\left|\\frac{x_1-x_2}{2}\\right|<\\frac{\\pi}{2}, \\frac{1}{2}\\left[f\\left(x_1\\right)+f\\left(x_2\\right)\\right]=\\frac{1}{2}\\left(\\sin x_1+\\sin x_2\\right)= \\sin \\frac{x_1+x_2}{2} \\cos \\frac{x_1-x_2}{2}<\\sin \\frac{x_1+x_2}{2}=f\\left(\\frac{x_1+x_2}{2}\\right)$, 所以 $f(x)=\\sin x$ 在 $(0$, $\\pi)$ 上是凸函数.\n在 $\\triangle A B C$ 中, $A, B, C \\in(0, \\pi)$, 得\n$$\n\\frac{\\sin A+\\sin B+\\sin C}{3} \\leqslant \\sin \\frac{A+B+C}{3}=\\sin \\frac{\\pi}{3}=\\frac{\\sqrt{3}}{2},\n$$\n所以 $\\sin A+\\sin B+\\sin C \\leqslant \\frac{3 \\sqrt{3}}{2}$, (等号当且仅当 $A=B=C=\\frac{\\pi}{3}$ 时成立)\n同理 $\\sin \\frac{A}{2}+\\sin \\frac{B}{2}+\\sin \\frac{C}{2} \\leqslant 3 \\sin \\frac{\\frac{A}{2}+\\frac{B}{2}+\\frac{C}{2}}{3}=\\frac{3}{2}$.\n证法二本题不利用凸函数性质也可证明:\n$$\n\\begin{aligned}\n& \\sin A+\\sin B+\\sin C=2 \\sin \\frac{A+B}{2} \\cos \\frac{A-B}{2}+\\sin C \\\\\n\\leqslant & 2 \\sin \\frac{\\pi-C}{2}+2 \\sin \\frac{C}{2} \\cos \\frac{C}{2}=2 \\cos \\frac{C}{2}\\left(1+\\sin \\frac{C}{2}\\right) \\\\\n= & 2 \\sqrt{1-\\sin ^2 \\frac{C}{2}} \\cdot\\left(1+\\sin \\frac{C}{2}\\right)=2 \\sqrt{\\left(1-\\sin \\frac{C}{2}\\right)\\left(1+\\sin \\frac{C}{2}\\right)^3}\n\\end{aligned}\n$$\n$$\n\\begin{aligned}\n& =2 \\sqrt{\\left(1-\\sin \\frac{C}{2}\\right)\\left(1+\\sin \\frac{C}{2}\\right)\\left(1+\\sin \\frac{C}{2}\\right)\\left(1+\\sin \\frac{C}{2}\\right)} \\\\\n& =2 \\sqrt{27\\left(1-\\sin \\frac{C}{2}\\right) \\cdot \\frac{1+\\sin \\frac{C}{2}}{3} \\cdot \\frac{1+\\sin \\frac{C}{2}}{3} \\cdot \\frac{1+\\sin \\frac{C}{2}}{3}} \\\\\n& \\leqslant 2 \\sqrt{27 \\cdot\\left(\\frac{1+1}{4}\\right)^4}=\\frac{3 \\sqrt{3}}{2} . \\\\\n& \\text { 等号当且仅当 }\\left\\{\\begin{array} { l } \n{ 1 - \\operatorname { s i n } \\frac { C } { 2 } = \\frac { 1 + \\operatorname { s i n } \\frac { C } { 2 } } { 3 } , } \\\\\n{ \\operatorname { c o s } \\frac { A - B } { 2 } = 1 , }\n\\end{array} \\text { 即 } \\left\\{\\begin{array}{l}\n\\sin \\frac{C}{2}=\\frac{1}{2}, \\\\\n\\cos \\frac{A-B}{2}=\\cos 0 .\n\\end{array} \\right.\\right.\n\\end{aligned}\n$$\n即$A=B=C=\\frac{\\pi}{3}$ 时成立.\n评注本题提及的凹、凸函数定义如下: 设 $y=f(x)$ 是 $[a, b]$ 上的连续函数, 若对 $(a, b)$ 内任意 $x_1 、 x_2\\left(x_1 \\neq x_2\\right)$, 恒有 $f\\left(\\frac{x_1+x_2}{2}\\right)>\\frac{1}{2}\\left[f\\left(x_1\\right)+\\right. \\left.f\\left(x_2\\right)\\right]$, 则称 $f(x)$ 在区间 $[a, b]$ 上是凸函数; 若恒有 $f\\left(\\frac{x_1+x_2}{2}\\right)<\\frac{1}{2}\\left[f\\left(x_1\\right)+\\right. \\left.f\\left(x_2\\right)\\right]$, 则称 $f(x)$ 在区间 $[a, b]$ 上是凹函数.\n它们的性质是:\n若 $f(x)$ 是 $[a, b]$ 上的凸函数, 则对于该区间内任意 $n$ 个自变量 $x_1$, $x_2, \\cdots, x_n$, 都有 $f\\left(\\frac{x_1+x_2+\\cdots+x_n}{n}\\right) \\geqslant \\frac{f\\left(x_1\\right)+f\\left(x_2\\right)+\\cdots+f\\left(x_n\\right)}{n}$; 同理, 若 $f(x)$ 为 $[a, b]$ 上的凹函数则取 \" $\\leqslant$ \" 号, 当且仅当 $x_1=x_2=\\cdots=x_n$ 时等号成立.\n证法二引用了不等式: $\\frac{a_1+a_2+\\cdots+a_n}{n} \\geqslant \\sqrt[n]{a_1 a_2 \\cdots a_n}$, 其中 $a_1, a_2, \\cdots$, $a_n$ 为正数.\n利用上述不等式以及本题结论, 想一想 $\\sin A \\sin B \\sin C \\leqslant \\frac{3}{8} \\sqrt{3}$, 如何证?", + "remark": "", + "figures": [] +} \ No newline at end of file diff --git a/processed_dataset/proof/1000.json b/processed_dataset/proof/1000.json new file mode 100644 index 0000000000000000000000000000000000000000..947254a462349560a735341cdd27051e810b51d1 --- /dev/null +++ b/processed_dataset/proof/1000.json @@ -0,0 +1,8 @@ +{ + "source_file": "./raw_volume-zh/volume3/chapter5.tex", + "problem_type": "proof", + "problem": "例12. 设 $\\alpha, \\beta \\in\\left(0, \\frac{\\pi}{2}\\right)$, 求证: $\\frac{1}{\\cos ^2 \\alpha}+\\frac{1}{\\sin ^2 \\alpha \\sin ^2 \\beta \\cos ^2 \\beta} \\geqslant 9$, 并讨论 $\\alpha 、 \\beta$ 为何值时, 等号成立.", + "solution": "分析:当不等式中出现两个变量时, 先通过局部调整, 消元化为一个变量再证明.\n证明因为 $\\sin ^2 \\beta \\cos ^2 \\beta=\\frac{1}{4} \\sin ^2 2 \\beta \\leqslant \\frac{1}{4}$,\n所以左边 $\\geqslant \\frac{1}{\\cos ^2 \\alpha}+\\frac{4}{\\sin ^2 \\alpha}=1+\\tan ^2 \\alpha+4+4 \\cot ^2 \\alpha$\n$$\n=5+\\tan ^2 \\alpha+\\frac{4}{\\tan ^2 \\alpha} \\geqslant 5+2 \\sqrt{4}=9 .\n$$\n当且仅当 $\\sin ^2 2 \\beta=1, \\tan ^2 \\alpha=\\frac{4}{\\tan ^2 \\alpha}$, 即 $\\alpha=\\arctan \\sqrt{2}, \\beta=\\frac{\\pi}{4}$ 时, 等号成立.\n评注本题还可加强为 $\\frac{1}{\\cos ^2 \\alpha}+\\frac{1}{\\sin ^2 \\alpha \\sin ^2 \\beta \\cos ^2 \\beta} \\geqslant 9+\\left(\\frac{2 \\cot \\alpha}{\\sin 2 \\beta}-\\tan \\alpha\\right)^2$.\n可这样证明: 因为 $\\left(\\cos ^2 \\alpha+\\sin ^2 \\alpha\\right)\\left(\\frac{1}{\\cos ^2 \\alpha}+\\frac{1}{\\sin ^2 \\alpha \\sin ^2 \\beta \\cos ^2 \\beta}\\right)-(1+ \\left.\\frac{1}{\\sin \\beta \\cos \\beta}\\right)^2=\\left(\\frac{\\cos \\alpha}{\\sin \\alpha \\sin \\beta \\cos \\beta}-\\frac{\\sin \\alpha}{\\cos \\alpha}\\right)^2$,\n所以 $\\frac{1}{\\cos ^2 \\alpha}+\\frac{1}{\\sin ^2 \\alpha \\sin ^2 \\beta \\cos ^2 \\beta}=\\left(1+\\frac{2}{\\sin 2 \\beta}\\right)^2+\\left(\\frac{2 \\cot \\alpha}{\\sin 2 \\beta}-\\tan \\alpha\\right)^2 \\geqslant 9+ \\left(\\frac{2 \\cot \\alpha}{\\sin 2 \\beta}-\\tan \\alpha\\right)^2$, 当且仅当 $\\sin 2 \\beta=1$, 即 $\\beta=\\frac{\\pi}{4}$ 时, 原不等式中等号成立.", + "remark": "", + "figures": [] +} \ No newline at end of file diff --git a/processed_dataset/proof/1001.json b/processed_dataset/proof/1001.json new file mode 100644 index 0000000000000000000000000000000000000000..a64513241f04fd10d1545a9d03e14e7fbabac181 --- /dev/null +++ b/processed_dataset/proof/1001.json @@ -0,0 +1,8 @@ +{ + "source_file": "./raw_volume-zh/volume3/chapter5.tex", + "problem_type": "proof", + "problem": "例13. (1) 求证: $\\tan ^2 \\alpha+\\tan ^2 \\beta+\\tan ^2 \\gamma \\geqslant \\tan \\alpha \\tan \\beta+\\tan \\beta \\tan \\gamma+\\tan \\gamma \\tan \\alpha$;\n (2)若 $\\alpha+\\beta+\\gamma=\\frac{\\pi}{2}$, 则 $\\tan ^2 \\alpha+\\tan ^2 \\beta+\\tan ^2 \\gamma \\geqslant 1$.", + "solution": "分析:利用基本不等式或拉格朗日恒等式证明.\n证明 (1) 根据 $a^2+b^2 \\geqslant 2 a b$, 得\n$$\n\\begin{aligned}\n& \\tan ^2 \\alpha+\\tan ^2 \\beta \\geqslant 2 \\tan \\alpha \\tan \\beta, \\\\\n& \\tan ^2 \\beta+\\tan ^2 \\gamma \\geqslant 2 \\tan \\beta \\tan \\gamma, \\\\\n& \\tan ^2 \\gamma+\\tan ^2 \\alpha \\geqslant 2 \\tan \\gamma \\tan \\alpha,\n\\end{aligned}\n$$\n三式相加, 即得\n$$\n\\tan ^2 \\alpha+\\tan ^2 \\beta+\\tan ^2 \\gamma \\geqslant \\tan \\alpha \\tan \\beta+\\tan \\beta \\tan \\gamma+\\tan \\gamma \\tan \\alpha .\n$$\n也可利用拉格朗日公式 $\\left(\\alpha_1^2+\\alpha_2^2+\\alpha_3^2\\right)\\left(\\beta_1^2+\\beta_2^2+\\beta_3^2\\right)-\\left(\\alpha_1 \\beta_1+\\alpha_2 \\beta_2+\\right. \\left.\\alpha_3 \\beta_3\\right)^2=\\left(\\alpha_1 \\beta_2-\\alpha_2 \\beta_1\\right)^2+\\left(\\alpha_1 \\beta_3-\\alpha_3 \\beta_1\\right)^2+\\left(\\alpha_2 \\beta_3-\\alpha_3 \\beta_2\\right)^2$ 证明.\n因为\n$$\n\\begin{aligned}\n& \\left(\\tan ^2 \\alpha+\\tan ^2 \\beta+\\tan ^2 \\gamma\\right)\\left(\\tan ^2 \\beta+\\tan ^2 \\gamma+\\tan ^2 \\alpha\\right) \\\\\n& -(\\tan \\alpha \\tan \\beta+\\tan \\beta \\tan \\gamma+\\tan \\gamma \\tan \\alpha)^2 \\\\\n= & \\left(\\tan \\alpha \\tan \\gamma-\\tan ^2 \\beta\\right)^2 \\\\\n& +\\left(\\tan ^2 \\alpha-\\tan \\beta \\tan \\gamma\\right)^2+\\left(\\tan \\alpha \\tan \\beta-\\tan ^2 \\gamma\\right)^2 \\\\\n\\geqslant & 0,\n\\end{aligned}\n$$\n所以\n$$\n\\left(\\tan ^2 \\alpha+\\tan ^2 \\beta+\\tan ^2 \\gamma\\right)^2 \\geqslant(\\tan \\alpha \\tan \\beta+\\tan \\beta \\tan \\gamma+\\tan \\gamma \\tan \\alpha)^2 .\n$$\n即\n$$\n\\tan ^2 \\alpha+\\tan ^2 \\beta+\\tan ^2 \\gamma \\geqslant \\tan \\alpha \\tan \\beta+\\tan \\beta \\tan \\gamma+\\tan \\gamma \\tan \\alpha .\n$$\n (2)因 $\\alpha+\\beta+\\gamma=\\frac{\\pi}{2}$, 得 $\\alpha+\\beta=\\frac{\\pi}{2}-\\gamma$, 所以\n$$\n\\tan (\\alpha+\\beta)=\\tan \\left(\\frac{\\pi}{2}-\\gamma\\right)\n$$\n即\n$$\n\\tan \\alpha \\tan \\beta+\\tan \\beta \\tan \\gamma+\\tan \\gamma \\tan \\alpha=1,\n$$\n由 (1) 得 $\\tan ^2 \\alpha+\\tan ^2 \\beta+\\tan ^2 \\gamma \\geqslant 1$.", + "remark": "", + "figures": [] +} \ No newline at end of file diff --git a/processed_dataset/proof/1002.json b/processed_dataset/proof/1002.json new file mode 100644 index 0000000000000000000000000000000000000000..af3e039b224550e2d2d5e705c3082bd2136c15bb --- /dev/null +++ b/processed_dataset/proof/1002.json @@ -0,0 +1,8 @@ +{ + "source_file": "./raw_volume-zh/volume3/chapter5.tex", + "problem_type": "proof", + "problem": "例15. 求实数 $a$ 的取值范围, 使不等式 $\\sin 2 \\theta-(2 \\sqrt{2}+\\sqrt{2} a) \\sin \\left(\\theta+\\frac{\\pi}{4}\\right)- \\frac{2 \\sqrt{2}}{\\cos \\left(\\theta-\\frac{\\pi}{4}\\right)}>-3-2 a$, 在 $\\theta \\in\\left[0, \\frac{\\pi}{2}\\right]$ 时恒成立.", + "solution": "分析:题中出现 $\\sin \\left(\\theta+\\frac{\\pi}{4}\\right)=\\cos \\left(\\theta-\\frac{\\pi}{4}\\right)=(\\sin \\theta+\\cos \\theta) \\frac{\\sqrt{2}}{2}$, 宜利用换元法,化归为有理不等式来求解.\n解设 $\\sin \\theta+\\cos \\theta=x$, 则由 $\\theta \\in\\left[0, \\frac{\\pi}{2}\\right]$ 得 $x \\in[1, \\sqrt{2}], \\sin 2 \\theta= x^2-1, \\sin \\left(\\theta+\\frac{\\pi}{4}\\right)=\\cos \\left(\\theta-\\frac{\\pi}{4}\\right)=\\frac{\\sqrt{2}}{2} x$, 原不等式化为\n$$\nx^2-1-(2+a) x-\\frac{4}{x}+3+2 a>0,\n$$\n因式分解得\n$$\n(x-2)\\left(x+\\frac{2}{x}-a\\right)>0,\n$$\n因为 $x \\in[1, \\sqrt{2}]$, 所以 $x+\\frac{2}{x}-a<0$, 即 $a>x+\\frac{2}{x}$.\n记 $f(x)=x+\\frac{2}{x}$, 可证 $f(x)$ 在 $[1, \\sqrt{2}]$ 上单调递减, 所以\n$$\nf(x)_{\\max }=f(1)=1+\\frac{2}{1}=3 .\n$$\n故 $a>3$.", + "remark": "", + "figures": [] +} \ No newline at end of file diff --git a/processed_dataset/proof/1003.json b/processed_dataset/proof/1003.json new file mode 100644 index 0000000000000000000000000000000000000000..f835e66005f52b2ee731792a0d5786d457d1e83a --- /dev/null +++ b/processed_dataset/proof/1003.json @@ -0,0 +1,8 @@ +{ + "source_file": "./raw_volume-zh/volume3/chapter5.tex", + "problem_type": "proof", + "problem": "例16. 函数 $F(x)=\\left|\\cos ^2 x+2 \\sin x \\cos x-\\sin ^2 x+A x+B\\right|$ 在 $0 \\leqslant x \\leqslant \\frac{3 \\pi}{2}$ 上的最大值 $M$ 与参数 $A 、 B$ 有关, 问 $A 、 B$ 取什么值时, $M$ 为最小? 证明你的结论.", + "solution": "分析:通过赋值猜想 $M$ 的最小值,再证明.\n解 $F(x)=|\\cos 2 x+\\sin 2 x+A x+B|=\\left|\\sqrt{2} \\sin \\left(2 x+\\frac{\\pi}{4}\\right)+A x+B\\right|$, 若 $A=B=0$, 则\n$$\nF(x)=\\left|\\sqrt{2} \\sin \\left(2 x+\\frac{\\pi}{4}\\right)\\right| .\n$$\n当 $x=\\frac{\\pi}{8}, \\frac{5 \\pi}{8}, \\frac{9 \\pi}{8}$ 时, $F(x)$ 取得最大值 $M=\\sqrt{2}$.\n下面证明对于任意实数 $A 、 B$,均有 $M \\geqslant \\sqrt{2}$.\n利用反证法.\n假设对于某一组实数 $A 、 B$, 有 $M \\leqslant \\sqrt{2}$, 则 $F\\left(\\frac{\\pi}{8}\\right) 、 F\\left(\\frac{5 \\pi}{8}\\right)$ 、\n$F\\left(\\frac{9 \\pi}{8}\\right)$ 均不大于 $\\sqrt{2}$, 从而\n$$\n\\left\\{\\begin{array}{l}\n\\sqrt{2}+\\frac{\\pi}{8} A+B \\leqslant \\sqrt{2}, \\quad\\quad(1)\\\\\n-\\sqrt{2}+\\frac{5 \\pi}{8} A+B \\geqslant-\\sqrt{2}, \\quad\\quad(2)\\\\\n\\sqrt{2}+\\frac{9 \\pi}{8} A+B \\leqslant \\sqrt{2}, \\quad\\quad(3)\n\\end{array}\\right.\n$$\n(1)+(3)-(2) $\\times 2$ 得 $0 \\leqslant 0$, 从而(1)(2)(3)等号成立, 有 $A=B=0$, 因此 $M \\geqslant \\sqrt{2}$, 当且仅当 $A=B=0$ 时, $M$ 取得最小值 $\\sqrt{2}$.", + "remark": "", + "figures": [] +} \ No newline at end of file diff --git a/processed_dataset/proof/1004.json b/processed_dataset/proof/1004.json new file mode 100644 index 0000000000000000000000000000000000000000..7c7a136b6cd8332ef850a9dc6f27a32cf72eca88 --- /dev/null +++ b/processed_dataset/proof/1004.json @@ -0,0 +1,8 @@ +{ + "source_file": "./raw_volume-zh/volume3/chapter5.tex", + "problem_type": "proof", + "problem": "例17. (1) 设函数 $f(x), g(x)$ 对所有 $x$ 满足 $-\\frac{\\pi}{2}\\sin (g(x))$;\n (2)利用或不利用(1), 证明:对所有 $x$, 有 $\\cos (\\cos x)>\\sin (\\sin x)$.", + "solution": "证明:(1) 不妨设 $g(x)>0$, 所以 $-\\frac{\\pi}{2}+g(x)\\cos \\left(\\frac{\\pi}{2}-\\right. g(x))$, 即 $\\cos (f(x))>\\sin (g(x))$;\n若 $f(x)<0$, 因为 $\\cos x$ 在 $\\left(-\\frac{\\pi}{2}, 0\\right)$ 内是增函数, 所以 $\\cos \\left(-\\frac{\\pi}{2}+g(x)\\right)< \\cos (f(x))$, 即 $\\cos (f(x))>\\sin (g(x))$;\n同理可证 $g(x) \\leqslant 0$ 时,仍有 $\\cos (f(x))>\\sin (g(x))$.\n综上所述, 对所有 $x$, 都有 $\\cos (f(x))>\\sin (g(x))$.\n(2) 因为\n$$\n\\begin{gathered}\n|\\cos x-\\sin x|=\\left|\\sqrt{2} \\cos \\left(x+\\frac{\\pi}{4}\\right)\\right| \\leqslant \\sqrt{2}<\\frac{\\pi}{2}, \\\\\n|\\cos x+\\sin x|=\\left|\\sqrt{2} \\sin \\left(x+\\frac{\\pi}{4}\\right)\\right| \\leqslant \\sqrt{2}<\\frac{\\pi}{2},\n\\end{gathered}\n$$\n所以令 $f(x)=\\cos x, g(x)=\\sin x$, 则符合第(1) 题条件, 于是对于所有 $x$, 都有 $\\cos (\\cos x)>\\sin (\\sin x)$.\n评注第(1)题的证明过程运用了分类讨论的数学思想以及三角函数的单调性;第 (2)题利用了重要不等式及三角函数的有界性.", + "remark": "", + "figures": [] +} \ No newline at end of file diff --git a/processed_dataset/proof/1005.json b/processed_dataset/proof/1005.json new file mode 100644 index 0000000000000000000000000000000000000000..583ac3754a89faa8366e87508f1a8ef32a27a93e --- /dev/null +++ b/processed_dataset/proof/1005.json @@ -0,0 +1,8 @@ +{ + "source_file": "./raw_volume-zh/volume3/chapter5.tex", + "problem_type": "proof", + "problem": "例18. 已知 $a 、 b 、 A 、 B$ 都是实数, 若对于一切实数 $x$, 都有\n$$\nf(x)=1-a \\cos x-b \\sin x-A \\cos 2 x-B \\sin 2 x \\geqslant 0 .\n$$\n求证: $a^2+b^2 \\leqslant 2, A^2+B^2 \\leqslant 1$.", + "solution": "分析:题中所给函数有较多的参数 $a 、 b 、 A 、 B$, 结合结论, 应引人辅助参数:\n$$\n\\begin{aligned}\na \\cos x+b \\sin x & =\\sqrt{a^2+b^2} \\sin (x+\\theta), \\\\\nA \\cos 2 x+B \\sin 2 x & =\\sqrt{A^2+B^2} \\sin (2 x+\\varphi),\n\\end{aligned}\n$$\n这样就使待证的参数较为集中, 再利用 $f(x) \\geqslant 0$ 恒成立, 选取适当的 $x$ 值以证明结论.\n证明若 $a^2+b^2=0, A^2+B^2=0$, 则结论显然成立;\n若 $a^2+b^2 \\neq 0, A^2+B^2 \\neq 0$, 令\n$$\n\\begin{aligned}\n& \\sin \\theta=\\frac{a}{\\sqrt{a^2+b^2}}, \\cos \\theta=-\\frac{b}{\\sqrt{a^2+b^2}}, \\\\\n& \\sin \\varphi=\\frac{A}{\\sqrt{A^2+B^2}}, \\cos \\varphi=\\frac{B}{\\sqrt{A^2+B^2}},\n\\end{aligned}\n$$\n于是\n$$\n\\begin{gathered}\nf(x)=1-\\sqrt{a^2+b^2} \\sin (x+\\theta)-\\sqrt{A^2+B^2} \\sin (2 x+\\varphi) \\geqslant 0, \\quad\\quad(1)\\\\\nf\\left(x+\\frac{\\pi}{2}\\right)=1-\\sqrt{a^2+b^2} \\cos (x+\\theta)+\\sqrt{A^2+B^2} \\sin (2 x+\\varphi) \\geqslant 0, \\quad\\quad(2)\n\\end{gathered}\n$$\n(1) +(2)得\n$$\n2-\\sqrt{a^2+b^2}[\\sin (x+\\theta)+\\cos (x+\\theta)] \\geqslant 0,\n$$\n即\n$$\n2-\\sqrt{2\\left(a^2+b^2\\right)} \\cdot \\sin \\left(x+\\theta+\\frac{\\pi}{4}\\right) \\geqslant 0,\n$$\n所以 $\\sqrt{a^2+b^2} \\sin \\left(x+\\theta+\\frac{\\pi}{4}\\right) \\leqslant \\sqrt{2}$ 对一切 $x$ 均成立.\n取 $x+\\theta+\\frac{\\pi}{4}=\\frac{\\pi}{2}$, 得 $x=\\frac{\\pi}{4}-\\theta$, 有 $\\sqrt{a^2+b^2} \\leqslant \\sqrt{2}$, 即 $a^2+b^2 \\leqslant 2$.\n又\n$$\nf(x+\\pi)=1+\\sqrt{a^2+b^2} \\sin (x+\\theta)-\\sqrt{A^2+B^2} \\sin (2 x+\\varphi) \\geqslant 0, \\quad\\quad(3)\n$$\n(1)+(3)得\n$$\n2-2 \\sqrt{A^2+B^2} \\sin (2 x+\\varphi) \\geqslant 0,\n$$\n即 $\\sqrt{A^2+B^2} \\sin (2 x+\\varphi) \\leqslant 1$, 取 $2 x+\\varphi=\\frac{\\pi}{2}$, 即 $x=\\frac{\\pi}{4}-\\frac{\\varphi}{2}$ 时, 有 $\\sqrt{A^2+B^2} \\leqslant 1$, 即 $A^2+B^2 \\leqslant 1$.\n评注对于恒等式或恒成立的不等式, 通常利用赋值法.", + "remark": "", + "figures": [] +} \ No newline at end of file diff --git a/processed_dataset/proof/1006.json b/processed_dataset/proof/1006.json new file mode 100644 index 0000000000000000000000000000000000000000..73c1f238ea43053a572a3c7407baf1dc84503ad9 --- /dev/null +++ b/processed_dataset/proof/1006.json @@ -0,0 +1,8 @@ +{ + "source_file": "./raw_volume-zh/volume3/chapter5.tex", + "problem_type": "proof", + "problem": "例19. 求证: $\\frac{1}{3}<\\sin 20^{\\circ}<\\frac{7}{20}$.", + "solution": "分析:因 $60^{\\circ}=3 \\times 20^{\\circ}$, 所以利用倍角公式将 $\\sin 20^{\\circ}$ 转化到 $\\sin 60^{\\circ}$ 后构造方程来证明.\n证明设 $\\sin 20^{\\circ}=x$, 则 $\\sin 60^{\\circ}=\\sin 3 \\times 20^{\\circ}=3 \\sin 20^{\\circ}-4 \\sin ^3 20^{\\circ}$, 从而 $4 x^3-3 x+\\frac{\\sqrt{3}}{2}=0$, 令 $f(x)=4 x^3-3 x+\\frac{\\sqrt{3}}{2}$, 则\n$$\n\\begin{aligned}\n& f(-1)=-1+\\frac{\\sqrt{3}}{2}<0, \\\\\n& f(0)=\\frac{\\sqrt{3}}{2}>0, \\\\\n& f\\left(\\frac{1}{3}\\right)=\\frac{4}{27}-1+\\frac{\\sqrt{3}}{2}>0, \\\\\n& f\\left(\\frac{7}{20}\\right)=4 \\times \\frac{343}{8000}-\\frac{21}{20}+\\frac{\\sqrt{3}}{2}<0, \\\\\n& f\\left(\\frac{1}{2}\\right)=\\frac{1}{2}-\\frac{3}{2}+\\frac{\\sqrt{3}}{2}<0, \\\\\n& f(1)=1+\\frac{\\sqrt{3}}{2}>0,\n\\end{aligned}\n$$\n所以 $f(x)$ 在 $(-1,0) 、\\left(\\frac{1}{3}, \\frac{7}{20}\\right) 、\\left(\\frac{1}{2}, 1\\right)$ 内各有一根, 但 $0<\\sin 20^{\\circ}< \\sin 30^{\\circ}$, 所以 $\\sin 20^{\\circ} \\in\\left(\\frac{1}{3}, \\frac{7}{20}\\right)$.\n即 $\\frac{1}{3}<\\sin 20^{\\circ}<\\frac{7}{20}$.\n评注方程的根与函数的图象有密切联系, 即函数图象与 $x$ 轴的交点横坐标就是方程的根.\n它可通过作出函数图象来观察根的分布情况.\n如函数 $y= f(x)$ 中, $f(a) f(b)<0$, 则方程 $f(x)=0$ 必有一根在 $(a, b)$ 中.", + "remark": "", + "figures": [] +} \ No newline at end of file diff --git a/processed_dataset/proof/1007.json b/processed_dataset/proof/1007.json new file mode 100644 index 0000000000000000000000000000000000000000..69ba37f092a723debd65d1d2779e327d9f30c6ce --- /dev/null +++ b/processed_dataset/proof/1007.json @@ -0,0 +1,10 @@ +{ + "source_file": "./raw_volume-zh/volume3/chapter5.tex", + "problem_type": "proof", + "problem": "例20. 平面上任给五个相异的点, 它们之间的最大距离与最小距离之比记为 $\\lambda$, 求证: $\\lambda \\geqslant 2 \\sin 54^{\\circ}$, 并讨论等号成立的充要条件.", + "solution": "分析:为什么出现 $54^{\\circ}$, 恰与正五边形有关, 因此, 应考虑这五个点与正五边形的关系.\n证明 (1)若五点中有三点共线, 不妨设 $A 、 B 、 C$ 三点共线,且 $B$ 在线段 $A C$ 上, 若 $A B \\leqslant B C$, 则 $\\frac{A C}{A B} \\geqslant 2>2 \\sin 54^{\\circ}$. 但这五点中最大的距离 $\\geqslant A C$, 最小的距离 $\\leqslant A B$, 从而 $\\lambda \\geqslant 2>2 \\sin 54^{\\circ}$.\n (2)若五点组成凸五边形, 则这个五边形必有一内角 $\\geqslant 108^{\\circ}$, 不妨设为 $A$,与 $A$ 相邻的顶点为 $B$ 和 $C$, 在 $\\triangle A B C$ 中, $A \\geqslant 108^{\\circ}$, 若 $B \\leqslant C$, 则 $B \\leqslant 36^{\\circ}$, 于是\n$$\n\\frac{B C}{A C}=\\frac{\\sin (B+C)}{\\sin B} \\geqslant \\frac{\\sin 2 B}{\\sin B}=2 \\cos B \\geqslant 2 \\cos 36^{\\circ}=2 \\sin 54^{\\circ},\n$$\n从而 $\\lambda \\geqslant \\frac{B C}{A C}>2 \\sin 54^{\\circ}$, 等号当且仅当这个五边形为正五边形时成立.\n(3) 若五点组成一个四边形 $A B C D$, 第五个点 $E$ 在四边形 $A B C D$ 内, 如图(), 连 $A C$, 则 $E$ 必在 $\\triangle A B C$ 或 $\\triangle A C D$ 内, 不妨设 $E$ 在 $\\triangle A B C$ 内, 则 $\\angle A E B 、 \\angle A E C$ 、 $\\angle B E C$ 中至少有一个角 $\\geqslant 120^{\\circ}>108^{\\circ}$, 可以化归为(2), 可证 $\\lambda>2 \\sin 54^{\\circ}$.\n (4)当这五点组成 $\\triangle A B C$, 而第四、五两点在 $\\triangle A B C$ 内, 任取其中一点 $E$, 则可化归为情形 (3), 仍可证 $\\lambda>2 \\sin 54^{\\circ}$.\n综上所述, $\\lambda \\geqslant 2 \\sin 54^{\\circ}$, 当且仅当这五点是正五边形的顶点时等号成立.\n评注分类讨论是本题的主要思想方法.", + "remark": "", + "figures": [ + "./images/volume3/figures/fig-c5i8.png" + ] +} \ No newline at end of file diff --git a/processed_dataset/proof/1008.json b/processed_dataset/proof/1008.json new file mode 100644 index 0000000000000000000000000000000000000000..06190ffbc657bb688c815258c2f54d2b5f9ff6f0 --- /dev/null +++ b/processed_dataset/proof/1008.json @@ -0,0 +1,10 @@ +{ + "source_file": "./raw_volume-zh/volume3/chapter6.tex", + "problem_type": "proof", + "problem": "例1. $ \\alpha$ 是 $(0,1)$ 内一个实数, 考虑数列 $\\left\\{x_n\\right\\}(n=0,1,2, \\cdots)$, 其中 $x_0=\\alpha, x_n=\\frac{4}{\\pi^2}\\left(\\arccos x_{n-1}+\\frac{\\pi}{2}\\right) \\cdot \\arcsin x_{n-1}, n=1,2,3, \\cdots$.\n求证: 当 $n$ 趋于无限时, 数列 $\\left\\{x_n\\right\\}, n=0,1,2, \\cdots$ 有一个极限, 并求这个极限.", + "solution": "证明:设 $\\arccos x_{n-1}=\\theta, \\arcsin x_{n-1}=\\varphi$, 其中 $\\theta \\in[0, \\pi], \\varphi \\in\\left[-\\frac{\\pi}{2}\\right.$, $\\left.\\frac{\\pi}{2}\\right]$, 因此有 $\\cos \\theta=x_{n-1}=\\sin \\varphi=\\cos \\left(\\frac{\\pi}{2}-\\varphi\\right)$.\n因为 $\\frac{\\pi}{2}-\\varphi \\in[0, \\pi], \\theta \\in[0, \\pi]$, 所以 $\\theta=\\frac{\\pi}{2}-\\varphi$.\n即\n$$\n\\arccos x_{n-1}=\\frac{\\pi}{2}-\\arcsin x_{n-1},\n$$\n故\n$$\nx_n=\\frac{4}{\\pi^2}\\left(\\arccos x_{n-1}+\\frac{\\pi}{2}\\right)\\left(\\frac{\\pi}{2}-\\arccos x_{n-1}\\right), \\quad\\quad(1)\n$$\n即\n$$\nx_n=1-\\frac{4}{\\pi^2} \\arccos ^2 x_{n-1},\n$$\n已知 $x_0 \\in(0,1)$, 设 $x_{n-1} \\in(0,1)$, 则 $0<\\left|\\arccos x_{n-1}\\right|<\\frac{\\pi}{2}$, 从而由 (1) 可知 $x_n \\in(0,1)$, 因此,对于任意非负整数 $n$, 都有 $x_n \\in(0,1)$.\n下面证明 $x_n$ 递增, 即证 $1-\\frac{4}{\\pi^2} \\arccos ^2 x_{n-1}>x_{n-1} \\Leftrightarrow \\frac{\\pi^2}{4}\\left(1-x_{n-1}\\right)> \\arccos ^2 x_{n-1}(n \\in \\mathbf{N})$, 因为 $\\arccos x_{n-1}=\\theta$, 上式等价于 $\\frac{\\pi^2}{4}(1-\\cos \\theta)>\\theta^2 \\Leftrightarrow \\frac{\\pi^2}{2} \\sin \\frac{2 \\theta}{2}>\\theta^2$, 从而只需证明\n$$\n\\sin \\frac{\\theta}{2}>\\frac{\\sqrt{2}}{\\pi} \\theta, \\theta \\in\\left(0, \\frac{\\pi}{2}\\right), \\quad\\quad(2)\n$$\n作函数 $y=\\sin \\frac{\\theta}{2}, \\theta \\in\\left(0, \\frac{\\pi}{2}\\right)$ 的图象(如图()), 连结这段正弦曲线的两个端点的直线段方程是 $y= \\frac{\\sqrt{2}}{\\pi} \\theta, 0<\\theta<\\frac{\\pi}{2}$, 由正弦函数的凸性可知, 这一段正弦曲线在这一个直线段的上方.\n因此, 当 $\\theta \\in(0$, $\\left.\\frac{\\pi}{2}\\right)$ 时,有 $\\sin \\frac{\\theta}{2}>\\frac{\\sqrt{2}}{\\pi} \\theta$, 从而 (2) 式成立.\n即 $x_n>$\n$x_{n-1}$,也就是说给定数列严格单调递增.\n再由 $x_n<1, n \\in \\mathrm{N}$. 即知, 当正整数 $n$ 趋于无限时, $x_n$ 的极限存在, 设 $\\lim _{n \\rightarrow \\infty} x_n=\\beta, 0<\\beta \\leqslant 1$.\n在(1)式两端同时取极限,有 $\\beta=1-\\frac{4}{\\pi^2} \\arccos ^2 \\beta$.\n令 $t=\\arccos \\beta, 0 \\leqslant t<\\frac{\\pi}{2}$, 代入前一式, 得 $\\cos t=1-\\frac{4}{\\pi^2} t^2$, 即 $\\sin ^2 \\frac{t}{2}= \\frac{2}{\\pi^2} t^2 \\Rightarrow \\sin \\frac{t}{2}=\\frac{\\sqrt{2}}{\\pi} t$, 由 (2) 式知, 上式只有一解 $t=0$, 从而 $\\beta=\\cos t=1$, 故所求极限为 1 .\n评注利用三角函数有界及单调性来证明极限的存在.", + "remark": "", + "figures": [ + "./images/volume3/figures/fig-c6i1.png" + ] +} \ No newline at end of file diff --git a/processed_dataset/proof/1009.json b/processed_dataset/proof/1009.json new file mode 100644 index 0000000000000000000000000000000000000000..f20c26f6e8f70a61cb713bbd8b6228d90723a25c --- /dev/null +++ b/processed_dataset/proof/1009.json @@ -0,0 +1,8 @@ +{ + "source_file": "./raw_volume-zh/volume3/chapter6.tex", + "problem_type": "proof", + "problem": "例2. 是否存在两个实数列 $\\left\\{a_n\\right\\},\\left\\{b_n\\right\\},(n \\in \\mathbf{N})$, 使得对于每一个 $n \\in \\mathbf{N}$ 和 $x \\in(0,1)$, 都有 $\\frac{3}{2} \\pi \\leqslant a_n \\leqslant b_n, \\cos a_n x+\\cos b_n x \\geqslant-\\frac{1}{n}$ ?", + "solution": "解:如果存在满足所给条件的数列 $\\left\\{a_n\\right\\}$ 和 $\\left\\{b_n\\right\\}$, 由于 $0<\\frac{\\pi}{a_n} \\leqslant \\frac{2}{3}<1$, 可得 $\\cos \\frac{b_n}{a_n} \\pi \\geqslant 1-\\frac{1}{n}$, 取 $x=\\frac{\\pi}{a_n}$, 再由 $\\frac{b_n}{a_n} \\pi \\geqslant \\pi$, 可知存在 $k_n \\in \\mathbf{N}$ 和 $\\varphi_n$ 使得\n$\\left|\\varphi_n\\right| \\leqslant \\arccos \\left(1-\\frac{1}{n}\\right) \\leqslant \\frac{\\pi}{2}$, 且 $\\pi \\leqslant \\frac{b_n}{a_n} \\pi=\\varphi_n+2 k_n \\pi. \\quad\\quad(1)$\n由(1)可得 $0<\\left(2 k_n-1\\right) \\pi<\\frac{b_n}{a_n} \\pi \\Rightarrow 0<\\frac{\\left(2 k_n-1\\right) \\pi}{b_n}<\\frac{\\pi}{a_n} \\leqslant \\frac{2}{3}<1$.\n于是, 对任意 $n \\in \\mathbf{N}$, 有 $\\cos \\frac{a_n}{b_n}\\left(2 k_n-1\\right) \\pi \\geqslant 1-\\frac{1}{n}$,\n由(1)和(2)得, $\\cos \\theta_n \\geqslant 1-\\frac{1}{n}$, 对于任意 $n \\in \\mathbf{N}$, 成立.\n其中 $\\theta_n=\\frac{\\left(2 k_n-1\\right) \\pi^2}{\\varphi_n+2 k_n \\pi}$.\n显然对于每一个 $n \\in \\mathbf{N}$, 都有 $0<\\frac{\\left(2 k_n-1\\right) \\pi}{\\left(2 k_n+1\\right)}<\\theta_n<\\pi$, 所以\n$$\n\\cos \\frac{2 k_n-1}{2 k_n+1} \\pi=\\cos \\left(1-\\frac{2}{2 k_n+1}\\right) \\pi \\geqslant 1-\\frac{1}{n} \\quad\\quad(2).\n$$\n由于 $k_n \\in \\mathbf{N}$, 从而 $\\frac{\\pi}{3} \\leqslant\\left(1-\\frac{2}{2 k_n+1}\\right) \\pi, n=1,2,3, \\cdots$. 于是对于任意 $n \\in \\mathbf{N}$, 有 $\\frac{1}{2}=\\cos \\frac{\\pi}{3} \\geqslant 1-\\frac{1}{n}$, 矛盾.\n以上证明了满足要求条件的数列 $\\left\\{a_n\\right\\},\\left\\{b_n\\right\\}$ 不存在.\n评注这类问题,假设存在,推出矛盾.\n从而否定存在,或举出满足条件的数列.", + "remark": "", + "figures": [] +} \ No newline at end of file diff --git a/processed_dataset/proof/1010.json b/processed_dataset/proof/1010.json new file mode 100644 index 0000000000000000000000000000000000000000..a6dc8ca08dbfa6f17a4d0bfaa8f2ef397969e9a1 --- /dev/null +++ b/processed_dataset/proof/1010.json @@ -0,0 +1,8 @@ +{ + "source_file": "./raw_volume-zh/volume3/chapter6.tex", + "problem_type": "proof", + "problem": "例5. 求证: 对任意 $n \\in \\mathbf{N}, \\alpha \\in \\mathbf{R}$, 并且 $n \\neq 1, \\sin \\alpha \\neq 0$, 多项式 $P(x)= x^n \\sin \\alpha-x \\sin n \\alpha+\\sin (n-1) \\alpha$ 被多项式 $Q(x)=x^2-2 x \\cos \\alpha+1$ 整除.", + "solution": "证明:因为 $Q(x)=x^2-2 x \\cos \\alpha+1=[x-(\\cos \\alpha+i \\sin \\alpha)][x- (\\cos \\alpha-i \\sin \\alpha)$ ], 由 $P(\\cos \\alpha \\pm i \\sin \\alpha)=(\\cos \\alpha \\pm i \\sin \\alpha)^n \\sin \\alpha-(\\cos \\alpha \\pm i \\sin \\alpha) \\sin n \\alpha+\\sin (n-1) \\alpha=(\\cos n \\alpha \\pm i \\sin n \\alpha) \\sin \\alpha-(\\cos \\alpha \\pm i \\sin \\alpha) \\sin n \\alpha+\\sin (n-$ 1) $\\alpha=\\cos n \\alpha \\sin \\alpha-\\cos \\alpha \\sin n \\alpha+\\sin (n-1) \\alpha=\\sin (\\alpha-n \\alpha)-\\sin (n-1) \\alpha=0$.\n所以 $P(x)$ 被 $x-(\\cos \\alpha+i \\sin \\alpha)$ 与 $x-(\\cos \\alpha-i \\sin \\alpha)$ 整除, 从而被 $[x- (\\cos \\alpha+i \\sin \\alpha)][x-(\\cos \\alpha-i \\sin \\alpha)]=x^2-2 x \\cos \\alpha+1=Q(x)$ 整除.\n评注如果 $P(x)$ 是一个多次式函数, 且 $P(x)=0$ 有异根 $x_1, x_2, \\cdots$, $x_n$, 则 $\\prod_{i=1}^n\\left(x-x_i\\right) \\mid P(x)$.", + "remark": "", + "figures": [] +} \ No newline at end of file diff --git a/processed_dataset/proof/1011.json b/processed_dataset/proof/1011.json new file mode 100644 index 0000000000000000000000000000000000000000..825d85abe708f0fc9ce97b33aa0d3164d15dd021 --- /dev/null +++ b/processed_dataset/proof/1011.json @@ -0,0 +1,8 @@ +{ + "source_file": "./raw_volume-zh/volume3/chapter6.tex", + "problem_type": "proof", + "problem": "例8. 已知 $x 、 y$ 都是正整数,且 $x-y=1$,\n$$\nA=\\left(\\sqrt{x}-\\frac{1}{\\sqrt{x}}\\right)\\left(\\sqrt{y}+\\frac{1}{\\sqrt{y}}\\right) \\cdot \\frac{1}{x},\n$$\n求证: $00)$ 时, 可设 $x=a \\sec ^2 \\alpha, y= a \\tan ^2 \\alpha$, 其中 $\\alpha \\in\\left(0, \\frac{\\pi}{2}\\right)$; 在形如 $\\sqrt{a^2-x^2}$ 中, 可设 $x=a \\cos \\alpha$, 其中 $\\alpha \\in[0$, $\\pi]$, 或设 $x=a \\sin \\alpha$, 其中 $\\alpha \\in\\left[-\\frac{\\pi}{2}, \\frac{\\pi}{2}\\right]$; 在形如 $\\sqrt{x^2-a^2}$ 中, 可设 $x=a \\sec \\alpha$, 其中 $\\alpha \\in\\left[0, \\frac{\\pi}{2}\\right) \\cup\\left(\\frac{\\pi}{2}, \\pi\\right]$.", + "remark": "", + "figures": [] +} \ No newline at end of file diff --git a/processed_dataset/proof/1012.json b/processed_dataset/proof/1012.json new file mode 100644 index 0000000000000000000000000000000000000000..0a53aa59626b701def3955243f181abdcfe4ebeb --- /dev/null +++ b/processed_dataset/proof/1012.json @@ -0,0 +1,8 @@ +{ + "source_file": "./raw_volume-zh/volume3/chapter6.tex", + "problem_type": "proof", + "problem": "例9. 证明: $\\frac{1}{\\sqrt{2006}}<\\underbrace{\\sin \\sin \\cdots \\sin }_{2005 } \\frac{\\sqrt{2}}{2}<\\frac{2}{\\sqrt{2006}}$.", + "solution": "分析:把问题一般化, 转证: 对任意 $n \\in \\mathbf{N}^*, n \\geqslant 2$, 都有\n$$\n\\sqrt{n+1}<\\underbrace{\\sin \\sin \\cdots \\sin }_{n } \\frac{\\sqrt{2}}{2}<\\frac{2}{\\sqrt{n+1}} \\quad\\quad(1).\n$$\n证明先证明下述引理.\n引理设 $m \\in \\mathbf{N}^*, m \\geqslant 2$, 则 $\\frac{1}{\\sqrt{m+1}}<\\sin \\frac{1}{\\sqrt{m}}, \\quad\\quad(2)$\n并且\n$$\n\\sin \\frac{2}{\\sqrt{m}}<\\frac{2}{\\sqrt{m+1}} . \\quad\\quad(3)\n$$\n引理证明对(2)式, 令 $m=\\cot ^2 \\alpha, \\alpha \\in\\left(0, \\frac{\\pi}{2}\\right)$, 则 (2) $\\Leftrightarrow \\sin \\alpha<\\operatorname{sintan} \\alpha \\Leftrightarrow \\alpha<\\tan \\alpha$, 由三角函数解可知当 $\\alpha \\in\\left(0, \\frac{\\pi}{2}\\right)$ 时, $\\sin \\alpha<\\alpha<\\tan \\alpha$, 故 (2) 成立.\n对于 (3) 式, 仍然令 $m=\\cot ^2 \\alpha, \\alpha \\in\\left(0, \\frac{\\pi}{2}\\right)$, 则 (3) $\\Leftrightarrow \\sin (2 \\tan \\alpha)< 2 \\sin \\alpha \\Leftrightarrow \\sin (\\tan \\alpha) \\cos (\\tan \\alpha)<\\sin \\alpha \\Leftrightarrow \\tan \\alpha \\cos (\\tan \\alpha)<\\sin \\alpha \\Leftrightarrow \\cos (\\tan \\alpha)< \\cos \\alpha \\Leftrightarrow \\tan \\alpha>\\alpha$, 所以 (3) 式成立.\n注意, 上述证明中都需要用到 $\\tan \\alpha<\\frac{\\pi}{2}$. 这可由 $\\tan \\alpha=\\frac{1}{\\sqrt{n}} \\leqslant \\frac{\\sqrt{2}}{2}<\\frac{\\pi}{2}$, 得到.\n现在回证(1)式, 由引理中的(2)可知,\n$$\n\\underbrace{\\sin \\sin \\cdots \\sin }_{n } \\frac{\\sqrt{2}}{2}=\\underbrace{\\sin \\sin \\cdots \\sin }_{n } \\frac{1}{\\sqrt{2}}>\\underbrace{\\sin \\cdots \\sin }_{n-1 } \\frac{1}{\\sqrt{3}}>\\cdots>\\sin \\frac{1}{\\sqrt{n}}>\\frac{1}{\\sqrt{n+1}},\n$$\n由引理中的(3)可知,\n$$\n\\underbrace{\\sin \\sin \\cdots \\sin }_{n } \\frac{\\sqrt{2}}{2}=\\underbrace{\\sin \\cdots \\sin }_{n } \\frac{2}{\\sqrt{8}}<\\underbrace{\\sin \\cdots \\cdot \\sin }_{n-} \\frac{2}{\\sqrt{9}}<\\cdots<\\sin \\frac{2}{\\sqrt{n+8}}<\\frac{2}{\\sqrt{n+9}} .\n$$\n又 $\\frac{2}{\\sqrt{n+9}}<\\frac{2}{\\sqrt{n+1}}$ 知 (1) 式成立.\n取 $n=2005$ 命题得证.\n评注此题本质上涉及函数迭代.\n事实上,记 $f(x)=\\sin x$, 用 $f^{(n)}(x)$ 表 $\\frac{\\sqrt{2}}{2}$. 这里 $h(x)=\\frac{x}{\\sqrt{1+x}}, g(x)=\\frac{x}{\\sqrt{1+\\left(\\frac{x}{2}\\right)^2}}$, 结合在区间 $\\left(0, \\frac{\\pi}{2}\\right)$ 上, $h(x) 、 f(x) 、 g(x)$ 都是单调递增函数, 可知 $01)$ 的三角形满足条件, 它的三个角分别为 $\\alpha 、 2 \\alpha 、 \\pi-3 \\alpha$. 显然 $0<\\alpha<\\frac{\\pi}{3}$.\n由于 $\\frac{\\sin (\\pi-3 \\alpha)}{\\sin \\alpha}=\\frac{\\sin 3 \\alpha}{\\sin \\alpha}=3-4 \\sin ^2 \\alpha=4 \\cos ^2 \\alpha-1=\\left(\\frac{\\sin 2 \\alpha}{\\sin \\alpha}\\right)^2-1$,\n 若 $A=\\alpha, B=2 \\alpha$, 则 $C=\\pi-3 \\alpha$. 由正弦定理得\n$$\n\\begin{gathered}\n\\frac{n-1}{\\sin \\alpha}=\\frac{n}{\\sin 2 \\alpha}=\\frac{n+1}{\\sin 3 \\alpha}, \\\\\n\\frac{n+1}{n-1}=\\left(\\frac{n}{n-1}\\right)^2-1,\n\\end{gathered}\n$$\n解得 $n=2$. 因此 $a=1, b=2, c=3$. 但此时不能构成三角形.\n 若 $A=\\alpha, C=2 \\alpha$, 则 $B=\\pi-3 \\alpha$, 由正弦定理得 $\\frac{n-1}{\\sin \\alpha}=\\frac{n}{\\sin 3 \\alpha}= \\frac{n+1}{\\sin 2 \\alpha}, \\frac{n}{n-1}=\\left(\\frac{n+1}{n-1}\\right)^2-1$, 解得 $n=5$.\n因此 $a=4, b=5, c=6$ 能构成三角形, 此时, 由余弦定理得 $\\cos A=\\frac{3}{4}$, $\\cos C=\\frac{1}{8}$, 满足 $\\cos C=\\cos 2 A$, 从而满足 $C=2 A$.\n 若 $B=\\alpha, C=2 \\alpha$, 则 $A=\\pi-3 \\alpha$, 由正弦定理得\n$$\n\\frac{n-1}{\\sin 3 \\alpha}=\\frac{n}{\\sin \\alpha}=\\frac{n+1}{\\sin 2 \\alpha}, \\frac{n-1}{n}=\\left(\\frac{n+2}{n}\\right)^2-1 .\n$$\n$n^2-3 n-1=0$,这个方程没有整数解.\n综上所述, 满足条件的三角形是唯一的, 它的三边的长分别为 $4 、 5 、 6$.", + "remark": "", + "figures": [] +} \ No newline at end of file diff --git a/processed_dataset/proof/1016.json b/processed_dataset/proof/1016.json new file mode 100644 index 0000000000000000000000000000000000000000..d982f1de5c6058035a751fce8d8db454e43df350 --- /dev/null +++ b/processed_dataset/proof/1016.json @@ -0,0 +1,10 @@ +{ + "source_file": "./raw_volume-zh/volume3/chapter6.tex", + "problem_type": "proof", + "problem": "例16. (Menelaus 定理) 直线 $l$ 与 $\\triangle A B C$ 的三边 $A B 、 B C 、 C A$ 或它们的延长线依次相交于 $D 、 E 、 F$, 求证: $\\frac{A D}{D B} \\cdot \\frac{B E}{E C} \\cdot \\frac{C F}{F A}=1$.", + "solution": "分析:结论的六条线段 $A D$ 和 $F A 、 B E$ 和 $D B 、 C F$ 和 $E C$ 分别是 $\\triangle A D F 、 \\triangle B D E 、 \\triangle C F E$ 的两条边, 可运用正弦定理, 将比值 $\\frac{A D}{F A} 、 \\frac{B E}{D B} 、 \\frac{C F}{E C}$ 表示成角的正弦的比值.\n证明如图(), 设 $\\angle A D F=\\alpha, \\angle C F E=\\beta$, $\\angle C E F=\\gamma$, 由正弦定理, 在 $\\triangle A D F$ 中, $\\frac{A D}{F A}=\\frac{\\sin \\beta}{\\sin \\alpha}$.\n在 $\\triangle B D E$ 中, $\\frac{B E}{D B}=\\frac{\\sin \\left(180^{\\circ}-\\alpha\\right)}{\\sin \\gamma}=\\frac{\\sin \\alpha}{\\sin \\gamma}$, 在 $\\triangle C F E$ 中, $\\frac{C F}{E C}=\\frac{\\sin \\gamma}{\\sin \\beta}$. 所以左边 $=\\frac{\\sin \\beta}{\\sin \\alpha}$.\n$\\frac{\\sin \\alpha}{\\sin \\gamma} \\cdot \\frac{\\sin \\gamma}{\\sin \\beta}=1$.\n评注可以证明, Menelaus 定理的逆命题也是正确的.\n在平面几何的竞赛中, 常用它们来证明有关三点共线的问题.", + "remark": "", + "figures": [ + "./images/volume3/figures/fig-c6i4.png" + ] +} \ No newline at end of file diff --git a/processed_dataset/proof/1017.json b/processed_dataset/proof/1017.json new file mode 100644 index 0000000000000000000000000000000000000000..9786c286ec345aaff8ca6fab2e90f6e7b30a84a0 --- /dev/null +++ b/processed_dataset/proof/1017.json @@ -0,0 +1,10 @@ +{ + "source_file": "./raw_volume-zh/volume3/chapter6.tex", + "problem_type": "proof", + "problem": "例17. (Ceva 定理) $P$ 是 $\\triangle A B C$ 外的一点,直线 $P A 、 P B 、 P C$ 依次与 $\\triangle A B C$ 的三边 $B C 、 C A 、 A B$ 或者它们的延长线相交于 $D 、 E 、 F$, 求证: $\\frac{A F}{F B} \\cdot \\frac{B D}{D C} \\cdot \\frac{C E}{E A}=1$.", + "solution": "分析:$A 、 B 、 F$ 在一条直线上, 且 $A F 、 F B$ 分别是 $\\triangle P A F 、 \\triangle P F B$ 的边, 可以用这两个三角形面积之比表示 $\\frac{A F}{F B}$.\n证明如图(), 设 $\\angle B P F=\\alpha, \\angle A P B=\\beta$,\n$$\n\\begin{gathered}\n\\frac{A F^2}{F B^2}=\\frac{\\frac{1}{2} P A \\cdot P F \\cdot \\sin (\\alpha+\\beta)}{\\frac{1}{2} P B \\cdot P F \\cdot \\sin \\alpha}=\\frac{P A \\sin (\\alpha+\\beta),}{P B \\cdot \\sin \\alpha}, \\\\\n\\frac{B D^2}{D C^2}=\\frac{\\frac{1}{2} P B \\cdot P D \\cdot \\sin \\beta}{\\frac{1}{2} P C \\cdot P D \\cdot \\sin \\left[180^{\\circ}-(\\alpha+\\beta)\\right]}=\\frac{P B \\sin \\beta}{P C \\sin (\\alpha+\\beta)}, \\\\\n\\frac{C E^2}{E A^2}=\\frac{\\frac{1}{2} P C \\cdot P E \\cdot \\sin \\alpha}{\\frac{1}{2} P A \\cdot P E \\cdot \\sin \\left(180^{\\circ}-\\beta\\right)}=\\frac{P C \\sin \\alpha}{P A \\sin \\beta},\n\\end{gathered}\n$$\n所以\n$$\n\\frac{A F}{F B} \\cdot \\frac{B D}{D C} \\cdot \\frac{C E}{E A}=1\n$$\n评注可以证明 Ceva 定理的逆命题也是正确的.\n在平面几何的竞赛题中, 常用它们来论证有关三线共点的问题.", + "remark": "", + "figures": [ + "./images/volume3/figures/fig-c6i5.png" + ] +} \ No newline at end of file diff --git a/processed_dataset/proof/1018.json b/processed_dataset/proof/1018.json new file mode 100644 index 0000000000000000000000000000000000000000..eeaba758f48888ce7509bee9a6b082403e7a86e3 --- /dev/null +++ b/processed_dataset/proof/1018.json @@ -0,0 +1,10 @@ +{ + "source_file": "./raw_volume-zh/volume3/chapter6.tex", + "problem_type": "proof", + "problem": "例18. 如图(), 设 $O 、 H$ 分别为锐角 $\\triangle A B C$ 的外心和垂心, 则在 $B C 、 C A 、 A B$ 上分别存在点 $D 、 E 、 F$, 使得 $O D+D H=O E+E H=O F+F H$, 且直线 $A D 、 B E 、 C F$ 共点.", + "solution": "证明:设 $A H$ 的延长线交 $\\triangle A B C$ 的外接圆于 $L$, 交 $B C$ 于 $K$, 连 $O L$ 交 $B C$ 于 $D$, 连 $H D$. 由于 $H K=K L$, 得 $H D=L D$, 于是 $O D+D H=O D+ D L=O L=R$, 其中 $R$ 为 $\\triangle A B C$ 外接圆半径.\n类似地, 可以在 $C A$ 和 $A B$ 上得到点 $E$ 和 $F$, 使\n$$\nO E+E H=O F+F H=R .\n$$\n连 $O B 、 O C$ 和 $B L$, 则 $\\angle O B C=90^{\\circ}-\\angle A, \\angle C B L=\\angle C A L=90^{\\circ}- \\angle C$, 所以\n$$\n\\angle O B L=90^{\\circ}-\\angle A+90^{\\circ}-\\angle C=\\angle B,\n$$\n由于 $O B=O L$, 则 $\\angle O L B=\\angle B$, 于是 $\\angle B O L=180^{\\circ}-2 \\angle B$,\n$$\n\\angle C O D=\\angle B O C-\\angle B O D=2 \\angle A+\\left(180^{\\circ}-2 \\angle B\\right)=180^{\\circ}-2 \\angle C .\n$$\n由正弦定理得\n$$\n\\begin{aligned}\n& \\frac{B D}{\\sin \\angle B O D}=\\frac{O D}{\\sin \\angle O B D}, \\\\\n& \\frac{C D}{\\sin \\angle C O D}=\\frac{O D}{\\sin \\angle O C D},\n\\end{aligned}\n$$\n于是有\n$$\n\\frac{B D}{C D}=\\frac{\\sin \\left(180^{\\circ}-2 \\angle B\\right)}{\\sin \\left(180^{\\circ}-2 \\angle C\\right)}=\\frac{\\sin 2 B}{\\sin 2 C}\n$$\n同理 $\\frac{C E}{E A}=\\frac{\\sin 2 C}{\\sin 2 A}, \\frac{A F}{F B}=\\frac{\\sin 2 A}{\\sin 2 B}$, 所以\n$$\n\\frac{B D}{D C} \\cdot \\frac{C E}{E A} \\cdot \\frac{A F}{F B}=1,\n$$\n由 Ceva 定理的逆定理得 $A D 、 B E 、 C F$ 三线共点.", + "remark": "", + "figures": [ + "./images/volume3/figures/fig-c6i6.png" + ] +} \ No newline at end of file diff --git a/processed_dataset/proof/1019.json b/processed_dataset/proof/1019.json new file mode 100644 index 0000000000000000000000000000000000000000..120f28a440307eadcafc7a503c11c879ea8c6f7e --- /dev/null +++ b/processed_dataset/proof/1019.json @@ -0,0 +1,11 @@ +{ + "source_file": "./raw_volume-zh/volume3/chapter6.tex", + "problem_type": "proof", + "problem": "例19. 如图(), $P$ 是 $\\triangle A B C$ 内任意一点 (包括在边界上),过 $P$ 作 $B C 、 C A 、 A B$ 的垂线,垂足为 $D 、 E 、 F$. 则\n $\\triangle A B C$ 为正三角形 $\\Leftrightarrow$ 对任意的 $P$ 都有\n$$\nS_{\\triangle P B D}+S_{\\triangle P C E}+S_{\\triangle P A F}=\\frac{1}{2} S_{\\triangle A B C} ;\n$$\n $\\triangle A B C$ 为正三角形 $\\Leftrightarrow$ 对任意的 $P$ 都有\n$B D+C E+A F=\\frac{1}{2}(B C+C A+A B)$.", + "solution": "证明: (1)必要性.\n过 $P$ 作 $M N / / B C$ 交 $A B 、 A C$ 于 $M 、 N$, 过 $B$ 作 $B Q \\perp M N$ 于 $Q$, 过 $C$ 作 $C R \\perp M N$ 于 $R$, 记正 $\\triangle A M N$ 的边长为 1 , 并设 $P M=x$, 则 $P N=1-x$. 于是,\n$$\n\\begin{aligned}\nS_{\\triangle P A F}+S_{\\triangle P N E} & =\\frac{1}{2} \\cdot \\frac{\\sqrt{3}}{2} x\\left(1-\\frac{x}{2}\\right)+\\frac{1}{2} \\cdot \\frac{\\sqrt{3}}{2}(1-x) \\cdot \\frac{1-x}{2} \\\\\n& =\\frac{\\sqrt{3}}{8}=\\frac{1}{2} S_{\\triangle A M N} .\n\\end{aligned}\n$$\n又 $\\triangle B M Q \\cong \\triangle C N R$, 所以\n$$\n\\begin{aligned}\nS_{\\triangle B P D}+S_{\\triangle P C N} & =\\frac{1}{2}\\left(S_{\\text {矩形 } B D P Q}+S_{\\text {矩形 } P D C R}\\right)-S_{\\triangle C R N} \\\\\n& =S_{\\triangle P C D}+S_{\\triangle B M P} . \\\\\n\\text { 从而 } \\quad S_{\\triangle P B D}+ & S_{\\triangle P C E}+S_{\\triangle P A F}=\\frac{1}{2} S_{\\triangle A B C} .\n\\end{aligned}\n$$\n充分性.\n当 $P$ 为 $\\angle A$ 的平分线与 $B C$ 的交点时, 因为 $\\triangle A P E \\cong \\triangle A P F$, $S_{\\triangle A P E}=S_{\\triangle A P F}$, 所以 $S_{\\triangle P C E}=S_{\\triangle P B F}$.\n故\n$$\nS_{\\triangle P A B}=S_{\\triangle P A C} .\n$$\n从而 $P$ 为 $B C$ 中点.\n故 $A B=A C$.\n同理\n$$\nA B=B C \\text {. }\n$$\n所以 $\\triangle A B C$ 为正三角形.\n (2)必要性.\n由正弦定理有\n$$\n\\begin{aligned}\n& P B \\sin \\alpha=P C \\sin \\left(60^{\\circ}-\\beta\\right), \\\\\n& P C \\sin \\beta=P A \\sin \\left(60^{\\circ}-\\gamma\\right), \\\\\n& P A \\sin \\gamma=P B \\sin \\left(60^{\\circ}-\\alpha\\right) .\n\\end{aligned}\n$$\n将以上三式相加并移项得\n$$\n\\begin{aligned}\n& P B\\left[\\sin \\alpha-\\sin \\left(60^{\\circ}-\\alpha\\right)\\right]+P C\\left[\\sin \\beta-\\sin \\left(60^{\\circ}-\\beta\\right)\\right] \\\\\n& +P A\\left[\\sin \\gamma-\\sin \\left(60^{\\circ}-\\gamma\\right)\\right] \\\\\n= & 2 \\cos 30^{\\circ} \\cdot\\left[P B \\sin \\left(\\alpha-30^{\\circ}\\right)+P C \\sin \\left(\\beta-30^{\\circ}\\right)+P A \\sin \\left(\\gamma-30^{\\circ}\\right)\\right] \\\\\n= & 0 .\n\\end{aligned}\n$$\n所以 $P B \\sin \\left(\\alpha-30^{\\circ}\\right)+P C \\sin \\left(\\beta-30^{\\circ}\\right)+P A \\sin \\left(\\gamma-30^{\\circ}\\right)=0$.\n又\n$$\n\\begin{aligned}\n& B D+C E+A F-D C-E A-F B \\\\\n= & P B \\cos \\alpha+P C \\cos \\beta+P A \\cos \\gamma-P B \\cos \\left(60^{\\circ}-\\alpha\\right) \\\\\n& -P C \\cos \\left(60^{\\circ}-\\beta\\right)-P A \\cos \\left(60^{\\circ}-\\gamma\\right) \\\\\n= & -2 \\sin 30^{\\circ}\\left[P B \\sin \\left(\\alpha-30^{\\circ}\\right)+P C \\sin \\left(\\beta-30^{\\circ}\\right)+P A \\sin \\left(\\gamma-30^{\\circ}\\right)\\right] \\\\\n= & 0,\n\\end{aligned}\n$$\n即\n$$\n\\begin{aligned}\nB D+C E+A F & =D C+E A+F B \\\\\n& =\\frac{1}{2}(B C+C A+A B) .\n\\end{aligned}\n$$\n充分性.\n如图(), 当 $P 、 P^{\\prime}$ 为 $B C$ 的中垂线上两个不同点时,有 $E E^{\\prime}=F F^{\\prime}$. 作 $P K \\perp P^{\\prime} F^{\\prime}$ 于 $K$, $P L \\perp P^{\\prime} E^{\\prime}$ 于 $L$, 则 $P K=P L$,\n$$\n\\triangle P K P^{\\prime} \\cong \\triangle P L P^{\\prime}, \\angle K P P^{\\prime}=\\angle L P P^{\\prime},\n$$\n即直线 $P P^{\\prime}$ 与 $A B 、 A C$ 所成角相等,有\n$$\n\\angle B=\\angle C, A B=A C \\text {. }\n$$\n同理 $A B=B C$.\n故 $\\triangle A B C$ 为正三角形.", + "remark": "", + "figures": [ + "./images/volume3/figures/fig-c6i7.png", + "./images/volume3/figures/fig-c6i8.png" + ] +} \ No newline at end of file diff --git a/processed_dataset/proof/1020.json b/processed_dataset/proof/1020.json new file mode 100644 index 0000000000000000000000000000000000000000..16491fbd18a7d3ec28e100b124aa14cc780e9cc1 --- /dev/null +++ b/processed_dataset/proof/1020.json @@ -0,0 +1,16 @@ +{ + "source_file": "./raw_volume-zh/volume3/chapter6.tex", + "problem_type": "proof", + "problem": "例20. 如图(), 在锐角 $\\triangle A B C$ 的 $B C$ 边上有两点 $E 、 F$, 满足 $\\angle B A E=\\angle C A F$, 作 $F M \\perp A B, F N \\perp A C$ ( $M 、 N$ 是垂足), 延长 $A E$ 交 $\\triangle A B C$ 的外接圆于点 $D$.\n证明: 四边形 $A M D N$ 与 $\\triangle A B C$ 的面积相等.", + "solution": "(注: 此题当 $A D$ 与 $A F$ 重合时, 即为第 28 届 IMO 第二题.\n)\n证法一如图(), 连 $B D$, 则 $\\triangle A B D \\backsim \\triangle A F C$, 所以 $A F \\cdot A D=A B \\cdot A C$.\n设 $\\angle B A E=\\angle C A F=\\alpha, \\angle E A F=\\beta$, 则\n$$\n\\begin{aligned}\n& S_{\\text {四边形 } A M D N}=\\frac{1}{2} A M \\cdot A D \\sin \\alpha+\\frac{1}{2} A D \\cdot A N \\sin (\\alpha+\\beta) \\\\\n&=\\frac{1}{2} A D[A F \\cos (\\alpha+\\beta) \\sin \\alpha+A F \\cos \\alpha \\sin (\\alpha+\\beta)] \\\\\n&=\\frac{1}{2} A D \\cdot A F \\sin (2 \\alpha+\\beta) \\\\\n&=\\frac{1}{2} A B \\cdot A C \\sin \\angle B A C \\\\\n&=S_{\\triangle A B C} . \\\\\n& \\text { 二如图 } 6-10, \\text { 作 } \\triangle A B C \\text { 外接圆的直径 } \\\\\n&, B G / / M F, C G / / N F, D G / / M N . \\text { 从而, } \\\\\n& S_{\\triangle G F M}, S_{\\triangle C F N}=S_{\\triangle G F N}, S_{\\triangle M N D}=S_{\\triangle M N G} . \\text { 所 }\n\\end{aligned}\n$$\n证法二如图(),作 $\\triangle A B C$ 外接圆的直径 $A G$. 易知, $B G / / M F, C G / / N F, D G / / M N$. 从而, $S_{\\triangle B F M}=S_{\\triangle G F M}, S_{\\triangle C F N}=S_{\\triangle G F N}, S_{\\triangle M N D}=S_{\\triangle M N G}$. 所以有\n$$\n\\begin{aligned}\nS_{\\text {四边形 } A M D N} & =S_{\\text {四边形 } A M G N} \\\\\n& =S_{\\text {四边形 } A M F N}+S_{\\triangle G F M}+S_{\\triangle G F N} \\\\\n& =S_{\\text {四边形 } A M F N}+S_{\\triangle B F M}+S_{\\triangle C F N} \\\\\n& =S_{\\triangle A B C .}\n\\end{aligned}\n$$\n证法三如图(), 作 $A H \\perp B C, H$ 为垂足, 则 $A 、 M 、 H 、 F 、 N$ 共圆.\n从而有 $\\angle M H B=\\angle B A F=\\angle C A D=\\angle C B D$, 所以 $M H / / B D$.\n同理 $N H / / C D$. 于是, 有\n$$\nS_{\\triangle B M H}=S_{\\triangle D M H}, S_{\\triangle C N H}=S_{\\triangle D N H} .\n$$\n故\n$S_{\\text {四边形 } A M D N}=S_{\\triangle A B C \\text {. }}$\n证法四如图(), 只要证明\n$$\nS_{\\triangle B M D}+S_{\\triangle C N D}=S_{\\triangle B C D} .\n$$\n设 $\\angle B A E=\\angle C A F=\\alpha$, 则\n$$\n\\begin{gathered}\nA F=\\frac{A B \\sin B}{\\sin (C+\\alpha)}=\\frac{2 R \\sin C \\cdot \\sin B}{\\sin (C+\\alpha)}, \\\\\nA M=A F \\cos (A-\\alpha) .\n\\end{gathered}\n$$\n其中 $R$ 为 $\\triangle A B C$ 外接圆半径.\n所以\n$$\nB M=A B-A M=2 R \\sin C \\cdot \\frac{\\cos B \\cdot \\sin (A-\\alpha)}{\\sin (C+\\alpha)} .\n$$\n故\n$$\nS_{\\triangle B M D}=\\frac{1}{2} B M \\cdot B D \\sin \\angle A B D=2 R^2 \\sin \\alpha \\cdot \\sin (A-\\alpha) \\cos B \\cdot \\sin C .\n$$\n同理\n$$\nS_{\\triangle C N D}=2 R^2 \\sin \\alpha \\cdot \\sin (A-\\alpha) \\cdot \\cos C \\cdot \\sin B .\n$$\n从而\n$$\n\\begin{aligned}\n& S_{\\triangle B M D}+S_{\\triangle C N D} \\\\\n= & 2 R^2 \\sin \\alpha \\cdot \\sin (A-\\alpha) \\cdot \\sin (B+C) \\\\\n= & \\frac{1}{2} \\cdot 2 R \\sin \\alpha \\cdot 2 R \\sin (A-\\alpha) \\cdot \\sin A \\\\\n= & \\frac{1}{2} B D \\cdot D C \\sin \\angle B D C=S_{\\triangle B C D} .\n\\end{aligned}\n$$\n证法五如图(), 作 $D K \\perp A B, D L \\perp A C$, 垂足分别为 $K 、 L$, 则只要证明 $S_{\\triangle F B M}+S_{\\triangle F C N}=S_{\\triangle F D M}+S_{\\triangle F D N}$.\n利用 $S_{\\triangle F D M}=S_{\\triangle F K N}, S_{\\triangle F D N}=S_{\\triangle F L N}$, 只需证明\n$$\nS_{\\triangle F B M}+S_{\\triangle F C N}=S_{\\triangle F K M}+S_{\\triangle F L N},\n$$\n即\n$$\nF M \\cdot B M+F N \\cdot C N=F M \\cdot M K+F N \\cdot N L .\n$$\n因此, 只需证明 $F M \\cdot B K=F N \\cdot C L$.\n由于 $\\triangle B K D \\backsim \\triangle C L D$, 所以\n$$\n\\frac{B K}{C L}=\\frac{D K}{D L}=\\frac{\\sin \\alpha}{\\sin (A-\\alpha)}=\\frac{F N}{F M} .\n$$\n故结论成立.\n其中 $\\alpha=\\angle B A E=\\angle C A F$.\n证法六如图(), 作 $D G / / M N$, 交 $A C$ 的延长线于 $G$, 只要证明\n$$\nS_{\\triangle A M G}=S_{\\triangle A B C} .\n$$\n由于 $\\angle A G D=\\angle A N M=\\angle A F M$, 所以 $\\triangle A G D \\backsim \\triangle A F M$. 从而 $A D$ • $A F=A M \\cdot A G$. 又用于 $\\triangle A B D \\backsim \\triangle A F C$, 有 $A D \\cdot A F=A B \\cdot A C$.\n故 $A M \\cdot A G=A B \\cdot A C$, 即\n$$\nS_{\\triangle A M G}=S_{\\triangle A B C} .\n$$\n证法七设 $\\angle B A E=\\angle C A F=\\alpha, \\angle E A F=\\beta$, 则由证法一知\n$$\nS_{\\text {四边形 } A M D N}=\\frac{1}{2} A D \\cdot A F \\sin (2 \\alpha+\\beta)=\\frac{A F}{4 R} \\cdot A D \\cdot B C \\text {. }\n$$\n又\n$$\n\\begin{aligned}\nS_{\\triangle A B C} & =\\frac{1}{2} A B \\cdot A F \\sin (\\alpha+\\beta)+\\frac{1}{2} A C \\cdot A F \\sin \\alpha \\\\\n& =\\frac{A F}{4 R}(A B \\cdot C D+A C \\cdot B D) .\n\\end{aligned}\n$$\n由托勒密定理知\n$$\nA B \\cdot C D+A C \\cdot B D=A D \\cdot B C,\n$$\n故结论成立.", + "remark": "", + "figures": [ + "./images/volume3/figures/fig-c6i9.png", + "./images/volume3/figures/fig-c6i9.png", + "./images/volume3/figures/fig-c6i10.png", + "./images/volume3/figures/fig-c6i11.png", + "./images/volume3/figures/fig-c6i12.png", + "./images/volume3/figures/fig-c6i12.png", + "./images/volume3/figures/fig-c6i13.png" + ] +} \ No newline at end of file diff --git a/processed_dataset/proof/1021.json b/processed_dataset/proof/1021.json new file mode 100644 index 0000000000000000000000000000000000000000..92338fca6484391ddf7efecea1422bc7b7bca3c9 --- /dev/null +++ b/processed_dataset/proof/1021.json @@ -0,0 +1,8 @@ +{ + "source_file": "./raw_volume-zh/volume3/exercise1.tex", + "problem_type": "proof", + "problem": "问题19 有四个函数: (1) $y=\\sin ^2 x$; (2) $y=\\sin |x|$; (3) $y=\\tan \\frac{x}{2}-\\cot \\frac{x}{2}$; (4) $y= |\\sin x|$. 其中周期为 $\\pi$, 且在 $\\left(0, \\frac{\\pi}{2}\\right)$ 上是增函数的为", + "solution": "(1)(3). 因 $y=\\sin ^2 x=\\frac{1}{2}(1-\\cos 2 x)$, 周期为 $\\pi ; y=\\tan \\frac{x}{2}-\\cot \\frac{x}{2}= \\frac{1-\\cos x}{\\sin x}-\\frac{1+\\cos x}{\\sin x}=-\\frac{2 \\cos x}{\\sin x}=-2 \\cot x$. 其定义域为 $\\{x \\mid x \\in \\mathbf{R}$, 且 $x \\neq k \\pi, k \\in \\mathbf{Z}\\}$, 周期为 $\\pi$; 可以验证 (1)(3)(4) 符合题意.", + "remark": "", + "figures": [] +} \ No newline at end of file diff --git a/processed_dataset/proof/1022.json b/processed_dataset/proof/1022.json new file mode 100644 index 0000000000000000000000000000000000000000..845b775fdfe8877a29c198f50475fc0c4bdef823 --- /dev/null +++ b/processed_dataset/proof/1022.json @@ -0,0 +1,8 @@ +{ + "source_file": "./raw_volume-zh/volume3/exercise1.tex", + "problem_type": "proof", + "problem": "问题25 正实数 $\\alpha 、 \\beta 、 a 、 b$ 满足条件 $\\alpha<\\beta, \\alpha+\\beta<\\pi, a+b<\\pi$ 并且 $\\frac{\\sin a}{\\sin b} \\leqslant \\frac{\\sin \\alpha}{\\sin \\beta}$, 求证: $a0, y>0, x+y<\\pi$ 的条件下, $\\sin x<\\sin y$ 的充要条件是 $x\\sin (\\pi-x)=\\sin x$. 这样即得到当 $x0$, 而 $0<\\frac{y-x}{2}<\\pi$ 得到 $y>x$. 利用上述结论由 $0<\\alpha<\\beta, \\alpha+\\beta<\\pi$ 可知 $\\sin \\alpha<\\sin \\beta$, 所以 $\\frac{\\sin a}{\\sin b} \\leqslant \\frac{\\sin \\alpha}{\\sin \\beta}<1$, 即 $\\sin a<\\sin b$, 再用一次上述结论可得 $af\\left(\\frac{x_1+x_2}{2}\\right)$.", + "solution": "要证明 $\\frac{1}{2}\\left[f\\left(x_1\\right)+f\\left(x_2\\right)\\right]>f\\left(\\frac{x_1+x_2}{2}\\right) \\Leftrightarrow \\frac{1}{2}\\left[\\tan x_1+\\tan x_2\\right]> \\tan \\frac{x_1+x_2}{2} \\Leftrightarrow \\tan \\frac{x_1+x_2}{2}-\\tan x_1<\\tan x_2-\\tan \\frac{x_1+x_2}{2} \\Leftrightarrow \\tan \\frac{x_2-x_1}{2}\\left(1+\\tan \\frac{x_1+x_2}{2} \\cdot \\tan x_1\\right)<\\tan \\frac{x_2-x_1}{2} \\cdot\\left(1+\\tan \\frac{x_1+x_2}{2} \\cdot \\tan x_2\\right)$, 不妨设 $x_1b, \\sin \\theta=\\frac{2 a b}{a^2+b^2}$, 其中 $\\theta \\in\\left(0, \\frac{\\pi}{2}\\right), A_n= \\left(a^2+b^2\\right)^n \\sin n \\theta$. 求证: 对于一切正整数 $n, A_n$ 均为整数.", + "solution": "设 $B_n=\\left(a^2+b^2\\right)^n \\cos n \\theta$. 由 $\\sin \\theta=\\frac{2 a b}{a^2+b^2}$, 得 $\\cos \\theta=\\frac{a^2-b^2}{a^2+b^2}(a> b>0)$, 所以 $\\left(a^2+b^2\\right) \\sin \\theta=2 a b,\\left(a^2+b^2\\right) \\cos \\theta=a^2-b^2$ 均为整数, 即 $A_1$ 、 $B_1$ 均为整数.\n设 $A_k 、 B_k$ 均为整数, 则 $A_{k+1}=\\left(a^2+b^2\\right)^{k+1} \\sin (k+1) \\theta=\\left(a^2+\\right. \\left.b^2\\right)^{k+1} \\sin k \\theta \\cos \\theta+\\left(a^2+b^2\\right)^{k+1} \\cos k \\theta \\sin \\theta=A_k \\cdot\\left(a^2-b^2\\right)+B_k \\cdot 2 a b, B_{k+1}= \\left(a^2+b^2\\right)^{k+1} \\cos (k+1) \\theta=\\left(a^2+b^2\\right)^{k+1} \\cos k \\theta \\cos \\theta-\\left(a^2+b^2\\right)^{k+1} \\sin k \\theta \\cos \\theta= B_k \\cdot\\left(a^2-b^2\\right)-A_k \\cdot 2 a b$. 由 $A_k 、 B_k 、 a 、 b$ 均为整数, 得 $A_{k+1} 、 B_{k+1}$ 均为整数.\n由归纳原理可知, 对于一切正整数 $n, A_n$ 为整数.", + "remark": "", + "figures": [] +} \ No newline at end of file diff --git a/processed_dataset/proof/1026.json b/processed_dataset/proof/1026.json new file mode 100644 index 0000000000000000000000000000000000000000..d78a2b0885a156daf1d86306e23a1f7431a8de72 --- /dev/null +++ b/processed_dataset/proof/1026.json @@ -0,0 +1,8 @@ +{ + "source_file": "./raw_volume-zh/volume3/exercise2.tex", + "problem_type": "proof", + "problem": "问题25 求证: $\\sum_{k=0}^n\\left(\\frac{1}{3}\\right)^k \\sin ^3\\left(3^k \\alpha\\right)=\\frac{3}{4} \\sin \\alpha-\\frac{1}{4 \\cdot 3^n} \\sin 3^{n+1} \\alpha$.", + "solution": "令 $a_n=\\sum_{k=0}^n\\left(\\frac{1}{3}\\right)^k \\sin ^3\\left(3^k \\alpha\\right)+\\frac{1}{4 \\cdot 3^n} \\sin 3^{n+1} \\alpha-\\frac{3}{4} \\sin \\alpha$, 容易验证 $a_{n+1}-a_n=0$, 所以成立.", + "remark": "", + "figures": [] +} \ No newline at end of file diff --git a/processed_dataset/proof/1027.json b/processed_dataset/proof/1027.json new file mode 100644 index 0000000000000000000000000000000000000000..473860935b4f20dcfb844c2863acdaa425953485 --- /dev/null +++ b/processed_dataset/proof/1027.json @@ -0,0 +1,8 @@ +{ + "source_file": "./raw_volume-zh/volume3/exercise2.tex", + "problem_type": "proof", + "problem": "问题27 已知 $\\frac{\\sin ^2 \\gamma}{\\sin ^2 \\alpha}=1-\\frac{\\tan (\\alpha-\\beta)}{\\tan \\alpha}$, 求证: $\\tan ^2 \\gamma=\\tan \\alpha \\cdot \\tan \\beta$.", + "solution": "$\\sin ^2 \\gamma=\\sin ^2 \\alpha \\cdot\\left[1-\\frac{\\tan (\\alpha-\\beta)}{\\tan \\alpha}\\right]=\\sin ^2 \\alpha \\frac{\\sin \\alpha \\cos (\\alpha-\\beta)-\\cos \\alpha \\sin (\\alpha-\\beta)}{\\sin \\alpha \\cos (\\alpha-\\beta)}= \\frac{\\sin \\alpha \\cdot \\sin \\beta}{\\cos (\\alpha-\\beta)}$, 所以 $\\tan ^2 \\gamma=\\frac{\\sin ^2 \\gamma}{1-\\sin ^2 \\gamma}=\\frac{\\frac{\\sin \\alpha \\sin \\beta}{\\cos (\\alpha-\\beta)}}{1-\\frac{\\sin \\alpha \\sin \\beta}{\\cos (\\alpha-\\beta)}}=\\frac{\\sin \\alpha \\sin \\beta}{\\cos (\\alpha-\\beta)-\\sin \\alpha \\sin \\beta}= \\frac{\\sin \\alpha \\sin \\beta}{\\cos \\alpha \\cos \\beta}=\\tan \\alpha \\cdot \\tan \\beta$.", + "remark": "", + "figures": [] +} \ No newline at end of file diff --git a/processed_dataset/proof/1028.json b/processed_dataset/proof/1028.json new file mode 100644 index 0000000000000000000000000000000000000000..fb73537d89fadf73b2457605add19beff6dc9a22 --- /dev/null +++ b/processed_dataset/proof/1028.json @@ -0,0 +1,8 @@ +{ + "source_file": "./raw_volume-zh/volume3/exercise3.tex", + "problem_type": "proof", + "problem": "问题21 锐角 $\\triangle A B C$ 中, 若 $\\cos ^2 A 、 \\cos ^2 B 、 \\cos ^2 C$ 的和等于 $\\sin ^2 A 、 \\sin ^2 B 、 \\sin ^2 C$ 中的某个值.\n证明: $\\tan A 、 \\tan B 、 \\tan C$ 必可按某顺序组成一个等差数列.", + "solution": "设 $\\cos ^2 A+\\cos ^2 B+\\cos ^2 C=\\sin ^2 B \\cdots$ (1). 据余弦定理: $\\sin ^2 B= \\sin ^2 A+\\sin ^2 C-2 \\sin A \\sin C \\cos B \\cdots(2) \\cdot \\sin A \\sin C=\\cos A \\cos C-\\cos (A+ C)=\\cos A \\cos C+\\cos B \\cdots$ (3). 由 (2)、(3) 得, $\\cos ^2 A+\\cos ^2 B+\\cos ^2 C=1- 2 \\cos A \\cos B \\cos C \\cdots$ (4). 由 (1)、(4) 得, $2 \\cos A \\cos B \\cos C=1-\\sin ^2 B=\\cos ^2 B$, $B$ 为锐角.\n则 $2 \\cos A \\cos C=\\cos B=-\\cos (A+C)=\\sin A \\sin C-\\cos A \\cos C$,\n所以 $\\tan A \\tan C=3$, 所以 $\\tan B=-\\tan (A+C)=\\frac{\\tan A+\\tan C}{\\tan A \\tan C-1}= \\frac{\\tan A+\\tan C}{2}$, 因此 $\\tan A 、 \\tan B 、 \\tan C$ 成等差数列.", + "remark": "", + "figures": [] +} \ No newline at end of file diff --git a/processed_dataset/proof/1029.json b/processed_dataset/proof/1029.json new file mode 100644 index 0000000000000000000000000000000000000000..44110f6167413e580dd7ca2696c157a3ddda14df --- /dev/null +++ b/processed_dataset/proof/1029.json @@ -0,0 +1,8 @@ +{ + "source_file": "./raw_volume-zh/volume3/exercise3.tex", + "problem_type": "proof", + "problem": "问题22 在锐角 $\\triangle A B C$ 中, 求证: $\\cos (B-C) \\cos (C-A) \\cos (A-B) \\geqslant 8 \\cos A \\cos B \\cos C$.", + "solution": "因为 $\\tan A \\tan B \\tan C=\\tan A+\\tan B+\\tan C$, 故\n$$\n\\begin{aligned}\n\\frac{\\cos (B-C)}{\\cos A} & =\\frac{\\cos (B-C)}{-\\cos (B+C)}=\\frac{\\sin B \\sin C+\\cos B \\cos C}{\\sin B \\sin C-\\cos B \\cos C}=\\frac{\\tan B \\tan C+1}{\\tan B \\tan C-1} \\\\\n& =\\frac{\\tan A \\tan B \\tan C+\\tan A}{\\tan A \\tan B \\tan C-\\tan A}=\\frac{2 \\tan A+\\tan B+\\tan C}{\\tan B+\\tan C} \\\\\n& \\geqslant \\frac{2 \\sqrt{(\\tan A+\\tan B)(\\tan C+\\tan A)}}{\\tan B+\\tan C},\n\\end{aligned}\n$$\n同理 $\\frac{\\cos (C-A)}{\\cos B} \\geqslant \\frac{2 \\sqrt{(\\tan B+\\tan C)(\\tan A+\\tan B)}}{\\tan C+\\tan A}$, $\\frac{\\cos (A-B)}{\\cos C} \\geqslant \\frac{2 \\sqrt{(\\tan C+\\tan A)(\\tan B+\\tan C)}}{\\tan A+\\tan B}$, 以上三式相乘, 即得 $\\frac{\\cos (B-C) \\cos (C-A) \\cos (A-B)}{\\cdot \\cos A \\cos B \\cos C} \\geqslant 8$, 当且仅当 $A=B=C=\\frac{\\pi}{3}$ 时等号成立, 原题得证.", + "remark": "", + "figures": [] +} \ No newline at end of file diff --git a/processed_dataset/proof/1030.json b/processed_dataset/proof/1030.json new file mode 100644 index 0000000000000000000000000000000000000000..0fe3224cc4bac9e01ff4aa9172730825eed07533 --- /dev/null +++ b/processed_dataset/proof/1030.json @@ -0,0 +1,8 @@ +{ + "source_file": "./raw_volume-zh/volume3/exercise3.tex", + "problem_type": "proof", + "problem": "问题23 $ \\triangle \\triangle A B C$ 的三条边长为 $a 、 b 、 c$, 证明:\n$$\n\\frac{\\left|a^2-b^2\\right|}{c}+\\frac{\\left|b^2-c^2\\right|}{a} \\geqslant \\frac{\\left|c^2-a^2\\right|}{b} .\n$$", + "solution": "由于 $a=2 R \\sin A, b=2 R \\sin B, c=2 R \\sin C$, 只要证 $\\frac{\\left|\\sin ^2 A-\\sin ^2 B\\right|}{\\sin C}+\\frac{\\left|\\sin ^2 B-\\sin ^2 C\\right|}{\\sin A} \\geqslant \\frac{\\left|\\sin ^2 C-\\sin ^2 A\\right|}{\\sin B}$... (1). 因为 $\\sin ^2 A- \\sin ^2 B=\\sin (A+B) \\sin (A-B)=\\sin C \\sin (A-B)$, 故由 (1), 只要证 $\\mid \\sin (A-$ B) $|+| \\sin (B-C)|\\geqslant| \\sin (C-A) \\mid \\cdots$ (2). $|\\sin (C-A)|=\\mid \\sin [(A-B)+ (B-C)]|=| \\sin (A-B) \\cos (B-C)+\\cos (A-B) \\sin (B-C) \\mid \\leqslant |\\sin (A-B) \\cos (B-C)|+|\\cos (A-B) \\sin (B-C)| \\leqslant|\\sin (A-B)|+ |\\sin (B-C)|$, 当且仅当 $A=B=C=\\frac{\\pi}{3}$ 时取等号, 此时 $\\triangle A B C$ 为正三角形, 即 $a=b=c$.", + "remark": "", + "figures": [] +} \ No newline at end of file diff --git a/processed_dataset/proof/1031.json b/processed_dataset/proof/1031.json new file mode 100644 index 0000000000000000000000000000000000000000..0ff7eed015d81e92bd0902f0a65597901f013071 --- /dev/null +++ b/processed_dataset/proof/1031.json @@ -0,0 +1,8 @@ +{ + "source_file": "./raw_volume-zh/volume3/exercise3.tex", + "problem_type": "proof", + "problem": "问题24 知 $\\triangle A B C$ 的外接圆、内切圆半径分别为 $R$ 和 $r$, 求证:\n$$\n\\frac{\\cos A}{\\sin ^2 A}+\\frac{\\cos B}{\\sin ^2 B}+\\frac{\\cos C}{\\sin ^2 C} \\geqslant \\frac{R}{r} \\text {. }\n$$", + "solution": "设 $a 、 b 、 c$ 为 $\\triangle A B C$ 的三边长, 则 $\\frac{b^2+c^2}{a}+2 a \\geqslant \\frac{(b+c)^2}{2 a}+2 a \\geqslant 2(b+c)$, 故 $\\frac{b^2+c^2}{a} \\geqslant 2(b+c-a)$, 同理可得 $\\frac{c^2+a^2}{b} \\geqslant 2(a+c-b)$, $\\frac{a^2+b^2}{c} \\geqslant 2(a+b-c)$, 以上三式相加得 $\\frac{b^2+c^2}{a}+\\frac{c^2+a^2}{b}+\\frac{a^2+b^2}{c} \\geqslant 2(a+ b+c)$. 根据正、余弦定理及 $a b c=2 R r(a+b+c)$, 并利用上式可得 $\\frac{\\cos A}{\\sin ^2 A}+ \\frac{\\cos B}{\\sin ^2 B}+\\frac{\\cos C}{\\sin ^2 C}=\\frac{2 R^2}{a b c}\\left(\\frac{a^2+b^2}{c}+\\frac{b^2+c^2}{a}+\\frac{c^2+a^2}{b}-a-b-c\\right) \\geqslant \\frac{2 R^2(a+b+c)}{a b c}=\\frac{R}{r}$, 当且仅当 $a=b=c$ 时等号成立.", + "remark": "", + "figures": [] +} \ No newline at end of file diff --git a/processed_dataset/proof/1032.json b/processed_dataset/proof/1032.json new file mode 100644 index 0000000000000000000000000000000000000000..de2123b3fc4803d94ec27cdf562cd3c3609629c8 --- /dev/null +++ b/processed_dataset/proof/1032.json @@ -0,0 +1,8 @@ +{ + "source_file": "./raw_volume-zh/volume3/exercise3.tex", + "problem_type": "proof", + "problem": "问题25 在 $\\triangle A B C$ 中, 实数 $x$ 满足 $\\sec ^2 x=\\csc ^2 A+\\csc ^2 B+\\csc ^2 C$, 求证: $\\cos [x+ \\left.(-1)^n A\\right] \\cdot \\cos \\left[x+(-1)^n B\\right] \\cdot \\cos \\left[x+(-1)^n C\\right]+\\cos ^3 x=0(n \\in \\mathbf{N})$.", + "solution": "因为 $\\cot A \\cot B+\\cot B \\cot C+\\cot C \\cdot \\cot A=1$, 因为 $\\sec ^2 x= \\csc ^2 A+\\csc ^2 B+\\csc ^2 C$, 所以 $\\tan ^2 x=\\cot ^2 A+\\cot ^2 B+\\cot ^2 C+2=\\cot ^2 A+ \\cot ^2 B+\\cot ^2 C+2(\\cot A \\cot B+\\cot B \\cot C+\\cot C \\cot A)=(\\cot A+\\cot B+ \\cot C)^2$, 所以 $\\tan x= \\pm(\\cot A+\\cot B+\\cot C)$. 当 $\\tan x==\\cot A+\\cot B+ \\cot C$ 时, $(\\tan x-\\cot A)(\\tan x-\\cot B)(\\tan x-\\cot C)=(\\cot B+\\cot C) (\\cot A+\\cot C)(\\cot B+\\cot A)=\\frac{\\sin (B+C)}{\\sin B \\sin C} \\cdot \\frac{\\sin (A+C)}{\\sin A \\sin C} \\cdot \\frac{\\sin (A+B)}{\\sin A \\sin B}= \\frac{1}{\\sin A \\sin B \\sin C}$, 两边乘以 $\\cos ^3 x \\sin A \\sin B \\sin C$ 得, $(\\sin x \\sin A-\\cos A \\cos x) (\\sin x \\sin B-\\cos B \\cos x)(\\sin x \\sin C-\\cos x \\cos C)=\\cos ^3 x \\Rightarrow \\cos (x+A) \\cos (x+B) \\cos (x+C)+\\cos ^3 x=0$, 当 $\\tan x=-(\\cot A+\\cot B+\\cot C)$ 时, $(\\tan x+\\cot A)(\\tan x+\\cot B)(\\tan x+\\cot C)=-(\\cot B+\\cot C)(\\cot C+ \\cot A)(\\cot A+\\cot B)=-\\frac{\\sin (B+C)}{\\sin B \\sin C} \\cdot \\frac{\\sin (C+A)}{\\sin C \\sin A} \\cdot \\frac{\\sin (A+B)}{\\sin A \\sin B}= -\\frac{1}{\\sin A \\sin B \\sin C}$, 两边乘以 $\\cos ^3 x \\sin A \\sin B \\sin C$ 得 $(\\sin x \\sin A+\\cos A \\cos x)(\\sin x \\sin B+\\cos x \\cos B)(\\sin x \\sin C+\\cos x \\cos C)=-\\cos ^3 x \\Rightarrow \\cos (x- A) \\cos (x-B) \\cos (x-C)+\\cos ^3 x=0$, 综上得证.", + "remark": "", + "figures": [] +} \ No newline at end of file diff --git a/processed_dataset/proof/1033.json b/processed_dataset/proof/1033.json new file mode 100644 index 0000000000000000000000000000000000000000..eaf41d4cf7abc47f2ccca2b191bef122aa73168f --- /dev/null +++ b/processed_dataset/proof/1033.json @@ -0,0 +1,8 @@ +{ + "source_file": "./raw_volume-zh/volume3/exercise3.tex", + "problem_type": "proof", + "problem": "问题26 设 $\\alpha 、 \\beta 、 \\gamma 、 \\varphi$ 为某四边形的四个内角, 设 $n(n \\geqslant 3)$ 为奇数, 若 $\\alpha 、 \\beta 、 \\gamma 、 \\varphi$ 满足 $\\sin \\alpha+\\sin \\beta+\\sin \\gamma+\\sin \\varphi=0$. 求证: 它们之中必有两个角之和在集合 $\\left\\{\\frac{2 \\pi}{n}, \\frac{4 \\pi}{n}, \\cdots, \\frac{(n-1) \\pi}{n}\\right\\}$ 中.", + "solution": "因为 $\\alpha+\\beta+\\gamma+\\varphi=2 \\pi$, 从而有 $\\frac{n \\alpha+n \\beta}{2}+\\frac{n \\gamma+n \\varphi}{2}=n \\pi$, 又 $n$ 为奇数, 所以 $\\sin \\frac{n \\alpha+n \\beta}{2}=\\sin \\frac{n \\gamma+n \\varphi}{2}, \\sin n \\alpha+\\sin n \\beta+\\sin n \\gamma+\\sin n \\varphi=0 \\Leftrightarrow 2 \\sin \\frac{n \\alpha+n \\beta}{2} \\cos \\frac{n \\alpha-n \\beta}{2}+2 \\sin \\frac{n \\gamma+n \\varphi}{2} \\cos \\frac{n \\gamma-n \\varphi}{2}=0 \\Leftrightarrow 2 \\sin \\frac{n \\alpha+n \\beta}{2} \\cos \\frac{n \\alpha-n \\beta}{2}+ 2 \\sin \\frac{n \\alpha+n \\beta}{2} \\cos \\frac{m \\gamma-n \\varphi}{2}=0 \\Leftrightarrow 2 \\sin \\frac{n \\alpha+n \\beta}{2}=0$ 或 $\\cos \\frac{n \\alpha-n \\beta}{2}+ \\cos \\frac{n \\gamma-n \\varphi}{2}=0$. (1) 若 $\\sin \\frac{n \\alpha+n \\beta}{2}=0$, 用例 5(1) 即 $\\alpha+\\beta=\\frac{2 \\pi}{n}, \\frac{4 \\pi}{n}, \\cdots$, $\\frac{2(n-1)}{n} \\pi$. (2) 若 $\\cos \\frac{n \\alpha-n \\beta}{2}+\\cos \\frac{n \\gamma-n \\varphi}{2}=0$, 则 $\\frac{n \\alpha-n \\beta}{2}+\\frac{n \\gamma-n \\varphi}{2}= 2 k \\pi+\\pi$ 或 $\\frac{n \\alpha-n \\beta}{2}=\\frac{n \\gamma-n \\varphi}{2}+\\pi+2 k \\pi(k \\in \\mathbf{Z})$, 由于 $\\alpha+\\beta+\\gamma+\\varphi=2 \\pi$, 故 $\\alpha+\\gamma=\\pi+\\frac{(2 k+1)}{n} \\pi$ 或 $\\alpha+\\varphi=\\pi+\\frac{(2 k+1)}{n} \\pi(k \\in \\mathbf{Z})$, 当 $\\alpha+\\gamma=\\pi+ \\frac{(2 k+1)}{n} \\pi(k \\in \\mathbf{Z})$ 时, 由 $0<\\alpha+\\gamma<2 \\pi$, 则有 $\\alpha+\\gamma=\\frac{2 \\pi}{n}, \\frac{4 \\pi}{n}, \\cdots$, $\\frac{2(n-1)}{n} \\pi$. 类似地, $\\alpha+\\varphi=\\frac{2 \\pi}{n}, \\frac{4 \\pi}{n}, \\cdots, \\frac{2(n-1)}{n} \\pi$, 综合 (1)、 (2), $\\alpha, \\beta 、 \\gamma 、 \\varphi$ 至少存在两个角之和在集合 $\\left\\{\\frac{2 \\pi}{n}, \\frac{4 \\pi}{n}, \\cdots, \\frac{2(n-1) \\pi}{n}\\right\\}$ 中.\n注意到 $n$ 为奇数, $\\pi$ 不在上面的集合中, 又因为 $\\frac{2 i \\pi}{n}+\\frac{2(n-i) \\pi}{n}=2 \\pi$, 结合四边形的内角之和为 $2 \\pi$. 因此 $\\alpha 、 \\beta 、 \\gamma 、 \\varphi$ 至少存在两个角之和在集合 $\\left\\{\\frac{2 \\pi}{n}, \\frac{4 \\pi}{n}, \\cdots, \\frac{2(n-1) \\pi}{n}\\right\\}$ 中, 得证.\n特别地, 当 $n=3$ 时,必有两角之和为 $\\frac{2 \\pi}{3}$.", + "remark": "", + "figures": [] +} \ No newline at end of file diff --git a/processed_dataset/proof/1034.json b/processed_dataset/proof/1034.json new file mode 100644 index 0000000000000000000000000000000000000000..6d95ab6f57c0827f5a5b8552b36b2ca28e08d531 --- /dev/null +++ b/processed_dataset/proof/1034.json @@ -0,0 +1,8 @@ +{ + "source_file": "./raw_volume-zh/volume3/exercise3.tex", + "problem_type": "proof", + "problem": "问题27 在 $\\triangle A B C$ 中, 若 $\\sin A \\cos ^2 \\frac{C}{2}+\\sin C \\cos ^2 \\frac{A}{2}=\\frac{3}{2} \\sin B$, 求证: (1) $a 、 b 、 c$ 成等差数列; $(2) 0<\\sin \\frac{B}{2} \\leqslant \\frac{1}{2}$.", + "solution": "(1) $\\sin A \\cdot(\\cos C+1)+\\sin C \\cdot(\\cos A+1)=3 \\sin B$. 即 $\\sin (A+ C)+\\sin A+\\sin C=3 \\sin B$. 所以 $\\sin A+\\sin C=2 \\sin B$, 即 $a+c=2 b$, 从而 $a 、 b 、 c$ 成等差数列.\n(2) 由 $\\sin A+\\sin C=2 \\sin B$ 得 $2 \\sin \\frac{A+C}{2} \\cos \\frac{A-C}{2}= 4 \\sin \\frac{B}{2} \\cos \\frac{B}{2}$, 所以 $\\sin \\frac{B}{2}=\\frac{1}{2} \\cos \\frac{A-C}{2} \\leqslant \\frac{1}{2}$, 显然 $\\sin \\frac{B}{2}>0$. 故 $0< \\sin \\frac{B}{2} \\leqslant \\frac{1}{2}$.", + "remark": "", + "figures": [] +} \ No newline at end of file diff --git a/processed_dataset/proof/1035.json b/processed_dataset/proof/1035.json new file mode 100644 index 0000000000000000000000000000000000000000..e2447cf7e310287e2da9b13018a8f2982a9caf00 --- /dev/null +++ b/processed_dataset/proof/1035.json @@ -0,0 +1,8 @@ +{ + "source_file": "./raw_volume-zh/volume3/exercise3.tex", + "problem_type": "proof", + "problem": "问题28 在锐角三角形 $A B C$ 中, 已知 $A),三个相同的正方形相接, 求证: $\\alpha+\\beta=45^{\\circ}$.", + "solution": "(1) 参照例 8 可证之.\n(2) 由条件得 $\\tan \\alpha=\\frac{1}{3}, \\tan \\beta=\\frac{1}{2}$, 于是 $\\tan (\\alpha+\\beta)=\\frac{\\frac{1}{2}+\\frac{1}{3}}{1-\\frac{1}{2} \\times \\frac{1}{3}}=\\frac{5}{5}=1$, 所以 $\\alpha+\\beta=45^{\\circ}$.", + "remark": "", + "figures": [ + "./images/volume3/figures/fig-c4p23.png" + ] +} \ No newline at end of file diff --git a/processed_dataset/proof/1038.json b/processed_dataset/proof/1038.json new file mode 100644 index 0000000000000000000000000000000000000000..633247083aa17827626b1987feb59e21faaaa07a --- /dev/null +++ b/processed_dataset/proof/1038.json @@ -0,0 +1,8 @@ +{ + "source_file": "./raw_volume-zh/volume3/exercise5.tex", + "problem_type": "proof", + "problem": "问题21. $ A 、 B 、 C$ 为 $\\triangle A B C$ 三内角, 求证:\n$$\n\\sin \\frac{A}{2} \\sin \\frac{B}{2} \\sin \\frac{C}{2} \\leqslant \\frac{1}{8} .\n$$", + "solution": "因为 $b+c \\geqslant 2 \\sqrt{b c}$ ( $b=c$ 时, 取等号), 所以 $\\frac{a}{2 \\sqrt{b c}} \\geqslant \\frac{a}{b+c}= \\frac{\\sin A}{\\sin B+\\sin C}=\\frac{2 \\sin \\frac{A}{2} \\cos \\frac{A}{2}}{2 \\sin \\frac{B+C}{2} \\cos \\frac{B-C}{2}}=\\frac{\\sin \\frac{A}{2}}{\\cos \\frac{B-C}{2}} \\geqslant \\sin \\frac{A}{2}(B=C$ 时取等号). 同理可得 $\\frac{b}{2 \\sqrt{a c}} \\geqslant \\sin \\frac{B}{2}, \\frac{C}{2 \\sqrt{a b}} \\geqslant \\sin \\frac{C}{2}$, 所以 $\\sin \\frac{A}{2} \\sin \\frac{B}{2} \\sin \\frac{C}{2} \\leqslant \\frac{a}{2 \\sqrt{b c}} \\cdot \\frac{b}{2 \\sqrt{a c}} \\cdot \\frac{c}{2 \\sqrt{b a}}=\\frac{1}{8}$.", + "remark": "", + "figures": [] +} \ No newline at end of file diff --git a/processed_dataset/proof/1039.json b/processed_dataset/proof/1039.json new file mode 100644 index 0000000000000000000000000000000000000000..2265fb2042bea869814b4e946336f8f89d2a996b --- /dev/null +++ b/processed_dataset/proof/1039.json @@ -0,0 +1,8 @@ +{ + "source_file": "./raw_volume-zh/volume3/exercise5.tex", + "problem_type": "proof", + "problem": "问题22. 已知 $0<\\theta<\\frac{\\pi}{2}, a 、 b>0$, 求证:\n$$\n\\frac{a}{\\sin \\theta}+\\frac{b}{\\cos \\theta} \\geqslant\\left(a^{\\frac{2}{3}}+b^{\\frac{2}{3}}\\right)^{\\frac{3}{2}}\n$$", + "solution": "$\\frac{a}{\\sin \\theta}+\\frac{b}{\\cos \\theta} \\geqslant\\left(a^{\\frac{2}{3}}+b^{\\frac{2}{3}}\\right)^{\\frac{3}{2}} \\Leftrightarrow\\left(\\frac{a}{\\sin \\theta}+\\frac{b}{\\cos \\theta}\\right)^2 \\geqslant\\left(a^{\\frac{2}{3}}+b^{\\frac{2}{3}}\\right)^3 \\Leftrightarrow \\frac{a^2}{\\sin ^2 \\theta}+\\frac{2 a b}{\\sin \\theta \\cos \\theta}+\\frac{b^2}{\\cos ^2 \\theta} \\geqslant a^2+b^2+3 \\sqrt[3]{a^4 b^2}+3 \\sqrt[3]{a^2 b^4} \\Leftrightarrow a^2 \\cot ^2 \\theta+ b^2 \\tan ^2 \\theta+2 a b \\frac{\\sin ^2 \\theta+\\cos ^2 \\theta}{\\sin \\theta \\cos \\theta} \\geqslant 3 \\sqrt[3]{a^4 b^2}+3 \\sqrt[3]{a^2 b^4} \\Leftrightarrow a^2 \\cot ^2 \\theta+b^2 \\tan ^2 \\theta+ 2 a b \\tan \\theta+2 a b \\cot \\theta \\geqslant 3 \\sqrt[3]{a^4 b^2}+3 \\sqrt[3]{\\sqrt{a^2 b^4}},(*)$ 因为 $a^2 \\cot ^2 \\theta+2 a b \\tan \\theta= a^2 \\cot ^2 \\theta+a b \\tan \\theta+a b \\tan \\theta \\geqslant 3 \\sqrt[3]{a^4 b^2}$, 同理 $b^2 \\tan ^2 \\theta+2 a b \\cot \\theta \\geqslant 3 \\sqrt[3]{a^2 b^4}$, 所以 $(*)$ 成立, 原不等式得证.", + "remark": "", + "figures": [] +} \ No newline at end of file diff --git a/processed_dataset/proof/1040.json b/processed_dataset/proof/1040.json new file mode 100644 index 0000000000000000000000000000000000000000..789901421feefab0d741890d7069c5e64a538baa --- /dev/null +++ b/processed_dataset/proof/1040.json @@ -0,0 +1,8 @@ +{ + "source_file": "./raw_volume-zh/volume3/exercise5.tex", + "problem_type": "proof", + "problem": "问题23. $ A 、 B 、 C$ 为 $\\triangle A B C$ 三内角, 证明:\n$$\n\\sqrt{\\tan \\frac{A}{2} \\tan \\frac{B}{2}+5}+\\sqrt{\\tan \\frac{B}{2} \\tan \\frac{C}{2}+5}+\\sqrt{\\tan \\frac{C}{2} \\tan \\frac{A}{2}+5} \\leqslant 4 \\sqrt{3} .\n$$", + "solution": "由 $\\tan \\frac{A}{2} \\tan \\frac{B}{2}+\\tan \\frac{B}{2} \\tan \\frac{C}{2}+\\tan \\frac{C}{2} \\tan \\frac{A}{2}=1$, 得 $\\left(\\tan \\frac{A}{2} \\tan \\frac{B}{2}+5\\right)+ \\left(\\tan \\frac{B}{2} \\tan \\frac{C}{2}+5\\right)+\\left(\\tan \\frac{C}{2} \\tan \\frac{A}{2}+5\\right)=16$, 所以 $\\left(\\sqrt{\\tan \\frac{A}{2} \\tan \\frac{B}{2}+5}+\\right. \\left.\\sqrt{\\tan \\frac{B}{2} \\tan \\frac{C}{2}+5}+\\sqrt{\\tan \\frac{A}{2} \\tan \\frac{C}{2}+5}\\right)^2=16+2 \\sqrt{\\tan \\frac{A}{2} \\tan \\frac{B}{2}+5} \\times \\sqrt{\\tan \\frac{B}{2} \\tan \\frac{C}{2}+5}+2 \\sqrt{\\tan \\frac{A}{2} \\tan \\frac{B}{2}+5} \\sqrt{\\tan \\frac{C}{2} \\tan \\frac{A}{2}+5}+2 \\sqrt{\\tan \\frac{B}{2} \\tan \\frac{C}{2}+5} \\sqrt{\\tan \\frac{C}{2} \\tan \\frac{A}{2}+5} \\leqslant 16+\\tan \\frac{A}{2} \\tan \\frac{B}{2}+5+\\tan \\frac{B}{2} \\tan \\frac{C}{2}+5+\\tan \\frac{A}{2} \\tan \\frac{B}{2}+ 5+\\tan \\frac{C}{2} \\tan \\frac{A}{2}+5+\\tan \\frac{B}{2} \\tan \\frac{C}{2}+5+\\tan \\frac{C}{2} \\tan \\frac{A}{2}+5=16+30+2=48$,\n所以 $\\sqrt{\\tan \\frac{A}{2} \\tan \\frac{B}{2}+5}+\\sqrt{\\tan \\frac{B}{2} \\tan \\frac{C}{2}+5}+\\sqrt{\\tan \\frac{C}{2} \\tan \\frac{A}{2}+5} \\leqslant \\sqrt{48}= 4 \\sqrt{3}$.", + "remark": "", + "figures": [] +} \ No newline at end of file diff --git a/processed_dataset/proof/1041.json b/processed_dataset/proof/1041.json new file mode 100644 index 0000000000000000000000000000000000000000..2b095a312f5958feb87978cbd6abfb65fa20c402 --- /dev/null +++ b/processed_dataset/proof/1041.json @@ -0,0 +1,8 @@ +{ + "source_file": "./raw_volume-zh/volume3/exercise5.tex", + "problem_type": "proof", + "problem": "问题24. 在锐角 $\\triangle A B C$ 中,求证:\n$$\n\\sec A+\\sec B+\\sec C \\geqslant \\csc \\frac{A}{2}+\\csc \\frac{B}{2}+\\csc \\frac{C}{2} .\n$$", + "solution": "当 $a 、 b \\in \\mathbf{R}^{+}$时, 有 $(a+b)\\left(\\frac{1}{a}+\\frac{1}{b}\\right) \\geqslant 4$, 又 $A 、 B 、 C$ 为锐角 $\\triangle A B C$ 的内角, 所以 $\\sec A+\\sec B=\\frac{1}{\\cos A}+\\frac{1}{\\cos B} \\geqslant \\frac{4}{\\cos A+\\cos B}= \\frac{4}{2 \\cos \\frac{A+B}{2} \\cos \\frac{A-B}{2}} \\geqslant \\frac{2}{\\cos \\frac{A+B}{2}}=\\frac{2}{\\sin \\frac{C}{2}}=2 \\csc \\frac{C}{2}$, 同理 $\\sec B+\\sec C \\geqslant 2 \\csc \\frac{A}{2}, \\sec C+\\sec A \\geqslant 2 \\csc \\frac{B}{2}$, 三式相加, 得证.", + "remark": "", + "figures": [] +} \ No newline at end of file diff --git a/processed_dataset/proof/1042.json b/processed_dataset/proof/1042.json new file mode 100644 index 0000000000000000000000000000000000000000..4078167cc6e20155cd044ddd04c8972e7e1f494f --- /dev/null +++ b/processed_dataset/proof/1042.json @@ -0,0 +1,8 @@ +{ + "source_file": "./raw_volume-zh/volume3/exercise5.tex", + "problem_type": "proof", + "problem": "问题25. 设 $\\triangle A B C$ 的外接圆半径为 $R$, 面积为 $S$, 角 $A 、 B 、 C$ 所对的边为 $a 、 b 、 c$.\n求证:\n$$\n\\frac{36 S}{(a+b+c)^2} \\leqslant \\tan \\frac{A}{2}+\\tan \\frac{B}{2}+\\tan \\frac{C}{2} \\leqslant \\frac{9 R^2}{4 S} .\n$$", + "solution": "(1) 因为 $\\left(\\tan \\frac{A}{2}+\\tan \\frac{B}{2}+\\tan \\frac{C}{2}\\right)\\left(\\cot \\frac{A}{2}+\\cot \\frac{B}{2}+\\cot \\frac{C}{2}\\right) \\geqslant 9$, 又 $\\cot \\frac{A}{2}+\\cot \\frac{B}{2}+\\cot \\frac{C}{2}=\\frac{(a+b+c)^2}{4 S}$, 所以 $\\tan \\frac{A}{2}+\\tan \\frac{B}{2}+\\tan \\frac{C}{2} \\geqslant \\frac{9}{\\frac{(a+b+c)^2}{4 S}}=\\frac{36 S}{(a+b+c)^2}$. (2) 设 $\\triangle A B C$ 内切圆半径为 $r$, 易知 $\\tan \\frac{A}{2}= \\frac{2 r}{b+c-a}$, 故 $\\tan \\frac{A}{2}+\\tan \\frac{B}{2}+\\tan \\frac{C}{2}=\\frac{2 r}{b+c-a}+\\frac{2 r}{c+a-b}+\\frac{2 r}{a+b-c}= \\frac{4 S}{(a+b+c)(b+c-a)}+\\frac{4 S}{(a+b+c)(c+a-b)}+\\frac{4 S}{(a+b+c)(a+b-c)} =\\frac{S}{p(p-a)}+\\frac{S}{p(p-b)}+\\frac{S}{p(p-c)}\\left(p=\\frac{a+b+c}{2}\\right)$, 又 $S= \\sqrt{p(p-a)(p-b)(p-c)}$, 所以 $4 S\\left(\\tan \\frac{A}{2}+\\tan \\frac{B}{2}+\\tan \\frac{C}{2}\\right)=4(p-b) (p-c)+4(p-c)(p-a)+4(p-a)(p-b) \\leqslant(p-b+p-c)^2+(p-c+ p-a)^2+(p-a+p-b)^2=a^2+b^2+c^2=4 R^2\\left(\\sin ^2 A+\\sin ^2 B+\\sin ^2 C\\right)$, 因为 $\\sin ^2 A+\\sin ^2 B+\\sin ^2 C \\leqslant \\frac{9}{4}$, 所以 $4 S\\left(\\tan \\frac{A}{2}+\\tan \\frac{B}{2}+\\tan \\frac{C}{2}\\right) \\leqslant 9 R^2$, 即 $\\tan \\frac{A}{2}+\\tan \\frac{B}{2}+\\tan \\frac{C}{2} \\leqslant \\frac{9 R^2}{4 S}$.", + "remark": "", + "figures": [] +} \ No newline at end of file diff --git a/processed_dataset/proof/1043.json b/processed_dataset/proof/1043.json new file mode 100644 index 0000000000000000000000000000000000000000..03fc355a8c2fededa85106572ed45c973120225e --- /dev/null +++ b/processed_dataset/proof/1043.json @@ -0,0 +1,8 @@ +{ + "source_file": "./raw_volume-zh/volume3/exercise5.tex", + "problem_type": "proof", + "problem": "问题26. 求证: $1 \\leqslant \\sqrt{|\\sin \\alpha|}+\\sqrt{|\\cos \\alpha|} \\leqslant 2^{\\frac{3}{4}}$.", + "solution": "原不等式等价于 $1 \\leqslant|\\sin \\alpha|+|\\cos \\alpha|+2 \\sqrt{|\\sin \\alpha \\cos \\alpha|} \\leqslant 2 \\sqrt{2}$, 因为 $(|\\sin \\alpha|+|\\cos \\alpha|)^2=\\sin ^2 \\alpha+\\cos ^2 \\alpha+2|\\sin \\alpha \\cos \\alpha|=1+|\\sin 2 \\alpha| \\leqslant 2$, 所以 $1 \\leqslant|\\sin \\alpha|+|\\cos \\alpha| \\leqslant \\sqrt{2}$, 又 $2 \\sqrt{|\\sin \\alpha \\cos \\alpha|}=\\sqrt{2|\\sin 2 \\alpha|} \\leqslant \\sqrt{2}$, 故原不等式显然成立.", + "remark": "", + "figures": [] +} \ No newline at end of file diff --git a/processed_dataset/proof/1044.json b/processed_dataset/proof/1044.json new file mode 100644 index 0000000000000000000000000000000000000000..882fb51291c6a10d0d48d42d9278cf9789e9adb5 --- /dev/null +++ b/processed_dataset/proof/1044.json @@ -0,0 +1,8 @@ +{ + "source_file": "./raw_volume-zh/volume3/exercise5.tex", + "problem_type": "proof", + "problem": "问题27. 求证: $\\cos x \\geqslant 1-\\frac{x^2}{2}$.", + "solution": "先证 $|\\sin x| \\leqslant|x|, \\cos x=1-2 \\sin ^2 \\frac{x}{2}=1-2\\left|\\sin \\frac{x}{2}\\right|^2 \\geqslant 1-$\n$$\n2\\left|\\frac{x}{2}\\right|^2=1-\\frac{x^2}{2}\n$$", + "remark": "", + "figures": [] +} \ No newline at end of file diff --git a/processed_dataset/proof/1045.json b/processed_dataset/proof/1045.json new file mode 100644 index 0000000000000000000000000000000000000000..9c47860a787a4499a392f71d9f3aff0e20928d3f --- /dev/null +++ b/processed_dataset/proof/1045.json @@ -0,0 +1,10 @@ +{ + "source_file": "./raw_volume-zh/volume3/exercise5.tex", + "problem_type": "proof", + "problem": "问题28. 试证明: 对于任何实数 $x,|\\sin x|$ 与 $|\\sin (x+1)|$ 中至少有一个大于 $\\frac{1}{3}$.", + "solution": "用反证法.\n设 $|\\sin x| \\leqslant \\frac{1}{3},|\\sin (x+1)| \\leqslant \\frac{1}{3}$, 如图(), 作两条平行直线 $y= \\pm \\frac{1}{3}$, 则角 $x, x+1$ 的终边与单位圆的交点都落在 $\\overparen{A B}$ 或 $\\overparen{B^{\\prime} A^{\\prime}}$ 上, 由于 $\\sin \\angle A O B=\\sin \\left(2 \\arcsin \\frac{1}{3}\\right)=\\frac{4 \\sqrt{2}}{9}<\\frac{\\sqrt{2}}{2}= \\sin \\frac{\\pi}{4}<\\sin 1$, 所以 $\\angle A O B<1(\\mathrm{rad})$, 导致矛盾.", + "remark": "", + "figures": [ + "./images/volume3/figures/fig-c5p28.png" + ] +} \ No newline at end of file diff --git a/processed_dataset/proof/1046.json b/processed_dataset/proof/1046.json new file mode 100644 index 0000000000000000000000000000000000000000..e798a3f32220e025feb3d6ba09772d9fb8b11b8c --- /dev/null +++ b/processed_dataset/proof/1046.json @@ -0,0 +1,10 @@ +{ + "source_file": "./raw_volume-zh/volume3/exercise5.tex", + "problem_type": "proof", + "problem": "问题30. 设 $x 、 y 、 z$ 为实数, $0\\sin 2 x+\\sin 2 y+\\sin 2 z .\n$$", + "solution": "原不等式等价于 $\\frac{\\pi}{2}>2 \\sin x(\\cos x- \\cos y)+2 \\sin y(\\cos y-\\cos z)+\\sin 2 z$, 即 $\\frac{\\pi}{4}> \\sin x(\\cos x-\\cos y)+\\sin y(\\cos y-\\cos z)+ \\sin z \\cos z$, 如图()在单位圆中, 设半径 $O A 、 O B 、 O C$ 与 $O x$ 的夹角分别为 $x 、 y 、 z$, 则 $S_1=\\sin x(\\cos x- \\cos y), S_2=\\sin y(\\cos y-\\cos z), S_3=\\sin z \\cdot \\cos z$, 显然 $S_1+S_2+S_3<\\frac{\\pi}{4}$, 故得证.", + "remark": "", + "figures": [ + "./images/volume3/figures/fig-c5p30.png" + ] +} \ No newline at end of file diff --git a/processed_dataset/proof/1047.json b/processed_dataset/proof/1047.json new file mode 100644 index 0000000000000000000000000000000000000000..b676c580d3edaf667f55ad488e6f09bed2db849d --- /dev/null +++ b/processed_dataset/proof/1047.json @@ -0,0 +1,8 @@ +{ + "source_file": "./raw_volume-zh/volume3/exercise6.tex", + "problem_type": "proof", + "problem": "问题3. 已知 $x, y, z \\in \\mathbf{R}$, 求证:\n$$\n\\frac{x-y}{1+x y}+\\frac{y-z}{1+y z}+\\frac{z-x}{1+x z}=\\frac{(x-y)(y-z)(z-x)}{(1+x y)(1+y z)(1+z x)} .\n$$", + "solution": "设 $x=\\tan \\alpha, y=\\tan \\beta, z=\\tan \\gamma$, 则 $\\frac{x-y}{1+x y}=\\tan \\left(\\alpha^{--} \\beta\\right), \\frac{y-z}{1+y z}= \\tan (\\beta-\\gamma), \\frac{z-x}{1+z x}=\\tan (\\gamma-\\alpha)$, 因 $(\\alpha-\\beta)+(\\beta-\\gamma)+(\\gamma-\\alpha)=0$, 所以 $\\tan (\\alpha-\\beta)+\\tan (\\beta-\\gamma)+\\tan (\\gamma-\\alpha)=\\tan (\\alpha-\\beta) \\tan (\\beta-\\gamma) \\tan (\\gamma-\\alpha)$, 故原式成立.", + "remark": "", + "figures": [] +} \ No newline at end of file diff --git a/processed_dataset/proof/1048.json b/processed_dataset/proof/1048.json new file mode 100644 index 0000000000000000000000000000000000000000..e79eb83c63bc7830ade8a8abb179fd601ff99e1b --- /dev/null +++ b/processed_dataset/proof/1048.json @@ -0,0 +1,8 @@ +{ + "source_file": "./raw_volume-zh/volume3/exercise6.tex", + "problem_type": "proof", + "problem": "问题4. 已知 $x 、 y 、 z$ 是非负实数,且 $x+y+z=1$, 求证:\n$$\n0 \\leqslant x y+y z+z x-2 x y z \\leqslant \\frac{7}{27} .\n$$", + "solution": "不妨设 $z<\\frac{1}{2}$, 作三角代换: $x=\\sin ^2 \\alpha \\cos ^2 \\beta, y=\\cos ^2 \\alpha \\cos ^2 \\beta, z=\\sin ^2 \\beta$, 则 $x y+y z+z x-2 x y z=x y(1-z)+z(x+y-x y)=\\sin ^2 \\alpha \\cos ^2 \\alpha \\cos ^6 \\beta+ \\sin ^2 \\beta \\cos ^2 \\beta\\left(1-\\sin ^2 \\alpha \\cos ^2 \\alpha \\cos ^2 \\beta\\right) \\geqslant 0$. 同理 $x y+y z+z x-2 x y z=x y(1- 2 z)+z(x+y)=\\sin ^2 \\alpha \\cos ^2 \\alpha \\cos ^4 \\beta \\cos 2 \\beta+\\frac{1}{4} \\sin ^2 2 \\beta=-\\frac{1}{4} \\sin ^2 2 \\alpha \\cdot \\frac{1}{4} (1+2 \\cos 2 \\beta)^2 \\cos 2 \\beta+\\frac{1}{4}\\left(1-\\cos ^2 2 \\beta\\right) \\leqslant \\frac{1}{4}+\\frac{1}{16} \\cos 2 \\beta(1+\\cos 2 \\beta)^2- \\frac{1}{4} \\cos ^2 2 \\beta=\\frac{1}{4}+\\frac{1}{32}\\left[2 \\cos 2 \\beta(1-\\cos 2 \\beta)^2\\right] \\leqslant \\frac{1}{4}+\\frac{1}{32} \\times\\left(\\frac{2}{3}\\right)^3=\\frac{7}{27}$.", + "remark": "", + "figures": [] +} \ No newline at end of file diff --git a/processed_dataset/proof/1049.json b/processed_dataset/proof/1049.json new file mode 100644 index 0000000000000000000000000000000000000000..da9ec9b0b2583ab6a80a3b774ca5cac7b8dc5054 --- /dev/null +++ b/processed_dataset/proof/1049.json @@ -0,0 +1,8 @@ +{ + "source_file": "./raw_volume-zh/volume3/exercise6.tex", + "problem_type": "proof", + "problem": "问题5. 数列 $a_0, a_1, a_2, \\cdots$ 与 $b_0, b_1, b_2, \\cdots$ 定义如下: $a_0=\\frac{\\sqrt{2}}{2}, a_{n+1}=\\frac{\\sqrt{2}}{2}$\n$$\n\\sqrt{1-\\sqrt{1-a_n^2}}, n=0,1,2, \\cdots, b_0=1, b_{n+1}=\\frac{\\sqrt{1+b_n^2}-1}{b_n}, n=0,1,2, \\cdots\n$$\n求证:不等式 $2^{n+2} a_n<\\pi<2^{n+2} b_n$ 对 $n=0,1,2, \\cdots$ 成立.", + "solution": "$a_0=\\frac{\\sqrt{2}}{2}=\\sin \\frac{\\pi}{2^2}, a_1=\\frac{\\sqrt{2}}{2} \\sqrt{1-\\sqrt{1-\\sin ^2 \\frac{\\pi}{2^2}}}=\\frac{\\sqrt{2}}{2} \\sqrt{1-\\cos \\frac{\\pi}{2^2}}=\\frac{\\sqrt{2}}{2} \\times \\sqrt{2 \\sin ^2 \\frac{\\pi}{2^3}}=\\sin \\frac{\\pi}{2^3}$. 若设 $a_k=\\sin \\frac{\\pi}{2^{k+2}}$, 则 $a_{k+1}=\\frac{\\sqrt{2}}{2} \\cdot \\sqrt{1-\\cos \\frac{\\pi}{2^{k+2}}}=\\sin \\frac{\\pi}{2^{k+3}}$, 从而 $a_n=\\sin \\frac{\\pi}{2^{n+2}}$. 同理 $b_n=\\tan \\frac{\\pi}{2^{n+2}}$. 当 $x \\in\\left(0, \\frac{\\pi}{2}\\right)$ 时, $\\sin x1$, 且 $\\frac{1}{x}+\\frac{1}{y}+\\frac{1}{z}=2$, 证明:\n$$\n\\sqrt{x+y+z} \\geqslant \\sqrt{x-1}+\\sqrt{y-1}+\\sqrt{z-1} .\n$$", + "solution": "设 $\\alpha, \\beta, \\gamma \\in\\left(0, \\frac{\\pi}{2}\\right)$, 则有 $0<\\cos ^2 \\alpha, \\cos ^2 \\beta, \\cos ^2 \\gamma<1$, 且 $\\frac{1}{\\cos ^2 \\alpha}>1, \\frac{1}{\\cos ^2 \\beta}>1, \\frac{1}{\\cos ^2 \\gamma}>1$, 令 $x=\\frac{1}{\\cos ^2 \\alpha}, y=\\frac{1}{\\cos ^2 \\beta}, z=\\frac{1}{\\cos ^2 \\gamma}$, 则 $\\frac{1}{x}+\\frac{1}{y}+ \\frac{1}{z}=\\cos ^2 \\alpha+\\cos ^2 \\beta+\\cos ^2 \\gamma=2$. 即 $\\sin ^2 \\alpha+\\sin ^2 \\beta+\\sin ^2 \\gamma=1 \\cdots$ (1). 待证的不等\n(1) 式, 对 (2) 式用柯西不等式有 $\\frac{\\sin \\alpha}{\\cos \\alpha}+\\frac{\\sin \\beta}{\\cos \\beta}+\\frac{\\sin \\gamma}{\\cos \\gamma} \\leqslant \\sqrt{\\sin ^2 \\alpha+\\sin ^2 \\beta+\\sin ^2 \\gamma} \\sin \\alpha \\cos \\alpha=\\sin \\beta \\cos \\beta=\\sin \\gamma \\cos \\gamma$. 即 $\\alpha=\\beta=\\gamma$ 时 (3) 等号成立,故 (2) 式成立, 所以原不等式得证.\n评注此题解三角代换 $x=\\frac{1}{\\sin ^2 \\alpha}, y=\\frac{1}{\\sin ^2 \\beta}, z=\\frac{1}{\\sin ^2 \\gamma}$ 证法一样.", + "remark": "", + "figures": [] +} \ No newline at end of file diff --git a/processed_dataset/proof/1059.json b/processed_dataset/proof/1059.json new file mode 100644 index 0000000000000000000000000000000000000000..90c3206edaa8104e35ec8a5cbf7961126e95d908 --- /dev/null +++ b/processed_dataset/proof/1059.json @@ -0,0 +1,10 @@ +{ + "source_file": "./raw_volume-zh/volume3/exercise6.tex", + "problem_type": "proof", + "problem": "问题24. 设 $\\triangle A B C$ 是锐角三角形, 其外接圆圆心为 $O$, 半径为 $R, A O$ 交 $\\triangle B O C$ 的外接圆于 $A^{\\prime}, B O$ 交 $\\triangle C O A$ 的外接圆于点 $B^{\\prime}, C O$ 交 $\\triangle A O B$ 的外接圆于点 $C^{\\prime}$. 证明: $O A^{\\prime} \\cdot O B^{\\prime} \\cdot O C^{\\prime} \\geqslant 8 R^3$, 并指出在什么条件下等号成立.", + "solution": "如图(), 作 $\\triangle B O C$ 的外接圆直径 $O D$, 连 $A^{\\prime} D 、 C D$, 则 $\\angle O A^{\\prime} D=\\angle O C D=90^{\\circ}$, 从而 $O A^{\\prime}=O D$. $\\cos \\angle A^{\\prime} O D=R \\cdot \\frac{\\cos \\angle A^{\\prime} O D}{\\cos \\angle C O D}$; 易知 $O D \\perp B C$, 于是 $\\angle C O D=\\angle A, \\angle A^{\\prime} O D=180^{\\circ}-\\angle C O D- \\angle A O C=180^{\\circ}-\\angle A-2 \\angle B=\\angle C-\\angle B$, 即 $O A^{\\prime}= R \\frac{\\cos (B-C)}{\\cos A}$, 同理 $O B^{\\prime}=R \\cdot \\frac{\\cos (A-C)}{\\cos B}, O C^{\\prime}= R \\frac{\\cos (B-A)}{\\cos C}$, 于是 $O A^{\\prime} \\cdot O B^{\\prime} \\cdot O C^{\\prime} \\geqslant 8 R^3 \\Leftrightarrow\\frac{\\cos (A-B)}{\\cos C} \\cdot \\frac{\\cos (A-C)}{\\cos B} \\cdot \\frac{\\cos (B-C)}{\\cos A} \\geqslant 8$, ( * ) 而 $\\frac{\\cos (A-B)}{\\cos C}= \\frac{\\cos A}{-\\cos A \\cos B+\\sin A \\sin B}=\\frac{1+\\cot A \\cdot \\cot B}{1-\\cot A \\cdot \\cot B}$, 令 $x=\\cot A \\cot B, y= \\cot B \\cot C, z=\\cot C \\cot A$. 则有 $x+y+z=1$, 又锐角三角形, 所以 $x, y$, $z>0, \\frac{1+x}{1-x}=\\frac{x+y+z+x}{x+y+z-x}=\\frac{(x+y)+(z+x)}{y+z} \\geqslant \\frac{2 \\sqrt{(x+y)(x+z)}}{y+z}$, 同理 $\\frac{1+y}{1-y} \\geqslant 2 \\frac{\\sqrt{(x+y)(y+z)}}{x+z}, \\frac{1+z}{1-z} \\geqslant 2 \\frac{\\sqrt{(x+z)(y+z)}}{x+y}$, 故( * ) 得证, 所以原结论获证.", + "remark": "", + "figures": [ + "./images/volume3/figures/fig-c6p24.png" + ] +} \ No newline at end of file diff --git a/processed_dataset/proof/1060.json b/processed_dataset/proof/1060.json new file mode 100644 index 0000000000000000000000000000000000000000..ce0f5676111a96a41aff1900df48a1cc43a0f296 --- /dev/null +++ b/processed_dataset/proof/1060.json @@ -0,0 +1,8 @@ +{ + "source_file": "./raw_volume-zh/volume3/exercise6.tex", + "problem_type": "proof", + "problem": "问题25. 已知正数 $m_i \\in \\mathbf{R}^{+}(i=1,2, \\cdots, n), p \\geqslant 2$ 且 $p \\in \\mathbf{N}$ 并满足 $\\sum_{i=1}^n \\frac{1}{1+m_i^p}=$ 1 , 求证: $\\prod_{i=1}^n m_i \\geqslant(n-1)^{\\frac{n}{p}}$.", + "solution": "令 $m_i^p=\\tan ^2 \\alpha_i, \\alpha_i \\in\\left(0, \\frac{\\pi}{2}\\right)$, 由已知条件应有 $\\sum_{i=1}^n \\cos ^2 \\alpha_i=1$, 于是 $\\sum_{i=1}^{n-1} \\cos ^2 \\alpha_i=\\sin ^2 \\alpha_n, \\cos ^2 \\alpha_1+\\cos ^2 \\alpha_2+\\cdots+\\cos ^2 \\alpha_{n-2}+\\cos ^2 \\alpha_n=\\sin ^2 \\alpha_{n-1}, \\cdots$, $\\cos ^2 \\alpha_2+\\cos ^2 \\alpha_3+\\cdots+\\cos ^2 \\alpha_n=\\sin ^2 \\alpha_1$, 以上各式利用均值不等式, 得 $(n-1) \\sqrt[n-1]{\\cos ^2 \\alpha_1 \\cos ^2 \\alpha_2 \\cdots \\cos ^2 \\alpha_{n-1}} \\leqslant \\sin ^2 \\alpha_n,(n-1) \\sqrt[n-1]{\\cos ^2 \\alpha_1 \\cos ^2 \\alpha_2 \\cdots \\cos ^2 \\alpha_{n-2} \\cos ^2 \\alpha_n} \\leqslant \\sin ^2 \\alpha_{n-1}, \\cdots, \\quad(n-1) \\sqrt[n-1]{\\cos ^2 \\alpha_2 \\cos ^2 \\alpha_3 \\cdots \\cos ^2 \\alpha_n} \\leqslant \\sin ^2 \\alpha_1$, 把上述几个不等式两边相乘得 $(n-1)^n\\left[\\cos ^2 \\alpha_1 \\cos ^2 \\alpha_2 \\cdots \\cos ^2 \\alpha_n\\right] \\leqslant \\sin ^2 \\alpha_1 \\sin ^2 \\alpha_2 \\cdots \\sin ^2 \\alpha_n$, 即 $\\tan ^2 \\alpha_1 \\tan ^2 \\alpha_2 \\cdots \\tan ^2 \\alpha_n \\geqslant(n-1)^n$, 由于 $m_i^p=\\tan ^2 \\alpha_i, i=1,2, \\cdots, n$, 故 $m_1$ • $m_2 \\cdots m_n \\geqslant(n-1)^{\\frac{n}{p}}$, 得证.\nEnd of \"cropped_page_174.png\"", + "remark": "", + "figures": [] +} \ No newline at end of file diff --git a/processed_dataset/proof/1061.json b/processed_dataset/proof/1061.json new file mode 100644 index 0000000000000000000000000000000000000000..ed58322fdb172e387fa08903d68ed01b5456cb63 --- /dev/null +++ b/processed_dataset/proof/1061.json @@ -0,0 +1,8 @@ +{ + "source_file": "./raw_volume-zh/volume4/chapter2-1.tex", + "problem_type": "proof", + "problem": "例1. 设 $f(x)=\\frac{a}{a^2-1}\\left(a^x-a^{-x}\\right)(a>0, a \\neq 1)$, 证明: 对正整数 $n \\geqslant$ 2 , 有\n$$\nf(n)>n .\n$$", + "solution": "证明:当 $n \\geqslant 2$ 时, 由平均值不等式, 得\n$$\n\\begin{aligned}\nf(n) & =\\frac{a}{a^2-1}\\left(a^n-a^{-n}\\right)=\\frac{a}{a^2-1}\\left(a^n-\\frac{1}{a^n}\\right) \\\\\n& =\\frac{a}{a^2-1}\\left(a-\\frac{1}{a}\\right)\\left(a^{n-1}+a^{n-2} \\frac{1}{a}+a^{n-3} \\frac{1}{a^2}+\\cdots+a \\frac{1}{a^{n-2}}+\\frac{1}{a^{n-1}}\\right) \\\\\n& \\geqslant \\frac{a}{a^2-1}\\left(a-\\frac{1}{a}\\right) n \\sqrt[n]{a^{n-1} a^{n-2} \\cdots a^2 a \\frac{1}{a} \\frac{1}{a^2} \\cdots \\frac{1}{a^{n-1}}}=n,\n\\end{aligned}\n$$\n当且仅当 $a=1$ 时等号成立,故命题成立.", + "remark": "", + "figures": [] +} \ No newline at end of file diff --git a/processed_dataset/proof/1062.json b/processed_dataset/proof/1062.json new file mode 100644 index 0000000000000000000000000000000000000000..c9685d648b3258d15b1f89e20da541a16225093e --- /dev/null +++ b/processed_dataset/proof/1062.json @@ -0,0 +1,8 @@ +{ + "source_file": "./raw_volume-zh/volume4/chapter2-1.tex", + "problem_type": "proof", + "problem": "例2. 设 $x>0$, 证明: $2^{12 \\sqrt{x}}+2^{4 \\sqrt[4]{x}} \\geqslant 2 \\cdot 2^{\\sqrt[6]{x}}$.", + "solution": "证明:由该不等式的外形, 很容易想到平均值不等式.\n由平均值不等式, 得\n$$\n\\begin{gathered}\n2^{12 \\sqrt{x}}+2^{\\sqrt[4]{x}} \\geqslant 2 \\cdot \\sqrt{2^{12 \\sqrt{x}} 2^{4 \\sqrt{x}}}=2 \\cdot 2^{\\frac{12 \\sqrt{x}+4 \\sqrt{x}}{2}} . \\\\\n\\frac{\\sqrt[12]{x}+\\sqrt[4]{x}}{2} \\geqslant\\left(x^{\\frac{1}{12}} x^{\\frac{1}{4}}\\right)^{\\frac{1}{2}}=x^{\\frac{1}{6}} .\n\\end{gathered}\n$$\n所以\n$$\n2^{12 \\sqrt{x}}+2^{4 \\sqrt{x}} \\geqslant 2 \\cdot 2^{\\sqrt[6]{x}} \\text {. }\n$$", + "remark": "", + "figures": [] +} \ No newline at end of file diff --git a/processed_dataset/proof/1063.json b/processed_dataset/proof/1063.json new file mode 100644 index 0000000000000000000000000000000000000000..c82573b596087244bd4f03cfc9615a7b381eda7f --- /dev/null +++ b/processed_dataset/proof/1063.json @@ -0,0 +1,8 @@ +{ + "source_file": "./raw_volume-zh/volume4/chapter2-1.tex", + "problem_type": "proof", + "problem": "例3. 设 $a_i>0, i=1,2, \\cdots, n$ 满足 $a_1 a_2 \\cdots a_n=1$. 证明:\n$$\n\\left(2+a_1\\right)\\left(2+a_2\\right) \\cdots\\left(2+a_n\\right) \\geqslant 3^n .\n$$", + "solution": "当 $n=1$ 时,则 $a_1=1$, 显然成立.\n假定当 $n=k$ 时成立,那么, 对于 $n= k+1$, 由于 $a_1 a_2 \\cdots a_k a_{k+1}=1$, 如果有某个 $a_i=1$, 则由归纳假设, 命题成立.\n如果 $a_i$ 都不为 1 , 则必有大于 1 的, 且必有小于 1 的, 不妨设 $a_k>1, a_{k+1}<1$. 则由归纳假设, 得\n$$\n\\left(2+a_1\\right)\\left(2+a_2\\right) \\cdots\\left(2+a_{k-1}\\right)\\left(2+a_k a_{k+1}\\right) \\geqslant 3^k .\n$$\n于是, 为了证明命题, 只要证明\n$$\n\\left(2+a_k\\right)\\left(2+a_{k+1}\\right) \\geqslant 3\\left(2+a_k a_{k+1}\\right)\n$$\n便可.\n因为\n$$\n\\left(2+a_k\\right)\\left(2+a_{k+1}\\right) \\geqslant 3\\left(2+a_k a_{k+1}\\right),\n$$\n等价于\n$$\n4+2 a_k+2 a_{k+1}+a_k a_{k+1} \\geqslant 6+3 a_k a_{k+1},\n$$\n等价于\n$$\n\\begin{gathered}\na_k+a_{k+1}-a_k a_{k+1}-1 \\geqslant 0, \\\\\n\\quad\\left(a_k-1\\right)\\left(1-a_{k+1}\\right) \\geqslant 0 .\n\\end{gathered}\n$$\n由假设最后不等式成立, 故命题成立.\n注:这里, 选取 $a_k>1, a_{k+1}<1$, 在平均值不等式的证明方法四中有过类似的考虑.\n证明:由于对任意的 $i$,\n$$\n2+a_i=1+1+a_i \\geqslant 3 \\sqrt[3]{a_i} .\n$$\n故 $\\quad\\left(2+a_1\\right)\\left(2+a_2\\right) \\cdots\\left(2+a_n\\right) \\geqslant 3^n \\sqrt[3]{a_1 a_2 \\cdots a_n}=3^n$.", + "remark": "", + "figures": [] +} \ No newline at end of file diff --git a/processed_dataset/proof/1064.json b/processed_dataset/proof/1064.json new file mode 100644 index 0000000000000000000000000000000000000000..5b5f90211ec4c49ed74f6496458cafe915b32de0 --- /dev/null +++ b/processed_dataset/proof/1064.json @@ -0,0 +1,8 @@ +{ + "source_file": "./raw_volume-zh/volume4/chapter2-1.tex", + "problem_type": "proof", + "problem": "例4. 设 $a>b>0$, 求证: $\\sqrt{2} a^3+\\frac{3}{a b-b^2} \\geqslant 10$.", + "solution": "证明:因为 $a b-b^2=b(a-b) \\leqslant \\frac{[b+(a-b)]^2}{4}=\\frac{a^2}{4}$, 所以\n$$\n\\begin{aligned}\n& \\sqrt{2} a^3+\\frac{3}{a b-b^2} \\geqslant \\sqrt{2} a^3+\\frac{12}{a^2} \\\\\n= & \\frac{\\sqrt{2}}{2} a^3+\\frac{\\sqrt{2}}{2} a^3+\\frac{4}{a^2}+\\frac{4}{a^2}+\\frac{4}{a^2} \\\\\n\\geqslant & 5 \\sqrt[5]{\\frac{\\sqrt{2}}{2} a^3 \\cdot \\frac{\\sqrt{2}}{2} a^3 \\cdot \\frac{4}{a^2} \\cdot \\frac{4}{a^2} \\cdot \\frac{4}{a^2}}=10,\n\\end{aligned}\n$$\n即命题成立.", + "remark": "注:为了消去 $a$, 将 $\\sqrt{2} a^3$ 写成两项, $\\frac{12}{a^2}$ 写成三项.\n这样, 利用平均值不等式, 它们的乘积为一个常数.", + "figures": [] +} \ No newline at end of file diff --git a/processed_dataset/proof/1065.json b/processed_dataset/proof/1065.json new file mode 100644 index 0000000000000000000000000000000000000000..f07ab7c054fcbad3666521407aae0956f280d6db --- /dev/null +++ b/processed_dataset/proof/1065.json @@ -0,0 +1,8 @@ +{ + "source_file": "./raw_volume-zh/volume4/chapter2-1.tex", + "problem_type": "proof", + "problem": "例5. 设 $a, b, c>0$, 求证:\n$$\n\\frac{c}{a}+\\frac{a}{b+c}+\\frac{b}{c} \\geqslant 2 .\n$$", + "solution": "证明:由平均值不等式, 得\n$$\n\\begin{aligned}\n\\frac{c}{a}+\\frac{a}{b+c}+\\frac{b}{c} & =\\frac{c}{a}+\\frac{a}{b+c}+\\frac{b+c}{c}-1 \\\\\n& \\geqslant 3 \\sqrt[3]{\\frac{c}{a} \\cdot \\frac{a}{b+c} \\cdot \\frac{b+c}{c}}-1 \\\\\n& =3-1=2,\n\\end{aligned}\n$$\n即命题成立.", + "remark": "", + "figures": [] +} \ No newline at end of file diff --git a/processed_dataset/proof/1066.json b/processed_dataset/proof/1066.json new file mode 100644 index 0000000000000000000000000000000000000000..ed1a819e60905e27cbed64aba986f62166fa320b --- /dev/null +++ b/processed_dataset/proof/1066.json @@ -0,0 +1,8 @@ +{ + "source_file": "./raw_volume-zh/volume4/chapter2-1.tex", + "problem_type": "proof", + "problem": "例6. 设 $x+y+z=0$, 求证:\n$$\n6\\left(x^3+y^3+z^3\\right)^2 \\leqslant\\left(x^2+y^2+z^2\\right)^3 .\n$$", + "solution": "证明:由 $x+y+z=0$ 及其对称性, 不妨假设 $x, y \\geqslant 0, z \\leqslant 0$, 由于 $x+y=-z$, 得 $z^2=(x+y)^2$, 从而\n$$\n\\left(x^2+y^2+z^2\\right)^3=8\\left(x^2+x y+y^2\\right)^3 .\n$$\n由 $A_3 \\geqslant G_3$, 得\n$$\n\\begin{aligned}\nx^2+x y+y^2 & =\\frac{x(x+y)}{2}+\\frac{y(x+y)}{2}+\\frac{x^2+y^2}{2} \\\\\n& \\geqslant 3 \\sqrt[3]{\\frac{x y(x+y)^2}{4} \\cdot \\frac{x^2+y^2}{2}} \\\\\n& \\geqslant 3 \\sqrt[3]{\\frac{x^2 y^2 z^2}{4}},\n\\end{aligned}\n$$\n所以\n$$\n\\left(x^2+y^2+z^2\\right)^3 \\geqslant 54 x^2 y^2 z^2=6\\left(x^3+y^3+z^3\\right)^2 .\n$$", + "remark": "", + "figures": [] +} \ No newline at end of file diff --git a/processed_dataset/proof/1067.json b/processed_dataset/proof/1067.json new file mode 100644 index 0000000000000000000000000000000000000000..70eb05d1e04dd820b3765f4f8b67c7b823476e01 --- /dev/null +++ b/processed_dataset/proof/1067.json @@ -0,0 +1,8 @@ +{ + "source_file": "./raw_volume-zh/volume4/chapter2-1.tex", + "problem_type": "proof", + "problem": "例7. 设 $a_1, a_2, \\cdots, a_n \\in \\mathbf{R}^{+}, S=a_1+a_2+\\cdots+a_n$. 求证:\n$$\n\\left(1+a_1\\right)\\left(1+a_2\\right) \\cdots\\left(1+a_n\\right) \\leqslant 1+S+\\frac{S^2}{2 !}+\\cdots+\\frac{S^n}{n !} .\n$$", + "solution": "证明:由于 $G_n \\leqslant A_n$, 得\n$$\n\\begin{aligned}\n& \\left(1+a_1\\right)\\left(1+a_2\\right) \\cdots\\left(1+a_n\\right) \\\\\n\\leqslant & \\left(\\frac{n+a_1+a_2+\\cdots+a_n}{n}\\right)^n=\\left(1+\\frac{S}{n}\\right)^n \\\\\n= & 1+\\mathrm{C}_n^1\\left(\\frac{S}{n}\\right)+\\mathrm{C}_n^2\\left(\\frac{S}{n}\\right)^2+\\cdots+\\mathrm{C}_n^m\\left(\\frac{S}{n}\\right)^m+\\cdots+\\mathrm{C}_n^n\\left(\\frac{S}{n}\\right)^n .\n\\end{aligned}\n$$\n因为 $n !=(n-m) !(n-m+1) \\cdots n \\leqslant(n-m) ! n^m$,\n所以\n$$\n\\mathrm{C}_n^m\\left(\\frac{S}{n}\\right)^m=\\frac{n !}{m !(n-m) !} \\cdot \\frac{1}{n^m} S^m \\leqslant \\frac{S^m}{m !},\n$$\n从而命题成立.", + "remark": "", + "figures": [] +} \ No newline at end of file diff --git a/processed_dataset/proof/1068.json b/processed_dataset/proof/1068.json new file mode 100644 index 0000000000000000000000000000000000000000..3e4a89621a35b7e2f7b063e9ec7398b26bf7dc82 --- /dev/null +++ b/processed_dataset/proof/1068.json @@ -0,0 +1,8 @@ +{ + "source_file": "./raw_volume-zh/volume4/chapter2-1.tex", + "problem_type": "proof", + "problem": "例8. 设 $k, n$ 为正整数, 且 $1 \\leqslant k \\leqslant n, a_i \\in \\mathbf{R}^{+}$, 满足 $a_1+a_2+\\cdots+ a_k=a_1 a_2 \\cdots a_k$. 求证:\n$$\na_1^{n-1}+a_2^{n-1}+\\cdots+a_k^{n-1} \\geqslant k n,\n$$\n并确定等号成立的充要条件.", + "solution": "证明:令 $a=a_1+a_2+\\cdots+a_k=a_1 a_2 \\cdots a_k$. 由平均值不等式, 得\n$$\na \\geqslant k a^{\\frac{1}{k}} \\text {, 即 } a \\geqslant k^{\\frac{k}{k-1}} \\text {. }\n$$\n又因为\n$$\na_1^{n-1}+a_2^{n-1}+\\cdots+a_k^{n-1} \\geqslant k\\left(a_1 a_2 \\cdots a_k\\right)^{\\frac{n-1}{k}}=k a^{\\frac{n-1}{k}} \\geqslant k \\cdot k^{\\frac{n-1}{k-1}},\n$$\n于是只需证明\n$$\nk^{\\frac{n-1}{k-1}} \\geqslant n\n$$\n再由平均值不等式, 得\n$$\nk=\\frac{(k-1) n+(n-k) \\times 1}{n-1} \\geqslant n^{\\frac{k-1}{n-1}},\n$$\n从而不等式成立.\n不难看出, 当 $k=n$ 且 $a_1=a_2=\\cdots=a_k$ 时等号成立.", + "remark": "", + "figures": [] +} \ No newline at end of file diff --git a/processed_dataset/proof/1069.json b/processed_dataset/proof/1069.json new file mode 100644 index 0000000000000000000000000000000000000000..3ea8a5b667ccd2a4959e1e817f1f1bfa59b394e8 --- /dev/null +++ b/processed_dataset/proof/1069.json @@ -0,0 +1,8 @@ +{ + "source_file": "./raw_volume-zh/volume4/chapter2-1.tex", + "problem_type": "proof", + "problem": "例9. 设 $a_i>0(i=1,2, \\cdots, n)$, 求证:\n$$\n\\sum_{k=1}^n k a_k \\leqslant \\frac{n(n-1)}{2}+\\sum_{k=1}^n a_k^k .\n$$", + "solution": "证明:因为 $\\frac{n(n-1)}{2}=\\sum_{k=1}^n(k-1)$, 所以由平均值不等式, 得\n$$\n\\frac{n(n-1)}{2}+\\sum_{k=1}^n a_k^k\n$$\n$$\n\\begin{aligned}\n& =\\sum_{k=1}^n\\left[(k-1)+a_k^k\\right] \\\\\n& =\\sum_{k=1}^n\\left(1+1+\\cdots+1+a_k^k\\right) \\\\\n& \\geqslant \\sum_{k=1}^n k \\sqrt[k]{1^{k-1} \\cdot a_k^k}=\\sum_{k=1}^n k a_k,\n\\end{aligned}\n$$\n故命题成立.", + "remark": "注:应用平均值不等式时, 通常要将乘幂看作连乘积, 有时还要巧妙地添上数 1 .", + "figures": [] +} \ No newline at end of file diff --git a/processed_dataset/proof/1070.json b/processed_dataset/proof/1070.json new file mode 100644 index 0000000000000000000000000000000000000000..621565aee903beb3d7c42527263b77658e411b5a --- /dev/null +++ b/processed_dataset/proof/1070.json @@ -0,0 +1,8 @@ +{ + "source_file": "./raw_volume-zh/volume4/chapter2-1.tex", + "problem_type": "proof", + "problem": "例10. 设 $a_i>0, b_i>0$ 且满足 $a_1+a_2+\\cdots+a_n \\leqslant 1, b_1+b_2+\\cdots+ b_n \\leqslant n$. 求证:\n$$\n\\left(\\frac{1}{a_1}+\\frac{1}{b_1}\\right)\\left(\\frac{1}{a_2}+\\frac{1}{b_2}\\right) \\cdots\\left(\\frac{1}{a_n}+\\frac{1}{b_n}\\right) \\geqslant(n+1)^n .\n$$", + "solution": "证明:由已知条件和平均值不等式, 得\n$$\n\\begin{gathered}\na_1 a_2 \\cdots a_n \\leqslant\\left(\\frac{a_1+a_2+\\cdots+a_n}{n}\\right)^n \\leqslant \\frac{1}{n^n}, \\\\\nb_1 b_2 \\cdots b_n \\leqslant\\left(\\frac{b_1+b_2+\\cdots+b_n}{n}\\right)^n \\leqslant 1 . \\\\\n\\frac{1}{a_i}+\\frac{1}{b_i}=\\frac{1}{n a_i}+\\cdots+\\frac{1}{n a_i}+\\frac{1}{b_i} \\\\\n\\geqslant(n+1) \\sqrt[n+1]{\\left(\\frac{1}{n a_i}\\right)^n\\left(\\frac{1}{b_i}\\right)}, \\\\\n\\left(\\frac{1}{a_1}+\\frac{1}{b_1}\\right)\\left(\\frac{1}{a_2}+\\frac{1}{b_2}\\right) \\cdots\\left(\\frac{1}{a_n}+\\frac{1}{b_n}\\right) \\\\\n\\geqslant(n+1)^n \\sqrt[n+1]{\\frac{1}{\\left(n^n\\right)^n} \\frac{1}{\\left(a_1 a_2 \\cdots a_n\\right)^n} \\frac{1}{b_1 b_2 \\cdots b_n}} \\\\\n\\geqslant(n+1)^n .\n\\end{gathered}\n$$\n故命题成立.", + "remark": "注:此题证明的关键是将 $\\frac{1}{a_i}$ 写成 $\\frac{1}{n a_i}+\\cdots+\\frac{1}{n a_i}$.", + "figures": [] +} \ No newline at end of file diff --git a/processed_dataset/proof/1071.json b/processed_dataset/proof/1071.json new file mode 100644 index 0000000000000000000000000000000000000000..a5d747fe03c3d947080f64d18e13d0bde1ec94fd --- /dev/null +++ b/processed_dataset/proof/1071.json @@ -0,0 +1,8 @@ +{ + "source_file": "./raw_volume-zh/volume4/chapter2-1.tex", + "problem_type": "proof", + "problem": "例11. 假设 $a 、 b 、 c$ 都是正数,证明:\n$$\na b c \\geqslant(a+b-c)(b+c-a)(c+a-b) .\n$$", + "solution": "证明:如果 $a+b-c, b+c-a, c+a-b$ 中有负数, 不妨设 $a+b-c<0$,\n则 $c>a+b$. 故 $b+c-a$ 与 $c+a-b$ 均为正数, 则结论显然成立.\n若 $a+b-c, b+c-a, c+a-b$ 均非负, 则由平均值不等式, 得\n$$\n\\sqrt{(a+b-c)(b+c-a)} \\leqslant \\frac{(a+b-c)+(b+c-a)}{2}=b .\n$$\n同理可得\n$$\n\\begin{aligned}\n& \\sqrt{(b+c-a)(c+a-b)} \\leqslant \\frac{(b+c-a)+(c+a-b)}{2}=c, \\\\\n& \\sqrt{(c+a-b)(a+b-c)} \\leqslant \\frac{(c+a-b)+(a+b-c)}{2}=a .\n\\end{aligned}\n$$\n将三式相乘,即得到我们要证明的问题,故命题成立.", + "remark": "注:通过对部分变量应用平均值不等式,而且轮换使用, 从而得到结论的证明.", + "figures": [] +} \ No newline at end of file diff --git a/processed_dataset/proof/1072.json b/processed_dataset/proof/1072.json new file mode 100644 index 0000000000000000000000000000000000000000..af203ec964cae7e1090293bd4c7c88c20b965f96 --- /dev/null +++ b/processed_dataset/proof/1072.json @@ -0,0 +1,8 @@ +{ + "source_file": "./raw_volume-zh/volume4/chapter2-1.tex", + "problem_type": "proof", + "problem": "例12. 假设正数 $a 、 b 、 c$ 满足 $(1+a)(1+b)(1+c)=8$. 证明 : $a b c \\leqslant 1$.", + "solution": "证明:由假设得\n$$\n1+(a+b+c)+(a b+b c+c a)+a b c=8 .\n$$\n再由平均值不等式, 得\n$$\na+b+c \\geqslant 3(a b c)^{\\frac{1}{3}}, a b+b c+c a \\geqslant 3(a b c)^{\\frac{2}{3}} .\n$$\n当且仅当 $a=b=c$ 时等式成立.\n于是\n$$\n8 \\geqslant 1+3(a b c)^{\\frac{1}{3}}+3(a b c)^{\\frac{2}{3}}+a b c=\\left[1+(a b c)^{\\frac{1}{3}}\\right]^3 .\n$$\n由此, 得\n$$\n(a b c)^{\\frac{1}{3}} \\leqslant 2-1=1 .\n$$\n所以, $a b c \\leqslant 1$, 当且仅当 $a=b=c$ 时等式成立.", + "remark": "", + "figures": [] +} \ No newline at end of file diff --git a/processed_dataset/proof/1073.json b/processed_dataset/proof/1073.json new file mode 100644 index 0000000000000000000000000000000000000000..f2b3451b7d8514dbe14a5786161ef3bdd12b695f --- /dev/null +++ b/processed_dataset/proof/1073.json @@ -0,0 +1,8 @@ +{ + "source_file": "./raw_volume-zh/volume4/chapter2-1.tex", + "problem_type": "proof", + "problem": "例13. 设 $n$ 为正整数,证明:\n$$\nn\\left[(n+1)^{\\frac{1}{n}}-1\\right] \\leqslant 1+\\frac{1}{2}+\\frac{1}{3}+\\cdots+\\frac{1}{n} \\leqslant n-(n-1)\\left(\\frac{1}{n}\\right)^{\\frac{1}{n-1}} .\n$$", + "solution": "证明:只证明不等式的左边,不等式的右边可同样处理.\n令 $A=\\frac{1+\\frac{1}{2}+\\frac{1}{3}+\\cdots+\\frac{1}{n}+n}{n}$, 则左边的不等式等价于\n$$\nA \\geqslant(n+1)^{\\frac{1}{n}}\n$$\n由平均值不等式, 得\n$$\n\\begin{aligned}\nA & =\\frac{(1+1)+\\left(1+\\frac{1}{2}\\right)+\\cdots+\\left(1+\\frac{1}{n}\\right)}{n} \\\\\n& =\\frac{2+\\frac{3}{2}+\\frac{4}{3}+\\cdots+\\frac{n+1}{n}}{n} \\\\\n& \\geqslant \\sqrt[n]{2 \\cdot \\frac{3}{2} \\cdot \\frac{4}{3} \\cdots \\frac{n+1}{n}}=(n+1)^{\\frac{1}{n}} .\n\\end{aligned}\n$$\n从而得\n$$\n1+\\frac{1}{2}+\\frac{1}{3}+\\cdots+\\frac{1}{n} \\geqslant n\\left[(n+1)^{\\frac{1}{n}}-1\\right] .\n$$\n不难看出, 当 $n=1$ 时等号成立.", + "remark": "", + "figures": [] +} \ No newline at end of file diff --git a/processed_dataset/proof/1074.json b/processed_dataset/proof/1074.json new file mode 100644 index 0000000000000000000000000000000000000000..c162e9d32f0ea399ffe95f1ff54797b91093f747 --- /dev/null +++ b/processed_dataset/proof/1074.json @@ -0,0 +1,8 @@ +{ + "source_file": "./raw_volume-zh/volume4/chapter2-1.tex", + "problem_type": "proof", + "problem": "例14. 设 $a_i>0, i=1,2, \\cdots, n, m>0$ 且满足 $\\sum_{i=1}^n \\frac{1}{1+a_i^m}=1$. 求证:\n$$\na_1 a_2 \\cdots a_n \\geqslant(n-1)^{\\frac{n}{m}} .\n$$", + "solution": "证明一令 $x_i=\\frac{1}{1+a_i^m}$, 则 $a_i^m=\\frac{1-x_i}{x_i}$, 且 $x_i>0, i=1,2, \\cdots, n$, $\\sum_{i=1}^n x_i=1$\n于是 $\\quad a_1^m a_2^m \\cdots a_n^m=\\frac{\\left(x_2+\\cdots+x_n\\right) \\cdots\\left(x_1+x_2+\\cdots+x_{n-1}\\right)}{x_1 x_2 \\cdots x_n}$\n$$\n\\begin{aligned}\n& \\geqslant \\frac{(n-1) \\sqrt[n-1]{x_2 x_3 \\cdots x_n} \\cdots(n-1) \\sqrt[n-1]{x_1 x_2 \\cdots x_{n-1}}}{x_1 x_2 \\cdots x_n} \\\\\n& =(n-1)^n\n\\end{aligned}\n$$\n故命题成立.", + "remark": "", + "figures": [] +} \ No newline at end of file diff --git a/processed_dataset/proof/1075.json b/processed_dataset/proof/1075.json new file mode 100644 index 0000000000000000000000000000000000000000..e8674c202362b0b952e891cf4b91d6e917f50f58 --- /dev/null +++ b/processed_dataset/proof/1075.json @@ -0,0 +1,8 @@ +{ + "source_file": "./raw_volume-zh/volume4/chapter2-1.tex", + "problem_type": "proof", + "problem": "例14. 设 $a_i>0, i=1,2, \\cdots, n, m>0$ 且满足 $\\sum_{i=1}^n \\frac{1}{1+a_i^m}=1$. 求证:\n$$\na_1 a_2 \\cdots a_n \\geqslant(n-1)^{\\frac{n}{m}} .\n$$", + "solution": "证明二令 \n$$\n\\begin{gathered}\na_i^m=\\tan ^2 \\alpha_i \\text {, 则 } \\sum_{i=1}^n \\frac{1}{1+a_i^m}=1 \\text { 等价于 } \\\\\n\\sum_{i=1}^n \\cos ^2 \\alpha_i=1 .\n\\end{gathered}\n$$\n其结论等价于\n$$\n\\tan ^2 \\alpha_1 \\tan ^2 \\alpha_2 \\cdots \\cdot \\tan ^2 \\alpha_n \\geqslant(n-1)^n,\n$$\n即\n$$\n\\sin ^2 \\alpha_1 \\sin ^2 \\alpha_2 \\cdots \\sin ^2 \\alpha_n \\geqslant(n-1)^n \\cos ^2 \\alpha_1 \\cos ^2 \\alpha_2 \\cdots \\cos ^2 \\alpha_n\n$$\n由平均值不等式, 得\n$$\n\\begin{aligned}\n\\sin ^2 \\alpha_1 & =1-\\cos ^2 \\alpha_1=\\cos ^2 \\alpha_2+\\cdots+\\cos ^2 \\alpha_n \\\\\n& \\geqslant(n-1) \\sqrt[n-1]{\\cos ^2 \\alpha_2 \\cos ^2 \\alpha_3 \\cdots \\cos ^2 \\alpha_n} .\n\\end{aligned}\n$$\n一般地,\n$$\n\\begin{aligned}\n\\sin ^2 \\alpha_i & =1-\\cos ^2 \\alpha_i=\\cos ^2 \\alpha_1+\\cdots+\\cos ^2 \\alpha_{i-1}+\\cos ^2 \\alpha_{i+1}+\\cdots+\\cos ^2 \\alpha_n \\\\\n& \\geqslant(n-1) \\sqrt[n-1]{\\cos ^2 \\alpha_1 \\cdots \\cos ^2 \\alpha_{i-1} \\cos ^2 \\alpha_{i+1} \\cdots \\cos ^2 \\alpha_n} .\n\\end{aligned}\n$$\n将它们相乘, 得\n$$\n\\sin ^2 \\alpha_1 \\sin ^2 \\alpha_2 \\cdots \\sin ^2 \\alpha_n \\geqslant(n-1)^n \\cos ^2 \\alpha_1 \\cos ^2 \\alpha_2 \\cdots \\cos ^2 \\alpha_n .\n$$\n故命题成立.", + "remark": "", + "figures": [] +} \ No newline at end of file diff --git a/processed_dataset/proof/1076.json b/processed_dataset/proof/1076.json new file mode 100644 index 0000000000000000000000000000000000000000..544571fd96fbee69aec34444d763c6f1f1aa3e48 --- /dev/null +++ b/processed_dataset/proof/1076.json @@ -0,0 +1,8 @@ +{ + "source_file": "./raw_volume-zh/volume4/chapter2-1.tex", + "problem_type": "proof", + "problem": "例15. 设 $a, b, c \\in \\mathbf{R}^{+}$, 且 $a^2+b^2+c^2=1$. 求证:\n$$\n\\frac{a}{1-a^2}+\\frac{b}{1-b^2}+\\frac{c}{1-c^2} \\geqslant \\frac{3 \\sqrt{3}}{2} \\text {. }\n$$", + "solution": "证明:原不等式\n$$\n\\frac{a}{1-a^2}+\\frac{b}{1-b^2}+\\frac{c}{1-c^2} \\geqslant \\frac{3 \\sqrt{3}}{2}\n$$\n等价于 $\\quad \\frac{a^2}{a\\left(1-a^2\\right)}+\\frac{b^2}{b\\left(1-b^2\\right)}+\\frac{c^2}{c\\left(1-c^2\\right)} \\geqslant \\frac{3 \\sqrt{3}}{2}$.\n由于 $a^2+b^2+c^2=1$, 如果能证明 $x\\left(1-x^2\\right) \\leqslant \\frac{2}{3 \\sqrt{3}}$, 则上述不等式成立.\n由平均值不等式, 得\n$$\n\\begin{aligned}\nx\\left(1-x^2\\right) & =\\sqrt{\\frac{2 x^2\\left(1-x^2\\right)\\left(1-x^2\\right)}{2}} \\\\\n& \\leqslant \\sqrt{\\frac{1}{2}\\left[\\frac{2 x^2+\\left(1-x^2\\right)+\\left(1-x^2\\right)}{3}\\right]^3} \\\\\n& =\\sqrt{\\frac{1}{2} \\cdot\\left(\\frac{2}{3}\\right)^3}=\\frac{2}{3 \\sqrt{3}},\n\\end{aligned}\n$$\n故不等式成立.", + "remark": "注:由于分子之和 $a^2+b^2+c^2=1$, 所以当各分母被控制在某个常数之内时, 便可以推出命题成立.\n这个方法在分式不等式证明中常常使用.", + "figures": [] +} \ No newline at end of file diff --git a/processed_dataset/proof/1077.json b/processed_dataset/proof/1077.json new file mode 100644 index 0000000000000000000000000000000000000000..2951a329d2ac0f04b7c16b885babacbf272b1c36 --- /dev/null +++ b/processed_dataset/proof/1077.json @@ -0,0 +1,8 @@ +{ + "source_file": "./raw_volume-zh/volume4/chapter2-1.tex", + "problem_type": "proof", + "problem": "例16. 设 $a_1, a_2, \\cdots, a_n$ 是 $1,2, \\cdots, n$ 的一个排列.\n求证:\n$$\n\\frac{1}{2}+\\frac{2}{3}+\\cdots+\\frac{n-1}{n} \\leqslant \\frac{a_1}{a_2}+\\frac{a_2}{a_3}+\\cdots+\\frac{a_{n-1}}{a_n} .\n$$", + "solution": "证明:因为 $a_1, a_2, \\cdots, a_n$ 是 $1,2, \\cdots, n$ 的一个排列, 所以\n$$\n\\begin{aligned}\n& \\left(1+a_1\\right)\\left(1+a_2\\right) \\cdots\\left(1+a_{n-1}\\right) \\\\\n\\geqslant & (1+1)(1+2) \\cdots[1+(n-1)] \\\\\n= & a_1 a_2 \\cdots a_n .\n\\end{aligned}\n$$\n于是\n$$\n\\begin{aligned}\n& \\frac{a_1}{a_2}+\\frac{a_2}{a_3}+\\cdots+\\frac{a_{n-1}}{a_n}+\\frac{1}{1}+\\frac{1}{2}+\\cdots+\\frac{1}{n} \\\\\n= & \\frac{a_1}{a_2}+\\frac{a_2}{a_3}+\\cdots+\\frac{a_{n-1}}{a_n}+\\frac{1}{a_1}+\\frac{1}{a_2}+\\cdots+\\frac{1}{a_n} \\\\\n= & \\frac{1}{a_1}+\\frac{1+a_1}{a_2}+\\frac{1+a_2}{a_3}+\\cdots+\\frac{1+a_{n-1}}{a_n} \\\\\n\\geqslant & n \\sqrt[n]{\\frac{\\left(1+a_1\\right)\\left(1+a_2\\right) \\cdots\\left(1+a_{n-1}\\right)}{a_1 a_2 \\cdots a_n}} \\geqslant n .\n\\end{aligned}\n$$\n又因为 $n=\\left(1+\\frac{1}{2}+\\frac{1}{3}+\\cdots+\\frac{1}{n}\\right)+\\left(\\frac{1}{2}+\\frac{2}{3}+\\cdots+\\frac{n-1}{n}\\right)$, 所以\n$$\n\\frac{a_1}{a_2}+\\frac{a_2}{a_3}+\\cdots+\\frac{a_{n-1}}{a_n} \\geqslant \\frac{1}{2}+\\frac{2}{3}+\\cdots+\\frac{n-1}{n} .\n$$", + "remark": "注:对于该不等式的证明, 首先要充分理解 $a_1, a_2, \\cdots, a_n$ 是 $1,2, \\cdots, n$ 的一个排列, 此外, 两边同时相加 $1+\\frac{1}{2}+\\frac{1}{3}+\\cdots+\\frac{1}{n}\\left(\\right.$ 即 $\\left.\\frac{1}{a_1}+\\frac{1}{a_2}+\\cdots+\\frac{1}{a_n}\\right)$ 也是很重要的一步.", + "figures": [] +} \ No newline at end of file diff --git a/processed_dataset/proof/1078.json b/processed_dataset/proof/1078.json new file mode 100644 index 0000000000000000000000000000000000000000..f68f74b75bee7601ecfeec89d5006e5e38cbec20 --- /dev/null +++ b/processed_dataset/proof/1078.json @@ -0,0 +1,8 @@ +{ + "source_file": "./raw_volume-zh/volume4/chapter2-1.tex", + "problem_type": "proof", + "problem": "例17. 设 $a, b, c$ 为正实数,求证:\n$$\n\\frac{a}{\\sqrt{a^2+8 b c}}+\\frac{b}{\\sqrt{b^2+8 a c}}+\\frac{c}{\\sqrt{c^2+8 a b}} \\geqslant 1 \\text {. }\n$$", + "solution": "证明:容易看出, 如果我们能证明 $\\frac{a}{\\sqrt{a^2+8 b c}} \\geqslant \\frac{a^{\\frac{4}{3}}}{a^{\\frac{4}{3}}+b^{\\frac{4}{3}}+c^{\\frac{4}{3}}}$, 那么, 将它们相加便得到所要证明的不等式.\n因为\n$$\n\\frac{a}{\\sqrt{a^2+8 b c}} \\geqslant \\frac{a^{\\frac{4}{3}}}{a^{\\frac{4}{3}}+b^{\\frac{4}{3}}+c^{\\frac{4}{3}}},\n$$\n等价于\n$$\n\\left(a^{\\frac{4}{3}}+b^{\\frac{4}{3}}+c^{\\frac{4}{3}}\\right)^2 \\geqslant a^{\\frac{2}{3}}\\left(a^2+8 b c\\right) .\n$$\n再由平均值不等式, 得\n$$\n\\begin{aligned}\n\\left(a^{\\frac{4}{3}}+b^{\\frac{4}{3}}+c^{\\frac{4}{3}}\\right)^2-\\left(a^{\\frac{4}{3}}\\right)^2 & =\\left(b^{\\frac{4}{3}}+c^{\\frac{4}{3}}\\right)\\left(a^{\\frac{4}{3}}+a^{\\frac{4}{3}}+b^{\\frac{4}{3}}+c^{\\frac{4}{3}}\\right) \\\\\n& \\geqslant 2 b^{\\frac{2}{3}} c^{\\frac{2}{3}} \\cdot 4 a^{\\frac{2}{3}} b^{\\frac{1}{3}} c^{\\frac{1}{3}}=8 a^{\\frac{2}{3}} b c .\n\\end{aligned}\n$$\n于是 $\\quad\\left(a^{\\frac{4}{3}}+b^{\\frac{4}{3}}+c^{\\frac{4}{3}}\\right)^2 \\geqslant\\left(a^{\\frac{4}{3}}\\right)^2+8 a^{\\frac{2}{3}} b c=a^{\\frac{2}{3}}\\left(a^2+8 b c\\right)$,\n从而\n$$\n\\frac{a}{\\sqrt{a^2}+8 b c} \\geqslant \\frac{a^{\\frac{4}{3}}}{a^{\\frac{4}{3}}+b^{\\frac{4}{3}}+c^{\\frac{4}{3}}} \\text {. }\n$$\n同理可得\n$$\n\\frac{b}{\\sqrt{b^2+8 a c}} \\geqslant \\frac{b^{\\frac{4}{3}}}{a^{\\frac{4}{3}}+b^{\\frac{4}{3}}+c^{\\frac{4}{3}}}, \\frac{c}{\\sqrt{c^2+8 a b}} \\geqslant \\frac{c^{\\frac{4}{3}}}{a^{\\frac{4}{3}}+b^{\\frac{4}{3}}+c^{\\frac{4}{3}}},\n$$\n于是 $\\quad \\frac{a}{\\sqrt{a^2+8 b c}}+\\frac{b}{\\sqrt{b^2+8 a c}}+\\frac{c}{\\sqrt{c^2+8 a b}} \\geqslant 1$.", + "remark": "注:这是一个 IMO 试题, 有多种不同的证明方法, 后面将再次遇到.\n这里的指数 $\\frac{4}{3}$ 是这样得到的.\n取 $x$ 为待定常数.\n设\n$$\n\\frac{a}{\\sqrt{a^2+8 b c}} \\geqslant \\frac{a^x}{a^x+b^x+c^x},\n$$\n上式等价于\n$$\n\\begin{aligned}\n& a^2\\left(a^x+b^x+c^x\\right)^2 \\geqslant a^{2 x}\\left(a^2+8 b c\\right) \\\\\n\\Leftrightarrow & \\left(a^x+b^x+c^x\\right)^2 \\geqslant a^{2 x-2}\\left(a^2+8 b c\\right) \\\\\n\\Leftrightarrow & a^{2 x}+2 a^x\\left(b^x+c^x\\right)+\\left(b^x+c^x\\right)^2 \\geqslant a^{2 x}+8 a^{2 x-2} b c \\\\\n\\Leftrightarrow & 2 a^x\\left(b^x+c^x\\right)+\\left(b^x+c^x\\right)^2 \\geqslant 8 a^{2 x-2} b c .\n\\end{aligned}\n$$\n由于\n$$\nb^x+c^x \\geqslant 2 b^{\\frac{x}{2}} c^{\\frac{x}{2}},\n$$\n只需\n$$\n2 a^x \\cdot 2 b^{\\frac{x}{2}} c^{\\frac{x}{2}}+\\left(2 b^{\\frac{x}{2}} c^{\\frac{x}{2}}\\right)^2 \\geqslant 8 a^{2 x-2} b c,\n$$\n即\n$$\na^x b^{\\frac{x}{2}} c^{\\frac{x}{2}}+b^x c^x \\geqslant 2 a^{2 x-2} b c .\n$$\n由于\n$$\na^x b^{\\frac{x}{2}} c^{\\frac{x}{2}}+b^x c^x \\geqslant 2 \\sqrt{a^x b^{\\frac{3}{2} x} c^{\\frac{3}{2} x}}=2 a^{\\frac{x}{2}} b^{\\frac{3}{4} x} c^{\\frac{3}{4} x},\n$$\n所以只需\n$$\na^{\\frac{x}{2}} b^{\\frac{3}{4} x} c^{\\frac{3}{4} x} \\geqslant a^{2 x-2} b c,\n$$\n显然取 $x=\\frac{4}{3}$ 满足要求.", + "figures": [] +} \ No newline at end of file diff --git a/processed_dataset/proof/1079.json b/processed_dataset/proof/1079.json new file mode 100644 index 0000000000000000000000000000000000000000..969d5afa8db661998a0aabae715cb0626db0f819 --- /dev/null +++ b/processed_dataset/proof/1079.json @@ -0,0 +1,8 @@ +{ + "source_file": "./raw_volume-zh/volume4/chapter2-1.tex", + "problem_type": "proof", + "problem": "例18. 已知正整数 $n \\geqslant 2$, 实数 $a_i, b_i$, 满足\n$$\na_1 \\geqslant a_2 \\geqslant \\cdots \\geqslant a_n>0, b_1 \\geqslant b_2 \\geqslant \\cdots \\geqslant b_n>0\n$$\n并且\n$$\n\\begin{gathered}\na_1 a_2 \\cdots a_n=b_1 b_2 \\cdots b_n, \\\\\n\\sum_{1 \\leqslant in-1$, 因为\n$$\n\\begin{aligned}\n& \\sum_{1 \\leqslant i2(n-1) b_n+2 b_2+a_1-a_{n-1} \\geqslant 2(n-1) b_n+2 b_2 \\geqslant 2 n b_n,\n$$\n即 $a_n>2 b_n$.\n又由 $a_1 a_2 \\cdots a_n=1$, 得 $a_n \\leqslant 1$, 所以\n$$\na_1-(n-1) a_n>n-1-(n-1)=0,\n$$\n于是 $\\quad 2 b_22 b_n b_2 a_1 a_2 \\cdots a_{n-2},\n$$\n即\n$$\nb_1 b_3 \\cdots b_{n-1}>2 a_1 a_2 \\cdots a_{n-2} \\text {. }\n$$\n而\n$$\n\\begin{aligned}\n& b_3 \\leqslant b_22 a_1,(n-1) \\sum_{i=1}^n b_i>2(n-1) a_1>n a_1 \\geqslant \\sum_{i=1}^n a_i .\n$$", + "remark": "", + "figures": [] +} \ No newline at end of file diff --git a/processed_dataset/proof/1080.json b/processed_dataset/proof/1080.json new file mode 100644 index 0000000000000000000000000000000000000000..28583a041683bd7e6cb9f82e9408138a8f5fd68a --- /dev/null +++ b/processed_dataset/proof/1080.json @@ -0,0 +1,8 @@ +{ + "source_file": "./raw_volume-zh/volume4/chapter2-1.tex", + "problem_type": "proof", + "problem": "例19. 给定 $n \\geqslant 2, n \\in \\mathbf{Z}^{+}$, 求所有 $m \\in \\mathbf{Z}^{+}$, 使得对 $a_i \\in \\mathbf{R}^{+}, i=1,2$, $\\cdots, n$, 满足 $a_1 a_2 \\cdots a_n=1$, 则\n$$\na_1^m+a_2^m+\\cdots+a_n^m \\geqslant \\frac{1}{a_1}+\\frac{1}{a_2}+\\cdots+\\frac{1}{a_n} .\n$$", + "solution": "解:取 $x=a_1=a_2=\\cdots=a_{n-1}>0, a_n=\\frac{1}{x^{n-1}}$, 则\n$$\n(n-1) x^m+\\frac{1}{x^{(n-1) m}} \\geqslant \\frac{n-1}{x}+x^{n-1} .\n$$\n由此得到 $m \\geqslant n-1$. 现在, 假设 $m \\geqslant n-1$, 则\n$$\n\\begin{aligned}\n& (n-1)\\left(a_1^m+a_2^m+\\cdots+a_n^m\\right)+n(m-n+1) \\\\\n= & \\left(a_1^m+a_2^m+\\cdots+a_{n-1}^m+1+1+\\cdots+1\\right)(\\text { 共 } m-n+1 \\text { 个 } 1) \\\\\n& +\\left(a_1^m+a_2^m+\\cdots+a_{n-2}^m+a_n^m+1+1+\\cdots+1\\right)+\\cdots\n\\end{aligned}\n$$\n$$\n\\begin{aligned}\n& +\\left(a_2^m+a_3^m+\\cdots+a_n^m+1+1+\\cdots+1\\right) \\\\\n\\geqslant & m \\sqrt[m]{\\left(a_1 a_2 \\cdots a_{n-1}\\right)^m}+\\cdots+m \\sqrt[m]{\\left(a_2 \\cdots a_n\\right)^m} \\\\\n= & m\\left(a_1 a_2 \\cdots a_{n-1}+\\cdots+a_2 a_3 \\cdots a_n\\right)=m\\left(\\frac{1}{a_1}+\\frac{1}{a_2}+\\cdots+\\frac{1}{a_n}\\right) .\n\\end{aligned}\n$$\n所以\n$$\na_1^m+a_2^m+\\cdots+a_n^m \\geqslant \\frac{m}{n-1}\\left(\\frac{1}{a_1}+\\frac{1}{a_2}+\\cdots+\\frac{1}{a_n}\\right)-\\frac{n}{n-1}(m-n+1) .\n$$\n于是, 只要证明\n$$\n\\begin{gathered}\n\\frac{m}{n-1}\\left(\\frac{1}{a_1}+\\frac{1}{a_2}+\\cdots+\\frac{1}{a_n}\\right)-\\frac{n}{n-1}(m-n+1) \\geqslant \\frac{1}{a_1}+\\frac{1}{a_2}+\\cdots+\\frac{1}{a_n}, \\\\\n(m-n+1)\\left(\\frac{1}{a_1}+\\frac{1}{a_2}+\\cdots+\\frac{1}{a_n}-n\\right) \\geqslant 0 .\n\\end{gathered}\n$$\n即\n$$\n(m-n+1)\\left(\\frac{1}{a_1}+\\frac{1}{a_2}+\\cdots+\\frac{1}{a_n}-n\\right) \\geqslant 0 .\n$$\n由假设以及平均值不等式, 得\n$$\n\\left(\\frac{1}{a_1}+\\frac{1}{a_2}+\\cdots+\\frac{1}{a_n}-n\\right) \\geqslant n \\sqrt[n]{\\frac{1}{a_1} \\cdot \\frac{1}{a_2} \\cdot \\cdots \\cdot \\frac{1}{a_n}}-n=0,\n$$\n所以原不等式成立.\n故对所有满足 $m \\geqslant n-1$ 的 $m \\in \\mathbf{Z}^{+}$均可.", + "remark": "", + "figures": [] +} \ No newline at end of file diff --git a/processed_dataset/proof/1081.json b/processed_dataset/proof/1081.json new file mode 100644 index 0000000000000000000000000000000000000000..9469ca6e05df186df4551d8d96de5322636323da --- /dev/null +++ b/processed_dataset/proof/1081.json @@ -0,0 +1,8 @@ +{ + "source_file": "./raw_volume-zh/volume4/chapter2-1.tex", + "problem_type": "proof", + "problem": "例20. 设 $n(n \\geqslant 2)$ 是整数, $a_1, a_2, \\cdots, a_n \\in \\mathbf{R}^{+}$, 求证:\n$$\n\\left(a_1^3+1\\right)\\left(a_2^3+1\\right) \\cdots\\left(a_n^3+1\\right) \\geqslant\\left(a_1^2 a_2+1\\right)\\left(a_2^2 a_3+1\\right) \\cdots\\left(a_n^2 a_1+1\\right) .\n$$", + "solution": "证明:先证对于正实数 $x_i, y_i(i=1,2,3)$, 有\n$$\n\\prod\\left(x_i^3+y_i^3\\right) \\geqslant\\left(\\prod x_i+\\prod y_i\\right)^3 .\n$$\n实际上, 由平均值不等式, 得\n$$\n\\begin{aligned}\n& \\sqrt[3]{\\frac{x_1^3 x_2^3 x_3^3}{\\prod\\left(x_i^3+y_i^3\\right)}} \\leqslant \\frac{1}{3}\\left(\\sum_{i=1}^3 \\frac{x_i^3}{x_i^3+y_i^3}\\right), \\\\\n& \\sqrt[3]{\\frac{y_1^3 y_2^3 y_3^3}{\\prod\\left(x_i^3+y_i^3\\right)}} \\leqslant \\frac{1}{3}\\left(\\sum_{i=1}^3 \\frac{y_i^3}{x_i^3+y_i^3}\\right),\n\\end{aligned}\n$$\n所以\n$$\n\\sqrt[3]{\\frac{x_1^3 x_2^3 x_3^3}{\\prod\\left(x_i^3+y_i^3\\right)}}+\\sqrt[3]{\\frac{y_1^3 y_2^3 y_3^3}{\\prod\\left(x_i^3+y_i^3\\right)}} \\leqslant 1,\n$$\n即\n$$\n\\Pi\\left(x_i^3+y_i^3\\right) \\geqslant\\left(\\prod x_i+\\prod y_i\\right)^3 .\n$$\n$$\n\\text { 令 } x_1=x_2=a_k, x_3=a_{k+1}, a_{n+1}=a_1, y_1=y_2=y_3=1, k=1,2, \\cdots \\text {, }\n$$\n$n$, 则\n$$\n\\left(a_k^3+1\\right)^2\\left(a_{k+1}^3+1\\right) \\geqslant\\left(a_k^2 a_{k+1}+1\\right)^3, k=1,2, \\cdots, n .\n$$\n将它们相乘, 则\n$$\n\\prod\\left(a_i^3+1\\right)^3 \\geqslant \\prod\\left(a_i^2 a_{i+1}+1\\right)^3,\n$$\n故\n$$\n\\left(a_1^3+1\\right)\\left(a_2^3+1\\right) \\cdots\\left(a_n^3+1\\right) \\geqslant\\left(a_1^2 a_2+1\\right)\\left(a_2^2 a_3+1\\right) \\cdots\\left(a_n^2 a_1+1\\right) .\n$$", + "remark": "", + "figures": [] +} \ No newline at end of file diff --git a/processed_dataset/proof/1082.json b/processed_dataset/proof/1082.json new file mode 100644 index 0000000000000000000000000000000000000000..c3746969a53c6e6f6a5d24b4d17d79218dcc0574 --- /dev/null +++ b/processed_dataset/proof/1082.json @@ -0,0 +1,8 @@ +{ + "source_file": "./raw_volume-zh/volume4/chapter2-1.tex", + "problem_type": "proof", + "problem": "例21. 设 $a, b, c>0, a+b+c=1$. 证明: 若正实数 $x_1, x_2, \\cdots, x_5$ 满足 $x_1 x_2 \\cdots x_5=1$, 则\n$$\n\\prod_{i=1}^5\\left(a x_i^2+b x_i+c\\right) \\geqslant 1 .\n$$", + "solution": "证明:将问题一般化.\n考虑表达式 $\\prod_{i=1}^n\\left(a x_i^2+b x_i+c\\right)$.\n则含 $a^i b^j c^k(i+j+k=n)$ 的项为\n$$\na^i b^j c^k\\left[\\left(x_1 x_2 \\cdots x_i\\right)^2\\left(x_{i+1} x_{i+2} \\cdots x_{i+j}\\right)+\\cdots\\right] .\n$$\n因此,共有 $\\mathrm{C}_n^i \\mathrm{C}_{n-i}^j$ 项求和 (取 $a x_t^2$ 的 $i$ 个因式, 从剩余的 $n-i$ 个因式中, 取 $b x_s$ 中的 $j$ 个因式).\n由对称性知,含 $x_i^2$ 的项数为常数, 即为含 $x_i$ 的项数.\n因此,当求和后的项是全部的乘积时,对某个 $p$ 有 $\\left(x_1 x_2 \\cdots x_n\\right)^p=1$.", + "remark": "注:对于本题, 不必求出 $p$. 事实上,\n$$\np=2 \\mathrm{C}_{n-1}^{i-1} \\mathrm{C}_{n-i}^j+\\mathrm{C}_{n-1}^{j-1} \\mathrm{C}_{n-j}^i=\\frac{2 i+j}{n} \\mathrm{C}_n^i \\mathrm{C}_{n-i}^j .\n$$\n回到原题;由代数一几何均值不等式得\n$$\na^i b^j c^k\\left[\\left(x_1 x_2 \\cdots x_i\\right)^2\\left(x_{i+1} x_{i+2} \\cdots x_{i+j}\\right)+\\cdots\\right] \\geqslant a^i b^j c^k \\mathrm{C}_n^i \\mathrm{C}_{n-i}^j .\n$$\n故 $\\prod_{i=1}^n\\left(a x_i^2+b x_i+c\\right) \\geqslant \\sum_{i+j+k=n} a^i b^j c^k \\mathrm{C}_n^i \\mathrm{C}_{n-i}^j=(a+b+c)^n=1$ :", + "figures": [] +} \ No newline at end of file diff --git a/processed_dataset/proof/1083.json b/processed_dataset/proof/1083.json new file mode 100644 index 0000000000000000000000000000000000000000..253a5992aad7f01e62eb6101a3821897d45529c1 --- /dev/null +++ b/processed_dataset/proof/1083.json @@ -0,0 +1,8 @@ +{ + "source_file": "./raw_volume-zh/volume4/chapter2-1.tex", + "problem_type": "proof", + "problem": "例22. 已知 $x, y, z \\in \\mathbf{R}^{+} \\cup\\{0\\}$, 且 $x+y+z=2$.", + "solution": "证明: $x^2 y^2+y^2 z^2+z^2 x^2+x y z \\leqslant 1$, 并求上式取等号时, $x 、 y 、 z$ 的值.\n证明注意到\n$$\n\\begin{aligned}\n& x^2 y^2+y^2 z^2+z^2 x^2+x y z \\\\\n= & \\frac{1}{2}\\left(2 x^2 y^2+2 y^2 z^2+2 z^2 x^2+2 x y z\\right) \\\\\n= & \\frac{1}{2}(x y \\cdot 2 x y+y z \\cdot 2 y z+z x \\cdot 2 z x+2 x y z) \\\\\n\\leqslant & \\frac{1}{2}\\left[x y\\left(x^2+y^2\\right)+y z\\left(y^2+z^2\\right)+z x\\left(z^2+x^2\\right)+2 x y z\\right] . \\label{(16)} \\\\\n= & \\frac{1}{2}\\left[(x y+y z+z x)\\left(x^2+y^2+z^2\\right)-x y z^2-y z x^2-z x y^2+2 x y z\\right] \\\\\n= & \\frac{1}{2}\\left[(x y+y z+z x)\\left(x^2+y^2+z^2\\right)-x y z(x+y+z-2)\\right] \\\\\n= & \\frac{1}{2}(x y+y z+z x)\\left(x^2+y^2+z^2\\right) .\n\\end{aligned}\n$$\n由此得到\n$$\nx^2 y^2+y^2 z^2+z^2 x^2+x y z \\leqslant \\frac{1}{2}\\left[(x y+y z+z x)\\left(x^2+y^2+z^2\\right)\\right] . \\label{(17)}\n$$\n由式(16)知,当 $x=y=z$ 或 $x=y, z=0$ 或 $y=z, x=0$ 或 $z=x$, $=0$ 时, 式(17)取等号.\n又 $x+y+z=2$, 因此, 当\n$(x, y, z)=\\left(\\frac{2}{3}, \\frac{2}{3}, \\frac{2}{3}\\right)$ 或 $(1,1,0)$ 或 $(1,0,1)$ 或 $(0,1,1)$ 时,式(17)取等号.\n运用常见不等式\n$$\n\\alpha \\beta \\leqslant\\left(\\frac{\\alpha+\\beta}{2}\\right)^2(\\alpha, \\beta \\in \\mathbf{R}) .\n$$\n令 $\\alpha=2 x y+2 y z+2 z x, \\beta=x^2+y^2+z^2$. 则\n$$\n\\begin{aligned}\n& \\frac{1}{2}(x y+y z+z x)\\left(x^2+y^2+z^2\\right) \\\\\n= & \\frac{1}{4}(2 x y+2 y z+2 z x)\\left(x^2+y^2+z^2\\right) \\\\\n\\leqslant & \\frac{1}{4}\\left(\\frac{2 x y+2 y z+2 z x+x^2+y^2+z^2}{2}\\right)^2 \\\\\n=\\frac{1}{16}(x+y+z)^4=1 . \\label{(18)}\n\\end{aligned}\n$$\n结合式(17)得\n$$\nx^2 y^2+y^2 z^2+z^2 x^2+x y z \\leqslant 1 . \\label{(19)}\n$$\n由式(18)取等号的条件知, 当\n$$\n\\alpha=\\beta \\Leftrightarrow 2 x y+2 y z+2 z x=x^2+y^2+z^2\n$$\n时,式(19)等号成立.\n故 $(x, y, z)=(1,1,0)$ 或 $(1,0,1)$ 或 $(0,1,1)$.", + "remark": "", + "figures": [] +} \ No newline at end of file diff --git a/processed_dataset/proof/1084.json b/processed_dataset/proof/1084.json new file mode 100644 index 0000000000000000000000000000000000000000..01ce1e73cc1e8948f8bb7ecf25128e0ac60e1ee2 --- /dev/null +++ b/processed_dataset/proof/1084.json @@ -0,0 +1,8 @@ +{ + "source_file": "./raw_volume-zh/volume4/chapter2-1.tex", + "problem_type": "proof", + "problem": "例23. 已知 $a 、 b 、 c$ 为正实数.\n证明:\n$$\n\\frac{a^2 b(b-c)}{a+b}+\\frac{b^2 c(c-a)}{b+c}+\\frac{c^2 a(a-b)}{c+a} \\geqslant 0 .\n$$", + "solution": "证明:$$\n\\begin{aligned}\n\\text { 原式 } & \\Leftrightarrow \\frac{a^2 b^2}{a+b}+\\frac{b^2 c^2}{b+c}+\\frac{c^2 a^2}{c+a} \\geqslant a b c\\left(\\frac{a}{a+b}+\\frac{b}{b+c}+\\frac{c}{c+a}\\right) \\\\\n& \\Leftrightarrow \\frac{a b}{c(a+b)}+\\frac{b c}{a(b+c)}+\\frac{a c}{b(c+a)} \\geqslant \\frac{a}{a+b}+\\frac{b}{b+c}+\\frac{c}{c+a} \\\\\n& \\Leftrightarrow(a b+b c+a c)\\left(\\frac{1}{a c+b c}+\\frac{1}{a b+a c}+\\frac{1}{b c+a b}\\right) \\\\\n& \\geqslant \\frac{a c}{a c+b c}+\\frac{a b}{a b+a c}+\\frac{b c}{b c+a b}+3 .\n\\end{aligned}\n$$\n下面进行换元.\n令\n$$\n\\left\\{\\begin{array}{l}\nx=a b+a c, \\\\\ny=b c+b a \\\\\nz=c a+c b\n\\end{array},\\left\\{\\begin{array}{l}\na c=\\frac{x+z-y}{2}, \\\\\na b=\\frac{x+y-z}{2}, \\\\\nb c=\\frac{y+z-x}{2}\n\\end{array} \\Rightarrow a b+b c+c a=\\frac{x+y+z}{2} .\\right.\\right.\n$$\n故\n$$\n\\begin{aligned}\n\\text { 原式 } & \\Leftrightarrow \\frac{1}{2}(x+y+z)\\left(\\frac{1}{x}+\\frac{1}{y}+\\frac{1}{z}\\right) \\geqslant \\frac{x+z-y}{2 z}+\\frac{x+y-z}{2 x}+\\frac{y+z-x}{2 y}+3 \\\\\n& \\Leftrightarrow(x+y+z)\\left(-\\frac{1}{x}+\\frac{1}{y}+\\frac{1}{z}\\right) \\geqslant \\frac{x-y}{z}+\\frac{y-z}{x}+\\frac{z-x}{y}+9\n\\end{aligned}\n$$\n$$\n\\begin{aligned}\n& \\Leftrightarrow 3+\\frac{y}{x}+\\frac{z}{x}+\\frac{x}{y}+\\frac{z}{y}+\\frac{x}{z}+\\frac{y}{z} \\geqslant \\frac{x-y}{z}+\\frac{y-z}{x}+\\frac{z-x}{y}+9 \\\\\n& \\Leftrightarrow \\frac{2 y}{z}+\\frac{2 z}{x}+\\frac{2 x}{y} \\geqslant 6 \\\\\n& \\Leftrightarrow \\frac{y}{z}+\\frac{z}{x}+\\frac{x}{y} \\geqslant 3 .\n\\end{aligned}\n$$\n由均值不等式即知结论成立.", + "remark": "", + "figures": [] +} \ No newline at end of file diff --git a/processed_dataset/proof/1085.json b/processed_dataset/proof/1085.json new file mode 100644 index 0000000000000000000000000000000000000000..123418783cb8a4879e076cd442c75a5e67dea27b --- /dev/null +++ b/processed_dataset/proof/1085.json @@ -0,0 +1,8 @@ +{ + "source_file": "./raw_volume-zh/volume4/chapter2-1.tex", + "problem_type": "proof", + "problem": "例24. 设正实数 $x_1, x_2, \\cdots, x_n$ 满足 $x_1 x_2 \\cdots x_n=1$. 证明:\n$$\n\\sum_{i=1}^n \\frac{1}{n-1+x_i} \\leqslant 1 \\text {. }\n$$", + "solution": "证明:用反证法.\n假设\n$$\n\\sum_{i=1}^n \\frac{1}{n-1+x_i}>1 . \\label{(20)}\n$$\n则对任意的 $k(k \\in\\{1,2, \\cdots, n\\})$, 由式(20)有\n$$\n\\begin{aligned}\n& \\frac{1}{n-1+x_k}>1-\\sum_{\\substack{1 \\leqslant i \\leqslant n \\\\\ni \\neq k}} \\frac{1}{n-1+x_i} \\\\\n&=\\sum_{\\substack{1 \\leqslant i \\leqslant n \\\\\ni \\neq k}}\\left(\\frac{1}{n-1}-\\frac{1}{n-1+x_i}\\right) \\\\\n&=\\sum_{\\substack{1 \\leqslant i \\leqslant n \\\\\ni \\neq k}} \\frac{x_i}{(n-1)\\left(n-1+x_i\\right)} \\\\\n& \\geqslant(n-1)\\left[\\prod_{\\substack{1 \\leqslant i \\leqslant n \\\\\ni \\neq k}} \\frac{x_i}{(n-1)\\left(n-1+x_i\\right)}\\right]^{\\frac{1}{n-1}} \\\\\n&=\\left(\\prod_{\\substack{1 \\leqslant i \\leqslant n \\\\\ni \\neq k}} \\frac{x_i}{n-1+x_i}\\right)^{\\frac{1}{n-1}},\n\\end{aligned}\n$$\n即对 $1 \\leqslant k \\leqslant n$, 均有\n$$\n\\frac{1}{n-1+x_k}>\\left(\\prod_{\\substack{1 \\leqslant i \\leqslant n \\\\ i \\neq k}} \\frac{x_i}{n-1+x_i}\\right)^{\\frac{1}{n-1}}\n$$\n取积得\n$$\n\\prod_{k=1}^n \\frac{1}{n-1+x_k}>\\prod_{k=1}^n\\left(\\prod_{\\substack{1 \\leqslant i \\leqslant n \\\\ i \\neq k}} \\frac{x_i}{n-1+x_i}\\right)^{\\frac{1}{n-1}}=\\prod_{k=1}^n \\frac{x_k}{n-1+x_k} .\n$$\n则 $\\prod_{k=1}^n x_k<1$, 这与 $\\prod_{k=1}^n x_k=1$ 矛盾.\n所以,假设不成立, 必有\n$$\n\\sum_{i=1}^n \\frac{1}{n-1+x_i} \\leqslant 1 .\n$$", + "remark": "", + "figures": [] +} \ No newline at end of file diff --git a/processed_dataset/proof/1086.json b/processed_dataset/proof/1086.json new file mode 100644 index 0000000000000000000000000000000000000000..2e33db7bb51b36a2f3760ec8e7be89a4304df6f2 --- /dev/null +++ b/processed_dataset/proof/1086.json @@ -0,0 +1,8 @@ +{ + "source_file": "./raw_volume-zh/volume4/chapter2-1.tex", + "problem_type": "proof", + "problem": "例26. 设 $x, y, z \\in \\mathbf{R}^{+}$, 求证:\n$$\n\\frac{x y}{z}+\\frac{y z}{x}+\\frac{z x}{y}>2 \\sqrt[3]{x^3+y^3+z^3} .\n$$", + "solution": "证明:欲证的不等式等价于\n$$\n\\begin{aligned}\n& \\left(\\frac{x y}{z}+\\frac{y z}{x}+\\frac{z x}{y}\\right)^3>8\\left(x^3+y^3+z^3\\right) \\\\\n\\Leftrightarrow & \\left(\\frac{x y}{z}\\right)^3+\\left(\\frac{y z}{x}\\right)^3+\\left(\\frac{z x}{y}\\right)^3+6 x y z+3 x^3\\left(\\frac{y}{z}+\\frac{z}{y}\\right) \\\\\n& +3 y^3\\left(\\frac{x}{z}+\\frac{z}{x}\\right)+3 z^3\\left(\\frac{y}{x}+\\frac{x}{y}\\right) \\\\\n> & 8\\left(x^3+y^3+z^3\\right) .\n\\end{aligned}\n$$\n因为 $\\frac{y}{z}+\\frac{z}{y} \\geqslant 2, \\frac{x}{z}+\\frac{z}{x} \\geqslant 2, \\frac{y}{x}+\\frac{x}{y} \\geqslant 2$, 所以只需证\n$$\n\\left(\\frac{x y}{z}\\right)^3+\\left(\\frac{y z}{x}\\right)^3+\\left(\\frac{z x}{y}\\right)^3+6 x y z>2\\left(x^3+y^3+z^3\\right) .\n$$\n不妨设 $x \\geqslant y \\geqslant z$, 记 $f(x, y, z)=\\left(\\frac{x y}{z}\\right)^3+\\left(\\frac{y z}{x}\\right)^3+\\left(\\frac{z x}{y}\\right)^3+6 x y z- 2\\left(x^3+y^3+z^3\\right)$, 下证 $f(x, y, z)-f(y, y, z) \\geqslant 0, f(y, y, z) \\geqslant 0$.\n事实上,\n$$\n\\begin{aligned}\n& f(x, y, z)-f(y, y, z) \\\\\n= & \\left(\\frac{x y}{z}\\right)^3+\\left(\\frac{y z}{x}\\right)^3+\\left(\\frac{z x}{y}\\right)^3+6 x y z-2\\left(x^3+y^3+z^3\\right) \\\\\n& -\\left[\\left(\\frac{y^2}{z}\\right)^3+z^3+z^3+6 y^2 z-2\\left(y^3+y^3+z^3\\right)\\right] \\\\\n= & \\left(\\frac{x y}{z}\\right)^3-\\frac{y^6}{z^3}+\\left(\\frac{y z}{x}\\right)^3+\\left(\\frac{z x}{y}\\right)^3-2 z^3+6 y z(x-y)-2\\left(x^3-y^3\\right) \\\\\n= & \\left(x^3-y^3\\right)\\left(\\frac{y^3}{z^3}+\\frac{z^3}{y^3}-2+\\frac{6 y z}{x^2+x y+y^2}-\\frac{z^3}{x^3}\\right),\n\\end{aligned}\n$$\n而 $x^3-y^3 \\geqslant 0, \\frac{y^3}{z^3}+\\frac{z^3}{y^3} \\geqslant 2, \\frac{6 y z}{x^2+x y+y^2}-\\frac{z^3}{x^3} \\geqslant \\frac{2 y z}{x^2}-\\frac{z^3}{x^3}=\\frac{z\\left(2 x y-z^2\\right)}{x^3}>$ 0 , 所以 $f(x, y, z)-f(y, y, z) \\geqslant 0$.\n又\n$$\nf(y, y, z)=\\left(\\frac{y^2}{z}\\right)^3+z^3+z^3+6 y^2 z-2\\left(y^3+y^3+z^3\\right) . \\label{(21)}\n$$\n$$\n\\begin{aligned}\n& =\\frac{y^6}{z^3}+2 y^2 z+2 y^2 z+2 y^2 z-4 y^3 \\\\\n& \\geqslant 4 \\sqrt[4]{2^3 y^{12}}-4 y^3=4(\\sqrt[4]{8}-1) y^3>0,\n\\end{aligned}\n$$\n从而 (21) 式得证, 原命题得证.", + "remark": "", + "figures": [] +} \ No newline at end of file diff --git a/processed_dataset/proof/1087.json b/processed_dataset/proof/1087.json new file mode 100644 index 0000000000000000000000000000000000000000..5e728eeb84b3ef4eba6c2d5e89e43cc15325c216 --- /dev/null +++ b/processed_dataset/proof/1087.json @@ -0,0 +1,8 @@ +{ + "source_file": "./raw_volume-zh/volume4/chapter2-1.tex", + "problem_type": "proof", + "problem": "例27. 设 $x 、 y 、 z$ 为非负实数,且 $x+y+z=1$, 求证:\n$$\nx y+y z+z x-2 x y z \\leqslant \\frac{7}{27} .\n$$", + "solution": "证明:不妨设 $x \\geqslant y \\geqslant z$.\n当 $x \\geqslant \\frac{1}{2}$ 时,则 $y z-2 x y z \\leqslant 0$, 所以\n$$\nx y+y z+z x-2 x y z \\leqslant x y+z x=x(1-x) \\leqslant \\frac{1}{4}<\\frac{7}{27} .\n$$\n当 $x<\\frac{1}{2}$ 时, 则 $y \\leqslant \\frac{1}{2}, z \\leqslant \\frac{1}{2}$.\n$$\n(1-2 x)(1-2 y)(1-2 z)=1-2+4(x y+y z+z x)-8 x y z .\n$$\n又由平均值不等式, 得\n$$\n\\begin{gathered}\n(1-2 x)(1-2 y)(1-2 z) \\leqslant\\left[\\frac{3-2(x+y+z)}{3}\\right]^3=\\frac{1}{27}, \\\\\nx y+y z+z x-2 x y z \\leqslant \\frac{1}{4}\\left(\\frac{1}{27}+1\\right)=\\frac{7}{27} .\n\\end{gathered}\n$$\n从而\n$$\nx y+y z+z x-2 x y z \\leqslant \\frac{1}{4}\\left(\\frac{1}{27}+1\\right)=\\frac{7}{27} .\n$$", + "remark": "", + "figures": [] +} \ No newline at end of file diff --git a/processed_dataset/proof/1088.json b/processed_dataset/proof/1088.json new file mode 100644 index 0000000000000000000000000000000000000000..e823b43cfc81548f3cd4d4e4afc1cc185f36c76a --- /dev/null +++ b/processed_dataset/proof/1088.json @@ -0,0 +1,8 @@ +{ + "source_file": "./raw_volume-zh/volume4/chapter2-1.tex", + "problem_type": "proof", + "problem": "例28. 设 $n$ 为正整数, $\\left(x_1, x_2, \\cdots, x_n\\right),\\left(y_1, y_2, \\cdots, y_n\\right)$ 为两个正数数列.\n假设正实数列 $\\left(z_1, z_2, \\cdots, z_{2 n}\\right)$, 满足\n$$\nz_{i+j}^2 \\geqslant x_i y_j, 1 \\leqslant i, j \\leqslant n .\n$$\n令 $M=\\max \\left\\{z_1, z_2, \\cdots, z_{2 n}\\right\\}$, 证明:\n$$\n\\left(\\frac{M+z_2+z_3+\\cdots+z_{2 n}}{2 n}\\right)^2 \\geqslant\\left(\\frac{x_1+x_2+\\cdots+x_n}{n}\\right)\\left(\\frac{y_1+y_2+\\cdots+y_n}{n}\\right) .\n$$", + "solution": "证明:令 $X=\\max \\left\\{x_1, x_2, \\cdots, x_n\\right\\}, Y=\\max \\left\\{y_1, y_2, \\cdots, y_n\\right\\}$, 不妨假设 $X=Y=1$ (否则用 $a_i=\\frac{x_i}{X}, b_i=\\frac{y_i}{Y}, c_i=\\frac{z_i}{\\sqrt{X Y}}$ 代替).\n我们将证明\n$$\nM+z_2+z_3+\\cdots+z_{2 n} \\geqslant x_1+x_2+\\cdots+x_n+y_1+y_2+\\cdots+y_n .\n$$\n于是\n$$\n\\frac{M+z_2+z_3+\\cdots+z_{2 n}}{2 n} \\geqslant \\frac{1}{2}\\left(\\frac{x_1+x_2+\\cdots+x_n}{n}+\\frac{y_1+y_2+\\cdots+y_n}{n}\\right) .\n$$\n由平均值不等式得到原不等式成立.\n为了证明上述不等式, 我们将证明: 对任意 $r>0$, 左边大于 $r$ 的项数不小于右边相应的项数, 那么, 对每个 $k$, 左边第 $k$ 个最大的项大于或等于右边第 $k$ 个最大的项,这样就证明了上述不等式成立.\n证明如下:\n如果 $r \\geqslant 1$, 则右边没有项大于 $r$, 所以只考虑 $r<1$.\n$$\n\\text { 令 } A=\\left\\{x_i \\mid x_i>r, 1 \\leqslant i \\leqslant n\\right\\}, a=|A|, B=\\left\\{y_i \\mid y_i>r, 1 \\leqslant i \\leqslant n\\right\\}, b=|B|\n$$\n由于 $X=Y=1$, 所以 $a, b$ 大于 0 .\n由于 $x_i>r, y_j>r$ 推出 $z_{i+j} \\geqslant \\sqrt{x_i y_j}>r$. 于是\n$$\nA+B=\\{\\alpha+\\beta \\mid \\alpha \\in A, \\beta \\in B\\} \\subseteq C=\\left\\{z_i \\mid z_i>r, 2 \\leqslant i \\leqslant 2 n\\right\\} .\n$$\n但是, 由于如果 $A=\\left\\{i_1, i_2, \\cdots, i_a\\right\\}, i_1r$, 那么 $M>r$. 所以上式的左边至少有 $a+b$ 项大于 $r$, 由于 $a+b$ 为右边大于 $r$ 的项数, 于是, 上述不等式成立.", + "remark": "注:在证明的过程中, 其实平均值不等式的作用是较小的.\n本题的证明有一定的难度, 是因为像这样证明不等式的方法和处理技巧并不多见.\n此解答由第 51 届 IMO 金牌获得者李嘉伦给出.", + "figures": [] +} \ No newline at end of file diff --git a/processed_dataset/proof/1089.json b/processed_dataset/proof/1089.json new file mode 100644 index 0000000000000000000000000000000000000000..c6098443729dc9ffc00e33d153185a7c170c0c3a --- /dev/null +++ b/processed_dataset/proof/1089.json @@ -0,0 +1,8 @@ +{ + "source_file": "./raw_volume-zh/volume4/chapter2-2.tex", + "problem_type": "proof", + "problem": "例4. 若 $x 、 y 、 z$ 是正实数, 求 $\\frac{x y z}{(1+5 x)(4 x+3 y)(5 y+6 z)(z+18)}$ 的最大值, 并证明你的结论.", + "solution": "解:在取定 $y$ 的情况下,\n$$\n\\begin{aligned}\n& \\frac{x}{(1+5 x)(4 x+3 y)} \\\\\n= & \\frac{x}{20 x^2+(15 y+4) x+3 y} \\\\\n= & \\frac{1}{20 x+\\frac{3 y}{x}+15 y+4} \\\\\n\\leqslant & \\frac{1}{2 \\sqrt{20 \\times 3 y}+15 y+4} \\\\\n= & \\frac{1}{(\\sqrt{15 y}+2)^2},\n\\end{aligned}\n$$\n当且仅当 $x=\\sqrt{\\frac{3 y}{20}}$ 时,等号成立.\n同理可得,\n$$\n\\frac{z}{(5 y+6 z)(z+18)} \\leqslant \\frac{1}{2 \\sqrt{6 \\times 90} y+5 y+108}=\\frac{1}{(\\sqrt{5 y}+6 \\sqrt{3})^2},\n$$\n当且仅当 $z=\\sqrt{15 y}$ 时, 等号成立.\n所以,\n$$\n\\begin{aligned}\n& \\frac{x y z}{(1+5 x)(4 x+3 y)(5 y+6 z)(z+18)} \\\\\n& \\leqslant \\frac{y}{(\\sqrt{15 y}+2)^2(\\sqrt{5 y}+6 \\sqrt{3})^2} \\\\\n& =\\left[\\frac{\\sqrt{y}}{(\\sqrt{15 y}+2)(\\sqrt{5 y}+6 \\sqrt{3})}\\right]^2 \\text {. } \\\\\n& =\\left[\\frac{1}{5 \\sqrt{3 y}+\\frac{12 \\sqrt{3}}{\\sqrt{y}}+20 \\sqrt{5}}\\right]^2 \\text {. } \\\\\n& \\leqslant\\left[\\frac{1}{2 \\sqrt{5 \\sqrt{3} \\times 12 \\sqrt{3}}+20 \\sqrt{5}}\\right]^2 \\\\\n& =\\left(\\frac{1}{32 \\sqrt{5}}\\right)^2=\\frac{1}{5120} \\text {, } \\\\\n&\n\\end{aligned}\n$$\n当且仅当 $x=\\frac{3}{5}, y=\\frac{12}{5}, z=6$ 时, 上式取得最大值 $\\frac{1}{5120}$.", + "remark": "", + "figures": [] +} \ No newline at end of file diff --git a/processed_dataset/proof/1090.json b/processed_dataset/proof/1090.json new file mode 100644 index 0000000000000000000000000000000000000000..4f26d34288d28c2c772ebb10b377f91512e04cca --- /dev/null +++ b/processed_dataset/proof/1090.json @@ -0,0 +1,8 @@ +{ + "source_file": "./raw_volume-zh/volume4/chapter2-2.tex", + "problem_type": "proof", + "problem": "例5. 若对于任何正实数, $\\frac{a^2}{\\sqrt{a^4+3 b^4+3 c^4}}+\\frac{k}{a^3} \\cdot\\left(\\frac{c^4}{b}+\\frac{b^4}{c}\\right) \\geqslant \\frac{2 \\sqrt{2}}{3}$. 均成立, 求实数 $k$ 的最小值.", + "solution": "解:$$\n\\begin{aligned}\n\\frac{a^2}{\\sqrt{a^4+3 b^4+3 c^4}} & =\\frac{\\sqrt{2} a^4}{\\sqrt{2 a^4\\left(a^4+3 b^4+3 c^4\\right)}} . \\\\\n& \\geqslant \\frac{\\sqrt{2} a^4}{\\frac{1}{2}\\left[2 a^4+\\left(a^4+3 b^4+3 c^4\\right)\\right]} \\\\\n& =\\frac{2 \\sqrt{2}}{3} \\cdot \\frac{a^4}{a^4+b^4+c^4} .\n\\end{aligned}\n$$\n从形式上猜测, 须证明 $\\frac{k}{a^3} \\cdot\\left(\\frac{c^4}{b}+\\frac{b^4}{c}\\right) \\geqslant \\frac{2 \\sqrt{2}}{3} \\cdot \\frac{b^4+c^4}{a^4+b^4+c^4}$, 又从等号成立的条件 $2 a^4=a^4+3 b^4+3 c^4$ 以及 $b, c$ 的对称性, 猜测 $k$ 可能在 $a^4=6 b^4= 6 c^4$ 时取到尽可能大的值, 而该值即为使不等式对任意 $a, b, c$ 成立的最小值.\n令 $a=\\sqrt[4]{6}, b=c=1$, 知 $\\frac{\\sqrt{2}}{2}+\\frac{k}{(\\sqrt[4]{6})^3} \\cdot 2 \\geqslant \\frac{2 \\sqrt{2}}{3} \\Rightarrow k \\geqslant \\frac{1}{\\sqrt[4]{24}}$.\n下面证明当 $k=\\frac{1}{\\sqrt[4]{24}}$ 时, $\\frac{k}{a^3} \\cdot\\left(\\frac{c^4}{b}+\\frac{b^4}{c}\\right) \\geqslant \\frac{2 \\sqrt{2}}{3} \\cdot \\frac{b^4+c^4}{a^4+b^4+c^4}$.\n等价于证明 $\\left(a^4+b^4+c^4\\right)\\left(b^5+c^5\\right) \\geqslant \\frac{4 \\sqrt[3]{6}}{3} a^3 b c\\left(b^4+c^4\\right)$.\n因为 $\\left(b^9+c^9\\right)-\\left(b^5 c^4+b^4 c^5\\right)=\\left(b^5-c^5\\right)\\left(b^4-c^4\\right) \\geqslant 0$, 所以 $b^9+c^9 \\geqslant b^5 c^4+b^4 c^5$.\n由加权平均值不等式, 得:\n$$\n\\begin{aligned}\na^4 b^5+2 b^5 c^4 & =6 \\cdot \\frac{a^4 b^5}{6}+2 \\cdot b^5 c^4 \\\\\n& \\geqslant 8 \\sqrt[8]{\\left(\\frac{a^4 b^5}{6}\\right)^6 \\cdot\\left(b^5 c^4\\right)^2} \\\\\n& =\\frac{4 \\sqrt[3]{6}}{3} a^3 b^5 c, \\\\\na^4 c^5+2 c^5 b^4 & =6 \\cdot \\frac{a^4 c^5}{6}+2 \\cdot c^5 b^4 \\\\\n& \\geqslant 8 \\sqrt[8]{\\left(\\frac{a^4 c^5}{6}\\right)^6 \\cdot\\left(c^5 b^4\\right)^2} \\\\\n& =\\frac{4 \\sqrt[3]{6}}{3} a^3 b c^5 .\n\\end{aligned}\n$$\n三式相加, 整理后即得\n$$\n\\left(a^4+b^4+c^4\\right)\\left(b^5+c^5\\right) \\geqslant \\frac{4 \\sqrt[3]{6}}{3} a^3 b c\\left(b^4+c^4\\right),\n$$\n故原左式 $\\geqslant \\frac{2 \\sqrt{2}}{3} \\cdot \\frac{a^4}{a^4+b^4+c^4}+\\frac{2 \\sqrt{2}}{3} \\cdot \\frac{b^4+c^4}{a^4+b^4+c^4}=\\frac{2 \\sqrt{2}}{3}=$ 原右式, 即所求 $k$ 的最小值为 $\\frac{1}{\\sqrt[4]{24}}$.", + "remark": "", + "figures": [] +} \ No newline at end of file diff --git a/processed_dataset/proof/1091.json b/processed_dataset/proof/1091.json new file mode 100644 index 0000000000000000000000000000000000000000..808f943501eef26b183e80ae9fc1e5b80c57458a --- /dev/null +++ b/processed_dataset/proof/1091.json @@ -0,0 +1,8 @@ +{ + "source_file": "./raw_volume-zh/volume4/chapter2-2.tex", + "problem_type": "proof", + "problem": "例7. (1) 如果 $a, b, c, d$ 是实数,求证:\n$$\na^6+b^6+c^6+d^6-6 a b c d \\geqslant-2,\n$$\n并指出等号何时成立;\n(2) 对于哪些正整数 $k$, 不等式\n$$\na^k+b^k+c^k+d^k-k a b c d \\geqslant M_k\n$$\n对所有实数 $a, b, c, d$ 成立? 求 $M_k$ 的最大可能值,并指出等号何时成立.", + "solution": "证明:(1) 给定不等式变形为\n$$\na^6+b^6+c^6+d^6+1+1 \\geqslant 6 a b c d .\n$$\n根据算术一几何平均值不等式, 得\n$$\n\\begin{aligned}\n& \\frac{a^6+b^6+c^6+d^6+1^6+1^6}{6} \\\\\n\\geqslant & \\sqrt[6]{|a|^6 \\cdot|b|^6 \\cdot|c|^6 \\cdot|d|^6 \\cdot 1^6 \\cdot 1^6} \\\\\n= & |a b c d| \\geqslant a b c d .\n\\end{aligned}\n$$\n因为算术一几何平均值不等式当\n$$\n|a|=|b|=|c|=|d|=1\n$$\n时等号成立.\n而最后的不等式, 当偶数个变量为负时等号成立.\n因此, 等号成立的情形是\n$$\n\\begin{aligned}\n(a, b, c, d)= & (1,1,1,1),(1,1,-1,-1),(1,-1,1,-1), \\\\\n& (-1,1,1,-1),(1,-1,-1,1),(-1,1,-1,1), \\\\\n& (-1,-1,1,1),(-1,-1,-1,-1)\n\\end{aligned}\n$$\n之一;\n(2)注意到,当 $k$ 是奇数时, 因为绝对值足够大的负值 $a, b, c, d$ 的选取得出了绝对值足够大的负值 $a^k+b^k+c^k+d^k-k a b c d$. 因此 $M_k$ 这样的数不存在.\n当 $k=2$ 时, 取 $a=b=c=d=r$, 得到 $a^2+b^2+c^2+d^2-2 a b c d= 4 r^2-2 r^4$.\n对足够大的正数 $r$ 的选取也得出绝对值任意大的负值.\n因此, $M_k$ 这样的数不存在.\n当 $k$ 是偶数,且 $k \\geqslant 4$ 时,取 $a=b=c=d=1$, 得 $a^k+b^k+c^k+d^k- k a b c d=4-k$.\n同(1)得\n$$\na^k+b^k+c^k+d^k-k a b c d \\geqslant 4-k,\n$$\n即 $\\frac{a^k+b^k+c^k+d^k+(k-4) \\cdot 1^k}{k} \\geqslant a b c d$.\n等号成立的条件与 (1) 相同.\n故此时 $M_k$ 的最大值为 $4-k$.", + "remark": "", + "figures": [] +} \ No newline at end of file diff --git a/processed_dataset/proof/1092.json b/processed_dataset/proof/1092.json new file mode 100644 index 0000000000000000000000000000000000000000..b68530755559a9f610a9e5c02d44e14ddb173a07 --- /dev/null +++ b/processed_dataset/proof/1092.json @@ -0,0 +1,8 @@ +{ + "source_file": "./raw_volume-zh/volume4/chapter2-2.tex", + "problem_type": "proof", + "problem": "例9. 对满足 $a b c=1$ 的正实数 $a 、 b 、 c$, 求\n$$\n\\left(a-1+\\frac{1}{b}\\right)\\left(b-1+\\frac{1}{c}\\right)\\left(c-1+\\frac{1}{a}\\right)\n$$\n的最大值.", + "solution": "解:由于表达式关于 $a 、 b 、 c$ 是对称的, 当 $a=b=c=1$ 时, 得\n$$\n\\left(a-1+\\frac{1}{b}\\right)\\left(b-1+\\frac{1}{c}\\right)\\left(c-1+\\frac{1}{a}\\right)=1 .\n$$\n下面我们证明最大值为 1 , 即证明对任意满足 $a b c=1$ 的实数 $a 、 b 、 c$, 有\n$$\n\\left(a-1+\\frac{1}{b}\\right)\\left(b-1+\\frac{1}{c}\\right)\\left(c-1+\\frac{1}{a}\\right) \\leqslant 1 .\n$$\n首先, 我们将非齐次的式子转换为齐次式, 即对正实数 $x 、 y 、 z$, 令 $a= \\frac{x}{y}, b=\\frac{y}{z}, c=\\frac{z}{x}$ (例如 $x=1, y=\\frac{1}{a}, z=\\frac{1}{a b}$ ), 则上式等价于证明\n$$\n(x-y+z)(y-z+x)(z-x+y) \\leqslant x y z .\n$$\n令 $u=x-y+z, v=y-z+x, w=z-x+y$, 由于 $u, v, w$ 的任意两个之和为正, 所以它们中最多有一个为负, 所以不妨假设 $u \\geqslant 0, v \\geqslant 0$, 由平均值不等式, 得\n$$\n\\begin{aligned}\n\\sqrt{u v} & =\\sqrt{(x-y+z)(y-z+x)} \\\\\n& \\leqslant \\frac{1}{2}(x-y+z)(y-z+x)=x .\n\\end{aligned}\n$$\n同理, $\\sqrt{v w} \\leqslant y, \\sqrt{w u} \\leqslant z$, 故 $u v w \\leqslant x y z$.", + "remark": "", + "figures": [] +} \ No newline at end of file diff --git a/processed_dataset/proof/1093.json b/processed_dataset/proof/1093.json new file mode 100644 index 0000000000000000000000000000000000000000..1792494bc999dba0f20e8564740bafe1694e981d --- /dev/null +++ b/processed_dataset/proof/1093.json @@ -0,0 +1,8 @@ +{ + "source_file": "./raw_volume-zh/volume4/chapter2-2.tex", + "problem_type": "proof", + "problem": "例10. 设 $a 、 b 、 c$ 为正实数, 满足\n$$\na+b+c+3 \\sqrt[3]{a b c} \\geqslant k(\\sqrt{a b}+\\sqrt{b c}+\\sqrt{c a}),\n$$\n求 $k$ 的最大值.", + "solution": "解:由于当 $a=b=c$ 时, 由 $6 \\geqslant 3 k$, 得 $k \\leqslant 2$. 下面证明\n$$\n\\begin{gathered}\na+b+c+3 \\sqrt[3]{a b c} \\geqslant 2(\\sqrt{a} \\bar{b}+\\sqrt{b c}+\\sqrt{c a}) . \\\\\n\\text { 令 } f(a, b, c)=a+b+c+3 \\sqrt[3]{a b c}-2(\\sqrt{a b}+\\sqrt{b c}+\\sqrt{c a}) .\n\\end{gathered}\n$$\n不妨假设 $a \\leqslant b \\leqslant c$, 作如下调整,\n$$\na=a^{\\prime}, b=b^{\\prime}=c^{\\prime}=\\sqrt{b c}=A,\n$$\n则\n$$\n\\begin{aligned}\nf\\left(a^{\\prime}, b^{\\prime}, c^{\\prime}\\right) & =\\left(a+2 A+3 \\cdot a^{\\frac{1}{3}} \\cdot A^{\\frac{2}{3}}\\right)-2\\left[A+(2 a A)^{\\frac{1}{2}}\\right] \\\\\n& =a+3 \\cdot a^{\\frac{1}{3}} \\cdot A^{\\frac{2}{3}}-4(a A)^{\\frac{1}{2}} \\geqslant 0 .\n\\end{aligned}\n$$\n等号成立当且仅当 $a=0$ 或 $a=b=c$.\n再证明 $f(a, b, c) \\geqslant f\\left(a^{\\prime}, b^{\\prime}, c^{\\prime}\\right)$. 因为\n$$\nf(a, b, c)-f\\left(a^{\\prime}, b^{\\prime}, c^{\\prime}\\right)=b+c-2 a^{\\frac{1}{2}}\\left(b^{\\frac{1}{2}}+c^{\\frac{1}{2}}-2 A^{\\frac{1}{2}}\\right)-2 A \\text {. }\n$$\n由于\n$$\na \\leqslant A, b^{\\frac{1}{2}}+c^{\\frac{1}{2}} \\geqslant 2 A^{\\frac{1}{2}} \\text {, }\n$$\n所以\n$$\n\\begin{aligned}\n& f(a, b, c)-f\\left(a^{\\prime}, b^{\\prime}, c^{\\prime}\\right) \\\\\n\\geqslant & b+c-2 \\sqrt{A}(\\sqrt{b}+\\sqrt{c}-2 \\sqrt{A})-2 A \\\\\n= & b+c-2(\\sqrt{b}+\\sqrt{c}) \\sqrt{A}+2 A \\\\\n= & (\\sqrt{b}-\\sqrt{A})^2+(\\sqrt{c}-\\sqrt{A})^2 \\\\\n\\geqslant & 0 .\n\\end{aligned}\n$$\n从而\n$$\nf(a, b, c) \\geqslant 0,\n$$\n故 $k$ 的最大值为 2 .", + "remark": "", + "figures": [] +} \ No newline at end of file diff --git a/processed_dataset/proof/1094.json b/processed_dataset/proof/1094.json new file mode 100644 index 0000000000000000000000000000000000000000..8e98143ba1ba7a88aca8ad61598f756db4616819 --- /dev/null +++ b/processed_dataset/proof/1094.json @@ -0,0 +1,8 @@ +{ + "source_file": "./raw_volume-zh/volume4/chapter2-3.tex", + "problem_type": "proof", + "problem": "例1. 对于任意一个 $\\triangle A B C$, 记其面积为 $S$, 周长为 $l, P 、 Q 、 R$ 依次为 $\\triangle A B C$ 内切圆在边 $B C 、 C A 、 A B$ 上的切点.\n证明 :\n$$\n\\left(\\frac{A B}{P Q}\\right)^3+\\left(\\frac{B C}{Q R}\\right)^3+\\left(\\frac{C A}{R P}\\right)^3 \\geqslant \\frac{2}{\\sqrt{3}} \\cdot \\frac{l^2}{S}\n$$", + "solution": "证明:记 $B C=a, C A=b, A B=c, Q R=p, R P=q, P Q=r$. 设 $A R=x, B P=y, C Q=z$. 由 $x+y=c, y+z=a, z+x=b$, 得\n$$\nx=t-a, y=t-b, z=t-c\\left(t=\\frac{a+b+c}{2}\\right) .\n$$\n在 $\\triangle A B C 、 \\triangle A R Q$ 中, 由余弦定理分别得\n$$\n\\begin{gathered}\na^2=b^2+c^2-2 b c \\cos A=(b-c)^2+2 b c(1-\\cos A), \\\\\np^2=2 x^2(1-\\cos A)=2(t-a)^2(1-\\cos A) .\n\\end{gathered}\n$$\n两式消去 $1-\\cos A$ 有\n$$\n\\begin{aligned}\np^2 & =(t-a)^2 \\frac{a^2-(b-c)^2}{b c} \\\\\n& =\\frac{4(t-a)(t-b)(t-c)}{a b c} a(t-a) . \\label{eq1}\n\\end{aligned}\n$$\n注意到\n$$\n4(t-a)(t-b)=(b+c-a)(a-b+c)=c^2-(b-a)^2 \\leqslant c^2 .\n$$\n同理, $4(t-b)(t-c) \\leqslant a^2, 4(t-c)(t-a) \\leqslant b^2$. 则\n$$\n8(t-a)(t-b)(t-c) \\leqslant a b c .\n$$\n代入式\\ref{eq1}得\n$$\np^2 \\leqslant \\frac{a(t-a)}{2} \\text { 或 }\\left(\\frac{a}{p}\\right)^3 \\geqslant 2 \\sqrt{2}\\left(\\frac{a}{t-a}\\right)^{\\frac{3}{2}} .\n$$\n同理, $\\left(\\frac{b}{q}\\right)^3 \\geqslant 2 \\sqrt{2}\\left(\\frac{b}{t-b}\\right)^{\\frac{3}{2}},\\left(\\frac{c}{r}\\right)^3 \\geqslant 2 \\sqrt{2}\\left(\\frac{c}{t-c}\\right)^{\\frac{3}{2}}$. 记 $M$ 为所证不等式左边.\n则\n$$\n\\begin{aligned}\nM & \\geqslant 2 \\sqrt{2}\\left[\\left(\\frac{a}{t-a}\\right)^{\\frac{3}{2}}+\\left(\\frac{b}{t-b}\\right)^{\\frac{3}{2}}+\\left(\\frac{c}{t-c}\\right)^{\\frac{3}{2}}\\right] \\\\\n& \\geqslant \\frac{2 \\sqrt{2}}{\\sqrt{3}}\\left(\\frac{a}{t-a}+\\frac{b}{t-b}+\\frac{c}{t-c}\\right)^{\\frac{3}{2}} .\n\\end{aligned} \\label{eq2}\n$$\n又 $a \\geqslant b \\geqslant c \\Leftrightarrow \\frac{1}{t-a} \\geqslant \\frac{1}{t-b} \\geqslant \\frac{1}{t-c}$.\n由切比雪夫不等式及均值不等式得\n$$\n\\frac{a}{t-a}+\\frac{b}{t-b}+\\frac{c}{t-c}\n$$\n$$\n\\begin{aligned}\n& \\geqslant \\frac{1}{3}(a+b+c)\\left(\\frac{1}{t-a}+\\frac{1}{t-b}+\\frac{1}{t-c}\\right) \\\\\n& \\geqslant \\frac{a+b+c}{[(t-a)(t-b)(t-c)]^{\\frac{1}{3}}} \\\\\n& =\\frac{(a+b+c) t^{\\frac{1}{3}}}{[t(t-a)(t-b)(t-c)]^{\\frac{1}{3}}} \\\\\n& =\\frac{1}{2^{\\frac{1}{3}}}\\left(\\frac{l^2}{S}\\right)^{\\frac{2}{3}} .\n\\end{aligned} \\label{eq3}\n$$\n由式\\ref{eq2}、\\ref{eq3}得\n$$\nM \\geqslant \\frac{2 \\sqrt{2}}{\\sqrt{3}} \\cdot \\frac{1}{\\sqrt{2}} \\cdot \\frac{l^2}{S}=\\frac{2}{\\sqrt{3}} \\cdot \\frac{l^2}{S} .\n$$", + "remark": "", + "figures": [] +} \ No newline at end of file diff --git a/processed_dataset/proof/1095.json b/processed_dataset/proof/1095.json new file mode 100644 index 0000000000000000000000000000000000000000..5d02fdd6f93a45d7700bf06f844c5b9dfd57b36f --- /dev/null +++ b/processed_dataset/proof/1095.json @@ -0,0 +1,8 @@ +{ + "source_file": "./raw_volume-zh/volume4/chapter2-3.tex", + "problem_type": "proof", + "problem": "例2. 设 $a 、 b 、 c$ 分别为一个三角形的三边长.\n令\n$$\n\\begin{aligned}\n& A=\\sum \\frac{a^2+b c}{b+c}, \\\\\n& B=\\sum \\frac{1}{\\sqrt{(a+b-c)(b+c-a)}},\n\\end{aligned}\n$$\n其中, \" $\\sum$ \" 表示轮换对称和.\n求证: $A B \\geqslant 9$.", + "solution": "证明:设 $a=y+z, b=x+z, c=x+y$ ( $x 、 y 、 z$ 为正数). 则\n$$\n\\begin{aligned}\nB & =\\sum \\frac{1}{2 \\sqrt{x y}}, \\\\\nA & =\\sum \\frac{x^2+y^2+z^2+x y+z x+3 y z}{2 x+y+z}, \\\\\nA B & =\\left(\\sum \\frac{1}{2 \\sqrt{y z}}\\right) \\sum \\frac{x^2+y^2+z^2+x y+3 y z+z x}{2 x+y+z} \\\\\n& \\geqslant\\left(\\sum \\sqrt{\\frac{1}{2 \\sqrt{y z}} \\cdot \\frac{x^2+y^2+z^2+x y+3 y z+z x}{2 x+y+z}}\\right)^2 .\n\\end{aligned}\n$$\n下面证明上式中每个根号内的数都大于或等于 1 . 只需证第一个根号内的数大于或等于 1 , 即\n$$\n\\frac{1}{2 \\sqrt{y z}} \\cdot \\frac{x^2+y^2+z^2+x y+z x+3 y z}{2 x+y+z} \\geqslant 1\n$$\n$$\n\\begin{aligned}\n\\Leftrightarrow & \\left(x^2+y^2+z^2+x y+3 y z+z x\\right)^2 \\geqslant 4 y z(2 x+y+z)^2 \\\\\n\\Leftrightarrow & x^4+y^4+z^4+3 x^2 y^2+3 x^2 z^2+3 y^2 z^2+2 x^3 y+2 x y^3 \\\\\n& +2 x^3 z+2 x z^3+2 y^3 z+2 y z^3 \\\\\n\\geqslant & 8 x y^2 z+8 x^2 y z+8 x y z^2 . \\label{eq1}\n\\end{aligned}\n$$\n由幂平均不等式得\n$$\nx^4+y^4+z^4 \\geqslant \\frac{1}{27}(x+y+z)^4 \\geqslant x y z(x+y+z)=x y^2 z+x^2 y z+x y z^2 .\n$$\n由均值不等式得\n$$\n\\begin{gathered}\n3 x^2 y^2+3 x^2 z^2+3 y^2 z^2 \\geqslant 3\\left(x y^2 z+x^2 y z+x y z^2\\right), \\\\\nx^3 y+x y^3+x^3 z+x z^3+y^3 z+y z^3-2 x y z(x+y+z) \\\\\n=\\left(x^3 y+y z^3-x y z^2-x^2 y z\\right)+\\left(y^3 z+x^3 z-x^2 y z-x y^2 z\\right) \\\\\n+\\left(z^3 x+x y^3-x y^2 z-x y z^2\\right) \\\\\n=y(x+z)(x-z)^2+z(x+y)(x-y)^2+x(y+z)(y-z)^2 \\geqslant 0 .\n\\end{gathered}\n$$\n相加即得式 \\ref{eq1} 成立.", + "remark": "", + "figures": [] +} \ No newline at end of file diff --git a/processed_dataset/proof/1096.json b/processed_dataset/proof/1096.json new file mode 100644 index 0000000000000000000000000000000000000000..f07a691e99332bc1c7780d18be40602dadb67c46 --- /dev/null +++ b/processed_dataset/proof/1096.json @@ -0,0 +1,8 @@ +{ + "source_file": "./raw_volume-zh/volume4/chapter2-3.tex", + "problem_type": "proof", + "problem": "例3. $\\triangle A B C$ 的三边长 $a, b, c$ 满足 $a+b+c=1$. 求证:\n$$\n5\\left(a^2+b^2+c^2\\right)+18 a b c \\geqslant \\frac{7}{3} .\n$$", + "solution": "证明:因为 $a^2+b^2+c^2$\n$$\n\\begin{aligned}\n& =(a+b+c)^2-2(a b+b c+c a) \\\\\n& =1-2(a b+b c+c a),\n\\end{aligned}\n$$\n所以, 欲证的不等式等价于\n$$\n\\frac{5}{9}(a b+b c+c a)-a b c \\leqslant \\frac{4}{27} .\n$$\n构造函数\n$$\nf(x)=(x-a)(x-b)(x-c),\n$$\n一方面,\n$$\nf(x)=x^3-(a+b+c) x^2+(a b+b c+c a) x-a b c,\n$$\n所以,\n$$\nf\\left(\\frac{5}{9}\\right)=\\left(\\frac{5}{9}\\right)^3-\\left(\\frac{5}{9}\\right)^2+\\frac{5}{9}(a b+b c+c a)-a b c .\n$$\n另一方面, 因为 $a, b, c$ 是三角形三边长, 所以 $0d$. 作\n$$\nF A_1 \\Perp A B, F E_1 \\Perp D E,\n$$\n连 $D E_1$, 连 $B A_1$ 延长交 $D E_1$ 于点 $C_1$, 则\n$$\n\\angle E_1 A_1 C_1=\\pi-A, \\angle A_1 C_1 E_1=\\pi-C, \\angle C_1 E_1 A_1=\\pi-E=\\pi-B\n$$\n且\n$$\n\\begin{aligned}\n& \\frac{\\sin A}{\\sin B}=\\frac{C_1 E_1}{A_1 C_1}, \\frac{\\sin B}{\\sin C}=\\frac{A_1 C_1}{A_1 E_1}, \\frac{\\sin C}{\\sin A}=\\frac{A_1 E_1}{C_1 E_1} . \\\\\n& a=d+A_1 E_1, e=b+E_1 C_1, c=f+C_1 A_1 .\n\\end{aligned}\n$$\n利用平均值不等式, 得\n$$\n\\begin{aligned}\n& 2\\left(R_A+R_B+R_C\\right) \\\\\n\\geqslant & 2(b+d+f)+\\frac{A_1 E_1 \\cdot A_1 C_1}{E_1 C_1}+\\frac{E_1 C_1 \\cdot A_1 E_1}{A_1 C_1}+\\frac{A_1 C_1 \\cdot E_1 C_1}{A_1 E_1} .\n\\end{aligned}\n$$\n再由排序不等式, 得\n$$\n\\frac{A_1 E_1 \\cdot A_1 C_1}{E_1 C_1}+\\frac{E_1 C_1 \\cdot A_1 E_1}{A_1 C_1}+\\frac{A_1 C_1 \\cdot E_1 C_1}{A_1 E_1} \\geqslant A_1 E_1+C_1 A_1+E_1 C_1 .\n$$\n从而\n$$\n2\\left(R_A+R_B+R_C\\right) \\geqslant 2(b+d+f)+A_1 E_1+C_1 A_1+E_1 C_1=p .\n$$\n综合 (1)、(2) 知命题成立.", + "remark": "", + "figures": [] +} \ No newline at end of file diff --git a/processed_dataset/proof/1102.json b/processed_dataset/proof/1102.json new file mode 100644 index 0000000000000000000000000000000000000000..b1213fdb74df53c36462d93d0c5e8e3fd157bf68 --- /dev/null +++ b/processed_dataset/proof/1102.json @@ -0,0 +1,8 @@ +{ + "source_file": "./raw_volume-zh/volume4/chapter2-4.tex", + "problem_type": "proof", + "problem": "例1. 设 $a, b, c$ 是正实数, 且满足 $a^2+b^2+c^2=3$. 证明:\n$$\n\\frac{1}{1+2 a b}+\\frac{1}{1+2 b c}+\\frac{1}{1+2 c a} \\geqslant 1 \\text {. }\n$$", + "solution": "证明:由算术平均值大于或等于几何平均值及算术平均值大于或等于 调和平均值可得\n$$\n\\begin{aligned}\n& \\frac{1}{1+2 a b}+\\frac{1}{1+2 b c}+\\frac{1}{1+2 c a} \\\\\n\\geqslant & \\frac{1}{1+a^2+b^2}+\\frac{1}{1+b^2+c^2}+\\frac{1}{1+c^2+a^2} \\\\\n\\geqslant & 3 \\cdot \\frac{3}{\\left(1+a^2+b^2\\right)+\\left(1+b^2+c^2\\right)+\\left(1+c^2+a^2\\right)} \\\\\n= & \\frac{9}{3+2\\left(a^2+b^2+c^2\\right)}=1 .\n\\end{aligned}\n$$", + "remark": "", + "figures": [] +} \ No newline at end of file diff --git a/processed_dataset/proof/1103.json b/processed_dataset/proof/1103.json new file mode 100644 index 0000000000000000000000000000000000000000..b149ca5babf599c332e8648704923b290c78e296 --- /dev/null +++ b/processed_dataset/proof/1103.json @@ -0,0 +1,8 @@ +{ + "source_file": "./raw_volume-zh/volume4/chapter2-4.tex", + "problem_type": "proof", + "problem": "例2. 已知正实数 $a 、 b 、 c$ 满足\n$$\na b+b c+c a \\leqslant 3 a b c .\n$$\n求证:\n$$\n\\begin{aligned}\n& \\sqrt{\\frac{a^2+b^2}{a+b}}+\\sqrt{\\frac{b^2+c^2}{b+c}}+\\sqrt{\\frac{c^2+a^2}{c+a}}+3 \\\\\n\\leqslant & \\sqrt{2}(\\sqrt{a+b}+\\sqrt{b+c}+\\sqrt{c+a}) .\n\\end{aligned}\n$$", + "solution": "证明:由 $Q_2 \\geqslant A_2$ 得\n$$\n\\begin{aligned}\n& \\sqrt{2} \\cdot \\sqrt{a+b}=2 \\sqrt{\\frac{a b}{a+b}} \\cdot \\sqrt{\\frac{1}{2}\\left(2+\\frac{a^2+b^2}{a b}\\right)} \\\\\n\\geqslant & 2 \\sqrt{\\frac{a b}{a+b}} \\cdot \\frac{1}{2}\\left(\\sqrt{2}+\\sqrt{\\frac{a^2+b^2}{a b}}\\right) \\\\\n= & \\sqrt{\\frac{2 a b}{a+b}}+\\sqrt{\\frac{a^2+b^2}{a+b}} .\n\\end{aligned}\n$$\n同理,\n$$\n\\begin{aligned}\n& \\sqrt{2} \\cdot \\sqrt{b+c} \\geqslant \\sqrt{\\frac{2 b c}{b+c}}+\\sqrt{\\frac{b^2+c^2}{b+c}}, \\\\\n& \\sqrt{2} \\cdot \\sqrt{c+a} \\geqslant \\sqrt{\\frac{2 c a}{c+a}}+\\sqrt{\\frac{c^2+a^2}{c+a}} .\n\\end{aligned}\n$$\n再由 $Q_3 \\geqslant H_3$ 得\n$$\n\\begin{gathered}\n\\sqrt{\\frac{\\left(\\sqrt{\\frac{a+b}{2 a b}}\\right)^2+\\left(\\sqrt{\\frac{b+c}{2 b c}}\\right)^2+\\left(\\sqrt{\\frac{c+a}{2 c a}}\\right)^2}{3}} \\\\\n\\geqslant \\frac{\\frac{1}{\\sqrt{\\frac{a+b}{2 a b}}}+\\frac{1}{\\sqrt{\\frac{b+c}{2 b c}}}+\\frac{1}{\\sqrt{\\frac{c+a}{2 c a}}}}{}\n\\end{gathered}\n$$\n所以\n$$\n\\begin{aligned}\n& \\sqrt{\\frac{2 a b}{a+b}}+\\sqrt{\\frac{2 b c}{b+c}}+\\sqrt{\\frac{2 c a}{c+a}} \\\\\n\\geqslant & 3 \\sqrt{\\frac{3}{\\left(\\sqrt{\\frac{a+b}{2 a b}}\\right)^2+\\left(\\sqrt{\\frac{b+c}{2 b c}}\\right)^2+\\left(\\sqrt{\\frac{c+a}{2 c a}}\\right)^2}} \\\\\n= & 3 \\sqrt{\\frac{3 a b c}{a b+b c+c a}} \\geqslant 3 .\n\\end{aligned}\n$$\n于是,原不等式成立.", + "remark": "", + "figures": [] +} \ No newline at end of file diff --git a/processed_dataset/proof/1104.json b/processed_dataset/proof/1104.json new file mode 100644 index 0000000000000000000000000000000000000000..1b14c1d1f5da08b0ad08bbe4834c343aa69da000 --- /dev/null +++ b/processed_dataset/proof/1104.json @@ -0,0 +1,8 @@ +{ + "source_file": "./raw_volume-zh/volume4/chapter2-4.tex", + "problem_type": "proof", + "problem": "例3. 设正实数 $x 、 y 、 z$ 满足 $x^2+y^2+z^2=1$. 求证:\n$$\nx^2 y z+y^2 x z+z^2 x y \\leqslant \\frac{1}{3} .\n$$", + "solution": "证明:因为 $\\sqrt[3]{x y z} \\leqslant \\frac{x+y+z}{3} \\leqslant \\sqrt{\\frac{x^2+y^2+z^2}{3}}=\\frac{1}{\\sqrt{3}}$, 所以\n$$\nx y z \\leqslant \\frac{1}{3 \\sqrt{3}}, x+y+z \\leqslant \\sqrt{3},\n$$\n故\n$$\nx^2 y z+y^2 x z+z^2 x y \\leqslant-\\frac{1}{3} .\n$$", + "remark": "", + "figures": [] +} \ No newline at end of file diff --git a/processed_dataset/proof/1105.json b/processed_dataset/proof/1105.json new file mode 100644 index 0000000000000000000000000000000000000000..d97effb7717da83138e41779cb53a76d63f7680f --- /dev/null +++ b/processed_dataset/proof/1105.json @@ -0,0 +1,8 @@ +{ + "source_file": "./raw_volume-zh/volume4/chapter2-4.tex", + "problem_type": "proof", + "problem": "例4. 设 $a, b, c, d \\in \\mathbf{R}^{+}$, 求证:\n$$\n\\sqrt[3]{\\frac{a b c+b c d+c d a+a d b}{4}} \\leqslant \\sqrt{\\frac{a^2+b^2+c^2+d^2}{4}} .\n$$", + "solution": "证明:首先两次应用 $G_2 \\leqslant A_2$, 得\n$\\frac{a b c+b c d+c d a+a d b}{4}$\n$$\n\\begin{aligned}\n& =\\frac{1}{2}\\left(a b \\cdot \\frac{c+d}{2}+c d \\cdot \\frac{a+b}{2}\\right) \\\\\n& \\leqslant \\frac{1}{2}\\left[\\left(\\frac{a+b}{2}\\right)^2 \\cdot \\frac{c+d}{2}+\\left(\\frac{c+d}{2}\\right)^2 \\cdot \\frac{a+b}{2}\\right] \\\\\n& =\\frac{a+b}{2} \\cdot \\frac{c+d}{2} \\cdot \\frac{a+b+c+d}{4} \\\\\n& \\leqslant\\left(\\frac{\\frac{a+b}{2}+\\frac{c+d}{2}}{2}\\right)^2 \\frac{a+b+c+d}{4}=\\left(\\frac{a+b+c+d}{4}\\right)^3 .\n\\end{aligned}\n$$\n即再由 $A_4 \\leqslant Q_4$, 得\n$$\n\\frac{a+b+c+d}{4} \\leqslant \\sqrt{\\frac{a^2+b^2+c^2+d^2}{4}} .\n$$\n故原不等式成立.", + "remark": "", + "figures": [] +} \ No newline at end of file diff --git a/processed_dataset/proof/1106.json b/processed_dataset/proof/1106.json new file mode 100644 index 0000000000000000000000000000000000000000..c7ee4de6e282c4ed9692737910a451515fb82b35 --- /dev/null +++ b/processed_dataset/proof/1106.json @@ -0,0 +1,8 @@ +{ + "source_file": "./raw_volume-zh/volume4/chapter2-4.tex", + "problem_type": "proof", + "problem": "例5. 设 $x_i \\geqslant 1(i=1,2, \\cdots, n)$, 证明:\n$$\n\\frac{\\prod\\left(x_i-1\\right)}{\\left(\\sum\\left(x_i-1\\right)\\right)^n} \\leqslant \\frac{\\prod x_i}{\\left(\\sum x_i\\right)^n} .\n$$", + "solution": "证明:原不等式等价于\n$$\n\\left(\\frac{\\Pi\\left(x_i-1\\right)}{\\prod x_i}\\right)^{\\frac{1}{n}} \\leqslant \\frac{\\sum\\left(x_i-1\\right)}{\\sum x_i},\n$$\n这个不等式可以由下面的事实推出.\n由平均值不等式,得\n$$\n\\begin{aligned}\n\\left(\\frac{\\prod\\left(x_i-1\\right)}{\\prod x_i}\\right)^{\\frac{1}{n}} & =\\left(\\prod \\frac{x_i-1}{x_i}\\right)^{\\frac{1}{n}} \\\\\n& \\leqslant \\frac{1}{n} \\sum \\frac{x_i-1}{x_i} \\\\\n& =1-\\frac{1}{n} \\sum \\frac{1}{x_i} .\n\\end{aligned}\n$$\n以及\n$$\n\\begin{gathered}\n\\frac{\\sum\\left(x_i-1\\right)}{\\sum x_i}=\\frac{\\sum x_i-n}{\\sum x_i}=1-\\frac{n}{\\sum x_i}, \\\\\n\\frac{1}{n} \\sum \\frac{1}{x_i} \\geqslant \\frac{n}{\\sum x_i} .\n\\end{gathered}\n$$\n从而可知命题成立.", + "remark": "", + "figures": [] +} \ No newline at end of file diff --git a/processed_dataset/proof/1107.json b/processed_dataset/proof/1107.json new file mode 100644 index 0000000000000000000000000000000000000000..020d442910723e6814b8e61ce3343c0e728d1024 --- /dev/null +++ b/processed_dataset/proof/1107.json @@ -0,0 +1,8 @@ +{ + "source_file": "./raw_volume-zh/volume4/chapter2-4.tex", + "problem_type": "proof", + "problem": "例6. 设 $x_i \\in\\left[0, \\frac{\\pi}{2}\\right], i=1,2, \\cdots, 10$, 满足 $\\sin ^2 x_1+\\sin ^2 x_2+\\cdots+ \\sin ^2 x_{10}=1$. 证明:\n$$\n3\\left(\\sin x_1+\\cdots+\\sin x_{10}\\right) \\leqslant \\cos x_1+\\cdots+\\cos x_{10} .\n$$", + "solution": "证明:由于 $\\sin ^2 x_1+\\sin ^2 x_2+\\cdots+\\sin ^2 x_{10}=1$ ,\n$$\n\\cos x_i=\\sqrt{\\sum_{j \\neq i} \\sin ^2 x_j}\n$$\n则对 $1 \\leqslant i \\leqslant 10$, 得\n$$\n\\cos x_i=\\sqrt{\\sum_{j \\neq i} \\sin ^2 x_j} \\geqslant \\frac{\\sum_{j \\neq i} \\sin x_j}{3} .\n$$\n从而\n$$\n\\sum_{i=1}^{10} \\cos x_i \\geqslant \\sum_{i=1}^{10} \\sum_{j \\neq i} \\frac{\\sin x_j}{3}=\\sum_{i=1}^{10} 9 \\cdot \\frac{\\sin x_i}{3}=3 \\sum_{i=1}^{10} \\sin x_i .\n$$\n故命题成立.", + "remark": "", + "figures": [] +} \ No newline at end of file diff --git a/processed_dataset/proof/1108.json b/processed_dataset/proof/1108.json new file mode 100644 index 0000000000000000000000000000000000000000..4c7df51bc6299f75817ab6b719c84e29f3ba5b10 --- /dev/null +++ b/processed_dataset/proof/1108.json @@ -0,0 +1,8 @@ +{ + "source_file": "./raw_volume-zh/volume4/chapter2-4.tex", + "problem_type": "proof", + "problem": "例8. 已知 $a, b, c \\in \\mathbf{R}^{+}$, 且满足 $\\frac{a^2}{1+a^2}+\\frac{b^2}{1+b^2}+\\frac{c^2}{1+c^2}=1$, 求证:\n$$\na b c \\leqslant \\frac{\\sqrt{2}}{4} \\text {. }\n$$", + "solution": "证明:令 $x=\\frac{a^2}{1+a^2}, y=\\frac{b^2}{1+b^2}, z=\\frac{c^2}{1+c^2}$, 则\n$$\n00$, 则称\n$$\nM_r=\\left(\\frac{\\sum_{i=1}^n a_i^r}{n}\\right)^{\\frac{1}{r}}\n$$\n为 $a_1, a_2, \\cdots, a_n$ 的 $r$ 次幂平均值.\n对于 $M_r$, 我们有幕平均不等式, 即:\n对 $\\alpha>\\beta$, 则 $M_\\alpha \\geqslant M_\\beta$, 即\n$$\n\\left(\\frac{\\sum a_i^\\alpha}{n}\\right)^{\\frac{1}{\\alpha}} \\geqslant\\left(\\frac{\\sum a_i^\\beta}{n}\\right)^{\\frac{1}{\\beta}}\n$$\n等号成立的充要条件是 $a_1=a_2=\\cdots=a_n$.\n特别当 $\\alpha>1, \\beta=1$ 时,\n$$\n\\frac{\\sum a_i^\\alpha}{n} \\geqslant\\left(\\frac{\\sum a_i}{n}\\right)^\\alpha\n$$", + "remark": "", + "figures": [] +} \ No newline at end of file diff --git a/processed_dataset/proof/1109.json b/processed_dataset/proof/1109.json new file mode 100644 index 0000000000000000000000000000000000000000..81644ae0d301ba082acc7ba2a0deb4a2f303cc70 --- /dev/null +++ b/processed_dataset/proof/1109.json @@ -0,0 +1,8 @@ +{ + "source_file": "./raw_volume-zh/volume4/chapter2-4.tex", + "problem_type": "proof", + "problem": "例10. 设三角形三边长分别为 $a 、 b 、 c$, 面积为 $S$, 则\n$$\na^n+b^n+c^n \\geqslant 2^n \\cdot 3^{\\frac{4-n}{4}} S^{\\frac{n}{2}}, n \\in \\mathbf{N} \\text {. }\n$$", + "solution": "证明:当 $n=1$ 时, $a+b+c \\geqslant 2 \\sqrt{3 \\sqrt{3} S}$ 是常见的几何不等式, 即 $n=1$ 时成立.\n假设当 $n=k$ 时命题成立, 即有不等式\n$$\na^k+b^k+c^k \\geqslant 2^k \\cdot 3^{\\frac{4-k}{4}} S^{\\frac{k}{2}},\n$$\n则当 $n=k+1$ 时, 由幂平均不等式,\n$$\n\\left(\\frac{a_1^\\alpha+a_2^\\alpha+\\cdots+a_n^\\alpha}{n}\\right)^{\\frac{1}{\\alpha}} \\geqslant\\left(\\frac{a_1^\\beta+a_2^\\beta+\\cdots+a_n^\\beta}{n}\\right)^{\\frac{1}{\\beta}},\n$$\n其中 $a_i \\in \\mathbf{R}^{+}, i=1,2, \\cdots, n, \\alpha \\geqslant \\beta$.\n得\n$$\n\\left(\\frac{a^{k+1}+b^{k+1}+c^{k+1}}{3}\\right)^{\\frac{1}{k+1}} \\geqslant\\left(\\frac{a^k+b^k+c^k}{3}\\right)^{\\frac{1}{k}} \\geqslant\\left(\\frac{2^k}{3} \\cdot 3^{\\frac{4-k}{4}} \\cdot S^{\\frac{k}{2}}\\right)^{\\frac{1}{k}} .\n$$\n从而\n$$\na^{k+1}+b^{k+1}+c^{k+1} \\geqslant 3\\left(\\frac{2^k}{3} \\cdot 3^{\\frac{4-k}{4}} \\cdot S^{\\frac{k}{2}}\\right)^{\\frac{k+1}{k}}=2^{k+1} \\cdot 3^{\\frac{4-(k+1)}{4}} S^{\\frac{k+1}{2}} .\n$$\n即当 $n=k+1$ 时, 原不等式成立.\n由于一般的幂平均不等式在竞赛中很少出现, 这里就不展开讨论了.\n等式或不等式的变形, 是证明数学问题和运算中常使用的方法和技巧.\n前面我们介绍了平均值不等式的证明和应用, 但对于某些问题, 通过变形处理可能比运用基本定理来证明更简单,下面我们以一个例子来说明,希望读者能有所体会和了解.", + "remark": "", + "figures": [] +} \ No newline at end of file diff --git a/processed_dataset/proof/1110.json b/processed_dataset/proof/1110.json new file mode 100644 index 0000000000000000000000000000000000000000..3e15cc287aa314ffaa9ef688b16c46fe68259ff4 --- /dev/null +++ b/processed_dataset/proof/1110.json @@ -0,0 +1,8 @@ +{ + "source_file": "./raw_volume-zh/volume4/chapter2-4.tex", + "problem_type": "proof", + "problem": "例11. 设 $a, b, c$ 是正实数,证明:\n$$\n\\frac{(2 a+b+c)^2}{2 a^2+(b+c)^2}+\\frac{(2 b+a+c)^2}{2 b^2+(c+a)^2}+\\frac{(2 c+a+b)^2}{2 c^2+(a+b)^2} \\leqslant 8 .\n$$", + "solution": "证明一通过变形直接证明.\n不妨假设 $a+b+c=1$. 则原不等式等价于\n$$\n\\begin{gathered}\n\\frac{(1+a)^2}{2 a^2+(1-a)^2}+\\frac{(1+b)^2}{2 b^2+(1-b)^2}+\\frac{(1+c)^2}{2 c^2+(1-c)^2} \\leqslant 8, \\\\\n\\frac{a^2+2 a+1}{3 a^2-2 a+1}+\\frac{b^2+2 b+1}{3 b^2-2 b+1}+\\frac{c^2+2 c+1}{3 c^2-2 c+1} \\leqslant 8 .\n\\end{gathered}\n$$\n两边同乘以 3 , 得\n$$\n\\frac{3 a^2+6 a+3}{3 a^2-2 a+1}+\\frac{3 b^2+6 b+3}{3 b^2-2 b+1}+\\frac{3 c^2+6 c+3}{3 c^2-2 c+1} \\leqslant 24 .\n$$\n消除分子的二次项,得\n$$\n\\frac{8 a+2}{3 a^2-2 a+1}+\\frac{8 b+2}{3 b^2-2 b+1}+\\frac{8 c+2}{3 c^2-2 c+1} \\leqslant 21 .\n$$\n因为 $3 x^2-2 x+1=3\\left(x-\\frac{1}{3}\\right)^2+\\frac{2}{3} \\geqslant \\frac{2}{3}$, 所以\n$$\n\\begin{aligned}\n& \\frac{8 a+2}{3 a^2-2 a+1}+\\frac{8 b+2}{3 b^2-2 b+1}+\\frac{8 c+2}{3 c^2-2 c+1} \\\\\n\\leqslant & \\frac{8 a+2}{\\frac{2}{3}+\\frac{8 b+2}{\\frac{2}{3}}+\\frac{8 c+2}{\\frac{2}{3}}=\\frac{3}{2}(8+6) \\leqslant 21 .}\n\\end{aligned}\n$$\n故命题成立.\n%%%%\n证明二对一个 $n$ 个变量的函数 $f$, 定义它的对称和\n$$\n\\sum_{s y m} f\\left(x_1, x_2, \\cdots, x_n\\right)=\\sum_\\sigma f\\left(x_{\\sigma(1)}, x_{\\sigma(2)}, \\cdots, x_{\\sigma(n)}\\right) .\n$$\n这里 $\\sigma$ 是 $1,2, \\cdots, n$ 的所有的排列, $s y m$ 表示对称求和.\n例如, 将 $x_1$, $x_2, x_3$ 记为 $x, y, z$, 当 $n=3$ 时,有\n$$\n\\begin{gathered}\n\\sum_{s y m} x^3=2 x^3+2 y^3+2 z^3, \\\\\n\\sum_{s y m} x^2 y=x^2 y+y^2 z+z^2 x+x^2 z+y^2 x+z^2 y\n\\end{gathered}\n$$\n$$\n\\sum_{s y m} x y z=6 x y z\n$$\n则\n$$\n8-\\frac{(2 a+b+c)^2}{2 a^2+(b+c)^2}+\\frac{(2 b+a+c)^2}{2 b^2+(c+a)^2}+\\frac{(2 c+a+b)^2}{2 c^2+(a+b)^2}=\\frac{A}{B},\n$$\n其中 $B>0$,\n$$\nA=\\sum_{\\text {sym }}\\left(4 a^6+4 a^5 b+a^4 b^2+5 a^4 b c+5 a^3 b^3-26 a^3 b^2 c+7 a^2 b^2 c^2\\right) .\n$$\n下面证明 $A>0$.\n由加权平均值不等式, 得\n$$\n4 a^6+b^6+c^6 \\geqslant 6 a^4 b c, 3 a^5 b+3 a^5 c+b^5 a+c^5 a \\geqslant 8 a^4 b c,\n$$\n得\n$$\n\\sum_{s y m} 6 a^6 \\geqslant \\sum_{s y m} 6 a^4 b c, \\sum_{s y m} 8 a^5 b \\geqslant \\sum_{s y m} 8 a^4 b c .\n$$\n于是 $\\quad \\sum_{s y m}\\left(4 a^6+4 a^5 b+5 a^4 b c\\right) \\geqslant \\sum_{s y m} 13 a^4 b c$.\n再由平均值不等式, 得\n$$\na^4 b^2+b^4 c^2+c^4 a^2 \\geqslant 3 a^2 b^2 c^2, a^3 b^3+b^3 c^3+c^3 a^3 \\geqslant 3 a^2 b^2 c^2,\n$$\n从而\n$$\n\\sum_{\\text {sym }}\\left(a^4 b^2+5 a^3 b^3\\right) \\geqslant \\sum_{s y m} 6 a^2 b^2 c^2,\n$$\n或者\n$$\n\\sum_{s y m}\\left(a^4 b^2+5 a^3 b^3+7 a^2 b^2 c^2\\right) \\geqslant \\sum_{s y m} 13 a^2 b^2 c^2 .\n$$\n回顾 Schur 不等式,\n$$\n\\begin{aligned}\n& a^3+b^3+c^3+3 a b c-\\left(a^2 b+b^2 c+c^2 a+a b^2+b c^2+c a^2\\right) \\\\\n= & a(a-b)(a-c)+b(b-a)(b-c)+c(c-a)(c-b) \\geqslant 0\n\\end{aligned}\n$$\n或者\n$$\n\\sum_{s y m}\\left(a^3-2 a^2 b+a b c\\right) \\geqslant 0,\n$$\n于是 $\\sum_{\\text {sym }}\\left(13 a^4 b c-26 a^3 b^2 c+13 a^2 b^2 c^2\\right) \\geqslant 13 a b c \\sum_{s y m}\\left(a^3-2 a^2 b+a b c\\right) \\geqslant 0$.\n综上可得 $A>0$. 证毕.", + "remark": "", + "figures": [] +} \ No newline at end of file diff --git a/processed_dataset/proof/1111.json b/processed_dataset/proof/1111.json new file mode 100644 index 0000000000000000000000000000000000000000..3b82b6cedc5c76066b5d77f2cf3c4cdd15b23e19 --- /dev/null +++ b/processed_dataset/proof/1111.json @@ -0,0 +1,8 @@ +{ + "source_file": "./raw_volume-zh/volume4/chapter2-5.tex", + "problem_type": "proof", + "problem": "例6. 求最小的正整数 $k$, 使得对满足 $0 \\leqslant a \\leqslant 1$ 的所有 $a$ 和所有正整数 $n$, 都有不等式\n$$\na^k(1-a)^n \\leqslant \\frac{1}{(n+1)^3} .\n$$", + "solution": "解:先设法消除参数 $a$, 然后求 $k$ 的最小值.\n由平均值不等式, 得\n$$\n\\begin{gathered}\n\\sqrt[n+k]{a^k\\left[\\frac{k}{n}(1-a)\\right]^n} \\leqslant \\frac{k a+n\\left[\\frac{k}{n}(1-a)\\right]}{k+n}=\\frac{k}{k+n}, \\\\\na^k(1-a)^n \\leqslant \\frac{k^k n^n}{(n+k)^{n+k}},\n\\end{gathered}\n$$\n所以\n$$\na^k(1-a)^n \\leqslant \\frac{k^k n^n}{(n+k)^{n+k}}\n$$\n当且仅当 $a=\\frac{k(1-a)}{n}$, 即 $a=\\frac{k}{n+k}$ 时, 等号成立.\n于是我们要求出最小的正整数 $k$, 使得对任何正整数 $n$ 都有\n$$\n\\frac{k^k n^n}{(n+k)^{n+k}}<\\frac{1}{(1+n)^3} \\text {. }\n$$\n当 $k=1$ 时,取 $n=1$, 上式为 $\\frac{1}{2^2}<\\frac{1}{2^3}$, 矛盾.\n当 $k=2$ 时,取 $n=1$, 则 $\\frac{4}{27}<\\frac{1}{8}$, 亦矛盾.\n当 $k=3$ 时,取 $n=3$, 则 $\\frac{1}{64}<\\frac{1}{64}$, 矛盾.\n因此 $k \\geqslant 4$. 下面证明 $k=4$ 时命题成立, 即\n$$\n4^4 n^n(n+1)^3<(n+4)^{n+4} \\text {. }\n$$\n当 $n=1,2,3$ 时,容易证明成立.\n当 $n \\geqslant 4$ 时, 再由平均值不等式, 得\n$$\n\\begin{aligned}\n\\sqrt[n+4]{4^4 n^n(n+1)^3} & =\\sqrt[n+4]{16(2 n)(2 n)(2 n)(2 n) n^{n-4}(n+1)^3} \\\\\n& \\leqslant \\frac{16+8 n+n(n-4)+3(n+1)}{n+4} \\\\\n& =\\frac{n^2+7 n+19}{n+4}<\\frac{n^2+8 n+16}{n+4}=n+4,\n\\end{aligned}\n$$\n故 $k$ 的最小值为 4 .", + "remark": "注:在第一次利用平均值不等式时, 引进参数 $\\alpha=\\frac{k}{n}$, 消除 $a$, 具有很强的技巧性.", + "figures": [] +} \ No newline at end of file diff --git a/processed_dataset/proof/1112.json b/processed_dataset/proof/1112.json new file mode 100644 index 0000000000000000000000000000000000000000..dc119d8aae721364574d8ddd9c08aac0fed024f0 --- /dev/null +++ b/processed_dataset/proof/1112.json @@ -0,0 +1,8 @@ +{ + "source_file": "./raw_volume-zh/volume4/chapter4-1.tex", + "problem_type": "proof", + "problem": "例1. 已知 $a, b, c \\in \\mathbf{R}^{+}$, 且 $a+b+c=1$, 求证:\n$$\n36 \\leqslant \\frac{1}{a}+\\frac{4}{b}+\\frac{9}{c} .\n$$", + "solution": "证明:由柯西不等式,得\n$$\n\\begin{aligned}\n\\frac{1}{a}+\\frac{4}{b}+\\frac{9}{c} & =\\left(\\frac{1}{a}+\\frac{4}{b}+\\frac{9}{c}\\right) \\cdot(a+b+c) \\\\\n& \\geqslant\\left(\\sqrt{a} \\cdot \\frac{1}{\\sqrt{a}}+\\sqrt{b} \\cdot \\frac{2}{\\sqrt{b}}+\\sqrt{c} \\cdot-\\frac{3}{\\sqrt{c}}\\right)^2=36,\n\\end{aligned}\n$$\n所以\n$$\n\\frac{1}{a}+\\frac{4}{b}+\\frac{9}{c} \\geqslant 36 \\text {. }\n$$", + "remark": "", + "figures": [] +} \ No newline at end of file diff --git a/processed_dataset/proof/1113.json b/processed_dataset/proof/1113.json new file mode 100644 index 0000000000000000000000000000000000000000..7471f7685ebf52a558fe12bcfec9a80d497c7198 --- /dev/null +++ b/processed_dataset/proof/1113.json @@ -0,0 +1,8 @@ +{ + "source_file": "./raw_volume-zh/volume4/chapter4-1.tex", + "problem_type": "proof", + "problem": "例2. 设 $a, b, c \\in \\mathbf{R}^{+}$, 满足 $a \\cos ^2 \\alpha+b \\sin ^2 \\alpha0(i=1,2, \\cdots, n)$ 满足 $\\sum_{i=1}^n a_i=1$, 求证:\n$$\n\\frac{a_1^2}{a_1+a_2}+\\frac{a_2^2}{a_2+a_3}+\\cdots+\\frac{a_n^2}{a_n+a_1} \\geqslant \\frac{1}{2} .\n$$", + "solution": "证明:令 $a_{n+1}=a_1$, 由柯西不等式, 得\n$$\n\\begin{aligned}\n&\\left(\\sum_{i=1}^n a_i\\right)^2=\\left(\\sum_{i=1}^n \\frac{a_i}{\\sqrt{a_i+a_{i+1}}} \\cdot \\sqrt{a_i+a_{i+1}}\\right)^2 \\\\\n& \\leqslant \\sum_{i=1}^n \\frac{a_i^2}{a_i+a_{i+1}} \\cdot \\sum_{i=1}^n\\left(a_i+a_{i+1}\\right) \\\\\n&=2 \\sum_{i=1}^n \\frac{a_i^2}{a_i+a_{i+1}} \\cdot \\sum_{i=1}^n a_i, \\\\\n& \\sum_{i=1}^n \\frac{a_i^2}{a_i+a_{i+1}} \\geqslant \\frac{1}{2} \\sum_{i=1}^n a_i=\\frac{1}{2} .\n\\end{aligned}\n$$\n于是\n$$\n\\sum_{i=1}^n \\frac{a_i^2}{a_i+a_{i+1}} \\geqslant \\frac{1}{2} \\sum_{i=1}^n a_i=\\frac{1}{2} .\n$$", + "remark": "注:在证明过程中,注意条件的利用和不等式的变形.", + "figures": [] +} \ No newline at end of file diff --git a/processed_dataset/proof/1115.json b/processed_dataset/proof/1115.json new file mode 100644 index 0000000000000000000000000000000000000000..6b3fd22819412c6adc765465443867fdbea13589 --- /dev/null +++ b/processed_dataset/proof/1115.json @@ -0,0 +1,8 @@ +{ + "source_file": "./raw_volume-zh/volume4/chapter4-1.tex", + "problem_type": "proof", + "problem": "例4. 设 $a 、 b 、 c$ 是正实数, 且满足 $a+b+c=1$. 证明:\n$$\n\\frac{a-b c}{a+b c}+\\frac{b-c a}{b+c a}+\\frac{c-a b}{c+a b} \\leqslant \\frac{3}{2} .\n$$", + "solution": "证明:注意到\n$$\n1-\\frac{a-b c}{a+b c}=\\frac{2 b c}{a+b c}=\\frac{2 b c}{1-b-c+b c}=\\frac{2 b c}{(1-b)(1-c)} .\n$$\n同理, $1-\\frac{b-c a}{b+c a}=\\frac{2 c a}{(1-c)(1-a)}, 1-\\frac{c-a b}{c+a b}=\\frac{2 a b}{(1-a)(1-b)}$. 故原不等式等价于\n$$\n\\frac{2 b c}{(1-b)(1-c)}+\\frac{2 c a}{(1-c)(1-a)}+\\frac{2 a b}{(1-a)(1-b)} \\geqslant \\frac{3}{2} .\n$$\n化简后得\n$$\n\\begin{aligned}\n& 4(b c+c a+a b-3 a b c) \\\\\n\\geqslant & 3(b c+c a+a b+1-a-b-c-a b c),\n\\end{aligned}\n$$\n即\n$$\na b+b c+c a \\geqslant 9 a b c .\n$$\n从而要证 $\\frac{1}{a}+\\frac{1}{b}+\\frac{1}{c} \\geqslant 9$.\n而 $\\frac{1}{a}+\\frac{1}{b}+\\frac{1}{c}=(a+b+c)\\left(\\frac{1}{a}+\\frac{1}{b}+\\frac{1}{c}\\right) \\geqslant 9$, 因此, 原不等式成立.", + "remark": "", + "figures": [] +} \ No newline at end of file diff --git a/processed_dataset/proof/1116.json b/processed_dataset/proof/1116.json new file mode 100644 index 0000000000000000000000000000000000000000..1b4038009917fa26df97ba8fd349ae49dd83af10 --- /dev/null +++ b/processed_dataset/proof/1116.json @@ -0,0 +1,8 @@ +{ + "source_file": "./raw_volume-zh/volume4/chapter4-1.tex", + "problem_type": "proof", + "problem": "例5. 设正实数 $a 、 b 、 c$ 满足 $a+b+c=3$. 证明:\n$$\n\\frac{1}{2+a^2+b^2}+\\frac{1}{2+b^2+c^2}+\\frac{1}{2+c^2+a^2} \\leqslant \\frac{3}{4} .\n$$", + "solution": "证明:用符号 $\\sum$ 表示循环和, 即证明:\n$$\n\\sum \\frac{1}{2+a^2+b^2} \\leqslant \\frac{3}{4} . \\label{eq1}\n$$\n由柯西不等式得\n$$\n\\left(\\sum \\frac{a^2+b^2}{2+a^2+b^2}\\right) \\sum\\left(2+a^2+b^2\\right) \\geqslant\\left(\\sum \\sqrt{a^2+b^2}\\right)^2 .\n$$\n又 $\\quad\\left(\\sum \\sqrt{a^2+b^2}\\right)^2=2 \\sum a^2+2 \\sum \\sqrt{\\left(a^2+b^2\\right)\\left(a^2+c^2\\right)}$,\n及\n$$\n\\sqrt{\\left(a^2+b^2\\right)\\left(a^2+c^2\\right)} \\geqslant a^2+b c,\n$$\n则\n$$\n\\begin{aligned}\n& \\left(\\sum \\sqrt{a^2+b^2}\\right)^2 \\\\\n\\geqslant & 2 \\sum a^2+2 \\sum a^2+2 \\sum a b \\\\\n= & 3 \\sum a^2+(a+b+c)^2=9+3 \\sum a^2 \\\\\n= & \\frac{3}{2}\\left(6+2 \\sum a^2\\right)=\\frac{3}{2} \\sum\\left(2+a^2+b^2\\right) .\n\\end{aligned}\n$$\n故 $\\left(\\sum \\frac{a^2+b^2}{2+a^2+b^2}\\right) \\sum\\left(2+a^2+b^2\\right) \\geqslant \\frac{3}{2} \\sum\\left(2+a^2+b^2\\right)$.\n所以,\n$$\n\\sum \\frac{a^2+b^2}{2+a^2+b^2} \\geqslant \\frac{3}{2} . \\label{eq2}\n$$\n式\\ref{eq2}两边乘以 -1 , 再加 3 , 再除以 2 即得式 \\ref{eq1}.", + "remark": "", + "figures": [] +} \ No newline at end of file diff --git a/processed_dataset/proof/1117.json b/processed_dataset/proof/1117.json new file mode 100644 index 0000000000000000000000000000000000000000..92d24348fe6d820d9d1e28e0f7107f318630f2f8 --- /dev/null +++ b/processed_dataset/proof/1117.json @@ -0,0 +1,8 @@ +{ + "source_file": "./raw_volume-zh/volume4/chapter4-1.tex", + "problem_type": "proof", + "problem": "例6. 已知 $x, y, z>0$, 且 $x y z=1$. 求证:\n$$\n\\frac{(x+y-1)^2}{z}+\\frac{(y+z-1)^2}{x}+\\frac{(z+x-1)^2}{y} \\geqslant 4(x+y+z)-12+\\frac{9}{x+y+z} .\n$$", + "solution": "证明:因为\n$$\n(a-b)^2=a^2-2 a b+b^2,\n$$\n所以\n$$\n\\begin{gathered}\na^2=2 a b-b^2+(a-b)^2, \\\\\n\\frac{a^2}{b}=2 a-b+\\frac{(a-b)^2}{b} .\n\\end{gathered}\n$$\n利用上式及柯西不等式,可知\n$$\n\\begin{aligned}\n& \\frac{(x+y-1)^2}{z}+\\frac{(y+z-1)^2}{x}+\\frac{(z+x-1)^2}{y} \\\\\n= & 2(x+y-1)-z+\\frac{(x+y-z-1)^2}{z} \\\\\n& +2(y+z-1)-x+\\frac{(y+z-x-1)^2}{x} \\\\\n& +2(z+x-1)-y+\\frac{(z+x-y-1)^2}{y} \\\\\n= & 3(x+y+z)-6+\\frac{(x+y-z-1)^2}{z}+\\frac{(y+z-x-1)^2}{x}+\\frac{(z+x-y-1)^2}{y} \\\\\n\\geqslant & 3(x+y+z)-6+\\frac{(x+y+z-3)^2}{x+y+z} \\\\\n= & 3(x+y+z)-6+\\frac{(x+y+z)^2-6(x+y+z)+9}{x+y+z} \\\\\n= & 4(x+y+z)-12+\\frac{9}{x+y+z} .\n\\end{aligned}\n$$", + "remark": "", + "figures": [] +} \ No newline at end of file diff --git a/processed_dataset/proof/1118.json b/processed_dataset/proof/1118.json new file mode 100644 index 0000000000000000000000000000000000000000..cc22a9ce119c111122f97b6b28f016a089235141 --- /dev/null +++ b/processed_dataset/proof/1118.json @@ -0,0 +1,8 @@ +{ + "source_file": "./raw_volume-zh/volume4/chapter4-1.tex", + "problem_type": "proof", + "problem": "例7. 设非负实数 $a_1, a_2, \\cdots, a_n$ 与 $b_1, b_2, \\cdots, b_n$ 同时满足以下条件:\n(1) $\\sum_{i=1}^n\\left(a_i+b_i\\right)=1$;\n(2) $\\sum_{i=1}^n i\\left(a_i-b_i\\right)=0$;\n(3) $\\sum_{i=1}^n i^2\\left(a_i+b_i\\right)=10$.\n求证: 对任意 $1 \\leqslant k \\leqslant n$, 都有 $\\max \\left\\{a_k, b_k\\right\\} \\leqslant \\frac{10}{10+k^2}$.", + "solution": "证明:对任意 $1 \\leqslant k \\leqslant n$, 有\n$$\n\\begin{aligned}\n\\left(k a_k\\right)^2 & \\leqslant\\left(\\sum_{i=1}^n i a_i\\right)^2=\\left(\\sum_{i=1}^n i b_i\\right)^2 \\\\\n& \\leqslant\\left(\\sum_{i=1}^n i^2 b_i\\right) \\cdot\\left(\\sum_{i=1}^n b_i\\right) \\quad \\text { (柯西不等式) } \\\\\n& =\\left(10-\\sum_{i=1}^n i^2 a_i\\right) \\cdot\\left(1-\\sum_{i=1}^n a_i\\right) \\\\\n& \\leqslant\\left(10-k^2 a_k\\right) \\cdot\\left(1-a_k\\right)=10-\\left(10+k^2\\right) a_k+k^2 a_k^2,\n\\end{aligned}\n$$\n(柯西不等式)\n从而 $a_k \\leqslant \\frac{10}{10+k^2}$.\n同理有 $b_k \\leqslant \\frac{10}{10+k^2}$, 所以 $\\max \\left\\{a_k, b_k\\right\\} \\leqslant \\frac{10}{10+k^2}$.", + "remark": "", + "figures": [] +} \ No newline at end of file diff --git a/processed_dataset/proof/1119.json b/processed_dataset/proof/1119.json new file mode 100644 index 0000000000000000000000000000000000000000..2c909beeac68a8e1efb53e6a417e10708fd999ba --- /dev/null +++ b/processed_dataset/proof/1119.json @@ -0,0 +1,8 @@ +{ + "source_file": "./raw_volume-zh/volume4/chapter4-1.tex", + "problem_type": "proof", + "problem": "例8. 设 $a_i \\in \\mathbf{R}^{+}(i=1,2, \\cdots, n)$, 如果对任意 $x_i \\geqslant 0$,\n$$\n\\sum_{i=1}^n r_i\\left(x_i-a_i\\right) \\leqslant \\sqrt{\\sum_{i=1}^n x_i^2}-\\sqrt{\\sum_{i=1}^n a_i^2} .\n$$\n求 $r_i(i=1,2, \\cdots, n)$.", + "solution": "解:令 $x_i=0$, 则\n$$\n\\sum_{i=1}^n r_i a_i \\geqslant \\sqrt{\\sum_{i=1}^n a_i^2}\n$$\n再令 $x_i=2 a_i$, 则 $\\sum_{i=1}^n r_i a_i \\leqslant \\sqrt{\\sum_{i=1}^n a_i^2}$.\n于是\n$$\n\\sum_{i=1}^n r_i a_i=\\sqrt{\\sum_{i=1}^n a_i^2}\n$$\n令 $x_i=r_i$, 则 $\\sum_{i=1}^n r_i\\left(r_i-a_i\\right) \\leqslant \\sqrt{\\sum_{i=1}^n r_i^2}-\\sqrt{\\sum_{i=1}^n a_i^2}$,\n推出\n$$\n\\sum_{i=1}^n r_i^2 \\leqslant \\sqrt{\\sum_{i=1}^n r_i^2}\n$$\n即 $\\sum_{i=1}^n r_i^2 \\leqslant 1$. 由柯西不等式, 得\n$$\n\\left(\\sum_{i=1}^n r_i a_i\\right)^2 \\leqslant\\left(\\sum_{i=1}^n r_i^2\\right)\\left(\\sum_{i=1}^n a_i^2\\right),\n$$\n等号成立充要条件是 $r_i=\\lambda a_i$.\n从而 $\\sum_{i=1}^n r_i^2 \\geqslant 1$, 于是\n$$\n\\sum_{i=1}^n r_i^2=1, \\lambda=\\frac{1}{\\sqrt{\\sum_{i=1}^n a_i^2}}, r_i=-\\frac{a_i}{\\sqrt{\\sum_{i=1}^n a_i^2}} .\n$$\n经验证, $r_i=\\frac{a_i}{\\sqrt{\\sum_{i=1}^n a_i^2}}(i=1,2, \\cdots, n)$ 为所求.\n上面的条件可以改为一般的形式:\n$$\n\\sum_{i=1}^n r_i\\left(x_i-a_i\\right) \\leqslant\\left(\\sum_{i=1}^n x_i^m\\right)^{\\frac{1}{m}}-\\left(\\sum_{i=1}^n a_i^m\\right)^{\\frac{1}{m}},\n$$\n其中 $m>1$ 为给定的常数.\n利用赫尔得不等式, 得\n$$\nr_i=\\left[\\frac{a_i^m}{\\sum_{i=1}^n a_i^m}\\right]^{\\frac{m-1}{m}}(i=1,2, \\cdots, n) .\n$$", + "remark": "", + "figures": [] +} \ No newline at end of file diff --git a/processed_dataset/proof/1120.json b/processed_dataset/proof/1120.json new file mode 100644 index 0000000000000000000000000000000000000000..ed65f005aefe0af1a8f258092f75d3a6774eaaab --- /dev/null +++ b/processed_dataset/proof/1120.json @@ -0,0 +1,8 @@ +{ + "source_file": "./raw_volume-zh/volume4/chapter4-1.tex", + "problem_type": "proof", + "problem": "例9. 设 $x_i, y_i, \\cdots, z_i \\in \\mathbf{R}(i=1,2, \\cdots, n)$, 求证:\n$$\n\\sum_{i=1}^n \\sqrt{x_i^2+y_i^2+\\cdots+z_i^2} \\geqslant \\sqrt{\\left(\\sum_{i=1}^n x_i\\right)^2+\\left(\\sum_{i=1}^n y_i\\right)^2+\\cdots+\\left(\\sum_{i=1}^n z_i\\right)^2} .\n$$", + "solution": "证明:令 $a=\\sum_{i=1}^n x_i, b=\\sum_{i=1}^n y_i, \\cdots, c=\\sum_{i=1}^n z_i$. 不妨设 $a^2+b^2+\\cdots+ c^2 \\neq 0$, 则由柯西不等式, 得\n$$\n\\left(a^2+b^2+\\cdots+c^2\\right)\\left(x_i^2+y_i^2+\\cdots+z_i^2\\right) \\geqslant\\left(a x_i+b y_i+\\cdots+c z_i\\right)^2,\n$$\n即\n$$\na x_i+b y_i+\\cdots+c z_i \\leqslant \\sqrt{a^2+b^2+\\cdots+c^2} \\cdot \\sqrt{x_i^2+y_i^2+\\cdots+z_i^2} .\n$$\n求和, 得\n$$\na^2+b^2+\\cdots+c^2 \\leqslant \\sqrt{a^2+b^2+\\cdots+c^2} \\sum_{i=1}^n \\sqrt{x_i^2+y_i^2+\\cdots+z_i^2} .\n$$\n故 $\\quad \\sum_{i=1}^n \\sqrt{x_i^2+y_i^2+\\cdots+z_i^2} \\geqslant \\sqrt{a^2+b^2+\\cdots+c^2}$.\n本例如果用向量方法证明, 会更简洁.", + "remark": "", + "figures": [] +} \ No newline at end of file diff --git a/processed_dataset/proof/1121.json b/processed_dataset/proof/1121.json new file mode 100644 index 0000000000000000000000000000000000000000..6c2b40500e6944dad454b1769e400d0eda76f981 --- /dev/null +++ b/processed_dataset/proof/1121.json @@ -0,0 +1,8 @@ +{ + "source_file": "./raw_volume-zh/volume4/chapter4-1.tex", + "problem_type": "proof", + "problem": "例10. 设 $a_i \\in \\mathbf{R}^{+}, 1 \\leqslant i \\leqslant n$. 证明:\n$$\n\\frac{1}{\\frac{1}{1+a_1}+\\frac{1}{1+a_2}+\\cdots+\\frac{1}{1+a_n}}-\\frac{1}{\\frac{1}{a_1}+\\frac{1}{a_2}+\\cdots+\\frac{1}{a_n}} \\geqslant \\frac{1}{n} \\text {. }\n$$", + "solution": "证明:令 $\\sum_{i=1}^n \\frac{1}{a_i}=a$, 则 $\\sum_{i=1}^n \\frac{1+a_i}{a_i}=n+a$. 由柯西不等式, 得\n$$\n\\sum_{i=1}^n \\frac{a_i}{1+a_i} \\cdot \\sum_{i=1}^n \\frac{1+a_i}{a_i} \\geqslant n^2 .\n$$\n所以 $\\sum_{i=1}^n \\frac{a_i}{a_i+1} \\geqslant \\frac{n^2}{n+a}$, 以及\n$$\n\\begin{aligned}\n\\sum_{i=1}^n \\frac{1}{a_i+1} & =\\sum_{i=1}^n\\left(1-\\frac{a_i}{a_i+1}\\right)=n-\\sum_{i=1}^n \\frac{a_i}{a_i+1} \\\\\n& \\leqslant n-\\frac{n^2}{n+a}=-\\frac{n a}{n+a} .\n\\end{aligned}\n$$\n于是\n$$\n\\begin{aligned}\n& \\frac{1}{\\frac{1}{1+a_1}+\\frac{1}{1+a_2}+\\cdots+\\frac{1}{1+a_n}}-\\frac{1}{\\frac{1}{a_1}+\\frac{1}{a_2}+\\cdots+\\frac{1}{a_n}} \\\\\n\\geqslant & \\frac{1}{\\frac{n a}{n+a}}-\\frac{1}{a}=\\frac{n+a}{n a}-\\frac{1}{a}=\\frac{a}{n a}=\\frac{1}{n},\n\\end{aligned}\n$$\n故命题成立.", + "remark": "", + "figures": [] +} \ No newline at end of file diff --git a/processed_dataset/proof/1122.json b/processed_dataset/proof/1122.json new file mode 100644 index 0000000000000000000000000000000000000000..70ccdda2d11c65422fa331ebf3d9837bcebdbd48 --- /dev/null +++ b/processed_dataset/proof/1122.json @@ -0,0 +1,8 @@ +{ + "source_file": "./raw_volume-zh/volume4/chapter4-1.tex", + "problem_type": "proof", + "problem": "例11. 设 $n$ 为正整数, $x_1 \\leqslant x_2 \\leqslant \\cdots \\leqslant x_n$ 为实数, 证明:\n(1) $\\left(\\sum_{i, j=1}^n\\left|x_i-x_j\\right|\\right)^2 \\leqslant \\frac{2\\left(n^2-1\\right)}{3} \\sum_{i, j=1}^n\\left(x_i-x_j\\right)^2$;\n(2) 第 (1) 小题等号成立的充要条件是 $x_1, x_2, \\cdots, x_n$ 为等差数列.", + "solution": "证明:(1) 不失一般性, 可设 $\\sum_{i=1}^n x_i=0$, 得\n$$\n\\sum_{i, j=1}^n\\left|x_i-x_j\\right|=2 \\sum_{i0, x_i y_i-z_i^2>0, i=1,2$, 求证:\n$$\n\\frac{8}{\\left(x_1+x_2\\right)\\left(y_1+y_2\\right)-\\left(z_1+z_2\\right)^2} \\leqslant \\frac{1}{x_1 y_1-z_1^2}+\\frac{1}{x_2 y_2-z_2^2} .\n$$", + "solution": "证明:注意到不等式的右边 $\\geqslant \\frac{2}{\\left[\\left(x_1 y_1-z_1^2\\right)\\left(x_2 y_2-z_2^2\\right)\\right]^{\\frac{1}{2}}}$, 考虑证明一个更强的结论:\n$$\n\\left(x_1+x_2\\right)\\left(y_1+y_2\\right)-\\left(z_1+z_2\\right)^2 \\geqslant 4\\left[\\left(x_1 y_1-z_1^2\\right)\\left(x_2 y_2-z_2^2\\right)\\right]^{\\frac{1}{2}} .\n$$\n令 $u_i=\\sqrt{x_i y_i-z_i^2}, i=1,2$, 由于 $4 u_1 u_2 \\leqslant\\left(u_1+u_2\\right)^2$, 则只要证明\n$$\n\\left(x_1+x_2\\right)\\left(y_1+y_2\\right)-\\left(z_1+z_2\\right)^2 \\geqslant\\left(u_1+u_2\\right)^2 \\text {, }\n$$\n等价于\n$$\n\\left(x_1+x_2\\right)\\left(y_1+y_2\\right) \\geqslant\\left(u_1+u_2\\right)^2+\\left(z_1+z_2\\right)^2 .\n$$\n由柯西不等式,得\n$$\n\\begin{aligned}\n\\left(x_1+x_2\\right)\\left(y_1+y_2\\right) & \\geqslant\\left(\\sqrt{x_1 y_1}+\\sqrt{x_2 y_2}\\right)^2 \\\\\n& =\\left(\\sqrt{u_1^2+z_1^2}+\\sqrt{u_2^2+z_2^2}\\right)^2 \\\\\n& =\\left(u_1^2+z_1^2\\right)+2 \\sqrt{u_1^2+z_1^2} \\sqrt{u_2^2+z_2^2}+\\left(u_2^2+z_2^2\\right) \\\\\n& \\geqslant\\left(u_1^2+z_1^2\\right)+2\\left(u_1 u_2+z_1 z_2\\right)+\\left(u_2^2+z_2^2\\right) \\\\\n& =\\left(u_1+u_2\\right)^2+\\left(z_1+z_2\\right)^2 .\n\\end{aligned}\n$$\n从而原不等式成立, 且等号成立的充分必要条件为 $x_1=x_2, y_1=y_2$, $z_1=z_2$.", + "remark": "注:.\n该例题也可以直接用柯西不等式证明, 关于它的推广, 可以参见引文或练习.", + "figures": [] +} \ No newline at end of file diff --git a/processed_dataset/proof/1127.json b/processed_dataset/proof/1127.json new file mode 100644 index 0000000000000000000000000000000000000000..83f125d5078e8afc1bd14aa801e144385db36dd1 --- /dev/null +++ b/processed_dataset/proof/1127.json @@ -0,0 +1,8 @@ +{ + "source_file": "./raw_volume-zh/volume4/chapter4-1.tex", + "problem_type": "proof", + "problem": "例16. 设 $a_i, b_i, c_i, d_i \\in \\mathbf{R}^{+}(i=1,2, \\cdots, n)$, 求证:\n$$\n\\left(\\sum a_i b_i c_i d_i\\right)^4 \\leqslant \\sum a_i^4 \\cdot \\sum b_i^4 \\cdot \\sum c_i^4 \\cdot \\sum d_i^4 .\n$$", + "solution": "证明:两次利用柯西不等式, 得\n$$\n\\begin{aligned}\n\\text { 左边 } & =\\left[\\sum\\left(a_i b_i\\right)\\left(c_i d_i\\right)\\right]^4 \\\\\n& \\leqslant\\left[\\sum\\left(a_i b_i\\right)^2\\right]^2 \\cdot\\left[\\sum\\left(c_i d_i\\right)^2\\right]^2=\\left[\\sum a_i^2 b_i^2\\right]^2\\left[\\sum c_i^2 d_i^2\\right]^2 \\\\\n& \\leqslant \\sum a_i^4 \\cdot \\sum b_i^4 \\cdot \\sum c_i^4 \\cdot \\sum d_i^4 .\n\\end{aligned}\n$$\n故命题成立.", + "remark": "", + "figures": [] +} \ No newline at end of file diff --git a/processed_dataset/proof/1128.json b/processed_dataset/proof/1128.json new file mode 100644 index 0000000000000000000000000000000000000000..a309524a26e728882d426c4c9e7b1876675d4b3c --- /dev/null +++ b/processed_dataset/proof/1128.json @@ -0,0 +1,8 @@ +{ + "source_file": "./raw_volume-zh/volume4/chapter4-1.tex", + "problem_type": "proof", + "problem": "例17. 设 $t_a 、 t_b 、 t_c$ 分别是 $\\triangle A B C$ 的 $\\angle A 、 \\angle B 、 \\angle C$ 的角平分线的长, 证明:\n$$\n\\sum \\frac{b c}{t_a^2} \\geqslant 4\n$$", + "solution": "证明:不难求得 $t_a^2=\\frac{b c\\left[(b+c)^2-a^2\\right]}{(b+c)^2}$, 则 $\\frac{b c}{t_a^2}=\\frac{(b+c)^2}{(b+c)^2 \\div a^2}$.\n同理可得\n$$\n\\frac{a c}{t_b^2}=\\frac{(a+c)^2}{(a+c)^2-b^2}, \\frac{a b}{t_c^2}=\\frac{(a+b)^2}{(a+b)^2-c^2}\n$$\n则\n$$\n\\sum \\frac{b c}{t_a^2}=\\sum \\frac{(a+b)^2}{(a+b)^2-c^2} \\geqslant \\frac{4(a+b+c)^2}{\\sum\\left[(a+b)^2-c^2\\right]}=\\frac{4(a+b+c)^2}{(a+b+c)^2}=4 .\n$$\n所以原不等式成立.", + "remark": "", + "figures": [] +} \ No newline at end of file diff --git a/processed_dataset/proof/1129.json b/processed_dataset/proof/1129.json new file mode 100644 index 0000000000000000000000000000000000000000..daa4a6ed8ae3ec398525a13b7bebe92c04d6146e --- /dev/null +++ b/processed_dataset/proof/1129.json @@ -0,0 +1,8 @@ +{ + "source_file": "./raw_volume-zh/volume4/chapter4-1.tex", + "problem_type": "proof", + "problem": "例18. 设 $a 、 b 、 c 、 d$ 为正实数, 满足 $a b+c d=1$, 点 $P_i\\left(x_i, y_i\\right)(i=1$, $2,3,4)$ 是以原点为圆心的单位圆上的四点.\n求证:\n$$\n\\begin{aligned}\n& \\left(a y_1+b y_2+c y_3+d y_4\\right)^2+\\left(a x_4+b x_3+c x_2+d x_1\\right)^2 \\\\\n\\leqslant & 2\\left(\\frac{a^2+b^2}{a b}+\\frac{c^2+d^2}{c d}\\right) .\n\\end{aligned}\n$$", + "solution": "证明:令 $\\alpha=a y_1+b y_2+c y_3+d y_4, \\beta=a x_4+b x_3+c x_2+d x_1$, 由柯西不等式, 得\n$$\n\\begin{aligned}\n\\alpha^2= & \\left(a y_1+b y_2+c y_3+d y_4\\right)^2 \\\\\n\\leqslant & {\\left[\\left(\\sqrt{a d} y_1\\right)^2+\\left(\\sqrt{b c} y_2\\right)^2+\\left(\\sqrt{b c} y_3\\right)^2+\\left(\\sqrt{a d} y_4\\right)^2\\right] } \\\\\n& {\\left[\\left(\\sqrt{\\frac{a}{d}}\\right)^2+\\left(\\sqrt{\\frac{b}{c}}\\right)^2+\\left(\\sqrt{\\frac{c}{b}}\\right)^2+\\left(\\sqrt{\\frac{d}{a}}\\right)^2\\right] } \\\\\n= & \\left(a d y_1^2+b c y_2^2+b c y_3^2+a d y_4^2\\right) \\cdot\\left(\\frac{a}{d}+\\frac{b}{c}+\\frac{c}{b}+\\frac{d}{a}\\right) .\n\\end{aligned}\n$$\n同理可得\n$$\n\\beta^2 \\leqslant\\left(a d x_4^2+b c x_3^2+b c x_2^2+a d x_1^2\\right) \\cdot\\left(\\frac{a}{d}+\\frac{b}{c}+\\frac{c}{b}+\\frac{d}{a}\\right) .\n$$\n将它们相加, 并利用 $x_i^2+y_i^2=1(i=1,2,3,4), a b+c d=1$, 得\n$$\n\\begin{aligned}\n\\alpha^2+\\beta^2 & \\leqslant(2 a d+2 b c)\\left(\\frac{a}{d}+\\frac{b}{c}+\\frac{c}{b}+\\frac{d}{a}\\right) \\\\\n& =2(a d+b c)\\left(\\frac{a b+c d}{b d}+\\frac{a b+c d}{a c}\\right) \\\\\n& =2(a d+b c)\\left(\\frac{1}{b d}+\\frac{1}{a c}\\right) \\\\\n& =2\\left(\\frac{a^2+b^2}{a b}+\\frac{c^2+d^2}{c d}\\right),\n\\end{aligned}\n$$\n故命题成立.", + "remark": "", + "figures": [] +} \ No newline at end of file diff --git a/processed_dataset/proof/1130.json b/processed_dataset/proof/1130.json new file mode 100644 index 0000000000000000000000000000000000000000..0c9cbbc64740f010434c64b5e2d090c3632042e3 --- /dev/null +++ b/processed_dataset/proof/1130.json @@ -0,0 +1,8 @@ +{ + "source_file": "./raw_volume-zh/volume4/chapter4-1.tex", + "problem_type": "proof", + "problem": "例19. 给定正整数 $n \\geqslant 2$, 设正整数 $a_i(i=1,2, \\cdots, n)$ 满足 $a_1< a_2<\\cdots0$, 且 $\\sum_{i=1}^n a_i=k$, 求证:\n$$\n\\sum_{i=1}^n\\left(a_i+\\frac{1}{a_i}\\right)^2 \\geqslant n\\left(\\frac{n^2+k^2}{n k}\\right)^2 .\n$$", + "solution": "证明:由柯西不等式, 得\n$$\n\\begin{aligned}\n& \\left(1^2+1^2+\\cdots+1^2\\right) \\sum_{i=1}^n\\left(a_i+\\frac{1}{a_i}\\right)^2 \\\\\n\\geqslant & {\\left[\\sum_{i=1}^n\\left(a_i+\\frac{1}{a_i}\\right)\\right]^2=\\left(k+\\sum_{i=1}^n \\frac{1}{a_i}\\right)^2 } \\\\\n\\geqslant & \\left(k+\\frac{n^2}{\\sum_{i=1}^n a_i}\\right]^2=\\left(\\frac{k^2+n^2}{k}\\right)^2,\n\\end{aligned}\n$$\n所以\n$$\n\\sum_{i=1}^n\\left(a_i+\\frac{1}{a_i}\\right)^2 \\geqslant n\\left(\\frac{n^2+k^2}{n k}\\right)^2 .\n$$\n利用变形的赫尔得不等式可以证明下列不等式.", + "remark": "", + "figures": [] +} \ No newline at end of file diff --git a/processed_dataset/proof/1135.json b/processed_dataset/proof/1135.json new file mode 100644 index 0000000000000000000000000000000000000000..ef68da7395a3290e8e3b184d091e3502cb22aea1 --- /dev/null +++ b/processed_dataset/proof/1135.json @@ -0,0 +1,8 @@ +{ + "source_file": "./raw_volume-zh/volume4/chapter4-1.tex", + "problem_type": "proof", + "problem": "例24. 证明: 对正实数 $a 、 b 、 c$, 有\n$$\n\\frac{a}{\\sqrt{a^2+8 b c}}+\\frac{b}{\\sqrt{b^2+8 a c}}+\\frac{c}{\\sqrt{c^2+8 a b}} \\geqslant 1 .\n$$", + "solution": "证明:由变形的柯西不等式,\n$$\n\\text { 左边 }=\\sum \\frac{a}{\\sqrt{a^2+8 b c}}=\\sum \\frac{a^{\\frac{3}{2}}}{\\sqrt{a^3+8 a b c}} \\geqslant \\frac{\\left(\\sum a\\right)^{\\frac{3}{2}}}{\\left[\\sum\\left(a^3+8 a b c\\right)\\right]^{\\frac{1}{2}}} \\text {. }\n$$\n所以要证明原不等式, 只需要证明\n$$\n\\frac{\\left(\\sum a\\right)^{\\frac{3}{2}}}{\\left[\\sum\\left(a^3+8 a b c\\right)\\right]^{\\frac{1}{2}}} \\geqslant 1,\n$$\n等价于\n$$\n\\left(\\sum a\\right)^3 \\geqslant \\sum a^3+24 a b c\n$$\n等价于\n$$\n\\sum a^3+3 \\sum\\left(a^2 b+a b^2\\right)+6 a b c \\geqslant \\sum a^3+24 a b c,\n$$\n等价于\n$$\n\\sum\\left(a^2 b+a b^2\\right) \\geqslant 6 a b c .\n$$\n易知该不等式成立,故原不等式成立.", + "remark": "注:前面, 我们用平均值不等式证明了这个不等式, 读者还可以用其他方法证明.", + "figures": [] +} \ No newline at end of file diff --git a/processed_dataset/proof/1136.json b/processed_dataset/proof/1136.json new file mode 100644 index 0000000000000000000000000000000000000000..3a18ba9720167ad6e733e1521da0d9d1ab0de61b --- /dev/null +++ b/processed_dataset/proof/1136.json @@ -0,0 +1,8 @@ +{ + "source_file": "./raw_volume-zh/volume4/chapter4-2.tex", + "problem_type": "proof", + "problem": "例4. 已知 $x 、 y 、 z$ 为实数, 且满足\n$$\nx+y+z=x y+y z+z x .\n$$\n求 $\\frac{x}{x^2+1}+\\frac{y}{y^2+1}+\\frac{z}{z^2+1}$ 的最小值.", + "solution": "解:令 $x=1, y=z=-1$. 则\n$$\n\\frac{x}{x^2+1}+\\frac{y}{y^2+1}+\\frac{z}{z^2+1}=-\\frac{1}{2} .\n$$\n猜想最小值为 $-\\frac{1}{2}$.\n只须证:\n$$\n\\begin{aligned}\n& \\frac{x}{x^2+1}+\\frac{y}{y^2+1}+\\frac{z}{z^2+1} \\geqslant-\\frac{1}{2} \\\\\n\\Leftrightarrow & \\frac{(x+1)^2}{x^2+1}+\\frac{(y+1)^2}{y^2+1} \\geqslant \\frac{(z-1)^2}{z^2+1} .\n\\end{aligned} \\label{(39)}\n$$\n注意到 $z(x+y-1)=x+y-x y$.\n若 $x+y-1=0$, 则 $x+y=x y=1$. 矛盾.\n故 $x+y-1 \\neq 0$.\n于是, $z=\\frac{x+y-x y}{x+y-1}$.\n代入不等式 (39)得\n$$\n\\begin{aligned}\n& \\frac{(x+-1)^2}{x^2+1}+\\frac{(y+1)^2}{y^2+1} \\\\\n\\geqslant & \\frac{(x y-1)^2}{(x+y-1)^2+(x+y-x y)^2} .\n\\end{aligned} \\label{(40)}\n$$\n由柯西不等式得式(40) 左边\n$$\n\\begin{aligned}\n& \\geqslant \\frac{[(1+x)(1-y)+(1+y)(1-x)]^2}{\\left(1+x^2\\right)(1-y)^2+\\left(1+y^2\\right)(1-x)^2} \\\\\n& =\\frac{4(x y-1)^2}{\\left(1+x^2\\right)(1-y)^2+\\left(1+y^2\\right)(1-x)^2} .\n\\end{aligned}\n$$\n于是, 只须证\n$$\n\\begin{aligned}\n& 4(x+y-1)^2+4(x+y-x y)^2 \\\\\n\\geqslant & \\left(1+x^2\\right)(1-y)^2+\\left(1+y^2\\right)(1-x)^2 \\\\\n\\Leftrightarrow & f(x)=\\left(y^2-3 y+3\\right) x^2-\\left(3 y^2-8 y+3\\right) x+3 y^2-3 y+1 \\geqslant 0 .\n\\end{aligned}\n$$\n由 $\\Delta=\\left(3 y^2-8 y+3\\right)^2-4\\left(y^2-3 y+3\\right)\\left(3 y^2-3 y+1\\right)= -3\\left(y^2-1\\right)^2 \\leqslant 0$, 故 $f(x) \\geqslant 0$ 恒成立.\n从而, 猜想成立, 即 $\\frac{x}{x^2+1}+\\frac{y}{y^2+1}+\\frac{z}{z^2+1}$ 的最小值为 $-\\frac{1}{2}$.", + "remark": "", + "figures": [] +} \ No newline at end of file diff --git a/processed_dataset/proof/1137.json b/processed_dataset/proof/1137.json new file mode 100644 index 0000000000000000000000000000000000000000..08ec90dff9dac78197eed8685535433c85725e77 --- /dev/null +++ b/processed_dataset/proof/1137.json @@ -0,0 +1,8 @@ +{ + "source_file": "./raw_volume-zh/volume4/chapter4-2.tex", + "problem_type": "proof", + "problem": "例8. 对满足 $a+b=1$ 的正实数 $a 、 b$, 求\n$$\n\\left(a+\\frac{1}{a}\\right)^2+\\left(b+\\frac{1}{b}\\right)^2\n$$\n的最小值.", + "solution": "解:当 $a=b=\\frac{1}{2}$ 时, 我们有\n$$\n\\left(a+\\frac{1}{a}\\right)^2+\\left(b+\\frac{1}{b}\\right)^2=\\frac{25}{2} .\n$$\n下面证明\n$$\n\\left(a+\\frac{1}{a}\\right)^2+\\left(b+\\frac{1}{b}\\right)^2 \\geqslant \\frac{25}{2} .\n$$\n从而最小值为 $\\frac{25}{2}$.\n令 $x=a+\\frac{1}{a}, y=b+\\frac{1}{b}$, 由\n$$\n\\frac{x^2+y^2}{2} \\geqslant\\left(\\frac{x+y}{2}\\right)^2 \\text {. }\n$$\n于是\n$$\n\\begin{aligned}\n\\frac{1}{2}\\left[\\left(a+\\frac{1}{a}\\right)^2+\\left(b+\\frac{1}{b}\\right)^2\\right] & \\geqslant\\left\\{\\frac{1}{2}\\left[\\left(a+\\frac{1}{a}\\right)+\\left(b+\\frac{1}{b}\\right)\\right]\\right\\}^2 \\\\\n& =\\left[\\frac{1}{2}\\left(1+\\frac{1}{a}+\\frac{1}{b}\\right)\\right]^2\n\\end{aligned}\n$$\n由柯西不等式, 得 $\\left(\\frac{1}{a}+\\frac{1}{b}\\right)(a+b) \\geqslant(1+1)^2=4$. 则\n$$\n\\left[\\frac{1}{2}\\left(1+\\frac{1}{a}+\\frac{1}{b}\\right)\\right]^2 \\geqslant\\left[\\frac{1}{2}\\left(1+\\frac{4}{a+b}\\right)\\right]^2=\\left(\\frac{1+4}{2}\\right)^2=\\frac{25}{4} \\text {. }\n$$\n从而命题成立.", + "remark": "", + "figures": [] +} \ No newline at end of file diff --git a/processed_dataset/proof/1138.json b/processed_dataset/proof/1138.json new file mode 100644 index 0000000000000000000000000000000000000000..fce2f7130369111838c1c74d6192b7e291f55f3b --- /dev/null +++ b/processed_dataset/proof/1138.json @@ -0,0 +1,8 @@ +{ + "source_file": "./raw_volume-zh/volume4/chapter4-2.tex", + "problem_type": "proof", + "problem": "例13. 设 $S=\\left\\{a_1, a_2, \\cdots, a_n\\right\\}, a_i \\in \\mathbf{Z}^{+}$, 且对任意 $S_1, S_2 \\subseteq S, S_1 \\neq S_2$, 有 $\\sum_{i \\in S_1} i \\neq \\sum_{j \\in S_2} j$. 求\n$$\n\\sqrt{a_1}+\\sqrt{a_2}+\\cdots+\\sqrt{a_n}\n$$\n的最小值.", + "solution": "解法一不妨设 $a_1%%\n解法二记 $b_1=1, b_2=2, \\cdots, b_n=2^{n-1}$, 则 $b_1\\frac{n-1}{n+2} .\n$$", + "solution": "证明:由柯西不等式, 得\n$$\n\\begin{gathered}\n{\\left[\\left(P_1+P_2\\right)+\\left(P_2+P_3\\right)+\\cdots+\\left(P_{n-1}+P_n\\right)\\right]} \\\\\n\\left(\\frac{1}{P_1+P_2}+\\frac{1}{P_2+P_3}+\\cdots+\\frac{1}{P_{n-2}+P_{n-1}}+\\frac{1}{P_{n-1}+P_n}\\right) \\geqslant(n-1)^2,\n\\end{gathered}\n$$\n所以\n$$\n\\begin{aligned}\n& \\frac{1}{P_1+P_2}+\\frac{1}{P_2+P_3}+\\cdots+\\frac{1}{P_{n-2}+P_{n-1}}+\\frac{1}{P_{n-1}+P_n} \\\\\n\\geqslant & \\frac{(n-1)^2}{2\\left(P_1+P_2+\\cdots+P_n\\right)-P_1-P_n} \\\\\n= & \\frac{(n-1)^2}{n(n+1)-P_1-P_n} \\\\\n\\geqslant & \\frac{(n-1)^2}{n(n+1)-1-2} \\\\\n= & \\frac{(n-1)^2}{(n-1)(n+2)-1} \\\\\n> & \\frac{(n-1)^2}{(n-1)(n+2)}=\\frac{n-1}{n+2} .\n\\end{aligned}\n$$", + "remark": "", + "figures": [] +} \ No newline at end of file diff --git a/processed_dataset/proof/1143.json b/processed_dataset/proof/1143.json new file mode 100644 index 0000000000000000000000000000000000000000..7516a9cb44cabf0dc2507073007fe8904be06c57 --- /dev/null +++ b/processed_dataset/proof/1143.json @@ -0,0 +1,8 @@ +{ + "source_file": "./raw_volume-zh/volume4/chapter4-3.tex", + "problem_type": "proof", + "problem": "例5. 设正数 $x_i$ 满足 $\\sum_{i=1}^n x_i==1$, 求证:\n$$\n\\sum_{i=1}^n \\frac{x_i}{\\sqrt{1-x_i}} \\geqslant \\frac{1}{\\sqrt{n-1}} \\sum_{i=1}^n \\sqrt{x_i} .\n$$", + "solution": "证明:由柯西不等式, 得\n$$\n\\sum_{i=1}^n \\frac{1}{\\sqrt{1-x_i}} \\cdot \\sum_{i=1}^n \\sqrt{1-x_i} \\geqslant n^2,\n$$\n以及\n$$\n\\sum_{i=1}^n \\sqrt{1-x_i} \\leqslant \\sqrt{\\sum_{i=1}^n 1 \\cdot \\sum_{i=1}^n\\left(1-x_i\\right)}=\\sqrt{n(n-1)},\n$$\n所以\n$$\n\\begin{aligned}\n\\sum_{i=1}^n \\frac{x_i}{\\sqrt{1-x_i}} & =\\sum_{i=1}^n \\frac{1}{\\sqrt{1-x_i}}-\\sum_{i=1}^n \\sqrt{1-x_i} \\\\\n& \\geqslant \\frac{n^2}{\\sum_{i=1}^n \\sqrt{1-x_i}}-\\sum_{i=1}^n \\sqrt{1-x_i} \\\\\n& \\geqslant \\frac{n^2}{\\sqrt{n(n-1)}}-\\sqrt{n(n-1)} \\\\\n& =\\sqrt{\\frac{n}{n-1}} .\n\\end{aligned}\n$$\n又由柯西不等式, 得\n$$\n\\sum_{i=1}^n \\sqrt{x_i} \\leqslant \\sqrt{\\sum_{i=1}^n 1 \\cdot \\sum_{i=1}^n x_i}=\\sqrt{n},\n$$\n故命题成立", + "remark": "", + "figures": [] +} \ No newline at end of file diff --git a/processed_dataset/proof/1144.json b/processed_dataset/proof/1144.json new file mode 100644 index 0000000000000000000000000000000000000000..6d2fb559eaeacaaa0147d016438ef4896c46a6a7 --- /dev/null +++ b/processed_dataset/proof/1144.json @@ -0,0 +1,8 @@ +{ + "source_file": "./raw_volume-zh/volume4/chapter4-3.tex", + "problem_type": "proof", + "problem": "例 6 设 $a 、 b 、 c$ 是大于 -1 的实数,证明:\n$$\n\\frac{1+a^2}{1+b+c^2}+\\frac{1+b^2}{1+c+a^2}+\\frac{1+c^2}{1+a+b^2} \\geqslant 2 \\text {. }\n$$", + "solution": "证明:由假设我们有 $1+a^2, 1+b^2, 1+c^2, 1+b+c^2, 1+c+a^2, 1+-a+ b^2$ 均大于零.\n由柯西不等式,得\n$$\n\\begin{aligned}\n& \\left(\\frac{1+a^2}{1+b+c^2}+\\frac{1+b^2}{1+c+a^2}+\\frac{1+c^2}{1+a+b^2}\\right) \\cdot\\left[\\left(1+a^2\\right)\\left(1+b+c^2\\right)+\\right. \\\\\n& \\left.\\left(1+b^2\\right)\\left(1+c+a^2\\right)+\\left(1+c^2\\right)\\left(1+a+b^2\\right)\\right] \\\\\n\\geqslant & \\left(1+a^2+1+b^2+1+c^2\\right)^2 .\n\\end{aligned}\n$$\n于是\n$$\n\\begin{aligned}\n& \\frac{1+a^2}{1+b+c^2}+\\frac{1+b^2}{1+c+a^2}+\\frac{1+c^2}{1+a+b^2} \\\\\n\\geqslant & \\frac{\\left(a^2+b^2+c^2+3\\right)^2}{\\left(1+a^2\\right)\\left(1+b+c^2\\right)+\\left(1+b^2\\right)\\left(1+c+a^2\\right)+\\left(1+c^2\\right)\\left(1+a+b^2\\right)} \\\\\n= & \\frac{a^4+b^4+c^4+9+2 a^2 b^2+2 b^2 c^2+2 c^2 a^2+6 a^2+6 b^2+6 c^2}{a^2 b^2+b^2 c^2+c^2 a^2+2\\left(a^2+b^2+c^2\\right)+a^2 b+b^2 c+c^2 a+a+b+c+3} \\\\\n= & 2+\\frac{a^4+b^4+c^4+3+2 a^2+2 b^2+2 c^2-2\\left(a^2 b+b^2 c+c^2 a+a+b+c\\right)}{a^2 b^2+b^2 c^2+c^2 a^2+2\\left(a^2+b^2+c^2\\right)+a^2 b+b^2 c+c^2 a+a+b+c+3}\n\\end{aligned}\n$$\n$$\n\\begin{aligned}\n& =2+\\frac{\\left(a^2-b\\right)^2+\\left(b^2-c\\right)^2+\\left(c^2-a\\right)^2+(a-1)^2+(b-1)^2+(c-1)^2}{a^2 b^2+b^2 c^2+c^2 a^2+2\\left(a^2+b^2+c^2\\right)+a^2 b+b^2 c+c^2 a+a+b+c+3} \\\\\n& \\geqslant 2\n\\end{aligned}\n$$\n当且仅当 $a=b=c=1$ 时等号成立.", + "remark": "", + "figures": [] +} \ No newline at end of file diff --git a/processed_dataset/proof/1145.json b/processed_dataset/proof/1145.json new file mode 100644 index 0000000000000000000000000000000000000000..5d4a3698abe68947372ea4f8d1daf1392045359b --- /dev/null +++ b/processed_dataset/proof/1145.json @@ -0,0 +1,8 @@ +{ + "source_file": "./raw_volume-zh/volume4/chapter4-3.tex", + "problem_type": "proof", + "problem": "例7. 正数 $a, b, c$ 满足 $a b c=1, n$ 为正整数,求证:\n(a) $\\frac{1}{1+2 a}+\\frac{1}{1+2 b}+\\frac{1}{1+2 c} \\geqslant 1$;\n(b) $\\frac{c^n}{a+b}+\\frac{b^n}{c+a}+\\frac{a^n}{b+c} \\geqslant \\frac{3}{2}$.", + "solution": "证明:(a) 首先来证明\n$$\n\\begin{aligned}\n& \\frac{1}{1+2 a} \\geqslant \\frac{a^{-\\frac{2}{3}}}{a^{-\\frac{2}{3}}+b^{-\\frac{2}{3}}+c^{-\\frac{2}{3}}} \\\\\n\\Leftrightarrow & a^{-\\frac{2}{3}}+b^{-\\frac{2}{3}}+c^{-\\frac{2}{3}} \\geqslant a^{-\\frac{2}{3}}+2 a^{\\frac{1}{3}} \\\\\n\\Leftrightarrow & b^{-\\frac{2}{3}}+c^{-\\frac{2}{3}} \\geqslant 2 b c^{-\\frac{1}{3}}\n\\end{aligned}\n$$\n这是显然的.\n同理有 $\\frac{1}{1+2 b} \\geqslant \\frac{b^{-\\frac{2}{3}}}{a^{-\\frac{2}{3}}+b^{-\\frac{2}{3}}+c^{-\\frac{2}{3}}}, \\frac{1}{1+2 c} \\geqslant \\frac{c^{-\\frac{2}{3}}}{a^{-\\frac{2}{3}}+b^{-\\frac{2}{3}}+c^{-\\frac{2}{3}}}$.\n所以 $\\frac{1}{1+2 a}+\\frac{1}{1+2 b}+\\frac{1}{1+2 c} \\geqslant 1$.\n(b) 不妨设 $a \\geqslant b \\geqslant c$, 那么 $a^{n-1} \\geqslant b^{n-1} \\geqslant c^{n-1}, \\frac{a}{b+c} \\geqslant \\frac{b}{c+a} \\geqslant \\frac{c}{a+b}$. 由排序不等式得到\n$$\n\\begin{aligned}\n& \\frac{c^n}{a+b}+\\frac{b^n}{c+a}+\\frac{a^n}{b+c} \\geqslant \\frac{c a^{n-1}}{a+b}+\\frac{b c^{n-1}}{c+a}+\\frac{a b^{n-1}}{b+c}, \\\\\n& \\frac{c^n}{a+b}+\\frac{b^n}{c+a}+\\frac{a^n}{b+c} \\geqslant \\frac{c b^{n-1}}{a+b}+\\frac{b a^{n-1}}{c+a}+\\frac{a c^{n-1}}{b+c},\n\\end{aligned}\n$$\n所以\n$$\n\\begin{aligned}\n& \\frac{c^n}{a+b}+\\frac{b^n}{c+a}+\\frac{a^n}{b+c} \\\\\n\\geqslant & \\frac{1}{3}\\left(\\frac{c}{a+b}+\\frac{b}{c+a}+\\frac{a}{b+c}\\right)\\left(a^{n-1}+b^{n-1}+c^{n-1}\\right),\n\\end{aligned}\n$$\n而显然有 $a^{n-1}+b^{n-1}+c^{n-1} \\geqslant 3$, 下面来证明 $\\frac{c}{a+b}+\\frac{b}{c+a}+\\frac{a}{b+c} \\geqslant \\frac{3}{2}$.\n即证 $\\frac{a+b+c}{a+b}+\\frac{a+b+c}{c+a}+\\frac{a+b+c}{b+c} \\geqslant \\frac{9}{2}$\n$$\n\\Leftrightarrow(a+b+c+a+b+c)\\left(\\frac{1}{a+b}+\\frac{1}{c+a}+\\frac{1}{b+c}\\right) \\geqslant 9 .\n$$\n这由柯西不等式可知是显然的.\n所以\n$$\n\\begin{aligned}\n& \\frac{c^n}{a+b}+\\frac{b^n}{c+a}+\\frac{a^n}{b+c} \\\\\n\\geqslant & \\frac{1}{3}\\left(\\frac{c}{a+b}+\\frac{b}{c+a}+\\frac{a}{b+c}\\right)\\left(a^{n-1}+b^{n-1}+c^{n-1}\\right) \\geqslant \\frac{3}{2} .\n\\end{aligned}\n$$\n证毕.", + "remark": "", + "figures": [] +} \ No newline at end of file diff --git a/processed_dataset/proof/1146.json b/processed_dataset/proof/1146.json new file mode 100644 index 0000000000000000000000000000000000000000..a524e047c0786eec1533a57e1c59ee2ead873f58 --- /dev/null +++ b/processed_dataset/proof/1146.json @@ -0,0 +1,8 @@ +{ + "source_file": "./raw_volume-zh/volume4/chapter4-3.tex", + "problem_type": "proof", + "problem": "例8. 证明: 对任意满足 $x+y+z=0$ 的实数 $x, y, z$ 都有\n$$\n\\frac{x(x+2)}{2 x^2+1}+\\frac{y(y+2)}{2 y^2+1}+\\frac{z(z+2)}{2 z^2+1} \\geqslant 0 .\n$$", + "solution": "证明:注意到 $\\frac{x(x+2)}{2 x^2+1}=\\frac{(2 x+1)^2}{2\\left(2 x^2+1\\right)}-\\frac{1}{2}$ 等式子, 所以原不等式等价于\n$$\n\\frac{(2 x+1)^2}{2 x^2+1}+\\frac{(2 y+1)^2}{2 y^2+1}+\\frac{(2 z+1)^2}{2 z^2+1} \\geqslant 3,\n$$\n由柯西不等式,我们有\n$$\n2 x^2=\\frac{4}{3} x^2+\\frac{2}{3}(y+z)^2 \\leqslant \\frac{4}{3} x^2+\\frac{4}{3}\\left(y^2+z^2\\right),\n$$\n所以\n$$\n\\sum \\frac{(2 x+1)^2}{2 x^2+1} \\geqslant 3 \\sum \\frac{(2 x+1)^2}{4\\left(x^2+y^2+z^2\\right)+3}=3 .\n$$", + "remark": "", + "figures": [] +} \ No newline at end of file diff --git a/processed_dataset/proof/1147.json b/processed_dataset/proof/1147.json new file mode 100644 index 0000000000000000000000000000000000000000..6683a258b80ded06b3263cdd7575a95e0aa35987 --- /dev/null +++ b/processed_dataset/proof/1147.json @@ -0,0 +1,8 @@ +{ + "source_file": "./raw_volume-zh/volume4/chapter4-3.tex", + "problem_type": "proof", + "problem": "例9. 已知正数 $a_1, a_2, \\cdots, a_n(n>2)$ 满足 $a_1+a_2+\\cdots+a_n=1$. 证明:\n$$\n\\frac{a_2 a_3 \\cdots a_n}{a_1+n-2}+\\frac{a_1 a_3 \\cdots a_n}{a_2+n-2}+\\cdots+\\frac{a_1 a_2 \\cdots a_{n-1}}{a_n+n-2} \\leqslant \\frac{1}{(n-1)^2} .\n$$", + "solution": "证明:由柯西不等式, 知对于正数 $x_1, x_2, \\cdots, x_n$, 有\n$$\n\\frac{1}{\\sum_{i=1}^n x_i} \\leqslant \\frac{1}{n^2} \\sum_{i=1}^n \\frac{1}{x_i} .\n$$\n又 $a_1+a_2+\\cdots+a_n=1(n>2)$, 则\n$$\n\\sum_{i=1}^n \\frac{1}{a_i\\left(a_i+n-2\\right)}\n$$\n$$\n\\begin{aligned}\n& =\\sum_{i=1}^n \\frac{1}{a_i \\sum_{\\substack{j=1 \\\\\nj \\neq i}}^n\\left(1-a_j\\right)} \\\\\n& \\leqslant \\sum_{i=1}^n \\frac{1}{(n-1)^2} \\sum_{\\substack{j=1 \\\\\nj \\neq i}}^n \\frac{1}{a_i\\left(1-a_j\\right)} \\\\\n& =\\frac{1}{(n-1)^2} \\sum_{j=1}^n \\sum_{\\substack{i=1 \\\\\ni \\neq j}}^n \\frac{1}{a_i\\left(1-a_j\\right)} .\n\\end{aligned}\n$$\n由已知得\n$$\na_i \\in(0,1)(i=1,2, \\cdots, n) .\n$$\n于是,对任意的 $j \\in\\{1,2, \\cdots, n\\}$, 有\n$$\n\\begin{array}{r}\na_i \\geqslant \\frac{\\prod_{k=1}^n a_k}{a_{i-1} a_j}, \\\\\na_{j+1} \\geqslant \\frac{\\prod_{k=1}^n a_k}{a_{j-1} a_j},\n\\end{array}\n$$\n其中, $i=1,2, \\cdots, n, i \\neq j, i \\neq j+1, a_0=a_n$.\n故 $\\quad \\sum_{\\substack{i=1 \\\\ i \\neq j}}^n a_i \\geqslant \\sum_{\\substack{i=1 \\\\ i \\neq j}}^n \\frac{\\prod_{k=1}^n a_k}{a_i a_j} \\Rightarrow\\left(1-a_j\\right) \\frac{a_j}{\\prod_{k=1}^n a_k} \\geqslant \\sum_{\\substack{i=1 \\\\ i \\neq j}}^n \\frac{1}{a_i}$, 即\n$$\n\\sum_{\\substack{i=1 \\\\ i \\neq j}}^n \\frac{1}{a_i\\left(1-a_j\\right)} \\leqslant \\frac{1}{\\prod_{\\substack{k=1 \\\\ k \\neq j}}^n a_k}\n$$\n则\n$$\n\\sum_{i=1}^n \\frac{1}{a_i\\left(a_i+n-2\\right)} \\leqslant \\frac{1}{(n-1)^2} \\sum_{j=1}^n \\frac{1}{\\prod_{\\substack{k=1 \\\\ k \\neq j}}^n a_k} .\n$$\n故\n$$\n\\sum_{i=1}^n \\frac{\\prod_{\\substack{k=1 \\\\ k \\neq i}}^n a_k}{a_i+n-2} \\leqslant \\frac{1}{(n-1)^2} \\sum_{j=1}^n a_j=\\frac{1}{(n-1)^2} .\n$$", + "remark": "", + "figures": [] +} \ No newline at end of file diff --git a/processed_dataset/proof/1148.json b/processed_dataset/proof/1148.json new file mode 100644 index 0000000000000000000000000000000000000000..acb36b410c41c21ebe42f6623a3e8f84c7fadbe5 --- /dev/null +++ b/processed_dataset/proof/1148.json @@ -0,0 +1,8 @@ +{ + "source_file": "./raw_volume-zh/volume4/chapter4-3.tex", + "problem_type": "proof", + "problem": "例10. 设 $n \\geqslant 2, a_1, a_2, \\cdots, a_n$ 是 $n$ 个正实数,满足:\n$$\n\\left(a_1+\\cdots+a_n\\right)\\left(\\frac{1}{a_1}+\\frac{1}{a_2}+\\cdots+-\\frac{1}{a_n}\\right) \\leqslant\\left(n+\\frac{1}{2}\\right)^2 .\n$$\n证明: $\\max \\left\\{a_1, \\cdots, a_n\\right\\} \\leqslant 4 \\min \\left\\{a_1, \\cdots, a_n\\right\\}$.", + "solution": "证明:不妨设\n$$\nm=a_1 \\leqslant a_2 \\leqslant \\cdots \\leqslant a_n=M,\n$$\n要证 $M \\leqslant 4 m$.\n当 $n=2$ 时,条件为\n$$\n(m+M)\\left(\\frac{1}{m}+\\frac{1}{M}\\right) \\leqslant \\frac{25}{4} .\n$$\n等价于\n$$\n4(m+M)^2 \\leqslant 25 m M,\n$$\n即\n$$\n(4 M-m)(M-4 m) \\leqslant 0,\n$$\n而\n$$\n4 M-m \\geqslant 3 M>0,\n$$\n故 $M \\leqslant 4 m$.\n当 $n \\geqslant 3$ 时,利用柯西不等式可知\n$$\n\\begin{gathered}\n\\left(n+\\frac{1}{2}\\right)^2 \\geqslant\\left(a_1+\\cdots+a_n\\right)\\left(\\frac{1}{a_1}+\\cdots+\\frac{1}{a_n}\\right) \\\\\n=\\left(m+a_2+\\cdots+a_{n-1}+M\\right)\\left(\\frac{1}{M}+\\frac{1}{a_2}+\\cdots+\\frac{1}{a_{n-1}}+\\frac{1}{m}\\right) \\\\\n\\geqslant(\\sqrt{\\frac{m}{M}}+\\underbrace{1+\\cdots+1}_{n-2 \\uparrow}+\\sqrt{\\frac{M}{m}})^2 .\n\\end{gathered}\n$$\n故\n$$\nn+\\frac{1}{2} \\geqslant \\sqrt{\\frac{m}{M}}+\\sqrt{\\frac{M}{m}}+n-2,\n$$\n于是\n$$\n\\sqrt{\\frac{M}{m}}+\\sqrt{\\frac{m}{M}} \\leqslant \\frac{5}{2} .\n$$\n从而\n$$\n2(m+M) \\leqslant 5 \\sqrt{m M},\n$$\n同 $n=2$ 的情形可得 $M \\leqslant 4 m$. 命题获证.", + "remark": "", + "figures": [] +} \ No newline at end of file diff --git a/processed_dataset/proof/1149.json b/processed_dataset/proof/1149.json new file mode 100644 index 0000000000000000000000000000000000000000..035e3ff72650111887fb8c57d9b1927ccd16e60f --- /dev/null +++ b/processed_dataset/proof/1149.json @@ -0,0 +1,8 @@ +{ + "source_file": "./raw_volume-zh/volume4/chapter4-3.tex", + "problem_type": "proof", + "problem": "例11. 设\n$$\nf(x, y, z)=\\frac{x(2 y-z)}{1+x+3 y}+\\frac{y(2 z-x)}{1+y+3 z}+\\frac{z(2 x-y)}{1+z+3 x},\n$$\n其中 $x, y, z \\geqslant 0$, 且 $x+y+z=1$. 求 $f(\\dot{x}, y, z)$ 的最大值和最小值.", + "solution": "解:先证 $f \\leqslant \\frac{1}{7}$, 当且仅当 $x=y=z=\\frac{1}{3}$ 时等号成立.\n因为\n$$\nf=\\sum \\frac{x(x+3 y-1)}{1+x+3 y}=1-2 \\sum \\frac{x}{1+x+3 y}, \\label{(41)}\n$$\n由柯西不等式\n$$\n\\sum \\frac{x}{1+x+3 y} \\geqslant \\frac{\\left(\\sum x\\right)^2}{\\sum x(1+x+3 y)}=\\frac{1}{\\sum x(1+x+3 y)},\n$$\n因为\n$$\n\\sum x(1+x+3 y)=\\sum x(2 x+4 y+z)=2+\\sum x y \\leqslant \\frac{7}{3} .\n$$\n从而\n$$\n\\begin{aligned}\n& \\sum \\frac{x}{1+x+3 y} \\geqslant \\frac{3}{7}, \\\\\n& f \\leqslant 1-2 \\times \\frac{3}{7}=\\frac{1}{7},\n\\end{aligned}\n$$\n$f_{\\text {max }}=\\frac{1}{7}$, 当且仅当 $x=y=z=\\frac{1}{3}$ 时等号成立.\n再证 $f \\geqslant 0$, 当 $x=1, y=z=0$ 时等号成立.\n事实上,\n$$\n\\begin{aligned}\nf(x, y, z)= & \\frac{x(2 y-z)}{1+x+3 y}+\\frac{y(2 z-x)}{1+y+3 z}+\\frac{z(2 x-y)}{1+z+3 x} \\\\\n= & x y\\left(\\frac{2}{1+x+3 y}-\\frac{1}{1+y+3 z}\\right) \\\\\n& +x z\\left(\\frac{2}{1+z+3 x}-\\frac{1}{1+x+3 y}\\right)\n\\end{aligned}\n$$\n$$\n\\begin{aligned}\n& +y z\\left(\\frac{2}{1+y+3 z}-\\frac{1}{1+z+3 x}\\right) \\\\\n= & \\frac{7 x y z}{(1+x+3 y)(1+y+3 z)} \\\\\n& +\\frac{7 x y z}{(1+z+3 x)(1+x+3 y)} \\\\\n& +\\frac{7 x y z}{(1+y+3 z)(1+z+3 x)} \\\\\n\\geqslant & 0 .\n\\end{aligned}\n$$\n故 $f_{\\min }=0$, 当 $x=1, y=z=0$ 时等号成立.\n另证: 设 $z=\\min \\{x, y, z\\}$, 若 $z=0$, 则\n$$\nf(x, y, 0)=\\frac{2 x y}{1+x+3 y}-\\frac{x y}{1+y}=\\frac{2 x y}{2 x+4 y}-\\frac{x y}{x+2 y}=0 .\n$$\n下设 $x, y \\geqslant z>0$, 由 (41) 式, 要证 $f \\geqslant 0$, 只要证\n$$\n\\sum \\frac{x}{1+x+3 y} \\leqslant \\frac{1}{2} . \\label{(42)}\n$$\n注意到\n$$\n\\frac{1}{2}=\\frac{x}{2 x+4 y}+\\frac{y}{x+2 y}\n$$\n于是(42)等价于\n$$\n\\begin{aligned}\n\\frac{z}{1+z+3 x} \\leqslant & \\left(\\frac{x}{2 x+4 y}-\\frac{x}{1+x+3 y}\\right)+\\left(\\frac{y}{x+2 y}-\\frac{y}{1+y+3 z}\\right) \\\\\n& =\\frac{z}{2 x+4 y}\\left(\\frac{x}{1+x+3 y}+\\frac{8 y}{1+y+3 z}\\right) \\\\\n& \\frac{2 x+4 y}{1+z+3 x} \\leqslant \\frac{x}{1+x+3 y}+\\frac{8 y}{1+y+3 z} .\n\\end{aligned}\n$$\n即\n$$\n\\frac{2 x+4 y}{1+z+3 x} \\leqslant \\frac{x}{1+x+3 y}+\\frac{8 y}{1+y+3 z} . \\label{(43)}\n$$\n而由柯西不等式, 可得\n$$\n\\begin{aligned}\n\\frac{x}{1+x+3 y}+\\frac{8 y}{1+y+3 z} & =\\frac{x^2}{x(1+x+3 y)}+\\frac{(2 y)^2}{y(1+y+3 z) / 2} \\\\\n& \\geqslant \\frac{(x+2 y)^2}{\\left(x+x^2+3 x y\\right)+\\left(y+y^2+3 y z\\right) / 2} \\\\\n& =\\frac{2 x+4 y}{1+z+3 x},\n\\end{aligned}\n$$\n即 (43) 成立, 从而 $f \\geqslant 0$, 故 $f_{\\text {min }}=0$, 当 $x=1, y=z=0$ 时等号成立.", + "remark": "", + "figures": [] +} \ No newline at end of file diff --git a/processed_dataset/proof/1150.json b/processed_dataset/proof/1150.json new file mode 100644 index 0000000000000000000000000000000000000000..deb282539c9913d145385e6f82c7d1882d7f85d3 --- /dev/null +++ b/processed_dataset/proof/1150.json @@ -0,0 +1,8 @@ +{ + "source_file": "./raw_volume-zh/volume4/chapter4-3.tex", + "problem_type": "proof", + "problem": "例12. 设 $x, y, z \\in \\mathbf{R}^{+}$, 且 $x+y+z=1$, 证明:\n$$\n\\sum_{x, y, z} \\frac{x^4}{y\\left(1-y^2\\right)} \\geqslant \\frac{1}{8} \\text {. }\n$$", + "solution": "证明:左边 $\\geqslant \\frac{\\left(\\sum x^2\\right)^2}{\\sum y\\left(1-y^2\\right)} \\geqslant \\frac{\\left[\\frac{\\left(\\sum x\\right)^2}{3}\\right]^2}{\\sum x-\\sum x^3}=\\frac{\\frac{1}{9}}{1-\\sum x^3}$.\n又 $\\quad \\sum x^3=\\sum \\frac{x^4}{x} \\geqslant \\frac{\\left(\\sum x^2\\right)^2}{\\sum x} \\geqslant\\left[\\frac{\\left(\\sum x\\right)^2}{3}\\right]^2=\\frac{1}{9}$,\n所以,左边 $\\geqslant \\frac{\\frac{1}{9}}{1-\\frac{1}{9}}=\\frac{1}{8}$, 故原不等式成立.", + "remark": "", + "figures": [] +} \ No newline at end of file diff --git a/processed_dataset/proof/1151.json b/processed_dataset/proof/1151.json new file mode 100644 index 0000000000000000000000000000000000000000..0e69e4ff9657c78a36f41ce9e746bd8f7c5cdc4a --- /dev/null +++ b/processed_dataset/proof/1151.json @@ -0,0 +1,8 @@ +{ + "source_file": "./raw_volume-zh/volume4/chapter4-3.tex", + "problem_type": "proof", + "problem": "例13. 设 $x, y, z, w \\in \\mathbf{R}^{+}$, 证明:\n$$\n\\frac{x}{y+2 z+3 w}+\\frac{y}{z+2 w+3 x}+\\frac{z}{w+2 x+3 y}+\\frac{w}{x+2 y+3 z} \\geqslant \\frac{2}{3} .\n$$", + "solution": "证明:左边 $=\\sum \\frac{x}{y+2 z+3 w}=\\sum \\frac{x^2}{x(y+2 z+3 w)}$\n$$\n\\begin{gathered}\n\\geqslant \\frac{\\left(\\sum x\\right)^2}{\\sum x(y+2 z+3 w)} \\\\\n=\\frac{\\left(\\sum x\\right)^2}{4 \\sum x y}, \\\\\n(x-y)^2+(x-z)^2+(x-w)^2+(y-z)^2+(y-w)^2+(z-w)^2 \\\\\n=3\\left(x^2+y^2+z^2+w^2\\right)-2(x y+x z+x w+y z+y w+z w) \\\\\n=3(x+y+z+w)^2-8(x y+x z+x w+y z+y w+z w) \\geqslant 0 .\n\\end{gathered}\n$$\n所以\n$$\n\\frac{\\left(\\sum x\\right)^2}{\\sum x y} \\geqslant \\frac{8}{3}\n$$\n故原不等式成立.", + "remark": "", + "figures": [] +} \ No newline at end of file diff --git a/processed_dataset/proof/1152.json b/processed_dataset/proof/1152.json new file mode 100644 index 0000000000000000000000000000000000000000..cc5e2c1e224ef66853617b0df4ef2f354c472833 --- /dev/null +++ b/processed_dataset/proof/1152.json @@ -0,0 +1,8 @@ +{ + "source_file": "./raw_volume-zh/volume4/chapter4-3.tex", + "problem_type": "proof", + "problem": "例14. 设 $x_1, x_2, \\cdots, x_n$ 为任意实数,证明:\n$$\n\\frac{x_1}{1+x_1^2}+\\frac{x_2}{1+x_1^2+x_2^2}+\\cdots+\\frac{x_n}{1+x_1^2+x_2^2+\\cdots+x_n^2}<\\sqrt{n} .\n$$", + "solution": "证明:由柯西不等式,得\n$$\n\\begin{aligned}\n& \\left(\\frac{x_1}{1+x_1^2}+\\frac{x_2}{1+x_1^2+x_2^2}+\\cdots+\\frac{x_n}{1+x_1^2+x_2^2+\\cdots+x_n^2}\\right)^2 \\\\\n\\leqslant & {\\left[\\left(\\frac{x_1}{1+x_1^2}\\right)^2+\\left(\\frac{x_2}{1+x_1^2+x_2^2}\\right)^2+\\cdots+\\left(\\frac{x_n}{1+x_1^2+x_2^2+\\cdots+x_n^2}\\right)^2\\right] \\cdot n . }\n\\end{aligned}\n$$\n对 $k \\geqslant 2$, 有\n$$\n\\begin{aligned}\n& \\left(\\frac{x_k}{1+x_1^2+\\cdots+x_k^2}\\right)^2 \\\\\n= & \\frac{x_k^2}{\\left(1+x_1^2+\\cdots+x_k^2\\right)^2} \\\\\n\\leqslant & \\frac{x_k^2}{\\left(1+x_1^2+\\cdots+x_{k-1}^2\\right)\\left(1+x_1^2+\\cdots+x_k^2\\right)} \\\\\n= & \\frac{1}{1+x_1^2+\\cdots+x_{k-1}^2}-\\frac{1}{1+x_1^2+\\cdots+x_k^2} .\n\\end{aligned}\n$$\n对于 $k=1$, 有\n$$\n\\left(\\frac{x_1}{1+x_1^2}\\right)^2 \\leqslant \\frac{x_1^2}{1+x_1^2}=1-\\frac{1}{1+x_1^2} .\n$$\n所以\n$$\n\\sum_{i=1}^n\\left(\\frac{x_k}{1+x_1^2+\\cdots+x_k^2}\\right)^2 \\leqslant 1-\\frac{1}{1+x_1^2+\\cdots+x_n^2}<1,\n$$\n从而\n$$\n\\left(\\frac{x_1}{1+x_1^2}+\\frac{x_2^2}{1+x_1^2+x_2^2}+\\cdots+\\frac{x_n^2}{1+x_1^2+\\cdots+x_n^2}\\right)^21$, 将证明 $\\sum_{i=1}^n x_i<\\sum_{i=1}^n \\frac{1}{x_i}$, 即等价于\n$$\n\\sum_{i=1}^n\\left[\\frac{1}{y_i}-(n-1)\\right]<\\sum_{i=1}^n \\frac{y_i}{1-(n-1) y_i} .\n$$\n对固定 $i$, 由柯西不等式, 得\n$$\n\\begin{aligned}\n& \\sum_{i \\neq j} \\frac{1-(n-1) y_i}{1-(n-1) y_j} \\\\\n\\geqslant & \\frac{\\left[1-(n-1) y_i\\right](n-1)^2}{\\sum_{i \\neq j}\\left[1-(n-1) y_i\\right]} \\\\\n> & \\frac{\\left[1-(n-1) y_i\\right](n-1)^2}{(n-1) y_i}=\\frac{(n-1)\\left[1-(n-1) y_i\\right]}{y_i} .\n\\end{aligned}\n$$\n对 $i$ 求和, 得\n$$\n\\sum_{i=1}^n \\sum_{i \\neq j} \\frac{1-(n-1) y_i}{1-(n-1) y_j} \\geqslant(n-1) \\sum_{i=1}\\left[\\frac{1}{y_i}-(n-1)\\right] .\n$$\n由于\n$$\n\\begin{gathered}\n\\sum_{i=1}^n \\sum_{i \\neq j} \\frac{1-(n-1) y_i}{1-(n-1) y_j} \\leqslant \\sum_{j=1}^n \\frac{(n-1) y_j}{1-(n-1) y_j}, \\\\\n\\sum_{i=1}^n \\frac{y_i}{1-(n-1) y_i}>\\sum_{j=1}^n\\left[\\frac{1}{y_j}-(n-1)\\right] .\n\\end{gathered}\n$$", + "remark": "", + "figures": [] +} \ No newline at end of file diff --git a/processed_dataset/proof/1154.json b/processed_dataset/proof/1154.json new file mode 100644 index 0000000000000000000000000000000000000000..181d5d01c08ad809f419fbd06ffe200b5165a85f --- /dev/null +++ b/processed_dataset/proof/1154.json @@ -0,0 +1,8 @@ +{ + "source_file": "./raw_volume-zh/volume4/chapter4-4.tex", + "problem_type": "proof", + "problem": "例1. 将 1650 个学生排成 22 行, 75 列的方阵, 已知任意给定的两列处于同一行的两个人中, 性别相同的学生不超过 11 对, 证明: 男生的人数不超过 928.", + "solution": "解:设第 $i$ 行的男生数为 $x_i$, 则女生数为 $75-x_i$, 依题意, 得\n$$\n\\sum_{i=1}^{22}\\left(\\mathrm{C}_{x_i}^2+\\mathrm{C}_{75-x_i}^2\\right) \\leqslant 11 \\times \\mathrm{C}_{75}^2\n$$\n于是\n$$\n\\sum_{i=1}^{22}\\left(x_i^2-75 x_i\\right) \\leqslant-30525,\n$$\n即 $\\sum_{i=1}^{22}\\left(2 x_i-75\\right)^2 \\leqslant 1650$. 由柯西不等式, 得\n$$\n\\left[\\sum_{i=1}^{22}\\left(2 x_i-750\\right)\\right]^2 \\leqslant 22 \\sum_{i=1}^{22}\\left(2 x_i-75\\right)^2 \\leqslant 36300,\n$$\n因此 $\\sum_{i=1}^{22}\\left(2 x_i-75\\right)<191$, 从而\n$$\n\\sum_{i=1}^{22} x_i<\\frac{191+1650}{2}<921,\n$$\n故男生的人数不超过 928 .", + "remark": "", + "figures": [] +} \ No newline at end of file diff --git a/processed_dataset/proof/1155.json b/processed_dataset/proof/1155.json new file mode 100644 index 0000000000000000000000000000000000000000..5ede6d1c1b845467f972c30376b3feb05159170d --- /dev/null +++ b/processed_dataset/proof/1155.json @@ -0,0 +1,8 @@ +{ + "source_file": "./raw_volume-zh/volume4/chapter4-4.tex", + "problem_type": "proof", + "problem": "例2. 在一群数学家中, 每一个人都有一些朋友 (关系是互相的). 证明: 存在一个数学家他所有的朋友的平均值不小于这群人的朋友的平均数.", + "solution": "证明:记 $M$ 为这群数学家的集合, $n=|M|, F(m)$ 表示数学家 $m$ 的朋友的集合, $f(m)$ 表示数学家 $m$ 的朋友数 $(f(m)=|F(m)|)$. 即命题等价于证明:必有一个 $m_0$ 使\n$$\n\\frac{1}{f\\left(m_0\\right)} \\sum_{m \\in F\\left(m_0\\right)} f(m) \\geqslant \\frac{1}{n} \\sum_{m \\in M} f(m) .\n$$\n我们用反证法来证明这个命题, 如果不存在这样的数学家 $m_0$. 则对任意的 $m_0$, 有\n$$\nn \\cdot \\sum_{m \\in F\\left(m_0\\right)} f(m)2 n\\left(n^2+1\\right) .\n$$\n于是矛盾, 从而证明了必有一条线段, 从它的两端点引出的线段数之和不小于 $2 n+1$. 不妨设 $A_1 A_2$ 是一条这样的线段, 从而又有 $A_k(k \\geqslant 3)$, 使线段 $A_1 A_k, A_2 A_k$ 都存在, 于是 $\\triangle A_1 A_2 A_k$ 即为所求.", + "remark": "", + "figures": [] +} \ No newline at end of file diff --git a/processed_dataset/proof/1157.json b/processed_dataset/proof/1157.json new file mode 100644 index 0000000000000000000000000000000000000000..5f601b1103eae650e13d393611a4aec6c00857cf --- /dev/null +++ b/processed_dataset/proof/1157.json @@ -0,0 +1,8 @@ +{ + "source_file": "./raw_volume-zh/volume4/chapter4-4.tex", + "problem_type": "proof", + "problem": "例5. 设 $A_1, A_2, \\cdots, A_{30}$ 是集 $\\{1,2, \\cdots, 2003\\}$ 的子集, 且 $\\left|A_i\\right| \\geqslant 660 (i=1,2, \\cdots, 30)$. 证明:存在 $i, j \\in\\{1,2, \\cdots, 30\\}, i \\neq j$, 使得\n$$\n\\left|A_i \\cap A_j\\right| \\geqslant 2003 \\text {. }\n$$", + "solution": "证明:不妨设每个 $A_i$ 的元素都为 660 个(否则去除一些元素),我们作一个集合、元素的关系表:表中每一行(除最上面的一行)表示 30 个集合, 表的 $n$ 列(最左面一列除外)表示 2003 个元素 $1,2, \\cdots, 2003$. 如果 $i \\in A_j(i=1$ , $2, \\cdots, 2003,1 \\leqslant j \\leqslant 30)$, 则在 $i$ 所在的列与 $A_j$ 所在的交叉处填上 1 , 如果 $i \\notin A_j$, 则写上 0 . 表中每一行有 660 个 1 , 因此共有 $30 \\times 660$ 个 1 . 第 $j$ 列有 $m_j$ 个 $1(j=1,2, \\cdots, 2003)$, 则\n$$\n\\sum_{j=1}^{2003} m_j=30 \\times 660 .\n$$\n由于每个元素 $j$ 属于 $\\mathrm{C}_{m_j}^2$ 个交集 $A_s \\cap A_t$, 因此\n$$\n\\sum_{j=1}^{2003} \\mathrm{C}_{m_j}^2=\\sum_{1 \\leqslant s2002,\n\\end{aligned}\n$$\n故 $\\left|A_i \\cap A_j\\right| \\geqslant 2003$.", + "remark": "", + "figures": [] +} \ No newline at end of file diff --git a/processed_dataset/proof/1158.json b/processed_dataset/proof/1158.json new file mode 100644 index 0000000000000000000000000000000000000000..2a4f76be7740ddc9af9de491eab72f937c0954b7 --- /dev/null +++ b/processed_dataset/proof/1158.json @@ -0,0 +1,8 @@ +{ + "source_file": "./raw_volume-zh/volume4/chapter4-4.tex", + "problem_type": "proof", + "problem": "例6. 给定平面上的 $n$ 个相异点.\n证明: 其中距离为单位长的点对少于 $2 \\sqrt{n^3}$ 对.", + "solution": "证明:对于平面上的点集 $\\left\\{P_1, P_2, \\cdots, P_n\\right\\}$, 令 $a_i$ 为与 $P_i$ 相距为单位长的点 $P_i$ 的个数.\n不妨设 $a_i \\geqslant 1$, 则相距为单位长的点对的对数是\n$$\nA=\\frac{a_1+a_2+\\cdots+a_n}{2} .\n$$\n设 $C_i$ 是以点 $P_i$ 为圆心, 以 1 为半径的圆.\n因为每对圆至多有 2 个交点,故所有的 $C_i$ 至多有\n$$\n2 \\mathrm{C}_n^2=n(n-1)\n$$\n个交点.\n点 $P_i$ 作为 $C_j$ 的交点出现 $\\mathrm{C}_{a_j}^2$ 次, 因此\n$$\nn(n-1) \\geqslant \\sum_{j=1}^n \\mathrm{C}_{a_j}^2=\\sum_{j=1}^n \\frac{a_j\\left(a_j-1\\right)}{2} \\geqslant \\frac{1}{2} \\sum_{j=1}^n\\left(a_j-1\\right)^2 .\n$$\n由柯西不等式, 得\n$$\n\\left[\\sum_{j=1}^n\\left(a_j-1\\right)\\right]^2 \\leqslant n \\cdot \\sum_{j=1}^n\\left(a_j-1\\right)^2 \\leqslant n \\cdot 2 n(n-1)<2 n^3,\n$$\n于是从而\n$$\n\\sum_{j=1}^n\\left(a_j-1\\right)<\\sqrt{2} \\cdot \\sqrt{n^3}\n$$\n$$\nA=\\frac{\\sum_{j=1}^n a_j}{2}<\\frac{n+\\sqrt{2 n^3}}{2}<2 \\sqrt{n^3},\n$$\n故命题成立.", + "remark": "", + "figures": [] +} \ No newline at end of file diff --git a/processed_dataset/proof/1159.json b/processed_dataset/proof/1159.json new file mode 100644 index 0000000000000000000000000000000000000000..f04e18b03a543e1afb1f6c10fb840fc97309da7c --- /dev/null +++ b/processed_dataset/proof/1159.json @@ -0,0 +1,8 @@ +{ + "source_file": "./raw_volume-zh/volume4/chapter4-4.tex", + "problem_type": "proof", + "problem": "例7. 在三维空间中给定一点 $O$ 以及由总长度为 1988 的若干条线段组成的有限集 $A$, 证明: 存在一个平面与集 $A$ 不相交且到点 $O$ 的距离不超过 574 .", + "solution": "证明:以点 $O$ 为原点建立直角坐标系, 并将所给的线段分别向 3 条坐标轴投影.\n设 $A$ 中共有 $n$ 条线段且它们在 3 条轴上的投影长分别为\n$$\nx_i, y_i, z_i, i=1,2, \\cdots, n \\text {. }\n$$\n记 $x=\\sum x_i, y=\\sum y_i, z=\\sum z_i$. 于是, 由柯西不等式, 得\n$$\n\\begin{aligned}\nx^2+y^2+z^2 & =\\left(\\sum x_i\\right)^2+\\left(\\sum y_i\\right)^2+\\left(\\sum z_i\\right)^2 \\\\\n& =\\sum_{i=1}^n \\sum_{j=1}^n\\left(x_i x_j+y_i y_j+z_i z_j\\right) \\\\\n& \\leqslant \\sum_{i=1}^n \\sum_{j=1}^n \\sqrt{\\left(x_i^2+y_i^2+z_i^2\\right)\\left(x_j^2+y_j^2+z_j^2\\right)} \\\\\n& =\\left(\\sum_{i=1}^n \\sqrt{x_i^2+y_i^2+z_i^2}\\right)^2=1988^2 .\n\\end{aligned}\n$$\n不妨设 $x=\\min \\{x, y, z\\}$, 于是\n$$\nx \\leqslant \\frac{1988}{\\sqrt{3}}<2 \\times 574 .\n$$\n从而在 $x$ 轴上的区间 $[-574,574]$ 内必有一点不在 $n$ 条给定线段的投影\n136. 上,过这点作与 $x$ 轴垂直的平面便满足题中的要求.", + "remark": "", + "figures": [] +} \ No newline at end of file diff --git a/processed_dataset/proof/1160.json b/processed_dataset/proof/1160.json new file mode 100644 index 0000000000000000000000000000000000000000..e7f6bdf5319c3c727a7ed8c422a2fa3c4e2c8dba --- /dev/null +++ b/processed_dataset/proof/1160.json @@ -0,0 +1,8 @@ +{ + "source_file": "./raw_volume-zh/volume4/chapter4-4.tex", + "problem_type": "proof", + "problem": "例8. 设 $O x y z$ 是空间直角坐标系, $S$ 是空间中一个有限点集, $S_x, S_y, S_z$ 分别是 $S$ 中所有点在 $O y z$ 平面, $O z x$ 平面和 $O x y$ 平面上的正投影所成的集合.\n求证:\n$$\n|S|^2 \\leqslant\\left|S_x\\right| \\cdot\\left|S_y\\right| \\cdot\\left|S_z\\right| .\n$$\n说明所谓一个点在一个平面上的正投影是指由点向平面所作垂线的 垂足.", + "solution": "证明:设共有 $n$ 个平行于 $O x y$ 平面的平面上有 $S$ 中的点, 这些平面分别记为 $M_1, M_2, \\cdots, M_n$. 对于平面 $M_i, 1 \\leqslant i \\leqslant n$, 设它与 $O z x, O z y$ 平面分别交于直线 $l_y$ 和 $l_x$, 并设 $M_i$ 上有 $m_i$ 个 $S$ 中的点.\n显然, $m_i \\leqslant\\left|S_z\\right|$.\n设 $M_i$ 上的点在 $l_x, l_y$ 上的正投影的集合分别为 $A_i$ 和 $B_i$, 记 $a_i=\\left|A_i\\right|$, $b_i=\\left|B_i\\right|$, 则有 $m_i \\leqslant a_i b_i$. 又因为\n$$\n\\sum_{i=1}^n a_i=\\left|S_y\\right|, \\sum_{i=1}^n b_i=\\left|S_x\\right|, \\sum_{i=1}^n m_i=|S|,\n$$\n从而由柯西不等式,得\n$$\n\\begin{aligned}\n\\left|S_x\\right| \\cdot\\left|S_y\\right| \\cdot\\left|S_z\\right| & =\\left(\\sum_{i=1}^n b_i\\right)\\left(\\sum_{i=1}^n a_i\\right) \\cdot\\left|S_z\\right| \\\\\n& \\geqslant\\left(\\sum_{i=1}^n \\sqrt{a_i b_i}\\right)^2 \\cdot\\left|S_z\\right| \\\\\n& =\\left(\\sum_{i=1}^n \\sqrt{a_i b_i \\mid S_z}\\right)^2 \\\\\n& \\geqslant\\left(\\sum_{i=1}^n m_i\\right)^2=|S|^2 .\n\\end{aligned}\n$$\n得证.", + "remark": "", + "figures": [] +} \ No newline at end of file diff --git a/processed_dataset/proof/1161.json b/processed_dataset/proof/1161.json new file mode 100644 index 0000000000000000000000000000000000000000..9d5ad8a2a5c2d5659b1f418fb4ab0b8246758f73 --- /dev/null +++ b/processed_dataset/proof/1161.json @@ -0,0 +1,8 @@ +{ + "source_file": "./raw_volume-zh/volume4/chapter4-5.tex", + "problem_type": "proof", + "problem": "例1. 已知正实数 $a 、 b 、 c 、 d$ 满足\n$$\na\\left(c^2-1\\right)=b\\left(b^2+c^2\\right),\n$$\n且 $d \\leqslant 1$. 证明:\n$$\nd\\left(a \\sqrt{1-d^2}+b^2 \\sqrt{1+d^2}\\right) \\leqslant \\frac{(a+b) c}{2} .\n$$", + "solution": "证明:设参数 $\\lambda>1$, 由柯西不等式得\n$$\n\\begin{aligned}\n& d\\left(a \\sqrt{1-d^2}+b^2 \\sqrt{1+d^2}\\right) \\\\\n\\leqslant & d \\sqrt{\\left(\\frac{a^2}{\\lambda}+b^4\\right)\\left[\\left(1-d^2\\right) \\lambda+\\left(1+d^2\\right)\\right]} \\\\\n= & \\sqrt{\\left(\\frac{a^2}{\\lambda}+b^4\\right)\\left[(1-\\lambda) d^4+(\\lambda+1) d^2\\right]} \\\\\n\\leqslant & \\sqrt{\\frac{1}{\\lambda-1}\\left(\\frac{a^2}{\\lambda}+b^4\\right)} \\cdot \\frac{\\lambda+1}{2} .\n\\end{aligned}\n$$\n由已知条件知 $c^2=\\frac{a+b^3}{a-b}$. 故 $a>b$, 取 $\\lambda=\\frac{a}{b}$. 则\n$$\n\\frac{\\lambda+1}{2} \\sqrt{\\frac{1}{\\lambda-1}\\left(\\frac{a^2}{\\lambda}+b^4\\right)}=\\frac{a+b}{2} \\sqrt{\\frac{a+b^3}{a-b}}=\\frac{(a+b) c}{2} .\n$$\n所以, 命题得证.", + "remark": "", + "figures": [] +} \ No newline at end of file diff --git a/processed_dataset/proof/1162.json b/processed_dataset/proof/1162.json new file mode 100644 index 0000000000000000000000000000000000000000..0468d4faca8234bf68257a401489d823e1736262 --- /dev/null +++ b/processed_dataset/proof/1162.json @@ -0,0 +1,8 @@ +{ + "source_file": "./raw_volume-zh/volume4/chapter4-5.tex", + "problem_type": "proof", + "problem": "例3. (1)设 3 个正实数 $a 、 b 、 c$ 满足\n$$\n\\left(a^2+b^2+c^2\\right)^2>2\\left(a^4+b^4+c^4\\right),\n$$\n求证: $a 、 b 、 c$ 一定是某个三角形的 3 条边长;\n(2)设 $n$ 个正实数 $a_1, a_2, \\cdots, a_n$ 满足\n$$\n\\left(a_1^2+a_2^2+\\cdots+a_n^2\\right)^2>(n-1)\\left(a_1^4+a_2^4+\\cdots+a_n^4\\right) .\n$$\n求证: 这些数中任意 3 个一定是某个三角形的 3 条边长.", + "solution": "证明:(1) 不妨设 $a \\geqslant b \\geqslant c>0$, 由题设,得\n$$\n\\left(a^2+b^2+c^2\\right)^2-2\\left(a^4+b^4+c^4\\right)>0 .\n$$\n分解因式, 得\n$$\n(a+b+c)(a+b-c)(a+c-b)(b+c-a)>0,\n$$\n所以 $b+c-a>0$, 即 $b+c>a$, 从而 $a, b, c$ 是某个三角形的 3 条边长;\n(2) 在 $a_1, a_2, \\cdots, a_n$ 中任取 3 个, 不妨设为 $a_1, a_2, a_3$. 由带参数的柯西不等式,得\n$$\n\\begin{aligned}\n&(n-1)\\left(\\sum a_i^4\\right) \\\\\n&<\\left(\\sum_{i=1}^n a_i^2\\right)^2 \\\\\n&= {\\left[\\lambda\\left(a_1^2+a_2^2+a_3^2\\right) \\cdot \\frac{1}{\\lambda}+\\sum_{i=4}^n a_i^2\\right]^2 } \\\\\n& \\leqslant {\\left[\\lambda^2\\left(a_1^2+a_2^2+a_3^2\\right)^2+\\sum_{i=4}^n a_i^4\\right]\\left(\\frac{1}{\\lambda^2}+n-3\\right) . } \\\\\n& \\text { 令 } \\frac{1}{\\lambda^2}+n-3=n-1, \\text { 即 } \\lambda=\\sqrt{\\frac{1}{2}}, \\text { 所以 } \\\\\n& \\quad\\left(a_1^2+a_2^2+a_3^2\\right)^2>2\\left(a_1^4+a_2^4+a_3^4\\right) .\n\\end{aligned}\n$$\n由 (1) 知, $a_1, a_2, a_3$ 为某个三角形的三边长.", + "remark": "", + "figures": [] +} \ No newline at end of file diff --git a/processed_dataset/proof/1163.json b/processed_dataset/proof/1163.json new file mode 100644 index 0000000000000000000000000000000000000000..af3d9f62968a3587aaed6043dcac62ad43b97e99 --- /dev/null +++ b/processed_dataset/proof/1163.json @@ -0,0 +1,8 @@ +{ + "source_file": "./raw_volume-zh/volume4/chapter4-5.tex", + "problem_type": "proof", + "problem": "例4. 设 $a=\\left(a_1, a_2, \\cdots, a_n\\right)$ 和 $b=\\left(b_1, b_2, \\cdots, b_n\\right)$ 是两个不成比例的实数序列, 又设 $x=\\left(x_1, x_2, \\cdots, x_n\\right)$ 是使\n$$\n\\sum_{i=1}^n a_i x_i=0, \\sum_{i=1}^n b_i x_i=1\n$$\n成立的任意实数序列.\n求证:\n$$\n\\sum_{i=1}^n x_i^2 \\geqslant \\frac{A}{A B-C^2},\n$$\n其中 $A=\\sum_{i=1}^n a_i^2, B=\\sum_{i=1}^n b_i^2, C=\\sum_{i=1}^n a_i b_i$.", + "solution": "证明:对任意实数 $\\lambda$,由柯西不等式,得\n$$\n\\left(\\sum_{i=1}^n x_i^2\\right) \\sum_{i=1}^n\\left(a_i \\lambda-b_i\\right)^2 \\geqslant\\left(\\lambda \\sum_{i=1}^n a_i x_i-\\sum_{i=1}^n b_i x_i\\right)^2=1 .\n$$\n从而\n$$\n\\left(\\sum_{i=1}^n x_i^2\\right)\\left(A \\lambda^2-2 C \\lambda+B\\right) \\geqslant 1,\n$$\n即对任意实数 $\\lambda$,有\n$$\nA \\lambda^2-2 C \\lambda+B-\\frac{1}{\\sum_{i=1}^n x_i^2} \\leqslant 0 .\n$$\n于是\n$$\n\\Delta=4 C^2-4 A B+\\frac{4 A}{\\sum_{i=1}^n x_i^2} \\leqslant 0,\n$$\n故命题成立.", + "remark": "注:该不等式的证明,也可通过构造一个新的序列 $\\left\\{y_i\\right\\}$ :\n$$\ny_i=\\frac{A b_i-C a_i}{A B-C^2}, i \\geqslant 1,\n$$\n则 $\\left\\{y_i\\right\\}$ 满足条件\n$$\n\\begin{gathered}\n\\sum_{i=1}^n x_i y_i=\\frac{A}{A B-C^2}, \\sum_{i=1}^n y_i^2=\\frac{A}{A B-C^2}, \\\\\n\\sum_{i=1}^n x_i^2-\\sum_{i=1}^n y_i^2=\\sum_{i=1}^n\\left(x_i-y_i\\right)^2,\n\\end{gathered}\n$$\n从而命题成立.", + "figures": [] +} \ No newline at end of file diff --git a/processed_dataset/proof/1164.json b/processed_dataset/proof/1164.json new file mode 100644 index 0000000000000000000000000000000000000000..db937f6840aefd87ed7956f668d9c4011cfc8807 --- /dev/null +++ b/processed_dataset/proof/1164.json @@ -0,0 +1,8 @@ +{ + "source_file": "./raw_volume-zh/volume4/chapter4-6.tex", + "problem_type": "proof", + "problem": "例1. 设 $a, b, c$ 为实数, 满足 $a^2+2 b^2+3 c^2=\\frac{3}{2}$, 求证:\n$$\n3^{-a}+9^{-b}+27^{-c} \\geqslant 1 .\n$$", + "solution": "证明:由平均值不等式, 得\n$$\n3^{-a}+9^{-b}+27^{-c} \\geqslant 3 \\sqrt[3]{3^{-a-2 b-3 c}}=3^{\\frac{3-a-2 b-3 c}{3}} .\n$$\n再由柯西不等式, 得\n$$\n\\begin{aligned}\n(a+2 b+3 c)^2 & =(a+\\sqrt{2} \\cdot \\sqrt{2} b+\\sqrt{3} \\cdot \\sqrt{3} c)^2 \\\\\n& \\leqslant(1+2+3)\\left(a^2+2 b^2+3 c^2\\right) \\\\\n& =6 \\cdot \\frac{3}{2}=9 .\n\\end{aligned}\n$$\n从而 $a+2 b+3 c \\leqslant 3,3-a-2 b-3 c \\geqslant 0,3^{\\frac{3-a-2 b-3 c}{3}} \\geqslant 3^0=1$. 故命题成立.", + "remark": "", + "figures": [] +} \ No newline at end of file diff --git a/processed_dataset/proof/1165.json b/processed_dataset/proof/1165.json new file mode 100644 index 0000000000000000000000000000000000000000..8029e93a0a1b3a1c08ef1fe0d85ef9387251eb0d --- /dev/null +++ b/processed_dataset/proof/1165.json @@ -0,0 +1,8 @@ +{ + "source_file": "./raw_volume-zh/volume4/chapter4-6.tex", + "problem_type": "proof", + "problem": "例3. 设 $a, b, c$ 为正数, 且满足 $a b c=1$, 求证:\n$$\n\\frac{1}{a^3(b+c)}+\\frac{1}{b^3(a+c)}+\\frac{1}{c^3(a+b)} \\geqslant \\frac{3}{2} .\n$$", + "solution": "证明:由柯西不等式, 得\n$$\n\\begin{aligned}\n& {\\left[\\frac{1}{a^3(b+c)}+\\frac{1}{b^3(a+c)}+\\frac{1}{c^3(a+b)}\\right] \\cdot[a(b+c)+b(a+c)+c(a+b)] } \\\\\n\\geqslant & \\left(\\frac{1}{a}+\\frac{1}{b}+\\frac{1}{c}\\right)^2=(a b+b c+a c)^2,\n\\end{aligned}\n$$\n所以由平均值不等式, 得\n$$\n\\begin{aligned}\n& \\frac{1}{a^3(b+c)}+\\frac{1}{b^3(a+c)}+\\frac{1}{c^3(a+b)} . \\\\\n\\geqslant & \\frac{1}{2}(a b+b c+c a) \\\\\n\\geqslant & \\frac{1}{2} \\cdot 3 \\cdot \\sqrt[3]{a^2 b^2 c^2}=\\frac{3}{2} .\n\\end{aligned}\n$$", + "remark": "", + "figures": [] +} \ No newline at end of file diff --git a/processed_dataset/proof/1166.json b/processed_dataset/proof/1166.json new file mode 100644 index 0000000000000000000000000000000000000000..2f19e4b6591ad3c2ddac5130f57d9331ba04bfae --- /dev/null +++ b/processed_dataset/proof/1166.json @@ -0,0 +1,8 @@ +{ + "source_file": "./raw_volume-zh/volume4/chapter4-6.tex", + "problem_type": "proof", + "problem": "例4. 设 $x_i, i=1,2, \\cdots, n$ 为正数, 且满足 $\\sum_{i=1}^n x_i=a, a \\in \\mathbf{R}^{+}, m, n \\in \\mathbf{N}^*, n \\geqslant 2$, 求证:\n$$\n\\sum_{i=1}^n \\frac{x_i^m}{a-x_i} \\geqslant \\frac{a^{m-1}}{(n-1) n^{m-2}} .\n$$", + "solution": "证明:当 $m=1$ 时, 即证明\n$$\n\\sum_{i=1}^n \\frac{x_i}{a-x_i} \\geqslant \\frac{n}{n-1} .\n$$\n由于\n$$\n\\sum_{i=1}^n \\frac{x_i}{a-x_i}=\\sum_{i=1}^n\\left[\\left(\\frac{a}{a-x_i}\\right)-1\\right]=\\sum_{i=1}^n \\frac{a}{a-x_i}-n,\n$$\n由柯西不等式, 得\n$$\n\\sum_{i=1}^n \\frac{a}{a-x_i} \\cdot \\sum_{i=1}^n\\left(a-x_i\\right) \\geqslant a n^2,\n$$\n即\n$$\n\\sum_{i=1}^n \\frac{a}{a-x_i} \\geqslant \\frac{a n^2}{\\sum_{i=1}^n\\left(a-x_i\\right)}=\\frac{a n^2}{(n-1) a},\n$$\n所以\n$$\n\\sum_{i=1}^n \\frac{x_i}{a-x_i} \\geqslant \\frac{a n^2}{n a-a}-n=\\frac{n}{n-1},\n$$\n于是命题成立.\n当 $m \\geqslant 2$ 时, 由柯西不等式, 得\n$$\n\\sum_{i=1}^n \\frac{x_i^m}{a-x_i} \\cdot \\sum_{i=1}^n\\left(a-x_i\\right) \\geqslant\\left(\\sum_{i=1}^n x_i^{\\frac{m}{2}}\\right)^2 .\n$$\n再由幕平均值不等式, 得\n$$\n\\left(\\frac{1}{n} \\sum_{i=1}^n x_i^{\\frac{m}{2}}\\right)^2 \\geqslant\\left[\\frac{1}{n}\\left(\\sum_{i=1}^n x_i\\right)^{\\frac{m}{2}}\\right]^2=\\frac{a^m}{n^m} .\n$$\n由于 $\\sum_{i=1}^n\\left(a-x_i\\right)=(n-1) a$, 于是\n$$\n\\sum_{i=1}^n \\frac{x_i^m}{a-x_i} \\geqslant \\frac{a^{m-1}}{(n-1) n^{m-2}} .\n$$", + "remark": "", + "figures": [] +} \ No newline at end of file diff --git a/processed_dataset/proof/1167.json b/processed_dataset/proof/1167.json new file mode 100644 index 0000000000000000000000000000000000000000..5a5408062bd808b5d8fe527fbed13c0fe5e73ac6 --- /dev/null +++ b/processed_dataset/proof/1167.json @@ -0,0 +1,8 @@ +{ + "source_file": "./raw_volume-zh/volume4/chapter4-6.tex", + "problem_type": "proof", + "problem": "例5. 设实数 $x_i$ 满足 $\\left|x_i\\right|<1(i=1,2, \\cdots, n), n \\geqslant 2$, 求证:\n$$\n\\sum_{i=1}^n \\frac{1}{1-\\left|x_i\\right|^n} \\geqslant \\frac{n}{1-\\prod_{i=1}^n x_i} .\n$$", + "solution": "证明:由柯西不等式, 得\n$$\n\\sum_{i=1}^n \\frac{1}{1-\\left|x_i\\right|^n} \\cdot \\sum_{i=1}^n\\left(1-\\left|x_i\\right|^n\\right) \\geqslant n^2 .\n$$\n因此欲证原不等式只要证明即证\n$$\n\\begin{gathered}\n\\frac{n^2}{\\sum_{i=1}^n\\left(1-\\left|x_i\\right|^n\\right)} \\geqslant \\frac{n}{1-\\prod_{i=1}^n x_i}, \\\\\nn-n \\prod_{i=1}^n x_i \\geqslant \\sum_{i=1}^n\\left(1-\\left|x_i\\right|^n\\right), \\\\\n\\sum_{i=1}^n\\left|x_i\\right|^n \\geqslant n \\prod_{i=1}^n x_i .\n\\end{gathered}\n$$\n由平均值不等式知上述不等式成立, 故原命题成立.", + "remark": "", + "figures": [] +} \ No newline at end of file diff --git a/processed_dataset/proof/1168.json b/processed_dataset/proof/1168.json new file mode 100644 index 0000000000000000000000000000000000000000..719e0d41b0dd55e8fb9a706a5a9c22e00452ceaa --- /dev/null +++ b/processed_dataset/proof/1168.json @@ -0,0 +1,8 @@ +{ + "source_file": "./raw_volume-zh/volume4/chapter4-6.tex", + "problem_type": "proof", + "problem": "例6. 已知正数 $x_i$ 满足 $\\sum_{i=1}^n \\frac{1}{1+x_i}=1$, 证明:\n$$\n\\prod_{i=1}^n x_i \\geqslant(n-1)^n \\text {. }\n$$", + "solution": "证明:由柯西不等式, 得\n$$\n\\sum_{i=1}^n \\frac{1}{1+x_i} \\cdot \\sum_{i=1}^n \\frac{1+x_i}{x_i} \\geqslant\\left(\\sum_{i=1}^n \\frac{1}{\\sqrt{x_i}}\\right)^2,\n$$\n即\n$$\n\\sum_{i=1}^n \\frac{1}{x_i}+n \\geqslant \\sum_{i=1}^n \\frac{1}{x_i}+2 \\sum_{1 \\leqslant i0$, 求证:\n$$\n\\sum \\sqrt{\\frac{5 a^2+8 b^2+5 c^2}{4 a c}} \\geqslant 3 \\sqrt[9]{\\frac{8(a+b)^2(b+c)^2(c+a)^2}{(a b c)^2}} .\n$$", + "solution": "证明:由柯西不等式及均值不等式有\n$$\n\\begin{aligned}\n5 a^2+8 b^2+5 c^2 & \\geqslant 4\\left(a^2+b^2\\right)+4\\left(b^2+c^2\\right) \\\\\n& \\geqslant 2(a+b)^2+2(b+c)^2 \\\\\n& \\geqslant 4(a+b)(b+c),\n\\end{aligned}\n$$\n所以\n$$\n\\sum \\sqrt{\\frac{5 a^2+8 b^2+5 c^2}{4 a c}} \\geqslant \\sum \\sqrt{\\frac{(a+b)(b+c)}{a c}} \\geqslant 3 \\sqrt[6]{\\frac{(a+b)^2(b+c)^2(c+a)^2}{(a b c)^2}},\n$$\n只需要证明\n$$\n\\sqrt[6]{\\frac{(a+b)^2(b+c)^2(c+a)^2}{(a b c)^2}} \\geqslant \\sqrt[9]{\\frac{8(a+b)^2(b+c)^2(c+a)^2}{(a b c)^2}},\n$$\n等价于 $(a+b)(b+c)(c+a) \\geqslant 8 a b c$, 即 $\\sum a(b-c)^2 \\geqslant 0$, 明显成立.", + "remark": "", + "figures": [] +} \ No newline at end of file diff --git a/processed_dataset/proof/1171.json b/processed_dataset/proof/1171.json new file mode 100644 index 0000000000000000000000000000000000000000..1b12a1f551fcf01202f89e08bb6a803ee061c784 --- /dev/null +++ b/processed_dataset/proof/1171.json @@ -0,0 +1,8 @@ +{ + "source_file": "./raw_volume-zh/volume4/chapter4-6.tex", + "problem_type": "proof", + "problem": "例9. 已知数列 $\\left\\{a_n\\right\\}$ 满足 $a_1>0, a_2>0, a_{n+2}=\\frac{2}{a_n+a_{n+1}} . M_n= \\max \\left\\{a_n, \\frac{1}{a_n}, \\frac{1}{a_{n+1}}, a_{n+1}\\right\\}$. 求证:\n$$\nM_{n+3} \\leqslant \\frac{3}{4} M_n+\\frac{1}{4} .\n$$", + "solution": "证明:由于\n$$\n\\begin{gathered}\nM_{n+3}=\\max \\left\\{a_{n+3}, a_{n+4}, \\frac{1}{a_{n+3}}, \\frac{1}{a_{n+4}}\\right\\}, \\\\\na_{n+3} \\leqslant \\frac{3}{4} M_n+\\frac{1}{4}, \\\\\na_{n+4} \\leqslant \\frac{3}{4} M_n+\\frac{1}{4},\n\\end{gathered}\n$$\n我们需证\n$$\n\\begin{aligned}\n& a_{n+3} \\leqslant \\frac{3}{4} M_n+\\frac{1}{4} \\\\\n& a_{n+4} \\leqslant \\frac{3}{4} M_n+\\frac{1}{4}\n\\end{aligned}\n$$\n$$\n\\begin{aligned}\n& \\frac{1}{a_{n+3}} \\leqslant \\frac{3}{4} M_n+\\frac{1}{4}, \\\\\n& \\frac{1}{a_{n+4}} \\leqslant \\frac{3}{4} M_n+\\frac{1}{4} .\n\\end{aligned}\n$$\n由于\n$$\n\\begin{aligned}\na_{n+3} & =\\frac{2}{a_{n+1}+a_{n+2}} \\leqslant \\frac{\\frac{1}{a_{n+1}}+\\frac{1}{a_{n+2}}}{2} \\\\\n& =\\frac{1}{2}\\left(\\frac{1}{a_{n+1}}+\\frac{a_n+a_{n+1}}{2}\\right) \\\\\n& =\\frac{1}{4}\\left(a_{n+1}+\\frac{1}{a_{n+1}}\\right)+\\frac{1}{4} \\cdot \\frac{1}{a_{n+1}}+\\frac{1}{4} a_n \\\\\n& \\leqslant \\frac{1}{4}\\left[\\min \\left(a_{n+1}, \\frac{1}{a_{n+1}}\\right)+\\max \\left(a_{n+1}, \\frac{1}{a_{n+1}}\\right)\\right]+\\frac{1}{4} M_n+\\frac{1}{4} M_n \\\\\n& \\leqslant \\frac{1}{4}\\left(1+M_n\\right)+\\frac{1}{2} M_n \\\\\n& =\\frac{3}{4} M_n+\\frac{1}{4} ; \\\\\n\\frac{1}{a_{n+3}} & =\\frac{a_{n+1}+a_{n+2}}{2}=\\frac{1}{2} \\cdot \\frac{1}{a_{n+1}}+\\frac{1}{a_n+a_{n+1}} \\\\\n& \\leqslant \\frac{1}{2}+\\frac{\\frac{1}{a_n}+\\frac{1}{a_{n+1}}}{4} \\\\\n& \\leqslant \\frac{1}{4}\\left(a_{n+1}+\\frac{1}{a_{n+1}}\\right)+\\frac{1}{4}\\left(\\frac{1}{a_n}+\\frac{1}{a_{n+1}}\\right) \\\\\n& =\\frac{1}{4}\\left[\\max \\left(a_{n+1}, \\frac{1}{a_{n+1}}\\right)+\\min \\left(a_{n+1}, \\frac{1}{a_{n+1}}\\right)\\right]+\\frac{1}{4}\\left(\\frac{1}{a_n}+\\frac{1}{a_{n+1}}\\right) \\\\\n& \\leqslant \\frac{1}{4}\\left(M_n+1\\right)+\\frac{1}{4} \\cdot 2 M_n \\\\\n& =\\frac{3}{4} M_n+\\frac{1}{4} ; \\\\\n& =\\frac{1}{a_{n+2}+a_{n+3}} \\leqslant \\frac{a_n+a_{n+1}}{a_n}+\\frac{1}{2} a_{n+1}+\\frac{1}{2} \\cdot \\frac{1}{a_n+a_{n+1}}+a_{n+2}\n\\end{aligned}\n$$\n$$\n\\begin{aligned}\n& \\leqslant \\frac{1}{4} a_n+\\frac{1}{2} a_{n+1}+\\frac{1}{8}\\left(\\frac{1}{a_n}+\\frac{1}{a_{n+1}}\\right) \\\\\n& =\\frac{1}{8}\\left(a_n+\\frac{1}{a_n}\\right)+\\frac{1}{8}\\left(a_{n+1}+\\frac{1}{a_{n+1}}\\right)+\\frac{1}{8} a_n+\\frac{3}{8} a_{n+1} \\\\\n& =\\frac{1}{8}\\left[\\max \\left(a_n, \\frac{1}{a_n}\\right)+\\min \\left(a_n, \\frac{1}{a_n}\\right)\\right] \\\\\n& +\\frac{1}{8}\\left[\\max \\left(a_{n+1}, \\frac{1}{a_{n+1}}\\right)+\\min \\left(a_{n+1}, \\frac{1}{a_{n+1}}\\right)\\right]+\\frac{1}{8} a_n+\\frac{3}{8} a_{n+1} \\\\\n& \\leqslant \\frac{1}{8}\\left(M_n+1\\right)+\\frac{1}{8}\\left(M_n+1\\right)+\\frac{1}{8} M_n+\\frac{3}{8} M_n \\\\\n& =\\frac{3}{4} M_n+\\frac{1}{4} ; \\\\\n& \\frac{1}{a_{n+4}}=\\frac{a_{n+2}+a_{n+3}}{2}=\\frac{1}{a_n+a_{n+1}}+\\frac{1}{a_{n+1}+a_{n+2}} \\\\\n& \\leqslant \\frac{\\frac{1}{a_n}+\\frac{1}{a_{n+1}}}{4}+\\frac{\\frac{1}{a_{n+1}}+\\frac{1}{a_{n+2}}}{4} \\\\\n& =\\frac{1}{4} \\cdot \\frac{1}{a_n}+\\frac{1}{2} \\cdot \\frac{1}{a_{n+1}}+\\frac{1}{4} \\cdot \\frac{1}{a_{n+2}} \\\\\n& =\\frac{1}{4} \\cdot \\frac{1}{a_n}+\\frac{1}{2} \\cdot \\frac{1}{a_{n+1}}+\\frac{1}{8}\\left(a_n+a_{n+1}\\right) \\\\\n& =\\frac{1}{8}\\left(a_n+\\frac{1}{a_n}\\right)+\\frac{1}{8}\\left(a_{n+1}+\\frac{1}{a_{n+1}}\\right)+\\frac{1}{8} \\cdot \\frac{1}{a_n}+\\frac{3}{8} \\cdot \\frac{1}{a_{n+1}} \\\\\n& =\\frac{1}{8}\\left[\\max \\left(a_n, \\frac{1}{a_n}\\right)+\\min \\left(a_n, \\frac{1}{a_n}\\right)\\right] \\\\\n& +\\frac{1}{8}\\left[\\max \\left(a_{n+1}, \\frac{1}{a_{n+1}}\\right)+\\min \\left(a_{n+1}, \\frac{1}{a_{n+1}}\\right)\\right]+\\frac{1}{8} \\cdot \\frac{1}{a_n}+\\frac{3}{8} \\cdot \\frac{1}{a_{n+1}} \\\\\n& \\leqslant \\frac{1}{8}\\left(M_n+1\\right)+\\frac{1}{8}\\left(M_n+1\\right)+\\frac{1}{8} M_n+\\frac{3}{8} M_n \\\\\n& =\\frac{3}{4} M_n+\\frac{1}{4} \\text {. } \\\\\n&\n\\end{aligned}\n$$\n因此, $M_{n+3} \\leqslant \\frac{3}{4} M_n+\\frac{1}{4}$.", + "remark": "注:当 $x, y>0$ 时, $x+y=\\max (x, y)+\\min (x, y)$; 当 $x>0$ 时, $\\min \\left(x, \\frac{1}{x}\\right) \\leqslant 1$.", + "figures": [] +} \ No newline at end of file diff --git a/processed_dataset/proof/1172.json b/processed_dataset/proof/1172.json new file mode 100644 index 0000000000000000000000000000000000000000..2a539bae9c50ceab5837fef1934c308f4cdf7f92 --- /dev/null +++ b/processed_dataset/proof/1172.json @@ -0,0 +1,8 @@ +{ + "source_file": "./raw_volume-zh/volume4/chapter4-6.tex", + "problem_type": "proof", + "problem": "例10. 已知正实数 $x 、 y 、 z$ 满足 $\\sqrt{x}+\\sqrt{y}+\\sqrt{z}=1$. 求证:\n$$\n\\frac{x^2+y z}{\\sqrt{2 x^2(y+z)}}+\\frac{y^2+z x}{\\sqrt{2 y^2(z+x)}}+\\frac{z^2+x y}{\\sqrt{2 z^2(x+y)}} \\geqslant 1 .\n$$", + "solution": "证明:证法 1 注意到\n$$\n\\begin{aligned}\n& \\frac{x^2+y z}{\\sqrt{2 x^2(y+z)}} \\\\\n= & \\frac{x^2-x(y+z)+y z}{\\sqrt{2 x^2(y+z)}}+\\frac{x(y+z)}{\\sqrt{2 x^2(y+z)}} \\\\\n= & \\frac{(x-y)(x-z)}{\\sqrt{2 x^2(y+z)}}+\\sqrt{\\frac{y+z}{2}} \\\\\n\\geqslant & -\\frac{(x-y)(x-z)}{\\sqrt{2 x^2(y+z)}}+\\frac{\\sqrt{y}+\\sqrt{z}}{2} .\n\\end{aligned}\n$$\n同理,\n$$\n\\begin{aligned}\n& \\frac{y^2+z x}{\\sqrt{2 y^2(z+x)}} \\geqslant \\frac{(y-z)(y-x)}{\\sqrt{2 y^2(z+x)}}+\\frac{\\sqrt{z}+\\sqrt{x}}{2}, \\\\\n& \\frac{z^2+x y}{\\sqrt{2 z^2(x+y)}} \\geqslant \\frac{(z-x)(z-y)}{\\sqrt{2 z^2(x+y)}}+\\frac{\\sqrt{x}+\\sqrt{y}}{2} .\n\\end{aligned}\n$$\n以上三式相加得\n$$\n\\begin{aligned}\n& \\frac{x^2+y z}{\\sqrt{2 x^2(y+z)}}+\\frac{y^2+z x}{\\sqrt{2 y^2(z+x)}}+\\frac{z^2+x y}{\\sqrt{2 z^2(x+y)}} \\\\\n\\geqslant & \\frac{(x-y)(x-z)}{\\sqrt{2 x^2(y+z)}}+\\frac{(y-z)(y-x)}{\\sqrt{2 y^2(z+x)}}+\\frac{(z-x)(z-y)}{\\sqrt{2 z^2(x+y)}}+\\sqrt{x}+\\sqrt{y}+\\sqrt{z} \\\\\n= & \\frac{(x-y)(x-z)}{\\sqrt{2 x^2(y+z)}}+\\frac{(y-z)(y-x)}{\\sqrt{2 y^2(z+x)}}+\\frac{(z-x)(z-y)}{\\sqrt{2 z^2(x+y)}}+1 .\n\\end{aligned}\n$$\n从而, 只需证明\n$$\n\\frac{(x-y)(x-z)}{\\sqrt{2 x^2(y+z)}}+\\frac{(y-z)(y-x)}{\\sqrt{2 y^2(z+x)}}+\\frac{(z-x)(z-y)}{\\sqrt{2 z^2(x+y)}} \\geqslant 0 .\n$$\n不失一般性, 设 $x \\geqslant y \\geqslant z$. 于是,\n$$\n\\frac{(x-y)(x-z)}{\\sqrt{2 x^2(y+z)}} \\geqslant 0,\n$$\n且\n$$\n\\begin{aligned}\n& \\frac{(y-z)(y-x)}{\\sqrt{2 y^2(z+x)}}+\\frac{(z-x)(z-y)}{\\sqrt{2 z^2(x+y)}} \\\\\n= & \\frac{(y-z)(x-z)}{\\sqrt{2 z^2(x+y)}}-\\frac{(y-z)(x-y)}{\\sqrt{2 y^2(z+x)}} \\\\\n\\geqslant & \\frac{(y-z)(x-y)}{\\sqrt{2 z^2(x+y)}}-\\frac{(y-z)(x-y)}{\\sqrt{2 y^2(z+x)}} \\\\\n= & (y-z)(x-y) \\cdot\\left[\\frac{1}{\\sqrt{2 z^2(x+y)}}-\\frac{1}{\\sqrt{2 y^2(z+x)}}\\right] . \\label{(44)}\n\\end{aligned}\n$$\n事实上,由\n$$\ny^2(z+x)=y^2 z+y^2 x \\geqslant y z^2+z^2 x=z^2(x+y)\n$$\n可知式 (44) 非负.\n从而,题中不等式成立.", + "remark": "", + "figures": [] +} \ No newline at end of file diff --git a/processed_dataset/proof/1173.json b/processed_dataset/proof/1173.json new file mode 100644 index 0000000000000000000000000000000000000000..6626bdcd1bcafedfa4610ecef4c558148768aaf0 --- /dev/null +++ b/processed_dataset/proof/1173.json @@ -0,0 +1,8 @@ +{ + "source_file": "./raw_volume-zh/volume4/chapter4-6.tex", + "problem_type": "proof", + "problem": "例10. 已知正实数 $x 、 y 、 z$ 满足 $\\sqrt{x}+\\sqrt{y}+\\sqrt{z}=1$. 求证:\n$$\n\\frac{x^2+y z}{\\sqrt{2 x^2(y+z)}}+\\frac{y^2+z x}{\\sqrt{2 y^2(z+x)}}+\\frac{z^2+x y}{\\sqrt{2 z^2(x+y)}} \\geqslant 1 .\n$$", + "solution": "证法 2 根据柯西不等式得\n$$\n\\begin{aligned}\n& {\\left[\\frac{x^2}{\\sqrt{2 x^2(y+z)}}+\\frac{y^2}{\\sqrt{2 y^2(z+x)}}+\\frac{z^2}{\\sqrt{2 z^2(x+y)}}\\right] . } \\\\\n& {[\\sqrt{2(y+z)}+\\sqrt{2(z+x)}+\\sqrt{2(x+y)}] } \\\\\n\\geqslant & (\\sqrt{x}+\\sqrt{y}+\\sqrt{z})^2=1 \\\\\n& {\\left[\\frac{y z}{\\sqrt{2 x^2(y+z)}}+\\frac{z x}{\\sqrt{2 y^2(z+x)}}+\\frac{x y}{\\sqrt{2 z^2(x+y)}}\\right] . } \\\\\n& {[\\sqrt{2(y+z)}+\\sqrt{2(z+x)}+\\sqrt{2(x+y)}] } \\\\\n\\geqslant & \\left(\\sqrt{\\frac{y z}{x}}+\\sqrt{\\frac{z x}{y}}+\\sqrt{\\frac{x y}{z}}\\right)^2 .\n\\end{aligned}\n$$\n和\n$$\n\\begin{aligned}\n& {\\left[\\frac{y z}{\\sqrt{2 x^2(y+z)}}+\\frac{z x}{\\sqrt{2 y^2(z+x)}}+\\frac{x y}{\\sqrt{2 z^2(x+y)}}\\right] . } \\\\\n& {[\\sqrt{2(y+z)}+\\sqrt{2(z+x)}+\\sqrt{2(x+y)}] } \\\\\n\\geqslant & \\left(\\sqrt{\\frac{y z}{x}}+\\sqrt{\\frac{z x}{y}}+\\sqrt{\\frac{x y}{z}}\\right)^2 .\n\\end{aligned}\n$$\n以上两式相加得\n$$\n\\begin{aligned}\n& {\\left[\\frac{x^2+y z}{\\sqrt{2 x^2(y+z)}}+\\frac{y^2+z x}{\\sqrt{2 y^2(z+x)}}+-\\frac{z^2+x y}{\\sqrt{2 z^2(x+y)}}\\right] . } \\\\\n& {[\\sqrt{2(y+z)}+\\sqrt{2(z+x)}+\\sqrt{2(x+y)}] } \\\\\n\\geqslant & 1+\\left(\\sqrt{\\frac{y z}{x}}+\\sqrt{\\frac{z x}{y}}+\\sqrt{\\frac{x y}{z}}\\right)^2 \\\\\n\\geqslant & 2\\left(\\sqrt{\\frac{y z}{x}}+\\sqrt{\\frac{z x}{y}}+\\sqrt{\\frac{x y}{z}}\\right) .\n\\end{aligned}\n$$\n从而, 只需证明\n$$\n\\begin{aligned}\n& 2\\left(\\sqrt{\\frac{y z}{x}}+\\sqrt{\\frac{z x}{y}}+\\sqrt{\\frac{x y}{z}}\\right) \\\\\n\\geqslant & \\sqrt{2(y+z)}+\\sqrt{2(z+x)}+\\sqrt{2(x+y)} .\n\\end{aligned}\n$$\n根据均值不等式得\n$$\n\\begin{aligned}\n& {\\left[\\sqrt{\\frac{y z}{x}}+\\left(\\frac{1}{2} \\sqrt{\\frac{z x}{y}}+\\frac{1}{2} \\sqrt{\\frac{x y}{z}}\\right)\\right]^2 } \\\\\n& \\geqslant 4 \\sqrt{\\frac{y z}{x}}\\left(\\frac{1}{2} \\sqrt{\\frac{z x}{y}}+\\frac{1}{2} \\sqrt{\\frac{x y}{z}}\\right)=2(y+z), \\\\\n& \\sqrt{\\frac{y z}{x}}+\\left(\\frac{1}{2} \\sqrt{\\frac{z x}{y}}+\\frac{1}{2} \\sqrt{\\frac{x y}{2}}\\right) \\geqslant \\sqrt{2(y+z)} .\n\\end{aligned}\n$$\n即同理,\n$$\n\\begin{aligned}\n& \\sqrt{\\frac{z x}{y}}+\\left(\\frac{1}{2} \\sqrt{\\frac{x y}{z}}+\\frac{1}{2} \\sqrt{\\frac{y z}{x}}\\right) \\geqslant \\sqrt{2(z+x)}, \\\\\n& \\sqrt{\\frac{x y}{2}}+\\left(\\frac{1}{2} \\sqrt{\\frac{y z}{x}}+\\frac{1}{2} \\sqrt{\\frac{z x}{y}}\\right) \\geqslant \\sqrt{2(x+y)} .\n\\end{aligned}\n$$\n以上三式相加得\n$$\n\\begin{aligned}\n& 2\\left(\\sqrt{\\frac{y z}{x}}+\\sqrt{\\frac{z x}{y}}+\\sqrt{\\frac{x y}{z}}\\right) \\\\\n\\geqslant & \\sqrt{2(y+z)}+\\sqrt{2(z+x)}+\\sqrt{2(x+y)} .\n\\end{aligned}\n$$\n从而, 题中不等式成立.", + "remark": "", + "figures": [] +} \ No newline at end of file diff --git a/processed_dataset/proof/1174.json b/processed_dataset/proof/1174.json new file mode 100644 index 0000000000000000000000000000000000000000..dfe148b0c034b1194bdb1aeafe0034f97dc76cfd --- /dev/null +++ b/processed_dataset/proof/1174.json @@ -0,0 +1,8 @@ +{ + "source_file": "./raw_volume-zh/volume4/chapter4-6.tex", + "problem_type": "proof", + "problem": "例11. 设正整数 $n \\geqslant 2$. 求常数 $C(n)$ 的最大值, 使得对于所有满足 $x_i \\in (0,1)(i=1,2, \\cdots, n)$, 且 $\\left(1-x_i\\right)\\left(1-x_j\\right) \\geqslant \\frac{1}{4}(1 \\leqslant i0, b>0, a+2 b=1$. 求证: $\\frac{1}{a}+\\frac{2}{b} \\geqslant 9$.", + "solution": "$1=a+2 b=a+b+b \\geqslant 3 \\sqrt[3]{a b^2}$, 故 $\\frac{1}{\\sqrt[3]{a b^2}} \\geqslant 3 . \\frac{1}{a}+\\frac{2}{b}=\\frac{1}{a}+\\frac{1}{b}+$\n$$\n\\frac{1}{b} \\geqslant 3 \\sqrt[3]{\\frac{1}{a b^2}} \\geqslant 3 \\times 3=9\n$$", + "remark": "", + "figures": [] +} \ No newline at end of file diff --git a/processed_dataset/proof/1176.json b/processed_dataset/proof/1176.json new file mode 100644 index 0000000000000000000000000000000000000000..db4a748b04e8bee8140cad1358a793d88d3fdc2c --- /dev/null +++ b/processed_dataset/proof/1176.json @@ -0,0 +1,8 @@ +{ + "source_file": "./raw_volume-zh/volume4/exercise1.tex", + "problem_type": "proof", + "problem": "问题2. 设 $a, b, c>0$, 求证: $\\left(a+\\frac{1}{b}\\right)\\left(b+\\frac{1}{c}\\right)\\left(c+\\frac{1}{a}\\right) \\geqslant 8$.", + "solution": "由均值不等式得 $\\left(a+\\frac{1}{b}\\right)\\left(b+\\frac{1}{c}\\right)\\left(c+\\frac{1}{a}\\right) \\geqslant 2 \\sqrt{\\frac{a}{b}} \\cdot 2 \\sqrt{\\frac{b}{c}}$. $2 \\sqrt{\\frac{c}{a}}=8$", + "remark": "", + "figures": [] +} \ No newline at end of file diff --git a/processed_dataset/proof/1177.json b/processed_dataset/proof/1177.json new file mode 100644 index 0000000000000000000000000000000000000000..89c0ba7e79bca52dc293dfb1148380eff3615c13 --- /dev/null +++ b/processed_dataset/proof/1177.json @@ -0,0 +1,8 @@ +{ + "source_file": "./raw_volume-zh/volume4/exercise1.tex", + "problem_type": "proof", + "problem": "问题3. 已知 $0a+b+c+6$. 综上, 原命题成立.", + "remark": "", + "figures": [] +} \ No newline at end of file diff --git a/processed_dataset/proof/1180.json b/processed_dataset/proof/1180.json new file mode 100644 index 0000000000000000000000000000000000000000..aff50f806ea802aac0b30bcbf1e98f82aafed1d2 --- /dev/null +++ b/processed_dataset/proof/1180.json @@ -0,0 +1,8 @@ +{ + "source_file": "./raw_volume-zh/volume4/exercise1.tex", + "problem_type": "proof", + "problem": "问题5. 正实数 $x 、 y 、 z$ 满足 $x y z=1$. 证明: $\\frac{x^3+y^3}{x^2+x y+y^2}+\\frac{y^3+z^3}{y^2+y z+z^2}+ \\frac{z^3+x^3}{z^2+z x+x^2} \\geqslant 2$.", + "solution": "注意到 $\\frac{x^2-x y+y^2}{x^2+x y+y^2} \\geqslant \\frac{1}{3} \\Leftrightarrow 3\\left(x^2-x y+y^2\\right) \\geqslant x^2+x y+y^2 \\Leftrightarrow 2(x- y)^2 \\geqslant 0$. 则 $\\frac{x^3+y^3}{x^2+x y+y^2}=\\frac{x^2-x y+y^2}{x^2+x y+y^2}(x+y) \\geqslant \\frac{x+y}{3}$. 故 $\\frac{x^3+y^3}{x^2+x y+y^2}+ \\frac{y^3+z^3}{y^2+y z+z^2}+\\frac{z^3+x^3}{z^2+z x+x^2} \\geqslant \\frac{1}{3}(x+y)+\\frac{1}{3}(y+z)+\\frac{1}{3}(z+x)=\\frac{2}{3}(x+ y+z) \\geqslant 2 \\sqrt[3]{x y z}=2$.", + "remark": "", + "figures": [] +} \ No newline at end of file diff --git a/processed_dataset/proof/1181.json b/processed_dataset/proof/1181.json new file mode 100644 index 0000000000000000000000000000000000000000..08c2312b8e3afc8d054ab5bb87dcc69ba0cd005e --- /dev/null +++ b/processed_dataset/proof/1181.json @@ -0,0 +1,8 @@ +{ + "source_file": "./raw_volume-zh/volume4/exercise1.tex", + "problem_type": "proof", + "problem": "问题6. 设 $a_1, a_2, \\cdots, a_n>0$ 且 $a_1+a_2+\\cdots+a_n=1$. 求证:\n$$\n\\left(\\frac{1}{a_1^2}-1\\right)\\left(\\frac{1}{a_2^2}-1\\right) \\cdots\\left(\\frac{1}{a_n^2}-1\\right) \\geqslant\\left(n^2-1\\right)^n \\text {. }\n$$", + "solution": "因为 $a_1+a_2+\\cdots+a_n=1$, 所以由均值不等式可得 $1+a_i=a_1+a_2 +\\cdots+a_n+a_i \\geqslant(n+1)\\left(a_1 a_2 \\cdots a_n a_i\\right)^{1 /(n+1)}, 1-a_i=a_1+a_2+\\cdots+a_n-a_i \\geqslant (n-1)\\left(a_1 a_2 \\cdots a_n / a_i\\right)^{1 /(n-1)}$. 取 $i=1,2, \\cdots, n$ 再将之分别累积后得 $\\prod_{i=1}^n(1- \\left.a_i^2\\right) \\geqslant\\left(n^2-1\\right)^n \\prod_{i=1}^n a_i^2$, 从而 $\\left(\\frac{1}{a_1^2}-1\\right)\\left(\\frac{1}{a_2^2}-1\\right) \\cdots\\left(\\frac{1}{a_n^2}-1\\right) \\geqslant\\left(n^2-1\\right)^n$.", + "remark": "", + "figures": [] +} \ No newline at end of file diff --git a/processed_dataset/proof/1182.json b/processed_dataset/proof/1182.json new file mode 100644 index 0000000000000000000000000000000000000000..8f1d62dccdb08912428b2765838fb02619bfb394 --- /dev/null +++ b/processed_dataset/proof/1182.json @@ -0,0 +1,8 @@ +{ + "source_file": "./raw_volume-zh/volume4/exercise1.tex", + "problem_type": "proof", + "problem": "问题7. 设 $a, b, c$ 为正数,且 $a+b+c=3$. 求证:\n$$\n\\sqrt{a}+\\sqrt{b}+\\sqrt{c} \\geqslant a b+b c+c a .\n$$", + "solution": "由条件等式有 $(a+b+c)^2=9$. 于是 $a b+b c+c a=\\frac{9-a^2-b^2-c^2}{2}$. 只需证明 $2 \\sqrt{a}+2 \\sqrt{b}+2 \\sqrt{c}+a^2+b^2+c^2 \\geqslant 9$ 为此先证 $2 \\sqrt{a}+a^2 \\geqslant 3 a$. 事实上, $2 \\sqrt{a}+a^2=\\sqrt{a}+\\sqrt{a}+a^2 \\geqslant 3 \\sqrt[3]{a^3}=3 a$. 类似可得其余两个不等式, 从而就有 $2 \\sqrt{a}+2 \\sqrt{b}+2 \\sqrt{c}+a^2+b^2+c^2 \\geqslant 3(a+b+c)=9$.", + "remark": "", + "figures": [] +} \ No newline at end of file diff --git a/processed_dataset/proof/1183.json b/processed_dataset/proof/1183.json new file mode 100644 index 0000000000000000000000000000000000000000..8d670ee29b81c5c6107afc07502c3baa67b692f8 --- /dev/null +++ b/processed_dataset/proof/1183.json @@ -0,0 +1,8 @@ +{ + "source_file": "./raw_volume-zh/volume4/exercise1.tex", + "problem_type": "proof", + "problem": "问题8. 已知 $x, y, z \\in \\mathbf{R}^{+}$, 且 $x+y+z=1$. 求证: $\\left(\\frac{1}{x}-x\\right)\\left(\\frac{1}{y}-y\\right)\\left(\\frac{1}{z}-z\\right) \\geqslant \\left(\\frac{8}{3}\\right)^3$.", + "solution": "设 $\\frac{1}{x}=a, \\frac{1}{y}=b, \\frac{1}{z}=c$, 代入已知条件等式得 $\\frac{1}{a}+\\frac{1}{b}+\\frac{1}{c}=1$, 即 $a b c=a b+b c+c a$, 由均值不等式易得到 $a b c \\geqslant 27$, 所以,\n$$\n\\begin{aligned}\n& \\left(\\frac{1}{x}-x\\right)\\left(\\frac{1}{y}-y\\right)\\left(\\frac{1}{z}-z\\right)=\\left(a-\\frac{1}{a}\\right)\\left(b-\\frac{1}{b}\\right)\\left(c-\\frac{1}{c}\\right)=\\frac{\\left(a^2-1\\right)\\left(b^2-1\\right)\\left(c^2-1\\right)}{a b c}= \\\\\n& \\frac{1}{a b c}\\left(a^2 b^2 c^2-a^2 b^2-b^2 c^2-c^2 a^2+a^2+b^2+c^2-1\\right)=\\frac{1}{a b c}\\left[(a b+b c+c a)^2-\\right. \\\\\n& \\left.a^2 b^2-b^2 c^2-c^2 a^2+a^2+b^2+c^2-1\\right]=\\frac{1}{a b c}\\left[2 a^2 b c+2 a b^2 c+2 a b c^2+a^2+b^2+\\right. \\\\\n& \\left.c^2-1\\right] \\geqslant \\frac{1}{a b c}\\left[2 a^2 b c+2 a b^2 c+2 a b c^2+a b+b c+c a-1\\right]=2(a+b+c)+1- \\\\\n& \\frac{1}{a b c} \\geqslant 6 \\sqrt[3]{a b c}+1-\\frac{1}{a b c} \\geqslant 6 \\times 3+1-\\frac{1}{27}=\\frac{512}{27}=\\left(\\frac{8}{3}\\right)^3 .\n\\end{aligned}\n$$", + "remark": "", + "figures": [] +} \ No newline at end of file diff --git a/processed_dataset/proof/1184.json b/processed_dataset/proof/1184.json new file mode 100644 index 0000000000000000000000000000000000000000..145d98e7d41868a1af57ac5758414c5b6e0e1e85 --- /dev/null +++ b/processed_dataset/proof/1184.json @@ -0,0 +1,8 @@ +{ + "source_file": "./raw_volume-zh/volume4/exercise1.tex", + "problem_type": "proof", + "problem": "问题9. 设 $x_1, x_2, \\cdots, x_n>0$, 且 $x_1 x_2 \\cdots x_n=1$. 求证: $\\frac{1}{x_1\\left(1+x_1\\right)}+\\frac{1}{x_2\\left(1+x_2\\right)} +\\cdots+\\frac{1}{x_n\\left(1+x_n\\right)} \\geqslant \\frac{n}{2}$.", + "solution": "证明: 显然原不等式等价于 $\\frac{1+x_1+x_1^2}{x_1\\left(1+x_1\\right)}+\\frac{1+x_2+x_2^2}{x_2\\left(1+x_2\\right)}+\\cdots+ \\frac{1+x_n+x_n^2}{x_n\\left(1+x_n\\right)} \\geqslant \\frac{3 n}{2}$. 注意到 $4\\left(1+x_i+x_i^2\\right) \\geqslant 3\\left(1+x_i\\right)^2$ 对任意的 $i=1,2, \\cdots$, $n$ 都成立, 因此要证明上式只需证明 $\\frac{3}{4}\\left(\\frac{1+x_1}{x_1}+\\frac{1+x_2}{x_2}+\\cdots+\\frac{1+x_n}{x_n}\\right) \\geqslant \\frac{3 n}{2}$, 即 $\\frac{1}{x_1}+\\frac{1}{x_2}+\\cdots+\\frac{1}{x_n} \\geqslant n \\cdots$ (3). 由 $x_1 x_2 \\cdots x_n=1$ 及均值不等式易知(3) 成立.", + "remark": "", + "figures": [] +} \ No newline at end of file diff --git a/processed_dataset/proof/1185.json b/processed_dataset/proof/1185.json new file mode 100644 index 0000000000000000000000000000000000000000..81ed1580dab85c8a662c215b6e6ce7b188a86e12 --- /dev/null +++ b/processed_dataset/proof/1185.json @@ -0,0 +1,8 @@ +{ + "source_file": "./raw_volume-zh/volume4/exercise1.tex", + "problem_type": "proof", + "problem": "问题10. 设 $a, b, c>0$ 且 $a^2+b^2+c^2+(a+b+c)^2 \\leqslant 4$. 求证:\n$$\n\\frac{a b+1}{(a+b)^2}+\\frac{b c+1}{(b+c)^2}+\\frac{c a+1}{(c+a)^2} \\geqslant 3 .\n$$", + "solution": "证明: 由 $a^2+b^2+c^2+(a+b+c)^2 \\leqslant 4$ 可知 $a^2+b^2+c^2+a b+b c+ c a \\leqslant 2$, 因此 $\\frac{2(a b+1)}{(a+b)^2} \\geqslant \\frac{2 a b+a^2+b^2+c^2+a b+b c+c a}{(a+b)^2}= \\frac{(a+b)^2+(c+a)(c+b)}{(a+b)^2}$ 即 $\\frac{a b+1}{(a+b)^2} \\geqslant \\frac{1}{2}\\left(1+\\frac{(c+a)(c+b)}{(a+b)^2}\\right) \\cdots$ (4). 同理可得 $\\frac{b c+1}{(b+c)^2} \\geqslant \\frac{1}{2}\\left(1+\\frac{(a+b)(a+c)}{(b+c)^2}\\right), \\frac{c a+1}{(c+a)^2} \\geqslant \\frac{1}{2}\\left(1+\\frac{(b+c)(b+a)}{(c+a)^2}\\right) \\cdots$ (5). 另外由均值不等式显然有 $\\frac{(c+a)(c+b)}{(a+b)^2}+\\frac{(a+b)(a+c)}{(b+c)^2}+\\frac{(b+c)(b+a)}{(c+a)^2} \\geqslant 3 \\cdots$ (6). 综合(4)(5)(6)可得 $\\frac{a b+1}{(a+b)^2}+\\frac{b c+1}{(b+c)^2}+\\frac{c a+1}{(c+a)^2} \\geqslant 3$.", + "remark": "", + "figures": [] +} \ No newline at end of file diff --git a/processed_dataset/proof/1186.json b/processed_dataset/proof/1186.json new file mode 100644 index 0000000000000000000000000000000000000000..675f72e745799d9cddee2f8772b4e8da31c58c20 --- /dev/null +++ b/processed_dataset/proof/1186.json @@ -0,0 +1,8 @@ +{ + "source_file": "./raw_volume-zh/volume4/exercise1.tex", + "problem_type": "proof", + "problem": "问题11. 已知 $a, b, c$ 为正实数,且 $a b c=8$, 求证:\n$$\n\\frac{a^2}{\\sqrt{\\left(1+a^3\\right)\\left(1+b^3\\right)}}+\\frac{b^2}{\\sqrt{\\left(1+b^3\\right)\\left(1+c^3\\right)}}+\\frac{c^2}{\\sqrt{\\left(1+c^3\\right)\\left(1+a^3\\right)}} \\geqslant \\frac{4}{3} \\text {. }\n$$", + "solution": "证明:注意到 $\\frac{a^2+2}{2}=\\frac{\\left(a^2-a+1\\right)+(a+1)}{2} \\geqslant \\sqrt{\\left(a^2-a+1\\right) \\cdot(a+1)}= \\sqrt{1+a^3}$. 要证原不等式只需证明 $\\frac{a^2}{\\left(a^2+2\\right)\\left(b^2+2\\right)}+\\frac{b^2}{\\left(b^2+2\\right)\\left(c^2+2\\right)}+ \\frac{c^2}{\\left(c^2+2\\right)\\left(a^2+2\\right)} \\geqslant \\frac{1}{3}$. 而上式等价于 $3 a^2\\left(c^2+2\\right)+3 b^2\\left(a^2+2\\right)+3 c^2\\left(b^2+\\right. 2) \\geqslant\\left(a^2+2\\right)\\left(b^2+2\\right)\\left(c^2+2\\right)$. 即 $\\left(a^2 b^2+b^2 c^2+c^2 a^2\\right)+2\\left(a^2+b^2+c^2\\right) \\geqslant a^2 b^2 c^2+8=64+8=72$. 而 $a^2 b^2+b^2 c^2+c^2 a^2 \\geqslant 3(a b c)^{\\frac{4}{3}}=48, a^2+b^2+ c^2 \\geqslant 3(a b c)^{\\frac{2}{3}}=12$, 则上式显然成立.\n故原不等式得证, 当且仅当 $a=b=c=$ 2 时取等号.", + "remark": "", + "figures": [] +} \ No newline at end of file diff --git a/processed_dataset/proof/1187.json b/processed_dataset/proof/1187.json new file mode 100644 index 0000000000000000000000000000000000000000..422e734e2c7b85c5474b908c71bbea01ca2cde55 --- /dev/null +++ b/processed_dataset/proof/1187.json @@ -0,0 +1,8 @@ +{ + "source_file": "./raw_volume-zh/volume4/exercise1.tex", + "problem_type": "proof", + "problem": "问题12. 设 $a, b \\in \\mathbf{R}, \\frac{1}{a}+\\frac{1}{b}=1$. 求证: 对一切正整数 $n$, 有\n$$\n(a+b)^n-a^n-b^n \\geqslant 2^{2 n}-2^{n+1} .\n$$", + "solution": "$n=1$ 时显然成立.\n假设 $n=k$ 时, 有 $(a+b)^k-a^k-b^k \\geqslant 2^{2 k}-2^{k+1}$.\n则对 $n=k+1$, 由 $\\frac{1}{a}+\\frac{1}{b}=1$, 有 $a+b=a b$, 于是 $a b=a+b \\geqslant 2 \\sqrt{a b}$ 即 $a b=a+b \\geqslant 4$. 从而, $(a+b)^{k+1}-a^{k+1}-b^{k+1}=(a+b)\\left[(a+b)^k-a^k-\\right. \\left.b^k\\right]+a^k b+a b^k \\geqslant 4\\left(2^{2 k}-2^{k+1}\\right)+2 \\sqrt{a^{k+1} b^{k+1}} \\geqslant 2^{2 k+2}-2^{k+3}+2^{k+2}=2^{2(k+1)}- 2^{(k+1)+1}$.", + "remark": "", + "figures": [] +} \ No newline at end of file diff --git a/processed_dataset/proof/1188.json b/processed_dataset/proof/1188.json new file mode 100644 index 0000000000000000000000000000000000000000..9e000f526bbd7fa61ff0ceb234c8506be21739c1 --- /dev/null +++ b/processed_dataset/proof/1188.json @@ -0,0 +1,8 @@ +{ + "source_file": "./raw_volume-zh/volume4/exercise1.tex", + "problem_type": "proof", + "problem": "问题13. 设 $a, b \\in \\mathbf{R}^{+}$, 求证: $\\sqrt{a}+1>\\sqrt{b}$ 成立的充要条件是对任意 $x>1$, 有\n$$\na x+\\frac{x}{x-1}>b \\text {. }\n$$", + "solution": "已知 $a b>0, x-1>0$, 则 $a x+\\frac{x}{x-1}=\\left[a(x-1)+\\frac{1}{x-1}\\right]+a+1 \\geqslant 2 \\sqrt{a}+a+1=(\\sqrt{a}+1)^2$. 当且仅当 $a(x-1)=\\frac{1}{x-1}$, 即 $x=1+\\frac{1}{\\sqrt{a}}$ 时, $a x+\\frac{x}{x-1}$ 的最小值为 $(\\sqrt{a}+1)^2$, 于是 $a x+\\frac{x}{x-1}>b$ 对任意 $x>1$ 成立的.\n充要条件是 $(\\sqrt{a}+1)^2>b$, 即 $\\sqrt{a}+1>\\sqrt{b}$.", + "remark": "", + "figures": [] +} \ No newline at end of file diff --git a/processed_dataset/proof/1189.json b/processed_dataset/proof/1189.json new file mode 100644 index 0000000000000000000000000000000000000000..007e25f65e79180ac8bdc2942ba288b078b06ff1 --- /dev/null +++ b/processed_dataset/proof/1189.json @@ -0,0 +1,8 @@ +{ + "source_file": "./raw_volume-zh/volume4/exercise1.tex", + "problem_type": "proof", + "problem": "问题14. 设 $x_1, x_2 \\in \\mathbf{R}$, 且 $x_1^2+x_2^2 \\leqslant 1$. 求证: 对任意 $y_1, y_2 \\in \\mathbf{R}$, 有 $\\left(x_1 y_1+\\right. \\left.x_2 y_2-1\\right)^2 \\geqslant\\left(x_1^2+x_2^2-1\\right)\\left(y_1^2+y_2^2-1\\right)$.", + "solution": "若 $y_1^2+y_2^2-1 \\geqslant 0$, 不等式显然成立.\n若 $y_1^2+y_2^2-1<0$, 则由平均值不等式, 得 $x_1 y_1 \\leqslant \\frac{x_1^2+y_1^2}{2}, x_2 y_2 \\leqslant \\frac{x_2^2+y_2^2}{2}$. 则 $x_1 y_1+x_2 y_2 \\leqslant \\frac{1}{2}\\left(x_1^2+x_2^2+y_1^2\\right. \\left.+y_2^2\\right) \\leqslant 1$. 因为 $1-x_1 y_1-x_2 y_2 \\geqslant 1-\\frac{x_1^2+y_1^2}{2}-\\frac{x_2^2+y_2^2}{2}=\\frac{1-x_1^2-x_2^2+1-y_1^2-y_2^2}{2} >0$, 所以, $\\left(1-x_1 y_1-x_2 y_2\\right)^2 \\geqslant\\left(\\frac{1-x_1^2-x_2^2+1-y_1^2-y_2^2}{2}\\right)^2 \\geqslant\\left(x_1^2+x_2^2-1\\right)$ $\\left(y_1^2+y_2^2-1\\right)$.", + "remark": "", + "figures": [] +} \ No newline at end of file diff --git a/processed_dataset/proof/1190.json b/processed_dataset/proof/1190.json new file mode 100644 index 0000000000000000000000000000000000000000..92b99d12b609e0bf461fc7cb836a3efe97c86015 --- /dev/null +++ b/processed_dataset/proof/1190.json @@ -0,0 +1,8 @@ +{ + "source_file": "./raw_volume-zh/volume4/exercise1.tex", + "problem_type": "proof", + "problem": "问题15. 设 $a 、 b 、 c$ 为正实数,求证:\n$$\n\\left(1+\\frac{a}{b}\\right)\\left(1+\\frac{b}{c}\\right)\\left(1+\\frac{c}{a}\\right) \\geqslant 2\\left(1+\\frac{a+b+c}{\\sqrt[3]{a b} c}\\right) .\n$$", + "solution": "由于 $\\left(1+\\frac{a}{b}\\right)\\left(1+\\frac{b}{c}\\right)\\left(1+\\frac{c}{a}\\right)=2+\\left(\\frac{a}{c}+\\frac{c}{b}+\\frac{b}{a}\\right)+ \\left(\\frac{a}{b}+\\frac{b}{c}+\\frac{c}{a}\\right)=2+\\left(\\frac{a}{c}+\\frac{a}{b}+\\frac{a}{a}\\right)+\\left(\\frac{b}{a}+\\frac{b}{c}+\\frac{b}{b}\\right)+\\left(\\frac{c}{b}+\\frac{c}{a}+\\frac{c}{c}\\right)- 3 \\geqslant-1+3 \\frac{a+b+c}{\\sqrt[3]{a b c}} \\geqslant 2\\left(1+\\frac{a+b+c}{\\sqrt[3]{a b c}}\\right)$.", + "remark": "", + "figures": [] +} \ No newline at end of file diff --git a/processed_dataset/proof/1191.json b/processed_dataset/proof/1191.json new file mode 100644 index 0000000000000000000000000000000000000000..a81ee718b70ffb0141fa77c5ae1675b9d2c5816b --- /dev/null +++ b/processed_dataset/proof/1191.json @@ -0,0 +1,8 @@ +{ + "source_file": "./raw_volume-zh/volume4/exercise1.tex", + "problem_type": "proof", + "problem": "问题16. 设 $x_1, x_2, x_3 \\in \\mathbf{R}^{+}$, 证明:\n$$\n\\frac{x_2}{x_1}+\\frac{x_3}{x_2}+\\frac{x_1}{x_3} \\leqslant\\left(\\frac{x_1}{x_2}\\right)^2+\\left(\\frac{x_2}{x_3}\\right)^2+\\left(\\frac{x_3}{x_1}\\right)^2 .\n$$", + "solution": "由平均值不等式, 得 $\\frac{x_2}{x_1}=\\frac{x_2}{x_3} \\cdot \\frac{x_3}{x_1} \\leqslant \\frac{1}{2}\\left[\\left(\\frac{x_2}{x_3}\\right)^2+\\left(\\frac{x_3}{x_1}\\right)^2\\right], \\frac{x_3}{x_2}=\\frac{x_3}{x_1}$. $\\frac{x_1}{x_2} \\leqslant \\frac{1}{2}\\left[\\left(\\frac{x_3}{x_1}\\right)^2+\\left(\\frac{x_1}{x_2}\\right)^2\\right], \\frac{x_1}{x_3}=\\frac{x_1}{x_2} \\cdot \\frac{x_2}{x_3} \\leqslant \\frac{1}{2}\\left[\\left(\\frac{x_1}{x_2}\\right)^2+\\left(\\frac{x_2}{x_3}\\right)^2\\right]$. 将它们相加, 便得到命题成立.", + "remark": "", + "figures": [] +} \ No newline at end of file diff --git a/processed_dataset/proof/1192.json b/processed_dataset/proof/1192.json new file mode 100644 index 0000000000000000000000000000000000000000..65297d89648cbbc5d800e7e85be2366cbb10dd89 --- /dev/null +++ b/processed_dataset/proof/1192.json @@ -0,0 +1,8 @@ +{ + "source_file": "./raw_volume-zh/volume4/exercise1.tex", + "problem_type": "proof", + "problem": "问题17. 设 $a 、 b 、 c$ 为正实数,且 $a+b+c=1$. 求证:\n$$\n(1+a)(1+b)(1+c) \\geqslant 8(1-a)(1-b)(1-c) .\n$$", + "solution": "由于 $1+a=2-b-c=1-b+1-c \\geqslant 2 \\sqrt{(1-b)(1-c)}$, 同理可得 $1+b \\geqslant 2 \\sqrt{(1-a)(1-c)}, 1+c \\geqslant 2 \\sqrt{(1-a)(1-b)}$. 将以上三式相乘便可以.", + "remark": "", + "figures": [] +} \ No newline at end of file diff --git a/processed_dataset/proof/1193.json b/processed_dataset/proof/1193.json new file mode 100644 index 0000000000000000000000000000000000000000..e784bfc2e32d1afddfb9364b494d56c59d04cda9 --- /dev/null +++ b/processed_dataset/proof/1193.json @@ -0,0 +1,8 @@ +{ + "source_file": "./raw_volume-zh/volume4/exercise1.tex", + "problem_type": "proof", + "problem": "问题18. 设 $x 、 y 、 z$ 为正实数,且 $x \\geqslant y \\geqslant z$. 求证:\n$$\n\\frac{x^2 y}{z}+\\frac{y^2 z}{x}+\\frac{z^2 x}{y} \\geqslant x^2+y^2+z^2 .\n$$", + "solution": "$\\frac{x^2 y}{z}+\\frac{y^2 z}{x}+\\frac{z^2 x}{y}-x^2-y^2-z^2=\\frac{x^2}{z}(y-z)+\\frac{y^2 z}{x}+\\frac{z^2 x}{y}-y^2-z^2 \\geqslant$\n$$\n\\begin{aligned}\n& \\frac{y^2}{z}(y-z)+2 z \\sqrt{y z}-y^2-z^2=\\frac{y-z}{z}\\left(y^2-y z+z^2-\\frac{2 z^2 \\sqrt{y}}{\\sqrt{y}+\\sqrt{z}}\\right)= \\\\\n& \\frac{(y-z)(\\sqrt{y}-\\sqrt{z})}{z(\\sqrt{y}+\\sqrt{z})}\\left[y(\\sqrt{y}+\\sqrt{z})^2-z^2\\right] \\geqslant 0 .\n\\end{aligned}\n$$", + "remark": "", + "figures": [] +} \ No newline at end of file diff --git a/processed_dataset/proof/1194.json b/processed_dataset/proof/1194.json new file mode 100644 index 0000000000000000000000000000000000000000..f024ebdb4ca8fc4edff54fde66609ad4d7a9af35 --- /dev/null +++ b/processed_dataset/proof/1194.json @@ -0,0 +1,8 @@ +{ + "source_file": "./raw_volume-zh/volume4/exercise1.tex", + "problem_type": "proof", + "problem": "问题19. 设 $a 、 b 、 c$ 为正实数,满足 $a^2+b^2+c^2=1$. 求证:\n$$\n\\frac{a b}{c}+\\frac{b c}{a}+\\frac{c a}{b} \\geqslant \\sqrt{3} \\text {. }\n$$", + "solution": "因 $\\left(\\frac{a b}{c}+\\frac{b c}{a}+\\frac{c a}{b}\\right)^2=\\frac{a^2 b^2}{c^2}+\\frac{b^2 c^2}{a^2}+\\frac{c^2 a^2}{b^2}+2\\left(a^2+b^2+c^2\\right)=$\n$$\n\\frac{1}{2}\\left(\\frac{a^2 b^2}{c^2}+\\frac{c^2 a^2}{b^2}\\right)+\\frac{1}{2}\\left(\\frac{b^2 c^2}{a^2}+\\frac{a^2 b^2}{c^2}\\right)+\\frac{1}{2}\\left(\\frac{c^2 a^2}{b^2}+\\frac{b^2 c^2}{a^2}\\right)+2 \\geqslant a^2+b^2+c^2+2=3.\n$$\n所以, 命题成立.", + "remark": "", + "figures": [] +} \ No newline at end of file diff --git a/processed_dataset/proof/1195.json b/processed_dataset/proof/1195.json new file mode 100644 index 0000000000000000000000000000000000000000..b1c5a9b7b41a5968ae0f21187125defb677486bd --- /dev/null +++ b/processed_dataset/proof/1195.json @@ -0,0 +1,8 @@ +{ + "source_file": "./raw_volume-zh/volume4/exercise1.tex", + "problem_type": "proof", + "problem": "问题20. 设 $a 、 b 、 c 、 d$ 是非负实数,满足 $a b+b c+c d+d a=1$. 求证:\n$$\n\\frac{a^3}{b+c+d}+\\frac{b^3}{a+c+d}+\\frac{c^3}{a+d+b}+\\frac{d^3}{a+b+c} \\geqslant \\frac{1}{3} \\text {. }\n$$", + "solution": "因为 $\\frac{a^3}{b+c+d}+\\frac{b+c+d}{18}+\\frac{1}{12} \\geqslant 3 \\sqrt[3]{\\frac{a^3}{a+c+d} \\cdot \\frac{b+c+d}{18} \\cdot \\frac{1}{12}}= \\frac{a}{2}$, 即 $\\frac{a^3}{b+c+d} \\geqslant \\frac{a}{2}-\\frac{b+c+d}{18}-\\frac{1}{12}$, 所以左 $\\geqslant \\frac{a+b+c+d}{2}-\\frac{1}{18}(3 a+ 3 b+3 c+3 d)-\\frac{4}{12}=\\frac{1}{3}(a+b+c+d)-\\frac{1}{3}$. 又由假设知 $a b+b c+c d+ d a=1$, 即 $(a+c)(b+d)=1$, 所以, $a+b+c+d=a+c+\\frac{1}{a+c} \\geqslant 2$. 故 $\\frac{a^3}{b+c+d}+\\frac{b^3}{a+c+d}+\\frac{c^3}{a+b+d}+\\frac{d^3}{a+b+c} \\geqslant \\frac{1}{3}$.", + "remark": "", + "figures": [] +} \ No newline at end of file diff --git a/processed_dataset/proof/1196.json b/processed_dataset/proof/1196.json new file mode 100644 index 0000000000000000000000000000000000000000..0ea8216ffd5edcad92169c2589451a28d5b1800e --- /dev/null +++ b/processed_dataset/proof/1196.json @@ -0,0 +1,8 @@ +{ + "source_file": "./raw_volume-zh/volume4/exercise2.tex", + "problem_type": "proof", + "problem": "问题2. 设 $a, b, c \\in \\mathbf{R}^{+}$, 且 $a+b+c=3$. 证明:\n$$\n\\sum \\frac{1}{a \\sqrt{2\\left(a^2+b c\\right)}} \\geqslant \\sum \\frac{1}{a+b c},\n$$\n其中, \" $\\sum$ \" 表示轮换对称和.", + "solution": "不失一般性, 令 $a \\geqslant b \\geqslant c$. 则 $\\frac{(c-a)(c-b)}{3(c+a b)} \\geqslant 0$ 及 $\\frac{(a-b)(a-c)}{3(a+b c)}+ \\frac{(b-a)(b-c)}{3(b+a c)}=\\frac{c(a-b)^2}{3}\\left[\\frac{1+a+b-c}{(a+b c)(b+a c)}\\right] \\geqslant 0$. 故 $\\sum \\frac{(a-b)(a-c)}{3(a+b c)} \\geqslant 0$. 而 $\\sum \\frac{1}{a+b c} \\leqslant \\frac{9}{2(a b+b c+a c)} \\Leftrightarrow \\sum \\frac{1}{a(a+b+c)+3 b c} \\leqslant$\n$$\n\\frac{3}{2(a b+b c+a c)} \\Leftrightarrow \\sum\\left[\\frac{1}{2(a b+b c+a c)}-\\frac{1}{a(a+b+c)+3 b c}\\right] \\geqslant 0 \\Leftrightarrow\n$$\n$\\sum \\frac{(a-b)(a-c)}{a(a+b+c)+3 b c}=\\sum \\frac{(a-b)(a-c)}{3(a+b c)} \\geqslant 0$. 由均值不等式知\n$\\frac{1}{a \\sqrt{2\\left(a^2+b c\\right)}}=\\frac{\\sqrt{b+c}}{\\sqrt{2 a} \\cdot \\sqrt{(a b+a c)\\left(a^2+b c\\right)}} \\geqslant \\frac{\\sqrt{2(b+c)}}{\\sqrt{a}(a+c)(a+b)}$. 只需证 $\\sum \\sqrt{\\frac{b+c}{2 a}} \\cdot \\frac{1}{(a+c) \\overline{(a+b)}} \\geqslant \\frac{9}{4(a b+b c+a c)}$. 又 $\\sqrt{\\frac{b+c}{2 a}} \\leqslant\\sqrt{\\frac{a+c}{2 b}} \\leqslant \\sqrt{\\frac{a+b}{2 c}}, \\frac{1}{(a+c)(a+b)} \\leqslant \\frac{1}{(b+c)(a+b)} \\leqslant \\frac{1}{(a+c)(c+b)}$,\n则由切比雪夫不等式知 $\\sum \\sqrt{\\frac{b+c}{2 a}} \\cdot \\frac{1}{(a+c)(a+b)} \\geqslant \\frac{1}{3}\\left(\\sum \\sqrt{\\frac{b+c}{2 a}}\\right)$.\n$\\sum \\frac{1}{(a+c)(a+b)}=\\frac{2}{(a+b)(b+c)(a+c)} \\sum \\sqrt{\\frac{b+c}{2 a}}$. 只需证 $\\sum \\sqrt{\\frac{b+c}{2 a}}\\geqslant \\frac{9(a+b)(b+c)(a+c)}{8(a b+b c+a c)}$. 令 $t=\\sqrt[6]{\\frac{(a+b)(b+c)(a+c)}{8 a b c}} \\geqslant 1$. 则\n$\\frac{9(a+b)(b+c)(a+c)}{8(a b+b c+a c)}=\\frac{27 t^6}{8 t^6+1}$. 由均值不等式知 $\\sum \\sqrt{\\frac{b+c}{2 a}} \\geqslant 3 t$. 故\n$3 t \\geqslant \\frac{27 t^6}{8 t^6+1} \\Leftrightarrow 8 t^6-9 t^5+1 \\geqslant 0$. 而当 $t \\geqslant 1$ 时, 上述不等式恒成立.", + "remark": "", + "figures": [] +} \ No newline at end of file diff --git a/processed_dataset/proof/1197.json b/processed_dataset/proof/1197.json new file mode 100644 index 0000000000000000000000000000000000000000..401d7a968076271b0d1be1413a0371700601714e --- /dev/null +++ b/processed_dataset/proof/1197.json @@ -0,0 +1,8 @@ +{ + "source_file": "./raw_volume-zh/volume4/exercise2.tex", + "problem_type": "proof", + "problem": "问题3. 设 $a_i \\in \\mathbf{R}^{+}(i=1,2, \\cdots, n, n \\geqslant 2)$, 求证:\n$$\n\\frac{a_1}{a_2+a_3}+\\frac{a_2}{a_3+a_4}+\\cdots+\\frac{a_{n-1}}{a_n+a_1}+\\frac{a_n}{a_1+a_2}>\\frac{n}{4} .\n$$", + "solution": "令 $a_{n+1}=a_1, a_{n+2}=a_2$. 又设 $a_{i_1}=\\max _{1 \\leqslant i \\leqslant n} a_i$, 原式左边为 $s, s$ 中分子含 $a_{i_1}$ 的项是 $\\frac{a_{i_1}}{a_{i_1+1}+a_{i_1+2}}$. 令 $a_{i_2}=\\max \\left\\{a_{i_1+1}, a_{i_1+2}\\right\\}$. 仿此取 $a_{i_3}=\\max \\left\\{a_{i_2+1}\\right.$, $\\left.a_{i_2+2}\\right\\}$. 将继续上面手续, 最终将回到 $a_{i_1}$. 即经 $r$ 次后, 有 $a_{i_r+1}=a_{i_1}$. 从取法易知 $r \\geqslant \\frac{n}{2}$. 于是, $s>\\frac{a_{i_1}}{2 a_{i_2}}+\\frac{a_{i_2}}{2 a_{i_3}}+\\cdots+\\frac{a_{i_r}}{2 a_{i_r+1}} \\geqslant \\frac{1}{2} r \\cdot \\sqrt[r]{\\frac{a_{i_1}}{a_{i_2}} \\cdot \\frac{a_{i_2}}{a_{i_3}} \\cdots \\cdots \\frac{a_{i_r}}{a_{i_1}}}= \\frac{1}{2} r \\geqslant \\frac{n}{4}$.", + "remark": "", + "figures": [] +} \ No newline at end of file diff --git a/processed_dataset/proof/1198.json b/processed_dataset/proof/1198.json new file mode 100644 index 0000000000000000000000000000000000000000..0d916ce58b81e1b37e93c88c3bec4741b4f7b449 --- /dev/null +++ b/processed_dataset/proof/1198.json @@ -0,0 +1,8 @@ +{ + "source_file": "./raw_volume-zh/volume4/exercise2.tex", + "problem_type": "proof", + "problem": "问题4. 设 $a_1, a_2, \\cdots, a_n(n \\geqslant 2)$ 是正实数,且满足 $a_1+a_2+\\cdots+a_n<1$. 证明: $\\frac{a_1 a_2 \\cdots a_n\\left[1-\\left(a_1+\\cdots+a_n\\right)\\right]}{\\left(a_1+a_2+\\cdots+a_n\\right)\\left(1-a_1\\right)\\left(1-a_2\\right) \\cdots\\left(1-a_n\\right)} \\leqslant \\frac{1}{n^{n+1}}$.", + "solution": "设 $a_{n+1}=1-\\left(a_1+a_2+\\cdots+a_n\\right)$, 则 $a_{n+1} \\geqslant 0, a_1+\\cdots+a_{n+1}=1$. 则原不等式等价于 $n^{n+1} a_1 a_2 \\cdots a_{n+1} \\leqslant\\left(1-a_1\\right)\\left(1-a_2\\right) \\cdots\\left(1-a_n\\right)\\left(1-a_{n+1}\\right)$. 对 $i=1,2, \\cdots, n+1$, 由平均值不等式, 得 $1-a_i=\\dot{a}_1+\\cdots+a_{i-1}+ a_{i+1}+\\cdots+a_{n+1} \\geqslant n \\sqrt[n]{a_1 \\cdots a_{i-1} a_{i+1} \\cdots a_{n+1}}$. 将 $n+1$ 个不等式相乘, 得 $(1- \\left.a_1\\right)\\left(1-a_2\\right) \\cdots\\left(1-a_{n+1}\\right) \\geqslant n^{n+1} \\sqrt[n]{a_1^n a_2^n \\cdots a_{n+1}^n}=n^{n+1} a_1 a_2 \\cdots a_{n+1}$, 当且仅当 $a_1=a_2=\\cdots=a_{n+1}=\\frac{1}{n+1}$ 时等号成立.", + "remark": "", + "figures": [] +} \ No newline at end of file diff --git a/processed_dataset/proof/1199.json b/processed_dataset/proof/1199.json new file mode 100644 index 0000000000000000000000000000000000000000..eb5d0db1c202ff0621d2a2ce2c76d8bb7771b1c5 --- /dev/null +++ b/processed_dataset/proof/1199.json @@ -0,0 +1,8 @@ +{ + "source_file": "./raw_volume-zh/volume4/exercise2.tex", + "problem_type": "proof", + "problem": "问题5. 设 $a_i, b_i \\in \\mathbf{R}^{+}(i=1,2, \\cdots, n)$, 求证:\n$$\n\\left[\\prod_{i=1}^n\\left(a_i+b_i\\right)\\right]^{\\frac{1}{n}} \\geqslant\\left(\\prod_{i=1}^n a_i\\right)^{\\frac{1}{n}}+\\left(\\prod_{i=1}^n b_i\\right)^{\\frac{1}{n}},\n$$\n其中 $\\prod_{i=1}^n a_i=a_1 a_2 \\cdots a_n$.", + "solution": "右 $=\\left(\\prod \\frac{a_i}{a_i+b_i}\\right)^{\\frac{1}{n}}+\\left(\\prod \\frac{b_i}{a_i+b_i}\\right)^{\\frac{1}{n}} \\leqslant \\frac{1}{n}\\left(\\sum \\frac{a_i}{a_i+b_i}+\\sum \\frac{b_i}{a_i+b_i}\\right)=$ 1. 所以左 $\\geqslant$ 右.", + "remark": "", + "figures": [] +} \ No newline at end of file diff --git a/processed_dataset/proof/1200.json b/processed_dataset/proof/1200.json new file mode 100644 index 0000000000000000000000000000000000000000..b4607ed987b573c9b28c468e150010bcc0c9236a --- /dev/null +++ b/processed_dataset/proof/1200.json @@ -0,0 +1,8 @@ +{ + "source_file": "./raw_volume-zh/volume4/exercise2.tex", + "problem_type": "proof", + "problem": "问题6. 设 $x, y \\in \\mathbf{R}^{+}, x \\neq y$. 令\n$$\nQ=\\sqrt{\\frac{x^2+y^2}{2}}, A=\\frac{x+y}{2}, G=\\sqrt{x y}, H=\\frac{2 x y}{x+y} .\n$$\n证明: $G-H\\sqrt{\\frac{x^2+y^2}{2}}-\\frac{x+y}{2} \\Leftrightarrow x+y>\\sqrt{\\frac{x^2+y^2}{2}}+\\sqrt{x y} \\Leftrightarrow \\\\\n& (x+y)^2>\\frac{x^2+y^2}{2}+x y+\\sqrt{2 x y\\left(x^2+y^2\\right)} \\Leftrightarrow \\frac{1}{2}(x+y)^2>\\sqrt{2 x y\\left(x^2+y^2\\right)} \\Leftrightarrow \\\\\n& (x+y)^4>8 x y\\left(x^2+y^2\\right) \\Leftrightarrow x^4+4 x^3 y+6 x^2 y^2+4 x y^3+y^4 \\geqslant 8 x^3 y+8 x y^3 \\Leftrightarrow\n\\end{aligned}\n$$\n$(x-y)^4>0$. 因为 $x \\neq y$, 所以上面最后一个不等式成立, 即 $A-G>Q-A$.\n下面证明 $Q-A>G-H$. 因为 $\\sqrt{\\frac{x^2+y^2}{2}}-\\frac{x+y}{2}>\\sqrt{x y}-\\frac{2 x y}{x+y} \\Leftrightarrow$\n$$\n\\sqrt{\\frac{x^2+y^2}{2}}-\\sqrt{x y}>\\frac{x+y}{2}-\\frac{2 x y}{x+y} \\Leftrightarrow \\frac{\\frac{1}{2}\\left(x^2+y^2\\right)-x y}{\\sqrt{\\frac{x^2+y^2}{2}}+\\sqrt{x y}}>\\frac{(x-y)^2}{2(x+y)} \\Leftrightarrow x+\n$$\n$y>\\sqrt{\\frac{x^2+y^2}{2}}+\\sqrt{x y}$. 此不等式前面已经证明成立, 所以 $Q-A>G-H$.", + "remark": "", + "figures": [] +} \ No newline at end of file diff --git a/processed_dataset/proof/1201.json b/processed_dataset/proof/1201.json new file mode 100644 index 0000000000000000000000000000000000000000..2bc372004488031a139a054d2fdc57927789e180 --- /dev/null +++ b/processed_dataset/proof/1201.json @@ -0,0 +1,8 @@ +{ + "source_file": "./raw_volume-zh/volume4/exercise2.tex", + "problem_type": "proof", + "problem": "问题7. 设 $a_1, a_2, \\cdots, a_{n+1}$ 为正等差数列(公差 $d \\geqslant 0$ ). 求证:\n$$\nn\\left(\\sqrt[n]{\\frac{a_{n+1}}{a_1}}-1\\right) \\leqslant \\sum_{i=1}^n \\frac{d}{a_i} \\leqslant \\frac{d}{d_1}+(n-1)\\left(1-\\sqrt[n-1]{\\frac{a_1}{a_n}}\\right) .\n$$", + "solution": "$\\sum_{k=1}^n \\frac{d}{a_k}=\\sum \\frac{a_{k+1}-a_k}{a_k}=\\sum \\frac{a_{k+1}}{a_k}-n \\geqslant n\\left(\\sqrt[n]{\\frac{a_2}{a_1} \\cdot \\frac{a_3}{a_2} \\cdot \\cdots \\cdot \\frac{a_{n+1}}{a_n}}-1\\right)=$\n$$\n\\begin{aligned}\n& n\\left(\\sqrt[n]{\\frac{a_{n+1}}{a_1}}-1\\right) . \\text { 又 } \\sum_{k=1}^n \\frac{d}{a_k}=\\frac{d}{a_1}+\\sum_{k=2}^n\\left(1-\\frac{a_{k-1}}{a_k}\\right) \\leqslant \\frac{d}{a_1}+(n-1)-(n-1) \\\\\n& \\sqrt[n-1]{\\frac{a_1}{a_n}}=\\frac{d}{a_1}+(n-1)\\left(1-\\sqrt[n-1]{\\frac{a_1}{a_n}}\\right) .\n\\end{aligned}\n$$", + "remark": "", + "figures": [] +} \ No newline at end of file diff --git a/processed_dataset/proof/1202.json b/processed_dataset/proof/1202.json new file mode 100644 index 0000000000000000000000000000000000000000..d38577848894498d24b682cc58859d0061ebd67c --- /dev/null +++ b/processed_dataset/proof/1202.json @@ -0,0 +1,8 @@ +{ + "source_file": "./raw_volume-zh/volume4/exercise2.tex", + "problem_type": "proof", + "problem": "问题8. 设 $x_1, x_2, \\cdots, x_n$ 为正有理数, 且各不相同.\n求证:\n$$\n\\left(\\frac{x_1^2+x_2^2+\\cdots+x_n^2}{x_1+x_2+\\cdots+x_n}\\right)^{x_1+x_2+\\cdots+x_n}>x_1^{x_1} x_2{ }^{x_2} \\cdots x_n^{x_n} .\n$$", + "solution": "首先设 $x_i \\in \\mathbf{N}$. 因为 $x_i$ 互不相同, 所以左边 $=$\n记 $m$ 为 $x_1, x_2, \\cdots, x_n$ 的各分母的最小公倍数, 则 $m x_1, m x_2, \\cdots, m x_n \\in \\mathbf{Z}^{+}$, 于是 $\\left[\\frac{\\left(m x_1\\right)^2+\\cdots+\\left(m x_n\\right)^2}{m x_1+m x_2+\\cdots+m x_n}\\right]^{m x_1+m x_2+\\cdots+m x_n}>\\left(m x_1\\right)^{m x_1} \\cdot\\left(m x_2\\right)^{m x_2} \\cdots \\cdots \\cdot \\left(m x_n\\right)^{m x_n}$. 两边开 $m$ 次方并除以 $m^{\\left(x_1+\\cdots+x_n\\right)}$, 即得到原不等式.", + "remark": "", + "figures": [] +} \ No newline at end of file diff --git a/processed_dataset/proof/1203.json b/processed_dataset/proof/1203.json new file mode 100644 index 0000000000000000000000000000000000000000..abedf68d448984a3a182ca8a310e048f35039f59 --- /dev/null +++ b/processed_dataset/proof/1203.json @@ -0,0 +1,8 @@ +{ + "source_file": "./raw_volume-zh/volume4/exercise2.tex", + "problem_type": "proof", + "problem": "问题9. 已知 $5 n$ 个实数 $r_i, s_i, t_i, u_i, v_i>1(1 \\leqslant i \\leqslant n)$, 记\n$$\nR=\\frac{1}{n} \\sum_{i=1}^n r_i, S=\\frac{1}{n} \\sum_{i=1}^n s_i, T=\\frac{1}{n} \\sum_{i=1}^n t_i, U=\\frac{1}{n} \\sum_{i=1}^n u_i, V=\\frac{1}{n} \\sum_{i=1}^n v_i .\n$$\n求证: $\\prod_{i=1}^n \\frac{r_i s_i t_i u_i v_i+1}{r_i s_i t_i u_i v_i-1} \\geqslant\\left(\\frac{R S T U V+1}{R S T U V-1}\\right)^n$.", + "solution": "设 $x_i>1(i=1,2, \\cdots, n)$, 则 $\\prod_{i=1}^n\\left(x_i+1\\right)^{\\frac{1}{n}} \\geqslant\\left(\\prod_{i=1}^n x_i\\right)^{\\frac{1}{n}}+1 \\cdots$ (1). $\\left(\\prod_{i=1}^n x_i\\right)^{\\frac{1}{n}} \\geqslant \\prod_{i=1}^n\\left(x_i-1\\right)^{\\frac{1}{n}}+1$, 即 $0<\\left[\\prod_{i=1}^n\\left(x_i-1\\right)\\right]^{\\frac{1}{n}} \\leqslant\\left(\\prod_{i=1}^n x_i\\right)^{\\frac{1}{n}}-1 \\cdots$ (2).\n由(1)、(2), 得 $\\prod_{i=1}^n \\frac{x_i+1}{x_i-1} \\geqslant\\left(\\frac{\\sqrt[n]{x_1 \\cdots x_n}+1}{\\sqrt[n]{x_1 \\cdots x_n}-1}\\right)^n \\cdots$ (3). 又函数 $f(x)=\\frac{x+1}{x-1}= 1+\\frac{2}{x-1}$ 在 $x>1$ 时是减函数.\n令 $x_i=r_i s_i t_i u_i v_i$, 由平均值不等式, 得 $\\left(\\prod_{i=1}^n x_i\\right)^{\\frac{1}{n}}=\\sqrt[n]{r_1 \\cdots r_n} \\cdots \\sqrt[n]{v_1 \\cdots v_n} \\leqslant R S T U V$. 代入(3)式, 得 $\\prod_{i=1}^n \\frac{r_i s_i t_i u_i v_i+1}{r_i s_i t_i u_i v_i-1} \\geqslant \\left(\\frac{R S T U V+1}{R S T U V-1}\\right)^n$", + "remark": "", + "figures": [] +} \ No newline at end of file diff --git a/processed_dataset/proof/1204.json b/processed_dataset/proof/1204.json new file mode 100644 index 0000000000000000000000000000000000000000..61efabf7a07d60513f4b144d0e9d08632d544c1b --- /dev/null +++ b/processed_dataset/proof/1204.json @@ -0,0 +1,8 @@ +{ + "source_file": "./raw_volume-zh/volume4/exercise2.tex", + "problem_type": "proof", + "problem": "问题10. 求证: $\\left(1-\\frac{1}{365}\\right)\\left(1-\\frac{2}{365}\\right) \\cdots\\left(1-\\frac{25}{365}\\right)<\\frac{1}{2}$.", + "solution": "由平均值不等式, 得左边 $\\leqslant\\left\\{\\frac{1}{25}\\left[\\left(1-\\frac{1}{365}\\right)+\\left(1-\\frac{2}{365}\\right)+\\cdots+\\right.\\right. \\left.\\left.\\left(1-\\frac{25}{365}\\right)\\right]\\right\\}^{25}=\\left(1-\\frac{13}{365}\\right)^{25}$. 在 $\\left(1-\\frac{13}{365}\\right)^{25}$ 的二项式展开中, 相邻两项的符号相反, 其绝对值的比为 $\\frac{\\mathrm{C}_{25}^{k+1}\\left(\\frac{13}{365}\\right)^{k+1}}{\\mathrm{C}_{25}^k\\left(\\frac{13}{365}\\right)^k}=\\frac{25+k}{k+1} \\cdot \\frac{13}{365} \\leqslant \\frac{25 \\times 13}{365}<1$.\n即 $\\mathrm{C}_{25}^k\\left(\\frac{13}{365}\\right)^k>\\mathrm{C}_{25}^{k+1}\\left(\\frac{13}{365}\\right)^{k+1}$. 所以 $\\left(1-\\frac{13}{365}\\right)^{25}=1- \\left[\\mathrm{C}_{25}^1\\left(\\frac{13}{365}\\right)-\\mathrm{C}_{25}^2\\left(\\frac{13}{365}\\right)^2\\right]-\\left[\\mathrm{C}_{25}^3\\left(\\frac{13}{365}\\right)^3-\\mathrm{C}_{25}^4\\left(\\frac{13}{365}\\right)^4\\right]-\\cdots-\\left(\\frac{13}{365}\\right)^{25}< 1-\\mathrm{C}_{25}^1 \\cdot \\frac{13}{365}+\\mathrm{C}_{25}^2\\left(\\frac{13}{365}\\right)^2=1-\\frac{65}{73}+\\frac{169 \\times 12}{75^2}=\\frac{2622}{5329}<\\frac{1}{2}$.", + "remark": "", + "figures": [] +} \ No newline at end of file diff --git a/processed_dataset/proof/1205.json b/processed_dataset/proof/1205.json new file mode 100644 index 0000000000000000000000000000000000000000..b598f17a964b3b96661f43d8459191f28c55ec98 --- /dev/null +++ b/processed_dataset/proof/1205.json @@ -0,0 +1,8 @@ +{ + "source_file": "./raw_volume-zh/volume4/exercise2.tex", + "problem_type": "proof", + "problem": "问题14. 设 $a_1, a_2, a_3 \\in \\mathbf{R}^{+}$, 求\n$$\n\\frac{a_1 a_2}{\\left(a_2+a_3\\right)\\left(a_3+a_1\\right)}+\\frac{a_2 a_3}{\\left(a_3+a_1\\right)\\left(a_1+a_2\\right)}+\\frac{a_3 a_1}{\\left(a_1+a_2\\right)\\left(a_2+a_3\\right)}\n$$\n的最小值.", + "solution": "所求的最小值为 $\\frac{3}{4}$. 当 $a_1=a_2=a_3$ 时, 其值为 $\\frac{3}{4}$. 下面证明:\n$$\n\\begin{aligned}\n& \\frac{a_1 a_2}{\\left(a_2+a_3\\right)\\left(a_3+a_1\\right)}+\\frac{a_2 a_3}{\\left(a_3+a_1\\right)\\left(a_1+a_2\\right)}+\\frac{a_3 a_1}{\\left(a_3+a_2\\right)\\left(a_2+a_3\\right)} \\geqslant \\\\\n& \\frac{3}{4} \\Leftrightarrow 4\\left[a_1 a_2\\left(a_1+a_2\\right)+a_2 a_3\\left(a_2+a_3\\right)+a_3 a_1\\left(a_3+a_1\\right)\\right] \\geqslant 3\\left(a_1+a_2\\right)\\left(a_2+\\right. \\\\\n& \\left.a_3\\right)\\left(a_3+a_1\\right) \\Leftrightarrow 4\\left[a_1\\left(a_2^2+a_3^2\\right)+a_2\\left(a_3^2+a_1^2\\right)+a_3\\left(a_1^2+a_2^2\\right)\\right] \\geqslant 3\\left[a_1\\left(a_2^2+a_3^2\\right)+\\right. \\\\\n& \\left.a_2\\left(a_3^2+a_1^2\\right)+a_3\\left(a_1^2+a_2^2\\right)+2 a_1 a_2 a_3\\right] \\Leftrightarrow a_1\\left(a_2^2+a_3^2\\right)+a_2\\left(a_3^2+a_1^2\\right)+a_3\\left(a_1^2+\\right. \\\\\n& \\left.a_2^2\\right) \\geqslant 6 a_1 a_2 a_3. \\circledast\n\\end{aligned}\n$$\n由平均值不等式, 得 $\\circledast$ 的左边 $\\geqslant a_1\\left(2 a_2 a_3\\right)+a_2\\left(2 a_1 a_3\\right)+ a_3\\left(2 a_1 a_2\\right)=6 a_1 a_2 a_3$. 故 $\\circledast$ 成立, 最小值为 $\\frac{3}{4}$.", + "remark": "", + "figures": [] +} \ No newline at end of file diff --git a/processed_dataset/proof/1206.json b/processed_dataset/proof/1206.json new file mode 100644 index 0000000000000000000000000000000000000000..8c3ea80a461e31602d9064310e106a25e56e8ebd --- /dev/null +++ b/processed_dataset/proof/1206.json @@ -0,0 +1,10 @@ +{ + "source_file": "./raw_volume-zh/volume4/exercise2.tex", + "problem_type": "proof", + "problem": "问题18. 证明: 对任意边长为 $a 、 b 、 c$, 且面积为 $S$ 的三角形, 有\n$$\n\\frac{a b+b c+c a}{4 S} \\geqslant \\sqrt{3} \\text {. }\n$$", + "solution": "如图()所示, 在 $\\triangle A B C$ 中, 设 $A B=c, A C=b$, $B C=a$, 且 $\\angle B A C=\\alpha$, 考虑 $\\triangle A B C$ 的外接圆周上边 $B C$ 的对弧 $\\overparen{B A C}$, 因为弧的中点 $D$ 是弧上离弦 $B C$ 最远的点, 所以对 $\\triangle A B C$ 与 $\\triangle D B C$ 的高 $A H=h$ 与 $D K$, 有 $h \\leqslant D K=B K \\cdot \\cot \\frac{\\angle B D C}{2}=\\frac{a}{2} \\cot \\frac{\\alpha}{2}$. 由平均值不等式, 得\n$\\frac{a b+a c+b c}{4 S} \\geqslant \\frac{3}{4 S} \\sqrt[3]{a^2 b^2 c^2}=\\frac{3}{4} \\sqrt[3]{\\frac{a^2 b^2 c^2}{\\left(\\frac{1}{2} b c \\sin \\alpha\\right)^2 \\cdot \\frac{1}{2} a h}}=\\frac{3}{2} \\sqrt[3]{\\frac{a}{h \\sin ^2 \\alpha}} \\geqslant \\frac{3}{2} \\sqrt[3]{\\frac{2}{\\sin ^2 \\alpha \\cot \\frac{\\alpha}{2}}}$. 令 $\\cos \\alpha=x$, 再由平均值不等式, 得 $\\frac{1}{2} \\sin ^2 \\alpha \\cot \\frac{\\alpha}{2}=\\sin \\alpha \\cos ^2 \\frac{\\alpha}{2}=\\frac{1}{2} \\sin \\alpha(1+\\cos \\alpha)=\\frac{1}{2} \\sqrt{1-x^2}(1+x)=\\frac{1}{2} \\sqrt{(1+x)^3(1-x)}=\\frac{1}{2} \\sqrt{27\\left(\\frac{1+x}{3}\\right)^3(1-x)} \\leqslant \\frac{1}{2} \\sqrt{27}\\left[\\frac{1}{4}\\left(3 \\cdot \\frac{1+x}{3}+(1-x)\\right)\\right]^2 =\\frac{1}{2} \\sqrt{27}\\left(\\frac{2}{4}\\right)^2=\\left(\\frac{\\sqrt{3}}{2}\\right)^3$. 从而 $\\frac{a b+b c+c a}{4 S} \\geqslant \\frac{3}{2} \\cdot-\\frac{2}{\\sqrt{3}}=\\sqrt{3}$.", + "remark": "", + "figures": [ + "./images/volume4/figures/fig-c2p18.png" + ] +} \ No newline at end of file diff --git a/processed_dataset/proof/1207.json b/processed_dataset/proof/1207.json new file mode 100644 index 0000000000000000000000000000000000000000..ca414a85647e90834ac772cf8b8972805becb9e2 --- /dev/null +++ b/processed_dataset/proof/1207.json @@ -0,0 +1,8 @@ +{ + "source_file": "./raw_volume-zh/volume4/exercise2.tex", + "problem_type": "proof", + "problem": "问题19. 证明: 如果 $A D 、 B E$ 与 $C F$ 是 $\\triangle A B C$ 的角平分线, 则 $\\triangle D E F$ 的面积不超过 $\\triangle A B C$ 的面积的四分之一.", + "solution": "记 $a=B C, b=A C, c=A B, S=S_{\\triangle A B C}, S_0=S_{\\triangle D E F}$. 由三角形平分线的性质, 得 $\\frac{A F}{b}=\\frac{B F}{a}=\\frac{A F+B F}{b+a}=\\frac{c}{a+b}$. 从而 $A F=\\frac{b c}{a+b}$, 同理可得 $A E=\\frac{b c}{a+c}$. 因此, $S_{\\triangle A E F}=\\frac{1}{2} A F \\cdot A E \\sin \\angle B A C=\\frac{1}{2} b c \\sin \\angle B A C \\cdot\\frac{b c}{(a+b)(a+c)}=\\frac{b c S}{(a+b)(a+c)}$. 同理可得 $S_{\\triangle B D F}=\\frac{a c S}{(a+b)(b+c)}, S_{\\triangle C D E}= \\frac{a b S}{(a+c)(b+c)}$. 由平均值不等式, 得 $S-S_0=S_{\\triangle A E F}+S_{\\triangle B D F}+S_{\\triangle C D E}= \\left[\\frac{b c}{(a+b)(a+c)}+\\frac{a c}{(b+a)(b+c)}+\\frac{a b}{(c+a)(c+b)}\\right] S \\geqslant \\frac{6 a b c S}{(a+b)(b+c)(c+a)}= 3\\left[1-\\frac{b c}{(a+b)(a+c)}-\\frac{a c}{(a+b)(c+b)}-\\frac{a b}{(a+c)(c+b)}\\right] S=3\\left(S-S_{\\triangle A E F}-\\right. \\left.S_{\\triangle B D F}-S_{\\triangle C D E}\\right)=3 S_0$. 于是, $S_0 \\leqslant \\frac{1}{4} S$, 即命题成立.", + "remark": "", + "figures": [] +} \ No newline at end of file diff --git a/processed_dataset/proof/1208.json b/processed_dataset/proof/1208.json new file mode 100644 index 0000000000000000000000000000000000000000..7c16efa1daa36596d6e206a0fda7f610efc957cf --- /dev/null +++ b/processed_dataset/proof/1208.json @@ -0,0 +1,8 @@ +{ + "source_file": "./raw_volume-zh/volume4/exercise2.tex", + "problem_type": "proof", + "problem": "问题20. 设 $\\triangle A B C$ 的外接圆 $K$ 的半径为 $R$, 内角平分线分别交圆 $K$ 于 $A^{\\prime}, B^{\\prime}$, $C^{\\prime}$, 证明: $16 Q^3 \\geqslant 27 R^4 P$. 其中, $Q 、 P$ 分别为 $\\triangle A^{\\prime} B^{\\prime} C^{\\prime}$ 与 $\\triangle A B C$ 的面积.", + "solution": "设 $\\triangle A B C$ 的三个内角为 $\\alpha 、 \\beta 、 \\gamma$, 则 $P=\\frac{1}{2} R^2(\\sin 2 \\alpha+\\sin 2 \\beta+ \\sin 2 \\gamma)$. 由于 $\\triangle A^{\\prime} B^{\\prime} C^{\\prime}$ 的内角为 $\\frac{\\beta+\\gamma}{2}, \\frac{\\alpha+\\gamma}{2}, \\frac{\\alpha+\\beta}{2}$, 所以 $Q=\\frac{1}{2} R^2[\\sin (\\beta+ \\gamma)+\\sin (\\alpha+\\gamma)+\\sin (\\alpha+\\beta)]$. 由平均值不等式, 得 $16 Q^3=2 R^6[\\sin (\\beta+\\gamma)+ \\sin (\\alpha+\\gamma)+\\sin (\\alpha+\\beta)]^3 \\geqslant 2 R^6 \\cdot 27 \\sin (\\beta+\\gamma) \\sin (\\alpha+\\gamma) \\sin (\\alpha+\\beta)= 27 R^6[\\cos (\\alpha-\\beta)+\\cos \\gamma] \\sin (\\alpha+\\beta)=\\frac{27}{2} R^6[\\sin (\\alpha+\\beta+\\gamma)+\\sin (\\alpha+\\beta-\\gamma)+ \\sin 2 \\alpha+\\sin 2 \\beta]=\\frac{27}{2} R^6(\\sin 2 \\alpha+\\sin 2 \\beta+\\sin 2 \\gamma)=27 R^4 P$.", + "remark": "", + "figures": [] +} \ No newline at end of file diff --git a/processed_dataset/proof/1209.json b/processed_dataset/proof/1209.json new file mode 100644 index 0000000000000000000000000000000000000000..6faa08ca93a1592fe97358ecd17bb4d6f56d92c5 --- /dev/null +++ b/processed_dataset/proof/1209.json @@ -0,0 +1,10 @@ +{ + "source_file": "./raw_volume-zh/volume4/exercise2.tex", + "problem_type": "proof", + "problem": "问题21. 设 $\\triangle A B C$ 的三边长为 $a 、 b 、 c$, 现将 $A B 、 A C$ 分别延长 $a$ 单位长度, 将 $B C 、 B A$ 分别延长 $b$ 单位长度.\n$C A, C B$ 分别延长 $c$ 单位长度.\n设这样得到六个端点所构成的凸多边形面积为 $G, \\triangle A B C$ 的面积为 $F$. 证明:\n$$\n\\frac{G}{F} \\geqslant 13 \\text {. }\n$$", + "solution": "如图()所示, 知 $S_{\\triangle A B_2 C_1}=S_{\\triangle B C_2 A_1}=S_{\\triangle C A_2 B_1}=S_{\\triangle A B C}$\n所以 $$\n\\begin{aligned}\n& \\frac{G}{F} \\\\\n& = \\frac{S_{A B C_2 C_1}+S_{B C A_2 A_2}+S_{A C B_1 B_2}+4 F}{F} \\\\\n& =\\frac{S_{\\triangle A A_1 A_2}+S_{\\triangle B B_1 B_2}+S_{\\triangle C C_1 C_2}+F}{F} \\\\\n& =1+ \\frac{(b+a)(c+a)}{b c}+\\frac{(a+b)(c+b)}{a c}+\\frac{(a+c)(b+c)}{a b}\\\\\n& =1+3+\\frac{a}{b}+\\frac{a}{c}+\\frac{b}{a}+\\frac{b}{c}+\\frac{c}{a}+\\frac{c}{b}+\\frac{a^2}{b c}+\\frac{b^2}{a c}+\\frac{c^2}{a b} \\geqslant 4+9 \\sqrt[9]{\\frac{a}{b} \\cdot \\frac{a}{c} \\cdot \\frac{b}{a} \\cdot \\frac{b}{c} \\cdot \\frac{c}{a} \\cdot \\frac{c}{b} \\cdot \\frac{a^2}{b c} \\cdot \\frac{b^2}{a c} \\cdot \\frac{c^2}{a b}} \\\\\n& =13\n\\end{aligned}\n$$", + "remark": "", + "figures": [ + "./images/volume4/figures/fig-c2p21.png" + ] +} \ No newline at end of file diff --git a/processed_dataset/proof/1210.json b/processed_dataset/proof/1210.json new file mode 100644 index 0000000000000000000000000000000000000000..955b4db59e7919b7c890c421cef2b974cf1f3db5 --- /dev/null +++ b/processed_dataset/proof/1210.json @@ -0,0 +1,8 @@ +{ + "source_file": "./raw_volume-zh/volume4/exercise2.tex", + "problem_type": "proof", + "problem": "问题24. 在 $\\triangle A B C$ 中, 三条边长分别为 $a 、 b 、 c$, 且 $a 、 b 、 c$ 为有理数, 求证:\n$$\n\\left(1+\\frac{b-c}{a}\\right)^a\\left(1+\\frac{c-a}{b}\\right)^b\\left(1+\\frac{a-b}{c}\\right)^c \\leqslant 1 .\n$$", + "solution": "因为 $a 、 b 、 c$ 为正有理数,故存在 $m \\in \\mathbf{N}$, 使 $m a, m b, m c$ 为正整数, 又 $a 、 b 、 c$ 为三边之长, 有 $1+\\frac{b-c}{a}>0,1+\\frac{c-a}{b}>0,1+\\frac{a-b}{c}>0$. 由平均值不等式, 得 $\\left[\\left(1+\\frac{b-c}{a}\\right)^{m a}\\left(1+\\frac{c-a}{b}\\right)^{m b}\\left(1+\\frac{a-b}{c}\\right)^{m c}\\right]^{\\frac{1}{m a+m b+m c}} \\leqslant \\frac{m a\\left(1+\\frac{b-c}{a}\\right)+m b\\left(1+\\frac{c-a}{b}\\right)+m c\\left(1+\\frac{a-b}{c}\\right)}{m a+m b+m c}=1$.", + "remark": "", + "figures": [] +} \ No newline at end of file diff --git a/processed_dataset/proof/1211.json b/processed_dataset/proof/1211.json new file mode 100644 index 0000000000000000000000000000000000000000..bafe068b8fa2813b7d4c347545627a047d98483e --- /dev/null +++ b/processed_dataset/proof/1211.json @@ -0,0 +1,8 @@ +{ + "source_file": "./raw_volume-zh/volume4/exercise2.tex", + "problem_type": "proof", + "problem": "问题27. 对每个正整数 $n$, 求证:\n$$\n\\sum_{j=1}^n \\frac{2 j+1}{j^2}>n\\left[(n+1)^{\\frac{2}{n}}-1\\right] .\n$$", + "solution": "显然 $2 j+1=(j+1)^2-j^2$, 由平均值不等式, 得 $\\sum_{j=1}^n \\frac{2 j+1}{j^2}= \\sum_{j=1}^n\\left[\\frac{(j+1)^2}{j^2}-1\\right]=\\sum_{j=1}^n \\frac{(j+1)^2}{j^2}-n \\geqslant n\\left[\\frac{2^2}{1^2} \\cdot \\frac{3^2}{2^2} \\cdot \\cdots \\cdot \\frac{(n+1)^2}{n^2}\\right]^{\\frac{1}{n}}-n= n\\left[(n+1)^{\\frac{2}{n}}-1\\right]$.", + "remark": "", + "figures": [] +} \ No newline at end of file diff --git a/processed_dataset/proof/1212.json b/processed_dataset/proof/1212.json new file mode 100644 index 0000000000000000000000000000000000000000..c81cb192d4d3fd25a57d4b98019396714667c10c --- /dev/null +++ b/processed_dataset/proof/1212.json @@ -0,0 +1,8 @@ +{ + "source_file": "./raw_volume-zh/volume4/exercise2.tex", + "problem_type": "proof", + "problem": "问题28. 设 $A 、 B 、 C$ 为三角形的三个内角, 求证:\n$$\n\\sin 3 A+\\sin 3 B+\\sin 3 C \\leqslant \\frac{3}{2} \\sqrt{3} .\n$$", + "solution": "不妨设 $A \\geqslant 60^{\\circ}$, 则 $B+C \\leqslant 180^{\\circ}-60^{\\circ}=120^{\\circ}$. $\\sin 3 A+\\sin 3 B+ \\sin 3 C=\\sin 3 A+2 \\sin \\frac{3}{2}(B+C) \\cos \\frac{3}{2}(B-C) \\leqslant \\sin 3 A+2 \\sin \\frac{3}{2}(B+C)$. 记 $\\alpha= \\frac{3}{2}(B+C)$, 则 $0 \\leqslant \\alpha \\leqslant 180^{\\circ}$, 且 $A=180^{\\circ}-(B+C)=180^{\\circ}-\\frac{2}{3} \\alpha$. 于是 $\\sin 3 A+ \\sin 3 B+\\sin 3 C \\leqslant \\sin \\left(3 \\times 180^{\\circ}-2 \\alpha\\right)+2 \\sin \\alpha=\\sin 2 \\alpha+2 \\sin \\alpha=2 \\sin \\alpha(1+ \\cos \\alpha)=8 \\sin \\frac{\\alpha}{2} \\cos ^3 \\frac{\\alpha}{2}$. 由平均值不等式, 得 $\\sin \\frac{\\alpha}{2} \\cos ^3 \\frac{\\alpha}{2}=\\sqrt{\\sin ^2 \\frac{\\alpha}{2} \\cos ^6 \\frac{\\alpha}{2}}= \\sqrt{\\frac{-1}{3} \\cdot 3 \\sin ^2 \\frac{\\alpha}{2} \\cos ^6 \\frac{\\alpha}{2}} \\leqslant \\sqrt{\\frac{1}{3}\\left[\\frac{3 \\sin ^2 \\frac{\\alpha}{2}+\\cos ^2 \\frac{\\alpha}{2}+\\cos ^2 \\frac{\\alpha}{2}+\\cos ^2 \\frac{\\alpha}{2}}{4}\\right]^4} \\leqslant \\frac{3 \\sqrt{3}}{16}$. 所以 $\\sin 3 A+\\sin 3 B+\\sin 3 C \\leqslant \\frac{3}{2} \\sqrt{3}$, 且当且仅当 $A=140^{\\circ}, B=C=20^{\\circ}$ 时, 等号成立.", + "remark": "", + "figures": [] +} \ No newline at end of file diff --git a/processed_dataset/proof/1213.json b/processed_dataset/proof/1213.json new file mode 100644 index 0000000000000000000000000000000000000000..46765b36446cdbde8984eb033515f179cc538238 --- /dev/null +++ b/processed_dataset/proof/1213.json @@ -0,0 +1,8 @@ +{ + "source_file": "./raw_volume-zh/volume4/exercise2.tex", + "problem_type": "proof", + "problem": "问题29. 设 $\\alpha 、 \\beta 、 \\gamma$ 为一个给定三角形的三个内角,求证:\n$$\n\\csc ^2 \\frac{\\alpha}{2}+\\csc ^2 \\frac{\\beta}{2}+\\csc ^2 \\frac{\\gamma}{2} \\geqslant 12,\n$$\n并求等号成立的条件.", + "solution": "由平均值不等式, 得 $\\csc ^2 \\frac{\\alpha}{2}+\\csc ^2 \\frac{\\beta}{2}+\\csc ^2 \\frac{\\gamma}{2} \\geqslant 3\\left(\\csc \\frac{\\alpha}{2} \\csc \\frac{\\beta}{2} \\csc \\frac{\\gamma}{2}\\right)^{\\frac{2}{3}}$, 当且仅当 $\\alpha=\\beta=\\gamma$ 时等号成立.\n再由平均值不等式及凸函数性质, 得 $\\left(\\sin \\frac{\\alpha}{2} \\sin \\frac{\\beta}{2} \\sin \\frac{\\gamma}{2}\\right)^{\\frac{1}{3}} \\leqslant \\frac{\\sin \\frac{\\alpha}{2}+\\sin \\frac{\\beta}{2}+\\sin \\frac{\\gamma}{2}}{3} \\leqslant \\sin \\frac{\\frac{\\alpha}{2}+\\frac{\\beta}{2}+\\frac{\\gamma}{2}}{3}=\\sin \\frac{\\pi}{6}=\\frac{1}{2}$, 因此 $\\csc ^2 \\frac{\\alpha}{2}+\\csc ^2 \\frac{\\beta}{2}+\\csc ^2 \\frac{\\gamma}{2} \\geqslant 3\\left(\\sin \\frac{\\alpha}{2} \\sin \\frac{\\beta}{2} \\sin \\frac{\\gamma}{2}\\right)^{-\\frac{2}{3}} \\geqslant 3\\left(\\frac{1}{2}\\right)^{-2}=12$. 当且仅当 $\\alpha=\\beta=\\gamma$ 时, 等号成立.", + "remark": "", + "figures": [] +} \ No newline at end of file diff --git a/processed_dataset/proof/1214.json b/processed_dataset/proof/1214.json new file mode 100644 index 0000000000000000000000000000000000000000..c2da238005d12224a1abeb34017447b908e1531c --- /dev/null +++ b/processed_dataset/proof/1214.json @@ -0,0 +1,8 @@ +{ + "source_file": "./raw_volume-zh/volume4/exercise2.tex", + "problem_type": "proof", + "problem": "问题30. 设 $x, y, z \\geqslant 0$, 且满足 $y z+z x+x y=1$, 求证:\n$$\nx\\left(1-y^2\\right)\\left(1-z^2\\right)+y\\left(1-z^2\\right)\\left(1-x^2\\right)+z\\left(1-x^2\\right)\\left(1-y^2\\right) \\leqslant \\frac{4}{9} \\sqrt{3} .\n$$", + "solution": "令 $x=\\tan \\frac{A}{2}, y=\\tan \\frac{B}{2}, z=\\tan \\frac{C}{2}$. 这里 $A, B, C \\in[0, \\pi)$. 由于 $\\tan \\left(\\frac{A}{2}+\\frac{B}{2}+\\frac{C}{2}\\right)=\\frac{\\tan \\frac{A}{2}+\\tan \\frac{B}{2}+\\tan \\frac{C}{2}-\\tan \\frac{A}{2} \\tan \\frac{B}{2} \\tan \\frac{C}{2}}{1-\\tan \\frac{A}{2} \\tan \\frac{B}{2}-\\tan \\frac{A}{2} \\tan \\frac{C}{2}-\\tan \\frac{B}{2} \\tan \\frac{C}{2}}$, 所以 $\\cot \\left(\\frac{A}{2}+\\frac{B}{2}+\\frac{C}{2}\\right)=\\frac{1-(x y+y z+z x)}{x+y+z-x y z}$. 由已知条件知 $y 、 z$ 不全为 0 且 $0 \\leqslant y z \\leqslant 1$, 从而 $x+y+z>x \\geqslant x y z \\geqslant 0$, 于是 $\\cot \\left(\\frac{A}{2}+\\frac{B}{2}+\\frac{C}{2}\\right)=0$. 所以 $0<\\frac{1}{2}(A+B+C)<\\frac{3 \\pi}{2}$, 得 $\\frac{1}{2}(A+B+C)=\\frac{\\pi}{2}$, 即 $A+B+C=\\pi$. 又 $x\\left(1-y^2\\right)\\left(1-z^2\\right)+y\\left(1-z^2\\right)\\left(1-x^2\\right)+z\\left(1-x^2\\right)\\left(1-y^2\\right)= =4 x y z$, 由平均值不等式, 得 $(x y z)^2 \\leqslant\\left(\\frac{x y+y z+z x}{3}\\right)^3=\\frac{1}{27}$, 从而 $x y z \\leqslant \\frac{1}{9} \\sqrt{3}$.", + "remark": "", + "figures": [] +} \ No newline at end of file diff --git a/processed_dataset/proof/1215.json b/processed_dataset/proof/1215.json new file mode 100644 index 0000000000000000000000000000000000000000..d5271680e9dcab6ca0e4ceac459f23115648b1a0 --- /dev/null +++ b/processed_dataset/proof/1215.json @@ -0,0 +1,8 @@ +{ + "source_file": "./raw_volume-zh/volume4/exercise2.tex", + "problem_type": "proof", + "problem": "问题31. 对 $a_i \\in \\mathbf{R}^{+}(i=1,2, \\cdots, n)$, 求证: $\\sum_{k=1}^n \\sqrt[k]{a_1 \\cdots a_k} \\leqslant \\mathrm{e} \\sum_{k=1}^n a_k$, 其中 $\\mathrm{e}= \\lim _{n \\rightarrow \\infty}\\left(1+\\frac{1}{n}\\right)^n$.", + "solution": "已知 $\\left(1+\\frac{1}{k}\\right)^k$ 单调增加且收玫于 e. 对任意 $i \\in \\mathbf{N}$, 有 $i\\left(1+\\frac{1}{i}\\right)^i \\leqslant i \\mathrm{e}$, 记 $b_i=i\\left(1+\\frac{1}{i}\\right)^i$. 则 $\\frac{b_i}{i} \\leqslant \\mathrm{e}$. 由 $b_1 b_2 \\cdots b_k=(1+k)^k$, 得 $\\sqrt[k]{a_1 \\cdots a_k}= \\frac{1}{1+k} \\sqrt[k]{\\left(a_1 b_1\\right) \\cdots\\left(a_k b_k\\right)}$. 由平均值不等式, 得 $\\sqrt[k]{a_1 \\cdots a_k} \\leqslant \\frac{1}{k(k+1)} \\sum_{i=1}^k a_i b_i= \\left(\\frac{1}{k}-\\frac{1}{k+1}\\right) \\sum_{i=1}^k a_i b_i, \\quad \\sum_{k=1}^n \\sqrt[k]{a_1 \\cdots a_k} \\leqslant \\sum_{k=1}^n\\left(\\frac{1}{k}-\\frac{1}{k+1}\\right) \\sum_{i=1}^k a_i b_i=$\n$$\n\\sum_{i=1}^n a_i b_i \\sum_{k=1}^i\\left(\\frac{1}{k}-\\frac{1}{k+1}\\right)=\\sum_{i=1}^n\\left(\\frac{1}{i}-\\frac{1}{n+1}\\right) b_i a_i<\\sum_{i=1}^n \\frac{b_i}{i} a_i<\\mathrm{e} \\sum_{i=1}^n a_i .\n$$", + "remark": "", + "figures": [] +} \ No newline at end of file diff --git a/processed_dataset/proof/1216.json b/processed_dataset/proof/1216.json new file mode 100644 index 0000000000000000000000000000000000000000..43b87981ac7bf16256938d1f80e2189ece3c339f --- /dev/null +++ b/processed_dataset/proof/1216.json @@ -0,0 +1,8 @@ +{ + "source_file": "./raw_volume-zh/volume4/exercise2.tex", + "problem_type": "proof", + "problem": "问题32. 设 $x_i \\in \\mathbf{R}(i=1,2, \\cdots, n, n \\geqslant 3)$. 令 $p=\\sum_{i=1}^n x_i, q=\\sum_{1 \\leqslant i0$ 且 $a+b+c+d=1$, 求证:\n$$\n\\frac{1}{4 a+3 b+c}+\\frac{1}{3 a+b+4 d}+\\frac{1}{a+4 c+3 d}+\\frac{1}{4 b+3 c+d} \\geqslant 2 .\n$$", + "solution": "由柯西不等式, 可得原式左边 $\\geqslant \\overline{(4 a+3 b+c)+(3 a+b+4 d)+(a+4 c+3 d)+(4 b+3 c+d)}=2$.", + "remark": "", + "figures": [] +} \ No newline at end of file diff --git a/processed_dataset/proof/1219.json b/processed_dataset/proof/1219.json new file mode 100644 index 0000000000000000000000000000000000000000..4997434be52ad4cd0532d6fbc9498d6c3c5c4acb --- /dev/null +++ b/processed_dataset/proof/1219.json @@ -0,0 +1,8 @@ +{ + "source_file": "./raw_volume-zh/volume4/exercise3.tex", + "problem_type": "proof", + "problem": "问题3. 已知 $a, b \\in \\mathbf{R}^{+}, n \\geqslant 2, n \\in \\mathbf{N}^{+}$. 求证:\n$$\n\\sum_{i=1}^n \\frac{1}{a+i b}<\\frac{n}{\\sqrt{a(a+n b)}} .\n$$", + "solution": "由柯西不等式, 有 $\\left(\\sum_{i=1}^n \\frac{1}{a+i b}\\right)^2 \\leqslant n \\sum_{i=1}^n\\left(\\frac{1}{a+i b}\\right)^2<$\n$$\n\\begin{aligned}\n& n\\left\\{\\frac{1}{a(a+b)}+\\frac{1}{(a+b)(a+2 b)}+\\cdots+\\frac{1}{[a+(n-1) b](a+n b)}\\right\\}= \\\\\n& \\frac{n}{b}\\left[\\left(\\frac{1}{a}-\\frac{1}{a+b}\\right)+\\left(\\frac{1}{a+b}-\\frac{1}{a+2 b}\\right)+\\cdots+\\left(\\frac{1}{a+(n-1) b}-\\frac{1}{a+n b}\\right)\\right]= \\\\\n& \\frac{n}{b}\\left(\\frac{1}{a}-\\frac{1}{a+n b}\\right)=\\frac{n^2}{a(a+n b)}, \\text { 即 } \\sum_{i=1}^n \\frac{1}{a+i b}<\\frac{n}{\\sqrt{a(a+n b)}} .\n\\end{aligned}\n$$", + "remark": "", + "figures": [] +} \ No newline at end of file diff --git a/processed_dataset/proof/1220.json b/processed_dataset/proof/1220.json new file mode 100644 index 0000000000000000000000000000000000000000..7ac9d2c608553fbb88fe624009b402f8cde5a517 --- /dev/null +++ b/processed_dataset/proof/1220.json @@ -0,0 +1,8 @@ +{ + "source_file": "./raw_volume-zh/volume4/exercise3.tex", + "problem_type": "proof", + "problem": "问题4. 已知 $a, b, c \\in \\mathbf{R}^{+}$, 且 $a+b+c=1$. 求证:\n$$\n\\frac{1}{1-a}+\\frac{1}{1-b}+\\frac{1}{1-c} \\geqslant \\frac{2}{1+a}+\\frac{2}{1+b}+\\frac{2}{1+c} \\text {. }\n$$", + "solution": "由柯西不等式, 当 $x, y>0$ 时, 有 $(x+y)\\left(\\frac{1}{x}+\\frac{1}{y}\\right) \\geqslant 4$, 于是, $\\frac{1}{x}+ \\frac{1}{y} \\geqslant \\frac{4}{x+y}$. 可以得到 $\\frac{1}{a+b}+\\frac{1}{b+c} \\geqslant \\frac{4}{a+2 b+c}, \\frac{1}{b+c}+\\frac{1}{c+a} \\geqslant \\frac{4}{b+2 c+a}$, $\\frac{1}{c+a}+\\frac{1}{a+b} \\geqslant \\frac{4}{c+2 a+b}$ 三式相加, 得 $\\frac{2}{b+c}+\\frac{2}{c+a}+\\frac{2}{a+b} \\geqslant \\frac{4}{a+2 b+c}+ \\frac{4}{b+2 c+a}+\\frac{4}{c+2 a+b}$, 将 $a+b+c=1$ 代入其中, 并约去 2 即得证.", + "remark": "", + "figures": [] +} \ No newline at end of file diff --git a/processed_dataset/proof/1221.json b/processed_dataset/proof/1221.json new file mode 100644 index 0000000000000000000000000000000000000000..c6860849bef41465723b83a33535e52d6763180e --- /dev/null +++ b/processed_dataset/proof/1221.json @@ -0,0 +1,8 @@ +{ + "source_file": "./raw_volume-zh/volume4/exercise3.tex", + "problem_type": "proof", + "problem": "问题5. 设正实数 $a 、 b 、 c$ 满足 $a b+b c+c a=\\frac{1}{3}$. 证明:\n$$\n\\frac{a}{a^2-b c+1}+\\frac{b}{b^2-c a+1}+\\frac{c}{c^2-a b+1} \\geqslant \\frac{1}{a+b+c} .\n$$", + "solution": "等式左边的分母显然为正数.\n由柯西不等式得 $\\frac{a}{a^2-b c+1}+$\n$$\n\\begin{aligned}\n& \\frac{b}{b^2-c a+1}+\\frac{c}{c^2-a b+1}=\\frac{a^2}{a^3-a b c+a}+\\frac{b^2}{b^3-a b c+b}+\\frac{c^2}{c^3-a b c+c} \\geqslant \\\\\n& \\frac{(a+b+c)^2}{a^3+b^3+c^3+a+b+c-3 a b c}=\\frac{(a+b+c)^2}{(a+b+c)\\left(a^2+b^2+c^2-a b-b c-c a\\right)+(a+b+c)} \\\\\n& =\\frac{a+b+c}{a^2+b^2+c^2-a b-b c-c a+1}=\\frac{a+b+c}{a^2+b^2+c^2+2(a b+b c+c a)}= \\\\\n& \\frac{1}{a+b+c} \\text {. 命题得证.\n}\n\\end{aligned}\n$$", + "remark": "", + "figures": [] +} \ No newline at end of file diff --git a/processed_dataset/proof/1222.json b/processed_dataset/proof/1222.json new file mode 100644 index 0000000000000000000000000000000000000000..a0e800635e3fea405c807ad79445c87b794e7b36 --- /dev/null +++ b/processed_dataset/proof/1222.json @@ -0,0 +1,8 @@ +{ + "source_file": "./raw_volume-zh/volume4/exercise3.tex", + "problem_type": "proof", + "problem": "问题6. 已知 $x 、 y, z$ 为正实数.\n证明:\n$$\n\\frac{1+x y+x z}{(1+y+z)^2}+\\frac{1+y z+y x}{(1+z+x)^2}+\\frac{1+z x+z y}{(1+x+y)^2} \\geqslant 1 .\n$$", + "solution": "由柯西不等式得 $\\left(1+\\frac{y}{x}+\\frac{z}{x}\\right)(1+x y+x z) \\geqslant(1+y+z)^2 \\Rightarrow \\frac{1+x y+x z}{(1+y+z)^2} \\geqslant \\frac{x}{x+y+z}$. 同理, $\\frac{1+y z+y x}{(1+z+x)^2} \\geqslant \\frac{y}{x+y+z} \\cdot \\frac{1+z x+z y}{(1+x+y)^2} \\geqslant \\frac{z}{x+y+z}$. 上述三个不等式相加即得 $\\frac{1+x y+x z}{(1+y+z)^2}+\\frac{1+y z+y x}{(1+z+x)^2}+ \\frac{1+z x+z y}{(1+x+y)^2} \\geqslant 1$", + "remark": "", + "figures": [] +} \ No newline at end of file diff --git a/processed_dataset/proof/1223.json b/processed_dataset/proof/1223.json new file mode 100644 index 0000000000000000000000000000000000000000..6f90542650c84187401798d56d1ecc685958d752 --- /dev/null +++ b/processed_dataset/proof/1223.json @@ -0,0 +1,8 @@ +{ + "source_file": "./raw_volume-zh/volume4/exercise3.tex", + "problem_type": "proof", + "problem": "问题7. $x, y, z \\in \\mathbf{R}^{+}$, 且 $x y z \\geqslant 1$. 求证:\n$$\n\\frac{x^5-x^2}{x^5+y^2+z^2}+\\frac{y^5-y^2}{y^5+z^2+x^2}+\\frac{z^5-z^2}{z^5+x^2+y^2} \\geqslant 0 \\text {. }\n$$", + "solution": "原不等式等价于 $\\sum \\frac{1}{x^5+y^2+z^2} \\leqslant \\frac{3}{x^2+y^2+z^2}$. 利用 $x y z \\geqslant 1$ 及柯西不等式得 $\\left(x^5+y^2+z^2\\right) \\cdot\\left(y z+y^2+z^2\\right) \\geqslant\\left(\\sum x^2\\right)^2$. 而 $\\sum\\left(y z+y^2+z^2\\right) \\leqslant \\sum\\left(\\frac{y^2+z^2}{2}+y^2+z^2\\right)=3 \\sum x^2$. 代入即得结果.", + "remark": "", + "figures": [] +} \ No newline at end of file diff --git a/processed_dataset/proof/1224.json b/processed_dataset/proof/1224.json new file mode 100644 index 0000000000000000000000000000000000000000..1a31c2414312b8907d795ac8ff3b306731698b7d --- /dev/null +++ b/processed_dataset/proof/1224.json @@ -0,0 +1,8 @@ +{ + "source_file": "./raw_volume-zh/volume4/exercise3.tex", + "problem_type": "proof", + "problem": "问题8. 设 $a, b, c \\in \\mathbf{R}^{+}$, 且 $a+b+c=3$. 证明:\n$$\n\\sum \\frac{a^4}{b^2+c} \\geqslant \\frac{3}{2} \\text {. }\n$$\n其中, \" $\\sum$ \" 表示轮换对称和.", + "solution": "由柯西不等式知 $\\left(b^2+c+c^2+a+a^2+b\\right)$.\n$\\left(\\frac{a^4}{b^2+c}+\\frac{b^4}{c^2+a}+\\frac{c^4}{c^2+b}\\right) \\geqslant\\left(a^2+b^2+c^2\\right)^2$. 故 $\\frac{a^4}{b^2+c}+\\frac{b^4}{c^2+a}+\\frac{c^4}{a^2+b} \\geqslant \\frac{\\left(a^2+b^2+c^2\\right)^2}{a^2+b^2+c^2+3}$. 令 $a^2+b^2+c^2=x$. 易证 $x \\geqslant 3$. 故 $\\frac{x^2}{3+x} \\geqslant \\frac{3}{2} \\Leftrightarrow 2 x^2 \\geqslant 9+3 x \\Leftrightarrow 2 x^2-3 x-9 \\geqslant 0 \\Leftrightarrow(2 x+3)(x-3) \\geqslant 0$. 显然成立.\n于是, $\\frac{a^4}{b^2+c}+ \\frac{b^4}{c^2+a}+\\frac{c^4}{a^2+b} \\geqslant \\frac{3}{2}$.", + "remark": "", + "figures": [] +} \ No newline at end of file diff --git a/processed_dataset/proof/1225.json b/processed_dataset/proof/1225.json new file mode 100644 index 0000000000000000000000000000000000000000..902ea94c5d405a56404818c833dcf8b81fe33d08 --- /dev/null +++ b/processed_dataset/proof/1225.json @@ -0,0 +1,8 @@ +{ + "source_file": "./raw_volume-zh/volume4/exercise3.tex", + "problem_type": "proof", + "problem": "问题9. 设实数 $a, b, c>0$, 且满足 $a+b+c=3$. 证明:\n$$\n\\frac{a^2+3 b^2}{a b^2(4-a b)}+\\frac{b^2+3 c^2}{b c^2(4-b c)}+\\frac{c^2+3 a^2}{c a^2(4-c a)} \\geqslant 4 .\n$$", + "solution": "记 $A=\\frac{a^2}{a b^2(4-a b)}+\\frac{b^2}{b c^2(4-b c)}+\\frac{c^2}{c a^2(4-c a)}, B=\\frac{b^2}{a b^2(4-a b)} +\\frac{c^2}{b c^2(4-b c)}+\\frac{a^2}{c a^2(4-c a)}$. 欲证明原不等式, 只需证明 $A \\geqslant 1, B \\geqslant 1$. 由柯西一施瓦兹不等式得 $\\left(\\frac{4-a b}{a}+\\frac{4-b c}{b}+\\frac{4-a c}{c}\\right) A \\geqslant\\left(\\frac{1}{a}+\\frac{1}{b}+\\frac{1}{c}\\right)^2$. 设 $k=\\frac{1}{a}+\\frac{1}{b}+\\frac{1}{c}$, 则 $A \\geqslant \\frac{k^2}{4 k-3}$. 由 $(a+b+c)\\left(\\frac{1}{a}+\\frac{1}{b}+\\frac{1}{c}\\right) \\geqslant 3^2 \\Rightarrow k= \\frac{1}{a}+\\frac{1}{b}+\\frac{1}{c} \\geqslant 3 \\Rightarrow(k-3)(k-1) \\geqslant 0 \\Rightarrow k^2-4 k+3 \\geqslant 0 \\Rightarrow A=\\frac{k^2}{4 k-3} \\geqslant 1$. 又 $B=\\frac{1}{a(4-a b)}+\\frac{1}{b(4-b c)}+\\frac{1}{c(4-c a)}$, 则 $\\left(\\frac{4-a b}{a}+\\frac{4-b c}{b}+\\frac{4-c a}{c}\\right) B \\geqslant\\left(\\frac{1}{a}+\\frac{1}{b}+\\frac{1}{c}\\right)^2$. 故 $B \\geqslant \\frac{k^2}{4 k-3} \\geqslant 1$. 因此, $A+3 B \\geqslant 4$.", + "remark": "", + "figures": [] +} \ No newline at end of file diff --git a/processed_dataset/proof/1226.json b/processed_dataset/proof/1226.json new file mode 100644 index 0000000000000000000000000000000000000000..bccae216293a8e51d323a856dc1746225ed07486 --- /dev/null +++ b/processed_dataset/proof/1226.json @@ -0,0 +1,8 @@ +{ + "source_file": "./raw_volume-zh/volume4/exercise3.tex", + "problem_type": "proof", + "problem": "问题10. 设 $a, b, c \\in \\mathbf{R}^{+}$, 且 $a b c=1$, 求证:\n$$\n\\frac{1}{1+2 a}+\\frac{1}{1+2 b}+\\frac{1}{1+2 c} \\geqslant 1 \\text {. }\n$$", + "solution": "令 $a=\\frac{x}{y}, b=\\frac{y}{z}, c=\\frac{z}{x}$, 则原不等式等价于 $\\frac{y}{2 x+y}+\\frac{z}{2 y+z}+ \\frac{x}{2 z+x} \\geqslant 1$. 由柯西不等式, 得 $[x(x+2 z)+y(y+2 x)+z(z+ 2 y)]\\left(\\frac{x}{x+2 z}+\\frac{y}{y+2 x}+\\frac{z}{z+2 y}\\right) \\geqslant(x+y+z)^2$, 即 $(x+y+ z)^2\\left(\\frac{x}{x+2 z}+\\frac{y}{y+2 x}+\\frac{z}{z+2 y}\\right) \\geqslant(x+y+z)^2$.", + "remark": "", + "figures": [] +} \ No newline at end of file diff --git a/processed_dataset/proof/1227.json b/processed_dataset/proof/1227.json new file mode 100644 index 0000000000000000000000000000000000000000..6c2f5ec45bdc1903c64eedbd534ab31ba7b452e8 --- /dev/null +++ b/processed_dataset/proof/1227.json @@ -0,0 +1,8 @@ +{ + "source_file": "./raw_volume-zh/volume4/exercise3.tex", + "problem_type": "proof", + "problem": "问题11. 设 $a_1, a_2, \\cdots, a_n$ 为实数,证明:\n$$\n\\sqrt[3]{a_1^3+a_2^3+\\cdots+a_n^3} \\leqslant \\sqrt{a_1^2+a_2^2+\\cdots+a_n^2} .\n$$", + "solution": "由 $\\left(\\sum_{i=1}^n a_i^3\\right)^2 \\leqslant \\sum_{i=1}^n a_i^2 \\sum_{i=1}^n a_i^4 \\leqslant \\sum_{i=1}^n a_i^2\\left(\\sum_{i=1}^n a_i^2\\right)^2=\\left(\\sum_{i=1}^n a_i^2\\right)^3$, 则 $\\left(\\sum_{i=1}^n a_i^3\\right)^{\\frac{1}{3}} \\leqslant\\left(\\sum_{i=1}^n a_i^2\\right)^{\\frac{1}{2}}$", + "remark": "", + "figures": [] +} \ No newline at end of file diff --git a/processed_dataset/proof/1228.json b/processed_dataset/proof/1228.json new file mode 100644 index 0000000000000000000000000000000000000000..99288493198f7e5f4510b40175b0464314028d0e --- /dev/null +++ b/processed_dataset/proof/1228.json @@ -0,0 +1,8 @@ +{ + "source_file": "./raw_volume-zh/volume4/exercise3.tex", + "problem_type": "proof", + "problem": "问题12. 已知 $a 、 b 、 c$ 为正实数,证明:\n$$\n\\frac{9}{a+b+c} \\leqslant 2\\left(\\frac{1}{a+b}+\\frac{1}{b+c}+\\frac{1}{c+a}\\right) .\n$$", + "solution": "由柯西不等式, 得 $2(a+b+c)\\left(\\frac{1}{a+b}+\\frac{1}{b+c}+\\frac{1}{c+a}\\right)=[(a+b)+(b+c)+(c+a)]\\left(\\frac{1}{a+b}+\\frac{1}{b+c}+\\frac{1}{c+a}\\right) \\geqslant 9$, 故命题成立.", + "remark": "", + "figures": [] +} \ No newline at end of file diff --git a/processed_dataset/proof/1229.json b/processed_dataset/proof/1229.json new file mode 100644 index 0000000000000000000000000000000000000000..d0ffe0fdadb379e4e487cb3455927134db6dcaf5 --- /dev/null +++ b/processed_dataset/proof/1229.json @@ -0,0 +1,8 @@ +{ + "source_file": "./raw_volume-zh/volume4/exercise3.tex", + "problem_type": "proof", + "problem": "问题13. 设 $a_i \\in \\mathbf{R}^{+}(i=1,2, \\cdots, n)$, 求证:\n$$\n\\frac{1}{a_1}+\\frac{2}{a_1+a_2}+\\cdots+\\frac{n}{a_1+\\cdots+a_n}<2 \\sum_{i=1}^n \\frac{1}{a_i} .\n$$", + "solution": "由柯西不等式, 得 $\\frac{k^2(k+1)^2}{4}=\\left(\\sum_{i=1}^k \\frac{i}{\\sqrt{a_i}} \\cdot \\sqrt{a_i}\\right)^2 \\leqslant \\sum_{i=1}^k \\frac{i^2}{a_i} \\sum_{i=1}^k a_i$.\n所以 $\\frac{k}{\\sum_{i=1}^k a_i} \\leqslant \\frac{4}{k(k+1)^2} \\sum_{i=1}^k \\frac{i^2}{a_i}$. 求和, 得 $\\sum_{k=1}^n \\frac{k}{\\sum_{i=1}^k a_i} \\leqslant \\sum_{k=1}^n\\left[\\frac{4}{k(k+1)^2} \\sum_{i=1}^k \\frac{i^2}{a_i}\\right]<2 \\sum_{i=1}^n\\left[\\frac{i^2}{a_i} \\sum_{k=i}^n \\frac{2 k+1}{k^2(k+1)^2}\\right]=2 \\sum_{i=1}^n\\left[\\frac{i^2}{a_i} \\sum_{k=i}^n\\left(\\frac{1}{k^2}-\\frac{1}{(k+1)^2}\\right)\\right]=2 \\sum_{i=1}^n \\frac{i^2}{a_i}\\left(\\frac{1}{i^2}-\\frac{1}{(n+1)^2}\\right)<2 \\cdot \\sum_{i=1}^n \\frac{i^2}{a_i} \\cdot \\frac{1}{i^2}=2 \\sum_{i=1}^n \\frac{1}{a_i}$.", + "remark": "", + "figures": [] +} \ No newline at end of file diff --git a/processed_dataset/proof/1230.json b/processed_dataset/proof/1230.json new file mode 100644 index 0000000000000000000000000000000000000000..53c0592aec25fe8fde87f580db2e3b936cabd7a0 --- /dev/null +++ b/processed_dataset/proof/1230.json @@ -0,0 +1,8 @@ +{ + "source_file": "./raw_volume-zh/volume4/exercise3.tex", + "problem_type": "proof", + "problem": "问题14. 设 $a_i, b_i, c_i, d_i$ 为正实数 $(i=1,2, \\cdots, n)$, 求证:\n$$\n\\left(\\sum_{i=1}^n a_i b_i c_i d_i\\right)^4 \\leqslant \\sum_{i=1}^n a_i^4 \\sum_{i=1}^n b_i^4 \\sum_{i=1}^n c_i^4 \\sum_{i=1}^n d_i^4 .\n$$", + "solution": "$\\left(\\sum_{i=1}^n a_i b_i c_i d_i\\right)^4 \\leqslant\\left[\\sum_{i=1}^n\\left(a_i b_i\\right)^2\\right]^2\\left[\\sum_{i=1}^n\\left(c_i d_i\\right)^2\\right]^2 \\leqslant \\sum_{i=1}^n a_i^4 \\sum_{i=1}^n b_i^4 \\sum_{i=1}^n c_i^4 \\sum_{i=1}^n d_i^4$.", + "remark": "", + "figures": [] +} \ No newline at end of file diff --git a/processed_dataset/proof/1231.json b/processed_dataset/proof/1231.json new file mode 100644 index 0000000000000000000000000000000000000000..816be460ba96d54317aa9268f652c84eeb8d002c --- /dev/null +++ b/processed_dataset/proof/1231.json @@ -0,0 +1,8 @@ +{ + "source_file": "./raw_volume-zh/volume4/exercise3.tex", + "problem_type": "proof", + "problem": "问题15. 设 $n(n \\geqslant 2)$ 为正整数,求证:\n$$\n\\frac{4}{7}<1-\\frac{1}{2}+\\frac{1}{3}-\\frac{1}{4}+\\cdots+\\frac{1}{2 n-1}-\\frac{1}{2 n}<\\frac{\\sqrt{2}}{2} .\n$$", + "solution": "$1-\\frac{1}{2}+\\frac{1}{3}-\\frac{1}{4}+\\cdots+\\frac{1}{2 n-1}-\\frac{1}{2 n}=\\frac{1}{n+1}+\\frac{1}{n+2}+\\cdots+\\frac{1}{2 n}$. 由柯西不等式, 得 $[(n+1)+(n+2)+\\cdots+(2 n)]\\left(\\frac{1}{n+1}+\\frac{1}{n+2}+\\cdots+\\frac{1}{2 n}\\right)> n^2$, 所以 $\\frac{1}{n+1}+\\frac{1}{n+2}+\\cdots+\\frac{1}{2 n}>\\frac{2 n}{3 n+1} \\geqslant \\frac{4}{7}$. 又 $\\frac{1}{n+1}+ \\frac{1}{n+2}+\\cdots+\\frac{1}{2 n}<\\left(1^2+1^2+\\cdots+1^2\\right)^{\\frac{1}{2}}\\left[\\frac{1}{(n+1)^2}+\\cdots+\\frac{1}{(2 n)^2}\\right]^{\\frac{1}{2}}< \\sqrt{n}\\left[\\frac{1}{n(n+1)}+\\frac{1}{(n+1)(n+2)}+\\cdots+\\frac{1}{(2 n-1) \\cdot 2 n}\\right]^{\\frac{1}{2}}=\\frac{\\sqrt{2}}{2}$.", + "remark": "", + "figures": [] +} \ No newline at end of file diff --git a/processed_dataset/proof/1232.json b/processed_dataset/proof/1232.json new file mode 100644 index 0000000000000000000000000000000000000000..ab005893ac322b88350a2346a5172de0c2f02194 --- /dev/null +++ b/processed_dataset/proof/1232.json @@ -0,0 +1,8 @@ +{ + "source_file": "./raw_volume-zh/volume4/exercise3.tex", + "problem_type": "proof", + "problem": "问题16. 设 $a_1, a_2, \\cdots, a_n$ 为正实数,证明:\n$$\n\\frac{\\left(\\sum_{i=1}^n a_i\\right)^2}{2 \\sum_{i=1}^n a_i^2} \\leqslant \\frac{a_1}{a_2+a_3}+\\frac{a_2}{a_3+a_4}+\\cdots+\\frac{a_n}{a_1+a_2} .\n$$", + "solution": "令 $a_{n+1}=a_1, a_{n+2}=a_2$, 则 $\\sum_{i=1}^n a_i\\left(a_{i+1}+a_{i+2}\\right) \\sum_{i=1}^n \\frac{a_i}{a_{i+1}+a_{i+2}} \\geqslant \\left(\\sum_{i=1}^n a_i\\right)^2$. 于是 $\\sum_{i=1}^n \\frac{a_i}{a_{i+1}+a_{i+2}} \\geqslant \\frac{\\left(\\sum_{i=1}^n a_i\\right)^2}{\\sum_{i=1}^n a_i\\left(a_{i+1}+a_{i+2}\\right)}$. 只需证 $2 \\sum_{i=1}^n a_i^2 \\geqslant \\sum_{i=1}^n a_i\\left(a_{i+1}+a_{i+2}\\right)$, 即 $\\frac{1}{2} \\sum_{i=1}^n\\left[\\left(a_i^2+a_{i+1}^2\\right)+\\left(a_i^2+a_{i+2}^2\\right)\\right] \\geqslant \\sum_{i=1}^n a_i\\left(a_{i+1}+a_{i+2}\\right)$. 由 $a_i^2+a_{i+1}^2 \\geqslant 2 a_i a_{i+1}$, 便得到命题成立.", + "remark": "", + "figures": [] +} \ No newline at end of file diff --git a/processed_dataset/proof/1233.json b/processed_dataset/proof/1233.json new file mode 100644 index 0000000000000000000000000000000000000000..a7fde852eb66046941bd4ea8e16b949e6745e6e1 --- /dev/null +++ b/processed_dataset/proof/1233.json @@ -0,0 +1,8 @@ +{ + "source_file": "./raw_volume-zh/volume4/exercise3.tex", + "problem_type": "proof", + "problem": "问题17. 设 $a 、 b 、 c 、 d$ 为正数,证明:\n$$\n\\sqrt{\\frac{a^2+b^2+c^2+d^2}{4}} \\geqslant \\sqrt[3]{\\frac{a b c+b c d+c d a+d a b}{4}} .\n$$", + "solution": "$\\frac{1}{4}(a b c+b c d+c d a+d a b)=\\frac{1}{4}[b c(a+d)+d a(b+c)] \\leqslant$\n$$\n\\begin{aligned}\n& \\frac{1}{4}\\left[\\left(\\frac{b+c}{2}\\right)^2(a+d)+\\left(\\frac{a+d}{2}\\right)^2(b+c)\\right]=\\frac{1}{16}(b+c)(a+d)(a+b+c+d) \\\\\n& \\leqslant \\frac{1}{64}(a+b+c+d)^3=\\left(\\frac{a+b+c+d}{4}\\right)^3 \\leqslant\\left(\\sqrt{\\frac{a^3+b^3+c^3+d^3}{4}}\\right)^3 .\n\\end{aligned}\n$$", + "remark": "", + "figures": [] +} \ No newline at end of file diff --git a/processed_dataset/proof/1234.json b/processed_dataset/proof/1234.json new file mode 100644 index 0000000000000000000000000000000000000000..925086501e5755e8aa342cbf5c27260301b712b3 --- /dev/null +++ b/processed_dataset/proof/1234.json @@ -0,0 +1,8 @@ +{ + "source_file": "./raw_volume-zh/volume4/exercise3.tex", + "problem_type": "proof", + "problem": "问题18. 设 $n$ 是大于 1 的自然数,求证:\n$$\n\\sqrt{\\mathrm{C}_n^1}+2 \\cdot \\sqrt{\\mathrm{C}_n^2}+\\cdots+n \\cdot \\sqrt{\\mathrm{C}_n^n}<\\sqrt{2^{n-1} \\cdot n^3} .\n$$", + "solution": "当 $n=2$ 时,则 $\\sqrt{2}<2$. 命题成立.\n当 $n=3$ 时, 则 $1<\\sqrt{3}$. 所以可设 $n \\geqslant 4$. 由柯西不等式, 得 $1 \\cdot \\sqrt{\\mathrm{C}_n^1}+2 \\cdot \\sqrt{\\mathrm{C}_n^2}+\\cdots+n \\cdot \\sqrt{\\mathrm{C}_n^n} \\leqslant\\left(1^2+2^2\\right. \\left.+\\cdots+n^2\\right)^{\\frac{1}{2}}\\left(\\mathrm{C}_n^1+\\mathrm{C}_n^2+\\cdots+\\mathrm{C}_n^n\\right)^{\\frac{1}{2}}=\\left[\\frac{n(n+1)(2 n+1)}{6}\\right]^{\\frac{1}{2}} \\cdot\\left(2^n-1\\right)^{\\frac{1}{2}}$. 即证明: $\\frac{n(n+1)(2 n+1)}{6} \\cdot\\left(2^n-1\\right)<2^{n-1} \\cdot n^3$ 便可.\n等价于 $\\left(2 n^2+3 n+1\\right) \\left(2^n-1\\right)<3 n^2 \\cdot 2^n$. 因为 $n \\geqslant 4$, 故 $n^2>3 n, n^2 \\geqslant 3 n+1$, 进而 $3 n^2 \\geqslant 2 n^2+ 3 n+1$. 所以 $\\left(2 n^2+3 n+1\\right)\\left(2^n-1\\right)<3 n^2 \\cdot 2^n$. 从而, 命题成立.", + "remark": "", + "figures": [] +} \ No newline at end of file diff --git a/processed_dataset/proof/1235.json b/processed_dataset/proof/1235.json new file mode 100644 index 0000000000000000000000000000000000000000..55bbac5fdc0e6239791ae092d890d54a07e86ec3 --- /dev/null +++ b/processed_dataset/proof/1235.json @@ -0,0 +1,8 @@ +{ + "source_file": "./raw_volume-zh/volume4/exercise4.tex", + "problem_type": "proof", + "problem": "问题1. 已知非负实数 $a_1, a_2, \\cdots, a_{100}$ 满足 $a_1^2+a_2^2+\\cdots+a_{100}^2=1$. 证明:\n$$\na_1^2 a_2+a_2^2 a_3+\\cdots+a_{100}^2 a_1<\\frac{12}{25} .\n$$", + "solution": "设 $S=\\sum_{k=1}^{100} a_k^2 a_{k+1}$, 其中, 定义 $a_{101}=a_1, a_{102}=a_2$. 由柯西不等式及均值不等式得 $(3 S)^2=\\left[\\sum_{k=1}^{100} a_{k+1}\\left(a_k^2+2 a_{k+1} a_{k+2}\\right)\\right]^2 \\leqslant\\left(\\sum_{k=1}^{100} a_{k+1}^2\\right) \\sum_{k=1}^{100}\\left(a_k^2+2 a_{k+1} a_{k+2}\\right)^2 =1 \\times \\sum_{k=1}^{100}\\left(a_k^2+2 a_{k+1} a_{k+2}\\right)^2=\\sum_{k=1}^{100}\\left(a_k^4+4 a_k^2 a_{k+1} a_{k+2}+4 a_{k+1}^2 a_{k+2}^2\\right) \\leqslant \\sum_{k=1}^{100}\\left[a_k^4+\\right. \\left.2 a_k^2\\left(a_{k+1}^2+a_{k+2}^2\\right)+4 a_{k+1}^2 a_{k+2}^2\\right]=\\sum_{k=1}^{100}\\left(a_k^4+6 a_k^2 a_{k+1}^2+2 a_k^2 a_{k+2}^2\\right)$. 又 $\\sum_{k=1}^{100}\\left(a_k^4+2 a_k^2 a_{k+1}^2\\right. \\left.+2 a_k^2 a_{k+2}^2\\right) \\leqslant\\left(\\sum_{k=1}^{100} a_k^2\\right)^2, \\sum_{k=1}^{100} a_k^2 a_{k+1}^2 \\leqslant\\left(\\sum_{i=1}^{50} a_{2 i-1}^2\\right)\\left(\\sum_{j=1}^{50} a_{2 j}^2\\right)$, 故 $(3 S)^2 \\leqslant\\left(\\sum_{k=1}^{100} a_k^2\\right)^2+ 4\\left(\\sum_{i=1}^{50} a_{2 i-1}^2\\right)\\left(\\sum_{j=1}^{50} a_{2 j}^2\\right) \\leqslant 1+\\left(\\sum_{i=1}^{50} a_{2 i-1}^2+\\sum_{j=1}^{50} a_{2 j}^2\\right)^2=2$. 从而, $S \\leqslant \\frac{\\sqrt{2}}{3} \\approx 0.4714< 0.48=\\frac{12}{25}$.", + "remark": "", + "figures": [] +} \ No newline at end of file diff --git a/processed_dataset/proof/1236.json b/processed_dataset/proof/1236.json new file mode 100644 index 0000000000000000000000000000000000000000..f0969eac3abbc1532105ad6a6425576e50692156 --- /dev/null +++ b/processed_dataset/proof/1236.json @@ -0,0 +1,8 @@ +{ + "source_file": "./raw_volume-zh/volume4/exercise4.tex", + "problem_type": "proof", + "problem": "问题3. 设 $x 、 y 、 z$ 为正实数, 满足\n$$\nx y+y z+z x=x+y+z .\n$$\n证明: $\\frac{1}{x^2+y+1}+\\frac{1}{y^2+z+1}+\\frac{1}{z^2+x+1} \\leqslant 1$, 并确定等号成立的条件.", + "solution": "由柯西不等式得 $\\frac{1}{x^2+y+1} \\leqslant \\frac{1+y+z^2}{(x+y+z)^2}, \\frac{1}{y^2+z+1} \\leqslant\\frac{1+z+x^2}{(x+y+z)^2}, \\frac{1}{z^2+x+1} \\leqslant \\frac{1+x+y^2}{(x+y+z)^2}$. 故 $\\frac{1}{x^2+y+1}+\\frac{1}{y^2+z+1}+ \\frac{1}{z^2+x+1} \\leqslant \\frac{3+x+y+z+x^2+y^2+z^2}{(x+y+z)^2}$. 记上不等式右边为 $S$. 只须证 $S \\leqslant 1$. 事实上, $S \\leqslant 1 \\Leftrightarrow 3+x+y+z \\leqslant 2(x y+y z+z x)$. 因为 $x+y+z= x y+y z+z x$, 所以, 只须证 $x+y+z \\geqslant 3$. 又 $x+y+z=x y+y z+z x \\leqslant \\frac{(x+y+z)^2}{3}$, 因此, $x+y+z \\geqslant 3$. 故原式得证.\n当且仅当 $x=y=z=1$ 时, 原式等号成立.", + "remark": "", + "figures": [] +} \ No newline at end of file diff --git a/processed_dataset/proof/1237.json b/processed_dataset/proof/1237.json new file mode 100644 index 0000000000000000000000000000000000000000..b9fcc8f6fc587e46a52f7f92d6d892d3567a126f --- /dev/null +++ b/processed_dataset/proof/1237.json @@ -0,0 +1,8 @@ +{ + "source_file": "./raw_volume-zh/volume4/exercise4.tex", + "problem_type": "proof", + "problem": "问题4. 设 $x 、 y 、 z$ 为正实数, 证明:\n$$\n\\sum \\frac{x}{\\sqrt{2\\left(x^2+y^2\\right)}}<\\sum \\frac{4 x^2+y^2}{x^2+4 y^2}<9,\n$$\n其中, \" $\\sum \"$ 表示轮换对称和.", + "solution": "令 $x \\geqslant y, z$. 则 $\\frac{4 z^2+x^2}{z^2+4 x^2} \\leqslant 1, \\frac{4 x^2+y^2}{x^2+4 y^2}<4, \\frac{4 y^2+z^2}{y^2+4 z^2}<4$. 三式相加, 知右边的不等式成立.\n由均值不等式知 $4 x y^2 \\leqslant y^3+4 x^2 y$. 则 $y^3+4 x^2 y+ 4 x^3+x y^2>4 x y^2+x^3 \\Rightarrow \\frac{4 x^2+y^2}{x^2+4 y^2}>\\frac{x}{x+y}$. 故 $\\sum \\frac{x}{x+y}<\\sum \\frac{4 x^2+y^2}{x^2+4 y^2}$. 由柯西一施瓦兹不等式知 $\\sum \\frac{x}{\\sqrt{2\\left(x^2+y^2\\right)}} \\leqslant \\sum \\frac{x}{x+y}<\\sum \\frac{4 x^2+y^2}{x^2+4 y^2}$. 故命题得证.", + "remark": "", + "figures": [] +} \ No newline at end of file diff --git a/processed_dataset/proof/1238.json b/processed_dataset/proof/1238.json new file mode 100644 index 0000000000000000000000000000000000000000..9f25d0b4e072039aa1db2e0559c9b14f10719430 --- /dev/null +++ b/processed_dataset/proof/1238.json @@ -0,0 +1,8 @@ +{ + "source_file": "./raw_volume-zh/volume4/exercise4.tex", + "problem_type": "proof", + "problem": "问题5. 设 $a, b, c>0$ 且 $a+b+c=3$. 求证:\n$$\n\\frac{a^2}{a+b^2}+\\frac{b^2}{b+c^2}+\\frac{c^2}{c+a^2} \\geqslant \\frac{3}{2} \\text {. }\n$$", + "solution": "由柯西不等式, 我们有 $\\sum \\frac{a^2}{a+b^2} \\sum a^2\\left(a+b^2\\right) \\geqslant\\left(a^2+b^2+c^2\\right)^2$, 因此只需要证明 $2\\left(a^2+b^2+c^2\\right)^2 \\geqslant 3\\left[a^2\\left(a+b^2\\right)+b^2\\left(b+c^2\\right)+c^2\\left(c+a^2\\right)\\right]$, 这等价于 $2\\left(a^4+b^4+c^4\\right)+a^2 b^2+b^2+c^2 \\geqslant 3\\left(a^3+b^3+c^3\\right)$, 由 $a+b+c=3$ 代入上式转化为 $a^4+b^4+c^4+a^2 b^2+b^2 c^2+c^2 a^2 \\geqslant a^3(b+c)+b^3(c+a)+ c^3(a+b)$, 由均值不等式 $\\left(a^4+a^2 b^2\\right)+\\left(a^4+a^2 c^2\\right) \\geqslant 2 a^3 b+2 a^3 c$, $\\left(b^4+b^2 c^2\\right)+\\left(b^4+b^2 a^2\\right) \\geqslant 2 b^3 c+2 b^3 a,\\left(c^4+c^2 a^2\\right)+\\left(c^4+c^2 b^2\\right) \\geqslant 2 c^3 a+ 2 c^3 b$. 上述三式相加除以 2 即得证.", + "remark": "", + "figures": [] +} \ No newline at end of file diff --git a/processed_dataset/proof/1239.json b/processed_dataset/proof/1239.json new file mode 100644 index 0000000000000000000000000000000000000000..064c343589ea83733e9ff4da5a1a268ad1b60f15 --- /dev/null +++ b/processed_dataset/proof/1239.json @@ -0,0 +1,8 @@ +{ + "source_file": "./raw_volume-zh/volume4/exercise4.tex", + "problem_type": "proof", + "problem": "问题6. 已知 $\\lambda$ 为正实数.\n求 $\\lambda$ 的最大值,使得对于所有满足条件\n$$\nu \\sqrt{w w}+v \\sqrt{w u}+w \\sqrt{u v} \\geqslant 1\n$$\n的正实数 $u 、 v 、 w$,均有\n$$\nu+v+w \\geqslant \\lambda .\n$$", + "solution": "首先, 易观察出当 $u=v=w=\\frac{\\sqrt{3}}{3}$ 时, $u \\sqrt{v w}+v \\sqrt{w u}+w \\sqrt{u v}=$ 1 及 $u+v+w=\\sqrt{3}$. 因此, $\\lambda$ 的最大值不超过 $\\sqrt{3}$. 下面证明: 对于所有 $u, v$, $w>0$, 且满足 $u \\sqrt{v w}+v \\sqrt{w u}+w \\sqrt{u v} \\geqslant 1$, 均有 $u+v+w \\geqslant \\sqrt{3}$. 由均值不等式及柯西不等式有 $\\frac{(u+v+w)^4}{9}=\\left(\\frac{u+v+w}{3}\\right)^3 \\cdot 3(u+v+w) \\geqslant 3 u v w(u+v+w)=(u w w+v w u+r u v)(u+v+w) \\geqslant(u \\sqrt{v w}+v \\sqrt{w u}+ w \\sqrt{u v})^2 \\geqslant 1$. 因此, $u+v+w \\geqslant \\sqrt{3}$. 当且仅当 $u=v=w=\\frac{\\sqrt{3}}{3}$ 时, 上式等号成立.\n综上, 所求 $\\lambda$ 的最大值为 $\\sqrt{3}$.", + "remark": "", + "figures": [] +} \ No newline at end of file diff --git a/processed_dataset/proof/1240.json b/processed_dataset/proof/1240.json new file mode 100644 index 0000000000000000000000000000000000000000..ff595638dd200f4bce720f9fb529ed3b1a6ef4f1 --- /dev/null +++ b/processed_dataset/proof/1240.json @@ -0,0 +1,8 @@ +{ + "source_file": "./raw_volume-zh/volume4/exercise4.tex", + "problem_type": "proof", + "problem": "问题7. 设 $x_1, x_2, \\cdots, x_n$ 为正实数, $x_{n+1}=x_1+x_2+\\cdots+x_n$, 证明:\n$$\nx_{n+1} \\sum_{i=1}^n\\left(x_{n+1}-x_i\\right) \\geqslant\\left(\\sum_{i=1}^n \\sqrt{x_i\\left(x_{n+1}-x_i\\right)}\\right)^2 .\n$$", + "solution": "由于 $\\sum_{i=1}^n\\left(x_{n+1}-x_i\\right)=n x_{n+1}-\\sum_{i=1}^n x_i=(n-1) x_{n+1}$, 于是, 只需证明, $x_{n+1} \\sqrt{n-1} \\geqslant \\sum_{i=1}^n \\sqrt{x_i\\left(x_{n+1}-x_i\\right)}$, 即证 $\\sum_{i=1}^n \\sqrt{\\frac{x_i}{x_{n+1}}\\left(1-\\frac{x_i}{x_{n+1}}\\right)} \\leqslant \\sqrt{n-1}$. 由柯西不等式, 得 $\\left[\\sum_{i=1}^n \\sqrt{\\frac{x_i}{x_{n+1}} \\cdot\\left(1-\\frac{x_i}{x_{n+1}}\\right)}\\right]^2 \\leqslant\\left(\\sum_{i=1}^n \\frac{x_i}{x_{n+1}}\\right)\\left[\\sum_{i=1}^n\\left(1-\\frac{x_i}{x_{n+1}}\\right)\\right]= \\left(\\frac{1}{x_{n+1}} \\sum_{i=1}^n x_i\\right)\\left(n-\\frac{1}{x_{n+1}} \\sum_{i=1}^n x_i\\right)=n-1$.", + "remark": "", + "figures": [] +} \ No newline at end of file diff --git a/processed_dataset/proof/1241.json b/processed_dataset/proof/1241.json new file mode 100644 index 0000000000000000000000000000000000000000..356d6b7392f3ab039708aa45fd580655eb07145d --- /dev/null +++ b/processed_dataset/proof/1241.json @@ -0,0 +1,8 @@ +{ + "source_file": "./raw_volume-zh/volume4/exercise4.tex", + "problem_type": "proof", + "problem": "问题8. 设 $x, y, z, w \\in \\mathbf{R}^{+}, \\alpha 、 \\beta 、 \\gamma 、 \\theta$ 满足 $\\alpha+\\beta+\\gamma+\\theta=(2 k+1) \\pi, k \\in \\mathbf{Z}$. 求证:\n$$\n(x \\sin \\alpha+y \\sin \\beta+z \\sin \\gamma+w \\sin \\theta)^2 \\leqslant \\frac{(x y+z w)(x z+y w)(x w+y z)}{x y z w},\n$$\n当且仅当 $x \\cos \\alpha=y \\cos \\beta=z \\cos \\gamma=w \\cos \\theta$ 时等号成立.", + "solution": "设 $u=x \\sin \\alpha+y \\sin \\beta, v=z \\sin \\gamma+x \\sin \\theta$, 则 $u^2= (x \\sin \\alpha+y \\sin \\beta)^2 \\leqslant(x \\sin \\alpha+y \\sin \\beta)^2+(x \\cos \\alpha-y \\cos \\beta)^2=x^2+y^2- 2 x y \\cos (\\alpha+\\beta)$. 所以 $\\cos (\\alpha+\\beta) \\leqslant \\frac{x^2+y^2-u^2}{2 x y}$. 同理 $\\cos (\\gamma+\\theta) \\leqslant \\frac{z^2+w^2-v^2}{2 z w}$. 由假设 $\\cos (\\alpha+\\beta)+\\cos (\\gamma+\\theta)=0$, 则 $\\frac{u^2}{x y}+\\frac{v^2}{z w} \\leqslant \\frac{x^2+y^2}{x y}+ \\frac{z^2+w^2}{z w}$. 于是 $(u+v)^2=\\left(u \\cdot \\frac{\\sqrt{x y}}{\\sqrt{x y}}+v \\cdot \\frac{\\sqrt{z w}}{\\sqrt{z w}}\\right)^2 \\leqslant\\left(\\frac{u^2}{x y}+\\frac{v^2}{z w}\\right)(x y+ z w) \\leqslant(x y+z w)\\left(\\frac{x^2+y^2}{x y}+\\frac{z^2+w^2}{z w}\\right)$. 等号成立 $\\Leftrightarrow x \\cos \\alpha=y \\cos \\beta$, $z \\cos \\gamma=w \\cos \\theta, \\frac{u}{x y}=\\frac{v}{z w} \\Leftrightarrow x \\cos \\alpha=y \\cos \\beta=z \\cos \\gamma=w \\cos \\theta$.", + "remark": "", + "figures": [] +} \ No newline at end of file diff --git a/processed_dataset/proof/1242.json b/processed_dataset/proof/1242.json new file mode 100644 index 0000000000000000000000000000000000000000..eafca6944765af604d4bfa16f42d41ea1923c5fc --- /dev/null +++ b/processed_dataset/proof/1242.json @@ -0,0 +1,8 @@ +{ + "source_file": "./raw_volume-zh/volume4/exercise4.tex", + "problem_type": "proof", + "problem": "问题10. 设复数 $z_k=x_k+\\mathrm{i} y_k, k=1,2, \\cdots, n, x_i$ 和 $y_i$ 为实数, $\\mathrm{i}=\\sqrt{-1}$. 令 $r$ 表示 $\\sqrt{z_1^2+z_2^2+\\cdots+z_n^2}$ 的实部的绝对值,求证:\n$$\nr \\leqslant\\left|x_1\\right|+\\left|x_2\\right|+\\cdots+\\left|x_n\\right| \\text {. }\n$$", + "solution": "设 $a+\\mathrm{i} b=\\sqrt{\\sum_{i=1}^n z_i^2}, a, b \\in \\mathbf{R}$, 则 $a^2-b^2=\\sum_{k=1}^n x_k^2-\\sum_{k=1}^n y_k^2, a b= \\sum_{k=1}^n x_k y_k$. 若 $r=|a|>\\sum_{k=1}^n\\left|x_k\\right|$, 由于 $\\sum_{k=1}^n\\left|x_k\\right| \\geqslant\\left(\\sum_{k=1}^n x_k^2\\right)^{\\frac{1}{2}}$, 则 $|a|> \\left(\\sum_{k=1}^n x_k^2\\right)^{\\frac{1}{2}}$. 由柯西不等式, 得 $|a| \\cdot|b| \\leqslant\\left(\\sum_{k=1}^n x_k^2\\right)^{\\frac{1}{2}}\\left(\\sum_{k=1}^n y_k^2\\right)^{\\frac{1}{2}}$, 从而 $|b| \\leqslant \\left(\\sum_{k=1}^n y_k^2\\right)^{\\frac{1}{2}}$, 于是 $a^2=\\sum_{k=1}^n x_k^2+b^2-\\sum_{k=1}^n y_k^2 \\leqslant \\sum_{k=1}^n x_k^2$ 与 $|a|>\\left(\\sum_{k=1}^n x_k^2\\right)^{\\frac{1}{2}}$ 矛盾.", + "remark": "", + "figures": [] +} \ No newline at end of file diff --git a/processed_dataset/proof/1243.json b/processed_dataset/proof/1243.json new file mode 100644 index 0000000000000000000000000000000000000000..6ba7f490d142f3ae3601b819e47d9e8d04974776 --- /dev/null +++ b/processed_dataset/proof/1243.json @@ -0,0 +1,8 @@ +{ + "source_file": "./raw_volume-zh/volume4/exercise4.tex", + "problem_type": "proof", + "problem": "问题11. 设 $A_n=\\frac{a_1+a_2+\\cdots+a_n}{n}, a_i>0, i=1,2, \\cdots, n$. 求证:\n$$\n\\left(A_n-\\frac{1}{A_n}\\right)^2 \\leqslant \\frac{1}{n} \\sum_{i=1}^n\\left(a_i-\\frac{1}{a_i}\\right)^2 .\n$$", + "solution": "$n \\sum_{i=1}^n\\left(a_i-\\frac{1}{a_i}\\right)^2=n \\sum_{i=1}^n a_i^2+n \\sum_{i=1}^n \\frac{1}{a_i^2}-2 n^2 \\geqslant\\left(\\sum_{i=1}^n a_i\\right)^2+\\left(\\sum_{i=1}^n \\frac{1}{a_i}\\right)^2- 2 n^2 \\geqslant n^2\\left(A_n^2+\\frac{1}{A_n^2}-2\\right)=n^2\\left(A_n-\\frac{1}{A_n}\\right)^2$.", + "remark": "", + "figures": [] +} \ No newline at end of file diff --git a/processed_dataset/proof/1244.json b/processed_dataset/proof/1244.json new file mode 100644 index 0000000000000000000000000000000000000000..71e7179061fffc8a439019a9e93c5c5b2e30fe89 --- /dev/null +++ b/processed_dataset/proof/1244.json @@ -0,0 +1,8 @@ +{ + "source_file": "./raw_volume-zh/volume4/exercise4.tex", + "problem_type": "proof", + "problem": "问题14. 设 $\\frac{1}{2} \\leqslant p \\leqslant 1, a_i \\geqslant 0,0 \\leqslant b_i \\leqslant p$ 且 $\\sum_{i=1}^n a_i=\\sum_{i=1}^n b_i=1$, 求证:\n$$\n\\sum_{i=1}^n b_i \\prod_{\\substack{1 \\leqslant j \\leqslant n \\\\ j \\neq i}} a_j \\leqslant \\frac{p}{(n-1)^{n-1}} .\n$$", + "solution": "不妨设 $a_1 \\leqslant a_2 \\leqslant \\cdots \\leqslant a_n, b_1 \\geqslant b_2 \\geqslant \\cdots \\geqslant b_n$. 令 $A_i=\\prod_{\\substack{j=1 \\\\ j \\neq i}}^n a_j$, 则 $A_1 \\geqslant A_2 \\geqslant \\cdots \\geqslant A_n \\geqslant 0$. 由排序不等式, 得 $\\sum_{i=1}^n b_i A_i \\leqslant b_1 A_1+\\left(1-b_1\\right) A_2= p A_1+p A_2+\\left(-p+1-b_1\\right) A_2+\\left(b_1-p\\right) A_1=p\\left(A_1+A_2\\right)-\\left(p-b_1\\right)\\left(A_1-\\right. \\left.A_2\\right)+(1-2 p) A_2 \\leqslant p\\left(A_1+A_2\\right)$ (因为 $\\frac{1}{2} \\leqslant p \\leqslant 1$ ). 由 $A_{n-1} \\geqslant G_{n-1}$, 得 $A_1+A_2=a_3 a_4 \\cdots a_n\\left(a_2+a_1\\right) \\leqslant\\left(\\frac{1}{n-1} \\sum_{i=1}^n a_i\\right)^{n-1}=\\frac{1}{(n-1)^{n-1}}$, 所以 $\\sum_{i=1}^n b_i A_i \\leqslant \\frac{p}{(n-1)^{n-1}}$.", + "remark": "", + "figures": [] +} \ No newline at end of file diff --git a/processed_dataset/proof/1245.json b/processed_dataset/proof/1245.json new file mode 100644 index 0000000000000000000000000000000000000000..875ce903c2aa2ce23897ad7456cff570e9deb1b3 --- /dev/null +++ b/processed_dataset/proof/1245.json @@ -0,0 +1,8 @@ +{ + "source_file": "./raw_volume-zh/volume4/exercise4.tex", + "problem_type": "proof", + "problem": "问题17. 设 $u 、 v$ 是正实数, 对于给定的正整数 $n$, 求: $u 、 v$ 满足的充分必要条件, 使得存在实数 $a_1 \\geqslant a_2 \\geqslant \\cdots \\geqslant a_n>0$ 满足\n$$\n\\sum_{i=1}^n a_i=u, \\sum_{i=1}^n a_i^2=v,\n$$\n当这些数存在时,求 $a_1$ 的最大值与最小值.", + "solution": "若存在 $a_1, a_2, \\cdots, a_n$, 由柯西不等式, 得 $\\left(\\sum_{i=1}^n a_i\\right)^2 \\leqslant n \\sum_{i=1}^n a_i^2$. 又 $\\left(\\sum_{i=1}^n a_i\\right)^2 \\geqslant \\sum_{i=1}^n a_i^2$, 所以 $u 、 v$ 满足的必要条件是 $v \\leqslant u^2 \\leqslant n v \\cdots$ (1). 可以证明, 以上也为充分条件.\n若(1)成立, 取正数 $a_1=\\frac{u+\\sqrt{(n-1)\\left(n v-u^2\\right)}}{n}$, 则 $a_1 \\leqslant u$. \n若 $n>1$ , 再取 $a_2=a_3=\\cdots=a_n=\\frac{u-a_1}{n-1}$, 则 $\\sum_{i=1}^n a_i=u, \\sum_{i=1}^n a_i^2=v, a_1 \\geqslant \\frac{u-a_1}{n-1}$. 可以证明, $a_1$ 的最大值为 $\\frac{u+\\sqrt{(n-1)\\left(n v-u^2\\right)}}{n}$. 事实上, 若 $a_1> \\frac{u+\\sqrt{(n-1)\\left(n v-u^2\\right)}}{n}$, 则 $n>1$, 且 $n a_1^2-2 u a_1+u^2-(n-1) v>0$, 即 $(n-$ 1) $\\left(v-a_1^2\\right)<\\left(u-a_1\\right)^2 \\cdots$ (2). 若有 $a_2 \\geqslant a_3 \\geqslant \\cdots \\geqslant a_n \\geqslant 0$, 使 $\\sum_{k=2}^n a_i=u-a_1$, $\\sum_{k=2}^n a_k^2=v-a_1^2$, 则由柯西不等式, 得 $\\left(u-a_1\\right)^2 \\leqslant(n-1)\\left(v-a_1^2\\right)$, 矛盾.\n以下求 $a_1$ 的最小值, 设 $a_1, a_2, \\cdots, a_n$ 满足所要之条件, 则对于任何 $1 \\leqslant i, j \\leqslant n$, 有 $a_i^2+a_j^2 \\leqslant\\left(a_i+a_j\\right)^2 \\cdots$ (3). $a_i^2+a_j^2 \\leqslant a_i^2+a_j^2+2\\left(a_1-a_i\\right)\\left(a_1-a_j\\right)=a_1^2+ \\left(a_i+a_j-a_1\\right)^2 \\cdots$ (4). 若 $n=1$, 显然 $u^2=v$, 且 $a_1=u$, 对 $n \\geqslant 2$, 显然 $\\frac{u}{n} \\leqslant a_1 \\leqslant u$. 若存在 $k \\in\\{1,2, \\cdots, n-1\\}$, 使得 $a_1 \\leqslant \\frac{u}{k}$, 则当 $a_i+a_j \\leqslant a_1$ 时, 使用(3), $a_i+a_j>a_1$ 时, 使用(4). 重复上述步骤有限次, 得 $v=\\sum_{i=1}^n a_i^2 \\leqslant k a_1^2+ \\left(u-k a_1\\right)^2 \\cdots$ (5). 进一步, 若 $\\frac{u}{k+1} \\leqslant a_1 \\leqslant \\frac{u}{k}$, 由(5)可得 $v=\\sum_{i=1}^n a_i^2 \\leqslant k a_1^2+ \\left(u-k a_1\\right)^2 \\leqslant \\frac{u^2}{k} \\cdots$ (6). 由(1)知, $\\frac{u^2}{n} \\leqslant v \\leqslant u^2$, 显然 $v=\\frac{u^2}{n}$ 的充要条件为 $a_1= a_2=\\cdots=a_n=\\frac{u}{n}$. 若 $v>\\frac{u^2}{n}$, 则存在 $k \\in\\{1,2, \\cdots, n-1\\}$, 使得 $\\frac{u^2}{k+1}< v \\leqslant \\frac{u^2}{k}$. 可以证明 $a_1 \\geqslant \\frac{k u+\\sqrt{k\\left[(k+1) v-u^2\\right]}}{k(k+1)} \\cdots$ (7). 如果(7)不成立, 则存在 $a_1, a_2, \\cdots, a_n$ 满足题设中的条件, 且 $a_1<\\frac{k u+\\sqrt{k\\left[(k+1) v-u^2\\right]}}{k(k+1)} \\cdots$ (8). 由于 $0 飞(k+1) v-u^2 \\leqslant \\frac{u^2}{k}$, 所以 $a_1 \\leqslant \\frac{u}{k}$. 由(5)可知 $v \\leqslant k a_1^2+\\left(u-k a_1\\right)^2$, 即 $k(k+1) a_1^2-2 k u a_1+u^2-v \\geqslant 0$. 再由 $k^2 u^2-k(k+1)\\left(u^2-v\\right)=k[(k+1) v- \\left.u^2\\right]>0$ 和(8)可推出 $a_1 \\leqslant \\frac{k u-\\sqrt{k\\left[(k+1) v-u^2\\right]}}{k(k+1)}<\\frac{u}{k+1}$. 于是存在 $k+ 1 \\leqslant m \\leqslant n-1$, 使得 $\\frac{u}{m+1} \\leqslant a_1<\\frac{u}{m}$. 由(6)可得 $v \\leqslant \\frac{u^2}{m} \\leqslant \\frac{u^2}{k+1}$ 与 $v>\\frac{u^2}{k+1}$ 矛盾.\n所以 (7)成立.\n另一方面, 在 $\\frac{u^2}{k+1}\\alpha$ 对所有正实数 $x 、 y 、 z$ 成立.", + "solution": "解法 1 令 $x=y, z \\rightarrow 0$, 则原式左端 $\\rightarrow 2$, 因此, 若 $\\alpha>2$, 将出现矛盾, 故 $\\alpha \\leqslant 2$.\n下面证明: $\\frac{x}{\\sqrt{y^2+z^2}}+\\frac{y}{\\sqrt{z^2+x^2}}+\\frac{z}{\\sqrt{x^2+y^2}}>2$.\n不妨设 $x \\leqslant y \\leqslant z$, 我们设法证明\n$$\n\\frac{x}{\\sqrt{y^2+z^2}}+\\frac{y}{\\sqrt{z^2+x^2}}+\\frac{z}{\\sqrt{x^2+y^2}}>\\frac{\\sqrt{x^2+y^2}}{\\sqrt{x^2+z^2}}+\\frac{\\sqrt{x^2+z^2}}{\\sqrt{x^2+y^2}} .\n$$\n将 $\\frac{y}{\\sqrt{z^2+x^2}} 、 \\frac{z}{\\sqrt{x^2+y^2}}$ 移到右边, 即证\n$$\n\\frac{x}{\\sqrt{y^2+z^2}}>\\frac{\\sqrt{x^2+y^2}-y}{\\sqrt{x^2+z^2}}+\\frac{\\sqrt{x^2+z^2}-z}{\\sqrt{x^2+y^2}} \\text {. }\n$$\n也即\n$$\n\\frac{x}{\\sqrt{y^2+z^2}}>\\frac{x^2}{\\sqrt{x^2+y^2}\\left(\\sqrt{x^2+y^2}+y\\right)}+\\frac{x^2}{\\sqrt{x^2+y^2}\\left(\\sqrt{x^2+z^2}+z\\right)} .\n$$\n两边约去 $x$, 并且由于 $\\sqrt{x^2+y^2}+y>2 y, \\sqrt{x^2+z^2}+z>2 z$, 所以, 只要证明\n$$\n\\frac{1}{\\sqrt{y^2+z^2}} \\geqslant \\frac{x}{2 y \\sqrt{x^2+z^2}}+\\frac{x}{2 z \\sqrt{x^2+y^2}} .\n$$\n由于 $\\frac{x}{\\sqrt{x^2+z^2}}=\\frac{1}{\\sqrt{1+\\left(\\frac{z}{x}\\right)^2}}$, 所以 $\\frac{x}{\\sqrt{x^2+z^2}}$ 随 $x$ 的增大而增大.\n同样, $\\frac{x}{\\sqrt{x^2+y^2}}$ 也随 $x$ 的增大而增大.\n所以我们只须考虑 $x=y$ 时的情况.\n令 $x=y$, 即证\n$$\n\\frac{1}{\\sqrt{y^2+z^2}} \\geqslant \\frac{1}{2 \\sqrt{y^2+z^2}}+\\frac{1}{2 \\sqrt{2} z}\n$$\n也就是 $\\frac{1}{2 \\sqrt{y^2+z^2}} \\geqslant \\frac{1}{2 \\sqrt{2} z}$, 即证 $\\sqrt{2} z \\geqslant \\sqrt{y^2+z^2}$.\n这是显然成立的.\n因此,\n$$\n\\frac{x}{\\sqrt{y^2+z^2}}+\\frac{y}{\\sqrt{x^2+z^2}}+\\frac{z}{\\sqrt{x^2+y^2}}>\\frac{\\sqrt{x^2+y^2}}{\\sqrt{x^2+z^2}}+\\frac{\\sqrt{x^2+z^2}}{\\sqrt{x^2+y^2}} \\geqslant 2 \\text {. }\n$$\n故 $\\alpha_{\\max }=2$.", + "remark": "", + "figures": [] +} \ No newline at end of file diff --git a/processed_dataset/proof/1261.json b/processed_dataset/proof/1261.json new file mode 100644 index 0000000000000000000000000000000000000000..1d9c13ebb50d34c70dc0cb979ecb788b80477bb9 --- /dev/null +++ b/processed_dataset/proof/1261.json @@ -0,0 +1,8 @@ +{ + "source_file": "./raw_volume-zh/volume5/chapter1.tex", + "problem_type": "proof", + "problem": "例9. 求最大的实数 $\\alpha$, 使得 $\\frac{x}{\\sqrt{y^2+z^2}}+\\frac{y}{\\sqrt{x^2+z^2}}+\\frac{z}{\\sqrt{x^2+y^2}}>\\alpha$ 对所有正实数 $x 、 y 、 z$ 成立.", + "solution": "解法 2 同样,我们来证明\n$$\n\\frac{x}{\\sqrt{y^2+z^2}}+\\frac{y}{\\sqrt{z^2+x^2}}+\\frac{z}{\\sqrt{x^2+y^2}}>2 .\n$$\n设\n$$\n\\frac{x}{\\sqrt{y^2+z^2}} \\geqslant \\frac{2 x^a}{x^a+y^a+z^a} . \\label{eq1}\n$$\n其中 $a$ 为待定参数.\n注意到式\\ref{eq1}等价于 $\\left(x^a+y^a+z^a\\right)^2 \\geqslant 4 x^{2 a-2}\\left(y^2+z^2\\right)$.\n上式左边 $\\geqslant 4 x^a\\left(y^a+z^a\\right)$, 故只须保证\n$$\ny^a+z^a \\geqslant x^{a-2}\\left(y^2+z^2\\right) .\n$$\n不难发现, 取 $a=2$ 即可.\n于是\n$$\n\\sum \\frac{x}{\\sqrt{y^2+z^2}} \\geqslant \\sum \\frac{2 x^2}{x^2+y^2+z^2}=2 \\text {. }\n$$\n而等号显然不可能成立, 所以 $\\sum \\frac{x}{\\sqrt{y^2+z^2}}>2$.\n故 $\\alpha_{\\max }=2$.", + "remark": "", + "figures": [] +} \ No newline at end of file diff --git a/processed_dataset/proof/1262.json b/processed_dataset/proof/1262.json new file mode 100644 index 0000000000000000000000000000000000000000..6d05e7da6029feeb16304cb94051c180ab44758c --- /dev/null +++ b/processed_dataset/proof/1262.json @@ -0,0 +1,8 @@ +{ + "source_file": "./raw_volume-zh/volume5/chapter1.tex", + "problem_type": "proof", + "problem": "例10. 设非负实数 $a_1, a_2, \\cdots, a_n$ 与 $b_1, b_2, \\cdots, b_n$ 同时满足以下条件:\n(1) $\\sum_{i=1}^n\\left(a_i+b_i\\right)=1$;\n(2) $\\sum_{i=1}^n i\\left(a_i-b_i\\right)=0$;\n(3) $\\sum_{i=1}^n i^2\\left(a_i+b_i\\right)=10$.\n求证: 对任意 $1 \\leqslant k \\leqslant n$, 都有 $\\max \\left\\{a_k, b_k\\right\\} \\leqslant \\frac{10}{10+k^2}$. (2010 年中国西部数学奥林匹克)", + "solution": "证明:对任意 $1 \\leqslant k \\leqslant n$, 有\n$$\n\\begin{aligned}\n\\left(k a_k\\right)^2 & \\leqslant\\left(\\sum_{i=1}^n i a_i\\right)^2=\\left(\\sum_{i=1}^n i b_i\\right)^2 \\leqslant\\left(\\sum_{i=1}^n i^2 b_i\\right) \\cdot\\left(\\sum_{i=1}^n b_i\\right) \\\\\n& =\\left(10-\\sum_{i=1}^n i^2 a_i\\right) \\cdot\\left(1-\\sum_{i=1}^n a_i\\right) \\\\\n& \\leqslant\\left(10-k^2 a_k\\right) \\cdot\\left(1-a_k\\right)=10-\\left(10+k^2\\right) a_k+k^2 a_k^2,\n\\end{aligned}\n$$\n从而 $a_k \\leqslant \\frac{10}{10+k^2}$.\n同理有 $b_k \\leqslant \\frac{10}{10+k^2}$, 所以 $\\max \\left\\{a_k, b_k\\right\\} \\leqslant \\frac{10}{10+k^2}$.", + "remark": "", + "figures": [] +} \ No newline at end of file diff --git a/processed_dataset/proof/1263.json b/processed_dataset/proof/1263.json new file mode 100644 index 0000000000000000000000000000000000000000..9d5e2a31981fd01b4499f031cca6329513143719 --- /dev/null +++ b/processed_dataset/proof/1263.json @@ -0,0 +1,8 @@ +{ + "source_file": "./raw_volume-zh/volume5/chapter1.tex", + "problem_type": "proof", + "problem": "例11. 正实数 $x 、 y 、 z$ 满足 $x y z \\geqslant 1$, 证明\n$$\n\\frac{x^5-x^2}{x^5+y^2+z^2}+\\frac{y^5-y^2}{y^5+z^2+x^2}+\\frac{z^5-z^2}{z^5+x^2+y^2} \\geqslant 0 \\text {. }\n$$", + "solution": "证明:原不等式可变形为\n$$\n\\frac{x^2+y^2+z^2}{x^5+y^2+z^2}+\\frac{x^2+y^2+z^2}{y^5+z^2+x^2}+\\frac{x^2+y^2+z^2}{z^5+x^2+y^2} \\leqslant 3 .\n$$\n由柯西不等式及题设条件 $x y z \\geqslant 1$, 得\n$\\left(x^5+y^2+z^2\\right)\\left(y z+y^2+z^2\\right) \\geqslant\\left(x^2(x y z)^{\\frac{1}{2}}+y^2+z^2\\right)^2 \\geqslant\\left(x^2+y^2+z^2\\right)^2$, 即\n$$\n\\begin{aligned}\n& \\frac{x^2+y^2+z^2}{x^5+y^2+z^2} \\leqslant \\frac{y z+y^2+z^2}{x^2+y^2+z^2}, \\\\\n& \\frac{x^2+y^2+z^2}{y^5+z^2+x^2} \\leqslant \\frac{z x+z^2+x^2}{x^2+y^2+z^2}, \\\\\n& \\frac{x^2+y^2+z^2}{z^5+x^2+y^2} \\leqslant \\frac{x y+x^2+y^2}{x^2+y^2+z^2},\n\\end{aligned}\n$$\n把上面三个不等式相加, 并利用 $x^2+y^2+z^2 \\geqslant x y+y z+z x$, 得\n$$\n\\frac{x^2+y^2+z^2}{x^5+y^2+z^2}+\\frac{x^2+y^2+z^2}{y^5+z^2+x^2}+\\frac{x^2+y^2+z^2}{z^5+x^2+y^2} \\leqslant 2+\\frac{x y+y z+z x}{x^2+y^2+z^2} \\leqslant 3 .\n$$\n摩尔多瓦选手 Boreico Iurie 的解法获得了特别奖.\n他的证法如下: 因为\n$$\n\\frac{x^5-x^2}{x^5+y^2+z^2}-\\frac{x^5-x^2}{x^3\\left(x^2+y^2+z^2\\right)}=\\frac{x^2\\left(x^3-1\\right)^2\\left(y^2+z^2\\right)}{x^3\\left(x^5+y^2+z^2\\right)\\left(x^2+y^2+z^2\\right)} \\geqslant 0,\n$$\n所以\n$$\n\\begin{aligned}\n\\sum_{c y c} \\frac{x^5-x^2}{x^5+y^2+z^2} & \\geqslant \\sum_{c y c} \\frac{x^5-x^2}{x^3\\left(x^2+y^2+z^2\\right)}=\\frac{1}{x^2+y^2+z^2} \\sum_{c y c}\\left(x^2-\\frac{1}{x}\\right) \\\\\n& \\left.\\geqslant \\frac{1}{x^2+y^2+z^2} \\sum_{c y c}\\left(x^2-y z\\right) \\text { (因为 } x y z \\geqslant 1\\right) \\\\\n& \\geqslant 0 .\n\\end{aligned}\n$$", + "remark": "", + "figures": [] +} \ No newline at end of file diff --git a/processed_dataset/proof/1264.json b/processed_dataset/proof/1264.json new file mode 100644 index 0000000000000000000000000000000000000000..d78e819e1e4468f6366dbd6dc4d724f2de4faa6b --- /dev/null +++ b/processed_dataset/proof/1264.json @@ -0,0 +1,8 @@ +{ + "source_file": "./raw_volume-zh/volume5/chapter1.tex", + "problem_type": "proof", + "problem": "例12. 若 $x, y \\in \\mathbf{R}, y \\geqslant 0$, 且 $y(y+1) \\leqslant(x+1)^2$, 求证: $y(y-1) \\leqslant x^2$.", + "solution": "证明:若 $0 \\leqslant y \\leqslant 1$, 则 $y(y-1) \\leqslant 0 \\leqslant x^2$.\n若 $y>1$, 由题设知\n$$\n\\begin{gathered}\ny(y+1) \\leqslant(x+1)^2, \\\\\ny \\leqslant \\sqrt{(x+1)^2+\\frac{1}{4}}-\\frac{1}{2} .\n\\end{gathered}\n$$\n要证明 $y(y-1) \\leqslant x^2$, 即只需证明\n$$\n\\begin{array}{lc} \n& \\sqrt{(x+1)^2+\\frac{1}{4}}-\\frac{1}{2} \\leqslant \\sqrt{x^2+\\frac{1}{4}}+\\frac{1}{2}, \\\\\n\\Leftrightarrow & (x+1)^2+\\frac{1}{4} \\leqslant x^2+\\frac{1}{4}+2 \\sqrt{x^2+\\frac{1}{4}}+1 \\\\\n\\Leftrightarrow & 2 x \\leqslant 2 \\sqrt{x^2+\\frac{1}{4}} .\n\\end{array}\n$$\n最后这个不等式是显然的, 从而原不等式得证.", + "remark": "", + "figures": [] +} \ No newline at end of file diff --git a/processed_dataset/proof/1265.json b/processed_dataset/proof/1265.json new file mode 100644 index 0000000000000000000000000000000000000000..032bc1e2a7159b2e2c85ae63abde8256af31b0de --- /dev/null +++ b/processed_dataset/proof/1265.json @@ -0,0 +1,8 @@ +{ + "source_file": "./raw_volume-zh/volume5/chapter1.tex", + "problem_type": "proof", + "problem": "例13. 设 $a, b, c \\in \\mathbf{R}^{+}$, 求证:\n$$\na+b+c-3 \\sqrt[3]{a b c} \\geqslant a+b-2 \\sqrt{a b} .\n$$", + "solution": "证明:注意到\n$$\na+b+c-3 \\sqrt[3]{a b c} \\geqslant a+b-2 \\sqrt{a b}\n$$\n$\\Leftrightarrow \\quad c+2 \\sqrt{a b} \\geqslant 3 \\sqrt[3]{a b c}$.\n因为\n$$\n\\begin{aligned}\nc+2 \\sqrt{a} \\bar{b} & =c+\\sqrt{a b}+\\sqrt{a b} \\\\\n& \\geqslant 3 \\sqrt[3]{c \\sqrt{a b} \\sqrt{a b}} \\\\\n& =3 \\sqrt[3]{a b c},\n\\end{aligned}\n$$\n从而\n$$\na+b+c-3 \\sqrt[3]{a b c} \\geqslant a+b-2 \\sqrt{a b} .\n$$\n说明在不等式的证明中, 分析法和综合法有时需交替使用.\n本题在用分析法得到 $c+2 \\sqrt{a b} \\geqslant 3 \\sqrt[3]{a b c}$, 再用分析法继续证明下去的话, 会使问题变得复杂, 此时结合综合法便使问题迎刃而解了.", + "remark": "", + "figures": [] +} \ No newline at end of file diff --git a/processed_dataset/proof/1266.json b/processed_dataset/proof/1266.json new file mode 100644 index 0000000000000000000000000000000000000000..dfb56c3e033b644df7400da3a0f3cd800f448a05 --- /dev/null +++ b/processed_dataset/proof/1266.json @@ -0,0 +1,8 @@ +{ + "source_file": "./raw_volume-zh/volume5/chapter1.tex", + "problem_type": "proof", + "problem": "例14. 已知 $n \\in \\mathbf{N}_{+}$, 求证:\n$$\n\\frac{1}{n+1}\\left(1+\\frac{1}{3}+\\cdots+\\frac{1}{2 n-1}\\right) \\geqslant \\frac{1}{n}\\left(\\frac{1}{2}+\\frac{1}{4}+\\cdots+\\frac{1}{2 n}\\right) . \\label{(1)}\n$$", + "solution": "证明:要证明(1), 我们只要证\n$$\nn\\left(1+\\frac{1}{3}+\\cdots+\\frac{1}{2 n-1}\\right) \\geqslant(n+1)\\left(\\frac{1}{2}+\\frac{1}{4}+\\cdots+\\frac{1}{2 n}\\right) . \\label{(2)}\n$$\n(2)的左边为\n$$\n\\frac{n}{2}+\\frac{n}{2}+n\\left(\\frac{1}{3}+\\cdots+\\frac{1}{2 n-1}\\right), \\label{(3)}\n$$\n(2)的右边为\n$$\n\\begin{aligned}\n& n\\left(\\frac{1}{2}+\\frac{1}{4}+\\cdots+\\frac{1}{2 n}\\right)+\\left(\\frac{1}{2}+\\frac{1}{4}+\\cdots+\\frac{1}{2 n}\\right) \\\\\n= & \\frac{n}{2}+n\\left(\\frac{1}{4}+\\cdots+\\frac{1}{2 n}\\right)+\\left(\\frac{1}{2}+\\frac{1}{4}+\\cdots+\\frac{1}{2 n}\\right) .\n\\end{aligned} \\label{(4)}\n$$\n比较(3)式和(4)式,若有\n$$\n\\frac{n}{2} \\geqslant \\frac{1}{2}+\\frac{1}{4}+\\cdots+\\frac{1}{2 n}, \\label{(5)}\n$$\n及\n$$\n\\frac{1}{3}+\\frac{1}{5}+\\cdots+\\frac{1}{2 n-1} \\geqslant \\frac{1}{4}+\\cdots+\\frac{1}{2 n}, \\label{(6)}\n$$\n则(2)得证.\n而(5)、(6)两式显然成立, 因此(1)得证.", + "remark": "", + "figures": [] +} \ No newline at end of file diff --git a/processed_dataset/proof/1267.json b/processed_dataset/proof/1267.json new file mode 100644 index 0000000000000000000000000000000000000000..bb528b412c52a1231b74aa90445280ff26b6b35c --- /dev/null +++ b/processed_dataset/proof/1267.json @@ -0,0 +1,8 @@ +{ + "source_file": "./raw_volume-zh/volume5/chapter1.tex", + "problem_type": "proof", + "problem": "例15. 已知 $a, b, c \\in \\mathbf{R}^{+}, a b c=1$. 求证:\n$$\n(a+b)(b+c)(c+a) \\geqslant 4(a+b+c-1) .\n$$", + "solution": "分析:想法是把 $a$ 当作参数, 将其看成是关于 $b+c$ 的一元二次方程, 用判别式的方法来证明.\n证明不妨设 $a \\geqslant 1$, 则原不等式等价于\n$$\na^2(b+c)+b^2(c+a)+c^2(a+b)+6 \\geqslant 4(a+b+c), \\label{(1)}\n$$\n即\n$$\n\\begin{aligned}\n& \\left(a^2-1\\right)(b+c)+b^2(c+a)+c^2(a+b)+6 \\\\\n\\geqslant & 4 a+3(b+c)\n\\end{aligned}\n$$\n由于\n$$\n(a+1)(b+c) \\geqslant 2 \\sqrt{a} \\cdot 2 \\sqrt{b c}=4,\n$$\n所以如果我们能够证明\n$$\n4(a-1)+b^2(c+a)+c^2(a+b)+6 \\geqslant 4 a+3(b+c), \\label{(2)}\n$$\n则(1)式成立.\n而(2)等价于\n$$\n2+a\\left(b^2+c^2\\right)+b c(b+c)-3(b+c) \\geqslant 0,\n$$\n故只需证\n$$\n\\frac{a}{2}(b+c)^2+(b c-3)(b+c)+2 \\geqslant 0 .\n$$\n记\n$$\nf(x)=\\frac{a}{2} \\cdot x^2+(b c-3) x+2,\n$$\n则其判别式\n$$\n\\Delta=(b c-3)^2-4 a .\n$$\n我们只要证明 $\\Delta \\leqslant 0$ 即可, 这相当于\n$$\n\\left(\\frac{1}{a}-3\\right)^2-4 a \\leqslant 0 .\n$$\n即\n$$\n\\begin{gathered}\n1-6 a+9 a^2-4 a^3 \\leqslant 0 . \\\\\n(a-1)^2(4 a-1) \\geqslant 0 . \\label{(3)}\n\\end{gathered}\n$$\n也即由 $a \\geqslant 1$, (3)显然成立, 进而(1)成立.\n由上知等号在 $a=b=c=1$ 时成立.", + "remark": "", + "figures": [] +} \ No newline at end of file diff --git a/processed_dataset/proof/1268.json b/processed_dataset/proof/1268.json new file mode 100644 index 0000000000000000000000000000000000000000..5f3be046e8c272e23d4fb6e1b6f48c9ffa024027 --- /dev/null +++ b/processed_dataset/proof/1268.json @@ -0,0 +1,8 @@ +{ + "source_file": "./raw_volume-zh/volume5/chapter1.tex", + "problem_type": "proof", + "problem": "例16. 设 $x 、 y 、 z$ 是 3 个不全为零的实数, 求 $\\frac{x y+2 y z}{x^2+y^2+z^2}$ 的最大值.", + "solution": "分析:欲求 $\\frac{\\dot{x} y+2 y z}{x^2+y^2+z^2}$ 的最大值, 只需先证明存在一个常数 $c$, 使\n$$\n\\frac{x y+2 y z}{x^2+y^2+z^2} \\leqslant c, \\label{(1)}\n$$\n且 $x 、 y 、 z$ 取某组数时, 等号成立.\n(1)式可化为 $x^2+y^2+z^2 \\geqslant \\frac{1}{c}(x y+2 y z)$. 由于右边两项为 $x y$ 和 $2 y z$, 所以左边的 $y^2$ 需拆成两项 $\\alpha y^2$ 和 $(1-\\alpha) y^2$. 由\n$$\n\\begin{gathered}\nx^2+\\alpha y^2 \\geqslant 2 \\sqrt{\\alpha} x y, \\\\\n(1-\\alpha) y^2+z^2 \\geqslant 2 \\sqrt{1-\\alpha} y z,\n\\end{gathered}\n$$\n又由 $\\frac{2 \\sqrt{1-\\alpha}}{2 \\sqrt{\\alpha}}=2$, 得 $\\alpha=\\frac{1}{5}$.\n从而解 因为\n$$\nx^2+y^2+z^2 \\geqslant \\frac{2}{\\sqrt{5}}(x y+2 y z) .\n$$\n$$\n\\begin{aligned}\n& x^2+\\frac{1}{5} y^2 \\geqslant \\frac{2}{\\sqrt{5}} x y, \\\\\n& \\frac{4}{5} y^2+z^2 \\geqslant \\frac{4}{\\sqrt{5}} y z,\n\\end{aligned}\n$$\n所以\n$$\nx^2+y^2+z^2 \\geqslant \\frac{2}{\\sqrt{5}}(x y+2 y z),\n$$\n即\n$$\n\\frac{x y+2 y z}{x^2+y^2+z^2} \\leqslant \\frac{\\sqrt{5}}{2} \\text {. }\n$$\n当 $x=1, y=\\sqrt{5}, z=2$ 时, 等号成立.\n所以, 欲求的最大值为 $\\frac{\\sqrt{5}}{2}$.", + "remark": "", + "figures": [] +} \ No newline at end of file diff --git a/processed_dataset/proof/1269.json b/processed_dataset/proof/1269.json new file mode 100644 index 0000000000000000000000000000000000000000..65ae8d8012a29ac6b9f2f24675938420f8e103d6 --- /dev/null +++ b/processed_dataset/proof/1269.json @@ -0,0 +1,8 @@ +{ + "source_file": "./raw_volume-zh/volume5/chapter1.tex", + "problem_type": "proof", + "problem": "例18. (Ostrowski) 设实数 $a_1, a_2, \\cdots, a_n$ 与 $b_1, b_2, \\cdots, b_n$ 不成比例.\n实数 $x_1, x_2, \\cdots, x_n$ 满足:\n$$\n\\left\\{\\begin{array}{l}\na_1 x_1+a_2 x_2+\\cdots+a_n x_n=0, \\\\\nb_1 x_1+b_2 x_2+\\cdots+b_n x_n=1 .\n\\end{array}\\right.\n$$\n求证: $x_1^2+x_2^2+\\cdots+x_n^2 \\geqslant \\frac{\\sum_{i=1}^n a_i^2}{\\sum_{i=1}^n a_i^2 \\cdot \\sum_{i=1}^n b_i^2-\\left(\\sum_{i=1}^n a_i b_i\\right)^2}$.", + "solution": "证法 1 设 $\\sum_{i=1}^n x_i^2=\\sum_{i=1}^n x_i^2+\\alpha \\cdot \\sum_{i=1}^n a_i x_i+\\beta\\left(\\sum_{i=1}^n b_i x_i-1\\right)$,\n其中 $\\alpha 、 \\beta$ 为待定系数.\n于是\n$$\n\\begin{aligned}\n\\sum_{i=1}^n x_i^2 & =\\sum_{i=1}^n\\left(x_i+\\frac{\\alpha a_i+\\beta b_i}{2}\\right)^2-\\sum_{i=1}^n \\frac{\\left(\\alpha a_i+\\beta b_i\\right)^2}{4}-\\beta \\\\\n& \\geqslant-\\sum_{i=1}^n \\frac{\\left(\\alpha a_i+\\beta b_i\\right)^2}{4}-\\beta .\n\\end{aligned}\n$$\n上述不等式等号成立, 当且仅当\n$$\nx_i=-\\frac{\\alpha a_i+\\beta b_i}{2}(i=1,2, \\cdots, n) . \\label{(1)}\n$$\n将(1)式代回 $\\sum_{i=1}^n a_i x_i=0$ 及 $\\sum_{i=1}^n b_i x_i=1$ 中, 有:\n$$\n\\left\\{\\begin{array}{l}\n-\\frac{1}{2} \\alpha A-\\frac{1}{2} \\beta C=0 \\\\\n-\\frac{1}{2} \\alpha C-\\frac{1}{2} \\beta B=1\n\\end{array}\\right.\n$$\n其中, $A=\\sum_{i=1}^n a_i^2, B=\\sum_{i=1}^n b_i^2, C=\\sum_{i=1}^n a_i b_i$. 因此,\n$$\n\\alpha=\\frac{2 C}{A B-C^2}, \\beta=-\\frac{2 A}{A B-C^2} .\n$$\n故\n$$\n\\sum_{i=1}^n x_i^2 \\geqslant-\\sum_{i=1}^n\\left(\\frac{\\alpha a_i+\\beta b_i}{2}\\right)^2-\\beta=\\frac{A}{A B-C^2} .\n$$", + "remark": "", + "figures": [] +} \ No newline at end of file diff --git a/processed_dataset/proof/1270.json b/processed_dataset/proof/1270.json new file mode 100644 index 0000000000000000000000000000000000000000..91df92748580d8d0fe6d121cbe8d7e3b60847f90 --- /dev/null +++ b/processed_dataset/proof/1270.json @@ -0,0 +1,8 @@ +{ + "source_file": "./raw_volume-zh/volume5/chapter1.tex", + "problem_type": "proof", + "problem": "例18. (Ostrowski) 设实数 $a_1, a_2, \\cdots, a_n$ 与 $b_1, b_2, \\cdots, b_n$ 不成比例.\n实数 $x_1, x_2, \\cdots, x_n$ 满足:\n$$\n\\left\\{\\begin{array}{l}\na_1 x_1+a_2 x_2+\\cdots+a_n x_n=0, \\\\\nb_1 x_1+b_2 x_2+\\cdots+b_n x_n=1 .\n\\end{array}\\right.\n$$\n求证: $x_1^2+x_2^2+\\cdots+x_n^2 \\geqslant \\frac{\\sum_{i=1}^n a_i^2}{\\sum_{i=1}^n a_i^2 \\cdot \\sum_{i=1}^n b_i^2-\\left(\\sum_{i=1}^n a_i b_i\\right)^2}$.", + "solution": "证法 2 由 Cauchy 不等式可得, 对任意 $t \\in \\mathbf{R}$,\n$$\n\\left[\\sum_{i=1}^n\\left(a_i t+b_i\\right)^2\\right] \\cdot\\left(x_1^2+x_2^2+\\cdots+x_n^2\\right) \\geqslant\\left[\\sum_{i=1}^n\\left(a_i t+b_i\\right) x_i\\right]^2=1,\n$$\n即\n$$\n\\left(x_1^2+x_2^2+\\cdots+x_n^2\\right)\\left(A t^2+2 C t+B\\right)-1 \\geqslant 0\n$$\n恒成立.\n由判别式 (关于 $t$ 的) $\\Delta \\leqslant 0$, 即有:\n$$\n\\sum_{i=1}^n x_i^2 \\geqslant \\frac{A}{A B-C^2} \\text {. }\n$$", + "remark": "", + "figures": [] +} \ No newline at end of file diff --git a/processed_dataset/proof/1271.json b/processed_dataset/proof/1271.json new file mode 100644 index 0000000000000000000000000000000000000000..3810ee1a963c934a6fe992afcccae67e5efad693 --- /dev/null +++ b/processed_dataset/proof/1271.json @@ -0,0 +1,8 @@ +{ + "source_file": "./raw_volume-zh/volume5/chapter1.tex", + "problem_type": "proof", + "problem": "例18. (Ostrowski) 设实数 $a_1, a_2, \\cdots, a_n$ 与 $b_1, b_2, \\cdots, b_n$ 不成比例.\n实数 $x_1, x_2, \\cdots, x_n$ 满足:\n$$\n\\left\\{\\begin{array}{l}\na_1 x_1+a_2 x_2+\\cdots+a_n x_n=0, \\\\\nb_1 x_1+b_2 x_2+\\cdots+b_n x_n=1 .\n\\end{array}\\right.\n$$\n求证: $x_1^2+x_2^2+\\cdots+x_n^2 \\geqslant \\frac{\\sum_{i=1}^n a_i^2}{\\sum_{i=1}^n a_i^2 \\cdot \\sum_{i=1}^n b_i^2-\\left(\\sum_{i=1}^n a_i b_i\\right)^2}$.", + "solution": "证法 3 (综合运用上述两种方法)\n由条件, 对任意 $\\lambda \\in \\mathbf{R}$, 有 $\\sum_{i=1}^n\\left(b_i-\\lambda a_i\\right) x_i=1$.\n利用 Cauchy 不等式可得\n$$\n\\sum_{i=1}^n x_i^2 \\cdot \\sum_{i=1}^n\\left(b_i-\\lambda a_i\\right)^2 \\geqslant\\left[\\sum_{i=1}^n\\left(b_i-\\lambda a_i\\right) x_i\\right]^2=1 .\n$$\n所以\n$$\n\\sum_{i=1}^n x_i^2 \\geqslant \\frac{1}{B+\\lambda^2 A-2 \\lambda C} .\n$$\n我们的目标是证明\n$$\n\\sum_{i=1}^n x_i^2 \\geqslant \\frac{1}{B-\\frac{C^2}{A}},\n$$\n因此, 只需即\n$$\n\\lambda^2 A-2 \\lambda C \\leqslant-\\frac{C^2}{A} .\n$$\n$\\lambda^2 A^2-2 \\lambda A C+C^2 \\leqslant 0$.\n取 $\\lambda=\\frac{C}{A}$ 即可满足上述条件.\n说明 2 可以从本题证明 Fan-Todd 定理:\n设 $a_k 、 b_k(k=1,2, \\cdots, n)$ 为两组不成比例的实数列, 已知 $a_i b_k \\neq a_k b_i(i \\neq k)$, 则\n$$\n\\frac{\\sum_{k=1}^n a_k^2}{\\sum_{k=1}^n a_k^2 \\sum_{k=1}^n b_k^2-\\left(\\sum_{k=1}^n a_k b_k\\right)^2} \\leqslant\\left(\\mathrm{C}_n^2\\right)^{-2} \\cdot \\sum_{k=1}^n\\left(\\sum_{i \\neq k} \\frac{a_k}{a_i b_k-a_k b_i}\\right)^2 .\n$$", + "remark": "", + "figures": [] +} \ No newline at end of file diff --git a/processed_dataset/proof/1272.json b/processed_dataset/proof/1272.json new file mode 100644 index 0000000000000000000000000000000000000000..a3f11953a36659b047e40c125ae9a28992cccc24 --- /dev/null +++ b/processed_dataset/proof/1272.json @@ -0,0 +1,8 @@ +{ + "source_file": "./raw_volume-zh/volume5/chapter1.tex", + "problem_type": "proof", + "problem": "例18. (Ostrowski) 设实数 $a_1, a_2, \\cdots, a_n$ 与 $b_1, b_2, \\cdots, b_n$ 不成比例.\n实数 $x_1, x_2, \\cdots, x_n$ 满足:\n$$\n\\left\\{\\begin{array}{l}\na_1 x_1+a_2 x_2+\\cdots+a_n x_n=0, \\\\\nb_1 x_1+b_2 x_2+\\cdots+b_n x_n=1 .\n\\end{array}\\right.\n$$\n求证: $x_1^2+x_2^2+\\cdots+x_n^2 \\geqslant \\frac{\\sum_{i=1}^n a_i^2}{\\sum_{i=1}^n a_i^2 \\cdot \\sum_{i=1}^n b_i^2-\\left(\\sum_{i=1}^n a_i b_i\\right)^2}$.", + "solution": "证明:只需在本题中令 $x_k=\\left(\\mathrm{C}_n^2\\right)^{-1} \\cdot \\sum_{r \\neq k} \\frac{a_r}{a_r b_k-a_k b_r}$, 读者不难自行验证 $1, x_2, \\cdots, x_n$ 满足全部条件.", + "remark": "", + "figures": [] +} \ No newline at end of file diff --git a/processed_dataset/proof/1273.json b/processed_dataset/proof/1273.json new file mode 100644 index 0000000000000000000000000000000000000000..d98f2cf0e85a0fe871314a18f9a9a6e7dd4061ff --- /dev/null +++ b/processed_dataset/proof/1273.json @@ -0,0 +1,8 @@ +{ + "source_file": "./raw_volume-zh/volume5/chapter1.tex", + "problem_type": "proof", + "problem": "例20. 设 $a 、 b 、 c$ 是正实数,求证:\n$$\n\\frac{(2 a+b+c)^2}{2 a^2+(b+c)^2}+\\frac{(2 b+c+a)^2}{2 b^2+(c+a)^2}+\\frac{(2 c+a+b)^2}{2 c^2+(a+b)^2} \\leqslant 8 .\n$$", + "solution": "证明:因为左边的式子是齐次的,所以不妨设 $a+b+c=3$, 于是只需证明\n$$\n\\begin{gathered}\n\\frac{(a+3)^2}{2 a^2+(3-a)^2}+\\frac{(b+3)^2}{2 b^2+(3-b)^2}+\\frac{(c+3)^2}{2 c^2+(3-c)^2} \\leqslant 8 . \\\\\nf(x)=\\frac{(x+3)^2}{2 x^2+(3-x)^2}, x \\in \\mathbf{R}^{\\perp} .\n\\end{gathered}\n$$\n令\n$$\nf(x)=\\frac{(x+3)^2}{2 x^2+(3-x)^2}, x \\in \\mathbf{R}^{+} .\n$$\n则\n$$\n\\begin{aligned}\nf(x) & =\\frac{x^2+6 x+9}{3\\left(x^2-2 x+3\\right)} \\\\\n& =\\frac{1}{3}\\left(1+\\frac{8 x+6}{x^2-2 x+3}\\right)=\\frac{1}{3}\\left(1+\\frac{8 x+6}{(x-1)^2+2}\\right) \\\\\n& \\leqslant \\frac{1}{3}\\left(1+\\frac{8 x+6}{2}\\right)=\\frac{1}{3}(4 x+4),\n\\end{aligned}\n$$\n所以\n$$\nf(a)+f(b)+f(c) \\leqslant \\frac{1}{3}(4 a+4+4 b+4+4 c+4)=8 .\n$$", + "remark": "", + "figures": [] +} \ No newline at end of file diff --git a/processed_dataset/proof/1274.json b/processed_dataset/proof/1274.json new file mode 100644 index 0000000000000000000000000000000000000000..05d302eabe8478a9c48018343c6f71871699f408 --- /dev/null +++ b/processed_dataset/proof/1274.json @@ -0,0 +1,8 @@ +{ + "source_file": "./raw_volume-zh/volume5/chapter1.tex", + "problem_type": "proof", + "problem": "例21. 已知 $a+b+c>0, a x^2+b x+c=0$ 有实根, 求证:\n$$\n4 \\min \\{a, b, c\\} \\leqslant a+b+c \\leqslant \\frac{9}{4} \\max \\{a, b, c\\} .\n$$", + "solution": "证明:不妨设 $a+b+c=1$, 否则可用 $\\frac{a}{a+b+c} 、 \\frac{b}{a+b+c} 、 \\frac{c}{a+b+c}$ 代替 $a, b 、 c$.\n先证明: $\\max \\{a, b, c\\} \\geqslant \\frac{4}{9}$.\n(1) 若 $b \\geqslant \\frac{4}{9}$, 则结论成立.\n(2) 若 $b<\\frac{4}{9}$, 因为 $b^2 \\geqslant 4 a c$, 有 $a c<\\frac{4}{81}$.\n又 $a+c=1-b>\\frac{5}{9}$, 所以如果 $a<0$ 或 $c<0$, 即有 $c>\\frac{5}{9}$ 或 $a>\\frac{5}{9}$, 结论成立.\n如果 $a, c \\geqslant 0$, 则 $\\left(\\frac{5}{9}-c\\right) \\cdot c\\frac{4}{9}$.\n若 $c<\\frac{1}{9}$, 此时 $a>\\frac{4}{9}$, 故结论成立.\n再证明: $\\min \\{a, b, c\\} \\leqslant \\frac{1}{4}$.\n(1) 若 $a \\leqslant \\frac{1}{4}$, 则无须证明.\n(2) 若 $a>\\frac{1}{4}$, 则有 $b^2 \\geqslant 4 a c \\geqslant c, b+c=1-a<\\frac{3}{4}$.\n不妨设 $c \\geqslant 0$ (否则 $c<0$, 结论已得), 故 $\\sqrt{c}+c \\leqslant b+c<\\frac{3}{4}$, 于是 $\\left(\\sqrt{c}+\\frac{3}{4}\\right)\\left(\\sqrt{c}-\\frac{1}{2}\\right)<0$.\n因此 $c<\\frac{1}{4}$, 结论成立.\n说明本题的结论是最佳的.\n方程 $\\frac{4}{9} x^2+\\frac{4}{9} x+\\frac{1}{9}=0$ 表明 $\\frac{9}{4}$ 不能改为更小的数; 而方程 $\\frac{1}{4} x^2+ \\frac{2}{4} x+\\frac{1}{4}=0$ 表明 4 不能改为更大的数.", + "remark": "", + "figures": [] +} \ No newline at end of file diff --git a/processed_dataset/proof/1275.json b/processed_dataset/proof/1275.json new file mode 100644 index 0000000000000000000000000000000000000000..b236ef1adf345d5b6298c593aac8147604ed565e --- /dev/null +++ b/processed_dataset/proof/1275.json @@ -0,0 +1,8 @@ +{ + "source_file": "./raw_volume-zh/volume5/chapter1.tex", + "problem_type": "proof", + "problem": "例22. 非负实数 $a 、 b 、 c 、 d$ 满足: $a^2+b^2+c^2+d^2=4$, 求证:\n$$\na^3+b^3+c^3+d^3+a b c+b c d+c d a+d a b \\leqslant 8 .\n$$", + "solution": "证明:原不等式等价于\n$$\n\\left(a^3+b^3+c^3+d^3+a b c+b c d+c d a+d a b\\right)^2 \\leqslant\\left(a^2+b^2+c^2+d^2\\right)^3 .\n$$\n因为\n$$\n\\begin{aligned}\n& a^3+b^3+c^3+d^3+a b c+b c d+c d a+d a b=a\\left(a^2+b c\\right)+b\\left(b^2+c d\\right)+ \\\\\n& c\\left(c^2+d a\\right)+d\\left(d^2+a b\\right) .\n\\end{aligned}\n$$\n所以, 由柯西不等式, 得\n$$\n\\begin{aligned}\n& \\quad\\left(a^3+b^3+c^3+d^3+a b c+b c d+c d a+d a b\\right)^2 \\leqslant\\left(a^2+b^2+c^2+d^2\\right) \\\\\n& {\\left[\\left(a^2+b c\\right)^2+\\left(b^2+c d\\right)^2+\\left(c^2+d a\\right)^2+\\left(d^2+a b\\right)^2\\right] .}\n\\end{aligned}\n$$\n于是只需证明\n$$\n\\begin{gathered}\n\\left(a^2+b c\\right)^2+\\left(b^2+c d\\right)^2+\\left(c^2+d a\\right)^2+\\left(d^2+a b\\right)^2 \\leqslant\\left(a^2+b^2+c^2+d^2\\right)^2 . \\\\\n2\\left(a^2 b c+b^2 c d+c^2 d a+d^2 a b\\right) \\leqslant a^2 b^2+c^2 d^2+a^2 d^2+b^2 c^2+2\\left(a^2 c^2+b^2 d^2\\right), \\\\\n(a b-a c)^2+(a c-c d)^2+(b c-b d)^2+(a d-b d)^2 \\geqslant 0 .\n\\end{gathered}\n$$\n从而命题得证.\n说明本题把右边的常数 8 利用已知条件化为关于 $a 、 b 、 c 、 d$ 的表达式,使得两边的次数一样, 从而有利于解题.", + "remark": "", + "figures": [] +} \ No newline at end of file diff --git a/processed_dataset/proof/1276.json b/processed_dataset/proof/1276.json new file mode 100644 index 0000000000000000000000000000000000000000..d07ea214b17a6e23e508c7c9540d7d1bdfc75f36 --- /dev/null +++ b/processed_dataset/proof/1276.json @@ -0,0 +1,8 @@ +{ + "source_file": "./raw_volume-zh/volume5/chapter1.tex", + "problem_type": "proof", + "problem": "例23. 给定整数 $n \\geqslant 4$, 对任意满足\n$$\na_1+a_2+\\cdots+a_n=b_1+b_2+\\cdots+b_n>0\n$$\n的非负实数 $a_1, a_2, \\cdots, a_n, b_1, b_2, \\cdots, b_n$, 求 $\\frac{\\sum_{i=1}^n a_i\\left(a_i+b_i\\right)}{\\sum^n b_i\\left(a_i+b_i\\right)}$ 的最大值.", + "solution": "解:由齐次性可知, 不妨假设 $\\sum_{i=1}^n a_i=\\sum_{i=1}^n b_i=1$. 首先, 当 $a_1=1, a_2= a_3=\\cdots=a_n=0, b_1=0, b_2=b_3=\\cdots=b_n=\\frac{1}{n-1}$ 时, $\\sum_{i=1}^n a_i\\left(a_i+b_i\\right)=$ 1, $\\sum_{i=1}^n b_i\\left(a_i+b_i\\right)=\\frac{1}{n-1}$, 故\n$$\n\\frac{\\sum_{i=1}^n a_i\\left(a_i+b_i\\right)}{\\sum_{i=1}^n b_i\\left(a_i+b_i\\right)}=n-1 .\n$$\n下证对任意满足 $\\sum_{i=1}^n a_i=\\sum_{i=1}^n b_i=1$ 的 $a_1, a_2, \\cdots, a_n, b_1, b_2, \\cdots, b_n$, 都有\n$$\n\\frac{\\sum_{i=1}^n a_i\\left(a_i+b_i\\right)}{\\sum_{i=1}^n b_i\\left(a_i+b_i\\right)} \\leqslant n-1\n$$\n由于分母是正数,故上式等价于\n$$\n\\begin{aligned}\n& \\sum_{i=1}^n a_i\\left(a_i+b_i\\right) \\leqslant(n-1) \\sum_{i=1}^n b_i\\left(a_i+b_i\\right), \\\\\n& (n-1) \\sum_{i=1}^n b_i^2+(n-2) \\sum_{i=1}^n a_i b_i \\geqslant \\sum_{i=1}^n a_i^2 .\n\\end{aligned}\n$$\n即由对称性, 不妨设 $b_1$ 是 $b_1, b_2, \\cdots, b_n$ 中最小的一个, 则有\n$$\n\\begin{aligned}\n(n-1) \\sum_{i=1}^n b_i^2+(n-2) \\sum_{i=1}^n a_i b_i \\geqslant(n-1) b_1^2+(n-1) \\sum_{i=2}^n b_i^2+(n-2) \\sum_{i=1}^n a_i b_1 \\\\\n\\geqslant(n-1) b_1^2+\\left(\\sum_{i=2}^n b_i\\right)^2+(n-2) b_1 \\\\\n=(n-1) b_1^2+\\left(1-b_1\\right)^2+(n-2) b_1 \\\\\n=n b_1^2+(n-4) b_1+1 \\\\\n\\geqslant 1=\\sum_{i=1}^n a_i \\geqslant \\sum_{i=1}^n a_i^2\n\\end{aligned}\n$$\n所以, 所求的最大值为 $n-1$.", + "remark": "", + "figures": [] +} \ No newline at end of file diff --git a/processed_dataset/proof/1277.json b/processed_dataset/proof/1277.json new file mode 100644 index 0000000000000000000000000000000000000000..6f3dcf7643c6c4cbc820c7cd5ffbe74203c6fc1b --- /dev/null +++ b/processed_dataset/proof/1277.json @@ -0,0 +1,8 @@ +{ + "source_file": "./raw_volume-zh/volume5/chapter1.tex", + "problem_type": "proof", + "problem": "例24. 证明: 在 $\\triangle A B C$ 中, 有\n$$\n\\sum_{c y c} a^3-2 \\sum_{c y c} a^2(b+c)+9 a b c \\leqslant 0 .\n$$", + "solution": "证明:令 $x=\\frac{b+c-a}{2}, y=\\frac{c+a-b}{2}, z=\\frac{a+b-c}{2}$, 则由 Schur 不等式可得\n$$\n\\begin{aligned}\n& \\sum_{c y c}\\left(\\frac{b+c-a}{2}\\right)(b-a)(c-a) \\geqslant 0, \\\\\n& \\sum_{c y c}(a-b)(a-c)(b+c-a) \\geqslant 0, \\\\\n& -\\sum_{c y c} a^3+2 \\sum_{c y c} a^2(b+c)-9 a b c \\geqslant 0,\n\\end{aligned}\n$$\n所以\n$$\n\\sum_{\\text {cyc }} a^3-2 \\sum_{c y c} a^2(b+c)+9 a b c \\leqslant 0 .\n$$", + "remark": "", + "figures": [] +} \ No newline at end of file diff --git a/processed_dataset/proof/1278.json b/processed_dataset/proof/1278.json new file mode 100644 index 0000000000000000000000000000000000000000..342279c2ce6ff54d422b90d8dc6775b912254927 --- /dev/null +++ b/processed_dataset/proof/1278.json @@ -0,0 +1,8 @@ +{ + "source_file": "./raw_volume-zh/volume5/chapter1.tex", + "problem_type": "proof", + "problem": "例25. 设 $x, y, z \\geqslant 0$, 且 $x+y+z=1$, 求证:\n$$\n0 \\leqslant y z+z x+x y-2 x y z \\leqslant \\frac{7}{27} .\n$$", + "solution": "证明:由 Schur 不等式的变形 II , 得\n$$\n\\left(\\sum_{\\mathrm{cyc}} x\\right)^3-4\\left(\\sum_{c y c} x\\right)\\left(\\sum_{c y c} y z\\right)+9 x y z \\geqslant 0,\n$$\n由题设条件 $\\sum_{c y c} x=1$, 得\n$$\n\\begin{gathered}\n1-4 \\cdot \\sum_{c y c} y z+9 x y z \\geqslant 0 \\\\\n\\sum_{c y c} y z-2 x y z \\leqslant \\frac{1}{4}+\\frac{1}{4} x y z \\leqslant \\frac{1}{4}+\\frac{1}{4}\\left(\\frac{x+y+z}{3}\\right)^3=\\frac{7}{27} .\n\\end{gathered}\n$$\n另一方面, $\\sum_{c y c} y z-2 x y z \\geqslant \\sum_{c y c} y z-x y-y z=z x \\geqslant 0$.\n从而命题得证.", + "remark": "", + "figures": [] +} \ No newline at end of file diff --git a/processed_dataset/proof/1279.json b/processed_dataset/proof/1279.json new file mode 100644 index 0000000000000000000000000000000000000000..f2744640fc1cdc0a6e46b99707b62f9229ff3f12 --- /dev/null +++ b/processed_dataset/proof/1279.json @@ -0,0 +1,8 @@ +{ + "source_file": "./raw_volume-zh/volume5/chapter1.tex", + "problem_type": "proof", + "problem": "例26. 设 $x, y, z \\in \\mathbf{R}^{+}$, 且 $x+y+z=x y z$, 求证:\n$$\nx^2+y^2+z^2-2(x y+y z+z x)+9 \\geqslant 0 . \\label{(3)}\n$$", + "solution": "证明:因为 $x+y+z=x y z$, 所以(3)等价于\n$$\n\\begin{gathered}\n{\\left[x^2+y^2+z^2-2(x y+y z+z x)\\right](x+y+z)+9 x y z \\geqslant 0} \\\\\n\\Leftrightarrow x^3+y^3+z^3-\\left(x^2 y+y^2 z+z^2 x+x y^2+y z^2+z x^2\\right)+3 x y z \\geqslant 0\n\\end{gathered}\n$$\n即\n$$\n\\sum_{c y c} x^3-\\sum_{c y c} x^2(y+z)+3 x y z \\geqslant 0,\n$$\n这就是 Schur 不等式的变形 I. 故命题得证.", + "remark": "", + "figures": [] +} \ No newline at end of file diff --git a/processed_dataset/proof/1280.json b/processed_dataset/proof/1280.json new file mode 100644 index 0000000000000000000000000000000000000000..c63bea5ba7c6df348d54d0f97ba9257dec256d37 --- /dev/null +++ b/processed_dataset/proof/1280.json @@ -0,0 +1,8 @@ +{ + "source_file": "./raw_volume-zh/volume5/chapter1.tex", + "problem_type": "proof", + "problem": "例27. 设 $a, b, c \\in \\mathbf{R}^{+}$, 求证:\n$$\n\\sqrt{a b c}(\\sqrt{a}+\\sqrt{b}+\\sqrt{c})+(a+b+c)^2 \\geqslant 4 \\sqrt{3 a b c(a+b+c)} .\n$$", + "solution": "证明:由 Schur 不等式(在(2)中, 令 $r=2$ ), 得\n$$\n\\sum_{c y c} x^2(x-y)(x-z) \\geqslant 0, x, y, z \\in \\mathbf{R}^{+},\n$$\n所以\n$$\n\\sum_{c y c} x^4+x y z \\sum_{c y c} x \\geqslant \\sum_{c y c} x^3(y+z) . \\label{(4)}\n$$\n又因为 $\\sum_{c y c} x^3(y+z)=2 \\sum_{c y c} y^2 z^2+\\sum_{c y c} y z(y-z)^2 \\geqslant 2 \\sum_{c y c} y^2 z^2$, 所以\n$$\n\\sum_{c y c} x^4+x y z \\sum_{c y c} x \\geqslant 2 \\sum_{c y c} y^2 z^2 .\n$$\n在(4)式中, 令 $x=\\sqrt{a}, y=\\sqrt{b}, z=\\sqrt{c}$, 得\n$$\n\\begin{gathered}\n\\sum_{c y c} a^2+\\sqrt{a b c} \\sum_{c y c} \\sqrt{a} \\geqslant 2 \\sum_{c y c} b c, \\\\\n\\sqrt{a b c} \\sum_{c y c} \\sqrt{a}+\\left(\\sum_{c y c} a\\right)^2 \\geqslant 4 \\sum_{c y c} b c .\n\\end{gathered}\n$$\n下证 $\\sum_{c y c} b c \\geqslant \\sqrt{3 a b c(a+b+c)}$.\n事实上, 由 $(u+v+w)^2 \\geqslant 3(u v+v w+w u)$, 得\n$$\n\\begin{aligned}\n(a b+b c+c a)^2 & \\geqslant 3(a b \\cdot b c+b c \\cdot c a+c a \\cdot a b) \\\\\n& =3 a b c(a+b+c),\n\\end{aligned}\n$$\n所以\n$$\n\\sum_{c y c} b c \\geqslant \\sqrt{3 a b c(a+b+c)},\n$$\n故 $\\quad \\sqrt{a b c}(\\sqrt{a}+\\sqrt{b}+\\sqrt{c})+(a+b+c)^2 \\geqslant 4 \\sqrt{3 a b c(a+b+c)}$.", + "remark": "", + "figures": [] +} \ No newline at end of file diff --git a/processed_dataset/proof/1281.json b/processed_dataset/proof/1281.json new file mode 100644 index 0000000000000000000000000000000000000000..04ff693e1cc4486f918aacc3269ef168921194c6 --- /dev/null +++ b/processed_dataset/proof/1281.json @@ -0,0 +1,8 @@ +{ + "source_file": "./raw_volume-zh/volume5/chapter1.tex", + "problem_type": "proof", + "problem": "例29. 设 $a, b, c \\in \\mathbf{R}^{+}$, 求证:\n$$\n\\frac{a+b+c}{3} \\geqslant \\sqrt[3]{\\frac{(a+b)(b+c)(c+a)}{8}} \\geqslant \\frac{\\sqrt{a b}+\\sqrt{b c}+\\sqrt{c a}}{3} .\n$$", + "solution": "证明:由平均不等式, 得\n$$\n\\begin{gathered}\n\\frac{(a+b)+(b+c)+(c+a)}{3} \\geqslant \\sqrt[3]{(a+b)(b+c)(c+a)}, \\\\\n\\frac{a+b+c}{3} \\geqslant \\sqrt[3]{\\frac{(a+b)(b+c)(c+a)}{8}} .\n\\end{gathered}\n$$\n所以\n$$\n\\frac{a+b+c}{3} \\geqslant \\sqrt[3]{\\frac{(a+b)(b+c)(c+a)}{8}} .\n$$\n由(2)(Hölder 不等式), 有\n$$\n\\begin{aligned}\n\\frac{(a+b)(b+c)(c+a)}{8} & =\\frac{1}{27}\\left(\\frac{a+b}{2}+b+a\\right)\\left(b+\\frac{b+c}{2}+c\\right)\\left(a+c+\\frac{a+c}{2}\\right) \\\\\n& \\geqslant \\frac{1}{27}\\left(\\sqrt[3]{\\frac{a+b}{2} \\cdot b \\cdot a}+\\sqrt[3]{b \\cdot \\frac{b+c}{2} \\cdot c}+\\sqrt[3]{a \\cdot c \\cdot \\frac{a+c}{2}}\\right)^3 \\\\\n& \\geqslant \\frac{1}{27}(\\sqrt[3]{\\sqrt{a b} \\cdot a \\cdot b}+\\sqrt[3]{\\sqrt{b c} \\cdot b \\cdot c}+\\sqrt[3]{\\sqrt{c a} \\cdot c \\cdot a})^3 \\\\\n& =\\frac{1}{27}(\\sqrt{a b}+\\sqrt{b c}+\\sqrt{c a})^3,\n\\end{aligned}\n$$\n所以 $\\sqrt[3]{\\frac{(a+b)(b+c)(c+a)}{8}} \\geqslant \\frac{\\sqrt{a b}+\\sqrt{b c}+\\sqrt{c a}}{3}$.", + "remark": "", + "figures": [] +} \ No newline at end of file diff --git a/processed_dataset/proof/1282.json b/processed_dataset/proof/1282.json new file mode 100644 index 0000000000000000000000000000000000000000..a17ac9e3ba12eba184a34b398c900589d9db2974 --- /dev/null +++ b/processed_dataset/proof/1282.json @@ -0,0 +1,8 @@ +{ + "source_file": "./raw_volume-zh/volume5/chapter1.tex", + "problem_type": "proof", + "problem": "例30. 设 $a 、 b 、 c$ 是正实数,求证:\n$$\n\\left(a^5-a^2+3\\right)\\left(b^5-b^2+3\\right)\\left(c^5-c^2+3\\right) \\geqslant(a+b+c)^3 .\n$$", + "solution": "证明:对于 $x \\in \\mathbf{R}^{+}, x^2-1$ 与 $x^3-1$ 具有相同的符号, 所以\n$$\n\\left(x^2-1\\right)\\left(x^3-1\\right) \\geqslant 0 \\text {, }\n$$\n即\n$$\nx^5-x^2+3 \\geqslant x^3+2 .\n$$\n于是 $\\left(a^5-a^2+3\\right)\\left(b^5-b^2+3\\right)\\left(c^5-c^2+3\\right) \\geqslant\\left(a^3+2\\right)\\left(b^3+2\\right)\\left(c^3+2\\right)$.\n而由(2)(Hölder 不等式),有\n$$\n\\begin{aligned}\n\\left(a^3+2\\right)\\left(b^3+2\\right)\\left(c^3+2\\right) & =\\left(a^3+1+1\\right)\\left(1+b^3+1\\right)\\left(1+1+c^3\\right) \\\\\n& \\geqslant(a+b+c)^3,\n\\end{aligned}\n$$\n从而命题得证.", + "remark": "", + "figures": [] +} \ No newline at end of file diff --git a/processed_dataset/proof/1283.json b/processed_dataset/proof/1283.json new file mode 100644 index 0000000000000000000000000000000000000000..688df518119b813bc9612dae7c601975e1521e35 --- /dev/null +++ b/processed_dataset/proof/1283.json @@ -0,0 +1,8 @@ +{ + "source_file": "./raw_volume-zh/volume5/chapter1.tex", + "problem_type": "proof", + "problem": "例31.(幂平均不等式) 设 $a_1, a_2, \\cdots, a_n$ 是正实数, $\\alpha>\\beta>0$, 则\n$$\n\\left(\\frac{1}{n} \\sum_{i=1}^n a_i^\\beta\\right)^{\\frac{1}{\\beta}} \\leqslant\\left(\\frac{1}{n} \\sum_{i=1}^n a_i^\\alpha\\right)^{\\frac{1}{\\alpha}}\n$$", + "solution": "证明:在(6)中 (Hölder 不等式), 令 $x^i=1, i=1,2, \\cdots, n$, 有\n$$\n\\sum_{i=1}^n y_i \\leqslant n^{\\frac{1}{p}}\\left(\\sum_{i=1}^n y_i^q\\right)^{\\frac{1}{q}}\n$$\n由于 $\\frac{1}{p}=1-\\frac{1}{q}$, 所以上式可以写成\n$$\n\\frac{1}{n} \\sum_{i=1}^n y_i \\leqslant\\left(\\frac{1}{n} \\sum_{i=1}^n y_i^q\\right)^{\\frac{1}{q}} .\n$$\n在上式中, 令 $y_i=a_i^\\beta, q=\\frac{\\alpha}{\\beta}>1$, 得 $\\frac{1}{n} \\sum_{i=1}^n a_i^\\beta \\leqslant\\left(\\frac{1}{n} \\sum_{i=1}^n a_i^\\alpha\\right)^{\\frac{\\beta}{\\alpha}}$, 于是 $\\left(\\frac{1}{n} \\sum_{i=1}^n a_i^\\beta\\right)^{\\frac{1}{\\beta}} \\leqslant\\left(\\frac{1}{n} \\sum_{i=1}^n a_i^\\alpha\\right)^{\\frac{1}{\\alpha}}$.", + "remark": "", + "figures": [] +} \ No newline at end of file diff --git a/processed_dataset/proof/1284.json b/processed_dataset/proof/1284.json new file mode 100644 index 0000000000000000000000000000000000000000..b5d8db028e702ecbcb2024ec66d996083ccd675a --- /dev/null +++ b/processed_dataset/proof/1284.json @@ -0,0 +1,8 @@ +{ + "source_file": "./raw_volume-zh/volume5/chapter2.tex", + "problem_type": "proof", + "problem": "例1. 证明 Lagrange 恒等式:\n$$\n\\left(\\sum_{i=1}^n a_i^2\\right) \\cdot\\left(\\sum_{i=1}^n b_i^2\\right)=\\left(\\sum_{i=1}^n a_i b_i\\right)^2+\\sum_{1 \\leqslant is \\geqslant r>q, p+q=r+s, a_1, a_2, \\cdots, a_n>0$, 则\n$$\n\\left(\\sum_{i=1}^n a_i^p\\right) \\cdot\\left(\\sum_{i=1}^n a_i^q\\right) \\geqslant\\left(\\sum_{i=1}^n a_i^s\\right) \\cdot\\left(\\sum_{i=1}^n a_i^r\\right)\n$$", + "solution": "证明:$$\n\\begin{aligned}\n& \\left(\\sum_{i=1}^n a_i^p\\right) \\cdot\\left(\\sum_{i=1}^n a_i^q\\right)-\\left(\\sum_{i=1}^n a_i^s\\right) \\cdot\\left(\\sum_{i=1}^n a_i^r\\right) \\\\\n= & \\sum_{i=1}^n \\sum_{j=1}^n\\left(a_i^p a_j^q-a_i^s a_j^r\\right)\n\\end{aligned}\n$$\n$$\n\\begin{aligned}\n& =\\frac{1}{2} \\sum_{i=1}^n \\sum_{j=1}^n\\left(a_i^p a_j^q-a_i^s a_j^r\\right)+\\frac{1}{2} \\sum_{i=1}^n \\sum_{j=1}^n\\left(a_j^p a_i^q-a_j^s a_i^r\\right) \\\\\n& =\\frac{1}{2} \\sum_{i=1}^n \\sum_{j=1}^n\\left(a_i^p a_j^q+a_j^p a_i^q-a_i^s a_j^r-a_j^s a_i^r\\right) \\\\\n& =\\frac{1}{2} \\sum_{i=1}^n \\sum_{j=1}^n a_i^q a_j^q\\left(a_i^{p-q}+a_j^{p-q}-a_i^{s-q} a_j^{r-q}-a_j^{s-q} a_i^{r-q}\\right) \\\\\n& =\\frac{1}{2} \\sum_{i=1}^n \\sum_{j=1}^n a_i^q a_j^q\\left(a_i^{p-s}-a_j^{p-s}\\right)\\left(a_i^{s-q}-a_j^{s-q}\\right) \\\\\n& \\geqslant 0 .\n\\end{aligned}\n$$\n故原不等式成立.", + "remark": "", + "figures": [] +} \ No newline at end of file diff --git a/processed_dataset/proof/1286.json b/processed_dataset/proof/1286.json new file mode 100644 index 0000000000000000000000000000000000000000..22a75ce9732e778a216cba093c7591ae5951be05 --- /dev/null +++ b/processed_dataset/proof/1286.json @@ -0,0 +1,8 @@ +{ + "source_file": "./raw_volume-zh/volume5/chapter2.tex", + "problem_type": "proof", + "problem": "例3. 若 $a_i>0, b_i>0, a_i b_i=c_i^2+d_i^2(i=1,2, \\cdots, n)$, 则\n$$\n\\left(\\sum_{i=1}^n a_i\\right)\\left(\\sum_{i=1}^n b_i\\right) \\geqslant\\left(\\sum_{i=1}^n c_i\\right)^2+\\left(\\sum_{i=1}^n d_i\\right)^2,\n$$\n等号成立当且仅当 $\\frac{a_i}{a_j}=\\frac{b_i}{b_j}=\\frac{c_i}{c_j}=\\frac{d_i}{d_j}(1 \\leqslant i2\\left(\\frac{a_2}{2}+\\frac{a_3}{3}+\\cdots+\\frac{a_n}{n}\\right) .\n$$", + "solution": "分析:若直接通过 $a_n$ 的表达式来证将非常复杂, 但通过建立其递推公式,可以使问题很容易得到解决, 我们便可从此处人手.\n证明\n$$\n\\begin{aligned}\na_n^2-a_{n-1}^2 & =\\left(1+\\frac{1}{2}+\\cdots+\\frac{1}{n}\\right)^2-\\left(1+\\frac{1}{2}+\\cdots+\\frac{1}{n-1}\\right)^2 \\\\\n& =\\frac{1}{n^2}+2 \\cdot \\frac{1}{n}\\left(1+\\frac{1}{2}+\\cdots+\\frac{1}{n-1}\\right) \\\\\n& =\\frac{1}{n^2}+\\frac{2}{n}\\left(a_n-\\frac{1}{n}\\right) \\\\\n& =2 \\cdot \\frac{a_n}{n}-\\frac{1}{n^2} .\n\\end{aligned}\n$$\n故 $\\quad a_n^2-a_1^2=2\\left(\\frac{a_2}{2}+\\frac{a_3}{3}+\\cdots+\\frac{a_n}{n}\\right)-\\left(\\frac{1}{2^2}+\\frac{1}{3^2}+\\cdots+\\frac{1}{n^2}\\right)$.\n所以 $a_n^2=2\\left(\\frac{a_2}{2}+\\frac{a_3}{3}+\\cdots+\\frac{a_n}{n}\\right)+\\left(1-\\frac{1}{2^2}-\\cdots-\\frac{1}{n^2}\\right)$\n$$\n\\begin{aligned}\n& >2\\left(\\frac{a_2}{2}+\\frac{a_3}{3}+\\cdots+\\frac{a_n}{n}\\right)+\\left(1-\\frac{1}{1 \\times 2}-\\frac{1}{2 \\times 3}-\\cdots-\\frac{1}{(n-1) n}\\right) \\\\\n& =2\\left(\\frac{a_2}{2}+\\frac{a_3}{3}+\\cdots+\\frac{a_n}{n}\\right)+\\frac{1}{n} \\\\\n& >2\\left(\\frac{a_2}{2}+\\frac{a_3}{3}+\\cdots+\\frac{a_n}{n}\\right) .\n\\end{aligned}\n$$\n故原不等式成立.\n说明本题也可以用数学归纳法证明加强的命题:\n$$\na_n^2>2\\left(\\frac{a_2}{2}+\\frac{a_3}{3}+\\cdots+\\frac{a_n}{n}\\right)+\\frac{1}{n} .\n$$", + "remark": "", + "figures": [] +} \ No newline at end of file diff --git a/processed_dataset/proof/1289.json b/processed_dataset/proof/1289.json new file mode 100644 index 0000000000000000000000000000000000000000..623f7db24ed050ef51b0b45c0013e7a288206027 --- /dev/null +++ b/processed_dataset/proof/1289.json @@ -0,0 +1,8 @@ +{ + "source_file": "./raw_volume-zh/volume5/chapter2.tex", + "problem_type": "proof", + "problem": "例6. 设数列 $\\left\\{a_k\\right\\}$ 满足: $a_{k+1}=a_k+f(n) \\cdot a_k^2, 0 \\leqslant k \\leqslant n$. 其中 $00, f(n)>0$, 易知 $a_p>0(1 \\leqslant p \\leqslant n)$, 于是\n$$\na_{k+1}=a_k+f(n) a_k^2>a_k(0 \\leqslant k \\leqslant n),\n$$\n故\n$$\na_{k+1}=a_k+f(n) a_k^2\\frac{1}{a_0}-p f(n) \\geqslant \\frac{1}{a_0}-p \\cdot \\frac{1}{n}\\left(\\frac{1}{a_0}-1\\right) \\geqslant 1$,\n于是 $\\quad a_p<\\frac{1}{\\frac{1}{a_0}-p f(n)}=\\frac{a_0}{1-a_0 p f(n)} \\leqslant 1(1 \\leqslant p \\leqslant n)$.\n另一方面, 由 $a_p^2\\frac{1}{1+f(n)} a_{k+1},\n$$\n故\n$$\n\\begin{gathered}\na_{k+1}>a_k+f(n) \\frac{a_k \\cdot a_{k+1}}{1+f(n)}, \\\\\n\\frac{1}{a_k}-\\frac{1}{a_{k+1}}>\\frac{f(n)}{1+f(n)},\n\\end{gathered}\n$$\n则于是\n$$\n\\begin{gathered}\n\\sum_{k=0}^{p-1}\\left(\\frac{1}{a_k}-\\frac{1}{a_{k+1}}\\right)>p \\cdot \\frac{f(n)}{1+f(n)}(1 \\leqslant p \\leqslant n), \\\\\n\\frac{1}{a_0}-\\frac{1}{a_p}>p \\cdot \\frac{f(n)}{1+f(n)},\n\\end{gathered}\n$$\n也即\n$$\n\\frac{1}{a_p}<\\frac{1}{a_0}-\\frac{p \\cdot f(n)}{1+f(n)}=\\frac{1+\\left(1-a_0 p\\right) f(n)}{a_0(1+f(n))},\n$$\n所以\n$$\na_p>\\frac{a_0(1+f(n))}{1+\\left(1-a_0 p\\right) f(n)}(1 \\leqslant p \\leqslant n) .\n$$\n综合两方面情况, 命题得证.", + "remark": "", + "figures": [] +} \ No newline at end of file diff --git a/processed_dataset/proof/1290.json b/processed_dataset/proof/1290.json new file mode 100644 index 0000000000000000000000000000000000000000..202a74416d8b1d8e4343bac5571a8e75e9046d67 --- /dev/null +++ b/processed_dataset/proof/1290.json @@ -0,0 +1,8 @@ +{ + "source_file": "./raw_volume-zh/volume5/chapter2.tex", + "problem_type": "proof", + "problem": "例7. (钟开莱不等式) 设 $a_1 \\geqslant a_2 \\geqslant \\cdots \\geqslant a_n>0$, 且 $\\sum_{i=1}^k a_i \\leqslant \\sum_{i=1}^k b_i(1 \\leqslant k \\leqslant n)$, 则\n(1) $\\sum_{i=1}^n a_i^2 \\leqslant \\sum_{i=1}^n b_i^2$;\n(2) $\\sum_{i=1}^n a_i^3 \\leqslant \\sum_{i=1}^n a_i b_i^2$.", + "solution": "证明:(1)由 Abel 变换公式,\n$$\n\\begin{aligned}\n\\sum_{i=1}^n a_i^2 & =a_n\\left(\\sum_{i=1}^n a_i\\right)+\\sum_{k=1}^{n-1}\\left(\\sum_{i=1}^k a_i\\right)\\left(a_k-a_{k+1}\\right) \\\\\n& \\leqslant a_n\\left(\\sum_{i=1}^n b_i\\right)+\\sum_{k=1}^{n-1}\\left(\\sum_{i=1}^k b_i\\right)\\left(a_k-a_{k+1}\\right) \\\\\n& =\\sum_{i=1}^n a_i b_i .\n\\end{aligned}\n$$\n再由 Cauchy 不等式, 有\n$$\n\\sum_{i=1}^n a_i b_i \\leqslant\\left(\\sum_{i=1}^n a_i^2\\right)^{\\frac{1}{2}} \\cdot\\left(\\sum_{i=1}^n b_i^2\\right)^{\\frac{1}{2}},\n$$\n即得\n$$\n\\sum_{i=1}^n a_i^2 \\leqslant \\sum_{i=1}^n b_i^2 .\n$$\n$$\n\\begin{aligned}\n& \\sum_{i=1}^n a_i^3=a_n^2\\left(\\sum_{i=1}^n a_i\\right)+\\sum_{k=1}^{n-1}\\left(\\sum_{i=1}^k a_i\\right)\\left(a_k^2-a_{k+1}^2\\right) \\\\\n& \\leqslant a_n^2\\left(\\sum_{i=1}^n b_i\\right)+\\sum_{k=1}^{n-1}\\left(\\sum_{i=1}^k b_i\\right)\\left(a_k^2-a_{k+1}^2\\right) \\\\\n&=\\sum_{i=1}^n a_i^2 b_i \\\\\n&=\\sum_{i=1}^n\\left(a_i^{\\frac{3}{2}} \\cdot a_i^{\\frac{1}{2}} b_i\\right) \\\\\n& \\leqslant\\left(\\sum_{i=1}^n a_i^3\\right)^{\\frac{1}{2}} \\cdot\\left(\\sum_{i=1}^n a_i b_i^2\\right)^{\\frac{1}{2}} . \\\\\n& \\sum_{i=1}^n a_i^3 \\leqslant \\sum_{i=1}^n a_i b_i^2 .\n\\end{aligned}\n$$\n即$\\sum_{i=1}^n a_i^3 \\leqslant \\sum_{i=1}^n a_i b_i^2$.", + "remark": "", + "figures": [] +} \ No newline at end of file diff --git a/processed_dataset/proof/1291.json b/processed_dataset/proof/1291.json new file mode 100644 index 0000000000000000000000000000000000000000..b698b63bf2d35cd54d324a544ca5933315f191fc --- /dev/null +++ b/processed_dataset/proof/1291.json @@ -0,0 +1,8 @@ +{ + "source_file": "./raw_volume-zh/volume5/chapter2.tex", + "problem_type": "proof", + "problem": "例8. 设 $a_1, a_2, \\cdots$ 是正实数列, 且对所有 $i, j=1,2, \\cdots$ 满足 $a_{i+j} \\leqslant a_i+a_j$. 求证: 对于正整数 $n$, 有\n$$\na_1+\\frac{a_2}{2}+\\frac{a_3}{3}+\\cdots+\\frac{a_n}{n} \\geqslant a_n\n$$", + "solution": "分析:由条件, 当 $i+j=k$ 时, 有 $a_i+a_{k-i} \\geqslant a_k$, 于是得到关于和 $s_{k-1}= a_1+\\cdots+a_{k-1}$ 的估计, 而差 $\\frac{1}{i}-\\frac{1}{i+1}$ 是易求的,提示我们用 Abel 变换公式.\n证明利用 Abel 变换法.\n记 $s_i=a_1+a_2+\\cdots+a_i, i=1,2, \\cdots, n$.\n约定 $s_0=0$, 则\n$$\n2 s_i=\\left(a_1+a_i\\right)+\\cdots+\\left(a_i+a_1\\right) \\geqslant i a_{i+1} .\n$$\n即\n$$\ns_i \\geqslant \\frac{i}{2} \\cdot a_{i+1} .\n$$\n故\n$$\n\\begin{aligned}\n\\sum_{i=1}^n \\frac{a_i}{i} & =\\sum_{i=1}^n \\frac{s_i-s_{i-1}}{i} \\\\\n& =\\sum_{i=1}^{n-1} s_i\\left(\\frac{1}{i}-\\frac{1}{i+1}\\right)+\\frac{1}{n} \\cdot s_n \\\\\n& \\geqslant \\frac{1}{2} s_1+\\sum_{i=1}^{n-1} \\frac{i a_{i+1}}{2}\\left(\\frac{1}{i}-\\frac{1}{i+1}\\right)+\\frac{1}{n} s_n \\\\\n& =\\frac{1}{2} s_1+\\frac{1}{2} \\cdot \\sum_{i=1}^{n-1} \\frac{a_{i+1}}{i+1}+\\frac{1}{n} \\cdot s_n\n\\end{aligned}\n$$\n$$\n=\\frac{1}{2} \\cdot \\sum_{i=1}^n \\frac{a_i}{i}+\\frac{1}{n} s_n\n$$\n因此\n$$\n\\begin{aligned}\n& \\sum_{i=1}^n \\frac{a_i}{i} \\geqslant \\frac{2}{n} s_n \\\\\n= & \\frac{2}{n}\\left(s_{n-1}+a_n\\right) \\\\\n\\geqslant & \\frac{2}{n} \\cdot\\left(\\frac{n-1}{2} a_n+a_n\\right) \\\\\n= & \\frac{n+1}{n} a_n>a_n .\n\\end{aligned}\n$$\n故原不等式成立.\n说明如果去掉 $a_1, a_2, \\cdots$, 是正的这一条件, 则可用数学归纳法证明本题 .", + "remark": "", + "figures": [] +} \ No newline at end of file diff --git a/processed_dataset/proof/1292.json b/processed_dataset/proof/1292.json new file mode 100644 index 0000000000000000000000000000000000000000..dcaacc3a7f52856938118ce808bcbb44fd904695 --- /dev/null +++ b/processed_dataset/proof/1292.json @@ -0,0 +1,8 @@ +{ + "source_file": "./raw_volume-zh/volume5/chapter2.tex", + "problem_type": "proof", + "problem": "例9. (排序不等式) 设有两个有序数组: $a_1 \\leqslant a_2 \\leqslant \\cdots \\leqslant a_n$ 及 $b_1 \\leqslant b_2 \\leqslant \\cdots \\leqslant b_n$. 求证:\n$$\n\\begin{aligned}\n& a_1 b_1+a_2 b_2+\\cdots+a_n b_n(\\text { 顺序和 }) \\\\\n\\geqslant & a_1 b_{j_1}+a_2 b_{j_2}+\\cdots+a_n b_{j_n}(\\text { 乱序和 }) \\\\\n\\geqslant & a_1 b_n+a_2 b_{n-1}+\\cdots+a_n b_1(\\text { 逆序和 }),\n\\end{aligned}\n$$\n其中 $j_1, j_2, \\cdots, j_n$ 是 $1,2, \\cdots, n$ 的任意一个排列.", + "solution": "证明:$$\ns_i=b_1+b_2+\\cdots+b_i,\n$$\n$$\ns_i^{\\prime}=b_{j_1}+b_{j_2}+\\cdots+b_{j_i}(i=1,2, \\cdots, n) \\text {. }\n$$\n由题设易知\n$$\ns_i \\leqslant s_i^{\\prime}(i=1,2, \\cdots, n-1) \\text {, }\n$$\n$$\ns_n=s_n^{\\prime} \\text {. }\n$$\n又因为 $a_i-a_{i+1} \\leqslant 0$, 故 $s_i\\left(a_i-a_{i+1}\\right) \\geqslant s_i^{\\prime}\\left(a_i-a_{i+1}\\right)$.\n所以\n$$\n\\begin{aligned}\n\\sum_{i=1}^n a_i b_i & =\\sum_{i=1}^{n-1} s_i\\left(a_i-a_{i+1}\\right)+a_n s_n \\\\\n& \\geqslant \\sum_{i=1}^{n-1} s_i^{\\prime}\\left(a_i-a_{i+1}\\right)+a_n s_n^{\\prime} \\\\\n& =\\sum_{i=1}^n a_i b_{j_i} .\n\\end{aligned}\n$$\n此即左端不等式.\n类似可证得右端不等式.", + "remark": "", + "figures": [] +} \ No newline at end of file diff --git a/processed_dataset/proof/1293.json b/processed_dataset/proof/1293.json new file mode 100644 index 0000000000000000000000000000000000000000..a46dbdca3427618bc6abf3d566119cd4b5add8e9 --- /dev/null +++ b/processed_dataset/proof/1293.json @@ -0,0 +1,8 @@ +{ + "source_file": "./raw_volume-zh/volume5/chapter2.tex", + "problem_type": "proof", + "problem": "例10. 将 $1,2,3, \\cdots, 2007$ 这 2007 个数任意排列可得 2007 ! 个不同数列, 问其中是否存在 4 个数列:\n$$\na_1, a_2, \\cdots, a_{2007} ; b_1, b_2, \\cdots, b_{2007} ; c_1, c_2, \\cdots, c_{2007} ; d_1, d_2, \\cdots, d_{2007},\n$$\n使得 $\\quad a_1 b_1+a_2 b_2+\\cdots+a_{2007} b_{2007}=2\\left(c_1 d_1+c_2 d_2+\\cdots+c_{2007} d_{2007}\\right)$ ?\n并证明你的结论.", + "solution": "解:由排序不等式\n$$\n\\begin{aligned}\na_1 b_1+a_2 b_2+\\cdots+a_{2007} b_{2007} & \\leqslant 1 \\cdot 1+2 \\cdot 2+\\cdots+2007 \\cdot 2007 \\\\\n& =\\frac{2007 \\cdot 2008 \\cdot 4015}{6}=2696779140, \\\\\nc_1 d_1+c_2 d_2+\\cdots+c_{2007} d_{2007} & \\geqslant 1 \\cdot 2007+2 \\cdot 2006+\\cdots+2007 \\cdot 1 \\\\\n& =\\sum_{k=1}^{2007} k(2008-k)=2008 \\sum_{k=1}^{2007} k-\\sum_{k=1}^{2007} k^2 \\\\\n& =2008 \\cdot \\frac{2007 \\cdot 2008}{2}-2696779140 \\\\\n& =1349397084,\n\\end{aligned}\n$$\n于是 $2\\left(c_1 d_1+c_2 d_2+\\cdots+c_{2007} d_{2007}\\right) \\geqslant 2698794168>a_1 b_1+a_2 b_2+\\cdots+ a_{2007} b_{2007}$.\n由此可见,满足条件的四个数列不存在.", + "remark": "", + "figures": [] +} \ No newline at end of file diff --git a/processed_dataset/proof/1294.json b/processed_dataset/proof/1294.json new file mode 100644 index 0000000000000000000000000000000000000000..4da2a8faba467ac2a9695b613081a117330ed406 --- /dev/null +++ b/processed_dataset/proof/1294.json @@ -0,0 +1,8 @@ +{ + "source_file": "./raw_volume-zh/volume5/chapter2.tex", + "problem_type": "proof", + "problem": "例11. 求证: 对每个正整数 $n$, 有\n$$\n\\frac{2 n+1}{3} \\sqrt{n} \\leqslant \\sum_{i=1}^n \\sqrt{i} \\leqslant \\frac{4 n+3}{6} \\sqrt{n}-\\frac{1}{6} .\n$$\n不等式两边等号成立当且仅当 $n=1$.", + "solution": "分析:对于 $\\sqrt{i}$, 既可看成是 $1 \\cdot \\sqrt{i}$, 也可看成是 $i \\cdot \\frac{1}{\\sqrt{i}}$, 这样就得到两种估计方法.\n证明容易验证当 $n=1$ 时,两个不等式都取等号.\n下面不妨设 $n \\geqslant 2$.\n先证左边不等式.\n令 $a_i=1, b_i=\\sqrt{i}(1 \\leqslant i \\leqslant n)$, 则\n$$\ns_i=a_1+a_2+\\cdots+a_i=i .\n$$\n利用 Abel 分部求和公式,得\n$$\n\\begin{aligned}\ns & =\\sum_{i=1}^n \\sqrt{i}=\\sum_{i=1}^n a_i b_i=\\sum_{i=1}^{n-1} i(\\sqrt{i}-\\sqrt{i+1})+n \\sqrt{n} \\\\\n& =n \\sqrt{n}-\\sum_{i=1}^{n-1} \\frac{i}{\\sqrt{i}+\\sqrt{i+1}} .\n\\end{aligned}\n$$\n由 $\\frac{i}{\\sqrt{i}+\\sqrt{i+1}}<\\frac{i}{2 \\sqrt{i}}=\\frac{1}{2} \\sqrt{i}$, 有\n$$\n\\begin{gathered}\n\\sum_{i=1}^{n-1} \\frac{i}{\\sqrt{i}+\\sqrt{i+1}}<\\frac{1}{2} \\sum_{i=1}^{n-1} \\sqrt{i}=\\frac{1}{2}(s-\\sqrt{n}), \\\\\ns>n \\sqrt{n}-\\frac{1}{2}(s-\\sqrt{n}), \\\\\ns>\\frac{2 n+1}{3} \\cdot \\sqrt{n} .\n\\end{gathered}\n$$\n所以解之, 得\n$$\ns>\\frac{2 n+1}{3} \\cdot \\sqrt{n}\n$$\n下证右边不等式.\n令 $a_i=i, b_i=\\frac{1}{\\sqrt{i}}(1 \\leqslant i \\leqslant n)$, 则\n$$\ns_i=a_1+a_2+\\cdots+a_i=\\frac{1}{2} i(i+1) .\n$$\n利用 Abel 分部求和公式, 有\n$$\n\\begin{aligned}\ns & =\\sum_{i=1}^{n-1} \\frac{i(i+1)}{2}\\left(\\frac{1}{\\sqrt{i}}-\\frac{1}{\\sqrt{i+1}}\\right)+\\frac{1}{\\sqrt{n}} \\cdot \\frac{n(n+1)}{2} \\\\\n& =\\frac{n+1}{2} \\cdot \\sqrt{n}+\\sum_{i=1}^{n-1} \\frac{i(i+1)}{2} \\cdot \\frac{1}{\\sqrt{i(i+1)(\\sqrt{i}+\\sqrt{i+1})}} \\\\\n& =\\frac{n+1}{2} \\cdot \\sqrt{n}+\\frac{1}{2} \\sum_{i=1}^{n-1} \\frac{\\sqrt{i(i+1)}}{\\sqrt{i}+\\sqrt{i+1}} .\n\\end{aligned}\n$$\n因为 $\\frac{\\sqrt{i(i+1)}}{\\sqrt{i}+\\sqrt{i+1}}<\\frac{1}{4}(\\sqrt{i}+\\sqrt{i+1})(i=1,2, \\cdots, n-1)$,\n故有\n$$\n\\begin{gathered}\n\\sum_{i=1}^{n-1} \\frac{\\sqrt{i(i+1)}}{\\sqrt{i}+\\sqrt{i+1}}<\\frac{1}{4} \\cdot \\sum_{i=1}^{n-1}(\\sqrt{i}+\\sqrt{i+1}) \\\\\n=\\frac{1}{4}(2 s-\\sqrt{n}-1) \\\\\ns<\\frac{1}{8}(2 s-\\sqrt{n}-1)+\\frac{n+1}{2} \\cdot \\sqrt{n} \\\\\ns<\\frac{4 n+3}{6} \\cdot \\sqrt{n}-\\frac{1}{6} .\n\\end{gathered}\n$$\n于是得到\n$$\ns<\\frac{1}{8}(2 s-\\sqrt{n}-1)+\\frac{n+1}{2} \\cdot \\sqrt{n},\n$$\n从而\n$$\ns<\\frac{4 n+3}{6} \\cdot \\sqrt{n}-\\frac{1}{6}\n$$\n命题获证.", + "remark": "", + "figures": [] +} \ No newline at end of file diff --git a/processed_dataset/proof/1295.json b/processed_dataset/proof/1295.json new file mode 100644 index 0000000000000000000000000000000000000000..1d4e920079fd9ef79fbc90aa0e9d299b9ca517c3 --- /dev/null +++ b/processed_dataset/proof/1295.json @@ -0,0 +1,8 @@ +{ + "source_file": "./raw_volume-zh/volume5/chapter2.tex", + "problem_type": "proof", + "problem": "例12. 设 $\\left\\{a_n\\right\\}$ 为无穷正数列, 若存在常数 $C$, 使得 $\\sum_{k=1}^n \\frac{1}{a_k} \\leqslant C$ 对所有正整数 $n$ 成立.\n求证: 存在常数 $M$, 使得 $\\sum_{k=1}^n \\frac{k^2 \\cdot a_k}{\\left(a_1+a_2+\\cdots+a_k\\right)^2} \\leqslant M$ 对所有正整数 $n$ 成立.", + "solution": "证明:记 $S_n=\\sum_{k=1}^n \\frac{k^2 \\cdot a_k}{\\left(a_1+a_2+\\cdots+a_k\\right)^2}, A_n=a_1+a_2+\\cdots+a_n(n \\geqslant 1)$, $A_0=0$. 所以\n$$\n\\begin{aligned}\nS_n & =\\sum_{k=1}^n \\frac{k^2 \\cdot\\left(A_k-A_{k-1}\\right)}{A_k^2} \\leqslant \\frac{1}{a_1}+\\sum_{k=2}^n \\frac{k^2 \\cdot\\left(A_k-A_{k-1}\\right)}{A_k A_{k-1}} \\\\\n& =\\frac{1}{a_1}+\\sum_{k=2}^n \\frac{k^2}{A_{k-1}}-\\sum_{k=2}^n \\frac{k^2}{A_k}=\\frac{1}{a_1}+\\sum_{k=1}^{n-1} \\frac{(k+1)^2}{A_k}-\\sum_{k=2}^n \\frac{k^2}{A_k} \\\\\n& =\\frac{1}{a_1}+2 \\sum_{k=2}^{n-1} \\frac{k}{A_k}+\\sum_{k=2}^{n-1} \\frac{1}{A_k}+\\frac{4}{A_1}-\\frac{n^2}{A_n} \\leqslant \\frac{5}{a_1}+2 \\sum_{k=1}^n \\frac{k}{A_k}+\\sum_{k=1}^n \\frac{1}{a_k} .\n\\end{aligned}\n$$\n而 $\\sum_{k=1}^n \\frac{k}{A_k}=\\sum_{k=1}^n \\frac{k}{A_k} \\sqrt{a_k} \\cdot \\frac{1}{\\sqrt{a_k}} \\leqslant\\left[\\sum_{k=1}^n\\left(\\frac{k}{A_k} \\sqrt{a_k}\\right)^2\\right]^{1 / 2} \\cdot\\left[\\sum_{k=1}^n\\left(\\frac{1}{\\sqrt{a_k}}\\right)^2\\right]^{1 / 2}$\n$$\n=\\left[\\sum_{k=1}^n \\frac{k^2}{A_k^2} a_k \\cdot \\sum_{k=1}^n \\frac{1}{a_k}\\right]^{1 / 2}=\\left[S_n \\cdot \\sum_{k=1}^n \\frac{1}{a_k}\\right]^{1 / 2},\n$$\n所以\n$$\nS_n \\leqslant \\frac{5}{a_1}+2\\left[S_n \\cdot \\sum_{k=1}^n \\frac{1}{a_k}\\right]^{1 / 2}+\\sum_{k=1}^n \\frac{1}{a_k}\n$$\n由此解出\n$$\n\\sqrt{S_n} \\leqslant \\sqrt{\\sum_{k=1}^n \\frac{1}{a_k}}+\\sqrt{\\frac{5}{a_1}+2 \\sum_{k=1}^n \\frac{1}{a_k}} .\n$$\n取 $M=\\left(\\sqrt{C}+\\sqrt{\\frac{5}{a_1}+2 C}\\right)^2$ 即可.", + "remark": "", + "figures": [] +} \ No newline at end of file diff --git a/processed_dataset/proof/1296.json b/processed_dataset/proof/1296.json new file mode 100644 index 0000000000000000000000000000000000000000..80442aa0e2c9bbad723f57012d71e37e86426632 --- /dev/null +++ b/processed_dataset/proof/1296.json @@ -0,0 +1,8 @@ +{ + "source_file": "./raw_volume-zh/volume5/chapter2.tex", + "problem_type": "proof", + "problem": "例13. 设 $n$ 是一个正整数, 实数 $a_1, a_2, \\cdots, a_n$ 和 $r_1, r_2, \\cdots, r_n$ 满足: $a_1 \\leqslant a_2 \\leqslant \\cdots \\leqslant a_n$ 和 $0 \\leqslant r_1 \\leqslant r_2 \\leqslant \\cdots \\leqslant r_n$, 求证:\n$$\n\\sum_{i=1}^n \\sum_{j=1}^n a_i a_j \\min \\left(r_i, r_j\\right) \\geqslant 0 .\n$$", + "solution": "证明:作一张 $n \\times n$ 的表:\n$$\nA_1=\\left(\\begin{array}{ccc}\na_1 a_1 r_1 & a_1 a_2 r_1 & a_1 a_3 r_1 \\cdots a_1 a_n r_1 \\\\\na_2 a_1 r_1 & a_2 a_2 r_2 & a_2 a_3 r_2 \\cdots a_2 a_n r_2 \\\\\na_3 a_1 r_1 & a_3 a_2 r_2 & a_3 a_3 r_3 \\cdots a_3 a_n r_3 \\\\\n\\cdots & \\cdots & \\\\\n\\cdots & \\cdots & \\\\\na_n a_1 r_1 & a_n a_2 r_2 & a_n a_3 r_3 \\cdots a_n a_n r_n\n\\end{array}\\right)\n$$\n由于 $\\sum_{i=1}^n \\sum_{j=1}^n a_i a_j \\min \\left(r_i, r_j\\right)=\\sum_{j=1}^n a_1 a_j \\min \\left(r_1, r_j\\right)+\\sum_{j=1}^n a_2 a_j \\min \\left(r_2, r_j\\right)+\\cdots+ \\sum_{j=1}^n a_k a_j \\min \\left(r_k, r_j\\right)+\\cdots+\\sum_{j=1}^n a_n a_j \\min \\left(r_n, r_j\\right)$, 它的第 $k$ 项 $\\sum_{j=1}^n a_k a_j \\min \\left(r_k, r_j\\right)=a_k a_1 r_1+a_k a_2 r_2+\\cdots+a_k a_k r_k+a_k a_{k+1} r_k+\\cdots+a_k a_n r_k$ 就是表中第 $k$ 行各元素的和, $k=1,2, \\cdots, n$.\n因此, $\\sum_{i=1}^n \\sum_{j=1}^n a_i a_j \\min \\left(r_i, r_j\\right)$ 就是表 $A_1$ 中所有元素的和.\n另外,此和也可以按以下方式求得: 先取出表 $A_1$ 中第一行、第一列的各元素, 并求其和; 剩下的表记为 $A_2$ (相当于删去 $A_1$ 中的第一行和第一列而得到 $A_2$ ), 再取出表 $A_2$ 中第一行、第一列的各元素, 并求其和; 剩下的表记为 $A_3$ (相当于删去 $A_2$ 中的第一行和第一列而得到 $A_3$ ), 再取出表 $A_3$ 中第一行、第一列的各元素, 并求其和; $\\cdots \\cdots$, 如此得\n$$\n\\begin{aligned}\n& \\sum_{i=1}^n \\sum_{j=1}^n a_i a_j \\min \\left(r_i, r_j\\right) \\\\\n= & \\sum_{k=1}^n r_k\\left(a_k^2+2 a_k\\left(a_{k+1}+a_{k+2}+\\cdots+a_n\\right)\\right) \\text { (这是 } A_k \\text { 中第一行第一列各元素的和) } \\\\\n= & \\sum_{k=1}^n r_k\\left(\\left(a_k+\\sum_{i=k+1}^n a_i\\right)^2-\\left(\\sum_{i=k+1}^n a_i\\right)^2\\right)=\\sum_{k=1}^n r_k\\left(\\left(\\sum_{i=k}^n a_i\\right)^2-\\left(\\sum_{i=k+1}^n a_i\\right)^2\\right) \\\\\n= & r_1\\left(\\sum_{i=1}^n a_i\\right)^2+r_2\\left(\\sum_{i=2}^n a_i\\right)^2+r_3\\left(\\sum_{i=3}^n a_i\\right)^2+\\cdots+r_n\\left(\\sum_{i=n}^n a_i\\right)^2 \\\\\n& -r_1\\left(\\sum_{i=2}^n a_i\\right)^2-r_2\\left(\\sum_{i=3}^n a_i\\right)^2-\\cdots-r_{n-1}\\left(\\sum_{i=n}^n a_i\\right)^2 \\\\\n= & \\sum_{i==}^n\\left(r_i-r_{i-1}\\right)\\left(\\sum_{i=k}^n a_i\\right)^2 \\geqslant 0\\left(\\text { 此处约定 } r_0=0\\right) .\n\\end{aligned}\n$$\n因此结论得证.", + "remark": "", + "figures": [] +} \ No newline at end of file diff --git a/processed_dataset/proof/1297.json b/processed_dataset/proof/1297.json new file mode 100644 index 0000000000000000000000000000000000000000..cfb070462fce76daf58ee5ee0b6447d8faa1943b --- /dev/null +++ b/processed_dataset/proof/1297.json @@ -0,0 +1,8 @@ +{ + "source_file": "./raw_volume-zh/volume5/chapter3.tex", + "problem_type": "proof", + "problem": "例1. 已知 $0^{\\circ} \\leqslant \\alpha \\leqslant 90^{\\circ}$, 求证:\n$$\n2 \\leqslant \\sqrt{5-4 \\sin \\alpha}+\\sin \\alpha \\leqslant \\frac{9}{4} .\n$$", + "solution": "证明:令 $x=\\sqrt{5-4 \\sin \\alpha}$, 则 $\\sin \\alpha=\\frac{5-x^2}{4}$, 由于 $0 \\leqslant \\sin \\alpha \\leqslant 1$, 所以 $1 \\leqslant x \\leqslant \\sqrt{5}$. 令\n$$\n\\begin{aligned}\ny & =\\sqrt{5-4 \\sin \\alpha}+\\sin \\alpha \\\\\n& =x+\\frac{5-x^2}{4} \\\\\n& =-\\frac{1}{4}(x-2)^2+\\frac{9}{4},\n\\end{aligned}\n$$\n由 $1 \\leqslant x \\leqslant \\sqrt{5}$ 便知 $2 \\leqslant y \\leqslant \\frac{9}{4}$. 从而\n$$\n2 \\leqslant \\sqrt{5-4 \\sin \\alpha}+\\sin \\alpha \\leqslant \\frac{9}{4} .\n$$\n说明本题中令 $x=\\sqrt{5-4 \\sin \\alpha}$, 可以使式子变成容易处理的二次函数形式, 从而获得解决.", + "remark": "", + "figures": [] +} \ No newline at end of file diff --git a/processed_dataset/proof/1298.json b/processed_dataset/proof/1298.json new file mode 100644 index 0000000000000000000000000000000000000000..fa51bd69d43236f024ab51b008ef0b68796d3abe --- /dev/null +++ b/processed_dataset/proof/1298.json @@ -0,0 +1,8 @@ +{ + "source_file": "./raw_volume-zh/volume5/chapter3.tex", + "problem_type": "proof", + "problem": "例2. 已知实数 $x 、 y$ 满足 $x^2+y^2-4 x-6 y+9=0$, 求证:\n$$\n19 \\leqslant x^2+y^2+12 x+6 y \\leqslant 99 .\n$$", + "solution": "证明:题设条件可化为\n即\n$$\n\\begin{aligned}\n& (x-2)^2+(y-3)^2=4, \\\\\n& \\left(\\frac{x-2}{2}\\right)^2+\\left(\\frac{y-3}{2}\\right)^2=1 .\n\\end{aligned}\n$$\n令 $\\frac{x-2}{2}=\\cos \\theta, \\frac{y-3}{2}=\\sin \\theta$, 其中 $\\theta \\in[0,2 \\pi)$, 所以\n$$\n\\begin{aligned}\n& x^2+y^2+12 x+6 y \\\\\n= & (4 x+6 y-9)+12 x+6 y \\\\\n= & 16 x+12 y-9 \\\\\n= & 16(2 \\cos \\theta+2)+12(2 \\sin \\theta+3)-9 \\\\\n= & 32 \\cos \\theta+24 \\sin \\theta+59 \\\\\n= & 8(4 \\cos \\theta+3 \\sin \\theta)+59 \\\\\n= & \\left.40 \\cos (\\theta+\\varphi)+59 \\text { (其中 } \\tan \\varphi=\\frac{3}{4}\\right), \\\\\n& 19 \\leqslant 40 \\cos (\\theta+\\varphi)+59 \\leqslant 99, \\\\\n& 19 \\leqslant x^2+y^2+12 x+6 y \\leqslant 99 .\n\\end{aligned}\n$$", + "remark": "", + "figures": [] +} \ No newline at end of file diff --git a/processed_dataset/proof/1299.json b/processed_dataset/proof/1299.json new file mode 100644 index 0000000000000000000000000000000000000000..9926218eb9a3805e1c8df56924c81c2261d51797 --- /dev/null +++ b/processed_dataset/proof/1299.json @@ -0,0 +1,10 @@ +{ + "source_file": "./raw_volume-zh/volume5/chapter3.tex", + "problem_type": "proof", + "problem": "例3. 设 $a 、 b 、 c$ 是三角形的三边长,求证:\n$$\na^2 b(a-b)+b^2 c(b-c)+c^2 a(c-a) \\geqslant 0 .\n$$", + "solution": "证明:令 $a=y+z, b=z+x, c=x+y, x, y, z \\in \\mathbf{R}^{+}$, 则欲证的不等式等价于\n$$\n\\begin{aligned}\n& (y+z)^2(z+x)(y-x)+(z+x)^2(x+y) \\\\\n& (z-y)+(x+y)^2(y+z)(x-z) \\geqslant 0 \\\\\n& \\Leftrightarrow \\quad x y^3+y z^3+z x^3 \\geqslant x^2 y z+x y^2 z+x y z^2 \\\\\n& \\Leftrightarrow \\\\\n& \\frac{x^2}{y}+\\frac{y^2}{z}+\\frac{z^2}{x} \\geqslant x+y+z . \\\\\n&\n\\end{aligned}\n$$\n因为\n$$\n\\frac{x^2}{y}+y \\geqslant 2 x, \\frac{y^2}{z}+z \\geqslant 2 y, \\frac{z^2}{x}+x \\geqslant 2 z,\n$$\n所以\n$$\n\\frac{x^2}{y}+\\frac{y^2}{z}+\\frac{z^2}{x} \\geqslant x+y+z .\n$$\n从而原不等式得证.\n说明在涉及到三角形三边长 $a 、 b 、 c$ 的不等式时,常常作代换 $a=y+z, b=z+x, c=x+y$, 其中 $x, y, z \\in \\mathbf{R}^{+}$. 其实, 如图() 所示, $D 、 E 、 F$ 分别是 $\\triangle A B C$ 的内切圆与边 $B C 、 C A 、 A B$ 的切点, 令 $A E=A F=x, B D=B F=y, C D=C E=z$, 则 $a=y+z, b=z+x, c=x+y$.\n通过代换,关于 $a 、 b 、 c$ 的不等式就转化为关于正实数 $x 、 y 、 z$ 的不等式了.", + "remark": "", + "figures": [ + "./images/volume5/figures/fig-c3i1.png" + ] +} \ No newline at end of file diff --git a/processed_dataset/proof/1300.json b/processed_dataset/proof/1300.json new file mode 100644 index 0000000000000000000000000000000000000000..1432c96f13696c32c9cd5e982394c7fe862284f3 --- /dev/null +++ b/processed_dataset/proof/1300.json @@ -0,0 +1,8 @@ +{ + "source_file": "./raw_volume-zh/volume5/chapter3.tex", + "problem_type": "proof", + "problem": "例4. 设 $a, b, c, d \\in \\mathbf{R}^{+}$, 且\n$$\n\\frac{a^2}{1+a^2}+\\frac{b^2}{1+b^2}+\\frac{c^2}{1+c^2}+\\frac{d^2}{1+d^2}=1 .\n$$\n求证: $a b c d \\leqslant \\frac{1}{9}$.", + "solution": "证明:令 $a=\\tan \\alpha, b=\\tan \\beta, c=\\tan \\gamma, d=\\tan \\delta . \\alpha, \\beta, \\gamma, \\delta \\in \\left(0, \\frac{\\pi}{2}\\right)$. 于是 $\\sin ^2 \\alpha+\\sin ^2 \\beta+\\sin ^2 \\gamma+\\sin ^2 \\delta=1$. 所以\n$3 \\cdot \\sqrt[3]{\\sin ^2 \\alpha \\sin ^2 \\beta \\sin ^2 \\gamma} \\leqslant \\sin ^2 \\alpha+\\sin ^2 \\beta+\\sin ^2 \\gamma=\\cos ^2 \\delta$,\n$3 \\cdot \\sqrt[3]{\\sin ^2 \\alpha \\sin ^2 \\beta \\sin ^2 \\delta} \\leqslant \\sin ^2 \\alpha+\\sin ^2 \\beta+\\sin ^2 \\delta=\\cos ^2 \\gamma$,\n$3 \\cdot \\sqrt[3]{\\sin ^2 \\alpha \\sin ^2 \\gamma \\sin ^2 \\delta} \\leqslant \\sin ^2 \\alpha+\\sin ^2 \\gamma+\\sin ^2 \\delta=\\cos ^2 \\beta$,\n$3 \\cdot \\sqrt[3]{\\sin ^2 \\beta \\sin ^2 \\gamma \\sin ^2 \\delta} \\leqslant \\sin ^2 \\beta+\\sin ^2 \\gamma+\\sin ^2 \\delta=\\cos ^2 \\alpha$.\n上面四式相乘, 得 $\\tan ^2 \\alpha \\tan ^2 \\beta \\tan ^2 \\gamma \\tan ^2 \\delta \\leqslant \\frac{1}{81}$.\n因此 $\\quad a b c d=\\tan \\alpha \\tan \\beta \\tan \\gamma \\tan \\delta \\leqslant \\frac{1}{9}$.", + "remark": "", + "figures": [] +} \ No newline at end of file diff --git a/processed_dataset/proof/1301.json b/processed_dataset/proof/1301.json new file mode 100644 index 0000000000000000000000000000000000000000..e87a08a8c759103df6ad3b76170a2dd0d7aa46d2 --- /dev/null +++ b/processed_dataset/proof/1301.json @@ -0,0 +1,8 @@ +{ + "source_file": "./raw_volume-zh/volume5/chapter3.tex", + "problem_type": "proof", + "problem": "例6. 已知 $a, b, c \\geqslant 0, a+b+c=1$. 求证:\n$$\n\\sqrt{a+\\frac{1}{4}(b-c)^2}+\\sqrt{b}+\\sqrt{c} \\leqslant \\sqrt{3} .\n$$", + "solution": "证法 1 不妨设 $b \\geqslant c$. 令 $\\sqrt{b}=x+y, \\sqrt{c}=x-y$, 则\n$$\nb-c=4 x y, a=1-2 x^2-2 y^2, x \\leqslant \\frac{1}{\\sqrt{2}} .\n$$\n原式左边 $=\\sqrt{1-2 x^2-2 y^2+4 x^2 y^2}+2 x$\n$$\n\\leqslant \\sqrt{1-2 x^2}+x+x \\leqslant \\sqrt{3} \\text {. }\n$$\n最后一步由柯西不等式得到.", + "remark": "", + "figures": [] +} \ No newline at end of file diff --git a/processed_dataset/proof/1302.json b/processed_dataset/proof/1302.json new file mode 100644 index 0000000000000000000000000000000000000000..3424d0ad67080ccb4218447cd5c44c38e5bf11d0 --- /dev/null +++ b/processed_dataset/proof/1302.json @@ -0,0 +1,8 @@ +{ + "source_file": "./raw_volume-zh/volume5/chapter3.tex", + "problem_type": "proof", + "problem": "例6. 已知 $a, b, c \\geqslant 0, a+b+c=1$. 求证:\n$$\n\\sqrt{a+\\frac{1}{4}(b-c)^2}+\\sqrt{b}+\\sqrt{c} \\leqslant \\sqrt{3} .\n$$", + "solution": "证法 2 令 $a=u^2, b=v^2, c=w^2$, 则 $u^2+v^2+w^2=1$, 于是待证不等式变为\n$$\n\\sqrt{u^2+\\frac{\\left(v^2-w^2\\right)^2}{4}}+v+w \\leqslant \\sqrt{3} . \\label{(1)}\n$$\n注意到\n$$\n\\begin{aligned}\nu^2+\\frac{\\left(v^2-w^2\\right)^2}{4} & =1-\\left(v^2+w^2\\right)+\\frac{\\left(v^2-w^2\\right)^2}{4}=\\frac{4-4\\left(v^2+w^2\\right)+\\left(v^2-w^2\\right)^2}{4} \\\\\n& =\\frac{4-4\\left(v^2+w^2\\right)+\\left(v^2+w^2\\right)^2-4 v^2 w^2}{4} \\\\\n& =\\frac{\\left(2-v^2-w^2\\right)^2-4 v^2 w^2}{4} \\\\\n& =\\frac{\\left(2-v^2-w^2-2 w w\\right)\\left(2-v^2-w^2+2 w w\\right)}{4} \\\\\n& =\\frac{\\left[2-(v+w)^2\\right]\\left[2-(v-w)^2\\right]}{4} \\leqslant 1-\\frac{(v+w)^2}{2} .\n\\end{aligned}\n$$\n(注意 $(v+w)^2 \\leqslant 2\\left(v^2+w^2\\right) \\leqslant 2$) 将上式代入 (1), 得\n$$\n\\sqrt{1-\\frac{(v+w)^2}{2}}+v+w \\leqslant \\sqrt{3}\n$$\n令 $\\frac{v+w}{2}=x$, 将上述不等式改写为 $\\sqrt{1-2 x^2}+2 x \\leqslant \\sqrt{3}$, 以下同证法 1 . 说明证法 2 解释了证法 1 中替换的动机.", + "remark": "", + "figures": [] +} \ No newline at end of file diff --git a/processed_dataset/proof/1303.json b/processed_dataset/proof/1303.json new file mode 100644 index 0000000000000000000000000000000000000000..fa88c4164914eba7420291fe7dcfa2277f1d9351 --- /dev/null +++ b/processed_dataset/proof/1303.json @@ -0,0 +1,8 @@ +{ + "source_file": "./raw_volume-zh/volume5/chapter3.tex", + "problem_type": "proof", + "problem": "例7. 设 $a, b, c \\in \\mathbf{R}^{+}$, 且 $a b c=1$, 求证:\n(1) $\\frac{1}{1+2 a}+\\frac{1}{1+2 b}+\\frac{1}{1+2 c} \\geqslant 1$;\n(2) $\\frac{1}{1+\\frac{1}{a+b}}+\\frac{1}{1+b+c}+\\frac{1}{1+c+a} \\leqslant 1$.", + "solution": "证明:(1) 证法 1 设 $a=\\frac{x}{y}, b=\\frac{y}{z}, c=\\frac{z}{x}, x, y, z \\in \\mathbf{R}^{+}$, 则原不等式等价于\n$$\ns=\\frac{y}{y+2 x}+\\frac{z}{z+2 y}+\\frac{x}{x+2 z} \\geqslant 1 .\n$$\n利用 Cauchy 不等式, 得\n$$\ns \\cdot[y(y+2 x)+z(z+2 y)+x(x+2 z)] \\geqslant(x+y+z)^2,\n$$\n即有 $s \\geqslant 1$ 成立.\n证法 2 首先我们证明\n$$\n\\frac{1}{1+2 a} \\geqslant \\frac{a^{-\\frac{2}{3}}}{a^{-\\frac{2}{3}}+b^{-\\frac{2}{3}}+c^{-\\frac{2}{3}}} . \\label{eq1}\n$$\n式\\ref{eq1}等价于\n$$\nb^{-\\frac{2}{3}}+c^{-\\frac{2}{3}} \\geqslant 2 a^{\\frac{1}{3}} \\text {. }\n$$\n又由于 $b^{-\\frac{2}{3}}+c^{-\\frac{2}{3}} \\geqslant 2 \\cdot(b c)^{-\\frac{1}{3}}=2 a^{\\frac{1}{3}}$, 故(1)成立.\n同理, 有\n$$\n\\begin{aligned}\n& \\frac{1}{1+2 b} \\geqslant \\frac{b^{-\\frac{2}{3}}}{a^{-\\frac{2}{3}}+b^{-\\frac{2}{3}}+c^{-\\frac{2}{3}}}, \\label{eq2} \\\\\n& \\frac{1}{1+2 c} \\geqslant \\frac{c^{-\\frac{2}{3}}}{a^{-\\frac{2}{3}}+b^{-\\frac{2}{3}}+c^{-\\frac{2}{3}}} . \\label{eq3}\n\\end{aligned}\n$$\n将式\\ref{eq1},\\ref{eq2},式\\ref{eq3}相加即得原不等式成立.\n(2) 令 $a=x^3, b=y^3, c==z^3, x, y, z \\in \\mathbf{R}^{+}$. 那么, 由题设得 $x y z=1$. 利用 $x^3+y^3 \\geqslant x^2 y+y^2 x$, 有\n$$\n\\begin{aligned}\n\\frac{1}{1+a+b} & =\\frac{1}{1+x^3+y^3} \\leqslant \\frac{1}{1+x^2 y+x y^2} \\\\\n& =\\frac{1}{x y z+x^2 y+y^2 x}=\\frac{1}{x y(x+y+z)} \\\\\n& =\\frac{z}{x+y+z} .\n\\end{aligned}\n$$\n同理,有\n$$\n\\begin{aligned}\n& \\frac{1}{1+b+c} \\leqslant \\frac{x}{x+y+z}, \\\\\n& \\frac{1}{1+c+a} \\leqslant \\frac{y}{x+y+z} .\n\\end{aligned}\n$$\n三式相加即得原不等式成立.\n说明当三数的乘积为 1 时, 本题的两种代换方法都是常用的.", + "remark": "", + "figures": [] +} \ No newline at end of file diff --git a/processed_dataset/proof/1304.json b/processed_dataset/proof/1304.json new file mode 100644 index 0000000000000000000000000000000000000000..1dfc14315be1a25022afb9938d5822c0814bad91 --- /dev/null +++ b/processed_dataset/proof/1304.json @@ -0,0 +1,8 @@ +{ + "source_file": "./raw_volume-zh/volume5/chapter3.tex", + "problem_type": "proof", + "problem": "例8. 设 $x, y, z \\in \\mathbf{R}^{+}$, 且 $\\frac{1}{x}+\\frac{1}{y}+\\frac{1}{z}=1$, 求证:\n$$\n\\sqrt{x+y z}+\\sqrt{y+z x}+\\sqrt{z+x y} \\geqslant \\sqrt{x y z}+\\sqrt{x}+\\sqrt{y}+\\sqrt{z} .\n$$", + "solution": "证明:令 $x=\\frac{1}{\\alpha}, y=\\frac{1}{\\beta}, z=\\frac{1}{\\gamma}$, 则 $\\alpha+\\beta+\\gamma=1$.\n原不等式等价于\n$$\n\\sum_{c y c} \\sqrt{\\frac{1}{\\alpha}+\\frac{1}{\\beta \\gamma}} \\geqslant \\sqrt{\\frac{1}{\\alpha \\beta \\gamma}}+\\sum_{c y c} \\sqrt{\\frac{1}{\\alpha}},\n$$\n即\n$$\n\\sum_{c y c} \\sqrt{\\alpha+\\beta \\gamma} \\geqslant \\sum_{c y c} \\sqrt{\\alpha^2}+\\sum_{c y c} \\sqrt{\\beta \\gamma},\n$$\n即\n$$\n\\begin{aligned}\n& \\sum_{c y c} \\sqrt{\\alpha \\cdot \\sum_{c y c} \\alpha+\\beta \\gamma} \\geqslant \\sum_{c y c} \\sqrt{\\alpha^2}+\\sum_{c y c} \\sqrt{\\beta \\gamma}, \\\\\n& \\sum_{c y c} \\sqrt{(\\alpha+\\beta)(\\alpha+\\gamma)} \\geqslant \\sum_{c y c}\\left(\\sqrt{\\alpha^2}+\\sqrt{\\beta \\gamma}\\right) .\n\\end{aligned}\n$$\n又不难证明 $\\quad \\sqrt{(\\alpha+\\beta)(\\alpha+\\gamma)} \\geqslant \\sqrt{\\alpha^2}+\\sqrt{\\beta \\gamma}$.\n故原不等式成立.", + "remark": "", + "figures": [] +} \ No newline at end of file diff --git a/processed_dataset/proof/1305.json b/processed_dataset/proof/1305.json new file mode 100644 index 0000000000000000000000000000000000000000..65bb1637a4ca201a9325c816b4e8bff4fde5b2a3 --- /dev/null +++ b/processed_dataset/proof/1305.json @@ -0,0 +1,8 @@ +{ + "source_file": "./raw_volume-zh/volume5/chapter3.tex", + "problem_type": "proof", + "problem": "例9. 设 $x, y, z \\in \\mathbf{R}^{+}$, 求证:\n$$\n\\frac{y^2-x^2}{z+x}+\\frac{z^2-y^2}{x+y}+\\frac{x^2-z^2}{y+z} \\geqslant 0 \\text {. (W. Janous 不等式) }\n$$", + "solution": "分析:左端式子分母是变量和的形式, 难以直接处理, 故先将它们代换掉,简化分母.\n证明令 $x+y=c, y+z=a, z+x=b$, 原不等式等价于\n$$\n\\frac{c(a-b)}{b}+\\frac{a(b-c)}{c}+\\frac{b(c-a)}{a} \\geqslant 0,\n$$\n即\n$$\n\\frac{a c^2(a-b)+a^2 b(b-c)+b^2 c(c-a)}{a b c} \\geqslant 0 .\n$$\n故只须证明\n$$\na^2 c^2+a^2 b^2+b^2 c^2-a b c^2-a^2 b c-a b^2 c \\geqslant 0 .\n$$\n这是很显然的.", + "remark": "", + "figures": [] +} \ No newline at end of file diff --git a/processed_dataset/proof/1306.json b/processed_dataset/proof/1306.json new file mode 100644 index 0000000000000000000000000000000000000000..2d72297da617e6f4bffc2129bbd265ea47f57dc0 --- /dev/null +++ b/processed_dataset/proof/1306.json @@ -0,0 +1,8 @@ +{ + "source_file": "./raw_volume-zh/volume5/chapter3.tex", + "problem_type": "proof", + "problem": "例10. 设 $x_1 、 x_2 、 x_3$ 是正数,求证:\n$$\nx_1 x_2 x_3 \\geqslant\\left(x_2+x_3-x_1\\right)\\left(x_1+x_3-x_2\\right)\\left(x_1+x_2-x_3\\right) .\n$$", + "solution": "证明:不妨设 $x_1 \\geqslant x_2 \\geqslant x_3>0$.\n令 $x_1=x_3+\\delta_1, x_2=x_3+\\delta_2$, 则 $\\delta_1 \\geqslant \\delta_2 \\geqslant 0$. 于是\n$$\n\\begin{aligned}\n& x_1 x_2 x_3-\\left(x_2+x_3-x_1\\right)\\left(x_1+x_3-x_2\\right)\\left(x_1+x_2-x_3\\right) \\\\\n= & \\left(x_3+\\delta_1\\right)\\left(x_3+\\delta_2\\right) x_3-\\left(x_3+\\delta_2-\\delta_1\\right)\\left(x_3+\\delta_1-\\delta_2\\right)\\left(x_3+\\delta_1+\\delta_2\\right) \\\\\n= & \\left(x_3^2+\\delta_1 x_3+\\delta_2 x_3+\\delta_1 \\delta_2\\right) x_3-\\left[x_3^2-\\left(\\delta_1-\\delta_2\\right)^2\\right]\\left(x_3+\\delta_1+\\delta_2\\right) \\\\\n= & x_3 \\delta_1 \\delta_2+x_3^2\\left(x_3+\\delta_1+\\delta_2\\right)-\\left[x_3^2-\\left(\\delta_1-\\delta_2\\right)^2\\right]\\left(x_3+\\delta_1+\\delta_2\\right) \\\\\n= & x_3 \\delta_1 \\delta_2+\\left(\\delta_1-\\delta_2\\right)^2\\left(x_3+\\delta_1+\\delta_2\\right) \\\\\n\\geqslant & 0 .\n\\end{aligned}\n$$\n所以\n$$\nx_1 x_2 x_3 \\geqslant\\left(x_2+x_3-x_1\\right)\\left(x_1+x_3-x_2\\right)\\left(x_1+x_2-x_3\\right) .\n$$\n说明本题用的代换方法称为\"增量代换法\".", + "remark": "", + "figures": [] +} \ No newline at end of file diff --git a/processed_dataset/proof/1307.json b/processed_dataset/proof/1307.json new file mode 100644 index 0000000000000000000000000000000000000000..ea5303d129bc2d5415f1748a024332da74b1c35d --- /dev/null +++ b/processed_dataset/proof/1307.json @@ -0,0 +1,8 @@ +{ + "source_file": "./raw_volume-zh/volume5/chapter3.tex", + "problem_type": "proof", + "problem": "例12. 已知 $x 、 y 、 z$ 都是正数,求证:\n$$\n\\begin{aligned}\n& x(y+z-x)^2+y(z+x-y)^2+z(x+y-z)^2 \\\\\n\\geqslant & 2 x y z\\left(\\frac{x}{y+z}+\\frac{y}{z+x}+\\frac{z}{x+y}\\right),\n\\end{aligned}\n$$\n等号当且仅当 $x=y=z$ 时成立.", + "solution": "证明:令 $a=y+z-x, b=x+z-y, c=x+y-z$, 则\n$$\nx=\\frac{b+c}{2}, y=\\frac{a+c}{2}, z=\\frac{a+b}{2} .\n$$\n于是原不等式等价于\n$$\n\\frac{1}{2} \\sum a^2(b+c) \\geqslant 2 \\frac{(a+b)(b+c)(c+a)}{8} \\cdot \\sum \\frac{\\frac{b+c}{2}}{a+\\frac{b+c}{2}},\n$$\n即\n$$\n\\begin{aligned}\n& 2\\left[a^2(b+c)+b^2(c+a)+c^2(a+b)\\right] \\\\\n\\geqslant & (a+b)(b+c)(c+a) \\cdot \\sum \\frac{b+c}{2 a+b+c} .\n\\end{aligned}\n$$\n下面我们证明\n$$\na^2(b+c)+b^2(c+a) \\geqslant(a+b)(b+c)(c+a) \\cdot \\frac{a+b}{2 c+a+b} . \\label{(1)}\n$$\n注意到(1)等价于 $\\frac{a^2}{a+c}+\\frac{b^2}{b+c} \\geqslant \\frac{(a+b)^2}{a+b+2 c}$.\n由 Cauchy 不等式,这是显然的.\n同理还有类似(1)的其他两式, 相加即得原不等式成立.", + "remark": "", + "figures": [] +} \ No newline at end of file diff --git a/processed_dataset/proof/1308.json b/processed_dataset/proof/1308.json new file mode 100644 index 0000000000000000000000000000000000000000..421b188ec7e546a3fcb894de72c19b6b8011bc34 --- /dev/null +++ b/processed_dataset/proof/1308.json @@ -0,0 +1,8 @@ +{ + "source_file": "./raw_volume-zh/volume5/chapter3.tex", + "problem_type": "proof", + "problem": "例13. 设 $a, b, c \\in \\mathbf{R}^{+}$, 求证:\n$$\n\\frac{b^3}{a^2+8 b c}+\\frac{c^3}{b^2+8 c a}+\\frac{a^3}{c^2+8 a b} \\geqslant \\frac{a+b+c}{9} .\n$$", + "solution": "证明:记不等式的左端为 $M$, 令\n$$\n\\begin{aligned}\nS & =\\left(a^2+8 b c\\right)+\\left(b^2+8 c a\\right)+\\left(c^2+8 a b\\right) \\\\\n& =(a+b+c)^2+6(a b+b c+c a) \\\\\n& \\leqslant 3(a+b+c)^2 .\n\\end{aligned}\n$$\n所以\n$$\n\\begin{aligned}\n3= & \\frac{1}{M} \\cdot\\left(\\frac{b^3}{a^2+8 b c}+\\frac{c^3}{b^2+8 c a}+\\frac{a^3}{c^2+8 a b}\\right)+\\frac{1}{S}\\left(\\left(a^2+8 b c\\right)\\right. \\\\\n& \\left.+\\left(b^2+8 c a\\right)+\\left(c^2+8 a b\\right)\\right)+\\frac{1+1+1}{3} \\\\\n= & \\sum\\left[\\frac{b^3}{M\\left(a^2+8 b c\\right)}+\\frac{a^2+8 b c}{S}+\\frac{1}{3}\\right] \\\\\n\\geqslant & 3 \\sqrt[3]{\\frac{b^3}{3 S M}}+3 \\sqrt[3]{\\frac{a^3}{3 S M}}+3 \\sqrt[3]{\\frac{c^3}{3 S M}} \\\\\n= & \\frac{3(a+b+c)}{\\sqrt[3]{3 S M}} .\n\\end{aligned}\n$$\n因此 $3 S M \\geqslant(a+b+c)^3$, 故有 $M \\geqslant \\frac{1}{9}(a+b+c)$ 成立.", + "remark": "", + "figures": [] +} \ No newline at end of file diff --git a/processed_dataset/proof/1309.json b/processed_dataset/proof/1309.json new file mode 100644 index 0000000000000000000000000000000000000000..7c587f5a94c8169f15959982cf7b6dfb93b36c2c --- /dev/null +++ b/processed_dataset/proof/1309.json @@ -0,0 +1,8 @@ +{ + "source_file": "./raw_volume-zh/volume5/chapter3.tex", + "problem_type": "proof", + "problem": "例14. 已知非负实数 $a 、 b 、 c$ 满足: $a+b+c=1$, 求证:\n$$\n\\begin{aligned}\n2 & \\leqslant\\left(1-a^2\\right)^2+\\left(1-b^2\\right)^2+\\left(1-c^2\\right)^2 \\\\\n& \\leqslant(1+a)(1+b)(1+c),\n\\end{aligned}\n$$\n并求出等号成立的条件.", + "solution": "证明:设 $a b+b c+c a=m, a b c=n$, 则\n$$\n(x-a)(x-b)(x-c)=x^3-x^2+m x-n .\n$$\n令 $x=a$, 则有 $a^3=a^2-m a+n$ (注意: 这起到了降次的作用!). 于是\n$$\n\\begin{aligned}\n\\sum_{c y c} a^3 & =\\sum_{c y c} a^2-m \\cdot \\sum_{c y c} a+3 n . \\\\\n\\sum_{c y c} a^4 & =\\sum_{c y c} a^3-m \\sum_{c y c} a^2+n \\sum_{c y c} a \\\\\n& =(1-m) \\sum_{c y c} a^2-m \\sum_{c y c} a+n \\sum_{c y c} a+3 n \\\\\n& =(1-m)(1-2 m)-m+n+3 n .\n\\end{aligned}\n$$\n所以\n$$\n\\begin{aligned}\n\\sum_{c y c}\\left(1-a^2\\right)^2 & =3-2 \\sum_{c y c} a^2+\\sum_{c y c} a^4 \\\\\n& =3-2(1-2 m)+2 m^2-3 m+1-m+4 n \\\\\n& =2 m^2+4 n+2 \\geqslant 2,\n\\end{aligned}\n$$\n等号当 $a 、 b 、 c$ 中有 2 个为 0 时取到.\n又因为\n$$\n(1+a)(1+b)(1+c)=2+m+n,\n$$\n则 $2 m^2+4 n+2 \\leqslant 2+m+n$ 相当于 $3 n \\leqslant m-2 m^2$, 即\n$$\n3 a b c \\leqslant m(1-2 m)=(a b+b c+c a)\\left(a^2+b^2+c^2\\right),\n$$\n即\n$$\n3 \\leqslant\\left(\\frac{1}{a}+\\frac{1}{b}+\\frac{1}{c}\\right)\\left(a^2+b^2+c^2\\right) . \\label{(1)}\n$$\n而由 Cauchy 不等式可得\n$$\n3\\left(a^2+b^2+c^2\\right) \\geqslant(a+b+c)^2=a+b+c .\n$$\n于是\n$$\n\\begin{aligned}\n& 3\\left(a^2+b^2+c^2\\right)\\left(\\frac{1}{a}+\\frac{1}{b}+\\frac{1}{c}\\right) \\\\\n\\geqslant & (a+b+c)\\left(\\frac{1}{a}+\\frac{1}{b}+\\frac{1}{c}\\right) \\geqslant 9 .\n\\end{aligned}\n$$\n故(1)成立,且等号当 $a=b=c=\\frac{1}{3}$ 时成立.", + "remark": "", + "figures": [] +} \ No newline at end of file diff --git a/processed_dataset/proof/1310.json b/processed_dataset/proof/1310.json new file mode 100644 index 0000000000000000000000000000000000000000..a808ddf371db5f7ce6cd8e404342c390545630f2 --- /dev/null +++ b/processed_dataset/proof/1310.json @@ -0,0 +1,8 @@ +{ + "source_file": "./raw_volume-zh/volume5/chapter3.tex", + "problem_type": "proof", + "problem": "例15. (1)设实数 $x 、 y 、 z$ 都不等于 $1, x y z=1$, 求证:\n$$\n\\frac{x^2}{(x-1)^2}+\\frac{y^2}{(y-1)^2}+\\frac{z^2}{(z-1)^2} \\geqslant 1 \\text {. }\n$$\n(2) 求证: 存在无穷多组三元有理数组 $(x, y, z)$, 使得上述不等式等号成立.", + "solution": "证法 1 (1)\n$$\n\\begin{gathered}\n\\text { 令 } \\frac{x}{x-1}=a, \\frac{y}{y-1}=b, \\frac{z}{z-1}=c, \\text { 则 } \\\\\nx==\\frac{a}{a-1}, y=\\frac{b}{b-1}, z=\\frac{c}{c-1} .\n\\end{gathered}\n$$\n由题设条件 $x y z=1$ 得,\n$$\n\\begin{aligned}\n& a b c=(a-1)(b-1)(c-1), \\\\\n& a+b+c-1=a b+b c+c a,\n\\end{aligned}\n$$\n即\n$$\na+b+c-1=a b+b c+c a\n$$\n所以\n$$\n\\begin{aligned}\na^2+b^2+c^2 & =(a+b+c)^2-2(a b+b c+c a) \\\\\n& =(a+b+c)^2-2(a+b+c-1) \\\\\n& =(a+b+c-1)^2+1 \\geqslant 1, \\\\\n\\frac{x^2}{(x-1)^2} & +\\frac{y^2}{(y-1)^2}+\\frac{z^2}{(z-1)^2} \\geqslant 1 .\n\\end{aligned}\n$$\n从而\n$$\n\\frac{x^2}{(x-1)^2}+\\frac{y^2}{(y-1)^2}+\\frac{z^2}{(z-1)^2} \\geqslant 1 \\text {. }\n$$\n(2) 令 $(x, y, z)=\\left(-\\frac{k}{(k-1)^2}, k-k^2, \\frac{k-1}{k^2}\\right), k$ 是正整数, 则 $(x, y, z)$ 是三元有理数组, $x 、 y 、 z$ 都不等于 1 , 且对于不同的正整数 $k$, 三元有理数组 $(x, y, z)$ 是互不相同的.\n此时\n$$\n\\begin{aligned}\n& \\frac{x^2}{(x-1)^2}+\\frac{y^2}{(y-1)^2}+\\frac{z^2}{(z-1)^2} \\\\\n= & \\frac{k^2}{\\left(k^2-k+1\\right)^2}+\\frac{\\left(k-k^2\\right)^2}{\\left(k^2-k+1\\right)^2}+\\frac{(k-1)^2}{\\left(k^2-k+1\\right)^2} \\\\\n= & \\frac{k^4-2 k^3+3 k^2-2 k+1}{\\left(k^2-k+1\\right)^2}=1,\n\\end{aligned}\n$$\n从而命题得证.", + "remark": "", + "figures": [] +} \ No newline at end of file diff --git a/processed_dataset/proof/1311.json b/processed_dataset/proof/1311.json new file mode 100644 index 0000000000000000000000000000000000000000..a0ea847dbd3189a8222798918d02ab1c60e1216b --- /dev/null +++ b/processed_dataset/proof/1311.json @@ -0,0 +1,8 @@ +{ + "source_file": "./raw_volume-zh/volume5/chapter3.tex", + "problem_type": "proof", + "problem": "例15. (1)设实数 $x 、 y 、 z$ 都不等于 $1, x y z=1$, 求证:\n$$\n\\frac{x^2}{(x-1)^2}+\\frac{y^2}{(y-1)^2}+\\frac{z^2}{(z-1)^2} \\geqslant 1 \\text {. }\n$$\n(2) 求证: 存在无穷多组三元有理数组 $(x, y, z)$, 使得上述不等式等号成立.", + "solution": "证法 2 (1) 由 $x y z=1$, 可设 $p=x, q=1, r=\\frac{1}{y}$, 得 $x=\\frac{p}{q}, y=\\frac{q}{r}, z= \\frac{1}{x y}=\\frac{r}{p}, p 、 q 、 r$ 互不相等.\n故\n$$\n\\begin{gathered}\n\\frac{x^2}{(x-1)^2}+\\frac{y^2}{(y-1)^2}+\\frac{z^2}{(z-1)^2} \\geqslant 1 \\\\\n\\Leftrightarrow \\frac{p^2}{(p-q)^2}+\\frac{q^2}{(q-r)^2}+\\frac{r^2}{(r-p)^2} \\geqslant 1 .\n\\end{gathered} \\label{eq1}\n$$\n令 $a=\\frac{p}{p-q}, b=\\frac{q}{q-r}, c=\\frac{r}{r-p}$, 则 式\\ref{eq1} 化为 $\\sum_{c y c} a^2 \\geqslant 1$. 由于\n$$\n\\begin{gathered}\n\\frac{-1+a}{a}=\\frac{q}{p}, \\frac{-1+b}{b}=\\frac{r}{q}, \\frac{-1+c}{c}=\\frac{p}{r}, \\\\\n\\frac{-1+a}{a} \\cdot \\frac{-1+b}{b} \\cdot \\frac{-1+c}{c}=1, \\\\\n1-\\sum_{c y c} a+\\sum_{c y c} a b=0,\n\\end{gathered} \\label{eq2}\n$$\n由式\\ref{eq2}可得\n$$\n1-\\sum_{c y c} a^2=-(a+b+c-1)^2 \\leqslant 0\n$$\n所以 $\\sum_{c y c} a^2 \\geqslant 1$, 从而\\ref{eq1}式成立.\n(2) 令 $b=\\frac{t^2+t}{t^2+t+1}, c=\\frac{t+1}{t^2+t+1}, a=-\\frac{b c}{b+c}$, 其中 $t$ 可取除 0、- 1 外的一切有理数, 改变 $t$, 其中使得 $b 、 c 、 a$ 中有某个为 1 的至多只有有限个, 这样就得到无穷多组三元有理数组 $(a, b, c), a 、 b 、 c$ 都不等于 1 , 使得 $\\sum_{c y c} a= \\sum_{c y c} a^2=1$, 而由 $(x, y, z)=\\left(\\frac{a}{a-1}, \\frac{b}{b-1}, \\frac{c}{c-1}\\right)$ 知 (2)成立.", + "remark": "", + "figures": [] +} \ No newline at end of file diff --git a/processed_dataset/proof/1312.json b/processed_dataset/proof/1312.json new file mode 100644 index 0000000000000000000000000000000000000000..fab9a9d3dc6406309924878c74c1ab11873c3b8c --- /dev/null +++ b/processed_dataset/proof/1312.json @@ -0,0 +1,8 @@ +{ + "source_file": "./raw_volume-zh/volume5/chapter4.tex", + "problem_type": "proof", + "problem": "例1. 求证: 对任何实数 $x 、 y 、 z$,下述三个不等式不可能同时成立:\n$$\n|x|<|y-z|,|y|<|z-x|,|z|<|x-y| .\n$$", + "solution": "证明:用反证法,假设三个不等式都成立,那么则有\n$$\n\\left\\{\\begin{array}{l}\nx^2<(y-z)^2, \\\\\ny^2<(z-x)^2, \\\\\nz^2<(x-y)^2 .\n\\end{array}\\right.\n$$\n$\\left\\{\\begin{array}{l}(x-y+z)(x+y-z)<0, \\\\ (y-z+x)(y+z-x)<0, \\\\ (z-x+y)(z+x-y)<0 .\\end{array}\\right.$\n上面三个不等式相乘即得\n$$\n(x+y-z)^2(y+z-x)^2(z+x-y)^2<0 .\n$$\n矛盾!", + "remark": "", + "figures": [] +} \ No newline at end of file diff --git a/processed_dataset/proof/1313.json b/processed_dataset/proof/1313.json new file mode 100644 index 0000000000000000000000000000000000000000..d849d54cc84a6927263088f08b221bcb7607ad8f --- /dev/null +++ b/processed_dataset/proof/1313.json @@ -0,0 +1,8 @@ +{ + "source_file": "./raw_volume-zh/volume5/chapter4.tex", + "problem_type": "proof", + "problem": "例2. 若 $a 、 b 、 c 、 d$ 为非负整数, 且 $(a+b)^2+3 a+2 b=(c+d)^2+3 c+ 2 d$. 求证:\n$$\na=c, b=d .\n$$", + "solution": "证明:先证明 $a+b=c+d$. 用反证法.\n若 $a+b \\neq c+d$, 不妨设 $a+b>c+d$, 则 $a+b \\geqslant c+d+1$. 故\n$$\n\\begin{aligned}\n(a+b)^2+3 a+2 b & =(a+b)^2+2(a+b)+a \\\\\n& \\geqslant(c+d+1)^2+2(c+d+1)+a \\\\\n& =(c+d)^2+4(c+d)+3+a \\\\\n& >(c+d)^2+3 c+2 d .\n\\end{aligned}\n$$\n矛盾! 所以 $a+b=c+d$, 代入原式即得 $a=c$, 进而有 $b=d$.\n说明对于整数 $x 、 y$, 若 $x>y$, 则 $x \\geqslant y+1$. 这一性质在处理与整数有关的不等式时很有用.", + "remark": "", + "figures": [] +} \ No newline at end of file diff --git a/processed_dataset/proof/1314.json b/processed_dataset/proof/1314.json new file mode 100644 index 0000000000000000000000000000000000000000..80d5f899a2cecb2434f6859de8c5b50aa44cd00d --- /dev/null +++ b/processed_dataset/proof/1314.json @@ -0,0 +1,8 @@ +{ + "source_file": "./raw_volume-zh/volume5/chapter4.tex", + "problem_type": "proof", + "problem": "例3. 已知 12 个实数 $a_1, a_2, \\cdots, a_{12}$ 满足:\n$$\n\\left\\{\\begin{array}{l}\na_2\\left(a_1-a_2+a_3\\right)<0, \\\\\na_3\\left(a_2-a_3+a_4\\right)<0, \\\\\n\\cdots \\cdots \\\\\na_{11}\\left(a_{10}-a_{11}+a_{12}\\right)<0 .\n\\end{array}\\right.\n$$\n求证: 从这些数中至少可找到 3 个正数和 3 个负数.", + "solution": "证明:用反证法, 不妨设 $a_1, a_2, \\cdots, a_{12}$ 中至多有两个负数, 则存在 $1 \\leqslant k \\leqslant 9$ 使 $a_k 、 a_{k+1} 、 a_{k+2} 、 a_{k+3}$ 都是非负实数.\n由题设可得 $\\left\\{\\begin{array}{l}a_{k+1}\\left(a_k-a_{k+1}+a_{k+2}\\right)<0, \\\\ a_{k+2}\\left(a_{k+1}-a_{k+2}+a_{k+3}\\right)<0 .\\end{array}\\right.$\n又因为 $a_{k+1} \\geqslant 0, a_{k+2} \\geqslant 0$, 则 $a_{k+1}>0, a_{k+2}>0$, 且\n$$\n\\left\\{\\begin{array}{l}\na_k-a_{k+1}+a_{k+2}<0, \\\\\na_{k+1}-a_{k+2}+a_{k+3}<0 .\n\\end{array}\\right.\n$$\n两式相加得 $a_k+a_{k+3}<0$, 此式与 $a_k \\geqslant 0, a_{k+3} \\geqslant 0$ 矛盾! 所以 $a_1, a_2, \\cdots, a_{12}$ 中至少有 3 个正数和 3 个负数.", + "remark": "", + "figures": [] +} \ No newline at end of file diff --git a/processed_dataset/proof/1315.json b/processed_dataset/proof/1315.json new file mode 100644 index 0000000000000000000000000000000000000000..b7dadecded79f82de6dbf4fd36063535184e9be7 --- /dev/null +++ b/processed_dataset/proof/1315.json @@ -0,0 +1,8 @@ +{ + "source_file": "./raw_volume-zh/volume5/chapter4.tex", + "problem_type": "proof", + "problem": "例4. 已知正整数 $a 、 b 、 c 、 d 、 n$ 满足: $n^21$, 有 $p \\geqslant q+1$, 则\n$$\n\\frac{p}{q} \\geqslant 1+\\frac{1}{q} . \\label{eq1}\n$$\n又由 $b=\\frac{a p}{q}$ 得出 $q \\mid a p$, 故 $q \\mid a$, 同理有 $q \\mid c$. 于是 $q \\mid c-a$, 所以 $c-a \\geqslant q$, $c \\geqslant a+q$, 因此\n$$\n\\frac{p}{q}=\\frac{d}{c} \\leqslant \\frac{d}{a+q}<\\frac{(n+1)^2}{n^2+q} . \\label{eq2}\n$$\n由式\\ref{eq1},\\ref{eq2}可知 $\\frac{(n+1)^2}{n^2+q}>1+\\frac{1}{q}$, 即 $2 n>q+\\frac{n^2}{q}$. 矛盾!故 $a d \\neq b c$.", + "remark": "", + "figures": [] +} \ No newline at end of file diff --git a/processed_dataset/proof/1316.json b/processed_dataset/proof/1316.json new file mode 100644 index 0000000000000000000000000000000000000000..597829381a593f515bee01719695493d2479de3d --- /dev/null +++ b/processed_dataset/proof/1316.json @@ -0,0 +1,8 @@ +{ + "source_file": "./raw_volume-zh/volume5/chapter4.tex", + "problem_type": "proof", + "problem": "例5. 已知 $a 、 b 、 c$ 是正实数,满足 $a+b+c \\geqslant a b c$. 求证: 在下列三个式子中至少有两个成立\n$$\n\\frac{6}{a}+\\frac{3}{b}+\\frac{2}{c} \\geqslant 2, \\frac{6}{b}+\\frac{3}{c}+\\frac{2}{a} \\geqslant 2, \\frac{6}{c}+\\frac{3}{a}+\\frac{2}{b} \\geqslant 2 .\n$$", + "solution": "证明:用反证法.\n(i) 如果 $\\frac{6}{a}+\\frac{3}{b}+\\frac{2}{c}<2, \\frac{6}{b}+\\frac{3}{c}+\\frac{2}{a}<2, \\frac{6}{c}+\\frac{3}{a}+\\frac{2}{b}<2$, 则 $11\\left(\\frac{1}{a}+\\frac{1}{b}+\\frac{1}{c}\\right)<6$, 与 $\\left(\\frac{1}{a}+\\frac{1}{b}+\\frac{1}{c}\\right)^2 \\geqslant 3\\left(\\frac{1}{a b}+\\frac{1}{b c}+\\frac{1}{c a}\\right) \\geqslant 3$ 矛盾.\n(ii) 不妨设三式中仅有 2 个小于 2 , 即设\n$$\n\\left\\{\\begin{array}{l}\n\\frac{6}{b}+\\frac{3}{c}+\\frac{2}{a}<2, \\label{eq1} \\\\\n\\frac{6}{c}+\\frac{3}{a}+\\frac{2}{b}<2, \\label{eq2} \\\\\n\\frac{6}{a}+\\frac{3}{b}+\\frac{2}{c} \\geqslant 2 . \\label{eq3}\n\\end{array}\\right.\n$$\n由式\\ref{eq1} +\\ref{eq2} $\\times 7$-式\\ref{eq3} 可得 $\\frac{43}{c}+\\frac{17}{b}+\\frac{17}{a}<14$.\n但上式左端 $>17\\left(\\frac{1}{a}+\\frac{1}{b}+\\frac{1}{c}\\right) \\geqslant 17 \\sqrt{3}>14$, 矛盾!\n因此结论成立.", + "remark": "", + "figures": [] +} \ No newline at end of file diff --git a/processed_dataset/proof/1317.json b/processed_dataset/proof/1317.json new file mode 100644 index 0000000000000000000000000000000000000000..bc2b277e29094a39c9d19f08a4a73317621c6b6c --- /dev/null +++ b/processed_dataset/proof/1317.json @@ -0,0 +1,8 @@ +{ + "source_file": "./raw_volume-zh/volume5/chapter4.tex", + "problem_type": "proof", + "problem": "例6. 4 个实数 $a 、 b 、 c 、 d$ 满足:\n(1) $a \\geqslant b \\geqslant c \\geqslant d$;\n(2) $a+b+c+d=9$;\n(3) $a^2+b^2+c^2+d^2=21$.\n求证: $a b-c d \\geqslant 2$.", + "solution": "证明:若 $a+b<5$, 则 $4\\sin (b \\cos x)$, 求证:\n$$\na^2+b^2<\\frac{\\pi^2}{4}\n$$", + "solution": "证明:用反证法.\n设 $a^2+b^2 \\geqslant \\frac{\\pi^2}{4}$, 将 $a \\sin x+b \\cos x$ 表示为 $\\sqrt{a^2+b^2} \\sin (x+ \\varphi)$ 的形式.\n其中, $\\cos \\varphi=\\frac{a}{\\sqrt{a^2+b^2}}, \\sin \\varphi=\\frac{b}{\\sqrt{a^2+b^2}}$.\n由于 $\\sqrt{a^2+b^2} \\geqslant \\frac{\\pi}{2}$, 故存在实数 $x_0$, 使得\n$$\n\\sqrt{a^2+b^2} \\sin \\left(x_0+\\varphi\\right)=\\frac{\\pi}{2} \\text {. }\n$$\n即\n$$\na \\sin x_0+b \\cos x_0=\\frac{\\pi}{2} .\n$$\n由此即得 $\\cos \\left(a \\sin x_0\\right)=\\sin \\left(b \\cos x_0\\right)$, 与题设矛盾!\n所以\n$$\na^2+b^2<\\frac{\\pi^2}{4} .\n$$", + "remark": "", + "figures": [] +} \ No newline at end of file diff --git a/processed_dataset/proof/1320.json b/processed_dataset/proof/1320.json new file mode 100644 index 0000000000000000000000000000000000000000..6dd47b1508476369d1ea868e4f05228cf75978bf --- /dev/null +++ b/processed_dataset/proof/1320.json @@ -0,0 +1,8 @@ +{ + "source_file": "./raw_volume-zh/volume5/chapter4.tex", + "problem_type": "proof", + "problem": "例9. 将一些整数排在数轴的一切有理点上, 求证: 可以找到这样一个区间,使这区间的两个端点上的数之和不大于区间中点上的数的 2 倍.", + "solution": "证明:用反证法.\n设存在一些整数的这样的排列, 使得对于含中点 $C$ 的任意区间 $[A, B]$, 有不等式 $c<\\frac{a+b}{2}$ 成立, 其中 $a 、 b 、 c$ 分别表示置于 $A 、 B 、$ C上的整数.\n设 $A 、 B 、 C 、 A_n 、 B_n$ 分别代表数轴上的点 $-1 、 1 、 0 、-\\frac{1}{2^n}$ 及 $\\frac{1}{2^n}(n=1$, $2,3, \\cdots)$, 并设置于它们上的整数分别为 $a 、 b 、 c 、 a_n 、 b_n$,\n则\n$$\n\\begin{array}{r}\na_1<\\frac{a+c}{2}, a_2<\\frac{a_1+c}{2}, \\\\\n\\max \\left\\{a_1, a_2\\right\\}<\\max \\{a, c\\} .\n\\end{array}\n$$\n同理, 有\n$$\n\\begin{aligned}\n& \\max \\{a, c\\}>\\max \\left\\{a_1, a_2\\right\\}>\\max \\left\\{a_3, a_4\\right\\}>\\cdots, \\\\\n& \\max \\{b, c\\}>\\max \\left\\{b_1, b_2\\right\\}>\\max \\left\\{b_3, b_4\\right\\}>\\cdots .\n\\end{aligned}\n$$\n所以存在 $m$, 使得\n$$\na_{2 m} \\leqslant \\max \\{a, c\\}-m, b_{2 m} \\leqslant \\max \\{b, c\\}-m .\n$$\n故\n$$\na_{2 m}+b_{2 m} \\leqslant 1-2 m<0 \\text {. }\n$$\n但 0 为区间 $\\left[a_{2 m}, b_{2 m}\\right]$ 的中点,矛盾!", + "remark": "", + "figures": [] +} \ No newline at end of file diff --git a/processed_dataset/proof/1321.json b/processed_dataset/proof/1321.json new file mode 100644 index 0000000000000000000000000000000000000000..6b6818cf10241d9e5d2e33602de1874f9e4b2b5f --- /dev/null +++ b/processed_dataset/proof/1321.json @@ -0,0 +1,8 @@ +{ + "source_file": "./raw_volume-zh/volume5/chapter4.tex", + "problem_type": "proof", + "problem": "例10. 设 $p$ 是两个大于 2 的连续整数之积, 求证: 没有整数 $x_1, x_2, \\cdots$, $x_p$ 适合方程\n$$\n\\sum_{i=1}^p x_i^2-\\frac{4}{4 p+1}\\left(\\sum_{i=1}^p x_i\\right)^2=1 .\n$$", + "solution": "证明:用反证法.\n设 $p=k(k+1), k \\geqslant 3$, 则 $p \\geqslant 12,4 p+1 \\geqslant 4 p$.\n假设有整数 $x_1 \\geqslant x_2 \\geqslant \\cdots \\geqslant x_p$ 满足等式:\n$$\n\\begin{aligned}\n4 p+1 & =(4 p+1) \\sum_{i=1}^p x_i^2-4\\left(\\sum_{i=1}^p x_i\\right)^2 \\\\\n& =4\\left[p \\sum_{i=1}^p x_i^2-\\left(\\sum_{i=1}^p x_i\\right)^2\\right]+\\sum_{i=1}^p x_i^2 \\\\\n& =4 \\sum_{1 \\leqslant ix_{l+1} \\geqslant \\cdots \\geqslant x_p$, 其中 $l \\in \\mathbf{Z}^{+}$. 我们分两种情形来讨论:\n(i)当 $2 \\leqslant l4 p+1 \\text {, 矛盾! }\n$$\n因而这种情况不可能.\n(ii) 当 $l=1$ 或 $l=p-1$ 时.\n不妨设 $l=1$. 即\n$$\nx_1>x_2=x_3=\\cdots=x_{p-1} \\geqslant x_p .\n$$\n则 $4 p+1=4 \\sum_{s=2}^p\\left(x_1-x_s\\right)^2+4 \\sum_{s=2}^{p-1}\\left(x_s-x_p\\right)^2+\\sum_{i=1}^p x_i^2$\n$$\n\\begin{aligned}\n= & 4(p-2)\\left(x_1-x_2\\right)^2+4\\left(x_1-x_p\\right)^2 \\\\\n& +4(p-2)\\left(x_2-x_p\\right)^2+\\sum_{i=1}^p x_i^2,\n\\end{aligned}\n$$\n故有 $x_1-x_2=1$, 于是, $9=4\\left(x_1-x_p\\right)^2+4(p-2)\\left(x_2-x_p\\right)^2+\\sum_{i=1}^p x_i^2$, 故 $x_2=x_p(p \\geqslant 12)$ 且 $x_1-x_p=1$, 所以 $5=x_1^2+(p-1)^2 x_2^2$.\n由于 $p \\geqslant 12$, 则 $x_2=0$,于是 $x_1^2=5$, 矛盾!", + "remark": "", + "figures": [] +} \ No newline at end of file diff --git a/processed_dataset/proof/1322.json b/processed_dataset/proof/1322.json new file mode 100644 index 0000000000000000000000000000000000000000..0a8f22495d98a2e4c849dff2ee79145ddf845e07 --- /dev/null +++ b/processed_dataset/proof/1322.json @@ -0,0 +1,8 @@ +{ + "source_file": "./raw_volume-zh/volume5/chapter4.tex", + "problem_type": "proof", + "problem": "例11. 设 $a_1, a_2, \\cdots, a_n$ 为正实数, 满足 $a_1+a_2+\\cdots+a_n=\\frac{1}{a_1}+ \\frac{1}{a_2}+\\cdots+\\frac{1}{a_n}$. 求证:\n$$\n\\frac{1}{n-1+a_1}+\\frac{1}{n-1+a_2}+\\cdots+\\frac{1}{n-1+a_n} \\geqslant 1 .\n$$", + "solution": "证明:令 $b_i=\\frac{1}{n-1+a_i}, i=1,2, \\cdots, n$, 则 $b_i<\\frac{1}{n-1}$, 且\n$$\na_i=\\frac{1-(n-1) b_i}{b_i}, i=1,2, \\cdots, n \\text {. }\n$$\n故条件转化为\n$$\n\\sum_{i=1}^n \\frac{1-(n-1) b_i}{b_i}=\\sum_{i=1}^n \\frac{b_i}{1-(n-1) b_i} .\n$$\n下面用反证法, 假设\n$$\nb_1+b_2+\\cdots+b_n<1 . \\label{(1)}\n$$\n由 Cauchy 不等式可得\n$$\n\\sum_{j \\neq i}\\left(1-(n-1) b_j\\right) \\cdot \\sum_{j \\neq i} \\frac{1}{1-(n-1) b_j} \\geqslant(n-1)^2,\n$$\n由(1),\n$$\n\\sum_{j \\neq i}\\left(1-(n-1) b_j\\right)<(n-1) b_i,\n$$\n所以\n$$\n\\sum_{j \\neq i} \\frac{1}{1-(n-1) b_j}>\\frac{n-1}{b_i},\n$$\n故 $\\quad \\sum_{j \\neq i} \\frac{1-(n-1) b_i}{1-(n-1) b_j}>(n-1) \\cdot \\frac{1-(n-1) b_i}{b_i}$.\n上式对 $i=1,2, \\cdots, n$ 求和, 有即\n$$\n\\begin{gathered}\n\\sum_{i=1}^n \\sum_{j \\neq i} \\frac{1-(n-1) b_i}{1-(n-1) b_j}>(n-1) \\sum_{i=1}^n \\frac{1-(n-1) b_i}{b_i}, \\\\\n\\sum_{j=1}^n \\sum_{j \\neq i} \\frac{1-(n-1) b_i}{1-(n-1) b_j}>(n-1) \\sum_{i=1}^n \\frac{1-(n-1) b_i}{b_i}, \\label{(2)}\n\\end{gathered}\n$$\n而由(1),\n$$\n\\sum_{i \\neq j}\\left(1-(n-1) b_i\\right)(n-1) \\sum_{i=1}^n \\frac{1-(n-1) b_i}{b_i} .\n$$\n矛盾!", + "remark": "", + "figures": [] +} \ No newline at end of file diff --git a/processed_dataset/proof/1323.json b/processed_dataset/proof/1323.json new file mode 100644 index 0000000000000000000000000000000000000000..a4eecc68b56acb2a4c33588ddf4ae51ef86789ce --- /dev/null +++ b/processed_dataset/proof/1323.json @@ -0,0 +1,8 @@ +{ + "source_file": "./raw_volume-zh/volume5/chapter4.tex", + "problem_type": "proof", + "problem": "例12. 对正整数 $n(n \\geqslant 2)$, 假设 $f(x)=a_n x^n+a_{n-1} x^{n-1}+\\cdots+a_1 x+a_0$ 的系数全为实数, $f(x)$ 的全部复根有负实部, 且 $f(x)$ 有一对相等实根.\n求证: 一定存在 $k, 1 \\leqslant k \\leqslant n-1$, 满足:\n$$\na_k^2-4 a_{k-1} a_{k+1} \\leqslant 0 .\n$$", + "solution": "证明:当 $n=2$ 时, $f(x)=a_2(x+a)^2$, 这里 $a$ 是一个正实数,所以\n$$\nf(x)=a_2\\left(x^2+2 a x+a^2\\right) .\n$$\n故 $a_1=2 a a_2, a_0=a^2 a_2, a_1^2-4 a_0 a_2=0$, 结论成立.\n下设正整数 $n>2$, 且设 $a_n>0$ (若否, 则每个系数都乘以 $(-1)$ ), 则\n$$\nf(x)=(x+a)^2\\left(b_{n-2} x^{n-2}+b_{n-3} x^{n-3}+\\cdots+b_1 x+b_0\\right)\\left(a \\in \\mathbf{R}^{+}\\right) . \\label{(1)}\n$$\n因为实系数多项式的复根是成对出现的, 假设有 $k$ 对共轭复根, 记为 $x_j \\pm \\mathrm{i} y_j, j=1,2, \\cdots, k$, 则 $x_j<0$. 其余 $n-2-2 k$ 个根为负实根, 记为 $z_1, z_2, \\cdots, z_{n-2-2 k}$. 则\n$$\n\\begin{aligned}\n& \\frac{1}{b_{n-2}} \\cdot f(x) \\\\\n= & (x+a)^2 \\cdot \\prod_{j=1}^k\\left[x-\\left(x_j+\\mathrm{i} y_j\\right)\\right]\\left[x-\\left(x_j-\\mathrm{i} y_j\\right)\\right] \\cdot \\prod_{l=1}^{n-2-2 k}\\left(x-z_l\\right) \\\\\n= & (x+a)^2 \\cdot \\prod_{j=1}^k\\left(x^2-2 x_j x+x_j^2+y_j^2\\right) \\cdot \\prod_{l=1}^{n-2-2 k}\\left(x-z_l\\right),\n\\end{aligned} \\label{(2)}\n$$\n从(1)和(2)可得, $b_j>0, j=0,1,2, \\cdots, n-2$. 且\n$$\n\\begin{aligned}\nf(x) & =(x+a)^2\\left(b_{n-2} x^{n-2}+b_{n-3} x^{n-3}+\\cdots+b_1 x+b_0\\right) \\\\\n& =b_{n-2} x^n+\\left(2 a b_{n-2}+b_{n-3}\\right) x^{n-1}+\\left(a^2 b_{n-2}\\right.\n\\end{aligned}\n$$\n$$\n\\begin{aligned}\n& \\left.+2 a b_{n-3}+b_{n-4}\\right) x^{n-2}+\\cdots+\\left(a^2 b_2+2 a b_1\\right. \\\\\n& \\left.+b_0\\right) x^2+\\left(a^2 b_1+2 a b_0\\right) x+a^2 b_0 .\n\\end{aligned}\n$$\n为便于统一书写, 引人 $b_i$ : 当 $i<0$ 时, $b_i=0$; 当 $i>n-2$ 时, $b_i=0$. 故\n$$\na_j=a^2 b_j+2 a b_{j-1}+b_{j-2}, j=0,1,2, \\cdots, n .\n$$\n当 $n=3$ 时,可用反证法证明.\n设\n$$\n\\begin{gathered}\na_1^2-4 a_0 a_2>0, a_2^2-4 a_1 a_3>0 . \\\\\na_0=a^2 b_0, a_1=a^2 b_1+2 a b_0,\n\\end{gathered}\n$$\n因为\n$$\na_2=2 a b_1+b_0, a_3=b_1 .\n$$\n所以\n$$\n\\begin{aligned}\n0 & 4 b_0 . \\label{(3)}\n$$\n又由于\n$$\n\\begin{aligned}\n0 & 4 a b_1 . \\label{(4)}\n$$\n(3)与(4)矛盾!\n当 $n>3$ 时,也使用反证法证明.\n设对于 $k \\in\\{1,2, \\cdots, n-1\\}$ 都有 $a_k^2-4 a_{k-1} a_{k+1}>0$, 则\n$$\n\\begin{gathered}\n00$, 有 $a b_1>4 b_0, r_1=b_0-4 a b_1<0$.\n当 $k=n-1$ 时, 从 (5) 有 $b_{n-3} r_{n-2}>0$, 故 $r_{n-2}>0$.\n用 $u$ 表示使 $r_u>0$ 的最小的下标 $(2 \\leqslant u \\leqslant n-2)$, 即 $r_{u-1} \\leqslant 0$.\n在(5)中令 $k=u$, 由 $00$, 此时有\n$$\n\\begin{aligned}\n& q_u=a b_u-4 b_{u-1}>0, \\text { 则 } a b_u>4 b_{u-1} . \\label{(6)} \\\\\n& r_u=b_{u-1}-4 a b_u>0, \\text { 则 } b_{u-1}>4 a b_u . \\label{(7)}\n\\end{aligned}\n$$\n由(6)和(7)即得矛盾!", + "remark": "", + "figures": [] +} \ No newline at end of file diff --git a/processed_dataset/proof/1324.json b/processed_dataset/proof/1324.json new file mode 100644 index 0000000000000000000000000000000000000000..4ce34980550bf1e401d17d5a5e98b024a024cb48 --- /dev/null +++ b/processed_dataset/proof/1324.json @@ -0,0 +1,8 @@ +{ + "source_file": "./raw_volume-zh/volume5/chapter5.tex", + "problem_type": "proof", + "problem": "例1. 已知 $a^2+b^2+c^2+d^2=1$, 求证:\n$$\n\\begin{gathered}\n(a+b)^4+(a+c)^4+(a+d)^4+(b+c)^4 \\\\\n+(b+d)^4+(c+d)^4 \\leqslant 6 .\n\\end{gathered} \\label{(1)}\n$$", + "solution": "分析:由已知可得 $\\left(a^2+b^2+c^2+d^2\\right)^2=1$, 我们要设法挖掘它与四次式 (1) 间的关系.\n注意到要使 $(a+b)^4$ 中 $a$ 的奇次项不在 $\\left(a^2+b^2+c^2+d^2\\right)^2$ 的展开式中出现, 可以配上 $(a-b)^4$ 与之相消, 这样就找到了突破口.\n证明考虑和式: $(a-b)^4+(a-c)^4+(a-d)^4+(b-c)^4+(b-d)^4+ (c-d)^4$, 不难发现它与(1)左端恰好构成恒等式, 即 :\n$$\n\\begin{gathered}\n(a+b)^4+(a-b)^4+(a+c)^4+(a-c)^4+(a+d)^4+(a-d)^4 \\\\\n+(b+c)^4+(b-c)^4+(b+d)^4+(b-d)^4+(c+d)^4+(c-d)^4 \\\\\n=6\\left(a^2+b^2+c^2+d^2\\right)^2\n\\end{gathered} \\label{(2)}\n$$\n由(2)立刻证得(1)成立.", + "remark": "", + "figures": [] +} \ No newline at end of file diff --git a/processed_dataset/proof/1325.json b/processed_dataset/proof/1325.json new file mode 100644 index 0000000000000000000000000000000000000000..41a3a961a93940a6bcf0d5ebdf27df41a8ce50e6 --- /dev/null +++ b/processed_dataset/proof/1325.json @@ -0,0 +1,8 @@ +{ + "source_file": "./raw_volume-zh/volume5/chapter5.tex", + "problem_type": "proof", + "problem": "例2. 设 $\\triangle A_1 A_2 A_3$ 与 $\\triangle B_1 B_2 B_3$ 的边长分别为 $a_1 、 a_2 、 a_3$ 与 $b_1 、 b_2 、 b_3$, 面积分别为 $S_1 、 S_2$, 又记\n$$\nH=a_1^2\\left(-b_1^2+b_2^2+b_3^2\\right)+a_2^2\\left(b_1^2-b_2^2+b_3^2\\right)+a_3^2\\left(b_1^2+b_2^2-b_3^2\\right) .\n$$\n则对于 $\\lambda \\in\\left\\{\\frac{b_1^2}{a_1^2}, \\frac{b_2^2}{a_2^2}, \\frac{b_3^2}{a_3^2}\\right\\}$, 求证: $H \\geqslant 8\\left(\\lambda S_1^2+\\frac{1}{\\lambda} S_2^2\\right)$.", + "solution": "证明:由海伦公式, 有\n$$\n\\begin{gathered}\n16 S_1^2=2 a_1^2 a_2^2+2 a_2^2 a_3^2+2 a_3^2 a_1^2-a_1^4-a_2^4-a_3^4, \\\\\n16 S_2^2=2 b_1^2 b_2^2+2 b_2^2 b_3^2+2 b_3^2 b_1^2-b_1^4-b_2^4-b_3^4 . \\\\\n\\text { 记 } D_1=\\sqrt{\\lambda} a_1^2-\\sqrt{\\frac{1}{\\lambda}} b_1^2, D_2=\\sqrt{\\lambda} a_2^2-\\sqrt{\\frac{1}{\\lambda}} b_2^2, D_3=\\sqrt{\\lambda} a_3^2-\\sqrt{\\frac{1}{\\lambda}} b_3^2 . \\text { 则 }\n\\end{gathered}\n$$\n有恒等式\n$$\n\\begin{aligned}\n& H-8\\left(\\lambda S_1^2+\\frac{1}{\\lambda} S_2^2\\right) \\\\\n= & \\frac{1}{2}\\left(D_1^2+D_2^2+D_3^2\\right)-\\left(D_1 D_2+D_2 D_3+D_3 D_1\\right) .\n\\end{aligned} \\label{(1)}\n$$\n当 $\\lambda=\\frac{b_1^2}{a_1^2}$ 时, $D_1=0$, (1)式即为\n$$\nH-8\\left(\\lambda S_1^2+\\frac{1}{\\lambda} S_2^2\\right)=\\frac{1}{2}\\left(D_2-D_3\\right)^2 .\n$$\n故结论成立.\n同理原不等式对 $\\lambda=\\frac{b_2^2}{a_2^2}$ 或 $\\frac{b_3^2}{a_3^2}$ 也成立.", + "remark": "", + "figures": [] +} \ No newline at end of file diff --git a/processed_dataset/proof/1326.json b/processed_dataset/proof/1326.json new file mode 100644 index 0000000000000000000000000000000000000000..a974753bc598fcb9cc753d602302c1fd476981e2 --- /dev/null +++ b/processed_dataset/proof/1326.json @@ -0,0 +1,8 @@ +{ + "source_file": "./raw_volume-zh/volume5/chapter5.tex", + "problem_type": "proof", + "problem": "例3. 已知 $a, b, c \\in(-2,1)$, 求证:\n$$\na b c>a+b+c-2 .\n$$", + "solution": "分析:不等式的两边是关于 $a 、 b 、 c$ 对称的,且 $a 、 b 、 c$ 都是一次的,所以可以尝试构造一次函数.\n证明设 $f(x)=(b c-1) x-b-c+2$, 则有\n$$\n\\begin{aligned}\nf(-2) & =-2 b c-b-c+4 \\\\\n& =-2\\left(b+\\frac{1}{2}\\right)\\left(c+\\frac{1}{2}\\right)+\\frac{9}{2} .\n\\end{aligned}\n$$\n因为 $b, c \\in(-2,1)$, 所以 $b+\\frac{1}{2}, c+\\frac{1}{2} \\in\\left(-\\frac{3}{2}, \\frac{3}{2}\\right)$, 故\n$$\n\\left(b+\\frac{1}{2}\\right)\\left(c+\\frac{1}{2}\\right) \\leqslant\\left|b+\\frac{1}{2}\\right| \\cdot\\left|c+\\frac{1}{2}\\right|<\\frac{9}{4},\n$$\n从而\n$$\n\\begin{gathered}\nf(-2)=-2\\left(b+\\frac{1}{2}\\right)\\left(c+\\frac{1}{2}\\right)+\\frac{9}{2} \\\\\n>-2 \\cdot \\frac{9}{4}+\\frac{9}{2}=0, \\\\\nf(1)=b c-b-c+1=(1-b)(1-c)>0,\n\\end{gathered}\n$$\n所以, 当 $x \\in(-2,1)$ 时, $f(x)$ 恒大于 0 ,于是 $f(a)>0$, 即\n$$\na b c>a+b+c-2 \\text {. }\n$$", + "remark": "", + "figures": [] +} \ No newline at end of file diff --git a/processed_dataset/proof/1327.json b/processed_dataset/proof/1327.json new file mode 100644 index 0000000000000000000000000000000000000000..72fd88becd2fb36800c35779c624aa6eb4928c4f --- /dev/null +++ b/processed_dataset/proof/1327.json @@ -0,0 +1,8 @@ +{ + "source_file": "./raw_volume-zh/volume5/chapter5.tex", + "problem_type": "proof", + "problem": "例4. 设 $x_1, x_2, x_3, y_1, y_2, y_3 \\in \\mathbf{R}$, 且满足 $x_1^2+x_2^2+x_3^2 \\leqslant 1$, 求证:\n$$\n\\begin{aligned}\n& \\left(x_1 y_1+x_2 y_2+x_3 y_3-1\\right)^2 \\\\\n\\geqslant & \\left(x_1^2+x_2^2+x_3^2-1\\right)\\left(y_1^2+y_2^2+y_3^2-1\\right) .\n\\end{aligned}\n$$", + "solution": "证明:当 $x_1^2+x_2^2+x_3^2=1$ 时,原不等式显然成立.\n当 $x_1^2+x_2^2+x_3^2<1$ 时, 构造二次函数\n$$\n\\begin{aligned}\nf(t) & =\\left(x_1^2+x_2^2+x_3^2-1\\right) t^2-2\\left(x_1 y_1+x_2 y_2+x_3 y_3-1\\right) t+\\left(y_1^2+y_2^2+y_3^2-1\\right) \\\\\n& =\\left(x_1 t-y_1\\right)^2+\\left(x_2 t-y_2\\right)^2+\\left(x_3 t-y_3\\right)^2-(t-1)^2\n\\end{aligned}\n$$\n这是一个开口向下的抛物线, 又因为\n$$\nf(1)=\\left(x_1-y_1\\right)^2+\\left(x_2-y_2\\right)^2+\\left(x_3-y_3\\right)^2 \\geqslant 0,\n$$\n所以,此抛物线的图象与 $x$ 轴一定有交点,从而\n$$\n\\Delta=4\\left(x_1 y_1+x_2 y_2+x_3 y_3-1\\right)^2-4\\left(x_1^2+x_2^2+x_3^2-1\\right)\\left(y_1^2+y_2^2+y_3^2-1\\right) \\geqslant 0,\n$$\n故\n$$\n\\begin{aligned}\n& \\left(x_1 y_1+x_2 y_2+x_3 y_3-1\\right)^2 \\\\\n\\geqslant & \\left(x_1^2+x_2^2+x_3^2-1\\right)\\left(y_1^2+y_2^2+y_3^2-1\\right) .\n\\end{aligned}\n$$\n说明对于要证明 \" $A \\cdot C \\geqslant($ 或 $\\leqslant) B^2$ \" 这类不等式, 我们先把不等式变形为\n$$\n4 A \\cdot C \\geqslant(\\text { 或 } \\leqslant)(2 B)^2,\n$$\n然后构造一个二次函数 $f(x)=A x^2-(2 B) x+C$, 再设法证明其判别式 $\\Delta \\leqslant$ 0 (或 $\\geqslant 0)$.", + "remark": "", + "figures": [] +} \ No newline at end of file diff --git a/processed_dataset/proof/1328.json b/processed_dataset/proof/1328.json new file mode 100644 index 0000000000000000000000000000000000000000..5b88c6ab4642372e32f23789a31637386e6d1f69 --- /dev/null +++ b/processed_dataset/proof/1328.json @@ -0,0 +1,8 @@ +{ + "source_file": "./raw_volume-zh/volume5/chapter5.tex", + "problem_type": "proof", + "problem": "例5. 设 $\\triangle A B C$ 的三边长 $a 、 b 、 c$ 满足: $a+b+c=1$, 求证:\n$$\n5\\left(a^2+b^2+c^2\\right)+18 a b c \\geqslant \\frac{7}{3} .\n$$", + "solution": "证明:由 $a^2+b^2+c^2=(a+b+c)^2-2(a b+b c+c a)$\n$$\n=1-2(a b+b c+c a) \\text {, }\n$$\n可知原不等式等价于\n$$\n\\begin{gathered}\n\\frac{5}{9}(a b+b c+c a)-a b c \\leqslant \\frac{4}{27} . \\label{(1)} \\\\\n\\text { 令 } f(x)=(x-a)(x-b)(x-c)=x^3-x^2+(a b+b c+c a) x-a b c,\n\\end{gathered}\n$$\n则\n$$\nf\\left(\\frac{5}{9}\\right)=\\left(\\frac{5}{9}\\right)^3-\\left(\\frac{5}{9}\\right)^2+\\frac{5}{9}(a b+b c+c a)-a b c .\n$$\n由于 $a 、 b 、 c$ 为三角形三边长, 有 $a, b, c \\in\\left(0, \\frac{1}{2}\\right)$, 故 $\\frac{5}{9}-a 、 \\frac{5}{9}-b$ 及 $\\frac{5}{9}-c$ 都大于 0 ,所以\n$$\n\\begin{aligned}\nf\\left(\\frac{5}{9}\\right) & =\\left(\\frac{5}{9}-a\\right)\\left(\\frac{5}{9}-b\\right)\\left(\\frac{5}{9}-c\\right) \\\\\n& \\leqslant \\frac{1}{27} \\cdot\\left[\\left(\\frac{5}{9}-a\\right)+\\left(\\frac{5}{9}-b\\right)+\\left(\\frac{5}{9}-c\\right)\\right]^3 \\\\\n& =\\frac{8}{27^2} .\n\\end{aligned}\n$$\n因此 $\\frac{8}{27^2} \\geqslant\\left(\\frac{5}{9}\\right)^3-\\left(\\frac{5}{9}\\right)^2+\\frac{5}{9}(a b+b c+c a)-a b c$, 整理即得(1)式, 故原不等式得证.", + "remark": "", + "figures": [] +} \ No newline at end of file diff --git a/processed_dataset/proof/1329.json b/processed_dataset/proof/1329.json new file mode 100644 index 0000000000000000000000000000000000000000..0fb5c6dd1fc4c52106afb8af7c0cb455a03c2923 --- /dev/null +++ b/processed_dataset/proof/1329.json @@ -0,0 +1,8 @@ +{ + "source_file": "./raw_volume-zh/volume5/chapter5.tex", + "problem_type": "proof", + "problem": "例6. 已知不等式\n$$\n\\sqrt{2}(2 a+3) \\cos \\left(\\theta-\\frac{\\pi}{4}\\right)+\\frac{6}{\\sin \\theta+\\cos \\theta}-2 \\sin 2 \\theta<3 a+6\n$$\n对于 $\\theta \\in\\left[0, \\frac{\\pi}{2}\\right]$ 恒成立,求 $a$ 的取值范围.", + "solution": "解:设 $\\sin \\theta+\\cos \\theta=x$, 则 $x \\in[1, \\sqrt{2}]$, 且\n$$\n\\begin{aligned}\n\\sin 2 \\theta & =2 \\sin \\theta \\cos \\theta=x^2-1, \\\\\n\\cos \\left(\\theta-\\frac{\\pi}{4}\\right) & =\\frac{\\sqrt{2}}{2} \\cos \\theta+\\frac{\\sqrt{2}}{2} \\sin \\theta=\\frac{\\sqrt{2}}{2} x .\n\\end{aligned}\n$$\n从而原不等式可化为\n$$\n\\begin{gathered}\n(2 a+3) x+\\frac{6}{x}-2\\left(x^2-1\\right)<3 a+6, \\\\\n2 x^3-(2 a+3) x^2+(3 a+4) x-6>0, \\\\\n(2 x-3)\\left(x^2-a x+2\\right)>0,\n\\end{gathered}\n$$\n即因为 $x \\in[1, \\sqrt{2}]$, 所以 $2 x-3<0$, 从而不等式 $x^2-a x+2<0$ 对于 $x \\in[1$, $\\sqrt{2}]$ 恒成立, 即 $a>x+\\frac{2}{x}, x \\in[1, \\sqrt{2}]$ 恒成立.\n$$\n\\text { 令 } f(x)=x+\\frac{2}{x}, x \\in[1, \\sqrt{2}] \\text {, 则 } a>f_{\\text {max }}(x) \\text {. }\n$$\n因为 $f(x)=x+\\frac{2}{x}$ 在 $[1, \\sqrt{2}]$ 上单调递减, 所以 $f_{\\text {max }}(x)=f(1)=3$, 所以 $a$ 的取值范围为 $a>3$.\n说明利用函数的单调性, 以及求函数最值的方法可以帮助我们来证明不等式.", + "remark": "", + "figures": [] +} \ No newline at end of file diff --git a/processed_dataset/proof/1330.json b/processed_dataset/proof/1330.json new file mode 100644 index 0000000000000000000000000000000000000000..11376b7322fd7b76c5f9d4fc2038c37bb9581b6d --- /dev/null +++ b/processed_dataset/proof/1330.json @@ -0,0 +1,8 @@ +{ + "source_file": "./raw_volume-zh/volume5/chapter5.tex", + "problem_type": "proof", + "problem": "例7. 求证: 对任意正实数 $a 、 b 、 c$, 都有\n$$\n1<\\frac{a}{\\sqrt{a^2+b^2}}+\\frac{b}{\\sqrt{b^2+c^2}}+\\frac{c}{\\sqrt{c^2+a^2}} \\leqslant \\frac{3 \\sqrt{2}}{2} .\n$$", + "solution": "证明:令 $x=\\frac{b^2}{a^2}, y=\\frac{c^2}{b^2}, z=\\frac{a^2}{c^2}$, 则 $x, y, z \\in \\mathbf{R}^{+}, x y z=1$. 于是只需证明\n$$\n1<\\frac{1}{\\sqrt{1+x}}+\\frac{1}{\\sqrt{1+y}}+\\frac{1}{\\sqrt{1+z}} \\leqslant \\frac{3 \\sqrt{2}}{2} .\n$$\n不妨设 $x \\leqslant y \\leqslant z$, 令 $A=x y$, 则 $z=\\frac{1}{A}, A \\leqslant 1$. 于是\n$$\n\\begin{aligned}\n\\frac{1}{\\sqrt{1+x}}+\\frac{1}{\\sqrt{1+y}}+\\frac{1}{\\sqrt{1+z}} & >\\frac{1}{\\sqrt{1+x}}+\\frac{1}{\\sqrt{1+\\frac{1}{x}}} \\\\\n& =\\frac{1+\\sqrt{x}}{\\sqrt{1+x}}>1 .\n\\end{aligned}\n$$\n设 $u=\\frac{1}{\\sqrt{1+A+x+\\frac{A}{x}}}$, 则 $u \\in\\left(0, \\frac{1}{1+\\sqrt{A}}\\right]$, 当且仅当 $x=\\sqrt{A}$ 时,\n$u=\\frac{1}{1+\\sqrt{A}}$. 于是\n$$\n\\begin{aligned}\n& \\left(\\frac{1}{\\sqrt{1+x}}+\\frac{1}{\\sqrt{1+y}}\\right)^2 \\\\\n= & \\left(\\frac{1}{\\sqrt{1+x}}+\\frac{1}{\\sqrt{1+\\frac{A}{x}}}\\right)^2 \\\\\n= & \\frac{1}{1+x}+\\frac{1}{1+\\frac{A}{x}}+\\frac{2}{\\sqrt{1+A+x+\\frac{A}{x}}} \\\\\n= & \\frac{2+x+\\frac{A}{x}}{1+A+x+\\frac{A}{\\dot{x}}}+\\frac{2}{\\sqrt{1+A+x+\\frac{A}{x}}} \\\\\n= & 1+(1-A) u^2+2 u .\n\\end{aligned}\n$$\n构造函数, 令 $f(u)=(1-A) u^2+2 u+1$, 则 $f(u)$ 在 $u \\in\\left(0, \\frac{1}{1+\\sqrt{A}}\\right]$ 上是增函数, 所以\n$$\n\\begin{aligned}\n\\frac{1}{\\sqrt{1+x}}+ & \\frac{1}{\\sqrt{1+y}}-\\leqslant \\sqrt{f\\left(\\frac{1}{1+\\sqrt{A}}\\right)}=\\frac{2}{\\sqrt{1+\\sqrt{A}}} . \\\\\n\\text { 令 } \\sqrt{A}=v, \\text { 则 } & \\frac{1}{\\sqrt{1+x}}+\\frac{1}{\\sqrt{1+y}}+\\frac{1}{\\sqrt{1+z}} \\\\\n& \\leqslant \\frac{2}{\\sqrt{1+\\sqrt{A}}}+\\frac{1}{\\sqrt{1+\\frac{1}{A}}} \\\\\n& =\\frac{2}{\\sqrt{1+v}}+\\frac{\\sqrt{2} v}{\\sqrt{2(1+v)}} \\\\\n& \\leqslant \\frac{2}{\\sqrt{1+v}}+\\frac{\\sqrt{2} v}{1+v} \\\\\n& =\\frac{2}{\\sqrt{1+v}}+\\sqrt{2}-\\frac{\\sqrt{2}}{1+v} \\\\\n& =-\\sqrt{2}\\left(\\frac{1}{\\sqrt{1+v}}-\\frac{\\sqrt{2}}{2}\\right)^2+\\frac{3 \\sqrt{2}}{2} \\\\\n& \\leqslant \\frac{3 \\sqrt{2}}{2} .\n\\end{aligned}\n$$", + "remark": "", + "figures": [] +} \ No newline at end of file diff --git a/processed_dataset/proof/1331.json b/processed_dataset/proof/1331.json new file mode 100644 index 0000000000000000000000000000000000000000..95d7958b4ef80eed37a258996c809b61d632e47e --- /dev/null +++ b/processed_dataset/proof/1331.json @@ -0,0 +1,10 @@ +{ + "source_file": "./raw_volume-zh/volume5/chapter5.tex", + "problem_type": "proof", + "problem": "例8. 求证: 对任意实数 $x$,均有\n$$\n\\left|\\sqrt{x^2+x+1}-\\sqrt{x^2-x+1}\\right|<1 .\n$$", + "solution": "证明:因为\n$$\n\\begin{aligned}\n& \\left|\\sqrt{x^2+x+1}-\\sqrt{x^2-x+1}\\right| \\\\\n= & \\left|\\sqrt{\\left(x+\\frac{1}{2}\\right)^2+\\left(\\frac{\\sqrt{3}}{2}\\right)^2}-\\sqrt{\\left(x-\\frac{1}{2}\\right)^2+\\left(\\frac{\\sqrt{3}}{2}\\right)^2}\\right| .\n\\end{aligned}\n$$\n上式可看作直角坐标系中点 $P\\left(x, \\frac{\\sqrt{3}}{2}\\right)$ 到点 $A\\left(-\\frac{1}{2}, 0\\right)$ 与点 $B\\left(\\frac{1}{2}, 0\\right)$ 的距离的差, 如图()所示.\n根据三角形两边之差小于第三边及 $A B=1$, 得\n$$\n\\left|\\sqrt{x^2+x+1}-\\sqrt{x^2-x+1}\\right|<1 .\n$$", + "remark": "", + "figures": [ + "./images/volume5/figures/fig-c5i1.png" + ] +} \ No newline at end of file diff --git a/processed_dataset/proof/1332.json b/processed_dataset/proof/1332.json new file mode 100644 index 0000000000000000000000000000000000000000..ab019e03eeff7bea8308359fd3827b8fabb9cf0f --- /dev/null +++ b/processed_dataset/proof/1332.json @@ -0,0 +1,10 @@ +{ + "source_file": "./raw_volume-zh/volume5/chapter5.tex", + "problem_type": "proof", + "problem": "例9. 设 $x 、 y 、 z$ 为实数, $0\\sin 2 x+\\sin 2 y+\\sin 2 z .\n$$", + "solution": "证明:原不等式等价于\n$$\n\\begin{gathered}\n\\frac{\\pi}{4}>\\sin x(\\cos x-\\cos y)+ \\\\\n\\sin y(\\cos y-\\cos z)+\\sin z \\cos z .\n\\end{gathered}\n$$\n构造图形如图() 所示.\n圆 $O$ 是单位圆, $S_1 、 S_2 、 S_3$ 分别是三个小矩形的面积, 则\n$$\n\\begin{aligned}\n& S_1=\\sin x(\\cos x-\\cos y), \\\\\n& S_2=\\sin y(\\cos y-\\cos z), \\\\\n& S_3=\\sin z \\cos z .\n\\end{aligned}\n$$\n由于 $S_1+S_2+S_3<\\frac{1}{4} \\cdot \\pi \\cdot 1^2=\\frac{1}{4} \\pi$, 故有\n$$\n\\frac{\\pi}{4}>\\sin x(\\cos x-\\cos y)+\\sin y(\\cos y-\\cos z)+\\sin z \\cos z,\n$$\n故原不等式成立.", + "remark": "", + "figures": [ + "./images/volume5/figures/fig-c5i2.png" + ] +} \ No newline at end of file diff --git a/processed_dataset/proof/1333.json b/processed_dataset/proof/1333.json new file mode 100644 index 0000000000000000000000000000000000000000..b0f595db1a766591402bbd3c547e83f5a9425da2 --- /dev/null +++ b/processed_dataset/proof/1333.json @@ -0,0 +1,10 @@ +{ + "source_file": "./raw_volume-zh/volume5/chapter5.tex", + "problem_type": "proof", + "problem": "例10. 设 $x 、 y 、 z 、 \\alpha 、 \\beta 、 \\gamma$ 为正数, $\\alpha 、 \\beta 、 \\gamma$ 中任意两数之和大于第三个且属于区间 $[0, \\pi)$, 求证:\n$$\n\\sqrt{x^2+y^2-2 x y \\cos \\alpha}+\\sqrt{y^2+z^2-2 y z \\cos \\beta} \\geqslant \\sqrt{z^2+x^2-2 z x \\cos \\gamma} .\n$$", + "solution": "分析:不等式中的项让我们联想到余弦定理的形式, 提示我们去构造一些三角形.\n证明因为 $\\alpha<\\beta+\\gamma<\\pi, \\beta<\\gamma+\\alpha<\\pi, \\gamma<\\alpha+\\beta<\\pi, \\gamma<\\alpha+\\beta<\\pi$, 故从空间一点 $P$ 可作一个三角面 $P-A B C$ 使得:\n$$\n\\begin{gathered}\n\\angle A P B=\\alpha, \\\\\n\\angle B P C=\\beta, \\\\\n\\angle C P A=\\gamma ; \\\\\nP A=x, P B=y, \\\\\nP C=z \n\\end{gathered}\n$$\n如图().\n这样一来,利用 $A B+B C \\geqslant A C$, 有原不等式成立.", + "remark": "", + "figures": [ + "./images/volume5/figures/fig-c5i3.png" + ] +} \ No newline at end of file diff --git a/processed_dataset/proof/1334.json b/processed_dataset/proof/1334.json new file mode 100644 index 0000000000000000000000000000000000000000..8396fc2a3cd556f82b8c74ac9085e9140446538e --- /dev/null +++ b/processed_dataset/proof/1334.json @@ -0,0 +1,8 @@ +{ + "source_file": "./raw_volume-zh/volume5/chapter5.tex", + "problem_type": "proof", + "problem": "例11. 已知 $|u| \\leqslant \\sqrt{2}, v$ 是正实数,求证:\n$$\nS=(u-v)^2+\\left(\\sqrt{2-u^2}-\\frac{9}{v}\\right)^2 \\geqslant 8 .\n$$", + "solution": "证明:关键是看出 $S$ 的表达式恰为直角坐标系中点 $A\\left(u, \\sqrt{2-u^2}\\right)$ 与点 $B\\left(u, \\frac{9}{v}\\right)$ 之间距离的平方.\n显然, 点 $A$ 在圆 $x^2+y^2=2$ 上, 点 $B$ 在双曲线 $x y=9$ 上.\n因此, 问题就转化为求圆 $x^2+y^2=2$ 到双曲线 $x y=9$ 之间的最短距离,\n在图形上易见此最短距离即点 $A(1,1)$ 与 $B(3,3)$ 的距离, 长为 $2 \\sqrt{2}$.\n所以 $S$ 的最小值为 $(2 \\sqrt{2})^2=8$.", + "remark": "", + "figures": [] +} \ No newline at end of file diff --git a/processed_dataset/proof/1335.json b/processed_dataset/proof/1335.json new file mode 100644 index 0000000000000000000000000000000000000000..0225e9f949186550916df238ca6666b8e10a28b6 --- /dev/null +++ b/processed_dataset/proof/1335.json @@ -0,0 +1,8 @@ +{ + "source_file": "./raw_volume-zh/volume5/chapter5.tex", + "problem_type": "proof", + "problem": "例12. 若 $a_1+a_2+\\cdots+a_n=1$, 求证:\n$$\n\\begin{gathered}\n\\overline{a_1^3+a_1^2 a_2}+\\frac{a_1^4}{+a_1 a_2^2+a_2^3}+\\frac{a_2^4}{a_2^3+a_2^2 a_3+a_2 a_3^2+a_3^3}+\\cdots \\\\\n+\\frac{a_n^4}{a_n^3+a_n^2 a_1+a_1^2 a_n+a_1^3} \\geqslant \\frac{1}{4} .\n\\end{gathered}\n$$", + "solution": "证明:记原不等式左端为 $A$.\n构造对偶式\n$$\n\\begin{aligned}\n& B=\\frac{a_2^4}{a_1^3+a_1^2 a_2+a_1 a_2^2+a_2^3}+\\cdots+\\frac{a_1^4}{a_n^3+a_n^2 a_1+a_1^2 a_n+a_1^3}, \\\\\n& A-B=\\frac{\\left(a_1^2+a_2^2\\right)\\left(a_1+a_2\\right)\\left(a_1-a_2\\right)}{\\left(a_1^2+a_2^2\\right)\\left(a_1+a_2\\right)}+\\cdots \\\\\n& +\\frac{\\left(a_n^2+a_1^2\\right)\\left(a_n+a_1\\right)\\left(a_n-a_1\\right)}{\\left(a_n^2+a_1^2\\right)\\left(a_n+a_1\\right)} \\\\\n& =\\left(a_1-a_2\\right)+\\left(a_2-a_3\\right)+\\cdots+\\left(a_n-a_1\\right) \\\\\n& =0 \\text {, } \\\\\n&\n\\end{aligned}\n$$\n那么\n$$\n\\begin{aligned}\nA-B= & \\frac{\\left(a_1^2+a_2^2\\right)\\left(a_1+a_2\\right)\\left(a_1-a_2\\right)}{\\left(a_1^2+a_2^2\\right)\\left(a_1+a_2\\right)}+\\cdots \\\\\n& +\\frac{\\left(a_n^2+a_1^2\\right)\\left(a_n+a_1\\right)\\left(a_n-a_1\\right)}{\\left(a_n^2+a_1^2\\right)\\left(a_n+a_1\\right)} \\\\\n= & \\left(a_1-a_2\\right)+\\left(a_2-a_3\\right)+\\cdots+\\left(a_n-a_1\\right) \\\\\n= & 0,\n\\end{aligned}\n$$\n故 $A=B$.\n又因为\n$$\n\\begin{aligned}\n& \\frac{a_1^4+a_2^4}{\\left(a_1^2+a_2^2\\right)\\left(a_1+a_2\\right)} \\geqslant \\frac{\\left(a_1^2+a_2^2\\right)^2}{2\\left(a_1^2+a_2^2\\right)\\left(a_1+a_2\\right)} \\\\\n&=\\frac{a_1^2+a_2^2}{2\\left(a_1+a_2\\right)} \\geqslant \\frac{\\left(a_1+a_2\\right)^2}{4\\left(a_1+a_2\\right)} \\\\\n&=\\frac{a_1+a_2}{4},\n\\end{aligned}\n$$\n所以\n$$\n\\begin{aligned}\nA & =\\frac{1}{2}(A+B) \\\\\n& \\geqslant \\frac{1}{2}\\left[\\frac{1}{4}\\left(a_1+a_2\\right)+\\frac{1}{4}\\left(a_2+a_3\\right)+\\cdots+\\frac{1}{4}\\left(a_n+a_1\\right)\\right] \\\\\n& =\\frac{1}{4} .\n\\end{aligned}\n$$", + "remark": "", + "figures": [] +} \ No newline at end of file diff --git a/processed_dataset/proof/1336.json b/processed_dataset/proof/1336.json new file mode 100644 index 0000000000000000000000000000000000000000..1f55c806a1894ad70f8962ed7df77508b0904dde --- /dev/null +++ b/processed_dataset/proof/1336.json @@ -0,0 +1,8 @@ +{ + "source_file": "./raw_volume-zh/volume5/chapter5.tex", + "problem_type": "proof", + "problem": "例13. 设 $x_n=\\sqrt{2+\\sqrt[3]{3+\\cdots+\\sqrt[n]{n}}}$, 求证:\n$$\nx_{n+1}-x_n<\\frac{1}{n !}, n=2,3, \\cdots \\text {. }\n$$", + "solution": "证明:当 $n=2$ 时, $x_3-x_2=\\sqrt{2+\\sqrt[3]{3}}-\\sqrt{2}<\\frac{1}{2 !}$.\n当 $n \\geqslant 3$ 时,构造数列 $\\left\\{a_i\\right\\} 、\\left\\{b_i\\right\\} 、\\left\\{c_i\\right\\}$ 如下:\n$$\n\\begin{aligned}\n& a_i=\\sqrt[i]{i+\\sqrt[i+1]{(i+1)+\\cdots+\\sqrt[n]{n+\\sqrt[n]{1+1}}}}, i=2, \\cdots, n+1 ; \\\\\n& b_i=\\sqrt[i]{i+\\sqrt[i+1]{(i+1)+\\cdots+\\sqrt[n]{n}}}, i=2,3, \\cdots, n, b_{n+1}=0 \\\\\n& c_i=a_i^{i-1}+a_i^{i-2} b_i+\\cdots+a_i b_i^{i-2}+b_i^{i-1}, i=2,3, \\cdots .\n\\end{aligned}\n$$\n显然, $x_{n+1}=a_2, x_n=b_2$, 且\n$$\n\\left(a_i-b_i\\right) c_i=a_i^i-b_i^i=a_{i+1}-b_{i+1} .\n$$\n故\n$$\na_i-b_i=\\frac{a_{i+1}-b_{i+1}}{c_i}, i=2,3, \\cdots, n .\n$$\n将以上 $n-1$ 个等式相乘, 并注意到\n$$\na_{n+1}-b_{n+1}=(n+1)^{\\frac{1}{n+1}},\n$$\n则有\n$$\na_2-b_2=\\frac{a_{n+1}-b_{n+1}}{c_2 c_3 \\cdots c_n}=\\frac{(n+1)^{\\frac{1}{n+1}}}{c_2 c_3 \\cdots c_n} .\n$$\n又因为 $a_k>b_k \\geqslant \\sqrt[k]{k}$, 故 $c_k \\geqslant k \\cdot k^{\\frac{k-1}{k}}>k \\cdot k^{\\frac{k-1}{k+1}}$, 于是\n$$\nx_{n+1}-x_n=a_2-b_2<\\frac{1}{n !} \\cdot \\frac{(n+1)^{\\frac{1}{n+1}}}{n^{\\frac{n+1}{n+1}}}<\\frac{1}{n !} .\n$$\n上式中当 $n>2$ 时, $\\frac{n+1}{n^{n-1}}<\\frac{2 n}{n^2}<1$ 是明显的.", + "remark": "", + "figures": [] +} \ No newline at end of file diff --git a/processed_dataset/proof/1337.json b/processed_dataset/proof/1337.json new file mode 100644 index 0000000000000000000000000000000000000000..67c7f0a265b1f031f27148bbd796f8d2ef42c1f5 --- /dev/null +++ b/processed_dataset/proof/1337.json @@ -0,0 +1,8 @@ +{ + "source_file": "./raw_volume-zh/volume5/chapter5.tex", + "problem_type": "proof", + "problem": "例14. 实数 $a_1, a_2, \\cdots, a_n$ 满足: $a_1+a_2+\\cdots+a_n=0$, 求证:\n$$\n\\max _{1 \\leqslant k \\leqslant n}\\left(a_k^2\\right) \\leqslant \\frac{n}{3} \\sum_{i=1}^{n-1}\\left(a_i-a_{i+1}\\right)^2 .\n$$", + "solution": "证明:只需对任意 $1 \\leqslant k \\leqslant n$, 证明不等式成立即可.\n记 $d_k=a_k-a_{k+1}, k=1,2, \\cdots, n-1$, 则\n$$\n\\begin{gathered}\na_k=a_k, \\\\\na_{k+1}=a_k-d_k, a_{k+2}=a_k-d_k-d_{k+1}, \\cdots, a_n=a_k-d_k-d_{k+1}-\\cdots-d_{n-1}, \\\\\nk=a_k+d_{k-1}, a_{k-2}=a_k+d_{k-1}+d_{k-2}, \\cdots, a_1=a_k+d_{k-1}+d_{k-2}+\\cdots+d_1,\n\\end{gathered}\n$$\n把上面这 $n$ 个等式相加, 并利用 $a_1+a_2+\\cdots+a_n=0$, 可得\n$$\n\\begin{gathered}\nn a_k-(n-k) d_k-(n-k-1) d_{k+1}-\\cdots-d_{n-1}+ \\\\\n(k-1) d_{k-1}+(k-2) d_{k-2}+\\cdots+d_1=0 .\n\\end{gathered}\n$$\n由 Cauchy 不等式可得\n$$\n\\begin{aligned}\n\\left(n a_k\\right)^2= & \\left((n-k) d_k+(n-k-1) d_{k+1}+\\cdots+d_{n-1}\\right. \\\\\n& \\left.-(k-1) d_{k-1}-(k-2) d_{k-2}-\\cdots-d_1\\right)^2 \\\\\n\\leqslant & \\left(\\sum_{i=1}^{k-1} i^2+\\sum_{i=1}^{n-k} i^2\\right)\\left(\\sum_{i=1}^{n-1} d_i^2\\right) \\\\\n\\leqslant & \\left(\\sum_{i=1}^{n-1} i^2\\right)\\left(\\sum_{i=1}^{n-1} d_i^2\\right)=\\frac{n(n-1)(2 n-1)}{6}\\left(\\sum_{i=1}^{n-1} d_i^2\\right) \\\\\n\\leqslant & \\frac{n^3}{3}\\left(\\sum_{i=1}^{n-1} d_i^2\\right), \\\\\n& a_k^2 \\leqslant \\frac{n}{3} \\sum_{i=1}^{n-1}\\left(a_i-a_{i+1}\\right)^2 .\n\\end{aligned}\n$$", + "remark": "", + "figures": [] +} \ No newline at end of file diff --git a/processed_dataset/proof/1338.json b/processed_dataset/proof/1338.json new file mode 100644 index 0000000000000000000000000000000000000000..1c03256e302795d6149a90ce5181fcb1c3085678 --- /dev/null +++ b/processed_dataset/proof/1338.json @@ -0,0 +1,8 @@ +{ + "source_file": "./raw_volume-zh/volume5/chapter5.tex", + "problem_type": "proof", + "problem": "例15. 给定两组数 $x_1, x_2, \\cdots, x_n$ 和 $y_1, y_2, \\cdots, y_n$, 现知\n$$\n\\begin{gathered}\nx_1>x_2>\\cdots>x_n>0, y_1>y_2>\\cdots>y_n>0, \\\\\nx_1>y_1, x_1+x_2>y_1+y_2, \\cdots, \\\\\nx_1+x_2+\\cdots+x_n>y_1+y_2+\\cdots+y_n .\n\\end{gathered}\n$$\n求证: 对于任何自然数 $k$,都有\n$$\nx_1^k+x_2^k+\\cdots+x_n^k>y_1^k+y_2^k+\\cdots+y_n^k .\n$$", + "solution": "分析:我们猜想是否有如下的递推关系:\n$$\n\\begin{aligned}\n& x_1^k+x_2^k+\\cdots+x_n^k>x_1^{k-1} y_1+x_2^{k-1} y_2+\\cdots+x_n^{k-1} y_n, \\\\\n& x_1^{k-1} y_1+x_2^{k-1} y_2+\\cdots+x_n^{k-1} y_n>x_1^{k-2} y_1^2+x_2^{k-2} y_2^2+\\cdots+x_n^{k-2} y_n^2 \\text {, } \\\\\n& x_1 y_1^{k-1}+x_2 y_2^{k-1}+\\cdots+x_n y_n^{k-1}>y_1^k+y_2^k+\\cdots+y_n^k . \\\\\n&\n\\end{aligned}\n$$\n从而联想到构造辅助命题:\n若 $a_1>a_2>\\cdots>a_n>0$, 且满足题设的条件,那么:\n$$\na_1 x_1+a_2 x_2+\\cdots+a_n x_n>a_1 y_1+a_2 y_2+\\cdots+a_n y_n . \\label{(1)}\n$$\n证明因为 $a_1>a_2>\\cdots>a_n>0$, 故存在正数 $b_1, b_2, \\cdots b_{n-1}, b_n$ 使得:\n$$\n\\begin{gathered}\na_n=b_1, \\\\\na_{n-1}=b_1+b_2, \\\\\n\\cdots \\cdots \\cdots \\cdots \\cdots \\cdots \\cdots \\cdot \\cdots \\cdot \\cdots \\cdot \\cdots \\cdot \\cdots+b_{n-1}, \\\\\na_2=b_1+b_2+\\cdots \\cdots+b_n . \\\\\na_1=b_1+b_2+\\cdots+\n\\end{gathered}\n$$\n于是 $\\quad a_1 x_1+a_2 x_2+\\cdots+a_n x_n$\n$$\n\\begin{aligned}\n& =\\left(b_1+b_2+\\cdots+b_n\\right) x_1+\\left(b_1+b_2+\\cdots+b_{n-1}\\right) x_2+\\cdots+b_1 x_n \\\\\n& =b_1\\left(x_1+x_2+\\cdots+x_n\\right)+b_2\\left(x_1+x_2+\\cdots+x_{n-1}\\right)+\\cdots+b_n x_1 \\\\\n& >b_1\\left(y_1+y_2+\\cdots+y_n\\right)+b_2\\left(y_1+y_2+\\cdots+y_{n-1}\\right)+\\cdots+b_n y_1 \\\\\n& =\\left(b_1+b_2+\\cdots+b_n\\right) y_1+\\left(b_1+b_2+\\cdots+b_{n-1}\\right) y_2+\\cdots+b_1 y_n \\\\\n& =a_1 y_1+a_2 y_2+\\cdots+a_n y_n .\n\\end{aligned}\n$$\n故(1)式成立.\n再依次取 $a_i=x_i^{k-1}, x_i^{k-2} y_i, \\cdots, y_i^{k-1}(i=1,2, \\cdots, n)$, 利用不等式的传递性, 自大到小逐渐缩小, 即得所要证的不等式.", + "remark": "", + "figures": [] +} \ No newline at end of file diff --git a/processed_dataset/proof/1339.json b/processed_dataset/proof/1339.json new file mode 100644 index 0000000000000000000000000000000000000000..5db3cd81df86996b32ec709073db34703352d02b --- /dev/null +++ b/processed_dataset/proof/1339.json @@ -0,0 +1,8 @@ +{ + "source_file": "./raw_volume-zh/volume5/chapter5.tex", + "problem_type": "proof", + "problem": "例17. 求所有大于 1 的正整数 $n$, 使得对任意正实数 $x_1, x_2, \\cdots, x_n$, 都有不等式\n$$\n\\left(x_1+x_2+\\cdots+x_n\\right)^2 \\geqslant n\\left(x_1 x_2+x_2 x_3+\\cdots+x_n x_1\\right) .\n$$", + "solution": "解:当 $n=2$ 时, 不等式为 $\\left(x_1+x_2\\right)^2 \\geqslant 2\\left(x_1 x_2+x_2 x_1\\right)$, 即 $\\left(x_1-x_2\\right)^2 \\geqslant$ 0 , 故 $n=2$ 满足题意.\n当 $n=3$ 时,不等式 $\\quad\\left(x_1+x_2+x_3\\right)^2 \\geqslant 3\\left(x_1 x_2+x_2 x_3+x_3 x_1\\right) ,$\n等价于\n$$\n\\left(x_1-x_2\\right)^2+\\left(x_2-x_3\\right)^2+\\left(x_3-x_1\\right)^2 \\geqslant 0,\n$$\n故 $n=3$ 满足题意.\n当 $n=4$ 时, 不等式为\n$$\n\\begin{gathered}\n\\left(x_1+x_2+x_3+x_4\\right)^2 \\geqslant 4\\left(x_1 x_2+x_2 x_3+x_3 x_4+x_4 x_1\\right) \\\\\n\\Leftrightarrow\\left(x_1-x_2+x_3-x_4\\right)^2 \\geqslant 0 .\n\\end{gathered}\n$$\n故 $n=4$ 满足题意.\n下证当 $n>4$ 时, 不等式不可能对任意正实数 $x_1, x_2, \\cdots, x_n$ 都成立.\n取\n$$\nx_1=x_2=1, x_3=x_4=\\cdots=x_n=\\frac{1}{5(n-2)},\n$$\n则原不等式为\n$$\n\\left[1+1+(n-2) \\cdot \\frac{1}{5(n-2)}\\right]^2 \\geqslant n\\left(1+\\frac{2}{5(n-2)}+\\frac{n-3}{25(n-2)^2}\\right)\n$$\n$$\n\\Leftrightarrow \\frac{121}{25} \\geqslant n+\\frac{2 n}{5(n-2)}+\\frac{n(n-3)}{25(n-2)^2},\n$$\n这与 $\\frac{121}{25}<5 \\leqslant n$ 矛盾.\n所以满足题意的正整数 $n$ 为 $2 、 3 、 4$.", + "remark": "", + "figures": [] +} \ No newline at end of file diff --git a/processed_dataset/proof/1340.json b/processed_dataset/proof/1340.json new file mode 100644 index 0000000000000000000000000000000000000000..8a725c609479f5ebfa6556b81ca335d3f7931d79 --- /dev/null +++ b/processed_dataset/proof/1340.json @@ -0,0 +1,8 @@ +{ + "source_file": "./raw_volume-zh/volume5/chapter5.tex", + "problem_type": "proof", + "problem": "例18. 已知正整数 $n \\geqslant 2$, 实数 $a_1 \\geqslant a_2 \\geqslant \\cdots \\geqslant a_n>0, b_1 \\geqslant b_2 \\geqslant \\cdots \\geqslant b_n>0$, 并且有: $a_1 a_2 \\cdots a_n=b_1 b_2 \\cdots b_n, \\sum_{1 \\leqslant i0$, 故只须 $k-h \\geqslant 1$, 即 $k \\geqslant h+1$ 即可.\n而 $\\sum_{i=1}^n a_i>(n-2) \\sum_{i=1}^n b_i$ 等价于\n$$\n(n-1) h+\\frac{1}{h^{n-1}}>(n-2)\\left[k+(n-2)+\\frac{1}{k}\\right] .\n$$\n为简便起见, 取 $k=h+1$, 仅需 $(n-1) h>(n-2)[h+1+(n-2)+1]$, 即 $h>n^2-2 n$ 即可.\n取 $h=n^2-2 n+1=(n-1)^2$, 此时, 便有 $\\sum_{i=1}^n a_i> (n-2) \\sum_{i=1}^n b_i$.\n说明.\n事实上, 我们可以证明 $\\sum_{i=1}^n a_i \\leqslant(n-1) \\sum_{i=1}^n b_i$.", + "remark": "", + "figures": [] +} \ No newline at end of file diff --git a/processed_dataset/proof/1341.json b/processed_dataset/proof/1341.json new file mode 100644 index 0000000000000000000000000000000000000000..04b12c75e15332f8f650c6eeca1b34f1998507dc --- /dev/null +++ b/processed_dataset/proof/1341.json @@ -0,0 +1,8 @@ +{ + "source_file": "./raw_volume-zh/volume5/chapter6.tex", + "problem_type": "proof", + "problem": "例1. 设 $a, b, c, d \\in \\mathbf{R}^{+}$, 求证:\n$$\n\\begin{aligned}\n& \\frac{a^3+b^3+c^3}{a+b+c}+\\frac{b^3+c^3+d^3}{b+c+d}+\\frac{c^3+d^3+a^3}{c+d+a}+\\frac{d^3+a^3+b^3}{d+a+b} \\\\\n\\geqslant & a^2+b^2+c^2+d^2 .\n\\end{aligned}\n$$", + "solution": "分析:直接证明这个不等式, 有点难以下手, 我们若能证明(局部不等式)\n$$\n\\frac{a^3+b^3+c^3}{a+b+c} \\geqslant \\frac{a^2+b^2+c^2}{3}, \\label{(1)}\n$$\n则同理可得 $\\frac{b^3+c^3+d^3}{b+c+d} \\geqslant \\frac{b^2+c^2+d^2}{3}, \\frac{c^3+d^3+a^3}{c+d+a} \\geqslant \\frac{c^2+d^2+a^2}{3}$, $\\frac{d^3+a^3+b^3}{d+a+b} \\geqslant \\frac{d^2+a^2+b^2}{3}$, 把这些不等式相加, 便得所要证明的式子.\n所以, 要证明本题, 只需证明一个局部不等式(1).\n证明先证: 对 $x, y, z \\in \\mathbf{R}^{+}$, 有\n$$\n\\frac{x^3+y^3+z^3}{x+y+z} \\geqslant \\frac{x^2+y^2+z^2}{3} .\n$$\n事实上,由柯西不等式, 得\n$$\n\\begin{aligned}\n(x+y+z)\\left(x^3+y^3+z^3\\right) & \\geqslant\\left(x^2+y^2+z^2\\right)^2 \\\\\n& \\geqslant\\left(x^2+y^2+z^2\\right) \\cdot \\frac{(x+y+z)^2}{3},\n\\end{aligned}\n$$\n所以\n$$\n\\frac{x^3+y^3+z^3}{x+y+z} \\geqslant \\frac{x^2+y^2+z^2}{3} .\n$$\n于是有\n$$\n\\begin{aligned}\n& \\frac{a^3+b^3+c^3}{a+b+c} \\geqslant \\frac{a^2+b^2+c^2}{3}, \\frac{b^3+c^3+d^3}{b+c+d} \\geqslant \\frac{b^2+c^2+d^2}{3}, \\\\\n& \\frac{c^3+d^3+a^3}{c+d+a} \\geqslant \\frac{c^2+d^2+a^2}{3}, \\frac{d^3+a^3+b^3}{d+a+b} \\geqslant \\frac{d^2+a^2+b^2}{3} .\n\\end{aligned}\n$$\n把上面这 4 个不等式相加便得\n$$\n\\begin{aligned}\n& \\frac{a^3+b^3+c^3}{a+b+c}+\\frac{b^3+c^3+d^3}{b+c+d}+\\frac{c^3+d^3+a^3}{c+d+a}+\\frac{d^3+a^3+b^3}{d+a+b} \\\\\n\\geqslant & a^2+b^2+c^2+d^2 .\n\\end{aligned}\n$$", + "remark": "", + "figures": [] +} \ No newline at end of file diff --git a/processed_dataset/proof/1342.json b/processed_dataset/proof/1342.json new file mode 100644 index 0000000000000000000000000000000000000000..0a1e7013965c52f679093a730624da0ec456eb9f --- /dev/null +++ b/processed_dataset/proof/1342.json @@ -0,0 +1,8 @@ +{ + "source_file": "./raw_volume-zh/volume5/chapter6.tex", + "problem_type": "proof", + "problem": "例2. 设 $x, y, z \\in \\mathbf{R}^{+}$, 求证:\n$$\n\\sqrt{\\frac{x}{y+z}}+\\sqrt{\\frac{y}{z+x}}+\\sqrt{\\frac{z}{x+y}} \\geqslant 2 .\n$$", + "solution": "分析:对不等式左边进行处理似有难度, 我们也拟从局部考虑, 先证 $\\sqrt{\\frac{x}{y+z}} \\geqslant \\frac{2 x}{x+y+z}$ 即可.\n证明先证明\n$$\n\\sqrt{\\frac{x}{y+z}} \\geqslant \\frac{2 x}{x+y+z} .\n$$\n事实上,由平均不等式可得\n$$\n\\sqrt{\\frac{-x}{y+z}}=\\frac{x}{\\sqrt{x} \\sqrt{y+z}} \\geqslant \\frac{x}{\\frac{x+y+z}{2}}=\\frac{2 x}{x+y+z} .\n$$\n同理可得\n$$\n\\begin{aligned}\n& \\sqrt{\\frac{y}{z+x}} \\geqslant \\frac{2 y}{x+y+z}, \\\\\n& \\sqrt{\\frac{z}{x+y}} \\geqslant \\frac{2 z}{x+y+z} .\n\\end{aligned}\n$$\n把上面 3 个不等式相加, 便得\n$$\n\\sqrt{\\frac{x}{y+z}}+\\sqrt{\\frac{y}{z+x}}+\\sqrt{\\frac{z}{x+y}} \\geqslant 2 .\n$$\n说明本题中的等号是不成立的.\n因为若等号成立, 则 $x=y+z, y= z+x, z=x+y$, 从而 $x+y+z=0$, 不可能.", + "remark": "", + "figures": [] +} \ No newline at end of file diff --git a/processed_dataset/proof/1343.json b/processed_dataset/proof/1343.json new file mode 100644 index 0000000000000000000000000000000000000000..860fc2a82051908ba4db4535ef615fb0f80f297a --- /dev/null +++ b/processed_dataset/proof/1343.json @@ -0,0 +1,8 @@ +{ + "source_file": "./raw_volume-zh/volume5/chapter6.tex", + "problem_type": "proof", + "problem": "例3. 设 $0 \\leqslant a, b, c \\leqslant 1$, 求证:\n$$\n\\frac{a}{b c+1}+\\frac{b}{c a+1}+\\frac{c}{a b+1} \\leqslant 2 .\n$$", + "solution": "分析:我们发现等号成立并不是当 $a=b=c$, 而是当 $a 、 b 、 c$ 中有一个为 0 , 另两个为 1 时取到, 因此, 不应从整体去考虑问题, 而应考虑局部的性质, 即单项性质.\n证明我们先证明\n$$\n\\frac{a}{b c+1} \\leqslant \\frac{2 a}{a+b+c} . \\label{(1)}\n$$\n注意到(1)等价于 $a+b+c \\leqslant 2 b c+2$, 即\n$$\n(b-1)(c-1)+b c+1 \\geqslant a .\n$$\n而 $a, b, c \\in[0,1]$, 上式显然成立,因此(1)成立.\n同理, 我们有\n$$\n\\begin{aligned}\n& \\frac{b}{c a+1} \\leqslant \\frac{2 b}{a+b+c}, \\label{(2)} \\\\\n& \\frac{c}{a b+1} \\leqslant \\frac{2 c}{a+b+c} . \\label{(3)}\n\\end{aligned}\n$$\n三式相加即得原不等式成立.", + "remark": "", + "figures": [] +} \ No newline at end of file diff --git a/processed_dataset/proof/1344.json b/processed_dataset/proof/1344.json new file mode 100644 index 0000000000000000000000000000000000000000..a0656fc8edba84fd4487a753108a2fd05ebd774d --- /dev/null +++ b/processed_dataset/proof/1344.json @@ -0,0 +1,8 @@ +{ + "source_file": "./raw_volume-zh/volume5/chapter6.tex", + "problem_type": "proof", + "problem": "例4. 已知 $x_i \\geqslant 1, i=1,2, \\cdots, n$, 且 $x_1 x_2 \\cdots x_n=a^n$. 记 $a_{n+1}=x_1$. 求证: 当 $n>2$ 时,有 $\\sum_{i=1}^n x_i x_{i+1}-\\sum_{i=1}^n x_i \\geqslant \\frac{n}{2}\\left(a^2-1\\right)$.", + "solution": "分析:为了联系 $x_i x_{i+1}$ 与 $x_i$, 自然想到 $\\left(x_i-1\\right)\\left(x_{i+1}-1\\right)$, 利用条件, 此式 $\\geqslant 0$.\n证明由已知可得 $\\left(x_i-1\\right)\\left(x_{i+1}-1\\right) \\geqslant 0$, 故\n$$\n2 x_i x_{i+1}-x_i-x_{i+1} \\geqslant x_i x_{i+1}-1,\n$$\n上式对 $i$ 从 1 到 $n$ 求和,有 $2\\left(\\sum_{i=1}^n x_i x_{i+1}-\\sum_{i=1}^n x_i\\right) \\geqslant \\sum_{i=1}^n x_i x_{i+1}-n$.\n利用平均不等式, 有\n$$\n\\sum_{i=1}^n x_i x_{i+1} \\geqslant n \\cdot \\sqrt[n]{\\left(a^n\\right)^2}=n a^2 .\n$$\n因此 $2\\left(\\sum_{i=1}^n x_i x_{i+1}-\\sum_{i=1}^n x_i\\right) \\geqslant n\\left(a^2-1\\right)$, 所以原不等式成立.", + "remark": "", + "figures": [] +} \ No newline at end of file diff --git a/processed_dataset/proof/1345.json b/processed_dataset/proof/1345.json new file mode 100644 index 0000000000000000000000000000000000000000..e4392e887b97c6fedd0e18d81648aaecb89b873b --- /dev/null +++ b/processed_dataset/proof/1345.json @@ -0,0 +1,8 @@ +{ + "source_file": "./raw_volume-zh/volume5/chapter6.tex", + "problem_type": "proof", + "problem": "例5. 设实数 $a_1, a_2, \\cdots, a_n \\in(-1,1]$, 求证:\n$$\n\\sum_{i=1}^n \\frac{1}{1+a_i a_{i+1}} \\geqslant \\sum_{i=1}^n \\frac{1}{1+a_i^2} \\text { (约定 } a_{n+1}=a_1 \\text { ). }\n$$", + "solution": "证明:首先证明:若 $x, y \\in(-1,1]$, 则\n$$\n\\frac{2}{1+x y} \\geqslant \\frac{1}{1+x^2}+\\frac{1}{1+y^2} . \\label{(1)}\n$$\n(1)等价于 $2\\left(1+x^2\\right)\\left(1+y^2\\right)-(1+x y)\\left(2+x^2+y^2\\right) \\geqslant 0$,\n即 $(x-y)^2-x y(x-y)^2 \\geqslant 0$. 故(1)成立.\n因此\n$$\n\\begin{aligned}\n& \\frac{2}{1+a_1 a_2} \\geqslant \\frac{1}{1+a_1^2}+\\frac{1}{1+a_2^2}, \\\\\n& \\frac{2}{1+a_2 a_3} \\geqslant \\frac{1}{1+a_2^2}+\\frac{1}{1+a_3^2} \\text {, } \\\\\n& \\frac{2}{1+a_n a_1} \\geqslant \\frac{1}{1+a_n^2}+\\frac{1}{1+a_1^2} \\text {. } \\\\\n&\n\\end{aligned}\n$$\n上述 $n$ 个式子相加即得原不等式成立.", + "remark": "", + "figures": [] +} \ No newline at end of file diff --git a/processed_dataset/proof/1346.json b/processed_dataset/proof/1346.json new file mode 100644 index 0000000000000000000000000000000000000000..09f8afa9aec50f6c089d1e06eaa1911cd081f7a8 --- /dev/null +++ b/processed_dataset/proof/1346.json @@ -0,0 +1,8 @@ +{ + "source_file": "./raw_volume-zh/volume5/chapter6.tex", + "problem_type": "proof", + "problem": "例6. 设 $x, y, z \\geqslant 0$, 且 $x^2+y^2+z^2=1$, 求证:\n$$\n\\frac{x}{1+y z}+\\frac{y}{1+z x}+\\frac{z}{1+x y} \\geqslant 1 . \\label{(1)}\n$$", + "solution": "证明:我们只须证明局部不等式\n而\n$$\n\\begin{aligned}\n& \\frac{x}{1+y z} \\geqslant x^2 . \\\\\n& x+x y z \\leqslant 1 .\n\\end{aligned}\n$$\n(1)等价于\n$$\n\\begin{aligned}\nx+x y z & \\leqslant x+\\frac{1}{2} x\\left(y^2+z^2\\right)=\\frac{1}{2}\\left(3 x-x^3\\right) \\\\\n& =\\frac{1}{2}\\left[2-(x-1)^2(x+2)\\right] \\leqslant 1 .\n\\end{aligned}\n$$\n故(1)成立, 进而原不等式获证.\n说明我们可得一串不等式\n$$\n\\begin{aligned}\n1 & \\leqslant \\frac{x}{1+y z}+\\frac{y}{1+z x}+\\frac{z}{1+x y} \\\\\n& \\leqslant \\frac{x}{1-y z}+\\frac{y}{1-z x}+\\frac{z}{1-x y} \\\\\n& \\leqslant \\frac{3 \\sqrt{3}}{2} .\n\\end{aligned}\n$$", + "remark": "", + "figures": [] +} \ No newline at end of file diff --git a/processed_dataset/proof/1347.json b/processed_dataset/proof/1347.json new file mode 100644 index 0000000000000000000000000000000000000000..a10878dc42b24358665ba3f11a0260c0c9ef6b17 --- /dev/null +++ b/processed_dataset/proof/1347.json @@ -0,0 +1,8 @@ +{ + "source_file": "./raw_volume-zh/volume5/chapter6.tex", + "problem_type": "proof", + "problem": "例7. 已知 $x, y, z \\in \\mathbf{R}^{+}$, 求证:\n$$\n\\frac{x z}{x^2+x z+y z}+\\frac{x y}{y^2+x y+x z}+\\frac{y z}{z^2+y z+x y} \\leqslant 1 . \\label{(1)}\n$$", + "solution": "证明:不妨设 $x y z=1$.\n首先证明\n$$\n\\frac{x z}{x^2+x z+y z} \\leqslant \\frac{1}{1+y+\\frac{1}{z}} .\n$$\n(1)等价于\n$$\nx^2+x z+y z \\geqslant x z+1+x,\n$$\n即 $x^2+\\frac{1}{x} \\geqslant 1+x$, 也即 $(x-1)^2(x+1) \\geqslant 0$. 故(1)成立.\n・又由于\n$$\n\\frac{1}{1+y+\\frac{1}{z}}=\\frac{z}{y z+z+1},\n$$\n且同理有\n$$\n\\frac{x y}{y^2+x y+x z} \\leqslant \\frac{1}{1+z+\\frac{1}{x}}=\\frac{1}{1+z+y z}\n$$\n$$\n\\frac{y z}{z^2+y z+x y} \\leqslant \\frac{1}{1+x+\\frac{1}{y}}=\\frac{x y z}{x y z+x+x z}=\\frac{y z}{1+z+y z} \\text {. }\n$$\n故 $\\sum_{c y c} \\frac{x z}{x^2+x z+y z} \\leqslant \\frac{1+z+y z}{1+z+y z}=1$, 结论成立.", + "remark": "", + "figures": [] +} \ No newline at end of file diff --git a/processed_dataset/proof/1348.json b/processed_dataset/proof/1348.json new file mode 100644 index 0000000000000000000000000000000000000000..025c46a932ba4b6802c228ed66b2eb0a21436e60 --- /dev/null +++ b/processed_dataset/proof/1348.json @@ -0,0 +1,8 @@ +{ + "source_file": "./raw_volume-zh/volume5/chapter6.tex", + "problem_type": "proof", + "problem": "例8. 设实数 $a 、 b 、 c$ 满足: $a+b+c=3$. 求证:\n$$\n\\frac{1}{5 a^2-4 a+11}+\\frac{1}{5 b^2-4 b+11}+\\frac{1}{5 c^2-4 c+11} \\leqslant \\frac{1}{4} .\n$$", + "solution": "证明:若 $a 、 b 、 c$ 都小于 $\\frac{9}{5}$, 则可以证明\n$$\n-\\frac{1}{5 a^2-4 a+11} \\leqslant \\frac{1}{24}(3-a) . \\label{(1)}\n$$\n事实上,\n$$\n\\begin{aligned}\n(1) & \\Leftrightarrow(3-a)\\left(5 a^2-4 a+11\\right) \\geqslant 24 \\\\\n& \\Leftrightarrow 5 a^3-19 a^2+23 a-9 \\leqslant 0 \\\\\n& \\Leftrightarrow(a-1)^2(5 a-9) \\leqslant 0 \\\\\n& \\Leftrightarrow a<\\frac{9}{5} .\n\\end{aligned}\n$$\n同理, 对 $b 、 c$ 也有类似的不等式, 相加便得\n$$\n\\begin{gathered}\n\\frac{1}{5 a^2-4 a+11}+\\frac{1}{5 b^2-4 b+11}+\\frac{1}{5 c^2-4 c+11} \\\\\n\\leqslant \\frac{1}{24}(3-a)+\\frac{1}{24}(3-b)+\\frac{1}{24}(3-c)=\\frac{1}{4} .\n\\end{gathered}\n$$\n若 $a 、 b 、 c$ 中有一个不小于 $\\frac{9}{5}$, 不妨设 $a \\geqslant \\frac{9}{5}$, 则\n$$\n\\begin{gathered}\n5 a^2-4 a+11=5 a\\left(a-\\frac{4}{5}\\right)+11 \\\\\n\\geqslant 5 \\cdot \\frac{9}{5} \\cdot\\left(\\frac{9}{5}-\\frac{4}{5}\\right)+11=20, \\\\\n\\frac{1}{5 a^2-4 a+11} \\leqslant \\frac{1}{20} .\n\\end{gathered}\n$$\n由于 $5 b^2-4 b+11 \\geqslant 5\\left(\\frac{2}{5}\\right)^2-4 \\cdot\\left(\\frac{2}{5}\\right)+11=11-\\frac{4}{5}>10$, 所以 $\\frac{1}{5 b^2-4 b+11}<\\frac{1}{10}$, 同理, $\\frac{1}{5 c^2-4 c+11}<\\frac{1}{10}$, 所以\n$$\n\\frac{1}{5 a^2-4 a+11}+\\frac{1}{5 b^2-4 b+11}+\\frac{1}{5 c^2-4 c+11}<\\frac{1}{20}+\\frac{1}{10}+\\frac{1}{10}=\\frac{1}{4} .\n$$\n因此, 总有 $\\frac{1}{5 a^2-4 a+11}+\\frac{1}{5 b^2-4 b+11}+\\frac{1}{5 c^2-4 c+11} \\leqslant \\frac{1}{4}$, 当且仅当 $a=b=c=1$ 时等号成立.", + "remark": "", + "figures": [] +} \ No newline at end of file diff --git a/processed_dataset/proof/1349.json b/processed_dataset/proof/1349.json new file mode 100644 index 0000000000000000000000000000000000000000..054b286f55d20977e2840750407a079130efcab1 --- /dev/null +++ b/processed_dataset/proof/1349.json @@ -0,0 +1,8 @@ +{ + "source_file": "./raw_volume-zh/volume5/chapter6.tex", + "problem_type": "proof", + "problem": "例9. 设 $n(\\geqslant 3)$ 是整数,求证: 对正实数 $x_1 \\leqslant x_2 \\leqslant \\cdots \\leqslant x_n$, 有不等式\n$$\n\\frac{x_n x_1}{x_2}+\\frac{x_1 x_2}{x_3}+\\cdots+\\frac{x_{n-1} x_n}{x_1} \\geqslant x_1+x_2+\\cdots+x_n .\n$$", + "solution": "证明:先证明一个引理: 若 $02 \\sqrt{\\frac{a_{2004}}{2} \\cdot \\frac{1}{a_{2004}}}=\\sqrt{2} .\n$$\n下面证明: 对一切正整数 $n$, 有\n$$\n\\sqrt{2}0$, 求证: 对任意正整数 $n$, 有不等式\n$$\n\\frac{1+a^2+\\cdots+a^{2 n}}{a+a^3+\\cdots+a^{2 n-1}} \\geqslant \\frac{n+1}{n} .\n$$", + "solution": "证明:对 $n$ 用数学归纳法.\n当 $n=1$ 时, $\\frac{1+a^2}{a} \\geqslant 2$, 故不等式成立.\n设当 $n=k$ 时不等式成立, 即\n$$\nA=\\frac{1+a^2+\\cdots+a^{2 k}}{a+a^3+\\cdots+a^{2 k-1}}>\\frac{k+1}{k} .\n$$\n我们的目标是证明:\n$$\nB=\\frac{1+a^2+\\cdots+a^{2 k+2}}{a+a^3+\\cdots+a^{2 k+1}}>\\frac{k+2}{k+1} .\n$$\n$A$ 和 $B$ 的分母不一样, 怎么把它们联系起来呢? 一个自然的想法是取 $A$ 的倒数.\n注意到: $\\frac{1}{A}<\\frac{k}{k+1}$.\n$$\n\\begin{aligned}\n\\text { 而 } \\frac{1}{A}+B & =\\frac{a+a^3+\\cdots+a^{2 k-1}}{1+a^2+\\cdots+a^{2 k}}+\\frac{1+a^2+\\cdots+a^{2 k+2}}{a+a^3+\\cdots+a^{2 k+1}} \\\\\n& =\\frac{1+2 a^2+2 a^4+\\cdots+2 a^{2 k}+a^{2 k+2}}{a+a^3+\\cdots+a^{2 k+1}}- \\\\\n& =\\frac{\\left(1+a^2\\right)+\\left(a^2+a^4\\right)+\\cdots+\\left(a^{2 k-2}+a^{2 k}\\right)+\\left(a^{2 k}+a^{2 k+2}\\right)}{a+a^3+\\cdots+a^{2 k+1}} \\\\\n& \\geqslant 2 .\n\\end{aligned}\n$$\n故\n$$\nB>2-\\frac{1}{A}>2-\\frac{k}{k+1}=\\frac{k+2}{k+1} .\n$$\n因此当 $n=k+1$ 时不等式成立.\n进而对一切正整数 $n$, 原不等式成立.", + "remark": "", + "figures": [] +} \ No newline at end of file diff --git a/processed_dataset/proof/1353.json b/processed_dataset/proof/1353.json new file mode 100644 index 0000000000000000000000000000000000000000..38fd7c22ee013bbbc01908df9f85e1c1085a712e --- /dev/null +++ b/processed_dataset/proof/1353.json @@ -0,0 +1,8 @@ +{ + "source_file": "./raw_volume-zh/volume5/chapter7.tex", + "problem_type": "proof", + "problem": "例3. 已知正项数列 $\\left\\{a_n\\right\\}$ 对 $n \\in \\mathbf{N}_{+}$都有 $a_n^2 \\leqslant a_n-a_{n+1}$ 成立.\n求证: $a_n<\\frac{1}{n}$.", + "solution": "证法 1 首先, 由 $a_{n+1} \\leqslant a_n-a_n^2=a_n\\left(1-a_n\\right)$ 以及 $\\left\\{a_n\\right\\}$ 每一项恒正, 有:\n$$\na_n\\left(1-a_n\\right)>0 \\Rightarrow 0a_{n+1}$.\n故 $\\quad \\frac{1}{a_{n+1}} \\geqslant \\frac{1}{a_n\\left(1-a_n\\right)}=\\frac{1}{1-a_n}+\\frac{1}{a_n}$,\n即\n$$\n\\frac{1}{a_{n+1}}-\\frac{1}{a_n}=\\frac{1}{1-a_n}>1 \\text {. }\n$$\n于是\n$$\n\\frac{1}{a_n}-\\frac{1}{a_1}=\\sum_{k=1}^n\\left(\\frac{1}{a_{k+1}}-\\frac{1}{a_k}\\right)>n-1 \\text {. }\n$$\n所以 $\\frac{1}{a_n}>(n-1)+\\frac{1}{a_1}>(n-1)+1=n \\Rightarrow a_n<\\frac{1}{n}$.", + "remark": "", + "figures": [] +} \ No newline at end of file diff --git a/processed_dataset/proof/1356.json b/processed_dataset/proof/1356.json new file mode 100644 index 0000000000000000000000000000000000000000..c8c5543a9e5f4ee76574d1ad4cd8b56eddd3c3d1 --- /dev/null +++ b/processed_dataset/proof/1356.json @@ -0,0 +1,8 @@ +{ + "source_file": "./raw_volume-zh/volume5/chapter7.tex", + "problem_type": "proof", + "problem": "例4. 设非负数列 $a_1, a_2, \\cdots$ 满足条件: $a_{m+n} \\leqslant a_n+a_m, m, n \\in \\mathbf{N}_{+}$, 求证: 对任意正整数 $n$, 均有 $a_n \\leqslant m a_1+\\left(\\frac{n}{m}-1\\right) a_m$.", + "solution": "证法 1 令 $m=1$, 可得,\n$$\n0 \\leqslant a_n \\leqslant a_{n-1}+a_1 \\leqslant \\cdots \\leqslant n a_1 . \\label{(1)}\n$$\n下面用归纳法证明, 对任何 $n, m \\in \\mathbf{N}_{+}$命题成立.\n固定 $m \\in \\mathbf{N}_{+}$, 取 $n=1$, 要证明的是\n$$\n\\begin{aligned}\n& \\left(1-\\frac{1}{m}\\right) a_m \\leqslant(m-1) a_1, \\\\\n& (m-1) a_m \\leqslant(m-1) m a_1 .\n\\end{aligned}\n$$\n即由(1)即知此式成立.\n因此当 $n=1$ 时命题成立.\n现假设当 $1 \\leqslant n \\leqslant k$ 时命题成立, 分以下两种情况:\n(i) $k1$.\n现在设命题 $P_k$ 成立, 即\n$$\n\\left(a_1+a_2+\\cdots+a_k\\right)^21$ 符合条件, 进而由归纳法原理知本题结论成立.\n说明本题用了数学归纳法的另一种形式—-螺旋归纳法:\n设 $P(n) 、 Q(n)$ 是两列关于正整数 $n$ 的命题, 如果:\n(1) 命题 $P(1)$ 成立;\n(2)对任何正整数 $k$, 若命题 $P(k)$ 成立, 则命题 $Q(k)$ 成立; 若命题 $Q(k)$ 成立, 则命题 $P(k+1)$ 成立, 那么对所有正整数 $n$, 命题 $P(n)$ 及 $Q(n)$ 都成立.", + "remark": "", + "figures": [] +} \ No newline at end of file diff --git a/processed_dataset/proof/1360.json b/processed_dataset/proof/1360.json new file mode 100644 index 0000000000000000000000000000000000000000..8709e9ef82b3051efd95106d7d7fc2d88b7c337b --- /dev/null +++ b/processed_dataset/proof/1360.json @@ -0,0 +1,8 @@ +{ + "source_file": "./raw_volume-zh/volume5/chapter7.tex", + "problem_type": "proof", + "problem": "例7. 假设 $a_10$ 除以 $a_1, a_{n-1}, a_n$ 中的每一个而不影响不等式.\n利用关于凸函数 $x^4$ 的 Jensen 不等式 (详见第 11 章), 有:\n$$\na_1^4\\left(a_n-a_{n-1}\\right)+a_n^4\\left(a_{n-1}-a_1\\right) \\geqslant\\left(a_1\\left(a_n-a_{n-1}\\right)+a_n\\left(a_{n-1}-a_1\\right)\\right)^4,\n$$\n展开即得结论成立.", + "remark": "", + "figures": [] +} \ No newline at end of file diff --git a/processed_dataset/proof/1361.json b/processed_dataset/proof/1361.json new file mode 100644 index 0000000000000000000000000000000000000000..a5e88a5319ec9927048508149caea0d0c1b22956 --- /dev/null +++ b/processed_dataset/proof/1361.json @@ -0,0 +1,8 @@ +{ + "source_file": "./raw_volume-zh/volume5/chapter7.tex", + "problem_type": "proof", + "problem": "例8. 给定正整数 $n$, 及实数 $x_1 \\leqslant x_2 \\leqslant \\cdots \\leqslant x_n, y_1 \\geqslant y_2 \\geqslant \\cdots \\geqslant y_n$, 满足 :\n$$\n\\sum_{i=1}^n i x_i=\\sum_{i=1}^n i y_i .\n$$\n求证: 对任意实数 $\\alpha$, 有\n$$\n\\sum_{i=1}^n x_i[i \\alpha] \\geqslant \\sum_{i=1}^n y_i[i \\alpha] .\n$$\n这里 $[\\beta]$ 表示不超过实数 $\\beta$ 的最大整数.", + "solution": "证明:我们先证明一个引理: 对任意实数 $x$ 和正整数 $n$, 有\n$$\n\\sum_{i=1}^{n-1}[i \\alpha] \\leqslant \\frac{n-1}{2}[n \\alpha] .\n$$\n引理证明: 只需要将 $[i \\alpha]+[(n-i) \\alpha] \\leqslant[n \\alpha]$ 对 $i=1,2, \\cdots, n-1$ 求和即得.\n回到原题,我们采用归纳法对 $n$ 进行归纳, 当 $n=1$ 时显然正确.\n假设当 $n=k$ 时原命题成立, 考虑当 $n=k+1$ 时.\n令 $a_i=x_i+\\frac{2}{k} x_{k+1}$, $b_i=y_i+\\frac{2}{k} y_{k+1}$, 其中 $i=1,2, \\cdots, k$. 显然我们有 $a_1 \\leqslant a_2 \\leqslant \\cdots \\leqslant a_k, b_1 \\geqslant b_2 \\geqslant \\cdots \\geqslant b_k$, 并且通过计算得知 $\\sum_{i=1}^k i a_i=\\sum_{i=1}^k i b_i$, 由归纳假设知 $\\sum_{i=1}^k a_i[i \\alpha] \\geqslant \\sum_{i=1}^k b_i[i \\alpha]$. 又由于 $x_{k+1} \\geqslant y_{k+1}$, 否则若 $x_{k+1}\\frac{f_{k+2}}{f_{k+3}}$, 那么 $x_1 \\in\\left(\\frac{f_{k+2}}{f_{k+3}}, 1\\right)$. 故\n$$\nx_1+x_2=x_1+\\frac{1}{x_1}-1<\\frac{f_{k+2}}{f_{k+3}}+\\frac{f_{k+3}}{f_{k+2}}-1=\\frac{f_{k+2}}{f_{k+3}}+\\frac{f_{k+1}}{f_{k+2}} .\n$$\n由(2)亦有 $x_1+x_2+\\cdots+x_{k+2}<\\frac{f_1}{f_2}+\\frac{f_2}{f_3}+\\cdots+\\frac{f_{k+2}}{f_{k+3}}$.\n所以结论对 $n=k+2$ 成立.", + "remark": "", + "figures": [] +} \ No newline at end of file diff --git a/processed_dataset/proof/1363.json b/processed_dataset/proof/1363.json new file mode 100644 index 0000000000000000000000000000000000000000..046c27ce399c34edf177a59cec4c5b85508deade --- /dev/null +++ b/processed_dataset/proof/1363.json @@ -0,0 +1,8 @@ +{ + "source_file": "./raw_volume-zh/volume5/chapter7.tex", + "problem_type": "proof", + "problem": "例9. 定义数列 $x_1, x_2, \\cdots, x_n$ 如下: $x_1 \\in[0,1)$, 且\n$$\nx_{n+1}= \\begin{cases}\\frac{1}{x_n}-\\left[\\frac{1}{x_n}\\right], & \\text { 若 } x_n \\neq 0, \\\\ 0, & \\text { 若 } x_n=0 .\\end{cases}\n$$\n求证: 对一切正整数 $n$, 有\n$$\nx_1+x_2+\\cdots+x_n<\\frac{f_1}{f_2}+\\frac{f_2}{f_3}+\\cdots+\\frac{f_n}{f_{n+1}},\n$$\n其中 $\\left\\{f_n\\right\\}$ 为 Fibonacci 数列, $f_1=f_2=1, f_{n+2}=f_{n+1}+f_n, n \\in \\mathbf{N}_{+}$.", + "solution": "证法 2 设 $f(x)=\\frac{1}{1+x}$, 令\n$$\ng_n(x)=x+f(x)+f^{(2)}(x)+\\cdots+f^{(n)}(x), n=0,1,2, \\cdots .\n$$\n注意到 $\\frac{f_1}{f_2}==1$, 且 $f\\left(\\frac{f_i}{f_{i+1}}\\right)=\\frac{1}{1+\\frac{f_i}{f_{i+1}}}=\\frac{f_{i+1}}{f_{i+2}}$, 可得\n$$\n\\begin{gathered}\nf^{(k)}(1)=f^{(k)}\\left(\\frac{f_1}{f_2}\\right)==f^{(k-1)}\\left(\\frac{f_2}{f_3}\\right)=\\cdots=f\\left(\\frac{f_k}{f_{k+1}}\\right)=\\frac{f_{k+1}}{f_{k+2}}, \\\\\nk=1,2, \\cdots .\n\\end{gathered}\n$$\n所以\n$$\ng_{n-1}(1)=\\frac{f_1}{f_2}+\\frac{f_2}{f_3}+\\cdots+\\frac{f_n}{f_{n+1}} .\n$$\n下面先证明一个引理:\n引理( I ) 对任何 $x, y \\in[0,1]$, 若 $x \\neq y$, 则 $|f(x)-f(y)|<|x-y|$, 并且 $f(x)-f(y)$ 与 $x-y$ 符号相反.\n(II) $g_n(x)$ 在 $[0,1]$ 中单调递增.\n引理证明: ( I ) 由 $f(x)-f(y)=\\frac{y-x}{(1+x)(1+y)}$ 易证.\n对于 (II),若 $x>y$, 由 ( I ) 知, 表达式\n$$\n\\begin{aligned}\ng_n(x)-g_n(y)= & (x-y)+(f(x)-f(y)) \\\\\n& +\\cdots+\\left(f^{(n)}(x)-f^{(n)}(y)\\right)\n\\end{aligned}\n$$\n中, 每个差的绝对值小于前一个差的绝对值, 且符号相反.\n又因为 $x-y>0$, 所以 $g_n(x)-g_n(y)>0$.\n回到原题, 如果每个 $x_i(i2, S$ 在 $a_{i-1}=a_{i-2}=\\cdots=a_2=1$ 时不依赖于 $x_n, a_n$, $a_{n-1}, \\cdots, a_i$ 之值取得最大值.\n此时 $S$ 中只有后面 $i-1$ 项中含有 $a_i$, 且这 $i-1$ 项之和为\n$$\ng_{i-2}\\left(\\frac{1}{a_i+\\frac{1}{a_{i+1}+\\frac{1}{\\ddots+\\frac{1}{a_n+x_n}}}}\\right) .\n$$\n由于 $g_{i-2}$ 是增函数,故在 $a_i$ 最小时取到最大值, 即当 $a_i=1$ 时达到最大.\n最后,由前所述,有\n$$\n\\begin{aligned}\nx_n+x_{n-1}+\\cdots+x_1 \\leqslant & x_n+\\frac{1}{1+x_n}+\\frac{1}{1+\\frac{1}{1+x_n}}+\\cdots \\\\\n& +\\frac{1}{1+\\frac{1}{1+\\frac{1}{\\ddots+\\frac{1}{1+x_n}}}} \\\\\n& =g_{n-1}\\left(x_n\\right)\\frac{1}{r_1}+\\frac{1}{r_2}+\\cdots+\\frac{1}{r_n} . \\label{(2)}\n$$\n不妨设\n$$\na_1 \\leqslant a_2 \\leqslant \\cdots \\leqslant a_n \\text {. }\n$$\n那么, 由归纳假设,\n$$\n\\begin{aligned}\n& \\frac{1}{a_1} \\leqslant \\frac{1}{r_1}, \\\\\n& \\frac{1}{a_1}+\\frac{1}{a_2} \\leqslant \\frac{1}{r_1}+\\frac{1}{r_2}, \\\\\n& \\frac{1}{a_1}+\\frac{1}{a_2}+\\cdots+\\frac{1}{a_{n-1}} \\leqslant \\frac{1}{r_1}+\\frac{1}{r_2}+\\cdots+\\frac{1}{r_{n-1}} . \\\\\n&\n\\end{aligned}\n$$\n将以上各式分别乘以非正数 $a_1-a_2, a_2-a_3, \\cdots, a_{n-1}-a_n$; 将(2)乘以 $a_n$, 然后相加得\n$$\nn>\\frac{a_1}{r_1}+\\frac{a_2}{r_2}+\\cdots+\\frac{a_n}{r_n}\n$$\n于是\n$$\n1>\\frac{1}{n}\\left(\\frac{a_1}{r_1}+\\cdots+\\frac{a_n}{r_n}\\right) \\geqslant \\sqrt[n]{\\frac{a_1 a_2 \\cdots a_n}{r_1 r_2 \\cdots r_n}},\n$$\n即\n$$\nr_1 r_2 \\cdots r_n \\geqslant a_1 a_2 \\cdots a_n . \\label{(3)}\n$$\n另一方面, 正数 $1-\\left(\\frac{1}{a_1}+\\frac{1}{a_2}+\\cdots+\\frac{1}{a_n}\\right) \\geqslant \\frac{1}{a_1 a_2 \\cdots a_n}$, 所以由(1)(2)得\n$$\n\\frac{1}{r_1 r_2 \\cdots r_n} \\geqslant \\frac{1}{a_1 a_2 \\cdots a_n} . \\label{(4)}\n$$\n由(3)(4)即可得出矛盾!\n因此(2)不能成立,故结论成立.", + "remark": "说明读者可以进一步推出等号成立当且仅当 $\\left\\{a_1, a_2, \\cdots, a_n\\right\\}=\\left\\{r_1\\right.$, $\\left.r_2, \\cdots, r_n\\right\\}$.\n此题为 Erdös 的一个猜想, 以上证明在归纳法中运用反证法(等于加了条件), 十分巧妙.\n从证明过程中不难验证以下两个命题:\n(A) 设 $x_1 \\geqslant x_2 \\geqslant \\cdots \\geqslant x_n>0, y_1 \\geqslant y_2 \\geqslant \\cdots \\geqslant y_n>0$, 且 $\\sum_{i=1}^n x_i \\leqslant \\sum_{i=1}^n y_i$;\n$$\n\\sum_{i=1}^k x_i \\geqslant \\sum_{i=1}^k y_i(k=1,2, \\cdots, n-1) \\text {, 则 } \\prod_{i=1}^n x_i \\leqslant \\prod_{i=1}^n y_i \\text {. }\n$$\n(B) 设 $a_1 \\geqslant a_2 \\geqslant \\cdots \\geqslant a_n>0$, 且 $\\prod_{i=1}^k b_i \\geqslant \\prod_{i=1}^k a_i(1 \\leqslant k \\leqslant n)$, 则\n$$\n\\sum_{i=1}^n b_i \\geqslant \\sum_{i=1}^n a_i\n$$", + "figures": [] +} \ No newline at end of file diff --git a/processed_dataset/proof/1365.json b/processed_dataset/proof/1365.json new file mode 100644 index 0000000000000000000000000000000000000000..d22d52892f78dc9c643cb3524aa1f2bf4dfa7c7d --- /dev/null +++ b/processed_dataset/proof/1365.json @@ -0,0 +1,8 @@ +{ + "source_file": "./raw_volume-zh/volume5/chapter8.tex", + "problem_type": "proof", + "problem": "例2. 设 $A, B, C$ 为非负实数, $A+B+C=\\frac{\\pi}{2}$, 且 $M=\\sin A+\\sin B+ \\sin C, N=\\sin ^2 A+\\sin ^2 B+\\sin ^2 C$, 则 $M^2+N \\leqslant 3$.", + "solution": "证明:不妨设 $C \\leqslant A, B$, 则 $C \\in\\left[0, \\frac{\\pi}{6}\\right]$,\n$$\n\\begin{gathered}\nM=2 \\sin \\frac{A+B}{2} \\cos \\frac{A-B}{2}+\\sin C \\\\\n=2 \\sin \\left(\\frac{\\pi}{4}-\\frac{C}{2}\\right) \\cos \\frac{A-B}{2}+\\sin C, \\\\\nN=\\frac{1}{2}(1-\\cos 2 A)+\\frac{1}{2}(1-\\cos 2 B)+\\sin ^2 C .\n\\end{gathered}\n$$\n固定 $C$, 有\n$$\n\\begin{aligned}\nM^2+N= & 1+2 \\sin ^2 C+4 \\sin C \\cdot \\sin \\left(\\frac{\\pi}{4}-\\frac{C}{2}\\right) \\cos \\frac{A-B}{2} \\\\\n& +4 \\sin ^2\\left(\\frac{\\pi}{4}-\\frac{C}{2}\\right) \\cdot \\cos ^2 \\frac{A-B}{2}-\\sin C \\cdot \\cos (A-B) \\\\\n= & 1+2 \\sin ^2 C+4 \\sin C \\cdot \\sin \\left(\\frac{\\pi}{4}-\\frac{C}{2}\\right) \\cos \\frac{A-B}{2} \\\\\n& +4 \\sin ^2\\left(\\frac{\\pi}{4}-\\frac{C}{2}\\right) \\cdot \\cos ^2 \\frac{A-B}{2}+\\sin C \\\\\n& -2 \\sin C \\cos ^2 \\frac{A-B}{2} \\\\\n= & 1+2 \\sin ^2 C+\\sin C+4 \\sin \\left(\\frac{\\pi}{2}-\\frac{C}{2}\\right) \\sin C \\\\\n& +\\left[4 \\sin ^2\\left(\\frac{\\pi}{4}-\\frac{C}{2}\\right)-2 \\sin C\\right] \\cos ^2 \\frac{A-B}{2} \\\\\n= & 1+2 \\sin ^2 C+\\sin C+4 \\sin \\left(\\frac{\\pi}{4}-\\frac{C}{2}\\right) \\sin C \\\\\n& +2(1-2 \\sin C) \\cos ^2 \\frac{A-B}{2} \\\\\n\\leqslant & 1+2 \\sin ^2 C+\\sin C+4 \\sin \\left(\\frac{\\pi}{4}-\\frac{C}{2}\\right) \\sin C\n\\end{aligned}\n$$\n$$\n\\begin{aligned}\n& +2(1-2 \\sin C) \\\\\n= & 3+2 \\sin ^2 C-3 \\sin C+4 \\sin \\left(\\frac{\\pi}{4}-\\frac{C}{2}\\right) \\sin C .\n\\end{aligned}\n$$\n令 $t=\\frac{\\pi}{4}-\\frac{C}{2}$, 则 $t \\in\\left[\\frac{\\pi}{6}, \\frac{\\pi}{4}\\right]$, 于是\n$$\n\\begin{aligned}\nM^2+N & =3+2 \\cos ^2 2 t-3 \\cos 2 t+4 \\sin t \\cos 2 t \\\\\n& =3+\\cos 2 t(2 \\cos 2 t-3+4 \\sin t) \\\\\n& =3+\\cos 2 t\\left(-4 \\sin ^2 t+4 \\sin t-1\\right) \\\\\n& =3-\\cos 2 t(2 \\sin t-1)^2 \\\\\n& \\leqslant 3\n\\end{aligned}\n$$\n当 $A=B, \\alpha=\\frac{\\pi}{4}$ (或 $\\frac{\\pi}{6}$ ) 时, 即 $A=B=C=\\frac{\\pi}{6}$ 或 $A=B=\\frac{\\pi}{4}, C=$ 0 , 不等式等号成立.", + "remark": "", + "figures": [] +} \ No newline at end of file diff --git a/processed_dataset/proof/1366.json b/processed_dataset/proof/1366.json new file mode 100644 index 0000000000000000000000000000000000000000..fa12b3f12c5b1f317de45b3161610ceb5cb6b662 --- /dev/null +++ b/processed_dataset/proof/1366.json @@ -0,0 +1,8 @@ +{ + "source_file": "./raw_volume-zh/volume5/chapter8.tex", + "problem_type": "proof", + "problem": "例6. 设 $x 、 y 、 z$ 都是非负实数,且 $x+y+z=1$, 求证:\n$$\ny z+z x+x y-2 x y z \\leqslant \\frac{7}{27} .\n$$", + "solution": "证明:易见, 当 $x=y=z=\\frac{1}{3}$ 时, 不等式中等号成立.\n不妨设 $x \\geqslant y \\geqslant z$, 则 $x \\geqslant \\frac{1}{3} \\geqslant z$.\n令 $\\quad x^{\\prime}=\\frac{1}{3}, y^{\\prime}=y, z^{\\prime}=x+z-\\frac{1}{3}$,\n则\n$$\nx^{\\prime}+z^{\\prime}=x+z, x^{\\prime} \\cdot z^{\\prime} \\geqslant x \\cdot z .\n$$\n所以 $y z+z x+x y-2 x y z=y(x+z)+(1-2 y) x z$\n$$\n\\begin{aligned}\n& \\leqslant y^{\\prime}\\left(x^{\\prime}+z^{\\prime}\\right)+\\left(1-2 y^{\\prime}\\right) x^{\\prime} z^{\\prime} \\\\\n& =\\frac{1}{3}\\left(y^{\\prime}+z^{\\prime}\\right)+\\frac{1}{3} y^{\\prime} z^{\\prime} \\leqslant \\frac{2}{9}+\\frac{1}{27}=\\frac{7}{27} .\n\\end{aligned}\n$$", + "remark": "", + "figures": [] +} \ No newline at end of file diff --git a/processed_dataset/proof/1367.json b/processed_dataset/proof/1367.json new file mode 100644 index 0000000000000000000000000000000000000000..88d97a220bb360cff2f1b2a19b83fa6c42f0fb16 --- /dev/null +++ b/processed_dataset/proof/1367.json @@ -0,0 +1,8 @@ +{ + "source_file": "./raw_volume-zh/volume5/chapter8.tex", + "problem_type": "proof", + "problem": "例8. 设 $a, b, c, d \\geqslant 0$, 且 $a+b+c+d=1$, 求证:\n$$\nb c d+c d a+d a b+a b c \\leqslant \\frac{1}{27}+\\frac{176}{27} a b c d .\n$$", + "solution": "证明:若 $d=0$, 则 $a b c \\leqslant \\frac{1}{27}$, 不等式显然成立.\n若 $a, b, c, d>0$, 只要证明:\n$$\n\\begin{gathered}\nf(a, b, c, d)=\\sum_{c y c} \\frac{1}{a}-\\frac{1}{27} \\frac{1}{a b c d} \\leqslant \\frac{176}{27} . \\\\\na \\leqslant \\frac{1}{4} \\leqslant b, a^{\\prime}=\\frac{1}{4}, b^{\\prime}=a+b-\\frac{1}{4},\n\\end{gathered}\n$$\n令 $\\quad a \\leqslant \\frac{1}{4} \\leqslant b, a^{\\prime}=\\frac{1}{4}, b^{\\prime}=a+b-\\frac{1}{4}$,\n则\n$$\na+b=a^{\\prime}+b^{\\prime}, a^{\\prime} b^{\\prime} \\geqslant a b .\n$$\n于是\n$$\n\\begin{aligned}\n& f(a, b, c, d)-f\\left(a^{\\prime}, b^{\\prime}, c, d\\right) \\\\\n= & \\left(\\frac{1}{a}+\\frac{1}{b}-\\frac{1}{a^{\\prime}}-\\frac{1}{b^{\\prime}}\\right)-\\frac{1}{27} \\cdot \\frac{1}{c d} \\cdot\\left(\\frac{1}{a b}-\\frac{1}{a^{\\prime} b^{\\prime}}\\right)\n\\end{aligned}\n$$\n$$\n\\begin{aligned}\n& =\\frac{a^{\\prime} b^{\\prime}-a b}{a b a^{\\prime} b^{\\prime}}(a+b)\\left(1-\\frac{1}{27} \\frac{1}{c d(a+b)}\\right) \\\\\n& \\leqslant \\frac{a^{\\prime} b^{\\prime}-a b}{a b a^{\\prime} b^{\\prime}}(a+b)\\left[1-\\frac{1}{27} \\cdot\\left(\\frac{a+b+c+d}{3}\\right)^{-3}\\right] \\\\\n& =0 .\n\\end{aligned}\n$$\n故\n$$\nf(a, b, c, d) \\leqslant f\\left(a^{\\prime}, b^{\\prime}, c, d\\right) .\n$$\n至多再进行两次磨光变换, 即可得\n$$\n\\begin{aligned}\nf(a, b, c, d) & \\leqslant f\\left(-\\frac{1}{4}, a+b-\\frac{1}{4}, c, d\\right) \\\\\n& \\leqslant f\\left(\\frac{1}{4}, \\frac{1}{4}, a+b+c-\\frac{1}{2}, d\\right) \\\\\n& \\leqslant f\\left(\\frac{1}{4}, \\frac{1}{4}, \\frac{1}{4}, \\frac{1}{4}\\right)=\\frac{176}{27} .\n\\end{aligned}\n$$\n所以原不等式成立.", + "remark": "", + "figures": [] +} \ No newline at end of file diff --git a/processed_dataset/proof/1368.json b/processed_dataset/proof/1368.json new file mode 100644 index 0000000000000000000000000000000000000000..e71c88eb636457e28ca4975058209e4a282b1935 --- /dev/null +++ b/processed_dataset/proof/1368.json @@ -0,0 +1,8 @@ +{ + "source_file": "./raw_volume-zh/volume5/chapter8.tex", + "problem_type": "proof", + "problem": "例9. 设正实数 $a 、 b 、 c 、 d$ 满足: $a b c d=1$, 求证:\n$$\n\\frac{1}{a}+\\frac{1}{b}+\\frac{1}{c}+\\frac{1}{d}+\\frac{9}{a+b+c+d} \\geqslant \\frac{25}{4} \\text {. }\n$$", + "solution": "证法 1 首先我们证明, 当 $a 、 b 、 c 、 d$ 中有两个相等时, 不等式成立.\n不妨设 $a=b$, 令 $s=a+b+c+d$, 则有\n$$\n\\begin{aligned}\n& \\frac{1}{a}+\\frac{1}{b}+\\frac{1}{c}+\\frac{1}{d}+\\frac{9}{a+b+c}+d=\\frac{2}{a}+\\frac{c+d}{c d}+\\frac{9}{s} \\\\\n& =\\frac{2}{a}+a^2(s-2 a)+\\frac{9}{s}=\\frac{2}{a}-2 a^3+\\left(a^2 s+\\frac{9}{s}\\right) .\n\\end{aligned}\n$$\n下面对 $a$ 的取值分情况讨论:\n若 $a \\geqslant \\frac{\\sqrt{2}}{2}$, 则 $s=a+b+c+d \\geqslant 2 a+\\frac{2}{a} \\geqslant \\frac{3}{a}$, 因此将 $s$ 视为变量, 上式最小值在 $s=2 a+\\frac{2}{a}$ 时取到, 此时\n$$\n\\begin{aligned}\n& \\frac{2}{a}-2 a^3+\\left(a^2 s+\\frac{9}{s}\\right)=\\frac{2}{a}-2 a^3+a^2\\left(2 a+\\frac{2}{a}\\right)+\\frac{9}{s}=\\frac{2}{a}+2 a+\\frac{9}{s}=s+\\frac{9}{s}= \\\\\n& \\frac{7}{16} s+\\frac{9}{16} s+\\frac{9}{s} \\geqslant \\frac{7}{16} \\times 4+2 \\sqrt{\\frac{9}{16} s \\cdot \\frac{9}{s}}=\\frac{7}{4}+\\frac{9}{2}=\\frac{25}{4} \\text {. (这里用到了 } s=2 a+ \\\\\n& \\left.\\frac{2}{a} \\geqslant 4\\right)\n\\end{aligned}\n$$\n若 $0\\frac{2}{a}+5 a \\\\\n& \\geqslant 2 \\sqrt{\\frac{2}{a} \\cdot 5 a}=2 \\sqrt{10}>\\frac{25}{4} .\n\\end{aligned}\n$$\n因此当 $a 、 b 、 c 、 d$ 中有两个相等时,不等式成立.\n下面假设 $a 、 b 、 c 、 d$ 两两不等, 不妨设 $a>b>c>d$. 由于 $\\frac{a d}{c} \\cdot b \\cdot c \\cdot c= a b c d=1$, 故由上面的分析得\n$$\n\\frac{1}{\\frac{a d}{c}}+\\frac{1}{b}+\\frac{1}{c}+\\frac{1}{c}+\\frac{9}{\\frac{a d}{c}+b+c+c} \\geqslant \\frac{25}{4} .\n$$\n下面我们只需证明\n$$\n\\frac{1}{a}+\\frac{1}{b}+\\frac{1}{c}+\\frac{1}{d}+\\frac{9}{a+b+c+d} \\geqslant \\frac{1}{\\frac{a d}{c}}+\\frac{1}{b}+\\frac{1}{c}+\\frac{1}{c}+\\frac{9}{\\frac{a d}{c}+b+c+c} . \\label{(1)}\n$$\n而 (1)\n$$\n\\begin{aligned}\n& \\Leftrightarrow \\frac{1}{a}+\\frac{1}{d}+\\frac{9}{a+b+c+d} \\geqslant \\frac{c}{a d}+\\frac{1}{c}+\\frac{9}{\\frac{a d}{c}+b+2 c} \\\\\n& \\Leftrightarrow \\frac{a c+c d-c^2-a d}{a c d} \\geqslant \\frac{9}{(a+b+c+d)\\left(\\frac{a d}{c}+b+2 c\\right)} \\cdot\\left(a+d-\\frac{a d}{c}-c\\right) \\\\\n& \\Leftrightarrow \\frac{(a-c)(c-d)}{a c d} \\geqslant \\frac{9}{(a+b+c+d)\\left(\\frac{a d}{c}+b+2 c\\right)} \\cdot \\frac{(a-c)(c-d)}{c}, \\\\\n& \\Leftrightarrow \\frac{1}{a d} \\geqslant \\frac{9}{(a+b+c+d)\\left(\\frac{a d}{c}+b+2 c\\right)} \\\\\n& \\Leftrightarrow(a+b+c+d)\\left(\\frac{a d}{c}+b+2 c\\right) \\geqslant 9 a d \\\\\n&\\left.\\Leftrightarrow \\frac{a d}{c}+b+2 c \\geqslant \\sqrt{9 a d} \\text { (因为 } a+b+c+d>\\frac{a d}{c}+b+2 c\\right) \\\\\n& \\Leftrightarrow \\frac{a d}{c}+3 c \\geqslant \\sqrt{9 a d} .\n\\end{aligned}\n$$\n而最后一式可以用均值不等式推出, 这样就证明了结论.", + "remark": "", + "figures": [] +} \ No newline at end of file diff --git a/processed_dataset/proof/1369.json b/processed_dataset/proof/1369.json new file mode 100644 index 0000000000000000000000000000000000000000..e0846dd14c8b0052a9dce30308708436779712d4 --- /dev/null +++ b/processed_dataset/proof/1369.json @@ -0,0 +1,8 @@ +{ + "source_file": "./raw_volume-zh/volume5/chapter8.tex", + "problem_type": "proof", + "problem": "例9. 设正实数 $a 、 b 、 c 、 d$ 满足: $a b c d=1$, 求证:\n$$\n\\frac{1}{a}+\\frac{1}{b}+\\frac{1}{c}+\\frac{1}{d}+\\frac{9}{a+b+c+d} \\geqslant \\frac{25}{4} \\text {. }\n$$", + "solution": "证法 2 不妨设 $a \\leqslant b \\leqslant c \\leqslant d$, 并记 $f(a, b, c, d)=\\frac{1}{a}+\\frac{1}{b}+\\frac{1}{c}+\\frac{1}{d}+\\frac{9}{a+b+c+d} $.\n先证: $f(a, b, c, d) \\geqslant f(\\sqrt{a c}, b, \\sqrt{a c}, d), \\label{(*)}$.\n事实上,上式等价于\n$$\n\\begin{aligned}\n& \\frac{1}{a}+\\frac{1}{c}+\\frac{9}{a+b+c+d} \\geqslant \\frac{1}{\\sqrt{a c}}+\\frac{1}{\\sqrt{a c}}+\\frac{9}{2 \\sqrt{a c}+b+d} \\\\\n\\Leftrightarrow & \\frac{(\\sqrt{a}-\\sqrt{c})^2}{a c} \\geqslant \\frac{9(\\sqrt{a}-\\sqrt{c})^2}{(a+b+c+d)(2 \\sqrt{a c}+b+d)} \\quad\\left(\\text { 因为 }(\\sqrt{a}-\\sqrt{c})^2 \\geqslant 0\\right) \\\\\n\\Leftarrow & (a+b+c+d)(2 \\sqrt{a c}+b+d) \\geqslant 9 a c \\quad \\text { (因为 } b+d \\geqslant 2 \\sqrt{b d}=\\frac{2}{\\sqrt{a c}} \\text { ) } \\\\\n\\leftarrow & \\left(a+c+\\frac{2}{\\sqrt{a c}}\\right)\\left(2 \\sqrt{a c}+\\frac{2}{\\sqrt{a c}}\\right) \\geqslant 9 a c \n\\end{aligned} \\label{(1)}\n$$\n而 $1=a b c d \\geqslant a \\cdot a \\cdot c \\cdot c \\Rightarrow a c \\leqslant 1 \\Rightarrow \\frac{2}{\\sqrt{a c}} \\geqslant 2 \\sqrt{a c}$. 且 $a+c \\geqslant 2 \\sqrt{a c}$, 故\n(1)左边 $\\geqslant\\left(2 \\sqrt{a c}+-\\frac{2}{\\sqrt{a c}}\\right)\\left(2 \\sqrt{a c}+\\frac{2}{\\sqrt{a c}}\\right)>4 \\sqrt{a c} \\cdot 4 \\sqrt{a c}=16 a c> 9 a c=$ (1)右边.\n所以 (*) 成立.\n(*) 说明 $f(a, b, c, d)$ (其中 $a \\leqslant b \\leqslant c \\leqslant d)$ 的最小值 (或极小值) 总是在 $a=c$, 即 $a=b=c$ 时取得.\n欲得到该四元函数的下界, 我们就可不妨设 $(a, b$, $c, d)=\\left(\\frac{1}{t}, \\frac{1}{t}, \\frac{1}{t}, t^3\\right)$, 这里 $t \\geqslant 1$; 这也说明了只需证明对任意的 $t \\geqslant 1$, 总有\n$$\nf\\left(\\frac{1}{t}, \\frac{1}{t}, \\frac{1}{t}, t^3\\right) \\geqslant \\frac{25}{4} , \\label{(**)}\n$$\n就证明了原不等式成立.\n代入, 可知\n$$\n\\begin{aligned}\n& f\\left(\\frac{1}{t}, \\frac{1}{t}, \\frac{1}{t}, t^3\\right) \\geqslant \\frac{25}{4} \\\\\n\\Leftrightarrow & 3 t+\\frac{1}{t^3}+\\frac{9}{t^3+\\frac{3}{t}} \\geqslant \\frac{25}{4} \\\\\n\\Leftrightarrow & 12 t^8-25 t^7+76 t^4-75 t^3+12 \\geqslant 0 \\\\\n\\Leftrightarrow & (t-1)^2\\left(12 t^6-t^5-14 t^4-27 t^3+36 t^2+24 t+12\\right) \\geqslant 0 \\\\\n\\Leftrightarrow & 12 t^6-t^5-14 t^4-27 t^3+36 t^2+24 t+12 \\geqslant 0 \\\\\n\\Leftrightarrow(t-1)\\left(12 t^5+11 t^4-3 t^3-30 t^2+6 t+30\\right)+42 \\geqslant 0 .\n\\end{aligned} \\label{(2)}\n$$\n而 $t \\geqslant 1,12 t^5+6 t \\geqslant 2 \\sqrt{12 t^5 \\cdot 6 t}=12 \\sqrt{2} t^3>3 t^3$,\n$$\n11 t^4+30 \\geqslant 2 \\sqrt{11 t^4 \\cdot 30}=2 \\sqrt{330} t^2>30 t^2 .\n$$\n故 $(t-1)\\left(12 t^5+11 t^4-3 t^3-30 t^2+6 t+30\\right)+42>0$, 所以 (2) 成立.\n至此, $(* *)$ 成立, 原不等式得证.", + "remark": "", + "figures": [] +} \ No newline at end of file diff --git a/processed_dataset/proof/1370.json b/processed_dataset/proof/1370.json new file mode 100644 index 0000000000000000000000000000000000000000..44abbae32c21fef350f9a520764919352e73d7e9 --- /dev/null +++ b/processed_dataset/proof/1370.json @@ -0,0 +1,12 @@ +{ + "source_file": "./raw_volume-zh/volume5/chapter8.tex", + "problem_type": "proof", + "problem": "例10. 求证: 在周长为定值 $l$ 的一切 $n$ 边形中, 正 $n$ 边形有最大的面积.", + "solution": "证明:(1) 首先, 凹多边形不可能具有最大面积.\n如图(), 设 $A_1 A_2 \\cdots A_n$ 为一凹多边形, 则当将 $\\triangle A_{i-1} A_i A_{i+1}$ 以线段 $A_{i+1} A_{i-1}$ 的中点为中心, 中心对称为 $\\triangle A_{i-1} A_i^{\\prime} A_{i+1}$ 时, 得到的凸多边形 $A_1 \\cdots A_{i-1} A_i^{\\prime} A_{i+1} \\cdots A_n$ 的周长也是 $l$, 但面积却变大了.\n因此, 我们在以下的讨论中只考虑凸多边形的情形.\n(2) 设凸多边形 $A_1 A_2 \\cdots A_n$ 不等边, 且其中存在两条邻边, 一边小于 $\\frac{l}{n}$ 而另一边大于 $\\frac{l}{n}$, 不妨设是边 $A_1 A_2$ 和 $A_2 A_3$, 连接 $A_1 A_3$, 以 $A_1 A_3, \\frac{l}{n}, A_1 A_2+ A_2 A_3-\\frac{l}{n}$ 为三边长作 $\\triangle A_1 A_2^{\\prime} A_3$ (如图() 所示).\n则 $A_1 A_2S_{\\triangle A_1 A_2 A_3}$, 从而 $A_1 A_2^{\\prime} A_3 \\cdots A_n$ 的面积大于 $A_1 A_2 \\cdots A_n$ 的面积, 且前者中有一边 $A_1 A_2^{\\prime}$ 的长度为 $\\frac{l}{n}$.\n若多边形 $A_1 A_2^{\\prime} A_3 \\cdots A_n$ 仍不等边, 又可重复上述磨光变换而使边长为 1 的边数每次至少增加 1 条, 故至多经过 $n-1$ 次变换, 就可变为等边 $n$ 边形.\n于是, 不等边 $n$ 边形的面积必小于某等边 $n$ 边形的面积.\n(3) 设凸 $n$ 边形 $A_1 A_2 \\cdots A_n$ 不等边, 于是 $n$ 边中存在两边 $A_i A_{i+1}<\\frac{l}{n}) 所示).\n于是, 得到的新 $n$ 边形与原 $n$ 边形周长、面积都相等, 但新多边形中有两条邻边, 一边大于\n$\\frac{l}{n}$, 另一边小于 $\\frac{l}{n}$, 这样就化为了情形 (2).\n故知任一周长为 $l$ 的不等边 $n$ 边形的面积都小于某一个周长为 $l$ 的等边 $n$ 边形的面积.\n(4) 由克拉美定理, 在 $n$ 条边长都为定值的 $n$ 边形中, 内接于圆的多边形面积最大.\n故在周长为 $l$ 的所有等边 $n$ 边形中, 正 $n$ 边形面积最大.\n综上所述, 在周长为 $l$ 的所有等边 $n$ 边形中, 正 $n$ 边形面积最大.", + "remark": "", + "figures": [ + "./images/volume5/figures/fig-c8i1.png", + "./images/volume5/figures/fig-c8i2.png", + "./images/volume5/figures/fig-c8i3.png" + ] +} \ No newline at end of file diff --git a/processed_dataset/proof/1371.json b/processed_dataset/proof/1371.json new file mode 100644 index 0000000000000000000000000000000000000000..1c13cc77ff0a1021305d6609c923bf5275ff022d --- /dev/null +++ b/processed_dataset/proof/1371.json @@ -0,0 +1,8 @@ +{ + "source_file": "./raw_volume-zh/volume5/chapter9.tex", + "problem_type": "proof", + "problem": "例1. 求证: $\\left|\\sum_{k=1}^n(-1)^k\\left\\{\\frac{n}{k}\\right\\}\\right| \\leqslant 3 \\sqrt{n}\\left(n \\in \\mathbf{N}_{+}\\right)$.", + "solution": "分析:对于单调递降、正负交错的数列, 我们可以采用下列处理方法:\n设 $a_1>a_2>\\cdots>a_k>0$, 则有\n$$\n-a_1t} \\sqrt{\\frac{k_i}{2^i}},\n$$\n其中\n$$\n\\left(\\sum_{i \\leqslant t} \\sqrt{\\frac{k_i}{2^i}}\\right)^2 \\leqslant \\sum_{i=1}^t 1 \\cdot \\sum_{i=1}^t \\frac{k_i}{2^i} \\leqslant t,\n$$\n即\n$$\n\\begin{aligned}\n& \\sum_{i \\leqslant t} \\sqrt{\\frac{k_i}{2^i}} \\leqslant \\sqrt{t}, \\\\\n& )^2 \\leqslant \\sum_{i=t+1}^l k_i \\cdot \\sum_{i=t+1}^l \\frac{1}{2^i} \\leqslant n \\cdot \\frac{1}{2^t},\n\\end{aligned}\n$$\n$$\n\\begin{gathered}\n\\left(\\sum_{i>t} \\sqrt{\\frac{k_i}{2^i}}\\right)^2 \\leqslant \\sum_{i=t+1}^l k_i \\cdot \\sum_{i=t+1}^l \\frac{1}{2^i} \\leqslant n \\cdot \\frac{1}{2^t}, \\\\\n\\sum_{i>t} \\sqrt{\\frac{k_i}{2^i}} \\leqslant \\sqrt{n} \\cdot \\frac{1}{2^{\\frac{t}{2}}},\n\\end{gathered}\n$$\n因此原不等式左边 $\\leqslant \\sqrt{t}+\\frac{\\sqrt{n}}{2^{\\frac{t}{2}}}$.\n取 $t=\\left[\\log _2 n\\right]$, 即有原不等式成立.", + "remark": "", + "figures": [] +} \ No newline at end of file diff --git a/processed_dataset/proof/1374.json b/processed_dataset/proof/1374.json new file mode 100644 index 0000000000000000000000000000000000000000..96cc6e9d9786d213d309ca9db48b308eea3c5e06 --- /dev/null +++ b/processed_dataset/proof/1374.json @@ -0,0 +1,8 @@ +{ + "source_file": "./raw_volume-zh/volume5/chapter9.tex", + "problem_type": "proof", + "problem": "例4. 设正整数 $n \\geqslant 2, x_1, x_2, \\cdots, x_n \\in[0,1]$, 求证: 存在某个 $i, 1 \\leqslant i \\leqslant n-1$, 使得不等式\n$$\nx_i\\left(1-x_{i+1}\\right) \\geqslant \\frac{1}{4} x_1\\left(1-x_n\\right)\n$$\n成立.", + "solution": "证明:令 $m=\\min \\left\\{x_1, x_2, \\cdots, x_n\\right\\}$, 且设 $x_r=m, 0 \\leqslant m \\leqslant 1$, 分两种情况讨论:\n(1) 如果 $x_2 \\leqslant \\frac{1}{2}(m+1)$, 取 $i=1$, 就有\n$$\n\\begin{aligned}\nx_1\\left(1-x_2\\right) & \\geqslant x_1\\left(1-\\frac{1+m}{2}\\right)=\\frac{1}{2} x_1(1-m) \\\\\n& \\geqslant \\frac{1}{2} x_1\\left(1-x_n\\right)\\left(\\text { 利用了 } m \\leqslant x_n \\leqslant 1\\right) \\\\\n& \\geqslant \\frac{1}{4} x_1\\left(1-x_n\\right) .\n\\end{aligned}\n$$\n(2) 如果 $x_2>\\frac{1}{2}(m+1)$, 则有以下两种可能:\n(i) $x_1=m, x_2>\\frac{1}{2}(1+m), \\cdots, x_n>\\frac{1}{2}(1+m)$, 设 $x_k$ 是 $x_2, x_3, \\cdots$, $x_n$ 中的最小值.\n取 $i=k-1$, 就有\n$$\nx_{k-1}\\left(1-x_k\\right) \\geqslant x_1\\left(1-x_n\\right) \\geqslant \\frac{1}{4} x_1\\left(1-x_n\\right),\n$$\n其中第一个不等号利用了 $x_{k-1} \\geqslant x_1$ 及 $1-x_k \\geqslant 1-x_n$.\n(ii) 存在某个 $t, 3 \\leqslant t \\leqslant n$, 使得 $x_t=m \\leqslant \\frac{1}{2}(1+m)$.\n于是一定存在某个正整数 $i, 2 \\leqslant i \\leqslant n-1$, 满足:\n$$\nx_i>\\frac{1}{2}(1+m), x_{i+1} \\leqslant \\frac{1}{2}(1+m) .\n$$\n对于这个 $i$, 有\n$$\n\\begin{aligned}\nx_i\\left(1-x_{i+1}\\right) & >\\frac{1}{2}(1+m)\\left(1-\\frac{m+1}{2}\\right) \\\\\n& =\\frac{1}{2}(1+m) \\cdot \\frac{1}{2}(1-m) \\\\\n& =\\frac{1}{4}\\left(1-m^2\\right) \\geqslant \\frac{1}{4}(1-m) \\geqslant \\frac{1}{4} x_1\\left(1-x_n\\right) .\n\\end{aligned}\n$$\n综上所述, 结论成立.", + "remark": "", + "figures": [] +} \ No newline at end of file diff --git a/processed_dataset/proof/1375.json b/processed_dataset/proof/1375.json new file mode 100644 index 0000000000000000000000000000000000000000..71e39ceb1f7e26bbb9dcc9313a0ff4cc1c80a30d --- /dev/null +++ b/processed_dataset/proof/1375.json @@ -0,0 +1,8 @@ +{ + "source_file": "./raw_volume-zh/volume5/chapter9.tex", + "problem_type": "proof", + "problem": "例5. 已知 $x_1 、 x_2 、 x_3 、 x_4 、 y_1 、 y_2$ 满足:\n$$\n\\begin{gathered}\ny_2 \\geqslant y_1 \\geqslant x_4 \\geqslant x_3 \\geqslant x_2 \\geqslant x_1 \\geqslant 2, \\\\\nx_1+x_2+x_3+x_4 \\geqslant y_1+y_2,\n\\end{gathered}\n$$\n求证: $x_1 x_2 x_3 x_4 \\geqslant y_1 y_2$.", + "solution": "证明:保持 $y_1+y_2$ 不变, 将 $y_1$ 调大至 $y_1 、 y_2$ 相等, 且相等于 $\\frac{y_1+y_2}{2}$, 则 $y_1 y_2$ 在此过程中增大, 因此我们只需对 $y_1=y_2=y$ 的情况作出证明.\n即已知\n$$\n\\begin{gathered}\ny \\geqslant x_4 \\geqslant x_3 \\geqslant x_2 \\geqslant x_1 \\geqslant 2, \\\\\nx_1+x_2+x_3+x_4 \\geqslant 2 y,\n\\end{gathered}\n$$\n要证明:\n$$\nx_1 x_2 x_3 x_4 \\geqslant y^2 \\text {. }\n$$\n下面分三种情况讨论:\n(1) $y \\leqslant 4$, 显然 $x_1 x_2 x_3 x_4 \\geqslant 2^4 \\geqslant y^2$.\n(2) $46$. 保持和 $x_1+x_4 、 x_2+x_3$ 不变, 分情况作调整:\n(i) 如果这两个和均 $\\geqslant y+2$, 则将 $x_4 、 x_3$ 调整为 $y$, 此时结论显然成立.\n(ii) 如果两个和中只有 1 个 $\\geqslant y+2$. 不妨设 $x_1+x_4 \\geqslant y+2$, 则将 $x_4$ 调整为 $y, x_2$ 调整为 2 , 再保持 $x_1+x_3$ 不变, 将 $x_1$ 调整为 2 , 则 $x_1 x_2 x_3 x_4 \\geqslant 2^2 y(y-4)>y^2$, 结论成立.\n(iii) 如果两个和都 $y^2$, 故结论成立.", + "remark": "", + "figures": [] +} \ No newline at end of file diff --git a/processed_dataset/proof/1376.json b/processed_dataset/proof/1376.json new file mode 100644 index 0000000000000000000000000000000000000000..a0293662c170d66f4d13fa35b427428c5b2e7ef9 --- /dev/null +++ b/processed_dataset/proof/1376.json @@ -0,0 +1,8 @@ +{ + "source_file": "./raw_volume-zh/volume5/chapter9.tex", + "problem_type": "proof", + "problem": "例6. 设 $a_1, a_2, \\cdots, a_n(n \\geqslant 3)$ 是实数,求证:\n$$\n\\sum_{i=1}^n a_i^2-\\sum_{i=1}^n a_i a_{i+1} \\leqslant\\left[\\frac{n}{2}\\right](M-m)^2,\n$$\n其中 $a_{n+1}=a_1, M=\\max _{1 \\leqslant i \\leqslant n} a_i, m=\\min _{1 \\leqslant i \\leqslant n} a_i,[x]$ 表示不超过 $x$ 的最大整数.", + "solution": "证明:若 $n=2 k$ ( $k$ 为正整数), 则\n$$\n2\\left(\\sum_{i=1}^n a_i^2-\\sum_{i=1}^n a_i a_{i+1}\\right)=\\sum_{i=1}^n\\left(a_i-a_{i+1}\\right)^2 \\leqslant n \\times(M-m)^2,\n$$\n从而\n$$\n\\sum_{i=1}^n a_i^2-\\sum_{i=1}^n a_i a_{i+1} \\leqslant \\frac{n}{2}(M-m)^2=\\left[\\frac{n}{2}\\right](M-m)^2 .\n$$\n若 $n=2 k+1$ ( $k$ 为正整数), 则对于循环排列的 $2 k+1$ 个数, 必有连续三项递增或递减 (因为 $\\prod_{i=1}^{2 k+1}\\left(a_i-a_{i-1}\\right)\\left(a_{i+1}-a_i\\right)=\\prod_{i=1}^{2 k+1}\\left(a_i-a_{i-1}\\right)^2 \\geqslant 0$, 所以不可能对于每一个 $i$, 都有 $a_i-a_{i-1}$ 与 $a_{i+1}-a_i$ 异号), 不妨设为 $a_1 、 a_2 、 a_3$, 则有\n$$\n\\left(a_1-a_2\\right)^2+\\left(a_2-a_3\\right)^2 \\leqslant\\left(a_1-a_3\\right)^2,\n$$\n从而\n$$\n2\\left(\\sum_{i=1}^n a_i^2-\\sum_{i=1}^n a_i a_{i+1}\\right)=\\sum_{i=1}^n\\left(a_i-a_{i+1}\\right)^2 \\leqslant\\left(a_1-a_3\\right)^2+\\sum_{i=3}^n\\left(a_i-a_{i+1}\\right)^2,\n$$\n这就将问题化为了 $2 k$ 个数的情形.\n我们有\n$$\n2\\left(\\sum_{i=1}^n a_i^2-\\sum_{i=1}^n a_i a_{i+1}\\right) \\leqslant\\left(a_1-a_3\\right)^2+\\sum_{i=3}^n\\left(a_i-a_{i+1}\\right)^2 \\leqslant 2 k(M-m)^2,\n$$\n即\n$$\n\\left(\\sum_{i=1}^n a_i^2-\\sum_{i=1}^n a_i a_{i+1}\\right) \\leqslant k(M-m)^2=\\left[\\frac{n}{2}\\right](M-m)^2,\n$$\n证毕.", + "remark": "", + "figures": [] +} \ No newline at end of file diff --git a/processed_dataset/proof/1377.json b/processed_dataset/proof/1377.json new file mode 100644 index 0000000000000000000000000000000000000000..6bb774a6db181d8972c8cad100358dc11c25b7a0 --- /dev/null +++ b/processed_dataset/proof/1377.json @@ -0,0 +1,8 @@ +{ + "source_file": "./raw_volume-zh/volume5/chapter9.tex", + "problem_type": "proof", + "problem": "例8. 设 $a_1, a_2, \\cdots$ 为无限实数序列, 满足: 存在一个实数 $c$, 对所有 $i$ 有\n$0 \\leqslant a_i \\leqslant c$, 并且 $\\left|a_i-a_j\\right| \\geqslant \\frac{1}{i+j}$ 成立 (对所有 $i \\neq j$ ), 求证: $c \\geqslant 1$.", + "solution": "证明:对固定的 $n \\geqslant 2$, 设序列前 $n$ 项可排序为\n$$\n0 \\leqslant a_{\\sigma(1)}n$, 求证: $\\left(1+\\frac{1}{n}\\right)^n<\\left(1+\\frac{1}{m}\\right)^m$.", + "solution": "我们只需证明: $\\left(1+\\frac{1}{n}\\right)^n<\\left(1+\\frac{1}{n+1}\\right)^{n+1}$.\n当 $n=1$ 时, 上式显然成立, 当 $n \\geqslant 2$ 时,由二项式定理,\n$$\n\\begin{aligned}\n\\left(1+\\frac{1}{n}\\right)^n=\\sum_{k=2}^n & \\frac{1}{k !} \\cdot\\left(1-\\frac{1}{n}\\right)\\left(1-\\frac{2}{n}\\right) \\cdots\\left(1-\\frac{k-1}{n}\\right)+2, \\\\\n\\left(1+\\frac{1}{n+1}\\right)^{n+1}= & \\sum_{k=2}^{n+1} \\frac{1}{k !}\\left(1-\\frac{1}{n+1}\\right)\\left(1-\\frac{2}{n+1}\\right) \\cdots\\left(1-\\frac{k-1}{n+1}\\right)+2 \\\\\n= & \\sum_{k=2}^n \\frac{1}{k !}\\left(1-\\frac{1}{n+1}\\right)\\left(1-\\frac{2}{n+1}\\right) \\cdots \\\\\n& \\left(1-\\frac{k-1}{n+1}\\right)+\\left(\\frac{1}{n+1}\\right)^{n+1}+2,\n\\end{aligned}\n$$\n于是,显然有 $\\left(1+\\frac{1}{n}\\right)^n<\\left(1+\\frac{1}{n+1}\\right)^{n+1}$.", + "remark": "", + "figures": [] +} \ No newline at end of file diff --git a/processed_dataset/proof/1381.json b/processed_dataset/proof/1381.json new file mode 100644 index 0000000000000000000000000000000000000000..c4e8d736104bb4fd1d5e4f20c528e25f25115b15 --- /dev/null +++ b/processed_dataset/proof/1381.json @@ -0,0 +1,8 @@ +{ + "source_file": "./raw_volume-zh/volume5/exercise1.tex", + "problem_type": "proof", + "problem": "问题3. 给定大于 1 的自然数 $a 、 b 、 n, A_{n-1}$ 和 $A_n$ 是 $a$ 进制数, $B_{n-1}$ 和 $B_n$ 是 $b$ 进制数, $A_{n-1} 、 A_n 、 B_{n-1} 、 B_n$ 定义为:\n$$\n\\begin{aligned}\n& A_n=x_n x_{n-1} \\cdots x_0, A_{n-1}=x_{n-1} x_{n-2} \\cdots x_0(\\text { 按 } a \\text { 进制写出 }) \\\\\n& B_n=x_n x_{n-1} \\cdots x_0, B_{n-1}=x_{n-1} x_{n-2} \\cdots x_0(\\text { 按 } b \\text { 进制写出 })\n\\end{aligned}\n$$\n其中 $x_n \\neq 0, x_{n-1} \\neq 0$. 求证: 当 $a>b$ 时,有 $\\frac{A_{n-1}}{A_n}<\\frac{B_{n-1}}{B_n}$.", + "solution": "由于 $A_n>0, B_n>0$, 所以只需证 $A_n B_{n-1}-A_{n-1} B_n>0$, 而\n$$\n\\begin{aligned}\n& A_n B_{n-1}-A_{n-1} B_n \\\\\n= & \\left(x_n \\cdot a^n+A_{n-1}\\right) B_{n-1}-A_{n-1}\\left(x_n b^n+B_{n-1}\\right) \\\\\n= & x_n\\left(a^n B_{n-1}-b^n A_{n-1}\\right) \\\\\n= & x_n\\left[x_{n-1}\\left(a^n b^{n-1}-a^{n-1} b^n\\right)+x_{n-2}\\left(a^n b^{n-2}-a^{n-2} b^n\\right)+\\cdots+x_0\\left(a^n-b^n\\right)\\right] .\n\\end{aligned}\n$$\n因为 $a>b$, 故当 $1 \\leqslant k \\leqslant n$ 时, $a^n \\cdot b^{n-k}>a^{n-k} \\cdot b^n$, 并且 $x_i \\geqslant 0(i=0$, $1,2, \\cdots, n-2), x_{n-1}, x_n \\geqslant 0$, 于是 $A_n B_{n-1}-A_{n-1} B_n>0$.", + "remark": "", + "figures": [] +} \ No newline at end of file diff --git a/processed_dataset/proof/1382.json b/processed_dataset/proof/1382.json new file mode 100644 index 0000000000000000000000000000000000000000..b132f9b8b06a97d68c095abec0a9c688370ca748 --- /dev/null +++ b/processed_dataset/proof/1382.json @@ -0,0 +1,8 @@ +{ + "source_file": "./raw_volume-zh/volume5/exercise1.tex", + "problem_type": "proof", + "problem": "问题4. 设 $a, b, c \\in \\mathbf{R}^{+}$, 求证:\n$$\n\\frac{1}{a}+\\frac{1}{b}+\\frac{1}{c} \\leqslant \\frac{a^8+b^8+c^8}{a^3 b^3 c^3} .\n$$", + "solution": "反复利用 $a^2+b^2+c^2 \\geqslant a b+b c+c a(a, b, c \\in \\mathbf{R})$, 有\n$$\na^8+b^8+c^8 \\geqslant a^4 b^4+b^4 c^4+c^4 a^4 \\geqslant a^2 b^4 c^2+b^2 c^4 a^2+c^2 a^4 b^2\n$$\n$$\n==a^2 b^2 c^2\\left(a^2+b^2+c^2\\right) \\geqslant a^2 b^2 c^2(a b+b c+c a) .\n$$\n两边同除以 $a^3 b^3 c^3$ 即得原不等式成立.", + "remark": "", + "figures": [] +} \ No newline at end of file diff --git a/processed_dataset/proof/1383.json b/processed_dataset/proof/1383.json new file mode 100644 index 0000000000000000000000000000000000000000..735eee6d4912357a20d85c714cab55c955d15b10 --- /dev/null +++ b/processed_dataset/proof/1383.json @@ -0,0 +1,8 @@ +{ + "source_file": "./raw_volume-zh/volume5/exercise1.tex", + "problem_type": "proof", + "problem": "问题6. 已知 $5 n$ 个实数 $r_i 、 s_i 、 t_i 、 u_i 、 v_i$ 都大于 $1(1 \\leqslant i \\leqslant n)$, 记 $R=\\frac{1}{n} \\cdot \\sum_{i=1}^n r_i$, $S=\\frac{1}{n} \\cdot \\sum_{i=1}^n s_i, T=\\frac{1}{n} \\cdot \\sum_{i=1}^n t_i, U=\\frac{1}{n} \\cdot \\sum_{i=1}^n u_i, V=\\frac{1}{n} \\cdot \\sum_{i=1}^n v_i$. 求证下述不等式成立:\n$$\n\\prod_{i=1}^n\\left(\\frac{r_i s_i t_i u_i v_i+1}{r_i s_i t_i u_i v_i-1}\\right) \\geqslant\\left(\\frac{R S T U V+1}{R S T U V-1}\\right)^n .\n$$", + "solution": "先证明一个引理:\n引理: 设 $x_1, x_2, \\cdots, x_n$ 为 $n$ 个大于 1 的实数, $A=\\sqrt[n]{x_1} \\overline{x_2 \\cdots x_n}$, 则\n$$\n\\prod_{i=1}^n \\frac{\\left(x_i+1\\right)}{\\left(x_i-1\\right)} \\geqslant\\left(\\frac{A+1}{A-1}\\right)^n \\text {. }\n$$\n证明: 不妨设 $x_1 \\leqslant x_2 \\leqslant \\cdots \\leqslant x_n$, 则 $x_1 \\leqslant A \\leqslant x_n$.\n用通分不难证明: $\\frac{\\left(x_1+1\\right)\\left(x_n+1\\right)}{\\left(x_1-1\\right)\\left(x_n-1\\right)} \\geqslant\\left(\\frac{A+1}{A-1}\\right) \\cdot\\left(\\frac{\\frac{x_1 x_n}{A}+1}{\\frac{x_1 x_n}{A}-1}\\right)$.\n于是, $\\prod_{i=1}^n\\left(\\frac{x_i+1}{x_i-1}\\right) \\geqslant \\prod_{i=2}^{n-1}\\left(\\frac{x_i+1}{x_i-1}\\right)\\left(\\frac{\\frac{x_1 x_n}{A}+1}{\\frac{x_1 x_n}{A}-1}\\right)\\left(\\frac{A+1}{A-1}\\right)$.\n再考虑剩下的 $n-1$ 个实数: $x_2, x_3, \\cdots, x_{n-1}$ 和 $x_1 x_n$, 它们的几何平均值仍为 $A$, 故这 $n-1$ 个数中亦存在最大、最小值, 且最大值不小于 $A$, 最小值不大于 $A$. 采取同上的做法, 经过 $n-1$ 次即可得: $\\prod_{i=1}^n\\left(\\frac{x_i+1}{x_i-1}\\right) \\geqslant\\left(\\frac{A+1}{A-1}\\right)^n$.\n下面证明原命题.\n令 $x_i=r_i s_i t_i u_i v_i(1 \\leqslant i \\leqslant n)$, 由引理可得\n$$\n\\prod_{i=1}^n \\frac{r_i s_i t_i u_i v_i+1}{r_i s_i t_i u_i v_i-1} \\geqslant\\left(\\frac{B+\\dot{1}}{B-1}\\right)^n,\n$$\n其中 $B=\\sqrt[n]{\\prod_{i=1}^n\\left(r_i s_i t_i u_i v_i\\right)}$. 因此只需证明: $\\frac{B+1}{B-1} \\geqslant \\frac{R S T U V+1}{R S T U V-1}$, 而\n$$\n\\begin{aligned}\n\\text { RSTUV } & =\\frac{1}{n} \\cdot\\left(\\sum_{i=1}^n r_i\\right) \\cdot \\frac{1}{n} \\cdot\\left(\\sum_{i=1}^n s_i\\right) \\cdot \\frac{1}{n}\\left(\\sum_{i=1}^n t_i\\right) \\cdot \\frac{1}{n}\\left(\\sum_{i=1}^n u_i\\right) \\cdot \\frac{1}{n}\\left(\\sum_{i=1}^n v_i\\right) \\\\\n& \\geqslant \\sqrt[n]{\\prod_{i=1}^n r_i} \\cdot \\sqrt[n]{\\prod_{i=1}^n s_i} \\cdot \\sqrt[n]{\\prod_{i=1}^n t_i} \\cdot \\sqrt[n]{\\prod_{i=1}^n u_i} \\cdot \\sqrt[n]{\\prod_{i=1}^n v_i}=B\n\\end{aligned}\n$$\n故 $(B+1)(R S T U V-1)-(B-1)(R S T U V+1)=2(R S T U V-B) \\geqslant 0$.\n所以结论成立.\n原不等式得证.", + "remark": "注::我们也可以用 Jensen 不等式来证.\n首先, 不难证明, 对任意 $a, b>1$, 有 $\\left(\\frac{a+1}{a-1}\\right) \\cdot\\left(\\frac{b+1}{b-1}\\right) \\geqslant\\left(\\frac{\\sqrt{a b}+1}{\\sqrt{a b}-1}\\right)^2$,\n故函数 $y=\\ln \\left(\\frac{\\mathrm{e}^x+1}{\\mathrm{e}^x-1}\\right)$ 在区间 $(0,+\\infty)$ 上是凸函数, 于是\n$$\n\\prod_{i=1}^n\\left(\\frac{r_i s_i t_i u_i v_i+1}{r_i s_i t_i u_i v_i-1}\\right) \\geqslant \\frac{\\sqrt[n]{\\prod_{i=1}^n r_i s_i t_i u_i v_i}+1}{\\sqrt[n]{\\prod_{i=1}^n r_i s_i t_i u_i v_i}-1} \\geqslant\\left(\\frac{R S T U V+1}{R S T U V-1}\\right)^n .\n$$", + "figures": [] +} \ No newline at end of file diff --git a/processed_dataset/proof/1384.json b/processed_dataset/proof/1384.json new file mode 100644 index 0000000000000000000000000000000000000000..5bf636d66a423370f902126e9e7017b52f645021 --- /dev/null +++ b/processed_dataset/proof/1384.json @@ -0,0 +1,8 @@ +{ + "source_file": "./raw_volume-zh/volume5/exercise1.tex", + "problem_type": "proof", + "problem": "问题7. 设 $k 、 n$ 是正整数, $1 \\leqslant k0$ (否则命题显然成立). 此时 $S_1>0, S_3<0$, 故\n$$\n\\begin{aligned}\n& 27\\left(S_1 S_2-S_3\\right)^2-64 S_2^3>27 S_1^2 S_2^2-64 S_2^3 \\\\\n= & S_2^2 \\cdot\\left(27 S_1^2-64 S_2\\right) \\\\\n= & S_2^2 \\cdot\\left[27 a^2+22\\left(b^2+c^2\\right)+5(b-c)^2-10 a(b+c)\\right] \\\\\n> & 0 .\n\\end{aligned}\n$$\n题中等号成立, 当且仅当 $a=b=c$.", + "remark": "", + "figures": [] +} \ No newline at end of file diff --git a/processed_dataset/proof/1386.json b/processed_dataset/proof/1386.json new file mode 100644 index 0000000000000000000000000000000000000000..7353eb8cf7871f6ff13b9b55a0556e1be4373e84 --- /dev/null +++ b/processed_dataset/proof/1386.json @@ -0,0 +1,8 @@ +{ + "source_file": "./raw_volume-zh/volume5/exercise1.tex", + "problem_type": "proof", + "problem": "问题9. 求证: 对任意 $c>0$, 存在正整数 $n$ 和复数列 $a_1, a_2, \\cdots, a_n$, 使\n$$\nc \\cdot \\frac{1}{2^n} \\sum_{\\varepsilon_1, \\varepsilon_2, \\cdots, \\varepsilon_n}\\left|\\varepsilon_1 a_1+\\varepsilon_2 a_2+\\cdots+\\varepsilon_n a_n\\right|<\\left(\\sum_{j=1}^n\\left|a_j\\right|^{\\frac{3}{2}}\\right)^{\\frac{2}{3}} .\n$$\n其中 $\\varepsilon_j \\in\\{-1,1\\}, j=1,2, \\cdots, n$.", + "solution": "考虑 $S=\\sum_{\\varepsilon_1, \\varepsilon_2, \\cdots, \\varepsilon_n}\\left|\\left(a_1 \\varepsilon_1+a_2 \\varepsilon_2+\\cdots+a_n \\varepsilon_n\\right)\\right|^2$\n$$\n\\begin{aligned}\n& =\\sum_{\\varepsilon_1, \\varepsilon_2, \\cdots, \\varepsilon_n}\\left(a_1 \\varepsilon_1+a_2 \\varepsilon_2+\\cdots+a_n \\varepsilon_n\\right)\\left(\\bar{a}_1 \\varepsilon_1+\\bar{a}_2 \\varepsilon_2+\\cdots+\\bar{a}_n \\varepsilon_n\\right) \\\\\n& =\\sum_{\\varepsilon_1, \\varepsilon_2, \\cdots, \\varepsilon_n}\\left(\\left|a_1\\right|^2+\\left|a_2\\right|^2+\\cdots+\\left|a_n\\right|^2+\\sum_{i \\neq j} a_i \\bar{a}_j \\varepsilon_i \\varepsilon_j\\right) .\n\\end{aligned}\n$$\n不妨取 $\\left|a_i\\right|=1, i=1,2, \\cdots, n$, 则\n$$\n\\begin{aligned}\nS & =n \\cdot 2^n+\\sum_{\\varepsilon_1, \\varepsilon_2, \\cdots, \\varepsilon_n} \\sum_{i \\neq j} a_i \\bar{a}_j \\varepsilon_i \\varepsilon_j \\\\\n& =n \\cdot 2^n+\\sum_{i \\neq j} a_i \\bar{a}_j \\sum_{\\varepsilon_1, \\varepsilon_2, \\cdots, \\varepsilon_n} \\varepsilon_i \\varepsilon_j \\\\\n& =n \\cdot 2^n .\n\\end{aligned}\n$$\n现要求 $n^{\\frac{2}{3}}>c \\cdot \\frac{1}{2^n} \\cdot \\sum_{\\varepsilon_1, \\varepsilon_2, \\cdots, \\varepsilon_n}\\left|\\varepsilon_1 a_1+\\varepsilon_2 a_2+\\cdots+\\varepsilon_n a_n\\right|$, 只需 $c \\cdot \\sqrt{\\frac{S}{2^n}}= c \\cdot \\sqrt{n} \\leqslant n^{\\frac{2}{3}}$, 即 $n^{\\frac{1}{6}} \\geqslant c$, 故取 $n \\geqslant c^6$ 即可.", + "remark": "", + "figures": [] +} \ No newline at end of file diff --git a/processed_dataset/proof/1387.json b/processed_dataset/proof/1387.json new file mode 100644 index 0000000000000000000000000000000000000000..0f405f7e5d271f7aa626f42cb3cc081b56254246 --- /dev/null +++ b/processed_dataset/proof/1387.json @@ -0,0 +1,8 @@ +{ + "source_file": "./raw_volume-zh/volume5/exercise1.tex", + "problem_type": "proof", + "problem": "问题11. 设 $n$ 个实数,它们的绝对值都小于等于 2 , 其立方和为 0 . 求证: 它们的和 $\\leqslant \\frac{2}{3} n$.", + "solution": "记这 $n$ 个实数为 $y_1, y_2, \\cdots, y_n$. 令 $x_i=\\frac{y_i}{2}$, 则 $\\left|x_i\\right| \\leqslant 1, \\sum_{i=1}^n x_i^3=$ 0. 要证明: $\\sum_{i=1}^n x_i \\leqslant \\frac{1}{3} n$.\n用待定系数法: 设 $x_i \\leqslant \\frac{1}{3}+\\lambda x_i^3$. 令 $x=\\cos \\theta$, 由三倍角公式易知 $\\lambda=\\frac{4}{3}$ 可使上式成立, 从而原不等式获证.", + "remark": "", + "figures": [] +} \ No newline at end of file diff --git a/processed_dataset/proof/1388.json b/processed_dataset/proof/1388.json new file mode 100644 index 0000000000000000000000000000000000000000..acb13ec0780a438ecfecdae64db9b76953ea2b15 --- /dev/null +++ b/processed_dataset/proof/1388.json @@ -0,0 +1,8 @@ +{ + "source_file": "./raw_volume-zh/volume5/exercise1.tex", + "problem_type": "proof", + "problem": "问题12. 已知正整数 $n \\geqslant 3,[-1,1]$ 中的实数 $x_1, x_2, \\cdots, x_n$ 满足: $\\sum_{k=1}^n x_k^5=0$. 求证: $\\sum_{k=1}^n x_k \\leqslant \\frac{8}{15} n$.", + "solution": "由于 $x_k \\in[-1,1]$, 则 $0 \\leqslant\\left(1+x_k\\right)\\left(B x_k-1\\right)^4$, 这里 $B$ 是一个待定实数, 展开, 得: $B^4 x_k^5+\\left(B^4-4 B^3\\right) x_k^4+B^2(6-4 B) x_k^3+B(4 B-6) x_k^2+(1-$ 4B) $x_k+1 \\geqslant 0$.\n令 $B=\\frac{3}{2}$, 从上式得 $\\frac{81}{16} x_k^5-\\frac{135}{16} x_k^4+\\frac{15}{2} x_k^2-5 x_k+1 \\geqslant 0$. 对 $k$ 从 1 到 $n$ 求和, 即有\n$$\n\\begin{aligned}\n5 \\sum_{k=1}^n x_k & \\leqslant n-\\frac{135}{16} \\sum_{k=1}^n x_k^4+\\frac{15}{2} \\sum_{k=1}^n x_k^2 \\\\\n& =n+\\frac{15}{2}\\left(\\sum_{k=1}^n x_k^2-\\frac{9}{8} \\sum_{k=1}^n x_k^4\\right) \\\\\n& \\leqslant n+\\frac{15}{2} \\cdot \\frac{2}{9} n=\\frac{8}{3} n,\n\\end{aligned}\n$$\n故 $\\sum_{k=1}^n x_k \\leqslant \\frac{8}{15} n$.", + "remark": "", + "figures": [] +} \ No newline at end of file diff --git a/processed_dataset/proof/1389.json b/processed_dataset/proof/1389.json new file mode 100644 index 0000000000000000000000000000000000000000..1f48de38fbaad1e4ad7b96ab4f3df6884e49ade0 --- /dev/null +++ b/processed_dataset/proof/1389.json @@ -0,0 +1,8 @@ +{ + "source_file": "./raw_volume-zh/volume5/exercise1.tex", + "problem_type": "proof", + "problem": "问题13. 已知 $n$ 个实数 $x_1, x_2, \\cdots, x_n$ 的算术平均值为 $a$. 证明:\n$$\n\\sum_{k=1}^n\\left(x_k-a\\right)^2 \\leqslant \\frac{1}{2}\\left(\\sum_{k=1}^n\\left|x_k-a\\right|\\right)^2 .\n$$", + "solution": "先证明 $a=0$ 时的情况.\n此时, $\\sum_{k=1}^n x_k^2=-2 \\sum_{1 \\leqslant i0, b_1 \\geqslant b_2 \\geqslant \\cdots \\geqslant b_n>0$, 并且, 有 $a_1 a_2 \\cdots a_n=b_1 b_2 \\cdots b_n ; \\sum_{1 \\leqslant i2 a_1$, 故 $2\\left(b_1+b_2+b_3\\right)>2 b_1>4 a_1> a_1+a_2+a_3$, 结论仍成立.\n对当 $n \\geqslant 3$ 时的一般情况, 无妨设 $b_1 b_2 \\cdots b_n=1$.\n如果 $a_1 \\leqslant n-1$, 则 $\\sum_{i=1}^n a_i \\leqslant n(n-1) \\leqslant(n-1) \\sum_{i=1}^n b_i$, 结论成立.\n下设 $a_1>n-1$, 则\n$$\n\\begin{aligned}\n\\sum_{1 \\leqslant i2(n-1) b_n+2 b_2 \\geqslant 2 n b_n$, 故 $a_n>2 b_n$. 又由 $a_1 a_2 \\cdots a_n=1$\n得 $a_n \\leqslant 1$, 所以 $a_1-(n-1) a_n>(n-1)-(n-1)=0$. 故 $2 b_22 b_n \\cdot b_2 \\cdot a_1 a_2 \\cdots a_{n-2}$, 即有 $b_1 b_3 b_4 \\cdots b_{n-1}>2 a_1 a_2 \\cdots a_{n-2}$.\n而 $b_3 \\leqslant b_22 a_1$.\n所以 $(n-1) \\sum_{i=1}^n b_i>2(n-1) a_1>n a_1 \\geqslant \\sum_{i=1}^n a_i(n \\geqslant 3)$.\n结论也成立.", + "remark": "", + "figures": [] +} \ No newline at end of file diff --git a/processed_dataset/proof/1392.json b/processed_dataset/proof/1392.json new file mode 100644 index 0000000000000000000000000000000000000000..af3f3070200b8c42f66c17335548694b0c1429e9 --- /dev/null +++ b/processed_dataset/proof/1392.json @@ -0,0 +1,8 @@ +{ + "source_file": "./raw_volume-zh/volume5/exercise1.tex", + "problem_type": "proof", + "problem": "问题16. 设 $x, y, z \\in \\mathbf{R}^{+}$, 求证:\n$$\n(x y+y z+z x)\\left[\\frac{1}{(x+y)^2}+\\frac{1}{(y+z)^2}+\\frac{1}{(z+x)^2}\\right] \\geqslant \\frac{9}{4} .\n$$", + "solution": "易见,此题等价于证明: $\\sum_{c y c} \\frac{y z}{x(y+z)^2} \\geqslant \\frac{9}{4(x+y+z)}$.\n不妨设 $x \\geqslant y \\geqslant z$,且 $x+y+z=1$. 则\n$$\n\\begin{aligned}\n\\sum_{c y c} \\frac{4 y z}{x(y+z)^2} & =\\sum_{c y c} \\frac{(y+z)^2-(y-z)^2}{x(y+z)^2} \\\\\n& =\\sum_{c y c} \\frac{1}{x}-\\sum_{c y c} \\frac{(y-z)^2}{x(y+z)^2} \\\\\n& =\\sum_{c y c} \\frac{1}{x} \\cdot \\sum_{c y c} x-\\sum_{c y c} \\frac{(y-z)^2}{x(y+z)^2} \\\\\n& =9+\\sum_{c y c}\\left(\\frac{\\sqrt{z}}{\\sqrt{y}}-\\sqrt{\\frac{y}{z}}\\right)^2-\\sum_{c y c} \\frac{(y-z)^2}{x(y+z)^2} \\\\\n& =9+\\sum_{c y c}\\left(\\frac{(y-z)^2}{y z}-\\frac{(y-z)^2}{x(y+z)^2}\\right) \\\\\n& =9+S,\n\\end{aligned}\n$$\n式中, $S=\\sum_{c y c}(y-z)^2 \\cdot\\left(\\frac{1}{y z}-\\frac{1}{x(y+z)^2}\\right)$.\n由于 $x>\\frac{1}{4}$, 所以\n$$\n\\frac{1}{y z} \\geqslant \\frac{4}{(y+z)^2}>\\frac{1}{x(y+z)^2},\n$$\n又由于\n$$\n\\begin{gathered}\n\\frac{1}{x z}-\\frac{1}{y(x+z)^2} \\geqslant 0 \\\\\ny(x+z)^2 \\geqslant x z(x+y+z) \\\\\nx^2(y-z)+x z(y-z)+y z^2 \\geqslant 0\n\\end{gathered}\n$$\n因此 $S \\geqslant(x-y)^2 \\cdot\\left(\\frac{1}{x z}-\\frac{1}{y(x+z)^2}+\\frac{1}{x y}-\\frac{1}{z(x+y)^2}\\right)$,\n而\n$$\n\\begin{aligned}\n& \\frac{1}{x z}-\\frac{1}{y(x+z)^2}+\\frac{1}{x y}-\\frac{1}{z(x+y)^2} \\\\\n= & \\frac{(x+z)^2-x}{x y(x+z)^2}+\\frac{(x+y)^2-x}{x z(x+y)^2}\n\\end{aligned}\n$$\n$$\n\\begin{aligned}\n& \\geqslant \\frac{(x+y)^2-x+(x+z)^2-x}{x y(x+z)^2} \\\\\n& =\\frac{2 x^2+2 x y+2 x z+y^2+z^2-2 x(x+y+z)}{x y(x+z)^2} \\geqslant 0,\n\\end{aligned}\n$$\n于是结论成立.", + "remark": "", + "figures": [] +} \ No newline at end of file diff --git a/processed_dataset/proof/1393.json b/processed_dataset/proof/1393.json new file mode 100644 index 0000000000000000000000000000000000000000..def3d59db4e7ac5ca178e8a1ef20dde3dcbd770c --- /dev/null +++ b/processed_dataset/proof/1393.json @@ -0,0 +1,8 @@ +{ + "source_file": "./raw_volume-zh/volume5/exercise1.tex", + "problem_type": "proof", + "problem": "问题17. 求证: 在锐角 $\\triangle A B C$ 中, 有\n$$\n\\begin{gathered}\n\\cot ^3 A+\\cot ^3 B+\\cot ^3 C+6 \\cot A \\cot B \\cot C \\\\\n\\geqslant \\cot A+\\cot B+\\cot C .\n\\end{gathered}\n$$", + "solution": "令 $x=\\cot A, y=\\cot B, z=\\cot C$, 由 $A+B+C=\\pi$ 知, $\\cot A \\cot B+ \\cot B \\cot C+\\cot C \\cot A=1$, 即 $x y+y z+z x=1$. 所以, 欲证的不等式等价是\n$$\nx^3+y^3+z^3+6 x y z \\geqslant(x+y+z)(x y+y z+z x),\n$$\n此即 $x(x-y)(x-z)+y(y-z)(y-x)+z(z-x)(z-y) \\geqslant 0$.\n由 Schur 不等式知原不等式成立.", + "remark": "", + "figures": [] +} \ No newline at end of file diff --git a/processed_dataset/proof/1394.json b/processed_dataset/proof/1394.json new file mode 100644 index 0000000000000000000000000000000000000000..ee04bdd99a2df1923e09c4d89fc40aa1388df88e --- /dev/null +++ b/processed_dataset/proof/1394.json @@ -0,0 +1,8 @@ +{ + "source_file": "./raw_volume-zh/volume5/exercise1.tex", + "problem_type": "proof", + "problem": "问题18. 设 $a, b, c \\in\\left(0, \\frac{\\pi}{2}\\right)$, 求证:\n$$\n\\begin{gathered}\n\\frac{\\sin a \\sin (a-b) \\sin (a-c)}{\\sin (b+c)}+\\frac{\\sin b \\sin (b-c) \\sin (b-a)}{\\sin (c+a)} \\\\\n+\\frac{\\sin c \\sin (c-a) \\sin (c-b)}{\\sin (a+b)} \\geqslant 0 .\n\\end{gathered}\n$$", + "solution": "因为 $\\sin (x-y) \\sin (x+y)=\\frac{1}{2}(\\cos 2 \\beta-\\cos 2 \\alpha)=\\sin ^2 \\alpha-\\sin ^2 \\beta$, 所以\n$$\n\\begin{aligned}\n& \\sin a \\sin (a-b) \\sin (a-c) \\sin (a+b) \\sin (a+c) \\\\\n&= \\sin a\\left(\\sin ^2 a-\\sin ^2 b\\right)\\left(\\sin ^2 a-\\sin ^2 c\\right) . \\\\\n& \\sin ^2 a, y=\\sin ^2 b, z=\\sin ^2 c, \\text { 则原不等式等价于 } \\\\\n&y)(x-z)+y^{\\frac{1}{2}}(y-z)(y-x)+z^{\\frac{1}{2}}(z-x)(z-y) \\geqslant 0,\n\\end{aligned}\n$$\n令 $x=\\sin ^2 a, y=\\sin ^2 b, z=\\sin ^2 c$, 则原不等式等价于\n$$\nx^{\\frac{1}{2}}(x-y)(x-z)+y^{\\frac{1}{2}}(y-z)(y-x)+z^{\\frac{1}{2}}(z-x)(z-y) \\geqslant 0,\n$$\n此即 Schur 不等式.", + "remark": "", + "figures": [] +} \ No newline at end of file diff --git a/processed_dataset/proof/1395.json b/processed_dataset/proof/1395.json new file mode 100644 index 0000000000000000000000000000000000000000..ed219a18473ef4d0086f8bd068645c0a2c12adb7 --- /dev/null +++ b/processed_dataset/proof/1395.json @@ -0,0 +1,8 @@ +{ + "source_file": "./raw_volume-zh/volume5/exercise1.tex", + "problem_type": "proof", + "problem": "问题19. 设正实数 $a\\cdot b\\cdot c$ 满足: $a^{2}+b^{2}+c^{2}+(a+b+c)^{2}\\leq4$ ,求证:\n$$\n\\frac{a b+1}{(a+b)^2}+\\frac{b c+1}{(b+c)^2}+\\frac{c a+1}{(c+a)^2} \\geqslant 3 .\n$$", + "solution": "由已知得: $a^2+b^2+c^2+a b+b c+a c \\leqslant 2$, 所以\n$$\n\\begin{aligned}\n\\sum_{c y c} \\frac{2 a b+2}{(a+b)^2} & \\geqslant \\sum_{c y c} \\frac{2 a b+a^2+b^2+c^2+a b+b c+a c}{(a+b)^2} \\\\\n& =\\sum_{c y c} \\frac{(a+b)^2+(c+a)(c+b)}{(a+b)^2} \\\\\n& =3+\\sum_{c y c} \\frac{(c+a)(c+b)}{(a+b)^2} \\geqslant 6,\n\\end{aligned}\n$$\n其中最后一个不等号是利用了平均值不等式.\n两边同时除以 2 即知原不等式成立.", + "remark": "", + "figures": [] +} \ No newline at end of file diff --git a/processed_dataset/proof/1396.json b/processed_dataset/proof/1396.json new file mode 100644 index 0000000000000000000000000000000000000000..c94f6e104526cc5ab916b5809615ca8ac4903f47 --- /dev/null +++ b/processed_dataset/proof/1396.json @@ -0,0 +1,8 @@ +{ + "source_file": "./raw_volume-zh/volume5/exercise1.tex", + "problem_type": "proof", + "problem": "问题20. 设正实数 $a 、 b 、 c$ 满足: $a b c=1$, 求证: 对于整数 $k \\geqslant 2$, 有 $\\frac{a^k}{a+b}+\\frac{b^k}{b+c}+\\frac{c^k}{c+a} \\geqslant \\frac{3}{2}$. (2007 年中国东南数学奥林匹克)", + "solution": "因为 $\\frac{a^k}{a+b}+\\frac{1}{4}(a+b)+\\underbrace{\\frac{1}{2}+\\frac{1}{2}+\\cdots+\\frac{1}{2}}_{k-2 \\uparrow \\frac{1}{2}} \\geqslant k \\cdot \\sqrt[k]{\\frac{a^k}{2^k}}=\\frac{k}{2} a$,\n所以 $\\frac{a^k}{a+b} \\geqslant \\frac{k}{2} a-\\frac{1}{4}(a+b)-\\frac{k-2}{2}$.\n同理可得 $\\frac{b^k}{b+c} \\geqslant \\frac{k}{2} b-\\frac{1}{4}(b+c)-\\frac{k-2}{2}, \\frac{c^k}{c+a} \\geqslant \\frac{k}{2} c-\\frac{1}{4}(c+a)-\\frac{k-2}{2}$\n三式相加可得 $\\frac{a^k}{a+b}+\\frac{b^k}{b+c}+\\frac{c^k}{c+a} \\geqslant \\frac{k}{2}(a+b+c)-\\frac{1}{2}(a+b+c)-$\n$$\n\\frac{3}{2}(k-2)=\\frac{(k-1)}{2}(a+b+c)-\\frac{3}{2}(k-2) \\geqslant \\frac{3}{2}(k-1)-\\frac{3}{2}(k-2)=\\frac{3}{2} \\text {. }\n$$", + "remark": "", + "figures": [] +} \ No newline at end of file diff --git a/processed_dataset/proof/1397.json b/processed_dataset/proof/1397.json new file mode 100644 index 0000000000000000000000000000000000000000..d1023d69e4123bdaef73bc351d41847a39eabce8 --- /dev/null +++ b/processed_dataset/proof/1397.json @@ -0,0 +1,8 @@ +{ + "source_file": "./raw_volume-zh/volume5/exercise2.tex", + "problem_type": "proof", + "problem": "问题1. 设 $a, b, p, q>0$. 求证:\n$$\n\\frac{a^{p+q}+b^{p+q}}{2} \\geqslant\\left(\\frac{a^p+b^p}{2}\\right)\\left(\\frac{a^q+b^q}{2}\\right) .\n$$", + "solution": "原不等式等价于 $\\left(a^p-b^p\\right)\\left(a^q-b^q\\right) \\geqslant 0$.\n而 $a^p-b^p$ 与 $a^q-b^q$ 同号或同为零,故结论成立.", + "remark": "", + "figures": [] +} \ No newline at end of file diff --git a/processed_dataset/proof/1398.json b/processed_dataset/proof/1398.json new file mode 100644 index 0000000000000000000000000000000000000000..9705c3e92bd1a3987a2fc6608868a1f71674cfa1 --- /dev/null +++ b/processed_dataset/proof/1398.json @@ -0,0 +1,8 @@ +{ + "source_file": "./raw_volume-zh/volume5/exercise2.tex", + "problem_type": "proof", + "problem": "问题5. 设 $a_1, a_2, \\cdots, a_n$ 是不全相等的 $n$ 个正数 $(n \\geqslant 2)$, 且满足 $\\sum_{k=1}^n a_k^{-2 n}=1$, 求证:\n$$\n\\sum_{k=1}^n a_k^{2 n}-n^2 \\sum_{1 \\leqslant in^2\n$$", + "solution": "原不等式等价于 $\\sum_{k=1}^n a_k^{2 n} \\cdot \\sum_{k=1}^n \\frac{1}{a_k^{2 n}}-n^2 \\sum_{in^2$.\n由 Lagrange 恒等式, $\\sum_{k=1}^n a_k^{2 n} \\cdot \\sum_{k=1}^n \\frac{1}{a_k^{2 n}}-n^2=\\sum_{i0$, 有 $\\left(x^n-\\frac{1}{x^n}\\right)^2 \\geqslant n^2\\left(x-\\frac{1}{x}\\right)^2$, 因此 $\\sum_{k=1}^n a_k^{2 n} \\cdot \\sum_{k=1}^n \\frac{1}{a_k^{2 n}}- n^2 \\geqslant n^2 \\sum_{i\\left[\\sum_{i=1}^{n-1}(n-i) x_i\\right]\\left[\\sum_{j=2}^n(j-1) x_j\\right] .\n$$", + "solution": "令 $y_i==\\sum_{j=i+1}^n x_j, y=\\sum_{j=2}^n(j-1) x_j, z_i=\\frac{n(n-1)}{2} y_i-(n-i) y(1 \\leqslant i \\leqslant n-1)$. 于是\n$$\n\\begin{aligned}\n& \\frac{n(n-1)}{2} \\cdot \\sum_{1 \\leqslant i0$.\n注意到 $\\sum_{i=1}^{n-1} y_i=y, \\sum_{i=1}^{n-1} z_i=0, y=\\sum_{j=2}^n(j-1) x_j<\\sum_{j=2}^n(j-1) x_n= \\frac{n(n-1)}{2} x_n, z_{n-1}=\\frac{n(n-1)}{2} y_{n-1}-y=\\frac{n(n-1)}{2} x_n-y>0$.\n又由于 $\\frac{z_{i+1}}{\\frac{n(n-1)}{2}(n-i-1)}-\\frac{z_i}{\\frac{n(n-1)}{2}(n-i)}=\\frac{y_{i+1}}{n-i-1}-\\frac{y_i}{n-i}=$\n$$\n\\frac{x_{i+2}+x_{i+3}+\\cdots+x_n}{n-i-1}-\\frac{x_{i+1}+x_{i+2}+\\cdots+x_n}{n-i}>0,\n$$\n有: $\\frac{z_1}{n-1}<\\frac{z_2}{n-2}<\\cdots<\\frac{z_{n-2}}{2}0(k \\leqslant n-2)$. 所以 $\\left(x_i-x_k\\right) z_i \\geqslant 0$.\n故 $\\sum_{i=1}^{n-1} x_i z_i \\geqslant x_k \\sum_{i=1}^{n-1} z_i=0$, 但等号不能成立.\n证毕.", + "remark": "", + "figures": [] +} \ No newline at end of file diff --git a/processed_dataset/proof/1402.json b/processed_dataset/proof/1402.json new file mode 100644 index 0000000000000000000000000000000000000000..4960cc90410d3efa6d6e0467bdb16ff7275bd9b0 --- /dev/null +++ b/processed_dataset/proof/1402.json @@ -0,0 +1,8 @@ +{ + "source_file": "./raw_volume-zh/volume5/exercise2.tex", + "problem_type": "proof", + "problem": "问题9. 设 $a_1 \\geqslant a_2 \\geqslant \\cdots \\geqslant a_n \\geqslant a_{n+1}=0$ 是实数序列, 求证:\n$$\n\\sqrt{\\sum_{k=1}^n a_k} \\leqslant \\sum_{k=1}^n \\sqrt{k}\\left(\\sqrt{a_k}-\\sqrt{a_{k+1}}\\right) .\n$$", + "solution": "$\\left[\\sum_{k=1}^n(\\sqrt{k}-\\sqrt{k-1}) \\sqrt{a_k}\\right]^2=\\sum_{i=1}^n(\\sqrt{i}-\\sqrt{i-1})^2 a_i+2 \\sum_{im$ 时, $a_k=0$. 注:下面再给出两种证法.", + "remark": "", + "figures": [] +} \ No newline at end of file diff --git a/processed_dataset/proof/1403.json b/processed_dataset/proof/1403.json new file mode 100644 index 0000000000000000000000000000000000000000..b5b0a79530c6d56c4112fca27414860af4511369 --- /dev/null +++ b/processed_dataset/proof/1403.json @@ -0,0 +1,8 @@ +{ + "source_file": "./raw_volume-zh/volume5/exercise2.tex", + "problem_type": "proof", + "problem": "问题9. 设 $a_1 \\geqslant a_2 \\geqslant \\cdots \\geqslant a_n \\geqslant a_{n+1}=0$ 是实数序列, 求证:\n$$\n\\sqrt{\\sum_{k=1}^n a_k} \\leqslant \\sum_{k=1}^n \\sqrt{k}\\left(\\sqrt{a_k}-\\sqrt{a_{k+1}}\\right) .\n$$", + "solution": "证法 2:用 Abel 求和公式能把不等式转化为:\n$\\sqrt{\\sum_{k=1}^n a_k} \\leqslant \\sum_{k=1}^n \\sqrt{a_k}(\\sqrt{k}-\\sqrt{k-1})=\\sum_{k=1}^n \\sqrt{a_k c_k}$, 此处 $c_k=\\sqrt{k}-\\sqrt{k-1}$.\n由于 $a_1 \\geqslant a_2 \\geqslant \\cdots \\geqslant a_n, c_1 \\geqslant c_2 \\geqslant \\cdots \\geqslant c_n$, 诱使我们用 Chebyshev 不等式, 即\n$$\n\\sum_{k=1}^n \\sqrt{a_k} \\cdot c_k \\geqslant \\frac{1}{n} \\cdot \\sum_{k=1}^n \\sqrt{a_k} \\cdot \\sum_{k=1}^n c_k=\\frac{1}{\\sqrt{n}} \\sum_{k=1}^n \\sqrt{a_k},\n$$\n并无多大用处,现将欲证结论改述如下:\n$$\n\\sqrt{\\sum_{k=1}^n a_k} \\leqslant \\sum_{k=1}^{n-1} \\sqrt{k}\\left(\\sqrt{a_k}-\\sqrt{a_{k+1}}\\right)+\\sqrt{n a_n} . \\label{(1)}\n$$\n对 $n$ 用归纳法.\n当 $n=1$ 时, (1)显然成立.\n假设对某个 $n \\geqslant 1$, 每个非增、非负、长为 $n$ 的实数列有 (1) 成立, 考察长为 $n+1$, 满足 $a_1 \\geqslant a_2 \\geqslant \\cdots \\geqslant a_{n+1} \\geqslant 0$ 的实数列.\n由归纳假设, 只需证明: $\\sqrt{\\sum_{k=1}^{n+1} a_k}- \\sqrt{\\sum_{k=1}^n a_k} \\leqslant-\\sqrt{n a_{n+1}}+\\sqrt{(n+1) a_{n+1}}$. 不妨设 $a_{n+1}>0, S=\\sum_{k=1}^n a_k, m=\\frac{S}{a_{n+1}}$, 则上式等价于 $\\sqrt{m+1}-\\sqrt{m} \\leqslant \\sqrt{n+1}-\\sqrt{n}$, 显然成立.", + "remark": "", + "figures": [] +} \ No newline at end of file diff --git a/processed_dataset/proof/1404.json b/processed_dataset/proof/1404.json new file mode 100644 index 0000000000000000000000000000000000000000..e561e017b473e943ead124df80b0caa847b212eb --- /dev/null +++ b/processed_dataset/proof/1404.json @@ -0,0 +1,8 @@ +{ + "source_file": "./raw_volume-zh/volume5/exercise2.tex", + "problem_type": "proof", + "problem": "问题9. 设 $a_1 \\geqslant a_2 \\geqslant \\cdots \\geqslant a_n \\geqslant a_{n+1}=0$ 是实数序列, 求证:\n$$\n\\sqrt{\\sum_{k=1}^n a_k} \\leqslant \\sum_{k=1}^n \\sqrt{k}\\left(\\sqrt{a_k}-\\sqrt{a_{k+1}}\\right) .\n$$", + "solution": "证法 3: 令 $x_i=\\sqrt{a_i}-\\sqrt{a_{i+1}}, i=1,2, \\cdots, n$.\n则 $a_i=\\left(x_i+x_{i+1}+\\cdots+x_n\\right)^2, 1 \\leqslant i \\leqslant n$.\n故 $\\sum_{k=1}^n a_k=\\sum_{k=1}^n k x_k^2+2 \\sum_{1 \\leqslant k\\frac{1}{4} \\cdot\\left(1+\\frac{1}{2}+\\cdots+\\frac{1}{n}\\right) .\n\\end{aligned}\n$$", + "remark": "", + "figures": [] +} \ No newline at end of file diff --git a/processed_dataset/proof/1407.json b/processed_dataset/proof/1407.json new file mode 100644 index 0000000000000000000000000000000000000000..e7646a9790341f12feeb5a01a70708674524a10a --- /dev/null +++ b/processed_dataset/proof/1407.json @@ -0,0 +1,8 @@ +{ + "source_file": "./raw_volume-zh/volume5/exercise2.tex", + "problem_type": "proof", + "problem": "问题13. 试证: 对任意实数 $x$, 有 $\\sum_{k=1}^n \\frac{[k x]}{k} \\leqslant[n x]$, 其中 $[x]$ 表示不超过 $x$ 的最大整数.", + "solution": "令 $A_n=\\sum_{k=1}^n \\frac{[k x]}{k}$, 下用数学归纳法证明: $A_n \\leqslant[n x]$.\n当 $n=1$ 时, 显然成立.\n假设对 $1 \\leqslant k \\leqslant n-1$, 有 $A_k \\leqslant[k x]$, 则由分部求和公式,\n$$\n\\begin{aligned}\nn A_n & =\\sum_{k=1}^n k \\cdot \\frac{[k x]}{k}-\\sum_{k=1}^{n-1} A_k(k-(k+1)) \\\\\n& =\\sum_{k=1}^n[k x]+\\sum_{k=1}^{n-1} A_k \\leqslant \\sum_{k=1}^n[k x]+\\sum_{k=1}^{n-1}[k x] \\\\\n& =[n x]+\\sum_{k=1}^{n-1}([(n-k) x]+[k x]) \\\\\n& \\left.\\leqslant[n x]+\\sum_{k=1}^{n-1}[(n-k) x+k x] \\text { (因为 }[x]+[y] \\leqslant[x+y]\\right) \\\\\n& =n[n x],\n\\end{aligned}\n$$\n故 $A_n \\leqslant[n x]$, 命题得证.", + "remark": "", + "figures": [] +} \ No newline at end of file diff --git a/processed_dataset/proof/1408.json b/processed_dataset/proof/1408.json new file mode 100644 index 0000000000000000000000000000000000000000..6283a00a1b07b0e05b4b343756dc46cc458260b9 --- /dev/null +++ b/processed_dataset/proof/1408.json @@ -0,0 +1,8 @@ +{ + "source_file": "./raw_volume-zh/volume5/exercise2.tex", + "problem_type": "proof", + "problem": "问题14. 已知 $a_k \\geqslant 0, k=1,2, \\cdots, n$. 定义 $A_k=\\frac{1}{k} \\cdot \\sum_{i=1}^k a_i$, 求证:\n$$\n\\sum_{k=1}^n A_k^2 \\leqslant 4 \\sum_{k=1}^n a_k^2\n$$", + "solution": "如果设 $\\frac{1}{c} \\sum_{k=1}^n A_k^2 \\leqslant \\sum_{k=1}^n A_k \\cdot a_k$, 则有 $\\sum_{k=1}^n A_k \\cdot a_k \\leqslant c \\cdot \\sum_{k=1}^n a_k^2$, 于是可将问题转为 Abel 方法处理:\n$$\n\\begin{aligned}\n\\sum_{k=1}^n A_k a_k & =\\sum_{k=1}^n A_k\\left[k A_k-(k-1) A_{k-1}\\right] \\\\\n& =\\sum_{k=1}^n k A_k^2-\\sum_{k=1}^n(k-1) A_k A_{k-1} \\\\\n& \\geqslant \\sum_{k=1}^n k A_k^2-\\frac{1}{2}\\left[\\sum_{k=1}^n(k-1) A_k^2+\\sum_{k=1}^n(k-1) A_{k-1}^2\\right] \\\\\n& =\\frac{1}{2} \\cdot \\sum_{k=1}^n A_k^2+\\frac{1}{2} n A_n^2 \\geqslant \\frac{1}{2} \\sum_{k=1}^n A_k^2 .\n\\end{aligned}\n$$\n故\n$$\n\\sum_{k=1}^n A_k^2 \\leqslant 4 \\sum_{k=1}^n a_k^2\n$$", + "remark": "注:: 由此思想可以证明 Hölder 型不等式: 设 $p>1$, 则 $\\sum_{k=1}^n A_k^p \\leqslant\\left(\\frac{p}{p-1}\\right)^p$. $\\sum_{k=1}^n a_k^p$, 即类似的有: $\\frac{1}{c} \\sum_{k=1}^n A_k^p \\leqslant \\sum_{k=1}^n A_k^{p-1} \\cdot a_k \\leqslant c^{p-1} \\sum_{k=1}^n a_k^p$, 然后可证得 $c= \\frac{p}{p-1}$ 是可使不等式成立的待定系数.\n此题是哈代一道不等式的初等形式.", + "figures": [] +} \ No newline at end of file diff --git a/processed_dataset/proof/1409.json b/processed_dataset/proof/1409.json new file mode 100644 index 0000000000000000000000000000000000000000..63e59d4bd30dee916502499ac2616f0ff10a403b --- /dev/null +++ b/processed_dataset/proof/1409.json @@ -0,0 +1,8 @@ +{ + "source_file": "./raw_volume-zh/volume5/exercise3.tex", + "problem_type": "proof", + "problem": "问题3. 在 $\\triangle A B C$ 中, 求证:\n$$\n\\frac{c-a}{b+c-a}+\\frac{a-b}{c+a-b}+\\frac{b-c}{a+b-c} \\leqslant 0 .\n$$", + "solution": "设 $b+c-a=2 x, c+a-b=2 y, a+b-c=2 z$, 则 $x, y, z \\in \\mathbf{R}^{+}$, 且 $a=y+z, b=x+z, c=x+y$, 故原不等式等价于 $\\frac{x-z}{2 x}+\\frac{y-x}{2 y}+\\frac{z-y}{2 z} \\leqslant$ 0 , 即 $\\frac{z}{x}+\\frac{x}{y}+\\frac{y}{z} \\geqslant 3$, 显然成立.", + "remark": "", + "figures": [] +} \ No newline at end of file diff --git a/processed_dataset/proof/1410.json b/processed_dataset/proof/1410.json new file mode 100644 index 0000000000000000000000000000000000000000..f55d6172de459eb31cf4225664c641f88febdca1 --- /dev/null +++ b/processed_dataset/proof/1410.json @@ -0,0 +1,8 @@ +{ + "source_file": "./raw_volume-zh/volume5/exercise3.tex", + "problem_type": "proof", + "problem": "问题4. 设 $x, y, z \\in \\mathbf{R}^{+}$, 求证:\n$$\n\\frac{x+y+z}{3} \\cdot \\sqrt[3]{x y z} \\leqslant\\left(\\frac{x+y}{2} \\cdot \\frac{y+z}{2} \\cdot \\frac{z+x}{2}\\right)^{\\frac{2}{3}} .\n$$", + "solution": "设 $x+y=2 a, y+z=2 b, z+x=2 c$. 则 $a 、 b 、 c$ 可组成一个三角形, 设其面积为 $S$, 外接圆半径为 $R$, 则易见原不等式等价于 $a+b+c \\leqslant 3 \\sqrt{3} R$.\n由 $2 R(\\sin A+\\sin B+\\sin C)=a+b+c \\leqslant 2 R \\cdot 3 \\cdot \\sin \\frac{A+B+C}{3}=3 \\sqrt{3} R$, 因此原不等式成立.", + "remark": "", + "figures": [] +} \ No newline at end of file diff --git a/processed_dataset/proof/1411.json b/processed_dataset/proof/1411.json new file mode 100644 index 0000000000000000000000000000000000000000..82c6757e82bf0dfd5dc1c7de879282a8578a9f18 --- /dev/null +++ b/processed_dataset/proof/1411.json @@ -0,0 +1,8 @@ +{ + "source_file": "./raw_volume-zh/volume5/exercise3.tex", + "problem_type": "proof", + "problem": "问题5. 若 $x, y \\in \\mathbf{R}^{+}$, 求证:\n$$\n\\left(\\frac{2 x+y}{3} \\cdot \\frac{x+2 y}{3}\\right)^2 \\geqslant \\sqrt{x y} \\cdot\\left(\\frac{x+y}{2}\\right)^3 .\n$$", + "solution": "设 $x+y=a, x y=b$, 则 $a^2 \\geqslant 4 b>0$. 故\n$$\n\\begin{aligned}\n\\text { 左边 } & =\\left(\\frac{2 a^2+b}{9}\\right)^2=\\left(\\frac{\\frac{1}{4} a^2+\\frac{1}{4} a^2+\\cdots+\\frac{1}{4} a^2+b}{9}\\right)^2 \\\\\n& \\geqslant \\sqrt[9]{\\frac{1}{4^{16}} \\cdot a^{32} \\cdot b^2} \\geqslant \\sqrt[9]{\\frac{1}{2^{27}} \\cdot a^{27} \\cdot b^{\\frac{9}{2}}} \\\\\n& =\\frac{1}{8} a^3 \\cdot b^{\\frac{1}{2}}=\\text { 右边.\n}\n\\end{aligned}\n$$", + "remark": "", + "figures": [] +} \ No newline at end of file diff --git a/processed_dataset/proof/1412.json b/processed_dataset/proof/1412.json new file mode 100644 index 0000000000000000000000000000000000000000..eac1ba1ebabeffbd6e8fa12ab67992ee5ecd019a --- /dev/null +++ b/processed_dataset/proof/1412.json @@ -0,0 +1,8 @@ +{ + "source_file": "./raw_volume-zh/volume5/exercise3.tex", + "problem_type": "proof", + "problem": "问题6. 设实数 $a 、 b$ 满足 $a b>0$, 求证: $\\sqrt[3]{\\frac{a^2 b^2(a+b)^2}{4}} \\leqslant \\frac{a^2+10 a b+b^2}{12}$, 并确定等号成立的条件.\n般地,对任意实数 $a 、 b$, 求证:\n$$\n\\sqrt[3]{\\frac{a^2 b^2(a+b)^2}{4}} \\leqslant \\frac{a^2+a b+b^2}{3} .\n$$", + "solution": "(1) 设 $a b=x>0, a+b=y$, 则 $y^2 \\geqslant 4 x$. 因此不等式右端 $=\\frac{y^2+8 x}{12}= \\frac{y^2}{12}+\\frac{x}{3}+\\frac{x}{3} \\geqslant 3 \\cdot \\sqrt[3]{\\frac{x^2 y^2}{12 \\cdot 3^2}}=\\sqrt[3]{\\frac{x^2 y^2}{4}}=$ 左边, 故不等式成立, 且当 $a=b$ 时等号才成立.\n(2)当 $x \\geqslant 0$ 时, 由于 $\\frac{y^2+8 x}{12} \\leqslant \\frac{y^2-x}{3}$, 结论仍成立.\n此时等号成立, 仅当 $a=b$;\n当 $x<0$ 时, $-x>0$, 故不等式右端 $=\\frac{y^2-x}{3}=\\frac{y^2}{3}-\\frac{x}{6}-\\frac{x}{6} \\geqslant 3$. $\\sqrt[3]{\\frac{y^2}{3} \\cdot\\left(-\\frac{x}{6}\\right)^2}=\\sqrt[3]{\\frac{x^2 y^2}{4}}$, 所以不等式也成立, 此时等号当 $b=-2 a$ 或 $a= -2 b$ 时取到.", + "remark": "", + "figures": [] +} \ No newline at end of file diff --git a/processed_dataset/proof/1413.json b/processed_dataset/proof/1413.json new file mode 100644 index 0000000000000000000000000000000000000000..e4140e09f7a65f4bc7118d50844f8d28b77392f1 --- /dev/null +++ b/processed_dataset/proof/1413.json @@ -0,0 +1,8 @@ +{ + "source_file": "./raw_volume-zh/volume5/exercise3.tex", + "problem_type": "proof", + "problem": "问题7. 设 $a, b, c \\in \\mathbf{R}^{+}, a b c=1$, 求证:\n$$\n\\frac{1}{1+a+b}+\\frac{1}{1+b+c}+\\frac{1}{1+c+a} \\leqslant \\frac{1}{2+a}+\\frac{1}{2+b}+\\frac{1}{2+c} .\n$$", + "solution": "令 $x=a+b+c \\geqslant 3 \\sqrt[3]{a b c}=3, y=\\frac{1}{a}+\\frac{1}{b}+\\frac{1}{c}=a b+b c+c a \\geqslant 3 \\sqrt[3]{a^2 b^2 c^2}=3$, 则原不等式等价于\n$$\n\\begin{aligned}\n& \\frac{(1+b+c)(1+c+a)+(1+c+a)(1+a+b)+(1+a+b)}{(1+a+b)(1+b+c)(1+c+a)}(1+b+c) \\\\\n& \\qquad \\frac{(2+b)(2+c)+(2+c)(2+a)+(2+a)(2+b)}{(2+a)(2+b)(2+c)}, \\\\\n& \\frac{3+4 x+y+x^2}{2 x+x^2+y+x y} \\leqslant \\frac{12+4 x+y}{9+4 x+y} . \\\\\n& \\text { 即 } \\quad \\text { 上式等价于 }\\left(\\frac{5 x^2 y}{3}-5 x^2\\right)+\\left(\\frac{x y^2}{3}-y^2\\right)+\\left(\\frac{4}{3} x^2 y-12 x\\right)+(4 x y-12 x)+ \\\\\n& \\left(\\frac{1}{3} x y^2-3 y\\right)+\\left(\\frac{1}{3} x y^2-9\\right)+(2 x y-18) \\geqslant 0 .\n\\end{aligned}\n$$\n即\n$$\n\\frac{3+4 x+y+x^2}{2 x+x^2+y+x y} \\leqslant \\frac{12+4 x+y}{9+4 x+y} \\text {. }\n$$\n上式等价于 $\\left(\\frac{5 x^2 y}{3}-5 x^2\\right)+\\left(\\frac{x y^2}{3}-y^2\\right)+\\left(\\frac{4}{3} x^2 y-12 x\\right)+(4 x y-12 x)+ \\left(\\frac{1}{3} x y^2-3 y\\right)+\\left(\\frac{1}{3} x y^2-9\\right)+(2 x y-18) \\geqslant 0$.\n注意 $x y \\geqslant 9$, 故此式成立, 原不等式得证.", + "remark": "", + "figures": [] +} \ No newline at end of file diff --git a/processed_dataset/proof/1414.json b/processed_dataset/proof/1414.json new file mode 100644 index 0000000000000000000000000000000000000000..e9288134aae080244ffa7856f261da0b4ca863bf --- /dev/null +++ b/processed_dataset/proof/1414.json @@ -0,0 +1,8 @@ +{ + "source_file": "./raw_volume-zh/volume5/exercise3.tex", + "problem_type": "proof", + "problem": "问题8. 已知 $a 、 b 、 c 、 d 、 e$ 为正数,且 $a b c d e=1$, 求证:\n$$\n\\begin{gathered}\n\\frac{a+a b c}{1+a b+a b c d}+\\frac{b+b c d}{1+b c+b c d e}+\\frac{c+c d e}{1+c d+c d e a}+ \\\\\n\\frac{d+d e a}{1+d e+d e a b}+\\frac{e+e a b}{1+e a+e a b c} \\geqslant \\frac{10}{3} .\n\\end{gathered}\n$$", + "solution": "令 $a=\\frac{y}{x}, b=\\frac{z}{y}, c=\\frac{u}{z}, d=\\frac{v}{u}, e=\\frac{x}{v}, x, y, z, u, v \\in \\mathbf{R}^{+}$, 则原不等式等价于 $\\frac{u+y}{x+z+v}+\\frac{z+v}{x+y+u}+\\frac{x+u}{y+z+v}+\\frac{y+v}{x+z+u}+$\n$$\n\\frac{x+z}{y+u+v} \\geqslant \\frac{10}{3}\n$$\n两边同加 5 ,再乘以 3 ,上式等价于\n$$\n\\begin{aligned}\n& 3(x+y+z+u+v) \\cdot\\left(\\frac{1}{x+z+v}+\\frac{1}{x+y+u}+\\frac{1}{y+z+v}+\\frac{1}{x+z+u}+\\right. \\\\\n& \\left.\\frac{1}{y+u+v}\\right) \\geqslant 25 .\n\\end{aligned}\n$$\n利用 Cauchy 不等式, 上式是显然的.", + "remark": "", + "figures": [] +} \ No newline at end of file diff --git a/processed_dataset/proof/1415.json b/processed_dataset/proof/1415.json new file mode 100644 index 0000000000000000000000000000000000000000..2e62be609635c74bb06674523ffc033cdad71f26 --- /dev/null +++ b/processed_dataset/proof/1415.json @@ -0,0 +1,8 @@ +{ + "source_file": "./raw_volume-zh/volume5/exercise3.tex", + "problem_type": "proof", + "problem": "问题9. 设 $a 、 b 、 c$ 是正实数,求证:\n$$\na^2+b^2+c^2 \\geqslant \\frac{c\\left(a^2+b^2\\right)}{a+b}+\\frac{b\\left(c^2+a^2\\right)}{c+a}+\\frac{a\\left(b^2+c^2\\right)}{b+c} .\n$$", + "solution": "不妨设 $a \\geqslant b \\geqslant c$, 令 $\\frac{a}{c}=x, \\frac{b}{c}=y$, 则 $x \\geqslant y \\geqslant 1$.\n原不等式转化为 $x^2+y^2+1 \\geqslant \\frac{x^2+y^2}{x+y}+\\frac{y\\left(1+x^2\\right)}{1+x}+\\frac{x\\left(1+y^2\\right)}{1+y}$.\n去分母, 整理得 $\\left(x^4 y+x y^4\\right)+\\left(x^4+y^4+x+y\\right) \\geqslant\\left(x^3 y^2+x^2 y^3\\right)+\\left(x^3+\\right. \\left.y^3+x^2+y^2\\right)$, 即\n$$\nx y(x+y)(x-y)^2+x(x+1)(x-1)^2+y(y+1)(y-1)^2 \\geqslant 0 .\n$$\n故原不等式成立.", + "remark": "注:: 本题也可以直接证.\n证法如下:\n设 $a \\geqslant b \\geqslant c, a^2-\\frac{a\\left(b^2+c^2\\right)}{b+c}=\\frac{a^2 b+a^2 c-a b^2-a c^2}{b+c}=\\frac{a b(a-b)}{b+c}+ \\frac{a c(a-c)}{b+c}$, 由于 $\\frac{1}{b+c} \\geqslant \\frac{1}{a+c} \\geqslant \\frac{1}{a+b}$, 则左边 - 右边 $=\\left[\\frac{a b(a-b)}{b+c}-\\right. \\left.\\frac{a b(a-b)}{c+a}\\right]+\\left[\\frac{a c(a-c)}{b+c}-\\frac{a c(a-c)}{a+b}\\right]+\\left[\\frac{b c(b-c)}{c+a}-\\frac{b c(b-c)}{a+b}\\right] \\geqslant 0$, 故原不等式成立.", + "figures": [] +} \ No newline at end of file diff --git a/processed_dataset/proof/1416.json b/processed_dataset/proof/1416.json new file mode 100644 index 0000000000000000000000000000000000000000..fdf9fdbaf0069d3434da0b7326c90f7e121a7701 --- /dev/null +++ b/processed_dataset/proof/1416.json @@ -0,0 +1,8 @@ +{ + "source_file": "./raw_volume-zh/volume5/exercise3.tex", + "problem_type": "proof", + "problem": "问题11. 求证:在开区间 $(0,1)$ 内一定能找到四对两两不同的正数 $(a, b)(a \\neq b)$, 满足:\n$$\n\\sqrt{\\left(1-a^2\\right)\\left(1-b^2\\right)}>\\frac{a}{2 b}+\\frac{b}{2 a}-a b-\\frac{1}{8 a b} .\n$$", + "solution": "令 $a=\\cos \\alpha, b=\\cos \\beta, \\alpha, \\beta \\in\\left(0, \\frac{\\pi}{2}\\right)$, 则\n$$\na b+\\sqrt{\\left(1-a^2\\right)\\left(1-b^2\\right)}=\\cos (\\alpha-\\beta) .\n$$\n两边平方, 有 $\\sqrt{\\left(1-a^2\\right)\\left(1-b^2\\right)}=\\frac{1}{2 a b} \\cdot\\left[\\cos ^2(\\alpha-\\beta)-1\\right]+\\frac{a}{2 b}+\\frac{b}{2 a}-a b$.\n当 $0<|\\alpha-\\beta|<\\frac{\\pi}{6}$ 时, $\\cos (\\alpha-\\beta)>\\frac{\\sqrt{3}}{2}$, 则\n$$\n\\frac{1}{2 a b} \\cdot\\left[\\cos ^2(\\alpha-\\beta)-1\\right]>-\\frac{1}{8 a b},\n$$\n原不等式成立.\n显见, 在开区间 $\\left(0, \\frac{\\pi}{2}\\right)$ 内选择 4 对两两不同的角对 $\\left(\\alpha_i, \\beta_i\\right)$, 使得存在某两个角对 $(\\alpha, \\beta)$, 满足 $0<|\\alpha-\\beta|<\\frac{\\pi}{6}$ 是可以办到的, 因此结论成立.", + "remark": "", + "figures": [] +} \ No newline at end of file diff --git a/processed_dataset/proof/1417.json b/processed_dataset/proof/1417.json new file mode 100644 index 0000000000000000000000000000000000000000..e3502ae5257829f89979df14273a491ed7eb5026 --- /dev/null +++ b/processed_dataset/proof/1417.json @@ -0,0 +1,8 @@ +{ + "source_file": "./raw_volume-zh/volume5/exercise3.tex", + "problem_type": "proof", + "problem": "问题12. 设 $s$ 是所有满足下列条件的三角形集合:\n$$\n5\\left(\\frac{1}{A P}+\\frac{1}{B Q}+\\frac{1}{C R}\\right)-\\frac{3}{\\min \\{A P, B Q, C R\\}}=\\frac{6}{r},\n$$\n其中 $r$ 为 $\\triangle A B C$ 内切圆半径, $P 、 Q 、 R$ 分别是内切圆切边 $A B 、 B C 、 C A$ 的切点.\n求证: $s$ 中所有三角形都是等腰三角形并且均相似.", + "solution": "设 $a=\\max \\{a, b, c\\}$, 则 $A P=\\min \\{A P, B Q, C R\\}$, 由题意可得\n$$\n\\begin{aligned}\n& \\frac{4}{-a+b+c}+\\frac{10}{-b+a+c}+\\frac{10}{-c+a+b}=\\frac{6}{r} . \\\\\n& \\text { 令 }-a+b+c=2 x,-b+a+c=2 y,-c+a+b=2 z, x, y, z>0 .\n\\end{aligned}\n$$\n则上式等价于: $\\frac{2}{x}+\\frac{5}{y}+\\frac{5}{z}=6 \\cdot \\sqrt{\\frac{1}{x y}+\\frac{1}{y z}+\\frac{1}{z x}}$, 故\n$$\n\\begin{gathered}\n2\\left(\\frac{1}{x}-\\frac{4}{y}\\right)^2+2\\left(\\frac{1}{x}-\\frac{4}{z}\\right)^2=7\\left(\\frac{1}{y}-\\frac{1}{z}\\right)^2 . \\\\\n\\text { 令 } p=-\\frac{1}{x}-\\frac{4}{y}, q=\\frac{1}{x}-\\frac{4}{z} \\text {, 则 } 25 p^2+14 p q+25 q^2=0 .\n\\end{gathered}\n$$\n易证 $p=q=0$, 故 $y=z=4 x$, 于是易见结论成立.", + "remark": "", + "figures": [] +} \ No newline at end of file diff --git a/processed_dataset/proof/1418.json b/processed_dataset/proof/1418.json new file mode 100644 index 0000000000000000000000000000000000000000..da076d49fc1110f49a0715b01586fbab91dc0bbc --- /dev/null +++ b/processed_dataset/proof/1418.json @@ -0,0 +1,8 @@ +{ + "source_file": "./raw_volume-zh/volume5/exercise3.tex", + "problem_type": "proof", + "problem": "问题13. 设 $a, b, c \\in \\mathbf{R}^{+}$, 且满足 $a b c=1$, 求证:\n$$\n\\left(a-1+\\frac{1}{b}\\right)\\left(b-1+\\frac{1}{c}\\right)\\left(c-1+\\frac{1}{a}\\right) \\leqslant 1 .\n$$", + "solution": "令 $a=\\frac{x}{y}, b=\\frac{y}{z}, c=\\frac{z}{x}$, 且 $x, y, z \\in \\mathbf{R}^{+}$, 则原不等式等价于 $\\left(\\frac{x}{y}-1+\\frac{z}{y}\\right)\\left(\\frac{y}{z}-1+\\frac{x}{z}\\right)\\left(\\frac{z}{x}-1+\\frac{y}{x}\\right) \\leqslant 1$, 即 $(x-y+z)(y-z+x) (z-x+y) \\leqslant x y z . x-y+z, y-z+x, z-x+y$ 中任意 2 个之和 $>0$, 故至多只有 1 个 $\\leqslant 0$.\n(1) 若其中恰有 1 个 $\\leqslant 0$, 结论显然成立.\n(2)若每个都 $>0$, 由于 $(x-y+z)(y-z+x) \\leqslant x^2,(x-y+z)(z- x+y) \\leqslant z^2,(y-z+x)(z-x+y) \\leqslant y^2$, 相乘即得不等式成立.", + "remark": "", + "figures": [] +} \ No newline at end of file diff --git a/processed_dataset/proof/1419.json b/processed_dataset/proof/1419.json new file mode 100644 index 0000000000000000000000000000000000000000..abfed3357b3bd61437ca0f8a3288aff64abcb056 --- /dev/null +++ b/processed_dataset/proof/1419.json @@ -0,0 +1,8 @@ +{ + "source_file": "./raw_volume-zh/volume5/exercise3.tex", + "problem_type": "proof", + "problem": "问题14. 设 $a, b, c \\in \\mathbf{R}^{+}$, 求证:\n$$\n\\frac{a}{\\sqrt{a^2+8 b c}}+\\frac{b}{\\sqrt{b^2+8 a c}}+\\frac{c}{\\sqrt{c^2+8 a b}} \\geqslant 1 \\text {. }\n$$", + "solution": "记 $x=\\frac{a}{\\sqrt{a^2+8 b c}}, y=-\\frac{b}{\\sqrt{b^2+8 a c}}, z=\\frac{c}{\\sqrt{c^2+8 a b}}, x, y, z \\in \\mathbf{R}^{+}$, 则 $\\left(\\frac{1}{x^2}-1\\right)\\left(\\frac{1}{y^2}-1\\right)\\left(\\frac{1}{z^2}-1\\right)=512$.\n反设 $x+y+z<1$, 则 $0\\frac{\\left[(x+y+z)^2-x^2\\right] \\cdot\\left[(x+y+z)^2-y^2\\right] \\cdot\\left[(x+y+z)^2-z^2\\right]}{x^2 y^2 z^2} \\\\\n& =\\frac{(y+z+x+x)(y+z)(x+y+y+z)(x+z)(x+y+z+z)(x+y)}{x^2 y^2 z^2} \\\\\n& \\geqslant \\frac{4 \\sqrt[4]{x^2 y z} \\cdot 2 \\sqrt{y z} \\cdot 4 \\sqrt[4]{x y^2 z} \\cdot 2 \\sqrt{x z} \\cdot 4 \\sqrt[4]{x y z^2} \\cdot 2 \\sqrt{x y}}{x^2 y^2 z^2}=512,\n\\end{aligned}\n$$\n矛盾!\n因此 $x+y+z \\geqslant 1$.", + "remark": "注::也可以先证明: $\\frac{a}{\\sqrt{a^2+8 b c}} \\geqslant \\frac{a^{\\frac{1}{3}}}{a^{\\frac{1}{3}}+b^{\\frac{1}{3}}+c^{\\frac{4}{3}}}$ 等.\n进而易证得不等式成立.", + "figures": [] +} \ No newline at end of file diff --git a/processed_dataset/proof/1420.json b/processed_dataset/proof/1420.json new file mode 100644 index 0000000000000000000000000000000000000000..6e4f1b4d661c7408e2fa5017b3427feb4cbd4b08 --- /dev/null +++ b/processed_dataset/proof/1420.json @@ -0,0 +1,8 @@ +{ + "source_file": "./raw_volume-zh/volume5/exercise4.tex", + "problem_type": "proof", + "problem": "问题1. 已知 $a, b, c \\in \\mathbf{R}$, 且 $a+b+c>0, a b+b c+c a>0, a b c>0$. 求证:\n$$\na>0, b>0, c>0 \\text {. }\n$$", + "solution": "若不然, 由于 $a b c>0$, 不妨设 $a<0, b<0, c>0$. 由 $a+b+c>0$ 得 $c>|a+b|$, 而 $a b>c \\cdot|a+b|$, 则 $a b>|a+b|^2$,矛盾!", + "remark": "", + "figures": [] +} \ No newline at end of file diff --git a/processed_dataset/proof/1421.json b/processed_dataset/proof/1421.json new file mode 100644 index 0000000000000000000000000000000000000000..6016cd70b67850a705de67674800551753d00505 --- /dev/null +++ b/processed_dataset/proof/1421.json @@ -0,0 +1,8 @@ +{ + "source_file": "./raw_volume-zh/volume5/exercise4.tex", + "problem_type": "proof", + "problem": "问题2. 求证:下列不等式组无实数解\n$$\n\\left\\{\\begin{array}{l}\n|x|>|y-z+t|, \\\\\n|y|>|x-z+t|, \\\\\n|z|>|x-y+t|, \\\\\n|t|>|x-y+z| .\n\\end{array}\\right.\n$$", + "solution": "反设存在实数 $x 、 y 、 z 、 t$ 满足不等式组, 则两边平方后可得: $(x+y- z+t)(x-y+z-t)>0 ;(y+x-z+t)(y-x+z-t)>0 ;(z+x-y+$ t) $(z-x+y-t)>0 ;(t+x-y+z)(t-x+y-z)>0$. 从而有 $-(x+y- z+t)^2(x-y+z-t)^2(y-x+z-t)^2(z+x-y+t)^2>0$, 矛盾!", + "remark": "", + "figures": [] +} \ No newline at end of file diff --git a/processed_dataset/proof/1422.json b/processed_dataset/proof/1422.json new file mode 100644 index 0000000000000000000000000000000000000000..d990c57ea6c517880ec5863fc7156f9bc9efc602 --- /dev/null +++ b/processed_dataset/proof/1422.json @@ -0,0 +1,8 @@ +{ + "source_file": "./raw_volume-zh/volume5/exercise4.tex", + "problem_type": "proof", + "problem": "问题3. 设实数 $a 、 b 、 c 、 d 、 p 、 q$ 满足:\n$$\na b+c d=2 p q, a c \\geqslant p^2>0,\n$$\n求证: $b d \\leqslant q^2$.", + "solution": "如果 $b d>q^2$, 则 $4 a b c d=4(a c)(b d)>4 p^2 q^2=(a b+c d)^2=a^2 b^2+ 2 a b c d+c^2 d^2$, 故 $(a b-c d)^2<0$. 矛盾!", + "remark": "", + "figures": [] +} \ No newline at end of file diff --git a/processed_dataset/proof/1423.json b/processed_dataset/proof/1423.json new file mode 100644 index 0000000000000000000000000000000000000000..f3454b440b5d8605167ea11ccce3fc235ae8c6e7 --- /dev/null +++ b/processed_dataset/proof/1423.json @@ -0,0 +1,8 @@ +{ + "source_file": "./raw_volume-zh/volume5/exercise4.tex", + "problem_type": "proof", + "problem": "问题4. 设 $a 、 b 、 c$ 为正实数,且 $a+b+c \\geqslant a b c$, 求证:\n$$\na^2+b^2+c^2 \\geqslant a b c .\n$$", + "solution": "若 $a^2+b^2+c^2a_2>\\cdots>a_m$, 下面证明: 对任意满足 $1 \\leqslant i \\leqslant m$ 的正整数 $i$, 有\n$$\na_i+a_{n+1-i} \\geqslant n+1 . \\label{(1)}\n$$\n如果(1)成立, 则 $2\\left(a_1+a_2+\\cdots+a_m\\right)=\\left(a_1+a_m\\right)+\\left(a_2+a_{m-1}\\right)+\\cdots+ \\left(a_m+a_1\\right) \\geqslant m(n+1)$, 因此结论成立.\n对(1)可以用反证法, 若存在某个正整数 $i, 1 \\leqslant i \\leqslant m$, 使得 $a_i+a_{m+1-i} \\leqslant n$,于是 $a_i \\max _{1 \\leqslant i \\leqslant n}\\left|a_i\\right|$. 记 $A=\\max _{1 \\leqslant i \\leqslant n}\\left|a_i\\right|$.\n补充定义 $S_0=-S_n$, 则 $S_0$ 与 $S_n$ 中有一个 $>A$, 另一个 $<-A$. 不妨设 $S_0<-A, S_n>A$ (否则可用 $-a_i$ 代替 $a_i$ ), 于是, 存在 $j, 0 \\leqslant j \\leqslant n-1$, 使得 $S_j<-A, S_{j+1}>A$. 故 $\\left|S_{j+1}-S_j\\right|>2 A$, 即 $2\\left|a_{j+1}\\right|>2 A$, 则 $\\left|a_{j+1}\\right|> A$,与 $A$ 的定义矛盾!", + "remark": "", + "figures": [] +} \ No newline at end of file diff --git a/processed_dataset/proof/1428.json b/processed_dataset/proof/1428.json new file mode 100644 index 0000000000000000000000000000000000000000..a46cc64c68b62b7a727b09626fa569a378ceebb2 --- /dev/null +++ b/processed_dataset/proof/1428.json @@ -0,0 +1,8 @@ +{ + "source_file": "./raw_volume-zh/volume5/exercise4.tex", + "problem_type": "proof", + "problem": "问题9. 设 $x_k, y_k \\in \\mathbf{R}, j_k=x_k+\\mathrm{i} y_k(k=1,2, \\cdots, n, \\mathrm{i}=\\sqrt{-1}) . r$ 是 $\\pm \\sqrt{j_1^2+j_2^2+\\cdots+j_n^2}$ 的实部的绝对值.\n求证:\n$$\nr \\leqslant\\left|x_1\\right|+\\left|x_2\\right|+\\cdots+\\left|x_n\\right| .\n$$", + "solution": "设 $a+\\mathrm{i} b$ 是 $j_{\\mathrm{i}}^2+j_2^2+\\cdots+j_n^2$ 的任一平方根, 则 $r=|a|$, 且 $(a+\\mathrm{i} b)^2= \\sum_{k=1}^n j_k^2=\\sum_{k=1}^n\\left(x_k+\\mathrm{i} y_k\\right)^2$.\n故 $a^2-b^2=\\sum_{k=1}^n x_k^2-\\sum_{k=1}^n y_k^2, a b=\\sum_{k=1}^n x_k y_k$.\n反设 $r>\\sum_{k=1}^n\\left|x_k\\right|$, 则 $a^2=r^2,\\left(\\sum_{k=1}^n\\left|x_k\\right|\\right)^2 \\geqslant \\sum_{k=1}^n x_k^2$.\n于是 $b^2>\\sum_{k=1}^n y_k^2$,从而 $a^2 b^2>\\sum_{k=1}^n x_k^2 \\cdot \\sum_{k=1}^n y_k^2 \\geqslant\\left(\\sum_{k=1}^n x_k y_k\\right)^2=a^2 b^2$,矛盾!", + "remark": "", + "figures": [] +} \ No newline at end of file diff --git a/processed_dataset/proof/1429.json b/processed_dataset/proof/1429.json new file mode 100644 index 0000000000000000000000000000000000000000..41fba6aeb23dea5710c5aa0761dfe81028243003 --- /dev/null +++ b/processed_dataset/proof/1429.json @@ -0,0 +1,8 @@ +{ + "source_file": "./raw_volume-zh/volume5/exercise4.tex", + "problem_type": "proof", + "problem": "问题10. 给定非增的正数列 $a_1 \\geqslant a_2 \\geqslant a_3 \\geqslant \\cdots \\geqslant a_n \\geqslant \\cdots$, 其中 $a_1=\\frac{1}{2 k}(k \\in \\mathbf{N}$, $k \\geqslant 2)$, 且 $a_1+a_2+\\cdots+a_n+\\cdots=1$. 求证: 从数列中可找出 $k$ 个数, 最小数超过最大数的一半.", + "solution": "用反证法.\n若不存在这样的 $k$ 个数, 则对 $a_1, a_2, \\cdots, a_k$, 有 $a_k \\leqslant \\frac{1}{2} a_1$; 对 $a_k, a_{k+1}, \\cdots, a_{2 k-1}$, 有 $a_{2 k-1} \\leqslant \\frac{1}{2} a_k \\leqslant \\frac{1}{2^2} a_1 ; \\cdots$; 对 $a_{(n-1)(k-1)+1}$, $a_{(n-1)(k-1)+2}, \\cdots, a_{n(k-1)+1}$, 有 $a_{n(k-1)+1} \\leqslant \\frac{1}{2^n} a_1 . n \\in \\mathbf{Z}^{+}$, 则\n$$\n\\begin{gathered}\nS_1=a_1+a_k+a_{2 k-1}+\\cdots \\leqslant a_1+\\frac{1}{2} a_1+\\frac{1}{2^2} a_1+\\cdots=2 a_1 ; \\\\\nS_2=a_2+a_{k+1}+a_{2 k}+\\cdots \\leqslant 2 a_2 \\leqslant 2 a_1 ; \\\\\n\\cdots \\cdots \\\\\nS_{k-1}=a_{k-1}+a_{2 k-2}+a_{3 k-3}+\\cdots \\leqslant 2 a_{k-1} \\leqslant 2 a_1 .\n\\end{gathered}\n$$\n故 $S=S_1+S_2+\\cdots+S_{k-1} \\leqslant 2(k-1) a_1=\\frac{k-1}{k}<1$,矛盾!", + "remark": "", + "figures": [] +} \ No newline at end of file diff --git a/processed_dataset/proof/1430.json b/processed_dataset/proof/1430.json new file mode 100644 index 0000000000000000000000000000000000000000..6bb095b1f40e8cc14a8c22bf402a8c0a621dd07b --- /dev/null +++ b/processed_dataset/proof/1430.json @@ -0,0 +1,8 @@ +{ + "source_file": "./raw_volume-zh/volume5/exercise4.tex", + "problem_type": "proof", + "problem": "问题11. 证明或否定命题: 若 $x 、 y$ 为实数且 $y \\geqslant 0, y(y+1) \\leqslant(x+1)^2$, 则 $y(y-1) \\leqslant x^2$.", + "solution": "反设 $y(y-1)>x^2$, 则由 $y \\geqslant 0$ 知 $y>1$. 进一步有 $y>\\frac{1}{2}+ \\sqrt{\\frac{1}{4}+x^2}$. 由假设 $y(y+1) \\leqslant(x+1)^2$ 和 $y>1$ 可知, $y \\leqslant-\\frac{1}{2}+ \\sqrt{\\frac{1}{4}+(x+1)^2}$, 于是得到 $\\frac{1}{2}+\\sqrt{\\frac{1}{4}+x^2}<-\\frac{1}{2}+\\sqrt{\\frac{1}{4}+(x+1)^2}$.\n由此不难推出 $\\sqrt{\\frac{1}{4}+x^2}a_1$, 均有\n$$\n\\left(x-a_1\\right)\\left(x-a_2\\right) \\cdots\\left(x-a_n\\right) \\leqslant x^n-a_1^n .\n$$", + "solution": "对第一个结论用反证法.\n因为 $a_1 \\geqslant a_2 \\geqslant \\cdots \\geqslant a_n$, 则 $\\max \\left\\{\\left|a_1\\right|,\\left|a_2\\right|, \\cdots,\\left|a_n\\right|\\right\\}=a_1$ 或者 $\\left|a_n\\right|$ (显然 $a_1>0$ ). 而若 $\\max \\left\\{\\left|a_1\\right|,\\left|a_2\\right|, \\cdots,\\left|a_n\\right|\\right\\}=\\left|a_n\\right|$, 则 $a_n<0$, $\\left|a_n\\right|>a_1$.\n下面令 $a_1 \\geqslant a_2 \\geqslant \\cdots \\geqslant a_{n-k}>a_{n-k+1}=a_{n-k+2}=\\cdots=a_n$.\n由于 $0 \\leqslant\\left|\\frac{a_i}{a_n}\\right|<1$, 故存在 $l$, 使得 $\\left|\\frac{a_i}{a_n}\\right|^{2 l+1}<\\frac{1}{n}(1 \\leqslant i \\leqslant n-k)$, 于是\n$$\n\\begin{aligned}\n& \\left(\\frac{a_1}{a_n}\\right)^{2 l+1}+\\left(\\frac{a_2}{a_n}\\right)^{2 l+1}+\\cdots+\\left(\\frac{a_n}{a_n}\\right)^{2 l+1} \\\\\n= & \\left(\\frac{a_1}{a_n}\\right)^{2 l+1}+\\left(\\frac{a_2}{a_n}\\right)^{2 l+1}+\\cdots+\\left(\\frac{a_{n-k}}{a_n}\\right)^{2 l+1}+k \\\\\n\\geqslant & k-\\left|\\frac{a_1}{a_n}\\right|^{2 l+1}-\\left|\\frac{a_2}{a_n}\\right|^{2 l+1}-\\cdots-\\left|\\frac{a_{n-k}}{a_n}\\right|^{2 l+1} \\\\\n> & k-\\frac{n-k}{n}=k-1+\\frac{k}{n}>0 .\n\\end{aligned}\n$$\n从而 $a_1^{2 l+1}+a_2^{2 l+1}+\\cdots+a_n^{2 l+1}<0$,矛盾!\n下面来证明第二个结论.\n当 $x>a_1$ 时, $x-a_j>0,1 \\leqslant j \\leqslant n,\\left(x-a_1\\right)\\left(x-a_2\\right) \\cdots\\left(x-a_n\\right) \\leqslant(x-$\n$$\n\\left.a_1\\right) \\cdot\\left[\\frac{\\left(x-a_2\\right)+\\cdots+\\left(x-a_n\\right)}{n-1}\\right]^{n-1}=\\left(x-a_1\\right) \\cdot\\left(x-\\frac{a_2+a_3+\\cdots+a_n}{n-1}\\right)^{n-1} \\text {. }\n$$\n由于 $a_1+a_2+\\cdots+a_n \\geqslant 0$, 即 $a_1 \\geqslant-\\left(a_2+a_3+\\cdots+a_n\\right)$.\n故 $x+\\frac{1}{n-1} a_1 \\geqslant x-\\frac{a_2+a_3+\\cdots+a_n}{n-1}(>0)$, 则有\n$$\n\\begin{aligned}\n\\left(x-a_1\\right)\\left(x-a_2\\right) \\cdots\\left(x-a_n\\right) & \\leqslant\\left(x-a_1\\right)\\left(x+\\frac{a_1}{n-1}\\right)^{n-1} \\\\\n& =\\left(x-a_1\\right) \\cdot \\sum_{S=0}^{n-1} \\mathrm{C}_{n-1}^S\\left(\\frac{a_1}{n-1}\\right)^S \\cdot x^{n-1-S} \\\\\n& =\\left(x-a_1\\right) \\cdot \\sum_{S=0}^{n-1} \\frac{\\mathrm{C}_{n-1}^S}{(n-1)^S} a_1^S \\cdot x^{n-1-S} .\n\\end{aligned}\n$$\n易见, 当 $0 \\leqslant S \\leqslant n-1$ 时, $\\frac{\\mathrm{C}_{n-1}^S}{(n-1)^S} \\leqslant 1$, 于是\n$$\n\\begin{aligned}\n\\left(x-a_1\\right)\\left(x-a_2\\right) \\cdots\\left(x-a_n\\right) & \\leqslant\\left(x-a_1\\right) \\cdot \\sum_{S=0}^{n-1} a_1^S x^{n-1-S}\\left(x>a_1 \\geqslant 0\\right) \\\\\n& =\\left(x-a_1\\right) \\cdot \\frac{x^{n-1}-a_1^{n-1} \\cdot \\frac{a_1}{x}}{1-\\frac{a_1}{x}}=x^n-a_1^n .\n\\end{aligned}\n$$", + "remark": "", + "figures": [] +} \ No newline at end of file diff --git a/processed_dataset/proof/1432.json b/processed_dataset/proof/1432.json new file mode 100644 index 0000000000000000000000000000000000000000..d0fa9dd9710a31214039743976a083d796ed773b --- /dev/null +++ b/processed_dataset/proof/1432.json @@ -0,0 +1,8 @@ +{ + "source_file": "./raw_volume-zh/volume5/exercise4.tex", + "problem_type": "proof", + "problem": "问题13. 设实数 $a_1, a_2, \\cdots, a_n(n \\geqslant 2)$ 和 $A$ 满足: $A+\\sum_{i=1}^n a_i^2<\\frac{1}{n-1}\\left(\\sum_{i=1}^n a_i\\right)^2$. 求证: 对于 $1 \\leqslant ik)$ 在 $n$ 趋向于无穷大时也趋向于无穷大, 矛盾! 故有 $a_k-$\n$$\n\\begin{aligned}\n& a_{k+1} \\geqslant 0, k=1,2, \\cdots \\\\\n& \\text { 令 } b_k=a_k-a_{k+1} \\geqslant 0, k=1,2, \\cdots . \\\\\n& 1 \\geqslant a_1+a_2+\\cdots+a_k \\\\\n& =b_1+2 b_2+3 b_3+\\cdots+k b_k+a_{k+1} \\\\\n& \\geqslant(1+2+3+\\cdots+k) b_k=\\frac{k(k+1)}{2} b_k .\n\\end{aligned}\n$$\n所以 $b_k \\leqslant \\frac{2}{k(k+1)}<\\frac{2}{k^2}$, 因此 $0 \\leqslant a_k-a_{k+1}<\\frac{2}{k^2}$.", + "remark": "", + "figures": [] +} \ No newline at end of file diff --git a/processed_dataset/proof/1434.json b/processed_dataset/proof/1434.json new file mode 100644 index 0000000000000000000000000000000000000000..79f4b266be165debdf77dab66b59d38be99e1740 --- /dev/null +++ b/processed_dataset/proof/1434.json @@ -0,0 +1,8 @@ +{ + "source_file": "./raw_volume-zh/volume5/exercise4.tex", + "problem_type": "proof", + "problem": "问题15. 若方程 $x^4+a x^3+b x+c=0$ 的根都是实数, 求证: $a b \\leqslant 0$.", + "solution": "反设 $a b>0$, 不妨设 $a>0$, 则 $b>0$. 分三种情况讨论:\n(1) 若 $c>0, x^4+a x^3+b x+c=0$ 的根均为负根, 与 $x^2$ 前系数为 0 矛盾.\n(2) 若 $c<0$, 四个实根乘积为 $c<0$, 正根为 1 个或 3 个, 其余为负根, 再分别讨论:\n(i) 如果有 3 个正根 $x_2 、 x_3 、 x_4$, 负根为 $x_1$, 则 $x_1+x_2+x_3+x_4=-a<$ 0 , 故 $-x_1>-x_1-a=x_2+x_3+x_4$. 由于 $x^2$ 前系数为 0 , 应当有 $x_1\\left(x_2+\\right. \\left.x_3+x_4\\right)+x_2 x_3+x_2 x_4+x_3 x_4=0$.\n而 $x_2 x_3+x_2 x_4+x_3 x_4=-x_1\\left(x_2+x_3+x_4\\right)>\\left(x_2+x_3+x_4\\right)^2$, 矛盾!\n(ii) 如果仅有一个正根, 不妨设 $x_1$ 为正根, $x_2 、 x_3 、 x_4$ 为负根, $x_1 x_2 x_3 x_4\\left(\\frac{1}{x_1}+\\frac{1}{x_2}+\\frac{1}{x_3}+\\frac{1}{x_4}\\right)=-b<0$, 又由于 $x_1 x_2 x_3 x_4<0$, 则 $\\frac{1}{x_1}+\\frac{1}{x_2}+ \\frac{1}{x_3}+\\frac{1}{x_4}>0$. 由于 $-x_1\\left(x_2+x_3+x_4\\right)=x_2 x_3+x_3 x_4+x_2 x_4>0$, 两式相乘, 得到一 $\\left(x_2+x_3+x_4\\right)>\\left(-\\frac{1}{x_2}-\\frac{1}{x_3}-\\frac{1}{x_4}\\right)\\left(x_2 x_3+x_3 x_4+x_4 x_2\\right)=-2\\left(x_2+\\right. \\left.x_3+x_4\\right)-\\left(\\frac{x_2 x_3}{x_4}+\\frac{x_3 x_4}{x_2}+\\frac{x_4 x_2}{x_3}\\right)$,矛盾!\n(3) 若 $c=0, x^3+a x^2+b=0$ 有三个实根, 由于 $a>0, b>0$, 三个实数均为负根, 由于 $x$ 前面系数为 0 , 则根的两两乘积之和为 0 , 矛盾!\n综上所述, $a b \\leqslant 0$.", + "remark": "", + "figures": [] +} \ No newline at end of file diff --git a/processed_dataset/proof/1435.json b/processed_dataset/proof/1435.json new file mode 100644 index 0000000000000000000000000000000000000000..1a4d75221293308e9d88a11f2add673655a59129 --- /dev/null +++ b/processed_dataset/proof/1435.json @@ -0,0 +1,8 @@ +{ + "source_file": "./raw_volume-zh/volume5/exercise5.tex", + "problem_type": "proof", + "problem": "问题1. 已知 $k>a>b>c>0$, 求证:\n$$\nk^2-(a+b+c) k+(a b+b c+c a)>0 .\n$$", + "solution": "考察恒等式 $(k-a)(k-b)(k-c)=k^3-(a+b+c) k^2+(a b+b c+ c a) k-a b c$.", + "remark": "", + "figures": [] +} \ No newline at end of file diff --git a/processed_dataset/proof/1436.json b/processed_dataset/proof/1436.json new file mode 100644 index 0000000000000000000000000000000000000000..4bea2d0abfae829461f8f8952cb2b39438dc320e --- /dev/null +++ b/processed_dataset/proof/1436.json @@ -0,0 +1,8 @@ +{ + "source_file": "./raw_volume-zh/volume5/exercise5.tex", + "problem_type": "proof", + "problem": "问题2. 方程 $x^3+a x^2+b x+c=0$ 的三根 $\\alpha 、 \\beta 、 \\gamma$ 均为实数, 且 $a^2=2 b+2$, 求证: $|a-c| \\leqslant 2$.", + "solution": "考察恒等式 $(\\alpha+\\beta+\\gamma-\\alpha \\beta \\gamma)^2=2(\\alpha \\beta-1)(\\beta \\gamma-1)\\left(\\gamma_\\alpha-1\\right)+2- \\alpha^2 \\beta^2 \\gamma^2+\\alpha^2+\\beta^2+\\gamma^2$.", + "remark": "", + "figures": [] +} \ No newline at end of file diff --git a/processed_dataset/proof/1437.json b/processed_dataset/proof/1437.json new file mode 100644 index 0000000000000000000000000000000000000000..15b9c880548e4a2de86093dd0fb68168777b507a --- /dev/null +++ b/processed_dataset/proof/1437.json @@ -0,0 +1,8 @@ +{ + "source_file": "./raw_volume-zh/volume5/exercise5.tex", + "problem_type": "proof", + "problem": "问题4. 设 $a 、 b 、 c$ 都是实数,求证:\n$$\n\\frac{|a+b+c|}{1+|a+b+c|} \\leqslant \\frac{|a|}{1+|a|}+\\frac{|b|}{1+|b|}+\\frac{|c|}{1+|c|} .\n$$", + "solution": "构造函数 $f(x)=\\frac{x}{1+x}$, 易证 $f(x)$ 在 $[0,+\\infty)$ 上是增函数, 于是\n$f(|a+b+c|) \\leqslant f(|a|+|b|+|c|)$, 即\n$$\n\\begin{aligned}\n\\frac{|a+b+c|}{1+|a+b+c|} \\leqslant & \\frac{|a|+|b|+|c|}{1+|a|+|b|+|c|} \\\\\n= & \\frac{|a|}{1+|a|+|b|+|c|}+\\frac{|b|}{1+|a|+|b|+|c|} \\\\\n& +\\frac{|c|}{1+|a|+|b|+|c|} \\\\\n\\leqslant & \\frac{|a|}{1+|a|}+\\frac{|b|}{1+|b|}+\\frac{|c|}{1+|c|},\n\\end{aligned}\n$$\n故结论成立.", + "remark": "", + "figures": [] +} \ No newline at end of file diff --git a/processed_dataset/proof/1438.json b/processed_dataset/proof/1438.json new file mode 100644 index 0000000000000000000000000000000000000000..f84d24c09aa6ed84b837f61bf146a516f434e812 --- /dev/null +++ b/processed_dataset/proof/1438.json @@ -0,0 +1,8 @@ +{ + "source_file": "./raw_volume-zh/volume5/exercise5.tex", + "problem_type": "proof", + "problem": "问题5. 求所有的实数 $a$, 使得不等式 $x^2+y^2+z^2 \\leqslant a(x y+y z+z x)$ 的任何正整数解都是某个三角形的三边长.", + "solution": "取 $x=2, y=z=1$, 有 $a \\geqslant \\frac{6}{5}$. 因此, 当 $a \\geqslant \\frac{6}{5}$ 时, 原不等式有整根 $(2,1,1)$, 但 $(x, y, z)$ 不能组成一个三角形, 因此有 $a<\\frac{6}{5}$.\n又由于当 $a<1$ 时, $x^2+y^2+z^2 \\leqslant a(x y+y z+z x)0$ 时, $f(x)$ 单调递增.\n令 $g(x)=f(x)- x=\\left(x-r_1\\right)\\left(x-r_2\\right)$. 其中 $r_1=\\frac{5-\\sqrt{5}}{10}, r_2=\\frac{5+\\sqrt{5}}{10}, 0a^4=a_{n-5}^4$.\n其中利用了 $a \\leqslant f(a) \\leqslant f^{(2)}(a) \\leqslant \\cdots \\leqslant f^{(10)}(a)=a_{n+5}$.\n(2) $a \\in\\left[r_2, 1\\right]$, 也有 $a_{n+5}=f^{(10)}(a)>f^{(9)}(a)>\\cdots>a>a^4=a_{n-5}^4$.\n(3) $a \\in\\left(r_1, r_2\\right), r_1=f\\left(r_1\\right)r_1$, 故只须证 $r_1>r_2^4$ (容易验证):\n故 $a_{n+5}>r_1>r_2^4>a^4=a_{n-5}^4$.\n(4) $a>1$, 则 $a_{n+5}=f^{(10)}(a)>f^{(9)}(a)>\\cdots>f^{(2)}(a)=\\left(a^2+\\frac{1}{5}\\right)^2+ \\frac{1}{5}>a^4=a_{n-5}^2$, 因此结论也成立.", + "remark": "", + "figures": [] +} \ No newline at end of file diff --git a/processed_dataset/proof/1441.json b/processed_dataset/proof/1441.json new file mode 100644 index 0000000000000000000000000000000000000000..ada73c7808b7d9077c936642deaba5c4ec36834a --- /dev/null +++ b/processed_dataset/proof/1441.json @@ -0,0 +1,8 @@ +{ + "source_file": "./raw_volume-zh/volume5/exercise5.tex", + "problem_type": "proof", + "problem": "问题7. 设 $a_n \\in \\mathbf{R}$ 满足: $a_{n+1} \\geqslant a_n^2+\\frac{1}{5}, n \\geqslant 0$, 求证:\n$$\n\\sqrt{a_{n+5}} \\geqslant a_{n-5}^2, n \\geqslant 5 .\n$$", + "solution": "证法 2 : 对于任何非负整数 $k$, 因为 $\\left(a_k-\\frac{1}{2}\\right)^2 \\geqslant 0$, 故有\n$$\na_{k+1}+\\frac{1}{20} \\geqslant a_k^2+\\frac{1}{5}+\\frac{1}{20}=a_k^2+\\frac{1}{4} \\geqslant a_k .\n$$\n因此 $a_{k+1}+\\frac{1}{20} \\geqslant a_k$.\n对 $k=n+1, n+2, n+3, n+4$, 求和得 $a_{n+5}+\\frac{1}{5} \\geqslant a_{n+1} \\geqslant a_n^2+\\frac{1}{5}$.\n类似地, 我们有 $a_n \\geqslant a_{n-5}^2$. 于是 $a_{n+5} \\geqslant a_n^2 \\geqslant a_{n-5}^4$, 结论成立.", + "remark": "", + "figures": [] +} \ No newline at end of file diff --git a/processed_dataset/proof/1442.json b/processed_dataset/proof/1442.json new file mode 100644 index 0000000000000000000000000000000000000000..673239f8c1c5667f040aa6b96640741ea47a237c --- /dev/null +++ b/processed_dataset/proof/1442.json @@ -0,0 +1,8 @@ +{ + "source_file": "./raw_volume-zh/volume5/exercise5.tex", + "problem_type": "proof", + "problem": "问题7. 设 $a_n \\in \\mathbf{R}$ 满足: $a_{n+1} \\geqslant a_n^2+\\frac{1}{5}, n \\geqslant 0$, 求证:\n$$\n\\sqrt{a_{n+5}} \\geqslant a_{n-5}^2, n \\geqslant 5 .\n$$", + "solution": "证法 3 : 只要证明 $a_{n+5} \\geqslant a_n^2(n \\geqslant 5)$.\n因为 $a_{n+5} \\geqslant a_{n+4}^2+\\frac{1}{5} ; \\cdots ; a_{n+1} \\geqslant a_n^2+\\frac{1}{5}$, 相加, 得\n$$\n\\begin{aligned}\na_{n+5} & \\geqslant \\sum_{i=1}^4\\left(a_{n+i}^2-a_{n+i}\\right)+1+a_n^2 \\\\\n& =\\sum_{i=1}^4\\left(a_{n+i}-\\frac{1}{2}\\right)^2+a_n^2 \\geqslant a_n^2 .\n\\end{aligned}\n$$\n因此结论成立.", + "remark": "", + "figures": [] +} \ No newline at end of file diff --git a/processed_dataset/proof/1443.json b/processed_dataset/proof/1443.json new file mode 100644 index 0000000000000000000000000000000000000000..15d3ad635de22ef5929f0edc4058ebded0b69150 --- /dev/null +++ b/processed_dataset/proof/1443.json @@ -0,0 +1,8 @@ +{ + "source_file": "./raw_volume-zh/volume5/exercise5.tex", + "problem_type": "proof", + "problem": "问题8. 设 $x 、 y 、 z$ 为任意实数,求证:\n$$\n\\sqrt{x^2+x y+y^2}+\\sqrt{x^2+x z+z^2} \\geqslant \\sqrt{y^2+y z+z^2} .\n$$", + "solution": "建立平面直角坐标系 $x O y$, 取三点 $A(x, 0) 、 B\\left(-\\frac{y}{2},-\\frac{\\sqrt{3}}{2} y\\right) 、 C \\left(-\\frac{z}{2}, \\frac{\\sqrt{3}}{2} z\\right)$, 则原不等式转化为: $|A B|+|A C| \\geqslant|B C|$, 这是显然的.", + "remark": "", + "figures": [] +} \ No newline at end of file diff --git a/processed_dataset/proof/1444.json b/processed_dataset/proof/1444.json new file mode 100644 index 0000000000000000000000000000000000000000..aa86f64a76f24d77cb28fb24db60e2c79261fa3f --- /dev/null +++ b/processed_dataset/proof/1444.json @@ -0,0 +1,10 @@ +{ + "source_file": "./raw_volume-zh/volume5/exercise5.tex", + "problem_type": "proof", + "problem": "问题9. 设 $x, y, z>0$, 求证:\n$$\n\\begin{aligned}\n& 3 \\sqrt{x y+y z+z x} \\\\\n\\leqslant & \\sqrt{x^2+x y+y^2}+\\sqrt{y^2+y z+z^2}+\\sqrt{z^2+z x+x^2} \\\\\n\\leqslant & 2(x+y+z) .\n\\end{aligned}\n$$", + "solution": "由条件, 可构造如图(), 在 $\\triangle A B C$ (内有一点 $P$, 满足条件 $\\angle A P B=\\angle B P C=\\angle C P A=120^{\\circ}$. 设 $P A=x, P B=y, P C=z$, 则 $A B=\\sqrt{x^2+x y+y^2}$, $B C=\\sqrt{y^2+y z+z^2}, C A=\\sqrt{z^2+z x+x^2}$. \n由三角形两边之和大于第三边, 有 $\\sqrt{x^2+x y+y^2}+ \\sqrt{y^2+y z+z^2}+\\sqrt{z^2+z x+x^2} \\leqslant 2(x+y+z)$).\n又由 $a^2+b^2+c^2 \\geqslant 4 \\sqrt{3} s, a b+b c+c a \\geqslant 4 \\sqrt{3} s$,\n得 $(a+b+c)^2 \\geqslant 12 \\sqrt{3} s$, 故 $a+b+c \\geqslant 2 \\sqrt{3 \\sqrt{3} s}=3 \\sqrt{x y+y z+z x}$.", + "remark": "", + "figures": [ + "./images/volume5/figures/fig-c5p9.png" + ] +} \ No newline at end of file diff --git a/processed_dataset/proof/1445.json b/processed_dataset/proof/1445.json new file mode 100644 index 0000000000000000000000000000000000000000..93a363cf0dabdcfd238040ff70cc4c89bb3cbeee --- /dev/null +++ b/processed_dataset/proof/1445.json @@ -0,0 +1,10 @@ +{ + "source_file": "./raw_volume-zh/volume5/exercise5.tex", + "problem_type": "proof", + "problem": "问题10. 已知 $\\alpha 、 \\beta 、 \\gamma$ 都是锐角, 且 $\\cos ^2 \\alpha+\\cos ^2 \\beta+\\cos ^2 \\gamma=1$, 求证:\n$$\n\\frac{3 \\pi}{4}<\\alpha+\\beta+\\gamma<\\pi \\text {. }\n$$", + "solution": "由条件, 作一个长、宽、高分别为 $\\cos \\alpha$ 、 $\\cos \\beta 、 \\cos \\gamma$ 的长方体 $A B C D-A_1 B_1 C_1 D_1$. 如图()所示, $A B=\\cos \\alpha, B C=\\cos \\beta, B B_1=\\cos \\gamma$. 则此长方体对角线长恰为 1 . 同时, 易见 $\\angle A B D_1=\\alpha$, $\\angle C B B_1=\\beta, \\angle B_1 B D_1=\\gamma$.\n在三面角 $B-A D_1 C$ 中, 有 $\\angle A B D_1+\\angle D_1 B C> \\angle A B C=\\frac{\\pi}{2}$, 故 $\\alpha+\\beta>\\frac{\\pi}{2}$, 同理, $\\beta+\\gamma>\\frac{\\pi}{2}, \\gamma+ \\alpha>\\frac{\\pi}{2}$, 则 $\\alpha+\\beta+\\gamma>\\frac{3 \\pi}{4}$.\n取 $B D_1$ 的中点 $O$, 则 $\\angle A O D_1=2 \\alpha, \\angle C O D_1=2 \\beta, \\angle B B_1 D_1=2 \\gamma$. 易证 $\\angle C O B_1=\\angle A O D_1=2 \\alpha$, 考虑三面角 $O-C B_1 D_1$, 有 $\\angle C O B_1+\\angle B_1 O D_1+ \\angle C O D_1<2 \\pi$, 于是即得 $\\alpha+\\beta+\\gamma<\\pi$.", + "remark": "", + "figures": [ + "./images/volume5/figures/fig-c5p10.png" + ] +} \ No newline at end of file diff --git a/processed_dataset/proof/1446.json b/processed_dataset/proof/1446.json new file mode 100644 index 0000000000000000000000000000000000000000..3514574a68185fbecdfcfe381af538891e4b4afd --- /dev/null +++ b/processed_dataset/proof/1446.json @@ -0,0 +1,8 @@ +{ + "source_file": "./raw_volume-zh/volume5/exercise5.tex", + "problem_type": "proof", + "problem": "问题11. 若 $p 、 q$ 为实数, 且对于 $0 \\leqslant x \\leqslant 1$, 成立不等式:\n$$\n\\left|\\sqrt{1-x^2}-p x-q\\right| \\leqslant \\frac{\\sqrt{2}-1}{2} \\text {. }\n$$\n求证: $p=-1, q=\\frac{1+\\sqrt{2}}{2}$.", + "solution": "原不等式即 $\\sqrt{1-x^2}-\\frac{\\sqrt{2}-1}{2} \\leqslant p x+q \\leqslant \\sqrt{1-x^2}+\\frac{\\sqrt{2}-1}{2}(0 \\leqslant x \\leqslant 1)$.\n分别以点 $A\\left(0, \\frac{\\sqrt{2}-1}{2}\\right)$ 及 $B\\left(0,-\\frac{\\sqrt{2}-1}{2}\\right)$ 为圆心, 半径为 1 作圆 $A$ 和圆 $B$, 则它们的两个端点分别为 $\\left(0, \\frac{\\sqrt{2}+1}{2}\\right) 、\\left(1, \\frac{\\sqrt{2}-1}{2}\\right)$ 及 $\\left(0, \\frac{3-\\sqrt{2}}{2}\\right)$ 、 $\\left(1,-\\frac{\\sqrt{2}-1}{2}\\right)$\n记圆 $A$ 在第一象限内的弧为 $l_1$, 圆 $B$ 在第一、四象限内的弧为 $l_2$, 于是, 当 $0 \\leqslant x \\leqslant 1$ 时,直线 $y=p x+q$ 位于 $l_1$ 和 $l_2$ 之间.\n由于连接 $l_1$ 两个端点的直线恰与圆 $B$ 相切, 从而与 $l_2$ 相切.\n因此, 直线 $y=p x+q$ 必过 $l_1$ 两个端点, 故 $p=-1, q=\\frac{\\sqrt{2}+1}{2}$.", + "remark": "", + "figures": [] +} \ No newline at end of file diff --git a/processed_dataset/proof/1447.json b/processed_dataset/proof/1447.json new file mode 100644 index 0000000000000000000000000000000000000000..37573f4c3ff6956b9c43aa566f90ea8ddf394dc4 --- /dev/null +++ b/processed_dataset/proof/1447.json @@ -0,0 +1,10 @@ +{ + "source_file": "./raw_volume-zh/volume5/exercise5.tex", + "problem_type": "proof", + "problem": "问题12. (Minkovski 不等式) 求证: 对于任意 $2 n$ 个正数 $a_1, a_2, \\cdots, a_n$ 及 $b_1, b_2, \\cdots, b_n$, 有\n$$\n\\begin{aligned}\n& \\sqrt{a_1^2+b_1^2}+\\sqrt{a_2^2+b_2^2}+\\cdots+\\sqrt{a_n^2+b_n^2} \\\\\n\\geqslant & \\sqrt{\\left(a_1+a_2+\\cdots+a_n\\right)^2+\\left(b_1+b_2+\\cdots+b_n\\right)^2},\n\\end{aligned}\n$$\n等号当且仅当 $\\frac{b_1}{a_1}=\\frac{b_2}{a_2}=\\cdots=\\frac{b_n}{a_n}$ 时成立.", + "solution": "把形如 $\\sqrt{x^2+y^2}$ 的项看作是一个直角三角形斜边的长, 可构造图形如图()所示.\n于是, 不等式左边 $=O A_1+A_1 A_2+\\cdots+A_{n-1} A_n$, 不等式右边 $=O A_n$.\n由于折线 $O A_1 \\cdots A_n$ 的长不小于直线段 $O A_n$ 的长, 故 $O A_1+A_1 A_2+\\cdots+ A_{n-1} A_n \\geqslant O A_n$, 因此不等式成立.", + "remark": "", + "figures": [ + "./images/volume5/figures/fig-c5p12.png" + ] +} \ No newline at end of file diff --git a/processed_dataset/proof/1448.json b/processed_dataset/proof/1448.json new file mode 100644 index 0000000000000000000000000000000000000000..23784cb02546ceed1acbb1a4f0fd1cc5be027407 --- /dev/null +++ b/processed_dataset/proof/1448.json @@ -0,0 +1,10 @@ +{ + "source_file": "./raw_volume-zh/volume5/exercise5.tex", + "problem_type": "proof", + "problem": "问题13. 设 $0)构造正方形 $A B C D$, 边长为 $a$.\n取 $U, V$ 使 $S D=a_1, C R=a_2, B Q=a_3, A P=a_4$.\n则 $S D R M$ 与 $C R V Q$ 互不重叠, $B P N Q$ 与 $A P U S$ 互不重叠, 因此它们至多将正方形 $A B C D$ 覆盖两次,故原不等式获证.\n(2) 和 (1) 类似, 6 个矩形至多将正方形 ( $a \\times a$ 的) 覆盖 3 次.", + "remark": "", + "figures": [ + "./images/volume5/figures/fig-c5p13.png" + ] +} \ No newline at end of file diff --git a/processed_dataset/proof/1449.json b/processed_dataset/proof/1449.json new file mode 100644 index 0000000000000000000000000000000000000000..e1206c80d315e3273e061850de7dbcfdd8912a68 --- /dev/null +++ b/processed_dataset/proof/1449.json @@ -0,0 +1,10 @@ +{ + "source_file": "./raw_volume-zh/volume5/exercise5.tex", + "problem_type": "proof", + "problem": "问题14. 已知 100 个正数 $x_1, x_2, \\cdots, x_{100}$, 满足:\n(1) $x_1^2+x_2^2+\\cdots+x_{100}^2>10000$;\n(2) $x_1+x_2+\\cdots+x_{100} \\leqslant 300$.\n求证: 可在它们之中找出 3 个数,使得这三个数之和大于 100 .", + "solution": "不妨设 $x_1 \\geqslant x_2 \\geqslant \\cdots \\geqslant x_{100}$, 下面我们借助图形()来说明, $x_1+x_2+x_3>100$ 成立.\n把 $x_i^2$ 看作是边长为 $x_i$ 的正方形的面积 $(1 \\leqslant i \\leqslant 100)$.\n由于 $x_1+x_2+\\cdots+x_{100} \\leqslant 300$, 这些正方形可以一个接一个地排起来,其总长 $\\leqslant 300$, 它们全落在一个长为 300 , 宽为 100 的矩形中.\n这个矩形可分为 3 个边长为 100 的正方形, 如图所示.\n如果 $x_1+x_2+x_3 \\leqslant 100$, 第一个边长为 100 的正方形中含有三个带形,互不重叠, 宽分别为 $x_1 、 x_2 、 x_3$.\n由于 $x_i$ 递减,第 2 个边长为 100 的正方形中所含的小正方形(包括不完整的, 如图中阴影部分所示) 的边长 $x_i$ 均 $\\leqslant x_3$, 它们可以移至上述宽为 $x_2$ 的带形中.\n同样,第 3 个边长为 100 的正方形中所含的小正方形可以移至上述宽为 $x_3$ 的带形中, 于是, 面积之和 $x_1^2+x_2^2+\\cdots+x_{100}^2<100^2$, 矛盾!\n故 $x_1+x_2+x_3>100$.", + "remark": "注:: 下面给出第 2 种证法.\n证法 2 : 不妨设 $x_1 \\geqslant x_2 \\geqslant \\cdots \\geqslant x_{100}$. 记 $s=x_1+x_2+x_3, \\lambda=x_3$, 令 $\\delta= x_2-x_3 \\geqslant 0$, 由 $x_1 \\geqslant x_2$, 易知 $\\left(x_1+\\delta\\right)^2+\\left(x_2-\\delta\\right)^2 \\geqslant x_1^2+x_2^2$. 所以 $x_1^2+x_2^2+ x_3^2 \\leqslant(s-2 \\lambda)^2+2 \\lambda^2$.\n由 $\\lambda \\geqslant x_4 \\geqslant x_5 \\geqslant \\cdots \\geqslant x_{100}$ 和 $x_4+x_5+\\cdots+x_{100}<300-s$ 可得\n$$\nx_4^2+x_5^2+\\cdots+x_{100}^2 \\leqslant \\lambda\\left(x_4+x_5+\\cdots+x_{100}\\right) \\leqslant(300-s) \\lambda .\n$$\n故 $x_1^2+x_2^2+\\cdots+x_{100}^2 \\leqslant 6 \\lambda^2+(300-5 s) \\lambda+s^2$.\n记 $f(\\lambda)=6 \\lambda^2+(300-5 s) \\lambda+s^2$. 则由于 $0<\\lambda \\leqslant \\frac{s}{3}$, 又由于 $f(0)=s^2$, $f\\left(\\frac{s}{3}\\right)=100 s$, 有 $x_1^2+x_2^2+\\cdots+x_{100}^2 \\leqslant \\max \\left\\{s^2, 100 s\\right\\}$. 再由 $x_1^2+x_2^2+\\cdots+ x_{100}^2>10000$, 即可得 $s=x_1+x_2+x_3>100$.", + "figures": [ + "./images/volume5/figures/fig-c5p14.png" + ] +} \ No newline at end of file diff --git a/processed_dataset/proof/1450.json b/processed_dataset/proof/1450.json new file mode 100644 index 0000000000000000000000000000000000000000..c4ab11ffd980646e8acc46679542fbf1b7ba657f --- /dev/null +++ b/processed_dataset/proof/1450.json @@ -0,0 +1,8 @@ +{ + "source_file": "./raw_volume-zh/volume5/exercise5.tex", + "problem_type": "proof", + "problem": "问题15. 设 $n$ 为一个大于 2 的奇数,求证: 当且仅当 $n=3$ 或 5 时,对于任意 $a_1$, $a_2, \\cdots, a_n \\in \\mathbf{R}$, 有下面不等式成立:\n$$\n\\begin{gathered}\n\\left(a_1-a_2\\right)\\left(a_1-a_3\\right) \\cdots\\left(a_1-a_n\\right)+\\left(a_2-a_1\\right)\\left(a_2-a_3\\right) \\cdots\\left(a_2-a_n\\right) \\\\\n+\\cdots+\\left(a_n-a_1\\right)\\left(a_n-a_2\\right) \\cdots\\left(a_n-a_{n-1}\\right) \\geqslant 0 .\n\\end{gathered}\n$$", + "solution": "不妨设 $a_1 \\leqslant a_2 \\leqslant \\cdots \\leqslant a_n$.\n则 $A_3=a_1^2+a_2^2+a_3^2-a_1 a_2-a_2 a_3-a_3 a_1 \\geqslant 0$.\n$$\n\\begin{aligned}\n& \\quad A_5=\\left(a_1-a_2\\right)\\left(a_1-a_3\\right)\\left(a_1-a_4\\right)\\left(a_1-a_5\\right)+\\left(a_2-a_1\\right)\\left(a_2-a_3\\right)\\left(a_2-\\right. \\\\\n& \\left.a_4\\right)\\left(a_2-a_5\\right)+\\left(a_3-a_1\\right)\\left(a_3-a_2\\right)\\left(a_3-a_4\\right)\\left(a_3-a_5\\right)+\\left(a_4-a_1\\right)\\left(a_4-a_2\\right) \\\\\n& \\left(a_4-a_3\\right)\\left(a_4-a_5\\right)+\\left(a_5-a_1\\right)\\left(a_5-a_2\\right)\\left(a_5-a_3\\right)\\left(a_5-a_4\\right) .\n\\end{aligned}\n$$\n由于 $\\left(a_3-a_1\\right)\\left(a_3-a_2\\right)\\left(a_3-a_4\\right)\\left(a_3-a_5\\right) \\geqslant 0$, 而 $A_5$ 前 2 项之和为 $\\left(a_2-a_1\\right)\\left[\\left(a_3-a_1\\right)\\left(a_4-a_1\\right)\\left(a_5-a_1\\right)-\\left(a_3-a_2\\right)\\left(a_4-a_2\\right)\\left(a_5-a_2\\right)\\right] \\geqslant 0$, 同理, $A_5$ 后 2 项之和也 $\\geqslant 0$, 故 $A_5 \\geqslant 0$.\n当 $n \\geqslant 7$ 时,构造反例如下: 令 $a_1=a_2=a_31+a_1+a_2+\\cdots+a_n .\n$$", + "solution": "构造数列 $x_n=\\left(1+a_1\\right)\\left(1+a_2\\right) \\cdots\\left(1+a_n\\right)-\\left(1+a_1+a_2+\\cdots+a_n\\right) (n \\geqslant 2)$, 则 $x_{n+1}-x_n=a_{n+1}\\left[\\left(1+a_1\\right) \\cdots\\left(1+a_n\\right)-1\\right]$.\n若 $a_i>0(i=1,2, \\cdots, n+1)$, 由上式易见 $x_{n+1}>x_n$; 若 $-1x_n$.\n因此 $\\left\\{x_n\\right\\}$ 是一个单调递增序列 $(n \\geqslant 2)$. 由于 $x_2=\\left(1+a_1\\right)\\left(1+a_2\\right)-1- a_1-a_2=a_1 a_2>0$, 则对一切 $n \\geqslant 2, x_n>0$, 从而原不等式成立.", + "remark": "", + "figures": [] +} \ No newline at end of file diff --git a/processed_dataset/proof/1452.json b/processed_dataset/proof/1452.json new file mode 100644 index 0000000000000000000000000000000000000000..ab153c4ffe6b6b796f30d96e83829973d1bbe755 --- /dev/null +++ b/processed_dataset/proof/1452.json @@ -0,0 +1,8 @@ +{ + "source_file": "./raw_volume-zh/volume5/exercise5.tex", + "problem_type": "proof", + "problem": "问题17. 设 $a_i$ 为正实数 $(i=1,2, \\cdots, n)$, 令:\n$$\n\\begin{gathered}\nk b_k=a_1+a_2+\\cdots+a_k(k=1,2, \\cdots, n), \\\\\nC_n=\\left(a_1-b_1\\right)^2+\\left(a_2-b_2\\right)^2+\\cdots+\\left(a_n-b_n\\right)^2, \\\\\nD_n=\\left(a_1-b_n\\right)^2+\\left(a_2-b_n\\right)^2+\\cdots+\\left(a_n-b_n\\right)^2 .\n\\end{gathered}\n$$\n求证: $C_n \\leqslant D_n \\leqslant 2 C_n$.", + "solution": "构造数列 $x_n=2 C_n-D_n, y_n=D_n-C_n, n \\in \\mathbf{N}_{+}$. 则\n$$\n\\begin{aligned}\nx_{n+1}-x_n= & 2\\left(C_{n+1}-C_n\\right)-\\left(D_{n+1}-D_n\\right) \\\\\n= & 2\\left(a_{n+1}-b_{n+1}\\right)^2-\\left(a_{n+1}-b_{n+1}\\right)^2-n\\left(b_{n+1}^2-b_n^2\\right) \\\\\n& +2\\left(b_{n+1}-b_n\\right)\\left(a_1+a_2+\\cdots+a_n\\right) \\\\\n= & \\left(a_{n+1}-b_{n+1}\\right)^2-n\\left(b_{n+1}^2-b_n^2\\right)+2 n b_n\\left(b_{n+1}-b_n\\right) \\\\\n= & {\\left[(n+1) b_{n+1}-n b_n-b_{n+1}\\right]^2-n\\left(b_{n+1}^2-b_n^2\\right)+2 n\\left(b_n b_{n+1}-b_n^2\\right) } \\\\\n= & \\left(n^2-n\\right)\\left(b_{n+1}-b_n\\right)^2 \\geqslant 0 .\n\\end{aligned}\n$$\n又由于 $x_1=2 C_1-D_1=\\left(a_1-b_1\\right)^2=0$, 故对一切 $n, x_n \\geqslant 0\\left(n \\in \\mathbf{N}_{+}\\right)$, 同样, $y_{n+1}-y_n=n\\left(b_{n+1}^2-b_n^2\\right)-2\\left(b_{n+1}-b_n\\right)\\left(a_1+a_2+\\cdots+a_n\\right)=n\\left(b_{n+1}-\\right. \\left.b_n\\right)^2 \\geqslant 0$, 又 $y_1=D_1-C_1=0$, 故对一切 $n \\in \\mathbf{N}^{+}, y_n \\geqslant 0$.\n综上所述, $C_n \\leqslant D_n \\leqslant 2 C_n$.", + "remark": "", + "figures": [] +} \ No newline at end of file diff --git a/processed_dataset/proof/1453.json b/processed_dataset/proof/1453.json new file mode 100644 index 0000000000000000000000000000000000000000..ce5a6c0044796def3d1be58a3dafc83ab1bccfed --- /dev/null +++ b/processed_dataset/proof/1453.json @@ -0,0 +1,8 @@ +{ + "source_file": "./raw_volume-zh/volume5/exercise6.tex", + "problem_type": "proof", + "problem": "问题1. 设 $x_1, x_2, \\cdots, x_n$ 都是正实数,求证:\n$$\n\\begin{aligned}\n& \\frac{1}{x_1}+\\frac{1}{x_2}+\\cdots+\\frac{1}{x_n} \\\\\n\\geqslant & 2\\left(\\frac{1}{x_1+x_2}+\\frac{1}{x_2+x_3}+\\cdots+\\frac{1}{x_{n-1}+x_n}+\\frac{1}{x_n+x_1}\\right) .\n\\end{aligned}\n$$", + "solution": "不难证明: $\\frac{1}{x_i}+\\frac{1}{x_{i+1}} \\geqslant \\frac{4}{x_i+x_{i+1}}\\left(i=1,2, \\cdots, n, x_{n+1}=x_1\\right)$, 把 $n$ 个式子相加即得原不等式成立.", + "remark": "", + "figures": [] +} \ No newline at end of file diff --git a/processed_dataset/proof/1454.json b/processed_dataset/proof/1454.json new file mode 100644 index 0000000000000000000000000000000000000000..fd3e70fd168774b8a88ce4f63369ac3cc7e3993b --- /dev/null +++ b/processed_dataset/proof/1454.json @@ -0,0 +1,8 @@ +{ + "source_file": "./raw_volume-zh/volume5/exercise6.tex", + "problem_type": "proof", + "problem": "问题2. 设 $a, b, c \\in \\mathbf{R}^{+}$, 求证:\n$$\n\\frac{1}{a^3+b^3+a b c}+\\frac{1}{b^3+c^3+a b c}+\\frac{1}{c^3+a^3+a b c} \\leqslant \\frac{1}{a b c} .\n$$", + "solution": "由于 $a^3+b^3 \\geqslant a^2 b+b^2 a$, 故 $\\frac{1}{a^3+b^3+a b c} \\leqslant \\frac{1}{a b(a+b+c)}$. 同理 $\\frac{1}{b^3+c^3+a b c} \\leqslant \\frac{1}{b c(a+b+c)} ; \\frac{1}{c^3+a^3+a b c} \\leqslant \\frac{1}{c a(a+b+c)}$. 三式相加即得原不等式成立.", + "remark": "", + "figures": [] +} \ No newline at end of file diff --git a/processed_dataset/proof/1455.json b/processed_dataset/proof/1455.json new file mode 100644 index 0000000000000000000000000000000000000000..c6ca93baff4dbb465f79936867cba014ebae9430 --- /dev/null +++ b/processed_dataset/proof/1455.json @@ -0,0 +1,8 @@ +{ + "source_file": "./raw_volume-zh/volume5/exercise6.tex", + "problem_type": "proof", + "problem": "问题3. 已知 $0 \\leqslant x, y, z \\leqslant 1$, 解方程:\n$$\n\\frac{x}{1+y+z x}+\\frac{y}{1+z+x y}+\\frac{z}{1+x+y z}=\\frac{3}{x+y+z} .\n$$", + "solution": "不难证明: $\\frac{x}{1+y+z x} \\leqslant \\frac{1}{x+y+z}, \\frac{y}{1+z+x y} \\leqslant \\frac{1}{x+y+z}$,\n$\\frac{z}{1+x+y z} \\leqslant \\frac{1}{x+y+z}$. 因此不等式等号成立, 则易得 $x=y=z=1$.", + "remark": "注:: 本题也可以这样解: 先证明 $\\frac{x}{1+y+z x} \\leqslant \\frac{x}{x+y+z}$ 等, 这样就可以得到 $x+y+z \\geqslant 3$, 因而 $x=y=z=1$.", + "figures": [] +} \ No newline at end of file diff --git a/processed_dataset/proof/1456.json b/processed_dataset/proof/1456.json new file mode 100644 index 0000000000000000000000000000000000000000..905456754edcd1325dec6d79316d85d961e9a37a --- /dev/null +++ b/processed_dataset/proof/1456.json @@ -0,0 +1,8 @@ +{ + "source_file": "./raw_volume-zh/volume5/exercise6.tex", + "problem_type": "proof", + "problem": "问题4. 设 $a, b, c \\in \\mathbf{R}^{+}$, 且 $a b c=1$. 求证:\n$$\n\\sum_{c y c} \\frac{a b}{a^5+b^5+a b} \\leqslant 1\n$$\n并问等号何时成立?", + "solution": "由于 $a^5+b^5-a^2 b^2(a+b)=\\left(a^2-b^2\\right)\\left(a^3-b^3\\right) \\geqslant 0$, 故 $a^5+b^5 \\geqslant a^2 b^2(a+b)$, 于是\n$$\n\\begin{aligned}\n\\frac{a b}{a^5+b^5+a b} & =\\frac{a b \\cdot a b c}{a^5+b^5+a b \\cdot a b c}==\\frac{a^2 b^2 c}{a^5+b^5+a^2 b^2 c} \\\\\n& \\leqslant \\frac{a^2 b^2 c}{a^2 b^2(a+b)+a^2 b^2 c}=\\frac{c}{a+b+c} .\n\\end{aligned}\n$$\n同理, 有 $\\frac{b c}{b^5+c^5+b c} \\leqslant \\frac{a}{a+b+c}, \\frac{c a}{c^5+a^5+c a} \\leqslant \\frac{b}{a+b+c}$.\n三式相加即得不等式成立, 且等号当 $a=b=c=1$ 时取到.", + "remark": "", + "figures": [] +} \ No newline at end of file diff --git a/processed_dataset/proof/1457.json b/processed_dataset/proof/1457.json new file mode 100644 index 0000000000000000000000000000000000000000..513b5ada4f277258404bf872da746002f3588a7e --- /dev/null +++ b/processed_dataset/proof/1457.json @@ -0,0 +1,8 @@ +{ + "source_file": "./raw_volume-zh/volume5/exercise6.tex", + "problem_type": "proof", + "problem": "问题5. 给定 $\\alpha, \\beta>0, x, y, z \\in \\mathbf{R}^{+}, x y z=2004$. 求 $u$ 的最大值, 其中, $u= \\sum_{c y c} \\frac{1}{2004^{\\alpha+\\beta}+x^\\alpha\\left(y^{2 \\alpha+3 \\beta}+z^{2 \\alpha+3 \\beta}\\right)}$.", + "solution": "首先, 不难证明 $y^{2 \\alpha+3 \\beta}+z^{2 \\alpha+3 \\beta} \\geqslant y^{\\alpha+2 \\beta} z^{\\alpha+\\beta}+y^{\\alpha+\\beta} z^{2 \\alpha+\\beta}$.\n又由于 $2004^{\\alpha+\\beta}+x^\\alpha\\left(y^{2 \\alpha+3 \\beta}+z^{2 \\alpha+3 \\beta}\\right)=x^\\alpha\\left[x^\\beta y^{\\alpha+\\beta} z^{\\alpha+\\beta}+y^{2 \\alpha+3 \\beta}+z^{2 \\alpha+3 \\beta}\\right] \\geqslant x^\\alpha\\left[x^\\beta y^{\\alpha+\\beta} z^{\\alpha+\\beta}+y^{\\alpha+2 \\beta} z^{\\alpha+\\beta}+y^{\\alpha+\\beta} z^{\\alpha+2 \\beta}\\right]=x^\\alpha y^{\\alpha+\\beta} z^{\\alpha+\\beta}\\left(x^\\beta+y^\\beta+z^\\beta\\right)$.\n因此, $u \\leqslant \\frac{x^\\beta}{2004^{\\alpha+\\beta}\\left(x^\\beta+y^\\beta+z^\\beta\\right)}+\\frac{y^\\beta}{2004^{\\alpha+\\beta}\\left(x^\\beta+y^\\beta+z^\\beta\\right)}$\n$$\n+\\frac{z^\\beta}{2004^{\\alpha+\\beta}} \\frac{1}{\\left(x^\\beta+y^\\beta+z^\\beta\\right)}=\\frac{1}{2004^{\\alpha+\\beta}},\n$$\n故 $u_{\\max }=2004^{-(\\alpha+\\beta)}$.", + "remark": "", + "figures": [] +} \ No newline at end of file diff --git a/processed_dataset/proof/1458.json b/processed_dataset/proof/1458.json new file mode 100644 index 0000000000000000000000000000000000000000..3f6f2f84c353873e0307f00fe4bd293fde10f473 --- /dev/null +++ b/processed_dataset/proof/1458.json @@ -0,0 +1,8 @@ +{ + "source_file": "./raw_volume-zh/volume5/exercise6.tex", + "problem_type": "proof", + "problem": "问题6. 设 $x_1, x_2, \\cdots, x_n$ 都是正数, 且 $x_1+x_2+\\cdots+x_n=a$. 对 $m, n \\in \\mathbf{Z}^{+}, m$, $n>1$, 求证: $\\frac{x_1^m}{a-x_1}+\\frac{x_2^m}{a-x_2}+\\cdots+\\frac{x_n^m}{a-x_n} \\geqslant \\frac{a^{m-1}}{(n-1) n^{m-2}}$.", + "solution": "由平均不等式, $\\frac{x_i^m}{a-x_i}+\\frac{\\left(a-x_i\\right) a^{m-2}}{(n-1)^2 n^{m-2}}+\\underbrace{\\frac{a^{m-1}}{(n-1) n^{m-1}}+\\cdots+\\frac{a^{m-1}}{(n-1) n^{m-1}}}_{m-2 \\uparrow} \\geqslant$\n$$\nm \\cdot \\frac{x_i \\cdot a^{m-2}}{(n-1)} n^{n-2} \\text {. }\n$$\n故 $\\sum_{i=1}^n \\frac{x_i^m}{a-x_i} \\geqslant \\sum_{i=1}^n\\left[\\frac{m x_i a^{m-2}}{(n-1) n^{m-2}}-\\frac{\\left(a-x_i\\right) a^{m-2}}{(n-1)^2 n^{m-2}}-\\frac{(m-2) a^{m-1}}{(n-1) n^{m-1}}\\right]= \\frac{a^{m-1}}{(n-1) n^{m-2}}$.", + "remark": "注:: 本题也可利用 Cauchy 不等式及幕平均不等式加以解决.", + "figures": [] +} \ No newline at end of file diff --git a/processed_dataset/proof/1459.json b/processed_dataset/proof/1459.json new file mode 100644 index 0000000000000000000000000000000000000000..41e0b9f5195b1dcca043c7c70a5fe53789f8fcfc --- /dev/null +++ b/processed_dataset/proof/1459.json @@ -0,0 +1,8 @@ +{ + "source_file": "./raw_volume-zh/volume5/exercise6.tex", + "problem_type": "proof", + "problem": "问题7. 已知 $a, b, c \\in \\mathbf{R}^{+}$, 求证:\n(1) $\\sqrt[3]{\\frac{a}{b+c}}+\\sqrt[3]{\\frac{b}{c+a}}+\\sqrt[3]{\\frac{c}{a+b}}>\\frac{3}{2}$;\n(2) $\\sqrt[3]{\\frac{a^2}{(b+c)^2}}+\\sqrt[3]{\\frac{b^2}{(c+a)^2}}+\\sqrt[3]{\\frac{c^2}{(a+b)^2}} \\geqslant \\frac{3}{\\sqrt[3]{4}}$.", + "solution": "(1) $\\sqrt[3]{\\frac{a}{b+c}}=\\frac{a}{\\sqrt[3]{a \\cdot a \\cdot(b+c)}} \\geqslant \\frac{3 a}{2 a+b+c}>\\frac{3 a}{2 a+2 b+2 c}$,\n同理, 有 $\\sqrt[3]{\\frac{b}{a+c}}>\\frac{3 b}{2 a+2 b+2 c} ; \\sqrt[3]{\\frac{c}{a+b}}>\\frac{3 c}{2 a+2 b+2 c}$.\n三式相加即得原不等式成立.\n(2) $\\sqrt[3]{\\frac{a^2}{(b+c)^2}}=-\\frac{\\sqrt[3]{2} \\cdot a}{\\sqrt[3]{2 a(b+c)(b+c)}} \\geqslant \\frac{3 \\sqrt[3]{2} a}{2 a+2 b+2 c}$. 同理, 有\n$$\n\\sqrt[3]{\\frac{b^2}{(a+c)^2}} \\geqslant \\frac{3 \\sqrt[3]{2} b}{2 a+2 b+2 c} ; \\sqrt[3]{\\frac{c^2}{(a+b)^2}} \\geqslant \\frac{3 \\sqrt[3]{2} c}{2 a+2 b+2 c} .\n$$\n三式相加, 原不等式成立 (注意等号是可以取到的).", + "remark": "", + "figures": [] +} \ No newline at end of file diff --git a/processed_dataset/proof/1460.json b/processed_dataset/proof/1460.json new file mode 100644 index 0000000000000000000000000000000000000000..abed9035ca3f4287b3cb18745da8cad7621e297e --- /dev/null +++ b/processed_dataset/proof/1460.json @@ -0,0 +1,8 @@ +{ + "source_file": "./raw_volume-zh/volume5/exercise6.tex", + "problem_type": "proof", + "problem": "问题8. 设正整数 $n \\geqslant 2$, 已知 $n$ 个正数 $v_1, v_2, \\cdots, v_n$ 满足下列两个条件:\n(1) $v_1+v_2+\\cdots+v_n=1$;\n(2) $v_1 \\leqslant v_2 \\leqslant \\cdots \\leqslant v_n \\leqslant 2 v_1$.\n求 $v_1^2+v_2^2+\\cdots+v_n^2$ 的最大值.", + "solution": "可以把条件推广为: $v_1+v_2+\\cdots+v_n=1, v_1 \\leqslant v_2 \\leqslant \\cdots \\leqslant v_n \\leqslant r v_1$.\n下面证明: $v_1^2+v_2^2+\\cdots+v_n^2 \\leqslant \\frac{(r+1)^2}{4 m}$, 其中 $r>1, r \\in \\mathbf{R}^{+}$.\n对于任意 $j \\in \\mathbf{Z}^{+}, 1 \\leqslant j \\leqslant n$, 由条件, 有 $\\left(v_j-v_1\\right)\\left(r v_1-v_j\\right) \\geqslant 0$, 故 $r v_1 v_j-r v_1^2-v_j^2+v_1 v_j \\geqslant 0$.\n上式关于 $j$ 从 1 到 $n$ 求和, 可得 $\\sum_{j=1}^n v_j^2 \\leqslant(r+1) v_1-m v_1^2= -m\\left(v_1-\\frac{r+1}{2 m}\\right)^2+\\frac{(r+1)^2}{4 m} \\leqslant \\frac{(r+1)^2}{4 m}$.", + "remark": "", + "figures": [] +} \ No newline at end of file diff --git a/processed_dataset/proof/1461.json b/processed_dataset/proof/1461.json new file mode 100644 index 0000000000000000000000000000000000000000..14531b1f01eaf683d5a0d56195f36e944729ceb2 --- /dev/null +++ b/processed_dataset/proof/1461.json @@ -0,0 +1,8 @@ +{ + "source_file": "./raw_volume-zh/volume5/exercise6.tex", + "problem_type": "proof", + "problem": "问题9. 已知 $a>1, b>1, c>1$, 求证:\n(1) $\\frac{a^5}{b^2-1}+\\frac{b^5}{c^2-1}+\\frac{c^5}{a^2-1} \\geqslant \\frac{26}{6} \\sqrt{15}$;\n(2) $\\frac{a^5}{b^3-1}+\\frac{b^5}{c^3-1}+\\frac{c^5}{a^3-1} \\geqslant \\frac{5}{2} \\sqrt[3]{50}$.", + "solution": "(1) 利用平均不等式,有\n$$\n\\begin{gathered}\n\\quad \\frac{a^5}{b^2-1}+\\frac{25(5+\\sqrt{15})}{12}(b-1)+\\frac{25(5-\\sqrt{15})}{12}(b+1)+\\frac{25}{18} \\sqrt{15}+ \\\\\n\\frac{25}{18} \\sqrt{15} \\geqslant \\frac{125}{6} a, \\text { 故 } \\frac{a^5}{b^2-1} \\geqslant \\frac{125}{6}(a-b)+\\frac{25}{18} \\sqrt{15} .\n\\end{gathered}\n$$\n同理, $\\frac{b^5}{c^2-1} \\geqslant \\frac{125}{6}(b-c)+\\frac{25}{18} \\sqrt{15} ; \\frac{c^5}{a^2-1} \\geqslant \\frac{125}{6}(c-a)+\\frac{25}{18} \\sqrt{15}$ 相加即得原不等式成立.\n(2) 由于 $\\left(b^3-1\\right)\\left(b^3-1\\right)\\left(b^3-1\\right) \\times \\frac{3}{2} \\times \\frac{3}{2} \\leqslant \\frac{3^5 \\cdot b^{15}}{5^5}$, 故 $b^3-1 \\leqslant\\frac{3 \\cdot \\sqrt[3]{20} \\cdot b^5}{25}$, 则 $\\frac{a^5}{b^3-1} \\geqslant \\frac{5 \\cdot \\sqrt[3]{50} \\cdot a^5}{6 b^5}$. 同理可得 $\\frac{b^5}{c^3-1} \\geqslant \\frac{5 \\cdot \\sqrt[3]{50} \\cdot b^5}{6 c^5}$; $\\frac{c^5}{a^3-1} \\geqslant \\frac{5 \\cdot \\sqrt[3]{50} \\cdot c^5}{6 a^5}$. 因此, 不等式左端 $\\geqslant \\frac{5 \\sqrt[3]{50}}{6}\\left(\\frac{a^5}{b^5}+\\frac{b^5}{c^5}+\\frac{c^5}{a^5}\\right) \\geqslant \\frac{5}{2} \\cdot \\sqrt[3]{50}$.", + "remark": "注::第(1) 小题也可以采用与第 (2) 小题类似的方法.\n由于 $\\left(b^2-1\\right)\\left(b^2-1\\right) \\cdot \\frac{2}{3} \\cdot \\frac{2}{3} \\cdot \\frac{2}{3} \\leqslant\\left(\\frac{2 b^2}{5}\\right)^5$, 有 $b^2-1 \\leqslant \\frac{6 \\sqrt{3} b^5}{25 \\sqrt{5}}$, 则 $\\frac{a^5}{b^2-1} \\geqslant \\frac{25 \\sqrt{5} a^5}{6 \\sqrt{3} b^5}$, 以此不难证得原不等式成立.", + "figures": [] +} \ No newline at end of file diff --git a/processed_dataset/proof/1462.json b/processed_dataset/proof/1462.json new file mode 100644 index 0000000000000000000000000000000000000000..be70c30ddb38519ae4ffa2f620095e9757cb4f1d --- /dev/null +++ b/processed_dataset/proof/1462.json @@ -0,0 +1,8 @@ +{ + "source_file": "./raw_volume-zh/volume5/exercise6.tex", + "problem_type": "proof", + "problem": "问题11. 已知 $\\sum_{i=1}^n x_i=1$, 求证: $\\sum_{i=1}^n \\sqrt{\\frac{1}{x_i}-1} \\geqslant(n-1) \\cdot \\sum_{i=1}^n \\frac{1}{\\sqrt{\\frac{1}{x_i}-1}}$.", + "solution": "证明:局部不等式: $x_i \\cdot \\sqrt{\\frac{1-x_j}{x_j}}+x_j \\cdot \\sqrt{\\frac{1-x_i}{x_i}} \\geqslant\\left(1-x_i\\right) \\sqrt{\\frac{x_j}{1-x_j}}+ \\left(1-x_j\\right) \\sqrt{\\frac{x_i}{1-x_i}}$. 进而易证原不等式成立.", + "remark": "", + "figures": [] +} \ No newline at end of file diff --git a/processed_dataset/proof/1463.json b/processed_dataset/proof/1463.json new file mode 100644 index 0000000000000000000000000000000000000000..cf91cf88e966dca25c8c2d22344a4b66c9bea774 --- /dev/null +++ b/processed_dataset/proof/1463.json @@ -0,0 +1,8 @@ +{ + "source_file": "./raw_volume-zh/volume5/exercise6.tex", + "problem_type": "proof", + "problem": "问题12. 设 $n$ 个集合 $S_1, S_2, \\cdots, S_n$ 的元素由非负整数构成, $x_i$ 为 $S_i$ 的所有元素之和.\n求证: 若对某个自然数 $k, 1\\frac{1}{k+1} \\cdot\\left[k \\cdot \\frac{n(n+1)(2 n+1)}{6}-(k+1)^2 \\frac{n(n+1)}{2}\\right] .\n$$\n不妨设 $x_1 \\leqslant x_2 \\leqslant \\cdots \\leqslant x_n$.\n由假设, $x_j-x_i$ 互不相等 $(1 \\leqslant i\\frac{1}{2} m^2 k^2-\\frac{1}{2} k[k(k+1)+1] m .\n\\end{aligned}\n$$\n又由于 $M=\\left(x_m-x_1\\right)+\\left(x_m+x_{m-1}-x_2-x_1\\right)+\\cdots+\\left(x_m+x_{m-1}+\\cdots+\\right. \\left.x_{m-k+1}-x_k-x_{k-1}-\\cdots-x_1\\right) \\leqslant k x_m+(k-1) x_{m-1}+\\cdots+x_{m-k-1} \\leqslant k x_m+ (k-1) x_m+\\cdots+x_m=\\frac{k(k+1)}{2} x_m$.\n由此即得 $x_m>\\frac{k}{k+1} m^2-\\frac{k(k+1)+1}{k+1} m \\geqslant \\frac{k}{k+1} m^2-(k+1) m(k< m \\leqslant n)$.\n当 $1 \\leqslant m \\leqslant k$ 时, 上式显然也成立.\n于是, $\\sum_{i=1}^n x_i>\\frac{k}{k+1} \\sum_{i=1}^n i^2-(k+1) \\sum_{i=1}^n i=\\frac{1}{k+1} \\cdot\\left[k \\frac{n(n+1)(2 n+1)}{6}-\\right. \\left.(k+1)^2 \\frac{n(n+1)}{2}\\right]$ 对一切 $10$.", + "solution": "用数学归纳法去证明 $a_n>n$.", + "remark": "", + "figures": [] +} \ No newline at end of file diff --git a/processed_dataset/proof/1465.json b/processed_dataset/proof/1465.json new file mode 100644 index 0000000000000000000000000000000000000000..4a4c6fddfc60c9e9aa02fda678c9b5a982036120 --- /dev/null +++ b/processed_dataset/proof/1465.json @@ -0,0 +1,8 @@ +{ + "source_file": "./raw_volume-zh/volume5/exercise7.tex", + "problem_type": "proof", + "problem": "问题2. 设数列 $a_1, a_2, \\cdots, a_{2 n+1}$ 满足: $a_i-2 a_{i+1}+a_{i+2} \\geqslant 0(i=1,2, \\cdots, 2 n-1)$, 求证:\n$$\n\\frac{a_1+a_3+\\cdots+a_{2 n+1}}{n+1} \\geqslant \\frac{a_2+a_4+\\cdots+a_{2 n}}{n} .\n$$", + "solution": "设 $(k-1)\\left(a_1+a_3+\\cdots+a_{2 k-1}\\right) \\geqslant k\\left(a_2+a_4+\\cdots+a_{2 k-2}\\right)$, 要证命题对 $n$ 成立, 只须证 $a_1+a_2+\\cdots+a_{2 k-1}+k a_{2 k+2} \\geqslant a_2+a_4+\\cdots+a_{2 k-2}+(k+$ 1) $a_{2 k}$, 即 $k\\left(a_{2 k+1}-a_{2 k}\\right) \\geqslant\\left(a_2-a_1\\right)+\\cdots+\\left(a_{2 k}-a_{2 k-1}\\right)$, 由条件 $a_{i+2}-a_{i+1} \\geqslant a_{i+1}-a_i$, 这是显然的.", + "remark": "", + "figures": [] +} \ No newline at end of file diff --git a/processed_dataset/proof/1466.json b/processed_dataset/proof/1466.json new file mode 100644 index 0000000000000000000000000000000000000000..1a2c343d1da86f79623e5f159fd5ac8e777d163c --- /dev/null +++ b/processed_dataset/proof/1466.json @@ -0,0 +1,8 @@ +{ + "source_file": "./raw_volume-zh/volume5/exercise7.tex", + "problem_type": "proof", + "problem": "问题3. 设 $\\left\\{a_n\\right\\}$ 是各项为正的数列, 若 $a_{n+1} \\leqslant a_n-a_n^2$, 求证: 对一切 $n \\geqslant 2$, 都有\n$$\na_n \\leqslant \\frac{1}{n+2} .\n$$", + "solution": "利用 $y=-x^2+x$ 在 $\\left(0, \\frac{1}{4}\\right)$ 上的单调性即可完成从 $k$ 到 $k+1$ 的过渡.", + "remark": "", + "figures": [] +} \ No newline at end of file diff --git a/processed_dataset/proof/1467.json b/processed_dataset/proof/1467.json new file mode 100644 index 0000000000000000000000000000000000000000..b17f926525e1da00c56c3c4a7ac302e410b1c405 --- /dev/null +++ b/processed_dataset/proof/1467.json @@ -0,0 +1,8 @@ +{ + "source_file": "./raw_volume-zh/volume5/exercise7.tex", + "problem_type": "proof", + "problem": "问题4. 已知数列 $\\left\\{a_n\\right\\}, a_1=a_2=1, a_{n+2}=a_{n+1}+a_n$. 求证:对任意 $n \\in \\mathbf{N}_{+}$, 有 $\\operatorname{arccot} a_n \\leqslant \\operatorname{arccot} a_{n+1}+\\operatorname{arccot} a_{n+2}$,\n并指出等号成立的条件.", + "solution": "由 $a_n>0, \\operatorname{arccot} a_{n+1}+\\operatorname{arccot} a_{n+2} \\in(0, \\pi), \\cot x$ 在 $(0, \\pi)$ 上是减函数, 故只须证明: $\\cot \\left(\\operatorname{arccot} a_n\\right) \\geqslant \\cot \\left(\\operatorname{arccot} a_{n+1}+\\operatorname{arccot} a_{n+2}\\right)$, 即证 $\\left(a_{n+1}+\\right. \\left.a_{n+2}\\right) a_n \\geqslant a_{n+1} \\cdot a_{n+2}-1$.\n而 $a_{n+2}=a_{n+1}+a_n$, 故只要证明: $a_n a_{n+2}-a_{n+1}^2 \\geqslant-1$.\n用数学归纳法, 容易验证 $a_n a_{n+2}-a_{n+1}^2==(-1)^{n+1}$. 因此原不等式成立, 易见等号成立当且仅当 $n$ 为偶数.", + "remark": "", + "figures": [] +} \ No newline at end of file diff --git a/processed_dataset/proof/1468.json b/processed_dataset/proof/1468.json new file mode 100644 index 0000000000000000000000000000000000000000..c5b1ad619b00337f1290215fc948fb7917e7fbac --- /dev/null +++ b/processed_dataset/proof/1468.json @@ -0,0 +1,8 @@ +{ + "source_file": "./raw_volume-zh/volume5/exercise7.tex", + "problem_type": "proof", + "problem": "问题5. 设 $a_1, a_2, \\cdots$ 是实数列, 且对所有 $i 、 j=1,2, \\cdots$ 满足: $a_{i+j} \\leqslant a_i+a_j$, 求证: 对于正整数 $n$, 有\n$$\na_1+\\frac{a_2}{2}+\\frac{a_3}{3}+\\cdots+\\frac{a_n}{n} \\geqslant a_n .\n$$", + "solution": "对 $n$ 用数学归纳法.\n当 $n=1$ 时, $a_1 \\geqslant a_1$, 不等式显然成立.\n假设当 $n=1,2, \\cdots, k-1$ 时不等式成立, 即有\n$$\n\\left\\{\\begin{array}{l}\na_1 \\geqslant a_1, \\\\\na_1+\\frac{a_2}{2} \\geqslant a_2, \\\\\n\\cdots \\ldots \\ldots \\ldots \\ldots \\ldots \\ldots . . . \\cdots \\cdots \\\\\na_1+\\frac{a_2}{2}+\\cdots+\\frac{a_{k-1}}{k-1} \\geqslant a_{k-1} .\n\\end{array}\\right.\n$$\n相加得 $(k-1) a_1+(k-2) \\frac{a_2}{2}+\\cdots+(k-(k-1)) \\frac{a_{k-1}}{k-1} \\geqslant a_1+ a_2+\\cdots+a_{k-1}$, 即 $k\\left(a_1+\\frac{a_n}{2}+\\cdots+\\frac{a_{k-1}}{k-1}\\right) \\geqslant 2\\left(a_1+a_2+\\cdots+a_{k-1}\\right)=\\left(a_1+\\right. \\left.a_{k-1}\\right)+\\left(a_2+a_{k-2}\\right)+\\cdots+\\left(a_{k-1}+a_1\\right) \\geqslant k a_k-a_k$.\n故 $a_1+\\frac{a_2}{2}+\\cdots+\\frac{a_k}{k} \\geqslant a_k$, 因此原不等式成立.", + "remark": "", + "figures": [] +} \ No newline at end of file diff --git a/processed_dataset/proof/1469.json b/processed_dataset/proof/1469.json new file mode 100644 index 0000000000000000000000000000000000000000..9c2bce2f45adee43d1645cd1ae92ac567024344a --- /dev/null +++ b/processed_dataset/proof/1469.json @@ -0,0 +1,8 @@ +{ + "source_file": "./raw_volume-zh/volume5/exercise7.tex", + "problem_type": "proof", + "problem": "问题6. 设非负整数 $a_1, a_2, \\cdots, a_{2004}$ 满足 $a_i+a_j \\leqslant a_{i+j} \\leqslant a_i+a_j+1(1 \\leqslant i 、 j$, $i+j \\leqslant 2004)$. 求证: 存在 $x \\in \\mathbf{R}$, 对所有 $n(1 \\leqslant n \\leqslant 2004)$, 有 $a_n= [n x]$.", + "solution": "若存在 $x$, 使得 $a_n=[n x]$, 则 $\\frac{a_n}{n} \\leqslant x<\\frac{a_n+1}{n}$, 此不等式应对 $n=1$, $2, \\cdots, 2004$ 都成立, 于是, $x$ 应同时属于 2004 个区间: $\\left[\\frac{a_n}{n}, \\frac{a_n+1}{n}\\right)$. 如果 $x$ 存在, $x$ 可取为 $\\max \\left\\{\\frac{a_m}{m}\\right\\}$.\n这样, 只要证明对一切 $n \\in\\{1,2, \\cdots, 2004\\}$, 都有 $\\frac{a_n+1}{n}>x \\geqslant \\frac{a_m}{m}$.\n下面证明: 若 $m 、 n$ 是正整数,且 $m 、 n \\leqslant 2004$, 则有\n$$\nm a_n+m>n a_m . \\label{(1)}\n$$\n当 $m=n$ 时,(1)式成立; 又由于当 $m=1, n=2$ 和 $m=2, n=1$ 时,有 $a_2+1>2 a_1$ 及 $2 a_1+2>a_2$, 由题设知(1)式成立.\n假设当 $m 、 n$ 都小于 $k(3 \\leqslant k \\leqslant 2004)$ 时命题成立.\n当 $m=k$ 时, 设 $m=n q+r, q \\in \\mathbf{N}_{+}, 0 \\leqslant rn a_r$, 故 $n a_mr a_m$, 故 $m a_n+m>n a_m$, 因此(1)式成立, 故 $\\frac{a_n+1}{n}>\\frac{a_m}{m}$. 令 $x= \\max _{1 \\leqslant m \\leqslant 2004} \\frac{a_m}{m}$, 则 $\\frac{a_n+1}{n}>x$, 故 $n x-10$, 求证:\n$$\n\\sqrt{a+\\sqrt{2 a+\\sqrt{3 a+\\cdots+\\sqrt{n a}}}}<\\sqrt{a}+1 .\n$$", + "solution": "我们用归纳法来证: 当 $1 \\leqslant k \\leqslant n$ 时,\n$$\n\\sqrt{k a+\\sqrt{(k+1) a+\\cdots+\\sqrt{n a}}}<1+\\sqrt{k a} . \\label{(1)}\n$$\n当 $k=n$ 时, 上式显然成立.\n设 $\\sqrt{(k+1) a+\\sqrt{(k+2) a+\\cdots+\\sqrt{n a}}}<1+ \\sqrt{(k+1) a}$, 则 $\\sqrt{k a+\\sqrt{(k+1) a+\\cdots+\\sqrt{n a}}}<\\sqrt{k a+1+\\sqrt{(k+1) a}}<\\sqrt{k a+1+2 \\sqrt{k a}}=1+\\sqrt{k a}$.\n因此, (1)对一切 $k(1 \\leqslant k \\leqslant n)$ 成立.\n特别地, 当 $k=1$ 时有原不等式成立.", + "remark": "", + "figures": [] +} \ No newline at end of file diff --git a/processed_dataset/proof/1472.json b/processed_dataset/proof/1472.json new file mode 100644 index 0000000000000000000000000000000000000000..4349890622113fb38ad501817e8c77376bfba8c4 --- /dev/null +++ b/processed_dataset/proof/1472.json @@ -0,0 +1,8 @@ +{ + "source_file": "./raw_volume-zh/volume5/exercise7.tex", + "problem_type": "proof", + "problem": "问题9. 若 $a_i>0(i=1,2,3, \\cdots, n)$, 且 $a_1 \\cdot a_2 \\cdots \\cdots a_n=1$, 求证:\n$$\n\\sum_{i=1}^n a_i \\geqslant n\n$$", + "solution": "当 $n=1$ 时, $a_1=1$, 结论成立.\n设当 $n=k$ 时结论成立.\n当 $n=k+1$ 时, $a_1 a_2 \\cdots a_{k+1}=1$, 由归纳假设, 有\n$$\n\\begin{aligned}\n& \\quad a_{k+1} \\cdot a_1+a_2+\\cdots+a_n \\geqslant k ; a_{k+1} \\cdot a_2+a_1+a_3+\\cdots+a_k \\geqslant k ; \\cdots, a_{k+1} \\cdot \\\\\n& a_k+a_1+\\cdots+a_{k-1} \\geqslant k .\n\\end{aligned}\n$$\n全部相加, 得\n$$\n\\left(a_1+a_2+\\cdots+a_k\\right)\\left(a_{k+1}+k-1\\right) \\geqslant k^2 .\n$$\n故 $\\quad \\sum_{i=1}^{k+1} a_i=\\sum_{i=1}^k a_i+a_{k+1} \\geqslant \\frac{k^2}{a_{k+1}+k-1}+a_{k+1} \\geqslant k+1$.\n因此结论成立.", + "remark": "", + "figures": [] +} \ No newline at end of file diff --git a/processed_dataset/proof/1473.json b/processed_dataset/proof/1473.json new file mode 100644 index 0000000000000000000000000000000000000000..8c6614903d39f815983c83fb82d9e61c50704f07 --- /dev/null +++ b/processed_dataset/proof/1473.json @@ -0,0 +1,8 @@ +{ + "source_file": "./raw_volume-zh/volume5/exercise7.tex", + "problem_type": "proof", + "problem": "问题10. 设数列 $\\left\\{a_n\\right\\}$ 满足: $a_1=a_2=1, a_{n+2}=a_{n+1}+a_n, S_n$ 为数列 $\\left\\{a_n\\right\\}$ 的前 $n$ 项之和.\n求证: $S_n=\\sum_{k=1}^n \\frac{a_k}{2^k}<2$.", + "solution": "设 $b_k=\\frac{a_k}{2^k}$, 则 $b_{k+2}=\\frac{1}{2} b_{k+1}+\\frac{1}{4} b_k, b_1=\\frac{1}{2}, b_2=\\frac{1}{4}$, 故\n$$\nb_{k+1}=\\frac{1}{2} b_k+\\frac{1}{4} b_{k-1}, b_k=\\frac{1}{2} b_{k-1}+\\frac{1}{4} b_{k-2}, \\cdots, b_3=\\frac{1}{2} b_2+\\frac{1}{4} b_1,\n$$\n相加, 得 $S_{k+2}-b_1-b_2=\\frac{1}{2} S_{k+1}-\\frac{1}{2} b_1+\\frac{1}{4} S_k, S_{k+2}=\\frac{1}{2} S_{k+1}+\\frac{1}{4} S_k+\\frac{1}{2}$, 接着用数学归纳法即可证明 $S_n<2$.", + "remark": "", + "figures": [] +} \ No newline at end of file diff --git a/processed_dataset/proof/1474.json b/processed_dataset/proof/1474.json new file mode 100644 index 0000000000000000000000000000000000000000..363d98265e634a70bb179173105a64f0fc3715c8 --- /dev/null +++ b/processed_dataset/proof/1474.json @@ -0,0 +1,8 @@ +{ + "source_file": "./raw_volume-zh/volume5/exercise7.tex", + "problem_type": "proof", + "problem": "问题11. 设 $r_1, r_2, \\cdots, r_n$ 为 $\\geqslant 1$ 的实数,证明:\n$$\n\\frac{1}{r_1+1}+\\frac{1}{r_2+1}+\\cdots+\\frac{1}{r_n+1} \\geqslant \\frac{n}{\\sqrt[n]{r_1 r_2 \\cdots r_n}+1} .\n$$", + "solution": "当 $n=1$ 时,不等式显然成立.\n下面用归纳法证明当 $n=2^k$ 时, 不等式成立.\n当 $k=1$ 时,有\n$$\n\\frac{1}{r_1+1}+\\frac{1}{r_2+1}-\\frac{2}{\\sqrt{r_1 r_2}+1}=\\frac{\\left(\\sqrt{r_1 r_2}-1\\right)\\left(\\sqrt{r_1}-\\sqrt{r_2}\\right)^2}{\\left(r_1+1\\right)\\left(r_2+1\\right)\\left(\\sqrt{r_1 r_2}+1\\right)} \\geqslant 0 .\n$$\n若当 $k=m$ 时, 不等式成立, 考虑 $k=m+1$ 的情况.\n即要证明:\n若对 $n$ 个数原不等式成立, 则对 $2 n$ 个数不等式也成立.\n如果 $r_1, r_2, \\cdots, r_{2 n}$ 均大于 1 , 则有\n$$\n\\begin{aligned}\n\\sum_{i=1}^{2 n} \\frac{1}{r_i+1} & =\\sum_{i=1}^n \\frac{1}{r_i+1}+\\sum_{i=n+1}^{2 n} \\frac{1}{r_i+1} \\\\\n& \\geqslant \\frac{n}{\\sqrt[n]{r_1 r_2 \\cdots r_n}+1}+\\frac{n}{\\sqrt[n]{r_{n+1} \\cdots r_{2 n}}+1} \\\\\n& \\geqslant \\frac{2 n}{\\sqrt[2 n]{r_1 r_2 \\cdots r_{2 n}}+1} .\n\\end{aligned}\n$$\n故当 $n=2^k(k=1,2, \\cdots)$ 时, 原不等式成立.\n对任意自然数 $n$, 存在正整数 $k$, 满足 $m=2^k>n$.\n设 $r_{n+1}=r_{n+2}=\\cdots=r_m=\\sqrt[n]{r_1 r_2 \\cdots r_n}$, 那么\n$$\n\\frac{1}{r_1+1}+\\frac{1}{r_2+1}+\\cdots+\\frac{1}{r_n+1}+\\frac{m-n}{\\sqrt[n]{r_1 r_2 \\cdots r_n}+1} \\geqslant \\frac{m}{\\sqrt[n]{r_1 r_2 \\cdots r_n}+1},\n$$\n因此原不等式成立.", + "remark": "", + "figures": [] +} \ No newline at end of file diff --git a/processed_dataset/proof/1475.json b/processed_dataset/proof/1475.json new file mode 100644 index 0000000000000000000000000000000000000000..623c961cf67e2f920efe158aee67e5f943b958cd --- /dev/null +++ b/processed_dataset/proof/1475.json @@ -0,0 +1,8 @@ +{ + "source_file": "./raw_volume-zh/volume5/exercise7.tex", + "problem_type": "proof", + "problem": "问题12. 设 $f(n)$ 定义在正整数集合上, 且满足 $f(1)=2, f(n+1)=f^2(n)- f(n)+1, n=1,2, \\cdots$. 求证: 对所有整数 $n>1$, 有\n$$\n1-\\frac{1}{2^{2^{n^{-1}}}}<\\frac{1}{f(1)}+\\frac{1}{f(2)}+\\cdots+\\frac{1}{f(n)}<1-\\frac{1}{2^{2^n}} .\n$$", + "solution": "由已知条件可得 $\\frac{1}{f(n)}=\\frac{1}{f(n)-1}-\\frac{1}{f(n+1)-1}$,\n故 $\\quad \\sum_{k=1}^n \\frac{1}{f(k)}=\\frac{1}{f(1)-1}-\\frac{1}{f(n+1)-1}=1-\\frac{1}{f(n+1)-1}$.\n下面证明: $2^{2^{n-1}}9$, 注意到 $2 \\lambda_5>9$, 定义函数 $f_0: \\mathbf{N}_{+} \\rightarrow \\mathbf{R}$ 如下:\n$$\n\\begin{gathered}\nf_0(1)=2, \\\\\nf_0(n)=2 \\lambda_k\\left(2^k9$. 故问题解决, $M=10$.", + "remark": "", + "figures": [] +} \ No newline at end of file diff --git a/processed_dataset/proof/1477.json b/processed_dataset/proof/1477.json new file mode 100644 index 0000000000000000000000000000000000000000..d63cf2c2c19a518eea6d5dc1384ae2b6c103a5b4 --- /dev/null +++ b/processed_dataset/proof/1477.json @@ -0,0 +1,8 @@ +{ + "source_file": "./raw_volume-zh/volume5/exercise7.tex", + "problem_type": "proof", + "problem": "问题14. 设 $a_1, a_2, \\cdots, a_n$ 是非负实数,满足: $\\sum_{i=1}^n a_i=4, n \\geqslant 3$. 求证:\n$$\na_1^3 a_2+a_2^3 a_3+\\cdots+a_{n-1}^3 a_n+a_n^3 a_1 \\leqslant 27 \\text {. }\n$$", + "solution": "对 $n$ 用数学归纳法.\n当 $n=3$ 时, 由于原不等式关于 $a_1 、 a_2 、 a_3$ 轮换对称, 不妨设 $a_1$ 为最大者, 若 $a_20$, 则(1)等价于\n$$\n1+\\frac{y^{s+2}}{s+1} \\geqslant y^2 . \\label{(2)}\n$$\n令 $f(y)=1+y^2\\left(\\frac{y^s}{s+1}-1\\right)$, 这里 $y>0$.\n如果 $y \\geqslant(s+1)^{\\frac{1}{s}}$, 则 $f(y) \\geqslant 1>0$, (2)显然成立.\n当 $y \\in(0,1]$ 时, (2)也自然成立.\n现在考虑 $\\left(1,(s+1)^{\\frac{1}{s}}\\right)$ 中的 $y$, 要证明 $f(y)>0$.\n而当 $y \\in\\left(1,(s+1)^{\\frac{1}{s}}\\right)$ 时, $\\frac{y^s}{s+1}<1$, 故 $f(y)=1-y^2\\left(1-\\frac{y^s}{s+1}\\right)$\n取 $A$ 为待定系数,于是\n$$\n\\begin{aligned}\nA^2\\left[y^2\\left(1-\\frac{y^s}{s+1}\\right)\\right]^s & =\\left(A y^s\\right)\\left(A y^s\\right)\\left(1-\\frac{y^s}{s+1}\\right) \\cdots\\left(1-\\frac{y^s}{s+1}\\right) \\\\\n& \\leqslant\\left\\{\\frac{1}{s+2}\\left[2 A y^s+s\\left(1-\\frac{y^s}{s+1}\\right)\\right]\\right\\}^{s+2} .\n\\end{aligned}\n$$\n取 $A=\\frac{s}{2(s+1)}$, 有 $\\left(\\frac{s}{2(s+1)}\\right)^2 \\cdot\\left[y^2\\left(1-\\frac{y^s}{1+s}\\right)\\right]^s \\leqslant\\left(\\frac{s}{s+2}\\right)^{s+2}$.\n所以 $y^2\\left(1-\\frac{y^s}{s+1}\\right) \\leqslant \\frac{s}{s+2}\\left[\\frac{2(s+1)}{s+2}\\right]^{\\frac{2}{s}}$.\n当 $s=1$ 时, 从上式有 $y^2\\left(1-\\frac{y}{2}\\right) \\leqslant \\frac{16}{27}<1$, 此时 $f(y)>0$.\n下面证明: 当正整数 $s \\geqslant 2$ 时, $\\left[\\frac{2(s+1)}{s+2}\\right]^{\\frac{2}{s}}<\\frac{s+2}{s}$, 如果上式成立, 则 $f(y)>0$.\n上面不等式等价于 $\\frac{2(s+1)}{s+2}<\\left(1+\\frac{2}{s}\\right)^{\\frac{s}{2}}$. 如果能证明 $\\left(1+\\frac{2}{s}\\right)^{\\frac{s}{2}} \\geqslant 2$, 则问题解决.\n事实上, $\\left(1+\\frac{2}{s}\\right)^s=1+s \\cdot \\frac{2}{s}+\\mathrm{C}_s^2 \\cdot\\left(\\frac{2}{s}\\right)^2+\\cdots \\geqslant 1+2+\\frac{2(s-1)}{s} \\geqslant$ 4. 故结论成立.", + "remark": "", + "figures": [] +} \ No newline at end of file diff --git a/processed_dataset/proof/1479.json b/processed_dataset/proof/1479.json new file mode 100644 index 0000000000000000000000000000000000000000..de732fe55287b5e2956f9bb4d8c9863a3579404d --- /dev/null +++ b/processed_dataset/proof/1479.json @@ -0,0 +1,8 @@ +{ + "source_file": "./raw_volume-zh/volume5/exercise7.tex", + "problem_type": "proof", + "problem": "问题16. 设 $z_i(1 \\leqslant i \\leqslant n)$ 是 $n$ 个复数, $s_i=z_1+z_2+\\cdots+z_i, 1 \\leqslant i \\leqslant n$. 求证:\n$$\n\\sum_{1 \\leqslant i0, \\sin \\frac{1}{6}\\left(\\lambda_1 A+\\lambda_2 B+4 \\lambda_3 C\\right)>0, \\sin \\frac{1}{3}\\left(\\lambda_1 A+\\lambda_2 B+\\lambda_3 C\\right)>0$.\n于是 $\\quad \\sin \\lambda_1 A+\\sin \\lambda_2 B+\\sin \\lambda_3 C+\\sin \\frac{\\lambda_1 A+\\lambda_2 B+\\lambda_3 C}{3}$\n$$\n\\begin{aligned}\n= & 2 \\sin \\frac{\\lambda_1 A+\\lambda_2 B}{2} \\cdot \\cos \\frac{\\lambda_1 A-\\lambda_2 B}{2} \\\\\n& +2 \\sin \\frac{\\lambda_1 A+\\lambda_2 B+4 \\lambda_3 C}{6} \\cdot \\cos \\frac{2 \\lambda_3 C-\\lambda_1 A-\\lambda_2 B}{6} \\\\\n\\leqslant & 2 \\sin \\frac{\\lambda_1 A+\\lambda_2 B}{2}+2 \\sin \\frac{\\lambda_1 A+\\lambda_2 B+4 \\lambda_3 C}{6} \\\\\n= & 4 \\sin \\frac{4 \\lambda_1 A+4 \\lambda_2 B+4 \\lambda_3 C}{12} \\cos \\frac{2 \\lambda_1 A+2 \\lambda_2 B-4 \\lambda_3 C}{12} \\\\\n\\leqslant & 4 \\sin \\frac{\\lambda_1 A+\\lambda_2 B+\\lambda_3 C}{3},\n\\end{aligned}\n$$\n等号成立当且仅当 $\\lambda_1 A=\\lambda_2 B==\\lambda_3 C$.", + "remark": "", + "figures": [] +} \ No newline at end of file diff --git a/processed_dataset/proof/1481.json b/processed_dataset/proof/1481.json new file mode 100644 index 0000000000000000000000000000000000000000..b5b8828bdfe81a91a1c2fe70cec33a206f7a8503 --- /dev/null +++ b/processed_dataset/proof/1481.json @@ -0,0 +1,8 @@ +{ + "source_file": "./raw_volume-zh/volume5/exercise8.tex", + "problem_type": "proof", + "problem": "问题4. 设 $a, b, c, d \\geqslant 0$, 且 $a+b+c+d==4$. 求证:\n$$\nb c d+c d a+d a b+a b c-a b c d \\leqslant \\frac{1}{2}(a b+a c+a d+b c+b d+c d) .\n$$", + "solution": "当 $a 、 b 、 c 、 d$ 中有 1 个为 0 时, 不妨设 $d=0$, 则只须证明: $a b c \\leqslant \\frac{1}{2}(a b+b c+c a)$. 由于 $\\frac{1}{a}+\\frac{1}{b}+\\frac{1}{c}+a+b+c \\geqslant 6$, 有 $\\frac{1}{a}+\\frac{1}{b}+\\frac{1}{c} \\geqslant 2$, 故上式成立.\n若 $a 、 b 、 c 、 d$ 均不为 0 , 则只须证: $f=\\sum \\frac{1}{a}-\\frac{1}{2} \\sum \\frac{1}{a b} \\leqslant 1$.\n不妨设 $a \\leqslant 1 \\leqslant b$, 则令 $a^{\\prime}=1, b^{\\prime}=a+b-1$, 有 $a^{\\prime} b^{\\prime} \\geqslant a b$, 于是\n$$\n\\begin{aligned}\n& f(a, b, c, d)-f\\left(a^{\\prime}, b^{\\prime}, c^{\\prime}, d^{\\prime}\\right) \\\\\n= & (a+b)\\left(\\frac{1}{a b}-\\frac{1}{a^{\\prime} b^{\\prime}}\\right)\\left[1-\\frac{1}{2}\\left(\\frac{1}{a+b}+\\frac{1}{c}+\\frac{1}{d}\\right)\\right] \\\\\n\\leqslant & (a+b) \\frac{a^{\\prime} b^{\\prime}-a b}{a^{\\prime} b^{\\prime} a b}\\left(1-\\frac{1}{2} \\cdot \\frac{32}{4}\\right) \\leqslant 0 .\n\\end{aligned}\n$$\n故 $f(a, b, c, d) \\leqslant f\\left(a^{\\prime}, b^{\\prime}, c^{\\prime}, d^{\\prime}\\right)==f(1, a+b-1, c, d)$.\n再经过上述两次磨光变换, 得\n$$\n\\begin{aligned}\nf(1, a+b-1, c, d) & \\leqslant f(1,1, a+b+c-2, d) \\\\\n& \\leqslant f(1,1,1, a+b+c+d-3) \\\\\n& =f(1,1,1,1)=1 .\n\\end{aligned}\n$$", + "remark": "", + "figures": [] +} \ No newline at end of file diff --git a/processed_dataset/proof/1482.json b/processed_dataset/proof/1482.json new file mode 100644 index 0000000000000000000000000000000000000000..7b4242912cdebbdd2e8b38ce584bd0eaf25fdd11 --- /dev/null +++ b/processed_dataset/proof/1482.json @@ -0,0 +1,8 @@ +{ + "source_file": "./raw_volume-zh/volume5/exercise8.tex", + "problem_type": "proof", + "problem": "问题5. 已知 $x_i$ 是非负实数, $i=1,2,3,4 . x_1+x_2+x_3+x_4=1$. 记 $S=1-\\sum_{i=1}^4 x_i^3-6 \\sum_{1 \\leqslant i1$ (若 $a_1=1$, 则 $n= 55)$, 则 $a_1-1$ 不在原来 10 个数中,将 $a_1$ 换成 $a_1-1$, 和至少减少了 $a_1(1+2)- \\left(a_1-1\\right)(1+2)=3$.\n而若 $a_1, a_2, \\cdots, a_{10}$ 不是相连的正整数,则必有 1 个 $a_i$, 使得 $a_i-1$ 不在原来的 10 个数中,将 $a_i$ 换成 $a_i-1$, 和也至少减少了 3. 因此, 对 $a_1+a_2+\\cdots+ a_{10}=n+1$ 的每一种排法 $S(n+1)$, 可找到 $a_1, a_2, \\cdots, a_i-1, \\cdots, a_{10}$ 的一个排法, 使 $S(n+1) \\geqslant S(n)+3 \\geqslant g(n)+3$, 故 $\\min S(n+1) \\geqslant g(n)+3$, 即 $g(n+1) \\geqslant g(n)+3$, 因而 $g(2002) \\geqslant 6065$, 并且等号可以取到: $a_1=1, a_2=9$, $a_3=3, a_4=7, a_5=5, a_6=6, a_7=4, a_8=8, a_9=2, a_{10}=1957$.", + "remark": "", + "figures": [] +} \ No newline at end of file diff --git a/processed_dataset/proof/1485.json b/processed_dataset/proof/1485.json new file mode 100644 index 0000000000000000000000000000000000000000..d66dbcb52aa21fa6f389a44be79511114b92d1f8 --- /dev/null +++ b/processed_dataset/proof/1485.json @@ -0,0 +1,8 @@ +{ + "source_file": "./raw_volume-zh/volume5/exercise8.tex", + "problem_type": "proof", + "problem": "问题10. 非负实数 $a 、 b 、 c$ 满足: $a b+b c+c a=1$, 求 $\\frac{1}{a+b}+\\frac{1}{b+c}+\\frac{1}{c+a}$ 的最小值.", + "solution": "记 $f(a, b, c)=\\frac{1}{a+b}+\\frac{1}{b+c}+\\frac{1}{c+a}$, 不妨设 $a \\leqslant b \\leqslant c$, 我们先证明: $f\\left(0, a+b, c^{\\prime}\\right) \\leqslant f(a, b, c)$. 这里 $c^{\\prime}=\\frac{1}{a+b}, a b+b c+c a=1$. 事实上, $f\\left(0, a+b, c^{\\prime}\\right) \\leqslant f(a, b, c)$ 等价于\n$$\n\\frac{1}{a+b}+\\frac{1}{a+b+c^{\\prime}}+\\frac{1}{c^{\\prime}} \\leqslant \\frac{1}{a+b}+\\frac{1}{b+c}+\\frac{1}{c+a} .\n$$\n由于 $c^{\\prime}=\\frac{1}{a+b}, \\frac{1-a b}{a+b}=c$, 不难化简得上式等价于\n$$\n(a+b)^2 a b \\leqslant 2(1-a b) .\n$$\n注意到 $2(1-a b)=2 c(a+b) \\geqslant \\frac{2(a+b)^2}{2} \\geqslant(a+b)^2 a b$, 故结论成立.\n因此, $f(a, b, c) \\geqslant \\frac{1}{a+b}+\\frac{1}{a+b+\\frac{1}{a+b}}+a+b$.\n不难验证在 $a=0, b=c=1$ 时, $f$ 取得最小值 $\\frac{5}{2}$.", + "remark": "注:: 也可用累次求极值法来解:\n显然, 当 $a, b, c$ 均大于 $\\frac{\\sqrt{3}}{3}$ 时, 有 $a b+b c+c a>1$, 不符题意, 则 $a 、 b 、 c$ 中必有一个不大于 $\\frac{\\sqrt{3}}{3}$, 不妨设 $b \\leqslant \\frac{\\sqrt{3}}{3}$, 则\n$$\n\\begin{aligned}\nS & =\\frac{1}{a+b}+\\frac{1}{b+c}+\\frac{1}{c+a} \\\\\n& =\\frac{a+2 b+c}{1+b^2}+\\frac{1}{c+a} \\\\\n& =\\frac{2 b}{1+b^2}+\\frac{a+c}{1+b^2}+\\frac{1}{a+c} .\n\\end{aligned}\n$$\n下面先固定 $b$, 求出 $\\frac{a+c}{1+b^2}+\\frac{1}{a+c}$ 的最小值.\n令 $x=a+c$, 则 $f(x)=\\frac{x}{1+b^2}+\\frac{1}{x}$ 在 $x \\geqslant \\sqrt{1+b^2}$ 时单调递增, 而 $1- b(a+c)=a c \\leqslant \\frac{(a+c)^2}{4}$, 故 $\\frac{x^2}{4} \\geqslant 1-b x$, 则 $x \\geqslant 2 \\sqrt{b^2+1}-2 b$. 又 $b^2+1 \\geqslant 4 b^2$, 故 $x \\geqslant \\sqrt{b^2+1}$, 从而 $f(x) \\geqslant f\\left(2 \\sqrt{b^2+1}-2 b\\right)$.\n综上可知, $S \\geqslant \\frac{2 b}{1+b^2}+f\\left(2 \\sqrt{b^2+1}-2 b\\right)=\\frac{2}{\\sqrt{b^2+1}}+\\frac{\\sqrt{b^2+1}+b}{2}$, 令 $y=\\sqrt{b^2+1}-b>0$, 则 $b=\\frac{1-y^2}{2 y}$.\n于是, $S \\geqslant \\frac{2}{y+b}+\\frac{1}{2 y}=\\frac{9 y^2+1}{2 y\\left(1+y^2\\right)}=\\frac{5}{2}+\\frac{(1-y)\\left(5\\left(y-\\frac{2}{5}\\right)^2+\\frac{1}{5}\\right)}{2 y\\left(1+y^2\\right)} \\geqslant \\frac{5}{2}$ (注意 $\\sqrt{b^2+1}-b \\leqslant 1$ ).\n等号当 $b=0$ 时取到, 此时 $a=c=1$, 故所求最小值为 $\\frac{5}{2}$.", + "figures": [] +} \ No newline at end of file diff --git a/processed_dataset/proof/1486.json b/processed_dataset/proof/1486.json new file mode 100644 index 0000000000000000000000000000000000000000..04ffb9d0000c72690310afb5f93f51fa948119a1 --- /dev/null +++ b/processed_dataset/proof/1486.json @@ -0,0 +1,8 @@ +{ + "source_file": "./raw_volume-zh/volume5/exercise8.tex", + "problem_type": "proof", + "problem": "问题11. 设 $a_1, a_2, \\cdots, a_{2001}$ 都是非负实数,满足:\n(1) $a_1+a_2+\\cdots+a_{2001}=2$;\n(2) $a_1 a_2+a_2 a_3+\\cdots+a_{2000} a_{2001}+a_{2001} a_1=1$.\n求 $S=a_1^2+a_2^2+\\cdots+a_{2001}^2$ 的最值.", + "solution": "下面先计算 $f\\left(a_1, a_2, \\cdots, a_{2001}\\right)=a_1 a_2+a_2 a_3+\\cdots+a_{2000} a_{2001}+ a_{2001} a_1$ 的最大值.\n引理:存在一个 $i \\in\\{1,2, \\cdots, 2001\\}$, 使 $a_i>a_{i+4}$ (记 $a_{2001+i}=a_i$ ).\n证明: 若不然, 有 $a_1 \\leqslant a_5 \\leqslant a_9 \\leqslant \\cdots \\leqslant a_{2001} \\leqslant a_4 \\leqslant a_8 \\leqslant \\cdots \\leqslant a_{2000} \\leqslant a_3 \\leqslant a_7 \\leqslant \\cdots \\leqslant a_{1999} \\leqslant a_2 \\leqslant a_6 \\leqslant \\cdots \\leqslant a_{1998} \\leqslant a_1$.\n于是, 有 $a_1=a_2=\\cdots=a_{2001}=\\frac{2}{2001}$, 不合题目条件,引理证毕.\n不妨设 $a_{1998}>a_1$. 对于给定的 $a_1, a_2, \\cdots, a_{1998}, a_{2000}$, 令 $a_1^{\\prime}=a_1, a_2^{\\prime}=$\n$$\n\\begin{aligned}\n& a_2, \\cdots, a_{1998}^{\\prime}=a_{1998}, a_{2000}^{\\prime}=a_{2000}, a_{2001}^{\\prime}=0, a_{1999}^{\\prime}=a_{1999}+a_{2001} \\text {, 则 } \\sum_i a_i^{\\prime}=2 . \\\\\n& \\quad f\\left(a_1^{\\prime}, a_2^{\\prime}, \\cdots, a_{2001}^{\\prime}\\right)=a_1 a_2+a_2 a_3+\\cdots+a_{1997} a_{1998}+a_{1998}\\left(a_{1999}+a_{2001}\\right)+ \\\\\n& \\left(a_{1999}+a_{2001}\\right) \\cdot a_{2000}>f\\left(a_1, a_2, \\cdots, a_{2001}\\right) .\n\\end{aligned}\n$$\n接着, 令 $a_1^{\\prime \\prime}=a_1^{\\prime}, \\cdots, a_{1998}^{\\prime \\prime}=a_{1998}^{\\prime}+a_{2000}^{\\prime}, a_{1999}^{\\prime \\prime}=a_{1999}^{\\prime}, a_{2000}^{\\prime}=0$, 则易证 $f\\left(a_1^{\\prime \\prime}, a_2^{\\prime \\prime}, \\cdots, a_{2000}^{\\prime \\prime}, 0\\right)>f\\left(a_1^{\\prime}, a_2^{\\prime}, \\cdots, a_{2000}^{\\prime}, 0\\right)$.\n即每次将末尾一个非零的 $a_i$ 变为 0 , 将 $a_i$ 与 $a_{i-2}$ 的和变为新的 $a_{i-2}, f$ 值增大, 直至只剩下 3 个数时, $a_1+a_2+a_3=2, f\\left(a_1, a_2, a_3, 0, \\cdots, 0\\right)= a_1 a_2+a_2 a_3=a_2\\left(2-a_2\\right) \\leqslant 1$, 即 $f\\left(a_1, a_2, \\cdots, a_{2001}\\right)$ 的最大值为 1 , 当且仅当 $a_4=a_5=\\cdots=a_{2001}=0, a_2=1$ 时取到.\n因此, $S=a_1^2+a_2^2+a_3^2=1+a_1^2+a_3^2 \\geqslant 1+\\frac{\\left(a_1+a_3\\right)^2}{2}=\\frac{3}{2}$.\n且 $S=2\\left(a_1-1\\right)^2+\\frac{3}{2} \\leqslant 2\\left(0 \\leqslant a_1 \\leqslant 1\\right)$. 故 $S_{\\max }=2, S_{\\min }=\\frac{3}{2}$.", + "remark": "", + "figures": [] +} \ No newline at end of file diff --git a/processed_dataset/proof/1487.json b/processed_dataset/proof/1487.json new file mode 100644 index 0000000000000000000000000000000000000000..a78d430020f9ec934c6a93e350b30bf70bda9edc --- /dev/null +++ b/processed_dataset/proof/1487.json @@ -0,0 +1,8 @@ +{ + "source_file": "./raw_volume-zh/volume5/exercise8.tex", + "problem_type": "proof", + "problem": "问题13. 设 $n$ 为一个固定的整数, $n \\geqslant 2$.\n(1)确定最小的常数 $c$, 使得不等式\n$$\n\\sum_{1 \\leqslant i1400$, 故 $1 \\leqslant l_1 \\leqslant 60$.\n注意到 $1997-l_1$ 随 $l_1$ 增大每次小 1 , 但 $\\sqrt{l_1\\left(3994-l_1\\right)}$ 上升很快, 猜测最小值必在 $l_1$ 较小时取到.\n(1) $l_1=1,(m+n)(m-n)=3993, m+n \\geqslant 121$.\n即当 $l=1996$ 时, $m+n+l \\geqslant 1996+121=2117$.\n(2)当 $200 \\leqslant l_1 \\leqslant 600$ 时, $l+m+n \\geqslant 1997-600+\\sqrt{200 \\times 3794}>2117$.\n(3)当 $50 \\leqslant l_1 \\leqslant 200$ 时,也易见 $l+m+n \\geqslant 1997-200+\\sqrt{50 \\times 3994}>$ 2117.\n(4)当 $10 \\leqslant l_1 \\leqslant 50$ 时, $l+m+n \\geqslant 1997-50+\\sqrt{10 \\times 3984}>2117$.\n(5)当 $5 \\leqslant l_1 \\leqslant 10$ 时, $l+m+n \\geqslant 1997-10+\\sqrt{5 \\times 3989}>2117$.\n(6) $l_1=4, l+m+n \\geqslant 1997-4+2 \\times 63>2117$.\n(7) $l_1=3,(m-n)(m+n)=3 \\times 3991$, 有 $(m+n)_{\\text {min }}=307$, 则 $l+m+ n=1997-3+307=2301$.\n(8) $l_1=2,(m-n)(m+n)=998$, 则 $l+m+n \\geqslant 1997-2+998>2117$.\n综上所述, $(l+m+n)_{\\text {min }}=2117$, 等号当 $l=1996, m=77, n=44$ 时取到.\n再来求 $l+m+n$ 的最大值.\n我们有 $(m-n)(m+n)=l_1\\left(3994-l_1\\right)$, 故 $l_1\\cdots>a_k \\geqslant 1$, 满足: 对任意 $i 、 j$,有 $\\left[a_i, a_j\\right] \\leqslant n$ 成立.\n求证:\n$$\nk \\leqslant 2 \\sqrt{n}+1 \\text {. }\n$$", + "solution": "证法 1 : 取定一个参量 $t, a_i \\leqslant t$ 的至多有 $t$ 个, 下面估计大于 $t$ 的那些 $a_i$ 的个数.\n首先,对任意两个 $a_i, a_j$, 有 $\\left[a_i, a_j\\right] \\leqslant n$, 即\n$$\n\\frac{a_i a_j}{\\left(a_i, a_j\\right)} \\leqslant n . \\label{(1)}\n$$\n而 $\\left(a_i, a_j\\right) \\mid a_i-a_j(\\neq 0)$, 故 $\\left|a_i-a_j\\right| \\geqslant\\left(a_i, a_j\\right)$.\n由(1), $\\frac{\\left(a_i, a_j\\right)}{a_i a_j} \\geqslant \\frac{1}{n}$, 故 $\\frac{\\left|a_i-a_j\\right|}{a_i a_j} \\geqslant \\frac{1}{n}$, 即\n$$\n\\left|\\frac{1}{a_i}-\\frac{1}{a_j}\\right| \\geqslant \\frac{1}{n} . \\label{(2)}\n$$\n对于 $a_1>a_2>\\cdots>a_l>t$, 有 $\\frac{1}{n}<\\frac{1}{a_1}<\\cdots<\\frac{1}{a_l}<\\frac{1}{t}$.\n结合(2)知 $l \\leqslant\\left(\\frac{1}{t}-\\frac{1}{n}\\right) \\cdot n$.\n综上所述, $k \\leqslant t+\\left(\\frac{1}{t}-\\frac{1}{n}\\right) \\cdot n=\\frac{n}{t}+t-1$.\n取 $t=[\\sqrt{n}]$, 即有 $k \\leqslant 2 \\sqrt{n}+1$.", + "remark": "", + "figures": [] +} \ No newline at end of file diff --git a/processed_dataset/proof/1490.json b/processed_dataset/proof/1490.json new file mode 100644 index 0000000000000000000000000000000000000000..989f26f61b53a338511ca59718b609bf77e5a398 --- /dev/null +++ b/processed_dataset/proof/1490.json @@ -0,0 +1,8 @@ +{ + "source_file": "./raw_volume-zh/volume5/exercise9.tex", + "problem_type": "proof", + "problem": "问题1. 设 $n \\geqslant a_1>\\cdots>a_k \\geqslant 1$, 满足: 对任意 $i 、 j$,有 $\\left[a_i, a_j\\right] \\leqslant n$ 成立.\n求证:\n$$\nk \\leqslant 2 \\sqrt{n}+1 \\text {. }\n$$", + "solution": "证法 2 : 我们来估计每一项有些什么性质:\n由 $\\left[a_1, a_2\\right]=\\frac{a_1 \\cdot a_2}{\\left(a_1, a_2\\right)} \\leqslant n$, 故 $a_2 \\cdot\\left[\\frac{a_1}{\\left(a_1, a_2\\right)}\\right] \\leqslant n$.\n由于 $a_1>a_2$, 易证 $\\frac{a_1}{\\left(a_1, a_2\\right)} \\geqslant 2$, 故 $2 a_2 \\leqslant n$.\n于是, 我们已有 $a_1 \\leqslant n, 2 a_2 \\leqslant n$, 猜测对所有 $i(1 \\leqslant i \\leqslant k)$, 都有\n$$\ni a_i \\leqslant n . \\label{(3)}\n$$\n假设(3)是正确的,那么\n$k=$ 不超过 $a_t$ 的数的个数 + 大于 $a_t$ 的数的个数\n$\\leqslant$ 不超过 $a_t$ 的数的个数 $+t$\n$$\n\\leqslant a_t+t \\leqslant \\frac{n}{t}+t \\text {. }\n$$\n取 $t=[\\sqrt{n}]+1$, 亦有 $k \\leqslant 2 \\sqrt{n}+1$. 可见(3)可能是正确的.\n用数学归纳法.\n设 $(i-1) a_{i-1} \\leqslant n$, 要证 $i a_i \\leqslant n$.\n事实上, $\\left[a_i, a_{i-1}\\right]=A a_i=B a_{i-1}$, 并且 $\\left[a_i, a_{i-1}\\right] \\leqslant n$.\n(1) 如果 $A \\geqslant i$, 则 $i a_i \\leqslant n$ 成立.\n(2) 如果 $A\\frac{2}{\\pi} x$, 故 $x \\in(0, \\pi)$ 时, $\\sin \\frac{x}{2}>\\frac{2}{\\pi} \\cdot \\frac{x}{2}= \\frac{x}{\\pi}$\n于是, $\\left|\\sum_{k=m+1}^n \\frac{\\sin k x}{k}\\right| \\leqslant \\frac{\\pi}{x} \\cdot \\frac{1}{m+1} \\leqslant \\frac{\\pi}{x} \\cdot \\frac{x}{\\sqrt{\\pi}}=\\sqrt{\\pi}$.\n所以, $\\left|\\sum_{k=1}^n \\frac{\\sin k x}{k}\\right| \\leqslant \\sqrt{\\pi}+\\sqrt{\\pi}=2 \\sqrt{\\pi}$.", + "remark": "注: 利用此法可以证明下述命题:\n设 $\\left\\{a_n\\right\\}$ 为非增的正数数列, 求证: 若对 $n \\geqslant 2001, n a_n \\leqslant 1$, 则对任意自然数 $m \\geqslant 2001, x \\in \\mathbf{R}$, 有 $\\left|\\sum_{k=2001}^m a_k \\sin k x\\right| \\leqslant 1+\\pi$.\n证明如下:\n令 $f_{m, n}(x)=\\sum_{k=n}^m a_k \\sin k x(n=2001)$, 则 $f_{m, n}(x)$ 为奇函数, 且为周期函数,故只要在 $[0, \\pi]$ 上考虑即可.\n(i) $x \\in\\left[\\frac{\\pi}{n}, \\pi\\right]$, 则\n$$\n|\\sin n x+\\cdots+\\sin m x|=\\left|\\frac{\\cos \\left(n-\\frac{1}{2}\\right) x-\\cos \\left(m+\\frac{1}{2}\\right) x}{2 \\sin \\frac{x}{2}}\\right|\n$$\n$$\n\\leqslant-\\frac{1}{\\sin \\frac{x}{2}} \\leqslant \\frac{\\pi}{x}\n$$\n因此, $\\left|f_{m, n}(x)\\right| \\leqslant a_n \\cdot \\frac{\\pi}{x} \\leqslant n a_n \\leqslant 1$.\n(ii) $x \\in\\left[0, \\frac{\\pi}{m}\\right)$, 则 $\\left|f_{m, n}(x)\\right| \\leqslant a_n n x+\\cdots+a_m m x \\leqslant(m-n) x \\leqslant \\pi$.\n(iii) $x \\in\\left[\\frac{\\pi}{m}, \\frac{\\pi}{n}\\right]$, 则 $n \\leqslant \\frac{\\pi}{x} \\leqslant m$, 记 $k=\\left[\\frac{\\pi}{x}\\right]$.\n因此, $\\left|f_{m, n}(x)\\right| \\leqslant\\left|a_n \\sin n x+\\cdots+a_k \\sin k x\\right|+\\mid a_{k+1} \\sin (k+1) x+\\cdots+ a_m \\sin m x \\mid$.\n由 $00$, 且 $a+b+c=3$. 求证:\n$$\n\\frac{a}{c}+\\frac{b}{a}+\\frac{c}{b} \\geqslant 3+Q\n$$\n其中 $Q=|(a-1)(b-1)(c-1)|$.", + "solution": "(1) 当 $a \\geqslant 1 \\geqslant b \\geqslant c$ 时, $\\theta=2+a b c-a b-b c-c a$.\n因为 $\\frac{a}{c}+a c+\\frac{b}{a}+a b+\\frac{c}{b}+b c \\geqslant 2 a+2 b+2 c=6$, 又 $a+b+c \\geqslant 3 \\sqrt[3]{a b c}$, $a b c \\leqslant 1$, 故左边 $\\geqslant 6+a b c-1-a b-b c-a c=3+\\theta$.\n(2)当 $a \\geqslant b \\geqslant 1 \\geqslant c$ 时, $\\theta=a b+b c+c a-2-a b c \\geqslant 0$, 故 $a b+b c+c a \\geqslant 2+a b c$.\n又由于 $\\frac{a}{c}+a b^2 c \\geqslant 2 a b, \\frac{b}{a}+a b c^2 \\geqslant 2 b c, \\frac{c}{b}+a^2 b c \\geqslant 2 a c$, 故左边 $\\geqslant 2(a b+b c+c a)-3 a b c \\geqslant a b+b c+c a+2-2 a b c \\geqslant a b+b c+c a+1-a b c= 3+\\theta$.", + "remark": "注:: 我们可以证明更强的结论: $\\frac{a}{c}+\\frac{b}{a}+\\frac{c}{b} \\geqslant 3+4 \\theta$.\n证明: 由于 $\\left(\\frac{a}{c}+\\frac{b}{a}+\\frac{c}{b}\\right)-\\left(\\frac{a}{b}+\\frac{b}{c}+\\frac{c}{a}\\right)=\\frac{(a-b)(b-c)}{b c}+ \\frac{(b-c)(b-a)}{a b}=\\frac{(a-b)(b-c)(a-c)}{a b c} \\geqslant 0$,\n故 $\\frac{a}{c}+\\frac{b}{a}+\\frac{c}{b}-3 \\geqslant \\frac{1}{2}\\left(\\frac{a}{c}+\\frac{b}{a}+\\frac{c}{b}+\\frac{a}{b}+\\frac{b}{c}+\\frac{c}{a}\\right)-3=\\frac{a(b-c)^2}{+b(c-a)^2+c(a-b)^2}-$, 以下分两种情况:\n(i) $a \\geqslant 1 \\geqslant b \\geqslant c$, 设 $1-b=\\alpha, 1-c=\\beta, \\alpha \\beta \\geqslant 0$, 则 $a-1=\\alpha+\\beta$, $\\alpha+\\beta \\leqslant 2$. 那么\n$$\n\\begin{aligned}\n& \\frac{a(b-c)^2+b(c-a)^2+c(a-b)^2}{2 a b c} \\\\\n= & \\frac{a(\\alpha-\\beta)^2+b(\\alpha+2 \\beta)^2+c(\\beta+2 \\alpha)^2}{2 a b c} \\\\\n\\geqslant & \\frac{b(\\alpha+2 \\beta)^2+c(\\beta+2 \\alpha)^2}{2 a b c} \\\\\n\\geqslant & \\frac{2 \\sqrt{b c} \\cdot(\\alpha+2 \\beta)(\\beta+2 \\alpha)}{2 a b c} \\\\\n= & \\frac{(\\alpha+2 \\beta)(\\beta+2 \\alpha)}{a \\sqrt{b c} \\geqslant \\frac{9 \\alpha \\beta}{a \\sqrt{b c}}}=\\frac{9 \\alpha \\beta}{\\sqrt{4 \\cdot\\left(\\frac{a}{2}\\right)^2 b c}} \\\\\n\\geqslant & \\frac{\\sqrt{9 \\alpha \\beta}}{\\sqrt{4 \\cdot\\left(\\frac{a}{2}+\\frac{a}{2}+b+c\\right.}}=\\frac{9 \\alpha \\beta}{\\frac{9}{8}} \\\\\n= & 8 \\alpha \\beta \\geqslant 4(\\alpha+\\beta)_{\\alpha \\beta}=4 \\theta .\n\\end{aligned}\n$$\n(ii) $a \\geqslant b \\geqslant 1 \\geqslant c$, 设 $1+\\alpha=a, 1+\\beta=b, c=1-\\alpha-\\beta$, 则 $\\alpha+\\beta \\leqslant 1$, $\\alpha-\\beta \\geqslant 0$. 此时\n$$\n\\begin{aligned}\n& \\frac{a(b-c)^2+b(c-a)^2+c(a-b)^2}{2 a b c} \\\\\n= & \\frac{a(2 \\beta+\\alpha)^2+b(2 \\alpha+\\beta)^2+c(\\alpha-\\beta)^2}{2 a b c} \\\\\n\\geqslant & \\frac{a(2 \\beta+\\alpha)^2+b(2 \\alpha+\\beta)^2}{2 a b c} \\\\\n\\geqslant & \\frac{2 \\sqrt{a b}(2 \\beta+\\alpha)(2 \\alpha+\\beta)}{2 a b c} \\\\\n\\geqslant & 9 \\alpha \\beta \\geqslant 9(\\alpha+\\beta) \\alpha \\beta=9 \\theta .\n\\end{aligned}\n$$\n综上, $\\frac{a}{c}+\\frac{b}{a}+\\frac{c}{b} \\geqslant 3+4 \\theta$.", + "figures": [] +} \ No newline at end of file diff --git a/processed_dataset/proof/1494.json b/processed_dataset/proof/1494.json new file mode 100644 index 0000000000000000000000000000000000000000..2f239abca13d942fe01c0c7a7dfaf27563b5a456 --- /dev/null +++ b/processed_dataset/proof/1494.json @@ -0,0 +1,8 @@ +{ + "source_file": "./raw_volume-zh/volume5/exercise9.tex", + "problem_type": "proof", + "problem": "问题6. 已知 $a, b, c \\in \\mathbf{R}^{+}$, 且 $a b c \\leqslant 1$, 求证:\n$$\n\\frac{a}{c}+\\frac{b}{a}+\\frac{c}{b} \\geqslant Q+a+b+c,\n$$\n其中 $Q=|(a-1)(b-1)(c-1)|$.", + "solution": "(1) 当 $a \\leqslant 1, b \\leqslant 1, c \\leqslant 1$ 时, 我们来证左式 $\\geqslant 3 \\geqslant a+b+c+\\theta$. 这等价于 $a b+b c+c a \\leqslant 2+a b c$.\n利用 $(1-a)(1-b) \\geqslant 0$, 得 $2-a-b \\geqslant 1-a b \\geqslant c(1-a b)$, 故\n$$\n2+a b c \\geqslant a+b+c \\geqslant a b+b c+c a .\n$$\n(2)当 $a \\geqslant 1, b \\leqslant 1, c \\leqslant 1$ 时, 由 $\\theta \\geqslant 0$, 有\n$$\na+b+c \\geqslant a b+b c+c a+1-a b c .\n$$\n而左式 $+a c+a b+b c \\geqslant 2(a+b+c)$, 则\n$$\n\\text { 左式 } \\geqslant 2 \\sum a-\\sum a c+a b c-1=a+b+c+\\theta \\text {. }\n$$\n(3)当 $a \\geqslant 1, b \\geqslant 1, c \\leqslant 1$ 时,易证 $a+b+c \\geqslant a+1+b c \\geqslant 2+a b c$.\n又由 $\\theta \\geqslant 0$, 得 $a b+b c+c a \\geqslant a+b+c+a b c-1$.\n$$\n\\begin{aligned}\n& \\sum a^2 b+\\sum b \\geqslant 2 \\sum a b \\geqslant \\sum a b+\\sum a+a b c-1, \\\\\n& \\sum a^2 b \\geqslant \\sum a b+a b c-1 \\geqslant a b c\\left(\\sum a b+1-a b c\\right),\n\\end{aligned}\n$$\n故因此\n$$\n\\text { 左端 } \\geqslant \\sum a b+1-a b c=a+b+c+\\theta \\text {. }\n$$\n对 $a \\geqslant 1, c \\geqslant 1, b \\leqslant 1$ 的情况也可类似证明.\n综上所述, 原不等式成立.", + "remark": "", + "figures": [] +} \ No newline at end of file diff --git a/processed_dataset/proof/1495.json b/processed_dataset/proof/1495.json new file mode 100644 index 0000000000000000000000000000000000000000..b340aa8d31fb3ca5a3263b689d165e44a4b14af1 --- /dev/null +++ b/processed_dataset/proof/1495.json @@ -0,0 +1,8 @@ +{ + "source_file": "./raw_volume-zh/volume5/exercise9.tex", + "problem_type": "proof", + "problem": "问题7. 在平面上, 将半径分别为 $1 、 2 、 3 、 4 、 5 、 6$ 的六个圆, 沿直线 $l$ 排成一串 (即六圆与 $l$ 外切于六点, 切点相邻的两圆外切). 求首尾两圆外公切线长的最值.", + "solution": "(1)下面证明: 依次按半径为1、3、5、6、4、2或2、4、6、5、3、1的顺序排列6个圆时,首尾两圆外公切线为最长.\n设六个圆依次为 $O_1, O_2, \\cdots, O_6, O_i$ 半径为 $r_i$, 且与 $l$ 相切于 $T_i(1 \\leqslant i \\leqslant 6)$, 对于 $1 \\leqslant i \\leqslant 5$, 由勾股定理易知: $T_i T_{i+1}^2=4 r_i r_{i+1}$.\n因此, $T_1 T_6=2\\left(\\sqrt{r_1 r_2}+\\sqrt{r_2 r_3}+\\sqrt{r_3 r_4}+\\sqrt{r_5 r_6}\\right)$.\n由于 $r_1, r_2, \\cdots, r_6$ 是 $1,2, \\cdots, 6$ 的一个排列, 故 $T_1 T_6$ 的表达式可写为\n$$\nT_1 T_6=2\\left(\\sqrt{1 r_1^{\\prime}}+\\sqrt{2 r_2^{\\prime}}+\\sqrt{3 r_3^{\\prime}}+\\sqrt{4 r_4^{\\prime}}+\\sqrt{5 r_5^{\\prime}}\\right) .\n$$\n由上式及排序原理易知, 在 $r_1^{\\prime} \\leqslant r_2^{\\prime} \\leqslant r_3^{\\prime} \\leqslant r_4^{\\prime} \\leqslant r_5^{\\prime}$ 且每个数尽可能大时, $T_1 T_6$ 值最大, 于是试着取 $r_5^{\\prime}=r_4^{\\prime}=6$.\n另一方面, $r_1^{\\prime} 、 r_2^{\\prime} 、 r_3^{\\prime}$ 只能是 $1 、 2 、 3 、 4 、 5$ 中的某三个, 且互不相等, 所以取 $r_1^{\\prime}=3, r_2^{\\prime}=4, r_3^{\\prime}=5$, 按前面所说的, 这样选取的 $r_1^{\\prime}, r_2^{\\prime}, \\cdots, r_6^{\\prime}$ 将使 $T_1 T_6$ 值最大.\n由于此时 $\\sum_{i=1}^5 \\sqrt{i_i^{\\prime}}=\\sqrt{1 \\times 3}+\\sqrt{2 \\times 4}+\\sqrt{3 \\times 5}+\\sqrt{4 \\times 6}+\\sqrt{5 \\times 6}= \\sqrt{1 \\times 3}+\\sqrt{3 \\times 5}+\\sqrt{5 \\times 6}+\\sqrt{6 \\times 4}+\\sqrt{4 \\times 2}=\\sqrt{2 \\times 4}+\\sqrt{4 \\times 6}+ \\sqrt{6 \\times 5}+\\sqrt{5 \\times 3}+\\sqrt{3 \\times} \\overline{1}$, 故上述取法是符合要求的.\n(2) 下面证明: 依次按半径为 $6 、 1 、 4 、 3 、 2 、 5$ 或 $5 、 2 、 3 、 4 、 1 、 6$ 的顺序排时, 将使 $M=T_1 T_6$ 的值最小.\n求 $T_1 T_6=2\\left(\\sum_{i=1}^6 \\sqrt{r_i r_{i+1}}-\\sqrt{r_n r_{n+1}}\\right)\\left(r_1 \\sim r_6\\right.$ 的圆周排列断开 $)$ 时, 由于 $1 \\times 6 \\leqslant 2 \\times 3$, 易见 1 不能在两端.\n由对称性知需研究 1 在第 2 、第 3 个位置中的最小者即可.\n当 $\\left\\{r_2, r_3, \\cdots, r_6\\right\\}=\\{2,3, \\cdots, 6\\}$ 时, 排法 $r_2, r_3, 1, r_4, r_5, r_6$ 的 $T_1 T_6=\\sqrt{r_2 r_3}+\\sqrt{r_3}+\\sqrt{r_4}+\\sqrt{r_4 r_5}+\\sqrt{r_5 r_6}$, 设 $\\{a, b\\} \\subset\\{2,3,4,5,6\\}$ 且 $ar_3$ 时 $T_1 T_6$ 较小;\n(ii) 当 $r_2 、 r_3 、 r_6$ 固定时, 设 $\\left\\{r_4, r_5\\right\\}=\\{a, b\\}$, 相应 $T_1 T_6$ 值之差为 $\\sqrt{a}+\\sqrt{b r_6}-\\sqrt{b}-\\sqrt{a r_6}=(\\sqrt{b}-\\sqrt{a})\\left(\\sqrt{r_6}-1\\right)>0$, 从而 $r_4>r_5$ 时 $T_1 T_6$ 值较小.\n这时, 设 $\\left\\{r_4, r_5, r_6\\right\\}=\\{a, b, c\\}$, 且 $a\\sqrt{c}+\\sqrt{c a}+\\sqrt{a b}>\\sqrt{b}+\\sqrt{b a}+\\sqrt{a c}$, 故 $r_6> r_4>r_5$ 时, $T_1 T_6$ 值较小.\n(iii) 当 $r_3 、 r_4 、 r_5$ 固定时, 设 $\\left\\{r_2, r_6\\right\\}=\\{a, b\\}$, 相应 $T_1 T_6$ 值之差为 $\\sqrt{a r_3}+\\sqrt{r_5 b}-\\sqrt{b r_3}-\\sqrt{r_5 a}=(\\sqrt{b}-\\sqrt{a})\\left(\\sqrt{r_5}-\\sqrt{r_3}\\right)$.\n由此知当 $r_5>r_3$ 且 $r_2>r_6$, 或 $r_5r_6>r_4>r_5>r_3$ 或 $r_6>r_4>r_5, r_6>r_2>r_3>r_5$ 时, $T_1 T_6$ 值较小, 它们分别对应如下排法:\n(a) $6,2,1,4,3,5$;\n(b) $4,3,1,5,2,6$;\n(c) $5,3,1,4,2,6$; (d) $5,4,1,3,2,6$.\n对应 $T_1 T_6$ 的值分别为 $M_1=4 \\sqrt{3}+\\sqrt{2}+2+\\sqrt{15}, M_2=5 \\sqrt{3}+\\sqrt{5}+ \\sqrt{10}, M_3=\\sqrt{15}+3 \\sqrt{3}+2+2 \\sqrt{2}, M_4=2 \\sqrt{5}+2+\\sqrt{6}+3 \\sqrt{3}$.\n此时 $M_3$ 最小, 即以排法 $5,3,1,4,2,6$ 时 $T_1 T_6$ 值最小.\n下面再研究排法 $r_2, 1, r_3, r_4, r_5, r_6$ 的 $T_1 T_6$ 值.\n此时,\n$$\nT_1 T_6=\\sqrt{r_2}+\\sqrt{r_3}+\\sqrt{r_3 r_4}+\\sqrt{r_4 r_5}+\\sqrt{r_5 r_6} .\n$$\n用同样的方法可得在 $r_2>r_6>r_3>r_4>r_5$ 或 $r_2>r_3>r_6>r_5>r_4$ 时 $T_1 T_6$ 值较小, 相应排法为:\n(a) $6,1,4,3,2,5$;\n(b) $6,1,5,2,3,4$.\n对应 $T_1 T_6$ 值分别为 $M_1=2 \\sqrt{6}+2+2 \\sqrt{3}+\\sqrt{10}0, y>0$.\n$$\n\\begin{aligned}\n\\text { 左式 } & =\\frac{y x(x+y)}{(2 x+y+2 c)(x+2 c)(x+y+2 c)} \\\\\n& <\\frac{x^2 y+y^2 x}{(2 x+3 y)(x+2 y)(x+3 y)} \\\\\n& =\\frac{x^2 y+y^2 x}{2 x^3+13 x^2 y+27 x y^2+18 y^3} .\n\\end{aligned}\n$$\n令 $x=k y, k>0$, 则\n$$\n\\begin{aligned}\n& 2 x^3+13 x^2 y+27 x y^2+18 y^3 \\\\\n= & 2 x^2 \\cdot k y+13 x^2 y+22 x y^2+\\frac{5}{k} x^2 y+\\frac{18}{k^2} x^2 y \\\\\n= & \\left(2 k+\\frac{5}{k}+\\frac{18}{k^2}\\right) x^2 y+13 x^2 y+22 x y^2 \\\\\n\\geqslant & 5 \\cdot \\sqrt[5]{\\left(\\frac{2}{3} k\\right)^3 \\cdot \\frac{5}{k} \\cdot \\frac{18}{k^2}} \\cdot x^2 y+13 x^2 y+22 x y^2 \\\\\n> & 22\\left(x^2 y+y^2 x\\right),\n\\end{aligned}\n$$\n故原不等式成立.", + "remark": "", + "figures": [] +} \ No newline at end of file diff --git a/processed_dataset/proof/1497.json b/processed_dataset/proof/1497.json new file mode 100644 index 0000000000000000000000000000000000000000..4c364de85bde758e139443c350e592e7c2c100a3 --- /dev/null +++ b/processed_dataset/proof/1497.json @@ -0,0 +1,8 @@ +{ + "source_file": "./raw_volume-zh/volume5/exercise9.tex", + "problem_type": "proof", + "problem": "问题13. (1) 求证:对任意实数 $p 、 q$, 有 $p^2+q^2+1>p(q+1)$;\n(2)求最大的实数 $b$, 使得对任意实数 $p 、 q$, 都有 $p^2+q^2+1>b p(q+1)$;\n(3)求最大的实数 $c$, 使得对任意整数 $p 、 q$, 都有 $p^2+q^2+1>c p(q+1)$.", + "solution": "(1) $p^2+q^2+1>p(q+1)$ 等价于 $\\left(q-\\frac{p}{2}\\right)^2+\\left(\\frac{p}{2}-1\\right)^2+\\frac{p^2}{2}>0$, 显然成立.\n(2) 令 $p=\\sqrt{2}, q=1$, 则 $b \\leqslant \\sqrt{2}$, 下证不等式 $p^2+q^2+1 \\geqslant \\sqrt{2} p(q+1)$ 对所有实数 $p 、 q$ 都成立.\n事实上, 由 $\\left(\\frac{p}{\\sqrt{2}}-q\\right)^2+\\left(\\frac{p}{\\sqrt{2}}-1\\right)^2 \\geqslant 0$, 即得此不等式成立, 因而 $b_{\\max }=\\sqrt{2}$.\n(3) 令 $p=q=1$, 有 $c \\leqslant \\frac{3}{2}$. 下证不等式 $p^2+q^2+1 \\geqslant \\frac{3}{2} p(q+1)$ 对所有整数 $p 、 q$ 都成立.\n上式等价于 $(3 p-4 q)^2+\\left(7 p^2-24 p+16\\right) \\geqslant 0$.\n由于 $p \\geqslant 3$ 或 $p \\leqslant 0$ 时, $7 p^2-24 p+16 \\geqslant 0$, 故结论成立.\n当 $p=1,2$ 时,也不难验证结论成立,故 $c_{\\text {max }}=\\frac{3}{2}$.", + "remark": "", + "figures": [] +} \ No newline at end of file diff --git a/processed_dataset/proof/1498.json b/processed_dataset/proof/1498.json new file mode 100644 index 0000000000000000000000000000000000000000..ca87036c8e3eaa677185b7f6bb5d94590c5f9df6 --- /dev/null +++ b/processed_dataset/proof/1498.json @@ -0,0 +1,8 @@ +{ + "source_file": "./raw_volume-zh/volume5/exercise9.tex", + "problem_type": "proof", + "problem": "问题14. (1) 设 $a 、 b 、 c$ 为 $\\triangle A B C$ 的三边长, $n \\geqslant 2$ 为整数, 求证:\n$$\n\\frac{\\sqrt[n]{a^n+b^n}+\\sqrt[n]{b^n+c^n}+\\sqrt[n]{c^n+a^n}}{a+b+c}<1+\\frac{\\sqrt[n]{2}}{2} .\n$$\n(2)设 $a 、 b 、 c$ 为三角形的三边长, 求最小正实数 $k$, 使得:\n$$\n\\frac{\\sqrt[3]{a^3+b^3}+\\sqrt[3]{b^3+c^3}+\\sqrt[3]{c^3+a^3}}{(\\sqrt{a}+\\sqrt{b}+\\sqrt{c})^2}c$. 原不等式等价于\n$$\n\\sqrt[n]{a^n+b^n}+\\sqrt[n]{b^n+c^n}+\\sqrt[n]{c^n+a^n}<(a+b+c)+\\frac{\\sqrt[n]{2}}{2}(a+b+c) .\n$$\n由于 $\\frac{\\sqrt[n]{2}}{2}(a+b+c)>\\sqrt[n]{2} \\cdot c=\\sqrt[n]{c^n+c^n} \\geqslant \\sqrt{b^n+c^n}$ ,\n又不难证明 $\\sqrt[n]{a^n+b^n} \\leqslant \\frac{a}{2}+b$ 及 $\\sqrt[n]{c^n+a^n} \\leqslant \\frac{a}{2}+c$, 相加即得原不等式成立.\n(2) 令 $a=b, c \\rightarrow 0$, 有左边 $\\rightarrow \\frac{2+\\sqrt[3]{2}}{4}$, 故 $k \\geqslant \\frac{2+\\sqrt[3]{2}}{4}$.\n下证: 不等式左端 $<\\frac{2+\\sqrt[3]{2}}{4}$.\n首先注意到, 若 $a 、 b 、 c$ 为三角形三边长, 则 $\\sqrt{a} 、 \\sqrt{b} 、 \\sqrt{c}$ 也为三角形的三边长.\n不妨设 $\\sqrt{a}+\\sqrt{b}+\\sqrt{c}=2$, 且 $x=\\sqrt{a}, y=\\sqrt{b}, z=\\sqrt{c}$, 则 $x+y+z= 2, x, y, z \\in(0,1)$.\n下证: $\\quad \\sqrt[3]{x^6+y^6}+\\sqrt[3]{y^6+z^6}+\\sqrt[3]{z^6+x^6}<2+\\sqrt[3]{2}, \\label{eq1}$.\n由于 $x, y, z<1$, 故 式\\ref{eq1} 左边 $<\\sqrt[3]{x^3+y^3}+\\sqrt[3]{y^3+z^3}+\\sqrt[3]{z^3+x^3}$, 而当 $x \\geqslant y>0$ 时, 不难证明: $x^3+y^3 \\leqslant[x+(\\sqrt[3]{2}-1) y]^3$.\n现在设 $x \\geqslant y \\geqslant z$, 则\\ref{eq1}式\n$$\n\\begin{aligned}\n\\text { 左边 } & \\leqslant x+(\\sqrt[3]{2}-1) y+y+(\\sqrt[3]{2}-1) z+x+(\\sqrt[3]{2}-1) z \\\\\n& =2 x+\\sqrt[3]{2} y+2(\\sqrt[3]{2}-1) z(\\text { 希望 } x, y \\rightarrow 1, z \\rightarrow 0) \\\\\n& =2 x+\\sqrt[3]{2} y+(2 \\sqrt[3]{2}-2)(2-x-y) \\\\\n& =4 \\sqrt[3]{2}-4+(4-2 \\sqrt[3]{2}) x+(2-\\sqrt[3]{2}) y \\\\\n& \\leqslant 4 \\sqrt[3]{2}-4+4-2 \\sqrt[3]{2}+2-\\sqrt[3]{2} \\\\\n& =2+\\sqrt[3]{2}\n\\end{aligned}\n$$\n因此式\\ref{eq1}成立.", + "remark": "", + "figures": [] +} \ No newline at end of file diff --git a/processed_dataset/proof/1499.json b/processed_dataset/proof/1499.json new file mode 100644 index 0000000000000000000000000000000000000000..70de058907157700ea457be45754b37009831fe4 --- /dev/null +++ b/processed_dataset/proof/1499.json @@ -0,0 +1,8 @@ +{ + "source_file": "./raw_volume-zh/volume6/chapter1-1.tex", + "problem_type": "proof", + "problem": "例1. 证明: 对任意 $n \\in \\mathbf{N}^*$, 都有\n$$\n\\frac{1}{1 \\times 2}+\\frac{1}{2 \\times 3}+\\cdots+\\frac{1}{n(n+1)}=1-\\frac{1}{n+1} . \\label{eq1}\n$$", + "solution": "证明:当 $n=1$ 时, \\ref{eq1} 式左边 $=\\frac{1}{2}$, \\ref{eq1} 式右边 $=1-\\frac{1}{1+1}=\\frac{1}{2}$, 故 $n=$ 1 时, \\ref{eq1} 式成立.\n现设\\ref{eq1}式对 $n$ 成立, 考虑 $n+1$ 的情形.\n利用 $\\frac{1}{k(k+1)}=\\frac{1}{k}-\\frac{1}{k+1}$, 知\n$$\n\\begin{aligned}\n& \\frac{1}{1 \\times 2}+\\frac{1}{2 \\times 3}+\\cdots+\\frac{1}{(n+1)(n+2)} \\\\\n= & \\left(1-\\frac{1}{2}\\right)+\\left(\\frac{1}{2}-\\frac{1}{3}\\right)+\\cdots+\\left(\\frac{1}{n+1}-\\frac{1}{n+2}\\right) \\\\\n= & 1-\\frac{1}{n+2} .\n\\end{aligned} \\label{eq2}\n$$\n所以, \\ref{eq1}式对 $n+1$ 成立.\n综上所述, 由数学归纳法原理知, \\ref{eq1}式对一切正整数 $n$ 成立.\n说明这是一个错误的证明, 其错误在于证明\\ref{eq1}式对 $n+1$ 成立时, 并没有用到归纳假设.\n正确的过程如下:\n由归纳假设知\n$$\n\\begin{aligned}\n& \\frac{1}{1 \\times 2}+\\frac{1}{2 \\times 3}+\\cdots+\\frac{1}{n(n+1)}+\\frac{1}{(n+1)(n+2)} \\\\\n= & \\left(1-\\frac{1}{n+1}\\right)+\\frac{1}{(n+1)(n+2)} \\\\\n= & \\left(1-\\frac{1}{n+1}\\right)+\\left(\\frac{1}{n+1}-\\frac{1}{n+2}\\right) \\\\\n= & 1-\\frac{1}{n+2} .\n\\end{aligned}\n$$\n所以, \\ref{eq1}式对 $n+1$ 成立.\n事实上, \\ref{eq2}式的得到是正确的, 但这是对\\ref{eq1}式的一个直接证明, 不应该套上数学归纳法这顶帽子.", + "remark": "", + "figures": [] +} \ No newline at end of file diff --git a/processed_dataset/proof/1500.json b/processed_dataset/proof/1500.json new file mode 100644 index 0000000000000000000000000000000000000000..61589eae67e7a153ff1836c608f5b822b8e5377c --- /dev/null +++ b/processed_dataset/proof/1500.json @@ -0,0 +1,11 @@ +{ + "source_file": "./raw_volume-zh/volume6/chapter1-1.tex", + "problem_type": "proof", + "problem": "例2. 设 $n \\in \\mathbf{N}^*$. 证明: 去掉 $2^n \\times 2^n$ 的方格表的任何一个方格后, 剩余的部分都可以用\" $\\square$ \" \"形状的 L 型无重叠地完全覆盖.", + "solution": "证明:当 $n=1$ 时, 由于一个\" $\\square$ \" 字型去掉任何一个方格后都是一个 \"回\"型,故命题对 $n=1$ 成立.\n现设 $n=k$ 时, 命题成立, 即去掉一个 $2^k \\times 2^k$ 的方格表的任何一个方格后,剩余部分都可用\" $\\square$ 型覆盖, 我们考虑 $n=k+1$ 的情形.\n如图 () 所示, 将 $2^{k+1} \\times 2^{k+1}$ 的方格表依中心所在的两条方格线把表格分割为 4 个 $2^k \\times 2^k$ 的方格表, 则题设中要求去掉的那个小方格必落在某个 $2^k \\times 2^k$ 的方格表中.\n在剩余的部分先绕中心摆一个\" $\\square$ \" 型, 去掉图 () 中所示的 4 个阴影方格后, 每个 $2^k \\times 2^k$ 的子表格都去掉了一个方格, 而由归纳假设可知, 它们都可以用 \" $\\square$ \" 型覆盖, 再补上绕中心所摆的那个\" $\\square$ \" 型就得出命题对 $n=k+1$ 成立.\n综上可知, 命题对一切正整数 $n$ 成立.\n说明本题采用的是数学归纳法证题时的常用表述方式.\n当然了, 表达方式可依个人的表达风格而定, 但都需要在归纳假设和结论之间进行正确地过渡, 它是完成数学归纳法证题时的关键步骤.", + "remark": "", + "figures": [ + "./images/volume6/figures/fig-c1i1.png", + "./images/volume6/figures/fig-c1i1.png" + ] +} \ No newline at end of file diff --git a/processed_dataset/proof/1501.json b/processed_dataset/proof/1501.json new file mode 100644 index 0000000000000000000000000000000000000000..c0f6e90970d0fda042d07d685a47d008f3184659 --- /dev/null +++ b/processed_dataset/proof/1501.json @@ -0,0 +1,8 @@ +{ + "source_file": "./raw_volume-zh/volume6/chapter1-1.tex", + "problem_type": "proof", + "problem": "例3. 设 $x 、 y$ 是实数, 使得 $x+y 、 x^2+y^2 、 x^3+y^3 、 x^4+y^4$ 都是整数.\n证明: 对任意 $n \\in \\mathbf{N}^*$, 数 $x^n+y^n$ 都为整数.", + "solution": "证明:此题要用到第一数学归纳法的一种变形: 设 $P(n)$ 是关于正整数 $n$ 的一个命题(或性质), 如果\n(1) 当 $n=1,2$ 时, $P(n)$ 成立;\n(2) 由 $P(n) 、 P(n+1)$ 成立可以推出 $P(n+2)$ 成立.\n那么, 对任意 $n \\in \\mathbf{N}^*, P(n)$ 都成立.\n事实上, 这种变形只是调整了归纳过程中的跨度, 这样的例子在后面的讨论中会频繁出现.\n回到原题, 由条件 $x+y$ 与 $x^2+y^2$ 都是整数可知,命题对 $n=1,2$ 成立.\n设命题对 $n, n+1$ 成立, 即 $x^n+y^n$ 与 $x^{n+1}+y^{n+1}$ 都是整数, 考虑 $n+2$ 的情形.\n此时\n$$\nx^{n+2}+y^{n+2}=(x+y)\\left(x^{n+1}+y^{n+1}\\right)-x y\\left(x^n+y^n\\right) .\n$$\n因此, 为证 $x^{n+2}+y^{n+2} \\in \\mathbf{Z}$, 结合归纳假设及条件中的 $x+y \\in \\mathbf{Z}$, 我们只需证明 $x y \\in \\mathbf{Z}$.\n注意到 $x+y 、 x^2+y^2 \\in \\mathbf{Z}$, 故 $2 x y=(x+y)^2-\\left(x^2+y^2\\right) \\in \\mathbf{Z}$. 若 $x y \\notin \\mathbf{Z}$, 则可设 $x y=\\frac{m}{2}, m$ 为奇数, 再由 $x^2+y^2 、 x^4+y^4 \\in \\mathbf{z}$, 知 $2 x^2 y^2=\\left(x^2+\\right. \\left.y^2\\right)^2-\\left(x^4+y^4\\right) \\in \\mathbf{Z}$, 即 $2 \\times\\left(\\frac{m}{2}\\right)^2=\\frac{m^2}{2} \\in \\mathbf{Z}$. 但 $m$ 为奇数, 这是一个矛盾.\n所以 $x y \\in \\mathbf{Z}$. 进而, 命题对 $n+2$ 成立.\n综上所述, 对任意 $n \\in \\mathbf{N}^*$, 数 $x^n+y^n \\in \\mathbf{Z}$.", + "remark": "", + "figures": [] +} \ No newline at end of file diff --git a/processed_dataset/proof/1502.json b/processed_dataset/proof/1502.json new file mode 100644 index 0000000000000000000000000000000000000000..c3f4e3d5d425d164f7ceafe4c2fff9d0c043e418 --- /dev/null +++ b/processed_dataset/proof/1502.json @@ -0,0 +1,8 @@ +{ + "source_file": "./raw_volume-zh/volume6/chapter1-1.tex", + "problem_type": "proof", + "problem": "例4. 设 $\\theta \\in\\left(0, \\frac{\\pi}{2}\\right), n$ 是大于 1 的正整数.\n证明:\n$$\n\\left(\\frac{1}{\\sin ^n \\theta}-1\\right)\\left(\\frac{1}{\\cos ^n \\theta}-1\\right) \\geqslant 2^n-2^{\\frac{n}{2}+1}+1 . \\label{eq1}\n$$", + "solution": "证明:当 $n=2$ 时,\\ref{eq1}式左右两边相等,故 $n=2$ 时命题成立.\n假设命题对 $n(\\geqslant 2)$ 成立, 则\n$$\n\\begin{aligned}\n& \\left(\\frac{1}{\\sin ^{n+1} \\theta}-1\\right)\\left(\\frac{1}{\\cos ^{n+1} \\theta}-1\\right) \\\\\n= & \\frac{1}{\\sin ^{n+1} \\theta \\cos ^{n+1} \\theta}\\left(1-\\sin ^{n+1} \\theta\\right)\\left(1-\\cos ^{n+1} \\theta\\right) \\\\\n= & \\frac{1}{\\sin ^{n+1} \\theta \\cos ^{n+1} \\theta}\\left(1-\\sin ^{n+1} \\theta-\\cos ^{n+1} \\theta\\right)+1 \\\\\n= & \\frac{1}{\\sin \\theta \\cos \\theta}\\left(\\frac{1}{\\sin ^n \\theta \\cos ^n \\theta}-\\frac{\\cos \\theta}{\\sin ^n \\theta}-\\frac{\\sin \\theta}{\\cos ^n \\theta}\\right)+1 \\\\\n= & \\frac{1}{\\sin \\theta \\cos \\theta}\\left[\\left(\\frac{1}{\\sin ^n \\theta}-1\\right)\\left(\\frac{1}{\\cos ^n \\theta}-1\\right)+\\frac{1-\\cos \\theta}{\\sin ^n \\theta}+\\frac{1-\\sin \\theta}{\\cos ^n \\theta}-1\\right]+1 \\\\\n\\geqslant & \\frac{1}{\\sin \\theta \\cos \\theta}\\left[\\left(2^n-2^{\\frac{n}{2}+1}\\right)+2 \\sqrt{\\frac{(1-\\cos \\theta)(1-\\sin \\theta)}{\\sin ^n \\theta \\cos ^n \\theta}}\\right]+1,\n\\end{aligned} \\label{eq2}\n$$\n这里式\\ref{eq2}由归纳假设和均值不等式得到.\n注意到 $\\sin \\theta \\cos \\theta=\\frac{1}{2}-\\sin 2 \\theta \\leqslant \\frac{1}{2}$, 而\n$$\n\\frac{(1-\\cos \\theta)(1--\\sin \\theta)}{\\sin ^n \\theta \\cos ^n \\theta}=\\left(\\frac{1}{\\sin \\theta \\cos \\theta}\\right)^{n-2} \\cdot \\frac{1}{(1+\\sin \\theta)(1+\\cos \\theta)},\n$$\n其中 $\\quad(1+\\sin \\theta)(1+\\cos \\theta)=1+\\sin \\theta+\\cos \\theta+\\sin \\theta \\cos \\theta$\n$$\n\\begin{aligned}\n& =1+t+\\frac{t^2-1}{2} \\\\\n& =\\frac{1}{2}(t+1)^2 \\leqslant \\frac{1}{2}(\\sqrt{2}+1)^2 .\n\\end{aligned}\n$$\n(这里用到 $t=\\sin \\theta+\\cos \\theta=\\sqrt{2} \\sin \\left(\\theta+\\frac{\\pi}{4}\\right) \\in(1, \\sqrt{2}]$ ).\n所以 $\\sqrt{\\frac{(1-\\cos \\theta)(1-\\sin \\theta)}{\\sin ^n \\theta \\cos ^n \\theta}} \\geqslant \\frac{2^{\\frac{n-1}{2}}}{\\sqrt{2}+1}=2^{\\frac{n}{2}}-2^{\\frac{n-1}{2}}$. 从而, 由式\\ref{eq2}可得\n$$\n\\begin{aligned}\n& \\left(\\frac{1}{\\sin ^{n+1} \\theta}-1\\right)\\left(\\frac{1}{\\cos ^{n+1} \\theta}-1\\right) \\geqslant 2\\left[\\left(2^n-2^{\\frac{n}{2}+1}\\right)+2\\left(2^{\\frac{n}{2}}-2^{\\frac{n-1}{2}}\\right)\\right]+1 \\\\\n= & 2\\left(2^n-2^{\\frac{n+1}{2}}\\right)+1=2^{n+1}-2^{\\frac{n+1}{2}+1}+1 .\n\\end{aligned}\n$$\n即命题对 $n+1$ 成立.\n综上所述,命题对一切 $n \\in \\mathbf{N}^*(n \\geqslant 2)$ 成立.", + "remark": "", + "figures": [] +} \ No newline at end of file diff --git a/processed_dataset/proof/1503.json b/processed_dataset/proof/1503.json new file mode 100644 index 0000000000000000000000000000000000000000..374d8f6d2c2431f5899500e2ee8f3be719a7cf59 --- /dev/null +++ b/processed_dataset/proof/1503.json @@ -0,0 +1,8 @@ +{ + "source_file": "./raw_volume-zh/volume6/chapter1-1.tex", + "problem_type": "proof", + "problem": "例5. 数列 $\\left\\{a_n\\right\\}$ 定义如下:\n$$\na_1=1, a_n=a_{n-1}+a_{\\left[\\frac{n}{2}\\right]}, n=2,3, \\cdots . \n$$\n证明: 该数列中有无穷多项是 7 的倍数.", + "solution": "证明:直接由递推式计算, 可得 $a_1=1, a_2=2, a_3=3, a_4=5, a_5=7$.\n现设 $a_n(n \\geqslant 5)$ 是 7 的倍数, 我们寻找下标 $m>n$, 使得 $7 \\mid a_m$.\n由 $a_n \\equiv 0(\\bmod 7)$, 知 $a_{2 n}=a_{2 n-1}+a_n \\equiv a_{2 n-1}(\\bmod 7), a_{2 n+1}=a_{2 n}+a_n \\equiv a_{2 n}(\\bmod 7)$, 故 $a_{2 n-1} \\equiv a_{2 n} \\equiv a_{2 n+1}(\\bmod 7)$. 记 $a_{2 n-1}$ 除以 7 所得余数为 $r$. 如果 $r=0$, 那么取 $m=2 n-1$ 即可; 如果 $r \\neq 0$, 考虑下面的 7 个数:\n$$\na_{4 n-3}, a_{4 n-2}, \\cdots, a_{4 n+3} \\text {. }\n$$\n注意到 $a_{4 n-2}=a_{4 n-3}+a_{2 n-1} \\equiv a_{4 n-3}+r(\\bmod 7), a_{4 n-1}=a_{4 n-2}+a_{2 n-1} \\equiv a_{4 n-2}+r(\\bmod 7) \\equiv a_{4 n-3}+2 r(\\bmod 7), a_{4 n}=a_{4 n-1}+a_{2 n} \\equiv a_{4 n-1}+r \\equiv a_{4 n-3}+ 3 r, \\cdots, a_{4 n+3}=a_{4 n+2}+a_{2 n+1} \\equiv a_{4 n+2}+r \\equiv a_{4 n-3}+6 r(\\bmod 7)$. 因此, $a_{4 n-3}$, $a_{4 n-2}, \\cdots, a_{4 n+3}$ 构成模 7 的一个完全剩余系.\n故存在 $m \\in\\{4 n-3,4 n-2, \\cdots$, $4 n+3\\}$, 使得 $a_m \\equiv 0(\\bmod 7)$.\n这样, 我们从 $a_5$ 出发结合上面的推导可知, $\\left\\{a_n\\right\\}$ 中有无穷多项是 7 的倍数.", + "remark": "", + "figures": [] +} \ No newline at end of file diff --git a/processed_dataset/proof/1504.json b/processed_dataset/proof/1504.json new file mode 100644 index 0000000000000000000000000000000000000000..c22c377f9e369e1eca1050171c2fb9434119c769 --- /dev/null +++ b/processed_dataset/proof/1504.json @@ -0,0 +1,8 @@ +{ + "source_file": "./raw_volume-zh/volume6/chapter1-1.tex", + "problem_type": "proof", + "problem": "例6. (1) 证明: 对任意正整数 $n(\\geqslant 2)$, 存在 $n$ 个不同的正整数 $a_1, \\cdots$, $a_n$, 使得对任意 $1 \\leqslant i1$, 有 $\\left(a_j-a_1\\right) \\mid\\left(a_j+a_1\\right)$, 故 $\\left(a_j-a_1\\right) \\mid 2 a_1$. 而由 $a_1\n$$\nc_1 \\leqslant 2 \\text {, 而当 } i \\geqslant 2 \\text { 时, } c_i \\leqslant 4 \\times 3^{i-2} . \n$$\n事实上, 若 $c_1>1$, 取 $(m, n)=\\left(c_1-1,1\\right)$, 知存在 $a_1 \\in \\mathbf{N}^*$, 使得 $c_1- 1=\\frac{c_1}{a_1}$, 即 $a_1=\\frac{c_1}{c_1-1}=1+\\frac{1}{c_1-1}$, 仅当 $c_1=2$ 时, $a_1$ 为整数, 故 $c_1 \\leqslant 2$.\n现设<1>对 $i=1,2, \\cdots, k-1(k \\geqslant 2)$ 都成立, 取 $(m, n)=\\left(c_k, k\\right)$, 则存在 $a_1, \\cdots, a_k \\in \\mathbf{N}^*$, 使得 $c_k=\\frac{c_1}{a_1}+\\cdots+\\frac{c_k}{a_k}$. 这要求 $a_k \\geqslant 2$, 否则 $\\sum_{i=1}^{k-1} \\frac{c_i}{a_i}=$ 0 与 $a_i 、 c_i$ 为正整数矛盾.\n从而 $c_k \\leqslant \\frac{c_k}{2}+\\sum_{i=1}^{k-1} c_i$, 即 $c_k \\leqslant 2 \\sum_{i=1}^{k-1} c_i$. 所以 $c_k \\leqslant 2\\left(2+4+4 \\times 3+\\cdots+4 \\times 3^{k-3}\\right)=4 \\times 3^{k-2}$. 因此, 由第二数学归纳法知, 结论<1>成立.\n再证:\n<2> 当 $c_1=2, c_i=4 \\times 3^{i-2}(i \\geqslant 2)$ 时, 数列 $\\left\\{c_i\\right\\}$ 具有题给的性质.\n对 $n$ 归纳.\n当 $n=1$ 时, $m \\leqslant c_1=2$, 故 $m=1$ 或 2 . 若 $m=1$, 取 $a_1=2$ 即可,若 $m=2$, 取 $a_1=1$ 即可.\n假设当 $1,2, \\cdots, n-1$ 时, 题给性质满足.\n考虑 $n$ 的情形, 此时 $1 \\leqslant m \\leqslant \\sum_{i=1}^n c_i$.\n若 $m=1$, 取 $a_i=n c_i, i=1,2, \\cdots, n$ 即可;\n若 $2 \\leqslant m \\leqslant \\frac{c_n}{2}+1=\\left(\\sum_{i=1}^{n-1} c_i\\right)+1$, 令 $a_n=c_n$, 并对 $m-\\frac{c_n}{a_n}=m-1$ 用归纳假设, 可知<2>成立;\n若 $\\frac{1}{2} c_n+1成立.\n综上可知, $c_1$ 的最大值为 2 , 而当 $i \\geqslant 2$ 时, $c_i$ 的最大值是 $4 \\times 3^{i-2}$.\n说明对比两个例子可发现, 用第二数学归纳法证题时, 一个思路是整体处理: 例 1 中对归纳假设中的 $n-1$ 个不等式求和; 另一个思路是将 $n$ 的情形归人 $1,2, \\cdots, n-1$ 中的某一种情形,这在例 2 的后半部分有明显的体现.", + "remark": "", + "figures": [] +} \ No newline at end of file diff --git a/processed_dataset/proof/1507.json b/processed_dataset/proof/1507.json new file mode 100644 index 0000000000000000000000000000000000000000..057db7fe63e8d54d6777bfd8940e2bd560e75b35 --- /dev/null +++ b/processed_dataset/proof/1507.json @@ -0,0 +1,8 @@ +{ + "source_file": "./raw_volume-zh/volume6/chapter1-2.tex", + "problem_type": "proof", + "problem": "例3. 设 $p(x)$ 是一个 $n$ 次实系数多项式, $a$ 是一个不小于 3 的实数.\n证明:下面的 $n+2$ 个数中至少有一个数不小于 1 .\n$$\n\\left|a^0-p(0)\\right|,\\left|a^1-p(1)\\right|, \\cdots,\\left|a^{n+1}-p(n+1)\\right| .\n$$", + "solution": "证明:对 $p(x)$ 的次数 $n$ 进行归纳.\n当 $n=0$ 时, $p(x)$ 是常数多项式, 设 $p(x)=c$, 此时, 由 $|1-c|+\\mid a- c|\\geqslant| a-1 \\mid \\geqslant 2$, 可知 $\\max \\{|1-c|,|a-c|\\} \\geqslant 1$, 即命题对 $n=0$ 成立.\n假设命题对所有次数小于 $n$ 的多项式都成立, 考虑次数为 $n$ 的多项式 $p(x)$.\n令 $f(x)=\\frac{1}{a-1}[p(x+1)-p(x)]$, 则 $f(x)$ 的次数 $\\leqslant n-1$. 由归纳假设知, 存在 $m \\in\\{0,1,2, \\cdots, n\\}$, 使得 $\\left|a^m-f(m)\\right| \\geqslant 1$, 即 $\\mid a^m-\\frac{1}{a-1}[p(m+$ 1) $-p(m)] \\mid \\geqslant 1$. 故\n$$\n\\left|a^{m+1}-p(m+1)+p(m)-a^m\\right| \\geqslant a-1 \\geqslant 2,\n$$\n从而 $\\max \\left\\{\\left|a^{m+1}-p(m+1)\\right|,\\left|a^m-p(m)\\right|\\right\\} \\geqslant 1$, 即存在 $r \\in\\{0,1,2, \\cdots$, $n+1\\}$, 使得 $\\left|a^r--p(r)\\right| \\geqslant 1$, 命题对 $n$ 成立.\n综上可知, 对任意次数为 $n$ 的多项式 $p(x)$, 命题都成立.\n说明在对多项式的次数用数学归纳法时, 常采用第二数学归纳法的形式,因为首项系数相同的两个 $n$ 次多项式之差的次数不一定是 $n-1$ 次, 但一定是一个次数小于 $n$ 的多项式.\n运用第二数学归纳法处理时就避开了讨论.", + "remark": "", + "figures": [] +} \ No newline at end of file diff --git a/processed_dataset/proof/1508.json b/processed_dataset/proof/1508.json new file mode 100644 index 0000000000000000000000000000000000000000..862150929b091823da5d3bb286778c5464e2a081 --- /dev/null +++ b/processed_dataset/proof/1508.json @@ -0,0 +1,11 @@ +{ + "source_file": "./raw_volume-zh/volume6/chapter1-2.tex", + "problem_type": "proof", + "problem": "例4. 证明: 任意一个凸 $n$ 边形都可以被它的三条边张成的三角形或它的四条边张成的平行四边形所覆盖.", + "solution": "证明:对 $n$ 归纳.\n当 $n=3$ 时,结论是显然的; 当 $n=4$ 时, 如果该四边形是平行四边形则已完成, 如果它不是平行四边形, 则有一组对边不平行, 将它们延长相交后, 总可以用另两条边中的一条合成一个三角形, 它覆盖这个四边形 (如图 () 所示).\n现假设对任一凸 $m$ 边形结论成立, 这里 $m) 所示的图形), 设它们相交于点 $U$. 现在用折线 $B U C$ 代替被 $\\angle B U C$ 覆盖的折线 $A D$ 及边 $B A$ 和 $C D$, 便得到一个边数少于 $n$ 并将 $M$ 覆盖的凸多边形 $M_1$, 对 $M_1$ 用归纳假设, 可知命题对 $n$ 成立.\n综上可知, 命题成立.\n说明数学归纳法在平面几何中也有广泛的应用.\n此题的结论可进一步加强为: 若凸 $n$ 边形不是平行四边形, 则它可被由其三条边张成的三角形所覆盖.", + "remark": "", + "figures": [ + "./images/volume6/figures/fig-c1i2.png", + "./images/volume6/figures/fig-c1i3.png" + ] +} \ No newline at end of file diff --git a/processed_dataset/proof/1509.json b/processed_dataset/proof/1509.json new file mode 100644 index 0000000000000000000000000000000000000000..f350f2fba0d2c011cec857768b2df453f7182c6e --- /dev/null +++ b/processed_dataset/proof/1509.json @@ -0,0 +1,8 @@ +{ + "source_file": "./raw_volume-zh/volume6/chapter1-2.tex", + "problem_type": "proof", + "problem": "例5. 设 $a_1, a_2, \\cdots, a_n$ 为一个倒三角形的第 1 行, 其中 $a_i \\in\\{0,1\\}, i= 1,2, \\cdots, n$. 数 $b_1, b_2, \\cdots, b_{n-1}$ 为这个倒三角形的第 2 行, 使得若 $a_k=a_{k+1}$, 则 $b_k=0$; 若 $a_k \\neq a_{k+1}$, 则 $b_k=1, k=1,2, \\cdots, n-1$. 类似定义该倒三角形的其余各行,直到第 $n$ 行为止.\n求该三角形中 1 的个数的最大值.", + "solution": "解:我们设该三角形中 1 的个数的最大值为 $f_n$. 容易得到 $f_1=1, f_2= 2, f_3=4$. 例子为\n$$\n\\begin{array}{cccccc}\n1, & 1 & 1 & 1 & 0 \\\\\n0 & & 0 & 1 \\\\\n& & & 1\n\\end{array} .\n$$\n得到上述值可以从表中第一行内 0 的个数出发, 但是随着 $n$ 变大时, 难以从第一行出发来处理.\n试着做 $n=5,6$ 时的情形,可以发现下面的表中 1 的个数比较多.\n上表中有一个特点, 即每三行重复出现 (只是 \"规模\" 小一些). 于是, 引导我们利用数学归纳法来求 $f_n$ 的值.\n先证明一个引理.\n引理当 $n \\geqslant 3$ 时, 考虑该倒三角形最上面的 3 行\n$$\n\\begin{gathered}\na_1, a_2, a_3, \\cdots, a_n \\\\\nb_1, b_2, \\cdots, b_{n-1} \\\\\nc_1, \\cdots, c_{n-2}\n\\end{gathered}\n$$\n则此 3 行中至少出现 $n-1$ 个 0 .\n证明对 $n$ 归纳予以证明.\n初始情况的验证留给读者, 我们来看如何实现归纳过渡.\n注意到, 在 $\\bmod 2$ 的意义下, 前 3 行为\n$$\n\\begin{gathered}\na_1, a_2, a_3, a_4, a_5, \\cdots, a_n \\\\\na_1+a_2, a_2+a_3, a_3+a_4, \\cdots, a_{n-1}+a_n \\\\\na_1+a_3, a_2+a_4, \\cdots, a_{n-2}+a_n\n\\end{gathered}\n$$\n如果 $a_1, a_1+a_2, a_1+a_3$ 不全为 1 , 那么去掉这 3 个数, 归为 $n-1$ 的情形,利用归纳假设可知,结论成立.\n如果 $a_1=a_1+a_2=a_1+a_3=1$, 那么 $a_1=1, a_2=a_3=0$, 此时,表格的前三行前面部分为\n其中被平行四边形框住的 9 个数 (前面的 3 个斜行) 中至少有 3 个 0 ,于是, 去掉这 9 个数后, 利用归纳假设可知,引理成立.\n由上述引理可知 $f_n \\leqslant 2(n-1)+f_{n-3}, n \\geqslant 4$. 结合 $f_1=1, f_2=2$, $f_3=4$, 可知 $f_n \\leqslant\\left\\lceil\\frac{n(n+1)}{3}\\right\\rceil$, 这里 $\\lceil x\\rceil$ 表示不小于 $x$ 的最小整数.\n利用前面的例子, 可知 $f_n=\\left\\lceil\\frac{n(n+1)}{3}\\right\\rceil$.\n所以,该倒三角形中, 1 的个数的最大值为 $\\left\\lceil\\frac{n(n+1)}{3}\\right\\rceil$.\n说明此题找到取最大值的例子是一个关键, 但经一定的尝试后不难得到.\n解答难在对每三行作为一个整体来进行处理不易想到, 它是从例子中得到启发后形成的思路.", + "remark": "", + "figures": [] +} \ No newline at end of file diff --git a/processed_dataset/proof/1510.json b/processed_dataset/proof/1510.json new file mode 100644 index 0000000000000000000000000000000000000000..4e6150a1ce24d16ab22099a51048fb78803ddb2d --- /dev/null +++ b/processed_dataset/proof/1510.json @@ -0,0 +1,8 @@ +{ + "source_file": "./raw_volume-zh/volume6/chapter1-2.tex", + "problem_type": "proof", + "problem": "例6. 设 $n \\in \\mathbf{N}^*$, 函数 $f:\\left\\{1,2,3, \\cdots, 2^{n-1}\\right\\} \\rightarrow \\mathbf{N}^*$ 满足: 对 $1 \\leqslant i \\leqslant 2^{n-1}$, 都有 $1 \\leqslant f(i) \\leqslant i$. 证明: 存在一个正整数数列 $a_1, a_2, \\cdots, a_n$, 使得 $1 \\leqslant a_12^{n-j-1}$, 则存在 $1 \\leqslant i_1f\\left(i_q\\right)$ (否则, 若 $f\\left(i_p\\right) \\leqslant f\\left(i_q\\right)$, 则在从 $i_q$ 出发的递增 $f$ 数列的前面加人 $f\\left(i_p\\right)$, 将导致 $t\\left(i_p\\right) \\geqslant t\\left(i_q\\right)+1$, 矛盾 $)$, 故 $f\\left(i_1\\right)> f\\left(i_2\\right)>\\cdots>f\\left(i_r\\right)$, 进而 $f\\left(i_1\\right) \\geqslant r=2^{n-j-1}+1$, 结合 $1 \\leqslant f\\left(i_1\\right) \\leqslant i_1$, 得 $i_1 \\geqslant 2^{n-j-1}+1$.\n现在, 由 $t\\left(i_1\\right)$ 的定义知, 存在 $i_1=a_1<\\cdots0$.\n如果式\\ref{eq2}有整数解 $\\left(a_0, b_0, c_0, m_0\\right), m_0>0$, 我们证明方程(2)有一组整数解 $\\left(a_1, b_1, c_1, m_1\\right), m_1>0$, 且 $m_1m_1>m_2>\\cdots$, 从而导致矛盾.\n事实上,若 $m_0$ 为奇数, 则 $m_0^2 \\equiv 1(\\bmod 8)$, 即 $a_0^2+b_0^2+c_0^2 \\equiv 7(\\bmod 8)$, 但是一个数的完全平方数 $\\equiv 0,1$ 或 $4(\\bmod 8)$, 从而 $a_0^2+b_0^2+c_0^2 \\equiv 0,1,2,3,4$, $5,6(\\bmod 8)$, 不会出现 $a_0^2+b_0^2+c_0^2 \\equiv 7(\\bmod 8)$ 的情形, 矛盾, 故 $m_0$ 为偶数.\n这时 $a_0^2+b_0^2+c_0^2=7 m_0^2 \\equiv 0(\\bmod 4)$, 又完全平方数 $\\equiv 0$ 或 $1(\\bmod 4)$, 故 $a_0$ 、 $b_0 、 c_0$ 都必须为偶数, 这样令 $a_1=\\frac{1}{2} a_0, b_1=\\frac{1}{2} b_0, c_1=\\frac{1}{2} c_0, m_1=\\frac{1}{2} m_0$, 就得到了一组满足 $0) 所示). 则 $P_2$ 到直线 $P_1 P_3$ 的距离不超过 $Q$ 到直线 $P_1 P_3$ 的距离 $Q R$, 而 $Q R|\\boldsymbol{s}|^2,\n$$\n这与 $\\boldsymbol{u}_1, \\cdots, \\boldsymbol{u}_n$ 的取法矛盾.\n所以, $\\left|M_2\\right| \\leqslant n-1$.\n另一方面, 由 (1) 知存在 $\\boldsymbol{u}_1^{\\prime}, \\cdots, \\boldsymbol{u}_n^{\\prime} \\in M$, 使得\n$$\n\\left|\\boldsymbol{u}_1+\\cdots+\\boldsymbol{u}_n+\\boldsymbol{u}_1^{\\prime}+\\cdots+\\boldsymbol{u}_n^{\\prime}\\right|=0,\n$$\n即 $\\boldsymbol{u}_1^{\\prime}+\\cdots+\\boldsymbol{u}_n^{\\prime}=-\\boldsymbol{s}$, 由条件 (2) 知, $M$ 中存在向量 $\\boldsymbol{v}_1^{\\prime}, \\cdots, \\boldsymbol{v}_{n-1}^{\\prime}$, 使得\n$$\nu_1^{\\prime}+\\cdots+u_n^{\\prime}+v_1^{\\prime}+\\cdots+v_{n-1}^{\\prime}=\\mathbf{0} \\text {, 即 } v_1^{\\prime}+\\cdots+v_{n-1}^{\\prime}=s \\text {. }\n$$\n用上面类似的方法证明: 不存在向量 $v^{\\prime} \\in M_1$, 但 $v^{\\prime} \\notin\\left\\{v_1^{\\prime}, \\cdots, v_{n-1}^{\\prime}\\right\\}$. 因此 $\\left|M_1\\right| \\leqslant n-1$.\n综上, 将导致 $|M| \\leqslant 2 n-2$, 矛盾.\n所以, 不存在符合条件的 $M$.", + "remark": "", + "figures": [] +} \ No newline at end of file diff --git a/processed_dataset/proof/1516.json b/processed_dataset/proof/1516.json new file mode 100644 index 0000000000000000000000000000000000000000..a291f8056a9f78bb1ac9d2cbb7256390f6d62de3 --- /dev/null +++ b/processed_dataset/proof/1516.json @@ -0,0 +1,8 @@ +{ + "source_file": "./raw_volume-zh/volume6/chapter1-3.tex", + "problem_type": "proof", + "problem": "例8. 桌子上有两堆硬币,已知这两堆硬币的总重量相同, 并且对任意正整数 $k$ (这里 $k$ 不超过每堆硬币的个数), 第一堆硬币中最重的 $k$ 枚硬币的重量之和不超过第二堆中最重的 $k$ 枚硬币的重量之和.\n证明: 对任意正实数 $x$, 若将两堆硬币中每一枚重量不小于 $x$ 的硬币都用重量为 $x$ 的硬币替换,则完成此操作后,第一堆的总重量不比第二堆轻.", + "solution": "证明:我们用排序原理来处理.\n设第一堆硬币的重量依次为 $x_1 \\geqslant \\cdots \\geqslant x_n$; 第二堆硬币的重量依次为 $y_1 \\geqslant \\cdots \\geqslant y_m$. 则由条件知, 对任意 $k \\leqslant \\min \\{m, n\\}$, 都有 $x_1+\\cdots+x_k \\leqslant y_1+\\cdots+y_k$.\n对任意 $x \\in \\mathbf{R}$, 设 $x_1 \\geqslant \\cdots \\geqslant x_s \\geqslant x>x_{s+1} \\geqslant \\cdots \\geqslant x_n, y_1 \\geqslant \\cdots \\geqslant y_t \\geqslant x>y_{t+1} \\geqslant \\cdots \\geqslant y_m$. 要证明:\n$$\ns x+x_{s+1}+\\cdots+x_n \\geqslant t x+y_{t+1}+\\cdots+y_m . \\label{eq1}\n$$\n显然, 当 $s$ 或 $t$ 不存在时 (注意, 由条件知, 若 $t$ 不存在则 $s$ 也不存在), 不等式\\ref{eq1}可由 $x_1+\\cdots+x_n=y_1+\\cdots+y_m$ 得到.\n下面考虑 $s$ 与 $t$ 都存在的情形.\n记 $x_1+\\cdots+x_n=y_1+\\cdots+y_m=A$, 则式\\ref{eq1}等价于\n$$\n\\begin{gathered}\ns x+\\left(A-x_1-\\cdots-x_s\\right) \\geqslant t x+\\left(A-y_1-\\cdots-y_t\\right) \\\\\n\\Leftrightarrow x_1+\\cdots+x_s+(t-s) x \\leqslant y_1+\\cdots+y_t .\n\\end{gathered} \\label{eq2}\n$$\n如果 $t \\geqslant s$, 那么\n$$\n\\begin{gathered}\nx_1+\\cdots+x_s+(t-s) x=x_1+\\cdots+x_s+\\underbrace{x+\\cdots+x}_{t-s \\uparrow} \\\\\n\\leqslant y_1+\\cdots+y_s+y_{s+1}+\\cdots+y_t .\n\\end{gathered}\n$$\n不等式\\ref{eq2}获证.\n如果 $t0$.\n现在对条件式作变形\n$$\n\\frac{1}{a_k}=\\frac{n}{n a_{k-1}+a_{k-1}^2}=\\frac{1}{a_{k-1}}-\\frac{1}{a_{k-1}+n},\n$$\n将上式移项得\n$$\n\\frac{1}{a_{k-1}+n}=\\frac{1}{a_{k-1}}-\\frac{1}{a_k} . \\label{eq1}\n$$\n对式\\ref{eq1}将下标 $k$ 从 1 到 $n$ 求和, 得\n$$\n\\begin{aligned}\n\\sum_{k=1}^n \\frac{1}{a_{k-1}+n} & =\\left(\\frac{1}{a_0}-\\frac{1}{a_1}\\right)+\\left(\\frac{1}{a_1}-\\frac{1}{a_2}\\right)+\\cdots+\\left(\\frac{1}{a_{n-1}}-\\frac{1}{a_n}\\right) \\\\\n& =\\frac{1}{a_0}-\\frac{1}{a_n}=2-\\frac{1}{a_n} .\n\\end{aligned}\n$$\n结合 $a_{k-1}>0$, 可知\n$$\n2-\\frac{1}{a_n}=\\sum_{k=1}^n \\frac{1}{a_{k-1}+n}<\\sum_{k=1}^n \\frac{1}{n}=1,\n$$\n于是 $a_n<1$.\n再由 $a_k>a_{k-1}$ 知 $0\\sum_{k=1}^n \\frac{1}{1+n}=\\frac{n}{n+1},\n$$\n得 $a_n>\\frac{n+1}{n+2}=1-\\frac{1}{n+2}>1-\\frac{1}{n}$.\n所以,命题成立.\n说明这里对条件式\"取倒数\"是基于 \"裂项\"的思想, 在数列求和中经常会先\"裂项\",在求和时达到前后相消的效果.", + "remark": "", + "figures": [] +} \ No newline at end of file diff --git a/processed_dataset/proof/1518.json b/processed_dataset/proof/1518.json new file mode 100644 index 0000000000000000000000000000000000000000..e552898e5ac171bf048b68597aaac1ebd746f1d8 --- /dev/null +++ b/processed_dataset/proof/1518.json @@ -0,0 +1,8 @@ +{ + "source_file": "./raw_volume-zh/volume6/chapter1-4.tex", + "problem_type": "proof", + "problem": "例3. 对 $n \\in \\mathbf{N}^*$, 设 $a_n=\\frac{n}{(n-1)^{\\frac{4}{3}}+n^{\\frac{4}{3}}+(n+1)^{\\frac{4}{3}}}$. 证明: $a_1+ a_2+\\cdots+a_{999}<50$.", + "solution": "证明:基本的想法是从局部往整体去处理, 为此, 对 $a_n$ 作恰当放大, 达到裂项相消的目的.\n注意到 $x^3-y^3=(x-y)\\left(x^2+x y+y^2\\right)$, 令 $x=(n+1)^{\\frac{2}{3}}, y=(n-1)^{\\frac{2}{3}}$, 则 $x y=\\left(n^2-1\\right)^{\\frac{2}{3}}0)$ 为公差的等差数列, 一个是以 $q(>1)$ 为公比的等比数列; 这里 $d 、 q$ 互素.\n证明: 如果这两个数列有一项相同, 那么存在无穷多项相同.", + "solution": "证明:可设所给的两个数列分别为 $\\{a+n d\\}, n=0,1,2, \\cdots ;\\left\\{b q^m\\right\\}$, $m=0,1,2, \\cdots$. 这里 $a 、 b 、 d 、 q$ 都是正整数,且 $q>1$.\n如果它们有一项相同, 不妨设两个数列的第一项相同, 否则去掉各数列中的前面的有限项后再讨论, 即 $a=b$. 此时, 为证两个数列有无穷多项相同, 只需证明:存在无穷多个 $m \\in \\mathbf{N}^*$, 使得\n$$\na q^m \\equiv a(\\bmod d),\n$$\n这只需 $q^m \\equiv 1(\\bmod d)$.\n注意到 $1, q, q^2, \\cdots, q^d$ 除以 $d$ 所得余数只有 $d$ 种不同取值, 由抽庶原则可知, 存在 $0 \\leqslant ib_m$, 当然更不能满足 式\\ref{eq1}, 本题讨论的是有穷数列, 其构造思路是让 $\\left\\{b_m\\right\\}$ 的公比尽量靠近 1 , 但在相邻两项之间又有足够的空间.\n考察由下面方式定义的数列 $\\left\\{a_n\\right\\}$ 和 $\\left\\{b_n\\right\\}$ :\n$$\n\\begin{gathered}\nb_1=x^n, b_2=x^{n-1}(1+x), \\cdots, b_n=x(1+x)^{n-1} ; \\\\\na_m=x^{n-1}(1+x)-1+(m-1) x^{n-1}, m=1,2, \\cdots, n .\n\\end{gathered}\n$$\n其中 $x$ 为待定的正整数.\n则 $\\left\\{a_m\\right\\}$ 是以 $x^{n-1}$ 为公差的等差数列, $\\left\\{b_m\\right\\}$ 是以 $1+\\frac{1}{x}$\n为公比的等比数列.\n故只需证明:存在正整数 $x$,使得式\\ref{eq1}成立.\n一方面, 对 $1 \\leqslant m \\leqslant n$, 由于 $a_m=x^n+x^{n-1}-1+(m-1) x^{n-1}$, 故当 $x>1$ 时, 都有 $a_m>x^n$. 因此\n$$\na_{m+1}=a_m+x^{n-1}1)$, 使得对任意 $1 \\leqslant m \\leqslant n$, 都有 $b_m1$, 此时 $M-1$ 在数列中出现, 可设 $a_{n-1}=M-1$. 若 $a_{n-1} \\in[n, 2 n-3]$, 则 $M=a_{n-1}+1=a_{n+1}$,矛盾.\n因此 $a_{n-1} \\leqslant n-1$, 结合 $M>1$, 知 $a_{n-1} \\in[1, n-1]$, 此时有 $a_{n+1}=a_{n-1}-1 \\in[0, n-2]$, 进而 $a_{n+3}=a_{n+1}-1$ (除非 $\\left.a_{n+1}=0\\right)$, 依次下去, 可得一个子数列 $a_{n-1}>a_{n+1}>a_{n+3}>\\cdots>a_{s-1}=0$, 这里 $s \\geqslant n+2$.\n结合 $a_{n-1} \\leqslant n-1$ 知 $M \\leqslant n$, 而 $a_{s-1}=0$, 由 式\\ref{eq2} 知 $a_{s+1}=2 s+1>s+2$, 进而,有 $a_{s+2}=a_{s+1}-(s+2)=s-1 \\in[0, s+1]$, 同上可知 $a_{s+1} \\in\\{0$, $\\left.a_{s+2}-1\\right\\}, \\cdots$ 因此, 必有一个下标 $t$, 使得 $a_t=M$ (因为 $s-1 \\geqslant n+1 \\geqslant M$ ), 从而 $M$ 亦为 $\\left\\{a_n\\right\\}$ 中的项,矛盾.\n综上可知,每一个非负整数都在 $\\left\\{a_n\\right\\}$ 中出现.\n说明本题的关键在于对题给的递推式作出恰当改写, 变为式\\ref{eq2}的形式, 从而出现隔一个数加上 1 或者减去 1 的特点, 为证明数列遍经每一个非负整数打下坚实的基础.", + "remark": "", + "figures": [] +} \ No newline at end of file diff --git a/processed_dataset/proof/1533.json b/processed_dataset/proof/1533.json new file mode 100644 index 0000000000000000000000000000000000000000..e6a3e8ec6afc849260af9b21e4e34a12a37f3929 --- /dev/null +++ b/processed_dataset/proof/1533.json @@ -0,0 +1,8 @@ +{ + "source_file": "./raw_volume-zh/volume6/chapter1-7.tex", + "problem_type": "proof", + "problem": "例5. 用 $A_n$ 表示一些由 $a 、 b 、 c$ 组成的字长为 $n$ 的词组成的集合,其中每一个词中都没有连续两个字同时为 $a$ 或者同时为 $b ; B_n$ 表示一些由 $a 、 b 、 c$ 组成的字长为 $n$ 的词组成的集合, 其中每一个词中都没有连续的三个字是两两不同的.\n证明: 对任意正整数 $n$, 都有 $\\left|B_{n+1}\\right|=3\\left|A_n\\right|$.", + "solution": "证明:我们采用递推的方法来处理.\n用 $c_n$ 表示集合 $A_n$ 中以 $c$ 开头的词的个数, $d_n$ 表示以 $a$ 或 $b$ 开头的词的个数.\n对于 $A_{n+1}$ 中的词, 依第 1 个字分类, 如果为 $c$, 那么去掉它后所得的词仍属于 $A_n$; 如果为 $a$, 那么第 2 个字为 $c$ 或 $b$; 如果为 $b$, 那么第 2 个字为 $c$ 或 $a$. 所以,成立如下递推关系式\n$$\n\\left\\{\\begin{array}{l}\nc_{n+1}=\\left|A_n\\right|=c_n+d_n, \\\\\nd_{n+1}=2 c_n+d_n .\n\\end{array}\\right. \\label{eq1}\n$$\n再用 $c_n^{\\prime}$ 表示 $B_n$ 中最前面的两个字相同的词的个数, $d_n^{\\prime}$ 表示 $B_n$ 中最前面的两个字不同的词的个数.\n对于 $B_{n+1}$ 中的词,我们依最前面的两个字是否相同分类.\n如果相同,那么第 3 个字可以任取,此时,去掉第 1 个字后, 所得词属于 $B_n$; 如果不同, 那么第 3 个字与前面两个字中的某一个相同, 在与第 1 个字相同时, 去掉第 1 个字后, 共有 $d_n^{\\prime}$ 个词.\n在与第 2 个字相同时, 去掉第 1 个字后, 共有 $2 c_n^{\\prime}$ 个词 (这里系数为 2 是因为 $a b b \\cdots$ 与 $c b b \\cdots$ 去掉第 1 个字后所得的词相同), 所以,它们之间的递推关系式为\n$$\n\\left\\{\\begin{array}{l}\nc_{n+1}^{\\prime}=\\left|B_n\\right|=c_n^{\\prime}+d_n^{\\prime} ; \\\\\nd_{n+1}^{\\prime}=2 c_n^{\\prime}+d_n^{\\prime} .\n\\end{array}\\right. \\label{eq2}\n$$\n注意到,递推关系式\\ref{eq1}与\\ref{eq2}完全相同,不同的只是它们的初始条件.\n直接枚举可知 $c_1=1, d_1=2 ; c_2^{\\prime}=3, d_2^{\\prime}=6$. 因此 $c_2^{\\prime}=3 c_1, d_2^{\\prime}=3 d_1$, 从而由递推关系式,可知 $c_{n+1}^{\\prime}=3 c_n, d_{n+1}^{\\prime}=3 d_n$. 结合 $\\left|A_n\\right|=c_{n+1}$ 及 $\\left|B_n\\right|=c_{n+1}^{\\prime}$, 可得 $\\left|B_{n+1}\\right|=3\\left|A_n\\right|$.\n命题获证.\n说明利用递推思想处理组合计数问题是一个重要的方法.\n这里建立的递推式可化为常系数齐次线性递推关系, 可求解出 $\\left|A_n\\right|$ 的具体数值.", + "remark": "", + "figures": [] +} \ No newline at end of file diff --git a/processed_dataset/proof/1534.json b/processed_dataset/proof/1534.json new file mode 100644 index 0000000000000000000000000000000000000000..3197c33c6cfb907231f2060abfd804295130d807 --- /dev/null +++ b/processed_dataset/proof/1534.json @@ -0,0 +1,8 @@ +{ + "source_file": "./raw_volume-zh/volume6/chapter1-7.tex", + "problem_type": "proof", + "problem": "例6. 实数数列 $\\left\\{a_n\\right\\}$ 满足: 对任意不同的正整数 $i 、 j$, 都有 $\\left|a_i-a_j\\right| \\geqslant \\frac{1}{i+j}$, 且存在正实数 $c$, 使得对任意 $n \\in \\mathbf{N}^*$, 都有 $0 \\leqslant a_n \\leqslant c$.\n求证: $c \\leqslant 1$.", + "solution": "证明:此题不是以等式形式给出的数列各项之间的关系,它只是用一个不等式刻画了项与项之间的差距.\n整个解决过程有一定的分析味道, 基于裂项求和的思想.\n对 $n \\geqslant 2$, 设 $\\pi(1), \\cdots, \\pi(n)$ 是 $1,2, \\cdots, n$ 的一个排列, 使得\n$$\n0 \\leqslant a_{\\pi(1)}\\frac{(n-1)^2}{n(n+1)-2}=\\frac{n-1}{n+2}=1-\\frac{3}{n+2} .\n\\end{aligned}\n$$\n所以, 我们有\n$$\nc \\geqslant 1-\\frac{3}{n+-2}\n$$\n令 $n \\rightarrow+\\infty$, 即可得 $c \\geqslant 1$.\n命题获证.", + "remark": "", + "figures": [] +} \ No newline at end of file diff --git a/processed_dataset/proof/1535.json b/processed_dataset/proof/1535.json new file mode 100644 index 0000000000000000000000000000000000000000..1b8ba88c1c26dbdcbef2567c189bc8c7dcae0c38 --- /dev/null +++ b/processed_dataset/proof/1535.json @@ -0,0 +1,8 @@ +{ + "source_file": "./raw_volume-zh/volume6/chapter1-7.tex", + "problem_type": "proof", + "problem": "例7. 由实数组成的无穷数列 $\\left\\{a_n\\right\\}$ 定义如下: $a_0 、 a_1$ 是两个不同的正实数, 且 $a_n=\\left|a_{n+1}-a_{n+2}\\right|, n=0,1,2, \\cdots$. 问: 该数列是否可能是一个有界数列? 证明你的结论.", + "solution": "解:此数列一定是一个无界数列.\n证明的基本思想是从 $\\left\\{a_n\\right\\}$ 中取出一个递增的无界数列.\n事实上, 如果存在 $n \\in \\mathbf{N}^*$, 使得 $a_n=a_{n+1}$, 则由递推关系式, 知 $a_{n-1}=0$, 进而 $a_{n-2}=a_{n-3}$ (注意, 这里用到 $\\left\\{a_n\\right\\}$ 的每一项都是非负实数), 这样依次倒推, 可知 $a_1=a_2$ 或者 $a_1 、 a_2$ 中有一个等于零,但这与 $a_1 、 a_2$ 是两个不同的正实数矛盾.\n因此, 对任意 $n \\in \\mathbf{N}^*$, 都有 $a_n \\neq a_{n+1}$ (即 $\\left\\{a_n\\right\\}$ 中没有相邻两项是相等的), 从而结合递推式知, 对任意 $n \\in \\mathbf{N}^*$, 都有 $a_n>0$.\n现在我们来从 $\\left\\{a_n\\right\\}$ 中挑出一个递增的子数列 $\\left\\{b_m\\right\\}$.\n由条件,知 $a_{n+2}=a_n+a_{n+1}$ 或 $a_{n+2}=a_{n+1}-a_n$. 若为前者,则 $a_{n+2}>a_{n+1}$; 若为后者, 则 $a_{n+2}a_{n+1}$. 这一段讨论表明: 要么 $a_{n+2}>a_{n+1}$, 要么 $a_{n+2}a_n$ ) 的项 $a_{n+1}$, 当然, 如果 $a_1>a_2$, 那么去掉 $a_1$ 保留 $a_2$ 后再做去项操作.\n这样留下的项依次记为 $b_1, b_2, \\cdots$ 所得数列 $\\left\\{b_m\\right\\}$ 是一个递增数列.\n最后, 我们证明: $\\left\\{b_m\\right\\}$ 必为无界数列.\n只需证明: 对任意 $m \\in \\mathbf{N}^*, b_{m+2}-b_{m+1} \\geqslant b_{m+1}-b_m$ (因为这时, 利用裂项求和可得 $b_{m+2}-b_2 \\geqslant m\\left(b_2-b_1\\right)$, 让 $m \\rightarrow+\\infty$, 即可知 $\\left\\{b_m\\right\\}$ 为无界数列).\n由 $\\left\\{b_m\\right\\}$ 的定义, 可设 $b_{m+2}=a_{n+2}$ (注意 $n$ 不一定为 $m$ ), 则由于 $a_{n+2}$ 是未被去掉的项,故 $a_{n+2}>a_{n+1}$, 如果 $a_{n+1}>a_n$, 那么 $b_{m+1}=a_{n+1}$, 而 $b_m=a_n$ 或者 $a_{n-1}$ (若为前者, 则 $a_n>a_{n-1}$ ), 于是, 总有 $b_m \\geqslant a_{n-1}$, 得\n$$\nb_{m+2}-b_{m+1}=a_{n+2}-a_{n+1}=a_n=a_{n+1}-a_{n-1} \\geqslant b_{m+1}-b_m .\n$$\n如果 $a_{n+1} a_{n-1}$, 否则 $a_{n-1}$ 不是去掉的项), 所以\n$$\nb_{m+2}-b_{m+1}=a_{n+2}-a_n=a_{n+1}=a_n-a_{n-1} \\geqslant b_{m+1}-b_m .\n$$\n命题获证.\n说明建议读者在阅读解答时, 手边写一个具体数列, 便于对比 $\\left\\{a_n\\right\\}$ 与 $\\left\\{b_m\\right\\}$ 之间的关系.\n与上题类似, 它也不是一个由确定性关系式定义的递推数列.\n处理上都涉及不等式估计,它正是一种分析能力的体现.", + "remark": "", + "figures": [] +} \ No newline at end of file diff --git a/processed_dataset/proof/1536.json b/processed_dataset/proof/1536.json new file mode 100644 index 0000000000000000000000000000000000000000..01f4bbbf3faa52fcd7b94a4e35fc5ee0beb709a4 --- /dev/null +++ b/processed_dataset/proof/1536.json @@ -0,0 +1,8 @@ +{ + "source_file": "./raw_volume-zh/volume6/chapter1-7.tex", + "problem_type": "proof", + "problem": "例8. 数列 $\\left\\{a_n\\right\\}$ 满足递推式\n$$\na_{n+1}=\\frac{a_n^2-1}{n+1}, n=0,1,2, \\cdots .\n$$\n问 : 是否存在正实数 $a$, 使得下面的结论都成立?\n(1) 若 $a_0 \\geqslant a$, 则极限 $\\lim _{n \\rightarrow \\infty} a_n$ 不存在;\n(2) 若 $00$,结合 $11$. 现在设 $a_0=2-\\varepsilon(0<\\varepsilon<1)$, 利用递推式及数学归纳法可证: 对任意 $n \\in \\mathbf{N}^*, a_nm$, 都有 $a_n \\leqslant \\frac{(m+1)^2-1}{n-1}$, 这在 $n$ 充分大时, 导致 $a_n \\leqslant 1$, 矛盾.\n因此, 必存在 $n \\in \\mathbf{N}^*$, 使得 $a_n \\leqslant 0$, 归人前面的情形.\n综上可知, $a=2$ 符合要求.", + "remark": "", + "figures": [] +} \ No newline at end of file diff --git a/processed_dataset/proof/1537.json b/processed_dataset/proof/1537.json new file mode 100644 index 0000000000000000000000000000000000000000..2dcac040682c79780138e89a5588f5a1a703e810 --- /dev/null +++ b/processed_dataset/proof/1537.json @@ -0,0 +1,8 @@ +{ + "source_file": "./raw_volume-zh/volume6/chapter1-8.tex", + "problem_type": "proof", + "problem": "例3. 设 $f(x)$ 是一个整系数多项式, 数列 $\\left\\{a_n\\right\\}$ 依如下方式定义\n$$\na_0=0, a_{n+1}=f\\left(a_n\\right), n=0,1,2, \\cdots .\n$$", + "solution": "证明: 若 $\\left\\{a_n\\right\\}$ 是一个纯周期数列, 则其最小正周期不大于 2 .\n证明问题可转化为证明: 若存在 $m \\in \\mathbf{N}^*$, 使得 $a_m=0$, 则 $a_1$ 或 $a_2$ 中有一个等于 0 .\n利用因式定理, 由于 $f(x)$ 为整系数多项式, 可知对 $m 、 n \\in \\mathbf{Z}(m \\neq n)$, 都有 $m-n \\mid f(m)-f(n)$.\n现令 $b_n=a_{n+1}-a_n, n=0,1,2, \\cdots$, 由上述结论及数列 $\\left\\{a_n\\right\\}$ 的定义可知 $b_n \\mid b_{n+1}$ (注意, 这里若 $b_n=0$, 则有 $b_{n+1}=f\\left(a_{n+1}\\right)-f\\left(a_n\\right)=0$ ).\n因为 $a_m=a_0=0$, 故 $a_{m+1}=f\\left(a_0\\right)=a_1$, 所以 $b_m=b_0$.\n如果 $b_0=0$, 那么 $a_0=a_1=\\cdots=a_m$, 命题已成立; 否则 $\\left|b_0\\right|=\\left|b_m\\right| \\neq$ 0 , 结合 $b_0\\left|b_1, b_1\\right| b_2, \\cdots, b_{m-1} \\mid b_m$, 可得 $\\left|b_0\\right|=\\left|b_1\\right|=\\cdots=\\left|b_m\\right|$.\n注意到\n$$\nb_0+b_1+\\cdots+b_{m-1}=a_m-a_0=0,\n$$\n因此, $b_0, b_1, \\cdots, b_{m-1}$ 中有一半为正整数, 另一半为负整数, 从而, 存在 $k \\in \\{1,2, \\cdots, m-2\\}$, 使得 $b_{k-1}=-b_k$, 得 $a_{k-1}=a_{k+1}$, 依 $\\left\\{a_n\\right\\}$ 的定义知, 对 $n \\geqslant k-1$, 都有 $a_{n+2}=a_n$. 取 $n=m$, 就有\n$$\n\\begin{aligned}\na_0 & =a_m=a_{m+2}=f\\left(a_{m+1}\\right)=f\\left(f\\left(a_m\\right)\\right) \\\\\n& =f\\left(f\\left(a_0\\right)\\right)=a_2 .\n\\end{aligned}\n$$\n即有 $a_2=0$.\n所以, 命题成立.", + "remark": "", + "figures": [] +} \ No newline at end of file diff --git a/processed_dataset/proof/1538.json b/processed_dataset/proof/1538.json new file mode 100644 index 0000000000000000000000000000000000000000..f726d068a2da79f5fb723ef1fffbc5acbbe1cd81 --- /dev/null +++ b/processed_dataset/proof/1538.json @@ -0,0 +1,8 @@ +{ + "source_file": "./raw_volume-zh/volume6/chapter1-8.tex", + "problem_type": "proof", + "problem": "例4. 设 $m$ 是一个给定的大于 1 的正整数,数列 $\\left\\{x_n\\right\\}$ 定义如下 $x_1=1$,\n$$\nx_2=2, \\cdots, x_m=m \\text {, 而 }\n$$\n$$\nx_{n+m}=x_{n+m-1}+x_n, n=1,2, \\cdots . \\label{eq1}\n$$", + "solution": "证明: 数列 $\\left\\{x_n\\right\\}$ 中存在连续的 $m-1$ 项, 它们都是 $m$ 的倍数.\n证明考察数列 $\\left\\{x_k(\\bmod m)\\right\\}$, 这里 $x_k(\\bmod m)$ 表示 $x_k$ 除以 $m$ 所得的余数, 将它记为 $y_k$. 转为证明: 数列 $\\left\\{y_k\\right\\}$ 中有连续 $m-1$ 个零.\n利用定理 2, 由 式\\ref{eq1} 可知, 存在 $n_0$ 及 $T \\in \\mathbf{N}^*$, 使得对任意 $k \\geqslant n_0$, 都有 $y_{k+T}=y_k$. 特别地, 有\n$$\ny_{n_0+m-1}=y_{n_0+m-1+T}, y_{n_0+m-2}=y_{n_0+n-2+T} .\n$$\n两式相减, 结合 式\\ref{eq1}及 $y_k$ 的定义可知 $y_{n_0-1}=y_{n_0-1+T}$, 依此倒推可知, 对任意 $k \\geqslant$ 1 , 都有 $y_k=y_{k+T}$.\n为得到我们的结论及计算上的方便, 我们将数列 $\\left\\{x_n\\right\\}$ 的下标依式\\ref{eq1}确定的递推关系向负整数延拓, 结合上面的讨论, 可知对任意 $k \\in \\mathbf{Z}$, 都有 $y_k=y_{k+T}$.\n现在由 $x_n=x_{n+m}-x_{n+m-1}$ 可知 $x_0=x_{-1}=\\cdots=x_{-(m-2)}=1$ (这里用到初始条件: 对任意 $1 \\leqslant j \\leqslant m$, 都有 $\\left.x_j=j\\right)$, 进而, 有 $x_{-(m-1)}=x_{-m}=\\cdots= x_{-(2 m-3)}=0$. 结合 $y_k=y_{k+T}$, 可知\n$$\n\\begin{aligned}\n& \\left(y_{-(2 m-3)+T}, \\cdots, y_{-(m-1)+T}\\right) \\\\\n= & \\left(y_{-(2 m-3)}, \\cdots, y_{-(m-1)}\\right)=(0, \\cdots, 0) .\n\\end{aligned}\n$$\n而 $y_{-(m-2)}=\\cdots=y_0=1$, 故 $-(2 m-3)+T \\geqslant 1$, 这表明: 数列 $\\left\\{y_k\\right\\}$ 中存在下标为正整数的连续 $m-1$ 项都等于零.\n所以, 命题成立.", + "remark": "", + "figures": [] +} \ No newline at end of file diff --git a/processed_dataset/proof/1539.json b/processed_dataset/proof/1539.json new file mode 100644 index 0000000000000000000000000000000000000000..8e80a8adb19928632151291e310d4f21e1d5d5b9 --- /dev/null +++ b/processed_dataset/proof/1539.json @@ -0,0 +1,8 @@ +{ + "source_file": "./raw_volume-zh/volume6/chapter1-8.tex", + "problem_type": "proof", + "problem": "例5. 设 $m$ 为给定的正整数, 对任意正整数 $n$, 用 $S_m(n)$ 表示 $n$ 在十进制表示下各数码的 $m$ 次方之和.\n例如 $S_3(172)=1^3+7^3+2^3=352$. 考虑数列: $n_0$ 为正整数, $n_k=S_m\\left(n_{k-1}\\right), k=1,2, \\cdots$.\n(1) 证明: 对任意正整数 $n_0$, 数列 $\\left\\{n_k\\right\\}$ 都是一个周期数列;\n(2)证明: 当 $n_0$ 变化时, (1) 中数列的最小正周期构成的集合为有限集.", + "solution": "证明:注意到, 对正整数 $n \\geqslant 10^{m+1}$, 存在 $p \\in \\mathbf{N}^*, p \\geqslant m+1$, 使得 $10^p \\leqslant n<10^{p+1}$, 此时 $n$ 为十进制中的 $p+1$ 位数, 故\n$$\n\\begin{aligned}\nS_m(n) & \\leqslant(p+1) \\cdot 9^m<(p+1) \\cdot 9^{p-1} \\\\\n& <9^p+\\mathrm{C}_p^1 \\cdot 9^{p-1}>(9+1)^p=10^p \\leqslant n .\n\\end{aligned}\n$$\n这表明数列 $\\left\\{n_k\\right\\}$ 中的项满足: 若 $n_k \\geqslant 10^{m+1}$, 则 $n_{k+1}=S_m\\left(n_k\\right)s \\geqslant k_0$, 使得 $n_r=n_s$. 利用 $\\left\\{n_k\\right\\}$ 的定义知, 对 $k \\geqslant s$, 都有 $n_k=n_{k+T}$, 这里 $T=r-s$, 并可使得 $T \\leqslant 10^{m+1}-1$.\n所以, 对任意 $n_0 \\in \\mathbf{N}^*$, 数列 $\\left\\{n_k\\right\\}$ 都为周期数列, 其最小正周期 $\\leqslant 10^{m+1}-1$. 从而, (1) 与 (2)都成立.", + "remark": "", + "figures": [] +} \ No newline at end of file diff --git a/processed_dataset/proof/1540.json b/processed_dataset/proof/1540.json new file mode 100644 index 0000000000000000000000000000000000000000..e6a5f9f07c8270a5f0ce457944a44fe1e877cf13 --- /dev/null +++ b/processed_dataset/proof/1540.json @@ -0,0 +1,8 @@ +{ + "source_file": "./raw_volume-zh/volume6/chapter1-8.tex", + "problem_type": "proof", + "problem": "例6. 任意选定一个正整数 $a_0$, 再任取 $a_1 \\in\\left\\{a_0+54, a_0+77\\right\\}$,如此下去, 当 $a_k$ 确定后, 再选取 $a_{k+1} \\in\\left\\{a_k+54, a_k+77\\right\\}$ 得到无穷数列 $\\left\\{a_n\\right\\}$. 证明: 该数列中总有一项, 其末两位数字相同.", + "solution": "证明:在模 100 的意义下讨论.\n我们用 $b_n$ 表示 $a_n$ 除以 100 所得的余数, 这里将 $b_n$ 都理解为两位数, 即 $b_n$ 是 $00,01, \\cdots, 99$ 中数.\n依数列 $\\left\\{a_n\\right\\}$ 的定义可知, 对任意 $n \\in \\mathbf{N}^*$, 都有 $b_{n+1} \\equiv b_n+77$ 或 $b_n+2 \\times 77(\\bmod 100)$.\n注意到 $(77,100)=1$, 故当 $j$ 跑遍模 100 的完系时, 00 $77 j$ 也跑遍模 100 的完系, 对 $j=0,1,2, \\cdots, 99$, 我们将 $77 j$ 除以 100 所得的余数排成右边所示的圆圈.\n那么, 由 $\\left\\{b_n\\right\\}$ 的结构可知, $b_n$ 与 $b_{n+1}$ 是圆圈上相邻的数或者中间隔一个数.\n因此, 圆圈上任意相邻的两个数中必有一个是 $\\left\\{b_n\\right\\}$ 中的项.\n而圆圈上 00 与 77 相邻, 故存在 $n \\in \\mathbf{N}^*$, 使得 $b_n=00$ 或 77 ,也就是 $a_n$ 的末两位数字是 00 或 77 .\n所以, 命题成立.\n说明尽管数列 $\\left\\{a_n\\right\\}$ 的每一项都有两种选择, 在模 100 的意义下也不是周期变化的,但跳跃性有限,组合方法的引人使问题迎刃而解.", + "remark": "", + "figures": [] +} \ No newline at end of file diff --git a/processed_dataset/proof/1541.json b/processed_dataset/proof/1541.json new file mode 100644 index 0000000000000000000000000000000000000000..004ebd3b2adfbe66b67e11fe761b5041a6774b2e --- /dev/null +++ b/processed_dataset/proof/1541.json @@ -0,0 +1,8 @@ +{ + "source_file": "./raw_volume-zh/volume6/chapter2-1.tex", + "problem_type": "proof", + "problem": "例1. 证明: 对任意 $m, n \\in \\mathbf{N}^*$, 都有 $\\left(F_m, F_n\\right)=F_{(m, n)}$. 即针对 Fibonaccia 数列的项求最大公因数可以转化到下标上去.", + "solution": "证明:当 $m=n$ 时显然成立.\n考虑 $m \\neq n$ 的情形, 不妨设 $m>n$.\n利用 Fibonaccia 数列的递推式, 可知\n$$\n\\begin{aligned}\nF_m= & F_{m-1}+F_{m-2}=F_2 F_{m-1}+F_1 F_{m-2} \\\\\n= & F_2\\left(F_{m-2}+F_{m-3}\\right)+F_1 F_{m-2} \\\\\n= & \\left(F_2+F_1\\right) F_{m-2}+F_2 F_{m-3} \\\\\n= & F_3 F_{m-2}+F_2 F_{m-3} \\\\\n& \\cdots \\\\\n= & F_n F_{m-n+1}+F_{n-1} F_{m-n} .\n\\end{aligned}\n$$\n于是 $\\left(F_m, F_n\\right)=\\left(F_{n-1} F_{m-n}, F_n\\right)=\\left(F_{m-n}, F_n\\right)$ (这里用到 $\\left(F_{n-1}, F_n\\right)=1$, 它可以通过对 $n$ 用数学归纳法证得, 具体过程留给读者).\n在上面的结论中, 用 $(m-n, n)$ 代替 $(m, n)$ 继续讨论, 表明求 $F_m$ 与 $F_n$ 的最大公因数的过程实质上是对下标 $m 、 n$ 作辗转相除.\n所以 $\\left(F_m, F_n\\right)= F_{(m, n)}$.\n说明利用本题的结论可证出下述命题: 如果 $F_n$ 为素数, 那么 $n=4$ 或者 $n$ 为素数.\n事实上, 如果 $n \\neq 4$ 且 $n$ 不是素数, 那么可写 $n=p q, 2 \\leqslant p \\leqslant q$, 并且\n$q \\geqslant 3$. 此时 $\\left(F_n, F_q\\right)=F_{(n, q)}=F_q$, 而 $F_q \\geqslant 2, F_n>F_q$, 由此导出 $F_n$ 为合数.", + "remark": "", + "figures": [] +} \ No newline at end of file diff --git a/processed_dataset/proof/1542.json b/processed_dataset/proof/1542.json new file mode 100644 index 0000000000000000000000000000000000000000..db658c2e865e49a343f1418199318534e78ba599 --- /dev/null +++ b/processed_dataset/proof/1542.json @@ -0,0 +1,8 @@ +{ + "source_file": "./raw_volume-zh/volume6/chapter2-1.tex", + "problem_type": "proof", + "problem": "例2. 证明: 每一个正整数 $m$, 都可以唯一地表示为如下形式\n$$\n\\begin{aligned}\nm & =\\left(a_n a_{n-1} \\cdots a_2\\right)_F \\\\\n& =a_n F_n+a_{n-1} F_{n-1}+\\cdots+a_2 F_2 . \n\\end{aligned} \\label{eq1}\n$$\n这里 $a_i=0$ 或 $1, a_n=1$, 并且不存在下标 $2 \\leqslant i \\leqslant n-1$, 使得 $a_i=a_{i+1}=1$, 其中 $F_i$ 为 Fibonaccia 数列中的第 $i$ 项.", + "solution": "证明:形如式\\ref{eq1}的正整数表示可称为 $m$ 的 $F$-表示, 它类似于二进制, 此结论是著名的 Zerkendorf 定理.\n对 $m$ 归纳来予以证明.\n当 $m=1$ 时, $m=F_2$, 命题成立.\n现设对所有小于 $m$ 的正整数 $k$ 命题都成立.\n由于存在唯一的 $n \\in \\mathbf{N}^*$, 使得 $F_n \\leqslant m0$, 那么由归纳假设, $m-F_n$ 有形如 式\\ref{eq1} 的表示, 设\n$$\nm-F_n=\\left(a_l a_{l-1} \\cdots a_2\\right)_F=a_l F_l+\\cdots+a_2 F_2,\n$$\n其中 $a_l=1$, 则 $m=F_n+a_l F_l+\\cdots+a_2 F_2$. 现在若 $l \\geqslant n-1$, 则 $m \\geqslant F_n+ F_{n-1}=F_{n+1}$, 矛盾, 所以 $l \\leqslant n-2$, 从而 $m$ 有满足式\\ref{eq1} 的表示.\n下证 $m$ 的形如式\\ref{eq1}的表示是唯一的.\n事实上,若\n$$\nm=\\left(a_n \\cdots a_2\\right)_F=\\left(b_l \\cdots b_2\\right)_F, \\label{eq2}\n$$\n这里 $a_n=b_l=1$, 且 $n \\geqslant l$.\n若 $n>l$, 则由于不存在下标 $1 \\leqslant i \\leqslant l-1$, 使得 $b_i=b_{i+1}=1$, 结合 $\\left\\{F_n\\right\\}$ 的定义, 可知\n$$\n\\begin{aligned}\n\\left(b_l \\cdots b_2\\right)_F & \\leqslant\\left\\{\\begin{array}{l}\nF_l+F_{l-2}+\\cdots+F_3, m \\text { 为偶数, } \\\\\nF_l+F_{l-2}+\\cdots+F_4+F_2, m \\text { 为奇数, }\n\\end{array}\\right. \\\\\n& <\\left\\{\\begin{array}{l}\nF_l+F_{l-2}+\\cdots+F_3+F_2=F_{l+1}, m \\text { 为偶数, } \\\\\nF_l+F_{l-2}+\\cdots+F_4+F_2+F_1=F_{l+1}, m \\text { 为奇数.\n}\n\\end{array}\\right.\n\\end{aligned}\n$$\n因此 $\\left(b_l \\cdots b_2\\right)_F0)$. 证明:\n(1) 对任意正整数 $n$, 都有 $g_n(x)=x+f(x)+f(f(x))+\\cdots+ \\underbrace{f(f(\\cdots f(x)))}_{n \\uparrow f}$ 是 $(0,+\\infty)$ 上的递增函数;\n(2) $g_n(1)=\\frac{F_1}{F_2}+\\frac{F_2}{F_3}+\\cdots+\\frac{F_{n+1}}{F_{n+2}}$, 这里 $\\left\\{F_n\\right\\}$ 是 Fibonacci 数列.", + "solution": "证明:为表述方便, 我们记 $f^{(n)}(x)=\\underbrace{f(f(\\cdots f(x)))}_{n \\uparrow f}$, 从局部出发来讨论这个函数迭代问题.\n(1)熟知函数 $y=x+\\frac{1}{x}$ 在 $(1,+\\infty)$ 上单调递增, 因此, 函数 $h(x)= x+f(x)=x+\\frac{1}{1+x}=(1+x)+\\frac{1}{1+x}-1$ 在 $(0,+\\infty)$ 上单调递增.\n注意到, $f(f(x))=\\frac{1}{1+f(x)}=\\frac{1}{1+\\frac{1}{1+x}}=\\frac{1+x}{2+x}=1-\\frac{1}{2+x}$ 是 $(0$, $+\\infty)$ 上的增函数, 依此可知, 对任意 $k \\in \\mathbf{N}^*$, 函数 $f^{(2 k)}(x)$ 都是 $(0,+\\infty)$ 上的增函数, 结合 $h(x)$ 在 $(0,+\\infty)$ 上递增, 可知 $f^{(2 k)}(x)+f^{(2 k+1)}(x)$ 也是 $(0$, $+\\infty)$ 上的增函数.\n利用上述结论可知,\n当 $n$ 为奇数时,\n$$\ng_n(x)=(x+f(x))+\\left(f^{(2)}(x)+f^{(3)}(x)\\right)+\\cdots+\\left(f^{(n-1)}(x)+f^{(n)}(x)\\right)\n$$\n是 $\\frac{n+1}{2}$ 个 $(0,+\\infty)$ 上的增函数之和.\n当 $n$ 为偶数时, $f^{(n)}(x)$ 与 $g_n(x)-f^{(n)}(x)$ 都是 $(0,+\\infty)$ 上的增函数, 因此, $g_n(x)$ 也是 $(0,-\\infty)$ 上的增函数.\n所以,对任意 $n \\in \\mathbf{N}^*, g_n(x)$ 都是 $(0,+\\infty)$ 上的增函数.\n(2) 由 $g_n(x)$ 的定义可知, 我们只需证明: 对任意 $n \\in \\mathbf{N}$, 都有 $f^{(n)}(1)= \\frac{F_{n+1}}{F_{n+2}}\\left(\\right.$ 这里 $\\left.f^{(0)}(x)=x\\right)$.\n利用 $1=\\frac{F_1}{F_2}, f(1)=\\frac{1}{2}=\\frac{F_2}{F_3}$ 可知当 $n=0 、 1$ 时命题成立.\n现设 $f^{(n)}(1)= \\frac{F_{n+1}}{F_{n+2}}$ (即命题对 $n$ 成立), 则由 $f^{(n+1)}(x)=\\frac{1}{1+f^{(n)}(x)}$ 可知 $f^{(n+1)}(1)= \\frac{1}{1+f^{(n)}(1)}$, 故\n$$\nf^{(n+1)}(x)=\\frac{1}{1+\\frac{F_{n+1}}{F_{n+2}}}=\\frac{F_{n+2}}{F_{n+2}+F_{n+1}}=\\frac{F_{n+2}}{F_{n+3}} .\n$$\n所以, (2) 成立.", + "remark": "", + "figures": [] +} \ No newline at end of file diff --git a/processed_dataset/proof/1545.json b/processed_dataset/proof/1545.json new file mode 100644 index 0000000000000000000000000000000000000000..12531f7eccdb79c752c2e2e908608464e05e6ceb --- /dev/null +++ b/processed_dataset/proof/1545.json @@ -0,0 +1,8 @@ +{ + "source_file": "./raw_volume-zh/volume6/chapter2-1.tex", + "problem_type": "proof", + "problem": "例5. 考虑数列 $\\left\\{x_n\\right\\}: x_1=a, x_2=b, x_{n+2}=x_{n+1}+x_n, n=1,2$, $3, \\cdots$, 这里 $a 、 b$ 为实数.\n若存在正整数 $k 、 m, k \\neq m$, 使得 $x_k=x_m=c$, 则称实数 $c$ 为\"双重值\". 证明: 存在实数 $a 、 b$, 使得至少存在 2000 个不同的\"双重值\". 进一步,证明:不存在 $a 、 b$, 使得存在无穷多个\"双重值\".", + "solution": "证明:我们利用 Fibonaccia 数列来构造一个具有 2000 个不同的\"双重值\"的数列.\n想法是将 $\\left\\{F_n\\right\\}$ 依现有的递推式向负整数下标延拓, 可得\n$$\n\\begin{aligned}\n& F_0=F_2-F_1=0, \\\\\n& F_{-1}=F_1-F_0=1=F_1, \\\\\n& F_{-2}=F_0-F_{-1}=-1=-F_2, \\\\\n& F_{-3}=F_{-1}-F_{-2}=2=F_3,\n\\end{aligned}\n$$\n依此下去, 可知 $F_{-2 m}=-F_{2 m}, F_{-(2 m+1)}=F_{2 m+1}, m=1,2, \\cdots$.\n于是, 对任意 $m \\in \\mathbf{N}^*$, 令 $a=F_{2 m+1}, b=-F_{2 m}$, 那么, 数列 $\\left\\{x_n\\right\\}$ 为\n$$\n\\begin{aligned}\n& F_{2 m+1},-F_{2 m}, F_{2 m-1},-F_{2 m-2}, \\cdots,-F_2, F_1, F_0, F_1, F_2, \\cdots, F_{2 m-1} \\\\\n& F_{2 m}, F_{2 m+1}, \\cdots\n\\end{aligned}\n$$\n数 $F_1, F_3, \\cdots, F_{2 m+1}$ 都是 $\\left\\{x_n\\right\\}$ 的 \"双重值\". 特别地, 取 $m=1999$ 即可找到符合要求的 $\\left\\{x_n\\right\\}$.\n另一方面, 若存在 $a 、 b$, 使得 $\\left\\{x_n\\right\\}$ 有无穷多个不同的\"双重值\", 则 $\\left\\{x_n\\right\\}$ 中任意相邻两项不同号 (否则, 数列从这相邻两项的下一项起变为一个严格递增 (或严格递减)的数列,不能出现无穷多个不同的\"双重值\").\n注意到, $\\left\\{x_n\\right\\}$ 的特征方程 (也就是 Fibonaccia 数列的特征方程) 为 $\\lambda^2= \\lambda+1$, 有两个不同的实根, 因而可设\n$$\nx_n=A \\cdot\\left(\\frac{1+\\sqrt{5}}{2}\\right)^n+B \\cdot\\left(\\frac{1-\\sqrt{5}}{2}\\right)^n, n=1,2, \\cdots .\n$$\n由于 $\\left|\\frac{1-\\sqrt{5}}{2}\\right|<1$, 而 $\\frac{1+\\sqrt{5}}{2}>1$, 如果 $A>0$, 那么 $n$ 充分大时, 都有 $x_n>0$, 从而会出现都为正数的相邻两项; 同样地, 若 $A<0$, 则 $\\left\\{x_n\\right\\}$ 中会出现同为负数的相邻两项.\n均导致矛盾.\n所以 $A=0$, 进而 $x_n=B \\cdot\\left(\\frac{1-\\sqrt{5}}{2}\\right)^n$, 结合 $\\left|\\frac{1-\\sqrt{5}}{2}\\right|<1$ 知, 数列 $\\left\\{\\left|x_n\\right|\\right\\}$ 是一个单调递减的数列, 在 $B \\neq 0$ 时不出现 \"双重数\",而 $B=0$ 时, 只有一个\"双重数\".\n综上可知,命题成立.\n说明利用 Fibonaccia 数列的特征方程及初始条件可求得通项公式为 $F_n=\\frac{1}{\\sqrt{5}}\\left(\\frac{1+\\sqrt{5}}{2}\\right)^n-\\frac{1}{\\sqrt{5}}\\left(\\frac{1-\\sqrt{5}}{2}\\right)^n, n=1,2, \\cdots$. 然而, 在实际问题的解决中递推式比通项公式用得更多.", + "remark": "", + "figures": [] +} \ No newline at end of file diff --git a/processed_dataset/proof/1546.json b/processed_dataset/proof/1546.json new file mode 100644 index 0000000000000000000000000000000000000000..aadc1f776500cbb877487fb22127ad67ad4085fd --- /dev/null +++ b/processed_dataset/proof/1546.json @@ -0,0 +1,8 @@ +{ + "source_file": "./raw_volume-zh/volume6/chapter2-1.tex", + "problem_type": "proof", + "problem": "例6. 将 Fibonacci 数列的项依次排列 $1,1,2,3,5,8, \\cdots$; 将所有的孪生素数 (若 $p$ 与 $p+2$ 都是素数,则称 $p$ 与 $p+2$ 为孪生素数) 从小到大排列 3 , $5,7,11,13,17,19,29,31, \\cdots$. 求在这两个数列中都出现的正整数.", + "solution": "解:对比两个数列的前面若干项, 可发现只有 $3 、 5$ 和 13 在两个数列中出现,猜测这是所有要求的正整数.\n鉴于孪生素数组成的数列的规律性难以把握, 要证上述猜测, 应从 Fibonacci 数列的性质着手, 如果 $n$ 比较大时, 要么 $F_n$ 为合数, 要么 $F_n \\pm 2$ 都是合数, 那么 $F_n$ 不在孪生素数数列中出现.\n依此想法着手, 先要猜出 Fibonacci 数列的一些性质.\n将 Fibonacci 数列的前面一些项列出\n\\begin{tabular}{|c|c|c|c|c|c|c|c|c|c|c|c|c|c|c|c|c|}\n\\hline$n$ & 1 & 2 & 3 & 4 & 5 & 6 & 7 & 8 & 9 & 10 & 11 & 12 & 13 & 14 & 15 & $\\cdots$ \\\\\n\\hline$F_n$ & 1 & 1 & 2 & 3 & 5 & 8 & 13 & 21 & 34 & 55 & 89 & 144 & 233 & 377 & 610 & $\\cdots$ \\\\\n\\hline\n\\end{tabular}\n发现 $F_{2 n}\\left(n \\geqslant 3\\right.$ 时) 都是合数, 而 $F_{2 n+1} \\pm 2$ ( $n \\geqslant 4$ 时) 也都是合数, 并且有如下的一些关系式\n(1) $F_{2 n}=F_n\\left(F_{n+1}+F_{n-1}\\right)$, 这里 $F_0=0$;\n(2) $F_{4 n+1}+2=F_{2 n-1}\\left(F_{2 n+1}+F_{2 n+3}\\right)$;\n(3) $F_{4 n+1}-2=F_{2 n+2}\\left(F_{2 n-2}+F_{2 n}\\right)$;\n(4) $F_{4 n+3}+2=F_{2 n+3}\\left(F_{2 n+1}+F_{2 n-1}\\right)$;\n(5) $F_{4 n+3}-2=F_{2 n}\\left(F_{2 n+2}+F_{2 n+4}\\right)$.\n注意到, 如果上述 5 个关系式成立, 那么在两个数列中出现的数只有 $3 、 5$ 和 13 . 现在用数学归纳法证明 (1)-(5)都成立.\n当 $n=1$ 时,利用前表中所列数据可知 (1)-(5)都成立.\n现设 (1)-(5) 对不超过 $n$ 的情形都成立, 则由 Fibonacci 数列的递推式, 对 $n+1$ 的情形, 有\n$$\n\\begin{aligned}\nF_{4 n+2} & =F_{4 n+1}+F_{4 n}=F_{4 n+1}+F_{4 n-1}+F_{4 n-2} \\\\\n& =\\left(F_{4 n+1}+2\\right)+\\left(F_{4 n-1}-2\\right)+F_{4 n-2} \\\\\n& =F_{2 n-1}\\left(F_{2 n+1}+F_{2 n+3}\\right)+F_{2 n-2}\\left(F_{2 n}+F_{2 n+2}\\right)+F_{2 n-1}\\left(F_{2 n-2}+F_{2 n}\\right) \\\\\n& =F_{2 n+1} F_{2 n-1}+F_{2 n-1} F_{2 n+3}+F_{2 n-2}\\left(F_{2 n}+F_{2 n-1}\\right)+\\left(F_{2 n-2} F_{2 n+2}+F_{2 n-1} F_{2 n}\\right) \\\\\n& =F_{2 n+1} F_{2 n-1}+F_{2 n-2} F_{2 n+1}+F_{2 n-1}\\left(F_{2 n+3}+F_{2 n}\\right)+F_{2 n-2} F_{2 n+2} \\\\\n& =F_{2 n+1} F_{2 n}+2 F_{2 n-1} F_{2 n+2}+\\left(F_{2 n}-F_{2 n-1}\\right) F_{2 n+2} \\\\\n& =F_{2 n+1} F_{2 n}+F_{2 n+2}\\left(F_{2 n-1}+F_{2 n}\\right) \\\\\n& =F_{2 n+1}\\left(F_{2 n}+F_{2 n+2}\\right)\n\\end{aligned}\n$$\n即\n$$\nF_{2(2 n+1)}==F_{2 n+1}\\left(F_{2 n+1-1}+F_{2 n+1+1}\\right) . \\label{eq1}\n$$\n同理可证即\n即\n$$\n\\begin{gathered}\nF_{4 n+4}=F_{2 n+2}\\left(F_{2 n+1}+F_{2 n+3}\\right), \\\\\nF_{2(2 n+2)}=F_{2 n+2}\\left(F_{2 n+2-1}+F_{2 n+2+1}\\right) . \\label{eq2}\n\\end{gathered}\n$$\n故由式\\ref{eq1}\\ref{eq2}可知, (1)对 $2 n+1 、 2 n+2$ 成立, 因此, 对所有 $n \\in \\mathbf{N}^*$ 成立.\n$$\n\\begin{aligned}\nF_{4 n+5}+2 & =F_{4 n+4}+\\left(F_{4 n+3}+2\\right) \\\\\n& =F_{2 n+2}\\left(F_{2 n+1}+F_{2 n+3}\\right)+F_{2 n+3}\\left(F_{2 n+1}+F_{2 n-1}\\right) \\\\\n& =F_{2 n+1}\\left(F_{2 n+2}+F_{2 n+3}\\right)+F_{2 n+3}\\left(F_{2 n+2}+F_{2 n-1}\\right) \\\\\n& =F_{2 n+1} F_{2 n+4}+2 F_{2 n+3} F_{2 n+1} \\\\\n& =F_{2 n+1}\\left(F_{2 n+3}+F_{2 n+5}\\right),\n\\end{aligned}\n$$\n即 (2) 对 $n+1$ 成立.\n类似地可证 (3)、(4)、(5) 对 $n+1$ 成立 (具体验证过程请读者完成).\n综上可知, (1)-(5)对任意 $n \\in \\mathbf{N}^*$ 成立.\n所以, 只有 $3 、 5$ 和 13 在两个数列中同时出现.\n说明利用例 1 的说明可得出当 $n \\geqslant 3$ 时, $F_{2 n}$ 为合数, 这里的结论更强一些.", + "remark": "", + "figures": [] +} \ No newline at end of file diff --git a/processed_dataset/proof/1547.json b/processed_dataset/proof/1547.json new file mode 100644 index 0000000000000000000000000000000000000000..8b27cbe686f7825025fe961c0e1f2d9458a38a81 --- /dev/null +++ b/processed_dataset/proof/1547.json @@ -0,0 +1,8 @@ +{ + "source_file": "./raw_volume-zh/volume6/chapter2-2.tex", + "problem_type": "proof", + "problem": "例1. 设函数 $f: \\mathbf{N}^* \\rightarrow[1,+\\infty)$ 满足:\n(1) $f(2)=2$;\n(2) 对任意 $m, n \\in \\mathbf{N}^*$, 有 $f(m n)=f(m) f(n)$;\n(3)当 $ml$, 我们取 $m>\\frac{l}{n-l}$, 就有\n$$\n\\left(\\frac{n}{l}\\right)^m=\\left(1+\\frac{n-l}{l}\\right)^m \\geqslant 1+m \\cdot \\frac{n-l}{l}>2,\n$$\n与式\\ref{eq1}矛盾.\n同样地, 若 $n\\frac{n}{l-n}$, 就有 $\\left(\\frac{l}{n}\\right)^m>2$, 即 $\\left(\\frac{n}{l}\\right)^m<\\frac{1}{2}$, 也与 式\\ref{eq1}矛盾.\n所以, 只能是 $n=l$.\n综上可知, 对任意 $n \\in \\mathbf{N}^*$, 都有 $f(n)=n$.\n说明如果函数 $f$ 是 $\\mathbf{N}^*$ 到 $\\mathbf{N}^*$ 的映射, 那么问题要简单得多, 请读者给出证明.\n类似地,用此方法还可证明著名的 Jenson 不等式.", + "remark": "", + "figures": [] +} \ No newline at end of file diff --git a/processed_dataset/proof/1548.json b/processed_dataset/proof/1548.json new file mode 100644 index 0000000000000000000000000000000000000000..54f461817a44606fba9141a584c89f13b7b1c00c --- /dev/null +++ b/processed_dataset/proof/1548.json @@ -0,0 +1,8 @@ +{ + "source_file": "./raw_volume-zh/volume6/chapter2-2.tex", + "problem_type": "proof", + "problem": "例2. 求所有的函数 $f: \\mathbf{N}^* \\rightarrow \\mathbf{N}^*$, 使得对任意 $m 、 n \\in \\mathbf{N}^*$, 都有\n$$\nf(m)^2+f(n) \\mid\\left(m^2+n\\right)^2 . \\label{eq1}\n$$", + "solution": "解:设 $f$ 是一个满足条件的函数, 在式\\ref{eq1}中令 $m=n==1$, 就有 $\\left(f(1)^2+\\right. f(1)) \\mid 4$, 即 $f(1)(f(1)+1) \\mid 4$, 由于 $f(1) \\geqslant 2$ 导致 $f(1)(f(1)+1) \\geqslant 6$, 故只能是 $f(1)=1$.\n下面我们先证明: 对任意素数 $p$, 都有 $f(p-1)=p-1, \\label{eq2}$.\n事实上, 对任意素数 $p$, 在 式\\ref{eq1} 中令 $m=1, n=p-1$, 则 $(f(1)+ f(p-1)) \\mid p^2$, 即 $(1+f(p-1)) \\mid p^2$, 从而 $f(p-1)+1=p$ 或 $p^2$, 若为前者, 则 (2) 已成立; 若为后者, 即 $f(p-1)=p^2-1$, 此时, 在 式\\ref{eq1} 中令 $m=p-1, n=$ 1 , 就有 $\\left(f(p-1)^2+f(1)\\right) \\mid\\left((p-1)^2+1\\right)^2$, 即 $\\left(\\left(p^2-1\\right)^2+1\\right) \\mid\\left((p-1)^2+1\\right)^2$, 但是 $\\left((p-1)^2+1\\right)^2 \\leqslant\\left((p-1)^2+(p-1)\\right)^2=p^2(p-1)^2<(p+1)^2(p- 1)^2+1=\\left(p^2-1\\right)^2+1$,矛盾.\n所以, 式\\ref{eq2} 成立.\n再证明: 对任意 $n \\in \\mathbf{N}^*$, 都有 $f(n)=n$.\n事实上,对任意正整数 $n$, 取 $k \\in \\mathbf{N}^*$, 使得 $k+1$ 为素数 (这样的 $k$ 有无穷多个), 在 式\\ref{eq1} 中令 $m=k$, 结合 式\\ref{eq2} 就有\n$$\n\\left(k^2+f(n)\\right) \\mid\\left(k^2+n\\right)^2 . \\label{eq3}\n$$\n注意到 $\\left(k^2+n\\right)^2=\\left(k^2+f(n)+n-f(n)\\right)^2=A\\left(k^2+f(n)\\right)+(n-f(n))^2$, 这里 $A$ 是某个整数.\n这样, 由式\\ref{eq3}知\n$$\n\\left(k^2+f(n)\\right) \\mid(n-f(n))^2 .\n$$\n上式表明, 数 $(n-f(n))^2$ 可以被无穷多个正整数整除(因为 $k$ 有无穷多种取法), 所以 $(n-f(n))^2=0$, 即 $f(n)=n$.\n综上可知, 只有一个函数满足条件, 即 $f(n)=n$.\n说明此题本质上只需证出对无穷多个 $k \\in \\mathbf{N}^*$, 有 $f(k)=k$, 然后将其余的漏洞补上, 选择式\\ref{eq2}作为突破口是希望让被除数的因数个数尽量少, 这个技巧在整除理论中经常用到.", + "remark": "", + "figures": [] +} \ No newline at end of file diff --git a/processed_dataset/proof/1549.json b/processed_dataset/proof/1549.json new file mode 100644 index 0000000000000000000000000000000000000000..2ab1720691c19e322e9aca5635e896b73b9439fa --- /dev/null +++ b/processed_dataset/proof/1549.json @@ -0,0 +1,8 @@ +{ + "source_file": "./raw_volume-zh/volume6/chapter2-2.tex", + "problem_type": "proof", + "problem": "例3. 求所有的函数 $f: \\mathbf{N}^* \\rightarrow \\mathbf{N}^*$, 使得对任意 $n \\in \\mathbf{N}^*$ 及素数 $p$, 都有\n$$\nf(n)^p \\equiv n(\\bmod f(p)) . \\label{eq1}\n$$", + "solution": "解:对任意素数 $p$, 在式\\ref{eq1}中取 $n=p$, 可知\n$$\np \\equiv f(p)^p \\equiv 0(\\bmod f(p)) .\n$$\n故 $f(p) \\mid p$, 从而 $f(p)=1$ 或者 $p$.\n现在记 $S=\\{p \\mid p$ 为素数, $f(p)=p\\}$, 依下面的三种情形讨论:\n情形一 $S$ 是一个无限集.\n我们利用上例的方法证明: 对任意 $n \\in \\mathbf{N}^*$, 都有 $f(n)=n$.\n事实上, 此时存在无穷多个素数 $p$, 使得 $f(p)=p$, 因此, 对任意 $n \\in \\mathbf{N}^*$, 都存在无穷多个素数 $p$, 使得 $n \\equiv f(n)^p(\\bmod p)$. 利用费马 (Fermat) 小定理, 有 $f(n)^p \\equiv f(n)(\\bmod p)$. 所以 $f(n) \\equiv n(\\bmod p)$, 这表明 $f(n)-n$ 是无穷多个素数的倍数,故 $f(n)=n$.\n情形二 $S$ 为空集.\n则对任意素数 $p$, 都有 $f(p)=1$. 此时, 对其余的正整数 $n, f(n)$ 可取任意正整数 (\\ref{eq1} 式都满足).\n情形三 $S$ 是一个非空有限集.\n设 $p$ 为 $S$ 中最大的素数, 若 $p \\geqslant 3$, 我们证明这将导致矛盾, 从而, 得到 $S=\\{2\\}$.\n由 $p$ 的最大性, 知对任意素数 $q>p$, 都有 $f(q)=1$, 由 式\\ref{eq1} 得, $q \\equiv f(q)^p \\equiv 1(\\bmod p)$, 即 $q \\equiv 1(\\bmod p)$.\n现在记 $Q$ 为所有不超过 $p$ 的奇素数之积, 则 $Q+2$ 的每一个素因子都大于 $p$ (注意, 这里用到 $p \\geqslant 3$ ), 这样, 结合上面得出的结论知 $Q+-2 \\equiv 1(\\bmod p)$, 导致 $p \\mid Q+1$,与 $p \\mid Q$ 矛盾.\n上述讨论表明 $S=\\{2\\}$, 知 $f(2)=2$, 而对奇.\n数 $p$, 都有 $f(p)=1$. 由 式\\ref{eq1} 知, 只需 $f(n)^2 \\equiv n(\\bmod 2)$. 因此, 对其余的正整数 $n, f(n)$ 只需取与 $n$ 同奇偶的正整数即可.\n直接验证, 可知每一种情形所得的函数都符合要求, 它们即为所求.\n说明求 $\\mathbf{N}^* \\rightarrow \\mathbf{N}^*$ 上的函数, 本质上是讨论正整数数列 $\\{f(n)\\}$ 相关的问题, 这里都采用了素因数分析的方法, 它属于数论方法的迁移, 同样, 求解函数方程的一些思路也可用来讨论这类问题.", + "remark": "", + "figures": [] +} \ No newline at end of file diff --git a/processed_dataset/proof/1550.json b/processed_dataset/proof/1550.json new file mode 100644 index 0000000000000000000000000000000000000000..4e45a12232ee74403102448cd8ecf0d07e433b63 --- /dev/null +++ b/processed_dataset/proof/1550.json @@ -0,0 +1,8 @@ +{ + "source_file": "./raw_volume-zh/volume6/chapter2-2.tex", + "problem_type": "proof", + "problem": "例4. 设 $k$ 为给定的正整数, 求所有的函数 $f: \\mathbf{N}^* \\rightarrow \\mathbf{N}^*$, 使得对任意 $m$ 、 $n \\in \\mathbf{N}^*$, 都有\n$$\n(f(m)+f(n)) \\mid(m+n)^k . \\label{eq1}\n$$", + "solution": "解:显然, 函数 $f(n)=n$ 符合条件, 它是否为满足条件的唯一函数呢? 下证明之.\n先证一个结论: $f$ 是一个单射.\n事实上, 若存在 $a 、 b \\in \\mathbf{N}^*$, 使得 $a \\neq b$, 但是 $f(a)=f(b)$, 则由 式\\ref{eq1} 知, 对任意 $n \\in \\mathbf{N}^*$, 都有\n$$\n(f(a)+f(n))\\left|(a+n)^k,(f(b)+f(n))\\right|(b+n)^k .\n$$\n因此,对任意 $n \\in \\mathbf{N}^*$, 数 $f(a)+f(n)$ 都是 $(a+n)^k$ 与 $(b+n)^k$ 的公因数.\n现在取一个素数 $p>\\max \\{a,|b-a|\\}$, 然后令 $n=p-a$. 由于 $(a+n$, $b+n)=(a+n, b-a)=(p, b-a)=1$, 故 $\\left((a+n)^k,(b+n)^k\\right)=1$, 导致 $f(a)+f(n)=1$, 与 $f(a) 、 f(n)$ 都为正整数矛盾.\n所以, $f$ 是一个单射.\n再证明: 对任意 $m \\in \\mathbf{N}^*$, 有 $|f(m+1)-f(m)|=1$.\n在式\\ref{eq1}中分别用 $(m, n)$ 和 $(m+1, n)$ 的结论, 有\n$$\n(f(m)+f(n))\\left|(m+n)^k,(f(m+1)+f(n))\\right|(m+1+n)^k . \n$$\n而 $(m+n, m+1+n)=1$, 故 $(f(m)+f(n), f(m+1)+f(n))=1$, 进而, 当 $m$ 固定时,对任意 $n \\in \\mathbf{N}^*$, 有\n$$\n(f(n)+f(m), f(m+1)-f(m))=1 . \\label{eq2}\n$$\n如果 $|f(m+1)-f(m)| \\neq 1$, 那么存在素数 $p$, 使得 $p|| f(m+1)- f(m) \\mid$. 现在取 $\\alpha \\in \\mathbf{N}^*$, 使得 $n=p^\\alpha-m$ 为正整数, 则由 式\\ref{eq1} 知 $f(n)+f(m) \\mid p^{\\alpha k}$, 从而 $f(n)+f(m)=p^l$, 这里 $l$ 为某个正整数, 导致\n$$\n(f(n)+f(m), f(m+1)-f(m))=\\left(p^l, f(m+1)-f(m)\\right) \\geqslant p .\n$$\n与\\ref{eq2}式矛盾.\n最后,我们证明: 对任意 $n \\in \\mathbf{N}^*$, 都有 $f(n)=n$.\n由前面的结论知, 对任意 $m \\in \\mathbf{N}^*$, 都有 $f(m+1)-f(m)=1$ 或者 $f(m+1)-f(m)=-1$. 如果这两种情形在同一个符合要求的函数 $f$ 中都出现, 那么存在 $m \\in \\mathbf{N}^*$, 使得 $(f(m+1)-f(m), f(m+2)-f(m+1))= (1,-1)$ 或者 $(-1,1)$, 都导致 $f(m+2)=f(m)$, 与 $f$ 为单射矛盾.\n所以,要么对任意 $m \\in \\mathbf{N}^*$, 都有 $f(m+1)-f(m)=1$, 要么对任意 $m \\in \\mathbf{N}^*$, 都有\n$$\nf(m+1)-f(m)=-1 \\text {. }\n$$\n注意到, $f$ 是 $\\mathbf{N}^* \\rightarrow \\mathbf{N}^*$ 上的函数, 知只能是 : 对任意 $m \\in \\mathbf{N}^*$, 都有 $f(m+ 1)-f(m)=1$. 依此可知, 对任意 $n \\in \\mathbf{N}^*$, 都有 $f(n)=n+c$ (这里 $c= f(1)-1 \\geqslant 0)$.\n如果 $c>0$, 我们取一个素数 $p>2 c$, 利用 (1) 知 $f(1)+f(p-1) \\mid p^k$, 得 $p+2 c \\mid p^k$, 这要求 $p+2 c$ 为 $p$ 的幂次, 从而 $p \\mid p+2 c$, 导致 $p \\mid 2 c$, 矛盾.\n所以 $c=0$.\n综上可知, 只有函数 $f(n)=n$ 符合要求.", + "remark": "", + "figures": [] +} \ No newline at end of file diff --git a/processed_dataset/proof/1551.json b/processed_dataset/proof/1551.json new file mode 100644 index 0000000000000000000000000000000000000000..2ad1419e697643e89fc4b00b5c7565ca60156467 --- /dev/null +++ b/processed_dataset/proof/1551.json @@ -0,0 +1,8 @@ +{ + "source_file": "./raw_volume-zh/volume6/chapter2-3.tex", + "problem_type": "proof", + "problem": "例1. 证明: 对任意正整数 $n \\geqslant 3$,都存在一个完全立方数,它可以表示为 $n$ 个正整数的立方和.", + "solution": "证明:对比不定方程 $x^3+y^3=z^3$ 没有正整数解的结论, 可了解问题的背景.\n当 $n=3$ 时,由 $3^3+4^3+5^3=6^3$ 可知命题对 $n=3$ 成立;\n当 $n=4$ 时, 由 $5^3+7^3+9^3+10^3=13^3$ (这个等式是 Euler 最早发现的), 可知命题对 $n=4$ 成立.\n现设命题对 $n(\\geqslant 3)$ 成立, 即存在正整数 $x_1) 所示, 设 $O$ 为正三角形 $A B C$ 的外心, $D 、 E$ 分别是 $B C 、 C A$ 的中点, $F$ 为 $B O$ 的中点.\n利用直角三角形斜边上的中线等于斜边的一半, 可知 $\\triangle A B O 、 \\triangle B F D 、 \\triangle F O D 、 \\triangle D E C$ 和 $\\triangle A D E$ 都是等腰三角形.\n故命题对 $n=5$ 成立.\n现设每一个正三角形都能剖分为 $n(\\geqslant 3)$ 个等腰三角形 (即命题对 $n$ 成立), 则对正三角形 $A B C$, 设 $D$ 、 $E 、 F$ 分别为 $B C 、 C A 、 A B$ 的中点, 并将正三角形 $A E F$\n依归纳假设剖分为 $n$ 个等腰三角形, 将这 $n$ 个三角形与 $\\triangle B D F 、 \\triangle C D E$ 、 $\\triangle D E F$ 合并, 即构成正三角形 $A B C$ 的一个个数为 $n+3$ 的等腰三角形剖分.\n故命题对 $n+3$ 成立.\n综上所述, 对任意 $n \\geqslant 3$, 命题成立.", + "remark": "", + "figures": [ + "./images/volume6/figures/fig-c2i5.png" + ] +} \ No newline at end of file diff --git a/processed_dataset/proof/1553.json b/processed_dataset/proof/1553.json new file mode 100644 index 0000000000000000000000000000000000000000..4b347895378d51f15777510d3a50d3068dc6eef0 --- /dev/null +++ b/processed_dataset/proof/1553.json @@ -0,0 +1,8 @@ +{ + "source_file": "./raw_volume-zh/volume6/chapter2-3.tex", + "problem_type": "proof", + "problem": "例3. 证明: 对任意 $n \\in \\mathbf{N}^*$, 不定方程\n$$\nx^2+y^2=z^n . \\label{eq1}\n$$\n有无穷多组正整数解.", + "solution": "证明:当 $n=1$ 时, 对任意 $x 、 y \\in \\mathbf{N}^*,\\left(x, y, x^2+y^2\\right)$ 都是 式\\ref{eq1} 的正整数解; 当 $n=2$ 时, 取 $m>n \\geqslant 1, m 、 n \\in \\mathbf{N}^*$, 令 $x=m^2-n^2, y=2 m m$, $z=m^2+n^2$, 就有 $x^2+y^2=z^2$, 故命题对 $n=1 、 2$ 成立.\n现设命题对 $n$ 成立, 对正整数 $x 、 y 、 z$, 若 $x^2+y^2=z^n$, 则 $(x z)^2+ (y z)^2=z^{n+2}$, 因此不定方程 $x^2+y^2=z^{n+2}$ 有无穷多组正整数解.\n结合命题对 $n=1,2$ 成立, 可知命题对任意 $n \\in \\mathbf{N}^*$ 成立.\n说明此题还可依下述方法处理: 令 $z=a+b \\mathrm{i}$, 其中 $a 、 b \\in \\mathbf{N}^*$ 且 $0< \\arg z<\\frac{\\pi}{n}$ (这样的 $a 、 b$ 对有无穷多对使得 $a^2+b^2$ 的值彼此不同), 则由二项式定理, 可写 $z^n=(a+b \\mathrm{i})^n=x+y \\mathrm{i}, x, y \\in \\mathbf{Z}$, 且 $x y \\neq 0$ (这是因为 $\\left.\\arg z^n \\in(0, \\pi)\\right)$, 两边取模, 可知 $\\left(\\sqrt{a^2+b^2}\\right)^n=\\sqrt{x^2+y^2}$, 即有 $x^2+y^2= \\left(a^2+b^2\\right)^n$, 故 $\\left(|x|,|y|, a^2+b^2\\right)$ 是 $x^2+y^2=z^n$ 的正整数解.", + "remark": "", + "figures": [] +} \ No newline at end of file diff --git a/processed_dataset/proof/1554.json b/processed_dataset/proof/1554.json new file mode 100644 index 0000000000000000000000000000000000000000..396aa32305836c4382a3fb0e904b2abbd620722f --- /dev/null +++ b/processed_dataset/proof/1554.json @@ -0,0 +1,8 @@ +{ + "source_file": "./raw_volume-zh/volume6/chapter2-3.tex", + "problem_type": "proof", + "problem": "例4. 求所有的函数 $f: \\mathbf{N} \\rightarrow \\mathbf{N}$, 使得\n(1) 对任意 $m 、 n \\in \\mathbf{N}$, 都有 $f\\left(m^2+n^2\\right)=f(m)^2+f(n)^2$;\n(2) $f(1)>0$.", + "solution": "解:在 (1) 中令 $m=n=0$, 得 $f(0)=2 f(0)^2$, 故 $f(0)=0$ 或 $\\frac{1}{2}$, 但 $f(0) \\in \\mathbf{N}$, 故 $f(0)=0$. 于是, 由 (1) 知, 对任意 $m \\in \\mathbf{N}$, 都有 $f\\left(m^2\\right)=f(m)^2$. 现在先计算 $n \\in\\{1,2, \\cdots, 10\\}$ 时, $f(n)$ 的值.\n由条件及前面推出的结论知 $f(1)=f\\left(1^2\\right)=f(1)^2$, 而 $f(1)>0$, 故 $f(1)=1$. 进而, 依次有\n$$\n\\begin{aligned}\n& f(2)=f\\left(1^2+1^2\\right)=f(1)^2+f(1)^2=1+1=2 ; \\\\\n& f(4)=f\\left(2^2\\right)=f(2)^2=4 ; \\\\\n& f(5)=f\\left(2^2+1^2\\right)=f(2)^2+f(1)^2=5 ; \\\\\n& f(8)=f\\left(2^2+2^2\\right)=f(2)^2+f(2)^2=8 .\n\\end{aligned}\n$$\n又由\n$$\n25=f(5)^2=f\\left(5^2\\right)=f\\left(3^2+4^2\\right)=f(3)^2+f(4)^2=f(3)^2+16,\n$$\n结合 $f(3) \\in \\mathbf{N}$, 知 $f(3)=3$. 进而 $f(9)=f(3)^2=9, f(10)=f\\left(3^2+1^2\\right)= f(3)^2+f(1)^2=10$.\n利用 $7^2+1^2=5^2+5^2$ 及条件(1) 可算出 $f(7)=7$, 再由 $10^2=6^2+8^2$, 知 $f(10)^2=f(6)^2+f(8)^2$, 解得 $f(6)=6$.\n所以, 对任意 $0 \\leqslant n \\leqslant 10$, 都有 $f(n)=n$.\n下面选用跨度为 5 的方法来证明: 对任意 $n \\in \\mathbf{N}$, 都有 $f(n)=n$.\n为此需要用到下面的一些等式\n$$\n\\begin{aligned}\n& (5 k+1)^2+2^2=(4 k+2)^2+(3 k-1)^2 ; \\\\\n& (5 k+2)^2+1^2=(4 k+1)^2+(3 k+2)^2 ; \\\\\n& (5 k+3)^2+1^2=(4 k+3)^2+(3 k+1)^2 ; \\\\\n& (5 k+4)^2+2^2=(4 k+2)^2+(3 k+4)^2 ; \\\\\n& (5 k+5)^2=(4 k+4)^2+(3 k+3)^2 .\n\\end{aligned}\n$$\n这些等式中右边的每一项在 $k \\geqslant 2$ 时都小于左边的第一项, 因此, 利用条件 (1) 及归纳假设,我们每次可以确定这些等式左边第一项的函数值.\n即每次归纳向后推 5 个数都成立.\n所以,对每个 $n \\in \\mathbf{N}$, 都有 $f(n)=n$.\n说明从上面的例子可以发现, 所谓用跨度为 $k$ 的方法去证 $P(n)$ 成立, 本质上是将 $\\{P(n)\\}$ 分划为 $k$ 组命题再分别予以证明, 当然, 如果将此想法与第二数学归纳法结合, 各组命题之间还可以相互利用, 本例中就体现了这个思想.", + "remark": "", + "figures": [] +} \ No newline at end of file diff --git a/processed_dataset/proof/1555.json b/processed_dataset/proof/1555.json new file mode 100644 index 0000000000000000000000000000000000000000..ddbdb5d07976b15d6bb26c012af68f08265bb3cf --- /dev/null +++ b/processed_dataset/proof/1555.json @@ -0,0 +1,8 @@ +{ + "source_file": "./raw_volume-zh/volume6/chapter2-4.tex", + "problem_type": "proof", + "problem": "例1. 设 $m, n \\in \\mathbf{N}^*$. 证明: 对任意正实数 $x_1, \\cdots, x_n ; y_1, \\cdots, y_n$. 若 $x_i+ y_i=1, i=1,2, \\cdots, n$, 则\n$$\n\\left(1-x_1 \\cdots x_n\\right)^m+\\left(1-y_1^m\\right)\\left(1-y_2^m\\right) \\cdots\\left(1-y_n^m\\right) \\geqslant 1 . \\label{eq1}\n$$", + "solution": "证明:对 $n$ 归纳.\n当 $n=1$ 时, 由条件知\n$$\n\\left(1-x_1\\right)^m+\\left(1-y_1^m\\right)=y_1^m+\\left(1-y_1^m\\right)=1 .\n$$\n故式\\ref{eq1}对 $n=1$ 成立.\n现设式\\ref{eq1}对 $n-1(n \\geqslant 2)$ 成立, 考虑 $n$ 的情形.\n$$\n\\begin{aligned}\n& \\left(1-x_1 \\cdots x_n\\right)^m+\\left(1-y_1^m\\right) \\cdots\\left(1-y_n^m\\right) \\\\\n= & \\left(1-x_1 \\cdots x_{n-1}\\left(1-y_n\\right)\\right)^m+\\left(1-y_1^m\\right) \\cdots\\left(1-y_n^m\\right) \\\\\n\\geqslant & \\left(1-x_1 \\cdots x_{n-1}+x_1 \\cdots x_{n-1} y_n\\right)^m+\\left(1-\\left(1-x_1 \\cdots x_{n-1}\\right)^m\\right)\\left(1-y_n^m\\right) .\n\\end{aligned}\n$$\n记 $a=1-x_1 \\cdots x_{n-1}, b=y_n$, 由上式知为证 式\\ref{eq1} 对 $n$ 成立, 只需证明:\n$$\n(a+b-a b)^m+\\left(1-a^m\\right)\\left(1-b^m\\right) \\geqslant 1\n$$\n对任意 $a, b \\in(0,1)$ 都成立.\n即证\n$$\n(a+b-a b)^m \\geqslant a^m+b^m-a^m b^m . \\label{eq2}\n$$\n对式\\ref{eq2}处理时,我们通过对 $m$ 归纳来进行.\n当 $m=1$ 时, 式\\ref{eq2} 显然成立; 现设 式\\ref{eq2} 对 $m-1(m \\geqslant 2)$ 成立, 则\n$$\n\\begin{aligned}\n& (a+b-a b)^m-a^m-b^m+a^m b^m \\\\\n\\geqslant & \\left(a^{m-1}+b^{m-1}-a^{m-1} b^{m-1}\\right)(a+b-a b)-a^m-b^m+a^m b^m \\\\\n= & 2 a^m b^m+a b^{m-1}+b a^{m-1}-a^m b^{m-1}-a^{m-1} b^m-a^m b-a b^m \\\\\n= & \\left(b^{m-1}-b^m\\right)\\left(a-a^m\\right)+\\left(a^{m-1}-a^m\\right)\\left(b-b^m\\right) .\n\\end{aligned}\n$$\n注意到 $a, b \\in(0,1)$, 故 $b^{m-1} \\geqslant b^m, a \\geqslant a^m, a^{m-1} \\geqslant a^m, b \\geqslant b^m$, 所以 $(a+b- a b)^m \\geqslant a^m+b^m-a^m b^m$, 即式\\ref{eq2}对 $m$ 成立.\n综上可知, 命题 式\\ref{eq1}成立.\n说明这个与两个正整数变量有关的问题, 选择对 $n$ 归纳 (视 $m$ 为常数) 是容易想到的, 因为这时式\\ref{eq1}左边的第 2 个加项在作归纳过渡时显得容易处理些.", + "remark": "", + "figures": [] +} \ No newline at end of file diff --git a/processed_dataset/proof/1556.json b/processed_dataset/proof/1556.json new file mode 100644 index 0000000000000000000000000000000000000000..3e7473d1fd3bd3d7e410393635eb8510ff7179c3 --- /dev/null +++ b/processed_dataset/proof/1556.json @@ -0,0 +1,8 @@ +{ + "source_file": "./raw_volume-zh/volume6/chapter2-4.tex", + "problem_type": "proof", + "problem": "例2. 证明: 对任意 $m, n \\in \\mathbf{N}^*$, 数\n$$\nS(m, n)=\\sum_{i=0}^{2^{n-1}-1}\\left(\\tan \\frac{(2 i+1) \\pi}{2^{n+1}}\\right)^{2 m}\n$$\n都为正整数.", + "solution": "证明:选择 $n$ 作为归纳对象.\n当 $n=1$ 时, $S(m, 1)=\\left(\\tan \\frac{\\pi}{4}\\right)^{2 m}=1$, 故命题对 $n=1$ 及 $m \\in \\mathbf{N}^*$ 成立.\n现设命题对 $n-1$ 及 $m \\in \\mathbf{N}^*$ 成立, 考虑 $n$ 的情形.\n由 $\\cot 2 \\alpha=\\frac{1-\\tan ^2 \\alpha}{2 \\tan \\alpha}=\\frac{1}{2}(\\cot \\alpha-\\tan \\alpha)$ 可知\n$$\n\\tan ^2\\left(\\frac{\\pi}{2}-2 \\alpha\\right)=\\frac{1}{4}(\\cot \\alpha-\\tan \\alpha)^2=\\frac{1}{4}\\left(\\tan ^2 \\alpha+\\cot ^2 \\alpha-2\\right),\n$$\n所以,利用首尾配对求和, 知\n$$\n\\begin{aligned}\nS(1, n) & =\\frac{1}{2} \\sum_{i=0}^{2^{n-1}-1}\\left(\\tan ^2 \\frac{(2 i+1) \\pi}{2^{n+1}}+\\tan ^2 \\frac{\\left(2\\left(2^{n-1}-1-i\\right)+1\\right) \\pi}{2^{n+1}}\\right) \\\\\n& =\\frac{1}{2} \\sum_{i=0}^{2^{n-1}-1}\\left(\\tan ^2 \\frac{(2 i+1) \\pi}{2^{n+1}}+\\tan ^2\\left(\\frac{\\pi}{2}-\\frac{(2 i+1) \\pi}{2^{n+1}}\\right)\\right) \\\\\n& =\\frac{1}{2} \\sum_{i=0}^{2^{n-1}-1}\\left(\\tan ^2 \\frac{(2 i+1) \\pi}{2^{n+1}}+\\cot ^2 \\frac{(2 i+1) \\pi}{2^{n+1}}\\right) \\\\\n& =\\frac{1}{2} \\sum_{i=0}^{2^{n-1}-1}\\left(4 \\tan ^2 \\frac{(2 i+1) \\pi}{2^n}+2\\right) \\\\\n& =2 \\sum_{i=0}^{2^{n-1}-1} \\tan ^2 \\frac{(2 i+1) \\pi}{2^n}+2^{n-1} \\\\\n& =2 \\sum_{i=0}^{2^{n-2}-1}\\left(\\tan ^2 \\frac{(2 i+1) \\pi}{2^n}+\\tan ^2 \\frac{\\left(2\\left(2^{n-1}-1-i\\right)+1\\right) \\pi}{2^n}\\right)+2^{n-1} \\\\\n& =2 \\sum_{i=0}^{2^{n-2}-1}\\left(\\tan ^2 \\frac{(2 i+1) \\pi}{2^n}+\\tan ^2\\left(\\pi-\\frac{(2 i+1) \\pi}{2^n}\\right)\\right)+2^{n-1} \\\\\n& =4 \\sum_{i=0}^{2^{n-2}-1} \\tan ^2 \\frac{(2 i+1) \\pi}{2^n}+2^{n-1}=4 S(1, n-1)+2^n .\n\\end{aligned}\n$$\n从而 $S(1, n) \\in \\mathbf{N}^*$.\n下面设 $S(1, n), S(2, n), \\cdots, S(m-1, n)$ 都为正整数, 考虑 $S(m, n)$.\n注意到, 对 $k \\in \\mathbf{N}$, 由二项式定理有\n$$\n\\left(x+x^{-1}\\right)^k=\\mathrm{C}_k^0\\left(x^k+x^{-k}\\right)+\\mathrm{C}_k^1\\left(x^{k-1}+x^{-(k-1)}\\right)+\\cdots, \\label{eq1}\n$$\n因此\n$$\n\\begin{aligned}\n& \\left(\\frac{1}{4}\\left(x+x^{-1}-2\\right)\\right)^m \\\\\n= & \\frac{1}{4^m} \\sum_{k=0}^m \\mathrm{C}_m^k\\left(x+x^{-1}\\right)^k \\cdot(-2)^{m-k} \\\\\n= & \\frac{1}{4^m}\\left(\\left(x^m+x^{-m}\\right)+b_1\\left(x^{m-1}+x^{-(m-1)}\\right)+\\cdots+b_{m-1}\\left(x+x^{-1}\\right)+b_m\\right),\n\\end{aligned}\n$$\n这里 $b_1, \\cdots, b_m \\in \\mathbf{Z}$, 并用到式\\ref{eq1}的结论.\n在上式中令 $x=\\tan ^2 \\frac{(2 i+1) \\pi}{2^{n+1}}$, 并对 $i=0,1,2, \\cdots, 2^{n-2}-1$ 求和, 利用 $S(1, n)$ 中类似的计算, 可知\n$$\nS(m, n-1)=\\frac{1}{4^m}\\left(S(m, n)+b_1 S(m-1, n)+\\cdots+b_{m-1} S(1, n)+b_m\\right),\n$$\n得\n$$\nS(m, n)=4^m \\cdot S(m, n-1)-b_1 S(m-1, n)-\\cdots-b_{m-1} S(1, n)-b_m .\n$$\n从而 $S(m, n) \\in \\mathbf{Z}$, 而 $S(m, n)$ 中每一项都大于零, 故 $S(m, n) \\in \\mathbf{N}^*$.\n综上可知, 对任意 $m 、 n \\in \\mathbf{N}^*$, 数 $S(m, n)$ 都为正整数,命题获证.\n说明本质上在从 $n-1$ 过渡到 $n$ 的过程中, 我们采用了对 $m$ 再归纳的方法.\n在双变量命题中, 这种处理是利用数学归纳法处理时常见的方法.", + "remark": "", + "figures": [] +} \ No newline at end of file diff --git a/processed_dataset/proof/1557.json b/processed_dataset/proof/1557.json new file mode 100644 index 0000000000000000000000000000000000000000..5a4e8bfe641a709b2b0f085289b168958d542e52 --- /dev/null +++ b/processed_dataset/proof/1557.json @@ -0,0 +1,8 @@ +{ + "source_file": "./raw_volume-zh/volume6/chapter2-4.tex", + "problem_type": "proof", + "problem": "例3. 设 $t$ 个非负整数 $a_1, a_2, \\cdots, a_t$ 满足\n$$\na_i+a_j \\leqslant a_{i+j} \\leqslant a_i+a_j+1,\n$$\n这里 $1 \\leqslant i, j \\leqslant t, i+j \\leqslant t$.\n求证: 存在 $x \\in \\mathbf{R}$, 使得对任意 $n \\in\\{1,2, \\cdots, t\\}$, 都有 $a_n=[n x]$.", + "solution": "证明:记 $I_n=\\left[\\frac{a_n}{n}, \\frac{a_n+1}{n}\\right), n=1,2, \\cdots, t$, 要求证明存在实数\n$$\nx \\in \\bigcap_{n=1}^t I_n . \\label{eq1}\n$$\n现设 $L=\\max _{1 \\leqslant n \\leqslant t} \\frac{a_n}{n}, U=\\min _{1 \\leqslant n \\leqslant t} \\frac{a_n+1}{n}$, 若能证明: $Ln$, 那么由归纳假设知 $(m-n) a_nt$, 注意到 $(s-t, t)=1$, 于是, 由归纳假设知, 存在有理数数列 $b_1, b_2, \\cdots, b_{(s-1) d-1}$, 使得其中任意连续 $(s-t) d$ 项之和为负数, 连续 $t d$ 项之和为正数.\n我们证明: 存在有理数 $a_1, a_2, \\cdots, a_{t d}$, 使得下述不等式组成立.\n$$\n\\left\\{\\begin{array}{l}a_{d+1}+\\cdots+a_1+b_1+\\cdots+b_{(s-1) d-1}<0, \\\\ a_{d+2}+\\cdots+a_1+b_1+\\cdots+b_{(s-1) d-2}<0, \\\\ \\cdots \\cdots \\cdots \\cdots \\cdots \\cdots \\cdots \\cdots \\cdots \\cdots \\cdots \\cdots \\cdots \\cdots \\cdots \\cdots \\cdots \\cdots \\cdots \\cdots \\cdots \\cdots \\cdots \\cdots \\cdots \\cdots \\cdots \\cdots \\cdots \\cdots \\cdots \\cdots \\cdots \\\\ a_{t d}+\\cdots+a_1+b_1+\\cdots+b_{(s-t) d}<0 .\\end{array}\\right. \\label{eq1}\n$$\n而且\n$$\n\\left\\{\\begin{array}{l}\na_{t d}+\\cdots+a_1>0, \\\\\na_{t d-1}+\\cdots+a_1+b_1>0, \\\\\n\\cdots \\cdots \\cdots \\cdots \\cdots \\cdots \\cdots \\cdots \\cdots \\cdots \\cdots \\cdots \\cdots \\cdots \\cdots \\cdots \\cdots \\cdots \\\\\na_1+b_1+\\cdots+b_{t d-1}>0 .\n\\end{array}\\right. \\label{eq2}\n$$\n这样, 数列 $a_{t d}, a_{t d-1}, \\cdots, a_1, b_1, \\cdots, b_{(s-1) d-1}$ 是一个满足命题的长为 $(s+t-$ 1) $d-1$ 的数列, 从而命题获证.\n事实上,要不等式组 式\\ref{eq1}、\\ref{eq2} 同时成立, 我们只需分别选取有理数 $a_1$, $a_2, \\cdots, a_d$, 使得 $a_1>-\\left(b_1+\\cdots+b_{t d-1}\\right), a_2>-\\left(a_1+b_1+\\cdots+b_{t d-2}\\right), \\cdots$, $a_d>-\\left(a_{d-1}+\\cdots+a_1+b_1+\\cdots+b_{(t-1) d}\\right)$, 再取 $a_{d+1}$, 使得 $a_{d+1}$ 为有理数, 且\n$$\n\\begin{gathered}\n-\\left(a_d+\\cdots+a_1+b_1+\\cdots+b_{(t-1) d-1}\\right) \\\\\n0$ (这里用到归纳假设), 故满足条件的 $a_{d+1}$ 存在.\n依此类推, 可知满足不等式组 式\\ref{eq1} 与 \\ref{eq2} 的有理数 $a_1, \\cdots, a_{t d}$ 存在.\n回到原题, 满足条件的整数数列最多有 $m+n-(m, n)-1$ 项.\n说明此题中 $m=11, n=6$ 的情形曾作为竞赛题出现过, 由于 $m==11$,\n$n=6$ 时的例子容易得到, 而对一般的 $m 、 n$ 例子却非常难找到, 因此当此例在 2000 年国家集训队的测验中出现时,做出的同学非常少.", + "remark": "", + "figures": [] +} \ No newline at end of file diff --git a/processed_dataset/proof/1559.json b/processed_dataset/proof/1559.json new file mode 100644 index 0000000000000000000000000000000000000000..e331bae26facfc7a4c4931a0fb07b25cd4deed71 --- /dev/null +++ b/processed_dataset/proof/1559.json @@ -0,0 +1,8 @@ +{ + "source_file": "./raw_volume-zh/volume6/chapter2-5.tex", + "problem_type": "proof", + "problem": "例1. 证明: 对任意正整数 $n$, 都有\n$$\n\\frac{1}{2} \\cdot \\frac{3}{4} \\cdot \\cdots \\cdot \\frac{2 n-1}{2 n}<\\frac{1}{\\sqrt{3 n}} . \\label{eq1}\n$$", + "solution": "证明:如果直接处理, 那么为实现归纳过渡, 需要不等式 $\\frac{2 n+1}{2(n+1)} \\cdot \\frac{1}{\\sqrt{3 n}} \\leqslant \\frac{1}{\\sqrt{3(n+1)}}$ 成立, 这要求 $(n+1)(2 n+1)^2 \\leqslant n(2 n+2)^2$, 而这等价于 $(2 n+1)^2 \\leqslant n(4 n+3)$. 但此不等式不成立.\n所以, 直接用数学归纳法难以证出 式\\ref{eq1} 成立.\n我们证明式\\ref{eq1}的加强命题:\n$$\n\\frac{1}{2} \\cdot \\frac{3}{4} \\cdot \\cdots \\cdot \\frac{2 n-1}{2 n} \\leqslant \\frac{1}{\\sqrt{3 n+1}} . \\label{eq2}\n$$\n当 $n=1$ 时,\\ref{eq2} 式左边 $=\\frac{1}{2}$, 右边 $=\\frac{1}{2}$, 故 式\\ref{eq2} 对 $n=1$ 成立.\n现设式\\ref{eq2}对 $n$ 成立, 则 $n+1$ 时, 有\n$$\n\\frac{1}{2} \\cdot \\frac{3}{4} \\cdot \\cdots \\cdot \\frac{2 n-1}{2 n} \\cdot \\frac{2 n+1}{2(n+1)} \\leqslant \\frac{1}{\\sqrt{3 n+1}} \\cdot \\frac{2 n+1}{2(n+1)} .\n$$\n为证式\\ref{eq2}对 $n+1$ 成立, 只需证明\n$$\n\\frac{1}{\\sqrt{3 n+1}} \\cdot \\frac{2 n+1}{2 n+2} \\leqslant \\frac{1}{\\sqrt{3 n+4}} .\n$$\n即证\n$$\n(3 n+4)(2 n+1)^2 \\leqslant(3 n+1)(2 n+2)^2 . \\label{eq3}\n$$\n注意到, 式\\ref{eq3}等价于\n$$\n\\begin{aligned}\n3(2 n+1)^2 & \\leqslant(3 n+1)\\left((2 n+2)^2-(2 n+1)^2\\right)=(3 n+1)(4 n+3) \\\\\n& \\Leftrightarrow 12 n^2+12 n+3 \\leqslant 12 n^2+13 n+3 \\\\\n& \\Leftrightarrow n \\geqslant 0 .\n\\end{aligned}\n$$\n所以,式\\ref{eq3}成立.\n从而式\\ref{eq2}对 $n+1$ 也成立, 即对任意 $n \\in \\mathbf{N}^*$, 都有 式\\ref{eq2} 成立.\n结合 $\\sqrt{3 n+1}>\\sqrt{3 n}$, 可知 式\\ref{eq1} 对任意 $n \\in \\mathbf{N}^*$ 成立.\n说明有些关于正整数 $n$ 的命题 $P(n)$ 直接用数学归纳法处理时难以实现 $n$ 到 $n+1$ 的过渡, 然而对比 $P(n)$ 更强的命题 $Q(n)$, 在用数学归纳法证明时反而简单, 因此需要对命题主动去加强.\n当然, 主动加强命题时通常需在把握问题本质的前提下恰当选择, 目的是便于实现归纳过渡.", + "remark": "", + "figures": [] +} \ No newline at end of file diff --git a/processed_dataset/proof/1560.json b/processed_dataset/proof/1560.json new file mode 100644 index 0000000000000000000000000000000000000000..5fb1cefa1b93b39e484cefa5047367b05183d070 --- /dev/null +++ b/processed_dataset/proof/1560.json @@ -0,0 +1,8 @@ +{ + "source_file": "./raw_volume-zh/volume6/chapter2-5.tex", + "problem_type": "proof", + "problem": "例2. 设 $A_1, A_2, \\cdots, A_r$ 是 $\\mathbf{N}^*$ 的任意一个 $r$-分划 (即 $A_1, \\cdots, A_r$ 中任两个的交集是空集, 且 $\\bigcup_{i=1}^r A_i=\\mathbf{N}^*$ ). 证明: 在 $A_1, \\cdots, A_r$ 中有一个集合 $A$ 具有下述性质: 存在 $m \\in \\mathbf{N}^*$, 使得对任意 $k \\in \\mathbf{N}^*$, 在 $A$ 中都可取出 $k$ 个数 $a_1, \\cdots$, $a_k$ 满足: 对 $1 \\leqslant j \\leqslant k-1$, 都有 $1 \\leqslant a_{j+1}-a_j \\leqslant m$.", + "solution": "证明:设 $P \\subseteq \\mathbf{N}^*$, 如果 $P$ 中含有任意长的相继正整数段, 那么称 $P$ 为长子集.\n我们将命题加强为: 对任意长子集 $P$ 的任何 $r-$ 分划 $A_1, A_2, \\cdots, A_r$. 集合 $A_1, \\cdots, A_r$ 中必有一个集合 $A$ 具有题设的性质.\n对 $r$ 运用数学归纳法.\n当 $r=1$ 时,由长子集的定义, 取 $m=1$ 可知命题成立;\n设命题对 $r=n$ 的情形成立, 考虑 $r=n+1$ 的情形.\n设 $P=\\left(A_1 \\cup A_2 \\cup \\cdots \\cup A_n\\right) \\cup A_{n+1}, Q=A_1 \\cup A_2 \\cup \\cdots \\cup A_n$. 如果 $Q$ 为长子集, 由归纳假设可知命题成立; 如果 $Q$ 不是长子集, 则必存在 $l \\in \\mathbf{N}^*$, 使 $Q$ 中没有长为 $l$ 的相继正整数段, 由于 $P$ 为长子集, 故对任意 $k \\in \\mathbf{N}^*, P$ 中存在长为 $k l$ 的相继正整数段, 该正整数段中至少有 $k$ 个数属于 $A_{n+1}$, 现在将这个长为 $k l$ 的相继正整数段中属于 $A_{n+1}$ 的最小 $k$ 个数取出, 则相邻两数之差不超过 $2 l$. 于是, 取 $m=2 l$, 则集合 $A_{n+1}$ 具有题给的性质.\n综上可知, 加强的命题获证.\n由于 $\\mathbf{N}^*$ 本身是一个长子集, 所以, 原命题成立.\n说明问题本质上要求证明: 对 $\\mathbf{N}^*$ 的每一个 $r$-分划而言, 都存在集合 $A$ 及 $m \\in \\mathbf{N}^*$, 使得将 $\\mathbf{N}^*$ 中的数分为长度为 $\\frac{m}{2}$ 的相继整数段后, 对任意 $k \\in \\mathbf{N}^*$, 都有相邻的 $k$ 个\"相继整数段\", 满足其中每个\"相继整数段\" 内都有一个数属于 $A$. 因此如果其他子集的并集中不含有任意长度的相继整数段, 那么 $A$ 中就能找到满足条件的 $k$ 个数, 依此想到引入 \"长子集\" 的概念, 进而得到问题的恰当加强.", + "remark": "", + "figures": [] +} \ No newline at end of file diff --git a/processed_dataset/proof/1561.json b/processed_dataset/proof/1561.json new file mode 100644 index 0000000000000000000000000000000000000000..19486ed5e8908bdade8d627a5586a963fbff204e --- /dev/null +++ b/processed_dataset/proof/1561.json @@ -0,0 +1,8 @@ +{ + "source_file": "./raw_volume-zh/volume6/chapter2-5.tex", + "problem_type": "proof", + "problem": "例3. 证明 : 存在无穷多个 $n \\in \\mathbf{N}^*$, 使得\n$$\nn \\mid\\left(2^n+2\\right) . \\label{(1)}\n$$", + "solution": "证明:$n=2$ 满足(1), 下一个满足(1)的正整数 $n=6$, 两者之间的关系是 $6=2^2+2$. 这提示我们用下面的方法来处理.\n设 $n(>1)$ 是具有性质 (1) 的正整数, 如果能证明: $\\left(2^n+2\\right) \\mid\\left(2^{2^n+2}+2\\right)$, 那么依此递推, 可知有无穷多个正整数 $n$ 满足 (1).\n注意到 $\\left(2^{n-1}+1\\right) \\mid\\left(2^{2^n+1}+1\\right)$ 在 $(n-1) \\mid\\left(2^n+1\\right)$ 条件下成立.\n我们通过增加一个要求的方法来处理.\n下证: (2) 存在无穷多个 $n \\in \\mathbf{N}^*(n>1)$, 使得 $n \\mid\\left(2^n+2\\right)$, 并且 $(n-1) \\mid \\left(2^n+1\\right)$.\n注意到, $n=2$ 具有上述性质.\n现设 $n(\\geqslant 2)$ 具有上面的性质, 令 $m=2^n+$ 2 , 我们证明 $m$ 也具有上述性质.\n事实上, 由于 $(n-1) \\mid\\left(2^n+1\\right)$, 而 $2^n+1$ 为奇数, 故可设 $2^n+1=(n-$ 1) $q, q$ 为奇数, 则\n$$\n\\begin{aligned}\n2^{m-1}+1 & =2^{2^n+1}+1=\\left(2^{n-1}\\right)^q+1 \\\\\n& =\\left(2^{n-1}+1\\right)\\left(\\left(2^{n-1}\\right)^{q-1}-\\left(2^{n-1}\\right)^{q-2}+\\cdots+1\\right),\n\\end{aligned}\n$$\n故 $\\left(2^{n-1}+1\\right) \\mid\\left(2^{m-1}+1\\right)$, 从而 $\\left(2^n+2\\right) \\mid\\left(2^m+2\\right)$, 即 $m \\mid\\left(2^m+2\\right)$.\n另一方面, 由 $(n-1) \\mid\\left(2^n+1\\right)$, 知 $n-1$ 为奇数, 故 $n$ 为偶数, 这样, 由 $n \\mid\\left(2^n+2\\right)$, 我们可设 $2^n+2=n p$, 这里 $p$ 为奇数 (这里用到 $\\left.4 \\nmid\\left(2^n+2\\right)\\right)$, 于是\n$$\n2^m+1=\\left(2^n\\right)^p+1=\\left(2^n+1\\right)\\left(\\left(2^n\\right)^{p-1}-\\left(2^n\\right)^{p-2}+\\cdots+1\\right),\n$$\n即有 $\\left(2^n+1\\right) \\mid\\left(2^m+1\\right)$, 也就是说 $(m-1) \\mid\\left(2^m+1\\right)$.\n综上可知, 命题成立.", + "remark": "", + "figures": [] +} \ No newline at end of file diff --git a/processed_dataset/proof/1562.json b/processed_dataset/proof/1562.json new file mode 100644 index 0000000000000000000000000000000000000000..a01c854d573213a1d6de27ee77b963df76ac49f6 --- /dev/null +++ b/processed_dataset/proof/1562.json @@ -0,0 +1,8 @@ +{ + "source_file": "./raw_volume-zh/volume6/chapter2-5.tex", + "problem_type": "proof", + "problem": "例4. 求所有的函数 $f: \\mathbf{Z} \\rightarrow \\mathbf{Z}$, 使得对任意 $x 、 y 、 z \\in \\mathbf{Z}$, 都有\n$$\nf\\left(x^3+y^3+z^3\\right)=f(x)^3+f(y)^3+f(z)^3 .\n$$", + "solution": "解:容易看到下面的 3 个函数\n$$\nf(x)=0, f(x)=x, f(x)=-x\n$$\n满足题中的条件.\n下证: 它们是所有满足条件的函数.\n取 $(x, y, z)=(0,0,0)$, 得 $f(0)=3 f(0)^3$, 这个关于 $f(0)$ 的三次方程只有一个整数解, 所以 $f(0)=0$. 再取 $(x, y, z)=(x,-x, 0)$ 可得 $f(x)= -f(-x)$, 故 $f(x)$ 为奇函数.\n而令 $(x, y, z)=(1,0,0)$, 得 $f(1)=f(1)^3$, 于是 $f(1) \\in\\{-1,0,1\\}$.\n下面用数学归纳法证明:\n对任意 $x \\in \\mathbf{Z}$, 都有 $f(x)=f(1) x$ (这样结合 $f(1)$ 的取值, 就完成了本题的解答). \\label{eq1}\n对 $|x|$ 予以归纳, 令 $(x, y, z)=(1,1,0)$, 得 $f(2)=2 f(1)^3=2 f(1)$, 令 $(x, y, z)=(1,1,1)$ 又有 $f(3)=3 f(1)$. 这样, 结合 $f(x)$ 为奇函数, 可知结论 式\\ref{eq1} 对 $|x| \\leqslant 3$ 都成立.\n现设对 $|x|3\\right)$, 都有 $f(x)=f(1) x$. 讨论 $f(k)$ 与 $f(-k)$ 的情形, 由 $f(x)$ 为奇函数, 只要证明 $f(k)=f(1) k$.\n为此, 我们需要用到下面的辅助命题.\n命题对任意 $k \\in \\mathbf{N}^*, k \\geqslant 4$, 数 $k^3$ 都可以表示为 5 个立方数之和, 并且 5 个加项中的每一项的绝对值都小于 $k^3$.\n事实上,由\n$$\n\\begin{aligned}\n& 4^3=3^3+3^3+2^3+1^3+1^3, 5^3=4^3+4^3+(-1)^3+(-1)^3+(-1)^3, \\\\\n& 6^3=5^3+4^3+3^3+0^3+0^3, 7^3=6^3+5^3+1^3+1^3+0^3 .\n\\end{aligned}\n$$\n及对不小于 9 的奇数 $2 m+1\\left(m \\in \\mathbf{N}^*, m \\geqslant 4\\right)$ 有\n$$\n(2 m+1)^3=(2 m-1)^3+(m+4)^3+(4-m)^3+(-5)^3+(-1)^3 . \\label{eq2}\n$$\n所以,命题对 $k=4$ 或 6 及 $k$ 为不小于 3 的奇数成立.\n注意到,对任意 $k>3, k \\in \\mathbf{N}^*$, 都存在分解式 $k=m y$, 这里 $m \\in \\mathbf{N}^*$, $y=4$ 或 6 或大于 3 的奇数.\n而由前所证,有表示 $y^3=y_1^3+\\cdots+y_5^3$, 其中 $\\left|y_i\\right|3, k \\in \\mathbf{N}^*$, 可写 $k^3=x_1^3+\\cdots+x_5^3,\\left|x_i\\right|2$, 记 $q(k)$ 为所有小于 $p(k)$ 的素数的乘积.\n若 $p(k)=2$, 则令 $q(k)=1$.\n定义数列 $\\left\\{x_n\\right\\}$ 如下 $x_0=1$, 而\n$$\nx_{n+1}=\\frac{x_n p\\left(x_n\\right)}{q\\left(x_n\\right)}, n=0,1,2, \\cdots .\n$$\n求所有的 $n \\in \\mathbf{N}^*$, 使 $x_n=111111$.", + "solution": "解:试算最初的一些 $x_n$ 的值,列表如下:\n\\begin{tabular}{|c|c|c|c|c|c|c|c|c|c|c|c|c|c|}\n\\hline$n$ & 0 & 1 & 2 & 3 & 4 & 5 & 6 & 7 & 8 & 9 & 10 & 11 & $\\cdots$ \\\\\n\\hline$x_n$ & 1 & 2 & 3 & $2 \\times 3$ & 5 & $2 \\times 5$ & $3 \\times 5$ & $2 \\times 3 \\times 5$ & 7 & $2 \\times 7$ & $3 \\times 7$ & $2 \\times 3 \\times 7$ & $\\cdots$ \\\\\n\\hline\n\\end{tabular}\n如果将 $n$ 写为二进制数, 那么由上面的数据, 可知 $n$ 在二进制表示中有几个 1 , 那么 $x_n$ 就是几个素数的乘积.\n进一步, 将素数从小到大排列, 设依次为 $p_0r_2>\\cdots>r_k \\geqslant 0 .\n$$\n即 $n$ 所对应的二进制数共 $\\left(r_1+1\\right)$ 位, 其中第 $r_k+1, r_{k-1}+1, \\cdots, r_1+1$ 位上的元素为 1 , 其余位上的元素全为 0 . 则 $x_n=p_{r_1} p_{r_2} \\cdots p_{r_k}$, 其中 $p_{r_i}$ 表示所有素数中第 $r_i+1$ 大的素数 (1).\n我们通过对 $n$ 归纳来证明上述结论.\n当 $n=1$ 时,由 $x_1=2=p_0$, 可知 (1) 成立.\n现设命题对 $n$ 成立, 即 $x_n=p_{r_1} p_{r_2} \\cdots p_{r_k}$, 考虑 $n+1$ 的情形.\n如果 $r_k \\geqslant 1$, 即 $n$ 对应的二进制数末位为 0 , 那么 $n+1=2^{r_1}+2^{r_2}+\\cdots+ 2^r+2^0$, 此时 $x_n$ 为奇数, 故 $p\\left(x_n\\right)=2$, 进而 $q\\left(x_n\\right)=1$, 由归纳假设, 可知\n$$\nx_{n+1}=\\frac{x_n p\\left(x_n\\right)}{q\\left(x_n\\right)}=\\frac{x_n \\cdot p_0}{1}=p_{r_1} \\cdots p_{r_k} p_0 .\n$$\n如果 $r_k=0$, 设 $i$ 是使得 $r_{i-1} \\geqslant r_i+2$ 的最大的正整数, 即 $n$ 对应的二进制数从右端第二位起往左数, 所有的二进制位中, 只有第 $\\left(r_i+1\\right)$ 位是第一个, 其左边的二进制数位至少含有一个 0 . 即此时 $r_{k-j}=j$, 其中 $0 \\leqslant j \\leqslant k-i$. 那么\n$$\nn+1=2^{r_1}+2^{r_2}+\\cdots+2^{r_{i-1}}+2^{r_i+1} \\text { (若 } i \\text { 不存在, 则 } n+1=2^{r_1+1} \\text { ). }\n$$\n这时由归纳假设知 $p\\left(x_n\\right)=p_{r_i+1}$, 从而 $q\\left(x_n\\right)=p_0 p_1 \\cdots p_{r_i}=p_0 p_1 \\cdots p_{k-i}= p_{r_k} p_{r_{k-1}} \\cdots p_{r_i}$. 所以\n$$\nx_{n+1}=\\frac{x_n \\cdot p\\left(x_n\\right)}{q\\left(x_n\\right)}=\\frac{p_{r_1} \\cdots p_{r_{i-1}} p_{r_i+1} p_{r_i} \\cdots p_{r_k}}{p_{r_i} \\cdots p_{r_k}}=p_{r_1} \\cdots p_{r_{i-1}} p_{r_i+1} .\n$$\n所以, (1)对 $n+1$ 成立, 即对任意 $n \\in \\mathbf{N}^*$, (1)都成立.\n现在由 $111111=3 \\times 7 \\times 11 \\times 13 \\times 37=p_1 p_3 p_4 p_5 p_{11}$, 可得满足 $x_n=$ 111111 的正整数 $n$ 对应的二进制表示为 $n=2^{11}+2^5+2^4+2^3+2=2106$. 所以,所求的 $n=2106$.", + "remark": "", + "figures": [] +} \ No newline at end of file diff --git a/processed_dataset/proof/1567.json b/processed_dataset/proof/1567.json new file mode 100644 index 0000000000000000000000000000000000000000..162fb3ca52d9d96585ff87a79b3934ca6c3c7af9 --- /dev/null +++ b/processed_dataset/proof/1567.json @@ -0,0 +1,8 @@ +{ + "source_file": "./raw_volume-zh/volume6/chapter2-6.tex", + "problem_type": "proof", + "problem": "例3. 整数数列 $\\left\\{a_n\\right\\}$ 定义如下 $a_1=2, a_2=7$,\n$$\n-\\frac{1}{2}2 a_{n-1}$. 所以\n$$\n\\left|3 a_{n+1}+2 a_n-\\frac{a_{n+1}^2}{a_n}\\right|<\\frac{1}{2} .\n$$\n利用 $a_{n+2}$ 为整数, 且 $\\left|a_{n+2}-\\frac{a_{n+1}^2}{a_n}\\right| \\leqslant \\frac{1}{2}$, 得\n$$\n\\begin{aligned}\n& \\left|a_{n+2}-\\left(3 a_{n+1}+2 a_n\\right)\\right| \\\\\n= & \\left|a_{n+2}-\\frac{a_{n+1}^2}{a_n}\\right|+\\left|\\frac{a_{n+1}^2}{a_n}-\\left(3 a_{n+1}+2 a_n\\right)\\right|<\\frac{1}{2}+\\frac{1}{2}=1 .\n\\end{aligned}\n$$\n所以 $a_{n+2}=3 a_{n+1}+2 a_n$. 于是猜想对 $k=n+1$ 的情形成立.\n综上可知,数列 $\\left\\{a_n\\right\\}$ 满足 $a_1=2, a_2=7, a_n=3 a_{n-1}+2 a_{n-2}, n=3$, $4, \\cdots$. 利用特征方程求解这个常系数齐次线性递推式,可得\n$$\na_n=\\frac{17+5 \\sqrt{17}}{68}\\left(\\frac{3+\\sqrt{17}}{2}\\right)^n+\\frac{17-5 \\sqrt{17}}{68}\\left(\\frac{3-\\sqrt{17}}{2}\\right)^n .\n$$", + "remark": "", + "figures": [] +} \ No newline at end of file diff --git a/processed_dataset/proof/1568.json b/processed_dataset/proof/1568.json new file mode 100644 index 0000000000000000000000000000000000000000..ebfda8287c9f7ff1aa4974bed1269dc4692523e7 --- /dev/null +++ b/processed_dataset/proof/1568.json @@ -0,0 +1,8 @@ +{ + "source_file": "./raw_volume-zh/volume6/chapter2-6.tex", + "problem_type": "proof", + "problem": "例4. 函数 $f: \\mathbf{N}^* \\rightarrow \\mathbf{N}^*$ 定义如下 $f(1)=1$, 对 $n \\in \\mathbf{N}^*$, 数 $f(n+1)$ 是满足下述条件的最大正整数 $m$ : 存在一个由正整数组成的等差数列 $a_1, a_2, \\cdots$, $a_m$ (这里项数小于 3 的数列也认为是等差数列), 使得 $a_14$, 那么由归纳假设及所列 $f(1)$ 至 $f(36)$ 的值可知 $d \\geqslant 8$, 此时 $m \\leqslant 1+\\frac{(4 n+1)-1}{8}q$ 两种情形讨论.\n情形一 $py_{k+1}$, 与 $pq$, 同情形一类似讨论.\n综上可知, 结论 3 成立.\n由结论 1 、结论 3 及 $x_0=0$, 可知命题成立.", + "remark": "", + "figures": [] +} \ No newline at end of file diff --git a/processed_dataset/proof/1570.json b/processed_dataset/proof/1570.json new file mode 100644 index 0000000000000000000000000000000000000000..3c0b712d7473e6c4bcff05af831cf745b7dcbdac --- /dev/null +++ b/processed_dataset/proof/1570.json @@ -0,0 +1,8 @@ +{ + "source_file": "./raw_volume-zh/volume6/chapter2-7.tex", + "problem_type": "proof", + "problem": "例1. 设 $a 、 b$ 是两个大于 2 的整数.\n证明: 存在正整数 $k$ 及由正整数组成的有穷数列 $n_1, n_2, \\cdots, n_k$, 使得 $n_1=a, n_k=b$, 而对 $1 \\leqslant i \\leqslant k-1$, 都有\n$$\n\\left(n_i+n_{i+1}\\right) \\mid n_i n_{i+1} .\n$$", + "solution": "证明:\"我们用 \" $a \\sim b$ \" 表示正整数 $a 、 b$ 可以用上述数列连接, 那么\"若 $a \\sim b$ 成立, 则 $b \\sim a$ 亦成立\".\n一个自然的想法是证明: 任意两个相邻正整数 (都大于 2) 之间是\"可达\" 的.\n利用下面的两个结论可达此目的.\n结论 1 对任意 $n \\in \\mathbf{N}^*, n \\geqslant 3$, 都有 $n \\sim 2 n$.\n下面的数列表明结论 1 成立.\n$$\nn, n(n-1), n(n-1)(n-2), n(n-2), 2 n \\text {. }\n$$\n结论 2 对任意 $n \\in \\mathbf{N}^*, n \\geqslant 4$, 都有 $n \\sim n-1$.\n利用数列\n$$\nn, n(n-1), n(n-1)(n-2), n(n-1)(n-2)(n-3), 2(n-1)(n-2) .\n$$\n结合结论 1 知 $2(n-1)(n-2) \\sim(n-1)(n-2)$, 而 $(n-1)(n-2)+(n-1)= (n-1)^2$ 是 $(n-1)(n-2) \\cdot(n-1)$ 的约数.\n故结论 2 成立.\n对大于 2 的整数 $a 、 b$, 不妨设 $a \\leqslant b$, 如果 $a=b$, 那么利用 $a \\sim a+1 \\sim b(=a)$ 可知命题成立; 如果 $a0$. 那么对 $n \\in \\mathbf{Z}$, 有\n$$\na_k=a^k \\cdot n+\\left(a^{k-1}+\\cdots+1\\right) b . \\label{eq1}\n$$\n此结论可通过对 $k$ 归纳得到.\n若 $b=0$, 则对任意大于 1 的正整数 $n$, 由 式\\ref{eq1}可知数列 $\\left\\{a_k\\right\\}$ 中每一项都是 $n$ 的倍数, 从而没有两项是互素的.\n若 $b \\neq 0$, 由于 $a$ 为正整数, 知存在 $k \\in \\mathbf{N}^*$, 使得 $\\left|\\left(a^{k-1}+\\cdots+1\\right) b\\right|>1$, 记 $c=\\left(a^{k-1}+\\cdots+1\\right) b$, 我们取 $n$ 为 $|c|$ 的素因子, 则对应于这个 $n$ 的 $a_k$ 是 $n$ 的倍数, 由式\\ref{eq1} 知 $a_{2 k}=a^{2 k} \\cdot n+\\left(a^{2 k-1}+\\cdots+1\\right) b=a^{2 k} \\cdot n+\\left(a^k+1\\right) \\cdot\\left(a^{k-1}+\\cdots+1\\right)$ $b=a^{2 k} \\cdot n+\\left(a^k+1\\right) c$, 故 $n$ 也是 $a_{2 k}$ 的约数, 导致 $a_k$ 与 $a_{2 k}$ 不互素.\n所以, 在 $m=1$ 时, 不存在符合要求的整系数多项式.\n下证: 当 $m \\geqslant 2$ 时,都存在这样的多项式.\n我们证明: 当 $f(x)=x^{m-1}(x-1)+1$ 时, 对任意 $n \\in \\mathbf{Z}$, 相应的数列 $\\left\\{a_k\\right\\}$ 中任意两项都互素.\n注意到, 对任意 $k \\in \\mathbf{N}^*$, 有\n$$\na_{k+1}=a_k^{m-1}\\left(a_k-1\\right)+1 \\equiv 1\\left(\\bmod a_k\\right),\n$$\n而且\n$$\na_{k+2}=a_{k+1}^{m-1}\\left(a_{k+1}-1\\right)+1 \\equiv 1^{m-1} \\cdot 0+1=1\\left(\\bmod a_k\\right)\n$$\n依此结合数学归纳法可知, 对任意正整数 $t>k$, 都有 $a_t \\equiv 1\\left(\\bmod a_k\\right)$. 所以, 数列 $\\left\\{a_k\\right\\}$ 中任意两项都互素.\n综上可知, 当 $m=1$ 时,不存在; 而 $m \\geqslant 2$ 时,都存在.\n说明对 $m \\geqslant 2$ 的情形, 任取一个 $m-2$ 次的整系数多项式 $g(x)$, 令 $f(x)=x(x-1) g(x)+1$, 仿上可证: 对 $n \\in \\mathbf{Z}$, 相应的数列 $\\left\\{a_k\\right\\}$ 中任意两项互素.", + "remark": "", + "figures": [] +} \ No newline at end of file diff --git a/processed_dataset/proof/1572.json b/processed_dataset/proof/1572.json new file mode 100644 index 0000000000000000000000000000000000000000..2b14c50ed7d030956d6cf83d7591c92beddf39c5 --- /dev/null +++ b/processed_dataset/proof/1572.json @@ -0,0 +1,8 @@ +{ + "source_file": "./raw_volume-zh/volume6/chapter2-7.tex", + "problem_type": "proof", + "problem": "例3. 设 $q$ 为一个给定的实数, 满足 $\\frac{1+\\sqrt{5}}{2}q+1$ (因为 $\\frac{1+\\sqrt{5}}{2}0$ (注意, 若 $a_1=0$, 则可知对任意 $n \\in \\mathbf{N}^*$, 都有 $\\left.a_n=0\\right)$, 此时, 对任意 $m \\in \\mathbf{N}^*$, 都有 $a_1+a_2+\\cdots+a_m>0$.\n由条件 $a_1+\\cdots+a_m \\equiv 0(\\bmod m)$, 可设 $a_1+\\cdots+a_m=d_m \\cdot m$, 结合反设中没有下标 $k$ 符合 (1), 可知对任意 $m \\in \\mathbf{N}^*$, 都有 $d_m \\geqslant m$, 故 $a_1+\\cdots+a_m \\geqslant m^2$. 利用 $m \\geqslant 2$ 时, 有 $a_m \\leqslant m-1$, 得\n$$\nm^2 \\leqslant a_1+\\cdots+a_m \\leqslant a_1+1+2+\\cdots+(m-1)=a_1+\\frac{m(m-1)}{2} .\n$$\n导致 $a_1 \\geqslant \\frac{m(m+1)}{2}$, 此式不能对所有 $m \\in \\mathbf{N}^*$ 都成立, 所得矛盾表明 (1) 成立.\n综上可知, 命题成立.\n说明利用反证法(或抽屉原则等)是间接得到存在性的基本方法, 在处理不存在问题时就更常用了.", + "remark": "", + "figures": [] +} \ No newline at end of file diff --git a/processed_dataset/proof/1574.json b/processed_dataset/proof/1574.json new file mode 100644 index 0000000000000000000000000000000000000000..2a3e94508d7b0c17213afc95eff4b7415a6cd414 --- /dev/null +++ b/processed_dataset/proof/1574.json @@ -0,0 +1,8 @@ +{ + "source_file": "./raw_volume-zh/volume6/chapter2-7.tex", + "problem_type": "proof", + "problem": "例6. 数列 $\\left\\{a_n\\right\\}$ 定义如下: 若正整数 $n$ 在二进制表示下,数码 1 出现偶数次, 则 $a_n=0$; 否则 $a_n=1$. 证明: 不存在正整数 $k 、 m$, 使得对任意 $j \\in\\{0,1$, $2, \\cdots, m-1\\}$, 都有\n$$\na_{k+j}=a_{k+m+j}=a_{k+2 m+j} . \\label{eq1}\n$$", + "solution": "证明:利用 $\\left\\{a_n\\right\\}$ 的定义可知\n$$\n\\left\\{\\begin{array}{l}\na_{2 n} \\equiv a_n(\\bmod 2), \\\\\na_{2 n+1} \\equiv a_{2 n}+1 \\equiv a_n+1(\\bmod 2) .\n\\end{array}\\right. \\label{eq2}\n$$\n如果存在 $k 、 m \\in \\mathbf{N}^*$, 使得对 $j \\in\\{0,1, \\cdots, m-1\\}$ 都有式\\ref{eq1}成立, 那么由最小数原理,我们可设 $(k, m)$ 是这样的正整数对中使 $k+m$ 最小的数对.\n情形一 $m$ 为偶数, 设 $m=2 t, t \\in \\mathbf{N}^*$.\n若 $k$ 为偶数, 在式\\ref{eq1}中取 $j=0,2, \\cdots, 2(t-1)$, 则 $0 \\leqslant \\frac{j}{2} \\leqslant t-1$, 且\n$$\na_{k+j}=a_{k+m+j}=a_{k+2 m+j},\n$$\n由式\\ref{eq2}得 $a \\frac{k}{2}+\\frac{j}{2}=a \\frac{k}{2}+t+\\frac{j}{2}=a \\frac{k}{2}+2 t+\\frac{j}{2}$, 这表明 $\\left(\\frac{k}{2}, \\frac{m}{2}\\right)$ 也是使 式\\ref{eq1} 对 $0 \\leqslant j \\leqslant \\frac{m}{2}-$ 1 都成立的正整数对, 与 $k+m$ 的最小性矛盾.\n若 $k$ 为奇数,在式\\ref{eq1}中取 $j=1,3, \\cdots, 2 t-1$, 同上讨论可知\n$$\na \\frac{k+1}{2}+\\frac{j-1}{2}=a \\frac{k+1}{2}+\\iota+\\frac{j-1}{2}=a \\frac{k+1}{2}+2 t+\\frac{j-1}{2},\n$$\n表明 $\\left(\\frac{k+1}{2}, \\frac{m}{2}\\right)$ 也使(1)对 $0 \\leqslant j \\leqslant \\frac{m}{2}-1$ 都成立, 与 $k+m$ 的最小性矛盾.\n情形二 $m$ 为奇数.\n当 $m=1$ 时, 要求 $a_k=a_{k+1}=a_{k+2}$, 这时如果 $k$ 为偶数, 那么 $a_{2 n}= a_{2 n+1} \\equiv a_{2 n}+1(\\bmod 2)$, 矛盾; 如果 $k$ 为奇数, 设 $k=2 n+1$, 那么 $a_{2 n+2}= a_{2 n+3} \\equiv a_{2 n+2}+1(\\bmod 2)$,亦矛盾.\n当 $m \\geqslant 3$ 时, 在 式\\ref{eq1} 中令 $j=0,1,2$, 可得\n$$\n\\left\\{\\begin{array}{l}\na_k=a_{k+m}=a_{k+2 m}, \\label{eq3}\\\\\na_{k+1}=a_{k+m+1}=a_{k+2 m+1}, \\label{eq4}\\\\\na_{k+2}=a_{k+m+2}=a_{k+2 m+2} . \\label{eq5}\n\\end{array}\\right.\n$$\n如果 $k$ 为偶数, 设 $k=2 n, m=2 t+1$, 那么由 式\\ref{eq2} 知 $a_{k+1} \\neq a_k, a_{k+m+1} \\neqa_{k+m+2}$, 这样结合 式\\ref{eq3}、\\ref{eq4}、式\\ref{eq5} 可知\n$$\na_k=a_{k+m+2}=a_{k+2} . \\label{eq6}\n$$\n(注意,这里用到数列之中的每一项都为 0 或 1.)\n现在, 若 $n$ 为偶数, 设 $n=2 t$, 则 $a_{k+2}=a_{4 t+2}=a_{2 t+1} \\equiv a_{2 t}+1 \\equiv a_{4 t}+1 \\equiv a_k+1(\\bmod 2)$, 与 式\\ref{eq6} 矛盾; 若 $n$ 为奇数, 则由 $m$ 为奇数可知 $k+2 m \\equiv 0(\\bmod$ 4 ), 类似讨论可得 $a_{k+2 m} \\neq a_{k+2 m+2}$, 结合 式\\ref{eq3}、\\ref{eq5}、式\\ref{eq6} 亦得矛盾.\n如果 $k$ 为奇数,结合 $m$ 为奇数, 由式\\ref{eq2}可知 $a_{k+m} \\neq a_{k+m+1}, a_{k+1} \\neq a_{k+2}$, 利用 式\\ref{eq3}、\\ref{eq4}、式\\ref{eq5} 得\n$$\na_k=a_{k+m}=a_{k+2} .\n$$\n现在, 若 $k \\equiv 1(\\bmod 4)$, 则由 式\\ref{eq7} 的 $a_k=a_{k+2}$ 可推出矛盾; 若 $k \\equiv 3(\\bmod 4)$, 则由 $m$ 为奇数可知 $k+2 m \\equiv 1(\\bmod 4)$, 故 $a_{k+2 m} \\neq a_{k+2 m+2}$, 即 $a_k \\neq a_{k+2}$ 与 式\\ref{eq7}矛盾.\n综上可知, 命题成立.", + "remark": "", + "figures": [] +} \ No newline at end of file diff --git a/processed_dataset/proof/1575.json b/processed_dataset/proof/1575.json new file mode 100644 index 0000000000000000000000000000000000000000..ae1e146b6d9c39dcda8a6b6510f90c8e4c5f016f --- /dev/null +++ b/processed_dataset/proof/1575.json @@ -0,0 +1,8 @@ +{ + "source_file": "./raw_volume-zh/volume6/exercise1.tex", + "problem_type": "proof", + "problem": "问题1. 证明: 对任意非空有限集, 都可以将它的所有子集排成一列, 使得任意两个相邻的子集的元素个数相差 1 .", + "solution": "对该非空有限集的元素个数 $n$ 归纳.\n记该集合为 $S_n$, 当 $n=1$ 时, 其子集可排列为 $\\varnothing, S_1$, 符合要求.\n设命题对 $n$ 成立, 即 $S_n$ 的子集可排列为 $A_1$, $A_2, \\cdots, A_{2^n}$, 使相邻两个集合元素个数相差 1 . 考虑 $S_{n+1}=\\left\\{a_1, \\cdots, a_{n+1}\\right\\}$, 对其 $n$ 元子集 $S_n=\\left\\{a_1, \\cdots, a_n\\right\\}$, 依归纳假设对 $S$ 的子集有符合要求的排列 $A_1, \\cdots, A_{2^n}$;于是, 下面的排列:\n$$\nA_1, \\cdots, A_{2^n}, A_{2^n} \\cup\\left\\{a_{n+1}\\right\\}, \\cdots, A_1 \\cup\\left\\{a_{n+1}\\right\\} .\n$$\n是 $S_{n+1}$ 所有子集的排列, 它们符合要求.\n说明这里构造的集合列中相邻两个子集的不同元素都恰好只有一个, 比要求的结论更强.", + "remark": "", + "figures": [] +} \ No newline at end of file diff --git a/processed_dataset/proof/1576.json b/processed_dataset/proof/1576.json new file mode 100644 index 0000000000000000000000000000000000000000..64709f3d4032deb13fbb86577fc2affb16dcb520 --- /dev/null +++ b/processed_dataset/proof/1576.json @@ -0,0 +1,8 @@ +{ + "source_file": "./raw_volume-zh/volume6/exercise1.tex", + "problem_type": "proof", + "problem": "问题2. 数列 $\\left\\{a_n\\right\\}$ 满足 $a_0=0, a_n+a_{n-2} \\geqslant 2 a_{n-1}, n=2,3, \\cdots$.\n证明: 对任意 $n \\in \\mathbf{N}^*$ 及 $k \\in \\mathbf{Z}$, 只要 $0 \\leqslant k \\leqslant n$, 就有 $n a_k \\leqslant k a_n$.", + "solution": "当 $k=0$ 时, 命题显然成立.\n对 $k>0$ 的情形, 要证的结论等价于 $\\frac{a_k}{k} \\leqslant \\frac{a_n}{n}$, 它是下述命题的推论: 对 $k \\geqslant 0$, 都有(1)\n$$\n(k+1) a_k \\leqslant k a_{k+1} .\n$$\n对 $k$ 归纳来证明(1)成立: 在 $k=0$ 时, 由 $a_0=0$, 知(1)成立; 现设(1)对 $k$ 成立, 则由条件知\n$$\n\\begin{aligned}\n(k+2) a_{k+1} & =2(k+1) a_{k+1}-k a_{k+1} \\leqslant 2(k+1) a_{k+1}-(k+1) a_k \\\\\n& =(k+1)\\left(2 a_{k+1}-a_k\\right) \\leqslant(k+1) a_{k+2} .\n\\end{aligned}\n$$\n所以, (1) 对 $k+1$ 也成立.\n命题获证.", + "remark": "", + "figures": [] +} \ No newline at end of file diff --git a/processed_dataset/proof/1577.json b/processed_dataset/proof/1577.json new file mode 100644 index 0000000000000000000000000000000000000000..580cea033959e241c92352177820666b0c20f61e --- /dev/null +++ b/processed_dataset/proof/1577.json @@ -0,0 +1,8 @@ +{ + "source_file": "./raw_volume-zh/volume6/exercise1.tex", + "problem_type": "proof", + "problem": "问题3. 一个由正实数组成的数列 $\\left\\{a_n\\right\\}$ 满足 $a_n^2 \\leqslant a_n-a_{n+1}, n=1,2, \\cdots$. 证明: 对任意 $n \\in \\mathbf{N}^*$, 都有 $a_n<\\frac{1}{n}$.", + "solution": "当 $n=1$ 时, $a_1^2 \\leqslant a_1-a_2\\frac{1}{2}-\\frac{1}{n} \\geqslant 0$, 因此 $a_{n+1}<\\frac{1}{4}-\\left(\\frac{1}{2}-\\frac{1}{n}\\right)^2=\\frac{1}{n}-\\frac{1}{n^2}=\\frac{n-1}{n^2}< \\frac{1}{n+1}$. 即命题对 $n+1$ 成立, 获证.", + "remark": "", + "figures": [] +} \ No newline at end of file diff --git a/processed_dataset/proof/1578.json b/processed_dataset/proof/1578.json new file mode 100644 index 0000000000000000000000000000000000000000..1e65617521f533841da320d7739d69dc070ac271 --- /dev/null +++ b/processed_dataset/proof/1578.json @@ -0,0 +1,8 @@ +{ + "source_file": "./raw_volume-zh/volume6/exercise1.tex", + "problem_type": "proof", + "problem": "问题4. 设实数 $a_1, \\cdots, a_n(n \\geqslant 2)$ 满足 $a_1\\sqrt{2 n}$.", + "solution": "由条件, 知 $a_{n+1}=a_n+\\frac{1}{a_{n-1}}$, 而结合初始条件及数学归纳法可知, 对任意 $n \\in \\mathbf{N}^*$, 有 $a_n>0$, 从而 $n \\geqslant 2$ 时,有 $a_{n+1}=a_n+\\frac{1}{a_{n-1}}>a_n$, 结合 $a_1a_n+\\frac{1}{a_n} .\n$$\n由 $a_3=\\frac{a_2 a_1+1}{a_1}=3$, 知 $a_3>\\sqrt{6}$, 即 $n=3$ 时, 有 $a_n>\\sqrt{2 n}$. 现设当 $n= m(\\geqslant 3)$ 时, 有 $a_m>\\sqrt{2 m}$, 则由 (1) 知 $a_{m+1}^2>\\left(a_m+\\frac{1}{a_m}\\right)^2=a_m^2+2+\\frac{1}{a_m^2}> a_m^2+2>2 m+2$, 故 $a_{m+1}>\\sqrt{2(m+1)}$, 即命题对 $m+1$ 成立.\n获证.", + "remark": "", + "figures": [] +} \ No newline at end of file diff --git a/processed_dataset/proof/1580.json b/processed_dataset/proof/1580.json new file mode 100644 index 0000000000000000000000000000000000000000..a14b9a72c00c18abf453a0be8a4b430fffb54017 --- /dev/null +++ b/processed_dataset/proof/1580.json @@ -0,0 +1,8 @@ +{ + "source_file": "./raw_volume-zh/volume6/exercise1.tex", + "problem_type": "proof", + "problem": "问题6. 设 $a$ 为正实数.\n证明: 对任意 $n \\in \\mathbf{N}^*$, 都有\n$$\n\\frac{1+a^2+a^4+\\cdots+a^{2 n}}{a+a^3+a^5+\\cdots+a^{2 n-1}} \\geqslant \\frac{n+1}{n} .\n$$", + "solution": "当 $n=1$ 时,由 $\\frac{1+a^2}{a}=\\frac{1}{a}+a \\geqslant 2 \\sqrt{\\frac{1}{a} \\cdot a}=2$ 知命题成立.\n现设 $n$ 时命题成立, 即 $\\frac{1+a^2+\\cdots+a^{2 n}}{a+a^3+\\cdots+a^{2 n-1}} \\geqslant \\frac{n+1}{n}$, 则 $\\frac{a+a^3+\\cdots+a^{2 n-1}}{1+a^2+\\cdots+a^{2 n}} \\leqslant \\frac{n}{n+1}$.\n注意到\n$$\n\\begin{aligned}\n& \\frac{1+a^2+\\cdots+a^{2 n+2}}{a+a^3+\\cdots+a^{2 n+1}}+\\frac{a+a^3+\\cdots+a^{2 n-1}}{1+a^2+\\cdots+a^{2 n}} \\\\\n= & \\frac{1+a^2+\\cdots+a^{2 n+2}}{a\\left(1+a^2+\\cdots+a^{2 n}\\right)}+\\frac{a+a^3+\\cdots+a^{2 n-1}}{1+a^2+\\cdots+a^{2 n}} \\\\\n= & \\frac{1+a^2+\\cdots+a^{2 n+2}+a\\left(a+a^3+\\cdots+a^{2 n-1}\\right)}{a\\left(1+a^2+\\cdots+a^{2 n}\\right)} \\\\\n= & \\frac{\\left(1+a^2+\\cdots+a^{2 n}\\right)+a^2\\left(1+a^2+\\cdots+a^{2 n}\\right)}{a\\left(1+a^2+\\cdots+a^{2 n}\\right)} \\\\\n= & \\frac{a^2+1}{a}=a+\\frac{1}{a} \\geqslant 2 .\n\\end{aligned}\n$$\n所以\n$$\n\\frac{1+a^2+\\cdots+a^{2 n+2}}{a+a^3+\\cdots+a^{2 n+1}} \\geqslant 2-\\frac{n}{n+1}=\\frac{n+2}{n+1}\n$$\n即命题对 $n+1$ 成立, 获证.", + "remark": "", + "figures": [] +} \ No newline at end of file diff --git a/processed_dataset/proof/1581.json b/processed_dataset/proof/1581.json new file mode 100644 index 0000000000000000000000000000000000000000..c9b528c8762e841f14756d6d8f690c60884370ce --- /dev/null +++ b/processed_dataset/proof/1581.json @@ -0,0 +1,8 @@ +{ + "source_file": "./raw_volume-zh/volume6/exercise1.tex", + "problem_type": "proof", + "problem": "问题7. 证明: 对任意 $n \\in \\mathbf{N}^*, n \\geqslant 2$, 都有\n$$\n\\lg (n !)>\\frac{3 n}{10}\\left(\\frac{1}{2}+\\frac{1}{3}+\\cdots+\\frac{1}{n}\\right) .\n$$", + "solution": "当 $n=2$ 时, 由于 $2^{10}>1000$, 故 $\\lg 2>\\frac{3}{10}$, 命题成立.\n现设命题对 $n (n \\geqslant 2)$ 成立, 由均值不等式知 $\\frac{1+2+\\cdots+n}{n}>\\sqrt[n]{1 \\times 2 \\times \\cdots \\times n}$, 即 $n+1> 2(n !)^{\\frac{1}{n}}$, 于是\n$$\n\\begin{aligned}\n\\lg ((n+1) !) & >\\lg \\left((n !) \\cdot 2(n !)^{\\frac{1}{n}}\\right) \\\\\n& =\\lg 2+\\frac{n+1}{n} \\lg (n !) \\\\\n& >\\lg 2+\\frac{n+1}{n} \\times \\frac{3 n}{10}\\left(\\frac{1}{2}+\\cdots+\\frac{1}{n}\\right) \\\\\n& >\\frac{3}{10}+\\frac{3(n+1)}{10}\\left(\\frac{1}{2}+\\cdots+\\frac{1}{n}\\right)\n\\end{aligned}\n$$\n$$\n=\\frac{3(n+1)}{10}\\left(\\frac{1}{2}+\\cdots+\\frac{1}{n+1}\\right) .\n$$\n所以,命题对 $n+1$ 成立, 获证.", + "remark": "", + "figures": [] +} \ No newline at end of file diff --git a/processed_dataset/proof/1582.json b/processed_dataset/proof/1582.json new file mode 100644 index 0000000000000000000000000000000000000000..0a038d1dcc5bccb3932393e5a4801b1d38e7c23c --- /dev/null +++ b/processed_dataset/proof/1582.json @@ -0,0 +1,8 @@ +{ + "source_file": "./raw_volume-zh/volume6/exercise1.tex", + "problem_type": "proof", + "problem": "问题8. 正实数数列 $\\left\\{a_n\\right\\}$ 满足: 对任意正整数 $n$, 都有 $\\sum_{j=1}^n a_j^3=\\left(\\sum_{j=1}^n a_j\\right)^2$.\n证明: 对任意 $n \\in \\mathbf{N}^*$, 都有 $a_n=n$.", + "solution": "当 $n=1$ 时, $a_1^3=a_1^2$, 而 $a_1>0$, 故 $a_1=1$, 即命题对 $n=1$ 成立.\n现设命题对 $1,2, \\cdots, n-1$ 都成立, 即 $a_k=k, k=1,2, \\cdots, n-1$. 则\n$$\n\\left(\\sum_{k=1}^{n-1} k^3\\right)+a_n^3=\\sum_{k=1}^n a_k^3=\\left(\\sum_{k=1}^n a_k\\right)^2=\\left(\\left(\\sum_{k=1}^{n-1} k\\right)+a_n\\right)^2,\n$$\n于是 $a_n^3=a_n^2+n(n-1) a_n$, 解得 $a_n=0,-(n-1)$ 或 $n$, 结合 $a_n>0$, 知 $a_n= n$. 所以, 命题对 $n$ 成立, 获证.", + "remark": "", + "figures": [] +} \ No newline at end of file diff --git a/processed_dataset/proof/1583.json b/processed_dataset/proof/1583.json new file mode 100644 index 0000000000000000000000000000000000000000..37dd85d54f6416557968ac427911cfc9d5e13ac8 --- /dev/null +++ b/processed_dataset/proof/1583.json @@ -0,0 +1,8 @@ +{ + "source_file": "./raw_volume-zh/volume6/exercise1.tex", + "problem_type": "proof", + "problem": "问题9. 设 $a_1, \\cdots, a_n$ 是 $n$ 个不同的正整数.\n证明: $a_1^2+\\cdots+a_n^2 \\geqslant \\frac{2 n+1}{3}\\left(a_1+\\cdots+a_n\\right)$.", + "solution": "不妨设 $a_1m$ 时,注意到, 对任意 $k \\in \\mathbf{N}^*, k>m$,均有\n$$\n10$. 进一步, 由 $2 n x_n= 2(n-1) x_{n-1}-x_{n-1}$, 得 $x_{n-1}=2(n-1) x_{n-1}-2 n x_n$. 求和, 得\n$$\n\\begin{aligned}\nx_1+\\cdots+x_n & =\\sum_{k=2}^{n+1}\\left(2(k-1) x_{k-1}-2 k x_k\\right) \\\\\n& =2 \\sum_{k=1}^n\\left(k x_k-(k+1) x_{k+1}\\right)=2\\left(x_1-(n+1) x_{n+1}\\right) \\\\\n& =1-2(n+1) x_{n+1}<1 .\n\\end{aligned}\n$$\n命题获证.", + "remark": "", + "figures": [] +} \ No newline at end of file diff --git a/processed_dataset/proof/1591.json b/processed_dataset/proof/1591.json new file mode 100644 index 0000000000000000000000000000000000000000..ff2f44222d333c54c6a55ec7a45a88a145f9462b --- /dev/null +++ b/processed_dataset/proof/1591.json @@ -0,0 +1,8 @@ +{ + "source_file": "./raw_volume-zh/volume6/exercise1.tex", + "problem_type": "proof", + "problem": "问题17. 数列 $\\{f(n)\\}$ 满足\n$$\nf(1)=2, f(n+1)=(f(n))^2-f(n)+1, n=1,2,3, \\cdots .\n$$\n证明: 对任意整数 $n>1$, 都有 $1-\\frac{1}{2^{2^{n-1}}}<\\frac{1}{f(1)}+\\frac{1}{f(2)}+\\cdots+\\frac{1}{f(n)}<1-\\frac{1}{2^{2^{n^2}}}$.", + "solution": "由条件可知 $f(n+1)=(f(n)-1) f(n)+1$, 结合数学归纳法及 $a_1>$ 1 , 可得对任意 $n \\in \\mathbf{N}^*$, 都有 $f(n)>1$,于是, 取倒数就有\n$$\n\\frac{1}{f(n+1)-1}=\\frac{1}{f(n)(f(n)-1)}=\\frac{1}{f(n)-1}-\\frac{1}{f(n)} .\n$$\n即 $\\frac{1}{f(n)}=\\frac{1}{f(n)-1}-\\frac{1}{f(n+1)-1}$, 裂项求和得\n$$\n\\sum_{k=1}^n \\frac{1}{f(k)}=\\frac{1}{f(1)-1}-\\frac{1}{f(n+1)-1}=1-\\frac{1}{f(n+1)-1} .\n$$\n回到递推关系式, 知 $f(n+1)-1=f(n)(f(n)-1)>(f(n)-1)^2> (f(n-1)-1)^{2^2}>\\cdots>(f(2)-1)^{2^{n-1}}=\\left(2^2-2\\right)^{2^{n-1}}=2^{2^{n-1}}$.\n$$\n\\text { 故 } \\sum_{k=1}^n \\frac{1}{f(k)}>1-\\frac{1}{2^{2^{n-1}}} \\text {. }\n$$\n另一方面, $f(n+1)=f(n)^2-(f(n)-1)1$ 时, 都有 $21$ 时, 有\n$$\nx_n y_n=\\cot \\frac{\\alpha}{2^{n-1}} \\tan \\frac{\\beta}{2^{n-1}}=\\cot \\frac{\\pi}{2^n \\times 3} \\tan \\frac{\\pi}{2^{n-1} \\times 3}\n$$\n$$\n=\\frac{2}{1-\\tan ^2 \\frac{\\pi}{2^n \\times 3}} .\n$$\n由于 $\\tan ^2 \\frac{\\pi}{2^n \\times 3} \\in\\left(0, \\tan ^2 \\frac{\\pi}{6}\\right)$, 即 $\\tan ^2 \\frac{\\pi}{2^n \\times 3} \\in\\left(0, \\frac{1}{3}\\right)$, 故 $2f\\left(\\frac{n}{\\sqrt{n-1}}\\right)=\\frac{n}{\\sqrt{n-1}}>\\sqrt{n+1}$. 故对一切 $n \\in \\mathbf{N}^*, n \\geqslant 3$,均有 $\\sqrt{n}f\\left(\\frac{n}{\\sqrt{n-1}}\\right)=\\frac{n}{\\sqrt{n-1}}$, 故当 $n \\geqslant 4$ 时,有 $a_n>\\frac{n-1}{\\sqrt{n-2}}$. 进而, 当 $n \\geqslant 4$ 时, 有\n$$\na_{n+1}=f\\left(a_n\\right)0\\right)$, 而 $a_4=\\frac{13}{6}<\\sqrt{6}$ 是显然的.\n于是, 当 $n \\geqslant 4$ 时,均有 $\\sqrt{n}0$, 则用 $-x_i$ 代替 $x_i$ 后讨论), 排序后, 设 $y_1 \\geqslant \\cdots \\geqslant y_k \\geqslant 0 \\geqslant y_{k+1} \\geqslant \\cdots \\geqslant y_{2011}$. 那么\n$$\n\\begin{aligned}\nS & =\\sum_{i=1}^{2011}\\left|x_i\\right|-\\left|\\sum_{i=1}^{2011} x_i\\right| \\\\\n& =\\left(y_1+\\cdots+y_k\\right)-\\left(y_{k+1}+\\cdots+y_{2011}\\right)+\\left(y_1+\\cdots+y_{2011}\\right) \\\\\n& =2\\left(y_1+\\cdots+y_k\\right) .\n\\end{aligned}\n$$\n为证明(1)成立,我们只需证明:(2)\n$$\ny_1+\\cdots+y_k \\leqslant 1+2+\\cdots+1005 \\text {. }\n$$\n分两种情形来处理.\n情形一: 若 $k \\geqslant 1006$, 则由 $y_1+\\cdots+y_{2011} \\leqslant 0$, 知\n$$\ny_1+\\cdots+y_k \\leqslant-\\left(y_{k+1}+\\cdots+y_{2011}\\right),\n$$\n结合 $y_{k+1} \\geqslant y_k-1, \\cdots, y_{2011} \\geqslant y_k-(2011-k)$ 可知\n$$\n\\begin{aligned}\n& y_1+\\cdots+y_k \\leqslant-\\left(\\left(y_k-1\\right)+\\cdots+\\left(y_k-(2011-k)\\right)\\right) \\\\\n= & -(2011-k) y_k+1+2+\\cdots+(2011-k) \\\\\n\\leqslant & 1+2+\\cdots+(2011-k) \\leqslant 1+2+\\cdots+1005 .\n\\end{aligned}\n$$\n此时, (2)成立.\n情形二:若 $k \\leqslant 1005$, 则同上可知\n$$\n\\begin{aligned}\ny_1+\\cdots+y_k & \\leqslant\\left(y_{k+1}+k\\right)+\\left(y_{k+1}+(k-1)\\right)+\\cdots+\\left(y_{k+1}+1\\right) \\\\\n& =k y_{k+1}+1+\\cdots+k \\leqslant 1+2+\\cdots+k \\leqslant 1+2+\\cdots+1005 .\n\\end{aligned}\n$$\n(2)亦成立.\n综上可知,所求的最大值为 1011030 .", + "remark": "", + "figures": [] +} \ No newline at end of file diff --git a/processed_dataset/proof/1597.json b/processed_dataset/proof/1597.json new file mode 100644 index 0000000000000000000000000000000000000000..95bdfe056362ca55fff0ca5b556586e4c0bf13b4 --- /dev/null +++ b/processed_dataset/proof/1597.json @@ -0,0 +1,8 @@ +{ + "source_file": "./raw_volume-zh/volume6/exercise1.tex", + "problem_type": "proof", + "problem": "问题24. 设 $a_0, a_1, a_2, \\cdots$ 是一个由正实数组成的无穷数列.\n证明: 存在无穷多个正整数 $n$, 使得 $1+a_n>\\sqrt[n]{2} a_{n-1}$.", + "solution": "采用反证法处理, 如果命题不成立, 那么, 存在正整数 $N$, 使得对任意 $n \\geqslant N$, 都有(1)\n$$\n1+a_n \\leqslant \\sqrt[n]{2} \\cdot a_{n-1} .\n$$\n现在定义一个正实数数列 $\\left\\{c_n\\right\\}: c_0=1, c_n=\\frac{a_{n-1}}{1+a_n} c_{n-1}, n=1,2, \\cdots$.\n则由 (1) 可知, (2):对 $n \\geqslant N$, 都有 $c_n \\geqslant 2^{-\\frac{1}{n}} \\cdot c_{n-1}$.\n注意到, 对 $n \\in \\mathbf{N}^*$, 有 $c_n\\left(1+a_n\\right)=a_{n-1} c_{n-1}$, 即 $c_n=a_{n-1} c_{n-1}-a_n c_n$, 裂项求和, 得\n$$\nc_1+\\cdots+c_n=a_0-a_n c_nN$ 时,有\n$$\n\\begin{aligned}\nc_n & \\geqslant c_{n-1} \\cdot 2^{-\\frac{1}{n}} \\geqslant c_{n-2} \\cdot 2^{-\\left(\\frac{1}{n-1}+\\frac{1}{n}\\right)} \\geqslant \\cdots \\geqslant c_N \\cdot 2^{-\\left(\\frac{1}{N+1}+\\cdots+\\frac{1}{n}\\right)} \\\\\n& =C \\cdot 2^{-\\left(1+\\frac{1}{2}+\\cdots+\\frac{1}{n}\\right)} .\n\\end{aligned}\n$$\n这里 $C=c_N \\cdot 2^{-\\left(1+\\frac{1}{2}+\\cdots+\\frac{1}{N}\\right)}$ 为常数.\n对任意 $k \\in \\mathbf{N}^*$, 若 $2^{k-1} \\leqslant n<2^k$, 则\n$$\n\\begin{aligned}\n1+\\frac{1}{2}+\\cdots+\\frac{1}{n} \\leqslant & 1+\\left(\\frac{1}{2}+\\frac{1}{3}\\right)+\\left(\\frac{1}{4}+\\cdots+\\frac{1}{7}\\right)+\\cdots \\\\\n& +\\left(\\frac{1}{2^{k-1}}+\\cdots+\\frac{1}{2^k-1}\\right) \\\\\n\\leqslant & 1+\\left(\\frac{1}{2}+\\frac{1}{2}\\right)+\\left(\\frac{1}{4}+\\cdots+\\frac{1}{4}\\right)+\\cdots \\\\\n& +\\left(\\frac{1}{2^{k-1}}+\\cdots+\\frac{1}{2^{k-1}}\\right)=k .\n\\end{aligned}\n$$\n所以, 此时有 $c_n \\geqslant C \\cdot 2^{-k}\\left(2^{k-1} \\leqslant n<2^k\\right)$.\n现在设 $2^{r-1} \\leqslant N<2^r, r \\in \\mathbf{N}^*$, 那么对任意 $m>r$, 有\n$$\n\\begin{aligned}\nc_{2^r}+c_{2^r+1}+\\cdots+c_{2^m-1} & =\\left(c_{2^r}+\\cdots+c_{2^{r+1}-1}\\right)+\\cdots+\\left(c_{2^{m-1}}+\\cdots+c_{2^m-1}\\right) \\\\\n& \\geqslant\\left(C \\cdot 2^{-(r+1)}\\right) \\cdot 2^r+\\cdots+\\left(C \\cdot 2^{-(m+1)}\\right) \\cdot 2^m \\\\\n& =\\frac{C(m-r)}{2} .\n\\end{aligned}\n$$\n这表明 $s_{2^m-1}>\\frac{C(m-r)}{2}$, 当 $m \\rightarrow+\\infty$ 时, 有 $s_{2^m-1} \\rightarrow+\\infty$, 与 $\\left\\{s_n\\right\\}$ 为有界数列矛盾.\n所以, 命题成立.", + "remark": "", + "figures": [] +} \ No newline at end of file diff --git a/processed_dataset/proof/1598.json b/processed_dataset/proof/1598.json new file mode 100644 index 0000000000000000000000000000000000000000..0c85f1dcd2fe3b2ba8f77935da575bb255a4dda7 --- /dev/null +++ b/processed_dataset/proof/1598.json @@ -0,0 +1,8 @@ +{ + "source_file": "./raw_volume-zh/volume6/exercise1.tex", + "problem_type": "proof", + "problem": "问题25. 函数 $F: \\mathbf{N} \\rightarrow \\mathbf{N}$, 具有下述性质: 对任意 $n \\in \\mathbf{N}$, 都有\n(1) $F(4 n)=F(2 n)+F(n)$;\n(2) $F(4 n+2)=F(4 n)+1$;\n(3) $F(2 n+1)=F(2 n)+1$.\n证明: 对任意正整数 $m$, 满足 $0 \\leqslant n \\leqslant 2^m$, 且 $F(4 n)=F(3 n)$ 的整数 $n$ 的个数为 $F\\left(2^{m+1}\\right)$.", + "solution": "在条件 (1) 中令 $n=0$, 可知 $F(0)=0$. 对 $n \\in \\mathbf{N}^*$, 设 $n$ 的二进制表示为 $n=\\left(n_k n_{k-1} \\cdots n_0\\right)=n_k \\cdot 2^k+\\cdots+n_0 \\cdot 2^0$, 这里 $n_k=1$, 而对 $0 \\leqslant i \\leqslant k-$ 1 , 有 $n_i \\in\\{0,1\\}$.\n我们对 $k$ 归纳来证明: 对任意 $n \\in \\mathbf{N}^*$, 都有(1):\n$$\nF(n)=n_k F_k+n_{k-1} F_{k-1}+\\cdots+n_0 F_0,\n$$\n这里数列 $\\left\\{F_m\\right\\}$ 定义为 $F_0=F_1=1, F_{m+2}=F_{m+1}+F_m, m=0,1,2, \\cdots$ (它是本书中 Fibonacci 数列下标向前平移一项所得).\n事实上, 由 $F(0)=0$ 及条件 (3) 可知 $F(1)=1$, 进而可得 $F(2)=1$, $F(3)=F(2)+1=F_0+F_1, F(4)=F(2)+F(1)=2=F_2$. 所以, 命题对 $k=0,1$ 成立.\n现设 (1) 对 $k$ 和 $k+1$ 成立, 考虑 $k+2$ 的情形, 此时可设 $n= \\left(n_{k+2} n_{k+1} \\cdots n_0\\right)_2$. 如果 $\\left(n_1, n_0\\right)=(0,0)$, 那么由 (1) 知 $F(n)= F\\left(\\left(n_{k+2} n_{k+1} \\cdots n_1\\right)_2\\right)+F\\left(\\left(n_{k+2} \\cdots n_2\\right)_2\\right)=n_{k+2} F_{k+1}+\\cdots+n_1 F_0+n_{k+2} F_k+\\cdots+ n_2 F_0=n_{k+2}\\left(F_{k+1}+F_k\\right)+\\cdots+n_2\\left(F_1+F_0\\right)+n_1 F_0=n_{k+2} F_{k+2}+\\cdots+n_2 F_2+ n_1 F_1+n_0 F_0$ (这里用到 $\\left.n_1=n_0=0\\right)$, (1) 对 $k+2$ 成立; 如果 $\\left(n_1, n_0\\right)=(1,0)$, 那么由 (2) 知 $F(n)=F\\left(\\left(n_{k+2} n_{k+1} \\cdots n_2 n_1^{\\prime} n_0^{\\prime}\\right)_2\\right)+1$, 这里 $n_1^{\\prime}=n_0^{\\prime}=0$, 于是, $F(n)=n_{k+2} F_{k+2}+\\cdots+n_2 F_2+1=n_{k+2} F_{k+2}+\\cdots+n_2 F_2+n_1 F_1+n_0 F_0$, (1) 也成立; 如果 $\\left(n_1, n_0\\right)=(0,1)$, 那么 $F(n)=F\\left(\\left(n_{k+2} \\cdots n_2 n_1^{\\prime} n_0^{\\prime}\\right)_2\\right)+1$, 这里 $n_1^{\\prime}=n_0^{\\prime}=0$, 利用条件 (1) 及前面的结论可知 (1) 成立; 如果 $\\left(n_1, n_0\\right)=(1$, $1)$, 那么 $F(n)=F\\left(\\left(n_{k+2} \\cdots n_1 n_0^{\\prime}\\right)\\right)+1$, 这里 $n_0^{\\prime}=0$, 利用条件 (2) 及前面的结论可知 (1) 成立.\n所以, (1) 对任意 $n \\in \\mathbf{N}^*$ 都成立.\n利用(1)可知 $F(4 n)=F(3 n)$ 的充要条件是 $n$ 的二进制表示 $\\left(n_k n_{k-1} \\cdots n_0\\right)_2$ 中没有相邻的两个数都等于 1 (这里还用到数列 $\\left\\{F_m\\right\\}$ 的定义). 在 $0 \\leqslant n<2^m$ 中, 记二进制表示中没有相邻的两个 1 的数的个数为 $f_m$, 则 $f_0=1, f_1=2$, 而去掉 $n$ 的末位数字 $n_0$ 后, 依 $n_0=0 、 1$ 分类, 分别有 $f_{m-1}$ 和 $f_{m-2}$ (因为 $n_0=1$ 时, 必有 $\\left.n_1=0\\right)$, 故 $f_m=f_{m-1}+f_{m-2}$. 这表明, 在 $0 \\leqslant n<2^m$ 中, 有 $F_{m+1}(= \\left.F\\left(2^{m+1}\\right)\\right)$ 个数 $n$ 满足 $F(4 n)=F(3 n)$, 从而命题成立.", + "remark": "", + "figures": [] +} \ No newline at end of file diff --git a/processed_dataset/proof/1599.json b/processed_dataset/proof/1599.json new file mode 100644 index 0000000000000000000000000000000000000000..e5b5706f1005fbf3f7ea1fcdc0e3f17a4f56ef0a --- /dev/null +++ b/processed_dataset/proof/1599.json @@ -0,0 +1,8 @@ +{ + "source_file": "./raw_volume-zh/volume6/exercise1.tex", + "problem_type": "proof", + "problem": "问题26. 函数 $f: \\mathbf{N}^* \\rightarrow \\mathbf{N}^*$ 定义如下 $f(1)=1$, 且对任意正整数 $n$, 都有\n$$\nf(n+1)=\\left\\{\\begin{array}{l}\nf(n)+2, \\text { 若 } n=f(f(n)-n+1), \\\\\nf(n)+1, \\text { 其他 } n .\n\\end{array}\\right.\n$$\n(1)证明: 对任意正整数 $n$, 都有 $f(f(n)-n+1) \\in\\{n, n+1\\}$;\n(2)求 $f(n)$ 的表达式.", + "solution": "由递推式, 可知 $f(n) \\leqslant f(n-1)+2 \\leqslant \\cdots \\leqslant f(1)+2(n-1)= 2 n-1$. 故 $f(n)-n+1 \\leqslant n$. 因此, 如果 $f(1), \\cdots, f(n)$ 的值确定了, 那么 $f(n+1)$ 的值唯一确定.\n从而, 存在唯一的函数 $f$ 满足条件.\n现在, 令 $g(n)=\\left[\\frac{1+\\sqrt{5}}{2} n\\right]$, 记 $\\alpha=\\frac{1+\\sqrt{5}}{2}$, 则 $g(1)=1$, 且对任意 $n \\in \\mathbf{N}^*$, 都有\n$$\ng(n+1)-g(n)=[\\alpha(n+1)]-[\\alpha n]=[\\alpha+\\varepsilon],\n$$\n这里 $\\varepsilon=\\{\\alpha n\\}=\\alpha n-[\\alpha n]$.\n另一方面, $g(g(n)-n+1)=[\\alpha(g(n)-n+1)]=[\\alpha(\\alpha n-\\varepsilon-n+1)]= \\left[\\left(\\alpha^2-\\alpha\\right) n+\\alpha(1-\\varepsilon)\\right]=n+[\\alpha(1-\\varepsilon)]$, 这里用到 $\\alpha^2-\\alpha-1=0$.\n注意到 $\\varepsilon \\neq 2-\\alpha=\\frac{3-\\sqrt{5}}{2}$ (否则 $1=\\frac{[\\alpha n]+\\varepsilon}{\\alpha}=\\frac{[\\alpha n]+2}{\\alpha}-1$, 导致 $\\alpha$ 为有理数,矛盾), 利用上述结论, 若 $0 \\leqslant \\varepsilon<2-\\alpha$, 则 $\\alpha(1-\\varepsilon)>\\alpha(\\alpha-1)=1$,\n从而 $g(g(n)-n+1)=n+1$, 此时 $1<\\alpha+\\varepsilon<\\alpha+2-\\alpha=2$, 即有 $g(n+ 1)-g(n)=1$; 若 $2-\\alpha<\\varepsilon<1$, 则 $\\alpha(1-\\varepsilon)<\\alpha(\\alpha-1)=1$, 从而 $g(g(n)- n+1)=n$, 这时 $2<\\alpha+\\varepsilon<3$, 即 $g(n+1)-g(n)=2$.\n上述讨论表明: $g: \\mathbf{N}^* \\rightarrow \\mathbf{N}^*$ 满足 $f$ 所满足的所有条件, 从而对任意 $n \\in \\mathbf{N}^*$, 有 $f(n)=g(n)$, 这给出了 (2) 要求的答案.\n结合(1)知 (1)成立.\n问题获解.", + "remark": "", + "figures": [] +} \ No newline at end of file diff --git a/processed_dataset/proof/1600.json b/processed_dataset/proof/1600.json new file mode 100644 index 0000000000000000000000000000000000000000..52bf7bcdd2f709038a98b4cf4d435a684a84c1df --- /dev/null +++ b/processed_dataset/proof/1600.json @@ -0,0 +1,8 @@ +{ + "source_file": "./raw_volume-zh/volume6/exercise1.tex", + "problem_type": "proof", + "problem": "问题27. 数列 $\\left\\{a_n\\right\\}$ 定义如下 $a_1=0, a_n=a_{\\left[\\frac{n}{2}\\right]}+(-1)^{\\frac{n(n+1)}{2}}, n=2,3, \\cdots$. 对每个非负整数 $k$, 求满足 $2^k \\leqslant n<2^{k+1}$, 且 $a_n=0$ 的下标 $n$ 的个数.", + "solution": "对每个 $n \\in \\mathbf{N}^*$, 设 $n$ 在二进制表示下, 相邻数对中 00 与 11 出现的次数和为 $x_n$, 相邻数对中 01 与 10 出现的次数和为 $y_n$. 我们证明: $a_n=x_n-y_n$, (1).\n事实上,当 $n=1$ 时, $x_1=y_1=0$,故 (1) 对 $n=1$ 成立.\n现设(1)对下标 $1,2, \\cdots, n-1(n \\geqslant 2)$ 都成立, 考虑 $n$ 的情形.\n如果二进制表示下, $n$ 的末两位为 00 或 11 ,则 $n \\equiv 0$ 或 $3(\\bmod 4)$,这时, $a_n=a_{\\left[\\frac{n}{2}\\right]}+1$, 而此时, $x_n=x_{\\left[\\frac{n}{2}\\right]}+1, y_n=y_{\\left[\\frac{n}{2}\\right]}$. 所以, (1) 对 $n$ 成立.\n如果二进制表示下, $n$ 的末两位为 01 或 10 , 则 $n \\equiv 1$ 或 $2(\\bmod 4)$; 这时 $a_n=a_{\\left[\\frac{n}{2}\\right]}-1$, 此时 $x_n=x_{\\left[\\frac{n}{2}\\right]}, y_n=y_{\\left[\\frac{n}{2}\\right]}+1$, 所以, (1) 对 $n$ 成立.\n综上, (1) 对任意 $n \\in \\mathbf{N}^*$ 都成立.\n现在需要计算 $2^k \\leqslant n<2^{k+1}$ 中, 在二进制表示下使得 $x_n$ 与 $y_n$ 相等的 $n$ 的个数.\n这时 $n$ 在二进制表示下是一个 $k+1$ 位数, 设为 $B_n$, 当 $k \\geqslant 1$ 时, 将 $B_n$ 的从左到右每一位减去它的下一位数, 然后取绝对值, 可得一个 $k$ 元的 $0 、 1$ 数组 $C_n$ (例如: 若 $B_n=(1101)_2$, 则 $C_n=(011)_2$ ), 注意到, $B_n$ 每一个相邻数对 00 与 11 变为 $C_n$ 中的一个 0 , 而 01 与 10 变为 $C_n$ 中的一个 1 . 所以,若 $x_n=y_n$, 则 $C_n$ 中 1 的个数与 0 的个数相同.\n反过来, 对一个由 $0 、 1$ 组成的 $k$ 元数组 $C_n=\\left(C_1 C_2 \\cdots C_k\\right)$, 则在 $\\bmod 2$ 意义下求下面的和 $b_1=1+c_1, b_2=b_1+c_2, \\cdots$, $b_k=b_{k-1}+b_k$, 这里 $b_0=1$, 那么, $B_n=\\left(b_0 b_1 \\cdots b_k\\right)_2$ 是一个满足 $2^k \\leqslant n<2^{k+1}$ 的数 $n$ 的二进制表示.\n这表明 $B_n$ 与 $C_n$ 之间是一个一一对应.\n所以, 原题中所求答案等于 $k$ 元 $0 、 1$ 数组中 0 与 1 的个数相等的数组的个数.\n因此,\n当 $k$ 为奇数时, 答案为 0 ; 当 $k$ 为偶数时, 答案为 $\\mathrm{C}_k^{\\frac{k}{2}}$ (注意, 这里认为 $\\mathrm{C}_0^0=1$ ).", + "remark": "", + "figures": [] +} \ No newline at end of file diff --git a/processed_dataset/proof/1601.json b/processed_dataset/proof/1601.json new file mode 100644 index 0000000000000000000000000000000000000000..2ecfdb0c2df3877c1a9e296a56fea306f640e362 --- /dev/null +++ b/processed_dataset/proof/1601.json @@ -0,0 +1,8 @@ +{ + "source_file": "./raw_volume-zh/volume6/exercise1.tex", + "problem_type": "proof", + "problem": "问题28. 数列 $\\left\\{x_n\\right\\}$ 满足 $x_1=1, x_{n+1}=\\left\\{\\begin{array}{l}x_n-2, \\text { 若 } x_n-2>0, \\text { 且 } x_n-2 \\notin \\\\ x_n+3, \\text { 其他情形, }\\end{array}\\right. \\left\\{x_1, \\cdots, x_n\\right\\}$, 证明: 对任意大于 1 的正整数 $k$, 存在下标 $n$, 使得 $x_{n+1}= x_n+3=k^2$.", + "solution": "我们证明: 对任意 $n \\in \\mathbf{N}$, 都有(1):\n$$\n\\begin{aligned}\n& x_{5 n+1}=5 n+1, x_{5 n+2}=5 n+4, x_{5 n+3}=5 n+2, \\\\\n& x_{5 n+4}=5 n+5, x_{5 n+5}=5 n+3 .\n\\end{aligned}\n$$\n(利用此结果及 $k^2 \\equiv 0,1$ 或 $4(\\bmod 5)$, 即可知命题成立).\n事实上,当 $n=0$ 时, 由 $a_1=1$ 知 $a_2=4, a_3=2, a_4=5, a_5=3$, 故 (1) 对 $n=0$ 成立.\n现设(1)对 $n=0,1,2, \\cdots, m-1\\left(m \\in \\mathbf{N}^*\\right)$ 都成立, 考虑 $n=m$ 的情形, 由 (1) 的结构 $\\left(a_{5 n+1}, \\cdots, a_{5 n+5}\\right.$ 是 $5 n+1, \\cdots, 5 n+5$ 的一个排列) 可知 $a_1$, $a_2, \\cdots, a_{5 m}$ 是 $1,2, \\cdots, 5 m$ 的一个排列, 利用递推关系式可知 $a_{5 m+1}=a_{5 m}- 2=5 m+1, a_{5 m+2}=a_{5 m+1}+3=5 m+4, a_{5 m+3}=a_{5 m+2}-2=5 m+2, a_{5 m+4}= a_{5 m+3}+3=5 m+5, a_{5 m+5}=a_{5 m+4}-2=5 m+2$. 所以, 结论 (1) 对 $m$ 也成立.", + "remark": "", + "figures": [] +} \ No newline at end of file diff --git a/processed_dataset/proof/1602.json b/processed_dataset/proof/1602.json new file mode 100644 index 0000000000000000000000000000000000000000..60876596917462bbaac7be7aa109d87061ea927b --- /dev/null +++ b/processed_dataset/proof/1602.json @@ -0,0 +1,8 @@ +{ + "source_file": "./raw_volume-zh/volume6/exercise1.tex", + "problem_type": "proof", + "problem": "问题29. 设 $n$ 是一个正奇数, $\\theta$ 是一个实数, 满足: $\\frac{\\pi}{\\theta}$ 是一个无理数.\n记 $a_k= \\tan \\left(\\theta+\\frac{k \\pi}{n}\\right), k=1,2, \\cdots, n$. 证明: $\\frac{a_1+a_2+\\cdots+a_n}{a_1 a_2 \\cdots a_n}$ 是一个整数,并确定其值.", + "solution": "利用 $\\frac{\\pi}{\\theta}$ 为无理数, 可知 $a_1, a_2, \\cdots, a_n$ 是 $n$ 个两两不同的实数.\n为确定所求代数式的值, 我们去寻找一个以 $a_1, \\cdots, a_n$ 为根的 $n$ 次多项式.\n注意到 $\\mathrm{e}^{\\mathrm{i} \\theta}=\\cos \\theta+\\mathrm{i} \\sin \\theta, \\mathrm{e}^{-\\mathrm{i} \\theta}=\\cos \\theta-\\mathrm{i} \\sin \\theta$, 于是, 有 $\\sec \\theta \\mathrm{e}^{\\mathrm{i} \\theta}= 1+\\mathrm{i} \\tan \\theta, \\sec \\theta \\cdot \\mathrm{e}^{-\\mathrm{i} \\theta}=1-\\mathrm{i} \\tan \\theta$, 所以(1)\n$$\n1+\\mathrm{itan} \\theta=\\mathrm{e}^{2 \\mathrm{i} \\theta}(1-\\mathrm{i} \\tan \\theta) .\n$$\n令 $\\omega=\\mathrm{e}^{2 i n \\theta}$, 则多项式 $Q_n(x)=(1+\\mathrm{i} x)^n-\\omega(1-\\mathrm{i} x)^n$ 有 $n$ 个根 $a_1, a_2, \\cdots$, $a_n$. (这一点由 (1) 可知, 因为 $\\omega$ 的 $n$ 次方根为 $\\left.\\mathrm{e}^{2 \\mathrm{i}\\left(\\theta+\\frac{k}{n} \\pi\\right)}, k=1,2, \\cdots, n\\right)$, 而 $Q_n(x)$ 是一个 $n$ 次多项式, 所以, $a_1, \\cdots, a_n$ 是 $Q_n(x)$ 的所有根.\n记 $Q_n(x)=c_n x^n+\\cdots+c_0$, 则由韦达定理知 $a_1+\\cdots+a_n=-\\frac{c_{n-1}}{c_n}, a_1 \\cdots a_n=(-1)^n \\cdot \\frac{c_0}{c_n}$, 所以 $\\frac{a_1+\\cdots+a_n}{a_1 \\cdots a_n}=(-1)^{n-1} \\cdot \\frac{c_{n-1}}{c_0}$.\n对 $Q_n(x)$ 用二项式定理, 知 $c_{n-1}=n \\cdot \\mathrm{i}^{n-1}-\\omega n(-\\mathrm{i})^{n-1}=n \\mathrm{i}^{n-1}(1-\\omega), c_0= 1-\\omega$, 从而 $\\frac{a_1+\\cdots+a_n}{a_1 \\cdots a_n}=n \\cdot(-\\mathrm{i})^{n-1}$, 结合 $n$ 为奇数, 可得\n$$\n\\frac{a_1+\\cdots+a_n}{a_1 \\cdots a_n}=(-1)^{\\frac{n-1}{2}} \\cdot n \\text {. }\n$$\n问题获解.", + "remark": "", + "figures": [] +} \ No newline at end of file diff --git a/processed_dataset/proof/1603.json b/processed_dataset/proof/1603.json new file mode 100644 index 0000000000000000000000000000000000000000..951e79bfa1968c175bf204db9e516567a7cb9d84 --- /dev/null +++ b/processed_dataset/proof/1603.json @@ -0,0 +1,8 @@ +{ + "source_file": "./raw_volume-zh/volume6/exercise1.tex", + "problem_type": "proof", + "problem": "问题30. 证明: 对任意 $n \\in \\mathrm{N}^*$, 存在一个首项系数为 1 的 $n$ 次整系数多项式 $P(x)$, 使得 $2 \\cos n \\varphi=P(2 \\cos \\varphi)$, 这里 $\\varphi$ 为任意实数.", + "solution": "当 $n=1$ 时, 取 $P(x)=x$ 即可; 当 $n=2$ 时, $2 \\cos 2 \\varphi=(2 \\cos \\varphi)^2-2$, 命题也成立.\n假设命题对 $n=k$ 和 $k+1$ 成立, 即存在首项系数为 1 的整系数多项式 $f(x)$ 和 $g(x)$, 使得\n$$\n2 \\cos k \\varphi=f(2 \\cos \\varphi), 2 \\cos (k+1) \\varphi=g(2 \\cos \\varphi) .\n$$\n其中 $f 、 g$ 的次数分别为 $k$ 和 $k+1$.\n下面考虑 $n=k+2$ 的情形.\n注意到\n$$\n\\begin{aligned}\n2 \\cos (k+2) \\varphi=2 \\cos [(k+1) \\varphi+\\varphi] \\\\\n& =2 \\cos (k+1) \\varphi \\cos \\varphi-2 \\sin (k+1) \\varphi \\sin \\varphi . \\label{(1)}\\\\\n2 \\cos k \\varphi & =2 \\cos [(k+1) \\varphi-\\varphi] \\\\\n& =2 \\cos (k+1) \\varphi \\cos \\varphi+2 \\sin (k+1) \\varphi \\cos \\varphi . \\label{(2)}\n\\end{aligned}\n$$\n将(1)与(2)相加, 得\n$$\n2 \\cos (k+2) \\varphi+2 \\cos k \\varphi=4 \\cos (k+1) \\varphi \\cos \\varphi .\n$$\n利用归纳假设, 可知\n$$\n2 \\cos (k+2) \\varphi=(2 \\cos \\varphi) g(2 \\cos \\varphi)-f(2 \\cos \\varphi) .\n$$\n故令 $h(x)=x g(x)-f(x)$ (易知 $h(x)$ 是一个首项系数为 1 的整系数多项式), 就有\n$$\n2 \\cos (k+2) \\varphi=h(2 \\cos \\varphi)\n$$\n命题对 $k+2$ 成立.\n所以, 命题成立.", + "remark": "", + "figures": [] +} \ No newline at end of file diff --git a/processed_dataset/proof/1604.json b/processed_dataset/proof/1604.json new file mode 100644 index 0000000000000000000000000000000000000000..7347f51b0bf2ff9cebd4dbc7ba451b91cc102d04 --- /dev/null +++ b/processed_dataset/proof/1604.json @@ -0,0 +1,8 @@ +{ + "source_file": "./raw_volume-zh/volume6/exercise1.tex", + "problem_type": "proof", + "problem": "问题32. 单位圆上是否存在无穷多个点, 使得其中任意两点之间的距离都是有理数?", + "solution": "不妨设所给单位圆方程为 $x^2+y^2=1$, 现取 $\\theta=\\arccos \\frac{3}{5}$, 则 $\\cos \\theta= \\frac{3}{5}, \\sin \\theta=\\frac{4}{5}$. 考虑由 $P_n(\\cos 2 n \\theta, \\sin 2 n \\theta)$ 组成的点集 $M, n=1,2, \\cdots$.\n对任意 $i 、 j \\in \\mathbf{N}^*$, 我们有\n$$\n\\begin{aligned}\n\\left|P_i P_j\\right|^2 & =(\\cos 2 i \\theta-\\cos 2 j \\theta)^2+(\\sin 2 i \\theta-\\sin 2 j \\theta)^2 \\\\\n& =2-2 \\cos 2(i-j) \\theta \\\\\n& =4 \\sin ^2(i-j) \\theta\n\\end{aligned}\n$$\n所以, $\\left|P_i P_j\\right|=2|\\sin (i-j) \\theta|$.\n注意到, $\\cos \\theta 、 \\sin \\theta \\in \\mathbf{Q}$, 由 $\\sin (n+1) \\theta=\\sin n \\theta \\cos \\theta+\\cos n \\theta \\sin \\theta$ 及 $\\cos (n+-1) \\theta=\\cos n \\theta \\cos \\theta-\\sin n \\theta \\sin \\theta$ 结合数学归纳法易证: 对任意 $n \\in \\mathbf{N}^*$, 都有 $\\sin n \\theta 、 \\cos n \\theta \\in \\mathbf{Q}$. 因此, $M$ 中任意两点之间的距离都是有理数.\n现在还需要证明: $M$ 是一个无穷点集.\n若否, 设 $M$ 是一个有限集, 则存在 $m 、 n \\in \\mathbf{N}^*, m \\neq n$, 使得 $2 m \\theta=2 n \\theta+ 2 k \\pi, k \\in \\mathbf{Z}$, 这表明 $\\theta=\\alpha \\pi, \\alpha \\in \\mathbf{Q}$. 又 $\\cos \\theta=\\frac{3}{5} \\in \\mathbf{Q}$, 由上题的结论, 知 $\\cos \\alpha \\pi \\in\\left\\{0, \\pm \\frac{1}{2}, \\pm 1\\right\\}$, 但 $\\cos \\theta=\\frac{3}{5} \\notin\\left\\{0, \\pm \\frac{1}{2}, \\pm 1\\right\\}$, 矛盾.\n所以, $M$ 是一个无限集.\n综上, 存在满足条件的无穷多个点.", + "remark": "", + "figures": [] +} \ No newline at end of file diff --git a/processed_dataset/proof/1605.json b/processed_dataset/proof/1605.json new file mode 100644 index 0000000000000000000000000000000000000000..34789415e0527b99d09b8d63dbe0253b5ba7b607 --- /dev/null +++ b/processed_dataset/proof/1605.json @@ -0,0 +1,8 @@ +{ + "source_file": "./raw_volume-zh/volume6/exercise1.tex", + "problem_type": "proof", + "problem": "问题33. 设 $n$ 为不小于 2 的正整数.\n求所有的实系数多项式\n$$\nP(x)=a_n x^n+a_{n-1} x^{n-1}+\\cdots+a_0,\\left(a_n \\neq 0\\right),\n$$\n使得 $P(x)$ 恰有 $n$ 个不大于- 1 的实根, 并且\n$$\na_0^2+a_1 a_n=a_n^2+a_0 a_{n-1} .\n$$", + "solution": "由条件,可设\n$$\nP(x)=a_n\\left(x+\\beta_1\\right)\\left(x+\\beta_2\\right) \\cdots\\left(x+\\beta_n\\right) .\n$$\n这里 $\\beta_i \\geqslant 1, i=1,2, \\cdots, n$,且 $a_n \\neq 0$.\n利用 $a_0^2+a_1 a_n=a_n^2+a_0 a_{n-1}$ 可知\n$$\na_n^2\\left(\\prod_{i=1}^n \\beta_i\\right)^2+a_n^2\\left(\\prod_{i=1}^n \\beta_i\\right) \\sum_{i=1}^n \\frac{1}{\\beta_i}=a_n^2+\\left(\\prod_{i=1}^n \\beta_i\\right)\\left(\\sum_{i=1}^n \\beta_i\\right) a_n^2 .\n$$\n于是\n$$\n\\prod_{i=1}^n \\beta_i-\\frac{1}{\\prod_{i=1}^n \\beta_i}=\\sum_{i=1}^n \\beta_i-\\sum_{i=1}^n \\frac{1}{\\beta_i} . \\label{(1)}\n$$\n下面对 $n$ 运用数学归纳法证明: 当 $\\beta_i \\geqslant 1, i=1,2, \\cdots, n$ 时,都有\n$$\n\\prod_{i=1}^n \\beta_i-\\frac{1}{\\prod_{i=1}^n \\beta_i} \\geqslant \\sum_{i=1}^n \\beta_i-\\sum_{i=1}^n \\frac{1}{\\beta_i} .\n$$\n等号当且仅当 $\\beta_1, \\cdots, \\beta_n$ 中有 $n-1$ 个数等于 1 时成立.\n当 $n=2$ 时,若 $\\beta_1 、 \\beta_2 \\geqslant 1$, 则有如下等价关系成立\n$$\n\\begin{aligned}\n\\beta_1 \\beta_2-\\frac{1}{\\beta_1 \\beta_2} & \\geqslant\\left(\\beta_1+\\beta_2\\right)-\\left(\\frac{1}{\\beta_1}+\\frac{1}{\\beta_2}\\right) \\\\\n& \\Leftrightarrow\\left(\\beta_1 \\beta_2\\right)^2-1 \\geqslant\\left(\\beta_1+\\beta_2\\right)\\left(\\beta_1 \\beta_2-1\\right) \\\\\n& \\Leftrightarrow\\left(\\beta_1 \\beta_2-1\\right)\\left(\\beta_1-1\\right)\\left(\\beta_2-1\\right) \\geqslant 0 .\n\\end{aligned}\n$$\n所以 $n=2$ 时, 上述命题成立.\n设命题对 $n=k$ 时成立, 当 $n=k+1$ 时, 我们令 $\\alpha=\\beta_k \\beta_{k+1}$, 由归纳假设, 可知\n$$\n\\prod_{i=1}^{k+1} \\beta_i-\\frac{1}{\\prod_{i=1}^{k+1} \\beta_i} \\geqslant\\left(\\sum_{i=1}^{k-1} \\beta_i-\\sum_{i=1}^{k-1} \\frac{1}{\\beta_i}\\right)+\\alpha-\\frac{1}{\\alpha},\n$$\n等号当且仅当 $\\beta_1, \\beta_2, \\cdots, \\beta_{k-1}, \\alpha$ 中有 $k-1$ 个等于 1 时成立.\n又由 $n=2$ 的情形, 可知 $\\alpha-\\frac{1}{\\alpha}=\\beta_k \\beta_{k+1}-\\frac{1}{\\beta_k \\beta_{k+1}} \\geqslant \\beta_k+\\beta_{k+1}-\\frac{1}{\\beta_k}-\\frac{1}{\\beta_{k+1}}$. 于是\n$$\n\\prod_{i=1}^{k+1} \\beta_i-\\frac{1}{\\prod_{i=1}^{k+1} \\beta_i} \\geqslant \\sum_{i=1}^{k+1} \\beta_i-\\sum_{i=1}^{k+1} \\frac{1}{\\beta_i},\n$$\n等号当且仅当 $\\beta_1, \\cdots, \\beta_{k-1}, \\alpha$ 中有 $k-1$ 个为 1 , 并且 $\\beta_k$ 与 $\\beta_{k+1}$ 中有一个为 1 时成立,而这等价于 $\\beta_1, \\cdots, \\beta_{k+1}$ 中有 $k$ 个为 1 时成立.\n由上述结论及(1)式可知, 形如 $P(x)=a_n(x+1)^{n-1}(x+\\beta), a_n \\neq 0$, $\\beta \\geqslant 1$ 的多项式为所有满足条件的多项式.", + "remark": "", + "figures": [] +} \ No newline at end of file diff --git a/processed_dataset/proof/1606.json b/processed_dataset/proof/1606.json new file mode 100644 index 0000000000000000000000000000000000000000..72aa1d96e3c057714a939da62011f43953182168 --- /dev/null +++ b/processed_dataset/proof/1606.json @@ -0,0 +1,8 @@ +{ + "source_file": "./raw_volume-zh/volume6/exercise1.tex", + "problem_type": "proof", + "problem": "问题34. 设 $P(x)$ 是一个整系数多项式, 满足: 对任意 $n \\in \\mathbf{N}^*$, 都有 $P(n)>n$. 并且对任意 $m \\in \\mathbf{N}^*$, 数列\n$$\nP(1), P(P(1)), P(P(P(1))), \\cdots\n$$\n中都有一项是 $m$ 的倍数.\n证明: $P(x)=x+1$.", + "solution": "记 $x_1=1, x_{n+1}=P\\left(x_n\\right), n=1,2, \\cdots$, 对固定的 $n \\in \\mathbf{N}^*, n \\geqslant 2$, 记 $x_n-1=M$, 则 $x_1 \\equiv 1 \\equiv x_n(\\bmod M)$, 从而 $P\\left(x_1\\right) \\equiv P\\left(x_n\\right)(\\bmod M)$, 即 $x_2 \\equiv x_{n+1}(\\bmod M)$. 这样利用数学归纳法, 可证: 对任意 $k \\in \\mathbf{N}^*$, 都有\n$$\nx_k \\equiv x_{n+k-1}(\\bmod M) . \\label{(1)}\n$$\n由条件 $x_1, x_2, \\cdots$ 中有一项为 $M$ 的倍数,故存在 $r \\in \\mathbf{N}^*$, 使得 $x_r \\equiv 0 (\\bmod M)$. 而由 (1) 知数列 $\\left\\{x_k\\right\\}$ 在 $\\bmod M$ 的意义下是一个以 $n-1$ 为周期的数列,故可设 $1 \\leqslant r \\leqslant n-1$.\n现由 $P(n)>n$, 可知 $x_13$, 令 $q=p^3$. 定义数列 $a_n$ 如下\n$$\na_n=\\left\\{\\begin{array}{l}\nn, n=0,1,2, \\cdots p-1, \\\\\na_{n-1}+a_{n-p}, n>p-1 .\n\\end{array}\\right.\n$$\n求 $a_q$ 除以 $p$ 所得的余数.", + "solution": "引理设 $n 、 k \\in \\mathbf{N}^*, n \\geqslant k p$, 则(1)\n$$\na_n=\\sum_{i=0}^k \\mathrm{C}_k^i a_{n-i(p-1)-k} .\n$$\n对 $k$ 归纳予以证明.\n当 $k=1$ 时, (1) 就是 $a_n=a_{n-1}+a_{n-p}$, 故 (1) 对 $k=1$ 成立.\n现设 (1) 对 $k$ 成立, 考虑 $k+1$ 的情形.\n此时 $n \\geqslant(k+1) p$,下标 $n-i(p- 1)-k(0 \\leqslant i \\leqslant k)$ 的最小值在 $i=k$ 时取到, 该最小值为 $n-k p \\geqslant p$, 所以, 下面求和式中的每一项都可用条件中的递推式.\n由归纳假设知, 当 $n \\geqslant(k+1) p$ 时, 有\n$$\n\\begin{aligned}\na_n & =\\sum_{i=0}^k \\mathrm{C}_k^i a_{n-i(p-1)-k} \\\\\n& =\\sum_{i=0}^k \\mathrm{C}_k^i\\left(a_{n-i(p-1)-k-1}+a_{n-i(p-1)-k-p}\\right) \\\\\n& =\\mathrm{C}_k^0 a_{n-k-1}+\\sum_{i=1}^k \\mathrm{C}_k^i a_{n-i(p-1)-k-1}+\\sum_{i=0}^{k-1} \\mathrm{C}_k^i a_{n-i(p-1)-k-p}+\\mathrm{C}_k^k a_{n-(k+1) p} \\\\\n& =\\mathrm{C}_{k+1}^0 a_{n-(k+1)}+\\sum_{i=0}^{k-1} \\mathrm{C}_k^{i+1} a_{n-(i+1)(p-1)-(k+1)}+\\sum_{i=0}^{k-1} \\mathrm{C}_k^i a_{n-(i+1)(p-1)-(k+1)}+\\mathrm{C}_{k+1}^{k+1} a_{n-(k+1) p} \\\\\n& =\\mathrm{C}_{k+1}^0 a_{n-(k+1)}+\\sum_{i=0}^{k-1}\\left(\\mathrm{C}_k^{i+1}+\\mathrm{C}_k^i\\right) a_{n-(i+1)(p-1)-(k+1)}+\\mathrm{C}_{k+1}^{k+1} a_{n-(k+1) p} \\\\\n& =\\sum_{i=0}^{k+1} \\mathrm{C}_{k+1}^i a_{n-(i+1)(p-1)-(k+1)} .\n\\end{aligned}\n$$\n最后一步,用到 $\\mathrm{C}_k^{i+1}+\\mathrm{C}_k^i=\\mathrm{C}_{k+1}^{i+1}$. 所以, (1)对 $k+1$ 成立,引理获证.\n下面利用引理来处理原题.\n当 $n \\geqslant p^2$ 时, 在引理中令 $k=p$, 就有\n$$\na_n=\\sum_{i=0}^p \\mathrm{C}_p^i a_{n-i(p-1)-p},\n$$\n熟知, 当 $1 \\leqslant i \\leqslant p-1$ 时, 有 $\\mathrm{C}_p^i \\equiv 0(\\bmod p)$, 所以, $a_n \\equiv a_{n-p}+a_{n-p^2}(\\bmod p)$, 这时结合 $a_n=a_{n-1}+a_{n-p}$, 可得 $a_{n-1} \\equiv a_{n-p^2}(\\bmod p)$, 这表明对任意 $t \\geqslant p^2-$ 1 , 有 $a_t \\equiv a_{t+p^2-1}(\\bmod p)$.\n由于 $p^3=p\\left(p^2-1\\right)+p$, 故 $a_{p^3}=a_{p+p\\left(p^2-1\\right)} \\equiv a_p(\\bmod p)$, 而 $a_p=a_0+ a_{p-1}=p-1$, 所以 $a_{p^3} \\equiv p-1(\\bmod p)$, 即 $a_{p^3}$ 除以 $p$ 所得的余数为 $p-1$.", + "remark": "", + "figures": [] +} \ No newline at end of file diff --git a/processed_dataset/proof/1614.json b/processed_dataset/proof/1614.json new file mode 100644 index 0000000000000000000000000000000000000000..5ff62bc9e4752c86d40e859ebefcd4caf155a1df --- /dev/null +++ b/processed_dataset/proof/1614.json @@ -0,0 +1,8 @@ +{ + "source_file": "./raw_volume-zh/volume6/exercise1.tex", + "problem_type": "proof", + "problem": "问题43. 设 $n$ 为不小于 2 的正整数, $2 \\leqslant b_0 \\leqslant 2 n-1, b_0$ 为整数,考虑由递推式\n$$\nb_{i+1}=\\left\\{\\begin{array}{l}\n2b_i-1, \\text { 若 } b_i \\leqslant n, \\\\\n2b_i-2 n, \\text { 若 } b_i>n\n\\end{array}\\right.\n$$\n定义的数列 $\\left\\{b_i\\right\\}$. 用 $p\\left(b_0, n\\right)$ 表示满足 $b_p=b_0$ 的最小下标 $p$.\n(1) 设 $k$ 为给定的正整数,求 $p\\left(2,2^k\\right)$ 和 $p\\left(2,2^k+1\\right)$ 的值;\n(2) 证明: 对任意 $n$ 和 $b_0$ 都有 $p\\left(b_0, n\\right) \\mid p(2, n)$.", + "solution": "为方便计, 记 $m=n-1, b_i=a_i+1$, 则 $1 \\leqslant a_0 \\leqslant 2 m$, 且\n$$\na_{i+1}= \\begin{cases}2 a_i, & \\text { 若 } a_i \\leqslant m, \\\\ 2 a_i-(2 m+1), & \\text { 若 } a_i>m .\\end{cases}\n$$\n这表明 $a_{i+1} \\equiv 2 a_i(\\bmod 2 m+1)$, 且 $1 \\leqslant a_i \\leqslant 2 m, i=1,2, \\cdots$.\n(1) 题中的 $p\\left(2,2^k\\right)$ 和 $p\\left(2,2^k+1\\right)$ 等价于针对 $\\left\\{a_i\\right\\}$ 求 $p\\left(1,2^k-1\\right)$ 和 $p\\left(1,2^k\\right)$. 前者等价于求最小 $l \\in \\mathbf{N}^*$, 使得 $2^l \\equiv 1\\left(\\bmod 2\\left(2^k-1\\right)+1\\right)$, 后者等价于求最小的 $l \\in \\mathbf{N}^*$, 使得 $2^l \\equiv 1\\left(\\bmod 2^{k+1}+1\\right)$.\n显然 $2^{k+1} \\equiv 1\\left(\\bmod 2\\left(2^k-1\\right)+1\\right)$, 而对 $1 \\leqslant t \\leqslant k$, 都有 $1 \\leqslant 2^t-1< 2^{k+1}-1=2\\left(2^k-1\\right)+1$, 故 $p\\left(1,2^k-1\\right)=k+1$.\n又 $2^{2(k+1)} \\equiv 1\\left(\\bmod 2^{k+1}+1\\right)$, 从而 $p\\left(1,2^k\\right) \\mid 2(k+1)$, 又对 $1 \\leqslant t \\leqslant k+$ 1 , 都有 $1 \\leqslant 2^t-1<2^{k+1}+1$, 于是 $p\\left(1,2^k\\right)>k+1$, 故 $p\\left(1,2^k\\right)=2(k+1)$.\n所以, 针对 $\\left\\{b_i\\right\\}$ 有 $p\\left(2,2^k\\right)=k+1, p\\left(2,2^k+1\\right)=2(k+1)$.\n(2) 还是转到 $\\left\\{a_i\\right\\}$ 上讨论, 要求证明: $p\\left(a_0, m\\right) \\mid p(1, m)$. 现设 $p(1$, $m)=t$, 则 $2^t \\equiv 1(\\bmod 2 m+1)$, 从而 $2^t a_0 \\equiv a_0(\\bmod 2 m+1)$, 这表明 $p\\left(a_0\\right.$, $m) \\mid t$ (这里用到类似于初等数论中阶的性质), 即有 $p\\left(a_0, m\\right) \\mid p(1, m)$, 命题成立.", + "remark": "", + "figures": [] +} \ No newline at end of file diff --git a/processed_dataset/proof/1615.json b/processed_dataset/proof/1615.json new file mode 100644 index 0000000000000000000000000000000000000000..f3e29b590e58b8af3e1bd7d7908324d00037c4ce --- /dev/null +++ b/processed_dataset/proof/1615.json @@ -0,0 +1,10 @@ +{ + "source_file": "./raw_volume-zh/volume6/exercise1.tex", + "problem_type": "proof", + "problem": "问题44. 在坐标平面上任给一条起点为 $(0,0)$, 终点为 $(1,0)$ 的折线.\n证明: 对任意 $n \\in \\mathbf{N}^*$, 在该折线上存在两点, 它们的纵坐标相同, 而横坐标相差 $\\frac{1}{n}$.", + "solution": "先建立一个引理 : 对任意 $\\alpha \\in(0,1)$, 折线上存在两点, 它们的纵坐标相同,横坐标相差 $\\alpha$ 或 $1-\\alpha$.\n事实上, 设 $\\Gamma$ 为题中所给的折线, $\\Gamma_1$ 为 $\\Gamma$ 向左平移 $\\alpha$ 个单位得到的折线, $\\Gamma_2$ 为 $\\Gamma$ 向右平移 $1-\\alpha$ 个单位得到的折线, 容易得到 $\\Gamma$ 与 $\\Gamma_1 U \\Gamma_2$ 至少有一个交点, 而这就是引理要求的结果 (如图 () 所示, 从 $\\Gamma$ 的最高点与最低点出发讨论\n 即可知 $\\Gamma$ 与 $\\Gamma_1 \\cup \\Gamma_2$ 有交点).\n下面利用引理来证明需要的结论.\n取 $\\alpha=\\frac{1}{2}$, 可知 $n=2$ 时, 结论成立; 取 $\\alpha=\\frac{1}{3}$, 则折线上有两点 $A 、 B$, 使得 $A B / / x$ 轴, 且 $|A B|=\\frac{1}{3}$ 或 $|A B|=\\frac{2}{3}$, 若 $|A B|=\\frac{1}{3}$, 则 $n=3$ 已成立, 若 $|A B|=\\frac{2}{3}$, 则考虑连结 $A 、 B$ 的 $\\Gamma$ 的子折线, 利用引理及 $n=2$ 的结论, 可知该子折线上存在点 $C 、 D$, 使 $C D / / A B$, 且 $|C D|=\\frac{1}{2}|A B|=\\frac{1}{3}$, 故 $n=3$ 时, 结论也成立.\n依此类推, 结合数学归纳法, 可知结论对任意 $n \\geqslant 2$ 均成立.", + "remark": "", + "figures": [ + "./images/volume6/figures/fig-c1p44.png" + ] +} \ No newline at end of file diff --git a/processed_dataset/proof/1616.json b/processed_dataset/proof/1616.json new file mode 100644 index 0000000000000000000000000000000000000000..d57a8b5e0c45190297a885784ffd6fc2421af579 --- /dev/null +++ b/processed_dataset/proof/1616.json @@ -0,0 +1,8 @@ +{ + "source_file": "./raw_volume-zh/volume6/exercise1.tex", + "problem_type": "proof", + "problem": "问题45. 有一个黑盒和标号为 $1,2, \\cdots, n$ 的 $n$ 个白盒, 在 $n$ 个白盒中共放了 $n$ 个白球, 允许进行如下操作: 若标号为 $k$ 的白盒中恰有 $k$ 个白球, 则从中取出这 $k$ 个球, 分别在黑盒和标号为 $1,2, \\cdots, k-1$ 的白盒中各放人一个球.\n证明: 对任意 $n \\in \\mathbf{N}^*$, 存在唯一的一种放置方式, 使得 $n$ 个球最初全在白盒中, 但经有限次操作后, $n$ 个球全部在黑盒中.", + "solution": "先用数学归纳法证明存在性.\n当 $n=1$ 时,显然存在; 设 $n$ 时,存在满足条件的放法 $T$, 考虑 $n+1$ 的情形, 这时先将 $n$ 个球依放法 $T$ 放人标号为 $1,2, \\cdots, n$ 的白盒中, 并设放好后, 最小的空盒号码为 $i(1 \\leqslant i \\leqslant n)$, 则依下法放人第 $n+1$ 个球: 从 $1,2, \\cdots, i-1$号白盒中各取一个球放人第 $i$ 号盒中, 并将第 $n+1$ 个球也放人 $i$ 号盒中, 易知这样的放置方法满足条件.\n再用数学归纳法 (仍对 $n$ 归纳) 证明: 放法是唯一的.\n当 $n=1 、 2$ 时, 唯一性显然成立; 设对 $n(\\geqslant 2)$ 时, 满足条件的放置方法只有一种, 记为 $T$.\n易知 $n+1 \\geqslant 3$ 时, 满足条件的放法中, 第 $n+1$ 个白盒子必为空盒, 于是, 若 $n+1$ 时存在两种满足条件的放法 $T_1$ 和 $T_2$. 注意到, 第 $n+1$ 号白盒 (为空盒) 可以拆走, 并且 $T_1$ 与 $T_2$ 经一步操作后, 白盒中有 $n$ 个球, 白盒个数也为 $n$ 个, 故它们都变为 $T$.\n设 $T_1 、 T_2$ 的第一次操作的白盒号分别为 $i_1 、 i_2$. 若 $i_1>i_2$, 则 $T_1$ 经第一次操作后第 $i_2$ 号白盒中有至少 1 个球,而 $T_2$ 经第一次操作后第 $i_2$ 号白盒中没有球, 不能都变为 $T$, 所以 $i_1 \\leqslant i_2$, 同理 $i_2 \\leqslant i_1$, 即有 $i_1=i_2$. 这时 $T_1 、 T_2$ 中盒号大于 $i_1$ 的白盒子中的球数相同, 小于 $i_1$ 的白盒子中的球数也相同 (否则, $T_1$ 与 $T_2$ 经一次操作后, 不能都变为 $T$ ), 因此 $i_1$ 号盒中的球数也相同, 从而 $T_1=T_2$. 这表明,存在唯一的满足条件的放置方法.", + "remark": "", + "figures": [] +} \ No newline at end of file diff --git a/processed_dataset/proof/1617.json b/processed_dataset/proof/1617.json new file mode 100644 index 0000000000000000000000000000000000000000..bb0b852a48940bb49b335147db5e0a4f4679d329 --- /dev/null +++ b/processed_dataset/proof/1617.json @@ -0,0 +1,8 @@ +{ + "source_file": "./raw_volume-zh/volume6/exercise1.tex", + "problem_type": "proof", + "problem": "问题46. 设 $R_0$ 是一个 $n$ 元数组, 其中每个数都属于 $\\{A, B, C\\}$. 定义序列 $R_0, R_1$, $R_2, \\cdots$ 如下: 如果 $R_j=\\left(x_1, \\cdots, x_n\\right)$, 那么 $R_{j+1}=\\left(y_1, y_2, \\cdots, y_n\\right)$, 这里\n$$\ny_i=\\left\\{\\begin{array}{l}\nx_i, \\text { 若 } x_i=x_{i+1}, \\\\\n\\{A, B, C\\} \\backslash\\left\\{x_i, x_{i+1}\\right\\}, \\text { 若 } x_i \\neq x_{i+1} .\n\\end{array}\\right.\n$$\n其中 $x_{n+1}=x_1$. 例如: 若 $R_0=(A, A, B, C)$, 则 $R_1=(A, C, A, B)$, $R_2=(B, B, C, C), \\cdots$.\n(1) 求所有的 $n \\in \\mathbf{N}^*$, 使得存在 $m \\in \\mathbf{N}^*$, 满足: 对任意 $R_0$ 都有 $R_m=R_0$.\n(2) 对 $n=3^k\\left(k \\in \\mathbf{N}^*\\right)$, 求满足 (1) 的最小正整数 $m$.", + "solution": "(1) 分别用 $0 、 1 、 2$ 表示 $A 、 B 、 C$, 在模 3 的意义来把握序列 $R_0$, $R_1, \\cdots$ 的变化情况.\n设 $R_j=\\left(x_1, \\cdots, x_n\\right), R_{j+1}=\\left(y_1, y_2, \\cdots, y_n\\right)$, 则对 $1 \\leqslant i \\leqslant n$, 均有 $y_i \\equiv-\\left(x_i+x_{i+1}\\right)(\\bmod 3)$.\n如果 $n$ 为偶数, 取 $R_0=(1,2,1,2, \\cdots, 1,2)$, 那么对任意 $m \\geqslant 1$, 均有 $R_m=(0,0, \\cdots, 0,0)$, 所以此时不存在符合要求的正整数 $m$. 如果 $n$ 为奇数, 由于不同的 $n$ 元数组 $\\left(x_1, \\cdots, x_n\\right)$ 至多 $3^n$ 组, 故对任意 $R_0$, 存在 $m_{R_0} \\in \\mathbf{N}^*$, 及 $k \\in \\mathbf{N}$, 使得 $R_k=R_{m_{R_0}+k}$. 我们证明: 若 $k \\geqslant 1$, 则 $R_{k-1}=R_{m_{R_0}+k-1}$ (从而依此类推可知 $R_0=R_{m_{R_0}}$ ).\n事实上, 设 $R_{k-1}=\\left(x_1, \\cdots, x_n\\right), R_{m_{R_0}+k-1}=\\left(y_1, \\cdots, y_n\\right)$, 则由 $R_k= R_{m_{R_0}+k}$, 可知 $-\\left(x_i+x_{i+1}\\right) \\equiv-\\left(y_i+y_{i+1}\\right)(\\bmod 3)$, 所以 $\\sum_{j=1}^n(-1)^j\\left(x_j+\\right. \\left.x_{j+1}\\right) \\equiv \\sum_{j=1}^n(-1)^j\\left(y_j+y_{j+1}\\right)(\\bmod 3)$, 结合 $n$ 为奇数, 可知 $-x_1+(-1)^n x_{n+1} \\equiv -y_1+(-1)^n y_{n+1}(\\bmod 3)$, 即 $-2 x_1 \\equiv-2 y_1(\\bmod 3), x_1 \\equiv y_1(\\bmod 3)$, 所以 $x_1=y_1$, 同理可证对 $i \\in\\{2, \\cdots, n\\}$, 均有 $x_i=y_i$, 所以 $R_{k-1}=R_{m_{R_0}+k-1}$.\n依上可知, 对任意 $R_0$, 存在 $m_{R_0}$, 使得 $R_0=R_{m_{R_0}}$, 于是, 在 $R_0$ 变化时, 取所有 $m_{R_0}$ 的最小公倍数 $m$, 则对任意 $R_0$, 均有 $R_0=R_m$.\n综上可知, 当且仅当 $n$ 为奇数时, 存在满足条件的 $m$.\n(2) 对 $n=3^k, k \\in \\mathbf{N}^*$, 满足条件((1) 中的条件) 的 $m$ 的最小值 $m=3^k$. 事实上, 对任意 $R_0=\\left(x_1, \\cdots, x_n\\right)$, 设 $R_{3^k}=\\left(y_1, \\cdots, y_n\\right)$, 则由前推出的模 3 意义下的关系式,易知\n$$\ny_p \\equiv-\\sum_{i=0}^{3^k} \\mathrm{C}_{3^i k} x_{i+p}(\\bmod 3),\n$$\n这里 $x_{i+p}$ 的下标在模 $n$ 的意义下取值, $p=1,2, \\cdots, n$. 注意到对 $1 \\leqslant i \\leqslant 3^k-1, \\mathrm{C}_{3^k}^i \\equiv 0(\\bmod 3)$, 所以 $y_p \\equiv-x_p-x_{3^k+p}=-2 x_p \\equiv x_p(\\bmod 3)$, 从而 $R_{3^k}=R_0$.\n另一方面, 设 $R_0=(0,0, \\cdots, 0,1)$, 则对 $00, P(-1)<0$, 故 $P\\left(x_1\\right)$ 不为常数, 有一项为 $x_1$ 的倍式,命题成立.\n假设命题对符合条件的含 $n-1$ 个变量的多项式都成立,考虑 $n$ 的情形.\n对满足条件的 $P\\left(x_1, x_2, \\cdots, x_n\\right)$, 我们令\n$$\nQ\\left(x_1, x_2, \\cdots, x_{n-1}\\right)=\\frac{1}{2}\\left[P\\left(x_1, x_2, \\cdots, x_{n-1}, 1\\right)-P\\left(x_1, \\cdots, x_{n-1},-1\\right)\\right],\n$$\n它是视 $P$ 为 $x_n$ 的多项式时 (其余变量 $x_1, \\cdots, x_{n-1}$ 视为常数), $x_n$ 的奇次项的系数和.\n由于当 $x_1, \\cdots, x_{n-1}$ 都用 +1 或 -1 代替时, 如果 -1 的个数为偶数,则 $P\\left(x_1, \\cdots, x_{n-1}, 1\\right)>0, P\\left(x_1, \\cdots, x_{n-1},-1\\right)<0$, 故 $Q\\left(x_1, \\cdots, x_{n-1}\\right)>0$ ; 类似地, 如果 -1 的个数为奇数, 那么 $Q\\left(x_1, \\cdots, x_{n-1}\\right)<0$. 利用归纳假设可知, $Q\\left(x_1, \\cdots, x_{n-1}\\right)$ 中有一项为 $x_1 x_2 \\cdots x_{n-1}$ 的倍式.\n注意到 $P\\left(x_1, \\cdots, x_n\\right)$ 是 $Q\\left(x_1, \\cdots, x_{n-1}\\right)$ 的每一项乘以 $x_n$ 的某个奇次幂(不同的项可能幕次不同)求和后得到, 所以, $P\\left(x_1, \\cdots, x_n\\right)$ 中有一项为 $x_1 \\cdots x_n$ 的倍式.\n综上可知, 命题成立.", + "remark": "", + "figures": [] +} \ No newline at end of file diff --git a/processed_dataset/proof/1623.json b/processed_dataset/proof/1623.json new file mode 100644 index 0000000000000000000000000000000000000000..9a19887dc80ab006665d14646f0d5abdb944bf05 --- /dev/null +++ b/processed_dataset/proof/1623.json @@ -0,0 +1,8 @@ +{ + "source_file": "./raw_volume-zh/volume6/exercise2.tex", + "problem_type": "proof", + "problem": "问题6. 设 $a_1, \\cdots, a_n$ 是一个由非负实数 (不全为零) 组成的数列, 定义\n$$\nm_k=\\max _{1 \\leqslant i \\leqslant k} \\frac{a_{k-i+1}+a_{k-i+2}+\\cdots+a_k}{i}, k=1,2, \\cdots, n .\n$$\n证明: 对任意正实数 $\\mu$, 满足 $m_k>\\mu$ 的下标 $k$ 的个数小于 $\\frac{a_1+a_2+\\cdots+a_n}{\\mu}$.", + "solution": "当 $n=1$ 时, $m_1=a_1$, 若 $\\mu \\geqslant a_1$, 则满足 $m_k>\\mu$ 的下标 $k$ 不存在,此时命题显然, 若 $\\mu\\mu$ 的下标 $k$ 的个数.\n如果 $m_n \\leqslant \\mu$, 那么对数列 $a_1, \\cdots, a_{n-1}$ 而言, 满足 $m_k>\\mu$ 的下标 $k$ 的个数也为 $r$, 此时由归纳假设知\n$$\nr<\\frac{a_1+\\cdots+a_{n-1}}{\\mu} \\leqslant \\frac{a_1+\\cdots+a_n}{\\mu} .\n$$\n命题对 $n$ 成立.\n如果 $m_n>\\mu$, 那么, 存在 $i \\in\\{1,2, \\cdots, n\\}$, 使得 $\\frac{a_{n-i+1}+\\cdots+a_n}{i}>\\mu$. 对这个 $i$, 就数列 $a_1, a_2, \\cdots, a_{n-i}$ 而言, 至少有 $r-i$ 个下标 $k$ 满足 $m_k>\\mu$, 从而, 由归纳假设知\n$$\nr-i<\\frac{a_1+\\cdots+a_{n-i}}{\\mu}\n$$\n于是\n$$\n\\left(a_1+\\cdots+a_{n-i}\\right)+\\left(a_{n-i+1}+\\cdots+a_n\\right)>(r-i) \\mu+i \\mu=r \\mu,\n$$\n故 $r<\\frac{a_1+a_2+\\cdots+a_n}{\\mu}$.\n命题获证.", + "remark": "", + "figures": [] +} \ No newline at end of file diff --git a/processed_dataset/proof/1624.json b/processed_dataset/proof/1624.json new file mode 100644 index 0000000000000000000000000000000000000000..fed9511d2b722a85a1eb2f9763a387904be3a06b --- /dev/null +++ b/processed_dataset/proof/1624.json @@ -0,0 +1,8 @@ +{ + "source_file": "./raw_volume-zh/volume6/exercise2.tex", + "problem_type": "proof", + "problem": "问题7. (Jenson 不等式) 设 $f(x)$ 是 $[a, b]$ 上的凸函数 (即对任意 $x 、 y \\in[a, b]$, 都有 $\\left.f\\left(\\frac{x+y}{2}\\right) \\leqslant \\frac{1}{2}(f(x)+f(y))\\right)$.\n证明: 对任意 $n$ 个数 $x_1, \\cdots, x_n \\in[a, b]$,都有\n$$\nf\\left(\\frac{x_1+\\cdots+x_n}{n}\\right) \\leqslant \\frac{1}{n}\\left(f\\left(x_1\\right)+\\cdots+f\\left(x_n\\right)\\right) .\n$$", + "solution": "对比第 10 节中平均值不等式的证明二, 用其中出现的方法来证这个应用广泛的 Jenson 不等式.\n当 $n=1,2$ 时,不等式显然成立.\n现设不等式对 $n=2^k\\left(k \\in \\mathbf{N}^*\\right)$ 成立, 则由 $f$ 的定义, 可知\n$$\n\\begin{aligned}\nf\\left(\\frac{x_1+\\cdots+x_{2^{k+1}}}{2^{k+1}}\\right) & \\leqslant \\frac{1}{2}\\left(f\\left(\\frac{x_1+\\cdots+x_{2^k}}{2^k}\\right)+f\\left(\\frac{x_{2^k+1}+\\cdots+x_{2^{k+1}}}{2^k}\\right)\\right) \\\\\n& \\leqslant \\frac{1}{2}\\left(\\frac{1}{2^k} \\sum_{j=1}^{2^k} f\\left(x_j\\right)+\\frac{1}{2^k} \\sum_{j=1}^{2^k} f\\left(x_{2^k+j}\\right)\\right) \\\\\n& =\\frac{1}{2^{k+1}} \\sum_{j=1}^{2^{k+1}} f\\left(x_j\\right) .\n\\end{aligned}\n$$\n因此,不等式对任意 $n=2^k\\left(k \\in \\mathbf{N}^*\\right)$ 都成立.\n对一般的 $n \\in \\mathbf{N}^*(n \\geqslant 3)$, 设 $2^k \\leqslant n<2^{k+1}, k \\in \\mathbf{N}^*$, 记 $A=\\frac{1}{n}\\left(x_1+\\cdots+\\right. x_n$ ), 则由不等式对 $2^{k+1}$ 成立, 知\n$$\nf\\left(\\frac{x_1+\\cdots+x_n+\\left(2^{k+1}-n\\right) A}{2^{k+1}}\\right) \\leqslant \\frac{1}{2^{k+1}}\\left(\\sum_{j=1}^n f\\left(x_j\\right)+\\left(2^{k+1}-n\\right) f(A)\\right),\n$$\n而 $\\frac{1}{2^{k+1}}\\left(x_1+\\cdots+x_n+\\left(2^{k+1}-n\\right) A\\right)=\\frac{1}{2^{k+1}}\\left(n A+\\left(2^{k+1}-n\\right) A\\right)=A$, 于是, 我们有\n$$\n2^{k+1} f(A) \\leqslant \\sum_{j=1}^n f\\left(x_j\\right)+\\left(2^{k+1}-n\\right) f(A),\n$$\n故 $f(A) \\leqslant \\frac{1}{n} \\sum_{j=1}^n f\\left(x_j\\right)$, 即不等式对 $n$ 成立.\n命题获证.", + "remark": "", + "figures": [] +} \ No newline at end of file diff --git a/processed_dataset/proof/1625.json b/processed_dataset/proof/1625.json new file mode 100644 index 0000000000000000000000000000000000000000..64ba25255ac55f273350fd8f780d7b02a04d1b77 --- /dev/null +++ b/processed_dataset/proof/1625.json @@ -0,0 +1,8 @@ +{ + "source_file": "./raw_volume-zh/volume6/exercise2.tex", + "problem_type": "proof", + "problem": "问题8. 设正实数 $x_1, \\cdots, x_n$ 满足 $x_1+\\cdots+x_n=1$, 这里 $n \\in \\mathbf{N}^*, n \\geqslant 2$. 证明:\n$$\n\\prod_{k=1}^n\\left(1+\\frac{1}{x_k}\\right) \\geqslant \\prod_{k=1}^n\\left(\\frac{n-x_k}{1-x_k}\\right) .\n$$", + "solution": "引理设 $f(x)$ 是 $(0,1)$ 上的凸函数, $n \\in \\mathbf{N}^*, n \\geqslant 2$, 正实数 $x_1, \\cdots, x_n$ 满足 $x_1+\\cdots+x_n=1$, 则\n$$\n\\sum_{i=1}^n f\\left(x_i\\right) \\geqslant \\sum_{i=1}^n f\\left(\\frac{1-x_i}{n-1}\\right) .\n$$\n引理的证明: 由 Jenson 不等式, 知\n$$\n\\begin{aligned}\n\\sum_{i=1}^n f\\left(x_i\\right) & =\\sum_{i=1}^n\\left(\\frac{1}{n-1} \\sum_{j \\neq i} f\\left(x_j\\right)\\right) \\geqslant \\sum_{i=1}^n f\\left(\\frac{1}{n-1} \\sum_{j \\neq i} x_j\\right) \\\\\n& =\\sum_{i=1}^n f\\left(\\frac{1-x_i}{n-1}\\right) .\n\\end{aligned}\n$$\n于是引理成立.\n回证原题.\n令 $f(x)=\\ln \\frac{1+x}{x}$, 注意到, 对任意 $x, y \\in(0, \\cdot 1)$, 都有\n$$\n\\begin{aligned}\nf(x)+f(y) & =\\ln \\frac{1+x}{x}+\\ln \\frac{1+y}{y}=\\ln \\frac{1+x y+x+y}{x y} \\\\\n& =\\ln \\left(\\frac{1}{x y}+\\frac{x+y}{x y}+1\\right) \\\\\n& \\geqslant \\ln \\left(\\frac{1}{\\left(\\frac{x+y}{2}\\right)^2}+\\frac{x+y}{\\left(\\frac{x+y}{2}\\right)^2}+1\\right) \\\\\n& =\\ln \\left(\\frac{4}{(x+y)^2}+\\frac{4}{x+y}+1\\right)=\\ln \\left(\\frac{(x+y+2)^2}{(x+y)^2}\\right) \\\\\n& =2 \\ln \\left(1+\\frac{1}{\\frac{x+y}{2}}\\right)=2 f\\left(\\frac{x+y}{2}\\right) .\n\\end{aligned}\n$$\n所以, $f(x)=\\ln \\frac{1+x}{x}$ 是 $(0,1)$ 上的凸函数, 依此结合前面所得可知命题成立.", + "remark": "", + "figures": [] +} \ No newline at end of file diff --git a/processed_dataset/proof/1626.json b/processed_dataset/proof/1626.json new file mode 100644 index 0000000000000000000000000000000000000000..7ec04fd798cf367f619abcec14496e1dc04b80f0 --- /dev/null +++ b/processed_dataset/proof/1626.json @@ -0,0 +1,8 @@ +{ + "source_file": "./raw_volume-zh/volume6/exercise2.tex", + "problem_type": "proof", + "problem": "问题9. 斐波那契数列 $\\left\\{F_n\\right\\}$ 满足: $F_1=F_2=1, F_{n+2}=F_{n+1}+F_n$. 证明: $\\sum_{i=1}^n \\frac{F_i}{2^i}<2$.", + "solution": "记 $S_n=\\sum_{i=1}^n \\frac{F_i}{2^i}$, 则 $S_1=\\frac{1}{2}, S_2=\\frac{1}{2}+\\frac{1}{4}=\\frac{3}{4}$, 而当 $n \\geqslant 3$ 时, 有\n$$\n\\begin{aligned}\nS_n & =\\frac{1}{2}+\\frac{1}{4}+\\sum_{i=3}^n \\frac{F_i}{2^i} \\\\\n& =\\frac{3}{4}+\\sum_{i=3}^n \\frac{F_{i-1}+F_{i-2}}{2^i} \\\\\n& =\\frac{3}{4}+\\frac{1}{2} \\sum_{i=3}^n \\frac{F_{i-1}}{2^{i-1}}+\\frac{1}{4} \\sum_{i=3}^n \\frac{F_{i-2}}{2^{i-2}} \\\\\n& =\\frac{3}{4}+\\frac{1}{2} \\sum_{i=2}^{n-1} \\frac{F_i}{2^i}+\\frac{1}{4} \\sum_{i=1}^{n-2} \\frac{F_i}{2^i} \\\\\n& =\\frac{3}{4}+\\frac{1}{2}\\left(S_{n-1}-\\frac{1}{2}\\right)+\\frac{1}{4} S_{n-2} \\\\\n& =\\frac{1}{2}+\\frac{1}{2} S_{n-1}+\\frac{1}{4} S_{n-2} .\n\\end{aligned}\n$$\n利用 $S_1=\\frac{1}{2}$ 及 $S_2=\\frac{3}{4}$ 可知对 $n=1,2$ 都有 $S_n<2$; 现设对 $n=k, k+1$ 都有 $S_n<2$, 那么有\n$$\nS_{k+2}=\\frac{1}{2}+\\frac{1}{2} S_{k+1}+\\frac{1}{4} S_k<\\frac{1}{2}+\\frac{1}{2} \\times 2+\\frac{1}{4} \\times 2=2 .\n$$\n所以, 命题成立.", + "remark": "", + "figures": [] +} \ No newline at end of file diff --git a/processed_dataset/proof/1627.json b/processed_dataset/proof/1627.json new file mode 100644 index 0000000000000000000000000000000000000000..e33b23fb69df0b8ab53d9519c705b87e6e879369 --- /dev/null +++ b/processed_dataset/proof/1627.json @@ -0,0 +1,8 @@ +{ + "source_file": "./raw_volume-zh/volume6/exercise2.tex", + "problem_type": "proof", + "problem": "问题11. Fibonacci 数列 $\\left\\{F_n\\right\\}$ 定义如下: $F_1=F_2=1, F_{n+2}=F_{n+1}+F_n, n=1$, $2, \\cdots$. 求所有的正整数数对 $(k, m), m>k$. 使得如下定义的数列 $\\left\\{x_n\\right\\}$ :\n$$\nx_1=\\frac{F_k}{F_m}, x_{n+1}=\\left\\{\\begin{array}{ll}\n\\frac{2 x_n-1}{1-x_n}, & \\text { 若 } x_n \\neq 1, \\\\\n1, & \\text { 若 } x_n=1\n\\end{array}(n=1,2, \\cdots)\\right.\n$$\n包含等于 1 的项.", + "solution": "若 $m \\geqslant k+2$, 则 $F_m \\geqslant F_{k+2}=F_{k+1}+F_k \\geqslant 2 F_k$ (因为数列 $\\left\\{F_n\\right\\}$ 是不减数列), 于是 $x_1 \\leqslant \\frac{1}{2}$, 结合 $\\left\\{x_n\\right\\}$ 的定义可知 $x_2 \\leqslant 0$, 进而利用数学归纳法易证: 对 $n \\geqslant 2$, 都有 $x_n \\leqslant 0$. 此时, $\\left\\{x_n\\right\\}$ 中不包含等于 1 的项.\n所以 $mk$, 故只能是 $m=k+1$.\n另一方面, 对任意 $k \\in \\mathbf{N}^*$, 若 $m=k+1$, 则由 $\\left\\{x_n\\right\\}$ 的定义可知 $x_2=$\n130 $\\frac{2 F_k-F_{k+1}}{F_{k+1}-F_k}$ (除非 $k=1, m=2$, 此时 $x_1=1$, 数列中已包含 1 ), 得 $x_2= \\frac{F_k-F_{k-1}}{F_{k-1}}=\\frac{F_{k-2}}{F_{k-1}}$, 依此递推, 当 $k$ 为奇数时, 设 $k=2 n+1$, 则有 $x_3=\\frac{F_{2 n-3}}{F_{2 n-2}}, \\cdots x_{n+1}=\\frac{F_1}{F_2}=1$, 符合题意; 当 $k$ 为偶数时, 设 $k=2 n$, 则有 $x_3=\\frac{F_{2 n-4}}{F_{2 n-3}}, \\cdots, x_n= \\frac{F_2}{F_3}=\\frac{1}{2}$, 此后数列的每一项都不大于零, 不符合题意.\n综上可知,所求正整数对 $(k, m)=(2 n-1,2 n), n \\in \\mathbf{N}^*$.", + "remark": "", + "figures": [] +} \ No newline at end of file diff --git a/processed_dataset/proof/1628.json b/processed_dataset/proof/1628.json new file mode 100644 index 0000000000000000000000000000000000000000..7c5434a886e33c407e27686231393739eeb17d86 --- /dev/null +++ b/processed_dataset/proof/1628.json @@ -0,0 +1,8 @@ +{ + "source_file": "./raw_volume-zh/volume6/exercise2.tex", + "problem_type": "proof", + "problem": "问题12. 小张从 $\\{1,2, \\cdots, 144\\}$ 中任取一个数, 小王希望有偿地知道小张所取的数.\n小王每次可从 $\\{1,2, \\cdots, 144\\}$ 中取一个子集 $M$, 然后问小张: \"你取的数是否属于 $M$ ?\" 如果答案是 Yes, 则小王付给小张 2 元钱, 答案是 $\\mathrm{No}$, 则付 1 元.\n问: 小王至少需要支付多少元钱,才能保证可以知道小张所取的数?", + "solution": "答案是 11 元钱.\n设 $f(n)$ 是从 $\\{1,2, \\cdots, n\\}$ 中确定小张所取的数所需支付的最少钱数, 则 $f(n)$ 是一个不减数列.\n并且如果小王第一次所取的子集是一个 $m$ 元集, 那么 $f(n) \\leqslant \\max \\{f(m)+2, f(n-m)+1\\}$.\n下面我们利用 Fibonacci 数列 $\\left\\{F_n\\right\\}$, 证明下述结论: 设 $x$ 为正整数, 并且 $F_ns a_s$, 即 $a_{s+1}> s\\left(a_s-a_{s+1}\\right)$, 则 $\\frac{a_{s+1}}{a_s-a_{s+1}}>s$.\n注意到 $\\left[a_s, a_{s+1}\\right]=\\frac{a_s a_{s+1}}{\\left(a_s, a_{s+1}\\right)}$, 利用 $\\frac{a_{s+1}}{\\left(a_s, a_{s+1}\\right)} \\geqslant \\frac{a_{s+1}}{a_s-a_{s+1}}>s$, 结合 $\\frac{a_{s+1}}{\\left(a_s, a_{s+1}\\right)} \\in \\mathbf{N}^*$, 可知 $\\frac{a_{s+1}}{\\left(a_s, a_{s+1}\\right)} \\geqslant s+1$. 于是\n$$\n(s+1) a_{s+1}<(s+1) a_s \\leqslant \\frac{a_{s+1}}{\\left(a_s, a_{s+1}\\right)} \\cdot a_s=\\left[a_s, a_{s+1}\\right] \\leqslant k .\n$$\n故命题对 $s+1$ 也成立.", + "remark": "", + "figures": [] +} \ No newline at end of file diff --git a/processed_dataset/proof/1632.json b/processed_dataset/proof/1632.json new file mode 100644 index 0000000000000000000000000000000000000000..e9d235014f010c339b498637da50b58d884e9ccf --- /dev/null +++ b/processed_dataset/proof/1632.json @@ -0,0 +1,8 @@ +{ + "source_file": "./raw_volume-zh/volume6/exercise2.tex", + "problem_type": "proof", + "problem": "问题16. 设 $a_0u_n$;\n(2) 数 $u_0, u_1, \\cdots, u_{n+1}$ 中没有 3 个数成等差数列.\n求 $u_{100}$ 的值.", + "solution": "设二进制表示下, $n=\\left(a_k a_{k-1} \\cdots a_0\\right)_2$, 这里 $a_i \\in\\{0,1\\}, a_k=1$, 令 $t_n=\\left(a_k a_{k-1} \\cdots a_0\\right)_3$ (为一个 3 进制表示下的正整数), $t_0=0$.\n我们用数学归纳法证明: 对任意的 $n \\in \\mathbf{N}^*$, 均有 $u_n=t_n$.\n当 $n=1$ 时,命题显然成立.\n设对任意的 $m1)$ 满足 $\\left(b^n-1\\right) \\mid a$.\n证明: 在 $b$ 进制表示下,数 $a$ 的表示中至少出现 $n$ 个非零数码.", + "solution": "在 $b$ 进制表示下予以讨论.\n设能被 $b^n-1$ 整除的所有数中, 其 $b$ 进制表示下出现的非零数字个数的最小值为 $s$, 并在所有这些非零数字个数为 $s$ 的数中, 取数码和最小的数 $A$.\n设 $A=a_1 b^{n_1}+a_2 b^{n_2}+\\cdots+a_s b^{n_s}$ 为 $A$ 的 $b$ 进制表示, 这里\n$n_1>n_2>\\cdots>n_s \\geqslant 0,1 \\leqslant a_i1$, 记 $h(n)$ 为 $n$ 的最大素因数.\n证明: 存在无穷多个 $n \\in \\mathbf{N}^*$, 使得(1)\n$$\nh(n)1$, 记 $w(n)$ 为 $n$ 的不同素因数的个数.\n证明:存在无穷多个 $n \\in \\mathbf{N}^*$, 使得\n$$\nw(n)1$.\n事实上,若 $2^k+1=p^m, p$ 为素数, $m \\in \\mathbf{N}^*$, 写 $k=2^\\alpha \\cdot \\beta, \\alpha \\geqslant 0, \\beta>1$, $\\beta$ 为奇数.\n分两种情形:\n(1) $\\alpha=0$, 则由 $k \\neq 3$ 知 $\\beta>3$, 此时, $2^\\beta+1=(2+1)\\left(2^{\\beta-1}-2^{\\beta-2}+\\cdots+\\right.1)$ 是 3 的倍数, 且 $2^\\beta+1>9$, 若 $2^\\beta+1=3^\\gamma$, 则 $\\gamma \\geqslant 3$, 此时, 两边 $\\bmod 4$, 知 $(-1)^\\gamma \\equiv 1(\\bmod 4)$, 从而 $\\gamma$ 为偶数, 记 $\\gamma=2 \\delta$, 则 $2^\\beta=\\left(3^\\delta-1\\right)\\left(3^\\delta+1\\right)$, 而 $3^\\delta-1$ 与 $3^\\delta+1$ 是相邻偶数, 其积为 2 的幂, 只能是 $3^\\delta-1=2$, 得 $\\delta=1, \\gamma=$ 2 ,矛盾.\n故 $\\alpha=0$ 时, 引理成立.\n(2) $\\alpha>0$, 此时, 利用因式分解知 $2^{2^\\alpha}+1 \\mid 2^k+1$. 若 $w\\left(2^k+1\\right)=1$, 则 $p=2^{2^\\alpha}+1$ 为素数, 此时, 设 $2^{2^\\alpha \\cdot \\beta}+1=p^u$, 即 $(p-1)^\\beta+1=p^u, u \\geqslant 2$. 两边 $\\bmod p^2$, 利用二项式定理, 可知 $p \\mid \\beta$, 进一步, 设 $\\beta=p^v \\cdot x, p \\nmid x$, 由二项式定理, 可知\n$$\np^u=p^\\beta-\\mathrm{C}_\\beta^{\\beta-1} p^{\\beta-1}+\\cdots+\\mathrm{C}_\\beta^2 p^2-\\beta \\cdot p,\n$$\n右边最后一项为 $p^{v+1}$ 的倍数, 但不是 $p^{v+2}$ 的倍数, 而其余每一项都是 $p^{v+2}$ 的倍数.\n故上式不能成立.\n所以 $\\alpha>0$ 时,引理也成立.\n利用上述引理, 可知当 $k \\neq 3$ 且 $k$ 不是 2 的方幂时, 有 $w\\left(2^k\\right)5$, 对每个 $k \\in\\left\\{k_0+1, \\cdots, 2 k_0-1\\right\\}$ 都有 $w\\left(2^k+1\\right) \\geqslant w\\left(2^k+2\\right)=1+ w\\left(2^{k-1}+1\\right)$. 于是, 有\n$$\nw\\left(2^{2 k_0-1}+1\\right) \\geqslant 1+w\\left(2^{2 k_0-2}+1\\right) \\geqslant \\cdots \\geqslant\\left(k_0-1\\right)+w\\left(2^{k_0}+1\\right) \\geqslant k_0 .\n$$\n这要求 $2^{2 k_0-1}+1 \\geqslant p_1 \\cdots p_{k_0}$, 这里 $p_1, \\cdots, p_{k_0}$ 是最初的 $k_0$ 个素数.\n但是 $p_1 \\cdots p_{k_0} \\geqslant(2 \\times 3 \\times 5 \\times 7 \\times 11) \\times\\left(p_6 \\cdots p_{k_0}\\right)>4^5 \\cdot 4^{k_0-5}=2^{2 k_0}$, 矛盾.\n所以, 命题成立.", + "remark": "", + "figures": [] +} \ No newline at end of file diff --git a/processed_dataset/proof/1637.json b/processed_dataset/proof/1637.json new file mode 100644 index 0000000000000000000000000000000000000000..cc27e840a9f0f431cdda4b919614f5321182551a --- /dev/null +++ b/processed_dataset/proof/1637.json @@ -0,0 +1,8 @@ +{ + "source_file": "./raw_volume-zh/volume6/exercise2.tex", + "problem_type": "proof", + "problem": "问题21. 用 $\\dot{a}_n$ 表示前 $n$ 个素数之和.\n证明: 对任意 $n \\in \\mathbf{N}^*$, 区间 $\\left[a_n, a_{n+1}\\right]$ 中至少有一个完全平方数.", + "solution": "设素数从小到大的排列为 $p_1, p_2, \\cdots, p_n^{\\prime}, \\cdots$. 则 $a_n=p_1+ p_2+\\cdots+p_n$.\n当 $n=1,2,3,4$ 时, 直接验证, 可知命题成立.\n现设 $n-1$ 时命题成立, 即存在正整数 $x$, 使得 $a_{n-1} \\leqslant x^2 \\leqslant a_n$, 取其中最大的 $x$, 记为 $y$, 则 $y^2 \\leqslant a_n$, 而 $(y+ 1)^2>a_n$. 这里 $n \\geqslant 5$.\n写 $p_{n+1}=2 k+1$, 则当 $n \\geqslant 5$ 时, 利用相邻两个奇素数至少差 2 可知\n$$\np_1+p_2+\\cdots+p_n<1+3+5+\\cdots+(2 k-1)=k^2 .\n$$\n从而, $y^2 \\leqslant a_n$ 1. 另外, $5 \\nmid a_0, 3 \\nmid a_1, 2 \\nmid a_3$, 而且每一个大于 5 的素数至多整除 $\\left\\{a_n\\right\\}$ 中的一项, 因此, 没有一个大于 1 的正整数能整除 $\\left\\{a_n\\right\\}$ 中的每一项, 故 (2) 亦满足.", + "remark": "", + "figures": [] +} \ No newline at end of file diff --git a/processed_dataset/proof/1641.json b/processed_dataset/proof/1641.json new file mode 100644 index 0000000000000000000000000000000000000000..69d7454f2f9f4efb3ff9a1b9d264984e40ef110c --- /dev/null +++ b/processed_dataset/proof/1641.json @@ -0,0 +1,8 @@ +{ + "source_file": "./raw_volume-zh/volume6/exercise2.tex", + "problem_type": "proof", + "problem": "问题25. 设 $p$ 为奇素数, $a_1, a_2, \\cdots, a_{p-2}$ 是一个正整数数列, 满足: 对任意 $k \\in\\{1$, $2, \\cdots, p-2\\}$ 都有 $p \\nmid a_k\\left(a_k^k-1\\right)$. 证明: 可以从 $a_1, a_2, \\cdots, a_{p-2}$ 中取出若干个数,使得它们的乘积 $\\equiv 2(\\bmod p)$.", + "solution": "我们证明: 当 $k=2, \\cdots, p-1$ 时, 存在一个集 $\\left\\{b_{k, 1}, b_{k, 2}, \\cdots\\right.$, $\\left.b_{k, k}\\right\\}$, 其中 $b_{k, i}=1$ 或者某些 $a_1, \\cdots, a_{k-1}$ 中的数之积, 满足: (1) 对 $1 \\leqslant if\\left(a+2^k n\\right)$, 那么取 $d=2^k n$, 可知 (1) 成立.\n所以, 对 $k \\in \\mathbf{N}$, 都有 $f\\left(a+2^{k+1} n\\right)f(a+2 n)>\\cdots$, 但由于是满射知小于 $f(a+n)$ 的 $f$ 的值只有有限个, 矛盾.\n(2) 不一定存在.\n例如: 令 $f: \\mathbf{N}^* \\rightarrow \\mathbf{N}^*$ 如下\n$$\n\\begin{aligned}\n& n=1 ; 2 ; 3,4 ; 5,6,7,8 ; 9,10, \\cdots \\\\\n& f(n)=1 ; 2 ; 4,3 ; 8,7,6,5 ; 16,15, \\cdots\n\\end{aligned}\n$$\n上述定义中, 对 $n \\in \\mathbf{N}^*$, 有 $f\\left(2^n+1\\right)=2^{n+1}, f\\left(2^n+2\\right)=2^{n+1}-1, \\cdots$, $f\\left(2^{n+1}\\right)=2^n+1$, 而 $f(1)=1, f(2)=2$.\n下证: 在 $m \\geqslant 5$ 时,对 $a 、 d \\in \\mathbf{N}^*$, 都有 $f(a+(m-2) d)>f(a+-(m- 1) d)$ 或者 $f(a+(m-1) d)>f(a+m d)$.\n事实上, 若否, 则 $f(a+(m-2) d)1$, 则对任意 $j \\in \\mathbf{N}^*$, 有 $\\prod_{k=j}^{+\\infty}\\left(1+\\frac{1}{n_k}\\right) \\in\\left(1+\\frac{1}{n_j}, 1+\\frac{1}{n_j-1}\\right]$. 引理的证明: 由条件, 可知\n$$\n\\begin{aligned}\n\\prod_{k=j}^{+\\infty}\\left(1+\\frac{1}{n_k}\\right) & \\leqslant \\prod_{k=0}^{+\\infty}\\left(1+\\frac{1}{n_j^{2^k}}\\right)=\\prod_{k=0}^{+\\infty}\\left(1+\\left(\\frac{1}{n_j}\\right)^{2^k}\\right) \\\\\n& =\\sum_{k=0}^{+\\infty}\\left(\\frac{1}{n_j}\\right)^k=\\frac{1}{1-\\frac{1}{n_j}}=1+\\frac{1}{n_j-1} .\n\\end{aligned}\n$$\n所以,引理成立.\n由此引理可证得唯一性(事实上, 若 $\\alpha=\\prod_{k=1}^{+\\infty}\\left(1+\\frac{1}{n_k}\\right)=\\prod_{k=1}^{+\\infty}\\left(1+\\frac{1}{m_k}\\right)$, 而 $n_1=m_1, \\cdots, n_j=m_j$, 则 $\\alpha / \\prod_{k=1}^j\\left(1+\\frac{1}{n_k}\\right)=\\alpha / \\prod_{k=1}^j\\left(1+\\frac{1}{m_k}\\right)$, 前者由引理知 $\\in \\left(1+\\frac{1}{n_{j+1}}, 1+\\frac{1}{n_{j+1}-1}\\right]$, 后者 $\\in\\left(1+\\frac{1}{m_{j+1}}, 1+\\frac{1}{m_{j+1}-1}\\right]$, 这可得出 $n_{j+1}= \\left.m_{j+1}\\right)$.\n存在性可由下面的方式得到, 记 $\\alpha_1=\\alpha \\in(1,2]$, 则存在唯一的 $n_1 \\in \\mathbf{N}^*$, 使得 $\\alpha_1 \\in\\left(1+\\frac{1}{n_1}, 1+\\frac{1}{n_1-1}\\right]$, 写 $\\alpha_2=\\frac{\\alpha_1}{1+\\frac{1}{n_1}}$, 则 $1<\\alpha_2<\\alpha_1 \\leqslant 2$, 对此 $\\alpha_2$, 存在唯一的 $n_2$, 使 $\\alpha_2 \\in\\left(1+\\frac{1}{n_2}, 1+\\frac{1}{n_2-1}\\right]$, 依次递推, 可定义数列 $\\left\\{n_k\\right\\}_{k=1}^{+\\infty}$. 下证 : $n_k^2 \\leqslant n_{k+1}$.\n事实上 $1+\\frac{1}{n_{k+1}}<\\alpha_{k+1}=\\frac{\\alpha_k}{1+\\frac{1}{n_k}} \\leqslant \\frac{1+\\frac{1}{n_k-1}}{1+\\frac{1}{n_k}}=1+\\frac{1}{n_k^2-1}$, 故 $n_k^2 \\leqslant n_{k+1}$. 最后, 由 $n_k$ 的定义可知 $1<\\frac{\\alpha}{\\prod_{k=1}^N\\left(1+\\frac{1}{n_k}\\right)}=\\prod_{k=N}^{+\\infty}\\left(1+\\frac{1}{n_k}\\right) \\leqslant 1+\\frac{1}{n_{N+1}-1}$. 令 $N \\rightarrow+\\infty$, 即可得 $\\alpha=\\prod_{k=1}^{+\\infty}\\left(1+\\frac{1}{n_k}\\right)$.", + "remark": "", + "figures": [] +} \ No newline at end of file diff --git a/processed_dataset/proof/1644.json b/processed_dataset/proof/1644.json new file mode 100644 index 0000000000000000000000000000000000000000..7609a269a4bc1628b07fc51fc6b46d1de5b41af2 --- /dev/null +++ b/processed_dataset/proof/1644.json @@ -0,0 +1,8 @@ +{ + "source_file": "./raw_volume-zh/volume6/exercise2.tex", + "problem_type": "proof", + "problem": "问题28. 设 $m$ 为给定的正整数, 数列 $\\left\\{a_n\\right\\}$ 的每一项都是正整数, 且对任意正整数 $n$, 都有 $0s_2, s_1, s_2 \\in A$ (当 $s_1, s_2 \\in B$ 时类似讨论), 分三种情形讨论.\n情形一:若 $s_1$ 右起第 $m+1$ 位为 1 , 则 $s_2$ 右起第 $m+1$ 位必为 0 . 考察这两个数右起的第 1 至第 $m$ 位, 其中 $s_1$ 有奇数个 1 , 而 $s_2$ 有偶数个 1 , 令 $s_1^{\\prime}=s_2+2^m$, $s_2^{\\prime}=s_1-2^m$, 那么 $s_1^{\\prime}$ 与 $s_2^{\\prime}$ 都有奇数个 1 , 并且 $s_1^{\\prime}>s_2^{\\prime}, s_1^{\\prime}+s_2^{\\prime}=n, s_1^{\\prime}, s_2^{\\prime} \\in B$. 反过来, 当 $s_1 、 s_2 \\in B$ 时, 亦有 $s_1^{\\prime} 、 s_2^{\\prime} \\in A$. 故这部分两个集合中的表示方法数相同.\n情形二: 若 $s_1 、 s_2$ 右起第 $m+1$ 位都为 0 , 而右起第 $m$ 位都为 1 , 同上讨论, 可知 $s_1^{\\prime}=s_1-2^{m-1} \\in B, s_2^{\\prime}=s_2-2^{m-1} \\in B$. 故 $\\left(s_1^{\\prime}, s_2^{\\prime}\\right)$ 构成 $B$ 中对 $n-2^m$ 的一个表示.\n反过来, 当 $s_1 、 s_2 \\in B$ 时, $\\left(s_1^{\\prime}, s_2^{\\prime}\\right)$ 构成 $A$ 中对 $n-2^m$ 的一个表示.\n利用 $r_A\\left(n-2^m\\right)=r_B\\left(n-2^m\\right)$ (归纳假设), 可知这部分两个集合中的表示方法数亦相同.\n情形三: 若 $s_1, s_2$ 右起第 $m+1$ 位都为 0 , 右起第 $m$ 位不全为 1 , 这时 $s_1$ 右起第 $m$ 位为 1 , 而 $s_2$ 右起第 $m$ 位为 0 . 此时考察两数右起第 1 位至 $m-1$ 位中 1 的个数, $s_1$ 中有奇数个, $s_2$ 中有偶数个.\n令 $s_1^{\\prime}=s_2+2^{m-1}, s_2^{\\prime}=s_1-2^{m-1}$. 同情形一可知,这部分两个集合中的表示方法数相同.\n综上可知, 对 $m+1$ 位数 $n$,亦有 $r_A(n)=r_B(n)$. 所以, $A 、 B$ 符合.", + "remark": "", + "figures": [] +} \ No newline at end of file diff --git a/processed_dataset/proof/1646.json b/processed_dataset/proof/1646.json new file mode 100644 index 0000000000000000000000000000000000000000..f3f64e8504e4005ce9feb466fe767422b57108cc --- /dev/null +++ b/processed_dataset/proof/1646.json @@ -0,0 +1,8 @@ +{ + "source_file": "./raw_volume-zh/volume6/exercise2.tex", + "problem_type": "proof", + "problem": "问题30. 证明: 任何一个大于 1 的整数都可以表示为符合下述条件的有限个正整数的和的形式:\n(1) 每个加项的素因数都是 2 或 3 ;\n(2) 任意两个加项中没有一个是另一个的倍数.", + "solution": "设能表示的数构成的集合为 $S$, 令 $T=S \\cup\\{1\\}$, 并记 $S$ 中 3 的幕次不超过 $h$ 的元素构成的集合为 $S_h, T_h=S_h \\cup\\{1\\}$, 则下面的结论显然成立.\n(1) $2 T \\subseteq T, 3 T \\subseteq T$. 这里 $x T=\\{x t \\mid t \\in T\\}$.\n(2) 若 $h\\frac{n^2}{16}$.", + "solution": "对 $n \\in \\mathbf{N}^*$, 设 $a_n$ 是二进制表示中仅在偶数位上出现数码 1 或仅在奇数位上出现数码 1 的正整数从小到大的排列中的第 $n$ 项.\n我们证明: 此数列 $\\left\\{a_n\\right\\}$ 符合条件.\n利用正整数的二进制表示可知 (1) 成立, 只需证明 (2)亦成立.\n考虑所有小于 $2^{2 r}$ 的非负整数, 它们在二进制表示下都为 $2 r$ 位数 (不足位的前面补上 0 ), 其中偶数位都为零的数有 $2^r$ 个, 奇数位都为零的数有 $2^r$ 个, 只有 0 在两类数中同时出现, 因此, 数列 $\\left\\{a_n\\right\\}$ 中恰有 $2^{r+1}-1$ 个数小于 $2^{2 r}$, 故 $a_{2^{r+1}-1}=2^{2 r}$.\n对 $n \\in \\mathbf{N}^*$, 设 $2^{r+1}-1 \\leqslant n<2^{r+2}-1, r \\in \\mathbf{N}$, 则由 $\\left\\{a_n\\right\\}$ 的定义知 $a_n \\geqslant a_{2^{r+1}-1}=2^{2 r}=\\frac{1}{16} \\times 2^{2(r+2)}>\\frac{n^2}{16}$.\n所以, 存在满足的数列 $\\left\\{a_n\\right\\}$.", + "remark": "", + "figures": [] +} \ No newline at end of file diff --git a/processed_dataset/proof/1649.json b/processed_dataset/proof/1649.json new file mode 100644 index 0000000000000000000000000000000000000000..627cb5b4cc1d9c7204871a5740b4a1c401ea955b --- /dev/null +++ b/processed_dataset/proof/1649.json @@ -0,0 +1,11 @@ +{ + "source_file": "./raw_volume-zh/volume7/chapter1.tex", + "problem_type": "proof", + "problem": "例1. 如图(), 设点 $P$ 在 $\\triangle A B C$ 的外接圆上, 直线 $C P$ 和 $A B$ 相交于点 $E$, 直线 $B P$ 和 $A C$ 相交于点 $F$, 边 $A C$ 的垂直平分线交边 $A B$ 于点 $J$, 边 $A B$ 的垂直平分线交边 $A C$ 于点 $K$, 求证:\n$$\n\\frac{C E^2}{B F^2}=\\frac{A J \\cdot J E}{A K \\cdot K F} \\text {. }\n$$", + "solution": "证明:如图(), 连结 $B K, C J$.\n$$\n\\angle E=\\angle A B P-\\angle B P E,\n$$\n而由 $A, B, P, C$ 四点共圆, 知 $\\angle B P E=\\angle A$, 故 $\\angle E=\\angle A B P-\\angle A$, 又由 $K A=K B$, 知 $\\angle A= \\angle A B K$, 故\n$$\n\\angle E=\\angle A B P-\\angle A B K=\\angle K B F . \\label{eq1}\n$$\n同理 $\\angle F=\\angle J C E, \\label{eq2}$.\n由式\\ref{eq1}, \\ref{eq2}得 $\\triangle J E C \\backsim \\triangle K B F$.\n由此,\n$$\n\\begin{aligned}\n& \\frac{C E}{B F}=\\frac{J E}{K B}=\\frac{J E}{A K}, \\label{eq3}\\\\\n& \\frac{C E}{B F}=\\frac{J C}{K F}=\\frac{A J}{K F} . \\label{eq4}\n\\end{aligned}\n$$\n将式\\ref{eq3}, \\ref{eq4}两式的左端和右端分别相乘即得结论.", + "remark": "", + "figures": [ + "./images/volume7/figures/fig-c1i2.png", + "./images/volume7/figures/fig-c1i2.png" + ] +} \ No newline at end of file diff --git a/processed_dataset/proof/1650.json b/processed_dataset/proof/1650.json new file mode 100644 index 0000000000000000000000000000000000000000..21514e782bc18b0189072c14fb6e587dbcfbb30d --- /dev/null +++ b/processed_dataset/proof/1650.json @@ -0,0 +1,10 @@ +{ + "source_file": "./raw_volume-zh/volume7/chapter1.tex", + "problem_type": "proof", + "problem": "例2 $\\triangle P Q R$ 和 $\\triangle P^{\\prime} Q^{\\prime} R$ 是两个全等的等边三角形,六边形 $A B C D E F$ 的边长分别记为 $A B=a_1, B C=b_1, C D=a_2, D E=b_2, E F=a_3, F A=b_3$. 求证: $a_1^2+a_2^2+a_3^2=b_1^2+b_2^2+b_3^2$.", + "solution": "证明:如图(), 因为 $\\angle P=\\angle Q=\\angle R= \\angle P^{\\prime}=\\angle Q^{\\prime}=\\angle R^{\\prime}=60^{\\circ}$, 再根据各组对顶角相等知\n$\\triangle P A B \\backsim \\triangle Q^{\\prime} C B \\backsim \\triangle Q C D \\backsim \\triangle R^{\\prime} E D \\backsim \\triangle R E F \\backsim \\triangle P^{\\prime} A F$.\n依次设上述六个三角形面积为: $S_1, S_1^{\\prime}, S_2, S_2^{\\prime}$, $S_3, S_3^{\\prime}$, 则有\n$$\n\\frac{S_1}{a_1^2}=\\frac{S_1^{\\prime}}{b_1^2}=\\frac{S_2}{a_2^2}=\\frac{S_2^{\\prime}}{b_2^2}=\\frac{S_3}{a_3^2}=\\frac{S_3^{\\prime}}{b_3^2} .\n$$\n设其比值为 $k$, 则由 $S_1+S_2+S_3=S_1^{\\prime}+S_2^{\\prime}+S_3^{\\prime}$ 得\n$$\nk\\left(a_1^2+a_2^2+a_3^2\\right)=k\\left(b_1^2+b_2^2+b_3^2\\right) \\text {, 即 } a_1^2+a_2^2+a_3^2=b_1^2+b_2^2+b_3^2 \\text {. }\n$$", + "remark": "", + "figures": [ + "./images/volume7/figures/fig-c1i3.png" + ] +} \ No newline at end of file diff --git a/processed_dataset/proof/1651.json b/processed_dataset/proof/1651.json new file mode 100644 index 0000000000000000000000000000000000000000..46c0dfcfcef61b7c7b19fa5ff2075a76be938eb1 --- /dev/null +++ b/processed_dataset/proof/1651.json @@ -0,0 +1,11 @@ +{ + "source_file": "./raw_volume-zh/volume7/chapter1.tex", + "problem_type": "proof", + "problem": "例3. 如图(), 圆 $\\Gamma_1 、 \\Gamma_2$ 内切于点 $S$, 圆 $\\Gamma_2$ 的弦 $A B$ 与圆 $\\Gamma_1$ 切于点 $C, M$ 是弧 $A B$ (不含点 $S$ ) 的中点, 过点 $M$ 作 $M N \\perp A B$, 垂足为 $N$, 记圆 $\\Gamma_1$ 的半径为 $r$.\n求证: $A C \\cdot C B=2 r \\cdot M N$.", + "solution": "证明:如图作出圆 $\\Gamma_1$ 的直径 $C D$.\n因 $S$ 是两圆 $\\Gamma_1 、 \\Gamma_2$ 的切点, 即位似中心, 而 $C 、 M$ 为两圆上的位似对应点,故 $S 、 C 、 M$ 三点共线.\n由相交弦定理得 $A C \\cdot C B=S C \\cdot C M$.\n又由 Rt $\\triangle S C D \\circlearrowleft \\mathrm{Rt} \\triangle N M C$, 得 $S C \\cdot C M=C D$ • $M N=2 r \\cdot M N$.", + "remark": "注:此题本身并不难,但利用 $S 、 C 、 M$ 共线这个命题, 并结合圆的 Pascal 定理可以证明如下结论:\n设三角形 $A B C$ 的外接圆为圆 $O_1$, 另有一圆 $O$ 同时与边 $A B$, 边 $A C$, 弧 $\\overparen{B C}$ 相切于点 $D 、 E 、 F$, 则 $D E$ 中点 $I$ 为三角形 $A B C$ 内心.\n(如图())", + "figures": [ + "./images/volume7/figures/fig-c1i4.png", + "./images/volume7/figures/fig-c1i5.png" + ] +} \ No newline at end of file diff --git a/processed_dataset/proof/1652.json b/processed_dataset/proof/1652.json new file mode 100644 index 0000000000000000000000000000000000000000..941fc74f60a5d89ca5e84840946950ab57434fbe --- /dev/null +++ b/processed_dataset/proof/1652.json @@ -0,0 +1,10 @@ +{ + "source_file": "./raw_volume-zh/volume7/chapter1.tex", + "problem_type": "proof", + "problem": "例4. 凸五边形 $A B C D E$ 满足 $\\angle B A C=\\angle C A D= \\angle D A E, \\angle A B C=\\angle A C D==\\angle A D E, P$ 是 $B D$ 和 $C E$的交点.\n求证: $A P$ 平分线段 $C D$.", + "solution": "证明:如图(), 由条件知 $\\triangle A B C \\backsim \\triangle A C D \\backsim \\triangle A D E$, 所以四边形 $A B C D \\sim$ 四边形 $A C D E$.\n设 $A C \\cap B D=X, A D \\cap C E=Y$, 可知\n$$\nA X: C X=A Y: D Y .\n$$\n设 $A P \\cap C D=M$, 由 Ceva 定理知\n$$\n\\frac{C M}{D M}=\\frac{C X}{D Y} \\cdot A Y=1 \n$$\n所以 $M$ 为 $C D$ 中点,故 $A P$ 平分线段 $C D$, 证毕.", + "remark": "", + "figures": [ + "./images/volume7/figures/fig-c1i6.png" + ] +} \ No newline at end of file diff --git a/processed_dataset/proof/1653.json b/processed_dataset/proof/1653.json new file mode 100644 index 0000000000000000000000000000000000000000..137b5cf96a16db529c7f19680dc75e7314e0ba00 --- /dev/null +++ b/processed_dataset/proof/1653.json @@ -0,0 +1,10 @@ +{ + "source_file": "./raw_volume-zh/volume7/chapter1.tex", + "problem_type": "proof", + "problem": "例5. 已知凸四边形 $A B C D$ 满足 $A B=B C, A D=D C$. $E$ 是线段 $A B$ 上一点, $F$ 是线段 $A D$ 上一点, 满足 $B 、 E 、 F 、 D$ 四点共圆.\n作 $\\triangle D P E$ 顺向相似于 $\\triangle A D C$; 作 $\\triangle B Q F$ 顺向相似于 $\\triangle A B C$. 求证: $A 、 P 、 Q$ 三点共线.\n(注: 两个三角形顺向相似是指它们的对应顶点同按顺时针方向或同按逆时针方向排列.)", + "solution": "证明:如图(), 将 $B 、 E 、 F 、 D$ 四点所共圆的圆心记作 $O$, 连结 $O B 、 O E 、 O F 、 O D 、 B D$.\n在 $\\triangle B D F$ 中, $O$ 是外心, 故 $\\angle B O F=2 \\angle B D A$.\n又 $\\triangle A B D \\cong \\triangle C B D, \\angle C D A=2 \\angle B D A$.\n于是 $\\angle B O F=\\angle C D A=\\angle E P D$,\n由此可知 $\\triangle B O F \\backsim \\triangle E P D, \\label{eq1}$.\n另一方面, 由 $B 、 E 、 F 、 D$ 四点共圆知\n$\\triangle A B F \\backsim \\triangle A D E, \\label{eq2}$.\n综合 式\\ref{eq1}, \\ref{eq2} 可知, 四边形 $A B O F \\backsim$ 四边形\n$A D P E$, 由此得\n$$\n\\angle B A O=\\angle D A P . \\label{eq3}\n$$\n同理,可得\n$$\n\\angle B A O=\\angle D A Q . \\label{eq4}\n$$\n式\\ref{eq3},\\ref{eq4}表明 $A 、 P 、 Q$ 三点共线.", + "remark": "", + "figures": [ + "./images/volume7/figures/fig-c1i7.png" + ] +} \ No newline at end of file diff --git a/processed_dataset/proof/1654.json b/processed_dataset/proof/1654.json new file mode 100644 index 0000000000000000000000000000000000000000..5569fba340afede45b42e7a342c08e676000ce82 --- /dev/null +++ b/processed_dataset/proof/1654.json @@ -0,0 +1,10 @@ +{ + "source_file": "./raw_volume-zh/volume7/chapter1.tex", + "problem_type": "proof", + "problem": "例6. 设凸四边形 $A B C D$ 对角线交于 $O$ 点.\n$\\triangle O A D 、 \\triangle O B C$ 的外接圆交于 $O 、 M$ 两点,直线 $O M$ 分别交 $\\triangle O A B 、 \\triangle O C D$ 的外接圆于 $T 、 S$ 两点.\n求证: $M$ 是线段 $T S$ 的中点.", + "solution": "证明:如图(), 连结 $B T, C S, M A, M B$, $M C, M D$.\n由 $\\angle B T O=\\angle B A O, \\angle B C O=\\angle B M O$, 故 $\\triangle B T M \\backsim \\triangle B A C$, 得\n$$\n\\frac{T M}{A C}=\\frac{B M}{B C} . \\label{eq1}\n$$\n同理, $\\triangle C M S \\backsim \\triangle C B D$, 得\n$$\n\\frac{M S}{B D}=\\frac{C M}{B C} . \\label{eq2}\n$$\n式\\ref{eq1} $\\div$ 式\\ref{eq2} 得 \n$$\n\\frac{T M}{M S}=\\frac{B M}{C M} \\cdot \\frac{A C}{B D} . \\label{eq3}\n$$\n又 $\\angle M B D=\\angle M C A, \\angle M D B=\\angle M A C$.\n故 $\\triangle M B D \\backsim \\triangle M C A$, 得\n$$\n\\frac{B M}{C M}=\\frac{B D}{A C}, \\label{eq4}\n$$\n将式\\ref{eq4}代入\\ref{eq3}, 即得 $T M=M S$.", + "remark": "", + "figures": [ + "./images/volume7/figures/fig-c1i8.png" + ] +} \ No newline at end of file diff --git a/processed_dataset/proof/1655.json b/processed_dataset/proof/1655.json new file mode 100644 index 0000000000000000000000000000000000000000..de5f804efec60cc2eb07376973f9120848bc7dac --- /dev/null +++ b/processed_dataset/proof/1655.json @@ -0,0 +1,10 @@ +{ + "source_file": "./raw_volume-zh/volume7/chapter1.tex", + "problem_type": "proof", + "problem": "例7. 如图(), 设 $D$ 是锐角 $\\triangle A B C$ 的边 $B C$ 上一点, 以线段 $B D$ 为直径的圆分别交直线 $A B$ 、 $A D$ 于点 $X 、 P$ (异于点 $B 、 D$ ), 以线段 $C D$ 为直径的圆分别交直线 $A C 、 A D$ 于点 $Y 、 Q$ (异于点 $C 、 D$ ). 过点 $A$ 作直线 $P X 、 Q Y$ 的垂线, 垂足分别为 $M 、 N$. 求证: $\\triangle A M N$ 相似 $\\triangle A B C$ 的充分必要条件是直线 $A D$ 过 $\\triangle A B C$ 的外心.", + "solution": "证明:由已知有 $B 、 P 、 D 、 X$ 及 $C 、 Y 、 Q 、 D$ 分别四点共圆.\n故 $\\angle A X M=\\angle B X P=\\angle B D P=\\angle Q D C=\\angle A Y N$.\n所以 Rt $\\triangle A M X \\backsim$ Rt $\\triangle A N Y$.\n于是 $\\angle M A X=\\angle N A Y, \\frac{A M}{A X}=\\frac{A N}{A Y}$.\n从而 $\\angle M A N=\\angle X A Y$, 结合 $\\frac{A M}{A X}=\\frac{A N}{A Y}$, 得 $\\triangle A M N \\backsim \\triangle A X Y$.\n故 $\\triangle A M N \\backsim \\triangle A B C \\Leftrightarrow \\triangle A X Y \\backsim \\triangle A B C \\Leftrightarrow X Y / / B C \\Leftrightarrow \\angle D X Y=\\angle X D B$.\n而由 $A 、 X 、 D 、 Y$ 四点共圆知 $\\angle D X Y= \\angle D A Y$.\n又 $\\angle X D B=90^{\\circ}-\\angle A B C$, 则\n$\\angle D X Y=\\angle X D B \\Leftrightarrow \\angle D A C=90^{\\circ}-\\angle A B C$.\n$\\Leftrightarrow$ 直线 $A D$ 过 $\\triangle A B C$ 的外心.", + "remark": "", + "figures": [ + "./images/volume7/figures/fig-c1i9.png" + ] +} \ No newline at end of file diff --git a/processed_dataset/proof/1656.json b/processed_dataset/proof/1656.json new file mode 100644 index 0000000000000000000000000000000000000000..192871b68b4aa1a0748bfceeead786fada64370f --- /dev/null +++ b/processed_dataset/proof/1656.json @@ -0,0 +1,10 @@ +{ + "source_file": "./raw_volume-zh/volume7/chapter1.tex", + "problem_type": "proof", + "problem": "例8. 已知 $\\triangle A B C$ 中, $O$ 是三角形内一点满足: $\\angle B A O=\\angle C A O=\\angle C B O=\\angle A C O$. 求证: $\\triangle A B C$ 三边长成等比数列.", + "solution": "证明:如图(), 过 $O$ 作 $A C$ 平行线交 $B C$, $A B$ 于 $D, E$, 设 $\\angle A O E=\\angle 1, \\angle C O D=\\angle 2$.\n则 $\\angle O A C=\\angle 1=\\angle B A O$, 而 $\\angle O A C=\\angle O C A$,\n所以 $A O=O C, A E=O E$, 且 $\\triangle A O E \\backsim \\triangle A C O$, 于是\n$$\n\\frac{A C}{A O}=\\frac{O C}{O E} . \\label{eq1}\n$$\n又因 $D E / / A C$, 所以\n$$\n\\frac{A B}{C B}=\\frac{A E}{C D}, \\label{eq2}\n$$\n再注意到 $\\angle 2=\\angle O B C, \\angle B C O=\\angle B C O$, 所以 $\\triangle O C D \\backsim \\triangle B C D$,\n$$\n\\frac{O C}{B C}=\\frac{C D}{O C} . \\label{eq3}\n$$\n式\\ref{eq1} $\\times$ 式\\ref{eq2} $\\times$ 式\\ref{eq3}得\n$$\n\\frac{A C}{A O} \\cdot \\frac{A B}{B C} \\cdot \\frac{O C}{B C}=\\frac{O C}{O E} \\cdot \\frac{A E}{C D} \\cdot \\frac{C D}{O C},\n$$\n即 $\\frac{A C \\cdot A B}{B C^2}=1(A O=O C, A E=O E), B C^2=A C \\cdot A B$.\n所以 $\\triangle A B C$ 三边成等比数列.", + "remark": "", + "figures": [ + "./images/volume7/figures/fig-c1i11.png" + ] +} \ No newline at end of file diff --git a/processed_dataset/proof/1657.json b/processed_dataset/proof/1657.json new file mode 100644 index 0000000000000000000000000000000000000000..33d29608fd0a753ed55089464d1151c9bcc0670f --- /dev/null +++ b/processed_dataset/proof/1657.json @@ -0,0 +1,11 @@ +{ + "source_file": "./raw_volume-zh/volume7/chapter1.tex", + "problem_type": "proof", + "problem": "例10. 如图(), 在三角形 $A B C$ 的内部有四个半径相等的 $\\odot K_1$, $\\odot K_2, \\odot K_3, \\odot K_4$, 其中 $\\odot K_1, \\odot K_2, \\odot K_3$ 均与三角形 $A B C$ 的两边相切, 且与 $\\odot K_4$ 外切.\n证明: 三角形 $A B C$ 的内心 、外心和 $K_4$ 在一条直线上.", + "solution": "证明:如图(), 设三角形的内心为 $I$, 外心为 $O$, 连结 $A I 、 B I 、 C I 、 K_1 K_2 、 K_1 K_3 、 K_3 K_2$ 、 $K_1 K_4 、 K_4 K_3 、 K_4 K_2$.\n因为三角形的三边与 $\\odot K_1, \\odot K_2, \\odot K_3$ 相切, 所以 $K_1$ 在 $A I$ 上, $K_2$ 在 $B I$ 上, $K_3$ 在 $C I$ 上.\n设圆的半径为 $r$, 注意到 $A B$ 是圆 $K_1$ 和圆 $K_2$ 的公切线, 且圆 $K_1$ 和圆 $K_2$ 是等圆, 所以 $K_1$ 和 $K_2$ 到 $A B$ 的距离都是 $r$.\n故 $K_1 K_2 / / A B$, 同理, $K_2 K_3 / / B C, K_1 K_3 / / A C$.\n所以\n$$\n\\frac{I K_1}{I A}=\\frac{I K_2}{I B}=\\frac{I K_3}{I C} .\n$$\n故三角形 $A B C$ 与三角形 $K_1 K_2 K_3$ 关于 $I$ 位似.\n因为 $K_1 K_4=K_4 K_3=K_4 K_2=2 r$, 所以 $K_4$ 是三角形 $K_1 K_2 K_3$ 的外心, 又 $O$ 是三角形 $A B C$ 的外心, 所以 $I 、 K_4 、 O$ 在一条直线上.", + "remark": "", + "figures": [ + "./images/volume7/figures/fig-c1i13.png", + "./images/volume7/figures/fig-c1i13.png" + ] +} \ No newline at end of file diff --git a/processed_dataset/proof/1658.json b/processed_dataset/proof/1658.json new file mode 100644 index 0000000000000000000000000000000000000000..eaf537b32e688b0ffc6a66675b0723d8fc8518a7 --- /dev/null +++ b/processed_dataset/proof/1658.json @@ -0,0 +1,10 @@ +{ + "source_file": "./raw_volume-zh/volume7/chapter1.tex", + "problem_type": "proof", + "problem": "例11. 求证: Euler 公式, 即 $O I^2=R^2-2 R r$. 其中 $R, r$ 分别为 $\\triangle A B C$ 的外接圆和内切圆半径.", + "solution": "证明:如图(), 延长 $A I$ 交 $\\overparen{B C}$ 于 $D$, 作 $D$ 的对径点 $E$, 作 $I F \\perp A B$ 于 $F$, 连结 $E C 、 D C 、 E D$, 则有 $\\angle E D C=\\frac{\\pi-A}{2}=\\angle A I F$, 于是 $\\triangle E D C \\backsim \\triangle A I F$.\n不难证明 $\\angle I C D=\\angle C I D=\\frac{A+C}{2}$, 即 $D I= D C$, 由 $\\triangle E D C \\backsim \\triangle A I F$, 知\n$$\n2 \\mathrm{R} r=I F \\cdot E D=A I \\cdot C D=A I \\cdot D I=R^2-O I^2 .\n$$\n最后一步用到了圆幂定理.\n有关圆幂定理读者可参看第 5 章.", + "remark": "", + "figures": [ + "./images/volume7/figures/fig-c1i14.png" + ] +} \ No newline at end of file diff --git a/processed_dataset/proof/1659.json b/processed_dataset/proof/1659.json new file mode 100644 index 0000000000000000000000000000000000000000..0773814efaf3c814b22815f40a9d651ad9a8d939 --- /dev/null +++ b/processed_dataset/proof/1659.json @@ -0,0 +1,10 @@ +{ + "source_file": "./raw_volume-zh/volume7/chapter1.tex", + "problem_type": "proof", + "problem": "例12. 求证: 圆外切四边形的圆心位于两个对角线中点的连线上.\n(牛顿定理)", + "solution": "证明:如图(), 设四边形 $A B C D$ 内切圆圆心为 $O, A C$ 中点为 $M$,\n$B D$ 中点为 $N$, 设 $A B$ 延长线和 $D C$ 延长线交于点 $E$, 过 $O$ 作与 $O E$ 垂直的 $X Y$ 交 $A B$ 于 $X$, 交 $C D$ 于 $Y$, 注意到 $\\angle A O D=\\angle A X Y=\\angle D Y X=90^{\\circ}+\\frac{1}{2} \\angle A E D, \\angle O A D= \\angle O A X, \\angle O D A=\\angle O D Y, \\triangle A O D$ \\& $\\triangle A X O \\backsim \\triangle O Y D$, 从而 $\\angle O A X=\\angle D O Y, \\angle A O X=\\angle O D Y$.\n即 $\\triangle O A X \\backsim \\triangle D O Y$, 于是\n$$\nA X \\cdot D Y=O X \\cdot O Y,\n$$\n同理可证,\n$$\nB X \\cdot C Y=O X \\cdot O Y .\n$$\n于是, $A X B \\backsim C Y D$ (这里的相似是两个线段间的相似, $X$ 分 $A B$ 的比等于 $Y$ 分 $C D$ 的比), 注意到两相似图形的对应顶点连线中点构成的图形与原来两个图形相似,则有 $M O N$ 构成线段, 且有 $\\frac{M O}{O N}=\\frac{A X}{X B}=\\frac{C Y}{Y D}$.", + "remark": "注:这是一个非常困难的问题, 在想到上述解答之前, 笔者始终没能找到简单的解答.\n如果线段的相似超出读者的理解, 也可以用解析几何中的定比分点公式来刻画这些点的位置.", + "figures": [ + "./images/volume7/figures/fig-c1i15.png" + ] +} \ No newline at end of file diff --git a/processed_dataset/proof/1660.json b/processed_dataset/proof/1660.json new file mode 100644 index 0000000000000000000000000000000000000000..ca6f4b412b72beadb1d065b513b6c30d5b627f05 --- /dev/null +++ b/processed_dataset/proof/1660.json @@ -0,0 +1,10 @@ +{ + "source_file": "./raw_volume-zh/volume7/chapter1.tex", + "problem_type": "proof", + "problem": "例13. 设四边形 $A B C D$ 内接于圆 $O, A B$ 延长线与 $D C$ 延长线交于 $E$,\n$A D$ 延长线与 $B C$ 延长线交于 $F, A C$ 中点为 $M, B D$ 中点为 $N$.\n求证: $\\frac{M N}{A B}=\\frac{1}{2}\\left(\\frac{A C}{B D}-\\frac{B D}{A C}\\right)$.", + "solution": "证明:如图(), 首先延长 $M N$ 交 $E F$ 于 $P, M N P$ 为牛顿线 $(A C 、 B D 、 E F$ 中点共线, 称为牛顿线), 于是 $P$ 为 $E F$ 中点, 下面证明\n$$\n2 \\cdot \\frac{M P}{E F}=\\frac{A C}{B D} \\text {. }\n$$\n取 $E C$ 中点 $Q$, 由于 $\\angle M Q P=\\angle M Q D+\\angle D Q P=\\angle A E D+\\angle D C F=\\angle A E D+\\angle E A D= \\angle E D F, \\frac{A E}{D E}=\\frac{\\sin \\angle A D E}{\\sin \\angle E A D}=\\frac{\\sin \\angle E D F}{\\sin \\angle D C F}=\\frac{C F}{D F}$, 从而 $\\frac{A E}{C F}=\\frac{D E}{D F}, \\frac{M Q}{Q P}=\\frac{D E}{D F}$, 故 $\\triangle M Q P \\backsim \\triangle E D F$.\n于是 $M P=\\frac{E F \\cdot M Q}{E D}=\\frac{E F \\cdot A E}{2 \\cdot E D}$, 即 $2 \\cdot \\frac{M P}{E F}=\\frac{A C}{B D}$.\n同理, $2 \\cdot \\frac{N P}{E F}=\\frac{B D}{A C}$, 两式相减即得原命题.", + "remark": "", + "figures": [ + "./images/volume7/figures/fig-c1i16.png" + ] +} \ No newline at end of file diff --git a/processed_dataset/proof/1661.json b/processed_dataset/proof/1661.json new file mode 100644 index 0000000000000000000000000000000000000000..f454aefb6a8163257bc77a790782c147783ed5cc --- /dev/null +++ b/processed_dataset/proof/1661.json @@ -0,0 +1,10 @@ +{ + "source_file": "./raw_volume-zh/volume7/chapter10.tex", + "problem_type": "proof", + "problem": "例1. 设一条平面闭折线周长为 1 . 证明: 可以用一个半径为 $\\frac{1}{4}$ 的圆完全盖住这条折线.", + "solution": "证明:如图(), 令 $A B$ 平分折线周长, $O$ 为线段 $A B$ 的中点,任取折线上一点 $M$, 则 $M A+M B$ 不超过 $A$ 与 $B$ 之间的折线总长 $\\frac{1}{2}$, 故 $O M \\leqslant \\frac{1}{2}(M A+M B) \\leqslant \\frac{1}{4}$.\n于是以 $O$ 为圆心 $\\frac{1}{4}$ 为半径的圆可以盖住这条折线.", + "remark": "", + "figures": [ + "./images/volume7/figures/fig-c10i1.png" + ] +} \ No newline at end of file diff --git a/processed_dataset/proof/1662.json b/processed_dataset/proof/1662.json new file mode 100644 index 0000000000000000000000000000000000000000..50899c99b276835e9fe4861fce7d96134a344edc --- /dev/null +++ b/processed_dataset/proof/1662.json @@ -0,0 +1,10 @@ +{ + "source_file": "./raw_volume-zh/volume7/chapter10.tex", + "problem_type": "proof", + "problem": "例2. 设 $\\triangle A B C$ 的三边分别为 $a 、 b 、 c$, 三边上的中线长分别为 $m_a 、 m_b 、 m_c$, 求证: $m_a\\left(b c-a^2\\right)+ m_b\\left(c a-b^2\\right)+m_c\\left(a b-c^2\\right) \\geqslant 0$.", + "solution": "证明:如图(), 设 $\\triangle A B C$ 的三条中线分别为 $A D 、 B E 、 C F$, 重心为 $G$, 对四边形 $B D G F$ 应用托勒密不等式可得\n$$\nB G \\cdot D F \\leqslant G F \\cdot D B+D G \\cdot B F .\n$$\n即 $2 b m_b \\leqslant a m_c+c m_a$, 故 $2 b^2 m_b \\leqslant a b m_c+b c m_a$ 等等.\n三式相加即得 $m_a\\left(b c-a^2\\right)+m_b\\left(c a-b^2\\right)+m_c\\left(a b-c^2\\right) \\geqslant 0$.", + "remark": "", + "figures": [ + "./images/volume7/figures/fig-c10i2.png" + ] +} \ No newline at end of file diff --git a/processed_dataset/proof/1663.json b/processed_dataset/proof/1663.json new file mode 100644 index 0000000000000000000000000000000000000000..4c2ed18768793c4212cfc25042fbee6c22fa8416 --- /dev/null +++ b/processed_dataset/proof/1663.json @@ -0,0 +1,10 @@ +{ + "source_file": "./raw_volume-zh/volume7/chapter10.tex", + "problem_type": "proof", + "problem": "例3. 设 $P$ 为平行四边形 $A B C D$ 内一点, 求证: $P A \\cdot P C+P B \\cdot P D \\geqslant A B \\cdot B C$, 并指出等号成立条件.", + "solution": "证明:如图(), 取点 $P^{\\prime}$ 使得 $\\overrightarrow{P P^{\\prime}}=\\overrightarrow{A B}$,\n于是原命题等价于 $P^{\\prime} P \\cdot B C \\leqslant P C \\cdot P^{\\prime} B+P B \\cdot P^{\\prime} C$, 即四边形 $P B P^{\\prime} C$ 的托勒密不等式, 等号成立的充要条件是 $P B P^{\\prime} C$ 为圆内接四边形, 即 $\\angle A P D+\\angle C P B=\\pi$.", + "remark": "", + "figures": [ + "./images/volume7/figures/fig-c10i3.png" + ] +} \ No newline at end of file diff --git a/processed_dataset/proof/1664.json b/processed_dataset/proof/1664.json new file mode 100644 index 0000000000000000000000000000000000000000..03c2109c7703322b6b1117ea3922dca0d8ff2158 --- /dev/null +++ b/processed_dataset/proof/1664.json @@ -0,0 +1,10 @@ +{ + "source_file": "./raw_volume-zh/volume7/chapter10.tex", + "problem_type": "proof", + "problem": "例4. 设 $O$ 为 $\\triangle A B C$ 内一点, 且 $\\angle A O B=\\angle B O C=\\angle C O A=120^{\\circ}, P$ 为任意一点 (不是 $O$ ), 求证:\n$$\nP A+P B+P C>O A+O B+O C .\n$$", + "solution": "证明:如图(), 过 $\\triangle A B C$ 的顶点 $A, B, C$ 分别引 $O A, O B, O C$ 的垂线.\n设这三条垂线的交点为 $A_1, B_1, C_1$, 考虑四边形 $A O B C_1$. 因为 $\\angle O A C_1=\\angle O B C_1=90^{\\circ}, \\angle A O B= 120^{\\circ}$, 所以 $\\angle C_1=60^{\\circ}$. 同理, $\\angle A_1=\\angle B_1=60^{\\circ}$, 所以 $\\triangle A_1 B_1 C_1$ 为正三角形.\n设 $P$ 到 $\\triangle A_1 B_1 C_1$ 三边 $B_1 C_1 、 C_1 A_1 、 A_1 B_1$ 的距离分别为 $h_a 、 h_b 、 h_c$, 且 $\\triangle A_1 B_1 C_1$ 的边长为 $a$, 高为$h$. 由等式 $S_{\\triangle A_1 B_1 C_1}=S_{\\triangle P B_1 C_1}+S_{\\triangle P C_1 A_1}+S_{\\triangle P A_1 B_1}$ 知 $\\frac{1}{2} h a=\\frac{1}{2} h_a \\cdot a+\\frac{1}{2} h_b \\cdot a+\\frac{1}{2} h_c \\cdot a$, 所以 $h=h_a+ h_b+h_c$.\n这说明正三角形 $A_1 B_1 C_1$ 内任一点 $P$ 到三边的距离和等于 $\\triangle A_1 B_1 C_1$ 的高 $h$, 这是一个定值, 所以 $O A+O B+O C=h=$ 定值.\n显然, $P A+P B+ P C>P$ 到 $\\triangle A_1 B_1 C_1$ 三边距离和, 所以 $P A+P B+P C>h=O A+O B+ O C$. 这就是我们所要证的结论.\n由这个结论可知 $O$ 点具有如下性质: 它到三角形三个顶点的距离和小于其他点到三角形顶点的距离和, 这个点叫费马点.", + "remark": "注:当 $\\triangle A B C$ 的三个角 $\\angle A 、 \\angle B 、 \\angle C$ 都小于 $120^{\\circ}$ 时, 在它的内部一定存在一点 $O$, 使得 $\\angle A O B=\\angle B O C=\\angle C O A=120^{\\circ}$. 当 $\\angle A 、 \\angle B 、 \\angle C$ 中有一个 $\\geqslant 120^{\\circ}$ 时, 不妨设 $\\angle A \\geqslant 120^{\\circ}$, 则对于任意一点 $P$ 都有 $P A+P B+P C \\geqslant A B+$ AC.", + "figures": [ + "./images/volume7/figures/fig-c10i4.png" + ] +} \ No newline at end of file diff --git a/processed_dataset/proof/1665.json b/processed_dataset/proof/1665.json new file mode 100644 index 0000000000000000000000000000000000000000..a491a3896fd0eae0fc2ea02a88f7b5a898572b75 --- /dev/null +++ b/processed_dataset/proof/1665.json @@ -0,0 +1,10 @@ +{ + "source_file": "./raw_volume-zh/volume7/chapter10.tex", + "problem_type": "proof", + "problem": "例5. 已知四边形 $A B C D$ 是圆的内接四边形.\n证明: $|A B-C D|+|A D-B C| \\geqslant 2|A C-B D|$.", + "solution": "证明:如图(), 设四边形 $A B C D$ 的外心为 $O$, 且圆 $O$ 的半径为 1 .\n设 $\\angle A O B=2 \\alpha, \\angle B O C=2 \\beta, \\angle C O D=2 \\gamma, \\angle D O A=2 \\delta$, 则 $\\alpha+\\beta+\\gamma+\\delta=\\pi$.\n不妨设 $\\alpha \\geqslant \\gamma, \\beta \\geqslant \\delta$, 则\n$$\n|A B-C D|=4\\left|\\sin \\frac{\\alpha-\\gamma}{2} \\sin \\frac{\\beta+\\delta}{2}\\right| \\text {. }\n$$\n同理,\n$$\n\\begin{aligned}\n& |A D-B C|=4\\left|\\sin \\frac{\\alpha+\\gamma}{2} \\sin \\frac{\\beta-\\delta}{2}\\right|, \\\\\n& |A C-B D|=4\\left|\\sin \\frac{\\alpha-\\gamma}{2} \\sin \\frac{\\beta-\\delta}{2}\\right|,\n\\end{aligned}\n$$\n则\n$$\n|A B-C D|-|A C-B D|=8\\left|\\sin \\frac{\\alpha-\\gamma}{2}\\right| \\cdot \\cos \\frac{\\beta}{2} \\sin \\frac{\\delta}{2} \\geqslant 0 .\n$$\n即 $|A B-C D| \\geqslant|A C-B D|$. 同理可证 $|A D-B C| \\geqslant.|A C-B D|$.\n所以 $|A B-C D|+|A D-B C| \\geqslant 2|A C-B D|$.", + "remark": "", + "figures": [ + "./images/volume7/figures/fig-c10i5.png" + ] +} \ No newline at end of file diff --git a/processed_dataset/proof/1666.json b/processed_dataset/proof/1666.json new file mode 100644 index 0000000000000000000000000000000000000000..5303669062864c0f01c6277f2f1162facf8fc437 --- /dev/null +++ b/processed_dataset/proof/1666.json @@ -0,0 +1,10 @@ +{ + "source_file": "./raw_volume-zh/volume7/chapter10.tex", + "problem_type": "proof", + "problem": "例6. 设 $P 、 Q 、 R$ 分别位于 $\\triangle A B C$ 的三条边 $B C 、 C A 、 A B$ 上,且将三角形周长三等分, 求证: $Q R+R P+P Q \\geqslant \\frac{1}{2}(a+b+c) . a 、 b 、 c$ 表示三角形三边长.", + "solution": "证明:如图(), 分别作 $R 、 Q$ 在底边 $B C$ 上的投影 $M 、 N$, 则 $Q R \\geqslant M N=a-(B R \\cdot \\cos B+ C Q \\cdot \\cos C)$, 同理有, $R P \\geqslant b-(C P \\cdot \\cos C+A R \\cdot \\cos A), P Q \\geqslant c-(A Q \\cdot \\cos A+B P \\cdot \\cos B)$.\n将三式相加, 并注意到 $A Q+A R=B R+B P= C P+C Q=\\frac{1}{3}(a+b+c)$, 即得\n$$\nQ R+R P+P Q \\geqslant \\frac{1}{3}(a+b+c)(3-\\cos A-\\cos B-\\cos C) \\geqslant \\frac{1}{2}(a+\n$$\n$b+c$ ). (这步用到 $\\cos A+\\cos B+\\cos C \\leqslant \\frac{3}{2}$.)", + "remark": "注:$\\cos A+\\cos B+\\cos C \\leqslant \\frac{3}{2}$ 的证明:\n$$\n\\begin{aligned}\n\\cos A+\\cos B+\\cos C & =2 \\cos \\frac{A+B}{2} \\cos \\frac{A-B}{2}-\\cos (A+B) \\\\\n& =2 \\cos \\frac{A+B}{2} \\cos \\frac{A-B}{2}-\\left(2 \\cos ^2 \\frac{A+B}{2}-1\\right)\n\\end{aligned}\n$$\n$$\n\\begin{aligned}\n& \\leqslant 2 \\cos \\frac{A+B}{2}-2 \\cos ^2 \\frac{A+B}{2}+1 \\\\\n& =-2\\left(\\cos \\frac{A+B}{2}-\\frac{1}{2}\\right)^2+\\frac{3}{2} \\\\\n& \\leqslant \\frac{3}{2}\n\\end{aligned}\n$$\n等号成立当且仅当 $\\triangle A B C$ 为等边三角形.", + "figures": [ + "./images/volume7/figures/fig-c10i6.png" + ] +} \ No newline at end of file diff --git a/processed_dataset/proof/1667.json b/processed_dataset/proof/1667.json new file mode 100644 index 0000000000000000000000000000000000000000..b42b3aa064af074e2fa91da890a9fa57f0a2ce94 --- /dev/null +++ b/processed_dataset/proof/1667.json @@ -0,0 +1,10 @@ +{ + "source_file": "./raw_volume-zh/volume7/chapter10.tex", + "problem_type": "proof", + "problem": "例7. (Erdös-Mordell 不等式) 设 $P$ 为三角形 $A B C$ 内任意一点, $P$ 到三边 $B C 、 C A 、 A B$ 的距离分别为 $P D=p, P E=q, P F=r$, 并记 $P A=x$, $P B=y, P C=z$, 证明: $x+y+z \\geqslant 2(p+q+r)$, 等号成立当且仅当 $\\triangle A B C$ 为正三角形并且 $P$ 为此三角形的中心.", + "solution": "证明:如图(), 过点 $P$ 作直线 $M N$, 使得 $\\angle A M N=\\angle A C B$, 于是 $\\triangle A M N \\backsim \\triangle A C B$, 从而 $\\frac{A N}{M N}=\\frac{c}{a}, \\frac{A M}{M N}=\\frac{b}{a}$.\n由于 $S_{\\triangle A M N}=S_{\\triangle A M P}+S_{\\triangle A N P}$, 所以有\n$$\nA P \\cdot M N \\geqslant q \\cdot A N+r \\cdot A M,\n$$\n所以 $x=A P \\geqslant q \\cdot \\frac{A N}{M N}+r \\cdot \\frac{A M}{M N}$.\n即 $x \\geqslant \\frac{c}{a} q+\\frac{b}{a} r$, 等等.\n于是 $x+y+z \\geqslant p\\left(\\frac{c}{b}+\\frac{b}{c}\\right)+q\\left(\\frac{c}{a}+\\frac{a}{c}\\right)+r\\left(\\frac{b}{a}+\\frac{a}{b}\\right) \\geqslant 2(p+q+r)$.\n第一个等号成立的条件是 $A P \\perp M N$, 即 $\\angle P A C=90^{\\circ}-\\angle B$, 以及 $\\angle P B A=90^{\\circ}-\\angle C, \\angle P C B=90^{\\circ}-\\angle A$.\n第二个等号成立的条件是 $a=b=c$, 所以 $x+y+z \\geqslant 2(p+q+r)$ 的等号成立条件是 $\\triangle A B C$ 为正三角形,且 $P$ 为其中心.", + "remark": "", + "figures": [ + "./images/volume7/figures/fig-c10i7.png" + ] +} \ No newline at end of file diff --git a/processed_dataset/proof/1668.json b/processed_dataset/proof/1668.json new file mode 100644 index 0000000000000000000000000000000000000000..f91e800b724abc41cbf3fd0f5701980bccf241c4 --- /dev/null +++ b/processed_dataset/proof/1668.json @@ -0,0 +1,8 @@ +{ + "source_file": "./raw_volume-zh/volume7/chapter10.tex", + "problem_type": "proof", + "problem": "例8. Neuberg-Pedoe 不等式: 设 $a_1 、 a_2 、 a_3 、 b_1 、 b_2 、 b_3$ 分别是位于同一平面上的两个三角形 $\\triangle A_1 B_1 C_1$ 和 $\\triangle A_2 B_2 C_2$ 的各边长, $F 、 F^{\\prime}$ 分别是它们的面积, 记\n$$\nM=b_1^2\\left(-a_1^2+a_2^2+a_3^2\\right)+b_2^2\\left(a_1^2-a_2^2+a_3^2\\right)+b_3^2\\left(a_1^2+a_2^2-a_3^2\\right) .\n$$\n求证: $M \\geqslant 16 F F^{\\prime}$.", + "solution": "证明:由柯西不等式, $16 F F^{\\prime}+2\\left(a_1^2 b_1^2+a_2^2 b_2^2+a_3^2 b_3^2\\right) \\leqslant\\left(\\left(16 F^2+2 a_1^4+\\right.\\right. \\left.\\left.2 a_2^4+2 a_3^4\\right)\\left(16 F^{\\prime 2}+2 b_1^4+2 b_2^4+2 b_3^4\\right)\\right)^{\\frac{1}{2}}=\\left(a_1^4+a_2^4+a_3^4+2 a_1^2 a_2^2+2 a_2^2 a_3^2+2 a_3^2 a_1^2\\right)^{\\frac{1}{2}} \\cdot\\left(b_1^4+b_2^4+b_3^4+2 b_1^2 b_2^2+2 b_2^2 b_3^2+2 b_3^2 b_1^2\\right)^{\\frac{1}{2}}$ (这步用到海伦公式) $=\\left(a_1^2+a_2^2+a_3^2\\right)\\left(b_1^2+b_2^2+b_3^2\\right)$.\n即 $M \\geqslant 16 F F^{\\prime}$.", + "remark": "注:海伦公式是指三角形的面积 $S=\\sqrt{p(p-a)(p-b)(p-c)}= \\frac{1}{4} \\sqrt{2 a^2 b^2+2 b^2 c^2+2 c^2 a^2-a^4-b^4-c^4}$, 其中 $p=\\frac{a+b+c}{2}$.", + "figures": [] +} \ No newline at end of file diff --git a/processed_dataset/proof/1669.json b/processed_dataset/proof/1669.json new file mode 100644 index 0000000000000000000000000000000000000000..f808ac61f141fd4b0bd54822300ec79c55b2c740 --- /dev/null +++ b/processed_dataset/proof/1669.json @@ -0,0 +1,10 @@ +{ + "source_file": "./raw_volume-zh/volume7/chapter10.tex", + "problem_type": "proof", + "problem": "例9. 设 $M$ 为 $\\triangle A B C$ 所在平面上一点, $H 、 O 、 R$ 分别为 $\\triangle A B C$ 的垂心、 外心、外接圆半径.\n求证: $S=\\min \\left(M A^3+M B^3+M C^3-\\frac{3}{2} R \\cdot M H^2\\right)$.", + "solution": "证明:$S=3 R^3-\\frac{3}{2} R \\cdot O H^2$.\n如图(),一方面, 当 $M=O$ 时等号成立.\n另一方面, 由均值不等式有\n$$\n\\frac{M A^3}{R}+\\frac{R^2+M A^2}{2} \\geqslant \\frac{M A^3}{R}+R \\cdot M A \\geqslant 2 M A^2 .\n$$\n所以 $\\frac{M A^3}{R} \\geqslant \\frac{3}{2} M A^2-\\frac{R^2}{2}$.\n类似三式相加得 $\\frac{1}{R}\\left(M A^3+M B^3+M C^3\\right) \\geqslant \\frac{3}{2}\\left(M A^2+M B^2+M C^2\\right)- \\frac{3}{2} R^2$. \\label{eq1}\n由 Leibniz 公式 (见注(1)), 设三角形的重心为 $G$, 有\n$$\n\\begin{aligned}\n& A M^2+B M^2+C M^2=3 M G^2+\\frac{1}{3}\\left(B C^2+C A^2+A B^2\\right), \\\\\n& A O^2+B O^2+C O^2=3 O G^2+\\frac{1}{3}\\left(B C^2+C A^2+A B^2\\right) .\n\\end{aligned}\n$$\n两式相减得\n$$\nA M^2+B M^2+C M^2=3 R^2+3 M G^2-3 O G^2,\n$$\n又由 Stewart 定理知\n$$\n\\begin{aligned}\n3 R^2+3 M G^2-3 O G^2 & =2 M O^2+3 R^2+M H^2-O H^2 (\\text { 见注 (2)} )\\\\\n& \\geqslant 3 R^2+M H^2-O H^2 .\n\\end{aligned}\n$$\n代入式\\ref{eq1}即得 $M A^3+M B^3+M C^3-\\frac{3}{2} R \\cdot M H^2 \\geqslant 3 R^3-\\frac{3}{2} R \\cdot O H^2$, 得证.", + "remark": "注(1) Leibniz 公式是指: 对于 $\\triangle A B C$ 所在平面上任意一点 $M$, 设三角形 $A B C$ 的重心为 $G$, 则\n$$\n\\begin{aligned}\nA M^2+B M^2+C M^2 & =3 M G^2+\\frac{1}{3}\\left(B C^2+C A^2+A B^2\\right) \\\\\n& =3 M G^2+A G^2+B G^2+C G^2,\n\\end{aligned}\n$$\n(可以用解析法或 Stewart 定理证明).\n(2) 由 Stewart 定理 (见第一章知识点) 知, $M G^2= \\frac{2 O G \\cdot M O^2+O G \\cdot M H^2}{3 O G}-2 O G^2=\\frac{2}{3} M O^2+\\frac{1}{3} M H^2-2 O G^2$, 于是 $3 M G^2- 3 O G^2=2 M O^2+M H^2-9 O G^2=2 M O^2+M H^2-O H^2$.\n(3) 此类最值问题一般思路是: 先找出最值点, 算出 (猜测)最值, 再用不等式、几何关系证明您的结论.", + "figures": [ + "./images/volume7/figures/fig-c10i8.png" + ] +} \ No newline at end of file diff --git a/processed_dataset/proof/1670.json b/processed_dataset/proof/1670.json new file mode 100644 index 0000000000000000000000000000000000000000..fe5a078eb9e3e6a4244d85a8aed306c7fc9eb52d --- /dev/null +++ b/processed_dataset/proof/1670.json @@ -0,0 +1,10 @@ +{ + "source_file": "./raw_volume-zh/volume7/chapter10.tex", + "problem_type": "proof", + "problem": "例10. 求证: 四条边给定的四边形中, 内接于圆的四边形面积最大.", + "solution": "证明:先提出一个引理: 如图(),设凸四边形 $A B C D$ 的边长为 $a 、 b 、 c 、 d$, 对角和为 $2 \\varphi$ (任一组), 设四边形的面积为 $S_0$, 则 $S_0^2=(s-a)(s-b)(s-c)(s-d)- a b c d \\cos ^2 \\varphi$. 其中 $2 s=a+b+c+d$.\n引理证明: 由余弦定理, $B D^2=a^2+d^2-2 a d \\cos A= b^2+c^2-2 b c \\cos C$, 所以 $\\frac{a^2+d^2-b^2-c^2}{2}=a d \\cdot \\cos A- b c \\cdot \\cos C$.\n又 $2 S_0=a d \\sin A+b c \\sin C$, 上面两式各自平方后相加得\n$$\nS_0^2=(s-a)(s-b)(s-c)(s-d)-a b c d \\cos ^2 \\varphi .\n$$\n回到原题, 对于凹四边形 $A B C D$, 不妨设 $D$ 在三角形 $A B C$ 内, 则作 $D$ 关于 $B C$ 的反射点 $D^{\\prime}$, 四边形 $A B C D^{\\prime}$ 与 $A B C D$ 四边各自相同, 但后者面积更大.\n对于凸四边形 $A B C D, S_0 \\leqslant(s-a)(s-b)(s-c)(s-d)$ 等号成立时当且仅当 $\\cos \\varphi=0$, 即它为圆内接四边形.\n综上,对于给定四边长为 $a 、 b 、 c 、 d$ 的四边形, 当且仅当它为圆内接四边形时, 它有最大面积 $\\sqrt{(s-a)(s-b)(s-c)(s-d)}$, 其中 $S= \\frac{a+b+c+d}{2}$.", + "remark": "", + "figures": [ + "./images/volume7/figures/fig-c10i9.png" + ] +} \ No newline at end of file diff --git a/processed_dataset/proof/1671.json b/processed_dataset/proof/1671.json new file mode 100644 index 0000000000000000000000000000000000000000..2882b945cbdcf2baeb9306a7e9d528b9cde34c6c --- /dev/null +++ b/processed_dataset/proof/1671.json @@ -0,0 +1,10 @@ +{ + "source_file": "./raw_volume-zh/volume7/chapter11.tex", + "problem_type": "proof", + "problem": "例1. $ \\triangle A B C$ 中, $A H \\perp B C$, 分别以 $A C 、 A B$ 为直径作两个圆, $D$ 是 $B C$ 上一点,过 $D$ 分别作 $A B 、 A C$ 的平行线, 在圆的上方交于 $F 、 E$. 求证: $D 、 H 、 F 、 E$ 四点共圆.", + "solution": "证明:如图(), 延长 $F A$ 交以 $A B$ 为直径的圆于 $E^{\\prime}$, 连结 $D E^{\\prime} 、 H E^{\\prime}$, 则 $\\angle H D F=\\angle H B A= \\angle H E^{\\prime} A=\\angle H E^{\\prime} F$, 于是 $H 、 D 、 E^{\\prime} 、 F$ 四点共圆, 故 $\\angle E^{\\prime} D B=\\angle E^{\\prime} F H=\\angle A F H=\\angle A C H= \\angle A C B=\\angle E D B$, 故 $E^{\\prime} 、 E$ 重合.\n从而 $D, H, F, E$ 四点共圆.", + "remark": "注:: 本题若不用反证法则很难证明,读者不妨一试.", + "figures": [ + "./images/volume7/figures/fig-c11i1.png" + ] +} \ No newline at end of file diff --git a/processed_dataset/proof/1672.json b/processed_dataset/proof/1672.json new file mode 100644 index 0000000000000000000000000000000000000000..383273c61eb779d5c9d567280dd5b5718b89a6ca --- /dev/null +++ b/processed_dataset/proof/1672.json @@ -0,0 +1,10 @@ +{ + "source_file": "./raw_volume-zh/volume7/chapter11.tex", + "problem_type": "proof", + "problem": "例2. 如图(), 设 $P$ 为 $\\triangle A B C$ 的一个内点, $P A 、 P B 、 P C$ 分别交边 $B C 、 C A 、 A B$ 于 $D 、 E 、 F$. 证明 $S_{\\triangle P A F}+S_{\\triangle P B D}+S_{\\triangle P C E}=\\frac{1}{2} S_{\\triangle A B C}$ 成立当且仅当 $P$ 至少位于 $\\triangle A B C$ 的一条中线上.", + "solution": "证明:设 $a=\\frac{A F}{F B}, b=\\frac{B D}{D C}, c=\\frac{C E}{E A}$, 则由塞瓦定理(对 $\\triangle A B C$ 和 $D 、 E 、 F$ ) 得\n$$\na b c=1 \\Rightarrow c=\\frac{1}{a b} \\text {. }\n$$\n由梅氏定理(对 $\\triangle A B D$ 和 $F C$ 使用)\n$$\n\\frac{A F}{F B} \\frac{B C}{C D} \\frac{D P}{P A}=1 \\text {. }\n$$\n所以 $\\frac{A P}{P D}=\\frac{A F}{F B} \\frac{B C}{C D}=a(b+1)=a+a b$.\n则 $\\frac{A P}{A D}=\\frac{A P}{A P+P D}=\\frac{a+a b}{1+a+a b}$.\n从而 $\\frac{S_{\\triangle A F P}}{S_{\\triangle A B C}}=\\frac{A F}{A B} \\frac{A P}{A D} \\frac{B D}{B C}=\\frac{a}{a+1} \\frac{a+a b}{1+a+a b} \\frac{b}{b+1}$\n$$\n=\\frac{a b(a+a b)}{(1+a)(1+b)(1+a+a b)} \\text {. }\n$$\n同理可求出 $\\frac{S_{\\triangle P B D}}{S_{\\triangle A B C}}$ 及 $\\frac{S_{\\triangle P C E}}{S_{\\triangle A B C}}$.\n故 $S_{\\triangle P A F}+S_{\\triangle P B D}+S_{\\triangle P C E}=\\frac{1}{2} S_{\\triangle A B C} \\Leftrightarrow \\frac{S_{\\triangle A F P}}{S_{\\triangle A B C}}+\\frac{S_{\\triangle P B D}}{S_{\\triangle A B C}}+\\frac{S_{\\triangle P C E}}{S_{\\triangle A B C}}=\\frac{1}{2}$\n$$\n\\begin{aligned}\n\\Leftrightarrow & \\frac{a b(a+a b)}{(1+a)(1+b)(1+a+a b)}+\\frac{b c(b+b c)}{(1+b)(1+c)(1+b+b c)}+ \\\\\n& \\frac{c a(c+c a)}{(1+c)(1+a)(1+c+c a)}=\\frac{1}{2} \\\\\n\\Leftrightarrow & \\frac{a b(a+a b)}{(1+a)(1+b)(1+a+a b)}+\\frac{b(1+a b)}{(1+b)(1+a b)(1+a+a b)}+ \\\\\n& \\frac{a(1+a)}{(1+a)(1+a b)(1+a+a b)}=\\frac{1}{2} \\text { (将 (1) 代入) } \\\\\n\\Leftrightarrow & a^3 b^3-a^2 b^3-a^3 b+a^2+b-1=0 \\text { (展开, 实际上去分母不是太困难) } \\\\\n\\Leftrightarrow & (a-1)(b-1)(a b-1)(a b+a+1)=0 \\\\\n\\Leftrightarrow & (1-a)(1-b)(1-c)(a b+a+1)=0\n\\end{aligned}\n$$\n$\\Leftrightarrow a^3 b^3-a^2 b^3-a^3 b+a^2+b-1=0$ (展开, 实际上去分母不是太困难)\n$\\Leftrightarrow(a-1)(b-1)(a b-1)(a b+a+1)=0\\Leftrightarrow(1-a)(1-b)(1-c)(a b+a+1)=0\\Leftrightarrow a, b, c$ 中至少有一个为 1 (因为 $a b+a+1>0$ )\n$\\Leftrightarrow p$ 至少位于 $\\triangle A B C$ 的一条中线上, 证毕.", + "remark": "", + "figures": [ + "./images/volume7/figures/fig-c11i2.png" + ] +} \ No newline at end of file diff --git a/processed_dataset/proof/1673.json b/processed_dataset/proof/1673.json new file mode 100644 index 0000000000000000000000000000000000000000..944753a7f55c064b2a234b2f8f61ce0eabfd3ced --- /dev/null +++ b/processed_dataset/proof/1673.json @@ -0,0 +1,10 @@ +{ + "source_file": "./raw_volume-zh/volume7/chapter11.tex", + "problem_type": "proof", + "problem": "例3. 如图(), $D$ 是 $\\triangle A B C$ 内的一点, 满足 $\\angle D A C=\\angle D C A=30^{\\circ}, \\angle D B A=60^{\\circ}, E$ 是边 $B C$ 的中点, $F$ 是边 $A C$ 的三等分点, 满足 $A F=2 F C$. 求证: $D E \\perp E F$. (2007 第六届女子数学奥林匹克)", + "solution": "证明:建立复平面, 令 $B=0, D=1, A= -\\omega^2 k$. 这里 $\\omega=-\\frac{1}{2}+\\frac{\\sqrt{3}}{2} \\mathrm{i}, k \\in \\mathbf{R}$.\n经计算可得\n$$\nC=1-\\omega^2-\\omega k,\n$$\n$$\n\\begin{gathered}\nE=\\frac{B+C}{2}=\\frac{1-\\omega^2-\\omega k}{2}, \\\\\nF=\\frac{2 C+A}{3}=\\frac{2-2 \\omega^2-2 \\omega k-\\omega^2 k}{3} .\n\\end{gathered}\n$$\n于是,\n$$\n\\begin{gathered}\nE-1=-\\frac{1+\\omega^2+\\omega k}{2}, \\\\\nF-E=\\frac{1-\\omega^2-\\left(\\omega+2 \\omega^2\\right) k}{6} .\n\\end{gathered}\n$$\n故\n$$\n\\begin{aligned}\n\\frac{F-E}{E-1} & =\\frac{1}{3} \\cdot \\frac{\\omega^2-1+\\left(\\omega+2 \\omega^2\\right) k}{1+\\omega^2+\\omega k} \\\\\n& =\\frac{\\omega-\\omega^2}{3} \\cdot \\frac{k+1}{k-1}=\\frac{\\mathrm{i}}{\\sqrt{3}} \\cdot \\frac{k+1}{k-1} .\n\\end{aligned}\n$$\n因此, $D E \\perp E F$, 即 $\\angle D E F=90^{\\circ}$.", + "remark": "", + "figures": [ + "./images/volume7/figures/fig-c11i3.png" + ] +} \ No newline at end of file diff --git a/processed_dataset/proof/1674.json b/processed_dataset/proof/1674.json new file mode 100644 index 0000000000000000000000000000000000000000..916ea626b39f1a4b416b5dd1fa65aa45cccdeac9 --- /dev/null +++ b/processed_dataset/proof/1674.json @@ -0,0 +1,11 @@ +{ + "source_file": "./raw_volume-zh/volume7/chapter11.tex", + "problem_type": "proof", + "problem": "例4. 如图(), 在 $\\triangle A B C$ 的三边上向外作 $\\triangle B P C 、 \\triangle C Q A 、 \\triangle A R B$, 使 $\\angle P B C= \\angle C A Q=45^{\\circ}, \\angle B C P=\\angle Q C A=30^{\\circ}$, $\\angle A B R=\\angle R A B=15^{\\circ}$. 求证: $\\angle P R Q=90^{\\circ}$, $Q R=P R$.", + "solution": "证明:建立如图() 所示复平面, 只需证明 $z_Q=\\mathrm{i} \\cdot z_P$.\n设 $z_A=-1$, 则 $z_B=\\cos 30^{\\circ}+\\mathrm{i} \\sin 30^{\\circ}$.\n因为 $\\frac{B P}{B C}=\\frac{A Q}{A C}=\\frac{\\sin 30^{\\circ}}{\\sin 105^{\\circ}}=\\frac{\\sqrt{2}}{1+\\sqrt{3}}$, 所\n$$\n\\begin{aligned}\n& \\text { 以 } z_P=z_B+\\frac{\\sqrt{2}}{1+\\sqrt{3}}\\left(z_C-z_B\\right) \\cdot \\mathrm{e}^{-\\frac{\\pi}{4} \\mathrm{i}}, z_Q=z_A+\\frac{\\sqrt{2}}{1+\\sqrt{3}}\\left(z_C-z_A\\right) \\cdot \\mathrm{e}^{\\frac{\\pi}{4} \\mathrm{i}} \\cdot \\\\\n& \\begin{aligned}\nz_P \\cdot \\mathrm{i} & =\\left[z_B\\left(1-\\frac{\\sqrt{2}}{1+\\sqrt{3}} \\mathrm{e}^{-\\frac{\\pi}{4} \\mathrm{i}}\\right)+\\frac{\\sqrt{2}}{1+\\sqrt{3}} \\mathrm{e}^{-\\frac{\\pi}{4} \\mathrm{i}} \\cdot z_C\\right] \\cdot \\mathrm{i} \\\\\n& =\\mathrm{e}^{\\frac{2}{3} \\mathrm{ri}}\\left(1-\\frac{\\sqrt{2}}{1+\\sqrt{3}} \\mathrm{e}^{-\\frac{\\pi}{4} \\mathrm{i}}\\right)+\\frac{\\sqrt{2}}{1+\\sqrt{3}} \\mathrm{e}^{\\frac{\\pi}{4} \\mathrm{i}} \\cdot z_C \\\\\n& =-\\frac{1}{2}+\\frac{\\sqrt{3}}{2} \\mathrm{i}-\\frac{\\sqrt{2}}{1+\\sqrt{3}}\\left(\\frac{\\sqrt{6}-\\sqrt{2}}{4}+\\mathrm{i} \\cdot \\frac{\\sqrt{6}+\\sqrt{2}}{4}\\right)+\\frac{\\sqrt{2}}{1+\\sqrt{3}} \\mathrm{e}^{\\frac{\\pi}{4} \\mathrm{i}} \\cdot z_C \\\\\n& =\\frac{-3+\\sqrt{3}}{2}+\\frac{\\sqrt{3}-1}{2} \\mathrm{i}+\\frac{\\sqrt{2}}{1+\\sqrt{3}} \\cdot \\mathrm{e}^{\\frac{\\pi}{4} \\mathrm{i}} \\cdot z_C ; \\\\\n& z_Q=\\frac{\\sqrt{2}}{1+\\sqrt{3}}\\left(\\frac{\\sqrt{2}}{2}+\\frac{\\sqrt{2}}{2} \\mathrm{i}\\right)-1+\\frac{\\sqrt{2}}{1+\\sqrt{3}} \\mathrm{e}^{\\frac{\\pi}{4} \\mathrm{i}} \\cdot z_C\n\\end{aligned}\n\\end{aligned}\n$$\n$$\n=\\frac{-3+\\sqrt{3}}{2}+\\frac{\\sqrt{3}-1}{2} \\mathrm{i}+\\frac{\\sqrt{2}}{1+\\sqrt{3}} \\mathrm{e}^{\\frac{\\pi}{4} \\mathrm{i}} \\cdot z_C .\n$$\n所以 $z_Q=z_P \\cdot \\mathrm{i}$.\n故 $\\angle P R Q=90^{\\circ}, Q R=P R$.", + "remark": "", + "figures": [ + "./images/volume7/figures/fig-c11i4.png", + "./images/volume7/figures/fig-c11i4.png" + ] +} \ No newline at end of file diff --git a/processed_dataset/proof/1675.json b/processed_dataset/proof/1675.json new file mode 100644 index 0000000000000000000000000000000000000000..be50fc0f9ff89c38fd869d4b52cac23c06ea9e54 --- /dev/null +++ b/processed_dataset/proof/1675.json @@ -0,0 +1,8 @@ +{ + "source_file": "./raw_volume-zh/volume7/chapter11.tex", + "problem_type": "proof", + "problem": "例5. 设 $P_1, P_2, \\cdots, P_n$ 是圆内接正 $n$ 边形, $P$ 是圆周上的任一点, 求证: $P P_1^4 \\cdot-P P_2^4+\\cdots+P P_n^4$ 是常数.", + "solution": "证明:设圆心在原点, 圆的半径为 $r$.\n显然可令 $P_k=r \\mathrm{e}^{\\mathrm{i} \\cdot \\frac{2 k \\pi}{n}}(k=1,2, \\cdots, n), P=r \\mathrm{e}^{\\mathrm{i} \\theta}$, 于是\n$$\n\\begin{aligned}\n\\left|P P_k\\right|^4 & =\\left|P-P_k\\right|^4=\\left|r \\mathrm{e}^{\\mathrm{i} \\theta}-r \\mathrm{e}^{\\mathrm{i} \\cdot \\frac{2 k \\pi}{n}}\\right|^4 \\\\\n& =r^4\\left[\\left(\\mathrm{e}^{\\mathrm{i} \\theta}-\\mathrm{e}^{\\mathrm{i} \\cdot \\frac{2 k \\pi}{n}}\\right)\\left(\\mathrm{e}^{-\\mathrm{i} \\theta}-\\mathrm{e}^{-\\mathrm{i} \\cdot \\frac{2 k \\pi}{n}}\\right)\\right]^2\\left(\\text { 这里用到 }|z|^2=z \\cdot \\bar{z}\\right) \\\\\n& =r^4\\left[2-\\mathrm{e}^{\\mathrm{i} \\theta} \\mathrm{e}^{-\\mathrm{i} \\cdot \\frac{2 k \\pi}{n}}-\\mathrm{e}^{-\\mathrm{i} \\theta} \\mathrm{e}^{\\mathrm{i} \\cdot \\frac{2 k \\pi}{n}}\\right]^2 \\\\\n& =r^4\\left[6+\\mathrm{e}^{2 i \\theta} \\mathrm{e}^{-\\mathrm{i} \\cdot \\frac{4 k \\pi}{n}}+\\mathrm{e}^{-2 \\mathrm{i} \\theta} \\mathrm{e}^{\\mathrm{i} \\cdot \\frac{k k \\pi}{n}}-4 \\mathrm{e}^{\\mathrm{i} \\theta} \\mathrm{e}^{-\\mathrm{i} \\cdot \\frac{2 k \\pi}{n}}-4 \\mathrm{e}^{-\\mathrm{i} \\theta} \\mathrm{e}^{\\mathrm{i} \\cdot \\frac{2 k \\pi}{n}}\\right] .\n\\end{aligned}\n$$\n但是 $\\sum_{k=1}^n \\mathrm{e}^{ \\pm \\mathrm{i} \\cdot \\frac{4 k \\pi}{n}}=\\frac{\\mathrm{e}^{ \\pm \\mathrm{i} \\cdot \\frac{4 \\pi}{n}}\\left(1-\\mathrm{e}^{ \\pm \\mathrm{i} \\cdot \\frac{4 n \\pi}{n}}\\right)}{1-\\mathrm{e}^{\\mathrm{i} \\cdot \\frac{4 \\pi}{n}}}=0$,\n$$\n\\sum_{k=1}^n \\mathrm{e}^{ \\pm \\frac{2 k \\pi}{n}}=\\frac{\\mathrm{e}^{ \\pm \\mathrm{i} \\cdot \\frac{2 \\pi}{n}}\\left(1-\\mathrm{e}^{ \\pm \\mathrm{i} \\cdot \\frac{2 n \\pi}{n}}\\right)}{1-\\mathrm{e}^{ \\pm \\mathrm{i} \\cdot \\frac{2 \\pi}{n}}}=0,\n$$\n所以 $\\sum_{k=1}^n P P_k^4=6 n r^4$.\n它与点 $P$ 无关, 即它是一常数.", + "remark": "注:(1) 从证明中易看出, $P P_1^2+P P_2^2+\\cdots+P P_n^2$ 也是一常数, 为 $2 n r^2$.\n(2) $\\mathrm{e}^{\\mathrm{i} \\theta}=\\cos \\theta+\\mathrm{i} \\sin \\theta$ 是复数欧拉公式.", + "figures": [] +} \ No newline at end of file diff --git a/processed_dataset/proof/1676.json b/processed_dataset/proof/1676.json new file mode 100644 index 0000000000000000000000000000000000000000..c5ef329effecbc891749321d50127dc49f113933 --- /dev/null +++ b/processed_dataset/proof/1676.json @@ -0,0 +1,8 @@ +{ + "source_file": "./raw_volume-zh/volume7/chapter11.tex", + "problem_type": "proof", + "problem": "例6. 设 $P$ 是锐角三角形 $A B C$ 内一点, $A P 、 B P 、 C P$ 分别交边 $B C$ 、 $C A 、 A B$ 于点 $D 、 E 、 F$, 已知 $\\triangle D E F \\backsim \\triangle A B C$. 求证: $P$ 是 $\\triangle A B C$ 的重心.", + "solution": "证明:本题的结论对 $\\triangle A B C$ 为一般的三角形都成立.\n我们采用复数方法予以证明.\n设 $P$ 为复平面上的原点, 并直接用 $X$ 表示点 $X$ 对应的复数, 则存在正实数 $\\alpha, \\beta, \\gamma$, 使得 $\\alpha A+\\beta B+\\gamma C=0$, 且 $\\alpha+\\beta+\\gamma=1$.\n由于 $D$ 为 $A P$ 与 $B C$ 的交点, 可解得 $D=-\\frac{\\alpha}{1-\\alpha} A$, 同样地, $E= -\\frac{\\beta}{1-\\beta} B, F=-\\frac{\\gamma}{1-\\gamma} C$. 利用 $\\triangle D E F \\backsim \\triangle A B C$ 可知 $\\frac{D-E}{A-B}=\\frac{E-F}{B-C}$, 于是\n$$\n\\frac{\\gamma B C}{1-\\gamma}+\\frac{\\beta A B}{1-\\beta}+\\frac{\\alpha B C}{1-\\alpha}-\\frac{\\alpha A B}{1-\\alpha}-\\frac{\\beta B C}{1-\\beta}-\\frac{\\gamma C A}{1-\\gamma}=0 .\n$$\n化简得: $\\left(\\gamma^2-\\beta^2\\right) B(C-A)+\\left(\\alpha^2-\\gamma^2\\right) A(C-B)=0$. 这时, 若 $\\gamma^2 \\neq \\beta^2$, 则 $\\frac{B(C-A)}{A(C-B)} \\in \\mathbf{R}$, 因此, $\\frac{\\frac{C-A}{C-B}}{\\frac{P-A}{P-B}} \\in \\mathbf{R}$, 这要求 $P$ 在 $\\triangle A B C$ 的外接圆上, 与 $P$ 在 $\\triangle A B C$ 内矛盾, 所以 $\\gamma^2=\\beta^2$, 进而 $\\alpha^2=\\gamma^2$, 得 $\\alpha=\\beta=\\gamma=\\frac{1}{3}$. 即 $P$ 为 $\\triangle A B C$ 的重心.\n命题获证.", + "remark": "", + "figures": [] +} \ No newline at end of file diff --git a/processed_dataset/proof/1677.json b/processed_dataset/proof/1677.json new file mode 100644 index 0000000000000000000000000000000000000000..f7b33d57c12fcf62c644a0caa755ac1dedcd9404 --- /dev/null +++ b/processed_dataset/proof/1677.json @@ -0,0 +1,10 @@ +{ + "source_file": "./raw_volume-zh/volume7/chapter11.tex", + "problem_type": "proof", + "problem": "例7. 如图(), 已知圆内接四边形 $A B C D$ 的两条对角线的交点为 $S, S$ 在边 $A B$ 、 $C D$ 上的投影分别为点 $E 、 F$. 证明: $E F$ 的中垂线平分线段 $B C$ 和 $D A$.", + "solution": "证明:设 $A D$ 的中点为 $M$, 则 $2 \\overrightarrow{S M}=\\overrightarrow{S A}+\\overrightarrow{S D}$.\n由于 $\\overrightarrow{S E} \\perp \\overrightarrow{E A}, \\overrightarrow{E A}=\\overrightarrow{S A}-\\overrightarrow{S E}$, 所以,\n$$\n\\overrightarrow{S E} \\cdot(\\overrightarrow{S A}-\\overrightarrow{S E})=\\mathbf{0},\n$$\n即 $\\overrightarrow{S E} \\cdot \\overrightarrow{S A}-\\overrightarrow{S E} \\cdot \\overrightarrow{S E}=\\mathbf{0}$. 类似地, 可得 $\\overrightarrow{S F} \\cdot \\overrightarrow{S D}- \\overrightarrow{S F} \\cdot \\overrightarrow{S F}=\\mathbf{0}$.\n由于 $\\angle E A S=\\angle F D S, \\angle A E S=\\angle D F S=90^{\\circ}$, 所以 $\\triangle A S E \\backsim \\triangle D S F$.\n于是, $|\\overrightarrow{S A}| \\cdot|\\overrightarrow{S F}|=|\\overrightarrow{S D}| \\cdot|\\overrightarrow{S E}|$.\n又 $\\angle A S F=\\angle D S E$, 易得 $\\overrightarrow{S A} \\cdot \\overrightarrow{S F}=\\overrightarrow{S D} \\cdot \\overrightarrow{S E}$.\n故 $(\\overrightarrow{S M}-\\overrightarrow{S F})^2-(\\overrightarrow{S M}-\\overrightarrow{S E})^2=2 \\overrightarrow{S M} \\cdot \\overrightarrow{S E}-2 \\overrightarrow{S M} \\cdot \\overrightarrow{S F}-\\overrightarrow{S E} \\cdot \\overrightarrow{S E}+ \\overrightarrow{S F} \\cdot \\overrightarrow{S F}=(\\overrightarrow{S A}+\\overrightarrow{S D}) \\cdot \\overrightarrow{S E}-(\\overrightarrow{S A}+\\overrightarrow{S D}) \\cdot \\overrightarrow{S F}-\\overrightarrow{S E} \\cdot \\overrightarrow{S E}+\\overrightarrow{S F} \\cdot \\overrightarrow{S F}=\\overrightarrow{S A} \\cdot \\overrightarrow{S E}-\\overrightarrow{S E} \\cdot \\overrightarrow{S E}-(\\overrightarrow{S D} \\cdot \\overrightarrow{S F}-\\overrightarrow{S F} \\cdot \\overrightarrow{S F})-(\\overrightarrow{S A} \\cdot \\overrightarrow{S F}-\\overrightarrow{S D} \\cdot \\overrightarrow{S E})=0$.\n这就表明 $D A$ 的中点在 $E F$ 的中垂线上.\n同理, $B C$ 的中点也在 $E F$ 的中垂线上.\n故 $E F$ 的中垂线平分线段 $B C$ 和 $D A$.", + "remark": "", + "figures": [ + "./images/volume7/figures/fig-c11i5.png" + ] +} \ No newline at end of file diff --git a/processed_dataset/proof/1678.json b/processed_dataset/proof/1678.json new file mode 100644 index 0000000000000000000000000000000000000000..0fb344e0f6b8258b5b3a9d4a356d7059502593fd --- /dev/null +++ b/processed_dataset/proof/1678.json @@ -0,0 +1,10 @@ +{ + "source_file": "./raw_volume-zh/volume7/chapter11.tex", + "problem_type": "proof", + "problem": "例8. 如图(), 凸四边形 $A B C D$ 中, $A B$ 、 $D C$ 的延长线交于 $E, A D 、 B C$ 的延长线交于 $F$. $P 、 Q 、 R$ 依次为 $A C 、 B D 、 E F$ 的中点.\n求证: $P$ 、 $Q 、 R$ 三点共线.", + "solution": "证明:设 $\\overrightarrow{A B}=\\vec{a}, \\overrightarrow{A D}=\\vec{b}, \\overrightarrow{B E}=\\lambda \\vec{a}$, $\\overrightarrow{D F}=u \\vec{b}, \\overrightarrow{E C}=m \\overrightarrow{E D}, \\overrightarrow{F C}=n \\overrightarrow{F B}$\n有 $\\overrightarrow{A C}=\\overrightarrow{A E}+\\overrightarrow{E C}=\\overrightarrow{A E}+m(\\overrightarrow{A D}-\\overrightarrow{A E})=(1+\\lambda)(1-m) \\vec{a}+m \\vec{b}$.\n又 $\\overrightarrow{A C}=\\overrightarrow{A F}+\\overrightarrow{F C}=\\overrightarrow{A F}+n(\\overrightarrow{A B}-\\overrightarrow{A F})=(1+u)(1-n) \\vec{b}+n \\vec{a}$, 则有\n$$\n\\begin{gathered}\n\\left\\{\\begin{array}{l}\n(1+\\lambda)(1-m)=n, \\\\\n(1+u)(1-n)=m,\n\\end{array}\\right. \\\\\n\\left\\{\\begin{array}{l}\nm=\\frac{(1+u) \\lambda}{\\lambda+u+\\lambda u}, \\\\\nn=\\frac{(1+\\lambda) u}{\\lambda+u+\\lambda u} .\n\\end{array}\\right.\n\\end{gathered}\n$$\n又因为\n$$\n\\begin{aligned}\n\\overrightarrow{A R} & =\\frac{1}{2}(\\overrightarrow{A E}+\\overrightarrow{A F})=\\frac{1}{2}[(1+\\lambda) \\vec{a}+(1+u) \\vec{b}], \\\\\n\\overrightarrow{A Q} & =\\frac{1}{2}(\\overrightarrow{A B}+\\overrightarrow{A D})=\\frac{1}{2}(\\vec{a}+\\vec{b}), \\\\\n\\overrightarrow{A P}=\\frac{1}{2} \\overrightarrow{A C} & =\\frac{1}{2}\\left[\\frac{(1+\\lambda) u}{\\lambda+u+\\lambda u} \\cdot \\vec{a}+\\frac{(1+u) \\lambda}{\\lambda+u+\\lambda u} \\cdot \\vec{b}\\right] .\n\\end{aligned}\n$$\n所以\n$$\n\\overrightarrow{Q R}=\\overrightarrow{A R}-\\overrightarrow{A Q}=\\frac{1}{2}(\\lambda \\vec{a}+u \\vec{b}),\n$$\n$$\n\\begin{aligned}\n\\overrightarrow{P R}=\\overrightarrow{A R}-\\overrightarrow{A P} & =\\frac{1}{2} \\cdot \\frac{(1+\\lambda)(1+u)}{\\lambda+u+\\lambda u} \\cdot(\\lambda \\vec{a}+u \\vec{b}) . \\\\\n\\overrightarrow{P R} & =\\frac{(1+\\lambda)(1+u)}{\\lambda+u+\\lambda u} \\cdot \\overrightarrow{Q R} .\n\\end{aligned}\n$$\n有 $\\overrightarrow{P R} / / \\overrightarrow{Q R}$, 即 $P 、 Q 、 R$ 三点共线.", + "remark": "注:此题我们曾经在完全四边形中用梅氏定理的逆定理证明过.\n用向量解决平面几何问题, 首先是在图形中选出一对不平行的有向线段, 设为 $\\vec{a} 、 \\vec{b}$, 则平面内的其他有向线段均可用 $\\vec{a} 、 \\vec{b}$ 唯一表示, 即 $\\overrightarrow{A B}=p \\vec{a}+ q \\vec{b}$. 有序实数对 $(p, q)$ 可看成 $\\overrightarrow{A B}$ 的 \"坐标\", 这里近似于复数, 但它的优点在于直观性, $\\vec{a} 、 \\vec{b}$ 可以是不互相垂直, 同时起始点可以任意选定, 从而对于解决几何问题有着较大的自由度.", + "figures": [ + "./images/volume7/figures/fig-c11i6.png" + ] +} \ No newline at end of file diff --git a/processed_dataset/proof/1679.json b/processed_dataset/proof/1679.json new file mode 100644 index 0000000000000000000000000000000000000000..2a02a0ed4d254b7f236f9ecc81bbe23e4e4501eb --- /dev/null +++ b/processed_dataset/proof/1679.json @@ -0,0 +1,10 @@ +{ + "source_file": "./raw_volume-zh/volume7/chapter11.tex", + "problem_type": "proof", + "problem": "例9. 如图(), 梯形 $A B C D$ 中, $A B$ 平行于 $C D$, 作点 $F \\in A B$, 使 $C F=D F$. 设 $A C$ 与 $B D$ 相交于点 $E, O_1 、 O_2$ 分别为 $\\triangle A D F 、 \\triangle B C F$ 的外心.\n求证: $E F \\perp \\mathrm{O}_1 \\mathrm{O}_2$.", + "solution": "证明:取 $D C$ 中点为 $O$, 由 $C F==D F$, 所以 $O F \\perp D C$.\n以 $D C$ 为 $x$ 轴, $O F$ 为 $y$ 轴建立直角坐标系, 不妨设 $D(-1,0), C(1,0), F(0, b)$, $A(d, b), B(c, b)$.\n$1^{\\circ}$ 若 $c \\neq-d$, 则直线 $A C$ :\n$$\ny=\\frac{b}{d-1} x-\\frac{b}{d-1} . \\label{eq1}\n$$\n直线 $B D$ :\n$$\ny=\\frac{b}{c+1} x+\\frac{b}{c+1} . \\label{eq2}\n$$\n联立 式\\ref{eq1}、\\ref{eq2}知 $E\\left(\\frac{c+d}{c+2-d}, \\frac{2 b}{c+2-d}\\right)$, 所以\n$$\nk_{\\mathrm{EF}}=\\frac{\\frac{2 b}{c+2-d}-b}{\\frac{c+d}{c+2-d}}=\\frac{b d-b c}{c+d} . \\label{eq3}\n$$\n而直线 $A F$ 中垂线方程为 $x=\\frac{d}{2}$.\n直线 $D F$ 中垂线方程为\n$$\ny=-\\frac{1}{b}\\left(x+\\frac{1}{2}\\right)+\\frac{b}{2} .\n$$\n所以 $O_1\\left(\\frac{d}{2},-\\frac{1}{b}\\left(\\frac{d}{2}+\\frac{1}{2}\\right)+\\frac{b}{2}\\right)$.\n同理, $O_2\\left(\\frac{c}{2}, \\frac{1}{b}\\left(\\frac{c}{2}-\\frac{1}{2}\\right)+\\frac{b}{2}\\right)$.\n所以\n$$\nK_{O_1 O_2}=\\frac{\\frac{1}{b}\\left(\\frac{c}{2}-\\frac{1}{2}+\\frac{d}{2}+\\frac{1}{2}\\right)}{\\frac{c}{2}-\\frac{d}{2}}=\\frac{\\frac{1}{b}\\left(\\frac{c+d}{2}\\right)}{\\frac{c-d}{2}}=\\frac{c+d}{b(c-d)} . \\label{eq4}\n$$\n由式\\ref{eq3}\\ref{eq4}知 $K_{E F} \\cdot K_{O_1 O_2}=\\frac{b d-b c}{c+d} \\cdot \\frac{c+d}{b(c-d)}=-1$.\n所以 $E F \\perp O_1 O_2$.\n$2^{\\circ}$ 若 $c=-d$, 则由对称性知 $E F \\quad\\left\\llcorner O_1 O_2\\right.$.\n综上, $E F \\perp O_1 O_2$.", + "remark": "", + "figures": [ + "./images/volume7/figures/fig-c11i7.png" + ] +} \ No newline at end of file diff --git a/processed_dataset/proof/1680.json b/processed_dataset/proof/1680.json new file mode 100644 index 0000000000000000000000000000000000000000..4a173e08b28061aa5883de548b41bdadb307ba94 --- /dev/null +++ b/processed_dataset/proof/1680.json @@ -0,0 +1,10 @@ +{ + "source_file": "./raw_volume-zh/volume7/chapter11.tex", + "problem_type": "proof", + "problem": "例10. 在 $\\triangle A B C$ 中, $A B=A C$, 有一圆内切于 $\\triangle A B C$ 的外接圆,且与 $A B$ 和 $A C$ 分别相切于点 $P$ 和 $Q$. 求证: 点 $P$ 和 $Q$ 连线的中点是 $\\triangle A B C$ 的内切圆圆心.", + "solution": "分析:设 $P Q$ 中点为 $O$, 则 $O$ 在 $\\angle B A C$ 的平分线 $A D$ 上.\n如图() 建立直角坐标系, 设 $O A=1, \\angle B A O=\\alpha$. 设 $\\triangle A B C$ 的外接圆的圆心为 $K$ 内切圆的圆心为 $M$. 连结 $P K$. 则 $O$ 到 $A B$ 与 $A C$ 的距离等于 $\\sin \\alpha$, 故只需证明 $O$ 到 $B C$ 的距离也等于 $\\sin \\alpha$, 即\n$$\ny_B=y_C=-\\sin \\alpha .\n$$\n因为 $\\triangle A B C$ 的外接圆直径\n$$\n\\begin{aligned}\n2 R & =O A+O K+K D=O A+O K+K P \\\\\n& =1+\\tan ^2 \\alpha+\\tan \\alpha \\cdot \\sec \\alpha=\\frac{1+\\sin \\alpha}{\\cos ^2 \\alpha},\n\\end{aligned}\n$$\n所以, $R=\\frac{1+\\sin \\alpha}{2 \\cos ^2 \\alpha}$,\n$$\ny_M=1-R=\\frac{\\cos 2 \\alpha-\\sin \\alpha}{2 \\cos ^2 \\alpha} .\n$$\n从而, $\\odot M$ 的方程为\n$$\nx^2+\\left(y-\\frac{\\cos 2 \\alpha-\\sin \\alpha}{2 \\cos ^2 \\alpha}\\right)^2=\\left(\\frac{1+\\sin \\alpha}{2 \\cos ^2 \\alpha}\\right)^2 .\n$$\n而 $A B$ 的方程为 $y=\\cot \\alpha \\cdot x+1$.\n解上述两方程得\n$$\ny_A=1, y_B=-\\sin \\alpha .\n$$\n故命题成立.", + "remark": "注:此题我们在前面几章曾经多次出现过,这里用的是解析法,也是一种不错的方法.", + "figures": [ + "./images/volume7/figures/fig-c11i8.png" + ] +} \ No newline at end of file diff --git a/processed_dataset/proof/1681.json b/processed_dataset/proof/1681.json new file mode 100644 index 0000000000000000000000000000000000000000..63127555dc416b9cef24eab1ac22d6061d1ef2d0 --- /dev/null +++ b/processed_dataset/proof/1681.json @@ -0,0 +1,10 @@ +{ + "source_file": "./raw_volume-zh/volume7/chapter11.tex", + "problem_type": "proof", + "problem": "例11. $ \\odot O_1$ 和 $\\odot O_2$ 被包含在 $\\odot O$ 内, 且分别与 $\\odot O$ 相切于两个不同的点 $M$ 和 $N . \\odot O_1$ 经过点 $O_2$, 经过 $\\odot O_1$ 和 $\\odot O_2$ 的两个交点的直线与 $\\odot O$ 相交于点 $A$ 和 $B$. 直线 $M A$ 和 $M B$ 分别与 $\\odot O_1$ 相交于 $C$ 和 $D$. 证明: $C D$ 与 $\\odot O_2$\n相切.", + "solution": "分析:如图(), 所以为坐标原点, $M O$ 为 $x$ 轴正半轴, 建立如图所示坐标系.\n设 $\\odot O 、 \\odot O_1$ 、 $\\odot O_2$ 的半径分别为 $r 、 r_1 、 r_2, \\angle O_2 M O=\\alpha$. 连结 $M O_2, O O_2, N O_2$, 则 $\\odot O_1$ 的方程为\n$$\n\\left(x-r_1\\right)^2+y^2=r_1^2,\n$$\n$\\odot \\mathrm{O}_2$ 方程为\n$$\n\\left(x-r_1-r_1 \\cos 2 \\alpha\\right)^2+\\left(y-r_1 \\sin 2 \\alpha\\right)^2=r_2^2 .\n$$\n所以, $A B$ 的方程为\n$$\n\\begin{aligned}\n& \\left(x-r_1\\right)^2+y^2-r_1^2 \\\\\n= & \\left(x-r_1-r_1 \\cos 2 \\alpha\\right)^2+\\left(y-r_1 \\sin 2 \\alpha\\right)^2-r_2^2,\n\\end{aligned}\n$$\n即\n$$\n2 r_1\\left[\\cos 2 \\alpha \\cdot\\left(x-r_1\\right)+\\sin 2 \\alpha \\cdot y\\right]+r_2^2-2 r_1^2=0 \\text {. }\n$$\n又 $\\odot O$ 与 $\\odot O_1$ 关于原点 $M$ 成位似图形 (位似比为 $\\frac{r}{r_1}$ ), 所以, $C D$ 的方程为\n$$\n2 r_1\\left[\\cos 2 \\alpha\\left(\\frac{r}{r_1} x-r_1\\right)+\\sin 2 \\alpha \\frac{r}{r_1} y\\right]+r_2^2-2 r_1^2=0,\n$$\n即\n$$\n2 r \\cos 2 \\alpha \\cdot x+2 r \\sin 2 \\alpha \\cdot y+r_2^2-2 r_1^2(1+\\cos \\alpha)=0 . \\label{eq1}\n$$\n$$\n\\text { 又 } O O_2^2=\\left(r-r_1-r_1 \\cos 2 \\alpha\\right)^2+\\left(r_1 \\sin 2 \\alpha\\right)^2=\\left(r-r_2\\right)^2 \\text {, 所以, }\n$$\n$$\nr_2^2-2 r_1^2(1+\\cos 2 \\alpha)=2 r r_2-2 r r_1(1+\\cos 2 \\alpha) .\n$$\n将上式代入式\\ref{eq1}得 $C D$ 的方程\n$$\n\\cos 2 \\alpha \\cdot x+\\sin 2 \\alpha \\cdot y+r_2-r_1(1+\\cos 2 \\alpha)=0 .\n$$\n从而, $O_2$ 到 $C D$ 的距离为 (注意 $O_2$ 的坐标为 $\\left(r_1+r_1 \\cos 2 \\alpha, r_1 \\sin 2 \\alpha\\right.$ ))\n$$\nd=\\cos 2 \\alpha \\cdot\\left(r_1+r_1 \\cos 2 \\alpha\\right)+\\sin 2 \\alpha \\cdot r_1 \\sin 2 \\alpha+r_2-r_1(1+\\cos 2 \\alpha)=r_2 \\text {. }\n$$\n因此, $C D$ 与 $\\odot O_2$ 相切.", + "remark": "", + "figures": [ + "./images/volume7/figures/fig-c11i9.png" + ] +} \ No newline at end of file diff --git a/processed_dataset/proof/1682.json b/processed_dataset/proof/1682.json new file mode 100644 index 0000000000000000000000000000000000000000..e5298815f2445b88bd2c8425dc9692e2612ffda0 --- /dev/null +++ b/processed_dataset/proof/1682.json @@ -0,0 +1,10 @@ +{ + "source_file": "./raw_volume-zh/volume7/chapter11.tex", + "problem_type": "proof", + "problem": "例12. 求最小常数 $a>1$, 使得对正方形 $A B C D$ 内部任一点 $P$, 都存在 $\\triangle P A B 、 \\triangle P B C 、 \\triangle P C D, \\triangle P D A$ 中的某两个三角形,使得它们的面积之比属于区间 $\\left[a^{-1}, a\\right]$. (2008 第七届女子数学奥林匹克)", + "solution": "解:$a_{\\min }=\\frac{1+\\sqrt{5}}{2}$.\n首先证明 $a_{\\min } \\leqslant \\frac{1+\\sqrt{5}}{2}$, 记 $\\varphi=\\frac{1+\\sqrt{5}}{2}$. 如图 (), 不妨设正方形边长为 $\\sqrt{2}$. 对正方形 $A B C D$ 内部一点 $P$, 令 $S_1, S_2, S_3, S_4$ 分别表示 $\\triangle P A B, \\triangle P B C, \\triangle P C D$, $\\triangle P D A$ 的面积, 不妨设 $S_1 \\geqslant S_2 \\geqslant S_4 \\geqslant S_3$.\n令 $\\lambda=\\frac{S_1}{S_2}, \\mu=\\frac{S_2}{S_4}$, 如果 $\\lambda, \\mu>\\varphi$, 由\n$$\nS_1+S_3=S_2+S_4=1 \\text {, 得 } \\frac{S_2}{1-S_2}=\\mu \\text {, 得 } S_2=\\frac{\\mu}{1+\\mu} \\text {. }\n$$\n故 $S_1=\\lambda S_2=\\frac{\\lambda \\mu}{1+\\mu}=\\frac{\\lambda}{1+\\frac{1}{\\mu}}>\\frac{\\varphi}{1+\\frac{1}{\\varphi}}=\\frac{\\varphi^2}{1+\\varphi}=1$, 矛盾.\n故 $\\min \\{\\lambda, \\mu\\} \\leqslant \\varphi$, 这表明 $a_{\\min } \\leqslant \\varphi$.\n反过来对于任意 $a \\in(1, \\varphi)$, 取定 $t \\in\\left(a, \\frac{1+\\sqrt{5}}{2}\\right)$, 使得 $b=\\frac{t^2}{1+t}>\\frac{8}{9}$. 我们在正方形 $A B C D$ 内取点 $P$, 使得 $S_1=b, S_2=\\frac{b}{t}, S_3=\\frac{b}{t^2}, S_4=1-b$, 则我们有\n$$\n\\frac{S_1}{S_2}=\\frac{S_2}{S_3}=t \\in\\left(a, \\frac{1+\\sqrt{5}}{2}\\right), \\frac{S_3}{S_4}=\\frac{b}{t^2(1-b)}>\\frac{b}{4(1-b)}>2>a,\n$$\n由此我们得到对任意 $i, j \\in\\{1,2,3,4\\}$, 有 $\\frac{S_i}{S_j} \\notin\\left[a^{-1}, a\\right]$. 这表明 $a_{\\min }=\\varphi$.", + "remark": "", + "figures": [ + "./images/volume7/figures/fig-c11i10.png" + ] +} \ No newline at end of file diff --git a/processed_dataset/proof/1683.json b/processed_dataset/proof/1683.json new file mode 100644 index 0000000000000000000000000000000000000000..7f45fc1a98521723f4a2fb21ab9df753fd413d71 --- /dev/null +++ b/processed_dataset/proof/1683.json @@ -0,0 +1,10 @@ +{ + "source_file": "./raw_volume-zh/volume7/chapter2.tex", + "problem_type": "proof", + "problem": "例1. 如图(), $K$ 是 $\\triangle A B C$ 边 $B C$ 上一点且不为 $B C$ 中点, $D_1 、 D_2$ 是 $A K$ 延长线上不同的两点, $B D_i$ 与 $A C$ 交于点 $N_i, C D_i$ 与 $A B$ 交于点 $M_i, i=1$, 2. 求证: $M_1 N_1$ 不平行于 $M_2 N_2$.", + "solution": "证明:由塞瓦定理, 有\n$$\n\\frac{M_1 B}{B A} \\cdot \\frac{A C}{C N_1}=\\frac{M_1 E_1}{E_1 N_1}, \\label{eq1}\n$$\n其中 $E_1$ 为直线 $A K$ 与直线 $M_1 N_1$ 交点;\n$$\n\\frac{M_2 B}{B A} \\cdot \\frac{A C}{C N_2}=\\frac{M_2 E_2}{E_2 N_2} \\label{eq2}\n$$\n其中 $E_2$ 为直线 $A K$ 与直线 $M_2 N_2$ 交点.\n注意到 $A 、 E_1 、 E_2$ 共线于 $A K$, 假设 $M_1 N_1 / / M_2 N_2$, 于是有\n$$\n\\frac{M_1 E_1}{E_1 N_1}=\\frac{M_2 E_2}{E_2 N_2} . \\label{eq3}\n$$\n由式\\ref{eq1}、\\ref{eq2}、式\\ref{eq3}, $\\frac{M_1 B}{C N_1}=\\frac{M_1 E_1}{E_1 N_1} \\cdot \\frac{B A}{A C}=\\frac{M_2 E_2}{E_2} \\frac{B A}{N_2} \\cdot \\frac{B A}{A C}=\\frac{M_2 B}{C N_2}$ 或 $\\frac{M_1 B}{M_1 M_2}=\\frac{N_1 C}{N_1 N_2}$,\n结合 $M_1 N_1 / / M_2 N_2$ 有 $M_1 N_1 / / M_2 N_2 / / B C$. 因此 $\\frac{M_1 B}{B A}=\\frac{N_1 C}{C A}$.\n回到第一个等式: $\\frac{M_1 B}{B A} \\cdot \\frac{A C}{C N_1}=\\frac{M_1 E_1}{E_1 N_1}$, 左边等于 1 , 但右边 $\\frac{M_1 E_1}{E_1 N_1}=\\frac{B K}{K C} \\neq$ 1. 矛盾! 故假设不成立, 即 $M_1 N_1$ 不可能平行于 $M_2 N_2$.", + "remark": "", + "figures": [ + "./images/volume7/figures/fig-c2i6.png" + ] +} \ No newline at end of file diff --git a/processed_dataset/proof/1684.json b/processed_dataset/proof/1684.json new file mode 100644 index 0000000000000000000000000000000000000000..e678c2b366f053aeb7ce0718433a2489ca8f7402 --- /dev/null +++ b/processed_dataset/proof/1684.json @@ -0,0 +1,10 @@ +{ + "source_file": "./raw_volume-zh/volume7/chapter2.tex", + "problem_type": "proof", + "problem": "例2. 如图(), $\\triangle A B C$ 中, $D$ 为线段 $B C$ 上一点, 满足 $A D \\perp B C$, 取边 $A B$ 上点 $E$, 边 $A C$ 上点 $F$, 连结 $D E 、 D F$, 满足 $\\angle E D A=\\angle F D A$, 求证: $A D 、 B F 、 C E$ 三线共点.", + "solution": "证明:法一: 过 $A$ 作 $B C$ 的平行线 $l$, 并与 $D E$ 延长线、 $D F$ 延长线分别交于 $G 、 H, l / / B C$ 以及 $A D \\perp B C$, 则 $l \\perp A D$, 结合 $\\angle E D A=\\angle F D A$, 有 $A$ 为等腰三角形 $D G H$ 底边 $G H$ 的中点, 即 $G A=A H$, 所以\n$$\n\\frac{A E}{E B} \\cdot \\frac{B D}{D C} \\cdot \\frac{C F}{F A}=\\frac{A G}{B D} \\cdot \\frac{B D}{D C} \\cdot \\frac{C D}{A H}=\\frac{A G}{A H}=1 .\n$$\n由角元塞瓦定理的逆定理知 $A D 、 B F 、 C E$ 三线共点.", + "remark": "", + "figures": [ + "./images/volume7/figures/fig-c2i7.png" + ] +} \ No newline at end of file diff --git a/processed_dataset/proof/1685.json b/processed_dataset/proof/1685.json new file mode 100644 index 0000000000000000000000000000000000000000..7940e2baf05eb08d0107e39f58d98eed4a6263ad --- /dev/null +++ b/processed_dataset/proof/1685.json @@ -0,0 +1,10 @@ +{ + "source_file": "./raw_volume-zh/volume7/chapter2.tex", + "problem_type": "proof", + "problem": "例2. 如图(), $\\triangle A B C$ 中, $D$ 为线段 $B C$ 上一点, 满足 $A D \\perp B C$, 取边 $A B$ 上点 $E$, 边 $A C$ 上点 $F$, 连结 $D E 、 D F$, 满足 $\\angle E D A=\\angle F D A$, 求证: $A D 、 B F 、 C E$ 三线共点.", + "solution": "法二: 设 $\\angle E D A=\\angle F D A=\\alpha$, 则 $\\triangle A E D$ 中, 由正弦定理 $\\frac{\\sin \\alpha}{A E}= \\frac{\\sin \\angle A E D}{A D}$.\n同理 $\\triangle B E D$ 中, $\\frac{\\sin \\left(90^{\\circ}-\\alpha\\right)}{B E}=\\frac{\\sin \\angle B E D}{B D}$, 由于 $\\sin \\angle A E D= \\sin \\angle B E D$,\n所以\n$$\n\\tan \\alpha=\\frac{B D \\cdot A E}{B E \\cdot A D}, \\label{eq1}\n$$\n同理 $\\triangle A D F, \\triangle D F C$ 中,\n$$\n\\tan \\alpha=\\frac{C D \\cdot A F}{C F \\cdot A D} . \\label{eq2}\n$$\n由式\\ref{eq1}, \\ref{eq2}, $1=\\frac{A E}{E B} \\cdot \\frac{B D}{D C} \\cdot \\frac{C F}{F A} \\Leftrightarrow A D 、 B F 、 C E$ 三线共点.", + "remark": "", + "figures": [ + "./images/volume7/figures/fig-c2i7.png" + ] +} \ No newline at end of file diff --git a/processed_dataset/proof/1686.json b/processed_dataset/proof/1686.json new file mode 100644 index 0000000000000000000000000000000000000000..bbc3fcd2b93640b617106316695dac8935e3ac39 --- /dev/null +++ b/processed_dataset/proof/1686.json @@ -0,0 +1,10 @@ +{ + "source_file": "./raw_volume-zh/volume7/chapter2.tex", + "problem_type": "proof", + "problem": "例3. 如图(), $A_1 、 B_1 、 C_1$ 分别是 $\\triangle A B C$ 的边 $B C 、 C A 、 A B$ 内任意一点, $G_a, G_b, G_c$ 分别为 $\\triangle A B_1 C_1, \\triangle B C_1 A_1, \\triangle C A_1 B_1$ 的重心.\n求证: $A G_a$, $B G_b, C G_c$ 三线共点的充要条件是 $A A_1, B B_1, C C_1$ 三线共点.", + "solution": "证明:由角元塞瓦定理知 $A G_a, B G_b, C G_c$ 三线共点的充分必要条件为\n$$\n\\left(\\frac{\\sin \\angle B A G_a}{\\sin \\angle G_a A C}\\right) \\cdot\\left(\\frac{\\sin \\angle A C G_c}{\\sin \\angle G_c C B}\\right) \\cdot\\left(\\frac{\\sin \\angle C B G_b}{\\sin \\angle G_b B A}\\right)=1, \\label{eq1}\n$$\n又注意到 $G_a$ 为 $\\triangle A B_1 C_1$ 重心, 因此 $S_{\\triangle G_a A C_1}=S_{\\triangle G_a A B_1}$, 即\n$$\n\\frac{1}{2} \\cdot A C_1 \\cdot A G_a \\cdot \\sin \\angle C_1 A G_a=\\frac{1}{2} \\cdot A B_1 \\cdot A G_a \\cdot \\sin \\angle B_1 A G_a,\n$$\n由此可得\n$$\n\\frac{\\sin \\angle B A G_a}{\\sin \\angle G_a A C}=\\frac{\\sin \\angle C_1 A G_a}{\\sin \\angle G_a A B_1}=\\frac{A B_1}{A C_1},\n$$\n同理可知\n$$\n\\frac{\\sin \\angle A C G_c}{\\sin \\angle G_c C B}=\\frac{A_1 C}{B_1} \\frac{C}{\\operatorname{Con} \\angle C B G_b}=\\frac{B C_1}{\\sin \\angle G_b B A} .\n$$\n则 式\\ref{eq1}就等价于 $\\frac{A B_1}{B_1 C} \\cdot \\frac{C A_1}{A_1 B} \\cdot \\frac{B C_1}{C_1 A}=1$, 由塞瓦定理, 这就等价于 $A A_1 、 B B_1$ 、 $C C_1$ 三线共点.", + "remark": "注:此题完美地将塞瓦定理的边元形式与角元形式结合起来.\n角元塞瓦定理的使用是自然的.", + "figures": [ + "./images/volume7/figures/fig-c2i8.png" + ] +} \ No newline at end of file diff --git a/processed_dataset/proof/1687.json b/processed_dataset/proof/1687.json new file mode 100644 index 0000000000000000000000000000000000000000..82033725bd72c1e874675a0d2d1759816b550d48 --- /dev/null +++ b/processed_dataset/proof/1687.json @@ -0,0 +1,10 @@ +{ + "source_file": "./raw_volume-zh/volume7/chapter2.tex", + "problem_type": "proof", + "problem": "例4. 如图(), $P$ 为 $\\triangle A B C$ 内一点, 使得 $\\angle P A B=10^{\\circ}, \\angle P B A=20^{\\circ}, \\angle P C A=30^{\\circ}$, $\\angle P A C=40^{\\circ}$. 求证: $\\triangle A B C$ 是等腰三角形.", + "solution": "证明:设 $\\angle A C B=x$, 则 $\\angle B C P=x-30^{\\circ}$.\n对 $\\triangle A P C$ 和点 $B$ 应用角元塞瓦定理有\n$$\n1=\\frac{\\sin \\angle A P}{\\sin \\angle B P C} \\cdot \\frac{\\sin \\angle P C B}{\\sin \\angle B C A} \\cdot \\frac{\\sin \\angle C A B}{\\sin \\angle B A P}\n$$\n$$\n\\begin{aligned}\n=\\frac{\\sin 150^{\\circ}}{\\sin 100^{\\circ}} \\cdot \\frac{\\sin \\left(x-30^{\\circ}\\right)}{\\sin x} & \\cdot \\frac{\\sin 50^{\\circ}}{\\sin 10^{\\circ}} \\\\\n\\frac{\\sin \\left(x-30^{\\circ}\\right)}{\\sin x} & =\\frac{2 \\cos 10^{\\circ} \\cdot \\sin 10^{\\circ}}{\\sin 50^{\\circ}} \\\\\n& =\\frac{\\sin \\left(50^{\\circ}-30^{\\circ}\\right)}{\\sin 50^{\\circ}} .\n\\end{aligned}\n$$\n则\n$$\n\\begin{aligned}\n\\frac{\\sin \\left(x-30^{\\circ}\\right)}{\\sin x} & =\\frac{2 \\cos 10^{\\circ} \\cdot \\sin 10^{\\circ}}{\\sin 50^{\\circ}} \\\\\n& =\\frac{\\sin \\left(50^{\\circ}-30^{\\circ}\\right)}{\\sin 50^{\\circ}} .\n\\end{aligned}\n$$\n故 $\\cos 30^{\\circ}-\\cot x \\cdot \\sin 30^{\\circ}=\\cos 30^{\\circ}-\\cot 50^{\\circ} \\cdot \\sin 30^{\\circ}$.\n所以, $\\cot x=\\cot 50^{\\circ}$.\n因此 $x=50^{\\circ}$. 又因为 $\\angle B A C=10^{\\circ}+40^{\\circ}=50^{\\circ}=x=\\angle A C B$, 所以 $\\triangle A B C$ 为等腰三角形.", + "remark": "", + "figures": [ + "./images/volume7/figures/fig-c2i9.png" + ] +} \ No newline at end of file diff --git a/processed_dataset/proof/1688.json b/processed_dataset/proof/1688.json new file mode 100644 index 0000000000000000000000000000000000000000..1792c699c239c132f083ffff23a48b36e67e501e --- /dev/null +++ b/processed_dataset/proof/1688.json @@ -0,0 +1,10 @@ +{ + "source_file": "./raw_volume-zh/volume7/chapter2.tex", + "problem_type": "proof", + "problem": "例6. 如图(), 点 $D 、 E 、 F$ 分别在锐角 $\\triangle A B C$ 的边 $B C 、 C A 、 A B$ 上 (均不是端点), 满足 $B C / / E F, D_1$ 是边 $B C$ 上一点 (不同于 $B 、 D 、 C$ ), 过 $D_1$ 作 $D_1 E_1 / / D E, D_1 F_1 / / D F$, 分别交 $A C 、 A B$ 两边于点 $E_1 、 F_1$, 连结 $E_1 F_1$, 再在 $B C$ 上方 (与 $A$ 同侧) 作 $\\triangle P B C$, 使得 $\\triangle P B C \\backsim \\triangle D E F$, 连结 $P D_1$. 求证: $E F 、 E_1 F_1 、 P D_1$ 三线共点.", + "solution": "证明:连结 $P E_1 、 P F_1$, 设 $E_1 F_1$ 交 $E F$ 于 $K$, 易知 $K$ 在线段 $E_1 F_1$ 上.\n以下证 $K$ 在 $P D_1$ 上.\n注意 $\\triangle D E F \\backsim \\triangle P B C, E F / / B C$, 故 $D E / / B P / / D_1 E_1, D F / / C P / / D_1 F_1$.\n再连结 $C F_1 、 B E_1$, 由梅氏定理得\n$$\n\\begin{aligned}\n\\frac{F_1 K}{K E_1}= & \\frac{F_1 F}{F A} \\cdot \\frac{A E}{E} \\frac{D D_1}{E_1}=\\frac{B F_1}{B D_1} \\frac{A E}{F A} \\frac{A E}{D D_1 \\cdot \\frac{C E_1}{C D_1}}=\\frac{A E}{A F} \\cdot \\frac{C D_1}{B D_1} \\cdot \\frac{B F_1}{C E_1} \\\\\n= & \\frac{\\frac{1}{2} C D_1 \\cdot B F_1 \\sin B}{\\frac{1}{2} B D_1 \\cdot C E_1 \\sin C} \\\\\n= & \\frac{S_{\\triangle C D_1 F_1}}{S_{\\triangle B D_1 E_1}}=\\frac{S_{\\triangle D_1}}{S_{\\triangle P D_1} E_1} \\text {. (由 } C P / / D_1 F_1 \\text { 及 } B P / / D_1 E_1 \\text { ). }\n\\end{aligned}\n$$\n现仅需再注意 $K$ 在 $E_1 F_1$ 上(因 $D_1 F_1$ 与 $B$ 在 $C P$ 同侧, 而 $D_1 E_1$ 与 $C$ 在 $B P$ 同侧) 即可由上式知 $K$ 在 $P D_1$ 上.\n所以欲证结论成立,证毕.", + "remark": "", + "figures": [ + "./images/volume7/figures/fig-c2i11.png" + ] +} \ No newline at end of file diff --git a/processed_dataset/proof/1689.json b/processed_dataset/proof/1689.json new file mode 100644 index 0000000000000000000000000000000000000000..eb533daddfc04425771cd05fd01e945d1811a453 --- /dev/null +++ b/processed_dataset/proof/1689.json @@ -0,0 +1,10 @@ +{ + "source_file": "./raw_volume-zh/volume7/chapter2.tex", + "problem_type": "proof", + "problem": "例7. 在凸五边形 $A B C D E$ 中, $\\angle A E D=\\angle A B C=90^{\\circ}, \\angle B A C= \\angle E A D . B D \\cap C E=F$. 求证: $A F \\perp B E$.", + "solution": "证明:如图(), 过点 $A$ 作 $A H \\perp B E$ 于 $H$,于是只需证明 $A H 、 B D 、 C E$ 三线共点.\n因为 $\\triangle A B C \\backsim \\triangle A E D$, 所以\n$$\n\\frac{A B}{A E}=\\frac{A C}{A D}=\\frac{B C}{D E} .\n$$\n又 $\\angle B A C=\\angle E A D$, 则\n$$\n\\angle B A D=\\angle C A E \\text {. }\n$$\n所以 $S_{\\triangle A B D}=S_{\\triangle A C E}$.\n故\n$$\n\\frac{\\sin \\angle A B D}{\\sin \\angle C E A}=\\frac{A E \\cdot E C}{A B \\cdot B D} . \\label{eq1}\n$$\n在 $\\triangle B C E$ 和 $\\triangle B D E$ 中应用正弦定理有\n$$\n\\frac{\\sin \\angle B E C}{\\sin \\angle E B C}=\\frac{B C}{E C}, \\frac{\\sin \\angle B E D}{\\sin \\angle D B E}=\\frac{B D}{E D} . \\label{eq2}\n$$\n又 $\\angle H A B=\\angle E B C, \\angle E A H=\\angle B E D$, 则\n$$\n\\begin{aligned}\n& \\frac{\\sin \\angle E A H}{\\sin \\angle H A B} \\cdot \\frac{\\sin \\angle A B D}{\\sin \\angle D B E} \\cdot \\frac{\\sin \\angle B E C}{\\sin \\angle C E A} \\\\\n= & \\frac{\\sin \\angle B E D}{\\sin \\angle E B C} \\cdot \\frac{\\sin \\angle A B D}{\\sin \\angle D B E} \\cdot \\frac{\\sin \\angle B E C}{\\sin \\angle C E A} \\\\\n= & \\frac{\\sin \\angle B E D}{\\sin \\angle D B E} \\cdot \\frac{\\sin \\angle A B D}{\\sin \\angle C E A} \\cdot \\frac{\\sin \\angle B E C}{\\sin \\angle E B C} \\\\\n= & \\frac{B D \\cdot A E \\cdot E C \\cdot B C}{E D \\cdot A B \\cdot B D \\cdot E C}=\\frac{A E \\cdot B C}{E D \\cdot A B}=1 .\n\\end{aligned}\n$$\n由关于 $\\triangle A B E$ 的角元塞瓦定理的逆定理知 $A H 、 B D 、 C E$ 三线共点 $F$. 因为 $A H \\perp B E$, 所以 $A F \\perp B E$.", + "remark": "", + "figures": [ + "./images/volume7/figures/fig-c2i12.png" + ] +} \ No newline at end of file diff --git a/processed_dataset/proof/1690.json b/processed_dataset/proof/1690.json new file mode 100644 index 0000000000000000000000000000000000000000..a562e6eed193ff4b8e1a9f661eed4aa287c7a23d --- /dev/null +++ b/processed_dataset/proof/1690.json @@ -0,0 +1,10 @@ +{ + "source_file": "./raw_volume-zh/volume7/chapter2.tex", + "problem_type": "proof", + "problem": "例8. 如图(), 点 $P 、 Q$ 是 $\\triangle A B C$ 的外接圆上(异于 $A 、 B 、 C$ ) 的两点, 点 $P$ 关于直线 $B C 、 C A$ 、 $A B$ 的对称点分别是 $U 、 V 、 W$, 连结 $Q U 、 Q V 、 Q W$ 分别与直线 $B C 、 C A 、 A B$ 交于点 $D 、 E 、 F$.\n求证: (1) $U 、 V 、 W$ 三点共线;\n(2) $D 、 E 、 F$ 三点共线.", + "solution": "证明:(1) 设从点 $P$ 向 $B C 、 C A 、 A B$ 作垂线, 垂足分别为 $X 、 Y 、 Z$.\n由对称知 $X Y$ 为 $\\triangle P U V$ 的中位线, 故 $U V / / X Y$.\n同理 $V W / / Y Z, W U / / X Z$.\n又由西姆松定理知 $X 、 Y 、 Z$ 三点共线.\n故 $U 、 V 、 W$ 三点共线.\n(2) 因为 $P 、 C 、 A 、 B$ 四点共圆, 所以 $\\angle P C E=\\angle A B P$.\n所以 $\\angle P C V=2 \\angle P C E=2 \\angle A B P=\\angle P B W$.\n又 $\\angle P C Q=\\angle P B Q$, 故 $\\angle P C V+\\angle P C Q=\\angle P B W+\\angle P B Q$, 即 $\\angle Q C V=\\angle Q B W$.\n从而 $\\frac{S_{\\triangle Q C V}}{S_{\\triangle Q B W}}=\\frac{C V \\cdot Q C}{Q B \\cdot B W}$.\n同理 $\\frac{S_{\\triangle Q A W}}{S_{\\triangle Q C U}}=\\frac{A W \\cdot A Q}{C Q \\cdot C U}, \\frac{S_{\\triangle Q B U}}{S_{\\triangle Q A V}}=\\frac{B Q \\cdot B U}{A Q \\cdot A V}$.\n所以 $\\frac{S_{\\triangle Q C V}}{S_{\\triangle Q B W}} \\cdot \\frac{S_{\\triangle Q A W}}{S_{\\triangle Q C U}} \\cdot \\frac{S_{\\triangle Q B U}}{S_{\\triangle Q A V}}=1$, (这里注意到 $O U=C V, A W=A V$, $B U=B W)$. 于是\n$$\n\\frac{B D}{D C} \\cdot \\frac{C E}{E A} \\cdot \\frac{A F}{F B}=\\frac{S_{\\triangle Q B U}}{S_{\\triangle Q C U}} \\cdot \\frac{S_{\\triangle Q C V}}{S_{\\triangle Q A V}} \\cdot \\frac{S_{\\triangle W A Q}}{S_{\\triangle W B Q}}=1 .\n$$\n故由梅氏定理逆定理知 $D 、 E 、 F$ 三点共线.", + "remark": "", + "figures": [ + "./images/volume7/figures/fig-c2i13.png" + ] +} \ No newline at end of file diff --git a/processed_dataset/proof/1691.json b/processed_dataset/proof/1691.json new file mode 100644 index 0000000000000000000000000000000000000000..72d0e3c0c213463f0bd68a9e854675cebc8efc71 --- /dev/null +++ b/processed_dataset/proof/1691.json @@ -0,0 +1,10 @@ +{ + "source_file": "./raw_volume-zh/volume7/chapter2.tex", + "problem_type": "proof", + "problem": "例9. 如图(),一圆与 $\\triangle A B C$ 的三边 $B C$ 、 $C A 、 A B$ 的交点依次为 $D_1 、 D_2 、 E_1 、 E_2 、 F_1 、 F_2$. 线段 $D_1 E_1$ 与 $D_2 F_2$ 交于点 $L, E_1 F_1$ 与 $D_2 E_2$ 交于点 $M, F_1 D_1$ 与 $F_2 E_2$ 交于点 $N$. 求证: $A L 、 B M$ 、 $C N$ 三线共点.", + "solution": "证明:连结 $D_1 E_2 、 E_1 F_2 、 F_1 D_2$, 于是, 有\n$$\n\\begin{aligned}\n\\angle D_1 E_1 F_2= & \\angle D_1 E_2 F_2, \\angle D_2 F_2 F_1=\\angle D_2 D_1 F_1, \\\\\n\\angle E_2 E_1 D_1= & \\angle E_2 D_2 D_1, \\angle E_1 F_2 D_2=\\angle E_1 F_1 D_2, \\\\\n& \\angle F_2 F_1 E_1=\\angle F_2 E_2 E_1, \\angle F_1 D_2 E_2=\\angle F_1 D_1 E_2 .\n\\end{aligned}\n$$\n分别对 $\\triangle A F_2 E_1$ 和点 $L 、 \\triangle B D_2 F_1$ 和点 $M 、 \\triangle C E_2 D_1$ 和点 $N$ 应用角元塞瓦定理有\n$$\n\\frac{\\sin \\angle F_2 A L}{\\sin \\angle L A E_1} \\cdot \\frac{\\sin \\angle A E_1 L}{\\sin \\angle L E_1 F_2} \\cdot \\frac{\\sin \\angle E_1 F_2 L}{\\sin \\angle L F_2 A}=1 .\n$$\n则\n$$\n\\frac{\\sin \\angle B A L}{\\sin \\angle L A C}=\\frac{\\sin \\angle D_1 E_1 F_2}{\\sin \\angle E_2 E_1 D_1} \\cdot \\frac{\\sin \\angle D_2 F_2 F_1}{\\sin \\angle E_1 F_2 D_2} . \\label{eq1}\n$$\n同理,有\n$$\n\\begin{aligned}\n& \\frac{\\sin \\angle C B M}{\\sin \\angle M B A}=\\frac{\\sin \\angle E_1 F_1 D_2}{\\sin \\angle F_2 F_1 E_1} \\cdot \\frac{\\sin \\angle E_2 D_2 D_1}{\\sin \\angle F_1 D_2 E_2} . \\label{eq2} \\\\\n& \\frac{\\sin \\angle A C N}{\\sin \\angle N C B}=\\frac{\\sin \\angle F_1 D_1 E_2}{\\sin \\angle D_2 D_1 F_1} \\cdot \\frac{\\sin \\angle F_2 E_2 E_1}{\\sin \\angle D_1 E_2 F_2} . \\label{eq3}\n\\end{aligned}\n$$\n式\\ref{eq1} $\\times$ 式\\ref{eq2} $\\times$ 式\\ref{eq3}并利用前面的六个等式,有\n$$\n\\frac{\\sin \\angle B A L}{\\sin \\angle L A C} \\cdot \\frac{\\sin \\angle A C N}{\\sin \\angle N C B} \\cdot \\frac{\\sin \\angle C B M}{\\sin \\angle M B A}=1 .\n$$\n由角元塞瓦定理的逆定理知 $A L 、 B M 、 C N$ 三线共点.", + "remark": "", + "figures": [ + "./images/volume7/figures/fig-c2i14.png" + ] +} \ No newline at end of file diff --git a/processed_dataset/proof/1692.json b/processed_dataset/proof/1692.json new file mode 100644 index 0000000000000000000000000000000000000000..a88f4b8fa0bb3f81e8969341341f3b89d776578b --- /dev/null +++ b/processed_dataset/proof/1692.json @@ -0,0 +1,11 @@ +{ + "source_file": "./raw_volume-zh/volume7/chapter2.tex", + "problem_type": "proof", + "problem": "例10. 在平面上给定四个点 $A_1 、 A_2 、 A_3 、 A_4$, 其中任意三点不共线, 使得 $A_1 A_2 \\cdot A_3 A_4=A_1 A_3 \\cdot A_2 A_4=A_1 A_4 \\cdot A_2 A_3$.\n记 $O_i$ 是 $\\triangle A_k A_j A_l$ 的外心, 这里 $\\{i, j, k, l\\}=\\{1,2,3,4\\}$. 假设对每个下标 $i$, 都有 $A_i \\neq Q_i$. 证明: 四条直线 $A_i O_i$ 共点或平行.", + "solution": "证明:如图(), 若 $A_1 、 A_2 、 A_3 、 A_4$ 构成一个凹四边形.\n不妨设 $A_4$ 在 $\\triangle A_1 A_2 A_3$ 中, 如图.\n作 $\\triangle A_1 A_3 P \\backsim \\triangle A_1 A_2 A_4$, 则 $\\angle A_3 A_1 P=\\angle A_4 A_1 A_2$.\n于是, $\\angle A_4 A_1 P=\\angle A_2 A_1 A_3$, 且 $\\frac{A_1 P}{A_1 A_3}=\\frac{A_1 A_4}{A_1 A_2}$.\n则 $\\triangle A_1 A_2 A_3 \\backsim \\triangle A_1 A_4 P$, 因此 $\\frac{A_4 P}{A_2 A_3}=\\frac{A_1 A_4}{A_1 A_2}$.\n即 $A_4 P=\\frac{A_1 A_4 \\cdot A_2 A_3}{A_1 A_2}=A_3 A_4$.\n又 $\\frac{A_3 P}{A_1 A_3}=\\frac{A_2 A_4}{A_1 A_2}$, 则\n$$\nA_3 P=\\frac{A_1 A_3 \\cdot A_2 A_4}{A_1 A_2}=A_3 A_4 .\n$$\n因此, $A_3 P=A_4 P=A_3 A_4$, 即 $\\triangle A_3 A_4 P$ 是正三角形.\n故 $\\angle A_1 A_2 A_4+\\angle A_1 A_3 A_4=\\angle A_1 A_3 P+\\angle A_1 A_3 A_4=60^{\\circ}$.\n同理,\n$$\n\\begin{aligned}\n& \\angle A_3 A_2 A_4+\\angle A_3 A_1 A_4=60^{\\circ}, \\\\\n& \\angle A_2 A_1 A_4+\\angle A_2 A_3 A_4=60^{\\circ} .\n\\end{aligned}\n$$\n设 $\\angle A_1 A_2 A_4=\\alpha, \\angle A_2 A_3 A_4=\\beta, \\angle A_3 A_1 A_4=\\gamma$.\n则 $\\angle A_1 A_3 A_4=60^{\\circ}-\\alpha, \\angle A_2 A_1 A_4=60^{\\circ}-\\beta, \\angle A_3 A_2 A_4=60^{\\circ}-\\gamma$.\n又如图(), 因为 $O_1$ 是 $\\triangle A_2 A_3 A_4$ 的外心, 所以, $\\angle A_4 A_2 O_1=90^{\\circ}-\\beta$. 于是, $\\angle A_1 A_2 O_1=90^{\\circ}+\\alpha-$ . 同理, $\\angle A_2 A_3 O_2=90^{\\circ}+\\beta-\\gamma, \\angle A_3 A_1 O_3=90^{\\circ}+ \\gamma-\\alpha$. 又 $\\angle A_4 A_3 O_1=90^{\\circ}-\\angle A_4 A_2 A_3=30^{\\circ}+\\gamma$, 则 $\\angle A_1 A_3 O_1=90^{\\circ}+\\gamma-\\alpha$.\n同理, $\\angle A_2 A_1 O_2=90^{\\circ}+\\alpha-\\beta, \\angle A_3 A_2 O_3= 90^{\\circ}+\\beta-\\gamma$.\n由角元塞瓦定理得\n$$\n\\frac{\\sin \\angle A_2 A_1 O_1}{\\sin \\angle O_1 A_1 A_3} \\cdot \\frac{\\sin \\angle A_3 A_2 O_1}{\\sin \\angle O_1 A_2 A_1} \\cdot \\frac{\\sin \\angle A_1 A_3 O_1}{\\sin \\angle O_1 A_3 A_2}=1 .\n$$\n因为 $\\angle O_1 A_3 A_2=\\angle O_1 A_2 A_3$, 所以\n$$\n\\frac{\\sin \\angle A_2 A_1 O_1}{\\sin \\angle A_3 A_1 O_1}=\\frac{\\sin \\angle O_1 A_2 A_1}{\\sin \\angle O_1 A_3 A_1}\n$$\n$$\n=\\frac{\\sin \\left(90^{\\circ}+\\alpha-\\beta\\right)}{\\sin \\left(90^{\\circ}+\\gamma-\\alpha\\right)}\n$$\n同理,\n$$\n\\begin{aligned}\n& \\frac{\\sin \\angle A_3 A_2 O_2}{\\sin \\angle A_1 A_2 O_2}=\\frac{\\sin \\left(90^{\\circ}+\\beta-\\gamma\\right)}{\\sin \\left(90^{\\circ}+\\alpha-\\beta\\right)}, \\\\\n& \\frac{\\sin \\angle A_1 A_3 O_3}{\\sin \\angle A_2 A_3 O_3}=\\frac{\\sin \\left(90^{\\circ}+\\gamma-\\alpha\\right)}{\\sin \\left(90^{\\circ}+\\beta-\\gamma\\right)} .\n\\end{aligned}\n$$\n因此, $A_1 O_1 、 A_2 O_2 、 A_3 O_3$ 三线共点 (或者互相平行).\n若四个点 $A_1 、 A_2 、 A_3 、 A_4$ 构成一个凸四边形 $A_1 A_2 A_3 A_4$, 类似可得 $A_1 O_1 、 A_2 O_2 、 A_3 O_3$ 三线共点(或者互相平行).\n同理, $A_1 O_1 、 A_2 O_2 、 A_4 O_4$ 三线共点(或者互相平行).\n综上, 四条直线 $A_i O_i$ 共点或平行.", + "remark": "", + "figures": [ + "./images/volume7/figures/fig-c2i15.png", + "./images/volume7/figures/fig-c2i16.png" + ] +} \ No newline at end of file diff --git a/processed_dataset/proof/1693.json b/processed_dataset/proof/1693.json new file mode 100644 index 0000000000000000000000000000000000000000..369d589ba233fd55f228c125784e070fc88cabb3 --- /dev/null +++ b/processed_dataset/proof/1693.json @@ -0,0 +1,10 @@ +{ + "source_file": "./raw_volume-zh/volume7/chapter3.tex", + "problem_type": "proof", + "problem": "例1. 如图(), $\\odot I$ 切 $\\triangle A B C$ 的边 $B C 、 C A$ 、 $A B$ 于 $A^{\\prime} 、 B^{\\prime} 、 C^{\\prime}$. 求证: $A A^{\\prime} 、 B B^{\\prime} 、 C C^{\\prime}$ 必交于一点 $Q$, 则 $\\sum \\frac{A Q}{A A^{\\prime}}=2$.", + "solution": "证明:由切线性质, 可设 $A C^{\\prime}=A B^{\\prime}=x$, $B C^{\\prime}=B A^{\\prime}=y, C A^{\\prime}=C B^{\\prime}=z$, 则\n$$\n\\frac{A^{\\prime} B}{A^{\\prime} C} \\cdot \\frac{B^{\\prime} C}{B^{\\prime} A} \\cdot \\frac{C^{\\prime} A}{C^{\\prime} B}=\\frac{y}{z} \\cdot \\frac{z}{x} \\cdot \\frac{x}{y}=1,\n$$\n由 ceva 定理逆定理知, $A A^{\\prime} 、 B B^{\\prime} 、 C C^{\\prime}$ 共点 $\\mathrm{Q}$.\n考虑直线 $C C^{\\prime}$ 截 $\\triangle A B A^{\\prime}$, 由梅氏定理有\n$$\n\\frac{A C^{\\prime}}{C^{\\prime} B} \\cdot \\frac{B C}{C A^{\\prime}} \\cdot \\frac{A^{\\prime} Q}{Q A}=\\frac{x}{y} \\cdot \\frac{y+z}{z} \\cdot \\frac{A^{\\prime} Q}{Q A}=1 \\Rightarrow \\frac{A^{\\prime} Q}{Q A}=\\frac{y z}{x(y+z)} .\n$$\n所以\n$$\n\\frac{A A^{\\prime}}{A Q}=\\frac{A Q+Q A^{\\prime}}{A Q}=\\frac{x y+y z+z x}{x(y+z)} .\n$$\n同理 $\\frac{B B^{\\prime}}{B Q}=\\frac{x y+y z+z x}{y(z+x)}, \\frac{C C^{\\prime}}{C Q}=\\frac{x y+y z+z x}{z(x+y)}$.\n故 $\\quad \\sum \\frac{A Q}{A A^{\\prime}}=\\frac{x(y+z)+y(z+x)+z(x+y)}{x y+y z+z x}=2$.", + "remark": "注:这一点 $Q$ 通常称之为\"切心\".", + "figures": [ + "./images/volume7/figures/fig-c3i3.png" + ] +} \ No newline at end of file diff --git a/processed_dataset/proof/1694.json b/processed_dataset/proof/1694.json new file mode 100644 index 0000000000000000000000000000000000000000..f4b5934ac5346b7a7cad21bf9780fef823bf7d8c --- /dev/null +++ b/processed_dataset/proof/1694.json @@ -0,0 +1,10 @@ +{ + "source_file": "./raw_volume-zh/volume7/chapter3.tex", + "problem_type": "proof", + "problem": "例2. 求证: 当 $P$ 为三角形外心 $O$ 时,则 $\\sin 2 A \\overrightarrow{O A}+\\sin 2 B \\cdot \\overrightarrow{O B}+\\sin 2 C \\cdot \\overrightarrow{O C}=\\overrightarrow{0}$.", + "solution": "证设 $\\triangle A B C$ 的外心为 $O$, 如图(),连结 $A O$ 交 $B C$ 于 $D$, 交外接圆于 $E$, 连结 $C O$ 交 $A B$ 于 $F$.\n由共边定理可得\n$$\n\\begin{aligned}\n\\frac{B D}{C D} & =\\frac{S_{\\triangle A B D}}{S_{\\triangle A C D}} \\\\\n& =\\frac{A B \\cdot A D \\sin \\angle B A D}{A C \\cdot A D \\sin \\angle C A D} \\\\\n& =\\frac{2 R \\sin C}{2 R \\sin B} \\cdot \\frac{\\sin \\angle B A D}{\\sin \\angle C A D} \\\\\n& =\\frac{2 \\sin C}{2 \\sin B} \\cdot \\frac{\\sin \\angle B C E}{\\sin \\angle C B E} \\\\\n& =\\frac{\\sin 2 C}{\\sin 2 B} .\n\\end{aligned}\n$$\n同理可得\n$$\n\\frac{A F}{F B}=\\frac{\\sin 2 B}{\\sin 2 A} .\n$$\n所以\n$$\n\\begin{aligned}\n\\frac{B D}{B C} & =\\frac{\\sin 2 C}{\\sin 2 B+\\sin 2 C}, \\\\\n\\frac{C D}{B C} & =\\frac{\\sin 2 B}{\\sin 2 B+\\sin 2 C}\n\\end{aligned}\n$$\n在 $\\triangle A B D$ 中由梅涅劳斯定理可得:\n$$\n\\begin{aligned}\n& \\frac{A F}{F B} \\cdot \\frac{B C}{C D} \\cdot \\frac{D O}{O A}=1 \\\\\n& \\Rightarrow \\frac{D O}{O A}=\\frac{F B}{A F} \\cdot \\frac{C D}{B C} \\\\\n& =\\frac{\\sin 2 A}{\\sin 2 B} \\cdot \\frac{\\sin 2 B}{\\sin 2 B+\\sin 2 C} \\\\\n& =\\frac{\\sin 2 A}{\\sin 2 B+\\sin 2 C} .\n\\end{aligned}\n$$\n过 $D$ 作 $D M / / O B, D N / / O C$, 则由三角形相似可知\n$$\n\\overrightarrow{O N}=\\frac{C D}{B C} \\cdot \\overrightarrow{O B}, \\overrightarrow{O M}=\\frac{B D}{B C} \\cdot \\overrightarrow{O C}\n$$\n因为\n$$\n\\overrightarrow{O D}=\\overrightarrow{O M}+\\overrightarrow{O N}\n$$\n又\n$$\n\\overrightarrow{O D}=-\\frac{\\sin 2 A}{\\sin 2 B+\\sin 2 C} \\overrightarrow{O A} \\text {, }\n$$\n所以 $-\\frac{\\sin 2 A}{\\sin 2 B+\\sin 2 C} \\overrightarrow{O A}=\\frac{\\sin 2 B}{\\sin 2 B+\\sin 2 C} \\overrightarrow{O B}+\\frac{\\sin 2 C}{\\sin 2 B+\\sin 2 C} \\overrightarrow{O C}$.\n故 $\\sin 2 A \\cdot \\overrightarrow{O A}+\\sin 2 B \\cdot \\overrightarrow{O B}+\\sin 2 C \\cdot \\overrightarrow{O C}=\\overrightarrow{0}$.", + "remark": "", + "figures": [ + "./images/volume7/figures/fig-c3i4.png" + ] +} \ No newline at end of file diff --git a/processed_dataset/proof/1695.json b/processed_dataset/proof/1695.json new file mode 100644 index 0000000000000000000000000000000000000000..6302b043dddd360e8cd2074fd64087dd1f6f0d01 --- /dev/null +++ b/processed_dataset/proof/1695.json @@ -0,0 +1,11 @@ +{ + "source_file": "./raw_volume-zh/volume7/chapter3.tex", + "problem_type": "proof", + "problem": "例3. 过不等边三角形外心和内心的直线是具有以下性质的点的轨迹: 该点在三角形三边或其延长线上的射影将三边分为六段, 其中相互间隔的三个有向线段的长度的代数和等于另外三个有向线段的长度的代数和.\n如图() 所示, $O 、 I$ 分别为 $\\triangle A B C$ 的外心和内心, $P$ 为 $\\triangle A B C$ 所在平面内的一点, 从 $P$ 作 $P D \\perp B C, P E \\perp C A, P F \\perp A B$, 垂足分别为 $D 、 E 、 F$, 若\n$$\nA F+B D+C E=F B+D C+E A, \\label{eq1}\n$$\n则 $P$ 点的轨迹为直线 $O I$.\n式\\ref{eq1}中的线段均为有向线段, 它们的正方向分别为 $A \\rightarrow B, B \\rightarrow C$ 和 $C \\rightarrow A$. 例如, 若 $F$ 在线段 $A B$ 的内部, 则 $A F$ 和 $F B$ 的长度均为正值, 若 $F$ 在 $A B$ 的延长线上, 则 $A F$ 的长度为正, $F B$ 的长度为负.\n以下证明和讨论中涉及到的线段, 凡属于三角形的边所在直线的, 其长度的正负号均服从这一规定.", + "solution": "证明:首先证明: 直线 $O I$ 上的任意点 $P$ 都满足式\\ref{eq1}. 为方便起见, 设 $O P: O I=k, O P$ 和 $O I$ 的方向以 $O \\rightarrow I$ 为正.\n设 $O 、 P 、 I$ 在三边上的射影分别为 $D_1 、 D 、 D_2 ; E_1 、 E 、 E_2$ 和 $F_1 、 F 、 F_2$, 如图所示, 则由外心和内心的性质可知\n$$\n\\begin{aligned}\n& A F_1+B D_1+C E_1=F_1 B+D_1 C+E_1 A . \\label{eq2} \\\\\n& A F_2+B D_2+C E_2=F_2 B+D_2 C+E_2 A . \\label{eq3}\n\\end{aligned}\n$$\n从而\n$$\n\\begin{aligned}\n& 2\\left(F_1 F_2+D_1 D_2+E_1 E_2\\right) \\\\\n= & \\left(A F_2-A F_1\\right)+\\left(F_1 B-F_2 B\\right)+\\left(B D_2-B D_1\\right) \\\\\n& +\\left(D_1 C-D_2 C\\right)+\\left(C E_2-C E_1\\right)+\\left(E_1 A-E_2 A\\right) \\\\\n= & 0 .\n\\end{aligned} \\label{eq4}\n$$\n此外, 由于 $O D_1 / / P D / / I D_2, O E_1 / / P E / / I E_2, O F_1 / / P F / / I F_2$, 有以下比例关系:\n$$\n\\begin{aligned}\nD_1 D: D_1 D_2 & =E_1 E: E_1 E_2=F_1 F: F_1 F_2 \\\\\n& =O P: O I=k .\n\\end{aligned} \\label{eq5}\n$$\n由式\\ref{eq2}、\\ref{eq4}、式\\ref{eq5}可得\n$$\n\\begin{aligned}\n& (A F+B D+C E)-(F B+D C+E A) \\\\\n= & \\left(A F_1+F_1 F+B D_1+D_1 D+C E_1+E_1 E\\right) \\\\\n& -\\left(F_1 B-F_1 F+D_1 C-D_1 D+E_1 A-E_1 E\\right) \\\\\n= & \\left(A F_1+B D_1+C E_1\\right)-\\left(F_1 B+D_1 C+E_1 A\\right) \\\\\n& +2\\left(F_1 F+D_1 D+E_1 E\\right) \\\\\n= & 2 k\\left(F_1 F_2+D_1 D_2+E_1 E_2\\right) \\\\\n= & 0,\n\\end{aligned}\n$$\n因此式\\ref{eq1}成立.\n如图() 中,外心 $O$ 在 $\\triangle A B C$ 的内部, $P$ 为线段 $O I$ 内部的点,这并非必要.\n对于其他情况, 例如外心在三角形的外部以及 $P$ 在 $O I$ 或 $I O$ 延长线上的情况,包括 $P$ 在三角形外部的情况, 只要统一执行上述关于线段长度的符号规定,证明过程都是相同的,这里不一一论述.\n其次, 可证明: 若 $P$ 不是直线 $O I$ 上的点,则式\\ref{eq1}一定不成立.\n由此可知,直线 $O I$ 就是 $P$ 点的轨迹.", + "remark": "", + "figures": [ + "./images/volume7/figures/fig-c3i5.png", + "./images/volume7/figures/fig-c3i6.png" + ] +} \ No newline at end of file diff --git a/processed_dataset/proof/1696.json b/processed_dataset/proof/1696.json new file mode 100644 index 0000000000000000000000000000000000000000..f874a43f6ac8231497ef0413cfc55f3a1b3ed474 --- /dev/null +++ b/processed_dataset/proof/1696.json @@ -0,0 +1,10 @@ +{ + "source_file": "./raw_volume-zh/volume7/chapter3.tex", + "problem_type": "proof", + "problem": "例4. 平面内两条直线 $l_1 / / l_2$, 它们之间的距离等于 $a$. 一块正方形的硬纸板 $A B C D$ 的边长也等于 $a$. 现将这块硬纸板平放在两条平行线上, 使得 $l_1$ 与 $A B 、 A D$ 都相交, 交点为 $E 、 F ; l_2$ 与 $C B 、 C D$ 都相交, 交点为 $G 、 H$. 设 $\\triangle A E F$ 的周长为 $m_1, \\triangle C G H$ 的周长为 $m_2$. 证明: 无论怎样放置正方形硬纸板 $A B C D, m_1+m_2$ 总是一个定值.", + "solution": "证明:如图(), 连结 $E H 、 F G$ 得交点 $O$.\n因为点 $H$ 到 $A B 、 l_1$ 距离相等, 所以 $E H$ 平分 $\\angle B E F$, 也平分 $\\angle D H G$.\n又点 $G$ 到 $A D 、 l_1$ 等距离, 所以 $F G$ 平分 $\\angle D F E$, 也平分 $\\angle B G H$.\n由此可知, $O$ 既是 $\\triangle A E F$ 的旁心, 又是 $\\triangle C G H$ 的旁心, 作出两个旁切圆, 易知它们是同心圆.\n设 $P 、 M 、 Q 、 N$ 分别是 $A B 、 A D 、 C D 、 C B$ 上的切点, 易证 $P 、 Q 、 O$ 共线; $M 、 O 、 N$ 共线, 且 $P Q= A D=a, M N=A B=a$.\n由旁心性质(4)知\n$$\nA P=A M=\\frac{1}{2} m_1, C Q=C N=\\frac{1}{2} m_2\n$$\n故 $m_1+m_2=2 A P+2 C Q=2 O M+2 O N=2 M N=2 a$ 为定值.", + "remark": "", + "figures": [ + "./images/volume7/figures/fig-c3i7.png" + ] +} \ No newline at end of file diff --git a/processed_dataset/proof/1697.json b/processed_dataset/proof/1697.json new file mode 100644 index 0000000000000000000000000000000000000000..b2013ecfeacfd0ab1f6879a9d242c04d0650856e --- /dev/null +++ b/processed_dataset/proof/1697.json @@ -0,0 +1,10 @@ +{ + "source_file": "./raw_volume-zh/volume7/chapter3.tex", + "problem_type": "proof", + "problem": "例5. 设点 $O$ 是锐角 $\\triangle A B C$ 的外心.\n分别以 $\\triangle A B C$ 三边的中点为圆心作过点 $O$ 的圆, 这三个圆两两的异于 $O$ 的交点分别为 $K 、 L 、 M$. 证明 : 点 $O$ 是 $\\triangle K L M$ 的内心.", + "solution": "证明:如图() 设三边中点分别为 $A^{\\prime} 、 B^{\\prime}$ 、 $C^{\\prime}$, 我们发现 $B^{\\prime} C^{\\prime}$ 垂直平分公共弦 $O K$, 并设交点为 $K^{\\prime}$, 那么 $O K^{\\prime}=\\frac{1}{2} \\cdot O K$, 类似地定义 $L^{\\prime} 、 M^{\\prime}$, 我们有 $\\triangle K^{\\prime} L^{\\prime} M^{\\prime}$ 位似于 $\\triangle K L M$, 相似比为 $\\frac{1}{2}$, 位似中心为 $O$, 于是原命题 $\\Leftrightarrow O$ 是 $\\triangle K^{\\prime} L^{\\prime} M^{\\prime}$ 的内心 , 结合前面的性质: 三角形的垂心是其垂足三角形的内心.\n只需证明, $O$ 为 $\\triangle A^{\\prime} B^{\\prime} C^{\\prime}$ 的垂心, 且 $K^{\\prime} 、 L^{\\prime}$ 、$M^{\\prime}$ 分别是 $O$ 在三边上的垂足.\n$A^{\\prime}$ 为边 $B C$ 中点, 故 $O A^{\\prime} \\perp B C \\Rightarrow O A^{\\prime} \\perp B^{\\prime} C^{\\prime}$, 又 $O K^{\\prime} \\perp B^{\\prime} C^{\\prime}$, 所以 $A^{\\prime}$ 、 $O 、 K^{\\prime}$ 共线且该线垂直于 $B^{\\prime} C^{\\prime}$.\n故原命题成立.", + "remark": "", + "figures": [ + "./images/volume7/figures/fig-c3i8.png" + ] +} \ No newline at end of file diff --git a/processed_dataset/proof/1698.json b/processed_dataset/proof/1698.json new file mode 100644 index 0000000000000000000000000000000000000000..2147b02b814685ea9d67318d123177efc21bd095 --- /dev/null +++ b/processed_dataset/proof/1698.json @@ -0,0 +1,10 @@ +{ + "source_file": "./raw_volume-zh/volume7/chapter3.tex", + "problem_type": "proof", + "problem": "例6. 如图(), 在锐角三角形 $\\triangle A B C$ 中, $A B), 在 $\\triangle A B C$ 中, 设 $A B> A C$, 过 $A$ 作 $\\triangle A B C$ 的外接圆的切线 $l$, 又以 $A$ 为圆心, $A C$ 为半径作圆分别交线段 $A B$ 于 $D$; 交直线 $l$ 于 $E 、 F$. 证明:直线 $D E 、 D F$ 分别通过 $\\triangle A B C$ 的内心与一个旁心.\n(注: 与三角形的一边及另两边的延长线均相切的圆称为三角形的旁切圆, 旁切圆的圆心称为旁心.)", + "solution": "证明:(1) 先证 $D E$ 过 $\\triangle A B C$ 的内心.\n如图(), 连结 $D E 、 D C$, 作 $\\angle B A C$ 的平分线分别交 $D C$ 于 $G 、 D E$ 于 $I$, 连结 $I C$, 则由 $A D=A C$, 得 $A G \\perp D C, I D=I C$.\n又 $D 、 C 、 E$ 在 $\\odot A$ 上, 所以 $\\angle I A C=\\frac{1}{2} \\angle D A C=\\angle I E C$, 因而 $A 、 I 、 C 、 E$ 四点共圆.\n从而 $\\angle C I E=\\angle C A E=\\angle A B C$, 而 $\\angle C I E=2 \\angle I C D$, 则 $\\angle I C D= \\frac{1}{2} \\angle A B C$.\n故 $\\angle A I C=\\angle I G C+\\angle I C G=90^{\\circ}+\\frac{1}{2} \\angle A B C$, 所以 $\\angle A C I=\\frac{1}{2} \\angle A C B$,\n故 $I$ 为 $\\triangle A B C$ 的内心.\n(2) 再证 $D F$ 过 $\\triangle A B C$ 的一个旁心.\n连结 $F D$ 并延长交 $\\angle A B C$ 的外角平分线于 $I_1$, 连结 $I I_1 、 B I_1 、 B I$, 由 (1) 知, $I$ 为内心.\n所以 $\\angle I B I_1=90^{\\circ}=\\angle E D I_1$, 故 $D 、 B 、 I_1 、 I$ 四点共圆.\n因为 $\\angle B I I_1=\\angle B D I_1=90^{\\circ}-\\angle A D I$\n$$\n=\\left(\\frac{1}{2} \\angle B A C+\\angle A D G\\right)-\\angle A D I=\\frac{1}{2} \\angle B A C+\\angle I D G,\n$$\n所以 $A 、 I 、 I_1$ 共线.\n故 $I_1$ 是 $\\triangle A B C$ 的 $B C$ 边外的旁心.", + "remark": "", + "figures": [ + "./images/volume7/figures/fig-c3i10.png", + "./images/volume7/figures/fig-c3i10.png" + ] +} \ No newline at end of file diff --git a/processed_dataset/proof/1700.json b/processed_dataset/proof/1700.json new file mode 100644 index 0000000000000000000000000000000000000000..1656f254e94d80be6c5fd1f146964f47ce02210c --- /dev/null +++ b/processed_dataset/proof/1700.json @@ -0,0 +1,10 @@ +{ + "source_file": "./raw_volume-zh/volume7/chapter3.tex", + "problem_type": "proof", + "problem": "例8. 如图() 在锐角三角形 $A B C$ 中, $A A_1$ 、 $B B_1$ 是两条角平分线, $I 、 O 、 H$ 分别是 $\\triangle A B C$ 的内心、外心、垂心, 连结 $H O$, 分别交 $A C 、 B C$ 于点 $P$, Q. 已知 $C 、 A_1 、 I 、 B_1$ 四点共圆.\n求证: (1) $\\angle C= 60^{\\circ}$; (2) $P Q=A P+B Q$.", + "solution": "证明:(1) 因为 $C 、 A_1 、 I 、 B_1$ 四点共圆, 所以\n$$\n\\begin{aligned}\n\\angle C & =180^{\\circ}-\\angle A I B=\\angle I A B+\\angle I B A \\\\\n& =\\frac{1}{2} \\angle A+\\frac{1}{2} \\angle B=90^{\\circ}-\\frac{1}{2} \\angle C,\n\\end{aligned}\n$$\n所以\n$$\n\\angle C=60^{\\circ} \\text {. }\n$$\n(2)因为\n$$\n\\begin{aligned}\n& \\angle A H B=180^{\\circ}-\\angle C=120^{\\circ}, \\\\\n& \\angle A O B=2 \\angle A C B=120^{\\circ},\n\\end{aligned}\n$$\n所以 $A 、 H 、 O 、 B$ 四点共圆,于是\n$$\n\\angle P H A=\\angle O B A=\\frac{1}{2}\\left(180^{\\circ}-\\angle A O B\\right)=30^{\\circ},\n$$\n又所以\n于是同理可得\n故\n$$\n\\angle P A H=90^{\\circ}-\\angle C=30^{\\circ},\n$$\n$$\n\\begin{aligned}\n\\angle P A H & =\\angle P H A, \\\\\nA P & =P H, \\\\\nB Q & =Q H,\n\\end{aligned}\n$$\n$$\nP Q=A P+B Q .\n$$", + "remark": "", + "figures": [ + "./images/volume7/figures/fig-c3i11.png" + ] +} \ No newline at end of file diff --git a/processed_dataset/proof/1701.json b/processed_dataset/proof/1701.json new file mode 100644 index 0000000000000000000000000000000000000000..8c515243040215f41fb4e6b926f4402ccb2bf5a3 --- /dev/null +++ b/processed_dataset/proof/1701.json @@ -0,0 +1,10 @@ +{ + "source_file": "./raw_volume-zh/volume7/chapter3.tex", + "problem_type": "proof", + "problem": "例9. 如图(), 圆 $O$ 、圆 $I$ 分别是 $\\triangle A B C$ 的外接圆和内切圆, 圆 $O$ 半径为 $R$, 圆 $I$ 半径为 $r$, 圆 $I$ 分别切 $A B 、 A C 、 B C$ 于点 $F 、 E 、 D$, 若 $M$ 为 $\\triangle D E F$ 的重心, 试求 $\\frac{I M}{O M}$ 的值 (其中 $R \\neq 2 r$ ).", + "solution": "解:取 $\\triangle D E F$ 的垂心 $H$, 设 $D H 、 E H 、 F H$ 分别交 $\\odot I$ 于 $A^{\\prime} 、 B^{\\prime} 、 C^{\\prime}$.\n则 $\\angle H A^{\\prime} C^{\\prime}=\\angle D F C^{\\prime}=\\angle D E B^{\\prime}=\\angle D A^{\\prime} B^{\\prime}$.\n同理 $\\angle H C^{\\prime} A^{\\prime}=\\angle H C^{\\prime} B^{\\prime}$. 故 $H$ 为 $\\triangle A^{\\prime} B^{\\prime} C^{\\prime}$ 的内心.\n注意到 $D$ 是 $\\bar{B}^{\\prime} D C^{\\prime}$ 的中心, 则 $I D \\perp B^{\\prime} C^{\\prime}$.\n又 $I D \\perp B C$, 所以 $B^{\\prime} C^{\\prime} / / B C$.\n同理 $A^{\\prime} B^{\\prime} / / A B, A^{\\prime} C^{\\prime} / / A C$, 所以 $\\triangle A^{\\prime} B^{\\prime} C^{\\prime} \\backsim \\triangle A B C$.\n而 $O 、 I$ 分别是 $\\triangle A B C$ 的外心和内心, $I 、 H$ 分别是 $\\triangle A^{\\prime} B^{\\prime} C^{\\prime}$ 的外心和内心.\n所以 $\\frac{O I}{I H}=k, k$ 为 $\\triangle A B C$ 与 $\\triangle A^{\\prime} B^{\\prime} C^{\\prime}$ 的相似比.\n又 $k=\\frac{R}{r}$, 则 $\\frac{O I}{I H}=\\frac{R}{r}$.\n又 $O I 、 I H$ 为 $\\triangle A B C$ 与 $\\triangle A^{\\prime} B^{\\prime} C^{\\prime}$ 中的对应线段.\n则 $O I$ 与 $B C$ 所成的角等于 $I H$ 与 $B^{\\prime} C^{\\prime}$ 所成的角, 则 $O 、 I 、 H$ 共线.\n又由欧拉定理知 $\\triangle D E F$ 中, $I 、 M 、 H$ 分别为外心、重心和垂心.\n所以 $\\frac{I M}{M H}=\\frac{1}{2}, \\frac{I M}{I H}=\\frac{1}{3}$.\n从而 $O M=O I+I M=\\frac{R}{r} I H+I M=\\left(\\frac{R}{r} \\cdot 3+1\\right) \\cdot I M$.\n故 $\\frac{I M}{O M}=\\frac{1}{\\frac{3 R}{r}+1}=\\frac{r}{3 R+r}$, 得证.", + "remark": "", + "figures": [ + "./images/volume7/figures/fig-c3i12.png" + ] +} \ No newline at end of file diff --git a/processed_dataset/proof/1702.json b/processed_dataset/proof/1702.json new file mode 100644 index 0000000000000000000000000000000000000000..7be41441b635f333a31aed0f6d16b342a11ac9fc --- /dev/null +++ b/processed_dataset/proof/1702.json @@ -0,0 +1,10 @@ +{ + "source_file": "./raw_volume-zh/volume7/chapter3.tex", + "problem_type": "proof", + "problem": "例10. 如图(), 锐角 $\\triangle A B C$ 中, $B C> A C>A B, A C$ 上的点 $E$ 与 $B C$ 上的点 $D$ 满足 $A E=B D, C D+C E=A B, B E$ 交 $A D$ 于 $K$. 求证: $K H=2 I O$.", + "solution": "证设 $H 、 I 、 O 、 G$ 分别为 $\\triangle A B C$ 垂心、内心、外心、重心.\n注意到 $H 、 G 、 O$ 三点共线且 $H G=2 O G$.\n故我们只需证 $K 、 G 、 I$ 共线且 $K G=2 I G$.\n取 $B C$ 中点 $M, A C$ 中点 $L$, 延长 $A I$ 交 $B C$ 于 $N$.\n设 $B C=a, C A=b, A B=c$, 则由条件易知\n$$\nC M=\\frac{1}{2} a, C D=\\frac{1}{2}(a+c-b) .\n$$\n由角平分线性质定理 $\\frac{C N}{B N}=\\frac{A C}{A B}$ 知 $C N=\\frac{a b}{b+c}$ 及\n$$\n\\frac{N I}{A I}=\\frac{B N}{A B}=\\frac{C N}{A C}=\\frac{B N+C N}{A B+A C}=\\frac{a}{b+c} .\n$$\n则 $\\frac{N M}{M D}=\\frac{C N-C M}{C M-C D}=\\frac{\\frac{a b}{b+c}-\\frac{1}{2} a}{\\frac{1}{2} a-\\frac{1}{2}(a+c-b)}=\\frac{a}{b+c}=\\frac{N I}{I A}$.\n所以 $I M / / A D$, 同理 $I L / / B E$.\n结合 $M L / / A B$, 故 $\\triangle I M L$ 与 $\\triangle K A B$ 对应边均平行.\n故两三角形位似, 位似中心为 $A M$ 与 $B L$ 交点 $G$, 位似比为 $\\frac{M L}{A B}=\\frac{1}{2}$.\n故 $I 、 G 、 K$ 三点共线且 $I G=\\frac{1}{2} G K$.\n结合前面的讨论知原命题成立.", + "remark": "", + "figures": [ + "./images/volume7/figures/fig-c3i13.png" + ] +} \ No newline at end of file diff --git a/processed_dataset/proof/1703.json b/processed_dataset/proof/1703.json new file mode 100644 index 0000000000000000000000000000000000000000..ffce90152140c6e7b61b137735f553ddf7d1e26e --- /dev/null +++ b/processed_dataset/proof/1703.json @@ -0,0 +1,10 @@ +{ + "source_file": "./raw_volume-zh/volume7/chapter4.tex", + "problem_type": "proof", + "problem": "例1. 如图(), 设 $B$ 是圆 $S_1$ 上的点, 过 $B$ 作圆 $S_1$ 的切线, $A$ 为该切线上异于 $B$ 的点, 又 $C$ 不是圆 $S_1$ 上的点, 且线段 $A C$ 交圆 $S_1$ 于两个不同的点.\n圆 $S_2$ 与 $A C$ 相切于点 $C$, 与圆 $S_1$ 相切于点 $D$, 且 $D$ 与 $B$ 在直线 $A C$ 的两侧.\n证明: $\\triangle B C D$ 的外心在 $\\triangle A B C$ 的外接圆上.", + "solution": "证明:作两圆公切线 $T T^{\\prime}$ 满足 $T$ 与 $A$ 在 $B D$ 同侧, 取 $B D$ 中点 $E, C D$ 中点 $F$, 连结$B K 、 E K 、 D K 、 F K 、 C K$.\n因 $\\angle T D B=\\angle A B D, \\angle T^{\\prime} D C=\\angle D C A$, 则\n$$\n\\begin{aligned}\n\\angle B D C & =180^{\\circ}-\\angle T D B+\\angle T^{\\prime} D C \\\\\n& =180^{\\circ}-\\angle A B D+\\angle D C A \\\\\n& =180^{\\circ}-(\\angle A B C-\\angle D B C)+(\\angle D C B-\\angle A C B) \\\\\n& =180^{\\circ}-\\angle A B C-\\angle A C B+\\angle D B C+\\angle D C B \\\\\n& =\\angle B A C+180^{\\circ}-\\angle B D C .\n\\end{aligned}\n$$\n于是\n$$\n2 \\angle B D C=180^{\\circ}+\\angle B A C .\n$$\n故\n$$\n\\begin{aligned}\n\\angle B K C & =\\angle B K D+\\angle D K C \\\\\n& =2 \\cdot(\\angle E K D+\\angle D K F) \\\\\n& =2 \\cdot \\angle E K F \\\\\n& =2 \\cdot\\left(180^{\\circ}-\\angle B D C\\right) \\\\\n& =180^{\\circ}-\\angle B A C .\n\\end{aligned}\n$$\n因此 $K$ 在 $\\triangle A B C$ 的外接圆上.", + "remark": "", + "figures": [ + "./images/volume7/figures/fig-c4i2.png" + ] +} \ No newline at end of file diff --git a/processed_dataset/proof/1704.json b/processed_dataset/proof/1704.json new file mode 100644 index 0000000000000000000000000000000000000000..2fc3ff4a7216b287a469d8532888284a24821b9e --- /dev/null +++ b/processed_dataset/proof/1704.json @@ -0,0 +1,10 @@ +{ + "source_file": "./raw_volume-zh/volume7/chapter4.tex", + "problem_type": "proof", + "problem": "例2. 如图(), $\\triangle A B C$ 的内切圆 $I$ 切 $B C 、 C A$ 于 $D 、 E, K 、 L$ 为 $A B 、 A C$ 的中点, 则 $D E$ 与 $K L$ 的交点 $T$ 在 $B I$ 的延长线上.", + "solution": "证明:设直线 $B I$ 分别交 $E D 、 K L$ 于 $T^{\\prime \\prime} 、 T^{\\prime}$, 连结 $T^{\\prime} B 、 T^{\\prime \\prime} B 、 A T^{\\prime} 、 A I 、 I D 、 E T^{\\prime}$.\n易知 $\\angle K B T^{\\prime}=\\angle K T^{\\prime} B$.\n所以 $B K=K T^{\\prime}=A K$, 从而 $\\angle A T^{\\prime} B=90^{\\circ}$.\n又 $\\angle A E I=90^{\\circ}$, 故 $A 、 E 、 T^{\\prime} 、 I$ 四点共圆.\n又 $\\angle C E T^{\\prime}=\\frac{\\pi-\\angle A C B}{2}=\\angle A B C+\\angle B A C=\\angle A I T^{\\prime \\prime}$,\n故 $A 、 E 、 T^{\\prime \\prime} 、 I$ 四点共圆.\n又 $T^{\\prime} 、 T^{\\prime \\prime}$ 都在直线 $B I$ 上, 所以 $\\triangle A E I$ 的外接圆与直线 $B I$ 的交点为 $I$ 、 $T^{\\prime} 、 T^{\\prime \\prime}$, 故 $T^{\\prime}$ 与 $T^{\\prime \\prime}$ 重合, 即 $T$ 在直线 $B I$ 上.", + "remark": "", + "figures": [ + "./images/volume7/figures/fig-c4i3.png" + ] +} \ No newline at end of file diff --git a/processed_dataset/proof/1705.json b/processed_dataset/proof/1705.json new file mode 100644 index 0000000000000000000000000000000000000000..fbb4c0dc00fd73cacc932d24c3f2feb5c16fecba --- /dev/null +++ b/processed_dataset/proof/1705.json @@ -0,0 +1,10 @@ +{ + "source_file": "./raw_volume-zh/volume7/chapter4.tex", + "problem_type": "proof", + "problem": "例3. 如图(), 设 $A 、 B$ 是定点, $C$ 是动点, 且 $\\angle A C B=\\alpha$ 是定角, 其中, $0^{\\circ}<\\alpha<180^{\\circ} . \\triangle A B C$ 的内切圆 $\\odot I$ 在边 $B C 、 C A 、 A B$ 上的切点分别为 $F 、 E 、 D$, $E F$ 分别与直线 $A I 、 B I$ 交于点 $M 、 N$. 证明: 线段 $M N$ 的长是定长,且 $\\triangle D M N$ 的外接圆过一个定点.", + "solution": "证明:取线段 $A B$ 的中点 $O$.\n因 $\\angle C E F=90^{\\circ}-\\frac{1}{2} \\angle C=180^{\\circ}-\\angle A I B= \\angle A I N$.\n所以 $I 、 N 、 E 、 A$ 四点共圆.\n又 $I 、 E 、 A 、 D$ 四点共圆, 则 $I 、 N 、 E 、 A 、 D$ 五点共圆.\n同理, $B 、 F 、 M 、 I 、 D$ 五点共圆.\n从而, $\\angle A N B=\\angle A E I=90^{\\circ}, \\angle A M B=\\angle I F B=90^{\\circ}$.\n因此, 点 $M 、 N$ 均在以 $A B$ 为直径的圆上.\n则\n$$\n\\begin{aligned}\n\\angle M O N & =2 \\angle M A N=2 \\angle M B N \\\\\n& =\\angle N A M+\\angle N B M \\\\\n& =\\angle N D I+\\angle M D I=\\angle M D N .\n\\end{aligned}\n$$\n故 $M 、 N 、 O 、 D$ 四点共圆.\n因此, $\\triangle M N D$ 的外接圆过定点 $O$.\n另一方面, $M N=A B \\sin \\angle N A M=A B \\sin \\angle I E F=A B \\sin \\frac{C}{2}=A B \\sin \\frac{\\alpha}{2} \\text { 为定值.\n}$", + "remark": "", + "figures": [ + "./images/volume7/figures/fig-c4i4.png" + ] +} \ No newline at end of file diff --git a/processed_dataset/proof/1706.json b/processed_dataset/proof/1706.json new file mode 100644 index 0000000000000000000000000000000000000000..707a7da0ab8254354c2852ab5d29b31c519b8be7 --- /dev/null +++ b/processed_dataset/proof/1706.json @@ -0,0 +1,10 @@ +{ + "source_file": "./raw_volume-zh/volume7/chapter4.tex", + "problem_type": "proof", + "problem": "例4. 如图(), $A B$ 是圆 $O$ 的直径.\n$C$ 与 $D$ 是互异的圆 $O$ 上的两点, 且在 $A B$ 的一侧.\n过 $C 、 D$ 作圆的切线交于点 $E$. 线段 $A D$ 与 $B C$ 交于点 $F$, 直线 $E F$ 交 $A B$ 于 $M$. 求证: $E, C, M, D$ 共圆.", + "solution": "证明:连结 $E O 、 C O 、 D O 、 C A$. 由 $\\angle C O E= \\angle C A F$, 知 Rt $\\triangle O O E \\backsim$ Rt $\\triangle C A F$.\n所以, $\\frac{C E}{C F}=\\frac{C O}{C A}$. 又 $\\angle E C F=90^{\\circ}-\\angle B C O=\\angle O C A$,\n则 $\\triangle E C F \\backsim \\triangle O C A$.\n故 $\\angle C A O=\\angle C F E=\\angle B F M$. 于是, $\\angle F M B=\\angle A C B=90^{\\circ}$.\n因此, $O 、 M 、 D 、 E 、 C$ 五点共圆.", + "remark": "", + "figures": [ + "./images/volume7/figures/fig-c4i5.png" + ] +} \ No newline at end of file diff --git a/processed_dataset/proof/1707.json b/processed_dataset/proof/1707.json new file mode 100644 index 0000000000000000000000000000000000000000..1e399fe56823b4a4cd5777989df624349e14f3d3 --- /dev/null +++ b/processed_dataset/proof/1707.json @@ -0,0 +1,10 @@ +{ + "source_file": "./raw_volume-zh/volume7/chapter4.tex", + "problem_type": "proof", + "problem": "例5. 如图(), 给定锐角三角形 $A B C, O$ 为外心, 直线 $A O$ 交边 $B C$ 于 $D$, 动点 $E 、 F$ 在 $A B 、 A C$ 上, 使得 $A 、 E 、 D 、 F$ 四点共圆.\n求证: 线段 $E F$ 在 $B C$ 上的投影为恒定长度.", + "solution": "证明:取 $A B$ 上一点 $E^{\\prime}, A C$ 上一点 $F^{\\prime}$ 满足 $A 、 E^{\\prime} 、 D 、 F^{\\prime}$ 四点共圆.\n下面证明: $E F$ 在 $B C$ 上的投影长度等于 $E^{\\prime} F^{\\prime}$ 在 $B C$ 上的投影长度, 由正弦定理,\n$$\n\\begin{aligned}\n\\frac{D E}{D F} & =\\frac{\\sin \\angle E F D}{\\sin \\angle D E F} \\\\\n& =\\frac{\\sin \\angle E A D}{\\sin \\angle D A F} \\\\\n& =\\frac{\\sin \\left(\\frac{\\pi}{2}-C\\right)}{\\sin \\left(\\frac{\\pi}{2}-B\\right)} \\\\\n& =\\frac{\\cos C}{\\cos B},\n\\end{aligned}\n$$\n连结 $D E 、 D E^{\\prime} 、 D F 、 D F^{\\prime}$, 则 $\\angle D E^{\\prime} B=\\angle D F^{\\prime} A, \\angle D E B=\\angle D F A$, 所以 $\\triangle D E E^{\\prime} \\backsim \\triangle D F F^{\\prime}$, 于是 $\\frac{E E^{\\prime}}{F F^{\\prime}}=\\frac{D E}{D F}=\\frac{\\cos C}{\\cos B}$, 即 $E E^{\\prime} \\cdot \\cos B=F F^{\\prime} \\cdot \\cos C$, 故 $E F$ 在 $B C$ 上的投影长度 $=A E$ 在 $B C$ 上的投影长度 $+A F$ 在 $B C$ 上的投影长度\n$$\n\\begin{aligned}\n= & A E \\cdot \\cos B+A F \\cdot \\cos C=A E \\cdot \\cos B+A F \\\\\n& \\cdot \\cos C+E E^{\\prime} \\cdot \\cos B-F F^{\\prime} \\cdot \\cos C \\\\\n= & A E^{\\prime} \\cdot \\cos B+A F^{\\prime} \\cdot \\cos C \\\\\n= & E^{\\prime} F^{\\prime} \\text { 在 } B C \\text { 上的投影长度, 为与 } E 、 F \\text { 具体位 }\n\\end{aligned}\n$$\n$=E^{\\prime} F^{\\prime}$ 在 $B C$ 上的投影长度, 为与 $E 、 F$ 具体位置无关的常数.", + "remark": "", + "figures": [ + "./images/volume7/figures/fig-c4i6.png" + ] +} \ No newline at end of file diff --git a/processed_dataset/proof/1708.json b/processed_dataset/proof/1708.json new file mode 100644 index 0000000000000000000000000000000000000000..645f91625c443b451440d86eafebc7c8bf0c50fb --- /dev/null +++ b/processed_dataset/proof/1708.json @@ -0,0 +1,10 @@ +{ + "source_file": "./raw_volume-zh/volume7/chapter4.tex", + "problem_type": "proof", + "problem": "例6. 如图() 在 $\\triangle A B C$ 中, $\\angle B A C= 90^{\\circ}$, 点 $E$ 在 $\\triangle A B C$ 的外接圆 $\\Gamma$ 的弧 $B C$ (不含点 $A)$ 内, $A E>E C$, 连结 $E C$ 并延长至点 $F$, 使得 $\\angle E A C=\\angle C A F$, 连结 $B F$ 交圆 $\\Gamma$ 于点 $D$, 连结 $E D$, 记 $\\triangle D E F$ 的外心为 $O$, 求证 : $A 、 C 、 O$ 三点共线.", + "solution": "证明:作 $\\triangle A E F$ 的外接圆交 $A C$ 延长线于点 $O^{\\prime}$, 连结 $O^{\\prime} E 、 O^{\\prime} F$. 注意到 $A C$ 平分 $\\angle E A F$, 所以 $A O^{\\prime}$ 在圆 $A E F$ 内平分 $\\angle A$, 则 $O^{\\prime}$ 为 $\\overparen{E F}$ 的中点, $O^{\\prime} E=O^{\\prime} F$.\n由于 $\\angle E O^{\\prime} F=180^{\\circ}-\\angle E A F=180^{\\circ}-2 \\angle E A O^{\\prime}, \\label{eq1}$,\n$$\n\\angle E D B=\\angle E A B=90^{\\circ}-\\angle E A O, \\label{eq2}\n$$\n由 式\\ref{eq1}、\\ref{eq2} 知\n$$\n\\angle E D B=\\frac{1}{2} \\angle E O^{\\prime} F \\text {. }\n$$\n从而 $D$ 在以 $O^{\\prime}$ 为圆心, $O^{\\prime} E$ 为半径的圆上.\n所以 $O^{\\prime} D=O^{\\prime} E=O^{\\prime} F, O^{\\prime}$ 与 $O$ 重合, 于是 $A 、 C 、 O$ 共线.", + "remark": "", + "figures": [ + "./images/volume7/figures/fig-c4i7.png" + ] +} \ No newline at end of file diff --git a/processed_dataset/proof/1709.json b/processed_dataset/proof/1709.json new file mode 100644 index 0000000000000000000000000000000000000000..323c891c790d6345d628f83e621cec982a4c22ef --- /dev/null +++ b/processed_dataset/proof/1709.json @@ -0,0 +1,11 @@ +{ + "source_file": "./raw_volume-zh/volume7/chapter4.tex", + "problem_type": "proof", + "problem": "例7. 如图(), $M 、 N$ 分别为锐角 $\\triangle A B C (\\angle A<\\angle B)$ 的外接圆 $\\Gamma$ 上弧 $B C$ 、弧 $A C$ 的中点.\n过点 $C$ 作 $P C / / M N$ 交圆 $\\Gamma$ 于点 $P, I$ 为 $\\triangle A B C$ 的内心, 连结 $P I$ 并延长交圆 $\\Gamma$ 于 $T$.\n(1) 求证: $M P \\cdot M T=N P \\cdot N T$;\n(2) 在弧 $A B$ (不含点 $C)$ 上任取一点 $Q(\\neq A, T$, $B)$, 记 $\\triangle A Q C 、 \\triangle Q C B$ 的内心分别为 $I_1 、 I_2$. 求证 : $Q$ 、 $I_1 、 I_2 、 T$ 四点共圆.", + "solution": "证明:(1) $P C / / N M \\Rightarrow$ 等腰梯形 $P C M N$, 连结 $I M 、 I C 、 C M 、 A I 、 C N$, $I$ 为内心, 故 $A I$ 延长线过 $\\overparen{B C}$ 中点 $M$, 于是\n$$\n\\begin{aligned}\n\\angle C I M & =\\angle C A I+\\angle I C A \\\\\n& =\\angle B A I+\\angle I C A \\\\\n& =\\angle B A M+\\angle I C A \\\\\n& =\\angle B C M+\\angle I C B \\\\\n& =\\angle I C M,\n\\end{aligned}\n$$\n故 $I M=C M$, 又 $P C M N$ 为等腰梯形,有 $C M=P N$, 于是 $I M=N P$, 同理可证 $P M=I N$.\n由此可得四边形 $M I N P$ 为平行四边形, 即 $P I$ 平分 $M N$.\n所以 $T I$ 平分线段 $M N, S_{\\triangle P N T}=S_{\\triangle P M T} \\Rightarrow \\frac{1}{2} \\cdot P M \\cdot T M \\cdot \\sin \\angle P M T= \\frac{1}{2} \\cdot P N \\cdot T N \\cdot \\sin \\angle P N T$, 又 $\\angle P M T$ 与 $\\angle P N T$ 互补, $\\sin \\angle P M T= \\sin \\angle P N T$, 于是 $P M \\cdot T M=P N \\cdot T N$.\n(2)易知 $Q 、 I_1 、 N$ 共线; $Q 、 I_2 、 M$ 共线, 连结 $N Q 、 M Q 、 I_1 T 、 I_2 T$, 首先证明 $\\triangle I_1 N T \\backsim \\triangle I_2 M T$, 这是由于 $N I_1=N C . M I_2=M C$, 又 $\\frac{N I_1}{N T}=\\frac{N C}{N T}=\\frac{M P}{N T}= \\frac{N P}{M T}=\\frac{M C}{M T}=\\frac{M I_2}{M T}$,\n$\\angle I_1 N T=\\angle Q N T=\\angle Q M T=\\angle I_2 M T$, 由此可得 $\\triangle I_1 N T \\backsim \\triangle I_2 M T$, 从而有 $\\angle Q I_1 T=180^{\\circ}-\\angle N I_1 T=180^{\\circ}-\\angle M I_2 T=\\angle Q I_2 T$.\n于是, $Q 、 I_1 、 I_2 、 T$ 四点共圆.", + "remark": "注:1. (2)这道题多次在数学竟赛中出现, 是一道较难的问题,但是给出命题 (1)以后,两部分都不算太难.\n2. 如图(), $I$ 为 $\\triangle A B C$ 内心, $A I$ 与 $\\triangle A B C$ 外接圆交于 $D$, 则 $D B=D I=D C$, 本题用到了这个内心的重要性质, 有的人称之为\"鸡爪定理\".", + "figures": [ + "./images/volume7/figures/fig-c4i8.png", + "./images/volume7/figures/fig-c4i9.png" + ] +} \ No newline at end of file diff --git a/processed_dataset/proof/1710.json b/processed_dataset/proof/1710.json new file mode 100644 index 0000000000000000000000000000000000000000..5195aeced130067e1d6a62121f11ce3ccb2b83f4 --- /dev/null +++ b/processed_dataset/proof/1710.json @@ -0,0 +1,11 @@ +{ + "source_file": "./raw_volume-zh/volume7/chapter4.tex", + "problem_type": "proof", + "problem": "例8. 设 $L$ 在 $\\triangle A B C$ 的边 $B A$ 上,延长 $C A$ 至 $K$ 使 $\\angle C K B=\\frac{1}{2} \\angle C L B$, 延长 $C B$ 至 $M$ 使 $\\angle C M A=\\frac{1}{2} \\angle C L A$. 设 $\\triangle C M K$ 的外心为 $O$, 则 $O L \\perp A B$.", + "solution": "证明:如图(),作 $\\triangle C M K$ 外接圆 $W$, 设 $M A \\cap W=\\{S\\}, K B \\cap W=\\{R\\}$, 则 $\\angle C O S=2$ • $\\angle C M S=\\angle C L A, \\angle C O R=2 \\cdot \\angle C K R=\\angle C L B$. 从而 $\\angle C O S+\\angle C O R=\\angle C L A+\\angle C L B=180^{\\circ}$. 所以 $S, O 、 R$ 三点共线.\n对 $C 、 L 、 M 、 S 、 R 、 K$ 使用帕斯卡定理知 $C P \\cap S R=P, C P$ 表示过 $C$ 的圆 $W$ 的切线, $C M \\cap K R=B, C K \\cap S M=A$, 则 $P 、 B 、 A$ 三点共线.\n从而过 $C$ 的切线、SR、 $A B$ 交于点 $P$. 由 $\\angle O O R= \\angle C L B$ 知 $C 、 O 、 L 、 P$ 共圆.\n从而 $\\angle O L B=\\angle O L A=90^{\\circ}$, 即 $O L \\perp A B$. 注此题用到了帕斯卡定理:\n如图(), 对圆内接六边形 $A B C D E F$, 设 $A B \\cap D E=\\{X\\}, B C \\cap E F=\\{Y\\}, C D \\cap F A=\\{Z\\}$, 则 $X$ 、 $Y 、 Z$ 三点共线.\n这是一个很有用的结论.\n其证明见习题 2 第 16 题.", + "remark": "", + "figures": [ + "./images/volume7/figures/fig-c4i10.png", + "./images/volume7/figures/fig-c4i11.png" + ] +} \ No newline at end of file diff --git a/processed_dataset/proof/1711.json b/processed_dataset/proof/1711.json new file mode 100644 index 0000000000000000000000000000000000000000..2b3fd64440c47ed25077685628eab33c39fbc726 --- /dev/null +++ b/processed_dataset/proof/1711.json @@ -0,0 +1,10 @@ +{ + "source_file": "./raw_volume-zh/volume7/chapter4.tex", + "problem_type": "proof", + "problem": "例9. 如图(), 已知 $\\triangle A B C$ 内切圆 $\\odot I$ 分别与边 $A B 、 B C$ 切于点 $F 、 D$, 直线 $A D 、 C F$ 分别与 $\\odot I$ 交于另一点 $H 、 K$. 求证 : $\\frac{F D \\cdot H K}{F H \\cdot D K}=3$.", + "solution": "证明:设 $A F=x, B F=y, C D=z$. 由斯特瓦尔特定理得 $A D^2=\\frac{B D}{B C} \\cdot A C^2+\\frac{C D}{B} \\frac{D}{C} \\cdot A B^2- B D \\cdot D C=\\frac{y(x+z)^2+z(x+y)^2}{y+z}-y z=x^2+ \\frac{4 x y z}{y+z}$. 由切割线定理得: $A H=\\frac{A F^2}{A D}=\\frac{x^2}{A D}$. 故 $H D=A D-A H=\\frac{A D^2-x^2}{A D}=\\frac{4 x y z}{A D(y+z)}$.\n同理\n$$\nK F=\\frac{4 x y z}{C F(x+y)} .\n$$\n因为 $\\triangle C D K \\backsim \\triangle C F D$, 所以 $D K=\\frac{D F \\cdot C D}{C F}=\\frac{D F}{C F} \\cdot z$.\n又因为 $\\triangle A F H \\backsim \\triangle A D F$, 所以 $F H=\\frac{D F \\cdot A F}{A D}=\\frac{D F}{A D} \\cdot x$.\n由余弦定理得:\n$$\n\\begin{gathered}\nD F^2=B D^2+B F^2-2 B D \\cdot B F \\cos B \\\\\n=2 y^2\\left[1-\\frac{(y+z)^2+(x+y)^2-(x+z)^2}{2(x+y)}\\right] \\\\\n=\\frac{4 x y^2 z}{(x+y)(y+z)} . \\\\\n\\text { 故 } \\frac{K F \\cdot H \\cdot D}{F H \\cdot D K}=\\frac{\\frac{4 x y z}{C F(x+y)} \\cdot \\frac{4 x y z}{\\frac{A D(y+z)}{D F} \\cdot x \\cdot \\frac{D F}{C F} \\cdot z}}{\\frac{D D}{C D}} \\\\\n=\\frac{16 x y^2 z}{D F^2(x+y)(y+z)}=4\n\\end{gathered} \\label{eq1}\n$$\n对圆内接四边形 $D K H F$ 应用托勒密定理得\n$$\nK F \\cdot H D=D F \\cdot H K+F H \\cdot D K . \n$$\n再结合式\\ref{eq1}得\n$$\n\\frac{F D \\cdot H K}{F H \\cdot D K}=3 .\n$$", + "remark": "", + "figures": [ + "./images/volume7/figures/fig-c4i12.png" + ] +} \ No newline at end of file diff --git a/processed_dataset/proof/1712.json b/processed_dataset/proof/1712.json new file mode 100644 index 0000000000000000000000000000000000000000..31f6cbb4bdc551cd437d53e95ba9189bf8a10c64 --- /dev/null +++ b/processed_dataset/proof/1712.json @@ -0,0 +1,10 @@ +{ + "source_file": "./raw_volume-zh/volume7/chapter4.tex", + "problem_type": "proof", + "problem": "例10. 已知 $\\odot O_1$ 与 $\\odot O_2$ 外切于点 $T$, 一直线与 $\\odot O_2$ 相切于点 $X$, 与 $\\odot O_1$ 交于点 $A 、 B$, 且点 $B$ 在线段 $A X$ 的内部, 直线 $X T$ 与 $\\odot O_1$ 交于另一点 $S, C$ 是不包含点 $A 、 B$ 的 $\\overparen{T S}$ 上的一点, 过点 $C$ 作 $\\odot O_2$ 的切线, 切点为 $Y$, 且线段 $C Y$ 与线段 $S T$ 不相交, 直线 $S C$ 与 $X Y$ 交于点 $I$. 证明: (1) $C 、 T 、 I 、 Y$ 四点共圆; (2) $I$ 是 $\\triangle A B C$ 的 $\\angle A$ 内的旁切圆的圆心.", + "solution": "证明:(1) 如图(), $T$ 为 $\\odot O_1$ 与 $\\odot O_2$ 的公切点, 则 $\\overparen{S T}$ 对应的度数等于 $\\overparen{X T}$ 对应的度数, 即 $\\angle S A T=\\angle X Y T$ <1>.\n而 $\\angle T C I=\\angle S A T$, 所以 $\\angle T C I=\\angle X Y T$, 即 $\\angle I Y T=\\angle I C T$.\n故 $C 、 I 、 T 、 Y$ 四点共圆 <2>. \n(2) 因为 $\\angle S A T=\\angle S B T, \\angle A X S=\\angle X Y T$, 结合 <1> 式得\n$$\n\\angle S A T=\\angle A X S, \\angle S B T=\\angle A X S .\n$$\n又 $\\angle A S T=\\angle X S A, \\angle B S T=\\angle B S X$, 所以 $\\triangle S A T \\backsim \\triangle S X A, \\triangle S B T \\backsim \\triangle S X B$.\n则 $S A^2=S T \\cdot S X, S B^2=S T \\cdot S X$.\n又 $\\angle S I T=\\angle C Y T$ (由 <2> 得), $\\angle S I T=\\angle S X I$, 而 $\\angle I S T=\\angle X S I$, 所以 $\\triangle S I T \\backsim \\triangle S X I$, 从而 $S I^2=S T \\cdot S X$.\n则 $S A=S I=S B$, 即 $S$ 为 $\\triangle A B I$ 的外心.\n所以 $\\angle X B I=\\frac{1}{2} \\angle A S I=\\frac{1}{2} \\angle A S C=\\frac{1}{2} \\angle C B X$.\n故 $B I$ 平分 $\\angle C B X$, 而 $\\angle B C I=\\angle B A S$, 又 $S$ 为 $\\triangle A B I$ 的外心, 则 $\\angle B A S=-\\frac{1}{2} \\angle A S B+90^{\\circ}$, 则 $2 \\angle B A S+\\angle A S B-90^{\\circ}$.\n又 $\\angle B A S=\\angle B C I, \\angle A S B=\\angle A C B$, 即 $2 \\angle B C I+\\angle A C B=90^{\\circ}$.\n所以 $C I$ 是 $\\angle A C B$ 的外角平分线, 又 $B I$ 平分 $\\angle C B X$.\n故 $I$ 为 $\\triangle A B C$ 的 $\\angle A$ 内的旁切圆圆心.\n证毕.", + "remark": "", + "figures": [ + "./images/volume7/figures/fig-c4i13.png" + ] +} \ No newline at end of file diff --git a/processed_dataset/proof/1713.json b/processed_dataset/proof/1713.json new file mode 100644 index 0000000000000000000000000000000000000000..79b52af831418bafb7b30ee3908af23175574421 --- /dev/null +++ b/processed_dataset/proof/1713.json @@ -0,0 +1,10 @@ +{ + "source_file": "./raw_volume-zh/volume7/chapter5.tex", + "problem_type": "proof", + "problem": "例1. 如图(), $\\triangle A B C$ 中, $A B>B C$, 外接圆上点 $B$ 处的切线交直线 $A C$ 于点 $P, D$ 是 $B$ 点关于点 $P$ 的对称点, $E$ 是点 $C$ 关于直线 $B P$ 的对称点.\n求证: 四边形 $A B E D$ 是圆内接四边形.", + "solution": "证明:由于 $P B$ 是切线, 由切割线定理\n$$\nP A \\cdot P C=P B^2=P D^2 \\Rightarrow \\frac{P A}{P D}=\\frac{P D}{P C},\n$$\n又 $\\angle D P A=\\angle C P D$, 所以 $\\triangle D P A \\backsim \\triangle C P D . \\Rightarrow \\angle C A D=\\angle P D C$.\n再由 $P B$ 是切线知, $\\angle P B C=\\angle B A C$.\n故 $\\angle B A D=\\angle P B C+\\angle P D C=180^{\\circ}-\\angle B C D$.\n再根据 $C 、 E$ 的对称性知, $\\angle B C D=\\angle B E D$.\n于是, $\\angle B A D+\\angle B E D=180^{\\circ}$.\n从而, 四边形 $A B E D$ 是圆内接四边形.", + "remark": "", + "figures": [ + "./images/volume7/figures/fig-c5i2.png" + ] +} \ No newline at end of file diff --git a/processed_dataset/proof/1714.json b/processed_dataset/proof/1714.json new file mode 100644 index 0000000000000000000000000000000000000000..75983af4b486cb85217ee60f792527ad4df45dae --- /dev/null +++ b/processed_dataset/proof/1714.json @@ -0,0 +1,11 @@ +{ + "source_file": "./raw_volume-zh/volume7/chapter5.tex", + "problem_type": "proof", + "problem": "例2. 如图(), 圆 $\\Gamma$ 与 $\\triangle A B C$ 的外接圆相切于点 $A$, 与边 $A B$ 交于点 $K$, 且和边 $B C$ 相交.\n过点 $C$ 作圆 $\\Gamma$ 的切线, 切点为 $L$, 连结 $K L$, 交边 $B C$ 于点 $T$. 求证: 线段 $B T$ 的长等于点 $B$ 到圆 $\\Gamma$ 的切线长.", + "solution": "分析:由圆幂定理知: 点 $B$ 到圆 $\\Gamma$ 的切线长的平方等于 $B K \\cdot B A$. 故问题等价于求证: $B T^2=B K$ ・ $B A$.\n证明过 $A$ 作两圆的公切线 $D E$, 连结 $A C$ 交圆 $\\Gamma$ 于 $M$.\n则 $\\angle D A B=\\angle A C B=\\angle A M K \\Rightarrow K M / / B C$.\n注意到 $A 、 K 、 L 、 M$ 四点共圆, 所以 $\\angle A M K=\\angle A L K=\\angle A C B \\Rightarrow A$ 、 $C 、 L 、 T$ 四点共圆,所以 $\\angle A L C=\\angle A T C$.\n又因 $C L$ 是圆 $\\Gamma$ 的切线, $\\angle A L C=\\angle A K L$.\n所以 $\\angle A T C=\\angle A K L$, 从而它们的补角也相等, 即 $\\angle B K T=\\angle B T A$. 又 $\\angle K B T=\\angle T B A$, 故 $\\triangle A B T \\backsim \\triangle T B K \\Rightarrow B T^2==B K \\cdot B A$.\n由圆幂定理知, $B T$ 的长等于 $B$ 到圆 $\\Gamma$ 的切线长.", + "remark": "注:若两圆相内切于点 $A$, 过 $A$ 出发的两条射线与两圆分别交于 $B_1 、 C_1$ 和 $B_2 、 C_2$, 则 $B_1 C_1 / / B_2 C_2$. 这个基本结论常用到.\n(如图())", + "figures": [ + "./images/volume7/figures/fig-c5i3.png", + "./images/volume7/figures/fig-c5i4.png" + ] +} \ No newline at end of file diff --git a/processed_dataset/proof/1715.json b/processed_dataset/proof/1715.json new file mode 100644 index 0000000000000000000000000000000000000000..ec39773af7eb49f0bcf4d8fb08a3084f878ff736 --- /dev/null +++ b/processed_dataset/proof/1715.json @@ -0,0 +1,10 @@ +{ + "source_file": "./raw_volume-zh/volume7/chapter5.tex", + "problem_type": "proof", + "problem": "例3. 如图(), 圆内接四边形 $A B C D$ 中, $A D=A B$. 求证: $A B^2+B C C D=A C^2$.", + "solution": "证明:将 $\\triangle A C D$ 绕着 $A$ 点逆时针旋转, 使 $A D$ 与 $A B$ 重合.\n由 $A D=A B$, 且 $A B C D$ 是圆内接四边形知: $C 、 B 、 E$ 三点共线.\n以 $A$ 为圆心, $A C$ 为半径作圆, 考虑 $B$ 对此圆的幂: 一方面, 这个幂是 $B A^2-C A^2$; 另一方面, 它也是 $\\overrightarrow{B E} \\cdot \\overrightarrow{B C}$, 从而 $A C^2-A B^2=B E \\cdot B C$ (这里是线段的长度 $)=C D \\cdot B C \\Rightarrow A B^2+B C \\cdot C D=A C^2$.", + "remark": "注:此题的证法很多,也可通过相似的方法去证明.\n留给读者思考.", + "figures": [ + "./images/volume7/figures/fig-c5i5.png" + ] +} \ No newline at end of file diff --git a/processed_dataset/proof/1716.json b/processed_dataset/proof/1716.json new file mode 100644 index 0000000000000000000000000000000000000000..5ae353e20ef257cb24137cef8c22be498414fe26 --- /dev/null +++ b/processed_dataset/proof/1716.json @@ -0,0 +1,10 @@ +{ + "source_file": "./raw_volume-zh/volume7/chapter5.tex", + "problem_type": "proof", + "problem": "例4. 如图(), $\\odot O_1$ 与 $\\odot O_2$ 相交于点 $C 、 D$, 过点 $D$ 的一条直线分别与 $\\odot O_1 、 \\odot O_2$ 相交于点 $A 、 B$, 点 $P$ 在 $\\odot O_1$ 的弧 $A D$ 上, $P D$ 与线段 $A C$ 的延长线交于点 $M$, 点 $Q$ 在 $\\odot O_2$ 的弧 $B D$ 上, $Q D$ 与线段 $B C$ 的延长线交于点 $N . O$ 是 $\\triangle A B C$ 的外心.\n求证: $O D \\perp M N$ 的充要条件是 $P 、 Q 、 M 、 N$ 四点共圆.", + "solution": "证明:设 $\\triangle A B C$ 的外接圆 $O$ 的半径为 $R$, 则 $M, N$ 对 $\\odot O$ 的幂分别为\n$$\n\\begin{aligned}\n& M O^2-R^2=M C \\cdot M A, \\label{eq1} \\\\\n& N O^2-R^2=N C \\cdot N B . \\label{eq2}\n\\end{aligned}\n$$\n又因为 $A 、 C 、 D 、 P$ 四点共圆, 所以\n$$\nM C \\cdot M A=M D \\cdot M P, \\label{eq3}\n$$\n同理 $Q 、 D 、 C 、 B$ 四点共圆, 所以\n$$\nN C \\cdot N B=N D \\cdot N Q . \\label{eq4}\n$$\n由式\\ref{eq1}、\\ref{eq2}、式\\ref{eq3}、\\ref{eq4}得\n$$\n\\begin{aligned}\nN C^2-M O^2 & =N D \\cdot N Q-M D \\cdot M P \\\\\n& =N D \\cdot(N D+D Q)-M D \\cdot(M D+D P) \\\\\n& =N D^2-M D^2+(N D \\cdot D Q-M D \\cdot D P) .\n\\end{aligned}\n$$\n所以, $O D \\perp M N \\Leftrightarrow N O^2-M O^2=N D^2-M D^2 \\Leftrightarrow N D \\cdot D Q=M D$ • $D P \\Leftrightarrow P 、 Q 、 M 、 N$ 四点共圆.", + "remark": "注:上面证题的过程中, 用到了这样一个结论: $O D \\perp M N \\Leftrightarrow N O^2- M O^2=N D^2-M D^2$. 它是一个非常重要的结论, 在证明垂直一类问题中常用到它.", + "figures": [ + "./images/volume7/figures/fig-c5i6.png" + ] +} \ No newline at end of file diff --git a/processed_dataset/proof/1717.json b/processed_dataset/proof/1717.json new file mode 100644 index 0000000000000000000000000000000000000000..7df0ee6d2a389f468eda5948f6ad83ecbe1e2a4b --- /dev/null +++ b/processed_dataset/proof/1717.json @@ -0,0 +1,10 @@ +{ + "source_file": "./raw_volume-zh/volume7/chapter5.tex", + "problem_type": "proof", + "problem": "例5. 如图(), 从半圆上的一点 $C$ 向直径 $A B$ 引垂线, 设垂足为 $D$, 作圆 $O_1$ 分别切 $\\overparen{B C} 、 C D 、 D B$ 于点 $E 、 F 、 G$. 求证 : $A C=A G$.", + "solution": "证明:设半圆的圆心为 $O$, 则 $O 、 O_1 、 E$ 三点共线.\n连结 $O_1 F$ 知 $O_1 F \\perp C D$, 且 $O_1 F / / A B$, 连结 $E F 、 A E$.\n由 $\\angle F E O_1=\\frac{1}{2} \\angle F O_1 O=\\frac{1}{2} \\angle E O B=\\angle O E A \\Rightarrow E 、 F 、 A$ 三点共线.\n又因为 $\\angle A C B=90^{\\circ}$, 且 $C D \\perp A B$, 所以 $\\angle A C F=\\angle A B C=\\angle A E C$.\n从而 $A C$ 是 $\\triangle C E F$ 外接圆的切线, 故点 $A$ 对 $\\triangle C E F$ 外接圆的幕 $A C^2$ 等于点 $A$ 对 $\\odot O_1$ 的幕 $A G^2$ (也等于 $A E \\cdot A F$ ), 即 $A C=A G$.\n上面几道例题都是和幕相关的问题, 以下的问题都和根轴有关.", + "remark": "", + "figures": [ + "./images/volume7/figures/fig-c5i7.png" + ] +} \ No newline at end of file diff --git a/processed_dataset/proof/1718.json b/processed_dataset/proof/1718.json new file mode 100644 index 0000000000000000000000000000000000000000..e8b4ff40e3a064cf9dcfc7e12afe1890b1684eb3 --- /dev/null +++ b/processed_dataset/proof/1718.json @@ -0,0 +1,10 @@ +{ + "source_file": "./raw_volume-zh/volume7/chapter5.tex", + "problem_type": "proof", + "problem": "例6. 如图(), 设 $D 、 E$ 是 $\\triangle A B C$ 中 $A B 、 A C$ 上的点.\n求证: 以 $B E 、 C D$ 为直径的两圆的根轴必通过 $\\triangle A B C$ 的垂心.", + "solution": "证明:设以 $B E$ 为直径的圆为 $\\odot O_1$, 以 $C D$ 为直径的圆为 $\\odot O_2, B M 、 C N$ 是高线, $H$ 为垂心.\n显然 $M$ 在 $\\odot O_1$ 上, $N$ 在 $\\odot O_2$ 上.\n又因 $B 、 C 、 M 、 N$ 四点共圆, 所以 $H B \\cdot H M= H C \\cdot H N$.\n而 $H B \\cdot H M$ 是 $H$ 对 $\\odot O_1$ 的幕, $H C \\cdot H N$ 是 $H$ 对 $\\odot \\mathrm{O}_2$ 的幂.\n由根轴定理知: $H$ 在它们的根轴上, 即以 $B E 、 C D$ 为直径的两圆的根轴通过 $\\triangle A B C$ 的垂心.", + "remark": "", + "figures": [ + "./images/volume7/figures/fig-c5i8.png" + ] +} \ No newline at end of file diff --git a/processed_dataset/proof/1719.json b/processed_dataset/proof/1719.json new file mode 100644 index 0000000000000000000000000000000000000000..2ed653c8faef5bba915c2ffccb6d9f4df8099c6d --- /dev/null +++ b/processed_dataset/proof/1719.json @@ -0,0 +1,11 @@ +{ + "source_file": "./raw_volume-zh/volume7/chapter5.tex", + "problem_type": "proof", + "problem": "例7. 如图(), 已知两个半径不相等的圆 $O_1$ 与圆 $O_2$ 相交于 $M 、 N$ 两点, 且 $\\odot O_1$ 与 $\\odot O_2$ 分别与 $\\odot O$ 内切于 $S 、 T$ 两点.\n求证: $O M \\perp M N$ 的充分必要条件是 $S 、 N 、 T$ 三点共线.", + "solution": "证明:如图(), 连结 $O S 、 O T 、 S T 、 S M$, 作公切线 $S P 、 T P$, 由根轴定理知, $P 、 M 、 N$ 三点共线.\n又 $\\angle O S P=\\angle O T P=90^{\\circ}$, 所以 $O 、 S 、 P 、 T$ 四点共圆.\n$O M \\perp M N \\Leftrightarrow \\angle O M P=\\angle O S P=90^{\\circ} \\Leftrightarrow O 、 M 、 T 、 P 、 S$ 五点共圆.\n注意到 $S P 、 T P$ 为切线, $\\angle N S P=\\angle S M P, \\angle N T P=\\angle T M P$.\n故 $O 、 M 、 T 、 P 、 S$ 共圆 $\\Leftrightarrow \\angle S M T+\\angle S P T=180^{\\circ} \\Leftrightarrow \\angle S M P+\\angle T M P+ \\angle S P T=180^{\\circ} \\Leftrightarrow \\angle S P T+\\angle P S N+\\angle P T N=180^{\\circ} \\Leftrightarrow S 、 N 、 T$ 共线.", + "remark": "", + "figures": [ + "./images/volume7/figures/fig-c5i9.png", + "./images/volume7/figures/fig-c5i9.png" + ] +} \ No newline at end of file diff --git a/processed_dataset/proof/1720.json b/processed_dataset/proof/1720.json new file mode 100644 index 0000000000000000000000000000000000000000..3f6e20302a80a4e5b45e7d8b3b257793fe7e7006 --- /dev/null +++ b/processed_dataset/proof/1720.json @@ -0,0 +1,10 @@ +{ + "source_file": "./raw_volume-zh/volume7/chapter5.tex", + "problem_type": "proof", + "problem": "例8. 设 $O$ 和 $I$ 分别为 $\\triangle A B C$ 的外心和内心, $\\triangle A B C$ 的内切圆与边 $B C 、 C A 、 A B$ 分别相切于点 $D 、 E 、 F$, 直线 $F D$ 和 $C A$ 相交于点 $P$, 直线 $D E$ 与 $A B$ 相交于点 $Q$, 点 $M, N$ 分别为线段 $P E 、 Q F$ 的中点.\n求证: $O I \\perp M N$.", + "solution": "证明:如图(), 考虑 $\\triangle A B C$ 与截线 $P F D$. 由梅氏定理:\n$$\n\\frac{C P}{P A} \\cdot \\frac{A F}{F B} \\cdot \\frac{B D}{D C}=1 \\Rightarrow \\frac{P A}{C P}=\\frac{A F}{D C}=\\frac{A F}{E C} .\n$$\n记 $\\triangle A B C$ 的三边分别为 $a 、 b 、 c$, 令 $p= \\frac{a+b+c}{2}$,并不妨设 $a>c$, 则\n$$\n\\frac{P A}{C P}=\\frac{P A}{C A+P A}=\\frac{P A}{P A+b}=\\frac{p-a}{p-c}, P A=\\frac{(p-a) b}{a-c} .\n$$\n$$\n\\begin{aligned}\n& \\text { 而 } P E=P A+A E=\\frac{(p-a) b}{a-c}+p-a=\\frac{2(p-a)(p-c)}{a-c}, \\\\\n& M E=\\frac{1}{2} P E=\\frac{(p-a)(p-c)}{a-c}, \\\\\n& M A=M E-A E=\\frac{(p-a)(p-c)}{a-c}-(p-a)=\\frac{(p-a)^2}{a-c}, \\\\\n& M C=M E+E C=\\frac{(p-a)(p-c)}{a-c}+(p-c)=\\frac{(p-c)^2}{a-c} .\n\\end{aligned}\n$$\n于是 $M A \\cdot M C=M E^2$.\n由于 $M E$ 是 $M$ 到 $\\triangle A B C$ 内切圆切线长, $M E^2$ 是点 $M$ 到内切圆 $I$ 的幂, 而 $M A \\cdot M C$ 是 $M$ 到 $\\triangle A B C$ 外接圆 $O$ 的幂.\n等式 \" $M A \\cdot M C=M E^2$ \" 表示点 $M$ 到 $\\triangle A B C$ 外接圆与内切圆的幕相等, 因而点 $M$ 在 $\\triangle A B C$ 外接圆 $O$ 与内切圆 $I$ 的根轴上.\n同理, 点 $N$ 也在 $\\triangle A B C$ 的外接圆 $O$ 与内切圆 $I$ 的根轴上, 由根轴定理知 $O I \\perp M N$.", + "remark": "", + "figures": [ + "./images/volume7/figures/fig-c5i10.png" + ] +} \ No newline at end of file diff --git a/processed_dataset/proof/1721.json b/processed_dataset/proof/1721.json new file mode 100644 index 0000000000000000000000000000000000000000..4309250a0f302c24f9822c26d6336d5d7a0b3c5b --- /dev/null +++ b/processed_dataset/proof/1721.json @@ -0,0 +1,10 @@ +{ + "source_file": "./raw_volume-zh/volume7/chapter5.tex", + "problem_type": "proof", + "problem": "例9. 如图(), 以 $O$ 为圆心的圆通过 $\\triangle A B C$ 的两个顶点 $A 、 C$, 且与 $A B 、 B C$ 两边分别相交于 $K$ 、 $N$ 两点, $\\triangle A B C$ 和 $\\triangle K B N$ 的两外接圆交于 $B 、 M$ 两点.\n证明: $\\angle O M B$ 为直角.", + "solution": "证明:设 $\\triangle A B C 、 \\triangle B K N$ 的外接圆圆心分别为 $O_1 、 O_2$, 由题设推知 $O 、 O_1 、 O_2$ 三点不共线 (否则 $B$ 和 $M$ 重合), 而直线 $A C 、 K N 、 B M$ 分别为这三个圆中两两圆的根轴, 故它们必相交于一点, 不妨设交于点 $P$.\n由 $\\angle P M N=\\angle B K N=\\angle N C A$, 知 $P 、 M 、 N 、 C$ 四点共圆, 故点 $B$ 对此圆的幂等于点 $B$ 对 $\\odot O$ 的幂.\n设 $R$ 为 $\\odot O$ 的半径, 则有\n$$\nB M \\cdot B P=B N \\cdot B C=B O^2-R^2 . \\label{eq1}\n$$\n又点 $P$ 对 $\\odot O_2$ 的幕等于点 $P$ 对 $\\odot O$ 的幕, 即\n$$\nP M \\cdot P B=P N \\cdot P K=P O^2-R^2 . \\label{eq2}\n$$\n式\\ref{eq2} - \\ref{eq1}得 $P O^2-B O^2=B P(P M-B M)$\n$$\n=(P M+B M)(P M-B M)=P M^2-B M^2 .\n$$\n故 $O M \\perp B P, \\angle B M O=90^{\\circ}$.", + "remark": "", + "figures": [ + "./images/volume7/figures/fig-c5i11.png" + ] +} \ No newline at end of file diff --git a/processed_dataset/proof/1722.json b/processed_dataset/proof/1722.json new file mode 100644 index 0000000000000000000000000000000000000000..e9308d0a2165703aee60f71cffe18391ceba73eb --- /dev/null +++ b/processed_dataset/proof/1722.json @@ -0,0 +1,10 @@ +{ + "source_file": "./raw_volume-zh/volume7/chapter5.tex", + "problem_type": "proof", + "problem": "例10. 如图(), 设圆 $O_1$ 和圆 $O_2$ 相离, 引它们的一条外公切线切圆 $O_1$ 于 $A$, 切圆 $O_2$ 于 $C$,引它们的一条内公切线切圆 $O_1$ 于 $B$, 切圆 $O_2$ 于 $D$, 求证: 直线 $A B$ 和 $C D$ 的交点在两圆的连心线上.", + "solution": "证明:设 $A B$ 和 $C D$ 的交点为 $K, A C$ 与 $B D$ 的交点为 $E$, 连结 $O_1 E$, 则 $A B \\perp O_1 E$,$\\mathrm{CD} \\perp \\mathrm{O}_2 E$.\n由于 $O_1 E$ 平分 $\\angle A E B, O_2 E$ 平分 $\\angle C E D$, 所以 $O_1 E \\perp O_2 E$, 且 $A B \\perp C D$, 即 $K$ 是分别以 $A C$ 和 $B D$ 为直径的两圆 $S_1$ 和 $S_2$ 的交点.\n所以 $K$ 在圆 $S_1$ 和圆 $S_2$ 的根轴上.\n下面证明 $O_1 O_2$ 是圆 $S_1$ 和圆 $S_2$ 的根轴.\n因 $O_1 A \\perp A C$, 所以 $O_1 A$ 是圆 $S_1$ 的切线, $O_1$ 关于圆 $S_1$ 的幂是 $O_1 A^2$.\n同理, $O_1 B$ 是圆 $S_2$ 的切线, $O_1$ 关于圆 $S_2$ 的幂是 $O_1 B^2$.\n由于 $O_1 A^2=O_1 B^2$, 所以 $O_1$ 是关于圆 $S_1$ 和 $S_2$ 的等幕点.\n同理, $O_2$ 是关于圆 $S_1$ 和圆 $S_2$ 的等幕点, 故 $O_1 O_2$ 是圆 $S_1$ 和圆 $S_2$ 的根轴.\n于是, $K$ 在连心线 $O_1 O_2$ 上.", + "remark": "", + "figures": [ + "./images/volume7/figures/fig-c5i12.png" + ] +} \ No newline at end of file diff --git a/processed_dataset/proof/1723.json b/processed_dataset/proof/1723.json new file mode 100644 index 0000000000000000000000000000000000000000..e8512977ff6000e987019ecdbf83f6d774c73538 --- /dev/null +++ b/processed_dataset/proof/1723.json @@ -0,0 +1,10 @@ +{ + "source_file": "./raw_volume-zh/volume7/chapter6.tex", + "problem_type": "proof", + "problem": "例1. 已知六边形 $A C_1 B A_1 C B_1$ 中, $A C_1=A B_1, B C_1=B A_1, C A_1= C B_1, \\angle A+\\angle B+\\angle C=\\angle A_1+\\angle B_1+\\angle C_1$.\n求证: $\\triangle A B C$ 面积是六边形 $A C_1 B A_1 C B_1$ 的一半.", + "solution": "证明:如图(), 旋转 $\\triangle B C A_1$ 至 $B A_1^{\\prime} C_1$, 则 $\\triangle B C A_1 \\cong \\triangle B A_1^{\\prime} C_1$, 显然 $\\angle A C_1 B+\\angle B A_1 C+ \\angle C B_1 A=360^{\\circ}, \\angle A C_1 B+\\angle B C_1 A_1^{\\prime}+\\angle A C_1 A_1^{\\prime}= 360^{\\circ}$, 所以 $\\angle A C_1 B=\\angle A C_1 A_1^{\\prime}$.\n又 $C_1 A_1^{\\prime}=C A_1=B_1 C, A C_1=A B_1$, 则 $\\triangle A C_1 A_1^{\\prime} \\cong \\triangle A B_1 C$.\n又 $A A_1^{\\prime}=A C, A_1^{\\prime} B=B C, A B=A B$, 所以 $\\triangle A B C \\cong \\triangle A B A_1^{\\prime}$.\n故 $S_{\\triangle A B C}=S_{\\triangle A B A_1^{\\prime}}=S_{\\triangle A C_1 B}+S_{\\triangle A B_1 C}+S_{\\triangle B C A_1} \\Rightarrow S_{\\triangle A B C}=\\frac{1}{2} S_{A C_1 B A_1 C B_1}$.", + "remark": "", + "figures": [ + "./images/volume7/figures/fig-c6i1.png" + ] +} \ No newline at end of file diff --git a/processed_dataset/proof/1724.json b/processed_dataset/proof/1724.json new file mode 100644 index 0000000000000000000000000000000000000000..f3aca1572e0edf5fc661732be21988fef27efb4a --- /dev/null +++ b/processed_dataset/proof/1724.json @@ -0,0 +1,10 @@ +{ + "source_file": "./raw_volume-zh/volume7/chapter6.tex", + "problem_type": "proof", + "problem": "例2. $P$ 是平行四边形 $A B C D$ 内一点, 且 $\\angle P A B=\\angle P C B$. 求证: $\\angle P B A=\\angle P D A$.", + "solution": "证明:将 $\\triangle A B P$ 沿向量 $\\overrightarrow{A D}$ 平移至 $D C P^{\\prime}$, 如图() 设角, 则四边形 $A P P^{\\prime} D$ 和四边形 $B P P^{\\prime} C$ 均为平行四边形, $\\angle 8=\\angle 2=\\angle 1= \\angle 5$, 于是四边形 $P D P^{\\prime} C$ 为圆内接四边形, 因此, $\\angle 4=\\angle 7=\\angle 6=\\angle 3$.\n即 $\\angle P B A=\\angle P D A$.", + "remark": "", + "figures": [ + "./images/volume7/figures/fig-c6i2.png" + ] +} \ No newline at end of file diff --git a/processed_dataset/proof/1725.json b/processed_dataset/proof/1725.json new file mode 100644 index 0000000000000000000000000000000000000000..95984ed72fbc136950d963e6e430b25e82fe5491 --- /dev/null +++ b/processed_dataset/proof/1725.json @@ -0,0 +1,10 @@ +{ + "source_file": "./raw_volume-zh/volume7/chapter6.tex", + "problem_type": "proof", + "problem": "例3. Fagnano 问题: 设 $\\triangle D E F$ 的三顶点分别在 $\\triangle A B C$ 的三边上, 则 $\\triangle D E F$ 称为 $\\triangle A B C$ 的内接三角形.\n证明 : 在锐角三角形的所有内接三角形中, 垂足三角形的周长最短.", + "solution": "证明:如图() 所示, 首先以 $A B$ 为轴将 $\\triangle A B C$ 反射为 $\\triangle A B C_1$, 再以 $B C_1$ 为轴将 $\\triangle A B C_1$ 反射为 $\\triangle A_1 B C_1$, 再以 $A_1 C_1$ 为反射轴反射成 $\\triangle A_1 B_1 C_1$, 如此类推, 并设 $E$ 最终被反射成 $E^{\\prime}$, 设 $\\triangle D E F$ 的三边分别为 $d 、 e 、 f$, 不难由反射变换保距离知, 图中标注的几条边长分别为 $d 、 e 、 f 、 d 、 e 、 f$, 于是 $2(d+ e+f) \\geqslant E E^{\\prime}$, 不难知道 $A C / / A_2 C_2$, 于是 $E E^{\\prime}$ 是与 $D 、 E 、 F$ 无关的只与三角形本身有关的常数, 另一方面, 若 $\\triangle D E F$ 为垂足三角形, 则有 $\\angle D F B= \\angle E F A$ 等等, 于是 $E E^{\\prime}$ 折线上六点共线, 取到等号.\n所以垂足三角形的周长 $=\\frac{E E^{\\prime}}{2}$ 是所有内接三角形中周长最短的.", + "remark": "", + "figures": [ + "./images/volume7/figures/fig-c6i3.png" + ] +} \ No newline at end of file diff --git a/processed_dataset/proof/1726.json b/processed_dataset/proof/1726.json new file mode 100644 index 0000000000000000000000000000000000000000..657e446941525adba6b6afb78069c118774acbf9 --- /dev/null +++ b/processed_dataset/proof/1726.json @@ -0,0 +1,10 @@ +{ + "source_file": "./raw_volume-zh/volume7/chapter6.tex", + "problem_type": "proof", + "problem": "例4. 已知点 $A 、 B 、 C$ 在某平面上.\n设 $D 、 E 、 F 、 G 、 H 、 I$ 是同一平面上的点,且使得 $\\triangle A B D 、 \\triangle B A E 、 \\triangle C A F 、 \\triangle D F G 、 \\triangle E C H 、 \\triangle G H I$ 为正定向等边三角形.\n证明: 点 $E$ 是线段 $A I$ 的中点.", + "solution": "证明:如图() 所示, 连结 $C G 、 E I$ 在 $\\triangle A D F$ 和 $\\triangle C G F$ 中, 有 $A F=C F, D F=G F$, 又 $\\angle D F G= \\angle A F C=60^{\\circ}$, 于是, 绕点 $F$ 顺时针旋转 $60^{\\circ}, \\triangle A D F$ 变换为 $\\triangle C G F$.\n类似地, 绕点 $H$ 顺时针旋转 $60^{\\circ}$ 的几何变换中, $\\triangle H C G$ 变为 $\\triangle H E I$.\n又绕 $A$ 顺时针旋转 $120^{\\circ}$, 线段 $A D$ 变为线段 $A E$, 所以, $A E=A D=C G=E I$, 且 $A E$ 和 $E I$ 与 $A D$ 的夹角都等于 $120^{\\circ}$, 即 $A 、 E 、 I$ 三点共线, 综上,点 $E$ 是线段 $A I$ 的中点.", + "remark": "", + "figures": [ + "./images/volume7/figures/fig-c6i4.png" + ] +} \ No newline at end of file diff --git a/processed_dataset/proof/1727.json b/processed_dataset/proof/1727.json new file mode 100644 index 0000000000000000000000000000000000000000..681a53b750a0a69df42cfde22551da9e0725c311 --- /dev/null +++ b/processed_dataset/proof/1727.json @@ -0,0 +1,11 @@ +{ + "source_file": "./raw_volume-zh/volume7/chapter6.tex", + "problem_type": "proof", + "problem": "例5. 如图(), 以 $B_0 、 B_1$ 为焦点的椭圆与 $\\triangle A B_0 B_1$ 的边 $A B_i$ 交于 $C_i(i=0,1)$. 在 $A B_0$ 的延长线上任取点 $P_0$, 以 $B_0$ 为圆心、 $B_0 P_0$ 为半径作圆弧 $\\overparen{P_0 Q_0}$ 交 $C_1 B_0$ 的延长线于点 $Q_0$; 以 $C_1$ 为圆心 $C_1 Q_0$ 为半径作圆弧 $\\overparen{Q_0 P_1}$ 交 $B_1 A$ 的延长线于点 $P_1$; 以 $B_1$ 为圆心 $B_1 P_1$ 为半径作圆弧 $\\overparen{P_1 Q_1}$ 交 $B_1 C_0$ 的延长线于点 $Q_1$; 以 $C_0$ 为圆心、 $C_0 Q_1$ 为半径作圆弧 $Q_1 P_0^{\\prime}$ 交 $A B_0$ 的延长线于 $P_0^{\\prime}$. 求证:\n(1) 点 $P_0^{\\prime}$ 与点 $P_0$ 重合, 且圆弧 $\\overparen{P_0 Q_0}$ 与 $\\overparen{P_0 Q_1}$ 相内切于点 $P_0$ ;\n(2) $P_0 、 Q_0 、 Q_1 、 P_1$ 四点共圆.", + "solution": "证明:如图(), $\\angle Q_0 B_0 P_0$ 的角平分线与 $\\angle A C_1 B_0$ 的角平分线的交点 $O$ 即为由点 $P_0$ 到点 $P_1$ 的旋转变换的旋转中心, 旋转角度为 $\\angle P_0 B_0 Q_0+ \\angle Q_0 C_1 P_1$, 且 $O P_0=O P_1$.\n同理, $\\angle P_1 B_1 Q_1$ 的角平分线与 $\\angle Q_1 C_0 P_0^{\\prime}$ 的角平分线的交点 $O^{\\prime}$ 即为由点 $P_1$ 到点 $P_0^{\\prime}$ 的旋转变换的旋转中心, 旋转角度为 $\\angle P_1 B_1 Q_1+\\angle Q_1 C_0 P_0^{\\prime}$, 且\n$O^{\\prime} P_1=O^{\\prime} P_0^{\\prime}$.\n于是, 有 $\\angle P_0 B_0 Q_0+\\angle Q_0 C_1 P_1=\\pi-\\angle A=\\angle P_1 B_1 Q_1+\\angle Q_1 C_0 P_0^{\\prime}$.\n设点 $O 、 O^{\\prime}$ 在 $A B_1$ 上的投影分别为 $D 、 D^{\\prime}$, 则 $A D=\\frac{A C_1+A B_0-B_0 C_1}{2}$, $A D^{\\prime}=\\frac{A B_1+A C_0-B_1 C_0}{2}$.\n由于 $B_1 C_1+B_0 C_1=B_1 C_0+B_0 C_0$, 所以, $A D=A D^{\\prime}$, 即 $D$ 与 $D^{\\prime}$ 重合.\n又因为点 $O 、 O^{\\prime}$ 均在 $\\angle B_1 A B_0$ 的角平分线上, 所以 $O$ 与 $O^{\\prime}$ 重合.\n从而点 $P_0$ 与点 $P_0^{\\prime}$ 重合, 圆弧 $\\overparen{P_0 Q_0}$ 与 $\\overparen{P_0 Q_1}$ 相内切于点 $P_0$, 且 $P_0 、 Q_0$ 、 $Q_1 、 P_1$ 四点共圆.", + "remark": "", + "figures": [ + "./images/volume7/figures/fig-c6i5.png", + "./images/volume7/figures/fig-c6i5.png" + ] +} \ No newline at end of file diff --git a/processed_dataset/proof/1728.json b/processed_dataset/proof/1728.json new file mode 100644 index 0000000000000000000000000000000000000000..a6dbd65f059651cacd66ded064f70b08377e1d2a --- /dev/null +++ b/processed_dataset/proof/1728.json @@ -0,0 +1,10 @@ +{ + "source_file": "./raw_volume-zh/volume7/chapter6.tex", + "problem_type": "proof", + "problem": "例6. 一个以点 $O$ 为圆心的圆经过 $\\triangle A B C$ 的顶点 $A 、 C$, 又与边 $A B 、 B C$ 分别相交于点 $K 、 N, \\triangle A B C$ 与 $\\triangle K B N$ 的外接圆交于点 $B 、 M$. 求证: $\\angle O M B=90^{\\circ}$.", + "solution": "证明:如图(), 设过点 $O$ 且垂直于 $B M$ 的直线为 $l$.\n于是, 只需证点 $M$ 在直线 $l$ 上.\n以 $l$ 为反射轴, 作轴反射变换 $S(l)$.\n设 $C \\rightarrow C^{\\prime}, K \\rightarrow K^{\\prime}$.\n则 $C C^{\\prime} \\perp l, K K^{\\prime} \\perp l$.\n所以, $C C^{\\prime} / / K K^{\\prime} / / B M$.\n连结 $C^{\\prime} K 、 K M 、 C K^{\\prime} 、 C M 、 C C^{\\prime}$.\n又 $\\angle K C^{\\prime} C=\\angle K A C=\\angle B N K=\\angle B M K$, 所以 $C^{\\prime} 、 K 、 M$ 三点共线.\n由 $\\angle B M C+\\angle C^{\\prime} C K^{\\prime}=\\angle B M C+\\angle C C^{\\prime} K$\n$$\n=\\angle B M C+\\angle B A C=180^{\\circ},\n$$\n知 $C 、 K^{\\prime} 、 M$ 三点共线.\n因此, $C^{\\prime} K 、 C K^{\\prime}$ 交于点 $M$.\n故点 $M$ 在直线 $l$ 上.", + "remark": "", + "figures": [ + "./images/volume7/figures/fig-c6i6.png" + ] +} \ No newline at end of file diff --git a/processed_dataset/proof/1729.json b/processed_dataset/proof/1729.json new file mode 100644 index 0000000000000000000000000000000000000000..b8854071d63829d3fa3dd13d12442528a3eab8dd --- /dev/null +++ b/processed_dataset/proof/1729.json @@ -0,0 +1,10 @@ +{ + "source_file": "./raw_volume-zh/volume7/chapter6.tex", + "problem_type": "proof", + "problem": "例7. 在 $\\triangle A B C$ 中, $A B=A C$, 圆 $O$ 是它的外接圆, $B N$ 平分 $\\angle A B C$, 点 $N$ 在圆 $O$ 上, 点 $E 、 F$ 分别在边 $A B 、 A C$ 上, 满足 $E O \\perp B N, E F \\perp E O$. 求证: $A E^2= B E \\cdot A F$.", + "solution": "证明:如图(), 因为 $E F \\perp E O, B N \\perp E O$, 则 $E F / / B N$.\n所以, $\\frac{A E}{B E}=\\frac{A F}{F D}$.\n故 $A E \\cdot F D=B E \\cdot A F$.\n于是, 只需证 $A E=F D$.\n由于线段 $A E 、 F D$ 不在同一个三角形中, 故可考虑作平移变换.\n作沿向量 $\\overrightarrow{F E}$ 平移变换, 则四边形 $F E D^{\\prime} D$ 为 $\\square$, 设 $D$ 变为 $D^{\\prime}$.\n连结 $A O 、 B O 、 D^{\\prime} O 、 E D^{\\prime}$. 因为\n$$\n\\begin{aligned}\n& \\angle E D^{\\prime} B=\\angle F D B=\\angle C+-\\frac{1}{2} \\angle B, \\\\\n& \\angle E B O=\\angle E A O=\\frac{1}{2} \\angle A, \\\\\n& \\angle B E O=90^{\\circ}-\\angle A E F=90^{\\circ}-\\frac{1}{2} \\angle B,\n\\end{aligned}\n$$\n则 $\\angle E O B=180^{\\circ}-\\angle B E O-\\angle E B O$\n$$\n\\begin{aligned}\n& =180^{\\circ}-\\left(90^{\\circ}-\\frac{1}{2} \\angle B\\right)-\\frac{1}{2} \\angle A \\\\\n& =\\frac{1}{2}(\\angle A+\\angle B+\\angle C)+\\frac{1}{2} \\angle B-\\frac{1}{2} \\angle A \\\\\n& =\\angle B+\\frac{1}{2} \\angle C .\n\\end{aligned}\n$$\n易知 $\\angle B=\\angle C$.\n则 $\\angle E O B=\\angle E D^{\\prime} B$, 所以, $E, O, D^{\\prime}, B$ 四点共圆.\n故\n$$\n\\begin{aligned}\n\\text { 故 } & \\angle E D^{\\prime} O=\\angle E B O=\\angle E A O, \\\\\n\\angle B O D^{\\prime}+\\angle E O B+\\angle A O E & =\\angle B E D^{\\prime}+\\angle B+\\frac{1}{2} \\angle C+\\angle A O E \\\\\n& =\\angle A+\\angle B+\\frac{1}{2} \\angle C+\\angle A O E \\\\\n& =\\angle A O E+90^{\\circ}+\\frac{1}{2} \\angle B+\\frac{1}{2} \\angle A \\\\\n& =\\angle A O E+90^{\\circ}+\\angle A E F+\\angle E A O=180^{\\circ} .\n\\end{aligned}\n$$\n因此, $D^{\\prime}, O, A$ 三点共线.\n又 $\\angle E D^{\\prime} O=\\angle E A O$, 则 $A E=E D^{\\prime}=F D$.", + "remark": "", + "figures": [ + "./images/volume7/figures/fig-c6i7.png" + ] +} \ No newline at end of file diff --git a/processed_dataset/proof/1730.json b/processed_dataset/proof/1730.json new file mode 100644 index 0000000000000000000000000000000000000000..059b7f0f9e6c1bd58074282eb1133988c003648e --- /dev/null +++ b/processed_dataset/proof/1730.json @@ -0,0 +1,10 @@ +{ + "source_file": "./raw_volume-zh/volume7/chapter6.tex", + "problem_type": "proof", + "problem": "例8. $\\odot O_1$ 与 $\\odot O_2$ 交于 $A 、 B$ 两点, 过 $A$ 作任一割线与两圆交于 $P 、 Q$. 两圆在 $P 、 Q$ 外切线交于 $R$, 直线 $B R$ 交 $\\odot\\left(O_1 O_2 B\\right)$ 于另一点 $S$. 求证: $R S$ 等于 $\\odot\\left(O_1 O_2 B\\right)$ 直径长.", + "solution": "证明:如图(), 以 $B$ 为中心作位似旋转变换使 $\\odot O_1 \\rightarrow \\odot O_2$, 则 $P 、 Q$ 为变换的对应点, $P R \\rightarrow R Q, P O_1 \\rightarrow Q O_2$.\n所以 $\\angle P B Q=\\angle O_1 B O_2=\\angle(R Q, P R)= \\pi-\\angle P R Q=\\angle\\left(O_2 Q, O_1 P\\right)$.\n所以 $P 、 R 、 Q 、 B$ 共圆, 且 $P O_1 、 Q O_2$ 交于 $C, \\angle O_1 C O_2=\\angle O_1 B O_2$.\n所以 $C \\in \\odot\\left(O_1 O_2 B\\right)$.\n因为 $\\angle B S O_2=\\angle O_2 C B=\\angle Q C B= \\angle Q R B$, 所以 $\\mathrm{SO}_2 / / R Q$.\n又因为 $R Q \\perp C Q$, 故 $S O_2 \\perp C Q$. 即 $C S$ 为$\\odot O_1 O_2 B$ 直径.\n注意到 $\\angle C S B=\\angle C O_2 B=2 \\angle O_2 Q B=2 \\angle C Q B=2 \\angle C R B$ (这里用到 $\\angle C B S=\\angle C Q R=90^{\\circ}, R, Q, B, C$ 四点共圆, 从而 $\\left.\\angle C Q B=\\angle C R B\\right)$, 于是 $R S=C S$.\n故 $R S$ 等于 $\\odot\\left(O_1 O_2 B\\right)$ 直径长.", + "remark": "", + "figures": [ + "./images/volume7/figures/fig-c6i8.png" + ] +} \ No newline at end of file diff --git a/processed_dataset/proof/1731.json b/processed_dataset/proof/1731.json new file mode 100644 index 0000000000000000000000000000000000000000..5e4e84fce82713740e4b38630641e1bd21a6c458 --- /dev/null +++ b/processed_dataset/proof/1731.json @@ -0,0 +1,10 @@ +{ + "source_file": "./raw_volume-zh/volume7/chapter7.tex", + "problem_type": "proof", + "problem": "例1. (四边形的余弦定理) 设凸四边形 $A B C D$ 对角线交于点 $P, \\angle A P B= \\theta$, 求证:\n$$\n\\cos \\theta=\\frac{A D^2+B C^2-A B^2-C D^2}{2 A C \\cdot B D} .\n$$", + "solution": "证明:如图(), 设 $P A 、 P B 、 P C 、 P D$ 的长分别为 $a 、 b 、 c 、 d$, 则有\n$$\n\\begin{aligned}\n& A D^2=a^2+d^2+2 a d \\cos \\theta, \\\\\n& B C^2=b^2+c^2+2 b c \\cos \\theta \\\\\n& A B^2=a^2+b^2-2 a b \\cos \\theta \\\\\n& C D^2=c^2+d^2+2 c d \\cos \\theta\n\\end{aligned}\n$$\n前两式之和减去后两式之和, 得\n$$\n\\begin{aligned}\nA D^2+B C^2-A B^2-C D^2 & =2(a d+b c+a b+c d) \\cos \\theta \\\\\n& =2 A C \\cdot B D \\cos \\theta .\n\\end{aligned}\n$$", + "remark": "", + "figures": [ + "./images/volume7/figures/fig-c7i1.png" + ] +} \ No newline at end of file diff --git a/processed_dataset/proof/1732.json b/processed_dataset/proof/1732.json new file mode 100644 index 0000000000000000000000000000000000000000..e7c8654c15f171515de6436407502258c8de1b3b --- /dev/null +++ b/processed_dataset/proof/1732.json @@ -0,0 +1,10 @@ +{ + "source_file": "./raw_volume-zh/volume7/chapter7.tex", + "problem_type": "proof", + "problem": "例2. 如图(), 给定凸四边形 $A B C D, \\angle B+ \\angle D<180^{\\circ}, P$ 是平面上的动点, 令 $f(P)=P A B C+P D \\cdot C A+P C \\cdot A B$.\n(1) 求证: 当 $f(P)$ 达到最小值时, $P 、 A 、 B 、 C$ 四点共圆;\n(2) 设 $E$ 是 $\\triangle A B C$ 外接圆 $O$ 的 $A B$ 上一点, 满足: $\\frac{A E}{A B}=\\frac{\\sqrt{3}}{2}, \\frac{B C}{E C}=\\sqrt{3}-1, \\angle E C B=\\frac{1}{2} \\angle E C A$, 又 $D A 、 D C$ 是圆 $O$ 的切线, $A C=\\sqrt{2}$, 求 $f(P)$ 的最小值.", + "solution": "(1) 证: 由托勒密不等式, 对平面上的任意点 $P$, 有\n$$\nP A \\cdot B C+P C \\cdot A B \\geqslant P B \\cdot A C .\n$$\n因此 $f(P)=P A \\cdot B C+P C \\cdot A B+P D \\cdot C A$\n$$\n\\geqslant P B \\cdot C A+P D \\cdot C A=(P B+P D) \\cdot C A .\n$$\n因为上面不等式当且仅当 $P 、 A 、 B 、 C$ 顺次共圆时取等号, 因此当且仅当 $P$ 在 $\\triangle A B C$ 的外接圆且在 $A C$ 上时, $f(P)=(P B+P D) \\cdot C A$.\n又因 $P B+P D \\geqslant B D$, 此不等式当且仅当 $B 、 P 、 D$ 共线且 $P$ 在 $B D$ 上时取等号.\n因此当且仅当 $P$ 为 $\\triangle A B C$ 的外接圆与 $B D$ 的交点时, $f(P)$ 取最小值 $f(P)_{\\min }=A C \\cdot B D$.\n故当 $f(P)$ 达最小值时, $P 、 A 、 B 、 C$ 四点共圆.\n(2) 记 $\\angle E C B=\\dot{\\alpha}$, 则 $\\angle E C A=2 \\alpha$, 由正弦定理有\n$$\n\\frac{A E}{A B}=\\frac{\\sin 2 \\alpha}{\\sin 3 \\alpha}=\\frac{\\sqrt{3}}{2},\n$$\n从而\n$$\n\\sqrt{3} \\sin 3 \\alpha=2 \\sin 2 \\alpha,\n$$\n即\n$$\n\\sqrt{3}\\left(3 \\sin \\alpha-4 \\sin ^3 \\alpha\\right)=4 \\sin \\alpha \\cos \\alpha,\n$$\n所以\n$$\n3 \\sqrt{3}-4 \\sqrt{3}\\left(1-\\cos ^2 \\alpha\\right)-4 \\cos \\alpha=0,\n$$\n整理得\n$$\n4 \\sqrt{3} \\cos ^2 \\alpha-4 \\cos \\alpha-\\sqrt{3}=0,\n$$\n解得\n$$\n\\cos \\alpha=\\frac{\\sqrt{3}}{2} \\text { 或 } \\cos \\alpha=-\\frac{1}{2 \\sqrt{3}} \\text { (舍去), }\n$$\n故\n$$\n\\alpha=30^{\\circ}, \\angle A C E=60^{\\circ} \\text {. }\n$$\n由已知 $\\frac{B C}{E C}=\\sqrt{3}-1=\\frac{\\sin \\left(\\angle E A C-30^{\\circ}\\right)}{\\sin \\angle E A C}$, 有\n$$\n\\sin \\left(\\angle E A C-30^{\\circ}\\right)=(\\sqrt{3}-1) \\sin \\angle E A C,\n$$\n即\n$$\n\\frac{\\sqrt{3}}{2} \\sin \\angle E A C-\\frac{1}{2} \\cos \\angle E A C=(\\sqrt{3}-1) \\sin \\angle E A C,\n$$\n整理得\n$$\n\\frac{2-\\sqrt{3}}{2}-\\sin \\angle E A C=\\frac{1}{2} \\cos \\angle E A C,\n$$\n故\n$$\n\\tan \\angle E A C=\\frac{1}{2-\\sqrt{3}}=2+\\sqrt{3}\n$$\n可得\n$$\n\\angle E A C=75^{\\circ} \\text {, }\n$$\n从而 $\\angle E=45^{\\circ}, \\angle D A C=\\angle D C A=\\angle E=45^{\\circ}, \\triangle A D C$ 为等腰直角三角形.\n因 $A C=\\sqrt{2}$, 则 $C D=1$.\n又 $\\triangle A B C$ 也是等腰直角三角形, 故 $B C=\\sqrt{2}, B D^2=1+2-2 \\cdot 1 \\cdot \\sqrt{2} \\cos 135^{\\circ}=5, B D=\\sqrt{5}$.\n故 $f(P)_{\\min }=B D \\cdot A C=\\sqrt{5} \\cdot \\sqrt{2}=\\sqrt{10}$.", + "remark": "", + "figures": [ + "./images/volume7/figures/fig-c7i2.png" + ] +} \ No newline at end of file diff --git a/processed_dataset/proof/1733.json b/processed_dataset/proof/1733.json new file mode 100644 index 0000000000000000000000000000000000000000..d3ee78e4fa44d99a1ce7af9b75ccc12a901e7139 --- /dev/null +++ b/processed_dataset/proof/1733.json @@ -0,0 +1,10 @@ +{ + "source_file": "./raw_volume-zh/volume7/chapter7.tex", + "problem_type": "proof", + "problem": "例3. 如图(), 在三角形 $A B C$ 中, $\\angle B A C=40^{\\circ}, \\angle A B C=60^{\\circ}, D$ 和 $E$ 分别是边 $A C$ 和 $A B$ 上点, 使得 $\\angle C B D=40^{\\circ}, \\angle B C E=70^{\\circ}, F$ 是直线 $B D$ 和 $C E$ 的交点.\n证明: 直线 $A F$ 和直线 $B C$ 垂直.", + "solution": "证明:设 $B C=1$, 分别在 $\\triangle A B C$ 和 $\\triangle B C F$ 中用正弦定理, 得\n$A B=\\frac{\\sin 80^{\\circ}}{\\sin 40^{\\circ}}, A C=\\frac{\\sin 60^{\\circ}}{\\sin 40^{\\circ}}, B F=\\frac{\\sin 70^{\\circ}}{\\sin 70^{\\circ}}, C F=\\frac{\\sin 40^{\\circ}}{\\sin 70^{\\circ}}$,\n而\n$$\n\\begin{aligned}\n& A F \\perp B C \\Leftrightarrow A B^2-A C^2=B F^2-C F^2 \\\\\n\\Leftrightarrow & \\frac{\\sin ^2 80^{\\circ}-\\sin ^2 60^{\\circ}}{\\sin ^2 40^{\\circ}}=\\frac{\\sin ^2 70^{\\circ}-\\sin ^2 40^{\\circ}}{\\sin ^2 70^{\\circ}} \\\\\n\\Leftrightarrow & \\frac{\\frac{1}{2}\\left(\\cos 120^{\\circ}-\\cos 160^{\\circ}\\right)}{\\sin ^2 40^{\\circ}}=\\frac{\\frac{1}{2}\\left(\\cos 80^{\\circ}-\\cos 140^{\\circ}\\right)}{\\sin ^2 70^{\\circ}} \\\\\n\\Leftrightarrow & \\frac{\\sin 140^{\\circ} \\sin 20^{\\circ}}{\\sin ^2 40^{\\circ}}=\\frac{\\sin 110^{\\circ} \\sin 30^{\\circ}}{\\sin ^2 70^{\\circ}} \\\\\n\\Leftrightarrow & \\sin 40^{\\circ} \\sin 30^{\\circ}=\\sin 20^{\\circ} \\sin 70^{\\circ} \\\\\n\\Leftrightarrow & \\sin 40^{\\circ}=2 \\sin 20^{\\circ} \\cos 20^{\\circ} .\n\\end{aligned}\n$$\n这是显然的,故命题得证.", + "remark": "", + "figures": [ + "./images/volume7/figures/fig-c7i3.png" + ] +} \ No newline at end of file diff --git a/processed_dataset/proof/1734.json b/processed_dataset/proof/1734.json new file mode 100644 index 0000000000000000000000000000000000000000..ebfd7139f4ac69afc15ba43b5735a818377cb59d --- /dev/null +++ b/processed_dataset/proof/1734.json @@ -0,0 +1,11 @@ +{ + "source_file": "./raw_volume-zh/volume7/chapter7.tex", + "problem_type": "proof", + "problem": "例4. 如图(),已知 $\\triangle A B C$ 中, $O$ 是三角形内一点满足: $\\angle B A O=\\angle C A O=\\angle C B O=\\angle A C O$. 求证: $\\triangle A B C$ 三边长成等比数列.", + "solution": "证明:如图(), 设 $\\angle B A O==\\angle C B O=\\angle A C O=\\alpha$.\n先证 $\\cot \\alpha=\\cot A+\\cot B+\\cot C, \\label{*}$.\n在 $\\triangle O A B$ 中, 由正弦定理: $\\frac{A B}{\\sin \\angle A O B}=\\frac{O B}{\\sin \\angle B A O}$,\n而 $\\sin \\angle A O B=\\sin (\\angle B A O+\\angle A B O)=\\sin (\\angle C B O+\\angle A B O)=\\sin B$, 所以\n$$\n\\frac{A B}{\\sin B}=\\frac{O B}{\\sin \\alpha} . \\label{eq1}\n$$\n同理 $\\triangle O B C$ 中有 $\\frac{B C}{\\sin C}=\\frac{O B}{\\sin (C-\\alpha)}, \\label{eq2}$.\n$\\frac{式\\ref{eq1}}{\\ref{eq2}}$, 有 $\\frac{A B \\sin C}{B C \\sin B}=\\frac{\\sin (C-\\alpha)}{\\sin \\alpha}$, 即 $\\frac{\\sin ^2 C}{\\sin A \\sin B}=\\frac{\\sin (C-\\alpha)}{\\sin \\alpha}$,\n而 $\\frac{\\sin (C-\\alpha)}{\\sin \\alpha}=\\frac{\\sin C \\cos \\alpha-\\cos C \\sin \\alpha}{\\sin \\alpha}=\\sin C \\cdot \\cot \\alpha-\\cos C$.\n所以 $\\frac{\\sin ^2 C}{\\sin A \\sin B}=\\sin C \\cdot \\cot \\alpha-\\cos C$, 即 $\\cot \\alpha=\\frac{\\sin C}{\\sin A \\sin B}+\\cot C= \\frac{\\sin (A+B)}{\\sin A \\sin B}+\\cot C=\\frac{\\sin A \\cos B+\\cos A \\sin B}{\\sin A \\sin B}+\\cot C=\\cot A+\\cot B+\\cot C$.\n\\ref{*}式得证, 由\\ref{*}式及已知条件知, $\\alpha=\\frac{A}{2}$. 从而有 $\\cot \\frac{A}{2}=\\cot A+\\cot B+\\cot C$,\n而 $\\cot \\frac{A}{2}-\\cot A=\\frac{\\cos \\frac{A}{2}}{\\sin \\frac{A}{2}}-\\frac{\\cos A}{\\sin A}=\\frac{\\sin \\frac{A}{2}}{\\sin \\frac{A}{2} \\sin A}=\\frac{1}{\\sin A}, \\cot B+\\cot C =\\frac{\\cos B}{\\sin B}+\\frac{\\cos C}{\\sin C}=\\frac{\\sin A}{\\sin B \\sin C}$,\n所以 $\\frac{1}{\\sin A}=\\frac{\\sin A}{\\sin B \\sin C}$, 所以 $\\sin ^2 A=\\sin B \\sin C$, 即 $b, a, c$ 成等比数列.", + "remark": "注: 若 $O$ 是 $\\triangle A B C$ 内一点, 满足 $\\angle B A O=\\angle C B O=\\angle A C O=\\alpha$ 这样的点 $O$ 称为布洛卡点, 布洛卡点的一个基本性质是: $\\cot \\alpha=\\cot A+\\cot B+ \\cot C$.", + "figures": [ + "./images/volume7/figures/fig-c7i4.png", + "./images/volume7/figures/fig-c7i4.png" + ] +} \ No newline at end of file diff --git a/processed_dataset/proof/1735.json b/processed_dataset/proof/1735.json new file mode 100644 index 0000000000000000000000000000000000000000..d340287f64dffc94cc93583194a11e95a472f95a --- /dev/null +++ b/processed_dataset/proof/1735.json @@ -0,0 +1,11 @@ +{ + "source_file": "./raw_volume-zh/volume7/chapter7.tex", + "problem_type": "proof", + "problem": "例5. 如图(), 设 $\\triangle A B C$ 是锐角三角形, 点 $D 、 E 、 F$ 分别在边 $B C 、 C A 、 A B$ 上,线段 $A D 、 B E$ 、 $C F$ 经过 $\\triangle A B C$ 的外心 $O$. 已知以下六个比值\n$$\n\\frac{B D}{D C}, \\frac{C E}{E A}, \\frac{A F}{F B}, \\frac{B F}{F A}, \\frac{A E}{E C}, \\frac{C D}{D B}\n$$\n中至少有两个是整数.\n求证: $\\triangle A B C$ 是等腰三角形.", + "solution": "证明:从六个比值中取出两个,共有两种类型:\n(1)涉及同一边; (2)涉及不同的边.\n(1) 如果同一边上的两个比值同时是整数, 不妨设为 $\\frac{B D}{D C} 、 \\frac{C D}{D B}$. 因它们互为倒数, 又同是整数, 所以, 必须都取 1 , 则 $B D=D C$.\n由于 $O$ 是 $\\triangle A B C$ 的外心,进而得 $A D$ 是边 $B C$ 的中垂线.\n于是, $A B=A C$.\n(2) 记 $\\angle C A B=\\alpha, \\angle A B C=\\beta, \\angle B C A=\\gamma$.\n因为 $\\triangle A B C$ 是锐角三角形, 所以,\n$$\n\\angle B O C=2 \\alpha, \\angle C O A=2 \\beta, \\angle A O B=2 \\gamma .\n$$\n于是, $\\frac{B D}{D C}=\\frac{S_{\\triangle O A B}}{S_{\\triangle O A C}}=\\frac{\\sin 2 \\gamma}{\\sin 2 \\beta}$.\n同理\n$$\n\\frac{C E}{E A}=\\frac{\\sin 2 \\alpha}{\\sin 2 \\gamma}, \\frac{A E}{F B}=\\frac{\\sin 2 \\beta}{\\sin 2 \\alpha} .\n$$\n若上述六个比值中有两个同时是整数且涉及不同的边时, 则存在整数 $m$ 、 $n$, 使得\n$$\n\\sin 2 x=m \\sin 2 z \\text { 且 } \\sin 2 y=n \\sin 2 z, \\label{eq1}\n$$\n或\n$$\n\\sin 2 z=m \\sin 2 x \\text { 且 } \\sin 2 z=n \\sin 2 y, \\label{eq2}\n$$\n其中, $x 、 y 、 z$ 是 $\\alpha 、 \\beta 、 \\gamma$ 的某种排列.\n以下构造 $\\triangle A_1 B_1 C_1$, 使得它的三个内角分别为 $180^{\\circ}-2 \\alpha, 180^{\\circ}-2 \\beta$, $180^{\\circ}-2 \\gamma$.\n如图(), 过点 $A 、 B 、 C$ 分别作 $\\triangle A B C$ 外接圆的切线, 所围成的 $\\triangle A_1 B_1 C_1$ 即满足要求.\n根据正弦定理, 知 $\\triangle A_1 B_1 C_1$ 的三边与 $\\sin 2 \\alpha 、 \\sin 2 \\beta 、 \\sin 2 \\gamma$ 成正比.\n在式\\ref{eq1}、\\ref{eq2}两种情况下, 可知其三边之比分别为 $1: m: n$ 或 $m: n: m n$.\n对于式\\ref{eq1}, 由三角形两边之和大于第三边, 可知必须 $m=n$;\n对于式\\ref{eq2}, 要保证 $m+n>m n$, 即 $(m-1)(n-1)<1$, 由此, $m 、 n$ 中必有一个为 1 .\n无论哪种情况, 都有 $\\triangle A_1 B_1 C_1$ 是等腰三角形.\n因此, $\\triangle A B C$ 也是等腰三角形.", + "remark": "", + "figures": [ + "./images/volume7/figures/fig-c7i5.png", + "./images/volume7/figures/fig-c7i5.png" + ] +} \ No newline at end of file diff --git a/processed_dataset/proof/1736.json b/processed_dataset/proof/1736.json new file mode 100644 index 0000000000000000000000000000000000000000..af7333646a372fe00bf88bb678337455fdb80710 --- /dev/null +++ b/processed_dataset/proof/1736.json @@ -0,0 +1,10 @@ +{ + "source_file": "./raw_volume-zh/volume7/chapter7.tex", + "problem_type": "proof", + "problem": "例6. 证明 Morley 定理: 如图(), 设 $\\triangle A B C$ 内有三点 $D 、 E 、 F, \\angle D B C=\\angle F B A=\\frac{1}{3} \\angle A B C$, $\\angle F A B=\\angle E A C=\\frac{1}{3} \\angle B A C, \\angle E C A=\\angle D C B= \\frac{1}{3} \\angle A C B$, 则 $\\triangle D E F$ 是正三角形.", + "solution": "证明:不妨设 $\\triangle A B C$ 对应角为 $\\angle A 、 \\angle B 、 \\angle C, R$ 为 $\\triangle A B C$ 外接圆半径.\n先证\n$$\nA F=8 R \\sin \\left(60^{\\circ}+\\frac{\\angle C}{3}\\right) \\sin \\frac{\\angle B}{3} \\sin \\frac{\\angle C}{3},\n$$\n这是因为 $\\frac{A F}{A B}=\\frac{\\sin \\frac{\\angle B}{3}}{\\sin \\frac{\\angle A+\\angle B}{3}}=\\frac{\\sin \\frac{\\angle B}{3}}{\\sin \\left(60^{\\circ}-\\frac{\\angle C}{3}\\right)}$,\n于是 $A F=A B \\cdot \\frac{\\sin \\frac{\\angle B}{3}}{\\sin \\left(60^{\\circ}-\\frac{\\angle C}{3}\\right)}=2 R \\cdot \\sin \\angle C \\cdot \\frac{\\sin \\frac{\\angle B}{3}}{\\sin \\left(60^{\\circ}-\\frac{\\angle C}{3}\\right)}$\n$$\n=8 R \\cdot \\sin \\angle C \\cdot \\sin \\left(60^{\\circ}+\\frac{\\angle C}{3}\\right) \\cdot \\sin \\frac{\\angle B}{3} \\text {. }\n$$\n(这里用到 $\\sin \\alpha \\cdot \\sin \\left(60^{\\circ}+\\alpha\\right) \\cdot \\sin \\left(60^{\\circ}-\\alpha\\right)=\\frac{1}{4} \\cdot \\sin 3 \\alpha$.)\n类似地有, $A E=8 R \\cdot \\sin \\left(60^{\\circ}+\\frac{\\angle B}{3}\\right) \\cdot \\sin \\frac{\\angle B}{3} \\cdot \\sin \\frac{\\angle C}{3}$, 于是 $E F^2= A E^2+A F^2-2 A E \\cdot A F \\cos \\frac{\\angle A}{3}=64 R^2 \\sin ^2 \\frac{\\angle B}{3} \\sin ^2 \\frac{\\angle C}{3}\\left(\\sin ^2\\left(60^{\\circ}+\\frac{\\angle C}{3}\\right)+\\right. \\left.\\sin ^2\\left(60^{\\circ}+\\frac{\\angle B}{3}\\right)\\right)-2 \\times 64 R^2 \\cdot \\sin ^2 \\frac{\\angle B}{3} \\sin ^2 \\frac{\\angle C}{3} \\sin \\left(60^{\\circ}+\\frac{\\angle B}{3}\\right) \\sin \\left(60^{\\circ}+\\frac{\\angle C}{3}\\right) \\cdot \\cos \\frac{\\angle A}{3}=64 R^2 \\sin ^2 \\frac{\\angle B}{3} \\sin ^2 \\frac{\\angle C}{3}\\left[\\sin ^2\\left(60^{\\circ}+\\frac{\\angle C}{3}\\right)+\\sin ^2\\left(60^{\\circ}+\\frac{\\angle B}{3}\\right)-\\right. \\left.2 \\sin \\left(60^{\\circ}+\\frac{\\angle B}{3}\\right) \\cdot \\sin \\left(60^{\\circ}+\\frac{\\angle C}{3}\\right) \\cdot \\cos \\frac{A}{3}\\right]=64 R^2 \\sin ^2 \\frac{\\angle B}{3} \\sin ^2 \\frac{\\angle C}{3}[1+\\left.\\cos \\frac{\\angle A}{3} \\cdot \\cos \\left(\\frac{\\angle C}{3}-\\frac{\\angle B}{3}\\right)-2 \\sin \\left(60^{\\circ}+\\frac{\\angle B}{3}\\right) \\sin \\left(60^{\\circ}+\\frac{\\angle C}{3}\\right) \\cos \\frac{A}{3}\\right]= 64 R^2 \\sin ^2 \\frac{\\angle B}{3} \\cdot \\sin ^2 \\frac{\\angle C}{3}\\left[1-\\cos ^2 \\frac{\\angle A}{3}\\right]=64 R^2 \\sin ^2 \\frac{\\angle A}{3} \\sin ^2 \\frac{\\angle B}{3} \\sin ^2 \\frac{\\angle C}{3}$, 于是 $E F=8 R \\sin \\frac{\\angle A}{3} \\sin \\frac{\\angle B}{3} \\sin \\frac{\\angle C}{3}$, 是关于 $A 、 B 、 C$ 对称的值, 所以 $D F= D E=E F$.", + "remark": "", + "figures": [ + "./images/volume7/figures/fig-c7i6.png" + ] +} \ No newline at end of file diff --git a/processed_dataset/proof/1737.json b/processed_dataset/proof/1737.json new file mode 100644 index 0000000000000000000000000000000000000000..8d90478af7060d0c6c74f227cdf69092dcde859b --- /dev/null +++ b/processed_dataset/proof/1737.json @@ -0,0 +1,10 @@ +{ + "source_file": "./raw_volume-zh/volume7/chapter7.tex", + "problem_type": "proof", + "problem": "例7. 如图(),已知 $O 、 I$ 分别是三角形 $A B C$ 的外心和内心, $B C=a, C A=b, A B=c$. 问当且仅当 $a, b, c$ 满足什么条件时,有 $O I \\perp I B$ ? 证明你的结论.\n(注: 若 $O 、 I$ 重合时, 也算成立.)", + "solution": "解:因为 $O I \\perp I B \\Leftrightarrow O I^2+B I^2=O B^2$. 令 $\\triangle A B C$ 内切圆半径 $r$, 外接圆半径为 $R$. 由欧拉定理 $O I^2=R^2-2 R r$, 且 $B I=\\frac{r}{\\sin \\frac{B}{2}}$. 所以 $R^2-2 R r+\\left(\\frac{r}{\\sin \\frac{B}{2}}\\right)^2=R^2$. 故 $\\frac{r^2}{\\sin ^2 \\frac{B}{2}}=2 R r, r=2 R \\sin ^2 \\frac{B}{2}$. 所以 $\\frac{r}{R}=1-\\cos B$. 又\n$\\frac{r}{R}=\\cos A+\\cos B+\\cos C-1$ (知识点 7). 所以 $\\cos A+\\cos B+\\cos C-1= 1-\\cos B \\cdot \\cos A+\\cos C=2-2 \\cos B$. 所以 $2 \\cos \\frac{A+C}{2} \\cos \\frac{A-C}{2}=4 \\sin ^2 \\frac{B}{2}$, $2 \\sin \\frac{B}{2} \\cos \\frac{A-C}{2}=4 \\sin ^2 \\frac{B}{2}$. 因为 $B \\in(0, \\pi)$, 所以 $\\sin \\frac{B}{2} \\in\\left(0, \\frac{\\pi}{2}\\right)$, $\\sin \\frac{B}{2} \\neq 0$. 所以 $\\cos \\frac{A-C}{2}=2 \\sin \\frac{B}{2}$.\n另一方面, $a+c=2 b \\Leftrightarrow \\sin A+\\sin C=2 \\sin B \\Leftrightarrow 2 \\sin \\frac{A+C}{2} \\cos \\frac{A-C}{2}= 4 \\sin \\frac{B}{2} \\cos \\frac{B}{2} \\Leftrightarrow \\cos \\frac{A-C}{2}=2 \\sin \\frac{B}{2}$. 综上, 当且仅当 $a+c=2 b$ 时, $O I \\perp I B$.", + "remark": "", + "figures": [ + "./images/volume7/figures/fig-c7i7.png" + ] +} \ No newline at end of file diff --git a/processed_dataset/proof/1738.json b/processed_dataset/proof/1738.json new file mode 100644 index 0000000000000000000000000000000000000000..b176a7e8bfeb993d4c26e9c1d8c8df5527bcbb00 --- /dev/null +++ b/processed_dataset/proof/1738.json @@ -0,0 +1,10 @@ +{ + "source_file": "./raw_volume-zh/volume7/chapter7.tex", + "problem_type": "proof", + "problem": "例8. 如图(), 设 $P$ 是锐角三角形 $A B C$ 内一点, $A P, B P, C P$ 分别交边 $B C, C A, A B$ 于点 $D, E, F$, 已知 $\\triangle D E F \\backsim \\triangle A B C$, 求证: $P$ 是 $\\triangle A B C$ 的重心.", + "solution": "证明:记 $\\angle E D C=\\alpha, \\angle A E F=\\beta, \\angle B F D= \\gamma$,用 $A, B, C$ 分别表示 $\\triangle A B C$ 的三个内角的大小.\n则\n$$\n\\begin{aligned}\n\\angle A F E & =\\angle B F E+\\angle B E F=(\\angle B-\\angle D B E)+(\\angle D E F-\\angle D E B) \\\\\n& =(\\angle B-\\angle D B E)+(\\angle B-\\angle D E B) \\cdot 2 B-(\\angle D B E+\\angle D E B) \\\\\n& =2 B-\\alpha .\n\\end{aligned}\n$$\n同理可证: $\\angle B D F=2 C-\\beta, \\angle C E D=2 A-\\gamma$.\n现在设 $\\triangle D E F$ 和 $\\triangle D E C$ 的外接圆半径为 $R_1$ 和 $R_2$, 则由正弦定理及 $\\angle E F D=C$, 可知 $2 R_1=\\frac{D E}{\\sin \\angle E F D}=\\frac{D E}{\\sin C}=2 R_2$, 故 $R_1=R_2$. 类似可得 $\\triangle D E F$ 和 $\\triangle A E F, \\triangle B D F$ 的外接圆半径相等.\n所以 $\\triangle D E F, \\triangle A E F$, $\\triangle B D F$ 和 $\\triangle D E C$ 这四个三角形的外接圆半径都相同, 记为 $R$.\n利用正弦定理得:\n$$\n\\frac{C E}{\\sin \\alpha}=\\frac{E A}{\\sin (2 B-\\alpha)}=\\frac{A F}{\\sin \\beta}=\\frac{F B}{\\sin (2 C-\\beta)}=\\frac{B D}{\\sin \\gamma}=\\frac{D C}{\\sin (2 A-\\gamma)}=2 R . \\label{eq1}\n$$\n再由 Ceva 定理可知 $\\frac{C E}{E A} \\cdot \\frac{A F}{F B} \\cdot \\frac{B D}{D C}=1$, 结合上式得\n$$\n\\frac{\\sin \\alpha \\sin \\beta \\sin \\gamma}{\\sin (2 B-\\alpha) \\sin (2 C-\\beta) \\sin (2 A-\\gamma)}=1 . \\label{eq2}\n$$\n若 $\\alphaB$, 可得 式\\ref{eq2}的左边小于右边,矛盾.\n所以, $\\alpha=B$. 同理 $\\beta=C, \\gamma=A$. 因此, 由 式\\ref{eq1}可知 $D, E, F$ 分别为 $B C, C A, A B$ 的中点.\n从而, $P$ 为 $\\triangle A B C$ 的重心.", + "remark": "", + "figures": [ + "./images/volume7/figures/fig-c7i8.png" + ] +} \ No newline at end of file diff --git a/processed_dataset/proof/1739.json b/processed_dataset/proof/1739.json new file mode 100644 index 0000000000000000000000000000000000000000..9600d7ecc964614130b69461aab11a4c3434eb8b --- /dev/null +++ b/processed_dataset/proof/1739.json @@ -0,0 +1,10 @@ +{ + "source_file": "./raw_volume-zh/volume7/chapter7.tex", + "problem_type": "proof", + "problem": "例9. 如图(), $A B$ 为圆 $\\omega$ 的直径, 直线 $l$ 切 $\\odot \\omega$ 于 $A . C 、 M 、 D$ 在 $I$ 上满足 $C M=D M$, 又设 $B C 、 B D$ 交 $\\odot \\omega$ 于 $P 、 Q, \\odot \\omega$ 切线 $P R 、 Q R$ 交于 $R$. 求证: $R$ 在 $B M$ 上.", + "solution": "证明:连结 $P A 、 Q A$, 设 $B R$ 交 $\\odot \\omega$ 于 $T$, 连结\n$P T, Q T$.\n在 $\\triangle B M C$ 与 $\\triangle B M D$ 中用正弦定理得\n$$\n\\frac{\\sin \\angle C B M}{\\sin C}=\\frac{C M}{B M}=\\frac{D M}{B M}=\\frac{\\sin \\angle D B M}{\\sin D} .\n$$\n于是\n$$\n\\frac{\\sin \\angle C B M}{\\sin \\angle D B M}=\\frac{\\sin C}{\\sin D} \\text {. }\n$$\n注意到 $\\angle B P A=\\angle B A C=\\angle B A D=\\angle B Q A=90^{\\circ}$.\n故\n$$\n\\angle C=\\angle B A P, \\angle D=\\angle B A Q .\n$$\n则\n$$\n\\frac{\\sin \\angle C B M}{\\sin \\angle \\overline{D B M}}=\\frac{\\sin \\angle B A P}{\\sin \\angle B A Q}=\\frac{B P}{B Q} . \\label{eq1}\n$$\n另一方面, 易知 $\\triangle R T P \\backsim \\triangle R P B, \\triangle R T Q \\backsim \\triangle R Q B$.\n因此\n$$\n\\begin{gathered}\n\\frac{B P}{P T}=\\frac{B R}{P R}=\\frac{B R}{Q R}=\\frac{B Q}{Q T} . \\\\\n\\frac{B P}{B Q}=\\frac{P T}{Q T}=\\frac{\\sin \\angle P B T}{\\sin \\angle Q B T}=\\frac{\\sin \\angle C B R}{\\sin \\angle D B R} . \\label{eq2}\n\\end{gathered}\n$$\n由式\\ref{eq1},\\ref{eq2}两式知 $\\frac{\\sin \\angle C B M}{\\sin \\angle D B M}=\\frac{\\sin \\angle C B R}{\\sin \\angle D B R}$.\n又 $\\quad \\angle C B M+\\angle D B M=\\angle C B R+\\angle D B R<\\pi$.\n由上式易知 $\\quad \\angle D B M=\\angle D B R$.\n(事实上, 上式等价于 $\\sin \\angle C B D \\cot \\angle D B M-\\cos \\angle C B D== \\sin \\angle C B D \\cos \\angle D B R-\\cos \\angle C B D)$.\n所以 $B 、 M 、 R$ 三点共线, 得证.", + "remark": "注:在有圆的情况下, 角度较易转化, 因此应尽量把线段比化为角度比, 再通过角度比求解题目.", + "figures": [ + "./images/volume7/figures/fig-c7i9.png" + ] +} \ No newline at end of file diff --git a/processed_dataset/proof/1740.json b/processed_dataset/proof/1740.json new file mode 100644 index 0000000000000000000000000000000000000000..bfdc5e1f77d0ec0ab21404acfca1ae0362681c67 --- /dev/null +++ b/processed_dataset/proof/1740.json @@ -0,0 +1,10 @@ +{ + "source_file": "./raw_volume-zh/volume7/chapter7.tex", + "problem_type": "proof", + "problem": "例10. 已知锐角三角形 $A B C, C D$ 是高, 点 $M$ 是 $A B$ 中点.\n过点 $M$ 的直线分别交射线 $C A 、 C B$ 于点 $K 、 L$, 且 $C K=C L$. 求证: 若 $\\triangle C K L$ 的外心为点 $S$, 则 $S D=S M$.", + "solution": "证明:如图(), 不妨设 $A C \\geqslant B C$, 易知此时点 $K$ 在 $A C$ 上,点 $L$ 在 $C B$ 延长线上.\n由正弦定理知\n$$\n\\frac{A K}{A M}=\\frac{\\sin \\angle A M K}{\\sin \\angle A K M}, \\frac{B L}{B M}=\\frac{\\sin \\angle B M L}{\\sin \\angle B L M},\n$$\n由对顶角相等及 $\\angle A K M+\\angle B L M=180^{\\circ}$, 得\n$$\n\\frac{A K}{A M}=\\frac{B L}{B M} \\text {, }\n$$\n即 $A K=B L$.\n这样一来,便有\n$$\nC K=C L=\\frac{A C+B C}{2}, C S=\\frac{C K}{2 \\cos \\frac{\\angle A C B}{2}}=\\frac{A C+B C}{4 \\cos \\frac{\\angle A C B}{2}},\n$$\n延长 $C S$ 交 $\\triangle A B C$ 外接圆 $\\overparen{A B}$ 于点 $E$, 则点 $E$ 为 $\\overparen{A B}$ 中点.\n若设 $\\triangle A B C$ 外接圆半径为 $R$, 则 $C E=2 R \\sin \\left(\\angle C A B+\\frac{\\angle A C B}{2}\\right)$, 于是\n$$\n\\begin{aligned}\n\\frac{C E}{C S} & =2 R \\sin \\left(\\angle C A B+\\frac{\\angle A C B}{2}\\right) \\cdot \\frac{2 \\cos \\frac{\\angle A C B}{2}}{R(\\sin \\angle A B C+\\sin \\angle C A B)} \\\\\n& =\\frac{4 \\sin \\left(\\angle C A B+\\frac{\\angle A C B}{2}\\right) \\cos \\frac{\\angle A C B}{2}}{\\sin \\angle C A B+\\sin \\angle A B C} \\\\\n& =\\frac{2(\\sin (\\angle C A B+\\angle A C B)+\\sin \\angle C A B)}{\\sin \\angle C A B+\\sin \\angle A B \\bar{C}} \\\\\n& =\\frac{2(\\sin \\angle A B C+\\sin \\angle C A B)}{\\sin \\angle C A B+\\sin \\angle A B C}=2 .\n\\end{aligned}\n$$\n这表明, 点 $S$ 为 $C E$ 中点.\n又因为 $M E \\perp A B, C D \\perp A B$, 故点 $S$ 在 $M D$ 的中垂线上.\n故 $S D=S M$.", + "remark": "", + "figures": [ + "./images/volume7/figures/fig-c7i10.png" + ] +} \ No newline at end of file diff --git a/processed_dataset/proof/1741.json b/processed_dataset/proof/1741.json new file mode 100644 index 0000000000000000000000000000000000000000..9ab4caaa5fcc682279e09826baea585b11a6f7f4 --- /dev/null +++ b/processed_dataset/proof/1741.json @@ -0,0 +1,10 @@ +{ + "source_file": "./raw_volume-zh/volume7/chapter7.tex", + "problem_type": "proof", + "problem": "例11. 如图(), 已知 $\\triangle A B C, \\angle C< \\angle A<90^{\\circ}, D \\in A C$, 且 $B D=B A, \\triangle A B C$ 内切圆与 $A B 、 A C$ 分别切于 $K 、 L$. 设 $J$ 是 $\\triangle B C D$ 内心.\n证明, $K L$ 平分线段 $A J$.", + "solution": "证明:设 $I$ 为 $\\triangle A B C$ 内心, $A I \\cap K L=P$, 连结 $I K 、 I L 、 B I 、 B J 、 I J$.\n设 $\\triangle A B C$ 内切圆半径为 $r$,三内角为 $A 、 B 、 C$.\n由于 $B I=\\frac{r}{\\sin \\frac{B}{2}}$ 且 $\\frac{B I}{\\sin \\angle B J I}=\\frac{I J}{\\sin \\angle I B J}$,\n$\\angle I B J=\\frac{\\angle A B C-\\angle D B C}{2}=\\frac{\\angle A B D}{2}=\\frac{180^{\\circ}-2 A}{2}=90^{\\circ}-A$,\n$\\angle I J B=180^{\\circ}-\\angle I B J-\\angle B I J=180^{\\circ}-\\left(90^{\\circ}-A\\right)-\\left(90^{\\circ}+\\frac{A}{2}\\right)=\\frac{A}{2}$. (这里用到 $C 、 I 、 J$ 三点共线)\n所以 $I J=\\frac{B I}{\\sin \\frac{A}{2}} \\sin \\left(90^{\\circ}-A\\right)=\\frac{B I}{\\sin \\frac{A}{2}} \\cos A$.\n而 $K L$ 平分 $A J \\Leftrightarrow A$ 到 $K L$ 的距离 $=J$ 到 $K L$ 的距离\n$\\Leftrightarrow A P-P I=\\frac{r \\cos A}{\\sin \\frac{A}{2}}, \\label{eq1}$.\n\\ref{eq1}式左边 $=r \\cot \\frac{A}{2} \\cos \\frac{A}{2}-r \\sin \\frac{A}{2}=r\\left(\\frac{\\cos ^2 \\frac{A}{2}}{\\sin \\frac{A}{2}}-\\sin \\frac{A}{2}\\right)=r$. $\\frac{\\cos ^2 \\frac{A}{2}-\\sin ^2 \\frac{A}{2}}{\\sin \\frac{A}{2}}=\\frac{r \\cos A}{\\sin \\frac{A}{2}}=$ \\ref{eq1} 式右边.\n故 $K L$ 平分 $A J$, 证毕.", + "remark": "", + "figures": [ + "./images/volume7/figures/fig-c7i11.png" + ] +} \ No newline at end of file diff --git a/processed_dataset/proof/1742.json b/processed_dataset/proof/1742.json new file mode 100644 index 0000000000000000000000000000000000000000..dc1f89ceb7649d9290ec1ba7e4e3ee7e4bc4359e --- /dev/null +++ b/processed_dataset/proof/1742.json @@ -0,0 +1,11 @@ +{ + "source_file": "./raw_volume-zh/volume7/chapter7.tex", + "problem_type": "proof", + "problem": "例12. 如图(), 凸四边形 $A B F D$ 中, $A B+ B F=A D+D F$. 延长 $A B$ 与 $D F$ 相交于点 $C$, 延长 $A D$ 与 $B F$ 相交于 $E$. 求证: $A C+C F=A E+E F$.", + "solution": "证明:连结 $A F$,并分别记角如图() 所示.\n首先, 在 $\\triangle A B F$ 中, 由正弦定理有:\n$$\n\\frac{A B}{\\sin \\gamma}=\\frac{B F}{\\sin \\alpha}=\\frac{A F}{\\sin (\\gamma+\\alpha)},\n$$\n所以 $A B+B F=A F \\cdot \\frac{\\sin \\gamma+\\sin \\alpha}{\\sin (\\gamma+\\alpha)}$.\n同理, $A D+D F=A F \\cdot \\frac{\\sin \\theta+\\sin \\beta}{\\sin (\\theta+\\beta)}$.\n所以 $A B+B F=A D+D F \\Leftrightarrow \\frac{\\sin \\gamma+\\sin \\alpha}{\\sin (\\gamma+\\alpha)}=\\frac{\\sin \\theta+\\sin \\beta}{\\sin (\\theta+\\beta)}\\Leftrightarrow \\cos \\frac{\\gamma-\\alpha}{2} \\cos \\frac{\\theta+\\beta}{2}=\\cos \\frac{\\theta-\\beta}{2} \\cos \\frac{\\gamma+\\alpha}{2}$\n$$\n\\begin{aligned}\n& \\Leftrightarrow \\cos \\frac{\\theta+\\beta+\\gamma-\\alpha}{2}+\\cos \\frac{\\theta+\\beta-\\gamma+\\alpha}{2} \\\\\n& =\\cos \\frac{\\gamma+\\alpha+\\theta-\\beta}{2}+\\cos \\frac{\\gamma+\\alpha-\\theta+\\beta}{2} .\n\\end{aligned}\n$$\n另一方面: $A C+C F=A E+E F \\Leftrightarrow \\frac{\\sin \\theta+\\sin \\alpha}{\\sin (\\theta-\\alpha)}=\\frac{\\sin \\gamma+\\sin \\beta}{\\sin (\\gamma-\\beta)}$,\n$$\n\\Leftrightarrow \\sin \\frac{\\theta+\\alpha}{2} \\sin \\frac{\\gamma-\\beta}{2}=\\sin \\frac{\\gamma+\\beta}{2} \\sin \\frac{\\theta-\\alpha}{2},\n$$\n即 $-\\frac{1}{2}\\left[\\cos \\frac{\\theta+\\alpha+\\gamma-\\beta}{2}-\\cos \\frac{\\theta+\\alpha-\\gamma+\\beta}{2}\\right]=-\\frac{1}{2}\\left[\\cos \\frac{\\gamma+\\beta+\\theta-\\alpha}{2}-\\right.$\n$$\n\\begin{aligned}\n& \\left.\\cos \\frac{\\gamma+\\beta-\\theta+\\alpha}{2}\\right] \\Leftrightarrow \\cos \\frac{\\theta+\\alpha+\\gamma-\\beta}{2}-\\cos \\frac{\\theta+\\alpha-\\gamma+\\beta}{2}=\\cos \\frac{\\gamma+\\beta+\\theta-\\alpha}{2}- \\\\\n& \\cos \\frac{\\gamma+\\beta-\\theta+\\alpha}{2} .\n\\end{aligned}\n$$\n故由 $A B+B F=A D+D F$ 可以推出 $A C+C F=A E+E F$.", + "remark": "", + "figures": [ + "./images/volume7/figures/fig-c7i12.png", + "./images/volume7/figures/fig-c7i12.png" + ] +} \ No newline at end of file diff --git a/processed_dataset/proof/1743.json b/processed_dataset/proof/1743.json new file mode 100644 index 0000000000000000000000000000000000000000..a90181c4d5f87d4e0d179c06f3aee4f3df38ec02 --- /dev/null +++ b/processed_dataset/proof/1743.json @@ -0,0 +1,10 @@ +{ + "source_file": "./raw_volume-zh/volume7/chapter7.tex", + "problem_type": "proof", + "problem": "例13. 如图() 已知锐角 $\\triangle A B C$ 的垂心为 $H$, 内心为 $I$, 且满足 $A C \\neq B C, C H, C I$ 分别与 $\\triangle A B C$ 的外接圆交于点 $D 、 L$. 证明: $\\angle C I H=90^{\\circ}$ 的充分必要条件是 $\\angle I D L=90^{\\circ}$.", + "solution": "证明:不妨设 $\\angle B>\\angle A$, 外接圆半径为 $R$,\n所以 $\\angle H C L=\\angle H C A-\\angle I C A=90^{\\circ}-$\n$$\n\\begin{aligned}\n& \\angle A-\\frac{\\angle C}{2} \\\\\n= & \\frac{1}{2} \\cdot\\left(180^{\\circ}-2 \\cdot \\angle A-180^{\\circ}+\\angle A+\\angle B\\right)=\\frac{1}{2} \\cdot \\mathbf{1 3} \\\\\n& (\\angle B-\\angle A) .\n\\end{aligned}\n$$\n由正弦定理: $C H=\\sin \\angle H A C \\cdot \\frac{A C}{\\sin \\angle A H C}=\\cos \\angle C \\cdot \\frac{A C}{\\sin \\angle B}=2 R \\cdot \\cos \\angle C $,\n同理,\n$$\n\\begin{aligned}\nC I & =\\frac{\\sin \\angle I A C \\cdot A C}{\\sin \\angle A I C}=\\frac{\\sin \\frac{\\angle A}{2} \\cdot(2 R \\sin \\angle B)}{\\sin \\left(90^{\\circ}+\\angle \\frac{B}{2}\\right)} \\\\\n& =\\frac{\\sin \\frac{\\angle A}{2} \\cdot 4 R \\cdot \\sin \\frac{\\angle B}{2} \\cdot \\cos \\frac{\\angle B}{2}}{\\cos \\frac{\\angle B}{2}}=4 R \\cdot \\sin \\frac{\\angle A}{2} \\cdot \\sin \\frac{\\angle B}{2} .\n\\end{aligned}\n$$\n$$\nL D=2 R \\cdot \\sin \\angle L C D=2 R \\cdot \\sin \\angle H C I=2 R \\cdot \\sin \\frac{\\angle B-\\angle A}{2} .\n$$\n由鸡爪定理: $L I=L B=2 R \\cdot \\sin \\angle L C B=2 R \\cdot \\sin \\frac{\\angle C}{2}$.\n$$\n\\angle C L D=\\angle C A D=\\angle A+\\angle D C B=\\angle A+90^{\\circ}-\\angle B .\n$$\n于是, $\\angle I D L=90^{\\circ}$ 的充分必要条件是 $L D=L I \\cdot \\cos \\angle D L C$, 即 $2 R \\cdot\\sin \\frac{\\angle B-\\angle A}{2}=2 R \\cdot \\sin \\frac{\\angle C}{2} \\cdot \\cos \\left(90^{\\circ}-\\angle B+\\angle A\\right)=2 R \\cdot \\sin \\frac{\\angle C}{2} \\cdot\\sin (\\angle B-\\angle A)=4 R \\cdot \\sin \\frac{\\angle C}{2} \\cdot \\sin \\frac{\\angle B-\\angle A}{2} \\cdot \\cos \\frac{\\angle B-\\angle A}{2}$ 或者 $1=2 \\cdot \\sin \\frac{\\angle C}{2} \\cdot \\cos \\frac{\\angle B-\\angle A}{2}$, 这就相当于 $\\cos \\frac{\\angle A+\\angle B}{2}=\\cos \\frac{\\angle B-\\angle A}{2}$.\n$2 \\cdot \\sin ^2 \\frac{\\angle C}{2}$.\n另一方面, $\\angle C I H=90^{\\circ}$ 即 $C I=C H \\cdot \\cos \\angle I C H$,\n等价于 $4 R \\cdot \\sin \\frac{\\angle A}{2} \\cdot \\sin \\frac{\\angle B}{2}=\\cos \\frac{\\angle B-\\angle A}{2} \\cdot 2 R \\cdot \\cos \\angle C$, 消去 $2 R$ 得\n$2 \\cdot \\sin \\frac{\\angle A}{2} \\cdot \\sin \\frac{\\angle B}{2}=\\cos \\frac{\\angle B-\\angle A}{2} \\cdot\\left(1-2 \\cdot \\sin ^2 \\frac{\\angle C}{2}\\right)$, 即 $\\cos \\frac{\\angle B-\\angle A}{2}-\\cos \\frac{\\angle A+\\angle B}{2}=\\cos \\frac{\\angle B-\\angle A}{2}\\left(1-2 \\sin ^2 \\frac{\\angle C}{2}\\right) \\Leftrightarrow \\cos \\frac{\\angle A+\\angle B}{2}= 2 \\cos \\frac{\\angle B-\\angle A}{2} \\cdot \\sin ^2 \\frac{\\angle C}{2}$.\n于是 $\\angle C I H=90^{\\circ}$ 的充要条件是 $\\angle I D L=90^{\\circ}$.", + "remark": "", + "figures": [ + "./images/volume7/figures/fig-c7i13.png" + ] +} \ No newline at end of file diff --git a/processed_dataset/proof/1744.json b/processed_dataset/proof/1744.json new file mode 100644 index 0000000000000000000000000000000000000000..a820856c0e7e32dc1be356221dfa76d100cc7e29 --- /dev/null +++ b/processed_dataset/proof/1744.json @@ -0,0 +1,10 @@ +{ + "source_file": "./raw_volume-zh/volume7/chapter8.tex", + "problem_type": "proof", + "problem": "例1. 如图(), $R K 、 R L$ 是圆的两条切线, 过 $R$ 的割线交圆于 $S 、 T$ 两点, 交 $K L$ 于 $V$, 则 $R 、 V 、 S 、 T$ 是调和点列.", + "solution": "证明:连结 $S L 、 T L$, 注意到 $\\angle S L R=\\angle L T R$, 于是 $\\triangle S L R \\backsim \\triangle L T R$,\n$\\frac{S L}{L T}=\\frac{S R}{R L}=\\frac{R L}{R T}=\\sqrt{\\frac{S R}{R T}}$, 同理可证, $\\frac{S K}{K T}= \\sqrt{\\frac{S R}{R T}}$, 另一方面, $\\frac{S V}{V T}=\\frac{S_{\\triangle S K L}}{S_{\\triangle T K L}}=\\frac{S K \\cdot S L}{T K \\cdot T L}= \\frac{S L}{L T} \\cdot \\frac{S K}{K T}=\\frac{S R}{R T}$, 即 $R 、 V 、 S 、 T$ 成调和点列.", + "remark": "", + "figures": [ + "./images/volume7/figures/fig-c8i12.png" + ] +} \ No newline at end of file diff --git a/processed_dataset/proof/1745.json b/processed_dataset/proof/1745.json new file mode 100644 index 0000000000000000000000000000000000000000..85242b8cc2093e37814e91d1b1542d6234338b2e --- /dev/null +++ b/processed_dataset/proof/1745.json @@ -0,0 +1,10 @@ +{ + "source_file": "./raw_volume-zh/volume7/chapter8.tex", + "problem_type": "proof", + "problem": "例2. 如图(), 已知 $P A 、 P B$ 是由圆 $O$ 外一点 $P$ 引出的两条切线, $M 、 N$ 分别为线段 $A P 、 A B$ 的中点, 延长 $M N$ 交圆 $O$ 于点 $C$, 点 $N$ 在 $M$ 与 $C$ 之间, $P C$ 交圆 $O$ 于点 $D$, 延长 $N D$ 交 $P B$ 于点 $Q$. 证明: 四边形 $M N Q P$ 为菱形.", + "solution": "证明:由例 1 结论知, $P 、 E 、 D 、 C$ 成调和点列.\n由于 $M N / / P B$, 由定理 1 , 以 $N$ 为中心, 由 $(P 、 E 、 D 、 C)$ 为调和点列可以得到 $(P 、 B 、 Q 、 \\infty)$ 为调和点列.\n所以, $Q$ 为 $P B$ 的中点.\n而 $P B=P A, M 、 N 、 Q$ 为 $P A 、 A B 、 P B$ 的中点, 故四边形 $M N Q P$ 为菱形.", + "remark": "", + "figures": [ + "./images/volume7/figures/fig-c8i13.png" + ] +} \ No newline at end of file diff --git a/processed_dataset/proof/1746.json b/processed_dataset/proof/1746.json new file mode 100644 index 0000000000000000000000000000000000000000..57723446e488ddcaf3afcaaa7902b738a7e4fe59 --- /dev/null +++ b/processed_dataset/proof/1746.json @@ -0,0 +1,10 @@ +{ + "source_file": "./raw_volume-zh/volume7/chapter8.tex", + "problem_type": "proof", + "problem": "例3. 求证: 以完全四边形的三条对角线为直径的圆共轴, 且完全四边形的四个三角形的垂心在这条根轴上.", + "solution": "证明:如图(), 不妨设 $H_1$ 为 $\\triangle D E F$ 的垂心, 以 $C F 、 B E 、 A D$ 为直径的圆依次为 $O_1 、 O_2 、 O_3$, 连结 $H_1 F$ 与 $\\odot O_1$ 交于 $K$, 显然 $K$ 在 $D E$ 延长线上.\n$H_1$ 对 $\\odot O_1$ 的幂为 $H_1 K \\cdot H_1 F, H_1$ 对 $\\odot O_2$ 的幕为 $H_1 E \\cdot H_1 L$.\n而由 $K 、 F 、 L 、 E$ 四点共圆知, $H_1 K H_1 F=H_1 E \\cdot H_1 L$, 即 $H_1$ 对 $\\odot O_1 、 \\odot O_2$ 的幂相等.\n同理 $H_1$ 对 $\\odot O_2 、 \\odot O_3$ 的幂相等, 故 $H_1$ 对 $\\odot O_1 、 \\odot O_2 、 \\odot O_3$ 等幂.\n同理 $H_2 、 H_3 、 H_4$.也对 $\\odot O_1 、 \\odot O_2 、 \\odot O_3$ 等幂.\n显然 $H_1 、 H_2 、 H_3 、 H_4$ 不重合, (从而不可能都是根心), 这里有三个圆两两根轴相同,且 $H_1 、 H_2 、 H_3 、 H_4$ 均在这条根轴上.", + "remark": "", + "figures": [ + "./images/volume7/figures/fig-c8i14.png" + ] +} \ No newline at end of file diff --git a/processed_dataset/proof/1747.json b/processed_dataset/proof/1747.json new file mode 100644 index 0000000000000000000000000000000000000000..ec3977ca55b11eff95957fad00da2449aa0cb5f8 --- /dev/null +++ b/processed_dataset/proof/1747.json @@ -0,0 +1,10 @@ +{ + "source_file": "./raw_volume-zh/volume7/chapter8.tex", + "problem_type": "proof", + "problem": "例4. 证明:\nCandy 定理: 设 $A B$ 为一圆任一条弦, $O$ 为 $A B$ 上任一点,过 $O$ 任作两条弦 $C D 、 E F$, 连结 $C F 、 E D$ 交 $A B$ 于 $G 、 H$, 则 $\\frac{1}{O G}-\\frac{1}{O H}=\\frac{1}{O A}-\\frac{1}{O B}$.", + "solution": "证明:如图(), 连结 $A F 、 B F 、 A D 、 B D$, 则 $\\angle A F C=\\angle A D C, \\angle C F E=\\angle C D E, \\angle E F B=\\angle E D B$.\n由上述交比性质结论知,\n$$\n\\frac{A G / G O}{A B / B O}=\\frac{A O / O H}{A B / B H} \\Rightarrow\\left(\\frac{A O}{G O}-1\\right) \\cdot B O=\\frac{A O}{O H} (B O-O H)=A O \\times\\left(\\frac{B O}{O H}-1\\right) \\Rightarrow \\frac{1}{G O}-\\frac{1}{A O}=\\frac{1}{O H}-\\frac{1}{B O}\n$$\n所以 $\\frac{1}{G O}-\\frac{1}{O H}= \\frac{1}{A O}-\\frac{1}{B O}$.", + "remark": "", + "figures": [ + "./images/volume7/figures/fig-c8i15.png" + ] +} \ No newline at end of file diff --git a/processed_dataset/proof/1748.json b/processed_dataset/proof/1748.json new file mode 100644 index 0000000000000000000000000000000000000000..c1e11594498fb12b3559217cceb2ab6622f6ce3f --- /dev/null +++ b/processed_dataset/proof/1748.json @@ -0,0 +1,10 @@ +{ + "source_file": "./raw_volume-zh/volume7/chapter8.tex", + "problem_type": "proof", + "problem": "例5. 如图(), 在 $\\triangle P B C$ 中, $\\angle P B C= 60^{\\circ}$, 过点 $P$ 作 $\\triangle P B C$ 的外接圆圆 $O$ 的切线, 与 $C B$ 的延长线交于点 $A$, 点 $D 、 E$ 分别在线段 $P A$ 和圆 $O$ 上,使得 $\\angle D B E=90^{\\circ}, P D=P E$, 连结 $B E$ 与 $P C$ 相交于点 $F$. 已知 $A F 、 B P 、 C D$ 三线共点.\n(1) 求证: $B F$ 是 $\\angle P B C$ 的角平分线;\n(2) 求 $\\tan \\angle P C B$ 的值.", + "solution": "解:(1) 设 $A F 、 B P 、 C D$ 三线共点于 $H$, 设 $A H$ 与 $B D$ 交于点 $G$. 在完全四边形 $A B C H P D$ 中, 由对角线调和分割性质知 $A H$ 被 $G 、 F$ 调和分割, 从而知 $B A 、 B H 、 B G 、 B F$ 为调和线束.\n而 $B D \\perp B E$, 故 $B F$ 平分 $\\angle A B P$ 的外角, 即 $B F$ 是 $\\angle P B C$ 的平分线.\n(2) 设 $\\angle P C B=\\alpha$, 则 $\\angle A P B=\\angle P E B=\\alpha$, 在 $\\triangle P E B$ 及 $\\triangle P D B$ 中分别由正弦定理并注意到 $P D=P E$ 有\n$$\n\\frac{\\sin 30^{\\circ}}{\\sin \\alpha}=\\frac{\\sin 60^{\\circ}}{\\sin \\left(120^{\\circ}-\\alpha\\right)} \\Rightarrow \\tan \\alpha=\\frac{6+\\sqrt{6}}{11} .\n$$", + "remark": "", + "figures": [ + "./images/volume7/figures/fig-c8i16.png" + ] +} \ No newline at end of file diff --git a/processed_dataset/proof/1749.json b/processed_dataset/proof/1749.json new file mode 100644 index 0000000000000000000000000000000000000000..763c2041b2aa2fd5629ea89b1ee4331f420337e8 --- /dev/null +++ b/processed_dataset/proof/1749.json @@ -0,0 +1,10 @@ +{ + "source_file": "./raw_volume-zh/volume7/chapter8.tex", + "problem_type": "proof", + "problem": "例6. 设 $O I$ 分别是 $\\triangle A B C$ 的外心、内心, $\\triangle A B C$ 的内切圆与 $B C 、 C A$ 、 $A B$ 分别切于点 $D 、 E 、 F$, 直线 $D F$ 与 $C A$ 交于点 $P$, 直线 $D E$ 与 $A B$ 交于点 $Q, M 、 N$ 分别是线段 $P E 、 Q F$ 的中点, 求证: $O I \\perp M N$. (2007 中国数学奥林匹克)", + "solution": "证明:如图(),易证 $\\frac{A F}{F B} \\cdot \\frac{B D}{D C} \\cdot \\frac{C E}{E A}=1$.\n所以, $A D 、 B E 、 C F$ 三线共点.\n由性质 2 知 $P 、 E 、 A 、 C$ 是调和点列.\n因为 $M$ 是线段 $P E$ 的中点, 所以, 由性质 7 得 $M E^2= M A \\cdot M C$. 同理, $N F^2=N A \\cdot N B$.\n因此, 点 $M 、 N$ 分别到 $\\triangle A B C$ 的内切圆和外接圆等幂, 即点 $M 、 N$ 在 $\\triangle A B C$ 的内切圆与外接圆的根轴上.\n故 $O I \\perp M N$.", + "remark": "注:: 本题在第 5 章中曾出现过, 但这里用的是不同的方法.", + "figures": [ + "./images/volume7/figures/fig-c8i17.png" + ] +} \ No newline at end of file diff --git a/processed_dataset/proof/1750.json b/processed_dataset/proof/1750.json new file mode 100644 index 0000000000000000000000000000000000000000..87a99b402d8968b8f62b1a16c3581861ed38ebbd --- /dev/null +++ b/processed_dataset/proof/1750.json @@ -0,0 +1,10 @@ +{ + "source_file": "./raw_volume-zh/volume7/chapter8.tex", + "problem_type": "proof", + "problem": "例7. 设凸四边形 $A B C D$ 的两组对边分别交于点 $E$ 、 $F$, 两条对角线的交点为 $P$, 过 $P$ 作 $P O \\perp E F$ 于点 $O$. 求证: $\\angle B O C= \\angle A O D$.", + "solution": "证明:如图(), 延长 $A C 、 D B$ 分别与 $E F$ 交于点 $Q 、 R$. 若 $B D$ 与 $E F$ 平行, 则视点 $R$ 在无穷远处.\n由性质 2 知 $P 、 Q$ 调和分割线段 $A C, P$ 、 $R$ 调和分割线段 $B D$.\n因为 $P O \\perp E F$, 所以, 根据性质 5 中 (3) 和 (4) $\\Rightarrow(1)$ 和 (2), 知 $\\angle P O A=\\angle P O C$,\n$\\angle P O B=\\angle P O D$. 因此, $\\angle B O C=\\angle A O D$.", + "remark": "", + "figures": [ + "./images/volume7/figures/fig-c8i18.png" + ] +} \ No newline at end of file diff --git a/processed_dataset/proof/1751.json b/processed_dataset/proof/1751.json new file mode 100644 index 0000000000000000000000000000000000000000..d215354cf65f83c4a42dbdd9e588d7cec6266028 --- /dev/null +++ b/processed_dataset/proof/1751.json @@ -0,0 +1,10 @@ +{ + "source_file": "./raw_volume-zh/volume7/chapter8.tex", + "problem_type": "proof", + "problem": "例8. 过锐角 $\\triangle A B C$ 的顶点 $A 、 B 、 C$ 的三条高分别交对边于点 $D 、 E$ 、 $F$, 过点 $D$ 平行于 $E F$ 的直线分别交 $A C 、 A B$ 于点 $Q 、 R, E F$ 交 $B C$ 于点 $P$. 证明: $\\triangle P Q R$ 的外接圆过 $B C$ 的中点.", + "solution": "证明:如图(), 取边 $B C$ 的中点 $M$.\n由性质 2 知 $B 、 C 、 D 、 P$ 是调和点列.\n又 $M$ 是 $B C$ 的中点, 因此 $D M D P=D B \\cdot D C$. (性质 4 的第(4)条)\n易证 $B 、 C 、 E 、 F$ 四点共圆.\n又因 $R Q / / E F$, 所以 $\\angle R Q C= \\angle P E C=\\angle R B C$.\n因此, $B 、 Q 、 C 、 R$ 四点共圆, 即 $D R \\cdot D Q=D B \\cdot D C=D M \\cdot D P$.\n由相交弦定理的逆定理知 $\\triangle P Q R$ 的外接圆过 $B C$ 的中点.", + "remark": "", + "figures": [ + "./images/volume7/figures/fig-c8i19.png" + ] +} \ No newline at end of file diff --git a/processed_dataset/proof/1752.json b/processed_dataset/proof/1752.json new file mode 100644 index 0000000000000000000000000000000000000000..8a302d135baf939991fc1070c81ed5ffaa00211e --- /dev/null +++ b/processed_dataset/proof/1752.json @@ -0,0 +1,10 @@ +{ + "source_file": "./raw_volume-zh/volume7/chapter8.tex", + "problem_type": "proof", + "problem": "例9. 凸四边形 $A B C D$ 的对角线交于点 $P$, 两组对边的直线分别交于点 $Q 、 R$, 经过 $P$ 的直线分别交 $A B 、 C D 、 Q R$ 于点 $M 、 N 、 G$.", + "solution": "证明: $\\frac{1}{M P}+\\frac{1}{M G}==\\frac{2}{M N}$.\n证明如图(), 设 $A C$ 与 $Q R$ 交于点 $S$, 则 $A$ 、 $C 、 P 、 S$ 成调和点列.\n连结 $Q P$. 考虑过 $Q$ 的四条线束 $Q A 、 Q P 、 Q D$ 、 $Q R$, 它们被两条直线 APCS 和 $M P N G$ 所截, 由于 $A$ 、 $C 、 P 、 S$ 成调和点列, 因此, $M 、 N 、 P 、 G$ 是调和点列, 即 $\\frac{1}{M P}+\\frac{1}{M G}=\\frac{2}{M N}$.", + "remark": "", + "figures": [ + "./images/volume7/figures/fig-c8i20.png" + ] +} \ No newline at end of file diff --git a/processed_dataset/proof/1753.json b/processed_dataset/proof/1753.json new file mode 100644 index 0000000000000000000000000000000000000000..ab2c72e9ea8bb3e3c052ad9bfe4b9d6adf1266d8 --- /dev/null +++ b/processed_dataset/proof/1753.json @@ -0,0 +1,10 @@ +{ + "source_file": "./raw_volume-zh/volume7/chapter8.tex", + "problem_type": "proof", + "problem": "例10. 如图(), 已知 $B 、 N 、 F$ 均在 $\\triangle A C E$ 的边上, $A N$ 分别交 $B F 、 B E 、 C F$ 于点 $M 、 G 、 H$. 求证: $\\frac{1}{A M}+\\frac{1}{A N}=-\\frac{1}{A G}+\\frac{1}{A H}$.", + "solution": "证明:设直线 $B F, C E$ 交于 $I$, 再设 $I D$ 与 $A N$ 交于 $J$, 由调和四边形的性质 2 知, $A 、 G 、 M 、 N$; $A 、 J 、 G 、 H$ 均为调和点列.\n所以左式 $=\\frac{2}{A J}=$ 右式.", + "remark": "", + "figures": [ + "./images/volume7/figures/fig-c8i21.png" + ] +} \ No newline at end of file diff --git a/processed_dataset/proof/1754.json b/processed_dataset/proof/1754.json new file mode 100644 index 0000000000000000000000000000000000000000..ebf118baabb6869b7868b669633c56b85db90176 --- /dev/null +++ b/processed_dataset/proof/1754.json @@ -0,0 +1,10 @@ +{ + "source_file": "./raw_volume-zh/volume7/chapter8.tex", + "problem_type": "proof", + "problem": "例11. 如图(), 圆 $O_1$ 和圆 $O_2$ 与 $\\triangle A B C$ 的三边所在的三条直线都相切, $E 、 F 、 G 、 H$ 为切点,并且 $E G 、 F H$ 的延长线交于点 $P$. 求证: 直线 $P A$ 与 $B C$ 垂直.", + "solution": "证明:设直线 $P A$ 交 $B C$ 于点 $D$.\n对 $\\triangle A B D$ 及截线 $P H F$, 对 $\\triangle A D C$ 及截线 $P G E$ 分别应用梅涅劳斯定理, 有\n$$\n\\frac{A H}{H} \\bar{B} \\cdot \\frac{B F}{F D} \\cdot \\frac{D P}{P A}=1=\\frac{D P}{P A} \\cdot \\frac{A G}{G} \\cdot \\frac{C E}{E D} .\n$$\n由切线性质, 有 $B F=H B, C E=G C$, 有 $\\frac{A H}{F D}=\\frac{A G}{E D}$, 即 $\\frac{E D}{D F}=\\frac{A G}{A H}$. 连结 $O_1 G 、 O_2 H$, 由 Rt $\\triangle A G O_1 \\backsim$ Rt $\\triangle A H O_2$, 知 $\\frac{A G}{A H}=\\frac{O_1 G}{O_2 H}$.\n连结 $O_1 E 、 O_2 F$, 则 $\\frac{A G}{A H}=\\frac{O_1 E}{O_2 F}$.\n连结 $O_1 D 、 O_2 D$, 则在 Rt $\\triangle O_1 E D$ 与 Rt $\\triangle O_2 F D$ 中, 有 $\\frac{E D}{D F}=\\frac{O_1 E}{O_2 F}$.\n于是, Rt $\\triangle O_1 E D \\backsim$ Rt $\\triangle O_2 F D$, 即有 $\\angle O_1 D E=\\angle O_2 D C$, 从而直线 $D F$ 为 $\\triangle O_1 D O_2$ 的 $\\angle O_1 D O_2$ 的外角平分线.\n设直线 $O_1 O_2$ 与直线 $E F$ 交于点 $Q$ (或无穷远点 $Q$ ), 从而点 $A 、 Q$ 调和分割 $O_1 O_2$ (由于 $A 、 Q$ 分别是内、外位似中心, $\\frac{O_1 A}{O_2 A}=\\frac{r_1}{r_2}=\\frac{O_1 Q}{Q O_2}$, 这里 $r_1 、 r_2$ 分别为 $\\odot O_1 、 \\odot O_2$ 的半径), 即 $D O_1 、 D O_2 、 D A 、 D Q$ 为调和线束, 于是知 $D A \\perp P Q$, 故 $P A \\perp B C$.", + "remark": "", + "figures": [ + "./images/volume7/figures/fig-c8i22.png" + ] +} \ No newline at end of file diff --git a/processed_dataset/proof/1755.json b/processed_dataset/proof/1755.json new file mode 100644 index 0000000000000000000000000000000000000000..95a9fa42aa78796daefe8afe9d226ac61bf04d76 --- /dev/null +++ b/processed_dataset/proof/1755.json @@ -0,0 +1,11 @@ +{ + "source_file": "./raw_volume-zh/volume7/chapter9.tex", + "problem_type": "proof", + "problem": "例1. 证明 Ptolemy 不等式:\n对平面上任意不共线的四点 $A 、 B 、 C 、 D$, 有 $A B \\cdot C D+B C \\cdot A D \\geqslant A C \\cdot B D$. 等号成立当且仅当 $A B C D$ 是圆内接凸四边形.", + "solution": "证明:如图(), 以 $A$ 为反演中心, 单位长度为反演半径, 设 $B 、 C 、 D$ 的反点分别为 $B^{\\prime} 、 C^{\\prime}$ 、 $D^{\\prime}$, 则\n$$\n\\begin{gathered}\nB^{\\prime} C^{\\prime}=\\frac{B C}{A B \\cdot A C}, C^{\\prime} D^{\\prime}=\\frac{C D}{A C \\cdot A D}, \\\\\nB^{\\prime} D^{\\prime}=\\frac{B D}{A B \\cdot \\bar{A} D},\n\\end{gathered}\n$$\n于是由 $B^{\\prime} C^{\\prime}+C^{\\prime} D^{\\prime} \\geqslant B^{\\prime} D^{\\prime}$ 得\n$$\n\\begin{aligned}\n& \\frac{B C}{A B \\cdot A C}+\\frac{C D}{A C \\cdot A D} \\geqslant \\frac{B D}{A B \\cdot A D}, \\\\\n& A B \\cdot C D+B C \\cdot A D \\geqslant A C \\cdot B D .\n\\end{aligned}\n$$\n等号成立条件是 $B^{\\prime} 、 C^{\\prime} 、 D^{\\prime}$ 共线且 $C^{\\prime}$ 在线段 $B^{\\prime} D^{\\prime}$ 上, 即 $A B C D$ 是圆内接凸四边形.", + "remark": "注:事实上, 对直线上顺次排列的四点 $A 、 B 、 C 、 D$, 有 Euler 恒等式:\n$$\nA B \\cdot C D+B C \\cdot A D==A C \\cdot B D .\n$$\n例 $2 A B$ 是圆 $O$ 的直径, $C$ 是 $A B$ 上一点, 过点 $C$ 作 $A B$ 的垂线交圆 $O$ 于点 $D$, 过点 $D$ 作圆 $O$ 的切线交 $A B$ 的延长线于点 $E, P$ 为圆 $O$ 上任意一点, 证明: $\\angle B P C=\\angle B P E$.\n证明如图(), 连结 $A D 、 A P 、 B D, D$ 点的极线过 $E$, 于是 $E$ 的极线过 $D$, 又 $E$ 的极线应与 $O E$ 垂直, 则 $C$ 在 $E$ 的极线上, 由性质 3 知, $A 、 B 、 C 、 E$ 成调和点列, 即 $P A 、 P B 、 P C$ 、 $P E$ 成调和线束, 再由 $A P \\perp P B$ 知 $\\angle C P B= \\angle B P E$.", + "figures": [ + "./images/volume7/figures/fig-c9i4.png", + "./images/volume7/figures/fig-c9i5.png" + ] +} \ No newline at end of file diff --git a/processed_dataset/proof/1756.json b/processed_dataset/proof/1756.json new file mode 100644 index 0000000000000000000000000000000000000000..bbfcb5663da6a9616ff91aef0beeb910ac392c9e --- /dev/null +++ b/processed_dataset/proof/1756.json @@ -0,0 +1,11 @@ +{ + "source_file": "./raw_volume-zh/volume7/chapter9.tex", + "problem_type": "proof", + "problem": "例3. 如图(), $Q$ 是以 $A B$ 为直径的圆上的一点, $Q \\neq A 、 B, Q$ 在 $A B$ 上的投影为 $H$. 以 $Q$ 为圆心、 $Q H$ 为半径的圆与以 $A B$ 为直径的圆交于点 $C$ 、 $D$. 证明: $C D$ 平分线段 $Q H$.", + "solution": "证明:作以 $Q$ 为反演中心、 $\\odot Q$ 为反演圆的反演变换.\n则 $\\odot O$ 反演为直线 $C D, A B$ 反演为以 $Q H$ 为直径且与 $\\odot Q$ 内切的圆 (如图()).\n因为 $A B$ 是 $\\odot O$ 的直径, 所以 $A B$ 与 $\\odot O$ 正交.\n由反演的保角性知, $C D$ 与以 $Q H$ 为直径的圆正交, 故 $C D$ 平分线段 $Q H$.", + "remark": "", + "figures": [ + "./images/volume7/figures/fig-c9i6.png", + "./images/volume7/figures/fig-c9i7.png" + ] +} \ No newline at end of file diff --git a/processed_dataset/proof/1757.json b/processed_dataset/proof/1757.json new file mode 100644 index 0000000000000000000000000000000000000000..be08bd9855e3bce04e70f5fb4586dcf3e4aa144f --- /dev/null +++ b/processed_dataset/proof/1757.json @@ -0,0 +1,10 @@ +{ + "source_file": "./raw_volume-zh/volume7/chapter9.tex", + "problem_type": "proof", + "problem": "例4. 四边形 $A B C D$ 内接于 $\\odot O$, 对角线 $A C$ 交 $B D$ 于 $P$. 设 $\\triangle A B P$ 、 $\\triangle B C P 、 \\triangle C D P 、 \\triangle D A P$ 的外接圆圆心分别为 $O_1 、 O_2 、 O_3 、 O_4$. 求证: $O P 、 \\mathrm{O}_1 \\mathrm{O}_3 、 \\mathrm{O}_2 \\mathrm{O}_4$ 三线共点.", + "solution": "证明:如图(), 作以 $P$ 为反演中心、 $P$ 关于 $\\odot O$ 的幂为反演幂的反演变换.\n则 $\\odot O$ 反演为本身, $\\odot O_i (i=1,2,3,4)$ 反演为四边形 $A B C D$ 各边所在的直线, 过点 $P$ 的直线也反演为本身.\n由于直线 $P O_2$ 与 $\\odot O_2$ 正交, 因此, 它们的反形也正交, 即 $\\mathrm{PO}_2 \\perp A D$.\n又易知 $O_4 O \\perp A D$, 则 $P_2 / / O_4 O$.\n同理, $\\mathrm{PO}_4 / / \\mathrm{O}_2 \\mathrm{O}$.\n因此, 四边形 $\\mathrm{PO}_2 \\mathrm{OO}_4$ 为平行四边形, $\\mathrm{PO}$ 与 $\\mathrm{O}_2 \\mathrm{O}_4$ 互相平分.\n同理, $P O$ 与 $O_1 O_3$ 互相平分.\n故 $\\mathrm{PO} 、 \\mathrm{O}_1 \\mathrm{O}_3 、 \\mathrm{O}_2 \\mathrm{O}_4$ 交于 $\\mathrm{PO}$ 的中点.", + "remark": "", + "figures": [ + "./images/volume7/figures/fig-c9i8.png" + ] +} \ No newline at end of file diff --git a/processed_dataset/proof/1758.json b/processed_dataset/proof/1758.json new file mode 100644 index 0000000000000000000000000000000000000000..36bb2dff6b44416d4a216d4ec6929987aed3f882 --- /dev/null +++ b/processed_dataset/proof/1758.json @@ -0,0 +1,10 @@ +{ + "source_file": "./raw_volume-zh/volume7/chapter9.tex", + "problem_type": "proof", + "problem": "例5. 已知圆 $O$ 外一点 $X$, 由 $X$ 向圆 $O$ 引两条切线, 切点分别为 $A 、 B$, 过点 $X$ 作直线, 与圆 $O$ 交于两点 $C 、 D$, 且满足 $C A \\perp B D$. 若 $C A$ 与 $B D$ 交于点 $F, C D$ 与 $A B$ 交于点 $G, B D$ 与 $G X$ 的中垂线交于点 $H$. 证明: $X 、 F 、 G 、 H$ 四点共圆.", + "solution": "证明:由配极性质 3 可知 $(X 、 G 、 D 、 C)$ 为调和点列, 而 $C A \\perp B D$, 故由调和点列的有关性质知 $\\angle G F D=\\angle D F X$.\n如图(), 设 $\\triangle G F X$ 的外接圆与 $B F$ 交于点 $H^{\\prime}$. 则 $G H^{\\prime}=X H^{\\prime}$, 即点 $H^{\\prime}$ 在 $G X$ 的中垂线上.\n从而, $H^{\\prime}=H$. 因此, $X 、 F 、 G 、 H$ 四点共圆.", + "remark": "", + "figures": [ + "./images/volume7/figures/fig-c9i9.png" + ] +} \ No newline at end of file diff --git a/processed_dataset/proof/1759.json b/processed_dataset/proof/1759.json new file mode 100644 index 0000000000000000000000000000000000000000..03a5c3d1c1eaf83ad14a936dfdaeab118feacd87 --- /dev/null +++ b/processed_dataset/proof/1759.json @@ -0,0 +1,10 @@ +{ + "source_file": "./raw_volume-zh/volume7/chapter9.tex", + "problem_type": "proof", + "problem": "例6. 如图(), $A B$ 为圆 $\\omega$ 的直径, 直线 $l$ 切 $\\odot \\omega$ 于 $A$. C、M、D 在 $l$ 上满足 $C M=D M$, 又设 $B C 、 B D$ 交 $\\odot \\omega$ 于 $P 、 Q, \\odot \\omega$ 切线 $P R 、 Q R$ 交于 $R$. 求证: $R$ 在 $B M$ 上.", + "solution": "证明:过 $B$ 作 $C D$ 平行线 $l^{\\prime}$, 则 $B C 、 B D, B M 、 l^{\\prime}$ 成调和线束, $A B$ 过圆心, $C D$ 为切线, $l^{\\prime} / / C D$, 所以 $l^{\\prime}$ 为圆的切线, 于是, $B B$ (过 $B$ 的切线)、 $B P$ 、 $B T 、 B Q$ 成调和线束, 因此结合定理 2 有, 四边形 $B P T Q$ 为调和四边形, 根据定理 $3, R$ 为直线 $P Q$ 的极点, 因此在直线 $B T(B M)$ 上.", + "remark": "", + "figures": [ + "./images/volume7/figures/fig-c9i10.png" + ] +} \ No newline at end of file diff --git a/processed_dataset/proof/1760.json b/processed_dataset/proof/1760.json new file mode 100644 index 0000000000000000000000000000000000000000..a866458c0046ea6fe622b2b7eb588cfe188bff87 --- /dev/null +++ b/processed_dataset/proof/1760.json @@ -0,0 +1,10 @@ +{ + "source_file": "./raw_volume-zh/volume7/chapter9.tex", + "problem_type": "proof", + "problem": "例7. 如图(), $\\triangle A B C$ 的内切圆 $\\odot I$ 切 $B C$ 、 $C A 、 A B$ 于 $D 、 E 、 F, A D$ 与 $\\odot I$ 的另一个交点 $X$, $B X 、 C X$ 分别交 $\\odot I$ 于 $P 、 Q$. 又记 $B C$ 中点为 $M$. 若 $A X=X D$, 求证: $F P / / E Q$.", + "solution": "证明:连结 $E Q$, 设 $E D$ 与 $C X$ 交于 $R$, 由定理 3 知, 四边形 $E Q D X$ 为调和四边形, 于是 $E E$ (过 $E$ 的切线)、 $E D, E Q 、 E X$ 成调和线束, 或 $E A 、 E D, E X 、 E Q$ 成调和线束, 结合 $A X=X D$ 即知 $E Q / / A D$, 同理, $P F / / A D$, 故 $P F / / E Q$.", + "remark": "", + "figures": [ + "./images/volume7/figures/fig-c9i11.png" + ] +} \ No newline at end of file diff --git a/processed_dataset/proof/1761.json b/processed_dataset/proof/1761.json new file mode 100644 index 0000000000000000000000000000000000000000..937917147878253a8b7e8fe2ff713abb50b6be9d --- /dev/null +++ b/processed_dataset/proof/1761.json @@ -0,0 +1,11 @@ +{ + "source_file": "./raw_volume-zh/volume7/chapter9.tex", + "problem_type": "proof", + "problem": "例8. 如图(), 设凸四边形 $A B C D$ 对角线交于 $O$ 点.\n$\\triangle O A D, \\triangle O B C$ 的外接圆交于 $O, M$ 两点, 直线 $O M$ 分别交 $\\triangle O A B, \\triangle O C D$ 的外接圆于 $T$, $S$ 两点.\n求证: $M$ 是线段 $T S$ 的中点.", + "solution": "证明:以 $O$ 为反演中心, 单位长度为反演半径作反演.\n反形如图() 所示, 则 $S M=T M$ 的充要条件是 $T O-O M=O S+O M$, 即 $T O+2 O M=O S$, 这就等价于 $\\frac{1}{O S^{\\prime}}+\\frac{2}{O M^{\\prime}}=\\frac{1}{O T^{\\prime}}$.\n由上章性质 2 知, $T^{\\prime} 、 S^{\\prime}, O 、 M^{\\prime}$ 成调和点列, 于是 $\\frac{1}{O S^{\\prime}}+\\frac{2}{O M^{\\prime}}=\\frac{1}{O T^{\\prime}}$.", + "remark": "注:两圆相交时, 通常以其中一个交点为反演中心, 则两圆反形为两条相交的直线, 交点为两圆的另一个交点的反演点.", + "figures": [ + "./images/volume7/figures/fig-c9i12.png", + "./images/volume7/figures/fig-c9i13.png" + ] +} \ No newline at end of file diff --git a/processed_dataset/proof/1762.json b/processed_dataset/proof/1762.json new file mode 100644 index 0000000000000000000000000000000000000000..744692dd46177c83f72d59b1b4e8c3e09d96b027 --- /dev/null +++ b/processed_dataset/proof/1762.json @@ -0,0 +1,10 @@ +{ + "source_file": "./raw_volume-zh/volume7/chapter9.tex", + "problem_type": "proof", + "problem": "例9. 如图(),已知圆 $O$ 中, $C G$ 为直径, 过点 $G$ 作一条直线.\n在直线上截取 $A G=B G(A$, $B$ 均在圆外). 连结 $A C, B C$. 分别交圆 $O$ 于点 $D$, $E$. 过 $D, E$ 分别作圆 $O$ 切线.\n交于一点 $P$, 连结 $P G$. 求证: $P G$ 垂直于 $A B$.", + "solution": "证明:过 $C$ 作 $A B$ 平行线交圆 $O$ 于 $Y$, 过 $G$ 做圆 $O$ 切线交直线 $D E$ 于 $X$,则 $X$ 在 $G$ 的极线即过 $G$ 的切线上, $X$ 还在 $P$ 的极线 $D E$ 上,所以 $X$ 的极线过 $G 、 P$, 又 $A G=G B$, 故 $A 、 B, G 、 \\infty$ 成调和点列, 所以 $C A 、 C B 、 C G 、 C \\infty$ 成调和线束,于是 $D G E Y$ 是调和四边形.\n从而 $X Y$ 是切线, 即 $X$ 在 $Y$ 的极线上, 那么 $X$ 的极线 $G P$ 过 $Y$ 点, 结合 $G Y$ 垂直于 $C Y$, 即 $P G$ 垂直于 $A B$.", + "remark": "", + "figures": [ + "./images/volume7/figures/fig-c9i14.png" + ] +} \ No newline at end of file diff --git a/processed_dataset/proof/1763.json b/processed_dataset/proof/1763.json new file mode 100644 index 0000000000000000000000000000000000000000..a174bad408a1589b01ade9e684e442aa4ee93d86 --- /dev/null +++ b/processed_dataset/proof/1763.json @@ -0,0 +1,10 @@ +{ + "source_file": "./raw_volume-zh/volume7/chapter9.tex", + "problem_type": "proof", + "problem": "例10. 给定 4 个圆 $\\odot S_1 、 \\odot S_2 、 \\odot S_3$ 、 $\\odot S_4$, 设 $\\odot S_1$ 和 $\\odot S_2 、 \\odot S_2$ 和 $\\odot S_3 、 \\odot S_3$ 和 $\\odot S_4 、 \\odot S_4$ 和 $\\odot S_1$ 分别交于点 $A_1$ 和 $A_2 、 B_1$ 和 $B_2 、 C_1$ 和 $C_2 、 D_1$ 和 $D_2$. 若 $A_1 、 B_1 、 C_1 、 D_1$ 四点共圆 (或共线), 证明: $A_2 、 B_2 、 C_2 、 D_2$ 四点共圆(或共线).", + "solution": "证明:作以 $A_1$ 为反演中心的反演变换, 于是, $\\odot S_1 、 \\odot S_2$ 反形为直线 $A_2^{\\prime} D_1^{\\prime} 、 A_2^{\\prime} B_1^{\\prime}, \\odot S_3$ 、 $\\odot S_4$ 反形为 $\\triangle B_2^{\\prime} C_1^{\\prime} B_1^{\\prime} 、 \\triangle D_2^{\\prime} C_1^{\\prime} D_1^{\\prime}$ 的外接圆, 这两个圆交于 $C_2^{\\prime}$. 如图() 只要证 $A_2^{\\prime} 、 B_2^{\\prime} 、 C_2^{\\prime} 、 D_2^{\\prime}$ 四点共圆即可.\n这就转化为三角形中的密克点问题:\n$\\triangle A_2^{\\prime} B_1^{\\prime} D_1^{\\prime}$ 中, $C_1^{\\prime} 、 B_2^{\\prime} 、 D_2^{\\prime}$ 分别在边 $D_1^{\\prime} B_1^{\\prime} 、 B_1^{\\prime} A_2^{\\prime} 、 A_2^{\\prime} D_1^{\\prime}$ 上, 若 $\\triangle D_2^{\\prime} D_1^{\\prime} C_1^{\\prime}$ 的外接圆与 $\\triangle B_1^{\\prime} B_2^{\\prime} C_1^{\\prime}$ 的外接圆交于点 $C_2^{\\prime}$, 则 $\\triangle A_2^{\\prime} D_2^{\\prime} B_2^{\\prime}$ 的外接圆也过该点.\n这个问题在圆的一章中就已经提到过.", + "remark": "", + "figures": [ + "./images/volume7/figures/fig-c9i15.png" + ] +} \ No newline at end of file diff --git a/processed_dataset/proof/1764.json b/processed_dataset/proof/1764.json new file mode 100644 index 0000000000000000000000000000000000000000..883be1565f458f585eb8b2ec7dbd46cf51178539 --- /dev/null +++ b/processed_dataset/proof/1764.json @@ -0,0 +1,10 @@ +{ + "source_file": "./raw_volume-zh/volume7/chapter9.tex", + "problem_type": "proof", + "problem": "例11. 如图(), 凸四边形 $A B C D$ 有内切圆, 且内切圆分别切边 $A B 、 B C 、 C D 、 D A$ 于 $A_1 、 B_1 、 C_1$ 、 $D_1$, 点 $E 、 F 、 G 、 H$ 分别为线段 $A_1 B_1 、 B_1 C_1 、 C_1 D_1$ 、 $D_1 A_1$ 的中点, 证明: 四边形 $E F G H$ 为矩形的充要条件是 $A 、 B 、 C 、 D$ 共圆.", + "solution": "证明:以 $A B C D$ 内切圆为反演圆作反演变换, 则由反演定理 4, $A 、 B 、 C 、 D$ 的反点分别为 $H 、 E 、 F 、 G$, 因为不过反演中心的圆的反形仍是一个圆, 于是 $A 、 B 、 C 、 D$ 共圆等价于 $E 、 F 、 G 、 H$ 共圆.\n注意 $E 、 F 、 G 、 H$ 分别是为四边形 $A_1 B_1 C_1 D_1$ 四边形的中点, 所以四边形 $E F G H$ 是一个平行四边形, 因而 $E 、 F 、 G 、 H$ 四点共圆的充要条件是平行四边形 $E F G H$ 是矩形, 这又等价于 $A 、 B 、 C 、 D$ 共圆.", + "remark": "注:此题曾在圆的初步中讲解过, 这里给出一种反演变换的解法.", + "figures": [ + "./images/volume7/figures/fig-c9i16.png" + ] +} \ No newline at end of file diff --git a/processed_dataset/proof/1765.json b/processed_dataset/proof/1765.json new file mode 100644 index 0000000000000000000000000000000000000000..2a216b0af507d3699960cce8e4fb6ecdc4cc00e4 --- /dev/null +++ b/processed_dataset/proof/1765.json @@ -0,0 +1,10 @@ +{ + "source_file": "./raw_volume-zh/volume7/chapter9.tex", + "problem_type": "proof", + "problem": "例12. 如图(), 圆内接四边形 $A B C D$ 内有一点 $P$ 满足 $\\angle A P D=\\angle A B P+\\angle D C P . P$ 在 $A B 、 B C 、 C D$ 上射影为 $E 、 F 、 G$. 证明: $\\triangle E F G \\backsim \\triangle A P D$.", + "solution": "证明:法一: 因为 $\\angle E F G=\\angle E F P+\\angle G F P= \\angle E B P+\\angle G C P=\\angle A P D$, 故只需证 $\\frac{A P}{P D}=\\frac{E F}{F G}$. 又 $\\frac{E F}{F G}=\\frac{P B \\sin B}{P C \\sin C}=\\frac{P B \\cdot A C}{P C \\cdot B D}$, 故只需证\n$$\n\\frac{A P}{P D}=\\frac{P B \\cdot A C}{P C \\cdot B D} \\Leftrightarrow \\frac{A P \\cdot P C}{A C}=\\frac{B P \\cdot P D}{B D} . \\label{eq1}\n$$\n又因 $\\angle A P D=\\angle A B P+\\angle D C P$, 所以 $\\triangle A B P$ 的外接圆与 $\\triangle D C P$ 外接圆外切于点 $P$.\n作以 $P$ 为反演中心, $P$ 对 $A B C D$ 外接圆的幂为反演幂作反演变换.\n则 $A 、 B 、 C 、 D$ 分别变为 $A^{\\prime} 、 B^{\\prime} 、 C^{\\prime} 、 D^{\\prime}$, 且 $A^{\\prime}$ 是 $A P$ 与 $A B C D$ 外接圆的交点, $B^{\\prime} 、 C^{\\prime} 、 D^{\\prime}$ 类似.\n因为 $\\triangle A B P 、 \\triangle C D P$ 外接圆外切于 $P$.\n故用反演性质知 $A^{\\prime} B^{\\prime} / / C^{\\prime} D^{\\prime} \\Rightarrow A^{\\prime} B^{\\prime} C^{\\prime} D^{\\prime}$ 为等腰梯形 $\\Rightarrow A^{\\prime} C^{\\prime}=B^{\\prime} D^{\\prime}$.\n由反演变换距离公式知\n$A^{\\prime} C^{\\prime}=A C \\times \\frac{|d|}{P A \\cdot P C}, B^{\\prime} D^{\\prime}=B D \\times \\frac{|d|}{P B \\cdot P D}$ ( $d$ 为反演幕 $)$.\n所以式\\ref{eq1} $\\Leftrightarrow A^{\\prime} C^{\\prime}=B^{\\prime} D^{\\prime}$, 此式已证明成立,故原题得证.", + "remark": "", + "figures": [ + "./images/volume7/figures/fig-c9i17.png" + ] +} \ No newline at end of file diff --git a/processed_dataset/proof/1766.json b/processed_dataset/proof/1766.json new file mode 100644 index 0000000000000000000000000000000000000000..4081b7d42781e42af560fb8c8b6fb6a52b48fc58 --- /dev/null +++ b/processed_dataset/proof/1766.json @@ -0,0 +1,11 @@ +{ + "source_file": "./raw_volume-zh/volume7/chapter9.tex", + "problem_type": "proof", + "problem": "例12. 如图(), 圆内接四边形 $A B C D$ 内有一点 $P$ 满足 $\\angle A P D=\\angle A B P+\\angle D C P . P$ 在 $A B 、 B C 、 C D$ 上射影为 $E 、 F 、 G$. 证明: $\\triangle E F G \\backsim \\triangle A P D$.", + "solution": "法二: 如图(), 作 $\\angle A P T=\\angle A B P$, 其中 $T$ 为 $A D$ 上的点, $T P$ 父 $B C$ 于 $S$.\n由 $\\angle A P D=\\angle A B P+\\angle D C P$ 知, $\\angle T P D=\\angle D C P$.\n易知 $P T$ 为 $\\triangle A B P 、 \\triangle D C P$ 外接圆的切线.\n由根轴定理知 $B A 、 C D 、 S T$ 交于一点, 设为 $R$.\n$$\n\\begin{aligned}\n\\angle P A D & =\\angle B A D-\\angle B A P=\\pi-\\angle B C R-\\angle B P S \\\\\n& =\\angle B P R-\\angle B C R(\\angle B P R=\\pi-\\angle B P S) \\\\\n& =\\angle B P R-(\\angle B S R-\\angle S R C) \\\\\n& =\\angle B P R-\\angle B S R+\\angle S R C \\\\\n& =\\angle P B S+\\angle P E G \\\\\n& =\\angle P E F+\\angle P E G=\\angle F E G .\n\\end{aligned} \\label{eq1}\n$$\n由已知条件易知 $P 、 E 、 B 、 F$ 和 $P 、 G 、 C 、 F$ 都有四点共圆.\n$$\n\\begin{aligned}\n\\angle A B P=\\angle E F P & , \\angle P C G=\\angle P F G, \\text { 故 } \\\\\n\\angle A P D & =\\angle A B P+\\angle P C G \\\\\n& =\\angle E F G+\\angle P F G=\\angle E F G\n\\end{aligned} \\label{eq2}\n$$\n由式\\ref{eq1}、\\ref{eq2}知 $\\triangle E F G \\backsim \\triangle A P D$.", + "remark": "", + "figures": [ + "./images/volume7/figures/fig-c9i17.png", + "./images/volume7/figures/fig-c9i18.png" + ] +} \ No newline at end of file diff --git a/processed_dataset/proof/1767.json b/processed_dataset/proof/1767.json new file mode 100644 index 0000000000000000000000000000000000000000..a5bec5508d0861995cd2274ea0eba2feff16ec54 --- /dev/null +++ b/processed_dataset/proof/1767.json @@ -0,0 +1,11 @@ +{ + "source_file": "./raw_volume-zh/volume7/chapter9.tex", + "problem_type": "proof", + "problem": "例13. 双心四边形 $A B C D, A C \\cap B D=E$, 内、外心为 $I 、 O$. 求证: $I 、 O 、 E$ 三点共线.", + "solution": "证明:先证一个引理.\n引理: 圆外切四边形 $A B C D$, 切点为 $M 、 N 、 K$ 、 $L$, 则 $A C 、 B D 、 M K 、 N L$ 四线共点.\n引理的证明: 如图(), 设 $A C \\cap K M=G$, $L N \\cap K M=G^{\\prime}$, 由正弦定理得\n$$\n\\begin{aligned}\n\\frac{G C}{A G} & =\\frac{C M \\frac{\\sin \\angle G M C}{\\sin \\angle C G M}}{A K \\frac{\\sin \\angle A K G}{\\sin \\angle A G K}} \\\\\n& =\\frac{C M}{A K} \\frac{\\sin \\angle G M C}{\\sin \\angle A K G} \\frac{\\sin \\angle A G K}{\\sin \\angle C G M}=\\frac{C M}{A K} .\n\\end{aligned}\n$$\n同理 $\\frac{G^{\\prime} C}{A G^{\\prime}}=\\frac{C L}{A N}$.\n所以 $\\frac{G^{\\prime} C}{A G^{\\prime}}=\\frac{C L}{A N}=\\frac{C M}{A K}=\\frac{C G}{A G}$, 即 $G=G^{\\prime}$.\n故 $A C 、 N L 、 K M$ 三线共点.\n同理 $B D 、 K M 、 L N$ 三线共点,引理得证.\n回到原题: 如图(), 切点仍记为 $K 、 L 、 M$ 、 $N$, 由引理 $K M \\cap L N=E$.\n以 $I$ 为中心, $\\odot(K N M)$ 为反演圆作反演, $A^{\\prime}$ 、 $B^{\\prime} 、 C^{\\prime} 、 D^{\\prime}$ 分别为 $K L M N$ 四边中点.\n由 $B^{\\prime} C^{\\prime} / / K M / / A^{\\prime} D^{\\prime}, A^{\\prime} B^{\\prime} / / N L / / D^{\\prime} C^{\\prime}$ 知 $A^{\\prime} B^{\\prime} C^{\\prime} D^{\\prime}$ 为平行四边形.\n而 $A 、 B 、 C 、 D$ 共圆知 $A^{\\prime} 、 B^{\\prime} 、 C^{\\prime} 、 D^{\\prime}$ 共圆, $A^{\\prime} B^{\\prime} C^{\\prime} D^{\\prime}$ 必为矩形, 其中心设为 $Q$, 且有 $K M \\perp L N$.\n由反演性质知 $Q 、 I 、 O$ 三点共线.\n设 $L N 、 K M$ 中点为 $P 、 R$, 则\n$$\n\\begin{aligned}\n\\overrightarrow{I Q^{\\prime}} & =\\frac{1}{4}\\left(\\overrightarrow{I A^{\\prime}}+\\overrightarrow{I B^{\\prime}}+\\overrightarrow{I C^{\\prime}}+\\overrightarrow{I D^{\\prime}}\\right) \\\\\n& =\\frac{1}{4}(\\overrightarrow{I K}+\\overrightarrow{I L}+\\overrightarrow{I M}+\\overrightarrow{I N})=\\frac{1}{2}(\\overrightarrow{I R}+\\overrightarrow{I P}) .\n\\end{aligned}\n$$\n由垂径定理知 $P I R E$ 为矩形.\n从而 $\\overrightarrow{I R}+\\overrightarrow{I P}=\\overrightarrow{I E}$.\n故 $\\overrightarrow{I Q}=\\frac{1}{2} \\overrightarrow{I E}$, 即 $I 、 Q 、 E$ 三点共线, 从而 $O 、 I 、 E$ 三点共线.", + "remark": "", + "figures": [ + "./images/volume7/figures/fig-c9i19.png", + "./images/volume7/figures/fig-c9i20.png" + ] +} \ No newline at end of file diff --git a/processed_dataset/proof/1768.json b/processed_dataset/proof/1768.json new file mode 100644 index 0000000000000000000000000000000000000000..17b912d2e7fc07605c5a9382cc10f940e6ce0553 --- /dev/null +++ b/processed_dataset/proof/1768.json @@ -0,0 +1,11 @@ +{ + "source_file": "./raw_volume-zh/volume7/exercise1.tex", + "problem_type": "proof", + "problem": "问题1. 在圆内接四边形 $A B C D$ 中, $F 、 G$ 分别为 $A C 、 B D$ 的中点.\n[1] 证明: 若 $\\angle B$ 与 $\\angle D$ 的平分线的交点恰好在 $A C$ 上, 则 $\\frac{1}{4} A C \\cdot B D= \\sqrt{A G \\cdot B F} \\cdot C G \\cdot D F$. (2) (1) 的逆命题一定成立吗?", + "solution": "证明: [1] 如图(), 设 $E$ 为 $\\angle B$ 与 $\\angle D$ 平分线的交点.\n由角分线定理有 $\\frac{A B}{B C}=$ i $\\frac{A E}{E C}=\\frac{A D}{D C} \\cdots$ (1). 由托勒密定理有 $A B \\cdot C D+A D \\cdot B C=A C \\cdot B D$. 结合式 (1) 有 $2 B C \\cdot A D=A C \\cdot B D \\cdots$ (2). $2 A B \\cdot C D=A C \\cdot B D \\cdots$ (3).\n由式(2)得 $\\frac{F A}{A D}= \\frac{A C}{2 A D}=\\frac{B C}{B D}$. 又 $\\angle F A D=\\angle C A D=\\angle C B D$, 故 $\\triangle F A D \\backsim \\triangle C B D$. 同理, $\\triangle F A B \\backsim \\triangle C D B$. 因此, $\\triangle F A D \\backsim \\triangle F B A$, 有 $\\frac{F A}{F D}=\\frac{F B}{F A}$. 故 $\\frac{1}{4} A C^2=F B \\cdot F D \\cdots$ (4). \n又由式 (1) 知 $\\frac{D A}{A B}=\\frac{D C}{C B}$, 这意味着 $\\angle A$ 与 $\\angle C$ 平分线的交点在 $B D$ 上.\n仿上述证法有, $\\frac{1}{4} B D^2=A G \\cdot C G \\cdots$ (5). (4) $\\times$(5)即可得证.\n[2] [1] 的逆命题不一定成立.\n如图(), 设四边形 $A B C D$ 为矩形, $A B>B C$. 显然, $A G=B F=C G=D F= \\frac{1}{2} A C=\\frac{1}{2} B D$, 满足 $\\left(\\frac{1}{4} A C \\cdot B D\\right)^2=A G \\cdot B F \\cdot C G \\cdot D F$. 但 $\\frac{A B}{B C}>1>\\frac{A D}{D C}$,\n说明 $\\angle B$ 与 $\\angle D$ 的平分线不在 $A C$ 相交.\n因此, 逆命题为假命题.", + "remark": "", + "figures": [ + "./images/volume7/figures/fig-c1a1-1.png", + "./images/volume7/figures/fig-c1a1-2.png" + ] +} \ No newline at end of file diff --git a/processed_dataset/proof/1769.json b/processed_dataset/proof/1769.json new file mode 100644 index 0000000000000000000000000000000000000000..c4f7f3f53946d3282bab30baccd73e5f9d601812 --- /dev/null +++ b/processed_dataset/proof/1769.json @@ -0,0 +1,11 @@ +{ + "source_file": "./raw_volume-zh/volume7/exercise1.tex", + "problem_type": "proof", + "problem": "问题2. 在锐角不等边 $\\triangle A B C$ 中, $A C>B C$. 设 $O 、 H$ 分别是 $\\triangle A B C$ 的外心、垂心, $C F \\perp A B$ 于点 $F$. 令 $P$ 是直线 $A B$ 上一点 $(P \\neq A)$, 满足 $A F=P F$. 记 $G$ 是边 $A C$ 的中点, 直线 $P H$ 与 $B C 、 O G$ 与 $F X 、 O F$ 与 $A C$ 分别交于点 $X 、 Y 、 Z$. 证明: $F 、 G 、 Z 、 Y$ 四点共圆.", + "solution": "证明: 如图(), , 首先, 由 $G$ 是 $A C$ 中点, $O 、 H$ 分别为 $\\triangle A B C$ 的外心, 垂心, 及欧拉定理知: $G 、 O 、 H 、 B$ 四点共线.\n要证 $Z 、 G 、 F 、 Y$ 四点共圆, 由 $\\angle Y G Z=\\angle O G Z=90^{\\circ}$, 知只需证 $\\angle O F Y=90^{\\circ}$, 即 $\\angle O F X=90^{\\circ}$.\n如图(), , 作 $O E \\perp A B$. 则 $C H=2 O E$. 由题设有 $P B=P F-B F=A F- B F=2 E F$. 另一方面, 由 $\\angle H P B=\\angle H A B=\\angle H C B$, 知 $P 、 B 、 H 、 C$ 四点共圆.\n因此, $\\triangle P X B \\backsim \\triangle C X H$. 作 $X L \\perp A B$ 于点 $L, X N \\perp C F$ 于点 $N$. 则 $\\frac{X L}{L F}= \\frac{X L}{X N}=\\frac{P B}{C H}=\\frac{2 E F}{2 O E}=\\frac{E F}{O E}$. 又 $\\angle X L F=\\angle F E O=90^{\\circ}$. 于是, $\\triangle X L F \\backsim \\triangle F E O$. 从而, $\\angle X F L=\\angle F O E$. 故 $\\angle X F O=180^{\\circ}-\\angle X F L-\\angle O F E= 180^{\\circ}-\\angle F O E-\\angle O F E=90^{\\circ}$.", + "remark": "", + "figures": [ + "./images/volume7/figures/fig-c1a2-1.png", + "./images/volume7/figures/fig-c1a2-2.png" + ] +} \ No newline at end of file diff --git a/processed_dataset/proof/1770.json b/processed_dataset/proof/1770.json new file mode 100644 index 0000000000000000000000000000000000000000..3e5eeebc84421aa4be12562edd1d28f687f6819e --- /dev/null +++ b/processed_dataset/proof/1770.json @@ -0,0 +1,10 @@ +{ + "source_file": "./raw_volume-zh/volume7/exercise1.tex", + "problem_type": "proof", + "problem": "问题3. 连结三角形内切圆的圆心和它的顶点的直线将原三角形分为三个三角形.\n若它们之中的一个三角形与原三角形相似,求三角形三个角的度数.", + "solution": "证明: 如图(), 设 $\\triangle A B C$ 的三个内角分别为 $\\alpha 、 \\beta$ 、 $\\gamma, O$ 为其内切圆圆心.\n点 $O$ 和 $\\triangle A B C$ 的三个顶点的连线是其三个内角的平分线.\n因为 $\\angle O B C=\\frac{\\beta}{2}$, $\\angle O C B=\\frac{\\gamma}{2}$, 则 $\\angle C O B=180^{\\circ}-\\frac{\\beta}{2}-\\frac{\\gamma}{2}=90^{\\circ}+\\frac{\\alpha}{2}$. 不失一般性, 设 $\\triangle A B C$ 和 $\\triangle C O B$ 相似.\n于是, $\\angle C O B$ 等于 $\\alpha, \\beta, \\gamma$ 中的一个.\n若 $\\angle C O B=\\alpha$, 则 $90^{\\circ}+\\frac{\\alpha}{2}=\\alpha$, 即\n$\\alpha=180^{\\circ}$, 这是不可能的.\n因此, $\\angle C O B=\\beta$ 或 $\\angle C O B=\\gamma$. 不妨设 $\\angle C O B=\\beta$, 则有 $\\angle O B C=\\alpha$ 或 $\\angle O B C=\\gamma$. 若是第一种情况, 有 $\\angle B C O=\\gamma$, 则 $\\frac{\\gamma}{2}=\\gamma$, 不可能.\n因此, $\\angle O B C=\\gamma, \\angle B C O=\\alpha$. 于是, $\\gamma=2 \\alpha, \\beta=2 \\gamma=4 \\alpha$. 因为 $\\alpha+\\beta+\\gamma= 180^{\\circ}$, 即 $\\alpha+2 \\alpha+4 \\alpha=180^{\\circ}$, 则 $\\alpha=\\left(\\frac{180}{7}\\right)^{\\circ}, \\beta=\\left(\\frac{720}{7}\\right)^{\\circ}, \\gamma=\\left(\\frac{360}{7}\\right)^{\\circ}$.", + "remark": "", + "figures": [ + "./images/volume7/figures/fig-c1a3.png" + ] +} \ No newline at end of file diff --git a/processed_dataset/proof/1771.json b/processed_dataset/proof/1771.json new file mode 100644 index 0000000000000000000000000000000000000000..c85cea526fdd9798e0fc8404eed93b1018fc04a6 --- /dev/null +++ b/processed_dataset/proof/1771.json @@ -0,0 +1,10 @@ +{ + "source_file": "./raw_volume-zh/volume7/exercise1.tex", + "problem_type": "proof", + "problem": "问题4. 半圆 $\\Gamma$ 的直径是 $A B, M$ 是 $A B$ 的中点.\n在半圆 $\\Gamma$ 的同侧, 以 $M B$ 为直径作半圆 $\\Gamma_1$. 设 $X 、 Y$ 是半圆 $\\Gamma_1$ 上的点, 且 $\\overparen{B X}=1.5 \\overparen{B Y}$. 直线 $M Y$ 交 $B X$ 于点 $D$, 交半圆 $\\Gamma$ 于点 $C$. 证明: $Y$ 是线段 $C D$ 的中点.", + "solution": "证明: 如图(), 取 $\\overparen{B Y}$ 的中点 $Z$, 设 $B Z 、 M Y$ 相交于点 $E$, 在半圆 $\\Gamma_1$ 中, $\\angle M Z B$ 是直径 $M B$ 所对的圆心角, 于是, $M Z \\perp B Z$. 因为 $\\overparen{B Z}=\\overparen{Z Y}$, 所以, $\\angle Y M Z=\\angle B M Z$. 在 $\\triangle M Z B$ 和 $\\triangle M Z E$ 中, 由 $\\angle E M Z=\\angle B M Z, M Z=M Z, \\angle M Z E=\\angle M Z B=90^{\\circ}$, 所以, $\\triangle M Z B \\cong \\triangle M Z E$. 故 $M E=M B$. 因此, 点 $E$ 和 $C$ 重合.\n又 $\\angle M Y B$ 是直径 $M B$ 所对的圆心角, 于是, $M Y \\perp B Y$. 因为 $\\overparen{Z Y}=\\overparen{Y X}$, 所以, $\\angle Z B Y=\\angle Y B X$. 同理, $\\triangle B Y C \\cong \\triangle B Y D$. 故 $C Y=D Y$.", + "remark": "", + "figures": [ + "./images/volume7/figures/fig-c1a4.png" + ] +} \ No newline at end of file diff --git a/processed_dataset/proof/1772.json b/processed_dataset/proof/1772.json new file mode 100644 index 0000000000000000000000000000000000000000..7844712f408f2b36b4e0136646b97e5183e24c71 --- /dev/null +++ b/processed_dataset/proof/1772.json @@ -0,0 +1,11 @@ +{ + "source_file": "./raw_volume-zh/volume7/exercise1.tex", + "problem_type": "proof", + "problem": "问题5. 设 $X 、 Y 、 Z$ 分别是菱形 $A B C D$ 边 $A B 、 B C 、 C D$ 上的点, 且使得 $X Y / / A Z$. 证明: $X Z 、 A Y 、 B D$ 三线共点.", + "solution": "证明: 如图(), 注意到 $\\angle B Y X=\\angle Z A D, \\angle X B Y=\\angle Z D A$, 所以, $\\triangle X B Y \\backsim \\triangle Z D A$. 故 $\\frac{B X}{B Y}=\\frac{D Z}{D A}$.\n设 $K 、 L$ 分别是对角线 $B D$ 与线段 $X Y 、 A Z$ 的交点.\n因为 $B D$ 是 $\\angle A B C$ 和 $\\angle A D Z$ 的平分线, 则 $\\frac{B X}{B Y}=\\frac{X K}{K Y}, \\frac{D Z}{D A}=\\frac{Z L}{L A}$. 从而, $\\frac{X K}{K Y}=\\frac{Z L}{L A}$ 或 $\\frac{X K}{Z L}= \\frac{K Y}{L A} \\cdots$ (1). 设线段 $A Y$ 与对角线 $B D$ 交于 $M_1$, 易知 $\\triangle K Y M_1 \\backsim \\triangle L A M_1$. 则 $\\frac{K Y}{L A}= \\frac{K M_1}{L M_1} \\cdots$ (2). 如图(), 设线段 $X Z$ 与对角线 $B D$ 交于 $M_2$, 易知 $\\triangle K X M_2 \\backsim \\triangle L Z M_2$, 则 $\\frac{X K}{Z L}=\\frac{K M_2}{L M_2} \\cdots$ (3). 由式(1)、(2)、(3)得 $\\frac{K M_1}{L M_1}=\\frac{K M_2}{L M_2}$, 即 $M_1=M_2$. 因此, $X Z 、 A Y 、 B D$ 三线共点.", + "remark": "", + "figures": [ + "./images/volume7/figures/fig-c1a5-1.png", + "./images/volume7/figures/fig-c1a5-2.png" + ] +} \ No newline at end of file diff --git a/processed_dataset/proof/1773.json b/processed_dataset/proof/1773.json new file mode 100644 index 0000000000000000000000000000000000000000..61c41af11bf6ec22532a009280f2d109321b322e --- /dev/null +++ b/processed_dataset/proof/1773.json @@ -0,0 +1,10 @@ +{ + "source_file": "./raw_volume-zh/volume7/exercise1.tex", + "problem_type": "proof", + "problem": "问题6. 已知等腰 $\\triangle A B C$ 和等腰 $\\triangle D B C$ 有公共的底边 $B C$, 且 $\\angle A B D=90^{\\circ} . M$ 是 $B C$ 的中点, $E$ 是线段 $A B$ 内部一点, $P$ 是线段 $M C$ 内部一点, $F$ 是 $A C$ 延长线上一点, 且满足 $\\angle B D E=\\angle A D P=\\angle C D F$. 证明: $P$ 是线段 $E F$ 的中点, 且 $D P \\perp E F$.", + "solution": "证明: 如图(), 连结 $E F$ 交 $B C$ 于 $P^{\\prime}$. 首先, $\\angle E B D= \\angle D C F=90^{\\circ}, B D=C D, \\angle B D E=\\angle C D F$, 故 $\\triangle B D E \\cong \\triangle C D F$. 进而, $D E=D F, \\triangle D E F$ 也是等腰三角形.\n则 $\\angle E D F=\\angle E D C+\\angle C D F=\\angle B D E+ \\angle E D C=\\angle B D C$. 因此, $\\triangle E D F \\backsim \\triangle B D C, \\angle D E F= \\angle D B C$. 于是, $E 、 B 、 D 、 P^{\\prime}$ 四点共圆.\n所以, $\\angle E P^{\\prime} D= \\angle E B D=90^{\\circ}$. 从而, $E P^{\\prime}=P^{\\prime} F, D P^{\\prime} \\perp E F$. 下面证明\n$P^{\\prime}$ 与 $P$ 重合.\n由 $\\angle B P^{\\prime} D=\\angle B E D, \\angle P^{\\prime} M D=\\angle E B D=90^{\\circ}$, 得 $\\triangle M P^{\\prime} D \\backsim \\triangle B E D, \\angle A D P^{\\prime}=\\angle B D E$. 所以, $P^{\\prime}$ 与 $P$ 重合.\n因此,所证结论成立.", + "remark": "", + "figures": [ + "./images/volume7/figures/fig-c1a6.png" + ] +} \ No newline at end of file diff --git a/processed_dataset/proof/1774.json b/processed_dataset/proof/1774.json new file mode 100644 index 0000000000000000000000000000000000000000..87fa7a7001e4dd7d81cf0e6bd646101c3a6817ba --- /dev/null +++ b/processed_dataset/proof/1774.json @@ -0,0 +1,10 @@ +{ + "source_file": "./raw_volume-zh/volume7/exercise1.tex", + "problem_type": "proof", + "problem": "问题7 $\\odot O_1 、 \\odot O_2$ 相交于 $A 、 B$ 两点, 过 $B$ 作直线分别交 $\\odot O_1 、 \\odot O_2$ 于点 $C$ 、 $E$, 过 $B$ 再作直线分别交 $\\odot O_1 、 \\odot O_2$ 于点 $D 、 F$. 点 $B$ 位于点 $C 、 E$ 和 $D$ 、 $F$ 之间, $M 、 N$ 分别为 $C E 、 D F$ 的中点.\n证明: $\\triangle A C D \\backsim \\triangle A E F \\backsim \\triangle A M N$.", + "solution": "证明: 如图(), 连结 $A B$. 显然, 四边形 $A C D B$ 和四边形 $A B E F$ 都是圆内接四边形, 所以, $\\angle C A D=\\angle C B D=\\angle E B F=\\angle E A F$, $\\angle A D C=\\angle A B C=180^{\\circ}-\\angle E B A=\\angle A F E$. 因此, $\\triangle A C D \\backsim \\triangle A E F$. 同理, $\\triangle A C E$ c $\\triangle A D F$, 所以, $A C: A M=A D: A N \\cdots$ (1). 且 $\\angle C A M=\\angle D A N$. 从而, $\\angle C A D=\\angle M A N \\cdots$ (2).\n由 (1)、(2) 知 $\\triangle A C D \\backsim \\triangle A M N$.", + "remark": "", + "figures": [ + "./images/volume7/figures/fig-c1a7.png" + ] +} \ No newline at end of file diff --git a/processed_dataset/proof/1775.json b/processed_dataset/proof/1775.json new file mode 100644 index 0000000000000000000000000000000000000000..9aa98164b2bf48ffed89f4002a5fb906159a0302 --- /dev/null +++ b/processed_dataset/proof/1775.json @@ -0,0 +1,10 @@ +{ + "source_file": "./raw_volume-zh/volume7/exercise1.tex", + "problem_type": "proof", + "problem": "问题8. 已知 $C$ 是线段 $A B$ 的中点, 过点 $A 、 C$ 的圆 $\\odot O_1$ 与过点 $B 、 C$ 的 $\\odot O_2$ 相交于 $C 、 D$ 两点, $P$ 是 $\\odot O_1$ 上 $\\overparen{A D}$ (不包含点 $C$ ) 的中点, $Q$ 是 $\\odot O_2$ 上 $\\overparen{B D}$ (不包含点 $C$ ) 的中点.\n求证: $P Q \\perp C D$.", + "solution": "证明: 如图(), 设 $A D 、 P C$ 相交于点 $E, B D$ 、 $Q C$ 相交于点 $F$. 由 $\\overparen{P A}=\\overparen{P D}$, 知 $\\angle P D E= \\angle P C D$. 又 $\\angle D P E=\\angle C P D$, 则 $\\triangle P D E \\backsim \\triangle P C D$. 于是, $\\frac{P D}{P C}=\\frac{P E}{P D}$, 即 $P D^2=P C \\cdot P E$. 同理, $Q D^2=Q C \\cdot Q F$, 由 $\\overparen{P A}=\\overparen{P D}$, 知 $\\angle A C P= \\angle D C E$. 又 $\\angle C P A=\\angle C D E$, 则 $\\triangle C P A \\backsim\\triangle C D E$. 于是, $\\frac{C A}{C E}=\\frac{C P}{C D}$, 即 $P C \\cdot C E=C A \\cdot C D$. 同理, $Q C \\cdot C F=C B \\cdot C D$. 而 $P D^2-Q D^2=P C \\cdot P E-Q C \\cdot Q F=P C(P C-C E)-Q C(Q C-C F)= P C^2-Q C^2+Q C \\cdot C F-P C \\cdot C E=P C^2-Q C^2+C B \\cdot C D-C A \\cdot C D= P C^2-Q C^2$. 所以, $P Q \\perp C D$.", + "remark": "", + "figures": [ + "./images/volume7/figures/fig-c1a8.png" + ] +} \ No newline at end of file diff --git a/processed_dataset/proof/1776.json b/processed_dataset/proof/1776.json new file mode 100644 index 0000000000000000000000000000000000000000..e26bf83276ef65e42cce6f64cda44cb4be6acdf6 --- /dev/null +++ b/processed_dataset/proof/1776.json @@ -0,0 +1,10 @@ +{ + "source_file": "./raw_volume-zh/volume7/exercise1.tex", + "problem_type": "proof", + "problem": "问题9. 已知梯形 $P R U S(P R / / S U)$, 满足 $\\angle P S R=2 \\angle R S U, \\angle S P U= 2 \\angle U P R$, 又点 $Q 、 T$ 分别在 $P R$ 和 $S U$ 上, 且 $S Q$ 和 $P T$ 分别是 $\\angle P S R$ 和 $\\angle S P U$ 的角平分线.\n$P T$ 与 $S Q$ 交于点 $E$, 过 $E$ 作 $S R$ 的平行线交 $P U$ 于点 $F$, 过 $E$ 作 $P U$ 的平行线交 $S R$ 于点 $G$, 又 $F G$ 分别交 $P R 、 S U$ 于点 $K$ 、\n$L$. 证明: $K F=F G=G L$.", + "solution": "证明: 如图(), 设 $P U$ 与 $R S$ 交于点 $D$. 由 $\\angle P S R=2 \\angle R S U, \\angle S P U=2 \\angle U P R$, 有 $\\angle P S D+\\angle S P D=2(\\angle R S U+\\angle U P R)= 2 \\angle P D S=180^{\\circ}-\\angle P D S$. 故 $\\angle P D S=60^{\\circ}$. 易知 $D E$ 平分 $\\angle P D S$, 故 $\\square D F E G$ 为菱形.\n又\n$\\angle P D S=60^{\\circ}$, 故 $D F=F E=E G=G D=F G$. 又 $\\angle K P F=\\frac{1}{2} \\angle S P U= \\angle E P F, \\angle K F P=\\angle D F G=60^{\\circ}=\\angle E F P$, 故 $\\triangle K F P \\cong \\triangle E F P$. 从而, $K F=E F=F G$. 同理, $G L=G E=F G$. 所以, $K F=F G=G L$.", + "remark": "", + "figures": [ + "./images/volume7/figures/fig-c1a9.png" + ] +} \ No newline at end of file diff --git a/processed_dataset/proof/1777.json b/processed_dataset/proof/1777.json new file mode 100644 index 0000000000000000000000000000000000000000..29942205ba7a6055c02fb6f4f96cadbde382d55e --- /dev/null +++ b/processed_dataset/proof/1777.json @@ -0,0 +1,10 @@ +{ + "source_file": "./raw_volume-zh/volume7/exercise1.tex", + "problem_type": "proof", + "problem": "问题12. 在四边形 $P Q R S$ 中, $A 、 B 、 C 、 D$ 分别为边 $P Q 、 Q R 、 R S 、 S P$ 的中点, $M$ 为 $C D$ 的中点.\n假设 $A M$ 上有一点 $H$, 满足 $H C=B C$. 证明: $\\angle B H M= 90^{\\circ}$.", + "solution": "证明: 易知四边形 $A B C D$ 为平行四边形.\n如图(), 延长 $A M 、 B C$ 交于点 $N$. 由 $A D / / C N$, 则 $\\angle M A D=\\angle M N C$. 又 $\\angle A M D=\\angle N M C$, 且 $M D=M C$. 于是, $\\triangle A M D \\cong \\triangle N M C$. 因此, $C N=D A=C B=H C$. 从而, 点 $H$ 位于以点 $C$ 为圆心、 $B N$ 为直径的圆上.\n所以, $\\angle B H M=90^{\\circ}$.", + "remark": "", + "figures": [ + "./images/volume7/figures/fig-c1a12.png" + ] +} \ No newline at end of file diff --git a/processed_dataset/proof/1778.json b/processed_dataset/proof/1778.json new file mode 100644 index 0000000000000000000000000000000000000000..8d7f03eb36ee017606f3a0f62d966b9f935881ee --- /dev/null +++ b/processed_dataset/proof/1778.json @@ -0,0 +1,10 @@ +{ + "source_file": "./raw_volume-zh/volume7/exercise1.tex", + "problem_type": "proof", + "problem": "问题13. 在凸四边形 $A B C D$ 中, $\\angle A D C 、 \\angle B C D$ 均大于 $90^{\\circ}$, 设点 $E$ 是直线 $A C$ 与过点 $B$ 而平行于 $A D$ 的直线的交点, 点 $F$ 是直线 $B D$ 与过点 $A$ 而平行于 $B C$ 的直线的交点.\n证明: $E F / / C D$.", + "solution": "证明: 如图(), 设 $P$ 是对角线 $A C$ 和 $B D$ 的交点.\n为证 $C D / / E F$, 只需证 $\\frac{P E}{P F}=\\frac{P C}{P D}$. 因为 $B C / / A F, \\triangle P B C \\backsim \\triangle P F A$, 所以, $\\frac{P F}{P B}=\\frac{P A}{P C}$, 即 $P F= \\frac{P A \\cdot P B}{P C}$. 因为 $A D / / B E, \\triangle P A D \\backsim \\triangle P E B$, 所以, $\\frac{P E}{P A}=\\frac{P B}{P D}$, 即 $P E= \\frac{P A \\cdot P B}{P D}$. 于是, $\\frac{P E}{P F}=\\frac{\\frac{P A \\cdot P B}{P D}}{\\frac{P A \\cdot P B}{P C}}=\\frac{P C}{P D}$. 因此, $C D / / E F$.", + "remark": "", + "figures": [ + "./images/volume7/figures/fig-c1a13.png" + ] +} \ No newline at end of file diff --git a/processed_dataset/proof/1779.json b/processed_dataset/proof/1779.json new file mode 100644 index 0000000000000000000000000000000000000000..2e8b6860eeec1aa8f85a39a77fab06bb1ce86277 --- /dev/null +++ b/processed_dataset/proof/1779.json @@ -0,0 +1,10 @@ +{ + "source_file": "./raw_volume-zh/volume7/exercise1.tex", + "problem_type": "proof", + "problem": "问题16. 设 $D$ 是 $\\triangle A B C$ 边 $B C$ 上一点, 且满足 $A B+B D=A C+C D$, 线段 $A D$ 与 $\\triangle A B C$ 的内切圆交于点 $X 、 Y$, 且 $X$ 距点 $A$ 更近一些, $\\triangle A B C$ 的内切圆与边 $B C$ 切于点 $E$. 证明: (1) $E Y \\perp A D$; (2) $X D=2 I A^{\\prime}$, 其中, $I$ 为 $\\triangle A B C$ 的内心, $A^{\\prime}$ 为边 $B C$ 的中点.", + "solution": "证明: (1) 如图(), 由条件可知 $D$ 为 $\\angle A$ 内的旁切圆与边 $B C$ 的切点, 且 $A$ 为 $\\angle A$ 内的旁切圆和内切圆的位似中心, $D$ 和 $X$ 是对应点.\n因此, 过点 $X$ 的切线与 $B C$ 平行.\n从而, $X E$ 为 $\\odot I$ 的直径, 则 $\\angle X Y E=90^{\\circ}$, 即 $E Y \\perp A D$.\n(2) 因为 $B E=D C=\\frac{1}{2}(A B+B C-A C)$, 所以, $A^{\\prime}$ 为 $E D$ 的中点.\n又 $I$ 为 $X E$ 的中点, 故 $X D=2 I A^{\\prime}$.", + "remark": "", + "figures": [ + "./images/volume7/figures/fig-c1a16.png" + ] +} \ No newline at end of file diff --git a/processed_dataset/proof/1780.json b/processed_dataset/proof/1780.json new file mode 100644 index 0000000000000000000000000000000000000000..0258d593cafa86d474b928268c17c2badf39372b --- /dev/null +++ b/processed_dataset/proof/1780.json @@ -0,0 +1,10 @@ +{ + "source_file": "./raw_volume-zh/volume7/exercise1.tex", + "problem_type": "proof", + "problem": "问题17. 设 $M 、 N$ 是 $\\triangle A B C$ 内部的两个点, 且满足 $\\angle M A B=\\angle N A C, \\angle M B A= \\angle N B C$. 证明:\n$$\n\\frac{A M \\cdot A N}{A B \\cdot A C}+\\frac{B M \\cdot B N}{B A \\cdot B C}+\\frac{C M \\cdot C N}{C A \\cdot C B}=1 .\n$$", + "solution": "证明: 如图(), 设 $K$ 是射线 $B N$ 上的点, 且满足 $\\angle B C K=\\angle B M A$. 因为 $\\angle B M A>\\angle A C B$, 则 $K$ 在 $\\triangle A B C$ 的外部.\n又因 $\\angle M B A=\\angle C B K$, 所以 $\\triangle A B M \\backsim \\triangle K B C$. 故 $\\frac{A B}{B K}=\\frac{B M}{B C}=\\frac{A M}{C K}$. 由 $\\angle A B K=\\angle M B C, \\frac{A B}{K B}=\\frac{B M}{B C}$, 可得 $\\triangle A B K \\backsim \\triangle M B C$, 于是, $\\frac{A B}{B M}=\\frac{B K}{B C}=\\frac{A K}{C M}$. 因为 $\\angle C K N=\\angle M A B=\\angle N A C$, 所以 $A 、 N 、 C 、 K$ 四点共圆.\n由托勒密 (Ptolemy) 定理, 有 $A C \\cdot N K=A N \\cdot C K+C N \\cdot A K$, 或 $A C(B K-B N)=A N \\cdot C K+C N \\cdot A K$. 将 $C K=\\frac{A M \\cdot B C}{B M}$, $A K=\\frac{A B \\cdot C M}{B M}, B K=\\frac{A B \\cdot B C}{B M}$ 代入, 得 $A C\\left(\\frac{A B \\cdot B C}{B M}-B N\\right)= \\frac{A N \\cdot A M \\cdot B C}{B M}+\\frac{C N \\cdot A B \\cdot C M}{B M}$, 即 $\\frac{A M \\cdot A N}{A B \\cdot A C}+\\frac{B M \\cdot B N}{B A \\cdot B C}+\\frac{C M \\cdot C N}{C A \\cdot C B}=1$.", + "remark": "", + "figures": [ + "./images/volume7/figures/fig-c1a17.png" + ] +} \ No newline at end of file diff --git a/processed_dataset/proof/1781.json b/processed_dataset/proof/1781.json new file mode 100644 index 0000000000000000000000000000000000000000..09612d6ecf3fedfb7f81483fc3ced338541f8c8f --- /dev/null +++ b/processed_dataset/proof/1781.json @@ -0,0 +1,10 @@ +{ + "source_file": "./raw_volume-zh/volume7/exercise1.tex", + "problem_type": "proof", + "problem": "问题18. 设 $A B C D E F$ 是凸六边形, $\\angle B+\\angle D+\\angle F=360^{\\circ}$, 且 $\\frac{A B}{B C} \\cdot \\frac{C D}{D E} \\cdot \\frac{E F}{F A}=1$.\n证明: $\\frac{B C}{C A} \\cdot \\frac{A E}{E F} \\cdot \\frac{F D}{D B}=1$.", + "solution": "证明: 如图(), 设点 $P$ 满足 $\\angle F E A=\\angle D E P$,\n$\\angle E F A=\\angle E D P$, 则 $\\triangle F E A \\backsim \\triangle D E P$. 于是, $\\frac{F A}{E F}=\\frac{D P}{D E} \\cdots$ (1), $\\frac{E F}{E D}=\\frac{E A}{E P} \\cdots$ (2). 由已知条件, 有 $\\angle A B C=\\angle P D C$. 又由 (1) 及已知条件得 $\\frac{A B}{B C}= \\frac{D E \\cdot F A}{C D \\cdot E F}=\\frac{D P}{C D} \\cdots$ (3). 所以, $\\triangle A B C \\backsim \\triangle P D C$, 故 $\\angle B C A=\\angle D C P$, 且 $\\frac{C B}{C D}=\\frac{C A}{C P}$. 因为 $\\angle F E D=\\angle A E P$, 由 (2) 知 $\\triangle F E D \\backsim \\triangle A E P$, 类似地, 由 $\\angle B C D=\\angle A C P$ 及 (3) 得 $\\triangle B C D \\backsim \\triangle A C P$. 于是 $\\frac{F D}{E F}=\\frac{P A}{A E}, \\frac{B C}{D B}=\\frac{C A}{P A}$. 两式相乘, 即得所求.", + "remark": "", + "figures": [ + "./images/volume7/figures/fig-c1a18.png" + ] +} \ No newline at end of file diff --git a/processed_dataset/proof/1782.json b/processed_dataset/proof/1782.json new file mode 100644 index 0000000000000000000000000000000000000000..cd69c757a3690b856c2cd24d9462d7003209b180 --- /dev/null +++ b/processed_dataset/proof/1782.json @@ -0,0 +1,10 @@ +{ + "source_file": "./raw_volume-zh/volume7/exercise10.tex", + "problem_type": "proof", + "problem": "问题1. 设 $R$ 与 $r$ 分别是锐角 $\\triangle A B C$ 的外接圆与内切圆的半径, 设 $\\angle A$ 是 $\\triangle A B C$\n的三个内角中最大的一个, $M$ 是边 $B C$ 的中点, 过点 $B 、 C$ 作 $\\triangle A B C$ 的外接圆的切线, 交于点 $X$. 证明: $\\frac{r}{R} \\geqslant \\frac{A M}{A X}$.", + "solution": "证明: 如图(), 设 $O$ 与 $I$ 分别是锐角 $\\triangle A B C$ 的外心与内心, 则 $O 、 M 、 X$ 三点共线, 且 $\\triangle O X C \\backsim \\triangle O C M$. 因此, $\\frac{O C}{O X}= \\frac{O M}{O C}$. 由于 $O C=R=O A$, 可得 $\\frac{O A}{O M}=\\frac{O X}{O A}$. 于是, $\\triangle O A M \\backsim \\triangle O X A$. 从而, $\\frac{A M}{A X}=\\frac{O M}{R}$.\n下面只需再证明 $O M \\leqslant r$. 比较 $\\angle O B M$ 与 $\\angle I B M$, 由于 $\\triangle A B C$ 是锐角三角形, $O$ 与 $I$ 位于 $\\triangle A B C$ 内部, 于是, 有 $\\angle O B M=\\frac{\\pi}{2}-\\angle A=\\frac{1}{2}(\\angle A+\\angle B+\\angle C)-\\angle A=\\frac{1}{2}(\\angle B+\\angle C-\\angle A) \\leqslant \\frac{\\angle B}{2}=\\angle I B M$. 类似可得 $\\angle O C M \\leqslant \\angle I C M$. 于是, 点 $O$ 位于 $\\triangle I B C$ 内部或边界上.\n因此, $O M \\leqslant r$.", + "remark": "", + "figures": [ + "./images/volume7/figures/fig-c10a1.png" + ] +} \ No newline at end of file diff --git a/processed_dataset/proof/1783.json b/processed_dataset/proof/1783.json new file mode 100644 index 0000000000000000000000000000000000000000..bd9af7e6102b3ac72aa9946aae67d13a3c7a5bc7 --- /dev/null +++ b/processed_dataset/proof/1783.json @@ -0,0 +1,8 @@ +{ + "source_file": "./raw_volume-zh/volume7/exercise10.tex", + "problem_type": "proof", + "problem": "问题2. 已知 $a 、 b 、 c$ 和 $R$ 分别为三角形的三边长和外接圆半径.\n证明:\n$$\n\\frac{1}{a b}+\\frac{1}{b c}+\\frac{1}{c a} \\geqslant \\frac{1}{R^2} \\text {. }\n$$", + "solution": "证明: 要证 $\\frac{1}{a b}+\\frac{1}{b c}+\\frac{1}{c a} \\geqslant \\frac{1}{R^2}$, 只要证 $\\frac{1}{4 R^2 \\sin A \\cdot \\sin B}+ \\frac{1}{4 R^2 \\sin B \\cdot \\sin C}+\\frac{1}{4 R^2 \\sin C \\cdot \\sin A} \\geqslant \\frac{1}{R^2} \\Leftrightarrow \\frac{1}{4 \\sin A \\cdot \\sin B}+\\frac{1}{4 \\sin B \\cdot \\sin C}+ \\frac{1}{4 \\sin C \\cdot \\sin A} \\geqslant 1 \\Leftrightarrow \\sin A+\\sin B+\\sin C \\geqslant 4 \\sin A \\cdot \\sin B \\cdot \\sin C \\Leftrightarrow 2 \\sin \\frac{A+B}{2}$. $\\cos \\frac{A-B}{2}+2 \\sin \\frac{A+B}{2} \\cdot \\cos \\frac{A+B}{2} \\geqslant 4 \\sin A \\cdot \\sin B \\cdot \\sin C \\Leftrightarrow \\cos \\frac{C}{2} \\cdot \\left(\\cos \\frac{A-B}{2}+\\cos \\frac{A+B}{2}\\right) \\geqslant 2 \\sin A \\cdot \\sin B \\cdot \\sin C \\Leftrightarrow 2 \\cos \\frac{A}{2} \\cdot \\cos \\frac{B}{2} \\cdot \\cos \\frac{C}{2} \\geqslant 2 \\sin A \\cdot \\sin B \\cdot \\sin C \\Leftrightarrow \\sin \\frac{A}{2} \\cdot \\sin \\frac{B}{2} \\cdot \\sin \\frac{C}{2} \\leqslant \\frac{1}{8} \\cdots(1)$.\n下面证明(1)成立.\n$\\sin \\frac{A}{2} \\cdot \\sin \\frac{B}{2} \\cdot \\sin \\frac{C}{2} \\leqslant\\left(\\frac{\\sin \\frac{A}{2}+\\sin \\frac{B}{2}+\\sin \\frac{C}{2}}{3}\\right)^3 \\leqslant \\sin ^3 \\frac{\\frac{A}{2}+\\frac{B}{2}+\\frac{C}{2}}{3}$ (由琴生不等式 $)=\\frac{1}{8}$. 则式(1)成立.\n因此,所证不等式成立.", + "remark": "", + "figures": [] +} \ No newline at end of file diff --git a/processed_dataset/proof/1784.json b/processed_dataset/proof/1784.json new file mode 100644 index 0000000000000000000000000000000000000000..d06565eac1b20b92140735ffac25910dc7a4ec56 --- /dev/null +++ b/processed_dataset/proof/1784.json @@ -0,0 +1,8 @@ +{ + "source_file": "./raw_volume-zh/volume7/exercise10.tex", + "problem_type": "proof", + "problem": "问题3. 已知 $\\triangle A B C$ 的三边长分别为 $a 、 b 、 c$, 点 $P$ 在 $\\triangle A B C$ 的内部, $P$ 到三条边的距离分别为 $p 、 q 、 r$. 证明: $R \\leqslant \\frac{a^2+b^2+c^2}{18 \\sqrt[3]{p q r}}$, 其中 $R$ 为 $\\triangle A B C$ 的外接圆半径, 并确定等号成立的条件.", + "solution": "证明: 设 $\\triangle A B C$ 的面积为 $S$, 则 $S=S_{\\triangle P B C}+S_{\\triangle P C A}+S_{\\triangle P A B}=\\frac{1}{2}(p a+q b+r c$ ). 由均值不等式有 $\\sqrt[3]{p q r}=\\frac{\\sqrt[3]{p a \\cdot q b \\cdot r c}}{\\sqrt[3]{a b c}} \\leqslant \\frac{p a+q b+r c}{3 \\sqrt[3]{a b c}}= \\frac{2 S}{3 \\sqrt[3]{a b c}} \\cdots$ (1). 故只需证 $R \\leqslant \\frac{\\left(a^2+b^2+c^2\\right) \\sqrt[3]{a b c}}{12 S}$. 又因为 $a^2+b^2+c^2 \\geqslant 3 \\sqrt[3]{a^2 b^2 c^2} \\cdots$ (2). 所以, 只需证 $R \\leqslant \\frac{a b c}{4 S} \\cdots$ (3). 而 $4 S R=2 R a b \\sin C=a b c$, 所以, 式 (3) 成立.\n因此,原不等式成立.\n式 (1) 的等号成立的条件是 $p a=q b=r c$, 式 (2) 的等号成立的条件是 $a= b=c$, 所以,原不等式等号成立的条件是 $a=b=c$ 且 $p=q=r$, 即 $\\triangle A B C$ 是正三角形且 $P$ 是 $\\triangle A B C$ 的中心.", + "remark": "", + "figures": [] +} \ No newline at end of file diff --git a/processed_dataset/proof/1785.json b/processed_dataset/proof/1785.json new file mode 100644 index 0000000000000000000000000000000000000000..bdeb82550484e275344e2a4c61bbb1ae9966bbda --- /dev/null +++ b/processed_dataset/proof/1785.json @@ -0,0 +1,10 @@ +{ + "source_file": "./raw_volume-zh/volume7/exercise10.tex", + "problem_type": "proof", + "problem": "问题4. 在 $\\triangle A B C$ 中, $\\angle A 、 \\angle C$ 的平分线分别与对边交于点 $D 、 E$. 若 $\\angle B> 60^{\\circ}$, 证明: $A E+C D), 作 $\\angle E^{\\prime} I A=\\angle E I A$ 交 $A C$ 于点 $E^{\\prime}$, 作 $\\angle D^{\\prime} I C=\\angle D I C$ 交 $A C$ 于点 $D^{\\prime}$. 因为 $D A$ 为 $\\angle B A C$ 的平分线, 故 $\\angle I A E=\\angle I A E^{\\prime}$. 又 $\\angle E I A=\\angle E^{\\prime} I A$, 所以, $\\triangle A I E \\cong \\triangle A I E^{\\prime}$. 于是, $A E=A E^{\\prime}$. 同理, $C D=C D^{\\prime}$. 故 $\\angle A I E^{\\prime}+\\angle C I D^{\\prime}=2 \\angle A I E=\\angle B A C+\\angle A C B, \\angle A I C=\\angle B+ \\frac{1}{2}(\\angle B A C+\\angle A C B)$. 因此, $\\angle A I C>\\angle A I E^{\\prime}+\\angle C I D^{\\prime}$ (这里用到 $\\angle B> 60^{\\circ}$ ). 从而, $A C=A E^{\\prime}+E^{\\prime} D^{\\prime}+D^{\\prime} C>A E^{\\prime}+D^{\\prime} C=A E+D C$.", + "remark": "", + "figures": [ + "./images/volume7/figures/fig-c10a4.png" + ] +} \ No newline at end of file diff --git a/processed_dataset/proof/1786.json b/processed_dataset/proof/1786.json new file mode 100644 index 0000000000000000000000000000000000000000..a183b48f5f653a5abba1f9edc5870d342335ac0f --- /dev/null +++ b/processed_dataset/proof/1786.json @@ -0,0 +1,8 @@ +{ + "source_file": "./raw_volume-zh/volume7/exercise10.tex", + "problem_type": "proof", + "problem": "问题5. (嵌入不等式) $A, B, C$ 为 $\\triangle A B C$ 的内角.\n求证: 对任意实数 $x 、 y 、 z$, $x^2+y^2+z^2-2 x y \\cos C-2 y z \\cos A-2 z x \\cos B \\geqslant 0$.", + "solution": "证明: $x^2+y^2+z^2-2 x y \\cos C-2 y z \\cos A-2 z x \\cos B=(x^2-2 x y \\cos C-2 z x \\cos B)+y^2+z^2-2 y z \\cos A=(x-y \\cos C-z \\cos B)^2+y^2 \\sin ^2 C+ z^2 \\sin ^2 B-2 y z \\cdot \\cos A-2 y z \\cos B \\cos C \\cdots$ (1).\n由于 $\\cos A=\\cos (\\pi-B-C)=-\\cos (B+C)=\\sin B \\sin C-\\cos B \\cos C$. 所以 (1) 式 $=(x-y \\cos C-z \\cos B)^2+(y \\sin C-z \\sin B)^2 \\geqslant 0$.", + "remark": "", + "figures": [] +} \ No newline at end of file diff --git a/processed_dataset/proof/1787.json b/processed_dataset/proof/1787.json new file mode 100644 index 0000000000000000000000000000000000000000..4cfe392fe095033a65f03b5525123d6b4d42d7cb --- /dev/null +++ b/processed_dataset/proof/1787.json @@ -0,0 +1,11 @@ +{ + "source_file": "./raw_volume-zh/volume7/exercise10.tex", + "problem_type": "proof", + "problem": "问题6. 设在凸四边形 $A B C D$ 中, $A B=A D+B C$. 在此四边形内, 距离 $C D$ 为 $h$ 的地方有一点 $P$, 使得 $A P=h+A D, B P=h+B C$. 求证: $\\frac{1}{\\sqrt{h}} \\geqslant \\frac{1}{\\sqrt{A D}}+\\frac{1}{\\sqrt{B C}}$.", + "solution": "证明: 设 $M$ 是线段 $A B$ 内的点,且满足 $A M= A D=r, B M=B C=R$. 因此条件也就等价于 $: \\odot P$ 半径为 $h$, 并且与边 $C D$ 和圆 $\\odot A, \\odot B$ 都相切, 其中 $\\odot A, \\odot B$ 分别是以 $A, B$ 为圆心, $r, R$ 为半径的圆, 且 $\\odot A, \\odot B$ 相切于 $M$ (如图()).\n我们需要证明 $\\frac{1}{\\sqrt{h}} \\geqslant \\frac{1}{\\sqrt{R}}+\\frac{1}{\\sqrt{r}}$. 当 $h$ 取最大值时, $\\frac{1}{\\sqrt{h}}$ 取最小值, 并且当 $D C$ 是 $\\odot A, \\odot B$ 的公切线时, 它取最小值.\n此时, 我们令 $h_0$ 是 $\\odot P$ 的半径.\n那么我们就需证明 $\\frac{1}{\\sqrt{h_0}}=\\frac{1}{\\sqrt{R}}+\\frac{1}{\\sqrt{r}}$.\n新的简化图(), 刻画了这时的情形.\n设 $E$ 是 $A$ 在 $B C$ 上的投影, $Q$ 是 $P$ 在 $C D$ 上的投影.\n我们有 $A E= \\sqrt{A B^2-B E^2}=\\sqrt{(R+r)^2-(R-r)^2}=2 \\sqrt{R r}$.\n另一方面, $A E=C D=D Q+Q C=\\sqrt{\\left(r+h_0\\right)^2-\\left(r-h_0\\right)^2}+ \\sqrt{\\left(R+h_0\\right)^2-\\left(R-h_0\\right)^2}=2 \\sqrt{r h_0}+2 \\sqrt{R h_0}$. 等式 $\\sqrt{R r}=\\sqrt{r h_0}+\\sqrt{R h_0}$ 等价于所要证明的等式.", + "remark": "", + "figures": [ + "./images/volume7/figures/fig-c10a6-1.png", + "./images/volume7/figures/fig-c10a6-2.png" + ] +} \ No newline at end of file diff --git a/processed_dataset/proof/1788.json b/processed_dataset/proof/1788.json new file mode 100644 index 0000000000000000000000000000000000000000..6c1f97ff4e73ffe46f4204549bb16a324815bf70 --- /dev/null +++ b/processed_dataset/proof/1788.json @@ -0,0 +1,10 @@ +{ + "source_file": "./raw_volume-zh/volume7/exercise10.tex", + "problem_type": "proof", + "problem": "问题7. 设 $\\triangle A B C$ 为锐角三角形,外接圆圆心为 $O$, 半径为 $R, A O$ 交 $\\triangle B O C$ 所在圆于另一点 $A^{\\prime}, B O$ 交 $\\triangle C O A$ 所在圆于另一点 $B^{\\prime}, C O$ 交 $\\triangle A O B$ 所在圆于另一点 $C^{\\prime}$. 证明: $O A^{\\prime} \\cdot O B^{\\prime} \\cdot O C \\geqslant 8 R^3$, 并指出在什么情况下等号成立.", + "solution": "证明: 如图(), 设 $A O$ 与 $B C, B O$ 与 $C A, C O$ 与 $A B$ 的交点依次为 $D 、 E 、 F, \\triangle A O B 、 \\triangle B O C 、 \\triangle C O A$ 的面积依次为 $S_1 、 S_2 、 S_3$. 由 $B 、 O 、 C 、 A^{\\prime}$ 四点共圆知 $\\angle O B C=\\angle O C B=\\angle B A^{\\prime} O$, 从而有 $\\triangle O B D \\backsim \\triangle O A^{\\prime} B$, 得 $O A^{\\prime}=\\frac{O B^2}{O D}=\\frac{R^2}{O D}$. 同理, $O B^{\\prime}=\\frac{R^2}{O E}, O C^{\\prime}=\\frac{R^2}{O F}$. 所以, $\\frac{O A^{\\prime} \\cdot O B^{\\prime} \\cdot O C^{\\prime}}{R^3}=\\frac{R^3}{O D \\cdot O E \\cdot O F}=\\frac{O A}{O D} \\cdot \\frac{O B}{O E} \\cdot \\frac{O C}{O F}=\\frac{S_1+S_3}{S_2} \\cdot \\frac{S_1+S_2}{S_3} \\cdot \\frac{S_2+S_3}{S_1}=\\left(\\frac{S_1}{S_2}+\\frac{S_3}{S_2}\\right)\\left(\\frac{S_1}{S_3}+\\frac{S_2}{S_3}\\right)\\left(\\frac{S_2}{S_1}+\\frac{S_3}{S_1}\\right)=\\left(\\frac{S_1}{S_2}+\\frac{S_2}{S_1}\\right)+ \\left(\\frac{S_2}{S_3}+\\frac{S_3}{S_2}\\right)+\\left(\\frac{S_3}{S_1}+\\frac{S_1}{S_3}\\right)+2 \\geqslant 8$, 等号当且仅当 $S_1=S_2=S_3$ 时成立, 此时 $\\triangle A B C$ 为正三角形.\n故 $O A^{\\prime} \\cdot O B^{\\prime} \\cdot O C^{\\prime} \\geqslant 8 R^3$, 等号当且仅当 $\\triangle A B C$ 为正三角形时成立.", + "remark": "", + "figures": [ + "./images/volume7/figures/fig-c10a7.png" + ] +} \ No newline at end of file diff --git a/processed_dataset/proof/1789.json b/processed_dataset/proof/1789.json new file mode 100644 index 0000000000000000000000000000000000000000..4c832404b1d50de8c30d9c68129ec2c8c121469e --- /dev/null +++ b/processed_dataset/proof/1789.json @@ -0,0 +1,8 @@ +{ + "source_file": "./raw_volume-zh/volume7/exercise10.tex", + "problem_type": "proof", + "problem": "问题8. 设 $A B C D E F$ 是凸六边形, 且 $A B=B C, C D=D E, E F=F A$, 证明: $\\frac{B C}{B E}+\\frac{D E}{D A}+\\frac{F A}{F C} \\geqslant \\frac{3}{2}$, 并指出等号成立的条件.", + "solution": "证明: 记 $A C=a, C E=b, A E=c$, 对四边形 $A C E F$ 运用 Ptolemy 不等式得 $A C \\cdot E F+C E \\cdot A F \\geqslant A E \\cdot C F$. 因为 $E F=A F$, 所以 $\\frac{F A}{F C} \\geqslant \\frac{c}{a+b}$. 同理 $\\frac{D E}{D A} \\geqslant \\frac{b}{c+a}, \\frac{B C}{B E} \\geqslant \\frac{a}{b+c}$. 故 $\\frac{B C}{B E}+\\frac{D E}{D A}+\\frac{F A}{F C} \\geqslant \\frac{a}{b+c}+\\frac{b}{c+a}+\\frac{c}{a+b}$, 令 $b+ c=x, c+a=y, a+b=z$, 则 $G=\\frac{y+z-x}{2}, b=\\frac{z+x-y}{2}, c=\\frac{x+y-z}{2}$, $\\frac{a}{b+c}+\\frac{b}{c+a}+\\frac{c}{a+b}=\\frac{1}{2}\\left(\\frac{y}{x}+\\frac{z}{x}+\\frac{z}{y}+\\frac{x}{y}+\\frac{x}{z}+\\frac{y}{z}-3\\right) \\geqslant \\frac{3}{2}$. 等号成立的条件为 $A B C D E F$ 是圆内接六边形且 $a=b=c$.", + "remark": "", + "figures": [] +} \ No newline at end of file diff --git a/processed_dataset/proof/1790.json b/processed_dataset/proof/1790.json new file mode 100644 index 0000000000000000000000000000000000000000..4f52b7384b92728f6340564eb297dde4e1f6b8af --- /dev/null +++ b/processed_dataset/proof/1790.json @@ -0,0 +1,10 @@ +{ + "source_file": "./raw_volume-zh/volume7/exercise10.tex", + "problem_type": "proof", + "problem": "问题9. 设在 $\\triangle A B C$ 中, $\\angle A 、 \\angle B$ 和 $\\angle C$ 的角平分线分别交 $\\triangle A B C$ 的外接圆于 $A_1 、 B_1 、 C_1$. 求证: $A A_1+B B_1+C C_1>A B+B C+C A$.", + "solution": "证明: 如图(), 对四边形 $A C A_1 B$ 应用 Ptolemy 定理, 可得 $A A_1 \\cdot B C=A B \\cdot A_1 C+A C \\cdot A_1 B$. 令 $A_1 B=A_1 C=x$, 注意到 $2 x=A_1 B+A_1 C>B C$, 有 $2 A A_1=2$ ・ $\\frac{A B x+A C x}{B C}=(A B+A C) \\cdot \\frac{2 x}{B C}>A B+A C$, 即 $A A_1> \\frac{1}{2}(A B+A C)$. 同理可得 $B B_1>\\frac{1}{2}(B A+B C), C C_1> \\frac{1}{2}(C A+C B)$, 三式相加即得所证结果.", + "remark": "", + "figures": [ + "./images/volume7/figures/fig-c10a9.png" + ] +} \ No newline at end of file diff --git a/processed_dataset/proof/1791.json b/processed_dataset/proof/1791.json new file mode 100644 index 0000000000000000000000000000000000000000..a889fb66a3aa8844bc969fcfd6d3cfa6b9d12ed5 --- /dev/null +++ b/processed_dataset/proof/1791.json @@ -0,0 +1,8 @@ +{ + "source_file": "./raw_volume-zh/volume7/exercise10.tex", + "problem_type": "proof", + "problem": "问题10. 两个凸四边形 $A B C D$ 和 $A^{\\prime} B^{\\prime} C^{\\prime} D^{\\prime}$ 的边长分别为 $a 、 b 、 c 、 d$ 和 $a^{\\prime} 、 b^{\\prime} 、 c^{\\prime}$ 、 $d^{\\prime}$, 面积分别为 $s$ 和 $s^{\\prime}$. 证明: $a a^{\\prime}+b b^{\\prime}+c c^{\\prime}+d d^{\\prime} \\geqslant 4 \\sqrt{s s^{\\prime}}$.", + "solution": "证明: 在边长给定的四边形中, 以内接于圆时其面积为最大.\n因此, 只需证两个凸四边形为圆内接四边形的情况.\n这时 $s= \\sqrt{(s-a)(s-b)(s-c)(s-d)}, s^{\\prime}$ 与之类似, 其中 $s=\\frac{1}{2}(a+b+c+d)=a+ c=b+d, s^{\\prime}=\\frac{1}{2}\\left(a^{\\prime}+b^{\\prime}+c^{\\prime}+d^{\\prime}\\right)=a^{\\prime}+c^{\\prime}=b^{\\prime}+d^{\\prime}$. 利用算术几何平均值不等式有 $a a^{\\prime}+b b^{\\prime}+c c^{\\prime}+d d^{\\prime}=(s-a)\\left(s^{\\prime}-a^{\\prime}\\right)+(s-b)\\left(s^{\\prime}-b^{\\prime}\\right)+(s-$ c) $\\left(s^{\\prime}-c^{\\prime}\\right)+(s-d)\\left(s^{\\prime}-d^{\\prime}\\right) \\geqslant 4\\left[(s-a)\\left(s^{\\prime}-a^{\\prime}\\right)(s-b) \\cdot\\left(s^{\\prime}-b^{\\prime}\\right)(s-\\right.$ c) $\\left.\\left(s^{\\prime}-c^{\\prime}\\right)(s-d)\\left(s^{\\prime}-d^{\\prime}\\right)\\right]^{\\frac{1}{4}}=4 \\sqrt{s s^{\\prime}}$.", + "remark": "", + "figures": [] +} \ No newline at end of file diff --git a/processed_dataset/proof/1792.json b/processed_dataset/proof/1792.json new file mode 100644 index 0000000000000000000000000000000000000000..91a0b80d767b4585c0c3ea773a9aa9233e0a7bcc --- /dev/null +++ b/processed_dataset/proof/1792.json @@ -0,0 +1,8 @@ +{ + "source_file": "./raw_volume-zh/volume7/exercise10.tex", + "problem_type": "proof", + "problem": "问题11. 已知 $\\triangle A B C$, 设 $I$ 是它的内心, 角 $A 、 B 、 C$ 的内角平分线分别与其对边交于 $A^{\\prime} 、 B^{\\prime} 、 C^{\\prime}$. 求证: $\\frac{5}{4}<\\frac{A I \\cdot B I}{A A^{\\prime} \\cdot B B^{\\prime}}+\\frac{B I \\cdot C I}{B B^{\\prime} \\cdot C C^{\\prime}}+\\frac{C I \\cdot A I}{C C^{\\prime} \\cdot A A^{\\prime}} \\leqslant \\frac{4}{3}$.", + "solution": "证明: 因为 $B I$ 平分 $\\angle A B C, C I$ 平分 $\\angle A C B$, 所以, $\\frac{A I}{A A^{\\prime}}= \\frac{A B}{A B+B A^{\\prime}}=\\frac{A C}{A C+C A}=\\frac{A B+A C}{A B+A C+B C}$.\n$$\n\\text { 记 } A B=c, A C=b, C B=a, s=a+b+c \\text {, 则 } \\frac{A I}{A A^{\\prime}}=\\frac{c+b}{a+b+c}=\\frac{s-a}{s} \\text {. }\n$$\n同理 $\\frac{B I}{B B^{\\prime}}=\\frac{a+c}{a+b+c}=\\frac{s-b}{s}, \\frac{C I}{C C^{\\prime}}=\\frac{a+b}{a+b+c}=\\frac{s-c}{s}$. 所以, $\\frac{A I \\cdot B I}{A A^{\\prime} \\cdot \\bar{B} B^{\\prime}}+ \\frac{B I \\cdot C I}{B B^{\\prime} \\cdot C C}+\\frac{C I \\cdot A I}{C C^{\\prime} \\cdot A A^{\\prime}}=\\frac{(s-a)(s-b)+(s-b)(s-c)+(s-c)(s-a)}{s^2}= \\frac{3 s^2-2(a+b+c) s+a b+b c+c a}{s^2}=1+\\frac{a b+b c+c a}{s^2}$. 欲证不等式等价于 $\\frac{1}{4}<\\frac{a b+b c+c a}{s^2} \\leqslant \\frac{1}{3} \\cdots$ (1). 因为 $(a-b)^2+(b-c)^2+(c-a)^2 \\geqslant 0$, 所以, $2\\left(a^2+b^2+c^2\\right) \\geqslant 2(a b+b c+c a) \\Leftrightarrow(a+b+c)^2 \\geqslant 3(a b+b c+c a)$. 所以 $\\frac{a b+b c+c a}{s^2} \\leqslant \\frac{1}{3}$, 此为(1)右端.\n另一方面, 不妨设 $a \\geqslant b \\geqslant c$, 则 $\\sqrt{a}+\\sqrt{b}+\\sqrt{c}, \\sqrt{a}+\\sqrt{b}-\\sqrt{c}, \\sqrt{a}-\\sqrt{b}+ \\sqrt{c}>0$. 又 $(\\sqrt{a})^2=a\\frac{1}{4}$. 此为(1)左端.", + "remark": "", + "figures": [] +} \ No newline at end of file diff --git a/processed_dataset/proof/1793.json b/processed_dataset/proof/1793.json new file mode 100644 index 0000000000000000000000000000000000000000..0df1c8e50b924acaf1a5f56e18edfea5fe307188 --- /dev/null +++ b/processed_dataset/proof/1793.json @@ -0,0 +1,10 @@ +{ + "source_file": "./raw_volume-zh/volume7/exercise10.tex", + "problem_type": "proof", + "problem": "问题12. 设 $P$ 为 $\\triangle A B C$ 内部或边上任一点, 记 $P A=x, P B=y, P C=z$, 求证: $x^2+y^2+z^2 \\geqslant \\frac{1}{3}\\left(a^2+b^2+c^2\\right)$.", + "solution": "证明: 如图(), 分别过 $A 、 B 、 C$ 作 $P A 、 P B$ 、 $P C$ 的垂线, 三垂线两两相交于 $A^{\\prime} 、 B^{\\prime} 、 C^{\\prime}$, 于是 $\\angle B P C=\\pi-A^{\\prime}, \\angle A P B=\\pi-C^{\\prime}, \\angle A P C=\\pi- B^{\\prime}$. 由余弦定理可得\n$$\n\\begin{aligned}\n& a^2=y^2+z^2+2 y z \\cos A^{\\prime}, \\\\\n& b^2=x^2+z^2+2 x z \\cos B^{\\prime}, \\\\\n& c^2=x^2+y^2+2 x y \\cos C^{\\prime},\n\\end{aligned}\n$$\n相加并应用第 5 题嵌入不等式便得 $a^2+b^2+c^2=2\\left(x^2+y^2+z^2\\right)+2 x y \\cos C^{\\prime}+ 2 x z \\cos B^{\\prime}+2 y z \\cos A^{\\prime} \\leqslant 2\\left(x^2+y^2+z^2\\right)+\\left(x^2+y^2+z^2\\right)=3\\left(x^2+y^2+z^2\\right)$. 得证.", + "remark": "", + "figures": [ + "./images/volume7/figures/fig-c10a12.png" + ] +} \ No newline at end of file diff --git a/processed_dataset/proof/1794.json b/processed_dataset/proof/1794.json new file mode 100644 index 0000000000000000000000000000000000000000..b4cc91d0c4c0ebdaf50bc6eb0b577c3eb44dba9d --- /dev/null +++ b/processed_dataset/proof/1794.json @@ -0,0 +1,10 @@ +{ + "source_file": "./raw_volume-zh/volume7/exercise10.tex", + "problem_type": "proof", + "problem": "问题13. 面积为 $M$ 的凸四边形内接于一圆, 圆心在四边形内部.\n证明: 以该四边形对角线交点在四边上的射影为顶点的四边形面积不超过 $\\frac{M}{2}$.", + "solution": "证明: 如图(), $O$ 是圆内接凸四边形 $A B C D$ 对角线交点, 它到四边的垂足分别是 $P 、 Q 、 R 、 S$, 则 $\\angle 2=\\angle 1=\\angle 4=\\angle 3$, 所以 $O P$ 平分 $\\angle S P Q$.\n同理可证: $O Q, O R, O S$ 分别平分 $\\angle P Q R, \\angle Q R S, \\angle R S P$. 所以四边形 $P Q R S$ 内心为 $O$. 由圆外切四边形面积公式 (见例 10 的解答过程) 得\n$S_{\\text {四边形 } \\mathrm{PQRS}}^2=P Q \\cdot O R \\cdot R S \\cdot S P \\cdot \\sin ^2 \\frac{\\angle S P Q+\\angle S R Q}{2}= P Q \\cdot Q R \\cdot R S \\cdot S P \\cdot \\sin ^2 \\angle A O P$.\n$\\frac{\\angle S P Q+\\angle S R Q}{2}= \\angle 2+\\angle 5=\\angle 1+\\angle 6=180^{\\circ}-\\angle A O D$.) 又因为 $S_{\\text {四边形 } P Q R S}=\\frac{1}{2} A C \\cdot B D \\cdot \\sin \\angle A O D=\\frac{1}{2}(A O+ O C)(B O+O D) \\cdot \\sin \\angle A O D=\\frac{1}{2}\\left(\\frac{S P}{\\sin A}+\\frac{Q R}{\\sin A}\\right) \\cdot\\left(\\frac{P Q}{\\sin B}\\right.\\left.+\\frac{R S}{\\sin B}\\right) \\cdot \\sin \\angle A O D \\geqslant \\frac{4 \\sqrt{P Q \\cdot Q R \\cdot R S \\cdot S P} \\sin \\angle A O D}{2 \\sin A \\sin B}=\\frac{2 S_{\\text {四边形PQRS }}}{\\sin A \\sin B} \\geqslant$\n所以 $S_{\\text {四边形 } P Q R S} \\leqslant \\frac{m}{2}$, 当且仅当 $A B C D$ 是矩形时等号成立.", + "remark": "", + "figures": [ + "./images/volume7/figures/fig-c10a13.png" + ] +} \ No newline at end of file diff --git a/processed_dataset/proof/1795.json b/processed_dataset/proof/1795.json new file mode 100644 index 0000000000000000000000000000000000000000..9a4f1fcb20215022543b6636fdbd6ff96be762e3 --- /dev/null +++ b/processed_dataset/proof/1795.json @@ -0,0 +1,10 @@ +{ + "source_file": "./raw_volume-zh/volume7/exercise10.tex", + "problem_type": "proof", + "problem": "问题14. 设 $G$ 为 $\\triangle A B C$ 的重心, $A_1 、 B_1 、 C_1$ 分别为 $A G 、 B G 、 C G$ 与 $\\triangle A B C$ 的外接圆的交点.\n求证: $G A_1+G B_1+G C_1 \\geqslant G A+G B+G C$, 等号成立当且仅当 $\\triangle A B C$ 为正三角形.", + "solution": "如图(), 设 $m_a$ 为边 $a$ 上中线长, $m_b$ 为边 $b$ 上中线长, $m_c$ 为边 $c$ 中线长.\n记 $M_a=A A_1, M_b=B B_1, M_c= C C_1$. 设 $A_0, B_0, C_0$ 分别平分边 $B C 、 C A 、 A B$. 由相交弦定理得 $\\frac{a^2}{4}=A_0 B \\cdot A_0 C=A_0 A_1 \\cdot A_0 A=\\left(M_a-\\right. \\left.m_a\\right) \\cdot m_a$, 又由中线长公式, $m_a^2=A A_0^2=\\frac{1}{2}\\left(b^2+c^2\\right)- \\frac{1}{4} a^2$. 即 $4 m_a^2=2\\left(b^2+c^2+a^2\\right)-3 a^2=8 k^2-3 a^2$, 其中\n$k=\\frac{1}{2} \\cdot \\sqrt{a^2+b^2+c^2}$, 且 $m_a^2+m_b^2+m_c^2=\\frac{3}{4}\\left(a^2+\\right. \\left.b^2+c^2\\right)$.\n于是 $8 k^2-4 m_a^2=3 a^2=12 \\cdot \\frac{a^2}{4}=12 \\cdot\\left(M_a-m_a\\right) \\cdot m_a$, 即 $M_a=m_a+ \\frac{8 k^2-4 m_a^2}{12 m_a}=\\frac{8 \\cdot\\left(k^2+m_a^2\\right)}{12 m_a}=\\frac{2 k}{3} \\cdot\\left(\\frac{k}{m_a}+\\frac{m_a}{k}\\right) \\geqslant \\frac{4 k}{3}$.\n同理, $M_b \\geqslant \\frac{4 k}{3}, M_c \\geqslant \\frac{4 k}{3}$.\n于是 $M_a+M_b+M_c \\geqslant 4 k=2 \\cdot \\sqrt{a^2+b^2+c^2}=\\frac{4}{3} \\cdot \\sqrt{3 \\cdot\\left(m_a^2+m_b^2+m_c^2\\right)}$ (由柯西不等式) $\\geqslant \\frac{4}{3} \\cdot\\left(m_a+m_b+m_c\\right)$. 得证.", + "remark": "", + "figures": [ + "./images/volume7/figures/fig-c10a14.png" + ] +} \ No newline at end of file diff --git a/processed_dataset/proof/1796.json b/processed_dataset/proof/1796.json new file mode 100644 index 0000000000000000000000000000000000000000..8ad8f473654d52733666bb1983519b7320aab1da --- /dev/null +++ b/processed_dataset/proof/1796.json @@ -0,0 +1,8 @@ +{ + "source_file": "./raw_volume-zh/volume7/exercise10.tex", + "problem_type": "proof", + "problem": "问题15. 如图, 设 $\\triangle A B C$ 内存在一点 $F$, 使得 $\\angle A F B= \\angle B F C=\\angle C F A$, 直线 $B F 、 C E$ 分别交 $A C$ 、 $A B$ 于 $D 、 E$. 证明: $A B+A C \\geqslant 4 D E$.", + "solution": "证明: 设 $A F=x, B F=y, C F=z$. 由 $S_{\\triangle A C F}=S_{\\triangle A D F}+S_{\\triangle C D F}$, 得 $D F=\\frac{x z}{x+z}$. 同理, $E F=\\frac{x y}{x+y}$. 于是, 只要证明 $\\sqrt{x^2+x y+y^2}+\\sqrt{x^2+x z+z^2} \\geqslant 4 \\sqrt{\\left(\\frac{x y}{x+y}\\right)^2+\\left(\\frac{x z}{x+z}\\right)^2+\\left(\\frac{x y}{x+y}\\right)\\left(\\frac{x z}{x+z}\\right)}$. 因为 $x+ y \\geqslant \\frac{4 x y}{x+y}, x+z \\geqslant \\frac{4 x z}{x+z}$, 所以, 只要证 $\\sqrt{x^2+x y+y^2}+\\sqrt{x^2+x z+z^2} \\geqslant \\sqrt{(x+y)^2+(x+z)^2+(x+y)(x+z)}$.\n平方化简后得 $2 \\sqrt{\\left(x^2+x y+y^2\\right)\\left(x^2+x z+z^2\\right)} \\geqslant x^2+2(y+z) x+y z$, 再平方化简后得 $3\\left(x^2-y z\\right)^2 \\geqslant 0$, 即原不等式成立.", + "remark": "", + "figures": [] +} \ No newline at end of file diff --git a/processed_dataset/proof/1797.json b/processed_dataset/proof/1797.json new file mode 100644 index 0000000000000000000000000000000000000000..ea40a98df429cc40bebec68be730e645c1b452cf --- /dev/null +++ b/processed_dataset/proof/1797.json @@ -0,0 +1,10 @@ +{ + "source_file": "./raw_volume-zh/volume7/exercise10.tex", + "problem_type": "proof", + "problem": "问题16. 设 $H$ 为锐角 $\\triangle A B C$ 的垂心, $\\triangle A B C$ 的三条高线中最长的一条记为 $h_{\\max }$. 证明: $A H+B H+ C H \\leqslant 2 h_{\\max }$.", + "solution": "证明: 不妨设 $A B=c, A C=b, B C=a$, 且不妨设 $a \\leqslant b \\leqslant c$. 如图(), 作 $H$ 关于 $B C$ 的对称点 $H^{\\prime}$, 连结 $H^{\\prime} H 、 H^{\\prime} B 、 H^{\\prime} C$. 因 $\\angle B C H^{\\prime}=\\angle B C H$, $\\angle B C H=\\angle B A H$, 所以, $A 、 B 、 H^{\\prime} 、 C$ 四点共圆.\n则由托勒密定理知 $A H^{\\prime}$. $B C=C H^{\\prime} \\cdot A B+B H^{\\prime} \\cdot A C \\Leftrightarrow\\left(2 h_a-A H\\right) B C=C H \\cdot A B+B H \\cdot A C \\geqslant C H \\cdot B C+B H \\cdot B C$. 故 $2 h_a-A H \\geqslant B H+C H$, 因此 $A H+B H+C H \\leqslant 2 h_a$. 因为 $h_a$ 是三条高线中最长的, 所以, $A H+B H+C H \\leqslant 2 h_{\\max }$.", + "remark": "", + "figures": [ + "./images/volume7/figures/fig-c10a16.png" + ] +} \ No newline at end of file diff --git a/processed_dataset/proof/1798.json b/processed_dataset/proof/1798.json new file mode 100644 index 0000000000000000000000000000000000000000..582d1cc6755e12c7bb32b2a24de70fc976725467 --- /dev/null +++ b/processed_dataset/proof/1798.json @@ -0,0 +1,10 @@ +{ + "source_file": "./raw_volume-zh/volume7/exercise10.tex", + "problem_type": "proof", + "problem": "问题17. 设 $\\triangle A B C$ 是等边三角形, $P$ 是其内部一点, 线段 $A P 、 B P 、 C P$ 依次交三边 $B C 、 C A 、 A B$ 于 $A_1 、 B_1 、 C_1$ 三点.\n证明: $A_1 B_1 \\cdot B_1 C_1 \\cdot C_1 A_1 \\geqslant A_1 B \\cdot B_1 C \\cdot C_1 A$.", + "solution": "如图(), 由余弦定理 $A_1 B_1^2=A_1 C^2+B_1 C^2-A_1 C \\cdot B_1 C \\geqslant 2 A_1 C \\cdot B_1 C- A_1 C \\cdot B_1 C=A_1 C \\cdot B_1 C$. 同理, $B_1 C_1^2 \\geqslant B_1 A \\cdot C_1 A, C_1 A_1^2 \\geqslant C_1 B \\cdot A_1 B$. 由塞瓦定理得 $\\frac{A_1 C}{A_1 B} \\cdot \\frac{B_1 A}{B_1 C} \\cdot \\frac{C_1 B}{C_1 A}=1$. $A_1 B \\cdot B_1 C \\cdot C_1 A \\cdot \\sqrt{\\frac{A_1 C \\cdot B_1 A \\cdot C_1 B}{A_1 B \\cdot B_1 C \\cdot C_1 A}}=A_1 B \\cdot B_1 C \\cdot C_1 A$.", + "remark": "", + "figures": [ + "./images/volume7/figures/fig-c10a17.png" + ] +} \ No newline at end of file diff --git a/processed_dataset/proof/1799.json b/processed_dataset/proof/1799.json new file mode 100644 index 0000000000000000000000000000000000000000..5b9d9a138178cd1196b7f2ae7275c928ef5721a9 --- /dev/null +++ b/processed_dataset/proof/1799.json @@ -0,0 +1,10 @@ +{ + "source_file": "./raw_volume-zh/volume7/exercise11.tex", + "problem_type": "proof", + "problem": "问题2. 如图(),在 $\\triangle A B C$ 中, $O$ 为外心, 三条高线交于 $H, D 、 E 、 F$ 为垂足, 直线 $E D 、 A B$ 交于 $M$, 直线 $F D 、 A C$ 交于 $N$. 求证:\n(1) $O B \\perp D F, O C \\perp D E$;\n(2) $\\mathrm{OH} \\perp M N$.", + "solution": "证明: (1) 设 $\\triangle A B C$ 外接圆半径为 $R$, 则有\n$$\n\\begin{aligned}\n\\overrightarrow{O B} \\cdot \\overrightarrow{D F} & =\\overrightarrow{O B} \\cdot(\\overrightarrow{D B}+\\overrightarrow{B F}) \\\\\n& =\\overrightarrow{O B} \\cdot \\overrightarrow{D B}+\\overrightarrow{O B} \\cdot \\overrightarrow{B F} \\\\\n& =\\overrightarrow{B O} \\cdot \\overrightarrow{B D}-\\overrightarrow{B O} \\cdot \\overrightarrow{B F} \\\\\n& =|\\overrightarrow{B O}| \\cdot|\\overrightarrow{B D}| \\cdot \\cos \\angle D B O-|\\overrightarrow{B O}| \\cdot|\\overrightarrow{B F}| \\cdot \\cos \\angle F B O \\\\\n& =R \\cdot|\\overrightarrow{B D}| \\sin \\angle B A C-R \\cdot|\\overrightarrow{B F}| \\sin \\angle A C B \\\\\n& =\\frac{1}{2}|\\overrightarrow{B D}| \\cdot|\\overrightarrow{B C}|-\\frac{1}{2}|\\overrightarrow{B F}| \\cdot|\\overrightarrow{B A}| .\n\\end{aligned}\n$$\n因为四边形 $A F D C$ 为圆内接四边形, 所以 $|\\overrightarrow{B F}| \\cdot|\\overrightarrow{B A}|=|\\overrightarrow{B D}| \\cdot|\\overrightarrow{B C}|$.\n则 $\\overrightarrow{O B} \\cdot \\overrightarrow{D F}=0$.\n故 $\\overrightarrow{O B} \\perp \\overrightarrow{D F}$, 即 $O B \\perp D F$.\n同理, $O C \\perp D E$.\n(2) $\\overrightarrow{O H} \\cdot \\overrightarrow{M N}=\\overrightarrow{O H} \\cdot(\\overrightarrow{A N}-\\overrightarrow{A M})=\\overrightarrow{O H} \\cdot \\overrightarrow{A N}-\\overrightarrow{O H} \\cdot \\overrightarrow{A M}$.\n而 $\\overrightarrow{O H} \\cdot \\overrightarrow{A N}=(\\overrightarrow{O A}+\\overrightarrow{O B}+\\overrightarrow{O C}) \\cdot \\overrightarrow{A N}$\n$$\n\\begin{aligned}\n& =(\\overrightarrow{O A}+\\overrightarrow{O C}) \\cdot \\overrightarrow{A N}+\\overrightarrow{O B} \\cdot \\overrightarrow{A N} \\\\\n& =\\overrightarrow{O B} \\cdot \\overrightarrow{A N}[\\text { 因为 }(\\overrightarrow{O A}+\\overrightarrow{O C}) \\perp \\overrightarrow{A N}] \\\\\n& =\\overrightarrow{O B} \\cdot(\\overrightarrow{F N}-\\overrightarrow{F A}) \\\\\n& =\\overrightarrow{O B} \\cdot \\overrightarrow{F N}-\\overrightarrow{O B} \\cdot \\overrightarrow{F A} \\\\\n& =-\\overrightarrow{O B} \\cdot \\overrightarrow{F A} \\text { (因为 } \\overrightarrow{O B} \\perp \\overrightarrow{F N}, \\text { 由 (1)) } \\\\\n& =\\overrightarrow{B O} \\cdot \\overrightarrow{F A}=|\\overrightarrow{B O}| \\cdot|\\overrightarrow{F A}| \\cdot \\cos \\angle O B A \\\\\n& =R \\cdot|\\overrightarrow{A C}| \\cdot \\cos \\angle B A C \\cdot \\sin \\angle A C B \\\\\n& =\\frac{1}{2}|\\overrightarrow{A C}| \\cdot|\\overrightarrow{A B}| \\cdot \\cos \\angle B A C=\\frac{1}{2} \\overrightarrow{A C} \\cdot \\overrightarrow{A B} .\n\\end{aligned}\n$$\n同理, $\\overrightarrow{O H} \\cdot \\overrightarrow{A M}=\\frac{1}{2} \\overrightarrow{A C} \\cdot \\overrightarrow{A B}$.\n则 $\\overrightarrow{O H} \\cdot \\overrightarrow{M N}=0$. 有 $\\overrightarrow{O H} \\perp \\overrightarrow{M N}$. 即 $O H \\perp M N$.", + "remark": "", + "figures": [ + "./images/volume7/figures/fig-c11p2.png" + ] +} \ No newline at end of file diff --git a/processed_dataset/proof/1800.json b/processed_dataset/proof/1800.json new file mode 100644 index 0000000000000000000000000000000000000000..8bcf82f2875771ec75ce1b83f9d2e3c0e70c8342 --- /dev/null +++ b/processed_dataset/proof/1800.json @@ -0,0 +1,10 @@ +{ + "source_file": "./raw_volume-zh/volume7/exercise11.tex", + "problem_type": "proof", + "problem": "问题3. 如图(),在 $\\square A B C D$ 两边 $B C 、 C D$ 向外分别作正方形 $B C N M 、 C D P Q$. 求证: $A C \\perp Q N$.", + "solution": "证明: $\\overrightarrow{A C} \\cdot \\overrightarrow{Q N}=(\\overrightarrow{A B}+\\overrightarrow{B C}) \\cdot(\\overrightarrow{Q C}+\\overrightarrow{C N})$\n$$\n=\\overrightarrow{A B} \\cdot \\overrightarrow{Q C}+\\overrightarrow{A B} \\cdot \\overrightarrow{C N}+\\overrightarrow{B C} \\cdot \\overrightarrow{Q C}+\\overrightarrow{B C} \\cdot \\overrightarrow{C N}\n$$\n$$\n\\begin{aligned}\n& =\\overrightarrow{A B} \\cdot \\overrightarrow{C N}+\\overrightarrow{B C} \\cdot \\overrightarrow{Q C} \\text { (因为 } \\overrightarrow{A B} \\perp \\overrightarrow{Q C}, \\overrightarrow{B C} \\perp \\overrightarrow{C N} \\text { ) } \\\\\n& =\\overrightarrow{C B} \\cdot \\overrightarrow{C Q}-\\overrightarrow{C D} \\cdot \\overrightarrow{C N} \\\\\n& =|\\overrightarrow{C B}| \\cdot|\\overrightarrow{C Q}| \\cdot \\cos \\angle B C Q-|\\overrightarrow{C D}| \\cdot|\\overrightarrow{C N}| \\cdot \\\\\n& =\\cos \\angle D C N .\n\\end{aligned}\n$$\n因为 $|\\overrightarrow{C B}|=|\\overrightarrow{C N}|,|\\overrightarrow{C Q}|=|\\overrightarrow{C D}|, \\angle B C Q=\\angle D C N$, 所以 $\\overrightarrow{A C}$. $\\overrightarrow{Q N}=0 \\Rightarrow \\overrightarrow{A C} \\perp \\overrightarrow{Q N}$, 即 $A C \\perp Q N$.", + "remark": "", + "figures": [ + "./images/volume7/figures/fig-c11p3.png" + ] +} \ No newline at end of file diff --git a/processed_dataset/proof/1801.json b/processed_dataset/proof/1801.json new file mode 100644 index 0000000000000000000000000000000000000000..2881bfe7e701704ff240f735894de61d55abe067 --- /dev/null +++ b/processed_dataset/proof/1801.json @@ -0,0 +1,10 @@ +{ + "source_file": "./raw_volume-zh/volume7/exercise11.tex", + "problem_type": "proof", + "problem": "问题4. 设 $A D$ 是 $\\triangle A B C$ 的中线, $l$ 是垂直于 $A D$ 的一条直线, $M$ 是 $l$ 上一点, $E$ 、 $F$ 分别为 $M B 、 M C$ 的中点, 过点 $E 、 F$ 且垂直于 $l$ 的直线分别与 $A B 、 A C$ 交于点 $P 、 Q, l^{\\prime}$ 是过点 $M$ 且垂直于 $P Q$ 的直线.\n证明: $l^{\\prime}$ 总过一定点.", + "solution": "证明: 如图(), 以 $A D$ 所在直线为 $y$ 轴、直线 $l$ 为 $x$ 轴建立直角坐标系.\n设点 $A(0, a) 、 D(0$, $-d) 、 M(m, 0) 、 B(-b,-d+c) 、 C(b,-d-$ c).\n故 $l_{A B}: y=\\frac{a+d-c}{b} x+a$.\n又 $x_E=\\frac{m-b}{2}$, 而 $P E \\perp x$ 轴, 则\n$$\nP\\left(\\frac{m-b}{2}, \\frac{m-b}{2} \\cdot \\frac{a+d-c}{b}+a\\right) .\n$$\n同理, $Q\\left(\\frac{m+b}{2}, \\frac{m+b}{2} \\cdot \\frac{-(a+d+c)}{b}+a\\right)$.\n故直线 $d^{\\prime}$ 的斜率为\n$$\n\\begin{aligned}\nk & =\\frac{-1}{k_{p q}} \\\\\n& =\\frac{\\frac{m+b}{2}-\\frac{m-b}{2}}{\\frac{m-b}{2} \\cdot \\frac{a+d-c}{b}+a+\\frac{a+d+c}{b} \\cdot \\frac{m+b}{2}-a} \\\\\n& =\\frac{b^2}{m a+m d+b c} .\n\\end{aligned}\n$$\n又直线 $l^{\\prime}$ 过点 $M$, 则 $l^{\\prime}$ 的方程为\n$$\ny=\\frac{b^2}{m a+m d+b c}(x-m), \\label{eq1}\n$$\n其中, $m$ 为变量, $a 、 d 、 b 、 c$ 均为常量.\n由式 \\ref{eq1} 得\n$$\nm\\left[y(a+d)+b^2\\right]=b(b x-c y) .\n$$\n因此, 令 $y_0=\\frac{-b^2}{a+d}$. 则 $x_0=\\frac{-b c}{a+d}$.\n所以, 直线 $l^{\\prime}$ 恒过定点 $\\left(\\frac{-b c}{a+d}, \\frac{-b^2}{a+d}\\right)$.", + "remark": "", + "figures": [ + "./images/volume7/figures/fig-c11a4.png" + ] +} \ No newline at end of file diff --git a/processed_dataset/proof/1802.json b/processed_dataset/proof/1802.json new file mode 100644 index 0000000000000000000000000000000000000000..18a6cdb2891d30d995f33b510400851751d0385e --- /dev/null +++ b/processed_dataset/proof/1802.json @@ -0,0 +1,10 @@ +{ + "source_file": "./raw_volume-zh/volume7/exercise11.tex", + "problem_type": "proof", + "problem": "问题5. 设点 $A$ 是圆 $O$ 外一点, 过点 $A$ 作圆 $O$ 的切线, 切点分别为 $B 、 C$. 圆 $O$ 的切线 $l$ 与 $A B 、 A C$ 分别交于点 $P 、 Q$, 过点 $P$ 且平行于 $A C$ 的直线与 $B C$ 交于点 $R$. 求证: 无论 $l$ 如何变化, $Q R$ 恒过一定点.", + "solution": "证明: 如图(), 以 $O$ 为原点, $O A$ 所在直线为 $y$ 轴建立右手直角坐标系, 且不妨设 $\\odot O$ 半径为 1 . $B\\left(-x_0, y_0\\right), C\\left(x_0, y_0\\right)\\left(x_0^2+y_0^2=1\\right)$.\n取 $A B$ 与 $x$ 轴交点 $S$, 则 $A\\left(0, \\frac{1}{y_0}\\right)$, $S\\left(-\\frac{1}{x_0}, 0\\right)$. 设直线 $P Q: x_1 x+y_1 y=1, x_1^2+\\left.\\frac{x_1+x_0}{x_1 y_0+y_1 x_0}\\right)$. 同理, $Q\\left(\\frac{y_0-y_1}{x_1 y_0-y_1 x_0}, \\frac{x_1-x_0}{x_1 y_0-y_1 x_0}\\right)$ 则\n$$\nl_{P R}: y=-\\frac{x_0}{y_0}\\left(x-\\frac{y_0-y_1}{x_1 y_0+y_1 x_0}\\right)+\\frac{x_1+x_0}{x_1 y_0+y_1 x_0} \\cdots \\text { (1). }\n$$\n在(1)中令 $y=y_0$, 所以\n$$\nR\\left(\\frac{2 x_0 y_0-x_0 y_1+x_1 y_0-x_1 y_0^3-y_1 x_0 y_0^2}{x_0\\left(x_1 y_0+y_1 x_0\\right)}, y_0\\right) .\n$$\n下证: $Q 、 R 、 S$ 三点共线.\n(*)\n$$\n\\begin{aligned}\n& (*) \\Leftrightarrow K_{R S}=K_{Q S} \\Leftrightarrow \\frac{y_0 x_0\\left(x_1 y_0+y_1 x_0\\right)}{2 x_0 y_0-x_0 y_1+x_1 y_0-x_1 y_0^3-y_1 x_0 y_0^2+x_1 y_0+y_1 x_0}= \\\\\n& \\frac{x_0 x_1-x_0^2}{x_0 y_0+x_1 y_0-2 x_0 y_1} \\Leftrightarrow 2 x_0 x_1 y_0-x_0 x_1 y_1+x_1^2 y_0-x_1^2 y_0^3-x_0 x_1 y_0^2 y_1+x_1^2 y_0+ \\\\\n& x_0 x_1 y_1-2 x_0^2 y_0+x_0^2 y_1-x_0 x_1 y_0+x_0 x_1 y_0^3+x_0^2 y_1 y_0^2-x_0 x_1 y_0-x_0^2 y_1= \\\\\n& x_0 x_1 y_0^3+x_1^2 y_0^3-2 x_0 x_1 y_1 y_0^2+x_0^2 y_0^2 y_1+x_0 x_1 y_0^2 y_1-2 x_0^2 y_0 y_1^2 \\Leftrightarrow 2 x_1^2 y_0-2 x_1^2 y_0^3- \\\\\n& 2 x_0^2 y_0=-2 x_0^2 y_0 y_1^2\n\\end{aligned}\n$$\n由于 $y_0^2=1-x_0^2, y_1^2=1-x_1^2$. * 式显然成立成立.\n所以 $(*)$ 得证.\n又 $A$ 为定点, 所以 $B 、 C$ 均为定点.\n所以 $S$ 为定点.\n因此 $Q R$ 恒过定点 $\\left(-\\frac{1}{x_0}, 0\\right)$, 得证.", + "remark": "", + "figures": [ + "./images/volume7/figures/fig-c11a5.png" + ] +} \ No newline at end of file diff --git a/processed_dataset/proof/1803.json b/processed_dataset/proof/1803.json new file mode 100644 index 0000000000000000000000000000000000000000..d10e07f22d0708aa24d77478ae98bf5cb58db2b2 --- /dev/null +++ b/processed_dataset/proof/1803.json @@ -0,0 +1,10 @@ +{ + "source_file": "./raw_volume-zh/volume7/exercise11.tex", + "problem_type": "proof", + "problem": "问题6. $A 、 B$ 为平面上的两个定点, $C$ 为平面上位于直线 $A B$ 同侧的一个动点, 以 $A C 、 B C$ 各为边,在 $\\triangle A B C$ 外作正方形 $C A D l 、 C B E J$. 证明: 无论 $C$ 点取在直线 $A B$ 同侧的任何位置, $D E$ 的中点 $M$ 位置不变.", + "solution": "如图(), 设图中各字母表示相应点的复数, 由题设, 应有\n$$\n\\begin{aligned}\n& D=A+(C-A) \\mathrm{i}, \\\\\n& E=B+(B-C) \\mathrm{i},\n\\end{aligned}\n$$\n从而 $M=\\frac{(D+E)}{2}=\\frac{A(1-\\mathrm{i})+B(1+\\mathrm{i})}{2}$ 与 $C$ 无关.\n(事实上, $\\triangle A M B$ 为等腰直角三角形).", + "remark": "", + "figures": [ + "./images/volume7/figures/fig-c11a6.png" + ] +} \ No newline at end of file diff --git a/processed_dataset/proof/1804.json b/processed_dataset/proof/1804.json new file mode 100644 index 0000000000000000000000000000000000000000..597bec86d437d211cf804aa1ecf630bf2aad4166 --- /dev/null +++ b/processed_dataset/proof/1804.json @@ -0,0 +1,10 @@ +{ + "source_file": "./raw_volume-zh/volume7/exercise11.tex", + "problem_type": "proof", + "problem": "问题7. 求证:任意凸四边形各边中点连线的中点必重合.", + "solution": "证明: 如图(), $a b c d$ 是任意四边形, $h, k, f, g$ 是各边的中点.\n因为 $h=\\frac{a+b}{2}, k=\\frac{b+c}{2}, f=\\frac{c+d}{2}, g=\\frac{d+a}{2}$\n故 $\\overrightarrow{h f}$ 的中点为 $O_1=\\frac{1}{2}(h+f)=\\frac{1}{4}(a+b+c+d)\\overrightarrow{k g}$ 的中点为 $O_2=\\frac{1}{2}(k+g)=\\frac{1}{4}(a+b+c+d)$\n由此知 $O_1=O_2$. 这就是要证得结果.", + "remark": "", + "figures": [ + "./images/volume7/figures/fig-c11a7.png" + ] +} \ No newline at end of file diff --git a/processed_dataset/proof/1805.json b/processed_dataset/proof/1805.json new file mode 100644 index 0000000000000000000000000000000000000000..7b7da292f7ba6a97b9a4d73ddb1a358249d78ffc --- /dev/null +++ b/processed_dataset/proof/1805.json @@ -0,0 +1,10 @@ +{ + "source_file": "./raw_volume-zh/volume7/exercise11.tex", + "problem_type": "proof", + "problem": "问题9. 凸四边形 $A B C D$ 中, 点 $M, N$ 在边 $A B$ 上, 使 $A M=M N=N B$, 点 $P 、 Q$ 在边 $C D$ 上, 使 $C P=P Q=Q D$. 求证: $S_{\\text {四边形 } A M C P}=S_{\\text {四边形 } M N P Q}== \\frac{1}{3} S_{\\text {四边形 } A B C D}$.", + "solution": "证明: 如图(), 连结 $A C 、 M P$. 因 $S_{\\triangle A C P}= \\frac{1}{3} S_{\\triangle A C D}, S_{\\triangle A C M}=\\frac{1}{3} S_{\\triangle A C B}$, 则 $S_{\\triangle A C P}+S_{\\triangle A C M}= \\frac{1}{3}\\left(S_{\\triangle A C D}+S_{\\triangle A C B}\\right)$, 即 $S_{\\text {四边形 } A M C P}=\\frac{1}{3} S_{\\text {四边形 } A B C D \\text {. 又 }} S_{\\triangle M P Q}=S_{\\triangle M P C}, S_{\\triangle M P N}=S_{\\triangle M P A}$, 则 $S_{\\triangle M P Q}+S_{\\triangle M P N}= S_{\\triangle M P C}+S_{\\triangle M P A}$, 即 $S_{\\text {四边形 } M N P Q}=S_{\\text {四边形 } A M C P}$. 综上,\n$S_{\\text {四边形 } A M C P}=S_{\\text {四边形 } M N P Q}=\\frac{1}{3} \\cdot S_{\\text {四边形 } A B C D}$.", + "remark": "", + "figures": [ + "./images/volume7/figures/fig-c11a9.png" + ] +} \ No newline at end of file diff --git a/processed_dataset/proof/1806.json b/processed_dataset/proof/1806.json new file mode 100644 index 0000000000000000000000000000000000000000..783d0d55ddce79a328ba9196fc31da5e445da905 --- /dev/null +++ b/processed_dataset/proof/1806.json @@ -0,0 +1,8 @@ +{ + "source_file": "./raw_volume-zh/volume7/exercise11.tex", + "problem_type": "proof", + "problem": "问题10. 凸六边形 $P_1 P_2 P_3 P_4 P_5 P_6$ 的各边之长相等, 每个顶点关于两个相邻顶点的连线的对称点分别为 $P_1^{\\prime} 、 P_2^{\\prime} 、 P_3^{\\prime} 、 P_4^{\\prime} 、 P_5^{\\prime} 、 P_6^{\\prime}$. 证明: $\\triangle P_1^{\\prime} P_3^{\\prime} P_5^{\\prime} \\cong \\triangle P_2^{\\prime} P_4^{\\prime} P_6^{\\prime}$.", + "solution": "证明: 将平面上的点视为复数, 由于四边形 $P_1 P_2 P_1^{\\prime} P_6$ 为菱形, 则 $P_1^{\\prime}=P_2+P_6-P_1$. 同理, $P_3^{\\prime}=P_4+P_2-P_3, P_5^{\\prime}=P_6+P_4-P_5$. 故 $P_1^{\\prime} P_3^{\\prime}= P_3^{\\prime}-P_1^{\\prime}=\\left(P_1+P_4\\right)-\\left(P_3+P_6\\right), P_3^{\\prime} P_5^{\\prime}=\\left(P_3+P_6\\right)-\\left(P_2+P_5\\right), P_5^{\\prime} P_1^{\\prime}= \\left(P_2+P_5\\right)-\\left(P_1+P_4\\right)$. 因此, $\\triangle P_1^{\\prime} P_3^{\\prime} P_5^{\\prime}$ 全等于 $P_1+P_4 、 P_2+P_5 、 P_3+P_6$ 三点所成的三角形.\n同样, 由 $P_2^{\\prime}=P_3+P_1-P_2, P_4^{\\prime}=P_5+P_3-P_4, P_6^{\\prime}= P_1+P_5-P_6$, 得到 $\\triangle P_2^{\\prime} P_4^{\\prime} P_6^{\\prime}$ 也全等于 $P_1+P_4 、 P_2+P_5 、 P_3+P_6$ 三点所成的三角形.\n因此, $\\triangle P_1^{\\prime} P_3^{\\prime} P_5^{\\prime} \\cong \\triangle P_2^{\\prime} P_4^{\\prime} P_6^{\\prime}$.", + "remark": "", + "figures": [] +} \ No newline at end of file diff --git a/processed_dataset/proof/1807.json b/processed_dataset/proof/1807.json new file mode 100644 index 0000000000000000000000000000000000000000..f24a410ffa4c3c7c2872350c4367c69629256204 --- /dev/null +++ b/processed_dataset/proof/1807.json @@ -0,0 +1,10 @@ +{ + "source_file": "./raw_volume-zh/volume7/exercise11.tex", + "problem_type": "proof", + "problem": "问题11. 已知梯形 $A B C D$,边 $A B / / C D$,对角线 $A C 、 B D$ 交于点 $O$, 在 $A D$ 上取一点 $P$, 使 $\\angle B P A=\\angle C P D$, 在 $B C$ 上取一点 $Q$, 使 $\\angle A Q B=\\angle D Q C$. 求证: $O$ 到 $P 、 Q$ 的距离相等.", + "solution": "证明: 如图(), 连结 $P N$, 过 $O$ 作 $M N / / A B$, 交 $A D$ 于 $M, B C$ 于 $N$, 则 $\\frac{B P}{A B}=\\frac{\\sin \\angle B A P}{\\sin \\angle B P A}=\\frac{\\sin \\angle A D C}{\\sin \\angle B P A}= \\frac{\\sin \\angle P D C}{\\sin \\angle B P A}=\\frac{\\sin \\angle P D C}{\\sin \\angle D P C}=\\frac{P C}{D C}$.\n于是 $\\frac{B P}{C P}=\\frac{A B}{D C}=\\frac{B O}{O D}=\\frac{B N}{N C}$. 故 $P N$ 平分 $\\angle B P C$, 结合 $\\angle B P A=\\angle C P D$ 知 $N P \\perp A D$, 又 $O M=A B \\cdot \\frac{D O}{D B}=A B \\cdot \\frac{C O}{C A}=O N$, 所以 $O P$ 为 Rt $\\triangle M N P$ 斜边 $M N$ 的中线, 故 $O P=\\frac{1}{2} \\cdot M N$, 同理 $O Q=\\frac{1}{2} M N$. 故 $O P=O Q$.", + "remark": "", + "figures": [ + "./images/volume7/figures/fig-c11a11.png" + ] +} \ No newline at end of file diff --git a/processed_dataset/proof/1808.json b/processed_dataset/proof/1808.json new file mode 100644 index 0000000000000000000000000000000000000000..3570d2df911f5e677a1604dd87eb65c732666851 --- /dev/null +++ b/processed_dataset/proof/1808.json @@ -0,0 +1,10 @@ +{ + "source_file": "./raw_volume-zh/volume7/exercise11.tex", + "problem_type": "proof", + "problem": "问题12. 已知锐角 $\\triangle A B C$, 其内切圆与边 $A B 、 A C$ 分别切于点 $D 、 E, X 、 Y$ 分别是 $\\angle A C B 、 \\angle A B C$ 的平分线与 $D E$ 的交点, $Z$ 是边 $B C$ 的中点.\n求证: 当且仅当 $\\angle A=60^{\\circ}$ 时, $\\triangle X Y Z$ 是等边三角形.", + "solution": "证明: 如图(), 设 $I$ 为 $\\triangle A B C$ 的内心.\n首先证明: $D 、 B 、 I 、 X$ 和 $E 、 I 、 C 、 Y$ 分别四点共圆.\n注意到 $\\angle X I B=180^{\\circ}-\\angle B I C=\\frac{1}{2} \\angle B+\\frac{1}{2} \\angle C=90^{\\circ}- \\frac{1}{2} \\angle A$. 由 $\\triangle A D E$ 为等腰三角形可得 $\\angle A D E=90^{\\circ}- \\frac{1}{2} \\angle A$. 所以, $\\angle X I B=\\angle A D E$. 因此, $D 、 B 、 I 、 X$ 四点共圆.\n同理, $E 、 I 、 C 、 Y$ 四点共圆.\n由此可知 $X Z=Y Z$, 且它们分别是 Rt $\\triangle X B C$ 和 Rt $\\triangle Y B C$ 的中线.\n所以, $\\triangle X Y Z$ 是等边三角形的充要条件是 $\\angle Y X Z=60^{\\circ}$. 易知 $\\angle Y X Z=60^{\\circ} \\Leftrightarrow \\angle Y X Z=\\angle Y X C+\\angle C X Z= \\angle A B Y+\\frac{1}{2} \\angle C=\\frac{1}{2}(\\angle B+\\angle C)=60^{\\circ} \\Leftrightarrow \\angle A=60^{\\circ}$. 因此, 所证命题成立.", + "remark": "", + "figures": [ + "./images/volume7/figures/fig-c11a12.png" + ] +} \ No newline at end of file diff --git a/processed_dataset/proof/1809.json b/processed_dataset/proof/1809.json new file mode 100644 index 0000000000000000000000000000000000000000..c4341a40045b52735a3bcf2844b3767c84a3b975 --- /dev/null +++ b/processed_dataset/proof/1809.json @@ -0,0 +1,10 @@ +{ + "source_file": "./raw_volume-zh/volume7/exercise11.tex", + "problem_type": "proof", + "problem": "问题13. 在 $\\triangle A B C$ 中, $\\angle B 、 \\angle C$ 的角平分线分别为 $B E 、 C D$, 其中点 $D 、 E$ 分别在边 $A B 、 A C$ 上, 设 $D E$ 的中点 $P$ 在边 $B C 、 A B 、 A C$ 上的投影分别为 $Q 、 M 、 N$. 证明: $P Q=P M+P N$.", + "solution": "证明: 如图(), 设 $B C=a, C A=b, A B=c$, 连结 $P A$ 、 $P B 、 P C$. 由 $S_{\\triangle P A C}=\\frac{1}{2} S_{\\triangle A C D}=\\frac{1}{2} \\cdot \\frac{b}{a+b} S_{\\triangle A B C}$, 知 $P N=\\frac{2 S_{\\triangle P A C}}{b}=\\frac{S_{\\triangle A B C}}{a+b}$. 由 $S_{\\triangle P A B}=\\frac{1}{2} S_{\\triangle A B E}=\\frac{1}{2}$. $\\frac{c}{a+c} S_{\\triangle A B C}$, 知 $P M=\\frac{2 S_{\\triangle P A B}}{c}=\\frac{S_{\\triangle A B C}}{a+c}$. 又 $S_{\\triangle P B C}= \\frac{1}{2}\\left(S_{\\triangle B C D}+S_{\\triangle B C E}\\right)=\\frac{1}{2}\\left(\\frac{a}{a+b}+\\frac{a}{a+c}\\right) S_{\\triangle A B C}=\\frac{a}{2}\\left(\\frac{1}{a+b}+\\frac{1}{a+c}\\right) S_{\\triangle A B C}$, 则 $P Q=\\frac{2 S_{\\triangle P B C}}{a}=\\left(\\frac{1}{a+b}+\\frac{1}{a+c}\\right) S_{\\triangle A B C}=P M+ P N$.", + "remark": "", + "figures": [ + "./images/volume7/figures/fig-c11a13.png" + ] +} \ No newline at end of file diff --git a/processed_dataset/proof/1810.json b/processed_dataset/proof/1810.json new file mode 100644 index 0000000000000000000000000000000000000000..5f43df75c9bc5290ffad0e8adc71d39ca47e7496 --- /dev/null +++ b/processed_dataset/proof/1810.json @@ -0,0 +1,10 @@ +{ + "source_file": "./raw_volume-zh/volume7/exercise11.tex", + "problem_type": "proof", + "problem": "问题14. 已知 $P$ 为正 $\\triangle A B C$ 内一点, $P$ 在边 $A B 、 B C 、 C A$ 上的投影分别为点 $A^{\\prime}$ 、 $B^{\\prime} 、 C^{\\prime}$. 记 $\\triangle A P C^{\\prime} 、 \\triangle B P A^{\\prime} 、 \\triangle C P B^{\\prime} 、 \\triangle A P A^{\\prime} 、 \\triangle B P B^{\\prime} 、 \\triangle C P C^{\\prime}$ 的内切圆半径分别为 $r_1 、 r_2 、 r_3 、 r_4 、 r_5 、 r_6$. 证明: $r_1+r_2+r_3=r_4+r_5+r_6$.", + "solution": "证明: 如图(), 连结 $A P 、 B P 、 C P, \\triangle A B P$ 中, $A P^2- A A^{\\prime 2}=B P^2-B A^{\\prime 2}\\left(=P A^{\\prime 2}\\right)$, 同理 $B P^2-B B^{\\prime 2}= C P^2-C B^{\\prime 2}, C P^2-C C^{\\prime 2}=A P^2-A C^{\\prime 2}$, 以上三式相加, 得 $A A^{\\prime 2}+B B^{\\prime 2}+C C^{\\prime 2}=B A^{\\prime 2}+C B^{\\prime 2}+A C^{\\prime 2}$, 即 $\\left(A A^{\\prime 2}-B A^{\\prime 2}\\right)+\\left(B B^{\\prime 2}-C B^{\\prime 2}\\right)+\\left(C C^{\\prime 2}-A C^{\\prime 2}\\right)= 0,\\left(A A^{\\prime}+B A^{\\prime}\\right)\\left(A A^{\\prime}-B A^{\\prime}\\right)+\\left(B B^{\\prime}+C B^{\\prime}\\right)\\left(B B^{\\prime}-\\right.\\left.C B^{\\prime}\\right)+\\left(C C^{\\prime}+A C^{\\prime}\\right)\\left(C C^{\\prime}-A C^{\\prime}\\right)=0$. 注意到 $A A^{\\prime}+B A^{\\prime}=B B^{\\prime}+C B^{\\prime}= C C^{\\prime}+A C^{\\prime}$, 所以 $A A^{\\prime}-B A^{\\prime}+B B^{\\prime}-C B^{\\prime}+C C^{\\prime}-A C^{\\prime}=0$.\n即 $A A^{\\prime}+B B^{\\prime}+C C^{\\prime}=B A^{\\prime}+C B^{\\prime}+A C^{\\prime}$ . *\n显然, $r_1=-\\frac{A C^{\\prime}+C^{\\prime} P-A P}{2}, \\quad r_2=\\frac{B A^{\\prime}+A^{\\prime} P-B P}{2}, \\quad r_3=$\n$$\n\\begin{aligned}\n& \\frac{C B^{\\prime}+P B^{\\prime}-P C}{2}, r_4=\\frac{A A^{\\prime}+A^{\\prime} P-A P}{2}, r_5=\\frac{B B^{\\prime}+B^{\\prime} P-P B}{2}, r_6= \\\\\n& \\frac{O C^{\\prime}+C^{\\prime} P-C P}{2} .\n\\end{aligned}\n$$\n由 $*$ 即有: $r_1+r_2+r_3=r_4+r_5+r_6$.", + "remark": "", + "figures": [ + "./images/volume7/figures/fig-c11a14.png" + ] +} \ No newline at end of file diff --git a/processed_dataset/proof/1811.json b/processed_dataset/proof/1811.json new file mode 100644 index 0000000000000000000000000000000000000000..a1379173cd5f2996a1d5becf7e2e5be4961628db --- /dev/null +++ b/processed_dataset/proof/1811.json @@ -0,0 +1,10 @@ +{ + "source_file": "./raw_volume-zh/volume7/exercise2.tex", + "problem_type": "proof", + "problem": "问题1. $\\triangle A B C$ 是一个三角形.\n一个过 $A 、 B$ 的圆交边 $A C 、 B C$ 于点 $D 、 E, A B$ 、 $D E$ 交于点 $F, B D 、 C F$ 交于点 $M$. 求证 : $M F=M C$. 的充要条件是 $M B$. $M D=M C^2$.", + "solution": "证明: 如图(), 因为 $A 、 B 、 E 、 D$ 四点共圆, 所以, $\\angle C B D=\\angle E A D$. 又 $A C 、 B M 、 F E$ 交于点 $D$, 由塞瓦定理有 $\\frac{F A}{A B} \\cdot \\frac{B E}{E C} \\cdot \\frac{C M}{M F}=1$. 因此, $M F=$\n$$\nM C \\Leftrightarrow \\frac{F A}{A B} \\cdot \\frac{B E}{E C}=1 \\Leftrightarrow A E / / C F \\Leftrightarrow \\angle F C A=\\angle E A C=\\angle M B C \\Leftrightarrow \\triangle M C D \\backsim\n$$\n$\\triangle M B C \\Leftrightarrow M B \\cdot M D=M C^2$.", + "remark": "", + "figures": [ + "./images/volume7/figures/fig-c2a1.png" + ] +} \ No newline at end of file diff --git a/processed_dataset/proof/1812.json b/processed_dataset/proof/1812.json new file mode 100644 index 0000000000000000000000000000000000000000..9ce93e213c5dd4a8e7e9fdbc1d81de749d0db649 --- /dev/null +++ b/processed_dataset/proof/1812.json @@ -0,0 +1,10 @@ +{ + "source_file": "./raw_volume-zh/volume7/exercise2.tex", + "problem_type": "proof", + "problem": "问题2. $M 、 N 、 P$ 分别是 $\\triangle A B C$ 的三边 $B C 、 C A 、 A B$ 的中点, $M_1 、 N_1 、 P_1$ 在 $\\triangle A B C$ 的边上, 且满足 $M M_1 、 N N_1 、 P P_1$ 分别平分 $\\triangle A B C$ 的周长.\n证明: $M M_1 、 N N_1 、 P P_1$ 交于同一点 $K$.", + "solution": "证明: 如图(),不妨设 $B C=a, A B=c$, $A C=b$, 且 $a \\geqslant c \\geqslant b$. 由 $M 、 N 、 P$ 为 $\\triangle A B C$ 三边中点, 有 $\\frac{P M_2}{P N}=\\frac{P M_2}{B M}=\\frac{P M_1}{B M_1}=\\frac{\\frac{b+c}{2}-\\frac{c}{2}}{\\frac{b+c}{2}}= \\frac{b}{b+c}$, 故 $\\frac{P M_2}{M_2 N}=\\frac{b}{c}$. 同理, $\\frac{N P_2}{P_2 M}=\\frac{a}{b}, \\frac{M N_2}{N_2 P}=\\frac{c}{a}$.\n所以, $\\frac{P M_2}{M_2 N} \\cdot \\frac{N P_2}{P_2} \\cdot \\frac{M N_2}{N_2 P}=1$, 因此, $M M_1 、 N N_1$ 、 $P P_1$ 交于同一点 $K$.", + "remark": "", + "figures": [ + "./images/volume7/figures/fig-c2a2.png" + ] +} \ No newline at end of file diff --git a/processed_dataset/proof/1813.json b/processed_dataset/proof/1813.json new file mode 100644 index 0000000000000000000000000000000000000000..05baa78b21102ec660a24297964a45afe978623f --- /dev/null +++ b/processed_dataset/proof/1813.json @@ -0,0 +1,10 @@ +{ + "source_file": "./raw_volume-zh/volume7/exercise2.tex", + "problem_type": "proof", + "problem": "问题3. 已知直线上的三个定点依次为 $A 、 B 、 C, \\Gamma$ 为过 $A 、 C$ 且圆心不在 $A C$ 上的圆.\n分别过 $A 、 C$ 两点且与圆 $\\Gamma$ 相切的直线交于点 $P, P B$ 与圆 $\\Gamma$ 交于点 $Q$. 证明: $\\angle A Q C$ 的平分线与 $A C$ 的交点不依赖于圆 $\\Gamma$ 的选取.", + "solution": "证明: 如图(), 假设 $\\angle A Q C$ 的平分线交 $A C$ 于点 $R$, 交圆 $\\Gamma$ 于点 $S$, 其中 $S$ 与 $Q$ 是不同的两点.\n因为 $\\triangle P A C$ 是等腰三角形, 所以, $\\frac{A B}{B C}=\\frac{\\sin \\angle A P B}{\\sin \\angle C P B}$, 同理,在 $\\triangle A S C$ 中, $\\frac{A R}{R C}=\\frac{\\sin \\angle A S Q}{\\sin \\angle C S Q}$. 在 $\\triangle P A C$ 中, 视 $Q$ 为其塞瓦点.\n由角元塞瓦定理, 有 $\\frac{\\sin \\angle A P B}{\\sin \\angle C P B} \\cdot \\frac{\\sin \\angle Q A C}{\\sin \\angle Q A P}$.\n$\\frac{\\sin \\angle Q C P}{\\sin \\angle Q C A}=1$. 因为 $\\angle P A Q=\\angle A S Q=\\angle Q C A$,\n$\\angle P C Q=\\angle C S Q=\\angle Q A C$, 则 $\\frac{\\sin \\angle A P B}{\\sin \\angle C P B}=\\frac{\\sin \\angle P A Q \\cdot \\sin \\angle Q C A}{\\sin \\angle Q A C \\cdot \\sin \\angle P C Q}= \\frac{\\sin ^2 \\angle A S Q}{\\sin ^2 \\angle C S Q}$. 故 $\\frac{A B}{B C}=\\frac{A R^2}{R C^2}$. 因此, 点 $R$ 不依赖于圆 $T$ 的选取.", + "remark": "", + "figures": [ + "./images/volume7/figures/fig-c2a3.png" + ] +} \ No newline at end of file diff --git a/processed_dataset/proof/1814.json b/processed_dataset/proof/1814.json new file mode 100644 index 0000000000000000000000000000000000000000..eba7064e05b19a94dee543000ab6ecf5a1580e68 --- /dev/null +++ b/processed_dataset/proof/1814.json @@ -0,0 +1,10 @@ +{ + "source_file": "./raw_volume-zh/volume7/exercise2.tex", + "problem_type": "proof", + "problem": "问题4. 已知非等边 $\\triangle A B C, \\angle A 、 \\angle B 、 \\angle C$ 的平分线分别交对边于点 $A^{\\prime} 、 B^{\\prime}$ 、 $C^{\\prime} . A A^{\\prime}$ 的中垂线与 $B C$ 交于点 $A^{\\prime \\prime}, B B^{\\prime}$ 的中垂线与 $A C$ 交于点 $B^{\\prime \\prime}, C C^{\\prime}$ 的中垂线交于点 $C^{\\prime \\prime}$. 证明: $A^{\\prime \\prime} 、 B^{\\prime \\prime} 、 C^{\\prime \\prime}$ 三点共线.", + "solution": "证明: 如图(), 注意到 $\\angle B A A^{\\prime}=\\angle A^{\\prime} A C$, $\\angle A^{\\prime \\prime} A A^{\\prime}=\\angle A^{\\prime \\prime} A^{\\prime} A$. 相减得 $\\angle A^{\\prime \\prime} A B=\\angle C$. 于是, $A A^{\\prime \\prime}$ 为 $\\triangle A B C$ 外接圆的切线.\n从而, $\\triangle A A^{\\prime \\prime} B \\backsim \\triangle C A^{\\prime \\prime} A$. 故 $\\frac{B A^{\\prime \\prime}}{A^{\\prime \\prime} C}=\\frac{B A^{\\prime \\prime}}{A A^{\\prime \\prime}} \\cdot \\frac{A A^{\\prime \\prime}}{A^{\\prime \\prime} C}=\\left(\\frac{A B}{A C}\\right)^2$. 同理, $\\frac{C B^{\\prime \\prime}}{B^{\\prime \\prime} A}=\\left(\\frac{B C}{B A}\\right)^2, \\frac{A C^{\\prime \\prime}}{C^{\\prime \\prime} B}=\\left(\\frac{A C}{C B}\\right)^2$. 所以, $\\frac{B A^{\\prime \\prime}}{A^{\\prime \\prime} C} \\cdot \\frac{C B^{\\prime \\prime}}{B^{\\prime \\prime} A}$.\n$\\frac{A C^{\\prime \\prime}}{C^{\\prime \\prime} B}=\\left(\\frac{A B}{A C} \\cdot \\frac{B C}{A B} \\cdot \\frac{A C}{C B}\\right)^2=1$. 由梅涅劳斯逆定理知 $A^{\\prime \\prime} 、 B^{\\prime \\prime} 、 C^{\\prime}$ 三点共线.", + "remark": "", + "figures": [ + "./images/volume7/figures/fig-c2a4.png" + ] +} \ No newline at end of file diff --git a/processed_dataset/proof/1815.json b/processed_dataset/proof/1815.json new file mode 100644 index 0000000000000000000000000000000000000000..4e77a83953d47bbc584cccce49ca7c185cbeebc0 --- /dev/null +++ b/processed_dataset/proof/1815.json @@ -0,0 +1,10 @@ +{ + "source_file": "./raw_volume-zh/volume7/exercise2.tex", + "problem_type": "proof", + "problem": "问题5. 已知 $\\triangle A B C$ 的三边 $B C 、 C A 、 A B$ 上各有一点 $D 、 E 、 F$, 且满足 $A D$ 、 $B E 、 C F$ 交于一点 $G$. 若 $\\triangle A G E 、 \\triangle C G D 、 \\triangle B G F$ 的面积相等, 证明: $G$ 是 $\\triangle A B C$ 的重心.", + "solution": "证明: 如图(), 设 $\\frac{A F}{F B}=x, \\frac{B D}{D C}=y, \\frac{C E}{E A}=z$. 由塞瓦定理得 $x y z==1$. 对于 $\\triangle B F C$ 和直线 $A G D$ 应用梅涅劳斯定理有 $\\frac{F G}{G C} \\cdot \\frac{C D}{D B} \\cdot \\frac{B A}{A F}=1$. 则 $\\frac{F G}{G C}=\\frac{B D}{D C} \\cdot \\frac{A F}{B A}= y \\cdot \\frac{x}{1+x}=\\frac{x y}{1+x}$. 所以, $\\frac{F G}{F C}=\\frac{x y}{1+x+x y}$. 故 $S_{\\triangle B F G}= \\frac{F G}{F C} S_{\\triangle B F C}=\\frac{F G}{F C} \\cdot \\frac{B F}{A B} S_{\\triangle A B C}=\\frac{x y}{(1+x+x y)(1+x)} S_{\\triangle A B C}$.\n同理, $S_{\\triangle C D G}=\\frac{y z}{(1+y+y z)(1+y)} S_{\\triangle A B C}$. 于是, $\\frac{x y}{(1+x+x y)(1+x)}= \\frac{y z}{(1+y+y z)(1+y)}$, 即 $x(1+y+y z)(1+y)=z(1+x+x y)(1+x)$. 因为 $x(1+y+y z)=x+x y+x y z=x+x y+1$, 所以, $1+y=z(1+x)=z+ z x$. 同理, $1+z=x+x y, 1+x=y+y z$. 三式相加得 $3=x y+y z+z x \\geqslant 3 \\sqrt[3]{x y \\cdot y z \\cdot z x}=3$. 当且仅当 $x y=y z=z x$ 时, 上式等号成立.\n所以, $x y= y z=z x$. 从而, $x=y=z$. 又 $x y z=1$, 则 $x=y=z=1$. 因此, $D 、 E 、 F$ 是三边的中点.\n故 $G$ 是 $\\triangle A B C$ 的重心.", + "remark": "", + "figures": [ + "./images/volume7/figures/fig-c2a5.png" + ] +} \ No newline at end of file diff --git a/processed_dataset/proof/1816.json b/processed_dataset/proof/1816.json new file mode 100644 index 0000000000000000000000000000000000000000..59f970110eb8fac0e747c3bb2ff5ad1bac8087a6 --- /dev/null +++ b/processed_dataset/proof/1816.json @@ -0,0 +1,10 @@ +{ + "source_file": "./raw_volume-zh/volume7/exercise2.tex", + "problem_type": "proof", + "problem": "问题6. 设 $\\triangle A B C$ 的边 $A B$ 的中点为 $N, \\angle A>\\angle B, D$ 是射线 $A C$ 上一点, 满足 $C D=B C, P$ 是射线 $D N$ 上一点, 且与点 $A$ 在边 $B C$ 的同侧, 满足\n$\\angle P B C=\\angle A, P C$ 与 $A B$ 交于点 $E, B C$ 与 $D P$ 交于点 $T$. 求表达式 $\\frac{B C}{T C}- \\frac{E A}{E B}$ 的值.", + "solution": "证明: 如图(), 延长 $B P$ 交直线 $A C$ 于 $F$. 于是, $\\triangle A C B \\backsim \\triangle B C F$. 从而, $A C \\cdot C F=B C^2$. 故 $C F=\\frac{B C^2}{A C}$. 注意到直线 $D T N$ 截 $\\triangle A B C$, 应用梅涅劳斯定理得 $\\frac{A N}{N B}$ ・ $\\frac{B T}{T C} \\cdot \\frac{C D}{D A}=1$. 则 $\\frac{B T}{T C}=\\frac{D A}{C D} \\cdot \\frac{B N}{A N}=\\frac{D A}{C D}=\\frac{A C}{B C}+1$. 故 $\\frac{B C}{T C}=\\frac{B T}{T C}+1=\\frac{A C}{B C}+2$. 同理, 由直线 $D N P$ 截 $\\triangle A B F$, 得 $\\frac{F P}{P B} \\cdot \\frac{B N}{N A} \\cdot \\frac{A D}{D F}=1$. 由直线 $C E P$ 截 $\\triangle A B F$, 得 $\\frac{F P}{P B} \\cdot\\frac{B E}{E A} \\cdot \\frac{A C}{C F}=1$. 故 $\\frac{E A}{B E}=\\frac{F P}{P B} \\cdot \\frac{A C}{C F}=\\frac{A N}{B N} \\cdot \\frac{F D}{A D} \\cdot \\frac{A C}{C F}= \\frac{F D}{A D} \\cdot \\frac{A C}{C F}=\\frac{\\frac{B C^2}{A C}+B C}{A C+B C} \\cdot \\frac{A C}{\\frac{B C^2}{A C}}=\\frac{B C}{A C} \\cdot \\frac{A C^2}{B C^2}=\\frac{A C}{B C}$. 因此, $\\frac{B C}{T C}-\\frac{E A}{B E}=2$.", + "remark": "", + "figures": [ + "./images/volume7/figures/fig-c2a6.png" + ] +} \ No newline at end of file diff --git a/processed_dataset/proof/1817.json b/processed_dataset/proof/1817.json new file mode 100644 index 0000000000000000000000000000000000000000..3b622de29b8796e18b9b8fe130e38ebddee4917b --- /dev/null +++ b/processed_dataset/proof/1817.json @@ -0,0 +1,8 @@ +{ + "source_file": "./raw_volume-zh/volume7/exercise2.tex", + "problem_type": "proof", + "problem": "问题7. 已知点 $B 、 C$ 分别在由点 $A$ 引出的两条射线上, 且 $A B+A C$ 为一定值.\n求证: $\\triangle A B C$ 的外接圆恒过不依赖于点 $B 、 C$ 的点 $D(D \\neq A)$.", + "solution": "证明: 作 $\\triangle A B C$ 的外接圆, 交 $\\angle B A C$ 的平分线于点 $D$. 下面证明: $D$ 是不依赖于点 $B 、 C$ 的定点.\n连结 $B C 、 B D 、 C D$. 在 $\\triangle B C D$ 中, 由正弦定理得\n$\\frac{B C}{\\sin A}=\\frac{B D}{\\sin \\frac{A}{2}}=\\frac{C D}{\\sin \\frac{A}{2}} \\cdots$ (1). 对圆内接四边形 $A B C D$ 应用托勒密定理得 $A B \\cdot C D+A C \\cdot B D=B C \\cdot A D \\cdots$ (2). 将式 (1) 代入式 (2) 得 $A D=(A B+A C) \\frac{\\sin \\frac{A}{2}}{\\sin A}=\\frac{A B+A C}{2 \\cos \\frac{A}{2}}$ 为定值.\n故所证结论成立.", + "remark": "", + "figures": [] +} \ No newline at end of file diff --git a/processed_dataset/proof/1818.json b/processed_dataset/proof/1818.json new file mode 100644 index 0000000000000000000000000000000000000000..5a00a2d29a8b3225ee943fb104f91b3529a4627e --- /dev/null +++ b/processed_dataset/proof/1818.json @@ -0,0 +1,10 @@ +{ + "source_file": "./raw_volume-zh/volume7/exercise2.tex", + "problem_type": "proof", + "problem": "问题8. 已知凸六边形 $A_1 A_2 A_3 A_4 A_5 A_6$ 所有的角都是钝角, 圆 $\\Gamma_i(1 \\leqslant i \\leqslant 6)$ 的圆心为 $A_i$, 且圆 $\\Gamma_i$ 分别与圆 $\\Gamma_{i-1}$ 和圆 $\\Gamma_{i+1}$ 相外切, 其中, $\\Gamma_0=\\Gamma_6, \\Gamma_1= \\Gamma_7$. 设过圆 $\\Gamma_1$ 的两个切点所连直线与过圆 $\\Gamma_3$ 的两个切点所连直线相交, 且过这个交点与点 $A_2$ 的直线为 $e$; 类似地, 由圆 $\\Gamma_3$ 、圆 $\\Gamma_5$ 和 $A_4$ 定义直线 $f$, 由圆 $\\Gamma_5$ 、圆 $\\Gamma_1$ 和 $A_6$ 定义直线 $g$. 证明 $: e 、 f 、 g$ 三线共点.", + "solution": "证明: 记这六个切点分别为 $B_1 、 B_2 、 B_3$ 、 $B_4 、 B_5 、 B_6$, 如图(), 设 $B_1 B_2 、 B_3 B_4 、 B_5 B_6$ 两两交于点 $P 、 Q 、 R$. 连结 $B_1 B_6 、 B_4 B_5 、 B_2 B_3$. 由角元塞瓦定理得 $\\frac{\\sin \\angle B_1 P A_6}{\\sin \\angle A_6 P B_6} \\cdot \\frac{\\sin \\angle P B_6 A_6}{\\sin \\angle A_6 B_6 B_1}$. $\\frac{\\sin \\angle B_6 B_1 A_6}{\\sin \\angle A_6 B_1 P}=1 \\cdots$ (1). 又 $A_6 B_6=A_6 B_1$, 则 $\\angle A_6 B_6 B_1=\\angle A_6 B_1 B_6$. 故式 (1) 为 $\\frac{\\sin \\angle B_1 P A_6}{\\sin \\angle A_6 P B_6}$.\n$\\frac{\\sin \\angle P B_6 A_6}{\\sin \\angle A_6 B_1 P}=1$. 完全类似地得 $\\frac{\\sin \\angle B_5 R A_4}{\\sin \\angle A_4 R B_4} \\cdot \\frac{\\sin \\angle R B_4 A_4}{\\sin \\angle A_4 B_5 R}=1, \\frac{\\sin \\angle B_3 Q A_2}{\\sin \\angle A_2 Q B_2}$. $\\frac{\\sin \\angle Q B_2 A_2}{\\sin \\angle A_2 B_3 Q}=1$. 以上三式相乘并由 $\\angle P B_6 A_6=\\angle A_5 B_6 B_5=\\angle A_5 B_5 B_6= \\angle R B_5 A_4, \\angle R B_4 A_4=\\angle A_3 B_4 B_3=\\angle A_3 B_3 B_4=\\angle Q B_3 A_2, \\angle Q B_2 A_2= \\angle A_1 B_2 B_1=\\angle A_1 B_1 B_2=\\angle P B_1 A_6$, 得 $\\frac{\\sin \\angle B_1 P A_6}{\\sin \\angle A_6 P B_6} \\cdot \\frac{\\sin \\angle B_5 R A_4}{\\sin \\angle A_4 R B_4}$. $\\frac{\\sin \\angle B_3 Q A_2}{\\sin \\angle A_2 Q B_2}=1$. 由角元塞瓦定理的逆定理知, $P A_6 、 Q A_2 、 R A_4$ 三线共点, 即 $e 、 f 、 g$ 三线共点.", + "remark": "", + "figures": [ + "./images/volume7/figures/fig-c2a8.png" + ] +} \ No newline at end of file diff --git a/processed_dataset/proof/1819.json b/processed_dataset/proof/1819.json new file mode 100644 index 0000000000000000000000000000000000000000..61efdb344c093d80b3f20dce1e8f6a01ab9bb05b --- /dev/null +++ b/processed_dataset/proof/1819.json @@ -0,0 +1,10 @@ +{ + "source_file": "./raw_volume-zh/volume7/exercise2.tex", + "problem_type": "proof", + "problem": "问题9. 设正方形 $P Q R S$ 内接于 $\\triangle A B C$, 其顶点 $P$ 和 $Q$ 在边 $B C$ 上, 顶点 $R$ 和 $S$ 分别在边 $C A$ 和 $A B$ 上, 记其中心为 $A_1$. 同样地, 定义两个顶点分别在边 $C A$ 和 $A B$ 上的内接正方形的中心依次为 $B_1$ 和 $C_1$. 求证: 直线 $A A_1$ 、 $B B_1 、 C C_1$ 三线共点.", + "solution": "证明:如图(),连结 $S A_1 、 R A_1$. 于是, $S A_1=R A_1$. 由正弦定理有\n$$\n\\frac{\\sin \\angle S A A_1}{\\sin \\angle A S A_1}=\\frac{S A_1}{A A_1}=\\frac{R A_1}{A A_1}=\\frac{\\sin \\angle A_1 A R}{\\sin \\angle A R A_1},\n$$\n则 $\\frac{\\sin \\angle B A A_1}{\\sin \\angle A_1 A C}=\\frac{\\sin \\angle A S A_1}{\\sin \\angle A R S_1}$. 因为 $A_1$ 为正方形\n$P Q R S$ 的中心, 所以, $\\angle S R A_1=\\angle R S A_1=45^{\\circ}$. 又因为 $S R / / B C$, 所以, $\\angle A S R=\\angle B, \\angle A R S=\\angle C$. 则 $\\angle A S A_1=\\angle B+45^{\\circ}, \\angle A R A_1=\\angle C+45^{\\circ}$.\n从而, $\\frac{\\sin \\angle B A A_1}{\\sin \\angle A_1 A C}=\\frac{\\sin \\left(B+45^{\\circ}\\right)}{\\sin \\left(C+45^{\\circ}\\right)}$. 同理, $\\frac{\\sin \\angle A C C_1}{\\sin \\angle C_1 C B}=\\frac{\\sin \\left(A+45^{\\circ}\\right)}{\\sin \\left(B+45^{\\circ}\\right)}$,\n$$\n\\frac{\\sin \\angle C B B_1}{\\sin \\angle B_1 B A}=\\frac{\\sin \\left(C+45^{\\circ}\\right)}{\\sin \\left(A+45^{\\circ}\\right)} \\text {. 故 } \\frac{\\sin \\angle B A A_1}{\\sin \\angle A_1 A C} \\cdot \\frac{\\sin \\angle A C C_1}{\\sin \\angle C_1 C B} \\cdot \\frac{\\sin \\angle C B B_1}{\\sin \\angle B_1 B A}=1 \\text {. }\n$$\n由角元 Ceva 定理的逆定理知结论成立.", + "remark": "", + "figures": [ + "./images/volume7/figures/fig-c2a9.png" + ] +} \ No newline at end of file diff --git a/processed_dataset/proof/1820.json b/processed_dataset/proof/1820.json new file mode 100644 index 0000000000000000000000000000000000000000..0190ee6fd572342fc543df1296afbe22755b4305 --- /dev/null +++ b/processed_dataset/proof/1820.json @@ -0,0 +1,10 @@ +{ + "source_file": "./raw_volume-zh/volume7/exercise2.tex", + "problem_type": "proof", + "problem": "问题10. 在 $\\triangle A B C$ 内部给定三点 $D 、 E 、 F$, 使得 $\\angle B A E=\\angle C A F, \\angle A B D= \\angle C B F$. 求证: $A D 、 B E 、 C F$ 三线共点的充分必要条件是 $\\angle A C D= \\angle B C E$.", + "solution": "证明: 如图(), 记 $\\angle B A E=\\angle C A F=\\alpha$, $\\angle A B D=\\angle C B F=\\beta, \\angle A C D=x, \\angle B C E=y$. 对 $\\triangle A B C$ 分别与点 $D 、 E 、 F$ 应用角元塞瓦定理有\n$$\n\\begin{aligned}\n1 & =\\frac{\\sin \\angle B A D}{\\sin \\angle D A C} \\cdot \\frac{\\sin \\angle A C D}{\\sin \\angle D C B} \\cdot \\frac{\\sin \\angle C B D}{\\sin \\angle D B A} \\\\\n& =\\frac{\\sin \\angle B A D}{\\sin \\angle D A C} \\cdot \\frac{\\sin x}{\\sin (C-x)} \\cdot \\frac{\\sin (B-\\beta)}{\\sin \\beta}, \\\\\n1 & =\\frac{\\sin \\angle A C F}{\\sin \\angle F C B} \\cdot \\frac{\\sin \\angle C B F}{\\sin \\angle F B A} \\cdot \\frac{\\sin \\angle B A F}{\\sin \\angle F A C} \\\\\n& =\\frac{\\sin \\angle A C F}{\\sin \\angle F C B} \\cdot \\frac{\\sin \\beta}{\\sin (B-\\beta)} \\cdot \\frac{\\sin (A-\\alpha)}{\\sin \\alpha}, \\\\\n1 & =\\frac{\\sin \\angle C B E}{\\sin \\angle E B A} \\cdot \\frac{\\sin \\angle B A E}{\\sin \\angle E A C} \\cdot \\frac{\\sin \\angle A C E}{\\sin \\angle E C B} \\\\\n& =\\frac{\\sin \\angle C B E}{\\sin \\angle E B A} \\cdot \\frac{\\sin \\alpha}{\\sin (A-\\alpha)} \\cdot \\frac{\\sin (C-y)}{\\sin y} .\n\\end{aligned}\n$$\n将三式相乘并整理得\n$1=\\frac{\\sin \\angle B A D}{\\sin \\angle D A C} \\cdot \\frac{\\sin \\angle A C F}{\\sin \\angle F C B} \\cdot \\frac{\\sin \\angle C B E}{\\sin \\angle E B A}=\\frac{\\sin (C-x)}{\\sin x} \\cdot \\frac{\\sin y}{\\sin (C-y)}$.\n即 $\\sin C \\cdot \\cot x-\\cos C=\\sin C \\cdot \\cot y-\\cos C, \\cot x=\\cot y, x=y$.\n由角元 Ceva 定理及其逆定理知, $A D 、 B E 、 C F$ 共线的充要条件是 $x= y$, 即 $\\angle A C D=\\angle B C E$.", + "remark": "", + "figures": [ + "./images/volume7/figures/fig-c2a10.png" + ] +} \ No newline at end of file diff --git a/processed_dataset/proof/1821.json b/processed_dataset/proof/1821.json new file mode 100644 index 0000000000000000000000000000000000000000..1e7e7a37df7461d2ba41e4aba1a11404558126cb --- /dev/null +++ b/processed_dataset/proof/1821.json @@ -0,0 +1,10 @@ +{ + "source_file": "./raw_volume-zh/volume7/exercise2.tex", + "problem_type": "proof", + "problem": "问题11. 以 $\\triangle A B C$ 的三边各为一边, 分别在形外作 $\\triangle C B D 、 \\triangle C A E 、 \\triangle A B F$, 使得 $\\angle B A F=\\angle C A E, \\angle A B F=\\angle C B D, \\angle A C E=\\angle B C D$.\n求证: $A D 、 B E 、 C F$ 三线共点.", + "solution": "证明: 如图(), 记 $\\angle B A F=\\angle C A E=\\alpha$, $\\angle A B F=\\angle C B D=\\beta, \\angle B C D=\\angle A C E=\\gamma$.\n关于 $\\triangle A B C$ 分别与点 $D 、 E 、 F$ 应用角元塞瓦定理有 $\\frac{\\sin \\angle B A D}{\\sin \\angle D A C} \\cdot \\frac{\\sin \\angle A C D}{\\sin \\angle D C B} \\cdot \\frac{\\sin \\angle C B D}{\\sin \\angle D B A}=1$. 则 $\\frac{\\sin \\angle B A D}{\\sin \\angle D A C}=\\frac{\\sin \\gamma \\cdot \\sin (B+\\beta)}{\\sin (C+\\gamma) \\cdot \\sin \\beta}$.\n同理, $\\frac{\\sin \\angle A C F}{\\sin \\angle F C B}=\\frac{\\sin \\beta \\cdot \\sin (A+\\alpha)}{\\sin (B+\\beta) \\cdot \\sin \\alpha}$,\n$$\n\\frac{\\sin \\angle C B E}{\\sin \\angle E B A}=\\frac{\\sin \\alpha \\cdot \\sin (C+\\gamma)}{\\sin (A+\\alpha) \\cdot \\sin \\gamma} \\text {. }\n$$\n以上三式相乘得 $\\frac{\\sin \\angle B A D}{\\sin \\angle D A C} \\cdot \\frac{\\sin \\angle A C F}{\\sin \\angle F C B} \\cdot \\frac{\\sin \\angle C B E}{\\sin \\angle E B A}=1$.\n由角元 Ceva 定理的逆定理知结论成立.", + "remark": "", + "figures": [ + "./images/volume7/figures/fig-c2a11.png" + ] +} \ No newline at end of file diff --git a/processed_dataset/proof/1822.json b/processed_dataset/proof/1822.json new file mode 100644 index 0000000000000000000000000000000000000000..22fb88819001b56eabbacd83ab40aab211cfc9b9 --- /dev/null +++ b/processed_dataset/proof/1822.json @@ -0,0 +1,10 @@ +{ + "source_file": "./raw_volume-zh/volume7/exercise2.tex", + "problem_type": "proof", + "problem": "问题12. 锐角 $\\triangle A B C$ 内接于圆 $O$, 分别过点 $B 、 C$ 作圆 $O$ 的切线, 并分别交过点 $A$ 所作圆 $O$ 的切线于点 $M 、 N, A D$ 为边 $B C$ 上的高.\n求证: $A D$ 平分 $\\angle M D N$.", + "solution": "证明: 如图(), 记 $\\angle M D A=\\alpha, \\angle N D A=\\beta$. 只需证明 $\\alpha=\\beta$.\n因为 $M N 、 M B 、 N C$ 都是圆 $O$ 的切线,所以,\n$$\n\\begin{gathered}\n\\angle M A B=\\angle M B A=\\angle A C B, \\\\\n\\angle N A C=\\angle N C A=\\angle A B C .\n\\end{gathered}\n$$\n对 $\\triangle D A B$ 和点 $M$ 应用角元塞瓦定理有\n$$\n\\begin{aligned}\n1 & =\\frac{\\sin \\angle A D M}{\\sin \\angle M D B} \\cdot \\frac{\\sin \\angle D B M}{\\sin \\angle M B A} \\cdot \\frac{\\sin \\angle B A M}{\\sin \\angle M A D} \\\\\n& =\\frac{\\sin \\alpha}{\\cos \\alpha} \\cdot \\frac{\\sin (B+C)}{\\sin \\angle M A D} \\\\\n& =\\tan \\alpha \\cdot \\frac{\\sin (B+C)}{\\sin \\angle M A D} . \n\\end{aligned} \\label{eq1}\n$$\n同理,对 $\\triangle D A C$ 和点 $N$ 应用角元塞瓦定理又有\n$$\n1=\\tan \\beta \\cdot \\frac{\\sin (B+C)}{\\sin \\angle N A D} . \\label{eq2}\n$$\n比较式\\ref{eq1}、\\ref{eq2}即得 $\\tan \\alpha=\\tan \\beta$.\n因此, $\\alpha=\\beta$, 即 $A D$ 平分 $\\angle M D N$.", + "remark": "", + "figures": [ + "./images/volume7/figures/fig-c2a12.png" + ] +} \ No newline at end of file diff --git a/processed_dataset/proof/1823.json b/processed_dataset/proof/1823.json new file mode 100644 index 0000000000000000000000000000000000000000..12c70e770da9bf3c4e8953b4794c88174aa4c269 --- /dev/null +++ b/processed_dataset/proof/1823.json @@ -0,0 +1,10 @@ +{ + "source_file": "./raw_volume-zh/volume7/exercise2.tex", + "problem_type": "proof", + "problem": "问题13. 在四边形 $A B C D$ 中, 对角线 $A C$ 平分 $\\angle B A D$. 在 $C D$ 上取一点 $E, B E$ 与 $A C$ 相交于 $F$, 延长 $D F$ 交 $B C$ 于 $G$. 求证: $\\angle G A C=\\angle E A C$.", + "solution": "证明: 如图(),考虑直线 $G F D$ 截 $\\triangle B C E$, 由梅氏定理知 $1=\\frac{B G}{G C} \\cdot \\frac{C D}{D E} \\cdot \\frac{E F}{F B}= \\frac{S_{\\triangle A B G}}{S_{\\triangle A C G}} \\cdot \\frac{S_{\\triangle A C D}}{S_{\\triangle A D E}} \\cdot \\frac{S_{\\triangle A E F}}{S_{\\triangle A F B}}$. 设 $\\angle B A C=\\angle D A C=\\theta, \\angle G A C=\\alpha, \\angle E A C=\\beta$, 则\n$$\n\\frac{S_{\\triangle A B G}}{S_{\\triangle A O G}}=\\frac{A B \\cdot \\sin (\\theta-\\alpha)}{A C \\cdot \\sin \\alpha}, \\frac{S_{\\triangle A C D}}{S_{\\triangle A D E}}=\\frac{A C \\cdot \\sin \\theta}{A E \\cdot \\sin (\\theta-\\beta)}, \\frac{S_{\\triangle A E F}}{S_{\\triangle A F B}}=\\frac{A E \\cdot \\sin \\beta}{A B \\cdot \\sin \\theta} .\n$$\n所以, $\\sin \\alpha \\cdot \\sin (\\theta-\\beta)=\\sin \\beta \\cdot \\sin (\\theta-\\alpha), \\sin \\alpha \\cdot \\sin \\theta \\cdot \\cos \\beta-\\sin \\alpha \\cos \\theta \\cdot \\sin \\beta=\\sin \\beta \\cdot \\sin \\theta \\cdot \\cos \\alpha-\\sin \\beta \\cdot \\cos \\theta \\cdot \\sin \\alpha, \\tan \\alpha=\\tan \\beta$, 显然 $\\alpha, \\beta \\in(0$, $\\pi), \\alpha=\\beta$.", + "remark": "", + "figures": [ + "./images/volume7/figures/fig-c2a13.png" + ] +} \ No newline at end of file diff --git a/processed_dataset/proof/1824.json b/processed_dataset/proof/1824.json new file mode 100644 index 0000000000000000000000000000000000000000..cbf3d5e65cb172f6bfa2b0adfe1e1ce98bdc6d27 --- /dev/null +++ b/processed_dataset/proof/1824.json @@ -0,0 +1,10 @@ +{ + "source_file": "./raw_volume-zh/volume7/exercise2.tex", + "problem_type": "proof", + "problem": "问题14. 设三角形 $A B C$ 的两条角平分线为 $B E, C F$, 求证: 若 $B E=C F$, 则 $A B=A C$.", + "solution": "证明: 如图(), 由 Stewart 定理知, $B E^2=B A \\cdot B C-E A \\cdot E C, C F^2= C B \\cdot C A-F B \\cdot F A$.\n设 $A B=c, B C=a, C A=b$, 则 $E A \\cdot E C^{\\prime}=\\frac{b c}{a+c} \\cdot \\frac{a b}{a+c}, F B \\cdot F A==\\frac{a c}{a+b} \\cdot \\frac{b c}{a+b}$, 从而, $a c-\\frac{a b^2 c}{(a+c)^2}=a b-\\frac{a b c^2}{(a+b)^2}$, 化简后有 $c-b=b c \\cdot \\frac{(b-c)\\left(a^2+b^2+c^2+2 a b+2 a c+b c\\right)}{(a+b)^2(a+c)^2}, * *$\n若 $c>b$, 则 $*$ 左边 $>0$, 右边 $<0$;\n若 $c0$;\n故 $c=b$.", + "remark": "", + "figures": [ + "./images/volume7/figures/fig-c2a14.png" + ] +} \ No newline at end of file diff --git a/processed_dataset/proof/1825.json b/processed_dataset/proof/1825.json new file mode 100644 index 0000000000000000000000000000000000000000..656c3c4b221b8ce86a88c10ce9021648a69d26f3 --- /dev/null +++ b/processed_dataset/proof/1825.json @@ -0,0 +1,10 @@ +{ + "source_file": "./raw_volume-zh/volume7/exercise2.tex", + "problem_type": "proof", + "problem": "问题16. 证明 Pascal 定理:\n圆内接六边形 $A B C D E F$ (不要求是凸的)三组对边 $A B$ 和 $D E, C D$ 和 $F A, E F$ 和 $B C$ 的交点 $L, M, N$ 共线.", + "solution": "证明: 如图(), 设三直线 $A B 、 C D 、 E F$ 两两相交成 $\\triangle U V W$, 对 $\\triangle U V W$ 及截线 $B C N 、 D E L 、 F A M$, 由梅涅劳斯定理\n$$\n\\begin{aligned}\n& \\frac{U N}{N V} \\cdot \\frac{V B}{B W} \\cdot \\frac{W C}{C U}=1, \\\\\n& \\frac{U E}{E V} \\cdot \\frac{V L}{L W} \\cdot \\frac{W D}{D U}=1,\n\\end{aligned}\n$$\n$$\n\\frac{U F}{F V} \\cdot \\frac{V A}{A W} \\cdot \\frac{W M}{M U}=1 .\n$$\n三式相乘.\n由相交弦定理与割线定理和乘积等式中的\n$$\n\\begin{aligned}\n& \\frac{V B}{B W} \\cdot \\frac{W C}{C U} \\cdot \\frac{U E}{E V} \\cdot \\frac{W D}{D U} \\cdot \\frac{U F}{F V} \\cdot \\frac{V A}{A W} \\\\\n= & \\frac{V B \\cdot V A}{E V \\cdot F V} \\cdot \\frac{W C \\cdot W D}{B W \\cdot A W} \\cdot \\frac{U E \\cdot U F}{C U \\cdot D U}=1\n\\end{aligned}\n$$\n故有 $\\frac{U N}{N V} \\cdot \\frac{V L}{L W} \\cdot \\frac{W M}{M U}=1$.\n由梅涅劳斯定理逆定理知, $L 、 M 、 N$ 共线.", + "remark": "", + "figures": [ + "./images/volume7/figures/fig-c2a16.png" + ] +} \ No newline at end of file diff --git a/processed_dataset/proof/1826.json b/processed_dataset/proof/1826.json new file mode 100644 index 0000000000000000000000000000000000000000..0b7131baa74433eb60dbd53454951bcc8f34f873 --- /dev/null +++ b/processed_dataset/proof/1826.json @@ -0,0 +1,10 @@ +{ + "source_file": "./raw_volume-zh/volume7/exercise2.tex", + "problem_type": "proof", + "problem": "问题17. 证明 Desargues 定理:\n若 $\\triangle A B C$ 与 $\\triangle A^{\\prime} B^{\\prime} C^{\\prime}$ 的对应顶点连线 $A A^{\\prime}, B B^{\\prime}, C C^{\\prime}$ 相交于一点 $O$, 则对应边 $B C$ 与 $B^{\\prime} C^{\\prime}, C A$ 与 $C^{\\prime} A^{\\prime}, A B$ 与 $A^{\\prime} B^{\\prime}$ 的交点 $D, E, F$ 共线.", + "solution": "证明: 如图(), 对 $\\triangle O B C 、 \\triangle O C A 、 \\triangle O A B$ 及相应的截线 $D B^{\\prime} C^{\\prime} 、 E C^{\\prime} A^{\\prime} 、 F A^{\\prime} B^{\\prime}$, 由梅涅劳斯定理得\n$$\n\\begin{aligned}\n& \\frac{B D}{D C} \\cdot \\frac{C C^{\\prime}}{C^{\\prime} O} \\cdot \\frac{O B^{\\prime}}{B^{\\prime} B}=1, \\\\\n& \\frac{C E}{E A} \\cdot \\frac{A A^{\\prime}}{A^{\\prime} O} \\cdot \\frac{O C^{\\prime}}{C^{\\prime} C}=1, \\\\\n& \\frac{A F}{F B} \\cdot \\frac{B B^{\\prime}}{B^{\\prime} O} \\cdot \\frac{O A^{\\prime}}{A^{\\prime} A}=1,\n\\end{aligned}\n$$\n三式相乘化简得 $\\quad \\frac{B D}{D C} \\cdot \\frac{C E}{E A} \\cdot \\frac{A F}{F B}=1$.\n故对 $\\triangle A B C$ 由梅涅劳斯定理逆定理知, $D 、 E 、 F$ 共线.", + "remark": "", + "figures": [ + "./images/volume7/figures/fig-c2a17.png" + ] +} \ No newline at end of file diff --git a/processed_dataset/proof/1827.json b/processed_dataset/proof/1827.json new file mode 100644 index 0000000000000000000000000000000000000000..6ac75b20b43f25e0440feb4217501da715f20e3b --- /dev/null +++ b/processed_dataset/proof/1827.json @@ -0,0 +1,10 @@ +{ + "source_file": "./raw_volume-zh/volume7/exercise3.tex", + "problem_type": "proof", + "problem": "问题1. 如图(),已知 $\\triangle A B C$ 内一点 $P$, 设 $D 、 E 、 F$ 分别为点 $P$ 在边 $B C 、 C A 、 A B$ 上的投影.\n假设 $A P^2+ P D^2=B P^2+P E^2=C P^2+P F^2$, 且 $\\triangle A B C$ 的三个旁心分别为 $I_A 、 I_B 、 I_C$. 证明: $P$ 是 $\\triangle I_A I_B I_C$ 的外心.", + "solution": "证明: 由已知条件可得, $B F^2-C E^2=\\left(B P^2-P F^2\\right)-\\left(C P^2-P E^2\\right)= \\left(B P^2+P E^2\\right)-\\left(C P^2+P F^2\\right)=0$. 从而, $B F=C E$. 设 $x=B F=C E$. 同理可设 $y=C D=A F, z=A E=B D$. 若 $D 、 E 、 F$ 中有一个点在三边的延长线上, 如点 $D$ 在 $B C$ 的延长线上, 则有 $A B+B C=(x+y)+(z-y)=x+z=A C$, 矛盾: 因此, $D 、 E 、 F$ 三个点都在 $\\triangle A B C$ 的三边上.\n设 $a=B C, b=C A, c= A B, p=\\frac{1}{2}(a+b+c)$, 则 $x=p-a, y=p-b, z=p-c$. 因为 $B D=p-c$, $C D=p-b$, 所以, $D$ 是 $\\triangle A B C$ 的 $\\angle B A C$ 内的旁切圆与边 $B C$ 的切点.\n同理, $E, F$ 分别是 $\\angle A B C 、 \\angle A C B$ 内的旁切圆与边 $C A 、 A B$ 的切点.\n由于 $P D$ 和 $I_A D$ 均垂直于 $B C$, 所以, $P 、 D 、 I_A$ 三点共线.\n同理, $P 、 E 、 I_B$ 和 $P 、 F 、 I_C$ 均三点共线.\n因为 $I_A 、 C 、 I_B$ 三点共线, 且 $\\angle P I_A C=\\angle P I_B C=\\frac{\\angle A C B}{2}$,\n所以, $P I_A=P I_C$. 同理可得, $P I_A=P I_B=P I_C$. 因此, $P$ 是 $\\triangle I_A I_B I_C$ 的外心.", + "remark": "", + "figures": [ + "./images/volume7/figures/fig-c3p1.png" + ] +} \ No newline at end of file diff --git a/processed_dataset/proof/1828.json b/processed_dataset/proof/1828.json new file mode 100644 index 0000000000000000000000000000000000000000..c96e57309189d30518f618c33a7ad6a99b679f11 --- /dev/null +++ b/processed_dataset/proof/1828.json @@ -0,0 +1,8 @@ +{ + "source_file": "./raw_volume-zh/volume7/exercise3.tex", + "problem_type": "proof", + "problem": "问题2. 已知圆内接四边形 $A B C D, K 、 L 、 M 、 N$ 分别是边 $A B 、 B C 、 C D 、 D A$ 的中点.\n证明: $\\triangle A K N$ 、 $\\triangle B K L 、 \\triangle C L M 、 \\triangle D M N$ 的垂心恰好是一个平行四边形的四个顶点.", + "solution": "证明: 由于 $N$ 是 $A D$ 的中点, 有 $O N \\perp A D$, 这里 $O$ 是圆心.\n由于 $K H_1$ 是一条高线, 所以, $K H_1 \\perp A D$. 因此, $K H_1 / / O N$. 同理可证 $O K / / N H_1$. 这表明四边形 $O N H_1 K$ 是平行四边形.\n类似地, 四边形 $O N H_4 M$ 也是平行四边形.\n于是, 四边形 $K H_1 H_4 M$ 也是平行四边形.\n考察 $K M$ 的另一侧, 易看出四边形 $\\mathrm{MKH}_2 \\mathrm{H}_3$ 也是平行四边形.\n利用上述结论, 便可断定四边形 $\\mathrm{H}_1 \\mathrm{H}_2 \\mathrm{H}_3 \\mathrm{H}_4$ 是平行四边形.", + "remark": "", + "figures": [] +} \ No newline at end of file diff --git a/processed_dataset/proof/1829.json b/processed_dataset/proof/1829.json new file mode 100644 index 0000000000000000000000000000000000000000..ab216fe2729cfdf5ff33ced382c292989040cd85 --- /dev/null +++ b/processed_dataset/proof/1829.json @@ -0,0 +1,10 @@ +{ + "source_file": "./raw_volume-zh/volume7/exercise3.tex", + "problem_type": "proof", + "problem": "问题3. 设 $D 、 E 、 F$ 分别为 $\\triangle A B C$ 的三边 $B C 、 C A 、 A B$ 上的点, 且满足 $\\frac{B D}{D C}=\\frac{C E}{E A}=\\frac{A F}{F B}$. 证明: 若 $\\triangle D E F$ 和 $\\triangle A B C$ 的外心重合,则 $\\triangle A B C$ 是正三角形.", + "solution": "证明: 如图(),记 $\\triangle A B C$ 与 $\\triangle D E F$ 的公共外心为 $O$, $B C 、 C A 、 A B$ 与小圆的另一个交点分别为 $D^{\\prime} 、 E^{\\prime} 、 F^{\\prime}$, 作 $O H \\perp B C$ 于点 $H$. 设 $\\frac{B D}{D C}=\\frac{C E}{E A}=\\frac{A F}{F B}=k$. 因为 $B H= H C, D H=H D^{\\prime}$, 所以, $B D^{\\prime}=D C=\\frac{1}{k+1} B C$. 同理, $B F=\\frac{1}{k+1} B A \\Rightarrow B F^{\\prime}=\\frac{k}{k+1} B A$. 由割线定理得: $B D^{\\prime}$ •\n$B D=B F \\cdot B F^{\\prime}$, 即 $\\frac{k}{(k+1)^2} B C^2=\\frac{k}{(k+1)^2} B A^2 \\Rightarrow B C= B A$. 同理, $B C=C A$. 故 $\\triangle A B C$ 为正三角形.", + "remark": "", + "figures": [ + "./images/volume7/figures/fig-c3a3.png" + ] +} \ No newline at end of file diff --git a/processed_dataset/proof/1830.json b/processed_dataset/proof/1830.json new file mode 100644 index 0000000000000000000000000000000000000000..02d22f9cb2360b0f8afb716c55b8bc7cb1c8de36 --- /dev/null +++ b/processed_dataset/proof/1830.json @@ -0,0 +1,10 @@ +{ + "source_file": "./raw_volume-zh/volume7/exercise3.tex", + "problem_type": "proof", + "problem": "问题4. 已知圆心分别为 $A 、 B$ 的两个圆交于点 $C 、 D$. 过点 $A 、 B 、 C$ 的圆与 $\\odot A$ 、 $\\odot B$ 分别交于点 $E 、 F$, 且不包含点 $C$ 的 $\\overparen{E F}$ 在 $\\odot A$ 和 $\\odot B$ 的外部.\n证明: $C D$ 平分这段弧 $\\overparen{E F}$.", + "solution": "证明: 如图(),因为 $\\angle C E D=\\frac{1}{2} \\angle C A D= \\angle C A B, \\angle C A B=\\angle C E B$, 所以, $\\angle C E D= \\angle C E B$, 即 $E 、 D 、 B$ 三点共线.\n因为 $\\overparen{C B}=\\overparen{B F}$, 所以, $\\angle C E B=\\angle B E F$, 即 $D$ 在 $\\angle C E F$ 的角平分线上.\n同理, $D$ 在 $\\angle C F E$ 的角平分线上.\n因此, $D$ 是 $\\triangle C E F$ 的内心.\n从而, $C D$ 是\n$\\angle E C F$ 的角平分线, 即平分 $\\overparen{E F}$.", + "remark": "", + "figures": [ + "./images/volume7/figures/fig-c3a4.png" + ] +} \ No newline at end of file diff --git a/processed_dataset/proof/1831.json b/processed_dataset/proof/1831.json new file mode 100644 index 0000000000000000000000000000000000000000..78b3251e9eab9bb776889092bb3d6e91640da2b5 --- /dev/null +++ b/processed_dataset/proof/1831.json @@ -0,0 +1,10 @@ +{ + "source_file": "./raw_volume-zh/volume7/exercise3.tex", + "problem_type": "proof", + "problem": "问题5. 设 $\\triangle A B C$ 为非直角三角形, 其垂心为 $H, M_1 、 M_2 、 M_3$ 分别为边 $B C$ 、 $C A 、 A B$ 的中点.\n令 $A_1 、 B_1 、 C_1$ 分别为 $H$ 关于 $M_1 、 M_2 、 M_3$ 的对称点, $A_2 、 B_2 、 C_2$ 分别为 $\\triangle B A_1 C 、 \\triangle C B_1 A 、 \\triangle A C_1 B$ 的垂心.\n求证: (1) $\\triangle A B C$ 与 $\\triangle A_2 B_2 C_2$ 的重心重合; (2) 由 $\\triangle A A_1 A_2 、 \\triangle B B_1 B_2 、 \\triangle C C_1 C_2$ 的重心所构成的三角形与 $\\triangle A B C$ 相似.", + "solution": "证明: 如图(), 因为 $\\triangle B H C$ 与 $\\triangle C A_1 B$ 关于 $M_1$ 对称,且 $A$ 为 $\\triangle B H C$ 的垂心, 所以, $A_2$ 为 $A$ 关于 $M_1$ 的对称点.\n(1) 对任意一点 $P$ 有 $\\overrightarrow{P B}+\\overrightarrow{P C}=2 \\overrightarrow{P M_1}=\\overrightarrow{P A}+ \\overrightarrow{P A_2}$. 将类似的关系式相加得 $\\sum \\overrightarrow{P A_2}=\\sum \\overrightarrow{P A}= 3 \\overrightarrow{P G}$. $G$ 为 $\\triangle A B C$ 重心, 由此可推知所证结论成立.\n(2) 设 $G_A 、 G_B 、 G_C$ 分别为 $\\triangle A A_1 A_2 、 \\triangle B B_1 B_2$ 、 $\\triangle C C_1 C_2$ 的重心.\n因此, $\\overrightarrow{H G_A}=\\frac{1}{3}\\left(\\overrightarrow{H A}+\\overrightarrow{H A_1}+\\right.\\left.\\overrightarrow{H A_2}\\right)=\\frac{1}{3}\\left(\\overrightarrow{H A}+2 \\overrightarrow{H M_1}+\\overrightarrow{H A}+2 \\overrightarrow{A M_1}\\right)=\\frac{4}{3} \\overrightarrow{H M_1}$, 即 $\\overrightarrow{G_A G_B}=\\frac{4}{3}\\left(\\overrightarrow{H M_2}-\\right. \\left.\\overrightarrow{H M_1}\\right)=\\frac{4}{3} \\overrightarrow{M_1 M_2}=\\frac{2}{3} \\overrightarrow{B A}$. 由此可知, 由 $\\triangle A A_1 A_2 、 \\triangle B B_1 B_2 、 \\triangle C C_1 C_2$ 的重心所构成的三角形与 $\\triangle A B C$ 相似, 且相似比为 $\\frac{2}{3}$.", + "remark": "", + "figures": [ + "./images/volume7/figures/fig-c3a5.png" + ] +} \ No newline at end of file diff --git a/processed_dataset/proof/1832.json b/processed_dataset/proof/1832.json new file mode 100644 index 0000000000000000000000000000000000000000..25c4c93a04880169e76fb6c548ca3773a14956d7 --- /dev/null +++ b/processed_dataset/proof/1832.json @@ -0,0 +1,10 @@ +{ + "source_file": "./raw_volume-zh/volume7/exercise3.tex", + "problem_type": "proof", + "problem": "问题6. 已知 $U$ 为 $\\triangle A B C$ 的内切圆的圆心, $O_1 、 O_2 、 O_3$ 分别为 $\\triangle B C U 、 \\triangle C A U$ 、 $\\triangle A B U$ 的外接圆的圆心.\n求证: $\\triangle A B C$ 的外接圆圆心与 $\\triangle O_1 O_2 O_3$ 的外接圆圆心重合.", + "solution": "证明: 如图(), 分别过 $\\triangle A B C$ 的顶点 $A 、 B 、 C$ 及其内切圆圆心 $U$ 的直线分别为角 $\\alpha 、 \\beta 、 \\gamma$ 的平分线, 其中 $\\angle C A B=\\alpha, \\angle A B C=\\beta, \\angle B C A=\\gamma$. 设 $O$ 为 $\\triangle A B C$ 的外接圆圆心.\n因为三角形外接圆圆心位于每条边的垂直平分线上, 点 $O$ 和 $O_1$ 位于边 $B C$ 的垂直平分线上, 所以, $O O_1$ 为边 $B C$ 的垂直平分线.\n类似地, $O_3 、 O_1 O_3$ 分别为边 $A B 、 U B$ 的垂直平分线.\n因为 $\\angle U B C$ 与 $\\angle O_3 O_1 O$ 的边互相垂直, 所以, $\\angle U B C=\\angle O_3 O_1 O, \\angle O_3 O_1 O=\\frac{\\beta}{2}$.\n同理, $\\angle O O_3 O_1=\\frac{\\beta}{2}$. 故 $\\angle O_3 O_1 O=\\angle O O_3 O_1$. 因此, $\\triangle O_1 O O_3$ 为等腰三角形, 即 $O O_1=O O_3$. 同理可得 $O O_1=O O_2$. 故 $O O_1=O O_2=O O_3$.\n所以, $O$ 为 $\\triangle O_1 O_2 O_3$ 的外接圆的圆心.", + "remark": "", + "figures": [ + "./images/volume7/figures/fig-c3a6.png" + ] +} \ No newline at end of file diff --git a/processed_dataset/proof/1833.json b/processed_dataset/proof/1833.json new file mode 100644 index 0000000000000000000000000000000000000000..40a90aa545f8ee36c32945434c3132618bb1cc9e --- /dev/null +++ b/processed_dataset/proof/1833.json @@ -0,0 +1,10 @@ +{ + "source_file": "./raw_volume-zh/volume7/exercise3.tex", + "problem_type": "proof", + "problem": "问题7. 已知在不等边 $\\triangle A B C$ 中, 三边 $B C 、 C A 、 A B$ 的长度成等差数列.\n$I 、 O$ 分别是 $\\triangle A B C$ 的内心、外心.\n证明: [1] $I O \\perp B I$; [2] 若 $B I$ 交 $A C$ 于点 $K$, $D 、 E$ 分别是边 $B C 、 A B$ 的中点, 则 $I$ 是 $\\triangle D E K$ 的外心.", + "solution": "证明: [1] 如图(), 作出 $\\triangle A B C$ 的外接圆 $\\odot O$, 记 $B I$ 的延长线与 $\\odot O$ 交于点 $P$, 连结 $A P 、 C P$, 则 $P A=C P= I P$. 对圆内接四边形 $A B C P$ 应用托勒密定理可得: $A C$. $B P=A B \\cdot C P+A P \\cdot B C=(A B+B C) \\cdot I P \\cdots$ (1). 由题意得 $2 A C=A B+B C$. 代入式 (1) 得 $B P=2 I P$, 即 $I$ 是 $B P$ 的中点.\n从而, $O I \\perp B I$.\n[2] 由三角形角平分线的性质知 $\\frac{A K}{K C}=\\frac{A B}{B C}$. 结合\n$A K+K C=A C, 2 A C=A B+B C$, 可得 $A K=\\frac{A B}{2}= A E, C K=\\frac{B C}{2}=C D$. 从而, $\\triangle A I E \\cong \\triangle A I K, \\triangle C I D \\cong \\triangle C I K$. 于是, $I E= I K=I D$. 所以, $I$ 是 $\\triangle D E K$ 的外心.", + "remark": "", + "figures": [ + "./images/volume7/figures/fig-c3a7.png" + ] +} \ No newline at end of file diff --git a/processed_dataset/proof/1834.json b/processed_dataset/proof/1834.json new file mode 100644 index 0000000000000000000000000000000000000000..bf198a5d735b8c647973160e6e3c756605913f20 --- /dev/null +++ b/processed_dataset/proof/1834.json @@ -0,0 +1,10 @@ +{ + "source_file": "./raw_volume-zh/volume7/exercise3.tex", + "problem_type": "proof", + "problem": "问题8. 当 $P$ 为三角形内心 $I$ 时,证明:\n$$\n\\sin A \\cdot \\overrightarrow{I A}+\\sin B \\cdot \\overrightarrow{I B}+\\sin C \\cdot \\overrightarrow{I C}=\\mathbf{0} .\n$$", + "solution": "证明: 设 $\\triangle A B C$ 的内心为 $I$, 如图(), 连结 $A I$ 交 $B C$ 于 $D$, 连结 $C I$ 交 $A B$ 于 $F$.\n由角的平分线定理可知\n$$\n\\frac{B D}{C D}=\\frac{A B}{A C} \\Rightarrow \\frac{B D}{B C}=\\frac{A B}{A B+A C}\n$$\n由正弦定理可知 $\\frac{B D}{B C}=\\frac{\\sin C}{\\sin C+\\sin B}$.\n同理: $\\frac{C D}{B C}=\\frac{A C}{A B+A C}=\\frac{\\sin B}{\\sin C+\\sin B}$.\n在 $\\triangle A B D$ 中由梅涅劳斯定理可得:\n$$\n\\begin{aligned}\n\\frac{D I}{I A} & =\\frac{F B}{A F} \\cdot \\frac{C D}{B C}=\\frac{B C}{A C} \\cdot \\frac{C D}{B C} \\\\\n& =\\frac{\\sin A}{\\sin B} \\cdot \\frac{\\sin B}{\\sin C+\\sin B} \\\\\n& =\\frac{\\sin A}{\\sin C+\\sin B} .\n\\end{aligned}\n$$\n所以 $\\overrightarrow{D I}=\\frac{\\sin A}{\\sin C+\\sin B} \\overrightarrow{I A}$.\n类似有\n$$\n\\overrightarrow{I D}=\\frac{C D}{B C} \\cdot \\overrightarrow{I B}+\\frac{B D}{B C} \\cdot \\overrightarrow{I C}\n$$\n所以 $-\\frac{\\sin A}{\\sin C+\\sin B} \\overrightarrow{I A}=\\frac{\\sin B}{\\sin C+\\sin B} \\overrightarrow{I B}+\\frac{\\sin C}{\\sin C+\\sin B} \\overrightarrow{I C}$,\n故 $\\sin A \\cdot \\overrightarrow{I A}+\\sin B \\cdot \\overrightarrow{I B}+\\sin C \\cdot \\overrightarrow{I C}=\\overrightarrow{0}$.", + "remark": "", + "figures": [ + "./images/volume7/figures/fig-c3a8.png" + ] +} \ No newline at end of file diff --git a/processed_dataset/proof/1835.json b/processed_dataset/proof/1835.json new file mode 100644 index 0000000000000000000000000000000000000000..f60190b7f0379b73ce0f7f9bbf55b6020868bffd --- /dev/null +++ b/processed_dataset/proof/1835.json @@ -0,0 +1,10 @@ +{ + "source_file": "./raw_volume-zh/volume7/exercise3.tex", + "problem_type": "proof", + "problem": "问题9. 在锐角 $\\triangle A B C$ 中, $A D$ 是高, $I 、 O$ 分别是内心、外心, 且 $D 、 I 、 O$ 三点共线.\n求证: $\\triangle A B C$ 的外接圆半径等于与边 $B C$ 相切的旁切圆半径.", + "solution": "解: 如图(),设与边 $B C$ 相切的旁切圆圆心为 $I_A$, 显然, $A 、 I 、 I_A$ 三点共线.\n作 $I E \\perp A B$ 于点 $E, I_A F \\perp A B$ 于点 $F$. 记 $I E=r$, $I_A F=r_a, B C=a, C A=b, A B=c, p$ 为 $\\triangle A B C$ 的半周长.\n设 $A D=h, \\triangle A B C$ 的外接圆半径 $O A=R$. 作 $I M \\perp B C$ 于点 $M, O N \\perp B C$ 于点 $N$.\n利用旁心性质 2 知 $A F=p$, 则\n$$\nA E=p-a .\n$$\n由 $\\triangle A I_A F \\backsim \\triangle A I E$, 得\n$$\n\\frac{r_a}{r}=\\frac{A F}{A E}=\\frac{p}{p-a} . \\label{eq1}\n$$\n由三角形外心性质易得 $\\angle B A D=\\angle O A C$. 但 $A I$ 平分 $\\angle B A C$, 知 $A I$ 平分\n$\\angle D A O$. 所以\n$$\n\\frac{R}{h}=\\frac{A O}{A D}=\\frac{O I}{I D}=\\frac{M N}{D M} . \\label{eq2}\n$$\n注意到\n$$\n\\begin{aligned}\nM N & =B N-B M \\\\\n& =\\frac{1}{2} a-\\frac{1}{2}(c+a-b)=\\frac{1}{2}(b-c), \\\\\nD M & =B M-B D=\\frac{1}{2}(c+a-b)-c \\cos B \\\\\n& =\\frac{1}{2}(c+a-b)-\\frac{1}{2 a}\\left(c^2+a^2-b^2\\right) \\\\\n& =\\frac{1}{2 a}(b-c)(b+c-a),\n\\end{aligned}\n$$\n代入式\\ref{eq2}得\n$$\n\\frac{R}{h}=\\frac{a}{b+c-a} .\n$$\n故\n$$\nR=\\frac{a h}{b+c-a}=\\frac{2 S_{\\triangle A B C}}{2(p-a)}=\\frac{p r}{p-a} .\n$$\n因此, $\\frac{R}{r}=\\frac{p}{p-a}$. 结合式 \\ref{eq1} 得 $r_a=R$.", + "remark": "", + "figures": [ + "./images/volume7/figures/fig-c3a9.png" + ] +} \ No newline at end of file diff --git a/processed_dataset/proof/1836.json b/processed_dataset/proof/1836.json new file mode 100644 index 0000000000000000000000000000000000000000..45190941fa8fb3b3a90bf788585a712f95b72172 --- /dev/null +++ b/processed_dataset/proof/1836.json @@ -0,0 +1,11 @@ +{ + "source_file": "./raw_volume-zh/volume7/exercise3.tex", + "problem_type": "proof", + "problem": "问题10. 如图(), 在 $\\triangle A B C$ 中, $A B=A C$, 一个圆内切于 $\\triangle A B C$ 的外接圆 $\\odot O$ 于 $M$, 并与 $A B 、 A C$ 分别相切于 $P$ 、 $Q$ 两点.\n求证: 线段 $P Q$ 的中点是 $\\triangle A B C$ 内切圆的圆心.", + "solution": "证明: 如图(),因为 $A B=A C$ 且都是 $\\odot O$ 的两条弦,所以 $O$ 点到 $A B 、 A C$ 的距离相等, 则 $O$ 在 $\\angle B A C$ 的平分线上.\n又因为小圆与 $A B, A C$ 都相切,所以小圆的圆心也在 $\\angle B A C$ 的平分线上, 所以小圆的圆心、 $O$ 点及 $A$ 点三点共线且该直线经过两圆切点 $M, A M$ 为图形对称轴.\n设 $A M$ 交 $P Q$ 于 $I$, 由对称性可知, $I$ 为 $P Q$ 中点.\n因为 $A M \\perp P Q, A M \\perp B C$, 所以 $P Q / / B C$. 设 $\\angle A P Q= 2 \\beta$, 则 $\\angle A B C=\\angle A P Q=2 \\beta$. 连结 $M P 、 M Q 、 M B 、 B I$, 则 $\\angle P M Q=\\angle A P Q=2 \\beta$, 由轴对称性知, $\\angle P M I=\\frac{1}{2} \\angle P M Q=\\beta$. 因为 $A M$ 为 $\\odot O$ 直径, 所以 $\\angle P B M= 90^{\\circ}$, 所以 $P 、 B 、 M 、 I$ 四点共圆.\n所以 $\\angle P B I=\\angle P M I=\\beta$, 所以 $B I$ 平分 $\\angle A B C$. 又因为 $A I$ 平分 $\\angle B A C$, 所以 $I$ 为 $\\triangle A B C$ 内心, 所以线段 $P Q$ 的中点是 $\\triangle A B C$ 内切圆的圆心.", + "remark": "", + "figures": [ + "./images/volume7/figures/fig-c3p10.png", + "./images/volume7/figures/fig-c3a10.png" + ] +} \ No newline at end of file diff --git a/processed_dataset/proof/1837.json b/processed_dataset/proof/1837.json new file mode 100644 index 0000000000000000000000000000000000000000..512893463e43b71a143d8c698580b143a53d8a7a --- /dev/null +++ b/processed_dataset/proof/1837.json @@ -0,0 +1,10 @@ +{ + "source_file": "./raw_volume-zh/volume7/exercise3.tex", + "problem_type": "proof", + "problem": "问题11. 在 $\\triangle A B C$ 的边 $A B 、 B C 、 C A$ 上分别取点 $P 、 Q 、 S$. 证明: 以 $\\triangle A P S 、 \\triangle B Q P 、 \\triangle C S Q$ 的外心为顶点的三角形与 $\\triangle A B C$ 相似.", + "solution": "证明: 如图(),设 $O_1 、 O_2 、 O_3$ 是 $\\triangle A P S 、 \\triangle B Q P 、 \\triangle C S Q$ 的外心, 作出六边形 $O_1 P O_2 Q O_3 S$ 后再由外心性质可知 $\\angle P O_1 S=2 \\angle A, \\angle Q O_2 P=2 \\angle B$,\n$$\n\\angle \\mathrm{SO}_3 \\mathrm{Q}=2 \\angle C \\text {. }\n$$\n所以 $\\angle P O_1 S+\\angle Q O_2 P+\\angle S O_3 Q=360^{\\circ}$. 从而又知\n$$\n\\angle O_1 P O_2+\\angle O_2 Q O_3+\\angle O_3 S O_1=360^{\\circ} \\text {. }\n$$\n将 $\\triangle O_2 Q_3$ 绕着 $O_3$ 点旋转到 $\\triangle K S O_3$,\n易判断 $\\triangle K S O_1 \\cong \\triangle O_2 P O_1$, 同时可得 $\\triangle \\mathrm{O}_1 \\mathrm{O}_2 \\mathrm{O}_3 \\cong \\triangle \\mathrm{O}_1 \\mathrm{KO}_3$.\n所以 $\\angle O_2 O_1 O_3=\\angle K O_1 O_3=\\frac{1}{2} \\angle O_2 O_1 K=\\frac{1}{2}\\left(\\angle O_2 O_1 S+\\angle S O_1 K\\right)= \\frac{1}{2}\\left(\\angle O_2 O_1 S+\\angle P O_1 O_2\\right)=\\frac{1}{2} \\angle P O_1 S=\\angle A$. 同理有 $\\angle O_1 O_2 O_3=\\angle B$. 故 $\\triangle O_1 O_2 O_3 \\backsim \\triangle A B C$.", + "remark": "", + "figures": [ + "./images/volume7/figures/fig-c3a11.png" + ] +} \ No newline at end of file diff --git a/processed_dataset/proof/1838.json b/processed_dataset/proof/1838.json new file mode 100644 index 0000000000000000000000000000000000000000..2fa5252b41d2062f7add827d4962cada4a545198 --- /dev/null +++ b/processed_dataset/proof/1838.json @@ -0,0 +1,11 @@ +{ + "source_file": "./raw_volume-zh/volume7/exercise3.tex", + "problem_type": "proof", + "problem": "问题12. 如图(), $ \\triangle A B C$ 的外心为 $O, A B=A C, D$ 是 $A B$ 中点, $E$ 是 $\\triangle A C D$ 的重心.\n证明: $O E \\perp C D$.", + "solution": "证明: 如图(), 设 $A M$ 为高亦为中线, 取 $A C$ 中点 $F, E$ 必在 $D F$ 上且 $D E: E F=2: 1$. 设 $C D$ 交 $A M$ 于 $G, G$ 必为 $\\triangle A B C$ 重心.\n连结 $G E, M F, M F$ 交 $D C$ 于 $K$.\n易证: $D G: G K=\\frac{1}{3} D C:\\left(\\frac{1}{2}-\\frac{1}{3}\\right) D C=2: 1$.\n所以 $D G: G K=D E: E F \\Rightarrow G E / / M F$.\n因为 $O D \\perp A B, M F / / A B$, 则 $O D \\perp M F \\Rightarrow O D \\perp G E$.\n但 $O G \\perp D E \\Rightarrow G$ 又是 $\\triangle O D E$ 之垂心.\n从而 $O E \\perp C D$.", + "remark": "", + "figures": [ + "./images/volume7/figures/fig-c3p12.png", + "./images/volume7/figures/fig-c3a12.png" + ] +} \ No newline at end of file diff --git a/processed_dataset/proof/1839.json b/processed_dataset/proof/1839.json new file mode 100644 index 0000000000000000000000000000000000000000..9a4b7d7044534bab34f1f50c9d5a8f8755db1c8e --- /dev/null +++ b/processed_dataset/proof/1839.json @@ -0,0 +1,10 @@ +{ + "source_file": "./raw_volume-zh/volume7/exercise3.tex", + "problem_type": "proof", + "problem": "问题13. $ \\odot O_1$ 交 $\\odot O_2$ 于点 $P 、 Q, \\angle O_1 P O_2<90^{\\circ}$, 过 $O_1 、 O_2$ 、 $P$ 三点的圆分别交 $\\odot O_1 、 \\odot O_2$ 于点 $A 、 B$. 证明: $Q$ 是$\\triangle A B P$ 的旁心.", + "solution": "如图(), 连结 $A Q 、 A O_2$, 由于 $\\pi-\\angle P A Q=\\frac{1}{2} \\angle P O_1 Q=\\angle P O_1 O_2= \\angle P A O_2$, 于是 $A 、 O_2 、 Q$ 三点共线, 同理, $B 、 O_1 、 Q$ 三点共线.\n连 $O_1 Q, \\angle Q A B= \\angle Q O_1 O_2=\\frac{1}{2} \\angle Q O_1 P=\\frac{1}{2} \\angle B O_1 P=\\frac{1}{2} \\cdot\\left(\\pi^{-} \\angle B A P\\right)$. 所以 $A Q$ 外角平分 $\\angle P A B$, 同理可证, $B Q$ 外角平分 $\\angle P B A$, 于是 $Q$ 为 $\\triangle P B A$ 在 $\\angle P$ 内的旁心.", + "remark": "", + "figures": [ + "./images/volume7/figures/fig-c3a13.png" + ] +} \ No newline at end of file diff --git a/processed_dataset/proof/1840.json b/processed_dataset/proof/1840.json new file mode 100644 index 0000000000000000000000000000000000000000..dd6d602013e0f37cbae2286d535d7fcdee9d5542 --- /dev/null +++ b/processed_dataset/proof/1840.json @@ -0,0 +1,10 @@ +{ + "source_file": "./raw_volume-zh/volume7/exercise3.tex", + "problem_type": "proof", + "problem": "问题14. 已知 $A B 、 A C$ 切 $\\odot O$ 于点 $B 、 C, O A$ 交 $B C$ 于点 $M$, 过 $M$ 作 $\\odot O$ 的另一弦 $E F$. 求证: $\\triangle A B C 、 \\triangle A E F$ 存在一个公共的旁心.", + "solution": "证明: 如图(),连结 $O B 、 O C 、 O E 、 O F 、 A E 、 A F$, 则 $\\angle O B A+\\angle O C A=90^{\\circ}+ 90^{\\circ}=180^{\\circ}$, 故 $A 、 B 、 O 、 C$ 共圆.\n$M B \\cdot M C=M O \\cdot M A$, 结合 $M B \\cdot M C= M E \\cdot M F$ 知 $M E \\cdot M F=M O \\cdot M A$, 于是 $A, E, O, F$ 共圆.\n令 $A O$ 直线与 $\\odot O$ 相交于 $X, Y$, 且 $X$ 在 $\\triangle A B C$ 内部, 那么 $\\angle X F M=\\angle X F E= \\frac{1}{2} \\angle X O E=\\frac{1}{2} \\angle A O E=\\frac{1}{2} \\angle A F E=\\frac{1}{2} \\angle A F M$, 于是 $X F$ 平分 $\\angle A F M$, 同理可证, $X E$ 平分 $\\angle A E M$, 故 $X$ 为 $\\triangle A E F$ 内心.\n又 $\\angle X F Y=90^{\\circ}, A 、 X$ 、 $Y$ 共线, 所以 $Y$ 为 $\\triangle A E F$ 的旁心.\n由 $E$ 点的任意性, 将 $E$ 移至 $B, F$ 移至 $C$ 有相同结论.\n$Y$ 为 $\\triangle A B C$ 的旁心.\n所以 $\\triangle A B C, \\triangle A E F$ 有一个公共的 $\\angle A$ 内的旁心.", + "remark": "", + "figures": [ + "./images/volume7/figures/fig-c3a14.png" + ] +} \ No newline at end of file diff --git a/processed_dataset/proof/1841.json b/processed_dataset/proof/1841.json new file mode 100644 index 0000000000000000000000000000000000000000..7d63a8ad83c9893b9c8c58f6eec33dfdb900ad12 --- /dev/null +++ b/processed_dataset/proof/1841.json @@ -0,0 +1,10 @@ +{ + "source_file": "./raw_volume-zh/volume7/exercise3.tex", + "problem_type": "proof", + "problem": "问题15. 已知 $\\triangle A B C$, 点 $D$ 在边 $B C$ 上, $O 、 O_1 、 O_2$ 分别是 $\\triangle A B C 、 \\triangle A B D$ 、 $\\triangle A C D$ 在 $\\angle A$ 内的旁心, $O E \\perp B C$ 于点 $E$. 求证: $E O_1 \\perp E O_2$.", + "solution": "证明: 如图(),设 $O_1 、 O_2$ 在边 $B C$ 上的垂足分别为 $M 、 N$, 则 $M E=B E-B M=\\frac{B C+C A-A B}{2}-\\frac{B D+D A-A B}{2}= \\frac{D C+C A-D A}{2}=D N$ 所以 $M D=E N$, 连结 $O_1 D, O_2 D$, $O_1 E, O_2 E, \\angle O_1 D M=\\frac{1}{2} \\angle A D C=90^{\\circ}-\\frac{1}{2} \\angle A D B= 90^{\\circ}-\\angle O_2 D N=\\angle D O_2 N$, 所以 $\\triangle M D O_1 \\backsim \\triangle N O_2 D$, 即 $M D \\cdot N D=M O_1 \\cdot N O_2$, 于是 $M E \\cdot E N=M D \\cdot N D= M O_1 \\cdot N O_2$. 即 $\\frac{M E}{M O_1}=\\frac{N O_2}{N E}$, 所以 $\\triangle M O_1 D \\backsim \\triangle N D O_2$, 从而有 $\\angle O_1 E O_2=180^{\\circ}-\\angle M E O_1-\\angle N E O_2=180^{\\circ}-\\angle M E O_1- \\angle M O_1 E=90^{\\circ}$. 即 $E O_1 \\perp E O_2$.", + "remark": "", + "figures": [ + "./images/volume7/figures/fig-c3a15.png" + ] +} \ No newline at end of file diff --git a/processed_dataset/proof/1842.json b/processed_dataset/proof/1842.json new file mode 100644 index 0000000000000000000000000000000000000000..cb7faa9d5b238742813a558f8a601746ed5bd15a --- /dev/null +++ b/processed_dataset/proof/1842.json @@ -0,0 +1,10 @@ +{ + "source_file": "./raw_volume-zh/volume7/exercise3.tex", + "problem_type": "proof", + "problem": "问题16. $A D$ 是直角三角形 $A B C$ 斜边 $B C$ 上的高, $(A B),因为 $\\angle B A C=90^{\\circ}$, 又 $O$ 为 $\\triangle A E F$ 外心, 所以 $O \\in E F$. 又 $O I_1==O I_2$, $2 \\angle I_1 A I_2=\\angle I_1 O I_2=90^{\\circ}$. 由此可得 $\\angle O I_2 I_1= \\angle O I_1 I_2=\\angle I_1 D O=\\angle I_2 D O=45^{\\circ}$. 所以, $O \\in A D$, 所以 $\\angle A I_1 O=\\angle I_1 A O=\\angle I_1 A E$, $\\angle I_1 O D=\\angle I_1 O M=\\angle E A D$. 故 $\\angle M O D$ 平分线为 $O I_1$, 又 $I_1 D$ 平分 $\\angle A D B$, 即 $I_1$ 为 $\\triangle O D M$ 内心.\n又 $\\angle I_1 O I_2=90^{\\circ}, O I_2$ 平分 $\\angle M O D$, 所以 $O I_2$ 平分 $\\angle F O D$, 而 $I_2 D$ 为 $\\angle A D C$ 平分线.\n所以 $I_2$ 为 $\\triangle O D M$ 旁心.", + "remark": "", + "figures": [ + "./images/volume7/figures/fig-c3a16.png" + ] +} \ No newline at end of file diff --git a/processed_dataset/proof/1843.json b/processed_dataset/proof/1843.json new file mode 100644 index 0000000000000000000000000000000000000000..cb9c7b60b561544790fd3090dabe5fba6d3e7f29 --- /dev/null +++ b/processed_dataset/proof/1843.json @@ -0,0 +1,11 @@ +{ + "source_file": "./raw_volume-zh/volume7/exercise4.tex", + "problem_type": "proof", + "problem": "问题1. 如图(), 设 $B$ 是圆 $S_1$ 上的点, 过 $B$ 作圆 $S_1$ 的切线, $A$ 为该切线上异于 $B$ 的点, 又 $C$ 不是圆 $S_1$ 上的点, 且线段 $A C$ 交圆 $S_1$ 于两个不同的点.\n圆 $S_2$ 与 $A C$ 相切于点 $C$, 与圆 $S_1$ 相切于点 $D$, 且 $D$ 与 $B$ 在直线 $A C$ 的两侧.\n证明: $\\triangle B C D$ 的外心在 $\\triangle A B C$ 的外接圆上.", + "solution": "证明: 如图(), 设 $E 、 F$ 分别是 $B D 、 C D$ 的中点, $K$ 是 $\\triangle B C D$ 的外心, $T D T^{\\prime}$ 是圆 $S_1$ 与圆 $S_2$ 的内公切线, 则 $E K$ 是 $B D$ 的中垂线.\n因 $\\angle T D B=\\angle A B D, \\angle T^{\\prime} D C=\\angle D C A$, 则 $\\angle B D C=180^{\\circ}-\\angle T D B+\\angle T^{\\prime} D C=180^{\\circ}- \\angle A B D+\\angle D C A=180^{\\circ}-(\\angle A B C-\\angle D B C)+ (\\angle D C B-\\angle A C B)=180^{\\circ}-\\angle A B C-\\angle A C B+ \\angle D B C+\\angle D C B=\\angle B A C+180^{\\circ}-\\angle B D C$.于是, $2 \\angle B D C=180^{\\circ}+\\angle B A C$. 故 $\\angle B K C= \\angle B K D+\\angle D K C=2(\\angle E K D+\\angle D K F)=2 \\angle E K F=2\\left(180^{\\circ}-\\angle B D C\\right)=180^{\\circ}-\\angle B A C$. 因此, $K$ 在 $\\triangle A B C$ 的外接圆上.", + "remark": "", + "figures": [ + "./images/volume7/figures/fig-c4p1.png", + "./images/volume7/figures/fig-c4a1.png" + ] +} \ No newline at end of file diff --git a/processed_dataset/proof/1844.json b/processed_dataset/proof/1844.json new file mode 100644 index 0000000000000000000000000000000000000000..2ce16b6074463c90f4957bde9ce9be479fd0637b --- /dev/null +++ b/processed_dataset/proof/1844.json @@ -0,0 +1,10 @@ +{ + "source_file": "./raw_volume-zh/volume7/exercise4.tex", + "problem_type": "proof", + "problem": "问题2. 在 $\\triangle A B C$ 的外接圆上, $\\overparen{B C} 、 \\overparen{C A} 、 \\overparen{A B}$ 的中点分别为 $D 、 E 、 F$, 其中 $A \\notin \\overparen{B C}, B \\notin \\overparen{C A}, C \\notin \\overparen{A B} . D E$ 分别交 $C B 、 C A$ 于点 $G 、 H, D F$ 分别交 $B C 、 B A$ 于点 $I 、 J, G H$ 和 $I J$ 的中点分别为 $M 、 N$.\n(1) 用 $\\triangle A B C$ 的内角表示 $\\triangle D M N$ 的三个内角;\n(2) 若 $O$ 为 $\\triangle D M N$ 的外心, $P$ 是 $A D$ 与 $E F$ 的交点, 证明: $O 、 M 、 P 、 N$ 四点共圆.", + "solution": "(1) 证明: 如图(), 图为 $\\angle B J I=\\frac{\\overparen{A F}^{\\circ}}{2}+\\frac{\\overparen{B D}^{\\circ}}{2}=\\frac{\\overparen{B F}^{\\circ}}{2}+ \\frac{\\widehat{C D}^{\\circ}}{2}=\\angle B I J$, 所以 $B I=B J$. 从而 $B N$ 是 $\\angle A B C$ 的平分线,则 $B 、 N 、 E$ 三点共线, $B N \\perp I J$. 同理, $C M \\perp G H$. 因此, $D 、 N 、 Q 、 M$ 四点共圆 (显然 $C M 、 B N$ 交于 $\\triangle A B C$ 的内心 $Q)$. 进而, 有 $\\angle D N M=\\angle D Q M=\\frac{\\overparen{A F}^{\\circ}+\\overparen{C D}^{\\circ}}{2}=\\frac{\\angle A+\\angle C}{2}, \\angle D M N=\\frac{\\angle A+\\angle B}{2}, \\angle M D N=\\frac{\\angle B+\\angle C}{2}$.\n(2) 显然, 直线 $A D$ 过 $Q$ 点, 且 $\\angle N P F=\\frac{\\angle B+\\angle C}{2}, \\angle E P M= \\frac{\\angle B+\\angle C}{2}$. 从而, $\\angle N O M=2 \\angle M D N=\\angle B+\\angle C, \\angle N P M=\\pi- \\angle N P F-\\angle E P M=\\pi-\\angle B-\\angle C$.\n所以, $N 、 O 、 M 、 P$ 四点共圆.", + "remark": "", + "figures": [ + "./images/volume7/figures/fig-c4a2.png" + ] +} \ No newline at end of file diff --git a/processed_dataset/proof/1845.json b/processed_dataset/proof/1845.json new file mode 100644 index 0000000000000000000000000000000000000000..a271bfb483506ac9677da462a197087790cbe9d3 --- /dev/null +++ b/processed_dataset/proof/1845.json @@ -0,0 +1,11 @@ +{ + "source_file": "./raw_volume-zh/volume7/exercise4.tex", + "problem_type": "proof", + "problem": "问题3. 如图(), 3 个圆有公共弦 $A B$. 任一条过点 $A$ 的直线 $l$ 与 3 个圆的交点依次为 $X 、 Y 、 Z$, 其中 $X \\neq B$. 证明: $\\frac{X Y}{Y} \\frac{Y}{}$ 为定值.", + "solution": "证明: 如图(), 因为 3 个圆有公共弦 $A B$, 故圆心 $O_1 、 O_2 、 O_3$ 共线.\n过 $O_1 、 O_2 、 O_3$ 分别作 $l$ 的垂线交 $l$ 于点 $H_1 、 H_2 、 H_3$. 易知 $A X=2 A H_1, A Y= 2 A H_2, A Z=2 A H_3$.\n故 $\\frac{X Y}{Y Z}=\\frac{H_1 H_2}{H_2 H_3}=\\frac{O_1 O_2}{O_2 O_3}$ 为定值.", + "remark": "", + "figures": [ + "./images/volume7/figures/fig-c4p3.png", + "./images/volume7/figures/fig-c4a3.png" + ] +} \ No newline at end of file diff --git a/processed_dataset/proof/1846.json b/processed_dataset/proof/1846.json new file mode 100644 index 0000000000000000000000000000000000000000..7252b599700f9ccc6fcf2e360280a89ce8b30e26 --- /dev/null +++ b/processed_dataset/proof/1846.json @@ -0,0 +1,10 @@ +{ + "source_file": "./raw_volume-zh/volume7/exercise4.tex", + "problem_type": "proof", + "problem": "问题4. 等腰 $\\triangle A B C$ 中, $A B=A C, M$ 为边 $B C$ 的中点, $X$ 是 $\\triangle A B M$ 外接圆的劣弧 $\\overparen{M A}$ 上的一个动点, $T$ 是 $\\angle B M A$ 内的一点, 且满足 $\\angle T M X=90^{\\circ}, T X=B X$. 证明: $\\angle M T B-\\angle C T M$ 的值不依赖于点 $X$.", + "solution": "证明: 如图(), 设 $N$ 是线段 $B T$ 的中点.\n则直线 $X N$ 是等腰 $\\triangle B X T$ 的对称轴.\n于是, $\\angle T N X=90^{\\circ}$, $\\angle B X N=\\angle N X T$. 又因为 $M N$ 是 $\\triangle B C T$ 中平行于 $C T$ 的中位线, 所以 $\\angle C T M=\\angle N M T$. 由于 $\\angle T N X= \\angle T M X=90^{\\circ}$, 则点 $M 、 N$ 在以 $X T$ 为直径的圆上.\n从而\n$\\angle M T B=\\angle M T N=\\angle M X N, \\angle C T M=\\angle N M T=\\angle N X T=\\angle B X N$. 故 $\\angle M T B-\\angle C T M=\\angle M X N-\\angle B X N=\\angle M X B=\\angle M A B$. 不依赖于点 $X$.", + "remark": "", + "figures": [ + "./images/volume7/figures/fig-c4a4.png" + ] +} \ No newline at end of file diff --git a/processed_dataset/proof/1847.json b/processed_dataset/proof/1847.json new file mode 100644 index 0000000000000000000000000000000000000000..6871e60a97e1dfa39136bf5d88db0cb335490a60 --- /dev/null +++ b/processed_dataset/proof/1847.json @@ -0,0 +1,10 @@ +{ + "source_file": "./raw_volume-zh/volume7/exercise4.tex", + "problem_type": "proof", + "problem": "问题5. 已知一个圆与 $\\triangle A B C$ 的边 $A B 、 B C$ 相切, 也和 $\\triangle A B C$ 的外接圆相切于点 $T$. 若 $I$ 是 $\\triangle A B C$ 的内心, 证明: $\\angle A T I=\\angle C T I$.", + "solution": "证明: 如图(), 设小圆圆心为 $O^{\\prime}$, 半径为 $r$, 大圆圆心为 $O$, 半径为 $R$, 且 $\\odot O^{\\prime}$ 与 $A B 、 B C$ 分别切于 $D 、 E$ 两点.\n连结 $D E 、 B O^{\\prime}$ 交于点 $I^{\\prime}$.\n下面证明 $I^{\\prime}=I$. 延长 $B O^{\\prime}$ 交 $\\odot O$ 于点 $F$, 易知 $B F$ 平分 $\\angle A B C$ 及 $\\overparen{A C}$. 则 $O^{\\prime}$ 关于 $\\odot O$ 的幂为 $r(2 R-r)= 2 R r-r^2=B O^{\\prime} \\cdot O^{\\prime} F=O^{\\prime} F \\cdot \\frac{r}{\\sin \\frac{B}{2}}$. 故 $O F=(2 R-r) \\sin \\frac{B}{2}$. 于是, 有 $F I^{\\prime}=F O^{\\prime}+O^{\\prime} I^{\\prime \\prime}=(2 R-r) \\sin \\frac{B}{2}+ r \\sin \\frac{B}{2}=2 R \\sin \\frac{B}{2}=A F$. 从而 $I^{\\prime}=I$.\n连结 $B T 、 O^{\\prime} T$ 有, $O^{\\prime} I^{\\prime} \\cdot O^{\\prime} B=O^{\\prime} E^2=r^2=O^{\\prime} T^2$. 从而, $\\triangle O^{\\prime} I^{\\prime} T \\backsim \\triangle O^{\\prime} T B$. 故 $\\angle O^{\\prime} T I^{\\prime}=\\angle O^{\\prime} B T$. 过点 $T$ 作两圆的公切线 $T G$, 于是有, $\\angle C T I+\\angle O^{\\prime} B C=\\angle C T I+\\angle O^{\\prime} B T+\\angle T B C=\\angle C T I+\\angle O^{\\prime} T I+ \\angle T B C=\\angle C T O^{\\prime}+\\angle T B C=\\angle C T O^{\\prime}+\\angle C T G=\\angle O^{\\prime} T G=90^{\\circ}$. 则 $\\angle C T I=90^{\\circ}-\\angle O^{\\prime} B C=90^{\\circ}-\\frac{1}{2} B$. 同理, $\\angle A T I=90^{\\circ}-\\frac{1}{2} B=\\angle C T I$.\n故命题得证.", + "remark": "", + "figures": [ + "./images/volume7/figures/fig-c4a5.png" + ] +} \ No newline at end of file diff --git a/processed_dataset/proof/1848.json b/processed_dataset/proof/1848.json new file mode 100644 index 0000000000000000000000000000000000000000..afd36a8f9ea43cdfb9a8ae461510b8904a73512c --- /dev/null +++ b/processed_dataset/proof/1848.json @@ -0,0 +1,10 @@ +{ + "source_file": "./raw_volume-zh/volume7/exercise4.tex", + "problem_type": "proof", + "problem": "问题6. 设 $\\triangle A B C$ 的外接圆为 $\\Gamma$, 圆心为 $O$ 的圆与线段 $B C$ 切于点 $P$, 与不含点 $A$ 的弧 $\\overparen{B C}$ 切于点 $Q$. 若 $\\angle B A O=\\angle C A O$, 证明: $\\angle P A O=\\angle Q A O$.", + "solution": "证明: 若 $A B=A C$, 则点 $P 、 Q$ 均在 $\\angle B A C$ 的角平分线上, $\\angle P A O=\\angle Q A O=O$. 若 $A B \\neq A C$, 如图(), 设 $\\triangle A B C$ 的外接圆的圆心为 $O^{\\prime}, B C$ 的中垂线与 $\\odot O^{\\prime}$ 交于点 $P^{\\prime} 、 M$, 其中, $P^{\\prime}, A$ 在 $B C$ 的同侧.\n则 $O^{\\prime} 、 O 、 Q$ 三点共线.\n由 $\\angle B A O=\\angle C A O$, 则 $A 、 O 、 M$ 三点共线.\n设 $P^{\\prime} Q$ 与 $\\odot O$ 交于点 $R$. 由 $\\angle M P^{\\prime} R=\\angle O^{\\prime} P^{\\prime} Q= \\angle O^{\\prime} Q R=\\angle O R Q$, 得 $O R / / M P^{\\prime}$. 因为 $M P^{\\prime} \\perp B C$, 所以, $O R \\perp B C$. 从而, $R$ 为 $\\odot O$ 与 $B C$ 的切点, 即 $R=P$. 这也就意味着 $P^{\\prime} 、 P 、 Q$ 三点共线, 且有 $\\angle Q P^{\\prime} M=\\angle P Q O=\\angle Q P O$. 又 $\\angle Q A O=\\angle Q A M=\\angle Q P^{\\prime} M$, 则 $\\angle Q A O=\\angle Q P O$. 从而, $A 、 P 、 O 、 Q$ 四点共圆.\n故 $\\angle P A O=\\angle P Q O=\\angle Q P O=\\angle Q A O$.", + "remark": "", + "figures": [ + "./images/volume7/figures/fig-c4a6.png" + ] +} \ No newline at end of file diff --git a/processed_dataset/proof/1849.json b/processed_dataset/proof/1849.json new file mode 100644 index 0000000000000000000000000000000000000000..4fffce2ef98beab01b206dc7b373206addb497ed --- /dev/null +++ b/processed_dataset/proof/1849.json @@ -0,0 +1,10 @@ +{ + "source_file": "./raw_volume-zh/volume7/exercise4.tex", + "problem_type": "proof", + "problem": "问题7. 设 $P$ 为 $\\triangle A B C$ 内一点, 且满足 $\\angle B P C=90^{\\circ}, \\angle B A P=\\angle B C P, M 、 N$ 分别是边 $A C 、 B C$ 的中点.\n若 $B P=2 P M$, 证明: $A 、 P 、 N$ 三点共线.", + "solution": "证明: 如图(), 作 $\\triangle P A B$ 的外接圆 $\\odot O$, 延长 $C P$ 交 $\\odot O$ 于点 $C^{\\prime}$, 连结\n$B C^{\\prime} 、 A C^{\\prime}$. 因为 $\\angle B C^{\\prime} P=\\angle B A P=\\angle B C P$, $\\angle B P C=90^{\\circ}$, 所以, $P C^{\\prime}=P C, B C^{\\prime}=B C$. 又 $M 、 P$ 分别为 $A C 、 C C^{\\prime}$ 的中点, 则 $A C^{\\prime} / / P M \\Rightarrow A C^{\\prime}=2 P M=P B$. 由 $A 、 C^{\\prime} 、 B 、 P$ 四点共圆, 故 $A P / / B C^{\\prime}$. 因为 $P$ 为 $C C^{\\prime}$ 的中点, 所以, $P N / / B C^{\\prime}$. 故 $A 、 P 、 N$ 三点共线.", + "remark": "", + "figures": [ + "./images/volume7/figures/fig-c4a7.png" + ] +} \ No newline at end of file diff --git a/processed_dataset/proof/1850.json b/processed_dataset/proof/1850.json new file mode 100644 index 0000000000000000000000000000000000000000..f5cfddb8b746c4889f2eaf0d49991a9160105675 --- /dev/null +++ b/processed_dataset/proof/1850.json @@ -0,0 +1,10 @@ +{ + "source_file": "./raw_volume-zh/volume7/exercise4.tex", + "problem_type": "proof", + "problem": "问题8. 设圆 $\\Gamma$ 和直线 $l$ 不相交, $A B$ 是圆 $\\Gamma$ 的直径, 且垂直于直线 $l$, 点 $B$ 比点 $A$ 更靠近直线 $l$. 在圆 $\\Gamma$ 上任意取一点 $C(C \\neq A 、 B)$, 直线 $A C$ 交直线 $l$ 于点 $D$, 直线 $D E$ 与圆 $\\Gamma$ 切于点 $E$, 且点 $B 、 E$ 在 $A C$ 的同一侧.\n设 $B E$ 交直线 $l$ 于点 $F, A F$ 交圆 $\\Gamma$ 于点 $G(G \\neq A)$. 证明: 点 $G$ 关于 $A B$ 的对称点在直线 $C F$ 上.", + "solution": "证明: 如图(), 设 $C F$ 交圆 $\\Gamma$ 于点 $H$. 因为直径 $A B \\perp l$, 所以,问题等价于证明 $G H / / l$.\n设 $A B \\perp l$, 垂足为点 $X$, 则 $\\angle A X F=\\angle A E F= 90^{\\circ}$. 所以, $A 、 F 、 X 、 E$ 四点共圆.\n于是, $\\angle E F D= \\angle E A B=\\angle F E D$. 从而, $D F=D E$. 又因为 $D F^2= D E^2=D C \\cdot D A$, 所以, $\\triangle D C F \\backsim \\triangle D F A$. 于是, $\\angle A F D=\\angle F C D=\\angle A C H=\\angle A G H$. 故 $G H / / l$.", + "remark": "", + "figures": [ + "./images/volume7/figures/fig-c4a8.png" + ] +} \ No newline at end of file diff --git a/processed_dataset/proof/1851.json b/processed_dataset/proof/1851.json new file mode 100644 index 0000000000000000000000000000000000000000..0871e154156986fea2f7eddfd88f26eb478316b2 --- /dev/null +++ b/processed_dataset/proof/1851.json @@ -0,0 +1,10 @@ +{ + "source_file": "./raw_volume-zh/volume7/exercise4.tex", + "problem_type": "proof", + "problem": "问题9. 在 $\\triangle A B C$ 中, $P 、 Q$ 分别是边 $A B 、 A C$ 上的点, 且使得 $\\angle A P C= \\angle A Q B=45^{\\circ}$. 过点 $P$ 作边 $A B$ 的垂线与 $B Q$ 交于点 $S$, 过点 $Q$ 作边 $A C$ 的垂线与 $C P$ 交于点 $R$. 设 $D$ 是 $B C$ 上的点, 且使得 $A D \\perp B C$. 证明: $P S$ 、 $A D 、 Q R$ 三线共点,且 $S R / / B C$.", + "solution": "证明: 如图(), 设 $Q R 、 A D$ 的延长线交于 $E$. 下面证明直线 $P E$ 和 $P S$ 重合.\n注意到 $\\angle A B Q= 135^{\\circ}-\\angle B A C=\\angle A C P$, 则 $B 、 P 、 Q 、 C$ 四点共圆.\n于是, $\\angle A P Q=\\angle A C B$. 又 $\\angle A E Q=90^{\\circ}- \\angle E A C=\\angle A C B$, 从而, $A 、 P 、 E 、 Q$ 四点共圆.\n于是, $\\angle A P E=180^{\\circ}-\\angle A Q E=90^{\\circ}$. \n所以, $P E \\perp A B$. 因此, $P E 、 P S$ 重合且 $P S 、 A D 、 Q R$ 三线共点.\n由 $\\angle B Q E=90^{\\circ}- 45^{\\circ}=\\angle C P E$, 知 $P 、 Q 、 R 、 S$ 四点共圆.\n从而 $\\angle Q P R=\\angle Q S R$. 因为 $B 、 P 、 Q 、 C$ 四点共圆, 所以, $\\angle Q P R=\\angle Q B C$.\n因此, $\\angle Q B C=\\angle Q S R$, 即 $S R / / B C$.", + "remark": "", + "figures": [ + "./images/volume7/figures/fig-c4a9.png" + ] +} \ No newline at end of file diff --git a/processed_dataset/proof/1852.json b/processed_dataset/proof/1852.json new file mode 100644 index 0000000000000000000000000000000000000000..c2a12edbe4192ad6965d5fec921184cbc5ca52f9 --- /dev/null +++ b/processed_dataset/proof/1852.json @@ -0,0 +1,10 @@ +{ + "source_file": "./raw_volume-zh/volume7/exercise4.tex", + "problem_type": "proof", + "problem": "问题10. 已知锐角 $\\triangle A B C$, 以 $A C$ 为直径的圆为圆 $\\Gamma_1$, 以 $B C$ 为直径的圆为圆 $\\Gamma_2, A C$ 与圆 $\\Gamma_2$ 相交于点 $E, B C$ 与圆 $\\Gamma_1$ 相交于点 $F$, 直线 $B E$ 和圆 $\\Gamma_1$ 相交于点 $L 、 N$, 其中点 $L$ 在线段 $B E$ 上, 直线 $A F$ 和圆 $\\Gamma_2$ 相交于点 $K 、 M$, 其中点 $K$ 在线段 $A F$ 上.\n证明: 四边形 $K L M N$ 是圆内接四边形.", + "solution": "证明: 如图(), 在圆 $\\Gamma_2$ 中, $B C$ 是直径, 点 $E$ 是 $A C$ 和圆 $\\Gamma_2$ 的交点, 则 $\\angle B E C=90^{\\circ}$. 同理, 由 $F$ 是 $B C$ 和圆 $\\Gamma_1$ 的交点, 得 $\\angle A F C=90^{\\circ}$. 因为 $\\angle A E B=\\angle A F B=90^{\\circ}$, 则点 $E 、 F$ 在以 $A B$ 为直径的圆上, 有 $C E \\cdot C A=C F \\cdot C B$. 易知 $\\triangle A C L$ 是直角三角形, $L E$ 是它的高线.\n故 $C E \\cdot C A=C L^2$. 同理, 在 $\\triangle B C K$ 中, 有 $C F \\cdot C B=C K^2$. 由 $C E \\cdot C A= C F \\cdot C B$, 有 $C L^2=C K^2$, 即 $C L=C K$. 又 $L N$ 垂直于圆 $\\Gamma_1$ 的直径 $A C$, 则 $C L=C N$. 同理, 在圆 $\\Gamma_2$ 中, 有 $C K=C M$. 综上, 线段 $C K 、 C L 、 C M$ 和 $C N$ 都相等.\n则点 $K 、 L 、 M 、 N$ 在以点 $C$ 为圆心的圆上.\n故四边形 $K L M N$ 是一个圆内接四边形.", + "remark": "", + "figures": [ + "./images/volume7/figures/fig-c4a10.png" + ] +} \ No newline at end of file diff --git a/processed_dataset/proof/1853.json b/processed_dataset/proof/1853.json new file mode 100644 index 0000000000000000000000000000000000000000..1fcd0c0b017e59f1fce475380fb4206c8995508f --- /dev/null +++ b/processed_dataset/proof/1853.json @@ -0,0 +1,10 @@ +{ + "source_file": "./raw_volume-zh/volume7/exercise4.tex", + "problem_type": "proof", + "problem": "问题11. 已知 $A A_1 、 B B_1 、 C C_1$ 是锐角 $\\triangle A B C$ 的三条高线.\n证明: $C_1$ 到线段 $A C$ 、 $B C 、 B B_1 、 A A_1$ 的垂足在同一直线上.", + "solution": "证明: 如图(), 设 $B_2 、 A_2 、 M 、 N$ 分别是点 $C_1$ 到线段 $A C 、 B C 、 B B_1 、 A A_1$ 的垂足.\n令 $\\angle A B C=\\beta$. 因为 $C_1 、 B_2 、 C 、 A_2$ 四点共圆, 则 $\\angle C B_2 A_2=\\angle C C_1 A_2= 90^{\\circ}-\\angle C_1 C A_2=\\angle A B C=\\beta$. 又 $B_2 、 A 、 C_1 、 N$ 四点共圆, 则 $\\angle A B_2 N=90^{\\circ}+\\angle C_1 B_2 N=90^{\\circ}+\\angle C_1 A A_1= 90^{\\circ}+90^{\\circ}-\\angle A B C=180^{\\circ}-\\beta$. 所以, $\\angle A B_2 N+\\angle C B_2 A_2=180^{\\circ} \\cdots$ (1). 同理, $\\angle B A_2 M+\\angle C A_2 B_2= 180^{\\circ}$...(2). 由式 (1)(2) 知, 点 $B_2 、 N 、 M 、 A_2$ 在同一直线上.", + "remark": "", + "figures": [ + "./images/volume7/figures/fig-c4a11.png" + ] +} \ No newline at end of file diff --git a/processed_dataset/proof/1854.json b/processed_dataset/proof/1854.json new file mode 100644 index 0000000000000000000000000000000000000000..8d4f1b501fea4aeb49711f9857d0b7dfb1fda5d8 --- /dev/null +++ b/processed_dataset/proof/1854.json @@ -0,0 +1,10 @@ +{ + "source_file": "./raw_volume-zh/volume7/exercise4.tex", + "problem_type": "proof", + "problem": "问题12. $D$ 是 $\\triangle A B C$ 内的一点, 满足 $\\angle D A C=\\angle D C A=30^{\\circ}, \\angle D B A=60^{\\circ}, E$ 是边 $B C$ 的中点, $F$ 是边 $A C$ 的三等分点, 满足 $A F=2 F C$. 求证: $D E \\perp E F$. (2007 第六届女子数学奥林匹克)", + "solution": "证明: 如图(), 作 $D M \\perp A C$ 于点 $M, F N \\perp C D$ 于点 $N$, 连结 $E M 、 E N$. 设 $C F=a, A F= 2 a$, 则 $C N=C F \\cos 30^{\\circ}=\\frac{\\sqrt{3} a}{2}=\\frac{1}{2} C D$, 即 $N$ 是 $C D$ 的中点.\n又因为 $M$ 是边 $A C$ 上的中点, $E$ 是边 $B C$ 上的中点, 所以, $E M / / A B, E N / / B D$, 得 $\\angle M E N=\\angle A B D=60^{\\circ}=\\angle M D C$. 故 $M 、 D$ 、 $E 、 N$ 四点共圆.\n又因 $D 、 M 、 F 、 N$ 四点共圆, 所以, $D 、 E 、 F 、 M 、 N$ 五点共圆.\n从而, $\\angle D E F=90^{\\circ}$.", + "remark": "", + "figures": [ + "./images/volume7/figures/fig-c4a12.png" + ] +} \ No newline at end of file diff --git a/processed_dataset/proof/1855.json b/processed_dataset/proof/1855.json new file mode 100644 index 0000000000000000000000000000000000000000..412d60b764baf1793045a34ff34f1ffef8c6e1b0 --- /dev/null +++ b/processed_dataset/proof/1855.json @@ -0,0 +1,10 @@ +{ + "source_file": "./raw_volume-zh/volume7/exercise4.tex", + "problem_type": "proof", + "problem": "问题13. 凸四边形 $A B C D$ 有内切圆, 该内切圆切边 $A B 、 B C 、 C D 、 D A$ 的切点分别为 $A_1 、 B_1 、 C_1 、 D_1$, 连结 $A_1 B_1 、 B_1 C_1 、 C_1 D_1 、 D_1 A_1$, 点 $E 、 F 、 G 、 H$ 分别为 $A_1 B_1 、 B_1 C_1 、 C_1 D_1 、 D_1 A_1$ 的中点.\n证明: 四边形 $E F G H$ 为矩形的充分必要条件是 $A 、 B 、 C 、 D$ 四点共圆.", + "solution": "证明: 如图(), 设 $I$ 为四边形 $A B C D$ 的内切圆圆心.\n由于 $H$ 为 $D_1 A_1$ 的中点.\n而 $A A_1$ 与 $A D_1$ 为过点 $A$ 所作的 $\\odot I$ 的切线.\n故 $H$ 在 $A I$ 上, 且 $A I \\perp A_1 D_1$. 又 $I D_1 \\perp A D_1$. 故由射影定理可知 $I H \\cdot I A=I D_1^2=r^2$. 其中 $r$ 为内切圆半径.\n同理可知.\n$E$ 在 $B I$ 上,且 $I E \\cdot I B= r^2$. 于是, $I E \\cdot I B=I H \\cdot I A$. 故 $A 、 H 、 E 、 B$ 四点共圆.\n所以, $\\angle E H I=\\angle A B E$. 类似地, 可证 $\\angle I H G= \\angle A D G ; \\angle I F E=\\angle C B E, \\angle I F G=\\angle C D G$. 将这四个式子相加得 $\\angle E H G+\\angle E F G=\\angle A B C+\\angle A D C$. 所以, $A 、 B 、 C 、 D$ 四点共圆的充要条件是 $E 、 F 、 G 、 H$ 四点共圆.\n而熟知一个四边形的各边中点围成的四边形是平行四边形.\n平行四边形为矩形的充要条件是该四边形的四个顶点共圆.\n因此, $E F G H$ 为矩形的充要条件是 $A 、 B$ 、 $C 、 D$ 四点共圆.", + "remark": "", + "figures": [ + "./images/volume7/figures/fig-c4a13.png" + ] +} \ No newline at end of file diff --git a/processed_dataset/proof/1856.json b/processed_dataset/proof/1856.json new file mode 100644 index 0000000000000000000000000000000000000000..e5b505a066220d2b46140e0de132a0c84476f111 --- /dev/null +++ b/processed_dataset/proof/1856.json @@ -0,0 +1,11 @@ +{ + "source_file": "./raw_volume-zh/volume7/exercise4.tex", + "problem_type": "proof", + "problem": "问题14. 如图(), 在 $\\triangle A B C$ 中, $A B=A C, D$ 是边 $B C$ 的中点, $E$ 是 $\\triangle A B C$ 外一点, 满足 $C E \\perp A B, B E=B D$. 过线段 $B E$ 的中点 $M$ 作直线 $M F \\perp B E$, 交 $\\triangle A B D$ 的外接圆的劣弧 $\\overparen{A D}$ 于点 $F$. 求证: $E D \\perp D F$. (2010 女子数学奥林匹克)", + "solution": "证明: 如图(), 易知 $A D \\perp B C$. 由此可知 $\\triangle A B D$ 的外接圆的圆心为线段 $A B$ 的中点 $O$. 延长 $F M$ 交 $\\odot O$ 于点 $L$, 连结 $O E$, 过点 $O$ 作 $O H \\perp F L, O K \\perp A D$, 分别交 $F L 、 A D$ 于点 $H 、 K$. 设直线 $F M$ 分别与直线 $E D 、 A B 、 A D$ 交于点 $S$ 、 $I 、 P$, 直线 $C E$ 与 $A B$ 交于点 $N$. 由条件知 $C N \\perp A B$. 所以, $A 、 N 、 D 、 C$ 四点共圆.\n故 $B D \\cdot B C=B N \\cdot A B$. 因为 $B C=2 B E, A B= 2 B O$, 所以, $B E^2=B N \\cdot B O$. 由射影定理得 $O E \\perp B E$. 从而, 四边形 $O E M H$ 是矩形.\n则 $O H=E M=\\frac{1}{2} B E$. 因为 $O$ 是 $A B$ 的中点, 且 $O K / / B D$, 所以, $O K=\\frac{1}{2} B D= \\frac{1}{2} B E=O H$. 于是, $F L=A D$. 从而, $\\overparen{L D}=\\overparen{A F} \\Rightarrow \\angle P F D=\\angle P D F$. 因为 $M F \\perp B E$, 所以, $\\angle B E D+\\angle M S E=90^{\\circ}$. 而 $\\angle P D S+\\angle B D E=90^{\\circ}$, 且 $\\angle B E D=\\angle B D E$, 于是, $\\angle P D S=\\angle M S E=\\angle D S P$. 因此, $\\angle F D S=90^{\\circ}$, 即 $E D \\perp F D$.", + "remark": "", + "figures": [ + "./images/volume7/figures/fig-c4p14.png", + "./images/volume7/figures/fig-c4a14.png" + ] +} \ No newline at end of file diff --git a/processed_dataset/proof/1857.json b/processed_dataset/proof/1857.json new file mode 100644 index 0000000000000000000000000000000000000000..586bc4ae8cc45b7fa10fa7caa00c644cc377bfa1 --- /dev/null +++ b/processed_dataset/proof/1857.json @@ -0,0 +1,11 @@ +{ + "source_file": "./raw_volume-zh/volume7/exercise4.tex", + "problem_type": "proof", + "problem": "问题15. 如图(), 在锐角 $\\triangle A B C$ 中, $A B>A C, M$ 为边 $B C$ 的中点, $\\angle B A C$ 的外角平分线交直线 $B C$ 于点 $P$. 点 $K 、 F$ 在直线 $P A$ 上, 使得 $M F \\perp B C, M K \\perp P A$. 求证: $B C^2=4 P F \\cdot A K$. (2010 女子数学奥林匹克)", + "solution": "证明: 如图(), 设 $\\triangle A B C$ 的外接圆 $\\odot O$ 交直线 $F M$ 于点 $D, A D$ 交 $B C$ 于点 $E$. 易知 $A D$ 平分 $\\angle B A C$. 所以, $A D \\perp A P, A D / / M K$. 故 $\\frac{M D}{F M}=\\frac{A K}{F K}$. 因为 $\\angle F M C=\\angle F A D=90^{\\circ}$, 所以, $F 、 M 、 E 、 A$ 四点共圆, 有 $\\angle A F D= \\angle A E C=\\angle A B C+\\frac{1}{2} \\angle B A C$. 又 $\\angle A B D=\\angle A B C+\\angle C B D=\\angle A B C+\\frac{1}{2} \\angle B A C=\\angle A F D$, 则 $A 、 F 、 B 、 D$ 四点共圆.\n故 $A 、 F 、 B 、 D 、 C$ 五点共圆.\n根据圆幂定理得 $P A \\cdot P F=P C \\cdot P B=(P M-M C)(P M+B M)=P M^2-B M^2 \\cdots$ (1). 对 Rt $\\triangle F M P$ 利用射影定理得 $P M^2=P K \\cdot P F \\cdots$ (2). (2)- (1) 得 $B M^2= P K \\cdot P F-P A \\cdot P F=P F(P K-P A)=P F \\cdot A K$. 因为 $B M^2=\\left(\\frac{B C}{2}\\right)^2= \\frac{B C^2}{4}$, 所以, 结论成立.", + "remark": "", + "figures": [ + "./images/volume7/figures/fig-c4p15.png", + "./images/volume7/figures/fig-c4a15.png" + ] +} \ No newline at end of file diff --git a/processed_dataset/proof/1858.json b/processed_dataset/proof/1858.json new file mode 100644 index 0000000000000000000000000000000000000000..0667205deae362d57e072dc9e50126ca15f7afa4 --- /dev/null +++ b/processed_dataset/proof/1858.json @@ -0,0 +1,10 @@ +{ + "source_file": "./raw_volume-zh/volume7/exercise4.tex", + "problem_type": "proof", + "problem": "问题16. 已知圆 $O$ 、圆 $I$ 分别是 $\\triangle A B C$ 的外接圆和内切圆.\n证明: 过圆 $O$ 上的任意一点 $D$, 都可以作一个三角形 $D E F$, 使得圆 $O$ 、圆 $I$ 分别是 $\\triangle D E F$ 的外接圆和内切圆.", + "solution": "证明: 如图(), 设 $O I=d, R, r$ 分别是 $\\triangle A B C$ 的外接圆和内切圆半径, 延长 $A I$ 交圆 $O$ 于 $K$, 则 $K I=K B=2 R \\sin \\frac{A}{2}, A I=\\frac{r}{\\sin \\frac{A}{2}}$, 延长 $O I$ 交 $\\odot O$ 于 $M 、 N$; 则 $(R+d)(R-d)=I M \\times I N= A I \\times K I=2 R r$, 即 $R^2-d^2=2 R r$. (注: 这实际上是所谓欧拉公式) 过 $D$ 分别作 $\\odot I$ 的切线 $D E, D F, E, F$ 在 $\\odot O$ 上, 连结 $E F$, 则 $D I$ 平分 $\\angle E D F$, 只要证 $E F$ 也与 $\\odot I$ 相切.\n设 $D I \\cap \\odot O=P$, 则 $P$ 是 $\\overparen{E F}$ 的中点, 连 $P E$, 则 $P E=2 R \\sin \\frac{D}{2}$, $D I=\\frac{r}{\\sin \\frac{D}{2}}, I D \\cdot I P=I M \\cdot I N=(R+d)(R-d)=R^2-d^2$, 所以 $P I= \\frac{R^2-d^2}{D I}=\\frac{R^2-d^2}{r} \\cdot \\sin \\frac{D}{2}=2 R \\sin \\frac{D}{2}=P E$, 由于 $I$ 在角 $D$ 的平分线上, 因此点 $I$ 是 $\\triangle D E F$ 的内心, (这是由于, $\\angle P E I=\\angle P I E=\\frac{1}{2}\\left(180^{\\circ}-\\angle P\\right)= \\frac{1}{2}\\left(180^{\\circ}-\\angle F\\right)=\\frac{D+E}{2}$, 而 $\\angle P E F=\\frac{D}{2}$, 所以 $\\angle F E I=\\frac{E}{2}$, 点 $I$ 是 $\\triangle D E F$ 的内心). 即弦 $E F$ 与 $\\odot I$ 相切.", + "remark": "", + "figures": [ + "./images/volume7/figures/fig-c4a16.png" + ] +} \ No newline at end of file diff --git a/processed_dataset/proof/1859.json b/processed_dataset/proof/1859.json new file mode 100644 index 0000000000000000000000000000000000000000..bd5764118b7350cf7adcf02141bdcbd498bd052b --- /dev/null +++ b/processed_dataset/proof/1859.json @@ -0,0 +1,10 @@ +{ + "source_file": "./raw_volume-zh/volume7/exercise4.tex", + "problem_type": "proof", + "problem": "问题17. 设 $H$ 为锐角 $\\triangle A B C$ 的垂心, $D$ 为边 $B C$ 的中点, 过点 $H$ 的直线分别交边 $A B 、 A C$ 于点 $F 、 E$, 使得 $A E=A F$, 射线 $D H$ 与 $\\triangle A B C$ 的外接圆交于点 $P$. 求证: $P 、 A 、 E 、 F$ 四点共圆.", + "solution": "证明: 如图(), 延长 $H D$ 至点 $M$, 使 $H D=D M$, 连结 $B M 、 C M 、 B H 、 C H$. 因为 $D$ 为边 $B C$ 的中点, 所以, 四边形 $B H C M$ 为平行四边形.\n于是, $\\angle B M C= \\angle B H C=180^{\\circ}-\\angle B A C$ (这里用到垂心、四点共圆), 即 $\\angle B M C+\\angle B A C=180^{\\circ}$. 因此, 点 $M$ 在 $\\triangle A B C$ 的外接圆上.\n连结 $P B 、 P C 、 P E 、 P F$. 因 $A E=A F, H$ 为 $\\triangle A B C$ 的垂心, 所以, $\\angle B F H=\\angle C E H \\cdots$ (1), $\\angle H B F= 90^{\\circ}-\\angle B A C=\\angle H C E \\cdots$ (2).\n综合式(1)、(2)知 $\\triangle B F H \\backsim \\triangle C E H \\Rightarrow \\frac{B F}{B H}=\\frac{C E}{C H}$. 由四边形 $B H C M$ 是平行四边形知 $B H=C M, C H=B M$. 于是, $\\frac{B F}{C M}=\\frac{C E}{B M} \\cdots$ (3). 又 $D$ 为边 $B C$ 的中点, 则 $S_{\\triangle P B M}=S_{\\triangle P C M}$. 故 $\\frac{1}{2} B P \\cdot B M \\sin \\angle M B P=\\frac{1}{2} C P \\cdot C M \\sin \\angle M C P$. 由 $\\angle M B P+\\angle M C P=180^{\\circ}$, 得 $B P \\cdot B M=C P \\cdot C M \\cdots$ (4). 结合 (3)、(4) 知 $\\frac{B F}{B P}=\\frac{C E}{C P}$. 因为 $\\angle P B F=\\angle P C E$, 所以, $\\triangle P B F \\backsim \\triangle P C E \\Rightarrow \\angle P F B= \\angle P E C$. 于是, $\\angle P F A=\\angle P E A$. 从而, $P 、 A 、 E 、 F$ 四点共圆.", + "remark": "", + "figures": [ + "./images/volume7/figures/fig-c4a17.png" + ] +} \ No newline at end of file diff --git a/processed_dataset/proof/1860.json b/processed_dataset/proof/1860.json new file mode 100644 index 0000000000000000000000000000000000000000..c5654a779b20e6767f139c2359bb72d0595c3612 --- /dev/null +++ b/processed_dataset/proof/1860.json @@ -0,0 +1,11 @@ +{ + "source_file": "./raw_volume-zh/volume7/exercise4.tex", + "problem_type": "proof", + "problem": "问题18. 如图(), 已知圆 $S_1$ 与圆 $S_2$ 交于 $P 、 Q$ 两点, $A_1 、 B_1$ 为圆 $S_1$ 上不同于 $P 、 Q$ 的两个点, 直线 $A_1 P 、 B_1 P$ 分别交圆 $S_2$ 于 $A_2 、 B_2$, 直线 $A_1 B_1$ 和 $A_2 B_2$ 交于点 $C$. 证明: 当点 $A_1$ 和点 $B_1$ 变化时, $\\triangle A_1 A_2 C$ 的外心总在一个定圆周上.", + "solution": "证明: 如图(), 因为 $\\angle A_1 C A_2+\\angle A_1 Q A_2= \\angle A_1 C A_2+\\angle A_1 Q P+\\angle P Q A_2=\\angle B_1 C B_2+\\angle C B_1 B_2+ \\angle C B_2 B_1=180^{\\circ}$, 则 $A_1 、 C 、 A_2 、 Q$ 四点共圆.\n设 $O$ 是 $\\triangle A_1 A_2 C$ 的外心, $O_1 、 O_2$ 分别为圆 $S_1$ 和圆 $S_2$ 的圆心, 则 $\\angle O O_1 Q=\\frac{1}{2} \\angle A_1 O_1 Q=180^{\\circ}-\\angle A_1 P Q$. 同理, $\\angle O O_2 Q=180^{\\circ}-\\angle A_2 P Q$. 所以, $\\angle O O_1 Q+\\angle O O_2 Q=180^{\\circ}$. 因此, $\\triangle A_1 A_2 C$ 的外心总在一个过 $O_1 、 O_2$ 和 $Q$ 的定圆上.", + "remark": "", + "figures": [ + "./images/volume7/figures/fig-c4p18.png", + "./images/volume7/figures/fig-c4a18.png" + ] +} \ No newline at end of file diff --git a/processed_dataset/proof/1861.json b/processed_dataset/proof/1861.json new file mode 100644 index 0000000000000000000000000000000000000000..75cd475b183a3b305aa1e454fb79344f78654a93 --- /dev/null +++ b/processed_dataset/proof/1861.json @@ -0,0 +1,11 @@ +{ + "source_file": "./raw_volume-zh/volume7/exercise4.tex", + "problem_type": "proof", + "problem": "问题19. 两圆外切于点 $A$ 且内切另一圆 $\\odot T$ 于点 $B 、 C$. 令 $D$ 是小圆内公切线割 $\\odot T$ 的弦的中点.\n证明: 当点 $B 、 C 、 D$ 不共线时, $A$ 是 $\\triangle B C D$ 的内心.", + "solution": "证明: 先证明一个引理: 如图(), $\\odot O$ 与弓形相切于点 $C 、 D$. 则 $C D$ 平分 $\\angle A C B$.\n引理的证明: 过 $C$ 作 $\\odot O$ 切线与 $B A$ 交于 $E$. 则由 $E C, E D$ 均为 $\\odot O$ 的切线知 $\\angle E C D=\\angle E D C$, 且 $\\angle E C A=\\angle C B D$. 注意到 $\\angle E C D= \\angle E C A+\\angle A C D, \\angle E D C=\\angle C B D+\\angle B C D$. 引理得证.\n回到原题 (这里的字母与上述引理字母表示不相同.\n)\n如图(), 设 $C A 、 B A$ 与 $\\odot T$ 分别相交于点 $E, F$. 连结 $B C$, $B E, C F, B D, E F, M B, M C, C N$. 由于 $\\odot T_2$ 与弓形 $M F N$ 相切, 由上述引理知 $C A$ 平分 $\\angle M C N$. 从而 $E$ 是优弧 $\\overparen{M N}$ 的中点.\n同理 $F$ 是劣弧 $\\overparen{M N}$ 的中点.\n而 $D$ 是 $M N$ 的中点.\n故 $E, T, D, F$ 四点共线, $\\angle E B F= \\angle E C F=90^{\\circ} ; \\angle M D E=\\angle M D F=90^{\\circ}$. 从而 $A 、 B 、 E 、 D$ 及 $A 、 D 、 F 、 C$ 分别四点共圆.\n再注意到 $C 、 B 、E 、 F$ 也四点共圆.\n因此, $\\angle C B F=\\angle C E F=\\angle D B A$. 即 $\\angle C B A=\\angle D B A$. 同理 $\\angle B C A=\\angle D C A$. 所以 $A$ 是 $\\triangle B C D$ 的内心.", + "remark": "", + "figures": [ + "./images/volume7/figures/fig-c4a19-1.png", + "./images/volume7/figures/fig-c4a19-2.png" + ] +} \ No newline at end of file diff --git a/processed_dataset/proof/1862.json b/processed_dataset/proof/1862.json new file mode 100644 index 0000000000000000000000000000000000000000..52124bd46a2c5efc33596874efb68c6637b5318f --- /dev/null +++ b/processed_dataset/proof/1862.json @@ -0,0 +1,10 @@ +{ + "source_file": "./raw_volume-zh/volume7/exercise5.tex", + "problem_type": "proof", + "problem": "问题1. 如图(),已知 $\\odot O_1$ 和 $\\odot O_2$ 相交于 $A$ 和 $B, P$ 是线段 $A B$ 上一点, $K M$ 是过 $P$ 点的 $\\odot O_1$ 的弦, $L N$ 是过 $P$ 点的 $\\odot O_2$ 的弦.\n求证: $K 、 L 、 M 、 N$ 四点共圆.", + "solution": "证明: 因为 $K M, A B$ 为 $\\odot O_1$ 的两条相交弦, 所以 $P K \\cdot P M=P A \\cdot P B$. 同理, $P L \\cdot P N=P A \\cdot P B$, 所以 $P K \\cdot P M=P L \\cdot P N$. 由相交弦定理的逆定理得到 $K 、 L 、 M 、 N$ 四点共圆.", + "remark": "", + "figures": [ + "./images/volume7/figures/fig-c5p1.png" + ] +} \ No newline at end of file diff --git a/processed_dataset/proof/1863.json b/processed_dataset/proof/1863.json new file mode 100644 index 0000000000000000000000000000000000000000..1b733822df71a8c01ea0988a1591689ac6b7c46b --- /dev/null +++ b/processed_dataset/proof/1863.json @@ -0,0 +1,11 @@ +{ + "source_file": "./raw_volume-zh/volume7/exercise5.tex", + "problem_type": "proof", + "problem": "问题2. 如图(), $A B C D$ 为 $\\odot O$ 的内接四边形, 延长 $A B$ 和 $D C$ 相交于 $E$, 延长 $A D$ 和 $B C$ 相交于 $F, E P$ 和 $F Q$ 分别切 $\\odot O$ 于 $P 、 Q$. 求证: $E P^2+F Q^2= E F^2$.", + "solution": "证明: 如图(), 作 $\\triangle B C E$ 的外接圆交 $E F$ 于 $G$, 连结 $C G$. 又因为 $\\angle F D C= \\angle A B C=\\angle E G C$, 故 $C 、 D 、 F 、 G$ 四点共圆.\n由切割线定理, 有 $E P^2=E C E D=E G \\cdot E F, F Q^2=F C \\cdot F B=F G \\cdot F E$, 所以 $E P^2+F Q^2=E G \\cdot E F+ F G \\cdot F E=E F(E G+F G)=E F^2$.", + "remark": "", + "figures": [ + "./images/volume7/figures/fig-c5p2.png", + "./images/volume7/figures/fig-c5a2.png" + ] +} \ No newline at end of file diff --git a/processed_dataset/proof/1864.json b/processed_dataset/proof/1864.json new file mode 100644 index 0000000000000000000000000000000000000000..bd3ad3028180f1dc3fcb5f708b04b49c982f0735 --- /dev/null +++ b/processed_dataset/proof/1864.json @@ -0,0 +1,10 @@ +{ + "source_file": "./raw_volume-zh/volume7/exercise5.tex", + "problem_type": "proof", + "problem": "问题3. 设 $A 、 B 、 C 、 D$ 是一条直线上依次排列的四个不同的点, 分别以 $A C 、 B D$ 为直径的圆相交于 $X$ 和 $Y$, 直线 $X Y$ 交 $B C$ 于 $Z$. 若 $P$ 为 $X Y$ 上开于 $Z$ 的一点, 直线 $C P$ 与以 $A C$ 为直径的圆相交于 $C$ 和 $M$, 直线 $B P$ 与以 $B D$ 为直径的圆相交于 $B$ 和 $N$. 试证: $A M 、 D N$ 和 $X Y$ 三线共点.", + "solution": "证明: 设 $A M$ 交直线 $X Y$ 于点 $Q$, 而 $D N$ 交直线 $X Y$ 于点 $Q^{\\prime}$ (如图(), 注意: 这里只画出了点 $P$ 在线段 $X Y$ 上的情形, 其他情况可类似证明). 需证: $Q$ 与 $Q^{\\prime}$ 重合.\n由于 $X Y$ 为两圆的根轴, 故 $X Y \\perp A D$, 而 $A C$ 为直径, 所以 $\\angle Q M C= \\angle P Z C=90^{\\circ}$. 进而, $Q 、 M 、 Z 、 C$ 四点共圆.\n同理 $Q^{\\prime} 、 N 、 Z 、 B$ 四点共圆.\n这样, 利用圆幂定理, 可知 $Q P \\cdot P Z=M P \\cdot P C=X P \\cdot P Y, Q^{\\prime} P \\cdot P Z= N P \\cdot P B=X P \\cdot P Y$. 所以, $Q P=Q^{\\prime} P$. 而 $Q$ 与 $Q^{\\prime}$ 都在直线 $X Y$ 上且在直线 $A D$ 同侧, 从而, $Q$ 与 $Q^{\\prime}$ 重合.\n命题获证.", + "remark": "", + "figures": [ + "./images/volume7/figures/fig-c5a3.png" + ] +} \ No newline at end of file diff --git a/processed_dataset/proof/1865.json b/processed_dataset/proof/1865.json new file mode 100644 index 0000000000000000000000000000000000000000..2f1cb84e6e09bec18cf7b5db590cb01d09000c25 --- /dev/null +++ b/processed_dataset/proof/1865.json @@ -0,0 +1,10 @@ +{ + "source_file": "./raw_volume-zh/volume7/exercise5.tex", + "problem_type": "proof", + "problem": "问题4. 圆 $\\Gamma_1$ 和圆 $\\Gamma_2$ 相交于点 $M$ 和 $N$. 设 $l$ 是圆 $\\Gamma_1$ 和圆 $\\Gamma_2$ 的两条公切线中距离 $M$ 较近的那条公切线.\n$l$ 与圆 $\\Gamma_1$ 相切于点 $A$, 与圆 $\\Gamma_2$ 相切于点 $B$. 设经过点 $M$ 且与 $l$ 平行的直线与圆 $\\Gamma_1$ 还相交于点 $C$, 与圆 $\\Gamma_2$ 还相交于点 $D$. 直线 $C A$ 和 $D B$ 相交于点 $E$, 直线 $A N$ 和 $C D$ 相交于点 $P$, 直线 $B N$ 和 $C D$ 相交于点 $Q$. 证明: $E P=E Q$.", + "solution": "证明: 如图(), 令 $K$ 为 $M N$ 和 $A B$ 的交点.\n根据圆幂定理: $A K^2=K N \\cdot K M=B K^2$. 换言之, $K$ 是 $A B$ 的中点, 因为 $P Q / / A B$, 所以 $M$ 是 $P Q$ 的中点, 故只需证明 $E M \\perp P Q$.\n因为 $C D // A B$, 所以点 $A$ 是圆 $\\Gamma_1$ 的弧 $C M$ 的中点, 点 $B$ 是圆 $\\Gamma_2$ 的弧 $D M$ 的中点.\n于是, $\\triangle A C M$ 与 $\\triangle B D M$ 都是等腰三角形.\n从而 $\\angle B A M= \\angle A M C=\\angle A C M=\\angle E A B, \\angle A B M= \\angle B M D=\\angle B D M=\\angle E B A$. 故 $E M \\perp A B$. 再由 $PQ // AB$, 即得 $E M\\perp P Q$ .", + "remark": "", + "figures": [ + "./images/volume7/figures/fig-c5a4.png" + ] +} \ No newline at end of file diff --git a/processed_dataset/proof/1866.json b/processed_dataset/proof/1866.json new file mode 100644 index 0000000000000000000000000000000000000000..cc13a8ecf72f7d265238344c22a3baacc07709c7 --- /dev/null +++ b/processed_dataset/proof/1866.json @@ -0,0 +1,10 @@ +{ + "source_file": "./raw_volume-zh/volume7/exercise5.tex", + "problem_type": "proof", + "problem": "问题5. 设 $A$ 是圆 $O$ 的直径 $B B^{\\prime}$ 上或其延长线上任一定点,过 $A$ 引圆 $O$ 的割线 $M A M^{\\prime}$ 或 $A M M^{\\prime}$, 过 $A$ 作 $B B^{\\prime}$ 的垂线交 $B M$ 的延长线于点 $N$, 交 $B M^{\\prime}$ 的延长线于点 $N^{\\prime}$. 求证: $A N \\cdot A N^{\\prime}$ 是定值.", + "solution": "证明: 如图(), $\\angle B N A=90^{\\circ}-\\angle M B A=\\angle B B^{\\prime}M= BM'M$.从而有 $M、N、M'、N'$ 共圆,记此圆为 $\\Gamma$. 注意到圆$O$与 $\\Gamma$ 的交点为$M、M'$ ,所以 $M M^{\\prime}$ 是圆$O$和 $\\Gamma$ 的根轴, 又 $A$ 在 $M M^{\\prime}$ 上, 所以 $A$ 关于两圆等幂, 即 $A B \\cdot A B^{\\prime}=A N \\cdot A N^{\\prime}$.", + "remark": "", + "figures": [ + "./images/volume7/figures/fig-c5a5.png" + ] +} \ No newline at end of file diff --git a/processed_dataset/proof/1867.json b/processed_dataset/proof/1867.json new file mode 100644 index 0000000000000000000000000000000000000000..e12584025be66b4507291b9d978de22b9c7828ff --- /dev/null +++ b/processed_dataset/proof/1867.json @@ -0,0 +1,11 @@ +{ + "source_file": "./raw_volume-zh/volume7/exercise5.tex", + "problem_type": "proof", + "problem": "问题6. 如图(), 某圆分别与凸四边形 $A B C D$ 的 $A B 、 B C$ 两边相切于 $G 、 H$ 两点, 与对角线 $A C$ 相交于 $E 、 F$ 两点: 问 $A B C D$ 应满足怎样的充要条件, 使得存在另一圆过 $E 、 F$ 两点,且分别与 $D A 、 D C$ 的延长线相切?", + "solution": "证明: 如图(), 所求的充分必要条件是 $A B+ A D=C B+C D$.\n(1)必要性的证明.\n设过 $E 、 F$ 两点的另一圆分别与 $D A$ 的延长线和 $D C$ 延长线相切于 $J$ 和 $K$ 两点, 由于 $A J, A G$ 分别是大圆, 小圆的切线,所以由圆幕定理知 $A G^2=A E \\cdot A F$ (对小圆) $A J^2=A E$ ・ $A F$ (对大圆), 故 $A G=A J$. 同理, $C H=C K$. 则有 $A B+A D=B G+G A+A D=B G+J D=B H+ 2^{\\circ}$ \n(2)充分性的证明.\n设凸四边形 $A B C D$ 满足条件 $A B+A D=C B+C D$. 在 $D A$ 延长线和 $D C$ 延长线上分别取 $J$ 点和 $K$ 点, 使 $A J=A G, C K=C H$, 于是\n$D J=J A+A D=A G+A D=A B+A D-B G=C B+C D-B H=C H+ C D=D K$. 过 $J$ 点和 $K$ 点分别作 $D J$ 和 $D K$ 的垂线, 以两垂线交点为圆心作通过 $J$ 点和 $K$ 点的圆.\n因为 $A J=A G, C K=C H$, 所以 $A$ 点和 $C$ 点关于原有圆的幂分别等于这两点关于所作圆的幕.\n又因为直线 $A C$ 与原有的圆相交于 $E$ 和 $F$.两点,所以 $E F$ 是两圆的公共弦 (直线 $A C$ 是两圆的根轴).\n至此, 我们证明了所作的与 $D A$ 延长线和 $D C$ 延长线相切的圆通过 $E 、 F$ 两点.", + "remark": "", + "figures": [ + "./images/volume7/figures/fig-c5p6.png", + "./images/volume7/figures/fig-c5a6.png" + ] +} \ No newline at end of file diff --git a/processed_dataset/proof/1868.json b/processed_dataset/proof/1868.json new file mode 100644 index 0000000000000000000000000000000000000000..553f6affa65d08d39ce80b84e016c9eb97f2a566 --- /dev/null +++ b/processed_dataset/proof/1868.json @@ -0,0 +1,11 @@ +{ + "source_file": "./raw_volume-zh/volume7/exercise5.tex", + "problem_type": "proof", + "problem": "问题7. 如图(), $\\triangle A B C$ 中, $E 、 F$ 分别为 $A B 、 A C$ 中点, $C M 、 B N$ 为高, $E F$ 交 $M N$ 于 $P$, $O 、 H$ 分别为三角形的外心与垂心.\n求证: $A P \\perp O H$.", + "solution": "证明:如图(), 由 $\\angle B M C=\\angle B N C=90^{\\circ}$ 知 $B 、 C$ 、 $N 、 M$ 四点共圆.\n所以 $A M \\cdot A B=A N \\cdot A C$. 又 $A E=\\frac{1}{2} A B, A F=\\frac{1}{2} A C$, 则 $A M \\cdot A E=A N \\cdot A F$, 即 $E 、 F 、 N 、 M$ 共圆.\n注意到由 $\\angle A M H= \\angle A N H=\\angle A E O=\\angle A F O=90^{\\circ}$ 知 $A H 、 A O$ 分别为 $\\triangle A M N 、 \\triangle A E F$ 外接圆的直径.\n过 $A H$ 中点 $H^{\\prime}$ 与 $A O$ 中点 $O^{\\prime}$ 分别为 $\\triangle A M N$ 与 $\\triangle A E F$ 的外心, 且易知 $O^{\\prime} H^{\\prime} / / O H$. 故只需证 $A P \\perp O^{\\prime} H^{\\prime}$, 只需证 $A 、 P$ 为 $\\triangle A M N 、 \\triangle A E F$ 外接圆的等幂点即可.\n注意到 $A$ 为两圆公共点, 而由 $E 、 F 、 N 、 M$ 共圆知 $P M \\cdot P N=P E \\cdot P F$. 故 $P$ 也为等幂点.\n综上所述, 原命题成立.", + "remark": "", + "figures": [ + "./images/volume7/figures/fig-c5p7.png", + "./images/volume7/figures/fig-c5a7.png" + ] +} \ No newline at end of file diff --git a/processed_dataset/proof/1869.json b/processed_dataset/proof/1869.json new file mode 100644 index 0000000000000000000000000000000000000000..6f3eeee1d966bff6becab52a268b0f40220544ae --- /dev/null +++ b/processed_dataset/proof/1869.json @@ -0,0 +1,12 @@ +{ + "source_file": "./raw_volume-zh/volume7/exercise5.tex", + "problem_type": "proof", + "problem": "问题8. 如图(), 已知圆 $O$ 与两条平行线 $l_1$ 和 $l_2$ 相切; 第二个圆圆 $O_1$ 切 $l_1$ 于点 $A$, 外切圆 $O$ 于点 $C$; 第三个圆圆 $O_2$ 切 $l_2$ 于点 $B$, 外切圆 $O$ 于点 $D$, 外切圆 $O_1$ 于点 $E, A D$ 交 $B C$ 于 $Q$. 求证: $Q$ 是 $\\triangle C D E$ 的外心.", + "solution": "引理 (字母与原题无关). 如图(), 设 $\\odot O_1 、 \\odot O_2$ 外切于点 $C$, 直线 $l_1 / / l_2$, 且 $l_1$ 切 $\\odot O_1$ 于点 $A, l_2$ 切 $\\odot O_2$ 于点 $B$, 那么 $A 、 C 、 B$ 三点共线.\n引理的证明: $l_1 / / l_2 \\Rightarrow O_1 A / / O_2 B \\Rightarrow \\angle A O_1 C=\\angle B O_2 C \\Rightarrow \\angle A C O_1= 90^{\\circ}-\\frac{\\angle A O_1 C}{2}=90^{\\circ}-\\frac{\\angle B O_2 C}{2}=\\angle O_2 C B \\Rightarrow A 、 C 、 B$ 共线.\n回到原题, 如图(), 令 $\\odot O$ 与 $l_1$ 切于 $G$, 与 $l_2$ 切于 $F$, 由引理知 $A, C, F$ 共线, $A$ 、 $E 、 B$ 共线,而 $\\angle A E C=\\angle G A C=\\angle C F B$. 故 $C 、 E 、 B 、 F$ 四点共圆, 由割线定理, $A C \\cdot A F=A E \\cdot A B$, 从而 $A$ 关于 $\\odot O 、 \\odot O_2$ 等幂,所以 $A$ 在两圆的根轴上, 所以直线 $A D$ 为 $\\odot O, \\odot O_2$ 的根轴, 又 $\\odot O$ 与 $\\odot O_2$ 切于点 $D$, 则 $A D$ 为切线, 即 $Q D$ 为两圆公切线, 同理 $Q C$ 为 $\\odot O$ 与 $\\odot O_2$ 公切线, 从而 $Q$ 同时位于两个根轴上, 所以 $Q$ 为三圆根心.\n从而 $Q E$ 为 $\\odot O_1 、 \\odot O_2$ 的公切线.\n$Q C= Q D=Q E, Q$ 为 $\\triangle C D E$ 外接圆圆心.", + "remark": "", + "figures": [ + "./images/volume7/figures/fig-c5p8.png", + "./images/volume7/figures/fig-c5a8-1.png", + "./images/volume7/figures/fig-c5a8-2.png" + ] +} \ No newline at end of file diff --git a/processed_dataset/proof/1870.json b/processed_dataset/proof/1870.json new file mode 100644 index 0000000000000000000000000000000000000000..65b97e8082e07f2f801bb616000ccb87850ac8cf --- /dev/null +++ b/processed_dataset/proof/1870.json @@ -0,0 +1,10 @@ +{ + "source_file": "./raw_volume-zh/volume7/exercise5.tex", + "problem_type": "proof", + "problem": "问题9. 设四边形 $A B C D$ 的对角线交于点 $O$, 点 $M$ 、 $N$ 分别是 $A D 、 B C$ 的中点, 点 $H_1 、 H_2$ (不重合)分别是 $\\triangle A O B$ 与 $\\triangle C O D$ 的垂心.\n求证: $H_1 H_2 \\perp M N$.", + "solution": "证明: 如图(), 以 $A B$ 为直径作圆 $S$ 交 $A C$ 于 $F$, 交 $B D$ 于 $E$, 那么 $A E \\perp B O, B F \\perp A O$, 从而有 $H_1$ 为 $A E 、 B F$ 交点, 显然, $F$ 在以 $N$ 为圆心、 $N B$ 为半径 (即以 $B C$ 为直径) 的 $O N$ 上, $E$ 在以 $A O$ 为直径的 $\\odot M$ 上, 因而, 直线 $B F$ 是 $\\odot S 、 \\odot N$ 的根轴, 直线 $A E$ 是 $\\odot S 、 \\odot M$ 的根轴.\n从而 $H_1=B F \\cap A E$ 是 $\\odot S 、 \\odot M 、 \\odot N$ 的根心 $\\Rightarrow H_1$ 在 $\\odot M 、 \\odot N$ 根轴上, 同理可证, $H_2$ 在 $\\odot M 、 \\odot N$ 根轴上, 故 $H_1 H_2 \\perp M N$ (根轴 $\\perp$ 连心线).", + "remark": "", + "figures": [ + "./images/volume7/figures/fig-c5a9.png" + ] +} \ No newline at end of file diff --git a/processed_dataset/proof/1871.json b/processed_dataset/proof/1871.json new file mode 100644 index 0000000000000000000000000000000000000000..660041ede9d8a2cffd029d24e27272a45af03f3a --- /dev/null +++ b/processed_dataset/proof/1871.json @@ -0,0 +1,10 @@ +{ + "source_file": "./raw_volume-zh/volume7/exercise5.tex", + "problem_type": "proof", + "problem": "问题10. 两个大圆圆 $A$ 、圆 $B$ 相等且相交, 两个小圆圆 $C$ 、圆 $D$ 不等亦相交,且交点为 $P 、 Q$. 若圆 $C$ 、圆 $D$ 既同时与圆 $A$ 内切, 又同时与圆 $B$ 外切.\n求证: 直线 $P Q$ 平分线段 $A B$.", + "solution": "证明: 如图(), 令 $A B$ 的中点为 $M$, 记圆 $C$ 与圆 $A$ 内切于 $S$, 与圆 $B$ 外切于 $T$, 设圆 $A$ 与圆 $B$ 的半径为 $R$, 圆 $C$ 的半径为 $r$, 则 $C A=S A-S C=R-r$, $C B=C T+T B=R+r$. 从而, $M$ 关于 $\\odot C$ 的幂 $= M C^2-r^2=\\frac{1}{2} \\cdot\\left(A C^2+B C^2\\right)-M A^2-r^2 \\text{(中线长公式)}=\\frac{1}{2}\\left((R+r)^2+(R-r)^2\\right)-M C^2-r^2=R^2-M A^2$ 为与 $\\odot C$ 无关的定值.\n同理 $M$ 关于 $\\odot D$ 的幂 $=R^2-M A^2$. 所以 $M$ 在 $\\odot C$ 与 $\\odot D$ 的根轴, 即直线 $P Q$ 上.\n即 $P Q$ 过 $A B$ 中点 $M$.", + "remark": "", + "figures": [ + "./images/volume7/figures/fig-c5a10.png" + ] +} \ No newline at end of file diff --git a/processed_dataset/proof/1872.json b/processed_dataset/proof/1872.json new file mode 100644 index 0000000000000000000000000000000000000000..a2e3ce4b02b5b99486dc4a4c963186c18c9ba167 --- /dev/null +++ b/processed_dataset/proof/1872.json @@ -0,0 +1,10 @@ +{ + "source_file": "./raw_volume-zh/volume7/exercise5.tex", + "problem_type": "proof", + "problem": "问题11. 在平面上有三个两两外离的圆 $\\Gamma_1 、 \\Gamma_2 、 \\Gamma_3$, 对于这三个圆外的任意一点 $P$, 将六个点 $A_1 、 B_1 、 A_2 、 B_2 、 A_3 、 B_3$ 定义如下: 对于 $i=1,2,3, A_i 、 B_i$ 是圆 $\\Gamma_i$ 上相异的两点, 使得直线 $P A_i 、 P B_i$ 均与圆 $\\Gamma_i$ 相切.\n若 $A_1 B_1$ 、 $A_2 B_2 、 A_3 B_3$ 三线共点,则称此时的点 $P$ 为 \"独特的\". 求证: 若平面上存在独特的点, 则所有这样的点落在同一个圆上.", + "solution": "证明: 记圆 $\\Gamma_i(i=1,2,3)$ 的圆心、半径分别为 $O_i 、 r_i$. 设 $P$ 为一个独特的点, 且与其相应的三条直线 $A_1 B_1 、 A_2 B_2 、 A_3 B_3$ 交于点 $Q$. 以线段 $P Q$ 为直径作圆并记为圆 $\\Gamma$, 其圆心、半径分别为 $O 、 r$.\n接下来证明: 所有独特的点都在圆 $\\Gamma$ 上.\n如图(), 记 $P O_1$ 交 $A_1 B_1$ 于点 $X_1$. 由 $P O_1 \\perp A_1 B_1$ 知, 点 $X_1$ 在圆 $\\Gamma$ 上.\n由 $P A_1$ 与圆 $\\Gamma_1$ 相切及射影定理知 $O_1 X_1 \\cdot O_1 P= O_1 A_1^2=r_1^2$. 另一方面, $O_1 X_1 \\cdot O_1 P$ 也是点 $O_1$ 对圆 $\\Gamma$ 的幕, 则 $r_1^2=O_1 X_1 \\cdot O_1 P=O_1 O^2-r^2 \\Rightarrow r^2=O O_1^2-r_1^2$. 因此, $r^2$ 是点 $O$ 对圆 $\\Gamma_1$ 的幕.\n同理, $r^2$ 也是点 $O$ 对圆 $\\Gamma_2 、 \\Gamma_3$ 的幂.\n综上, $O$ 是所给定的三个圆 $\\Gamma_1 、 \\Gamma_2 、 \\Gamma_3$ 的根心.\n因为点 $O$ 对这三个圆的幂的平方根 $r$ 与点 $P$ 的选取无关, 所以, 所有独特的点都在圆 $\\Gamma$ 上.", + "remark": "注:: 若三个圆的根心位于无穷远点 (相应的三条根轴两两平行), 则在平面上不存在独特的点, 这与题目中的论述是相容的.", + "figures": [ + "./images/volume7/figures/fig-c5a11.png" + ] +} \ No newline at end of file diff --git a/processed_dataset/proof/1873.json b/processed_dataset/proof/1873.json new file mode 100644 index 0000000000000000000000000000000000000000..068803f4a1be066149a4c1ceaf5681f9ec65f473 --- /dev/null +++ b/processed_dataset/proof/1873.json @@ -0,0 +1,10 @@ +{ + "source_file": "./raw_volume-zh/volume7/exercise5.tex", + "problem_type": "proof", + "problem": "问题12. 等腰 $\\triangle A B C, A B=A C, P$ 在边 $B C$ 的延长线上, $X$ 和 $Y$ 分别是直线 $A B$ 和 $A C$ 上的点.\n$P X / / A C, P Y / / A B, T$ 是 $\\triangle A B C$ 外接圆上弧 $\\overparen{B C}$ 的中点.\n证明: $P T \\perp X Y$.", + "solution": "证明: 如图(), 设 $M$ 和 $N$ 分别是 $T$ 在 $P X$ 和 $P Y$ 上的正交投影, 可以得到 $\\frac{P Y}{A B}=\\frac{P C}{B C}, P N=P B \\sin \\frac{A}{2}$. 所以, $P N \\cdot P Y=P B \\cdot P C \\cdot \\frac{A B}{B C} \\sin \\frac{A}{2}$. 同理可得 $P M \\cdot P X=P B \\cdot P C \\cdot \\frac{A C}{B C} \\sin \\frac{A}{2}$. 由于 $A B=A C$, 所以, $P X \\cdot P M=P N \\cdot P Y$. 因为 $M$ 和 $N$ 分别在以 $T X$ 和 $T Y$ 为直径的圆上,故点 $P$ 在分别以 $T X$ 和 $T Y$ 为直径的两圆的根轴上.\n设 $K$ 是分别以 $T X$ 和 $T Y$ 为直径的两圆的另外一个交点, 于是, $T 、 K$ 和 $P$ 三点共线.\n又 $\\overparen{Y K T}=\\overparen{X K T}=\\frac{\\pi}{2}$, 所以, $P T \\perp X Y$. 这样就证明了结论.", + "remark": "", + "figures": [ + "./images/volume7/figures/fig-c5a12.png" + ] +} \ No newline at end of file diff --git a/processed_dataset/proof/1874.json b/processed_dataset/proof/1874.json new file mode 100644 index 0000000000000000000000000000000000000000..ba27a21af0c7b704f45757c4219b8f161bcfeab8 --- /dev/null +++ b/processed_dataset/proof/1874.json @@ -0,0 +1,10 @@ +{ + "source_file": "./raw_volume-zh/volume7/exercise5.tex", + "problem_type": "proof", + "problem": "问题13. 已知非等腰锐角 $\\triangle A B C, A A_1 、 B B_1$ 是它的两条高, 又线段 $A_1 B_1$ 与平行于 $A B$ 的中位线相交于点 $C^{\\prime}$. 证明: 经过 $\\triangle A B C$ 的外心和垂心的直线与直线 $C C^{\\prime}$ 垂直.", + "solution": "证明: 如图(), 在 $\\triangle A B C$ 中, 分别将边 $B C$ 、 $C A$ 的中点记作 $A_0 、 B_0$, 将三角形的垂心记作 $H$, 外心记作 $O$. 因为点 $A 、 B 、 A_1 、 B_1$ 位于同一圆周上 ( $A B$ 为其直径), 所以, $\\angle C B_1 A_1= \\angle C B A=\\angle C A_0 B_0$. 故点 $A_0 、 B_0 、 A_1 、 B_1$ 位于同一圆周 $W_1$ 上.\n将以 $C H$ 为直径的圆周记作 $W_2$, 将以 $C O$ 为直径的圆周记作 $W_3$. 易知, 点\n$A_1 、 B_1$ 位于圆周 $W_2$ 上, 而点 $A_0 、 B_0$ 位于圆周 $W_3$ 上.\n因此, 点 $C^{\\prime}$ 关于圆 $W_1$ 和圆 $W_2$ 有相同的幂, 关于圆 $W_1$ 和圆 $W_3$ 也有相同的幕.\n从而, 点 $C^{\\prime}$ 关于圆 $W_2$ 和圆 $W_3$ 有相同的幕, 即位于它们的根轴之上.\n所以, 直线 $C C^{\\prime}$ 就是圆 $W_2$ 和圆 $W_3$ 的根轴.\n故 $C C^{\\prime}$ 垂直于这两个圆的圆心连线.\n又圆 $W_2$ 和圆 $W_3$ 的圆心分别为线段 $C H$ 和 $C O$ 的中点, 它们的连线平行于直线 $O H$, 则 $O H \\perp C C^{\\prime}$.", + "remark": "", + "figures": [ + "./images/volume7/figures/fig-c5a13.png" + ] +} \ No newline at end of file diff --git a/processed_dataset/proof/1875.json b/processed_dataset/proof/1875.json new file mode 100644 index 0000000000000000000000000000000000000000..6fdb04987f8cbe2383d78c487f2d99aba91f37cc --- /dev/null +++ b/processed_dataset/proof/1875.json @@ -0,0 +1,10 @@ +{ + "source_file": "./raw_volume-zh/volume7/exercise5.tex", + "problem_type": "proof", + "problem": "问题14. $\\triangle A B C$ 中, $\\odot I_1 、 \\odot I_2 、 \\odot I_3$ 分别是 $\\angle A 、 \\angle B 、 \\angle C$ 所对的旁切圆, $I 、 G$ 是 $\\triangle A B C$ 的内心、重心, 求证: $\\odot I_1 、 \\odot I_2 、 \\odot I_3$ 的根心在 $I G$ 上.", + "solution": "证明: 如图(), 作 $I_2 H_2 \\perp B C, I_3 H_3 \\perp B C$, 垂足分别为 $H_2 、 H_3$. 熟知 $B H_2=C_3=\\frac{A B+B C+C A}{2}$, 取 $B C$ 中点, 则 $M H_2=M_3$. 所以 $M$ 在 $\\odot I_2 、 \\odot I_3$ 根轴上.\n熟知 $A 、 G 、 M$ 共线且 $A G=2 G M$. 延长 $I G$ 至 $T$ 使 $I G=2 G T$, 则 $\\triangle A G I \\backsim \\triangle M G T$. 从而 $\\angle G M T= \\angle G A I$, 则 $M T / / A I$. 又 $A I \\perp I_2 I_3$, 所以 $M T \\perp I_2 I_3$, 故 $M T$ 为 $\\odot I_2 、 \\odot I_3$ 的根轴, $T$ 在根轴上.\n同理 $T$ 在 $\\odot I_1$ 与 $\\odot O_2 、 \\odot I_1$ 与 $\\odot O_3$ 的根轴上.\n故 $T$ 为三圆的根心, 且 $T$ 在 $I G$ 上,得证.", + "remark": "", + "figures": [ + "./images/volume7/figures/fig-c5a14.png" + ] +} \ No newline at end of file diff --git a/processed_dataset/proof/1876.json b/processed_dataset/proof/1876.json new file mode 100644 index 0000000000000000000000000000000000000000..2f2bff2e2f95ac29ece578e4d5284decbc14b8e4 --- /dev/null +++ b/processed_dataset/proof/1876.json @@ -0,0 +1,10 @@ +{ + "source_file": "./raw_volume-zh/volume7/exercise5.tex", + "problem_type": "proof", + "problem": "问题15. $A B 、 A C$ 为 $\\odot O$ 切线, $A D E$ 为一条割线, $M$ 为 $D E$ 中点, $P$ 为一动点, 满足 $M 、 O 、 P$ 三点共线, $\\odot P$ 为以 $P$ 点为圆心, $P D$ 为半径的圆.\n证明: $C$ 点在 $\\triangle B M P$ 外接圆与 $\\odot P$ 的根轴上.", + "solution": "证明: 如图(), 作 $P R \\perp A C$, 其延长线交 $B C$ 延长线于 $S$, 再过 $A$ 作 $A N \\perp B C$, 则 $N$ 为 $B C$ 的中点, 且 $\\triangle A C N \\backsim \\triangle S C R \\Rightarrow C B \\cdot C S=2 C A \\cdot C R$. 因为 $\\angle O M A=\\angle O B A=\\angle O C A=90^{\\circ}$. 所以 $A$ 、 $C 、 O 、 M 、 B$ 五点共圆.\n则 $\\angle B M P=\\angle B M A+ 90^{\\circ}=\\angle B C A+90^{\\circ}=180^{\\circ}-\\angle R S C$. 故 $B 、 M$ 、 $P 、 S$ 四点共圆.\n$C$ 点对 $\\triangle B M P$ 外接圆的幂为: $-C B \\cdot C S=-2 C A \\cdot C R$. 又因为 $P A^2-A O^2=\\left(A M^2+M P^2\\right)-\\left(A M^2+M O^2\\right)=M P^2-M O^2 . P D^2-O D^2=\\left(D M^2+\\right. \\left.M P^2\\right)-\\left(D M^2+M O^2\\right)=M P^2-M O^2$. 所以 $P A^2-A O^2=P D^2-D O^2 \\Rightarrow P A^2-P D^2=A O^2-D O^2 \\cdots$ (1). 而 $A$ 对 $\\odot O$ 的幕有: $A O^2-D O^2=A D \\cdot A E$.\n从而 $C$ 对 $\\odot P$ 的幂为: $C P^2-P D^2 \\stackrel{\\text { 由 (1) }}{=} C P^2-\\left[A P^2-\\left(A O^2-D O^2\\right)\\right]=C P^2- A P^2+A D \\cdot A E=C P^2-A P^2+A C^2=\\left(C R^2+R P^2\\right)-\\left(A R^2+R P^2\\right)+ A C^2=C R^2-(A C+C R)^2+A C^2=-2 C A \\cdot C R$. 所以点 $C$ 对 $\\odot P$ 的幕等于 $C$ 到 $\\triangle B M P$ 外接圆的幂.\n故由根轴定理知, $C$ 点在上述两圆根轴上.", + "remark": "", + "figures": [ + "./images/volume7/figures/fig-c5a15.png" + ] +} \ No newline at end of file diff --git a/processed_dataset/proof/1877.json b/processed_dataset/proof/1877.json new file mode 100644 index 0000000000000000000000000000000000000000..d32695acde9f225e2e7fe4404e0f7683f6ea9275 --- /dev/null +++ b/processed_dataset/proof/1877.json @@ -0,0 +1,10 @@ +{ + "source_file": "./raw_volume-zh/volume7/exercise5.tex", + "problem_type": "proof", + "problem": "问题16. 已知 $Q$ 为以 $A B$ 为直径的圆上的一点, $Q \\neq A 、 B, Q$ 在 $A B$ 上的投影为 $H$, 以 $Q$ 为圆心 $Q H$ 为半径的圆与以 $A B$ 为直径的圆交于点 $C 、 D$. 证明: $C D$ 平分线段 $Q H$.", + "solution": "证明: 如图(), 设直线 $Q H$ 与 $\\odot Q$ 交于点 $S$, 与 $\\odot O$ 交于点 $T$, 设 $C D$ 与 $Q H$ 交于点 $M$, 则 $M$ 在两圆根轴 $C D$ 上, 故 $M$ 关于 $\\odot Q, \\odot O$ 等幕, 即 $M H$. $M S=M C \\cdot M D=M Q \\cdot M T$, 又由垂径定理知 $Q H=H T, Q H=Q S$, 代入上式知 $M H \\cdot(M Q+ Q H)=M Q \\cdot(M H+Q H) \\Rightarrow M H=M Q$, 所以 $M$ 平分 $Q H$, 即 $C D$ 平分线段 $Q H$.", + "remark": "", + "figures": [ + "./images/volume7/figures/fig-c5a16.png" + ] +} \ No newline at end of file diff --git a/processed_dataset/proof/1878.json b/processed_dataset/proof/1878.json new file mode 100644 index 0000000000000000000000000000000000000000..dea39132e51404b461413e345d2a142873d984f7 --- /dev/null +++ b/processed_dataset/proof/1878.json @@ -0,0 +1,10 @@ +{ + "source_file": "./raw_volume-zh/volume7/exercise5.tex", + "problem_type": "proof", + "problem": "问题18. 凸四边形 $A B C D$ 的两条对角线交于点 $O, \\triangle A O B$ 和 $\\triangle C O D$ 的重心分别为 $M_1$ 和 $M_2, \\triangle B O C$ 和 $\\triangle A O D$ 的垂心分别为 $H_1$ 和 $H_2$. 证明: $M_1 M_2 \\perp \\mathrm{H}_1 \\mathrm{H}_2$.", + "solution": "如图(), 作 $\\triangle A O D$ 的两条高 $A A_1$ 和 $D D_1$, 作 $\\triangle B O C$ 的两条高 $B B_1$ 和\n$C C_1$. 因为 $\\angle A A_1 B=90^{\\circ}=\\angle B B_1 A$, 所以, $A 、 B 、 B_1$ 、 $A_1$ 四点共圆且圆心为 $A B$ 的中点 $E$. 同理, $C_1 、 C 、 D$ 、 $D_1$ 四点共圆且圆心为 $C D$ 的中点 $F$. 因此, $E F$ 为 $\\odot E$ 和 $\\odot F$ 的连心线.\n又 $A 、 D_1 、 A_1 、 D$ 四点共圆, 则有 $H_2 A \\cdot H_2 A_1=H_2 D_1 \\cdot H_2 D$. 由于 $H_2 A \\cdot H_2 A_1$ 和 $H_2 D_1 \\cdot H_2 D$ 恰分别为点 $H_2$ 关于 $\\odot E$ 和 $\\odot F$ 的幕, 所以, 点 $H_2$ 在 $\\odot E$ 和 $\\odot F$ 的根轴上.\n同理, 点 $H_1$ 也在这条根轴上.\n故直线 $H_1 H_2$ 就是 $\\odot E$ 和 $\\odot F$ 的根轴.\n从而 $H_1 H_2 \\perp E F$. 又 $M_1 M_2 / / E F$, 所以,\n$$\nM_1 M_2 \\perp H_1 H_2 .\n$$", + "remark": "", + "figures": [ + "./images/volume7/figures/fig-c5a18.png" + ] +} \ No newline at end of file diff --git a/processed_dataset/proof/1879.json b/processed_dataset/proof/1879.json new file mode 100644 index 0000000000000000000000000000000000000000..37449ee43c576a728bf9c4c6ee9a97a862a7b320 --- /dev/null +++ b/processed_dataset/proof/1879.json @@ -0,0 +1,10 @@ +{ + "source_file": "./raw_volume-zh/volume7/exercise5.tex", + "problem_type": "proof", + "problem": "问题19. 在凸五边形 $A B C D E$ 中, $A B=B C, \\angle B C D=\\angle E A B=90^{\\circ}, P$ 为形内一点, 使得 $A P \\perp B E, C P \\perp B D$. 证明: $B P \\perp D E$.", + "solution": "证法 1: 如图(), 过点 $P$ 作 $P H \\perp D E$ 于点 $H$. 因为\n$$\n\\angle P F D=\\angle P G E=90^{\\circ}=\\angle P H D=\\angle P H E,\n$$\n所以, $F 、 D 、 H 、 P$ 和 $P 、 H 、 E 、 G$ 分别四点共圆, 记两圆为 $\\odot M_1$ 和 $\\odot M_2$. 又 $B F \\cdot B D=B C^2=B A^2=B G \\cdot B E$, 所以, $F$ 、 $D 、 E 、 G$ 四点共圆, 记此圆为 $\\odot M_3$.\n易见, $\\odot M_1 、 \\odot M_2 、 \\odot M_3$ 两两之间的公共弦恰为 $P H 、 E G 、 F D$. 由根心定理知, 这三条根轴交于一点.\n又已知直线 $D F$ 和 $E G$ 交于点 $B$, 因此, 直线 $P H$ 过点 $B$.\n由 $P H \\perp D E$, 知 $B P \\perp D E$.", + "remark": "", + "figures": [ + "./images/volume7/figures/fig-c5a19.png" + ] +} \ No newline at end of file diff --git a/processed_dataset/proof/1880.json b/processed_dataset/proof/1880.json new file mode 100644 index 0000000000000000000000000000000000000000..d2a5921e68852a5749b7912337ecf3c96f408176 --- /dev/null +++ b/processed_dataset/proof/1880.json @@ -0,0 +1,8 @@ +{ + "source_file": "./raw_volume-zh/volume7/exercise5.tex", + "problem_type": "proof", + "problem": "问题19. 在凸五边形 $A B C D E$ 中, $A B=B C, \\angle B C D=\\angle E A B=90^{\\circ}, P$ 为形内一点, 使得 $A P \\perp B E, C P \\perp B D$. 证明: $B P \\perp D E$.", + "solution": "证法 2 : 记 $B P$ 的中点为 $O$.\n因为 $\\angle B F P=90^{\\circ}=\\angle B G P$, 所以, $B 、 F 、 P 、 G$ 四点共圆且圆心为点 $O$.\n又因为 $B A=B C, \\angle B C D=90^{\\circ}=\\angle B A E$, 故以点 $B$ 为圆心 $B C$ 为半径的 $\\odot B$ 过点 $A$, 且直线 $D C$ 和 $E A$ 都是 $\\odot B$ 的切线, 切点分别为 $C$ 和 $A$.\n所以, $D C^2=D F \\cdot D B$.\n故点 $D$ 在 $\\odot O$ 与 $\\odot B$ 的根轴上.\n同理, 点 $E$ 在 $\\odot O$ 与 $\\odot B$ 的根轴上.\n因此, 直线 $D E$ 为 $\\odot O$ 与 $\\odot B$ 的根轴.\n则 $B O \\perp D E$, 即 $B P \\perp D E$.", + "remark": "", + "figures": [] +} \ No newline at end of file diff --git a/processed_dataset/proof/1881.json b/processed_dataset/proof/1881.json new file mode 100644 index 0000000000000000000000000000000000000000..6bab0278fd3b55854c6ee7ec32b8c0dd2196b623 --- /dev/null +++ b/processed_dataset/proof/1881.json @@ -0,0 +1,10 @@ +{ + "source_file": "./raw_volume-zh/volume7/exercise5.tex", + "problem_type": "proof", + "problem": "问题20. 在 $\\angle A O B$ 内部取一点 $C$, 过点 $C$ 作 $C D \\perp O A$ 于点 $D$, 作 $C E \\perp O B$ 于点 $E$, 再过点 $D$ 作 $D N \\perp O B$ 于点 $N$, 过点 $E$ 作 $E M \\perp O A$ 于点 $M$. 证明: $O C \\perp M N$.", + "solution": "证明: 如图(), 过点 $C$ 作 $C H \\perp M N$ 于点 $H$.\n因为\n$$\n\\angle C D M=\\angle C E N=90^{\\circ},\n$$\n所以, $C 、 D 、 M 、 H$ 和 $C 、 H 、 N 、 E$ 分别四点共圆, 记两圆为 $\\odot O_1$ 和 $\\odot O_2$. 由\n$$\n\\angle D M E=90^{\\circ}=\\angle D N E,\n$$\n知 $D 、 M 、 N 、 E$ 四点共圆, 记之为 $\\odot O_3$.\n易见, 直线 $C H 、 E N 、 D M$ 恰为 $\\odot O_1 、 \\odot O_2 、 \\odot O_3$ 两两之间的三条根轴.\n又因前两条直线交于点 $O$, 故由根心定理知直线 $C H$ 过点 $O$, 即 $C 、 H 、 O$ 三点共线.\n又 $C H \\perp M N$, 所以, $O C \\perp M N$.", + "remark": "", + "figures": [ + "./images/volume7/figures/fig-c5a20.png" + ] +} \ No newline at end of file diff --git a/processed_dataset/proof/1882.json b/processed_dataset/proof/1882.json new file mode 100644 index 0000000000000000000000000000000000000000..efb76003fa04a6274042fd7042987cf736a6f007 --- /dev/null +++ b/processed_dataset/proof/1882.json @@ -0,0 +1,10 @@ +{ + "source_file": "./raw_volume-zh/volume7/exercise5.tex", + "problem_type": "proof", + "problem": "问题21. 设锐角 $\\triangle A B C$ 的外心为 $O, \\triangle B O C$ 的外心为 $T$, 点 $M$ 为边 $B C$ 的中点, 在边 $A B 、 A C$ 上分别取点 $D 、 E$, 使得 $\\angle A D M=\\angle A E M=\\angle B A C$. 证明: $A T \\perp D E$.", + "solution": "证明: 如图(), 由 $O$ 是 $\\triangle A B C$ 的外心, $T$ 是 $\\triangle B O C$ 的外心知, $O 、 M 、 T$ 三点共线, 且 $O T \\perp B C$.\n延长 $D M 、 A C$ 交于点 $G$, 延长 $E M 、 A B$ 交于点 $F$, 连结 $F T 、 B T 、 G T$. 于是, 有\n$$\n\\begin{aligned}\n\\angle B T O & =2 \\angle B C O=180^{\\circ}-\\angle B O C \\\\\n& =180^{\\circ}-2 \\angle B A C=\\angle A F E .\n\\end{aligned}\n$$\n故 $B 、 F 、 T 、 M$ 四点共圆.\n则 $\\angle B F T=180^{\\circ}-\\angle B M T=90^{\\circ}$.\n同理, $\\angle C G T=90^{\\circ}$.\n过点 $T$ 作 $T H \\perp D E$ 于点 $H$, 于是, $D 、 F 、 T 、 H$ 和 $H 、 T 、 G 、 E$ 分别四点共圆, 记两圆为 $\\odot S_1$ 和 $\\odot S_2$.\n又 $\\angle F D G=180^{\\circ}-\\angle A D G=180^{\\circ}-\\angle A E F=\\angle F E G$,\n所以, $D 、 F 、 G 、 E$ 四点共圆, 记之为 $\\odot S_3$.\n由于直线 $T H 、 G E 、 D F$ 恰为 $\\odot S_1 、 \\odot S_2 、 \\odot S_3$ 两两之间的三条根轴, 且 $G E$ 与 $D F$ 交于点 $A$, 故由根心定理知 $T H$ 过点 $A$.\n因为 $T H \\perp D E$, 所以, $A T \\perp D E$.", + "remark": "", + "figures": [ + "./images/volume7/figures/fig-c5a21.png" + ] +} \ No newline at end of file diff --git a/processed_dataset/proof/1883.json b/processed_dataset/proof/1883.json new file mode 100644 index 0000000000000000000000000000000000000000..5e4678888ec62d4bc8d7a1281cf9cc76840f0476 --- /dev/null +++ b/processed_dataset/proof/1883.json @@ -0,0 +1,10 @@ +{ + "source_file": "./raw_volume-zh/volume7/exercise6.tex", + "problem_type": "proof", + "problem": "问题1. 已知梯形 $A B C D$ 的对角线 $A C 、 B D$ 交于点 $P$, 点 $Q$ 在平行线 $B C 、 A D$ 之间, 满足 $\\angle A Q D=\\angle C Q B$, 且 $P 、 Q$ 在直线 $C D$ 的两侧.\n证明: $\\angle B Q P=\\angle D A Q$.", + "solution": "证明: 设 $t=\\frac{A D}{B C}$, 以 $P$ 为位似中心, $-t$ 为位似比的位似变换 $h$ 将 $\\triangle P B C$ 变换到 $\\triangle P D A$.\n如图(), 设 $Q^{\\prime}=h(Q)$, 则 $Q 、 P 、 Q^{\\prime}$ 三点共线.\n由于点 $P 、 Q$ 在边 $A D$ 的同侧, 也在边 $B C$ 的同侧, 于是, 点 $Q^{\\prime} 、 P$ 也在边 $h(B C)=D A$ 的同侧.\n从而, 点 $Q 、 Q^{\\prime}$ 也在边 $A D$ 的同侧.\n此外, 点 $Q 、 C$\n在 $B D$ 的同侧, 点 $Q^{\\prime} 、 A$ 在 $B D$ 的另一侧.\n由位似变换 $h$ 知, $\\angle A Q^{\\prime} D= \\angle C Q B=\\angle A Q D$. 于是, $A 、 Q^{\\prime} 、 Q 、 D$ 四点共圆.\n从而, $\\angle D A Q=\\angle D Q^{\\prime} Q= \\angle D Q^{\\prime} P=\\angle B Q P$.", + "remark": "", + "figures": [ + "./images/volume7/figures/fig-c6a1.png" + ] +} \ No newline at end of file diff --git a/processed_dataset/proof/1884.json b/processed_dataset/proof/1884.json new file mode 100644 index 0000000000000000000000000000000000000000..97ec452536b1990d75f9694ec1893323e8669bc8 --- /dev/null +++ b/processed_dataset/proof/1884.json @@ -0,0 +1,10 @@ +{ + "source_file": "./raw_volume-zh/volume7/exercise6.tex", + "problem_type": "proof", + "problem": "问题2. 在 $\\triangle A B C$ 中, $\\angle B<\\angle C$, 设经过点 $B 、 C$ 且与 $A C$ 切于点 $C$ 的圆为 $\\odot O$, 直线 $A B 、 C O$ 分别与 $\\odot O$ 交于点 $D(\\neq B) 、 P(\\neq C)$. 过点 $P$ 作 $A O$ 的平行线与 $A C$ 交于点 $E$. 直线 $E B$ 交 $\\odot O$ 于点 $L(\\neq B), B D$ 的中垂线与 $A C$ 交于点 $F, L F$ 交 $C D$ 于点 $K$. 证明: $E K / / C L$.", + "solution": "证明: 如图(), 设过点 $E$ 平行于 $C L$ 的直线与 $C D$ 交于点 $K^{\\prime}$.\n由 $\\angle E B D=\\angle D C L=\\angle E K^{\\prime} D$, 知 $B 、 D$ 、 $E 、 K^{\\prime}$ 四点共圆.\n设该圆为 $\\odot O_1$. 注意到 $A O$ 是 $\\triangle C E P$ 的中位线, 且 $C E$ 是切线, 则 $A E^2= A C^2=A D \\cdot A B$, 有 $A E$ 切 $\\odot O_1$ 于点 $E$. 考虑从 $\\odot O_1$ 到 $\\odot O$ 的位似变换, 位似中心为 $F$, 点 $E 、 O_1$ 分别对应点 $C 、 O, K^{\\prime}$ 对应点 $L$. 因此, $F 、 K^{\\prime} 、 L$ 三点共线,故 $K=K^{\\prime}$.", + "remark": "", + "figures": [ + "./images/volume7/figures/fig-c6a2.png" + ] +} \ No newline at end of file diff --git a/processed_dataset/proof/1885.json b/processed_dataset/proof/1885.json new file mode 100644 index 0000000000000000000000000000000000000000..2df5e162736f43b3d72a555cd61a4219498936d0 --- /dev/null +++ b/processed_dataset/proof/1885.json @@ -0,0 +1,10 @@ +{ + "source_file": "./raw_volume-zh/volume7/exercise6.tex", + "problem_type": "proof", + "problem": "问题3. 点 $O$ 是平行四边形 $A B C D$ 内的一个点, 使得 $\\angle A O B+\\angle C O D=180^{\\circ}$. 证明: $\\angle O B C=\\angle O D C$.", + "solution": "证明: 如图(), 考虑将点 $D$ 映射到点 $A$ 的平移.\n此平移也将点 $O$ 映射到 $O^{\\prime}$, 且 $\\overrightarrow{O O^{\\prime}}=\\overrightarrow{D A}$. 由于 $\\overrightarrow{C B}=\\overrightarrow{D A}$, 故也将点 $C$ 映射到点 $B$.\n平移保持角度不变, 所以 $\\angle A O^{\\prime} B=\\angle D O C= 180^{\\circ}-\\angle A O B$.\n因此, $A O B O^{\\prime}$ 是一个圆内接四边形, 进而有\n$$\n\\angle O D C=\\angle O^{\\prime} A B=\\angle O^{\\prime} O B .\n$$\n又因 $O^{\\prime} O$ 平行于 $B C$, 所以 $\\angle O^{\\prime} O B=\\angle O B C$, 故有\n$\\angle O D C=\\angle O B C$.", + "remark": "", + "figures": [ + "./images/volume7/figures/fig-c6a3.png" + ] +} \ No newline at end of file diff --git a/processed_dataset/proof/1886.json b/processed_dataset/proof/1886.json new file mode 100644 index 0000000000000000000000000000000000000000..28deb4cd9a558b558a8f17b16a91ee0a751ea986 --- /dev/null +++ b/processed_dataset/proof/1886.json @@ -0,0 +1,11 @@ +{ + "source_file": "./raw_volume-zh/volume7/exercise6.tex", + "problem_type": "proof", + "problem": "问题4. 如图(), 圆 $W_1 、 W_2$ 的圆心分别为 $O_1 、 O_2$, 两圆相交于点 $A 、 B$. 由点 $A$ 分别向圆 $W_1 、 W_2$ 作切线 $l_1 、 l_2$, 点 $T_1 、 T_2$ 分别位于圆 $W_1 、 W_2$ 上, 使得 $\\angle T_1 O_1 A=\\angle A O_2 T_2$. 圆 $W_1$ 上过点 $T_1$ 的切线与 $l_2$ 相交于点 $M_1$, 圆 $W_2$ 上过点 $T_2$ 的切线与 $l_1$ 相交于点 $M_2$. 证明: 线段 $M_1 M_2$ 的中点位于一条不依赖于点 $T_1 、 T_2$ 位置的直线上.", + "solution": "证明: 如图(), 以 $A$ 为中心, 作系数为 $k(k>0)$ 的同位相似,将圆 $W_1$ 变为与圆 $W_2$ 相等的圆 $W_1^{\\prime}$. 用带撤的同一字母 (连同原来的下标) 表示各个点和各条直线在该变换之下的像,如图.\n设 $B_1$ 是圆 $W_2$ 与圆 $W_1^{\\prime}$ 的 (不同于 $A$ ) 第二个交点.\n圆 $W_1^{\\prime}$ 与圆 $W_2$ 关于直线 $A B_1$ 相互对称.\n在此对称之下, $T_1^{\\prime}$ 变为 $T_2, l_1^{\\prime}$ 变为 $l_2, M_1^{\\prime}$ 变为 $M_2$. 从而, $A M_2=A M_1^{\\prime}=k A M_1(k$ 不依赖于点 $T_1$ 和 $T_2$ 的位置). 于是, 不论点 $T_1 、 T_2$ 的位置如何变化, 所得到的 $\\triangle A M_1 M_2$ 都彼此为同位相似 (因为 $M_2, M_1$ 分别位于直线 $l_1 、 l_2$ 上, 它们都经过点 $A$, 且位于直线 $A B_1$ 的不同侧, 而比值 $A M_2$ : $A M_1$ 为常数). 由此即知, 线段 $M_1 M_2$ 的中点位于一条固定的经过点 $A$ 的直线上.", + "remark": "", + "figures": [ + "./images/volume7/figures/fig-c6p4.png", + "./images/volume7/figures/fig-c6a4.png" + ] +} \ No newline at end of file diff --git a/processed_dataset/proof/1887.json b/processed_dataset/proof/1887.json new file mode 100644 index 0000000000000000000000000000000000000000..7780c517dbc64c34ba0bc5aca4c450ffb6cf8cf3 --- /dev/null +++ b/processed_dataset/proof/1887.json @@ -0,0 +1,10 @@ +{ + "source_file": "./raw_volume-zh/volume7/exercise6.tex", + "problem_type": "proof", + "problem": "问题5. 圆内接四边形 $A B C D$ 对角线 $B D$ 上的点 $K$ 满足 $\\angle A K B=\\angle A D C, I 、 I^{\\prime}$ 分别为 $\\triangle A C D 、 \\triangle A B K$ 的内心, 线段 $I I^{\\prime}$ 与 $B D$ 交于点 $X$. 证明 : $A 、 X 、 I 、 D$ 四点共圆.", + "solution": "证明: 如图(), 由 $\\angle A K B=\\angle A D C, \\angle A B K= \\angle A C D$, 知 $\\triangle A K B \\backsim \\triangle A D C . \\triangle A K B$ 绕点 $A$ 旋转 $\\angle B A C$ 再放缩变为 $\\triangle A D C$, 因此, $\\angle I A I^{\\prime}=\\angle B A C$, $\\frac{A I}{A I}=\\frac{A C}{A B}$. 故 $\\triangle A I I^{\\prime} \\backsim \\triangle A C B$. 于是, $\\angle A I X= \\angle A C B=\\angle A D X$. 从而, $A 、 X 、 I 、 D$ 四点共圆.", + "remark": "", + "figures": [ + "./images/volume7/figures/fig-c6a5.png" + ] +} \ No newline at end of file diff --git a/processed_dataset/proof/1888.json b/processed_dataset/proof/1888.json new file mode 100644 index 0000000000000000000000000000000000000000..f26e6319c32d47f997efadbca0ad42ff9fe252d8 --- /dev/null +++ b/processed_dataset/proof/1888.json @@ -0,0 +1,10 @@ +{ + "source_file": "./raw_volume-zh/volume7/exercise6.tex", + "problem_type": "proof", + "problem": "问题7. $\\triangle A B C$ 的外接圆的圆心为 $O, A^{\\prime}$ 是边 $B C$ 的中点, $A A^{\\prime}$ 与外接圆交于点 $A^{\\prime \\prime}, A^{\\prime} Q_a \\perp A O$, 点 $Q_a$ 在 $A O$ 上, 过点 $A^{\\prime \\prime}$ 的外接圆的切线与 $A^{\\prime} Q_a$ 相交于点 $P_a$. 用同样的方式, 可以构造点 $P_b$ 和 $P_c$. 证明: $P_a 、 P_b 、 P_c$ 三点共线.", + "solution": "证明: 可以证明它们都在 $\\odot O$ 与九点圆的根轴上.\n如图(), 把 $\\triangle A B C$ 位似变换到 $\\triangle A^{\\prime} B^{\\prime} C^{\\prime}$. $\\triangle A B C$ 的重心为位似中心, 位似比为 $-\\frac{1}{2}$. 在这种变换下, $A O$ 变成了 $A^{\\prime} N$, 其中 $N$ 是九点圆的圆心.\n所以, $A^{\\prime} N / / A O, A^{\\prime} P_a \\perp A^{\\prime} N$. 故 $A^{\\prime} P_a$. 是九点圆的切线.\n易知 $\\angle O A B+\\angle C=90^{\\circ}$, 则 $\\angle B A A^{\\prime}+ \\angle A^{\\prime} A O+\\angle C=90^{\\circ}$ (不妨设 $A B \\leqslant A C$ ). 又\n$\\angle P_a A^{\\prime \\prime} A^{\\prime}=\\angle B A A^{\\prime}+\\angle C, \\angle P_a A^{\\prime} A^{\\prime \\prime}=90^{\\circ}- \\angle A^{\\prime} A O$, 所以, $\\angle P_a A^{\\prime \\prime} A^{\\prime}=\\angle P_a A^{\\prime} A^{\\prime \\prime}$. 故 $A^{\\prime} P_a= A^{\\prime \\prime} P_a$. 所以, $P_a$ 在 $\\odot O$ 与九点圆的根轴上.\n同理, $P_b 、 P_c$ 也在 $\\odot O$ 与九点圆的根轴上.", + "remark": "", + "figures": [ + "./images/volume7/figures/fig-c6a7.png" + ] +} \ No newline at end of file diff --git a/processed_dataset/proof/1889.json b/processed_dataset/proof/1889.json new file mode 100644 index 0000000000000000000000000000000000000000..ba01ba8f489c82e08c278c5a28951a1de649134c --- /dev/null +++ b/processed_dataset/proof/1889.json @@ -0,0 +1,10 @@ +{ + "source_file": "./raw_volume-zh/volume7/exercise6.tex", + "problem_type": "proof", + "problem": "问题8. 学设 $E 、 F$ 分别为正方形 $A B C D$ 的边 $B C 、 C D$ 上的点, $A E 、 A F$ 分别与对角线 $B D$ 交于 $P 、 Q$ 两点, 且 $B E+D F=E F$. 求证: 五边形 $P E C F Q$ 内接于圆.", + "solution": "证明: 如图(), 以 $A$ 为旋转中心逆时针旋转 $90^{\\circ}$, 则 $B \\rightarrow D$, 设 $E \\rightarrow E^{\\prime}$, 则 $A E^{\\prime}$ 垂直且等于 $A E$, $D E^{\\prime}$ 垂直且等于 $B E$, 因而 $E^{\\prime}$ 在 $C D$ 的延长线上, 所以 $B E+F D=D E^{\\prime}+F D=E^{\\prime} F$, 于是, $B E+D F=E F \\Leftrightarrow E^{\\prime} F=E F \\Leftrightarrow \\triangle A E^{\\prime} F \\cong \\triangle A E F \\Leftrightarrow \\angle E^{\\prime} A F=\\angle E A F \\Leftrightarrow \\angle E A F=\\frac{1}{2} \\angle E A E^{\\prime} \\Leftrightarrow \\angle E A F=45^{\\circ}$. 注意到\"任意一条直线与其像直线的交角等于旋转角\" $A F$. 是 $A E$ 在旋转下的像.\n故 $\\angle A E Q=45^{\\circ}$, 所以 $E Q \\perp Q F$; 同理 $E P \\perp P F$. 即 $P 、 E$ 、 $C, F$ 和 $E 、 C 、 F 、 Q$ 分别四点共圆, 从而五边形 $P E C F Q$ 内接于圆.", + "remark": "", + "figures": [ + "./images/volume7/figures/fig-c6a8.png" + ] +} \ No newline at end of file diff --git a/processed_dataset/proof/1890.json b/processed_dataset/proof/1890.json new file mode 100644 index 0000000000000000000000000000000000000000..85db49da95c40afd700bce63a4dcd2786722ad08 --- /dev/null +++ b/processed_dataset/proof/1890.json @@ -0,0 +1,10 @@ +{ + "source_file": "./raw_volume-zh/volume7/exercise6.tex", + "problem_type": "proof", + "problem": "问题10. 将一张正方形纸片 $A B C D$ 折叠, 使 $D$ 点重合于边 $B C$ 上一点 $D^{\\prime}, A$ 点折叠后的位置是 $A^{\\prime}, A B$ 与 $A^{\\prime} D^{\\prime}$ 交于 $E$, 设 $\\triangle B D^{\\prime} E$ 的内切圆半径为 $r$. 证明: $A^{\\prime} E=r$.", + "solution": "证明: 如图(), 设折痕所在直线为 $l$, 轴反射变换, 则 $A \\rightarrow A^{\\prime}, D \\rightarrow D^{\\prime}$; 再设 $B \\rightarrow B^{\\prime}, C \\rightarrow C^{\\prime}$, 因 $D^{\\prime} \\rightarrow D$, 而 $D^{\\prime}$ 在 $B C$ 上, 所以 $D$ 在 $B^{\\prime} C^{\\prime}$ 上, 又 $D$ 到 $A B$ 、 $B C 、 A^{\\prime} D^{\\prime}$ 的距离都等于正方形的边长, 所以 $D$ 为 $\\triangle B E D^{\\prime}$ 的旁心.\n因 $\\triangle B E D^{\\prime}$ 为直角三角形, 于是, $r=\\frac{1}{2}\\left(B E+B D^{\\prime}-E D^{\\prime}\\right)=\\frac{1}{2}\\left(B E+B D^{\\prime}+E D^{\\prime}-\\right. \\left.2 E D^{\\prime}\\right)=\\frac{1}{2}\\left(A B+B C-2 E D^{\\prime}\\right)=A B-E D^{\\prime}=A^{\\prime} D^{\\prime}-E D^{\\prime}=A^{\\prime} E$.", + "remark": "", + "figures": [ + "./images/volume7/figures/fig-c6a10.png" + ] +} \ No newline at end of file diff --git a/processed_dataset/proof/1891.json b/processed_dataset/proof/1891.json new file mode 100644 index 0000000000000000000000000000000000000000..01c1c8b60ceb30826460a6ee57efbccbc33e7850 --- /dev/null +++ b/processed_dataset/proof/1891.json @@ -0,0 +1,10 @@ +{ + "source_file": "./raw_volume-zh/volume7/exercise6.tex", + "problem_type": "proof", + "problem": "问题11. 设 $B 、 C$ 是线段 $A D$ 上的两点,且 $A B=C D$. 求证: 对于平面上任意一点 $P$, 都有 $P A+P D \\geqslant P B+P C$.", + "solution": "证明: 如图(), 设线段 $A D$ 的中点为 $M$, 以 $M$ 为旋转中心作中心反射, 则 $D \\rightarrow A, C \\rightarrow B$, 设 $P \\rightarrow P^{\\prime}$, 则 $P^{\\prime} A=P D, P^{\\prime} B=P C$. 因 $B$ 是 $\\triangle A P^{\\prime} P$ 内部的一点, 所以 $P A+P^{\\prime} A \\geqslant P B+P^{\\prime} B$, 故 $P A+P D \\geqslant P B+P C$.", + "remark": "", + "figures": [ + "./images/volume7/figures/fig-c6a11.png" + ] +} \ No newline at end of file diff --git a/processed_dataset/proof/1892.json b/processed_dataset/proof/1892.json new file mode 100644 index 0000000000000000000000000000000000000000..f73e7ce503ddb920af915219bb86a31c8b0165e3 --- /dev/null +++ b/processed_dataset/proof/1892.json @@ -0,0 +1,10 @@ +{ + "source_file": "./raw_volume-zh/volume7/exercise6.tex", + "problem_type": "proof", + "problem": "问题12. 点 $D$ 是 $\\triangle A B C$ 的外接圆的不包含点 $A$ 的弧 $\\overparen{B C}$ 上的一点, 且 $D \\neq B$, $D \\neq C$, 在射线 $B D$ 和 $C D$ 上分别取点 $E 、 F$, 使 $B E=-A C, C F=A B$. 再设 $M$ 是线段 $E F$ 的中点.\n证明: $\\angle B M C$ 是直角.", + "solution": "证明: 如图(), $M$ 为旋转中心作中心反射, 则 $E \\rightarrow F, F \\rightarrow E$; 设 $B \\rightarrow B^{\\prime}$, $C \\rightarrow C^{\\prime}$, 则 $B^{\\prime} C^{\\prime}=B C$, 则 $E C^{\\prime}=C F=A B$, 又 $E C^{\\prime} / / C F$, 所以 $\\angle B E C^{\\prime}= \\angle B D F=\\angle B A C$, 因为 $E B=A C$, 则 $\\triangle E C^{\\prime} B \\cong \\triangle A B C$, 所以 $B C^{\\prime}=B C$; 同理, $B^{\\prime} C=B C$. 因此, $B C^{\\prime} B^{\\prime} C$ 是一个菱形, 从而 $B B^{\\prime} \\perp C C^{\\prime}$, 故 $B M \\perp M C$ 成立.", + "remark": "", + "figures": [ + "./images/volume7/figures/fig-c6a12.png" + ] +} \ No newline at end of file diff --git a/processed_dataset/proof/1893.json b/processed_dataset/proof/1893.json new file mode 100644 index 0000000000000000000000000000000000000000..45ae9868568ec3145f1da84dd6be68c565fc9adc --- /dev/null +++ b/processed_dataset/proof/1893.json @@ -0,0 +1,10 @@ +{ + "source_file": "./raw_volume-zh/volume7/exercise6.tex", + "problem_type": "proof", + "problem": "问题13. 已知边长分别为 $a 、 b 、 c$ 的 $\\triangle A B C$ 内接于 $\\odot O, \\odot O_1$ 内切于 $\\odot O$, 切点 $T$ 在 $B C$ 弧上, 由点 $A 、 B 、 C$ 分别引 $\\odot O_1$ 的切线长顺次为 $\\alpha 、 \\beta 、 \\lambda$. 证明: $a \\alpha=b \\beta+c \\gamma$.", + "solution": "证明: 如图(), 设两圆相切于点 $T, A T, B T$, $C T$ 分别交小圆于 $A_1, B_1, C_1$. 则小圆与大圆关于点 $T$ 位似.\n设小圆、大圆半径之比是 $k$, 那么 $k$ 就是位似比.\n由圆幂定理: $\\alpha^2=A A_1 \\cdot A T, A T$ 与 $A_1 T$ 是位似变换 $F$ 的对应线段, 故 $A_1 T=k \\cdot A T$. 所以 $A A_1= (1-k) A T$, 即 $A T=\\frac{\\alpha}{\\sqrt{1-k}}$. 同理 $B T=\\frac{\\beta}{\\sqrt{1-k}}$, $C T=\\frac{\\gamma}{\\sqrt{1-k}}$. 由托勒密定理得 $a \\cdot A T=b \\cdot B T+c \\cdot C T$. 将三式代入得: $a \\alpha=b \\beta+c \\gamma$.", + "remark": "", + "figures": [ + "./images/volume7/figures/fig-c6a13.png" + ] +} \ No newline at end of file diff --git a/processed_dataset/proof/1894.json b/processed_dataset/proof/1894.json new file mode 100644 index 0000000000000000000000000000000000000000..a171b4d9d894ffbd20091ee2bc8fcfa91fc49b93 --- /dev/null +++ b/processed_dataset/proof/1894.json @@ -0,0 +1,10 @@ +{ + "source_file": "./raw_volume-zh/volume7/exercise6.tex", + "problem_type": "proof", + "problem": "问题14. 由 $\\triangle A B C$ 向外作 $\\triangle B C D$ 和 $\\triangle A C E$, 使得: $A E=B D$ 且 $\\angle B D C+\\angle A E C= 180^{\\circ}, F$ 是线段 $A B$ 上的一点满足 $\\frac{A F}{F B}=\\frac{D C}{C E}$. 证明: $\\frac{D E}{C D+C E}=\\frac{E F}{B C}=\\frac{F D}{A C}$.", + "solution": "证明:如图(), 作 $\\triangle A E C_1 \\cong \\triangle B D C$, $\\triangle B D C_2 \\cong \\triangle A E C$, 因为 $A E=B D, \\angle B D C+ \\angle A E C=180^{\\circ}$, 所以 $C, E, C_1$ 及 $C, D, C_2$ 均三点共线.\n这样 $\\triangle A C C_1$ 与 $\\triangle B C C_2$ 三边对应相等.\n则 $\\triangle A C C_1 \\cong \\triangle B C C_2$.\n设 $C C_1, A B$ 的中垂线交于点 $O$. 可以证明: $O$ 即是将 $\\triangle A C_1 C$ 旋转至 $\\triangle B C C_2$ 的旋转中心.\n因为 $O C_1=O C, O A=O B, A C_1=B C$, 所以\n$\\triangle O A C_1 \\cong \\triangle O B C$. 所以 $\\angle C_1 O A=\\angle C O B$, $\\angle C_1 O C=\\angle A O B$. 而 $\\angle O A C_1=\\angle O B C, \\angle C_1 A C=\\angle C B C_2$, 所以 $\\angle O A C=\\angle O B C_2$. 又因为 $O A=O B, A C=B C_2, \\triangle O A C \\cong \\triangle O B C_2$. 所以\n$O C=O C_2, \\angle A O C=\\angle B O C_2$. 这表明 $C_1 、 A 、 C$ 绕 $O$ 旋转 $\\angle A O B$ 后所得的像点依次是 $C 、 B 、 C_2$. 所以 $\\triangle B C C_2$ 是 $\\triangle A C_1 C$ 的像.\n因为 $\\triangle O C_1 C, \\triangle O A B, \\triangle O C C_2$ 是顶角相同的等腰三角形, 故它们相似 (其中 $\\triangle O C_1 C$ 与 $\\triangle O C C_2$ 全等). 因为 $\\frac{C_1 E}{E C}=\\frac{C D}{D C_2}=\\frac{A F}{F B}=\\frac{D C}{C E}$, 由此知 $D$ 是 $E$ 旋转后的像.\n所以 $\\triangle O E C \\backsim \\triangle O F B$. 所以 $\\angle E O C=\\angle F O B, \\frac{O E}{O F}=\\frac{O C}{O B}$. 又得 $\\angle F O E=\\angle B O C, \\triangle F O E \\backsim \\triangle B O C$. 所以 $\\frac{E F}{B C}=\\frac{O E}{O C}$. 同理可证得: $\\frac{F D}{A C}=\\frac{O D}{O C}$.\n因为 $D$ 是 $E$ 旋转后的像, 所以 $O E=O D$, 且有 $\\triangle O E D \\backsim \\triangle O C C_1$.\n所以 $\\frac{D E}{C C_1}=\\frac{O E}{O C_1}=\\frac{O E}{O C}, C C_1=C_1 E+C E=C D+C E$. 即 $\\frac{D E}{C D+C E}=\\frac{E F}{B C}= \\frac{F D}{A C}=\\frac{O E}{O C}$.", + "remark": "", + "figures": [ + "./images/volume7/figures/fig-c6a14.png" + ] +} \ No newline at end of file diff --git a/processed_dataset/proof/1895.json b/processed_dataset/proof/1895.json new file mode 100644 index 0000000000000000000000000000000000000000..af1075bc58d11c474acafd36415b304e61e93dca --- /dev/null +++ b/processed_dataset/proof/1895.json @@ -0,0 +1,10 @@ +{ + "source_file": "./raw_volume-zh/volume7/exercise6.tex", + "problem_type": "proof", + "problem": "问题15. 已知圆 $W$ 的中心为 $O, B C$ 为直径.\n点 $A$ 位于圆 $W$ 上使得 $0^{\\circ}<\\angle A O B< 120^{\\circ}$. 设 $D$ 是不包含 $C$ 点的弧 $\\overparen{A B}$ 的中点.\n直线 $l$ 通过 $O$ 且平行于直线 $A D$, 设 $l$ 交直线 $A C$ 于 $J$. 线段 $O A$ 的垂直平分线交圆 $W$ 于 $E$ 和 $F$. 求证: $J$ 是 $\\triangle C E F$ 的内心.", + "solution": "证明: 如图(), 连结 $O D 、 D F 、 E J 、 O E 、 E A$. 我们首先证明 $J$ 位于 $\\angle F E C$ 的内角平分线上.\n事实上, 因为 $E$ 和 $F$ 是关于 $P$ 的一对反射点, $D 、 J$ 是关于 $P$ 的另一对反射点 (四边形 $A D O J$ 是平行四边形), 所以 $\\angle F E J=\\angle D F E$.\n因此我们只需证明 $\\angle D F E=\\frac{1}{2} \\angle F E C$. 这等价于证明 $\\angle D O E=\\frac{1}{2} \\angle F O C$. …11. 由已知条件易知\n$\\triangle A O E$ 是等边三角形, 由于 $\\angle D O E=\\angle A O E-\\angle A O D=60^{\\circ}-\\frac{1}{2} \\angle A O B= \\frac{1}{2}\\left(180^{\\circ}-\\left(\\angle A O B+60^{\\circ}\\right)\\right)=\\frac{1}{2}\\left(180^{\\circ}-(\\angle A O B+\\angle F O A)\\right)=\\frac{1}{2} \\angle F O C$.\n(1) 式得证, 从而证得 $J$ 位于 $\\angle F E C$ 的内角平分线上.\n又易见 $\\overparen{A F}=\\overparen{A E}$, 故 $J$ 位于 $\\angle F C E$ 的平分线上.\n故 $J$ 是 $\\triangle C E F$ 的内心.", + "remark": "", + "figures": [ + "./images/volume7/figures/fig-c6a15.png" + ] +} \ No newline at end of file diff --git a/processed_dataset/proof/1896.json b/processed_dataset/proof/1896.json new file mode 100644 index 0000000000000000000000000000000000000000..c4a7392779040d74834e1bc01ef55380fba6c16e --- /dev/null +++ b/processed_dataset/proof/1896.json @@ -0,0 +1,10 @@ +{ + "source_file": "./raw_volume-zh/volume7/exercise6.tex", + "problem_type": "proof", + "problem": "问题16. 设 $A B C D E F$ 是凸六边形, $A B=B C=C D, D E=E F=F A, \\angle B C D= \\angle E F A=60^{\\circ}, G 、 H$ 是六边形内两点, 使 $\\angle A G B=\\angle D H E=120^{\\circ}$. 求证: $A G+G B+G H+D H+H E \\geqslant C F$.", + "solution": "证明: 用旋转法来证明本题.\n如图(), 分别以 $A B 、 D E$ 为边向六边形外作正 $\\triangle A B M$ 和 $\\triangle D E N$. 将 $\\triangle A G B$ 绕 $A$ 逆时针旋转 $60^{\\circ}$ 到 $\\triangle A G^{\\prime} M$, 则 $\\triangle A G G^{\\prime}$ 为正三角形, 故 $A G=G G^{\\prime}, G B=G^{\\prime} M$. 同样, 将 $\\triangle E H D$ 顺时针旋转 $60^{\\circ}$ 到 $\\triangle E H^{\\prime} N$,\n则 $\\triangle E H H^{\\prime}$ 为正三角形.\n于是, $E H=H H^{\\prime}, H D=H^{\\prime} N$. 连 $M N$, 则多边形 $A M B C D N E F$ 关于轴 $B E$ 对称, $M N=C F$. 另一方面, 由 \"两点间线段最短\" 有 $A G+G B+G H+D H+H E=M G^{\\prime}+G^{\\prime} G+G H+H H^{\\prime}+H^{\\prime} N \\geqslant M N$. 从而 $A G+G B+G H+D H+H E \\geqslant C F$.", + "remark": "", + "figures": [ + "./images/volume7/figures/fig-c6a16.png" + ] +} \ No newline at end of file diff --git a/processed_dataset/proof/1897.json b/processed_dataset/proof/1897.json new file mode 100644 index 0000000000000000000000000000000000000000..6e537265abc1ccd8c9b98bbf024df4ecb279f3ce --- /dev/null +++ b/processed_dataset/proof/1897.json @@ -0,0 +1,8 @@ +{ + "source_file": "./raw_volume-zh/volume7/exercise7.tex", + "problem_type": "proof", + "problem": "问题1. 设与 $\\triangle A B C$ 的外接圆内切并与边 $A B 、 A C$ 相切的圆为 $C_a$, 记 $r_a$ 为圆 $C_a$ 的半径, $r$ 是 $\\triangle A B C$ 的内切圆半径.\n类似地定义 $r_b 、 r_c$. 证明: $r_a+r_b+r_c \\geqslant 4 r$.", + "solution": "证明: 设 $O_a 、 O_b 、 O_c$ 为圆 $C_a$ 、圆 $C_b$ 、圆 $C_c$ 的圆心.\n记 $M 、 N$ 为点 $O_a$ 在 $A B 、 A C$ 上的投影,则 $\\triangle A B C$ 的内心 $I$ 为 $M N$ 的中点.\n$\\frac{1}{\\cos ^2 \\frac{A}{2}}$. 同理, $\\frac{r_b}{r}=\\frac{1}{\\cos ^2 \\frac{B}{2}}, \\frac{r_C}{r}=\\frac{1}{\\cos ^2 \\frac{C}{2}}$. 令 $\\alpha=\\frac{A}{2}, \\beta=\\frac{B}{2}, \\gamma=\\frac{C}{2}$. 只需证当 $\\alpha+\\beta+\\gamma=\\frac{\\pi}{2}$ 时, 有 $\\frac{1}{\\cos ^2 \\alpha}+\\frac{1}{\\cos ^2 \\beta}+\\frac{1}{\\cos ^2 \\gamma} \\geqslant 4$, 即 $\\tan ^2 \\alpha+\\tan ^2 \\beta+ \\tan ^2 \\gamma \\geqslant 1$.\n由 Cauchy - Schwartz(柯西-许瓦尔兹) 不等式, 有 $3\\left(\\tan ^2 \\alpha+\\tan ^2 \\beta+\\right.\\left.\\tan ^2 \\gamma\\right) \\geqslant(\\tan \\alpha+\\tan \\beta+\\tan \\gamma)^2$. 故只需证 $\\tan \\alpha+\\tan \\beta+\\tan \\gamma \\geqslant \\sqrt{3}$. 因为 $\\tan x$ 在 $\\left(0, \\frac{\\pi}{2}\\right)$ 上是凸函数, 故由 Jensen 不等式得: $\\tan \\alpha+\\tan \\beta+\\tan \\gamma \\geqslant 3 \\tan \\frac{\\pi}{6}=\\sqrt{3}$. 因此, $r_a+r_b+r_c \\geqslant 4 r$.", + "remark": "", + "figures": [] +} \ No newline at end of file diff --git a/processed_dataset/proof/1898.json b/processed_dataset/proof/1898.json new file mode 100644 index 0000000000000000000000000000000000000000..2baf28bd12b99be3b8f16d08cfba6bc48bb7aee1 --- /dev/null +++ b/processed_dataset/proof/1898.json @@ -0,0 +1,10 @@ +{ + "source_file": "./raw_volume-zh/volume7/exercise7.tex", + "problem_type": "proof", + "problem": "问题2. 已知圆 $W$ 是等边 $\\triangle A B C$ 的外接圆, 设圆 $W$ 与圆 $W_1$ 外切且切点异于点 $A 、 B 、 C$, 点 $A_1 、 B_1 、 C_1$ 在圆 $W_1$ 上, 且使得 $A A_1 、 B B_1 、 C C_1$ 与圆 $W_1$ 相切.\n证明: 线段 $A A_1 、 B B_1 、 C C_1$ 中的一线段的长度等于另两线段长度之和.", + "solution": "证明: 如图(), 设 $r 、 r_1$ 分别是圆 $W$ 、圆 $W_1$ 的半径.\n不失一般性, 设圆 $W$ 和圆 $W_1$ 的切点位于 $A B$ 间靠近 $A$ 的一边.\n记 $O 、 O_1$ 分别是圆 $W$ 、圆 $W_1$ 的圆心.\n设 $\\angle O_1 O A=\\alpha$, 则 $\\angle O_1 O B=120^{\\circ}-\\alpha$, $\\angle O_1 O C=120^{\\circ}+\\alpha$. 由余弦定理得 $A A_1^2=A O_1^2- r_1^2=r^2+\\left(r+r_1\\right)^2-2 r\\left(r+r_1\\right) \\cos \\alpha-r_1^2=2 r(r+\\left.r_1\\right)(1-\\cos \\alpha)=4 \\cdot \\sin ^2 \\frac{\\alpha}{2} \\cdot r\\left(r+r_1\\right)$. 注意到 $0<\\alpha<120^{\\circ}$, 所以, $A A_1= 2 \\sin \\frac{\\alpha}{2} \\cdot \\sqrt{r\\left(r+r_1\\right)}$. 同理, $B B_1=2 \\sin \\left(60^{\\circ}-\\frac{\\alpha}{2}\\right) \\cdot \\sqrt{r\\left(r+r_1\\right)}, C C_1= 2 \\sin \\left(60^{\\circ}+\\frac{\\alpha}{2}\\right) \\cdot \\sqrt{r\\left(r+r_1\\right)}$. 因此, $A A_1+B B_1=2 \\sqrt{r\\left(r+r_1\\right)}\\left[\\sin \\frac{\\alpha}{2}+\\right.$\n$$\n\\left.\\sin \\left(60^{\\circ}-\\frac{\\alpha}{2}\\right)\\right]=2 \\sqrt{r\\left(r+r_1\\right)} \\sin \\left(60^{\\circ}+\\frac{\\alpha}{2}\\right)=O C_1 .\n$$", + "remark": "", + "figures": [ + "./images/volume7/figures/fig-c7a2.png" + ] +} \ No newline at end of file diff --git a/processed_dataset/proof/1899.json b/processed_dataset/proof/1899.json new file mode 100644 index 0000000000000000000000000000000000000000..b2103d5d3ddab9ea34adb2d33bf7c62eddea162e --- /dev/null +++ b/processed_dataset/proof/1899.json @@ -0,0 +1,10 @@ +{ + "source_file": "./raw_volume-zh/volume7/exercise7.tex", + "problem_type": "proof", + "problem": "问题3. 设 $O$ 是锐角 $\\triangle A B C$ 的外心, $\\angle B<\\angle C, A O$ 交边 $B C$ 于点 D. $\\triangle A B D$ 和 $\\triangle A C D$ 的外心分别为 $E 、 F$. 在 $B A$ 和 $C A$ 的延长线上分别取点 $G$ 和 $H$, 使得 $A G=A C, A H=A B$. 证明: 四边形 $E F G H$ 是矩形的充分必要条件是 $\\angle A C B-\\angle A B C=60^{\\circ}$.", + "solution": "证明: 如图(), 设 $E F$ 交 $A B$ 于点 $K$, 点 $E 、 F$ 在 $B C$ 上的射影分别为 $E^{\\prime}, F^{\\prime}$. 显然, $\\triangle A G H \\cong \\triangle A C B$, $E F=\\frac{E^{\\prime} F^{\\prime}}{\\sin \\angle E^{\\prime} E F}=\\frac{B C}{2 \\sin \\angle A D C}=\\frac{B C}{2 \\sin \\left(B+90^{\\circ}-C\\right)}= \\frac{B C}{2 \\cos (C-B)}$.\n必要性.\n由 $E F=G H=B C$, 得 $\\cos (C-B)=\\frac{1}{2}$, 故 $\\angle A C B-\\angle A B C=60^{\\circ}$.\n充分性.\n由 $\\angle A C B-\\angle A B C=60^{\\circ}$, 得 $E F=B C=G H$. 由 $E F \\perp A D$, 则 $\\angle F K A=90^{\\circ}-\\angle O A B=\\angle A C B=\\angle A G H$. 故 $E F / / G H$. 因此, 四边形 $E F G H$ 为平行四边形.\n因 $\\angle A D C=\\angle A B C+90^{\\circ}- \\angle A C B=30^{\\circ}$, 则 $\\angle A F C=60^{\\circ}$. 故 $\\triangle A F C$ 为正三角形, 有 $A F=A C=A G$. 又 $A K=\\frac{A D}{2 \\sin \\angle A K F}=\\frac{A D}{2 \\sin C}=A F$, 故 $\\angle K F G==90^{\\circ}$. 从而, 四边形 $E F G H$ 为矩形.", + "remark": "", + "figures": [ + "./images/volume7/figures/fig-c7a3.png" + ] +} \ No newline at end of file diff --git a/processed_dataset/proof/1900.json b/processed_dataset/proof/1900.json new file mode 100644 index 0000000000000000000000000000000000000000..cb746c89474750200b921a4fe8e0de00a45a4483 --- /dev/null +++ b/processed_dataset/proof/1900.json @@ -0,0 +1,10 @@ +{ + "source_file": "./raw_volume-zh/volume7/exercise7.tex", + "problem_type": "proof", + "problem": "问题4. 已知 $\\triangle A B C$ 的外心 $O, P$ 为劣弧 $\\overparen{A B}$ 上一点.\n由 $P$ 向 $B O$ 作垂线交 $A B$ 于 $S$, 交 $B C$ 于 $T$. 由 $P$ 向 $A O$ 作垂线交 $A B$ 于 $Q$, 交 $A C$ 于 $R$. 证明: (1) $\\triangle P Q S$ 是等腰三角形; (2) $P Q^2=Q R \\cdot S T$.", + "solution": "证明: (1) 如图(), 由 $P R \\perp O A, P T \\perp O B$, 有\n$\\angle P Q S=\\angle A Q R=90^{\\circ}-\\angle O A B=90^{\\circ}-\\angle O B A= \\angle B S T=\\angle P S Q$. 故 $P Q=P S$.\n(2) 设 $\\odot O$ 的半径为 $r, P R \\cap O A=G, P T \\cap O B=H$. 设 $\\angle P O A=\\alpha-\\beta, \\angle P O B=\\alpha+\\beta, \\angle A O B= 2 \\alpha$. 则 $\\angle Q P S=180^{\\circ}-2 \\alpha, \\angle P Q S=\\angle P S Q=\\alpha= \\angle C$. 故 $\\triangle A Q R \\backsim \\triangle A C B \\backsim \\triangle T S B$. 所以, $\\frac{A Q}{T S}=\\frac{Q R}{S B}$, 即 $Q R \\cdot S T=A Q \\cdot S B$. 又 $O G=O P \\cos \\angle P O G=r \\cos (\\alpha-\\beta)$,\n$P G=r \\sin (\\alpha-\\beta)$, 则 $A G=r[1-\\cos (\\alpha-\\beta)], A Q=\\frac{A G}{\\sin \\angle A Q G}= \\frac{r[1-\\cos (\\alpha-\\beta)]}{\\sin \\alpha}, Q G=A Q \\cos \\angle A Q G=\\frac{r[1-\\cos (\\alpha-\\beta)] \\cos \\alpha}{\\sin \\alpha}$ 故 $P Q=P G-Q G=r\\left\\{\\sin (\\alpha-\\beta)-\\frac{\\cos \\alpha[1-\\cos (\\alpha-\\beta)]}{\\sin \\alpha}\\right\\}=\\frac{r(\\cos \\beta-\\cos \\alpha)}{\\sin \\alpha}$. 同理, $S B=\\frac{r[1-\\cos (\\alpha+\\beta)]}{\\sin \\alpha}$.\n注意到 $P Q^2=Q R \\cdot S T \\Leftrightarrow P Q^2=A Q \\cdot S B \\Leftrightarrow(\\cos \\beta-\\cos \\alpha)^2=[1-$\n$$\n\\cos (\\alpha-\\beta)][1-\\cos (\\alpha+\\beta)] \\Leftrightarrow(\\cos \\beta-\\cos \\alpha)^2=2 \\sin ^2 \\frac{(\\alpha-\\beta)}{2} .\n$$\n$2 \\sin ^2 \\frac{(\\alpha+\\beta)}{2} \\Leftrightarrow \\cos \\beta-\\cos \\alpha=2 \\sin \\frac{\\alpha-\\beta}{2} \\cdot \\sin \\frac{\\alpha+\\beta}{2}$, 而最后一式显然成立, 故 $P Q^2=Q R \\cdot S T$.", + "remark": "", + "figures": [ + "./images/volume7/figures/fig-c7a4.png" + ] +} \ No newline at end of file diff --git a/processed_dataset/proof/1901.json b/processed_dataset/proof/1901.json new file mode 100644 index 0000000000000000000000000000000000000000..b293b5b4a0ebe3460e732d7e9447255e90e388ba --- /dev/null +++ b/processed_dataset/proof/1901.json @@ -0,0 +1,8 @@ +{ + "source_file": "./raw_volume-zh/volume7/exercise7.tex", + "problem_type": "proof", + "problem": "问题5. 在锐角 $\\triangle A B C$ 中, $\\angle A C B=2 \\angle A B C$, 点 $D$ 是 $B C$ 边上一点, 使得 $2 \\angle B A D=\\angle A B C$. 证明 : $\\frac{1}{B D}=\\frac{1}{A B}+\\frac{1}{A C}$.", + "solution": "证明: 记 $\\angle B A D=\\alpha$, 则 $\\angle A B C=2 \\alpha, \\angle A C B=4 \\alpha$, 利用正弦定理可知, $\\frac{B D}{A B}=\\frac{\\sin \\alpha}{\\sin 3 \\alpha}, \\frac{A B}{A C}=\\frac{\\sin 4 \\alpha}{\\sin 2 \\alpha}=2 \\cos 2 \\alpha$, 从而, 要证的式子等价于 $\\sin 3 \\alpha= \\sin \\alpha+2 \\sin \\alpha \\cos 2 \\alpha$, 最后一式是显然的.", + "remark": "", + "figures": [] +} \ No newline at end of file diff --git a/processed_dataset/proof/1902.json b/processed_dataset/proof/1902.json new file mode 100644 index 0000000000000000000000000000000000000000..99073074e300da662166dc0975ac2e5c4dd8301e --- /dev/null +++ b/processed_dataset/proof/1902.json @@ -0,0 +1,8 @@ +{ + "source_file": "./raw_volume-zh/volume7/exercise7.tex", + "problem_type": "proof", + "problem": "问题6. 设 $R 、 r$ 分别是 $\\triangle A B C$ 的外接圆半径和内切圆半径, $R^{\\prime} 、 r^{\\prime}$ 分别是 $\\triangle A^{\\prime} B^{\\prime} C^{\\prime}$ 的外接圆半径和内切圆半径.\n证明: 若 $\\angle C=\\angle C^{\\prime}, R r^{\\prime}=R^{\\prime} r$, 则 $\\triangle A B C \\backsim \\triangle A^{\\prime} B^{\\prime} C^{\\prime}$.", + "solution": "证明: 因为 $\\angle C==\\angle C^{\\prime}, R=\\frac{c}{2 \\sin C}, R^{\\prime}=\\frac{c^{\\prime}}{2 \\sin C}$, 所以 $c r^{\\prime}=c^{\\prime} r$, 有 $\\frac{c}{r}=\\frac{c^{\\prime}}{r^{\\prime}}$, 即 $\\cot \\frac{A}{2}+\\cot \\frac{B}{2}=\\cot \\frac{A^{\\prime}}{2}+\\cot \\frac{B^{\\prime}}{2}$.\n设 $\\frac{\\angle A}{2}=\\angle 1, \\frac{\\angle B}{2}=\\angle 2, \\frac{\\angle A^{\\prime}}{2}=\\angle 3, \\frac{\\angle B^{\\prime}}{2}=\\angle 4$, 则 $\\frac{\\cos \\angle 1}{\\sin \\angle 1}+\\frac{\\cos \\angle 2}{\\sin \\angle 2}= \\frac{\\cos \\angle 3}{\\sin \\angle 3}+\\frac{\\cos \\angle 4}{\\sin \\angle 4}, \\frac{\\sin (\\angle 1+\\angle 2)}{\\sin \\angle 1 \\cdot \\sin \\angle 2}=\\frac{\\sin (\\angle 3+\\angle 4)}{\\sin \\angle 3 \\cdot \\sin \\angle 4}$, 因为 $\\angle 1+\\angle 2=\\angle 3+ \\angle 4$, 所以 $\\sin \\angle 1 \\cdot \\sin \\angle 2=\\sin \\angle 3 \\cdot \\sin \\angle 4$, 即 $\\cos (\\angle 1+\\angle 2)-\\cos (\\angle 1- \\angle 2)=\\cos (\\angle 3+\\angle 4)-\\cos (\\angle 3-\\angle 4)$, 得 $\\cos (\\angle 1-\\angle 2)=\\cos (\\angle 3- \\angle 4)$, 有 $\\angle 1-\\angle 2=\\angle 3-\\angle 4$, 或 $\\angle 1-\\angle 2=\\angle 4-\\angle 3$. 又 $\\angle 1+\\angle 2= \\angle 3+\\angle 4$, 于是 $\\angle A=\\angle A^{\\prime}$ 或 $\\angle A=\\angle B^{\\prime}$.\n故 $\\triangle A B C \\backsim \\triangle A^{\\prime} B^{\\prime} C^{\\prime}$.", + "remark": "", + "figures": [] +} \ No newline at end of file diff --git a/processed_dataset/proof/1903.json b/processed_dataset/proof/1903.json new file mode 100644 index 0000000000000000000000000000000000000000..1aa711adac2213be850bbb4548210969da1eace5 --- /dev/null +++ b/processed_dataset/proof/1903.json @@ -0,0 +1,10 @@ +{ + "source_file": "./raw_volume-zh/volume7/exercise7.tex", + "problem_type": "proof", + "problem": "问题9. 设锐角 $\\triangle A B C$ 的外心为 $O$, 从 $A$ 作 $B C$ 的高, 垂足为 $P$, 且 $\\angle B C A \\geqslant \\angle A B C+30^{\\circ}$. 证明: $\\angle C A B+\\angle C O P<90^{\\circ}$.", + "solution": "证明: 如图(), 延长 $C O 、 A O 、 A P$ 分别交 $\\odot O$ 于 $D 、 E 、 F$, 连结 $E F 、 B D$. 则 $\\angle E=\\angle C A P+\\angle A B C= 90^{\\circ}-\\angle A C B+\\angle A B P$. 故 $\\angle O A P=90^{\\circ}-\\angle E= \\angle A C B-\\angle A B P$. 设 $\\odot O$ 的半径为 $R$. 因为 $C P= 2 R \\sin B \\cdot \\cos C, A P=2 R \\sin B \\cdot \\sin C$, 所以 $O P^2= A P^2+O A^2-2 O A \\cdot A P \\cdot \\cos \\angle O A P=4 R^2 \\sin ^2 B \\cdot \\sin ^2 C+R^2-4 R^2 \\cdot \\sin B \\cdot \\sin C \\cdot \\cos (C-B)= 4 R^2\\left[\\sin ^2 B \\cdot \\sin ^2 C+\\frac{1}{4}-\\sin ^2 B \\cdot \\sin ^2 C-\\sin B \\cdot \\cos B \\cdot\\right.\\sin C \\cdot \\cos C]=4 R^2\\left(\\frac{1}{4}-\\sin B \\cdot \\cos B \\cdot \\sin C \\cdot \\cos C\\right)$. 所以 $O P^2- C P^2=4 R^2\\left[\\frac{1}{4}-\\sin B \\cdot \\cos C \\cdot \\sin (B+C)\\right]=4 R^2\\left[\\frac{1}{4}-\\frac{1}{2} \\sin ^2 A+\\right.\\left.\\frac{1}{2} \\sin A \\cdot \\sin (C-B)\\right]$. 因为 $\\angle C-\\angle B \\geqslant 30^{\\circ}$, 且 $\\angle C 、 \\angle B$ 都为锐角, 所以上式 $\\geqslant 4 R^2\\left[\\frac{1}{4}-\\frac{1}{2} \\sin ^2 A+\\frac{1}{4} \\sin A\\right]=R^2(2 \\sin A+1)(1-\\sin A)>0$. 所以 $O P^2>C P^2, O P>C P$. 有 $\\angle C O P<\\angle O C P$. 故 $\\angle C O P+\\angle C A B< \\angle O C P+\\angle D=90^{\\circ}$.", + "remark": "", + "figures": [ + "./images/volume7/figures/fig-c7a9.png" + ] +} \ No newline at end of file diff --git a/processed_dataset/proof/1904.json b/processed_dataset/proof/1904.json new file mode 100644 index 0000000000000000000000000000000000000000..d5b514580fe5b8c3b74504fed62325cbc599c92d --- /dev/null +++ b/processed_dataset/proof/1904.json @@ -0,0 +1,10 @@ +{ + "source_file": "./raw_volume-zh/volume7/exercise7.tex", + "problem_type": "proof", + "problem": "问题10. 圆 $W$ 内切于四边形 $A B C D, I$ 是圆 $W$ 的圆心, 且有 $(A I+D I)^2+(B I+ C I)^2=(A B+C D)^2$. 证明 : 四边形 $A B C D$ 是等腰梯形.", + "solution": "证明: 如图(), 设角,由已知 $\\left(\\frac{r}{\\sin \\alpha}+\\frac{r}{\\sin \\theta}\\right)^2+ \\left(\\frac{r}{\\sin \\beta}+\\frac{r}{\\sin \\gamma}\\right)^2=\\left(r \\frac{\\cos \\alpha}{\\sin \\alpha}+r \\frac{\\cos \\beta}{\\sin \\beta}+r \\frac{\\cos \\gamma}{\\sin \\gamma}+\\right. \\left.r \\frac{\\cos \\theta}{\\sin \\theta}\\right)^2$, 所以 $\\left(\\frac{1}{\\sin \\alpha}+\\frac{1}{\\sin \\theta}\\right)^2-\\left(\\frac{\\cos \\alpha}{\\sin \\alpha}+\\frac{\\cos \\theta}{\\sin \\theta}\\right)^2+ \\left(\\frac{1}{\\sin \\beta}+\\frac{1}{\\sin \\gamma}\\right)^2-\\left(\\frac{\\cos \\beta}{\\sin \\beta}+\\frac{\\cos \\gamma}{\\sin \\gamma}\\right)^2=2\\left(\\frac{\\cos \\alpha}{\\sin \\alpha}+\\right. \\left.\\frac{\\cos \\theta}{\\sin \\theta}\\right)\\left(\\frac{\\cos \\beta}{\\sin \\beta}+\\frac{\\cos \\gamma}{\\sin \\gamma}\\right)$, 化简得 $2+\\frac{1-\\cos \\theta \\cos \\alpha}{\\sin \\alpha \\sin \\theta}+\\frac{1-\\cos \\beta \\cos \\gamma}{\\sin \\beta \\sin \\gamma}=\\frac{\\sin (\\alpha+\\theta) \\sin (\\beta+\\gamma)}{\\sin \\alpha \\sin \\beta \\sin \\gamma \\sin \\theta},[\\cos (\\beta-\\gamma)-\\cos (\\beta+\\gamma)][1+\\cos (\\beta+ \\gamma)]+[\\cos (\\alpha-\\theta)-\\cos (\\alpha+\\theta)] \\cdot[1+\\cos (\\alpha+\\theta)]=2 \\sin (\\alpha+\\theta) \\sin (\\beta+ \\gamma) \\cdots$ (1). 因为 $\\sin (\\beta+\\gamma)=\\sin (\\alpha+\\theta), \\cos (\\beta+\\gamma)=-\\cos (\\theta+\\alpha)$, 所以 (1) 式可化简为 $\\cos (\\alpha-\\theta) \\cos ^2 \\frac{\\beta+\\gamma}{2}+\\cos (\\beta-\\gamma) \\cdot \\cos ^2 \\frac{\\alpha+\\theta}{2}=1$. 所以 $\\cos (\\alpha- \\theta)=\\cos (\\beta-\\gamma)=1$. 所以 $\\alpha=\\theta, \\beta=\\gamma . \\angle A=\\angle D, \\angle B=\\angle C$.\n故 $A B C D$ 为等腰梯形.", + "remark": "", + "figures": [ + "./images/volume7/figures/fig-c7a10.png" + ] +} \ No newline at end of file diff --git a/processed_dataset/proof/1905.json b/processed_dataset/proof/1905.json new file mode 100644 index 0000000000000000000000000000000000000000..1ee5790ecf9f7f8042c07dec2a94d64063946c97 --- /dev/null +++ b/processed_dataset/proof/1905.json @@ -0,0 +1,10 @@ +{ + "source_file": "./raw_volume-zh/volume7/exercise7.tex", + "problem_type": "proof", + "problem": "问题11. 已知 $\\odot O$ 与 $\\triangle A B C$ 的外接圆、 $A B 、 A C$ 均相切, 切点分别为 $T 、 P 、 Q, I$ 是 $P Q$ 中点.\n证明: $I$ 是 $\\triangle A B C$ 的内心或旁心.", + "solution": "证明: 如图(), $\\odot O$ 可与 $\\odot O_1$ 外切 (旁心) 或内切 (内心). 两者证明类似.\n只证前者.\n因为 $A P 、 A Q$ 切 $\\odot O$ 于 $P 、 Q$. 所以 $A O \\perp P Q, A O$ 平分 $P Q$, 所以 $A 、 I 、 O$ 共线.\n从而 $A I$ 平分 $\\angle B A C$. 延长 $A O$ 交 $\\odot O_1$ 于 $E$, 延长 $T O_1$ 交 $\\odot O_1$ 于 $M$. 由相交弦定理, $A O \\cdot O E=O T \\cdot O M \\cdots$ (1). 设 $\\odot O$ 半径为 $r, \\odot O_1$ 半径为 $R$.\n则 $A O=\\frac{r}{\\sin \\frac{A}{2}}, O I=r \\sin \\frac{A}{2}, A I=\\frac{r}{\\sin \\frac{A}{2}}-r \\sin \\frac{A}{2}$,\n$A E=2 R \\sin \\left(B+\\frac{A}{2}\\right)$. 从而由 (1) 知 $\\frac{r}{\\sin \\frac{A}{2}} \\cdot\\left(2 R \\sin \\left(B+\\frac{A}{2}\\right)-\\frac{r}{\\sin \\frac{A}{2}}\\right)=$\n$$\n\\begin{aligned}\n& r(2 R-r) \\text {. 所以, } 2 R \\sin \\left(B+\\frac{A}{2}\\right)=(2 R-r) \\sin \\frac{A}{2}+\\frac{r}{\\sin \\frac{A}{2}} \\text {, } \\\\\n& 2 R\\left(\\sin \\left(B+\\frac{A}{2}\\right)-\\sin \\frac{A}{2}\\right)=\\frac{r}{\\sin \\frac{A}{2}}\\left(1-\\sin ^2 \\frac{A}{2}\\right), 4 R \\cos \\frac{B+A}{2} \\sin \\frac{B}{2}= \\\\\n& \\frac{r}{\\sin \\frac{A}{2}} \\cos ^2 \\frac{A}{2}, r=4 R \\frac{\\sin \\frac{A}{2} \\sin \\frac{B}{2} \\sin \\frac{C}{2}}{\\cos ^2 \\frac{A}{2}} \\text {, 所以, } \\frac{A I}{A E}=\\frac{\\frac{r}{\\sin \\frac{A}{2}}\\left(1-\\sin ^2 \\frac{A}{2}\\right)}{2 R \\sin \\left(B+\\frac{A}{2}\\right)}= \\\\\n& \\frac{\\frac{4 R}{\\sin \\frac{A}{2}} \\cdot \\cos ^2 \\frac{A}{2} \\cdot \\frac{\\sin \\frac{A}{2} \\sin \\frac{B}{2} \\sin \\frac{C}{2}}{\\cos ^2 \\frac{A}{2}}}{2 R \\sin \\left(B+\\frac{A}{2}\\right)}=\\frac{2 \\sin \\frac{B}{2} \\sin \\frac{C}{2}}{\\sin \\left(B+\\frac{A}{2}\\right)}=\\frac{2 \\sin \\frac{B}{2} \\sin \\frac{C}{2}}{\\cos \\frac{B-C}{2}} . \\\\\n& \\frac{A I}{I E}=\\frac{2 \\sin \\frac{B}{2} \\sin \\frac{C}{2}}{\\cos \\frac{B}{2} \\cos \\frac{C}{2}-\\sin \\frac{B}{2} \\sin \\frac{C}{2}}=\\frac{2 \\sin \\frac{B}{2} \\sin \\frac{C}{2}}{\\sin \\frac{A}{2}} \\text {. } \\\\\n&\n\\end{aligned}\n$$\n连结 $B I$, 设 $\\angle A B I=\\theta$, 则 $\\frac{A I}{I E}=\\frac{2 R \\sin C \\cdot \\sin \\theta}{2 R \\sin \\frac{A}{2} \\cdot \\sin \\left(B+\\frac{A}{2}-\\theta\\right)}$ 故\n$\\frac{2 \\sin \\frac{C}{2} \\cos \\frac{C}{2} \\sin \\theta}{\\sin \\frac{A}{2} \\sin \\left(B+\\frac{A}{2}-\\theta\\right)}$ 由 (2) $=\\frac{2 \\sin \\frac{B}{2} \\sin \\frac{C}{2}}{\\sin \\frac{A}{2}}, \\sin \\frac{A+B}{2} \\sin \\theta=\\sin \\frac{B}{2} \\sin (B+\\left.\\frac{A}{2}-\\theta\\right), \\cos \\left(\\frac{A+B}{2}-\\theta\\right)-\\cos \\left(\\frac{A+B}{2}+\\theta\\right)=\\cos \\left(\\frac{A+B}{2}-\\theta\\right)-\\cos \\left(\\frac{3}{2} B+\\right.\\frac{A}{2}-\\theta$ ) (注意 $0<\\theta),在锐角 $\\triangle A B C$ 的 $B C$ 边上有两点 $E 、 F$ 满足 $\\angle B A E=\\angle C A F$, 作 $F M \\perp A B, F N \\perp A C$, 垂足为 $M 、 N$. 延长 $A E$ 交 $\\triangle A B C$ 的外接圆于点 $D$. 证明: 四边形 $A M D N$ 与 $\\triangle A B C$ 的面积相等.", + "solution": "证明: : 连结 $B D$, 则 $\\triangle A B D \\backsim \\triangle A F C$, 所以 $A F \\cdot A D=A B \\cdot A C$. 设 $\\angle B A E=\\angle C A F=\\alpha, \\angle E A F=\\beta$, 则 $S_{\\text {四边形 } A M D N}=-\\frac{1}{2} A M \\cdot A D \\sin \\alpha+ \\frac{1}{2} A D \\cdot A N \\sin (\\alpha+\\beta)=\\frac{1}{2} A D[A F \\cos (\\alpha+\\beta) \\sin \\alpha+A F \\cos \\alpha \\sin (\\alpha+\\beta)]= \\frac{1}{2} A D \\cdot A F \\sin (2 \\alpha+\\beta)=\\frac{1}{2} A B \\cdot A C \\sin \\angle B A C=S_{\\triangle A B C}$.", + "remark": "", + "figures": [ + "./images/volume7/figures/fig-c7p12.png" + ] +} \ No newline at end of file diff --git a/processed_dataset/proof/1907.json b/processed_dataset/proof/1907.json new file mode 100644 index 0000000000000000000000000000000000000000..1557bbee30653b994f11900d027add1ef2966d4e --- /dev/null +++ b/processed_dataset/proof/1907.json @@ -0,0 +1,10 @@ +{ + "source_file": "./raw_volume-zh/volume7/exercise7.tex", + "problem_type": "proof", + "problem": "问题13. 已知三角形 $A B C$ 的内心为 $I$, 外心为 $O$, 点 $B$ 关于圆 $O$ 的对径点为 $K$, 在 $A B$ 的延长线上取点 $N$, $C B$ 的延长线上取点 $M$, 使得 $M C=N A=S, S$ 为三角形 $A B C$ 的半周长.\n证明: $I K \\perp M N$.", + "solution": "证明: 如图(), 设 $\\angle B M N=\\alpha, \\angle I K C=\\beta, r, R$ 分别为 $\\triangle A B C$ 的内切圆, 外接圆半径, 则 $\\angle K C I=90^{\\circ}-\\frac{C}{2}$.\n由正弦定理, $\\frac{K C}{I C}=\\frac{\\sin \\left(90^{\\circ}-\\frac{C}{2}+\\beta\\right)}{\\sin \\beta}= \\frac{\\cos \\left(\\frac{C}{2}-\\beta\\right)}{\\sin \\beta}=\\cos \\frac{C}{2} \\cdot \\cot \\beta+\\sin \\frac{C}{2}$. 又 $K C=2 R \\cdot \\cos \\angle B K C=2 R \\cdot \\cos A, I C=\\frac{r}{\\sin \\angle I C B}=\\frac{r}{\\sin \\frac{C}{2}}=4 R \\sin \\frac{B}{2} \\sin \\frac{A}{2}(r=\\left.4 R \\sin \\frac{A}{2} \\sin \\frac{B}{2} \\sin \\frac{C}{2}\\right)$, 故 $\\cos \\frac{C}{2} \\cdot \\cot \\beta+\\sin \\frac{C}{2}=\\frac{K C}{I C}=\\frac{\\cos A}{2 \\sin \\frac{B}{2} \\sin \\frac{A}{2}}$. 即\n$$\n\\begin{aligned}\n& \\cot \\beta=\\frac{\\cos A}{2 \\sin \\frac{B}{2} \\sin \\frac{A}{2} \\cos \\frac{C}{2}}-\\tan \\frac{C}{2} \\text {. } \\\\\n& \\text { 又 } B N=S-c=\\frac{1}{2}(a+b-c)=\\frac{1}{2} \\cdot 2 R(\\sin A+\\sin B-\\sin C)= \\\\\n& R\\left(2 \\sin \\frac{A+B}{2} \\cos \\frac{A-B}{2}-2 \\sin \\frac{A+B}{2} \\cos \\frac{A+B}{2}\\right)=2 R \\cdot \\cos \\frac{C}{2} \\text {. } \\\\\n& \\left(\\cos \\frac{A-B}{2}-\\cos \\frac{A+B}{2}\\right)=4 R \\cdot \\sin \\frac{A}{2} \\sin \\frac{B}{2} \\cos \\frac{C}{2} \\text {. 同理, } B M=4 R \\cdot \\sin \\frac{C}{2} \\sin \\\\\n& \\frac{B}{2} \\cos \\frac{A}{2} \\text {, 于是 } \\frac{B M}{B N}=\\frac{\\sin \\frac{C}{2} \\cos \\frac{A}{2}}{\\cos \\frac{C}{2} \\sin \\frac{A}{2}} \\text {, 另一方面, } \\frac{B M}{B N}=\\frac{\\sin (\\alpha+B)}{\\sin \\alpha} \\text {, 结合以上两 } \\\\\n&\n\\end{aligned}\n$$\n式, 得 $\\cos B+\\cot \\alpha \\cdot \\sin B=\\tan \\frac{C}{2} \\cdot \\cot \\frac{A}{2}, \\cot \\alpha=\\frac{\\tan \\frac{C}{2} \\cdot \\cot \\frac{A}{2}}{\\sin B}-\\cot B$. (2)\n下证 $\\cot \\alpha=\\cot \\beta$.\n由 (1), (2) $\\Leftrightarrow \\frac{\\cos A}{2 \\sin \\frac{B}{2} \\sin \\frac{A}{2} \\cos \\frac{C}{2}}-\\frac{\\sin \\frac{C}{2}}{\\cos \\frac{C}{2}}=-\\frac{\\tan \\frac{C}{2} \\cot \\frac{A}{2}}{\\sin B}-\\frac{\\cos B}{\\sin B} \\Leftrightarrow$\n$$\n\\frac{\\cos A-2 \\sin \\frac{A}{2} \\sin \\frac{B}{2} \\sin \\frac{C}{2}}{2 \\sin \\frac{A}{2} \\sin \\frac{B}{2} \\cos \\frac{C}{2}}=\\frac{\\tan \\frac{C}{2} \\cot \\frac{A}{2}-\\cos B}{\\sin B} \\Leftrightarrow\\left(\\cos A-2 \\sin \\frac{A}{2}\\right.\n$$\n$$\n\\begin{gathered}\n\\left.\\sin \\frac{B}{2} \\sin \\frac{C}{2}\\right) \\cos \\frac{B}{2}=\\sin \\frac{A}{2} \\cos \\frac{C}{2} \\cdot\\left(\\frac{\\sin \\frac{C}{2}}{\\cos \\frac{C}{2}} \\cdot \\frac{\\cos \\frac{A}{2}}{\\sin \\frac{A}{2}}-\\cos B\\right) \\Leftrightarrow \\cos A \\\\\n\\cos \\frac{B}{2}-\\sin \\frac{A}{2} \\sin B \\sin \\frac{C}{2}=\\sin \\frac{C}{2} \\cos \\frac{A}{2}-\\cos B \\sin \\frac{A}{2} \\cos \\frac{C}{2} \\Leftrightarrow \\cos A \\cos \\frac{B}{2}= \\\\\n\\cos \\frac{A}{2} \\sin \\frac{C}{2}-\\sin \\frac{A}{2} \\cos \\left(B+\\frac{C}{2}\\right) \\Leftrightarrow \\cos A \\cos \\frac{B}{2}=\\cos \\frac{A}{2} \\cos \\frac{A+B}{2}-\\sin \\frac{A}{2} \\cdot \\\\\n\\cos \\left[B+\\frac{\\pi-A-B}{2}\\right] * * \\\\\n* * \\text { 右边 }=\\frac{1}{2}\\left[\\cos \\left(A+\\frac{B}{2}\\right)+\\cos \\frac{B}{2}\\right]+\\left(-\\frac{1}{2}\\right)\\left[\\cos \\frac{B}{2}-\\cos \\left(A-\\frac{B}{2}\\right)\\right] \\\\\n=\\frac{1}{2}\\left[\\cos \\left(A+\\frac{B}{2}\\right)+\\cos \\left(A-\\frac{B}{2}\\right)\\right] \\\\\n=\\cos A \\cos \\frac{B}{2}=\\text { 左边.\n}\n\\end{gathered}\n$$\n从而 $M N$ 与 $B C$ 的夹角等于 $I K$ 与 $C K$ 夹角, 又 $B C \\perp C K$, 所以 $M N \\perp I K$.", + "remark": "", + "figures": [ + "./images/volume7/figures/fig-c7a13.png" + ] +} \ No newline at end of file diff --git a/processed_dataset/proof/1908.json b/processed_dataset/proof/1908.json new file mode 100644 index 0000000000000000000000000000000000000000..1764960e6cef1e13b115c3fd69dd67cf476857bf --- /dev/null +++ b/processed_dataset/proof/1908.json @@ -0,0 +1,10 @@ +{ + "source_file": "./raw_volume-zh/volume7/exercise8.tex", + "problem_type": "proof", + "problem": "问题1. 在完全四边形 $A B C D E F$ 中, 对角线 $A D$ 的延长线交对角线 $C E$ 于点 $G$. 记 $\\frac{A D}{D G}=p_1, \\frac{C D}{D F}=p_2, \\frac{E D}{D B}=p_3, \\frac{A B}{B C}=\\lambda_3, \\frac{C G}{G E}=\\lambda_2, \\frac{E F}{F A}=\\lambda_1$. 求证:\n$$\n\\begin{aligned}\n& \\lambda_1=\\frac{p_1 p_2-1}{1+p_1}=\\frac{1+p_1}{p_1 p_3-1}=\\frac{1+p_2}{1+p_3} \\label{eq1} \\\\\n& \\lambda_2=\\frac{p_2 p_3-1}{1+p_2}=\\frac{1+p_2}{p_2 p_1-1}=\\frac{1+p_3}{1+p_1} \\label{eq2} \\\\\n& \\lambda_3=\\frac{p_3 p_1-1}{1+p_3}=\\frac{1+p_3}{p_3 p_2-1}=\\frac{1+p_1}{1+p_2} \\label{eq3}\n\\end{aligned}\n$$", + "solution": "证明: 首先证明式\\ref{eq1}, 如图(), 过点 $D$ 作 $M N C E$ 交 $B C$ 于点 $M$, 交 $F E$ 于点 $N$, 则 $\\frac{C E}{D N}=\\frac{C F}{D F}= \\frac{C D+D F}{D F}=p_2+1, \\frac{G E}{D N}=\\frac{A G}{A D}=\\frac{A D+D G}{A D}=1+ \\frac{1}{p_1}$. 以上两式相除得, $\\frac{C E}{G E}=\\frac{p_1\\left(1+p_2\\right)}{1+p_1}$, 则 $\\lambda_1=\\frac{C G}{G E}= \\frac{C E-G E}{G E}=\\frac{p_1 p_2-1}{1+p_1}$. 又 $\\frac{C E}{M D}=\\frac{B E}{B D}=\\frac{B D+D E}{B D}=1+p_3, \\frac{C G}{M D}=\\frac{A G}{A D}=\\frac{G E}{D N}=\\frac{1+p_1}{p_1}$, 则有 $\\frac{C E}{C G}=\\frac{p_1\\left(1+p_3\\right)}{1+p_1}$, 从而, $\\lambda_1=\\frac{C G}{G E}= \\frac{1+p_1}{p_1 p_3-1}$. 对 $\\triangle C E D$ 及点 $A$ 应用塞瓦定理有 $\\frac{C G}{G E} \\cdot \\frac{E B}{B D} \\cdot \\frac{D F}{F C}=1$. 从而, $\\lambda_1= \\frac{C G}{G E}=\\frac{C F}{D F} \\cdot \\frac{B D}{B E}=\\frac{1+p_2}{1+p_3}$. 故 $\\lambda_1=\\frac{p_1 p_2-1}{1+p_1}=\\frac{1+p_1}{p_1 p_3-1}=\\frac{1+p_2}{1+p_3}$.\n同理可证式\\ref{eq2}和式\\ref{eq3}.", + "remark": "", + "figures": [ + "./images/volume7/figures/fig-c8a1.png" + ] +} \ No newline at end of file diff --git a/processed_dataset/proof/1909.json b/processed_dataset/proof/1909.json new file mode 100644 index 0000000000000000000000000000000000000000..7896bb2810ea07410f800536ea7f45a21fbda50f --- /dev/null +++ b/processed_dataset/proof/1909.json @@ -0,0 +1,10 @@ +{ + "source_file": "./raw_volume-zh/volume7/exercise8.tex", + "problem_type": "proof", + "problem": "问题2. 求证: 完全四边形 $A B C D E F$ 的三条对角线 $A D 、 B F 、 C E$ 的中点 $M 、 N$ 、 $P$ 三点共线.", + "solution": "证明: 如图(), 分别取 $C D 、 B D 、 B C$ 的中点 $Q 、 R 、 S$.\n于是, 在 $\\triangle A C D$ 中, $M 、 R 、 Q$ 三点共线; 在 $\\triangle B C F$ 中, $S 、 R 、 N$ 三点共线; 在 $\\triangle B C E$ 中, $S 、 Q 、 P$ 三点共线.\n由平行线性质有 $\\frac{M Q}{M R}=\\frac{A C}{A B}, \\frac{N R}{N S}=\\frac{F D}{F C}$,\n$\\frac{P S}{P Q}=\\frac{E B}{E D}$. 由于直线 $A F E$ 与 $\\triangle B C D$ 的边所在直线相截, 所以, 由梅涅劳斯定理知 $\\frac{A C}{A B} \\cdot \\frac{F D}{F C} \\cdot \\frac{E B}{E D}=1$. 从而, $\\frac{M Q}{M R} \\cdot \\frac{N R}{N S} \\cdot \\frac{P S}{P Q}=1$.\n再对 $\\triangle Q R S$ 应用梅涅劳斯定理的逆定理, 知 $N 、 M 、 P$ 三点共线.", + "remark": "", + "figures": [ + "./images/volume7/figures/fig-c8a2.png" + ] +} \ No newline at end of file diff --git a/processed_dataset/proof/1910.json b/processed_dataset/proof/1910.json new file mode 100644 index 0000000000000000000000000000000000000000..d62f73dc63bba6569f459d6a6a921543af4dd624 --- /dev/null +++ b/processed_dataset/proof/1910.json @@ -0,0 +1,11 @@ +{ + "source_file": "./raw_volume-zh/volume7/exercise8.tex", + "problem_type": "proof", + "problem": "问题3. 如图(), 凸四边形 $A B C D$ 的一组对边 $B A$ 和 $C D$ 的延长线交于 $M$, 且 $A D$ 不平行于 $B C$, 过 $M$ 作截线交另一组对边所在直线于 $H 、 L$, 交对角线所在直线于 $H^{\\prime} 、 L^{\\prime}$. 求证: $\\frac{1}{M H}+\\frac{1}{M L}=\\frac{1}{M H^{\\prime}}+\\frac{1}{M L}$.", + "solution": "证明: 如图(), 延长 $B C 、 A D$ 交于 $P$, 设 $B D 、 A C$ 交于 $Q$, 连结 $M P, P Q$ 分别与 $M C, M L$ 交于 $N$ 、 $K$,连结 $M Q$ 交 $B C$ 于 $R$. 交 $A D$ 于 $T$.\n由完全四边形的性质知, $M 、 Q 、 T 、 R$ 成调和点列, 考虑过 $P$ 点的四条直线 $P M 、 P A 、 P K$ 、 $P B$ 构成的直线束.\n由上述引理知 $M 、 K 、 H 、 L$ 成调和点列.\n所以\n$$\n\\frac{1}{M H}+\\frac{1}{M L}=\\frac{2}{M K} \\text { (调和点列的性质) } \\cdots \\text { (1). }\n$$\n同理, $B 、 C 、 R 、 P$ 为调和点列, 考虑过 $Q$ 的四条直线 $Q B 、 Q C 、 Q R 、 Q P$ 构成的直线束.\n现 $M L$ 去截直线束, 由上述引理知, $L^{\\prime} 、 H^{\\prime} 、 M 、 K$ 也成调和点列, 即 $M 、 K 、 L^{\\prime} 、 H^{\\prime}$ 也成调和点列.\n所以 $\\frac{1}{M L^{\\prime}}+\\frac{1}{M H^{\\prime}}=\\frac{2}{M K} \\cdots$ (2).\n由(1)、(2)知: $\\frac{1}{M H}+\\frac{1}{M L}=\\frac{1}{M H^{\\prime}}+\\frac{1}{M L^{\\prime}}$.", + "remark": "", + "figures": [ + "./images/volume7/figures/fig-c8p3.png", + "./images/volume7/figures/fig-c8a3.png" + ] +} \ No newline at end of file diff --git a/processed_dataset/proof/1911.json b/processed_dataset/proof/1911.json new file mode 100644 index 0000000000000000000000000000000000000000..3845ac58a593ca0421659db2f018d89794349beb --- /dev/null +++ b/processed_dataset/proof/1911.json @@ -0,0 +1,10 @@ +{ + "source_file": "./raw_volume-zh/volume7/exercise8.tex", + "problem_type": "proof", + "problem": "问题4. 已知四边形 $A B C D$ 内接于以 $B D$ 为直径的圆.\n设 $A^{\\prime}$ 为点 $A$ 关于 $B D$ 的对称点, $B^{\\prime}$ 为点 $B$ 关于 $A C$ 的对称点, 直线 $A^{\\prime} C$ 与 $B D 、 A C$ 与 $B^{\\prime} D$ 分别交于点 $P 、 Q$. 证明: $P Q \\perp A C$.", + "solution": "证明: 如图(), 设 $A C$ 与 $B D$ 交于点 $R^{\\circ}$. 因为四边形 $A B A^{\\prime} C$ 为圆内接四边形, 所以, $\\angle B A R=\\angle B A C=\\angle B A^{\\prime} P=\\angle B A P$, 即 $A B$ 为 $\\angle C A P$ 的角平分线.\n又 $B D$ 为直径, 则 $\\angle D A B=90^{\\circ}$. 故 $D A$ 为 $\\angle R A P$ 的外角平分线.\n因此, $P 、 R 、 B 、 D$ 为调和点列.\n进而, $Q P$ 、 $Q R 、 Q B 、 Q D$ 为调和线束.\n因为 $\\angle B Q R=\\angle B^{\\prime} Q R=\\angle D Q R$, 所以, $\\angle R Q P=90^{\\circ}$, 故 $P Q \\perp A C$.", + "remark": "", + "figures": [ + "./images/volume7/figures/fig-c8a4.png" + ] +} \ No newline at end of file diff --git a/processed_dataset/proof/1912.json b/processed_dataset/proof/1912.json new file mode 100644 index 0000000000000000000000000000000000000000..8d0f047411ea7291220d1746d307cc5af4512e71 --- /dev/null +++ b/processed_dataset/proof/1912.json @@ -0,0 +1,11 @@ +{ + "source_file": "./raw_volume-zh/volume7/exercise8.tex", + "problem_type": "proof", + "problem": "问题5. 如图(), 设 $D 、 E 、 F$ 分别为 $\\triangle A B C$ 的三边 $B C 、 C A 、 A B$ 上的点, 且 $A D$ 与 $E F$ 垂直相交于 $O$, 又 $D E 、 D F$ 分别平分 $\\angle A D C 、 \\angle A D B$, 则 $O D$ 平分 $\\angle B O C$.", + "solution": "证明: 如图(), 设直线 $E F$ 与 $B C$ 交于点 $G$ (可以是无穷远点, 以下同), 由角平分线定理有 $\\frac{C E}{A E} \\cdot \\frac{A F}{F B} \\cdot \\frac{B D}{D C}=\\frac{D C}{A D} \\cdot \\frac{A D}{B D} \\cdot \\frac{B D}{D C}=1$. 由 Ceva 定理知 $B E 、 A D$ 、 $C F$ 三线共点, 由性质 2 知 $B 、 C 、 D 、 G$ 成调和点列, 即 $O B 、 O C 、 O D 、 O G$ 成调和线束, 结合 $O D \\perp O G$ 知 $O D$ 平分 $\\angle B O C$.", + "remark": "", + "figures": [ + "./images/volume7/figures/fig-c8p5.png", + "./images/volume7/figures/fig-c8a5.png" + ] +} \ No newline at end of file diff --git a/processed_dataset/proof/1913.json b/processed_dataset/proof/1913.json new file mode 100644 index 0000000000000000000000000000000000000000..85cf65cd1649e9bbd0c46ad39c0b23d8120f49d1 --- /dev/null +++ b/processed_dataset/proof/1913.json @@ -0,0 +1,10 @@ +{ + "source_file": "./raw_volume-zh/volume7/exercise8.tex", + "problem_type": "proof", + "problem": "问题6. 已知 $\\triangle A B C$ 的外心为 $O, P$ 为 $O A$ 延长线上一点,直线 $l$ 与 $P B$ 关于 $B A$ 对称, 直线 $h$ 与 $P C$ 关于 $A C$ 对称, $l$ 与 $h$ 交于点 $Q$. 若 $P$ 在 $O A$ 的延长线上运动,求 $Q$ 的轨迹.", + "solution": "证明: 如图(), 延长 $A O$ 至 $R$, 使得 $O R=O A$, 连结 $R C, P C$, 设 $h$ 与 $A O$ 交于点 $Q_1$, 注意到 $A C \\perp C R$ 以及 $h 、 C P$ 关于 $A C$ 边对称, 于是 $C R 、 C A 、 h 、 C P$ 成调和线束, 于是 $R 、 A 、 Q_1 、 P$ 成调和点列, 因此 $h$ 过 $A O$ 上满足 $P 、 Q_0 、 A 、 R$ 成调和点列的点 $Q_0$, 同理, $l$ 也过该点, 即 $Q=Q_0$ 为 $A O$ 上满足 $P 、 Q_0 、 A 、 R$ 成调和点列的点,故 $Q$ 的轨迹为线段 $A O$ 内部.\n(注: 在学了下一章反演的知识后, 读者就会发现, $Q$ 与 $P$ 关于 $\\triangle A B C$ 外接圆互为反演点.)", + "remark": "", + "figures": [ + "./images/volume7/figures/fig-c8a6.png" + ] +} \ No newline at end of file diff --git a/processed_dataset/proof/1914.json b/processed_dataset/proof/1914.json new file mode 100644 index 0000000000000000000000000000000000000000..1befb54049f9c5f79bea2536370af564acd33674 --- /dev/null +++ b/processed_dataset/proof/1914.json @@ -0,0 +1,10 @@ +{ + "source_file": "./raw_volume-zh/volume7/exercise8.tex", + "problem_type": "proof", + "problem": "问题7. 在 $\\triangle A B C$ 中, 经过点 $B 、 C$ 的圆与边 $A C 、 A B$ 的另一个交点分别为 $E 、 F, B E$ 与 $C F$ 交于点 $P, A P$ 与 $B C$ 交于点 $D 、 M$ 是边 $B C$ 的中点, $D 、 M$ 不重和.\n求证: $D 、 M 、 E 、 F$ 四点共圆.", + "solution": "证明: 如图(), 设直线 $B C$ 与 $E F$ 交于点 $Q$, 由性质 2 知, $Q 、 D 、 B 、 C$ 成调和点列, 又 $M$ 为 $B C$ 中点, 于是, 不难证明 $Q D \\cdot Q M=Q B \\cdot Q C$, 因此, $Q E \\cdot Q F=Q B \\cdot Q C=Q D \\cdot Q M$, 即 $E 、 M$ 、 $D 、 F$ 四点共圆.", + "remark": "", + "figures": [ + "./images/volume7/figures/fig-c8a7.png" + ] +} \ No newline at end of file diff --git a/processed_dataset/proof/1915.json b/processed_dataset/proof/1915.json new file mode 100644 index 0000000000000000000000000000000000000000..224baed2d1ee5e54b2a054d3d2c6f7ad27b5751c --- /dev/null +++ b/processed_dataset/proof/1915.json @@ -0,0 +1,10 @@ +{ + "source_file": "./raw_volume-zh/volume7/exercise8.tex", + "problem_type": "proof", + "problem": "问题8. 在四边形 $A B C D$ 中, 对角线 $A C$ 平分 $\\angle B A D$, 在 $C D$ 上取一点 $E, B E$ 与 $A C$ 交于点 $F$, 延长 $D F$ 交 $B C$ 于点 $G$. 求证: $\\angle G A C=\\angle E A C$.", + "solution": "证明: 如图(), 设 $A C$ 交 $B D 、 G E$ 于点 $H 、 M$, 延长 $G E$ 与 $B D$ 交于点 $N$, 则 $D 、 B 、 H 、 N$ 成调和点列.\n由 $A C$ 平分 $\\angle B A D$ 知 $A H \\perp A N$, 又由定理 1, 以 $C$ 为中心, 知 $E 、 G 、 M 、 N$ 成调和点列, 且 $A M \\perp A N$, 所以, $A C$ 平分 $\\angle G A E$. 故 $\\angle G A C=\\angle E A C$.", + "remark": "", + "figures": [ + "./images/volume7/figures/fig-c8a8.png" + ] +} \ No newline at end of file diff --git a/processed_dataset/proof/1916.json b/processed_dataset/proof/1916.json new file mode 100644 index 0000000000000000000000000000000000000000..9da9be80b58af8d2176b07f4cf0d41e1c8161584 --- /dev/null +++ b/processed_dataset/proof/1916.json @@ -0,0 +1,10 @@ +{ + "source_file": "./raw_volume-zh/volume7/exercise8.tex", + "problem_type": "proof", + "problem": "问题9. 在 $\\triangle A B C$ 中, $A B>A C$, 它的内切圆切边 $B C$ 于点 $E$, 连结 $A E$ 交内切圆于点 $D$ (不同于点 $E$ ). 在线段 $A E$ 上取异于 $E$ 的一点 $F$, 使得 $C E=C F$, 连结 $C F$ 并延长交 $B D$ 于点 $G$. 求证: $C F=F G$.", + "solution": "证明: 如图(), 过 $D$ 作内切圆切线 $D H$ 交直线 $B C$ 于 $H$, 由 $C F=C E$, $H D=H E$ 知 $\\triangle C E F \\backsim \\triangle H E D$, 于是 $D H / / G C$, 由例 2 证明过程知, $B 、 C 、E 、 H$ 成调和点列, 于是 $D B 、 D C 、 D E 、 D H$ 成调和线束, 所以 $F$ 平分 $C G$.", + "remark": "", + "figures": [ + "./images/volume7/figures/fig-c8a9.png" + ] +} \ No newline at end of file diff --git a/processed_dataset/proof/1917.json b/processed_dataset/proof/1917.json new file mode 100644 index 0000000000000000000000000000000000000000..c5bb1addf692b474387cd4f8f9c6894421d33444 --- /dev/null +++ b/processed_dataset/proof/1917.json @@ -0,0 +1,11 @@ +{ + "source_file": "./raw_volume-zh/volume7/exercise8.tex", + "problem_type": "proof", + "problem": "问题10. 如图(), $O 、 I$ 分别是 $\\triangle A B C$ 的外心、内心, $A D$ 是边 $B C$ 上的高, $I$ 在线段 $O D$ 上.\n求证: $\\triangle A B C$ 的外接圆半径等于边 $B C$ 上的旁切圆半径.", + "solution": "证明: 如图(), 设 $I_A$ 为旁心, $A I_A$ 交 $B C$ 于点 $E$, 交 $\\odot O$ 于点 $M$, 则 $M$ 为 $\\overparen{B C}$ 的中点.\n连结 $O M$, 则 $O M \\perp B C$. 作 $I_A F \\perp B C$ 于 $F$, 则由平行线性质, 有\n$$\n\\frac{A D}{A I}=\\frac{O M}{M I}(*), \\frac{A D}{I_A F}=\\frac{A E}{I_A E} .\n$$\n由性质 7 的推论 2 , 有 $\\frac{A I}{I E}=\\frac{I_A M}{M E}$, 即有 $\\frac{A I}{I_A M}=\\frac{I E}{M E}= \\frac{A I+I E}{I_A M+M E}=\\frac{A E}{I_A E}$. 从而 $\\frac{A D}{I_A F}=\\frac{A I}{I_A M}$, 亦即 $\\frac{A D}{A I}=\\frac{I_A F}{I_A M}$. \n注意到 (*) 式及 $M I=I_A M$. 故 $O M=I_A F$. 即 $\\triangle A B C$ 的外接圆半径 $O M$ 等于边 $B C$ 上的旁切圆半径 $I_A F$.", + "remark": "", + "figures": [ + "./images/volume7/figures/fig-c8p10.png", + "./images/volume7/figures/fig-c8a10.png" + ] +} \ No newline at end of file diff --git a/processed_dataset/proof/1918.json b/processed_dataset/proof/1918.json new file mode 100644 index 0000000000000000000000000000000000000000..3b1342ba519cfe8ef40a9464650f1d47ac33d4ad --- /dev/null +++ b/processed_dataset/proof/1918.json @@ -0,0 +1,11 @@ +{ + "source_file": "./raw_volume-zh/volume7/exercise8.tex", + "problem_type": "proof", + "problem": "问题11. 如图(), 在 $\\triangle A B C$ 中, 设 $A B>A C$. 过点 $A$ 作 $\\triangle A B C$ 的外接圆的切线 $l$. 又以 $A$ 为圆心, $A C$ 为半径作圆分别交线段 $A B$ 于点 $D$, 交直线 $l$ 于点 $E 、 F$. 证明: 直线 $D E 、 D F$ 分别通过 $\\triangle A B C$ 的内心与一个旁心.", + "solution": "证明: 如图(), 作 $\\angle B A C$ 的平分线, 交 $D E$ 于 $I$, 易知 $\\triangle A D I \\cong \\triangle A C I$. 所以 $\\angle A C I=\\frac{1}{2}\\left(180^{\\circ}-\\angle B A C-\\right. \\angle A B C)=\\frac{1}{2} \\angle A C B$. 从而 $I$ 为 $\\triangle A B C$ 的内心.\n设射线 $A I$ 交 $B C$ 于 $M$, 交 $\\triangle A B C$ 的外接圆于 $A_1$, 交直线 $F D$ 于 $I_A$. 连 $C I_A$, 则知 $\\angle D I_A A=\\angle A I_A C$.\n延长 $C B$ 到 $P$, 使 $P B=B A$, 则 $\\angle A P C= \\frac{1}{2} \\angle A B C=\\frac{1}{2} \\angle B$.\n注意到 $\\frac{1}{2}(\\angle A+\\angle B+\\angle C)=90^{\\circ}=\\angle F D A+\\angle A D E= \\left(\\frac{1}{2} \\angle A+\\angle D I_A A\\right)+\\angle I C A=\\left(\\frac{1}{2} \\angle A+\\angle A I_A C\\right)+\\frac{1}{2} \\angle C$,\n从而 $\\angle A I_A C=\\frac{1}{2} \\angle B=\\angle A P C$, 于是, $A 、 P 、 I_A 、 C$ 四点共圆, 有 $\\angle A I_A P= \\angle A C P=\\angle A A_1 B$, 即有 $B A_1 / / P I_A$, 亦即有 $\\frac{I_A A_1}{A_1 M}=\\frac{P B}{B M}=\\frac{A B}{B M}=\\frac{A I}{I M}$.\n由性质 7 的推论 2 , 知 $I_A$ 是边 $B C$ 外的旁心.", + "remark": "", + "figures": [ + "./images/volume7/figures/fig-c8p11.png", + "./images/volume7/figures/fig-c8a11.png" + ] +} \ No newline at end of file diff --git a/processed_dataset/proof/1919.json b/processed_dataset/proof/1919.json new file mode 100644 index 0000000000000000000000000000000000000000..7b2403fc69aba445b763a6630314e73b1b786365 --- /dev/null +++ b/processed_dataset/proof/1919.json @@ -0,0 +1,11 @@ +{ + "source_file": "./raw_volume-zh/volume7/exercise8.tex", + "problem_type": "proof", + "problem": "问题12. 如图(), $A D$ 为 $\\triangle A B C$ 的内角平分线, $\\angle A D C=60^{\\circ}$, 点 $M$ 在 $A D$ 上, 满足 $D M=D B$, 射线 $B M 、 C M$ 交 $A C 、 A B$ 于点 $E 、 F$. 证明: $D F \\perp E F$.", + "solution": "证明: 如图(), 在 $A B$ 上取 $A S=A C$, 连结 $S M, D S$, 则由 $A D$ 平分 $\\angle B A C$ 知, $\\triangle A S D$ 与 $\\triangle A C D$ 关于 $A D$ 对称, 即有 $\\angle M D S= \\angle M D C=\\angle A D C=60^{\\circ}$, 亦即 $\\angle B D S=60^{\\circ}$. 又由 $D M=D B$, 知 $\\angle D S B=\\angle D S M=\\angle D C M$. 于是 $\\angle F C B+\\angle A C B=\\angle D C M+\\angle A C B=\\angle D S B+\\angle A S D= 180^{\\circ}$, 从而 $\\sin \\angle F C B=\\sin \\angle A C B$.\n由 $\\frac{F B}{F C}=\\frac{\\sin \\angle F C B}{\\sin \\angle F B C}=\\frac{\\sin \\angle A C B}{\\sin \\angle A B C}=\\frac{A B}{A C}=\\frac{B D}{D C}$, 知 $F D$ 平分 $\\angle B F C$.\n设直线 $F E$ 与直线 $B C$ 相交于点 $G$ (因角平分线 $A D$ 交 $B C$ 成 $60^{\\circ}$ 角, 必相交), 则由完全四边形对角线调和分割的性质, 知 $D 、 G$ 调和分割 $B C$, 即 $B$ 、 $C 、 D 、 G$ 为调和点列, 亦即 $F B 、 F C 、 F D 、 F G$ 为调和线束.\n而 $F D$ 平分 $\\angle B F C$, 则由性质 5 知 $D F \\perp E F$.", + "remark": "", + "figures": [ + "./images/volume7/figures/fig-c8p12.png", + "./images/volume7/figures/fig-c8a12.png" + ] +} \ No newline at end of file diff --git a/processed_dataset/proof/1920.json b/processed_dataset/proof/1920.json new file mode 100644 index 0000000000000000000000000000000000000000..c9af9d0b6dd76ac8bf589e1f7a5aa1a2b3afd153 --- /dev/null +++ b/processed_dataset/proof/1920.json @@ -0,0 +1,10 @@ +{ + "source_file": "./raw_volume-zh/volume7/exercise9.tex", + "problem_type": "proof", + "problem": "问题1. 设 $\\triangle A B C$ 的内切圆 $\\Gamma$ 与 $B C$ 切于点 $D, D^{\\prime}$ 是圆 $\\Gamma$ 上的点, 且 $D D^{\\prime}$ 为圆 $\\Gamma$ 的直径, 过 $D^{\\prime}$ 作圆 $\\Gamma$ 的切线与 $A D$ 交于点 $X$, 过 $X$ 作圆 $\\Gamma$ 的不同于 $X D^{\\prime}$ 的切线,切点为 $N$. 证明: $\\triangle B C N$ 的外接圆与圆 $\\Gamma$ 切于点 $N$.", + "solution": "证明: 显然, $A B \\neq A C$. 下设 $A B> A C$. 如图(), 设圆 $\\Gamma$ 与 $A C 、 A B$ 分别切于点 $E 、 F$, 且设 $F E$ 与 $B C$ 交于点 $K$. 则 $K$ 是点 $A$ 关于圆 $\\Gamma$ 的极线 $F E$ 上的点.\n由配极原则知, $A$ 也是点 $K$ 关于圆 $\\Gamma$ 的极线上的点.\n因为点 $D$ 在点 $K$ 关于圆 $\\Gamma$ 的极线上,所以, $K$ 关于圆 $\\Gamma$ 的极线为 $A D$. 同理, 设 $D^{\\prime} N$ 与 $B C$ 交于点 $K^{\\prime}$, 则 $K^{\\prime}$ 关于圆 $\\Gamma$ 的极线为 $D X$. 由于 $A D$\n与 $D X$ 为同一条直线, 因此, $K^{\\prime}=K$. 因为 $B, C ; D, K$ 是调和点列, 且 $\\angle D^{\\prime} N D=90^{\\circ}$, 所以, $N D$ 是 $\\angle B N C$ 的角平分线.\n设 $N B 、 N C$ 分别与圆 $\\Gamma$ 交于点 $P 、 Q$. 则 $D$ 为弧 $\\widehat{P Q}$ 的中点.\n于是, $P Q / / B C$. 由 $\\angle X N P=\\angle P Q N=\\angle B C N$, 知 $X N$ 与 $\\triangle B C N$ 的外接圆切于点 $N$. 从而, $\\triangle B C N$ 的外接圆与圆 $\\Gamma$ 切于点 $N$.", + "remark": "", + "figures": [ + "./images/volume7/figures/fig-c9a1.png" + ] +} \ No newline at end of file diff --git a/processed_dataset/proof/1921.json b/processed_dataset/proof/1921.json new file mode 100644 index 0000000000000000000000000000000000000000..3cbcb1b8e7b239b7b82657759318b06d522d80c4 --- /dev/null +++ b/processed_dataset/proof/1921.json @@ -0,0 +1,10 @@ +{ + "source_file": "./raw_volume-zh/volume7/exercise9.tex", + "problem_type": "proof", + "problem": "问题2. 圆 $O_1$ 与圆 $O_2$ 交于 $A 、 B$ 两点.\n过点 $O_1$ 的直线 $D C$ 交圆 $O_1$ 于 $D$ 且切圆 $O_2$ 于 $C, C A$ 切圆 $O_1$ 于 $A$, 圆 $O_1$ 的弦 $A E$ 与直线 $D C$ 垂直.\n过 $A$ 作 $A F$ 垂直于 $D E, F$ 为垂足.\n求证: $B D$ 平分线段 $A F$.", + "solution": "证明: 如图(), 延长 $C B$ 交 $\\odot O_1$ 于 $G$, 对于 $\\odot O_1$, 由 $C A$ 是切线, $C O_1 \\perp A E$ 知, 直线 $A E$ 是 $C$ 的极线,故 $E C$ 是过 $E$ 点的 $\\odot O_1$ 的切线.\n由配极定理 3 知四边形 $A B E G$ 为调和四边形.\n由配极定理 $2, D G 、 D B 、 D A 、 D E$ 成调和线束.\n设 $A E$ 与 $C D$ 交点为 $H$, 连 $A B 、 B E 、 G D$, 又 $\\angle B C D=\\angle B A C=\\angle B E A$, 所以 $H 、 E$ 、 $C 、 B$ 四点共圆.\n于是 $\\angle E B C=\\angle E H C=90^{\\circ}$, 故 $\\angle G D E=\\angle E B C=90^{\\circ}$, 即 $D G / / A F$, 结合\n$D G 、 D B 、 D A 、 D E$ 成调和线束知 $D B$ 平分 $A F$.", + "remark": "", + "figures": [ + "./images/volume7/figures/fig-c9a2.png" + ] +} \ No newline at end of file diff --git a/processed_dataset/proof/1922.json b/processed_dataset/proof/1922.json new file mode 100644 index 0000000000000000000000000000000000000000..fddd7aed3957325b8a41598b0067e9553a3a49a6 --- /dev/null +++ b/processed_dataset/proof/1922.json @@ -0,0 +1,10 @@ +{ + "source_file": "./raw_volume-zh/volume7/exercise9.tex", + "problem_type": "proof", + "problem": "问题3. 凸四边形 $A B C D$ 外切于 $\\odot O, A B 、 B C 、 C D 、 D A$ 上的切点分别是 $E 、 F 、 G 、 H$, 直线 $H E$ 与 $F G$ 相交于点 $P$. 求证: $O P \\perp A C$.", + "solution": "证明: 如图(), 以 $\\odot O$ 为基圆, 易知 $H E$ 是 $A$ 的极线, $F G$ 是 $C$ 的极线.\n而 $H E$ 与 $F G$ 交于 $P$, 因此 $P$ 点既在 $A$ 点极线上,也在 $C$ 点极线上由配极性质 1 知,故 $A 、 C$ 都在 $P$ 点极线上, 从而 $A C$ 即为 $P$ 点的极线.\n因此 $O P \\perp A C$. 原命题得证.", + "remark": "", + "figures": [ + "./images/volume7/figures/fig-c9a3.png" + ] +} \ No newline at end of file diff --git a/processed_dataset/proof/1923.json b/processed_dataset/proof/1923.json new file mode 100644 index 0000000000000000000000000000000000000000..a8e1d3efa3db6ff57edad52e335cb4090bc40a93 --- /dev/null +++ b/processed_dataset/proof/1923.json @@ -0,0 +1,10 @@ +{ + "source_file": "./raw_volume-zh/volume7/exercise9.tex", + "problem_type": "proof", + "problem": "问题5. 设 $P$ 为 $\\triangle A B C$ 内一点, 令 $\\alpha=\\angle B P C-\\angle A, \\beta=\\angle C P A-\\angle B, \\gamma= \\angle A P B-\\angle C$. 求证: $\\frac{P A \\cdot \\sin A}{\\sin \\alpha}=\\frac{P B \\cdot \\sin B}{\\sin \\beta}=\\frac{P C \\cdot \\sin C}{\\sin \\gamma}$.", + "solution": "证明: 如图(), 以 $P$ 为反演中心, 单位长度为反演幂, 设 $A 、 B 、 C$ 的反点分别为 $A^{\\prime} 、 B^{\\prime} 、 C^{\\prime}$, 因点 $P$ 在 $\\triangle A B C$ 内, 所以, 点 $P$ 也在 $\\triangle A^{\\prime} B^{\\prime} C^{\\prime}$ 内, 由定理 $1, \\angle B^{\\prime} A^{\\prime} P=\\angle P B A, \\angle P A^{\\prime} C^{\\prime}=\\angle A C P$, 所以 $\\angle B^{\\prime} A^{\\prime} C^{\\prime}=\\angle P B A+\\angle A C P=\\angle B P C-$. $\\angle A=\\alpha$; 同理 $\\angle C^{\\prime} B^{\\prime} A^{\\prime}=\\beta, \\angle A^{\\prime} C^{\\prime} B^{\\prime}=\\gamma$. 又由定理 2, 有 $B^{\\prime} C^{\\prime}=\\frac{B C}{P B \\cdot P C}, C^{\\prime} A^{\\prime}=\\frac{C A}{P C \\cdot P A}$, $B^{\\prime} A^{\\prime}=\\frac{A B}{P A \\cdot P B}$, 对 $\\triangle A^{\\prime} B^{\\prime} C^{\\prime}$ 用正弦定理并将上面三式代入即得\n$$\n\\frac{P A \\cdot B C}{\\sin \\alpha}=\\frac{P B \\cdot C A}{\\sin \\beta}=\\frac{P C \\cdot A B}{\\sin \\gamma} \\text {, 即等价于所证.\n}\n$$", + "remark": "", + "figures": [ + "./images/volume7/figures/fig-c9a5.png" + ] +} \ No newline at end of file diff --git a/processed_dataset/proof/1924.json b/processed_dataset/proof/1924.json new file mode 100644 index 0000000000000000000000000000000000000000..8a7e63cf6b801a5444e64d5845ef4251d15250b6 --- /dev/null +++ b/processed_dataset/proof/1924.json @@ -0,0 +1,11 @@ +{ + "source_file": "./raw_volume-zh/volume7/exercise9.tex", + "problem_type": "proof", + "problem": "问题6. 如图(), 在弓形中, 内接一对相切的圆, 对每一对相切的圆, 通过它们的切点引公切线.\n证明: 所有的切线通过一个点.", + "solution": "证明: 法一: 如图(), 设 $P$ 是两圆 $\\odot O_1 、 \\odot O_2$ 的切点.\n作以 $P$ 为反演中心的反演变换, 于是, 在点 $P$ 处相切的两圆反形为一对平行直线 $l_1 / / l_2$, 而和它们相切的弦和弧, 变为 $A^{\\prime} L^{\\prime} B^{\\prime}$ 和 $A^{\\prime} K^{\\prime} B^{\\prime}$, 且 $\\widehat{A^{\\prime} L^{\\prime} B^{\\prime}}=\\widehat{A^{\\prime} K^{\\prime} B^{\\prime}}$, 公切线 $K L$ 变为 $K^{\\prime} L^{\\prime}$, 且与 $l_1$ 、 $l_2$ 平行.\n所以,直线 $A^{\\prime} B^{\\prime}$ 垂直平分 $K^{\\prime} L^{\\prime}$. 换言之, 过点 $A 、 P 、 B$ 的弧平分弓形角 $A 、 B$ 且垂直直线 $K L$. 然而, 恰存在一个过点 $A 、 B$ 的圆, 平分 $\\angle A 、 \\angle B$ (它的中心 $O$ 是从点 $A 、 B$ 分别向 $\\angle A 、 \\angle B$ 的平分线引的垂线的交点), 直线 $K L$ 垂直这个圆, 因此, 通过它的中心.\n于是, 条件中所有直线都通过点 $O$.", + "remark": "", + "figures": [ + "./images/volume7/figures/fig-c9p6.png", + "./images/volume7/figures/fig-c9a6-1.png" + ] +} \ No newline at end of file diff --git a/processed_dataset/proof/1925.json b/processed_dataset/proof/1925.json new file mode 100644 index 0000000000000000000000000000000000000000..e977fc1cd6475a8ba065cef8218402e68bd17c90 --- /dev/null +++ b/processed_dataset/proof/1925.json @@ -0,0 +1,11 @@ +{ + "source_file": "./raw_volume-zh/volume7/exercise9.tex", + "problem_type": "proof", + "problem": "问题6. 如图(), 在弓形中, 内接一对相切的圆, 对每一对相切的圆, 通过它们的切点引公切线.\n证明: 所有的切线通过一个点.", + "solution": "法二: 如图(), 连结两个切点 $T 、 S$ 及 $V 、 U$. 设它们相交于 $M$. 则由 \"圆的初步\" 习题 19 的引理知 $M$ 为优弧 $A B$ 的中点 ( $M$ 为定点). 且由 $\\angle B A M=\\angle B T M= \\angle A T M$ 有 $\\triangle A S M \\backsim \\triangle T A M$, 从而 $M A^2=M S \\cdot M T$. 同理 $M B^2=M U \\cdot M V$. 故 $M S \\cdot M T=M U \\cdot M V, M$ 在 $\\odot O_1$ 与 $\\odot O_2$ 的根轴上.\n而 $K L$ 是两圆的公切线, 也是两圆的根轴.\n故 $M$ 在 $K L$ 上, 即所有切线都过定点 $M$.", + "remark": "", + "figures": [ + "./images/volume7/figures/fig-c9p6.png", + "./images/volume7/figures/fig-c9a6-2.png" + ] +} \ No newline at end of file diff --git a/processed_dataset/proof/1926.json b/processed_dataset/proof/1926.json new file mode 100644 index 0000000000000000000000000000000000000000..14cb4ba08adfcc344399b118ea90a22a01f4fd82 --- /dev/null +++ b/processed_dataset/proof/1926.json @@ -0,0 +1,11 @@ +{ + "source_file": "./raw_volume-zh/volume7/exercise9.tex", + "problem_type": "proof", + "problem": "问题7. 如图(), 在线段 $A B$ 上取点 $C$, 以线段 $A C 、 B C 、 A B$ 为直径分别作圆, $\\odot O$ 与这三个圆都相切.\n证明: $\\odot O$ 的直径等于它的圆心到直线 $A B$ 的距离.", + "solution": "证明: 如图(), 以点 $C$ 为反演中心作反演变换.\n以 $A C$ 、 $B C 、 A B$ 为直径的圆分别反演成直线 $A^{\\prime} D 、 B^{\\prime} E$ 、以 $A^{\\prime} B^{\\prime}$ 为直径的圆, 且直线 $A^{\\prime} D 、 B^{\\prime} E$ 与 $A^{\\prime} B^{\\prime}$ 垂直, $\\odot O$ 反演成 $\\odot O^{\\prime}$, 且与直线 $A^{\\prime} D 、 B^{\\prime} E$ 及以 $A^{\\prime} B^{\\prime}$ 为直径的圆都相切.\n由于 $\\odot O 、 \\odot O^{\\prime}$ 关于点 $C$ 位似, 所以, $\\odot O$ 的直径与圆心到 $A B$ 的距离的比等于 $\\odot O^{\\prime}$ 的直径与圆心到 $A^{\\prime} B^{\\prime}$ 的距离的比.\n易知后者的比值为 1 .", + "remark": "", + "figures": [ + "./images/volume7/figures/fig-c9p7.png", + "./images/volume7/figures/fig-c9a7.png" + ] +} \ No newline at end of file diff --git a/processed_dataset/proof/1927.json b/processed_dataset/proof/1927.json new file mode 100644 index 0000000000000000000000000000000000000000..435e6ef5a2f05f145dc0c8a5b0860b890d65047c --- /dev/null +++ b/processed_dataset/proof/1927.json @@ -0,0 +1,10 @@ +{ + "source_file": "./raw_volume-zh/volume7/exercise9.tex", + "problem_type": "proof", + "problem": "问题8. 已知圆内接四边形 $A B C D$, 直线 $A D$ 和 $B C$ 交于点 $E$, 且点 $C$ 在点 $B 、 E$ 之间, 对角线 $A C 、 B D$ 交于 $F$, 设点 $M$ 为边 $C D$ 的中点, 点 $N$ 是 $\\triangle A B M$ 的外接圆上的不同于 $M$ 的点, 且满足 $\\frac{A N}{B N}=\\frac{A M}{B M}$. 证明: $E 、 F 、 N$ 三点共线.", + "solution": "证明: 如图(), 延长 $C D 、 B A$ 交于点 $P$, 则直线 $E F$ 即为点 $P$ 关于 $\\odot O$ 的极线.\n(定理 1) 欲证 $N$ 在直线 $E F$ 上, 只需证 $N$ 对 $\\odot O$ 的极线过 $P$ 点.\n设 $\\triangle A M B$ 外接圆为 $\\odot O^{\\prime}$, 因为 $A 、 M 、 B 、 N$ 均在 $\\odot O^{\\prime}$ 上, 且 $\\frac{A M}{M B}= \\frac{A N}{N B} \\cdot A M \\cdot N B=A N \\cdot M B$, 故 $A M B N$ 为调和四边形.\n所以点 $M 、 N$\n处的两条切线交于直线 $A B$ 上,设为 $R$, 取 $A B$ 中点 $S$.\n可知, 点 $N$ 对 $\\odot O$ 的幂等于点 $N$ 对 $\\triangle O S M$ 外接圆的幂.\n过 $P$ 作 $P W \\perp O N$ 于 $W$, 则 $O 、 M 、 P 、 W 、 S$ 共圆.\n则 $N$ 为 $\\odot O$ 的幂等于 $N$ 对 $\\triangle O W P$ 外接圆的幂.\n从而 $P$ 点在 $N$ 关于 $\\odot O$ 的极线上,故结论成立,得证.", + "remark": "", + "figures": [ + "./images/volume7/figures/fig-c9a8.png" + ] +} \ No newline at end of file diff --git a/processed_dataset/proof/1928.json b/processed_dataset/proof/1928.json new file mode 100644 index 0000000000000000000000000000000000000000..5dcba137ad5723ed39c5d1be48c49d9569aeb285 --- /dev/null +++ b/processed_dataset/proof/1928.json @@ -0,0 +1,10 @@ +{ + "source_file": "./raw_volume-zh/volume7/exercise9.tex", + "problem_type": "proof", + "problem": "问题9. 已知三角形 $A B C$ 及其内切圆 $O, E 、 F 、 G$ 分别为 $B C 、 B A 、 A C$ 边上的切点, $H$ 为边 $B C$ 上高 $A D$ 的中心, $E H$ 交圆 $O$ 于 $I$. 求证: $I E$ 平分 $\\angle B I C$.", + "solution": "证明: 如图(), 设直线 $G F$ 与直线 $B C$ 交于点 $K$, 延长 $K I$ 交圆 $O$ 于 $M$,并设 $A E$ 与 $I M$ 交于点 $L$, 连结 $M E$. 由上章例 2 的证明过程知 $C 、 B 、 E 、 K$ 成调和点列, 又注意到 $A$ 的极线 $F G$ 过 $K, E$ 的极线 $B C$ 过 $K$, 所以 $K$ 的极线过 $A 、 E$, 即 $K$ 的极线为直线 $A E$,于是, $K 、 L$ 共轭, 故 $K 、 L 、 I 、 M$ 成调和点列, 于是从 $E$ 点出发的线束 $E K 、 E L 、 E I 、 E M$ 或 $E D 、 E A 、 E H 、 E M$ 成调和线束, 结合 $D H=H A$, 于是 $D 、 A 、 H 、 \\infty$ 成调和点列, 仍由 $E$ 点出发知, $E D 、 E A 、 E H 、 E \\infty$ (即过 $E$ 平行于直线 $D A$ 的直线) 成调和线束, 那么 $E \\infty$ 与 $E M$ 重合, 即 $E M / / D A$, 或 $E M \\perp B C$, 注意到 $B C$ 是 $E$ 点处切线即 $O E \\perp B C$, 故 $M 、 O 、 E$ 三点共线, 于是 $M E$ 是圆 $O$ 的直径, $K I \\perp I E$, 结合 $K 、 E 、 B 、 C$ 成调和点列, 于是 $I E$ 平分 $\\angle B I C$.", + "remark": "", + "figures": [ + "./images/volume7/figures/fig-c9a9.png" + ] +} \ No newline at end of file diff --git a/processed_dataset/proof/1929.json b/processed_dataset/proof/1929.json new file mode 100644 index 0000000000000000000000000000000000000000..f74a78dc99d6abe5e2ad9a8ce7a180a1694ab358 --- /dev/null +++ b/processed_dataset/proof/1929.json @@ -0,0 +1,10 @@ +{ + "source_file": "./raw_volume-zh/volume7/exercise9.tex", + "problem_type": "proof", + "problem": "问题10. 在四个圆中,每个圆都和其他的两个圆外切.\n证明:四个切点位于同一个圆上.", + "solution": "证明: 设这四个圆为圆 $A$ 、圆 $B$ 、圆 $C$ 、 圆 $D$,\n取 $A 、 B$ 的公切点 $E$, 以 $E$ 为反演中心, 单位长度为反演半径作反演变换得到以下命题:\n如图(), 已知直线 $l_1, l_2$ 平行, 有圆 $O_1$ 和圆 $O_2$ 相切于点 $X$, 且 $O_1$ 与 $l_1$ 切于 $Y, O_2$ 与 $l_2$ 切于 $Z$, 则 $X 、 Y 、 Z$ 三点共线,该结论由三角形的相似得证.", + "remark": "", + "figures": [ + "./images/volume7/figures/fig-c9a10.png" + ] +} \ No newline at end of file diff --git a/processed_dataset/proof/1930.json b/processed_dataset/proof/1930.json new file mode 100644 index 0000000000000000000000000000000000000000..88f41d72fb3be9444e675f0b842665f331f1e14f --- /dev/null +++ b/processed_dataset/proof/1930.json @@ -0,0 +1,10 @@ +{ + "source_file": "./raw_volume-zh/volume7/exercise9.tex", + "problem_type": "proof", + "problem": "问题11. 已知 $\\triangle A B C$ 的中线 $A M$ 交其内切圆 $\\Gamma$ 于点 $K 、 L$, 分别过 $K 、 L$ 且平行于 $B C$ 的直线交圆 $\\Gamma$ 于点 $X 、 Y, A X 、 A Y$ 分别交 $B C$ 于点 $P 、 Q$. 证明: $B P=C Q$.", + "solution": "证明: 如图(), 设内切圆圆心为 $I, \\odot I$ 在 $B C$ 上切点为 $H$. 过 $A$ 作平行于 $B C$ 的直线 $l$, 设内切圆在边 $A B 、 A C$ 上的切点分别为 $D 、 E$, 设直线 $D E$ 与 $l$ 交于点 $F$, 又设 $D E$ 交 $A M$ 于 $G$, 由于 $M$ 平分线段 $A B$, 故 $A C 、 A B 、 A M 、 l$ 成调和线束, 或 $A E 、 A D 、 A G 、 A F$ 成调和线束, 于是 $E 、 D 、 G 、 F$ 成调和点列, 结合配极性质 3 知 $F$ 的极线过 $G$, 又 $A$ 的极线 $D E$ 过 $F$, 于是 $F$ 的极线也过 $A$, 那么 $F$ 的极线是直线 $A G$, 又 $G$ 的极线过 $A$, 所以 $G$ 的极线为 $A F$, 又 $A F / / B C$, 所以 $G$ 的极线 $/ / H$ 的切线 $B C$, 于是 $G H$ 的极点为无穷远点 (无穷远点的极线过圆心), 因此 $G H$ 过内心 $I$. 也就是说 $G I \\perp B C$, 以及 $B C / / L Y / / X K, X 、 K ; L 、 Y$ 分别关于直线 $I G$ 对称, 故 $X 、 G 、 Y$ 共线, $M P=X K \\cdot \\frac{A K}{A M}=L Y \\cdot \\frac{K G}{G M} \\cdot \\frac{A K}{A M}=L Y \\cdot \\frac{A L}{A M}$ (这步用到 $A$ 、 $G 、 K 、 L$ 成调和点列 $)=M Q$. 于是 $B P=C Q$.", + "remark": "", + "figures": [ + "./images/volume7/figures/fig-c9a11.png" + ] +} \ No newline at end of file diff --git a/processed_dataset/proof/1931.json b/processed_dataset/proof/1931.json new file mode 100644 index 0000000000000000000000000000000000000000..b7ff2482ac7eb02207100f6b86ac961359dd7fb2 --- /dev/null +++ b/processed_dataset/proof/1931.json @@ -0,0 +1,10 @@ +{ + "source_file": "./raw_volume-zh/volume7/exercise9.tex", + "problem_type": "proof", + "problem": "问题12. 四边形 $A B C D$ 有内切圆 $\\odot I, E 、 F 、 G 、 H$ 分别是 $\\odot I$ 在四边 $A B 、 B C$ 、 $C D 、 D A$ 上的切点.\n求证: $A C 、 B D 、 E G 、 F H$ 四线共点.", + "solution": "证明: 如图(), 设 $A B$ 和 $C D$ 交于点 $O, A D$ 和 $B C$ 交于点 $P$. 则对于 $\\odot I, O$ 的极线为 $E G, P$ 的极线为 $F H$, 于是 $E G$ 和 $F H$ 交于点 $X, X$ 为 $O P$ 的极点.\n令 $E H$ 交 $F G$ 于点 $N, E F$ 交 $G H$ 于点 $M$, 则 $M$ 为 $D$ 的极线与 $B$ 的极线的交点, 因此 $M$ 的极线过 $B 、 D$, 即 $M$ 的极线为直线 $B D$. 同理, $N$ 的极线为直线 $A C$, 设 $A C$ 交 $B D$ 于 $Y$, 又由配极定理 1 知, $M$ 的极线过 $N, N$ 的极线过 $M$. 我们发现 $Y$ 的极线过 $M 、 N$. 故 $Y$ 的极线为直线 $M N$. 对六边形 $E E H G G F$ (退化六边形) 使用帕斯卡定理 $O 、 M 、 N$ 共线, 同理 $M 、 N 、 P$ 共线.\n于是 $X$ 的极线 $=Y$ 的极线, 所以 $X=Y$. 即 $A C$ 、 $B D 、 E G 、 F H$ 四线共点.", + "remark": "注:: 此题虽然可以由三角方法解决 (见本章例 13 引理), 但这里我们用配极方法给出另一种解答, 供读者欣赏.", + "figures": [ + "./images/volume7/figures/fig-c9a12.png" + ] +} \ No newline at end of file diff --git a/processed_dataset/proof/1932.json b/processed_dataset/proof/1932.json new file mode 100644 index 0000000000000000000000000000000000000000..8debd54c4ba13b22dba256925dbd2245e5a0a2f3 --- /dev/null +++ b/processed_dataset/proof/1932.json @@ -0,0 +1,8 @@ +{ + "source_file": "./raw_volume-zh/volume8/chapter1.tex", + "problem_type": "proof", + "problem": "例5. 已知两个复系数函数\n$$\nf(x)=\\sum_{k=0}^n a_k x^{n-k}, g(x)=\\sum_{k=0}^n b_k x^{n-k},\n$$\n其中 $a_0=b_0=1, \\sum_{k=1}^{\\left[\\frac{n}{2}\\right]} b_{2 k}$ 和 $\\sum_{k=1}^{\\left[\\frac{n+1}{2}\\right]} b_{2 k-1}$ 均为实数.\n若 $g(x)=0$ 的所有根的平方的相反数是 $f(x)=0$ 的全部根, 求证: $\\sum_{k=1}^n(-1)^k a_k$ 是实数.", + "solution": "分析:与解设方程 $g(x)=0$ 的 $n$ 个根为 $x_k(k=1,2, \\cdots, n)$, 则知方程 $f(x)=0$ 的 $n$ 个根为 $-x_k^2(k=1,2, \\cdots, n)$, 于是, 有\n$$\ng(x)=\\prod_{k=1}^n\\left(x-x_k\\right), f(x)=\\prod_{k=1}^n\\left(x+x_k^2\\right) .\n$$\n$$\n\\text { 从而 } \\begin{aligned}\nf(-1) & =\\prod_{k=1}^n\\left(-1+x_k^2\\right) \\\\\n& =\\prod_{k=1}^n\\left(-1-x_k\\right) \\prod_{k=1}^n\\left(1-x_k\\right) \\\\\n& =g(-1) g(1) .\n\\end{aligned} \\label{eq1}\n$$\n因为 $g(1)=\\sum_{k=0}^n b_k=b_0+\\sum_{k=1}^{\\left[\\frac{n}{2}\\right]} b_{2 k}+\\sum_{k=1}^{\\left[\\frac{n+1}{2}\\right]} b_{2 k-1}$,\n$$\n\\begin{aligned}\ng(-1)=\\sum_{k=0}^n b_k(-1)^{n-k}=(-1)^n b_0+(-1)^{n-1} \\sum_{k=1}^{\\left[\\frac{n+1}{2}\\right]} b_{2 k-1}+(-1)^{n-2} \\sum_{k=1}^{\\left[\\frac{n}{2}\\right]} b_{2 k} \\\\\nf(-1)=\\sum_{k=0}^n a_k(-1)^{n-k} \\\\\n=(-1)^n a_0+\\sum_{k=1}^n(-1)^{n-k} a_k \\\\\n=(-1)^n+(-1)^n \\sum_{k=1}^n(-1)^k a_k,\n\\end{aligned}\n$$\n所以\n$$\n\\sum_{k=1}^n(-1)^k a_k=(-1)^n f(-1)-1 . \\label{eq2}\n$$\n由题设条件知 $g(-1) 、 g(1)$ 均是实数, 注意到等式 \\ref{eq1}, 可知 $f(-1)$ 亦是实数, 从而 \\ref{eq2} 式的右端为实数.\n也即 $\\sum_{k=1}^n(-1)^k a_k$ 为实数, 证毕.", + "remark": "注:考虑 $f(-1)$ 是本题的关键, 它建立了 $f(x)$ 与 $g(x)$ 的一个关系.", + "figures": [] +} \ No newline at end of file diff --git a/processed_dataset/proof/1933.json b/processed_dataset/proof/1933.json new file mode 100644 index 0000000000000000000000000000000000000000..615980f022480a106d80d94505aa94f7f5aa8fc5 --- /dev/null +++ b/processed_dataset/proof/1933.json @@ -0,0 +1,8 @@ +{ + "source_file": "./raw_volume-zh/volume8/chapter1.tex", + "problem_type": "proof", + "problem": "例6. 设 $A 、 B 、 C$ 分别是复数 $z_0=a \\mathrm{i}, z_1=\\frac{1}{2}+b \\mathrm{i}, z_2=1+c \\mathrm{i}$ 对应的不共线的三点 ( $a 、 b 、 c$ 都是实数). 证明: 曲线 $z=z_0 \\cos ^4 t+2 z_1 \\cos ^2 t \\cdot \\sin ^2 t+ z_2 \\sin ^4 t(t \\in \\mathbf{R})$ 与 $\\triangle A B C$ 中平行于 $A C$ 的中位线只有一个公共点, 并求出此点.", + "solution": "分析:与解设 $D 、 E$ 分别为 $A B 、 B C$ 的中点, 则 $D 、 E$ 对应的复数分别为 $\\frac{1}{2}\\left(z_0+z_1\\right)=\\frac{1}{4}+\\frac{a+b}{2} \\mathrm{i}, \\frac{1}{2}\\left(z_1+z_2\\right)=\\frac{3}{4}+\\frac{b+c}{2} \\mathrm{i}$.\n于是,线段 $D E$ 上的点对应的复数 $z$ 满足\n$$\nz=\\lambda\\left(\\frac{1}{4}+\\frac{a+b}{2} \\mathrm{i}\\right)+(1-\\lambda)\\left(\\frac{3}{4}+\\frac{b+c}{2} \\mathrm{i}\\right), 0 \\leqslant \\lambda \\leqslant 1 .\n$$\n代入曲线方程\n$$\nz=z_0 \\cos ^4 t+2 z_1 \\cos ^2 t \\cdot \\sin ^2 t+z_2 \\sin ^4 t,\n$$\n对比两边实部和虚部, 得\n$$\n\\left\\{\\begin{array}{l}\n\\frac{3}{4}-\\frac{\\lambda}{2}=\\sin ^2 t \\cos ^2 t+\\sin ^4 t, \\\\\n\\frac{1}{2}[\\lambda a+b+(1-\\lambda) c]=a \\cos ^4 t+2 b \\sin ^2 t \\cos ^2 t+c \\sin ^4 t .\n\\end{array}\\right.\n$$\n两式中消去 $\\lambda$, 得\n$$\n\\begin{aligned}\n\\frac{3}{4}(a-c)+\\frac{b+c}{2} & =a \\cos ^4 t+(2 b+a-c) \\sin ^2 t \\cos ^2 t+a \\sin ^4 t \\\\\n& =a\\left(1-2 \\sin ^2 t \\cos ^2 t\\right)+(2 b+a-c) \\sin ^2 t \\cos ^2 t \\\\\n& =a+(2 b-a-c) \\sin ^2 t \\cos ^2 t .\n\\end{aligned}\n$$\n于是 $\\quad(2 b-a-c)\\left(\\sin ^2 t \\cos ^2 t-\\frac{1}{4}\\right)=0$.\n若 $2 b-a-c=0$, 则 $z_1=\\frac{1}{2}\\left(z_0+z_2\\right)$, 因此 $A 、 B 、 C$ 三点共线, 与假设矛盾! 所以 $2 b-a-c \\neq 0$, 故 $\\sin ^2 t \\cos ^2 t=\\frac{1}{4}$, 从而 $\\sin ^2 t\\left(1-\\sin ^2 t\\right)=\\frac{1}{4}$, 即 $\\left(\\sin ^2 t-\\frac{1}{2}\\right)^2=0, \\sin ^2 t=\\frac{1}{2}$.\n故 $\\frac{3}{4}-\\frac{2}{\\lambda}=\\frac{1}{4}+\\left(\\frac{1}{2}\\right)^2=-\\frac{1}{2}$, 即有 $\\lambda=\\frac{1}{2} \\in[0,1]$.\n这表明曲线与 $\\triangle A B C$ 的平行于 $A C$ 的中位线只有一个交点, 这个交点对应的复数为\n$$\nz=\\frac{1}{2}\\left(\\frac{1}{4}+\\frac{a+b}{2} \\mathrm{i}\\right)+\\frac{1}{2}\\left(\\frac{3}{4}+\\frac{b+c}{2} \\mathrm{i}\\right)=\\frac{1}{2}+\\frac{a+c+2 b}{4} \\mathrm{i} .\n$$", + "remark": "", + "figures": [] +} \ No newline at end of file diff --git a/processed_dataset/proof/1934.json b/processed_dataset/proof/1934.json new file mode 100644 index 0000000000000000000000000000000000000000..abb2f9ac5ed60013643ac09a004ba55fb1ce157a --- /dev/null +++ b/processed_dataset/proof/1934.json @@ -0,0 +1,8 @@ +{ + "source_file": "./raw_volume-zh/volume8/chapter1.tex", + "problem_type": "proof", + "problem": "例7. 设 $z=\\sum_{k=1}^n z_k^2, z_k=x_k+y_k \\mathrm{i}\\left(x_k 、 y_k \\in \\mathbf{R}, k=1,2, \\cdots, n\\right), p$ 是 $z$ 的平方根的实部, 求证: $|p| \\leqslant \\sum_{k=1}^n\\left|x_k\\right|$.", + "solution": "分析:与解设 $p+q \\mathrm{i}(p 、 q \\in \\mathbf{R})$ 是 $z$ 的平方根, 由\n$$\n\\begin{gathered}\n(p+q \\mathrm{i})^2=\\sum_{k=1}^n z_k^2=\\sum_{k=1}^n\\left(x_k^2-y_k^2\\right)+2 \\mathrm{i} \\sum_{k=1}^n x_k y_k, \\\\\np^2-q^2=\\sum_{k=1}^n\\left(x_k^2-y_k^2\\right), p q=\\sum_{k=1}^n x_k y_k . \\label{eq1}\n\\end{gathered}\n$$\n用反证法, 假设 $|p|>\\sum_{k=1}^n\\left|x_k\\right|$, 则 $p^2>\\left(\\sum_{k=1}^n\\left|x_k\\right|\\right)^2 \\geqslant \\sum_{k=1}^n x_k^2$, 由此推出 $q^2>\\sum_{k=1}^n y_k^2, \\label{eq2}$.\n由式\\ref{eq1}、\\ref{eq2}, 可得 $\\left(\\sum_{k=1}^n x_k y_k\\right)^2=p^2 q^2>\\left(\\sum_{k=1}^n x_k^2\\right) \\cdot\\left(\\sum_{k=1}^n y_k^2\\right)$, 这与柯西不等式相矛盾!\n故 $|p| \\leqslant \\sum_{k=1}^n\\left|x_k\\right|$, 证毕.", + "remark": "", + "figures": [] +} \ No newline at end of file diff --git a/processed_dataset/proof/1935.json b/processed_dataset/proof/1935.json new file mode 100644 index 0000000000000000000000000000000000000000..5dac6d9862b1bfaafeb703884364eca2533f2c02 --- /dev/null +++ b/processed_dataset/proof/1935.json @@ -0,0 +1,8 @@ +{ + "source_file": "./raw_volume-zh/volume8/chapter1.tex", + "problem_type": "proof", + "problem": "例8. 是否存在 $\\theta \\in\\left(-\\frac{\\pi}{2}, \\frac{\\pi}{2}\\right)$, 使 $z^2+8 z+9=(z-\\tan \\theta)(z-\\tan 3 \\theta)$ 对一切复数 $z$ 恒成立?", + "solution": "分析:与解结论是否定的.\n用反证法, 假设存在一个 $\\theta \\in\\left(-\\frac{\\pi}{2}, \\frac{\\pi}{2}\\right)$, 使题中的等式成立.\n特别地, 取 $z=\\mathrm{i}$, 得\n$$\n\\begin{aligned}\n8+8 i & =(i-\\tan \\theta)(i-\\tan 3 \\theta)=i(1+i \\tan \\theta) \\cdot i(1+i \\tan 3 \\theta) \\\\\n& =-\\frac{(\\cos \\theta+i \\sin \\theta)(\\cos 3 \\theta+i \\sin 3 \\theta)}{\\cos \\theta \\cos 3 \\theta}=-\\frac{\\cos 4 \\theta+i \\sin 4 \\theta}{\\cos \\theta \\cos 3 \\theta}\n\\end{aligned} \\label{eq1}\n$$\n从而, 有 $\\tan 4 \\theta=1, \\cos 4 \\theta= \\pm \\frac{\\sqrt{2}}{2}$.\n比较 式\\ref{eq1}的实部,便有\n$$\n\\begin{gathered}\n8=-\\frac{\\cos 4 \\theta}{\\cos \\theta \\cos 3 \\theta}=-\\frac{2 \\cos 4 \\theta}{\\cos 4 \\theta+\\cos 2 \\theta}, \\\\\n\\cos 4 \\theta+\\cos 2 \\theta=-\\frac{1}{4} \\cos 4 \\theta, \\\\\n-\\frac{5}{4} \\cos 4 \\theta=\\cos 2 \\theta, \\\\\n\\frac{25}{16} \\cos ^2 4 \\theta=\\cos ^2 2 \\theta=\\frac{1+\\cos 4 \\theta}{2},\n\\end{gathered}\n$$\n即有\n$$\n25 \\cos ^2 4 \\theta=8(1+\\cos 4 \\theta) .\n$$\n将 $\\cos 4 \\theta= \\pm \\frac{\\sqrt{2}}{2}$ 代入上式, 显然左端是有理数, 而右端是无理数,矛盾.\n故不存在这样的 $\\theta$,使等式恒成立.", + "remark": "", + "figures": [] +} \ No newline at end of file diff --git a/processed_dataset/proof/1936.json b/processed_dataset/proof/1936.json new file mode 100644 index 0000000000000000000000000000000000000000..ee7d69b3048d8f03f5c229e01d9403b2115126ff --- /dev/null +++ b/processed_dataset/proof/1936.json @@ -0,0 +1,8 @@ +{ + "source_file": "./raw_volume-zh/volume8/chapter2.tex", + "problem_type": "proof", + "problem": "例5. 设 $\\alpha 、 \\beta$ 为复数且 $|\\alpha|=k$, 证明:\n$$\n\\left|k| \\beta |^{\\frac{1}{2}}-\\alpha \\frac{\\beta}{|\\beta|^{\\frac{1}{2}}}\\right|^2=2 k[k|\\beta|-\\operatorname{Re}(\\alpha \\beta)] .\n$$", + "solution": "分析:与解给出两种证明方法.\n证法 1 先证明另一个形式上简单一些的恒等式:\n$$\n|| w|-w|^2=2|w|(|w|-\\operatorname{Re} w) \n$$\n这是因为令 $w=a+\\mathrm{i} b(a, b \\in \\mathbf{R})$, 则左边即为 $\\left(\\sqrt{a^2+b^2}-a\\right)^2+b^2$, 右边为 $2\\left(a^2+b^2\\right)-2 a \\sqrt{a^2+b^2}$, 两边显然是相等的.\n由此, 令 $w=\\alpha-\\frac{\\beta}{|\\beta|^{\\frac{1}{2}}}$, 注意到 $\\left|\\alpha \\frac{\\beta}{|\\beta|^{\\frac{1}{2}}}\\right|=k|\\beta|^{\\frac{1}{2}}$ 即知题目中的恒等式是成立的, 证毕.", + "remark": "", + "figures": [] +} \ No newline at end of file diff --git a/processed_dataset/proof/1937.json b/processed_dataset/proof/1937.json new file mode 100644 index 0000000000000000000000000000000000000000..c1246528ac6004a7c246b993c029dd2b7e49a96a --- /dev/null +++ b/processed_dataset/proof/1937.json @@ -0,0 +1,8 @@ +{ + "source_file": "./raw_volume-zh/volume8/chapter2.tex", + "problem_type": "proof", + "problem": "例5. 设 $\\alpha 、 \\beta$ 为复数且 $|\\alpha|=k$, 证明:\n$$\n\\left|k| \\beta |^{\\frac{1}{2}}-\\alpha \\frac{\\beta}{|\\beta|^{\\frac{1}{2}}}\\right|^2=2 k[k|\\beta|-\\operatorname{Re}(\\alpha \\beta)] .\n$$", + "solution": "证法 2 利用恒等式: $|\\alpha-\\beta|^2=|\\alpha|^2+|\\beta|^2-2 \\operatorname{Re} \\bar{\\alpha} \\beta$, 则\n$$\n\\begin{aligned}\n\\left|k| \\beta |^{\\frac{1}{2}}-\\alpha \\frac{\\beta}{|\\beta|^{\\frac{1}{2}}}\\right|^2 & =\\left(k|\\beta|^{\\frac{1}{2}}\\right)^2+\\left(k|\\beta|^{\\frac{1}{2}}\\right)^2-2 \\operatorname{Re}(k \\alpha \\beta) \\\\\n& =2 k[k|\\beta|-\\operatorname{Re}(\\alpha \\beta)],\n\\end{aligned}\n$$\n证毕.", + "remark": "注:事实上,证法 1 中的那个形式上简单一些的恒等式也可以由证法 2 中的那个恒等式推出.\n这是一个熟知的结论, 但是有相当多的学生不习惯, 或者说不喜欢用这个式子, 因为觉得这个式子破坏了对称性, 不如 $|\\alpha-\\beta|^2= |\\alpha|^2+|\\beta|^2-\\bar{\\alpha} \\beta-\\alpha \\bar{\\beta}$ 来得美观.\n但在本题中, 恰恰是这个实形式的恒等式对解题所起的作用要比对称形式的那个式子大 (读者可以自行比较).", + "figures": [] +} \ No newline at end of file diff --git a/processed_dataset/proof/1938.json b/processed_dataset/proof/1938.json new file mode 100644 index 0000000000000000000000000000000000000000..15269cbc8daa7db3dd379c4a49d28ad1a45a69d5 --- /dev/null +++ b/processed_dataset/proof/1938.json @@ -0,0 +1,8 @@ +{ + "source_file": "./raw_volume-zh/volume8/chapter2.tex", + "problem_type": "proof", + "problem": "例6. 记 $A=\\cos \\frac{\\pi}{11}+\\cos \\frac{3 \\pi}{11}+\\cos \\frac{5 \\pi}{11}+\\cos \\frac{7 \\pi}{11}+\\cos \\frac{9 \\pi}{11}$,\n$$\nB=\\sin \\frac{\\pi}{11}+\\sin \\frac{3 \\pi}{11}+\\sin \\frac{5 \\pi}{11}+\\sin \\frac{7 \\pi}{11}+\\sin \\frac{9 \\pi}{11},\n$$\n求证: $A=\\frac{1}{2}, B=\\frac{1}{2} \\cot \\frac{\\pi}{22}$.", + "solution": "分析:与解设 $z=\\cos \\frac{\\pi}{11}+i \\sin \\frac{\\pi}{11}$, 则\n$$\nA+B \\mathrm{i}=z+z^3+z^5+z^7+z^9=\\frac{z\\left(1-z^{10}\\right)}{1-z^2}=\\frac{z-z^{11}}{1-z^2}\n$$\n$$\n\\begin{aligned}\n& =\\frac{z-(\\cos \\pi-i \\sin \\pi)}{1-z^2}=\\frac{z+1}{1-z^2}=\\frac{1}{1-z} \\\\\n& =\\frac{1-\\bar{z}}{(1-z)(1-\\bar{z})}=\\frac{1-\\cos \\frac{\\pi}{11}+\\mathrm{i} \\sin \\frac{\\pi}{11}}{2-(z+\\bar{z})} \\\\\n& =\\frac{1-\\cos \\frac{\\pi}{11}+\\mathrm{i} \\sin \\frac{\\pi}{11}}{2\\left(1-\\cos \\frac{\\pi}{11}\\right)}=\\frac{1}{2}+\\frac{1}{2} \\cdot \\frac{\\sin \\frac{\\pi}{11}}{1-\\cos \\frac{\\pi}{11}} \\mathrm{i} \\\\\n& =\\frac{1}{2}+\\mathrm{i} \\cdot \\frac{1}{2} \\cdot \\cot \\frac{\\pi}{22} .\n\\end{aligned}\n$$\n所以 $A=\\frac{1}{2}, B=\\frac{1}{2} \\cot \\frac{\\pi}{22}$, 证毕.", + "remark": "", + "figures": [] +} \ No newline at end of file diff --git a/processed_dataset/proof/1939.json b/processed_dataset/proof/1939.json new file mode 100644 index 0000000000000000000000000000000000000000..929b938c1fed66b1c062c2c9bb1eba4be80b2fe0 --- /dev/null +++ b/processed_dataset/proof/1939.json @@ -0,0 +1,8 @@ +{ + "source_file": "./raw_volume-zh/volume8/chapter3.tex", + "problem_type": "proof", + "problem": "例4. 设 $p 、 q$ 是复数 $(q \\neq 0)$, 若关于 $x$ 的方程 $x^2+p x+q^2=0$ 的两根的模相等,求证: $\\frac{p}{q}$ 是实数.", + "solution": "分析:与解从韦达定理着手, 建立 $p 、 q$ 与方程根的联系.\n设方程 $x^2+p x+q^2=0$ 的两根是 $z_1 、 z_2$, 则\n$$\n\\left\\{\\begin{array}{l}\nz_1+z_2=-p, \\\\\nz_1 z_2=q^2\n\\end{array}\\right.\n$$\n根据题设, 有 $\\left|z_1\\right|=\\left|z_2\\right|$, 即 $\\left|z_1\\right|^2=\\left|z_2\\right|^2$, 有 $z_1 \\overline{z_1}=z_2 \\overline{z_2}$, 所以\n$$\n\\frac{p^2}{q^2}=\\frac{\\left(z_1+z_2\\right)^2}{z_1 z_2}=\\frac{z_1}{z_2}+\\frac{z_2}{z_1}+2=\\overline{\\left(\\frac{z_2}{z_1}\\right)}+\\left(\\frac{z_2}{z_1}\\right)+2=2 \\operatorname{Re}\\left(\\frac{z_2}{z_1}\\right)+2 \\in \\mathbf{R} .\n$$\n因为 $\\left|\\frac{z_2}{z_1}\\right|=1$, 所以 $\\left|\\operatorname{Re}\\left(\\frac{z_2}{z_1}\\right)\\right| \\leqslant\\left|\\frac{z_2}{z_1}\\right|=1$, 于是 $\\frac{p^2}{q^2} \\geqslant 0$. 故知 $\\frac{p}{q} \\in \\mathbf{R}$, 证毕.", + "remark": "", + "figures": [] +} \ No newline at end of file diff --git a/processed_dataset/proof/1940.json b/processed_dataset/proof/1940.json new file mode 100644 index 0000000000000000000000000000000000000000..0f8b243c8e719ab249aa04b89e4a384ea5fcd29a --- /dev/null +++ b/processed_dataset/proof/1940.json @@ -0,0 +1,8 @@ +{ + "source_file": "./raw_volume-zh/volume8/chapter3.tex", + "problem_type": "proof", + "problem": "例6.. 已知复数 $z$ 满足 $11 z^{100}+10 \\mathrm{i} z^{99}+10 \\mathrm{i} z-11=0$, 求证: $|z|=1$.", + "solution": "分析:与解将已知复数方程变形为 $z^{99}=\\frac{11-10 \\mathrm{i} z}{11 z+10 \\mathrm{i}}$,\n要证 $|z|=1$, 只要证 $\\left|\\frac{11-10 \\mathrm{i} z}{11 z+10 \\mathrm{i}}\\right|=1$ 就行了, 这可以用反证法.\n设 $z=a+b \\mathrm{i}(a 、 b \\in \\mathbf{R})$, 则\n$$\n\\left|z^{99}\\right|=\\left|\\frac{11-10 \\mathrm{i} z}{11 z+10 \\mathrm{i}}\\right|=\\sqrt{\\frac{121+220 b+100\\left(a^2+\\overline{b^2}\\right)}{121\\left(a^2+b^2\\right)+220 b+100}} .\n$$\n记\n$$\n\\begin{aligned}\n& f(a, b)=121+220 b+100\\left(a^2+b^2\\right), \\\\\n& g(a, b)=121\\left(a^2+b^2\\right)+220 b+100 .\n\\end{aligned}\n$$\n若 $a^2+b^2>1$, 则 $f(a, b)g(a, b)$, 即 $|z|^{99}>1$, 有 $|z|>1, a^2+ b^2>1$,矛盾.\n从而只能有 $a^2+b^2=1$, 故 $|z|=1$, 证毕.", + "remark": "注:本题处理技巧独特, 读者应仔细体会.", + "figures": [] +} \ No newline at end of file diff --git a/processed_dataset/proof/1941.json b/processed_dataset/proof/1941.json new file mode 100644 index 0000000000000000000000000000000000000000..d852628c552019e5ff1d436bc13ebb3f51812176 --- /dev/null +++ b/processed_dataset/proof/1941.json @@ -0,0 +1,8 @@ +{ + "source_file": "./raw_volume-zh/volume8/chapter3.tex", + "problem_type": "proof", + "problem": "例7. 设 $n$ 是不小于 2 的整数, $\\alpha$ 是多项式 $P(x)=x^n+a_{n-1} x^{n-1}+\\cdots+a_0$ 的一个根, 且 $0 \\leqslant a_i \\leqslant 1(i=0,1, \\cdots, n-1)$. 求证: $\\operatorname{Re} \\alpha<\\frac{1+\\sqrt{5}}{2}$.", + "solution": "分析:与解用反证法.\n若 $\\operatorname{Re} \\alpha \\geqslant \\frac{1+\\sqrt{5}}{2}$, 则\n$$\n\\begin{aligned}\n\\left|\\alpha^n+a_{n-1} \\alpha^{n-1}\\right| & =\\left|a_{n-2} \\alpha^{n-2}+\\cdots+a_0\\right| \\\\\n& \\leqslant|\\alpha|^{n-2}+\\cdots+|\\alpha|+1 \\\\\n& =\\frac{|\\alpha|^{n-1}-1}{|\\alpha|-1}<\\frac{|\\alpha|^{n-1}}{|\\alpha|-1} .\n\\end{aligned}\n$$\n所以 $\\left|\\alpha+a_{n-1}\\right|<\\frac{1}{|\\alpha|-1} \\leqslant \\frac{1}{\\frac{1+\\sqrt{5}}{2}-1}=\\frac{\\sqrt{5}+1}{2}$.\n而 $\\operatorname{Re}\\left(\\alpha+a_{n-1}\\right) \\geqslant \\frac{1+\\sqrt{5}}{2}$,矛盾!\n故 $\\operatorname{Re} \\alpha<\\frac{1+\\sqrt{5}}{2}$, 证毕.", + "remark": "", + "figures": [] +} \ No newline at end of file diff --git a/processed_dataset/proof/1942.json b/processed_dataset/proof/1942.json new file mode 100644 index 0000000000000000000000000000000000000000..eeefd1dc96fc6f68e73f26d29cbcf138921c956d --- /dev/null +++ b/processed_dataset/proof/1942.json @@ -0,0 +1,8 @@ +{ + "source_file": "./raw_volume-zh/volume8/chapter3.tex", + "problem_type": "proof", + "problem": "例8. 设正整数 $n_1|z|^2,\n\\end{gathered}\n$$\n由此得到, $|z|>1$,与 $|z|<\\frac{\\sqrt{5}-1}{2}$ 矛盾.\n综上所述, 原命题成立, 证毕.", + "remark": "注:以上两例的技巧性比较高, 均是反证法结合模的放缩, 利用不等式解决了问题.", + "figures": [] +} \ No newline at end of file diff --git a/processed_dataset/proof/1943.json b/processed_dataset/proof/1943.json new file mode 100644 index 0000000000000000000000000000000000000000..9cd14f02f985605b9fbb540d9007a130ed2deca0 --- /dev/null +++ b/processed_dataset/proof/1943.json @@ -0,0 +1,10 @@ +{ + "source_file": "./raw_volume-zh/volume8/chapter4.tex", + "problem_type": "proof", + "problem": "例2. 如图(), $P$ 点在 $\\triangle A B C$ 所在平面上, $\\overrightarrow{A P}=m \\overrightarrow{A B}+n \\overrightarrow{A C}$. 求证: $P$ 点在直线 $B C$ 上的充要条件是 $m+n=1$.", + "solution": "分析:与解 $\\overrightarrow{B P}=\\overrightarrow{B A}+\\overrightarrow{A P}=\\overrightarrow{A P}-\\overrightarrow{A B}= (m-1) \\overrightarrow{A B}+n \\overrightarrow{A C}$,\n$$\n\\overrightarrow{B C}=\\overrightarrow{B A}+\\overrightarrow{A C}=-\\overrightarrow{A B}+\\overrightarrow{A C}\n$$\n(1) 若 $m+n=1$, 则 $\\overrightarrow{B P}=-n \\overrightarrow{A B}+n \\overrightarrow{A C}=n \\overrightarrow{B C}$, 故 $B 、 P 、 C$ 共线;\n(2) 若 $B 、 P 、 C$ 共线, 则存在实数 $t$, 使 $\\overrightarrow{B P}=t \\overrightarrow{B C}$.\n即 $(m-1) \\overrightarrow{A B}+n \\overrightarrow{A C}=t(-\\overrightarrow{A B}+\\overrightarrow{A C})$, 所以 $m-1=-t, n=t$.\n从而 $(m-1)+n=0$, 即 $m+n=1$, 证毕.", + "remark": "", + "figures": [ + "./images/volume8/figures/fig-c4i1.png" + ] +} \ No newline at end of file diff --git a/processed_dataset/proof/1944.json b/processed_dataset/proof/1944.json new file mode 100644 index 0000000000000000000000000000000000000000..45abd832c3428cfa68e4a153794c14019027ca8a --- /dev/null +++ b/processed_dataset/proof/1944.json @@ -0,0 +1,8 @@ +{ + "source_file": "./raw_volume-zh/volume8/chapter4.tex", + "problem_type": "proof", + "problem": "例3. $ \\triangle A B C$ 中, 点 $O$ 为外心, $H$ 为垂心, 求证: $\\overrightarrow{O H}=\\overrightarrow{O A}+\\overrightarrow{O B}+\\overrightarrow{O C}$.", + "solution": "分析:与解作直径 $\\overrightarrow{B D}$, 连接 $D A 、 D C$, 有 $\\overrightarrow{O B}=-\\overrightarrow{O D}, D A \\perp A B, D C \\perp B C, A H \\perp B C, C H \\perp A B$.\n故 $C H / / D A, A H / / D C$, 得 $A H C D$ 是平行四边形, 进而 $\\overrightarrow{A H}=\\overrightarrow{D C}$.\n又 $\\overrightarrow{D C}=\\overrightarrow{O C}-\\overrightarrow{O D}=\\overrightarrow{O C}+\\overrightarrow{O B}$, 得 $\\overrightarrow{O H}=\\overrightarrow{O A}+\\overrightarrow{A H}=\\overrightarrow{O A}+\\overrightarrow{D C}=\\overrightarrow{O A}+ \\overrightarrow{O B}+\\overrightarrow{O C}$, 证毕.", + "remark": "", + "figures": [] +} \ No newline at end of file diff --git a/processed_dataset/proof/1945.json b/processed_dataset/proof/1945.json new file mode 100644 index 0000000000000000000000000000000000000000..545027fb89062fc3274311b83276e86d36159765 --- /dev/null +++ b/processed_dataset/proof/1945.json @@ -0,0 +1,8 @@ +{ + "source_file": "./raw_volume-zh/volume8/chapter4.tex", + "problem_type": "proof", + "problem": "例5. 是否存在 4 个平面向量, 两两不共线, 其中任意两个向量之和与其余两个向量之和垂直?", + "solution": "分析:与解在正 $\\triangle A B C$ 中, $O$ 为内心, $P$ 为内切圆周上一点, 满足 $\\overrightarrow{P A}$ 、 $\\overrightarrow{P B} 、 \\overrightarrow{P C} 、 \\overrightarrow{P O}$ 两两不共线, 则\n$$\n\\begin{aligned}\n(\\overrightarrow{P A}+\\overrightarrow{P B}) \\cdot(\\overrightarrow{P C}+\\overrightarrow{P O}) & =(\\overrightarrow{P O}+\\overrightarrow{O A}+\\overrightarrow{P O}+\\overrightarrow{O B}) \\cdot(\\overrightarrow{P O}+\\overrightarrow{O C}+\\overrightarrow{P O}) \\\\\n& =(2 \\overrightarrow{P O}+\\overrightarrow{O A}+\\overrightarrow{O B}) \\cdot(2 \\overrightarrow{P O}+\\overrightarrow{O C}) \\\\\n& =(2 \\overrightarrow{P O}-\\overrightarrow{O C}) \\cdot(2 \\overrightarrow{P O}+\\overrightarrow{O C}) \\\\\n& =4 \\overrightarrow{P O}^2-\\overrightarrow{O C}^2=4|P O|^2-|O C|^2=0,\n\\end{aligned}\n$$\n即 $(\\overrightarrow{P A}+\\overrightarrow{P B}) \\perp(\\overrightarrow{P C}+\\overrightarrow{P O})$.\n同理可证其他情况, 从而 $\\overrightarrow{P A} 、 \\overrightarrow{P B} 、 \\overrightarrow{P C} 、 \\overrightarrow{P O}$ 符合题意.", + "remark": "注:本题属于构造性问题,利用向量和的定义将一个向量拆成多个向量和的技巧,望读者切实掌握.", + "figures": [] +} \ No newline at end of file diff --git a/processed_dataset/proof/1946.json b/processed_dataset/proof/1946.json new file mode 100644 index 0000000000000000000000000000000000000000..7d196385eba1b806b153f17e4df5c80cd2f9dd83 --- /dev/null +++ b/processed_dataset/proof/1946.json @@ -0,0 +1,12 @@ +{ + "source_file": "./raw_volume-zh/volume8/chapter4.tex", + "problem_type": "proof", + "problem": "例6. 如图(), 在 $\\triangle A B C$ 的内部任选点 $O$, 证明: $S_A \\cdot \\overrightarrow{O A}+S_B \\cdot \\overrightarrow{O B}+S_C \\cdot \\overrightarrow{O C}=\\overrightarrow{0}$, 其中 $S_A$ 、 $S_B 、 S_C$ 分别为 $\\triangle B C O 、 \\triangle C A O 、 \\triangle A B O$ 的面积.", + "solution": "分析:与解如图(), 设 $\\overrightarrow{O A} 、 \\overrightarrow{O B} 、 \\overrightarrow{O C}$ 上的单位向量分别为 $\\overrightarrow{e_1} 、 \\overrightarrow{e_2} 、 \\overrightarrow{e_3}$, 作 $\\triangle P Q R$, 使 $P Q / / O A, Q R / / O B$, $P R / / O C$, (如图()) 则 $\\sin \\angle R=\\sin \\alpha, \\sin \\angle P= \\sin \\beta, \\sin \\angle Q=\\sin \\gamma$.\n因为 $\\overrightarrow{Q P}+\\overrightarrow{P R}+\\overrightarrow{R Q}=\\overrightarrow{Q Q}=\\overrightarrow{0}$, 所以 $|\\overrightarrow{Q P}| \\overrightarrow{e_1}+ |\\overrightarrow{P R}| \\overrightarrow{e_2}+|\\overrightarrow{R Q}| \\overrightarrow{e_3}=0$.\n设 $R$ 为 $\\triangle P Q R$ 的外接圆半径, 则\n$2 R \\sin \\alpha \\cdot \\overrightarrow{e_1}+2 R \\sin \\beta \\cdot \\overrightarrow{e_2}+2 R \\sin \\gamma \\cdot \\overrightarrow{e_3}=\\overrightarrow{0}$, $\\sin \\alpha \\cdot \\overrightarrow{e_1}+\\sin \\beta \\cdot \\overrightarrow{e_2}+\\sin \\gamma \\cdot \\overrightarrow{e_3}=\\overrightarrow{0}$.\n$$\n\\begin{gathered}\n\\frac{1}{2}|O A| \\cdot|O B| \\cdot|O C| \\cdot \\sin \\alpha \\overrightarrow{e_1}+\\frac{1}{2}|O A| \\cdot|O B| \\cdot|O C| \\cdot \\sin \\beta \\overrightarrow{e_2} \\\\\n+\\frac{1}{2}|O A| \\cdot|O B| \\cdot|O C| \\cdot \\sin \\gamma \\overrightarrow{e_3}=\\overrightarrow{0}\n\\end{gathered}\n$$\n$$\n\\begin{gathered}\n\\left(\\frac{1}{2}|O B| \\cdot|O C| \\sin \\alpha\\right)\\left(|O A| \\cdot \\overrightarrow{e_1}\\right)+\\left(\\frac{1}{2}|O A| \\cdot|O C| \\sin \\beta\\right)\\left(|O B| \\cdot \\overrightarrow{e_2}\\right) \\\\\n+\\left(\\frac{1}{2}|O A| \\cdot|O B| \\sin \\gamma\\right)\\left(|O C| \\cdot \\overrightarrow{e_3}\\right)=\\overrightarrow{0}\n\\end{gathered}\n$$\n所以 $S_A \\cdot \\overrightarrow{O A}+S_B \\cdot \\overrightarrow{O B}+S_C \\cdot \\overrightarrow{O C}=\\overrightarrow{0}$, 证毕.", + "remark": "", + "figures": [ + "./images/volume8/figures/fig-c4i2.png", + "./images/volume8/figures/fig-c4i2.png", + "./images/volume8/figures/fig-c4i3.png" + ] +} \ No newline at end of file diff --git a/processed_dataset/proof/1947.json b/processed_dataset/proof/1947.json new file mode 100644 index 0000000000000000000000000000000000000000..af7cd04155774818f23ae31072a6af4469aea0f1 --- /dev/null +++ b/processed_dataset/proof/1947.json @@ -0,0 +1,8 @@ +{ + "source_file": "./raw_volume-zh/volume8/chapter4.tex", + "problem_type": "proof", + "problem": "例7. 设 $O$ 是 $\\triangle A B C$ 内部一点.\n证明: 存在正整数 $p 、 q 、 r$, 使得\n$$\n|p \\cdot \\overrightarrow{O A}+q \\cdot \\overrightarrow{O B}+r \\cdot \\overrightarrow{O C}|<\\frac{1}{2007} \\text {. }\n$$", + "solution": "分析:与解由条件可知存在正实数 $\\beta 、 \\gamma$ 使得 $\\overrightarrow{O A}+\\beta \\overrightarrow{O B}+\\gamma \\overrightarrow{O C}=\\overrightarrow{0}$, 于是对任意正整数 $k$, 都有 $k \\overrightarrow{O A}+k \\beta \\overrightarrow{O B}+k \\gamma \\overrightarrow{O C}=\\overrightarrow{0}$, 记 $m(k)=[k \\beta], n(k)= [k \\gamma]$, 这里 $[x]$ 表示不超过实数 $x$ 的最大整数, $\\{x\\}=x-[x]$.\n利用 $\\beta 、 \\gamma$ 都是正实数可知 $m(k T)$ 和 $n(k T)$ 都是关于正整数 $k$ 的严格递增数列, 这里 $T$ 是某个大于 $\\max \\left\\{\\frac{1}{\\beta}, \\frac{1}{\\gamma}\\right\\}$ 的正整数.\n因此\n$$\n\\begin{aligned}\n|k T \\cdot \\overrightarrow{O A}+m(k T) \\cdot \\overrightarrow{O B}+n(k T) \\cdot \\overrightarrow{O C}| & =|-\\{k T \\beta\\} \\overrightarrow{O B}-\\{k T \\gamma\\} \\overrightarrow{O C}| \\\\\n& \\leqslant\\{k T \\beta\\}|\\overrightarrow{O B}|+\\{k T \\gamma\\}|\\overrightarrow{O C}| \\\\\n& \\leqslant|\\overrightarrow{O B}|+|\\overrightarrow{O C}| .\n\\end{aligned}\n$$\n这表明有无穷多个向量 $k T \\cdot \\overrightarrow{O A}+m(k T) \\cdot \\overrightarrow{O B}+n(k T) \\cdot \\overrightarrow{O C}$ 的终点落在一个以 $O$ 为圆心, $|\\overrightarrow{O B}|+|\\overrightarrow{O C}|$ 为半径的圆内, 因此, 其中必有两个向量的终点之间的距离小于 $\\frac{1}{2007}$, 也就是说, 这两个向量的差的模长小于 $\\frac{1}{2007}$. 即存在正整数 $k_1), 求证: 圆内接四边形 $A B C D$ 的两组对边 $A B$ 和 $C D$ 的交角平分线 $l_1$ 与 $A D$ 和 $B C$ 的交角平分线 $l_2$ 互相垂直.", + "solution": "分析:与解设 $A B 、 C D$ 交于 $O_1, A D 、 B C$ 交于 $O_2$, 取 $\\overrightarrow{O_1 D} 、 \\overrightarrow{O_1 A} 、 \\overrightarrow{O_2 D} 、 \\overrightarrow{O_2 C}$ 方向上单位向量分别为 $\\vec{i} 、 \\vec{j} 、 \\vec{l} 、 \\vec{k}$, 则\n$$\n\\begin{aligned}\n\\cos \\left(180^{\\circ}-\\angle D A B\\right) & =\\vec{i} \\cdot \\vec{j} \\Rightarrow \\cos \\angle D A B=-\\vec{\\imath} \\cdot \\vec{j}, \\\\\n\\cos \\left(180^{\\circ}-\\angle D C B\\right) & =\\vec{i} \\cdot \\vec{k} \\Rightarrow \\cos \\angle D C B=-\\vec{i} \\cdot \\vec{k}, \\\\\n\\cos \\angle C D A= & \\vec{i} \\cdot \\vec{l}, \\cos \\angle B=\\vec{k} \\cdot \\vec{j} .\n\\end{aligned}\n$$\n而 $\\angle B+\\angle C D A=180^{\\circ}, \\angle D C B+\\angle D A B=180^{\\circ}$,\n所以 $\\vec{k} \\cdot \\vec{j}+\\vec{i} \\cdot \\vec{l}=0,-\\vec{i} \\cdot \\vec{k}-\\vec{l} \\cdot \\vec{j}=0 \\Leftrightarrow \\vec{i} \\cdot \\vec{k}+\\vec{l} \\cdot \\vec{j}=0$.\n于是 $(\\vec{i}+\\vec{j})(\\vec{k}+\\vec{l})=\\vec{k} \\cdot \\vec{j}+\\vec{k} \\cdot \\vec{i}+\\vec{i} \\cdot \\vec{l}+\\vec{j} \\cdot \\vec{l}=(\\vec{k} \\cdot \\vec{j}+\\vec{i} \\cdot \\vec{l})+ (\\vec{i} \\cdot \\vec{k}+\\vec{l} \\cdot \\vec{j})=0$.\n设 $P$ 为 $l_1$ 和 $l_2$ 的交点, 则 $O_1 P 、 O_2 P$ 分别平分 $\\angle D O_1 A$ 和 $\\angle C O_2 D \\Leftrightarrow \\vec{i}+\\vec{j}$ 与 $\\overrightarrow{O_1 P}$ 同向, $\\vec{l}+\\vec{k}$ 与 $\\overrightarrow{O_2 P}$ 同向, 所以 $\\overrightarrow{O_1 P}=\\lambda_1(\\vec{i}+\\vec{j}), \\overrightarrow{O_2 P}=\\lambda_2(\\vec{l}+\\vec{k}) \\left(\\lambda_1 \\neq 0, \\lambda_2 \\neq 0\\right)$.\n所以 $\\overrightarrow{O_1 P} \\cdot \\overrightarrow{O_2 P}=0$, 因此 $l_1 \\perp l_2$, 证毕.", + "remark": "", + "figures": [ + "./images/volume8/figures/fig-c5i1.png" + ] +} \ No newline at end of file diff --git a/processed_dataset/proof/1950.json b/processed_dataset/proof/1950.json new file mode 100644 index 0000000000000000000000000000000000000000..b90e3193f1f8c847ad0c2a3e64f4ef6e23d60e59 --- /dev/null +++ b/processed_dataset/proof/1950.json @@ -0,0 +1,8 @@ +{ + "source_file": "./raw_volume-zh/volume8/chapter5.tex", + "problem_type": "proof", + "problem": "例5. 任给 8 个非零实数 $a_1, a_2, \\cdots, a_8$, 证明:下面 6 个数 $a_1 a_3+a_2 a_4$, $a_1 a_5+a_2 a_6, a_1 a_7+a_2 a_8, a_3 a_5+a_4 a_6, a_3 a_7+a_4 a_8, a_5 a_7+a_6 a_8$ 中,至少有一个是非负的.", + "solution": "分析:与解令向量 $\\overrightarrow{O A}=\\left(a_1, a_2\\right), \\overrightarrow{O B}=\\left(a_3, a_4\\right), \\overrightarrow{O C}=\\left(a_5, a_6\\right)$, $\\overrightarrow{O D}=\\left(a_7, a_8\\right)$, 这 4 个向量中至少有两个向量之间的最小正夹角 $\\alpha$ 小于或等于 $90^{\\circ}$, 不妨设这两个向量为 $\\overrightarrow{O A}$ 和 $\\overrightarrow{O B}$, 此时\n$$\na_1 a_3+a_2 a_4=\\overrightarrow{O A} \\cdot \\overrightarrow{O B}=|\\overrightarrow{O A}| \\cdot|\\overrightarrow{O B}| \\cdot \\cos \\alpha \\geqslant 0,\n$$\n证毕.", + "remark": "", + "figures": [] +} \ No newline at end of file diff --git a/processed_dataset/proof/1951.json b/processed_dataset/proof/1951.json new file mode 100644 index 0000000000000000000000000000000000000000..072c680474993f8ee74ca2a5348a61df6d8d215f --- /dev/null +++ b/processed_dataset/proof/1951.json @@ -0,0 +1,10 @@ +{ + "source_file": "./raw_volume-zh/volume8/chapter5.tex", + "problem_type": "proof", + "problem": "例7. 如图(), 在 $\\triangle A B C$ 中, $A B=A C, D$ 是 $B C$ 的中点, $E$ 是从 $D$ 作 $A C$ 的垂线的垂足, $F$ 是 $D E$ 的中点, 求证: $A F \\perp B E$.", + "solution": "分析:与解\n$$\n\\begin{aligned}\n\\overrightarrow{A F} \\cdot \\overrightarrow{B E} & =\\overrightarrow{A F} \\cdot(\\overrightarrow{B C}+\\overrightarrow{C E}) \\\\\n& =\\left(\\overrightarrow{A D}+\\frac{1}{2} \\overrightarrow{D E}\\right) \\cdot \\overrightarrow{B C}+\\left(\\overrightarrow{A E}+\\frac{1}{2} \\overrightarrow{E D}\\right) \\cdot \\overrightarrow{C E} \\\\\n& =\\frac{1}{2} \\overrightarrow{D E} \\cdot \\overrightarrow{B C}+\\overrightarrow{A E} \\cdot \\overrightarrow{C E}=\\overrightarrow{D E} \\cdot \\overrightarrow{D C}+\\overrightarrow{A E} \\cdot \\overrightarrow{C E} \\\\\n& =\\overrightarrow{D E} \\cdot(\\overrightarrow{D E}+\\overrightarrow{E C})-|\\overrightarrow{A E}| \\cdot|\\overrightarrow{C E}| \\\\\n& =\\overrightarrow{D E} \\cdot \\overrightarrow{D E}-|\\overrightarrow{A E}| \\cdot|\\overrightarrow{C E}|=0 .\n\\end{aligned}\n$$\n故 $A F \\perp B E$, 证毕.", + "remark": "", + "figures": [ + "./images/volume8/figures/fig-c5i2.png" + ] +} \ No newline at end of file diff --git a/processed_dataset/proof/1952.json b/processed_dataset/proof/1952.json new file mode 100644 index 0000000000000000000000000000000000000000..6479ca6957a5ab12ed654361c4566b3768a015b9 --- /dev/null +++ b/processed_dataset/proof/1952.json @@ -0,0 +1,10 @@ +{ + "source_file": "./raw_volume-zh/volume8/chapter5.tex", + "problem_type": "proof", + "problem": "例8. 如图(), $\\triangle A B C$ 中, $O$ 为外心, 三条高 $A D 、 B E 、 C F$ 交于点 $H$, 直线 $D E$ 和 $A B$ 交于点 $M, F D$ 和 $A C$ 交于点 $N$. 求证: $O H \\perp M N$.", + "solution": "分析:与解设点 $H^{\\prime}$ 满足 $\\overrightarrow{O H^{\\prime}}=\\overrightarrow{O A}+\\overrightarrow{O B}+\\overrightarrow{O C}$, 则\n$$\n\\begin{aligned}\n\\overrightarrow{A H^{\\prime}} \\cdot \\overrightarrow{B C} & =\\left(\\overrightarrow{O H^{\\prime}}-\\overrightarrow{O A}\\right) \\cdot(\\overrightarrow{O C}-\\overrightarrow{O B}) \\\\\n& =\\overrightarrow{O C}^2-\\overrightarrow{O B}^2=0,\n\\end{aligned}\n$$\n故 $A H^{\\prime} \\perp B C$.\n同理 $B H^{\\prime} \\perp A C$, 于是 $H^{\\prime}$ 与 $H$ 重合, 即 $\\overrightarrow{O H}=\\overrightarrow{O A}+\\overrightarrow{O B}+\\overrightarrow{O C}$.\n由 $\\angle D E C=\\angle A B C, \\angle O C E=\\frac{\\pi}{2}-\\angle A B C$ 知: $O C \\perp D E$, 同理, $O B \\perp D F$. 故\n$$\n\\begin{aligned}\n\\overrightarrow{O H} \\cdot \\overrightarrow{A M} & =(\\overrightarrow{O B}+\\overrightarrow{O A}) \\cdot \\overrightarrow{A M}+\\overrightarrow{O C} \\cdot \\overrightarrow{A M}=\\overrightarrow{O C} \\cdot \\overrightarrow{A M} \\\\\n& =\\overrightarrow{O C} \\cdot(\\overrightarrow{A E}+\\overrightarrow{E M})=\\overrightarrow{O C} \\cdot \\overrightarrow{A E}+\\overrightarrow{O C} \\cdot \\overrightarrow{E M} \\\\\n& =\\overrightarrow{O C} \\cdot \\overrightarrow{A E}=|\\overrightarrow{O C}| \\cdot|\\overrightarrow{A E}| \\cdot \\cos \\left(90^{\\circ}-B\\right) \\\\\n& =R \\cdot|\\overrightarrow{A B}| \\cos A \\sin B=2 R^2 \\cdot \\cos A \\sin B \\sin C\n\\end{aligned}\n$$\n同理 $\\overrightarrow{O H} \\cdot \\overrightarrow{A N}=2 R^2 \\cdot \\cos A \\sin B \\sin C$, 所以\n$$\n\\overrightarrow{O H} \\cdot \\overrightarrow{M N}=\\overrightarrow{O H} \\cdot(\\overrightarrow{A N}-\\overrightarrow{A M})=\\overrightarrow{O H} \\cdot \\overrightarrow{A N}-\\overrightarrow{O H} \\cdot \\overrightarrow{A M}=0\n$$\n故 $O H \\perp M N$, 证毕.", + "remark": "注:以上两例是向量法解平面几何问题的范例, 在第九章中还会有更多的介绍.", + "figures": [ + "./images/volume8/figures/fig-c5i3.png" + ] +} \ No newline at end of file diff --git a/processed_dataset/proof/1953.json b/processed_dataset/proof/1953.json new file mode 100644 index 0000000000000000000000000000000000000000..bb04f68174663432813cc8cd8648722237ab4a56 --- /dev/null +++ b/processed_dataset/proof/1953.json @@ -0,0 +1,11 @@ +{ + "source_file": "./raw_volume-zh/volume8/chapter6.tex", + "problem_type": "proof", + "problem": "例2. 如图() 所示, 在长方体 $A B C D-A_1 B_1 C_1 D_1$ 中, 点 $E 、 F$ 分别在 $B B_1 、 D D_1$ 上, 且 $A E \\perp A_1 B, A F \\perp A_1 D$.\n(1) 证明: $A_1 C \\perp$ 平面 $A E F$;\n(2) 若规定两个平面所成的角是这两个平面所组成的二面角中的锐角 (或直角), 在 $A B=4, A D=3, A A_1=5$ 时, 求平面 $A E F$ 与平面 $D_1 B_1 B D$ 所成角的大小 (用反三角函数值表示);\n(3) 条件同 (2), 计算 $A_1 D$ 和平面 $A E F$ 所成的角.", + "solution": "分析:与解 (1) 如图() 所示建立空间直角坐标系.\n因为 $A_1 B \\perp A E$, 即 $\\overrightarrow{A_1 B} \\cdot \\overrightarrow{A E}=0$, 即 $\\{0, a,-c\\} \\cdot \\left\\{0, a, h_1\\right\\}=0$, 所以 $a^2-h_1 c=0$.\n因为 $A_1 D \\perp A F$, 即 $\\overrightarrow{A_1 D} \\cdot \\overrightarrow{A F}=0$, 即 $\\{b, 0, c\\} \\cdot\\left\\{-b, 0, h_2\\right\\}=0$, 所以 $b^2-h_2 c=0$.\n因为 $\\overrightarrow{A_1 C} \\cdot \\overrightarrow{A E}=\\{-b, a,-c\\} \\cdot\\left\\{0, a, h_1\\right\\}=a^2-h_1 c=0$, 所以 $A_1 C \\perp A E$.\n因为 $\\overrightarrow{A_1 C} \\cdot \\overrightarrow{A F}=\\{-b, a,-c\\} \\cdot\\left\\{-b, 0, h_2\\right\\}=b^2-h_2 c=0$, 所以 $A_1 C \\perp A F$.\n所以 $A_1 C \\perp$ 平面 $A E F$, 证毕.\n(2) 在空间中有定理: 若两条直线分别垂直于两个平面, 则这两条直线所成的角与这两个平面所成的角的大小相等.\n设 $\\vec{a}=\\{x, y, z\\},|\\vec{a}| \\neq 0, \\vec{a} \\perp$ 平面 $D_1 B_1 B D, \\overrightarrow{D D_1}=\\{0,0,5\\}$, $\\overrightarrow{D B}=\\{3,4,0\\}, \\overrightarrow{A_1 C}=\\{-3,4,-5\\}$, 则\n$$\n\\left\\{\\begin{array} { l } \n{ \\vec { a } \\cdot \\vec { D D _ { 1 } } = 0 , } \\\\\n{ \\vec { a } \\cdot \\vec { D B } = 0 , } \\\\\n{ | \\vec { a } | \\neq 0 . }\n\\end{array} \\Rightarrow \\left\\{\\begin{array}{l}\n5 z=0, \\\\\n3 x+4 y=0, \\\\\nx^2+y^2+z^2 \\neq 0 .\n\\end{array}\\right.\\right.\n$$\n所以 $\\vec{a}=\\left\\{x,-\\frac{3}{4} x, 0\\right\\}(x \\neq 0)$, 而 $\\vec{a} \\perp$ 平面 $D_1 B_1 B D$, 由 $A_1 C \\perp$ 平面 $A E F$, 设 $\\vec{a}$ 和 $\\overrightarrow{A_1} \\vec{C}$ 所在直线所成的角为 $\\theta$, 则\n$$\n\\cos \\theta=\\left|\\frac{\\vec{a} \\cdot \\overrightarrow{A_1 C}}{|\\vec{a}| \\cdot\\left|\\overrightarrow{A_1 C}\\right|}\\right|=\\left|\\frac{-3 x-3 x}{5 \\sqrt{2} \\cdot \\frac{5}{4} \\cdot|x|}\\right|=\\frac{12 \\sqrt{2}}{25} .\n$$\n所以平面 $A E F$ 与平面 $D_1 B_1 B D$ 所成的角为 $\\arccos \\frac{12 \\sqrt{2}}{25}$.\n(3)与平面垂直的向量称为平面的法向量, 要求直线与平面所成角的大小只要求出直线与平面法向量所成的那个不超过 $90^{\\circ}$ 的角, 然后求出其余角即可.\n特别地当直线与平面的法向量平行时,直线与该平面垂直.\n显然 $\\overrightarrow{A_1 C}=\\{-3,4,-5\\}$ 为平面 $A E F$ 的法向量, 且 $\\overrightarrow{A D_1}=\\{3,0,5\\}$, 所以\n$$\n\\cos \\varphi=\\left|\\frac{\\overrightarrow{A_1 C} \\cdot \\overrightarrow{A_1 D}}{\\left|\\overrightarrow{A_1 C}\\right| \\cdot\\left|\\overrightarrow{A_1 D}\\right|}\\right|=\\left|\\frac{-9-25}{5 \\sqrt{2} \\cdot \\sqrt{34}}\\right|=\\frac{\\sqrt{17}}{5} .\n$$\n由 $\\frac{\\pi}{2}-\\arccos \\frac{\\sqrt{17}}{5}=\\arcsin \\frac{\\sqrt{17}}{5}$ 知, $A_1 D$ 和平面 $A E F$ 所成的角为 $\\arcsin \\frac{\\sqrt{17}}{5}$.", + "remark": "", + "figures": [ + "./images/volume8/figures/fig-c6i2.png", + "./images/volume8/figures/fig-c6i2.png" + ] +} \ No newline at end of file diff --git a/processed_dataset/proof/1954.json b/processed_dataset/proof/1954.json new file mode 100644 index 0000000000000000000000000000000000000000..a760578fb13bdb00600a4cf045ebe5024e283562 --- /dev/null +++ b/processed_dataset/proof/1954.json @@ -0,0 +1,10 @@ +{ + "source_file": "./raw_volume-zh/volume8/chapter6.tex", + "problem_type": "proof", + "problem": "例3. (1) 直线 $P A$ 交平面 $\\alpha$ 于点 $A$, 点 $P$ 在直线 $P A$ 上, $\\overrightarrow{n_0}$ 是垂直于平面 $\\alpha$ 的单位向量,试叙述 $\\left|\\overrightarrow{P A} \\cdot \\vec{n}_0\\right|$ 的几何意义;\n(2) 在长方体 $A B C D-A_1 B_1 C_1 D_1$ 中, $A B=6, A D=A A_1=4$, 求点 $B_1$ 到平面 $A C D_1$ 的距离;\n(3) 第 (2) 小题的条件下, 设 $P 、 Q 、 R$ 分别为 $A_1 B_1 、 B_1 C_1$ 和 $B B_1$ 的中点, 求证平面 $A C D_1$ 平行于平面 $P Q R$.", + "solution": "分析:与解 (1) <1> $P A \\perp$ 平面 $\\alpha$ 时, $\\overrightarrow{P A}$ 与 $\\overrightarrow{n_0}$ 的夹角为 0 或 $\\pi, \\overrightarrow{P A} \\cdot \\overrightarrow{n_0}=\\pm|\\overrightarrow{P A}|$, 所以 $\\left|\\overrightarrow{P A} \\cdot \\overrightarrow{n_0}\\right|=|\\overrightarrow{P A}|$;\n<2> $P A$ 不垂直于平面 $\\alpha$ 时, 过点 $P$ 作 $P O \\perp \\alpha$ 于点 $O$, 设向量 $\\overrightarrow{P A}$ 与 $\\overrightarrow{n_0}$ 的夹角为 $\\theta$, 则 $\\left|\\overrightarrow{P A} \\cdot \\vec{n}_0\\right|=|| \\overrightarrow{P A}|\\cdot 1 \\cdot \\cos \\theta|=|\\overrightarrow{P A}| \\cdot|\\cos \\theta|=|P O|$.\n所以由<1>和<2>可知 $\\left|\\overrightarrow{P A} \\cdot \\overrightarrow{n_0}\\right|$ 为点 $P$ 到平面 $\\alpha$ 的距离.\n(2) 如图(), 建立空间直角坐标系, 则 $A(4,0,0), B_1(4,6,4), C(0$, $6,0), D_1(0,0,4), \\overrightarrow{A D_1}=\\{-4,0,4\\}, \\overrightarrow{C D_1}=\\{0,-6,4\\}, B_1 C=\\{-4$, $0,-4\\}$, 设 $\\vec{n}_0=\\{x, y, z\\}$ 且 $x^2+y^2+z^2=1$ 为平面 $A C D_1$ 的法向量.\n因为\n$$\n\\left\\{\\begin{array}{l}\n\\overrightarrow{n_0} \\cdot \\overrightarrow{A D_1}=0, \\\\\n\\overrightarrow{n_0} \\cdot \\overrightarrow{C D_1}=0, \\\\\nx^2+y^2+z^2=1,\n\\end{array}\\right.\n$$\n所以\n$$\n\\left\\{\\begin{array}{l}\n-4 x+4 z=0, \\\\\n-6 y+4 z=0, \\\\\nx^2+y^2+z^2=1 .\n\\end{array}\\right.\n$$\n由此解得取 $\\overrightarrow{n_0}=\\left\\{\\frac{3}{\\sqrt{2 \\overline{2}}}, \\frac{2}{\\sqrt{22}}, \\frac{3}{\\sqrt{22}}\\right\\}, B_1$ 到平面 $A C D_1$ 的距离为\n$$\nd_1=\\left|\\overrightarrow{B_1 C} \\cdot \\overrightarrow{n_0}\\right|=\\left|-4 \\times \\frac{3}{\\sqrt{22}}-4 \\times \\frac{3}{\\sqrt{22}}\\right|=\\frac{12 \\sqrt{22}}{11} \\text {. }\n$$\n(3) $P(4,3,4), Q(2,6,4), R(4,6,2)$.\n设 $\\overrightarrow{m_0}=\\{x, y, z\\}$, 且 $x^2+y^2+z^2=1$ 为平面 $P Q R$ 的法向量, $\\overrightarrow{P Q}= \\{-2,3,0\\}, \\overrightarrow{P R}=\\{0,3,-2\\}$.\n取 $\\overrightarrow{m_0}=\\left\\{\\frac{3}{\\sqrt{22}}, \\frac{2}{\\sqrt{2} \\overline{2}}, \\frac{3}{\\sqrt{22}}\\right\\}$.\n又因为 $\\overrightarrow{n_0}=\\left\\{\\frac{3}{\\sqrt{22}}, \\frac{2}{\\sqrt{22}}, \\frac{3}{\\sqrt{22}}\\right\\}$, 显然 $\\overrightarrow{m_0} / / \\overrightarrow{n_0}$, 所以平面 $A C D_1$ 平行于平面 $P Q R$.", + "remark": "", + "figures": [ + "./images/volume8/figures/fig-c6i3.png" + ] +} \ No newline at end of file diff --git a/processed_dataset/proof/1955.json b/processed_dataset/proof/1955.json new file mode 100644 index 0000000000000000000000000000000000000000..acb77e87a7ab292c7cd9a2ebf3c7c4bbf2655d7a --- /dev/null +++ b/processed_dataset/proof/1955.json @@ -0,0 +1,11 @@ +{ + "source_file": "./raw_volume-zh/volume8/chapter6.tex", + "problem_type": "proof", + "problem": "例4. 如图() 所示,已知正四棱雉 $S-A B C D$ 的底面边长为 6 , 高为 $3, P 、 Q 、 R$ 分别在 $S C 、 S B$ 、 $S D$ 上, 且 $S P: P C=1: 2, S Q: Q B=2: 1, S R: R D=2: 1$.\n(1) 求证: $S A / /$ 平面 $P Q R$ 并求出 $S A$ 到平面 $P Q R$ 的距离;\n(2) 求点 $P$ 到直线 $B D$ 的距离;\n(3) 若 $M 、 N$ 分别是 $B D$ 和 $S C$ 上的动点, 求线段 $M N$ 长度的最小值.", + "solution": "分析:与解 (1) 如图() 所示, 建立空间直角坐标系, 则 $A(3,-3,0)$, $B(3,3,0), C(-3,3,0), D(-3,-3,0), S(0,0,3), P(-1,1,2)$, $Q(2,2,1), R(-2,-2,1)$, 设 $G$ 是 $S A$ 上任一点.\n因为 $\\overrightarrow{A G} / / \\overrightarrow{A S}$, 所以 $\\overrightarrow{A G}=k \\overrightarrow{A S}=k\\{-3,3,3\\}=\\{-3 k, 3 k, 3 k\\} (k \\in \\mathbf{R})$.\n所以 $\\overrightarrow{A G}=\\overrightarrow{O G}-\\overrightarrow{O A}=\\{-3 k, 3 k, 3 k\\}, \\overrightarrow{O G}=\\{3-3 k,-3+3 k, 3 k\\}$, $\\overrightarrow{G R}=\\{3 k-5,-3 k+1,-3 k+1\\}$.\n设 $\\overrightarrow{n_0}$ 为平面 $P Q R$ 的一个单位法向量, 且 $\\vec{n}_0=\\{x, y, z\\}$.\n$\\overrightarrow{P Q}=\\{3,1,-1\\}, \\overrightarrow{Q R}=\\{-4,-4,0\\}$, 则\n$$\n\\left\\{\\begin{array}{l}\n\\overrightarrow{n_0} \\cdot \\overrightarrow{P Q}=0, \\\\\n\\overrightarrow{n_0} \\cdot \\overrightarrow{Q R}=0, \\\\\n\\left|\\overrightarrow{n_0}\\right|=1 .\n\\end{array}\\right.\n$$\n所以\n$$\n\\left\\{\\begin{array}{l}\n3 x+y-z=0, \\\\\n-4 x-4 y=0, \\\\\nx^2+y^2+z^2=1 .\n\\end{array}\\right.\n$$\n由此解得\n$$\n\\left\\{\\begin{array} { l } \n{ x = \\frac { \\sqrt { 6 } } { 6 } , } \\\\\n{ y = - \\frac { \\sqrt { 6 } } { 6 } , } \\\\\n{ z = \\frac { \\sqrt { 6 } } { 3 } }\n\\end{array} \\left\\{\\begin{array}{l}\nx=-\\frac{\\sqrt{6}}{6}, \\\\\ny=\\frac{\\sqrt{6}}{6}, \\\\\nz=-\\frac{\\sqrt{6}}{3} .\n\\end{array}\\right.\\right.\n$$\n取 $\\overrightarrow{n_0}=\\left\\{\\frac{\\sqrt{6}}{6},-\\frac{\\sqrt{6}}{6}, \\frac{\\sqrt{6}}{3}\\right\\}$, 而 $\\overrightarrow{A S}=\\{-3,3,3\\}$, 则 $\\overrightarrow{n_0} \\cdot \\overrightarrow{A S}=-3 \\times \\frac{\\sqrt{6}}{6}- \\frac{\\sqrt{6}}{6} \\times 3+3 \\times \\frac{\\sqrt{6}}{3}=0$.\n所以 $A S / /$ 平面 $P Q R$, 又 $\\left|\\overrightarrow{G R} \\cdot \\overrightarrow{n_0}\\right|=\\frac{2 \\sqrt{6}}{3}$, 所以 $S A$ 到平面 $P Q R$ 的距离为 $\\frac{2 \\sqrt{6}}{3}$.\n(2) $\\overrightarrow{P O}=\\{1,-1,-2\\}, \\overrightarrow{B D}=\\{-6,-6,0\\}$, 所以 $\\overrightarrow{P O} \\cdot \\overrightarrow{B D}=\\{1,-1$, $-2\\} \\cdot\\{-6,-6,0\\}=0$, 即 $P O \\perp B D$, 点 $P$ 到 $B D$ 的距离即为线段 $P O$ 的长度,故点 $P$ 到 $B D$ 的距离 $P O=\\sqrt{1^2+1^2+2^2}=\\sqrt{6}$.\n(3) 因为 $\\overrightarrow{O N}-\\overrightarrow{O S}=\\overrightarrow{S N}=l\\{-3,3,-3\\}=\\{-3 l, 3 l,-3 l\\}$, 所以 $N(-3 l, 3 l, 3-3 l)$.\n同理, $M(t, t, 0)(t \\in \\mathbf{R})$.\n所以 $M N^2=(t+3 l)^2+(t-3 l)^2+(3 l-3)^2=2 t^2+27\\left(l-\\frac{1}{3}\\right)^2+6$.\n故 $t=0, l=\\frac{1}{3}$ 时, $M N_{\\text {min }}=\\sqrt{6}$.", + "remark": "", + "figures": [ + "./images/volume8/figures/fig-c6i4.png", + "./images/volume8/figures/fig-c6i4.png" + ] +} \ No newline at end of file diff --git a/processed_dataset/proof/1956.json b/processed_dataset/proof/1956.json new file mode 100644 index 0000000000000000000000000000000000000000..9b34c61b37a949e25cd984971d3ee30e91a74515 --- /dev/null +++ b/processed_dataset/proof/1956.json @@ -0,0 +1,10 @@ +{ + "source_file": "./raw_volume-zh/volume8/chapter6.tex", + "problem_type": "proof", + "problem": "例5. 如图() 所示, 平行六面体 $A B C D- A_1 B_1 C_1 D_1$ 的底面是菱形, 且 $\\angle C_1 C B=\\angle C_1 C D= \\angle B C D=60^{\\circ}$,\n(1) 证明: $C C_1 \\perp B D$;\n(2)当 $\\frac{C D}{C C_1}$ 为何值时 $A_1 C \\perp$ 平面 $C_1 B D$ ?", + "solution": "分析:与解 (1) 设 $\\overrightarrow{C D}=\\vec{x}, \\overrightarrow{C B}=\\vec{y}, \\overrightarrow{C C_1}= \\vec{z}$, 且 $|\\vec{x}|=a,|\\vec{y}|=a,|\\vec{z}|=b$, 则 $\\overrightarrow{B D}=\\vec{x}-\\vec{y}$,\n因为 $\\overrightarrow{C C_1} \\cdot \\overrightarrow{B D}=\\vec{z} \\cdot(\\vec{x}-\\vec{y})=\\vec{z} \\cdot \\vec{x}-\\vec{z} \\cdot \\vec{y}=a b \\cos 60^{\\circ}-a b \\cos 60^{\\circ}=0$,\n所以 $C C_1 \\perp B D$, 证毕.\n(2) $\\overrightarrow{C A_1}=\\vec{x}+\\vec{y}+\\vec{z}, \\overrightarrow{C_1 D}=\\vec{x}-\\vec{z}$.\n$$\n\\begin{aligned}\n\\overrightarrow{C A_1} \\cdot \\overrightarrow{C_1 D} & =(\\vec{x}+\\vec{y}+\\vec{z}) \\cdot(\\vec{x}-\\vec{z}) \\\\\n& =\\vec{x}^2-\\vec{x} \\cdot \\vec{z}+\\vec{y} \\cdot \\vec{x}-\\vec{y} \\cdot \\vec{z}+\\vec{z} \\cdot \\vec{x}-\\vec{z}^2 \\\\\n& =a^2+a^2 \\cos 60^{\\circ}-a b \\cos 60^{\\circ}-b^2 \\\\\n& =\\frac{3}{2} a^2-\\frac{1}{2} a b-b^2 \\\\\n& =(a-b)\\left(\\frac{3}{2} a+b\\right) .\n\\end{aligned}\n$$\n又因为 $A_1 C \\perp$ 平面 $C_1 B D$, 所以 $A_1 C \\perp C_1 D$, 即 $\\overrightarrow{C A_1} \\cdot \\overrightarrow{C_1 D}=0$, 且 $a>0$, $b>0$, 所以 $a=b$.\n另一方面,\n$$\n\\begin{aligned}\n\\overrightarrow{C A_1} \\cdot \\overrightarrow{B D} & =(\\vec{x}+\\vec{y}+\\vec{z}) \\cdot(\\vec{x}-\\vec{y}) \\\\\n& =x^2-\\vec{x} \\cdot \\vec{y}+\\vec{y} \\cdot \\vec{x}-\\vec{y}^2+\\vec{z} \\cdot \\vec{x}-\\vec{z} \\cdot \\vec{y} \\\\\n& =a^2-a^2+a b \\cos 60^{\\circ}-a b \\cos 60^{\\circ}=0,\n\\end{aligned}\n$$\n所以当 $\\frac{C D}{C C_1}=1$ 时, $A_1 C \\perp C_1 D$ 且 $A_1 C \\perp B D$ 即 $A_1 C \\perp$ 平面 $C_1 B D$.", + "remark": "", + "figures": [ + "./images/volume8/figures/fig-c6i5.png" + ] +} \ No newline at end of file diff --git a/processed_dataset/proof/1957.json b/processed_dataset/proof/1957.json new file mode 100644 index 0000000000000000000000000000000000000000..ad16a91398c17da4b7bcb5af3472edee52f97592 --- /dev/null +++ b/processed_dataset/proof/1957.json @@ -0,0 +1,8 @@ +{ + "source_file": "./raw_volume-zh/volume8/chapter7.tex", + "problem_type": "proof", + "problem": "例1. 已知单位圆的内接正 $n$ 边形 $A_1 A_2 \\cdots A_n$ 及圆周上一点 $P$, 求证: $\\sum_{k=1}^n\\left|P A_k\\right|^2=2 n$.", + "solution": "分析:与解设 $\\zeta=\\mathrm{e}^{\\frac{2 \\pi \\mathrm{i}}{n}}, A_1, \\cdots, A_n$ 对应的复数是 $1, \\zeta, \\zeta^2, \\cdots, \\zeta^{-1}$. 又设 $P$ 点 (对应的复数) 为 $z=\\mathrm{e}^{\\mathrm{i} \\theta}$. 则我们有\n$$\n\\begin{aligned}\n\\sum_{k=1}^n\\left|P A_k\\right|^2 & =\\sum_{k=0}^{n-1}\\left|z-\\zeta^k\\right|^2=\\sum_{k=0}^{n-1}\\left(z-\\zeta^k\\right)\\left(\\bar{z}-\\zeta^{-k}\\right) \\\\\n& =\\sum_{k=0}^{n-1}\\left(|z|^2-\\zeta^k \\bar{z}-\\zeta^{-k} z+1\\right) \\\\\n& =2 n-\\bar{z} \\sum_{k=0}^{n-1} \\zeta^k-z \\sum_{k=0}^{n-1} \\zeta^{-k}=2 n\n\\end{aligned}\n$$\n最后一步应用了 $1+\\zeta+\\zeta^2+\\cdots+\\zeta^{n-1}=0$, 证毕.", + "remark": "", + "figures": [] +} \ No newline at end of file diff --git a/processed_dataset/proof/1958.json b/processed_dataset/proof/1958.json new file mode 100644 index 0000000000000000000000000000000000000000..eca9dc990678ca35fc52ea313e43b7d88a8eca26 --- /dev/null +++ b/processed_dataset/proof/1958.json @@ -0,0 +1,8 @@ +{ + "source_file": "./raw_volume-zh/volume8/chapter7.tex", + "problem_type": "proof", + "problem": "例2. 设 $P(x), Q(x), R(x)$ 及 $S(x)$ 都是多项式, 且\n$$\nP\\left(x^5\\right)+x Q\\left(x^5\\right)+x^2 R\\left(x^5\\right)=\\left(x^4+x^3+x^2+x+1\\right) S(x), \\label{eq1}\n$$\n求证: $x-1$ 是 $P(x), Q(x), R(x)$ 及 $S(x)$ 的公因式.", + "solution": "分析:与解设 $\\zeta$ 是一个 5 次单位根 $(\\zeta \\neq 1)$, 在 式\\ref{eq1} 中取 $x=\\zeta, \\zeta^2, \\zeta^3, \\zeta^4$, 得出\n$$\n\\left(\\zeta^k\\right)^2 R(1)+\\zeta^k Q(1)+P(1)=0(k=1,2,3,4),\n$$\n这意味着多项式 $x^2 R(1)+x Q(1)+P(1)$ 有四个不同的零点, 从而必须 $R(1)=Q(1)=P(1)=0$.\n再将 $x=1$ 代入 (1), 得 $S(1)=0$.\n于是 $P(x), Q(x), R(x)$ 及 $S(x)$ 都有因式 $x-1$, 证毕.", + "remark": "", + "figures": [] +} \ No newline at end of file diff --git a/processed_dataset/proof/1959.json b/processed_dataset/proof/1959.json new file mode 100644 index 0000000000000000000000000000000000000000..9d0329135b6e39f247641976d2701f4c8bed5861 --- /dev/null +++ b/processed_dataset/proof/1959.json @@ -0,0 +1,8 @@ +{ + "source_file": "./raw_volume-zh/volume8/chapter7.tex", + "problem_type": "proof", + "problem": "例3. 求证: 不存在四个整系数多项式 $f_k(x)(k \\doteq 1,2,3,4)$, 使得恒等式\n$$\n9 x+4=f_1^3(x)+f_2^3(x)+f_3^3(x)+f_4^3(x) . \\label{eq1}\n$$\n成立.", + "solution": "分析:与解本题看上去平平常常,但自己做起来却未必顺顺当当.\n记 $\\omega$ 是三次单位根 $(\\omega \\neq 1)$, 则对任意整系数多项式 $f(x)$, 利用 $\\omega^3=1$ 及 $\\omega^2=-1-\\omega$ 可将 $f(\\omega)$ 化为 $a+b \\omega$ ( $a 、 b$ 是整数), 于是 (注意 $\\left.1+\\omega+\\omega^2=0\\right)$\n$$\nf^3(\\omega)=(a+b \\omega)^3=a^3+b^3-3 a b^2+3 a b(a-b) \\omega .\n$$\n由于 $a b(a-b)$ 总是偶数,故若存在形如 式\\ref{eq1} 的恒等式, 以 $x=\\omega$ 代入, 即得\n$$\n9 \\omega+4=A+B \\omega . \\label{eq2}\n$$\n这里 $A 、 B$ 都是整数,且 $B$ 是偶数.\n但由式\\ref{eq2}易知 $B=9$, 这显然不可能,证毕.", + "remark": "", + "figures": [] +} \ No newline at end of file diff --git a/processed_dataset/proof/1960.json b/processed_dataset/proof/1960.json new file mode 100644 index 0000000000000000000000000000000000000000..a329b4ab837a93babcc86295dda9b33c9e0682dd --- /dev/null +++ b/processed_dataset/proof/1960.json @@ -0,0 +1,8 @@ +{ + "source_file": "./raw_volume-zh/volume8/chapter7.tex", + "problem_type": "proof", + "problem": "例4. 设 $\\varepsilon=\\cos \\frac{2 \\pi}{n}+\\mathrm{i} \\sin \\frac{2 \\pi}{n}$, 求证:\n(1) $(1-\\varepsilon)\\left(1-\\varepsilon^2\\right) \\cdots\\left(1-\\varepsilon^{n-1}\\right)=n$;\n(2) $\\sin \\frac{\\pi}{n} \\sin \\frac{2 \\pi}{n} \\cdots \\sin \\frac{(n-1) \\pi}{n}=\\frac{n}{2^{n-1}}$.", + "solution": "分析:与解方程 $x^n-1=0$ 的 $n$ 个单位根是\n$$\n\\varepsilon_k=\\cos \\frac{2 k \\pi}{n}+\\mathrm{i} \\sin \\frac{2 k \\pi}{n},(k=0,1, \\cdots, n-1)\n$$\n注意到 $\\varepsilon=\\cos \\frac{2 \\pi}{n}+\\mathrm{i} \\sin \\frac{2 \\pi}{n}$, 从而有\n$$\n\\varepsilon_k=\\varepsilon^k\n$$\n于是, 由\n$$\nx^n-1=(x-1)(x-\\varepsilon)\\left(x-\\varepsilon^2\\right) \\cdots\\left(x-\\varepsilon^{n-1}\\right)\n$$\n得\n$$\n\\begin{aligned}\n& (x-\\varepsilon)\\left(x-\\varepsilon^2\\right) \\cdots\\left(x-\\varepsilon^{n-1}\\right) \\\\\n= & \\frac{x^n-1}{x-1} \\\\\n= & x^{n-1}+x^{n-2}+\\cdots+x+1 .\n\\end{aligned}\n$$\n即有\n$$\n(x-\\varepsilon)\\left(x-\\varepsilon^2\\right) \\cdots\\left(x-\\varepsilon^{n-1}\\right)=x^{n-1}+x^{n-2}+\\cdots+x+1 . \\label{eq1}\n$$\n(1) 在\\ref{eq1}式中, 令 $x=1$, 立得\n$$\n(1-\\varepsilon)\\left(1-\\varepsilon^2\\right) \\cdots\\left(1-\\varepsilon^{n-1}\\right)=n . \\label{eq2}\n$$\n(2) 对\\ref{eq2}式的两边取模,并注意到\n$$\n\\left|1-\\varepsilon^k\\right|=2 \\sin \\frac{k \\pi}{n},\n$$\n立得\n$$\n2^{n-1} \\sin \\frac{\\pi}{n} \\sin \\frac{2 \\pi}{n} \\cdots \\sin \\frac{(n-1) \\pi}{n}=n,\n$$\n即有\n$$\n\\sin \\frac{\\pi}{n} \\sin \\frac{2 \\pi}{n} \\cdots \\sin \\frac{(n-1) \\pi}{n}=\\frac{n}{2^{n-1}},\n$$\n证毕.", + "remark": "", + "figures": [] +} \ No newline at end of file diff --git a/processed_dataset/proof/1961.json b/processed_dataset/proof/1961.json new file mode 100644 index 0000000000000000000000000000000000000000..95dbd43e95adfe4fade496599cf7127f7618d543 --- /dev/null +++ b/processed_dataset/proof/1961.json @@ -0,0 +1,8 @@ +{ + "source_file": "./raw_volume-zh/volume8/chapter7.tex", + "problem_type": "proof", + "problem": "例6. 有 $m$ 个男孩与 $n$ 个女孩围坐在一个圆周上 $(m>0, n>0, m+ n \\geqslant 3)$, 将顺序相邻的 3 人中恰有 1 个男孩的组数记作 $a$, 顺序相邻的 3 人中恰有 1 个女孩的组数记作 $b$, 求证: $a-b$ 是 3 的倍数.", + "solution": "分析:与解用 $a_k$ 表示小孩, 且将 $a_k$ 赋值为 $a_k=\\left\\{\\begin{array}{l}\\omega, a_k \\text { 表示男孩时, } \\\\ \\bar{\\omega}, a_k \\text { 表示女孩时.\n}\\end{array}\\right.$\n其中 $\\omega=-\\frac{1}{2}+\\frac{\\sqrt{3}}{2} \\mathrm{i}$, 有 $\\omega^{3 m}=1$, 并且\n$$\na_k a_{k+1} a_{k+2}=\\left\\{\\begin{array}{l}\n\\omega^{-1},\\left(a_k, a_{k+1}, a_{k+2} \\text { 中恰有一个男孩 }\\right) \\\\\n\\omega,\\left(a_k, a_{k+1}, a_{k+2} \\text { 中恰有一个女孩 }\\right) \\\\\n1,\\left(a_k, a_{k+1}, a_{k+2} \\text { 中全都是男 (女) 孩 }\\right)\n\\end{array}\\right.\n$$\n从而得\n$$\n\\begin{aligned}\n1 & =\\left(a_1 a_2 \\cdots a_{m+n}\\right)^3 \\\\\n& =\\left(a_1 a_2 a_3\\right)\\left(a_2 a_3 a_4\\right) \\cdots\\left(a_{m+n} a_1 a_2\\right) \\\\\n& =\\omega^{b-a},\n\\end{aligned}\n$$\n故 $a-b$ 是 3 的倍数,证毕.", + "remark": "注:本题相当于是一个复数赋值问题.", + "figures": [] +} \ No newline at end of file diff --git a/processed_dataset/proof/1962.json b/processed_dataset/proof/1962.json new file mode 100644 index 0000000000000000000000000000000000000000..fdb429d101366b6196b20ea7c3a144471f3c2959 --- /dev/null +++ b/processed_dataset/proof/1962.json @@ -0,0 +1,8 @@ +{ + "source_file": "./raw_volume-zh/volume8/chapter7.tex", + "problem_type": "proof", + "problem": "例7. 设 $z_k(k=0,1, \\cdots, n-1)$ 是 $z^n-1=0$ 的 $n$ 个根, 定义\n$$\nf(x)=a_m x^m+a_{m-1} x^{m-1}+\\cdots+a_1 x+a_0,\n$$\n其中 $m$ 为小于 $n$ 的正整数, 求证: $-\\frac{1}{n} \\sum_{k=0}^{n-1} f\\left(z_k\\right)=a_0$.", + "solution": "分析:与解令 $z_k=\\cos \\frac{2 k \\pi}{n}+\\operatorname{isin} \\frac{2 k \\pi}{n}=z_1^k(k=0,1, \\cdots, n-1)$,\n则由 $l2$; 如 $f(z)$ 恒为 0 , 则当然有 $\\left|P\\left(\\zeta_j\\right)\\right|=2$. 这就证明了式\\ref{eq1}.\n上面的论证还表明,如果式\\ref{eq1}成立等号, 必须 $f\\left(\\zeta_j\\right)=0(j=1,2, \\cdots, n)$, 这意味着 $f(z)==0$, 即 $P(z)=z^n+1$, 所以 $z_1, \\cdots, z_n$ 构成正 $n$ 边形, 证毕.", + "remark": "", + "figures": [] +} \ No newline at end of file diff --git a/processed_dataset/proof/1964.json b/processed_dataset/proof/1964.json new file mode 100644 index 0000000000000000000000000000000000000000..b34192ae11b33739b82432f5af377b943b93199b --- /dev/null +++ b/processed_dataset/proof/1964.json @@ -0,0 +1,11 @@ +{ + "source_file": "./raw_volume-zh/volume8/chapter8.tex", + "problem_type": "proof", + "problem": "例2. 设 $n(\\geqslant 3)$ 个复数 $z_1, z_2, \\cdots, z_n$ 满足\n(1) $z_1+z_2+\\cdots+z_n=0$;\n(2) $\\left|z_i\\right|<1, i=1,2, \\cdots, n$.\n证明: 存在 $i 、 j$, 使得 $1 \\leqslant i) 所示.\n由条件 (1), $z_1+z_2+\\cdots+z_n=0$, 可知 $z_2, \\cdots$, $z_n$ 中必有一个复数的实部小于 0. 从而 $\\overrightarrow{O Z}_2, \\cdots, \\overrightarrow{O Z_n}$ 中必有一个向量落在 $\\angle y O A$ 或 $\\angle y^{\\prime} O B$ 内, 不妨设 $\\overrightarrow{O Z_2}$ 落在 $\\angle y O A$ 内.\n作射线 $O C$, 使得 $\\angle Z_2 O C= 120^{\\circ}$, 则 $z_3, \\cdots, z_n$ 对应的向量不能落在 $\\angle B O C$ 内.\n综上所述, 可知 $\\overrightarrow{O Z}_1, \\overrightarrow{O Z_2}, \\cdots, \\overrightarrow{O Z_n}$, 都落在 $\\angle A O C$ 内, 于是, 将该复平面适当旋转后, 可使向量 $\\overrightarrow{O Z}_1, \\overrightarrow{O Z}_2, \\cdots, \\overrightarrow{O Z_n}$ 都落在 $y$ 轴的右方, 它们的实部都不小于零,这与 (1)矛盾.\n所以, 在 $z_1, z_2, \\cdots, z_n$ 中, 存在 $i 、 j, 1 \\leqslant i) , 令 $z=z_i+z_j$, 则可知 $\\angle Z_i O Z$ 和 $\\angle Z O Z_j$ 中必有 -一个 $\\geqslant 60^{\\circ}$. 而 $\\overrightarrow{Z_i Z}=\\overrightarrow{O Z_j}, \\overrightarrow{Z_j Z}=\\overrightarrow{O Z}_i$ 及 $\\angle O Z_i Z= \\angle O Z_j Z=180^{\\circ}-\\angle Z_i O Z_j \\leqslant 60^{\\circ}$, 就可知\n$$\n|z| \\leqslant \\max \\left\\{\\left|z_i\\right|,\\left|z_j\\right|\\right\\}<1 .\n$$", + "figures": [ + "./images/volume8/figures/fig-c8i1.png", + "./images/volume8/figures/fig-c8i2.png" + ] +} \ No newline at end of file diff --git a/processed_dataset/proof/1965.json b/processed_dataset/proof/1965.json new file mode 100644 index 0000000000000000000000000000000000000000..712ac0326734264cc555b8271ea0121a2bc9b9fa --- /dev/null +++ b/processed_dataset/proof/1965.json @@ -0,0 +1,8 @@ +{ + "source_file": "./raw_volume-zh/volume8/chapter8.tex", + "problem_type": "proof", + "problem": "例3. 设 $p=\\overline{a_n a_{n-1} \\cdots a_0}=a_n \\times 10^n+a_{n-1} \\times 10^{n-1}+\\cdots+a_1 \\times 10+a_0$ 是十进制表示下的一个质数, 这里 $a_n>0$. 证明: $f(x)=a_n x^n+\\cdots+a_0$ 在整系数范围内不可约.", + "solution": "分析:与解从 $f(x)$ 的根 $x_0$ 出发, 先证明: $\\operatorname{Re}\\left(x_0\\right) \\leqslant 0$ 或者 $\\left|x_0\\right|<4$, 这里 $\\operatorname{Re}\\left(x_0\\right)$ 表示 $x_0$ 的实部.\n事实上, 若 $\\operatorname{Re}\\left(x_0\\right) \\leqslant 0$ 或 $\\left|x_0\\right| \\leqslant 1$, 则上述论断已成立.\n当 $\\operatorname{Re}\\left(x_0\\right)>0$, 且 $\\left|x_0\\right|>1$ 时,有 $\\operatorname{Re}\\left(\\frac{1}{x_0}\\right)=\\frac{\\operatorname{Re}\\left(x_0\\right)}{\\left|x_0\\right|^2}>0$. 于是, 有\n$$\n0=\\left|\\frac{f\\left(x_0\\right)}{x_0^n}\\right| \\geqslant\\left|a_n+\\frac{a_{n-1}}{x_0}\\right|-\\frac{a_{n-2}}{\\left|x_0\\right|^2}-\\cdots-\\frac{a_0}{\\left|x_0\\right|^n}\n$$\n$$\n\\begin{aligned}\n& \\geqslant \\operatorname{Re}\\left(a_n+\\frac{a_{n-1}}{x_0}\\right)-\\left(\\frac{9}{\\left|x_0\\right|^2}+\\cdots+\\frac{9}{\\left|x_0\\right|^n}\\right) \\\\\n& \\geqslant a_n-\\frac{9}{\\left|x_0\\right|^2-\\left|x_0\\right|} \\geqslant 1-\\frac{9}{\\left|x_0\\right|^2-\\left|x_0\\right|},\n\\end{aligned}\n$$\n于是 $\\left|x_0\\right|^2-\\left|x_0\\right|-9 \\leqslant 0$, 故 $\\left|x_0\\right| \\leqslant \\frac{1+\\sqrt{37}}{2}<4$.\n下面,利用上述论断证明 $f(x)$ 在整系数范围内中不可约.\n若存在非常数的整系数多项式 $g(x)$ 和 $h(x)$, 使得 $f(x)=g(x) h(x)$, 设 $g(x)=b_m\\left(x-r_1\\right) \\cdots\\left(x-r_m\\right)$. 对于 $g(10)$ 而言,一方面 $g(10) \\in \\mathbf{Z}$, 另一方面, 对 $1 \\leqslant i \\leqslant m$, 由于 $r_i$ 也是 $f(x)$ 的根, 如果 $r_i \\in \\mathbf{R}$, 则 $r_i \\leqslant 0$ (否则, 由 $f(x)$ 的系数均非负, 将导数 $\\left.f\\left(r_i\\right)>0,\\right)$ 故 $10-r_i \\geqslant 10$; 如果 $r_i \\notin \\mathbf{R}$, 则 $\\bar{r}_i$ 也是 $f(x)$ 的根, 这时\n$$\n\\left(10-r_i\\right)\\left(10-\\bar{r}_i\\right)=100-20 \\operatorname{Re}\\left(r_i\\right)+\\left|r_i\\right|^2>20,\n$$\n所以, 总有 $|g(10)|>\\left|b_m\\right| \\geqslant 1$, 同理 $|h(10)|>1$.\n但是, $f(10)=g(10) h(10)$ 为质数,矛盾.\n证毕.", + "remark": "注:从本题的证明过程中我们知道: 多项式根的分布情况对多项式的分解起着举足轻重的作用.", + "figures": [] +} \ No newline at end of file diff --git a/processed_dataset/proof/1966.json b/processed_dataset/proof/1966.json new file mode 100644 index 0000000000000000000000000000000000000000..30f6870fa065da2b67d5ce6bef3d1375cd974803 --- /dev/null +++ b/processed_dataset/proof/1966.json @@ -0,0 +1,8 @@ +{ + "source_file": "./raw_volume-zh/volume8/chapter8.tex", + "problem_type": "proof", + "problem": "例4. 是否存在 2002 个不同的正实数 $a_1, a_2, \\cdots, a_{2002}$, 使得对任意正整数 $k, 1 \\leqslant k \\leqslant 2002$, 多项式 $a_{k+2001} x^{2001}+a_{k+2000} x^{2000}+\\cdots+a_{k+1} x+a_k$ 的每个复根 $z$ 都满足 $|\\operatorname{Im} z| \\leqslant|\\operatorname{Re} z|$ ? (约定 $a_{2002+i}=a_i, i=1,2, \\cdots, 2001$.)", + "solution": "分析:与解不存在.\n用反证法.\n若存在正实数 $a_1, a_2, \\cdots, a_{2002}$ 满足题设要求, 对一固定的 $k$, 设 $a_{k+2001} x^{2001}+a_{k+2000} x^{2000}+\\cdots+a_{k+1} x+a_k=0$ 的复根为 $z_1, z_2, \\cdots, z_{2001}$, 那么由于 $\\left|\\operatorname{Im} z_j\\right| \\leqslant\\left|\\operatorname{Re} z_j\\right|(1 \\leqslant j \\leqslant 2001)$, 而\n$$\n\\begin{aligned}\nz_j^2 & =\\left(\\operatorname{Re} z_j+\\mathrm{i} \\operatorname{Im} z_j\\right)^2 \\\\\n& =\\left(\\operatorname{Re} z_j\\right)^2-\\left(\\operatorname{Im} z_j\\right)^2+2\\left(\\operatorname{Re} z_j\\right)\\left(\\operatorname{Im} z_j\\right) i,\n\\end{aligned}\n$$\n即 $z_j^2$ 的实部 $\\operatorname{Re}\\left(z_j^2\\right)=\\left(\\operatorname{Re} z_j\\right)^2-\\left(\\operatorname{Im} z_j\\right)^2 \\geqslant 0(1 \\leqslant j \\leqslant 2001)$, 所以\n$$\n\\operatorname{Re}\\left(z_1^2+z_2^2+\\cdots+z_{2001}^2\\right)=\\operatorname{Re}\\left(z_1^2\\right)+\\operatorname{Re}\\left(z_2^2\\right)+\\cdots+\\operatorname{Re}\\left(z_{2001}^2\\right) \\geqslant 0 . \\label{eq1}\n$$\n而由韦达定理\n$$\nz_1+z_2+\\cdots+z_{2001}=\\frac{-a_{k+2000}}{a_{k+2001}}, \\sum_{1 \\leqslant j \\leqslant l \\leqslant 2001} z_j z_l=\\frac{a_{k+1999}}{a_{k+2001}},\n$$\n所以\n$$\n\\begin{aligned}\nz_1^2+z_2^2+\\cdots+z_{2001}^2 & =\\left(z_1+z_2+\\cdots+z_{2001}\\right)^2-2 \\sum_{1 \\leqslant j \\leqslant l \\leqslant 2001} z_j z_l \\\\\n& =\\frac{a_{k+2000}^2-2 a_{k+1999} a_{k+2001}}{a_{k+2001}^2}\n\\end{aligned}\n$$\n即 $z_1^2+z_2^2+\\cdots+z_{2001}^2$ 是一个实数.\n又由 式\\ref{eq1} 知, 其实部 $\\geqslant 0$, 所以它是一个非负实数, 即\n$$\n\\frac{a_{k+2000}^2-2 a_{k+1999} a_{k+2001}}{a_{k+2001}^2} \\geqslant 0 \\Rightarrow a_{k+2000}^2-2 a_{k+1999} a_{k+2001} \\geqslant 0 .\n$$\n上式对每个 $1 \\leqslant k \\leqslant 2002$ 均成立, 即当 $1 \\leqslant j \\leqslant 2002$, 均有 $a_j^2- 2 a_{j-1} a_{j+1} \\geqslant 0$. 但这是不可能的,事实上:\n设 $a_{j_0}$ 是 $a_1, a_2, \\cdots, a_{2002}$ 中最小的一个, 那么 $a_{j_0}^2-2 a_{j_0-1} a_{j_0+1} \\leqslant a_{j_0}^2- 2 a_{j_0} a_{j_0}=-a_{j_0}^2<0$, 矛盾.", + "remark": "", + "figures": [] +} \ No newline at end of file diff --git a/processed_dataset/proof/1967.json b/processed_dataset/proof/1967.json new file mode 100644 index 0000000000000000000000000000000000000000..f19908d020e69f363b821244d5186d8496cbdd8a --- /dev/null +++ b/processed_dataset/proof/1967.json @@ -0,0 +1,10 @@ +{ + "source_file": "./raw_volume-zh/volume8/chapter8.tex", + "problem_type": "proof", + "problem": "例5. $n$ 是正整数, $a_j(j=1,2, \\cdots, n)$ 为复数, 且对集合 $\\{1,2, \\cdots, n\\}$ 的任一非空子集 $I$, 均有\n$$\n\\left|\\prod_{j \\in I}\\left(1+a_j\\right)-1\\right| \\leqslant \\frac{1}{2} . \\label{eq1}\n$$\n证明: $\\sum_{j=1}^n\\left|a_j\\right| \\leqslant 3$.", + "solution": "分析:与解设 $1+a_j=r_j \\mathrm{e}^{\\mathrm{i} \\theta_j},\\left|\\theta_j\\right| \\leqslant \\pi, j=1,2, \\cdots, n$, 则题设条件变为\n$$\n\\left|\\prod_{j \\in I} r_j \\cdot \\mathrm{e}^{\\mathrm{i} \\sum_{j \\in I} \\theta_j}-1\\right| \\leqslant \\frac{1}{2} .\n$$\n先证如下引理: 设 $r 、 \\theta$ 为实数, $r>0,|\\theta| \\leqslant \\pi$, $\\left|r \\mathrm{e}^{\\mathrm{i} \\theta}-1\\right| \\leqslant \\frac{1}{2}$, 则 $\\frac{1}{2} \\leqslant r \\leqslant \\frac{3}{2},|\\theta| \\leqslant \\frac{\\pi}{6}$, $\\left|r \\mathrm{e}^{\\mathrm{i} \\theta}-1\\right| \\leqslant|r-1|+|\\theta|$.\n引理的证明: 如图图(), 由复数的几何意义, 有 $\\frac{1}{2} \\leqslant r \\leqslant \\frac{3}{2},|\\theta| \\leqslant \\frac{\\pi}{6}$.\n又由\n$$\n\\begin{aligned}\n\\left|r \\mathrm{e}^{\\mathrm{i} \\theta}-1\\right| & =|r(\\cos \\theta+\\mathrm{i} \\sin \\theta)-1| \\\\\n& =|(r-1)(\\cos \\theta+\\mathrm{i} \\sin \\theta)+[(\\cos \\theta-1)+\\mathrm{i} \\sin \\theta]| \\\\\n& \\leqslant|r-1|+\\sqrt{(\\cos \\theta-1)^2+\\sin ^2 \\theta} \\\\\n& =|r-1|+\\sqrt{2(1-\\cos \\theta)}\n\\end{aligned}\n$$\n$$\n\\begin{aligned}\n& =|r-1|+2\\left|\\sin \\frac{\\theta}{2}\\right| \\\\\n& \\leqslant|r-1|+|\\theta|,\n\\end{aligned}\n$$\n得引理的另一部分.\n由式\\ref{eq1}及引理, 对 $|I|$ 用数学归纳法知:\n$$\n\\frac{1}{2} \\leqslant \\prod_{j \\in I} r_j \\leqslant \\frac{3}{2},\\left|\\sum_{j \\in I} \\theta_j\\right| \\leqslant \\frac{\\pi}{6}, \\label{eq2}\n$$\n由式\\ref{eq1}及引理知\n$$\n\\left|a_j\\right|=\\left|r_j \\mathrm{e}^{\\mathrm{i} \\theta_j}-1\\right| \\leqslant\\left|r_j-1\\right|+\\left|\\theta_j\\right|,\n$$\n因此\n$$\n\\begin{aligned}\n\\sum_{j=1}^n\\left|a_j\\right| & \\leqslant \\sum_{j=1}^n\\left|r_j-1\\right|+\\sum_{j=1}^n\\left|\\theta_j\\right| \\\\\n& =\\sum_{r_j \\geqslant 1}\\left|r_j-1\\right|+\\sum_{r_j<1}\\left|r_j-1\\right|+\\sum_{\\theta_j \\geqslant 0}\\left|\\theta_j\\right|+\\sum_{\\theta_j<0}\\left|\\theta_j\\right| .\n\\end{aligned}\n$$\n由式\\ref{eq2}知\n$$\n\\begin{aligned}\n\\sum_{r_j \\geqslant 1}\\left|r_j-1\\right| & =\\sum_{r_j \\geqslant 1}\\left(r_j-1\\right) \\leqslant \\prod_{r_j \\geqslant 1}\\left(1+r_j-1\\right)-1 \\\\\n& \\leqslant \\frac{3}{2}-1=\\frac{1}{2}, \\\\\n\\sum_{r_j<1}\\left|r_j-1\\right| & =\\sum_{r_j<1}\\left(1-r_j\\right) \\leqslant \\prod_{r_j<1}\\left(1-\\left(1-r_j\\right)\\right)^{-1}-1 \\\\\n& \\leqslant 2-1=1, \\\\\n\\sum_{j=1}^n\\left|\\theta_j\\right| & =\\sum_{\\theta_j \\geqslant 0} \\theta_j-\\sum_{\\theta_j<0} \\theta_j \\leqslant \\frac{\\pi}{6}-\\left(-\\frac{\\pi}{6}\\right) \\leqslant \\frac{\\pi}{3} .\n\\end{aligned}\n$$\n综上, 有\n$$\n\\sum_{j=1}^n\\left|a_j\\right| \\leqslant \\frac{1}{2}+1+\\frac{\\pi}{3}<3 .\n$$\n证毕.", + "remark": "", + "figures": [ + "./images/volume8/figures/fig-c8i3.png" + ] +} \ No newline at end of file diff --git a/processed_dataset/proof/1968.json b/processed_dataset/proof/1968.json new file mode 100644 index 0000000000000000000000000000000000000000..90a192cf381298dd791511974eb9c674f5dab6ef --- /dev/null +++ b/processed_dataset/proof/1968.json @@ -0,0 +1,10 @@ +{ + "source_file": "./raw_volume-zh/volume8/chapter8.tex", + "problem_type": "proof", + "problem": "例6. 设 $z_1 、 z_2 、 z_3$ 是 3 个模不大于 1 的复数, wr 、 $w_2$ 是方程 $\\left(z-z_1\\right)(z- \\left.z_2\\right)+\\left(z-z_2\\right)\\left(z-z_3\\right)+\\left(z-z_3\\right)\\left(z-z_1\\right)=0$ 的两个根.\n证明: 对 $j=1,2$, 3 , 都有\n$$\n\\min \\left\\{\\left|z_j-w_1\\right|,\\left|z_j-w_2\\right|\\right\\} \\leqslant 1 .\n$$", + "solution": "分析:与解由对称性, 只需证明: $\\min \\left\\{\\left|z_1-w_1\\right|,\\left|z_1-w_2\\right|\\right\\} \\leqslant 1$.\n不妨设 $z_1 \\neq w_1, w_2$. 令 $f(z)=\\left(z-z_1\\right)\\left(z-z_2\\right)+\\left(z-z_2\\right)\\left(z-z_3\\right)+ \\left(z-z_3\\right)\\left(z-z_1\\right)$, 由\n$$\nf(z)=3\\left(z-w_1\\right)\\left(z-w_2\\right),\n$$\n得\n$$\n3\\left(z_1-w_1\\right)\\left(z_1-w_2\\right)=\\left(z_1-z_2\\right)\\left(z_1-z_3\\right),\n$$\n因此,若 $\\left|z_1-z_2\\right|\\left|z_1-z_3\\right| \\leqslant 3$, 结论成立.\n另一方面, 由 $w_1+w_2=\\frac{2}{3}\\left(z_1+z_2+z_3\\right), w_1 w_2=\\frac{z_1 z_2+z_2 z_3+z_3 z_1}{3}$,\n又\n$$\n\\frac{1}{z-w_1}+\\frac{1}{z-w_2}=\\frac{2 z-\\left(w_1+w_2\\right)}{\\left(z-w_1\\right)\\left(z-w_2\\right)}=\\frac{3\\left(2 z-\\left(w_1+w_2\\right)\\right)}{f(z)},\n$$\n所以\n$$\n\\begin{aligned}\n\\frac{1}{z_1-w_1}+\\frac{1}{z_1-w_2} & =\\frac{3\\left(2 z_1-\\frac{2}{3}\\left(z_1+z_2+z_3\\right)\\right)}{\\left(z_1-z_2\\right)\\left(z_1-z_3\\right)} \\\\\n& =\\frac{2\\left(2 z_1-z_2-z_3\\right)}{\\left(z_1-z_2\\right)\\left(z_1-z_3\\right)},\n\\end{aligned}\n$$\n因此, 当 $\\left|\\frac{2 z_1-z_2-z_3}{\\left(z_1-z_2\\right)\\left(z_1-z_3\\right)}\\right| \\geqslant 1$ 时,结论成立.\n下设 $\\left|z_1-z_2\\right|\\left|z_1-z_3\\right|>3,\\left|\\frac{2 z_1-z_2-z_3}{\\left(z_1-z_2\\right)\\left(z_1-z_3\\right)}\\right|<1$.\n如图(), 考虑以 $A\\left(z_1\\right) 、 B\\left(z_2\\right) 、 C\\left(z_3\\right)$ 为顶点的三角形.\n记 $m_a$ 和 $h_a$ 分别是三角形 $A B C$ 的边 $B C$ 上的中线和高, 则 $b c>3,2 m_a6-4>0$, 所以 $\\angle A<90^{\\circ}$, 即 $\\triangle A B C$ 为锐角三角形.\n所以, $\\triangle A B C$ 为单位圆内的锐角三角形.\n平移 $\\triangle A B C$ \n使 $B 、 C$ 在单位圆周内(或圆周上), 延长 $C A$ 交单位圆于 $D$, 则由 $\\angle D \\leqslant \\angle A<\\frac{\\pi}{2}$ 得 $\\sin A \\geqslant \\sin D$, 所以 $2 k=\\frac{B C}{\\sin A} \\leqslant \\frac{B C}{\\sin D}=2$. 即 $\\triangle A B C$ 外接圆半径 $R \\leqslant 1$, 于是 $2 m_a), 设 $A A^{\\prime} 、 B B^{\\prime} 、 C C^{\\prime}$ 是 $\\triangle A B C$ 的外接圆的三条直径, $P$ 是 $\\triangle A B C$ 所在平面上任意一点, 点 $P$ 在 $B C 、 C A 、 A B$ 上的射影分别为 $D$ 、 $E 、 F, X$ 是点 $A^{\\prime}$ 关于点 $D$ 的对称点, $Y$ 是点 $B^{\\prime}$ 关于点 $E$ 的对称点, $Z$ 是点 $C^{\\prime}$ 关于点 $F$ 的对称点.\n求证: $\\triangle X Y Z \\backsim \\triangle A B C$.", + "solution": "分析:与解引人原点为 $O$ 的复平面, 设圆 $O$ 的半径为 1 , 仍以各点字母表示所在位置的复数,\n由 $A 、 B 、 C$ 在圆 $O$ 上知 $\\bar{A}=\\frac{1}{A}, \\bar{B}=\\frac{1}{B}, \\bar{C}=\\frac{1}{C}$, 于是由 $P D \\perp B C$ 于 $D$ 知\n$$\n\\begin{cases}\\frac{P-D}{C-B}=-\\frac{\\bar{P}-\\bar{D}}{\\bar{C}-\\bar{B}}, & (P D \\perp B C) \\\\ \\frac{C-D}{C-B}=\\frac{\\bar{C}}{\\bar{C}-\\bar{D}}, \\quad(D \\in B C) & \\end{cases}\n$$\n视为关于 $D 、 \\bar{D}$ 的方程, 解出 $D=\\frac{P+C-B C \\bar{P}+B}{2}$. 由 $A^{\\prime}$ 是圆 $O$ 中 $A$ 的对径点, 知 $A^{\\prime}=-A$, 故\n$$\n\\begin{aligned}\nX & =2 D-A^{\\prime}=2 D+A=(P+A+B+C)-B C \\bar{P} \\\\\n& =(P+A+B+C)-(A B C \\bar{P}) \\bar{A} .\n\\end{aligned}\n$$\n类似地有\n$$\n\\begin{aligned}\n& Y=(P+A+B+C)-(A B C \\bar{P}) \\bar{B}, \\\\\n& Z=(P+A+B+C)-(A B C \\bar{P}) \\bar{C} .\n\\end{aligned}\n$$\n但复平面上的变换 $\\phi: Z \\mapsto(A+B+C+P)+(-A B C \\bar{P}) Z$ 可以视为由平移, 对称, 旋转, 位似变换迭加的变换, 因此 $\\phi$ 是保角的, 故以 $\\phi(A), \\phi(B), \\phi(C)$ 为顶点的三角形 (顶点按顺序) 与 $\\triangle A B C$ 相似, 即 $\\triangle X Y Z \\backsim \\triangle A B C$. 证毕.", + "remark": "", + "figures": [ + "./images/volume8/figures/fig-c9i1.png" + ] +} \ No newline at end of file diff --git a/processed_dataset/proof/1976.json b/processed_dataset/proof/1976.json new file mode 100644 index 0000000000000000000000000000000000000000..24d3eae831b74821a8d9cddc6054374e9c8845c5 --- /dev/null +++ b/processed_dataset/proof/1976.json @@ -0,0 +1,8 @@ +{ + "source_file": "./raw_volume-zh/volume8/chapter9.tex", + "problem_type": "proof", + "problem": "例9. 设 $H=\\left\\{h(x) \\mid h(x)=\\frac{a x+b}{-b x+a}, a \\in \\mathbf{R}, b \\in \\mathbf{R}\\right\\}$, 求证: 若 $h_1(x)$ 、 $h_2(x) \\in H$, 则 $h_1\\left[h_2(x)\\right] 、 h_2\\left[h_1(x)\\right] \\in H$, 且 $h_1\\left[h_2(x)\\right]=h_2\\left[h_1(x)\\right]$.", + "solution": "分析:与解设 $h_1(x)=\\frac{a x+b}{-b x+a}, h_2(x)=\\frac{c x+d}{-d x+c}$, 则\n$$\n\\begin{aligned}\n& h_1\\left[h_2(x)\\right]=\\frac{a \\cdot \\frac{c x+d}{-d x+c}+b}{-b \\cdot \\frac{c x+d}{-d x+c}+a}=\\frac{(a c-b d) x+(a d+b c)}{-(a d+b c) x+(a c-b d)} \\in H, \\\\\n& h_2\\left[h_1(x)\\right]=\\frac{c \\cdot \\frac{a x+b}{-b x+a}+d}{-d \\cdot \\frac{a x+b}{-b x+a}+a}=\\frac{(a c-b d) x+(a d+b c)}{-(a d+b c) x+(a c-b d)} \\in H,\n\\end{aligned}\n$$\n且有 $h_1\\left[h_2(x)\\right]=h_2\\left[h_1(x)\\right]$.\n以上可以推广到任意个 $H$ 型函数, 即若 $h_1(x) 、 h_2(x) 、 \\cdots h_n(x) \\in H$, 则 $h_1\\left\\{h_2 \\cdots\\left[h_n(x)\\right]\\right\\} \\in H$, 且 $h_1\\left\\{h_2 \\cdots\\left[h_n(x)\\right]\\right\\}=h_1^{\\prime}\\left\\{h_2^{\\prime} \\cdots h_n^{\\prime}(x)\\right\\}$, 这里 $h_1^{\\prime}(x)$ 、 $h_2^{\\prime}(x) \\cdots h_n^{\\prime}(x)$ 是 $h_1(x) 、 h_2(x) \\cdots h_n(x)$ 的任一个排列.", + "remark": "注:从上面的证明过程中可受到启发:\n若 $h_1(x)=\\frac{a x+b}{-b x+a}$ 对应复数 $a+b \\mathrm{i}, h_2(x)=\\frac{c x+d}{-d x+c}$ 对应复数 $c+d \\mathrm{i}$, 则\n$$\nh_1\\left[h_2(x)\\right]=\\frac{(a c-b d) x+(a d+b c)}{-(a d+b c) x+(a c-b d)} \\text { 对应复数 }(a c-b d)+(a d+\n$$\n$b c) \\mathrm{i}$,\n且 $(a c-b d)+(a d+b c) \\mathrm{i}$ 恰好等于它们的乘积 $(a+b \\mathrm{i})(c+d \\mathrm{i})$.\n由此可得到解法如下:\n要求由任意个 $H$ 型函数迭代式所确定的函数表达式, 首先将已知函数所对应的复数写出, 然后加以相乘, 最后写出乘积复数所对应的 $H$ 型函数即为所求.", + "figures": [] +} \ No newline at end of file diff --git a/processed_dataset/proof/1977.json b/processed_dataset/proof/1977.json new file mode 100644 index 0000000000000000000000000000000000000000..41d260ae7b19680f4dab71e9b7e8d249cb4d0506 --- /dev/null +++ b/processed_dataset/proof/1977.json @@ -0,0 +1,8 @@ +{ + "source_file": "./raw_volume-zh/volume8/exercise1.tex", + "problem_type": "proof", + "problem": "问题4. 知复数 $z_1 、 z_2$ 满足 $2 z_1^2+z_2^2=2 z_1 z_2$, 且 $z_1+z_2$ 为纯虚数, 求证: 复数 $3 z_1-2 z_2$ 是实数.", + "solution": "令 $z_1+z_2=k \\mathrm{i}(k \\in \\mathbf{R}$, 且 $k \\neq 0)$, 由于 $2 z_1^2+z_2^2=2 z_1 z_2$ 等价于 $\\left(3 z_1-2 z_2\\right)^2=-\\left(z_1+z_2\\right)^2$.\n于是, 有 $3 z_1-2 z_2= \\pm \\mathrm{i}\\left(z_1+z_2\\right)= \\pm(k \\mathrm{i}) \\mathrm{i}= \\pm k \\in \\mathbf{R}$, 故知复数 $3 z_1-z_2$ 是实数.", + "remark": "", + "figures": [] +} \ No newline at end of file diff --git a/processed_dataset/proof/1978.json b/processed_dataset/proof/1978.json new file mode 100644 index 0000000000000000000000000000000000000000..a0c1cfd8db5b256fe80ba66c609ee54ebde0b718 --- /dev/null +++ b/processed_dataset/proof/1978.json @@ -0,0 +1,8 @@ +{ + "source_file": "./raw_volume-zh/volume8/exercise1.tex", + "problem_type": "proof", + "problem": "问题7. 求证: $[(2 a-b-c)+(b-c) \\sqrt{3} \\mathrm{i}]^3=[(2 b-c-a)+(c-a) \\sqrt{3} \\mathrm{i}]^3$.", + "solution": "设 $\\omega=-\\frac{1}{2}+\\frac{\\sqrt{3}}{2} \\mathrm{i}$, 则 $\\omega^3=1$.\n$$\n\\begin{aligned}\n\\text { 左边 } & =[2 a+b(-1+\\sqrt{3} \\mathrm{i})+c(-1-\\sqrt{3} \\mathrm{i})]^3=[2 a+2 b \\omega+2 c \\bar{\\omega}]^3, \\\\\n\\text { 右边 } & =[a(-1-\\sqrt{3} \\mathrm{i})+2 b+c(-1+\\sqrt{3} \\mathrm{i})]^3=(2 a \\bar{\\omega}+2 b+2 c \\omega)^3 \\\\\n& =\\left[\\bar{\\omega}\\left(2 a+2 b \\omega+2 c \\omega^2\\right)\\right]^3=(2 a+2 b \\omega+2 c \\bar{\\omega})^3 .\n\\end{aligned}\n$$\n所以左边 $=$ 右边, 等式成立, 证毕.", + "remark": "", + "figures": [] +} \ No newline at end of file diff --git a/processed_dataset/proof/1979.json b/processed_dataset/proof/1979.json new file mode 100644 index 0000000000000000000000000000000000000000..d3ddabdf579d7dc7c9191d2f50e49dac1b695c9d --- /dev/null +++ b/processed_dataset/proof/1979.json @@ -0,0 +1,8 @@ +{ + "source_file": "./raw_volume-zh/volume8/exercise2.tex", + "problem_type": "proof", + "problem": "问题6. 已知 $|z|=1, z^{11}+z=1$, 求复数 $z$.", + "solution": "由 $|z|=1$, 可设 $z=\\cos \\theta+i \\sin \\theta$, 且 $0 \\leqslant \\theta<2 \\pi$, 代入 $z^{11}+z=1$,\n得\n$$\n\\begin{gathered}\n(\\cos \\theta+i \\sin \\theta)^{11}+(\\cos \\theta+i \\sin \\theta)=1, \\\\\n(\\cos 11 \\theta+\\cos \\theta-1)+(\\sin 11 \\theta+\\sin \\theta) i=0,\n\\end{gathered}\n$$\n所以\n$$\n\\left\\{\\begin{array}{l}\n\\cos 11 \\theta+\\cos \\theta-1=0, \\\\\n\\sin 11 \\theta+\\sin \\theta=0,\n\\end{array}\\right.\n$$\n即\n$$\n\\cos 11 \\theta=1-\\cos \\theta, \\label{eq1}\n$$\n且\n$$\n\\sin 11 \\theta=-\\sin \\theta, \\label{eq2}\n$$\n由${ 式\\ref{eq1}}^2+{式\\ref{eq2}}^2$, 可知\n$$\n(1-\\cos \\theta)^2+(-\\sin \\theta)^2=1,\n$$\n于是\n$$\n\\begin{gathered}\n\\cos \\theta=\\frac{1}{2}, \\\\\n\\sin \\theta= \\pm \\frac{\\sqrt{3}}{2} .\n\\end{gathered}\n$$\n从而经验证知, $z=\\frac{1}{2} \\pm \\frac{\\sqrt{3}}{2} \\mathrm{i}$ 是原方程的解.", + "remark": "注:对 $z^{11}=1-z$ 两边取模, 得 $|z-1|=1$, 并结合 $|z|=1$, 亦可给出简明解法.", + "figures": [] +} \ No newline at end of file diff --git a/processed_dataset/proof/1980.json b/processed_dataset/proof/1980.json new file mode 100644 index 0000000000000000000000000000000000000000..ee633bf9cb2a142f23e2448cb9e6c3981f322204 --- /dev/null +++ b/processed_dataset/proof/1980.json @@ -0,0 +1,8 @@ +{ + "source_file": "./raw_volume-zh/volume8/exercise2.tex", + "problem_type": "proof", + "problem": "问题8. 求证: $\\sin (4 \\arcsin x)=4 x \\cdot \\sqrt{1-x^2} \\cdot\\left(1-2 x^2\\right) .(|x| \\leqslant 1)$", + "solution": "因为\n$$\n(\\cos \\alpha+i \\sin \\alpha)^4=1 \\cdot(\\cos 4 \\alpha+i \\sin 4 \\alpha)=\\cos 4 \\alpha+i \\sin 4 \\alpha,\n$$\n又因为\n$$\n\\begin{aligned}\n(\\cos \\alpha+\\mathrm{i} \\sin \\alpha)^4 & =\\left[(\\cos \\alpha+\\mathrm{i} \\sin \\alpha)^2\\right]^2=\\left[\\left(\\cos ^2 \\alpha-\\sin ^2 \\alpha\\right)+2 \\sin \\alpha \\cos \\alpha \\cdot \\mathrm{i}\\right]^2 \\\\\n& =\\left(\\cos ^4 \\alpha-6 \\sin ^2 \\alpha \\cos ^2 \\alpha+\\sin ^4 \\alpha\\right)+4 \\sin \\alpha \\cos \\alpha\\left(\\cos ^2 \\alpha-\\sin ^2 \\alpha\\right) \\mathrm{i},\n\\end{aligned}\n$$\n所以\n$$\n\\cos 4 \\alpha+\\operatorname{isin} 4 \\alpha=\\left(\\cos ^4 \\alpha-6 \\sin ^2 \\alpha \\cos ^2 \\alpha+\\sin ^4 \\alpha\\right)+4 \\sin \\alpha \\cdot \\cos \\alpha\\left(\\cos ^2 \\alpha-\\sin ^2 \\alpha\\right) \\text {. }\n$$\n根据复数相等的定义得\n$$\n\\sin 4 \\alpha=4 \\sin \\alpha \\cos \\alpha\\left(\\cos ^2 \\alpha-\\sin ^2 \\alpha\\right) .\n$$\n将 $\\alpha=\\arcsin x$ 代入上式则有\n$$\n\\begin{aligned}\n\\sin (4 \\arcsin x) & =4 \\sin (\\arcsin x) \\cdot \\cos (\\arcsin x) \\cdot\\left[\\cos ^2(\\arcsin x)-\\sin ^2(\\arcsin x)\\right] \\\\\n& =4 x \\cdot \\sqrt{1-x^2} \\cdot\\left[\\left(\\sqrt{1-x^2}\\right)^2-x^2\\right] \\\\\n& =4 x \\cdot \\sqrt{1-x^2} \\cdot\\left(1-2 x^2\\right) .\n\\end{aligned}\n$$\n即 $\\sin (4 \\arcsin x)=4 x \\cdot \\sqrt{1-x^2} \\cdot\\left(1-2 x^2\\right)$, 证毕.", + "remark": "", + "figures": [] +} \ No newline at end of file diff --git a/processed_dataset/proof/1981.json b/processed_dataset/proof/1981.json new file mode 100644 index 0000000000000000000000000000000000000000..4ce9d5366576c53d612f219da4f5d36fda4af2c6 --- /dev/null +++ b/processed_dataset/proof/1981.json @@ -0,0 +1,8 @@ +{ + "source_file": "./raw_volume-zh/volume8/exercise3.tex", + "problem_type": "proof", + "problem": "问题4. 设 $z$ 是模不为 1 的虚数, 记 $w=z+\\frac{1}{z}$, 设实数 $a$ 满足 $w^2+a w+1=0$, 证明: $-20, r \\neq 1, \\theta \\neq k \\pi)$, 则\n$$\n\\begin{aligned}\nw & =z+\\frac{1}{z}=r(\\cos \\theta+\\mathrm{i} \\sin \\theta)+\\frac{1}{r}(\\cos \\theta-\\mathrm{i} \\sin \\theta) \\\\\n& =\\left(r+\\frac{1}{r}\\right) \\cos \\theta+\\mathrm{i}\\left(r-\\frac{1}{r}\\right) \\sin \\theta,\n\\end{aligned}\n$$\n因为 $\\theta \\neq k \\pi, r>0$ 且 $r \\neq 1$, 所以 $\\left(r-\\frac{1}{r}\\right) \\sin \\theta \\neq 0$, 故 $w$ 是虚数, 即方程 $w^2+a w+1=0$ 有虚数根, 所以 $\\Delta=a^2-4<0$, 故 $-20$, 于是, 对任意 $x \\in \\mathbf{R}$, 有 $P(x)-Q(x)>0$. 而 $P(x) 、 Q(x)$ 亦是实数, 从而\n$$\nP(P(x))-Q(Q(x))=\\{P(P(x))-Q(P(x))\\}+\\{P(Q(x))-Q(Q(x))\\}>0 \\text {. }\n$$\n故方程 $P(P(x))=Q(Q(x))$ 无实根, 证毕.", + "remark": "", + "figures": [] +} \ No newline at end of file diff --git a/processed_dataset/proof/1983.json b/processed_dataset/proof/1983.json new file mode 100644 index 0000000000000000000000000000000000000000..f17dcf00f3a42cf9310648404b3f459d97395ec1 --- /dev/null +++ b/processed_dataset/proof/1983.json @@ -0,0 +1,8 @@ +{ + "source_file": "./raw_volume-zh/volume8/exercise3.tex", + "problem_type": "proof", + "problem": "问题8. 设 $a 、 b 、 c \\in \\mathbf{R}, b \\neq a c, a \\neq-c$. $z$ 是复数, $z^2-(a-c) z-b=0$. 求证 $\\left|\\frac{a^2+b-(a+c) z}{a c-b}\\right|=1$ 的充分必要条件是 $(a-c)^2+4 b \\leqslant 0$.", + "solution": "一方面, 若 $(a-c)^2+4 b \\leqslant 0$, 则由求根公式, 得\n$$\nz==\\frac{a-c \\pm \\sqrt{-(a-c)^2-4 b} \\cdot \\mathrm{i}}{2} .\n$$\n于是\n$$\n\\begin{aligned}\n\\left|\\frac{a^2+b-(a+c) z}{a c-b}\\right| & =\\left|\\frac{a^2+c^2+2 b \\mp(a+c) \\sqrt{-(a-c)^2-4 b} \\cdot \\mathrm{i}}{2(a c-b)}\\right| \\\\\n& =\\sqrt{\\frac{\\left(a^2+c^2+2 b\\right)^2-(a+c)^2\\left[(a-c)^2+4 b\\right]}{(2 a c-2 b)^2}} \\\\\n& =\\sqrt{\\frac{\\left(a^2+c^2+2 b\\right)^2-\\left(a^2-c^2\\right)^2-4 b(a+c)^2}{4(a c-b)^2}} \\\\\n& =\\sqrt{\\frac{\\left(a^2+b\\right)\\left(c^2+b\\right)-b\\left(a^2+c^2+2 a c\\right)}{(a c-b)^2}} \\\\\n& =\\sqrt{\\frac{a^2 c^2+b^2-2 a b c}{(a c-b)^2}}=1 .\n\\end{aligned}\n$$\n另一方面, 若 $\\left|\\frac{a^2+b-(a+c) z}{a c-b}\\right|=1$, 假设 $(a-c)^2+4 b>0$, 则 $z^2- (a-c) z-b=0$ 有两个不相等的实根.\n于是 $\\frac{a^2+b-(a+c) z}{a c-b}=1$ 或 -1 , 即 $z=a$ 或 $z=\\frac{-a^2+a c-2 b}{-(a+c)}$.\n当把 $z=a$ 代入 $z^2-(a-c) z-b=0$ 时, 得到 $a c-b=0$,与 $b \\neq a c$ 相矛盾.\n当把 $z=\\frac{-a^2+a c-2 b}{-(a+c)}$ 代入 $z^2-(a-c) z-b=0$ 时, 得到 $(b-a c)[(a- \\left.c)^2+4 b\\right]=0$, 这与条件 $b \\neq a c$ 及前面的假设 $(a-c)^2+4 b>0$ 相矛盾.\n于是, $a 、 \\frac{-a^2+a c-2 b}{-(a+c)}$ 均不是方程 $z^2-(a-c) z-b=0$ 的根,矛盾.\n故一定有 $(a-c)^2+4 b \\leqslant 0$, 证毕.", + "remark": "", + "figures": [] +} \ No newline at end of file diff --git a/processed_dataset/proof/1984.json b/processed_dataset/proof/1984.json new file mode 100644 index 0000000000000000000000000000000000000000..36f92db23544c2c34b6ba76a5297036ad7e2f165 --- /dev/null +++ b/processed_dataset/proof/1984.json @@ -0,0 +1,10 @@ +{ + "source_file": "./raw_volume-zh/volume8/exercise4.tex", + "problem_type": "proof", + "problem": "问题6. 如图(), 任意四边形 $A B C D$ 中, $M 、 N$ 分别为 $A D$ 、 $B C$ 的中点, $G$ 为 $M N$ 的中点, $O$ 为平面内的任意一点, 求证:\n(1) $\\overrightarrow{G A}+\\overrightarrow{G B}+\\overrightarrow{G C}+\\overrightarrow{G D}=\\overrightarrow{0}$;\n(2) $\\overrightarrow{O G}=\\frac{1}{4}(\\overrightarrow{O A}+\\overrightarrow{O B}+\\overrightarrow{O C}+\\overrightarrow{O D})$.", + "solution": "(1) 设 $\\overrightarrow{G M}=\\vec{a}, \\overrightarrow{M D}=\\vec{m}, \\overrightarrow{N C}=\\vec{n}$, 则 $\\overrightarrow{G N}=-\\vec{a}, \\overrightarrow{M A}=-\\vec{m}, \\overrightarrow{N B}= -\\vec{n}$\n从而 $\\overrightarrow{G A}=\\vec{a}-\\vec{m}, \\overrightarrow{G C}=-\\vec{a}+\\vec{n}, \\overrightarrow{G B}=-\\vec{a}-\\vec{n}, \\overrightarrow{G D}=\\vec{a}+\\vec{m}$.\n所以 $\\overrightarrow{G A}+\\overrightarrow{G B}+\\overrightarrow{G C}+\\overrightarrow{G D}=\\overrightarrow{0}$.\n(2) $\\overrightarrow{O A}=\\overrightarrow{O G}+\\overrightarrow{G A}, \\overrightarrow{O C}=\\overrightarrow{O G}+\\overrightarrow{G C}, \\overrightarrow{O B}=\\overrightarrow{O G}+\\overrightarrow{G B}, \\overrightarrow{O D}=\\overrightarrow{O G}+ \\overrightarrow{G D}$\n从而 $\\overrightarrow{O A}+\\overrightarrow{O B}+\\overrightarrow{O C}+\\overrightarrow{O D}=4 \\overrightarrow{O G}+\\overrightarrow{G A}+\\overrightarrow{G B}+\\overrightarrow{G C}+\\overrightarrow{G D}=4 \\overrightarrow{O G}$, 即 $\\overrightarrow{O G}=\\frac{1}{4}(\\overrightarrow{O A}+\\overrightarrow{O B}+\\overrightarrow{O C}+\\overrightarrow{O D})$, 证毕.", + "remark": "", + "figures": [ + "./images/volume8/figures/fig-c4p6.png" + ] +} \ No newline at end of file diff --git a/processed_dataset/proof/1985.json b/processed_dataset/proof/1985.json new file mode 100644 index 0000000000000000000000000000000000000000..7f92a65513957dd370ca4b47574ab0be28c950dd --- /dev/null +++ b/processed_dataset/proof/1985.json @@ -0,0 +1,8 @@ +{ + "source_file": "./raw_volume-zh/volume8/exercise4.tex", + "problem_type": "proof", + "problem": "问题7. 设 $A_1 A_2 A_3 A_4$ 为 $\\odot O$ 的内接四边形, $H_1 、 H_2 、 H_3 、 H_4$ 依次为 $\\triangle A_2 A_3 A_4$ 、 $\\triangle A_3 A_4 A_1 、 \\triangle A_4 A_1 A_2 、 \\triangle A_1 A_2 A_3$ 的垂心, 求证: $H_1 、 H_2 、 H_3 、 H_4$ 在同一个圆上,并定出该圆的圆心.", + "solution": "设 $\\odot O$ 的半径为 $R$, 设 $\\overrightarrow{O A_1}+\\overrightarrow{O A_2}+\\overrightarrow{O A_3}+\\overrightarrow{O A_4}=\\overrightarrow{O C}$, 则由例 3 知\n$$\n\\begin{gathered}\n\\overrightarrow{O H_1}=\\overrightarrow{O A_2}+\\overrightarrow{O A_3}+\\overrightarrow{O A_4}=\\overrightarrow{O C}-\\overrightarrow{O A_1}, \\\\\n\\left|\\overrightarrow{H_1 C}\\right|=\\left|\\overrightarrow{O C}-\\overrightarrow{O H_1}\\right|=\\left|\\overrightarrow{O A_1}\\right|=R .\n\\end{gathered}\n$$\n从而\n$$\n\\left|\\overrightarrow{H_1 C}\\right|=\\left|\\overrightarrow{O C}-\\overrightarrow{O H_1}\\right|=\\left|\\overrightarrow{O A_1}\\right|=R\n$$\n同理 $\\left|\\overrightarrow{H_2 C}\\right|=\\left|\\overrightarrow{O A_2}\\right|=R,\\left|\\overrightarrow{H_3 C}\\right|=\\left|\\overrightarrow{O A_3}\\right|=R,\\left|\\overrightarrow{H_4 C}\\right|=\\left|\\overrightarrow{O A_4}\\right|=R$. 所以 $H_1 、 H_2 、 H_3 、 H_4$ 在同一圆上, 以 $C$ 为圆心, 以 $R$ 为半径, 证毕.", + "remark": "", + "figures": [] +} \ No newline at end of file diff --git a/processed_dataset/proof/1986.json b/processed_dataset/proof/1986.json new file mode 100644 index 0000000000000000000000000000000000000000..f033aba2a7cc2c01b007b4ff58460a29eb7a596a --- /dev/null +++ b/processed_dataset/proof/1986.json @@ -0,0 +1,10 @@ +{ + "source_file": "./raw_volume-zh/volume8/exercise5.tex", + "problem_type": "proof", + "problem": "问题4. 如图(), 空间四边形 $A B C D$ 中, $P 、 Q$ 分别是对角线 $A C 、 B D$ 的中点.\n求证:\n(1) 若 $A B=C D, A D=B C$, 则 $P Q \\perp A C$, $P Q \\perp B D ;$\n(2) 若 $P Q \\perp A C, P Q \\perp B D$, 则 $A B=C D$, $A D=B C$.", + "solution": "(1) $\\overrightarrow{P Q}=\\overrightarrow{P A}+\\overrightarrow{A Q}=-\\frac{1}{2} \\overrightarrow{A C}+\\frac{1}{2}(\\overrightarrow{A B}+\\overrightarrow{A D})=\\frac{1}{2}(\\overrightarrow{A B}+\\overrightarrow{A D}- \\overrightarrow{A C}$ ),\n$$\n\\begin{aligned}\n& A B=C D \\Leftrightarrow \\overrightarrow{A B}^2=\\overrightarrow{C D}^2=(\\overrightarrow{A D}-\\overrightarrow{A C})^2=\\overrightarrow{A D}^2+\\overrightarrow{A C}^2-2 \\overrightarrow{A D} \\cdot \\overrightarrow{A C}, \\\\\n& A D=B C \\Leftrightarrow \\overrightarrow{A D}^2=\\overrightarrow{B C}^2=(\\overrightarrow{A C}-\\overrightarrow{A B})^2=\\overrightarrow{A C}^2+\\overrightarrow{A B}^2-2 \\overrightarrow{A C} \\cdot \\overrightarrow{A B},\n\\end{aligned}\n$$\n即\n$$\n\\overrightarrow{A D} \\cdot \\overrightarrow{A C}+\\overrightarrow{A B} \\cdot \\overrightarrow{A C}=\\overrightarrow{A C}^2\n$$\n所以\n$$\n\\begin{aligned}\n\\overrightarrow{P Q} \\cdot \\overrightarrow{A C} & =\\frac{1}{2}(\\overrightarrow{A B}+\\overrightarrow{A D}-\\overrightarrow{A C}) \\cdot \\overrightarrow{A C} \\\\\n& =\\frac{1}{2}\\left(\\overrightarrow{A B} \\cdot \\overrightarrow{A C}+\\overrightarrow{A D} \\cdot \\overrightarrow{A C}-\\overrightarrow{A C}^2\\right)=0 \\Rightarrow \\overrightarrow{P Q} \\perp \\overrightarrow{A C}\n\\end{aligned}\n$$\n同理可证 $\\overrightarrow{P Q} \\perp \\overrightarrow{B D}$, 证毕.\n$$\n\\begin{aligned}\nP Q \\perp A C & \\Rightarrow \\overrightarrow{P Q} \\cdot \\overrightarrow{A C}=0 \\Rightarrow \\frac{1}{2}(\\overrightarrow{A B}+\\overrightarrow{A D}-\\overrightarrow{A C}) \\cdot \\overrightarrow{A C}=0 \\\\\n& \\Rightarrow \\overrightarrow{A B} \\cdot \\overrightarrow{A C}+\\overrightarrow{A D} \\cdot \\overrightarrow{A C}=\\overrightarrow{A C}^2,\n\\end{aligned}\n$$\n$$\n\\begin{aligned}\nP Q \\perp B D & \\Rightarrow \\overrightarrow{P Q} \\cdot \\overrightarrow{B D}=0 \\Rightarrow \\frac{1}{2}(\\overrightarrow{A B}+\\overrightarrow{A D}-\\overrightarrow{A C}) \\cdot(\\overrightarrow{A D}-\\overrightarrow{A B})=0 \\\\\n& \\Rightarrow \\overrightarrow{A D}^2=\\overrightarrow{A B}^2+\\overrightarrow{A C} \\cdot \\overrightarrow{A D}-\\overrightarrow{A C} \\cdot \\overrightarrow{A B}\n\\end{aligned}\n$$\n$$\n\\begin{aligned}\n\\overrightarrow{C D}^2 & =(\\overrightarrow{A D}-\\overrightarrow{A C})^2=\\overrightarrow{A C}^2+\\overrightarrow{A D}^2-2 \\overrightarrow{A D} \\cdot \\overrightarrow{A C} \\\\\n& =(\\overrightarrow{A B} \\cdot \\overrightarrow{A C}+\\overrightarrow{A D} \\cdot \\overrightarrow{A C})+\\left(\\overrightarrow{A B}^2+\\overrightarrow{A C} \\cdot \\overrightarrow{A D}-\\overrightarrow{A C} \\cdot \\overrightarrow{A B}\\right)-2 \\overrightarrow{A D} \\cdot \\overrightarrow{A C} \\\\\n& =\\overrightarrow{A B}^2,\n\\end{aligned}\n$$\n所以\n$$\nC D=A B \\text {. }\n$$\n同理可证 $A D=B C$, 证毕.", + "remark": "", + "figures": [ + "./images/volume8/figures/fig-c5p4.png" + ] +} \ No newline at end of file diff --git a/processed_dataset/proof/1987.json b/processed_dataset/proof/1987.json new file mode 100644 index 0000000000000000000000000000000000000000..b559997c10d225eed0b0690f9c69e0f5a3c378b3 --- /dev/null +++ b/processed_dataset/proof/1987.json @@ -0,0 +1,8 @@ +{ + "source_file": "./raw_volume-zh/volume8/exercise5.tex", + "problem_type": "proof", + "problem": "问题8. 在平面上给定 $\\triangle A B C$, 对于平面上的一点 $P$, 建立如下的变换 $f: A P$ 的中点为 $Q, B Q$ 的中点为 $R, C R$ 的中点为 $P^{\\prime}, f(P)=P^{\\prime}$. 求证: $f$ 只有一个不动点 (指 $P$ 与 $P^{\\prime}$ 重合的点).", + "solution": "$\\overrightarrow{A Q}=\\frac{1}{2} \\overrightarrow{A P}, \\overrightarrow{A R}=\\frac{1}{2}(\\overrightarrow{A B}+\\overrightarrow{A Q})=\\frac{1}{2} \\overrightarrow{A B}+\\frac{1}{4} \\overrightarrow{A P}$,\n$$\n\\overrightarrow{A P^{\\prime}}=\\frac{1}{2}(\\overrightarrow{A C}+\\overrightarrow{A R})=\\frac{1}{2} \\overrightarrow{A C}+\\frac{1}{4} \\overrightarrow{A B}+-\\frac{1}{8} \\overrightarrow{A P}\n$$\n要使 $P$ 与 $P^{\\prime}$ 重合, 应有 $\\overrightarrow{A P}=\\frac{1}{2} \\overrightarrow{A C}+\\frac{1}{4} \\overrightarrow{A B}+\\frac{1}{8} \\overrightarrow{A P}$, 得 $\\overrightarrow{A P}=\\frac{1}{7}(4 \\overrightarrow{A C}+ 2 \\overrightarrow{A B})$, 对于给定的 $\\triangle A B C$, 满足条件的不动点 $P$ 只有一个, 证毕.", + "remark": "", + "figures": [] +} \ No newline at end of file diff --git a/processed_dataset/proof/1988.json b/processed_dataset/proof/1988.json new file mode 100644 index 0000000000000000000000000000000000000000..41b867d97e4043e58d9b3b075600f803b5aea0f0 --- /dev/null +++ b/processed_dataset/proof/1988.json @@ -0,0 +1,10 @@ +{ + "source_file": "./raw_volume-zh/volume8/exercise5.tex", + "problem_type": "proof", + "problem": "问题9. 如图(), 设 $O$ 是 $\\triangle A B C$ 的外心, $D$ 是 $A B$ 的中点, $E$ 是 $\\triangle A C D$ 的重心, 求证: 如果 $A B=A C$, 那么 $O E \\perp C D$.", + "solution": "$\\overrightarrow{O E}=\\frac{1}{3}(\\overrightarrow{O C}+\\overrightarrow{O A}+\\overrightarrow{O D})=\\frac{1}{3}\\left(\\overrightarrow{O C}+\\frac{3}{2} \\overrightarrow{O A}+\\frac{1}{2} \\overrightarrow{O B}\\right)$,\n$$\n\\overrightarrow{C D}=\\frac{1}{2}(\\overrightarrow{C A}+\\overrightarrow{C B})=\\frac{1}{2}(\\overrightarrow{O A}+\\overrightarrow{O B}-2 \\overrightarrow{O C}) .\n$$\n因为 $A B=A C$, 所以 $\\overrightarrow{A O} \\perp \\overrightarrow{B C}$, 从而\n$$\n\\begin{aligned}\n12 \\overrightarrow{O E} \\cdot \\overrightarrow{C D} & =(2 \\overrightarrow{O C}+3 \\overrightarrow{O A}+\\overrightarrow{O B}) \\cdot(\\overrightarrow{O A}+\\overrightarrow{O B}-2 \\overrightarrow{O C}) \\\\\n& =3 \\overrightarrow{O A}^2+\\overrightarrow{O B}^2-4 \\overrightarrow{O C}^2+4 \\overrightarrow{O A} \\cdot \\overrightarrow{O B}-4 \\overrightarrow{O C} \\cdot \\overrightarrow{O A} \\\\\n& =3 R^2+R^2-4 R^2+4 \\overrightarrow{O A} \\cdot(\\overrightarrow{O B}-\\overrightarrow{O C})=0 .\n\\end{aligned}\n$$\n故 $O E \\perp C D$, 证毕.", + "remark": "", + "figures": [ + "./images/volume8/figures/fig-c5p9.png" + ] +} \ No newline at end of file diff --git a/processed_dataset/proof/1989.json b/processed_dataset/proof/1989.json new file mode 100644 index 0000000000000000000000000000000000000000..e2ef4d96e9133db429eb5e9b97787624fe1ab106 --- /dev/null +++ b/processed_dataset/proof/1989.json @@ -0,0 +1,8 @@ +{ + "source_file": "./raw_volume-zh/volume8/exercise7.tex", + "problem_type": "proof", + "problem": "问题3. 求证: $\\sin 1+\\sin 2+\\cdots+\\sin n \\leqslant \\frac{1}{\\sin \\frac{1}{2}}$.", + "solution": "$\\left|\\sum_{k=1}^n \\sin k\\right| \\leqslant\\left|\\sum_{k=1}^n \\mathrm{e}^{\\mathrm{i} k}\\right|=\\left|\\frac{\\mathrm{e}^{\\mathrm{i}}\\left(1-\\mathrm{e}^{\\mathrm{i} n}\\right)}{1-\\mathrm{e}^{\\mathrm{i}}}\\right|=\\left|\\frac{1-\\mathrm{e}^{\\mathrm{i} n}}{1-\\mathrm{e}^{\\mathrm{i}}}\\right|=\\frac{\\left|\\mathrm{i}-\\mathrm{e}^{\\mathrm{i} n}\\right|}{2 \\sin \\frac{1}{2}} \\leqslant \\frac{1+\\left|\\mathrm{e}^{\\mathrm{in}}\\right|}{2 \\sin \\frac{1}{2}}=\\frac{1}{\\sin \\frac{1}{2}}$, 证毕.", + "remark": "", + "figures": [] +} \ No newline at end of file diff --git a/processed_dataset/proof/1990.json b/processed_dataset/proof/1990.json new file mode 100644 index 0000000000000000000000000000000000000000..442edaf8e747b67b9c835ddacdd27dac499d6512 --- /dev/null +++ b/processed_dataset/proof/1990.json @@ -0,0 +1,8 @@ +{ + "source_file": "./raw_volume-zh/volume8/exercise7.tex", + "problem_type": "proof", + "problem": "问题6. 设 $f(x)$ 是复系数多项式, $n$ 是正整数, 求证: 如果 $(x-1) \\mid f\\left(x^n\\right)$, 则 $\\left(x^n-1\\right) \\mid f\\left(x^n\\right)$.", + "solution": "$f\\left(x^n\\right)=(x-1) g(x)$.\n取 $\\zeta=\\mathrm{e}^{\\frac{2 \\pi j}{n}}$ 是一个 $n$ 次单位根, 由 $f(1)=0$ 知, $f\\left(\\zeta^{k n}\\right)=0(k=1, \\cdots, n)$.\n故 $f\\left(x^n\\right)$ 被 $(x-\\zeta)\\left(x-\\zeta^2\\right) \\cdots\\left(x-\\zeta^n\\right)=x^n-1$ 整除,证毕.", + "remark": "", + "figures": [] +} \ No newline at end of file diff --git a/processed_dataset/proof/1991.json b/processed_dataset/proof/1991.json new file mode 100644 index 0000000000000000000000000000000000000000..d607bdb8c74673685b2ce199544ccf30c382eab3 --- /dev/null +++ b/processed_dataset/proof/1991.json @@ -0,0 +1,8 @@ +{ + "source_file": "./raw_volume-zh/volume8/exercise7.tex", + "problem_type": "proof", + "problem": "问题7. 设 $g(\\theta)=\\lambda_1 \\cos \\theta+\\lambda_2 \\cos 2 \\theta+\\cdots+\\lambda_n \\cos n \\theta$, 其中 $\\lambda_1, \\lambda_2, \\cdots, \\lambda_n, \\theta$ 均为实数.\n若对一切实数 $\\theta$, 恒有 $g(\\theta) \\geqslant-1$. 求证: $\\lambda_1+\\lambda_2+\\cdots+\\lambda_n \\leqslant n$.", + "solution": "令 $\\theta_k=\\frac{2 k \\pi}{n+1}, k=0,1,2, \\cdots, n$, 则有\n$$\n\\sum_{k=0}^n \\cos m \\theta_k=\\sum_{k=0}^n \\sin m \\theta_k=0, m=1,2, \\cdots, n . \\label{eq1}\n$$\n(事实上, $\\sum_{k=0}^n \\mathrm{e}^{\\mathrm{i} n \\theta_k}=\\frac{1-\\mathrm{e}^{\\mathrm{i} m \\cdot 2 \\pi}}{1-\\mathrm{e}^{\\mathrm{i} m \\frac{2 \\pi}{n+1}}}=0$, 于是\\ref{eq1}式成立), 因此\n$$\n\\begin{aligned}\n& g(0)+g\\left(\\theta_1\\right)+g\\left(\\theta_2\\right)+\\cdots+g\\left(\\theta_n\\right) \\\\\n= & \\lambda_1\\left(\\cos 0+\\cos \\theta_1+\\cdots+\\cos \\theta_n\\right)+\\lambda_2\\left(\\cos 0+\\cos 2 \\theta_1+\\cdots+\\cos 2 \\theta_n\\right) \\\\\n& +\\cdots+\\lambda_n\\left(\\cos 0+\\cos n \\theta_1+\\cdots+\\cos n \\theta_n\\right)=0 .\n\\end{aligned}\n$$\n故由 $g\\left(\\theta_1\\right) \\geqslant-1, g\\left(\\theta_2\\right) \\geqslant-1, \\cdots, g\\left(\\theta_n\\right) \\geqslant-1$ 得\n$$\n\\lambda_1+\\lambda_2+\\cdots+\\lambda_n=g(0)=-\\left[g\\left(\\theta_1\\right)+g\\left(\\theta_2\\right)+\\cdots+g\\left(\\theta_n\\right)\\right] \\leqslant n,\n$$\n证毕.", + "remark": "", + "figures": [] +} \ No newline at end of file diff --git a/processed_dataset/proof/1992.json b/processed_dataset/proof/1992.json new file mode 100644 index 0000000000000000000000000000000000000000..9b236523ec18c74866198c84c17a2f10447875e1 --- /dev/null +++ b/processed_dataset/proof/1992.json @@ -0,0 +1,8 @@ +{ + "source_file": "./raw_volume-zh/volume8/exercise8.tex", + "problem_type": "proof", + "problem": "问题2. 设 $a 、 b 、 c$ 是给定复数, 记 $|a+b|=m,|a-b|=n$, 已知 $m n \\neq 0$, 求证:\n$$\n\\max \\{|a c+b|,|a+b c|\\} \\geqslant-\\frac{m m}{\\sqrt{m^2+n^2}} .\n$$", + "solution": "因为\n$$\n\\begin{aligned}\n\\max \\{|a c+b|,|a+b c|\\} & \\geqslant \\frac{|b||a c+b|+|a||a+b c|}{|b|+|a|} \\\\\n& \\geqslant \\frac{|b(a c+b)-a(a+b c)|}{|a|+|b|} \\\\\n& =\\frac{\\left|b^2-a^2\\right|}{|a|+|b|} \\\\\n& \\geqslant \\frac{|b+a||b-a|}{\\sqrt{2\\left(|a|^2+|b|^2\\right)}},\n\\end{aligned}\n$$\n又\n$$\nm^2+n^2=|a-b|^2+|a+b|^2=2\\left(|a|^2+|b|^2\\right),\n$$\n所以\n$$\n\\max \\{|a c+b|,|a+b c|\\} \\geqslant \\frac{m m}{\\sqrt{m^2+n^2}},\n$$\n证毕.", + "remark": "", + "figures": [] +} \ No newline at end of file diff --git a/processed_dataset/proof/1993.json b/processed_dataset/proof/1993.json new file mode 100644 index 0000000000000000000000000000000000000000..91a46378800dd5836ea72d74bfee644ff9c69820 --- /dev/null +++ b/processed_dataset/proof/1993.json @@ -0,0 +1,8 @@ +{ + "source_file": "./raw_volume-zh/volume8/exercise8.tex", + "problem_type": "proof", + "problem": "问题3. 设 $r \\in \\mathbf{N}^*$, 求证: 二次三项式 $x^2-r x-1$ 不可能是任何一个各项系数的绝对值都小于 $r$ 的非零整系数多项式的因式.", + "solution": "先证明: 若 $\\alpha$ 是多项式 $f(x)=a_n x^n+\\cdots+a_0\\left(a_n \\neq 0\\right)$ 的根, 则 $|\\alpha|< M+1$, 这里 $M=\\max _{0 \\leqslant i \\leqslant n-1}\\left|\\frac{a_i}{a_n}\\right|$.\n事实上, 若 $|\\alpha| \\leqslant 1$, 则上述结论显然成立.\n若 $|\\alpha|>1$, 由 $f(\\alpha)=0$, 可知 $-a_n \\alpha^n=a_0+a_1 \\alpha+\\cdots+a_{n-1} \\alpha^{n-1}$, 于是 $|\\alpha|^n=\\left|\\frac{a_0}{a_n}+\\frac{a_1}{a_n} \\alpha+\\cdots+\\frac{a_{n-1}}{a_n} \\alpha^{n-1}\\right| \\leqslant M\\left(1+|\\alpha|+\\cdots+|\\alpha|^{n-1}\\right)=\\frac{M\\left(|\\alpha|^n-1\\right)}{|\\alpha|-1}<\\frac{M \\cdot|\\alpha|^n}{|\\alpha|-1}$, 故 $|\\alpha|r)$ 是 $g(x)$ 的根, 利用上面的结论, 可知 $r<\\frac{r+\\sqrt{r^2+4}}{2} <1+\\max _{0 \\leqslant i \\leqslant n-1}\\left|\\frac{b_i}{b_n}\\right|$, 从而 $\\max _{0 \\leqslant i \\leqslant n-1}\\left|\\frac{b_i}{b_n}\\right|>r-1, \\max _{0 \\leqslant i \\leqslant n-1}\\left|b_i\\right|>r-1$. 由于 $b_i \\in \\mathbf{Z}$, 所以 $\\max _{0 \\leqslant i \\leqslant n-1}\\left|b_i\\right| \\geqslant r$, 证毕.", + "remark": "", + "figures": [] +} \ No newline at end of file diff --git a/processed_dataset/proof/1994.json b/processed_dataset/proof/1994.json new file mode 100644 index 0000000000000000000000000000000000000000..2b99912c8faf1fa69f9167828ebcc6263779d903 --- /dev/null +++ b/processed_dataset/proof/1994.json @@ -0,0 +1,8 @@ +{ + "source_file": "./raw_volume-zh/volume8/exercise8.tex", + "problem_type": "proof", + "problem": "问题4. 设 $b 、 k \\in \\mathbf{N}^*, 1\\sqrt{k}$.\n求证: $f(x)$ 在整系数范围内不可约.", + "solution": "若 $f(x)=g(x) h(x)$, 这里 $g(x), h(x)$ 为非常数的整系数多项式, 则 $k p=|f(b)|=|g(b)||h(b)|$, 于是质数 $p$ 整除 $|g(b)| 、|h(b)|$ 中的一个.\n不妨设 $p|| h(b) \\mid$, 则 $|g(b)| \\leqslant k$. 设 $g(x)=b_0\\left(x-r_1\\right) \\cdots\\left(x-r_j\\right)$. 因为 $\\left|b-r_i\\right|>\\sqrt{k}, 1 \\leqslant i \\leqslant j$, 所以 $k \\geqslant|g(b)|=\\left|b_0\\right|\\left|b-r_1\\right| \\cdots\\left|b-r_j\\right|> \\sqrt{k^j}$, 从而 $j=1$, 即 $g(x)=b_0 x+b_1, b_0 、 b_1 \\in \\mathbf{Z}$. 不妨设 $b_0>0$ (否则, 用 $-g(x) 、-h(x)$ 代替 $g(x) 、 h(x))$. 若 $b_1<0$, 则 $f(x)$ 有一个正实根, 与条件 (1) 矛盾,故 $b_1 \\geqslant 0$. 但这又导致 $b>k \\geqslant|g(b)|=b_0 b+b_1 \\geqslant b$. 矛盾.\n证毕.", + "remark": "", + "figures": [] +} \ No newline at end of file diff --git a/processed_dataset/proof/1995.json b/processed_dataset/proof/1995.json new file mode 100644 index 0000000000000000000000000000000000000000..269a845d991dc13464c3b2a31a55efe1c8c5ad26 --- /dev/null +++ b/processed_dataset/proof/1995.json @@ -0,0 +1,8 @@ +{ + "source_file": "./raw_volume-zh/volume8/exercise8.tex", + "problem_type": "proof", + "problem": "问题5. 设多项式 $P(x)=x^n+a_1 x^{n-1}+\\cdots+a_{n-1} x+a_n$ 有复根 $x_1 、 x_2 、 \\cdots 、 x_n$, $\\alpha=\\frac{1}{n} \\sum_{k=1}^n x_k, \\beta^2=\\frac{1}{n} \\sum_{k=1}^n\\left|x_k\\right|^2$, 且 $\\beta^2<1+|\\alpha|^2$. 若复数 $x_0$ 满足 $\\left|\\alpha-x_0\\right|^2<1-\\beta^2+|\\alpha|^2$, 求证: $\\left|P\\left(x_0\\right)\\right|<1$.", + "solution": "$\\left|P\\left(x_0\\right)\\right|^2=P\\left(x_0\\right) \\cdot \\overline{P\\left(x_0\\right)}=\\prod_{j=1}^n\\left(x_0-x_j\\right)\\left(\\overline{x_0}-\\overline{x_j}\\right)$\n$$\n=\\prod_{j=1}^n\\left(\\left|x_0\\right|^2-x_0 \\overline{x_j}-\\overline{x_0} x_j+\\left|x_j\\right|^2\\right) . \\label{eq1}\n$$\n由平均不等式有 $\\left[\\prod_{j=1}^n\\left(\\left|x_0\\right|^2-x_0 \\overline{x_j}-\\overline{x_0} x_j+\\left|x_j\\right|^2\\right)\\right]^{\\frac{1}{n}} \\leqslant \\frac{1}{n} \\sum_{j=1}^n\\left(\\left|x_0\\right|^2-\\right. \\left.x_0 \\overline{x_j}-\\overline{x_0} x_j+\\left|x_j\\right|^2\\right)=\\frac{1}{n}\\left[n\\left|x_0\\right|^2-x_0 \\sum_{j=1}^n \\overline{x_j}-\\overline{x_0} \\sum_{j=1}^n x_j+\\sum_{j=1}^n\\left|x_j\\right|^2\\right]= \\left|x_0-\\alpha\\right|^2+\\beta^2-|\\alpha|^2<1$, 故代入 式\\ref{eq1} 即知 $\\left|P\\left(x_0\\right)\\right|<1$, 证毕.", + "remark": "", + "figures": [] +} \ No newline at end of file diff --git a/processed_dataset/proof/1996.json b/processed_dataset/proof/1996.json new file mode 100644 index 0000000000000000000000000000000000000000..7702443420c9e1f84fdc6f41650fcccebc228ba5 --- /dev/null +++ b/processed_dataset/proof/1996.json @@ -0,0 +1,8 @@ +{ + "source_file": "./raw_volume-zh/volume8/exercise8.tex", + "problem_type": "proof", + "problem": "问题6. 设 $n$ 是正整数, $z_1, z_2, \\cdots, z_n, \\omega_1, \\omega_2, \\cdots, \\omega_n$ 为复数, 对任意的 $\\varepsilon_1$, $\\varepsilon_2, \\cdots, \\varepsilon_n \\in\\{-1,1\\}$, 不等式 $\\left|\\varepsilon_1 z_1+\\varepsilon_2 z_2+\\cdots+\\varepsilon_n z_n\\right| \\leqslant \\mid \\varepsilon_1 \\omega_1+ \\varepsilon_2 \\omega_2+\\cdots+\\varepsilon_n \\omega_n \\mid$ 成立.\n证明:\n$$\n\\left|z_1\\right|^2+\\left|z_2\\right|^2+\\cdots+\\left|z_n\\right|^2 \\leqslant\\left|\\omega_1\\right|^2+\\left|\\omega_2\\right|^2+\\cdots+\\left|\\omega_n\\right|^2 . \\label{eq1}\n$$", + "solution": "对于 $\\varepsilon_1, \\varepsilon_2, \\cdots, \\varepsilon_n \\in\\{-1,1\\}$ 的所有选择, 表达式 $\\mid \\varepsilon_1 z_1+\\varepsilon_2 z_2+\\cdots+ \\left.\\varepsilon_n z_n\\right|^2$ 可以加在一起, 有 $2^n$ 个加数.\n首先证明:\n$$\n\\sum_{\\varepsilon_1, \\cdots, \\varepsilon_n \\in\\{-1,1\\}}\\left|\\varepsilon_1 z_1+\\cdots+\\varepsilon_n z_n\\right|^2=2^n\\left(\\left|z_1\\right|^2+\\left|z_2\\right|^2+\\cdots+\\left|z_n\\right|^2\\right) .\n$$\n根据性质: 对于 $u 、 v \\in \\mathbf{C}$, 平行四边形中有等式 $|u+v|^2+|u-v|^2= 2|u|^2+2|v|^2$ 成立, 下面我们用数学归纳法证明式\\ref{eq1}.\n当 $n=1$ 时, \\ref{eq1}式显然成立.\n假设 $n>1$, 且对于 $n-1 \\in \\mathbf{N}^*$ ,\\ref{eq1}式成立.\n下面证明对于 $n$ ,\\ref{eq1}式也成立.\n$$\n\\begin{aligned}\n& \\sum_{\\varepsilon_1, \\cdots, \\varepsilon_n \\in\\{-1,1\\}}\\left|\\varepsilon_1 z_1+\\varepsilon_2 z_2+\\cdots+\\varepsilon_{n-1} z_{n-1}+\\varepsilon_n z_n\\right|^2 \\\\\n= & \\sum_{\\varepsilon_1}, \\cdots, \\varepsilon_{n-1} \\in\\{-1,1\\} \\\\\n= & \\sum_{\\varepsilon_1}, \\cdots, \\varepsilon_{n-1} \\in\\{-1,1\\} \\\\\n= & 2 \\sum_{\\varepsilon_1, \\cdots, \\varepsilon_{n-1} \\in\\{-1,1\\}}\\left(2\\left|\\varepsilon_1+\\cdots+\\varepsilon_{n-1} z_{n-1}+z_n\\right|^2+\\left|\\varepsilon_1 z_1+\\cdots+\\varepsilon_{n-1} z_{n-1}\\right|^2+2\\left|z_n\\right|^2\\right) \\\\\n= & 2\\left[2^{n-1}\\left(\\left|z_1\\right|^2+\\cdots+\\left.z_n\\right|^2\\right)\\right. \\\\\n= & 2^n\\left(\\left|z_1\\right|^2+\\left|z_2\\right|^2+\\cdots+\\left.\\varepsilon_{n-1} z_{n-1}\\right|^2+\\left|z_n\\right|^2\\right)\n\\end{aligned}\n$$\n故(1)式成立.\n对于 $\\varepsilon_1, \\varepsilon_2, \\cdots, \\varepsilon_n \\in\\{-1,1\\}$ 的所有选择,现将下面的不等式 $\\mid \\varepsilon_1 z_1+\\cdots+ \\left.\\varepsilon_n z_n\\right|^2 \\leqslant\\left|\\varepsilon_1 \\omega_1+\\cdots+\\varepsilon_n \\omega_n\\right|^2$ 相加得 $2^n\\left(\\left|z_1\\right|^2+\\cdots+\\left|z_n\\right|^2\\right) \\leqslant 2^n\\left(\\left|\\omega_1\\right|^2+\\cdots\\right. \\left.+\\left|\\omega_n\\right|^2\\right)$, 故 $\\left|z_1\\right|^2+\\cdots+\\left|z_n\\right|^2 \\leqslant\\left|\\omega_1\\right|^2+\\cdots+\\left|\\omega_n\\right|^2$, 证毕.", + "remark": "", + "figures": [] +} \ No newline at end of file diff --git a/processed_dataset/proof/1997.json b/processed_dataset/proof/1997.json new file mode 100644 index 0000000000000000000000000000000000000000..6db43f28bd3d1e3cdc1b9c9ed8172d8399a4a1ca --- /dev/null +++ b/processed_dataset/proof/1997.json @@ -0,0 +1,8 @@ +{ + "source_file": "./raw_volume-zh/volume8/exercise8.tex", + "problem_type": "proof", + "problem": "问题7. 已知复系数多项式 $P(z)=\\sum_{i=0}^n a_i z^i\\left(n \\in \\mathbf{N}^*\\right)$. 求证: 存在一个复数 $z$, 满足 $|z| \\leqslant 1$, 且\n$$\n|P(z)| \\geqslant\\left|a_0\\right|+\\frac{\\left|a_1\\right|}{n} .\n$$", + "solution": "将 $P(z)$ 乘以一个模长为 1 的单位向量, 可使 $a_0 \\geqslant 0$, 再将 $z$ 乘以一个单位向量, 可使 $a_1 \\geqslant 0$. 现在, 如果对任意 $|z| \\leqslant 1$, 均有 $|P(z)|), 由余弦定理,\n$$\n\\begin{aligned}\n\\cos \\angle A O B & =\\frac{t_1 \\overline{t_1}+t_2 \\overline{t_2}-\\left|t_1-t_2\\right|^2}{2\\left|t_1 t_2\\right|} \\\\\n& =\\frac{t_1 \\overline{t_1}+t_2 \\overline{t_2}-\\left(t_1-t_2\\right)\\left(\\overline{t_1}-\\overline{t_2}\\right)}{2\\left|t_1 t_2\\right|},\n\\end{aligned}\n$$\n化简即得求证式.\n至于面积, 不妨将 $A$ 顺时针转到横轴正方向, 即 $A \\rightarrow A^{\\prime}=t_1 \\mathrm{e}^{\\mathrm{i} \\theta}=$ 正实数 $=\\left|t_1\\right|$, 于是 $B \\rightarrow B^{\\prime}=t_2 \\mathrm{e}^{\\mathrm{i} \\theta}$.\n$$\nS_{\\triangle A O B}=S_{\\triangle A^{\\prime} O B^{\\prime}}=\\frac{1}{2}\\left|t_1\\right| \\cdot \\operatorname{Im}\\left(t_2 \\mathrm{e}^{\\mathrm{i} \\theta}\\right)=\\frac{1}{2}\\left|t_1\\right| \\cdot\n$$\n$\\operatorname{Im}\\left(\\frac{t_2}{t_1}\\left|t_1\\right|\\right)=\\frac{1}{2} \\operatorname{Im}\\left(\\overline{t_1} t_2\\right)=\\frac{\\overline{t_1} t_2-t_1 \\overline{t_2}}{4 \\mathrm{i}}, \\sin \\angle A O B$ 可即刻得出.\n证毕.", + "remark": "", + "figures": [ + "./images/volume8/figures/fig-c9a2.png" + ] +} \ No newline at end of file diff --git a/processed_dataset/proof/2000.json b/processed_dataset/proof/2000.json new file mode 100644 index 0000000000000000000000000000000000000000..345fca56bfcab88718bff92c94faf8b1dc3f7e2d --- /dev/null +++ b/processed_dataset/proof/2000.json @@ -0,0 +1,8 @@ +{ + "source_file": "./raw_volume-zh/volume8/exercise9.tex", + "problem_type": "proof", + "problem": "问题5. 设 $P$ 是锐角三角形 $A B C$ 内一点, $A P 、 B P 、 C P$ 分别交边 $B C 、 C A 、 A B$ 于点 $D 、 E 、 F$, 已知 $\\triangle D E F \\backsim \\triangle A B C$. 求证: $P$ 是 $\\triangle A B C$ 的重心.", + "solution": "本题的结论对 $\\triangle A B C$ 为一般三角形都成立.\n设 $P$ 为复平面上的原点, 并直接用 $X$ 表示点 $X$ 对应的复数, 则存在正实数 $\\alpha 、 \\beta 、 \\gamma$, 使得 $\\alpha A+\\beta B+\\gamma C=0$, 且 $\\alpha+\\beta+\\gamma=1$.\n由于 $D$ 为 $A P$ 与 $B C$ 的交点, 可解得 $D=-\\frac{\\alpha}{1-\\alpha}-A$. 同样地, $E= -\\frac{\\beta}{1-\\beta} B, F=-\\frac{\\gamma}{1-\\gamma} C$. 利用 $\\triangle D E F \\backsim \\triangle A B C$ 可知 $\\frac{D-E}{A-B}=\\frac{E-F}{B-C}$, 于是\n$$\n\\frac{\\gamma B C}{1-\\gamma}+\\frac{\\beta A B}{1-\\beta}+\\frac{\\alpha}{1-\\alpha}-\\frac{\\alpha A}{1-\\alpha}-\\frac{\\beta B C}{1-\\beta}-\\frac{\\gamma C A}{1-\\gamma}=0 .\n$$\n化简得\n$$\n\\left(\\gamma^2-\\beta^2\\right) B(C-A)+\\left(\\alpha^2-\\gamma^2\\right) A(C-B)=0 .\n$$\n这时, 若 $\\gamma^2 \\neq \\beta^2$, 则 $\\frac{B(C-A)}{A(C-B)} \\in \\mathbf{R}$, 因此, $\\frac{\\frac{C-A}{C-B}}{\\frac{P-A}{P-B}} \\in \\mathbf{R}$, 这要求 $P$ 在 $\\triangle A B C$ 的外接圆上,与 $P$ 在 $\\triangle A B C$ 内矛盾, 所以 $\\gamma^2=\\beta^2$,进而 $\\alpha^2=\\gamma^2$, 得 $\\alpha=\\beta=\\gamma=\\frac{1}{3}$.\n即 $P$ 为 $\\triangle A B C$ 的重心, 证毕.", + "remark": "", + "figures": [] +} \ No newline at end of file diff --git a/processed_dataset/proof/2001.json b/processed_dataset/proof/2001.json new file mode 100644 index 0000000000000000000000000000000000000000..9c8500f42d99d6d87b0d274df2ffd928bf6cf3a5 --- /dev/null +++ b/processed_dataset/proof/2001.json @@ -0,0 +1,8 @@ +{ + "source_file": "./raw_volume-zh/volume8/exercise9.tex", + "problem_type": "proof", + "problem": "问题6. 给定一个凸六边形, 其任意两条对边具有如下性质: 它们的中点之间的距离等于它们的长度和的 $\\frac{\\sqrt{3}}{2}$ 倍.\n证明: 该六边形的所有内角相等 (一个凸六边形 $A B C D E F$ 有 3 组对边: $A B$ 和 $D E, B C$ 和 $E F, C D$ 和 $F A$ ).", + "solution": "引理: $\\triangle P Q R$ 中, $\\angle Q P R \\geqslant 60^{\\circ}, L$ 为 $Q R$ 中点.\n则 $P L \\leqslant \\frac{\\sqrt{3}}{2} Q R$, 等号当且仅当 $\\triangle P Q R$ 为正三角形时取到.\n引理的证明: 设 $S$ 为平面上一点,使得 $P$ 与 $S$ 在 $Q R$ 的同侧, 而 $\\triangle Q R S$ 为正三角形.\n则由于 $\\angle Q P R \\geqslant 60^{\\circ}$, 故 $P$ 在 $\\triangle Q R S$ 的外接圆的内部 (包括边界). 而 $\\triangle Q R S$ 的外接圆落在以 $L$ 为圆心, $\\frac{\\sqrt{3}}{2} Q R$ 为半径的圆内.\n所以引理获证.\n设 $A B C D E F$ 为给定的凸六边形, 记 $\\vec{a}=\\overrightarrow{A B}, \\vec{b}=\\overrightarrow{B C}, \\cdots, \\vec{f}=\\overrightarrow{F A}$. 并设 $M 、 N$ 分别为 $A B$ 和 $D E$ 的中点.\n则\n$$\n\\begin{gathered}\n\\overrightarrow{M N}=\\frac{1}{2} \\vec{a}+\\vec{b}+\\vec{c}+\\frac{1}{2} \\vec{d}, \\\\\n\\overrightarrow{M N}=-\\frac{1}{2} \\vec{a}-\\vec{f}-\\vec{e}-\\frac{1}{2} \\vec{d} . \\\\\n\\overrightarrow{M N}=\\frac{1}{2}(\\vec{b}+\\vec{c}-\\vec{e}-\\vec{f}) .\n\\end{gathered}\n$$\n于是\n$$\n\\overrightarrow{M N}=\\frac{1}{2}(\\vec{b}+\\vec{c}-\\vec{e}-\\vec{f}) . \\label{eq1}\n$$\n由条件, 我们有\n$$\n\\overrightarrow{M N}=\\frac{\\sqrt{3}}{2}(|\\vec{a}|+|\\vec{d}|) \\geqslant \\frac{\\sqrt{3}}{2}|\\vec{a}-\\vec{d}| . \\label{eq2}\n$$\n记 $\\vec{x}=\\vec{a}-\\vec{d}, \\vec{y}=\\vec{c}-\\vec{f}, \\vec{z}=\\vec{e}-\\vec{b}$, 由(1)与(2)可得\n$$\n|\\vec{y}-\\vec{z}| \\geqslant \\sqrt{3}|\\vec{x}| . \\label{eq3}\n$$\n同理可知\n$$\n\\begin{aligned}\n& |\\vec{z}-\\vec{x}| \\geqslant \\sqrt{3}|\\vec{y}|, \\label{eq4}\\\\\n& |\\vec{x}-\\vec{y}| \\geqslant \\sqrt{3}|\\vec{z}| . \\label{eq5}\n\\end{aligned}\n$$\n注意到\n式\\ref{eq3} $\\Leftrightarrow|\\vec{y}|^2-2 \\vec{y} \\cdot \\vec{z}+|\\vec{z}|^2 \\geqslant 3|\\vec{x}|^2$;\n式\\ref{eq4} $\\Leftrightarrow|\\vec{z}|^2-2 \\vec{z} \\cdot \\vec{x}+|\\vec{x}|^2 \\geqslant 3|\\vec{y}|^2$;\n式\\ref{eq5} $\\Leftrightarrow|\\vec{x}|^2-2 \\vec{x} \\cdot \\vec{y}+|\\vec{y}|^2 \\geqslant 3|\\vec{z}|^2$.\n上述 3 式相加, 得\n$$\n-|\\vec{x}|^2-|\\vec{y}|^2-|\\vec{z}|^2-2 \\vec{y} \\cdot \\vec{z}-2 \\vec{z} \\cdot \\vec{x}-2 \\vec{x} \\cdot \\vec{y} \\geqslant 0 .\n$$\n即 $-|\\vec{x}+\\vec{y}+\\vec{z}| \\geqslant 0$. 因此 $\\vec{x}+-\\vec{y}+\\vec{z}=0$, 并且上述所有不等式全部取等号.\n于是\n$$\n\\begin{gathered}\n\\vec{x}+\\vec{y}+\\vec{z}=0, \\\\\n|\\vec{y}-\\vec{z}|=\\sqrt{3}|\\vec{x}|, \\vec{a} / / \\vec{d} / / \\vec{x}, \\\\\n|\\vec{z}-\\vec{x}|=\\sqrt{3}|\\vec{y}|, \\vec{c} / / \\vec{f} / / \\vec{y}, \\\\\n|\\vec{x}-\\vec{y}|=\\sqrt{3}|\\vec{z}|, \\vec{e} / / \\vec{b} / / \\vec{z} .\n\\end{gathered}\n$$\n现在设 $\\triangle P Q R$ 中, $\\overrightarrow{P Q}=\\vec{x}, \\overrightarrow{Q R}=\\vec{y}, \\overrightarrow{R P}=\\vec{z}$, 并不妨设 $\\angle Q P R \\geqslant 60^{\\circ}$. $L$ 为 $Q R$ 中点, 则 $P L=\\frac{1}{2}|\\vec{z}-\\vec{x}|=\\frac{\\sqrt{3}}{2}|\\vec{y}|=\\frac{\\sqrt{3}}{2} Q R$. 利用引理可知, $\\triangle P Q R$ 为正三角形.\n于是,\n$$\n\\angle A B C=\\angle B C D=\\cdots=\\angle F A B=120^{\\circ},\n$$\n证毕.", + "remark": "", + "figures": [] +} \ No newline at end of file diff --git a/processed_dataset/proof/2002.json b/processed_dataset/proof/2002.json new file mode 100644 index 0000000000000000000000000000000000000000..f41ccca1f01aac9d82e9dcd378b22318370c3320 --- /dev/null +++ b/processed_dataset/proof/2002.json @@ -0,0 +1,8 @@ +{ + "source_file": "./raw_volume-zh/volume8/exercise9.tex", + "problem_type": "proof", + "problem": "问题7. 设 $\\triangle A B C$ 内接于单位圆 $O, A 、 B 、 C$ 对应的复数分别为 $t_1 、 t_2 、 t_3$, 而 $P$ 为外接圆上任一点, 对应复数为 $t$, 证明: $P$ 关于 $\\triangle A B C$ 的西摩松线的方程为\n$$\nt z-s_3 \\bar{z}=\\frac{1}{2 t}\\left(t^3+t_1 t^2-s_2 t-s_3\\right), \\label{eq1}\n$$\n此处 $s_1=t_1+t_2+t_3, s_2=t_2 t_3+t_3 t_1+t_1 t_2, s_3=t_1 t_2 t_3$.", + "solution": "首先, 经过 $t_2 、 t_3$ 的直线方程为\n$$\n\\left(\\overline{t_2}-\\overline{t_3}\\right) z-\\left(t_2-t_3\\right) \\bar{z}=\\overline{t_2} t_3-t_2 \\overline{t_3} .\n$$\n我们只需证明 $P$ 在这直线上的垂足满足题设之方程, 由于题设方程是对称的, 同理可证另外两个垂足亦在其上.\n化简, 得 $z+t_2 t_3 \\bar{z}=t_2+t_3$, 当 $t_2+t_3 \\neq 0$ 时, 两边除以 $t_2+t_3$ 并转化, 于是 $\\frac{z}{t_2+t_3}+\\frac{\\bar{z}}{\\overline{t_2+t_3}}=1$. 由例 1 知, 过 $t$ 且与之垂直的直线方程是\n$$\n\\frac{z}{t_2+t_3}-\\frac{\\bar{z}}{\\overline{t_2+t_3}}=\\frac{t}{t_2+t_3}-\\frac{\\bar{t}}{\\overline{t_2+t_3}}\n$$\n或\n$$\nz-t_2 t_3 \\bar{z}=t-t_2 t_3 \\bar{t}=t-\\frac{t_2 t_3}{t} . \\label{eq2}\n$$\n注意\\ref{eq2}式也包括 $t_2+t_3=0$ 的情形,此式与 $z+t_2 t_3 \\bar{z}=t_2+t_3$ 联立,解出\n$$\nz=\\frac{1}{2}\\left(t+t_2+t_3-\\frac{t_2 t_3}{t}\\right)\n$$\n$$\n\\bar{z}=\\frac{1}{2}\\left(\\overline{t_2}+\\overline{t_3}+\\bar{t}-\\frac{\\overline{t_2 t_3}}{\\bar{t}}\\right)=\\frac{t_2+t_3+t_2 t_3-t^2}{2 t_2 t_3},\n$$\n于是 $t z-s_3 \\bar{z}=\\frac{1}{2}\\left(t^2+t_2+t_3-t_2 t_3\\right)-\\frac{1}{2 t}\\left(t_1 t_2+t_1 t_3+t_1 t_2 t_3-t_1 t^2\\right)$\n$$\n\\begin{aligned}\n& =\\frac{1}{2 t}\\left(t^3+t^2 t_2+t^2 t_3-t_2 t_3-t_1 t_2-t_1 t_3-t_1 t_2 t_3+t_1 t^2\\right) \\\\\n& =\\frac{1}{2 t}\\left(t^3+s_1 t^2-s_2 t-s_3\\right) .\n\\end{aligned}\n$$\n证毕.", + "remark": "", + "figures": [] +} \ No newline at end of file diff --git a/processed_dataset/proof/2003.json b/processed_dataset/proof/2003.json new file mode 100644 index 0000000000000000000000000000000000000000..7454ca22e5f6718f40191245bb8a1f6fba5c60f2 --- /dev/null +++ b/processed_dataset/proof/2003.json @@ -0,0 +1,8 @@ +{ + "source_file": "./raw_volume-zh/volume8/exercise9.tex", + "problem_type": "proof", + "problem": "问题8. 一个圆周上依次有 $A 、 B 、 C 、 D$ 四点, 则其中任一点关于其余三点为顶点的三角形的西摩松线交于一点.", + "solution": "设圆为单位圆, $A 、 B 、 C 、 D$ 对应的复数分别为 $t_1 、 t_2 、 t_3 、 t_4$. 则 $t_1$ 关于 $\\triangle B C D$ 的西摩松线方程为\n$$\nt_1 z-t_2 t_3 t_4 \\bar{z}=\\frac{1}{2 t_1}\\left[t_1^3+\\left(t_2+t_3+t_4\\right) t_1^2-\\left(t_3 t_4+t_4 t_2+t_2 t_3\\right) t_1-t_2 t_3 t_4\\right],\n$$\n将 $z=\\frac{1}{2}\\left(t_1+t_2+t_3+t_4\\right)$ 及 $\\bar{z}=\\frac{1}{2}\\left(\\overline{t_1}+\\overline{t_2}+\\overline{t_3}+\\overline{t_4}\\right)= \\frac{t_2 t_3 t_4+t_3 t_4 t_1+t_4 t_1 t_2+t_1 t_2 t_3}{2 t_1 t_2 t_3 t_4}$ 代入 (1) 式, 发现确实满足, 由于 $\\frac{1}{2}\\left(t_1+t_2+t_3+\\right. \\left.t_4\\right)$ 是一对称式,故另三条西摩松线也经过此点,证毕.", + "remark": "注:这一结论称为安宁定理, 若不用复数, 也许更费口舌.", + "figures": [] +} \ No newline at end of file diff --git a/processed_dataset/proof/2004.json b/processed_dataset/proof/2004.json new file mode 100644 index 0000000000000000000000000000000000000000..546bb0e573e2dd4e6fdabca1dcf5ecb2032ac42f --- /dev/null +++ b/processed_dataset/proof/2004.json @@ -0,0 +1,8 @@ +{ + "source_file": "./raw_volume-zh/volume9/chapter1.tex", + "problem_type": "proof", + "problem": "例1. 设 $a 、 b 、 c$ 是 $\\triangle A B C$ 的边长,求证:\n$$\n\\frac{a}{b+c}+\\frac{b}{c+a}+\\frac{c}{a+b}<2 .\n$$", + "solution": "证明:由三角不等式 $aP C .\n$$", + "solution": "证明:如图(), 延长 $C P$ 交 $A B$ 于点 $D$, 则 $\\angle A D C$ 和 $\\angle B D C$ 有一个不是锐角, 不妨设 $\\angle A D C$ 不是锐角, 则在 $\\triangle A D C$ 中, 由命题 3 得\n$$\nA C>C D \\text {, }\n$$\n因此\n$$\nA B \\geqslant A C>C D \\geqslant P C, \\label{eq1}\n$$\n又在 $\\triangle P A B$ 中, 由三角不等式\n$$\nP A+P B>A B, \\label{eq2}\n$$\n由式\\ref{eq1}、\\ref{eq2}即得求证的不等式.", + "remark": "注:(1) 若去掉条件\" $A B$ 是最长边\", 则结论不一定成立.\n(2) 当 $P$ 是正三角形 $A B C$ 所在平面上一点,且 $P$ 不在这个正三角形的外接圆上, 则 $P A 、 P B 、 P C$ 中任意两个之和大于第三个, 即它们构成某个三角形的三边.", + "figures": [ + "./images/volume9/figures/fig-c1i1.png" + ] +} \ No newline at end of file diff --git a/processed_dataset/proof/2006.json b/processed_dataset/proof/2006.json new file mode 100644 index 0000000000000000000000000000000000000000..972c7c0f1507d3377f8090fd1f72523e8929e38d --- /dev/null +++ b/processed_dataset/proof/2006.json @@ -0,0 +1,10 @@ +{ + "source_file": "./raw_volume-zh/volume9/chapter1.tex", + "problem_type": "proof", + "problem": "例3. 设一条平面闭折线的周长为 1 , 证明: 可以用一个半径是 $\\frac{1}{4}$ 的圆完全盖住这条折线.", + "solution": "分析:解决问题的关键是确定一个点 (圆心), 使得折线上的每一点到这个点的距离不超过 $\\frac{1}{4}$.\n证明如图(),设 $A$ 为闭折线上任意取定的一点, 在闭折线上取点 $B$, 使折线 $A B$ (不论哪一段) 的长恰为 $\\frac{1}{2}$. 连接 $A B$, 取 $A B$ 的中点 $O$, 则折线上任一点到 $O$ 的距离不超过 $\\frac{1}{4}$.\n事实上,设 $M$ 为折线上任一点,则由命题 4 可得\n$O M<\\frac{1}{2}(A M+M B) \\leqslant \\frac{1}{2}$ 折线 $A M+$ 折线 $\\left.B M\\right)=\\frac{1}{2}$ 折线 $A B=\\frac{1}{4}$.\n现以 $O$ 为圆心, $\\frac{1}{4}$ 为半径作圆, 则这个圆完全盖住了这条闭折线, 证毕.\n上面几个例题的证明方法实际上都体现了一种 \"化直\"的思想, 我们称其为 \"化直法\". 具体地说, 化直法是以命题 1 或它的推论为理论依据, 采用把曲线段化为折线段,再把折线段化为直线段来处理的方法.\n化直法是证明几何不等式,特别是距离不等式最为常用的方法之一.\n下面再看几个例子.\n首先,我们介绍经典的 Pólya 问题.", + "remark": "", + "figures": [ + "./images/volume9/figures/fig-c1i2.png" + ] +} \ No newline at end of file diff --git a/processed_dataset/proof/2007.json b/processed_dataset/proof/2007.json new file mode 100644 index 0000000000000000000000000000000000000000..6b06ada7803116a1d5fc441cc7787f4a53ad94f0 --- /dev/null +++ b/processed_dataset/proof/2007.json @@ -0,0 +1,10 @@ +{ + "source_file": "./raw_volume-zh/volume9/chapter1.tex", + "problem_type": "proof", + "problem": "例4. 求证:两端点在一圆周上且将此圆分成等面积的两部分的所有曲线中, 以此圆的直径具有最短的长度.", + "solution": "证明:设 $\\widehat{A B}$ 是一条满足题设条件的曲线.\n如果 $A 、 B$ 两点正好是某一条直径的两个端点, 那么显然 $\\widehat{A B}$ 的长度不会小于圆的直径.\n如果弦 $A B$ 不是直径, 如图(), 那么令与弦 $A B$ 平行的直径为 $C D$, 曲线 $\\widehat{A B}$ 至少与 $C D$ 交于不同的两点, 设不是圆心的那个交点为 $E$, 则曲线 $\\widehat{A B}$ 的长 $=$ 曲线 $\\widehat{A E}$ 的长 + 曲线 $\\widehat{E B}$ 的长 $\\geqslant A E+E B$. (这样将曲线化为了折线)\n下面再证折线 $(A E+E B)>$ 圆的直径.\n为此, 作 $B$ 关于 $C D$ 的对称点 $B^{\\prime}$, 则易证 $A B^{\\prime}$ 是圆的直径.\n于是\n$$\nA E+E B=A E+E B^{\\prime}>A B^{\\prime}=\\text { 圆的直径.\n}\n$$\n综上便知所证结论成立.\n下面的例题源于我们对垂足三角形极值性质的研究.", + "remark": "", + "figures": [ + "./images/volume9/figures/fig-c1i3.png" + ] +} \ No newline at end of file diff --git a/processed_dataset/proof/2008.json b/processed_dataset/proof/2008.json new file mode 100644 index 0000000000000000000000000000000000000000..6217cdfbd76f8867c4616e586b550e6b8f25e317 --- /dev/null +++ b/processed_dataset/proof/2008.json @@ -0,0 +1,11 @@ +{ + "source_file": "./raw_volume-zh/volume9/chapter1.tex", + "problem_type": "proof", + "problem": "例5. 设 $P$ 是 $\\triangle A B C$ 内一点, $P$ 在三边 $B C 、 C A 、 A B$ 上的射影分别为 $A^{\\prime} 、 B^{\\prime} 、 C^{\\prime}$, 直线 $A P 、 B P 、 C P$ 与三条对边的交点分别为 $A^{\\prime \\prime} 、 B^{\\prime \\prime} 、 C^{\\prime \\prime}$. 已知 $\\triangle A^{\\prime \\prime} B^{\\prime \\prime} C^{\\prime \\prime}$ 的周长 $=1$, 求证:\n折线 $A^{\\prime} B^{\\prime \\prime} C^{\\prime} A^{\\prime \\prime}+$ 折线 $A^{\\prime} C^{\\prime \\prime} B^{\\prime} A^{\\prime \\prime} \\leqslant 2$.", + "solution": "证明:所求证的不等式等价于\n$$\nA^{\\prime} B^{\\prime \\prime}+B^{\\prime \\prime} C^{\\prime}+C^{\\prime} A^{\\prime \\prime}+A^{\\prime} C^{\\prime \\prime}+C^{\\prime \\prime} B^{\\prime}+B^{\\prime} A^{\\prime \\prime} \\leqslant 2 . \\label{eq1}\n$$\n要证式\\ref{eq1}, 只需证明局部不等式\n$$\nA^{\\prime \\prime} B^{\\prime \\prime}+A^{\\prime \\prime} C^{\\prime \\prime} \\geqslant A^{\\prime} B^{\\prime \\prime}+A^{\\prime} C^{\\prime \\prime} . \\label{eq2}\n$$\n事实上, 把式\\ref{eq2}和类似的两个不等式\n$$\n\\begin{aligned}\n& B^{\\prime \\prime} A^{\\prime \\prime}+B^{\\prime \\prime} C^{\\prime \\prime} \\geqslant B^{\\prime} A^{\\prime \\prime}+B^{\\prime} C^{\\prime \\prime}, \\\\\n& C^{\\prime \\prime} A^{\\prime \\prime}+C^{\\prime \\prime} B^{\\prime \\prime} \\geqslant C^{\\prime} A^{\\prime \\prime}+C^{\\prime} B^{\\prime \\prime},\n\\end{aligned}\n$$\n相加便得式\\ref{eq1}.\n下面是式\\ref{eq2}的证明.\n为证式\\ref{eq2}, 我们需要下面的引理.\n引理如图(), 设 $P$ 是 $\\triangle A B C$ 的高 $A D$ 上的一点, 直线 $B P$ 交 $A C$ 于 $E$, 直线 $C P$ 交 $A B$ 于 $F$, 则\n$$\n\\angle F D A=\\angle E D A .\n$$\n证明过 $A$ 作 $B C$ 的平行线, 与直线 $D E$ 、 $D F$ 交于 $M 、 N$, 则\n$$\n\\frac{A F}{B F}=\\frac{A N}{B D}, \\frac{C E}{A E}=\\frac{C D}{A M} .\n$$\n由 Ceva 定理得\n$$\n\\frac{A F}{B F} \\cdot \\frac{B D}{D C} \\cdot \\frac{C E}{E A}=1,\n$$\n即\n$$\nA M=A N \\text {. }\n$$\n又由\n$A D \\perp M N$,\n所以\n$D M=D N$,\n故\n$$\n\\angle E D A=\\angle A D M=\\angle A D N=\\angle F D A .\n$$\n下面回转来证明式\\ref{eq2}:\n(1) 若 $P$ 位于 $\\triangle A B C$ 的高 $A D$ 上,则 $A^{\\prime}=A^{\\prime \\prime}$, (2)显然成立.\n(2) 若 $P$ 不位于 $\\triangle A B C$ 的高 $A D$ 上,如图(), 不妨设 $P 、 B$ 位于 $A D$ 同侧, 连接并延长 $A^{\\prime} P$ 交 $A B$ 于 $M$, 连接 $M C$ 交 $B B^{\\prime \\prime}$ 于 $M^{\\prime}$, 则由引理知\n$$\n\\angle B^{\\prime \\prime} A^{\\prime} P>\\angle M^{\\prime} A^{\\prime} P=\\angle C^{\\prime \\prime} A^{\\prime} P . \\label{eq3}\n$$\n作 $B^{\\prime \\prime}$ 关于 $B C$ 的对称点 $N$, 则\n$$\n\\angle N A^{\\prime} C=\\angle C A^{\\prime} B^{\\prime \\prime},\n$$\n又由式\\ref{eq3}可得\n$$\n\\begin{aligned}\n& \\angle N A^{\\prime} C+\\angle C^{\\prime \\prime} A^{\\prime} C \\\\\n= & \\angle N A^{\\prime} C+\\angle C^{\\prime \\prime} A^{\\prime} P+\\angle P A^{\\prime} C \\\\\n< & \\angle P A^{\\prime} B^{\\prime \\prime}+\\angle P A^{\\prime} N \\\\\n= & \\pi,\n\\end{aligned}\n$$\n所以 $A^{\\prime} 、 A^{\\prime \\prime}$ 在 $C^{\\prime \\prime} N$ 同侧, 即 $A^{\\prime}$ 在 $\\triangle C^{\\prime \\prime} A^{\\prime \\prime} N$ 内, 因此由命题 5 有\n$$\nA^{\\prime \\prime} C^{\\prime \\prime}+A^{\\prime \\prime} N>A^{\\prime} C^{\\prime \\prime}+A^{\\prime} N \\text {. }\n$$\n注意到 $A^{\\prime \\prime} B^{\\prime \\prime}=A^{\\prime \\prime} N, A^{\\prime} B^{\\prime \\prime}=A^{\\prime} N$. 上式即是\n$$\nA^{\\prime \\prime} B^{\\prime \\prime}+A^{\\prime \\prime} C^{\\prime \\prime}>A^{\\prime} B^{\\prime \\prime}+A^{\\prime} C^{\\prime \\prime} \\text {. }\n$$\n式\\ref{eq2}得证.", + "remark": "注:(1) 本例所用的反射对称方法是一种常用的化直手段.\n(2) 利用不等式\\ref{eq2}, 袁俊博士证明了刘健先生提出的一个猜想:\n$$\n\\triangle A^{\\prime} B^{\\prime} C^{\\prime} \\text { 的周长 } \\leqslant \\triangle A^{\\prime \\prime} B^{\\prime \\prime} C^{\\prime \\prime} \\text { 的周长.\n}\n$$\n下面的例题是一个很有难度的问题.", + "figures": [ + "./images/volume9/figures/fig-c1i4.png", + "./images/volume9/figures/fig-c1i5.png" + ] +} \ No newline at end of file diff --git a/processed_dataset/proof/2009.json b/processed_dataset/proof/2009.json new file mode 100644 index 0000000000000000000000000000000000000000..e09be31352f796adb39b9340df959cf6bda659fe --- /dev/null +++ b/processed_dataset/proof/2009.json @@ -0,0 +1,10 @@ +{ + "source_file": "./raw_volume-zh/volume9/chapter1.tex", + "problem_type": "proof", + "problem": "例6. 设 $P$ 为 $\\triangle A B C$ 内一点,证明:\n$$\n\\sqrt{P A}+\\sqrt{P B}+\\sqrt{P C}<\\frac{\\sqrt{5}}{2}(\\sqrt{B C}+\\sqrt{C A}+\\sqrt{A B}) . \\label{eq1}\n$$", + "solution": "证明:下面的引理可由命题 5 直接得到.\n引理设 $P$ 是凸四边形 $A B C D$ 的一个内点,则\n$$\nP B+P C), 作出 $\\triangle A B C$ 三边的中点 $A^{\\prime} 、 B^{\\prime}$ 、 $C^{\\prime}$, 则 $P$ 必位于平行四边形 $A^{\\prime} B^{\\prime} A C^{\\prime} 、 C^{\\prime} B^{\\prime} C A^{\\prime}$ 、 $B^{\\prime} A^{\\prime} B C^{\\prime}$ 某一个之中.\n不妨设 $P$ 位于平行四边形 $A^{\\prime} B^{\\prime} A C^{\\prime}$ 内, 则对凸四边形 $A B A^{\\prime} B^{\\prime}$ 应用引理有\n$$\nP A+P B2\\left(\\frac{a}{2}+\\frac{b+c}{2}+c\\right)>a+b+c=$ 式\\ref{eq7} 的左端.\n综上, 式\\ref{eq1}被证明.", + "remark": "", + "figures": [ + "./images/volume9/figures/fig-c1i6.png" + ] +} \ No newline at end of file diff --git a/processed_dataset/proof/2010.json b/processed_dataset/proof/2010.json new file mode 100644 index 0000000000000000000000000000000000000000..37da6a19799de63724630d8a6cbed1c13db5cb91 --- /dev/null +++ b/processed_dataset/proof/2010.json @@ -0,0 +1,10 @@ +{ + "source_file": "./raw_volume-zh/volume9/chapter1.tex", + "problem_type": "proof", + "problem": "例6. 设 $P$ 为 $\\triangle A B C$ 内一点,证明:\n$$\n\\sqrt{P A}+\\sqrt{P B}+\\sqrt{P C}<\\frac{\\sqrt{5}}{2}(\\sqrt{B C}+\\sqrt{C A}+\\sqrt{A B}) . \\label{eq1}\n$$", + "solution": "当然, 上面的例 6 还可用等高线方法来证明.\n所谓等高线就是在讨论极值问题时引进的特殊平面曲线,如圆、椭圆等.\n这里用的等高线是椭圆.\n另外的证法:\n设 $B C=a, C A=b, A B=c$, 且不妨设 $a \\leqslant b, c$.\n现过 $P$ 点作一个以 $B 、 C$ 为焦点的椭圆, 与 $A B$ 、 $A C$ 分别交于 $E$ 和 $F$,如图(),则\n$$\nP A \\leqslant \\max (E A, F A) \\text {. }\n$$\n不妨设 $E A \\geqslant F A$, 则 $P A \\leqslant E A$.\n又\n$\\sqrt{P B}+\\sqrt{P C} \\leqslant \\sqrt{2(P B+P C)}=\\sqrt{2(E B+E C)}$,\n因此\n$$\n\\begin{aligned}\n& \\sqrt{P A}+\\sqrt{P B}+\\sqrt{P C} \\\\\n< & \\sqrt{E A}+\\sqrt{2(E B+E C)} \\\\\n\\leqslant & {\\left[5 E A+\\frac{5}{2}(E B+E C)\\right]^{\\frac{1}{2}} } \\\\\n= & {\\left[5(E A+E B)+\\frac{5}{2}(E C-E B)\\right]^{\\frac{1}{2}} } \\\\\n< & \\sqrt{5}\\left(a+\\frac{a}{2}\\right)^{\\frac{1}{2}} \\\\\n< & \\frac{\\sqrt{5}}{2}(\\sqrt{a}+\\sqrt{b}+\\sqrt{c}),\n\\end{aligned}\n$$\n得证.", + "remark": "", + "figures": [ + "./images/volume9/figures/fig-c1i7.png" + ] +} \ No newline at end of file diff --git a/processed_dataset/proof/2011.json b/processed_dataset/proof/2011.json new file mode 100644 index 0000000000000000000000000000000000000000..3895943dd3903c598c389ebd90eda2487d06353c --- /dev/null +++ b/processed_dataset/proof/2011.json @@ -0,0 +1,10 @@ +{ + "source_file": "./raw_volume-zh/volume9/chapter10.tex", + "problem_type": "proof", + "problem": "Shum 的最小圆问题.\n这里介绍组合几何的一个计数极值问题, 它是 George F. Shum 在 Amer. Math. Monthly (1978,824,E2746) 上提出的一个公开问题(提出时无解答).\n问题设 $\\tau=\\left\\{A_1, A_2, \\cdots, A_n\\right\\}$ 是平面上 $n$ 个不共线的点的集合.\n若一个中心在 $O$, 半径为 $r$ 的圆过 $\\tau$ 中至少三个点, 且 $A_k O \\leqslant r$ 对所有 $k \\in\\{1,2$, $\\cdots, n\\}$ 都成立,则称这个圆是点集 $\\tau$ 的一个最小圆.\n对固定的 $n$, 问 $\\tau$ 的最小圆最多有多少个?\n这里我们分别介绍三种解法.", + "solution": "解法 1 问题的答案是 $n-2$.\n首先注意到出现在最小圆上的点 $A_i$ 一定在 $\\left\\{A_i\\right\\}$ 的凸包的边界上, 因此我们可假定这 $n$ 个点是一个凸 $n$ 边形 $P_n$ 的 $n$ 个顶点.\n假设 $A_i$ 和 $A_j$ 在最小圆 $C_1$ 上,而 $A_k$ 和 $A_h$ 在最小圆 $C_2$ 上.\n若线段 $A_i A_j$ 和 $A_k A_h$ 相交 (有公共内点), 则 $C_1=C_2$. 为了看出这一点, 只需考虑凸四边形 $Q=A_i A_k A_j A_h$. 因 $Q$ 在 $C_1$ 内,所以\n$$\n\\angle A_k A_j A_h+\\angle A_h A_i A_k \\leqslant 180^{\\circ}, \\label{eq1}\n$$\n又因 $Q$ 在 $C_2$ 内,所以\n$$\n\\angle A_i A_k A_j+\\angle A_j A_h A_i \\leqslant 180^{\\circ}, \\label{eq2}\n$$\n式\\ref{eq1}、\\ref{eq2}相加便得 $Q$ 的四个内角之和小于等于 $360^{\\circ}$, 因此式\\ref{eq1}、\\ref{eq2}必须同时取等号, 因此 $Q$ 是一个圆的内接四边形.\n这样可知决定不同最小圆的三角形的边没有交点 (公共内点). 因为 $P_n$ 最多能剖分成 $n-2$ 个没有公共内点且以原顶点为顶点的三角形, 因此 $P_n$ 的最小圆最多有 $n-2$ 个.\n下面证明 $n-2$ 个最小圆是可以达到的.\n我们归纳构造 $n$ 元点集 $\\left\\{A_k\\right\\}$ 使得它的所有最小圆是过 $A_1 、 A_{k-1} 、 A_k$ 的外接圆 $C_k(k=3, \\cdots, n)$. 首先选择非共线的三点组 $A_1 、 A_2 、 A_3$. 现假设 $A_1, \\cdots, A_k(k \\geqslant 3)$ 已被选择好, 它们的最小圆为 $C_3, \\cdots, C_k$. 现在以弦 $A_1 A_k$ 和圆 $C_k$ 上不包含点 $A_{k-1}$ 的弧所形成的区域 $S_k$ 中取一个内点作为 $A_{k+1}$, 如图(), 则圆 $C_{k+1}$ 包含 $S_k$ 在圆 $C_k$ 中的补集 $S_k^{\\prime}$, 因此包含了所有点 $A_1, \\cdots, A_{k+1}$. 另一方面, 我们有\n$$\nS_3 \\supset S_4 \\supset \\cdots \\supset S_k .\n$$\n因此 $A_k$ 都位于圆 $C_i$ 内 $(i=1,2, \\cdots, k)$. 这样我们就达到了目标.\n实际上不难看出, 没有四个顶点共圆的任何凸 $n$ 边形都存在 $n-2$ 个最小圆.", + "remark": "", + "figures": [ + "./images/volume9/figures/fig-c10i1.png" + ] +} \ No newline at end of file diff --git a/processed_dataset/proof/2012.json b/processed_dataset/proof/2012.json new file mode 100644 index 0000000000000000000000000000000000000000..6b2b10f53a27b8d8c2b1ce4c66e0d13ee792e85f --- /dev/null +++ b/processed_dataset/proof/2012.json @@ -0,0 +1,13 @@ +{ + "source_file": "./raw_volume-zh/volume9/chapter10.tex", + "problem_type": "proof", + "problem": "Shum 的最小圆问题.\n这里介绍组合几何的一个计数极值问题, 它是 George F. Shum 在 Amer. Math. Monthly (1978,824,E2746) 上提出的一个公开问题(提出时无解答).\n问题设 $\\tau=\\left\\{A_1, A_2, \\cdots, A_n\\right\\}$ 是平面上 $n$ 个不共线的点的集合.\n若一个中心在 $O$, 半径为 $r$ 的圆过 $\\tau$ 中至少三个点, 且 $A_k O \\leqslant r$ 对所有 $k \\in\\{1,2$, $\\cdots, n\\}$ 都成立,则称这个圆是点集 $\\tau$ 的一个最小圆.\n对固定的 $n$, 问 $\\tau$ 的最小圆最多有多少个?\n这里我们分别介绍三种解法.", + "solution": "解法 2 $\\tau$ 的最小圆至多为 $n-2$ 个.\n我们先对 $n$ 用归纳法证明: 存在凸 $n$ 边形至少有 $n-2$ 个最小圆.\n当 $n=3$ 时结论显然成立.\n假设结论在 $n=k$ 时成立, 即凸 $k$ 边形 $A_1 A_2 \\cdots A_k$ 有 $k-2$ 个最小圆.\n如图(), 延长 $A_2 A_1$ 与 $A_{k-1} A_k$, 我们在 $A_1 A_2 、 A_k A_{k-1}$ 的内侧和 $A_1 A_k$ 的外侧区域中取一点 $A_{k+1}$, 令 $A_{k+1}$ 到 $A_1 A_k$ 的距离足够小, 使 $A_{k+1}$ 在原凸 $k$ 边形的任一最小圆内, 且使 $\\triangle A_1 A_{k+1} A_k$ 的外接圆内部包含了所有 $A_i(i=2, \\cdots, k-1$, 只需使 $\\angle A_1 A_{k+1} A_k$ 大于所有 $\\angle A_1 A_i A_k$ 的补角即可). 这时凸\n$k+1$ 边形 $A_1 A_2 \\cdots A_k A_{k+1}$ 至少有 $k-1$ 个最小圆.\n下面再证明 $\\tau$ 至多有 $n-2$ 个最小圆.\n首先不妨设 $\\tau$ 构成一个凸 $n$ 边形 $P_n=A_1 A_2 \\cdots A_n$ (否则考虑其凸包集 $\\tau^{\\prime}$, 显然最小圆的点必须为凸包上的点), 并设其无四点共圆.\n下面先证明四条引理.\n引理 $1 P_n$ 的每条边上有且仅有一个最小圆过此边两端点.\n证明如图(), 对边 $A_1 A_2, \\tau$ 中其余点均在 $A_1 A_2$ 同侧.\n若 $\\triangle A_1 A_2 A_k$ 的外接圆为最小圆, 则 $A_i$ 在 $\\odot A_1 A_2 A_k$ 内且与 $A_k$ 在 $A_1 A_2$ 同侧, 所以 $\\angle A_1 A_k A_2$ 为所有 $\\angle A_1 A_i A_2(i=3, \\cdots, n)$ 中的最小者, 因此过 $A_1 A_2$ 只能有一个最小圆,引理 1 得证.\n引理 2 对 $P_n$ 的每一条对角线, 或者有两个最小圆或无最小圆过其两端点.\n证明如图(), 对于对角线 $A_1 A_k$, 在其一侧所有以 $P_n$ 的顶点为顶点的角 中, 设 $\\angle A_1 A_i A_k$ 最小, 在另一侧 $\\angle A_1 A_j A_k$ 最小.\n若 $\\angle A_1 A_i A_k+ \\angle A_1 A_j A_k<\\pi$, 则无覆盖圆过 $A_1 A_k$; 若 $\\angle A_1 A_i A_k+\\angle A_1 A_j A_k>\\pi$, 则 $\\triangle A_1 A_i A_k$ 和 $\\triangle A_1 A_j A_k$ 的外接圆均为最小圆,引理 2 得证.\n$\\boldsymbol{A}_j$\n引理 3 若有最小圆过对角线 $A_1 A_k$ 的两端, 则称 $A_1 A_k$ 为 \"好对角线\". $P_n$ 的\"好对角线\"不在非端点处相交.\n证明如图(), 假设 \"好对角线\" $A_i A_j 、 A_r A_s$ 相交于非端点处.\n因 $A_r 、 A_s$ 均在过 $A_i A_j$ 的最小圆内, 注意到这时 $A_i 、 A_r 、 A_s 、 A_j$ 不共圆, 因此\n$$\n\\angle A_i A_r A_j+\\angle A_i A_s A_j>\\pi,\n$$\n同理\n$$\n\\angle A_r A_i A_s+\\angle A_r A_j A_s>\\pi,\n$$\n相加即得凸四边形的内角和大于 $2 \\pi$,矛盾.\n引理 $4 P_n$ 的\"好对角线\"至多有 $n-3$ 条.\n证明设有 $k$ 条\"好对角线\", 则将 $P_n$ 分成 $k+1$ 个凸多边形.\n因 \"好对角线\"不相交于 $P_n$ 内部, 故这 $k+1$ 部分的内角和等于原 $n$ 边形的内角和, 而每部分内角之和大于等于 $\\pi$, 但 $P_n$ 的内角和为 $(n-2) \\pi$, 故有\n$$\n(k+1) \\pi \\leqslant(n-2) \\pi,\n$$\n因此得 $k \\leqslant n-3$.\n下再证 $P_n$ 的最小圆的个数不超过 $n-2$.\n因每个最小圆恰有三条弦以凸 $n$ 边形 $P_n$ 的顶点为端点, 且每条弦或为 $P_n$ 的边或为 \"好对角线\", 故最小圆个数 $\\leqslant \\frac{n+2(n-3)}{3}=n-2$.\n最后若 $P_n$ 中有四点共圆, 则有两条 \"好对角线\"交于内部.\n现抹去其中一条\"好对角线\",剩下的\"好对角线\"的条数小于 $n-2$, 这时同样可推得最小圆的个数小于等于 $n-2$.", + "remark": "", + "figures": [ + "./images/volume9/figures/fig-c10i2.png", + "./images/volume9/figures/fig-c10i3.png", + "./images/volume9/figures/fig-c10i4.png", + "./images/volume9/figures/fig-c10i5.png" + ] +} \ No newline at end of file diff --git a/processed_dataset/proof/2013.json b/processed_dataset/proof/2013.json new file mode 100644 index 0000000000000000000000000000000000000000..038f050ae44ece9fb256506344c2490944bcc7c0 --- /dev/null +++ b/processed_dataset/proof/2013.json @@ -0,0 +1,11 @@ +{ + "source_file": "./raw_volume-zh/volume9/chapter10.tex", + "problem_type": "proof", + "problem": "Shum 的最小圆问题.\n这里介绍组合几何的一个计数极值问题, 它是 George F. Shum 在 Amer. Math. Monthly (1978,824,E2746) 上提出的一个公开问题(提出时无解答).\n问题设 $\\tau=\\left\\{A_1, A_2, \\cdots, A_n\\right\\}$ 是平面上 $n$ 个不共线的点的集合.\n若一个中心在 $O$, 半径为 $r$ 的圆过 $\\tau$ 中至少三个点, 且 $A_k O \\leqslant r$ 对所有 $k \\in\\{1,2$, $\\cdots, n\\}$ 都成立,则称这个圆是点集 $\\tau$ 的一个最小圆.\n对固定的 $n$, 问 $\\tau$ 的最小圆最多有多少个?\n这里我们分别介绍三种解法.", + "solution": "解法 3 $\\tau$ 的最小圆最多有 $n-2$ 个.\n下面用归纳法来证明.\n(i) $n=3$ 时, $\\tau^{\\prime}=\\left\\{A_1, A_2, A_3\\right\\}$ 的最小圆恰有一个.\n(ii)假设 $n=k$ 时结论成立.\n设 $\\tau^{\\prime \\prime}=\\left\\{A_1, A_2, \\cdots, A_{k+1}\\right\\}$, 在 $\\tau^{\\prime \\prime}$ 的所有最小圆中, 不妨设 $\\odot O$ 是半径最大的一个, 且设 $\\tau$ \" 中的三个点 $A_1 、 A_2 、 A_3$ 在 $\\odot O$ 上, $\\tau^{\\prime \\prime}$ 中的其余点都在 $\\odot O$ 内或 $\\odot O$ 上.\n不妨设 $\\triangle A_1 A_2 A_3$ 中 $\\angle A_1$ 最大, 则 $\\angle A_2 、 \\angle A_3$ 均为锐角.\n下面证明: 除 $\\odot O$ 外, 不存在过 $A_1$ 的其他最小圆.\n否则若 $\\odot K$ 过 $A_1$, 因最小圆覆盖住了所有的点, 所以 $A_2 、 A_3$ 在 $\\odot K$ 上或 $\\odot K$ 内, 且至少有一个在 $\\odot K$ 内, 不妨设 $A_2$ 在 $\\odot K$ 内.\n延长 $A_2 A_3$ 交 $\\odot K$ 于 $B 、 C$, 如图(), 则\n$$\n\\angle B A_2 A_1>90^{\\circ} .\n$$\n因此 $A_1 B>A_1 A_2$ 且有 $\\angle A_1 C A_3 \\leqslant \\angle A_1 A_3 A_2$, 所以有\n$$\n\\odot K \\text { 的半径 }=\\frac{A_1 B}{2 \\sin \\angle A_1 C A_2}>\\frac{A_1 A_2}{2 \\sin \\angle A_1 A_3 A_2}=\\odot O \\text { 的半径, }\n$$\n矛盾! 因此过 $A_1$ 的仅有一个最小圆.\n因此 $\\tau$ \"中不过 $A_1$ 的最小圆均为 $\\left\\{A_2, \\cdots, A_{k+1}\\right\\}$ 的最小圆, 由归纳假设这样的圆的个数至多为 $k-2$ 个,故 $\\tau$ \"的最小圆至多为 $k-1$ 个.\n这就用归纳法证明了 $\\tau$ 的最小圆个数小于等于 $n-2$ 个.\n再证对任意的 $n \\geqslant 3, n \\in \\mathbf{N}$, 存在点集 $\\tau=\\left\\{A_1, A_2, \\cdots, A_n\\right\\}$, 其中 $A_1 A_2 \\cdots A_n$ 为凸 $n$ 边形, 使 $\\tau$ 的最小圆个数恰为 $n-2$ 个,下面也用归纳法来证明这一点.\n(i) 当 $n=3$ 时,取一个三角形的三个顶点便可.\n(ii)假设 $n=k$ 时,存在凸 $k$ 边形 $A_1 A_2 \\cdots A_k$, 其最小圆恰有 $k-2$ 个.\n当 $n=k+1$ 时, 如图(), 设 $A_1 A_2 \\cdots A_k$ 为归纳假设中的凸 $k$ 边形, 延长 $A_2 A_1$ 、 $A_{k-1} A_k$ 交于 $B$. 设 $\\angle A_1 A_i A_k(i=2,3, \\cdots, k-1)$ 中最小的为 $\\alpha$, 则在 $\\triangle B A_1 A_k$ 中取点 $A_{k+1}$ 使得\n$\\angle A_1 A_{k+1} A_k>180^{\\circ}-\\alpha$, 这时 $A_1 A_2 \\cdots A_k$ 的 $k-2$ 个最小圆仍为 $A_1 A_2 \\cdots A_{k+1}$ 的最小圆, 且 $\\triangle A_{k+1} A_1 A_k$ 的外接圆为新的最小圆, 故 $A_1 A_2 \\cdots A_{k+1}$ 中有 $k-1$ 个最小圆.\n综上, 问题全部解答完成.", + "remark": "注:上面的三个解法各有特色,都是好的解答, 龙云同学提供的解法更是集中了前两种解法的优点, 清晰简明, 给人以启迪.", + "figures": [ + "./images/volume9/figures/fig-c10i6.png", + "./images/volume9/figures/fig-c10i7.png" + ] +} \ No newline at end of file diff --git a/processed_dataset/proof/2014.json b/processed_dataset/proof/2014.json new file mode 100644 index 0000000000000000000000000000000000000000..bd18500798590145ff2c01fb70b34a4c70facd30 --- /dev/null +++ b/processed_dataset/proof/2014.json @@ -0,0 +1,10 @@ +{ + "source_file": "./raw_volume-zh/volume9/chapter11.tex", + "problem_type": "proof", + "problem": "例1. 设 $d$ 是一个四面体的三组相对棱距离的最小值, $h$ 是四面体的最小高, 求证:\n$$\n2 d>h\n$$", + "solution": "分析:本例是一个典型的立体几何问题, 解决的关键是寻找或者确定一个数量关系比较集中的平面, 从而将问题化归为平面问题来处理.\n证明如图(). 不妨设 $A H=h, A C$ 与 $B D$ 的距离为 $d$. 现作 $A F 、 C N$ 分别垂直 $B D$ 于 $F 、 N$, 显然\n$$\nH F / / C N \\text {, }\n$$\n于是可在平面 $B C D$ 内作矩形 $F E C N$.\n现考虑 $\\triangle A E F, A H$ 为其边 $E F$ 上的高, 边 $A E$ 上的高 $F G=d$, 高 $E M$ 为 $C$ 到面 $A B D$ 的距离, 因此 $E M \\geqslant A H$.\n这样一来, 题中的数量关系都集中到了平面 $A E F$ 内, 问题就转化为一个平面几何问题.\n即在 $\\triangle A E F$ 中求证 $\\frac{A H}{F G}<2$.\n这是不难的, 事实上由 $A H \\leqslant E M$ 可知\n$$\nA F \\leqslant E F .\n$$\n再注意到 $\\triangle A E H \\backsim \\triangle F E G$, 便有\n$$\n\\frac{h}{d}=\\frac{A H}{F G}=\\frac{A E}{E F}<\\frac{(A F+E F)}{E F} \\leqslant 2 .\n$$", + "remark": "注:本例的结果实际上给出了四面体宽度的最好的下界估计(常数 2 是最佳的), 这个结果被袁淑峰和笔者推广到了一般的 $n$ 维单形.\n般 $n$ 维单形宽度的严格上界估计已由杨路、张景中两位先生得到, 对于四面体, 这个结果就是\n$$\nd \\leqslant \\frac{9 \\sqrt{6}}{2} \\sqrt{\\sum_{i=1}^4 \\frac{1}{h_i^2}},\n$$\n等号成立当且仅当这个四面体为正四面体, 由此还可推出 \"一切维数相同体积相等的四面体中, 正四面体具有最大的宽度\". 有兴趣的读者可参看论文 \"杨路, 张景中.\n度量方程用于 Sallee 猜想.", + "figures": [ + "./images/volume9/figures/fig-c11i1.png" + ] +} \ No newline at end of file diff --git a/processed_dataset/proof/2015.json b/processed_dataset/proof/2015.json new file mode 100644 index 0000000000000000000000000000000000000000..97f97efb161c21ae4904c479ae954fc44570c1eb --- /dev/null +++ b/processed_dataset/proof/2015.json @@ -0,0 +1,10 @@ +{ + "source_file": "./raw_volume-zh/volume9/chapter11.tex", + "problem_type": "proof", + "problem": "例2. 设四面体 $A B C D$ 的内切球半径为 $r$,一组对棱 $A B=a, C D=b$, 求证:\n$$\nr<\\frac{a b}{2(a+b)}\n$$", + "solution": "证明:设四面体的体积为 $V$, 表面积为 $S$, 则熟知\n$$\nr=\\frac{3 V}{S}, \\label{eq1}\n$$\n又由熟知的 Steiner 定理\n$$\nV=\\frac{1}{6} a b d \\sin \\theta, \\label{eq2}\n$$\n其中 $d$ 为对棱 $A B$ 与 $C D$ 之间的距离, $\\theta$ 为它们所成的角.\n由式\\ref{eq1}、\\ref{eq2}有\n$$\nr \\leqslant \\frac{1}{2} \\frac{a b d}{S} . \\label{eq3}\n$$\n另一方面,如图(), 由四面体的棱 $A B$ 之端点到棱 $C D$ 的距离均不小于 $d$, 且其中必有一个大于 $d$, 这样一来\n$$\nS_{\\triangle A D C}+S_{\\triangle B D C}>b d\n$$\n同理\n$$\nS_{\\triangle D A B}+S_{\\triangle C A B}>a d\n$$\n相加即得\n$$\nS>(a+b) d\n$$\n由式\\ref{eq3}、\\ref{eq4}可得\n$$\nr<\\frac{a b d}{2(a+b) d}=\\frac{a b}{2(a+b)} .\n$$", + "remark": "", + "figures": [ + "./images/volume9/figures/fig-c11i2.png" + ] +} \ No newline at end of file diff --git a/processed_dataset/proof/2016.json b/processed_dataset/proof/2016.json new file mode 100644 index 0000000000000000000000000000000000000000..6aa985a2fbc518cfcb830db18805827a42460cc2 --- /dev/null +++ b/processed_dataset/proof/2016.json @@ -0,0 +1,10 @@ +{ + "source_file": "./raw_volume-zh/volume9/chapter11.tex", + "problem_type": "proof", + "problem": "例3. 设 $r$ 是四面体 $A_1 A_2 A_3 A_4$ 的内切球半径, $r_1 、 r_2 、 r_3 、 r_4$ 分别是四个面 $\\triangle A_2 A_3 A_4 、 \\triangle A_1 A_3 A_4 、 \\triangle A_1 A_2 A_4 、 \\triangle A_1 A_2 A_3$ 的内切圆半径, 求证\n$$\n\\frac{1}{r_1^2}+\\frac{1}{r_2^2}+\\frac{1}{r_3^2}+\\frac{1}{r_4^2} \\leqslant \\frac{2}{r^2}\n$$\n等号成立当且仅当 $A_1 A_2 A_3 A_4$ 是正四面体.", + "solution": "证明:先证一个简单的引理.\n引理设四面体 $A_1 A_2 A_3 A_4$ 的体积为 $V$, 记 $S_1$ 为 $\\triangle A_2 A_3 A_4$ 的面积, 等等.\n记 $\\widetilde{a_{12}}$ 表示棱 $A_3 A_4$ 的长, 等等.\n则对任意的 $1 \\leqslant i), 作 $A_1 H \\perp$ 面 $A_2 A_3 A_4$, 垂足为 $H$. 作 $H D \\perp A_3 A_4$, 则\n$$\n\\begin{aligned}\n3 V & =A_1 H \\cdot S_1 \\\\\n& =S_1 \\cdot A_1 D \\cdot \\sin \\theta_{12} \\\\\n& =S_1 \\frac{A_1 D \\cdot A_3 A_4}{\\widetilde{a_{12}}} \\sin \\theta_{12} \\\\\n& =\\frac{2 S_1 S_2 \\sin \\theta_{12}}{\\widetilde{a_{12}}} .\n\\end{aligned}\n$$\n引理得证.\n下证原题.\n设 $A_1 A_2 A_3 A_4$ 的表面积为 $S$, 由引理和公式 $3 V=r S$ 有\n$$\n\\widetilde{a_{i j}}=\\frac{2}{r S} S_i S_j \\sin \\theta_{i j},\n$$\n于是\n$$\n\\sum_{j \\neq i} \\widetilde{a_{i j}}=\\left(\\frac{2}{r S}\\right) S_i \\sum_{j \\neq i} S_j \\sin \\theta_{i j} . \\label{eq1}\n$$\n又由四面体的射影公式\n$$\nS_i=\\sum_{j \\neq i} S_j \\cos \\theta_{i j},\n$$\n并应用 Cauchy 公式有\n$$\n\\begin{aligned}\n\\sum_{j \\neq i} S_j \\sin \\theta_{i j} & =\\sum_{j \\neq i} \\sqrt{\\left(S_j+S_j \\cos \\theta_{i j}\\right)\\left(S_j-S_j \\cos \\theta_{i j}\\right)} \\\\\n& \\leqslant\\left(\\sum_{j \\neq i} S_j+S_j \\cos \\theta_{i j}\\right)^{\\frac{1}{2}}\\left(\\sum_{j \\neq i} S_j-S_j \\cos \\theta_{i j}\\right)^{\\frac{1}{2}} \\\\\n& =S^{\\frac{1}{2}}\\left(S-2 S_i\\right)^{\\frac{1}{2}}\n\\end{aligned} \\label{eq2}\n$$\n注意到\n$$\n\\frac{2 S_i}{\\sum_{j \\neq i} \\widetilde{a_{i j}}}=r_i, \\label{eq3}\n$$\n由式\\ref{eq1}、\\ref{eq2}、式\\ref{eq3}可得\n$$\n\\frac{1}{r_i} \\leqslant\\left(\\frac{1}{r S}\\right) \\cdot S^{\\frac{1}{2}}\\left(S-2 S_i\\right)^{\\frac{1}{2}}\n$$\n由此得\n$$\n\\frac{r}{r_i} \\leqslant\\left(\\frac{S-2 S_i}{S}\\right)^{\\frac{1}{2}}\n$$\n故有\n$$\n\\sum_{i=1} \\frac{r^2}{r_i^2} \\leqslant \\sum_{i=1}^4\\left(\\frac{S-2 S_i}{S}\\right)=2\n$$\n此即\n$$\n\\frac{1}{r_1^2}+\\frac{1}{r_2^2}+\\frac{1}{r_3^2}+\\frac{1}{r_4^2} \\leqslant \\frac{2}{r^2} .\n$$", + "remark": "注:问题产生的背景: 关于三角形的面积有著名的 Pölya 不等式\n$$\nS \\leqslant \\frac{\\sqrt{3}}{4}(a b c)^{\\frac{2}{3}} .\n$$\n在上世纪五十年代末到六十年代初, 几位作者同时独立的将 Pölya 不等式推广到了 $n$ 维单形, 现称为单形的体积优化定理.\n特别地, 对于四面体有\n$$\nV \\leqslant \\frac{2^{\\frac{3}{2}}}{3^{\\frac{7}{4}}}\\left(\\prod_{k=1}^4 S_k\\right)^{\\frac{3}{8}} .\n$$\n一个自然的平行问题是: 四面体的内切球半径和各个面的内切圆的半径是否有类似的优化不等式?\n为了回答这个问题, 笔者在 1992 年和加拿大数学家 Klamkin 的一次私人通信中介绍了上例中的不等式及推论 $r_1 r_2 r_3 r_4 \\geqslant 4 r^4$. 随后这个结果被 Klamkin 推荐发表在 Crux. Math. (Problem. 1990,1994) 上.\n这个结果不久便被唐立华和笔者推广到了 $n$ 维单形, 发表在 Geom. Dedicata, 1996:61 上.\n上面提供的证明和 Crux. Math. 上发表的证明有些不同,完全适用于 $n$ 维空间.\n值得指出,关于单形体积优化定理一类更深刻的推广被张景中和杨路两位先生得到,这已成为距离几何和几何不等式研究中被广泛引用的经典结果.", + "figures": [ + "./images/volume9/figures/fig-c11i3.png" + ] +} \ No newline at end of file diff --git a/processed_dataset/proof/2017.json b/processed_dataset/proof/2017.json new file mode 100644 index 0000000000000000000000000000000000000000..c91b8b4a44a773980c3fe570d7d9d8da68570690 --- /dev/null +++ b/processed_dataset/proof/2017.json @@ -0,0 +1,8 @@ +{ + "source_file": "./raw_volume-zh/volume9/chapter11.tex", + "problem_type": "proof", + "problem": "例4. 设 $R$ 是一个含外心的四面体的外接球半径, $R_1 、 R_2 、 R_3 、 R_4$ 分别是 $\\triangle A_2 A_3 A_4 、 \\triangle A_1 A_3 A_4 、 \\triangle A_1 A_2 A_4 、 \\triangle A_1 A_2 A_3$ 的外接圆半径, 求证\n$$\n1 \\leqslant \\frac{R}{\\max \\left(R_1, R_2, R_3, R_4\\right)} \\leqslant \\frac{3 \\sqrt{2}}{4} . \\label{eq1}\n$$", + "solution": "证明:记 $\\Sigma=A_1 A_2 A_3 A_4$ 并设 $O$ 是四面体 $\\Sigma$ 的外心且 $O$ 在体内.\n显见 $O$ 在某个面上的射影 $O_i$ 一定是面三角形的外心.\n由 $A_i O_i \\leqslant A_i O, i=1,2,3,4$ 便得 $R_i \\leqslant R$,且等号可以成立, 式\\ref{eq1} 左边的不等式得证.\n现以 $O$ 为中心作一个包含在 $\\Sigma$ 内的最大球, 设这个最大球的半径为 $d$, 这个球至少与 $\\Sigma$ 的某一个面相切, 不妨设与面 $A_2 A_3 A_4$ 相切, 则切点一定是 $A_2 A_3 A_4$ 的外心, 因此\n$$\nR_1^2=R^2-d^2 . \\label{eq2}\n$$\n因包含于 $\\Sigma$ 中的球以内切球半径为最大, 由著名不等式 $r \\leqslant \\frac{R}{3}$ 可得\n$$\nd \\leqslant \\frac{R}{3}. \\label{eq3}\n$$\n由式\\ref{eq1}、\\ref{eq2}、式\\ref{eq3}便得\n$$\nR^2-R_1^2 \\leqslant \\frac{R^2}{9}\n$$\n由此得\n$$\nR \\leqslant \\frac{3}{\\sqrt{8}} R_1 \\leqslant \\frac{3}{\\sqrt{8}}-\\max \\left(R_1, R_2, R_3, R_4\\right),\n$$\n右边的不等式得证.", + "remark": "", + "figures": [] +} \ No newline at end of file diff --git a/processed_dataset/proof/2018.json b/processed_dataset/proof/2018.json new file mode 100644 index 0000000000000000000000000000000000000000..e2a446ab8ca4f0b0d6e725d592142c42ea9eabbe --- /dev/null +++ b/processed_dataset/proof/2018.json @@ -0,0 +1,8 @@ +{ + "source_file": "./raw_volume-zh/volume9/chapter11.tex", + "problem_type": "proof", + "problem": "例5. 设 $G$ 为四面体 $A_1 A_2 A_3 A_4$ 的重心, $G$ 到棱 $A_i A_j$ 的距离为 $h_{i j}, G$ 到 $A_i$ 的距离为 $D_i$, 求证:\n$$\n\\sum_{1 \\leqslant i), 作平行四边形 $A B C D$ 和平行四边形 $A C B E$, 连接 $B D 、 C E$. 注意到 $D E=2 a$, $B D=2 m_b, C E=2 m_c$, 对四边形 $B C D E$ 应用 Ptolemy 不等式立得\n$$\nB C \\cdot D E+B E \\cdot C D \\geqslant B D \\cdot E C,\n$$\n这就是式\\ref{eq1}.", + "remark": "", + "figures": [ + "./images/volume9/figures/fig-c2i2.png" + ] +} \ No newline at end of file diff --git a/processed_dataset/proof/2020.json b/processed_dataset/proof/2020.json new file mode 100644 index 0000000000000000000000000000000000000000..c280b8b34f8f233233f9ebfdd50faf5cc21b62c0 --- /dev/null +++ b/processed_dataset/proof/2020.json @@ -0,0 +1,10 @@ +{ + "source_file": "./raw_volume-zh/volume9/chapter2.tex", + "problem_type": "proof", + "problem": "例2. 设 $\\triangle A B C$ 的三边分别为 $a 、 b 、 c$, 三边上的中线分别为 $m_a 、 m_b$ 、 $m_c$, 求证:\n$$\nm_a\\left(b c-a^2\\right)+m_b\\left(a c-b^2\\right)+m_c\\left(a b-c^2\\right) \\geqslant 0 . \\label{eq1}\n$$", + "solution": "下面的证法的关键在于寻找一个特殊的四边形.\n证明如图(), 设 $\\triangle A B C$ 的三条中线分别为 $A D 、 B E 、 C F$, 重心为 $G$.\n现对四边形 $B D G F$ 应用 Ptolemy 不等式可得\n$$\nB G \\cdot D F \\leqslant G F \\cdot D B+D G \\cdot B F . \\label{eq2}\n$$\n注意到 $B G=\\frac{2}{3} m_b, D G=\\frac{1}{3} m_a, G F=\\frac{1}{3} m_c$ 及 $D F=\\frac{1}{2} b$, 因此式\\ref{eq2}可改写为\n$$\n2 b m_b \\leqslant a m_c+c m_a \n$$\n因此\n$$\n2 b^2 m_b \\leqslant a b m_c+c b m_a. \\label{eq3}\n$$\n同理有\n$$\n2 c^2 m_c \\leqslant a c m_b+b c m_a. \\label{eq4}\n$$\n$$\n2 a^2 m_a \\leqslant a b m_c+a c m_b . \\label{eq5}\n$$\n式\\ref{eq3}、\\ref{eq4}、式\\ref{eq5}相加可得\n$$\n2\\left(m_a b c+m_b c a+m_c a b\\right) \\geqslant 2\\left(m_a a^2+m_b b^2+m_c c^2\\right),\n$$\n整理就得式\\ref{eq1}.", + "remark": "", + "figures": [ + "./images/volume9/figures/fig-c2i3.png" + ] +} \ No newline at end of file diff --git a/processed_dataset/proof/2021.json b/processed_dataset/proof/2021.json new file mode 100644 index 0000000000000000000000000000000000000000..7896719c535bd3bb95033531a72e26a9207fff86 --- /dev/null +++ b/processed_dataset/proof/2021.json @@ -0,0 +1,10 @@ +{ + "source_file": "./raw_volume-zh/volume9/chapter2.tex", + "problem_type": "proof", + "problem": "例3. 已知 $A_1 A_2 \\cdots A_n$ 是一个正 $n$ 边形, $M_1, M_2, \\cdots, M_n$ 是相应边的中点.\n设 $P$ 是这个 $n$ 边形所在平面上的任意一点.\n求证:\n$$\n\\sum_{i=1}^n P M_i \\geqslant\\left(\\cos \\frac{\\pi}{n}\\right) \\sum_{i=1}^n P A_i . \\label{eq1}\n$$", + "solution": "证明:如图(), $M_{i-1} 、 M_i$ 分别是这个正 $n$ 边形第 $i-1$ 条边和第 $i$ 条边的中点.\n对四边形 $P M_{i-1} A_i M_i$ 应用 Ptolemy 不等式可得局部不等式\n$$\nA_i M_{i-1} \\cdot P M_i^i+P M_{i-1} \\cdot A_i M_i \\geqslant P A_i \\cdot M_{i-1} M_i,\n$$\n由此可得\n$$\nP M_i+P M_{i-1} \\geqslant 2\\left(\\cos \\frac{\\pi}{n}\\right) \\cdot P A_i, \\label{eq2}\n$$\n这里 $i=1,2, \\cdots, n$, 并约定 $A_0=A_n, M_0=M_n$.\n现对式\\ref{eq2}求和可得\n$$\n\\sum_{i=1}^n\\left(P M_i+P M_{i-1}\\right) \\geqslant 2\\left(\\cos \\frac{\\pi}{n}\\right) \\cdot \\sum_{i=1}^n P A_i,\n$$\n这就是式\\ref{eq1}.", + "remark": "", + "figures": [ + "./images/volume9/figures/fig-c2i4.png" + ] +} \ No newline at end of file diff --git a/processed_dataset/proof/2022.json b/processed_dataset/proof/2022.json new file mode 100644 index 0000000000000000000000000000000000000000..bbc36b98cf9998daa45c1fdfde6f27fee6188273 --- /dev/null +++ b/processed_dataset/proof/2022.json @@ -0,0 +1,10 @@ +{ + "source_file": "./raw_volume-zh/volume9/chapter2.tex", + "problem_type": "proof", + "problem": "例4. 设 $P$ 为平行四边形 $A B C D$ 内一点, 求证:\n$$\nP A \\cdot P C+P B \\cdot P D \\geqslant A B \\cdot B C, \\label{eq1}\n$$\n并指出等号成立的条件.", + "solution": "证明:如图(), 作 $P Q$ 平行并等于 $C D$, 连接 $C Q 、 B Q$, 则 $C D P Q$ 与 $A B Q P$ 均是平行四边形,所以\n$$\nC Q=P D, B Q=P A, P Q=A B .\n$$\n在四边形 $P B Q C$ 中由 Ptolemy 不等式有\n$$\nB Q \\cdot P C+P B \\cdot C Q \\geqslant P Q \\cdot B C,\n$$\n即\n$$\nP A \\cdot P C+P B \\cdot P D \\geqslant A B \\cdot B C,\n$$\n等号成立当且仅当 $P 、 B 、 Q 、 C$ 四点共圆, 即 $\\angle C P B+\\angle C Q B=\\pi$, 而 $\\angle C Q B=\\angle A P D$, 所以式\\ref{eq1}等号成立的条件为\n$$\n\\angle A P D+\\angle C P B=\\pi .\n$$", + "remark": "", + "figures": [ + "./images/volume9/figures/fig-c2i5.png" + ] +} \ No newline at end of file diff --git a/processed_dataset/proof/2023.json b/processed_dataset/proof/2023.json new file mode 100644 index 0000000000000000000000000000000000000000..835ee4417892591588267dcebe8d6978721625db --- /dev/null +++ b/processed_dataset/proof/2023.json @@ -0,0 +1,11 @@ +{ + "source_file": "./raw_volume-zh/volume9/chapter2.tex", + "problem_type": "proof", + "problem": "例5. 在 $\\triangle A B C$ 中, $\\angle A=60^{\\circ}, P$ 为 $\\triangle A B C$ 所在平面上一点,且使得 $P A=6, P B=7, P C=10$, 求 $\\triangle A B C$ 面积的最大值.", + "solution": "解法 1 先证引理.\n引理在凸四边形 $X Y Z U$ 中, 对角线 $X Z$ 和 $Y U$ 交于点 $O, \\angle X O Y= \\theta$, 则\n$$\nY Z^2+U X^2-X Y^2-Z U^2=2 X Z \\cdot Y U \\cdot \\cos \\theta .\n$$\n证明如图(), 在 $\\triangle O Y Z 、 \\triangle O U X 、 \\triangle O X Y$ 及 $\\triangle O Z U$ 中分别应用余弦定理可得\n$$\n\\begin{aligned}\nY Z^2 & =O Y^2+O Z^2+2 O Y \\cdot O Z \\cdot \\cos \\theta, \\\\\nU X^2 & =O U^2+O X^2+2 O U \\cdot O X \\cdot \\cos \\theta, \\\\\nX Y^2 & =O X^2+O Y^2-2 O X \\cdot O Y \\cdot \\cos \\theta, \\\\\nZ U^2 & =O Z^2+O U^2-2 O Z \\cdot O U \\cdot \\cos \\theta,\n\\end{aligned}\n$$\n由这四个等式相加便可立得引理中的等式.\n下面求解原问题.\n如图(), 在 $\\triangle A B C$ 中, 过 $P$ 作 $A B$ 的平行线, 过 $A$ 作 $P B$ 的平行线, 两条直线交于 $D$. 设 $P D$ 交 $A C$ 于 $E$, 则 $\\angle C E P=60^{\\circ}$.\n设 $A C=x, A B=P D=y, C D=t$. 对四边形 $A P C D$ 应用引理可得\n$$\nt^2+6^2-10^2-7^2=2 \\cos 60^{\\circ} \\cdot x y,\n$$\n即\n$$\nx y=t^2-113 . \\label{eq1}\n$$\n另一方面, 对四边形 $A P C D$ 应用 Ptolemy 不等式可得\n$$\nx y \\leqslant 6 t+70 . \\label{eq2}\n$$\n由式\\ref{eq1}、\\ref{eq2}有\n$$\nt^2-6 t-183 \\leqslant 0 \\text {, }\n$$\n所以\n$$\n0 \\leqslant t \\leqslant 3+8 \\sqrt{3} . \\label{eq3}\n$$\n将式\\ref{eq3}代入\\ref{eq2}可得 $x y \\leqslant 88+48 \\sqrt{3}$, 故\n$$\nS_{\\triangle A B C}=\\frac{\\sqrt{3}}{4} x y \\leqslant 36+22 \\sqrt{3},\n$$\n等号成立当且仅当 $D 、 A 、 P 、 C$ 四点共圆, 即 $\\angle P B A=\\angle P C A$. 故 $S_{\\triangle A B C}$ 的最大值为 $36+22 \\sqrt{3}$.", + "remark": "", + "figures": [ + "./images/volume9/figures/fig-c2i6.png", + "./images/volume9/figures/fig-c2i7.png" + ] +} \ No newline at end of file diff --git a/processed_dataset/proof/2024.json b/processed_dataset/proof/2024.json new file mode 100644 index 0000000000000000000000000000000000000000..0898c1365c9042aa94b888b5a9b8b50463da04f6 --- /dev/null +++ b/processed_dataset/proof/2024.json @@ -0,0 +1,11 @@ +{ + "source_file": "./raw_volume-zh/volume9/chapter2.tex", + "problem_type": "proof", + "problem": "例5. 在 $\\triangle A B C$ 中, $\\angle A=60^{\\circ}, P$ 为 $\\triangle A B C$ 所在平面上一点,且使得 $P A=6, P B=7, P C=10$, 求 $\\triangle A B C$ 面积的最大值.", + "solution": "解法 2 先证引理.\n引理设 $P$ 为一个平行四边形 $A B C D$ 所在平面上的一点, 则\n$$\nP A^2+P C^2-P B^2-P D^2=2 \\overrightarrow{A B} \\cdot \\overrightarrow{A D} .\n$$\n证明如图(), 平移 $\\triangle B P C$ 至 $\\triangle A D P^{\\prime}$. 设 $\\overrightarrow{A P}=\\alpha, \\overrightarrow{P D}=\\beta, \\overrightarrow{D P^{\\prime}}=\\gamma$, 则\n$$\n\\begin{aligned}\nP A^2+P C^2 & =P A^2+P^{\\prime} D^2=\\alpha^2+\\gamma^2, \\\\\nP D^2+P B^2 & =P D^2+P^{\\prime} A^2 \\\\\n& =\\beta^2+(\\alpha+\\beta+\\gamma)^2 \\\\\n& =2 \\beta^2+\\alpha^2+\\gamma^2+2 \\alpha \\cdot \\beta+2 \\alpha \\cdot \\gamma+2 \\gamma \\cdot \\beta,\n\\end{aligned}\n$$\n因此\n$$\n\\begin{aligned}\nP A^2+P C^2-P D^2-P B^2 & =-2 \\beta^2-2 \\alpha \\cdot \\beta-2 \\gamma \\cdot \\beta-2 \\gamma \\cdot \\alpha \\\\\n& =-2(\\alpha+\\beta) \\cdot(\\beta+\\gamma) \\\\\n& =2 \\overrightarrow{A D} \\cdot \\overrightarrow{P^{\\prime} P} \\\\\n& =2 \\overrightarrow{A B} \\cdot \\overrightarrow{A D} .\n\\end{aligned}\n$$\n下面求解原问题.\n如图(), 平移 $\\triangle A P B$ 至 $\\triangle C P^{\\prime} D$, 则 $P^{\\prime} C=6$, $P^{\\prime} D=7, C D=A B, P P^{\\prime}=A C$.\n设 $P D=d$, 对四边形 $C P^{\\prime} D P$ 应用 Ptolemy 不等式可得\n$$\n70+6 d \\geqslant A B \\cdot A C . \\label{eq1}\n$$\n再对平行四边形 $A B D C$ 应用引理可得\n$$\n7^2+10^2-6^2-d^2=2 \\overrightarrow{B A} \\cdot \\overrightarrow{B D}=-A B \\cdot A C . \\label{eq2}\n$$\n由式\\ref{eq1}、\\ref{eq2}可得\n$$\nd^2-113 \\leqslant 6 d+70\n$$\n所以\n$$\n0 \\leqslant d \\leqslant 3+8 \\sqrt{3} \\text {. }\n$$\n故由式\\ref{eq1}可得 $A B \\cdot A C \\leqslant 88+48 \\sqrt{3}$, 进而得 $S_{\\triangle A B C} \\leqslant 36+22 \\sqrt{3}$, 等号成立当且仅当 $\\angle A C P=\\angle A P B$, 故 $S_{\\triangle A B C}$ 的最大值为 $36+22 \\sqrt{3}$.", + "remark": "", + "figures": [ + "./images/volume9/figures/fig-c2i8.png", + "./images/volume9/figures/fig-c2i9.png" + ] +} \ No newline at end of file diff --git a/processed_dataset/proof/2025.json b/processed_dataset/proof/2025.json new file mode 100644 index 0000000000000000000000000000000000000000..49a06e8984510d2852828046fc35440aac77e38a --- /dev/null +++ b/processed_dataset/proof/2025.json @@ -0,0 +1,10 @@ +{ + "source_file": "./raw_volume-zh/volume9/chapter2.tex", + "problem_type": "proof", + "problem": "例6. (Bottema 不等式)设 $a_1 、 a_2 、 a_3 、 b_1 、 b_2 、 b_3$ 分别是位于同一平面上的两个三角形 $\\triangle A_1 A_2 A_3$ 和 $\\triangle B_1 B_2 B_3$ 的三边, $F 、 F^{\\prime}$ 分别是它们的面积, $x_1 、 x_2 、 x_3$ 分别是空间任一点 $P$ 到 $\\triangle A_1 A_2 A_3$ 三顶点的距离, 记\n$$\nM=b_1^2\\left(-a_1^2+a_2^2+a_3^2\\right)+b_2^2\\left(a_1^2-a_2^2+a_3^2\\right)+b_3^2\\left(a_1^2+a_2^2-a_3^2\\right) .\n$$\n求证:\n$$\n\\sum_{i=1}^3 b_i x_i \\geqslant\\left(\\frac{M}{2}+8 F F^{\\prime}\\right)^{\\frac{1}{2}} . \\label{eq1}\n$$", + "solution": "证明:如图(), 设 $A_1 A_2=a_3$, 在直线 $A_1 A_2$ 位于 $\\triangle A_1 A_2 A_3$ 的异侧作 $\\triangle A_1 A_2 C$, 使得 $\\triangle A_1 A_2 C \\backsim \\triangle B_1 B_2 B_3$, 则\n$$\nA_1 C=\\frac{a_3 b_2}{b_3}, A_2 C=\\frac{a_3 b_1}{b_3} .\n$$\n对空间四边形 $P A_1 C A_2$ 应用 Ptolemy 不等式可得\n$$\nx_1 \\frac{a_3 b_1}{b_3}+x_2 \\frac{a_3 b_2}{b_3} \\geqslant a_3 \\cdot P C,\n$$\n即\n$$\nb_1 x_1+b_2 x_2 \\geqslant b_3 \\cdot P C \\text {. }\n$$\n因此\n$$\n\\begin{aligned}\nb_1 x_1+b_2 x_2+b_3 x_3 & \\geqslant b_3 \\cdot P C+b_3 x_3 \\\\\n& =b_3\\left(P C+x_3\\right) \\geqslant b_3 \\cdot A_3 C,\n\\end{aligned}\n$$\n即有\n$$\n2\\left(b_1 x_1+b_2 x_2+b_3 x_3\\right)^2 \\geqslant 2 b_3^2 \\cdot A_3 C^2 . \\label{eq2}\n$$\n另一方面, 在 $\\triangle A_1 A_3 C$ 中应用余弦定理可得\n$$\nA_3 C^2=a_2^2+\\left(\\frac{a_3 b_2}{b_3}\\right)^2-2 a_2 \\cdot \\frac{a_3 b_2}{b_3} \\cdot \\cos \\left(\\angle A_3 A_1 A_2+\\angle A_2 A_1 C\\right) .\n$$\n因此\n$$\n\\begin{aligned}\n2 b_3^2 \\cdot A_3 C^2= & 2 a_2^2 b_3^2+2 a_3^2 b_2^2-4 a_2 a_3 b_2 b_3 \\cos \\left(\\angle A_3 A_1 A_2+\\angle A_2 A_1 C\\right) \\\\\n= & 2 a_2^2 b_3^2+2 a_3^2 b_2^2-4 a_2 a_3 b_2 b_3 \\cdot\\left(\\cos \\angle A_3 A_1 A_2 \\cdot \\cos \\angle A_2 A_1 C-\\right. \\\\\n& \\left.\\sin \\angle A_3 A_1 A_2 \\cdot \\sin \\angle A_2 A_1 C\\right) \\\\\n= & 2 a_2^2 b_3^2+2 a_3^2 b_2^2-4 a_2 a_3 b_2 b_3 \\cdot \\frac{a_2^2+a_3^2-a_1^2}{2 a_2 a_3} \\cdot \\frac{b_2^2+b_3^2-b_1^2}{2 b_2 b_3}+ \\\\\n& 4\\left(a_2 a_3 \\sin \\angle A_3 A_1 A_2\\right)\\left(b_2 b_3 \\sin \\angle A_2 A_1 C\\right) \\\\\n= & 2 a_2^2 b_3^2+2 a_3^2 b_2^2-\\left(a_2^2+a_3^2-a_1^2\\right)\\left(b_2^2+b_3^2-b_1^2\\right)+16 F F^{\\prime} \\\\\n= & b_1^2\\left(-a_1^2+a_2^2+a_3^2\\right)+b_2^2\\left(a_1^2-a_2^2+a_3^2\\right)+b_3^2\\left(a_1^2+a_2^2-a_3^2\\right)+ \\\\\n& 16 F F^{\\prime} \\\\\n= & M+16 F F^{\\prime} . \\label{eq3}\n\\end{aligned}\n$$\n由式\\ref{eq2}、\\ref{eq3},式\\ref{eq1}得证.", + "remark": "注:由著名的 Neuberg-Pedoe 不等式: $M \\geqslant 16 F F^{\\prime}$, 从 Bottema 不等式可推出关于两个三角形的如下不等式\n$$\nb_1 x_1+b_2 x_2+b_3 x_3 \\geqslant 4 \\sqrt{F F^{\\prime}} .\n$$\n在这个不等式中取 $\\triangle B_1 B_2 B_3$ 为正三角形, 则可得关于一个三角形内点到顶点距离的费马不等式\n$$\nx_1+x_2+x_3 \\geqslant 2 \\sqrt{\\sqrt{3} F} .\n$$\n当然, 在 Bottema 不等式中取 $\\triangle B_1 B_2 B_3$ 为正三角形, 则可得费马不等式的如下加强形式\n$$\nx_1+x_2+x_3 \\geqslant\\left(\\frac{1}{2}\\left(a^2+b^2+c^2\\right)+2 \\sqrt{3} F\\right)^{\\frac{1}{2}}\n$$", + "figures": [ + "./images/volume9/figures/fig-c2i10.png" + ] +} \ No newline at end of file diff --git a/processed_dataset/proof/2026.json b/processed_dataset/proof/2026.json new file mode 100644 index 0000000000000000000000000000000000000000..22dcf47cb7db94c345ba6a99da6ff8c5caa0995e --- /dev/null +++ b/processed_dataset/proof/2026.json @@ -0,0 +1,10 @@ +{ + "source_file": "./raw_volume-zh/volume9/chapter3.tex", + "problem_type": "proof", + "problem": "例1. 已知四边形 $A B C D$ 是圆的内接四边形,证明:\n$$\n|A B-C D|+|A D-B C| \\geqslant 2|A C-B D| .\n$$", + "solution": "证明:如图(), 设四边形 $A B C D$ 外接圆的圆心为 $O$, 该外接圆的半径为 $1, \\angle A O B=2 \\alpha, \\angle B O C=2 \\beta$, $\\angle C O D=2 \\gamma, \\angle D O A=2 \\delta$, 则\n$$\n\\alpha+\\beta+\\gamma+\\delta=\\pi \\text {. }\n$$\n不妨设 $\\alpha \\geqslant \\gamma, \\beta \\geqslant \\delta$, 则\n$$\n\\begin{aligned}\n& |A B-C D|=2|\\sin \\alpha-\\sin \\gamma| \\\\\n= & 4\\left|\\sin \\frac{\\alpha-\\gamma}{2} \\cos \\frac{\\alpha+\\gamma}{2}\\right|=4\\left|\\sin \\frac{\\alpha-\\gamma}{2} \\sin \\frac{\\beta+\\delta}{2}\\right| .\n\\end{aligned}\n$$\n同理\n$$\n\\begin{aligned}\n& |A D-B C|=4\\left|\\sin \\frac{\\beta-\\delta}{2} \\sin \\frac{\\alpha+\\gamma}{2}\\right| \\\\\n& |A C-B D|=4\\left|\\sin \\frac{\\beta-\\delta}{2} \\sin \\frac{\\alpha-\\gamma}{2}\\right|\n\\end{aligned}\n$$\n因此\n$$\n\\begin{aligned}\n|A B-C D|-|A C-B D| & =4\\left|\\sin \\frac{\\alpha-\\gamma}{2}\\right|\\left(\\left|\\sin \\frac{\\beta+\\delta}{2}\\right|-\\left|\\sin \\frac{\\beta-\\delta}{2}\\right|\\right) \\\\\n& =4\\left|\\sin \\frac{\\alpha-\\gamma}{2}\\right|\\left(\\sin \\frac{\\beta+\\delta}{2}-\\sin \\frac{\\beta-\\delta}{2}\\right)\n\\end{aligned}\n$$\n$$\n\\begin{aligned}\n& =4\\left|\\sin \\frac{\\alpha-\\gamma}{2}\\right| \\cdot\\left(2 \\cos \\frac{\\beta}{2} \\cdot \\sin \\frac{\\delta}{2}\\right) \\\\\n& \\geqslant 0 .\n\\end{aligned}\n$$\n故\n$$\n|A B-C D| \\geqslant|A C-B D| \\text {. }\n$$\n同理\n$$\n|A D-B C| \\geqslant|A C-B D| \\text {. }\n$$\n将这两个不等式相加即得求证结果.\n圆内接四边形中又有一种更特殊的四边形叫双圆四边形.\n所谓双圆四边形是既有外接圆又有内切圆的四边形.", + "remark": "", + "figures": [ + "./images/volume9/figures/fig-c3i1.png" + ] +} \ No newline at end of file diff --git a/processed_dataset/proof/2027.json b/processed_dataset/proof/2027.json new file mode 100644 index 0000000000000000000000000000000000000000..521bfca400919299ea7d4191310e92d0e61a1fea --- /dev/null +++ b/processed_dataset/proof/2027.json @@ -0,0 +1,10 @@ +{ + "source_file": "./raw_volume-zh/volume9/chapter3.tex", + "problem_type": "proof", + "problem": "例2. 凸四边形 $A B C D$ 既有内切圆又有外接圆,已知它的外接圆半径为 $R$, 面积为 $S$, 四边形的边长分别为 $a 、 b 、 c 、 d$, 证明:\n$$\na b c+a b d+a c d+b c d \\leqslant 2 \\sqrt{S}\\left(S+2 R^2\\right) . \\label{eq1}\n$$", + "solution": "证明:1 如图(), 设四边形 $A B C D$ 的外接圆和内切圆的圆心分别为 $O$ 和 $I$. 内切圆与边 $A B=a 、 B C=b 、 C D=c 、 D A=d$ 的切点分别为 $K 、 L 、 M 、 N$. 设 $\\angle A I N=\\angle 1, \\angle B I K=\\angle 2, \\angle C I L=\\angle 3$, $\\angle D I M=\\angle 4$, 并记 $A K=A N=a^{\\prime}, B L=B K=b^{\\prime}$, $C L=C M=c^{\\prime}, D M=D N=d^{\\prime}$.\n不妨设内切圆 $I$ 的半径 $r$ 为 1 . 由 $A B C D$ 有内切圆知\n$$\na+c=b+d \\text {. }\n$$\n这时若记式\\ref{eq1}的左边为 $H$, 则\n$$\nH=(a+c) b d+(b+d) a c=\\frac{1}{2}(a+b+c+d)(a c+b d) . \\label{eq2}\n$$\n又 $\\quad a=a^{\\prime}+b^{\\prime}, b=b^{\\prime}+c^{\\prime}, c=c^{\\prime}+d^{\\prime}, d=d^{\\prime}+a^{\\prime}$,\n将这些表达式代入式\\ref{eq2}的右边便得\n$$\nH=\\left(a^{\\prime}+b^{\\prime}+c^{\\prime}+d^{\\prime}\\right)\\left[\\left(a^{\\prime}+b^{\\prime}\\right)\\left(c^{\\prime}+d^{\\prime}\\right)+\\left(b^{\\prime}+c^{\\prime}\\right)\\left(d^{\\prime}+a^{\\prime}\\right)\\right] . \\label{eq3}\n$$\n又由 $\\angle A+\\angle C=180^{\\circ}$, 可得\n$$\n\\angle 1+\\angle 3=90^{\\circ} .\n$$\n因此 $\\triangle A I N \\backsim \\triangle I C L$, 由此得\n$$\na^{\\prime} c^{\\prime}=A N \\cdot C L=N I \\cdot I L=1 . \\label{eq4}\n$$\n同理\n$$\nb^{\\prime} d^{\\prime}=1 . \\label{eq5}\n$$\n又注意到\n$$\nS=\\frac{a+b+c+d}{2} \\cdot r=a^{\\prime}+b^{\\prime}+c^{\\prime}+d^{\\prime}, \\label{eq6}\n$$\n因此由式\\ref{eq4}、\\ref{eq5}、式\\ref{eq6}可得\n$$\nH=S\\left[4+\\left(a^{\\prime}+c^{\\prime}\\right)\\left(b^{\\prime}+d^{\\prime}\\right)\\right] . \\label{eq7}\n$$\n另一方面, 由正弦定理并注意到 $\\angle B+2 \\angle 2=180^{\\circ}$, 有\n$$\n\\begin{aligned}\nR & =\\frac{A C}{2 \\sin \\angle B}=\\frac{A C}{2 \\sin 2 \\angle 2}=\\frac{A C}{4}\\left(\\tan \\angle 2+\\frac{1}{\\tan \\angle 2}\\right) \\\\\n& =\\frac{1}{4} A C(\\tan \\angle 2+\\tan \\angle 4) \\\\\n& =\\frac{1}{4} A C \\cdot\\left(b^{\\prime}+d^{\\prime}\\right) .\n\\end{aligned}\n$$\n同理\n$$\nR=\\frac{1}{4} B D \\cdot\\left(a^{\\prime}+c^{\\prime}\\right) .\n$$\n因此\n$$\nR^2=\\frac{1}{16} A C \\cdot B D\\left(a^{\\prime}+c^{\\prime}\\right)\\left(b^{\\prime}+d^{\\prime}\\right),\n$$\n但是\n$$\nS=\\frac{1}{2} A C \\cdot B D \\cdot \\sin \\alpha \\leqslant \\frac{1}{2} A C \\cdot B D,\n$$\n这里 $\\alpha$ 为对角线 $A C$ 与 $B D$ 夹角.\n因此\n$$\nR^2 \\geqslant \\frac{1}{8} S\\left(a^{\\prime}+c^{\\prime}\\right)\\left(b^{\\prime}+d^{\\prime}\\right) .\n$$\n由上可知\n$$\n\\text { 式\\ref{eq1} } \\begin{aligned}\n\\text { 的右边 } & \\geqslant 2 \\sqrt{S}\\left(S+\\frac{S}{4}\\left(a^{\\prime}+c^{\\prime}\\right)\\left(b^{\\prime}+d^{\\prime}\\right)\\right) \\\\\n& =\\frac{S^{\\frac{3}{2}}}{2}\\left[4+\\left(a^{\\prime}+c^{\\prime}\\right)\\left(b^{\\prime}+d^{\\prime}\\right)\\right] . \\label{eq8}\n\\end{aligned}\n$$\n因此由式\\ref{eq7}、\\ref{eq8}知, 要证式\\ref{eq1}, 只需证明 $\\frac{1}{2} S^{\\frac{1}{2}} \\geqslant 1$, 这等价于\n$$\n\\sqrt{a^{\\prime}+b^{\\prime}+c^{\\prime}+d^{\\prime}} \\geqslant 2 . \\label{eq9}\n$$\n而由 $a^{\\prime} c^{\\prime}=1, b^{\\prime} d^{\\prime}=1$ 可知\n$$\na^{\\prime}+b^{\\prime}+c^{\\prime}+d^{\\prime} \\geqslant 2 \\sqrt{a^{\\prime} c^{\\prime}}+2 \\sqrt{b^{\\prime} d^{\\prime}}=4,\n$$\n式\\ref{eq9}得证.", + "remark": "", + "figures": [ + "./images/volume9/figures/fig-c3i2.png" + ] +} \ No newline at end of file diff --git a/processed_dataset/proof/2028.json b/processed_dataset/proof/2028.json new file mode 100644 index 0000000000000000000000000000000000000000..7e4ede73a1e15cfd5670bdaeb7eab715a1b8a842 --- /dev/null +++ b/processed_dataset/proof/2028.json @@ -0,0 +1,10 @@ +{ + "source_file": "./raw_volume-zh/volume9/chapter3.tex", + "problem_type": "proof", + "problem": "例2. 凸四边形 $A B C D$ 既有内切圆又有外接圆,已知它的外接圆半径为 $R$, 面积为 $S$, 四边形的边长分别为 $a 、 b 、 c 、 d$, 证明:\n$$\na b c+a b d+a c d+b c d \\leqslant 2 \\sqrt{S}\\left(S+2 R^2\\right) . \\label{eq1}\n$$", + "solution": "证明 2 先证引理.\n引理设 $\\triangle A B C$ 中, $\\angle A \\geqslant 90^{\\circ}$, 则 $\\frac{b+c}{a} \\leqslant \\sqrt{2}$.\n证明\n$$\n\\begin{aligned}\n\\frac{b+c}{a} & =\\frac{\\sin B+\\sin C}{\\sin A}=2 \\frac{\\sin \\frac{B+C}{2} \\cos \\frac{B-C}{2}}{\\sin A} \\\\\n& \\leqslant \\frac{2 \\cos \\frac{A}{2}}{2 \\sin \\frac{A}{2} \\cos \\frac{A}{2}}=\\frac{1}{\\sin \\frac{A}{2}} \\leqslant \\sqrt{2} .\n\\end{aligned}\n$$\n下面证明原题中的不等式.\n如图(), 设四边形 $A B C D$ 的四边 $A B 、 B C 、 C D$ 、 $D A$ 的长分别为 $a 、 b 、 c 、 d$, 再设 $A B C D$ 的内切圆的半径为 1 . 注意到\n$$\na+c=b+d=\\frac{1}{2}(a+b+c+d) \\cdot 1=S,\n$$\n可得\n$$\n\\begin{aligned}\nH & =a b c+a b d+a c d+b c d \\\\\n& =a c(b+d)+b d(a+c)=(a c+b d) S . \\label{eq1}\n\\end{aligned}\n$$\n现设 $A C$ 的中垂线为 $l, D$ 关于 $l$ 的对称点为 $E$, 则\n$$\n\\triangle A C D \\cong \\triangle C A E .\n$$\n因此 $A E=c, C E=d$, 且 $\\angle E=\\angle D=\\pi-\\angle B$, 由此可知 $A 、 E 、 C 、 B$ 四点共圆, 故\n$$\nS=\\frac{1}{2}(a c+b d) \\sin \\alpha, \\label{eq2}\n$$\n其中 $\\alpha=\\angle E A B$.\n由式\\ref{eq1}、\\ref{eq2}知原不等式等价于\n$$\n\\frac{2 S}{\\sin \\alpha} \\cdot S \\leqslant 2 \\sqrt{S}\\left(S+2 R^2\\right) . \\label{eq3}\n$$\n注意到 $R=\\frac{B E}{2 \\sin \\alpha}$, 因此式\\ref{eq3}进一步等价于\n$$\nS^{\\frac{3}{2}} \\leqslant S \\sin \\alpha+\\frac{B E^2}{2 \\sin \\alpha}, \\label{eq4}\n$$\n但由平均值不等式\n\\ref{eq4} 的右端 $\\geqslant 2 \\sqrt{\\frac{S \\cdot B E^2}{2}}$.\n因此要证式\\ref{eq1}, 只需证明\n$$\n\\sqrt{2} B E \\geqslant S . \\label{eq5}\n$$\n事实上, 由 $\\angle E A B+\\angle E C B=180^{\\circ}$, 不妨设 $\\angle E A B \\geqslant 90^{\\circ}$. 对 $\\triangle A B E$ 应用引理可知 $\\frac{a+c}{B E} \\leqslant \\sqrt{2}$, 故\n$$\n\\sqrt{2} B E \\geqslant a+c=S,\n$$\n\\ref{eq1}式得证.\n上面的证法综合运用三角、几何的技巧,构造了一个新的共圆四边形,实现了问题的转化.", + "remark": "", + "figures": [ + "./images/volume9/figures/fig-c3i3.png" + ] +} \ No newline at end of file diff --git a/processed_dataset/proof/2029.json b/processed_dataset/proof/2029.json new file mode 100644 index 0000000000000000000000000000000000000000..90ba369a47beee5e7c192eaa168322419cf6f5ef --- /dev/null +++ b/processed_dataset/proof/2029.json @@ -0,0 +1,8 @@ +{ + "source_file": "./raw_volume-zh/volume9/chapter3.tex", + "problem_type": "proof", + "problem": "例3. (Popa 不等式) 如果一个凸四边形的四边满足 $a \\leqslant b \\leqslant c \\leqslant d$, 面积为 $F$, 求证:\n$$\nF \\leqslant \\frac{3 \\sqrt{3}}{4} c^2 . \\label{eq1}\n$$", + "solution": "证明:由于边长给定的四边形中, 圆内接四边形的面积最大, 因此我们仅需对圆内接四边形证明式\\ref{eq1}便可.\n这时\n$$\nF^2=(s-a)(s-b)(s-c)(s-d),\n$$\n其中 $s=\\frac{1}{2}(a+b+c+d)$. 但是 $s-d=(a+b+c)-s$, 因此由算术几何平均值不等式可得\n$$\n\\begin{aligned}\nF^2 & =3^3\\left(\\frac{1}{3} s-\\frac{1}{3} a\\right)\\left(\\frac{1}{3} s-\\frac{1}{3} b\\right)\\left(\\frac{1}{3} s-\\frac{1}{3} c\\right)(a+b+c-s) \\\\\n& \\leqslant 3^3\\left[\\frac{\\left(\\frac{1}{3} s-\\frac{1}{3} a\\right)+\\left(\\frac{1}{3} s-\\frac{1}{3} b\\right)+\\left(\\frac{1}{3} s-\\frac{1}{3} c\\right)+(a+b+c-s)}{4}\\right]^4 \\\\\n& =3^3\\left(\\frac{a+b+c}{3 \\cdot 2}\\right)^4 \\leqslant 3^3\\left(\\frac{c}{2}\\right)^4 .\n\\end{aligned}\n$$\n最后一步用了 $a \\leqslant b \\leqslant c$.\n两边开方, 由此便得\n$$\nF \\leqslant \\frac{3 \\sqrt{3}}{4} c^2\n$$\n得证.", + "remark": "", + "figures": [] +} \ No newline at end of file diff --git a/processed_dataset/proof/2030.json b/processed_dataset/proof/2030.json new file mode 100644 index 0000000000000000000000000000000000000000..b20bfbb8232447855a85aa9627086ea210fc37a5 --- /dev/null +++ b/processed_dataset/proof/2030.json @@ -0,0 +1,10 @@ +{ + "source_file": "./raw_volume-zh/volume9/chapter3.tex", + "problem_type": "proof", + "problem": "例4. (高灵不等式)设凸四边形 $A B C D$ 和 $A^{\\prime} B^{\\prime} C^{\\prime} D^{\\prime}$ 的四边分别为 $a$ 、 $b 、 c 、 d$ 和 $a^{\\prime} 、 b^{\\prime} 、 c^{\\prime} 、 d^{\\prime}$, 它们的面积分别为 $F 、 F^{\\prime}$. 令\n$$\nK=4(a d+b c)\\left(a^{\\prime} d^{\\prime}+b^{\\prime} c^{\\prime}\\right)-\\left(a^2-b^2-c^2+d^2\\right)\\left(a^{\\prime 2}-b^{\\prime 2}-c^{\\prime 2}+d^{\\prime 2}\\right) \\text {. }\n$$\n求证:\n$$\nK \\geqslant 16 F F^{\\prime} \\text {. }\n$$", + "solution": "证明:由于给定边长的四边形以圆的内接四边形具有最大面积, 因此仅需考虑 $A B C D$ 和 $A^{\\prime} B^{\\prime} C^{\\prime} D^{\\prime}$ 均为圆内接四边形的情况.\n如图(), 因为 $\\angle B+\\angle D=180^{\\circ}$, 所以\n$$\n2 F=(a d+b c) \\sin B , \\label{eq1}\n$$\n类似的有\n$$\n2 F^{\\prime}=\\left(a^{\\prime} d^{\\prime}+b^{\\prime} c^{\\prime}\\right) \\sin B^{\\prime} . \\label{eq2}\n$$\n另一方面, 由余弦定理\n$$\n\\begin{aligned}\nA C^2 & =b^2+c^2+2 b c \\cos B \\\\\n& =a^2+d^2-2 a d \\cos B,\n\\end{aligned}\n$$\n因此 $\\quad a^2-b^2-c^2+d^2=2(a d+b c) \\cos B, \\label{eq3}$,\n类似的有 $\\quad a^{\\prime 2}-b^{\\prime 2}-c^{\\prime 2}+d^{\\prime 2}=2\\left(a^{\\prime} d^{\\prime}+b^{\\prime} c^{\\prime}\\right) \\cos B^{\\prime}, \\label{eq4}$.\n由式\\ref{eq1}、\\ref{eq4}可得\n$$\nK-16 F F^{\\prime}=4(a d+b c)\\left(a^{\\prime} d^{\\prime}+b^{\\prime} c^{\\prime}\\right)\\left(1-\\cos \\left(B-B^{\\prime}\\right)\\right) \\geqslant 0,\n$$\n故原不等式成立.", + "remark": "注:由上面的证法, 实际上可把高灵不等式写的更一般一些:\n$$\n0 \\leqslant K-16 F F^{\\prime} \\leqslant 8(a d+b c)\\left(a^{\\prime} d^{\\prime}+b^{\\prime} c^{\\prime}\\right),\n$$\n左边的不等式即为高灵不等式.\n高灵不等式可看作是著名的 Neuberg-Pedoe 不等式在四边形中的推广.\n在这一节的最后, 我们研究一个难度较大的关于双圆四边形的极值问题, 这里要介绍的解法由向振同学 (原长沙市一中学生, 曾获 2003 年第 44 届 $\\mathrm{IMO}$ 金牌)给出.", + "figures": [ + "./images/volume9/figures/fig-c3i5.png" + ] +} \ No newline at end of file diff --git a/processed_dataset/proof/2031.json b/processed_dataset/proof/2031.json new file mode 100644 index 0000000000000000000000000000000000000000..00ec40dbe649b189aafcfd14951643af47cec427 --- /dev/null +++ b/processed_dataset/proof/2031.json @@ -0,0 +1,11 @@ +{ + "source_file": "./raw_volume-zh/volume9/chapter3.tex", + "problem_type": "proof", + "problem": "例5. 给定外接圆半径 $R$ 和面积 $S$ 不变的双圆四边形 $A B C D$ (这里 $S \\leqslant 2 R^2$ ), 求 $p l m$ 的最大值, 其中 $p$ 是四边形 $A B C D$ 的半周长, $l 、 m$ 分别为它的两条对角线长.", + "solution": "解:如图(), 我们可以用三个参数 $r 、 \\alpha 、 \\beta(\\gamma \\in(0,+\\infty), \\alpha 、 \\beta \\in \\left.\\left(0, \\frac{\\pi}{2}\\right)\\right)$ 来确定一个双圆四边形 $A B C D$, 这里的 $r$ 是四边形 $A B C D$ 的内切圆半径, $\\alpha=\\angle A I K, \\beta=\\angle B I K$, 其中 $I$ 是四边形 $A B C D$ 的内切圆的圆心, $K$ 是圆 $I$ 与边 $A B$ 的切点.\n下面证明:\n$$\n\\begin{gathered}\nS=r^2\\left(\\frac{2}{\\sin 2 \\alpha}+\\frac{2}{\\sin 2 \\beta}\\right), \\label{eq1}\\\\\nR^2=r^2\\left(\\frac{1}{\\sin 2 \\alpha \\sin 2 \\beta}+\\frac{1}{\\sin ^2 2 \\alpha \\sin ^2 2 \\beta}\\right) . \\label{eq2}\n\\end{gathered}\n$$\n先证式\\ref{eq1}. 因为半周长\n$$\np=r(\\tan \\alpha+\\cot \\alpha+\\tan \\beta+\\cot \\beta)=r\\left(\\frac{2}{\\sin 2 \\alpha}+\\frac{2}{\\sin 2 \\beta}\\right),\n$$\n所以\n$$\nS=r p=r^2\\left(\\frac{2}{\\sin 2 \\alpha}+\\frac{2}{\\sin 2 \\beta}\\right)\n$$\n这就是式\\ref{eq1}.\n再证式\\ref{eq2}. 在 $\\triangle A B D$ 中易知\n$$\nA B=r(\\tan \\alpha+\\tan \\beta), A D=r(\\tan \\alpha+\\cot \\beta), \\angle D A B=\\pi-2 \\alpha,\n$$\n故由余弦定理并通过三角化简可得\n$$\n\\begin{aligned}\nB D^2= & r^2\\left[(\\tan \\alpha+\\tan \\beta)^2+(\\tan \\alpha+\\cot \\beta)^2+\\right. \\\\\n& 2 \\cos 2 \\alpha(\\tan \\alpha+\\tan \\beta)(\\tan \\alpha+\\cot \\beta)] \\\\\n= & r^2\\left(\\tan \\alpha \\cdot \\frac{2}{\\sin 2 \\beta} \\cdot 4 \\cos ^2 \\alpha+\\frac{4}{\\sin ^2 2 \\beta}\\right),\n\\end{aligned}\n$$\n故\n$$\nR^2=\\frac{B D^2}{4 \\sin ^2 2 \\alpha}=r^2\\left(\\frac{1}{\\sin 2 \\alpha \\sin 2 \\beta}+\\frac{1}{\\sin ^2 2 \\alpha \\sin ^2 2 \\beta}\\right),\n$$\n这就是式\\ref{eq1}.\n现令 $a=\\sin 2 \\alpha, b=\\sin 2 \\beta$, 则 $a, b \\in(0,1]$, 且(1)、(2)可写为\n$$\n\\begin{aligned}\nS & =2 r^2 \\frac{a+b}{a b}, \\label{eq3}\\\\\nR^2 & =r^2 \\frac{1+a b}{a^2 b^2}, \\label{eq4}\n\\end{aligned}\n$$\n式\\ref{eq3}除以\\ref{eq4}可得\n$$\n\\frac{a b(a+b)}{1+a b}=\\frac{S}{2 R^2} . \\label{eq5}\n$$\n式\\ref{eq5}是 $a 、 b$ 所满足的约束条件, 我们在此条件下求 $p l m$ 的最大值.\n易知\n$$\n\\begin{gathered}\np=r \\cdot\\left(\\frac{2}{a}+\\frac{2}{b}\\right), \\\\\nl m=4 R^2 a b,\n\\end{gathered}\n$$\n所以 $(p l m)^2=64 R^4 r^2(a+b)^2=16 R^2 S^2(1+a b)$,\n因此\n$$\np l m=4 R S \\sqrt{1+a b} . \\label{eq6}\n$$\n由式\\ref{eq5}得\n$$\n\\frac{S}{2 R^2}=\\frac{a b(a+b)}{1+a b} \\geqslant \\frac{a b \\cdot 2 \\sqrt{a b}}{1+a b} . \\label{eq7}\n$$\n令 $\\sqrt{a b}=x$, 则 $x \\in(0,1]$, 于是式\\ref{eq7}可写为\n$$\n4 R^2 \\cdot x^3-S \\cdot x^2-S \\leqslant 0 . \\label{eq8}\n$$\n令函数 $f(x)=4 R^2 \\cdot x^3-S x^2-S$, 注意到\n$$\nf(0)=-S<0, f(1)=4 R^2-2 S \\geqslant 0,\n$$\n且\n$$\nf^{\\prime}(x)= \\begin{cases}\\geqslant 0, & x \\geqslant \\frac{S}{6 R^2}, \\\\ <0, & 0) 所示.\n由式\\ref{eq8}知 $f(\\sqrt{a b}) \\leqslant 0$, 因此\n$$\n\\sqrt{a b} \\leqslant t,\n$$\n于是 $a b \\leqslant t^2$, 从而代入式\\ref{eq6}知\n$$\np l m=4 R S \\sqrt{1+a b} \\leqslant 4 R S \\sqrt{1+t^2},\n$$\n当 $a=b=t$ 时, 等号成立.\n故所求的 $p l m$ 的最大值为 $4 R S \\sqrt{1+t^2}$, 其中 $t$ 是方程 $4 R^2 x^3-S x^2-S=0$ 在区间 $(0,1]$ 上的根.", + "remark": "", + "figures": [ + "./images/volume9/figures/fig-c3i6.png", + "./images/volume9/figures/fig-c3i7.png" + ] +} \ No newline at end of file diff --git a/processed_dataset/proof/2032.json b/processed_dataset/proof/2032.json new file mode 100644 index 0000000000000000000000000000000000000000..56d0bda280d4e9fce08997d4014daed2328117ef --- /dev/null +++ b/processed_dataset/proof/2032.json @@ -0,0 +1,11 @@ +{ + "source_file": "./raw_volume-zh/volume9/chapter4.tex", + "problem_type": "proof", + "problem": "例1. 设 $P$ 是 $\\triangle A B C$ 内一点, $P$ 的塞瓦三角形为 $D E F$, 求证: 总可以以 $\\triangle D E F$ 的某两边为邻边作一平行四边形使之位于 $\\triangle A B C$ 内.", + "solution": "证明:如图(), 设 $G$ 是 $\\triangle A B C$ 的重心, $N$ 、 $M$ 分别是边 $A C$ 和 $A B$ 的中点.\n不妨设 $P$ 在 $A N G M$ 内部或边界上,则 $E 、 F$ 分别在线段 $A N$ 、 $A M$ 的内部或端点处,所以\n$$\n\\frac{A F}{F B} \\leqslant 1, \\frac{A E}{E C} \\leqslant 1,\n$$\n又不妨设\n$$\n\\frac{A F}{F B} \\leqslant \\frac{A E}{E C} .\n$$\n由塞瓦定理可得\n$$\n\\frac{A F}{F B} \\cdot \\frac{B D}{D C} \\cdot \\frac{C E}{E A}=1,\n$$\n由此推得\n$$\n\\frac{B D}{D C}=\\frac{A E}{C E} \\cdot \\frac{F B}{A F} \\geqslant 1 .\n$$\n如图(), 作出以 $E F 、 E D$ 为邻边的平行四边形 $F E D E^{\\prime}$, 下面只需证 $E^{\\prime}$ 位于 $\\triangle A B C$ 内部或边界上.\n过 $F$ 作 $F F^{\\prime} / / B C, F^{\\prime}$ 落在 $A C$ 上, 因为\n$$\n\\frac{A F}{F B} \\leqslant \\frac{A E}{E C},\n$$\n故 $F^{\\prime}$ 在线段 $A E$ 内部或端点上.\n因为\n$$\n\\angle E^{\\prime} D F=\\angle E F D \\leqslant \\angle F^{\\prime} F D=\\angle F D B,\n$$\n所以 $D E^{\\prime}$ 在 $\\angle F D B$ 内部.\n同理\n$$\n\\frac{C E}{E A} \\geqslant 1 \\geqslant \\frac{C D}{D B}\n$$\n也可证明 $F E^{\\prime}$ 在 $\\angle B F D$ 的内部或边界上, 故 $E^{\\prime}$ 在 $\\triangle F D B$ 的内部.\n得证.", + "remark": "注:定理 1 和命题 1 通过例 1 联合起来了, 即由例 1 ,\n定理 $1 \\Rightarrow$ 命题 1 .\n一个自然的问题是, 内点 $P$ 的垂足三角形是否有类似于塞瓦三角形的扩张性质呢?\n易见针角三角形的内点的垂足三角形一般不具有扩张性质, 但对于锐角三角形有下面的正面回答.", + "figures": [ + "./images/volume9/figures/fig-c4i6.png", + "./images/volume9/figures/fig-c4i7.png" + ] +} \ No newline at end of file diff --git a/processed_dataset/proof/2033.json b/processed_dataset/proof/2033.json new file mode 100644 index 0000000000000000000000000000000000000000..d6691d89e2ce00e6f56e1e9125d127030b0a2524 --- /dev/null +++ b/processed_dataset/proof/2033.json @@ -0,0 +1,10 @@ +{ + "source_file": "./raw_volume-zh/volume9/chapter4.tex", + "problem_type": "proof", + "problem": "例2. 设 $P$ 是锐角三角形 $\\triangle A B C$ 内一点, 关于 $P$ 的垂足三角形为 $\\triangle D E F$. 求证: 总可以以 $\\triangle D E F$ 的某两边为邻边作一平行四边形使之位于 $\\triangle A B C$ 内.", + "solution": "证明:设 $O$ 是 $\\triangle A B C$ 的外心, 因为 $\\triangle A B C$ 为锐角三角形, 所以 $O$ 位于 $\\triangle A B C$ 内.\n不妨设 $P$. 落在 $\\triangle A O B$ 内, 如图().\n我们证明以 $F E 、 F D$ 为邻边作的平行四边形 $D F E G$ 位于 $\\triangle A B C$ 内.\n为此, 只需证明\n$$\n\\begin{aligned}\n& \\angle F E G \\leqslant \\angle F E C, \\label{eq1}\\\\\n& \\angle F D G \\leqslant \\angle F D C . \\label{eq2}\n\\end{aligned}\n$$\n下证式\\ref{eq1}, \\ref{eq2}类似可证.\n因为\n$$\n\\begin{gathered}\n\\angle F E G=\\angle A F E+\\angle B F D, \\\\\n\\angle F E C=\\angle A F E+\\angle A,\n\\end{gathered}\n$$\n因此要证式\\ref{eq1}, 只需证明\n$$\n\\angle B F D \\leqslant \\angle A . \\label{eq3}\n$$\n事实上, 由 $B 、 F 、 P 、 D$ 四点共圆知\n$$\n\\angle B F D=\\angle B P D . \\label{eq4}\n$$\n现过 $O$ 作 $O H \\perp B C$, 垂足为 $H$, 则由\n$$\n\\angle P B D \\geqslant \\angle O B H\n$$\n可知\n$$\n\\angle B P D \\leqslant \\angle B O H, \\label{eq5}\n$$\n而 $O$ 为 $\\triangle A B C$ 的外心, 因此\n$$\n\\angle B O H=\\angle B A C=\\angle A . \\label{eq6}\n$$\n由式\\ref{eq4}、\\ref{eq5}、式\\ref{eq6}, \\ref{eq3}得证.", + "remark": "注:由例 2 我们知, 对锐角三角形, 由定理 1 可推出命题 2 .\n上例曾被用作第 2 届中国西部数学奥林匹克试题 (笔者为了降低难度, 加上了 $P$ 位于 $\\triangle A O B$ 内这一条件).\n下面的话题转向三角形内的五点问题,这个问题是 A. Soifer 提供给 Colorado 数学奥林匹克的一个试题.\n他提出并证明了: 在单位面积的三角形内任给五点, 则至少有三点组成的三角形的面积不超过 $\\frac{1}{4}$.\n不难证明, 五点问题的点数不能减少, 但着眼于结论中三角形的个数, 我们仍能改进问题的结论.", + "figures": [ + "./images/volume9/figures/fig-c4i8.png" + ] +} \ No newline at end of file diff --git a/processed_dataset/proof/2034.json b/processed_dataset/proof/2034.json new file mode 100644 index 0000000000000000000000000000000000000000..ae95691426abdf2306bbc62a089c17c20db27f47 --- /dev/null +++ b/processed_dataset/proof/2034.json @@ -0,0 +1,12 @@ +{ + "source_file": "./raw_volume-zh/volume9/chapter4.tex", + "problem_type": "proof", + "problem": "例3. 在单位面积的三角形中任给五点,则其中必存在两个不同的三点组使得以它们为顶点构成的三角形的面积不超过 $\\frac{1}{4}$.", + "solution": "证明:我们需要下面常用的引理.\n引理设凸四边形位于一个单位面积的三角形内,则这个凸四边形的四个顶点中必有三个顶点组成的三角形的面积不超过 $\\frac{1}{4}$.\n凸四边形的四个顶点本质上都可化归到三角形的边上, 因此这个引理实质上就是大家熟知的首届冬令营的试题的第二题: 设 $P_1 、 P_2 、 P_3 、 P_4$ 位于 $\\triangle A B C$ 的三边上, 求证: $\\triangle P_1 P_2 P_3 、 \\triangle P_1 P_3 P_4 、 \\triangle P_2 P_3 P_4 、 \\triangle P_1 P_2 P_4$ 中必有一个面积小于或等于 $\\frac{1}{4} S_{\\triangle A B C}$.\n下面回证原题.\n当这五点的凸包为线段时,结论显然成立.\n当这五点的凸包为三角形时,则可以以这五个点为顶点作出五个互不相交的三角形, 如图(), 而且它们的总面积小于或等于 1 , 故必有两个三角形的面积不超过 $\\frac{1}{4}$.\n若这五点的凸包为凸四边形时, 不妨设五点分布如图(), 即 $P_5$ 位于凸四边形 $P_1 P_2 P_3 P_4$ 内, 由引理可知 $P_1 、 P_2 、 P_3 、 P_4$ 中必有三点组成的三角形的面积不超过 $\\frac{1}{4}$, 又\n$$\n\\begin{gathered}\nS_{\\triangle P_1 P_2 P_5}+S_{\\triangle P_2 P_3 P_5}+S_{\\triangle P_3 P_4 P_5}+S_{\\triangle P_4 P_1 P_5} \\\\\n\\leqslant S\\left(P_1 P_2 P_3 P_4\\right) \\leqslant S_{\\triangle A B C}=1,\n\\end{gathered}\n$$\n因此 $\\triangle P_1 P_2 P_5 、 \\triangle P_2 P_3 P_5 、 \\triangle P_3 P_4 P_5 、 \\triangle P_4 P_1 P_5$ 中必有一个的面积小于或等于 $\\frac{1}{4}$, 结论成立.\n若这五点的凸包为凸五边形, 则其中任何四顶点均可构成嵌人 $\\triangle A B C$ 中的凸四边形, 如图(), 这样的凸四边形共有 $\\mathrm{C}_5^4=5$ 个.\n因而包括重复计算, 必有 5 个面积不超过 $\\frac{1}{4}$ 的三角形, 又每个三角形至多重复两次,故面积不超过 $\\frac{1}{4}$ 的三角形的个数大于 $\\left[\\frac{5}{2}\\right]=2$,\n得证.", + "remark": "注:1 可以证明上例的结论还可以改进, 其中存在的两个三角形可改进为三个三角形 (不能改进为四个三角形) 不超过 $\\frac{1}{4}$. 但这个证明篇幅很大, 这里从略.\n注:2 任给一个图形 $F$, 令 $S(F)$ 表示满足下面条件的最小的正整数 $n$ : 在 $F$ 的内部 (含边界) 任给 $n$ 个点使得总存在其中的三个点, 它们构成的三角形的面积不超过 $\\frac{|F|}{4}$, 这里 $|F|$ 表示 $F$ 的面积.", + "figures": [ + "./images/volume9/figures/fig-c4i9.png", + "./images/volume9/figures/fig-c4i10.png", + "./images/volume9/figures/fig-c4i11.png" + ] +} \ No newline at end of file diff --git a/processed_dataset/proof/2035.json b/processed_dataset/proof/2035.json new file mode 100644 index 0000000000000000000000000000000000000000..084b979978ad523794141dd4cd3528531f5571a2 --- /dev/null +++ b/processed_dataset/proof/2035.json @@ -0,0 +1,12 @@ +{ + "source_file": "./raw_volume-zh/volume9/chapter4.tex", + "problem_type": "proof", + "problem": "例 4 证明: (1) 面积为 1 的凸多边形可被面积为 2 的平行四边形覆盖;\n(2) 面积为 1 的凸多边形可被面积为 2 的三角形覆盖.", + "solution": "证明:(1) 设面积为 1 的凸多边形 $M$ 位于它的一条支撑线 $A B$ 的一侧, 则 $M$ 中存在一点到直线 $A B$ 的距离最大, 记这个点为 $C(C$ 可能是 $M$ 的一个顶点,也可能是 $M$ 的一条平行于 $A B$ 的边上的任意一点). 现在连接 $A C$ (如图()), 将 $M$ 分成两部分 $M_1$ 和 $M_2$ (如果 $A C$ 是 $M$ 的一边, $M_1 、 M_2$ 中有一个不存在). 假设 $D_1$ 和 $D_2$ 是 $M$ 的点 (它们分别位于 $A C$ 的两侧) 且到 $A C$ 有最大的距离, 再过 $C$ 作直线平行于 $A B$, 过 $D_1 、 D_2$ 作直线 $l_1$ 和 $l_2$ 平行于 $A C$, 则直线 $A B 、 l 、 l_1 、 l_2$ 构成了包含 $M$ 的一个平行四边形 $P$.\n因为 $M_1$ 和 $M_2$ 是凸的, 所以它们分别包含 $\\triangle A D_1 C$ 和 $\\triangle A D_2 C$.\n设 $P_1 、 P_2$ 是直线 $A C$ 将 $P$ 分成的两个平行四边形, 则\n$$\nS_{\\triangle A D_1 C}=\\frac{1}{2} S\\left(P_1\\right), S_{\\triangle A D_2 C}=\\frac{1}{2} S\\left(P_2\\right),\n$$\n其中 $S(X)$ 表示 $X$ 的面积.\n因此\n$$\n\\begin{gathered}\nS(P)=S\\left(P_1\\right)+S\\left(P_2\\right)=2 S_{\\triangle A D_1 C}+2 S_{\\triangle A D_2 C} \\\\\n\\leqslant 2 S\\left(M_1\\right)+2 S\\left(M_2\\right)=2 S(M)=2 .\n\\end{gathered}\n$$\n(1) 得证.\n(2)设 $u$ 是给定的面积为 1 的多边形.\n现考虑 $u$ 的最大面积的内接 $\\triangle A_1 A_2 A_3$. 下面分两种情况讨论.\n(a) 若 $S_{\\triangle A_1 A_2 A_3} \\leqslant \\frac{1}{2}$. 这时如图(), 过 $\\triangle A_1 A_2 A_3$ 的顶点分别作对边的平行线, 这三条直线交成的三角形记作 $T$, 则 $T$ 的面积小于等于 2 .\n因此, 这时我们只需证明多边形 $u$ 位于 $T$ 内便可.\n假定 $u$ 的某个点 $M$ 位于 $T$ 外, 则 $M$ 到 $\\triangle A_1 A_2 A_3$ 某一边 (不妨设为 $A_1 A_2$ ) 的距离大于这个三角形另一个顶点 $\\left(A_3\\right)$ 到这一边的距离 (见图 4-13). 这时 $\\triangle A_1 A_2 M$ 的面积大于 $\\triangle A_1 A_2 A_3$ 的面积, 这与 $\\triangle A_1 A_2 A_3$ 是 $u$ 中的最大面积的内接三角形矛盾.\n这种情况得证.\n(b) 若 $S_{\\triangle A_1 A_2 A_3}>\\frac{1}{2}$. 这时在 $u$ 被 $\\triangle A_1 A_2 A_3$ 的每条边所在的直线切割的剩余部分内, 分别以 $\\triangle A_1 A_2 A_3$ 的一边为底构造面积最大的三角形.\n设这样的三个三角形分别为 $\\triangle B_1 A_2 B_3 、 \\triangle B_2 A_1 B_3$ 、 $\\triangle B_3 A_1 A_2$, 再过 $B_1 、 B_2 、 B_3$ 分别作 $A_2 A_3 、 A_1 A_3$ 、 $A_1 A_2$ 的平行线, 我们就得到一个较大的三角形 $\\triangle C_1 C_2 C_3$, 记为 $C$ (如图()). 同 (a) 可证, $u$ 一定在三角形 $C$ 内.\n注意到 $u$ 是一个凸多边形, 因此\n$$\nS\\left(A_1 B_3 A_2 B_1 A_3 B_2\\right) \\leqslant S(u)=1 .\n$$\n因此我们只需证明\n$$\nS_{\\triangle C_1 C_2 C_3} \\leqslant 2 S\\left(A_1 B_3 A_2 B_1 A_3 B_2\\right), \\label{eq1}\n$$\n便知结论成立.\n因为 $\\triangle C_1 C_2 C_3 \\backsim \\triangle A_1 A_2 A_3$, 所以为了计算 $\\triangle C_1 C_2 C_3$ 的面积, 我们记\n$$\n\\frac{S_{\\triangle A_1 A_2 B_3}}{A_{\\triangle A_1 A_2 A_3}}=\\lambda_3, \\frac{S_{\\triangle A_1 A_3 B_2}}{S_{\\triangle A_1 A_2 A_3}}=\\lambda_2, \\frac{S_{\\triangle A_2 A_3 B_1}}{S_{\\triangle A_1 A_2 A_3}}=\\lambda_1,\n$$\n则\n$$\n\\frac{S_{\\triangle C_1 C_2 C_3}}{S_{\\triangle A_1 A_2 A_3}}=\\left(\\lambda_1+\\lambda_2+\\lambda_3+1\\right)^2 . \\label{eq2}\n$$\n又由假设 $S_{\\triangle A_1 A_2 A_3}>\\frac{1}{2}$ 可知\n$$\n\\begin{aligned}\n\\lambda_1+\\lambda_2+\\lambda_3 & =\\frac{S_{\\triangle A_1 A_2 B_3}+S_{\\triangle A_1 A_3 B_2}+S_{\\triangle A_2 A_3 B_1}}{S_{\\triangle A_1 A_2 A_3}} \\\\\n& \\leqslant \\frac{S(u)-S_{\\triangle A_1 A_2 A_3}}{S_{\\triangle A_1 A_2 A_3}}=\\frac{1-S_{\\triangle A_1 A_2 A_3}}{S_{\\triangle A_1 A_2 A_3}} \\\\\n& <1 . \\label{eq3}\n\\end{aligned}\n$$\n又明显的有\n$$\n\\begin{aligned}\n& \\frac{S\\left(A_1 B_3 A_2 B_1 A_3 B_2\\right)}{S_{\\triangle A_1 A_2 A_3}} \\\\\n= & \\frac{S_{\\triangle A_1 A_2 A_3}+S_{\\triangle B_1 A_2 A_3}+S_{\\triangle B_2 A_1 A_3}+S_{\\triangle B_3 A_1 A_2}}{S_{\\triangle A_1 A_2 A_3}} \\\\\n= & \\lambda_1+\\lambda_2+\\lambda_3+1 . \\label{eq4}\n\\end{aligned}\n$$\n由式\\ref{eq2}、\\ref{eq3}、式\\ref{eq4}可得\n$$\n\\frac{S_{\\triangle C_1 C_2 C_3}}{S\\left(A_1 B_3 A_2 B_1 A_3 B_2\\right)}=\\lambda_1+\\lambda_2+\\lambda_3+1<2 .\n$$\n式\\ref{eq1}得证.\n(2)证完.\n现在我们考虑另一个有趣的问题:一个面积为 1 的凸多边形, 最大的内接三角形的面积有多大? 下面的例子回答了这个问题.", + "remark": "", + "figures": [ + "./images/volume9/figures/fig-c4i12.png", + "./images/volume9/figures/fig-c4i13.png", + "./images/volume9/figures/fig-c4i14.png" + ] +} \ No newline at end of file diff --git a/processed_dataset/proof/2036.json b/processed_dataset/proof/2036.json new file mode 100644 index 0000000000000000000000000000000000000000..968b5c811448a665d6a8b85adf7fe1b9c679c947 --- /dev/null +++ b/processed_dataset/proof/2036.json @@ -0,0 +1,11 @@ +{ + "source_file": "./raw_volume-zh/volume9/chapter4.tex", + "problem_type": "proof", + "problem": "例5. (1) 设 $M$ 是一个面积为 1 的凸多边形, $l$ 是任意给定的直线.\n求证: 存在 $M$ 的一个内接三角形, 它有一条边平行于 $l$, 且面积大于或等于 $\\frac{3}{8}$;\n(2) 如果 $M$ 是一个正六边形, $l$ 是任意给定的一条直线,证明 $M$ 中不存在有一边平行于 $l$ 且面积大于 $\\frac{3}{8} S(M)$ 的内接三角形.", + "solution": "证明:(1)如图(), 作两条平行于 $l$ 的 $M$ 的支撑线, 使得它们构成的带形包含 $M$, 且 $M$ 的顶点 $A$ 和 $B$ 分别在这两条直线上.\n记这两条直线为 $l_1 、 l_2$. 设 $l_1 、 l_2$ 间的宽度为 $d$, 再画三条直线 $l_1^{\\prime}$ 、 $l_0 、 l_2^{\\prime}$ 使得这个带形被分为四个等宽的小带形, 每个小带形的宽为 $\\frac{1}{4} d$.\n假设 $M$ 的边界与 $l_1^{\\prime}$ 相交于 $P$ 和 $Q, l_2^{\\prime}$ 与 $M$ 的边界相交于 $R$ 和 $S$ (因为 $M$ 是凸的, $M$ 不可能有整个边在这两条直线上). 设 $p$ 是 $M$ 的通过点 $P$ 的边 (如果 $P$ 是顶点则可以在两边中任选一条) 所在的直线, $q 、 r$ 和 $s$ 的记号意义类似.\n这时由 $p 、 q 、 l_0 、 l_1$ 为边界形成的梯形 $T_1$ 的面积是 $\\frac{d}{2} \\cdot P Q$. 类似的, 由 $l_0 、 l_2 、 r 、 s$ 为边界形成的梯形 $T_2$ 的面积是 $\\frac{d}{2} \\cdot R S$. 因为 $T_1$ 和 $T_2$ 的并集整个包含 $M$, 故有\n$$\n\\begin{aligned}\nS(M) & \\leqslant S\\left(T_1\\right)+S\\left(T_2\\right) \\\\\n& =\\frac{d}{2} \\cdot P Q+\\frac{d}{2} \\cdot R S=\\frac{d}{2}(P Q+R S) .\n\\end{aligned}\n$$\n现考虑两个三角形 $\\triangle A R S$ 和 $\\triangle B P Q$, 这两个三角形都是 $M$ 的内接三角形, 且\n$$\nS_{\\triangle A R S}=\\frac{1}{2} \\cdot R S \\cdot \\frac{3}{4} d, S_{\\triangle B P Q}=\\frac{1}{2} \\cdot P Q \\cdot \\frac{3}{4} d\n$$\n因此\n$$\n\\begin{aligned}\nS_{\\triangle A R S}+S_{\\triangle B P Q} & =(P Q+R S) \\cdot \\frac{3}{8} d \\\\\n& =\\frac{3}{4}(P Q+R S) \\cdot \\frac{1}{2} d \\\\\n& \\geqslant \\frac{3}{4} S(M)=\\frac{3}{4},\n\\end{aligned}\n$$\n故 $S_{\\triangle A R S} \\geqslant \\frac{3}{8}$ 和 $S_{\\triangle B P Q} \\geqslant \\frac{3}{8}$ 至少有一个成立, 结论得证.\n(2) 设 $M$ 是一个正六边形 $A B C D E F, l / / A B$, 如图(). 设 $\\triangle P Q R$ 是 $M$ 的最大面积的内接三角形且边 $P Q / / A B$. 不妨设 $P$ 和 $Q$ 分别位于 $F A$ 和 $B C$ 上, 则明显的 $R$ 一定位于 $D E$ 上.\n让我们假定正六边形 $M$ 的边长有单位长度, 并记 $A P=B Q=a$, 则\n$$\nP Q=A B+P G+Q H=1+\\frac{a}{2}+\\frac{a}{2}=1+a,\n$$\n且\n$$\nh(P Q R)=R S-A G=\\sqrt{3}-\\frac{a \\sqrt{3}}{2}=(2-a) \\frac{\\sqrt{3}}{2} .\n$$\n因此\n$$\n\\begin{aligned}\nS_{\\triangle P Q R} & =\\frac{1}{2}(1+a)(2-a) \\frac{\\sqrt{3}}{2} \\\\\n& =\\frac{\\sqrt{3}}{4}\\left(2+a-a^2\\right) \\\\\n& =\\frac{\\sqrt{3}}{4}\\left(2+\\frac{1}{4}-\\left(a-\\frac{1}{2}\\right)^2\\right) .\n\\end{aligned}\n$$\n由此知当 $a=\\frac{1}{2}$ 时, $S_{\\triangle P Q R}$ 的面积最大, 最大值为\n$$\n\\left(S_{\\triangle P Q R}\\right)_{\\max }=\\frac{9 \\sqrt{13}}{16},\n$$\n但这个六边形的面积等于\n$$\n6 S(O A B)=6 \\cdot \\frac{\\sqrt{3}}{4}=\\frac{3 \\sqrt{3}}{2},\n$$\n其中 $O$ 是正六边形 $M$ 的中心.\n这说明有一条边平行于给定直线 $l$ 的 $M$ 的最大内接三角形恰为 $\\frac{3}{8} S(M)$,因此结论成立.", + "remark": "", + "figures": [ + "./images/volume9/figures/fig-c4i15.png", + "./images/volume9/figures/fig-c4i16.png" + ] +} \ No newline at end of file diff --git a/processed_dataset/proof/2037.json b/processed_dataset/proof/2037.json new file mode 100644 index 0000000000000000000000000000000000000000..82131d90ee0b8fce7df53fcfd93fa3d01161fcbe --- /dev/null +++ b/processed_dataset/proof/2037.json @@ -0,0 +1,10 @@ +{ + "source_file": "./raw_volume-zh/volume9/chapter5.tex", + "problem_type": "proof", + "problem": "例1. (Erdös-Mordell 不等式) 设 $P$ 为 $\\triangle A B C$ 内任意一点, $P$ 到三边 $B C 、 C A 、 A B$ 的距离分别为 $P D=p 、 P E=q 、 P F=r$, 并记 $P A=x$, $P B=y, P C=z$, 则\n$$\nx+y+z \\geqslant 2(p+q+r),\n$$\n等号成立当且仅当 $\\triangle A B C$ 为正三角形并且 $P$ 为此三角形的中心.", + "solution": "证明 1 如图(), 注意到 $\\angle D P E=180^{\\circ}- \\angle C$, 由余弦定理\n$$\n\\begin{aligned}\nD E & =\\sqrt{p^2+q^2+2 p q \\cos C} \\\\\n& =\\sqrt{p^2+q^2+2 p q \\sin A \\sin B-2 p q \\cos A \\cos B} \\\\\n& =\\sqrt{(p \\sin B+q \\sin A)^2+(p \\cos B-q \\cos A)^2} \\\\\n& \\geqslant \\sqrt{(p \\sin B+q \\sin A)^2} \\\\\n& =p \\sin B+q \\sin A .\n\\end{aligned}\n$$\n又因 $P 、 D 、 C 、 E$ 四点共圆, 线段 $C P$ 为这圆的直径, 故\n$$\nz=\\frac{D E}{\\sin C} \\geqslant\\left(\\frac{\\sin B}{\\sin C}\\right) p+\\left(\\frac{\\sin A}{\\sin C}\\right) q,\n$$\n同理\n$$\n\\begin{aligned}\n& x \\geqslant\\left(\\frac{\\sin B}{\\sin A}\\right) r+\\left(\\frac{\\sin C}{\\sin A}\\right) q, \\\\\n& x \\geqslant\\left(\\frac{\\sin A}{\\sin B}\\right) r+\\left(\\frac{\\sin C}{\\sin A}\\right) p .\n\\end{aligned}\n$$\n三式相加便得\n$$\n\\begin{aligned}\nx+y+z & \\geqslant\\left(\\frac{\\sin B}{\\sin C}+\\frac{\\sin C}{\\sin B}\\right) p+\\left(\\frac{\\sin A}{\\sin C}+\\frac{\\sin C}{\\sin A}\\right) q+\\left(\\frac{\\sin B}{\\sin A}+\\frac{\\sin A}{\\sin B}\\right) r \\\\\n& \\geqslant 2(p+q+r),\n\\end{aligned}\n$$\n得证.", + "remark": "", + "figures": [ + "./images/volume9/figures/fig-c5i1.png" + ] +} \ No newline at end of file diff --git a/processed_dataset/proof/2038.json b/processed_dataset/proof/2038.json new file mode 100644 index 0000000000000000000000000000000000000000..acc48a9e83f0ad97f19606424bc20d31669e8f4b --- /dev/null +++ b/processed_dataset/proof/2038.json @@ -0,0 +1,10 @@ +{ + "source_file": "./raw_volume-zh/volume9/chapter5.tex", + "problem_type": "proof", + "problem": "例1. (Erdös-Mordell 不等式) 设 $P$ 为 $\\triangle A B C$ 内任意一点, $P$ 到三边 $B C 、 C A 、 A B$ 的距离分别为 $P D=p 、 P E=q 、 P F=r$, 并记 $P A=x$, $P B=y, P C=z$, 则\n$$\nx+y+z \\geqslant 2(p+q+r),\n$$\n等号成立当且仅当 $\\triangle A B C$ 为正三角形并且 $P$ 为此三角形的中心.", + "solution": "证明 2 如图(), 过点 $P$ 作直线 $M N$, 使得 $\\angle A M N=\\angle A C B$, 于是 $\\triangle A M N \\backsim \\triangle A C B$.\n从而 $\\frac{A N}{M N}=\\frac{c}{a}, \\frac{A M}{M N}=\\frac{b}{a}$.\n由于 $S_{\\triangle A M N}=S_{\\triangle A M P}+S_{\\triangle A N P}$,\n所以有 $A P \\cdot M N \\geqslant q \\cdot A N+r \\cdot A M$,\n所以 $x=A P \\geqslant q \\cdot \\frac{A N}{M N}+r \\cdot \\frac{A M}{M N}$.\n即\n$$\n\\begin{aligned}\n& x \\geqslant \\frac{c}{a} \\cdot q+\\frac{b}{a} \\cdot r, \\label{eq1}\\\\\n& y \\geqslant \\frac{c}{b} \\cdot p+\\frac{a}{b} \\cdot r, \\label{eq2}\\\\\n& z \\geqslant \\frac{b}{c} \\cdot p+\\frac{a}{c} \\cdot q . \\label{eq3}\n\\end{aligned}\n$$\n将式\\ref{eq1}、\\ref{eq2}、式\\ref{eq3}相加得\n$$\nx+y+z \\geqslant p\\left(\\frac{c}{b}+\\frac{b}{c}\\right)+q\\left(\\frac{c}{a}+\\frac{a}{c}\\right)+r\\left(\\frac{b}{a}+\\frac{a}{b}\\right) \\geqslant 2(p+q+r) .\n$$", + "remark": "", + "figures": [ + "./images/volume9/figures/fig-c5i2.png" + ] +} \ No newline at end of file diff --git a/processed_dataset/proof/2039.json b/processed_dataset/proof/2039.json new file mode 100644 index 0000000000000000000000000000000000000000..6893fc7998861c97b48df418402b3287b974a3da --- /dev/null +++ b/processed_dataset/proof/2039.json @@ -0,0 +1,10 @@ +{ + "source_file": "./raw_volume-zh/volume9/chapter5.tex", + "problem_type": "proof", + "problem": "例1. (Erdös-Mordell 不等式) 设 $P$ 为 $\\triangle A B C$ 内任意一点, $P$ 到三边 $B C 、 C A 、 A B$ 的距离分别为 $P D=p 、 P E=q 、 P F=r$, 并记 $P A=x$, $P B=y, P C=z$, 则\n$$\nx+y+z \\geqslant 2(p+q+r),\n$$\n等号成立当且仅当 $\\triangle A B C$ 为正三角形并且 $P$ 为此三角形的中心.", + "solution": "证明 3 如图(), 作点 $P$ 关于 $\\angle A$ 平分线的对称点 $P^{\\prime}$, 则易知 $P^{\\prime}$ 到 $C A 、 A B$ 的距离分别为 $r$ 、 $q$, 且 $P^{\\prime} A=P A=x$.\n设 $A 、 P^{\\prime}$ 到 $B C$ 的距离分别为 $h_1 、 r_1^{\\prime}$, 则\n$$\nP^{\\prime} A+r_1^{\\prime}=P A+r_1^{\\prime} \\geqslant h_1,\n$$\n两端乘 $a$ 可得\n$$\n\\begin{aligned}\na \\cdot P A+a r_1^{\\prime} & \\geqslant a h_1 \\\\\n& =2 S_{\\triangle A B C} \\\\\n& =a r_1^{\\prime}+c q+b r .\n\\end{aligned}\n$$\n因此同理\n$$\n\\begin{aligned}\n& x \\geqslant \\frac{c}{a} \\cdot q+\\frac{b}{a} \\cdot r, \\\\\n& y \\geqslant \\frac{a}{b} \\cdot r+\\frac{c}{b} \\cdot p, \\\\\n& z \\geqslant \\frac{a}{c} \\cdot q+\\frac{b}{c} \\cdot p .\n\\end{aligned}\n$$\n将这三个不等式相加可得\n$$\nx+y+z=\\left(\\frac{c}{b}+\\frac{b}{c}\\right) p+\\left(\\frac{c}{a}+\\frac{a}{c}\\right) q+\\left(\\frac{b}{a}+\\frac{a}{b}\\right) r \\geqslant 2(p+q+r) .\n$$", + "remark": "", + "figures": [ + "./images/volume9/figures/fig-c5i3.png" + ] +} \ No newline at end of file diff --git a/processed_dataset/proof/2040.json b/processed_dataset/proof/2040.json new file mode 100644 index 0000000000000000000000000000000000000000..37d03678bb591a5bd3bd844377a51391808279d1 --- /dev/null +++ b/processed_dataset/proof/2040.json @@ -0,0 +1,10 @@ +{ + "source_file": "./raw_volume-zh/volume9/chapter5.tex", + "problem_type": "proof", + "problem": "例1. (Erdös-Mordell 不等式) 设 $P$ 为 $\\triangle A B C$ 内任意一点, $P$ 到三边 $B C 、 C A 、 A B$ 的距离分别为 $P D=p 、 P E=q 、 P F=r$, 并记 $P A=x$, $P B=y, P C=z$, 则\n$$\nx+y+z \\geqslant 2(p+q+r),\n$$\n等号成立当且仅当 $\\triangle A B C$ 为正三角形并且 $P$ 为此三角形的中心.", + "solution": "要点是将三角形的高转化为内角平分线来处理,并运用嵌人不等式.\n证明 4 如图(), 设 $\\angle B P C=2 \\alpha, \\angle C P A=2 \\beta, \\angle A P B=2 \\gamma$, 设它们的内角平分线长分别是 $w_a 、 w_b 、 w_c$, 则我们只需证明更强的不等式\n$$\nx+y+z \\geqslant 2\\left(w_a+w_b+w_c\\right) .\n$$\n事实上,注意到内角平分线公式有\n$$\nw_a=\\frac{2 y z}{y+z} \\cos \\frac{1}{2} \\angle B P C \\leqslant \\sqrt{y z} \\cos \\alpha,\n$$\n同理\n$$\n\\begin{aligned}\n& w_b \\leqslant \\sqrt{x z} \\cos \\beta, \\\\\n& w_c \\leqslant \\sqrt{x y} \\cos \\gamma .\n\\end{aligned}\n$$\n由于 $\\alpha+\\beta+\\gamma=\\pi$, 所以由嵌人不等式可得\n$$\n\\begin{aligned}\n2\\left(w_a+w_b+w_c\\right) & \\leqslant 2(\\sqrt{y z} \\cos \\alpha+\\sqrt{x z} \\cos \\beta+\\sqrt{x y} \\cos \\gamma) \\\\\n& \\leqslant x+y+z .\n\\end{aligned}\n$$\n证完.", + "remark": "", + "figures": [ + "./images/volume9/figures/fig-c5i4.png" + ] +} \ No newline at end of file diff --git a/processed_dataset/proof/2041.json b/processed_dataset/proof/2041.json new file mode 100644 index 0000000000000000000000000000000000000000..028922a595b008ddb8c6557fa9536c9797569eae --- /dev/null +++ b/processed_dataset/proof/2041.json @@ -0,0 +1,10 @@ +{ + "source_file": "./raw_volume-zh/volume9/chapter5.tex", + "problem_type": "proof", + "problem": "例1. (Erdös-Mordell 不等式) 设 $P$ 为 $\\triangle A B C$ 内任意一点, $P$ 到三边 $B C 、 C A 、 A B$ 的距离分别为 $P D=p 、 P E=q 、 P F=r$, 并记 $P A=x$, $P B=y, P C=z$, 则\n$$\nx+y+z \\geqslant 2(p+q+r),\n$$\n等号成立当且仅当 $\\triangle A B C$ 为正三角形并且 $P$ 为此三角形的中心.", + "solution": "证明 5 如图(), 过 $D 、 E$ 作 $D T_1 \\perp F P$ 于 $T_1, E T_2 \\perp F P$ 于 $T_2$. 由\n$$\nD E \\geqslant D T_1+E T_2, D T_1=p \\sin B, E T_2=q \\sin A,\n$$\n可得\n$$\n\\begin{aligned}\nz & =\\frac{D E}{\\sin C} \\geqslant \\frac{p \\sin B+q \\sin A}{\\sin C} \\\\\n& =p \\frac{\\sin B}{\\sin C}+q \\frac{\\sin A}{\\sin C},\n\\end{aligned}\n$$\n所以\n$$\n\\begin{aligned}\n& x+y+z \\\\\n= & P A+P B+P C \\\\\n\\geqslant & \\left(p \\frac{\\sin B}{\\sin C}+q \\frac{\\sin A}{\\sin C}\\right)+\\left(q \\frac{\\sin C}{\\sin A}+r \\frac{\\sin B}{\\sin A}\\right)+\\left(r \\frac{\\sin A}{\\sin B}+p \\frac{\\sin C}{\\sin B}\\right) \\\\\n= & p\\left(\\frac{\\sin B}{\\sin C}+\\frac{\\sin C}{\\sin B}\\right)+q\\left(\\frac{\\sin A}{\\sin C}+\\frac{\\sin C}{\\sin A}\\right)+r\\left(\\frac{\\sin B}{\\sin A}+\\frac{\\sin A}{\\sin B}\\right) \\\\\n\\geqslant & 2(p+q+r) .\n\\end{aligned}\n$$\n证完.", + "remark": "注:关于 Erdös-Mordell 不等式研究已有众多成果, 其中平面上的推广较为简单, 很早由 N. Ozeki 和 H. Vigler 完成, 后又被其他人多次重新发现.\nErdös-Mordell 不等式在空间,特别是 $n$ 维空间的推广是一个困难的问题,据我所知至今还未得到理想的结果.", + "figures": [ + "./images/volume9/figures/fig-c5i5.png" + ] +} \ No newline at end of file diff --git a/processed_dataset/proof/2042.json b/processed_dataset/proof/2042.json new file mode 100644 index 0000000000000000000000000000000000000000..5d3e5210c320b345383a35075a953aa92a9a1a5f --- /dev/null +++ b/processed_dataset/proof/2042.json @@ -0,0 +1,10 @@ +{ + "source_file": "./raw_volume-zh/volume9/chapter5.tex", + "problem_type": "proof", + "problem": "例2. 设 $\\triangle A B C$ 的三边为 $a 、 b 、 c$, 则\n$$\nh_a+m_b+t_c \\leqslant \\frac{\\sqrt{3}}{2}(a+b+c),\n$$\n其中 $h_a 、 m_b 、 t_c$ 分别表示边 $B C 、 A C 、 A B$ 上的高、中线和内角平分线.", + "solution": "证明:如图(), 将高线 $h_a$ 转化为内角平分线 $t_a$ 来考虑.\n为此仅需证明更强的不等式\n$$\nt_a+m_b+t_c \\leqslant \\frac{\\sqrt{3}}{2}(a+b+c) . \\label{eq1}\n$$\n要证式\\ref{eq1}, 只需证明局部不等式\n$$\nm_b+2 t_a \\leqslant \\frac{\\sqrt{3}}{2}(b+2 c) . \\label{eq2}\n$$\n事实上,若式\\ref{eq2}成立, 则类似的有\n$$\nm_b+2 t_c \\leqslant \\frac{\\sqrt{3}}{2}(b+2 a) . \\label{eq3}\n$$\n\\ref{eq2}、式\\ref{eq3}相加便是式\\ref{eq1}.\n下证式\\ref{eq2}.\n由内角平分线公式易知\n$$\nt_a^2=\\frac{4}{(b+c)^2} \\cdot b c p(p-a) \\leqslant p(p-a)=\\frac{1}{4}\\left((b+c)^2-a^2\\right), \\label{eq4}\n$$\n又\n$$\nm_b^2=\\frac{1}{4}\\left(2 a^2+2 c^2-b^2\\right) . \\label{eq5}\n$$\n因此由 Cauchy 不等式及\\ref{eq4}、\\ref{eq5}可得\n$$\n\\begin{aligned}\nm_b+2 t_a & \\leqslant \\sqrt{3\\left(m_b^2+2 t_a^2\\right)} \\\\\n& \\leqslant \\sqrt{\\frac{3}{4}\\left(2 a^2+2 c^2-b^2+2(b+c)^2-2 a^2\\right)} \\\\\n& =\\frac{\\sqrt{3}}{2}(b+2 c) .\n\\end{aligned}\n$$\n式\\ref{eq2}得证,从而问题得证.", + "remark": "注:(1) 仔细观察例 1 和例 2 的各种证法,我们认为将整体的线性几何不等式归结为局部的线性几何不等式是一个共同的技巧.\n例 1 的各种证法的目标都在于寻找局部不等式\n$$\nx \\geqslant \\lambda_1 q+\\lambda_2 r,\n$$\n其中 $\\lambda_1 、 \\lambda_2$ 是与动点 $P$ 无关的几何量, 而例 2 却是通过转化为局部不等式\n$$\nm_b+2 t_a \\leqslant \\frac{\\sqrt{3}}{2}(b+2 c)\n$$\n来达到目标.\n(2)例 2 用内角平分线代替高线来加强命题的技巧已在例 1 的证法 4 中应用过,还将在本书最后一章\"四面体的不等式\"例 5 的证明中再次用到.\n这种主动加强命题的技巧是十分有用的.", + "figures": [ + "./images/volume9/figures/fig-c5i6.png" + ] +} \ No newline at end of file diff --git a/processed_dataset/proof/2043.json b/processed_dataset/proof/2043.json new file mode 100644 index 0000000000000000000000000000000000000000..808940fe05491fc9b8b52efd13ba8f3cb6aa9ff6 --- /dev/null +++ b/processed_dataset/proof/2043.json @@ -0,0 +1,10 @@ +{ + "source_file": "./raw_volume-zh/volume9/chapter5.tex", + "problem_type": "proof", + "problem": "例3. 给定一个锐角 $\\triangle A B C$. 设 $h_a 、 h_b 、 h_c$ 分别表示 $\\triangle A B C$ 的三边 $B C$ 、 $C A 、 A B$ 上的高, $s$ 表示半周长.\n证明\n$$\n\\sqrt{3} \\cdot \\max \\left\\{h_a, h_b, h_c\\right\\} \\geqslant s .\n$$", + "solution": "证明:如果三角形 $\\triangle A B C$ 是正三角形,等号成立.\n下面证明: 如果 $\\triangle A B C$ 不是正三角形, 则问题可化归为等腰三角形的情形来证明.\n事实上, 如果 $\\angle A \\geqslant \\angle B>\\angle C$, 则 $\\angle A>\\frac{\\pi}{3}$, 且\n$$\nh_c>h_b \\geqslant h_a .\n$$\n设 $h$ 表示最大高 $h_c$, 如图(),延长 $\\triangle A B C$ 的最短边 $A B$ 到 $D$ 使得 $A D=A C$. 连接 $C D$. 因此如果 $\\sqrt{3} h \\geqslant s$ 对等腰 $\\triangle A C D$ 成立,则对一般的锐角 $\\triangle A B C$ 也成立.\n现在等腰 $\\triangle A C D$ 中证明 $\\sqrt{3} h \\geqslant s$.\n因为 $s=A C+\\frac{1}{2} C D, C D=2 A C \\cdot \\sin \\frac{A}{2}, h_c= A C \\cdot \\sin A$, 因此 $\\sqrt{3} h \\geqslant s$ 等价于\n$$\n\\sqrt{3} \\sin A \\geqslant 1+\\sin \\frac{A}{2}\\left(\\frac{\\pi}{3}0, x+1>0,6 x^2-3 x-1 \\leqslant 0$, 故 式\\ref{eq2} 成立,得证.", + "remark": "注:本例将一般三角形化归为等腰三角形证题的技巧值得注意, 这样的化归有时可大大简化问题的处理.", + "figures": [ + "./images/volume9/figures/fig-c5i7.png" + ] +} \ No newline at end of file diff --git a/processed_dataset/proof/2044.json b/processed_dataset/proof/2044.json new file mode 100644 index 0000000000000000000000000000000000000000..596de1be0b63d7ba9df56595777fb40b1b2241a7 --- /dev/null +++ b/processed_dataset/proof/2044.json @@ -0,0 +1,10 @@ +{ + "source_file": "./raw_volume-zh/volume9/chapter5.tex", + "problem_type": "proof", + "problem": "例4. (Zirakzadeh 不等式) 设 $P 、 Q 、 R$ 分别位于 $\\triangle A B C$ 的三条边 $B C 、 C A 、 A B$ 上且将三角形的周长三等分, 则\n$$\nQ R+R P+P Q \\geqslant \\frac{1}{2}(a+b+c) .\n$$", + "solution": "证明:.下面用投影方法产生局部的线性几何不等式.\n如图(), 从 $Q 、 R$ 分别向直线 $B C$ 引垂线, 垂足分别记为 $M 、 N$, 则\n$$\nQ R \\geqslant M N=a-(B R \\cdot \\cos B+C Q \\cdot \\cos C),\n$$\n同理有\n$$\n\\begin{aligned}\n& R P \\geqslant b-(C P \\cdot \\cos C+A R \\cdot \\cos A), \\\\\n& P Q \\geqslant c-(A Q \\cdot \\cos A+B P \\cdot \\cos B) .\n\\end{aligned}\n$$\n将三式相加, 并注意到\n$$\nA Q+A R=B R+B P=C P+C Q=\\frac{1}{3}(a+b+c),\n$$\n即得\n$$\nQ R+R P+P Q \\geqslant \\frac{1}{3}(a+b+c)(3-\\cos A-\\cos B-\\cos C),\n$$\n又\n$$\n\\cos A+\\cos B+\\cos C \\leqslant \\frac{3}{2}\n$$\n立得\n$$\nQ R+R P+P Q \\geqslant \\frac{1}{2}(a+b+c) .\n$$", + "remark": "注:上面的优美解法是杨学枝先生给出的.\n这个问题曾在国内引起了较为广泛的讨论.\n下面的例 5 是王振先生发现并证明的一个结果, 难度稍大.", + "figures": [ + "./images/volume9/figures/fig-c5i8.png" + ] +} \ No newline at end of file diff --git a/processed_dataset/proof/2045.json b/processed_dataset/proof/2045.json new file mode 100644 index 0000000000000000000000000000000000000000..3c8cddcb3b521f5418f9079b82fc22ded537a2e2 --- /dev/null +++ b/processed_dataset/proof/2045.json @@ -0,0 +1,10 @@ +{ + "source_file": "./raw_volume-zh/volume9/chapter5.tex", + "problem_type": "proof", + "problem": "例5. 设 $I 、 G$ 分别是 $\\triangle A B C$ 的内心和重心,求证\n$$\nA I+B I+C I \\leqslant A G+B G+C G .\n$$", + "solution": "证明:令 $B C=a, A C=b, A B=c$, 不妨设 $a \\geqslant b \\geqslant c$, 如图(). 下面我们证明 $G$ 一定落在 $\\triangle B I C$ 内或边界上.\n先证 $G$ 不落在 $\\triangle A I B$ 内, 若不然, 假设 $G$ 落在 $\\triangle A I B$ 内, 则有\n$$\nS_{\\triangle A B G}A L$, 而 $A L=B L, \\frac{A T}{B T}=\\frac{b}{a} \\leqslant 1$, 所以 $A T \\leqslant \\frac{1}{2} A B=A L$,矛盾.\n因此 $G$ 落在 $\\triangle B I C$ 内或边界上, 且可证 $G$ 在 $A I$ 右侧.\n设 $\\angle A I G$ 的补角为 $\\theta$, 则 $0 \\leqslant \\theta \\leqslant \\frac{A+C}{2}$. 由此可知 $A G \\geqslant A I+G I \\cos \\theta$.\n同理\n$$\n\\begin{aligned}\n& B G \\geqslant B I+G I \\cos \\left(90^{\\circ}+\\frac{C}{2}-\\theta\\right), \\\\\n& C G \\geqslant C I-G I \\cos \\left(\\frac{A+C}{2}-\\theta\\right) .\n\\end{aligned}\n$$\n因此\n$$\n\\begin{aligned}\n& A G+B G+C G-(A I+B I+C I) \\\\\n\\geqslant & G I\\left(\\cos \\theta+\\cos \\left(90^{\\circ}+\\frac{C}{2}-\\theta\\right)-\\cos \\left(\\frac{A+C}{2}-\\theta\\right)\\right) \\\\\n= & G I\\left(\\cos \\theta-2 \\sin \\frac{B+C}{4} \\cos \\left(\\frac{B-C}{4}+\\theta\\right)\\right) .\n\\end{aligned}\n$$\n由于 $\\frac{B+C}{4} \\leqslant 30^{\\circ}, \\theta \\leqslant \\frac{B-C}{4}+\\theta<90^{\\circ}$, 所以\n$$\n\\cos \\theta-2 \\sin \\frac{B+C}{4} \\cos \\left(\\frac{B-C}{4}+\\theta\\right) \\geqslant \\cos \\theta-\\cos \\left(\\frac{B-C}{4}+\\theta\\right) \\geqslant 0,\n$$\n故 $A G+B G+C G \\geqslant A I+B I+C I$.", + "remark": "", + "figures": [ + "./images/volume9/figures/fig-c5i9.png" + ] +} \ No newline at end of file diff --git a/processed_dataset/proof/2046.json b/processed_dataset/proof/2046.json new file mode 100644 index 0000000000000000000000000000000000000000..f94c4e9d17fd0234e47f8c406b215599980fe4f6 --- /dev/null +++ b/processed_dataset/proof/2046.json @@ -0,0 +1,8 @@ +{ + "source_file": "./raw_volume-zh/volume9/chapter6.tex", + "problem_type": "proof", + "problem": "例1. 设 $P$ 是 $\\triangle A B C$ 所在平面上任一点,求证:\n$$\na \\cdot P B \\cdot P C+b \\cdot P C \\cdot P A+c \\cdot P A \\cdot P B \\geqslant a b c .\n$$", + "solution": "证明:视 $\\triangle A B C$ 所在平面为复平面, 设 $P 、 A 、 B 、 C$ 分别对应着复数 $z$ 、 $z_1, z_2, z_3$, 令\n$$\nf(z)=\\frac{\\left(z-z_2\\right)\\left(z-z_3\\right)}{\\left(z_1-z_2\\right)\\left(z_1-z_3\\right)}+\\frac{\\left(z-z_3\\right)\\left(z-z_1\\right)}{\\left(z_2-z_3\\right)\\left(z_2-z_1\\right)}+\\frac{\\left(z-z_1\\right)\\left(z-z_2\\right)}{\\left(z_3-z_1\\right)\\left(z_3-z_2\\right)},\n$$\n则 $f(z)$ 是关于 $z$ 的二次多项式,且易见\n$$\nf\\left(z_1\\right)=f\\left(z_2\\right)=f\\left(z_3\\right)=1,\n$$\n故 $f(z) \\equiv 1$. 因此\n$$\n\\begin{aligned}\n& \\frac{P B \\cdot P C}{b c}+\\frac{P C \\cdot P A}{c a}+\\frac{P A \\cdot P B}{a b} \\\\\n= & \\left|\\frac{\\left(z-z_2\\right)\\left(z-z_3\\right)}{\\left(z_1-z_2\\right)\\left(z_1-z_3\\right)}\\right|+\\left|\\frac{\\left(z-z_3\\right)\\left(z-z_1\\right)}{\\left(z_2-z_3\\right)\\left(z_2-z_1\\right)}\\right|+\\left|\\frac{\\left(z-z_1\\right)\\left(z-z_2\\right)}{\\left(z_3-z_1\\right)\\left(z_3-z_2\\right)}\\right| \\\\\n\\geqslant & |f(z)|=1 .\n\\end{aligned}\n$$\n由此立得所证不等式.", + "remark": "", + "figures": [] +} \ No newline at end of file diff --git a/processed_dataset/proof/2047.json b/processed_dataset/proof/2047.json new file mode 100644 index 0000000000000000000000000000000000000000..9848abe2c6f7dea6d348c747d104b502b9e81fb8 --- /dev/null +++ b/processed_dataset/proof/2047.json @@ -0,0 +1,11 @@ +{ + "source_file": "./raw_volume-zh/volume9/chapter6.tex", + "problem_type": "proof", + "problem": "例2. 设 $\\triangle A B C$ 和 $\\triangle A^{\\prime} B^{\\prime} C^{\\prime}$ 是同一个平面上的两个正三角形, 且顶点排列方向相同, 求证: 三条线段 $A A^{\\prime} 、 B B^{\\prime} 、 C C^{\\prime}$ 中任何两个之和大于或等于第三个.", + "solution": "证明:如图(), 对顶点排列方向相同的两个相似三角形 $\\triangle A B C$ 和 $\\triangle A^{\\prime} B^{\\prime} C^{\\prime}$ 总有一恒等式\n$$\n\\begin{aligned}\n& \\left(z_1^{\\prime}-z_1\\right)\\left(z_2-z_3\\right)+\\left(z_2^{\\prime}-z_2\\right)\\left(z_3-z_1\\right)+ \\\\\n& \\left(z_3^{\\prime}-z_3\\right)\\left(z_1-z_2\\right)=0, \\label{eq1}\n\\end{aligned}\n$$\n其中 $z_1 、 z_2 、 z_3$ 分别是 $A 、 B 、 C$ 对应的复数; $z_1^{\\prime}$ 、 $z_2^{\\prime} 、 z_3^{\\prime}$ 分别是 $A^{\\prime} 、 B^{\\prime} 、 C^{\\prime}$ 对应的复数.\n由复数模的性质从式\\ref{eq1}可得\n$$\n\\left|z_1^{\\prime}-z_1\\right| \\cdot\\left|z_2-z_3\\right|+\\left|z_2^{\\prime}-z_2\\right| \\cdot\\left|z_3-z_1\\right| \\geqslant\\left|\\left(z_3^{\\prime}-z_3\\right)\\left(z_1-z_2\\right)\\right| .\n$$\n注意到 $\\triangle A B C$ 是正三角形, 即\n$$\n\\left|z_2-z_3\\right|=\\left|z_3-z_1\\right|=\\left|z_1-z_2\\right|,\n$$\n故有\n$$\n\\left|z_1^{\\prime}-z_1\\right|+\\left|z_2^{\\prime}-z_2\\right| \\geqslant\\left|z_3^{\\prime}-z_3\\right|\n$$\n这就是\n$$\nA A^{\\prime}+B B^{\\prime} \\geqslant C C^{\\prime}\n$$\n同理可证另外的两个不等式.\n现回忆平面几何中一个简单的命题:三个正数 $a 、 b 、 c$ 构成一个三角形三边的充要条件是存在正数 $x 、 y 、 z$ 使得 $a=y+z, b=x+z, c=x+y$. (这个结论的充分性可直接验证,必要性可通过如图() 中的分解看出.)\n依据这个命题,关于三角形的几何不等式总可以通过代换 $x=-a+b+c, y=a-b+c, z=a+b-c$ 将问题转化为涉及正数 $x 、 y 、 z$ 的不等式.\n利用正数代换的一个十分典型的问题是第 24 届 IMO 试题:\n在 $\\triangle A B C$ 中, 求证:\n$$\nb^2 c(b-c)+c^2 a(c-a)+a^2 b(a-b) \\geqslant 0 .\n$$\n这个问题的一种简洁证法是利用正数代换转化为 $x 、 y 、 z$ 的不等式\n$$\n\\frac{x^2}{y}+\\frac{y^2}{z}+\\frac{z^2}{x} \\geqslant x+y+z,\n$$\n从而用一下 Cauchy 不等式便可.", + "remark": "", + "figures": [ + "./images/volume9/figures/fig-c6i1.png", + "./images/volume9/figures/fig-c6i2.png" + ] +} \ No newline at end of file diff --git a/processed_dataset/proof/2048.json b/processed_dataset/proof/2048.json new file mode 100644 index 0000000000000000000000000000000000000000..0e0bc8b28cef40893bd0a2c384d29aed54a23208 --- /dev/null +++ b/processed_dataset/proof/2048.json @@ -0,0 +1,8 @@ +{ + "source_file": "./raw_volume-zh/volume9/chapter6.tex", + "problem_type": "proof", + "problem": "例3. 设 $r_a 、 r_b 、 r_c$ 分别为 $\\triangle A B C$ 的三边 $a 、 b 、 c$ 相应的旁切圆半径, 求证:\n$$\n\\frac{a^2}{r_b^2+r_c^2}+\\frac{b^2}{r_c^2+r_a^2}+\\frac{c^2}{r_a^2+r_b^2} \\geqslant 2\n$$", + "solution": "证明:作正数代换\n$$\n\\begin{gathered}\nx=-a+b+c, \\\\\ny=a-b+c, \\\\\nz=a+b-c,\n\\end{gathered}\n$$\n则 $x, y, z>0$. 注意到\n$$\n\\begin{gathered}\nS_{\\triangle A B C}=\\frac{1}{4} \\sqrt{(x+y+z) x y z}, \\\\\nr_a=\\frac{2 S_{\\triangle A B C}}{b+c-a}=\\frac{1}{2 x} \\sqrt{(x+y+z) x y z},\n\\end{gathered}\n$$\n等等, 通过计算, 原不等式等价变为下面的代数不等式\n$$\n\\frac{y^2 z^2(y+z)^2}{y^2+z^2}+\\frac{z^2 x^2(z+x)^2}{z^2+x^2}+\\frac{x^2 y^2(x+y)^2}{x^2+y^2} \\geqslant 2 x y z(x+y+z) . \\label{eq1}\n$$\n为证式\\ref{eq1}, 只需证明下面有趣的局部不等式\n$$\n\\frac{y^2 z^2(y+z)^2}{y^2+z^2} \\geqslant \\frac{2 x y z(x+y+z) y^2 z^2}{x^2 y^2+y^2 z^2+z^2 x^2} . \\label{eq2}\n$$\n事实上, 如果\\ref{eq2}式成立, 将这样的三个不等式相加便得所证结果.\n下证式\\ref{eq2}.\n$$\n\\begin{aligned}\n式\\ref{eq2} & \\Leftrightarrow(y+z)^2\\left(x^2 y^2+y^2 z^2+z^2 x^2\\right) \\geqslant 2 x y z(x+y+z)\\left(y^2+z^2\\right), \\\\\n& \\Leftrightarrow\\left(y^2+z^2\\right) x^2(y+z)^2+y^2 z^2(y+z)^2 \\geqslant 2 x y z(x+y+z)\\left(y^2+z^2\\right), \\\\\n& \\Leftrightarrow\\left(y^2+z^2\\right)^2 x^2+y^2 z^2(y+z)^2 \\geqslant 2 x y z(y+z)\\left(y^2+z^2\\right), \\\\\n& \\Leftrightarrow\\left[\\left(y^2+z^2\\right) x-y z(y+z)\\right]^2 \\geqslant 0,\n\\end{aligned}\n$$\n得证.", + "remark": "", + "figures": [] +} \ No newline at end of file diff --git a/processed_dataset/proof/2049.json b/processed_dataset/proof/2049.json new file mode 100644 index 0000000000000000000000000000000000000000..010ad9b07de11195bec17f6d12b0d48d0f4336da --- /dev/null +++ b/processed_dataset/proof/2049.json @@ -0,0 +1,10 @@ +{ + "source_file": "./raw_volume-zh/volume9/chapter6.tex", + "problem_type": "proof", + "problem": "例4. 设 $P_i\\left(x_i, y_i\\right)\\left(i=1,2,3 ; x_1), 设直线 $O P_3 、 O P_2$ 的倾斜角为 $\\theta_3 、 \\theta_2, \\angle P_2 P_1 P_3=\\alpha$, 则 $\\theta_3- \\theta_2= \\pm \\alpha$, 从而\n$$\n\\begin{aligned}\n\\frac{1}{\\left|x_2-x_3\\right|}\\left|\\frac{y_3}{x_3}-\\frac{y_2}{x_2}\\right| & =\\frac{1}{\\left|x_2-x_3\\right|}\\left|\\tan \\theta_3-\\tan \\theta_2\\right| \\\\\n& =\\frac{1}{\\left|x_2-x_3\\right|}\\left|\\frac{\\sin \\left(\\theta_3-\\theta_2\\right)}{\\cos \\theta_3 \\cos \\theta_2}\\right| \\\\\n& =\\frac{\\sin \\alpha}{\\left|x_2-x_3\\right|}\\left|\\frac{1}{\\cos \\theta_3 \\cos \\theta_2}\\right| \\\\\n& >\\frac{\\sin \\alpha}{\\left|x_2-x_3\\right|} \\geqslant \\frac{\\sin \\alpha}{P_2 P_3} \\\\\n& =\\frac{1}{2 R}, \\label{eq1}\n\\end{aligned}\n$$\n从而原不等式得证.\n在上面的证明中, 若 $y_1=y_3=x_1=0$, 并且 $y_2 \\rightarrow 0$, 则 $\\cos \\theta_3=1$, $\\cos \\theta_2 \\rightarrow 1$. 此时 式\\ref{eq1}的左边 $\\rightarrow \\frac{1}{2 R}$, 故 2 是最优的.", + "remark": "", + "figures": [ + "./images/volume9/figures/fig-c6i3.png" + ] +} \ No newline at end of file diff --git a/processed_dataset/proof/2050.json b/processed_dataset/proof/2050.json new file mode 100644 index 0000000000000000000000000000000000000000..5c4fca3920644e2e8064bf0d90730606acd7101b --- /dev/null +++ b/processed_dataset/proof/2050.json @@ -0,0 +1,10 @@ +{ + "source_file": "./raw_volume-zh/volume9/chapter6.tex", + "problem_type": "proof", + "problem": "例5. 设 $P$ 是锐角 $\\triangle A B C$ 所在平面上任一点, $u 、 v 、 w$ 分别为点 $P$ 到 $A$ 、 $B 、 C$ 的距离.\n求证:\n$$\nu^2 \\tan A+v^2 \\tan B+w^2 \\tan C \\geqslant 4 \\Delta,\n$$\n并指出等号成立的条件, 其中 $\\triangle$ 为 $\\triangle A B C$ 的面积.", + "solution": "证明:如图(), 取 $B C$ 所在的直线为 $X$ 轴, 过 $A$ 的高线所在的直线为 $Y$ 轴, 建立平面直角坐标系.\n设 $A 、 B 、 C$ 的坐标分别为 $(0, a) 、(-b, 0)$ 、 $(c, 0)$ (这里 $a, b, c>0)$, 于是\n$$\n\\tan A=-\\tan (B+C)=\\frac{a(b+c)}{a^2-b c} .\n$$\n由 $\\angle A$ 为锐角知 $a^2-b c>0$.\n现设点 $P$ 的坐标为 $(x, y)$, 则\n$$\n\\begin{aligned}\n& u^2 \\tan A+v^2 \\tan B+w^2 \\tan C \\\\\n= & {\\left[x^2+(y-a)^2\\right] \\frac{a(b+c)}{a^2-b c}+\\frac{a}{b}\\left[(x+b)^2+y^2\\right]+\\frac{a}{c}\\left[(x-c)^2+y^2\\right] } \\\\\n= & \\left(x^2+y^2+a^2-2 a y\\right) \\frac{a(b+c)}{a^2-b c}+\\frac{a(b+c)}{b c}\\left(x^2+y^2+b c\\right) \\\\\n= & \\frac{a(b+c)}{b c\\left(a^2-b c\\right)}\\left[a^2 x^2+(a y-b c)^2+2 b c\\left(a^2-b c\\right)\\right] \\\\\n\\geqslant & \\frac{a(b+c)}{b c\\left(a^2-b c\\right)} \\cdot 2 b c\\left(a^2-b c\\right) \\\\\n= & 2 a(b+c)=4 \\Delta .\n\\end{aligned}\n$$\n从上面的证明过程可看出, 等号成立的充要条件是 $x=0$ 且 $y=\\frac{b c}{a}$, 即点 $P$ 为 $\\triangle A B C$ 的垂心 $\\left(0, \\frac{b c}{a}\\right)$.", + "remark": "", + "figures": [ + "./images/volume9/figures/fig-c6i4.png" + ] +} \ No newline at end of file diff --git a/processed_dataset/proof/2051.json b/processed_dataset/proof/2051.json new file mode 100644 index 0000000000000000000000000000000000000000..e9dc48d63f07ceb0eb5e1a8d6a034c4401cc5f54 --- /dev/null +++ b/processed_dataset/proof/2051.json @@ -0,0 +1,8 @@ +{ + "source_file": "./raw_volume-zh/volume9/chapter6.tex", + "problem_type": "proof", + "problem": "例6. 设 $\\triangle A B C$ 和 $\\triangle A^{\\prime} B^{\\prime} C^{\\prime}$ 的三边分别为 $a 、 b 、 c$ 及 $a^{\\prime} 、 b^{\\prime} 、 c^{\\prime}$, 面积分别为 $F$ 和 $F^{\\prime}$. 设\n$$\n\\mu=\\min \\left\\{\\frac{a^2}{a^{\\prime 2}}, \\frac{b^2}{b^{\\prime 2}}, \\frac{c^2}{c^{\\prime 2}}\\right\\}, v=\\max \\left\\{\\frac{a^2}{a^{\\prime 2}}, \\frac{b^2}{b^{\\prime 2}}, \\frac{c^2}{c^{\\prime 2}}\\right\\},\n$$\n则对 $\\mu \\leqslant \\lambda \\leqslant v$ 有\n$$\nH \\geqslant 8\\left(\\lambda F^{\\prime 2}+\\frac{1}{\\lambda} F^2\\right),\n$$\n其中 $H=a^{\\prime 2}\\left(-a^2+b^2+c^2\\right)+b^{\\prime 2}\\left(a^2-b^2+c^2\\right)+c^{\\prime 2}\\left(a^2+b^2-c^2\\right)$.", + "solution": "证明:由三角形面积的海伦(Heron)公式有\n$$\n\\begin{gathered}\n16 F^2=\\left(a^2+b^2+c^2\\right)^2-2\\left(a^4+b^4+c^4\\right), \\\\\n16 F^{\\prime 2}=\\left(a^{\\prime 2}+b^{\\prime 2}+c^{\\prime 2}\\right)^2-2\\left(a^{\\prime 4}+b^{\\prime 4}+c^{\\prime 4}\\right) .\n\\end{gathered}\n$$\n令 $D_1=\\sqrt{\\lambda} a^{\\prime 2}-\\frac{a^2}{\\sqrt{\\lambda}}, D_2=\\sqrt{\\lambda} b^{\\prime 2}-\\frac{b^2}{\\sqrt{\\lambda}}, D_3=\\sqrt{\\lambda} c^{\\prime 2}-\\frac{c^2}{\\sqrt{\\lambda}}$, 则有恒等式\n$$\nH-8\\left(\\lambda F^{\\prime 2}+\\frac{1}{\\lambda} F^2\\right)=\\frac{1}{2} D_1^2-D_1\\left(D_2+D_3\\right)+\\frac{1}{2}\\left(D_2-D_3\\right)^2 . \\label{eq1}\n$$\n注意到当 $\\lambda=\\mu=\\frac{a^2}{a^{\\prime 2}}$ 时, $D_1=0$, 因此由 式\\ref{eq1} 得\n$$\nH-8\\left(\\frac{a^2}{a^{\\prime 2}} F^{\\prime 2}+\\frac{a^{\\prime 2}}{a^2} F^2\\right)=\\frac{1}{2}\\left[\\frac{a}{a}\\left(b^{\\prime 2}-c^{\\prime 2}\\right)-\\frac{a^{\\prime}}{a}\\left(b^2-c^2\\right)\\right]^2 \\geqslant 0,\n$$\n即\n$$\n\\begin{aligned}\n& H \\geqslant 8\\left(\\mu F^2+\\frac{1}{\\mu} F^2\\right), \\label{eq2}\\\\\n& H \\geqslant 8\\left(u F^{\\prime 2}+\\frac{1}{v} F^2\\right) . \\label{eq3}\n\\end{aligned}\n$$\n同理对给定的 $\\lambda \\in[\\mu, v]$, 可令\n$$\n\\lambda=\\theta \\mu+(1-\\theta)_\\nu, 0 \\leqslant \\theta \\leqslant 1 .\n$$\n因此将式\\ref{eq2}、\\ref{eq3}两式分别乘 $\\theta$ 和 $1-\\theta$ 后相加可得\n$$\nH \\geqslant 8\\left[\\lambda F^{\\prime 2}+\\left(\\frac{\\theta}{\\mu}+\\frac{1-\\theta}{v}\\right) F^2\\right], \\label{eq4}\n$$\n易知\n$$\n\\frac{\\theta}{\\mu}+\\frac{1-\\theta}{v} \\geqslant \\frac{1}{\\lambda}\n$$\n因此由式\\ref{eq4}便得\n$$\nH \\geqslant 8\\left(\\lambda F^{\\prime 2}+\\frac{1}{\\lambda} F^2\\right)\n$$", + "remark": "注:上例证明中的代数恒等式\\ref{eq1}的发现是一个关键的难点.\n此例是陈计先生给出的著名的 Neuberg-Pedoe 不等式的一个加强.", + "figures": [] +} \ No newline at end of file diff --git a/processed_dataset/proof/2052.json b/processed_dataset/proof/2052.json new file mode 100644 index 0000000000000000000000000000000000000000..f4316ad9031627da1986e6b1976ed8d6904a0eb7 --- /dev/null +++ b/processed_dataset/proof/2052.json @@ -0,0 +1,10 @@ +{ + "source_file": "./raw_volume-zh/volume9/chapter7.tex", + "problem_type": "proof", + "problem": "例1. 给定半径为 $r$ 的圆上的定点 $P$ 的切线 $l$, 由此圆上动点 $R$ 引 $R Q$ 垂直于 $l$, 交 $l$ 于 $Q$. 试确定 $\\triangle P Q R$ 面积的最大值.", + "solution": "解:如图(), 注意到 $O P \\| R Q$, 作 $R S \\| l$, 交圆周于 $S$, 连接 $P S$. 易证\n$S_{\\triangle P Q R}=\\frac{1}{2} S_{\\triangle P R S}$.\n由定理 4 , 当圆内接三角形 $P R S$ 为正三角形时面积最大, 最大值为 $\\frac{3 \\sqrt{3}}{4} r^2$. 因此 $\\triangle P Q R$ 面积的最大值为 $\\frac{3 \\sqrt{3}}{8} r^2$", + "remark": "", + "figures": [ + "./images/volume9/figures/fig-c7i1.png" + ] +} \ No newline at end of file diff --git a/processed_dataset/proof/2053.json b/processed_dataset/proof/2053.json new file mode 100644 index 0000000000000000000000000000000000000000..7275931d271df55311709572728c6a5e7b0a409f --- /dev/null +++ b/processed_dataset/proof/2053.json @@ -0,0 +1,11 @@ +{ + "source_file": "./raw_volume-zh/volume9/chapter7.tex", + "problem_type": "proof", + "problem": "例2. 如图(),曲线 $L$ 将正三角形 $\\triangle A B C$ 分为两个等面积的部分.\n证明: $l \\geqslant \\frac{\\sqrt{\\pi} a}{2 \\sqrt[4]{3}}$, 其中 $l$ 为 $L$ 的长, $a$ 为正三角形的边长.", + "solution": "解:如图(), 我们将 $\\triangle A B C$ 连续翻转六次, 这时 $L$ 形成一条闭曲线.\n由于 $L$ 所围成的区域的面积等于 $3 S_{\\triangle A B C}$ 为一定值, 由定理 1 , 当此闭曲线为圆时周长为最小.\n因此\n$$\n6 l \\geqslant 2 \\pi \\sqrt{\\frac{3 S_{\\triangle A B C}}{\\pi}}=2 \\sqrt{\\pi} \\cdot \\sqrt{3 \\cdot \\frac{\\sqrt{3}}{4} a^2},\n$$\n由此即得 $l \\geqslant \\frac{\\sqrt{\\pi} a}{2 \\sqrt[4]{3}}$.", + "remark": "", + "figures": [ + "./images/volume9/figures/fig-c7i2.png", + "./images/volume9/figures/fig-c7i3.png" + ] +} \ No newline at end of file diff --git a/processed_dataset/proof/2054.json b/processed_dataset/proof/2054.json new file mode 100644 index 0000000000000000000000000000000000000000..c51e9f35f0aa405026b402df7f62e9f752923923 --- /dev/null +++ b/processed_dataset/proof/2054.json @@ -0,0 +1,10 @@ +{ + "source_file": "./raw_volume-zh/volume9/chapter7.tex", + "problem_type": "proof", + "problem": "例3. 设一个凸四边形 $Q$ 的四边满足 $a \\leqslant b \\leqslant c \\leqslant d$, 面积为 $F$, 求证\n$$\nF \\leqslant \\frac{3 \\sqrt{3}}{4} c^2 .\n$$\n这里用等周定理给出证明.", + "solution": "证明:如图(), 以四边形 $Q$ 的最大边为轴将 $Q$ 翻转过来, 则形成一个六边形 (有两种情况形成凸五边形和矩形, 这时结论同理可证). 注意到这个凸六边形的周长等于 $2(a+b+c)$, 为一定值.\n由等周定理 II,\n周长一定的 $n$ 边形中以正 $n$ 边形的面积为最大, 所以这个六边形的面积 $2 F$ 满足\n$$\n2 F \\leqslant \\frac{3 \\sqrt{3}}{2}\\left(\\frac{a+b+c}{3}\\right)^2,\n$$\n再应用 $a, b \\leqslant c$ 可得\n$$\nF \\leqslant \\frac{3 \\sqrt{3}}{4} c^2\n$$", + "remark": "注:由这个证明方法, 我们可将 Popa 不等式推广到 $n$ 边形中 .", + "figures": [ + "./images/volume9/figures/fig-c7i4.png" + ] +} \ No newline at end of file diff --git a/processed_dataset/proof/2055.json b/processed_dataset/proof/2055.json new file mode 100644 index 0000000000000000000000000000000000000000..00f21d4d5b506ec21923e8a754ef70df6318a167 --- /dev/null +++ b/processed_dataset/proof/2055.json @@ -0,0 +1,10 @@ +{ + "source_file": "./raw_volume-zh/volume9/chapter7.tex", + "problem_type": "proof", + "problem": "例4. 设两个 $n$ 边形 $\\Omega_1 、 \\Omega_2$ 的边分别为 $a_1 \\leqslant a_2 \\leqslant \\cdots \\leqslant a_{n-1} \\leqslant a_n$ 和 $b_1 \\leqslant b_2 \\leqslant \\cdots \\leqslant b_{n-1} \\leqslant b_n$, 它们的面积分别为 $F_1$ 和 $F_2$. 求证:\n$$\n\\frac{F_1}{a_n^2}+\\frac{F_2}{b_n^2}<\\frac{(n-1)^2}{4 \\pi}\\left(\\frac{a_{n-1}}{a_n}+\\frac{b_{n-1}}{b_n}\\right)^2 .\n$$", + "solution": "证明:不妨设 $A_n A_1$ 为 $\\Omega_1$ 的最大边,以 $A_n A_1$ 为一边,在与 $\\Omega_1$ 相异的一侧作一多边形 $\\Omega_2^{\\prime}$, 使得它与 $\\Omega_2$ 相似, 且 $\\Omega_2^{\\prime}$ 的最大边 $B_n^{\\prime} B_1^{\\prime}$ 与 $A_n A_1$ 重合, 如图(). 记 $\\widetilde{\\Omega}=\\Omega_1 \\cup \\Omega_2$, 那么 $\\widetilde{\\Omega}$ 的周长为\n$$\n\\sum_{i=1}^{n-1}\\left(a_i+\\frac{a_n}{b_n} b_i\\right)\n$$\n其面积为记 $\\angle A_n 、 \\angle A_1$ 为 $\\Omega_1$ 中以最长边为一边的两个角, $\\angle B_n 、 \\angle B_1$ 为 $\\Omega_2^{\\prime}$ 中相应的角.\n设 $\\Omega_1$ 中 $\\angle A_n$ 和 $\\angle A_1$ 的另一边分别为 $a_k 、 a_l, \\Omega_2^{\\prime}$ 中 $\\angle B_n$ 和 $\\angle B_1$ 的另一边分别为 $b_{k^{\\prime}}^{\\prime} 、 b_{l^{\\prime}}^{\\prime}$.\n如果 $\\angle A_n 、 \\angle A_1$ 分别和 $\\angle B_n 、 \\angle B_1$ 不是互为补角, 那么边 $a_k 、 a_l$ 和边 $b_{k^{\\prime}}^{\\prime}$ 、 $b_l^{\\prime}$ 分别不共线, 此时 $\\widetilde{\\Omega}$ 是一个 $2(n-1)$ 边形, 应用等周不等式(定理 2 ) 可得\n$$\n\\begin{aligned}\nF_1+\\frac{a_n^2}{b_n^2} F_2 & \\leqslant \\frac{\\left(\\sum_{i=1}^{n-1}\\left(a_i+\\frac{a_n}{b_n} b_i\\right)\\right)^2}{8(n-1)} \\cdot \\cot \\frac{\\pi}{2(n-1)} \\\\\n& \\leqslant \\frac{\\left((n-1) a_{n-1}+(n-1) \\frac{a_n}{b_n} b_{n-1}\\right)^2}{8(n-1)} \\cdot \\cot \\frac{\\pi}{2(n-1)}\n\\end{aligned}\n$$\n$$\n=\\frac{(n-1)\\left(a_{n-1}+\\frac{a_n}{b_n} b_{n-1}\\right)^2}{8} \\cdot \\cot \\frac{\\pi}{2(n-1)},\n$$\n也即\n$$\n\\frac{F_1}{a_n^2}+\\frac{F_2}{b_n^2} \\leqslant \\frac{n-1}{8}\\left(\\frac{a_{n-1}}{a_n}+\\frac{b_{n-1}}{b_n}\\right)^2 \\cdot \\cot \\frac{\\pi}{2(n-1)} . \\label{eq1}\n$$\n当 $\\angle A_n 、 \\angle A_1$ 其中之一和 $\\angle B_n 、 \\angle B_1$ 其中之一互为补角时, $\\widetilde{\\Omega}$ 是一个 $2 n-3$ 边形; 当 $\\angle A_n 、 \\angle A_1$ 和 $\\angle B_n 、 \\angle B_1$ 分别互为补角时, $\\widetilde{\\Omega}$ 是一个 $2(n-$ 2 )边形, 和上面的讨论类似, 在这两种情况下运用等周不等式分别可得\n$$\n\\begin{aligned}\n& \\frac{F_1}{a_n^2}+\\frac{F_2}{b_n^2} \\leqslant \\frac{(n-1)^2}{4(2 n-3)}\\left(\\frac{a_{n-1}}{a_n}+\\frac{b_{n-1}}{b_n}\\right)^2 \\cdot \\cot \\frac{\\pi}{2 n-3}, \\label{eq2}\\\\\n& \\frac{F_1}{a_n^2}+\\frac{F_2}{b_n^2} \\leqslant \\frac{(n-1)^2}{8(n-2)}\\left(\\frac{a_{n-1}}{a_n}+\\frac{b_{n-1}}{b_n}\\right)^2 \\cdot \\cot \\frac{\\pi}{2(n-2)} . \\label{eq3}\n\\end{aligned}\n$$\n注意到当 $x \\in\\left(0, \\frac{\\pi}{2}\\right)$, 有下式成立\n$$\n\\cot x<\\frac{1}{x} . \\label{eq4}\n$$\n将式\\ref{eq1}、\\ref{eq2}、式\\ref{eq3}分别和\\ref{eq4}式结合可得\n$$\n\\frac{F_1}{a_n^2}+\\frac{F_2}{b_n^2}<\\frac{(n-1)^2}{4 \\pi}\\left(\\frac{a_{n-1}}{a_n}+\\frac{b_{n-1}}{b_n}\\right)^2 .\n$$\n最后, 我们介绍 Ozeki 不等式的等周定理的证明.", + "remark": "", + "figures": [ + "./images/volume9/figures/fig-c7i5.png" + ] +} \ No newline at end of file diff --git a/processed_dataset/proof/2056.json b/processed_dataset/proof/2056.json new file mode 100644 index 0000000000000000000000000000000000000000..ced854cf491f862efd47bdd351900e1371d566e4 --- /dev/null +++ b/processed_dataset/proof/2056.json @@ -0,0 +1,10 @@ +{ + "source_file": "./raw_volume-zh/volume9/chapter7.tex", + "problem_type": "proof", + "problem": "例5. (Ozeki 不等式) 设 $\\varphi_1+\\varphi_2+\\cdots+\\varphi_n=\\pi$ (其中 $n \\geqslant 3$ ), $0<\\varphi_i< \\pi, i=1,2, \\cdots, n$. 求证对任意非负实数 $x_1, x_2, \\cdots, x_n$, 有\n$$\n\\sum_{i=1}^n x_i^2 \\geqslant \\sec \\frac{\\pi}{n}\\left(\\sum_{i=1}^n x_i x_{i+1} \\cos \\varphi_i\\right),\n$$\n其中 $x_{n+1}=x_1$.", + "solution": "证明:如图(), 从某一点 $O$ 出发作 $n$ 条射线 $O A_1, O A_2, \\cdots, O A_n$, 使得\n$$\n\\angle A_i O A_{i+1}=\\varphi_i+\\frac{\\pi}{n},\n$$\n其中 $A_{n+1}=A_1$, 再在这 $n$ 条射线上截取线段 $O A_i=x_i(i=1,2, \\cdots, n)$, 这样便得到一个 $n$ 边形 $A_1 A_2 \\cdots A_n$.\n现记 $A_i A_{i+1}=a_i$, 记这个 $n$ 边形的面积为 $F$.\n在 $\\triangle O A_i A_{i+1}$ 中由余弦定理可得\n$$\na_i^2=x_i^2+x_{i+1}^2-2 x_i x_{i+1} \\cos \\angle A_i O A_{i+1}, i=1,2, \\cdots, n, x_{i+1}=x_1 .\n$$\n因此\n$$\n\\sum_{i=1}^n a_i^2=2 \\sum_{i=1}^n x_i^2-2 \\sum_{i=1}^n x_i x_{i+1} \\cos \\left(\\varphi_i+\\frac{\\pi}{n}\\right), \\label{eq1}\n$$\n又由 Cauchy 不等式和等周不等式可得\n$$\n\\sum_{i=1}^n a_i^2 \\geqslant \\frac{1}{n}\\left(\\sum_{i=1}^n a_i\\right)^2 \\geqslant 4 F \\tan \\frac{\\pi}{n} . \\label{eq2}\n$$\n由式\\ref{eq1}和\\ref{eq2}可得\n$$\n\\begin{aligned}\n\\sum_{i=1}^n x_i^2 & \\geqslant 2 F \\tan \\frac{\\pi}{n}+\\sum_{i=1}^n x_i x_{i+1} \\cos \\left(\\varphi_i+\\frac{\\pi}{n}\\right) \\\\\n& =\\sum_{i=1}^n\\left[x_i x_{i+1} \\sin \\left(\\varphi_i+\\frac{\\pi}{n}\\right) \\tan \\frac{\\pi}{n}+x_i x_{i+1} \\cos \\left(\\varphi_i+\\frac{\\pi}{n}\\right)\\right] \\\\\n& =\\sec \\frac{\\pi}{n} \\sum_{i=1}^n x_i x_{i+1}\\left[\\sin \\left(\\varphi_i+\\frac{\\pi}{n}\\right) \\sin \\frac{\\pi}{n}+\\cos \\frac{\\pi}{n} \\cos \\left(\\varphi_i+\\frac{\\pi}{n}\\right)\\right] \\\\\n& =\\sec \\frac{\\pi}{n} \\sum_{i=1}^n x_i x_{i+1} \\cos \\varphi_i .\n\\end{aligned}\n$$", + "remark": "注:(1) Ozeki 不等式是 N. Ozeki 在研究著名的 Erdös-Mordell 不等式的多边形推广时提出的, 关于它的一些相关结果可参考\"Ozeki N. On the P. Erdös inequality for the triangle. J. College Arts Sci. Chiba Univ, 1957(2) : $247-250 \"$. 这个不等式 1961 年被 Lenhard 重新发现.\n(2) Ozeki 不等式是三角形嵌人不等式的多边形推广.\n三角形嵌人不等式可推广到三维甚至 $n$ 维空间中,其中关于四面体的结果为:\n设 $\\theta_{i j}(1 \\leqslant i0, x$ 、 $y, z$ 为任意实数, 求证:\n$$\n\\left(\\lambda_1 x+\\lambda_2 y+\\lambda_3 z\\right)^2 \\geqslant\\left(\\lambda_1 \\lambda_2+\\lambda_2 \\lambda_3+\\lambda_3 \\lambda_1\\right)\\left(2 x y+2 y z+2 z x-x^2-y^2-z^2\\right),\n$$\n其中等号当且仅当 $\\frac{x}{\\lambda_2+\\lambda_3}=\\frac{y}{\\lambda_1+\\lambda_3}=\\frac{z}{\\lambda_1+\\lambda_2}$ 时成立.", + "solution": "证明1 (应用嵌入不等式)\n由 $\\lambda_1 、 \\lambda_2 、 \\lambda_3$ 中至多只有一个负数及 $\\lambda_1\\left(\\lambda_2+\\lambda_3\\right)+\\lambda_2 \\lambda_3>0$ 易知 $\\lambda_2+ \\lambda_3>0$, 同理 $\\lambda_1+\\lambda_2>0, \\lambda_1+\\lambda_3>0$.\n设 $\\lambda_1+\\lambda_2=c^2, \\lambda_2+\\lambda_3=a^2, \\lambda_1+\\lambda_3=b^2(a, b, c>0)$, 则\n$$\n\\lambda_1=\\frac{1}{2}\\left(b^2+c^2-a^2\\right), \\lambda_2=\\frac{1}{2}\\left(a^2+c^2-b^2\\right), \\lambda_3=\\frac{1}{2}\\left(a^2+b^2-c^2\\right) .\n$$\n由 $\\lambda_1 \\lambda_2+\\lambda_2 \\lambda_3+\\lambda_3 \\lambda_1>0$ 展开得\n$$\n(a+b+c)(a-b+c)(a+b-c)(-a+b+c)>0 .\n$$\n从而 $a 、 b 、 c$ 构成某个三角形的三条边, 设这个三角形为 $\\triangle A B C$. 因此\n$$\n\\begin{aligned}\n& \\text { 原不等式 } \\Leftrightarrow \\sum \\lambda_1^2 x^2+2 \\sum \\lambda_1 \\lambda_2 x y \\geqslant\\left(\\sum \\lambda_1 \\lambda_2\\right)\\left(2 x y+2 y z+2 z x-x^2-\\right. \\\\\n&\\left.y^2-z^2\\right) \\\\\n& \\Leftrightarrow \\sum x^2\\left(\\lambda_1+\\lambda_2\\right)\\left(\\lambda_1+\\lambda_3\\right) \\geqslant \\sum \\lambda_1 \\lambda_2(2 y z+2 x z) \\\\\n& \\Leftrightarrow \\sum x^2 c^2 b^2 \\geqslant \\sum \\frac{c^4-a^4-b^4+2 a^2 b^2}{4}(2 y z+2 x z) \\\\\n& \\Leftrightarrow \\sum(x c b)^2 \\geqslant \\sum y z a^2\\left(b^2+c^2-a^2\\right) . \\label{eq1}\n\\end{aligned}\n$$\n事实上,由嵌入不等式\n$$\n\\begin{aligned}\n\\sum(x b c)^2 & \\geqslant \\sum 2(y c a)(z b a) \\cdot \\cos A \\\\\n& =\\sum 2(y c a)(z b a) \\cdot \\frac{b^2+c^2-a^2}{2 b c} \\\\\n& =\\sum y z a^2\\left(b^2+c^2-a^2\\right) .\n\\end{aligned}\n$$\n此即\\ref{eq1}式,从而原不等式成立.\n等号成立当且仅当\n$$\n\\begin{aligned}\n& \\frac{x b c}{\\sin A}=\\frac{y a c}{\\sin B}=\\frac{z a b}{\\sin C} \\\\\n\\Leftrightarrow & \\frac{x}{\\sin ^2 A}=\\frac{y}{\\sin ^2 B}=\\frac{z}{\\sin ^2 C} \\\\\n\\Leftrightarrow & \\frac{x}{\\lambda_2+\\lambda_3}=\\frac{y}{\\lambda_1+\\lambda_3}=\\frac{z}{\\lambda_1+\\lambda_2} .\n\\end{aligned}\n$$", + "remark": "", + "figures": [] +} \ No newline at end of file diff --git a/processed_dataset/proof/2058.json b/processed_dataset/proof/2058.json new file mode 100644 index 0000000000000000000000000000000000000000..caa943970d0daac043387aa2a1a21f7766863833 --- /dev/null +++ b/processed_dataset/proof/2058.json @@ -0,0 +1,8 @@ +{ + "source_file": "./raw_volume-zh/volume9/chapter8.tex", + "problem_type": "proof", + "problem": "例1. 设 $\\lambda_1 、 \\lambda_2 、 \\lambda_3$ 中至少有两个正数, 且满足 $\\lambda_1 \\lambda_2+\\lambda_2 \\lambda_3+\\lambda_3 \\lambda_1>0, x$ 、 $y, z$ 为任意实数, 求证:\n$$\n\\left(\\lambda_1 x+\\lambda_2 y+\\lambda_3 z\\right)^2 \\geqslant\\left(\\lambda_1 \\lambda_2+\\lambda_2 \\lambda_3+\\lambda_3 \\lambda_1\\right)\\left(2 x y+2 y z+2 z x-x^2-y^2-z^2\\right),\n$$\n其中等号当且仅当 $\\frac{x}{\\lambda_2+\\lambda_3}=\\frac{y}{\\lambda_1+\\lambda_3}=\\frac{z}{\\lambda_1+\\lambda_2}$ 时成立.", + "solution": "证明 2 (判别式法)\n同证法 1 可说明 $\\lambda_1+\\lambda_2>0, \\lambda_2+\\lambda_3>0, \\lambda_1+\\lambda_3>0$. 这时原不等式等价于\n$$\n\\begin{gathered}\n\\left(\\lambda_1+\\lambda_2\\right)\\left(\\lambda_1+\\lambda_3\\right) x^2-2\\left[\\lambda_3\\left(\\lambda_1+\\lambda_2\\right) y+\\lambda_2\\left(\\lambda_1+\\lambda_3\\right) z\\right] x+ \\\\\n{\\left[\\left(\\lambda_1+\\lambda_2\\right)\\left(\\lambda_2+\\lambda_3\\right) y^2+\\left(\\lambda_1+\\lambda_3\\right)\\left(\\lambda_2+\\lambda_3\\right) z^2-2 \\lambda_1\\left(\\lambda_2+\\lambda_3\\right) y z\\right] \\geqslant 0 .}\n\\end{gathered}\n$$\n将上式左边看作是关于 $x$ 的二次函数, 其二次项系数是正数,因此只需要证明它的判别式\n$$\n\\Delta \\leqslant 0,\n$$\n即\n$$\n\\begin{aligned}\n& \\lambda_3^2\\left(\\lambda_1+\\lambda_2\\right)^2 y^2+\\lambda_2^2\\left(\\lambda_1+\\lambda_3\\right)^2 z^2+2 \\lambda_3 \\lambda_2\\left(\\lambda_1+\\lambda_2\\right)\\left(\\lambda_1+\\lambda_3\\right) y z \\\\\n\\leqslant & \\left(\\lambda_1+\\lambda_2\\right)\\left(\\lambda_1+\\lambda_3\\right)\\left[\\left(\\lambda_1+\\lambda_2\\right)\\left(\\lambda_3+\\lambda_2\\right) y^2+\\left(\\lambda_1+\\lambda_3\\right)\\left(\\lambda_3+\\lambda_2\\right) z^2-\\right. \\\\\n& \\left.2 \\lambda_1\\left(\\lambda_3+\\lambda_2\\right) y z\\right] \\\\\n\\Leftrightarrow & \\left(\\lambda_1 \\lambda_2+\\lambda_2 \\lambda_3+\\lambda_3 \\lambda_1\\right)\\left[\\left(\\lambda_1+\\lambda_2\\right) y-\\left(\\lambda_1+\\lambda_3\\right) z\\right]^2 \\geqslant 0 .\n\\end{aligned}\n$$\n由题设条件这是成立的, 因此原不等式得证.\n等号成立当且仅当\n$$\n\\left(\\lambda_1+\\lambda_2\\right) y=\\left(\\lambda_1+\\lambda_3\\right) z \\Leftrightarrow \\frac{y}{\\lambda_1+\\lambda_3}=\\frac{z}{\\lambda_1+\\lambda_2},\n$$\n由 $x 、 y 、 z$ 地位的对称性即知等号成立当且仅当\n$$\n\\frac{x}{\\lambda_2+\\lambda_3}=\\frac{y}{\\lambda_1+\\lambda_3}=\\frac{z}{\\lambda_1+\\lambda_2} \\text {. }\n$$", + "remark": "", + "figures": [] +} \ No newline at end of file diff --git a/processed_dataset/proof/2059.json b/processed_dataset/proof/2059.json new file mode 100644 index 0000000000000000000000000000000000000000..89742e94cb797f0f7e5e7d68e2e6405d73e71c3f --- /dev/null +++ b/processed_dataset/proof/2059.json @@ -0,0 +1,8 @@ +{ + "source_file": "./raw_volume-zh/volume9/chapter8.tex", + "problem_type": "proof", + "problem": "例2. 设 $\\triangle A B C$ 和 $\\triangle A^{\\prime} B^{\\prime} C^{\\prime}$ 的边长分别为 $a 、 b 、 c$ 及 $a^{\\prime} 、 b^{\\prime} 、 c^{\\prime}$, 对应内角平分线分别为 $t_a 、 t_b 、 t_c$ 及 $t_a^{\\prime} 、 t_b^{\\prime} 、 t_c^{\\prime}$. 求证\n$$\nt_a t_a^{\\prime}+t_b t_b^{\\prime}+t_c t_c^{\\prime} \\leqslant \\frac{3}{4}\\left(a a^{\\prime}+b b^{\\prime}+c c^{\\prime}\\right) . \\label{eq1}\n$$", + "solution": "证明:由角平分线公式可得\n$$\nt_a=\\frac{2 b c}{b+c} \\cdot \\cos \\frac{A}{2} \\leqslant \\sqrt{b c} \\cos \\frac{A}{2},\n$$\n同理 $t_a^{\\prime} \\leqslant \\sqrt{b^{\\prime} c} \\cos \\frac{A}{2}$, 等等.\n因此\n$$\n\\begin{aligned}\nt_a t_a^{\\prime}+t_b t_b^{\\prime}+t_c t_c^{\\prime} & \\leqslant \\sum \\sqrt{b b^{\\prime} c c^{\\prime}} \\cdot \\cos \\frac{A}{2} \\cos \\frac{A^{\\prime}}{2} \\\\\n& =\\frac{1}{2} \\sum \\sqrt{b b^{\\prime} c c^{\\prime}}\\left(\\cos \\frac{A-A^{\\prime}}{2}+\\cos \\frac{A+A^{\\prime}}{2}\\right) \\\\\n& \\leqslant \\frac{1}{2} \\sum \\sqrt{b b^{\\prime} c c^{\\prime}}\\left(1+\\cos \\frac{A+A^{\\prime}}{2}\\right) \\\\\n& =\\frac{1}{2} \\sum \\sqrt{b b^{\\prime} c c^{\\prime}}+\\frac{1}{2} \\sum \\sqrt{b b^{\\prime} c c^{\\prime}} \\cdot \\cos \\frac{A+A^{\\prime}}{2} . \\label{eq2}\n\\end{aligned}\n$$\n现注意到 $\\frac{A+A^{\\prime}}{2}+\\frac{B+B^{\\prime}}{2}+\\frac{C+C^{\\prime}}{2}=\\pi$, 应用嵌入不等式并令 $x= \\sqrt{a a^{\\prime}}, y=\\sqrt{b b^{\\prime}}, z=\\sqrt{c c^{\\prime}}$ 可得\n$$\n2 \\sum \\sqrt{b b^{\\prime} c c^{\\prime}} \\cos \\frac{A+A^{\\prime}}{2} \\leqslant \\sum a a^{\\prime}, \\label{eq3}\n$$\n又由平均值不等式\n$$\n\\sum \\sqrt{b b^{\\prime} c c^{\\prime}} \\leqslant \\frac{1}{2} \\sum\\left(b b^{\\prime}+c c^{\\prime}\\right)=\\sum a a^{\\prime}, \\label{eq4}\n$$\n由式\\ref{eq2}、\\ref{eq3}及式\\ref{eq4}即得所证不等式.", + "remark": "", + "figures": [] +} \ No newline at end of file diff --git a/processed_dataset/proof/2060.json b/processed_dataset/proof/2060.json new file mode 100644 index 0000000000000000000000000000000000000000..3ff9ddbcb8fbdaebed071bb3664995d194843015 --- /dev/null +++ b/processed_dataset/proof/2060.json @@ -0,0 +1,10 @@ +{ + "source_file": "./raw_volume-zh/volume9/chapter8.tex", + "problem_type": "proof", + "problem": "例3. 设 $P$ 为 $\\triangle A B C$ 内部或边上任一点, 记 $P A=x, P B=y, P C=z$,\n求证\n$$\nx^2+y^2+z^2 \\geqslant \\frac{1}{3}\\left(a^2+b^2+c^2\\right) .\n$$", + "solution": "证明:如图(), 分别过 $A 、 B 、 C$ 作 $P A 、 P B$ 、 $P C$ 的垂线, 三垂线两两相交于 $A^{\\prime} 、 B^{\\prime} 、 C^{\\prime}$, 于是 $\\angle B P C=\\pi-A^{\\prime}, \\angle A P B=\\pi-C^{\\prime}, \\angle A P C=\\pi- B^{\\prime}$. 由余弦定理可得\n$$\n\\begin{aligned}\n& a^2=y^2+z^2+2 y z \\cos A^{\\prime}, \\\\\n& b^2=x^2+z^2+2 x z \\cos B^{\\prime}, \\\\\n& c^2=x^2+y^2+2 x y \\cos C^{\\prime},\n\\end{aligned}\n$$\n相加并应用嵌入不等式便得\n$$\n\\begin{aligned}\na^2+b^2+c^2 & =2\\left(x^2+y^2+z^2\\right)+2 x y \\cos C^{\\prime}+2 x z \\cos B^{\\prime}+2 y z \\cos A^{\\prime} \\\\\n& \\leqslant 2\\left(x^2+y^2+z^2\\right)+\\left(x^2+y^2+z^2\\right) \\\\\n& =3\\left(x^2+y^2+z^2\\right),\n\\end{aligned}\n$$\n得证.", + "remark": "", + "figures": [ + "./images/volume9/figures/fig-c8i1.png" + ] +} \ No newline at end of file diff --git a/processed_dataset/proof/2061.json b/processed_dataset/proof/2061.json new file mode 100644 index 0000000000000000000000000000000000000000..1f88efc527452b18dfb0d2ed5c123c1da971685c --- /dev/null +++ b/processed_dataset/proof/2061.json @@ -0,0 +1,8 @@ +{ + "source_file": "./raw_volume-zh/volume9/chapter8.tex", + "problem_type": "proof", + "problem": "例4. 设 $\\triangle A B C$ 的三条边为 $a 、 b 、 c$, 对应的内角分别为 $A 、 B 、 C$. 记 $s= a+b+c . \\triangle A^{\\prime} B^{\\prime} C^{\\prime}$ 的三条边为 $a^{\\prime} 、 b^{\\prime} 、 c^{\\prime}, s^{\\prime}=a^{\\prime}+b^{\\prime}+c^{\\prime}$. 求证\n$$\n\\frac{a}{a} \\tan \\frac{A}{2}+\\frac{b}{b^{\\prime}} \\tan \\frac{B}{2}+\\frac{c}{c} \\tan \\frac{C}{2} \\geqslant \\frac{\\sqrt{3} s}{2 s^{\\prime}},\n$$\n等号成立当且仅当 $\\triangle A B C$ 和 $\\triangle A^{\\prime} B^{\\prime} C^{\\prime}$ 均为正三角形.", + "solution": "证明:在加权正弦和的不等式中作代换 $A \\rightarrow \\frac{\\pi-A}{2}, B \\rightarrow \\frac{\\pi-B}{2}, C \\rightarrow \\frac{\\pi-C}{2}$, 再对所得不等式作代换 $x \\rightarrow y z, y \\rightarrow z x, z \\rightarrow x y$ 且同时作代换 $u \\rightarrow a^{\\prime}$, $v \\rightarrow b^{\\prime}, w \\rightarrow c^{\\prime}$, 可得\n$$\n\\frac{y z}{a^{\\prime} x}+\\frac{x z}{b^{\\prime} y}+\\frac{x y}{c^{\\prime} z} \\geqslant \\frac{2\\left(x \\cos \\frac{A}{2}+y \\cos \\frac{B}{2}+z \\cos \\frac{C}{2}\\right)}{\\sqrt{b^{\\prime} c^{\\prime}+a^{\\prime} c^{\\prime}+a^{\\prime} b^{\\prime}}}, \\label{eq1}\n$$\n再在式\\ref{eq1}中作代换 $x \\rightarrow \\frac{b c}{s} \\cos \\frac{A}{2}, y \\rightarrow \\frac{a c}{s} \\cos \\frac{B}{2}, z \\rightarrow \\frac{a b}{s} \\cos \\frac{C}{2}$, 并注意到\n$$\n\\frac{\\frac{a}{s} \\cos \\frac{B}{2} \\cos \\frac{C}{2}}{\\cos \\frac{A}{2}}=\\frac{2 R \\sin A \\cos \\frac{B}{2} \\cos \\frac{C}{2}}{2 R(\\sin A+\\sin B+\\sin C) \\cos \\frac{A}{2}}\n$$\n$$\n=\\frac{2 \\sin \\frac{A}{2} \\cos \\frac{A}{2} \\cos \\frac{B}{2} \\cos \\frac{C}{2}}{4 \\cos \\frac{A}{2} \\cos \\frac{B}{2} \\cos \\frac{C}{2} \\cos \\frac{A}{2}}=\\frac{1}{2} \\tan \\frac{A}{2},\n$$\n等等,可得\n$$\n\\frac{1}{2}\\left(\\frac{a}{a^{\\prime}} \\tan \\frac{A}{2}+\\frac{b}{b^{\\prime}} \\tan \\frac{B}{2}+\\frac{c}{c} \\tan \\frac{C}{2}\\right) \\geqslant \\frac{2\\left(\\frac{b c}{s} \\cos ^2 \\frac{A}{2}+\\frac{a c}{s} \\cos ^2 \\frac{B}{2}+\\frac{a b}{s} \\cos ^2 \\frac{C}{2}\\right)}{\\sqrt{b^{\\prime} c^{\\prime}+a^{\\prime} c^{\\prime}+a^{\\prime} b^{\\prime}}} . \\label{eq2}\n$$\n又因为\n$$\nb^{\\prime} c^{\\prime}+a^{\\prime} c^{\\prime}+a^{\\prime} b^{\\prime} \\leqslant \\frac{4}{3} s^{\\prime 2}, \\label{eq3}\n$$\n且\n$$\n\\begin{aligned}\n& 2\\left(\\frac{b c}{s} \\cos ^2 \\frac{A}{2}+\\frac{a c}{s} \\cos ^2 \\frac{B}{2}+\\frac{a b}{s} \\cos ^2 \\frac{C}{2}\\right) \\\\\n= & \\frac{1}{s}[b c(1+\\cos A)+a c(1+\\cos B)+a b(1+\\cos C)] \\\\\n= & \\frac{1}{2 s}\\left[2 b c+\\left(a^2+b^2-c^2\\right)+2 a c+\\left(a^2+c^2-b^2\\right)+2 a b+\\left(-a^2+b^2+c^2\\right)\\right] \\\\\n= & \\frac{1}{2 s}\\left[a^2+b^2+c^2+2(a b+b c+c a)\\right] \\\\\n= & \\frac{1}{2 s}(a+b+c)^2=\\frac{s}{2} .\n\\end{aligned} \\label{eq4}\n$$\n将式\\ref{eq3}和\\ref{eq4}代入式\\ref{eq2}立得所证不等式.", + "remark": "", + "figures": [] +} \ No newline at end of file diff --git a/processed_dataset/proof/2062.json b/processed_dataset/proof/2062.json new file mode 100644 index 0000000000000000000000000000000000000000..15cd8a073381611416f6623e61c2c168d41bb73b --- /dev/null +++ b/processed_dataset/proof/2062.json @@ -0,0 +1,8 @@ +{ + "source_file": "./raw_volume-zh/volume9/chapter8.tex", + "problem_type": "proof", + "problem": "例5. (Klamkin 不等式)设 $P 、 P^{\\prime}$ 是 $\\triangle A_1 A_2 A_3$ 所在平面上任意两点, 记 $P A_i=R_i, P^{\\prime} A_i=R_i^{\\prime}$, 三边 $a_i=A_{i-1} A_{i+1}$, 其中 $A_4=A_1, A_0=A_3, i=1$, 2 , 3. 求证\n$$\na_1 R_1 R_1^{\\prime}+a_2 R_2 R_2^{\\prime}+a_3 R_3 R_3^{\\prime} \\geqslant a_1 a_2 a_3, \\label{eq1}\n$$\n并指明等号成立的条件.", + "solution": "证明:应用惯性矩不等式.\n在惯性矩不等式中, 令 $x=\\frac{a_1 R_1^{\\prime}}{R_1}, y=\\frac{a_2 R_2^{\\prime}}{R_2}, z=\\frac{a_3 R_3^{\\prime}}{R_3}$, 可得\n$$\n\\left(\\sum \\frac{a_i R_i^{\\prime}}{R_i}\\right)\\left(\\sum a_i R_i^{\\prime} R_i\\right) \\geqslant \\sum a_1^2\\left(\\frac{a_2 R_2^{\\prime}}{R_2}\\right)\\left(\\frac{a_3 R_3^{\\prime}}{R_3}\\right),\n$$\n整理即得\n$$\n\\left(\\sum a_1 R_1^{\\prime} R_2 R_3\\right)\\left(\\sum a_1 R_1 R_1^{\\prime}\\right) \\geqslant a_1 a_2 a_3\\left(\\sum a_1 R_1 R_2^{\\prime} R_3^{\\prime}\\right) . \\label{eq2}\n$$\n类似的有\n$$\n\\left(\\sum a_1 R_1 R_2^{\\prime} R_3^{\\prime}\\right)\\left(\\sum a_1 R_1 R_1^{\\prime}\\right) \\geqslant a_1 a_2 a_3\\left(\\sum a_1 R_1^{\\prime} R_2 R_3\\right) . \\label{eq3}\n$$\n式\\ref{eq2}、\\ref{eq3}两式相加, 两边约去相同的项便得所证不等式\\ref{eq1}.\n注意到式\\ref{eq2}中等号成立的条件为\n$$\n\\frac{r_1 R_1}{R_1^{\\prime}}=\\frac{r_2 R_2}{R_2^{\\prime}}=\\frac{r_3 R_3}{R_3^{\\prime}}, \\label{eq4}\n$$\n而式\\ref{eq3}中等号成立的条件为\n$$\n\\frac{r_1^{\\prime} R_1^{\\prime}}{R_1}=\\frac{r_2^{\\prime} R_2^{\\prime}}{R_2}=\\frac{r_3^{\\prime} R_3^{\\prime}}{R_3}, \\label{eq5}\n$$\n这里 $r_i 、 r_i^{\\prime}$ 分别是 $P$ 和 $P^{\\prime}$ 到 $a_i$ 的距离.\n式\\ref{eq4}、\\ref{eq5}相等可得\n$$\nr_1 r_1^{\\prime}=r_2 r_2^{\\prime}=r_3 r_3^{\\prime},\n$$\n即 $P$ 和 $P^{\\prime}$ 到三边的距离成反比.\n这说明 $P$ 和 $P^{\\prime}$ 是关于 $\\triangle A_1 A_2 A_3$ 的一对等角共轭点.", + "remark": "注:(1) 关于等角共轭点的概念及性质可参见 Roger A. Johnson 的书 《Modern Geometry》的第八章(中译本: 单墫译, 上海教育出版社, 1999).\n(2)上例证法的巧妙之处在于:对称的应用惯性矩不等式.\n下面是唐立华建立的一个不等式.", + "figures": [] +} \ No newline at end of file diff --git a/processed_dataset/proof/2063.json b/processed_dataset/proof/2063.json new file mode 100644 index 0000000000000000000000000000000000000000..5acb47e0dedc6094ce13fe1f22f631d44899d8c0 --- /dev/null +++ b/processed_dataset/proof/2063.json @@ -0,0 +1,8 @@ +{ + "source_file": "./raw_volume-zh/volume9/chapter8.tex", + "problem_type": "proof", + "problem": "例6. 设 $P$ 为 $\\triangle A_1 A_2 A_3$ 所在平面上的任意一点, $P A_i=R_i(i=1,2$, 3), $\\triangle A_1 A_2 A_3$ 的面积为 $\\Delta, \\triangle A_1 A_2 A_3$ 的边长分别为 $a_1 、 a_2 、 a_3$, 则\n$$\n\\left(R_2^2+R_3^2-R_1^2\\right) \\sin A_1+\\left(R_3^2+R_1^2-R_2^2\\right) \\sin A_2+\\left(R_1^2+R_2^2-R_3^2\\right) \\sin A_3 \\geqslant 2 \\Delta, \\label{eq1}\n$$\n等号成立当且仅当 $a_1=a_2=a_3$ 且\n$$\nR_1: R_2: R_3=\\sin \\alpha_1: \\sin \\alpha_2: \\sin \\alpha_3,\n$$\n其中 $\\angle \\alpha_i=\\angle A_{i+1} P A_{i+2}\\left(A_4=A_1, A_5=A_2, i=1,2,3\\right)$(按同一方向取角).", + "solution": "证明:先证下面的引理.\n引理设 $\\triangle A_1 A_2 A_3$ 的边长分别为 $a_1 、 a_2 、 a_3$, 则\n$$\n\\begin{gathered}\na_1\\left(a_1+a_2-a_3\\right)\\left(a_1+a_3-a_2\\right)+a_2\\left(a_2+a_1-a_3\\right)\\left(a_2+a_3-a_1\\right)+ \\\\\na_3\\left(a_3+a_1-a_2\\right)\\left(a_3+a_2-a_1\\right) \\geqslant 3 a_1 a_2 a_3, \\label{eq2}\n\\end{gathered}\n$$\n等号成立当且仅当 $a_1=a_2=a_3$.\n证明令\n$$\nx=\\frac{1}{2}\\left(a_2+a_3-a_1\\right), y=\\frac{1}{2}\\left(a_3+a_1-a_2\\right), z=\\frac{1}{2}\\left(a_1+a_2-a_3\\right),\n$$\n则 $x, y, z>0$, 且\n$$\na_1=y+z, a_2=z+x, a_3=x+y .\n$$\n于是式\\ref{eq2}等价于\n$$\n\\begin{aligned}\n& 4[y z(y+z)+z x(z+x)+x y(x+y)] \\geqslant 3(x+y)(y+z)(z+x) \\\\\n\\Leftrightarrow & 4\\left[x^2(y+z)+y^2(z+x)+z^2(x+y)\\right] \\geqslant 3\\left[x^2(y+z)+\\right. \\\\\n& \\left.y^2(z+x)+z^2(x+y)\\right]+6 x y z \\\\\n\\Leftrightarrow & x^2(y+z)+y^2(z+x)+z^2(x+y) \\geqslant 6 x y z .\n\\end{aligned} \\label{eq3}\n$$\n而由均值不等式知式\\ref{eq3}成立, 故引理中的不等式\\ref{eq2}得证, 等号成立当且仅当 $x=y=z$, 即 $a_1=a_2=a_3$.\n下面证明式\\ref{eq1}.\n由正弦定理及 $\\Delta=\\frac{a_1 a_2 a_3}{4 R}$ 知, \\ref{eq1}式等价于\n$$\n\\left(a_2+a_3-a_1\\right) R_1^2+\\left(a_1+a_3-a_2\\right) R_2^2+\\left(a_1+a_2-a_3\\right) R_3^2 \\geqslant a_1 a_2 a_3 . \\label{eq4}\n$$\n不妨设 $a_1 \\geqslant a_2 \\geqslant a_3>0$, 则\n$$\n\\begin{gathered}\na_1+a_2-a_3 \\geqslant a_3+a_1-a_2 \\geqslant a_2+a_3-a_1>0 . \\\\\n\\text { 记 } \\lambda_i=\\left(a_1+a_2+a_3-2 a_i\\right)(i=1,2,3), \\text { 从而 } \\\\\n\\lambda_2 \\lambda_3 a_1 \\geqslant \\lambda_3 \\lambda_1 a_2 \\geqslant \\lambda_1 \\lambda_2 a_3,\n\\end{gathered}\n$$\n故由引理、惯性矩不等式及 Tchebychef(切比雪夫) 不等式可得\n$$\n\\begin{aligned}\n& \\left(a_2+a_3-a_1\\right) R_1^2+\\left(a_1+a_3-a_2\\right) R_2^2+\\left(a_1+a_2-a_3\\right) R_3^2 \\\\\n= & \\lambda_1 R_1^2+\\lambda_2 R_2^2+\\lambda_3 R_3^2 \\geqslant \\frac{\\sum_{i=1}^3 \\lambda_{i+1} \\lambda_{i+2} \\cdot a_i^2}{\\sum_{i=1}^3 \\lambda_i} \\\\\n= & \\frac{\\sum_{i=1}^3\\left(\\lambda_{i+1} \\lambda_{i+2} \\cdot a_i\\right) \\cdot a_i}{\\sum_{i=1}^3 a_i} \\geqslant \\frac{\\left(\\sum_{i=1}^3 \\lambda_{i+1} \\lambda_{i+2} \\cdot a_i\\right)\\left(\\sum_{i=1}^3 a_i\\right)}{3 \\sum_{i=1}^3 a_i} \\\\\n\\geqslant & a_1 a_2 a_3,\n\\end{aligned}\n$$\n故式\\ref{eq4}成立, 从而式\\ref{eq1}得证.\n由引理及 Tchebychef 不等式等号成立的条件易知式\\ref{eq1}等号成立当且仅当 $a_1=a_2=a_3$ 且 $R_1: R_2: R_3=\\sin \\alpha_1: \\sin \\alpha_2: \\sin \\alpha_3$.", + "remark": "", + "figures": [] +} \ No newline at end of file diff --git a/processed_dataset/proof/2064.json b/processed_dataset/proof/2064.json new file mode 100644 index 0000000000000000000000000000000000000000..ce29939c7cc021dab2266379e8e4e04467192d53 --- /dev/null +++ b/processed_dataset/proof/2064.json @@ -0,0 +1,11 @@ +{ + "source_file": "./raw_volume-zh/volume9/chapter9.tex", + "problem_type": "proof", + "problem": "Tsintsifas 的不等式轨迹问题.\n欧氏几何中下面的结论是熟知的.\n如果 $A 、 B 、 C 、 D$ 是一个平面上的任意四点, 则 $A D \\cdot B C 、 B D \\cdot C A$ 和 $C D \\cdot A B$ 是某一个三角形的三边长; 当 $A 、 B 、 C 、 D$ 四点共圆时, 这个三角形是退化的(在这种情况下,我们有著名的 Ptolemy 定理).\n这个结论的证明并不难, 用复数就更为简单.\n事实上,设 $A 、 B 、 C 、 D$ 对应的复数分别为 $z_1 、 z_2 、 z_3 、 z_4$, 用下面的恒等式\n$$\n\\left(z_1-z_4\\right)\\left(z_2-z_3\\right)+\\left(\\dot{z}_2-z_4\\right)\\left(z_3-z_1\\right)+\\left(z_3-z_4\\right)\\left(z_1-z_2\\right)=0,\n$$\n便可立得结论.\nTsintsifas 在 1983 年的 Crux. Math. (9) 上提出问题:\n问题设 $T$ 是一个给定的 $\\triangle A B C, P$ 是 $T$ 所在平面上的一点,现记以 $a \\cdot P A 、 b \\cdot P B 、 c \\cdot P C$ 为边的三角形为 $T_0$ (可能是退化的). 现设 $R 、 R_0$ 分别是 $T 、 T_0$ 的外接圆半径,求使得不等式\n$$\nP A \\cdot P B \\cdot P C \\leqslant R R_0\n$$\n成立的点 $P$ 的轨迹.", + "solution": "下面的解法 1 关键是应用关于一点 $P$ 的垂足三角形的面积公式.\n解法 1 先证引理.\n引理设 $P$ 为 $\\triangle A_1 A_2 A_3$ 所在平面上一点, 则 $P$ 关于 $\\triangle A_1 A_2 A_3$ 的垂足三角形的有向面积为\n$$\nF=\\frac{R^2-\\overline{O P^2}}{4 R^2} S_{\\triangle A_1 A_2 A_3},\n$$\n其中 $O$ 为 $\\triangle A_1 A_2 A_3$ 的外心.\n证明如图(), 设垂足三角形为 $\\triangle P_1 P_2 P_3$, 又设 $A_2 P$ 交 $\\triangle A_1 A_2 A_3$ 的外接圆于 $B_2$, 则\n$$\n\\begin{aligned}\n\\angle A_2 P A_3 & =\\angle P_2 P_1 P_3+\\angle A_2 A_1 A_3 \\\\\n& =\\angle A_2 B_2 A_3+\\angle B_2 A_3 P,\n\\end{aligned}\n$$\n注意, 这里的 $\\angle$ 表示有向角.\n因此\n$$\n\\angle P_2 P_1 P_3=\\angle B_2 A_3 P .\n$$\n这时\n$$\n\\begin{aligned}\nF & =\\frac{1}{2} \\overline{P_1 P_2} \\cdot \\overline{P_1 P_3} \\sin \\angle P_2 P_1 P_3 \\\\\n& =\\frac{1}{2} \\overline{P_1 P_2} \\cdot \\overline{P_1 P_3} \\sin \\angle B_2 A_3 P \\\\\n& =\\frac{1}{2} \\overline{P A_3} \\sin \\alpha_3 \\overline{P A_2} \\sin \\alpha_2 \\sin \\angle B_2 A_3 P,\n\\end{aligned}\n$$\n又\n$$\n\\frac{\\sin \\angle B_2 A_3 P}{\\sin \\angle A_2 B_2 A_3}=\\frac{\\overline{P B_2}}{\\overline{P A_3}},\n$$\n因此\n$$\n\\begin{aligned}\nF & =\\frac{1}{2} \\overline{P A_2} \\cdot \\overline{P B_2} \\sin \\angle A_2 B_2 A_3 \\sin \\alpha_2 \\sin \\alpha_3 \\\\\n& =\\frac{1}{2}\\left(R^2-\\overline{O P^2}\\right) \\sin \\alpha_1 \\sin \\alpha_2 \\sin \\alpha_3 \\\\\n& =\\frac{R^2-\\overline{O P}^2}{4 R^2} S_{\\triangle A_1 A_2 A_3},\n\\end{aligned}\n$$\n引理得证.\n下面回到原题.\n设 $P A=a^{\\prime}, P B=b^{\\prime}, P C=c^{\\prime}$, 则\n$$\nR=\\frac{a b c}{4 S_T}, R_0=\\frac{a b c a^{\\prime} b^{\\prime} c^{\\prime}}{4 S_{T_0}},\n$$\n因此\n$$\nR R_0 \\geqslant P A \\cdot P B \\cdot P C\n$$\n等价于\n$$\n\\begin{aligned}\n& \\frac{(a b c)^2}{16 S_T S_{T_0}} \\geqslant 1, \\\\\n& S_{T_0} \\leqslant \\frac{(a b c)^2}{16 S_T} . \n\\end{aligned} \\label{eq1}\n$$\n如图(), 设 $P$ 在 $B C 、 C A 、 A B$ 上的射影分别为 $A^{\\prime} 、 B^{\\prime} 、 C^{\\prime}$, 则\n$$\nB^{\\prime} C^{\\prime}=P A \\sin A=\\frac{a \\cdot P A}{2 R},\n$$\n同理\n$$\nC^{\\prime} A^{\\prime}=\\frac{b \\cdot P B}{2 R}, A^{\\prime} B^{\\prime}=\\frac{c \\cdot P C}{2 R},\n$$\n故 $\\triangle A^{\\prime} B^{\\prime} C^{\\prime}$ 与 $T_0$ 相似且相似比为 $2 R$. 因此由式\\ref{eq1}知\n$$\nS_{\\triangle A^{\\prime} B^{\\prime} C^{\\prime}}=\\frac{1}{4 R^2} S_{T_0} \\leqslant \\frac{1}{4 R^2} \\cdot \\frac{a^2 b^2 c^2}{16 S_T}, \\label{eq2}\n$$\n再应用引理得\n$$\nS_{\\triangle A^{\\prime} B^{\\prime} C^{\\prime}}=\\frac{\\left|R^2-\\overline{O P^2}\\right|}{4 R^2} S_T . \\label{eq3}\n$$\n因此由从式\\ref{eq2}、\\ref{eq3}可得\n$$\n\\frac{\\left\\lfloor R^2-\\overline{O P}^2 \\mid\\right.}{4 R^2} S_T \\leqslant \\frac{1}{4 R^2} \\cdot \\frac{a^2 b^2 c^2}{16 S_T} .\n$$\n这等价于\n$$\n\\left|R^2-\\overline{O P}^2\\right| \\leqslant \\frac{a^2 b^2 c^2}{16 S_T^2}=R^2\n$$\n故\n$$\n|\\overline{O P}| \\leqslant \\sqrt{2} R .\n$$\n这说明 $P$ 的轨迹是一个以 $T$ 的外心 $O$ 为圆心, $\\sqrt{2} R$ 为半径的圆及内部.", + "remark": "", + "figures": [ + "./images/volume9/figures/fig-c9i1.png", + "./images/volume9/figures/fig-c9i2.png" + ] +} \ No newline at end of file diff --git a/processed_dataset/proof/2065.json b/processed_dataset/proof/2065.json new file mode 100644 index 0000000000000000000000000000000000000000..c5595f41254b412c8c154741c3458108699b6b5d --- /dev/null +++ b/processed_dataset/proof/2065.json @@ -0,0 +1,8 @@ +{ + "source_file": "./raw_volume-zh/volume9/chapter9.tex", + "problem_type": "proof", + "problem": "Tsintsifas 的不等式轨迹问题.\n欧氏几何中下面的结论是熟知的.\n如果 $A 、 B 、 C 、 D$ 是一个平面上的任意四点, 则 $A D \\cdot B C 、 B D \\cdot C A$ 和 $C D \\cdot A B$ 是某一个三角形的三边长; 当 $A 、 B 、 C 、 D$ 四点共圆时, 这个三角形是退化的(在这种情况下,我们有著名的 Ptolemy 定理).\n这个结论的证明并不难, 用复数就更为简单.\n事实上,设 $A 、 B 、 C 、 D$ 对应的复数分别为 $z_1 、 z_2 、 z_3 、 z_4$, 用下面的恒等式\n$$\n\\left(z_1-z_4\\right)\\left(z_2-z_3\\right)+\\left(\\dot{z}_2-z_4\\right)\\left(z_3-z_1\\right)+\\left(z_3-z_4\\right)\\left(z_1-z_2\\right)=0,\n$$\n便可立得结论.\nTsintsifas 在 1983 年的 Crux. Math. (9) 上提出问题:\n问题设 $T$ 是一个给定的 $\\triangle A B C, P$ 是 $T$ 所在平面上的一点,现记以 $a \\cdot P A 、 b \\cdot P B 、 c \\cdot P C$ 为边的三角形为 $T_0$ (可能是退化的). 现设 $R 、 R_0$ 分别是 $T 、 T_0$ 的外接圆半径,求使得不等式\n$$\nP A \\cdot P B \\cdot P C \\leqslant R R_0\n$$\n成立的点 $P$ 的轨迹.", + "solution": "解法 2 设 $\\triangle A B C$ 的外心为 $O$, 现以 $P$ 为反演中心, $\\lambda=P A \\cdot P B \\cdot P C$ 为反演幂做反演变换.\n设 $A 、 B 、 C$ 反演后变为 $A^{\\prime} 、 B^{\\prime} 、 C^{\\prime}$, 则\n$$\nB^{\\prime} C^{\\prime}=B C \\cdot \\frac{\\lambda}{P B \\cdot P C}=a \\cdot P A .\n$$\n同理\n$$\nC^{\\prime} A^{\\prime}=b \\cdot P B, A^{\\prime} B^{\\prime}=c \\cdot P C .\n$$\n因此 $\\triangle A^{\\prime} B^{\\prime} C^{\\prime}$ 即为以 $a \\cdot P A 、 b \\cdot P B 、 c \\cdot P C$ 为边的三角形.\n而 $\\odot(O, R)$ 反演后的半径为\n$$\nR_0=R \\frac{\\lambda}{|P O|^2-R^2},\n$$\n因此 $P A \\cdot P B \\cdot P C \\leqslant R R_0$ 等价于\n$$\n\\lambda \\leqslant R^2 \\cdot \\frac{\\lambda}{|P O|^2-R^2},\n$$\n这又等价于\n$$\n|P O| \\leqslant \\sqrt{2} R\n$$\n这说明 $P$ 的轨迹是以 $O$ 为圆心, 半径为 $\\sqrt{2} R$ 的圆及内部.", + "remark": "注:由上面的解法看出存在 $\\triangle A B C$ 的反演像与点 $P$ 的垂足三角形相似.", + "figures": [] +} \ No newline at end of file diff --git a/processed_dataset/proof/2066.json b/processed_dataset/proof/2066.json new file mode 100644 index 0000000000000000000000000000000000000000..e516bb5414ed13b9015bc4929507f165baf3de7b --- /dev/null +++ b/processed_dataset/proof/2066.json @@ -0,0 +1,8 @@ +{ + "source_file": "./raw_volume-zh/volume9/exercise1.tex", + "problem_type": "proof", + "problem": "问题1. 设 $A^{\\prime}$ 为 $\\triangle A B C$ 的外角平分线 $A T$ 上的任意一点, 试证:\n$$\nA^{\\prime} B+A^{\\prime} C \\geqslant A B+A C .\n$$", + "solution": "延长 $B A$ 到 $C^{\\prime}$ 使得 $A C^{\\prime}=A C$, 连接 $A^{\\prime} C^{\\prime}$. 显然 $A^{\\prime} B+A^{\\prime} C^{\\prime}>B C^{\\prime}= B A+A C$. 再由 $\\triangle A A^{\\prime} C^{\\prime} \\cong \\triangle A A^{\\prime} C$ 便知 $A^{\\prime} C^{\\prime}=A^{\\prime} C$, 代入便得.", + "remark": "", + "figures": [] +} \ No newline at end of file diff --git a/processed_dataset/proof/2067.json b/processed_dataset/proof/2067.json new file mode 100644 index 0000000000000000000000000000000000000000..24cab4b75e3cc92e2868042359d75932e51b856b --- /dev/null +++ b/processed_dataset/proof/2067.json @@ -0,0 +1,8 @@ +{ + "source_file": "./raw_volume-zh/volume9/exercise1.tex", + "problem_type": "proof", + "problem": "问题2. 给定边长为 $a>b>c$ 的 $\\triangle A B C$ 及其任意内点 $O$. 设直线 $A O 、 B O 、 C O$ 与 $\\triangle A B C$ 的边交于点 $P 、 Q 、 R$. 证明:\n$$\nO P+O Q+O R 2 \\sin 70^{\\circ}$, 矛盾.\n若凸包为四边形 $A B C D, E$ 在其内部, 不妨设 $A C$ 和 $B D$ 交于$F$, 且 $E$ 在 $\\triangle A F B$ 内, 如图(), 假设 $\\lambda<2 \\sin 70^{\\circ}$. 先考虑两条引理: 引理 1 对任意 $\\triangle A B C, \\frac{B C}{\\min (B A, C A)} \\geqslant 2 \\sin \\frac{A}{2}$. 引理 2 设 $D$ 为 $\\triangle A B C$ 内一点, 则及假设, 必有 $\\angle A E C<140^{\\circ}, \\angle A E B<140^{\\circ}$, 于是 $\\angle B E C>80^{\\circ}$, 同理 $\\angle A E D>80^{\\circ} \\cdots$ (1). 又由引理 2 及假设, 有 $\\angle A B C<70^{\\circ}, \\angle B A D<70^{\\circ}$, 于是 $\\max (\\angle A D C, \\angle B C D)>110^{\\circ}$. 不妨设 $\\angle B C D>110^{\\circ}$, 由 (1) 有 $B C \\geqslant \\frac{B C}{\\min (C E, B E)} \\geqslant 2 \\sin \\frac{\\angle C E B}{2} \\geqslant 2 \\sin 40^{\\circ}$, 于是有 $\\lambda \\geqslant B D> \\sqrt{1+4 \\sin ^2 40^{\\circ}+4 \\sin 40^{\\circ} \\cos 70^{\\circ}}=2 \\sin 70^{\\circ}$, 矛盾! 因此, 五点构成一个凸五边形.", + "remark": "", + "figures": [ + "./images/volume9/figures/fig-c10a2.png" + ] +} \ No newline at end of file diff --git a/processed_dataset/proof/2070.json b/processed_dataset/proof/2070.json new file mode 100644 index 0000000000000000000000000000000000000000..1515a56a918eeac7f9db3fea390b01825244319d --- /dev/null +++ b/processed_dataset/proof/2070.json @@ -0,0 +1,8 @@ +{ + "source_file": "./raw_volume-zh/volume9/exercise11.tex", + "problem_type": "proof", + "problem": "问题1. 设 $P 、 Q$ 是正四面体 $A B C D$ 内的二点, 求证: $\\angle P A Q<60^{\\circ}$.", + "solution": "提示: 设 $\\angle P A Q$ 所在平面与正四面体各面的交线为 $A E 、 A F 、 E F$. 只要证明 $\\angle E A F \\leqslant 60^{\\circ}$ 便可, 这可通过证明在 $\\triangle A E F$ 中 $E F$ 是最小边来实现.", + "remark": "", + "figures": [] +} \ No newline at end of file diff --git a/processed_dataset/proof/2071.json b/processed_dataset/proof/2071.json new file mode 100644 index 0000000000000000000000000000000000000000..423b3b935f856850834e9c427898052c9ed370b3 --- /dev/null +++ b/processed_dataset/proof/2071.json @@ -0,0 +1,8 @@ +{ + "source_file": "./raw_volume-zh/volume9/exercise11.tex", + "problem_type": "proof", + "problem": "问题3. 半径为 1 的球面上的两点用球内长度小于 2 的曲线段连接起来.\n证明这条曲线段一定落在这个球的某个半球内.", + "solution": "提示: 先考虑平面上的类似问题, 探究解题方法.\n只要过球心 $O$ 作垂直于 $\\angle A O B$ 的平分线 $O C$ 的平面 $\\pi$, 再利用点的对称性设法证明 $\\overparen{A B}$ 上不可能有平面 $\\pi$ 上的点,即不穿过 $\\pi$ 便可.", + "remark": "", + "figures": [] +} \ No newline at end of file diff --git a/processed_dataset/proof/2072.json b/processed_dataset/proof/2072.json new file mode 100644 index 0000000000000000000000000000000000000000..ceb557374ff49a6de7d1845e982cf62da35638c7 --- /dev/null +++ b/processed_dataset/proof/2072.json @@ -0,0 +1,8 @@ +{ + "source_file": "./raw_volume-zh/volume9/exercise11.tex", + "problem_type": "proof", + "problem": "问题4. 设 $I$ 是四面体 $A_1 A_2 A_3 A_4$ 的内心, 并记 $\\triangle A_i I A_j$ 的面积为 $I_{i j}, A_i$ 所对的面三角形的面积为 $S_i$, 试证\n$$\n\\sum_{1 \\leqslant iA B+B C+C A .\n$$", + "solution": "对四边形 $A C A_1 B$ 应用 Ptolemy 定理, 可得 $A A_1 \\cdot B C=A B \\cdot A_1 C+ A C \\cdot A_1 B$. 令 $A_1 B=A_1 C=x$, 注意到 $2 x=A_1 B+A_1 C>B C$, 有 $2 A A_1= 2 \\frac{A B x+A C x}{B C}=(A B+A C) \\cdot \\frac{2 x}{B C}>A B+A C$, 即 $A A_1>\\frac{1}{2}(A B+A C)$. 同理可得 $B B_1>\\frac{1}{2}(B A+B C), C C_1>-\\frac{1}{2}(C A+C B)$, 三式相加即得所证结果.", + "remark": "", + "figures": [] +} \ No newline at end of file diff --git a/processed_dataset/proof/2078.json b/processed_dataset/proof/2078.json new file mode 100644 index 0000000000000000000000000000000000000000..87e0e063e606712041b24c5ae0bcfa6313f83008 --- /dev/null +++ b/processed_dataset/proof/2078.json @@ -0,0 +1,8 @@ +{ + "source_file": "./raw_volume-zh/volume9/exercise3.tex", + "problem_type": "proof", + "problem": "问题1. 设 $A B C D$ 是一个有内切圆的凸四边形, 它的每个内角和外角都不小于 $60^{\\circ}$. 证明:\n$$\n\\frac{1}{3}\\left|A B^3-A D^3\\right| \\leqslant\\left|B C^3-C D^3\\right| \\leqslant 3\\left|A B^3-A D^3\\right|,\n$$\n并指出等号成立的条件.", + "solution": "利用余弦定理, 知 $B D^2=A D^2+A B^2-2 A D \\cdot A B \\cos A=C D^2+ B C^2-2 C D \\cdot B C \\cos C$. 由条件知 $60^{\\circ} \\leqslant A \\leqslant 120^{\\circ}, 60^{\\circ} \\leqslant C \\leqslant 120^{\\circ}$, 故 $-\\frac{1}{2} \\leqslant \\cos A \\leqslant \\frac{1}{2},-\\frac{1}{2} \\leqslant \\cos C \\leqslant \\frac{1}{2}$, 于是 $3 B D^2-\\left(A B^2+A D^2+A B \\cdot A D\\right)= 2\\left(A D^2+A B^2\\right)-A D \\cdot A B(1+6 \\cos A) \\geqslant 2\\left(A D^2+A B^2\\right)-4 A D \\cdot A B= 2(A B-A D)^2 \\geqslant 0$, 即 $\\frac{1}{3}\\left(A B^2+A D^2+A B \\cdot A D\\right) \\leqslant B D^2=C D^2+B C^2- 2 C D \\cdot B C \\cos C \\leqslant C D^2+B C^2+C D \\cdot B C$. 再由 $A B C D$ 为圆外切四边形可知 $A D+B C=A B+C D$, 所以 $|A B-A D|=|C D-B C|$. 结合上式就有 $\\frac{1}{3}\\left|A B^3-A D^3\\right| \\leqslant\\left|B C^3-C D^3\\right|$, 等号成立的条件为 $\\cos A=\\frac{1}{2}, A B= A D, \\cos C=-\\frac{1}{2}$ 或者 $|A B-A D|=|C D-B C|=0$, 所以等号成立的条件是 $A B=A D$ 或者 $C D=B C$. 同理可证另一个不等式成立.", + "remark": "", + "figures": [] +} \ No newline at end of file diff --git a/processed_dataset/proof/2079.json b/processed_dataset/proof/2079.json new file mode 100644 index 0000000000000000000000000000000000000000..94f2335678e934d55e2c0788d88d8d5482fcde81 --- /dev/null +++ b/processed_dataset/proof/2079.json @@ -0,0 +1,8 @@ +{ + "source_file": "./raw_volume-zh/volume9/exercise3.tex", + "problem_type": "proof", + "problem": "问题2. 一个面积为 $S$ 的凸四边形有外接圆, 且其外接圆圆心在该四边形内部.\n从此四边形对角线的交点向四条边作垂线, 证明: 以四个垂足为顶点的四边形的面积不超过 $\\frac{S}{2}$.", + "solution": "设这个凸四边形为 $A B C D$, 其外接圆圆心为 $O$, 半径为 $R, E$ 为 $A C$ 、 $B D$ 的交点, $M 、 N 、 P 、 Q$ 分别是 $E$ 到边 $A B 、 B C 、 C D 、 D A$ 的投影, 作 $E F \\perp M N, F$ 为垂足.\n由于 $B 、 M 、 E 、 N$ 四点共圆, 且在以 $B E$ 为直径的圆上,所以 $M N=B E \\cdot \\sin B=\\frac{B E \\cdot A C}{2 R}$, 而 $E F=E M \\cdot \\sin \\angle E M N= \\frac{A E \\cdot B E \\cdot \\sin \\angle A E B}{A B} \\cdot \\sin \\angle C B E$, 结合 $B E \\cdot \\sin \\angle E B C=C E \\cdot \\sin \\angle B C E= \\frac{C E \\cdot A B}{2 R}$, 又由于 $A E \\cdot E C=R^2-O E^2$. 于是 $E F=\\frac{R^2-O E^2}{2 R} \\cdot \\sin \\angle A E B$, 所以 $S_{\\triangle M E N}=\\frac{1}{2} M N \\cdot E F=\\frac{\\left(R^2-O E^2\\right) \\cdot A C \\cdot B E \\cdot \\sin \\angle A E B}{8 R^2}$. 类似地计算 $S_{\\triangle M E P} 、 S_{\\triangle P E Q} 、 S_{\\triangle Q E M}$, 求和可得 $S_{M N P Q}=\\frac{\\left(R^2-O E^2\\right) \\cdot A C \\cdot B D \\cdot \\sin \\angle A E B}{4 R^2}= \\frac{\\left(R^2-O E^2\\right) \\cdot S}{2 R^2} \\leqslant \\frac{S}{2}$.", + "remark": "", + "figures": [] +} \ No newline at end of file diff --git a/processed_dataset/proof/2080.json b/processed_dataset/proof/2080.json new file mode 100644 index 0000000000000000000000000000000000000000..df0154b1a6019febcab8f0abe08e5878d6630bfb --- /dev/null +++ b/processed_dataset/proof/2080.json @@ -0,0 +1,8 @@ +{ + "source_file": "./raw_volume-zh/volume9/exercise3.tex", + "problem_type": "proof", + "problem": "问题3. 两个凸四边形 $A B C D$ 和 $A^{\\prime} B^{\\prime} C^{\\prime} D^{\\prime}$ 的边长分别为 $a 、 b 、 c 、 d$ 和 $a^{\\prime} 、 b^{\\prime} 、 c^{\\prime}$ 、 $d^{\\prime}$, 面积分别为 $S$ 和 $S^{\\prime}$. 证明:\n$$\na a^{\\prime}+b b^{\\prime}+c c^{\\prime}+d d^{\\prime} \\geqslant 4 \\sqrt{S S^{\\prime}} .\n$$", + "solution": "在边长给定的四边形中, 以内接于圆时其面积为最大.\n因此, 只需证两个凸四边形为圆内接四边形的情况.\n这时 $S= \\sqrt{(s-a)(s-b)(s-c)(s-d)}, S^{\\prime}$ 与之类似, 其中 $s=\\frac{1}{2}(a+b+c+d)= (a+c)=(b+d), s^{\\prime}=\\frac{1}{2}\\left(a^{\\prime}+b^{\\prime}+c^{\\prime}+d^{\\prime}\\right)=\\left(a^{\\prime}+c^{\\prime}\\right)=\\left(b^{\\prime}+d^{\\prime}\\right)$. 利用算术几何平均值不等式有 $a a^{\\prime}+b b^{\\prime}+c c^{\\prime}+d d^{\\prime}=(s-a)\\left(s^{\\prime}-a^{\\prime}\\right)+(s-b) \\left(s^{\\prime}-b^{\\prime}\\right)+(s-c)\\left(s^{\\prime}-c^{\\prime}\\right)+(s-d)\\left(s^{\\prime}-d^{\\prime}\\right) \\geqslant 4\\left[(s-a)\\left(s^{\\prime}-a^{\\prime}\\right)(s-b)\\right. \\left.\\left(s^{\\prime}-b^{\\prime}\\right)(s-c)\\left(s^{\\prime}-c^{\\prime}\\right)(s-d)\\left(s^{\\prime}-d^{\\prime}\\right)\\right]^{\\frac{1}{4}}=4 \\sqrt{S S^{\\prime}}$.", + "remark": "", + "figures": [] +} \ No newline at end of file diff --git a/processed_dataset/proof/2081.json b/processed_dataset/proof/2081.json new file mode 100644 index 0000000000000000000000000000000000000000..28631439f824d6a8da4086fb5d91ac1ca1b85b3b --- /dev/null +++ b/processed_dataset/proof/2081.json @@ -0,0 +1,8 @@ +{ + "source_file": "./raw_volume-zh/volume9/exercise3.tex", + "problem_type": "proof", + "problem": "问题5. 设 $A B C D$ 是一个圆内接四边形且它的边长分别为 $a 、 b 、 c$ 、 d. $\\rho_a$ 是该四边形外与边 $A B$ 及边 $C B 、 D A$ 的延长线相切的圆的半径, $\\rho_b$ 、 $\\rho_c 、 \\rho_d$ 与 $\\rho_a$ 的意义类似.\n求证:\n$$\n\\frac{1}{\\rho_a}+\\frac{1}{\\rho_b}+\\frac{1}{\\rho_c}+\\frac{1}{\\rho_d} \\geqslant \\frac{8}{\\sqrt[4]{a b c d}},\n$$\n当且仅当 $A B C D$ 是正方形时等号成立.", + "solution": "设 $A B=a, B C=b, C D=c, D A=d$. 易得 $a=\\rho_a\\left(\\tan \\frac{A}{2}+\\tan \\frac{B}{2}\\right)$, 即 $\\frac{a}{\\rho_a} \\geqslant 2 \\sqrt{\\tan \\frac{A}{2} \\tan \\frac{B}{2}}$. 同理 $\\frac{b}{\\rho_b} \\geqslant 2 \\sqrt{\\tan \\frac{B}{2} \\tan \\frac{C}{2}}, \\frac{c}{\\rho_c} \\geqslant 2 \\sqrt{\\tan \\frac{C}{2} \\tan \\frac{D}{2}}$, $\\frac{d}{\\rho_d} \\geqslant 2 \\sqrt{\\tan \\frac{D}{2} \\tan \\frac{A}{2}}$. 因为 $A+C=B+D=\\pi$, 所以 $\\tan \\frac{A}{2} \\tan \\frac{C}{2}=\\tan \\frac{B}{2} \\tan \\frac{D}{2}=1$, 因此 $\\frac{1}{\\rho_a}+\\frac{1}{\\rho_c} \\geqslant \\frac{2}{a} \\sqrt{\\tan \\frac{A}{2} \\tan \\frac{\\bar{B}}{2}}+\\frac{2}{c} \\sqrt{\\tan \\frac{C}{2} \\tan \\frac{D}{2}} \\geqslant 2 \\sqrt{\\frac{4}{a c} \\sqrt{\\tan \\frac{A}{2} \\tan \\frac{B}{2} \\tan \\frac{C}{2} \\tan \\frac{D}{2}}}=\\frac{4}{\\sqrt{a c}}$, 同理 $\\frac{1}{\\rho_b}+\\frac{1}{\\rho_d} \\geqslant \\frac{4}{\\sqrt{b d}}$, 所以 $\\frac{1}{\\rho_a}+ \\frac{1}{\\rho_b}+\\frac{1}{\\rho_c}+\\frac{1}{\\rho_d} \\geqslant \\frac{4}{\\sqrt{a c}}+\\frac{4}{\\sqrt{b d}} \\geqslant \\frac{8}{\\sqrt[4]{a b c d}}$. 等号成立当且仅当 $A=B=C= D$ 并且 $a=b=c=d$.", + "remark": "", + "figures": [] +} \ No newline at end of file diff --git a/processed_dataset/proof/2082.json b/processed_dataset/proof/2082.json new file mode 100644 index 0000000000000000000000000000000000000000..238aa929ea15461ab94f213cc7d568772cbab062 --- /dev/null +++ b/processed_dataset/proof/2082.json @@ -0,0 +1,8 @@ +{ + "source_file": "./raw_volume-zh/volume9/exercise4.tex", + "problem_type": "proof", + "problem": "问题1. 已知钝角三角形 $A B C$ 的外接圆半径为 1 , 证明: 存在一个斜边长为 $\\sqrt{2}+1$ 的等腰三角形覆盖 $\\triangle A B C$.", + "solution": "不妨设 $\\angle C>90^{\\circ}$, 于是 $\\min \\{\\angle A, \\angle B\\}<45^{\\circ}$. 不妨设 $\\angle A<45^{\\circ}$. 以 $A B$ 为直径, 在顶点 $C$ 的同侧作半圆 $O$, 则 $C$ 位于半圆 $O$ 内.\n作射线 $A T$ 使得 $\\angle B A T=45^{\\circ}$. 再作射线 $O E$ 使得 $\\angle B O E=45^{\\circ}$, 且与半圆相交于点 $E$. 过点 $E$ 作半圆的切线, 分别交 $A B$ 的延长线和 $A T$ 于点 $D$ 和 $F$, 则等腰直角三角形 $A D F$ 覆盖 $\\triangle A B C$, 并且 $A D=A O+O D=\\frac{1}{2} A B+\\frac{\\sqrt{2}}{2} A B=\\frac{1}{2}(1+\\sqrt{2}) \\cdot A B<\\frac{1}{2}(1+\\sqrt{2}) 2 R=1+\\sqrt{2}$.", + "remark": "", + "figures": [] +} \ No newline at end of file diff --git a/processed_dataset/proof/2083.json b/processed_dataset/proof/2083.json new file mode 100644 index 0000000000000000000000000000000000000000..6f91b93e2882dec1a68d5283979b7c93a62ad43a --- /dev/null +++ b/processed_dataset/proof/2083.json @@ -0,0 +1,8 @@ +{ + "source_file": "./raw_volume-zh/volume9/exercise4.tex", + "problem_type": "proof", + "problem": "问题4. 证明: 在边长为 1 的正方形内, 不可能无重叠的放人两个边长大于 $\\sqrt{\\frac{2}{3}}$ 的正方形.", + "solution": "提示: 只需证明边长大于 $\\frac{\\sqrt{2}}{3}$ 的正三角形放人边长为 1 的正方形后,一定包含该正方形的中心.", + "remark": "", + "figures": [] +} \ No newline at end of file diff --git a/processed_dataset/proof/2084.json b/processed_dataset/proof/2084.json new file mode 100644 index 0000000000000000000000000000000000000000..73805b132876e04e76b2a34661418333f75b9743 --- /dev/null +++ b/processed_dataset/proof/2084.json @@ -0,0 +1,8 @@ +{ + "source_file": "./raw_volume-zh/volume9/exercise4.tex", + "problem_type": "proof", + "problem": "问题5. 平面上任给 $n$ 个点,其中任何三点可组成一个三角形, 每个三角形都有一个面积.\n令最大面积与最小面积之比为 $u_n$, 求 $u_5$ 的最小值.", + "solution": "设平面内任意五点为 $A_1 、 A_2 、 A_3 、 A_4 、 A_5$, 其中任意 3 点不共线.\n(1) 若5 点的凸包不是凸五边形, 那么其中必有一点落在某个三角形内, 这时易证 $\\mu_5 \\geqslant 3$. (2) 5 点的凸包为凸五边形 $A_1 A_2 A_3 A_4 A_5$. 作 $M N / / A_3 A_4$ 分别交 $A_1 A_3 、 A_1 A_4$ 于 $M$ 和 $N$, 且使得 $\\frac{A_1 M}{M A_3}=\\frac{A_1 N}{N A_4}=\\frac{\\sqrt{5}-1}{2}$. (i) $A_2 、 A_5$ 中有一点, 比如 $A_2$ 与 $A_3 、 A_4$ 在直线 $M N$ 的同侧时有 $\\mu_5 \\geqslant \\frac{S_{\\triangle A_1 A_3 A_4}}{S_{\\triangle A_2 A_3 A_4}} \\geqslant \\frac{A_1 A_3}{M A_3}=1+\\frac{A_1 M}{M A_3}= \\frac{\\sqrt{5}+1}{2}$. (ii) $A_2 、 A_5$ 与 $A_1$ 均在直线 $M N$ 的同侧时, 设 $A_2 A_5$ 交 $A_1 A_3$ 于 $O$, 则 $A_1 O \\leqslant A_1 M$, 于是 $\\mu_5 \\geqslant \\frac{S_{\\triangle A_2 A_3 A_5}}{S_{\\triangle A_1 A_2 A_5}}=\\frac{O A_3}{O A_1} \\geqslant \\frac{M A_3}{M A_1}=\\frac{\\sqrt{5}+1}{2}$. 注意到 $3> \\frac{\\sqrt{5}+1}{2}$, 所以总有 $\\mu_5 \\geqslant \\frac{\\sqrt{5}+1}{2}$. 当 $A_1 、 A_2 、 A_3 、 A_4 、 A_5$ 为边长为 $a$ 的正五边形的 5 个顶点时, 有 $\\mu_5=\\frac{S_{\\triangle A_1 A_3 A_4}}{S_{\\triangle A_1 A_2 A_3}}=\\frac{\\frac{1}{2} A_1 A_3 \\cdot A_1 A_4 \\sin 36^{\\circ}}{\\frac{1}{2} A_1 A_2 \\cdot A_1 A_3 \\sin 36^{\\circ}}=\\frac{A_1 A_4}{A_1 A_2}= \\frac{\\sqrt{5}+1}{2}$. 综上可得 $\\mu_5$ 的最小值为 $\\frac{\\sqrt{5}+1}{2}$.", + "remark": "", + "figures": [] +} \ No newline at end of file diff --git a/processed_dataset/proof/2085.json b/processed_dataset/proof/2085.json new file mode 100644 index 0000000000000000000000000000000000000000..5564a368de5f107d3f8ab7f8a408b0f81a159c5f --- /dev/null +++ b/processed_dataset/proof/2085.json @@ -0,0 +1,8 @@ +{ + "source_file": "./raw_volume-zh/volume9/exercise5.tex", + "problem_type": "proof", + "problem": "问题1. 设 $G$ 为 $\\triangle A B C$ 的重心, $A_1 、 B_1 、 C_1$ 分别为 $A G 、 B G 、 C G$ 与 $\\triangle A B C$ 的外接圆的交点,求证\n$$\nG A_1+G B_1+G C_1 \\geqslant G A+G B+G C,\n$$\n等号成立当且仅当 $\\triangle A B C$ 为正三角形.", + "solution": "提示: 问题转化为证明 $A A_1+B B_1+C C_1 \\geqslant \\frac{4}{3}\\left(m_a+m_b+m_c\\right)$, 其中 $m_a$ 为边 $a$ 上的中线, 等等.\n设 $A A_1=M_a$, 等等.\n由相交弦定理可得 $m_a \\cdot\\left(M_a-\\right. \\left.m_a\\right)=\\left(\\frac{a}{2}\\right)^2$, 将其与等式 $4 m_a^2=8 k^2-3 a^2$ 联立消去 $a$ 可得 $M_a= \\frac{2 k}{3}\\left(\\frac{k}{m_a}+\\frac{m_a}{k}\\right) \\geqslant \\frac{4}{3} k$, 等等, 其中 $k^2=\\frac{1}{4}\\left(a^2+b^2+c^2\\right)(k>0)$.", + "remark": "", + "figures": [] +} \ No newline at end of file diff --git a/processed_dataset/proof/2086.json b/processed_dataset/proof/2086.json new file mode 100644 index 0000000000000000000000000000000000000000..fd3880a27db721ea1b589e18cbce282cfac29a9f --- /dev/null +++ b/processed_dataset/proof/2086.json @@ -0,0 +1,8 @@ +{ + "source_file": "./raw_volume-zh/volume9/exercise5.tex", + "problem_type": "proof", + "problem": "问题2. 在凸四边形内部标定四个点, 求证: 可在凸四边形的边界上找到一点使得它到四边形的各顶点的距离之和大于它到四个给定点的距离之和.", + "solution": "提示: 设凸四边形 $A B C D$ 内的 4 个标定点为 $M 、 N 、 P 、 Q$, 只需考虑 $M N P Q$ 为凸四边形的情形.\n对 $A B C D$ 周界上任意一点 $E$, 令 $f(E)=E A+ E B+E C+E D, g(E)=E M+E N+E P+E Q$, 于是只要证明周界上存在一点 $E$ 使 $f(E)>g(E)$. 作直线 $M P$, 交四边形 $A B C D$ 的周界于 $F 、 G$, 于是有 $f(F)+f(G)>g(F)+g(G)$. 这表明 $f(F)>g(F)$ 和 $f(G)>g(G)$ 中至少有一个成立.", + "remark": "", + "figures": [] +} \ No newline at end of file diff --git a/processed_dataset/proof/2087.json b/processed_dataset/proof/2087.json new file mode 100644 index 0000000000000000000000000000000000000000..899e0c473a23a5f7d313a20062083fad4be3d562 --- /dev/null +++ b/processed_dataset/proof/2087.json @@ -0,0 +1,8 @@ +{ + "source_file": "./raw_volume-zh/volume9/exercise5.tex", + "problem_type": "proof", + "problem": "问题4. 设 $a$ 是一个凸六边形 $A B C D E F$ 的最大边长, $d=\\min \\{A D$, $B E, C F\\}$, 求证 $d \\leqslant 2 \\alpha$.", + "solution": "不妨设 $A E \\leqslant A C \\leqslant C E$, 则由 Ptolemy 定理有 $A D \\cdot C E \\leqslant A C \\cdot D E+ A E \\cdot C D \\leqslant a(A C+A E) \\leqslant 2 a \\cdot C E$, 由此 $A D \\leqslant 2 a$.", + "remark": "", + "figures": [] +} \ No newline at end of file diff --git a/processed_dataset/proof/2088.json b/processed_dataset/proof/2088.json new file mode 100644 index 0000000000000000000000000000000000000000..9ff4a217c87354a9f1de579f8dd6005d17e92c30 --- /dev/null +++ b/processed_dataset/proof/2088.json @@ -0,0 +1,8 @@ +{ + "source_file": "./raw_volume-zh/volume9/exercise6.tex", + "problem_type": "proof", + "problem": "问题2. 分别以 $\\triangle A B C$ 的边 $A B$ 和 $A C$ 为边向外作正方形 $A B D E$ 和 $A C F G, P$ 、 $Q$ 为直线 $E G$ 上的两点使得 $B P$ 和 $C Q$ 垂直于 $B C$, 求证\n$$\nB P+C Q \\geqslant B C+E G,\n$$\n等号成立当且仅当 $A B=A C$.", + "solution": "不失一般性, 可设 $A=(0, h), B=(p, 0), C=(q, 0)(h>0, p$ 0 . 设 $\\triangle A B C$ 的面积为 $S$, 则 $2 S=a h_a=2(p-b) r_b$, 由此知 $\\frac{h_a}{r_b}=\\frac{2(p-b)}{a}= \\frac{2 y}{y+z}$, 故 $\\sum\\left(\\frac{h_a}{r_b}\\right)^2=4 \\sum \\frac{y^2}{(y+z)^2}$. 而 $\\sum \\sin ^2 \\frac{A}{2}=\\sum \\frac{(p-b)(p-c)}{b c}= \\sum \\frac{y z}{(x+y)(z+x)}$, 因此, 原不等式等价于 $\\sum \\frac{y^2}{(y+z)^2} \\geqslant \\sum \\frac{y z}{(x+y)(z+x)} \\Leftrightarrow \\sum y^2(x+y)^2(z+x)^2 \\geqslant \\sum y z(x+y)(z+x)(y+ z)^2 \\Leftrightarrow \\sum y^2\\left(y z+x \\sum x\\right)^2 \\geqslant \\sum y z\\left(y z+x \\sum x\\right)(y+z)^2 \\Leftrightarrow \\sum y^2 x^2\\left(\\sum x\\right)^2+ \\sum y^4 z^2+2 x y z \\sum x \\sum y^2 \\geqslant \\sum y^2 z^2(y+z)^2+x y z \\sum(y+ z)^2 \\sum x \\Leftrightarrow \\sum y^2 z^2\\left(x^2+2 x y+2 x z\\right)+\\sum y^4 z^2 \\geqslant 2 x y z \\sum x \\cdot \\sum x y= 6 x^2 y^2 z^2+2 x y z\\left(\\sum x^2(y+z)\\right) \\Leftrightarrow 3 x^2 y^2 z^2+2 x y z \\sum x^2(y+z) \\leqslant 2 x y z \\sum y z(y+z)+\\sum y^4 z^2 \\Leftrightarrow 3 x^2 y^2 z^2 \\leqslant \\sum y^4 z^2$, 由平均值不等式, 最后一式成立,故原不等式成立.", + "remark": "", + "figures": [] +} \ No newline at end of file diff --git a/processed_dataset/proof/2090.json b/processed_dataset/proof/2090.json new file mode 100644 index 0000000000000000000000000000000000000000..010b8acb550bf08cc8fec9af70eacb68378004b9 --- /dev/null +++ b/processed_dataset/proof/2090.json @@ -0,0 +1,8 @@ +{ + "source_file": "./raw_volume-zh/volume9/exercise6.tex", + "problem_type": "proof", + "problem": "问题5. 设 $A D 、 B E 、 C F$ 是 $\\triangle A B C$ 的角平分线, $\\triangle A B C$ 内的动点 $P$ 到其三边的距离的平方根构成某三角形的三条边长, 求证\n(1) $P$ 的轨迹是一个椭圆 $\\Gamma$ 的内部, 并且 $\\Gamma$ 与 $\\triangle A B C$ 的边 $B C 、 A B 、 A C$ 分别相切于 $D 、 E 、 F$;\n(2) 椭圆 $\\Gamma$ 的面积 $S_{\\Gamma}$ 满足\n$$\n\\frac{4 \\sqrt{3} \\pi}{9} S_{\\triangle D E F} \\leqslant S_{\\Gamma} \\leqslant \\frac{\\sqrt{3} \\pi}{9} S_{\\triangle A B C} .\n$$", + "solution": "(1) 提示: 用解析法证明 $\\Gamma$ 是一个椭圆及内部.\n(2) 新建一个直角坐标系.\n设 $\\Gamma$ 的方程为 $\\Gamma: \\frac{x^2}{a^2}+\\frac{y^2}{b^2}=1(a, b>0)$, 则 $\\triangle A B C$ 是 $\\Gamma$ 的外切三角形, $\\triangle D E F$ 是 $\\Gamma$ 的内接三角形.\n作变换 $x=a x^{\\prime}, y=b y^{\\prime}$, 则它将 $x O y$ 平面上的任意凸区域 $D$ 变成 $x^{\\prime} O^{\\prime} y^{\\prime}$ 平面上的凸区域 $D^{\\prime}$, 将 $\\Gamma$ 变为单位圆 $\\odot O^{\\prime}$, 且它们的面积有关系 $|D|=a b \\cdot\\left|D^{\\prime}\\right|$. 设此变换把 $\\triangle A B C 、 \\triangle D E F$ 依次变为 $\\triangle A^{\\prime} B^{\\prime} C^{\\prime} 、 \\triangle D^{\\prime} E^{\\prime} F^{\\prime}$, 则 $\\triangle A^{\\prime} B^{\\prime} C^{\\prime} 、 \\triangle D^{\\prime} E^{\\prime} F^{\\prime}$ 分别是 $\\odot O^{\\prime}$ 的外切三角形、内接三角形, 且 $S_{\\Gamma}=a b \\cdot S_{\\odot O^{\\prime}}, S_{\\triangle A B C}=a b \\cdot S_{\\triangle A^{\\prime} B^{\\prime} C^{\\prime}}, S_{\\triangle D E F}=a b \\cdot S_{\\triangle D^{\\prime} E^{\\prime} F^{\\prime}}$, 因此 $\\frac{S_{\\Gamma}}{S_{\\triangle D E F}}=\\frac{S_{\\odot O^{\\prime}}}{S_{\\triangle D^{\\prime} E^{\\prime} F^{\\prime}}} \\geqslant \\frac{4 \\sqrt{3} \\pi}{9}, \\frac{S_{\\Gamma}}{S_{\\triangle A B C}}=\\frac{S_{\\odot O^{\\prime}}}{S_{\\triangle A^{\\prime} B^{\\prime} C^{\\prime}}} \\leqslant \\frac{\\sqrt{3}}{9} \\pi$, 由此即得所证不等式.", + "remark": "", + "figures": [] +} \ No newline at end of file diff --git a/processed_dataset/proof/2091.json b/processed_dataset/proof/2091.json new file mode 100644 index 0000000000000000000000000000000000000000..0f97d65aa907ec08d33b83c3538090292dc24705 --- /dev/null +++ b/processed_dataset/proof/2091.json @@ -0,0 +1,8 @@ +{ + "source_file": "./raw_volume-zh/volume9/exercise7.tex", + "problem_type": "proof", + "problem": "问题1. 在已知边 $B C$ 和对角 $\\alpha$ 的所有三角形中, 证明 :\n(1) 底边为 $B C$ 的等腰三角形面积最大;\n(2) 底边为 $B C$ 的等腰三角形周长最大.", + "solution": "(1) 顶点 $A$ 在以 $B C$ 为弦的圆弧上, 圆弧上线段 $B C$ 的对角为 $\\alpha$. 如果点 $A$ 离开直线 $B C$ 最远, 即如果点 $A$ 在线段 $B C$ 的垂直平分线上, 则 $\\triangle A B C$ 面积最大.\n(2) 在固定 $B C$ 和 $\\alpha$ 的情况下, $\\triangle A B C$ 外接圆半径 $R$ 是定值.\n显然, $A B+A C=2 R(\\sin \\gamma+\\sin \\beta)=4 R \\cdot \\sin \\frac{\\pi-\\alpha}{2} \\cos \\frac{\\gamma-\\beta}{2}$, 当 $\\cos \\frac{\\gamma-\\beta}{2}=1$, 即 $\\gamma=\\beta$ 时, $A B+A C$ 最大.", + "remark": "", + "figures": [] +} \ No newline at end of file diff --git a/processed_dataset/proof/2092.json b/processed_dataset/proof/2092.json new file mode 100644 index 0000000000000000000000000000000000000000..58731f98e529faa8afd5e717147353d8328b6cba --- /dev/null +++ b/processed_dataset/proof/2092.json @@ -0,0 +1,8 @@ +{ + "source_file": "./raw_volume-zh/volume9/exercise7.tex", + "problem_type": "proof", + "problem": "问题2. 两个等边三角形内接于一个半径为 $r$ 的圆, 设 $K$ 为两个三角形重叠处的面积,求证 $2 K \\geqslant r^2 \\sqrt{3}$.", + "solution": "设等边 $\\triangle A B C$ 的两边 $A B 、 A C$ 与等边 $\\triangle P Q R$ 的边 $P Q$ 交于 $D 、 E$, 由旋转对称性可得 $K=S_{\\triangle A B C}-3 S_{\\triangle A D E}=\\frac{3 \\sqrt{3}}{4} r^2-3 S_{\\triangle A D E}$, 注意到 $\\triangle A D E$ 有固定的周长 $\\sqrt{3} r$, 利用等周定理知 $\\triangle A D E$ 为正三角形 (即其边长等于 $\\triangle A B C$ 边长的 $\\frac{1}{3}$ ) 时面积最大.\n由此得 $K \\geqslant \\frac{\\sqrt{3} r^2}{2}$.", + "remark": "", + "figures": [] +} \ No newline at end of file diff --git a/processed_dataset/proof/2093.json b/processed_dataset/proof/2093.json new file mode 100644 index 0000000000000000000000000000000000000000..483ff039069e9e082d7500300cd55560d1afa514 --- /dev/null +++ b/processed_dataset/proof/2093.json @@ -0,0 +1,8 @@ +{ + "source_file": "./raw_volume-zh/volume9/exercise7.tex", + "problem_type": "proof", + "problem": "问题4. 设 $n$ 边形 $A_1 A_2 \\cdots A_n$ 内有一点 $P$ 到边 $A_1 A_2, A_2 A_3, \\cdots, A_n A_1$ 的距离分别为 $d_1, d_2, \\cdots, d_n$, 求证: $\\sum_{i=1}^n \\frac{a_i}{d_i} \\geqslant 2 n \\tan \\frac{\\pi}{n}$, 其中 $a_i=A_i A_{i+1}$ (约定 $A_{n+1}=A_1$ ), 并指出等号成立的充要条件.", + "solution": "设 $n$ 边形 $A_1 A_2 \\cdots A_n$ 的面积为 $S$, 由面积关系有 $\\sum_{i=1}^n a_i d_i=2 S$, 于是由 Cauchy 不等式和等周不等式有 $\\sum_{i=1}^n \\frac{a_i}{d_i}=\\sum_{i=1}^n \\frac{a_i^2}{a_i d_i} \\geqslant \\frac{\\left(\\sum a_i\\right)^2}{\\sum_{i=1}^n a_i d_i}=\\frac{\\left(\\sum a_i\\right)^2}{2 S} \\geqslant \\frac{1}{2 S} \\cdot 4 n \\cdot S \\cdot \\tan \\frac{\\pi}{n}=2 n \\tan \\frac{\\pi}{n}$.", + "remark": "", + "figures": [] +} \ No newline at end of file diff --git a/processed_dataset/proof/2094.json b/processed_dataset/proof/2094.json new file mode 100644 index 0000000000000000000000000000000000000000..c57f27e62c78e08649ffd7c71e69c99a7ef990e3 --- /dev/null +++ b/processed_dataset/proof/2094.json @@ -0,0 +1,8 @@ +{ + "source_file": "./raw_volume-zh/volume9/exercise8.tex", + "problem_type": "proof", + "problem": "问题1. 设 $a 、 b 、 c$ 是任意三角形的三边, $x 、 y 、 z$ 是任意三个实数, 求证\n$$\na^2(x-y)(x-z)+b^2(y-x)(y-z)+c^2(z-x)(z-y) \\geqslant 0 .\n$$", + "solution": "提示: 把 $\\cos A=\\frac{b^2+c^2-a^2}{2 b c}$ 等代入嵌入不等式, 再作代数恒等变形即得.", + "remark": "", + "figures": [] +} \ No newline at end of file diff --git a/processed_dataset/proof/2095.json b/processed_dataset/proof/2095.json new file mode 100644 index 0000000000000000000000000000000000000000..3cb102bf27581dba37b7b903649d76aacab4f517 --- /dev/null +++ b/processed_dataset/proof/2095.json @@ -0,0 +1,8 @@ +{ + "source_file": "./raw_volume-zh/volume9/exercise8.tex", + "problem_type": "proof", + "problem": "问题2. 在 $\\triangle A B C$ 和 $\\triangle A^{\\prime} B^{\\prime} C^{\\prime}$ 中, 求证\n$$\n\\cot A+\\cot B+\\cot C \\geqslant \\frac{\\cos A^{\\prime}}{\\sin A}+\\frac{\\cos B^{\\prime}}{\\sin B}+\\frac{\\cos C^{\\prime}}{\\sin C} .\n$$", + "solution": "注意到公式 $\\cot A=\\frac{b^2+c^2-a^2}{4 \\Delta}$ 等等, 原不等式等价于 $a^2+b^2+c^2 \\geqslant \\frac{4 \\Delta \\cos A^{\\prime}}{\\sin A}+\\frac{4 \\Delta \\cos B^{\\prime}}{\\sin B}+\\frac{4 \\Delta \\cos C^{\\prime}}{\\sin C}$, 再由面积公式, 上式等价于 $a^2+b^2+c^2 \\geqslant 2 a b \\cos C^{\\prime}+2 a c \\cos B^{\\prime}+2 b c \\cos A^{\\prime}$. 这正是嵌入不等式的一个特例.", + "remark": "", + "figures": [] +} \ No newline at end of file diff --git a/processed_dataset/proof/2096.json b/processed_dataset/proof/2096.json new file mode 100644 index 0000000000000000000000000000000000000000..965d097c3f8a6a932ca4074d64adbfc87b0b296c --- /dev/null +++ b/processed_dataset/proof/2096.json @@ -0,0 +1,8 @@ +{ + "source_file": "./raw_volume-zh/volume9/exercise8.tex", + "problem_type": "proof", + "problem": "问题4. 证明:设 $x, y, z, w \\in \\mathbf{R}^{+}, \\alpha+\\beta+\\gamma+\\theta=(2 k+1) \\pi, k \\in \\mathbf{Z}$, 则\n$$\n|x \\sin \\alpha+y \\sin \\beta+z \\sin \\gamma+w \\sin \\theta| \\leqslant \\sqrt{\\frac{(x y+z w)(x z+y w)(x w+y z)}{x y z w}} .\n$$", + "solution": "记 $u=x \\sin \\alpha+y \\sin \\beta, v=z \\sin \\gamma+r \\sin \\theta$, 则 $u^2=(x \\sin \\alpha+y \\sin \\beta)^2 \\leqslant (x \\sin \\alpha+y \\sin \\beta)^2+(x \\cos \\alpha-y \\cos \\beta)^2=x^2+y^2-2 x y \\cos (\\alpha+\\beta)$. 同理可得 $v^2 \\leqslant z^2+w^2-2 z w \\cos (\\theta+\\gamma)$. 注意到已知条件 $\\alpha+\\beta+\\gamma+\\theta=(2 k+1) \\pi (k \\in \\mathbf{Z})$, 有 $\\cos (\\alpha+\\beta)+\\cos (\\gamma+\\theta)=0$, 因此 $\\frac{x^2+y^2-u^2}{2 x y}+\\frac{z^2+w^2-\\gamma^2}{2 z w} \\geqslant 0$, 即 $\\frac{u^2}{x y}+\\frac{v^2}{z w} \\leqslant \\frac{x^2+y^2}{x y}+\\frac{z^2+w^2}{z w}=\\frac{(x z+y w)(x w+y z)}{x y z w}$. 另外, 应用 Cauchy 不等式有 $|u+v| \\leqslant \\sqrt{\\left(\\frac{u^2}{x y}+\\frac{v^2}{z w}\\right)(x y+z w)} \\leqslant \\sqrt{\\frac{(x y+z w)(x z+y w)(x w+y z)}{x y z w}}$, 得证.", + "remark": "", + "figures": [] +} \ No newline at end of file diff --git a/processed_dataset/proof/2097.json b/processed_dataset/proof/2097.json new file mode 100644 index 0000000000000000000000000000000000000000..b48e40e5a84e96596458151ad06949a403fff082 --- /dev/null +++ b/processed_dataset/proof/2097.json @@ -0,0 +1,8 @@ +{ + "source_file": "./raw_volume-zh/volume9/exercise8.tex", + "problem_type": "proof", + "problem": "问题5. 证明:设 $P$ 是 $\\triangle A_1 A_2 A_3$ 所在平面上的任意一点,则\n$$\n\\left(a_2^2+a_3^2-a_1^2\\right) R_1^2+\\left(a_3^2+a_1^2-a_2^2\\right) R_2^2+\\left(a_1^2+a_2^2-a_3^2\\right) R_3^2 \\geqslant \\frac{16}{3} \\Delta^2,\n$$\n其中 $\\Delta$ 表示 $\\triangle A_1 A_2 A_3$ 的面积, $a_1 、 a_2 、 a_3$ 是它的边长, $P A_i=R_i, i=1$, 2 , 3 .", + "solution": "在惯性矩不等式中, 令 $x=a_2^2+a_3^2-a_1^2$, 等等, 可得左边 $\\geqslant \\sum\\left(a_3^2+\\right. \\left.a_1^2-a_2^2\\right)\\left(a_1^2+a_2^2-a_3^2\\right) \\cdot \\frac{a_1^2}{a_1^2+a_2^2+a_3^2} \\cdots$ (1), 又注意到 $a_1^2+a_2^2+a_3^2 \\leqslant 9 R^2, R= \\frac{a_1 a_2 a_3}{4 \\Delta}$, 结合可得 $\\frac{a_1^2 a_2^2 a_3^3}{a_1^2+a_2^2+a_3^2} \\geqslant \\frac{16}{9} \\Delta^2 \\cdots$ (2), 再注意到 Heron 公式 $16 \\Delta^2= 2\\left(a_1^2 a_2^2+a_2^2 a_3^2+a_3^2 a_1^2\\right)-\\left(a_1^4+a_2^4+a_3^4\\right) \\cdots$ (3), 因此由 (1)、(2)、(3) 可知 $\\sum\\left(a_3^2+\\right.$\n$$\n\\begin{aligned}\n& \\left.a_1^2-a_2^2\\right)\\left(a_1^2+a_2^2-a_3^2\\right) \\frac{a_1^2}{a_1^2+a_2^2+a_3^2} \\geqslant-16 \\Delta^2+\\frac{12 a_1^2 a_2^2 a_3^2}{a_1^2+a_2^2+a_3^2} \\geqslant-16 \\Delta^2+12 . \\\\\n& \\frac{16}{9} \\Delta^2=\\frac{16}{3} \\Delta^2 .\n\\end{aligned}\n$$", + "remark": "", + "figures": [] +} \ No newline at end of file diff --git a/processed_dataset/proof/2098.json b/processed_dataset/proof/2098.json new file mode 100644 index 0000000000000000000000000000000000000000..bb1d968991746c4e243afa55a43cb7283654b911 --- /dev/null +++ b/processed_dataset/proof/2098.json @@ -0,0 +1,8 @@ +{ + "source_file": "./raw_volume-zh/volume9/exercise9.tex", + "problem_type": "proof", + "problem": "问题2. (Tsintsifas) 设 $P$ 是 $\\triangle A B C$ 的一个内点, $\\triangle A^{\\prime} B^{\\prime} C^{\\prime}$ 是点 $P$ 的垂足三角形, $\\triangle A B C$ 和 $\\triangle A^{\\prime} B^{\\prime} C^{\\prime}$ 的三边分别是 $a 、 b 、 c$ 和 $a^{\\prime} 、 b^{\\prime} 、 c^{\\prime}$, 求证:\n$$\n\\frac{a^{\\prime}}{a}+\\frac{b^{\\prime}}{b}+\\frac{c^{\\prime}}{c}<2\n$$", + "solution": "提示: 用 $a^{\\prime}=P A \\sin A$ 等代入易证.", + "remark": "", + "figures": [] +} \ No newline at end of file diff --git a/processed_dataset/text/0001.json b/processed_dataset/text/0001.json new file mode 100644 index 0000000000000000000000000000000000000000..18b022d315b4285881c77505a6423e4be3f8ebdf --- /dev/null +++ b/processed_dataset/text/0001.json @@ -0,0 +1,5 @@ +{ + "source_file": "./raw_volume-zh/volume1/chapter1.tex", + "text": "一、集合的概念集合是一个原始的概念,是数学中一个不定义的概念.\n管如此,对一个具体的集合而言,很多情况下我们还是可以采用列举法或描述法给出它的一个准确而清晰的表示.\n用描述法表示一个集合基于下面的概括原则:\n概括原则对任给的一个性质 ${\\mathbf{}}P$ ,存在一个集合 ${\\mathsf{S}},$ 它的元素恰好是具有性质P的所有对象,即\n$S = \\{x | P(x)\\}$,\n其中 $\\textstyle P(x)$ 表示“$x$具有性质 ${\\mathbf{}}P$ ”\n由此,我们知道集合的元素是完全确定的,同时它的元素之间具有互异性和无序性.\n集合的元素个数为有限数的集合称为有限集,元素个数为无限数的集合称为无限集.\n果有限集 $A$的元素个数为$n$,则称 $A$为$n$元集,记作 $|A|=n.$ 空集不含任何元素.", + "figures": [] +} \ No newline at end of file diff --git a/processed_dataset/text/0002.json b/processed_dataset/text/0002.json new file mode 100644 index 0000000000000000000000000000000000000000..ddf625b201684b7f223be0c15e112a91a071ad4b --- /dev/null +++ b/processed_dataset/text/0002.json @@ -0,0 +1,5 @@ +{ + "source_file": "./raw_volume-zh/volume1/chapter1.tex", + "text": "二、集合与集合的关系在两个集合的关系中,子集是一个重要的概念,它的两个特例是真子集和集合相等.\n下面“充分必要条件”的角度来理解子集、真子集和集合相等的概念无疑是十分有益的:\n子集: $A\\subseteq B\\Longleftrightarrow$ 对任意 $x\\in A.$ ,恒有 $x\\in B$ ;\n真子集: $A\\subsetneq B\\Longleftrightarrow A\\subseteq B$, 且存在$x^{\\prime}\\in B$ ,但 $x^{\\prime}\\not\\in{\\boldsymbol{A}}$;\n集合相等: $A=B\\Longleftrightarrow A\\subseteq B$ ,且 $B\\subseteq A.$ \n容易证明两个集合关系的如下性质\n1. $\\emptyset \\subseteq A,\\,\\emptyset\\subseteq A\\ (A\\neq\\emptyset)$;\n2. $A\\subseteq B,\\,B\\subseteq C\\Rightarrow A\\subseteq C$ ;\n3. $n$ 元集$A$ 总共有 $2^n$ 个不同的子集.", + "figures": [] +} \ No newline at end of file diff --git a/processed_dataset/text/0003.json b/processed_dataset/text/0003.json new file mode 100644 index 0000000000000000000000000000000000000000..02a57ffc70e7b491abcf9bce3d5baac549eb56bb --- /dev/null +++ b/processed_dataset/text/0003.json @@ -0,0 +1,5 @@ +{ + "source_file": "./raw_volume-zh/volume1/chapter1.tex", + "text": "三、相关问题举例我们来研究一些与元素和集合有关的稍难的问题, 解决这些问题需要借助其他数学工具.", + "figures": [] +} \ No newline at end of file diff --git a/processed_dataset/text/0004.json b/processed_dataset/text/0004.json new file mode 100644 index 0000000000000000000000000000000000000000..dcefcc5d0f81663408c8e4e1893c62ce28943b27 --- /dev/null +++ b/processed_dataset/text/0004.json @@ -0,0 +1,5 @@ +{ + "source_file": "./raw_volume-zh/volume1/chapter2.tex", + "text": "集合的交集、并集、补集三种基本运算是通过元素与集合的关系来定义的:\n$$\n\\begin{aligned}\n& A \\cap B=\\{x \\mid x \\in A, \\text { 且 } x \\in B\\}, \\\\\n& A \\cup B=\\{x \\mid x \\in A, \\text { 或 } x \\in B\\}, \\\\\n& \\complement_U A=\\{x \\mid A \\subseteq U, x \\in U, \\text { 且 } x \\notin A\\} .\n\\end{aligned}\n$$\n请注意这里的逻辑关联词“且”、“或”, 它们在集合运算的定义中起了决定性的作用.\n有时, 我们还要用到集合的差集的概念.\n定义由属于集合 $A$ 但不属于集合 $B$ 的全体元素组成的集合叫做集合\n$A$ 对 $B$ 的差集, 记作 $A \\backslash B$ (或 $A-B$ ), 即\n$$\nA \\backslash B=\\{x \\mid x \\in A \\text {, 且 } x \\notin B\\} .\n$$\n由这个定义可以看出, 补集只是差集的一种特殊情况.\n记 $U$ 为全集, 容易证明集合的运算满足如下法则:\n(1) 等幂律: $A \\cap A=A, A \\cup A=A$;\n(2) 同一律: $A \\cap U=A, A \\cup U=U$,\n$$\nA \\cap \\varnothing=\\varnothing, A \\cup \\varnothing=A ;\n$$\n(3) 互补律: $A \\cap \\complement_U A=\\varnothing, A \\cup \\complement_U A=U$;\n(4) 交换律: $A \\cap B=B \\cap A, A \\cup B=B \\cup A$;\n(5) 结合律: $A \\cap(B \\cap C)=(A \\cap B) \\cap C$,\n$$\nA \\cup(B \\cup C)=(A \\cup B) \\cup C\n$$\n(6) 分配律: $A \\cap(B \\cup C)=(A \\cap B) \\cup(A \\cap C)$,\n$$\nA \\cup(B \\cap C)=(A \\cup B) \\cap(A \\cup C) \\text {; }\n$$\n(7) 吸收律: $A \\cup(A \\cap B)=A, A \\cap(A \\cup B)=A$;\n(8) 反演律 (摩根律) : $\\complement_U(A \\cap B)=\\complement_U A \\cup \\complement_U B$,\n$$\n\\complement_U(A \\cup B)=\\complement_U A \\cap \\complement_U B .\n$$\n利用维恩图可以清晰地理解集合的交、并、补、差运算及其运算律.\n维恩图为集合问题的解决提供了一个直观的工具.", + "figures": [] +} \ No newline at end of file diff --git a/processed_dataset/text/0005.json b/processed_dataset/text/0005.json new file mode 100644 index 0000000000000000000000000000000000000000..dca84c8315d781ee93d7f6cb8c46a16d1e0b614c --- /dev/null +++ b/processed_dataset/text/0005.json @@ -0,0 +1,5 @@ +{ + "source_file": "./raw_volume-zh/volume1/chapter3.tex", + "text": "我们知道集合可以分为有限集和无限集两类.\n研究无限集元素的“数目” 是一个困难而有趣的问题, 最出名的就是所谓 “连续统假设”, 但它不是我们的话题.\n我们要讨论的问题仅与有限集有关.\n一、有限集的阶有限集 $A$ 的元素的数目叫做这个集合的阶, 记作 $|A|$ (或 $n(A))$.", + "figures": [] +} \ No newline at end of file diff --git a/processed_dataset/text/0006.json b/processed_dataset/text/0006.json new file mode 100644 index 0000000000000000000000000000000000000000..ad6b5be43e8642097b8d53689378bfb0ee16b098 --- /dev/null +++ b/processed_dataset/text/0006.json @@ -0,0 +1,5 @@ +{ + "source_file": "./raw_volume-zh/volume1/chapter3.tex", + "text": "二、有关集合阶的不等式有些集合虽然不能准确求出其元素的数目,但是我们可以利用不等式来估计其阶的范围.", + "figures": [] +} \ No newline at end of file diff --git a/processed_dataset/text/0007.json b/processed_dataset/text/0007.json new file mode 100644 index 0000000000000000000000000000000000000000..fc00f7ed44e48c87f0cbc7a8820c3ab6c91e364c --- /dev/null +++ b/processed_dataset/text/0007.json @@ -0,0 +1,5 @@ +{ + "source_file": "./raw_volume-zh/volume1/chapter3.tex", + "text": "三、有关集合阶的最大(小)值对于满足一定条件的一组集合, 如何确定集合元素数目的最大 (小)值, 这也是一类常见的问题.", + "figures": [] +} \ No newline at end of file diff --git a/processed_dataset/text/0008.json b/processed_dataset/text/0008.json new file mode 100644 index 0000000000000000000000000000000000000000..fb1026fe77c72aca82ea8131a99f46202244092d --- /dev/null +++ b/processed_dataset/text/0008.json @@ -0,0 +1,5 @@ +{ + "source_file": "./raw_volume-zh/volume1/chapter4.tex", + "text": "集合的分划一个集合可以写成若干个集合的并集,例如集合 $\\{1,2, 3, 4, 5\\}$, 可以写成两个集合 $A=\\{1,2,3\\}, B=\\{3,4,5\\}$ 的并集 $A \\cup B$; 也可以写成三个集合 $C=\\{1,2\\}, D=\\{3,5\\}, E=\\{4\\}$ 的并集 $C \\cup D \\cup E$, 等等.\n下面我们来研究将一个集合表示成若干个集合的并集的一种特殊情形.\n定义把一个集合 $M$ 分成 $n$ 个非空的子集: $A_1, A_2, \\cdots, A_n$, 如果:\n(1) $A_i \\cap A_j=\\varnothing(1 \\leqslant i, j \\leqslant n, i \\neq j)$;\n(2) $\\bigcup_{i=1}^n A_i=M$,\n那么,这些子集的全体叫做集合 $M$ 的一个 $n$-分划.\n由集合分划的定义, 容易证明有限集的一个非常有用的性质:\n加法原理设 $A_1, A_2, \\cdots, A_n$ 是有限集 $M$ 的一个 $n$-分划, 则\n$$\n|M|=\\sum_{i=1}^n\\left|A_i\\right|\n$$\n这是一个基本的计数公式.\n集合的分划引出了大量有趣的数学问题.", + "figures": [] +} \ No newline at end of file diff --git a/processed_dataset/text/0009.json b/processed_dataset/text/0009.json new file mode 100644 index 0000000000000000000000000000000000000000..93e4ea516d332232fa08d54c506caaef62fea62d --- /dev/null +++ b/processed_dataset/text/0009.json @@ -0,0 +1,5 @@ +{ + "source_file": "./raw_volume-zh/volume1/chapter5.tex", + "text": "我们可以将某些集合取来作为元素构成一个新的集合, 如 $A^*=\\{\\{1\\},\\{0,1\\},\\{0\\}, \\varnothing\\}$ 就是一个含有 4 个元素 $\\{1\\} 、\\{0,1\\} 、\\{0\\} 、 \\varnothing$ 的集合.\n特别地, 将集合 $M$ 的若干子集作为元素构成的集合 $M^*$ 叫做原集合的一个子集族.\n例如前面的 $A^*$ 就是二元集 $A=\\{0,1\\}$ 的全部子集所构成的子集族.\n子集族中所含原来集合的子集的数目叫做该子集族的阶.\n例如子集族 $A^*$ 的阶为 4 , 即 $\\left|A^*\\right|=4$.\n一、C 族最简单的子集族是由有限集 $M$ 的全体子集所构成的子集族, 简称为 $C$ 族.\n$C$ 族有如下基本的性质:\n性质设 $|M|=n$, 则集合 $M$ 的全部子集构成的集合 $M^*$ 的阶为 $2^n$, 即\n$$\n\\left|M^*\\right|=\\mathrm{C}_n^0+\\mathrm{C}_n^1+\\cdots+\\mathrm{C}_n^n=2^n .\n$$", + "figures": [] +} \ No newline at end of file diff --git a/processed_dataset/text/0010.json b/processed_dataset/text/0010.json new file mode 100644 index 0000000000000000000000000000000000000000..d45dbecd297ddc3895bdfa22a7ab407f7024cf47 --- /dev/null +++ b/processed_dataset/text/0010.json @@ -0,0 +1,5 @@ +{ + "source_file": "./raw_volume-zh/volume1/chapter5.tex", + "text": "二、求解子集族求解子集族的问题主要有两类: 求子集族的阶, 或确定集合的满足特定条件的子集族中的每个集合.", + "figures": [] +} \ No newline at end of file diff --git a/processed_dataset/text/0011.json b/processed_dataset/text/0011.json new file mode 100644 index 0000000000000000000000000000000000000000..cefd6b3cc4dab4f86ba18daec691b7df0f2205f2 --- /dev/null +++ b/processed_dataset/text/0011.json @@ -0,0 +1,5 @@ +{ + "source_file": "./raw_volume-zh/volume1/chapter6.tex", + "text": "集合的性质很多集合问题实际上就是研究集合中元素的性质问题,前面的每一节都能找到这样的例子.\n面,我们再通过一些例子进一步探讨研究集合性质的技巧.\n一、集合中全部元素的性质已知集合 $S=\\{x \\mid P(x)\\}$. 如果由性质 $P$ 能推出 $S$ 中每个元素都满足的性质 $P^{\\prime}$, 那么 $P^{\\prime}$ 就是 $P$ 的一个必要条件.\n设 $S^{\\prime}=\\left\\{x \\mid P^{\\prime}(x)\\right\\}$, 显然有 $S \\subseteq S^{\\prime}$.", + "figures": [] +} \ No newline at end of file diff --git a/processed_dataset/text/0012.json b/processed_dataset/text/0012.json new file mode 100644 index 0000000000000000000000000000000000000000..8863c7baa50a8dfb97b4c63f03db6bc862fe5a48 --- /dev/null +++ b/processed_dataset/text/0012.json @@ -0,0 +1,5 @@ +{ + "source_file": "./raw_volume-zh/volume1/chapter6.tex", + "text": "二、集合子集元素的性质设集合 $S=\\{x \\mid P(x)\\}$. 如果条件 $P^*$ 是条件 $P$ 的充分条件,那么集合\n$$\nS^*=\\left\\{x \\mid P^*(x), x \\in S\\right\\}\n$$\n是集合 $S$ 的子集, 即 $S^* \\subseteq S$. 这里 $P^*$ 是集合 $S$ 中部分元素的性质.\n我们还可以通过增加 $S$ 的“内涵”的方式来缩小它的“外延”: $S$ 是所有具备性质 $P$ 的元素 $x$ 的集合,增加新的性质 $P^*$, 得到集合\n$$\nS^*=\\left\\{x \\mid P(x) \\text { 且 } P^*(x), x \\in S\\right\\},\n$$\n显然 $S^*=\\left\\{x \\mid P^*(x), x \\in S\\right\\}$, 它是 $S$ 的子集, 即 $S^* \\subseteq S$.\n一类典型的问题就是从集合 $S$ 中分离出所有满足性质 $P^*$ 的元素, 从而得到所求的 $S^*$.", + "figures": [] +} \ No newline at end of file diff --git a/processed_dataset/text/0013.json b/processed_dataset/text/0013.json new file mode 100644 index 0000000000000000000000000000000000000000..4103ac72303f747cd91b8763c936c69a94effd37 --- /dev/null +++ b/processed_dataset/text/0013.json @@ -0,0 +1,5 @@ +{ + "source_file": "./raw_volume-zh/volume1/chapter6.tex", + "text": "三、其他有关集合性质的问题有关集合性质的问题丰富多彩,除了上面两类典型的问题外很难作一个系统的分类.\n其实, 集合问题大多具有明显的组合色彩, 解题方法各异, 分类并没有实质意义.\n下面我们再看几个例子.", + "figures": [] +} \ No newline at end of file diff --git a/processed_dataset/text/0014.json b/processed_dataset/text/0014.json new file mode 100644 index 0000000000000000000000000000000000000000..b100a216c82fdb0c232e86db86bcd3b75372be63 --- /dev/null +++ b/processed_dataset/text/0014.json @@ -0,0 +1,5 @@ +{ + "source_file": "./raw_volume-zh/volume1/chapter7.tex", + "text": "分类原则自本节开始, 我们把注意力转向集合知识和由集合知识派生出来的数学方法的应用上.\n首先我们来看与集合的分划有关的所谓分类问题.\n一、分类原则在我们的经验中, 有些数学问题涉及的对象较复杂, 统一地解决有困难, 于是就将这些对象分成“不重不漏”的若干类, 然后逐类解决.\n这就是分类解决问题的方法.\n分类的基本原则就是每次分类必须不重不漏, 其理论依据就是集合的分划.\n分类原则设所研究的对象的全体形成集合 $M, A_1, A_2, \\cdots, A_n$ 是 $M$ 的一组非空子集, 且\n(1) $A_i \\cap A_j=\\varnothing, 1 \\leqslant i, j \\leqslant n, i \\neq j$;\n(2) $\\bigcup_{i=1}^n A_i=M$,\n那么,这组子集叫做研究对象的全体的一个 $n$-分类,其中每一个一集叫做所。 研究对象的一个类.", + "figures": [] +} \ No newline at end of file diff --git a/processed_dataset/text/0015.json b/processed_dataset/text/0015.json new file mode 100644 index 0000000000000000000000000000000000000000..8e082d8769b7bbbc5338f826d32ec574778d6655 --- /dev/null +++ b/processed_dataset/text/0015.json @@ -0,0 +1,5 @@ +{ + "source_file": "./raw_volume-zh/volume1/chapter8.tex", + "text": "二、代数与数论问题在代数不等式中, 有一类确定满足不等关系的量是否存在的问题, 通常可以尝试用最小数原理来解决.", + "figures": [] +} \ No newline at end of file diff --git a/processed_dataset/text/0016.json b/processed_dataset/text/0016.json new file mode 100644 index 0000000000000000000000000000000000000000..ba980304ef6f714e6a6b264746d0312016d4ddef --- /dev/null +++ b/processed_dataset/text/0016.json @@ -0,0 +1,5 @@ +{ + "source_file": "./raw_volume-zh/volume1/chapter9.tex", + "text": "容不原理本节我们进一步讨论如何计算有限集的阶的问题.\n设 $M$ 为非空有限集, 非空集合\n$$\nA_1, A_2, \\cdots, A_n\n$$\n是 $M$ 的一个子集族, 且满足\n$$\nA_1 \\cup A_2 \\cup \\cdots \\cup A_n=\\bigcup_{i=1}^n A_i=M,\n$$\n则称子集族 $\\mathscr{A}: A_1, A_2, \\cdots, A_n$ 是集合 $M$ 的一个覆盖.\n我们的问题是, 如何通过计算覆盖 $\\mathscr{A}: A_1, A_2, \\cdots, A_n$ 中每个子集的阶来计算有限集 $M$ 的阶.\n一、加法原理我们先来看一个简单的情形.\n如果子集族 $\\mathscr{A}$ 既满足 (i), 又满足\n$$\nA_i \\cap A_j=\\varnothing, 1 \\leqslant i \\neq j \\leqslant n,\n$$\n那么覆盖 $A_1, A_2, \\cdots, A_n$ 就是有限集 $M$ 的一个 $n$ 一分划.\n对于有限集 $M$ 的 $n$ 一分划, 我们有下面非常有用的结论.\n加法原理设 $M$ 为非空有限集, $A_1, A_2, \\cdots, A_n$ 是 $M$ 的一个由非空子集构成的 $n$-分划,那么\n$$\n|M|=\\left|A_1\\right|+\\left|A_2\\right|+\\cdots+\\left|A_n\\right| .\n$$\n加法原理是组合数学中一个基本的计数原理.\n在实际运用中可根据问题的不同背景赋予有限集 $M$ 的元素不同的含义.", + "figures": [] +} \ No newline at end of file diff --git a/processed_dataset/text/0017.json b/processed_dataset/text/0017.json new file mode 100644 index 0000000000000000000000000000000000000000..af9eba19bbba621d846c9d521a1129fcc2b61900 --- /dev/null +++ b/processed_dataset/text/0017.json @@ -0,0 +1,5 @@ +{ + "source_file": "./raw_volume-zh/volume1/chapter9.tex", + "text": "二、容斥原理的简单形式如果条件(ii)不一定满足, 也就是说可能存在 $1 \\leqslant p \\neq q \\leqslant n$, 使\n$$\nA_p \\cap A_q \\neq \\varnothing\n$$\n时, $\\left|A_1\\right|,\\left|A_2\\right|, \\cdots,\\left|A_n\\right|$ 与 $|M|$ 有什么关系呢? 我们还是先来看比较简单的情形.\n定理 $1\\left|A_1 \\cup A_2\\right|=\\left|A_1\\right|+\\left|A_2\\right|-\\left|A_1 \\cap A_2\\right|$.\n证明设 $A_1 \\cap A_2=B, A_1^{\\prime}=A_1 \\backslash B, A_2^{\\prime}=A_2 \\backslash B$, 则\n$$\nA_1 \\cup A_2=A_1^{\\prime} \\cup A_2^{\\prime} \\cup B \\text {. }\n$$\n由加法原理知, $\\left|A_1^{\\prime}\\right|=\\left|A_1\\right|-|B|,\\left|A_2^{\\prime}\\right|=\\left|A_2\\right|-|B|$, 所以\n$$\n\\begin{aligned}\n\\left|A_1 \\cup A_2\\right| & =\\left|A_1^{\\prime} \\cup A_2^{\\prime} \\cup B\\right|=\\left|A_1^{\\prime}\\right|+\\left|A_2^{\\prime}\\right|+|B| \\\\\n& =\\left(\\left|A_1\\right|-|B|\\right)+\\left(\\left|A_2\\right|-|B|\\right)+|B| \\\\\n& =\\left|A_1\\right|+\\left|A_2\\right|-\\left|A_1 \\cap A_2\\right| .\n\\end{aligned}\n$$\n定理 2 设 $A_1 、 A_2$ 是集合 $S$ 的子集,则\n$$\n\\left|\\complement_S A_1 \\cap \\complement_S A_2\\right|=|S|-\\left|A_1\\right|-\\left|A_2\\right|+\\left|A_1 \\cap A_2\\right| .\n$$\n证明由摩根定律及加法原理有\n$$\n\\left|\\complement_S A_1 \\cap \\complement_S A_2\\right|=\\left|\\complement_S\\left(A_1 \\cup A_2\\right)\\right|=|S|-\\left|A_1 \\cup A_2\\right| .\n$$\n又由定理 1 得\n$$\n\\left|\\complement_S A_1 \\cap \\complement_S A_2\\right|=|S|-\\left|A_1\\right|-\\left|A_2\\right|+\\left|A_1 \\cap A_2\\right| .\n$$\n定理 1 及定理 2 是容斥原理的简单形式, 可以用来解决一些简单的计数问题.", + "figures": [] +} \ No newline at end of file diff --git a/processed_dataset/text/0018.json b/processed_dataset/text/0018.json new file mode 100644 index 0000000000000000000000000000000000000000..8d729dfa501dec891c85e7cd9e25056ca63ded66 --- /dev/null +++ b/processed_dataset/text/0018.json @@ -0,0 +1,5 @@ +{ + "source_file": "./raw_volume-zh/volume1/chapter9.tex", + "text": "三、容有原理的一般形式定理 3\n$$\n\\begin{aligned}\n\\left|\\bigcup_{i=1}^n A_i\\right|= & \\sum_{i=1}^n\\left|A_i\\right|-\\sum_{1 \\leqslant i0), a$ 当然未必被 $b$ 整除,但我们有下面的结论一一带余除法, 这是初等数论中最为基本的一个结果.\n(4) (带余除法) 设 $a 、 b$ 为整数, $b>0$, 则存在整数 $q$ 和 $r$, 使得\n$$\na=b q+r, \\text { 其中 } 0 \\leqslant r0$ 时,可选择 $x$ 为正 (负) 数, 此时 $y$ 则相应地为负 (正) 数.\n由 (1)易于推出下面的\n(2) 两个整数 $a 、 b$ 互素的充分必要条件是存在整数 $x 、 y$,使得\n$$\na x+b y=1 \\text {. }\n$$\n事实上, 条件的必要性是 (1) 的特例.\n反过来, 若有 $x 、 y$ 使等式成立, 设 $(a, b)=d$, 则 $d \\mid a$ 且 $d \\mid b$, 故 $d \\mid a x$ 及 $d \\mid b y$,于是 $d \\mid(a x+b y)$, 即 $d \\mid 1$, 从而 $d=1$.\n由(1)及 (2)不难导出下面的几个基本结论:\n(3) 若 $m|a, m| b$, 则 $m \\mid(a, b)$, 即 $a 、 b$ 的任一个公约数都是它们的最大公约数的约数.\n(4) 若 $m>0$, 则 $(m a, m b)=m(a, b)$.\n(5) 若 $(a, b)=d$, 则 $\\left(\\frac{a}{d}, \\frac{b}{d}\\right)=1$. 因此, 由两个不互素的整数,可自然地产生一对互素的整数.\n(6) 若 $(a, m)=1,(b, m)=1$, 则 $(a b, m)=1$. 这表明,与一个固定整数互素的整数之集关于乘法封闭.\n由此可推出: 若 $(a, b)=1$, 则对任意 $k>0$ 有 $\\left(a^k, b\\right)=1$, 进而对任意 $l>0$ 有 $\\left(a^k, b^l\\right)=1$.\n(7) 设 $b \\mid a c$. 若 $(b, c)=1$, 则 $b \\mid a$.\n(8) 设正整数 $a 、 b$ 之积是一个整数的 $k$ 次幂 $(k \\geqslant 2)$. 若 $(a, b)=1$, 则 $a 、 b$ 都是整数的 $k$ 次幂.\n一般地, 设正整数 $a, b, \\cdots, c$ 之积是一个整数的 $k$ 次幕.\n若 $a, b, \\cdots, c$ 两两互素, 则 $a, b, \\cdots, c$ 都是整数的 $k$ 次幂.\n(6)、(7)、(8)表现了互素的重要性, 它们的应用也最为广泛.\n现在, 我们简单地谈谈最小公倍数.\n设 $a 、 b$ 是两个非零整数,一个同时为 $a 、 b$ 倍数的数称为它们的一个公倍数.\n$a 、 b$ 的公倍数显然有无穷多个, 这其中最小的正数称为 $a 、 b$ 的最小公倍数,记作 $[a, b]$. 对于多个非零整数 $a, b, \\cdots, c$, 可类似地定义它们的最小公倍数 $[a, b, \\cdots, c]$.\n下面是最小公倍数的主要性质.\n(9) $a$ 与 $b$ 的任一公倍数都是 $[a, b]$ 的倍数.\n对于多于两个整数的情形, 类似的结论也成立.\n(10) 两个整数 $a 、 b$ 的最大公约数与最小公倍数满足\n$$\n(a, b)[a, b]=|a b| .\n$$\n但请注意, 对于多于两个整数的情形, 类似的结论不成立 (请读者举出例子). 然而我们有下面的\n(11) 若 $a, b, \\cdots, c$ 两两互素,则有\n$$\n[a, b, \\cdots, c]=|a b \\cdots c| .\n$$\n由此及(9) 可知, 若 $a|d, b| d, \\cdots, c \\mid d$, 且 $a, b, \\cdots, c$ 两两互素, 则有 $a b{ }^{\\prime} \\cdot c \\mid d$.\n互素,在数论中相当重要, 往往是许多问题的关键或基础.\n数学竞赛中, 有一些问题要求证明两个整数互素 (或求它们的最大公约数), 下面几个例子表现了处理这些问题的一个基本方法.", + "figures": [] +} \ No newline at end of file diff --git a/processed_dataset/text/0022.json b/processed_dataset/text/0022.json new file mode 100644 index 0000000000000000000000000000000000000000..25a87f3c4be4bc8bc9107cfeb6895b9825094b41 --- /dev/null +++ b/processed_dataset/text/0022.json @@ -0,0 +1,5 @@ +{ + "source_file": "./raw_volume-zh/volume10/chapter3.tex", + "text": "素数及唯一分解定理.\n大于 1 的整数 $n$ 总有两个不同的正约数: 1 和 $n$. 若 $n$ 仅有这两个正约数(称 $n$ 没有真因子), 则称 $n$ 为素数 (或质数). 若 $n$ 有真因子, 即 $n$ 可表示为 $a \\cdot b$ 的形式 (这里 $a 、 b$ 为大于 1 的整数), 则称 $n$ 为合数.\n于是,正整数被分成三类: 数 1 单独作一类, 素数类及合数类.\n素数在正整数中特别重要, 我们常用字母 $p$ 表示素数.\n由定义易得出下面的基本结论:\n(1) 大于 1 的整数必有素约数.\n这是因为, 大于 1 的整数当然有大于 1 的正约数, 这些约数中的最小数必然没有真因子,从而是素数.\n(2) 设 $p$ 是素数, $n$ 是任意一个整数,则或者 $p$ 整除 $n$, 或者 $p$ 与 $n$ 互素.\n事实上, $p$ 与 $n$ 的最大公约数 $(p, n)$ 必整除 $p$, 故由素数的定义推知, 或者 $(p, n)=1$, 或者 $(p, n)=p$, 即或者 $p$ 与 $n$ 互素,或者 $p \\mid n$.\n素数的最为锐利的性质是下面的\n(3) 设 $p$ 是素数, $a 、 b$ 为整数.\n若 $p \\mid a b$, 则 $a 、 b$ 中至少有一个数被 $p$ 整除.\n实际上, 若 $p$ 不整除 $a$ 和 $b$, 则由上述的 (2), $p$ 与 $a 、 b$ 均互素, 从而 $p$ 与 $a b$ 互素 (见第 2 单元 (6)), 这与已知的 $p \\mid a b$ 相违!\n由 (3)特别地推出, 若素数 $p$ 整除 $a^n(n \\geqslant 1)$, 则 $p \\mid a$.\n关于素数的最为经典的一个结果是公元前欧几里得证明的:\n(4) 素数有无穷多个.\n我们用反证法来证明这一事实.\n假设素数只有有限多个, 设全体素数为 $p_1, p_2, \\cdots, p_k$. 考虑数 $N=p_1 p_2 \\cdots p_k+1$, 显然 $N>1$, 故 $N$ 有素因子 $p$. 因 $p_1, p_2, \\cdots, p_k$ 是全部素数, 故 $p$ 必等于某个 $p_i(1 \\leqslant i \\leqslant k)$, 从而 $p$ 整除 $N- p_1 p_2 \\cdots p_k$, 即 $p$ 整除 1 , 这不可能.\n因此素数有无穷多个.\n(请注意, $p_1 \\cdots p_k+1$ 并不一定是素数.)\n(4) 中的断言, 也可由第 2 单元例 3 推出来: 设 $F_k=2^{2^k}+1(k \\geqslant 0)$, 则\n$F_k>1$, 故 $F_k$ 有素约数.\n因已证明无穷数列 $\\left\\{F_k\\right\\}(k \\geqslant 0)$ 中的项两两互素, 故每个 $F_k$ 的素约数与这个数列中其他项的素约数不同, 因此素数必有无穷多个.\n现在我们转向初等数论中最为基本的一个结果, 即正整数的唯一分解定理,也称为算术基本定理,它表现了素数在正整数集合中的真正分量.\n(5) (唯一分解定理) 每个大于 1 的正整数均可分解为有限个素数的积; 并且, 若不计素因数在乘积中的次序, 这样的分解是唯一的.\n换句话说,设 $n>1$, 则 $n$ 必可表示为 $n=p_1 p_2 \\cdots p_k$, 其中 $p_i(1 \\leqslant i \\leqslant k)$ 都是素数; 并且,若 $n$ 有两种素因数分解\n$$\nn=p_1 p_2 \\cdots p_k=q_1 q_2 \\cdots q_l,\n$$\n则必有 $k=l$, 并且 $p_1, p_2, \\cdots, p_k$ 是 $q_1, q_2, \\cdots, q_l$ 的一个排列.\n将 $n$ 的素因数分解中的相同的素因子收集在一起,可知每个大于 1 的正整数 $n$ 可唯一地表示为\n$$\nn=p_1^{\\alpha_1} p_2^\\alpha \\cdots p_k^{\\alpha_k},\n$$\n其中 $p_1, p_2, \\cdots, p_k$ 是互不相同的素数, $\\alpha_1, \\alpha_2, \\cdots, \\alpha_k$ 是正整数,这称为 $n$ 的标准分解.\n若已知正整数 $n$ 的 (如上所述的)标准分解,则由唯一分解定理,可确定其全部的正约数:\n(6) $n$ 的全部正约数为 $p_1^{\\beta_1} p_2^{\\beta_2} \\cdots p_k^{\\beta_k}$, 其中 $\\beta_i$ 是满足 $0 \\leqslant \\beta_i \\leqslant \\alpha_i(i=1, \\cdots$, $k$ ) 的任意整数.\n由此易知, 若设 $\\tau(n)$ 为 $n$ 的正约数的个数, $\\sigma(n)$ 为 $n$ 的正约数之和, 则有\n$$\n\\begin{aligned}\n& \\tau(n)=\\left(\\alpha_1+1\\right)\\left(\\alpha_2+1\\right) \\cdots\\left(\\alpha_k+1\\right), \\\\\n& \\sigma(n)=\\frac{p_1^{\\alpha_1+1}-1}{p_1-1} \\cdot \\frac{p^{\\alpha_2+1}-1}{p_2-1} \\cdots \\cdots \\cdot \\frac{p^{\\alpha_k+1}-1}{p_k-1} .\n\\end{aligned}\n$$\n虽然素数有无穷多,但它们在自然数中的分布却极不规则 . 给定一个大整数, 判定它是否为素数, 通常是极其困难的, 要作出其标准分解, 则更为困难.\n下面 (7) 中的结果相当有趣, 它对任意 $n>1$, 给出了 $n$ ! 的标准分解.\n(7) 对任意正整数 $m$ 及素数 $p$, 记号 $p^\\alpha \\| m$ 表示 $p^\\alpha \\mid m$, 但 $p^{\\alpha+1} \\nmid m$, 即 $p^\\alpha$ 是 $m$ 的标准分解中出现的 $p$ 的幕.\n设 $n>1, p$ 为素数, $p^{\\alpha_p} \\| n !$, 则\n$$\n\\alpha_p=\\sum_{l=1}^{\\infty}\\left[\\frac{n}{p^l}\\right]\\left(==\\left[\\frac{n}{p}\\right]+\\left[\\frac{n}{p^2}\\right]+\\cdots\\right)\n$$\n这里 $[x]$ 表示不超过实数 $x$ 的最大整数.\n请注意, 由于当 $p^l>n$ 时, $\\left[\\frac{n}{p^l}\\right]=0$, 故上面和式中只有有限多个项非零.\n证明某些特殊形式的数不是素数 (或给出其为素数的必要条件), 是初等数论中较为基本的问题, 在数学竞赛中尤为常见.\n处理这类问题的基本方法是应用 (各种)分解技术,指出所说数的一个真因子.\n我们举几个这样的例子.", + "figures": [] +} \ No newline at end of file diff --git a/processed_dataset/text/0023.json b/processed_dataset/text/0023.json new file mode 100644 index 0000000000000000000000000000000000000000..7bd1cc8ae20e48eeea9b95c7a943fa8e56ff118b --- /dev/null +++ b/processed_dataset/text/0023.json @@ -0,0 +1,5 @@ +{ + "source_file": "./raw_volume-zh/volume10/chapter4.tex", + "text": "不定方程, 是指未知数的个数多于方程的个数, 而未知数的取值范围受某些限制 (如整数、正整数、有理数等) 的方程.\n不定方程是数论的一个重要课题,数学竞赛中也常涉及这方面的问题.\n初等范围内, 处理不定方程主要有三种方法: 分解方法, 同余方法, 以及 (不等式)估计方法.\n分解方法则是最为基本的方法.\n分解方法的主要功效, 大致地说, 是通过 \"分解\" 将原方程分解为若干个易于处理的方程.\n这里说的\"分解\"包含两个方面的手法: 其一, 是代数 (整式) 的分解; 其二, 是应用整数的某些性质 (唯一分解定理, 互素的性质等) 导出适用的分解.\n分解方法当然没有固定的程序可循.\n有时, 分解相当困难, 或分解方式较多而难以选择; 有时, 进一步的论证则很不容易.\n本节的一些例子就已表现了这些.\n分解方法常和别的方法结合使用, 请参考本单元及后面的一些例子.", + "figures": [] +} \ No newline at end of file diff --git a/processed_dataset/text/0024.json b/processed_dataset/text/0024.json new file mode 100644 index 0000000000000000000000000000000000000000..eb269ea02b39fc856e231c39276dc6aea12f70b0 --- /dev/null +++ b/processed_dataset/text/0024.json @@ -0,0 +1,5 @@ +{ + "source_file": "./raw_volume-zh/volume10/chapter5.tex", + "text": "从前面几个单元的内容, 可以看出初等数论的一个显著特点一一灵活多样,数学竞赛中的数论问题尤其如此.\n本单元再选取一些这样的例子.", + "figures": [] +} \ No newline at end of file diff --git a/processed_dataset/text/0025.json b/processed_dataset/text/0025.json new file mode 100644 index 0000000000000000000000000000000000000000..a9bb872eda40bd9ad5c5ebb6df85212b8453ed02 --- /dev/null +++ b/processed_dataset/text/0025.json @@ -0,0 +1,5 @@ +{ + "source_file": "./raw_volume-zh/volume10/chapter6.tex", + "text": "同余, 是数论中的一个重要概念, 应用极为广泛.\n设 $n$ 是给定的正整数, 若整数 $a 、 b$ 满足 $n \\mid(a-b)$, 则称 $a$ 和 $b$ 模 $n$ 同余, 记作\n$$\na \\equiv b(\\bmod n) .\n$$\n若 $n \\nmid(a-b)$, 则称 $a$ 和 $b$ 模 $n$ 不同余, 记作\n$$\na \\not \\equiv b(\\bmod n) .\n$$\n由带余除法易知, $a$ 和 $b$ 模 $n$ 同余的充分必要条件是 $a$ 与 $b$ 被 $n$ 除得的余数相同.\n对于固定的模 $n$, 模 $n$ 的同余式与通常的等式有许多类似的性质:\n(1)(反身性) $a \\equiv a(\\bmod n)$.\n(2)(对称性) 若 $a \\equiv b(\\bmod n)$, 则 $b \\equiv a(\\bmod n)$.\n(3)(传递性) 若 $a \\equiv b(\\bmod n), b \\equiv c(\\bmod n)$, 则 $a \\equiv c(\\bmod n)$.\n(4)(同余式相加) 若 $a \\equiv b(\\bmod n), c \\equiv d(\\bmod n)$, 则 $a \\pm c \\equiv b \\pm d(\\bmod n)$.\n(5)(同余式相乘) 若 $a \\equiv b(\\bmod n), c \\equiv d(\\bmod n)$, 则 $a c \\equiv b d(\\bmod n)$.\n不难看到, 反复用(4)或 (5), 可以对多于两个的(模相同的)同余式建立加、减和乘法的运算公式.\n特别地, 由 (5) 易推出: 若 $a \\equiv b(\\bmod n), k, c$ 为整数且 $k>0$, 则\n$$\na^k c \\equiv b^k c(\\bmod n) .\n$$\n请注意, 同余式的消去律一般并不成立, 即从 $a c \\equiv b c(\\bmod n)$ 未必能推出 $a \\equiv b(\\bmod n)$. 然而,我们有下面的结果:\n(6) 若 $a c \\equiv b c(\\bmod n)$, 则 $a \\equiv b\\left(\\bmod \\frac{n}{(n, c)}\\right)$. 由此推出, 若 $(c, n)=1$, 则有 $a \\equiv b(\\bmod n)$, 即在 $c$ 与 $n$ 互素时, 可以在原同余式两边约去 $c$ 而不改变模(这再一次表现了互素的重要性).\n现在提及几个涉及模的简单但有用的性质.\n(7) 若 $a \\equiv b(\\bmod n)$, 而 $d \\mid n$, 则 $a \\equiv b(\\bmod d)$.\n(8) 若 $a \\equiv b(\\bmod n), d \\neq 0$, 则 $d a \\equiv d b(\\bmod d n)$.\n(9) 若 $a \\equiv b\\left(\\bmod n_i\\right)(i=1,2, \\cdots, k)$, 则 $a \\equiv b\\left(\\bmod \\left[n_1, n_2, \\cdots, n_k\\right]\\right)$. 特别地, 若 $n_1, n_2, \\cdots, n_k$ 两两互素, 则有 $a \\equiv b\\left(\\bmod n_1 n_2 \\cdots n_k\\right)$.\n由上述的性质 (1)、(2)、(3) 可知, 整数集合可以按模 $n$ 来分类, 确切地说, 若 $a$ 和 $b$ 模 $n$ 同余, 则 $a$ 与 $b$ 属同一个类, 否则不属于同一个类, 每一个这样的类称为模 $n$ 的一个同余类.\n由带余除法,任一整数必恰与: $0,1, \\cdots, n-1$ 中的一个模 $n$ 同余, 而 0 , $1, \\cdots, n-1$ 这 $n$ 个数彼此模 $n$ 不同余, 因此模 $n$ 共有 $n$ 个不同的同余类, 即为\n$$\nM_i=\\{x \\mid x \\in \\mathbf{Z}, x \\equiv i(\\bmod n)\\}, i=0,1, \\cdots, n-1 .\n$$\n例如, 模 2 的同余类共有两个, 即通常说的偶数类与奇数类.\n两个类中的数分别具有形式 $2 k$ 与 $2 k+1$ ( $k$ 为任意整数).\n在 $n$ 个剩余类中各任取一个数作为代表, 这样的 $n$ 个数称为模 $n$ 的一个完全剩余系,简称模 $n$ 的完系.\n换句话说, $n$ 个数 $c_1, c_2, \\cdots, c_n$ 称为模 $n$ 的一个完系, 是指它们彼此模 $n$ 不同余.\n例如, $0,1, \\cdots, n-1$ 是模 $n$ 的一个完系, 这称作模 $n$ 的最小非负完系.\n易于看到,若 $i$ 和 $n$ 互素, 则同余类 $M_i$ 中的所有数都和 $n$ 互素,这样的同余类称为模 $n$ 的缩同余类.\n我们将模 $n$ 的缩同余类的个数记作 $\\varphi(n)$, 称为欧拉函数, 这是数论中的一个重要函数.\n显然, $\\varphi(1)=1$, 而对 $n>1, \\varphi(n)$ 为 1 , $2, \\cdots, n-1$ 中与 $n$ 互素的数的个数.\n例如, 若 $p$ 是素数, 则有 $\\varphi(p)=p-1$.\n在模 $n$ 的 $\\varphi(n)$ 个缩同余类中各任取一个数作为代表, 这样的 $\\varphi(n)$ 个数称为模 $n$ 的一个缩剩余系, 简称模 $n$ 的缩系, 于是 $\\varphi(n)$ 个数 $r_1, r_2, \\cdots, r_{\\varphi(n)}$ 称为模 $n$ 的一个缩系, 是指它们模 $n$ 互不同余, 且均与 $n$ 互素.\n不超过 $n$ 且与 $n$ 互素的 $\\varphi(n)$ 个正整数称为模 $n$ 的最小正缩系.\n下面的结果, 由模 $n$ 的一个完 (缩) 系, 产生模 $n$ 的另一个完(缩)系, 用处很多.\n(10) 设 $(a, n)=1, b$ 是任意整数.\n若 $c_1, c_2, \\cdots, c_n$ 是模 $n$ 的一个完系, 则 $a c_1+b, a c_2+b, \\cdots, a c_n+b$ 也是模 $n$ 的一个完系;\n若 $\\left.r_1, r_2, \\cdots, r_{\\varphi(n)}\\right)$ 是模 $n$ 的一个缩系, 则 $a r_1, a r_2, \\cdots, a r_{\\varphi(n)}$ 也是模 $n$ 的一个缩系.\n由 (10)中的第一个断言可推出:\n(11) 设 $(a, n)=1, b$ 是任意整数,则有整数 $x$, 使得 $a x \\equiv b(\\bmod n)$, 并易知所有这样的 $x$ 形成模 $n$ 的一个同余类.\n特别地, 有 $x$ 使得 $a x \\equiv 1(\\bmod n)$. 这样的 $x$ 称为 $a$ 关于模 $n$ 的逆, 记作 $a^*$ 或 $a^{-1}(\\bmod n)$, 它们形成模 $n$ 的一个同余类, 从而有一个 $a^{-1}$ 满足 $1 \\leqslant a^{-1}1$ 为整数, $a$ 是与 $m$ 互素的任一整数, $\\varphi(m)$ 为欧拉函数 (见第 6 单元), 则\n$$\na^{\\varphi(m)} \\equiv 1(\\bmod m) .\n$$\n欧拉定理可如下证明: 取 $r_1, r_2, \\cdots, r_{\\varphi(m)}$ 为模 $m$ 的一个缩系.\n因为 ( $a$ , $m)=1$, 故 $a r_1, a r_2, \\cdots, a r_{\\varphi(m)}$ 也是模 $m$ 的一个缩系 (见第 6 单元). 由于模 $m$ 的两个完 (缩) 系在模 $m$ 意义下互为排列, 因此特别地有\n$$\nr_1 \\cdots r_{\\varphi(m)} \\equiv a r_1 \\cdot a r_2 \\cdots \\cdot a r_{\\varphi(m)}=a^{\\varphi(m)} r_1 r_2 \\cdots r_{\\varphi(m)}(\\bmod m) .\n$$\n因 $\\left(r_i, m\\right)=1$, 故 $\\left(r_1 r_2 \\cdots r_{\\varphi(m)}, m\\right)=1$, 因此上式两边可约去 $r_1 \\cdots r_{\\varphi(m)}$, 即有 $a^{\\varphi(m)} \\equiv 1(\\bmod m)$.\n注1 当 $m=p$ 为素数时, 由于 $\\varphi(p)=p-1$,故由欧拉定理可推出费马小定理.\n注2 若已知 $m$ 的标准分解 $m=p_1^{\\alpha_1} \\cdots p_k^{\\alpha_k}$, 则欧拉函数 $\\varphi(m)$ 由下面公式确定(其证明这里略去):\n$$\n\\begin{aligned}\n\\varphi(m) & =p_1^{\\alpha_1-1}\\left(p_1-1\\right) p_2^{\\alpha_2-1}\\left(p_2-1\\right) \\cdots p_k^{\\alpha_k-1}\\left(p_k-1\\right) \\\\\n& =m\\left(1-\\frac{1}{p_1}\\right)\\left(1-\\frac{1}{p_2}\\right) \\cdots\\left(1-\\frac{1}{p_k}\\right) .\n\\end{aligned}\n$$\n(3)中国剩余定理设 $m_1, m_2, \\cdots, m_k$ 是 $k$ 个两两互素的正整数, $M== m_1 m_2 \\cdots m_k, M_i=\\frac{M}{m_i}(i=1,2, \\cdots, k), b_1, b_2, \\cdots, b_k$ 为任意整数, 则同余式组\n$$\nx \\equiv b_1\\left(\\bmod m_1\\right), \\cdots, x \\equiv b_k\\left(\\bmod m_k\\right)\n$$\n有唯一解 $x \\equiv M_1^* M_1 b_1+\\cdots+M_k^* M_k b_k(\\bmod M)$, 其中 $M_i^*$ 为满足 $M_i^* M_i \\equiv 1\\left(\\bmod m_i\\right)$ 任意整数 $(i=1,2, \\cdots, k)$.\n验证上述结论是一件容易的事情, 我们将这留给读者(注意, 对任意 $i$, 有 $\\left(m_i, M_i\\right)=1$, 以及对任意 $j \\neq i$ 有 $m_i \\mid M_j$). 中国剩余定理的主要力量在于, 它断言所说的同余式组当模两两互素时一定有解, 而解的具体形式通常并不重要.\n上述的几个数论定理是解决问题的有力工具, 它们往往和其他方法结合使用, 我们在后面将看到这一点, 这里先介绍几个较为直接的应用这些定理的例子.", + "figures": [] +} \ No newline at end of file diff --git a/processed_dataset/text/0027.json b/processed_dataset/text/0027.json new file mode 100644 index 0000000000000000000000000000000000000000..ead71b4b9afecbb046abdd4381848e63c5665281 --- /dev/null +++ b/processed_dataset/text/0027.json @@ -0,0 +1,5 @@ +{ + "source_file": "./raw_volume-zh/volume10/chapter8.tex", + "text": "阶及其应用.\n设 $n>1, a$ 是满足 $(a, n)=1$ 的整数,则必有一个 $r(1 \\leqslant r \\leqslant n-1)$ 使得 $a^r \\equiv 1(\\bmod n)$.\n事实上, 由于 $n$ 个数 $a^0, a^1, \\cdots, a^{n-1}$ 都与 $n$ 互素, 故它们模 $n$ 至多有 $n-$ 1 个不同的余数, 因此其中必有两个模 $n$ 同余, 即有 $0 \\leqslant im_0\\left( m_0\\left(n \\cdot \\frac{m}{n}=m$, 这与 $n$ 个抽屉内共有 $m$ 个物件矛盾,故结论成立.\n证毕.\n推广如果将 $m_1+m_2+\\cdots+m_n-1\\left(m_1, m_2, \\cdots, m_n\\right.$ 均为正整数 $)$ 个物件放人 $n$ 个抽㞕内, 那么或者第一个抽㞕内至多有 $m_1-1$ 个物件, 或者第二个抽㞕内至多有 $m_2-1$ 个物件 ......或者第 $n$ 个抽屉内至多有 $m_n-1$ 个物件.\n证明用反证法,如果第 $i$ 个抽㞎内至少有 $m_i$ 个物件 $(i=1,2, \\cdots, n)$,那么 $n$ 个抽屉内的物件的总数至少为 $m_1+m_2+\\cdots+m_n$, 这与 $n$ 个抽屉内共有 $m_1+m_2+\\cdots+m_n-1$ 个物件矛盾,故结论成立.\n证毕.\n二、平均值原理平均值原理 (1) 设 $a_1, a_2, \\cdots, a_n$ 是实数, $A=\\frac{1}{n}\\left(a_1+a_2+\\cdots+a_n\\right)$, 则 $a_1, a_2, \\cdots, a_n$ 中必有一个数不小于 $A$, 也有一个数不大于 $A$;\n(2) 设 $a_1, a_2, \\cdots, a_n$ 是正实数, $G=\\sqrt[n]{a_1 a_2 \\cdots a_n}$, 则 $a_1, a_2, \\cdots, a_n$ 中必有一个数不小于 $G$, 也有一个数不大于 $G$.\n证明 (1) $\\min _{1 \\leqslant i \\leqslant n}\\left\\{a_i\\right\\} \\leqslant A \\leqslant \\max _{1 \\leqslant i \\leqslant n}\\left\\{a_i\\right\\}$;\n(2) $\\min _{1 \\leqslant i \\leqslant n}\\left\\{a_i\\right\\} \\leqslant G \\leqslant \\max _{1 \\leqslant i \\leqslant n}\\left\\{a_i\\right\\}$.", + "figures": [] +} \ No newline at end of file diff --git a/processed_dataset/text/0042.json b/processed_dataset/text/0042.json new file mode 100644 index 0000000000000000000000000000000000000000..a498e697dfef711a4f936c42a219fa056da7b1a0 --- /dev/null +++ b/processed_dataset/text/0042.json @@ -0,0 +1,5 @@ +{ + "source_file": "./raw_volume-zh/volume11/chapter3.tex", + "text": "一、母函数的概念.\n设 $f(x)=(1+x)^n$, 由二项式定理,有\n$$\nf(x)=\\sum_{k=0}^n \\mathrm{C}_n^k x^k=\\mathrm{C}_n^0+\\mathrm{C}_n^1 x+\\mathrm{C}_n^2 x^2+\\cdots+\\mathrm{C}_n^n x^n .\n$$\n这时, $f(x)$ 对应了一个数列 $\\left\\{\\mathrm{C}_n^k, 0 \\leqslant k \\leqslant n\\right\\}$, 即生成数列 $\\left\\{\\mathrm{C}_n^k\\right\\}$, 因此, 我们把函数 $f(x)=(1+x)^n$ 称为数列 $\\left\\{\\mathrm{C}_n^k\\right\\}$ 的生成函数或母函数.\n一般地说, 对于有穷数列\n$$\na_0, a_1, a_2, \\cdots, a_n,\n$$\n多项式 $f(x)=\\sum_{k=0}^n a_k x^k=a_0+a_1 x+\\cdots+a_k x^k+\\cdots+a_n x^n$ 称为数列 $\\left\\{a_k\\right\\}$ 的母函数.\n更一般地, 对于无穷数列\n$$\na_0, a_1, \\cdots, a_n, \\cdots\n$$\n我们称下列形式幂级数\n$$\nf(x)=\\sum_{n=0}^{\\infty} a_n x^n=a_0+a_1 x+\\cdots+a_n x^n+\\cdots .\n$$\n为无穷数列 $\\left\\{a_n\\right\\}$ 的母函数.\n关于形式幂级数我们作如下的规定: 设 $f(x)=\\sum_{n=0}^{\\infty} a_n x^n, g(x)= \\sum_{n=0}^{\\infty} b_n x^n$ 是两个形式幂级数,我们规定\n(1) $f(x)=g(x)$, 当且仅当 $a_n==b_n(n=0,1,2, \\cdots)$ ;\n(2) $f(x) \\pm g(x)=\\sum_{n=0}^{\\infty}\\left(a_n \\pm b_n\\right) x^n$;\n(3) $\\alpha f(x)=\\sum_{n=0}^{\\infty}\\left(\\alpha a_n\\right) x^n$ ( $\\alpha$ 为常数);\n(4) $f(x) g(x)=\\sum_{n=0}^{\\infty} c_n x^n$, 其中 $c_n=\\sum_{k=0}^n a_k b_{n-k}, n=0,1,2, \\cdots$.\n二、几个重要公式在应用母函数解题时, 除了二项式定理以外, 还要用到下列几个公式: 公式 I (无穷递缩等比数列求和公式)\n$$\n\\frac{1}{1-x}=\\sum_{n=0}^{\\infty} x^n=1+x+x^2+\\cdots+x^n+\\cdots(|x|<1) .\n$$\n公式 II $\\quad(1-x)^{-k}=\\sum_{n=0}^{\\infty} \\mathrm{C}_{n+k-1}^{k-1} x^n=1+\\mathrm{C}_k^{k-1} x+\\mathrm{C}_{k+1}^{k-1} x^2+\\cdots+\\mathrm{C}_{n+k-1}^{k-1} x^n+ \\cdots$ ( $k$ 为正整数, $|x|<1$ ).\n公式 II 可由公式 I 两边求 $k-1$ 阶导数后除以 $(k-1)$ ! 而得到.", + "figures": [] +} \ No newline at end of file diff --git a/processed_dataset/text/0043.json b/processed_dataset/text/0043.json new file mode 100644 index 0000000000000000000000000000000000000000..687574939146f7f734baa045e649442db7433c52 --- /dev/null +++ b/processed_dataset/text/0043.json @@ -0,0 +1,5 @@ +{ + "source_file": "./raw_volume-zh/volume11/chapter4.tex", + "text": "一、递推数列对于一个数列 $\\left\\{x_n\\right\\}$, 若存在正整数 $k$ 和一个把 $x_{n+k}$ 和前面 $k$ 项 $x_{n+k-1}$, $x_{n+k-2}, \\cdots, x_n$ 联系起来的方程\n$$\n\\Phi\\left(x_{n+k}, x_{n+k-1}, \\cdots, x_n\\right)=0, k=0,1,2, \\cdots, \\label{eq1}\n$$\n则称数列 $\\left\\{x_n\\right\\}$ 为 $k$ 阶递推数列, 且称方程(1)是数列 $\\left\\{x_n\\right\\}$ 的递推方程.\n从式\\ref{eq1} 解出\n$$\nx_{n+k}=\\varphi\\left(x_{n+k-1}, x_{n+k-2}, \\cdots, x_n\\right), \\label{eq2}\n$$\n又称为数列 $\\left\\{x_n\\right\\}$ 的递推公式, 数列 $\\left\\{x_n\\right\\}$ 开头 $k$ 项的值.\n$$\nx_1=a_1, x_2=a_2, \\cdots, x_k=a_k\\left(a_1, a_2, \\cdots, a_k \\text { 为已知常数 }\\right) \\text {, } \\label{eq3}\n$$\n称为递推方程式\\ref{eq1} 或递推公式\\ref{eq2} 的初始条件或初始值, 显然, 一个 $k$ 阶递推数列 $\\left\\{x_n\\right\\}$ 可由递推公式\\ref{eq2}和初始值式\\ref{eq3}唯一确定.\n由递推公式\n$$\n\\begin{aligned}\n& x_{n+k}=p_1 x_{n+k-1}+p_2 x_{n+k-2}+\\cdots+p_k x_n+q, \\\\\n& \\left(n=1,2,3, \\cdots ; p_1, p_2, \\cdots, p_k \\text { 为常数且 } p_k \\neq 0\\right)\n\\end{aligned} \\label{eq4}\n$$\n及初始值式\\ref{eq3}确定的数列 $\\left\\{x_n\\right\\}$ 称为 $k$ 阶常系数线性递推数列, 特别 $q \\equiv 0$ 时, 称为 $k$ 阶常系数线性齐次递推数列.\n二、求递推数列通项的方法\n(1)换元方法这种方法的基本思想是: 选择适当的变换函数 $\\varphi(x)$, 令 $x_n=\\varphi\\left(y_n\\right)$ 或 $\\varphi\\left(x_n\\right)=y_n$, 代入到 $\\left\\{x_n\\right\\}$ 的递推关系中, 得到 $\\left\\{y_n\\right\\}$ 的一个新的递推关系.\n如果从这个新的递推关系中能求出 $y_n$ 的通项, 那么代入到 $x_n=\\varphi\\left(y_n\\right)$ 或 $\\varphi\\left(x_n\\right)=y_n$ 中, 便可求出 $x_n$ 的通项.\n因此, 换元的关键是选择变换函数 $\\varphi(x)$.\n(2)特征根法考虑二阶常系数线性齐次递推数列 $\\left\\{x_n\\right\\}$ :\n$$\nx_{n+2}=p x_{n+1}+q x_n(n=0,1,2, \\cdots, p, q \\text { 为常数, } q \\neq 0) . \\label{eq5}\n$$\n若有等比数列 $\\left\\{r^n\\right\\}$ 满足 式\\ref{eq5},则易知 $r$ 必须满足下列二次方程\n$$\nr^2=p r+q, \\label{eq6}\n$$\n我们称方程式\\ref{eq6}为\\ref{eq5}的特征方程, 并称式\\ref{eq6}的根为特征根, 反之若 $r_0$ 是式\\ref{eq6}的一个根, 则易证等比数列 $\\left\\{r_0^n\\right\\}$ 满足递推公式(5).\n若式\\ref{eq6}有两个不相等的根 $r_1$ 和 $r_2$, 则数列 $\\left\\{r_1^n\\right\\}$ 和 $\\left\\{r_2^n\\right\\}$ 都是式\\ref{eq5}的解, 并且对任意常数 $c_1, c_2$, 数列 $\\left\\{c_1 r_1^n+c_2 r_2^n\\right\\}$ 也是 式\\ref{eq5} 的解.\n如果给出初始值 $x_0=a$, $x_1=b$, 则由\n$$\n\\left\\{\\begin{array}{l}\nc_1+c_2=a, \\\\\nc_1 r_1+c_2 r_2=b,\n\\end{array}\\right.\n$$\n可唯一确定 $c_1$ 和 $c_2$, 从而得出式\\ref{eq5}的满足初始值 $x_0=a, x_1=b$ 的唯一解为\n$$\nx_n=c_1 r_1^n+c_2 r_2^n .\n$$\n若式\\ref{eq6}有二重根 $r=\\frac{p}{2}$, 则由\n$$\n\\left\\{\\begin{array}{l}\n2 r-p=0, \\\\\nr^2-p r-q=0, \\\\\nr^2=-q .\n\\end{array}\\right.\n$$\n可得 $p r+2 q=0$, 从而有\n$$\n\\begin{aligned}\n& n r^n-p(n-1) r^{n-1}-q(n-2) r^{n-2} \\\\\n= & n r^{n-2}\\left(r^2-p r-q\\right)+r^{n-2}(p r+2 q) \\\\\n= & 0 .\n\\end{aligned}\n$$\n即数列 $\\left\\{n r^{n-1}\\right\\}$ 是式\\ref{eq5}的解, 并且对任意常数 $c_1$ 和 $c_2 .\\left\\{c_1 r^n+c_2 n r^n\\right\\}$ 也是(5)的解.\n再由初始值 $x_0=a, x_1=b$, 可唯一确定 $c_1, c_2$, 从而得到式\\ref{eq5}的满足初始值\n$x_0=a, x_1=b$ 的唯一解为 $x_n=\\left(c_1+c_2 n\\right) r^n$.\n(3) 数学归纳法,这个方法的基本思想是 : 从初始值出发, 利用所给的递推关系逐次算出数列前面若干项的值, 从中找出规律, 归纳出通项的表达式, 再用数学归纳法给予证明.", + "figures": [] +} \ No newline at end of file diff --git a/processed_dataset/text/0044.json b/processed_dataset/text/0044.json new file mode 100644 index 0000000000000000000000000000000000000000..34a552a11a63ae8672c0b490eedcd10d5f3cc0a3 --- /dev/null +++ b/processed_dataset/text/0044.json @@ -0,0 +1,5 @@ +{ + "source_file": "./raw_volume-zh/volume11/chapter5.tex", + "text": "分类和分步.\n一、分类\n当被研究的数学问题出现多种不同的情形时, 常常可按出现的各种情形分别进行讨论和解答.\n得出各种情形下相应的结论, 综合起来就获得原问题的解答, 这就是分类的思想方法.\n应用分类的思想方法解题应遵循以下原则:\n(1)分类必须包含原题目中所有可能出现的各种情形,没有遗漏;\n(2)任何两类之间互相排斥,没有重叠;\n(3)每次分类必须使用同一标准;\n(4)选择分类标准的关键在于各类情形都比原问题易于解决.\n二、分步\n分步就是将一个较复杂的问题变成 (或改编成)一组互相关联的\"小问题\". 在这一组 \"小问题\" 中, 后面问题的解决常常依赖于前面小题的结果, 而当最后一个小问题解出时, 便得出了原问题的结论.", + "figures": [] +} \ No newline at end of file diff --git a/processed_dataset/text/0045.json b/processed_dataset/text/0045.json new file mode 100644 index 0000000000000000000000000000000000000000..1b0497dc96c843b193d135550915e8fe19798310 --- /dev/null +++ b/processed_dataset/text/0045.json @@ -0,0 +1,5 @@ +{ + "source_file": "./raw_volume-zh/volume11/chapter6.tex", + "text": "对应不仅是一个基本的数学概念, 而且是解题和证题的一种重要方法和技巧.\n对应是联系陌生问题和熟悉问题的桥梁, 通过对应往往使得一些隐蔽的关系变得明朗和具体, 使人们易于找到解决问题的途径.\n用对应方法解题的关键是构造对应关系, 而这并没有一般的通法, 而应根据不同问题的特点作具体分析才能确定.\n下面我们将通过具体例题说明各种对应方法在解题中的应用.\n一、配对 法所谓配对法, 是指这样一种解题和证题的思想方法: 按照一定的规则, 将所研究的对象两两配成一对, 从而使得计算比较容易或解题思路更加清晰, 达到化繁为简, 化难为易的目的.\n二、映射方法运用映射法解题, 主要是利用下列定理.\n定理设 $f$ 是从有限集合 $M$ 到有限集合 $N$ 的映射.\n$|M|,|N|$ 分别表示 $M, N$ 中元素个数.\n(1) 若 $f$ 是单射 (即对任意 $x_1, x_2 \\in M$, 当 $x_1 \\neq x_2$ 时有 $f\\left(x_1\\right) \\neq \\left.f\\left(x_2\\right)\\right)$, 则 $|M| \\leqslant|N|$;\n(2) 若 $f$ 为满射 (即对任意 $y \\in N$, 都存在 $x \\in M$ 使 $f(x)=y$ ), 则 $|M| \\geqslant|N|$;\n(3) 若 $f$ 为双射, 又称 $f$ 是从 $M$ 到 $N$ 上的一一对应(即 $f$ 既是单射, 又是满射), 则 $|M|=|N|$.\n当计算有限集合 $M$ 中的元素个数比较困难时,我们设法建立 $M$ 到另一集合 $N$ 上的双射, 如果 $N$ 中的元素个数 $|N|$ 容易算出, 于是由 $|M|=|N|$, 得出 $M$ 中元素的个数, 这就是计数中的映射方法.\n在某些组合证明中,除了要建立方程外,有时还要建立不等关系, 这时就可考虑构造单射、满射来进行论证.", + "figures": [] +} \ No newline at end of file diff --git a/processed_dataset/text/0046.json b/processed_dataset/text/0046.json new file mode 100644 index 0000000000000000000000000000000000000000..70709701afadd5a6a8b46c8a84c204baaa8d894b --- /dev/null +++ b/processed_dataset/text/0046.json @@ -0,0 +1,5 @@ +{ + "source_file": "./raw_volume-zh/volume11/chapter6.tex", + "text": "配对法除了用于直接计数和通过计数进行证明外, 还用于一类两人对策问题.\n这类问题常常涉及到几何图形的特征或数量的性质 (如整除性、同余等). 在这类对策问题中, 若某一步走后未被判输, 则称这样的步为活步.\n若某一方的策略能保证自己总能在对方走出活步后仍有步可走, 则必不败, 而在有限步对策问题中不败就必胜(假设无平局).\n保证有步可走的常用方法之一是将所有可走的位置(或可取的数)配对, 使每对位置 (或每对数) $a, b$ 满足: 只要对方走到一个位置(或取到一个数), 比如 $a$,自己就能走到另一个位置 $b$ (或取到另一个数 $b$ ).\n对涉及到几何图形的对策问题, 配对应根据几何图形的特征和走步规则来确定, 常用的有对称、相邻等条件, 应抢占多余的位置或对称中心.\n而对涉及数量性质的对策问题, 配对应根据问题条件中数的特征和走步规则来确定,常用的有整除、同余、几个数的和一定等条件.", + "figures": [] +} \ No newline at end of file diff --git a/processed_dataset/text/0047.json b/processed_dataset/text/0047.json new file mode 100644 index 0000000000000000000000000000000000000000..c43d2917984150b7784e68a3e1331f7d56a39b3c --- /dev/null +++ b/processed_dataset/text/0047.json @@ -0,0 +1,5 @@ +{ + "source_file": "./raw_volume-zh/volume11/chapter7.tex", + "text": "算二次方法.\n设 $A=\\left\\{a_1, a_2, \\cdots, a_m\\right\\}, B=\\left\\{b_1, b_2, \\cdots, b_n\\right\\}$ 是两个有限集合, 将所有形如 $\\left(a_i, b_j\\right)(1 \\leqslant i \\leqslant m, 1 \\leqslant j \\leqslant n)$ 的有序对构成的集合称为 $A$ 与 $B$ 的笛卡儿乘积, 并用记号 $A \\times B$ 表示.\n对任意 $a_i \\in A$, 设 $C_i=\\left\\{\\left(a_i, b\\right) \\mid b \\in B\\right\\}(i=1,2, \\cdots, m)$, 对任意 $b_j \\in B$, 设 $D_j=\\left\\{\\left(a, b_j\\right) \\mid a \\in A\\right\\}(j=1,2$, $\\cdots, n)$, 于是 $|A \\times B|=\\sum_{i=1}^m\\left|C_i\\right|=\\sum_{j=1}^n\\left|D_j\\right|$, 这个等式叫做富比尼(Fubini) 原理, 又叫做算二次原理.\n运用算二次原理的方法主要体现在对同一对象从两种不同的角度去进行计数, 再加以综合, 以便推出所欲取得的结果.", + "figures": [] +} \ No newline at end of file diff --git a/processed_dataset/text/0048.json b/processed_dataset/text/0048.json new file mode 100644 index 0000000000000000000000000000000000000000..256b7ff1c8f801a43f23793f2a14a6fef2906ac6 --- /dev/null +++ b/processed_dataset/text/0048.json @@ -0,0 +1,5 @@ +{ + "source_file": "./raw_volume-zh/volume11/chapter8.tex", + "text": "递推方法解组合问题的一般步骤是:\n(1) 用枚举法求初始值;\n(2) 建立递推关系;\n(3) 利用递推关系求解.\n当利用已建立的递推关系求解遇到困难时, 还应考虑建立新的递推关系, 而在建立递推关系遇到困难时, 则可列举简单情形寻求启示, 从中归纳得出要求的递推关系.", + "figures": [] +} \ No newline at end of file diff --git a/processed_dataset/text/0049.json b/processed_dataset/text/0049.json new file mode 100644 index 0000000000000000000000000000000000000000..569bd170f4428c2b783ba995fc47ee809803436e --- /dev/null +++ b/processed_dataset/text/0049.json @@ -0,0 +1,5 @@ +{ + "source_file": "./raw_volume-zh/volume11/chapter9.tex", + "text": "一、染色方法.\n染色方法就是根据问题的特点, 对研究的对象用几种颜色染色, 并通过对染色对象 (点、线、区域, …) 以及数种染色对象的组合结构 (两边同色的角、 两边异色的角、三边同色的三角形, 三边不全同色的三角形、同色点对, 异色点对, $\\cdots)$ 的数量和性质进行分析和比较, 从而使问题的解答能够比较容易地、直观地给出的一种解题方法.\n二、赋值方法赋值方法就是根据问题的特点, 对研究的对象分别赋不同的数值, 并通过对所赋的值进行分析、计算和比较,从而得到问题解答的一种解题方法.", + "figures": [] +} \ No newline at end of file diff --git a/processed_dataset/text/0050.json b/processed_dataset/text/0050.json new file mode 100644 index 0000000000000000000000000000000000000000..00fe45d769e3fb0e4df2c6fc4addf874bf397ee2 --- /dev/null +++ b/processed_dataset/text/0050.json @@ -0,0 +1,15 @@ +{ + "source_file": "./raw_volume-zh/volume12/chapter1.tex", + "text": "我们经常遇到这样一些现象或问题:\n在一群人中,有的两个人之间互相认识, 有的互不相识;\n一次足球锦标赛有若干个队参加, 其中有的两个队之间比赛过, 有的没有比赛过;\n有若干个大城市, 有的两个城市之间有航线相通, 有的没有航线相通;\n平面上的一个点集中, 其中任意两点之间, 有的距离为 1 , 有的距离不为 1 .\n在上面这些现象或问题中都包含两方面的内容: 其一是一些 \"对象\", 如人群、足球队、城市、点等等; 其二是这些对象两两之间的某种特定关系, 如 \"互相认识\"、\"比赛过\"、\"通航\"、\"距离为 1 \"等.\n为了表示这些对象和他们之间的关系, 我们可以用一个点表示一个对象, 称这些点为顶点, 如果两个对象之间有所讨论的关系, 就在相应的两点之间连上一条线, 称这些线为边, 这样就构成了一个图形.\n这个用来表示某类对象及它们间特定关系的, 由若干个顶点与连接某些顶点的边构成的图形, 我们直观地称之为图 *.\n图论是以图作为研究对象的一个数学分支.\n例如图() 中给出了 3 个图 $G_1 、 G_2 、 G_3$, 其中顶点由小圆圈表示.\n图的一般数学定义为: 一个图 $G$ 是一个三元组 $(V, E, \\phi)$, 其中 $V$ 和 $E$ 是两个不相交的集合, $V$ 非空, $\\phi$ 是 $E$ 到 $V$ 的一个映射, $V 、 E 、 \\phi$ 分别称为图 $G$ 的顶点集、边集和关联函数.\n我们注意到, 在直观地叙述图的定义中, 并没有规定这些顶点的位置以及边的曲直长短, 也没有规定这些顶点、边都要在同一平面中, 不过, 连结两点的边不能通过第三个顶点, 也不能与自己相交.\n在图论中, 如果两个图 $G$ 与 $G^{\\prime}$ 的顶点之间可以建立起一一对应, 并且 $G$ 中连接顶点 $v_i$ 与 $v_j$ 之间的边数 $k(k=0,1,2, \\cdots)$ 与连接 $G^{\\prime}$ 中相应的顶点 $v_i^{\\prime}$ 与 $v_j^{\\prime}$ 的边数相同时, 便称图 $G$ 与 $G^{\\prime}$ 是同构的,认为 $G$ 与 $G^{\\prime}$ 是相同的图.\n例如, 如图() 中的三个图 $G_1 、 G_2 、 G_3$ 是同构的.\n如果对图 $G=(V, E)$ 与 $G^{\\prime}=\\left(V^{\\prime}, E^{\\prime}\\right)$ 有 $V^{\\prime} \\subseteq V, E^{\\prime} \\subseteq E$, 即图 $G^{\\prime}$ 的顶点都是图 $G$ 的顶点, 图 $G^{\\prime}$ 的边也都是图 $G$ 的边, 则称 $G^{\\prime}$ 是 $G$ 的子图, 例如图() 中的 $G_1 、 G_2$ 都是 $G$ 的子图.\n若在一个图 $G$ 中的两个顶点 $v_i$ 与 $v_j$ 之间有边 $e$ 相连, 则称点 $v_i$ 与 $v_j$ 是相邻的, 否则就称点 $v_i$ 与 $v_j$ 是不相邻的.\n如果顶点 $v$ 是边 $e$ 的一个端点, 称点 $v$ 与边 $e$ 是关联的.\n如图() 中, 顶点 $v_1$ 与 $v_2$ 是相邻的, 而顶点 $v_2$ 与顶点 $v_5$ 是不相邻的.\n顶点 $v_3$ 与边 $e_4$ 是关联的.\n有些顶点本身也有边相连, 这样的边称为环.\n如图() 所示的边 $e_6$ 是环.\n连结两个顶点的边有时可能不止一条, 若两个顶点之间有 $k(k \\geqslant 2)$ 条边相连,则称这些边为平行边.\n例如图() 中的边 $e_1 、 e_2$ 是平行边.\n如果一个图没有环, 并且没有平行边, 这样的图称为简单图.\n如图() 中的 $G_1 、 G_2 、 G_3$ 都是简单图, 而如图() 所示的就不是一个简单图.\n在简单图中, 连结顶点 $v_i$ 与 $v_j$ 之间的边可用 $\\left(v_i, v_j\\right)$ 表示.\n当然, $\\left(v_i, v_j\\right)$ 与 $\\left(v_j, v_i\\right)$ 表示的是同一条边.\n如果一个简单图中, 每两个顶点之间都有一条边, 这样的图称为完全图.\n通常将有 $n$ 个顶点的完全图记为 $K_n$. 如图() 中是完全图 $K_3 、 K_4 、 K_5$. 完全图 $K_n$ 的边的数目是 $\\mathrm{C}_n^2=\\frac{1}{2} n(n-1)$.\n在图 $G=(V, E)$ 中, 若顶点个数 $|V|(|V|$ 也称为 $G$ 的阶 $)$ 和边数 $|E|$ 都是有限的, 则称图 $G$ 是有限图.\n如果 $|V|$ 或 $|E|$ 是无限的, 则称 $G$ 为无限图.\n本篇中,除非特别说明,我们所说的图都是指有限简单图.\n利用上述的这些基本概念可以帮助我们思考并解决一些问题.\n本书中的例题和习题包括图论问题和利用图论方法解决的问题.", + "figures": [ + "./images/volume12/figures/fig-c1i1.png", + "./images/volume12/figures/fig-c1i1.png", + "./images/volume12/figures/fig-c1i2.png", + "./images/volume12/figures/fig-c1i3.png", + "./images/volume12/figures/fig-c1i3.png", + "./images/volume12/figures/fig-c1i3.png", + "./images/volume12/figures/fig-c1i1.png", + "./images/volume12/figures/fig-c1i3.png", + "./images/volume12/figures/fig-c1i4.png" + ] +} \ No newline at end of file diff --git a/processed_dataset/text/0051.json b/processed_dataset/text/0051.json new file mode 100644 index 0000000000000000000000000000000000000000..18d733bbbc3dce4ac6e7fec3d471656a858c975e --- /dev/null +++ b/processed_dataset/text/0051.json @@ -0,0 +1,9 @@ +{ + "source_file": "./raw_volume-zh/volume12/chapter2.tex", + "text": "图 $G$ 中与顶点 $v$ 关联的边数 (约定环计两次) 称为图 $G$ 中顶点 $v$ 的度 (或次数), 记作 $d_G(v)$. 在不致混淆的时候, 简记为 $d(v)$. 我们用 $\\delta(G)$ 与 $\\Delta(G)$ 分别表示 $G$ 中顶点的最小度和最大度,分别简记为 $\\delta$ 和 $\\Delta$.\n如图() 中, $d\\left(v_1\\right)=1, d\\left(v_2\\right)=3, d\\left(v_3\\right)=d\\left(v_4\\right)=2, \\delta=1, \\Delta=3$.\n图 $G$ 中, 若顶点 $v$ 的度是奇数, 则称点 $v$ 为奇顶点; 若顶点 $v$ 的度是偶数, 则称点 $v$ 为偶顶点.\n图 $\\dot{2}-1$ 中, 点 $v_1, v_2$ 是奇顶点, 点 $v_3, v_4$ 是偶顶点.\n在图 $G=(V, E)$ 中,如果对任意的 $v \\in V$, 均有 $d(v)=k$, 则称图 $G$ 是 $k$ 正则的.\n完全图 $K_n$ 是 $(n-1)$ 正则图.\n如图() 中是一个 3 正则图.\n关于图 $G$ 中所有顶点的度之和与边数之间有如下结论.\n定理一设 $G$ 是 $n$ 阶图, 则 $G$ 中 $n$ 个顶点的度之和等于边数的两倍.\n记 $G$ 中 $n$ 个顶点为 $v_1, v_2, \\cdots, v_n$, 边数为 $e$, 则\n$$\nd\\left(v_1\\right)+d\\left(v_2\\right)+\\cdots+d\\left(v_n\\right)=2 e .\n$$\n证明所有顶点的度的和 $d\\left(v_1\\right)+d\\left(v_2\\right)+\\cdots+d\\left(v_n\\right)$ 表示以顶点 $v_1$, $v_2, \\cdots, v_n$ 中某个顶点为一个端点的边的总数.\n由于一条边有两个端点, 因此图 $G$ 的每条边在和 $d\\left(v_1\\right)+d\\left(v_2\\right)+\\cdots+d\\left(v_n\\right)$ 中被计人两次.\n所以所有顶点的度的和为边数的两倍.\n例如如图() 中, $e=4, d\\left(v_1\\right)+d\\left(v_2\\right)+d\\left(v_3\\right)+d\\left(v_4\\right)=1+3+2+ 2=8=2 e$.\n定理一通常称为握手引理, 在二百多年前欧拉就给出了这样一个著名的结论: 如果许多人在见面时握了手, 那么握手的次数为偶数.\n进而推得: 握过奇数次手的人有偶数个.\n这个推论就是定理二 对于任意的图 $G$, 奇顶点的个数一定是偶数.\n证明设 $G$ 中的顶点为 $v_1, v_2, \\cdots, v_n$, 且 $v_1, \\cdots, v_t$ 是奇顶点, $v_{t+1}$, $\\cdots, v_n$ 是偶顶点.\n由定理一,\n$$\n\\begin{aligned}\n& d\\left(v_1\\right)+\\cdots+d\\left(v_t\\right)+d\\left(v_{t+1}\\right)+\\cdots+d\\left(v_n\\right)=2 e, \\\\\n& d\\left(v_1\\right)+\\cdots+d\\left(v_t\\right)=2 e-d\\left(v_{t+1}\\right)-\\cdots-d\\left(v_n\\right) .\n\\end{aligned}\n$$\n因为 $d\\left(v_{t+1}\\right), \\cdots, d\\left(v_n\\right)$ 都是偶数, 故上式右边是偶数, 而 $d\\left(v_1\\right), \\cdots, d\\left(v_t\\right)$ 都是奇数, 要使它们的和为偶数, $t$ 必须是偶数.\n即 $G$ 中奇顶点个数为偶数.", + "figures": [ + "./images/volume12/figures/fig-c2i1.png", + "./images/volume12/figures/fig-c2i2.png", + "./images/volume12/figures/fig-c2i1.png" + ] +} \ No newline at end of file diff --git a/processed_dataset/text/0052.json b/processed_dataset/text/0052.json new file mode 100644 index 0000000000000000000000000000000000000000..c6378150db9df6f3ebf742b3680b5fa773844fc1 --- /dev/null +++ b/processed_dataset/text/0052.json @@ -0,0 +1,9 @@ +{ + "source_file": "./raw_volume-zh/volume12/chapter3.tex", + "text": "1941 年, 匈牙利数学家托兰 (Turán) 为了回答这样的问题: \" $n$ 个顶点的图 $G$ 不包含 $m$ 个顶点的完全图 $K_m$, 则图 $G$ 的最大边数是多少?\"而提出了他的著名定理, 从而开创了图论研究的一个新方向 \"极图理论\", 极图理论是近年来图论中比较活跃的分支之一.\n匈牙利数学家波洛巴斯 (B. Bollobás)在 1978 年专门写了一本《极图理论》, 是这方面最具权威的著作.\n下面先从 $k$ 部图的定义谈起.\n如果图 $G$ 的顶点集 $V$ 可以分解为 $k$ 个两两不交非空子集的并,即\n$$\nV=\\bigcup_{i=1}^k V_i, V_i \\cap V_j=\\varnothing, i \\neq j .\n$$\n并且没有一条边, 其两个端点都在上述同一子集内, 我们称这样的图 $G$ 为 $k$ 部图.\n记作 $G=\\left(V_1, V_2, \\cdots, V_k ; E\\right)$.\n如图() 所示的是一个 2 部图, 2 部图又称偶图.\n图()所示的是一个 3 部图.\n显然任何 $n$ 阶图是一个 $n$ 部图.\n如果在一个 $k$ 部图 $G=\\left(V_1, V_2, \\cdots, V_k ; E\\right)$ 中, $\\left|V_i\\right|=m_i$. 任何两点 $u \\in V_i, v \\in V_j, i \\neq j, i, j=1,2, \\cdots, k$, 均有 $u$ 和 $v$ 相邻, 则称 $G$ 是完全 $k$ 部图, 记作 $K_{m_1}, m_2, \\cdots, m_k$. 图()所示的是完全偶图 $K_{2,3}$.\n完全偶图 $K_{m, m}$ 和 $K_{m, m+1}$ 中分别有 $m^2$ 和 $m(m+1)$ 条边, 于是图中边数 $=\\left[\\frac{n^2}{4}\\right]$ (此处 $[x]$ 表示不超过 $x$ 的最大整数, $n$ 是图的阶), 完全偶图 $K_{m, m}$ 和 $K_{m, m+1}$ 中显然不含三角形, 下面的定理一表明, 在不含三角形的图中,这两类图中边的数目最多.\n定理一有 $n$ 个顶点且不含三角形的图 $G$ 的最大边数为 $\\left[\\frac{n^2}{4}\\right]$.\n证明设 $v_1$ 是 $G$ 中具有最大度数的顶点, $d\\left(v_1\\right)=d$. 又设与 $v_1$ 相邻的 $d$ 个顶点为\n$$\nv_n, v_{n-1}, \\cdots, v_{n-d+1} .\n$$\n由于 $G$ 不含三角形.\n所以 $v_n, v_{n-1}, \\cdots, v_{n-d+1}$ 中任意两点都不相邻, 故 $G$ 的边数 $e$ 满足\n$$\n\\begin{aligned}\ne & \\leqslant d\\left(v_1\\right)+d\\left(v_2\\right)+\\cdots+d\\left(v_{n-d}\\right) \\\\\n& \\leqslant(n-d) \\cdot d \\leqslant\\left(\\frac{n-d}{2}+d\\right)^2 \\\\\n& =\\frac{n^2}{4} .\n\\end{aligned}\n$$\n因为边数 $e$ 为整数, 所以 $e \\leqslant\\left[\\frac{n^2}{4}\\right]$.\n最大值是可以达到的, 当 $n=2 m$ 时, 取 $G=K_{m, m}$; 当 $n=2 m+1$ 时, 取 $G=K_{m, m+i}$.\n定理一的证明, 用数学归纳法也可完成, 留给读者作为习题.", + "figures": [ + "./images/volume12/figures/fig-c3i1.png", + "./images/volume12/figures/fig-c3i2.png", + "./images/volume12/figures/fig-c3i1.png" + ] +} \ No newline at end of file diff --git a/processed_dataset/text/0053.json b/processed_dataset/text/0053.json new file mode 100644 index 0000000000000000000000000000000000000000..524056dcefc5ea7e6c71ce5d28e0bfddb6253761 --- /dev/null +++ b/processed_dataset/text/0053.json @@ -0,0 +1,8 @@ +{ + "source_file": "./raw_volume-zh/volume12/chapter3.tex", + "text": "定理三设 $S=\\left\\{x_1, x_2, \\cdots, x_n\\right\\}$ 是平面上直径为 1 的点集,则距离大于 $\\frac{\\sqrt{2}}{2}$ 的点对的最大可能的数目是 $\\left[\\frac{n^2}{3}\\right]$. 并且对每个 $n$, 存在直径为 1 的一个点集 $\\left\\{x_1, x_2, \\cdots, x_n\\right\\}$, 它恰好有 $\\left[\\frac{n^2}{3}\\right]$ 个点对,其距离大于 $\\frac{\\sqrt{2}}{2}$.\n证明作图 $G: n$ 个顶点表示这 $n$ 个点, 两顶点相邻当且仅当这两点之间的距离大于 $\\frac{\\sqrt{2}}{2}$. 我们先证明 $G$ 不包含 $K_4$.\n对于平面上任意 4 个点,它们的凸包只有 3 种情况 : 线段、三角形、四边形,如图() 所示.\n显然在每一种情况下都有一个不小于 $90^{\\circ}$ 的角 $x_i x_j x_k$. 对于这 3 个点 $x_i, x_j, x_k$, 它们两两之间的距离不可能都大于 $\\frac{\\sqrt{2}}{2}$ 且小于等于 1 .\n因为若 $d\\left(x_i, x_j\\right)$ (此处用 $d(x, y)$ 表示 $x$ 和 $y$ 之间的距离) 和 $d\\left(x_j, x_k\\right)$ 都大于 $\\frac{\\sqrt{2}}{2}$, 且 $\\angle x_i x_j x_k \\geqslant 90^{\\circ}$, 则\n$$\nd\\left(x_i, x_k\\right) \\geqslant \\sqrt{d^2\\left(x_i, x_j\\right)+d^2\\left(x_j, \\overline{x_k}\\right)}>1 .\n$$\n由于点集 $S$ 的直径为 1 , 故 $G$ 中的任意 4 个点中, 至少有一对点不相邻, 即 $G$ 中不含 $K_4$.\n根据定理二, $G$ 的边数不超过 $e_3(n)=\\left[\\frac{n^3}{3}\\right]$.\n我们可以构作一个直径为 1 的点集 $\\left\\{x_1, x_2, \\cdots\\right.$, $\\left.x_n\\right\\}$, 其中恰有 $\\left[\\frac{n^2}{3}\\right]$ 个点对, 其距离大于 $\\frac{\\sqrt{2}}{2}$. 作法如下: 选择 $r$, 使 $0) 所示.\n把 $x_1, x_2, \\cdots, x_{\\left[\\frac{n}{3}\\right]}$ 放在一个圆内, $x_{\\left[\\frac{n}{3}\\right]+1}, \\cdots, x_{\\left[\\frac{2 n}{3}\\right]}$ 放在另一个圆内, $x_{\\left[\\frac{2 n}{3}\\right]+1}, \\cdots, x_n$ 放在第三个圆内, 并且使得 $x_1$ 与 $x_n$ 的距离为 1 . 显然该集的直径为 1 , 当且仅当 $x_i$ 和 $x_j$ 分属两个不同的圆时, $d\\left(x_i, x_j\\right)>\\frac{\\sqrt{2}}{2}$. 所以恰好存在 $\\left[\\frac{n^2}{3}\\right]$ 个点对 $\\left(x_i, x_j\\right)$, 使得 $d\\left(x_i, x_j\\right)>\\frac{\\sqrt{2}}{2}$.", + "figures": [ + "./images/volume12/figures/fig-c3i11.png", + "./images/volume12/figures/fig-c3i12.png" + ] +} \ No newline at end of file diff --git a/processed_dataset/text/0054.json b/processed_dataset/text/0054.json new file mode 100644 index 0000000000000000000000000000000000000000..888fef79a6f89271b235781d28c559cae72412be --- /dev/null +++ b/processed_dataset/text/0054.json @@ -0,0 +1,11 @@ +{ + "source_file": "./raw_volume-zh/volume12/chapter4.tex", + "text": "在各种各样的图中, 有一类简单但是很重要的图, 那就是\"树\". 树之所以重要, 不仅仅因为它在众多领域有着广泛的应用, 更在于图论本身.\n因为树是非常简单的图,所以在讨论关于图的一般性结论或猜想时, 可从先考虑树这种情形.\n先引人几个概念.\n在图 $G$ 中,一个由不同的边组成的序列:\n$$\ne_1, e_2, \\cdots, e_m .\n$$\n如果其中边 $e_i=\\left(v_{i-1}, v_i\\right), i=1,2, \\cdots, m$. 则称这个序列是从 $v_0$ 到 $v_m$ 的链.\n数 $m$ 称为这条链的长.\n$v_0$ 与 $v_m$ 称为这条链的端点.\n并且这条链记为 $v_0 v_1 \\cdots v_m$.\n如果一条链的两个端点 $v_0$ 与 $v_m$ 重合, 称这条链为圈 ${ }^*$.\n如图() 中, $e_1, e_2, e_3, e_4, e_5$ 组成一条链, $e_1, e_2, e_3$ 组成一个圈.\n如果图 $G$ 中的任意两个顶点 $u$ 与 $v$, 都有一条从 $u$ 到 $v$ 的链, 称这样的图 $G$ 为连通图.\n不是连通的图称为不连通图.\n如图() 中的图是连通图.\n如图() 中的图是不连通图.\n本书所指的 \"圈\" 实为 \"闭链\". 与通常图论书中的 \"圈\"有别.\n图论中的圈是指: 在 \"闭链\" $v_0 v_1 \\cdots v_m\\left(v_0=v_m\\right)$ 中, 点 $v_1, v_2, \\cdots, v_m$ 互不相同.\n现在给出树的定义.\n一个连通且没有圈的图称为树.\n通常用字母 $T$ 来表示树.\n根据树的定义, 树显然是简单图.\n如图() 是一个有 8 个顶点的树.\n显然,一个不含圈的图必定是由一个或数个顶点不交的树所组成的.\n我们称这样的图为森林.\n如图() 所示是一个森林, 它由 3 个树组成.\n度为 1 的顶点称为悬挂点 (或树叶).\n定理一如果树 $T$ 的顶点数 $\\geqslant 2$, 则 $T$ 中至少有两个悬挂点.\n证法一设想我们从某个顶点 $u$ 出发, 沿着 $T$ 的边走, 已经走过的边不再重复.\n由于树是没有圈的, 因此不会回到已经走过的点, 也就是说每个顶点至多走一次.\n如果我们走到的一个点不是悬挂点, 由于这个点的度大于 1 , 还可以继续走下去.\n但 $T$ 的顶点个数是有限的, 所以不可能永远走下去.\n如果在顶点 $v$ 处不能再继续走下去了, 则顶点 $v$ 就是一个悬挂点.\n我们从一个悬挂点 $v$ 出发, 又可以走到另一个悬挂点 $v^{\\prime}$, 所以树 $T$ 至少有两个悬挂点.\n证法二设 $\\mu=w v_1 v_2 \\cdots v_k v$ 是树 $T$ 中的一条最长的链, 可以证明 $d(u)=d(v)=1$, 即 $u 、 v$ 是悬挂点.\n事实上, 若 $d(u) \\geqslant 2$, 则存在不同于 $v_1$ 的顶点 $w$ 与 $u$ 相邻, 如果 $w$ 是 $v_2, \\cdots, v_k, v$ 中的一个, 则出现圈, 与树的定义矛盾.\n如果 $w$ 是不同于 $v_2, \\cdots$, $v_k, v$ 的点, 则 $r u v_1 \\cdots v_k v$ 是比 $\\mu$ 更长的链, 这与 $\\mu$ 的取法矛盾.\n从而 $d(u)=$ 1. 同样地, $d(v)=1$. 所以树 $T$ 至少有两个悬挂点.\n注:: 证明一是\"构造性\"的, 证明二是用\"最长链\"的方法, 这是两个很重要的解题方法.\n定理二设树 $T$ 的顶点数为 $n$, 则它的边数 $e=n-1$.\n证明对顶点数 $n$ 用数学归纳法.\n当 $n=1$ 时, $e=0$, 结论正确.\n假设当 $n=k$ 时结论成立.\n设 $T$ 是有 $k+1(k \\geqslant 1)$ 个顶点的树, 由定理一, $T$ 至少有两个悬挂点, 设 $v$ 是其中之一, 则去掉 $v$ 及与它关联的边, 就得到一个有 $k$ 个顶点的树 $T^{\\prime}$, 根据归纳假设 $T^{\\prime}$ 有 $k-1$ 条边, 所以 $T$ 的边数为 $k$, 从而结论对一切自然数 $n$ 都成立.", + "figures": [ + "./images/volume12/figures/fig-c4i1.png", + "./images/volume12/figures/fig-c4i1.png", + "./images/volume12/figures/fig-c4i2.png", + "./images/volume12/figures/fig-c4i3.png", + "./images/volume12/figures/fig-c4i4.png" + ] +} \ No newline at end of file diff --git a/processed_dataset/text/0055.json b/processed_dataset/text/0055.json new file mode 100644 index 0000000000000000000000000000000000000000..6baff28093b6580cf0758f34485e63475ca512a1 --- /dev/null +++ b/processed_dataset/text/0055.json @@ -0,0 +1,5 @@ +{ + "source_file": "./raw_volume-zh/volume12/chapter4.tex", + "text": "定理三设 $T$ 是有 $n$ 个顶点、 $e$ 条边的图.\n则下述三个命题是等价的:\n(1) 图 $T$ 是树;\n(2) 图 $T$ 无圈, 并且 $e=n-1$;\n(3) 图 $T$ 连通, 并且 $e=n-1$.\n证明由(1)推出 (2):\n设图 $T$ 是树, 则由树的定义知 $T$ 无圈, 由定理二知, $e=n-1$, 故 (2) 成立.\n由 (2) 推出 (3):\n只要证明 $T$ 是连通的即可, 用反证法.\n设 $T$ 是不连通的, 它有 $k(k \\geqslant 2)$ 个连通分支, 因为每个连通分支都无圈, 故每个连通分支都是树.\n若第 $i$ 个分支有 $p_i$ 个顶点, 根据定理二知, 第 $i$ 个分支有 $p_i-1$ 条边, 故\n$$\ne=\\left(p_1-1\\right)+\\cdots+\\left(p_k-1\\right)=n-k \\leqslant n-2 .\n$$\n这与 $e=n-1$ 矛盾.\n于是证得 $T$ 是连通的.\n由(3)推出 (1):\n只要证得 $T$ 无圈, 则 $T$ 便是树.\n当 $n=1$ 时结论显然成立.\n设 $n \\geqslant 2$, 那么 $T$ 必有悬挂点.\n否则, 因 $T$ 连通且 $n \\geqslant 2$, 故 $T$ 中每个顶点的度 $\\geqslant 2$,于是\n$$\ne=\\frac{1}{2}\\left[d\\left(v_1\\right)+d\\left(v_2\\right)+\\cdots+d\\left(v_n\\right)\\right] \\geqslant \\frac{1}{2} \\times 2 n=n .\n$$\n这与 $e=n-1$ 矛盾.\n现对 $n$ 用数学归纳法证明 $T$ 无圈.\n当 $n=2$ 时, $e=1$, 此时 $T$ 无圈.\n设 $n=k$ 时命题成立.\n$T$ 是有 $k+1$ 个顶点的图, 顶点 $v$ 是 $T$ 的悬挂点.\n在 $T$ 中去掉 $v$ 及与它关联的边得到图 $T^{\\prime}$, 由归纳假设可知 $T^{\\prime}$ 无圈, 在 $T^{\\prime}$ 中加人 $v$ 及与它关联的边又得到图 $T$, 故 $T$ 是无圈的.\n从而命题正确.\n定理三说明了\"连通\"、\"无圈\"及 \" $e=n-1$ \" 这三个性质中的任何两个都足以保证图 $T$ 是树, 所以也都可以作树的定义.", + "figures": [] +} \ No newline at end of file diff --git a/processed_dataset/text/0056.json b/processed_dataset/text/0056.json new file mode 100644 index 0000000000000000000000000000000000000000..cbd12be2dba14dbf21dbd58866f0732ecaa50186 --- /dev/null +++ b/processed_dataset/text/0056.json @@ -0,0 +1,11 @@ +{ + "source_file": "./raw_volume-zh/volume12/chapter5.tex", + "text": "欧拉问题到\n欧拉 (Euler) 问题起源于著名的七桥游戏.\n位于欧洲的哥尼斯堡 (Königsberg), 景致迷人,碧波荡漾的普莱格尔 (Pregel) 河横贯其境, 河中有两个岛 $A$ 与 $D$, 河上有七座桥连接这两个岛及河的两岸 $B 、 C$ (如图() 所示).\n问:一个旅游者能否通过每座桥一次且仅一次?\n这便是著名的哥尼斯堡七桥问题.\n天才的欧拉, 以其独具的慧眼, 看出了这个似乎是趣味几何问题的潜在意义.\n1736 年, 他发表了题为《哥尼斯堡的七座桥》的论文, 解决了七桥问题.\n通常认为这是图论的第一篇论文.\n欧拉把图()变成一个图 $G$, 如图() 所示.\n岛 $A 、 D$ 及河岸 $B 、 C$ 变成 4 个顶点,图 $G$ 中的 7 条边表示七座桥.\n于是七桥游戏就变成了一笔画问题: 能否一笔画出这个图,每条边都无遗漏也无重复地画到? 而一笔画出这个图并未一定要求最后回到原来的出发点,也就是说, 图 $G$ 能否一笔画成(且最后回到原出发点)的问题就等价于这个图是不是一个链(圈).\n如果图 $G$ 是一条从 $v_1$ 到 $v_{n+1}$ 的链, 那么每一个不同于 $v_1$ 及 $v_{n+1}$ 的顶点 $v_i(i=2,3, \\cdots, n)$ 都是偶顶点.\n因为对顶点 $v_i$ 来说, 有一条进人 $v_i$ 的边就有一条从 $v_i$ 引出的边, 而且进、出的边不能重复已走过的边.\n所以与 $v_i$ 相邻的边总是成双的.\n故图 $G$ 至多有两个奇顶点, 即 $v_1$ 与 $v_{n+1}$. 如果 $G$ 是一个圈, 根据上面的推理, $v_1$ 与 $v_{n+1}$ 也是偶顶点.\n因此, 如果图 $G$ 是一个链 (圈), 那么 $G$ 的奇顶点的个数等于 2 (等于 0 ). 这就是图 $G$ 是一条链(圈)的必要条件.\n换句话说, 如果图 $G$ 的奇顶点个数大于 2 , 那么图 $G$ 就不是一条链, 从而不能一笔画.\n如图() 中, $A 、 B 、 C 、 D$ 都是奇顶点, 因而这个图不是一条链, 所以不能一笔画出.\n也就是说一个旅游者要既无重复也无遗漏地走过如图() 中的七座桥是不可能的.\n下面给出\"一笔画定理\".\n定理一有限图 $G$ 是一条链或圈 (即可以一笔画成) 的充要条件是: $G$ 是连通的, 并且奇顶点个数等于 0 或 2 . 当且仅当奇顶点的个数等于 0 时, 连通图 $G$ 是一个圈.\n证明必要性上面已经证明了,下证充分性.\n如果 $G$ 连通,奇顶点的个数为 0 , 则 $G$ 一定是一个圈.\n从 $G$ 中任一顶点 $v_0$ 出发, 经关联的边 $e_1$ 进人 $v_1$, 因为 $d\\left(v_1\\right)$ 是偶数, 由 $v_1$ 再经关联的边 $e_2$ 可进人 $v_2$, 如此继续下去, 每条边仅取一次, 经过若干步后必可回到 $v_1$, 于是得到一个圈 $\\mu_1: v_0 v_1 \\cdots v_0$.\n如果 $\\mu_1$ 恰好是图 $G$, 则命题得证.\n否则在 $G$ 中去掉 $\\mu_1$ 后得子图 $G_1$, 则 $G_1$ 中每个顶点也都是偶顶点.\n因图 $G$ 是连通的, 所以在 $G_1$ 中必定存在一个和 $\\mu_1$ 公共的顶点 $u$, 在 $G_1$ 中存在一个从 $u$ 出发到 $u$ 的一个圈 $\\mu_2$, 于是 $\\mu_1$ 和 $\\mu_2$ 合起来仍是一个圈.\n重复上述过程, 因为 $G$ 中总共只有有限条边, 总有一个时侯, 得到的圈恰好是图 $G$.\n如果 $G$ 连通,奇顶点个数为 2. 不妨设 $u 、 v$ 是两个奇顶点, 在 $u 、 v$ 之间连一条边 $e$ 得图 $G^{\\prime}$. 于是 $G^{\\prime}$ 中奇顶点个数为 0 , 故 $G^{\\prime}$ 是一个圈.\n从而去掉 $e$ 后, $G$ 便是一条链.\n进一步有如下问题,若一个连通图 $G$ 的奇顶点个数不是 0 或 2 , 那么要多少笔才能画成呢? 我们已经知道, 一个图中的奇顶点个数是偶数, 于是有如下结论.\n定理二如果连通图 $G$ 有 $2 k$ 个奇顶点,则图 $G$ 可以用 $k$ 笔画成,并且至少要用 $k$ 笔才能画成.\n证明把这 $2 k$ 个奇顶点分成 $k$ 对: $v_1, v_1^{\\prime} ; v_2, v_2^{\\prime} ; \\cdots, v_k, v_k^{\\prime}$, 在每对点 $v_i, v_i^{\\prime}$ 之间添加一条边 $e_i$, 得图 $G^{\\prime}$. 图 $G^{\\prime}$ 没有奇顶点, 所以 $G^{\\prime}$ 是一个圈.\n再把这 $k$ 条新添的边去掉, 这个圈至多分为 $k$ 段, 即 $k$ 条链.\n这说明图 $G$ 是可以用 $k$ 笔画成的.\n设图 $G$ 可以分成 $h$ 条链,由定理一,每条链上至多有两个奇顶点,所以\n$$\n2 h \\geqslant 2 k \\text {, }\n$$\n即 $h \\geqslant k$. 图 $G$ 至少要用 $k$ 笔才能画成.", + "figures": [ + "./images/volume12/figures/fig-c5i1.png", + "./images/volume12/figures/fig-c5i1.png", + "./images/volume12/figures/fig-c5i2.png", + "./images/volume12/figures/fig-c5i2.png", + "./images/volume12/figures/fig-c5i1.png" + ] +} \ No newline at end of file diff --git a/processed_dataset/text/0057.json b/processed_dataset/text/0057.json new file mode 100644 index 0000000000000000000000000000000000000000..dfdefb49b39bbfd53b7fcb4052125f06ccc55e5c --- /dev/null +++ b/processed_dataset/text/0057.json @@ -0,0 +1,8 @@ +{ + "source_file": "./raw_volume-zh/volume12/chapter5.tex", + "text": "如图() 中的图需 5笔才能画成, 如图() 中的图 4 笔可以画成.", + "figures": [ + "./images/volume12/figures/fig-c5i5.png", + "./images/volume12/figures/fig-c5i6.png" + ] +} \ No newline at end of file diff --git a/processed_dataset/text/0058.json b/processed_dataset/text/0058.json new file mode 100644 index 0000000000000000000000000000000000000000..dfeb15a45f88f8371a4e984b8e731f43ffcfbe19 --- /dev/null +++ b/processed_dataset/text/0058.json @@ -0,0 +1,5 @@ +{ + "source_file": "./raw_volume-zh/volume12/chapter6.tex", + "text": "哈密顿问题.\n1856 年, 著名英国数学家哈密顿 (Willian Rowan Hamilton) 提出一个名为 \"环游世界\" 的游戏.\n他用一个正十二面体的二十个顶点代表二十个大城市, 要求沿着棱, 从一个城市出发,经过每个城市恰好一次, 然后回到出发点.\n这个游戏曾经风靡一时, 在这个游戏中提到了这样一种链(圈): 它经过图上各顶点一次并且仅仅一次.\n这种链 (圈) 称为哈密顿链 (圈),一个图若包含哈密顿圈,则称这个图是哈密顿图.\n从表面上看,哈密顿问题与欧拉问题很相似,但实际上有着本质的区别, 它是图论中尚未解决的困难问题之一.\n迄今为止还没有找到判断它的充分必要条件, 所以对不同类型的问题, 有不同的判断方法, 下面通过例题作些介绍.", + "figures": [] +} \ No newline at end of file diff --git a/processed_dataset/text/0059.json b/processed_dataset/text/0059.json new file mode 100644 index 0000000000000000000000000000000000000000..7b2d66d1b1039f1aad3ba8038dae7afb5113248b --- /dev/null +++ b/processed_dataset/text/0059.json @@ -0,0 +1,9 @@ +{ + "source_file": "./raw_volume-zh/volume12/chapter6.tex", + "text": "对于一个连通图, 是否存在哈密顿链 (圈) 的问题.\n虽然直到最后还不知道有什么充要条件, 然而许多第一流的数学家经过一个多世纪的努力, 已经知道一些必要条件和一些充分条件, 下面给出一个简单图具有哈密顿链的充分条件.\n定理二设 $G$ 是 $n(n \\geqslant 3)$ 阶简单图, 且对每一对顶点 $v, v^{\\prime}$ 有\n$$\nd(v)+d\\left(v^{\\prime}\\right) \\geqslant n-1,\n$$\n则图 $G$ 有哈密顿链.\n%%%%\n证明:先证明 $G$ 是连通图.\n若 $G$ 有两个或两个以上的连通部分, 设其中之一有 $n_1$ 个顶点, 另一部分有 $n_2$ 个顶点.\n分别从中各取一顶点 $v_1 、 v_2$, 则 $d\\left(v_1\\right) \\leqslant n_1-1, d\\left(v_2\\right) \\leqslant n_2-1$. 故\n$$\nd\\left(v_1\\right)+d\\left(v_2\\right) \\leqslant n_1+n_2-2) 所示.\n因为否则 $v_p$ 最多只和 $p-k-1$ 个顶点相邻, 即排除 $v_{j_1-1} 、 v_{j_2-1} 、 \\cdots 、 v_{j_k-1}$ 和 $v_p$ 自身, 这样\n$$\nd\\left(v_1\\right)+d\\left(v_p\\right) \\leqslant k+(p-k-1)=p-1) 所示.\n于是就得到一个包含 $v_1, v_2, \\cdots, v_p, v^{\\prime}$ 的圈: $v^{\\prime} v_k v_{k+1} \\cdots v_{l-1} v_p v_{p-1} \\cdots v_l v_1 v_2 \\cdots v_k v^{\\prime}$. 不断重复上面的步骤直到存在一条具有 $n-1$ 条边的链为止.\n易知定理二的条件对于图中哈密顿链的存在性只是充分的, 但并不是必要条件.\n设 $G$ 是 $n$ 边形, 如图(), 其中 $n=6$, 虽然任何两个顶点的度之和是 $4<6-1$, 但在 $G$ 中有一条哈密顿链.", + "figures": [ + "./images/volume12/figures/fig-c6i8.png", + "./images/volume12/figures/fig-c6i9.png", + "./images/volume12/figures/fig-c6i10.png" + ] +} \ No newline at end of file diff --git a/processed_dataset/text/0060.json b/processed_dataset/text/0060.json new file mode 100644 index 0000000000000000000000000000000000000000..064cf102944a48efeff7e8aa8655c735f75473fc --- /dev/null +++ b/processed_dataset/text/0060.json @@ -0,0 +1,7 @@ +{ + "source_file": "./raw_volume-zh/volume12/chapter6.tex", + "text": "定理三(Ore, 1960) $\\quad G$ 是 $n(n \\geqslant 3)$ 阶简单图, 且对每一对不相邻的顶点 $v 、 v^{\\prime}$ 有\n$$\nd(v)+d\\left(v^{\\prime}\\right) \\geqslant n,\n$$\n那么图 $G$ 有哈密顿圈.\n证明当 $n=3$ 时, 由所给条件知 $G$ 一定是完全图 $K_3$, 命题成立.\n设 $n \\geqslant 4$, 用反证法证明.\n设 $G$ 是有 $n$ 个顶点且满足度数条件却没有哈密顿圈的图.\n不妨设 $G$ 是具有这种性质的边数最大的图, 也就是说 $G$ 添上一条边就具有哈密顿圈 (否则 $G$ 可以添加一些边, 直到不能再添为止, 加边后顶点的度数条件仍满足), 由此得出在图 $G$ 中有一条包含图中每一个顶点的哈密顿链, 记为 $v_1 v_2 \\cdots v_n$. 则 $v_1$ 与 $v_n$ 不相邻, 于是\n$$\nd\\left(v_1\\right)+d\\left(v_n\\right) \\geqslant n .\n$$\n那么在 $v_2, v_3, \\cdots, v_{n-1}$ 中必有一点 $v_i$, 使 $v_1$ 与 $v_i$ 相邻, $v_n$ 与 $v_{i-1}$ 相邻, 如图() 所示.\n否则, 有 $d\\left(v_1\\right)=k$ 个点 $v_{i_1}, v_{i_2}, \\cdots, v_{i_k} \\left(2 \\leqslant i_1 \\leqslant i_2 \\leqslant \\cdots \\leqslant i_k \\leqslant n-1\\right)$ 与 $v_1$ 相邻, 而 $v_n$ 与 $v_{i_1-1}, v_{i_2-1}, \\cdots,v_{i_k-1}$ 都不相邻, 从而\n$$\nd\\left(v_n\\right) \\leqslant n-1-k,\n$$\n则\n$$\nd\\left(v_1\\right)+d\\left(v_n\\right) \\leqslant k+n-1-k=n-1), 表面看有几条边相交,但是把它画成与它同构的如图(), 则可看出它是一个平面图.\n我们说到一个平面图, 总假定它已经按这样的要求画好了.\n一个平面图的顶点和边把平面分成一个一个互相隔开的区域, 每一个这样的区域称为平面图的一个面.\n这些面中有一个在所有边的外面, 称为外部面,其余的就称为内部面.\n例如图() 中, $F_1 、 F_2 、 F_3 、 F_4$ 是内部面, $F_5$ 是外部面.\n在中学课本中已经见到过关于凸多面体的欧拉公式, 设凸多面体有 $v$ 个顶点、 $e$ 条棱和 $f$ 块面,则 $v-e+f=2$. 我们可以把这个公式推广到平面图上来.\n定理一(欧拉定理)如果一个连通的平面图 $G$ 有 $v$ 个顶点、 $e$ 条边、 $f$ 个面, 那么\n$$\nv-e+f=2 .\n$$\n证明对 $G$ 的边数用数学归纳法.\n若 $G$ 只有一个顶点, 则 $v=1, e=0, f=1$, 故 $v-e+f=2$ 成立.\n若 $G$ 为一条边, 则 $v=2, e=1, f=1$, 所以 $v-e+f=2$ 成立.\n设 $G$ 为 $k$ 条边时欧拉公式成立, 即 $v_k-e_k+f_k=2$. 现考察 $G$ 为 $k+1$ 条边时的情形.\n由于在 $k$ 条边的连通图上增加一条边, 使它仍为连通图, 只有两种情形:\n(i) 增加一个新顶点 $v^{\\prime}, v^{\\prime}$ 与图中的一点 $v$ 相邻, 如图() 所示, 此时 $v_k$ 与 $e_k$ 都增加 1 , 而面数 $f_k$ 不变, 故\n$$\n\\left(v_k+1\\right)-\\left(e_k+1\\right)+f_k=v_k-e_k+f_k=2 .\n$$\n(ii) 用一条边连接图中的两个顶点 $v$ 和 $v^{\\prime}$, 如图() 所示, 这时 $e_k$ 和 $f_k$ 都增加 1 而顶点数 $v_k$ 没有变, 故\n$$\nv_k-\\left(e_k+1\\right)+\\left(f_k+1\\right)=v_k-e_k+f_k=2 .\n$$\n按归纳法原理,定理对任何正整数 $e$ 成立.\n欧拉定理的一个重要应用是由此可以决定一个平面简单图中最多的边数.\n因为一个面上至少有 3 条边, $f$ 个面的边界上至少有 $3 f$ 条边.\n另外,一条边最多是 2 个面的边界,所以\n$$\n2 e \\geqslant 3 f, f \\leqslant \\frac{2}{3} e .\n$$\n代入欧拉公式:\n$$\n2=v-e+f \\leqslant v-e+\\frac{2}{3} e,\n$$\n即\n$$\ne \\leqslant 3 v-6\n$$\n这就证明了下面的定理.\n定理二一个连通的平面简单图 $G$ 有 $v(v \\geqslant 3)$ 个顶点及 $e$ 条边, 则 $e \\leqslant 3 v-6$.\n其实定理二对不连通的平面简单图也成立.\n应用定理二可以判定某些图是非平面图.", + "figures": [ + "./images/volume12/figures/fig-c7i1-1.png", + "./images/volume12/figures/fig-c7i1-2.png", + "./images/volume12/figures/fig-c7i2.png", + "./images/volume12/figures/fig-c7i3-1.png", + "./images/volume12/figures/fig-c7i3-2.png" + ] +} \ No newline at end of file diff --git a/processed_dataset/text/0062.json b/processed_dataset/text/0062.json new file mode 100644 index 0000000000000000000000000000000000000000..25ee26e6cee8f254b71ab72d291f7304b4b8c1e3 --- /dev/null +++ b/processed_dataset/text/0062.json @@ -0,0 +1,7 @@ +{ + "source_file": "./raw_volume-zh/volume12/chapter7.tex", + "text": "欧拉公式和导出的不等式虽然可以用来否定一些图是平面图, 但是它对于肯定一个图是平面图却无能为力.\n1930 年波兰数学家库拉托夫斯基证明了一个简洁而漂亮的结果: 所有非平面图都包含着 $K_5$ 或 $K_{3,}{ }_3$ 作为子图.\n为了明确地叙述这个结果, 先给出两个图\"同肧\"的概念.\n如果两个图中的一个图是由另一个图的边上插人一些新的顶点而得到的,那么,这两个图称为同胚的.\n如图() 中的两个图是同胚的.\n根据两个图同胚的概念, 可以知道一个图的边上插人或删去一些度数为 2 的顶点后,不影响图的平面性.\n下面给出库拉托夫斯基定理:\n定理三一个图是平面图当且仅当它不包含同胚于 $K_5$ 或 $K_{3,3}$ 的子图.\n这个定理虽然很基本, 但证明很长, 故从略.", + "figures": [ + "./images/volume12/figures/fig-c7i4.png" + ] +} \ No newline at end of file diff --git a/processed_dataset/text/0063.json b/processed_dataset/text/0063.json new file mode 100644 index 0000000000000000000000000000000000000000..c43b54add2c22e7c8a583a5bb246082a2e20c2c2 --- /dev/null +++ b/processed_dataset/text/0063.json @@ -0,0 +1,5 @@ +{ + "source_file": "./raw_volume-zh/volume12/chapter7.tex", + "text": "1968 年, 两位苏联数学家柯耶瑞夫 (Kozyrev) 和戈林伯格 (Grinberg) 给出了平面图具有哈密顿圈的一个必要条件.\n定理四如果一个平面图有哈密顿圈 $c$, 用 $f_i^{\\prime}$ 表示在 $c$ 的内部的 $i$ 边形的个数,用 $f_i^{\\prime \\prime}$ 表示在 $c$ 的外部的 $i$ 边形的个数,则\n(1) $1 \\cdot f_3^{\\prime}+2 \\cdot f_4^{\\prime}+3 \\cdot f_5^{\\prime}+\\cdots=n-2$;\n(2) $1 \\cdot f_3^{\\prime \\prime}+2 \\cdot f_4^{\\prime \\prime}+3 \\cdot f_5^{\\prime \\prime}+\\cdots=n-2$;\n(3) $1 \\cdot\\left(f_3^{\\prime}-f_3^{\\prime \\prime}\\right)+2 \\cdot\\left(f_4^{\\prime}-f_4^{\\prime \\prime}\\right)+3 \\cdot\\left(f_5^{\\prime}-f_5^{\\prime \\prime}\\right)+\\cdots=0$.\n其中 $n$ 为 $G$ 的顶点数, 显然也是 $c$ 的长.\n证明设 $c$ 的内部有 $d$ 条边.\n由于 $G$ 是平面图,它的边都不相交,所以一条边把它经过的面分成两部分.\n设想这些边是一条一条地放进图里去的, 每放进一条边就使 $c$ 内部的面增加一个, 因此 $d$ 条边把 $c$ 的内部分成了 $d+1$ 个面.\n于是 $c$ 的内部的面的总数为\n$$\nf_2^{\\prime}+f_3^{\\prime}+f_4^{\\prime}+f_5^{\\prime}+\\cdots=d+1 . \\label{eq1}\n$$\n在 $c$ 内每个 $i$ 边形中记上数字 $i$, 各面所记数字之和就是围成这些面的边的总数, $c$ 内部的每一条边都被数了两次, 而 $c$ 上的 $n$ 条边, 每条边都只数了一次,于是\n$$\n2 f_2^{\\prime}+3 f_3^{\\prime}+4 f_4^{\\prime}+5 f_5^{\\prime}+\\cdots=2 d+n . \\label{eq2}\n$$\n\\ref{eq2} 式减去\\ref{eq1}式的两倍,得\n$$\n1 \\cdot f_3^{\\prime}+2 \\cdot f_4^{\\prime}+3 \\cdot f_5^{\\prime}+\\cdots=n-2 . \\label{eq3}\n$$\n类似地可以推得\n$$\n1 \\cdot f_3^{\\prime \\prime}+2 \\cdot f_4^{\\prime \\prime}+3 \\cdot f_5^{\\prime \\prime}+\\cdots=n-2 . \\label{eq4}\n$$\n式\\ref{eq3}、\\ref{eq4}两式相减即得\n$$\n1 \\cdot\\left(f_3^{\\prime}-f_3^{\\prime \\prime}\\right)+2 \\cdot\\left(f_4^{\\prime}-f_4^{\\prime \\prime}\\right)+3 \\cdot\\left(f_5^{\\prime}-f_5^{\\prime \\prime}\\right)+\\cdots=0 .\n$$", + "figures": [] +} \ No newline at end of file diff --git a/processed_dataset/text/0064.json b/processed_dataset/text/0064.json new file mode 100644 index 0000000000000000000000000000000000000000..7f3d9911ed1d86c37e497243cb8798f12fc9469f --- /dev/null +++ b/processed_dataset/text/0064.json @@ -0,0 +1,5 @@ +{ + "source_file": "./raw_volume-zh/volume12/chapter8.tex", + "text": "拉姆赛问题.\n通常, 我们把与图的染色、拉姆赛 (Ramsey, 英国逻辑学家) 数、抽庶原则关联的问题称为拉姆赛问题.\n我们先从匈牙利的一个竞赛题谈起.\n1947 年匈牙利数学奥林匹克中出了这样一道试题:", + "figures": [] +} \ No newline at end of file diff --git a/processed_dataset/text/0065.json b/processed_dataset/text/0065.json new file mode 100644 index 0000000000000000000000000000000000000000..2a268b2226677e90aec7a785d92d956381cadc9a --- /dev/null +++ b/processed_dataset/text/0065.json @@ -0,0 +1,7 @@ +{ + "source_file": "./raw_volume-zh/volume12/chapter8.tex", + "text": "$r_k$ 的存在性是由英国数学家、数理逻辑学家拉姆赛首先证明的, 所以 $r_k$ 叫拉姆赛数,关于 $r_k$ 我们有如下结论.\n定理二 (1) 对每个正整数 $k$, 拉姆赛数 $r_k$ 存在, 并且当 $k \\geqslant 2$ 时,\n$$\nr_k \\leqslant k\\left(r_{k-1}-1\\right)+2 \\text {; }\n$$\n(2) 对一切自然数 $k$,\n$$\nr_k \\leqslant 1+1+k+k(k-1)+\\cdots+\\frac{k !}{2 !}+\\frac{k !}{1 !}+k ! .\n$$\n证明 (1) 对 $k$ 进行归纳.\n我们已知 $r_1 、 r_2$ 存在, 且 $r_1=3, r_2=6 \\leqslant2\\left(r_1-1\\right)+2$.\n设 $r_k$ 存在, 且 $r_k \\leqslant k\\left(r_{k-1}-1\\right)+2$ 成立.\n取 $n=(k+1)\\left(r_k-1\\right)+2$, 并设 $K_n$ 是 $k+1$ 色完全图, 它们的顶点为 $A_1, A_2, \\cdots, A_n$. 任取 $K_n$ 的一个顶点 $A_1$, 从它出发有 $n-1=(k+1)\\left(r_k-1\\right)+1$ 条边.\n这些边共有 $k+1$ 种颜色, 由抽庶原则, 这些边中至少有 $r_k$ 条同色.\n不妨设这 $r_k$ 条边是 $A_1 A_2, A_1 A_3, \\cdots$, $A_1 A_{r_k+1}$, 且都染 $c_1$ 色.\n考虑由顶点 $A_2, A_3, \\cdots, A_{r_k+1}$ 构成的 $r_k$ 阶子图 $K_{r_k}$. 若 $K_{r_k}$ 含有 $c_1$ 色边, 例如 $A_2 A_3$, 则 $\\triangle A_1 A_2 A_3$ 为同色三角形; 若 $K_{r_k}$ 不含有 $c_1$ 色边, 则 $K_{r_k}$ 的边只有 $k$ 种颜色, 即 $K_{r_k}$ 为 $k$ 色完全图, 按归纳假设, $K_{r_k}$ 含有同色三角形.\n总之, $K_n$ 含有同色三角形.\n于是知\n$M=\\{m \\mid$ 任何 $m$ 阶 $k+1$ 色完全图都含有同色三角形 $\\}$\n是自然数集 $N$ 的非空子集, 从而有最小值.\n即 $r_{k+1}$ 存在, 且 $r_{k+1} \\leqslant n=(k+$ 1) $\\left(r_k-1\\right)+2$.\n(2) 用归纳法.\n当 $k=1$ 时, $r_1=3 \\leqslant 1+1+1$. 设命题对 $k$ 成立, 则应用 (1) 的结果及归纳假设, 得\n$$\n\\begin{aligned}\nr_{k+1} \\leqslant & (k+1)\\left(r_k-1\\right)+2 \\\\\n\\leqslant & (k+1)\\left[1+k+k(k-1)+\\cdots+\\frac{k !}{2 !}+\\frac{k !}{1 !}+k !\\right]+2 \\\\\n= & (k+1)+(k+1) k+(k+1) k(k-1)+\\cdots+\\frac{(k+1) !}{2 !}+\\frac{(k+1) !}{1 !} \\\\\n& \\quad+(k+1) !+2 \\\\\n& \\quad 1+1+(k+1)+(k+1) k+\\cdots+\\frac{(k+1) !}{2 !}+\\frac{(k+1) !}{1 !}+(k+1) !,\n\\end{aligned}\n$$\n所以命题对 $k+1$ 也成立.\n如果利用高等数学中关于自然对数的底数 $\\mathrm{e}$ 的展开式\n$$\n\\mathrm{e}=1+\\frac{1}{1 !}+\\frac{1}{2 !}+\\cdots+\\frac{1}{n !}+\\cdots,\n$$\n可将 (2) 的结果简化成 $r_k \\leqslant[k$ ! $\\mathrm{e}]+1$. 这里 $[x]$ 表示不超过 $x$ 的最大整数.\n定理二虽然证明了 $r_k$ 的存在性,并给出了 $r_k$ 的一个上界.\n但是 $r_k$ 的准确值, 只知道三个, 除了前面提及的 $r_1=3, r_2=6$ 外, 还知道一个 $r_3=17$. 事实上, 根据定理二的 (1), 我们有 $r_3 \\leqslant 3\\left(r_2-1\\right)+2=3 \\times 5+2=17$. 直接仿照定理二的证明得出的这个结果还被编为 1964 年第六届国际数学奥林匹克试题:\n有 17 位科学家, 其中每一个人和其他的所有的人通信, 他们在通信中只讨论 3 个题目,而且每两个科学家之间只讨论一个题目.\n求证: 至少有三个科学家相互之间讨论同一个题目.\n另一方面,可以给完全图 $K_{16}$ 的边涂上三种颜色,使得图中没有一个同色三角形,如图() 所示,实线表示红边,虚线表示蓝边,未画线的表示黄边.\n这就是说, $r_3 \\geqslant 17$. 所以 $r_3=17$.\n我们来看定理一的另一种推广:\n设完全图 $K_n$ 的每条边被染以红、蓝两色, 即 $K_n$ 是染红、蓝两色的完全图,则对固定的自然数 $p, q$, 当 $n$ 充分大时,红、蓝两色完全图 $K_n$ 中, 必然出现红色 $K_p$, 或蓝色 $K_q$. 我们把满足上述性质的最小 $n$ 记为 $r(p, q), r(p, q)$ 也称为拉姆赛数.\n用子图、补图的概念, $r(p, q)$ 也可说成使完全图 $K_n$ 的任何 $n$ 阶子图 $G$ 或者包含一个完全子图 $K_p$, 或者它的补图 $G$ 包含一个完全子图 $K_q$ 的 $n$ 的最小值.\n由定义及定理一, 我们知 $r(3,3)=r_2=6$, 另外, 还容易发现 $r(1, q)= r(p, 1)=1$.\n为了更好地理解后面将要给出的 $r(p, q)$ 的一般结果, 我们证明一个具体的例子: $r(3,4)=9$. 先证 $r(3,4) \\leqslant 9$, 它也可以用一个类似于例 1 的形式给出.", + "figures": [ + "./images/volume12/figures/fig-c8i5.png" + ] +} \ No newline at end of file diff --git a/processed_dataset/text/0066.json b/processed_dataset/text/0066.json new file mode 100644 index 0000000000000000000000000000000000000000..7c155d9d3c8707343308dc9dbdd05622c0b90852 --- /dev/null +++ b/processed_dataset/text/0066.json @@ -0,0 +1,7 @@ +{ + "source_file": "./raw_volume-zh/volume12/chapter8.tex", + "text": "关于 $r(p, q)$ 我们有如下的结论.\n定理三 (1) $r(2, q)=q, r(p, 2)=p$;\n(2) $r(p, q)=r(q, p)$;\n(3) 在 $p \\geqslant 2, q \\geqslant 2$ 时,\n$$\nr(p, q) \\leqslant r(p, q-1)+r(p-1, q) .\n$$\n并且在 $r(p, q-1)$ 与 $r(p-1, q)$ 都是偶数时, 成立严格不等式.\n为了叙述方便,我们采用子图、补图的说法给出证明.\n(3) 的证明较难, 为了加深理解, 阅读时可参照例 4 的证明.\n证明 (1) 设 $G$ 是有 $q$ 个顶点的图.\n若 $G$ 中有两个顶点相邻, 这时 $G$ 含有 $K_2$, 否则 $\\bar{G}$ 就是 $K_q$, 所以 $r(2, q) \\leqslant q$. 又由 $q-1$ 个两两不相邻的点所成的图 $G$ 显然不含 $K_2$, 它的补图 $\\bar{G}$ 是 $K_{q-1}$, 不含 $K_q$, 所以 $r(2, q) \\geqslant q$.\n综上所述, $r(2, q)=q$. 同理可证, $r(p, 2)=p$.\n(2) 设图 $G$ 有 $r(p, q)$ 个顶点, 则 $\\bar{G}$ 也有 $r(p, q)$ 个顶点.\n于是 $\\bar{G}$ 中含有 $K_p$, 或者 $G$ 中含有 $K_q$. 换言之, $G$ 中含 $K_q$, 或 $\\bar{G}$ 中含 $K_p$, 故 $r(p, q) \\geqslant r(q, p)$.\n同理 $r(q, p) \\geqslant r(p, q)$, 所以 $r(p, q)=r(q, p)$.\n(3) 设图 $G$ 有 $r(p, q-1)+r(p-1, q)$ 个顶点, $v_1$ 是 $G$ 的一个顶点.\n若 $d\\left(v_1\\right) \\geqslant r(p-1, q)$. 取 $\\delta=r(p-1, q)$ 个与 $v_1$ 相邻的顶点 $v_2$, $v_3, \\cdots, v_\\delta, v_{\\delta+1}$. 将 $G$ 中其余的顶点以及关联的边去掉得 $G_1$, 根据 $\\delta=r(p- 1, q)$ 的定义, $G_1$ 中含有 $K_{p-1}$ 或 $\\bar{G}_1$ 中含有 $K_q$. 如果 $G_1$ 含有 $K_{p-1}$, 那么在 $G$ 中, $v_1$ 与这个 $K_{p-1}$ 组成完全图 $K_p$. 如果 $\\bar{G}_1$ 中含有 $K_q$, 则 $\\bar{G}$ 中也含有这个 $K_q$.\n若与 $v_1$ 相邻的顶点数 $13, r(4,4)>17$ 就够了.\n这里证明前者, 后者的证明从略.\n考察如图() 所示的图 $G$, 它不含 $K_3$, 它的补图 $\\bar{G}$ 不含 $K_5$, 所以 $r(3,5)>13$.\n应用定理三,我们可以得出 $r(p, q)$ 的一个简单的上界.", + "figures": [ + "./images/volume12/figures/fig-c8i7.png" + ] +} \ No newline at end of file diff --git a/processed_dataset/text/0067.json b/processed_dataset/text/0067.json new file mode 100644 index 0000000000000000000000000000000000000000..7da7b14ed074a24ba2f0231559ad2ce06148392f --- /dev/null +++ b/processed_dataset/text/0067.json @@ -0,0 +1,5 @@ +{ + "source_file": "./raw_volume-zh/volume12/chapter8.tex", + "text": "定理四当 $p \\geqslant 2, q \\geqslant 2$ 时,\n$$\nr(p, q) \\leqslant \\mathrm{C}_{p+q-2}^{p-1} .\n$$\n证明记 $l=p+q$, 我们对 $l$ 进行归纳.\n当 $l=4$ 时, $p=q=2$. 左边是 $r(2,2)=2$, 右边是 $\\mathrm{C}_{4-2}^1=2$, 命题成立.\n设当 $l=k(k \\geqslant 4)$ 时, 命题成立.\n当 $l=k+1$ 时, 在 $p=k-1, q=2$ 或 $p=2, q=k-1$ 的情况下,\n$$\nr(k-1,2)=r(2, k-1)=k-1=\\mathrm{C}_{k-1}^{k-2}=\\mathrm{C}_{k-1}^1 .\n$$\n命题成立.\n当 $p \\geqslant 3, q \\geqslant 3, p+q=k+1$ 的情况下, 应用定理三的 (3) 及归纳假设,有\n$$\n\\begin{aligned}\nr(p, q) & \\leqslant r(p-1, q)+r(p, q-1) \\\\\n& \\leqslant \\mathrm{C}_{p+q-3}^{p-2}+\\mathrm{C}_{p+q-3}^{p-1} \\\\\n& =\\mathrm{C}_{p+q-2}^{p-1} .\n\\end{aligned}\n$$\n命题仍成立.\n根据数学归纳法, 定理四得证.\n尽管如此, $r(p, q)$ 的准确值却不容易求得.\n除 $r(1, q)=r(p, 1)=1$, $r(p, 2)=p, r(2, q)=q$ 外, 目前已经确定的 $r(p, q)$ 仅有少数几个, 它们列在下面的表中.\n其中分数的分子表示下界,分母表示上界.\n如果把前面两种推广结合起来,可得到如下的推广:\n用 $l$ 种颜色 $c_1, c_2, \\cdots, c_l$ 去染完全图 $K_n$ 的边, 每边染且只染一种颜色, 得到一个 $l$ 色完全图.\n当 $n$ 充分大时, $l$ 色完全图 $K_n$ 中, 或者包含一个 $c_1$ 色的完全子图 $K_{p_1}$, 或者包含一个 $c_2$ 色的完全子图 $K_{p_2}, \\cdots$, 或者包含一个 $c_i$ 色的完全子图 $K_{p_l}$. 我们把满足上述性质的最小 $n$ 记为 $r\\left(p_1, p_2, \\cdots, p_l\\right)$, 它也称为拉姆赛数.\n如果利用所谓\"超图\"的概念, 那么还可作推广.\n这里就不深人介绍了.", + "figures": [] +} \ No newline at end of file diff --git a/processed_dataset/text/0068.json b/processed_dataset/text/0068.json new file mode 100644 index 0000000000000000000000000000000000000000..5453cb808a10d0b500e99d808165eadc37d65a7e --- /dev/null +++ b/processed_dataset/text/0068.json @@ -0,0 +1,7 @@ +{ + "source_file": "./raw_volume-zh/volume12/chapter9.tex", + "text": "在第一节里,我们曾说过图是描述一些对象之间的某种特定关系的工具.\n前面所说的图都是无向图, 它所描述的关系是对称性关系.\n在现实生活中有许多关系不是对称性的, 如认识关系, 甲认识乙, 并不意味着乙也认识甲, 比赛的胜负关系等也是这样, 由此我们可以抽象出有向图的概念.\n把一个图的每一条边都规定一个方向, 称这个图为有向图.\n有向图的边称为弧, 若顶点 $v_i$ 与 $v_j$ 有弧相连, 弧上的箭头方向从 $v_i$ 指向 $v_j$, 记这条弧为 $\\left(v_i\\right.$, $\\left.v_j\\right)$, 且称 $v_i$ 为起点, $v_j$ 为终点.\n通常将有向图记为 $D=(V, U)$, 其中 $V$ 表示 $D$ 的顶点集合, $U$ 表示 $D$ 的弧的集合.\n如图() 所示的有向图中,顶点集\n$$\nV=\\left\\{v_1, v_2, v_3, v_4, v_5, v_6\\right\\},\n$$\n弧的集合\n$$\n\\begin{aligned}\nU= & \\left\\{\\left(v_1, v_2\\right),\\left(v_2, v_3\\right),\\left(v_5, v_2\\right),\\left(v_4, v_2\\right),\\left(v_4, v_6\\right),\\right. \\\\\n& \\left.\\left(v_5, v_6\\right),\\left(v_5, v_4\\right),\\left(v_3, v_5\\right),\\left(v_4, v_5\\right)\\right\\} .\n\\end{aligned}\n$$\n这一节所说的有向图都是简单有向图, 也就是不含环 (即起点和终点相同的弧), 也不含多重弧 (即由一点向另一点的多于一条的弧) 的有向图.\n如果有向图 $G$ 的弧集 $U$ 中有弧 $\\left(v_i, v_j\\right)$ 或 $\\left(v_j, v_i\\right)$, 就称顶点 $v_i$ 和 $v_j$ 相邻, 否则称顶点 $v_i$ 和 $v_j$ 不相邻.\n以顶点 $v_i$ 为起点的弧的条数称为 $v_i$ 的出度, 记为 $d^{+}\\left(v_i\\right)$, 以 $v_i$ 为终点的弧的条数称为 $v_i$ 的人度, 记为 $d^{-}\\left(v_i\\right)$.\n有 $n$ 个顶点, 且每两个顶点都恰有一条弧相连的有向图称为竞赛图, 记作 $\\bar{K}_n$.\n定理一设 $n$ 阶竞赛图 $\\bar{K}_n$ 的顶点为 $v_1, v_2, \\cdots, v_n$, 则\n$$\n\\begin{aligned}\n& d^{+}\\left(v_1\\right)+d^{+}\\left(v_2\\right)+\\cdots+d^{+}\\left(v_n\\right) \\\\\n= & d^{-}\\left(v_1\\right)+d^{-}\\left(v_2\\right)+\\cdots+d^{-}\\left(v_n\\right)\n\\end{aligned}\n$$\n$$\n=\\frac{1}{2} n(n-1) .\n$$\n证明因为 $\\bar{K}_n$ 中每一条弧产生一个人度和一个出度, 并且每两点之间有且仅有一条弧, 所以 $\\bar{K}_n$ 中所有顶点的人度之和等于出度之和, 并且等于弧数.\n即\n$$\n\\begin{aligned}\n& d^{+}\\left(v_1\\right)+d^{+}\\left(v_2\\right)+\\cdots+d^{+}\\left(v_n\\right) \\\\\n= & d^{-}\\left(v_1\\right)+d^{-}\\left(v_2\\right)+\\cdots+d^{-}\\left(v_n\\right) \\\\\n= & \\frac{1}{2} n(n-1) .\n\\end{aligned}\n$$", + "figures": [ + "./images/volume12/figures/fig-c9i1.png" + ] +} \ No newline at end of file diff --git a/processed_dataset/text/0069.json b/processed_dataset/text/0069.json new file mode 100644 index 0000000000000000000000000000000000000000..469998917e292df1951c4fc76d354844aff01800 --- /dev/null +++ b/processed_dataset/text/0069.json @@ -0,0 +1,5 @@ +{ + "source_file": "./raw_volume-zh/volume12/chapter9.tex", + "text": "定理二竞赛图中总存在这样一个顶点, 使得从这一顶点出发, 通过长最多为 2 的路可以到达其他所有顶点.\n证明设竞赛图 $\\bar{K}_n$ 中出度最大的顶点为 $v_1$, 以 $v_1$ 为起点的弧的终点集合记为 $\\mathbf{N}^{+}\\left(v_1\\right)$. 若命题结论不真, 则必存在顶点 $v_2\\left(v_2 \\neq v_1\\right), v_2 \\notin \\mathbf{N}^{+}\\left(v_1\\right)$, 且对每一点 $u \\in \\mathbf{N}^{+}\\left(v_1\\right)$ 都有一条从 $v_2$ 到 $u$ 的弧 $\\left(v_2, u\\right)$, 又有弧 $\\left(v_2, v_1\\right)$, 故 $d^{+}\\left(v_2\\right) \\geqslant d^{+}\\left(v_1\\right)+1$, 这与 $v_1$ 是出度最大的顶点矛盾.\n定理得证.", + "figures": [] +} \ No newline at end of file diff --git a/processed_dataset/text/0070.json b/processed_dataset/text/0070.json new file mode 100644 index 0000000000000000000000000000000000000000..646fa796bda8c53cf1029aecba9d23ce7c9b0a82 --- /dev/null +++ b/processed_dataset/text/0070.json @@ -0,0 +1,7 @@ +{ + "source_file": "./raw_volume-zh/volume12/chapter9.tex", + "text": "若有向图 $D$ 中有一条路包含图 $D$ 的一切顶点, 则称这条路为哈密顿路.\n对竟赛图 $\\bar{K}_n$, 有如下结论.\n定理三竞赛图 $\\bar{K}_n$ 中存在一条长为 $n-1$ 的哈密顿路.\n证明对顶点数 $n$ 用数学归纳法证明.\n$n=2$ 时显然.\n设命题对 $\\leqslant k$ 个顶点的竞赛图成立.\n当 $n=k+1$ 时, 从 $k+1$ 个顶点中任取一个 $v$, 在 $\\bar{K}_{k+1}$ 中去掉 $v$ 及其相邻的弧, 根据归纳假设, $\\bar{K}_{k+1}-v$ 中存在哈密顿路,设为 $v_1, v_2, \\cdots, v_k$.\n如果有弧 $\\left(v_k, v\\right)$, 那么 $v_1, v_2, \\cdots, v_k, v$ 就是一条哈密顿路.\n如果有弧 $\\left(v, v_1\\right)$, 那么 $v, v_1, v_2, \\cdots, v_k$ 就是一条哈密顿路.\n否则就存在弧 $\\left(v, v_k\\right),\\left(v_1, v\\right)$, 那么一定有一个 $i(1 \\leqslant i \\leqslant k-1)$, 使弧 $\\left(v_i, v\\right)$ 与 $\\left(v, v_{i+1}\\right)$ 同时存在, 这时 $v_1, \\cdots, v_i, v, v_{i+1}, \\cdots, v_k$ 就是所求的哈密顿路, 如图() 所示.", + "figures": [ + "./images/volume12/figures/fig-c9i2.png" + ] +} \ No newline at end of file diff --git a/processed_dataset/text/0071.json b/processed_dataset/text/0071.json new file mode 100644 index 0000000000000000000000000000000000000000..bbd076ed1d38fa4fd67e3272e3c6f5da8914ab02 --- /dev/null +++ b/processed_dataset/text/0071.json @@ -0,0 +1,5 @@ +{ + "source_file": "./raw_volume-zh/volume12/chapter9.tex", + "text": "定理四竞赛图 $\\bar{K}_n(n \\geqslant 3)$ 中有一个回路是三角形的充分必要条件是有两个顶点 $v$ 与 $v^{\\prime}$, 满足\n$$\nd^{+}(v)=d^{+}\\left(v^{\\prime}\\right)\n$$\n证明设顶点 $v$ 与 $v^{\\prime}$, 满足 $d^{+}(v)=d^{+}\\left(v^{\\prime}\\right)$, 我们证明 $\\bar{K}_n$ 中有一个回路为三角形.\n不妨设有弧 $\\left(v, v^{\\prime}\\right)$, 并且从 $v^{\\prime}$ 到顶点 $v_1, v_2, \\cdots, v_k$ 各有一条弧, 其中 $k=d^{+}(v)$. 则必有一顶点 $v_j(1 \\leqslant j \\leqslant k)$, 从 $v_j$ 到 $v$ 有一条弧, 否则 $d^{+}(v) \\geqslant k+1>d^{+}\\left(v^{\\prime}\\right)$, 回路 $v, v^{\\prime}, v_j$ 就是一个三角形.\n于是充分性得证.\n若 $\\bar{K}_n$ 的各顶点的出度不同, 用数学归纳法证明 $\\bar{K}_n$ 不含三角形回路.\n当 $n=3$ 时, 易知顶点出度为 $0,1,2$ 的三角形不成回路.\n设命题在 $n=k$ 时正确.\n考察 $k+1$ 阶竞赛图 $\\bar{K}_{k+1}$, 若它的各个顶点出度不同, 那么它们依次为 $0,1,2, \\cdots, k$. 设 $d^{+}\\left(v^{\\prime}\\right)=k$, 去掉点 $v^{\\prime}$ 及相应的弧, 由归纳假设, $\\bar{K}_k-v^{\\prime}$ 不含三角形回路, 显然 $\\bar{K}_{k+1}$ 中也没有三角形回路, 必要性得证.", + "figures": [] +} \ No newline at end of file diff --git a/processed_dataset/text/0072.json b/processed_dataset/text/0072.json new file mode 100644 index 0000000000000000000000000000000000000000..830b3c88a31c2e32abf8c9e08a642b1f90db6345 --- /dev/null +++ b/processed_dataset/text/0072.json @@ -0,0 +1,5 @@ +{ + "source_file": "./raw_volume-zh/volume13/chapter1.tex", + "text": "小蓝本高中卷13组合极值不等式控制.\n组合极值的一个显著特点, 就是其约束条件或所求的极值的函数式较复杂.\n所谓不等式控制, 就是对约束条件或极值函数进行放缩, 使条件与极值函数之间的联系趋于明显.\n通过放缩, 使问题接近于一种标准形式: 在 $f(x, y)=$ 0 下,求 $u=g(x, y)$ 的最值, 从而将组合极值化归为一般的极值求解.\n不等式控制,通常有两种方式: 一是对约束条件进行放缩, 使隐蔽的约束条件明显化;二是对极值函数进行放缩, 使复杂的函数式简单化.", + "figures": [] +} \ No newline at end of file diff --git a/processed_dataset/text/0073.json b/processed_dataset/text/0073.json new file mode 100644 index 0000000000000000000000000000000000000000..9e90306ff8dcdb15b37f63e5ff04489897cd730e --- /dev/null +++ b/processed_dataset/text/0073.json @@ -0,0 +1,5 @@ +{ + "source_file": "./raw_volume-zh/volume13/chapter10.tex", + "text": "参数估计有些极值问题,因变动的因素较多,从表面上看情况相当复杂.\n但适当引人新的参数, 便可将极值函数用参数表出.\n这样, 离散极值问题被转化为一元或二元 (参数) 函数的极值问题, 从而使问题得到简化.\n我们称这种求极值的方法为参数估计.", + "figures": [] +} \ No newline at end of file diff --git a/processed_dataset/text/0074.json b/processed_dataset/text/0074.json new file mode 100644 index 0000000000000000000000000000000000000000..eb56f284899281310f84fe93439337ae859165ce --- /dev/null +++ b/processed_dataset/text/0074.json @@ -0,0 +1,5 @@ +{ + "source_file": "./raw_volume-zh/volume13/chapter11.tex", + "text": "估计的一种典型的方法是算两次, 它的基本模式是:\n对于集合 $X=\\left\\{a_1, a_2, \\cdots, a_n\\right\\}$, 设 $F=\\left\\{A_1, A_2, \\cdots, A_k\\right\\}$ 是 $X$ 的子集族 (常常是有交划分, 即可能有某两个子集的交非空), 其中第 $i$ 个子集 $A_i$ 的元素个数为 $r_i(i=1,2, \\cdots, k)$.\n用两种方法计算某种量 $\\Omega$ (称为\"中间量\") 的个数 $|\\Omega|$. 一方面,对整体 $X$ 计算, 常常是考虑每个元素对整体的贡献, 得 $|\\Omega|=f(n)$ (总个数). 另一方面, 从子集族 $F$ 计算, 由第 $i$ 子集 $A_i$ 得 $\\Omega$ 的个数为 $\\left|\\Omega_i\\right|=f_i(k, r)$,于是\n$$\n\\sum_{i=1}^n f_i(k, r)=\\sum_{i=1}^n\\left|\\Omega_i\\right| \\leqslant|\\Omega|=f(n) .\n$$\n这里要求,对不同的子集 $A_i 、 A_j$, 由它们得到的中间量 $\\Omega$ 是不同的.\n因此,常常要适当选取中间量 $\\Omega$, 以保证满足这一要求.\n如果对不同的子集 $A_i$ 、 $A_j$, 它们有公共的 $\\Omega$, 则要去掉重复计数.\n算两次的关键是\"算什么\". 对此,没有统一的模式,但中间量 $\\Omega$ 的选择常有如下一些方法:\n角:当估计的量与同色三角形有关时, 可计算同色角、异色角, 这是一种 \"减元\"策略.\n$r$ 子集:若各子集满足条件: $\\left|A_i \\cap A_j\\right| \\leqslant r$, 则计算 $r+1$ 元子集(我们称为 \"加元\" 策略). 此时, 对不同的子集 $A_i 、 A_j$, 它们的 $r+1$ 元子集是互异的.\n否则, $A_i 、 A_j$ 有公共的 $r+1$ 元子集, 则 $\\left|A_i \\cap A_j\\right| \\geqslant r+1$,与 $\\left|A_i \\cap A_j\\right| \\leqslant r$ 矛盾.\n对子:将具有特殊关系的 2 个元素配对 (并非任意的 2 元子集), 并称之为对子,计算这样的对子的个数.\n此时, 常常需要去掉重复计数.\n次数:比如, 某种元素出现的总次数, 参加某种活动的人次数.\n得分:某比赛选手所得的总分、各选手得分的总和.\n我们先看两个利用 \"算两次\" 技巧计数的例子, 因为计数常常是某些极值问题中的子问题.", + "figures": [] +} \ No newline at end of file diff --git a/processed_dataset/text/0075.json b/processed_dataset/text/0075.json new file mode 100644 index 0000000000000000000000000000000000000000..82bc7d8f3bc228e3baba6f67fdac189a1f8fa3ab --- /dev/null +++ b/processed_dataset/text/0075.json @@ -0,0 +1,5 @@ +{ + "source_file": "./raw_volume-zh/volume13/chapter12.tex", + "text": "缩小包围圈.\n先找一个合乎条件的\"大范围\",然后逐步缩小范围,使其范围仍然合乎条件,再思考范围在变化过程中为什么能继续合乎条件, 找到使范围合乎条件的本质因素, 由此使范围达到最佳估计; 或者分析确定范围的各种因素, 考察其中某些因素的\"功能\"(对范围的直接影响)是否可以优化或改进, 从而使范围的估计更精确.\n我们称这种估计方法为缩小包围圈.", + "figures": [] +} \ No newline at end of file diff --git a/processed_dataset/text/0076.json b/processed_dataset/text/0076.json new file mode 100644 index 0000000000000000000000000000000000000000..93a3be3ddebd1860c6f6a0ff3875d82bd67fc193 --- /dev/null +++ b/processed_dataset/text/0076.json @@ -0,0 +1,5 @@ +{ + "source_file": "./raw_volume-zh/volume13/chapter13.tex", + "text": "所谓考察特例, 是指考察问题包含的一些简单的特殊情形, 从中发现解题途径.\n它常包括如下 4 种情形:\n情形 1 考察\"最坏\"的特例——种最特殊的情况.\n情形 2 由充分条件、必要条件寻找特例.\n对此,一个找使性质 $P$ 成立的充分条件的方法是: 假设所求对象不满足要求 $P$, 由此导出若干性质, 然后设法破坏其中一个性质即可.\n情形 3 先考虑原问题在特殊情况下如何解决, 然后将一般情况变换到特殊情况处理.\n情形 4 由特殊情况发现一般规律, 猜想问题的结论, 最后用数学归纳法加以证明.", + "figures": [] +} \ No newline at end of file diff --git a/processed_dataset/text/0077.json b/processed_dataset/text/0077.json new file mode 100644 index 0000000000000000000000000000000000000000..742a28b1af650c486fc0228228471201aa8e7f63 --- /dev/null +++ b/processed_dataset/text/0077.json @@ -0,0 +1,5 @@ +{ + "source_file": "./raw_volume-zh/volume13/chapter2.tex", + "text": "累次极值.\n组合极值的一个特点是极值函数中变动的量较多, 难于发现函数的变化趋势.\n如果我们先冻结若干个变量, 即视若干个变量为常数, 则其函数的变化对剩下的变量的依赖关系就趋于明显, 由此可比较容易地求出第一次极值.\n然后\"解冻\"原来的变量, 进而求出函数的极值.\n冻结变量一般有两种方法: 一是冻结一个变量, 它通常用于求三元函数的极值: 对于三元函数 $f(x, y, z)$, 若固定变量 $z$, 则函数可看成是关于 $x 、 y$ 的二元函数.\n在此基础上求出二元函数的极值 $G(z)$, 再视 $z$ 为变量, 对 $G(z)$ 求极值.\n它的基本思路是:\n$$\nu=f(x, y, z)=g(x, y) \\leqslant G(z) \\leqslant C .\n$$\n但在有的情况下, $G(z)$ 的表达式是一种分段函数, 则上述思路又可表示为\n$$\n\\begin{aligned}\nu & =f(x, y, z)=g(x, y) \\leqslant G(z)=\\left\\{\\begin{array}{l}\nG_1(z),(z \\in A) \\\\\nG_2(z),(z \\in B)\n\\end{array}\\right. \\\\\n& \\leqslant\\left\\{\\begin{array}{l}\nA_1,(z \\in A) \\\\\nA_2,(z \\in B)\n\\end{array} \\Rightarrow u \\leqslant \\max \\left\\{A_1, A_2\\right\\} .\\right.\n\\end{aligned}\n$$\n特别地, 如果 $g(x, y) \\leqslant G(z) \\leqslant C$ 中的等式不同时成立, 则固定 $z$ 的取值时,须分类处理(单独讨论 $z$ 的若干特殊取值). 其基本思路为:\n$$\n\\begin{gathered}\nu=f(x, y, z)=\\left\\{\\begin{array}{cc}\ng_1(x, y) & \\left(z=z_1\\right) \\\\\n\\cdots & \\cdots \\\\\ng_k(x, y) & \\left(z=z_k\\right) \\\\\ng(x, y) & (z \\in A)\n\\end{array} \\leqslant\\left\\{\\begin{array}{ccc}\nG_1 & \\left(z=z_1\\right) \\\\\n\\cdots & \\cdots \\\\\nG_k & \\left(z=z_k\\right) \\\\\nG(z) \\leqslant G_A & (z \\in A)\n\\end{array}\\right.\\right. \\\\\n\\Rightarrow u \\leqslant G, \\text { 其中 } G=\\max \\left\\{G_1, G_2, \\cdots, G_k, G_A\\right\\} .\n\\end{gathered}\n$$\n二是冻结多个变量, 它通常用于求多 (超过 3) 元函数的极值: 在多元函数的解析式中, 选择其中一个字母为主变元, 冻结其他的所有变元, 则函数变为一完函数 $f(t)$. 至此, 先求出 $f(t)$ 的极值点 $f\\left(t_0\\right)$, 再对 $f\\left(t_0\\right)$ 冻结变元 (因为 $f\\left(t_0\\right)$ 是关于其他变元的函数), 又化为一元函数求解.\n如此下去, 直至求出函数的极值.\n从实质上看, 累次极值就是放缩法, 只是放缩方式是采用固定变量逐步消元.\n我们先看一个求一般函数的极值的例子.", + "figures": [] +} \ No newline at end of file diff --git a/processed_dataset/text/0078.json b/processed_dataset/text/0078.json new file mode 100644 index 0000000000000000000000000000000000000000..08a389dd22a73889d4c9bed7c731fb967ae1165a --- /dev/null +++ b/processed_dataset/text/0078.json @@ -0,0 +1,5 @@ +{ + "source_file": "./raw_volume-zh/volume13/chapter3.tex", + "text": "这种方法, 是先证明所求的极值存在, 然后由问题的直观性, 猜想出极值点.\n最后从反面证明函数在其他点不能达到极值: 假设函数在另外的点 $\\left(x_1\\right.$, $x_2, \\cdots, x_n$ ) 处达到极值, 经过适当调整(常常是将小的分量变大, 大的分量变小), 发现函数在 $\\left(x_1^{\\prime}, x_2^{\\prime}, \\cdots, x_n^{\\prime}\\right)$ 处的值更大或更小, 从而断定它不是极值点.\n它的基本步骤是:\n证明极值存在一一猜出极值点一一证明其他点非极值点一一得出结论.", + "figures": [] +} \ No newline at end of file diff --git a/processed_dataset/text/0079.json b/processed_dataset/text/0079.json new file mode 100644 index 0000000000000000000000000000000000000000..cb2624436fe17a2fce94d068b39d2edb540a037d --- /dev/null +++ b/processed_dataset/text/0079.json @@ -0,0 +1,5 @@ +{ + "source_file": "./raw_volume-zh/volume13/chapter4.tex", + "text": "对称处理.\n这种方法适应于求对称多项式型函数的极值.\n它的基本思想是, 先证明函数必定存在极大值或极小值.\n然后固定若干变元, 保留少数几个变元, 讨论函数关于这少数几个变元的极值点所具有的性质.\n再由对称性, 得出函数关于其他变元的极值点具有同样的性质, 进而确定极值点, 求出极值.\n基本步骤是: 证明最值存在一一尽可能多地固定变量, 化为一元或二元函数求出最值点一一利用对称性发现多元函数的最值点一一求出最值.", + "figures": [] +} \ No newline at end of file diff --git a/processed_dataset/text/0080.json b/processed_dataset/text/0080.json new file mode 100644 index 0000000000000000000000000000000000000000..6b7f8f9fc1f21aae86bcabbf00a6730c931ab248 --- /dev/null +++ b/processed_dataset/text/0080.json @@ -0,0 +1,5 @@ +{ + "source_file": "./raw_volume-zh/volume13/chapter5.tex", + "text": "磨光变换.\n有些函数的极值, 虽然不能证明其必定存在极值, 但由问题的直观, 可以发现其极值点.\n此时,我们可以对其施行一种变换: 先将变量组中的某个分量调整到极值点, 而将此分量与极值点相应分量的差转移到另外的分量中去, 进而验证这一变换保持函数值单调递增或递减.\n反复施行这一变换(必须论证变换有限次后终止), 直至变量组中的每一个分量都调整到了极值点, 得到函数的极值.\n这种变换称为磨光变换.\n磨光变换常采用如下一些变换方式:\n(1)对\"搭配型\"最值点, 比如取值最小的变量与取值最大的变量搭配, 可先将相搭配的一对变量调整到最值点,然后再调整其他变量.\n(2) 对 \"均匀型\"最值点, 比如各个变量取值相同, 可先将取值最小的变量调整到最值点, 不足部分在取值较大的变量中补足.\n(3)对 \"聚积型\"最值点, 比如一个变量取值最大, 其他变量取值都很小, 可先将取值最小的变量调整到最小点, 再调整其他变量到最值点.\n从实质上看, 磨光变换就是放缩法, 只是放缩形式采用磨光手段.", + "figures": [] +} \ No newline at end of file diff --git a/processed_dataset/text/0081.json b/processed_dataset/text/0081.json new file mode 100644 index 0000000000000000000000000000000000000000..31273ccd92bdb21aece9991ce3c9cb15be816b81 --- /dev/null +++ b/processed_dataset/text/0081.json @@ -0,0 +1,5 @@ +{ + "source_file": "./raw_volume-zh/volume13/chapter6.tex", + "text": "间距估计.\n考虑这样的问题: 设 $X$ 是给定的集合, $A$ 是 $X$ 的具有某种性质的子集, 求 $|A|$ 的最大值.\n对此, 可将集合中元素适当排序, 然后估计相邻元素间的距离, 由此得到元素个数的估计.\n这种估计方法简称间距估计.", + "figures": [] +} \ No newline at end of file diff --git a/processed_dataset/text/0082.json b/processed_dataset/text/0082.json new file mode 100644 index 0000000000000000000000000000000000000000..fdf10f1c94fd0f6bf916729fe6383375d02a097b --- /dev/null +++ b/processed_dataset/text/0082.json @@ -0,0 +1,5 @@ +{ + "source_file": "./raw_volume-zh/volume13/chapter7.tex", + "text": "划块估计.\n为了估计集合 $X$ 的具有某种性质的子集 $A$ 中含有元素的个数, 可将 $X$ 划分为若干块 $X_1, X_2, \\cdots, X_t$. 然后讨论每个 $X_i$ 至多(或至少)含有 $A$ 的多少个元素, 由此得到 $|A|$ 的范围估计.\n常有如下 3 种基本情形:\n情形 1 若 $X$ 的子集 $A$ 满足: $A$ 中任何 $r$ 元组都具有性质 $p$, 求 $|A|$ 的最大值.\n则可将 $X$ 划分为若干块 $X_1, X_2, \\cdots, X_t$, 使 $X_i$ 中任何 $r$ 元组都不具有性质 $p$, 从陑每个 $X_i$ 至多含有 $A$ 的 $r-1$ 个元素.\n情形 2 设 $X=X_1 \\cup X_2 \\cup \\cdots \\cup X_t$, 且每个 $X_i$ 至多含有 $A$ 的 $k_i$ 个元素, 从而 $|A| \\leqslant k_1+k_2+\\cdots+k_t$. 显然, $k_1+k_2+\\cdots+k_t$ 越小, 估计越精确(等号越有可能达到). 因此, 在 $X$ 的划分中, 应使 $k_1+k_2+\\cdots+k_t$ 尽可能小, 这就要使 $k_i$ 在 $A_i$ 中占的\"比重\": $\\frac{k_i}{\\left|A_i\\right|}$ 较小.\n这常常可通过列表试验, 找到估计较为精确的划分.\n情形 3 有些数集具有这样的性质: 只要集合 $A=\\left\\{a_1, a_2, \\cdots, a_n\\right\\}$ 具有性质 $p$, 则集合 $A+a=\\left\\{a_1+a, a_2+a, \\cdots, a_n+a\\right\\}$ 也具有性质 $p$, 我们称集合 $A$ 的这种性质 $p$ 具有平移不变性.\n此时, 可对 $X$ 进行均匀 (各块的元素个数相等)的划分,然后分块进行估计.", + "figures": [] +} \ No newline at end of file diff --git a/processed_dataset/text/0083.json b/processed_dataset/text/0083.json new file mode 100644 index 0000000000000000000000000000000000000000..564ab9e060f6e7c2b1c6adccc900778f6e304215 --- /dev/null +++ b/processed_dataset/text/0083.json @@ -0,0 +1,5 @@ +{ + "source_file": "./raw_volume-zh/volume13/chapter8.tex", + "text": "猜想与反证.\n有些具有某种性质 $p$ 的集合是很容易构造的, 常常只要把一类具有某种性质的元素构成一个集合 (类聚法) 即可.\n这时, 可先直接构造具有某种性质 $p$ 的集合,然后猜想得到的集合是 \"最大\"的.\n证明猜想的一种有效的方法是反面估计.\n假定集合 $A$ 是具有某种性质 $p$ 的集合,我们期望证明 $|A| \\leqslant r$. 反设有 $|A|>r$,则 $A$ 中必存在某些特殊元素破坏集合 $A$ 的性质 $p$. 对此,抽屉原理是常用的工具.", + "figures": [] +} \ No newline at end of file diff --git a/processed_dataset/text/0084.json b/processed_dataset/text/0084.json new file mode 100644 index 0000000000000000000000000000000000000000..4a52454a4413b622c7a10639f1ec1185f2728840 --- /dev/null +++ b/processed_dataset/text/0084.json @@ -0,0 +1,5 @@ +{ + "source_file": "./raw_volume-zh/volume13/chapter9.tex", + "text": "整体估计.\n为了估计某个变量的变化范围, 可将其放在若干个变量构成的整体中一起考虑, 从整体上估计它们的取值范围, 进而得到某变量的取值范围.\n这种估计方法称为整体估计.\n整体估计的一种特殊情况是估计平均数: 设 $A_1, A_2, \\cdots, A_n$ 的平均数为 $A$, 则 $A_1, A_2, \\cdots, A_n$ 中至少有一个 $A_i$ 不小于 $A$, 也至少有 - 个 $A_j$ 不大于 $A$.\n平均数估计中的一个重要的工具是\"集合元素关系表\": 设 $X=\\left\\{a_1, a_2\\right.$, $\\left.\\cdots, a_n\\right\\}, A_1, A_2, \\cdots, A_k$ 是 $X$ 的子集.\n所谓\"集合元素关系表\", 是指由 $n$ 行 $k$ 列数构成的如下数表:\n其中 $a_i \\in A_j$ 时, $x_{i j}=1$, 否则, $x_{i j}=0$. 这样,第 $i$ 行中 1 的个数就是元素 $a_i$ 在各子集中出现的次数, 称为 $a_i$ 的度, 记作 $d\\left(a_i\\right)$ 或 $m_i$, 即 $m_i=\\sum_{j=1}^k x_{i j}$.\n第 $j$ 列中 1 的个数就是集合 $A_j$ 中的元素的个数, 即 $\\left|A_j\\right|=\\sum_{i=1}^n x_{i j}$.\n在集合元素关系表中,有两个常用的关系式:\n(1) 考察各元素在 $F$ 中出现的总次数,即表中 1 的个数 $S$, 有\n$$\n\\sum_{i=1}^n m_i=S=\\sum_{j=1}^k\\left|A_j\\right|\n$$\n(2)考察各元素在集合对的交集中出现的总次数 $T$,有\n$$\n\\sum_{i=1}^n \\mathrm{C}_{m_i}^2=T=\\sum_{1 \\leqslant i0)$ 个单位后就得到函数 $y=f(x+h)$ (或 $y=f(x-h)$ ) 的图象; 将函数 $y=f(x)$ 的图象向上 (或向下) 平移 $k(k>0)$ 个单位后就得到函数 $y=f(x)+k$ (或 $y=f(x)-k)$ 的图象.\n(2) 伸缩变换将函数 $y=f(x)$ 的图象上所有点的横坐标变到原来的 $\\frac{1}{\\omega}$ 倍就得到函数 $y=f(\\omega x)$ 的图象; 将函数 $y=f(x)$ 的图象上所有点的纵坐标变到原来的 $\\frac{1}{\\omega}$ 倍就得到函数 $y=\\frac{f(x)}{\\omega}$. 的图象.\n(3) 对称变换函数 $y=f(x)$ 的图象与函数 $y=-f(x), y=f(-x) ,y=-f(-x)$ 的图象分别关于 $x$ 轴、 $y$ 轴、原点对称.\n(4) 翻转变换将函数 $y=f(x)$ 的图象在 $x$ 轴上方的部分不变, $x$ 轴下方的部分翻转到 $x$ 轴的上方就得到函数 $y=|f(x)|$ 的图象.", + "figures": [] +} \ No newline at end of file diff --git a/processed_dataset/text/0108.json b/processed_dataset/text/0108.json new file mode 100644 index 0000000000000000000000000000000000000000..0c59befbbb65d55f30b4bd842c6a91cc505753b1 --- /dev/null +++ b/processed_dataset/text/0108.json @@ -0,0 +1,5 @@ +{ + "source_file": "./raw_volume-zh/volume2/chapter2.tex", + "text": "函数的奇偶性、单调性、周期性是函数的最基本的性质, 它们反映了函数的重要的代数特征.\n2.1 奇偶性定义 2.1 设函数 $f(x)$ 的定义域为 $D$, 且 $D$ 是关于原点对称的数集.\n若对于任意的 $x \\in D$, 都有\n$$\nf(-x)=-f(x),\n$$\n则称 $f(x)$ 是奇函数.\n若对于任意的 $x \\in D$, 都有\n$$\nf(-x)=f(x),\n$$\n则称 $f(x)$ 是偶函数.", + "figures": [] +} \ No newline at end of file diff --git a/processed_dataset/text/0109.json b/processed_dataset/text/0109.json new file mode 100644 index 0000000000000000000000000000000000000000..68a2c1dc67ba208c5fe587fbccb218170b4562f3 --- /dev/null +++ b/processed_dataset/text/0109.json @@ -0,0 +1,8 @@ +{ + "source_file": "./raw_volume-zh/volume2/chapter2.tex", + "text": "2.2 单调性定义 2.2 设函数 $f(x)$ 的定义域为 $I$. 如果对于任意的 $x_1, x_2 \\in I$, 当$x_1).\n函数的定义域为 $I$, 如果对于任意的 $x_1, x_2 \\in I$, 当 $x_1f\\left(x_2\\right)$, 那么就说 $y=f(x)$ 在此区间上是减函数,如图().\n如果函数 $y=f(x)$ 在某区间上是增函数或减函数,那么就说 $y=f(x)$ 在此区间上有 (严格的) 单调性,这一区间叫做 $y=f(x)$ 的单调区间.\n对于函数的单调性, 我们应该注意以下儿点:\n(1)函数的单调性是函数的一个重要性质, 但讨论函数的单调性必须在定义域内进行, 即函数的单调区间是定义域的子区间.\n(2)函数的单调性是对某一区间而言的,要指明函数的单调性,必须指明是在其定义域的哪一个子区间上,有的函数在其整个定义域上都是增函数, 有的函数在其整个定义域上都是减函数; 而有的函数在定义域的一些子区间上是增函数,在另一些子区间上是减函数.\n(3)某个函数在一个区间上是增(减)函数,在另一区间上也是增(减) 函数,绝不能说它在这两个区间的并集上也是增(减) 函数.\n例如, $y=\\frac{1}{x}$ 在 $(0,+\\infty)$ 上是减函数, 在 $(-\\infty, 0)$ 上也是减函数, 它有两个减区间, 但绝不能说 $y=\\frac{1}{x}$ 在 $(-\\infty, 0) \\cup(0,+\\infty)$ 上也是减函数.\n(4)函数的单调性反映在其图象上是指函数图象的走势.\n在单调区间上, 增函数的图象是上升的, 减函数的图象是下降的.\n(5)中学数学教材中所指的单调性是严格单调的, 即必须是 $f\\left(x_1\\right)< f\\left(x_2\\right)$ 或 $f\\left(x_1\\right)>f\\left(x_2\\right)$, 绝不能是 $f\\left(x_1\\right) \\leqslant f\\left(x_2\\right)$ 或 $f\\left(x_1\\right) \\geqslant f\\left(x_2\\right)$.\n关于函数的单调性, 有如下性质:\n(1) 若函数 $y=f(x)$ 和 $y=g(x)$ 在公共区间 $A$ 上都是增(减) 函数, 则函数 $y=f(x)+g(x)$ 在 $A$ 上也是增(减) 函数.\n(2) 若两个正值函数 $y=f(x)$ 和 $y=g(x)$ 在公共区间 $A$ 上都是增(减)函数,则函数 $y=f(x) g(x)$ 在 $A$ 上也是增(减) 函数;\n若两个负值函数 $y=f(x)$ 和 $y=g(x)$ 在公共区间 $A$ 上都是增(减) 函数, 则函数 $y=f(x) g(x)$ 在 $A$ 上是减 (增) 函数.\n(3) 若函数 $y=f(x)$ 是区间 $A$ 上的增(减) 函数, 值域为 $C$, 则其反函数 $y=f^{-1}(x)$ 是 $C$ 上的增 (减) 函数.\n(4) 若函数 $y=f(u)$ 和 $u=g(x)$ 在相关区间上是单调函数, 则函数 $y= f(g(x))$ 在此区间上也是单调函数; 并且若 $y=f(u)$ 和 $u=g(x)$ 的单调性相同(相反), 则 $y=f(g(x))$ 是增(减) 函数.", + "figures": [ + "./images/volume2/figures/fig-c2d2-1.png", + "./images/volume2/figures/fig-c2d2-2.png" + ] +} \ No newline at end of file diff --git a/processed_dataset/text/0110.json b/processed_dataset/text/0110.json new file mode 100644 index 0000000000000000000000000000000000000000..34c9b4cf21c6d548ef0b717786dbf807444a5eaa --- /dev/null +++ b/processed_dataset/text/0110.json @@ -0,0 +1,5 @@ +{ + "source_file": "./raw_volume-zh/volume2/chapter2.tex", + "text": "2.3 周期性定义 2.3 设函数 $f(x)$ 的定义域为 $D$, 如果存在一个常数 $T \\neq 0$, 使得对每个 $x \\in D$, 都有\n$$\nf(x+T)=f(x),\n$$\n那么称 $f(x)$ 是周期函数, $T$ 为 $f(x)$ 的一个周期.\n如果 $f(x)$ 的所有正周期中存在最小值 $T_0$, 那么称 $T_0$ 为周期函数 $f(x)$ 的最小正周期.\n一般说函数的周期都是指最小正周期.\n函数 $f(x)=\\sin x, f(x)=\\cos x$ 的周期为 $2 \\pi$, 函数 $f(x)=\\tan x$, $f(x)=\\cot x$ 的周期为 $\\pi$. 函数 $f(x)=x-[x], x \\in(-\\infty,+\\infty)$ 的周期为 1. 常量函数 $f(x)=c$ 是以任何正数为周期的周期函数, 但不存在最小正周期.", + "figures": [] +} \ No newline at end of file diff --git a/processed_dataset/text/0111.json b/processed_dataset/text/0111.json new file mode 100644 index 0000000000000000000000000000000000000000..0b8d2f2d879261f591b91c92b0eeb6d2bf08e9aa --- /dev/null +++ b/processed_dataset/text/0111.json @@ -0,0 +1,5 @@ +{ + "source_file": "./raw_volume-zh/volume2/chapter3.tex", + "text": "几个常见的初等函数相这一节我们来讨论几个常见的初等函数的性质及其解题方法.\n3.1 二次函数二次函数在中学数学中占有重要地位.\n它形式简单, 应用极其广泛.\n$f(x)=a x^2+b x+c(a \\neq 0)$ 称为二次函数.\n也常常写成:\n$$\nf(x)=a(\\dot{x}-k)^2+m(a \\neq 0) \\text { (顶点式) }\n$$\n或\n$$\nf(x)=a\\left(x-x_1\\right)\\left(x-x_2\\right)(a \\neq 0) \\text { (零点式). }\n$$\n二次函数 $f(x)=a x^2+b x+c(a \\neq 0)$ 的性质.\n(1) 对称性对任意实数 $x$, 有\n$$\nf\\left(-\\frac{b}{2 a}+x\\right)=f\\left(-\\frac{b}{2 a}-x\\right) .\n$$\n(2) $f(0)=c$.\n(3) 若 $\\Delta=b^2-4 a c \\geqslant 0$, 则 $f\\left(\\frac{-b \\pm \\sqrt{\\Delta}}{2 a}\\right)=0$.\n(4) 当 $a>0$ 时, $f(x)$ 在区间 $\\left(-\\infty,-\\frac{b}{2 a}\\right]$ 上递减,在区间 $\\left[-\\frac{b}{2 a},+\\infty\\right)$上递增;\n当 $a<0$ 时, $f(x)$ 在区间 $\\left(-\\infty,-\\frac{b}{2 a}\\right]$ 上递增, 在区间 $\\left[-\\frac{b}{2 a},+\\infty\\right)$ 上递减.\n(5) 当 $a>0$ 时, $f(x)$ 有最小值 $f_{\\text {min }}(x)=f\\left(-\\frac{b}{2 a}\\right)=\\frac{4 a c-b^2}{4 a}$;\n当 $a<0$ 时, $f(x)$ 有最大值 $f_{\\text {max }}(x)=f\\left(-\\frac{b}{2 a}\\right)=\\frac{4 a c-b^2}{4 a}$.\n二次函数 $f(x)=a x^2+b x+c(a \\neq 0)$ 的图象.\n(1) 对称轴 $f(x)$ 关于直线 $x=-\\frac{b}{2 a}$ 对称.\n(2) 顶点对称轴与抛物线的交点 $\\left(-\\frac{b}{2 a}, \\frac{4 a c-b^2}{4 a}\\right)$ 称为顶点.\n当 $a>0$ 时, 顶点是抛物线的最低点; 当 $a<0$ 时, 顶点是抛物线的最高点.\n(3) 开口当 $a>0$ 时,开口向上; 当 $a<0$ 时,开口向下.\n开口大小由 $|a|$ 决定.\n(4) 二次函数图象与 $x$ 轴的位置关系当 $\\Delta>0$ 时, 二次函数图象与 $x$ 轴有两个不同的交点; 当 $\\Delta=0$ 时,二次函数图象与 $x$ 轴相切; 当 $\\Delta<0$ 时, 二次函数与 $x$ 轴无交点.", + "figures": [] +} \ No newline at end of file diff --git a/processed_dataset/text/0112.json b/processed_dataset/text/0112.json new file mode 100644 index 0000000000000000000000000000000000000000..a11afcc66d628693e34a2737ea064727e14cf04c --- /dev/null +++ b/processed_dataset/text/0112.json @@ -0,0 +1,5 @@ +{ + "source_file": "./raw_volume-zh/volume2/chapter3.tex", + "text": "3.2 幕函数、指数函数和对数函数形如 $y=x^\\alpha$ 的函数叫做幕函数,其中 $x$ 是自变量, $\\alpha$ 是常数,在中学阶段, 我们只研究 $\\alpha \\in \\mathbf{Q}$ 的情况.\n幂函数的图象是不通过第四象限的一条曲线.\n当 $\\alpha>0$ 时, 图象都通过 $(0,0),(1,1)$, 在 $(0,+\\infty)$ 上是增函数; 当 $\\alpha<0$ 时, 图象都通过点 $(1,1)$, 在 $(0,+\\infty)$ 上是减函数.\n形如 $y=a^x(a>0, a \\neq 1)$ 的函数叫做指数函数, 其定义域为 $\\mathbf{R}$, 值域为 $(0,+\\infty)$. 当 $01$ 时, $y=a^x$ 是增函数.\n形如 $y=\\log _a x(a>0, a \\neq 1)$ 的函数叫做对数函数, 其定义域为 $(0 , +\\infty)$, 值域为 $\\mathbf{R}$. 当 $01$ 时, $y=\\log _a x$ 是增函数.", + "figures": [] +} \ No newline at end of file diff --git a/processed_dataset/text/0113.json b/processed_dataset/text/0113.json new file mode 100644 index 0000000000000000000000000000000000000000..63d1bd68a0c886c158ae3c18330719b0cf383ff3 --- /dev/null +++ b/processed_dataset/text/0113.json @@ -0,0 +1,7 @@ +{ + "source_file": "./raw_volume-zh/volume2/chapter3.tex", + "text": "3.3 函数 $f(x)=x+\\frac{a^2}{x}$\n函数 $f(x)=x+\\frac{a^2}{x}(a>0)$ 具有很广泛的应用,下面我们来讨论它的一些性质.\n$f(x)=x+\\frac{a^2}{x}$ 的定义域为 $(-\\infty, 0) \\cup(0,+\\infty)$.\n当 $x>0$ 时, $f(x)=x+\\frac{a^2}{x} \\geqslant 2 a$;\n当 $x=a$ 时,等号成立;\n当 $x<0$ 时, $f(x)=-\\left((-x)+\\frac{a^2}{(-x)}\\right) \\leqslant-2 a$;\n当 $x=-a$ 时等号成立.\n所以, $f(x)=x+\\frac{a^2}{x}$ 的值域为 $(-\\infty,-2 a] \\cup[2 a,+\\infty)$.\n易知 $f(x)=x+\\frac{a^2}{x}$ 是奇函数.\n设 $x_1, x_2 \\in(0,+\\infty)$, 则\n$$\n\\begin{aligned}\nf\\left(x_2\\right)-f\\left(x_1\\right) & =\\left(x_2-x_1\\right)+a^2\\left(\\frac{1}{x_2}-\\frac{1}{x_1}\\right) \\\\\n& =\\left(x_2-x_1\\right)\\left(1-\\frac{a^2}{x_1 x_2}\\right) .\n\\end{aligned}\n$$\n当 $00$.\n所以, $f(x)=x+\\frac{a^2}{x}(a>0)$ 在 $(0, a]$ 上是单调递减的, 在 $[a,+\\infty)$ 上是单调递增的.\n用同样的方法可知, $f(x)=x+\\frac{a^2}{x}(a>0)$ 在 $(-\\infty,-a]$ 上是单调递增的, 在 $[-a, 0)$ 上是单调递减的.\n函数 $f(x)=x+\\frac{a^2}{x}(a>0)$ 的图象如图 () 所示.", + "figures": [ + "./images/volume2/figures/fig-c3d3.png" + ] +} \ No newline at end of file diff --git a/processed_dataset/text/0114.json b/processed_dataset/text/0114.json new file mode 100644 index 0000000000000000000000000000000000000000..24c272ec4fe6f3c68534077ba960696bf5356ecd --- /dev/null +++ b/processed_dataset/text/0114.json @@ -0,0 +1,5 @@ +{ + "source_file": "./raw_volume-zh/volume2/chapter4.tex", + "text": "函数的最大值与最小值我们经常会遇到各种各样的求最大值和最小值问题,这类问题在许多情况下可以归结为求函数的最大值和最小值.\n定义设函数 $f(x)$ 的定义域为 $D$. 若存在 $x_0 \\in D$, 使得对任意 $x \\in D$, 都有\n$$\nf(x) \\leqslant f\\left(x_0\\right),\n$$\n则称 $f\\left(x_0\\right)$ 为函数 $f(x)$ 在 $D$ 上的最大值, 简记为 $f_{\\text {max }}$. 若存在 $y_0 \\in D$, 使得对任意 $x \\in D$, 都有\n$$\nf(x) \\geqslant f\\left(y_0\\right),\n$$\n则称 $f\\left(y_0\\right)$ 为函数 $f(x)$ 在 $D$ 上的最小值, 简记为 $f_{\\min }$.\n求函数的最大值和最小值问题涉及的知识面较广, 解法也灵活多变, 需要我们有很好的综合能力.\n常用的方法有:\n(1)配方法;(2)判别式法;(3)不等式法;(4)换元法;(5)构造法;(6)利用函数性质.\n下面我们分别来介绍这些方法的应用.\n4. 1 配方法利用平方数恒大于或等于 0 , 将所给的函数配成若干个平方及一些常数的代数和的形式, 然后再求其最值就容易了.", + "figures": [] +} \ No newline at end of file diff --git a/processed_dataset/text/0115.json b/processed_dataset/text/0115.json new file mode 100644 index 0000000000000000000000000000000000000000..fde69bc9b61d32a6e17c70cb0169ad0399c11d4f --- /dev/null +++ b/processed_dataset/text/0115.json @@ -0,0 +1,5 @@ +{ + "source_file": "./raw_volume-zh/volume2/chapter4.tex", + "text": "4.2 判别式法利用实系数一元二次方程有实根, 则它的判别式 $\\Delta \\geqslant 0$, 从而可以确定系数中参数的范围, 进而求得最值.\n特别地, 对于分式函数 $y=\\frac{a_1 x^2+b_1 x+c_1}{a_2 x^2+b_2 x+c_2}$ 的最大值与最小值问题, 常用的方法是去分母后, 化为关于 $x$ 的二次方程, 然后用判别式 $\\Delta \\geqslant 0$, 得出 $y$ 的取值范围, 进而确定出 $y$ 的最大值和最小值.", + "figures": [] +} \ No newline at end of file diff --git a/processed_dataset/text/0116.json b/processed_dataset/text/0116.json new file mode 100644 index 0000000000000000000000000000000000000000..eac5c6bc59cc52c5680769001db9d97c3a2f1434 --- /dev/null +++ b/processed_dataset/text/0116.json @@ -0,0 +1,5 @@ +{ + "source_file": "./raw_volume-zh/volume2/chapter4.tex", + "text": "4.3 不等式法不等式与函数的最值问题有着密切的联系, 利用不等式取等号, 就可得到一个最值问题的解, 而许多不等式又可解释为最值问题的解.", + "figures": [] +} \ No newline at end of file diff --git a/processed_dataset/text/0117.json b/processed_dataset/text/0117.json new file mode 100644 index 0000000000000000000000000000000000000000..a260fdbdc6f0bfa5b6f7de5c84c6ac456384854d --- /dev/null +++ b/processed_dataset/text/0117.json @@ -0,0 +1,5 @@ +{ + "source_file": "./raw_volume-zh/volume2/chapter4.tex", + "text": "4.4 换元法通过换元, 把复杂的目标函数变形为较简单的函数形式, 或将不易求得最值的函数形式化为易求得最值的形式, 从而使问题得到解决.", + "figures": [] +} \ No newline at end of file diff --git a/processed_dataset/text/0118.json b/processed_dataset/text/0118.json new file mode 100644 index 0000000000000000000000000000000000000000..46b17e5ea8a1e782c4e3971c4c4a9c6c727f5456 --- /dev/null +++ b/processed_dataset/text/0118.json @@ -0,0 +1,5 @@ +{ + "source_file": "./raw_volume-zh/volume2/chapter4.tex", + "text": "4.5 构造法根据欲求最值的函数的特征, 构造反映函数关系的几何图形, 然后借助于图形可较容易地求得最大值和最小值.", + "figures": [] +} \ No newline at end of file diff --git a/processed_dataset/text/0119.json b/processed_dataset/text/0119.json new file mode 100644 index 0000000000000000000000000000000000000000..1cd8e2fba053f4a23ff1cd12f30b4b8faf927454 --- /dev/null +++ b/processed_dataset/text/0119.json @@ -0,0 +1,5 @@ +{ + "source_file": "./raw_volume-zh/volume2/chapter4.tex", + "text": "4.6 利用函数性质若函数 $f(x)$ 在 $[a, b]$ 上是增函数, 则 $f(x)$ 在 $[a, b]$ 上的最大值为 $f(b)$, 最小值为 $f(a)$; 若函数 $f(x)$ 在 $[a, b]$ 上是减函数, 则 $f(x)$ 在 $[a, b]$ 上的最大值为 $f(a)$, 最小值为 $f(b)$.\n若函数 $f(x)$ 满足: 当 $x \\leqslant x_0$ 时, $f(x)$ 是增函数, 当 $x \\geqslant x_0$ 时, $f(x)$ 是减函数, 则 $f\\left(x_0\\right)$ 是 $f(x)$ 的最大值;\n若函数 $f(x)$ 满足: 当 $x \\leqslant x_0$ 时, $f(x)$ 是减函数, 当 $x \\geqslant x_0$ 时, $f(x)$ 是增函数, 则 $f\\left(x_0\\right)$ 是 $f(x)$ 的最小值.", + "figures": [] +} \ No newline at end of file diff --git a/processed_dataset/text/0120.json b/processed_dataset/text/0120.json new file mode 100644 index 0000000000000000000000000000000000000000..0b635b330232d1dc790266f958b0dd55b096844d --- /dev/null +++ b/processed_dataset/text/0120.json @@ -0,0 +1,5 @@ +{ + "source_file": "./raw_volume-zh/volume2/chapter5.tex", + "text": "构造函数解题我们在处理某些方程、不等式、最值问题及一些组合问题时, 常常构造一个函数,然后利用函数的图象和性质来解决问题.\n5.1 构造函数证明不等式", + "figures": [] +} \ No newline at end of file diff --git a/processed_dataset/text/0121.json b/processed_dataset/text/0121.json new file mode 100644 index 0000000000000000000000000000000000000000..3a80a75e3d28aceefa47805bc525bdc66683a4e4 --- /dev/null +++ b/processed_dataset/text/0121.json @@ -0,0 +1,5 @@ +{ + "source_file": "./raw_volume-zh/volume2/chapter6.tex", + "text": "函数的选代\n6.1 函数迭代的定义我们利用了一个函数自身复合多次.\n这便是函数的迭代.\n定义 6.1 设 $f: D \\mapsto D$ 是一个函数, 对任意 $x \\in D$, 记\n$$\n\\begin{aligned}\n& f^{(0)}(x)=x, \\\\\n& f^{(1)}(x)=f(x), \\\\\n& f^{(2)}(x)=f(f(x)), \\\\\n& f^{(3)}(x)=f(f(f(x))),\n\\end{aligned}\n$$\n$$\nf^{(n+1)}(x)=f\\left(f^{(n)}(x)\\right),\n$$\n则称 $f^{(n)}(x)$ 是函数 $f(x)$ 在 $D$ 上的 $n$ 次迭代, 并称 $n$ 是 $f^{(n)}(x)$ 的迭代指数.\n如果 $f^{(n)}(x)$ 有反函数, 则记为 $f^{(-n)}(x)$, 于是, 迭代指数可取所有整数.\n求一个函数的 $n$ 次迭代, 是数学竞赛中的一种基本题型.\n对于一些简单的函数, 它的 $n$ 次迭代是容易得到的.\n若 $f(x)=x+c$, 则 $f^{(n)}(x)=x+n c, f^{-1}(x)=x \\div c, f^{(\\rightarrow n)}(x)=x-n c$.\n若 $f(x)=x^3$, 则 $f^{(n)}(x)=x^{3^n}, f^{(-1)}(x)=x^{\\frac{1}{3}}, f^{(-n)}(x)=x^{\\frac{1}{3^n}}$.\n若 $f(x)=a x+b$, 则 $f^{(n)}(x)=a^n\\left(x-\\frac{b}{1-a}\\right)+\\frac{b}{1-a}, f^{(-1)}(x)= \\frac{1}{a}\\left(x-\\frac{b}{1-a}\\right)+\\frac{b}{1-a}, f^{(-n)}(x)=\\frac{1}{a^n}\\left(x-\\frac{b}{1-a}\\right)+\\frac{b}{1-a}$.", + "figures": [] +} \ No newline at end of file diff --git a/processed_dataset/text/0122.json b/processed_dataset/text/0122.json new file mode 100644 index 0000000000000000000000000000000000000000..38aeea2de147bb652298e570b9d4deb80acf5eae --- /dev/null +++ b/processed_dataset/text/0122.json @@ -0,0 +1,5 @@ +{ + "source_file": "./raw_volume-zh/volume2/chapter6.tex", + "text": "6.2 $f^{(n)}(x)$ 的求法\n(1)数学归纳法这里用到的是先猜后证的想法, 即先对函数 $f(x)$ 迭代几次, 观察出其规律, 然后猜测出 $f^{(n)}(x)$ 的表达式, 最后用数学归纳法证之.\n这种方法只适用于一些较为简单的函数.\n面看一些例子.", + "figures": [] +} \ No newline at end of file diff --git a/processed_dataset/text/0123.json b/processed_dataset/text/0123.json new file mode 100644 index 0000000000000000000000000000000000000000..5e447d6b38672e0e95ee2287279a9957335687a6 --- /dev/null +++ b/processed_dataset/text/0123.json @@ -0,0 +1,5 @@ +{ + "source_file": "./raw_volume-zh/volume2/chapter6.tex", + "text": "(2) 递归法设 $f(x)$ 是定义在 $D$ 上且取值于 $D$ 的函数, 由此定义数列 $\\left\\{a_n\\right\\}: a_0$ 已知, 且 $a_0 \\in D, a_n=f\\left(a_{n-1}\\right), n \\geqslant 1$. 一方面, 若已求得 $f^{(n)}(x)=g(x)$, 则 $a_n= f\\left(a_{n-1}\\right)=f^{(2)}\\left(a_{n-2}\\right)=\\cdots=f^{(n)}\\left(a_0\\right)$, 即 $\\left\\{a_n\\right\\}$ 的通项公式; 另一方面, 如果已求得 $\\left\\{a_n\\right\\}$ 的通项公式 $a_n=g\\left(a_0\\right)$, 则取 $a_0=x, a_n=g(x)$, 而 $a_n=f\\left(a_{n-1}\\right)=\\cdots= f^{(n)}\\left(a_0\\right)=f^{(n)}(x)$, 从而 $f^{(n)}(x)=g(x)$, 即 $f^{(n)}(x)$ 的表达式.\n由上述知, 函数的 $n$ 次迭代可以通过构造数列的方法来解, 其步骤为\n(1) 设 $a_0=x, a_n=f^{(n)}(x)$ ;\n(2) 由 $a_n=f^{(n)}(x)=f\\left(a_{n-1}\\right)$, 求出 $a_n=g\\left(a_0\\right)$;\n(3) $f^{(n)}(x)=g\\left(a_0\\right)=g(x)$.", + "figures": [] +} \ No newline at end of file diff --git a/processed_dataset/text/0124.json b/processed_dataset/text/0124.json new file mode 100644 index 0000000000000000000000000000000000000000..887f401f7409fff7433cede063176f03b4d3e5a8 --- /dev/null +++ b/processed_dataset/text/0124.json @@ -0,0 +1,5 @@ +{ + "source_file": "./raw_volume-zh/volume2/chapter6.tex", + "text": "(3) 相似法相似法是求函数 $f(x)$ 的 $n$ 次迭代的一个重要方法.\n若存在一个函数 $\\varphi(x)$ 以及它的反函数 $\\varphi^{-1}(x)$, 使得\n$$\nf(x)=\\varphi^{-1}(g(\\varphi(x))),\n$$\n我们就称 $f(x)$ 通过 $\\varphi(x)$ 和 $g(x)$ 相似, 简称 $f(x)$ 和 $g(x)$ 相似, 记为 $f \\stackrel{\\varphi}{\\sim} g$, 其中 $\\varphi(x)$ 称为桥函数.\n相似关系是一个等价关系;也就是说它满足:\n(1) $f \\sim f($ 自身性);\n(2) 若 $f \\sim g$, 则 $g \\sim f$ (对称性);\n(3) 若 $f \\sim g, g \\sim h$, 则 $f \\sim h$ (传递性).\n如果 $f(x)$ 与 $g(x)$ 相似, 即\n$$\nf(x)=\\varphi^{-1}(g(\\varphi(x))),\n$$\n那么\n$$\n\\begin{aligned}\nf^{(2)}(x) & =f(f(x))=\\varphi^{-1}(g(\\varphi(f(x)))) \\\\\n& =\\varphi^{-1}\\left(g\\left(\\varphi\\left(\\varphi^{-1}(g(\\varphi(x)))\\right)\\right)\\right) \\\\\n& =\\varphi^{-1}\\left(g^{(2)}(\\varphi(x))\\right) .\n\\end{aligned}\n$$\n用数学归纳法可以证明\n$$\nf^{(n)}(x)=\\varphi^{-1}\\left(g^{(n)}(\\varphi(x))\\right) .\n$$\n事实上,\n$$\n\\begin{aligned}\nf^{(n+1)}(x) & =f\\left(f^{(n)}(x)\\right) \\\\\n& =f\\left(\\varphi^{-1}\\left(g^{(n)}(\\varphi(x))\\right)\\right) \\\\\n& =\\varphi^{-1} g\\left(\\varphi\\left(\\varphi^{-1}\\left(g^{(n)}(\\varphi(x))\\right)\\right)\\right) \\\\\n& =\\varphi^{-1}\\left(g^{(n+1)}(\\varphi(x))\\right) .\n\\end{aligned}\n$$\n这样一来, 我们便把 $f$ 的迭代问题转化为 $g$ 的迭代问题.", + "figures": [] +} \ No newline at end of file diff --git a/processed_dataset/text/0125.json b/processed_dataset/text/0125.json new file mode 100644 index 0000000000000000000000000000000000000000..1c9ac414768583efbe22a6d7d37832cb21c29901 --- /dev/null +++ b/processed_dataset/text/0125.json @@ -0,0 +1,5 @@ +{ + "source_file": "./raw_volume-zh/volume2/chapter6.tex", + "text": "以上两例是比较难的利用桥函数来解决的问题.\n在介绍完下一种方法后, 我们再来介绍一种寻找桥函数的方法.\n(4) 不动点法不动点法的基本思想是根据函数的不动点得出桥函数的一个性质, 进而确定桥函数的形状, 然后利用相似法求出函数的 $n$ 次迭代.\n我们先给出不动点的定义和性质.\n定义 $6.2 f(x)=x$ 的根称为 $f(x)$ 的不动点.\n函数的不动点具有如下性质:\n(1) 若 $x_0$ 是 $f(x)$ 的不动点, 则 $f^{(n)}\\left(x_0\\right)^x=x_0$, 即 $x_0$ 也是 $f^{(n)}(x)$ 的不动点.\n(2) 设 $f(x)=\\varphi^{-1}(g(\\varphi(x)))$, 因此有 $\\varphi(f(x))=g(\\varphi(x))$. 若 $f\\left(x_0\\right)= x_0$, 则有 $\\varphi\\left(x_0\\right)=g\\left(\\varphi\\left(x_0\\right)\\right)$, 即 $\\varphi\\left(x_0\\right)$ 是 $g(x)$ 的不动点.\n对于一些简单的函数, 利用不动点, 把函数变形后再迭代, 最后用数学归纳法证之,会使算法简单些.\n先看两个例子.", + "figures": [] +} \ No newline at end of file diff --git a/processed_dataset/text/0126.json b/processed_dataset/text/0126.json new file mode 100644 index 0000000000000000000000000000000000000000..a6a6ded3cd0427f74c5ce7f9e67644f9331338ae --- /dev/null +++ b/processed_dataset/text/0126.json @@ -0,0 +1,5 @@ +{ + "source_file": "./raw_volume-zh/volume2/chapter6.tex", + "text": "对于不动点这一重要方法还有两点需要说明:\n(1) 不动点在求解某些特殊的函数方程中往往会有意想不到的简便方法, 而且有时会是唯一的方法, 这会在后面的章节中举例.\n(2) 由于函数的迭代与数列的关系 (具体可见前面递归法的叙述), 利用不动点可以求一些数列的通项公式和研究数列通项具有的性质, 我们给读者留作练习题.", + "figures": [] +} \ No newline at end of file diff --git a/processed_dataset/text/0127.json b/processed_dataset/text/0127.json new file mode 100644 index 0000000000000000000000000000000000000000..c54a1fb951402f58f75240662794ced9273c3600 --- /dev/null +++ b/processed_dataset/text/0127.json @@ -0,0 +1,5 @@ +{ + "source_file": "./raw_volume-zh/volume2/chapter6.tex", + "text": "(2) 数列通项的估值用迭代估计数列通项,基本思想是根据以下定理:\n定理设 $f, g, h$ 都是定义在 $I$ 上且可迭代的函数, 如果 $g$ 和 $h$ 都是单调增函数, 且对 $x \\in I$, 有\n$$\ng(x) \\leqslant f(x) \\leqslant h(x),\n$$\n那么必有\n$$\ng^{(n)}(x) \\leqslant f^{(n)}(x) \\leqslant h^{(n)}(x) .\n$$\n定理由数学归纳法立得, 读者可自行完成.", + "figures": [] +} \ No newline at end of file diff --git a/processed_dataset/text/0128.json b/processed_dataset/text/0128.json new file mode 100644 index 0000000000000000000000000000000000000000..4188e619959e0cd8cc682705ec477e9fc4dfd164 --- /dev/null +++ b/processed_dataset/text/0128.json @@ -0,0 +1,5 @@ +{ + "source_file": "./raw_volume-zh/volume2/chapter7.tex", + "text": "函数方程的解法 \n7.1 代换法代换法是解函数方程的常用手段, 其基本思想是: 将函数方程中的自变量适当地以别的自变量代换 (当然在代换时应特别注意函数的定义域不能发生变化), 得到一个新的函数方程, 然后设法求得未知函数.\n代换法在单变量函数方程中尤为多用.", + "figures": [] +} \ No newline at end of file diff --git a/processed_dataset/text/0129.json b/processed_dataset/text/0129.json new file mode 100644 index 0000000000000000000000000000000000000000..92e17abf19cd152e92f24d092d82f43f3346d805 --- /dev/null +++ b/processed_dataset/text/0129.json @@ -0,0 +1,5 @@ +{ + "source_file": "./raw_volume-zh/volume2/chapter7.tex", + "text": "7.2 赋值法所谓赋值法, 就是对自变量赋予某些特殊的数值, 从而挖掘出题中隐含的条件,并且通过这些新条件简化函数方程, 逼近最终目标.\n先来看两道整数集上函数方程的例子.", + "figures": [] +} \ No newline at end of file diff --git a/processed_dataset/text/0130.json b/processed_dataset/text/0130.json new file mode 100644 index 0000000000000000000000000000000000000000..3eb79003d1d8f05fc1acb203989ab3228adbf99f --- /dev/null +++ b/processed_dataset/text/0130.json @@ -0,0 +1,5 @@ +{ + "source_file": "./raw_volume-zh/volume2/chapter7.tex", + "text": "7.3 柯西法用柯西法解函数方程的步骤是: 先求出对于自变量取所有正整数值时函数方程的解具有的形式, 然后依次证明对自变量取整数值、有理数值以及实数值时函数方程的解仍具有这种形式, 从而得到函数方程的解.", + "figures": [] +} \ No newline at end of file diff --git a/processed_dataset/text/0131.json b/processed_dataset/text/0131.json new file mode 100644 index 0000000000000000000000000000000000000000..62dec05f1397cd4d97b56ddf8a989eec02637707 --- /dev/null +++ b/processed_dataset/text/0131.json @@ -0,0 +1,5 @@ +{ + "source_file": "./raw_volume-zh/volume2/chapter7.tex", + "text": "7.4 递归法函数方程的递归解法, 是一种借助于数列对函数方程加以研究的方法.\n设 $f(n)$ 是定义在正整数集 $\\mathbf{N}_{+}$上的函数,如果存在一个递推关系 $S$ 和初始条件 $f(1)=a_1$, 当知道 $f(1), f(2), \\cdots, f(n)$ 的值后, 由 $S$ 可以唯一地确定 $f(n+1)$ 的值, 我们就称 $f(n)$ 为递归函数, 递归法主要解决递归函数.", + "figures": [] +} \ No newline at end of file diff --git a/processed_dataset/text/0132.json b/processed_dataset/text/0132.json new file mode 100644 index 0000000000000000000000000000000000000000..9adfeb006bc8ecf29211a6a5d93a86dd280980cf --- /dev/null +++ b/processed_dataset/text/0132.json @@ -0,0 +1,5 @@ +{ + "source_file": "./raw_volume-zh/volume3/chapter1.tex", + "text": "1. 三角函数性质奇偶性:正弦函数 $y=\\sin x$ 和正切函数 $y=\\tan x$ 、余切函数 $y=\\cot x$ 在其定义域上为奇函数,余弦函数 $y=\\cos x$ 在其定义域上为偶函数.\n一般判断三角函数的奇偶性时,有的需要先将三角函数解析式恒等变形化简, 有的需要将 $f(-x)$ 进行变形.\n单调性:三角函数单调性在平面几何、立体几何、解析几何、复数等分支中均有广泛地应用.\n解决三角函数的单调性问题时, 一般先将三角函数转化为基本三角函数,然后利用基本三角函数的单调性来解决.\n基本三角函数的单调性如下:\n$$\ny=\\sin x \\text { 在 }\\left[-\\frac{\\pi}{2}+2 k \\pi, \\frac{\\pi}{2}+2 k \\pi\\right] \\text { 上为增函数,在 }\\left[\\frac{\\pi}{2}+2 k \\pi, \\frac{3 \\pi}{2}+2 k \\pi\\right]\n$$\n上为减函数 $(k \\in \\mathbf{Z})$.\n$y=\\cos x$ 在 $[-\\pi+2 k \\pi, 2 k \\pi]$ 上为增函数,在 $[2 k \\pi, \\pi+2 k \\pi]$ 上为减函数 $(k \\in \\mathbf{Z})$.\n$y=\\tan x$ 在 $\\left(-\\frac{\\pi}{2}+k \\pi, \\frac{\\pi}{2}+k \\pi\\right)$ 上为增函数 $(k \\in \\mathbf{Z}) . y=\\cot x$ 在 $(k \\pi$, $\\pi+k \\pi)$ 上为减函数 $(k \\in \\mathbf{Z})$.\n周期性: 周期函数的本质是, 存在非零常数 $T$, 使定义域中的任意 $x$ 都有 $f(x+T)=f(x)$ 成立.\n下面列举与周期函数相关的几个结论:\n(1) 周期函数的定义域是无界的.\n(2) 定义域为 $\\mathbf{R}$ 的周期函数 $f(x)$, 若 $T$ 是周期,则 $n T(n \\in \\mathbf{Z}, n \\neq 0)$ 仍是函数的周期.\n(3) 设 $f(x)$ 是非常数的周期函数, 且定义域为 $D$, 若 $f(x)$ 在 $D$ 上, 则 $f(x)$ 有最小正周期.\n(4) 若函数 $f(x)$ 有最小正周期 $T$, 那么除 $n T(n \\in \\mathbf{Z}, n \\neq 0)$ 外, 函数无其他周期.\n(5) 函数 $y=f(x)$ 是数集 $M$ 上的周期函数, 则:\n(5-1) $a f(x)+b$ ( $a, b$ 是常数) 是 $M$ 上的周期函数;\n(5-2) $|f(x)|$ 是 $M$ 上的周期函数;\n(5-3) $\\frac{1}{f(x)}$ 是 $\\{x \\mid f(x) \\neq 0, x \\in M\\}$ 上的周期函数;\n(5-4) $f(a x+b)$ 是 $\\{x \\mid a x+b, x \\in M\\}$ 上的周期函数.\n(6) 设函数 $y=f(u)$ 的定义域为 $M, u=g(x)$ 是 $M$ 上的周期函数, 如果当 $x \\in M_1$ 时, $g(x) \\in M$, 那么 $f(g(x))$ 是 $M_1$ 上的周期函数.\n(7) 设函数 $y=f(x)$, 如果对任意实数 $x$, 都有 $f(a+x)=f(a-x)$, $f(b+x)=f(b-x)(a \\neq b)$, 则 $f(x)$ 是周期函数, 周期 $T=2(b-a)$.\n(8) 设函数 $y=f(x)$, 如果它的图形关于两点 $\\left(a_1, b\\right)$ 和 $\\left(a_2, b\\right)\\left(a_1 \\neq a_2\\right)$ 对称, 那么 $f(x)$ 是周期函数, 其周期 $T=2\\left(a_1-a_2\\right)$.\n(9) 设函数 $y=f(x)$, 如果对任意实数 $x$, 都有 $f(a+x)=f(a-x)$, $f(b+x)=-f(b-x),(a \\neq b)$, 则 $f(x)$ 是周期函数, 其周期为 $T=4(b-a)$.\n(10) $y=A \\sin (\\omega x+\\varphi)(\\omega \\neq 0)$ 的最小正周期是 $T=\\frac{2 \\pi}{|\\omega|} ; y=A \\cos (\\omega x+ \\varphi ( \\omega \\neq 0)$ 的最小正周期是 $T=\\frac{2 \\pi}{|\\omega|}, y=A \\tan (\\omega x+\\varphi)(\\omega \\neq 0)$ 的最小正周期是 $T=\\frac{\\pi}{|\\omega|}, y=A \\cot (\\omega x+\\varphi)(\\omega \\neq 0)$ 的最小正周期是 $T=\\frac{\\pi}{|\\omega|}$.\n对于复杂的三角函数,一般须先将其转化为基本三角函数, 然后可以得到它的周期性.\n这里要求在变形过程中必须是等价的, 特别要注意的是定义域的变化.\n2. 三角函数的图象变换\n(1) 平移变换\n(1-1) 左右平移: $y=\\sin x \\rightarrow y=\\sin (x+\\varphi)$\n$\\varphi>0$ 向左平移 $\\varphi$ 个单位, $\\varphi<0$ 向右平移 $|\\varphi|$ 个单位.\n(1-2) 上下平移: $y=\\sin x \\rightarrow y=\\sin x+k$\n$k>0$ 向上平移 $k$ 个单位, $k<0$ 向下平移 $|k|$ 个单位.\n(2) 周期变换: $y=\\sin x \\rightarrow y=\\sin \\omega x$\n(2-1) 当 $\\omega>1$ 时, 纵坐标不变, 横坐标缩短为原来的 $\\frac{1}{\\omega}$ 倍;\n(2-2) 当 $0<\\omega<1$ 时, 纵坐标不变, 横坐标伸长为原来的 $\\frac{1}{\\omega}$ 倍.\n(3) 振幅变换: $y=\\sin x \\rightarrow y=A \\sin x$\n(3-1) 当 $A>1$ 时, 横坐标不变,纵坐标伸长为原来的 $A$ 倍;\n(3-2) 当 $0b$ & 一解 & 一解 \\\\\n\\hline$a=b$ & 无解 & 一解 \\\\\n\\hline \\multirow{2}{*}{$ab \\sin A$ 两解 \\\\\n& & $a=b \\sin A$ 一解 \\\\\n& & $aa(|a|<1)$, 可作出如图() 所示的单位圆, 在纵轴上取点 $A(0, a)$, 过 $A$ 作 $M N / / O x$ 轴交单位圆于 $M$ 和 $N$, 根据三角函数的定义可知阴影部分的角度满足 $\\sin x>a$, 即其解集为 $\\{x \\mid 2 k \\pi+\\arcsin a) 所示, 在 $y$ 轴上取点 $A(0, a)$, 过 $A$ 作 $x$ 轴的平行线, 在这条平行线上方的图象,其三角函数值满足不等式 $\\sin x>a$, 从而其解集为 $\\{x \\mid 2 k \\pi+\\arcsin a0$, $b>0, c>0$, 则有 $a+b \\geqslant 2 \\sqrt{a b}$ (当且仅当 $a=b$ 时取等号); $a+b+c \\geqslant$\n$3 \\sqrt[3]{a b c}$ (当且仅当 $a=b=c$ 时取等号); $a^3+b^3+c^3 \\geqslant 3 a b c$ (当且仅当 $a= b=c$ 时取等号).\n另外, 在三角函数中, 有一个重要不等式: 若 $x \\in\\left(0, \\frac{\\pi}{2}\\right)$, 则 $\\sin x) 来证明.\n在单位圆上作 $\\angle A O M=x, A T \\perp O x$ 轴, $N M \\perp O x$ 轴, 则 $\\sin x=N M, \\tan x=A T$, 由 $S_{\\triangle A O M}0, i=1$, $2, \\cdots, k$, 有\n$$\n\\left(a_1 a_2 \\cdots a_k\\right)^{\\frac{1}{k}} \\leqslant \\frac{a_1+a_2+\\cdots+a_k}{k} .\n$$\n那么, 当 $n=k+1$ 时, 由于\n$$\nA_{k+1}=\\frac{a_1+a_2+\\cdots+a_{k+1}}{k+1}, G_{k+1}=\\sqrt[k+1]{a_1 a_2 \\cdots a_k a_{k+1}},\n$$\n关于 $a_1, a_2, \\cdots, a_{k+1}$ 是对称的,任意对调 $a_i$ 与 $a_j(i \\neq j)$, 即将 $a_i$ 写成 $a_j, a_j$ 写成 $a_i, A_{k+1}$ 和 $G_{k+1}$ 的值不改变, 因此不妨设 $a_1=\\min \\left\\{a_1, a_2, \\cdots, a_{k+1}\\right\\}$, $a_{k+1}=\\max \\left\\{a_1, a_2, \\cdots, a_{k+1}\\right\\}$, 显然 $a_1 \\leqslant A_{k+1} \\leqslant a_{k+1}$, 以及\n$$\nA_{k+1}\\left(a_1+a_{k+1}-A_{k+1}\\right)-a_1 a_{k+1}=\\left(a_1-A_{k+1}\\right)\\left(A_{k+1}-a_{k+1}\\right) \\geqslant 0,\n$$\n即\n$$\nA_{k+1}\\left(a_1+a_{k+1}-A_{k+1}\\right) \\geqslant a_1 a_{k+1} \\text {. }\n$$\n对 $k$ 个正数 $a_2, a_3, \\cdots, a_k, a_1+a_{k+1}-A_{k+1}$, 由归纳假设, 得\n$$\n\\frac{a_2+a_3+\\cdots+a_k+\\left(a_1+a_{k+1}-A_{k+1}\\right)}{k} \\geqslant \\sqrt[k]{a_2 a_3 \\cdots a_k\\left(a_1+a_{k+1}-A_{k+1}\\right)} .\n$$\n而\n$$\n\\frac{a_2+a_3+\\cdots+a_k+\\left(a_1+a_{k+1}-A_{k+1}\\right)}{k}=\\frac{(k+1) A_{k+1}-A_{k+1}}{k}=A_{k+1},\n$$\n于是\n$$\nA_{k+1}^k \\geqslant a_2 a_3 \\cdots a_k\\left(a_1+a_{k+1}-A_{k+1}\\right) .\n$$\n两边乘以 $A_{k+1}$, 得\n$$\nA_{k+1}^{k+1} \\geqslant a_2 a_3 \\cdots a_k A_{k+1}\\left(a_1+a_{k+1}-A_{k+1}\\right) \\geqslant a_2 a_3 \\cdots a_k\\left(a_1 a_{k+1}\\right)=G_{k+1}^{k+1} .\n$$\n从而, 有 $A_{k+1} \\geqslant G_{k+1}$.\n直接验证可知, 当且仅当所有的 $a_i$ 相等时等号成立, 故命题成立.\n说明这里, 利用了证明与正整数有关的命题的常用方法, 即数学归纳法.\n数学归纳法证题技巧的应用, 可以说是五彩缤纷, 千姿百态.\n应用数学归纳法, 除了需要验证当 $n=1$ 或 $n=n_0$ (这里 $n_0$ 为某个固定的正整数)外, 其关键是要在 $n=k$ 时成立的假设之下, 导出当 $n=k+1$ 时命题也成立, 要完成这一步,需要一定的技巧和处理问题的能力, 只有通过多做练习来实现理解和掌握.", + "figures": [] +} \ No newline at end of file diff --git a/processed_dataset/text/0140.json b/processed_dataset/text/0140.json new file mode 100644 index 0000000000000000000000000000000000000000..c887ef69bd642439d94affa698130bda6f896a76 --- /dev/null +++ b/processed_dataset/text/0140.json @@ -0,0 +1,5 @@ +{ + "source_file": "./raw_volume-zh/volume4/chapter1.tex", + "text": "平均值不等式的证明.\n一般地, 假设 $a_1, a_2, \\cdots, a_n$ 为 $n$ 个非负实数, 它们的算术平均值记为\n$$\nA_n=\\frac{a_1+a_2+\\cdots+a_n}{n},\n$$\n几何平均值记为\n$$\nG_n=\\left(a_1 a_2 \\cdots a_n\\right)^{\\frac{1}{n}}=\\sqrt[n]{a_1 a_2 \\cdots a_n} .\n$$\n算术平均值与几何平均值之间有如下的关系\n$$\n\\frac{a_1+a_2+\\cdots+a_n}{n} \\geqslant \\sqrt[n]{a_1 a_2 \\cdots a_n},\n$$\n即\n$$\nA_n \\geqslant G_n,\n$$\n当且仅当 $a_1=a_2=\\cdots=a_n$ 时,等号成立.\n证法二(归纳法,与证法一的不同处理)\n(1)当 $n=2$ 时,已知结论成立.\n(2)假设对 $n=k$ (正整数 $k \\geqslant 2$ ) 时命题成立, 即对于 $a_i>0, i=1$, $2, \\cdots, k$, 有\n$$\n\\left(a_1 a_2 \\cdots a_k\\right)^{\\frac{1}{k}} \\leqslant \\frac{a_1+a_2+\\cdots+a_k}{k} .\n$$\n那么, 当 $n=k+1$ 时,\n$$\n\\begin{aligned}\n& a_1+a_2+\\cdots+a_k+a_{k+1} \\\\\n= & a_1+a_2+\\cdots+a_k+\\left(a_{k+1}+G_{k+1+\\cdots+}^{(k-1) \\uparrow G_{k+1}} G_{k+1}\\right)-(k-1) G_{k+1} \\\\\n\\geqslant & k \\sqrt[k]{a_1 a_2 \\cdots a_k}+k \\sqrt[k]{a_{k+1} G_{k+1}^{k-1}}-(k-1) G_{k+1} \\\\\n\\geqslant & 2 k \\sqrt{\\sqrt[k]{a_1 a_2 \\cdots a_k} \\sqrt[k]{a_{k+1} G_{k+1}^{k-1}}}-(k-1) G_{k+1} \\\\\n= & 2 k \\sqrt[2 k]{a_1 a_2 \\cdots a_{k+1} G_{k+1}^{k-1}}-(k-1) G_{k+1} \\\\\n= & 2 k \\sqrt[2 k]{G_{k+1}^{k+1} G_{k+1}^{k-1}}-(k-1) G_{k+1} \\\\\n= & (k+1) G_{k+1}\n\\end{aligned}\n$$\n于是 $A_{k+1} \\geqslant G_{k+1}$.\n不难看出,当且仅当所有的 $a_i$ 相等时等号成立,故命题成立.", + "figures": [] +} \ No newline at end of file diff --git a/processed_dataset/text/0141.json b/processed_dataset/text/0141.json new file mode 100644 index 0000000000000000000000000000000000000000..6d5779725c7ee954a1bae412e76e4eeceee43e85 --- /dev/null +++ b/processed_dataset/text/0141.json @@ -0,0 +1,5 @@ +{ + "source_file": "./raw_volume-zh/volume4/chapter1.tex", + "text": "平均值不等式的证明.\n一般地, 假设 $a_1, a_2, \\cdots, a_n$ 为 $n$ 个非负实数, 它们的算术平均值记为\n$$\nA_n=\\frac{a_1+a_2+\\cdots+a_n}{n},\n$$\n几何平均值记为\n$$\nG_n=\\left(a_1 a_2 \\cdots a_n\\right)^{\\frac{1}{n}}=\\sqrt[n]{a_1 a_2 \\cdots a_n} .\n$$\n算术平均值与几何平均值之间有如下的关系\n$$\n\\frac{a_1+a_2+\\cdots+a_n}{n} \\geqslant \\sqrt[n]{a_1 a_2 \\cdots a_n},\n$$\n即\n$$\nA_n \\geqslant G_n,\n$$\n当且仅当 $a_1=a_2=\\cdots=a_n$ 时,等号成立.\n证法三 (归纳法, 另一种处理方式)\n(1)当 $n=2$ 时,已知结论成立.\n(2)假设对 $n=k$ (正整数 $k \\geqslant 2$ ) 时命题成立, 即对于 $a_i>0, i=1$, $2, \\cdots, k$, 有\n$$\n\\left(a_1 a_2 \\cdots a_k\\right)^{\\frac{1}{k}} \\leqslant \\frac{a_1+a_2+\\cdots+a_k}{k} .\n$$\n那么, 当 $n=k+1$ 时,\n$$\n\\begin{aligned}\nA_{k+1} & =\\frac{1}{2 k}\\left[(k+1) A_{k+1}+(k-1) A_{k+1}\\right] \\\\\n& =\\frac{1}{2 k}(a_1+a_2+\\cdots+a_{k+1}+\\underbrace{}_{\\frac{N_{k+1}-1 \\uparrow}{A_{k+1}}+A_{k+1}+\\cdots+A_{k+1}}) \\\\\n& \\geqslant \\frac{1}{2 k}\\left(k \\sqrt[k]{a_1 a_2 \\cdots a_k}+k \\sqrt[k]{a_{k+1} A_{k+1}^{k-1}}\\right) \\geqslant \\sqrt[2 k]{a_1 a_2 \\cdots a_k a_{k+1} A_{k+1}^{k-1}} .\n\\end{aligned}\n$$\n所以 $A_{k+1}^{2 k} \\geqslant a_1 a_2 \\cdots a_{k+1} A_{k+1}^{k-1}$, 故得 $A_{k+1} \\geqslant G_{k+1}$.\n说明.\n在上面的证明中, 将 $A_{k+1}$ 表示为 $A_{k+1}=\\frac{1}{2 k}\\left[(k+1) A_{k+1}+(k-\\right.$ 1) $\\left.A_{k+1}\\right]$ 是一步较为关键和重要的变形技巧.", + "figures": [] +} \ No newline at end of file diff --git a/processed_dataset/text/0142.json b/processed_dataset/text/0142.json new file mode 100644 index 0000000000000000000000000000000000000000..cce6bd63907ca8158083d8c611a2a6f08bf6c41a --- /dev/null +++ b/processed_dataset/text/0142.json @@ -0,0 +1,5 @@ +{ + "source_file": "./raw_volume-zh/volume4/chapter1.tex", + "text": "平均值不等式的证明.\n一般地, 假设 $a_1, a_2, \\cdots, a_n$ 为 $n$ 个非负实数, 它们的算术平均值记为\n$$\nA_n=\\frac{a_1+a_2+\\cdots+a_n}{n},\n$$\n几何平均值记为\n$$\nG_n=\\left(a_1 a_2 \\cdots a_n\\right)^{\\frac{1}{n}}=\\sqrt[n]{a_1 a_2 \\cdots a_n} .\n$$\n算术平均值与几何平均值之间有如下的关系\n$$\n\\frac{a_1+a_2+\\cdots+a_n}{n} \\geqslant \\sqrt[n]{a_1 a_2 \\cdots a_n},\n$$\n即\n$$\nA_n \\geqslant G_n,\n$$\n当且仅当 $a_1=a_2=\\cdots=a_n$ 时,等号成立.\n证法四 (归纳法和变换)\n在证明原命题之前, 首先令\n$$\ny_1=\\frac{a_1}{G_n}, y_2=\\frac{a_2}{G_n}, \\cdots, y_n==\\frac{a_n}{G_n},\n$$\n其中 $G_n=\\sqrt[n]{a_1 a_2 \\cdots a_n}$, 则 $y_1 y_2 \\cdots y_n=1\\left(y_i>0\\right)$, 且平均值不等式等价于\n$$\ny_1+y_2+\\cdots+y_n \\geqslant n \\text {. }\n$$\n即在条件 $y_1 y_2 \\cdots y_n=1\\left(y_i>0\\right)$ 之下, 证明 $y_1+y_2+\\cdots+y_n \\geqslant n$.\n我们用归纳法证明上述不等式.\n(1)当 $n=-1$ 时, $y_1=1 \\geqslant 1$, 显然成立.\n(2)假设当 $n=k$ 时不等式成立,则对于 $n=k+1$, 由于 $y_1 y_2 \\cdots y_n=1 \\left(y_i>0\\right)$, 那么 $y_i$ 中必有大于或等于 1 者, 也有小于或等于 1 者, 不妨设 $y_k \\geqslant 1, y_{k+1} \\leqslant 1$, 并令 $y=y_k y_{k+1}$, 则 $y_1 y_2 \\cdots y_{k-1} y=1$, 从而由归纳假设, 得\n$$\ny_1+y_2+\\cdots+y_{k-1}+y \\geqslant k \\text {. }\n$$\n于是\n$$\n\\begin{aligned}\n& y_1+y_2+\\cdots+y_{k-1}+y_k+y_{k+1} \\\\\n\\geqslant & k+y_k+y_{k+1}-y_k y_{k+1} \\\\\n= & k+1+\\left(y_k-1\\right)\\left(1-y_{k+1}\\right) \\\\\n\\geqslant & k+1 .\n\\end{aligned}\n$$\n不难看出, 当且仅当 $y_1=y_2=\\cdots=y_n=1$, 从而 $a_1=a_2=\\cdots=a_n$ 时, 等号成立.\n故当 $n=k+1$ 时,命题也成立.\n说明通过变量替换,将原问题化为一个与正整数有关的形式简单的不等式,在证明中运用了我们比较熟悉的手段和技巧.", + "figures": [] +} \ No newline at end of file diff --git a/processed_dataset/text/0143.json b/processed_dataset/text/0143.json new file mode 100644 index 0000000000000000000000000000000000000000..8ef1987dc234dd282d84dc7716912225919879d2 --- /dev/null +++ b/processed_dataset/text/0143.json @@ -0,0 +1,5 @@ +{ + "source_file": "./raw_volume-zh/volume4/chapter1.tex", + "text": "平均值不等式的证明.\n一般地, 假设 $a_1, a_2, \\cdots, a_n$ 为 $n$ 个非负实数, 它们的算术平均值记为\n$$\nA_n=\\frac{a_1+a_2+\\cdots+a_n}{n},\n$$\n几何平均值记为\n$$\nG_n=\\left(a_1 a_2 \\cdots a_n\\right)^{\\frac{1}{n}}=\\sqrt[n]{a_1 a_2 \\cdots a_n} .\n$$\n算术平均值与几何平均值之间有如下的关系\n$$\n\\frac{a_1+a_2+\\cdots+a_n}{n} \\geqslant \\sqrt[n]{a_1 a_2 \\cdots a_n},\n$$\n即\n$$\nA_n \\geqslant G_n,\n$$\n当且仅当 $a_1=a_2=\\cdots=a_n$ 时,等号成立.\n证法五 (归纳法和二项展开式)\n(1) 当 $n=2$ 时,已知结论成立.\n(2)假设对 $n=k$ (正整数 $k \\geqslant 2$ ) 时命题成立, 即对于 $a_i>0, i=1$, $2, \\cdots, k$, 有\n$$\n\\left(a_1 a_2 \\cdots a_k\\right)^{\\frac{1}{k}} \\leqslant \\frac{a_1+a_2+\\cdots+a_k}{k} .\n$$\n那么, 当 $n=k+1$ 时, 不妨假设 $a_{k+1}=\\max \\left\\{a_1, a_2, \\cdots, a_{k+1}\\right\\}$, 于是由归纳假设, 得\n$$\na_{k+1} \\geqslant \\frac{a_1+a_2+\\cdots+a_k}{k}=A_k \\geqslant G_k=\\sqrt[k]{a_1 a_2 \\cdots a_k} .\n$$\n从而, 得\n$$\nA_{k+1}^{k+1}=\\left(\\frac{a_1+a_2+\\cdots+a_k+a_{k+1}}{k+1}\\right)^{k+1}=\\left(\\frac{k A_k+a_{k+1}}{k+1}\\right)^{k+1}=\\left(A_k+\\frac{a_{k+1}-A_k}{k+1}\\right)^{k+1}\n$$\n$$\n\\begin{aligned}\n& =A_k^{k+1}+(k+1) A_k^k\\left(\\frac{a_{k+1}-A_k}{k+1}\\right)+\\cdots+\\left(\\frac{a_{k+1}-A_k}{k+1}\\right)^{k+1} \\\\\n& \\geqslant A_k^{k+1}+(k+1) A_k^k\\left(\\frac{a_{k+1}-A_k}{k+1}\\right)=A_k^{k+1}+A_k^k\\left(a_{k+1}-A_k\\right) \\\\\n& =A_k^k a_{k+1} \\geqslant G_k^k a_{k+1}=a_1 a_2 \\cdots a_k a_{k+1} \\\\\n& =G_{k+1}^{k+1} .\n\\end{aligned}\n$$\n所以 $A_{k+1} \\geqslant G_{k+1}$.\n不难看出, 当且仅当所有的 $a_i$ 相等时等号成立,故命题成立.\n说明在证明过程中, 考虑 $A_{k+1}^{k+1}$, 并通过一定的处理和运算, 导出所需要的结果.\n有时候可能利用到其他的有用结论.", + "figures": [] +} \ No newline at end of file diff --git a/processed_dataset/text/0144.json b/processed_dataset/text/0144.json new file mode 100644 index 0000000000000000000000000000000000000000..945f23167a63cbce8a49b17572c9f3fa86208e6f --- /dev/null +++ b/processed_dataset/text/0144.json @@ -0,0 +1,5 @@ +{ + "source_file": "./raw_volume-zh/volume4/chapter1.tex", + "text": "平均值不等式的证明.\n一般地, 假设 $a_1, a_2, \\cdots, a_n$ 为 $n$ 个非负实数, 它们的算术平均值记为\n$$\nA_n=\\frac{a_1+a_2+\\cdots+a_n}{n},\n$$\n几何平均值记为\n$$\nG_n=\\left(a_1 a_2 \\cdots a_n\\right)^{\\frac{1}{n}}=\\sqrt[n]{a_1 a_2 \\cdots a_n} .\n$$\n算术平均值与几何平均值之间有如下的关系\n$$\n\\frac{a_1+a_2+\\cdots+a_n}{n} \\geqslant \\sqrt[n]{a_1 a_2 \\cdots a_n},\n$$\n即\n$$\nA_n \\geqslant G_n,\n$$\n当且仅当 $a_1=a_2=\\cdots=a_n$ 时,等号成立.\n证法六 (倒向归纳法)\n倒向归纳法, 也称 \"留空回填\" 法.\n基本思想是先对自然数的一个子列 $\\left\\{n_m\\right\\}$ 证明命题成立, 然后再回过来证明 $\\{n\\} \\backslash\\left\\{n_m\\right\\}$ 相应的命题成立.\n首先证明当 $n=2^m$ ( $m$ 为正整数) 时, 平均值不等式成立.\n为此, 对 $m$ 用数学归纳法.\n当 $m=1$ 时,显然有 $\\sqrt{a_1 a_2} \\leqslant \\frac{a_1+a_2}{2}$.\n假设 $m=k$ 时命题成立,则当 $m=k+1$ 时,\n$$\n\\begin{aligned}\n& \\sqrt[2^{k+1}]{a_1 a_2 \\cdots a_{2^k} a_{2^k+1} \\cdots a_{2^{k+1}}} \\\\\n& =\\sqrt{\\sqrt[2^k]{a_1 a_2 \\cdots a_{2^k}} \\sqrt[2^k]{a_{2^k+1} \\cdots a_{2^{k+1}}}} \\\\\n& \\leqslant \\frac{1}{2}\\left(\\sqrt[2^k]{a_1 a_2 \\cdots a_{2^k}}+\\sqrt[2^k]{a_{2^k+1} \\cdots a_{2^{k+1}}}\\right) \\\\\n& \\leqslant \\frac{1}{2}\\left(\\frac{a_1+a_2+\\cdots+a_{2^k}}{2^k}+\\frac{a_{2^k}+1+\\cdots+a_{2^{k+1}}}{2^k}\\right) \\\\\n& =\\frac{a_1+a_2+\\cdots+a_{2^k}+a_{2^k+1}+\\cdots+a_{2^{k+1}}}{2^{k+1}} \\text {. } \\\\\n&\n\\end{aligned}\n$$\n所以对于具有 $n=2^m$ 形式的正整数 $n$, 平均值不等式成立, 即对无穷多个正整数 $2,4,8, \\cdots, 2^m, \\cdots$, 平均值不等式成立.\n现假设 $n=k+1$ 时, 平均值不等式成立.\n当 $n=k$ 时, $A_k= \\frac{a_1+a_2+\\cdots+a_k}{k}$, 则由假设,得\n$$\n\\sqrt[k+1]{a_1 a_2 \\cdots a_k A_k} \\leqslant \\frac{a_1+a_2+\\cdots+a_k+A_k}{k+1}=\\frac{k A_k+A_k}{k+1}=A_k,\n$$\n所以 $G_k \\leqslant A_k$ ,也就是说当 $n=k$ 时命题也成立.\n综上可知, 对一切正整数 $n$, 平均值不等式成立.\n不难看出, 当且仅当所有的 $a_i$ 相等时等号成立,故命题成立.", + "figures": [] +} \ No newline at end of file diff --git a/processed_dataset/text/0145.json b/processed_dataset/text/0145.json new file mode 100644 index 0000000000000000000000000000000000000000..79a8b73704908764099e15a93e4f381766e264bd --- /dev/null +++ b/processed_dataset/text/0145.json @@ -0,0 +1,5 @@ +{ + "source_file": "./raw_volume-zh/volume4/chapter1.tex", + "text": "平均值不等式的证明.\n一般地, 假设 $a_1, a_2, \\cdots, a_n$ 为 $n$ 个非负实数, 它们的算术平均值记为\n$$\nA_n=\\frac{a_1+a_2+\\cdots+a_n}{n},\n$$\n几何平均值记为\n$$\nG_n=\\left(a_1 a_2 \\cdots a_n\\right)^{\\frac{1}{n}}=\\sqrt[n]{a_1 a_2 \\cdots a_n} .\n$$\n算术平均值与几何平均值之间有如下的关系\n$$\n\\frac{a_1+a_2+\\cdots+a_n}{n} \\geqslant \\sqrt[n]{a_1 a_2 \\cdots a_n},\n$$\n即\n$$\nA_n \\geqslant G_n,\n$$\n当且仅当 $a_1=a_2=\\cdots=a_n$ 时,等号成立.\n证法七(利用排序不等式)\n为了利用与上面不同的方法证明平均值不等式, 我们首先介绍和证明另一个重要的结论,即排序不等式.\n引理 1 (排序不等式) 设两个实数组 $a_1, a_2, \\cdots, a_n$ 和 $b_1, b_2, \\cdots, b_n$, 满足\n$$\na_1 \\leqslant a_2 \\leqslant \\cdots \\leqslant a_n ; b_1 \\leqslant b_2 \\leqslant \\cdots \\leqslant b_n,\n$$\n则\n$a_1 b_1+a_2 b_2+\\cdots+a_n b_n$ (同序乘积之和)\n$\\geqslant a_1 b_{j_1}+a_2 b_{j_2}+\\cdots+a_n b_{j_n}$ (乱序乘积之和)\n$\\geqslant a_1 b_n+a_2 b_{n-1}+\\cdots+a_n b_1$ (反序乘积之和)\n其中 $j_1, j_2, \\cdots, j_n$ 是 $1,2, \\cdots, n$ 的一个排列, 并且等号同时成立的充分必要条件是 $a_1=a_2=\\cdots=a_n$ 或 $b_1=b_2=\\cdots=b_n$ 成立.\n证明令 $A=a_1 b_{j_1}+a_2 b_{j_2}+\\cdots+a_n b_{j_n}$. 如果 $j_n \\neq n$, 且假设此时 $b_n$ 所在的项是 $a_{j_m} b_n$, 则由 $\\left(b_n-b_{j_n}\\right)\\left(a_n-a_{j_m}\\right) \\geqslant 0$, 得\n$$\na_n b_n+a_{j_m} b_{j_n} \\geqslant a_{j_m} b_n+a_n b_{j_n},\n$$\n也就是说, $j_n \\neq n$ 时, 调换 $A$ 中 $b_n$ 与 $b_{j_n}$ 的位置, 其余都不动, 则得到 $a_n b_n$ 项, 并使 $A$ 变为 $A_1$, 且 $A_1 \\geqslant A$. 用同样的方法, 可以再得到 $a_{n-1} b_{n-1}$ 项, 并使 $A_1$ 变为 $A_2$, 且 $A_2 \\geqslant A_1$.\n继续这个过程, 至多经过 $n-1$ 次调换, 得 $a_1 b_1+a_2 b_2+\\cdots+a_n b_n$, 故\n$$\na_1 b_1+a_2 b_2+\\cdots+a_n b_n \\geqslant A \\text {. }\n$$\n同样可以证明 $A \\geqslant a_1 b_n+a_2 b_{n-1}+\\cdots+a_n b_1$.\n显然当 $a_1=a_2=\\cdots=a_n$ 或 $b_1=b_2=\\cdots=b_n$ 时,两个等号同时成立.\n反之, 如果 $\\left\\{a_1, a_2, \\cdots, a_n\\right\\}$ 及 $\\left\\{b_1, b_2, \\cdots, b_n\\right\\}$ 中的数都不全相同时, 则必有 $a_1 \\neq a_n, b_1 \\neq b_n$. 于是 $a_1 b_1+a_n b_n>a_1 b_n+a_n b_1$, 且 $a_2 b_2+\\cdots+a_{n-1} b_{n-1} \\geqslant a_2 b_{n-1}+\\cdots+a_{n-1} b_2$, 从而有 $a_1 b_n+a_2 b_2+\\cdots+a_n b_n>a_1 b_n+a_2 b_{n-1}+\\cdots+ a_n b_1$. 故这两个等式中至少有一个不成立.\n现在,利用引理 1 证明平均值不等式.\n令 $y_k=\\frac{a_1 a_2 \\cdots a_k}{G_n^k}, k=1,2, \\cdots, n$. 由排序不等式, 得\n$$\n\\begin{aligned}\n& y_1 \\times \\frac{1}{y_1}+y_2 \\times \\frac{1}{y_2}+\\cdots+y_n \\times \\frac{1}{y_n} \\\\\n\\leqslant & y_1 \\times \\frac{1}{y_n}+y_2 \\times \\frac{1}{y_1}+\\cdots+y_n \\times \\frac{1}{y_{n-1}} \\\\\n= & \\frac{a_1+a_2+\\cdots+a_n}{G_n}\n\\end{aligned}\n$$\n所以 $A_n \\geqslant G_n$.\n显然当 $a_1=a_2=\\cdots=a_n$ 时, $A_n=G_n$. 如果 $a_1, a_2, \\cdots, a_n$ 不全相等, 不妨设 $a_1 \\neq a_2$, 令 $b=\\frac{a_1+a_2}{2}$, 则 $a_1 a_20\\right)$, 且平均值不等式等价于\n$$\ny_1+y_2+\\cdots+y_n \\geqslant n \\text {. }\n$$\n下面利用排序不等式证明这个不等式.\n任取 $x_1>0$, 再取 $x_2>0$, 使得 $y_1=\\frac{x_1}{x_2}$, 再取 $x_3>0$, 使得 $y_2=\\frac{x_2}{x_3}, \\cdots$, 最后取 $x_n>0$, 使得 $y_{n-1}=\\frac{x_{n-1}}{x_n}$. 所以\n$$\ny_n=\\frac{1}{y_1 y_2 \\cdots y_{n-1}}=\\frac{1}{\\frac{x_1}{x_2} \\frac{x_2}{x_3} \\cdots \\frac{x_{n-1}}{x_n}}=\\frac{x_n}{x_1} .\n$$\n由引理 1 , 得\n$$\ny_1+y_2+\\cdots+y_n=\\frac{x_1}{x_2}+\\frac{x_2}{x_3}+\\cdots+\\frac{x_{n-1}}{x_n}+\\frac{x_n}{x_1} \\geqslant n .\n$$\n当且仅当 $x_1=x_2=\\cdots=x_n$ 时等号成立, 从而当且仅当 $y_1=y_2=\\cdots=y_n$ 时等号成立,所以当且仅当 $a_1=a_2=\\cdots=a_n$ 时等号成立.\n(2) 排序不等式是一个重要的基本的不等式, 可以利用排序不等式直接证明许多其他有关的不等式.\n例如:\n契比雪夫不等式设 $a_1, a_2, \\cdots, a_n, b_1, b_2, \\cdots, b_n$ 满足 $a_1 \\leqslant a_2 \\leqslant \\cdots \\leqslant a_n, b_1 \\leqslant b_2 \\leqslant \\cdots \\leqslant b_n$, 则\n$$\nn \\sum_{k=1}^n a_k b_{n-k+1} \\leqslant \\sum_{k=1}^n a_k \\sum_{k=1}^n b_k \\leqslant n \\sum_{k=1}^n a_k b_k,\n$$\n当且仅当 $a_1=a_2=\\cdots=a_n$ 或 $b_1=b_2=\\cdots=b_n$ 时等号成立.\n证明显然\n$$\n\\begin{aligned}\n& n \\sum_{k=1}^n a_k b_k-\\sum_{k=1}^n a_k \\sum_{k=1}^n b_k \\\\\n= & \\sum_{k=1}^n \\sum_{j=1}^n\\left(a_k b_k-a_k b_j\\right)=\\sum_{j=1}^n \\sum_{k=1}^n\\left(a_j b_j \\rightarrow a_j b_k\\right) \\\\\n= & \\frac{1}{2} \\sum_{k=1}^n \\sum_{j=1}^n\\left(a_k b_k+a_j b_j-a_k b_j-a_j b_k\\right) \\\\\n= & \\frac{1}{2} \\sum_{k=1}^n \\sum_{j=1}^n\\left(a_k-a_j\\right)\\left(b_k-b_j\\right) \\geqslant 0,\n\\end{aligned}\n$$\n故命题成立.", + "figures": [] +} \ No newline at end of file diff --git a/processed_dataset/text/0146.json b/processed_dataset/text/0146.json new file mode 100644 index 0000000000000000000000000000000000000000..d1e083dc35748c06b05afa4f3eb676e6c5219bce --- /dev/null +++ b/processed_dataset/text/0146.json @@ -0,0 +1,5 @@ +{ + "source_file": "./raw_volume-zh/volume4/chapter1.tex", + "text": "平均值不等式的证明.\n一般地, 假设 $a_1, a_2, \\cdots, a_n$ 为 $n$ 个非负实数, 它们的算术平均值记为\n$$\nA_n=\\frac{a_1+a_2+\\cdots+a_n}{n},\n$$\n几何平均值记为\n$$\nG_n=\\left(a_1 a_2 \\cdots a_n\\right)^{\\frac{1}{n}}=\\sqrt[n]{a_1 a_2 \\cdots a_n} .\n$$\n算术平均值与几何平均值之间有如下的关系\n$$\n\\frac{a_1+a_2+\\cdots+a_n}{n} \\geqslant \\sqrt[n]{a_1 a_2 \\cdots a_n},\n$$\n即\n$$\nA_n \\geqslant G_n,\n$$\n当且仅当 $a_1=a_2=\\cdots=a_n$ 时,等号成立.\n证法八(调整法)\n(1)首先,如果 $a_1=a_2=\\cdots=a_n$, 那么必有 $A_n=G_n$. 下设这些数不全等, 不妨设 $a_1=\\min \\left\\{a_1, a_2, \\cdots, a_n\\right\\}, a_2=\\max \\left\\{a_1, a_2, \\cdots, a_n\\right\\}$, 则 $a_1< A_n0,\n\\end{aligned}\n$$\n则 $G_n \\leqslant G_n^1=\\sqrt[n]{b_1 b_2 \\cdots b_n}$.\n(2) 如果 $b_1=b_2=\\cdots=b_n$, 则命题成立.\n若不全等,则必有最大和最小者, 而且它们都不等于 $A_n$, 仿照上面作法, 可以得到 $c_1, c_2, \\cdots, c_n$, 这组数中, 有两个数为 $A_n$, 且 $A_n^2=\\frac{c_1+c_2+\\cdots+c_n}{n}=\\frac{b_1+b_2+\\cdots+b_n}{n}=A_n^1=A_n$, $G_n^2=\\sqrt[n]{c_1 c_2 \\cdots c_n} \\geqslant G_n^1 \\geqslant G_n$. 如果 $c_1=c_2=\\cdots=c_n$, 那么 $A_n^2=G_n^2$, 从而 $A_n=A_n^2 \\geqslant G_n$. 如果 $c_1, c_2, \\cdots, c_n$ 仍然不全相等,再按上述方法,进行第三次变换,所得到的新的数组中必有 3 个数都为 $A_n$. 这样下去,一定存在某个数 $m(2 \\leqslant m \\leqslant n)$ 使得.\n$$\nA_n=A_n^1=\\cdots=A_n^m, G_n \\leqslant G_n^1 \\leqslant G_n^2 \\leqslant \\cdots \\leqslant G_n^m, A_n^m=G_n^m,\n$$\n从而得 $A_n \\geqslant G_n$, 且只要 $a_1, a_2, \\cdots, a_n$ 不全相等, 必有 $A_n>G_n$. 故命题成立.\n注:调整法是证明不等式或求最值的一种有效方法, 特别是对那些当变量相等时取等号或取到最值的有关问题.", + "figures": [] +} \ No newline at end of file diff --git a/processed_dataset/text/0147.json b/processed_dataset/text/0147.json new file mode 100644 index 0000000000000000000000000000000000000000..67aa200e257865367b4d2f00d108a26b99d37e12 --- /dev/null +++ b/processed_dataset/text/0147.json @@ -0,0 +1,5 @@ +{ + "source_file": "./raw_volume-zh/volume4/chapter1.tex", + "text": "平均值不等式的证明.\n一般地, 假设 $a_1, a_2, \\cdots, a_n$ 为 $n$ 个非负实数, 它们的算术平均值记为\n$$\nA_n=\\frac{a_1+a_2+\\cdots+a_n}{n},\n$$\n几何平均值记为\n$$\nG_n=\\left(a_1 a_2 \\cdots a_n\\right)^{\\frac{1}{n}}=\\sqrt[n]{a_1 a_2 \\cdots a_n} .\n$$\n算术平均值与几何平均值之间有如下的关系\n$$\n\\frac{a_1+a_2+\\cdots+a_n}{n} \\geqslant \\sqrt[n]{a_1 a_2 \\cdots a_n},\n$$\n即\n$$\nA_n \\geqslant G_n,\n$$\n当且仅当 $a_1=a_2=\\cdots=a_n$ 时,等号成立.\n证法九 \n为了证明平均值不等式, 需要证明一个引理.\n引理 2 假设 $x 、 y$ 为正实数, $n$ 为正整数,则\n$$\nx^{n+1}+n y^{n+1} \\geqslant(n+1) y^n x .\n$$\n证明由于 $x 、 y$ 与 $x^k 、 y^k(1 \\leqslant k \\leqslant n)$ 同序, 所以\n$$\n(x-y)\\left(x^k-y^k\\right) \\geqslant 0 .\n$$\n于是\n$$\n\\begin{aligned}\n& x^{n+1}+n y^{n+1}-(n+1) x y^n \\\\\n= & x\\left(x^n-y^n\\right)-n y^n(x-y) \\\\\n= & (x-y)\\left[x\\left(x^{n-1}+x^{n-2} y+\\cdots+y^{n-1}\\right)-n y^n\\right] \\\\\n= & (x-y)\\left[\\left(x^n-y^n\\right)+\\left(x^{n-1}-y^{n-1}\\right) y+\\cdots+(x-y) y^{n-1}\\right] \\\\\n\\geqslant & 0,\n\\end{aligned}\n$$\n故引理 2 成立.\n现在, 我们利用引理 2 和数学归纳法证明平均值不等式.\n(1) 当 $n=2$ 时,已知结论成立.\n(2)假设对 $n=k$ (正整数 $k \\geqslant 2$ ) 时命题成立, 即对于 $a_i>0, i=1$, $2, \\cdots, k$, 有\n$$\n\\left(a_1 a_2 \\cdots a_k\\right)^{\\frac{1}{k}} \\leqslant \\frac{a_1+a_2+\\cdots+a_k}{k} .\n$$\n那么, 当 $n=k+1$ 时, 为了利用引理 2 , 令 $a_1 a_2 \\cdots a_k=y^{k(k+1)}, a_{k+1}=x^{k+1}, x$, $y \\geqslant 0$, 则由归纳假设和引理 2 , 得\n$$\n\\begin{aligned}\n& a_1+a_2+\\cdots+a_{k+1}-(k+1) G_{k+1} \\\\\n= & \\frac{k\\left(a_1+a_2+\\cdots+a_k\\right)}{k}+x^{k+1}-(k+1) y^k x \\\\\n\\geqslant & k \\sqrt[k]{a_1 a_2 \\cdots a_k}+x^{k+1}-(k+1) y^k x \\\\\n= & k y^{k+1}+x^{k+1}-(k+1) y^k x \\geqslant 0 .\n\\end{aligned}\n$$\n不难看出, 当且仅当所有的 $a_i$ 相等时等号成立,故命题成立.\n说明 (1) 值得注意的是, 像引理 2 这样的结论及其证明, 为我们证明和解决一般的不等式问题提供了方法和技巧.\n前面, 我们利用数学归纳法与不同的处理方式,证明了平均值不等式,当然, 还可以用其他的方法来证明.\n(2) 我们也可以用排序不等式证明引理 1 .", + "figures": [] +} \ No newline at end of file diff --git a/processed_dataset/text/0148.json b/processed_dataset/text/0148.json new file mode 100644 index 0000000000000000000000000000000000000000..7ca2ed6166f245476a0b874fe93bca9028325295 --- /dev/null +++ b/processed_dataset/text/0148.json @@ -0,0 +1,5 @@ +{ + "source_file": "./raw_volume-zh/volume4/chapter1.tex", + "text": "平均值不等式的证明.\n一般地, 假设 $a_1, a_2, \\cdots, a_n$ 为 $n$ 个非负实数, 它们的算术平均值记为\n$$\nA_n=\\frac{a_1+a_2+\\cdots+a_n}{n},\n$$\n几何平均值记为\n$$\nG_n=\\left(a_1 a_2 \\cdots a_n\\right)^{\\frac{1}{n}}=\\sqrt[n]{a_1 a_2 \\cdots a_n} .\n$$\n算术平均值与几何平均值之间有如下的关系\n$$\n\\frac{a_1+a_2+\\cdots+a_n}{n} \\geqslant \\sqrt[n]{a_1 a_2 \\cdots a_n},\n$$\n即\n$$\nA_n \\geqslant G_n,\n$$\n当且仅当 $a_1=a_2=\\cdots=a_n$ 时,等号成立.\n证法十(构造数列)\n令 $f(n)=n\\left(\\frac{a_1+a_2+\\cdots+a_n}{n}-\\sqrt[n]{a_1 a_2 \\cdots a_n}\\right)$, 如果能证明 $f(n)$ 关于 $n$ 是单调增加的, 即\n$$\nf(n) \\leqslant f(n+1), n \\geqslant 2 .\n$$\n那么, 由 $f(2) \\geqslant 0$, 得到 $f(n) \\geqslant f(2) \\geqslant 0$, 则平均值不等式成立.\n现在, 证明 $f(n)$ 的单调性.\n同证法九, 设 $a_1 a_2 \\cdots a_n=y^{n(n+1)}, a_{n+1}=x^{n+1}, x, y \\geqslant 0$, 则由引理 2 , 得\n$$\n\\begin{aligned}\n& f(n+1)-f(n) \\\\\n= & (n+1)\\left(\\frac{a_1+a_2+\\cdots+a_{n+1}}{n+1}-\\sqrt[n+1]{a_1 a_2 \\cdots a_{n+1}}\\right)\n\\end{aligned}\n$$\n$$\n\\begin{aligned}\n& -n\\left(\\frac{a_1+a_2+\\cdots+a_n}{n}-\\sqrt[n]{a_1 a_2 \\cdots a_n}\\right) \\\\\n= & a_{n+1}-(n+1) \\sqrt[n+1]{a_1 a_2 \\cdots a_{n+1}}+n \\sqrt[n]{a_1 a_2 \\cdots a_n} \\\\\n= & x^{n+1}-(n+1) y^n x+n y^{n+1} \\\\\n\\geqslant & 0 .\n\\end{aligned}\n$$\n这表明 $f(n+1) \\geqslant f(n)$.\n另外, 由于 $f(2) \\geqslant 0$, 则对任意 $n \\geqslant 2$, 得\n$$\nf(n) \\geqslant f(n-1) \\geqslant \\cdots \\geqslant f(2) \\geqslant 0 .\n$$\n不难看出, 当且仅当所有的 $a_i$ 相等时等号成立, 故平均值不等式成立.", + "figures": [] +} \ No newline at end of file diff --git a/processed_dataset/text/0149.json b/processed_dataset/text/0149.json new file mode 100644 index 0000000000000000000000000000000000000000..fb037104bc3dfb20ffd74839361cb557c5cbc126 --- /dev/null +++ b/processed_dataset/text/0149.json @@ -0,0 +1,5 @@ +{ + "source_file": "./raw_volume-zh/volume4/chapter1.tex", + "text": "平均值不等式的证明.\n一般地, 假设 $a_1, a_2, \\cdots, a_n$ 为 $n$ 个非负实数, 它们的算术平均值记为\n$$\nA_n=\\frac{a_1+a_2+\\cdots+a_n}{n},\n$$\n几何平均值记为\n$$\nG_n=\\left(a_1 a_2 \\cdots a_n\\right)^{\\frac{1}{n}}=\\sqrt[n]{a_1 a_2 \\cdots a_n} .\n$$\n算术平均值与几何平均值之间有如下的关系\n$$\n\\frac{a_1+a_2+\\cdots+a_n}{n} \\geqslant \\sqrt[n]{a_1 a_2 \\cdots a_n},\n$$\n即\n$$\nA_n \\geqslant G_n,\n$$\n当且仅当 $a_1=a_2=\\cdots=a_n$ 时,等号成立.\n证法十一\n为了证明平均值不等式, 首先证明另一个不等式, 即引理 3 如果 $x_k \\geqslant 0$, 且 $x_k \\geqslant x_{k-1}(k=2,3, \\cdots, n)$, 则\n$$\nx_n^n \\geqslant x_1\\left(2 x_2-x_1\\right)\\left(3 x_3-2 x_2\\right) \\cdots\\left[n x_n-(n-1) x_{n-1}\\right],\n$$\n当且仅当 $x_1=x_2=\\cdots=x_n$ 时等号成立.\n证明因为 $x_k \\geqslant x_{k-1}$, 则\n$$\nx_k^{k-1}+x_k^{k-2} x_{k-1}+\\cdots+x_{k-1}^{k-1} \\geqslant k x_{k-1}^{k-1},\n$$\n所以\n$$\n\\begin{aligned}\nx_k^k-x_{k-1}^k & =\\left(x_k-x_{k-1}\\right)\\left(x_k^{k-1}+x_k^{k-2} x_{k-1}+\\cdots+x_{k-1}^{k-1}\\right) \\\\\n& \\geqslant k x_{k-1}^{k-1}\\left(x_k-x_{k-1}\\right) .\n\\end{aligned}\n$$\n即\n$$\nx_k^k \\geqslant x_{k-1}^{k-1}\\left[k x_k-(k-1) x_{k-1}\\right](k=1,2, \\cdots, n),\n$$\n当且仅当 $x_k=x_{k-1}$ 时等号成立.\n所以\n$$\nx_n^n=x_1 \\frac{x_2^2}{x_1} \\frac{x_3^3}{x_2^2} \\cdots \\frac{x_n^n}{x_{n-1}^{n-1}} \\geqslant x_1\\left(2 x_2-x_1\\right)\\left(3 x_3-2 x_2\\right) \\cdots\\left[n x_n-(n-1) x_{n-1}\\right] .\n$$\n现在利用引理 3 证明平均值不等式.\n不妨假设 $a_n \\geqslant a_{n-1} \\geqslant \\cdots \\geqslant a_2 \\geqslant a_1>0$. 由 $A_k=\\frac{a_1+a_2+\\cdots+a_k}{k}$, 则 $A_k \\geqslant A_{k-1}>0(k=2,3, \\cdots, n)$, 且 $k A_k-(k-1) A_{k-1}=a_k$. 由引理 3, 得\n$$\nA_n^n \\geqslant a_1 a_2 \\cdots a_n,\n$$\n即 $A_n \\geqslant G_n$. 当且仅当 $A_1=A_2=\\cdots=A_n$, 即 $a_1=a_2=\\cdots=a_n$ 时等号成立.", + "figures": [] +} \ No newline at end of file diff --git a/processed_dataset/text/0150.json b/processed_dataset/text/0150.json new file mode 100644 index 0000000000000000000000000000000000000000..1de2c0d0ee944716c2cdda4134c82857c0e61d05 --- /dev/null +++ b/processed_dataset/text/0150.json @@ -0,0 +1,5 @@ +{ + "source_file": "./raw_volume-zh/volume4/chapter1.tex", + "text": "平均值不等式的证明.\n一般地, 假设 $a_1, a_2, \\cdots, a_n$ 为 $n$ 个非负实数, 它们的算术平均值记为\n$$\nA_n=\\frac{a_1+a_2+\\cdots+a_n}{n},\n$$\n几何平均值记为\n$$\nG_n=\\left(a_1 a_2 \\cdots a_n\\right)^{\\frac{1}{n}}=\\sqrt[n]{a_1 a_2 \\cdots a_n} .\n$$\n算术平均值与几何平均值之间有如下的关系\n$$\n\\frac{a_1+a_2+\\cdots+a_n}{n} \\geqslant \\sqrt[n]{a_1 a_2 \\cdots a_n},\n$$\n即\n$$\nA_n \\geqslant G_n,\n$$\n当且仅当 $a_1=a_2=\\cdots=a_n$ 时,等号成立.\n证法十二(函数方法)\n引理 4 如果函数 $f(x):(a, b) \\rightarrow \\mathbf{R}$ 满足\n$$\nf\\left(\\frac{x+y}{2}\\right)>\\frac{f(x)+f(y)}{2}, x, y \\in(a, b), x \\neq y, \\label{eq1}\n$$\n那么\n$$\nf\\left(\\frac{x_1+x_2+\\cdots+x_n}{n}\\right)>\\frac{f\\left(x_1\\right)+f\\left(x_2\\right)+\\cdots+f\\left(x_n\\right)}{n}, \\label{eq2}\n$$\n其中 $x_i \\in(a, b)$, 且至少有一对 $(i, j)$, 使 $x_i \\neq x_j$.\n证明对 $n$ 用归纳法.\n当 $n=1,2$ 时,结论显然成立.\n设当 $n=k$ 时结论成立.\n对于 $n=k+1$, 有\n$$\nA_{k+1}=\\frac{a_1+a_2+\\cdots+a_k}{2 k}+\\frac{a_{k+1}+(k-1) A_{k+1}}{2 k},\n$$\n并记\n$$\nB=\\frac{a_{k+1}+(k-1) A_{k+1}}{k},\n$$\n则\n$$\n\\begin{aligned}\nf\\left(A_{k+1}\\right)= & f\\left(\\frac{A_k+B}{2}\\right) \\\\\n\\geqslant & \\frac{1}{2}\\left[f\\left(A_k\\right)+f(B)\\right] \\\\\n\\geqslant & \\frac{1}{2}\\left\\{\\frac{1}{k}\\left[f\\left(a_1\\right)+f\\left(a_2\\right)+\\cdots+f\\left(a_k\\right)\\right]\\right. \\\\\n& \\left.+\\frac{1}{k}\\left[f\\left(a_{k+1}\\right)+(k-1) f\\left(A_{k+1}\\right)\\right]\\right\\} .\n\\end{aligned}\n$$\n所以\n$$\nf\\left(\\frac{a_1+a_2+\\cdots+a_{k+1}}{k+1}\\right) \\geqslant \\frac{f\\left(a_1\\right)+f\\left(a_2\\right)+\\cdots+f\\left(a_{k+1}\\right)}{k+1} .\n$$\n我们称满足 \\ref{eq1} 式的函数为凹函数 (可以证明, 如果函数 $f$ 二阶可导, 则当 $f^{\\prime \\prime}(x)<0$ 时, $f$ 为凹函数). 特别的, 不难验证函数 $f(x)=\\ln x$ 在 (0, $+\\infty)$ 上是凹函数, 于是, 对 $a_i \\in(0,+\\infty), i=1,2, \\cdots, n$, 我们有\n$$\nf\\left(\\frac{a_1+a_2+\\cdots+a_n}{n}\\right) \\geqslant \\frac{f\\left(a_1\\right)+f\\left(a_2\\right)+\\cdots+f\\left(a_n\\right)}{n},\n$$\n从而\n$$\n\\ln \\frac{a_1+a_2+\\cdots+a_n}{n} \\geqslant \\ln \\left(a_1 a_2 \\cdots a_n\\right)^{\\frac{1}{n}} .\n$$\n由对数函数的单调性, 得\n$$\n\\frac{a_1+a_2+\\cdots+a_n}{n} \\geqslant\\left(a_1 a_2 \\cdots a_n\\right)^{\\frac{1}{n}},\n$$\n故命题成立.\n下面验证 $\\ln x$ 为凹函数.\n对任意 $x, y, x \\neq y$, 要使得: $f\\left(\\frac{x+y}{2}\\right)> \\frac{f(x)+f(y)}{2}$, 即\n$$\n\\ln \\frac{x+y}{2}>\\frac{\\ln (x)+\\ln (y)}{2}\n$$\n等价于\n$$\n\\ln \\frac{x+y}{2} \\geqslant \\ln (x y)^{\\frac{1}{2}} .\n$$\n由函数的单调性, 等价于\n$$\n\\frac{x+y}{2} \\geqslant(x y)^{\\frac{1}{2}}\n$$\n这个可以由 $(\\sqrt{x}-\\sqrt{y})^2>0$ 直接导出.\n另外, 由凹函数方法, 设 $p>0, q>0$, 且 $\\frac{1}{p}+\\frac{1}{q}=1$, 由于函数 $f(x)= \\ln x, x \\in \\mathbf{R}^{+}$为凹函数, 则对 $x, y>0$, 有\n$$\n\\begin{gathered}\n\\frac{1}{p} \\ln x+\\frac{1}{q} \\ln y \\leqslant \\ln \\left(\\frac{1}{p} x+\\frac{1}{q} y\\right), \\\\\nx^{\\frac{1}{p}} y^{\\frac{1}{q}} \\leqslant \\frac{1}{p} x+\\frac{1}{q} y .\n\\end{gathered}\n$$\n即\n$$\nx^{\\frac{1}{p}} y^{\\frac{1}{q}} \\leqslant \\frac{1}{p} x+\\frac{1}{q} y .\n$$\n等号成立的充分必要条件是 $x=y$.\n这个不等式称为 Young 不等式.\n注:引理 4 中的不等式 \\ref{eq2}, 称为 Jensen 不等式, 它的一般形式为设 $y=f(x), x \\in(a, b)$ 为凹函数, 则对任意 $x_i \\in(a, b)(i=1,2, \\cdots$, $n)$, 我们有\n$$\n\\frac{1}{p_1} f\\left(x_1\\right)+\\frac{1}{p_2} f\\left(x_2\\right)+\\cdots+\\frac{1}{p_n} f\\left(x_n\\right) \\leqslant f\\left(\\frac{x_1}{p_1}+\\frac{x_2}{p_2}+\\cdots+\\frac{x_n}{p_n}\\right) \\text {. }\n$$\n其中 $p_i>0(i=1,2, \\cdots, n)$ 且 $\\sum_{i=1}^n \\frac{1}{p_i}=1$.\n在这部分,我们利用不同的方法证明了平均值不等式成立.\n在证明过程中,利用了各种技巧和方法.", + "figures": [] +} \ No newline at end of file diff --git a/processed_dataset/text/0151.json b/processed_dataset/text/0151.json new file mode 100644 index 0000000000000000000000000000000000000000..0ab5d908bfed778eaa0d4a0362e8e09a43d054e5 --- /dev/null +++ b/processed_dataset/text/0151.json @@ -0,0 +1,5 @@ +{ + "source_file": "./raw_volume-zh/volume4/chapter2-1.tex", + "text": "平均值不等式的应用.\n2. 1 平均值不等式在不等式证明中的应用.\n下面举例说明平均值不等式在证明各种竞赛问题中的应用.\n在证明过程中,应用灵活,具有较高的技巧性.", + "figures": [] +} \ No newline at end of file diff --git a/processed_dataset/text/0152.json b/processed_dataset/text/0152.json new file mode 100644 index 0000000000000000000000000000000000000000..74337945958027216bec56094b602204f1f0075c --- /dev/null +++ b/processed_dataset/text/0152.json @@ -0,0 +1,5 @@ +{ + "source_file": "./raw_volume-zh/volume4/chapter2-2.tex", + "text": "2.2 平均值不等式在求极值中的应用.\n不等式在求极值中起着重要的作用, 在利用平均值不等式求极值的过程中, 要注意\"缩\"或 \"放\" 的结果是否为常数 (通常是和与积), 同时必须指出等号成立的条件.", + "figures": [] +} \ No newline at end of file diff --git a/processed_dataset/text/0153.json b/processed_dataset/text/0153.json new file mode 100644 index 0000000000000000000000000000000000000000..91485f64ed4342f9eebba16c430a630704623172 --- /dev/null +++ b/processed_dataset/text/0153.json @@ -0,0 +1,5 @@ +{ + "source_file": "./raw_volume-zh/volume4/chapter2-3.tex", + "text": "2.3 平均值不等式在几何不等式中的应用.\n对于几何中出现的不等式证明, 常用的方法有: 几何方法、代数方法和三角方法, 当然, 我们不能将它们截然地分开, 常常是要综合地运用各种知识.\n如果采用代数方法证明几何命题, 那么, 灵活运用平均值不等式和柯西不等式,对解决问题将有极大的帮助.", + "figures": [] +} \ No newline at end of file diff --git a/processed_dataset/text/0154.json b/processed_dataset/text/0154.json new file mode 100644 index 0000000000000000000000000000000000000000..46bd5730c52a13af3029e4c1a1831f147640b001 --- /dev/null +++ b/processed_dataset/text/0154.json @@ -0,0 +1,5 @@ +{ + "source_file": "./raw_volume-zh/volume4/chapter2-4.tex", + "text": "2.4 平均值不等式的变形及应用.\n对于平均值不等式,有各种不同的变形和推广, 由于这些问题可以包括在命题的证明和讨论中, 这里就不展开讨论了.\n对任意正数 $a_1, a_2, \\cdots, a_n$, 由平均值不等式, 得\n$$\n\\sum_{i=1}^n a_i \\cdot \\sum_{i=1}^n \\frac{1}{a_i} \\geqslant n \\sqrt[n]{a_1 a_2 \\cdots a_n} \\cdot n \\sqrt[n]{\\frac{1}{a_1} \\cdot \\frac{1}{a_2} \\cdot \\cdots \\cdot \\frac{1}{a_n}}=n^2 .\n$$\n从而\n$$\n\\frac{\\sum_{i=1}^n a_i}{n} \\geqslant \\frac{n}{\\sum_{i=1}^n \\frac{1}{a_i}} .\n$$\n令 $H_n=\\frac{n}{\\frac{1}{a_1}+\\frac{1}{a_2}+\\cdots+\\frac{1}{a_n}}$, 则称 $H_n$ 为 $n$ 个正实数 $a_1, a_2, \\cdots, a_n$ 的调和平均值.\n由于 $\\frac{x_1+x_2+\\cdots+x_n}{n} \\geqslant \\sqrt[n]{x_1 x_2 \\cdots x_n}$, 令 $x_i=\\frac{1}{a_i}$, 则\n$$\n\\frac{n}{\\frac{1}{a_1}+\\frac{1}{a_2}+\\cdots+\\frac{1}{a_n}} \\leqslant \\sqrt[n]{a_1 a_2 \\cdots a_n}\n$$\n即 $H_n \\leqslant G_n$, 调和平均值不大于几何平均值.\n对任意实数 $a_1, a_2, \\cdots, a_n$, 有\n$$\nn \\sum_{i=1}^n a_i^2-\\left(\\sum_{i=1}^n a_i\\right)^2=\\sum_{1 \\leqslant i0, \\lambda_i>0(i=1,2, \\cdots, n)$, 且 $\\prod_{i=1}^n \\lambda_i=1$ 时, 我们有\n$$\n\\frac{1}{n} \\sum_{i=1}^n \\lambda_i a_i \\geqslant \\sqrt[n]{a_1 a_2 \\cdots a_n} .\n$$", + "figures": [] +} \ No newline at end of file diff --git a/processed_dataset/text/0156.json b/processed_dataset/text/0156.json new file mode 100644 index 0000000000000000000000000000000000000000..6c9689ac611f909c2c668c49d50ab75acbfd659f --- /dev/null +++ b/processed_dataset/text/0156.json @@ -0,0 +1,5 @@ +{ + "source_file": "./raw_volume-zh/volume4/chapter3.tex", + "text": "前面我们介绍了平均值不等式及其在不等式证明中的一些应用, 同时, 也介绍了证明不等式的一些方法和技巧.\n但是, 任何一个结论的使用, 都有它的局限性, 平均值不等式也是如此.\n在不等式的证明过程中, 要求我们了解不等式的性质和证明不等式的常用方法, 需要掌握一些基本的结论和重要的定理, 并能灵活地应用有关知识.\n在这里, 我们将再介绍另一个重要的基本不等式, 即柯西不等式, 与平均值不等式类似, 它的表达形式简单, 它的证明方法多样,在应用中具有较强的灵活性和技巧性.\n柯西不等式及其证明.\n设 $a_1, a_2, \\cdots, a_n$ 及 $b_1, b_2, \\cdots, b_n$ 为任意实数, 则\n$$\n\\left(a_1 b_1+a_2 b_2+\\cdots+a_n b_n\\right)^2 \\leqslant\\left(a_1^2+a_2^2+\\cdots+a_n^2\\right)\\left(b_1^2+b_2^2+\\cdots+b_n^2\\right),\n$$\n当且仅当 $\\frac{a_1}{b_1}=\\frac{a_2}{b_2}=\\cdots=\\frac{a_n}{b_n}$ (规定 $a_i=0$ 时, $b_i=0$ )时等号成立.\n柯西不等式的证明方法很多,这里我们选择其中一些简单和具有一定技巧的证明.\n证法一不妨假设 $A_n=\\sum_{i=1}^n a_i^2 \\neq 0, C_n=\\sum_{i=1}^n b_i^2 \\neq 0$, 令 $x_i=\\frac{a_i}{\\sqrt{A_n}}, y_i=\\frac{b_i}{\\sqrt{C_n}}$,\n则\n$$\n\\sum_{i=1}^n x_i^2=\\sum_{i=1}^n y_i^2=1\n$$\n则原不等式等价于\n$$\nx_1 y_1+x_2 y_2+\\cdots+x_n y_n \\leqslant 1,\n$$\n即\n$$\n2\\left(x_1 y_1+x_2 y_2+\\cdots+x_n y_n\\right) \\leqslant x_1^2+x_2^2+\\cdots+x_n^2+y_1^2+y_2^2+\\cdots+y_n^2 \\text {. }\n$$\n又等价于\n$$\n\\left(x_1-y_1\\right)^2+\\left(x_2-y_2\\right)^2+\\cdots+\\left(x_n-y_n\\right)^2 \\geqslant 0 .\n$$\n这个不等式显然成立, 且等号成立的充要条件为 $x_i=y_i(i=1,2, \\cdots$, $n)$, 从而原不等式成立,且等号成立的充要条件是\n$$\nb_i=k a_i\\left(k=\\frac{\\sqrt{C_n}}{\\sqrt{A_n}}\\right) .\n$$", + "figures": [] +} \ No newline at end of file diff --git a/processed_dataset/text/0157.json b/processed_dataset/text/0157.json new file mode 100644 index 0000000000000000000000000000000000000000..b7b89437bdf5891e8c4e97d0046469427ef1742d --- /dev/null +++ b/processed_dataset/text/0157.json @@ -0,0 +1,5 @@ +{ + "source_file": "./raw_volume-zh/volume4/chapter3.tex", + "text": "柯西不等式及其证明.\n设 $a_1, a_2, \\cdots, a_n$ 及 $b_1, b_2, \\cdots, b_n$ 为任意实数, 则\n$$\n\\left(a_1 b_1+a_2 b_2+\\cdots+a_n b_n\\right)^2 \\leqslant\\left(a_1^2+a_2^2+\\cdots+a_n^2\\right)\\left(b_1^2+b_2^2+\\cdots+b_n^2\\right),\n$$\n当且仅当 $\\frac{a_1}{b_1}=\\frac{a_2}{b_2}=\\cdots=\\frac{a_n}{b_n}$ (规定 $a_i=0$ 时, $b_i=0$ )时等号成立.\n证法二(比值法)\n按上述证明方法和记号, 不妨假设 $A_n \\neq 0, C_n \\neq 0$, 令 $x_i=\\frac{\\left|a_i\\right|}{\\sqrt{A_n}}, y_i= \\frac{\\left|b_i\\right|}{\\sqrt{C_n}}$, 则\n$$\n\\sum_{i=1}^n x_i^2=\\sum_{i=1}^n y_i^2=1\n$$\n由于 $\\begin{aligned} \\frac{\\left|\\sum_{i=1}^n a_i b_i\\right|}{\\sqrt{A_n} \\cdot \\sqrt{C_n}} & \\leqslant \\sum_{i=1}^n x_i y_i \\leqslant \\sum_{i=1}^n \\frac{1}{2}\\left(x_i^2+y_i^2\\right) \\\\ & =\\frac{1}{2}\\left(\\sum_{i=1}^n x_i^2+\\sum_{i=1}^n y_i^2\\right)=1,\\end{aligned}$\n且等号成立当且仅当\n$$\n\\begin{gathered}\n\\left|\\sum_{i=1}^n a_i b_i\\right|=\\sum_{i=1}^n\\left|a_i b_i\\right|, \\\\\n\\frac{a_i^2}{\\sum_{i=1}^n a_i^2}=\\frac{b_i^2}{\\sum_{i=1}^n b_i^2}\n\\end{gathered}\n$$\n由第一个条件表明 $a_i b_i \\geqslant 0, i=1,2, \\cdots, n$, 即 $a_i$ 与 $b_i(i=1,2, \\cdots$, $n$ ) 同号.\n第二个条件成立的充分必要条件是 $\\frac{a_i^2}{b_i^2}=\\frac{A_n}{C_n}$, 即 $\\frac{\\left|a_i\\right|}{\\left|b_i\\right|}$ 为常数.\n由于 $a_i$ 与 $b_i(i=1,2, \\cdots, n)$ 同号, 从而命题成立.", + "figures": [] +} \ No newline at end of file diff --git a/processed_dataset/text/0158.json b/processed_dataset/text/0158.json new file mode 100644 index 0000000000000000000000000000000000000000..1490a5a106fe4c677964d9ee5197dbe4d0c73811 --- /dev/null +++ b/processed_dataset/text/0158.json @@ -0,0 +1,5 @@ +{ + "source_file": "./raw_volume-zh/volume4/chapter3.tex", + "text": "柯西不等式及其证明.\n设 $a_1, a_2, \\cdots, a_n$ 及 $b_1, b_2, \\cdots, b_n$ 为任意实数, 则\n$$\n\\left(a_1 b_1+a_2 b_2+\\cdots+a_n b_n\\right)^2 \\leqslant\\left(a_1^2+a_2^2+\\cdots+a_n^2\\right)\\left(b_1^2+b_2^2+\\cdots+b_n^2\\right),\n$$\n当且仅当 $\\frac{a_1}{b_1}=\\frac{a_2}{b_2}=\\cdots=\\frac{a_n}{b_n}$ (规定 $a_i=0$ 时, $b_i=0$ )时等号成立.\n证法三 (比值法, 类似证法二)\n令 $A_n=a_1^2+a_2^2+\\cdots+a_n^2, B_n=a_1 b_1+a_2 b_2+\\cdots+a_n b_n, C_n=b_1^2+ b_2^2+\\cdots+b_n^2$, 则\n$$\n\\begin{gathered}\n\\frac{A_n C_n}{B_n^2}+1=\\sum_{i=1}^n \\frac{a_i^2 C_n}{B_n^2}+\\sum_{i=1}^n \\frac{b_i^2}{C_n} \\\\\n=\\sum_{i=1}^n\\left(\\frac{a_i^2 C_n}{B_n^2}+\\frac{b_i^2}{C_n}\\right) \\\\\n\\geqslant \\sum_{i=1}^n 2 \\cdot \\frac{a_i b_i}{B_n}=2, \\\\\n\\frac{A_n C_n}{B_n^2}+1 \\geqslant 2, \\\\\nB_n^2 \\leqslant A_n C_n .\n\\end{gathered}\n$$\n所以即\n$B_n^2 \\leqslant A_n C_n$.\n等号成立当且仅当 $\\frac{a_i}{b_i}(i=1,2, \\cdots, n)$ 为一个常数.\n注:(1)这两个证明方法比较简单, 但是对于不等式的证明来讲, 怎样人手是十分重要的.\n比值法是证明不等式的一种常用、基本的方法.\n(2)上述两种方法也称为标准化方法,这个方法可以简化许多不等式的证明.\n在前面我们也使用过.\n如为了证明 $G_n \\leqslant A_n$, 令 $y_i=\\frac{a_i}{G_n}$, 则问题化为在条件 $y_1 y_2 \\cdots y_n=1\\left(y_i>0\\right)$ 下, 证明 $\\sum_{i=1}^n y_i \\geqslant n$.", + "figures": [] +} \ No newline at end of file diff --git a/processed_dataset/text/0159.json b/processed_dataset/text/0159.json new file mode 100644 index 0000000000000000000000000000000000000000..0eb6acde3a5904a596b538b7d59685460aeabf6b --- /dev/null +++ b/processed_dataset/text/0159.json @@ -0,0 +1,5 @@ +{ + "source_file": "./raw_volume-zh/volume4/chapter3.tex", + "text": "柯西不等式及其证明.\n设 $a_1, a_2, \\cdots, a_n$ 及 $b_1, b_2, \\cdots, b_n$ 为任意实数, 则\n$$\n\\left(a_1 b_1+a_2 b_2+\\cdots+a_n b_n\\right)^2 \\leqslant\\left(a_1^2+a_2^2+\\cdots+a_n^2\\right)\\left(b_1^2+b_2^2+\\cdots+b_n^2\\right),\n$$\n当且仅当 $\\frac{a_1}{b_1}=\\frac{a_2}{b_2}=\\cdots=\\frac{a_n}{b_n}$ (规定 $a_i=0$ 时, $b_i=0$ )时等号成立.\n证法四 (归纳法)\n众所周知, 归纳法是证明不等式的一种强有力和常用的方法, 这里, 利用归纳法证明一个更强的结论, 即\n$$\n\\sum_{i=1}^n\\left|a_i b_i\\right| \\leqslant \\sqrt{\\sum_{i=1}^n a_i^2} \\sqrt{\\sum_{i=1}^n b_i^2} .\n$$\n(1)当 $n=2$ 时,\n$$\n\\begin{aligned}\n\\left(a_1 b_1+a_2 b_2\\right)^2 & =a_1^2 b_1^2+2 a_1 b_1 a_2 b_2+a_2^2 b_2^2 \\\\\n& \\leqslant a_1^2 b_1^2+a_1^2 b_2^2+a_2^2 b_1^2+a_2^2 b_2^2 \\\\\n& =\\left(a_1^2+a_2^2\\right)\\left(b_1^2+b_2^2\\right),\n\\end{aligned}\n$$\n且等号成立当且仅当 $\\frac{a_1}{b_1}=\\frac{a_2}{b_2}$, 命题成立.\n(2)假设当 $n==k$ 时命题成立, 那么对于 $n=k+1$, 由归纳假设,\n$$\n\\begin{aligned}\n& \\sqrt{\\sum_{i=1}^{k+1} a_i^2} \\cdot \\sqrt{\\sum_{i=1}^{k+1} b_i^2} \\\\\n= & \\sqrt{\\sum_{i=1}^k a_i^2+a_{k+1}^2} \\cdot \\sqrt{\\sum_{i=1}^k b_i^2+b_{k+1}^2}\n\\end{aligned}\n$$\n$$\n\\begin{aligned}\n& \\geqslant \\sqrt{\\sum_{i=1}^k a_i^2} \\cdot \\sqrt{\\sum_{i=1}^k b_i^2}+\\left|a_{k+1} b_{k+1}\\right| \\\\\n& \\geqslant \\sum_{i=1}^k\\left|a_i b_i\\right|+\\left|a_{k+1} b_{k+1}\\right|=\\sum_{i=1}^{k+1}\\left|a_i b_i\\right| .\n\\end{aligned}\n$$\n所以对一切的 $n$ 命题成立.\n不难得到等号成立的充分必要条件.", + "figures": [] +} \ No newline at end of file diff --git a/processed_dataset/text/0160.json b/processed_dataset/text/0160.json new file mode 100644 index 0000000000000000000000000000000000000000..949d057124048ac6106e6f4e16f6fdf3b63bfc81 --- /dev/null +++ b/processed_dataset/text/0160.json @@ -0,0 +1,5 @@ +{ + "source_file": "./raw_volume-zh/volume4/chapter3.tex", + "text": "柯西不等式及其证明.\n设 $a_1, a_2, \\cdots, a_n$ 及 $b_1, b_2, \\cdots, b_n$ 为任意实数, 则\n$$\n\\left(a_1 b_1+a_2 b_2+\\cdots+a_n b_n\\right)^2 \\leqslant\\left(a_1^2+a_2^2+\\cdots+a_n^2\\right)\\left(b_1^2+b_2^2+\\cdots+b_n^2\\right),\n$$\n当且仅当 $\\frac{a_1}{b_1}=\\frac{a_2}{b_2}=\\cdots=\\frac{a_n}{b_n}$ (规定 $a_i=0$ 时, $b_i=0$ )时等号成立.\n证法五 (归纳与综合法)\n(1)当 $n=2$ 时,有\n$$\n\\begin{aligned}\n\\left(a_1 b_1+a_2 b_2\\right)^2 & =a_1^2 b_1^2+2 a_1 b_1 a_2 b_2+a_2^2 b_2^2 \\\\\n& \\leqslant a_1^2 b_1^2+a_1^2 b_2^2+a_2^2 b_1^2+a_2^2 b_2^2 \\\\\n& =\\left(a_1^2+a_2^2\\right)\\left(b_1^2+b_2^2\\right),\n\\end{aligned}\n$$\n且等号成立当且仅当 $\\frac{a_1}{b_1}=\\frac{a_2}{b_2}$, 命题成立.\n(2)假设当 $n=k$ 时命题成立.\n对于 $n=k+1$, 令 $A_k=a_1^2+a_2^2+\\cdots+a_k^2$, $B_k=a_1 b_1+a_2 b_2+\\cdots+a_k b_k, C_k=b_1^2+b_2^2+\\cdots+b_k^2$, 则由归纳假设\n$$\nB_k^2 \\leqslant A_k C_k .\n$$\n由于我们要证明\n$$\n\\begin{aligned}\n& \\left(a_1 b_1+a_2 b_2+\\cdots+a_k b_k+a_{k+1} b_{k+1}\\right)^2 \\\\\n\\leqslant & \\left(a_1^2+a_2^2+\\cdots+a_k^2+a_{k+1}^2\\right)\\left(b_1^2+b_2^2+\\cdots+b_k^2+b_{k+1}^2\\right),\n\\end{aligned}\n$$\n等价于证明\n$$\n\\begin{aligned}\n& \\left(B_k+a_{k+1} b_{k+1}\\right)^2 \\leqslant\\left(A_k+a_{k+1}^2\\right)\\left(C_k+b_{k+1}^2\\right) \\\\\n\\Leftrightarrow & B_k^2+2 B_k a_{k+1} b_{k+1} \\leqslant A_k C_k+A_k b_{k+1}^2+C_k a_{k+1}^2 \\\\\n\\Leftrightarrow & A_k C_k-B_k^2+A_k b_{k+1}^2+C_k a_{k+1}^2-2 B_k a_{k+1} b_{k+1} \\geqslant 0 \\\\\n\\Leftrightarrow & A_k C_k-B_k^2+\\left(\\sqrt{A_k} b_{k+1}-\\sqrt{C_k} a_{k+1}\\right)^2+2\\left(\\sqrt{A_k} \\sqrt{C_k}-B_k\\right) a_{k+1} b_{k+1} \\geqslant 0 .\n\\end{aligned}\n$$\n由归纳假设, 上述不等式成立, 且等式成立当且仅当 $\\frac{a_1}{b_1}=\\frac{a_2}{b_2}=\\cdots= \\frac{a_{k+1}}{b_{k+1}}$, 故对任意 $n \\geqslant 1$, 命题成立.", + "figures": [] +} \ No newline at end of file diff --git a/processed_dataset/text/0161.json b/processed_dataset/text/0161.json new file mode 100644 index 0000000000000000000000000000000000000000..964943335db9495a277cb54a88f8bb1782278ba1 --- /dev/null +++ b/processed_dataset/text/0161.json @@ -0,0 +1,5 @@ +{ + "source_file": "./raw_volume-zh/volume4/chapter3.tex", + "text": "柯西不等式及其证明.\n设 $a_1, a_2, \\cdots, a_n$ 及 $b_1, b_2, \\cdots, b_n$ 为任意实数, 则\n$$\n\\left(a_1 b_1+a_2 b_2+\\cdots+a_n b_n\\right)^2 \\leqslant\\left(a_1^2+a_2^2+\\cdots+a_n^2\\right)\\left(b_1^2+b_2^2+\\cdots+b_n^2\\right),\n$$\n当且仅当 $\\frac{a_1}{b_1}=\\frac{a_2}{b_2}=\\cdots=\\frac{a_n}{b_n}$ (规定 $a_i=0$ 时, $b_i=0$ )时等号成立.\n证法六 (归纳法和平均值不等式)\n(1)当 $n=2$ 时,有\n$$\n\\begin{aligned}\n\\left(a_1 b_1+a_2 b_2\\right)^2 & =a_1^2 b_1^2+2 a_1 b_1 a_2 b_2+a_2^2 b_2^2 \\\\\n& \\leqslant a_1^2 b_1^2+a_1^2 b_2^2+a_2^2 b_1^2+a_2^2 b_2^2 \\\\\n& =\\left(a_1^2+a_2^2\\right)\\left(b_1^2+b_2^2\\right),\n\\end{aligned}\n$$\n即命题成立.\n(2)假设当 $n=k$ 时命题成立.\n对于 $n=k+1$, 由于\n$$\n\\begin{aligned}\n& \\left(a_1 b_1+a_2 b_2+\\cdots+a_k b_k+a_{k+1} b_{k+1}\\right)^2 \\\\\n= & \\left(a_1 b_1+a_2 b_2+\\cdots+a_k b_k\\right)^2 \\\\\n& +2\\left(a_1 b_1+a_2 b_2+\\cdots+a_k b_k\\right) a_{k+1} b_{k+1}+a_{k+1}^2 b_{k+1}^2 .\n\\end{aligned}\n$$\n由平均值不等式, 得\n$$\n\\begin{aligned}\n& 2\\left(a_1 b_1+a_2 b_2+\\cdots+a_k b_k\\right) a_{k+1} b_{k+1} \\\\\n\\leqslant & a_{k+1}^2\\left(b_1^2+b_2^2+\\cdots+b_k^2\\right)+b_{k+1}^2\\left(a_1^2+a_2^2+\\cdots+a_k^2\\right) .\n\\end{aligned}\n$$\n由归纳假设,得\n$$\n\\begin{aligned}\n& \\left(a_1 b_1+a_2 b_2+\\cdots+a_k b_k+a_{k+1} b_{k+1}\\right)^2 \\\\\n= & \\left(a_1 b_1+a_2 b_2+\\cdots+a_k b_k\\right)^2+2\\left(a_1 b_1+a_2 b_2+\\cdots+a_k b_k\\right) a_{k+1} b_{k+1}+a_{k+1}^2 b_{k+1}^2 \\\\\n\\leqslant & \\left(a_1 b_1+a_2 b_2+\\cdots+a_k b_k\\right)^2+a_{k+1}^2\\left(b_1^2+b_2^2+\\cdots+b_k^2\\right) \\\\\n& +b_{k+1}^2\\left(a_1^2+a_2^2+\\cdots+a_k^2\\right)+a_{k+1}^2 b_{k+1}^2 \\\\\n= & \\left(a_1^2+a_2^2+\\cdots+a_{k+1}^2\\right)\\left(b_1^2+b_2^2+\\cdots+b_{k+1}^2\\right) .\n\\end{aligned}\n$$\n结合平均值不等式等号成立的条件, 不难得到柯西不等式等号成立的充要条件,故命题成立.\n注:(1)在上述的证明中, 我们反复利用了平均值不等式.\n(2)上述几种证明均用归纳法, 由于证明过程中, 对表达式的处理的不同,所以难易程度也就不同.", + "figures": [] +} \ No newline at end of file diff --git a/processed_dataset/text/0162.json b/processed_dataset/text/0162.json new file mode 100644 index 0000000000000000000000000000000000000000..f9708b2aa859c3d7938f18d5cdd54cf9072d34a8 --- /dev/null +++ b/processed_dataset/text/0162.json @@ -0,0 +1,5 @@ +{ + "source_file": "./raw_volume-zh/volume4/chapter3.tex", + "text": "柯西不等式及其证明.\n设 $a_1, a_2, \\cdots, a_n$ 及 $b_1, b_2, \\cdots, b_n$ 为任意实数, 则\n$$\n\\left(a_1 b_1+a_2 b_2+\\cdots+a_n b_n\\right)^2 \\leqslant\\left(a_1^2+a_2^2+\\cdots+a_n^2\\right)\\left(b_1^2+b_2^2+\\cdots+b_n^2\\right),\n$$\n当且仅当 $\\frac{a_1}{b_1}=\\frac{a_2}{b_2}=\\cdots=\\frac{a_n}{b_n}$ (规定 $a_i=0$ 时, $b_i=0$ )时等号成立.\n证法七(利用排序不等式) 由于\n$$\n\\sum_{i=1}^n a_i^2 \\sum_{i=1}^n b_i^2=a_1^2 \\sum_{i=1}^n b_i^2+a_2^2 \\sum_{i=1}^n b_i^2+\\cdots+a_n^2 \\sum_{i=1}^n b_i^2,\n$$\n则\n$$\n\\begin{gathered}\na_1 b_1, \\cdots, a_1 b_n, a_2 b_1, \\cdots, a_2 b_n, \\cdots, a_n b_1, \\cdots, a_n b_n, \\\\\na_1 b_1, \\cdots, a_1 b_n, a_2 b_1, \\cdots, a_2 b_n, \\cdots, a_n b_1, \\cdots, a_n b_n\n\\end{gathered}\n$$\n有两行相同, 共 $n^2$ 列, 且是同序的.\n另一方面,有乱序\n$$\n\\begin{aligned}\n& a_1 b_1, \\cdots, a_1 b_n, a_2 b_1, \\cdots, a_2 b_n, \\cdots, a_n b_1, \\cdots, a_n b_n \\\\\n& a_1 b_1, \\cdots, a_n b_1, a_1 b_2, \\cdots, a_n b_2, \\cdots, a_1 b_n, \\cdots, a_n b_n\n\\end{aligned}\n$$\n两行, 共 $n^2$ 列, 且两行为乱序, 其乘积为\n$$\n\\sum_{i=1}^n \\sum_{j=1}^n\\left(a_i b_j\\right)\\left(a_j b_i\\right)=\\left(\\sum_{i=1}^n a_i b_i\\right)^2 .\n$$\n由引理 1 , 得\n$$\n\\left(\\sum_{i=1}^n a_i b_i\\right)^2 \\leqslant \\sum_{i=1}^n a_i^2 \\sum_{i=1}^n b_i^2,\n$$\n当且仅当 $\\frac{a_1}{b_1}=\\frac{a_2}{b_2}=\\cdots=\\frac{a_n}{b_n}$ 时等号成立.", + "figures": [] +} \ No newline at end of file diff --git a/processed_dataset/text/0163.json b/processed_dataset/text/0163.json new file mode 100644 index 0000000000000000000000000000000000000000..1629508d2f3d867c1e67fee13c30cf3cb91f0de0 --- /dev/null +++ b/processed_dataset/text/0163.json @@ -0,0 +1,5 @@ +{ + "source_file": "./raw_volume-zh/volume4/chapter3.tex", + "text": "柯西不等式及其证明.\n设 $a_1, a_2, \\cdots, a_n$ 及 $b_1, b_2, \\cdots, b_n$ 为任意实数, 则\n$$\n\\left(a_1 b_1+a_2 b_2+\\cdots+a_n b_n\\right)^2 \\leqslant\\left(a_1^2+a_2^2+\\cdots+a_n^2\\right)\\left(b_1^2+b_2^2+\\cdots+b_n^2\\right),\n$$\n当且仅当 $\\frac{a_1}{b_1}=\\frac{a_2}{b_2}=\\cdots=\\frac{a_n}{b_n}$ (规定 $a_i=0$ 时, $b_i=0$ )时等号成立.\n证法八(利用参数平均值不等式)\n由于对 $m \\in \\mathbf{R}^{+}$, 得\n$$\na_i b_i \\leqslant \\frac{1}{2}\\left(m^2 a_i^2+\\frac{b_i^2}{m^2}\\right) .\n$$\n令 $m^2=\\sqrt{\\frac{\\sum_{i=1}^n b_i^2}{\\sum_{i=1}^n a_i^2}}$, 则\n$$\n\\left|a_i b_i\\right| \\leqslant \\frac{1}{2}\\left(\\sqrt{\\frac{\\sum_{i=1}^n b_i^2}{\\sum_{i=1}^n a_i^2} a_i^2}+\\sqrt{\\frac{\\sum_{i=1}^n a_i^2}{\\sum_{i=1}^n b_i^2}} b_i^2\\right),\n$$\n从而\n$$\n\\sum_{i=1}^n\\left|a_i b_i\\right| \\leqslant \\frac{1}{2}\\left(\\sqrt{\\frac{\\sum_{i=1}^n b_i^2}{\\sum_{i=1}^n a_i^2}} \\sum_{i=1}^n a_i^2+\\sqrt{\\frac{\\sum_{i=1}^n a_i^2}{\\sum_{i=1}^n b_i^2}} \\sum_{i=1}^n b_i^2\\right),\n$$\n故\n$$\n\\begin{aligned}\n\\sum_{i=1}^n a_i b_i & \\leqslant \\sum_{i=1}^n\\left|a_i b_i\\right| \\leqslant \\frac{1}{2}\\left(\\sqrt{\\sum_{i=1}^n b_i^2 \\sum_{i=1}^n a_i^2}+\\sqrt{\\sum_{i=1}^n a_i^2 \\sum_{i=1}^n b_i^2}\\right) \\\\\n& =\\left(\\sum_{i=1}^n a_i^2\\right)^{\\frac{1}{2}}\\left(\\sum_{i=1}^n b_i^2\\right)^{\\frac{1}{2}} .\n\\end{aligned}\n$$\n注:利用含参数的基本不等式来证明不等式, 具有较高的灵活性和技巧, 为了让大家熟悉这种证明方法, 后面, 我们将专门介绍.", + "figures": [] +} \ No newline at end of file diff --git a/processed_dataset/text/0164.json b/processed_dataset/text/0164.json new file mode 100644 index 0000000000000000000000000000000000000000..9cfae344abdac4a2c3878f61cf5ac7b333d7a072 --- /dev/null +++ b/processed_dataset/text/0164.json @@ -0,0 +1,5 @@ +{ + "source_file": "./raw_volume-zh/volume4/chapter3.tex", + "text": "柯西不等式及其证明.\n设 $a_1, a_2, \\cdots, a_n$ 及 $b_1, b_2, \\cdots, b_n$ 为任意实数, 则\n$$\n\\left(a_1 b_1+a_2 b_2+\\cdots+a_n b_n\\right)^2 \\leqslant\\left(a_1^2+a_2^2+\\cdots+a_n^2\\right)\\left(b_1^2+b_2^2+\\cdots+b_n^2\\right),\n$$\n当且仅当 $\\frac{a_1}{b_1}=\\frac{a_2}{b_2}=\\cdots=\\frac{a_n}{b_n}$ (规定 $a_i=0$ 时, $b_i=0$ )时等号成立.\n证法九(利用行列式性质)\n$$\n\\begin{aligned}\n& S=\\sum_{i=1}^n a_i^2 \\cdot \\sum_{i=1}^n b_i^2-\\left(\\sum_{i=1}^n a_i b_i\\right)^2 \\\\\n&=\\left|\\begin{array}{cc}\na_1^2+a_2^2+\\cdots+a_n^2 & a_1 b_1+a_2 b_2+\\cdots+a_n b_n \\\\\na_1 b_1+a_2 b_2+\\cdots+a_n b_n & b_1^2+b_2^2+\\cdots+b_n^2\n\\end{array}\\right| \\\\\n&=\\sum_{i=1}^n\\left|\\begin{array}{cc}\na_1^2+a_2^2+\\cdots+a_n^2 & a_i b_i \\\\\na_1 b_1+a_2 b_2+\\cdots+a_n b_n & b_i^2\n\\end{array}\\right| \\\\\n&=\\sum_{i=1}^n \\sum_{j=1}^n\\left|\\begin{array}{cc}\na_j^2 & a_i b_i \\\\\na_j b_j & b_i^2\n\\end{array}\\right| \\\\\n&=\\sum_{i=1}^n \\sum_{j=1}^n a_j b_i\\left|\\begin{array}{cc}\na_j & a_i \\\\\nb_j & b_i\n\\end{array}\\right|, \\\\\n& S=\\sum_{j=1}^n \\sum_{i=1}^n a_i b_j\\left|\\begin{array}{cc}\na_i & a_j \\\\\nb_i & b_j\n\\end{array}\\right| \\\\\n&=\\sum_{j=1}^n \\sum_{i=1}^n a_i b_j(-1)\\left|\\begin{array}{cc}\na_j & a_i \\\\\nb_j & b_i\n\\end{array}\\right| \\\\\n&=\\sum_{i=1}^n \\sum_{j=1}^n a_i b_j(-1)\\left|\\begin{array}{ll}\na_j & a_i \\\\\nb_j & b_i\n\\end{array}\\right| \\\\\n& 2 S=\\sum_{i=1}^n \\sum_{j=1}^n\\left(a_j b_i-a_i b_j\\right)\\left|\\begin{array}{ll}\na_j & a_i \\\\\nb_j & b_i\n\\end{array}\\right| \\\\\n&=\\sum_{i=1}^n \\sum_{j=1}^n\\left(a_j b_i-a_i b_j\\right)^2 \\geqslant 0,\n\\end{aligned}\n$$\n又\n$$\n\\begin{aligned}\nS & =\\sum_{j=1}^n \\sum_{i=1}^n a_i b_j\\left|\\begin{array}{cc}\na_i & a_j \\\\\nb_i & b_j\n\\end{array}\\right| \\\\\n& =\\sum_{j=1}^n \\sum_{i=1}^n a_i b_j(-1)\\left|\\begin{array}{ll}\na_j & a_i \\\\\nb_j & b_i\n\\end{array}\\right| \\\\\n& =\\sum_{i=1}^n \\sum_{j=1}^n a_i b_j(-1)\\left|\\begin{array}{cc}\na_j & a_i \\\\\nb_j & b_i\n\\end{array}\\right|,\n\\end{aligned}\n$$\n所以\n$$\n\\begin{aligned}\n2 S & =\\sum_{i=1}^n \\sum_{j=1}^n\\left(a_j b_i-a_i b_j\\right)\\left|\\begin{array}{ll}\na_j & a_i \\\\\nb_j & b_i\n\\end{array}\\right| \\\\\n& =\\sum_{i=1}^n \\sum_{j=1}^n\\left(a_j b_i-a_i b_j\\right)^2 \\geqslant 0\n\\end{aligned}\n$$\n即 $S \\geqslant 0$, 故不等式成立.", + "figures": [] +} \ No newline at end of file diff --git a/processed_dataset/text/0165.json b/processed_dataset/text/0165.json new file mode 100644 index 0000000000000000000000000000000000000000..f5e5220e7abb9cd0747c074dbbd7c4cfca773fdb --- /dev/null +++ b/processed_dataset/text/0165.json @@ -0,0 +1,5 @@ +{ + "source_file": "./raw_volume-zh/volume4/chapter3.tex", + "text": "柯西不等式及其证明.\n设 $a_1, a_2, \\cdots, a_n$ 及 $b_1, b_2, \\cdots, b_n$ 为任意实数, 则\n$$\n\\left(a_1 b_1+a_2 b_2+\\cdots+a_n b_n\\right)^2 \\leqslant\\left(a_1^2+a_2^2+\\cdots+a_n^2\\right)\\left(b_1^2+b_2^2+\\cdots+b_n^2\\right),\n$$\n当且仅当 $\\frac{a_1}{b_1}=\\frac{a_2}{b_2}=\\cdots=\\frac{a_n}{b_n}$ (规定 $a_i=0$ 时, $b_i=0$ )时等号成立.\n证法十(利用拉格朗日恒等式)\n对 $a_1, a_2, \\cdots, a_n$ 与 $b_1, b_2, \\cdots, b_n$, 我们有如下的拉格朗日恒等式\n$$\n\\left(\\sum_{i=1}^n a_i^2\\right) \\cdot\\left(\\sum_{i=1}^n b_i^2\\right)-\\left(\\sum_{i=1}^n a_i b_i\\right)^2=\\sum_{1 \\leqslant i0\n$$\n在证法十一中, 就是利用了这个性质.", + "figures": [] +} \ No newline at end of file diff --git a/processed_dataset/text/0168.json b/processed_dataset/text/0168.json new file mode 100644 index 0000000000000000000000000000000000000000..e46427ba24c0adb3f00194e61be45d3c9e59aac8 --- /dev/null +++ b/processed_dataset/text/0168.json @@ -0,0 +1,5 @@ +{ + "source_file": "./raw_volume-zh/volume4/chapter3.tex", + "text": "柯西不等式及其证明.\n设 $a_1, a_2, \\cdots, a_n$ 及 $b_1, b_2, \\cdots, b_n$ 为任意实数, 则\n$$\n\\left(a_1 b_1+a_2 b_2+\\cdots+a_n b_n\\right)^2 \\leqslant\\left(a_1^2+a_2^2+\\cdots+a_n^2\\right)\\left(b_1^2+b_2^2+\\cdots+b_n^2\\right),\n$$\n当且仅当 $\\frac{a_1}{b_1}=\\frac{a_2}{b_2}=\\cdots=\\frac{a_n}{b_n}$ (规定 $a_i=0$ 时, $b_i=0$ )时等号成立.\n证法十三(构造单调数列)\n构造数列 $\\left\\{S_n\\right\\}$, 其中\n$$\nS_n=\\left(a_1 b_1+a_2 b_2+\\cdots+a_n b_n\\right)^2-\\left(a_1^2+a_2^2+\\cdots+a_n^2\\right)\\left(b_1^2+b_2^2+\\cdots+b_n^2\\right) \\text {, }\n$$\n则\n$$\n\\begin{aligned}\n& S_1=\\left(a_1 b_1\\right)^2-a_1^2 b_1^2=0 \\\\\n& S_{n+1}-S_n= {\\left[\\left(a_1 b_1+a_2 b_2+\\cdots+a_{n+1} b_{n+1}\\right)^2\\right.} \\\\\n&\\left.-\\left(a_1^2+a_2^2+\\cdots+a_{n+1}^2\\right)\\left(b_1^2+b_2^2+\\cdots+b_{n+1}^2\\right)\\right] \\\\\n&-\\left[\\left(a_1 b_1+a_2 b_2+\\cdots+a_n b_n\\right)^2-\\left(a_1^2+a_2^2+\\cdots+a_n^2\\right)\\right. \\\\\n&\\left.\\left(b_1^2+b_2^2+\\cdots+b_n^2\\right)\\right]\n\\end{aligned}\n$$\n$$\n\\begin{aligned}\n= & 2\\left(a_1 b_1+a_2 b_2+\\cdots+a_n b_n\\right) a_{n+1} b_{n+1}+a_{n+1}^2 b_{n+1}^2 \\\\\n& -\\left(a_1^2+a_2^2+\\cdots+a_n^2\\right) b_{n+1}^2 \\\\\n& -a_{n+1}^2\\left(b_1^2+b_2^2+\\cdots+b_n^2\\right)-a_{n+1}^2 b_{n+1}^2 \\\\\n= & -\\left[\\left(a_1 b_{n+1}-b_1 a_{n+1}\\right)^2+\\left(a_2 b_{n+1}-b_2 a_{n+1}\\right)^2\\right. \\\\\n& \\left.+\\cdots+\\left(a_n b_{n+1}-b_n a_{n+1}\\right)^2\\right] \\leqslant 0,\n\\end{aligned}\n$$\n即 $S_{n+1} \\leqslant S_n$, 所以数列 $\\left\\{S_n\\right\\}$ 单调减少, 从而对一切 $n \\geqslant 1$, 有 $S_n \\leqslant S_1=0$, 故命题成立.", + "figures": [] +} \ No newline at end of file diff --git a/processed_dataset/text/0169.json b/processed_dataset/text/0169.json new file mode 100644 index 0000000000000000000000000000000000000000..e75edc4cfecf4e7953e510900df4d625bc8ec204 --- /dev/null +++ b/processed_dataset/text/0169.json @@ -0,0 +1,5 @@ +{ + "source_file": "./raw_volume-zh/volume4/chapter3.tex", + "text": "柯西不等式及其证明.\n设 $a_1, a_2, \\cdots, a_n$ 及 $b_1, b_2, \\cdots, b_n$ 为任意实数, 则\n$$\n\\left(a_1 b_1+a_2 b_2+\\cdots+a_n b_n\\right)^2 \\leqslant\\left(a_1^2+a_2^2+\\cdots+a_n^2\\right)\\left(b_1^2+b_2^2+\\cdots+b_n^2\\right),\n$$\n当且仅当 $\\frac{a_1}{b_1}=\\frac{a_2}{b_2}=\\cdots=\\frac{a_n}{b_n}$ (规定 $a_i=0$ 时, $b_i=0$ )时等号成立.\n证法十四 (二次函数的判别式)\n令 $A_n=a_1^2+a_2^2+\\cdots+a_n^2, B_n=a_1 b_1+a_2 b_2+\\cdots+a_n b_n, C_n=b_1^2+ b_2^2+\\cdots+b_n^2$, 作二次函数 $f(x)=A_n x^2+2 B_n x+C_n=\\sum_{i=1}^n\\left(a_i x+b_i\\right)^2 \\geqslant 0$, 且 $f(x)=0$ 的充要条件是 $\\frac{a_i}{b_i}=\\lambda$ 为常数.\n由于 $A_n>0, f(x) \\geqslant 0$, 则它的判别式 $\\Delta=4\\left(B_n^2-A_n C_n\\right) \\leqslant 0$, 即\n$$\nB_n^2 \\leqslant A_n C_n \\text {. }\n$$\n等号成立当且仅当 $\\frac{a_1}{b_1}=\\frac{a_2}{b_2}=\\cdots=\\frac{a_n}{b_n}$ 为常数.\n用类似的方法, 可以证明下列不等式:\n设 $a_i, b_i \\in \\mathbf{R}$, 满足 $a_1^2-a_2^2-\\cdots-a_n^2>0$ 或 $b_1^2-b_2^2-\\cdots-b_n^2>0$, 求证: $\\left(a_1 b_1-a_2 b_2-\\cdots-a_n b_n\\right)^2 \\geqslant\\left(a_1^2-a_2^2-\\cdots-a_n^2\\right)\\left(b_1^2-b_2^2-\\cdots-b_n^2\\right)$.\n证明按上述记号, 不妨设 $A_n>0$, 考虑函数 $g(x)=A_n x^2+2 B_n x+ C_n=\\left(a_1 x+b_1\\right)^2-\\sum_{i=2}^n\\left(a_i x+b_i\\right)^2$, 则存在 $x_0=-\\frac{b_1}{a_1}, a_1 \\neq 0$, 使得 $g\\left(x_0\\right) \\leqslant$ 0 , 由于二次函数开口向上, 从而存在 $x_1$ 充分大, 使得 $g\\left(x_1\\right)>0$. 则它的判别式 $\\Delta=4\\left(B_n^2-A_n C_n\\right) \\geqslant 0$, 即\n$$\nB_n^2 \\geqslant A_n C_n\n$$\n等号成立当且仅当 $\\frac{a_1}{b_1}=\\frac{a_2}{b_2}=\\cdots=\\frac{a_n}{b_n}$ 为常数.", + "figures": [] +} \ No newline at end of file diff --git a/processed_dataset/text/0170.json b/processed_dataset/text/0170.json new file mode 100644 index 0000000000000000000000000000000000000000..85e0539ac7d34a40c0e159a14ad83fea2894c925 --- /dev/null +++ b/processed_dataset/text/0170.json @@ -0,0 +1,5 @@ +{ + "source_file": "./raw_volume-zh/volume4/chapter3.tex", + "text": "柯西不等式及其证明.\n设 $a_1, a_2, \\cdots, a_n$ 及 $b_1, b_2, \\cdots, b_n$ 为任意实数, 则\n$$\n\\left(a_1 b_1+a_2 b_2+\\cdots+a_n b_n\\right)^2 \\leqslant\\left(a_1^2+a_2^2+\\cdots+a_n^2\\right)\\left(b_1^2+b_2^2+\\cdots+b_n^2\\right),\n$$\n当且仅当 $\\frac{a_1}{b_1}=\\frac{a_2}{b_2}=\\cdots=\\frac{a_n}{b_n}$ (规定 $a_i=0$ 时, $b_i=0$ )时等号成立.\n证法十五 (凹函数方法)\n令 $A_n=a_1^2+a_2^2+\\cdots+a_n^2, B_n=a_1 b_1+a_2 b_2+\\cdots+a_n b_n, C_n=b_1^2+ b_2^2+\\cdots+b_n^2$, 且不妨假设 $a_i>0, b_i>0$, 由前面的引理 4 , 对凹函数 $f(x)= \\ln x$, 有\n$$\n\\begin{gathered}\n\\frac{1}{2} \\ln \\frac{a_i^2}{A_n}+\\frac{1}{2} \\ln \\frac{b_i^2}{C_n} \\leqslant \\ln \\frac{\\frac{a_i^2}{A_n}+\\frac{b_i^2}{C_n}}{2} \\\\\n\\Leftrightarrow \\ln \\left(\\frac{a_i^2}{A_n} \\frac{b_i^2}{C_n}\\right)^{\\frac{1}{2}} \\leqslant \\ln \\frac{\\frac{a_i^2}{A_n}+\\frac{b_i^2}{C_n}}{2} \\\\\n\\Leftrightarrow\\left(\\frac{a_i^2}{A_n} \\frac{b_i^2}{C_n}\\right)^{\\frac{1}{2}} \\leqslant \\frac{\\frac{a_i^2}{A_n}+\\frac{b_i^2}{C_n}}{2} .\n\\end{gathered}\n$$\n于是\n$$\n\\begin{aligned}\n& \\sum_{i=1}^n \\frac{a_i}{A_n^{\\frac{1}{2}}} \\frac{b_i}{C_n^{\\frac{1}{2}}} \\leqslant \\frac{1}{2}\\left(\\frac{1}{A_n} \\sum_{i=1}^n a_i^2+\\frac{1}{C_n} \\sum_{i=1}^n b_i^2\\right)=1 \\\\\n\\Leftrightarrow & \\sum_{i=1}^n a_i b_i \\leqslant A_n^{\\frac{1}{2}} C_n^{\\frac{1}{2}} .\n\\end{aligned}\n$$\n不难得到, 等式成立的充要条件是 $\\frac{a_1}{b_1}=\\frac{a_2}{b_2}=\\cdots=\\frac{a_n}{b_n}$.\n另外, 如果令 $x=\\frac{a_i^2}{A_n}, y=\\frac{b_i^2}{C_n}, p=q=2$, 则由 Young 不等式,容易得到柯西不等式.", + "figures": [] +} \ No newline at end of file diff --git a/processed_dataset/text/0171.json b/processed_dataset/text/0171.json new file mode 100644 index 0000000000000000000000000000000000000000..f3e016c6645b87b46b0e197e01a90392db097a46 --- /dev/null +++ b/processed_dataset/text/0171.json @@ -0,0 +1,5 @@ +{ + "source_file": "./raw_volume-zh/volume4/chapter3.tex", + "text": "柯西不等式的变形和推广.\n变形 1 设 $a_i \\in \\mathbf{R}, b_i>0(i=1,2, \\cdots, n)$, 则\n$$\n\\sum_{i=1}^n \\frac{a_i^2}{b_i} \\geqslant \\frac{\\left(\\sum_{i=1}^n a_i\\right)^2}{\\sum_{i=1}^n b_i},\n$$\n等号成立的充分必要条件是 $a_i=\\lambda b_i(i=1,2, \\cdots, n)$.\n变形 2 设 $a_i, b_i(i=1,2, \\cdots, n)$ 同号且不为零, 则\n$$\n\\sum_{i=1}^n \\frac{a_i}{b_i} \\geqslant \\frac{\\left(\\sum_{i=1}^n a_i\\right)^2}{\\sum_{i=1}^n a_i b_i},\n$$\n等号成立的充分必要条件是 $b_1=b_2=\\cdots=b_n$.\n柯西不等式的推广为赫尔德 (Holder)不等式, 即赫尔德不等式 设 $a_i>0, b_i>0(i=1,2, \\cdots, n), p>0, q>0$, 满足 $\\frac{1}{p}+\\frac{1}{q}=1$, 则\n$$\n\\sum_{i=1}^n a_i b_i \\leqslant\\left(\\sum_{i=1}^n a_i^p\\right)^{\\frac{1}{p}}\\left(\\sum_{i=1}^n b_i^q\\right)^{\\frac{1}{q}} .\n$$\n等号成立的充分必要条件是 $a_i^p=\\lambda b_i^q(i=1,2, \\cdots, n, \\lambda>0)$.\n证明由 Young 不等式, 得\n$$\n\\begin{aligned}\n& \\sum_{i=1}^n\\left[\\frac{a_i^p}{\\sum_{i=1}^n a_i^p}\\right]^{\\frac{1}{p}} \\cdot\\left[\\frac{b_i^q}{\\sum_{i=1}^n b_i^q}\\right]^{\\frac{1}{q}} \\\\\n\\leqslant & \\sum_{i=1}^n\\left[\\frac{1}{p} \\frac{a_i^p}{\\sum_{i=1}^n a_i^p}\\right]+\\sum_{i=1}^n\\left[\\frac{1}{q} \\frac{b_i^q}{\\sum_{i=1}^n b_i^q}\\right]=\\frac{1}{p}+\\frac{1}{q}=1 .\n\\end{aligned}\n$$\n等号成立的充分必要条件是\n$$\n\\frac{a_i^p}{\\sum_{i=1}^n a_i^p}=\\frac{b_i^q}{\\sum_{i=1}^n b_i^q}\n$$\n即 $a_i^p=\\lambda b_i^q(i=1,2, \\cdots, n, \\lambda>0)$.\n赫尔德不等式也可以变形为\n$$\n\\sum_{i=1}^n \\frac{a_i^{m+1}}{b_i^m} \\geqslant \\frac{\\left(\\sum_{i=1}^n a_i\\right)^{m+1}}{\\left(\\sum_{i=1}^n b_i\\right)^m},\n$$\n等号成立的充分必要条件是 $a_i=\\lambda b_i(i=1,2, \\cdots, n)$. 其中 $a_i>0, b_i>0 (i=1,2, \\cdots, n), m>0$ 或 $m<-1$.\n证明当 $m>0$ 时, 由赫尔德不等式, 得\n$$\n\\begin{aligned}\n\\sum_{i=1}^n a_i & =\\sum_{i=1}^n\\left(\\frac{a_i}{b_i^{\\frac{m}{m+1}}}\\right) \\cdot b_i^{\\frac{m}{m+1}} \\\\\n& \\leqslant\\left[\\sum_{i=1}^n\\left(\\frac{a_i}{b_i^{\\frac{m}{m+1}}}\\right)^{m+1}\\right]^{\\frac{1}{m+1}} \\cdot\\left[\\sum_{i=1}^n\\left(b_i^{\\frac{m}{m+1}}\\right)^{\\frac{m+1}{m}}\\right]^{\\frac{m}{m+1}} \\\\\n& =\\left(\\sum_{i=1}^n \\frac{a_i^{m+1}}{b_i^m}\\right)^{\\frac{1}{m+1}} \\cdot\\left(\\sum_{i=1}^n b_i\\right)^{\\frac{m}{m+1}}\n\\end{aligned}\n$$\n故\n$$\n\\sum_{i=1}^n \\frac{a_i^{m+1}}{b_i^m} \\geqslant \\frac{\\left(\\sum_{i=1}^n a_i\\right)^{m+1}}{\\left(\\sum_{i=1}^n b_i\\right)^m} .\n$$\n当 $m<-1$ 时, $-(m+1)>0$, 对于数组 $\\left(b_1, b_2, \\cdots, b_n\\right)$ 和 $\\left(a_1, a_2, \\cdots\\right.$, $\\left.a_n\\right)$ 有\n$$\n\\begin{gathered}\n\\sum_{i=1}^n \\frac{b_i^{-(m+1)+1}}{a_i^{-(m+1)}} \\geqslant \\frac{\\left(\\sum_{i=1}^n b_i\\right)^{-(m+1)+1}}{\\left(\\sum_{i=1}^n a_i\\right)^{-(m+1)}} . \\\\\n\\sum_{i=1}^n \\frac{a_i^{m+1}}{b_i^m} \\geqslant \\frac{\\left(\\sum_{i=1}^n a_i\\right)^{m+1}}{\\left(\\sum_{i=1}^n b_i\\right)^m} .\n\\end{gathered}\n$$\n等号成立当且仅当 $\\left(\\frac{a_i}{b_i^{\\frac{m}{m+1}}}\\right)^{m+1}=\\mu\\left(b_i^{\\frac{m}{m+1}}\\right)^{\\frac{m+1}{m}}$, 即 $a_i=\\lambda b_i(i=1$, $2, \\cdots, n)$.\n由赫尔德不等式可以推出另一个重要的不等式, 即闵可夫斯基不等式 对 $a_i, b_i \\in \\mathbf{R}^{+}, k>1$, 则\n$$\n\\left[\\sum_{i=1}^n\\left(a_i+b^i\\right)^k\\right]^{\\frac{1}{k}} \\leqslant\\left(\\sum_{i=1}^n a_i^k\\right)^{\\frac{1}{k}}+\\left(\\sum_{i=1}^n b_i^k\\right)^{\\frac{1}{k}},\n$$\n当且仅当 $\\frac{a_1}{b_1}=\\frac{a_2}{b_2}=\\cdots=\\frac{a_n}{b_n}$ 时,等号成立.\n证明由赫尔德不等式, 得\n$$\n\\begin{aligned}\n\\sum_{i=1}^n\\left(a_i+b_i\\right)^k= & \\sum_{i=1}^n a_i\\left(a_i+b_i\\right)^{k-1}+\\sum_{i=1}^n b_i\\left(a_i+b_i\\right)^{k-1} \\\\\n\\leqslant & \\left(\\sum_{i=1}^n a_i^k\\right)^{\\frac{1}{k}}\\left[\\sum_{i=1}^n\\left(a_i+b_i\\right)^k\\right]^{\\frac{k-1}{k}}+\\left(\\sum_{i=1}^n b_i^k\\right)^{\\frac{1}{k}}\\left[\\sum_{i=1}^n\\left(a_i+b_i\\right)^k\\right]^{\\frac{k-1}{k}}, \\\\\n& \\\\\n\\text { 所以 } & {\\left[\\sum_{i=1}^n\\left(a_i+b_i\\right)^k\\right]^{\\frac{1}{k}} \\leqslant\\left(\\sum_{i=1}^n a_i^k\\right)^{\\frac{1}{k}}+\\left(\\sum_{i=1}^n b_i^k\\right)^{\\frac{1}{k}} . }\n\\end{aligned}\n$$\n所以 $\\quad\\left[\\sum_{i=1}^n\\left(a_i+b_i\\right)^k\\right]^{\\frac{1}{k}} \\leqslant\\left(\\sum_{i=1}^n a_i^k\\right)^{\\frac{1}{k}}+\\left(\\sum_{i=1}^n b_i^k\\right)^{\\frac{1}{k}}$.\n不难知, 当且仅当 $\\frac{a_1}{b_1}=\\frac{a_2}{b_2}=\\cdots=\\frac{a_n}{b_n}$ 时,等号成立.\n关于柯西不等式的复数形式, 就不在这里讨论了.", + "figures": [] +} \ No newline at end of file diff --git a/processed_dataset/text/0172.json b/processed_dataset/text/0172.json new file mode 100644 index 0000000000000000000000000000000000000000..89d0e91609dc2c94625b014c3b6ce08b613a043b --- /dev/null +++ b/processed_dataset/text/0172.json @@ -0,0 +1,5 @@ +{ + "source_file": "./raw_volume-zh/volume4/chapter4-1.tex", + "text": "4.1 柯西不等式在证明不等式中的应用.\n运用柯西不等式,证明其他不等式的关键是构造两组数,并按照柯西不等式形式进行探索, 巧妙选取两组数.", + "figures": [] +} \ No newline at end of file diff --git a/processed_dataset/text/0173.json b/processed_dataset/text/0173.json new file mode 100644 index 0000000000000000000000000000000000000000..d0853714ce6fde870e67a2323b64a92eb26c5795 --- /dev/null +++ b/processed_dataset/text/0173.json @@ -0,0 +1,5 @@ +{ + "source_file": "./raw_volume-zh/volume4/chapter4-2.tex", + "text": "4.2 柯西不等式在解方程组和求极值中的应用应用柯西不等式中等号成立的条件, 通过不等式夹逼, 求出方程组中各个未知数的值, 从而进一步求出有关代数式的值.\n极值问题往往是关于对称式的问题.\n先根据条件, 在各个未知元相等时的值得出极值, 然后证明相应的不等式.", + "figures": [] +} \ No newline at end of file diff --git a/processed_dataset/text/0174.json b/processed_dataset/text/0174.json new file mode 100644 index 0000000000000000000000000000000000000000..1d0ecd28fcb73b4620c3b6a2def9951ab085cf36 --- /dev/null +++ b/processed_dataset/text/0174.json @@ -0,0 +1,5 @@ +{ + "source_file": "./raw_volume-zh/volume4/chapter4-3.tex", + "text": "4.3 柯西不等式在证明分式不等式中的应用.\n在各种不等式中, 分式不等式的问题由于自身的特点, 证明它们需要有较灵活的技巧和方法.\n对于分式型的不等式,通常运用柯西不等式的一些变形.", + "figures": [] +} \ No newline at end of file diff --git a/processed_dataset/text/0175.json b/processed_dataset/text/0175.json new file mode 100644 index 0000000000000000000000000000000000000000..196f79446d7c9ac87498a81f7ae865962c9ea22e --- /dev/null +++ b/processed_dataset/text/0175.json @@ -0,0 +1,5 @@ +{ + "source_file": "./raw_volume-zh/volume4/chapter4-4.tex", + "text": "4.4 柯西不等式在组合计数估计中的应用.\n在研究组合, 特别是组合计数问题时, 常常需要由给定的条件, 对一些不等式进行估计.\n如果能灵活地应用, 柯西不等式在解决这些问题中能发挥很好的作用.", + "figures": [] +} \ No newline at end of file diff --git a/processed_dataset/text/0176.json b/processed_dataset/text/0176.json new file mode 100644 index 0000000000000000000000000000000000000000..8480ef8e48e25b86f57c977886f9a0cc00c428d1 --- /dev/null +++ b/processed_dataset/text/0176.json @@ -0,0 +1,5 @@ +{ + "source_file": "./raw_volume-zh/volume4/chapter4-5.tex", + "text": "4.5 带参数的柯西不等式如果 $a_i, b_i \\in \\mathbf{R}, \\lambda_i>0, i=1,2, \\cdots, n$, 则\n$$\n\\left(\\sum_{i=1}^n a_i b_i\\right)^2 \\leqslant \\sum_{i=1}^n \\lambda_i a_i^2 \\cdot \\sum_{i=1}^n \\frac{1}{\\lambda_i} b_i^2 .\n$$", + "figures": [] +} \ No newline at end of file diff --git a/processed_dataset/text/0177.json b/processed_dataset/text/0177.json new file mode 100644 index 0000000000000000000000000000000000000000..1afd742f23f42e5020b63eba7d4e93eb7312aab3 --- /dev/null +++ b/processed_dataset/text/0177.json @@ -0,0 +1,5 @@ +{ + "source_file": "./raw_volume-zh/volume4/chapter4-6.tex", + "text": "4.6 利用平均值不等式与柯西不等式解题.", + "figures": [] +} \ No newline at end of file diff --git a/processed_dataset/text/0178.json b/processed_dataset/text/0178.json new file mode 100644 index 0000000000000000000000000000000000000000..58e5ad2f224b9ad65040d56a7eb0ae1719b32715 --- /dev/null +++ b/processed_dataset/text/0178.json @@ -0,0 +1,5 @@ +{ + "source_file": "./raw_volume-zh/volume5/chapter1.tex", + "text": "证明不等式的基本万法现实世界中的量, 相等是局部的、相对的, 而不等则是普遍的、绝对的,不等式的本质是研究\"数量关系\"中的\"不等关系\".\n对于两个量, 我们常常要比较它们之间的大小, 或者证明一个量大于另一个量, 这就是不等式的证明.\n不等式的证明因题而异, 灵活多变, 常常要用到一些基本的不等式,如平均不等式、柯西不等式等,其中还需用到一些技巧性高的代数变形.\n本节将介绍证明不等式的一些最基本的方法.\n1.1 比较法比较法一般有两种形式:\n(1) 差值比较欲证 $A \\geqslant B$, 只需证 $A-B \\geqslant 0$;\n(2) 商值比较若 $B>0$, 欲证 $A \\geqslant B$, 只需证 $\\frac{A}{B} \\geqslant 1$.\n在用比较法时, 常常需要对式子进行适当变形, 如因式分解、拆项、合并项等.", + "figures": [] +} \ No newline at end of file diff --git a/processed_dataset/text/0179.json b/processed_dataset/text/0179.json new file mode 100644 index 0000000000000000000000000000000000000000..39affadda684e0b554f3c6973ef3c6b51942cdf4 --- /dev/null +++ b/processed_dataset/text/0179.json @@ -0,0 +1,5 @@ +{ + "source_file": "./raw_volume-zh/volume5/chapter1.tex", + "text": "1.2. 放缩 法有时我们直接证明不等式 $A \\leqslant B$ 比较困难, 可以试着去找一个中间量 $C$,\n如果有 $A \\leqslant C$ 及 $C \\leqslant B$ 同时成立, 自然就有 $A \\leqslant B$ 成立.\n所谓\"放缩\" 即将 $A$ 放大到 $C$, 再把 $C$ 放大到 $B$ 或者反过来把 $B$ 缩小到 $C$ 再缩小到 $A$. 不等式证明的技巧, 常体现在对放缩尺度的把握上.", + "figures": [] +} \ No newline at end of file diff --git a/processed_dataset/text/0180.json b/processed_dataset/text/0180.json new file mode 100644 index 0000000000000000000000000000000000000000..7259293ca79f09b84455ec0fe5d9b7a2cedb4b76 --- /dev/null +++ b/processed_dataset/text/0180.json @@ -0,0 +1,5 @@ +{ + "source_file": "./raw_volume-zh/volume5/chapter1.tex", + "text": "1.3 分析法就是先假定要证的不等式成立, 然后由它出发推出一系列与之等价的不等式 (即要求推理过程的每一步都可逆), 直到得到一个较容易证明的不等式或者一个明显成立的不等式.\n分析法是一种执果索因的证明方法, 在寻求证明思路时尤为有效.", + "figures": [] +} \ No newline at end of file diff --git a/processed_dataset/text/0181.json b/processed_dataset/text/0181.json new file mode 100644 index 0000000000000000000000000000000000000000..ee5dc592fe2406db47152825bee0eee243d65f2f --- /dev/null +++ b/processed_dataset/text/0181.json @@ -0,0 +1,5 @@ +{ + "source_file": "./raw_volume-zh/volume5/chapter1.tex", + "text": "1.4 待定系数法引人适当的参数, 根据题中式子的特点, 将参数确定, 从而使不等式获得证明.", + "figures": [] +} \ No newline at end of file diff --git a/processed_dataset/text/0182.json b/processed_dataset/text/0182.json new file mode 100644 index 0000000000000000000000000000000000000000..5e667ef7563c42483cf6b350684f0ed31f5cece2 --- /dev/null +++ b/processed_dataset/text/0182.json @@ -0,0 +1,5 @@ +{ + "source_file": "./raw_volume-zh/volume5/chapter1.tex", + "text": "1.5 标准化(归一化)\n当不等式为齐次式的时候, 常可设变量之和为 $k$ (某个常数), 这样不仅简化了式子,而且增加了条件,有助于我们解决问题.", + "figures": [] +} \ No newline at end of file diff --git a/processed_dataset/text/0183.json b/processed_dataset/text/0183.json new file mode 100644 index 0000000000000000000000000000000000000000..9965538dc16d4a10f27dbfae717155b88f6f1242 --- /dev/null +++ b/processed_dataset/text/0183.json @@ -0,0 +1,5 @@ +{ + "source_file": "./raw_volume-zh/volume5/chapter1.tex", + "text": "1.6 Schur 不等式\nSchur 不等式: 设 $x, y, z \\in \\mathbf{R}^{+}$, 则\n$$\nx(x-y)(x-z)+y(y-z)(y-x)+z(z-x)(z-y) \\geqslant 0 . \\label{(1)}\n$$\n(即: $\\sum_{c y c}[x(x-y)(x-z)] \\geqslant 0$. .).\n一般地, Schur 不等式为: 设 $x, y, z \\geqslant 0, r>0$, 则\n$$\n\\sum_{c y c} x^r(x-y)(x-z) \\geqslant 0 . \\label{(2)}\n$$\n证明不妨设 $x \\geqslant y \\geqslant z$, 则\n$$\n\\begin{aligned}\n\\text { 左边 } & \\geqslant x^r(x-y)(x-z)-y^r(x-y)(y-z) \\\\\n& \\geqslant y^r(x-y)^2 \\geqslant 0 .\n\\end{aligned}\n$$\nSchur 不等式的如下两个变形形式在解题中非常有用:\n变形 I : $\\quad \\sum_{c y c} x^3-\\sum_{c y c}\\left[x^2(y+z)\\right]+3 x y z \\geqslant 0$.\n变形 II : $\\quad\\left(\\sum_{c y c} x\\right)^3-4\\left(\\sum_{c y c} x\\right)\\left(\\sum_{c y c} y z\\right)+9 x y z \\geqslant 0$.\n事实上, 把(1)展开即得变形 $\\mathrm{I}$, 因为 $\\left(\\sum_{c x} x\\right)^3=\\sum_{c x} x^3+3 \\sum_{c x}\\left[x^2(y+z)\\right]+ 6 x y z$, 代入变形 $\\mathrm{I}$, 得\n$$\n\\begin{gathered}\n\\left(\\sum_{c y c} x\\right)^3-3 \\sum_{c y c}\\left[x^2(y+z)\\right]-6 x y z-\\sum_{c y c}\\left[x^2(y+z)\\right]+3 x y z \\geqslant 0, \\\\\n\\left(\\sum_{c y c} x\\right)^3-4 \\sum_{c y c}\\left[x^2(y+z)\\right]-3 x y z \\geqslant 0,\n\\end{gathered}\n$$\n所以\n$$\n\\left(\\sum_{c y c} x\\right)^3-4\\left(\\sum_{c y c} x\\right)\\left(\\sum_{c y c} y z\\right)+9 x y z \\geqslant 0 .\n$$", + "figures": [] +} \ No newline at end of file diff --git a/processed_dataset/text/0184.json b/processed_dataset/text/0184.json new file mode 100644 index 0000000000000000000000000000000000000000..6b7f723abe745f4f3c5f0f59264c6a3632a18e9e --- /dev/null +++ b/processed_dataset/text/0184.json @@ -0,0 +1,5 @@ +{ + "source_file": "./raw_volume-zh/volume5/chapter1.tex", + "text": "1.7 Hölder 不等式\nHölder 不等式: 设 $w_1, w_2, \\cdots, w_n$ 是正实数, $w_1+w_2+\\cdots+w_n=1$, 对任意正实数 $a_{i j}$, 有\n$$\n\\begin{aligned}\n& \\left(a_{11}+a_{12}+\\cdots+a_{1 m}\\right)^{w_1}\\left(a_{21}+a_{22}+\\cdots+a_{2 m}\\right)^{w_2} \\cdots\\left(a_{n 1}+a_{n 2}+\\cdots+a_{n m}\\right)^{w_n} \\\\\n\\geqslant & a_{11}^{w_1} a_{21}^{w_2} \\cdots a_{n 1}^{w_n}+a_{12}^{w_1} a_{22}^{w_2} \\cdots a_{n 2}^{w_n}+\\cdots+a_{1 m}^{w_1} a_{2 m}^{w_2} \\cdots a_{n m}^{w_n} .\n\\end{aligned} \\label{(1)}\n$$\n(即: $\\prod_{i=1}^n\\left(\\sum_{j=1}^m a_{i j}\\right)^{w_i} \\geqslant \\sum_{j=1}^m \\prod_{i=1}^n a_{i j}^{w_i}$. )\n证明记 $A_\\alpha=\\sum_{j=1}^m a_{\\alpha j}(\\alpha=1,2, \\cdots, n)$, 则(1)式为\n$$\n\\begin{gathered}\n\\left(A_1^{w_1} A_2^{w_2} \\cdots A_n^{w_n}\\right)^{-1} \\sum_{j=1}^m a_{1 j}^{w_1} a_{2 j}^{w_2} \\cdots a_{n j}^{w_n} \\leqslant 1, \\\\\n\\sum_{j=1}^m\\left(\\frac{a_{1 j}}{A_1}\\right)^{w_1}\\left(\\frac{a_{2 j}}{A_2}\\right)^{w_2} \\cdots\\left(\\frac{a_{n j}}{A_n}\\right)^{w_n} \\leqslant 1 .\n\\end{gathered}\n$$\n即因为 $f(x)=\\ln x(x>0)$ 是向上凸函数 (因为 $f^{\\prime \\prime}(x)=-\\frac{1}{x^2}<0$), 由加权的 Jensen 不等式, 可得\n$$\n\\begin{aligned}\n& w_1 \\ln \\frac{a_{1 j}}{A_1}+w_2 \\ln \\frac{a_{2 j}}{A_2}+\\cdots+w_n \\ln \\frac{a_{n j}}{A_n} \\\\\n= & \\frac{1}{w_1+w_2+\\cdots+w_n}\\left(w_1 \\ln \\frac{a_{1 j}}{A_1}+w_2 \\ln \\frac{a_{2 j}}{A_2}+\\cdots+w_n \\ln \\frac{a_{n j}}{A_n}\\right) \\\\\n\\leqslant & \\ln \\frac{w_1 \\frac{a_{1 j}}{A_1}+w_2 \\frac{a_{2 j}}{A_2}+\\cdots+w_n \\frac{a_{n j}}{A_n}}{w_1+w_2+\\cdots+w_n} \\\\\n\\leqslant & \\ln \\left(w_1 \\frac{a_{1 j}}{A_1}+w_2 \\frac{a_{2 j}}{A_2}+\\cdots+w_n \\frac{a_{n j}}{A_n}\\right),\n\\end{aligned}\n$$\n所以 $\\left(\\frac{a_{1 j}}{A_1}\\right)^{w_1}\\left(\\frac{a_{2 j}}{A_2}\\right)^{w_2} \\cdots\\left(\\frac{a_{n j}}{A_n}\\right)^{w_n} \\leqslant w_1 \\frac{a_{1 j}}{A_1}+w_2 \\frac{a_{2 j}}{A_2}+\\cdots+w_n \\frac{a_{n j}}{A_n}$,\n把上式对 $j$ 从 1 到 $m$ 求和, 得\n$$\n\\sum_{j=1}^m\\left(\\frac{a_{1 j}}{A_1}\\right)^{w_1}\\left(\\frac{a_{2 j}}{A_2}\\right)^{w_2} \\cdots\\left(\\frac{a_{n j}}{A_n}\\right)^{w_n} \\leqslant w_1+w_2+\\cdots+w_n=1,\n$$\n从而命题得证.\n特别地, 当 $w_1=w_2=\\cdots=w_n=\\frac{1}{n}$ 时, 有\n$$\n\\begin{aligned}\n& \\left(a_{11}^n+a_{12}^n+\\cdots+a_{1 m}^n\\right)\\left(a_{21}^n+a_{22}^n+\\cdots+a_{2 m}^n\\right) \\cdots\\left(a_{n 1}^n+a_{n 2}^n+\\cdots+a_{n m}^n\\right) \\\\\n\\geqslant & \\left(a_{11} a_{21} \\cdots a_{n 1}+a_{12} a_{22} \\cdots a_{n 2}+\\cdots+a_{1 m} a_{2 m} \\cdots a_{n m}\\right)^n . \\label{(2)}\n\\end{aligned}\n$$\n在(2)中, 取 $n=3, m=3$, 有\n$$\n\\begin{aligned}\n& \\left(a_{11}^3+a_{12}^3+a_{13}^3\\right)\\left(a_{21}^3+a_{22}^3+a_{23}^3\\right)\\left(a_{31}^3+a_{32}^3+a_{33}^3\\right) \\\\\n\\geqslant & \\left(a_{11} a_{21} a_{31}+a_{12} a_{22} a_{32}+a_{13} a_{23} a_{33}\\right)^3 . \\label{(3)}\n\\end{aligned}\n$$\n在(2)中, 取 $n=3, m=2$, 有\n$$\n\\left(a_{11}^3+a_{12}^3\\right)\\left(a_{21}^3+a_{22}^3\\right)\\left(a_{31}^3+a_{32}^3\\right) \\geqslant\\left(a_{11} a_{21} a_{31}+a_{12} a_{22} a_{32}\\right)^3 . \\label{(4)}\n$$\n在(1)中, 取 $n=2$, 有\n$$\n\\left(\\sum_{i=1}^m a_i\\right)^\\alpha\\left(\\sum_{i=1}^m b_i\\right)^\\beta \\geqslant \\sum_{i=1}^m a_i^\\alpha b_i^\\beta, \\label{(5)}\n$$\n其中 $\\alpha 、 \\beta$ 是正实数, 且 $\\alpha+\\beta=1$. 当 $\\alpha=\\beta=\\frac{1}{2}$ 时,(5) 即为 Cauchy 不等式.\n在(5)中, 令 $m=n, a_i^\\alpha=x_i, b_i^\\beta=y_i, \\alpha=\\frac{1}{p}, \\beta=\\frac{1}{q}$, 则(5)式为\n$$\n\\sum_{i=1}^n x_i y_i \\leqslant\\left(\\sum_{i=1}^n x_i^p\\right)^{\\frac{1}{p}}\\left(\\sum_{i=1}^n y_i^q\\right)^{\\frac{1}{q}}, \\label{(6)}\n$$\n其中 $p>0, q>0, \\frac{1}{p}+\\frac{1}{q}=1$.", + "figures": [] +} \ No newline at end of file diff --git a/processed_dataset/text/0185.json b/processed_dataset/text/0185.json new file mode 100644 index 0000000000000000000000000000000000000000..ae617771ded034e1b1dd4b22b0fa04839f07b503 --- /dev/null +++ b/processed_dataset/text/0185.json @@ -0,0 +1,5 @@ +{ + "source_file": "./raw_volume-zh/volume5/chapter2.tex", + "text": "和式的恒等变换.\n在不等式的证明过程中, 我们时常要对和式进行处理, 对和式作一些恒等变形.\n因此,有必要了解一下一些重要的恒等变换式以及变换法:\n(1) $a_i a_j+b_i b_j-a_i b_j-a_j b_i=\\left(a_i-b_i\\right)\\left(a_j-b_j\\right)$;\n(2) $\\left(\\sum_{i=1}^n a_i\\right)^2=\\sum_{i=1}^n a_i^2+2 \\sum_{1 \\leqslant i0, m \\leqslant \\sum_{k=1}^t a_k \\leqslant M, t=1,2, \\cdots, n$. 则有:\n$$\nb_1 m \\leqslant \\sum_{k=1}^n a_k b_k \\leqslant b_1 M . \\label{eq3}\n$$\n在实际证题的时候, 如果发现一列数和易求,一列数差易求, 就可以考虑采用 Abel 变换.", + "figures": [] +} \ No newline at end of file diff --git a/processed_dataset/text/0186.json b/processed_dataset/text/0186.json new file mode 100644 index 0000000000000000000000000000000000000000..f3b8fbfaeedead8222e3fb711ffff5e0de70e342 --- /dev/null +++ b/processed_dataset/text/0186.json @@ -0,0 +1,5 @@ +{ + "source_file": "./raw_volume-zh/volume5/chapter3.tex", + "text": "变量代换是数学中常用的解题方法之一, 将一个较复杂的式子视为一个整体,用一个字母去代换它, 从而使复杂问题简单化.\n有时候, 有些式子可以用三角换元,从而使问题简化.\n当问题的条件或结论中出现 \" $x^2+y^2=r^2$ \"、 \" $x^2+y^2 \\leqslant r^2$ \"、\" $\\sqrt{r^2-x^2}$ \"或\" $|x| \\leqslant 1$ \" 等形式时, 可以考虑用\" $\\sin \\alpha$ 与 \" $\\cos \\alpha$ \"代换; 当问题的条件或结论中出现 \" $\\sqrt{r^2+x^2}$ \"、 \" $\\sqrt{x^2-r^2}$ \"形式时,可作 \" $x=r \\tan \\alpha$ \"或 \" $x=r \\sec \\alpha$ \" 代换等.\n在作代换时,要特别注意 $\\alpha$ 的取值范围是由原变量 $x$ 的取值范围所决定的.", + "figures": [] +} \ No newline at end of file diff --git a/processed_dataset/text/0187.json b/processed_dataset/text/0187.json new file mode 100644 index 0000000000000000000000000000000000000000..40bd6ad080251fb436504cd93468e97a8195986f --- /dev/null +++ b/processed_dataset/text/0187.json @@ -0,0 +1,5 @@ +{ + "source_file": "./raw_volume-zh/volume5/chapter4.tex", + "text": "反证法是我们论证数学命题时常用的有力工具.\n有些问题从正面很难下手, 就应试着用反证法来考虑, 因为当我们从正面去看问题而发现条件不多时,反证假设就相当于又加了一个条件,这样自然更易人手.\n反证法有着广泛的应用,这一章我们就来看一下它在不等式证明中的应用.", + "figures": [] +} \ No newline at end of file diff --git a/processed_dataset/text/0188.json b/processed_dataset/text/0188.json new file mode 100644 index 0000000000000000000000000000000000000000..865c6180ce9ea3f24b4c39fa832144272eb83ed6 --- /dev/null +++ b/processed_dataset/text/0188.json @@ -0,0 +1,5 @@ +{ + "source_file": "./raw_volume-zh/volume5/chapter5.tex", + "text": "构造法是不等式证明中的一种重要方法.\n主要利用引人适当的恒等式、 函数、图形、数列等辅助手段, 使命题转化, 变成较为直观和本质的形式, 进而使不等式获证.\n5.1 构造恒等式恒等式可以看作是最强的不等式, 有时候, 通过补充不等式中略去的那些项或因式, 可以得到隐藏在其背后的恒等式, 这样往往能找到证题的突破口, 因为恒等式的结果是显然的.", + "figures": [] +} \ No newline at end of file diff --git a/processed_dataset/text/0189.json b/processed_dataset/text/0189.json new file mode 100644 index 0000000000000000000000000000000000000000..dcd46f208ed46d442d631abd7d8c8d063f70f0ec --- /dev/null +++ b/processed_dataset/text/0189.json @@ -0,0 +1,5 @@ +{ + "source_file": "./raw_volume-zh/volume5/chapter5.tex", + "text": "5.2 构造函数根据代数式的特征, 构造适当的函数, 利用一次函数、二次函数的性质, 以及函数的单调性等性质, 可以帮助我们来证明不等式.", + "figures": [] +} \ No newline at end of file diff --git a/processed_dataset/text/0190.json b/processed_dataset/text/0190.json new file mode 100644 index 0000000000000000000000000000000000000000..f0d2a47dc84c7506c36d5f7a85835a2d40731099 --- /dev/null +++ b/processed_dataset/text/0190.json @@ -0,0 +1,5 @@ +{ + "source_file": "./raw_volume-zh/volume5/chapter5.tex", + "text": "5.3 构造图形如果问题条件中的数量关系有明显的几何意义或以某种方式可与几何 图形建立联系, 那么通过作图构造图形, 将题设的条件及数量关系直接在图形中得到实现,然后在构造的图形中寻求所证的结论.", + "figures": [] +} \ No newline at end of file diff --git a/processed_dataset/text/0191.json b/processed_dataset/text/0191.json new file mode 100644 index 0000000000000000000000000000000000000000..c1f1704086e72bcc34ea0217455706e294e92dee --- /dev/null +++ b/processed_dataset/text/0191.json @@ -0,0 +1,5 @@ +{ + "source_file": "./raw_volume-zh/volume5/chapter5.tex", + "text": "5.4 构造对偶式在一些轮换不等式中, 构造一个新的对偶轮换式与原不等式一起考虑, 常常能起到意想不到的效果.", + "figures": [] +} \ No newline at end of file diff --git a/processed_dataset/text/0192.json b/processed_dataset/text/0192.json new file mode 100644 index 0000000000000000000000000000000000000000..7207a3f9525fd476a8fcb4576da1fb7a4e9e911a --- /dev/null +++ b/processed_dataset/text/0192.json @@ -0,0 +1,5 @@ +{ + "source_file": "./raw_volume-zh/volume5/chapter5.tex", + "text": "5.5 构造数列在遇到与 $n$ 有关的不等式时, 可以考虑构造辅助数列, 并通过数列的性质 (如单调性) 来证题.", + "figures": [] +} \ No newline at end of file diff --git a/processed_dataset/text/0193.json b/processed_dataset/text/0193.json new file mode 100644 index 0000000000000000000000000000000000000000..5e2c3bf07027f38f8a8dbc25670ddf35f86e991f --- /dev/null +++ b/processed_dataset/text/0193.json @@ -0,0 +1,5 @@ +{ + "source_file": "./raw_volume-zh/volume5/chapter5.tex", + "text": "5.6 构造辅助命题如果一个命题直接证比较困难, 可以试着考虑建立辅助命题来帮助证题.", + "figures": [] +} \ No newline at end of file diff --git a/processed_dataset/text/0194.json b/processed_dataset/text/0194.json new file mode 100644 index 0000000000000000000000000000000000000000..b22d2efc6e3e9545b8438e2a91e27b61495a997a --- /dev/null +++ b/processed_dataset/text/0194.json @@ -0,0 +1,5 @@ +{ + "source_file": "./raw_volume-zh/volume5/chapter6.tex", + "text": "局部不等式.\n对于和式类不等式, 如果从整体考虑较难人手的话, 不妨先估计局部的性质,导出一些局部的不等式,再综合这些局部不等式推断出整体的性质.\n这里所说的局部, 既可以是由一个单项,也可以是由几项组成.", + "figures": [] +} \ No newline at end of file diff --git a/processed_dataset/text/0195.json b/processed_dataset/text/0195.json new file mode 100644 index 0000000000000000000000000000000000000000..2fcb2c2bd424d4320a9b921ef121ac37d5d7952c --- /dev/null +++ b/processed_dataset/text/0195.json @@ -0,0 +1,5 @@ +{ + "source_file": "./raw_volume-zh/volume5/chapter7.tex", + "text": "数学归纳法与不等式证明.\n数学归纳法有很多表达形式, 其中最基本和最常用的是第一数学归纳法和第二数学归纳法.\n第一数学归纳法: 设 $P(n)$ 是一个 (关于正整数 $n$ ) 的命题.\n如果:\n(1) $P(1)$ 成立; (2) 设 $P(k)$ 成立, 可推出 $P(k+1)$ 成立, 那么 $P(n)$ 对一切正整数 $n$ 都成立.\n第二数学归纳法: 设 $P(n)$ 是一个 (关于正整数 $n$ ) 的命题.\n如果:\n(1) $P(1)$ 成立; (2) 设 $n \\leqslant k$ ( $k$ 为任意正整数) 时 $P(n)(1 \\leqslant n \\leqslant k)$ 成立, 可推出 $P(k+1)$ 成立,那么 $P(n)$ 对一切正整数 $n$ 都成立.\n在遇到与正整数 $n$ 有关的不等式时, 往往可以想到采用数学归纳法去证明.", + "figures": [] +} \ No newline at end of file diff --git a/processed_dataset/text/0196.json b/processed_dataset/text/0196.json new file mode 100644 index 0000000000000000000000000000000000000000..8b4ab5265db41ea24007aff55556e13a6deeca3c --- /dev/null +++ b/processed_dataset/text/0196.json @@ -0,0 +1,5 @@ +{ + "source_file": "./raw_volume-zh/volume5/chapter8.tex", + "text": "不等式与多变量函数最值.\n在这一章里我们主要学习最值问题中所常用的\"固定变量法\",即当我们遇到那些有很多\"互相制约因素和变量的最值问题时, 可以先固定大多数变量, 只允许一部分变量变动, 借机看清它们与目标量之间的依赖关系.\n然后, 让先前被固定冻结住的变量活化而重新变动起来,最后达到解决问题的目的.\n当然, 使用固定变法处理问题时, 可以从函数值人手, 让函数值逐步逐步接近最值; 也可以从自变量本身人手, 使其较快达到最值点; 还可以从正面推导,或从反面论证.\n针对这些不同的手段,下面我们将分别介绍 \"累次求最值法\"、\"磨光变换法\"以及\"调整法\".\n8.1 累次求最值法累次求最值法的关键在于求最值的最值, 即先固定一些变量, 对于剩下的那小部分变量求出最值 (当然与前面被固定的变量有关), 再解冻被固定住的变量, 对求得的最值继续作估计.\n如此一步一步, 直至求出目的最值.\n用累次求最值法证明不等式,也叫逐次逼近法,其技巧性很强.", + "figures": [] +} \ No newline at end of file diff --git a/processed_dataset/text/0197.json b/processed_dataset/text/0197.json new file mode 100644 index 0000000000000000000000000000000000000000..cfca826146f1e5b41b5479909f779029ac3ab18f --- /dev/null +++ b/processed_dataset/text/0197.json @@ -0,0 +1,5 @@ +{ + "source_file": "./raw_volume-zh/volume5/chapter8.tex", + "text": "8.2 磨光变换法前述的累次求最值法, 在每一次逼近过程中都保证了取到统一等号, 这就要求我们首先发现取到最值的条件.\n我们可以换一种方法来求解, 当已知等号成立的条件后, 就不必每步都取到等号, 只要使变量组 $\\left(x_1, x_2, \\cdots, x_n\\right)$ 逐步接近等号组 (即最值点), 且保证有限步可达到最值点即可.\n这种方法称为磨光变换法.\n在用磨光变换法解题时, 须注意确保函数值在磨光变换下向所需方向变化, 如欲求最大值, 则磨光变换必须保证函数值不减, 有时就要求我们适当地选取变量才能满足这一要求.", + "figures": [] +} \ No newline at end of file diff --git a/processed_dataset/text/0198.json b/processed_dataset/text/0198.json new file mode 100644 index 0000000000000000000000000000000000000000..9be0512899622d3dacd07c6404303dbc916670ac --- /dev/null +++ b/processed_dataset/text/0198.json @@ -0,0 +1,5 @@ +{ + "source_file": "./raw_volume-zh/volume5/chapter8.tex", + "text": "8.3 调整 法对于一些最值问题, 如离散极值问题, 它们只有有限多种情形, 自然存在最大、最小值.\n这样, 我们可以凭借最值存在这一点, 用调整法来反证而解决问题.\n由于最值必存在, 我们不必像累次求极值法那样每步都取极值, 也不必像磨光变换法那样保证迅速接近最值点.\n只要对各种情形作局部的适当调整, 说明它们不能取最值即可.", + "figures": [] +} \ No newline at end of file diff --git a/processed_dataset/text/0199.json b/processed_dataset/text/0199.json new file mode 100644 index 0000000000000000000000000000000000000000..d6e4344ded34fc7f5a47e847df097ea7f1e99087 --- /dev/null +++ b/processed_dataset/text/0199.json @@ -0,0 +1,5 @@ +{ + "source_file": "./raw_volume-zh/volume5/chapter9.tex", + "text": "一些特殊的证明万法和技巧.\n不等式形形色色,变化万千,在前面几节里我们已经学过了不少证明方法和技巧,这一章我们再学习几种证题方法, 以供读者借鉴.\n9.1 断开求和法.\n断开求和法是指把和式分为两部分, 分别用不同的估计方法处理, 最后再结合起来给出证明.\n这种方法的技巧性很强, 请读者认真体会.", + "figures": [] +} \ No newline at end of file diff --git a/processed_dataset/text/0200.json b/processed_dataset/text/0200.json new file mode 100644 index 0000000000000000000000000000000000000000..e2fd02123f41282de886907e81db9761b7d16777 --- /dev/null +++ b/processed_dataset/text/0200.json @@ -0,0 +1,5 @@ +{ + "source_file": "./raw_volume-zh/volume5/chapter9.tex", + "text": "9.2 枚举 法在应用枚举法证题时一定要细心, 不要漏了可能的情况.", + "figures": [] +} \ No newline at end of file diff --git a/processed_dataset/text/0201.json b/processed_dataset/text/0201.json new file mode 100644 index 0000000000000000000000000000000000000000..e83902b60e5d47feb1a1ee64332e0ebdf984b99b --- /dev/null +++ b/processed_dataset/text/0201.json @@ -0,0 +1,5 @@ +{ + "source_file": "./raw_volume-zh/volume5/chapter9.tex", + "text": "9.3 加\"序\"条件在无序的不等式中引人\"序关系\", 可以为我们增加有利条件, 帮助证题.", + "figures": [] +} \ No newline at end of file diff --git a/processed_dataset/text/0202.json b/processed_dataset/text/0202.json new file mode 100644 index 0000000000000000000000000000000000000000..011551fd555c713486ab9354ce34782f74a6b2f6 --- /dev/null +++ b/processed_dataset/text/0202.json @@ -0,0 +1,5 @@ +{ + "source_file": "./raw_volume-zh/volume5/chapter9.tex", + "text": "9.4 一些非 \"对称\"不等式的处理方法我们经常会遇到一些不等式题, 其变元的地位不尽相同, 我们在处理时就不能再用\"平均\"的方法了, 而了解等号成立的条件是大有益处的.", + "figures": [] +} \ No newline at end of file diff --git a/processed_dataset/text/0203.json b/processed_dataset/text/0203.json new file mode 100644 index 0000000000000000000000000000000000000000..60fee76e15648d1a118753907901a225da033e71 --- /dev/null +++ b/processed_dataset/text/0203.json @@ -0,0 +1,5 @@ +{ + "source_file": "./raw_volume-zh/volume6/chapter1-1.tex", + "text": "第一数学归纳法.\n数学归纳法是证明关于正整数 $n$ 的命题 $P(n)$ 成立与否时经常用到的方法.\n它是下面的归纳公理的一个直接推论.\n归纳公理设 $S$ 是正整数集 $\\mathbf{N}^*$ 的一个子集,满足条件:\n(1) $1 \\in S$;\n(2) 若 $n \\in S$, 则 $n+1 \\in S$.\n那么 $S=\\mathbf{N}^*$.\n归纳公理是由皮亚诺 (G. Peano, 1858-1932) 提出的关于正整数的五条公理中的一条, 它是数学归纳法的基础.\n第一数学归纳法是最常用的一种形式, 它就是我们高中课本中所提及的数学归纳法.\n第一数学归纳法设 $P(n)$ 是关于正整数 $n$ 的一个命题(或性质). 如果\n(1) 当 $n=1$ 时, $P(n)$ 成立;\n(2) 由 $P(n)$ 成立可以推出 $P(n+1)$ 成立.\n那么, 对任意 $n \\in \\mathbf{N}^*, P(n)$ 都成立.\n证明记 $S=\\left\\{n \\mid n \\in \\mathbf{N}^*, \\text{且} P(n) \\text{成立} \\right\\}$, 则 $S$ 为 $\\mathbf{N}^*$ 的子集.\n由 (1) 知 $1 \\in S$; 由 (2) 知, 若 $n \\in S$, 则 $n+1 \\in S$. 这样由归纳公理可知 $S=\\mathbf{N}^*$, 也就是说, 对任意 $n \\in \\mathbf{N}^*, P(n)$ 都成立.\n说明事实上,第一数学归纳法与归纳公理是等价的, 因此, 我们又称之为数学归纳法原理, 并把第一数学归纳法简称为数学归纳法.\n对中学生而言, 要接受数学归纳法的含义和正确性并不难, 但是要正确地用好数学归纳法却不是一件容易的事.\n数学归纳法中的两步缺一不可.\n验证 $P(1)$ 成立是奠基, 利用归纳假设结合已知的有关数学知识证出 $P(n+1)$ 成立是递推的根据.\n这两步对证明命题相辅相成, 构成数学归纳法证明过程的逻辑结构.\n尤为重要的是在证明过程中必须用到归纳假设, 这是检验是否用对了数学归纳法的一把尺.", + "figures": [] +} \ No newline at end of file diff --git a/processed_dataset/text/0204.json b/processed_dataset/text/0204.json new file mode 100644 index 0000000000000000000000000000000000000000..df6c8b45c5cea19014a4493b21d33a9c0883ea9d --- /dev/null +++ b/processed_dataset/text/0204.json @@ -0,0 +1,5 @@ +{ + "source_file": "./raw_volume-zh/volume6/chapter1-2.tex", + "text": "第二数学归纳法.\n第二数学归纳法设 $P(n)$ 是关于正整数 $n$ 的一个命题(或性质). 如果\n(1) 当 $n=1$ 时, $P(n)$ 成立;\n(2) 由 \"对一切小于 $n$ 的正整数 $k, P(k)$ 都成立\"可以推出 $P(n)$ 成立.\n那么,对任意 $n \\in \\mathbf{N}^*, P(n)$ 都成立.\n证明考虑命题 $Q(n)$ : \"对所有 $1 \\leqslant k \\leqslant n, k \\in \\mathbf{N}^*, P(k)$ 都成立\". 则由 $Q(n)$ 成立, 可知 $P(n)$ 成立.\n当 $n=1$ 时, 由 (1) 知 $Q(n)$ 成立.\n现设 $Q(n-1)(n \\geqslant 2)$ 成立, 即对所有 $1 \\leqslant k \\leqslant n-1, P(k)$ 都成立, 则由 (2) 知, $P(n)$ 成立.\n所以, 对任意 $1 \\leqslant k \\leqslant n, P(k)$ 都成立, 从而, $Q(n)$ 成立.\n于是, 由第一数学归纳法可知, 对任意 $n \\in \\mathbf{N}^*, Q(n)$ 都成立, 进而, $P(n)$ 成立.\n第二数学归纳法获证.\n第二数学归纳法是第一数学归纳法的推论, 在作归纳假设时, 我们假设了 $P(1), \\cdots, P(n-1)$ 都成立, 并在此前提下证出 $P(n)$ 成立, 这是区别于第一数学归纳法的地方, 有时会给证明带来很大的方便.", + "figures": [] +} \ No newline at end of file diff --git a/processed_dataset/text/0205.json b/processed_dataset/text/0205.json new file mode 100644 index 0000000000000000000000000000000000000000..09bab1c0e0ed91bdbfa1cb9cd2548684cb280e83 --- /dev/null +++ b/processed_dataset/text/0205.json @@ -0,0 +1,5 @@ +{ + "source_file": "./raw_volume-zh/volume6/chapter1-3.tex", + "text": "最小数原理与无穷递降法.\n最小数原理在数学竞赛中经常被用到,其最基本的表达形式如下:\n最小数原理:正整数集 $\\mathbf{N}^*$ 的任何一个非空子集 $T$ 必有最小元素, 即存在正整数 $t_0 \\in T$, 使对任意的 $t \\in T$, 都有 $t_0 \\leqslant t$.\n证明考虑集合 $S=\\left\\{x \\mid x \\in \\mathbf{N}^*, x \\notin T\\right\\}$, 即 $S=\\mathbf{N}^* \\backslash T$.\n若 $T$ 中没有最小元, 我们证明: 每一个正整数都属于 $S$, 从而导致 $T=\\varnothing$, 矛盾.\n首先, $1 \\in S$, 否则 $1 \\in T$, 则 1 是 $T$ 中的最小元.\n其次, 设 $1,2, \\cdots, n \\in S$, 即 $1,2, \\cdots, n$ 都不是 $T$ 的元素, 这时, 若 $n+ 1 \\in T$, 则 $n+1$ 为 $T$ 的最小元, 这与 $T$ 中没有最小元矛盾.\n所以 $n+1 \\in S$. 从而, 由第二数学归纳法知, 对任意 $n \\in \\mathbf{N}^*$, 都有 $n \\in S$. 所以,最小数原理成立.\n具体处理问题时,我们还会用到上述原理的一些其他形式或推论.\n1. 最大数原理设 $M$ 是正整数集 $\\mathbf{N}^*$ 的非空子集,且 $M$ 有上界,即存在 $a \\in \\mathbf{N}^*$, 使得对任意 $x \\in M$, 都有 $x \\leqslant a$. 则 $M$ 有最大元.\n2. 任意一个由实数组成的有限集中, 必有最小元素, 也必有最大元素.\n3. 排序原理由 $n$ 个实数组成的集合 $M$ 可以写为 $M=\\left\\{x_1, \\cdots, x_n\\right\\}$, 这里 $x_1a_{n+1}$ ), 那么称它为递增 (或者递减)数列; 如果只是 $a_n \\leqslant a_{n+1}$ (或者 $a_n \\geqslant a_{n+1}$ ), 那么称它为不减(或者不增)数列.\n如果存在常数 $M$,使得对任意 $n \\in \\mathbf{N}^*$, 都有 $\\left|a_n\\right| \\leqslant M$, 那么实数数列 $\\left\\{a_n\\right\\}$ 称为有界数列.", + "figures": [] +} \ No newline at end of file diff --git a/processed_dataset/text/0207.json b/processed_dataset/text/0207.json new file mode 100644 index 0000000000000000000000000000000000000000..4dca3757e5036e46ce1dabc8f55b1dd92a25a7c5 --- /dev/null +++ b/processed_dataset/text/0207.json @@ -0,0 +1,5 @@ +{ + "source_file": "./raw_volume-zh/volume6/chapter1-5.tex", + "text": "等差数列与等比数列.\n等差数列与等比数列是两类最简单的数列, 它们是其他数列化归的对象.\n如果一个数列从第 2 项起,每一项与它的前一项的差(比)等于同一个常数, 那么称它为等差 (比) 数列.\n这个常数叫做等差 (比) 数列的公差 (比), 通常用 $d$ 表示公差, $q$ 表示公比.\n注意, 由于零不能做分母, 因此, $q$ 不能等于零.\n相应于等差 (比)数列的通项与求和有下面的相关公式:\n1. 记等差数列 $\\left\\{a_n\\right\\}$ 的前 $n$ 项之和为 $S_n$, 则 $a_n=a_1+(n-1) d, S_n= \\frac{1}{2}\\left(a_1+a_n\\right) n=a_1 n+\\frac{n(n-1)}{2} d$.\n2. 记等比数列 $\\left\\{a_n\\right\\}$ 的前 $n$ 项之和为 $S_n$, 则 $a_n=a_1 \\cdot q^{n-1}$, 而\n$$\nS_n= \\begin{cases}n a_1, & \\text { 若 } q=1, \\\\ \\frac{a_1\\left(1-q^n\\right)}{1-q}, & \\text { 若 } q \\neq 1 .\\end{cases}\n$$\n3. 如果等比数列 $\\left\\{a_n\\right\\}$ 是一个无穷数列, 其公比 $q$ 满足 $|q|<1$, 那么称它为无穷递缩等比数列, 其所有项之和 $S=\\frac{a_1}{1-q}$.", + "figures": [] +} \ No newline at end of file diff --git a/processed_dataset/text/0208.json b/processed_dataset/text/0208.json new file mode 100644 index 0000000000000000000000000000000000000000..0ab324eebe8f564099f716bf8fd731dc5f660182 --- /dev/null +++ b/processed_dataset/text/0208.json @@ -0,0 +1,5 @@ +{ + "source_file": "./raw_volume-zh/volume6/chapter1-6.tex", + "text": "高阶等差数列与差分方法.\n对一个给定的数列 $\\left\\{a_n\\right\\}$ 的相邻两项作差, 得到一个新数列\n$$\na_2-a_1, a_3-a_2, \\cdots, a_{n+1}-a_n, \\cdots\n$$\n这个数列称为 $\\left\\{a_n\\right\\}$ 的一阶差数列.\n如果记该数列为 $\\left\\{b_n\\right\\}$, 其中 $b_n=a_{n+1}-a_n$, 那么再求 $\\left\\{b_n\\right\\}$ 的相邻两项之差, 所得数列\n$$\nb_2-b_1, b_3-b_2, \\cdots, b_{n+1}-b_n, \\cdots\n$$\n称为原数列 $\\left\\{a_n\\right\\}$ 的二阶差数列.\n依此类推,对任意 $p \\in \\mathbf{N}^*$, 可以定义数列 $\\left\\{a_n\\right\\}$ 的 $p$ 阶差数列.\n如果 $\\left\\{a_n\\right\\}$ 的 $p$ 阶差数列是一个非零常数数列, 那么称它为 $p$ 阶等差数列.\n特别地, 一阶等差数列就是我们通常说的等差数列, 二阶及二阶以上的等差数列统称为高阶等差数列.\n注意到, 数列是定义在 $\\mathrm{N}^*$ 上的函数, 将上述作差思想予以推广就得到了差分的概念.\n设 $f(x)$ 是定义在 $\\mathbf{R}$ 上的函数, 令 $\\Delta f(x)=f(x+1)-f(x)$, 则 $\\Delta f(x)$ 也是定义在 $\\mathbf{R}$ 上的函数, 它称为 $f(x)$ 的一阶差分, 与上类似, 我们可以递推地定义 $f(x)$ 的二阶, 三阶, $\\cdots \\cdots, p$ 阶差分\n$$\n\\begin{aligned}\n\\Delta^2 f(x)= & \\Delta(\\Delta f(x))=\\Delta(f(x+1)-f(x)) \\\\\n= & (f(x+2)-f(x+1))-(f(x+1)-f(x)) \\\\\n= & f(x+2)-2 f(x+1)+f(x), \\\\\n& \\cdots \\cdots, \\\\\n& \\Delta^p f(x)=\\Delta\\left(\\Delta^{p-1} f(x)\\right) .\n\\end{aligned}\n$$\n利用数学归纳法易证下面的定理:\n定理 1 设 $f(x)$ 是定义在 $\\mathbf{R}$ 上的函数,则\n$$\n\\begin{aligned}\n\\Delta^p f(x) & =\\sum_{i=0}^p(-1)^{p-i} \\mathrm{C}_p^i f(x+i) \\\\\n& =\\sum_{i=0}^p(-1)^i \\mathrm{C}_p^i f(x+p-i)\n\\end{aligned}\n$$\n如果函数 $f(x)(x \\in \\mathbf{R})$ 是关于 $x$ 的 $p$ 次多项式, 那么 $\\Delta f(x)$ 是关于 $x$ 的 $p-1$ 次多项式, $\\Delta^2 f(x)$ 是关于 $x$ 的 $p-2$ 次多项式, $\\cdots, \\Delta^p f(x)$ 是关于 $x$ 的零次多项式, 且 $\\Delta^p f(x)=p ! a_p$ (这里 $a_p$ 是 $f(x)$ 的首项系数), 而当 $m>p$, $m \\in \\mathbf{N}^*$ 时, $\\Delta^m f(x) \\equiv 0$.\n反过来, 对函数 $f(x)(x \\in \\mathbf{R})$, 如果 $\\Delta^{p+1} f(x) \\equiv 0$, 那么 $f(x)$ 是关于 $x$ 的一个次数不超过 $p$ 的多项式.\n将这些结论应用于高阶等差数列, 我们有定理 2 数列 $\\left\\{a_n\\right\\}$ 是一个 $p$ 阶等差数列的充要条件是数列的通项 $a_n$ 为 $n$ 的一个 $p$ 次多项式.", + "figures": [] +} \ No newline at end of file diff --git a/processed_dataset/text/0209.json b/processed_dataset/text/0209.json new file mode 100644 index 0000000000000000000000000000000000000000..36035b86f41005f41c56e4b9f368ff2b2cb13a5e --- /dev/null +++ b/processed_dataset/text/0209.json @@ -0,0 +1,5 @@ +{ + "source_file": "./raw_volume-zh/volume6/chapter1-7.tex", + "text": "递推数列.\n如果数列 $\\left\\{a_n\\right\\}$ 的第 $n$ 项 $a_n$ 由它的前面若干项所确定, 那么该数列就是一个递推数列.\n事实上, 等差数列与等比数列都是递推数列, 它们满足的递推关系式分别是 $a_n=2 a_{n-1}-a_{n-2}$ 和 $a_n=a_{n-1} \\cdot q$.\n一般地, 如果\n$$\na_{n+k}=F\\left(a_n, a_{n+1}, \\cdots, a_{n+k-1}\\right), \\label{eq1}\n$$\n即 $a_{n+k}$ 是 $a_n, a_{n+1}, \\cdots, a_{n+k-1}$ 的函数, 并且初始值 $a_1, \\cdots, a_k$ 是确定的, 那么称数列 $\\left\\{a_n\\right\\}$ 是一个 $k$ 阶递推数列, 式\\ref{eq1}称为 $\\left\\{a_n\\right\\}$ 的递推公式.\n与递推数列相关的问题有两大类:一类是已知递推公式求数列的通项 (或其他性质); 另一类是利用递推思想, 先建立递推公式再去讨论问题的本质.\n下面先讨论一些工具性结果.\n称满足下述递推公式的数列 $\\left\\{a_n\\right\\}$ 为常系数齐次线性递推数列\n$$\na_{n+k}=c_1 a_{n+k-1}+\\dot{c_2} a_{n+k-2}+\\cdots+c_k a_n, \\label{eq2}\n$$\n其中 $c_1, c_2, \\cdots, c_k$ 为常数.\n注意到,如果 $\\lambda$ 是方程\n$$\n\\lambda^k=c_1 \\lambda^{k-1}+c_2 \\lambda^{k-2}+\\cdots+c_k . \\label{eq3}\n$$\n的根, 那么数列 $\\left\\{\\lambda^n\\right\\}(n=1,2, \\cdots)$ 满足递推式\\ref{eq2}, 进一步, 如果式\\ref{eq3}的根两两不同,设为 $\\lambda_1, \\lambda_2, \\cdots, \\lambda_k$, 那么数列 $\\left\\{A_1 \\lambda_1^n+A_2 \\lambda_2^n+\\cdots+A_k \\lambda_k^n\\right\\}(n=1,2, \\cdots)$ 是满足式\\ref{eq2}的数列, 并且可以通过初始条件 $a_1, a_2, \\cdots, a_k$ 确定其中的系数 $A_1$, $A_2, \\cdots, A_k$ (解一个线性方程组), 这样我们就得到了满足式\\ref{eq2}及给定初始值 $a_1$, $a_2, \\cdots, a_k$ 的数列的通项.\n上述求解线性递推数列的方法称为 \"特征根法\", 其中式\\ref{eq3}称为 \\ref{eq2}的特征方程.\n在式\\ref{eq3}出现重根时结论相对复杂些, 我们会在例子中予以阐述.\n与之相对的, 还可以利用母函数方法来处理,一般地, 对数列 $\\left\\{a_n\\right\\}(n= 0,1,2, \\cdots)$, 我们称下面的形式级数\n$$\nf(x)=a_0+a_1 x+a_2 x^2+\\cdots\n$$\n为数列 $\\left\\{a_n\\right\\}$ 的母函数.\n例如: 常数数列 $a_n=-1, n=0,1,2, \\cdots$ 的母函数为 $f(x)=1+x+ x^2+\\cdots=\\frac{1}{1-x}(|x|<1)$, 它就是无穷递缩等比数列的求和公式.\n由于母函数方法中涉及级数收玫等高等数列方面的知识, 这里我们不加证明的给出下面的形式级数公式, 并且通过例子来说明使用的方法.\n对 $\\alpha \\in \\mathbf{R}$, 定义 $\\left(\\begin{array}{l}\\alpha \\\\ n\\end{array}\\right)=\\frac{\\alpha(\\alpha-1) \\cdots(\\alpha-n+1)}{n !}, n \\in \\mathbf{N}$ (它是组合数的一个推广, 在上一节的差分多项式中已经涉及, 这里还规定 $\\left.\\left(\\begin{array}{l}\\alpha \\\\ 0\\end{array}\\right)=1\\right)$. 则\n$$\n(1+x)^\\alpha=1+\\left(\\begin{array}{l}\n\\alpha \\\\\n\\end{array}\\right) x+\\left(\\begin{array}{l}\n\\alpha \\\\\n\\end{array}\\right) x^2+\\cdots . \\label{eq4}\n$$\n特别地, 当 $\\alpha \\in \\mathbf{N}^*$ 时, 式\\ref{eq4} 即为二项式定理.\n对于其他形式的递推数列没有统一的处理方法, 常见的处理方法还有不动点方法等.", + "figures": [] +} \ No newline at end of file diff --git a/processed_dataset/text/0210.json b/processed_dataset/text/0210.json new file mode 100644 index 0000000000000000000000000000000000000000..4aa415ad4aa4e909229183ece964fd041d2d534a --- /dev/null +++ b/processed_dataset/text/0210.json @@ -0,0 +1,5 @@ +{ + "source_file": "./raw_volume-zh/volume6/chapter1-8.tex", + "text": "周期数列.\n对一个数列 $\\left\\{a_n\\right\\}$, 如果存在正整数 $T$ 及 $n_0$, 使得对任意 $n \\geqslant n_0$, 都有 $a_n= a_{n+\\mathrm{T}}$, 那么称 $\\left\\{a_n\\right\\}$ 是一个周期数列.\n进一步, 若 $n_0=1$, 则称 $\\left\\{a_n\\right\\}$ 是一个纯周期数列.\n这里 $T$ 称为 $\\left\\{a_n\\right\\}$ 的一个周期.\n由周期数列的定义可知, 如果 $T$ 为 $\\left\\{a_n\\right\\}$ 的一个周期, 那么对任意 $m \\in \\mathbf{N}^*$, 数 $m T$ 也是 $\\left\\{a_n\\right\\}$ 的一个周期.\n利用这个性质结合数论中著名的 Bezout 定理可得下面的定理:\n定理 1 如果 $T_1 、 T_2$ 都是周期数列 $\\left\\{a_n\\right\\}$ 的周期, 那么 $\\left(T_1 、 T_2\\right)$ (指 $T_1$ 、 $T_2$ 的最大公因数) 也是 $\\left\\{a_n\\right\\}$ 的一个周期.\n由此定理可知, 如果 $\\left\\{a_n\\right\\}$ 是一个周期数列, 那么 $\\left\\{a_n\\right\\}$ 有最小正周期.\n这与周期函数不一定有最小正周期形成鲜明的对比.\n对于一个整数数列 $\\left\\{a_n\\right\\}$ 而言, 它本身可能不是一个周期数列, 但是对某些正整数 $m$, 在模 $m$ 的意义下是一个周期数列, 这就是模周期数列的概念.\n此时, 存在 $T 、 n_0 \\in \\mathbf{N}^*$, 使得对任意 $n \\geqslant n_0$, 都有 $a_{n+T} \\equiv a_n(\\bmod m)$.\n定理 2 整数数列 $\\left\\{a_n\\right\\}$ 如果是一个常系数递推数列, 那么对任意 $m \\in \\mathbf{N}^*$, 数列 $\\left\\{a_n\\right\\}$ 都是模 $m$ 下的一个周期数列.\n事实上,如果 $\\left\\{a_n\\right\\}$ 是一个常系数 $k$ 阶递推数列, 考察下面的数组\n$$\n\\left(a_1, a_2, \\cdots, a_k\\right),\\left(a_2, a_3, \\cdots, a_{k+1}\\right), \\cdots . \\label{eq1}\n$$\n由于在模 $m$ 的意义下, 数组 $\\left(x_1, \\cdots, x_k\\right)$ 中每个 $x_i$ 只取 $0,1,2, \\cdots, m-1$, 故式\\ref{eq1}中的数组在模 $m$ 的意义下至多只有 $m^k$ 种不同的情形.\n所以, 存在 $r 、 t \\in \\mathbf{N}^*(r))\n所谓九点圆, 是指三角形的九个特殊点:三个垂心在三边上的投影、三边中点、三个顶点与垂心的连线中点, 它们在一个圆上.\n这个问题在相似观点下几乎是显然的, 读者可以试着证明: 以上提到的 9 个点,全部位于以 $O H$ 中点为圆心, 外接圆半径的一半为半径的圆上.\n事实上,这两个圆位似,位似中心为 $H$, 位似比为 $1: 2$.\n位似是一种特殊的相似.\n所谓位似图形是指: 如果两个图形不仅是相似图形, 且对应点连线线相交于一点, 那么这样的两个图形叫做位似图形, 位似图形对应点连线的交点是位似中心.\n位似图形的任意一对对应点与位似中心在同一直线上, 它们到位似中心的距离之比等于相似比.\n位似图形的性质有:\n1. 位似图形对应线段的比等于相似比.\n2. 位似图形的对应角都相等.\n3. 位似图形对应点连线的交点是位似中心.\n4. 位似图形面积的比等于相似比的平方.\n5. 位似图形高、周长的比都等于相似比.", + "figures": [ + "./images/volume7/figures/fig-c1i1.png" + ] +} \ No newline at end of file diff --git a/processed_dataset/text/0219.json b/processed_dataset/text/0219.json new file mode 100644 index 0000000000000000000000000000000000000000..8394574ee8c485b5b5baae1dce42f0d72e3ff9f9 --- /dev/null +++ b/processed_dataset/text/0219.json @@ -0,0 +1,5 @@ +{ + "source_file": "./raw_volume-zh/volume7/chapter10.tex", + "text": "几何不等式是几何问题中难度较大的一类.\n想解决这类问题不光得有平面几何知识,还得有深厚的代数功底.\n因此,一般出现在较高层次的竞赛中, 如 IMO、CMO、国家集训队考试中.\n下面介绍其中比较经典的一些问题.", + "figures": [] +} \ No newline at end of file diff --git a/processed_dataset/text/0220.json b/processed_dataset/text/0220.json new file mode 100644 index 0000000000000000000000000000000000000000..0d10fed3018a81a0beacfa86fc0ca017141de1ab --- /dev/null +++ b/processed_dataset/text/0220.json @@ -0,0 +1,5 @@ +{ + "source_file": "./raw_volume-zh/volume7/chapter11.tex", + "text": "平面几何中的其他方法和问题选讲.\n除了前面几章介绍的内容之外, 还有一些方法也是平面几何中常用的, 比如: 同一法、代数法、复数法、向量法、解析法等.\n本章就这几种方法各举若干事例.\n同一法.", + "figures": [] +} \ No newline at end of file diff --git a/processed_dataset/text/0221.json b/processed_dataset/text/0221.json new file mode 100644 index 0000000000000000000000000000000000000000..8b8022830ca25da88caaab36fd8a2ab74b50bd5a --- /dev/null +++ b/processed_dataset/text/0221.json @@ -0,0 +1,11 @@ +{ + "source_file": "./raw_volume-zh/volume7/chapter2.tex", + "text": "三角形中的几个重要定理及其应用.\n梅涅劳斯定理, 塞瓦定理是平面几何中的两个极其重要的定理.\n它们常常联合起来同时使用.\n1. 梅涅劳斯定理: 一直线与 $\\triangle A B C$ 的三边 $A B 、 B C 、 C A$ 或它们的延长线分别相交于 $X 、 Y 、 Z$, 则 $\\frac{A X}{X B} \\cdot \\frac{B Y}{Y C} \\cdot \\frac{C Z}{Z A}=1$.\n事实上,如图() 过 $A 、 B 、 C$ 分别作直线 $X Y Z$ 的垂线, 设垂足分别为 $Q 、 P 、 S$. 由三角形相似有关知识有: : $\\frac{A X}{X B}=\\frac{A Q}{B P}, \\frac{B Y}{Y C}=\\frac{B P}{C S}, \\frac{C Z}{Z A}=\\frac{C S}{A Q}$. 三式相乘即得.\n梅涅劳斯定理的逆定理也成立, 即 \"在 $\\triangle A B C$ 的边 $A B 、 A C 、 B C$ (或其延长线上) 分别取 $X 、 Z 、 Y$. 如果 $\\frac{A X}{X B} \\cdot \\frac{B Y}{Y C} \\cdot \\frac{C Z}{Z A}=1$, 那么 $X, Y, Z$ 三点共线\". 梅氏定理的逆定理常用来证明三点共线.\n2. 塞瓦定理常可分为边元塞瓦定理和角元塞瓦定理.\n边元塞瓦定理: 如图(), $\\triangle A B C$ 内任取一点 $P$, 直线 $A P 、 B P 、 C P$ 分别与边 $B C 、 C A 、 A B$ 相交于点 $D 、 E 、 F$, 则 $\\frac{B D}{D C} \\cdot \\frac{C E}{E A} \\cdot \\frac{A F}{F B}=1$.\n事实上, $\\frac{B D}{D C}=\\frac{S_{\\triangle B P D}}{S_{\\triangle C P D}}=\\frac{S_{\\triangle A B D}}{S_{\\triangle A C D}}=\\frac{S_{\\triangle A B P}}{S_{\\triangle A C P}}$ (用到了分比性质).\n同理: $\\frac{C E}{E A}=\\frac{S_{\\triangle B P C}}{S_{\\triangle A B P}}, \\frac{A F}{F B}=\\frac{S_{\\triangle A C P}}{S_{\\triangle B P C}}$. 三式相乘即得.\n边元塞瓦定理的逆定理也成立.\n即 \"在 $\\triangle A B C$ 的边 $B C 、 C A 、 A B$ 上分别取点 $D 、 E 、 F$, 如果 $\\frac{B D}{D C} \\cdot \\frac{C E}{E A} \\cdot \\frac{A F}{F B}=1$. 那么直线 $A D 、 B E 、 C F$ 三线相交于同一点.\n塞瓦定理的逆定理常被用来证明三线共点.\n角元塞瓦定理: 如图(), 设 $D 、 E 、 F$ 分别是 $\\triangle A B C$ 的三边 $B C 、 C A 、 A B$ 上的点, 三条线段 $A D$ 、 $B E 、 C F$ 交于一点 $M$. 则\n(1) 对 $\\triangle A B C$ 与点 $M$, 有\n$$\n\\frac{\\sin \\angle B A M}{\\sin \\angle M A C} \\cdot \\frac{\\sin \\angle A C M}{\\sin \\angle M C B} \\cdot \\frac{\\sin \\angle C B M}{\\sin \\angle M B A}=1 ;\n$$\n(2) 对 $\\triangle M B C$ 与点 $A$, 有\n$$\n\\frac{\\sin \\angle B M D}{\\sin \\angle D M C} \\cdot \\frac{\\sin \\angle M C A}{\\sin \\angle A C B} \\cdot \\frac{\\sin \\angle C B A}{\\sin \\angle A B M}=1 ;\n$$\n(3) 对 $\\triangle M C A$ 与点 $B$, 有\n$$\n\\frac{\\sin \\angle C M E}{\\sin \\angle E M A} \\cdot \\frac{\\sin \\angle M A B}{\\sin \\angle B A C} \\cdot \\frac{\\sin \\angle A C B}{\\sin \\angle B C M}=1 ;\n$$\n(4) 对 $\\triangle M A B$ 与点 $C$, 有\n$$\n\\frac{\\sin \\angle A M F}{\\sin \\angle F M B} \\cdot \\frac{\\sin \\angle M B C}{\\sin \\angle C B A} \\cdot \\frac{\\sin \\angle B A C}{\\sin \\angle C A M}=1 .\n$$\n像边元塞瓦定理的情形一样, 角元塞瓦定理的逆定理也成立.\n如图(), 过 $\\triangle A B C$ 的三个顶点各引一条异于三角形三边的直线 $A D$ 、 $B E 、 C F$. 若\n$$\n\\frac{\\sin \\angle B A D}{\\sin \\angle D A C} \\cdot \\frac{\\sin \\angle A C F}{\\sin \\angle F C B} \\cdot \\frac{\\sin \\angle C B E}{\\sin \\angle E B A}=1,\n$$\n则 $A D 、 B E 、 C F$ 三线共点或互相平行.\n3. 斯台沃特定理: 如图(), $\\triangle A B C$ 的边 $B C$ 上任取一点 $D$, 若 $B D= u, C D=v, A D=t$, 则\n$$\nt^2=\\frac{b^2 u+c^2 v}{a}-u v\n$$\n事实上,由余弦定理\n$$\n\\cos \\angle A D B=\\frac{u^2+t^2-c^2}{2 u t}, \\cos \\angle A D C=\\frac{t^2+v^2-b^2}{2 t v},\n$$\n$$\nt^2=\\frac{b^2 u+c^2 \\dot{v}}{a}-u v\n$$\n特别地, 当 $A D$ 是 $\\triangle A B C$ 的中线时, $u=v=\\frac{1}{2} a$, 令 $A D=m_a$, 则 $m_a= \\frac{1}{2} \\sqrt{2 b^2+2 c^2-a^2}$, 此即中线长公式; 当 $A D$ 是 $\\triangle A B C$ 的内角平分线时, 由内角平分线性质: $u=\\frac{a c}{b+c}, v=\\frac{a b}{b+c}$, 设 $A D=t_a$, 可得 $t_a=\\frac{2}{b+c} \\cdot \\sqrt{b c \\cdot p(p-a)}$, 这里 $p=\\frac{a+b+c}{2}$. 此即角平分线长公式.", + "figures": [ + "./images/volume7/figures/fig-c2i1.png", + "./images/volume7/figures/fig-c2i2.png", + "./images/volume7/figures/fig-c2i3.png", + "./images/volume7/figures/fig-c2i4.png", + "./images/volume7/figures/fig-c2i5.png" + ] +} \ No newline at end of file diff --git a/processed_dataset/text/0222.json b/processed_dataset/text/0222.json new file mode 100644 index 0000000000000000000000000000000000000000..b4d9377252f09f561ce059e169b70f1a28adc4e6 --- /dev/null +++ b/processed_dataset/text/0222.json @@ -0,0 +1,8 @@ +{ + "source_file": "./raw_volume-zh/volume7/chapter3.tex", + "text": "三角形的重心、垂心、内心、外心、旁心称之为三角形的五心.\n五心有很多重要性质.\n1. 重心:三角形的三条中线交于一点,该点叫做三角形的重心.\n主要性质有:\n(1) 重心到顶点的距离与重心到对边中点的距离之比为 $2: 1$;\n(2) 重心和三角形任意两个顶点组成的 3 个三角形面积相等.\n即重心到三条边的距离与三条边的长成反比;\n(3) 重心到三角形 3 个顶点距离的平方和最小;\n(4) 在平面直角坐标系中, 重心的坐标是顶点坐标的算术平均数.\n即设 $A 、 B$ 、 $C$ 的坐标分别为 $\\left(x_i, y_i\\right)(i=1,2,3)$, 则重心 $G\\left(\\frac{x_1+x_2+x_3}{3}, \\frac{y_1+y_2+y_3}{3}\\right)$.\n2. 垂心:三角形的三条高(所在直线)交于一点,该点叫做三角形的垂心.\n023 与垂心有关性质:\n(1) 三角形三个顶点、三个垂足、垂心这 7 个点可以得到 6 个四点圆;\n(2) 三角形外心 $O$ 、重心 $G$ 、垂心 $H$ 三点共线, 且 $O G: G H=1: 2$; (此直线称为三角形的欧拉线 (Euler line))\n(3) 垂心到三角形一顶点距离等于此三角形外心到此顶点对边距离的 2 倍.\n(可用三角知识证得)\n3. 内心: 三角形内切圆的圆心,叫做三角形的内心.\n主要性质有:\n(1) 三角形的三条内角平分线交于一点,该点即为三角形的内心;\n(2) 直角三角形内心到边的距离等于两直角边的和减去斜边的差的二分之一;\n(3) 三角形的内心到边的距离 (即内切圆的半径 $r$ ) 与三边长及面积之间有关系: $r=\\frac{2 S_{\\triangle}}{a+b+c}$.\n4. 外心: 三角形外接圆的圆心, 叫做三角形的外心.\n有关性质:\n(1) 三角形的三条边的垂直平分线交于一点, 该点即为三角形外心;\n(2) 若 $O$ 是 $\\triangle A B C$ 的外心, 则 $\\angle B O C=2 \\angle A$ ( $\\angle A$ 为锐角或直角) 或\n$\\angle B O C=360^{\\circ}-2 \\angle A$ ( $\\angle A$ 为钝角 $)$;\n(3) 当三角形为锐角三角形时,外心在三角形内部; 当三角形为钝角三角形时,外心在三角形外部; 当三角形为直角三角形时,外心在斜边的中点上;\n(4) 外心到三顶点的距离相等.\n5. 旁心: 三角形的旁边圆 (与三角形的一边和其他两边的延长线相切的圆) 的圆心, 叫做三角形的旁心.\n有关性质:\n(1) 三角形一内角平分线和另外两顶点处的外角平分线交于一点, 该点即为三角形的旁心;\n(2) 每个三角形都有三个旁心;\n(3) 旁心到三边的距离相等;\n(4) 旁心与半周长 $(p)$ 形影不离.\n如图(), $I_A$ 是 $\\triangle A B C$ 的一个旁心.\n作 $I_A E \\perp A B$ 于点 $E, I_A F \\perp A C$ 于点 $F, I_A D \\perp B C$ 于点 $D$. 显然, $B E=B D, C F=C D$, $A E=A F . A E+A F=(A B+B D)+(A C+C D)= A B+B C+A C$. 即 $A E=A F=\\frac{a+b+c}{2}=p$.\n6. 三角形各心之间的相互联系.\n(1) 等腰三角形的内心、外心、重心、垂心共线 (均在对称轴上);\n(2) 等边三角形的内心、外心、重心、垂心共点;\n(3) $\\triangle A B C$ 的内心 $I$ 是切点 $\\triangle D E F$ 的外心;\n(4) $\\triangle A B C$ 的外心 $O$ 是中点 $\\triangle D E F$ 的垂心;\n(5) $\\triangle A B C$ 的垂心 $H$ 是垂足 $\\triangle D E F$ 的内心;\n(6) $\\triangle A B C$ 的重心 $G$ 是中点 $\\triangle D E F$ 的重心;\n(7) 若三角形中同时出现内心、旁心, 就构成了三组三点共线、三组四点共圆.\n如图(), $I$ 为 $\\triangle A B C$ 的内心, $I_A 、 I_B 、 I_C$ 是 $\\triangle A B C$ 的三个旁心.\n显然, $A 、 I 、 I_A$ 等三点共线; $I_A$ 、 $C 、 I 、 B$ 等四点共圆, 且 $I I_A$ 等是三个圆的直径.\n7. 三角形五心的一个向量统一表示:三角形五心有一个向量的表示 $\\lambda_1 \\cdot \\overrightarrow{P A}+\\lambda_2 \\cdot \\overrightarrow{P B}+\\lambda_3 \\cdot \\overrightarrow{P C}=\\overrightarrow{0}\\left(\\lambda_1\\right.$, $\\left.\\lambda_2, \\lambda_3 \\in k\\right)$.\n(1) 当 $P$ 为 $\\triangle A B C$ 的外心 $O$ 时, $\\sin 2 A \\cdot \\overrightarrow{O A}+\\sin 2 B \\cdot \\overrightarrow{O B}+\\sin 2 C \\cdot \\overrightarrow{O C}=\\overrightarrow{0}$; (证明见例 2)\n(2) 当 $P$ 为 $\\triangle A B C$ 的内心 $I$ 时, $\\sin A \\cdot \\overrightarrow{I A}+\\sin B \\cdot \\overrightarrow{I B}+\\sin C \\cdot \\overrightarrow{I C}=\\overrightarrow{0}$;\n(3) 当 $P$ 为非直角三角形的垂心 $H$ 时,\n$$\n\\tan A \\cdot \\overrightarrow{H A}+\\tan B \\cdot \\overrightarrow{H B}+\\tan C \\cdot \\overrightarrow{H C}=\\overrightarrow{0}\n$$\n(4) 当 $P$ 为三角形重心 $G$ 时,则 $\\overrightarrow{G A}+\\overrightarrow{G B}+\\overrightarrow{G C}=\\overrightarrow{0}$;\n(5) 当 $P$ 为三角形旁心 $I$ 时, 则: 对旁心 $I_A$, 有 $-\\sin A \\cdot \\overrightarrow{I_A A}+\\sin B \\cdot \\overrightarrow{I_A B}+ \\sin C \\cdot \\overrightarrow{I_A C}=\\overrightarrow{0}$; 对旁心 $I_B$ 有 $-\\sin B \\cdot \\overrightarrow{I_B B}+\\sin A \\cdot \\overrightarrow{I_B A}+\\sin C \\cdot \\overrightarrow{I_B C}=\\overrightarrow{0}$; 对旁心 $I_C$ 有 $\\sin A \\cdot \\overrightarrow{I_C A}+\\sin B \\cdot \\overrightarrow{I_C B}-\\sin C \\cdot \\overrightarrow{I_C C}=\\overrightarrow{0}$.\n关于旁心的向量性质我们只证(1)(例 2), (2)留作习题, 其余留着大家思考.", + "figures": [ + "./images/volume7/figures/fig-c3i1.png", + "./images/volume7/figures/fig-c3i2.png" + ] +} \ No newline at end of file diff --git a/processed_dataset/text/0223.json b/processed_dataset/text/0223.json new file mode 100644 index 0000000000000000000000000000000000000000..e39ebcc0129385bf1661c3c0a54c18723f3c0681 --- /dev/null +++ b/processed_dataset/text/0223.json @@ -0,0 +1,7 @@ +{ + "source_file": "./raw_volume-zh/volume7/chapter4.tex", + "text": "圆的初步.\n1. 圆的内容非常丰富, 许多平面儿何竞赛问题都和它有关, 其中四点共圆是圆的一个极其重要的问题.\n2. 圆和有关的角:\n(1) 同弧所对的圆周角相等;\n(2) 弦切角等于弦所对的圆周角;\n(3) 顶点在某圆内部的角, 叫做这圆的圆内角.\n圆的圆内角, 等于它本身及其对顶角包含的弧所对的圆周角之和; 顶点在某圆外部而两边与圆均有公共点的角, 叫做这圆的圆外角.\n圆的圆外角, 等于它包含的两弧所对的圆周角之差.\n3. 多值有向角: 我们知道, 射线绕着它的端点依逆时针的方向旋转为正角, 顺时针的方向为负角.\n假定有两直线 $l, l^{\\prime}$, 它们或相交或平行或重合.\n任意选定一点 $O$, 通过 $O$ 作两直线 $h, h^{\\prime}$ 使分别平行 (或重合) 于 $l, l^{\\prime}$, 然后将 $h$ 绕 $O$ 点依任何方向旋转, 而每当 $h$ 重合于 $h^{\\prime}$ 一次, $h$ 便旋过一个角度, 这个角度或小于等于直角, 或大于等于直角, 甚或大于若干周角.\n这些角度视旋转方向为正向或负向而规定它们的值是正的或负的.\n现在我们把这样得到的角度都当作 $l$ 与 $l^{\\prime}$ 所做成角的角度, 并用记号 \" $4 l l, l^{\\prime}$ \"来表示.\n凡两直线做成的角若是按这个方法来测定的, 那么称为多值有向角.\n应该指出, 这样的角只注意于旋转的方向, $l$ 与 $l^{\\prime}$ 本身的正负向是无需给定的.\n又书写记号 \" Х $l, l^{\\prime}$ \"时, 必须注意 $l$ 与 $l^{\\prime}$ 的先后次序, 不得错乱.\n假定两个多值有向角的通值能够一对一地对应相等, 那么我们就说这两个多值有向角相等.\n在这个定义中, 不难晓得多值有向角的相等具有反身性, 对称性,传递性.\n显然,若两个多值有向角相等, 则它们的最小非负值必相等.\n有了多值有向角这个概念, 就可以导出一个有关三点共线的命题: 三点 $A 、 B 、 C$ 共线的充要条件是: $\\measuredangle A B C=0$ 或 $\\backslash P A B=\\measuredangle \\backslash P A C$.\n4. 四点共圆的条件.\n四点 $A 、 B 、 C 、 D$ (不论次序) 共圆的必要且充分条件为 $\\Varangle . A C B= \\measuredangle A D B \\neq 0$.\n此外,切割线定理, 相交弦定理的逆定理都可作为四点共圆的依据.\n5. 与圆有关的两个著名定理:\n(1) 托勒密定理: 在凸四边形 $A B C D$ 中, $A B \\times C D+A D \\times B C \\geqslant A C \\times B D$. 当且仅当四边形 $A B C D$ 是圆内接四边形时,等号成立.\n(2) 西姆松定理: 过三角形外接圆上异于三角形顶点的任意一点作三边的垂线,则三垂足点共线 (此线常称为西姆松线).\n西姆松定理的逆定理也是成立的: 若一点在三角形三边所在直线上的射影共线, 则该点在此三角形的外接圆上.\n6. 密克 (Miquel) 定理: 设在 $\\triangle A B C$ 三边 $B C$ 、 $C A 、 A B$ 所在直线上任取一点 $X 、 Y 、 Z$ (如图()), 则三圆 $\\odot A Y Z 、 \\odot B Z X 、 \\odot C X Y$ 共点.\n事实上,令 $\\odot B Z X$ 与 $\\odot C X Y$ 的第二交点为 $O$, 连结 $O X, O Y 、 O Z$, 则 $\\measuredangle A Y O=\\not C X O=\\measuredangle B Z O$. 因知$O$ 点在 $\\odot A Y Z$.上.", + "figures": [ + "./images/volume7/figures/fig-c4i1.png" + ] +} \ No newline at end of file diff --git a/processed_dataset/text/0224.json b/processed_dataset/text/0224.json new file mode 100644 index 0000000000000000000000000000000000000000..1080a27e62a9292ab0ce4283736ba17a80911e22 --- /dev/null +++ b/processed_dataset/text/0224.json @@ -0,0 +1,7 @@ +{ + "source_file": "./raw_volume-zh/volume7/chapter5.tex", + "text": "圆幂与根轴.\n1. 点到圆的幂: 设 $P$ 是平面上一定点, 从 $P$ 向一定圆作割线, 从 $P$ 起到和圆周相交为止的两有向线段之积,称为 $P$ 点对这一定圆的幕.\n2. 相交弦定理和切割线定理统称为圆幕定理.\n3. 由相交弦定理和切割线定理知, 定点 $P$ 对于定圆的幕是一个定值.\n设 $P O=d$ ( $O$ 为圆心), $\\odot O$ 的半径为 $r$, 则 $d^2-r^2$ 就是点 $P$ 对于 $\\odot O$ 的幂.\n令 $k= d^2-r^2$, 则当 $P$ 在圆外时, $k>0$; 当 $P$ 在圆内时, $k<0$; 当 $P$ 在圆上时, $k=0$.\n与圆幂定理紧密相关的另一个概念是根轴问题.\n下面我们证明有关根轴的一个重要结论.\n4. 对于两已知圆有等幂的点的轨迹是一条垂直于连心线的直线.\n事实上, 设点 $A$ 到 $\\odot O_1$ 和 $\\odot O_2$ 的幕相等, $\\odot O_1 、 \\odot O_2$ 的半径分别为 $R_1$, $R_2\\left(R_1>R_2\\right)$, 则 $A O_1^2-R_1^2=A O_2^2-R_2^2$, 即\n$$\nA O_1^2-A O_2^2=R_1^2-R_2^2=\\text { 常数.\n}\n$$\n如图(), 设 $O_1 O_2$ 的中点为 $D, A M \\perp O_1 O_2$ 于 $M$, 则 $A O_1^2=A M^2+\\left(O_1 D+D M\\right)^2= A M^2+O_1 D^2+D M^2+2 O_1 D \\cdot D M=A D^2+ O_1 D^2+2 O_1 D \\cdot D M$.\n同理 $A O_2^2=A D^2+D O_2^2-2 D O_2 \\cdot D M$, 以上两式相减, $D M=\\frac{R_1^2-R_2^2}{2 O_1 O_2}=$ 常数.\n所以, 过定点 $M$ 的垂线即是两圆等幂点的轨迹.\n这条直线称为两圆的根轴.\n特别地, 若两圆同心 , 则 $O_1 O_2=0$. 即同心圆的根轴不存在; 又若 $R_2=0$, $\\odot O_2$ 变成一点 $O_2$, 则点 $A$ 对于 $\\odot O_2$ 的幕是 $A O_2^2$. 此时, 直线 (轨迹) 称为一圆与一定点的根轴.\n5. 根轴有下面重要性质:\n性质 1 若两圆相交, 其根轴就是公共弦所在的直线.\n性质 2 若两圆相切, 其根轴就是过两圆切点的公切线.\n性质 3 三个圆, 其两两的根轴或相交于一点,或互相平行.\n事实上,若三条根轴中有两条相交, 则这一交点对三个圆的幕均相等, 所以必在第三条根轴上.\n这一点,称为三个圆的根心.\n显然,当三个圆的圆心在一直线上时,三条根轴互相平行; 当三个圆的圆心不共线时,根心存在.", + "figures": [ + "./images/volume7/figures/fig-c5i1.png" + ] +} \ No newline at end of file diff --git a/processed_dataset/text/0225.json b/processed_dataset/text/0225.json new file mode 100644 index 0000000000000000000000000000000000000000..8693376e7336067a6465ee51bc1e058fb4ba9696 --- /dev/null +++ b/processed_dataset/text/0225.json @@ -0,0 +1,5 @@ +{ + "source_file": "./raw_volume-zh/volume7/chapter6.tex", + "text": "一、反射变换.\n反射变换是平面到自身的变换, 若存在一条直线 $l$, 使对于平面上的每一点 $P$ 及其对应点 $P^{\\prime}$, 其连线 $P P^{\\prime}$ 都被定直线 $l$ 垂直平分, 则称这种变换为反射变换, 定直线 $l$ 称为对称轴.\n反射变换有如下性质:\n(1) 把图形变为与之全等的图形;\n(2) 关于 $l$ 对称的两点连线被 $l$ 垂直平分.\n证题过程中使用反射变换, 可保留原有图形的性质, 且使原来分散条件相对集中, 以利于问题的解决.\n二、平移变换.\n平移变换是平面到自身的变换, 将平面上任一点 $P$ 变换到 $P^{\\prime}$, 使得: (1) 射线 $P P^{\\prime}$ 有给定的方向 ; (2) 线段 $P P^{\\prime}$ 有给定的长度.\n则称这种变换为平移变换.\n在平移变换下, 图形变为与之全等的图形,直线变为与之平行的直线.\n在解几何问题时, 常利用平移变换使分散的条件集中在一起, 具有更紧凑的位置关系或变换成更简单的基本图形.\n三、旋转变换.\n旋转变换是平面到它自身的变换, 使原点 $O$ 变换到它自身, 其他任何点 $X$ 变到 $X^{\\prime}$, 使得: (1) $O X^{\\prime}=O X$; (2) $\\angle X O X^{\\prime}=\\theta$ (定角). 则称这样的变换为旋转变换, $O$ 称为旋转中心.\n旋转变换保持图形全等, 但图形方位可能有变化.\n在几何解题中, 旋转的作用是使原有图形的性质得以保持, 但改变其位置, 使能组合成新的有利论证的图形.", + "figures": [] +} \ No newline at end of file diff --git a/processed_dataset/text/0226.json b/processed_dataset/text/0226.json new file mode 100644 index 0000000000000000000000000000000000000000..e394ef5617390ac5cb9e48fe62f9ac3c95f10269 --- /dev/null +++ b/processed_dataset/text/0226.json @@ -0,0 +1,5 @@ +{ + "source_file": "./raw_volume-zh/volume7/chapter7.tex", + "text": "三角法是平面几何的基本而又重要的方法之一.\n熟练掌握和运用公式是用三角法证明平面几何问题的基础.\n正弦定理和余弦定理是三角法证明平面几何问题中用得最多的两个基本定理.\n1. 正弦定理: $\\frac{a}{\\sin A}=\\frac{b}{\\sin \\bar{B}}=\\frac{c}{\\sin \\bar{C}}=2 R(a 、 b 、 c$ 是三角形的三边, $R$ 是 $\\triangle A B C$ 的外接圆半径).\n2. 余弦定理: $a^2=b^2+c^2-2 b c \\cos A, b^2=a^2+c^2-2 a c \\cos B, c^2= a^2+b^2-2 a b \\cos C$.\n3. 积化和差公式: $\\cos \\alpha \\cdot \\cos \\beta=\\frac{1}{2}[\\cos (\\alpha+\\beta)+\\cos (\\alpha-\\beta)], \\sin \\alpha \\cos \\beta= \\frac{1}{2}[\\sin (\\alpha+\\beta)+\\sin (\\alpha-\\beta)], \\cos \\alpha \\sin \\beta=\\frac{1}{2}[\\sin (\\alpha+\\beta)-\\sin (\\alpha-\\beta)]$, $\\sin \\alpha \\sin \\beta=-\\frac{1}{2}[\\cos (\\alpha+\\beta)-\\cos (\\alpha-\\beta)]$.\n4. 和差化积公式: $\\sin \\alpha+\\sin \\beta=2 \\sin \\frac{\\alpha+\\beta}{2} \\cos \\frac{\\alpha-\\beta}{2}, \\sin \\alpha-\\sin \\beta= 2 \\cos \\frac{\\alpha+\\beta}{2} \\sin \\frac{\\alpha-\\beta}{2}, \\cos \\alpha+\\cos \\beta=2 \\cos \\frac{\\alpha+\\beta}{2} \\cos \\frac{\\alpha-\\beta}{2}, \\cos \\alpha-\\cos \\beta= -2 \\sin \\frac{\\alpha+\\beta}{2} \\sin \\frac{\\alpha-\\beta}{2}$.\n5. 三倍角公式: $\\sin 3 \\alpha=3 \\sin \\alpha-4 \\sin ^3 \\alpha, \\cos 3 \\alpha=4 \\cos ^3 \\alpha-3 \\cos \\alpha$,\n$$\n\\begin{aligned}\n& \\sin \\alpha \\sin \\left(60^{\\circ}+\\alpha\\right) \\sin \\left(60^{\\circ}-\\alpha\\right)=\\frac{1}{4} \\cdot \\sin 3 \\alpha, \\cos \\alpha \\cdot \\cos \\left(60^{\\circ}+\\alpha\\right) \\cdot \\cos \\left(60^{\\circ}-\\alpha\\right)= \\\\\n& \\frac{1}{4} \\cos 3 \\alpha, \\tan \\alpha \\cdot \\tan \\left(60^{\\circ}+\\alpha\\right) \\cdot \\tan \\left(60^{\\circ}-\\alpha\\right)=\\tan 3 \\alpha .\n\\end{aligned}\n$$\n6. 三角形中的恒等式很多, 其中用得较多的有:\n$$\n\\begin{aligned}\n& \\cos ^2 A+\\cos ^2 B+\\cos ^2 C=1-2 \\cos A \\cos B \\cdot \\cos C, \\\\\n& \\tan A+\\tan B+\\tan C=\\tan A \\cdot \\tan B \\cdot \\tan C, \\\\\n& \\tan \\frac{A}{2} \\tan \\frac{B}{2}+\\tan \\frac{B}{2} \\tan \\frac{C}{2}+\\tan \\frac{C}{2} \\cdot \\tan \\frac{A}{2}=1 .\n\\end{aligned}\n$$\n7. 设 $r 、 R$ 分别为 $\\triangle A B C$ 的内切圆, 外接圆半径, 则有 $\\frac{r}{R}=4 \\sin \\frac{A}{2}$\n$$\n\\sin \\frac{B}{2} \\sin \\frac{C}{2}=\\cos A+\\cos B+\\cos C-1 \\text {. }\n$$\n事实上, 设 $I$ 是内心, $\\triangle B I C$ 中, $r=B I \\sin \\frac{B}{2}$, 而由正弦定理, $\\frac{B I}{\\sin \\frac{C}{2}}= \\frac{B C}{\\sin \\angle B I C}=\\frac{2 R \\sin A}{\\sin \\left(\\frac{A}{2}+\\frac{\\pi}{2}\\right)}$, 所以 $r=\\frac{2 R \\sin A}{\\cos \\frac{A}{2}} \\cdot \\sin \\frac{C}{2} \\cdot \\sin \\frac{B}{2}=4 R \\sin \\frac{A}{2} \\sin \\frac{B}{2} \\sin \\frac{C}{2}$, 所以 $\\frac{r}{R}=4 \\sin \\frac{A}{2} \\sin \\frac{B}{2} \\sin \\frac{C}{2}$.", + "figures": [] +} \ No newline at end of file diff --git a/processed_dataset/text/0227.json b/processed_dataset/text/0227.json new file mode 100644 index 0000000000000000000000000000000000000000..cf838a90ef334c21ad7fe1a2ae869426828c6773 --- /dev/null +++ b/processed_dataset/text/0227.json @@ -0,0 +1,9 @@ +{ + "source_file": "./raw_volume-zh/volume7/chapter8.tex", + "text": "本节介绍完全四边形以及调和点列的性质.\n完全四边形:\n我们把两两相交, 且没有三线共点的四条直线及它们的六个交点所构成的图形, 叫做完全四边形.\n如图(), 直线 $A B C 、 B D E 、 C D F 、 A F E$ 两两交于 $A 、 B 、 C 、 D 、 E 、 F$ 六点, 则四边形 $A B C D E F$ 即为完全四边形, 线段 $A D 、 B F 、 C E$ 为其三条对角线.\n性质 1 在完全四边形 $A B C D E F$ 中, 四个三角形 $\\triangle A B E 、 \\triangle B C D 、 \\triangle A C F 、 \\triangle D E F$ 的外接圆共点 (这点称为 Miquel 点).\n证明如图(), 设 $\\triangle B C D$ 与 $\\triangle D E F$ 的外接圆除交于点 $D$ 外, 还交于点 $M$.\n设点 $M$ 在直线 $C B 、 C D 、 B D$ 上的射影分别为 $P 、 Q 、 R$.\n由西姆松定理,知 $P 、 Q 、 R$ 三点共线.\n同样, 点 $M$ 在直线 $D F 、 D E 、 F E$ 上的射影分别为 $Q 、 R 、 S$, 则 $Q 、 R 、 S$ 三点也共线.\n故 $P 、 Q 、 R 、 S$ 四点共线.\n在 $\\triangle A C F$ 中, 点 $P$ 在直线 $A C$ 上, 点 $Q$ 在直线 $C F$ 上, 点 $S$ 在直线 $A F$ 上, 且 $P 、 Q 、 S$ 三点共线, 由西姆松定理的逆定理, 知点 $M$ 在 $\\triangle A C F$ 的外接圆上.\n同理, 点 $M$ 在 $\\triangle A B E$ 的外接圆上.\n故 $\\triangle A B E 、 \\triangle B C D 、 \\triangle A C F 、 \\triangle D E F$ 的四个外接圆共点.\n以下的性质 2 极为重要:\n性质 2 完全四边形的一条对角线所在直线与其他两条对角线所在直线相交,则该线被其他两条对角线所在直线调和分割.\n设四边形 $A B C D$ 是平面四边形,对角线 $A C$ 和 $B D$ 交于点 $P$, 对边 $A B$ 和 $D C 、 A D$ 和 $B C$ 分别交于点 $Q 、 R, A C 、 B D$ 分别与 $Q R$ 交于点 $X 、 Y$, 则 $Q$ 、 $R 、 X 、 Y ; B 、 D 、 P 、 Y ; A 、 C 、 P 、 X$ 均为调和点列.\n证明如图(), 对于直线 $B D Y$ 截 $\\triangle A Q R$ 、 直线 $Q R Y$ 截 $\\triangle A B D$ 、直线 $Q X R$ 截 $\\triangle A B C$, 分别应用梅涅劳斯定理得\n$$\n\\begin{aligned}\n& \\frac{A B}{B Q} \\cdot \\frac{Q Y}{Y R} \\cdot \\frac{R D}{D A}=1, \\label{eq1} \\\\\n& \\frac{A Q}{Q B} \\cdot \\frac{B Y}{Y D} \\cdot \\frac{D R}{R A}=1, \\label{eq2} \\\\\n& \\frac{B Q}{Q A} \\cdot \\frac{A X}{X C} \\cdot \\frac{C R}{R B}=1 . \\label{eq3}\n\\end{aligned}\n$$\n对于点 $C$ 与 $\\triangle A Q R$ 、点 $C$ 与 $\\triangle A B D$ 、点 $D$ 与 $\\triangle A B C$, 分别应用塞瓦定理得\n$$\n\\begin{aligned}\n& \\frac{A B}{B Q} \\cdot \\frac{Q X}{X R} \\cdot \\frac{R D}{D A}=1, \\label{eq4} \\\\\n& \\frac{A Q}{Q B} \\cdot \\frac{B P}{P D} \\cdot \\frac{D R}{R A}=1, \\label{eq5} \\\\\n& \\frac{A P}{P C} \\cdot \\frac{C R}{R B} \\cdot \\frac{B Q}{Q A}=1 . \\label{eq6}\n\\end{aligned}\n$$\n比较式\\ref{eq1}和\\ref{eq4}、式\\ref{eq2}和\\ref{eq5}、式\\ref{eq3}和\\ref{eq6}分别得\n$$\n\\frac{Q Y}{Y R}=\\frac{Q X}{X R}, \\frac{B Y}{Y D}=\\frac{B P}{P D}, \\frac{A X}{X C}=\\frac{A P}{P C} .\n$$\n所以, $Q 、 R 、 X 、 Y ; B 、 D 、 P 、 Y ; A 、 C 、 P 、 X$ 分别为调和点列.\n特别地, 若 $B D / / Q R$, 则视交点 $Y$ 在无穷远处, 此时, $\\frac{B P}{P D}=\\frac{Q X}{X R}=1$.\n上述三组调和点列仍成立.", + "figures": [ + "./images/volume7/figures/fig-c8i1.png", + "./images/volume7/figures/fig-c8i2.png", + "./images/volume7/figures/fig-c8i3.png" + ] +} \ No newline at end of file diff --git a/processed_dataset/text/0228.json b/processed_dataset/text/0228.json new file mode 100644 index 0000000000000000000000000000000000000000..a2075c22b641aef89e6df1e55dad227f5bb0f8cf --- /dev/null +++ b/processed_dataset/text/0228.json @@ -0,0 +1,8 @@ +{ + "source_file": "./raw_volume-zh/volume7/chapter8.tex", + "text": "性质 3 在完全四边形 $A B C D E F$ 中, 若 $G 、 H$ 分别是 $C F 、 B E$ 的中点, 则\n$$\nS_{\\text {四边形 } B C E F}=4 S_{\\triangle A G H} \\text {. }\n$$\n证明如图(), 连结 $C H 、 H F$ 得\n$$\n\\begin{aligned}\nS_{\\triangle A G H} & =S_{\\triangle A C H}-S_{\\triangle C G H}-S_{\\triangle A C G} \\\\\n& =S_{\\triangle A B H}+S_{\\triangle B C H}-\\frac{1}{2} S_{\\triangle C H F}-\\frac{1}{2} S_{\\triangle A C F}\n\\end{aligned}\n$$\n$$\n\\begin{aligned}\n& =\\frac{1}{2} S_{\\triangle A B E}+\\frac{1}{2} S_{\\triangle B C E}-\\frac{1}{2} S_{\\text {四边形 } A C H F} \\\\\n& =\\frac{1}{2} S_{\\text {四边形 } H C E F}=\\frac{1}{4}\\left(S_{\\triangle B E F}+S_{\\triangle B C E}\\right) \\\\\n& ==\\frac{1}{4} S_{\\text {四边形 } B C E F} .\n\\end{aligned}\n$$\n调和点列调和点列是射影几何学的重要内容, 它在平面几何中也有着广泛的应用.\n对于线段 $A B$ 的内分点 $C$ 和外分点 $D$ 满足 $\\frac{A C}{C B}=\\frac{A D}{D B}$, 则称点 $C 、 D$ 调和分割线段 $A B$ 或者 $A 、 B 、 C 、 D$ 是调和点列.\n我们允许无穷远点的存在, 即规定如果 $D$ 为无穷远点, 则 $\\frac{A D}{D B}=1$, 也可以说, 当 $C$ 平分线段 $A B$ 时, $A 、 B 、 C$ 以及直线 $A C$ 上的无穷远点四点成调和点列.\n性质 4 对于线段 $A B$ 的内分点 $C$ 和外分点 $D$ 满足 $C 、 D$ 调和分割线段 $A B, M$ 是线段 $A B$ 的中点,则有以下结论成立:\n(1) 点 $A 、 B$ 调和分割线段 $C D$;\n(2) $\\frac{1}{A C}+\\frac{1}{A D}=\\frac{2}{A B}$;\n(3) $A B \\cdot C D=2 A D \\cdot B C$;\n(4) $C A \\cdot C B=C M \\cdot C D$.\n以上证明并不难,留给读者完成.\n性质 5 对线段 $A B$ 的内分点 $C$ 和外分点 $D$ 以及直线 $A B$ 外一点 $P$, 给出四个论断:\n(1) $P C$ 是 $\\angle A P B$ 的平分线;\n(2) $P D$ 是 $\\angle A P B$ 的外角平分线;\n(3) $C 、 D$ 调和分割线段 $A B$;\n(4) $P C \\perp P D$.\n以上四个论断中, 任意选取两个作题设、另外两个作结论组成的六个命题均为真命题.\n这里仅对由论断 (3) 和 (4) 作题设, (1) 和 (2) 作结论的命题给出证明.\n证明如图(), 不妨设 $\\angle A P C=\\alpha$, $\\angle B P C=\\beta$.\n由 $P C \\perp P D$ 知\n$$\n\\angle A P D=90^{\\circ}+\\alpha, \\angle B P D=90^{\\circ}-\\beta .\n$$\n故 $\\frac{A C}{C B}=\\frac{P A \\sin \\angle A P C}{P B \\sin \\angle B P C}=\\frac{P A \\sin \\alpha}{P B \\sin \\beta}$,\n$\\frac{A D}{D B}=\\frac{P A \\sin \\angle A P D}{P B \\sin \\angle B P D}=\\frac{P A}{P B} \\frac{\\cos \\alpha}{\\cos \\beta}$.\n所以 $\\frac{\\sin \\alpha}{\\sin \\beta}=\\frac{\\cos \\alpha}{\\cos \\beta}$, 即 $\\alpha=\\beta$.\n因此,结论(1)成立.\n接下来易证结论 (2), 略.", + "figures": [ + "./images/volume7/figures/fig-c8i4.png", + "./images/volume7/figures/fig-c8i5.png" + ] +} \ No newline at end of file diff --git a/processed_dataset/text/0229.json b/processed_dataset/text/0229.json new file mode 100644 index 0000000000000000000000000000000000000000..29911069b473681eca53437e5564574caf154d31 --- /dev/null +++ b/processed_dataset/text/0229.json @@ -0,0 +1,13 @@ +{ + "source_file": "./raw_volume-zh/volume7/chapter8.tex", + "text": "性质 6 如图(), 过 $O$ 引出四条给定的直线, 直线 $L$ 与这四条直线相交, 交点分别为 $A 、 B$ 、 $C 、 D$, 则 $\\overrightarrow{A B} / \\overrightarrow{A B} / \\overrightarrow{C D}$ 为定值, 这个比例称为交比.\n证明如图设角, 则 $\\frac{\\overrightarrow{A B}}{\\overrightarrow{C B}}=-\\frac{S_{\\triangle O A B}}{S_{\\triangle O B C}}= -\\frac{O A \\cdot \\sin \\alpha}{O C \\cdot \\sin \\beta}$\n同理, $\\frac{\\overrightarrow{A D}}{\\overrightarrow{C D}}=-\\frac{O A \\cdot \\sin (\\alpha+\\beta+\\gamma)}{O C \\cdot \\sin \\gamma}$.\n所以 $\\frac{\\overrightarrow{A B} / \\overrightarrow{C B}}{\\overrightarrow{A D} / \\overrightarrow{C D}}=\\frac{\\sin \\alpha \\cdot \\sin \\gamma}{\\sin \\beta \\cdot \\sin (\\alpha+\\beta+\\gamma)}$ 为定值.\n特别地, 若上述定值为 -1 时, 则 $A 、 C 、 B 、 D$ 成调和点列.\n此时称直线 $O A 、 O B 、 O C 、 O D$ 成调和线束.\n容易发现,共点的四条直线成调和线束的充要条件是任作一不过它们交点的直线截四条直线所得的交点成调和点列.\n(注意: 如果该直线与四条直线之一平行, 命题仍有效, 这时有一点为无穷远点)\n由此可得定理 1 如图(), 设过 $O$ 的线束 $O A 、 O B 、 O C 、 O D$ 分别交不过 $O$ 的两条直线 $l_1$ 与 $l_2$ 于 $A 、 B 、 C 、 D 、 A^{\\prime} 、 B^{\\prime} 、 C^{\\prime} 、 D^{\\prime}$, 其中 $A^{\\prime}$ 在直线 $O A$ 上, 等等.\n那么 $A 、 B 、 C 、 D$ 成调和点列的充要条件是 $A^{\\prime} 、 B^{\\prime} 、 C^{\\prime} 、 D^{\\prime}$ 成调和点列.\n以后使用该结论时统一称 $O$ 为中心.\n下面介绍两个比较常用的基本图形:\n如图(), 若线段 $A B$ 的中点为 $C, O$ 为直线 $A B$ 外一点, 则 $O A 、 O C$ 、 $O B$ 以及过 $O$ 且平行于 $A B$ 的直线成调和线束.\n如图(), 若四条直线 $l_1 、 l_2 、 l_3 、 l_4$ 成调和线束, 则 $l_1 \\perp l_3$ 的充要条件是 $l_2 、 l_4$ 与 $l_3$ 的夹角相等.\n性质 7 设 $A 、 B 、 C 、 D$ 共线, 则 $A 、 B 、 C 、 D$ 为调和点列的充要条件是, 从线段 $C D$ 的中点 $O$ 起, 截同向线段 $O A$ 及 $O B$, 使这线段的一半长为比例中项, 即 $O C^2=O A \\cdot O B$. (如图() 所示)\n推论 1 一圆的直径被另一圆周调和分割的充要条件是,这两个圆正交.\n(两圆正交是指过它分别作两圆切线,则这两条线垂直)\n推论 2 如图() 设点 $C$ 是 $\\triangle A E F$ 的内心, 角平分线 $A C$ 交边 $E F$ 于点 $B$, 射线 $A B$ 交 $\\triangle A E F$ 的外接圆于点 $O$, 则射线 $A B$ 上的点 $D$ 为 $\\triangle A E F$ 的旁心的充要条件是 $\\frac{A C}{C B}=\\frac{D O}{O B}$.\n事实上, 若 $D$ 为 $\\triangle A E F$ 的旁心, 如图(), 则易知, 三角形的角平分线被其内心和相应的旁心调和分割, 于是有 $\\frac{A C}{C B}=\\frac{A D}{D B}$. 显然 $C 、 E 、 D 、 F$ 共圆.\n且圆心为 $O$. 于是 $\\frac{A C}{C B}=\\frac{A D}{D B}=\\frac{A D-A C}{D B-C B}$ (分比定理) $= \\frac{C D}{(D O+O B)-(O C-O B)}=\\frac{C D}{2 O B}=\\frac{2 O D}{2 O B}=\\frac{O D}{O B}$. 反之, 若 $\\frac{A C}{C B}==\\frac{D O}{O B}$, 可用同一法证得 $D$ 为 $\\triangle A E F$ 的旁心.", + "figures": [ + "./images/volume7/figures/fig-c8i6.png", + "./images/volume7/figures/fig-c8i7.png", + "./images/volume7/figures/fig-c8i8.png", + "./images/volume7/figures/fig-c8i9.png", + "./images/volume7/figures/fig-c8i10.png", + "./images/volume7/figures/fig-c8i11.png", + "./images/volume7/figures/fig-c8i11.png" + ] +} \ No newline at end of file diff --git a/processed_dataset/text/0230.json b/processed_dataset/text/0230.json new file mode 100644 index 0000000000000000000000000000000000000000..29193dbd33c1e0a69aeb1df658a06734289a37c3 --- /dev/null +++ b/processed_dataset/text/0230.json @@ -0,0 +1,7 @@ +{ + "source_file": "./raw_volume-zh/volume7/chapter9.tex", + "text": "本章我们将介绍反演变换与配极理论.\n配极与前面所讲的调和点列有着密不可分的关联, 作为对与上章的补充.\n反演是一种全新的几何变换, 它的性质很独特,主要作用是将大量的圆变成直线,减少图形的复杂性.\n反演定义 设 $O$ 是平面 $\\pi$ 上的一个定点, $k$ 是一个非零常数,如果平面 $\\pi$ 的一个变换, 使得对于平面 $\\pi$ 上任意异于 $O$ 的点 $A$ 与其像点 $A^{\\prime}$, 恒有\n(i) $A^{\\prime} 、 O 、 A$ 共线;\n(ii) $\\overrightarrow{O A^{\\prime}} \\cdot \\overrightarrow{O A}=k$.\n则这个变换称为平面 $\\pi$ 的一个反演变换, 记作 $I(O, k)$, 其中定点 $O$ 称为反演中心, 常数 $k$ 称为反演幕, 点 $A^{\\prime}$ 称为 $A$ 的反点.\n这里要注意, 反演中心本身不参与反演变换.\n反演后, 反演中心 $O$ 仍记为 $O$, 位置不动.\n当反演幂 $k>0$ 时, 反演变换 $I(O, k)$ 称为双曲型反演变换; 当 $k<0$ 时, 反演变换 $I(O, k)$ 称为椭圆型反演变换.\n对于反演变换 $I(O, k)$, 令 $r=\\sqrt{|k|}$, 则以反演中心 $O$ 为圆心, $r$ 为半径的圆称为反演变换 $I(O, k)$ 的反演圆或基圆, $r$ 称为反演半径.\n显然, 当点 $A^{\\prime}$ 是点 $A$ 的反点时, 点 $A$ 也是点 $A^{\\prime}$ 的反点, 因而点 $A$ 与点 $A^{\\prime}$ 互为反点,由此可见,反演变换是可逆的,且其逆变换就是自身.\n平面 $\\pi$ 上的图形 $F$ 在反演变换下的像 $F^{\\prime}$ 称为图形 $F$ 关于这个反演变换的反形,简单图形 $F^{\\prime}$ 是图形 $F$ 的反形.\n显然, 如果图形 $F^{\\prime}$ 是图形 $F$ 的反形, 则图形 $F$ 是图形 $F^{\\prime}$ 的反形, 因而图形 $F$ 与图形 $F^{\\prime}$ 互为反形.\n反演变换的不动点称为自反点, 而反演变换的不变图形称为自反图形.\n定理 1 设 $A 、 B$ 为平面上两点且 $A 、 B 、 O$ 不共线, 在反演变换 $I(O, k)$ 下, 设 $A 、 B$ 两点的反点分别为 $A^{\\prime} 、 B^{\\prime}$, 则 $A 、 B 、 A^{\\prime} 、 B^{\\prime}$ 四点共圆.\n证明如图(), 设 $A \\stackrel{I(O, k)}{\\rightarrow} A^{\\prime}, B \\stackrel{I(O, k)}{\\rightarrow} B^{\\prime}$, 且 $A 、 B 、 A^{\\prime} 、 B^{\\prime}$ 不共线, 由反演变换的定义, 有 $\\overrightarrow{O A^{\\prime}} \\cdot \\overrightarrow{O A}=k=\\overrightarrow{O B^{\\prime}} \\cdot \\overrightarrow{O B}$, 故 $A 、 B 、 A^{\\prime} 、 B^{\\prime}$ 共圆.\n定理 2 在反演变换 $I(O, k)$ 下, 设 $A 、 B$ (均不同于反演中心 $O)$ 两点的反点分别为 $A^{\\prime} 、 B^{\\prime}$, 则有 $A^{\\prime} B^{\\prime}=\\frac{|k|}{O A \\cdot O B} \\cdot A B$.\n证明若 $O 、 A 、 B$ 共线, 则由 $\\overrightarrow{O A^{\\prime}} \\cdot \\overrightarrow{O A}=k, \\overrightarrow{O B^{\\prime}} \\cdot \\overrightarrow{O B}=k$, 可得 $\\overrightarrow{A^{\\prime} B^{\\prime}}=\\overrightarrow{O B^{\\prime}}-\\overrightarrow{O A^{\\prime}}=\\frac{k}{\\overrightarrow{O B}}-\\frac{k}{\\overrightarrow{O A}}=\\frac{k(\\overrightarrow{O A}-\\overrightarrow{O B})}{\\overrightarrow{O A} \\cdot \\overrightarrow{O B}}=\\frac{k \\overrightarrow{B A}}{\\overrightarrow{O A} \\cdot \\overrightarrow{O B}}$.\n若 $O 、 A 、 B$ 不共线, 则由 $\\triangle O B^{\\prime} A^{\\prime}$ 相似 $\\triangle O A B$, 有\n$$\n\\frac{A^{\\prime} B^{\\prime}}{A B}=\\frac{O A^{\\prime}}{O B}=\\frac{O A \\cdot O A^{\\prime}}{O A \\cdot O B}=\\frac{|k|}{O A \\cdot O B} .\n$$\n由此可见,无论哪种情形,结论都成立.\n定理 3 除反演中心外, 平面上的每一个点, 都有唯一的反演点, 且这种关系是对称的, 即如果点 $P$ 是 $P^{\\prime}$ 的反演点, 那么, $P^{\\prime}$ 也是 $P$ 的反演点.\n位于反演圆上的点, 保持在原处; 位于反演圆内的点, 变换为圆外部的点; 位于反演圆外的点, 变换为圆内部的点.\n定理 4 设 $P$ 为反演圆 $O(r)$ 外的一点, 则它的反演点 $P^{\\prime}$ 是 $O P$ 与 $P$ 到圆的切线的切点连线的交点.\n定理 5 过反演中心的直线反演后为自身.\n(这条直线不包含反演中心, 即挖去反演中心)任意一条不过反演中心的直线, 它的反形是经过反演中心的圆, 反之亦然.\n特别地, 过反演中心相交的圆, 变为不过反演中心的相交直线.\n定理 6 不过反演中心的圆, 它的反形是一个圆, 反演中心是这两个互为反形的圆的一个位似中心, 任一对反演点是逆对应点.\n定理 7 两条直线或曲线的夹角在反演变换下是不变的(两条曲线之间的夹角是指它们的切线之间的夹角).", + "figures": [ + "./images/volume7/figures/fig-c9i1.png" + ] +} \ No newline at end of file diff --git a/processed_dataset/text/0231.json b/processed_dataset/text/0231.json new file mode 100644 index 0000000000000000000000000000000000000000..5bec2d5340839478bc3f372f7ff7b0d701dbf19c --- /dev/null +++ b/processed_dataset/text/0231.json @@ -0,0 +1,8 @@ +{ + "source_file": "./raw_volume-zh/volume7/chapter9.tex", + "text": "配极定义 在平面上取定一个以 $O$ 为圆心、 $r$ 为半径的圆.\n对于不同于 $O$ 的任一点 $P$, 作一直线 $l$ 通过 $P$ 的反演像 $P^{\\prime}$ (即 $O 、 P 、 P^{\\prime}$ 三点共线, 且 $O P \\left.O P^{\\prime}=r^2\\right)$ 且垂直于射线 $O P$. 则称直线 $l$ 为点 $P$ 的极线, $P$ 为直线 $l$ 的极点.\n性质 1 若点 $A$ 在 $B$ 的极线上, 则点 $B$ 在 $A$ 的极线上.\n这时称 $A 、 B$ 共轭.\n(这是因为 $A$ 在 $B$ 的极线上意味着 $A B^{\\prime} \\perp B B^{\\prime}$, 如图(), 而 $O B^{\\prime} \\cdot O B=r^2=O A^{\\prime}$ ・ $O A$, 故 $A 、 B^{\\prime} 、 B 、 A^{\\prime}$ 四点共圆, 从而 $A A^{\\prime} \\perp A^{\\prime} B$ )\n性质 2 若点 $P$ 在圆 $O$ 之外, 过 $P$ 作圆 $O$ 的两条切线与圆 $O$ 切于点 $M 、 N$, 则 $M N$ 是 $P$ 的极线.\n$(M 、 N$ 的极线分别是过 $M 、 N$ 的圆 $O$ 的切线, 均过 $P$,于是 $P$ 的极线过 $M 、 N$ )\n性质 3 若过圆 $O$ 外一点 $P$ 作一直线与圆 $O$ 交于点 $R 、 S$, 线段 $R S$ 与 $P$ 的极线交于点 $Q$, 则 $(P 、 S 、 Q 、 R)$ 为调和点列.\n定理 1 过一点 $A$ 任作两割线交圆 $O$ 于 $P 、 Q$ 和 $R 、 S$, 连结 $P R$ 与 $Q S$ 、 $P S$ 与 $Q R$ 分别交于 $B 、 C$, 则 $B C$ 必是 $A$ 关于圆 $O$ 的极线.\n证明如图(), 设直线 $B C$ 与直线 $A S$ 交于 $N$, 与直线 $A Q$ 交于 $M$, 利用上一章性质 2 , 即完全四边形的调和分割性知, $S 、 R 、 N$ 、 $A$ 成调和点列, 结合性质 3 以及确定三点后, 第四调和点的唯一性, $A$ 的极线过 $N$, 同理, $A$ 的极线过 $M$,于是 $A$ 的极线为 $M N$, 所以 $B C$ 是 $A$ 的极线.\n我们称一个圆内接四边形为调和四边形, 如果它满足对边乘积相等.\n通过正弦定理以及调和线束中的 $1=\\frac{\\overrightarrow{A B} / \\overrightarrow{C B}}{\\overrightarrow{A D} / \\overrightarrow{C D}}=\\frac{\\sin \\alpha \\cdot \\sin \\gamma}{\\sin \\beta \\cdot \\sin (\\alpha+\\beta+\\gamma)}$, 我们发现:\n定理 2 圆 $O$ 内接四边形 $A B C D$ 为调和四边形的充要条件是对圆上一点 $P, P A 、 P C, P B 、 P D$ 成调和线束.\n不难证明:\n定理 3 对圆外一点 $P$, 过 $P$ 作圆 $O$ 的两条切线, 切点分别是 $A 、 B$, 再任作割线 $P C D$ 交圆 $O$ 于 $C 、 D$, 则 $A C B D$ 为调和四边形.", + "figures": [ + "./images/volume7/figures/fig-c9i2.png", + "./images/volume7/figures/fig-c9i3.png" + ] +} \ No newline at end of file diff --git a/processed_dataset/text/0232.json b/processed_dataset/text/0232.json new file mode 100644 index 0000000000000000000000000000000000000000..78c2c169bbee58ca3553163930099c508d55c94e --- /dev/null +++ b/processed_dataset/text/0232.json @@ -0,0 +1,5 @@ +{ + "source_file": "./raw_volume-zh/volume8/chapter1.tex", + "text": "复数的概念及代数运算.\n复数概念的引人最初是为了求解\n$$\nx^2+1=0\n$$\n这样的没有实根的方程, 因此复数集可以看作实数集的一个自然的扩充.\n为此, 首先引进一个\"新数\" $i$, 使它满足\n$$\n\\mathrm{i}^2=-1,\n$$\n即 $\\mathrm{i}$ 适合方程 $x^2+1=0$. 这个新数 $\\mathrm{i}$ 称为虚数单位.\n将 $\\mathrm{i}$ 添加到实数集中去, 定义:形如 $z=a+b \\mathrm{i}$ ( $a 、 b$ 均是实数) 的表达式称为一个复数.\n其中的 $a$ 和 $b$ 分别叫做复数 $z$ 的实部和虚部, 分别记作\n$$\na=\\operatorname{Re}(z), b=\\operatorname{Im}(z) .\n$$\n一、复数 $z=a+b \\mathrm{i}(a 、 b \\in \\mathrm{R})$ 的分类.\n当虚部 $b=0$ 时,复数 $z$ 是实数;\n当虚部 $b \\neq 0$ 时,复数 $z$ 是虚数;\n当虚部 $b \\neq 0$, 且实部 $a=0$ 时,复数 $z$ 是纯虚数.\n如果记\n$\\mathbf{R}$ 一一实数集\n$\\mathbf{C}$ 一一复数集\n$\\mathbf{P}$ 一一虚数集\n$\\mathbf{Q}$ 一一纯虚数集\n就有关系\n$$\n\\mathbf{R} \\cap \\mathbf{P}=\\varnothing \\quad \\mathbf{R} \\cup \\mathbf{P}=\\mathbf{C} \\quad \\mathbf{Q} \\varsubsetneqq \\mathbf{P} \\varsubsetneqq \\mathbf{C}\n$$\n二、复数相等的充要条件.\n对于两个复数 $z_1=a+b \\mathrm{i}(a 、 b \\in \\mathbf{R}), z_2=c+d \\mathrm{i}(c 、 d \\in \\mathbf{R})$, 二者相等的充要条件是 $a=c$ 且 $b=d$, 即\n$$\na+b \\mathrm{i}=c+d \\mathrm{i} \\Leftrightarrow\\left\\{\\begin{array}{l}\na=c, \\\\\nb=d .\n\\end{array}\\right.\n$$\n复数相等的充要条件是复数问题化归为实数问题的理论依据, \"化虚为实\"是解决复数问题的通性通法.\n三、复数的运算法则.\n对于两个复数 $a+b \\mathrm{i} 、 c+d \\mathrm{i}(a, b, c, d \\in \\mathbf{R})$.\n加法: $(a+b \\mathrm{i})+(c+d \\mathrm{i})=(a+c)+(b+\\dot{d}) \\mathrm{i}$;\n减法: $(a+b \\mathrm{i})-(c+d \\mathrm{i})=(a-c)+(b-d) \\mathrm{i}$;\n乘法: $(a+b \\mathrm{i})(c+d \\mathrm{i})=(a c-b d)+(b c+a d) \\mathrm{i}$;\n除法: $\\frac{a+b \\mathrm{i}}{c+\\bar{d}}=\\frac{a c+b d}{c^2+d^2}+\\frac{b c-a d}{c^2+d^2} \\mathrm{i}(c+d \\mathrm{i} \\neq 0)$.\n四、复数的运算定律.\n复数的加法满足交换律、结合律, 也就是说, 对于任何复数 $z_1 、 z_2 、 z_3$, 均有\n$$\n\\begin{gathered}\nz_1+z_2=z_2+z_1, \\\\\n\\left(z_1+z_2\\right)+z_3=z_1+\\left(z_2+z_3\\right) .\n\\end{gathered}\n$$\n复数的乘法满足交换律、结合律, 以及乘法对于加法的分配律.\n也就是说, 对于复数 $z_1 、 z_2 、 z_3$,均有\n$$\n\\begin{aligned}\nz_1 z_2 & =z_2 z_1, \\\\\n\\left(z_1 z_2\\right) z_3 & =z_1\\left(z_2 z_3\\right), \\\\\nz_1\\left(z_2+z_3\\right) & =z_1 z_2+z_1 z_3 .\n\\end{aligned}\n$$\n五、共轭复数的性质.\n当两个复数的实部相等, 虚部互为相反数时, 就称其互为共轭复数.\n特别地, 若复数的虚部不为零时, 也称作互为共轭虚数.\n对于复数 $z=a+b \\mathrm{i}(a$ 、 $b \\in \\mathbf{R})$, 它的共轭复数用 $\\bar{z}=a-b \\mathrm{i}(a 、 b \\in \\mathbf{R})$ 来表示.\n共轭复数有如下基本性质:\n(1) $\\overline{z_1 \\pm z_2}=\\overline{z_1} \\pm \\overline{z_2}$;\n(2) $\\overline{z_1 z_2}=\\overline{z_1} \\overline{z_2}$;\n(3) $\\overline{\\left(\\frac{z_1}{z_2}\\right)}=\\frac{\\overline{z_1}}{\\overline{z_2}}\\left(z_2 \\neq 0\\right)$;\n(4) $\\overline{z^n}=(\\bar{z})^n$;\n(5) $z+\\bar{z}=2 \\operatorname{Re}(z), z-\\bar{z}=2 \\mathrm{i} \\operatorname{Im}(z)$;\n(6) $\\overline{\\bar{z}}=z$;\n(7) $z$ 是实数的充要条件是 $\\bar{z}=z ; z$ 是纯虚数的充要条件是 $\\bar{z}=-z$ 且 $z \\neq 0$.\n六、复数的几何形式.\n复数 $a+b \\mathrm{i}(a 、 b \\in \\mathbf{R})$ 与复平面上的点 $Z(a, b)$ 是一一对应的, 点 $Z(a, b)$ 和向量 $\\overrightarrow{O Z}$ 也构成一一对应关系, 点 $Z$ 和间量 $\\overrightarrow{O Z}$ 均是复数 $z=a+b \\mathrm{i}$ 的几何形式.\n向量 $\\overrightarrow{O Z}$ 的模 $r$ 称为复数 $z=a+b \\mathrm{i}$ 的模 $|z|$, 即\n$$\nr=|z|=\\sqrt{a^2+b^2} .\n$$\n这种对应关系的构建, 揭示了复数问题与向量问题之间的相互转化, 说明了向量方法是解决复数问题的一条有效途径.\n关于复数的模,有如下的基本性质:\n(1) $z \\bar{z}=|z|^2=|\\bar{z}|^2$;\n(2) ||$z_1|-| z_2|| \\leqslant\\left|z_1 \\pm z_2\\right| \\leqslant\\left|z_1\\right|+\\left|z_2\\right|$;\n(3) $|z| \\geqslant \\max \\{|\\operatorname{Re}(z)|,|\\operatorname{Im}(z)|\\}$.", + "figures": [] +} \ No newline at end of file diff --git a/processed_dataset/text/0233.json b/processed_dataset/text/0233.json new file mode 100644 index 0000000000000000000000000000000000000000..6f4385640ad2d7818391d69f6de4b05f9bf37785 --- /dev/null +++ b/processed_dataset/text/0233.json @@ -0,0 +1,5 @@ +{ + "source_file": "./raw_volume-zh/volume8/chapter2.tex", + "text": "复数的模与幅角.\n关于复数的模的概念, 在第一章中已有定义.\n在复数的三角形式表示中, 出现了复数的辐角的概念.\n有关复数辐角的问题是近年高中数学竞赛的热点问题之一.\n面给出复数辐角的定义和一些性质.\n设复数 $z=a+b \\mathrm{i}(a, b \\in \\mathbf{R})$ 所对应的向量为 $\\overrightarrow{O Z}$, 我们称始边是 $x$ 轴正半轴, 终边是 $\\overrightarrow{O Z}$ 的角称为复数 $z$ 的辐角, 记为 $\\operatorname{Arg} z$. 在 $[0,2 \\pi)$ 内的辐角叫做复数 $z$ 的辐角主值, 记为 $\\arg z$. 且有\n$$\n\\operatorname{Arg} z=\\arg z+2 k \\pi(k \\in \\mathbf{Z}) .\n$$\n当 $a \\in \\mathbf{R}^{+}$时,有\n$$\n\\begin{gathered}\n\\arg a=0, \\arg (-a)=\\pi, \\\\\n\\arg (a \\mathrm{i})=\\frac{\\pi}{2}, \\arg (-a \\mathrm{i})=\\frac{3 \\pi}{2} .\n\\end{gathered}\n$$\n0 的辐角是任意的.\n非零复数与它的模和辐角主值构成一一对应关系.\n两个非零复数相等, 当且仅当它们的模与幅角主值分别相等.\n关于复数辐角的运算,有如下结论:\n$$\n\\begin{gathered}\n\\operatorname{Arg}\\left(z_1 z_2\\right)=\\operatorname{Arg}\\left(z_1\\right)+\\operatorname{Arg}\\left(z_2\\right), \\\\\n\\operatorname{Arg}\\left(\\frac{z_1}{z_2}\\right)=\\operatorname{Arg}\\left(z_1\\right)-\\operatorname{Arg}\\left(z_2\\right)\\left(z_2 \\neq 0\\right), \\\\\n\\operatorname{Arg}\\left(z^n\\right)=n \\operatorname{Arg}(z)(n \\in \\mathbf{Z}) .\n\\end{gathered}\n$$\n若复数 $z=a+b \\mathrm{i}(a 、 b \\in \\mathbf{R}, a b \\neq 0)$, 则\n$$\n\\arg z= \\begin{cases}\\arctan \\frac{b}{a}, & \\text { 点 }(a, b) \\text { 在第 I 象限 } \\\\ \\pi+\\arctan \\frac{b}{a}, & \\text { 点 }(a, b) \\text { 在第 II 、III 象限 } \\\\ 2 \\pi+\\arctan \\frac{b}{a}, \\text { 点 }(a, b) \\text { 在第 IV 象限 }\\end{cases}\n$$\n有了上述准备工作, 我们可以定义复数的三角形式:\n设复数 $z=a+b \\mathrm{i}(a 、 b \\in \\mathbf{R})$ 的模等于 $r$, 辐角等于 $\\theta$, 则称 $z=r(\\cos \\theta+ i \\sin \\theta)$ 为复数 $z=a+b \\mathrm{i}(a 、 b \\in \\mathbf{R})$ 的三角形式.\n以下介绍复数在三角形式下的乘法、乘方、除法、开方等运算的法则.\n一、复数的乘法与乘方.\n若 $z_1=r_1\\left(\\cos \\theta_1+\\mathrm{i} \\sin \\theta_1\\right), z_2=r_2\\left(\\cos \\theta_2+\\mathrm{i} \\sin \\theta_2\\right)$, 则\n$$\n\\begin{aligned}\nz_1 z_2 & =r_1\\left(\\cos \\theta_1+\\mathrm{i} \\sin \\theta_1\\right) \\cdot r_2\\left(\\cos \\theta_2+\\mathrm{i} \\sin \\theta_2\\right) \\\\\n& =r_1 r_2\\left[\\cos \\left(\\theta_1+\\theta_2\\right)+\\mathrm{i} \\sin \\left(\\theta_1+\\theta_2\\right)\\right] .\n\\end{aligned}\n$$\n两个复数相乘,积的模等于各复数的模的积, 积的辐角等于各复数的辐角的和.\n若 $z=r(\\cos \\theta+i \\sin \\theta), n \\in \\mathbf{N}^*$, 则\n$$\nz^n=[r(\\cos \\theta+\\mathrm{i} \\sin \\theta)]^n=r^n(\\cos n \\theta+\\mathrm{i} \\sin n \\theta) .\n$$\n复数 $n$ 次幂的模等于这个复数模的 $n$ 次幂, 它的辐角等于这个复数辐角的 $n$ 倍, 这个定理叫做棣莫佛 (Abraham de Moivre, 1667-1754 年)定理.\n二、复数的除法.\n若 $z_1=r_1\\left(\\cos \\theta_1+i \\sin \\theta_1\\right), z_2=r_2\\left(\\cos \\theta_2+i \\sin \\theta_2\\right)$, 则\n$$\n\\begin{aligned}\n\\frac{z_1}{z_2} & =\\frac{r_1\\left(\\cos \\theta_1+\\mathrm{i} \\sin \\theta_1\\right)}{r_2\\left(\\cos \\theta_2+\\mathrm{i} \\sin \\theta_2\\right)} \\\\\n& =\\frac{r_1}{r_2}\\left[\\cos \\left(\\theta_1-\\theta_2\\right)+\\mathrm{i} \\sin \\left(\\theta_1-\\theta_2\\right)\\right] .\n\\end{aligned}\n$$\n两个复数相除, 商的模等于被除数的模除以除数的模所得的商, 商的辐角等于被除数的辐角减去除数的辐角.\n三、复数的开方.\n复数 $r(\\cos \\theta+\\mathrm{i} \\sin \\theta)$ 的 $n\\left(n \\in \\mathbf{N}^*\\right)$ 次方根是\n$$\n\\sqrt[n]{r}\\left(\\cos \\frac{\\theta+2 k \\pi}{n}+\\mathrm{i} \\sin \\frac{\\theta+2 k \\pi}{n}\\right)(k=0,1, \\cdots, n-1) .\n$$\n复数的 $n$ 次方根是 $n$ 个复数, 它们的模都等于这个复数的模的 $n$ 次算术根, 它们的辐角分别等于这个复数的辐角与 $2 \\pi$ 的 $0,1, \\cdots, n-1$ 倍的和的 $n$ 分之一.\n由此可知,方程 $x^n=b(b \\in \\mathbf{C})$ 的根的几何意义是复平面内的 $n$ 个点, 这些点均匀分布在以原点为圆心 、以 $\\sqrt[n]{\\mid b T}$ 为半径的圆周上.\n四、辐角的三角函数.\n设复数 $z=\\cos \\theta+\\mathrm{i} \\sin \\theta=\\mathrm{e}^{\\mathrm{i} \\theta}$, 则\n$$\n\\begin{gathered}\n\\cos n \\theta=\\operatorname{Re}\\left(z^n\\right)=\\frac{z^{2 n}+1}{2 z^n} ; \\\\\n\\sin n \\theta=\\operatorname{Im}\\left(z^n\\right)=\\frac{z^{2 n}-1}{2 z^n \\mathrm{i}} ; \\\\\n\\tan n \\theta=\\frac{\\operatorname{Im}\\left(z^n\\right)}{\\operatorname{Re}\\left(z^n\\right)}=\\frac{z^{2 n}-1}{\\left(z^{2 n}+1\\right) \\mathrm{i}} .\n\\end{gathered}\n$$", + "figures": [] +} \ No newline at end of file diff --git a/processed_dataset/text/0234.json b/processed_dataset/text/0234.json new file mode 100644 index 0000000000000000000000000000000000000000..5d8ce80ee7a8e059213b777ee7d3b34083eb45d4 --- /dev/null +++ b/processed_dataset/text/0234.json @@ -0,0 +1,5 @@ +{ + "source_file": "./raw_volume-zh/volume8/chapter3.tex", + "text": "复数与方程.\n在复数集中,有关方程的试题常考常新, 对于复系数方程, 其韦达定理仍然适用,而实系数方程的虚根以共轭形式成对出现.\n一、实系数方程 $a x^2+b x+c(a \\neq 0)$ 在复数集 $\\mathrm{C}$ 中有两个根\n$$\nx=\\frac{-b \\pm \\mathrm{i} \\sqrt{-\\left(b^2-4 a c\\right)}}{2 a}\\left(b^2-4 a c<0\\right) .\n$$\n二、复平面上的曲线方程.\n如果复数 $z$ 对应着复平面上一点 $Z(x, y)$, 就可得出一些常用曲线的复数形式的方程:\n(1) 方程 $\\left|z-z_0\\right|=r$ 表示以 $Z_0$ 为圆心 $r$ 为半径的圆.\n(2) 方程 $\\left|z-z_1\\right|=\\left|z-z_2\\right|$ 表示线段 $Z_1 Z_2$ 的垂直平分线.\n(3) 方程 $\\left|z-z_1\\right|+\\left|z-z_2\\right|=2 a\\left(a>0,2 a>\\left|Z_1 Z_2\\right|\\right)$ 表示以 $Z_1 、 Z_2$ 为焦点, $a$ 为长半轴的椭圆.\n若 $2 a=\\left|Z_1 Z_2\\right|$, 则此方程表示以 $Z_1 、 Z_2$ 为端点的线段.\n(4) 方程 ||$z-z_1|-| z-z_2||=2 a\\left(0<2 a<\\left|Z_1 Z_2\\right|\\right)$ 表示以 $Z_1 、 Z_2$ 为焦点, 实轴长为 $2 a$ 的双曲线.\n(5)复平面上的特殊区域.\n用一些复数模的不等式, 就可表示复平面上的特殊区域.\n1) $\\left|z-z_0\\right|0$ 表示复平面的右半平面, $\\operatorname{Im}(z)<0$ 表示复平面的下半平面.", + "figures": [] +} \ No newline at end of file diff --git a/processed_dataset/text/0235.json b/processed_dataset/text/0235.json new file mode 100644 index 0000000000000000000000000000000000000000..847314ca334d20cc6291f6dd1f71197f1520117b --- /dev/null +++ b/processed_dataset/text/0235.json @@ -0,0 +1,5 @@ +{ + "source_file": "./raw_volume-zh/volume8/chapter4.tex", + "text": "向量的加减法.\n一、向量的有关概念.\n1. 向量: 既有大小又有方向的量叫做向量.\n记作 $\\overrightarrow{A B}$, 其中 $A$ 是向量的起点, $B$ 是向量的终点.\n也可以记作 $\\vec{a}$.\n2. 向量的模: 向量 $\\overrightarrow{A B}$ 的大小亦即线段 $A B$ 的长度叫做向量的模, 记作 $|\\overrightarrow{A B}|$ (向量 $\\vec{a}$ 的模记作 $|\\vec{a}|$ ). 向量的模又叫做向量的长度.\n3. 单位向量: 模为 1 的向量叫做单位向量.\n4. 零向量: 模为 0 的向量叫做零向量, 记作 $\\overrightarrow{0}$. 零向量的方向任意, 所有的零向量都相等.\n5. 平行向量: 方向相同或相反的向量叫做平行向量.\n向量 $\\vec{a}$ 和 $\\vec{b}$ 平行记作 $\\vec{a} / / \\vec{b}$. 我们规定 $\\overrightarrow{0}$ 与任一向量平行.\n平行向量又叫做共线向量.\n6. 相等向量: 模相等且方向相同的向量叫做相等向量.\n向量 $\\vec{a}$ 和 $\\vec{b}$ 相等记作 $\\vec{a}=\\vec{b}$. 零向量与零向量相等.\n任意两个相等的非零向量, 都可用同一条有向线段来表示,并且与有向线段的起点无关.\n7. 相反向量: 与 $\\vec{a}$ 模相等, 方向相反的向量, 叫做 $\\vec{a}$ 的相反向量, 记作 $-\\vec{a}$. $\\vec{a}$ 和一 $\\vec{a}$ 互为相反向量.\n我们规定 $\\overrightarrow{0}$ 的相反向量仍是 $\\overrightarrow{0}$. 于是任一向量与它的相反向量之和是零向量, 即 $\\vec{a}+(-\\vec{a})=\\overrightarrow{0}$.\n8. 向量的夹角: 已知两个非零向量 $\\vec{a}$ 和 $\\vec{b}$, 作 $\\overrightarrow{O A}=\\vec{a}, \\overrightarrow{O B}=\\vec{b}$, 则 $\\angle A O B=\\theta\\left(0^{\\circ} \\leqslant \\theta \\leqslant 180^{\\circ}\\right)$ 叫做向量 $\\vec{a}$ 和 $\\vec{b}$ 的夹角.\n向量 $\\vec{a}$ 和 $\\vec{b}$ 的夹角也记作 $\\langle\\vec{a}, \\vec{b}\\rangle$.\n二、向量的运算.\n1. 向量的加法: 已知向量 $\\vec{a}, \\vec{b}$, 在平面内任取一点 $A$, 作 $\\overrightarrow{A B}=\\vec{a}, \\overrightarrow{B C}= \\vec{b}$, 则向量 $\\overrightarrow{A C}$ 叫做向量 $\\vec{a}$ 与 $\\vec{b}$ 的和, 记作 $\\vec{a}+\\vec{b}$, 即 $\\vec{a}+\\vec{b}=\\overrightarrow{A B}+\\overrightarrow{B C}=\\overrightarrow{A C}$.\n求两个向量和的运算, 叫做向量的加法.\n对于零向量和任一向量 $\\vec{a}$, 有 $\\vec{a}+\\overrightarrow{0}=\\overrightarrow{0}+\\vec{a}=\\vec{a}$.\n以同一点 $A$ 为起点的两个已知向量 $\\vec{a} 、 \\vec{b}$ 为邻边作平行四边形 $A B C D$, 则以 $A$ 为起点的对角线 $\\overrightarrow{A C}$ 就是 $\\vec{a}$ 与 $\\vec{b}$ 的和, 我们把这种作两个向量和的方法叫做向量加法的平行四边形法则.\n而前面根据向量加法的定义得出的求向量和的方法, 称为向量加法的三角形法则.\n这个法则可以推广到多个向量的求和一一多边形法则.\n2. 向量的减法: 向量 $\\vec{a}$ 加上 $\\vec{b}$ 的相反向量, 叫做 $\\vec{a}$ 与 $\\vec{b}$ 的差.\n即\n$$\n\\vec{a}-\\vec{b}=\\vec{a}+(-\\vec{b}) .\n$$\n求两个向量差的运算, 叫做向量的减法.\n因为 $(\\vec{a}-\\vec{b})+\\vec{b}=\\vec{a}+(-\\vec{b})+\\vec{b}=\\vec{a}+\\overrightarrow{0}=\\vec{a}$, 所以求 $\\vec{a}-\\vec{b}$ 就是求这样一个量, 它与 $\\vec{b}$ 的和等于 $\\vec{a}$. 因此可得如下求 $\\vec{a}-\\vec{b}$ 的作图方法.\n已知 $\\vec{a}$ 和 $\\vec{b}$, 在平面内任取一点 $O$, 作 $\\overrightarrow{O A}=\\vec{a}, \\overrightarrow{O B}=\\vec{b}$, 则 $\\overrightarrow{B A}=\\vec{a}-\\vec{b}$. 即 $\\vec{a}-\\vec{b}$ 可以表示为从向量 $\\vec{b}$ 的终点指向向量 $\\vec{a}$ 的终点的向量.\n3. 实数与向量的积: 实数 $\\lambda$ 与向量 $\\vec{a}$ 的积是一个向量, 记作 $\\lambda \\vec{a}$, 它的模与方向规定如下:\n(1) $|\\lambda \\vec{a}|=|\\lambda||\\vec{a}|$;\n(2)当 $\\lambda>0$ 时, $\\lambda \\vec{a}$ 的方向与 $\\vec{a}$ 相同; 当 $\\lambda<0$ 时, $\\lambda \\vec{a}$ 的方向与 $\\vec{a}$ 相反; 当 $\\lambda=0$ 时, $\\lambda \\vec{a}=\\overrightarrow{0}$.\n4. 向量的数量积: 已知两个非零向量 $\\vec{a}$ 和 $\\vec{b}$, 它们的夹角为 $\\theta$, 我们把数量 $|\\vec{a}||\\vec{b}| \\cos \\theta$ 叫做 $\\vec{a}$ 与 $\\vec{b}$ 的数量积, 记作 $\\vec{a} \\cdot \\vec{b}$, 即\n$$\n\\vec{a} \\cdot \\vec{b}=|\\vec{a}||\\vec{b}| \\cos \\theta=|\\vec{a}||\\vec{b}| \\cos \\langle\\vec{a}, \\vec{b}\\rangle .\n$$\n并且规定,零向量与任一向量的数量积为 0 . 向量的数量积又叫做内积.\n设 $\\overrightarrow{O A}=\\vec{a}, \\overrightarrow{O B}=\\vec{b}$, 过点 $B$ 作 $B B_1$ 垂直于直线 $O A$, 垂足为 $B_1$, 则\n$$\nO B_1=|\\vec{b}| \\cos \\theta\n$$\n$|\\vec{b}| \\cos \\theta$ 叫做向量 $\\vec{b}$ 在 $\\vec{a}$ 方向上的投影, 当 $\\theta$ 为锐角时, 它是正值; 当 $\\theta$ 为钝角时, 它是负值; 当 $\\theta$ 为直角时, 它是 0 . 当 $\\theta=0^{\\circ}$ 时, 它是 $|\\vec{b}|$; 当 $\\theta=180^{\\circ}$ 时, 它是 $-|\\vec{b}|$.\n因此, 我们得到 $\\vec{a} \\cdot \\vec{b}$ 的几何意义:数量积 $\\vec{a} \\cdot \\vec{b}$ 等于 $\\vec{a}$ 的长度 $|\\vec{a}|$ 与 $\\vec{b}$ 在 $\\vec{a}$ 方向上的投影 $|\\vec{b}| \\cos \\theta$ 的乘积.\n三、向量的运算法则.\n1. 加法的交换律: $\\vec{a}+\\vec{b}=\\vec{b}+\\vec{a}$;\n加法的结合律: $(\\vec{a}+\\vec{b})+\\vec{c}=\\vec{a}+(\\vec{b}+\\vec{c})$.\n2. $\\lambda(\\mu \\vec{a})=(\\lambda \\mu) \\vec{a}$,\n分配律: $(\\lambda+\\mu) \\vec{a}=\\lambda \\vec{a}+\\mu \\vec{a}$;\n分配律: $\\lambda(\\vec{a}+\\vec{b})=\\lambda \\vec{a}+\\lambda \\vec{b}$.\n3. 数量积的交换律: $\\vec{a} \\cdot \\vec{b}=\\vec{b} \\cdot \\vec{a}, \\vec{a} \\cdot(\\lambda \\vec{b})=\\lambda(\\vec{a} \\cdot \\vec{b})$.\n分配律: $(\\vec{a}+\\vec{b}) \\cdot \\vec{c}=\\vec{a} \\cdot \\vec{c}+\\vec{b} \\cdot \\vec{c}$.\n4. 平方公式:\n$$\n\\begin{aligned}\n& (\\vec{a}+\\vec{b})^2=\\vec{a}^2+2 \\vec{a} \\cdot \\vec{b}+\\vec{b}^2, \\\\\n& (\\vec{a}-\\vec{b})^2=\\vec{a}^2-2 \\vec{a} \\cdot \\vec{b}+\\vec{b}^2 .\n\\end{aligned}\n$$\n5. 平方差公式:\n$$\n(\\vec{a}+\\vec{b}) \\cdot(\\vec{a}-\\vec{b})=\\vec{a}^2-\\vec{b}^2 .\n$$\n四、向量的共线与垂直.\n1. 不共线的四点 $A 、 B 、 C 、 D$ 组成平行四边形的充要条件是 $\\overrightarrow{A B}=\\overrightarrow{C D}$ 或 $\\overrightarrow{A B}=\\overrightarrow{D C}$.\n2. 向量 $\\vec{b}$ 与非零向量 $\\vec{a}$ 共线的充要条件是有且仅有一个实数 $\\lambda$, 使得 $\\vec{b}=\\lambda \\vec{a}$.\n3. 两个非零向量 $\\vec{a} 、 \\vec{b}$ 垂直的充要条件是 $\\vec{a} \\cdot \\vec{b}=0$.\n4. 对于共线三点 $P_1 、 P_2 、 P$ 一定存在实数 $\\lambda$, 使得 $\\overrightarrow{P_1 P}=\\lambda \\vec{P} \\overrightarrow{P_2}$, 若 $P_1$ 、 $P_2$ 是已知点, 则点 $P$ 位置由 $\\lambda$ 确定, $\\lambda>0$ 时, $P$ 为 $\\overrightarrow{P_1 P_2}$ 内分点; $\\lambda<0$ 时, $P$ 为 $\\overrightarrow{P_1 P_2}$ 外分点; $|\\lambda|=\\frac{\\left|\\overrightarrow{P_1 P}\\right|}{\\left|\\overrightarrow{P_2}\\right|}$, 称 $\\lambda$ 为 $P$ 分 $\\overrightarrow{P_1 P_2}$ 所成的比.\n并且有\n$$\n\\overrightarrow{O P}=\\frac{1}{1+\\lambda} \\overrightarrow{O P_1}+\\frac{\\lambda}{1+\\lambda} \\overrightarrow{O P_2}\n$$", + "figures": [] +} \ No newline at end of file diff --git a/processed_dataset/text/0236.json b/processed_dataset/text/0236.json new file mode 100644 index 0000000000000000000000000000000000000000..1af69134e0fbed270cd46b3c817f3fa9495cd05b --- /dev/null +++ b/processed_dataset/text/0236.json @@ -0,0 +1,5 @@ +{ + "source_file": "./raw_volume-zh/volume8/chapter5.tex", + "text": "关于向量内积的定义在第 4 章中已有介绍, 在本章中, 我们着重通过例题来说明向量内积在向量问题及各种其它问题中的应用.", + "figures": [] +} \ No newline at end of file diff --git a/processed_dataset/text/0237.json b/processed_dataset/text/0237.json new file mode 100644 index 0000000000000000000000000000000000000000..a8768f34d4fbd5b1b1f5806583700555f0dddd7c --- /dev/null +++ b/processed_dataset/text/0237.json @@ -0,0 +1,5 @@ +{ + "source_file": "./raw_volume-zh/volume8/chapter6.tex", + "text": "本章中,我们将利用空间向量解决各种立体几何中的计算问题.", + "figures": [] +} \ No newline at end of file diff --git a/processed_dataset/text/0238.json b/processed_dataset/text/0238.json new file mode 100644 index 0000000000000000000000000000000000000000..d5cf1fb88bb50fba0f1f8a6eb7e2e5b3fb26e5cf --- /dev/null +++ b/processed_dataset/text/0238.json @@ -0,0 +1,5 @@ +{ + "source_file": "./raw_volume-zh/volume8/chapter7.tex", + "text": "单位根及其应用.\n对于方程\n$$\nx^n-1=0,\\left(n \\in \\mathbf{N}^*, n \\geqslant 2\\right)\n$$\n由复数开方法则, 就得到它的 $n$ 个根\n$$\n\\varepsilon_k=\\cos \\frac{2 k \\pi}{n}+\\operatorname{isin} \\frac{2 k \\pi}{n} . \\quad(k=0,1,2, \\cdots, n-1)\n$$\n它们显然是 1 的 $n$ 次方根, 称为 $n$ 次单位根.\n利用复数乘方公式,有\n$$\n\\varepsilon_k=\\left(\\cos \\frac{2 \\pi}{n}+i \\sin \\frac{2 \\pi}{n}\\right)^k=\\varepsilon_1^k .\n$$\n这说明, $n$ 个 $n$ 次单位根可以表示为\n$$\n1, \\varepsilon_1, \\varepsilon_1^2, \\cdots, \\varepsilon_1^{n-1} .\n$$\n关于 $n$ 次单位根, 有如下一些性质:\n(1) $\\left|\\varepsilon_k\\right|=1 . \\quad(k \\in \\mathbf{N})$\n(2) $\\varepsilon_j \\varepsilon_k=\\varepsilon_{j+k} . \\quad(j, k \\in \\mathbf{N})$\n(3) $1+\\varepsilon_1+\\varepsilon_1^2+\\cdots+\\varepsilon_1^{n-1}=0 . \\quad(n \\geqslant 2)$\n(4) 设 $m$ 是整数,则\n$$\n1+\\varepsilon_1^m+\\varepsilon_2^m+\\cdots+\\varepsilon_{n-1}^m=\\left\\{\\begin{array}{l}\nn, \\text { 当 } m \\text { 是 } n \\text { 的倍数时; } \\\\\n0, \\text { 当 } m \\text { 不是 } n \\text { 的倍数时.\n}\n\\end{array}\\right.\n$$", + "figures": [] +} \ No newline at end of file diff --git a/processed_dataset/text/0239.json b/processed_dataset/text/0239.json new file mode 100644 index 0000000000000000000000000000000000000000..afe6c1578622e3b16504a3d91e773f5c208b6b51 --- /dev/null +++ b/processed_dataset/text/0239.json @@ -0,0 +1,5 @@ +{ + "source_file": "./raw_volume-zh/volume8/chapter8.tex", + "text": "复数的模与幅角.\n本章中, 我们将通过例题介绍一些关于复数的模和幅角的较高难度的技巧和方法.", + "figures": [] +} \ No newline at end of file diff --git a/processed_dataset/text/0240.json b/processed_dataset/text/0240.json new file mode 100644 index 0000000000000000000000000000000000000000..1ba52c0a08d1df40e1051bd0e6a78f8680fb6453 --- /dev/null +++ b/processed_dataset/text/0240.json @@ -0,0 +1,5 @@ +{ + "source_file": "./raw_volume-zh/volume8/chapter9.tex", + "text": "复数与向量的应用.\n本章中主要介绍复数与向量的一些应用, 特别是其在平面几何中的应用.\n另外还将运用复数来解决一类函数的迭代问题.\n复数的几何意义构建了代数与几何之间的相互联系, 当中的要害之处在于怎样选取恰当的坐标系, 进而建立几何元素的复数表示, 以借助复数的运算来探究平面几何问题的解决方案.\n一、设复平面上两点 $Z_1 、 Z_2$ 对应的复数分别是 $z_1 、 z_2$, 那么这两点间的距离满足\n$$\n\\begin{aligned}\n\\left|Z_1 Z_2\\right|^2 & =\\left|z_1-z_2\\right|^2 \\\\\n& =\\left(z_1-z_2\\right)\\left(\\overline{z_1}-\\overline{z_2}\\right) \\\\\n& =\\left|z_1\\right|^2+\\left|z_2\\right|^2-\\left(z_1 \\overline{z_2}+\\overline{z_1} z_2\\right) .\n\\end{aligned}\n$$\n二、设复平面上两点 $Z_1 、 Z_2$ 对应的复数分别是 $z_1 、 z_2$, 那么线段 $Z_1 Z_2$ 定比分点 $Z$ 对应的复数 $z$ 可以表示为\n$$\nz=\\frac{z_1+\\lambda z_2}{1+\\lambda} .(\\lambda \\in \\mathbf{R}, \\lambda \\neq-1)\n$$\n三、设复平面上三点 $Z_1 、 Z_2 、 Z_3$ 对应的复数分别是 $z_1 、 z_2 、 z_3$, 这三点共线的充要条件是存在不全为零的实数 $\\lambda_1 、 \\lambda_2 、 \\lambda_3$, 使如下两式同时成立:\n$$\n\\left\\{\\begin{array}{l}\n\\lambda_1+\\lambda_2+\\lambda_3=0, \\\\\n\\lambda_1 z_1+\\lambda_2 z_2+\\lambda_3 z_3=0 .\n\\end{array}\\right.\n$$\n四、设不共线的四点 $A 、 B 、 C 、 D$ 对应的复数分别是 $z_1 、 z_2 、 z_3 、 z_4$, 则 $A 、 B 、 C 、 D$ 四点共圆的充要条件是\n$$\n\\frac{z_3-z_1}{z_4-z_1}: \\frac{z_3-z_2}{z_4-z_2}=\\lambda\n$$\n其中 $\\lambda$ 是非零实数.\n五、设不共线的三点 $A 、 B 、 C$ 对应的复数分别是 $z_1 、 z_2 、 z_3$, 则 $\\triangle A B C$\n的面积公式是\n$$\nS_{\\triangle A B C}=\\frac{\\mathrm{i}}{4} \\cdot\\left|\\begin{array}{ccc}\n1 & 1 & 1 \\\\\nz_1 & z_2 & z_3 \\\\\n\\overline{z_1} & \\overline{z_2} & \\overline{z_3}\n\\end{array}\\right| .\n$$", + "figures": [] +} \ No newline at end of file diff --git a/processed_dataset/text/0241.json b/processed_dataset/text/0241.json new file mode 100644 index 0000000000000000000000000000000000000000..0240aa7bc035b9520f70af6537569c19994e7cb0 --- /dev/null +++ b/processed_dataset/text/0241.json @@ -0,0 +1,5 @@ +{ + "source_file": "./raw_volume-zh/volume9/chapter1.tex", + "text": "距离不等式中的化直法.\n在几何量 (长度、角度、面积、体积等) 的大小比较中, 线段长度的比较是最基本的.\n我们把仅涉及到线段长度的几何不等式叫做距离不等式.\n欧氏几何中一些简单的不等公理和定理常常是解决距离不等式的出发点, 其中最常用的工具有:\n命题 1 连接 $A 、 B$ 两点的最短线是线段 $A B$.\n这个命题的一个直接推论就是命题 2 (三角不等式) 如果 $A 、 B 、 C$ 为任意三点, 则 $A B \\leqslant A C+C B$, 当且仅当 $C$ 位于线段 $A B$ 上时等号成立.\n由这个命题还可产生下面一些常用的推论.\n命题 3 三角形中大边对大角, 大角对大边.\n命题 4 三角形中线的长度小于夹它的两边长度之和的一半.\n命题 5 如果一个凸多边形位于另一个凸多边形的内部, 则外面的凸多边形的周长大于里面凸多边形的周长.\n命题 6 凸多边形内的线段长度, 或者不超过凸多边形的最大边长, 或者不超过凸多边形的最大对角线长.\n下面看一些例题.", + "figures": [] +} \ No newline at end of file diff --git a/processed_dataset/text/0242.json b/processed_dataset/text/0242.json new file mode 100644 index 0000000000000000000000000000000000000000..922736d9a3a00c917f0aca71c0a66266fa8861fd --- /dev/null +++ b/processed_dataset/text/0242.json @@ -0,0 +1,5 @@ +{ + "source_file": "./raw_volume-zh/volume9/chapter11.tex", + "text": "四面体中的不等式.\n三角形是平面上最简单的多边形, 四面体是三维空间中最简单的多面体, 因此四面体可看作是三角形在空间的推广.\n三角形中的许多不等式都可推广到四面体中.\n关于四面体的几何不等式和极值问题已有丰富的结果, 这里介绍几个典型例题.", + "figures": [] +} \ No newline at end of file diff --git a/processed_dataset/text/0243.json b/processed_dataset/text/0243.json new file mode 100644 index 0000000000000000000000000000000000000000..3702d857c88b8980eecb1ee74ecfd49ad79b587a --- /dev/null +++ b/processed_dataset/text/0243.json @@ -0,0 +1,7 @@ +{ + "source_file": "./raw_volume-zh/volume9/chapter2.tex", + "text": "Ptolemy 不等式及其应用.\n著名的 Ptolemy 不等式是关于任意四边形的一个距离不等式, 它可表述为\n定理 (Ptolemy 不等式) 在四边形 $A B C D$ 中有\n$$\nA B \\cdot C D+A D \\cdot B C \\geqslant A C \\cdot B D,\n$$\n等号成立当且仅当 $A 、 B 、 C 、 D$ 四点共圆.\n证明如图(), 在四边形 $A B C D$ 内取点 $E$, 使 $\\angle B A E=\\angle C A D, \\angle A B E=\\angle A C D$, 则 $\\triangle A B E \\backsim \\triangle A C D$. 因此 $A B \\cdot C D=A C \\cdot B E$. 又 $\\angle B A C= \\angle E A D$, 且 $\\frac{A B}{A E}=\\frac{A C}{A D}$, 所以 $\\triangle A B C \\backsim \\triangle A E D, A D$ ・ $B C=A C \\cdot D E$. 故\n$$\nA B \\cdot C D+A D \\cdot B C=A C(B E+D E) \\geqslant A C \\cdot B D,\n$$\n等号成立当且仅当点 $E$ 在 $B D$ 上, 此时 $\\angle A B D=\\angle A C D$, 故四边形 $A B C D$ 内接于圆.\n应用 Ptolemy 不等式, 我们可给出一些距离不等式的简洁证明.", + "figures": [ + "./images/volume9/figures/fig-c2i1.png" + ] +} \ No newline at end of file diff --git a/processed_dataset/text/0244.json b/processed_dataset/text/0244.json new file mode 100644 index 0000000000000000000000000000000000000000..951592a11cbf8928cf1cc91f03a631b1a29ae6c8 --- /dev/null +++ b/processed_dataset/text/0244.json @@ -0,0 +1,5 @@ +{ + "source_file": "./raw_volume-zh/volume9/chapter3.tex", + "text": "圆内接四边形中的不等式.\n圆内接四边形不仅有着丰富的几何等量关系, 也有很多有趣的极值性质.\n因为圆内接四边形的边可用对应的圆心角的三角函数表示, 这就使得三角方法在处理圆内接四边形的几何不等式时能派上用场.\n下面就是这样的一个例子.", + "figures": [] +} \ No newline at end of file diff --git a/processed_dataset/text/0245.json b/processed_dataset/text/0245.json new file mode 100644 index 0000000000000000000000000000000000000000..290a98dd8002656369dd70ca8e46d3793542e5ca --- /dev/null +++ b/processed_dataset/text/0245.json @@ -0,0 +1,7 @@ +{ + "source_file": "./raw_volume-zh/volume9/chapter3.tex", + "text": "圆内接四边形有一个著名的极值性质: 四条边给定的四边形中, 内接于圆的四边形面积最大.\n一个给定边长的圆内接四边形的面积有很好的解析公式, 这就是下面的定理.\n定理设一个圆的内接凸四边形的边长依次为 $a 、 b 、 c 、 d$, 又设 $s$ 为该四边形周长的一半, 则四边形的面积 $F$ 为\n$$\nF=\\sqrt{(s-a)(s-b)(s-c)(s-d)} .\n$$\n这个定理是三角形熟知结果的推广.\n如果取 $d=0$, 我们便得到通常的三角形面积的海伦公式.\n证明设定理中的四边形为 $A B C D$, 如图() $A B=a, B C=b, C D=c, D A=d$.\n如果 $A B C D$ 是长方形, 证明立即得出.\n如果 $A B C D$ 不是长方形, 设 $B C$ 与 $A D$ 相交于圆外的点 $E$. 记 $C E=x, D E=y$, 则由三角形面积公式有\n$$\nS_{\\triangle C D E}=\\frac{1}{4} \\sqrt{(x+y+c)(x+y-c)(x-y+c)(-x+y+c)} . \\label{eq1}\n$$\n但 $\\triangle A B E \\backsim \\triangle C D E$, 所以\n$$\n\\frac{S_{\\triangle A B E}}{S_{\\triangle C D E}}=\\frac{a^2}{c^2}\n$$\n由此推得\n$$\n\\frac{F}{S_{\\triangle C D E}}=\\frac{c^2-a^2}{c^2}, \\label{eq2}\n$$\n又由比例式\n$$\n\\frac{x}{c}=\\frac{y-d}{a}, \\frac{y}{c}=\\frac{x-\\underline{b}}{a},\n$$\n相加解出 $x+y$, 得\n$$\nx+y+c=\\frac{c}{c-a}(-a+b+c+d),\n$$\n$x+y-c$ 等等的类似表达式, 都可以立即得到.\n将它们代入式\\ref{eq1}并化简, 得\n$$\nS_{\\triangle C D E}=\\frac{c^2}{c^2-a^2} \\sqrt{(s-a)(s-b)(s-c)(s-d)},\n$$\n代入式\\ref{eq2}, 便知结论成立.\n推广可以证明任一边长为 $a 、 b 、 c 、 d$,一对对角的和为 $2 u$ 的凸四边形, 面积 $F$ 可由下式给出\n$$\nF^2=(s-a)(s-b)(s-c)(s-d)-a b c d \\cos ^2 u .\n$$\n由于证明包含长而乏味的三角化简, 我们不在这里给出.\n由这个公式立即看出 : 四条边给定的四边形中, 内接于圆的面积最大.", + "figures": [ + "./images/volume9/figures/fig-c3i4.png" + ] +} \ No newline at end of file diff --git a/processed_dataset/text/0246.json b/processed_dataset/text/0246.json new file mode 100644 index 0000000000000000000000000000000000000000..ea935bd0e12f1e22ba4434bbf16a992063ef0609 --- /dev/null +++ b/processed_dataset/text/0246.json @@ -0,0 +1,11 @@ +{ + "source_file": "./raw_volume-zh/volume9/chapter4.tex", + "text": "特殊多边形的面积不等式.\n多边形的面积不等式与极值问题一直备受关注.\n一些特殊多边形,如三角形、平行四边形的面积不等式更是在中学数学竞赛中经常出现.\n这节介绍一些有趣的结果, 并力求体现处理面积问题的一般方法.\n首先我们研究一下平行四边形和它内含的三角形的面积之间的关系.\n关于这个问题的一个熟知结论是: 任一平行四边形的内含三角形的面积不超过这个平行四边形面积的一半.\n这个结论的证明十分简单, 如图(). 只需过 $\\triangle P Q R$ 的顶点 $Q$ 作 $A B$ 的平行线, 并考虑被平行线分成的小平行四边形和小三角形的面积关系 便可.\n现考虑这个问题的反问题,三角形与其内含的平行四边形的面积有何关系? 关于它的回答是下面有用的定理.\n定理 1 任意一个三角形的内含平行四边形的面积不超过三角形面积的一半.\n证明设平行四边形 $P_1 P_2 P_3 P_4$ 是 $\\triangle A B C$ 内的平行四边形.\n如图(), 不妨设直线 $P_1 P_2 、 P_3 P_4$ 交边 $B C$ 于两点, 分别记为 $M_2 、 M_3$, 在这两直线上分别截取线段 $M_2 M_1$ 和 $M_3 M_4$ 使得\n$$\nM_2 M_1=P_2 P_1, M_3 M_4=P_3 P_4,\n$$\n则四边形 $M_1 M_2 M_3 M_4$ 是平行四边形且\n$$\nS\\left(M_1 M_2 M_3 M_4\\right)=S\\left(P_1 P_2 P_3 P_4\\right) .\n$$\n设直线 $M_1 M_4$ 分别交边 $A B 、 A C$ 于两点 $D 、 E$, 过点 $E$ 作 $A B$ 的平行线交 $B C$ 于 $F$, 则得平行四边形 $B D E F$, 易见\n$$\nS(B D E F) \\geqslant S\\left(M_1 M_2 M_3 M_4\\right)=S\\left(P_1 P_2 P_3 P_4\\right) .\n$$\n因此要证\n$$\nS\\left(P_1 P_2 P_3 P_4\\right) \\leqslant \\frac{1}{2} S_{\\triangle A B C},\n$$\n只需证明\n$S(B D E F) \\leqslant \\frac{1}{2} S_{\\triangle A B C} , \\label{(*)}$.\n下证 式\\ref{(*)}.\n如图(), 设 $\\lambda=\\frac{A D}{A B}$, 则由\n$\\triangle A D E \\backsim \\triangle A B C$\n可知\n$S_{\\triangle A D E}=\\lambda^2 S_{\\triangle A B C}$.\n同理\n$$\nS_{\\triangle E F C}=(1-\\lambda)^2 S_{\\triangle A B C} \\text {. }\n$$\n因此 $S_{\\triangle A D E}+S_{\\triangle E F C}=\\left[\\lambda^2+(1-\\lambda)^2\\right] S_{\\triangle A B C} \\geqslant \\frac{1}{2} S_{\\triangle A B C}$,\n所以 $\\quad S(B D E F)=S_{\\triangle A B C}-\\left(S_{\\triangle A D E}+S_{\\triangle E F C}\\right) \\leqslant \\frac{1}{2} S_{\\triangle A B C}$.\n式\\ref{(*)} 得证,且当 $D 、 E 、 F$ 分别为三边的中点时等号成立.\n注:上面的证法是典型的化归法, 即将一般的平行四边形 $P_1 P_2 P_3 P_4$ 转化为有一组边与边 $B C$ 平行的平行四边形 $M_1 M_2 M_3 M_4$, 再转化为两组边分别平行于三角形两边的非常特殊的平行四边形 $B D E F$, 从而使问题大大得到简化.\n如图(), 设 $P$ 是 $\\triangle A B C$ 内的一点, 直线 $A P 、 B P 、 C P$ 与三边的交点分别为 $D 、 E 、 F$, 则 $\\triangle D E F$ 叫做点 $P$ 的塞瓦 (Ceva) 三角形;\n如图(), 若内点 $P$ 在三边 $B C 、 C A 、 A B$ 上的射影分别为 $D 、 E 、 F$, 则 $\\triangle D E F$ 叫做点 $P$ 的垂足三角形.\n关于点 $P$ 的塞瓦三角形和垂足三角形有下面著名的命题.\n命题 1 若 $P$ 是 $\\triangle A B C$ 的内点, 则点 $P$ 的塞瓦三角形 $D E F$ 的面积不超过 $\\frac{1}{4} S_{\\triangle A B C}$.\n命题 2 若 $P$ 是 $\\triangle A B C$ 的内点,则点 $P$ 的垂足三角形 $D E F$ 的面积不超过 $\\frac{1}{4} S_{\\triangle A B C}$.", + "figures": [ + "./images/volume9/figures/fig-c4i1.png", + "./images/volume9/figures/fig-c4i2.png", + "./images/volume9/figures/fig-c4i3.png", + "./images/volume9/figures/fig-c4i4.png", + "./images/volume9/figures/fig-c4i5.png" + ] +} \ No newline at end of file diff --git a/processed_dataset/text/0247.json b/processed_dataset/text/0247.json new file mode 100644 index 0000000000000000000000000000000000000000..bd3967f54260d5f36a9838f8e898e87d9f4f2cf0 --- /dev/null +++ b/processed_dataset/text/0247.json @@ -0,0 +1,5 @@ +{ + "source_file": "./raw_volume-zh/volume9/chapter4.tex", + "text": "A. Soifer 的五点问题等价于下面的命题 3 .\n命题 3 对于任意的三角形 $T, S(T)=5$.\nA. Soifer 进一步证明了命题 4 对任意的平行四边形 $P, S(P)=5$.\n一个自然的问题是 : 是否对任意的图形 $F$ 都有 $S(F)=5$ ?\n答案是否定的, A. Soifer 证明了命题 5 对正五边形 $F, S(F)=6$.\n対任意的凸的图形 $F, S(F)$ 都可以取什么样的值呢? A. Soifer 证明了 $S(F)$ 只能在很小的范围内取值, 即有命题 6 对任意的凸的图形 $F, 4 \\leqslant S(F) \\leqslant 6$.\n关于命题 6 的更为进一步的结论是:\n命题 7 对任意的凸的图形 $F, S(F) \\neq 4$.\n命题 8 对任意的凸的图形 $F, S(F)=5$, 或者 $S(F)=6$.\n一个有趣的但尚未解决的问题是: 什么样的凸图形 $F$ 使得 $S(F)=5$, 什么样的凸图形 $F$ 使得 $S(F)=6$ ?\n下面讨论的话题是面积为 1 的凸多边形能被怎样的平行四边形和三角形覆盖的问题.", + "figures": [] +} \ No newline at end of file diff --git a/processed_dataset/text/0248.json b/processed_dataset/text/0248.json new file mode 100644 index 0000000000000000000000000000000000000000..3ff7a7e8aed0a5e6361f45a56a5d9a07e764a378 --- /dev/null +++ b/processed_dataset/text/0248.json @@ -0,0 +1,5 @@ +{ + "source_file": "./raw_volume-zh/volume9/chapter5.tex", + "text": "线性几何不等式.\n许多线性几何不等式给人的印象是: 简单而不平凡, 特别容易被记住.\n线性几何不等式的证明要么平凡,要么使人棘手.\n数学竞赛中出现的线性几何不等式大都是富有挑战性的.\nErdös-Mordell 不等式是最著名的线性几何不等式之一,下面首先介绍这个不等式.", + "figures": [] +} \ No newline at end of file diff --git a/processed_dataset/text/0249.json b/processed_dataset/text/0249.json new file mode 100644 index 0000000000000000000000000000000000000000..7cfef8cb481fab531014f288c00e6248b94ef923 --- /dev/null +++ b/processed_dataset/text/0249.json @@ -0,0 +1,5 @@ +{ + "source_file": "./raw_volume-zh/volume9/chapter6.tex", + "text": "构造代数恒等式来证明一些距离不等式是十分方便的,这方面一个典型的例子是 M. S. Klamkin 早年的一个不等式.", + "figures": [] +} \ No newline at end of file diff --git a/processed_dataset/text/0250.json b/processed_dataset/text/0250.json new file mode 100644 index 0000000000000000000000000000000000000000..1a9188ebcdf17956820a7aecb85d4bd61591128f --- /dev/null +++ b/processed_dataset/text/0250.json @@ -0,0 +1,5 @@ +{ + "source_file": "./raw_volume-zh/volume9/chapter7.tex", + "text": "等周极值问题.\n各种空间,各种形式的等周问题似乎是几何学永恒的研究主题之一.\n平面几何中的等周定理反映着一些特殊几何图形,如圆,正 $n$ 边形等特有的极值性质,因此特别引人注意.\n下面叙述几个中学数学竞赛中常要用到的等周定理.\n定理 1 (等周定理 I )周长一定的所有图形中, 以圆的面积为最大; 反之,面积一定的所有图形中, 以圆的周长为最小.\n定理 2 (等周定理 II) 周长一定的所有 $n$ 边形中, 以正 $n$ 边形的面积为最大; 反之,面积一定的所有 $n$ 边形中, 以正 $n$ 边形的周长为最小.\n设 $a_1, a_2, \\cdots, a_n$ 为任 $-n$ 边形的边长, $F$ 为其面积,则由等周定理立得如下的等周不等式\n$$\n\\left(\\sum_{i=1}^n a_i\\right)^2 \\geqslant 4 n \\tan \\frac{\\pi}{n} \\cdot F\n$$\n等号成立当且仅当 $a_1=a_2=\\cdots=a_n$.\n下面的定理 3 是第 3 章的圆内接四边形极值性质的推广.\n定理 3 (Steiner 定理) 边长给定的 $n$ 边形中以存在外接圆者的面积为最大.\n定理 4 圆内接 $n$ 边形中以正 $n$ 边形的面积为最大.\n设 $a_1, a_2, \\cdots, a_n$ 是半径为 $R$ 的圆内接 $n$ 边形的边长,则由定理 4 可得\n$$\n\\sum_{i=1}^n a_i \\leqslant 2 n R \\cdot \\sin \\frac{\\pi}{n}\n$$\n等号成立当且仅当 $a_1=a_2=\\cdots=a_n$.\n先看几个简单的例子.", + "figures": [] +} \ No newline at end of file diff --git a/processed_dataset/text/0251.json b/processed_dataset/text/0251.json new file mode 100644 index 0000000000000000000000000000000000000000..61be0c501ac84834832f637c2078a33726ee5db1 --- /dev/null +++ b/processed_dataset/text/0251.json @@ -0,0 +1,5 @@ +{ + "source_file": "./raw_volume-zh/volume9/chapter8.tex", + "text": "三角形嵌入不等式 (简称嵌入不等式) 在近年来初等几何不等式研究中扮演着一个重要的角色, 是产生新的几何不等式的一个源头, 嵌入不等式可叙述为\n定理 1 (嵌入不等式)设 $A+B+C=(2 k+1) \\pi, x, y, z \\in \\mathbf{R}$, 则\n$$\nx^2+y^2+z^2 \\geqslant 2 y z \\cos A+2 z x \\cos B+2 x y \\cos C, \\label{eq1}\n$$\n等号成立当且仅当 $x: y: z=\\sin A: \\sin B: \\sin C$.\n简证 式\\ref{eq1} 的左右两边之差 $=(x-y \\cos C-z \\cos B)^2+(y \\sin C- z \\sin B)^2 \\geqslant 0$, 得证.\n顾名思义,不等式(1)被称为嵌入不等式的几何解释是: 如果 $0

),标号为 $1 、 2 、 3 、 4$ 的 4 个凸多边形中每两个有一条公共边但没有公共的内部, 故所求 $n$ 的最大值不小于 4. 另一方面, 设在平面内存在 5 个凸多边形满足条件, 取定其中一个记为 $M_0$, 其他 4 个按它们的公共点在 $M_0$ 上的位置顺序记为 $M_1 、 M_2 、 M_3 、 M_4$. 按已知条件, $M_1$ 与 $M_3$ 有一条公共边, 因此 $M_0 、 M_1 、 M_3$ 这 3 个凸多边形或将 $M_2$ 包围在中间而 $M_4$ 在外,或将 $M_4$ 包围在中间而 $M_2$ 在外, 无论哪种情形 $M_2$ 与 $M_4$ 不可能有公共边, 矛盾, 故 $n \\leqslant 4$. 综上可知, 所求 $n$ 的最大值为 4 .", + "remark": "", + "figures": [ + "./images/volume9/figures/fig-c10a1.png" + ] +} \ No newline at end of file diff --git a/processed_dataset/calculation/0997.json b/processed_dataset/calculation/0997.json new file mode 100644 index 0000000000000000000000000000000000000000..ca70b8d957b1c94d45e95980d31177ca33ce46ed --- /dev/null +++ b/processed_dataset/calculation/0997.json @@ -0,0 +1,8 @@ +{ + "source_file": "./raw_volume-zh/volume9/exercise2.tex", + "problem_type": "calculation", + "problem": "问题2. $\\triangle A B C$ 的边 $B C$ 和边 $A C$ 分别取定长 $a$ 和 $b$, 而边 $A B$ 的长度可变动.\n以边 $A B$ 作为正方形的一边向三角形外作正方形.\n设 $O$ 是所作正方形的中心, 并设 $B C$ 和 $A C$ 的中点分别为 $M$ 和 $N$. 试求 $O M+O N$ 的最大值.", + "solution": "设正方形为 $A B D E$, 则 $O M=\\frac{C E}{2}, O N=\\frac{C D}{2}$, 所以 $O M+O N= \\frac{1}{2}(C D+C E)$. 设 $A B=c$, 则 $B D=A E=c, A D=B E=\\sqrt{2} c$. 对四边形 $A C B D$ 和 $A C B E$ 分别应用广义 Ptolemy 定理可得 $O M+O N=\\frac{1}{2}(C D+ C E) \\leqslant \\frac{\\sqrt{2}+1}{2}(a+b)$. 所以 $O M+O N$ 的最大值为 $\\frac{\\sqrt{2}+1}{2}(a+b)$.", + "remark": "", + "figures": [] +} \ No newline at end of file diff --git a/processed_dataset/calculation/0998.json b/processed_dataset/calculation/0998.json new file mode 100644 index 0000000000000000000000000000000000000000..5b9ab8d56544b79c5e9d604900dca5ef94617649 --- /dev/null +++ b/processed_dataset/calculation/0998.json @@ -0,0 +1,8 @@ +{ + "source_file": "./raw_volume-zh/volume9/exercise5.tex", + "problem_type": "calculation", + "problem": "问题5. 设 $I$ 是 $\\triangle A B C$ 的内心.\n记 $\\triangle I B C 、 \\triangle I C A 、 \\triangle I A B$ 的内切圆半径分别为 $r_1 、 r_2 、 r_3$, 则\n$$\n3 \\sqrt{3}(2-\\sqrt{3}) r \\leqslant r_1+r_2+r_3 \\leqslant \\frac{3 \\sqrt{3}(2-\\sqrt{3})}{2} R,\n$$\n这里 $r 、 R$ 分别是 $\\triangle A B C$ 的内切圆半径和外接圆半径.", + "solution": "提示: 需要用到如下三角形不等式 $\\sin \\alpha \\sin \\beta \\sin \\gamma \\leqslant \\frac{3 \\sqrt{6}-5 \\sqrt{2}}{16} \\cdot \\cos (\\beta+ \\gamma) \\cos \\alpha+\\cos (\\gamma+\\alpha) \\cos \\beta+\\cos (\\alpha+\\beta) \\cos \\gamma \\leqslant \\frac{3 \\sqrt{6}+9 \\sqrt{2}}{8}, \\tan \\alpha \\cdot \\tan \\beta+ \\tan \\beta \\tan \\gamma+\\tan \\gamma \\tan \\alpha \\leqslant 3(7-4 \\sqrt{3})$, 其中 $A=4 \\alpha, B=4 \\beta, C=4 \\gamma$.", + "remark": "", + "figures": [] +} \ No newline at end of file diff --git a/processed_dataset/calculation/0999.json b/processed_dataset/calculation/0999.json new file mode 100644 index 0000000000000000000000000000000000000000..1cf9f41b70d9381d6335cb925ef3cc83e409e225 --- /dev/null +++ b/processed_dataset/calculation/0999.json @@ -0,0 +1,8 @@ +{ + "source_file": "./raw_volume-zh/volume9/exercise7.tex", + "problem_type": "calculation", + "problem": "问题3. 在 3 条边长为 1 、一个内角是 $30^{\\circ}$ 的四边形中, 找出具有最大面积 $S$ 的四边形, 并求出 $S$.", + "solution": "设四边形的三边 $A B=B C=C D=1$, 显然具有最大面积的四边形为凸四边形.\n由对称性, 可只考虑 $\\angle A$ 或 $\\angle B$ 等于 $30^{\\circ}$ 的情形.\n(1) 若 $\\angle B=30^{\\circ}$, 则易知这时四边形 $A B C D$ 的最大值 $S_1=\\frac{\\sqrt{6}-\\sqrt{2}+1}{4}$. (2) 若 $\\angle A=30^{\\circ}$, 作 $B 、 C$ 关于直线 $A D$ 的对称点 $B^{\\prime} 、 C^{\\prime}$. 连接 $A B^{\\prime} 、 B^{\\prime} C^{\\prime} 、 C^{\\prime} D 、 B B^{\\prime}$, 则 $\\triangle A B B^{\\prime}$ 是边长为 1 的正三角形, 五边形 $B C D C^{\\prime} B^{\\prime}$ 是边长均为 1 的五边形, 当且仅当它是正五边形时面积取得最大值 $\\frac{\\sqrt{25+10 \\sqrt{5}}}{4}$. 故四边形 $A B C D$ 的最大值 $S_2=\\frac{1}{2}\\left[\\frac{\\sqrt{3}}{4}+\\frac{\\sqrt{25+10 \\sqrt{5}}}{4}\\right]=\\frac{\\sqrt{3}}{8}+\\frac{\\sqrt{25+10 \\sqrt{5}}}{8}$. 易知 $S_2>S_1$, 故所求最大值 $S=\\frac{\\sqrt{3}}{8}+\\frac{\\sqrt{25+10 \\sqrt{5}}}{8}$, 且具有最大面积 $S^{\\prime}$ 的四边形 $A B C D$ 中, $\\angle A=30^{\\circ}$, $\\angle B=168^{\\circ}, \\angle C=108^{\\circ}, \\angle D=54^{\\circ}, A B=B C=C D=1$.", + "remark": "", + "figures": [] +} \ No newline at end of file diff --git a/processed_dataset/calculation/1000.json b/processed_dataset/calculation/1000.json new file mode 100644 index 0000000000000000000000000000000000000000..d90c2dcb51d24c37ea2e769a787a60bc64965b01 --- /dev/null +++ b/processed_dataset/calculation/1000.json @@ -0,0 +1,8 @@ +{ + "source_file": "./raw_volume-zh/volume9/exercise8.tex", + "problem_type": "calculation", + "problem": "问题3 (Garfunkel-Baukoff)\n$$\n\\tan ^2 \\frac{A}{2}+\\tan ^2 \\frac{B}{2}+\\tan ^2 \\frac{C}{2} \\geqslant 2-8 \\sin \\frac{A}{2} \\sin \\frac{B}{2} \\sin \\frac{C}{2} .\n$$", + "solution": "提示: 在嵌入不等式中令 $x=\\tan \\frac{A}{2}, y=\\tan \\frac{B}{2}, z=\\tan \\frac{C}{2}$, 再通过一系列三角恒等变形即得.", + "remark": "", + "figures": [] +} \ No newline at end of file diff --git a/processed_dataset/calculation/1001.json b/processed_dataset/calculation/1001.json new file mode 100644 index 0000000000000000000000000000000000000000..703497cf324c3e798fac02e165aa5102abc1abc7 --- /dev/null +++ b/processed_dataset/calculation/1001.json @@ -0,0 +1,8 @@ +{ + "source_file": "./raw_volume-zh/volume9/exercise9.tex", + "problem_type": "calculation", + "problem": "问题1. 设 $P$ 是 $\\triangle A B C$ 所在平面上的一个动点, 点 $P$ 的垂足三角形的面积为 $F_P$, $\\triangle A B C$ 的面积为 $F$, 求平面上使得不等式\n$$\nF_P \\leqslant \\frac{1}{4} F\n$$\n成立的点 $P$ 的轨迹.", + "solution": "利用 Gergonne 公式可求得 $P$ 的轨迹是以 $O$ 为圆心, 半径为 $\\sqrt{2} R$ 的圆及其内部.", + "remark": "", + "figures": [] +} \ No newline at end of file diff --git a/processed_dataset/images/volume1/figures/fig-c1p4.png b/processed_dataset/images/volume1/figures/fig-c1p4.png new file mode 100644 index 0000000000000000000000000000000000000000..4ed6a0ccf5a38209c52b74e6198b5149f95a6de0 --- /dev/null +++ b/processed_dataset/images/volume1/figures/fig-c1p4.png @@ -0,0 +1,3 @@ +version https://git-lfs.github.com/spec/v1 +oid sha256:7363b79fe422aa42dc8e2cefd64c279c95902e9642ee9a0b7fc36075aa8d58b1 +size 61898 diff --git a/processed_dataset/images/volume1/figures/fig-c2e3.png b/processed_dataset/images/volume1/figures/fig-c2e3.png new file mode 100644 index 0000000000000000000000000000000000000000..bbaadd484afc33f9636d24caf010adf6048b7700 --- /dev/null +++ b/processed_dataset/images/volume1/figures/fig-c2e3.png @@ -0,0 +1,3 @@ +version https://git-lfs.github.com/spec/v1 +oid sha256:2874ce9748b231a0597704b62b020c661ecb4182b081149bba19915bad5563b6 +size 36457 diff --git a/processed_dataset/images/volume1/figures/fig-c2e7.png b/processed_dataset/images/volume1/figures/fig-c2e7.png new file mode 100644 index 0000000000000000000000000000000000000000..8a499d45a066ea7117b775e6db198ce5b5defd29 --- /dev/null +++ b/processed_dataset/images/volume1/figures/fig-c2e7.png @@ -0,0 +1,3 @@ +version https://git-lfs.github.com/spec/v1 +oid sha256:c21b797af73203bd6d9f5b91c42517fc310d139056ac6a4cd2976f2fea629563 +size 35632 diff --git a/processed_dataset/images/volume1/figures/fig-c2p10.png b/processed_dataset/images/volume1/figures/fig-c2p10.png new file mode 100644 index 0000000000000000000000000000000000000000..7d710bf589c14b5f97c4181ba9b742b7901be890 --- /dev/null +++ b/processed_dataset/images/volume1/figures/fig-c2p10.png @@ -0,0 +1,3 @@ +version https://git-lfs.github.com/spec/v1 +oid sha256:3d305e8d125b05e81187567b9c10cc32ab3e665d3c0e5dec1453055fa00a8132 +size 59323 diff --git a/processed_dataset/images/volume1/figures/fig-c2p8.png b/processed_dataset/images/volume1/figures/fig-c2p8.png new file mode 100644 index 0000000000000000000000000000000000000000..4241aa05c3aa0f501e87f13d3b5bc39713178dd6 --- /dev/null +++ b/processed_dataset/images/volume1/figures/fig-c2p8.png @@ -0,0 +1,3 @@ +version https://git-lfs.github.com/spec/v1 +oid sha256:4a647fb765cea6e7b0a65288fb84c576b8d9d6a4602b71f995ce679d8d40b36a +size 33284 diff --git a/processed_dataset/images/volume1/figures/fig-c5e6.png b/processed_dataset/images/volume1/figures/fig-c5e6.png new file mode 100644 index 0000000000000000000000000000000000000000..efdeda88088bf7f8c8caf8897b2e28511c5b09aa --- /dev/null +++ b/processed_dataset/images/volume1/figures/fig-c5e6.png @@ -0,0 +1,3 @@ +version https://git-lfs.github.com/spec/v1 +oid sha256:48c6cc919e7457c75e2223ce75ee6e32ef47b169881351226cb0dff65917e64e +size 35208 diff --git a/processed_dataset/images/volume1/figures/fig-c6p12-1.png b/processed_dataset/images/volume1/figures/fig-c6p12-1.png new file mode 100644 index 0000000000000000000000000000000000000000..a0061d365f06c9334118579b728b44576e0c2516 --- /dev/null +++ b/processed_dataset/images/volume1/figures/fig-c6p12-1.png @@ -0,0 +1,3 @@ +version https://git-lfs.github.com/spec/v1 +oid sha256:1619e48b613b83ab6bd1722728276c9520be42386ac4a35dc683223f1bb2dfa9 +size 37755 diff --git a/processed_dataset/images/volume1/figures/fig-c6p12-2.png b/processed_dataset/images/volume1/figures/fig-c6p12-2.png new file mode 100644 index 0000000000000000000000000000000000000000..4ea92bc00efe24ac560ad7efa5ed61215a25cd5c --- /dev/null +++ b/processed_dataset/images/volume1/figures/fig-c6p12-2.png @@ -0,0 +1,3 @@ +version https://git-lfs.github.com/spec/v1 +oid sha256:538413a16d98b438502d3e58c2f34867dcd0ed10fedecc2f86f8440e60b7f8bc +size 19602 diff --git a/processed_dataset/images/volume1/figures/fig-c6p12-3.png b/processed_dataset/images/volume1/figures/fig-c6p12-3.png new file mode 100644 index 0000000000000000000000000000000000000000..b7631efcd4f2edc3e8269a3f6cf0baf85a0cbf40 --- /dev/null +++ b/processed_dataset/images/volume1/figures/fig-c6p12-3.png @@ -0,0 +1,3 @@ +version https://git-lfs.github.com/spec/v1 +oid sha256:0787542c8d9a58a2814a794251b5e7be5120d3b19d3d1d2a58c813a107421ddb +size 36191 diff --git a/processed_dataset/images/volume1/figures/fig-c6p18.png b/processed_dataset/images/volume1/figures/fig-c6p18.png new file mode 100644 index 0000000000000000000000000000000000000000..dc555f5a9a9416172a858c1909f8048f74f789fb --- /dev/null +++ b/processed_dataset/images/volume1/figures/fig-c6p18.png @@ -0,0 +1,3 @@ +version https://git-lfs.github.com/spec/v1 +oid sha256:55e7640637f12f6bb51d8e926a089405cc986edbf0c5502d5c775700e258c803 +size 19290 diff --git a/processed_dataset/images/volume1/figures/fig-c7e5-1.png b/processed_dataset/images/volume1/figures/fig-c7e5-1.png new file mode 100644 index 0000000000000000000000000000000000000000..1b95c5ea05cc3f233700c3fae7e35f713dd43215 --- /dev/null +++ b/processed_dataset/images/volume1/figures/fig-c7e5-1.png @@ -0,0 +1,3 @@ +version https://git-lfs.github.com/spec/v1 +oid sha256:50f4f465aa4ba2dda6360808616c71ed68f400833b255701465993fe1e8f64d2 +size 31326 diff --git a/processed_dataset/images/volume1/figures/fig-c7e5-2.png b/processed_dataset/images/volume1/figures/fig-c7e5-2.png new file mode 100644 index 0000000000000000000000000000000000000000..c77b79bf24382ecd6e28ebe1bfcaf0efc94c73c1 --- /dev/null +++ b/processed_dataset/images/volume1/figures/fig-c7e5-2.png @@ -0,0 +1,3 @@ +version https://git-lfs.github.com/spec/v1 +oid sha256:24c0f253fe1b76353642691bf2ad2854d511f237190279cf4b924a6e9b7d5081 +size 34221 diff --git a/processed_dataset/images/volume1/figures/fig-c7e5-3.png b/processed_dataset/images/volume1/figures/fig-c7e5-3.png new file mode 100644 index 0000000000000000000000000000000000000000..e063d913b6beeb28f540bb567b01ae71b021d28e --- /dev/null +++ b/processed_dataset/images/volume1/figures/fig-c7e5-3.png @@ -0,0 +1,3 @@ +version https://git-lfs.github.com/spec/v1 +oid sha256:d898b1bb08679dbc8502718f5815afb8796ba6249fc0dacc08e65ee41aee3b46 +size 31585 diff --git a/processed_dataset/images/volume1/figures/fig-c7e5-4.png b/processed_dataset/images/volume1/figures/fig-c7e5-4.png new file mode 100644 index 0000000000000000000000000000000000000000..3d88bce54665fdac8536e4fdc2fdd89acffafe8c --- /dev/null +++ b/processed_dataset/images/volume1/figures/fig-c7e5-4.png @@ -0,0 +1,3 @@ +version https://git-lfs.github.com/spec/v1 +oid sha256:e4184d66d7e23694e2f0c6a47c9a7ba8414ce91e766c3de9cd48ff5814aaa027 +size 36971 diff --git a/processed_dataset/images/volume1/figures/fig-c8e7-1.png b/processed_dataset/images/volume1/figures/fig-c8e7-1.png new file mode 100644 index 0000000000000000000000000000000000000000..51331f2fad5889aacef075519c2997ea5383a424 --- /dev/null +++ b/processed_dataset/images/volume1/figures/fig-c8e7-1.png @@ -0,0 +1,3 @@ +version https://git-lfs.github.com/spec/v1 +oid sha256:68ba5ec23ffdfc1291ce9c4f2ccfb754ad2ef08e4c92fd727b127664dc31daab +size 31421 diff --git a/processed_dataset/images/volume1/figures/fig-c8e7-2.png b/processed_dataset/images/volume1/figures/fig-c8e7-2.png new file mode 100644 index 0000000000000000000000000000000000000000..ba8146afd8ee727bbb76ad12e50a8965e972fc3e --- /dev/null +++ b/processed_dataset/images/volume1/figures/fig-c8e7-2.png @@ -0,0 +1,3 @@ +version https://git-lfs.github.com/spec/v1 +oid sha256:729d66a7abe2717f577bcd2ec39fc701183491ba9057a0a91497af2085d9d070 +size 21745 diff --git a/processed_dataset/images/volume1/figures/fig-c8p11.png b/processed_dataset/images/volume1/figures/fig-c8p11.png new file mode 100644 index 0000000000000000000000000000000000000000..ff039d6bc166ba972fe497ae7a630d5387a6d3ab --- /dev/null +++ b/processed_dataset/images/volume1/figures/fig-c8p11.png @@ -0,0 +1,3 @@ +version https://git-lfs.github.com/spec/v1 +oid sha256:cf6020001900c2785913e9a2361575920f48e773ccdd791b71624258401e95b4 +size 29365 diff --git a/processed_dataset/images/volume1/figures/fig-c8p12.png b/processed_dataset/images/volume1/figures/fig-c8p12.png new file mode 100644 index 0000000000000000000000000000000000000000..5dd9d0067fa0380a957752c9d7072e6e6370b027 --- /dev/null +++ b/processed_dataset/images/volume1/figures/fig-c8p12.png @@ -0,0 +1,3 @@ +version https://git-lfs.github.com/spec/v1 +oid sha256:a2f80257d0d049579cdbfecc2968d226bfdb24d74481d46ca725ff81e84b913a +size 36999 diff --git a/processed_dataset/images/volume1/figures/fig-c9p1.png b/processed_dataset/images/volume1/figures/fig-c9p1.png new file mode 100644 index 0000000000000000000000000000000000000000..d03b0e09aab34e7f90a65ebb399b3a9a087ec8f4 --- /dev/null +++ b/processed_dataset/images/volume1/figures/fig-c9p1.png @@ -0,0 +1,3 @@ +version https://git-lfs.github.com/spec/v1 +oid sha256:95851f711b175945c5e77c5d70600035d94421bceb727e0c46cab853db32414e +size 24795 diff --git a/processed_dataset/images/volume11/figures/fig-c10a4.png b/processed_dataset/images/volume11/figures/fig-c10a4.png new file mode 100644 index 0000000000000000000000000000000000000000..be3d260a1b61ca79f917598ed8a8e81618f28062 --- /dev/null +++ b/processed_dataset/images/volume11/figures/fig-c10a4.png @@ -0,0 +1,3 @@ +version https://git-lfs.github.com/spec/v1 +oid sha256:fdf3bf247d5093de14cc9f05d1d1be4b2142948070e05cfba6d604fd0a358479 +size 36364 diff --git a/processed_dataset/images/volume11/figures/fig-c10i1.png b/processed_dataset/images/volume11/figures/fig-c10i1.png new file mode 100644 index 0000000000000000000000000000000000000000..57a90fcb2258cf13055f6f4d83b1e7b6e1c09c1e --- /dev/null +++ b/processed_dataset/images/volume11/figures/fig-c10i1.png @@ -0,0 +1,3 @@ +version https://git-lfs.github.com/spec/v1 +oid sha256:57e1c2856f105747ec6552cff53354c1d206522ef897fe81c2afad2ff8298447 +size 32742 diff --git a/processed_dataset/images/volume11/figures/fig-c11a3.png b/processed_dataset/images/volume11/figures/fig-c11a3.png new file mode 100644 index 0000000000000000000000000000000000000000..818bb06649865b7c1327a023c3f49dd3db23da8d --- /dev/null +++ b/processed_dataset/images/volume11/figures/fig-c11a3.png @@ -0,0 +1,3 @@ +version https://git-lfs.github.com/spec/v1 +oid sha256:1f957e9098b7d1846cc0ff782ad97ce02ca759f0e6cf7dc34ddd1a1681d28a7b +size 53903 diff --git a/processed_dataset/images/volume11/figures/fig-c12a3.png b/processed_dataset/images/volume11/figures/fig-c12a3.png new file mode 100644 index 0000000000000000000000000000000000000000..21001be85bf01db1c4431ae2f6e442dd123d1881 --- /dev/null +++ b/processed_dataset/images/volume11/figures/fig-c12a3.png @@ -0,0 +1,3 @@ +version https://git-lfs.github.com/spec/v1 +oid sha256:1a82b21ce27dc06fcf9ce6dcc72b3bfae8a778b5b9038a3eea514b1eb59cebae +size 29444 diff --git a/processed_dataset/images/volume11/figures/fig-c12a4-1.png b/processed_dataset/images/volume11/figures/fig-c12a4-1.png new file mode 100644 index 0000000000000000000000000000000000000000..d16b86ce63abfed1f5c676a34853548bffdc21d8 --- /dev/null +++ b/processed_dataset/images/volume11/figures/fig-c12a4-1.png @@ -0,0 +1,3 @@ +version https://git-lfs.github.com/spec/v1 +oid sha256:b9902b25086fd4074bbbc67f4570a8dec1638d70dbc0b10f4ae1d818be4bd5ad +size 23967 diff --git a/processed_dataset/images/volume11/figures/fig-c12a4-2.png b/processed_dataset/images/volume11/figures/fig-c12a4-2.png new file mode 100644 index 0000000000000000000000000000000000000000..1539ee742b041941aadef4fd543c4d47bcc67e66 --- /dev/null +++ b/processed_dataset/images/volume11/figures/fig-c12a4-2.png @@ -0,0 +1,3 @@ +version https://git-lfs.github.com/spec/v1 +oid sha256:91f2a716b1dd6255a514f577248d0c78bdcd293091d822458ab3a9c1f1309e4c +size 22744 diff --git a/processed_dataset/images/volume11/figures/fig-c12a4-3.png b/processed_dataset/images/volume11/figures/fig-c12a4-3.png new file mode 100644 index 0000000000000000000000000000000000000000..39dfbbe0545ad88b828e62d53c3aafb4f65a4c78 --- /dev/null +++ b/processed_dataset/images/volume11/figures/fig-c12a4-3.png @@ -0,0 +1,3 @@ +version https://git-lfs.github.com/spec/v1 +oid sha256:1af1d90733e7f41b68875aaf4c06ed80acb0fec413c2c4af8d8d708ac2b16287 +size 24013 diff --git a/processed_dataset/images/volume11/figures/fig-c12i1.png b/processed_dataset/images/volume11/figures/fig-c12i1.png new file mode 100644 index 0000000000000000000000000000000000000000..453c7a5329dd7aefb66bb91ec7ae52055f2a625a --- /dev/null +++ b/processed_dataset/images/volume11/figures/fig-c12i1.png @@ -0,0 +1,3 @@ +version https://git-lfs.github.com/spec/v1 +oid sha256:9da39fd86dd1d79d559c06c6fdfedb1b4dcef4f15d88e14985e54590ef4702f2 +size 15931 diff --git a/processed_dataset/images/volume11/figures/fig-c12i2.png b/processed_dataset/images/volume11/figures/fig-c12i2.png new file mode 100644 index 0000000000000000000000000000000000000000..11c659cd6b312a78b58d4e2189c9478ff3388df8 --- /dev/null +++ b/processed_dataset/images/volume11/figures/fig-c12i2.png @@ -0,0 +1,3 @@ +version https://git-lfs.github.com/spec/v1 +oid sha256:e90084c2c374c156e623ed9671dbf05ae705238da1a80ccb2f0acdd167dbab67 +size 24992 diff --git a/processed_dataset/images/volume11/figures/fig-c12i3.png b/processed_dataset/images/volume11/figures/fig-c12i3.png new file mode 100644 index 0000000000000000000000000000000000000000..21859fbf85725cd2970517adc8d865ddfacee09a --- /dev/null +++ b/processed_dataset/images/volume11/figures/fig-c12i3.png @@ -0,0 +1,3 @@ +version https://git-lfs.github.com/spec/v1 +oid sha256:b8673e3cc1149f2e2d072e98a6bd3e60fdfd2d659c2d04a986f4b4994d0152c5 +size 92177 diff --git a/processed_dataset/images/volume11/figures/fig-c12i4.png b/processed_dataset/images/volume11/figures/fig-c12i4.png new file mode 100644 index 0000000000000000000000000000000000000000..11664e8b8f6ba9dac4bd53ab3eb42075b25e89d8 --- /dev/null +++ b/processed_dataset/images/volume11/figures/fig-c12i4.png @@ -0,0 +1,3 @@ +version https://git-lfs.github.com/spec/v1 +oid sha256:ccd181794ef6f28433a037241fe30b9aaff20738693e0e3d7080ebcec255be63 +size 27841 diff --git a/processed_dataset/images/volume11/figures/fig-c13a6.png b/processed_dataset/images/volume11/figures/fig-c13a6.png new file mode 100644 index 0000000000000000000000000000000000000000..126ae7a60c7c7e70d651fa21982ab14e2206fe52 --- /dev/null +++ b/processed_dataset/images/volume11/figures/fig-c13a6.png @@ -0,0 +1,3 @@ +version https://git-lfs.github.com/spec/v1 +oid sha256:332bc17601d91c3cd27a80c07328d1990221a78d22747467eba718c0430ff59d +size 30348 diff --git a/processed_dataset/images/volume11/figures/fig-c13i1.png b/processed_dataset/images/volume11/figures/fig-c13i1.png new file mode 100644 index 0000000000000000000000000000000000000000..e4ad29c82ce1788faf4ad83821c399eb4127f452 --- /dev/null +++ b/processed_dataset/images/volume11/figures/fig-c13i1.png @@ -0,0 +1,3 @@ +version https://git-lfs.github.com/spec/v1 +oid sha256:42df173fdd5b9fcada9258231e693205eff485efc40d6807e7157f716f168b2e +size 37434 diff --git a/processed_dataset/images/volume11/figures/fig-c15a7-1.png b/processed_dataset/images/volume11/figures/fig-c15a7-1.png new file mode 100644 index 0000000000000000000000000000000000000000..baf64eb27cef6013ec20649a2a3eaa81548ef848 --- /dev/null +++ b/processed_dataset/images/volume11/figures/fig-c15a7-1.png @@ -0,0 +1,3 @@ +version https://git-lfs.github.com/spec/v1 +oid sha256:1d93558beb76b1435a9fee161d80d6c1a6095893b42595688666a21a965ae4b9 +size 23455 diff --git a/processed_dataset/images/volume11/figures/fig-c15a7-2.png b/processed_dataset/images/volume11/figures/fig-c15a7-2.png new file mode 100644 index 0000000000000000000000000000000000000000..a3993d91affe030f96b4fe196f8db23240b93a67 --- /dev/null +++ b/processed_dataset/images/volume11/figures/fig-c15a7-2.png @@ -0,0 +1,3 @@ +version https://git-lfs.github.com/spec/v1 +oid sha256:c1cf984fc4de2d6e9d087b226f080e6fcd21edc8d21f0e13337d80c80edd420b +size 32046 diff --git a/processed_dataset/images/volume11/figures/fig-c15a8.png b/processed_dataset/images/volume11/figures/fig-c15a8.png new file mode 100644 index 0000000000000000000000000000000000000000..791c4dd0efa2ee3564daec4f1796597ad77cc62d --- /dev/null +++ b/processed_dataset/images/volume11/figures/fig-c15a8.png @@ -0,0 +1,3 @@ +version https://git-lfs.github.com/spec/v1 +oid sha256:0c9c219c4e145ebb4a1cd4290a8c22906cd9218e94d73047612aa92bb7644209 +size 32709 diff --git a/processed_dataset/images/volume11/figures/fig-c15i1.png b/processed_dataset/images/volume11/figures/fig-c15i1.png new file mode 100644 index 0000000000000000000000000000000000000000..933ed4cac29d9eb8040dd7b51c04b51230374672 --- /dev/null +++ b/processed_dataset/images/volume11/figures/fig-c15i1.png @@ -0,0 +1,3 @@ +version https://git-lfs.github.com/spec/v1 +oid sha256:1f2207fcc5f0fa805d9a8b01affe1efbdaebf7f3dfa757d73af63cf07701f1f7 +size 43672 diff --git a/processed_dataset/images/volume11/figures/fig-c15i2.png b/processed_dataset/images/volume11/figures/fig-c15i2.png new file mode 100644 index 0000000000000000000000000000000000000000..b717b864d0f80be50f10b678904a6885f4c6f7b8 --- /dev/null +++ b/processed_dataset/images/volume11/figures/fig-c15i2.png @@ -0,0 +1,3 @@ +version https://git-lfs.github.com/spec/v1 +oid sha256:be3b7a5eb2d888c926c3bba262c6b02fe876795dabb01cf761edab59c12132c2 +size 31322 diff --git a/processed_dataset/images/volume11/figures/fig-c15i3.png b/processed_dataset/images/volume11/figures/fig-c15i3.png new file mode 100644 index 0000000000000000000000000000000000000000..04b89a66c2c783185416861e9241ccf232680409 --- /dev/null +++ b/processed_dataset/images/volume11/figures/fig-c15i3.png @@ -0,0 +1,3 @@ +version https://git-lfs.github.com/spec/v1 +oid sha256:ba839b640c1f6cca34cbc91765732e5e1ad530a947ef2c574485c2cf5f9aeec5 +size 24992 diff --git a/processed_dataset/images/volume11/figures/fig-c15i4.png b/processed_dataset/images/volume11/figures/fig-c15i4.png new file mode 100644 index 0000000000000000000000000000000000000000..ac07bea0a3bd36df2b4e076f7d58faf6df7e27f9 --- /dev/null +++ b/processed_dataset/images/volume11/figures/fig-c15i4.png @@ -0,0 +1,3 @@ +version https://git-lfs.github.com/spec/v1 +oid sha256:353a54021b314dcd1ae7d51e441e4aa86e291ce9cab8894949ef04313bac57f9 +size 31633 diff --git a/processed_dataset/images/volume11/figures/fig-c15i5.png b/processed_dataset/images/volume11/figures/fig-c15i5.png new file mode 100644 index 0000000000000000000000000000000000000000..4ee3e7e4b369a02ca68dcd3d0f69f50b788103f6 --- /dev/null +++ b/processed_dataset/images/volume11/figures/fig-c15i5.png @@ -0,0 +1,3 @@ +version https://git-lfs.github.com/spec/v1 +oid sha256:5aa12e22c1aefb0fd129b971c43a417e56d28d66a4820a1162cd07b0fb3cd215 +size 31894 diff --git a/processed_dataset/images/volume11/figures/fig-c1a8.png b/processed_dataset/images/volume11/figures/fig-c1a8.png new file mode 100644 index 0000000000000000000000000000000000000000..9c4ff71228f9ee47da5653611d70025334e4643d --- /dev/null +++ b/processed_dataset/images/volume11/figures/fig-c1a8.png @@ -0,0 +1,3 @@ +version https://git-lfs.github.com/spec/v1 +oid sha256:82c56ec34499499c790fadc9ef216ba38915888c310a0c512fee64029e07f0cf +size 15433 diff --git a/processed_dataset/images/volume11/figures/fig-c1p10.png b/processed_dataset/images/volume11/figures/fig-c1p10.png new file mode 100644 index 0000000000000000000000000000000000000000..2c804be3f157b941d6bee17579e95072100be98d --- /dev/null +++ b/processed_dataset/images/volume11/figures/fig-c1p10.png @@ -0,0 +1,3 @@ +version https://git-lfs.github.com/spec/v1 +oid sha256:55989f791c946132ba75be6c50a9540feaf1e5d7e6db1832d0b0bcd196462f03 +size 20999 diff --git a/processed_dataset/images/volume11/figures/fig-c2a3.png b/processed_dataset/images/volume11/figures/fig-c2a3.png new file mode 100644 index 0000000000000000000000000000000000000000..c2af5eeb64dc8423be35ed7211523b5ff2ab7903 --- /dev/null +++ b/processed_dataset/images/volume11/figures/fig-c2a3.png @@ -0,0 +1,3 @@ +version https://git-lfs.github.com/spec/v1 +oid sha256:6174a785241038cd96ab9983482249e8b68b5a43dcc1ffb5b069573dabae60c9 +size 20363 diff --git a/processed_dataset/images/volume11/figures/fig-c2i1.png b/processed_dataset/images/volume11/figures/fig-c2i1.png new file mode 100644 index 0000000000000000000000000000000000000000..51612f941a56a5c0aa2f96d9de1810c73e2f228a --- /dev/null +++ b/processed_dataset/images/volume11/figures/fig-c2i1.png @@ -0,0 +1,3 @@ +version https://git-lfs.github.com/spec/v1 +oid sha256:aa700c2e7a4e292a809c4ca66b24cd305faba45f727386fa40ff2069e6c483d3 +size 23713 diff --git a/processed_dataset/images/volume11/figures/fig-c5a7.png b/processed_dataset/images/volume11/figures/fig-c5a7.png new file mode 100644 index 0000000000000000000000000000000000000000..3e818c016e050f106681f9a2f8a7b0520eadd887 --- /dev/null +++ b/processed_dataset/images/volume11/figures/fig-c5a7.png @@ -0,0 +1,3 @@ +version https://git-lfs.github.com/spec/v1 +oid sha256:15949ea749b10549b2129bb699ebbe094d98ec1deb93fd13dcebe526cd17ac39 +size 56930 diff --git a/processed_dataset/images/volume11/figures/fig-c5a8.png b/processed_dataset/images/volume11/figures/fig-c5a8.png new file mode 100644 index 0000000000000000000000000000000000000000..a79bffa32ca20a88b501241baa7c6aa8d771086d --- /dev/null +++ b/processed_dataset/images/volume11/figures/fig-c5a8.png @@ -0,0 +1,3 @@ +version https://git-lfs.github.com/spec/v1 +oid sha256:0c2e93f10055f290b903c1899cd279f0a2954060896f2046c6ea31d212d67796 +size 21079 diff --git a/processed_dataset/images/volume11/figures/fig-c5a9.png b/processed_dataset/images/volume11/figures/fig-c5a9.png new file mode 100644 index 0000000000000000000000000000000000000000..7407cbae8ef612a8f6004bfa82f21facc6cba131 --- /dev/null +++ b/processed_dataset/images/volume11/figures/fig-c5a9.png @@ -0,0 +1,3 @@ +version https://git-lfs.github.com/spec/v1 +oid sha256:08d51c9ab5f187e6f0a2d1719184bbaa0210b01fc19551f982865d21e4b7780b +size 35219 diff --git a/processed_dataset/images/volume11/figures/fig-c6i1.png b/processed_dataset/images/volume11/figures/fig-c6i1.png new file mode 100644 index 0000000000000000000000000000000000000000..b7dabb6d862aacb36e74731ad6b76d940ea23d2c --- /dev/null +++ b/processed_dataset/images/volume11/figures/fig-c6i1.png @@ -0,0 +1,3 @@ +version https://git-lfs.github.com/spec/v1 +oid sha256:059866a803eef2585222f32da0f25562e67975f345ebb85c9ef82447ef03b445 +size 160419 diff --git a/processed_dataset/images/volume11/figures/fig-c6i2.png b/processed_dataset/images/volume11/figures/fig-c6i2.png new file mode 100644 index 0000000000000000000000000000000000000000..d87974aae5418c5d9a8fc8fde1fe34b4ec9b5385 --- /dev/null +++ b/processed_dataset/images/volume11/figures/fig-c6i2.png @@ -0,0 +1,3 @@ +version https://git-lfs.github.com/spec/v1 +oid sha256:c746415bc256769cfb653af9ce70ef68eaadc2a06d817b9844df9ebe81a256eb +size 19237 diff --git a/processed_dataset/images/volume11/figures/fig-c6i3.png b/processed_dataset/images/volume11/figures/fig-c6i3.png new file mode 100644 index 0000000000000000000000000000000000000000..193176d5d76c51ef49b56ea28af91026825c20c1 --- /dev/null +++ b/processed_dataset/images/volume11/figures/fig-c6i3.png @@ -0,0 +1,3 @@ +version https://git-lfs.github.com/spec/v1 +oid sha256:6e4e44adf493fff7ea6795706439efe33183e21faff19d1adc1d3854838ebb41 +size 24075 diff --git a/processed_dataset/images/volume11/figures/fig-c6i4.png b/processed_dataset/images/volume11/figures/fig-c6i4.png new file mode 100644 index 0000000000000000000000000000000000000000..d5f34e744d60884c6ecac9aa0d713efc6f9447d5 --- /dev/null +++ b/processed_dataset/images/volume11/figures/fig-c6i4.png @@ -0,0 +1,3 @@ +version https://git-lfs.github.com/spec/v1 +oid sha256:d2424babec2a12b186e9c31d627cee579322122d4a95a93cbf8a99751d4c3f55 +size 40029 diff --git a/processed_dataset/images/volume11/figures/fig-c6i5-1.png b/processed_dataset/images/volume11/figures/fig-c6i5-1.png new file mode 100644 index 0000000000000000000000000000000000000000..374bc4263bd92c86af31662f192d5a913090e284 --- /dev/null +++ b/processed_dataset/images/volume11/figures/fig-c6i5-1.png @@ -0,0 +1,3 @@ +version https://git-lfs.github.com/spec/v1 +oid sha256:c8444dae9a922520685b03ee5245e1dd95b87f59f5caf97088e59504d09569b5 +size 36342 diff --git a/processed_dataset/images/volume11/figures/fig-c6i5-2.png b/processed_dataset/images/volume11/figures/fig-c6i5-2.png new file mode 100644 index 0000000000000000000000000000000000000000..a3b7e29d4c910b31b3002069323f542ae3197bc4 --- /dev/null +++ b/processed_dataset/images/volume11/figures/fig-c6i5-2.png @@ -0,0 +1,3 @@ +version https://git-lfs.github.com/spec/v1 +oid sha256:7c2a18cdcb85b9190b1cd240cfc52a2ba4e220d14e575c09541ed60efbe41a0a +size 33965 diff --git a/processed_dataset/images/volume11/figures/fig-c6i5-3.png b/processed_dataset/images/volume11/figures/fig-c6i5-3.png new file mode 100644 index 0000000000000000000000000000000000000000..ee4b7441f9f59cb3d6f4fc9c32e0a186bc91c922 --- /dev/null +++ b/processed_dataset/images/volume11/figures/fig-c6i5-3.png @@ -0,0 +1,3 @@ +version https://git-lfs.github.com/spec/v1 +oid sha256:ad50f2fb82a2cf70330b00bd163ec5bb34f4b1f634f67d9adb38c6e9682a34e6 +size 28098 diff --git a/processed_dataset/images/volume11/figures/fig-c9a1.png b/processed_dataset/images/volume11/figures/fig-c9a1.png new file mode 100644 index 0000000000000000000000000000000000000000..4f61f02ef78e142c35a152d4c2a1e019df3cdeb8 --- /dev/null +++ b/processed_dataset/images/volume11/figures/fig-c9a1.png @@ -0,0 +1,3 @@ +version https://git-lfs.github.com/spec/v1 +oid sha256:fa0f8ad92ececacd6cb6f5a27d11fb7b0bd8c40b75947d739266eb24312935cf +size 58150 diff --git a/processed_dataset/images/volume11/figures/fig-c9a2-1.png b/processed_dataset/images/volume11/figures/fig-c9a2-1.png new file mode 100644 index 0000000000000000000000000000000000000000..cae9bed757b8d9f4d5fb0f4c94940cb3a87128af --- /dev/null +++ b/processed_dataset/images/volume11/figures/fig-c9a2-1.png @@ -0,0 +1,3 @@ +version https://git-lfs.github.com/spec/v1 +oid sha256:7b09116bb8778747f9619703987ad6d9ec8449ca184c8f9e3dd512366e31e9dd +size 12544 diff --git a/processed_dataset/images/volume11/figures/fig-c9a2-2.png b/processed_dataset/images/volume11/figures/fig-c9a2-2.png new file mode 100644 index 0000000000000000000000000000000000000000..34142bc251294f1869944d68dd5a4dc22b449ce0 --- /dev/null +++ b/processed_dataset/images/volume11/figures/fig-c9a2-2.png @@ -0,0 +1,3 @@ +version https://git-lfs.github.com/spec/v1 +oid sha256:4a68c6f8fab638f9fff9e03d292f40e8dcc3caae7428f2cba52823ddb7cd2f0c +size 21405 diff --git a/processed_dataset/images/volume11/figures/fig-c9a3.png b/processed_dataset/images/volume11/figures/fig-c9a3.png new file mode 100644 index 0000000000000000000000000000000000000000..08ad148781d3ac9105f078415d152c68016e858a --- /dev/null +++ b/processed_dataset/images/volume11/figures/fig-c9a3.png @@ -0,0 +1,3 @@ +version https://git-lfs.github.com/spec/v1 +oid sha256:2c31887a451ec616d1769fdc229437d74229dabcfcad0a75a31b342f5212b1c2 +size 24910 diff --git a/processed_dataset/images/volume11/figures/fig-c9a4.png b/processed_dataset/images/volume11/figures/fig-c9a4.png new file mode 100644 index 0000000000000000000000000000000000000000..84cb3c10372b7768c470940c9ba518cedef678d0 --- /dev/null +++ b/processed_dataset/images/volume11/figures/fig-c9a4.png @@ -0,0 +1,3 @@ +version https://git-lfs.github.com/spec/v1 +oid sha256:e860f9b30be6241267fb309240e59f3ff6f28ed186ef034491179a3640f7e951 +size 63572 diff --git a/processed_dataset/images/volume11/figures/fig-c9i1.png b/processed_dataset/images/volume11/figures/fig-c9i1.png new file mode 100644 index 0000000000000000000000000000000000000000..08cef6c14a6578f78dd75cd72a5e7a37056981d6 --- /dev/null +++ b/processed_dataset/images/volume11/figures/fig-c9i1.png @@ -0,0 +1,3 @@ +version https://git-lfs.github.com/spec/v1 +oid sha256:c60f8e076e71b95099729b7432eed40d9b2370c3a8a3feb693942a883b105748 +size 22958 diff --git a/processed_dataset/images/volume11/figures/fig-c9i10.png b/processed_dataset/images/volume11/figures/fig-c9i10.png new file mode 100644 index 0000000000000000000000000000000000000000..c7278e8c3885b7f36f3a14c612c0255be6684f49 --- /dev/null +++ b/processed_dataset/images/volume11/figures/fig-c9i10.png @@ -0,0 +1,3 @@ +version https://git-lfs.github.com/spec/v1 +oid sha256:93a13c69d31b42f7f6689e9652dc1d8de83c8b0bebd45f1e6443ec5afca5cb5b +size 28302 diff --git a/processed_dataset/images/volume11/figures/fig-c9i2.png b/processed_dataset/images/volume11/figures/fig-c9i2.png new file mode 100644 index 0000000000000000000000000000000000000000..a292f056c660cb41a95f0cdf2e5e43a6c5ac1fff --- /dev/null +++ b/processed_dataset/images/volume11/figures/fig-c9i2.png @@ -0,0 +1,3 @@ +version https://git-lfs.github.com/spec/v1 +oid sha256:b2c81df770a8ea3be5b1ce3a8e220558d7bfd003d8a6e5f660fedf99935aa5f5 +size 20801 diff --git a/processed_dataset/images/volume11/figures/fig-c9i3.png b/processed_dataset/images/volume11/figures/fig-c9i3.png new file mode 100644 index 0000000000000000000000000000000000000000..341c1e209fd4fa75eda1779515bdd8ccabc27f72 --- /dev/null +++ b/processed_dataset/images/volume11/figures/fig-c9i3.png @@ -0,0 +1,3 @@ +version https://git-lfs.github.com/spec/v1 +oid sha256:89c635e6972294c531e4b96515fecfeb3c83cae930471989ae40ec9a675b1ed2 +size 8519 diff --git a/processed_dataset/images/volume11/figures/fig-c9i4.png b/processed_dataset/images/volume11/figures/fig-c9i4.png new file mode 100644 index 0000000000000000000000000000000000000000..b54e0a0439935709cd83fb24c2d82bd96e5b244a --- /dev/null +++ b/processed_dataset/images/volume11/figures/fig-c9i4.png @@ -0,0 +1,3 @@ +version https://git-lfs.github.com/spec/v1 +oid sha256:6d1479b7134a56b3f874746d888010a2773bfe98fe027f997f35545f4a33f491 +size 8030 diff --git a/processed_dataset/images/volume11/figures/fig-c9i5.png b/processed_dataset/images/volume11/figures/fig-c9i5.png new file mode 100644 index 0000000000000000000000000000000000000000..2d2f311d70d641e8b0406799ef59db366d657012 --- /dev/null +++ b/processed_dataset/images/volume11/figures/fig-c9i5.png @@ -0,0 +1,3 @@ +version https://git-lfs.github.com/spec/v1 +oid sha256:988ea82a5a3e8525e9d6c4fed90793d7730b118fcfd93524c776f21378052caf +size 54143 diff --git a/processed_dataset/images/volume11/figures/fig-c9i6.png b/processed_dataset/images/volume11/figures/fig-c9i6.png new file mode 100644 index 0000000000000000000000000000000000000000..e37574ada149a5956c7b6e24f14f217fe2126d1b --- /dev/null +++ b/processed_dataset/images/volume11/figures/fig-c9i6.png @@ -0,0 +1,3 @@ +version https://git-lfs.github.com/spec/v1 +oid sha256:d9bb0c3c67f95a2613eaa38aaef19b16c55a8d154e0a31d4e612051f29864815 +size 73872 diff --git a/processed_dataset/images/volume11/figures/fig-c9i7-1.png b/processed_dataset/images/volume11/figures/fig-c9i7-1.png new file mode 100644 index 0000000000000000000000000000000000000000..a410071696f54a021fb381af9053cb742cfeefba --- /dev/null +++ b/processed_dataset/images/volume11/figures/fig-c9i7-1.png @@ -0,0 +1,3 @@ +version https://git-lfs.github.com/spec/v1 +oid sha256:b77852046744c1eb1773b600216064c58df44571f337e3e90d50130b32394a9d +size 21101 diff --git a/processed_dataset/images/volume11/figures/fig-c9i7-2.png b/processed_dataset/images/volume11/figures/fig-c9i7-2.png new file mode 100644 index 0000000000000000000000000000000000000000..57890dc62e1cb8694a1047aa540e61ffa87bc87d --- /dev/null +++ b/processed_dataset/images/volume11/figures/fig-c9i7-2.png @@ -0,0 +1,3 @@ +version https://git-lfs.github.com/spec/v1 +oid sha256:342e4789365833fa10f03476da8c65550f9b6dbcd6697b638d0287f8c4cf4b30 +size 17752 diff --git a/processed_dataset/images/volume11/figures/fig-c9i8-1.png b/processed_dataset/images/volume11/figures/fig-c9i8-1.png new file mode 100644 index 0000000000000000000000000000000000000000..5965118e1fc325596be7dbce05b6abc14a10504b --- /dev/null +++ b/processed_dataset/images/volume11/figures/fig-c9i8-1.png @@ -0,0 +1,3 @@ +version https://git-lfs.github.com/spec/v1 +oid sha256:fb69fb4a4220bc8d0c9616ff53d0cf894925b11eb113806faa627b3318181ecb +size 17636 diff --git a/processed_dataset/images/volume11/figures/fig-c9i8-2.png b/processed_dataset/images/volume11/figures/fig-c9i8-2.png new file mode 100644 index 0000000000000000000000000000000000000000..f8f77dc995cc1143c5d2aee0c7a0b4a83aef917d --- /dev/null +++ b/processed_dataset/images/volume11/figures/fig-c9i8-2.png @@ -0,0 +1,3 @@ +version https://git-lfs.github.com/spec/v1 +oid sha256:65bcb8ff134304eac1d0044874e28262b0ba4baafdd7f94dbbf53fc5699b8c09 +size 22183 diff --git a/processed_dataset/images/volume11/figures/fig-c9i9.png b/processed_dataset/images/volume11/figures/fig-c9i9.png new file mode 100644 index 0000000000000000000000000000000000000000..21711078f030eb20f643cd69703da940cec66408 --- /dev/null +++ b/processed_dataset/images/volume11/figures/fig-c9i9.png @@ -0,0 +1,3 @@ +version https://git-lfs.github.com/spec/v1 +oid sha256:c8a6aeafa5f009a3cacef8cd1cbbe76b30191b091a1da272b5562c7b8468e48c +size 52442 diff --git a/processed_dataset/images/volume11/figures/fig-c9p2-1.png b/processed_dataset/images/volume11/figures/fig-c9p2-1.png new file mode 100644 index 0000000000000000000000000000000000000000..0f894529e44d605aea8eb5da14b8f6abf367e4ae --- /dev/null +++ b/processed_dataset/images/volume11/figures/fig-c9p2-1.png @@ -0,0 +1,3 @@ +version https://git-lfs.github.com/spec/v1 +oid sha256:07d47042128b45d8ed85a94c57076a86e6b4e73268cb7d17e41fb5401bb156bb +size 5913 diff --git a/processed_dataset/images/volume11/figures/fig-c9p2-2.png b/processed_dataset/images/volume11/figures/fig-c9p2-2.png new file mode 100644 index 0000000000000000000000000000000000000000..89d392aac421f03fdc501bcc41478ec3f7e610f9 --- /dev/null +++ b/processed_dataset/images/volume11/figures/fig-c9p2-2.png @@ -0,0 +1,3 @@ +version https://git-lfs.github.com/spec/v1 +oid sha256:bae620704b9dd5da3844169849010686a33aa7bda81497d597669fa26b9273a8 +size 6044 diff --git a/processed_dataset/images/volume12/figures/fig-c1a1.png b/processed_dataset/images/volume12/figures/fig-c1a1.png new file mode 100644 index 0000000000000000000000000000000000000000..2fc41e5f3c392bf25e6bc4ddf17ad158b1bbe9b4 --- /dev/null +++ b/processed_dataset/images/volume12/figures/fig-c1a1.png @@ -0,0 +1,3 @@ +version https://git-lfs.github.com/spec/v1 +oid sha256:3128003dd8b214e58b083db8c22beaaf772cf93830e7d92985f5f5f81befbe3f +size 17100 diff --git a/processed_dataset/images/volume12/figures/fig-c1a3-1.png b/processed_dataset/images/volume12/figures/fig-c1a3-1.png new file mode 100644 index 0000000000000000000000000000000000000000..41157391294f0ffb6065da4a1498087c1880d335 --- /dev/null +++ b/processed_dataset/images/volume12/figures/fig-c1a3-1.png @@ -0,0 +1,3 @@ +version https://git-lfs.github.com/spec/v1 +oid sha256:04292604ac7a3fd1e5188d1487732811cc2a24e90701e8b395a509541ecf4749 +size 24228 diff --git a/processed_dataset/images/volume12/figures/fig-c1a3-2.png b/processed_dataset/images/volume12/figures/fig-c1a3-2.png new file mode 100644 index 0000000000000000000000000000000000000000..edece69664aca6431d83b717adc56a9422e46cda --- /dev/null +++ b/processed_dataset/images/volume12/figures/fig-c1a3-2.png @@ -0,0 +1,3 @@ +version https://git-lfs.github.com/spec/v1 +oid sha256:bdea0bc95eb96f25bf2459bbb26b46aa43028ec7624439603498962d91aba1f3 +size 28464 diff --git a/processed_dataset/images/volume12/figures/fig-c1a8.png b/processed_dataset/images/volume12/figures/fig-c1a8.png new file mode 100644 index 0000000000000000000000000000000000000000..5ec5cb2396788dc7160b8b6f40b96c9f4acda5c9 --- /dev/null +++ b/processed_dataset/images/volume12/figures/fig-c1a8.png @@ -0,0 +1,3 @@ +version https://git-lfs.github.com/spec/v1 +oid sha256:37b26e204b6f0fe7dbac6e1979eb5d3a8bcc0bf0a6d659ef109c83196e9044e0 +size 36890 diff --git a/processed_dataset/images/volume12/figures/fig-c1i1.png b/processed_dataset/images/volume12/figures/fig-c1i1.png new file mode 100644 index 0000000000000000000000000000000000000000..9f5dbe05a556f58ba01519fbdbfcf031236eb5bd --- /dev/null +++ b/processed_dataset/images/volume12/figures/fig-c1i1.png @@ -0,0 +1,3 @@ +version https://git-lfs.github.com/spec/v1 +oid sha256:3fc988e66a128ec4d40a974cf75e5075d9354e6e594643fd044ebe5a188f0e91 +size 58254 diff --git a/processed_dataset/images/volume12/figures/fig-c1i10.png b/processed_dataset/images/volume12/figures/fig-c1i10.png new file mode 100644 index 0000000000000000000000000000000000000000..0d26e861831e5d5ad636f064e43aed5a97e210c2 --- /dev/null +++ b/processed_dataset/images/volume12/figures/fig-c1i10.png @@ -0,0 +1,3 @@ +version https://git-lfs.github.com/spec/v1 +oid sha256:2eb69c9e89e4df25a30f3068ecec5d7ddfa6df3b94514ae2750c7a25507e1b92 +size 22459 diff --git a/processed_dataset/images/volume12/figures/fig-c1i11.png b/processed_dataset/images/volume12/figures/fig-c1i11.png new file mode 100644 index 0000000000000000000000000000000000000000..ddaa2c06fd1ee8b833ec3eed978d1d1b2116aaa4 --- /dev/null +++ b/processed_dataset/images/volume12/figures/fig-c1i11.png @@ -0,0 +1,3 @@ +version https://git-lfs.github.com/spec/v1 +oid sha256:9497c0d0861777787dd3e403749b8268b4e3b77d09aaed75902dae363641fe6d +size 22526 diff --git a/processed_dataset/images/volume12/figures/fig-c1i12.png b/processed_dataset/images/volume12/figures/fig-c1i12.png new file mode 100644 index 0000000000000000000000000000000000000000..e45f4d7d3d4d2219b015264764a244c92f671257 --- /dev/null +++ b/processed_dataset/images/volume12/figures/fig-c1i12.png @@ -0,0 +1,3 @@ +version https://git-lfs.github.com/spec/v1 +oid sha256:cda9d53cd5f4844a6c672b7b9e96894bbf7883e60f0cbe5088d9626c2fef51b8 +size 18755 diff --git a/processed_dataset/images/volume12/figures/fig-c1i13.png b/processed_dataset/images/volume12/figures/fig-c1i13.png new file mode 100644 index 0000000000000000000000000000000000000000..0a4a99e65f597cc18f9f17a41cf0bde22f7565f7 --- /dev/null +++ b/processed_dataset/images/volume12/figures/fig-c1i13.png @@ -0,0 +1,3 @@ +version https://git-lfs.github.com/spec/v1 +oid sha256:6cd2305fa6fc5f3fda6b073dc540ccf8ba53b7f6a2c44de63cafe84b840ce94d +size 32924 diff --git a/processed_dataset/images/volume12/figures/fig-c1i14.png b/processed_dataset/images/volume12/figures/fig-c1i14.png new file mode 100644 index 0000000000000000000000000000000000000000..e498c92aee4a3b41d9972f092db5ccbf77157daa --- /dev/null +++ b/processed_dataset/images/volume12/figures/fig-c1i14.png @@ -0,0 +1,3 @@ +version https://git-lfs.github.com/spec/v1 +oid sha256:903af32806743be7fa6fb678a1c762ce16b2c3fc5796a3ce0b2a6dcef03543b7 +size 22426 diff --git a/processed_dataset/images/volume12/figures/fig-c1i2.png b/processed_dataset/images/volume12/figures/fig-c1i2.png new file mode 100644 index 0000000000000000000000000000000000000000..0c33cbd401e13df724584f7a99d5bb685775b1be --- /dev/null +++ b/processed_dataset/images/volume12/figures/fig-c1i2.png @@ -0,0 +1,3 @@ +version https://git-lfs.github.com/spec/v1 +oid sha256:582edf934970fa7e15bef00c3f628ec86c79a97fa0d6a6340447da1bec480363 +size 41551 diff --git a/processed_dataset/images/volume12/figures/fig-c1i3.png b/processed_dataset/images/volume12/figures/fig-c1i3.png new file mode 100644 index 0000000000000000000000000000000000000000..8ab29f16fdae2e94d12e3db181bb48a1270060ca --- /dev/null +++ b/processed_dataset/images/volume12/figures/fig-c1i3.png @@ -0,0 +1,3 @@ +version https://git-lfs.github.com/spec/v1 +oid sha256:c8ee51aa94f96018e225ff13e3ec9d2a04074b7d003fa38f8b83fbd18c40c351 +size 24906 diff --git a/processed_dataset/images/volume12/figures/fig-c1i4.png b/processed_dataset/images/volume12/figures/fig-c1i4.png new file mode 100644 index 0000000000000000000000000000000000000000..96bc269c5c8eb5529939a01d1d67f2c15844426a --- /dev/null +++ b/processed_dataset/images/volume12/figures/fig-c1i4.png @@ -0,0 +1,3 @@ +version https://git-lfs.github.com/spec/v1 +oid sha256:947935f198929a78e9f4bedc4e60b20dbe7c13fd6bc80020db9b687f481bc4d5 +size 41736 diff --git a/processed_dataset/images/volume12/figures/fig-c1i5.png b/processed_dataset/images/volume12/figures/fig-c1i5.png new file mode 100644 index 0000000000000000000000000000000000000000..b848c33e266ec3529f3ac6eb2f88cb3c31f84436 --- /dev/null +++ b/processed_dataset/images/volume12/figures/fig-c1i5.png @@ -0,0 +1,3 @@ +version https://git-lfs.github.com/spec/v1 +oid sha256:b557b83f25459b40f1e1a059b320562b34637da8ffc5fdd4f4a760b91d415faf +size 19557 diff --git a/processed_dataset/images/volume12/figures/fig-c1i6.png b/processed_dataset/images/volume12/figures/fig-c1i6.png new file mode 100644 index 0000000000000000000000000000000000000000..0c8e75ee60c40957e374701b13ccbf4451930206 --- /dev/null +++ b/processed_dataset/images/volume12/figures/fig-c1i6.png @@ -0,0 +1,3 @@ +version https://git-lfs.github.com/spec/v1 +oid sha256:ea8c43cc15ad67c8b25a5e73fca65e1dcdd95033bef743eb8f3e9c70767be366 +size 22903 diff --git a/processed_dataset/images/volume12/figures/fig-c1i7.png b/processed_dataset/images/volume12/figures/fig-c1i7.png new file mode 100644 index 0000000000000000000000000000000000000000..556db7e9594a9380a7109a9d29d14a8220d4f9e4 --- /dev/null +++ b/processed_dataset/images/volume12/figures/fig-c1i7.png @@ -0,0 +1,3 @@ +version https://git-lfs.github.com/spec/v1 +oid sha256:f9bb3dfb0fd2c523764f3d20258e30563d9a8002d30fe0928c6057c002a4b3f4 +size 23606 diff --git a/processed_dataset/images/volume12/figures/fig-c1i8.png b/processed_dataset/images/volume12/figures/fig-c1i8.png new file mode 100644 index 0000000000000000000000000000000000000000..bb293636e09cc108398fac682eeaf941fae81160 --- /dev/null +++ b/processed_dataset/images/volume12/figures/fig-c1i8.png @@ -0,0 +1,3 @@ +version https://git-lfs.github.com/spec/v1 +oid sha256:ba701b6b025317d6deaeb754f35b79744d6edfd9a05f96c99326e3424c87a524 +size 21303 diff --git a/processed_dataset/images/volume12/figures/fig-c1i9.png b/processed_dataset/images/volume12/figures/fig-c1i9.png new file mode 100644 index 0000000000000000000000000000000000000000..34eaf13760aaaf3af22fdc3b08105b528ffafd5e --- /dev/null +++ b/processed_dataset/images/volume12/figures/fig-c1i9.png @@ -0,0 +1,3 @@ +version https://git-lfs.github.com/spec/v1 +oid sha256:4c86c0ea7c4451c4ab2cd36238d2a40a9ff143b43544a6f9d93c2b838c19598e +size 19168 diff --git a/processed_dataset/images/volume12/figures/fig-c1p3-1.png b/processed_dataset/images/volume12/figures/fig-c1p3-1.png new file mode 100644 index 0000000000000000000000000000000000000000..7fa478d6ec9b861030e41fda0294b261a624b49f --- /dev/null +++ b/processed_dataset/images/volume12/figures/fig-c1p3-1.png @@ -0,0 +1,3 @@ +version https://git-lfs.github.com/spec/v1 +oid sha256:cc5715a79641548f79565735dd165b7274add6ea81c24eb4374c0e3af253021f +size 27500 diff --git a/processed_dataset/images/volume12/figures/fig-c1p3-2.png b/processed_dataset/images/volume12/figures/fig-c1p3-2.png new file mode 100644 index 0000000000000000000000000000000000000000..132fbfeb5ca6894afe2168f8cd834e05f36c9167 --- /dev/null +++ b/processed_dataset/images/volume12/figures/fig-c1p3-2.png @@ -0,0 +1,3 @@ +version https://git-lfs.github.com/spec/v1 +oid sha256:4b5113b7a4ed2d80159dab2bf8d7e65e55ef91477a511253bdc200f3ed5b5d9b +size 22653 diff --git a/processed_dataset/images/volume12/figures/fig-c2a4.png b/processed_dataset/images/volume12/figures/fig-c2a4.png new file mode 100644 index 0000000000000000000000000000000000000000..bed0ac819cca82c12bc3832679057b5dc53446ad --- /dev/null +++ b/processed_dataset/images/volume12/figures/fig-c2a4.png @@ -0,0 +1,3 @@ +version https://git-lfs.github.com/spec/v1 +oid sha256:46339bd9d482930cabc36736df22d5bba28e92011059ddfa3255655dfde23b8a +size 18495 diff --git a/processed_dataset/images/volume12/figures/fig-c2i1.png b/processed_dataset/images/volume12/figures/fig-c2i1.png new file mode 100644 index 0000000000000000000000000000000000000000..be30b282c2be8456a435e2267ad42f50fe70b8e8 --- /dev/null +++ b/processed_dataset/images/volume12/figures/fig-c2i1.png @@ -0,0 +1,3 @@ +version https://git-lfs.github.com/spec/v1 +oid sha256:886da46ee1e3d665e7ddb8f3e2367edf1419a49bf2c42d21a8781ac56d97b963 +size 16999 diff --git a/processed_dataset/images/volume12/figures/fig-c2i2.png b/processed_dataset/images/volume12/figures/fig-c2i2.png new file mode 100644 index 0000000000000000000000000000000000000000..54616050083c97d6d42f8f2131b00e3a3e59cdf6 --- /dev/null +++ b/processed_dataset/images/volume12/figures/fig-c2i2.png @@ -0,0 +1,3 @@ +version https://git-lfs.github.com/spec/v1 +oid sha256:746d885b2719e79a03306363b6cff2d2f13a776d6f2d595a76173167abe5254c +size 29211 diff --git a/processed_dataset/images/volume12/figures/fig-c2i3.png b/processed_dataset/images/volume12/figures/fig-c2i3.png new file mode 100644 index 0000000000000000000000000000000000000000..43242e1e60b66cf7530a53578b97cabffbd234a8 --- /dev/null +++ b/processed_dataset/images/volume12/figures/fig-c2i3.png @@ -0,0 +1,3 @@ +version https://git-lfs.github.com/spec/v1 +oid sha256:4c638b5e8e9dde3d3f2d36af4e737fbb106b84ef3f6cb772cd321d79fe85bfbc +size 66739 diff --git a/processed_dataset/images/volume12/figures/fig-c2i4.png b/processed_dataset/images/volume12/figures/fig-c2i4.png new file mode 100644 index 0000000000000000000000000000000000000000..18bc92fb99bfbf7be3cc4554d60ae5507bf86633 --- /dev/null +++ b/processed_dataset/images/volume12/figures/fig-c2i4.png @@ -0,0 +1,3 @@ +version https://git-lfs.github.com/spec/v1 +oid sha256:55085e206830208af4bbc42f509122e79a5e8220be7d025ef80eae61651be369 +size 52685 diff --git a/processed_dataset/images/volume12/figures/fig-c2i5.png b/processed_dataset/images/volume12/figures/fig-c2i5.png new file mode 100644 index 0000000000000000000000000000000000000000..a777b4d1955c2ec20ca42bafb4734ff52fc4937d --- /dev/null +++ b/processed_dataset/images/volume12/figures/fig-c2i5.png @@ -0,0 +1,3 @@ +version https://git-lfs.github.com/spec/v1 +oid sha256:9cd5517d9aac72c94ce485def4fc85cc6b45c87fda0c67395d6f797049260648 +size 80942 diff --git a/processed_dataset/images/volume12/figures/fig-c3a10.png b/processed_dataset/images/volume12/figures/fig-c3a10.png new file mode 100644 index 0000000000000000000000000000000000000000..b338aa6df1d539df27337da08db57158108c6636 --- /dev/null +++ b/processed_dataset/images/volume12/figures/fig-c3a10.png @@ -0,0 +1,3 @@ +version https://git-lfs.github.com/spec/v1 +oid sha256:eda97122a0dc3a907cf6692036390d3cbf46501b1ccde15c9b33250dbce7ff5b +size 32107 diff --git a/processed_dataset/images/volume12/figures/fig-c3i1.png b/processed_dataset/images/volume12/figures/fig-c3i1.png new file mode 100644 index 0000000000000000000000000000000000000000..927a1a9a868a2470e3ff2cc94ccb132d4db9f900 --- /dev/null +++ b/processed_dataset/images/volume12/figures/fig-c3i1.png @@ -0,0 +1,3 @@ +version https://git-lfs.github.com/spec/v1 +oid sha256:5c5f8aa0dda0f31ed6baeebf090d510de3fb98b3c8d26e844dcebd844d64ff02 +size 29819 diff --git a/processed_dataset/images/volume12/figures/fig-c3i10.png b/processed_dataset/images/volume12/figures/fig-c3i10.png new file mode 100644 index 0000000000000000000000000000000000000000..30615ed45a0f911110aa9632c0fcec71d3743a7f --- /dev/null +++ b/processed_dataset/images/volume12/figures/fig-c3i10.png @@ -0,0 +1,3 @@ +version https://git-lfs.github.com/spec/v1 +oid sha256:897e45ef1e8472f9e2a54836c74a6775159df65941481e3230e6da0217a61cf4 +size 29903 diff --git a/processed_dataset/images/volume12/figures/fig-c3i11.png b/processed_dataset/images/volume12/figures/fig-c3i11.png new file mode 100644 index 0000000000000000000000000000000000000000..bc4f533b4b750bbd2ec9bce192c563e6ceb7ae29 --- /dev/null +++ b/processed_dataset/images/volume12/figures/fig-c3i11.png @@ -0,0 +1,3 @@ +version https://git-lfs.github.com/spec/v1 +oid sha256:3a05e1d3429db9af3664359e09c2237f28994c160bb8d82cf34360c72b45c517 +size 46531 diff --git a/processed_dataset/images/volume12/figures/fig-c3i12.png b/processed_dataset/images/volume12/figures/fig-c3i12.png new file mode 100644 index 0000000000000000000000000000000000000000..e112c7e54a06bc4896a16668d490a49a25a5cf44 --- /dev/null +++ b/processed_dataset/images/volume12/figures/fig-c3i12.png @@ -0,0 +1,3 @@ +version https://git-lfs.github.com/spec/v1 +oid sha256:93dd839324bc4f464e89637ea35b92536c87f7af4248010ab7014850277247f8 +size 28349 diff --git a/processed_dataset/images/volume12/figures/fig-c3i2.png b/processed_dataset/images/volume12/figures/fig-c3i2.png new file mode 100644 index 0000000000000000000000000000000000000000..87a9bc50db954ea1667e3a72478518e6ae028383 --- /dev/null +++ b/processed_dataset/images/volume12/figures/fig-c3i2.png @@ -0,0 +1,3 @@ +version https://git-lfs.github.com/spec/v1 +oid sha256:07bd29b37dbf5f433e57be4b874e20227e9da3d19b54a91e3f7e0ee66f604111 +size 30639 diff --git a/processed_dataset/images/volume12/figures/fig-c3i3.png b/processed_dataset/images/volume12/figures/fig-c3i3.png new file mode 100644 index 0000000000000000000000000000000000000000..299a6224ce92c15d1e29536a9e3e1c47201f0096 --- /dev/null +++ b/processed_dataset/images/volume12/figures/fig-c3i3.png @@ -0,0 +1,3 @@ +version https://git-lfs.github.com/spec/v1 +oid sha256:3a7197cc6351dfbb1ae4d708432f6c0d690b344a0b6e1bb355e2ea7327cc6af3 +size 23711 diff --git a/processed_dataset/images/volume12/figures/fig-c3i4.png b/processed_dataset/images/volume12/figures/fig-c3i4.png new file mode 100644 index 0000000000000000000000000000000000000000..8592ce906b646d6dc1b6bb1bdf22142e67766efe --- /dev/null +++ b/processed_dataset/images/volume12/figures/fig-c3i4.png @@ -0,0 +1,3 @@ +version https://git-lfs.github.com/spec/v1 +oid sha256:3b71206d4de74b23954f8cb4409d4e5222481419df7a366a76826a7684a87395 +size 45612 diff --git a/processed_dataset/images/volume12/figures/fig-c3i5.png b/processed_dataset/images/volume12/figures/fig-c3i5.png new file mode 100644 index 0000000000000000000000000000000000000000..07995ac26e4eb178063feb151908fbf639b2f533 --- /dev/null +++ b/processed_dataset/images/volume12/figures/fig-c3i5.png @@ -0,0 +1,3 @@ +version https://git-lfs.github.com/spec/v1 +oid sha256:38a408e346249221cb42bae863752e4ec9f64aa28a1c332d4b8cd8268c6ea7a6 +size 30504 diff --git a/processed_dataset/images/volume12/figures/fig-c3i6.png b/processed_dataset/images/volume12/figures/fig-c3i6.png new file mode 100644 index 0000000000000000000000000000000000000000..c50f901dbd7a44b17db6073d7245e4dd0c459ac9 --- /dev/null +++ b/processed_dataset/images/volume12/figures/fig-c3i6.png @@ -0,0 +1,3 @@ +version https://git-lfs.github.com/spec/v1 +oid sha256:d6bd9ba28144602361edac38615cb9f7a0325a34411a24b526c15fcce7768ead +size 38569 diff --git a/processed_dataset/images/volume12/figures/fig-c3i7.png b/processed_dataset/images/volume12/figures/fig-c3i7.png new file mode 100644 index 0000000000000000000000000000000000000000..50359650d0772277202975c7fa904b03258bf0ac --- /dev/null +++ b/processed_dataset/images/volume12/figures/fig-c3i7.png @@ -0,0 +1,3 @@ +version https://git-lfs.github.com/spec/v1 +oid sha256:374f5e08e0f24b8b4aabdc107cd643289084693edb876eb42bdca395b875c020 +size 40001 diff --git a/processed_dataset/images/volume12/figures/fig-c3i8.png b/processed_dataset/images/volume12/figures/fig-c3i8.png new file mode 100644 index 0000000000000000000000000000000000000000..e5f4737472354b526979075abed6b660638b39fa --- /dev/null +++ b/processed_dataset/images/volume12/figures/fig-c3i8.png @@ -0,0 +1,3 @@ +version https://git-lfs.github.com/spec/v1 +oid sha256:e8d9fa4d4536712d6800f0bceb9c6a2c2ea93ff3c0d0e41e3e188d616f1ec0ba +size 19532 diff --git a/processed_dataset/images/volume12/figures/fig-c3i9.png b/processed_dataset/images/volume12/figures/fig-c3i9.png new file mode 100644 index 0000000000000000000000000000000000000000..913afab5630f2f4ee6fa50584ba8e2cc485cfba4 --- /dev/null +++ b/processed_dataset/images/volume12/figures/fig-c3i9.png @@ -0,0 +1,3 @@ +version https://git-lfs.github.com/spec/v1 +oid sha256:cfc077b3d1f908510ed29dd2bfbb45c69ed9e2662838ca55ca505c1b0e4e9e21 +size 25161 diff --git a/processed_dataset/images/volume12/figures/fig-c4a4.png b/processed_dataset/images/volume12/figures/fig-c4a4.png new file mode 100644 index 0000000000000000000000000000000000000000..bb0a9bc867831445954696f24e2d8c49a29c2722 --- /dev/null +++ b/processed_dataset/images/volume12/figures/fig-c4a4.png @@ -0,0 +1,3 @@ +version https://git-lfs.github.com/spec/v1 +oid sha256:cf386c0dd48c5d9c0a37cae47d38c6625232739d02bedc702040a250cd7e3c9d +size 29162 diff --git a/processed_dataset/images/volume12/figures/fig-c4a9.png b/processed_dataset/images/volume12/figures/fig-c4a9.png new file mode 100644 index 0000000000000000000000000000000000000000..4523007f538b7a9725936795832660d998f003d8 --- /dev/null +++ b/processed_dataset/images/volume12/figures/fig-c4a9.png @@ -0,0 +1,3 @@ +version https://git-lfs.github.com/spec/v1 +oid sha256:b8cb49a06f045bed6928782523b5f3fd7f47db8549a40143869674af564d8306 +size 20483 diff --git a/processed_dataset/images/volume12/figures/fig-c4i1.png b/processed_dataset/images/volume12/figures/fig-c4i1.png new file mode 100644 index 0000000000000000000000000000000000000000..95a5b78bbad89dd18feac1146be415365e2c991b --- /dev/null +++ b/processed_dataset/images/volume12/figures/fig-c4i1.png @@ -0,0 +1,3 @@ +version https://git-lfs.github.com/spec/v1 +oid sha256:84bd0863470edb09e3de403439bf679fd2197defa69a5d6ad765cc88cbf18922 +size 26160 diff --git a/processed_dataset/images/volume12/figures/fig-c4i2.png b/processed_dataset/images/volume12/figures/fig-c4i2.png new file mode 100644 index 0000000000000000000000000000000000000000..8944a97e19d1fb34b5260c4b33a645a658e690e9 --- /dev/null +++ b/processed_dataset/images/volume12/figures/fig-c4i2.png @@ -0,0 +1,3 @@ +version https://git-lfs.github.com/spec/v1 +oid sha256:f2dec4e107f44fa7c97e2e865cd97938be60ec47eb300f65b55c8c1e03e17507 +size 17019 diff --git a/processed_dataset/images/volume12/figures/fig-c4i3.png b/processed_dataset/images/volume12/figures/fig-c4i3.png new file mode 100644 index 0000000000000000000000000000000000000000..b2a9d7048aadc5fc65c111734b3167b6faed4b43 --- /dev/null +++ b/processed_dataset/images/volume12/figures/fig-c4i3.png @@ -0,0 +1,3 @@ +version https://git-lfs.github.com/spec/v1 +oid sha256:763e2ab4be74152d6a6914f3317420aae07cd379a3f357b2167c15fd78062c25 +size 17294 diff --git a/processed_dataset/images/volume12/figures/fig-c4i4.png b/processed_dataset/images/volume12/figures/fig-c4i4.png new file mode 100644 index 0000000000000000000000000000000000000000..f24e3885fc0a0fc7d30ce83c9952eefe811f3054 --- /dev/null +++ b/processed_dataset/images/volume12/figures/fig-c4i4.png @@ -0,0 +1,3 @@ +version https://git-lfs.github.com/spec/v1 +oid sha256:73eed79d94d4362f1516ff41d3b75a59b6404910cb93cde2e99be5303342c264 +size 32062 diff --git a/processed_dataset/images/volume12/figures/fig-c4i5.png b/processed_dataset/images/volume12/figures/fig-c4i5.png new file mode 100644 index 0000000000000000000000000000000000000000..e4daebf5e6eacef94d5d67fd3508456b39880380 --- /dev/null +++ b/processed_dataset/images/volume12/figures/fig-c4i5.png @@ -0,0 +1,3 @@ +version https://git-lfs.github.com/spec/v1 +oid sha256:ce7af84d4ab7cc13ac8cb20e71d32735640031239f654dbc8c1484abc35936c9 +size 28667 diff --git a/processed_dataset/images/volume12/figures/fig-c4i6.png b/processed_dataset/images/volume12/figures/fig-c4i6.png new file mode 100644 index 0000000000000000000000000000000000000000..fbe01e16e637034dc14f9ee3fd02281b40eea271 --- /dev/null +++ b/processed_dataset/images/volume12/figures/fig-c4i6.png @@ -0,0 +1,3 @@ +version https://git-lfs.github.com/spec/v1 +oid sha256:2e563e91f8a09129b88b0ae1bd8e06f1a885673f6a99ec926a6367442e953444 +size 31274 diff --git a/processed_dataset/images/volume12/figures/fig-c4i7.png b/processed_dataset/images/volume12/figures/fig-c4i7.png new file mode 100644 index 0000000000000000000000000000000000000000..fc320833499842b43c029038a4d51cbe40f6823b --- /dev/null +++ b/processed_dataset/images/volume12/figures/fig-c4i7.png @@ -0,0 +1,3 @@ +version https://git-lfs.github.com/spec/v1 +oid sha256:529d8b99a8a4b3e5f20bfe30a05dbc579e5c961fc4f6cd7c13776c204bad4cd7 +size 28641 diff --git a/processed_dataset/images/volume12/figures/fig-c5a6.png b/processed_dataset/images/volume12/figures/fig-c5a6.png new file mode 100644 index 0000000000000000000000000000000000000000..332c1e6bfdb61f85668b5b29dbba28fa7d904ce2 --- /dev/null +++ b/processed_dataset/images/volume12/figures/fig-c5a6.png @@ -0,0 +1,3 @@ +version https://git-lfs.github.com/spec/v1 +oid sha256:acdd6e6fcefdfad5760d4dd889efe3b456a9f282a74b2c68d49993459c62aa9f +size 21455 diff --git a/processed_dataset/images/volume12/figures/fig-c5a7.png b/processed_dataset/images/volume12/figures/fig-c5a7.png new file mode 100644 index 0000000000000000000000000000000000000000..8d6a0ee282591fa02a915c39c24847ba6ce91323 --- /dev/null +++ b/processed_dataset/images/volume12/figures/fig-c5a7.png @@ -0,0 +1,3 @@ +version https://git-lfs.github.com/spec/v1 +oid sha256:a936dfce85108ee9492bfb10a7419d393ab8545ac530afe6d7789f862edbb51c +size 62456 diff --git a/processed_dataset/images/volume12/figures/fig-c5i1.png b/processed_dataset/images/volume12/figures/fig-c5i1.png new file mode 100644 index 0000000000000000000000000000000000000000..7ff31400c445b060613e53da7f3841080eb535e8 --- /dev/null +++ b/processed_dataset/images/volume12/figures/fig-c5i1.png @@ -0,0 +1,3 @@ +version https://git-lfs.github.com/spec/v1 +oid sha256:0cdc6bc66862dda939daabdb3dbcd6dc2e0fd08305a58599fff1df97cee9c42a +size 89168 diff --git a/processed_dataset/images/volume12/figures/fig-c5i10.png b/processed_dataset/images/volume12/figures/fig-c5i10.png new file mode 100644 index 0000000000000000000000000000000000000000..4173337f42998bd4561349658d300642fb609d87 --- /dev/null +++ b/processed_dataset/images/volume12/figures/fig-c5i10.png @@ -0,0 +1,3 @@ +version https://git-lfs.github.com/spec/v1 +oid sha256:a184af0f15f208843591c104de0030bcd0c2a7c0f14e736ffe976a878dd0b802 +size 22728 diff --git a/processed_dataset/images/volume12/figures/fig-c5i11.png b/processed_dataset/images/volume12/figures/fig-c5i11.png new file mode 100644 index 0000000000000000000000000000000000000000..ea4c8ea8a96e35c56abaf5ee64ea5b11e5ab02cc --- /dev/null +++ b/processed_dataset/images/volume12/figures/fig-c5i11.png @@ -0,0 +1,3 @@ +version https://git-lfs.github.com/spec/v1 +oid sha256:58eaadb944483db04853cc833dbc92cf4369e8bc1eb83318167c35d5e0161339 +size 22234 diff --git a/processed_dataset/images/volume12/figures/fig-c5i12.png b/processed_dataset/images/volume12/figures/fig-c5i12.png new file mode 100644 index 0000000000000000000000000000000000000000..0c6ef351ad71bd4706ce9e033ba6b651c58c30ca --- /dev/null +++ b/processed_dataset/images/volume12/figures/fig-c5i12.png @@ -0,0 +1,3 @@ +version https://git-lfs.github.com/spec/v1 +oid sha256:5a4b0e1efbea9c161a77fe84d4cb8bcceaf3152a90b135082a1bd207aba55c03 +size 28882 diff --git a/processed_dataset/images/volume12/figures/fig-c5i2.png b/processed_dataset/images/volume12/figures/fig-c5i2.png new file mode 100644 index 0000000000000000000000000000000000000000..3bcb2e18b281c02aba20a3eb684a1688ee200fa0 --- /dev/null +++ b/processed_dataset/images/volume12/figures/fig-c5i2.png @@ -0,0 +1,3 @@ +version https://git-lfs.github.com/spec/v1 +oid sha256:7cf7fefc0d29894cdbe308ddf72e77136aa7eb62cfc842f358047ef3a1037e03 +size 22912 diff --git a/processed_dataset/images/volume12/figures/fig-c5i3.png b/processed_dataset/images/volume12/figures/fig-c5i3.png new file mode 100644 index 0000000000000000000000000000000000000000..eb76c9bdc7572418a8cfa2b9183b8b27ae319494 --- /dev/null +++ b/processed_dataset/images/volume12/figures/fig-c5i3.png @@ -0,0 +1,3 @@ +version https://git-lfs.github.com/spec/v1 +oid sha256:228e4788f08fdb21410e0ce4956ea9471ed5139a57566eba991057aa30d96505 +size 26099 diff --git a/processed_dataset/images/volume12/figures/fig-c5i4.png b/processed_dataset/images/volume12/figures/fig-c5i4.png new file mode 100644 index 0000000000000000000000000000000000000000..b9c2a0e266420acd6a2313e4eb92659485cab321 --- /dev/null +++ b/processed_dataset/images/volume12/figures/fig-c5i4.png @@ -0,0 +1,3 @@ +version https://git-lfs.github.com/spec/v1 +oid sha256:52d68dd4b02b5df13d55aa3652ffe661e90dd8575f228fbba452d81e77ff7f8e +size 17673 diff --git a/processed_dataset/images/volume12/figures/fig-c5i5.png b/processed_dataset/images/volume12/figures/fig-c5i5.png new file mode 100644 index 0000000000000000000000000000000000000000..8f15e09d57cd6d5ad9b6c75a8c7bd8548c0ceb18 --- /dev/null +++ b/processed_dataset/images/volume12/figures/fig-c5i5.png @@ -0,0 +1,3 @@ +version https://git-lfs.github.com/spec/v1 +oid sha256:c2f2af8d7976d73d6bf4828550d9535a60fc03a1dd8cf81c065fb71f4c383615 +size 24476 diff --git a/processed_dataset/images/volume12/figures/fig-c5i6.png b/processed_dataset/images/volume12/figures/fig-c5i6.png new file mode 100644 index 0000000000000000000000000000000000000000..c53b0f4bc6cba2e09ee4d96d5a66d790d7bba420 --- /dev/null +++ b/processed_dataset/images/volume12/figures/fig-c5i6.png @@ -0,0 +1,3 @@ +version https://git-lfs.github.com/spec/v1 +oid sha256:61163d8e85369e805e1f5785b4add8fcc4e2ed52b682a36e0280670c0b09914c +size 16720 diff --git a/processed_dataset/images/volume12/figures/fig-c5i7.png b/processed_dataset/images/volume12/figures/fig-c5i7.png new file mode 100644 index 0000000000000000000000000000000000000000..811d8ebe8196a36009d1bcf0cccdd6bc3a27c832 --- /dev/null +++ b/processed_dataset/images/volume12/figures/fig-c5i7.png @@ -0,0 +1,3 @@ +version https://git-lfs.github.com/spec/v1 +oid sha256:224b1f20bc085741ce17988a7073291b1fb05559480ee5cfc64e8d5a91be1154 +size 75526 diff --git a/processed_dataset/images/volume12/figures/fig-c5i8.png b/processed_dataset/images/volume12/figures/fig-c5i8.png new file mode 100644 index 0000000000000000000000000000000000000000..4e57aacc76bdda43b21ffccc5bba1961fb83a4c2 --- /dev/null +++ b/processed_dataset/images/volume12/figures/fig-c5i8.png @@ -0,0 +1,3 @@ +version https://git-lfs.github.com/spec/v1 +oid sha256:7deb82a90aaf93adffeb918ad79583b9abd5112fcaf31302f9c968eab247b7e4 +size 75930 diff --git a/processed_dataset/images/volume12/figures/fig-c5i9.png b/processed_dataset/images/volume12/figures/fig-c5i9.png new file mode 100644 index 0000000000000000000000000000000000000000..0f7c9ed3dc9554a81c0c6fb9984b21d8c939ef73 --- /dev/null +++ b/processed_dataset/images/volume12/figures/fig-c5i9.png @@ -0,0 +1,3 @@ +version https://git-lfs.github.com/spec/v1 +oid sha256:02bdda4c30446e9f2f36e9c323964a9abd47d26ceffa616ae5d41da5a4177c98 +size 28163 diff --git a/processed_dataset/images/volume12/figures/fig-c5p3.png b/processed_dataset/images/volume12/figures/fig-c5p3.png new file mode 100644 index 0000000000000000000000000000000000000000..2546f014c7513e4faab2c25e2559420fbbe9e2dd --- /dev/null +++ b/processed_dataset/images/volume12/figures/fig-c5p3.png @@ -0,0 +1,3 @@ +version https://git-lfs.github.com/spec/v1 +oid sha256:ec9435397522f71e4632dfb72783a64d6ff8252852608227962b49a4f94a682a +size 35702 diff --git a/processed_dataset/images/volume12/figures/fig-c5p6.png b/processed_dataset/images/volume12/figures/fig-c5p6.png new file mode 100644 index 0000000000000000000000000000000000000000..432f6dc087395ec8db62afc1e5987f5e7b335598 --- /dev/null +++ b/processed_dataset/images/volume12/figures/fig-c5p6.png @@ -0,0 +1,3 @@ +version https://git-lfs.github.com/spec/v1 +oid sha256:4f462084d00d5f85deb95aab3010937ccbe9257c6b1b1e06ea10d366caeddde5 +size 29045 diff --git a/processed_dataset/images/volume12/figures/fig-c5p7.png b/processed_dataset/images/volume12/figures/fig-c5p7.png new file mode 100644 index 0000000000000000000000000000000000000000..641068b98840ceb9f5385d8baa33e902c125e862 --- /dev/null +++ b/processed_dataset/images/volume12/figures/fig-c5p7.png @@ -0,0 +1,3 @@ +version https://git-lfs.github.com/spec/v1 +oid sha256:1cab81941b41c1031c9defad241c5dc91afc86de41ad930564dbbc89259080e3 +size 19527 diff --git a/processed_dataset/images/volume12/figures/fig-c6a10-1.png b/processed_dataset/images/volume12/figures/fig-c6a10-1.png new file mode 100644 index 0000000000000000000000000000000000000000..18c200d72e09d1e060a98cc6f9062cb41dee69b1 --- /dev/null +++ b/processed_dataset/images/volume12/figures/fig-c6a10-1.png @@ -0,0 +1,3 @@ +version https://git-lfs.github.com/spec/v1 +oid sha256:54fb5e3f369b423138fd9fd93adb84311ee8e6fa6d644021c6887200b917641b +size 22343 diff --git a/processed_dataset/images/volume12/figures/fig-c6a10-2.png b/processed_dataset/images/volume12/figures/fig-c6a10-2.png new file mode 100644 index 0000000000000000000000000000000000000000..bbc70557c81d38c278ef6c73bc07cf7b6cb23128 --- /dev/null +++ b/processed_dataset/images/volume12/figures/fig-c6a10-2.png @@ -0,0 +1,3 @@ +version https://git-lfs.github.com/spec/v1 +oid sha256:51b494763dc1ddd8e7e001cacb39556fd2239d120bb5a00ed923e2120945d30d +size 22383 diff --git a/processed_dataset/images/volume12/figures/fig-c6a10-3.png b/processed_dataset/images/volume12/figures/fig-c6a10-3.png new file mode 100644 index 0000000000000000000000000000000000000000..6d6a636f3a80c7fa14c4141965fb670665f3345d --- /dev/null +++ b/processed_dataset/images/volume12/figures/fig-c6a10-3.png @@ -0,0 +1,3 @@ +version https://git-lfs.github.com/spec/v1 +oid sha256:62b1d693814743f4b9ed7fc21c0629ce491f624cc6ea7929b5e34f1bf68f17ba +size 27240 diff --git a/processed_dataset/images/volume12/figures/fig-c6a3.png b/processed_dataset/images/volume12/figures/fig-c6a3.png new file mode 100644 index 0000000000000000000000000000000000000000..a4f049afb5288894dfc69a113cbdfdb2f6cc1536 --- /dev/null +++ b/processed_dataset/images/volume12/figures/fig-c6a3.png @@ -0,0 +1,3 @@ +version https://git-lfs.github.com/spec/v1 +oid sha256:1b48cec5e0612d39b81942e11c363bf76cfc584e27bf3da2471ddc22fc38c9f3 +size 50460 diff --git a/processed_dataset/images/volume12/figures/fig-c6a9-1.png b/processed_dataset/images/volume12/figures/fig-c6a9-1.png new file mode 100644 index 0000000000000000000000000000000000000000..2aaf914697d7be2d7bb5c52adea5c59179e60877 --- /dev/null +++ b/processed_dataset/images/volume12/figures/fig-c6a9-1.png @@ -0,0 +1,3 @@ +version https://git-lfs.github.com/spec/v1 +oid sha256:46e38f85f562eee35934d7eea4ed02bb6619b861d8ba13b8ea7172838fda840b +size 6938 diff --git a/processed_dataset/images/volume12/figures/fig-c6a9-2.png b/processed_dataset/images/volume12/figures/fig-c6a9-2.png new file mode 100644 index 0000000000000000000000000000000000000000..ed626ff99c8df4c45abf4131ed27299fd7c3f950 --- /dev/null +++ b/processed_dataset/images/volume12/figures/fig-c6a9-2.png @@ -0,0 +1,3 @@ +version https://git-lfs.github.com/spec/v1 +oid sha256:2cbef60784fa50d8b30cfae837de5272d3f70019536200b64819a40bb2ebe04a +size 13893 diff --git a/processed_dataset/images/volume12/figures/fig-c6a9-3.png b/processed_dataset/images/volume12/figures/fig-c6a9-3.png new file mode 100644 index 0000000000000000000000000000000000000000..965c0f6a3f53dfbbedf5b83cc2012b12fbeb915f --- /dev/null +++ b/processed_dataset/images/volume12/figures/fig-c6a9-3.png @@ -0,0 +1,3 @@ +version https://git-lfs.github.com/spec/v1 +oid sha256:c9a3dedf4a5096642d0a5f2707bbed7a117db1d872e229d59d2216b9a887bcda +size 26298 diff --git a/processed_dataset/images/volume12/figures/fig-c6a9-4.png b/processed_dataset/images/volume12/figures/fig-c6a9-4.png new file mode 100644 index 0000000000000000000000000000000000000000..43a086e1c6e71df9c804e9caeb62ab26b5a7a714 --- /dev/null +++ b/processed_dataset/images/volume12/figures/fig-c6a9-4.png @@ -0,0 +1,3 @@ +version https://git-lfs.github.com/spec/v1 +oid sha256:56afd8c9d4b743dc19952af24bbb2cb7ce89190c1dfc2f7a4495430c39132c04 +size 60228 diff --git a/processed_dataset/images/volume12/figures/fig-c6i1.png b/processed_dataset/images/volume12/figures/fig-c6i1.png new file mode 100644 index 0000000000000000000000000000000000000000..e3611ff8c8d096f56c5e69365b5dd912a38cce53 --- /dev/null +++ b/processed_dataset/images/volume12/figures/fig-c6i1.png @@ -0,0 +1,3 @@ +version https://git-lfs.github.com/spec/v1 +oid sha256:4239fd33e75b6367d21b6a8356a6d2a0efb569f275c23460e8eada004154eb21 +size 36575 diff --git a/processed_dataset/images/volume12/figures/fig-c6i10.png b/processed_dataset/images/volume12/figures/fig-c6i10.png new file mode 100644 index 0000000000000000000000000000000000000000..c5773ba35eb423d850e499d9b3207ad17f9eb76f --- /dev/null +++ b/processed_dataset/images/volume12/figures/fig-c6i10.png @@ -0,0 +1,3 @@ +version https://git-lfs.github.com/spec/v1 +oid sha256:ab58d3956626a4cdea4362f3bf84a088c20297b31a09aa9f8d3a27b7da11c0d8 +size 13876 diff --git a/processed_dataset/images/volume12/figures/fig-c6i11.png b/processed_dataset/images/volume12/figures/fig-c6i11.png new file mode 100644 index 0000000000000000000000000000000000000000..78490f02b4133d67092f128600b0f9020352ec9b --- /dev/null +++ b/processed_dataset/images/volume12/figures/fig-c6i11.png @@ -0,0 +1,3 @@ +version https://git-lfs.github.com/spec/v1 +oid sha256:81a31fb04cd7a92199dd7fb34046dbaf95f2fd1f90c563292e68d23c149a5283 +size 26574 diff --git a/processed_dataset/images/volume12/figures/fig-c6i12.png b/processed_dataset/images/volume12/figures/fig-c6i12.png new file mode 100644 index 0000000000000000000000000000000000000000..18ebb4bf1818c3b0fdddd37a786f88d77cdfd78b --- /dev/null +++ b/processed_dataset/images/volume12/figures/fig-c6i12.png @@ -0,0 +1,3 @@ +version https://git-lfs.github.com/spec/v1 +oid sha256:ad6fb92831ee7434f736d2ad58828e080963fb722616361442cf91334f41a218 +size 42031 diff --git a/processed_dataset/images/volume12/figures/fig-c6i13.png b/processed_dataset/images/volume12/figures/fig-c6i13.png new file mode 100644 index 0000000000000000000000000000000000000000..1225b3779c650d210229ef5f5a06b875ab273de7 --- /dev/null +++ b/processed_dataset/images/volume12/figures/fig-c6i13.png @@ -0,0 +1,3 @@ +version https://git-lfs.github.com/spec/v1 +oid sha256:fab9c83b2fe83bec4405a5701e302eaa2fe421b9a5d4a6a9a6a8263819501f0f +size 28829 diff --git a/processed_dataset/images/volume12/figures/fig-c6i2.png b/processed_dataset/images/volume12/figures/fig-c6i2.png new file mode 100644 index 0000000000000000000000000000000000000000..5087efd3b42f462f3c87616dee391ca136f69f1a --- /dev/null +++ b/processed_dataset/images/volume12/figures/fig-c6i2.png @@ -0,0 +1,3 @@ +version https://git-lfs.github.com/spec/v1 +oid sha256:dac795f9c1f8aa20ec14fc7b565f634d3ff0ae3bd89afffb385ae03eff33bc24 +size 47433 diff --git a/processed_dataset/images/volume12/figures/fig-c6i3.png b/processed_dataset/images/volume12/figures/fig-c6i3.png new file mode 100644 index 0000000000000000000000000000000000000000..62c9e5c2f32eee94a654015e321b27d0865b86e2 --- /dev/null +++ b/processed_dataset/images/volume12/figures/fig-c6i3.png @@ -0,0 +1,3 @@ +version https://git-lfs.github.com/spec/v1 +oid sha256:9923da1531ae7f283249509b6fe7edfd331f849e04d3f7d5c8e6e87d128f2743 +size 34186 diff --git a/processed_dataset/images/volume12/figures/fig-c6i4.png b/processed_dataset/images/volume12/figures/fig-c6i4.png new file mode 100644 index 0000000000000000000000000000000000000000..be712063fb0b3be0b162a3f77257101303c59b1a --- /dev/null +++ b/processed_dataset/images/volume12/figures/fig-c6i4.png @@ -0,0 +1,3 @@ +version https://git-lfs.github.com/spec/v1 +oid sha256:6790b822e421bc616c0da3c47a586bba866a4bdf7870758fbf58298591a90942 +size 32194 diff --git a/processed_dataset/images/volume12/figures/fig-c6i5.png b/processed_dataset/images/volume12/figures/fig-c6i5.png new file mode 100644 index 0000000000000000000000000000000000000000..e6ce395cb80d88870d371cdbc4997d6b766fb823 --- /dev/null +++ b/processed_dataset/images/volume12/figures/fig-c6i5.png @@ -0,0 +1,3 @@ +version https://git-lfs.github.com/spec/v1 +oid sha256:aba2e7cb9203d12b1f046d21d43636585bebc442c8398305c724f03e821d321c +size 33096 diff --git a/processed_dataset/images/volume12/figures/fig-c6i6.png b/processed_dataset/images/volume12/figures/fig-c6i6.png new file mode 100644 index 0000000000000000000000000000000000000000..9a9ee2b3e5201868c176c2b01c314cf4dff7af81 --- /dev/null +++ b/processed_dataset/images/volume12/figures/fig-c6i6.png @@ -0,0 +1,3 @@ +version https://git-lfs.github.com/spec/v1 +oid sha256:39794c3a60681c02e652740a13b671be0146e446cdff41ec1c783cb47d4e6d7a +size 51685 diff --git a/processed_dataset/images/volume12/figures/fig-c6i7.png b/processed_dataset/images/volume12/figures/fig-c6i7.png new file mode 100644 index 0000000000000000000000000000000000000000..d34ed81049ee38e384564529f11c4e9957d9cacb --- /dev/null +++ b/processed_dataset/images/volume12/figures/fig-c6i7.png @@ -0,0 +1,3 @@ +version https://git-lfs.github.com/spec/v1 +oid sha256:d473f3660b8e5e382a687505da3c71f6dcdf1bdfabd379585b6572b2e0405487 +size 48685 diff --git a/processed_dataset/images/volume12/figures/fig-c6i8.png b/processed_dataset/images/volume12/figures/fig-c6i8.png new file mode 100644 index 0000000000000000000000000000000000000000..8b4f4024cf9780be3b6e45ca85082e1eb0664087 --- /dev/null +++ b/processed_dataset/images/volume12/figures/fig-c6i8.png @@ -0,0 +1,3 @@ +version https://git-lfs.github.com/spec/v1 +oid sha256:1f062301f996dddc75e307dcb3c4b2a717c5466502a2ff5e51d3f90ea459f56c +size 30436 diff --git a/processed_dataset/images/volume12/figures/fig-c6i9.png b/processed_dataset/images/volume12/figures/fig-c6i9.png new file mode 100644 index 0000000000000000000000000000000000000000..65ad388aa52506ed4ee9602dc58cb8ee4be4d4fb --- /dev/null +++ b/processed_dataset/images/volume12/figures/fig-c6i9.png @@ -0,0 +1,3 @@ +version https://git-lfs.github.com/spec/v1 +oid sha256:1bd4ef4b9e37e6f5623699740c89ac17f25197a88c6ef550174625ab69e3b4a7 +size 37212 diff --git a/processed_dataset/images/volume12/figures/fig-c7a12.png b/processed_dataset/images/volume12/figures/fig-c7a12.png new file mode 100644 index 0000000000000000000000000000000000000000..782ad9f38cb234c341a4809aa7acea60d9b580c8 --- /dev/null +++ b/processed_dataset/images/volume12/figures/fig-c7a12.png @@ -0,0 +1,3 @@ +version https://git-lfs.github.com/spec/v1 +oid sha256:937ca92bcefb7035055cdf01172ba43f8cbc8dc05d3221249a5da2de0627ab91 +size 16864 diff --git a/processed_dataset/images/volume12/figures/fig-c7i1-1.png b/processed_dataset/images/volume12/figures/fig-c7i1-1.png new file mode 100644 index 0000000000000000000000000000000000000000..1ce1da17491ffbb06893f071be699603683de481 --- /dev/null +++ b/processed_dataset/images/volume12/figures/fig-c7i1-1.png @@ -0,0 +1,3 @@ +version https://git-lfs.github.com/spec/v1 +oid sha256:0880127a022d37a3dc80c75b3609b18c2dde265a661d0aa78d3772048323521d +size 20642 diff --git a/processed_dataset/images/volume12/figures/fig-c7i1-2.png b/processed_dataset/images/volume12/figures/fig-c7i1-2.png new file mode 100644 index 0000000000000000000000000000000000000000..825305c28ad3cf9d65bf65fb6c3251dce5a1e0fa --- /dev/null +++ b/processed_dataset/images/volume12/figures/fig-c7i1-2.png @@ -0,0 +1,3 @@ +version https://git-lfs.github.com/spec/v1 +oid sha256:b60ff4b77cc40bb36cb9dde981b1247878549d5168a00b9dab1c989312da99b7 +size 23932 diff --git a/processed_dataset/images/volume12/figures/fig-c7i2.png b/processed_dataset/images/volume12/figures/fig-c7i2.png new file mode 100644 index 0000000000000000000000000000000000000000..188ec372e1be90e40880e60e2911ab87b6af7122 --- /dev/null +++ b/processed_dataset/images/volume12/figures/fig-c7i2.png @@ -0,0 +1,3 @@ +version https://git-lfs.github.com/spec/v1 +oid sha256:3751acf67a1e45c4df163d6b7f6a2bf02b0b1d0f8bcacee9e978d172d5615289 +size 29942 diff --git a/processed_dataset/images/volume12/figures/fig-c7i3-1.png b/processed_dataset/images/volume12/figures/fig-c7i3-1.png new file mode 100644 index 0000000000000000000000000000000000000000..268c94bca7cf3447cc3f541aab7811422f23861c --- /dev/null +++ b/processed_dataset/images/volume12/figures/fig-c7i3-1.png @@ -0,0 +1,3 @@ +version https://git-lfs.github.com/spec/v1 +oid sha256:572b8afe568a4490494ee47a37e300b42a416d6f7656245958b54297626f3c66 +size 17121 diff --git a/processed_dataset/images/volume12/figures/fig-c7i3-2.png b/processed_dataset/images/volume12/figures/fig-c7i3-2.png new file mode 100644 index 0000000000000000000000000000000000000000..e9c5ef389335e8e735028581ecfc860a6b6d8ab2 --- /dev/null +++ b/processed_dataset/images/volume12/figures/fig-c7i3-2.png @@ -0,0 +1,3 @@ +version https://git-lfs.github.com/spec/v1 +oid sha256:b9de3dd81f4299c3bf285fbb0d6129260752e80749708d9e09573455489ea1de +size 15556 diff --git a/processed_dataset/images/volume12/figures/fig-c7i4.png b/processed_dataset/images/volume12/figures/fig-c7i4.png new file mode 100644 index 0000000000000000000000000000000000000000..251cc4a96cf186486420b9766f57ff1308b132cb --- /dev/null +++ b/processed_dataset/images/volume12/figures/fig-c7i4.png @@ -0,0 +1,3 @@ +version https://git-lfs.github.com/spec/v1 +oid sha256:fed0d969a031f20aeef354af95741f2af1dd1f98a5757a801e5fcb59c2bea160 +size 37971 diff --git a/processed_dataset/images/volume12/figures/fig-c7i5.png b/processed_dataset/images/volume12/figures/fig-c7i5.png new file mode 100644 index 0000000000000000000000000000000000000000..d3ceae6a9fe8a6ffaac17da7e0e035047a91b225 --- /dev/null +++ b/processed_dataset/images/volume12/figures/fig-c7i5.png @@ -0,0 +1,3 @@ +version https://git-lfs.github.com/spec/v1 +oid sha256:2718cb3208939fe6d17ea40f5eb94b4005cbb587e5a7be1bcd6c396ef2640bcd +size 23805 diff --git a/processed_dataset/images/volume12/figures/fig-c7i6.png b/processed_dataset/images/volume12/figures/fig-c7i6.png new file mode 100644 index 0000000000000000000000000000000000000000..20d1c27647f196dd441830f0fbacfcb4c4d275e5 --- /dev/null +++ b/processed_dataset/images/volume12/figures/fig-c7i6.png @@ -0,0 +1,3 @@ +version https://git-lfs.github.com/spec/v1 +oid sha256:f593fe7647c90d17eff217bdfda75ca7b18c205f6bd9996329a777619f343a00 +size 20298 diff --git a/processed_dataset/images/volume12/figures/fig-c7i7.png b/processed_dataset/images/volume12/figures/fig-c7i7.png new file mode 100644 index 0000000000000000000000000000000000000000..c7c31d3c566d1d50b956eda1016682cc4efdca73 --- /dev/null +++ b/processed_dataset/images/volume12/figures/fig-c7i7.png @@ -0,0 +1,3 @@ +version https://git-lfs.github.com/spec/v1 +oid sha256:bdb7fc0d930207e68c7d381191f23df1aca30c4683f7ce07cf21bca77f988c65 +size 92868 diff --git a/processed_dataset/images/volume12/figures/fig-c7i8.png b/processed_dataset/images/volume12/figures/fig-c7i8.png new file mode 100644 index 0000000000000000000000000000000000000000..5b1e2a2e104077686977fd805cfa3e5f893a1da7 --- /dev/null +++ b/processed_dataset/images/volume12/figures/fig-c7i8.png @@ -0,0 +1,3 @@ +version https://git-lfs.github.com/spec/v1 +oid sha256:65d753ebc16b5e38a2f5ecd1e099a6af9dd6767afec2f3a9f6ab5bc18df9f794 +size 57107 diff --git a/processed_dataset/images/volume12/figures/fig-c7p10.png b/processed_dataset/images/volume12/figures/fig-c7p10.png new file mode 100644 index 0000000000000000000000000000000000000000..b267e0c9db29410d60cb703579449f585364e9d8 --- /dev/null +++ b/processed_dataset/images/volume12/figures/fig-c7p10.png @@ -0,0 +1,3 @@ +version https://git-lfs.github.com/spec/v1 +oid sha256:6334b8e68484eb42f916ef9181a0f9b04d29848f1e5182d3231f8de14bdf144a +size 22231 diff --git a/processed_dataset/images/volume12/figures/fig-c7p11.png b/processed_dataset/images/volume12/figures/fig-c7p11.png new file mode 100644 index 0000000000000000000000000000000000000000..6d1bb195aa47f70ffcb85c3e70c6638e0e858948 --- /dev/null +++ b/processed_dataset/images/volume12/figures/fig-c7p11.png @@ -0,0 +1,3 @@ +version https://git-lfs.github.com/spec/v1 +oid sha256:4b7a2d2973c9a3fed7e39aed78c1c5bdef03e16aee26335bcce2eb25efc2be5b +size 23276 diff --git a/processed_dataset/images/volume12/figures/fig-c8a1.png b/processed_dataset/images/volume12/figures/fig-c8a1.png new file mode 100644 index 0000000000000000000000000000000000000000..7f1789e6d1ac939256b2be35a6dc18116bbfeee2 --- /dev/null +++ b/processed_dataset/images/volume12/figures/fig-c8a1.png @@ -0,0 +1,3 @@ +version https://git-lfs.github.com/spec/v1 +oid sha256:d655192c04d35987b9f02a9ab966749986fa90fca2a209a89ff1e7cad35b604a +size 29632 diff --git a/processed_dataset/images/volume12/figures/fig-c8a12-1.png b/processed_dataset/images/volume12/figures/fig-c8a12-1.png new file mode 100644 index 0000000000000000000000000000000000000000..5b7ef8ca4dbcfe351b326dc100539aa0c73873fd --- /dev/null +++ b/processed_dataset/images/volume12/figures/fig-c8a12-1.png @@ -0,0 +1,3 @@ +version https://git-lfs.github.com/spec/v1 +oid sha256:0f766222c6d9ee14438ccfe0ba3e5e1724e3f6f6e1a26a03f13ef07e6581abd4 +size 39172 diff --git a/processed_dataset/images/volume12/figures/fig-c8a12-2.png b/processed_dataset/images/volume12/figures/fig-c8a12-2.png new file mode 100644 index 0000000000000000000000000000000000000000..4fcb498b63272f2d2418a623719e7467e8f1bfa3 --- /dev/null +++ b/processed_dataset/images/volume12/figures/fig-c8a12-2.png @@ -0,0 +1,3 @@ +version https://git-lfs.github.com/spec/v1 +oid sha256:55abba1229d9b8332069336b07956086f25720751a33e3e706a97bf2ca96e20b +size 34539 diff --git a/processed_dataset/images/volume12/figures/fig-c8a2.png b/processed_dataset/images/volume12/figures/fig-c8a2.png new file mode 100644 index 0000000000000000000000000000000000000000..09d50fce0c868ea1d8937c06ea1e87bfaa3da27e --- /dev/null +++ b/processed_dataset/images/volume12/figures/fig-c8a2.png @@ -0,0 +1,3 @@ +version https://git-lfs.github.com/spec/v1 +oid sha256:89a32da20458f46c0dfa828dc16e5a08f1e854108b45e875a1a4b56abb6b4dbf +size 30770 diff --git a/processed_dataset/images/volume12/figures/fig-c8a5.png b/processed_dataset/images/volume12/figures/fig-c8a5.png new file mode 100644 index 0000000000000000000000000000000000000000..0439a51ce8ec62d54fc849b2c19de0368ded3750 --- /dev/null +++ b/processed_dataset/images/volume12/figures/fig-c8a5.png @@ -0,0 +1,3 @@ +version https://git-lfs.github.com/spec/v1 +oid sha256:0444872db2c316562b97da189e07bb7fd065da5fe252feac68473d8a31858295 +size 25362 diff --git a/processed_dataset/images/volume12/figures/fig-c8i1.png b/processed_dataset/images/volume12/figures/fig-c8i1.png new file mode 100644 index 0000000000000000000000000000000000000000..236e1d67b443517c7514001472d5c969fce9ddde --- /dev/null +++ b/processed_dataset/images/volume12/figures/fig-c8i1.png @@ -0,0 +1,3 @@ +version https://git-lfs.github.com/spec/v1 +oid sha256:65f4ac5b374ec7d27a8da6320a4e46507c0b72274f9a0e71cfd532d779daf1af +size 21877 diff --git a/processed_dataset/images/volume12/figures/fig-c8i2.png b/processed_dataset/images/volume12/figures/fig-c8i2.png new file mode 100644 index 0000000000000000000000000000000000000000..73d65c0dd4b6f846de8876e8863370d9a2c4707b --- /dev/null +++ b/processed_dataset/images/volume12/figures/fig-c8i2.png @@ -0,0 +1,3 @@ +version https://git-lfs.github.com/spec/v1 +oid sha256:70b721688ef07f818f277f9954a86d0cfdd628d34ed7fd3bc9c1445e4e5da543 +size 18017 diff --git a/processed_dataset/images/volume12/figures/fig-c8i3.png b/processed_dataset/images/volume12/figures/fig-c8i3.png new file mode 100644 index 0000000000000000000000000000000000000000..4bce2a1f6a0c728ce01ebaad10b2d3c591040a45 --- /dev/null +++ b/processed_dataset/images/volume12/figures/fig-c8i3.png @@ -0,0 +1,3 @@ +version https://git-lfs.github.com/spec/v1 +oid sha256:0ca459a66969ccc603cc60fbe36609bfeaf6b8269f4c5667bae19b19017a67af +size 23874 diff --git a/processed_dataset/images/volume12/figures/fig-c8i4.png b/processed_dataset/images/volume12/figures/fig-c8i4.png new file mode 100644 index 0000000000000000000000000000000000000000..1b5c0a73e79c5664ff5704bee64b68748f602e5e --- /dev/null +++ b/processed_dataset/images/volume12/figures/fig-c8i4.png @@ -0,0 +1,3 @@ +version https://git-lfs.github.com/spec/v1 +oid sha256:a3113d69223a482fbc29e699fe7e95c4bd63857ba5ecf70ac83aadf7ad3810c4 +size 67799 diff --git a/processed_dataset/images/volume12/figures/fig-c8i5.png b/processed_dataset/images/volume12/figures/fig-c8i5.png new file mode 100644 index 0000000000000000000000000000000000000000..40b2d8281e7a391c0b7c124c80f33a95b1909f29 --- /dev/null +++ b/processed_dataset/images/volume12/figures/fig-c8i5.png @@ -0,0 +1,3 @@ +version https://git-lfs.github.com/spec/v1 +oid sha256:ed9e923951069d8ee3e73a5782a52dab69968448b4dd133509b02d0e817ab2a5 +size 100834 diff --git a/processed_dataset/images/volume12/figures/fig-c8i6.png b/processed_dataset/images/volume12/figures/fig-c8i6.png new file mode 100644 index 0000000000000000000000000000000000000000..6488bfa183d39d2fb89a1e4911a8670a20a23eae --- /dev/null +++ b/processed_dataset/images/volume12/figures/fig-c8i6.png @@ -0,0 +1,3 @@ +version https://git-lfs.github.com/spec/v1 +oid sha256:1c61fbef6cc1f8157c401a778f5aede4233a37f71c0bd9da181f77e7cc0f82fa +size 41691 diff --git a/processed_dataset/images/volume12/figures/fig-c8i7.png b/processed_dataset/images/volume12/figures/fig-c8i7.png new file mode 100644 index 0000000000000000000000000000000000000000..e46fd2a320938d467373598dd47b1a49a0d560d4 --- /dev/null +++ b/processed_dataset/images/volume12/figures/fig-c8i7.png @@ -0,0 +1,3 @@ +version https://git-lfs.github.com/spec/v1 +oid sha256:aad0b9fb55e4c2f3b67f7d4dbcc915a997ed044f895377b37c4e649d5930880e +size 43642 diff --git a/processed_dataset/images/volume12/figures/fig-c8i8.png b/processed_dataset/images/volume12/figures/fig-c8i8.png new file mode 100644 index 0000000000000000000000000000000000000000..d52ecd7786c91a52684a8d357703fd37325667a9 --- /dev/null +++ b/processed_dataset/images/volume12/figures/fig-c8i8.png @@ -0,0 +1,3 @@ +version https://git-lfs.github.com/spec/v1 +oid sha256:4238cefe132a55c3b9a6471aa7b72beb3f73d23c2615c566dc7a428a223a987c +size 20125 diff --git a/processed_dataset/images/volume12/figures/fig-c9a1-1.png b/processed_dataset/images/volume12/figures/fig-c9a1-1.png new file mode 100644 index 0000000000000000000000000000000000000000..bf01ace7dce32fc5100fc4106d39857a3b341a1d --- /dev/null +++ b/processed_dataset/images/volume12/figures/fig-c9a1-1.png @@ -0,0 +1,3 @@ +version https://git-lfs.github.com/spec/v1 +oid sha256:6cbea8c8aa5452aed859becb2ef568bcf1e97178442b5d9602dbf44d6e758e60 +size 25200 diff --git a/processed_dataset/images/volume12/figures/fig-c9a1-2.png b/processed_dataset/images/volume12/figures/fig-c9a1-2.png new file mode 100644 index 0000000000000000000000000000000000000000..271aaf293b9db7ca5f42e28904535384c140afcc --- /dev/null +++ b/processed_dataset/images/volume12/figures/fig-c9a1-2.png @@ -0,0 +1,3 @@ +version https://git-lfs.github.com/spec/v1 +oid sha256:45d558cb3aa2ee25419bc1f44d9a492cec040e7e3b3a18bfcccd5fb2f085704e +size 31397 diff --git a/processed_dataset/images/volume12/figures/fig-c9i1.png b/processed_dataset/images/volume12/figures/fig-c9i1.png new file mode 100644 index 0000000000000000000000000000000000000000..e25867d8c862c6ff99266b3bc68a43bcb4dc9a25 --- /dev/null +++ b/processed_dataset/images/volume12/figures/fig-c9i1.png @@ -0,0 +1,3 @@ +version https://git-lfs.github.com/spec/v1 +oid sha256:9aea6a1531d89fcf66a5112bb4ad9794878db6cc1ca02add7465fd8b48fce5f3 +size 25620 diff --git a/processed_dataset/images/volume12/figures/fig-c9i2.png b/processed_dataset/images/volume12/figures/fig-c9i2.png new file mode 100644 index 0000000000000000000000000000000000000000..2574c817d8c54518d2672108cbb2436a71340dff --- /dev/null +++ b/processed_dataset/images/volume12/figures/fig-c9i2.png @@ -0,0 +1,3 @@ +version https://git-lfs.github.com/spec/v1 +oid sha256:f332b59a177c44acaf2e11c4f13da40afd5210b65cbf6580d948d3f0f610f713 +size 28305 diff --git a/processed_dataset/images/volume13/figures/fig-c11a4.png b/processed_dataset/images/volume13/figures/fig-c11a4.png new file mode 100644 index 0000000000000000000000000000000000000000..2858369a2924780cb741475d4d7826e998f7e65e --- /dev/null +++ b/processed_dataset/images/volume13/figures/fig-c11a4.png @@ -0,0 +1,3 @@ +version https://git-lfs.github.com/spec/v1 +oid sha256:5924f3f9ef9f609bbcc9938efd14d49a840dff93caab5dd2bb1ee49b3289888f +size 24967 diff --git a/processed_dataset/images/volume13/figures/fig-c11i1.png b/processed_dataset/images/volume13/figures/fig-c11i1.png new file mode 100644 index 0000000000000000000000000000000000000000..d856b1a7d9864d44d67e7ce2c262ae876f5138ab --- /dev/null +++ b/processed_dataset/images/volume13/figures/fig-c11i1.png @@ -0,0 +1,3 @@ +version https://git-lfs.github.com/spec/v1 +oid sha256:33f75442fa35ca47554848f24f67a6aca36db214e8f428d0575925631b67a40e +size 23856 diff --git a/processed_dataset/images/volume13/figures/fig-c13i1.png b/processed_dataset/images/volume13/figures/fig-c13i1.png new file mode 100644 index 0000000000000000000000000000000000000000..c1ae6022a035c18095b1c40fdae3cce1880c1eb9 --- /dev/null +++ b/processed_dataset/images/volume13/figures/fig-c13i1.png @@ -0,0 +1,3 @@ +version https://git-lfs.github.com/spec/v1 +oid sha256:0e9d5393ed63de585ba25a214e9deaf46196e26516b0b386fc9cc4b8e130de18 +size 32932 diff --git a/processed_dataset/images/volume13/figures/fig-c13i2.png b/processed_dataset/images/volume13/figures/fig-c13i2.png new file mode 100644 index 0000000000000000000000000000000000000000..cfcef07d68f716a95363c9ffa1eb39d0abbc54c2 --- /dev/null +++ b/processed_dataset/images/volume13/figures/fig-c13i2.png @@ -0,0 +1,3 @@ +version https://git-lfs.github.com/spec/v1 +oid sha256:5787c1889a8eb683b1ef15d3c331b0eebd011b841a2fcdb738be1bd0ece8820e +size 31351 diff --git a/processed_dataset/images/volume13/figures/fig-c13i3.png b/processed_dataset/images/volume13/figures/fig-c13i3.png new file mode 100644 index 0000000000000000000000000000000000000000..e8c8013d11c9eea6a825a1e53f5beccb4acd9f46 --- /dev/null +++ b/processed_dataset/images/volume13/figures/fig-c13i3.png @@ -0,0 +1,3 @@ +version https://git-lfs.github.com/spec/v1 +oid sha256:3e51b74710bbed5225f3067443c37cccde6eda09cc49bfef7157b128ae4a5826 +size 36148 diff --git a/processed_dataset/images/volume13/figures/fig-c13i4.png b/processed_dataset/images/volume13/figures/fig-c13i4.png new file mode 100644 index 0000000000000000000000000000000000000000..0945619f1a0a2527093406317bfd26bdd402c54d --- /dev/null +++ b/processed_dataset/images/volume13/figures/fig-c13i4.png @@ -0,0 +1,3 @@ +version https://git-lfs.github.com/spec/v1 +oid sha256:73c1db6425047329587d3a2e0e6682428f6e61012686e45e505209020c0c80b6 +size 38306 diff --git a/processed_dataset/images/volume13/figures/fig-c13i5.png b/processed_dataset/images/volume13/figures/fig-c13i5.png new file mode 100644 index 0000000000000000000000000000000000000000..83ab39e2efbbef10c04e91388ac892ee979df04a --- /dev/null +++ b/processed_dataset/images/volume13/figures/fig-c13i5.png @@ -0,0 +1,3 @@ +version https://git-lfs.github.com/spec/v1 +oid sha256:894a6d6b466242a13cb383f3caf15aab473c7663a3910f732fedc93a224074f5 +size 20079 diff --git a/processed_dataset/images/volume13/figures/fig-c13i6.png b/processed_dataset/images/volume13/figures/fig-c13i6.png new file mode 100644 index 0000000000000000000000000000000000000000..f4d33440db55b97f4cf0da1b4f63e2032bca6a98 --- /dev/null +++ b/processed_dataset/images/volume13/figures/fig-c13i6.png @@ -0,0 +1,3 @@ +version https://git-lfs.github.com/spec/v1 +oid sha256:08415af5bd553aac74cafae7ab6c7a919ae0138cce8071cab4be93d60df07291 +size 42575 diff --git a/processed_dataset/images/volume13/figures/fig-c13i7.png b/processed_dataset/images/volume13/figures/fig-c13i7.png new file mode 100644 index 0000000000000000000000000000000000000000..42c746541d22b0f7de51acd1eb1595964fe550a0 --- /dev/null +++ b/processed_dataset/images/volume13/figures/fig-c13i7.png @@ -0,0 +1,3 @@ +version https://git-lfs.github.com/spec/v1 +oid sha256:99838e85107125d3f33c5eb6bd48a975f02e6ba43eeb50d475606846954bad47 +size 39273 diff --git a/processed_dataset/images/volume13/figures/fig-c13i8.png b/processed_dataset/images/volume13/figures/fig-c13i8.png new file mode 100644 index 0000000000000000000000000000000000000000..6a2e6cbe586444e0cf11e9b3331c3c046e89c2a1 --- /dev/null +++ b/processed_dataset/images/volume13/figures/fig-c13i8.png @@ -0,0 +1,3 @@ +version https://git-lfs.github.com/spec/v1 +oid sha256:1d4a9eacff697667a99fa50b5e79b8fc09b3ea1672ebb8531f9417be586a460c +size 55359 diff --git a/processed_dataset/images/volume13/figures/fig-c7i1.png b/processed_dataset/images/volume13/figures/fig-c7i1.png new file mode 100644 index 0000000000000000000000000000000000000000..ead2d8f3ba623751acee07ebc5cd5650e68f5cf6 --- /dev/null +++ b/processed_dataset/images/volume13/figures/fig-c7i1.png @@ -0,0 +1,3 @@ +version https://git-lfs.github.com/spec/v1 +oid sha256:b9a5359ab67c651208b3c8697816c484dff89b0f1a2c0a235396a53bdce7661f +size 47136 diff --git a/processed_dataset/images/volume13/figures/fig-c7i2.png b/processed_dataset/images/volume13/figures/fig-c7i2.png new file mode 100644 index 0000000000000000000000000000000000000000..36fefebbb27d0abdeec4c0a3ad40d39370a1c163 --- /dev/null +++ b/processed_dataset/images/volume13/figures/fig-c7i2.png @@ -0,0 +1,3 @@ +version https://git-lfs.github.com/spec/v1 +oid sha256:fdc352d531f2e71c777c13462b212d9fc24ea5ac357f6d779bd748853f325bac +size 45050 diff --git a/processed_dataset/images/volume13/figures/fig-c7i3.png b/processed_dataset/images/volume13/figures/fig-c7i3.png new file mode 100644 index 0000000000000000000000000000000000000000..4baac6c6c7a25fcefbd8a0559d0301af00aa1636 --- /dev/null +++ b/processed_dataset/images/volume13/figures/fig-c7i3.png @@ -0,0 +1,3 @@ +version https://git-lfs.github.com/spec/v1 +oid sha256:8e3d501b8dd95ab5528b32b566d916a75e05ceaa8439db30f4ba17bdda1ee2ba +size 61514 diff --git a/processed_dataset/images/volume13/figures/fig-c7i4.png b/processed_dataset/images/volume13/figures/fig-c7i4.png new file mode 100644 index 0000000000000000000000000000000000000000..030bde20b442a6dcd011f7300fac41c6e6ebdf53 --- /dev/null +++ b/processed_dataset/images/volume13/figures/fig-c7i4.png @@ -0,0 +1,3 @@ +version https://git-lfs.github.com/spec/v1 +oid sha256:b7f20e21c1dafa6df2f7bfaff2a75813b26e444b9f1edcbc9059592a927f6f5d +size 60799 diff --git a/processed_dataset/images/volume13/figures/fig-c7i5.png b/processed_dataset/images/volume13/figures/fig-c7i5.png new file mode 100644 index 0000000000000000000000000000000000000000..56891212893ab32a0ebed58b321ae546a0bba159 --- /dev/null +++ b/processed_dataset/images/volume13/figures/fig-c7i5.png @@ -0,0 +1,3 @@ +version https://git-lfs.github.com/spec/v1 +oid sha256:449c5a24693cea2ce923d0c04127d45fa8ab56e16e778612f1766c577a900b3c +size 24239 diff --git a/processed_dataset/images/volume13/figures/fig-c7i6.png b/processed_dataset/images/volume13/figures/fig-c7i6.png new file mode 100644 index 0000000000000000000000000000000000000000..3b80836453a0636d0a884c7be1e532aaa3afc6b4 --- /dev/null +++ b/processed_dataset/images/volume13/figures/fig-c7i6.png @@ -0,0 +1,3 @@ +version https://git-lfs.github.com/spec/v1 +oid sha256:22a4499fe523f126e91fa15b99ac12510a7a485e72caf669c5e4255a5b52ed95 +size 66753 diff --git a/processed_dataset/images/volume13/figures/fig-c7i7.png b/processed_dataset/images/volume13/figures/fig-c7i7.png new file mode 100644 index 0000000000000000000000000000000000000000..017facc035b14a168f1e0f985ba4dd5639340ee9 --- /dev/null +++ b/processed_dataset/images/volume13/figures/fig-c7i7.png @@ -0,0 +1,3 @@ +version https://git-lfs.github.com/spec/v1 +oid sha256:ce875eddc05e708fb65c7c32b65c883db9c31544f318e1df229db5f5f2a464fd +size 38898 diff --git a/processed_dataset/images/volume13/figures/fig-c9a6.png b/processed_dataset/images/volume13/figures/fig-c9a6.png new file mode 100644 index 0000000000000000000000000000000000000000..f1f39f5841835b41e1497f1c2271bc2316965a8c --- /dev/null +++ b/processed_dataset/images/volume13/figures/fig-c9a6.png @@ -0,0 +1,3 @@ +version https://git-lfs.github.com/spec/v1 +oid sha256:213eed7de0d2b58ffa693196ee4527273387740405254967813ee64e73f55280 +size 25554 diff --git a/processed_dataset/images/volume14/figures/fig-c10a8.png b/processed_dataset/images/volume14/figures/fig-c10a8.png new file mode 100644 index 0000000000000000000000000000000000000000..948f199557670f7805f991089bf27d94737ff7c3 --- /dev/null +++ b/processed_dataset/images/volume14/figures/fig-c10a8.png @@ -0,0 +1,3 @@ +version https://git-lfs.github.com/spec/v1 +oid sha256:e71c00b080e9b69b802170948d2b4249c1269c2d22b1285690167f39c5b65cbf +size 34359 diff --git a/processed_dataset/images/volume14/figures/fig-c10i1.png b/processed_dataset/images/volume14/figures/fig-c10i1.png new file mode 100644 index 0000000000000000000000000000000000000000..e21b182494acbc6c6cebd19dad850ae365af71a5 --- /dev/null +++ b/processed_dataset/images/volume14/figures/fig-c10i1.png @@ -0,0 +1,3 @@ +version https://git-lfs.github.com/spec/v1 +oid sha256:5e238ffe2baca6a48b07a299abb1c44ee8a9c68b31323742acfc38d459720418 +size 34319 diff --git a/processed_dataset/images/volume14/figures/fig-c11a1.png b/processed_dataset/images/volume14/figures/fig-c11a1.png new file mode 100644 index 0000000000000000000000000000000000000000..8e7184933e5fca81fe70ebfa1e4df604e0e0502a --- /dev/null +++ b/processed_dataset/images/volume14/figures/fig-c11a1.png @@ -0,0 +1,3 @@ +version https://git-lfs.github.com/spec/v1 +oid sha256:e45cbe168d24f7aa119c188c43d9c6003d5f4f3117bd659f3c24a4069908d494 +size 30434 diff --git a/processed_dataset/images/volume14/figures/fig-c11a2.png b/processed_dataset/images/volume14/figures/fig-c11a2.png new file mode 100644 index 0000000000000000000000000000000000000000..5835664f1da2744e729462f8e4324a4efbe0aef5 --- /dev/null +++ b/processed_dataset/images/volume14/figures/fig-c11a2.png @@ -0,0 +1,3 @@ +version https://git-lfs.github.com/spec/v1 +oid sha256:bad51f38dca0dffb6c405fcbd8ae628399d1694a6e5191095413973adfbd0fe8 +size 38270 diff --git a/processed_dataset/images/volume14/figures/fig-c11a6.png b/processed_dataset/images/volume14/figures/fig-c11a6.png new file mode 100644 index 0000000000000000000000000000000000000000..46b5c31da8994670f76313e4d4681f4d3659ee56 --- /dev/null +++ b/processed_dataset/images/volume14/figures/fig-c11a6.png @@ -0,0 +1,3 @@ +version https://git-lfs.github.com/spec/v1 +oid sha256:bed754cb370268cda7b7e9b678ac1f569d4badbc2c57ad3e379bd2b8740c9b29 +size 24994 diff --git a/processed_dataset/images/volume14/figures/fig-c11i1.png b/processed_dataset/images/volume14/figures/fig-c11i1.png new file mode 100644 index 0000000000000000000000000000000000000000..891681ad5552b5f57bf93854ad53640838a3c96d --- /dev/null +++ b/processed_dataset/images/volume14/figures/fig-c11i1.png @@ -0,0 +1,3 @@ +version https://git-lfs.github.com/spec/v1 +oid sha256:f9e7bb0afb81c5a187f7b7850968a87a98ee3e096018446a1735277a8cfb0d26 +size 29045 diff --git a/processed_dataset/images/volume14/figures/fig-c11i2.png b/processed_dataset/images/volume14/figures/fig-c11i2.png new file mode 100644 index 0000000000000000000000000000000000000000..181a45f8a6a8ee5269391838f631b3abb3669fec --- /dev/null +++ b/processed_dataset/images/volume14/figures/fig-c11i2.png @@ -0,0 +1,3 @@ +version https://git-lfs.github.com/spec/v1 +oid sha256:1eb12ac06f9beca04e5a6898f3e4145a5cb097fc9d70e688a0d702ab66fee76c +size 25898 diff --git a/processed_dataset/images/volume14/figures/fig-c11i3.png b/processed_dataset/images/volume14/figures/fig-c11i3.png new file mode 100644 index 0000000000000000000000000000000000000000..4ae7debed6734a724e4a4e9d8cc3f4195db45620 --- /dev/null +++ b/processed_dataset/images/volume14/figures/fig-c11i3.png @@ -0,0 +1,3 @@ +version https://git-lfs.github.com/spec/v1 +oid sha256:0e458f54bf2f122d6d4965afe343bfaab7c0eb7c8afacd72bc8f55547f8d922f +size 20973 diff --git a/processed_dataset/images/volume14/figures/fig-c11i4.png b/processed_dataset/images/volume14/figures/fig-c11i4.png new file mode 100644 index 0000000000000000000000000000000000000000..2d4548981811714c78baebd4cb15e1937e38448a --- /dev/null +++ b/processed_dataset/images/volume14/figures/fig-c11i4.png @@ -0,0 +1,3 @@ +version https://git-lfs.github.com/spec/v1 +oid sha256:98b72a0744401a32a26c59a6afe75c759990330fd85643a5eeb7d9bc7a05aec4 +size 35172 diff --git a/processed_dataset/images/volume14/figures/fig-c13i1.png b/processed_dataset/images/volume14/figures/fig-c13i1.png new file mode 100644 index 0000000000000000000000000000000000000000..5a134f8c8b058dbd63658ebbd25472511e569336 --- /dev/null +++ b/processed_dataset/images/volume14/figures/fig-c13i1.png @@ -0,0 +1,3 @@ +version https://git-lfs.github.com/spec/v1 +oid sha256:64574937e6c8fc635e40ee13c4f5fbbdd25fcb6e2cd801a3b84c5dd137294483 +size 20667 diff --git a/processed_dataset/images/volume14/figures/fig-c13p7.png b/processed_dataset/images/volume14/figures/fig-c13p7.png new file mode 100644 index 0000000000000000000000000000000000000000..ad759a44b596a0d98958b471fbae371d411ab4dd --- /dev/null +++ b/processed_dataset/images/volume14/figures/fig-c13p7.png @@ -0,0 +1,3 @@ +version https://git-lfs.github.com/spec/v1 +oid sha256:714e580ffdd4c1192460aa69bd5529557ea597052a45904fd8a5058d9374c274 +size 49165 diff --git a/processed_dataset/images/volume14/figures/fig-c14a2.png b/processed_dataset/images/volume14/figures/fig-c14a2.png new file mode 100644 index 0000000000000000000000000000000000000000..ddb9ec961ba36b52f83b8bb2792faaed651f3fe7 --- /dev/null +++ b/processed_dataset/images/volume14/figures/fig-c14a2.png @@ -0,0 +1,3 @@ +version https://git-lfs.github.com/spec/v1 +oid sha256:d211dcbbf6cac0a87069488977e676d136805acd6c214e7753aa6dee58646afd +size 47999 diff --git a/processed_dataset/images/volume14/figures/fig-c14a3.png b/processed_dataset/images/volume14/figures/fig-c14a3.png new file mode 100644 index 0000000000000000000000000000000000000000..4dd01748d9af9c37d5c9c76104f151801ad5da88 --- /dev/null +++ b/processed_dataset/images/volume14/figures/fig-c14a3.png @@ -0,0 +1,3 @@ +version https://git-lfs.github.com/spec/v1 +oid sha256:8a22d10227d7449e07014a1028ac3264ae512c4fbbfb97d7c22ccd2bd4f89eb4 +size 33178 diff --git a/processed_dataset/images/volume14/figures/fig-c14a4.png b/processed_dataset/images/volume14/figures/fig-c14a4.png new file mode 100644 index 0000000000000000000000000000000000000000..0bfd351ec34902c8e63acfc67895ed4b6b41b020 --- /dev/null +++ b/processed_dataset/images/volume14/figures/fig-c14a4.png @@ -0,0 +1,3 @@ +version https://git-lfs.github.com/spec/v1 +oid sha256:fe733af0a3a3ed16fe4bde02226bd5eafe01fec4609d7c744afa3bc1b91ad7dc +size 33466 diff --git a/processed_dataset/images/volume14/figures/fig-c14a5.png b/processed_dataset/images/volume14/figures/fig-c14a5.png new file mode 100644 index 0000000000000000000000000000000000000000..f341f0a784c06b4c4de37513894b5f155416aba3 --- /dev/null +++ b/processed_dataset/images/volume14/figures/fig-c14a5.png @@ -0,0 +1,3 @@ +version https://git-lfs.github.com/spec/v1 +oid sha256:d6391a32d031eab5704faa13d66d45e231671304a6f0f9a3a03758d3baae40d0 +size 20590 diff --git a/processed_dataset/images/volume14/figures/fig-c14a8-1.png b/processed_dataset/images/volume14/figures/fig-c14a8-1.png new file mode 100644 index 0000000000000000000000000000000000000000..43701d2c46fc1adbea220cd74107d055ffde7ed9 --- /dev/null +++ b/processed_dataset/images/volume14/figures/fig-c14a8-1.png @@ -0,0 +1,3 @@ +version https://git-lfs.github.com/spec/v1 +oid sha256:cd208613005ca557cff3970e7412af802c243a22c31eac9b81dcb68412aa81b4 +size 49022 diff --git a/processed_dataset/images/volume14/figures/fig-c14a8-2.png b/processed_dataset/images/volume14/figures/fig-c14a8-2.png new file mode 100644 index 0000000000000000000000000000000000000000..18356f1e48429fe84cf79825c03eb72b07354664 --- /dev/null +++ b/processed_dataset/images/volume14/figures/fig-c14a8-2.png @@ -0,0 +1,3 @@ +version https://git-lfs.github.com/spec/v1 +oid sha256:1271a45952914629c9e38c87a258a2aa7a5c952c34bbed9ac598d00d328bc0c6 +size 50039 diff --git a/processed_dataset/images/volume14/figures/fig-c14a9.png b/processed_dataset/images/volume14/figures/fig-c14a9.png new file mode 100644 index 0000000000000000000000000000000000000000..2eab70493147ea30847820813e0499a0f7f8e73e --- /dev/null +++ b/processed_dataset/images/volume14/figures/fig-c14a9.png @@ -0,0 +1,3 @@ +version https://git-lfs.github.com/spec/v1 +oid sha256:7614fcc3bb568131e7171c5dcc288c88aa1597b880783c36e67cc447ba75b209 +size 88215 diff --git a/processed_dataset/images/volume14/figures/fig-c14e1.png b/processed_dataset/images/volume14/figures/fig-c14e1.png new file mode 100644 index 0000000000000000000000000000000000000000..915c2bd83ec7c3a0ca31576c7bc96c02c397fefd --- /dev/null +++ b/processed_dataset/images/volume14/figures/fig-c14e1.png @@ -0,0 +1,3 @@ +version https://git-lfs.github.com/spec/v1 +oid sha256:91b97cc848714d8b732a4e5beb92cd065afe82cd1899edaa6c4eb3764683979f +size 6274 diff --git a/processed_dataset/images/volume14/figures/fig-c14e2.png b/processed_dataset/images/volume14/figures/fig-c14e2.png new file mode 100644 index 0000000000000000000000000000000000000000..2ecb6ae62c27a7f61046d1c253fe1665278676e5 --- /dev/null +++ b/processed_dataset/images/volume14/figures/fig-c14e2.png @@ -0,0 +1,3 @@ +version https://git-lfs.github.com/spec/v1 +oid sha256:940b32b70e1db1b7a32d6bfac1ab307704981f6c5d15cca7f2cd84c8334907b6 +size 6491 diff --git a/processed_dataset/images/volume14/figures/fig-c14e3.png b/processed_dataset/images/volume14/figures/fig-c14e3.png new file mode 100644 index 0000000000000000000000000000000000000000..b386b124a71019522fa58060595495a49dd2507c --- /dev/null +++ b/processed_dataset/images/volume14/figures/fig-c14e3.png @@ -0,0 +1,3 @@ +version https://git-lfs.github.com/spec/v1 +oid sha256:abede94257e843e33d2e763b80683eb180aeadec3212586158d3fd476302e792 +size 6503 diff --git a/processed_dataset/images/volume14/figures/fig-c14i1.png b/processed_dataset/images/volume14/figures/fig-c14i1.png new file mode 100644 index 0000000000000000000000000000000000000000..f4d3302759b938b8d2d29518eeb91ca3e0bfaa3e --- /dev/null +++ b/processed_dataset/images/volume14/figures/fig-c14i1.png @@ -0,0 +1,3 @@ +version https://git-lfs.github.com/spec/v1 +oid sha256:a5d80a7b145745913b78da516a1bd84657565363c5400ae0fb2677a606b7c58f +size 49543 diff --git a/processed_dataset/images/volume14/figures/fig-c14i2.png b/processed_dataset/images/volume14/figures/fig-c14i2.png new file mode 100644 index 0000000000000000000000000000000000000000..a9b62fcbc2a88519bc76d55ce37bac470e851d3f --- /dev/null +++ b/processed_dataset/images/volume14/figures/fig-c14i2.png @@ -0,0 +1,3 @@ +version https://git-lfs.github.com/spec/v1 +oid sha256:a97520a1ac86a99b6b7c9e608616690b59c49648ae56b3ddef32823763ac38ab +size 42484 diff --git a/processed_dataset/images/volume14/figures/fig-c14i3.png b/processed_dataset/images/volume14/figures/fig-c14i3.png new file mode 100644 index 0000000000000000000000000000000000000000..27d4fefecb4243c7c0d1734a070d0d988e6d3d2e --- /dev/null +++ b/processed_dataset/images/volume14/figures/fig-c14i3.png @@ -0,0 +1,3 @@ +version https://git-lfs.github.com/spec/v1 +oid sha256:559a406b267b9c9b3e6d3edb1ea470d722074017f28ec268073da30cd4be65a2 +size 41315 diff --git a/processed_dataset/images/volume14/figures/fig-c14i4.png b/processed_dataset/images/volume14/figures/fig-c14i4.png new file mode 100644 index 0000000000000000000000000000000000000000..2f8a8b2c4302b4252f4e9f656875fdb720e42fbc --- /dev/null +++ b/processed_dataset/images/volume14/figures/fig-c14i4.png @@ -0,0 +1,3 @@ +version https://git-lfs.github.com/spec/v1 +oid sha256:c23563cc766f21f45b037d48fd46a3cc6352d28c9512322869adb22fd9edd5ae +size 60124 diff --git a/processed_dataset/images/volume14/figures/fig-c14i5.png b/processed_dataset/images/volume14/figures/fig-c14i5.png new file mode 100644 index 0000000000000000000000000000000000000000..6fce1a7b15d10c247d28900cfd499eb57370cfa0 --- /dev/null +++ b/processed_dataset/images/volume14/figures/fig-c14i5.png @@ -0,0 +1,3 @@ +version https://git-lfs.github.com/spec/v1 +oid sha256:c010416adc37c227bc466727d2c8865a3de7ab432c3fbaba60d5c3dd54a55fb5 +size 25245 diff --git a/processed_dataset/images/volume14/figures/fig-c14i6.png b/processed_dataset/images/volume14/figures/fig-c14i6.png new file mode 100644 index 0000000000000000000000000000000000000000..40e2d314bda442ad1f194010e4b37b5cec93fef7 --- /dev/null +++ b/processed_dataset/images/volume14/figures/fig-c14i6.png @@ -0,0 +1,3 @@ +version https://git-lfs.github.com/spec/v1 +oid sha256:840166ae9af4d5fdf5c0311cc2c21aed311136535096f395233de39f010b0140 +size 12319 diff --git a/processed_dataset/images/volume14/figures/fig-c14i7.png b/processed_dataset/images/volume14/figures/fig-c14i7.png new file mode 100644 index 0000000000000000000000000000000000000000..33b36c46c79166648f6e615d47de3550d4259f09 --- /dev/null +++ b/processed_dataset/images/volume14/figures/fig-c14i7.png @@ -0,0 +1,3 @@ +version https://git-lfs.github.com/spec/v1 +oid sha256:518b56a769febdb82600e44d89571dc4098ebd525df095b17205ed1146993094 +size 15619 diff --git a/processed_dataset/images/volume14/figures/fig-c14i8.png b/processed_dataset/images/volume14/figures/fig-c14i8.png new file mode 100644 index 0000000000000000000000000000000000000000..f5c6328608b84217c3eece6352fc4af88ff9615b --- /dev/null +++ b/processed_dataset/images/volume14/figures/fig-c14i8.png @@ -0,0 +1,3 @@ +version https://git-lfs.github.com/spec/v1 +oid sha256:5cff5958234fd3ad113cf09f5be47796c37aff44d646ab2ef5e6d02ca0aafb8c +size 37905 diff --git a/processed_dataset/images/volume14/figures/fig-c14p2.png b/processed_dataset/images/volume14/figures/fig-c14p2.png new file mode 100644 index 0000000000000000000000000000000000000000..2e6736a56eacbd1b2e410ef1bb6e368ffd3cddc5 --- /dev/null +++ b/processed_dataset/images/volume14/figures/fig-c14p2.png @@ -0,0 +1,3 @@ +version https://git-lfs.github.com/spec/v1 +oid sha256:a2c778b2ed306ee574b8f058017f419926325bb60c88f854d894cb595ff5eef0 +size 24210 diff --git a/processed_dataset/images/volume14/figures/fig-c14p9.png b/processed_dataset/images/volume14/figures/fig-c14p9.png new file mode 100644 index 0000000000000000000000000000000000000000..0309a7fd4cdc6f5f424ef62890ee085a64d531c5 --- /dev/null +++ b/processed_dataset/images/volume14/figures/fig-c14p9.png @@ -0,0 +1,3 @@ +version https://git-lfs.github.com/spec/v1 +oid sha256:8201a55ba0006412078f51e6e9af40623c963ab6549c49dea336b8dd69bc58c8 +size 34837 diff --git a/processed_dataset/images/volume14/figures/fig-c15i1.png b/processed_dataset/images/volume14/figures/fig-c15i1.png new file mode 100644 index 0000000000000000000000000000000000000000..474c9023934fbc88cb61ea1c4d48603813d773e2 --- /dev/null +++ b/processed_dataset/images/volume14/figures/fig-c15i1.png @@ -0,0 +1,3 @@ +version https://git-lfs.github.com/spec/v1 +oid sha256:43515e3a3e5fc1de5df68504e7d6f559568dc2e7df0f257b050db4ad8f98d67d +size 12434 diff --git a/processed_dataset/images/volume14/figures/fig-c15i2.png b/processed_dataset/images/volume14/figures/fig-c15i2.png new file mode 100644 index 0000000000000000000000000000000000000000..aed17e238b27c2ce8b8491d0853f303751841a20 --- /dev/null +++ b/processed_dataset/images/volume14/figures/fig-c15i2.png @@ -0,0 +1,3 @@ +version https://git-lfs.github.com/spec/v1 +oid sha256:f11ee3efc4ed9a6abf5490017dca5ed526a2f34f8607cec96d4c37dd5bd21061 +size 36927 diff --git a/processed_dataset/images/volume14/figures/fig-c15i3.png b/processed_dataset/images/volume14/figures/fig-c15i3.png new file mode 100644 index 0000000000000000000000000000000000000000..5ddef9682dd83cee51aafed6c399ed32a16bc82e --- /dev/null +++ b/processed_dataset/images/volume14/figures/fig-c15i3.png @@ -0,0 +1,3 @@ +version https://git-lfs.github.com/spec/v1 +oid sha256:7d7940aa575caf56359249766c99ea4ce06e8990ac5c914dcde97069caeed6da +size 38479 diff --git a/processed_dataset/images/volume14/figures/fig-c15i4.png b/processed_dataset/images/volume14/figures/fig-c15i4.png new file mode 100644 index 0000000000000000000000000000000000000000..bb8e15ec75bff34de66eccc8452a7e446d764919 --- /dev/null +++ b/processed_dataset/images/volume14/figures/fig-c15i4.png @@ -0,0 +1,3 @@ +version https://git-lfs.github.com/spec/v1 +oid sha256:61c8ee966a915b56e7fafe7515c8522f85f6498e1b2f9d576f03c673410a5667 +size 35636 diff --git a/processed_dataset/images/volume14/figures/fig-c15i5.png b/processed_dataset/images/volume14/figures/fig-c15i5.png new file mode 100644 index 0000000000000000000000000000000000000000..b70e75972544a79e7ebdcc90a06beef6864ea41f --- /dev/null +++ b/processed_dataset/images/volume14/figures/fig-c15i5.png @@ -0,0 +1,3 @@ +version https://git-lfs.github.com/spec/v1 +oid sha256:303c576b83d2f07ee709bee7cda19d7fa120c368d853b295ce1c8d00ac6c6197 +size 18183 diff --git a/processed_dataset/images/volume14/figures/fig-c15p4.png b/processed_dataset/images/volume14/figures/fig-c15p4.png new file mode 100644 index 0000000000000000000000000000000000000000..ae6c40af348f22b2b4a30a891a8d9fa29be7c01e --- /dev/null +++ b/processed_dataset/images/volume14/figures/fig-c15p4.png @@ -0,0 +1,3 @@ +version https://git-lfs.github.com/spec/v1 +oid sha256:826f2fa055b59afc10f4eded2356590e9212fc0ed5a85d395ada2e28ff5c33a4 +size 24412 diff --git a/processed_dataset/images/volume14/figures/fig-c15p5.png b/processed_dataset/images/volume14/figures/fig-c15p5.png new file mode 100644 index 0000000000000000000000000000000000000000..82f128ace7be8629158c1d3d7229b91211748991 --- /dev/null +++ b/processed_dataset/images/volume14/figures/fig-c15p5.png @@ -0,0 +1,3 @@ +version https://git-lfs.github.com/spec/v1 +oid sha256:df318c0c55526c1b09b770a5a60af44648d949b85e56b68ed7764ba1f48baeea +size 28656 diff --git a/processed_dataset/images/volume14/figures/fig-c18a10-1.png b/processed_dataset/images/volume14/figures/fig-c18a10-1.png new file mode 100644 index 0000000000000000000000000000000000000000..3d5fa1866b2ecd08884c67c6392ee207f189f219 --- /dev/null +++ b/processed_dataset/images/volume14/figures/fig-c18a10-1.png @@ -0,0 +1,3 @@ +version https://git-lfs.github.com/spec/v1 +oid sha256:620ffa0e66747c0e8aacbb6c6cacc5c25d7bfc00b6f210be97a49b98aa1c7e53 +size 13812 diff --git a/processed_dataset/images/volume14/figures/fig-c18a10-2.png b/processed_dataset/images/volume14/figures/fig-c18a10-2.png new file mode 100644 index 0000000000000000000000000000000000000000..d97f9b5f9c15653a22d645a1d18f284565edc043 --- /dev/null +++ b/processed_dataset/images/volume14/figures/fig-c18a10-2.png @@ -0,0 +1,3 @@ +version https://git-lfs.github.com/spec/v1 +oid sha256:070432aa8c7eb22bb657ae4b1791bd453aa6763856156fb6f1d0f99f7fb25fe2 +size 51746 diff --git a/processed_dataset/images/volume14/figures/fig-c18a3.png b/processed_dataset/images/volume14/figures/fig-c18a3.png new file mode 100644 index 0000000000000000000000000000000000000000..bfa275e1b5b52a68997688edd5fdd75ad1ff13d0 --- /dev/null +++ b/processed_dataset/images/volume14/figures/fig-c18a3.png @@ -0,0 +1,3 @@ +version https://git-lfs.github.com/spec/v1 +oid sha256:02ac5f38adc2ee2fc1133ce20eb2be3ca65f84b29715832108ca0fdca91bfae4 +size 19290 diff --git a/processed_dataset/images/volume14/figures/fig-c18i1.png b/processed_dataset/images/volume14/figures/fig-c18i1.png new file mode 100644 index 0000000000000000000000000000000000000000..a25ac3178661ff04d578df3a07bf1f93cf143963 --- /dev/null +++ b/processed_dataset/images/volume14/figures/fig-c18i1.png @@ -0,0 +1,3 @@ +version https://git-lfs.github.com/spec/v1 +oid sha256:efbf7c7a35697ada8ae779c2d4a5ffa6bfe0e0122245b22a606abebbc28caaa0 +size 25114 diff --git a/processed_dataset/images/volume14/figures/fig-c18i2.png b/processed_dataset/images/volume14/figures/fig-c18i2.png new file mode 100644 index 0000000000000000000000000000000000000000..dad02004bb0920e84be3bd5fbd3f618093cd5242 --- /dev/null +++ b/processed_dataset/images/volume14/figures/fig-c18i2.png @@ -0,0 +1,3 @@ +version https://git-lfs.github.com/spec/v1 +oid sha256:97ebdd324298d9358644e93af93bbd91722b9cb57578d3018cfad69ca46f9538 +size 26926 diff --git a/processed_dataset/images/volume14/figures/fig-c18p10.png b/processed_dataset/images/volume14/figures/fig-c18p10.png new file mode 100644 index 0000000000000000000000000000000000000000..1df6949fbcc13cce4e0bab597cb7c7b5bab0ac9d --- /dev/null +++ b/processed_dataset/images/volume14/figures/fig-c18p10.png @@ -0,0 +1,3 @@ +version https://git-lfs.github.com/spec/v1 +oid sha256:2e52f28615f6bee1331df8ba8b9e44b1aebf2f1da0d466276b9f304d9be9a4cc +size 29652 diff --git a/processed_dataset/images/volume14/figures/fig-c19a3.png b/processed_dataset/images/volume14/figures/fig-c19a3.png new file mode 100644 index 0000000000000000000000000000000000000000..78d44bba6b32f49fbb0359292bcaca14b2588c7f --- /dev/null +++ b/processed_dataset/images/volume14/figures/fig-c19a3.png @@ -0,0 +1,3 @@ +version https://git-lfs.github.com/spec/v1 +oid sha256:4dd928536568443ff5248f6ea06c941c28ccc76438245f57467b351d76d524aa +size 16191 diff --git a/processed_dataset/images/volume14/figures/fig-c19a6.png b/processed_dataset/images/volume14/figures/fig-c19a6.png new file mode 100644 index 0000000000000000000000000000000000000000..5bd79245d8c49c85d81ada7c91f7e234b9101cdd --- /dev/null +++ b/processed_dataset/images/volume14/figures/fig-c19a6.png @@ -0,0 +1,3 @@ +version https://git-lfs.github.com/spec/v1 +oid sha256:6d765c07ab77836dae9dc72d6701bfc76c9fac6a9ca7158f918c45bbcec49dd7 +size 14333 diff --git a/processed_dataset/images/volume14/figures/fig-c19i1.png b/processed_dataset/images/volume14/figures/fig-c19i1.png new file mode 100644 index 0000000000000000000000000000000000000000..03e656e6c73fca6e94ce73181b7a400b9cd329dd --- /dev/null +++ b/processed_dataset/images/volume14/figures/fig-c19i1.png @@ -0,0 +1,3 @@ +version https://git-lfs.github.com/spec/v1 +oid sha256:4fe7d4197dccc0cd1aa614e9d6ae74b2c02a9809565f3c9840d06815244803c2 +size 27314 diff --git a/processed_dataset/images/volume14/figures/fig-c1i1.png b/processed_dataset/images/volume14/figures/fig-c1i1.png new file mode 100644 index 0000000000000000000000000000000000000000..58ffd0c999f6e7f030cca199a0370c1e56691be3 --- /dev/null +++ b/processed_dataset/images/volume14/figures/fig-c1i1.png @@ -0,0 +1,3 @@ +version https://git-lfs.github.com/spec/v1 +oid sha256:6d9fa18694ae5782e519943621ea3d7eaa67787f917c0841c2e7f8a638224506 +size 22528 diff --git a/processed_dataset/images/volume14/figures/fig-c1i2.png b/processed_dataset/images/volume14/figures/fig-c1i2.png new file mode 100644 index 0000000000000000000000000000000000000000..5bbe8cdbc6cbc8e4e21f18742c1728aa14b8fbda --- /dev/null +++ b/processed_dataset/images/volume14/figures/fig-c1i2.png @@ -0,0 +1,3 @@ +version https://git-lfs.github.com/spec/v1 +oid sha256:939e37cf36fc0a97fa376566dec567530a6279c21524f8ae07466a2513c0d610 +size 22584 diff --git a/processed_dataset/images/volume14/figures/fig-c1i3.png b/processed_dataset/images/volume14/figures/fig-c1i3.png new file mode 100644 index 0000000000000000000000000000000000000000..32ddb03f73b114f6706e26747f6edddc68dce601 --- /dev/null +++ b/processed_dataset/images/volume14/figures/fig-c1i3.png @@ -0,0 +1,3 @@ +version https://git-lfs.github.com/spec/v1 +oid sha256:0ae7be3f6e4677e049a065b1f9769bd3628641ed91e2753b0863acf2d6436505 +size 37858 diff --git a/processed_dataset/images/volume14/figures/fig-c20i1.png b/processed_dataset/images/volume14/figures/fig-c20i1.png new file mode 100644 index 0000000000000000000000000000000000000000..ff8d465ef4f8a9a12674a8093d6dd0e41e9c3751 --- /dev/null +++ b/processed_dataset/images/volume14/figures/fig-c20i1.png @@ -0,0 +1,3 @@ +version https://git-lfs.github.com/spec/v1 +oid sha256:70205531a62560444e534b32f12ea7cb4aab3077f5b0ac3d8fbc52c069b442d7 +size 28812 diff --git a/processed_dataset/images/volume14/figures/fig-c20i2.png b/processed_dataset/images/volume14/figures/fig-c20i2.png new file mode 100644 index 0000000000000000000000000000000000000000..094172b8e1043bfe881b67346ccba525a390bfa5 --- /dev/null +++ b/processed_dataset/images/volume14/figures/fig-c20i2.png @@ -0,0 +1,3 @@ +version https://git-lfs.github.com/spec/v1 +oid sha256:664f3e61feb8477004d0656608a091950e4bb0bc3f16f21759721095d038f4e6 +size 23099 diff --git a/processed_dataset/images/volume14/figures/fig-c2i1.png b/processed_dataset/images/volume14/figures/fig-c2i1.png new file mode 100644 index 0000000000000000000000000000000000000000..b204824c310b4385440f55f01c215a9f3c35cc61 --- /dev/null +++ b/processed_dataset/images/volume14/figures/fig-c2i1.png @@ -0,0 +1,3 @@ +version https://git-lfs.github.com/spec/v1 +oid sha256:af7a01cff7970dbbe8ef4cd1a35c0c4da53c38dd38070439a0a7457ac4c90ac1 +size 36099 diff --git a/processed_dataset/images/volume14/figures/fig-c2i2.png b/processed_dataset/images/volume14/figures/fig-c2i2.png new file mode 100644 index 0000000000000000000000000000000000000000..60660f4bfd4d666392a7c2177d2751bf41679431 --- /dev/null +++ b/processed_dataset/images/volume14/figures/fig-c2i2.png @@ -0,0 +1,3 @@ +version https://git-lfs.github.com/spec/v1 +oid sha256:5c8541edd67dc635c063cdb7464d841316b688cd4b2ec52d1a43e29b1faffc2f +size 52813 diff --git a/processed_dataset/images/volume14/figures/fig-c3a3.png b/processed_dataset/images/volume14/figures/fig-c3a3.png new file mode 100644 index 0000000000000000000000000000000000000000..f94cc6c688c33197920c8b075e14460ba87e682c --- /dev/null +++ b/processed_dataset/images/volume14/figures/fig-c3a3.png @@ -0,0 +1,3 @@ +version https://git-lfs.github.com/spec/v1 +oid sha256:d8e0d8a16c0103914a8bad6ef0e6c6b3b4bd38eb69f949ceaaa3bdff847a0844 +size 38277 diff --git a/processed_dataset/images/volume14/figures/fig-c4a4.png b/processed_dataset/images/volume14/figures/fig-c4a4.png new file mode 100644 index 0000000000000000000000000000000000000000..71e14f7e6986c123d2c5370bae224fdf398bff92 --- /dev/null +++ b/processed_dataset/images/volume14/figures/fig-c4a4.png @@ -0,0 +1,3 @@ +version https://git-lfs.github.com/spec/v1 +oid sha256:05936bf4411df3b6a6bc59dbb459bd6c74f35d2937f67c8def5268b58bad1a10 +size 19083 diff --git a/processed_dataset/images/volume14/figures/fig-c4i1.png b/processed_dataset/images/volume14/figures/fig-c4i1.png new file mode 100644 index 0000000000000000000000000000000000000000..a3c092617bad07792847edf0af0bd21d0a9eebaa --- /dev/null +++ b/processed_dataset/images/volume14/figures/fig-c4i1.png @@ -0,0 +1,3 @@ +version https://git-lfs.github.com/spec/v1 +oid sha256:d90e642ec29b1dbe42c1e0133b381d02558b37a561d8b7b07a6381e3620259e0 +size 21472 diff --git a/processed_dataset/images/volume14/figures/fig-c4i2.png b/processed_dataset/images/volume14/figures/fig-c4i2.png new file mode 100644 index 0000000000000000000000000000000000000000..9de2b479322c3f1eeff83f8ca563f5f2385c0181 --- /dev/null +++ b/processed_dataset/images/volume14/figures/fig-c4i2.png @@ -0,0 +1,3 @@ +version https://git-lfs.github.com/spec/v1 +oid sha256:a9d8bd3f26773834d58b2453fbd95bf599d6b05d4f816223181f3e95ecb8cd2f +size 22797 diff --git a/processed_dataset/images/volume14/figures/fig-c6a2.png b/processed_dataset/images/volume14/figures/fig-c6a2.png new file mode 100644 index 0000000000000000000000000000000000000000..5dd401dbda53ff8971320f33311b5e9074588ef5 --- /dev/null +++ b/processed_dataset/images/volume14/figures/fig-c6a2.png @@ -0,0 +1,3 @@ +version https://git-lfs.github.com/spec/v1 +oid sha256:a722564ac513dc011387daa948d84c38cbdeb05cc0639e5c8820e2079c82d920 +size 20168 diff --git a/processed_dataset/images/volume14/figures/fig-c6i1.png b/processed_dataset/images/volume14/figures/fig-c6i1.png new file mode 100644 index 0000000000000000000000000000000000000000..dc646898bf08e31822a7c58f7f8e89e99a9808fc --- /dev/null +++ b/processed_dataset/images/volume14/figures/fig-c6i1.png @@ -0,0 +1,3 @@ +version https://git-lfs.github.com/spec/v1 +oid sha256:5ed2a187d090559cef669e93ccde7587ad30891fbe4ad1c11a2af8c40ad507e2 +size 30708 diff --git a/processed_dataset/images/volume14/figures/fig-c6i2.png b/processed_dataset/images/volume14/figures/fig-c6i2.png new file mode 100644 index 0000000000000000000000000000000000000000..ce29e35f6f146ba0bbeb4fefdea643ffeded7eeb --- /dev/null +++ b/processed_dataset/images/volume14/figures/fig-c6i2.png @@ -0,0 +1,3 @@ +version https://git-lfs.github.com/spec/v1 +oid sha256:ed4a4c31d10d06ed031be244766b7becd2e52ed150b0b6fc6465d6c954c3b007 +size 23408 diff --git a/processed_dataset/images/volume14/figures/fig-c7i1.png b/processed_dataset/images/volume14/figures/fig-c7i1.png new file mode 100644 index 0000000000000000000000000000000000000000..b7a2fc8f7131c6f03c9238aeb5ba520ad5ac3510 --- /dev/null +++ b/processed_dataset/images/volume14/figures/fig-c7i1.png @@ -0,0 +1,3 @@ +version https://git-lfs.github.com/spec/v1 +oid sha256:3c8068cffaa907cc9ee863924f8d92b3f883bde8e488efcc709d04185bec585c +size 24183 diff --git a/processed_dataset/images/volume14/figures/fig-c8a2.png b/processed_dataset/images/volume14/figures/fig-c8a2.png new file mode 100644 index 0000000000000000000000000000000000000000..ee4e7b147d658d34b675f1d1f805ba5d62555b1a --- /dev/null +++ b/processed_dataset/images/volume14/figures/fig-c8a2.png @@ -0,0 +1,3 @@ +version https://git-lfs.github.com/spec/v1 +oid sha256:697c7a8f3452139909cc7d58a71493605d32fd92f85303e64d7fe4ab9498f3ad +size 24248 diff --git a/processed_dataset/images/volume14/figures/fig-c8a3.png b/processed_dataset/images/volume14/figures/fig-c8a3.png new file mode 100644 index 0000000000000000000000000000000000000000..2224559d7e24074e7e11487d1abaac13e9ee45b5 --- /dev/null +++ b/processed_dataset/images/volume14/figures/fig-c8a3.png @@ -0,0 +1,3 @@ +version https://git-lfs.github.com/spec/v1 +oid sha256:16f7bba776deabcc5b45b2df7d6a972daecd8196deca59fd477f42d64b1a1f17 +size 27273 diff --git a/processed_dataset/images/volume14/figures/fig-c8a4.png b/processed_dataset/images/volume14/figures/fig-c8a4.png new file mode 100644 index 0000000000000000000000000000000000000000..5d42d24ec054bb66f8063219824c040de50d3fdb --- /dev/null +++ b/processed_dataset/images/volume14/figures/fig-c8a4.png @@ -0,0 +1,3 @@ +version https://git-lfs.github.com/spec/v1 +oid sha256:e7d333121355fa4c96ae534bd9317fcebcb18347f75601d10f1b329f3af8b46a +size 25297 diff --git a/processed_dataset/images/volume14/figures/fig-c8a5.png b/processed_dataset/images/volume14/figures/fig-c8a5.png new file mode 100644 index 0000000000000000000000000000000000000000..1284a3bca33433ddbfa93d8fd0fc90ff38e01fe2 --- /dev/null +++ b/processed_dataset/images/volume14/figures/fig-c8a5.png @@ -0,0 +1,3 @@ +version https://git-lfs.github.com/spec/v1 +oid sha256:af2522608254421d26de354189ec74bf86fe4df06e04886191d19005a49a9961 +size 38241 diff --git a/processed_dataset/images/volume14/figures/fig-c8a7.png b/processed_dataset/images/volume14/figures/fig-c8a7.png new file mode 100644 index 0000000000000000000000000000000000000000..2fd68594bb885549013e3dcb27750944748aaf67 --- /dev/null +++ b/processed_dataset/images/volume14/figures/fig-c8a7.png @@ -0,0 +1,3 @@ +version https://git-lfs.github.com/spec/v1 +oid sha256:40d3d0a9fc2a302ea1bf9aef407fbca02008310759f32ec622621370483a984b +size 31516 diff --git a/processed_dataset/images/volume14/figures/fig-c8a8.png b/processed_dataset/images/volume14/figures/fig-c8a8.png new file mode 100644 index 0000000000000000000000000000000000000000..62f366d2fb56f0e6cb30e4650ad0d3428ec62ddb --- /dev/null +++ b/processed_dataset/images/volume14/figures/fig-c8a8.png @@ -0,0 +1,3 @@ +version https://git-lfs.github.com/spec/v1 +oid sha256:9520d8a485a2ce38dcbac2674c114a8bca5508f61b275b220a5eb237521cb60f +size 46838 diff --git a/processed_dataset/images/volume14/figures/fig-c8i1.png b/processed_dataset/images/volume14/figures/fig-c8i1.png new file mode 100644 index 0000000000000000000000000000000000000000..eddace1929afd7d129c5f0aa6305b001a6bcf592 --- /dev/null +++ b/processed_dataset/images/volume14/figures/fig-c8i1.png @@ -0,0 +1,3 @@ +version https://git-lfs.github.com/spec/v1 +oid sha256:2fcaf28880037e06024c767cb0a898d9ab2a93d8349d8e989354127db5a7c413 +size 27116 diff --git a/processed_dataset/images/volume14/figures/fig-c8i2.png b/processed_dataset/images/volume14/figures/fig-c8i2.png new file mode 100644 index 0000000000000000000000000000000000000000..a9fffeefe303e5feb96007b62b5b30c4ecb231ef --- /dev/null +++ b/processed_dataset/images/volume14/figures/fig-c8i2.png @@ -0,0 +1,3 @@ +version https://git-lfs.github.com/spec/v1 +oid sha256:8432a5b8c42d42784a01cd66b19f124f17526a480bb4406e82de3df847b1ab6c +size 18865 diff --git a/processed_dataset/images/volume14/figures/fig-c8i3.png b/processed_dataset/images/volume14/figures/fig-c8i3.png new file mode 100644 index 0000000000000000000000000000000000000000..f4486ecfb0f31b8d66f87522bf268244f9acc569 --- /dev/null +++ b/processed_dataset/images/volume14/figures/fig-c8i3.png @@ -0,0 +1,3 @@ +version https://git-lfs.github.com/spec/v1 +oid sha256:642a78163cf9bfe3613d26777c37266bf3d0a409f49fab7bfe5eeef9ce19774e +size 29868 diff --git a/processed_dataset/images/volume14/figures/fig-c8i4.png b/processed_dataset/images/volume14/figures/fig-c8i4.png new file mode 100644 index 0000000000000000000000000000000000000000..6ddd796ad1d128155fe8d6b1bb21d6eb27818dc5 --- /dev/null +++ b/processed_dataset/images/volume14/figures/fig-c8i4.png @@ -0,0 +1,3 @@ +version https://git-lfs.github.com/spec/v1 +oid sha256:642726850582cd6dd845b369c6c774c26c47343f66ba9cdb48ef014c2eba6319 +size 32735 diff --git a/processed_dataset/images/volume14/figures/fig-c8i5.png b/processed_dataset/images/volume14/figures/fig-c8i5.png new file mode 100644 index 0000000000000000000000000000000000000000..7bcdb8f6095b6243e7637fd3f6f30843902a5f09 --- /dev/null +++ b/processed_dataset/images/volume14/figures/fig-c8i5.png @@ -0,0 +1,3 @@ +version https://git-lfs.github.com/spec/v1 +oid sha256:737ddff13d809a4aa5cb94955a616588e73b617812051f8b990bfefe3adbc957 +size 36287 diff --git a/processed_dataset/images/volume14/figures/fig-c8i6.png b/processed_dataset/images/volume14/figures/fig-c8i6.png new file mode 100644 index 0000000000000000000000000000000000000000..bd600796a4b7bb0fb715fe9a6071bd5eaa187b9b --- /dev/null +++ b/processed_dataset/images/volume14/figures/fig-c8i6.png @@ -0,0 +1,3 @@ +version https://git-lfs.github.com/spec/v1 +oid sha256:a9d75bf516bb24b46801bfcf62f3771d19b2dc817a14b5062fe2bcf9e2698671 +size 22389 diff --git a/processed_dataset/images/volume14/figures/fig-c8p3.png b/processed_dataset/images/volume14/figures/fig-c8p3.png new file mode 100644 index 0000000000000000000000000000000000000000..19403ff4a5e8dcb6691176c3fc24b73606b60fb2 --- /dev/null +++ b/processed_dataset/images/volume14/figures/fig-c8p3.png @@ -0,0 +1,3 @@ +version https://git-lfs.github.com/spec/v1 +oid sha256:d8b90e62790359c925e8bde8143f40d21c4e05036d26243a0d59f30d6fedff6b +size 26267 diff --git a/processed_dataset/images/volume14/figures/fig-c8p7.png b/processed_dataset/images/volume14/figures/fig-c8p7.png new file mode 100644 index 0000000000000000000000000000000000000000..d719097d47ad61bfe0c7a6ad71943f5fdb36358d --- /dev/null +++ b/processed_dataset/images/volume14/figures/fig-c8p7.png @@ -0,0 +1,3 @@ +version https://git-lfs.github.com/spec/v1 +oid sha256:4ce05d155003ecc51ffd341a98b089a21c62a6ffb4aeb37ef6058e0d8ffb9143 +size 28223 diff --git a/processed_dataset/images/volume14/figures/fig-c9a6.png b/processed_dataset/images/volume14/figures/fig-c9a6.png new file mode 100644 index 0000000000000000000000000000000000000000..df5b0bb27b99524cc12a521d22b99e742dde0ac8 --- /dev/null +++ b/processed_dataset/images/volume14/figures/fig-c9a6.png @@ -0,0 +1,3 @@ +version https://git-lfs.github.com/spec/v1 +oid sha256:7b4b40adee72ee560c9aebe67723ac8ba31c389149f85ba4a74a45ae2bd6ffd6 +size 101944 diff --git a/processed_dataset/images/volume2/figures/fig-c1e13.png b/processed_dataset/images/volume2/figures/fig-c1e13.png new file mode 100644 index 0000000000000000000000000000000000000000..da1ecb3586c5e34f06e33efab547e37eb6ed3378 --- /dev/null +++ b/processed_dataset/images/volume2/figures/fig-c1e13.png @@ -0,0 +1,3 @@ +version https://git-lfs.github.com/spec/v1 +oid sha256:dcc9b3ea1412c6c1c51cd3b5be686e7595e36fd11a49323b883ba65a8cee76e0 +size 23116 diff --git a/processed_dataset/images/volume2/figures/fig-c1e15-1.png b/processed_dataset/images/volume2/figures/fig-c1e15-1.png new file mode 100644 index 0000000000000000000000000000000000000000..c8e2b49a8ce1a768b543a32a98aae43da63a27d7 --- /dev/null +++ b/processed_dataset/images/volume2/figures/fig-c1e15-1.png @@ -0,0 +1,3 @@ +version https://git-lfs.github.com/spec/v1 +oid sha256:94a55c9355a17bb62f450365900e89655df4021e37fac2914b205d4f5b5ed1ff +size 13961 diff --git a/processed_dataset/images/volume2/figures/fig-c1e15-2.png b/processed_dataset/images/volume2/figures/fig-c1e15-2.png new file mode 100644 index 0000000000000000000000000000000000000000..9ca02d8d1ce3ad6f0629effbf08c93b7bcfdf537 --- /dev/null +++ b/processed_dataset/images/volume2/figures/fig-c1e15-2.png @@ -0,0 +1,3 @@ +version https://git-lfs.github.com/spec/v1 +oid sha256:de65ec529f08abf25883914678dc13338d8e1cde8d5a4c2bd1381e5b5c6f7d4f +size 16053 diff --git a/processed_dataset/images/volume2/figures/fig-c1e15-3.png b/processed_dataset/images/volume2/figures/fig-c1e15-3.png new file mode 100644 index 0000000000000000000000000000000000000000..95218c79723ec6e5cfdc024cb362c054ea6ff096 --- /dev/null +++ b/processed_dataset/images/volume2/figures/fig-c1e15-3.png @@ -0,0 +1,3 @@ +version https://git-lfs.github.com/spec/v1 +oid sha256:57f9731e9e274942dc3e5d52f00bc4a67f60b4b4cc8e533d08a88bb6939e3bba +size 15971 diff --git a/processed_dataset/images/volume2/figures/fig-c2d2-1.png b/processed_dataset/images/volume2/figures/fig-c2d2-1.png new file mode 100644 index 0000000000000000000000000000000000000000..3067ac7fb22945e28b94d908ec4da81b0582daaf --- /dev/null +++ b/processed_dataset/images/volume2/figures/fig-c2d2-1.png @@ -0,0 +1,3 @@ +version https://git-lfs.github.com/spec/v1 +oid sha256:9416d9238cb819b4ed42822372e83e6867751ee302371bde34edb1c089acb30a +size 17537 diff --git a/processed_dataset/images/volume2/figures/fig-c2d2-2.png b/processed_dataset/images/volume2/figures/fig-c2d2-2.png new file mode 100644 index 0000000000000000000000000000000000000000..0bce55255b0ef85e18ef5d9cf185f7aafa4bc125 --- /dev/null +++ b/processed_dataset/images/volume2/figures/fig-c2d2-2.png @@ -0,0 +1,3 @@ +version https://git-lfs.github.com/spec/v1 +oid sha256:4aaaccbb294eceb06796f0ea6ddb53894bab5cc13bf20de2ff4ec94e5adbd745 +size 18085 diff --git a/processed_dataset/images/volume2/figures/fig-c2p8.png b/processed_dataset/images/volume2/figures/fig-c2p8.png new file mode 100644 index 0000000000000000000000000000000000000000..17d888828b496e75bc52aa64ae24a3a3cd64901e --- /dev/null +++ b/processed_dataset/images/volume2/figures/fig-c2p8.png @@ -0,0 +1,3 @@ +version https://git-lfs.github.com/spec/v1 +oid sha256:b5d8a91574fa5f5c2c84f4e4c2f0289bca9a6e6a1e75cc2ab15b31c4bd8ceb3b +size 26486 diff --git a/processed_dataset/images/volume2/figures/fig-c3d3.png b/processed_dataset/images/volume2/figures/fig-c3d3.png new file mode 100644 index 0000000000000000000000000000000000000000..deac45fc76802d78d9c722da0c732f43aad3d2da --- /dev/null +++ b/processed_dataset/images/volume2/figures/fig-c3d3.png @@ -0,0 +1,3 @@ +version https://git-lfs.github.com/spec/v1 +oid sha256:3f1488141ec4a6e0d2834f568985eb278954a2ed858d109f1c1255595938dc98 +size 19406 diff --git a/processed_dataset/images/volume2/figures/fig-c3p6.png b/processed_dataset/images/volume2/figures/fig-c3p6.png new file mode 100644 index 0000000000000000000000000000000000000000..3b4e954d3651696966979f13c9c6f18347d38515 --- /dev/null +++ b/processed_dataset/images/volume2/figures/fig-c3p6.png @@ -0,0 +1,3 @@ +version https://git-lfs.github.com/spec/v1 +oid sha256:4940e53e31a26407020570c21cfd19c1e9cbeaedc1ec3026d285ec21724aedbf +size 27081 diff --git a/processed_dataset/images/volume2/figures/fig-c3p9.png b/processed_dataset/images/volume2/figures/fig-c3p9.png new file mode 100644 index 0000000000000000000000000000000000000000..3492222d3e2ca00ebff152ca588afe69662fc921 --- /dev/null +++ b/processed_dataset/images/volume2/figures/fig-c3p9.png @@ -0,0 +1,3 @@ +version https://git-lfs.github.com/spec/v1 +oid sha256:cad901c85369523fb556aa7414b581d95b89f7094f8a08e275554fae6c2d4c37 +size 23302 diff --git a/processed_dataset/images/volume2/figures/fig-c4e14.png b/processed_dataset/images/volume2/figures/fig-c4e14.png new file mode 100644 index 0000000000000000000000000000000000000000..f4fcb81e383263a7a5f714e88361860d586f3be5 --- /dev/null +++ b/processed_dataset/images/volume2/figures/fig-c4e14.png @@ -0,0 +1,3 @@ +version https://git-lfs.github.com/spec/v1 +oid sha256:72ba4854f5e9a1e6edbc1fbcfe82d13ce41f48695ec4b2b06af537aaeffc3f5c +size 19861 diff --git a/processed_dataset/images/volume2/figures/fig-c4e15.png b/processed_dataset/images/volume2/figures/fig-c4e15.png new file mode 100644 index 0000000000000000000000000000000000000000..da42a22577938c8650134dd666e9175d86f16748 --- /dev/null +++ b/processed_dataset/images/volume2/figures/fig-c4e15.png @@ -0,0 +1,3 @@ +version https://git-lfs.github.com/spec/v1 +oid sha256:a64db4219afe38efc1966461cce5eca3cd7bbea644127cbf6f989f194db80578 +size 22781 diff --git a/processed_dataset/images/volume2/figures/fig-c4e16.png b/processed_dataset/images/volume2/figures/fig-c4e16.png new file mode 100644 index 0000000000000000000000000000000000000000..10575ee0285e06951e755f8a24f3d4ee92f062ef --- /dev/null +++ b/processed_dataset/images/volume2/figures/fig-c4e16.png @@ -0,0 +1,3 @@ +version https://git-lfs.github.com/spec/v1 +oid sha256:62465020287afda6397eb21ad95ce1e08b9a84e1e96eeb6b8e135b067b864f04 +size 36781 diff --git a/processed_dataset/images/volume2/figures/fig-c4e3-1.png b/processed_dataset/images/volume2/figures/fig-c4e3-1.png new file mode 100644 index 0000000000000000000000000000000000000000..303e9a3fddaf11346cb4ffcaf3cc134839f6a897 --- /dev/null +++ b/processed_dataset/images/volume2/figures/fig-c4e3-1.png @@ -0,0 +1,3 @@ +version https://git-lfs.github.com/spec/v1 +oid sha256:3cb3eaef7bb85e8b7388b37f5d5760c3f96cc3442b7bb031e26d94a17a55352d +size 17873 diff --git a/processed_dataset/images/volume2/figures/fig-c4e3-2.png b/processed_dataset/images/volume2/figures/fig-c4e3-2.png new file mode 100644 index 0000000000000000000000000000000000000000..984eba2b5dcc977c2c99bd9c358500f7c94e53dd --- /dev/null +++ b/processed_dataset/images/volume2/figures/fig-c4e3-2.png @@ -0,0 +1,3 @@ +version https://git-lfs.github.com/spec/v1 +oid sha256:f96aa884d429f16da0396ba7ca70d8af3c78e49107e78020d8c02fd73a0b7058 +size 17866 diff --git a/processed_dataset/images/volume2/figures/fig-c4p11.png b/processed_dataset/images/volume2/figures/fig-c4p11.png new file mode 100644 index 0000000000000000000000000000000000000000..417f15e6e94bed05ffecd4661199377dd6ea0cc2 --- /dev/null +++ b/processed_dataset/images/volume2/figures/fig-c4p11.png @@ -0,0 +1,3 @@ +version https://git-lfs.github.com/spec/v1 +oid sha256:c307d3e0aad907504da8ace75e48b6c98f451f95eae0c975aa04b62d60527194 +size 28632 diff --git a/processed_dataset/images/volume2/figures/fig-c6e23.png b/processed_dataset/images/volume2/figures/fig-c6e23.png new file mode 100644 index 0000000000000000000000000000000000000000..be672e25d82a9f8c563c6634980b1347cfe98772 --- /dev/null +++ b/processed_dataset/images/volume2/figures/fig-c6e23.png @@ -0,0 +1,3 @@ +version https://git-lfs.github.com/spec/v1 +oid sha256:2715b40f53abb3e65648c49b4115a837369e5aff549068899ef7faea8b6153e0 +size 33458 diff --git a/processed_dataset/images/volume2/figures/fig-c6p6.png b/processed_dataset/images/volume2/figures/fig-c6p6.png new file mode 100644 index 0000000000000000000000000000000000000000..86b143db67b259b42fedafc67b82e7d48bdb85bb --- /dev/null +++ b/processed_dataset/images/volume2/figures/fig-c6p6.png @@ -0,0 +1,3 @@ +version https://git-lfs.github.com/spec/v1 +oid sha256:ae38f01fa605e9c741ad293ebb9e21b4200dd5b30d9eef022011b0c01a863955 +size 20031 diff --git a/processed_dataset/images/volume3/figures/fig-c1e11-1.png b/processed_dataset/images/volume3/figures/fig-c1e11-1.png new file mode 100644 index 0000000000000000000000000000000000000000..1c45ab04ef311463f7d9eaf16b7ff0daf1e9d52b --- /dev/null +++ b/processed_dataset/images/volume3/figures/fig-c1e11-1.png @@ -0,0 +1,3 @@ +version https://git-lfs.github.com/spec/v1 +oid sha256:361d44baa3545ab3d1279fc3c505ba67322cc618841be22a2fbf563ee35bc522 +size 15717 diff --git a/processed_dataset/images/volume3/figures/fig-c1e11-2.png b/processed_dataset/images/volume3/figures/fig-c1e11-2.png new file mode 100644 index 0000000000000000000000000000000000000000..25a25d1ff8fc34505f08f3032358bfca5c05db83 --- /dev/null +++ b/processed_dataset/images/volume3/figures/fig-c1e11-2.png @@ -0,0 +1,3 @@ +version https://git-lfs.github.com/spec/v1 +oid sha256:baa2110550bc4a8cc3e872719e0edd3669c6610a94377342ebf7339947082b68 +size 17078 diff --git a/processed_dataset/images/volume3/figures/fig-c1e11-3.png b/processed_dataset/images/volume3/figures/fig-c1e11-3.png new file mode 100644 index 0000000000000000000000000000000000000000..0e5a04d17ccf813f1811c44c184c462593a4bad9 --- /dev/null +++ b/processed_dataset/images/volume3/figures/fig-c1e11-3.png @@ -0,0 +1,3 @@ +version https://git-lfs.github.com/spec/v1 +oid sha256:cf216444bda5f2c21abc1c3d73f1144bde0a6387223222d969cec24099d73854 +size 16092 diff --git a/processed_dataset/images/volume3/figures/fig-c1e2.png b/processed_dataset/images/volume3/figures/fig-c1e2.png new file mode 100644 index 0000000000000000000000000000000000000000..b35469ed2e7b328be304cb337b9b4aeff64f44e2 --- /dev/null +++ b/processed_dataset/images/volume3/figures/fig-c1e2.png @@ -0,0 +1,3 @@ +version https://git-lfs.github.com/spec/v1 +oid sha256:c0a7d471983b499e89053731f251329f1e09ce4992630bab667a05e7eae2a954 +size 25670 diff --git a/processed_dataset/images/volume3/figures/fig-c3e1-1.png b/processed_dataset/images/volume3/figures/fig-c3e1-1.png new file mode 100644 index 0000000000000000000000000000000000000000..6beef2f62c73ecc9f0a27c05c4d546a83b7c75b0 --- /dev/null +++ b/processed_dataset/images/volume3/figures/fig-c3e1-1.png @@ -0,0 +1,3 @@ +version https://git-lfs.github.com/spec/v1 +oid sha256:2814bd6b3e23569397bdfad8c487856cfa093c9e64f8fe8ab9ef4b24d2526932 +size 17592 diff --git a/processed_dataset/images/volume3/figures/fig-c3e1-2.png b/processed_dataset/images/volume3/figures/fig-c3e1-2.png new file mode 100644 index 0000000000000000000000000000000000000000..5ae2a1ec34ea26e1fcc35fbab9be788b27d635df --- /dev/null +++ b/processed_dataset/images/volume3/figures/fig-c3e1-2.png @@ -0,0 +1,3 @@ +version https://git-lfs.github.com/spec/v1 +oid sha256:28d5d541510292838ad389142c92e24096614c984606cb4ab2c743e6ba13d0c8 +size 18175 diff --git a/processed_dataset/images/volume3/figures/fig-c3e14-1.png b/processed_dataset/images/volume3/figures/fig-c3e14-1.png new file mode 100644 index 0000000000000000000000000000000000000000..dd9c845b022403ffab60046790ed08e6c346c1ee --- /dev/null +++ b/processed_dataset/images/volume3/figures/fig-c3e14-1.png @@ -0,0 +1,3 @@ +version https://git-lfs.github.com/spec/v1 +oid sha256:0cd359bd09d6a1d00902458835e25144b1409a3a0c83709092e046e85562474b +size 16611 diff --git a/processed_dataset/images/volume3/figures/fig-c3e14-2.png b/processed_dataset/images/volume3/figures/fig-c3e14-2.png new file mode 100644 index 0000000000000000000000000000000000000000..1028940ecb289e55099be881e6fd181489b1ccf9 --- /dev/null +++ b/processed_dataset/images/volume3/figures/fig-c3e14-2.png @@ -0,0 +1,3 @@ +version https://git-lfs.github.com/spec/v1 +oid sha256:339c9f4389566d78886b9ace5505fe667719b45fca04e9d2637640767df4fce8 +size 19763 diff --git a/processed_dataset/images/volume3/figures/fig-c3e17.png b/processed_dataset/images/volume3/figures/fig-c3e17.png new file mode 100644 index 0000000000000000000000000000000000000000..76c2ddbc85581debce8f5fdaaae475dc914a78ba --- /dev/null +++ b/processed_dataset/images/volume3/figures/fig-c3e17.png @@ -0,0 +1,3 @@ +version https://git-lfs.github.com/spec/v1 +oid sha256:0fedf8ce78926810fca4aa98a013f6c8c2340a397d569b80a30e008529e4423c +size 21490 diff --git a/processed_dataset/images/volume3/figures/fig-c3e19.png b/processed_dataset/images/volume3/figures/fig-c3e19.png new file mode 100644 index 0000000000000000000000000000000000000000..2e0e9ae5578ea2ba7daa7292490ef02761fa11a3 --- /dev/null +++ b/processed_dataset/images/volume3/figures/fig-c3e19.png @@ -0,0 +1,3 @@ +version https://git-lfs.github.com/spec/v1 +oid sha256:cafc512b6a5b1a4d00b1432bd90b31b8007bc9424fe0375442682e2095220844 +size 19841 diff --git a/processed_dataset/images/volume3/figures/fig-c3e2.png b/processed_dataset/images/volume3/figures/fig-c3e2.png new file mode 100644 index 0000000000000000000000000000000000000000..da54453183c52a75d04d9638ee5116f1bc9d916f --- /dev/null +++ b/processed_dataset/images/volume3/figures/fig-c3e2.png @@ -0,0 +1,3 @@ +version https://git-lfs.github.com/spec/v1 +oid sha256:f5e500413ac089c922e9e4107290c5c8ee5cef4f8cbba1b5335b0b68ecd429b1 +size 22115 diff --git a/processed_dataset/images/volume3/figures/fig-c3e3.png b/processed_dataset/images/volume3/figures/fig-c3e3.png new file mode 100644 index 0000000000000000000000000000000000000000..15194f305c92e3cd2ded387298a9b2760dc9a068 --- /dev/null +++ b/processed_dataset/images/volume3/figures/fig-c3e3.png @@ -0,0 +1,3 @@ +version https://git-lfs.github.com/spec/v1 +oid sha256:e5e790df680d1e1cd1f0c89d68f6fb92b8090b4be22d44c8f7e2c0dfa170bd8d +size 24526 diff --git a/processed_dataset/images/volume3/figures/fig-c3e4.png b/processed_dataset/images/volume3/figures/fig-c3e4.png new file mode 100644 index 0000000000000000000000000000000000000000..4a818e3e26fc9c1f4e57ab7210d17fd5b324a7e0 --- /dev/null +++ b/processed_dataset/images/volume3/figures/fig-c3e4.png @@ -0,0 +1,3 @@ +version https://git-lfs.github.com/spec/v1 +oid sha256:ebfdaa04a9baa7a8af44c0c97f845fe92b06f2c072c8bd60a7d3171685bae1c3 +size 25477 diff --git a/processed_dataset/images/volume3/figures/fig-c3e6.png b/processed_dataset/images/volume3/figures/fig-c3e6.png new file mode 100644 index 0000000000000000000000000000000000000000..22bdd291b92384eda2cfbbb7217d4c4f4ff0bf45 --- /dev/null +++ b/processed_dataset/images/volume3/figures/fig-c3e6.png @@ -0,0 +1,3 @@ +version https://git-lfs.github.com/spec/v1 +oid sha256:ab4c0b99701b96b037c2e97302fa74bccfea105e754bc7d981c9fb8f9197941a +size 22875 diff --git a/processed_dataset/images/volume3/figures/fig-c4e17.png b/processed_dataset/images/volume3/figures/fig-c4e17.png new file mode 100644 index 0000000000000000000000000000000000000000..ad47fa8ab5434a4456f6305e108729f953e4c1b0 --- /dev/null +++ b/processed_dataset/images/volume3/figures/fig-c4e17.png @@ -0,0 +1,3 @@ +version https://git-lfs.github.com/spec/v1 +oid sha256:26c68db3ff55cba0b92242d1e25bc21663e3bd5c87b565faa39aacdb6d348ffe +size 35331 diff --git a/processed_dataset/images/volume3/figures/fig-c4e18.png b/processed_dataset/images/volume3/figures/fig-c4e18.png new file mode 100644 index 0000000000000000000000000000000000000000..b1f2c59c56c6da8a790f498223552a6edc7d0970 --- /dev/null +++ b/processed_dataset/images/volume3/figures/fig-c4e18.png @@ -0,0 +1,3 @@ +version https://git-lfs.github.com/spec/v1 +oid sha256:b2f5204f8b76dbc5dac0467f1a3a35270ebaf52f3855afb7fbc1b0bc105a94f4 +size 42906 diff --git a/processed_dataset/images/volume3/figures/fig-c4p23.png b/processed_dataset/images/volume3/figures/fig-c4p23.png new file mode 100644 index 0000000000000000000000000000000000000000..ba68d23653def65f7d23b67f1be9617b8a452e56 --- /dev/null +++ b/processed_dataset/images/volume3/figures/fig-c4p23.png @@ -0,0 +1,3 @@ +version https://git-lfs.github.com/spec/v1 +oid sha256:ab284888a3f5d42acf3e718e8d5c12ac695e815f2366cb39ca13217eb16dde1a +size 19065 diff --git a/processed_dataset/images/volume3/figures/fig-c5i1.png b/processed_dataset/images/volume3/figures/fig-c5i1.png new file mode 100644 index 0000000000000000000000000000000000000000..913d1cc92d92f9192c23789aac90e8cb6d09d3d9 --- /dev/null +++ b/processed_dataset/images/volume3/figures/fig-c5i1.png @@ -0,0 +1,3 @@ +version https://git-lfs.github.com/spec/v1 +oid sha256:22ed393b01b045e5861ff301857d57fd1fe617e133e70f75ae9e9eff85172c27 +size 36586 diff --git a/processed_dataset/images/volume3/figures/fig-c5i2.png b/processed_dataset/images/volume3/figures/fig-c5i2.png new file mode 100644 index 0000000000000000000000000000000000000000..0bb29b1f0657427a9d903394484f44a7f9baaf42 --- /dev/null +++ b/processed_dataset/images/volume3/figures/fig-c5i2.png @@ -0,0 +1,3 @@ +version https://git-lfs.github.com/spec/v1 +oid sha256:4917cea56b777c32c5cde977a909f96a204e304cbecf38fb00b3c69eb7c76915 +size 25991 diff --git a/processed_dataset/images/volume3/figures/fig-c5i3.png b/processed_dataset/images/volume3/figures/fig-c5i3.png new file mode 100644 index 0000000000000000000000000000000000000000..1fe385c88606b5a16a21ac913b8f958517cf7071 --- /dev/null +++ b/processed_dataset/images/volume3/figures/fig-c5i3.png @@ -0,0 +1,3 @@ +version https://git-lfs.github.com/spec/v1 +oid sha256:fa18875de914f4b2663f60ae45189523b09c6f30937b3db21ee356df30bcb9ee +size 24728 diff --git a/processed_dataset/images/volume3/figures/fig-c5i4.png b/processed_dataset/images/volume3/figures/fig-c5i4.png new file mode 100644 index 0000000000000000000000000000000000000000..99ccce6c7300f20792b0b855f95027c522a30da9 --- /dev/null +++ b/processed_dataset/images/volume3/figures/fig-c5i4.png @@ -0,0 +1,3 @@ +version https://git-lfs.github.com/spec/v1 +oid sha256:ccd4622d72e44af8e69e2bad5be1ab41b824c10df9471ea62e32f9769fc008b2 +size 42214 diff --git a/processed_dataset/images/volume3/figures/fig-c5i5.png b/processed_dataset/images/volume3/figures/fig-c5i5.png new file mode 100644 index 0000000000000000000000000000000000000000..2530d59002dd4d16853a1e2a7c880f8ffe28f0b0 --- /dev/null +++ b/processed_dataset/images/volume3/figures/fig-c5i5.png @@ -0,0 +1,3 @@ +version https://git-lfs.github.com/spec/v1 +oid sha256:926eed57010a1fbe2598c9415299781ca9831ea5b86cadaf35ba34496f2bd667 +size 42621 diff --git a/processed_dataset/images/volume3/figures/fig-c5i6.png b/processed_dataset/images/volume3/figures/fig-c5i6.png new file mode 100644 index 0000000000000000000000000000000000000000..813a59fa69b073d0f11b56c276c4e7d15afda597 --- /dev/null +++ b/processed_dataset/images/volume3/figures/fig-c5i6.png @@ -0,0 +1,3 @@ +version https://git-lfs.github.com/spec/v1 +oid sha256:4a161269c9afb41ba49707cbf2d4a9d57964043a919514a15700038f03bf6f83 +size 31552 diff --git a/processed_dataset/images/volume3/figures/fig-c5i7.png b/processed_dataset/images/volume3/figures/fig-c5i7.png new file mode 100644 index 0000000000000000000000000000000000000000..97a7bb57659913edfdcf1272aac6b33a49a7649b --- /dev/null +++ b/processed_dataset/images/volume3/figures/fig-c5i7.png @@ -0,0 +1,3 @@ +version https://git-lfs.github.com/spec/v1 +oid sha256:0f695528cccae10b28f9ad76843292183b022ed3a960c4c22952413aa21215fd +size 33906 diff --git a/processed_dataset/images/volume3/figures/fig-c5i8.png b/processed_dataset/images/volume3/figures/fig-c5i8.png new file mode 100644 index 0000000000000000000000000000000000000000..a0ba7a9283ea864c2cb792852a619a6c04f5158f --- /dev/null +++ b/processed_dataset/images/volume3/figures/fig-c5i8.png @@ -0,0 +1,3 @@ +version https://git-lfs.github.com/spec/v1 +oid sha256:4a000e7e640189aa40591d42c8e5c10a52922cfe0aee606477f9e9ff59cd72fa +size 22446 diff --git a/processed_dataset/images/volume3/figures/fig-c5p28.png b/processed_dataset/images/volume3/figures/fig-c5p28.png new file mode 100644 index 0000000000000000000000000000000000000000..3bbd3ec3480ad0118845625497830b5e1dce1452 --- /dev/null +++ b/processed_dataset/images/volume3/figures/fig-c5p28.png @@ -0,0 +1,3 @@ +version https://git-lfs.github.com/spec/v1 +oid sha256:09d55ef98d11005b1fd6c40f32f17bed4a11612c5fef3d0ddf83bb5b0590416a +size 32765 diff --git a/processed_dataset/images/volume3/figures/fig-c5p30.png b/processed_dataset/images/volume3/figures/fig-c5p30.png new file mode 100644 index 0000000000000000000000000000000000000000..87121e2f647c41945cedc3b19f7e3628e2d5985c --- /dev/null +++ b/processed_dataset/images/volume3/figures/fig-c5p30.png @@ -0,0 +1,3 @@ +version https://git-lfs.github.com/spec/v1 +oid sha256:bfe2b28997e249308899ff693bfb88a3faf5398eb2b5d964a4f335308866275f +size 38460 diff --git a/processed_dataset/images/volume3/figures/fig-c6i1.png b/processed_dataset/images/volume3/figures/fig-c6i1.png new file mode 100644 index 0000000000000000000000000000000000000000..8312d408174b05c0601390604b4ab512951549f8 --- /dev/null +++ b/processed_dataset/images/volume3/figures/fig-c6i1.png @@ -0,0 +1,3 @@ +version https://git-lfs.github.com/spec/v1 +oid sha256:cfa23c16a302a21af533d610968cf78bca4b97e0ec6c1aa7392506ad7c8cece6 +size 17560 diff --git a/processed_dataset/images/volume3/figures/fig-c6i10.png b/processed_dataset/images/volume3/figures/fig-c6i10.png new file mode 100644 index 0000000000000000000000000000000000000000..4003598b96ca89d5fb6af7bdebfef6c044696b04 --- /dev/null +++ b/processed_dataset/images/volume3/figures/fig-c6i10.png @@ -0,0 +1,3 @@ +version https://git-lfs.github.com/spec/v1 +oid sha256:6c4e2d2ee7068ed047f602518c1a33306393e5acff58a75561068b86f3a1dac7 +size 35772 diff --git a/processed_dataset/images/volume3/figures/fig-c6i11.png b/processed_dataset/images/volume3/figures/fig-c6i11.png new file mode 100644 index 0000000000000000000000000000000000000000..42c9ae1d4cda4d72fb74c1830b26922ed9233e52 --- /dev/null +++ b/processed_dataset/images/volume3/figures/fig-c6i11.png @@ -0,0 +1,3 @@ +version https://git-lfs.github.com/spec/v1 +oid sha256:9d094c6de154e730af1d3b2f9dc7288f3c9cccf21085f4f04a66f579a5b5faac +size 35090 diff --git a/processed_dataset/images/volume3/figures/fig-c6i12.png b/processed_dataset/images/volume3/figures/fig-c6i12.png new file mode 100644 index 0000000000000000000000000000000000000000..d1caff457b15094ac85c1820288c729c0d58ddee --- /dev/null +++ b/processed_dataset/images/volume3/figures/fig-c6i12.png @@ -0,0 +1,3 @@ +version https://git-lfs.github.com/spec/v1 +oid sha256:ce7377ee22f6f435542a9c336368ea2080347df8c08605d36703efcbbd5316b7 +size 37230 diff --git a/processed_dataset/images/volume3/figures/fig-c6i13.png b/processed_dataset/images/volume3/figures/fig-c6i13.png new file mode 100644 index 0000000000000000000000000000000000000000..5ac1d273050c5e4c93bcc413fbf3a869a939f0e7 --- /dev/null +++ b/processed_dataset/images/volume3/figures/fig-c6i13.png @@ -0,0 +1,3 @@ +version https://git-lfs.github.com/spec/v1 +oid sha256:78f5392e0b09905d24a237f01ace9da46af0a3f76e17b4310ab213142ad3ca15 +size 34533 diff --git a/processed_dataset/images/volume3/figures/fig-c6i2.png b/processed_dataset/images/volume3/figures/fig-c6i2.png new file mode 100644 index 0000000000000000000000000000000000000000..fefd0aad5a4ec916d94222b4d6f03f6038e59fad --- /dev/null +++ b/processed_dataset/images/volume3/figures/fig-c6i2.png @@ -0,0 +1,3 @@ +version https://git-lfs.github.com/spec/v1 +oid sha256:30098c3a68b108118ddd9c58d1534202f3926a15b3684dd6b0b8522a6f5d04c1 +size 20266 diff --git a/processed_dataset/images/volume3/figures/fig-c6i3-1.png b/processed_dataset/images/volume3/figures/fig-c6i3-1.png new file mode 100644 index 0000000000000000000000000000000000000000..4767ad343560e2e142bf3d4a9c9af5d853a0d131 --- /dev/null +++ b/processed_dataset/images/volume3/figures/fig-c6i3-1.png @@ -0,0 +1,3 @@ +version https://git-lfs.github.com/spec/v1 +oid sha256:a1a3a7e8be5ec6e557352fac9c50afafb2109906d730e10ad4c2928d5a3aeb6f +size 15982 diff --git a/processed_dataset/images/volume3/figures/fig-c6i3-2.png b/processed_dataset/images/volume3/figures/fig-c6i3-2.png new file mode 100644 index 0000000000000000000000000000000000000000..b54414c3b18a5c8e4b6e2733e2cbe6493d52d657 --- /dev/null +++ b/processed_dataset/images/volume3/figures/fig-c6i3-2.png @@ -0,0 +1,3 @@ +version https://git-lfs.github.com/spec/v1 +oid sha256:0978bbdaadbf1a7e48df47d27ab5a3b40d73c45556d01c0a10645b67dbc32054 +size 17928 diff --git a/processed_dataset/images/volume3/figures/fig-c6i3-3.png b/processed_dataset/images/volume3/figures/fig-c6i3-3.png new file mode 100644 index 0000000000000000000000000000000000000000..227507867fb5b4d86e4779a021f9040e7e6b0dcf --- /dev/null +++ b/processed_dataset/images/volume3/figures/fig-c6i3-3.png @@ -0,0 +1,3 @@ +version https://git-lfs.github.com/spec/v1 +oid sha256:f90d46461fb5b51b6b156bdf5d2c26f6ee3f82b14f424d20d5664fa6da6408ba +size 16128 diff --git a/processed_dataset/images/volume3/figures/fig-c6i3-4.png b/processed_dataset/images/volume3/figures/fig-c6i3-4.png new file mode 100644 index 0000000000000000000000000000000000000000..2e6746f109f5f208604998bf5d4665386248861f --- /dev/null +++ b/processed_dataset/images/volume3/figures/fig-c6i3-4.png @@ -0,0 +1,3 @@ +version https://git-lfs.github.com/spec/v1 +oid sha256:1bc824cf23c135c2bfba8f6a0b89ac5f307662f734edb6f6e54452ba94b00794 +size 19601 diff --git a/processed_dataset/images/volume3/figures/fig-c6i4.png b/processed_dataset/images/volume3/figures/fig-c6i4.png new file mode 100644 index 0000000000000000000000000000000000000000..0301cb6c79cdbed3d6081a247808b3a311cfbbd0 --- /dev/null +++ b/processed_dataset/images/volume3/figures/fig-c6i4.png @@ -0,0 +1,3 @@ +version https://git-lfs.github.com/spec/v1 +oid sha256:894c1bc04119f786efc2d23a1acad39b8788ac50a52df1ea96d0416712208739 +size 22086 diff --git a/processed_dataset/images/volume3/figures/fig-c6i5.png b/processed_dataset/images/volume3/figures/fig-c6i5.png new file mode 100644 index 0000000000000000000000000000000000000000..79d2383ab1e8506c1169ac21c323e56e0249a45f --- /dev/null +++ b/processed_dataset/images/volume3/figures/fig-c6i5.png @@ -0,0 +1,3 @@ +version https://git-lfs.github.com/spec/v1 +oid sha256:685239fded6d4d3c00fab6816cb8f3fa92f57c75272607a8717a18efb01f3368 +size 24165 diff --git a/processed_dataset/images/volume3/figures/fig-c6i6.png b/processed_dataset/images/volume3/figures/fig-c6i6.png new file mode 100644 index 0000000000000000000000000000000000000000..7ea0c74ca38138769692b2058fc702ac243669ac --- /dev/null +++ b/processed_dataset/images/volume3/figures/fig-c6i6.png @@ -0,0 +1,3 @@ +version https://git-lfs.github.com/spec/v1 +oid sha256:d12112556d7db5ed6aa09f04c0b3914afdaa062425047d8cabbaf3e5a16ba5ad +size 39640 diff --git a/processed_dataset/images/volume3/figures/fig-c6i7.png b/processed_dataset/images/volume3/figures/fig-c6i7.png new file mode 100644 index 0000000000000000000000000000000000000000..54011e0c376765736c965f7b5e390f05ed4613ee --- /dev/null +++ b/processed_dataset/images/volume3/figures/fig-c6i7.png @@ -0,0 +1,3 @@ +version https://git-lfs.github.com/spec/v1 +oid sha256:5422425468591afea229da07ccbd9366bdbdc408d3489bc9f4e62142af00ea80 +size 30920 diff --git a/processed_dataset/images/volume3/figures/fig-c6i8.png b/processed_dataset/images/volume3/figures/fig-c6i8.png new file mode 100644 index 0000000000000000000000000000000000000000..4c26f23f5ec9b348e37711ee90dbf485d41200c9 --- /dev/null +++ b/processed_dataset/images/volume3/figures/fig-c6i8.png @@ -0,0 +1,3 @@ +version https://git-lfs.github.com/spec/v1 +oid sha256:d4681e67c68734d22350baeffe6ba6bbdc906961142328586b08723533341336 +size 26958 diff --git a/processed_dataset/images/volume3/figures/fig-c6i9.png b/processed_dataset/images/volume3/figures/fig-c6i9.png new file mode 100644 index 0000000000000000000000000000000000000000..3110a8edaeba05d5fc8da894f9c402d32d6567be --- /dev/null +++ b/processed_dataset/images/volume3/figures/fig-c6i9.png @@ -0,0 +1,3 @@ +version https://git-lfs.github.com/spec/v1 +oid sha256:0f9b1311eaab64caa1acba50ba3194e14b4318a593cfd4c4aeb1ad8dc21f7675 +size 30695 diff --git a/processed_dataset/images/volume3/figures/fig-c6p15.png b/processed_dataset/images/volume3/figures/fig-c6p15.png new file mode 100644 index 0000000000000000000000000000000000000000..5226e32b3729906011c177d9e60e70f422a3ca93 --- /dev/null +++ b/processed_dataset/images/volume3/figures/fig-c6p15.png @@ -0,0 +1,3 @@ +version https://git-lfs.github.com/spec/v1 +oid sha256:6f978729aa4ce724347e8c12d7a888b320118b505393b80a4c3051b9b247d8c4 +size 20019 diff --git a/processed_dataset/images/volume3/figures/fig-c6p16.png b/processed_dataset/images/volume3/figures/fig-c6p16.png new file mode 100644 index 0000000000000000000000000000000000000000..f6da35a7f84936a398583efc85bac7c544aa573c --- /dev/null +++ b/processed_dataset/images/volume3/figures/fig-c6p16.png @@ -0,0 +1,3 @@ +version https://git-lfs.github.com/spec/v1 +oid sha256:ec0743ff8103add0fc7ce48cb995cb6ecf925e2b8f5d1c8b720e7c173f3ccb31 +size 22333 diff --git a/processed_dataset/images/volume3/figures/fig-c6p17.png b/processed_dataset/images/volume3/figures/fig-c6p17.png new file mode 100644 index 0000000000000000000000000000000000000000..49092bd508b8916d527044bb466a692c0cc048bf --- /dev/null +++ b/processed_dataset/images/volume3/figures/fig-c6p17.png @@ -0,0 +1,3 @@ +version https://git-lfs.github.com/spec/v1 +oid sha256:af206b0cb6fe56b0918a87757ca1d9cbcc6eefcf2bc2b7f87acd82fd2c819e9b +size 24860 diff --git a/processed_dataset/images/volume3/figures/fig-c6p18.png b/processed_dataset/images/volume3/figures/fig-c6p18.png new file mode 100644 index 0000000000000000000000000000000000000000..1e0fb75f63e48141f702079bf15f745367000655 --- /dev/null +++ b/processed_dataset/images/volume3/figures/fig-c6p18.png @@ -0,0 +1,3 @@ +version https://git-lfs.github.com/spec/v1 +oid sha256:b58f341ba186af639c293e376950f9d7ccdbc4a7bc41ff9ddae3da0ed63e7b7d +size 34288 diff --git a/processed_dataset/images/volume3/figures/fig-c6p24.png b/processed_dataset/images/volume3/figures/fig-c6p24.png new file mode 100644 index 0000000000000000000000000000000000000000..d9980d7cf35494a20ddff9969f9af432ea2df319 --- /dev/null +++ b/processed_dataset/images/volume3/figures/fig-c6p24.png @@ -0,0 +1,3 @@ +version https://git-lfs.github.com/spec/v1 +oid sha256:2f1e4a9c0bdfca1cb9415b2bd439a1cb99e9dad2ae35d3d63a250daa5d249b44 +size 44723 diff --git a/processed_dataset/images/volume4/figures/fig-c2p18.png b/processed_dataset/images/volume4/figures/fig-c2p18.png new file mode 100644 index 0000000000000000000000000000000000000000..490f3d95048ec058cf8365b92e6f8fc1c0bf3aef --- /dev/null +++ b/processed_dataset/images/volume4/figures/fig-c2p18.png @@ -0,0 +1,3 @@ +version https://git-lfs.github.com/spec/v1 +oid sha256:1313e7608dd3220f2703f23f6fd17f5051076906dea16e8b330ecf9bebcd66af +size 35513 diff --git a/processed_dataset/images/volume4/figures/fig-c2p21.png b/processed_dataset/images/volume4/figures/fig-c2p21.png new file mode 100644 index 0000000000000000000000000000000000000000..3c9af2404b099ba5d6afb7ffad3d10bea782eef3 --- /dev/null +++ b/processed_dataset/images/volume4/figures/fig-c2p21.png @@ -0,0 +1,3 @@ +version https://git-lfs.github.com/spec/v1 +oid sha256:3044fd5a5a13460c9ea5053fe0c9d6c0c5083396f9b842fa7e1972a29d4db283 +size 37352 diff --git a/processed_dataset/images/volume4/figures/fig-c2p22.png b/processed_dataset/images/volume4/figures/fig-c2p22.png new file mode 100644 index 0000000000000000000000000000000000000000..466e1db481ae84a9e29b20603f0ce58a4162d045 --- /dev/null +++ b/processed_dataset/images/volume4/figures/fig-c2p22.png @@ -0,0 +1,3 @@ +version https://git-lfs.github.com/spec/v1 +oid sha256:204811386a4ceeddd2e31577c6838ce885c6637d12402d9aae826857b41a976e +size 22679 diff --git a/processed_dataset/images/volume5/figures/fig-c3i1.png b/processed_dataset/images/volume5/figures/fig-c3i1.png new file mode 100644 index 0000000000000000000000000000000000000000..cc1b7c67e5a3254405bbcedd5dd2d778a487a32f --- /dev/null +++ b/processed_dataset/images/volume5/figures/fig-c3i1.png @@ -0,0 +1,3 @@ +version https://git-lfs.github.com/spec/v1 +oid sha256:502d42e1d98fe17ace0bc34b1f9c9c1f88ed810d5e0e857a4220aa50dcc29dfe +size 33083 diff --git a/processed_dataset/images/volume5/figures/fig-c5i1.png b/processed_dataset/images/volume5/figures/fig-c5i1.png new file mode 100644 index 0000000000000000000000000000000000000000..c507a93d826ec1b5d7548b56223d74e9aaedc288 --- /dev/null +++ b/processed_dataset/images/volume5/figures/fig-c5i1.png @@ -0,0 +1,3 @@ +version https://git-lfs.github.com/spec/v1 +oid sha256:53ed3b737ce6ee32e2a8724de6e5c6583fce1c29dcfc0a5eb6d70f3ec530d90a +size 40853 diff --git a/processed_dataset/images/volume5/figures/fig-c5i2.png b/processed_dataset/images/volume5/figures/fig-c5i2.png new file mode 100644 index 0000000000000000000000000000000000000000..bd675cde0311085c3f4277b3e3edd44de6023179 --- /dev/null +++ b/processed_dataset/images/volume5/figures/fig-c5i2.png @@ -0,0 +1,3 @@ +version https://git-lfs.github.com/spec/v1 +oid sha256:fda023f35cc3c83720d6387bed762ea958a3a8c0abc65c6c3e1eed4a5b52f86f +size 44862 diff --git a/processed_dataset/images/volume5/figures/fig-c5i3.png b/processed_dataset/images/volume5/figures/fig-c5i3.png new file mode 100644 index 0000000000000000000000000000000000000000..505173cefe4d9747d52c200e59c70d8914a84313 --- /dev/null +++ b/processed_dataset/images/volume5/figures/fig-c5i3.png @@ -0,0 +1,3 @@ +version https://git-lfs.github.com/spec/v1 +oid sha256:0c33ddc4d6e19da2bb4186a83ab14030a7bb68bee556ff6d86793926812e81dc +size 39958 diff --git a/processed_dataset/images/volume5/figures/fig-c5p10.png b/processed_dataset/images/volume5/figures/fig-c5p10.png new file mode 100644 index 0000000000000000000000000000000000000000..4bbff2626dc95cc4fa24dc6c320a3c43ac4cc7af --- /dev/null +++ b/processed_dataset/images/volume5/figures/fig-c5p10.png @@ -0,0 +1,3 @@ +version https://git-lfs.github.com/spec/v1 +oid sha256:1a84bf0a3c262250a83e6e1844ed8b21f02f5429158d0384b408bf67c38f3324 +size 48340 diff --git a/processed_dataset/images/volume5/figures/fig-c5p12.png b/processed_dataset/images/volume5/figures/fig-c5p12.png new file mode 100644 index 0000000000000000000000000000000000000000..5e08d99b1a6c3ef82e561124e18cf71c1a5663e6 --- /dev/null +++ b/processed_dataset/images/volume5/figures/fig-c5p12.png @@ -0,0 +1,3 @@ +version https://git-lfs.github.com/spec/v1 +oid sha256:0e894136765c61074058563ae456a449f12a8625a245569f23e6af7035e87e74 +size 51473 diff --git a/processed_dataset/images/volume5/figures/fig-c5p13.png b/processed_dataset/images/volume5/figures/fig-c5p13.png new file mode 100644 index 0000000000000000000000000000000000000000..7547ad7cccc554c94a9af944aa2475a80642cc19 --- /dev/null +++ b/processed_dataset/images/volume5/figures/fig-c5p13.png @@ -0,0 +1,3 @@ +version https://git-lfs.github.com/spec/v1 +oid sha256:a2046c605ee691e4b3c5a918d7646506280d79158e792dc4a45d90df358bba43 +size 33165 diff --git a/processed_dataset/images/volume5/figures/fig-c5p14.png b/processed_dataset/images/volume5/figures/fig-c5p14.png new file mode 100644 index 0000000000000000000000000000000000000000..50da13f76ce22c7be9f30965049f8761f5112106 --- /dev/null +++ b/processed_dataset/images/volume5/figures/fig-c5p14.png @@ -0,0 +1,3 @@ +version https://git-lfs.github.com/spec/v1 +oid sha256:6ed3bae5a20d24595f0aa2dfd6b26778da864b411d2a7dec498c883e5c675d49 +size 50034 diff --git a/processed_dataset/images/volume5/figures/fig-c5p9.png b/processed_dataset/images/volume5/figures/fig-c5p9.png new file mode 100644 index 0000000000000000000000000000000000000000..82b699eecd2b9d157b528ed2ce850cf5577945e9 --- /dev/null +++ b/processed_dataset/images/volume5/figures/fig-c5p9.png @@ -0,0 +1,3 @@ +version https://git-lfs.github.com/spec/v1 +oid sha256:1d344f1a470622095f33eb4854d40560a391f2e9db2c69b52ced49de4957c71e +size 40069 diff --git a/processed_dataset/images/volume5/figures/fig-c8i1.png b/processed_dataset/images/volume5/figures/fig-c8i1.png new file mode 100644 index 0000000000000000000000000000000000000000..40487d9ead5801b406245de8101be8e29209a508 --- /dev/null +++ b/processed_dataset/images/volume5/figures/fig-c8i1.png @@ -0,0 +1,3 @@ +version https://git-lfs.github.com/spec/v1 +oid sha256:4eb35ac24495b046b45be7429ec287552211ee7b662ecb55650e569b5ec520d1 +size 37598 diff --git a/processed_dataset/images/volume5/figures/fig-c8i2.png b/processed_dataset/images/volume5/figures/fig-c8i2.png new file mode 100644 index 0000000000000000000000000000000000000000..336148dd36f1519c84f64dc989f058af8267b49d --- /dev/null +++ b/processed_dataset/images/volume5/figures/fig-c8i2.png @@ -0,0 +1,3 @@ +version https://git-lfs.github.com/spec/v1 +oid sha256:fefe9323b19d78fd324d4a532361c2b8e59b645115064290caf9b6ce447a88b7 +size 34647 diff --git a/processed_dataset/images/volume5/figures/fig-c8i3.png b/processed_dataset/images/volume5/figures/fig-c8i3.png new file mode 100644 index 0000000000000000000000000000000000000000..36df847419249f96ff47e309483d4baeb8de952b --- /dev/null +++ b/processed_dataset/images/volume5/figures/fig-c8i3.png @@ -0,0 +1,3 @@ +version https://git-lfs.github.com/spec/v1 +oid sha256:8344c4383c849681816d8f4a22f3aee0983addff539c3defcfc915e639713b9c +size 36608 diff --git a/processed_dataset/images/volume6/figures/fig-c1i1.png b/processed_dataset/images/volume6/figures/fig-c1i1.png new file mode 100644 index 0000000000000000000000000000000000000000..ae4665c0797c958ad31765829a1ee6f494119e51 --- /dev/null +++ b/processed_dataset/images/volume6/figures/fig-c1i1.png @@ -0,0 +1,3 @@ +version https://git-lfs.github.com/spec/v1 +oid sha256:47d1971c9400a4932bd9cc678e9896821411fbb0a587f2a29b64bdc5e1891e5b +size 28678 diff --git a/processed_dataset/images/volume6/figures/fig-c1i2.png b/processed_dataset/images/volume6/figures/fig-c1i2.png new file mode 100644 index 0000000000000000000000000000000000000000..7658a98ef2c8f81703eb2687011062bdefe1472b --- /dev/null +++ b/processed_dataset/images/volume6/figures/fig-c1i2.png @@ -0,0 +1,3 @@ +version https://git-lfs.github.com/spec/v1 +oid sha256:f14464768544d2d1f3770475f351315902911aeb5ffb55da0d44a7a6e8da89d5 +size 23040 diff --git a/processed_dataset/images/volume6/figures/fig-c1i3.png b/processed_dataset/images/volume6/figures/fig-c1i3.png new file mode 100644 index 0000000000000000000000000000000000000000..f45709c39d9be625ba003c01ed7abc451f4c995c --- /dev/null +++ b/processed_dataset/images/volume6/figures/fig-c1i3.png @@ -0,0 +1,3 @@ +version https://git-lfs.github.com/spec/v1 +oid sha256:528ba902d6c1c86b57147731ad1619fb57ef42737249a7f8062aaeb1750a39e8 +size 28112 diff --git a/processed_dataset/images/volume6/figures/fig-c1i4.png b/processed_dataset/images/volume6/figures/fig-c1i4.png new file mode 100644 index 0000000000000000000000000000000000000000..7734b9f4a9bf181553dc669dcb5690f961170a9d --- /dev/null +++ b/processed_dataset/images/volume6/figures/fig-c1i4.png @@ -0,0 +1,3 @@ +version https://git-lfs.github.com/spec/v1 +oid sha256:2448086ad3196085a74dc9c570018055e8a6100d0b01dddafb380ce6e3558513 +size 28813 diff --git a/processed_dataset/images/volume6/figures/fig-c1p44.png b/processed_dataset/images/volume6/figures/fig-c1p44.png new file mode 100644 index 0000000000000000000000000000000000000000..fe45aa9dd510276e3e839f70b7c60fd9622ea6dd --- /dev/null +++ b/processed_dataset/images/volume6/figures/fig-c1p44.png @@ -0,0 +1,3 @@ +version https://git-lfs.github.com/spec/v1 +oid sha256:b87a536b72560a7ee95fffd95d398a30961478cbb1cda8caa24903be63801815 +size 32117 diff --git a/processed_dataset/images/volume6/figures/fig-c2i5.png b/processed_dataset/images/volume6/figures/fig-c2i5.png new file mode 100644 index 0000000000000000000000000000000000000000..31e33a453b33842b0f932ee1902ece07a74c3365 --- /dev/null +++ b/processed_dataset/images/volume6/figures/fig-c2i5.png @@ -0,0 +1,3 @@ +version https://git-lfs.github.com/spec/v1 +oid sha256:c1eb09a52e17b5b643ab8011ba685893e1b4aa0ceda405c51757866a409ce2bf +size 33940 diff --git a/processed_dataset/images/volume7/figures/fig-c10a1.png b/processed_dataset/images/volume7/figures/fig-c10a1.png new file mode 100644 index 0000000000000000000000000000000000000000..e02e26d8b59446950eeb152bc2de3bd64b75852d --- /dev/null +++ b/processed_dataset/images/volume7/figures/fig-c10a1.png @@ -0,0 +1,3 @@ +version https://git-lfs.github.com/spec/v1 +oid sha256:1a960ba34f44d525cae9a316013873f4f889218d001626d1259e9e4e0b12dc93 +size 33578 diff --git a/processed_dataset/images/volume7/figures/fig-c10a12.png b/processed_dataset/images/volume7/figures/fig-c10a12.png new file mode 100644 index 0000000000000000000000000000000000000000..2c9900e8461fd000112786fe73ea2e46d63ec441 --- /dev/null +++ b/processed_dataset/images/volume7/figures/fig-c10a12.png @@ -0,0 +1,3 @@ +version https://git-lfs.github.com/spec/v1 +oid sha256:14301d4b0be82be9c3ff9e614a0761c48bfb88d2921b520159382e4c863e11ca +size 25282 diff --git a/processed_dataset/images/volume7/figures/fig-c10a13.png b/processed_dataset/images/volume7/figures/fig-c10a13.png new file mode 100644 index 0000000000000000000000000000000000000000..a6d819fd8978a77baf3ad335bf2b83ab999d3191 --- /dev/null +++ b/processed_dataset/images/volume7/figures/fig-c10a13.png @@ -0,0 +1,3 @@ +version https://git-lfs.github.com/spec/v1 +oid sha256:b496a4c110e3b0dff9e1970ff1c5cedae32c976b324333b7b4cdd51a96c8f207 +size 42214 diff --git a/processed_dataset/images/volume7/figures/fig-c10a14.png b/processed_dataset/images/volume7/figures/fig-c10a14.png new file mode 100644 index 0000000000000000000000000000000000000000..645e62eda60a284f9a3304ad8a2bb3c8adf90c48 --- /dev/null +++ b/processed_dataset/images/volume7/figures/fig-c10a14.png @@ -0,0 +1,3 @@ +version https://git-lfs.github.com/spec/v1 +oid sha256:54abb2aa60491fdca7f0a11c0814733da645290281a70f20bc58fe38cad156c1 +size 37421 diff --git a/processed_dataset/images/volume7/figures/fig-c10a16.png b/processed_dataset/images/volume7/figures/fig-c10a16.png new file mode 100644 index 0000000000000000000000000000000000000000..65eb9dc386ef555fb77ac518265d262b2c41b2da --- /dev/null +++ b/processed_dataset/images/volume7/figures/fig-c10a16.png @@ -0,0 +1,3 @@ +version https://git-lfs.github.com/spec/v1 +oid sha256:5a69c70f3c7cca6bdc5bca64a2bba30fe741c536daf401880019f9db10d8ea30 +size 28329 diff --git a/processed_dataset/images/volume7/figures/fig-c10a17.png b/processed_dataset/images/volume7/figures/fig-c10a17.png new file mode 100644 index 0000000000000000000000000000000000000000..4477541aa51eee3ea93744a4c063ca7d753466c9 --- /dev/null +++ b/processed_dataset/images/volume7/figures/fig-c10a17.png @@ -0,0 +1,3 @@ +version https://git-lfs.github.com/spec/v1 +oid sha256:9cb6a7cccbc6ecbff08c19e2ffa362ad40bac2a014e63fb2df2195c23f111575 +size 30546 diff --git a/processed_dataset/images/volume7/figures/fig-c10a4.png b/processed_dataset/images/volume7/figures/fig-c10a4.png new file mode 100644 index 0000000000000000000000000000000000000000..49760531f0f534152ee9c2010cb004dece34b521 --- /dev/null +++ b/processed_dataset/images/volume7/figures/fig-c10a4.png @@ -0,0 +1,3 @@ +version https://git-lfs.github.com/spec/v1 +oid sha256:e4eaa0ee19a72f2358c7f806bbeffffdce6897d5cb311c26d2d2524009f95982 +size 25401 diff --git a/processed_dataset/images/volume7/figures/fig-c10a6-1.png b/processed_dataset/images/volume7/figures/fig-c10a6-1.png new file mode 100644 index 0000000000000000000000000000000000000000..b55a38abf4d89effc684e73f536fda7390941743 --- /dev/null +++ b/processed_dataset/images/volume7/figures/fig-c10a6-1.png @@ -0,0 +1,3 @@ +version https://git-lfs.github.com/spec/v1 +oid sha256:daf76d6cb558da1dc884bc60fdbd1b0e4413d5c528ce1d1faa7a2f2197fc37b8 +size 24240 diff --git a/processed_dataset/images/volume7/figures/fig-c10a6-2.png b/processed_dataset/images/volume7/figures/fig-c10a6-2.png new file mode 100644 index 0000000000000000000000000000000000000000..df85ac76da28c2c99d18208bf28da6a5a55c1d12 --- /dev/null +++ b/processed_dataset/images/volume7/figures/fig-c10a6-2.png @@ -0,0 +1,3 @@ +version https://git-lfs.github.com/spec/v1 +oid sha256:7f312bdbe9ee8a05fc19dfd253507f7b38d3cb57ad8a383db5a6e592600426be +size 30757 diff --git a/processed_dataset/images/volume7/figures/fig-c10a7.png b/processed_dataset/images/volume7/figures/fig-c10a7.png new file mode 100644 index 0000000000000000000000000000000000000000..40c2e41a62d65c2edc95d676aa7b442e0a9dcbf2 --- /dev/null +++ b/processed_dataset/images/volume7/figures/fig-c10a7.png @@ -0,0 +1,3 @@ +version https://git-lfs.github.com/spec/v1 +oid sha256:5fea7fc6305b66f01dac517c939dde985ec67c0f405173ebf977babd93c63a3c +size 32467 diff --git a/processed_dataset/images/volume7/figures/fig-c10a9.png b/processed_dataset/images/volume7/figures/fig-c10a9.png new file mode 100644 index 0000000000000000000000000000000000000000..f35ea99c452789bf6ce8b70794d8279e216d238a --- /dev/null +++ b/processed_dataset/images/volume7/figures/fig-c10a9.png @@ -0,0 +1,3 @@ +version https://git-lfs.github.com/spec/v1 +oid sha256:97c4077c465e90d5bf9dfc223c536b1c1ac5cf7a9a715d0546cdf614dab10824 +size 31918 diff --git a/processed_dataset/images/volume7/figures/fig-c10i1.png b/processed_dataset/images/volume7/figures/fig-c10i1.png new file mode 100644 index 0000000000000000000000000000000000000000..4a2591b233590188ee72e5069bdb25c434f3c7a4 --- /dev/null +++ b/processed_dataset/images/volume7/figures/fig-c10i1.png @@ -0,0 +1,3 @@ +version https://git-lfs.github.com/spec/v1 +oid sha256:2c3f03ac25f97253797c57d8be4ed19faea2bdfbbba2634fd3a0cf043ac82058 +size 25568 diff --git a/processed_dataset/images/volume7/figures/fig-c10i2.png b/processed_dataset/images/volume7/figures/fig-c10i2.png new file mode 100644 index 0000000000000000000000000000000000000000..f0492fb4b91a8eef004655ae50a3c7b0f59da7c1 --- /dev/null +++ b/processed_dataset/images/volume7/figures/fig-c10i2.png @@ -0,0 +1,3 @@ +version https://git-lfs.github.com/spec/v1 +oid sha256:2c1bc9fe3f0f11b6e20e39458e9f2fb3ec99fccd4e1a1fa1f7780b028c689727 +size 27566 diff --git a/processed_dataset/images/volume7/figures/fig-c10i3.png b/processed_dataset/images/volume7/figures/fig-c10i3.png new file mode 100644 index 0000000000000000000000000000000000000000..a535db3685d7bef2f4621589290b23d746857c0b --- /dev/null +++ b/processed_dataset/images/volume7/figures/fig-c10i3.png @@ -0,0 +1,3 @@ +version https://git-lfs.github.com/spec/v1 +oid sha256:d97158e7c307cc1c975691a3e3317ff7eb76b9ba445e1297e5b2ab67a3a84cc7 +size 20282 diff --git a/processed_dataset/images/volume7/figures/fig-c10i4.png b/processed_dataset/images/volume7/figures/fig-c10i4.png new file mode 100644 index 0000000000000000000000000000000000000000..1c71298232024687c27ad15bbf197ebba250ffe9 --- /dev/null +++ b/processed_dataset/images/volume7/figures/fig-c10i4.png @@ -0,0 +1,3 @@ +version https://git-lfs.github.com/spec/v1 +oid sha256:e5b60a8abbd5698f407e6adbee977bfd01d27dac196240ef42a25dcfaf65a112 +size 27804 diff --git a/processed_dataset/images/volume7/figures/fig-c10i5.png b/processed_dataset/images/volume7/figures/fig-c10i5.png new file mode 100644 index 0000000000000000000000000000000000000000..876fbfba2d1953a79db73b3a1eed8334a81d827d --- /dev/null +++ b/processed_dataset/images/volume7/figures/fig-c10i5.png @@ -0,0 +1,3 @@ +version https://git-lfs.github.com/spec/v1 +oid sha256:901fafee01ed652ebe0ff99a3ac8488a5999144f350738482899b4cd575f2ed0 +size 33502 diff --git a/processed_dataset/images/volume7/figures/fig-c10i6.png b/processed_dataset/images/volume7/figures/fig-c10i6.png new file mode 100644 index 0000000000000000000000000000000000000000..054d51f7d8c492c6444b7a60c930cc245871100a --- /dev/null +++ b/processed_dataset/images/volume7/figures/fig-c10i6.png @@ -0,0 +1,3 @@ +version https://git-lfs.github.com/spec/v1 +oid sha256:20c8f3c7ca489950d7790c09fdcfc2d6d081cb3779372b724e040ae31fb48d51 +size 26413 diff --git a/processed_dataset/images/volume7/figures/fig-c10i7.png b/processed_dataset/images/volume7/figures/fig-c10i7.png new file mode 100644 index 0000000000000000000000000000000000000000..48de27374fd029dad14a994198d76ddc48cf1bec --- /dev/null +++ b/processed_dataset/images/volume7/figures/fig-c10i7.png @@ -0,0 +1,3 @@ +version https://git-lfs.github.com/spec/v1 +oid sha256:6139b595d8b2fc4b6695dca84cb8707ccba9e82250d354bfebd6635470f63095 +size 29263 diff --git a/processed_dataset/images/volume7/figures/fig-c10i8.png b/processed_dataset/images/volume7/figures/fig-c10i8.png new file mode 100644 index 0000000000000000000000000000000000000000..2e1294a3ecc9e3b68b46d3782c47548fe631b592 --- /dev/null +++ b/processed_dataset/images/volume7/figures/fig-c10i8.png @@ -0,0 +1,3 @@ +version https://git-lfs.github.com/spec/v1 +oid sha256:60c6262929f7b55e3c2b60619739985bd704e5d8e8453386a4580824418c2bdf +size 26843 diff --git a/processed_dataset/images/volume7/figures/fig-c10i9.png b/processed_dataset/images/volume7/figures/fig-c10i9.png new file mode 100644 index 0000000000000000000000000000000000000000..56b2e123f65ed37b7fe5b957a1b297afc677a254 --- /dev/null +++ b/processed_dataset/images/volume7/figures/fig-c10i9.png @@ -0,0 +1,3 @@ +version https://git-lfs.github.com/spec/v1 +oid sha256:5b58a485783d9a4daf54908328a9cbe464f90374f84c62d621025a3ad44b2cdc +size 24819 diff --git a/processed_dataset/images/volume7/figures/fig-c11a11.png b/processed_dataset/images/volume7/figures/fig-c11a11.png new file mode 100644 index 0000000000000000000000000000000000000000..1ee12efc794ff72223170acab33b7dbcb339fd17 --- /dev/null +++ b/processed_dataset/images/volume7/figures/fig-c11a11.png @@ -0,0 +1,3 @@ +version https://git-lfs.github.com/spec/v1 +oid sha256:02ce2ea47b4be6b50d8722c3a16af3ad34960b2aa648db254d4e25c7cd82f34c +size 36292 diff --git a/processed_dataset/images/volume7/figures/fig-c11a12.png b/processed_dataset/images/volume7/figures/fig-c11a12.png new file mode 100644 index 0000000000000000000000000000000000000000..683537e2aba5a5cd97ab6739f63e39ea2623712a --- /dev/null +++ b/processed_dataset/images/volume7/figures/fig-c11a12.png @@ -0,0 +1,3 @@ +version https://git-lfs.github.com/spec/v1 +oid sha256:c607ceae18f77a08a51b37bda5050b48a4f0d6c43844337a0ed10a23b5affabc +size 33102 diff --git a/processed_dataset/images/volume7/figures/fig-c11a13.png b/processed_dataset/images/volume7/figures/fig-c11a13.png new file mode 100644 index 0000000000000000000000000000000000000000..be870a41d0deb7106e47a45c378e968d2848ca8d --- /dev/null +++ b/processed_dataset/images/volume7/figures/fig-c11a13.png @@ -0,0 +1,3 @@ +version https://git-lfs.github.com/spec/v1 +oid sha256:f3845f4a44383a3d327602caf8da03d3172501ed94d32b88f113c8325534f641 +size 32442 diff --git a/processed_dataset/images/volume7/figures/fig-c11a14.png b/processed_dataset/images/volume7/figures/fig-c11a14.png new file mode 100644 index 0000000000000000000000000000000000000000..af0f222ff770f85f04f46ede4df68de2a4e9d350 --- /dev/null +++ b/processed_dataset/images/volume7/figures/fig-c11a14.png @@ -0,0 +1,3 @@ +version https://git-lfs.github.com/spec/v1 +oid sha256:800c455608dbcad7059183ae6f77809b56a127efb7d84027665b77fdcfe35f33 +size 25207 diff --git a/processed_dataset/images/volume7/figures/fig-c11a4.png b/processed_dataset/images/volume7/figures/fig-c11a4.png new file mode 100644 index 0000000000000000000000000000000000000000..7347e051137226965e612a85ea720dd229223517 --- /dev/null +++ b/processed_dataset/images/volume7/figures/fig-c11a4.png @@ -0,0 +1,3 @@ +version https://git-lfs.github.com/spec/v1 +oid sha256:4118527d146d3e403cec7bdd1eee910064e45894b96ec627a4eb0920d170e589 +size 40497 diff --git a/processed_dataset/images/volume7/figures/fig-c11a5.png b/processed_dataset/images/volume7/figures/fig-c11a5.png new file mode 100644 index 0000000000000000000000000000000000000000..7fd648c91f8013448e6ac9972a47024d6d313f33 --- /dev/null +++ b/processed_dataset/images/volume7/figures/fig-c11a5.png @@ -0,0 +1,3 @@ +version https://git-lfs.github.com/spec/v1 +oid sha256:56e81c95f6bbe6f8d7888bbe06ff079eab77261cccbc92dad089e4be4fcccef8 +size 36644 diff --git a/processed_dataset/images/volume7/figures/fig-c11a6.png b/processed_dataset/images/volume7/figures/fig-c11a6.png new file mode 100644 index 0000000000000000000000000000000000000000..ce13ca5ee15494d79c241809b472baf649198f7b --- /dev/null +++ b/processed_dataset/images/volume7/figures/fig-c11a6.png @@ -0,0 +1,3 @@ +version https://git-lfs.github.com/spec/v1 +oid sha256:db959938085a21b56e649e39af194e443c570f1245bb6d7e45305453e4bf22b6 +size 24994 diff --git a/processed_dataset/images/volume7/figures/fig-c11a7.png b/processed_dataset/images/volume7/figures/fig-c11a7.png new file mode 100644 index 0000000000000000000000000000000000000000..37d188422661678140b677664d668ac220b3faa9 --- /dev/null +++ b/processed_dataset/images/volume7/figures/fig-c11a7.png @@ -0,0 +1,3 @@ +version https://git-lfs.github.com/spec/v1 +oid sha256:45b0db2f7ce892ab47542771abc3086dca418c69f11995245a88c1662ead9421 +size 23491 diff --git a/processed_dataset/images/volume7/figures/fig-c11a8.png b/processed_dataset/images/volume7/figures/fig-c11a8.png new file mode 100644 index 0000000000000000000000000000000000000000..32614666917ff6609c4a25f59f8597844a3755d5 --- /dev/null +++ b/processed_dataset/images/volume7/figures/fig-c11a8.png @@ -0,0 +1,3 @@ +version https://git-lfs.github.com/spec/v1 +oid sha256:3a8664e46bc2d449b9eca72cc79a3e7fd026c69abbc6e1570e6a6a33df64a733 +size 75747 diff --git a/processed_dataset/images/volume7/figures/fig-c11a9.png b/processed_dataset/images/volume7/figures/fig-c11a9.png new file mode 100644 index 0000000000000000000000000000000000000000..f0041e80f7d7fd48fc8a40a3bdcf09bbf0fe5994 --- /dev/null +++ b/processed_dataset/images/volume7/figures/fig-c11a9.png @@ -0,0 +1,3 @@ +version https://git-lfs.github.com/spec/v1 +oid sha256:33c64656ce72127017b4dd7c985d1c2a5f15667bb5e6c034377dc6ec6e87b9a2 +size 32681 diff --git a/processed_dataset/images/volume7/figures/fig-c11i1.png b/processed_dataset/images/volume7/figures/fig-c11i1.png new file mode 100644 index 0000000000000000000000000000000000000000..7c42c6e68013a875bcc6ce8ca43a8b3062d23970 --- /dev/null +++ b/processed_dataset/images/volume7/figures/fig-c11i1.png @@ -0,0 +1,3 @@ +version https://git-lfs.github.com/spec/v1 +oid sha256:22693abe175c50e49c174b149203b18db223f91e98db50318bd9c1965f03531c +size 37411 diff --git a/processed_dataset/images/volume7/figures/fig-c11i10.png b/processed_dataset/images/volume7/figures/fig-c11i10.png new file mode 100644 index 0000000000000000000000000000000000000000..e16e85560ac2c462db468d3f83a8d01b9a1fc259 --- /dev/null +++ b/processed_dataset/images/volume7/figures/fig-c11i10.png @@ -0,0 +1,3 @@ +version https://git-lfs.github.com/spec/v1 +oid sha256:651cbd03bc59be5c7bdafa6b3c7563656dadb480fa44703dad611284f395a3c6 +size 19849 diff --git a/processed_dataset/images/volume7/figures/fig-c11i2.png b/processed_dataset/images/volume7/figures/fig-c11i2.png new file mode 100644 index 0000000000000000000000000000000000000000..b7f1843e8aa66ee04056fa110f59ecdea51c2531 --- /dev/null +++ b/processed_dataset/images/volume7/figures/fig-c11i2.png @@ -0,0 +1,3 @@ +version https://git-lfs.github.com/spec/v1 +oid sha256:304d3bf1b2bbd906627bf9fba52a6dcf3fa7979129a0f236a1102b9b784cd1a0 +size 33322 diff --git a/processed_dataset/images/volume7/figures/fig-c11i3.png b/processed_dataset/images/volume7/figures/fig-c11i3.png new file mode 100644 index 0000000000000000000000000000000000000000..ea120361dc3208c236c5ec31c3bc4f3d9546e41a --- /dev/null +++ b/processed_dataset/images/volume7/figures/fig-c11i3.png @@ -0,0 +1,3 @@ +version https://git-lfs.github.com/spec/v1 +oid sha256:9f55748a00b957df17b4749a7f74aaf1a6a1dba411e680eb54dad46d18025c9a +size 31901 diff --git a/processed_dataset/images/volume7/figures/fig-c11i4.png b/processed_dataset/images/volume7/figures/fig-c11i4.png new file mode 100644 index 0000000000000000000000000000000000000000..036f657516911849beb3fea891ed7472b223c322 --- /dev/null +++ b/processed_dataset/images/volume7/figures/fig-c11i4.png @@ -0,0 +1,3 @@ +version https://git-lfs.github.com/spec/v1 +oid sha256:d33610ebc863046cdaaad8ad1d92445fd60ed42d68e919f175b14fa7b02a77b5 +size 36198 diff --git a/processed_dataset/images/volume7/figures/fig-c11i5.png b/processed_dataset/images/volume7/figures/fig-c11i5.png new file mode 100644 index 0000000000000000000000000000000000000000..f7f57bf526e94df1ff9dfbab2ddca02c381c1dde --- /dev/null +++ b/processed_dataset/images/volume7/figures/fig-c11i5.png @@ -0,0 +1,3 @@ +version https://git-lfs.github.com/spec/v1 +oid sha256:dbdd11a115aabb369fe2af723598546e1d13ea46e422da7767cde229a64fd947 +size 36792 diff --git a/processed_dataset/images/volume7/figures/fig-c11i6.png b/processed_dataset/images/volume7/figures/fig-c11i6.png new file mode 100644 index 0000000000000000000000000000000000000000..0f1889f5307f706f20d45119e773194f7880de94 --- /dev/null +++ b/processed_dataset/images/volume7/figures/fig-c11i6.png @@ -0,0 +1,3 @@ +version https://git-lfs.github.com/spec/v1 +oid sha256:1c5f4304f43089e5c8b10ef7cc8ea2e66c58f2faf33a89f3176b0489f7417d20 +size 24976 diff --git a/processed_dataset/images/volume7/figures/fig-c11i7.png b/processed_dataset/images/volume7/figures/fig-c11i7.png new file mode 100644 index 0000000000000000000000000000000000000000..10c1822101ddee0dd2b1ca03de344f7d5f5bcf73 --- /dev/null +++ b/processed_dataset/images/volume7/figures/fig-c11i7.png @@ -0,0 +1,3 @@ +version https://git-lfs.github.com/spec/v1 +oid sha256:ffe7e563c7c379720f61a8ae0603b83f09536704183278eda166bd2336e04b35 +size 34986 diff --git a/processed_dataset/images/volume7/figures/fig-c11i8.png b/processed_dataset/images/volume7/figures/fig-c11i8.png new file mode 100644 index 0000000000000000000000000000000000000000..b6e68e4529e261a5d7b0f9e6e10320e5b3e52c4b --- /dev/null +++ b/processed_dataset/images/volume7/figures/fig-c11i8.png @@ -0,0 +1,3 @@ +version https://git-lfs.github.com/spec/v1 +oid sha256:d48fa7e9bc33e6f56c435f8f344db81709598c0ed74f8840cc3d88ca46562c08 +size 36148 diff --git a/processed_dataset/images/volume7/figures/fig-c11i9.png b/processed_dataset/images/volume7/figures/fig-c11i9.png new file mode 100644 index 0000000000000000000000000000000000000000..176d0d4d7a99ae6ff0e0b4e249da40a76a6abd3b --- /dev/null +++ b/processed_dataset/images/volume7/figures/fig-c11i9.png @@ -0,0 +1,3 @@ +version https://git-lfs.github.com/spec/v1 +oid sha256:7bad8bc43b00d28ca5ae97f724e938c28167d1cc766984ed0bf82bfa8d074edd +size 37781 diff --git a/processed_dataset/images/volume7/figures/fig-c11p2.png b/processed_dataset/images/volume7/figures/fig-c11p2.png new file mode 100644 index 0000000000000000000000000000000000000000..48f097f039bb6931b44fd8a7767d04f7a520940e --- /dev/null +++ b/processed_dataset/images/volume7/figures/fig-c11p2.png @@ -0,0 +1,3 @@ +version https://git-lfs.github.com/spec/v1 +oid sha256:14a93b7952a5fd7eabf23469fa06e4076352f0892ef4230d6331ff272684d13e +size 30489 diff --git a/processed_dataset/images/volume7/figures/fig-c11p3.png b/processed_dataset/images/volume7/figures/fig-c11p3.png new file mode 100644 index 0000000000000000000000000000000000000000..ea0b360dc397483e808c32baf753c1db16cd7071 --- /dev/null +++ b/processed_dataset/images/volume7/figures/fig-c11p3.png @@ -0,0 +1,3 @@ +version https://git-lfs.github.com/spec/v1 +oid sha256:378b628ac8dbaae02f1c2459febd321e887aa0c57feef63832067ac856c9a606 +size 17879 diff --git a/processed_dataset/images/volume7/figures/fig-c1a1-1.png b/processed_dataset/images/volume7/figures/fig-c1a1-1.png new file mode 100644 index 0000000000000000000000000000000000000000..e9cc9a05ca9af77e592e68ecbcc6e5da41b7c7a9 --- /dev/null +++ b/processed_dataset/images/volume7/figures/fig-c1a1-1.png @@ -0,0 +1,3 @@ +version https://git-lfs.github.com/spec/v1 +oid sha256:df2868d397307afa3211a4c2656da179aa326a9f2dadb78dcb97aeb2df28279e +size 31261 diff --git a/processed_dataset/images/volume7/figures/fig-c1a1-2.png b/processed_dataset/images/volume7/figures/fig-c1a1-2.png new file mode 100644 index 0000000000000000000000000000000000000000..41bec7830f6760769b75fa354ec826a7909f44d4 --- /dev/null +++ b/processed_dataset/images/volume7/figures/fig-c1a1-2.png @@ -0,0 +1,3 @@ +version https://git-lfs.github.com/spec/v1 +oid sha256:afacef62fe8c153379f2fdaeedb12af333dfcecb46ddc94ca4d34da3413e7ba5 +size 20829 diff --git a/processed_dataset/images/volume7/figures/fig-c1a11.png b/processed_dataset/images/volume7/figures/fig-c1a11.png new file mode 100644 index 0000000000000000000000000000000000000000..57cf41b47f63552ac0f9c93ce4fb97f5973e8e14 --- /dev/null +++ b/processed_dataset/images/volume7/figures/fig-c1a11.png @@ -0,0 +1,3 @@ +version https://git-lfs.github.com/spec/v1 +oid sha256:55995e1fd4b13f015e1092f16304a6873cbf4f1be8bc1f75d91b2a2a0091db1a +size 28035 diff --git a/processed_dataset/images/volume7/figures/fig-c1a12.png b/processed_dataset/images/volume7/figures/fig-c1a12.png new file mode 100644 index 0000000000000000000000000000000000000000..097c6a0ea828e8e45b4137fa60b153c3de096176 --- /dev/null +++ b/processed_dataset/images/volume7/figures/fig-c1a12.png @@ -0,0 +1,3 @@ +version https://git-lfs.github.com/spec/v1 +oid sha256:73673c85d1cf2fe0c59199013ee9590a296a81c3c67601c597243d18df3ddf48 +size 30822 diff --git a/processed_dataset/images/volume7/figures/fig-c1a13.png b/processed_dataset/images/volume7/figures/fig-c1a13.png new file mode 100644 index 0000000000000000000000000000000000000000..5f9f53afd274897c16253750e7dc4660c0cff895 --- /dev/null +++ b/processed_dataset/images/volume7/figures/fig-c1a13.png @@ -0,0 +1,3 @@ +version https://git-lfs.github.com/spec/v1 +oid sha256:eff4927f5fc1c116a646f21e4a33dfb22c0578233b34153b57f18fab75f0f61c +size 22940 diff --git a/processed_dataset/images/volume7/figures/fig-c1a14.png b/processed_dataset/images/volume7/figures/fig-c1a14.png new file mode 100644 index 0000000000000000000000000000000000000000..a6aa21eea8faff18c1822144bdbb6d8b43b47be3 --- /dev/null +++ b/processed_dataset/images/volume7/figures/fig-c1a14.png @@ -0,0 +1,3 @@ +version https://git-lfs.github.com/spec/v1 +oid sha256:8c84366deab0cf650b5e28f4aa2b63e71d218a7ec3753cab26bb63ab21715927 +size 21834 diff --git a/processed_dataset/images/volume7/figures/fig-c1a15.png b/processed_dataset/images/volume7/figures/fig-c1a15.png new file mode 100644 index 0000000000000000000000000000000000000000..a3f96460eb9d79e00bdb93291bfbd7fca93bddba --- /dev/null +++ b/processed_dataset/images/volume7/figures/fig-c1a15.png @@ -0,0 +1,3 @@ +version https://git-lfs.github.com/spec/v1 +oid sha256:52da64f1be8017ec99aadef51211ca4bfefadda4842783de1d5b5b7913aac6e2 +size 39390 diff --git a/processed_dataset/images/volume7/figures/fig-c1a16.png b/processed_dataset/images/volume7/figures/fig-c1a16.png new file mode 100644 index 0000000000000000000000000000000000000000..cd9cea2aecd5782835e7c8b8cb31dc599e4359a5 --- /dev/null +++ b/processed_dataset/images/volume7/figures/fig-c1a16.png @@ -0,0 +1,3 @@ +version https://git-lfs.github.com/spec/v1 +oid sha256:d045c04a40568325a6c0df8d295cf62a8d48d60ac3e49acfd18098ba16578cb1 +size 23106 diff --git a/processed_dataset/images/volume7/figures/fig-c1a17.png b/processed_dataset/images/volume7/figures/fig-c1a17.png new file mode 100644 index 0000000000000000000000000000000000000000..6257f36ab2e5dd40886307a8286f7bd63f3377ca --- /dev/null +++ b/processed_dataset/images/volume7/figures/fig-c1a17.png @@ -0,0 +1,3 @@ +version https://git-lfs.github.com/spec/v1 +oid sha256:805a4ec2a3b90075a31b135f746d11735a5d86e9707a2c46bdeb9f78af404df3 +size 27432 diff --git a/processed_dataset/images/volume7/figures/fig-c1a18.png b/processed_dataset/images/volume7/figures/fig-c1a18.png new file mode 100644 index 0000000000000000000000000000000000000000..e21fc128bc9fa1ebcc968170a33556d1a0b7256e --- /dev/null +++ b/processed_dataset/images/volume7/figures/fig-c1a18.png @@ -0,0 +1,3 @@ +version https://git-lfs.github.com/spec/v1 +oid sha256:35decab244e9df825d7f8efaefe77bd3cca800386cbcc2a88a598b157d64cac5 +size 32131 diff --git a/processed_dataset/images/volume7/figures/fig-c1a2-1.png b/processed_dataset/images/volume7/figures/fig-c1a2-1.png new file mode 100644 index 0000000000000000000000000000000000000000..3cf2a9217efd86925f7e93a61d5e8de5ee520b32 --- /dev/null +++ b/processed_dataset/images/volume7/figures/fig-c1a2-1.png @@ -0,0 +1,3 @@ +version https://git-lfs.github.com/spec/v1 +oid sha256:bfedda4c97e12c1a5bd5877c607d835b1110dbcb6b0be9e79c0895a8bee20c68 +size 26072 diff --git a/processed_dataset/images/volume7/figures/fig-c1a2-2.png b/processed_dataset/images/volume7/figures/fig-c1a2-2.png new file mode 100644 index 0000000000000000000000000000000000000000..535dbaf57f474a0e1c1d1cf7502f07ace660649f --- /dev/null +++ b/processed_dataset/images/volume7/figures/fig-c1a2-2.png @@ -0,0 +1,3 @@ +version https://git-lfs.github.com/spec/v1 +oid sha256:f492abb99116be9dd81b5b13fbc88065fb2fb607fe7dbba90d9946cf875ba4c1 +size 26061 diff --git a/processed_dataset/images/volume7/figures/fig-c1a3.png b/processed_dataset/images/volume7/figures/fig-c1a3.png new file mode 100644 index 0000000000000000000000000000000000000000..ff32f1357d5f9de65d960b67a5901019cfff25e7 --- /dev/null +++ b/processed_dataset/images/volume7/figures/fig-c1a3.png @@ -0,0 +1,3 @@ +version https://git-lfs.github.com/spec/v1 +oid sha256:4ec913c38a320b8d7ab808b0daf1e2f2d71679100a8cc59ae5e5eda7c43072e4 +size 28548 diff --git a/processed_dataset/images/volume7/figures/fig-c1a4.png b/processed_dataset/images/volume7/figures/fig-c1a4.png new file mode 100644 index 0000000000000000000000000000000000000000..0130402faaa68937c622d16fc025d8a6f8698718 --- /dev/null +++ b/processed_dataset/images/volume7/figures/fig-c1a4.png @@ -0,0 +1,3 @@ +version https://git-lfs.github.com/spec/v1 +oid sha256:d0af8ce8145c7f6e8496badc50a4ac8a11ccb27cb50c43a6138a3be81434a80c +size 26519 diff --git a/processed_dataset/images/volume7/figures/fig-c1a5-1.png b/processed_dataset/images/volume7/figures/fig-c1a5-1.png new file mode 100644 index 0000000000000000000000000000000000000000..4da92188e8839e110d538ec7a4d88751ba0c57cd --- /dev/null +++ b/processed_dataset/images/volume7/figures/fig-c1a5-1.png @@ -0,0 +1,3 @@ +version https://git-lfs.github.com/spec/v1 +oid sha256:8febc85278421b5109f60c9af33346a67c17fc040ee3ad6ade4f00c5ed4c3f6d +size 22388 diff --git a/processed_dataset/images/volume7/figures/fig-c1a5-2.png b/processed_dataset/images/volume7/figures/fig-c1a5-2.png new file mode 100644 index 0000000000000000000000000000000000000000..2371e15ef50276f22089adac23a260c1fa94e617 --- /dev/null +++ b/processed_dataset/images/volume7/figures/fig-c1a5-2.png @@ -0,0 +1,3 @@ +version https://git-lfs.github.com/spec/v1 +oid sha256:d2d9fcfd293c39d34d2cd2d1031945b5acece446c97725937bb763278fddc244 +size 22693 diff --git a/processed_dataset/images/volume7/figures/fig-c1a6.png b/processed_dataset/images/volume7/figures/fig-c1a6.png new file mode 100644 index 0000000000000000000000000000000000000000..d7f387c326571f6cb228b7c664412c85023d6c51 --- /dev/null +++ b/processed_dataset/images/volume7/figures/fig-c1a6.png @@ -0,0 +1,3 @@ +version https://git-lfs.github.com/spec/v1 +oid sha256:f1fe6cf5c0d1fb200e01f5cda37ce64230b1b1eb8628b47c86251c210135c9cb +size 25718 diff --git a/processed_dataset/images/volume7/figures/fig-c1a7.png b/processed_dataset/images/volume7/figures/fig-c1a7.png new file mode 100644 index 0000000000000000000000000000000000000000..cc209d9d241ca663314c7a67e41b43bfaf6dbb54 --- /dev/null +++ b/processed_dataset/images/volume7/figures/fig-c1a7.png @@ -0,0 +1,3 @@ +version https://git-lfs.github.com/spec/v1 +oid sha256:f5fb9337e2acaa329e13e7ab84ebae165258cbe31968cb87675d605d56cabfae +size 37285 diff --git a/processed_dataset/images/volume7/figures/fig-c1a8.png b/processed_dataset/images/volume7/figures/fig-c1a8.png new file mode 100644 index 0000000000000000000000000000000000000000..2a25b528f1789365c56b4f6aab0bdb96d6c776c4 --- /dev/null +++ b/processed_dataset/images/volume7/figures/fig-c1a8.png @@ -0,0 +1,3 @@ +version https://git-lfs.github.com/spec/v1 +oid sha256:5071fb0e4c5a50670d0c8c9a38de9f8dbdcb1716fc18e8bff7407c74e489eca6 +size 36318 diff --git a/processed_dataset/images/volume7/figures/fig-c1a9.png b/processed_dataset/images/volume7/figures/fig-c1a9.png new file mode 100644 index 0000000000000000000000000000000000000000..08a9677af726d5792e5be8dd7c01fd6dd7d103df --- /dev/null +++ b/processed_dataset/images/volume7/figures/fig-c1a9.png @@ -0,0 +1,3 @@ +version https://git-lfs.github.com/spec/v1 +oid sha256:246825de526aba890831580fbe223d2095165536c7b8252fb1cb3541baf1a12a +size 28039 diff --git a/processed_dataset/images/volume7/figures/fig-c1i1.png b/processed_dataset/images/volume7/figures/fig-c1i1.png new file mode 100644 index 0000000000000000000000000000000000000000..215335b9a86e816c976dc2280859383819e0b97d --- /dev/null +++ b/processed_dataset/images/volume7/figures/fig-c1i1.png @@ -0,0 +1,3 @@ +version https://git-lfs.github.com/spec/v1 +oid sha256:bbe997db9e96d40be53aceb86f2ec7d8129f2142f559fc6715ad95504b2dbcf3 +size 40896 diff --git a/processed_dataset/images/volume7/figures/fig-c1i10.png b/processed_dataset/images/volume7/figures/fig-c1i10.png new file mode 100644 index 0000000000000000000000000000000000000000..9dd0edfa99011d75deb3ec8c89580a2b8b4271ee --- /dev/null +++ b/processed_dataset/images/volume7/figures/fig-c1i10.png @@ -0,0 +1,3 @@ +version https://git-lfs.github.com/spec/v1 +oid sha256:1de5c5c5c45aa629c6267565fb6159149b2b200fc396043e285f94219cdbab2e +size 50065 diff --git a/processed_dataset/images/volume7/figures/fig-c1i11.png b/processed_dataset/images/volume7/figures/fig-c1i11.png new file mode 100644 index 0000000000000000000000000000000000000000..af1a02c9f176537a7977ebc7b14c483f3e0c75f4 --- /dev/null +++ b/processed_dataset/images/volume7/figures/fig-c1i11.png @@ -0,0 +1,3 @@ +version https://git-lfs.github.com/spec/v1 +oid sha256:ed49aeeca4c257ec79ad6c508dccc1b0cbc6179f3d7545aed28a2e0c8d64a875 +size 24083 diff --git a/processed_dataset/images/volume7/figures/fig-c1i12.png b/processed_dataset/images/volume7/figures/fig-c1i12.png new file mode 100644 index 0000000000000000000000000000000000000000..a3804679fc9f94bfd740979fe6628bae7559a21f --- /dev/null +++ b/processed_dataset/images/volume7/figures/fig-c1i12.png @@ -0,0 +1,3 @@ +version https://git-lfs.github.com/spec/v1 +oid sha256:7bd082b0ea4fdfd9c478c08ee74a7519fbea89c511b1986dba76a6b7aa72509b +size 37616 diff --git a/processed_dataset/images/volume7/figures/fig-c1i13.png b/processed_dataset/images/volume7/figures/fig-c1i13.png new file mode 100644 index 0000000000000000000000000000000000000000..484c4ce7999e4f462877369b195100892773ad06 --- /dev/null +++ b/processed_dataset/images/volume7/figures/fig-c1i13.png @@ -0,0 +1,3 @@ +version https://git-lfs.github.com/spec/v1 +oid sha256:45a753d83d2d105af11c08ceab4ce6f12f3574a5a073e14eda6b79dce0d71440 +size 36568 diff --git a/processed_dataset/images/volume7/figures/fig-c1i14.png b/processed_dataset/images/volume7/figures/fig-c1i14.png new file mode 100644 index 0000000000000000000000000000000000000000..7feddff579cebf3292db520829db460467c3de65 --- /dev/null +++ b/processed_dataset/images/volume7/figures/fig-c1i14.png @@ -0,0 +1,3 @@ +version https://git-lfs.github.com/spec/v1 +oid sha256:0ae15c2ff64630d87f937bcac65304544cce9cbfdea9c811388872a3460c07da +size 37479 diff --git a/processed_dataset/images/volume7/figures/fig-c1i15.png b/processed_dataset/images/volume7/figures/fig-c1i15.png new file mode 100644 index 0000000000000000000000000000000000000000..5e2d2774e978afa45c7ee6d59ca1a752a8d8b822 --- /dev/null +++ b/processed_dataset/images/volume7/figures/fig-c1i15.png @@ -0,0 +1,3 @@ +version https://git-lfs.github.com/spec/v1 +oid sha256:957e7c2383b835de9c1cebf801e1fe4d9259888b4f7f2c50e0a64f98838c878d +size 33654 diff --git a/processed_dataset/images/volume7/figures/fig-c1i16.png b/processed_dataset/images/volume7/figures/fig-c1i16.png new file mode 100644 index 0000000000000000000000000000000000000000..f29bfc2c324b13c5392050f115c981c49b90bcea --- /dev/null +++ b/processed_dataset/images/volume7/figures/fig-c1i16.png @@ -0,0 +1,3 @@ +version https://git-lfs.github.com/spec/v1 +oid sha256:8eeb61cf877ebd065df9c67e524a029cf4c9a91bb07fd5607a10e6db45924b45 +size 33408 diff --git a/processed_dataset/images/volume7/figures/fig-c1i2.png b/processed_dataset/images/volume7/figures/fig-c1i2.png new file mode 100644 index 0000000000000000000000000000000000000000..70c11949a142dba5ce7828af811b0e3459e6bb5e --- /dev/null +++ b/processed_dataset/images/volume7/figures/fig-c1i2.png @@ -0,0 +1,3 @@ +version https://git-lfs.github.com/spec/v1 +oid sha256:31be7a212a189297b20ce0fe38a8f068cd76b00f8a30ecd3c978b8d0999a9fc3 +size 37610 diff --git a/processed_dataset/images/volume7/figures/fig-c1i3.png b/processed_dataset/images/volume7/figures/fig-c1i3.png new file mode 100644 index 0000000000000000000000000000000000000000..b780314311c6af04f8ec8489a82725cbb1f9fd10 --- /dev/null +++ b/processed_dataset/images/volume7/figures/fig-c1i3.png @@ -0,0 +1,3 @@ +version https://git-lfs.github.com/spec/v1 +oid sha256:a712c263045b04df6dff3014fba4127d3f153d4a861376ed45738418ea7910b2 +size 31030 diff --git a/processed_dataset/images/volume7/figures/fig-c1i4.png b/processed_dataset/images/volume7/figures/fig-c1i4.png new file mode 100644 index 0000000000000000000000000000000000000000..6a07ae7e6109077e6356601b8b6b85fe68cad450 --- /dev/null +++ b/processed_dataset/images/volume7/figures/fig-c1i4.png @@ -0,0 +1,3 @@ +version https://git-lfs.github.com/spec/v1 +oid sha256:739a8479ff84d5a635c237d630a9a21f839f52a4cf198551924b1c27184e81ea +size 32498 diff --git a/processed_dataset/images/volume7/figures/fig-c1i5.png b/processed_dataset/images/volume7/figures/fig-c1i5.png new file mode 100644 index 0000000000000000000000000000000000000000..2b2317ebd616854194b7ff4d530dbca3e8933000 --- /dev/null +++ b/processed_dataset/images/volume7/figures/fig-c1i5.png @@ -0,0 +1,3 @@ +version https://git-lfs.github.com/spec/v1 +oid sha256:7131f3dcf24c88a67683b15b4bc7c06218f2f40d4a741a1cd26bb99e968c529e +size 41493 diff --git a/processed_dataset/images/volume7/figures/fig-c1i6.png b/processed_dataset/images/volume7/figures/fig-c1i6.png new file mode 100644 index 0000000000000000000000000000000000000000..6ebde943521ef9a56c1ba5bdfd592953d58b15c4 --- /dev/null +++ b/processed_dataset/images/volume7/figures/fig-c1i6.png @@ -0,0 +1,3 @@ +version https://git-lfs.github.com/spec/v1 +oid sha256:5961ee98d8426eba35be0a5559bfc0c7d033f1924393c8a853903164b2236ab8 +size 33738 diff --git a/processed_dataset/images/volume7/figures/fig-c1i7.png b/processed_dataset/images/volume7/figures/fig-c1i7.png new file mode 100644 index 0000000000000000000000000000000000000000..b11ded7ec11e949f46400f3344dbaca51b09a5ad --- /dev/null +++ b/processed_dataset/images/volume7/figures/fig-c1i7.png @@ -0,0 +1,3 @@ +version https://git-lfs.github.com/spec/v1 +oid sha256:4b9948e2249130a4d5354348c37bdaf0cc2451860451576349e685f44e62a0e7 +size 41092 diff --git a/processed_dataset/images/volume7/figures/fig-c1i8.png b/processed_dataset/images/volume7/figures/fig-c1i8.png new file mode 100644 index 0000000000000000000000000000000000000000..54e6015af5069a47e10b235c3c86012b9ffc73b2 --- /dev/null +++ b/processed_dataset/images/volume7/figures/fig-c1i8.png @@ -0,0 +1,3 @@ +version https://git-lfs.github.com/spec/v1 +oid sha256:5a1e56d2b6e43cedb13f47de7da5fb41f5d3784c68f5bd455156dcad20ebdf0d +size 49205 diff --git a/processed_dataset/images/volume7/figures/fig-c1i9.png b/processed_dataset/images/volume7/figures/fig-c1i9.png new file mode 100644 index 0000000000000000000000000000000000000000..787819515cabf81ef82f9e0cda27c42427744c9b --- /dev/null +++ b/processed_dataset/images/volume7/figures/fig-c1i9.png @@ -0,0 +1,3 @@ +version https://git-lfs.github.com/spec/v1 +oid sha256:0c937648c9e828b0f6cb8668cb8eb4589df191ae53c68707ca500ea6bd7e4018 +size 37620 diff --git a/processed_dataset/images/volume7/figures/fig-c1p10.png b/processed_dataset/images/volume7/figures/fig-c1p10.png new file mode 100644 index 0000000000000000000000000000000000000000..dcb6b9848070dfd3c55a7e236f1b98cf13bbba99 --- /dev/null +++ b/processed_dataset/images/volume7/figures/fig-c1p10.png @@ -0,0 +1,3 @@ +version https://git-lfs.github.com/spec/v1 +oid sha256:55f47b148713f6e87961cb8719b34a45f65a3996a8d4d4b6d2be7107d3ba1b6d +size 21164 diff --git a/processed_dataset/images/volume7/figures/fig-c2a1.png b/processed_dataset/images/volume7/figures/fig-c2a1.png new file mode 100644 index 0000000000000000000000000000000000000000..a4c50d631d02c827d24472bacc3a664042462183 --- /dev/null +++ b/processed_dataset/images/volume7/figures/fig-c2a1.png @@ -0,0 +1,3 @@ +version https://git-lfs.github.com/spec/v1 +oid sha256:1d834dcb0c813580385536bce6ae9d19cbe4186777d41c72e8d2993461c4d015 +size 23643 diff --git a/processed_dataset/images/volume7/figures/fig-c2a10.png b/processed_dataset/images/volume7/figures/fig-c2a10.png new file mode 100644 index 0000000000000000000000000000000000000000..8928e47ebd17efd5f04f810db35fdf239e860442 --- /dev/null +++ b/processed_dataset/images/volume7/figures/fig-c2a10.png @@ -0,0 +1,3 @@ +version https://git-lfs.github.com/spec/v1 +oid sha256:2d79c396e99a5b93dd9cd1a0b7aba281bd6d0402e9574dab7951937402fe7a61 +size 31285 diff --git a/processed_dataset/images/volume7/figures/fig-c2a11.png b/processed_dataset/images/volume7/figures/fig-c2a11.png new file mode 100644 index 0000000000000000000000000000000000000000..34937513ee465c2a07a288dacbf60b3855340352 --- /dev/null +++ b/processed_dataset/images/volume7/figures/fig-c2a11.png @@ -0,0 +1,3 @@ +version https://git-lfs.github.com/spec/v1 +oid sha256:fc9899895823ffb9ecb9077dafd43582ef671253e794350ff3229fd737bda4ea +size 37747 diff --git a/processed_dataset/images/volume7/figures/fig-c2a12.png b/processed_dataset/images/volume7/figures/fig-c2a12.png new file mode 100644 index 0000000000000000000000000000000000000000..db5d8e13dd15a15fc743e8c31be1c4bd78813fbc --- /dev/null +++ b/processed_dataset/images/volume7/figures/fig-c2a12.png @@ -0,0 +1,3 @@ +version https://git-lfs.github.com/spec/v1 +oid sha256:5d8c89dca7fd0e8cca8a9a9fec96ecfcbb073432e523fb91e13240c20cb8e825 +size 32680 diff --git a/processed_dataset/images/volume7/figures/fig-c2a13.png b/processed_dataset/images/volume7/figures/fig-c2a13.png new file mode 100644 index 0000000000000000000000000000000000000000..e6a1497f1d1fc85005a33c52835bce016ae8bd61 --- /dev/null +++ b/processed_dataset/images/volume7/figures/fig-c2a13.png @@ -0,0 +1,3 @@ +version https://git-lfs.github.com/spec/v1 +oid sha256:74383971b2b60cb8c759231445a88f7b014d83f76252855049ee948166ffc07f +size 36640 diff --git a/processed_dataset/images/volume7/figures/fig-c2a14.png b/processed_dataset/images/volume7/figures/fig-c2a14.png new file mode 100644 index 0000000000000000000000000000000000000000..10710a4dbe6c8a62bcfd576c41e7e39f4567ce97 --- /dev/null +++ b/processed_dataset/images/volume7/figures/fig-c2a14.png @@ -0,0 +1,3 @@ +version https://git-lfs.github.com/spec/v1 +oid sha256:4825a157c95f9823be9943948df33ca35b2e08e033bff8649614efab41aa766a +size 23515 diff --git a/processed_dataset/images/volume7/figures/fig-c2a15.png b/processed_dataset/images/volume7/figures/fig-c2a15.png new file mode 100644 index 0000000000000000000000000000000000000000..978c68ef20b589083431a5a0b417ba6e2801d4e0 --- /dev/null +++ b/processed_dataset/images/volume7/figures/fig-c2a15.png @@ -0,0 +1,3 @@ +version https://git-lfs.github.com/spec/v1 +oid sha256:908a09e800a5bc7f03b51073e3e8a1da1570ca8542453038d89d7711bcec22e6 +size 22234 diff --git a/processed_dataset/images/volume7/figures/fig-c2a16.png b/processed_dataset/images/volume7/figures/fig-c2a16.png new file mode 100644 index 0000000000000000000000000000000000000000..468aa6473eb9182c3f5b067ffa05b8f05d87fcc9 --- /dev/null +++ b/processed_dataset/images/volume7/figures/fig-c2a16.png @@ -0,0 +1,3 @@ +version https://git-lfs.github.com/spec/v1 +oid sha256:213d944782a220669f1ffd5b922376b89f40abd572438005339859de2bdd75e3 +size 38028 diff --git a/processed_dataset/images/volume7/figures/fig-c2a17.png b/processed_dataset/images/volume7/figures/fig-c2a17.png new file mode 100644 index 0000000000000000000000000000000000000000..115ae586eace489dc9468e20a3ee1f58f0105b7d --- /dev/null +++ b/processed_dataset/images/volume7/figures/fig-c2a17.png @@ -0,0 +1,3 @@ +version https://git-lfs.github.com/spec/v1 +oid sha256:ef6a74836ae32de7fe58c351721ad230c65af0eace4c590d8df56c96f86f8e8a +size 35848 diff --git a/processed_dataset/images/volume7/figures/fig-c2a2.png b/processed_dataset/images/volume7/figures/fig-c2a2.png new file mode 100644 index 0000000000000000000000000000000000000000..1d2ceb0495c9dcf00e4f4cf356a87afeb825f738 --- /dev/null +++ b/processed_dataset/images/volume7/figures/fig-c2a2.png @@ -0,0 +1,3 @@ +version https://git-lfs.github.com/spec/v1 +oid sha256:fdcc25feda6bbb751b5bc6bd0b611c256d45105e5012c1c9ae55658d5dac04b7 +size 27709 diff --git a/processed_dataset/images/volume7/figures/fig-c2a3.png b/processed_dataset/images/volume7/figures/fig-c2a3.png new file mode 100644 index 0000000000000000000000000000000000000000..0abce5cfbc1066b2208df29f17024c57cba806cd --- /dev/null +++ b/processed_dataset/images/volume7/figures/fig-c2a3.png @@ -0,0 +1,3 @@ +version https://git-lfs.github.com/spec/v1 +oid sha256:c4929d335ba6ee61c1f5995783e1892af7a3248dcab9425bdfb8d1c8e3f3b42b +size 31547 diff --git a/processed_dataset/images/volume7/figures/fig-c2a4.png b/processed_dataset/images/volume7/figures/fig-c2a4.png new file mode 100644 index 0000000000000000000000000000000000000000..532e9419b748b7213a82d0b077fbfb65921339c5 --- /dev/null +++ b/processed_dataset/images/volume7/figures/fig-c2a4.png @@ -0,0 +1,3 @@ +version https://git-lfs.github.com/spec/v1 +oid sha256:71f5b6e4465321bdf3f224a09d0900938ae9ac7fcfe05082ea34149ea524a2ec +size 21698 diff --git a/processed_dataset/images/volume7/figures/fig-c2a5.png b/processed_dataset/images/volume7/figures/fig-c2a5.png new file mode 100644 index 0000000000000000000000000000000000000000..ee68839cd626c9c92c91bc17bcfff3f335c54df8 --- /dev/null +++ b/processed_dataset/images/volume7/figures/fig-c2a5.png @@ -0,0 +1,3 @@ +version https://git-lfs.github.com/spec/v1 +oid sha256:ff4f9b85279954617a0fa858e2138a2217254e2c5d4bd1dde3fdd3f2e09d9804 +size 31356 diff --git a/processed_dataset/images/volume7/figures/fig-c2a6.png b/processed_dataset/images/volume7/figures/fig-c2a6.png new file mode 100644 index 0000000000000000000000000000000000000000..690bbfeba2d1196fb8d21f1d6bd3c3ad7e957755 --- /dev/null +++ b/processed_dataset/images/volume7/figures/fig-c2a6.png @@ -0,0 +1,3 @@ +version https://git-lfs.github.com/spec/v1 +oid sha256:f7b1daca12a13a2bf0dc24a4d11210dfc7e7c1dcd8c038e408dc6e4e47bb963f +size 28698 diff --git a/processed_dataset/images/volume7/figures/fig-c2a8.png b/processed_dataset/images/volume7/figures/fig-c2a8.png new file mode 100644 index 0000000000000000000000000000000000000000..83837648da954ef367ba38b91fa1af7444c6c2fa --- /dev/null +++ b/processed_dataset/images/volume7/figures/fig-c2a8.png @@ -0,0 +1,3 @@ +version https://git-lfs.github.com/spec/v1 +oid sha256:0fcc2c2f8e6d6a2dde8a1114fa7b7cb666c43b494ddfab2badb4e3765b6f6e70 +size 38717 diff --git a/processed_dataset/images/volume7/figures/fig-c2a9.png b/processed_dataset/images/volume7/figures/fig-c2a9.png new file mode 100644 index 0000000000000000000000000000000000000000..bdeb6520527ebad660fa593e2cfa332b057fd06d --- /dev/null +++ b/processed_dataset/images/volume7/figures/fig-c2a9.png @@ -0,0 +1,3 @@ +version https://git-lfs.github.com/spec/v1 +oid sha256:f1dc332f6e43018fa96cbd4a10ceb91537a720159cb2606fbfb1ef6f7938b6be +size 23180 diff --git a/processed_dataset/images/volume7/figures/fig-c2i1.png b/processed_dataset/images/volume7/figures/fig-c2i1.png new file mode 100644 index 0000000000000000000000000000000000000000..a7793c9346a5fa046d2fe6240e69d5f92f06748b --- /dev/null +++ b/processed_dataset/images/volume7/figures/fig-c2i1.png @@ -0,0 +1,3 @@ +version https://git-lfs.github.com/spec/v1 +oid sha256:21047c69b433ca2bd6802e64af33e01ae88e5258bd0a74ff671e92fb57d8bdeb +size 24170 diff --git a/processed_dataset/images/volume7/figures/fig-c2i10.png b/processed_dataset/images/volume7/figures/fig-c2i10.png new file mode 100644 index 0000000000000000000000000000000000000000..b08365edcd2b010aae5397b1ce297581f3d60e7a --- /dev/null +++ b/processed_dataset/images/volume7/figures/fig-c2i10.png @@ -0,0 +1,3 @@ +version https://git-lfs.github.com/spec/v1 +oid sha256:e2fe1d3a96e5674868a08b1a7d1dea0510579d795d3bbee661025dec0842871c +size 24561 diff --git a/processed_dataset/images/volume7/figures/fig-c2i11.png b/processed_dataset/images/volume7/figures/fig-c2i11.png new file mode 100644 index 0000000000000000000000000000000000000000..d977d0ecee82c6eeb0da3d60a7efbc9c8172d74b --- /dev/null +++ b/processed_dataset/images/volume7/figures/fig-c2i11.png @@ -0,0 +1,3 @@ +version https://git-lfs.github.com/spec/v1 +oid sha256:3a5ec8d244e01ac5c7c76ffbd1229af1b5ea9b7e5b9ae9ef50ca1e1f2978d919 +size 34443 diff --git a/processed_dataset/images/volume7/figures/fig-c2i12.png b/processed_dataset/images/volume7/figures/fig-c2i12.png new file mode 100644 index 0000000000000000000000000000000000000000..dba6686132b584c5db4d5c30ecfeb57cef95cdbe --- /dev/null +++ b/processed_dataset/images/volume7/figures/fig-c2i12.png @@ -0,0 +1,3 @@ +version https://git-lfs.github.com/spec/v1 +oid sha256:4be79610be463e85051b7fafc4fd0ba73d8a8ffd9df22e9eec973831acf7e1b5 +size 28760 diff --git a/processed_dataset/images/volume7/figures/fig-c2i13.png b/processed_dataset/images/volume7/figures/fig-c2i13.png new file mode 100644 index 0000000000000000000000000000000000000000..dd174f70a6a0f87e249a90d9e8fc16afb7f1a72d --- /dev/null +++ b/processed_dataset/images/volume7/figures/fig-c2i13.png @@ -0,0 +1,3 @@ +version https://git-lfs.github.com/spec/v1 +oid sha256:65ec7f7f989c5b0a57cc6d1babf82cc97b42c321081ddac28f4d0d5a2e09f19f +size 49717 diff --git a/processed_dataset/images/volume7/figures/fig-c2i14.png b/processed_dataset/images/volume7/figures/fig-c2i14.png new file mode 100644 index 0000000000000000000000000000000000000000..130ed5a67ecceb788cda748e243fd7339e7b1699 --- /dev/null +++ b/processed_dataset/images/volume7/figures/fig-c2i14.png @@ -0,0 +1,3 @@ +version https://git-lfs.github.com/spec/v1 +oid sha256:0346b616d02f2fb1bff234f3a1076f8d7ca04bddf8989392b664f98b1061469f +size 42275 diff --git a/processed_dataset/images/volume7/figures/fig-c2i15.png b/processed_dataset/images/volume7/figures/fig-c2i15.png new file mode 100644 index 0000000000000000000000000000000000000000..f0b85a2469b67bb19fe5362a49885013df27feb9 --- /dev/null +++ b/processed_dataset/images/volume7/figures/fig-c2i15.png @@ -0,0 +1,3 @@ +version https://git-lfs.github.com/spec/v1 +oid sha256:b3ec3e8e8f21762130fea7756e212c581724eabd6199aaea878251fcb8c6f120 +size 21028 diff --git a/processed_dataset/images/volume7/figures/fig-c2i16.png b/processed_dataset/images/volume7/figures/fig-c2i16.png new file mode 100644 index 0000000000000000000000000000000000000000..ac48243815aeb6c276b36e2706be905753b28557 --- /dev/null +++ b/processed_dataset/images/volume7/figures/fig-c2i16.png @@ -0,0 +1,3 @@ +version https://git-lfs.github.com/spec/v1 +oid sha256:cbab899b0d82255a17d48dedcfb97ea218b55355d03b4094a691c448a2370f6f +size 32356 diff --git a/processed_dataset/images/volume7/figures/fig-c2i2.png b/processed_dataset/images/volume7/figures/fig-c2i2.png new file mode 100644 index 0000000000000000000000000000000000000000..49e30ea51f2562a955cf46c8c0c4a50ab5701a49 --- /dev/null +++ b/processed_dataset/images/volume7/figures/fig-c2i2.png @@ -0,0 +1,3 @@ +version https://git-lfs.github.com/spec/v1 +oid sha256:1b6b993b1e1b109c7c50b76e2d973e96d4e189f199777ba7198ace65739cc75b +size 24102 diff --git a/processed_dataset/images/volume7/figures/fig-c2i3.png b/processed_dataset/images/volume7/figures/fig-c2i3.png new file mode 100644 index 0000000000000000000000000000000000000000..b2c58ff48d3da0250e12415a56a97d3b9e195f34 --- /dev/null +++ b/processed_dataset/images/volume7/figures/fig-c2i3.png @@ -0,0 +1,3 @@ +version https://git-lfs.github.com/spec/v1 +oid sha256:bbeb84011291bfaf43b06e1310ae8940571a837474b67a78dfdcaa50ac957b49 +size 25269 diff --git a/processed_dataset/images/volume7/figures/fig-c2i4.png b/processed_dataset/images/volume7/figures/fig-c2i4.png new file mode 100644 index 0000000000000000000000000000000000000000..58f48ab4dfc9b34c26fe3b41fe283305f6a5b608 --- /dev/null +++ b/processed_dataset/images/volume7/figures/fig-c2i4.png @@ -0,0 +1,3 @@ +version https://git-lfs.github.com/spec/v1 +oid sha256:f2adaa84c972d6b27083699b7525a3ffeb41b8c1d4e122c0090623a367699da7 +size 54107 diff --git a/processed_dataset/images/volume7/figures/fig-c2i5.png b/processed_dataset/images/volume7/figures/fig-c2i5.png new file mode 100644 index 0000000000000000000000000000000000000000..f9e373c2f219bc98afbd186f95ff4761836761d3 --- /dev/null +++ b/processed_dataset/images/volume7/figures/fig-c2i5.png @@ -0,0 +1,3 @@ +version https://git-lfs.github.com/spec/v1 +oid sha256:1c2fcbda1f7c81a82dfca8dfe50e7a6367d39fb7a97ef01cdbd9f2ce3a88de3e +size 20578 diff --git a/processed_dataset/images/volume7/figures/fig-c2i6.png b/processed_dataset/images/volume7/figures/fig-c2i6.png new file mode 100644 index 0000000000000000000000000000000000000000..4cf589247aae497a9723f6b54dcf3eff12b8fd01 --- /dev/null +++ b/processed_dataset/images/volume7/figures/fig-c2i6.png @@ -0,0 +1,3 @@ +version https://git-lfs.github.com/spec/v1 +oid sha256:fec39c5811aad740576fea46cbf1b4474ab250f0e1c1869a5e1f6e200e62ef90 +size 34222 diff --git a/processed_dataset/images/volume7/figures/fig-c2i7.png b/processed_dataset/images/volume7/figures/fig-c2i7.png new file mode 100644 index 0000000000000000000000000000000000000000..724e504627022cf7e4c8aa5325e46a86b307c146 --- /dev/null +++ b/processed_dataset/images/volume7/figures/fig-c2i7.png @@ -0,0 +1,3 @@ +version https://git-lfs.github.com/spec/v1 +oid sha256:98aa7662d4163e5af90b3bd28b44f0d7c0ba68ff0b660451e27b63fa5447b54f +size 31190 diff --git a/processed_dataset/images/volume7/figures/fig-c2i8.png b/processed_dataset/images/volume7/figures/fig-c2i8.png new file mode 100644 index 0000000000000000000000000000000000000000..f3215012c5c31a892cd3a2e68e4d094e863d889e --- /dev/null +++ b/processed_dataset/images/volume7/figures/fig-c2i8.png @@ -0,0 +1,3 @@ +version https://git-lfs.github.com/spec/v1 +oid sha256:754a8b525b310e0a8324362d3df601454e9e099d9ada0b5159369f328b3c95bc +size 31069 diff --git a/processed_dataset/images/volume7/figures/fig-c2i9.png b/processed_dataset/images/volume7/figures/fig-c2i9.png new file mode 100644 index 0000000000000000000000000000000000000000..bfc3b446500f4a43667bd72683f7f842b23a0cd6 --- /dev/null +++ b/processed_dataset/images/volume7/figures/fig-c2i9.png @@ -0,0 +1,3 @@ +version https://git-lfs.github.com/spec/v1 +oid sha256:f68528119449752a6eecfd94a8a947f58f4283becab7527012c1a13b6e93e1d8 +size 17669 diff --git a/processed_dataset/images/volume7/figures/fig-c3a10.png b/processed_dataset/images/volume7/figures/fig-c3a10.png new file mode 100644 index 0000000000000000000000000000000000000000..df425aa8cfb7d9d4c3f00a9da7f51ae699847b4a --- /dev/null +++ b/processed_dataset/images/volume7/figures/fig-c3a10.png @@ -0,0 +1,3 @@ +version https://git-lfs.github.com/spec/v1 +oid sha256:4ee292ffb391e40a53fdf52c24954c9484194934940c68749afb8626ab359f40 +size 36220 diff --git a/processed_dataset/images/volume7/figures/fig-c3a11.png b/processed_dataset/images/volume7/figures/fig-c3a11.png new file mode 100644 index 0000000000000000000000000000000000000000..18b7185ffb71b9b512c8c5e78fdc2e6911a91f61 --- /dev/null +++ b/processed_dataset/images/volume7/figures/fig-c3a11.png @@ -0,0 +1,3 @@ +version https://git-lfs.github.com/spec/v1 +oid sha256:eb8e2f723351bad4f51e43ae22db3f5f9f41f9e442359618cc77fb66a1915b37 +size 34830 diff --git a/processed_dataset/images/volume7/figures/fig-c3a12.png b/processed_dataset/images/volume7/figures/fig-c3a12.png new file mode 100644 index 0000000000000000000000000000000000000000..41bf23153c57f97e44e33cf4f9e99944925d6f51 --- /dev/null +++ b/processed_dataset/images/volume7/figures/fig-c3a12.png @@ -0,0 +1,3 @@ +version https://git-lfs.github.com/spec/v1 +oid sha256:39c264c0cb3e24cdfe38e159bdf6d7984f0ca7af8c5b5a7b5260f2d730e768ad +size 28092 diff --git a/processed_dataset/images/volume7/figures/fig-c3a13.png b/processed_dataset/images/volume7/figures/fig-c3a13.png new file mode 100644 index 0000000000000000000000000000000000000000..41c7f4a339dcd3bb502bc4d10069f6c05c6f45f7 --- /dev/null +++ b/processed_dataset/images/volume7/figures/fig-c3a13.png @@ -0,0 +1,3 @@ +version https://git-lfs.github.com/spec/v1 +oid sha256:059c86f1f2865b97a7bd4463ab162b0c9656805c0221915c9f75bd09ccc09bbb +size 34863 diff --git a/processed_dataset/images/volume7/figures/fig-c3a14.png b/processed_dataset/images/volume7/figures/fig-c3a14.png new file mode 100644 index 0000000000000000000000000000000000000000..025faae1aee9a6077cf368673655105fea9c2612 --- /dev/null +++ b/processed_dataset/images/volume7/figures/fig-c3a14.png @@ -0,0 +1,3 @@ +version https://git-lfs.github.com/spec/v1 +oid sha256:eea24091d7a60e98e20bcb1a9e9472b1a07d082e147283a823220f02e32ef12e +size 39128 diff --git a/processed_dataset/images/volume7/figures/fig-c3a15.png b/processed_dataset/images/volume7/figures/fig-c3a15.png new file mode 100644 index 0000000000000000000000000000000000000000..bded8556cfd2dd0769dfa9bab86ae3084a76388f --- /dev/null +++ b/processed_dataset/images/volume7/figures/fig-c3a15.png @@ -0,0 +1,3 @@ +version https://git-lfs.github.com/spec/v1 +oid sha256:2f79f7bcc09312973083f8b62dccd1a7ee9341c312552899c3f995dd533bbaaf +size 27937 diff --git a/processed_dataset/images/volume7/figures/fig-c3a16.png b/processed_dataset/images/volume7/figures/fig-c3a16.png new file mode 100644 index 0000000000000000000000000000000000000000..fe17a4c3be5ce4946e9ca3e997766483feb503c7 --- /dev/null +++ b/processed_dataset/images/volume7/figures/fig-c3a16.png @@ -0,0 +1,3 @@ +version https://git-lfs.github.com/spec/v1 +oid sha256:256f66b299627bbf7d0d5e514822981886cda218e64197b373f01180122f356a +size 29416 diff --git a/processed_dataset/images/volume7/figures/fig-c3a3.png b/processed_dataset/images/volume7/figures/fig-c3a3.png new file mode 100644 index 0000000000000000000000000000000000000000..5b1220a996f87f15210a54507f54f843d3766802 --- /dev/null +++ b/processed_dataset/images/volume7/figures/fig-c3a3.png @@ -0,0 +1,3 @@ +version https://git-lfs.github.com/spec/v1 +oid sha256:1c7b11814bceea597ef05385af3382c7397ade9c556191c7d6ced73d73476bac +size 37874 diff --git a/processed_dataset/images/volume7/figures/fig-c3a4.png b/processed_dataset/images/volume7/figures/fig-c3a4.png new file mode 100644 index 0000000000000000000000000000000000000000..05bc8cdeafbb8f897bb4e1587e06db67becffe4d --- /dev/null +++ b/processed_dataset/images/volume7/figures/fig-c3a4.png @@ -0,0 +1,3 @@ +version https://git-lfs.github.com/spec/v1 +oid sha256:67d0772898bc8501e76d387ce9d8b678932daa239ba835a3b1fc0a45267ae697 +size 39910 diff --git a/processed_dataset/images/volume7/figures/fig-c3a5.png b/processed_dataset/images/volume7/figures/fig-c3a5.png new file mode 100644 index 0000000000000000000000000000000000000000..5238e7e5dd37e997a4f790c6af1292324e7e4f5c --- /dev/null +++ b/processed_dataset/images/volume7/figures/fig-c3a5.png @@ -0,0 +1,3 @@ +version https://git-lfs.github.com/spec/v1 +oid sha256:2608f2382e0d3e6df75e9ba1060831603c68c9b1b451ad9bd02fef20984bce98 +size 35131 diff --git a/processed_dataset/images/volume7/figures/fig-c3a6.png b/processed_dataset/images/volume7/figures/fig-c3a6.png new file mode 100644 index 0000000000000000000000000000000000000000..71cea0373bc352cc49bc3f837ced7b3e61049d7e --- /dev/null +++ b/processed_dataset/images/volume7/figures/fig-c3a6.png @@ -0,0 +1,3 @@ +version https://git-lfs.github.com/spec/v1 +oid sha256:86207e365ae55f1bf1f9bcf1aeab263424c955a6a3dcb49c42dadbc35bb9c669 +size 34787 diff --git a/processed_dataset/images/volume7/figures/fig-c3a7.png b/processed_dataset/images/volume7/figures/fig-c3a7.png new file mode 100644 index 0000000000000000000000000000000000000000..1533a50606a501129a77d1af13fcb5e88bab6d4f --- /dev/null +++ b/processed_dataset/images/volume7/figures/fig-c3a7.png @@ -0,0 +1,3 @@ +version https://git-lfs.github.com/spec/v1 +oid sha256:7cec3860505177125ce7a793f21d30ea616665fd302d46f6101c013096a3141a +size 34980 diff --git a/processed_dataset/images/volume7/figures/fig-c3a8.png b/processed_dataset/images/volume7/figures/fig-c3a8.png new file mode 100644 index 0000000000000000000000000000000000000000..a953d9812d515351f898dedc8c980216f50ddd52 --- /dev/null +++ b/processed_dataset/images/volume7/figures/fig-c3a8.png @@ -0,0 +1,3 @@ +version https://git-lfs.github.com/spec/v1 +oid sha256:a8c239a5abe37b9935cabe5e6d2447031fc71e2d32bdc9840904a6cf8f57698a +size 24676 diff --git a/processed_dataset/images/volume7/figures/fig-c3a9.png b/processed_dataset/images/volume7/figures/fig-c3a9.png new file mode 100644 index 0000000000000000000000000000000000000000..5f06c712a70a9d14e5b18a14f517878fb3f863b6 --- /dev/null +++ b/processed_dataset/images/volume7/figures/fig-c3a9.png @@ -0,0 +1,3 @@ +version https://git-lfs.github.com/spec/v1 +oid sha256:6468385845efe846a6ada41795fe515b0bb78f0d51276b7a91c075aa71ab02fb +size 32024 diff --git a/processed_dataset/images/volume7/figures/fig-c3i1.png b/processed_dataset/images/volume7/figures/fig-c3i1.png new file mode 100644 index 0000000000000000000000000000000000000000..9fd5d806396ebe73c7bb09a8930936dc1d39426d --- /dev/null +++ b/processed_dataset/images/volume7/figures/fig-c3i1.png @@ -0,0 +1,3 @@ +version https://git-lfs.github.com/spec/v1 +oid sha256:4f966f1b7b58d4a35767fca3b6404392b29830054a8362345cb30c3d59c1d120 +size 24342 diff --git a/processed_dataset/images/volume7/figures/fig-c3i10.png b/processed_dataset/images/volume7/figures/fig-c3i10.png new file mode 100644 index 0000000000000000000000000000000000000000..ae213aaf61e267b3f8a0bf43e7c2c24cc541318a --- /dev/null +++ b/processed_dataset/images/volume7/figures/fig-c3i10.png @@ -0,0 +1,3 @@ +version https://git-lfs.github.com/spec/v1 +oid sha256:b5c9262b80caf96edbdb70ada13639d8d79282621ca46d80c498eb29274891f1 +size 35898 diff --git a/processed_dataset/images/volume7/figures/fig-c3i11.png b/processed_dataset/images/volume7/figures/fig-c3i11.png new file mode 100644 index 0000000000000000000000000000000000000000..2ec7169baba92a280424e1f65b62094b64c749a0 --- /dev/null +++ b/processed_dataset/images/volume7/figures/fig-c3i11.png @@ -0,0 +1,3 @@ +version https://git-lfs.github.com/spec/v1 +oid sha256:630c7f7e72c80ce4a19cd1757a69e96d12045f351972b64d3111d19815d55e68 +size 27242 diff --git a/processed_dataset/images/volume7/figures/fig-c3i12.png b/processed_dataset/images/volume7/figures/fig-c3i12.png new file mode 100644 index 0000000000000000000000000000000000000000..2994a9696e7f855e2ce63c223cea5ce9b8f85907 --- /dev/null +++ b/processed_dataset/images/volume7/figures/fig-c3i12.png @@ -0,0 +1,3 @@ +version https://git-lfs.github.com/spec/v1 +oid sha256:2d28e603fdd15a9cd9f97942accd42d652c6cc1d4b42306efc9cadc13c8fa2a6 +size 40673 diff --git a/processed_dataset/images/volume7/figures/fig-c3i13.png b/processed_dataset/images/volume7/figures/fig-c3i13.png new file mode 100644 index 0000000000000000000000000000000000000000..0285ebb24dd0da61ecfe6b52bf6dd5a7e2d6a565 --- /dev/null +++ b/processed_dataset/images/volume7/figures/fig-c3i13.png @@ -0,0 +1,3 @@ +version https://git-lfs.github.com/spec/v1 +oid sha256:c4d558f2d8f688ae8085af3093b1b74b731d70b0766de95569b42e2d74643ea0 +size 28207 diff --git a/processed_dataset/images/volume7/figures/fig-c3i2.png b/processed_dataset/images/volume7/figures/fig-c3i2.png new file mode 100644 index 0000000000000000000000000000000000000000..d3e76968bc112b83a1cdae7a4194d8d58c83efcf --- /dev/null +++ b/processed_dataset/images/volume7/figures/fig-c3i2.png @@ -0,0 +1,3 @@ +version https://git-lfs.github.com/spec/v1 +oid sha256:d18508d0ac2fca79e3adc668d6a139b03464666b69b9a93c48dcec688a859d7d +size 25253 diff --git a/processed_dataset/images/volume7/figures/fig-c3i3.png b/processed_dataset/images/volume7/figures/fig-c3i3.png new file mode 100644 index 0000000000000000000000000000000000000000..c3c0734e4aab704b70330ccef0c0612825828e0c --- /dev/null +++ b/processed_dataset/images/volume7/figures/fig-c3i3.png @@ -0,0 +1,3 @@ +version https://git-lfs.github.com/spec/v1 +oid sha256:fd9cdc0b4b86d927458b2327c154e31e2f33204244dfad1a783038c4eb9eb4a0 +size 29254 diff --git a/processed_dataset/images/volume7/figures/fig-c3i4.png b/processed_dataset/images/volume7/figures/fig-c3i4.png new file mode 100644 index 0000000000000000000000000000000000000000..7f9cdbd4631e1c61f415371b6a6c3426357f4a83 --- /dev/null +++ b/processed_dataset/images/volume7/figures/fig-c3i4.png @@ -0,0 +1,3 @@ +version https://git-lfs.github.com/spec/v1 +oid sha256:a12c61c8ae759e7215f4882139b0c6752472a51f8d0b10163675347798b48177 +size 33525 diff --git a/processed_dataset/images/volume7/figures/fig-c3i5.png b/processed_dataset/images/volume7/figures/fig-c3i5.png new file mode 100644 index 0000000000000000000000000000000000000000..91af83eb45120d540408d6a94e75d6d7d25f22cd --- /dev/null +++ b/processed_dataset/images/volume7/figures/fig-c3i5.png @@ -0,0 +1,3 @@ +version https://git-lfs.github.com/spec/v1 +oid sha256:c6ef48aa739a5da7844721c1e328d4e9a83da713408f6da996a8a6d6cdf06138 +size 22687 diff --git a/processed_dataset/images/volume7/figures/fig-c3i6.png b/processed_dataset/images/volume7/figures/fig-c3i6.png new file mode 100644 index 0000000000000000000000000000000000000000..af348b323ffbf4980eb7f99f60b885f94247b98e --- /dev/null +++ b/processed_dataset/images/volume7/figures/fig-c3i6.png @@ -0,0 +1,3 @@ +version https://git-lfs.github.com/spec/v1 +oid sha256:f190f8dd779dcf4b6f435cbe1e5c45ddab05d8564bff561cbd66ccaebbc06b73 +size 29545 diff --git a/processed_dataset/images/volume7/figures/fig-c3i7.png b/processed_dataset/images/volume7/figures/fig-c3i7.png new file mode 100644 index 0000000000000000000000000000000000000000..e4fcb636d14736c88d3f38baacfb80ddeb342793 --- /dev/null +++ b/processed_dataset/images/volume7/figures/fig-c3i7.png @@ -0,0 +1,3 @@ +version https://git-lfs.github.com/spec/v1 +oid sha256:8493d6a51f106158a8b9f67e079c014671fbf0ce5ac3b57a1bef2b8a14fe15d6 +size 32836 diff --git a/processed_dataset/images/volume7/figures/fig-c3i8.png b/processed_dataset/images/volume7/figures/fig-c3i8.png new file mode 100644 index 0000000000000000000000000000000000000000..efa173611aff5ab05eb45c23c200c3b07b2e0a6c --- /dev/null +++ b/processed_dataset/images/volume7/figures/fig-c3i8.png @@ -0,0 +1,3 @@ +version https://git-lfs.github.com/spec/v1 +oid sha256:8862d0f4967075a0ed585ebdf5811a19abfeb6d239d907c8f51b1e9a1a8da25d +size 35931 diff --git a/processed_dataset/images/volume7/figures/fig-c3i9.png b/processed_dataset/images/volume7/figures/fig-c3i9.png new file mode 100644 index 0000000000000000000000000000000000000000..644b077dbac6d77f543665c8ba84a3cb404c2ccd --- /dev/null +++ b/processed_dataset/images/volume7/figures/fig-c3i9.png @@ -0,0 +1,3 @@ +version https://git-lfs.github.com/spec/v1 +oid sha256:c07263d1e27afe5a5702d8a0fd7845b89b1b7c54cd9227b13d85e81965ab02b0 +size 32956 diff --git a/processed_dataset/images/volume7/figures/fig-c3p1.png b/processed_dataset/images/volume7/figures/fig-c3p1.png new file mode 100644 index 0000000000000000000000000000000000000000..7fadea3831040b7991108760e1c016bddef891c6 --- /dev/null +++ b/processed_dataset/images/volume7/figures/fig-c3p1.png @@ -0,0 +1,3 @@ +version https://git-lfs.github.com/spec/v1 +oid sha256:dbdab3b8fee8417c2a823e51969ae1fabb8c32711fc4f54abcf3df8b0ef59b05 +size 24587 diff --git a/processed_dataset/images/volume7/figures/fig-c3p10.png b/processed_dataset/images/volume7/figures/fig-c3p10.png new file mode 100644 index 0000000000000000000000000000000000000000..7e38fdfa5aaa5a99ca89e937bf47d5b182327e70 --- /dev/null +++ b/processed_dataset/images/volume7/figures/fig-c3p10.png @@ -0,0 +1,3 @@ +version https://git-lfs.github.com/spec/v1 +oid sha256:aeedb98bf3a85447e0b1ba6cb1146ba0510cfdb4cd4273130bd469c71a655a7b +size 30279 diff --git a/processed_dataset/images/volume7/figures/fig-c3p12.png b/processed_dataset/images/volume7/figures/fig-c3p12.png new file mode 100644 index 0000000000000000000000000000000000000000..22a1c0f68967caf286a700c6a93d43e8ec67aa35 --- /dev/null +++ b/processed_dataset/images/volume7/figures/fig-c3p12.png @@ -0,0 +1,3 @@ +version https://git-lfs.github.com/spec/v1 +oid sha256:af20892f53c96f693adcc14048e5ed0a3bc7e760872a60c0f05226e740722a29 +size 19305 diff --git a/processed_dataset/images/volume7/figures/fig-c4a1.png b/processed_dataset/images/volume7/figures/fig-c4a1.png new file mode 100644 index 0000000000000000000000000000000000000000..7a637d36c88f524c34f43f300b6daf5f54a735d6 --- /dev/null +++ b/processed_dataset/images/volume7/figures/fig-c4a1.png @@ -0,0 +1,3 @@ +version https://git-lfs.github.com/spec/v1 +oid sha256:16329745538c1e0cc516ce27531362f3c3fabd29ae114a50321a496b11a65326 +size 40714 diff --git a/processed_dataset/images/volume7/figures/fig-c4a10.png b/processed_dataset/images/volume7/figures/fig-c4a10.png new file mode 100644 index 0000000000000000000000000000000000000000..db13d5e858e3350c94be45d65cbdabab57df9389 --- /dev/null +++ b/processed_dataset/images/volume7/figures/fig-c4a10.png @@ -0,0 +1,3 @@ +version https://git-lfs.github.com/spec/v1 +oid sha256:42818fe3cbe92828a7938c12c92d6f163c9baa16c9f5a9e3adcb4bf7fab3f79e +size 43339 diff --git a/processed_dataset/images/volume7/figures/fig-c4a11.png b/processed_dataset/images/volume7/figures/fig-c4a11.png new file mode 100644 index 0000000000000000000000000000000000000000..f2fc70ef060ce156c55a3e7209e4e9edad617cde --- /dev/null +++ b/processed_dataset/images/volume7/figures/fig-c4a11.png @@ -0,0 +1,3 @@ +version https://git-lfs.github.com/spec/v1 +oid sha256:ebbacac50e7378353105b6ba1823fae08a5c9f938ced7fce9cf3416d1312236b +size 28672 diff --git a/processed_dataset/images/volume7/figures/fig-c4a12.png b/processed_dataset/images/volume7/figures/fig-c4a12.png new file mode 100644 index 0000000000000000000000000000000000000000..304a5011812ef30a981be0637be88808d2d51918 --- /dev/null +++ b/processed_dataset/images/volume7/figures/fig-c4a12.png @@ -0,0 +1,3 @@ +version https://git-lfs.github.com/spec/v1 +oid sha256:a892aa0efd35e931f614a25ac6a0841f7a44e1275ff3c9863044b9b74a64cac7 +size 35411 diff --git a/processed_dataset/images/volume7/figures/fig-c4a13.png b/processed_dataset/images/volume7/figures/fig-c4a13.png new file mode 100644 index 0000000000000000000000000000000000000000..55d00e7c472338c86b22762911ebe0e937c13746 --- /dev/null +++ b/processed_dataset/images/volume7/figures/fig-c4a13.png @@ -0,0 +1,3 @@ +version https://git-lfs.github.com/spec/v1 +oid sha256:749b3e4a6c8ffed28516ddfe429135692011595f7d6e7e0323e8477c4488c573 +size 46110 diff --git a/processed_dataset/images/volume7/figures/fig-c4a14.png b/processed_dataset/images/volume7/figures/fig-c4a14.png new file mode 100644 index 0000000000000000000000000000000000000000..6c521216acd8879fbbef9fa406fd157f61ffa6c8 --- /dev/null +++ b/processed_dataset/images/volume7/figures/fig-c4a14.png @@ -0,0 +1,3 @@ +version https://git-lfs.github.com/spec/v1 +oid sha256:77862f30c0f25a7c02578a790a6043e0c2530af742e71bf8a9289c96765aee03 +size 38668 diff --git a/processed_dataset/images/volume7/figures/fig-c4a15.png b/processed_dataset/images/volume7/figures/fig-c4a15.png new file mode 100644 index 0000000000000000000000000000000000000000..ae44b7adcce25000e574f12c63a247ffc4a44b08 --- /dev/null +++ b/processed_dataset/images/volume7/figures/fig-c4a15.png @@ -0,0 +1,3 @@ +version https://git-lfs.github.com/spec/v1 +oid sha256:48f2c11dda19d8bb89b3e3d7d3150bc9e99af20027ac5be654e809d88751a049 +size 34356 diff --git a/processed_dataset/images/volume7/figures/fig-c4a16.png b/processed_dataset/images/volume7/figures/fig-c4a16.png new file mode 100644 index 0000000000000000000000000000000000000000..f54ce5ccff900d8c87da9a041af539ed289a50ff --- /dev/null +++ b/processed_dataset/images/volume7/figures/fig-c4a16.png @@ -0,0 +1,3 @@ +version https://git-lfs.github.com/spec/v1 +oid sha256:3c70b4980445b226800e724c474fc1f88fe455bf16b7586c06f1ff53c25be1fd +size 48455 diff --git a/processed_dataset/images/volume7/figures/fig-c4a17.png b/processed_dataset/images/volume7/figures/fig-c4a17.png new file mode 100644 index 0000000000000000000000000000000000000000..f3782e5175e166e443fee1f3d48ef0545b0013da --- /dev/null +++ b/processed_dataset/images/volume7/figures/fig-c4a17.png @@ -0,0 +1,3 @@ +version https://git-lfs.github.com/spec/v1 +oid sha256:bbc041ce39d598ad14e21f210c15a4668f39af77c6705ac3489be6437c3ad24b +size 45708 diff --git a/processed_dataset/images/volume7/figures/fig-c4a18.png b/processed_dataset/images/volume7/figures/fig-c4a18.png new file mode 100644 index 0000000000000000000000000000000000000000..a953c10ba6caa8be507d76b59e2ecdefee165184 --- /dev/null +++ b/processed_dataset/images/volume7/figures/fig-c4a18.png @@ -0,0 +1,3 @@ +version https://git-lfs.github.com/spec/v1 +oid sha256:291eb2e5052775d6c5ea4af05c8071b93f5c04201f07eaa7396e13d54d8b0e4e +size 49111 diff --git a/processed_dataset/images/volume7/figures/fig-c4a19-1.png b/processed_dataset/images/volume7/figures/fig-c4a19-1.png new file mode 100644 index 0000000000000000000000000000000000000000..e62658722c84a276bbb354dd3ec08b21eeb621df --- /dev/null +++ b/processed_dataset/images/volume7/figures/fig-c4a19-1.png @@ -0,0 +1,3 @@ +version https://git-lfs.github.com/spec/v1 +oid sha256:89ff18556aacb5c6fd68f9eba6e9fa578d81dbf2d143e30ee0fb9c6bbd77fbea +size 22002 diff --git a/processed_dataset/images/volume7/figures/fig-c4a19-2.png b/processed_dataset/images/volume7/figures/fig-c4a19-2.png new file mode 100644 index 0000000000000000000000000000000000000000..cfc53c9093b7e915bbcfc531572d478ed46a2db5 --- /dev/null +++ b/processed_dataset/images/volume7/figures/fig-c4a19-2.png @@ -0,0 +1,3 @@ +version https://git-lfs.github.com/spec/v1 +oid sha256:79cdc1d4ed4884df60dd4981122a02df2f2f4776b645ad8413fd006379946457 +size 43362 diff --git a/processed_dataset/images/volume7/figures/fig-c4a2.png b/processed_dataset/images/volume7/figures/fig-c4a2.png new file mode 100644 index 0000000000000000000000000000000000000000..9a47adc0044bc938f53438121059597daba46972 --- /dev/null +++ b/processed_dataset/images/volume7/figures/fig-c4a2.png @@ -0,0 +1,3 @@ +version https://git-lfs.github.com/spec/v1 +oid sha256:cd84232dd20501148874ce82858591798a33916752528c9b1ddbd89ac94652f1 +size 43244 diff --git a/processed_dataset/images/volume7/figures/fig-c4a3.png b/processed_dataset/images/volume7/figures/fig-c4a3.png new file mode 100644 index 0000000000000000000000000000000000000000..97fa5a979abd06921ec87a4952a69db7c5ef0560 --- /dev/null +++ b/processed_dataset/images/volume7/figures/fig-c4a3.png @@ -0,0 +1,3 @@ +version https://git-lfs.github.com/spec/v1 +oid sha256:2fe7d8ecaa20c73b64ad90694bebf88233bbefd03fc77ced566cd8e0741d4b49 +size 39404 diff --git a/processed_dataset/images/volume7/figures/fig-c4a4.png b/processed_dataset/images/volume7/figures/fig-c4a4.png new file mode 100644 index 0000000000000000000000000000000000000000..c7d7cc360c3890ef06da412a5b96eec09b9f9ffb --- /dev/null +++ b/processed_dataset/images/volume7/figures/fig-c4a4.png @@ -0,0 +1,3 @@ +version https://git-lfs.github.com/spec/v1 +oid sha256:8f0cd84e9b2f523f353fb993c095837b84a5631f8736638be4e5765cf434693c +size 38421 diff --git a/processed_dataset/images/volume7/figures/fig-c4a5.png b/processed_dataset/images/volume7/figures/fig-c4a5.png new file mode 100644 index 0000000000000000000000000000000000000000..2e5f7d45a4a4e3bee402e27c8f0740cb3499ed0d --- /dev/null +++ b/processed_dataset/images/volume7/figures/fig-c4a5.png @@ -0,0 +1,3 @@ +version https://git-lfs.github.com/spec/v1 +oid sha256:819f79d31f89aef06a90bdd361f15539ab031c42a9320c57f60e730ad7abc656 +size 39820 diff --git a/processed_dataset/images/volume7/figures/fig-c4a6.png b/processed_dataset/images/volume7/figures/fig-c4a6.png new file mode 100644 index 0000000000000000000000000000000000000000..e89baeb39fd12b697442d09fb399a82e488a20dc --- /dev/null +++ b/processed_dataset/images/volume7/figures/fig-c4a6.png @@ -0,0 +1,3 @@ +version https://git-lfs.github.com/spec/v1 +oid sha256:6323be56a4eaff55ad7df59f90efd24263d87f110abf32a7826faae0c1463d09 +size 34745 diff --git a/processed_dataset/images/volume7/figures/fig-c4a7.png b/processed_dataset/images/volume7/figures/fig-c4a7.png new file mode 100644 index 0000000000000000000000000000000000000000..d102358c4b10beeea72c88ce4fd4c7af3719c4ad --- /dev/null +++ b/processed_dataset/images/volume7/figures/fig-c4a7.png @@ -0,0 +1,3 @@ +version https://git-lfs.github.com/spec/v1 +oid sha256:e937687599af0cce313e7c94f463abc56bfbbef929caed6d0514c3382398ffeb +size 32007 diff --git a/processed_dataset/images/volume7/figures/fig-c4a8.png b/processed_dataset/images/volume7/figures/fig-c4a8.png new file mode 100644 index 0000000000000000000000000000000000000000..8f786fafd9b9f995b789db4f5b7af2be35208c51 --- /dev/null +++ b/processed_dataset/images/volume7/figures/fig-c4a8.png @@ -0,0 +1,3 @@ +version https://git-lfs.github.com/spec/v1 +oid sha256:68ffcc2805e7e6067c03cb7fb6295264edc0a6fc53399a6464787e27305ad696 +size 32167 diff --git a/processed_dataset/images/volume7/figures/fig-c4a9.png b/processed_dataset/images/volume7/figures/fig-c4a9.png new file mode 100644 index 0000000000000000000000000000000000000000..57ba8a624e76760adaab61cdad06008402dc9fc1 --- /dev/null +++ b/processed_dataset/images/volume7/figures/fig-c4a9.png @@ -0,0 +1,3 @@ +version https://git-lfs.github.com/spec/v1 +oid sha256:03c249c1424b9aa8320df47483d89d387825423fdcd7723fe8254dbd27c000eb +size 30885 diff --git a/processed_dataset/images/volume7/figures/fig-c4i1.png b/processed_dataset/images/volume7/figures/fig-c4i1.png new file mode 100644 index 0000000000000000000000000000000000000000..b8c5657d37055b30c3cb99fd7a7306505dcd4678 --- /dev/null +++ b/processed_dataset/images/volume7/figures/fig-c4i1.png @@ -0,0 +1,3 @@ +version https://git-lfs.github.com/spec/v1 +oid sha256:1387fc800ce5774a78cd5eff4d74d693640c0da4bd56ba7b75aef2df2de0cad5 +size 32802 diff --git a/processed_dataset/images/volume7/figures/fig-c4i10.png b/processed_dataset/images/volume7/figures/fig-c4i10.png new file mode 100644 index 0000000000000000000000000000000000000000..40a01eaa18a934c9269542cde2d1a95c3a6b5cab --- /dev/null +++ b/processed_dataset/images/volume7/figures/fig-c4i10.png @@ -0,0 +1,3 @@ +version https://git-lfs.github.com/spec/v1 +oid sha256:ea8f674ad267741d062e783d245ebae990100576b097b00897ec79ff5b4cc44f +size 35433 diff --git a/processed_dataset/images/volume7/figures/fig-c4i11.png b/processed_dataset/images/volume7/figures/fig-c4i11.png new file mode 100644 index 0000000000000000000000000000000000000000..63ee9e0da1ebc5cc4494ea4bd9a44498fd55a97b --- /dev/null +++ b/processed_dataset/images/volume7/figures/fig-c4i11.png @@ -0,0 +1,3 @@ +version https://git-lfs.github.com/spec/v1 +oid sha256:0225134850eb690b88e3e5e7f849ad8fea83be4c605ae4dd6ea654c493e80426 +size 33229 diff --git a/processed_dataset/images/volume7/figures/fig-c4i12.png b/processed_dataset/images/volume7/figures/fig-c4i12.png new file mode 100644 index 0000000000000000000000000000000000000000..ec1eb391f449ea1563e870b0431aac7c62ad6b61 --- /dev/null +++ b/processed_dataset/images/volume7/figures/fig-c4i12.png @@ -0,0 +1,3 @@ +version https://git-lfs.github.com/spec/v1 +oid sha256:dc093e7c7104c65c1b616b3e1b200785bd7a41ef0b784d7a4d6baa2ebfbd53b6 +size 29626 diff --git a/processed_dataset/images/volume7/figures/fig-c4i13.png b/processed_dataset/images/volume7/figures/fig-c4i13.png new file mode 100644 index 0000000000000000000000000000000000000000..a05643f617fd48b86df10a21764c1560544ad090 --- /dev/null +++ b/processed_dataset/images/volume7/figures/fig-c4i13.png @@ -0,0 +1,3 @@ +version https://git-lfs.github.com/spec/v1 +oid sha256:0f1b798f8c25aeb9ccbd9db7be3cb1957b0838301dc1618343a4d97d12252da9 +size 36695 diff --git a/processed_dataset/images/volume7/figures/fig-c4i2.png b/processed_dataset/images/volume7/figures/fig-c4i2.png new file mode 100644 index 0000000000000000000000000000000000000000..b0c1ce34728357211d97d94189d23292f181c14a --- /dev/null +++ b/processed_dataset/images/volume7/figures/fig-c4i2.png @@ -0,0 +1,3 @@ +version https://git-lfs.github.com/spec/v1 +oid sha256:34605fe24cd6086a624d4a5ab62d65905ee16b8327cef5e971ae9a57d73b8e99 +size 42893 diff --git a/processed_dataset/images/volume7/figures/fig-c4i3.png b/processed_dataset/images/volume7/figures/fig-c4i3.png new file mode 100644 index 0000000000000000000000000000000000000000..e1b6df7af756c2a84e9a37952c3c24cb4b70fd01 --- /dev/null +++ b/processed_dataset/images/volume7/figures/fig-c4i3.png @@ -0,0 +1,3 @@ +version https://git-lfs.github.com/spec/v1 +oid sha256:3c7a2e286cb645b972881ad88df308bc68b6cd32eb7b15d81a72f4a8cfa79c5c +size 34686 diff --git a/processed_dataset/images/volume7/figures/fig-c4i4.png b/processed_dataset/images/volume7/figures/fig-c4i4.png new file mode 100644 index 0000000000000000000000000000000000000000..92fd421982c066706548cb34dae265729658c1e1 --- /dev/null +++ b/processed_dataset/images/volume7/figures/fig-c4i4.png @@ -0,0 +1,3 @@ +version https://git-lfs.github.com/spec/v1 +oid sha256:3cb173c275604b52e8d31f69d656aad16313ba3d0202523e00b0d2dcfbd37814 +size 33763 diff --git a/processed_dataset/images/volume7/figures/fig-c4i5.png b/processed_dataset/images/volume7/figures/fig-c4i5.png new file mode 100644 index 0000000000000000000000000000000000000000..906c955d147bf7003746518ce66d7d339d54dc66 --- /dev/null +++ b/processed_dataset/images/volume7/figures/fig-c4i5.png @@ -0,0 +1,3 @@ +version https://git-lfs.github.com/spec/v1 +oid sha256:20f213e2d8224d70e13832c88ce4dc56dba0c9371ddbfa2ae940896ad09059d8 +size 34457 diff --git a/processed_dataset/images/volume7/figures/fig-c4i6.png b/processed_dataset/images/volume7/figures/fig-c4i6.png new file mode 100644 index 0000000000000000000000000000000000000000..432d0a0f056f1232f627e27eb6f7e68d7a6acd4d --- /dev/null +++ b/processed_dataset/images/volume7/figures/fig-c4i6.png @@ -0,0 +1,3 @@ +version https://git-lfs.github.com/spec/v1 +oid sha256:f85ae020e6f7e4f3bf48939ef2626f1d085dafbefe00db614a6754fa9e2f5a8b +size 41837 diff --git a/processed_dataset/images/volume7/figures/fig-c4i7.png b/processed_dataset/images/volume7/figures/fig-c4i7.png new file mode 100644 index 0000000000000000000000000000000000000000..a5b82e93d71dcee4e678c71dd4851e587bcecfca --- /dev/null +++ b/processed_dataset/images/volume7/figures/fig-c4i7.png @@ -0,0 +1,3 @@ +version https://git-lfs.github.com/spec/v1 +oid sha256:938e54b6e13207e6509bb8bb7ff34a78880958972bb66e221c31406dacf49aee +size 44028 diff --git a/processed_dataset/images/volume7/figures/fig-c4i8.png b/processed_dataset/images/volume7/figures/fig-c4i8.png new file mode 100644 index 0000000000000000000000000000000000000000..55278904812f014356e54d5d814a1c6bf1d954ee --- /dev/null +++ b/processed_dataset/images/volume7/figures/fig-c4i8.png @@ -0,0 +1,3 @@ +version https://git-lfs.github.com/spec/v1 +oid sha256:60b57909c6daf09f9873d8afa237c9486a83380cee1d0ccbb782ebbfbee191e4 +size 39841 diff --git a/processed_dataset/images/volume7/figures/fig-c4i9.png b/processed_dataset/images/volume7/figures/fig-c4i9.png new file mode 100644 index 0000000000000000000000000000000000000000..ac54c47fbef36fe6a153b4cb793ad38def05150b --- /dev/null +++ b/processed_dataset/images/volume7/figures/fig-c4i9.png @@ -0,0 +1,3 @@ +version https://git-lfs.github.com/spec/v1 +oid sha256:151fb740540e76924cd964e453c80d81c6e4b0f2fca5f49cd050d0cd2884cec4 +size 49732 diff --git a/processed_dataset/images/volume7/figures/fig-c4p1.png b/processed_dataset/images/volume7/figures/fig-c4p1.png new file mode 100644 index 0000000000000000000000000000000000000000..8e9e3f532ac50c0940c15c2131eabac3d9f47501 --- /dev/null +++ b/processed_dataset/images/volume7/figures/fig-c4p1.png @@ -0,0 +1,3 @@ +version https://git-lfs.github.com/spec/v1 +oid sha256:be63048aa1b47cd381194e154188225d59ae6e5fb1a8dcda37438824cd8b2de7 +size 30650 diff --git a/processed_dataset/images/volume7/figures/fig-c4p14.png b/processed_dataset/images/volume7/figures/fig-c4p14.png new file mode 100644 index 0000000000000000000000000000000000000000..5358c7925faa9803636238d4ab8bf41412ff5433 --- /dev/null +++ b/processed_dataset/images/volume7/figures/fig-c4p14.png @@ -0,0 +1,3 @@ +version https://git-lfs.github.com/spec/v1 +oid sha256:5e4c54c6d9f6decd3e1ec7543fdd02f7f59222c73a53cb2765556898cdd8c8a3 +size 30805 diff --git a/processed_dataset/images/volume7/figures/fig-c4p15.png b/processed_dataset/images/volume7/figures/fig-c4p15.png new file mode 100644 index 0000000000000000000000000000000000000000..1c8cdcd08c7f040baeeecfc784eec989ef7b5ff3 --- /dev/null +++ b/processed_dataset/images/volume7/figures/fig-c4p15.png @@ -0,0 +1,3 @@ +version https://git-lfs.github.com/spec/v1 +oid sha256:d2982d9c39b5afc9808b4225f54d4c35f1e01b84505a370d10a97bf239756599 +size 20787 diff --git a/processed_dataset/images/volume7/figures/fig-c4p18.png b/processed_dataset/images/volume7/figures/fig-c4p18.png new file mode 100644 index 0000000000000000000000000000000000000000..578941894ecdd7642a9339b639a7303116575ef8 --- /dev/null +++ b/processed_dataset/images/volume7/figures/fig-c4p18.png @@ -0,0 +1,3 @@ +version https://git-lfs.github.com/spec/v1 +oid sha256:8844277876769713256fb078385fbcf19060439d84b4043320750bfb09f7b07b +size 37296 diff --git a/processed_dataset/images/volume7/figures/fig-c4p3.png b/processed_dataset/images/volume7/figures/fig-c4p3.png new file mode 100644 index 0000000000000000000000000000000000000000..9f017d62f6b7b1bcfb4a90b884b3cac977c1e58a --- /dev/null +++ b/processed_dataset/images/volume7/figures/fig-c4p3.png @@ -0,0 +1,3 @@ +version https://git-lfs.github.com/spec/v1 +oid sha256:479cc8e145f92284119ab8d38eb04d955dbfc9a77f439867b8ed3c938162ff17 +size 30276 diff --git a/processed_dataset/images/volume7/figures/fig-c5a10.png b/processed_dataset/images/volume7/figures/fig-c5a10.png new file mode 100644 index 0000000000000000000000000000000000000000..2deb7eb87ac91bd4743e3f70322179bb229e1717 --- /dev/null +++ b/processed_dataset/images/volume7/figures/fig-c5a10.png @@ -0,0 +1,3 @@ +version https://git-lfs.github.com/spec/v1 +oid sha256:7447d92ed55b9422d99322cda1ebba8000cb0ca58cbbf4d43ccb4138db0f1df4 +size 41212 diff --git a/processed_dataset/images/volume7/figures/fig-c5a11.png b/processed_dataset/images/volume7/figures/fig-c5a11.png new file mode 100644 index 0000000000000000000000000000000000000000..32a55c6125a2beb53c539ae33e4beb70a112052e --- /dev/null +++ b/processed_dataset/images/volume7/figures/fig-c5a11.png @@ -0,0 +1,3 @@ +version https://git-lfs.github.com/spec/v1 +oid sha256:df45702e643803af3a833586142bbe0847a085f3856224c32851210684ff5830 +size 33955 diff --git a/processed_dataset/images/volume7/figures/fig-c5a12.png b/processed_dataset/images/volume7/figures/fig-c5a12.png new file mode 100644 index 0000000000000000000000000000000000000000..d5b8e7c62923d78523a7469bed0e76ae5ec6abd2 --- /dev/null +++ b/processed_dataset/images/volume7/figures/fig-c5a12.png @@ -0,0 +1,3 @@ +version https://git-lfs.github.com/spec/v1 +oid sha256:9919929d3a81620c2099125b2b88fd08f2a969d0e8c0c58781d44079e68e1dcc +size 36277 diff --git a/processed_dataset/images/volume7/figures/fig-c5a13.png b/processed_dataset/images/volume7/figures/fig-c5a13.png new file mode 100644 index 0000000000000000000000000000000000000000..a97273eae38b018219137438044f5e92b19e034d --- /dev/null +++ b/processed_dataset/images/volume7/figures/fig-c5a13.png @@ -0,0 +1,3 @@ +version https://git-lfs.github.com/spec/v1 +oid sha256:88db2fb226728a3abde38d0a39100ea421879c60c1d0dd4986fe063e97ad9fb5 +size 43830 diff --git a/processed_dataset/images/volume7/figures/fig-c5a14.png b/processed_dataset/images/volume7/figures/fig-c5a14.png new file mode 100644 index 0000000000000000000000000000000000000000..d1805921540dbe99e4f215c118c0a881c031df11 --- /dev/null +++ b/processed_dataset/images/volume7/figures/fig-c5a14.png @@ -0,0 +1,3 @@ +version https://git-lfs.github.com/spec/v1 +oid sha256:ee79de16ec856f3668a756478451765cc8f1668650fd5bedff4607e73eb75542 +size 32959 diff --git a/processed_dataset/images/volume7/figures/fig-c5a15.png b/processed_dataset/images/volume7/figures/fig-c5a15.png new file mode 100644 index 0000000000000000000000000000000000000000..67cde0fcfc2d66e7b166243bfa30b2ec8e8f2b5f --- /dev/null +++ b/processed_dataset/images/volume7/figures/fig-c5a15.png @@ -0,0 +1,3 @@ +version https://git-lfs.github.com/spec/v1 +oid sha256:58438f6a399fe406e0f43864c2b84e05be5ef6a45406eb2198938bdaf9d788c0 +size 38691 diff --git a/processed_dataset/images/volume7/figures/fig-c5a16.png b/processed_dataset/images/volume7/figures/fig-c5a16.png new file mode 100644 index 0000000000000000000000000000000000000000..be79bca889e398ea0b6d4edc4b4d48243c1e5b10 --- /dev/null +++ b/processed_dataset/images/volume7/figures/fig-c5a16.png @@ -0,0 +1,3 @@ +version https://git-lfs.github.com/spec/v1 +oid sha256:af7dd04c45391382bc164a126b4177ac91185be192bd6382e70a13847bb88173 +size 35653 diff --git a/processed_dataset/images/volume7/figures/fig-c5a17.png b/processed_dataset/images/volume7/figures/fig-c5a17.png new file mode 100644 index 0000000000000000000000000000000000000000..f9491ec973ce8733bdc3c684bcc4522f0d1a2f15 --- /dev/null +++ b/processed_dataset/images/volume7/figures/fig-c5a17.png @@ -0,0 +1,3 @@ +version https://git-lfs.github.com/spec/v1 +oid sha256:3b77a51a4b4ee25155b7e699c48bf26a262d86ea73ee0511071614d7854f3282 +size 37547 diff --git a/processed_dataset/images/volume7/figures/fig-c5a18.png b/processed_dataset/images/volume7/figures/fig-c5a18.png new file mode 100644 index 0000000000000000000000000000000000000000..9f9c5f03d44d4ffe2a2a6a4a34a05d4f9bb14ccc --- /dev/null +++ b/processed_dataset/images/volume7/figures/fig-c5a18.png @@ -0,0 +1,3 @@ +version https://git-lfs.github.com/spec/v1 +oid sha256:6939f266c78ae9c1e56e4980f465df48f81207f6b36ab46b963f5a685ea473e8 +size 40977 diff --git a/processed_dataset/images/volume7/figures/fig-c5a19.png b/processed_dataset/images/volume7/figures/fig-c5a19.png new file mode 100644 index 0000000000000000000000000000000000000000..1a8edc40dec1507040b12c8f686ffa7971dbd829 --- /dev/null +++ b/processed_dataset/images/volume7/figures/fig-c5a19.png @@ -0,0 +1,3 @@ +version https://git-lfs.github.com/spec/v1 +oid sha256:71eb41e638fdb6b9cfd3bd54a0c1a6820e556a776162d2ad78caeedc24a694e8 +size 29476 diff --git a/processed_dataset/images/volume7/figures/fig-c5a2.png b/processed_dataset/images/volume7/figures/fig-c5a2.png new file mode 100644 index 0000000000000000000000000000000000000000..b68ca8b1dc4c4575e0ce30b36e1c1d28b06a0aba --- /dev/null +++ b/processed_dataset/images/volume7/figures/fig-c5a2.png @@ -0,0 +1,3 @@ +version https://git-lfs.github.com/spec/v1 +oid sha256:ec5bf8cc1b5427291f022ba7b9f34a7e32736bb79e5d5ab518d9387d49855613 +size 34192 diff --git a/processed_dataset/images/volume7/figures/fig-c5a20.png b/processed_dataset/images/volume7/figures/fig-c5a20.png new file mode 100644 index 0000000000000000000000000000000000000000..18dfc018100c3947bfb7a5332f6e7bc67cb39fa3 --- /dev/null +++ b/processed_dataset/images/volume7/figures/fig-c5a20.png @@ -0,0 +1,3 @@ +version https://git-lfs.github.com/spec/v1 +oid sha256:9581057f11ce61daccb54a7aff1f46b60ad00f6a20b1ef88f048667924aeecca +size 26200 diff --git a/processed_dataset/images/volume7/figures/fig-c5a21.png b/processed_dataset/images/volume7/figures/fig-c5a21.png new file mode 100644 index 0000000000000000000000000000000000000000..b10fc0de4e9c1b39448c53e9c9d7d2bb1bebe699 --- /dev/null +++ b/processed_dataset/images/volume7/figures/fig-c5a21.png @@ -0,0 +1,3 @@ +version https://git-lfs.github.com/spec/v1 +oid sha256:41e41bfa09d90d38c9e7d365d35a8e89a270615ccb36248a7386850da36b7aa8 +size 37315 diff --git a/processed_dataset/images/volume7/figures/fig-c5a3.png b/processed_dataset/images/volume7/figures/fig-c5a3.png new file mode 100644 index 0000000000000000000000000000000000000000..7f99c06fa718b8f07ff05afbb8e25be8fe0a487f --- /dev/null +++ b/processed_dataset/images/volume7/figures/fig-c5a3.png @@ -0,0 +1,3 @@ +version https://git-lfs.github.com/spec/v1 +oid sha256:1830ef3da6882f3b5040c15da6f74c21aac7a50583436be71e643f708ae5d9b7 +size 36292 diff --git a/processed_dataset/images/volume7/figures/fig-c5a4.png b/processed_dataset/images/volume7/figures/fig-c5a4.png new file mode 100644 index 0000000000000000000000000000000000000000..c078360ddb6980e3e0a218eb025dd15e3433f72e --- /dev/null +++ b/processed_dataset/images/volume7/figures/fig-c5a4.png @@ -0,0 +1,3 @@ +version https://git-lfs.github.com/spec/v1 +oid sha256:7d2a793c51bb9e99b7d605123eaa78d0d2b87b6cf34fcb78e6706a6472f78b30 +size 51458 diff --git a/processed_dataset/images/volume7/figures/fig-c5a5.png b/processed_dataset/images/volume7/figures/fig-c5a5.png new file mode 100644 index 0000000000000000000000000000000000000000..64743fc1d70434961ca22fac2225b88f2a9aac16 --- /dev/null +++ b/processed_dataset/images/volume7/figures/fig-c5a5.png @@ -0,0 +1,3 @@ +version https://git-lfs.github.com/spec/v1 +oid sha256:d34d29969f881b66c213372da0e5340aba03067719adc7d36db9b7149615a119 +size 35505 diff --git a/processed_dataset/images/volume7/figures/fig-c5a6.png b/processed_dataset/images/volume7/figures/fig-c5a6.png new file mode 100644 index 0000000000000000000000000000000000000000..bd577ffb38fce969c6e9bcb00b025ea35c3cb44d --- /dev/null +++ b/processed_dataset/images/volume7/figures/fig-c5a6.png @@ -0,0 +1,3 @@ +version https://git-lfs.github.com/spec/v1 +oid sha256:ad491a01441ce6963dcfe227c6552b6212b62b34b719fa882df6e62baa5d11e7 +size 29532 diff --git a/processed_dataset/images/volume7/figures/fig-c5a7.png b/processed_dataset/images/volume7/figures/fig-c5a7.png new file mode 100644 index 0000000000000000000000000000000000000000..9481fd2f592b0445b77073b0e100b495edcf66b4 --- /dev/null +++ b/processed_dataset/images/volume7/figures/fig-c5a7.png @@ -0,0 +1,3 @@ +version https://git-lfs.github.com/spec/v1 +oid sha256:c64c8da0f2a7f10ebaa7ca87105660586ae67dfd63ceb2b18157a8dd0f8787e2 +size 46219 diff --git a/processed_dataset/images/volume7/figures/fig-c5a8-1.png b/processed_dataset/images/volume7/figures/fig-c5a8-1.png new file mode 100644 index 0000000000000000000000000000000000000000..c0ff0a956815393caae4e1ee2da48003a5191499 --- /dev/null +++ b/processed_dataset/images/volume7/figures/fig-c5a8-1.png @@ -0,0 +1,3 @@ +version https://git-lfs.github.com/spec/v1 +oid sha256:b1568a6629a43a0fac0bb2e07ce34d6ac2b2df71908146a668675f28790bd20d +size 30899 diff --git a/processed_dataset/images/volume7/figures/fig-c5a8-2.png b/processed_dataset/images/volume7/figures/fig-c5a8-2.png new file mode 100644 index 0000000000000000000000000000000000000000..fda430feb0f5ce428bb0e2190296bbef6262900c --- /dev/null +++ b/processed_dataset/images/volume7/figures/fig-c5a8-2.png @@ -0,0 +1,3 @@ +version https://git-lfs.github.com/spec/v1 +oid sha256:3b080a159f1fe6f6d81c31c73aaf43e9630a5f27fea8702b2981d632a703c366 +size 37582 diff --git a/processed_dataset/images/volume7/figures/fig-c5a9.png b/processed_dataset/images/volume7/figures/fig-c5a9.png new file mode 100644 index 0000000000000000000000000000000000000000..4d87f29877a2882a4d563ea21353e3f28ee1a2ba --- /dev/null +++ b/processed_dataset/images/volume7/figures/fig-c5a9.png @@ -0,0 +1,3 @@ +version https://git-lfs.github.com/spec/v1 +oid sha256:e1b0661c3afe312e8ef9134cb351f9f77c46d5062a2faa4523b1159acdca9d27 +size 45034 diff --git a/processed_dataset/images/volume7/figures/fig-c5i1.png b/processed_dataset/images/volume7/figures/fig-c5i1.png new file mode 100644 index 0000000000000000000000000000000000000000..49b6c88fdb7ea42730abd678eca4df3e03a5dbeb --- /dev/null +++ b/processed_dataset/images/volume7/figures/fig-c5i1.png @@ -0,0 +1,3 @@ +version https://git-lfs.github.com/spec/v1 +oid sha256:f5b0d2cded8db1a328f33f38f565137cc2dfa818e79dd8f89261be664e606988 +size 28239 diff --git a/processed_dataset/images/volume7/figures/fig-c5i10.png b/processed_dataset/images/volume7/figures/fig-c5i10.png new file mode 100644 index 0000000000000000000000000000000000000000..b49ec6cd93aefc58b64d44e622f39d0eb3be444e --- /dev/null +++ b/processed_dataset/images/volume7/figures/fig-c5i10.png @@ -0,0 +1,3 @@ +version https://git-lfs.github.com/spec/v1 +oid sha256:0755b16d57f3ecc7660f2ebecd86914355d82def89dae3b7a4419bf1f92eea37 +size 31707 diff --git a/processed_dataset/images/volume7/figures/fig-c5i11.png b/processed_dataset/images/volume7/figures/fig-c5i11.png new file mode 100644 index 0000000000000000000000000000000000000000..949fb36d25a23c8c6f9f93a86fb6a3b7fbb1bb6e --- /dev/null +++ b/processed_dataset/images/volume7/figures/fig-c5i11.png @@ -0,0 +1,3 @@ +version https://git-lfs.github.com/spec/v1 +oid sha256:1e8b46a05a6b36999408d1e019bc664ef81fd4d4ec630e7ba93cf32feefef609 +size 51789 diff --git a/processed_dataset/images/volume7/figures/fig-c5i12.png b/processed_dataset/images/volume7/figures/fig-c5i12.png new file mode 100644 index 0000000000000000000000000000000000000000..f2f60f8e6615138577eee37ec3fb9bc3f6ae899d --- /dev/null +++ b/processed_dataset/images/volume7/figures/fig-c5i12.png @@ -0,0 +1,3 @@ +version https://git-lfs.github.com/spec/v1 +oid sha256:fe00e9524bf05bcca4c76e7d8ec99feb9421660d502a9b6bfbd9fec8b1b6d3e5 +size 33915 diff --git a/processed_dataset/images/volume7/figures/fig-c5i2.png b/processed_dataset/images/volume7/figures/fig-c5i2.png new file mode 100644 index 0000000000000000000000000000000000000000..c51bc7f3d7d86dd63b90d1d3d475c2a25b6cb1a8 --- /dev/null +++ b/processed_dataset/images/volume7/figures/fig-c5i2.png @@ -0,0 +1,3 @@ +version https://git-lfs.github.com/spec/v1 +oid sha256:37b9e5ab1978a13f060dcbbdee862e57521d57cc0d8c0fced853ddc9e9ece5c8 +size 32484 diff --git a/processed_dataset/images/volume7/figures/fig-c5i3.png b/processed_dataset/images/volume7/figures/fig-c5i3.png new file mode 100644 index 0000000000000000000000000000000000000000..eb89ea34b9fb22cff119cb26e3fa4f07e20a15d0 --- /dev/null +++ b/processed_dataset/images/volume7/figures/fig-c5i3.png @@ -0,0 +1,3 @@ +version https://git-lfs.github.com/spec/v1 +oid sha256:eebc2f72954fed23570e89a05d0b0b47680f2727df7489ee5d03f635685973ea +size 35602 diff --git a/processed_dataset/images/volume7/figures/fig-c5i4.png b/processed_dataset/images/volume7/figures/fig-c5i4.png new file mode 100644 index 0000000000000000000000000000000000000000..e1a1bf4d51b9d958362c15a035ed39fd8a87b447 --- /dev/null +++ b/processed_dataset/images/volume7/figures/fig-c5i4.png @@ -0,0 +1,3 @@ +version https://git-lfs.github.com/spec/v1 +oid sha256:0bc2a2708a9cc40df8b70a459bb49e075510c2885c1df73fb800cae1299f0aaa +size 26495 diff --git a/processed_dataset/images/volume7/figures/fig-c5i5.png b/processed_dataset/images/volume7/figures/fig-c5i5.png new file mode 100644 index 0000000000000000000000000000000000000000..d94ce8bf67400d6649122ebaefa118c860d7e002 --- /dev/null +++ b/processed_dataset/images/volume7/figures/fig-c5i5.png @@ -0,0 +1,3 @@ +version https://git-lfs.github.com/spec/v1 +oid sha256:8b8586a8780cea388bb513fff6d5460609b6c5efd574e556a6d11e84d5ea1827 +size 28063 diff --git a/processed_dataset/images/volume7/figures/fig-c5i6.png b/processed_dataset/images/volume7/figures/fig-c5i6.png new file mode 100644 index 0000000000000000000000000000000000000000..2309b1085df00256842a861cd86ca72edc89d12f --- /dev/null +++ b/processed_dataset/images/volume7/figures/fig-c5i6.png @@ -0,0 +1,3 @@ +version https://git-lfs.github.com/spec/v1 +oid sha256:827954799ec09d39deaf41380d625fdd054c148f9dde88667f8466dd6d06f170 +size 38851 diff --git a/processed_dataset/images/volume7/figures/fig-c5i7.png b/processed_dataset/images/volume7/figures/fig-c5i7.png new file mode 100644 index 0000000000000000000000000000000000000000..68ad4792721da6a5b4c23fd6f94ba07e16d43ec9 --- /dev/null +++ b/processed_dataset/images/volume7/figures/fig-c5i7.png @@ -0,0 +1,3 @@ +version https://git-lfs.github.com/spec/v1 +oid sha256:a5547e6a2452a5d4425c795e3f4687993b41e06f7e1b5f5196b3798228705c09 +size 31866 diff --git a/processed_dataset/images/volume7/figures/fig-c5i8.png b/processed_dataset/images/volume7/figures/fig-c5i8.png new file mode 100644 index 0000000000000000000000000000000000000000..bf1be79f336d3f24e7b3c88f737c1e0ccf845672 --- /dev/null +++ b/processed_dataset/images/volume7/figures/fig-c5i8.png @@ -0,0 +1,3 @@ +version https://git-lfs.github.com/spec/v1 +oid sha256:b4a3ed94cf286eac3bd013de46be6ed273a754393985db1bd8990cde09779a5d +size 40984 diff --git a/processed_dataset/images/volume7/figures/fig-c5i9.png b/processed_dataset/images/volume7/figures/fig-c5i9.png new file mode 100644 index 0000000000000000000000000000000000000000..8a70d6b0bb718460cfa44a59518d40e7564c11d6 --- /dev/null +++ b/processed_dataset/images/volume7/figures/fig-c5i9.png @@ -0,0 +1,3 @@ +version https://git-lfs.github.com/spec/v1 +oid sha256:c0d509b1de00d3fc1cca1b226f2a16c241d20f16993fb378ae54287808b2d5fd +size 37907 diff --git a/processed_dataset/images/volume7/figures/fig-c5p1.png b/processed_dataset/images/volume7/figures/fig-c5p1.png new file mode 100644 index 0000000000000000000000000000000000000000..8379f65b70a6b6bad0caba31687d9e16b522c2b8 --- /dev/null +++ b/processed_dataset/images/volume7/figures/fig-c5p1.png @@ -0,0 +1,3 @@ +version https://git-lfs.github.com/spec/v1 +oid sha256:1330cd7f533c3789f42330eb3aabfed001764e093325753d3fdedd1b4199bd57 +size 31223 diff --git a/processed_dataset/images/volume7/figures/fig-c5p2.png b/processed_dataset/images/volume7/figures/fig-c5p2.png new file mode 100644 index 0000000000000000000000000000000000000000..5edc7d4479ad42b36ed9d09bbfa2db0b03c5eebf --- /dev/null +++ b/processed_dataset/images/volume7/figures/fig-c5p2.png @@ -0,0 +1,3 @@ +version https://git-lfs.github.com/spec/v1 +oid sha256:3c37e6bf0ec0b6e8ee4ca593bb4178facc339628f92ac957411c9dfd3f0536e1 +size 31014 diff --git a/processed_dataset/images/volume7/figures/fig-c5p6.png b/processed_dataset/images/volume7/figures/fig-c5p6.png new file mode 100644 index 0000000000000000000000000000000000000000..d1654bd20df41cb61e09ccb4026a6c4004b8a2ce --- /dev/null +++ b/processed_dataset/images/volume7/figures/fig-c5p6.png @@ -0,0 +1,3 @@ +version https://git-lfs.github.com/spec/v1 +oid sha256:53b465e5571c47cfec3b353231cf5f923271a790d514246353d7a55ce3d0a033 +size 31187 diff --git a/processed_dataset/images/volume7/figures/fig-c5p7.png b/processed_dataset/images/volume7/figures/fig-c5p7.png new file mode 100644 index 0000000000000000000000000000000000000000..e803a471c52667e56b06eb10cf036b79a5a56f97 --- /dev/null +++ b/processed_dataset/images/volume7/figures/fig-c5p7.png @@ -0,0 +1,3 @@ +version https://git-lfs.github.com/spec/v1 +oid sha256:2680b59ed7172393517ce7b23338fef399effedd86efa51c76b5cc23204bb0f7 +size 29257 diff --git a/processed_dataset/images/volume7/figures/fig-c5p8.png b/processed_dataset/images/volume7/figures/fig-c5p8.png new file mode 100644 index 0000000000000000000000000000000000000000..251936e3f8dcfb465f74f2d84495840fa3bf179f --- /dev/null +++ b/processed_dataset/images/volume7/figures/fig-c5p8.png @@ -0,0 +1,3 @@ +version https://git-lfs.github.com/spec/v1 +oid sha256:e59a6c823e0d5a77482436e565c3c0502ebce22781ded013a9cc43e4ededcd3c +size 33360 diff --git a/processed_dataset/images/volume7/figures/fig-c6a1.png b/processed_dataset/images/volume7/figures/fig-c6a1.png new file mode 100644 index 0000000000000000000000000000000000000000..2ca4cfbdb5303fc2896ce8e3855994ddc01d3b29 --- /dev/null +++ b/processed_dataset/images/volume7/figures/fig-c6a1.png @@ -0,0 +1,3 @@ +version https://git-lfs.github.com/spec/v1 +oid sha256:fe9e563aed3d25977adbbc44d0bead5b5088c488dc953f790a0f267f8b50e526 +size 37572 diff --git a/processed_dataset/images/volume7/figures/fig-c6a10.png b/processed_dataset/images/volume7/figures/fig-c6a10.png new file mode 100644 index 0000000000000000000000000000000000000000..c2b1b1193687708cbbe56e002bb099dc6d5c496c --- /dev/null +++ b/processed_dataset/images/volume7/figures/fig-c6a10.png @@ -0,0 +1,3 @@ +version https://git-lfs.github.com/spec/v1 +oid sha256:7c5da10b310b8c44ef42cc4f40f2ced6d7aff3b46ce23d7c33cdefcbe4a0a540 +size 33616 diff --git a/processed_dataset/images/volume7/figures/fig-c6a11.png b/processed_dataset/images/volume7/figures/fig-c6a11.png new file mode 100644 index 0000000000000000000000000000000000000000..cdb5c83934d0d8981c9be53a432dc601b9b93a30 --- /dev/null +++ b/processed_dataset/images/volume7/figures/fig-c6a11.png @@ -0,0 +1,3 @@ +version https://git-lfs.github.com/spec/v1 +oid sha256:bcb5bb7dde2408f3eb626c52bd3ca3efc6fb724bb98e9689796a4fd804df8f39 +size 31373 diff --git a/processed_dataset/images/volume7/figures/fig-c6a12.png b/processed_dataset/images/volume7/figures/fig-c6a12.png new file mode 100644 index 0000000000000000000000000000000000000000..0300fccf757efb15acbe88d29d620df0cd9c91c1 --- /dev/null +++ b/processed_dataset/images/volume7/figures/fig-c6a12.png @@ -0,0 +1,3 @@ +version https://git-lfs.github.com/spec/v1 +oid sha256:cb6ad8ddf0f48c4bd8993b4be79db21ebf34c98312323cc4ec0123052ec9f9e3 +size 39527 diff --git a/processed_dataset/images/volume7/figures/fig-c6a13.png b/processed_dataset/images/volume7/figures/fig-c6a13.png new file mode 100644 index 0000000000000000000000000000000000000000..c6448005aaf453f18cdeb7089cafb6afc2382d01 --- /dev/null +++ b/processed_dataset/images/volume7/figures/fig-c6a13.png @@ -0,0 +1,3 @@ +version https://git-lfs.github.com/spec/v1 +oid sha256:700e6ff89781382dceea9568093426a5b55fb9168aed4e9722080bde0b8ccfb1 +size 39653 diff --git a/processed_dataset/images/volume7/figures/fig-c6a14.png b/processed_dataset/images/volume7/figures/fig-c6a14.png new file mode 100644 index 0000000000000000000000000000000000000000..fa4dbd9b522e80bbcbd27a28b23e10b4a58794fe --- /dev/null +++ b/processed_dataset/images/volume7/figures/fig-c6a14.png @@ -0,0 +1,3 @@ +version https://git-lfs.github.com/spec/v1 +oid sha256:ce463624684fa1e64f9c2e89b66bc65a3f1260fdb0b13baa941ac9407963f7c4 +size 39360 diff --git a/processed_dataset/images/volume7/figures/fig-c6a15.png b/processed_dataset/images/volume7/figures/fig-c6a15.png new file mode 100644 index 0000000000000000000000000000000000000000..9a95249c156611dcc6cebde606436cfff404ee3f --- /dev/null +++ b/processed_dataset/images/volume7/figures/fig-c6a15.png @@ -0,0 +1,3 @@ +version https://git-lfs.github.com/spec/v1 +oid sha256:24c96e728cfd27074b4d4b3702477667274fe35fb750bdffaa23d3c2f0ef1dc8 +size 39920 diff --git a/processed_dataset/images/volume7/figures/fig-c6a16.png b/processed_dataset/images/volume7/figures/fig-c6a16.png new file mode 100644 index 0000000000000000000000000000000000000000..ebf18567061c1b6f56e7be2fc9ebdc8ebefffbd6 --- /dev/null +++ b/processed_dataset/images/volume7/figures/fig-c6a16.png @@ -0,0 +1,3 @@ +version https://git-lfs.github.com/spec/v1 +oid sha256:cd49b7cd721dd96d91bf4c1cdf9abcb160bde2c6115ac5ff3f66de27333bd0f2 +size 40846 diff --git a/processed_dataset/images/volume7/figures/fig-c6a2.png b/processed_dataset/images/volume7/figures/fig-c6a2.png new file mode 100644 index 0000000000000000000000000000000000000000..8859c23d1fe199a408deb9b6cfae498d8a9f6861 --- /dev/null +++ b/processed_dataset/images/volume7/figures/fig-c6a2.png @@ -0,0 +1,3 @@ +version https://git-lfs.github.com/spec/v1 +oid sha256:88b4cec7be0c5dbc63b62cdc0974043e0930a059c38d855ae72fe644c7abd13f +size 42665 diff --git a/processed_dataset/images/volume7/figures/fig-c6a3.png b/processed_dataset/images/volume7/figures/fig-c6a3.png new file mode 100644 index 0000000000000000000000000000000000000000..c0b0d414f3ac5766999451f9d4f961af3166745d --- /dev/null +++ b/processed_dataset/images/volume7/figures/fig-c6a3.png @@ -0,0 +1,3 @@ +version https://git-lfs.github.com/spec/v1 +oid sha256:698e7fd81766a5885a754e70f2a3e81ef9d36a7ea463408b4462c402fe0ec6fe +size 31556 diff --git a/processed_dataset/images/volume7/figures/fig-c6a4.png b/processed_dataset/images/volume7/figures/fig-c6a4.png new file mode 100644 index 0000000000000000000000000000000000000000..85308862be1d8341fbcc1b89f3a0e27f3f65be56 --- /dev/null +++ b/processed_dataset/images/volume7/figures/fig-c6a4.png @@ -0,0 +1,3 @@ +version https://git-lfs.github.com/spec/v1 +oid sha256:eb4fa81a95a54d9a675f71f1576a5262daa1a735f321c7560bff0adda41fe538 +size 51330 diff --git a/processed_dataset/images/volume7/figures/fig-c6a5.png b/processed_dataset/images/volume7/figures/fig-c6a5.png new file mode 100644 index 0000000000000000000000000000000000000000..9e9239f32eae83f1071239cdcb7a3c27c41fc2ef --- /dev/null +++ b/processed_dataset/images/volume7/figures/fig-c6a5.png @@ -0,0 +1,3 @@ +version https://git-lfs.github.com/spec/v1 +oid sha256:9b7f97f140c07b6598d71580fb82586f3cfc2a97cbf0b9e0728644b2761c693c +size 34268 diff --git a/processed_dataset/images/volume7/figures/fig-c6a6.png b/processed_dataset/images/volume7/figures/fig-c6a6.png new file mode 100644 index 0000000000000000000000000000000000000000..3be1624de9ee9738713cf857efb1b52820c6cbb3 --- /dev/null +++ b/processed_dataset/images/volume7/figures/fig-c6a6.png @@ -0,0 +1,3 @@ +version https://git-lfs.github.com/spec/v1 +oid sha256:6ce59d4db1e721de5384db456a8cde2c210497949b25d6b9b9d6884f63ab8d4a +size 35946 diff --git a/processed_dataset/images/volume7/figures/fig-c6a7.png b/processed_dataset/images/volume7/figures/fig-c6a7.png new file mode 100644 index 0000000000000000000000000000000000000000..606db7f026dfd05c007a478d8998bfa3402620da --- /dev/null +++ b/processed_dataset/images/volume7/figures/fig-c6a7.png @@ -0,0 +1,3 @@ +version https://git-lfs.github.com/spec/v1 +oid sha256:fea0eae2a378ff25f99b0de1b20f89f9248b1e02e94ea7f98fe92e5f6a4be14a +size 39809 diff --git a/processed_dataset/images/volume7/figures/fig-c6a8.png b/processed_dataset/images/volume7/figures/fig-c6a8.png new file mode 100644 index 0000000000000000000000000000000000000000..68da89e345585b3f4cf13e95b425447851126148 --- /dev/null +++ b/processed_dataset/images/volume7/figures/fig-c6a8.png @@ -0,0 +1,3 @@ +version https://git-lfs.github.com/spec/v1 +oid sha256:f6802d9cf1a34568d3a9fd747e87d59152a4d4c2770b09c4ee586c637b97299d +size 32854 diff --git a/processed_dataset/images/volume7/figures/fig-c6a9.png b/processed_dataset/images/volume7/figures/fig-c6a9.png new file mode 100644 index 0000000000000000000000000000000000000000..4d03cc040feb808013e2d818b7f56ce12417c6e4 --- /dev/null +++ b/processed_dataset/images/volume7/figures/fig-c6a9.png @@ -0,0 +1,3 @@ +version https://git-lfs.github.com/spec/v1 +oid sha256:33da86a09427bdeb526935b5d2bf1eeab797948a7c87493ae637407f1c3e7a90 +size 37434 diff --git a/processed_dataset/images/volume7/figures/fig-c6i1.png b/processed_dataset/images/volume7/figures/fig-c6i1.png new file mode 100644 index 0000000000000000000000000000000000000000..19bbe8a49a2e6376717433f93a0ecaa4d9a17537 --- /dev/null +++ b/processed_dataset/images/volume7/figures/fig-c6i1.png @@ -0,0 +1,3 @@ +version https://git-lfs.github.com/spec/v1 +oid sha256:98731d42f809e25204ae0002154523c08066444de47efa95e6ab740d6b2e8569 +size 30747 diff --git a/processed_dataset/images/volume7/figures/fig-c6i2.png b/processed_dataset/images/volume7/figures/fig-c6i2.png new file mode 100644 index 0000000000000000000000000000000000000000..5d55b86bec22925be4b0425ab4926e1ecc19b274 --- /dev/null +++ b/processed_dataset/images/volume7/figures/fig-c6i2.png @@ -0,0 +1,3 @@ +version https://git-lfs.github.com/spec/v1 +oid sha256:5435fa3cab87b5e575f12a320bbe648321bc96e02e3cd44126261a683eec4378 +size 29475 diff --git a/processed_dataset/images/volume7/figures/fig-c6i3.png b/processed_dataset/images/volume7/figures/fig-c6i3.png new file mode 100644 index 0000000000000000000000000000000000000000..a290445b6547442e68b6eeca2a77cd0d42a871ca --- /dev/null +++ b/processed_dataset/images/volume7/figures/fig-c6i3.png @@ -0,0 +1,3 @@ +version https://git-lfs.github.com/spec/v1 +oid sha256:f353e0f79e33fedf363388d9d447ba26ba3ffce1c38f1742027da58766145278 +size 69280 diff --git a/processed_dataset/images/volume7/figures/fig-c6i4.png b/processed_dataset/images/volume7/figures/fig-c6i4.png new file mode 100644 index 0000000000000000000000000000000000000000..de13e9bcb608b32cc057a92a0389d6d5af7176b7 --- /dev/null +++ b/processed_dataset/images/volume7/figures/fig-c6i4.png @@ -0,0 +1,3 @@ +version https://git-lfs.github.com/spec/v1 +oid sha256:1ca7a49676393026a0fb8b41d19ea2b40032f6abfa91fa56cf43f4b9e7037467 +size 37492 diff --git a/processed_dataset/images/volume7/figures/fig-c6i5.png b/processed_dataset/images/volume7/figures/fig-c6i5.png new file mode 100644 index 0000000000000000000000000000000000000000..c19675002cd3ba3ca03ea5e1ebfc9f2d6ef6dc93 --- /dev/null +++ b/processed_dataset/images/volume7/figures/fig-c6i5.png @@ -0,0 +1,3 @@ +version https://git-lfs.github.com/spec/v1 +oid sha256:0b409bd1de27ee7e87d8442fb3db1adb093e9e3561f37009b80c69f5528d2c38 +size 40862 diff --git a/processed_dataset/images/volume7/figures/fig-c6i6.png b/processed_dataset/images/volume7/figures/fig-c6i6.png new file mode 100644 index 0000000000000000000000000000000000000000..b1be3a03660dfdd7281919d69dd46b10eee53383 --- /dev/null +++ b/processed_dataset/images/volume7/figures/fig-c6i6.png @@ -0,0 +1,3 @@ +version https://git-lfs.github.com/spec/v1 +oid sha256:f89c186ffb0078ab67b28c9b64d2e0eb76f31cec05d4bd7652cfb1b788ca6acc +size 41093 diff --git a/processed_dataset/images/volume7/figures/fig-c6i7.png b/processed_dataset/images/volume7/figures/fig-c6i7.png new file mode 100644 index 0000000000000000000000000000000000000000..232bd7a563f95cba0db7af8720e3d485462e920e --- /dev/null +++ b/processed_dataset/images/volume7/figures/fig-c6i7.png @@ -0,0 +1,3 @@ +version https://git-lfs.github.com/spec/v1 +oid sha256:742ea9694ed729bd2186d5847ef76cb21118eb7a5b097e3e2bd036c04d351c65 +size 32143 diff --git a/processed_dataset/images/volume7/figures/fig-c6i8.png b/processed_dataset/images/volume7/figures/fig-c6i8.png new file mode 100644 index 0000000000000000000000000000000000000000..fd4314453525ffe9a4cedd020107e4a8f0312eef --- /dev/null +++ b/processed_dataset/images/volume7/figures/fig-c6i8.png @@ -0,0 +1,3 @@ +version https://git-lfs.github.com/spec/v1 +oid sha256:73d89a93bcd102d703c5c303c92123a3f0f42f320e1ed88d32cb8a92f9fd0971 +size 47302 diff --git a/processed_dataset/images/volume7/figures/fig-c6p4.png b/processed_dataset/images/volume7/figures/fig-c6p4.png new file mode 100644 index 0000000000000000000000000000000000000000..36d2d9488c624d9db7da40f537dab94362829681 --- /dev/null +++ b/processed_dataset/images/volume7/figures/fig-c6p4.png @@ -0,0 +1,3 @@ +version https://git-lfs.github.com/spec/v1 +oid sha256:9efcd43f81e970c6bcbdb17072c54e2cf28625d6776f5861e2b8401a009ce824 +size 36526 diff --git a/processed_dataset/images/volume7/figures/fig-c7a10.png b/processed_dataset/images/volume7/figures/fig-c7a10.png new file mode 100644 index 0000000000000000000000000000000000000000..31a672baf32017b02cd88eda049fd77c567c92a9 --- /dev/null +++ b/processed_dataset/images/volume7/figures/fig-c7a10.png @@ -0,0 +1,3 @@ +version https://git-lfs.github.com/spec/v1 +oid sha256:a69e37e012dbb653162fdb80fee2233bffd466b1f574fb73e2696ea1ca06b7d6 +size 38894 diff --git a/processed_dataset/images/volume7/figures/fig-c7a11.png b/processed_dataset/images/volume7/figures/fig-c7a11.png new file mode 100644 index 0000000000000000000000000000000000000000..cec8e5d4b7dcdd8e27b052f8bf640c4d85d41dfc --- /dev/null +++ b/processed_dataset/images/volume7/figures/fig-c7a11.png @@ -0,0 +1,3 @@ +version https://git-lfs.github.com/spec/v1 +oid sha256:d2c7451192dd3b7c34948e73488b084a5f5180fc9101da9d5e53bb69c88d795a +size 41887 diff --git a/processed_dataset/images/volume7/figures/fig-c7a13.png b/processed_dataset/images/volume7/figures/fig-c7a13.png new file mode 100644 index 0000000000000000000000000000000000000000..b12fd1430681fe665a3577b8a4fa072e60660345 --- /dev/null +++ b/processed_dataset/images/volume7/figures/fig-c7a13.png @@ -0,0 +1,3 @@ +version https://git-lfs.github.com/spec/v1 +oid sha256:0f7cf7afac47e17c0adcb4a85282d1bc1f3fffae2d74abec2aafe0a2b203040e +size 39126 diff --git a/processed_dataset/images/volume7/figures/fig-c7a2.png b/processed_dataset/images/volume7/figures/fig-c7a2.png new file mode 100644 index 0000000000000000000000000000000000000000..997ebcfe37d2a87184cb5fefda59b574a7f7b8ca --- /dev/null +++ b/processed_dataset/images/volume7/figures/fig-c7a2.png @@ -0,0 +1,3 @@ +version https://git-lfs.github.com/spec/v1 +oid sha256:78646b3c424fc79e0f0ddceffbfc93e26ee28723e495123365dfcc532bbdc4f8 +size 36696 diff --git a/processed_dataset/images/volume7/figures/fig-c7a3.png b/processed_dataset/images/volume7/figures/fig-c7a3.png new file mode 100644 index 0000000000000000000000000000000000000000..ccabeaa7728c0614f263a74d409b88532b447239 --- /dev/null +++ b/processed_dataset/images/volume7/figures/fig-c7a3.png @@ -0,0 +1,3 @@ +version https://git-lfs.github.com/spec/v1 +oid sha256:bd9763fd23707e399f8514c589f9fba4933ab384cb15e8af5043802d97665407 +size 35312 diff --git a/processed_dataset/images/volume7/figures/fig-c7a4.png b/processed_dataset/images/volume7/figures/fig-c7a4.png new file mode 100644 index 0000000000000000000000000000000000000000..59eb9117c8563237b2498141a2e00c645e4231d0 --- /dev/null +++ b/processed_dataset/images/volume7/figures/fig-c7a4.png @@ -0,0 +1,3 @@ +version https://git-lfs.github.com/spec/v1 +oid sha256:8c9a093af5667150e68972d06a465a7083e3fe459b359554ebfb6fb1d14f8564 +size 35835 diff --git a/processed_dataset/images/volume7/figures/fig-c7a9.png b/processed_dataset/images/volume7/figures/fig-c7a9.png new file mode 100644 index 0000000000000000000000000000000000000000..4bf2923eaccc75e9129c3dfc561dc3e42a7d064f --- /dev/null +++ b/processed_dataset/images/volume7/figures/fig-c7a9.png @@ -0,0 +1,3 @@ +version https://git-lfs.github.com/spec/v1 +oid sha256:dc7b9ccc19b38a59238347b87a4835670d4adcd784d6c4146e37f0dc2126f2b4 +size 37239 diff --git a/processed_dataset/images/volume7/figures/fig-c7i1.png b/processed_dataset/images/volume7/figures/fig-c7i1.png new file mode 100644 index 0000000000000000000000000000000000000000..eb08c22a926300ee8e4e60eb9b73755c572c4e35 --- /dev/null +++ b/processed_dataset/images/volume7/figures/fig-c7i1.png @@ -0,0 +1,3 @@ +version https://git-lfs.github.com/spec/v1 +oid sha256:887941b2f03dfc353816d7ccb3b91f35042d4a69ad131c85431f14b158bc4474 +size 22496 diff --git a/processed_dataset/images/volume7/figures/fig-c7i10.png b/processed_dataset/images/volume7/figures/fig-c7i10.png new file mode 100644 index 0000000000000000000000000000000000000000..557426cd245c271a3752a3fd44353dcdfb4e9806 --- /dev/null +++ b/processed_dataset/images/volume7/figures/fig-c7i10.png @@ -0,0 +1,3 @@ +version https://git-lfs.github.com/spec/v1 +oid sha256:6eacb3a4c251647f3e8a97241b5f528c7432a54fab4b35a392bf5fb875c19508 +size 37110 diff --git a/processed_dataset/images/volume7/figures/fig-c7i11.png b/processed_dataset/images/volume7/figures/fig-c7i11.png new file mode 100644 index 0000000000000000000000000000000000000000..4d7ee3a6f4d4c1b8245934d2a7f60ba5a8d7615a --- /dev/null +++ b/processed_dataset/images/volume7/figures/fig-c7i11.png @@ -0,0 +1,3 @@ +version https://git-lfs.github.com/spec/v1 +oid sha256:dba53816896b096ebd7bd5c79f0435faf0408da5d3ed72796021912b03e3f777 +size 38792 diff --git a/processed_dataset/images/volume7/figures/fig-c7i12.png b/processed_dataset/images/volume7/figures/fig-c7i12.png new file mode 100644 index 0000000000000000000000000000000000000000..c6e6301534ef7c71ff3bd5a2ee8fe82baf0bdd19 --- /dev/null +++ b/processed_dataset/images/volume7/figures/fig-c7i12.png @@ -0,0 +1,3 @@ +version https://git-lfs.github.com/spec/v1 +oid sha256:8522aeead87614d6c4a6aa122fc74983ed11ab04cc82dbfa283a4e6d943c372d +size 30886 diff --git a/processed_dataset/images/volume7/figures/fig-c7i13.png b/processed_dataset/images/volume7/figures/fig-c7i13.png new file mode 100644 index 0000000000000000000000000000000000000000..367e47570409f5dc98e38793b4977f9479acb6e5 --- /dev/null +++ b/processed_dataset/images/volume7/figures/fig-c7i13.png @@ -0,0 +1,3 @@ +version https://git-lfs.github.com/spec/v1 +oid sha256:8faa7443bfdcfcc4eddeba15ee921daf209cca474d99b2c7b833792fdfbb87d7 +size 37512 diff --git a/processed_dataset/images/volume7/figures/fig-c7i2.png b/processed_dataset/images/volume7/figures/fig-c7i2.png new file mode 100644 index 0000000000000000000000000000000000000000..55a907d7a08f50f9be8c0d0c94b060f1d89afe97 --- /dev/null +++ b/processed_dataset/images/volume7/figures/fig-c7i2.png @@ -0,0 +1,3 @@ +version https://git-lfs.github.com/spec/v1 +oid sha256:c71d139b5b9b0ebb0ff6723f078f968f200d30e3127e907658135353e7cf2960 +size 34453 diff --git a/processed_dataset/images/volume7/figures/fig-c7i3.png b/processed_dataset/images/volume7/figures/fig-c7i3.png new file mode 100644 index 0000000000000000000000000000000000000000..e35a98def711098be61c31b626bdc906143cfdba --- /dev/null +++ b/processed_dataset/images/volume7/figures/fig-c7i3.png @@ -0,0 +1,3 @@ +version https://git-lfs.github.com/spec/v1 +oid sha256:d3a2f35c7341793e1962292cf74954d64da0fefb692fb85e8cdbda7610d0599e +size 27393 diff --git a/processed_dataset/images/volume7/figures/fig-c7i4.png b/processed_dataset/images/volume7/figures/fig-c7i4.png new file mode 100644 index 0000000000000000000000000000000000000000..4f9009a64bdd86ea2ba6e831824f553145701b0c --- /dev/null +++ b/processed_dataset/images/volume7/figures/fig-c7i4.png @@ -0,0 +1,3 @@ +version https://git-lfs.github.com/spec/v1 +oid sha256:a468063a240de718117ba1aa0f993cfce5e96e47c23724adc7a6dc4e52790816 +size 23136 diff --git a/processed_dataset/images/volume7/figures/fig-c7i5.png b/processed_dataset/images/volume7/figures/fig-c7i5.png new file mode 100644 index 0000000000000000000000000000000000000000..3fbb491fb5d8e86c9a092db92fb0ba41fb6d67d8 --- /dev/null +++ b/processed_dataset/images/volume7/figures/fig-c7i5.png @@ -0,0 +1,3 @@ +version https://git-lfs.github.com/spec/v1 +oid sha256:7769f417184b279728269b0b8d82955be254d3d15382bc822fb39435b4a2dcd8 +size 30304 diff --git a/processed_dataset/images/volume7/figures/fig-c7i6.png b/processed_dataset/images/volume7/figures/fig-c7i6.png new file mode 100644 index 0000000000000000000000000000000000000000..642ea4e9263911a38e89f61ce7a305a8b5f47174 --- /dev/null +++ b/processed_dataset/images/volume7/figures/fig-c7i6.png @@ -0,0 +1,3 @@ +version https://git-lfs.github.com/spec/v1 +oid sha256:09c5896826d1d2d4124aa1576319bae2def5d7a6c180bd74ee88807b22b3b87f +size 28185 diff --git a/processed_dataset/images/volume7/figures/fig-c7i7.png b/processed_dataset/images/volume7/figures/fig-c7i7.png new file mode 100644 index 0000000000000000000000000000000000000000..e6b53b69b687f581c757b56a2c83a1fc05fe2889 --- /dev/null +++ b/processed_dataset/images/volume7/figures/fig-c7i7.png @@ -0,0 +1,3 @@ +version https://git-lfs.github.com/spec/v1 +oid sha256:0d71331fd3979bda4162e2f26b638fb37c1e7802cc4a39819e6013dc686ea2a0 +size 23247 diff --git a/processed_dataset/images/volume7/figures/fig-c7i8.png b/processed_dataset/images/volume7/figures/fig-c7i8.png new file mode 100644 index 0000000000000000000000000000000000000000..93cf5a250f5a6ee67d0c445658f80c5fa68dfaee --- /dev/null +++ b/processed_dataset/images/volume7/figures/fig-c7i8.png @@ -0,0 +1,3 @@ +version https://git-lfs.github.com/spec/v1 +oid sha256:401b6971cefa0f5e308d438bf091f615da53a259cd704ae60d257573ba6ce1f1 +size 29869 diff --git a/processed_dataset/images/volume7/figures/fig-c7i9.png b/processed_dataset/images/volume7/figures/fig-c7i9.png new file mode 100644 index 0000000000000000000000000000000000000000..e964ab414aa9fb9d0fd839154d4827bbf82dacfa --- /dev/null +++ b/processed_dataset/images/volume7/figures/fig-c7i9.png @@ -0,0 +1,3 @@ +version https://git-lfs.github.com/spec/v1 +oid sha256:90f6aa4e6df277707b539b8d6c27a673edf8f8e57438afd9d7e4efe5dfc72127 +size 38600 diff --git a/processed_dataset/images/volume7/figures/fig-c7p12.png b/processed_dataset/images/volume7/figures/fig-c7p12.png new file mode 100644 index 0000000000000000000000000000000000000000..20d0a2ba3e377a489997845f1da84dc6720a0310 --- /dev/null +++ b/processed_dataset/images/volume7/figures/fig-c7p12.png @@ -0,0 +1,3 @@ +version https://git-lfs.github.com/spec/v1 +oid sha256:cffd10a90136423abadb76d878dff5ba28386955cfdd02774bef2dd7d18afc4c +size 39598 diff --git a/processed_dataset/images/volume7/figures/fig-c8a1.png b/processed_dataset/images/volume7/figures/fig-c8a1.png new file mode 100644 index 0000000000000000000000000000000000000000..5ae546ba40ae467de6d9ac10983a1784357d012d --- /dev/null +++ b/processed_dataset/images/volume7/figures/fig-c8a1.png @@ -0,0 +1,3 @@ +version https://git-lfs.github.com/spec/v1 +oid sha256:cf22bc1f9b8aa5c49247f8315bc233f63b6a0939217917567ad8b5e546628604 +size 29375 diff --git a/processed_dataset/images/volume7/figures/fig-c8a10.png b/processed_dataset/images/volume7/figures/fig-c8a10.png new file mode 100644 index 0000000000000000000000000000000000000000..4e1fc0e6e587b701a87d4c22924d167fd00bf4fb --- /dev/null +++ b/processed_dataset/images/volume7/figures/fig-c8a10.png @@ -0,0 +1,3 @@ +version https://git-lfs.github.com/spec/v1 +oid sha256:d514aae5edca386d9dc8385a290476a94dcbab8fa1b7b3b7e628728430f8e066 +size 31472 diff --git a/processed_dataset/images/volume7/figures/fig-c8a11.png b/processed_dataset/images/volume7/figures/fig-c8a11.png new file mode 100644 index 0000000000000000000000000000000000000000..fbb2310c82b6ac4076f8b687420da6ccc52260ea --- /dev/null +++ b/processed_dataset/images/volume7/figures/fig-c8a11.png @@ -0,0 +1,3 @@ +version https://git-lfs.github.com/spec/v1 +oid sha256:9121a84c98dc111785211fbe26abc7f3d41cf78572e9123e1a3cfc7bdce4ee4c +size 42347 diff --git a/processed_dataset/images/volume7/figures/fig-c8a12.png b/processed_dataset/images/volume7/figures/fig-c8a12.png new file mode 100644 index 0000000000000000000000000000000000000000..a6e91eb0b7658ad6c446629d7cde1349da542a7f --- /dev/null +++ b/processed_dataset/images/volume7/figures/fig-c8a12.png @@ -0,0 +1,3 @@ +version https://git-lfs.github.com/spec/v1 +oid sha256:4c42b23487d54a4f9bfaf75eb5aa99c0d5e254e25d88f722d15af2ffffea83ca +size 27701 diff --git a/processed_dataset/images/volume7/figures/fig-c8a2.png b/processed_dataset/images/volume7/figures/fig-c8a2.png new file mode 100644 index 0000000000000000000000000000000000000000..65da6810b61a7f3b3d30b654976964af31e9e976 --- /dev/null +++ b/processed_dataset/images/volume7/figures/fig-c8a2.png @@ -0,0 +1,3 @@ +version https://git-lfs.github.com/spec/v1 +oid sha256:ea9c25b170ea77f781d786df4945760a0269483059b6c2a019a311ca68b1f7db +size 32513 diff --git a/processed_dataset/images/volume7/figures/fig-c8a3.png b/processed_dataset/images/volume7/figures/fig-c8a3.png new file mode 100644 index 0000000000000000000000000000000000000000..59c40dd0eb5985bce77b7820a914957d6fb92ccd --- /dev/null +++ b/processed_dataset/images/volume7/figures/fig-c8a3.png @@ -0,0 +1,3 @@ +version https://git-lfs.github.com/spec/v1 +oid sha256:fc298cbdc7559cd268b959303256ac1e88e7dca5cd08fa8c6a31faa72c21dfd3 +size 36096 diff --git a/processed_dataset/images/volume7/figures/fig-c8a4.png b/processed_dataset/images/volume7/figures/fig-c8a4.png new file mode 100644 index 0000000000000000000000000000000000000000..ba6729b0087adb6487bbf968df8d19f170b58653 --- /dev/null +++ b/processed_dataset/images/volume7/figures/fig-c8a4.png @@ -0,0 +1,3 @@ +version https://git-lfs.github.com/spec/v1 +oid sha256:ca0964432cd2ef9725dfd520fb87525cf99196b15ace93dc51b9d2dee93b003d +size 42997 diff --git a/processed_dataset/images/volume7/figures/fig-c8a5.png b/processed_dataset/images/volume7/figures/fig-c8a5.png new file mode 100644 index 0000000000000000000000000000000000000000..decf61f5a85d7362d7744a9f0a46c32ed91dd5c0 --- /dev/null +++ b/processed_dataset/images/volume7/figures/fig-c8a5.png @@ -0,0 +1,3 @@ +version https://git-lfs.github.com/spec/v1 +oid sha256:41c56862ed5c3b1548713095e7df89d9ab4e2ad5263bf1389dae67d2aa0198e7 +size 30803 diff --git a/processed_dataset/images/volume7/figures/fig-c8a6.png b/processed_dataset/images/volume7/figures/fig-c8a6.png new file mode 100644 index 0000000000000000000000000000000000000000..5e43ed32c5e7103c84077be13eac777363b4b6fd --- /dev/null +++ b/processed_dataset/images/volume7/figures/fig-c8a6.png @@ -0,0 +1,3 @@ +version https://git-lfs.github.com/spec/v1 +oid sha256:eca204d44130d5a23874f93b3ff2443d8db2e24e94a5ef7af68701a64d2e5ee2 +size 37409 diff --git a/processed_dataset/images/volume7/figures/fig-c8a7.png b/processed_dataset/images/volume7/figures/fig-c8a7.png new file mode 100644 index 0000000000000000000000000000000000000000..04f5005f482a6d198c3549561c55aa94db4fdc88 --- /dev/null +++ b/processed_dataset/images/volume7/figures/fig-c8a7.png @@ -0,0 +1,3 @@ +version https://git-lfs.github.com/spec/v1 +oid sha256:bffd0d2b0fd89907257d42560d24cff4a578fdf40f7adf197233ad60daa97bf3 +size 39298 diff --git a/processed_dataset/images/volume7/figures/fig-c8a8.png b/processed_dataset/images/volume7/figures/fig-c8a8.png new file mode 100644 index 0000000000000000000000000000000000000000..8cb9aecc65642e7b7840e7001684ca687f07e7c8 --- /dev/null +++ b/processed_dataset/images/volume7/figures/fig-c8a8.png @@ -0,0 +1,3 @@ +version https://git-lfs.github.com/spec/v1 +oid sha256:c8272395c9286b5ebda1e77262a52eca00440ba77fed7be2de8f5f9f5ce025fb +size 38533 diff --git a/processed_dataset/images/volume7/figures/fig-c8a9.png b/processed_dataset/images/volume7/figures/fig-c8a9.png new file mode 100644 index 0000000000000000000000000000000000000000..25cafac9be5ed3583cc08dec86978f948518c8d9 --- /dev/null +++ b/processed_dataset/images/volume7/figures/fig-c8a9.png @@ -0,0 +1,3 @@ +version https://git-lfs.github.com/spec/v1 +oid sha256:af26972f09727d8e645b88ffc3a84eeedced6fca9b9aab4fda1ab4c039142645 +size 32222 diff --git a/processed_dataset/images/volume7/figures/fig-c8i1.png b/processed_dataset/images/volume7/figures/fig-c8i1.png new file mode 100644 index 0000000000000000000000000000000000000000..b7ac26bb9dbf8910c9201a4b0decde5c500b905a --- /dev/null +++ b/processed_dataset/images/volume7/figures/fig-c8i1.png @@ -0,0 +1,3 @@ +version https://git-lfs.github.com/spec/v1 +oid sha256:8595dddd2590de7bd6395a6a946af2a6aa71301c86b4da9aa6bbe3c551e00cc3 +size 23601 diff --git a/processed_dataset/images/volume7/figures/fig-c8i10.png b/processed_dataset/images/volume7/figures/fig-c8i10.png new file mode 100644 index 0000000000000000000000000000000000000000..079ee368a8eb28acdee934710328aa235aaa7a26 --- /dev/null +++ b/processed_dataset/images/volume7/figures/fig-c8i10.png @@ -0,0 +1,3 @@ +version https://git-lfs.github.com/spec/v1 +oid sha256:e9a3ea6a9018d5f62a61b7cabd4118438f6dbb2c406b0bc22a3e3bc22e9e66b0 +size 27883 diff --git a/processed_dataset/images/volume7/figures/fig-c8i11.png b/processed_dataset/images/volume7/figures/fig-c8i11.png new file mode 100644 index 0000000000000000000000000000000000000000..eba13bf90915ff5651bbfafa2dbc40f2e6568129 --- /dev/null +++ b/processed_dataset/images/volume7/figures/fig-c8i11.png @@ -0,0 +1,3 @@ +version https://git-lfs.github.com/spec/v1 +oid sha256:36a2a61d36c6fcaec89c624c715ac216207090ba3ce695741a3824085ff7f328 +size 34078 diff --git a/processed_dataset/images/volume7/figures/fig-c8i12.png b/processed_dataset/images/volume7/figures/fig-c8i12.png new file mode 100644 index 0000000000000000000000000000000000000000..26861d921e3bf52162a86531f8e4feb4fadb4f67 --- /dev/null +++ b/processed_dataset/images/volume7/figures/fig-c8i12.png @@ -0,0 +1,3 @@ +version https://git-lfs.github.com/spec/v1 +oid sha256:cd17f0cdabf76ddf76c1efeba8827eda731789610558ef9b7147c58aa9f04d4f +size 30110 diff --git a/processed_dataset/images/volume7/figures/fig-c8i13.png b/processed_dataset/images/volume7/figures/fig-c8i13.png new file mode 100644 index 0000000000000000000000000000000000000000..116dff09070487b2f6c8f24caba0f98d5d7982cb --- /dev/null +++ b/processed_dataset/images/volume7/figures/fig-c8i13.png @@ -0,0 +1,3 @@ +version https://git-lfs.github.com/spec/v1 +oid sha256:c91de3d53b115a6b37427c9d222b28b6c623bd20dcc30456352337e91305de1a +size 32200 diff --git a/processed_dataset/images/volume7/figures/fig-c8i14.png b/processed_dataset/images/volume7/figures/fig-c8i14.png new file mode 100644 index 0000000000000000000000000000000000000000..526e1b6d9394c425ca41631c860443cd7efca24a --- /dev/null +++ b/processed_dataset/images/volume7/figures/fig-c8i14.png @@ -0,0 +1,3 @@ +version https://git-lfs.github.com/spec/v1 +oid sha256:d3c3aa9ed76ade1b9ec907e1011e1307c4e5616baa53d8902d16031e8873d942 +size 65886 diff --git a/processed_dataset/images/volume7/figures/fig-c8i15.png b/processed_dataset/images/volume7/figures/fig-c8i15.png new file mode 100644 index 0000000000000000000000000000000000000000..158446d235011a939c9a82577d65163b90045ffb --- /dev/null +++ b/processed_dataset/images/volume7/figures/fig-c8i15.png @@ -0,0 +1,3 @@ +version https://git-lfs.github.com/spec/v1 +oid sha256:246e905bc6bb1405d32ec999a4d83a41953fe3f001362b33ca5278547f205a7b +size 32752 diff --git a/processed_dataset/images/volume7/figures/fig-c8i16.png b/processed_dataset/images/volume7/figures/fig-c8i16.png new file mode 100644 index 0000000000000000000000000000000000000000..fd8ddc2915cfa01da8701304a128867686c49e6f --- /dev/null +++ b/processed_dataset/images/volume7/figures/fig-c8i16.png @@ -0,0 +1,3 @@ +version https://git-lfs.github.com/spec/v1 +oid sha256:6950522312ca35d0d870107a79ccbbe969f2921caa3b81da204f679f8d5b66ec +size 33091 diff --git a/processed_dataset/images/volume7/figures/fig-c8i17.png b/processed_dataset/images/volume7/figures/fig-c8i17.png new file mode 100644 index 0000000000000000000000000000000000000000..8d082d116c92253a40daf86419fc50edceeba6fd --- /dev/null +++ b/processed_dataset/images/volume7/figures/fig-c8i17.png @@ -0,0 +1,3 @@ +version https://git-lfs.github.com/spec/v1 +oid sha256:80c0272cbfdb89bae339044b5934c95edb034868e32433cba51e22951cc1428f +size 27197 diff --git a/processed_dataset/images/volume7/figures/fig-c8i18.png b/processed_dataset/images/volume7/figures/fig-c8i18.png new file mode 100644 index 0000000000000000000000000000000000000000..f7a1fb0ae3475134f5a6c69704aab140db40550b --- /dev/null +++ b/processed_dataset/images/volume7/figures/fig-c8i18.png @@ -0,0 +1,3 @@ +version https://git-lfs.github.com/spec/v1 +oid sha256:a4c7a55c8060edb7ee08bc7058eb5975694f300d5c0363f16386da58c2d7b3d1 +size 32279 diff --git a/processed_dataset/images/volume7/figures/fig-c8i19.png b/processed_dataset/images/volume7/figures/fig-c8i19.png new file mode 100644 index 0000000000000000000000000000000000000000..fc9303bd0df4097d679b667bc6bd086ec3c2f84a --- /dev/null +++ b/processed_dataset/images/volume7/figures/fig-c8i19.png @@ -0,0 +1,3 @@ +version https://git-lfs.github.com/spec/v1 +oid sha256:e52f0968cc61655a28d83c0b764b71243407eb959b8c5174294584a079a0ea4c +size 39215 diff --git a/processed_dataset/images/volume7/figures/fig-c8i2.png b/processed_dataset/images/volume7/figures/fig-c8i2.png new file mode 100644 index 0000000000000000000000000000000000000000..8b4e547e20e688a8490a8f3a9cf0ea5cdc6d97ed --- /dev/null +++ b/processed_dataset/images/volume7/figures/fig-c8i2.png @@ -0,0 +1,3 @@ +version https://git-lfs.github.com/spec/v1 +oid sha256:0eac6985dd1617b44f40f8231842cf81fab492a7c36a5389364af4c650710a4f +size 42040 diff --git a/processed_dataset/images/volume7/figures/fig-c8i20.png b/processed_dataset/images/volume7/figures/fig-c8i20.png new file mode 100644 index 0000000000000000000000000000000000000000..e1d594772e27fc425c63d58642842b3f14a4f55d --- /dev/null +++ b/processed_dataset/images/volume7/figures/fig-c8i20.png @@ -0,0 +1,3 @@ +version https://git-lfs.github.com/spec/v1 +oid sha256:7ef82e03f64d3de0534c2d32e6bfa7780e19a7bcb2f70a4e3b602aff144edbf7 +size 30444 diff --git a/processed_dataset/images/volume7/figures/fig-c8i21.png b/processed_dataset/images/volume7/figures/fig-c8i21.png new file mode 100644 index 0000000000000000000000000000000000000000..aee51674aa324887ab879aeec8421b5e648a38c8 --- /dev/null +++ b/processed_dataset/images/volume7/figures/fig-c8i21.png @@ -0,0 +1,3 @@ +version https://git-lfs.github.com/spec/v1 +oid sha256:f49cdc219c6ef9d9fcfdec36910198612efdd93c2398f82387e43a67ace1d56c +size 29828 diff --git a/processed_dataset/images/volume7/figures/fig-c8i22.png b/processed_dataset/images/volume7/figures/fig-c8i22.png new file mode 100644 index 0000000000000000000000000000000000000000..9ccedfef5ca8d2017c6a795e01b2cde434b45cf5 --- /dev/null +++ b/processed_dataset/images/volume7/figures/fig-c8i22.png @@ -0,0 +1,3 @@ +version https://git-lfs.github.com/spec/v1 +oid sha256:017512369077d8bbf30fcba23dbcf0cc75c8c223623b86bc5ec78f97c764a81d +size 39236 diff --git a/processed_dataset/images/volume7/figures/fig-c8i3.png b/processed_dataset/images/volume7/figures/fig-c8i3.png new file mode 100644 index 0000000000000000000000000000000000000000..9d3961ad605d4ccff61fae49eb98e61874c4e6a1 --- /dev/null +++ b/processed_dataset/images/volume7/figures/fig-c8i3.png @@ -0,0 +1,3 @@ +version https://git-lfs.github.com/spec/v1 +oid sha256:df7db15227f0a7fd5c28da33d5868dfe366c8465d3ad3dda5ad1907a79ef60ec +size 26148 diff --git a/processed_dataset/images/volume7/figures/fig-c8i4.png b/processed_dataset/images/volume7/figures/fig-c8i4.png new file mode 100644 index 0000000000000000000000000000000000000000..381447967431f728c35b4f49679349f89f62ca23 --- /dev/null +++ b/processed_dataset/images/volume7/figures/fig-c8i4.png @@ -0,0 +1,3 @@ +version https://git-lfs.github.com/spec/v1 +oid sha256:3d54e12381cccbe9db4b85aa20bd01ce2565b7ff71e3ad3c91ae9a8e5ea544e3 +size 30463 diff --git a/processed_dataset/images/volume7/figures/fig-c8i5.png b/processed_dataset/images/volume7/figures/fig-c8i5.png new file mode 100644 index 0000000000000000000000000000000000000000..08df08a977c8f0c8a9e0406417d5f4fe8a6e4192 --- /dev/null +++ b/processed_dataset/images/volume7/figures/fig-c8i5.png @@ -0,0 +1,3 @@ +version https://git-lfs.github.com/spec/v1 +oid sha256:1fcf8b09c4b3b5f15ec158ce9803ff103fe86f95598ff587d0695a296b1b3c2d +size 19222 diff --git a/processed_dataset/images/volume7/figures/fig-c8i6.png b/processed_dataset/images/volume7/figures/fig-c8i6.png new file mode 100644 index 0000000000000000000000000000000000000000..3ab8ebff277dd0e080a696058d37f273ae065a57 --- /dev/null +++ b/processed_dataset/images/volume7/figures/fig-c8i6.png @@ -0,0 +1,3 @@ +version https://git-lfs.github.com/spec/v1 +oid sha256:c7bdeca05efd299f9607330e3e732d40ccf1168c47039b281ee5ac499e6118f4 +size 24386 diff --git a/processed_dataset/images/volume7/figures/fig-c8i7.png b/processed_dataset/images/volume7/figures/fig-c8i7.png new file mode 100644 index 0000000000000000000000000000000000000000..52adb5508662d07b937b3d94190ce1531cc04759 --- /dev/null +++ b/processed_dataset/images/volume7/figures/fig-c8i7.png @@ -0,0 +1,3 @@ +version https://git-lfs.github.com/spec/v1 +oid sha256:8e2608bacc16f4c58b2cc31d1eec621e93f7a7d2cfac2594391c53412888e366 +size 24628 diff --git a/processed_dataset/images/volume7/figures/fig-c8i8.png b/processed_dataset/images/volume7/figures/fig-c8i8.png new file mode 100644 index 0000000000000000000000000000000000000000..7021d9715d4ec38613c0171c6fd922fc8f627ce6 --- /dev/null +++ b/processed_dataset/images/volume7/figures/fig-c8i8.png @@ -0,0 +1,3 @@ +version https://git-lfs.github.com/spec/v1 +oid sha256:3234a1239bb912801aef606f743d90ea59d4b7b0b7f4053bd0c07a069d89f583 +size 17515 diff --git a/processed_dataset/images/volume7/figures/fig-c8i9.png b/processed_dataset/images/volume7/figures/fig-c8i9.png new file mode 100644 index 0000000000000000000000000000000000000000..6cd3e8248e82b6324c73f0359b2de5675ff0cc70 --- /dev/null +++ b/processed_dataset/images/volume7/figures/fig-c8i9.png @@ -0,0 +1,3 @@ +version https://git-lfs.github.com/spec/v1 +oid sha256:494085f6042b9554c0f21c2a800f98d6f8cca5813a0ab0ba6b6143363e685315 +size 18948 diff --git a/processed_dataset/images/volume7/figures/fig-c8p10.png b/processed_dataset/images/volume7/figures/fig-c8p10.png new file mode 100644 index 0000000000000000000000000000000000000000..a42d7b75586f3c1c2e5c06124847cf2fa8ef1c5d --- /dev/null +++ b/processed_dataset/images/volume7/figures/fig-c8p10.png @@ -0,0 +1,3 @@ +version https://git-lfs.github.com/spec/v1 +oid sha256:0e8f65932f229b92b08375ddb9039a105cbf1d6ba8524c06f1cba57307439270 +size 38706 diff --git a/processed_dataset/images/volume7/figures/fig-c8p11.png b/processed_dataset/images/volume7/figures/fig-c8p11.png new file mode 100644 index 0000000000000000000000000000000000000000..8f04e2545848845d09d34339fdfe6f5ff9fe5ba5 --- /dev/null +++ b/processed_dataset/images/volume7/figures/fig-c8p11.png @@ -0,0 +1,3 @@ +version https://git-lfs.github.com/spec/v1 +oid sha256:ce0e64a8629a0345ce8ab9933e899eddf2dccc591bbdcaa737a4003749f4257b +size 38916 diff --git a/processed_dataset/images/volume7/figures/fig-c8p12.png b/processed_dataset/images/volume7/figures/fig-c8p12.png new file mode 100644 index 0000000000000000000000000000000000000000..db6b42211cbf5caa8e462b3ac3fdb99a034b9e6e --- /dev/null +++ b/processed_dataset/images/volume7/figures/fig-c8p12.png @@ -0,0 +1,3 @@ +version https://git-lfs.github.com/spec/v1 +oid sha256:dc4131dd04b744a44c30ef9509164d603724242cb53f26b0c43176d2c431736f +size 26299 diff --git a/processed_dataset/images/volume7/figures/fig-c8p3.png b/processed_dataset/images/volume7/figures/fig-c8p3.png new file mode 100644 index 0000000000000000000000000000000000000000..fa92edfa4214c8bea48a24f53d7a2d9da6449377 --- /dev/null +++ b/processed_dataset/images/volume7/figures/fig-c8p3.png @@ -0,0 +1,3 @@ +version https://git-lfs.github.com/spec/v1 +oid sha256:a699c143aa4156c2bb9ff205c9b9a3ef45728c93e5c8bad8ae718f3885dda4f4 +size 26751 diff --git a/processed_dataset/images/volume7/figures/fig-c8p5.png b/processed_dataset/images/volume7/figures/fig-c8p5.png new file mode 100644 index 0000000000000000000000000000000000000000..1ceb6b4af941a9d641cc098326b36846d2552ccc --- /dev/null +++ b/processed_dataset/images/volume7/figures/fig-c8p5.png @@ -0,0 +1,3 @@ +version https://git-lfs.github.com/spec/v1 +oid sha256:a44dc3a3c7fa6c5cd7e13869d38b1396ab12c37b21384111bede8daf97852ed9 +size 26162 diff --git a/processed_dataset/images/volume7/figures/fig-c9a1.png b/processed_dataset/images/volume7/figures/fig-c9a1.png new file mode 100644 index 0000000000000000000000000000000000000000..4d95c54f51cd510be8f06b7ff779a117771671c1 --- /dev/null +++ b/processed_dataset/images/volume7/figures/fig-c9a1.png @@ -0,0 +1,3 @@ +version https://git-lfs.github.com/spec/v1 +oid sha256:25b78e55a058636d5fa22f50af2c262fc10b0f6c367ffa1b386e0fc5464245bf +size 47644 diff --git a/processed_dataset/images/volume7/figures/fig-c9a10.png b/processed_dataset/images/volume7/figures/fig-c9a10.png new file mode 100644 index 0000000000000000000000000000000000000000..ef5ce10253212d2ca3a20308551d5117624f06e5 --- /dev/null +++ b/processed_dataset/images/volume7/figures/fig-c9a10.png @@ -0,0 +1,3 @@ +version https://git-lfs.github.com/spec/v1 +oid sha256:f8f7e25744436f85ccf6e6789c7aa93b0d8d0415866b75c405cee7bb4a6d6bcd +size 28620 diff --git a/processed_dataset/images/volume7/figures/fig-c9a11.png b/processed_dataset/images/volume7/figures/fig-c9a11.png new file mode 100644 index 0000000000000000000000000000000000000000..076dd9ea5d187fde0971ce08cb17462900eedd93 --- /dev/null +++ b/processed_dataset/images/volume7/figures/fig-c9a11.png @@ -0,0 +1,3 @@ +version https://git-lfs.github.com/spec/v1 +oid sha256:697685cd371791e4608b7cca901355de67d3b75fe5617d338bdcfd2853a070ee +size 44075 diff --git a/processed_dataset/images/volume7/figures/fig-c9a12.png b/processed_dataset/images/volume7/figures/fig-c9a12.png new file mode 100644 index 0000000000000000000000000000000000000000..1435c781689148948751434dc6c5264a85e27860 --- /dev/null +++ b/processed_dataset/images/volume7/figures/fig-c9a12.png @@ -0,0 +1,3 @@ +version https://git-lfs.github.com/spec/v1 +oid sha256:f78c91bbcaff539a5279703cb7b93a1b59a07a7ca99c0d7846780ce22007faf0 +size 39625 diff --git a/processed_dataset/images/volume7/figures/fig-c9a2.png b/processed_dataset/images/volume7/figures/fig-c9a2.png new file mode 100644 index 0000000000000000000000000000000000000000..095333fbd038ae83c265dfe88c48725d898e9464 --- /dev/null +++ b/processed_dataset/images/volume7/figures/fig-c9a2.png @@ -0,0 +1,3 @@ +version https://git-lfs.github.com/spec/v1 +oid sha256:797417919172625f3679386f244c980607532078158865acc808442a6b2bb51c +size 45813 diff --git a/processed_dataset/images/volume7/figures/fig-c9a3.png b/processed_dataset/images/volume7/figures/fig-c9a3.png new file mode 100644 index 0000000000000000000000000000000000000000..058cf422e535a2ec5e0cbc42e0be64e9338a8b78 --- /dev/null +++ b/processed_dataset/images/volume7/figures/fig-c9a3.png @@ -0,0 +1,3 @@ +version https://git-lfs.github.com/spec/v1 +oid sha256:8e42aab3f776533ff59b7d66da7a287cc4aafc7349f8e85ae892c9f88419c0be +size 28117 diff --git a/processed_dataset/images/volume7/figures/fig-c9a4-1.png b/processed_dataset/images/volume7/figures/fig-c9a4-1.png new file mode 100644 index 0000000000000000000000000000000000000000..2002a4a34c211bb5e95ed6dbeb27a1f8ed6be9de --- /dev/null +++ b/processed_dataset/images/volume7/figures/fig-c9a4-1.png @@ -0,0 +1,3 @@ +version https://git-lfs.github.com/spec/v1 +oid sha256:ad0c2b51f41ee26361e8d76be90b79d6cc2cfb9fe77150d8e852f9597acdf9fe +size 45538 diff --git a/processed_dataset/images/volume7/figures/fig-c9a4-2.png b/processed_dataset/images/volume7/figures/fig-c9a4-2.png new file mode 100644 index 0000000000000000000000000000000000000000..b91a378c6169457736876a478a2627397a590a4b --- /dev/null +++ b/processed_dataset/images/volume7/figures/fig-c9a4-2.png @@ -0,0 +1,3 @@ +version https://git-lfs.github.com/spec/v1 +oid sha256:8114deb1dad6109d877038fed2bc13e182499514300ebb506a8397b83b05ba4c +size 29044 diff --git a/processed_dataset/images/volume7/figures/fig-c9a5.png b/processed_dataset/images/volume7/figures/fig-c9a5.png new file mode 100644 index 0000000000000000000000000000000000000000..70813abe2b48c889a5a573e0a9ea132b64e954c7 --- /dev/null +++ b/processed_dataset/images/volume7/figures/fig-c9a5.png @@ -0,0 +1,3 @@ +version https://git-lfs.github.com/spec/v1 +oid sha256:1abd29660d00976f89c08507338a16608d06b9bfdf47214ee71ed33616c80d80 +size 31846 diff --git a/processed_dataset/images/volume7/figures/fig-c9a6-1.png b/processed_dataset/images/volume7/figures/fig-c9a6-1.png new file mode 100644 index 0000000000000000000000000000000000000000..7ba1887794b751098849010a194286afcbbdadb6 --- /dev/null +++ b/processed_dataset/images/volume7/figures/fig-c9a6-1.png @@ -0,0 +1,3 @@ +version https://git-lfs.github.com/spec/v1 +oid sha256:d4bf021477a233e30b3075856297a57c398db8369233fd07ab427396fbaa8cf8 +size 29026 diff --git a/processed_dataset/images/volume7/figures/fig-c9a6-2.png b/processed_dataset/images/volume7/figures/fig-c9a6-2.png new file mode 100644 index 0000000000000000000000000000000000000000..657720fcc86326b606e8cf2ec30144bc841ce332 --- /dev/null +++ b/processed_dataset/images/volume7/figures/fig-c9a6-2.png @@ -0,0 +1,3 @@ +version https://git-lfs.github.com/spec/v1 +oid sha256:62204ac8306fc2b768d1f8a7fa59088c4a1550d7ebe9a903a6c6cc4019c160c2 +size 39238 diff --git a/processed_dataset/images/volume7/figures/fig-c9a7.png b/processed_dataset/images/volume7/figures/fig-c9a7.png new file mode 100644 index 0000000000000000000000000000000000000000..4b29b141e22ec4c45bdb9db68ccef5083954c22c --- /dev/null +++ b/processed_dataset/images/volume7/figures/fig-c9a7.png @@ -0,0 +1,3 @@ +version https://git-lfs.github.com/spec/v1 +oid sha256:90f4aa1d73132e2983ba74838e30ec43133895a1660699f8189af101a57b1e68 +size 35656 diff --git a/processed_dataset/images/volume7/figures/fig-c9a8.png b/processed_dataset/images/volume7/figures/fig-c9a8.png new file mode 100644 index 0000000000000000000000000000000000000000..78fc01ee1f7fb024ad1d0dd4f83010298314cdad --- /dev/null +++ b/processed_dataset/images/volume7/figures/fig-c9a8.png @@ -0,0 +1,3 @@ +version https://git-lfs.github.com/spec/v1 +oid sha256:d5e5c2128b36c75bf543decfafcec76be81123be2dfea1c0a56f90dbc90e6d55 +size 51383 diff --git a/processed_dataset/images/volume7/figures/fig-c9a9.png b/processed_dataset/images/volume7/figures/fig-c9a9.png new file mode 100644 index 0000000000000000000000000000000000000000..ba44262eee8b54b056d31ecdabcaab4381f16e7d --- /dev/null +++ b/processed_dataset/images/volume7/figures/fig-c9a9.png @@ -0,0 +1,3 @@ +version https://git-lfs.github.com/spec/v1 +oid sha256:3d383c39f8db4a193ec952ec9b300d18ecee4b0c78ad1748a7405878a38f9604 +size 47754 diff --git a/processed_dataset/images/volume7/figures/fig-c9i1.png b/processed_dataset/images/volume7/figures/fig-c9i1.png new file mode 100644 index 0000000000000000000000000000000000000000..50f1b60a57292528d25b5eb749c0b196486c1045 --- /dev/null +++ b/processed_dataset/images/volume7/figures/fig-c9i1.png @@ -0,0 +1,3 @@ +version https://git-lfs.github.com/spec/v1 +oid sha256:7e11cd007d6bbab5ccbf2fc40ecc7e884b49ec210fdec10236d04b35c3dccb5d +size 43896 diff --git a/processed_dataset/images/volume7/figures/fig-c9i10.png b/processed_dataset/images/volume7/figures/fig-c9i10.png new file mode 100644 index 0000000000000000000000000000000000000000..2d4e88c6bb9fa5164f5c0ccd607979083de1435d --- /dev/null +++ b/processed_dataset/images/volume7/figures/fig-c9i10.png @@ -0,0 +1,3 @@ +version https://git-lfs.github.com/spec/v1 +oid sha256:d9d5da4c41550f353bf6253e369d1ca898d56f1d6cb21a5d13762cf6019902d6 +size 40590 diff --git a/processed_dataset/images/volume7/figures/fig-c9i11.png b/processed_dataset/images/volume7/figures/fig-c9i11.png new file mode 100644 index 0000000000000000000000000000000000000000..f78acfb04a4332fa8b1bc4b05c03300907905797 --- /dev/null +++ b/processed_dataset/images/volume7/figures/fig-c9i11.png @@ -0,0 +1,3 @@ +version https://git-lfs.github.com/spec/v1 +oid sha256:3071ebfebc8853c439b4071d5f7f5b8834d4e4c7352c0f7aff8ab60c080e694e +size 34788 diff --git a/processed_dataset/images/volume7/figures/fig-c9i12.png b/processed_dataset/images/volume7/figures/fig-c9i12.png new file mode 100644 index 0000000000000000000000000000000000000000..5da0db8031c28068eeb9d97d2e55ef6d40949fcb --- /dev/null +++ b/processed_dataset/images/volume7/figures/fig-c9i12.png @@ -0,0 +1,3 @@ +version https://git-lfs.github.com/spec/v1 +oid sha256:070e9f9d3b93f726a76ced378693f396f8c49c8752eead980c791a2dbaaef969 +size 42555 diff --git a/processed_dataset/images/volume7/figures/fig-c9i13.png b/processed_dataset/images/volume7/figures/fig-c9i13.png new file mode 100644 index 0000000000000000000000000000000000000000..437d9b861dfbc67f52461754425e2bd957882d92 --- /dev/null +++ b/processed_dataset/images/volume7/figures/fig-c9i13.png @@ -0,0 +1,3 @@ +version https://git-lfs.github.com/spec/v1 +oid sha256:4182690b87d924430431d73e8b41c4dae4cbef5f6e00ef72c9997e9d9e7ba771 +size 27073 diff --git a/processed_dataset/images/volume7/figures/fig-c9i14.png b/processed_dataset/images/volume7/figures/fig-c9i14.png new file mode 100644 index 0000000000000000000000000000000000000000..b0a95e9c35a34a76c51019af271e5b7ee915825f --- /dev/null +++ b/processed_dataset/images/volume7/figures/fig-c9i14.png @@ -0,0 +1,3 @@ +version https://git-lfs.github.com/spec/v1 +oid sha256:c83a70e010b5d6c043f00bf1597be9c51d0a9a514aa3c1c780feb9831fa90c34 +size 45476 diff --git a/processed_dataset/images/volume7/figures/fig-c9i15.png b/processed_dataset/images/volume7/figures/fig-c9i15.png new file mode 100644 index 0000000000000000000000000000000000000000..2c71108ae5a8c5d9d72ec8dbfcda58dfaf908b31 --- /dev/null +++ b/processed_dataset/images/volume7/figures/fig-c9i15.png @@ -0,0 +1,3 @@ +version https://git-lfs.github.com/spec/v1 +oid sha256:186f711b7c49e2917ae1cf8caa6f90f09672fcbbd9a9b0d3e1a5856696822ec9 +size 40879 diff --git a/processed_dataset/images/volume7/figures/fig-c9i16.png b/processed_dataset/images/volume7/figures/fig-c9i16.png new file mode 100644 index 0000000000000000000000000000000000000000..c14eef243b0b85d1bb6e2d34b07ce35d7411904f --- /dev/null +++ b/processed_dataset/images/volume7/figures/fig-c9i16.png @@ -0,0 +1,3 @@ +version https://git-lfs.github.com/spec/v1 +oid sha256:45a958066c4b59dee3f6c3480e8b76b68ffc2926f5e5affb766c7a2c2c4f1356 +size 40692 diff --git a/processed_dataset/images/volume7/figures/fig-c9i17.png b/processed_dataset/images/volume7/figures/fig-c9i17.png new file mode 100644 index 0000000000000000000000000000000000000000..cfe8b018f919fbf4d0d48eacc8176cd9e2a5f893 --- /dev/null +++ b/processed_dataset/images/volume7/figures/fig-c9i17.png @@ -0,0 +1,3 @@ +version https://git-lfs.github.com/spec/v1 +oid sha256:1c4b2585c2e054cfb22ab5df18b08eb8535505be9e14eb1a55e30eaff492400d +size 38465 diff --git a/processed_dataset/images/volume7/figures/fig-c9i18.png b/processed_dataset/images/volume7/figures/fig-c9i18.png new file mode 100644 index 0000000000000000000000000000000000000000..daea89b115de5a7a35653e0c9dd56985c9f4f9ca --- /dev/null +++ b/processed_dataset/images/volume7/figures/fig-c9i18.png @@ -0,0 +1,3 @@ +version https://git-lfs.github.com/spec/v1 +oid sha256:f036bd14ba0cafd5e96a85b422d7cf7ff7742f9fb57c5ed3539e27fc03125d29 +size 49739 diff --git a/processed_dataset/images/volume7/figures/fig-c9i19.png b/processed_dataset/images/volume7/figures/fig-c9i19.png new file mode 100644 index 0000000000000000000000000000000000000000..1cb990b8e06e54cbd5452aed4c1ddcd2d1bf9955 --- /dev/null +++ b/processed_dataset/images/volume7/figures/fig-c9i19.png @@ -0,0 +1,3 @@ +version https://git-lfs.github.com/spec/v1 +oid sha256:053ce557548ee55fb0d5751b0f15b836af13336e21958ad1caf33a469c72bb3c +size 33213 diff --git a/processed_dataset/images/volume7/figures/fig-c9i2.png b/processed_dataset/images/volume7/figures/fig-c9i2.png new file mode 100644 index 0000000000000000000000000000000000000000..a05a390c467f4d17b175874d7c1260a144085467 --- /dev/null +++ b/processed_dataset/images/volume7/figures/fig-c9i2.png @@ -0,0 +1,3 @@ +version https://git-lfs.github.com/spec/v1 +oid sha256:990e31a10b61a2d38d168deafc8d9d491a6839646b227dfe88667d10ff0d55c6 +size 26450 diff --git a/processed_dataset/images/volume7/figures/fig-c9i20.png b/processed_dataset/images/volume7/figures/fig-c9i20.png new file mode 100644 index 0000000000000000000000000000000000000000..358477a85068f2e269858ce59476e9737e1d9b79 --- /dev/null +++ b/processed_dataset/images/volume7/figures/fig-c9i20.png @@ -0,0 +1,3 @@ +version https://git-lfs.github.com/spec/v1 +oid sha256:eed6706c033880434213d0f598433d249a46301f4c70cb834ecf3aad151fcb60 +size 37302 diff --git a/processed_dataset/images/volume7/figures/fig-c9i3.png b/processed_dataset/images/volume7/figures/fig-c9i3.png new file mode 100644 index 0000000000000000000000000000000000000000..dc62140c0db4f4576615e7c422a0a1d74105d532 --- /dev/null +++ b/processed_dataset/images/volume7/figures/fig-c9i3.png @@ -0,0 +1,3 @@ +version https://git-lfs.github.com/spec/v1 +oid sha256:3c652df8900f5fd60505f949cd8f6355276fe6e7a3064f0cdcb7f170371c6aab +size 38582 diff --git a/processed_dataset/images/volume7/figures/fig-c9i4.png b/processed_dataset/images/volume7/figures/fig-c9i4.png new file mode 100644 index 0000000000000000000000000000000000000000..76eec7b795d40e8e5970a976e09b6ffa23d109dd --- /dev/null +++ b/processed_dataset/images/volume7/figures/fig-c9i4.png @@ -0,0 +1,3 @@ +version https://git-lfs.github.com/spec/v1 +oid sha256:176103740da62176b6c89df2bd9f18e1bedd66c925d32b8019c7bd55c092b6ed +size 26868 diff --git a/processed_dataset/images/volume7/figures/fig-c9i5.png b/processed_dataset/images/volume7/figures/fig-c9i5.png new file mode 100644 index 0000000000000000000000000000000000000000..ba2b3e4b3b1272e62c15556ca40a904a082a5b26 --- /dev/null +++ b/processed_dataset/images/volume7/figures/fig-c9i5.png @@ -0,0 +1,3 @@ +version https://git-lfs.github.com/spec/v1 +oid sha256:a66c2beab87cece3f384f162006f2c4b5fdd3bc92479785e65723281f3309003 +size 33898 diff --git a/processed_dataset/images/volume7/figures/fig-c9i6.png b/processed_dataset/images/volume7/figures/fig-c9i6.png new file mode 100644 index 0000000000000000000000000000000000000000..e8b49cb6febc44c4dfbcca0aa944920ffa833186 --- /dev/null +++ b/processed_dataset/images/volume7/figures/fig-c9i6.png @@ -0,0 +1,3 @@ +version https://git-lfs.github.com/spec/v1 +oid sha256:610ea1cdda6eee349dd4672736f1f2e3131b29cc56d99296e7b335f8a601212e +size 24422 diff --git a/processed_dataset/images/volume7/figures/fig-c9i7.png b/processed_dataset/images/volume7/figures/fig-c9i7.png new file mode 100644 index 0000000000000000000000000000000000000000..97f323303d36a691ed6080546eb509410c4f9da1 --- /dev/null +++ b/processed_dataset/images/volume7/figures/fig-c9i7.png @@ -0,0 +1,3 @@ +version https://git-lfs.github.com/spec/v1 +oid sha256:aa22a71d712e01b419684f283121f4d1a5d1f7b2329c5e735e0483d719f98cfe +size 20449 diff --git a/processed_dataset/images/volume7/figures/fig-c9i8.png b/processed_dataset/images/volume7/figures/fig-c9i8.png new file mode 100644 index 0000000000000000000000000000000000000000..fe4954663d4904bac98ddb4bc60c582e37255c96 --- /dev/null +++ b/processed_dataset/images/volume7/figures/fig-c9i8.png @@ -0,0 +1,3 @@ +version https://git-lfs.github.com/spec/v1 +oid sha256:21876860b65538daa70c3276d79852d91c91c00dd65b5c12c24d7e86941aec37 +size 32509 diff --git a/processed_dataset/images/volume7/figures/fig-c9i9.png b/processed_dataset/images/volume7/figures/fig-c9i9.png new file mode 100644 index 0000000000000000000000000000000000000000..b775efb0c1fb6a6de236227c35b53a3430c3462c --- /dev/null +++ b/processed_dataset/images/volume7/figures/fig-c9i9.png @@ -0,0 +1,3 @@ +version https://git-lfs.github.com/spec/v1 +oid sha256:6adb12b565af892a322ab96d5601eb97abcb371231277cd9c61a8aa146432b63 +size 31297 diff --git a/processed_dataset/images/volume7/figures/fig-c9p6.png b/processed_dataset/images/volume7/figures/fig-c9p6.png new file mode 100644 index 0000000000000000000000000000000000000000..fcd6d07f62acbf602fd20d1299615b372bb240be --- /dev/null +++ b/processed_dataset/images/volume7/figures/fig-c9p6.png @@ -0,0 +1,3 @@ +version https://git-lfs.github.com/spec/v1 +oid sha256:4814a99bd458e57f2aa4771cf74b71125963a88a89584abe9b9f5a396690c2a7 +size 20159 diff --git a/processed_dataset/images/volume7/figures/fig-c9p7.png b/processed_dataset/images/volume7/figures/fig-c9p7.png new file mode 100644 index 0000000000000000000000000000000000000000..4df2e33f80a3e6d76bdfc4b6667e3785bc62b6dd --- /dev/null +++ b/processed_dataset/images/volume7/figures/fig-c9p7.png @@ -0,0 +1,3 @@ +version https://git-lfs.github.com/spec/v1 +oid sha256:0af3100d72d9779f711aa2ada9dec5f1277087a9b785c6f585ca819c1bb5f776 +size 29082 diff --git a/processed_dataset/images/volume8/figures/fig-c2a2.png b/processed_dataset/images/volume8/figures/fig-c2a2.png new file mode 100644 index 0000000000000000000000000000000000000000..06ac259a56249a9d4e239ab8e74692ff2ae5394c --- /dev/null +++ b/processed_dataset/images/volume8/figures/fig-c2a2.png @@ -0,0 +1,3 @@ +version https://git-lfs.github.com/spec/v1 +oid sha256:00293f2692135d0c95fc2953ff8adbb875dd9927dc9075367098450e50c39d56 +size 16364 diff --git a/processed_dataset/images/volume8/figures/fig-c2a3-1.png b/processed_dataset/images/volume8/figures/fig-c2a3-1.png new file mode 100644 index 0000000000000000000000000000000000000000..fe7dc419cffdd0ffa09cbef883e9aedc76373b5a --- /dev/null +++ b/processed_dataset/images/volume8/figures/fig-c2a3-1.png @@ -0,0 +1,3 @@ +version https://git-lfs.github.com/spec/v1 +oid sha256:44d6d611ead747c1b5ab7127b40dfd8962a66ff93628143cc8c6ba9b431c45b4 +size 14853 diff --git a/processed_dataset/images/volume8/figures/fig-c2a3-2.png b/processed_dataset/images/volume8/figures/fig-c2a3-2.png new file mode 100644 index 0000000000000000000000000000000000000000..d77e9913a3e7886ca8f0f2f082958fc1712b9162 --- /dev/null +++ b/processed_dataset/images/volume8/figures/fig-c2a3-2.png @@ -0,0 +1,3 @@ +version https://git-lfs.github.com/spec/v1 +oid sha256:65526a0178a8b9e2dd9699bb69b6f9dda6c083b7eaee29df440023f8d3a867ec +size 15706 diff --git a/processed_dataset/images/volume8/figures/fig-c2a7.png b/processed_dataset/images/volume8/figures/fig-c2a7.png new file mode 100644 index 0000000000000000000000000000000000000000..770bec66a56457048dd1f6411f42d8d5f9db5b49 --- /dev/null +++ b/processed_dataset/images/volume8/figures/fig-c2a7.png @@ -0,0 +1,3 @@ +version https://git-lfs.github.com/spec/v1 +oid sha256:7d3df7001c6992782793327673c053170c7b234a8b5b8e9a3b50057a1b25c0d1 +size 12821 diff --git a/processed_dataset/images/volume8/figures/fig-c4i1.png b/processed_dataset/images/volume8/figures/fig-c4i1.png new file mode 100644 index 0000000000000000000000000000000000000000..185e381b4d1e98609620a118501f110b6f2567af --- /dev/null +++ b/processed_dataset/images/volume8/figures/fig-c4i1.png @@ -0,0 +1,3 @@ +version https://git-lfs.github.com/spec/v1 +oid sha256:72d957b1a79b18e7ee1cd22ea961afccff189bcbc796bfa4b0f15b25326ee7f7 +size 12810 diff --git a/processed_dataset/images/volume8/figures/fig-c4i2.png b/processed_dataset/images/volume8/figures/fig-c4i2.png new file mode 100644 index 0000000000000000000000000000000000000000..586ac088ad76ed596998b95ca0f81977af6d9213 --- /dev/null +++ b/processed_dataset/images/volume8/figures/fig-c4i2.png @@ -0,0 +1,3 @@ +version https://git-lfs.github.com/spec/v1 +oid sha256:77b0c91c8f2c988f0b7d0e2abac8ae1da23186cc8cb4a0f6a3e68a315f01a4f8 +size 16833 diff --git a/processed_dataset/images/volume8/figures/fig-c4i3.png b/processed_dataset/images/volume8/figures/fig-c4i3.png new file mode 100644 index 0000000000000000000000000000000000000000..538c7c4025a65339e9d8ccd481841e2594727a1e --- /dev/null +++ b/processed_dataset/images/volume8/figures/fig-c4i3.png @@ -0,0 +1,3 @@ +version https://git-lfs.github.com/spec/v1 +oid sha256:4eae2aa905939524efce8d53f1a35699ea90e132f2c5fde6f46cc8de3dbf6d31 +size 16270 diff --git a/processed_dataset/images/volume8/figures/fig-c4p6.png b/processed_dataset/images/volume8/figures/fig-c4p6.png new file mode 100644 index 0000000000000000000000000000000000000000..68b6f3ac3d5243201f6cf90e4cf69cd24307bf48 --- /dev/null +++ b/processed_dataset/images/volume8/figures/fig-c4p6.png @@ -0,0 +1,3 @@ +version https://git-lfs.github.com/spec/v1 +oid sha256:a8bc1e3a028644add2559aa5acdf54e7c27f4ae2d8b93682dd22b4cf1289fdf2 +size 17821 diff --git a/processed_dataset/images/volume8/figures/fig-c5i1.png b/processed_dataset/images/volume8/figures/fig-c5i1.png new file mode 100644 index 0000000000000000000000000000000000000000..3aadfa2c35c527d7e7805cb17d24439515153612 --- /dev/null +++ b/processed_dataset/images/volume8/figures/fig-c5i1.png @@ -0,0 +1,3 @@ +version https://git-lfs.github.com/spec/v1 +oid sha256:00db1687e1347c3354c93bd2a68b38bdefec98aac65a5c90ec61cd2a50c4430e +size 22317 diff --git a/processed_dataset/images/volume8/figures/fig-c5i2.png b/processed_dataset/images/volume8/figures/fig-c5i2.png new file mode 100644 index 0000000000000000000000000000000000000000..4871728ea7a3cd1cbdeaa4536a9dad4df7f7a48e --- /dev/null +++ b/processed_dataset/images/volume8/figures/fig-c5i2.png @@ -0,0 +1,3 @@ +version https://git-lfs.github.com/spec/v1 +oid sha256:710a13c118ef9589cf098f23bc34fff4f514edd5d3ea16bb21a173c0ff6d959b +size 12639 diff --git a/processed_dataset/images/volume8/figures/fig-c5i3.png b/processed_dataset/images/volume8/figures/fig-c5i3.png new file mode 100644 index 0000000000000000000000000000000000000000..a09351c960665dbf897181928f96b2de9b78e47c --- /dev/null +++ b/processed_dataset/images/volume8/figures/fig-c5i3.png @@ -0,0 +1,3 @@ +version https://git-lfs.github.com/spec/v1 +oid sha256:ad1ee753f84b7bf3747ffa5ae7780ad8f7fe2ce81fbb46dffcbc55e3804f83db +size 17788 diff --git a/processed_dataset/images/volume8/figures/fig-c5p4.png b/processed_dataset/images/volume8/figures/fig-c5p4.png new file mode 100644 index 0000000000000000000000000000000000000000..2069f4ba45cb4dbb3aab7c0a9e31364695e35e60 --- /dev/null +++ b/processed_dataset/images/volume8/figures/fig-c5p4.png @@ -0,0 +1,3 @@ +version https://git-lfs.github.com/spec/v1 +oid sha256:aa64ec1a0e694f4191f1f70faf87645d4195dfca543f9a4adbc6628da6e1e1e0 +size 15429 diff --git a/processed_dataset/images/volume8/figures/fig-c5p9.png b/processed_dataset/images/volume8/figures/fig-c5p9.png new file mode 100644 index 0000000000000000000000000000000000000000..dfc52ff465a08914282fb04ad7ee7844b3b0eeb4 --- /dev/null +++ b/processed_dataset/images/volume8/figures/fig-c5p9.png @@ -0,0 +1,3 @@ +version https://git-lfs.github.com/spec/v1 +oid sha256:a0fe9977c20e6d78c222d7dfdf47fba0a85f47bd33bea5763fc59705c8dcccd6 +size 15592 diff --git a/processed_dataset/images/volume8/figures/fig-c6a1.png b/processed_dataset/images/volume8/figures/fig-c6a1.png new file mode 100644 index 0000000000000000000000000000000000000000..ed68beda99cb97a8c39762cf18b25d8240f2cdf4 --- /dev/null +++ b/processed_dataset/images/volume8/figures/fig-c6a1.png @@ -0,0 +1,3 @@ +version https://git-lfs.github.com/spec/v1 +oid sha256:e5a768d2e681297d4ecfb13f0772b24d3b0ce8671f8fe7afa58ae17cb928d717 +size 21436 diff --git a/processed_dataset/images/volume8/figures/fig-c6a3.png b/processed_dataset/images/volume8/figures/fig-c6a3.png new file mode 100644 index 0000000000000000000000000000000000000000..8108491c3fd606de503dcbee59c111141bfffd04 --- /dev/null +++ b/processed_dataset/images/volume8/figures/fig-c6a3.png @@ -0,0 +1,3 @@ +version https://git-lfs.github.com/spec/v1 +oid sha256:043baf74b249f6ee43af4688905beecc86dd9e9f994f0077af10b6eca1a8aff1 +size 23483 diff --git a/processed_dataset/images/volume8/figures/fig-c6a4.png b/processed_dataset/images/volume8/figures/fig-c6a4.png new file mode 100644 index 0000000000000000000000000000000000000000..963960412d39772c7f6df079f5065913c6043ad9 --- /dev/null +++ b/processed_dataset/images/volume8/figures/fig-c6a4.png @@ -0,0 +1,3 @@ +version https://git-lfs.github.com/spec/v1 +oid sha256:8e7565b7c3638c30ea2d08df56d42a4cbc823e2725ffe6b8e9afe225c8ee55c2 +size 18150 diff --git a/processed_dataset/images/volume8/figures/fig-c6i1.png b/processed_dataset/images/volume8/figures/fig-c6i1.png new file mode 100644 index 0000000000000000000000000000000000000000..cf9cfa5d9ae1ab2aa7ac120de888de6a425bcb0c --- /dev/null +++ b/processed_dataset/images/volume8/figures/fig-c6i1.png @@ -0,0 +1,3 @@ +version https://git-lfs.github.com/spec/v1 +oid sha256:b2436cf32296c7205e08207c52bf6712dd67420445a2f7bfc239437ddb715a1c +size 21911 diff --git a/processed_dataset/images/volume8/figures/fig-c6i2.png b/processed_dataset/images/volume8/figures/fig-c6i2.png new file mode 100644 index 0000000000000000000000000000000000000000..69d9a5ffc06bd2d86c95b90ba4ddae9aeb54e6d0 --- /dev/null +++ b/processed_dataset/images/volume8/figures/fig-c6i2.png @@ -0,0 +1,3 @@ +version https://git-lfs.github.com/spec/v1 +oid sha256:e0c6c5f3ff8cd889699e2909c416ed1f6876e6ef17c7e147fd1af7d0dcc8ac18 +size 29649 diff --git a/processed_dataset/images/volume8/figures/fig-c6i3.png b/processed_dataset/images/volume8/figures/fig-c6i3.png new file mode 100644 index 0000000000000000000000000000000000000000..3390e32069f011b12d8bc9f932edeaebe4726051 --- /dev/null +++ b/processed_dataset/images/volume8/figures/fig-c6i3.png @@ -0,0 +1,3 @@ +version https://git-lfs.github.com/spec/v1 +oid sha256:476362360e7ca1222c4f64aa8852ad4e709442a1ef0cda4e62137dd57d13350b +size 26451 diff --git a/processed_dataset/images/volume8/figures/fig-c6i4.png b/processed_dataset/images/volume8/figures/fig-c6i4.png new file mode 100644 index 0000000000000000000000000000000000000000..1b5a524a9e38a1b6f82f0b85aea1bd75df3ba20b --- /dev/null +++ b/processed_dataset/images/volume8/figures/fig-c6i4.png @@ -0,0 +1,3 @@ +version https://git-lfs.github.com/spec/v1 +oid sha256:d078349bff9caf349e4978f7581fbcc7339eb950adae2d951b24f7ed120ecc57 +size 27802 diff --git a/processed_dataset/images/volume8/figures/fig-c6i5.png b/processed_dataset/images/volume8/figures/fig-c6i5.png new file mode 100644 index 0000000000000000000000000000000000000000..e70d111b89bf9dc0433779a17ac25e8aea4ac7e1 --- /dev/null +++ b/processed_dataset/images/volume8/figures/fig-c6i5.png @@ -0,0 +1,3 @@ +version https://git-lfs.github.com/spec/v1 +oid sha256:77257180aa18e61ddbd9376106546d5aed4b513c105873884a6e0dfc83799bb0 +size 22206 diff --git a/processed_dataset/images/volume8/figures/fig-c6i6.png b/processed_dataset/images/volume8/figures/fig-c6i6.png new file mode 100644 index 0000000000000000000000000000000000000000..f6852a87588dc744d5f703b76d14dda4a3017f55 --- /dev/null +++ b/processed_dataset/images/volume8/figures/fig-c6i6.png @@ -0,0 +1,3 @@ +version https://git-lfs.github.com/spec/v1 +oid sha256:102aa957afc074b16aa753f04f6730203deaa4180c31a33f3fe31295f7fbddfb +size 22362 diff --git a/processed_dataset/images/volume8/figures/fig-c6i7.png b/processed_dataset/images/volume8/figures/fig-c6i7.png new file mode 100644 index 0000000000000000000000000000000000000000..1ee8348b7d7e81d58e8a372c30f94832213b3fe1 --- /dev/null +++ b/processed_dataset/images/volume8/figures/fig-c6i7.png @@ -0,0 +1,3 @@ +version https://git-lfs.github.com/spec/v1 +oid sha256:c9d6a37c95abda8a694331fee4675aa306eee83180c719957146bbc633140511 +size 26864 diff --git a/processed_dataset/images/volume8/figures/fig-c6p3.png b/processed_dataset/images/volume8/figures/fig-c6p3.png new file mode 100644 index 0000000000000000000000000000000000000000..ea445e2c2865423869fbe58ff43d43e93e1b205b --- /dev/null +++ b/processed_dataset/images/volume8/figures/fig-c6p3.png @@ -0,0 +1,3 @@ +version https://git-lfs.github.com/spec/v1 +oid sha256:cecfa277f1f40999111477edb9af9b6017c14405d0aabfb09ebf056fa11f5278 +size 22363 diff --git a/processed_dataset/images/volume8/figures/fig-c8i1.png b/processed_dataset/images/volume8/figures/fig-c8i1.png new file mode 100644 index 0000000000000000000000000000000000000000..a5e6ccc1759fe739db26110011d7323813f1a315 --- /dev/null +++ b/processed_dataset/images/volume8/figures/fig-c8i1.png @@ -0,0 +1,3 @@ +version https://git-lfs.github.com/spec/v1 +oid sha256:e34c9c50acbd4404d470c664fb344e70ca2f6a455b368454e6178c67a92d59e5 +size 17868 diff --git a/processed_dataset/images/volume8/figures/fig-c8i2.png b/processed_dataset/images/volume8/figures/fig-c8i2.png new file mode 100644 index 0000000000000000000000000000000000000000..969c93553021fe784fa32b053a44df0ca83f0b2e --- /dev/null +++ b/processed_dataset/images/volume8/figures/fig-c8i2.png @@ -0,0 +1,3 @@ +version https://git-lfs.github.com/spec/v1 +oid sha256:d356dc48ad184c1d726c43c40d088f7a5a05903cff6bf28335badede825f2090 +size 13561 diff --git a/processed_dataset/images/volume8/figures/fig-c8i3.png b/processed_dataset/images/volume8/figures/fig-c8i3.png new file mode 100644 index 0000000000000000000000000000000000000000..b1f77de0722c3f81d7c64a4d24cad7607c7aaffa --- /dev/null +++ b/processed_dataset/images/volume8/figures/fig-c8i3.png @@ -0,0 +1,3 @@ +version https://git-lfs.github.com/spec/v1 +oid sha256:5e0ffb89da6c53cfd7b9105fc7f5c86b84439d59752c40e91aa3a3ea0010b16b +size 20114 diff --git a/processed_dataset/images/volume8/figures/fig-c8i4.png b/processed_dataset/images/volume8/figures/fig-c8i4.png new file mode 100644 index 0000000000000000000000000000000000000000..caf67a7e8e7f433b0bd455cbd35e4f9cfcf3e135 --- /dev/null +++ b/processed_dataset/images/volume8/figures/fig-c8i4.png @@ -0,0 +1,3 @@ +version https://git-lfs.github.com/spec/v1 +oid sha256:ba776f2fd4cb3937320c6b23546f9ab49d0b2f57a0261ed88657d126a161ff72 +size 18536 diff --git a/processed_dataset/images/volume8/figures/fig-c9a2.png b/processed_dataset/images/volume8/figures/fig-c9a2.png new file mode 100644 index 0000000000000000000000000000000000000000..7002887256a2f432ffe1a508f4be6edbbd4ed96f --- /dev/null +++ b/processed_dataset/images/volume8/figures/fig-c9a2.png @@ -0,0 +1,3 @@ +version https://git-lfs.github.com/spec/v1 +oid sha256:aeccbe7c0913369d08fb59df1b312193af9f2cfe5b0996197a7dde0ae139e7b9 +size 12552 diff --git a/processed_dataset/images/volume8/figures/fig-c9a4.png b/processed_dataset/images/volume8/figures/fig-c9a4.png new file mode 100644 index 0000000000000000000000000000000000000000..0b6cebc7f226e89d4eef264da775c36d99e6ded2 --- /dev/null +++ b/processed_dataset/images/volume8/figures/fig-c9a4.png @@ -0,0 +1,3 @@ +version https://git-lfs.github.com/spec/v1 +oid sha256:1bcdb268776b91d27e957447fd99ea94fbc1a561379ddcef855a754028c7c1ff +size 13042 diff --git a/processed_dataset/images/volume8/figures/fig-c9i1.png b/processed_dataset/images/volume8/figures/fig-c9i1.png new file mode 100644 index 0000000000000000000000000000000000000000..6a3171940d88cae9267282dc1c42758d255c4fc3 --- /dev/null +++ b/processed_dataset/images/volume8/figures/fig-c9i1.png @@ -0,0 +1,3 @@ +version https://git-lfs.github.com/spec/v1 +oid sha256:a34b44bf744236dfed35033fe78a1c2609b89d21976bd26ae8a8544ead6dd988 +size 27327 diff --git a/processed_dataset/images/volume9/figures/fig-c10a1.png b/processed_dataset/images/volume9/figures/fig-c10a1.png new file mode 100644 index 0000000000000000000000000000000000000000..515e3995dbbff9304c709994c87602ee4bf13f78 --- /dev/null +++ b/processed_dataset/images/volume9/figures/fig-c10a1.png @@ -0,0 +1,3 @@ +version https://git-lfs.github.com/spec/v1 +oid sha256:79baffbe7f96d5a749f9c6e7bee197998815775e0d095d1b7061d65a232c888a +size 21110 diff --git a/processed_dataset/images/volume9/figures/fig-c10a2.png b/processed_dataset/images/volume9/figures/fig-c10a2.png new file mode 100644 index 0000000000000000000000000000000000000000..01654b9898f5b9cf84c19c2c2e9f150fe596c485 --- /dev/null +++ b/processed_dataset/images/volume9/figures/fig-c10a2.png @@ -0,0 +1,3 @@ +version https://git-lfs.github.com/spec/v1 +oid sha256:f970da5e043e6f0c3f92e145717501fa9efff35a5ad4a820b48d76d8dba88a3e +size 28045 diff --git a/processed_dataset/images/volume9/figures/fig-c10i1.png b/processed_dataset/images/volume9/figures/fig-c10i1.png new file mode 100644 index 0000000000000000000000000000000000000000..0f304c784dd6a6b5c7664929f8f8fe6d31187ff6 --- /dev/null +++ b/processed_dataset/images/volume9/figures/fig-c10i1.png @@ -0,0 +1,3 @@ +version https://git-lfs.github.com/spec/v1 +oid sha256:8d39e35560a809278f42d19ed752c67e8a94416837674ad498bd65401eef7279 +size 26028 diff --git a/processed_dataset/images/volume9/figures/fig-c10i2.png b/processed_dataset/images/volume9/figures/fig-c10i2.png new file mode 100644 index 0000000000000000000000000000000000000000..dadb86347a7a78c1184d3fb782de47d97bf95adf --- /dev/null +++ b/processed_dataset/images/volume9/figures/fig-c10i2.png @@ -0,0 +1,3 @@ +version https://git-lfs.github.com/spec/v1 +oid sha256:1d7ce0e4d3334a3b22b1959ab5bea251137ff22baf03dfdb890a308b6432abe4 +size 20951 diff --git a/processed_dataset/images/volume9/figures/fig-c10i3.png b/processed_dataset/images/volume9/figures/fig-c10i3.png new file mode 100644 index 0000000000000000000000000000000000000000..35ad92844e45f989aaf2180f717de3e9235f54e2 --- /dev/null +++ b/processed_dataset/images/volume9/figures/fig-c10i3.png @@ -0,0 +1,3 @@ +version https://git-lfs.github.com/spec/v1 +oid sha256:2624f5e891ed7235e17035a708ce2ed415e79f0cfb552c3138f9478c30108abc +size 28559 diff --git a/processed_dataset/images/volume9/figures/fig-c10i4.png b/processed_dataset/images/volume9/figures/fig-c10i4.png new file mode 100644 index 0000000000000000000000000000000000000000..2c6a8b6e51d2a595faeff0617bd5c41ac25cf9b6 --- /dev/null +++ b/processed_dataset/images/volume9/figures/fig-c10i4.png @@ -0,0 +1,3 @@ +version https://git-lfs.github.com/spec/v1 +oid sha256:29663b3cc78e2bed08e5fc6248574b1b67720c78110f87a2b58f8abdb7c3b857 +size 11973 diff --git a/processed_dataset/images/volume9/figures/fig-c10i5.png b/processed_dataset/images/volume9/figures/fig-c10i5.png new file mode 100644 index 0000000000000000000000000000000000000000..ebbff901c8cd93ea603ff573cc21c7cf72ed885e --- /dev/null +++ b/processed_dataset/images/volume9/figures/fig-c10i5.png @@ -0,0 +1,3 @@ +version https://git-lfs.github.com/spec/v1 +oid sha256:112ecfad38d795ceb23f354558e83c7990b5f5c83fa0c95bf66e4e394f48e2b6 +size 25732 diff --git a/processed_dataset/images/volume9/figures/fig-c10i6.png b/processed_dataset/images/volume9/figures/fig-c10i6.png new file mode 100644 index 0000000000000000000000000000000000000000..ea26c77b791723f0a5d010ed69534401f3b6f158 --- /dev/null +++ b/processed_dataset/images/volume9/figures/fig-c10i6.png @@ -0,0 +1,3 @@ +version https://git-lfs.github.com/spec/v1 +oid sha256:f07d1d3dfc73175f69a167a03a527ecb01a59cbdec396fa97b38980dcfc51dd3 +size 27259 diff --git a/processed_dataset/images/volume9/figures/fig-c10i7.png b/processed_dataset/images/volume9/figures/fig-c10i7.png new file mode 100644 index 0000000000000000000000000000000000000000..c1b05bad308f82c13a0dc7534f2974b7c1d97ee5 --- /dev/null +++ b/processed_dataset/images/volume9/figures/fig-c10i7.png @@ -0,0 +1,3 @@ +version https://git-lfs.github.com/spec/v1 +oid sha256:715f94a850b6424127d5338fd8541706f64b340b86d5fb1a202b4a7f8949ccb3 +size 23474 diff --git a/processed_dataset/images/volume9/figures/fig-c11i1.png b/processed_dataset/images/volume9/figures/fig-c11i1.png new file mode 100644 index 0000000000000000000000000000000000000000..193d34245996c8ea93d1200e7f84757ca745a030 --- /dev/null +++ b/processed_dataset/images/volume9/figures/fig-c11i1.png @@ -0,0 +1,3 @@ +version https://git-lfs.github.com/spec/v1 +oid sha256:98e28a176b56752e88a15c08e09ba4ce006e33c003c4e4da6de82dd3e29ad89d +size 39776 diff --git a/processed_dataset/images/volume9/figures/fig-c11i2.png b/processed_dataset/images/volume9/figures/fig-c11i2.png new file mode 100644 index 0000000000000000000000000000000000000000..5d7d6885a50359727d31dca04e0669ce9c11d68a --- /dev/null +++ b/processed_dataset/images/volume9/figures/fig-c11i2.png @@ -0,0 +1,3 @@ +version https://git-lfs.github.com/spec/v1 +oid sha256:ab75131bd9a3f6553ef4704ecada074fd2afda15090fd7f3e1d9ea611693d4a7 +size 26011 diff --git a/processed_dataset/images/volume9/figures/fig-c11i3.png b/processed_dataset/images/volume9/figures/fig-c11i3.png new file mode 100644 index 0000000000000000000000000000000000000000..3b08ec2fddb12e97bb959408603b4d6636909bd5 --- /dev/null +++ b/processed_dataset/images/volume9/figures/fig-c11i3.png @@ -0,0 +1,3 @@ +version https://git-lfs.github.com/spec/v1 +oid sha256:54867568621bfdb0e5d89af4b6f851896b59cfae6c7d23e445bd9e9c648bb75a +size 27704 diff --git a/processed_dataset/images/volume9/figures/fig-c1i1.png b/processed_dataset/images/volume9/figures/fig-c1i1.png new file mode 100644 index 0000000000000000000000000000000000000000..567e69f647c9f237e65fcbd97681ff4c5afcc8c0 --- /dev/null +++ b/processed_dataset/images/volume9/figures/fig-c1i1.png @@ -0,0 +1,3 @@ +version https://git-lfs.github.com/spec/v1 +oid sha256:417225490d84988f20b6bf2da3502c030599b013364f3191ff7e6f5342037bc3 +size 23138 diff --git a/processed_dataset/images/volume9/figures/fig-c1i2.png b/processed_dataset/images/volume9/figures/fig-c1i2.png new file mode 100644 index 0000000000000000000000000000000000000000..c33209f8194c2f04444b1e5d5c671089e1b8ce45 --- /dev/null +++ b/processed_dataset/images/volume9/figures/fig-c1i2.png @@ -0,0 +1,3 @@ +version https://git-lfs.github.com/spec/v1 +oid sha256:46ce7b335550e4e6479cf14324dfeb567d55484a8ed5c75848ef2895001deb9e +size 25294 diff --git a/processed_dataset/images/volume9/figures/fig-c1i3.png b/processed_dataset/images/volume9/figures/fig-c1i3.png new file mode 100644 index 0000000000000000000000000000000000000000..45f731b6c20b3b4d169d1095443a2f32997c6f2a --- /dev/null +++ b/processed_dataset/images/volume9/figures/fig-c1i3.png @@ -0,0 +1,3 @@ +version https://git-lfs.github.com/spec/v1 +oid sha256:a1a79523e45bc2c701a7562c38ad23d96b6a8a36152851714754cb14b71145a8 +size 26048 diff --git a/processed_dataset/images/volume9/figures/fig-c1i4.png b/processed_dataset/images/volume9/figures/fig-c1i4.png new file mode 100644 index 0000000000000000000000000000000000000000..45def32542faa384f46e00e473abab1d1ceb856b --- /dev/null +++ b/processed_dataset/images/volume9/figures/fig-c1i4.png @@ -0,0 +1,3 @@ +version https://git-lfs.github.com/spec/v1 +oid sha256:9d91497648cfab30355435767bd84aac585a4bc4ada294347fa4f5a79115f174 +size 31982 diff --git a/processed_dataset/images/volume9/figures/fig-c1i5.png b/processed_dataset/images/volume9/figures/fig-c1i5.png new file mode 100644 index 0000000000000000000000000000000000000000..00ecf76528bce6c36b2087027c72d08bc4469cf5 --- /dev/null +++ b/processed_dataset/images/volume9/figures/fig-c1i5.png @@ -0,0 +1,3 @@ +version https://git-lfs.github.com/spec/v1 +oid sha256:158c8513939877a53b063b0c7b1cbcd70ac1f2345999f207909f20ca16dd6b89 +size 44883 diff --git a/processed_dataset/images/volume9/figures/fig-c1i6.png b/processed_dataset/images/volume9/figures/fig-c1i6.png new file mode 100644 index 0000000000000000000000000000000000000000..5ea4b18f0194edc2572d3704990c1e3b4e96505f --- /dev/null +++ b/processed_dataset/images/volume9/figures/fig-c1i6.png @@ -0,0 +1,3 @@ +version https://git-lfs.github.com/spec/v1 +oid sha256:28c7a5e963d377fa87081f6a025294ace8d70fe0c777a1b051474b9b4ec5d5da +size 26243 diff --git a/processed_dataset/images/volume9/figures/fig-c1i7.png b/processed_dataset/images/volume9/figures/fig-c1i7.png new file mode 100644 index 0000000000000000000000000000000000000000..b55e5fabce6c2b8cf77f71dc3085ceb9e6e93550 --- /dev/null +++ b/processed_dataset/images/volume9/figures/fig-c1i7.png @@ -0,0 +1,3 @@ +version https://git-lfs.github.com/spec/v1 +oid sha256:dfcec966162abd7b59c7f4b88358821684906181c14224279e1d068bf1f50d08 +size 25096 diff --git a/processed_dataset/images/volume9/figures/fig-c2i1.png b/processed_dataset/images/volume9/figures/fig-c2i1.png new file mode 100644 index 0000000000000000000000000000000000000000..f080afcf874be1395dc6b996bad87147cd845f63 --- /dev/null +++ b/processed_dataset/images/volume9/figures/fig-c2i1.png @@ -0,0 +1,3 @@ +version https://git-lfs.github.com/spec/v1 +oid sha256:6b44e723bb116600768fec36b68b63ff53404bc79e079e896fa87bcff36eb26e +size 28233 diff --git a/processed_dataset/images/volume9/figures/fig-c2i10.png b/processed_dataset/images/volume9/figures/fig-c2i10.png new file mode 100644 index 0000000000000000000000000000000000000000..4077d540db8f142f712bce4107fdedaa47d4fbc5 --- /dev/null +++ b/processed_dataset/images/volume9/figures/fig-c2i10.png @@ -0,0 +1,3 @@ +version https://git-lfs.github.com/spec/v1 +oid sha256:2a76db9b7486af0c88d300603f8edb97d1d7a8a4fed027c8da045f619ad8c42a +size 22639 diff --git a/processed_dataset/images/volume9/figures/fig-c2i2.png b/processed_dataset/images/volume9/figures/fig-c2i2.png new file mode 100644 index 0000000000000000000000000000000000000000..605615b0ae85b515d177b01d2df6d0784f1d8306 --- /dev/null +++ b/processed_dataset/images/volume9/figures/fig-c2i2.png @@ -0,0 +1,3 @@ +version https://git-lfs.github.com/spec/v1 +oid sha256:c81a6875129bdcc6b920b48dec78d964e38aeb702a1e75d12c4ffd7637697566 +size 26003 diff --git a/processed_dataset/images/volume9/figures/fig-c2i3.png b/processed_dataset/images/volume9/figures/fig-c2i3.png new file mode 100644 index 0000000000000000000000000000000000000000..0c6f72b1e81a3174a3bd104ef6e46d795216f49a --- /dev/null +++ b/processed_dataset/images/volume9/figures/fig-c2i3.png @@ -0,0 +1,3 @@ +version https://git-lfs.github.com/spec/v1 +oid sha256:993a799b2c8c316eea454d0faaa4c4b0c3dcbdca028652145ce538416858350f +size 25893 diff --git a/processed_dataset/images/volume9/figures/fig-c2i4.png b/processed_dataset/images/volume9/figures/fig-c2i4.png new file mode 100644 index 0000000000000000000000000000000000000000..c0b78bb12ea6b00fb06dfbd3c5791690f3aae116 --- /dev/null +++ b/processed_dataset/images/volume9/figures/fig-c2i4.png @@ -0,0 +1,3 @@ +version https://git-lfs.github.com/spec/v1 +oid sha256:dd5b82fa8c0926ae17301e94f357011ef56451e88ac501bf239bef8f9a6c9542 +size 28553 diff --git a/processed_dataset/images/volume9/figures/fig-c2i5.png b/processed_dataset/images/volume9/figures/fig-c2i5.png new file mode 100644 index 0000000000000000000000000000000000000000..89ffeac796db2f79703b2592d514db00c7c469ec --- /dev/null +++ b/processed_dataset/images/volume9/figures/fig-c2i5.png @@ -0,0 +1,3 @@ +version https://git-lfs.github.com/spec/v1 +oid sha256:788a6d41b3b001c5b63bf692db09b6c1006632ef38cf132ded10ec7d53c007fc +size 24927 diff --git a/processed_dataset/images/volume9/figures/fig-c2i6.png b/processed_dataset/images/volume9/figures/fig-c2i6.png new file mode 100644 index 0000000000000000000000000000000000000000..af03d30abfcbdec9574837262f84de0f545fc265 --- /dev/null +++ b/processed_dataset/images/volume9/figures/fig-c2i6.png @@ -0,0 +1,3 @@ +version https://git-lfs.github.com/spec/v1 +oid sha256:7c3aaf2520646511b5b941b34bd19f4493ede6ef0b046feab86f90d130202af4 +size 23814 diff --git a/processed_dataset/images/volume9/figures/fig-c2i7.png b/processed_dataset/images/volume9/figures/fig-c2i7.png new file mode 100644 index 0000000000000000000000000000000000000000..7465ee528d311bb846790f50a48ec250d6e25f4e --- /dev/null +++ b/processed_dataset/images/volume9/figures/fig-c2i7.png @@ -0,0 +1,3 @@ +version https://git-lfs.github.com/spec/v1 +oid sha256:94fe287b81ebc7fab9ee1108be75e156c0ba23d87b64ad8a6d95efe6fbd735e1 +size 25031 diff --git a/processed_dataset/images/volume9/figures/fig-c2i8.png b/processed_dataset/images/volume9/figures/fig-c2i8.png new file mode 100644 index 0000000000000000000000000000000000000000..edbb216adb9c7d4f5fc5c8e674edfbf259a38aad --- /dev/null +++ b/processed_dataset/images/volume9/figures/fig-c2i8.png @@ -0,0 +1,3 @@ +version https://git-lfs.github.com/spec/v1 +oid sha256:44a58784f780f2bc8d36a1e273c6156b80df870a35d4f5de76b88b57c805c79b +size 26131 diff --git a/processed_dataset/images/volume9/figures/fig-c2i9.png b/processed_dataset/images/volume9/figures/fig-c2i9.png new file mode 100644 index 0000000000000000000000000000000000000000..4f3a555cfe0ad63f0f2a6aced3fddef20882729a --- /dev/null +++ b/processed_dataset/images/volume9/figures/fig-c2i9.png @@ -0,0 +1,3 @@ +version https://git-lfs.github.com/spec/v1 +oid sha256:cdf69a2c2c53ae047678d516c53ec9faeb9892342d8b689c3ed20931b551f741 +size 29393 diff --git a/processed_dataset/images/volume9/figures/fig-c3i1.png b/processed_dataset/images/volume9/figures/fig-c3i1.png new file mode 100644 index 0000000000000000000000000000000000000000..239e9630051e3249adad2f9e90a867118bc0b312 --- /dev/null +++ b/processed_dataset/images/volume9/figures/fig-c3i1.png @@ -0,0 +1,3 @@ +version https://git-lfs.github.com/spec/v1 +oid sha256:c9e4cd2fcab6b4ed053eb2288a52808d67c4d825855d18a6a3245b61110d2073 +size 28100 diff --git a/processed_dataset/images/volume9/figures/fig-c3i2.png b/processed_dataset/images/volume9/figures/fig-c3i2.png new file mode 100644 index 0000000000000000000000000000000000000000..cad44f873f8c4d6dfe5e0993a56a483644a8e8a0 --- /dev/null +++ b/processed_dataset/images/volume9/figures/fig-c3i2.png @@ -0,0 +1,3 @@ +version https://git-lfs.github.com/spec/v1 +oid sha256:bf476ffcb3383e77c0064513589aca45f1938f1fc99349aac82418190f4c6a90 +size 35194 diff --git a/processed_dataset/images/volume9/figures/fig-c3i3.png b/processed_dataset/images/volume9/figures/fig-c3i3.png new file mode 100644 index 0000000000000000000000000000000000000000..0e090d7c9e853a1981f658cc936d63f911974010 --- /dev/null +++ b/processed_dataset/images/volume9/figures/fig-c3i3.png @@ -0,0 +1,3 @@ +version https://git-lfs.github.com/spec/v1 +oid sha256:26695db43fc270268d7ecd065caaa4c3d7d9222f690520f3aef3b8d18da0e1c2 +size 30623 diff --git a/processed_dataset/images/volume9/figures/fig-c3i4.png b/processed_dataset/images/volume9/figures/fig-c3i4.png new file mode 100644 index 0000000000000000000000000000000000000000..495127869556c76ae449ca292b5922035a23ed25 --- /dev/null +++ b/processed_dataset/images/volume9/figures/fig-c3i4.png @@ -0,0 +1,3 @@ +version https://git-lfs.github.com/spec/v1 +oid sha256:099f4459bc906e36e7fa06d613713d07703c5c947def4c50ff1958775a43f689 +size 28739 diff --git a/processed_dataset/images/volume9/figures/fig-c3i5.png b/processed_dataset/images/volume9/figures/fig-c3i5.png new file mode 100644 index 0000000000000000000000000000000000000000..f1378060670db65126a521b66e7d8fe766092c73 --- /dev/null +++ b/processed_dataset/images/volume9/figures/fig-c3i5.png @@ -0,0 +1,3 @@ +version https://git-lfs.github.com/spec/v1 +oid sha256:9fec5c3ed70b43386b25cd02206a5c6d520f69c660952fc735b56c770b311cb1 +size 25219 diff --git a/processed_dataset/images/volume9/figures/fig-c3i6.png b/processed_dataset/images/volume9/figures/fig-c3i6.png new file mode 100644 index 0000000000000000000000000000000000000000..767529652b21c25f49b0482ba268173eb7b0e39f --- /dev/null +++ b/processed_dataset/images/volume9/figures/fig-c3i6.png @@ -0,0 +1,3 @@ +version https://git-lfs.github.com/spec/v1 +oid sha256:17b89db4f186caba6998afd34c8f3f314dba6b1a1f7d32fa82650b6534dacf6d +size 28799 diff --git a/processed_dataset/images/volume9/figures/fig-c3i7.png b/processed_dataset/images/volume9/figures/fig-c3i7.png new file mode 100644 index 0000000000000000000000000000000000000000..9529d66a9e061bd38cf9e061efffafaaca5c393b --- /dev/null +++ b/processed_dataset/images/volume9/figures/fig-c3i7.png @@ -0,0 +1,3 @@ +version https://git-lfs.github.com/spec/v1 +oid sha256:4d5a17297d99caea7b5adffc5b609cc7c58fd955a45a32aefb32cf3f4104979e +size 21027 diff --git a/processed_dataset/images/volume9/figures/fig-c4i1.png b/processed_dataset/images/volume9/figures/fig-c4i1.png new file mode 100644 index 0000000000000000000000000000000000000000..d9cc3cb3b89adb9c132f9208bbb5e4974a7655e0 --- /dev/null +++ b/processed_dataset/images/volume9/figures/fig-c4i1.png @@ -0,0 +1,3 @@ +version https://git-lfs.github.com/spec/v1 +oid sha256:d19c3b540c66f32f851717b6a4fd31d37188a567eaa06536cf42d85436eb4840 +size 20256 diff --git a/processed_dataset/images/volume9/figures/fig-c4i10.png b/processed_dataset/images/volume9/figures/fig-c4i10.png new file mode 100644 index 0000000000000000000000000000000000000000..76c2448ea9a19e7e6f8828f38b59e037139cc25d --- /dev/null +++ b/processed_dataset/images/volume9/figures/fig-c4i10.png @@ -0,0 +1,3 @@ +version https://git-lfs.github.com/spec/v1 +oid sha256:e29aacd7e650bd5f4e83b012a0199cf4d2567888d5231dcfe2bcb37e754e84d0 +size 21981 diff --git a/processed_dataset/images/volume9/figures/fig-c4i11.png b/processed_dataset/images/volume9/figures/fig-c4i11.png new file mode 100644 index 0000000000000000000000000000000000000000..b9024173192c13f6ee161b1ca7791f3ba89a2e3f --- /dev/null +++ b/processed_dataset/images/volume9/figures/fig-c4i11.png @@ -0,0 +1,3 @@ +version https://git-lfs.github.com/spec/v1 +oid sha256:41584ec8586850551a3ec35227024bb6e3f82114a00aaeb1b73ac5e8bfbd9b02 +size 22407 diff --git a/processed_dataset/images/volume9/figures/fig-c4i12.png b/processed_dataset/images/volume9/figures/fig-c4i12.png new file mode 100644 index 0000000000000000000000000000000000000000..65afcbf449d3c8b9f0cf5ecfaac21a23376abbbf --- /dev/null +++ b/processed_dataset/images/volume9/figures/fig-c4i12.png @@ -0,0 +1,3 @@ +version https://git-lfs.github.com/spec/v1 +oid sha256:37e33cc579c023110dd14ec4523a8c7be125f408a23fb236dfa187cf0304c82b +size 32249 diff --git a/processed_dataset/images/volume9/figures/fig-c4i13.png b/processed_dataset/images/volume9/figures/fig-c4i13.png new file mode 100644 index 0000000000000000000000000000000000000000..6b5fc296fd4f5e78c3a224d6d17f63703b967a21 --- /dev/null +++ b/processed_dataset/images/volume9/figures/fig-c4i13.png @@ -0,0 +1,3 @@ +version https://git-lfs.github.com/spec/v1 +oid sha256:e39ec276ea993b93cd5f2bb914fbe07d014b14ab65106d3544478ec4957277b4 +size 28524 diff --git a/processed_dataset/images/volume9/figures/fig-c4i14.png b/processed_dataset/images/volume9/figures/fig-c4i14.png new file mode 100644 index 0000000000000000000000000000000000000000..769364f993be9516e1736d3894ab639162ace16e --- /dev/null +++ b/processed_dataset/images/volume9/figures/fig-c4i14.png @@ -0,0 +1,3 @@ +version https://git-lfs.github.com/spec/v1 +oid sha256:3b875f94c0b088641c082a5d3c699620e12c3bfe99ad5c799ddcaf205d995712 +size 36336 diff --git a/processed_dataset/images/volume9/figures/fig-c4i15.png b/processed_dataset/images/volume9/figures/fig-c4i15.png new file mode 100644 index 0000000000000000000000000000000000000000..2ea4c5053a3c17f41bdd6f017b3d7a4ff44b8b99 --- /dev/null +++ b/processed_dataset/images/volume9/figures/fig-c4i15.png @@ -0,0 +1,3 @@ +version https://git-lfs.github.com/spec/v1 +oid sha256:9cbbcbb2e9e7fcf9f0a3525d6c694b2b66d429c7523aff99c1d228889d32bdde +size 58045 diff --git a/processed_dataset/images/volume9/figures/fig-c4i16.png b/processed_dataset/images/volume9/figures/fig-c4i16.png new file mode 100644 index 0000000000000000000000000000000000000000..cba762b6a210dffbbf1d1a8face91d9d0bb49c1a --- /dev/null +++ b/processed_dataset/images/volume9/figures/fig-c4i16.png @@ -0,0 +1,3 @@ +version https://git-lfs.github.com/spec/v1 +oid sha256:5e0963176df027bd7ac8af7ecbe708d07e5bdacae1b39a61210a4d45d7e8c823 +size 32829 diff --git a/processed_dataset/images/volume9/figures/fig-c4i2.png b/processed_dataset/images/volume9/figures/fig-c4i2.png new file mode 100644 index 0000000000000000000000000000000000000000..aea40b29c32c2c3bb2ad4b1f0a85d96ecd05136a --- /dev/null +++ b/processed_dataset/images/volume9/figures/fig-c4i2.png @@ -0,0 +1,3 @@ +version https://git-lfs.github.com/spec/v1 +oid sha256:44c0d58b3d4c9ce069e4e95df07a08282088ac957fe4c1518a476010970c671d +size 29277 diff --git a/processed_dataset/images/volume9/figures/fig-c4i3.png b/processed_dataset/images/volume9/figures/fig-c4i3.png new file mode 100644 index 0000000000000000000000000000000000000000..d47cfc7a3628522e1b9a47e20e8ec003c114f037 --- /dev/null +++ b/processed_dataset/images/volume9/figures/fig-c4i3.png @@ -0,0 +1,3 @@ +version https://git-lfs.github.com/spec/v1 +oid sha256:9b2fa07e1bff60b4fae0ce6554c84a9f3bf699f0c12de175437bf3f9fe42f283 +size 19921 diff --git a/processed_dataset/images/volume9/figures/fig-c4i4.png b/processed_dataset/images/volume9/figures/fig-c4i4.png new file mode 100644 index 0000000000000000000000000000000000000000..1294271f757ea6253e335cc871b0a5808c6e0884 --- /dev/null +++ b/processed_dataset/images/volume9/figures/fig-c4i4.png @@ -0,0 +1,3 @@ +version https://git-lfs.github.com/spec/v1 +oid sha256:496ef025f3a5e708244993f7e72b21d22d827b2f1bf3e880bacec3120f991692 +size 27963 diff --git a/processed_dataset/images/volume9/figures/fig-c4i5.png b/processed_dataset/images/volume9/figures/fig-c4i5.png new file mode 100644 index 0000000000000000000000000000000000000000..e011b419c44876daabe6c1690526af1c0f37049c --- /dev/null +++ b/processed_dataset/images/volume9/figures/fig-c4i5.png @@ -0,0 +1,3 @@ +version https://git-lfs.github.com/spec/v1 +oid sha256:ef3ce00933acb940a1bfa010376c77bd8e0795d6f46a707b2d730c39056c8ae2 +size 23364 diff --git a/processed_dataset/images/volume9/figures/fig-c4i6.png b/processed_dataset/images/volume9/figures/fig-c4i6.png new file mode 100644 index 0000000000000000000000000000000000000000..ab8e1333222652375de9eeaf5ba57318cf51c49b --- /dev/null +++ b/processed_dataset/images/volume9/figures/fig-c4i6.png @@ -0,0 +1,3 @@ +version https://git-lfs.github.com/spec/v1 +oid sha256:802c9f59426c3ae85652e91e35cfb2f0326d869b2c98d742ccccbd4539c17060 +size 31728 diff --git a/processed_dataset/images/volume9/figures/fig-c4i7.png b/processed_dataset/images/volume9/figures/fig-c4i7.png new file mode 100644 index 0000000000000000000000000000000000000000..64a72c51b6267071656373023448c812d45958e5 --- /dev/null +++ b/processed_dataset/images/volume9/figures/fig-c4i7.png @@ -0,0 +1,3 @@ +version https://git-lfs.github.com/spec/v1 +oid sha256:7f3b5f15f3dd3c3744e1d1257f0040bfde21cc808cf240d9862ac6ad873bb20d +size 33108 diff --git a/processed_dataset/images/volume9/figures/fig-c4i8.png b/processed_dataset/images/volume9/figures/fig-c4i8.png new file mode 100644 index 0000000000000000000000000000000000000000..95cbdd5905eee62d4e9b151a19d8738a57666b32 --- /dev/null +++ b/processed_dataset/images/volume9/figures/fig-c4i8.png @@ -0,0 +1,3 @@ +version https://git-lfs.github.com/spec/v1 +oid sha256:b3bce303243246ab4ce597d2713856b235d15cb61b8400801450557e8972f5d8 +size 33020 diff --git a/processed_dataset/images/volume9/figures/fig-c4i9.png b/processed_dataset/images/volume9/figures/fig-c4i9.png new file mode 100644 index 0000000000000000000000000000000000000000..6ec6888e618af7fe92c89e6239f2007d0ff9a0c5 --- /dev/null +++ b/processed_dataset/images/volume9/figures/fig-c4i9.png @@ -0,0 +1,3 @@ +version https://git-lfs.github.com/spec/v1 +oid sha256:e03b7af8f5531659caa41ef5e26b316d50f7e85e70195a9935710db74b3561ce +size 27385 diff --git a/processed_dataset/images/volume9/figures/fig-c5i1.png b/processed_dataset/images/volume9/figures/fig-c5i1.png new file mode 100644 index 0000000000000000000000000000000000000000..80d71d676bcd680a8240d7a77b38e656ca729a26 --- /dev/null +++ b/processed_dataset/images/volume9/figures/fig-c5i1.png @@ -0,0 +1,3 @@ +version https://git-lfs.github.com/spec/v1 +oid sha256:6857c4d0d80535f0b4d4b03e16c8c18494605c8731da266b72e65932bded5b4c +size 29258 diff --git a/processed_dataset/images/volume9/figures/fig-c5i2.png b/processed_dataset/images/volume9/figures/fig-c5i2.png new file mode 100644 index 0000000000000000000000000000000000000000..2040c4262e59600e9c6047b03fc26ff5844ff94a --- /dev/null +++ b/processed_dataset/images/volume9/figures/fig-c5i2.png @@ -0,0 +1,3 @@ +version https://git-lfs.github.com/spec/v1 +oid sha256:ec8f5295cb7668baf2ab8ad3153bf281a3ee62dac87dfc264b3dbf9d8ddc122c +size 26042 diff --git a/processed_dataset/images/volume9/figures/fig-c5i3.png b/processed_dataset/images/volume9/figures/fig-c5i3.png new file mode 100644 index 0000000000000000000000000000000000000000..7e13f3fbb8388e76b480bd1c69899fca2f07274e --- /dev/null +++ b/processed_dataset/images/volume9/figures/fig-c5i3.png @@ -0,0 +1,3 @@ +version https://git-lfs.github.com/spec/v1 +oid sha256:b412edb2f76ae3bb3bc0ec525400a69ba9d2aa657c73ba174f3f028230555a94 +size 27611 diff --git a/processed_dataset/images/volume9/figures/fig-c5i4.png b/processed_dataset/images/volume9/figures/fig-c5i4.png new file mode 100644 index 0000000000000000000000000000000000000000..479982dbbb128fe3ce56bb4b2b29ad0f13097988 --- /dev/null +++ b/processed_dataset/images/volume9/figures/fig-c5i4.png @@ -0,0 +1,3 @@ +version https://git-lfs.github.com/spec/v1 +oid sha256:9649dd1cda7646a8e30cb08b46e8500bd035788e5d3ccdda5b89eeb1b414d229 +size 24487 diff --git a/processed_dataset/images/volume9/figures/fig-c5i5.png b/processed_dataset/images/volume9/figures/fig-c5i5.png new file mode 100644 index 0000000000000000000000000000000000000000..0e47c73114e69d51e78ff2afa86b26352a567212 --- /dev/null +++ b/processed_dataset/images/volume9/figures/fig-c5i5.png @@ -0,0 +1,3 @@ +version https://git-lfs.github.com/spec/v1 +oid sha256:86b329669649816b52df76f89715162f82f204439bd79c086bb85234141489a6 +size 24456 diff --git a/processed_dataset/images/volume9/figures/fig-c5i6.png b/processed_dataset/images/volume9/figures/fig-c5i6.png new file mode 100644 index 0000000000000000000000000000000000000000..d75e948ba39a494040989f35b7fc3a8e114b5510 --- /dev/null +++ b/processed_dataset/images/volume9/figures/fig-c5i6.png @@ -0,0 +1,3 @@ +version https://git-lfs.github.com/spec/v1 +oid sha256:bf64cd7e2ce1e4c93eb7f9d494552d26d7ecc6cebc586f9786b2a47efa19602a +size 20175 diff --git a/processed_dataset/images/volume9/figures/fig-c5i7.png b/processed_dataset/images/volume9/figures/fig-c5i7.png new file mode 100644 index 0000000000000000000000000000000000000000..f582d8ecbddf45a93f713e94da182beae0eb271e --- /dev/null +++ b/processed_dataset/images/volume9/figures/fig-c5i7.png @@ -0,0 +1,3 @@ +version https://git-lfs.github.com/spec/v1 +oid sha256:b2f34a9867729599096cdc788d80c44574dafe32a8611d6688fb518ec4b11648 +size 21643 diff --git a/processed_dataset/images/volume9/figures/fig-c5i8.png b/processed_dataset/images/volume9/figures/fig-c5i8.png new file mode 100644 index 0000000000000000000000000000000000000000..75ccf98aaecc7c7fece0021339ce1015ad036c3a --- /dev/null +++ b/processed_dataset/images/volume9/figures/fig-c5i8.png @@ -0,0 +1,3 @@ +version https://git-lfs.github.com/spec/v1 +oid sha256:97f6a2d35f5d9f9d251a6ca99f32628dfc779d545072c344b8028058d0b41621 +size 26256 diff --git a/processed_dataset/images/volume9/figures/fig-c5i9.png b/processed_dataset/images/volume9/figures/fig-c5i9.png new file mode 100644 index 0000000000000000000000000000000000000000..737e76368feb42a2439827cdfb438aa707d02c90 --- /dev/null +++ b/processed_dataset/images/volume9/figures/fig-c5i9.png @@ -0,0 +1,3 @@ +version https://git-lfs.github.com/spec/v1 +oid sha256:702cb311a8565943e06843c7d432525c413b59606ee92de8a648e649e3379bc6 +size 24839 diff --git a/processed_dataset/images/volume9/figures/fig-c6i1.png b/processed_dataset/images/volume9/figures/fig-c6i1.png new file mode 100644 index 0000000000000000000000000000000000000000..4dda217224f27a40d5d9b4007c51a913a3b6bc87 --- /dev/null +++ b/processed_dataset/images/volume9/figures/fig-c6i1.png @@ -0,0 +1,3 @@ +version https://git-lfs.github.com/spec/v1 +oid sha256:45f32e4a15ef0f23e502d51fb189aaac87f026508949797c5ee36b2c03d4210a +size 24456 diff --git a/processed_dataset/images/volume9/figures/fig-c6i2.png b/processed_dataset/images/volume9/figures/fig-c6i2.png new file mode 100644 index 0000000000000000000000000000000000000000..fd6e3c86e3bc523e135b6bc5aed7be04ce3fcb6f --- /dev/null +++ b/processed_dataset/images/volume9/figures/fig-c6i2.png @@ -0,0 +1,3 @@ +version https://git-lfs.github.com/spec/v1 +oid sha256:16b6379899e3bf299508f20d08922a47e09daa15473122616f4a83bef7cd098f +size 24029 diff --git a/processed_dataset/images/volume9/figures/fig-c6i3.png b/processed_dataset/images/volume9/figures/fig-c6i3.png new file mode 100644 index 0000000000000000000000000000000000000000..f581d8b31fb9bdbb70e9d10e6fdc60cca4d112af --- /dev/null +++ b/processed_dataset/images/volume9/figures/fig-c6i3.png @@ -0,0 +1,3 @@ +version https://git-lfs.github.com/spec/v1 +oid sha256:3a4d46615f2b3244b964c2e6c93df0a2043a36b73c0cbe016b17039a682eb375 +size 23996 diff --git a/processed_dataset/images/volume9/figures/fig-c6i4.png b/processed_dataset/images/volume9/figures/fig-c6i4.png new file mode 100644 index 0000000000000000000000000000000000000000..9316348906931a9c4ae8898cf7236ab6ef7a3eaf --- /dev/null +++ b/processed_dataset/images/volume9/figures/fig-c6i4.png @@ -0,0 +1,3 @@ +version https://git-lfs.github.com/spec/v1 +oid sha256:d26843d79c43ab308cb46317c6ee8904d06d4fb4c7d2c3e2e7e0181e4331c0b8 +size 21943 diff --git a/processed_dataset/images/volume9/figures/fig-c7i1.png b/processed_dataset/images/volume9/figures/fig-c7i1.png new file mode 100644 index 0000000000000000000000000000000000000000..d22dab278fa422d23c3707f8e85604018784cde2 --- /dev/null +++ b/processed_dataset/images/volume9/figures/fig-c7i1.png @@ -0,0 +1,3 @@ +version https://git-lfs.github.com/spec/v1 +oid sha256:391f7333dcf7f865f337a765ee852a36ffec6622aec9dbcad55055d2ff6368ea +size 28642 diff --git a/processed_dataset/images/volume9/figures/fig-c7i2.png b/processed_dataset/images/volume9/figures/fig-c7i2.png new file mode 100644 index 0000000000000000000000000000000000000000..7392bb24cf04e028e666210de7b7e54798560409 --- /dev/null +++ b/processed_dataset/images/volume9/figures/fig-c7i2.png @@ -0,0 +1,3 @@ +version https://git-lfs.github.com/spec/v1 +oid sha256:b01b8af79b84b486945293d0bc52b11349d934bf7cba532e2fb730f78ffeb345 +size 18929 diff --git a/processed_dataset/images/volume9/figures/fig-c7i3.png b/processed_dataset/images/volume9/figures/fig-c7i3.png new file mode 100644 index 0000000000000000000000000000000000000000..dd765256fb45d557978de46841eaa36074abf7f5 --- /dev/null +++ b/processed_dataset/images/volume9/figures/fig-c7i3.png @@ -0,0 +1,3 @@ +version https://git-lfs.github.com/spec/v1 +oid sha256:8eb3f7c0922dd1f4bc12706f9d175c4c78fdbb13f06bc76840fe0f9ebaad332d +size 30815 diff --git a/processed_dataset/images/volume9/figures/fig-c7i4.png b/processed_dataset/images/volume9/figures/fig-c7i4.png new file mode 100644 index 0000000000000000000000000000000000000000..2c7726bae82df7325e64f4cab2aceefbbfdfcfc9 --- /dev/null +++ b/processed_dataset/images/volume9/figures/fig-c7i4.png @@ -0,0 +1,3 @@ +version https://git-lfs.github.com/spec/v1 +oid sha256:e039a585620e1b777b0c3ca4719ecc200019f45200d030f21a6ff26731254e42 +size 17563 diff --git a/processed_dataset/images/volume9/figures/fig-c7i5.png b/processed_dataset/images/volume9/figures/fig-c7i5.png new file mode 100644 index 0000000000000000000000000000000000000000..b95e4fb0ab93d123c3b271362238d9a65111e883 --- /dev/null +++ b/processed_dataset/images/volume9/figures/fig-c7i5.png @@ -0,0 +1,3 @@ +version https://git-lfs.github.com/spec/v1 +oid sha256:82590f70b5b3471c76d580dada1f3edc68d25a597a520fc95c29ddf2acaa2a09 +size 23611 diff --git a/processed_dataset/images/volume9/figures/fig-c7i6.png b/processed_dataset/images/volume9/figures/fig-c7i6.png new file mode 100644 index 0000000000000000000000000000000000000000..cb8138867ce053b2a5baafba68b0cdb2386e61f3 --- /dev/null +++ b/processed_dataset/images/volume9/figures/fig-c7i6.png @@ -0,0 +1,3 @@ +version https://git-lfs.github.com/spec/v1 +oid sha256:c606c294c39a9563d3c224ff5d14d987bf944aea7d43defd58c1eb8c6a018d30 +size 29822 diff --git a/processed_dataset/images/volume9/figures/fig-c8i1.png b/processed_dataset/images/volume9/figures/fig-c8i1.png new file mode 100644 index 0000000000000000000000000000000000000000..fe996360f64409cdf263e8f2abb7c8461485a364 --- /dev/null +++ b/processed_dataset/images/volume9/figures/fig-c8i1.png @@ -0,0 +1,3 @@ +version https://git-lfs.github.com/spec/v1 +oid sha256:6f9cba2779fac4da8358185683b748b8e720702e9615446bfe46e6538f55f803 +size 27006 diff --git a/processed_dataset/images/volume9/figures/fig-c9i1.png b/processed_dataset/images/volume9/figures/fig-c9i1.png new file mode 100644 index 0000000000000000000000000000000000000000..29e4da79aeeef2183a71d1f680d8061b317e527f --- /dev/null +++ b/processed_dataset/images/volume9/figures/fig-c9i1.png @@ -0,0 +1,3 @@ +version https://git-lfs.github.com/spec/v1 +oid sha256:cb18e630b460b370b9adaa9871ee00aa3ba3f137128e28c1bf551f151c88782e +size 30873 diff --git a/processed_dataset/images/volume9/figures/fig-c9i2.png b/processed_dataset/images/volume9/figures/fig-c9i2.png new file mode 100644 index 0000000000000000000000000000000000000000..5ed690fc626d4e30514ffe89228749e709355755 --- /dev/null +++ b/processed_dataset/images/volume9/figures/fig-c9i2.png @@ -0,0 +1,3 @@ +version https://git-lfs.github.com/spec/v1 +oid sha256:53c0b5599801323a894d8a5d12b5ca679cd070c6f0afd00972eecbd826843b76 +size 24117 diff --git a/processed_dataset/proof/0001.json b/processed_dataset/proof/0001.json new file mode 100644 index 0000000000000000000000000000000000000000..652a2d0330cdbfb9928f445d578ff378d7212edc --- /dev/null +++ b/processed_dataset/proof/0001.json @@ -0,0 +1,8 @@ +{ + "source_file": "./raw_volume-zh/volume1/chapter1.tex", + "problem_type": "proof", + "problem": "例2. 设 $A$ 是两个整数平方差的集合,即 $A=\\left\\{x\\mid x=m^{2}-n^{2},\\,m,n\\in\\mathbf{Z}\\right\\}$.证明:\n(1) 若 $s,\\ t\\in A$ ,则 $s t\\in A.$ \n(2) 若 $s,\\ t\\in A,\\ t\\neq0$,则$\\frac{s}{t}=p^{2}-q^{2}.$ ,其中 $p,q$ 是有理数.", + "solution": "分析: 想办法将 $st$ 表示为两个整数的平方差.\n证明: (1)由 $s,t\\in A$ ,可设\n$$\ns=m_{1}^{2}-n_{1}^{2}\\,,\\;t=m_{2}^{2}-n_{2}^{2}\\,, \n$$\n其中 $m_{1}, n_{1}, m_{2}, n_{2}$ 均为整数.\n是\n$$\n\\begin{array}{l}{{s t=(m_{1}^{2}-n_{1}^{2})\\,(m_{2}^{2}-n_{2}^{2})}}\\\\ {{=m_{1}^{2}m_{2}^{2}+2m_{1}m_{2}n_{1}n_{2}+n_{1}^{2}n_{2}^{2}-m_{1}^{2}n_{2}^{2}-2m_{1}m_{2}n_{1}n_{2}-m_{2}^{2}n_{1}^{2}}}\\\\ {{=(m_{1}m_{2}+n_{1}n_{2})^{2}-(m_{1}n_{2}+m_{2}n_{1})^{2}\\,,}}\\end{array} \n$$\n即 $st$ 是两个整数的平方差,故 $s t\\in A.$ \n(2) 由于 $s, t \\in A$ ,由(1)知 $s t\\in A.$ 令 $s t=m^{2}-n^{2},m,n$ 是整数.\n $t\\neq 0$,因此\n$$\n{\\frac{s}{t}}\\,={\\frac{s t}{t^{2}}}=\\left({\\frac{m}{t}}\\right)^{2}-\\left({\\frac{n}{t}}\\right)^{2}. \n$$\n而 ${\\frac{m}{t}},{\\frac{n}{t}}$ 均为有理数,故命题得证.", + "remark": "", + "figures": [] +} \ No newline at end of file diff --git a/processed_dataset/proof/0002.json b/processed_dataset/proof/0002.json new file mode 100644 index 0000000000000000000000000000000000000000..a9434ece9a319325ae8069882f611b64f1014652 --- /dev/null +++ b/processed_dataset/proof/0002.json @@ -0,0 +1,8 @@ +{ + "source_file": "./raw_volume-zh/volume1/chapter1.tex", + "problem_type": "proof", + "problem": "例3. 设函数 $f(x)=x^{2}+a x+b\\ (a,\\,b\\in\\mathbb{R})$ ,集合 $A=\\{x\\mid x=f(x)$ $x\\in\\mathbb{R})\\,,\\,B=\\{x\\mid x=f(f(x))\\,,\\,x\\in\\mathbb{R}\\}.$ \n(1)证明: $A\\subset B$ ;\n(2)当 $A=\\{-1,\\,3\\}$ 时,求集合 $B$ .", + "solution": "分析: 欲证 $A\\subseteq B$, 只需证明方程 $x=f(x)$ 的根必是方程 $x=f(f(x))$ 的根.\n解: (1)对任意的 $x_{0}\\in A$ ,有 $x_{0}=f(x_{0}), \\, x_0 \\in \\mathbb{R}.$\n于是\n$$\nf(f(x_{0}))=f(x_{0})=x_{0}. \n$$\n故 $x_{0}\\in B$ ,所以 $A\\subseteq B$. \n(2)因 $A=\\{-1,\\,3\\}$ ,所以\n$$\n\\begin{align*}\n\\left\\{\n\\begin{aligned}\n (-\\,1)^{2}+a*(-\\,1)+b=-\\,1, \\\\\n 3^{2}+a*3+b=3,\n\\end{aligned}\n\\right.\n\\end{align*}\n$$\n解之得 $a=-1$, $b=-3$,故 $f(x)=x^{2}-x-3.$ 由 $x=f(f(x))$ 得\n$(x^2-x-3)^2-(x^2-x-3)-x-3= 0.$\n即\n$$\n(x^{2}-2x-3)\\,(x^{2}-3)\\,=\\,0\\,. \n$$\n解得 $x=-1,\\,3,\\,\\pm{\\sqrt{3}}.$ \n所以 $,B=\\{-1,\\ 3,-{\\sqrt{3}}\\,,{\\sqrt{3}}\\}$ .", + "remark": "", + "figures": [] +} \ No newline at end of file diff --git a/processed_dataset/proof/0003.json b/processed_dataset/proof/0003.json new file mode 100644 index 0000000000000000000000000000000000000000..3555246a0b4a98740c1dd52c753c44a5be92d3ea --- /dev/null +++ b/processed_dataset/proof/0003.json @@ -0,0 +1,8 @@ +{ + "source_file": "./raw_volume-zh/volume1/chapter1.tex", + "problem_type": "proof", + "problem": "例6. 求所有的角 $\\alpha$, 使得集合\n$$\n\\{\\sin \\alpha, \\sin 2 \\alpha, \\sin 3 \\alpha\\}=\\{\\cos \\alpha, \\cos 2 \\alpha, \\cos 3 \\alpha\\} .\n$$", + "solution": "解: 设 $\\alpha \\in[0,2 \\pi)$. 由已知得\n$$\n\\sin \\alpha+\\sin 2 \\alpha+\\sin 3 \\alpha=\\cos \\alpha+\\cos 2 \\alpha+\\cos 3 \\alpha,\n$$\n即\n$$\n\\begin{aligned}\n2 \\sin 2 \\alpha \\cos \\alpha+\\sin 2 \\alpha & =2 \\cos 2 \\alpha \\cos \\alpha+\\cos 2 \\alpha, \\\\\n\\sin 2 \\alpha(2 \\cos \\alpha+1) & =\\cos 2 \\alpha(2 \\cos \\alpha+1) .\n\\end{aligned}\n$$\n所以 $\\sin 2 \\alpha=\\cos 2 \\alpha$ 或 $\\cos \\alpha=-\\frac{1}{2}$ (舍去).\n从而\n$$\n\\begin{aligned}\n0 & =\\sin 2 \\alpha-\\cos 2 \\alpha \\\\\n& =\\sin 2 \\alpha-\\sin \\left(\\frac{\\pi}{2}-2 \\alpha\\right) \\\\\n& =2 \\cos \\frac{\\pi}{4} \\sin \\left(2 \\alpha-\\frac{\\pi}{4}\\right) .\n\\end{aligned}\n$$\n于是 $\\alpha=\\frac{\\pi}{8}, \\frac{5 \\pi}{8}, \\frac{9 \\pi}{8}, \\frac{13 \\pi}{8}$.\n又 $\\sin \\alpha \\sin 2 \\alpha \\sin 3 \\alpha=\\cos \\alpha \\cos 2 \\alpha \\cos 3 \\alpha$, 且 $\\sin 2 \\alpha=\\cos 2 \\alpha$, 因此\n$$\n\\begin{aligned}\n\\cos 4 \\alpha & =0, \\\\\n\\alpha=\\frac{(2 k-1) \\pi}{8}, k & =1,2, \\cdots, 8 .\n\\end{aligned}\n$$\n经验证, $\\alpha=\\frac{k \\pi}{4}+\\frac{\\pi}{8}(k \\in \\mathbf{Z})$ 满足题意.\n说明: 元素之和(积)相等只是两个集合相等的必要条件, 因此这里还必须检查集合的元素是否互异.", + "remark": "", + "figures": [] +} \ No newline at end of file diff --git a/processed_dataset/proof/0004.json b/processed_dataset/proof/0004.json new file mode 100644 index 0000000000000000000000000000000000000000..95ca7e4ddb599540274272f5531b37b9b9aac0b7 --- /dev/null +++ b/processed_dataset/proof/0004.json @@ -0,0 +1,8 @@ +{ + "source_file": "./raw_volume-zh/volume1/chapter1.tex", + "problem_type": "proof", + "problem": "例7. 设 $S$ 为非空数集, 且满足: (i) $2 \\notin S$; (ii) 若 $a \\in S$, 则 $\\frac{1}{2-a} \\in S$. 证明:\n(1) 对一切 $n \\in \\mathbf{N}^*, n \\geqslant 3$, 有 $\\frac{n}{n-1} \\notin S$;\n(2) $S$ 或者是单元素集,或者是无限集.", + "solution": "分析: 对于 (1), 因为 $n \\in \\mathbf{N}^*$, 可以考虑采用数学归纳法.\n证明: (1) 因为 $S$ 非空, 所以存在 $a \\in S$, 且 $a \\neq 2$.\n我们用数学归纳法证明下面的命题:\n若 $a \\in S$, 则对 $k \\in \\mathbf{N}^*, \\frac{(k-1)-(k-2) a}{k-(k-1) a} \\in S$, 且 $a \\neq \\frac{k+1}{k}$.\n当 $k=1$ 时, 显然 $a \\in S$, 且 $a \\neq 2$ 成立.\n设 $k \\in \\mathbf{N}^*, \\frac{(k-1)-(k-2) a}{k-(k-1) a} \\in S$ 且 $a \\neq \\frac{k+1}{k}$ 成立.\n由 (ii) 得\n$$\n\\begin{gathered}\n\\frac{1}{2-\\frac{(k-1)-(k-2) a}{k-(k-1) a}} \\in S, \\\\\n\\frac{k-(k-1) a}{(k+1)-k a} \\in S .\n\\end{gathered}\n$$\n化简得\n$$\n\\frac{k-(k-1) a}{(k+1)-k a} \\in S \\text {. }\n$$\n又 $\\frac{k-(k-1) a}{(k+1)-k a} \\neq 2$, 所以 $a \\neq \\frac{k+2}{k+1}$.\n综上, 由归纳原理知, 对 $k \\in \\mathbf{N}^*$ 命题成立.\n从而, 对一切 $n \\in \\mathbf{N}^*, n \\geqslant 3$, $\\frac{n}{n-1} \\notin S$ 成立.\n(2) 由 (1) 知, 若 $a \\in S, a \\neq \\frac{m}{m-1}\\left(m \\in \\mathbf{N}^*, m \\geqslant 3\\right)$, 则 $\\frac{(m-1)-(m-2) a}{m-(m-1) a} \\in S$.\n所以, 当 $n \\geqslant 2, m \\geqslant 2, m \\neq n$ 时,\n$$\n\\begin{aligned}\n& \\frac{(n-1)-(n-2) a}{n-(n-1) a}=\\frac{(m-1)-(m-2) a}{m-(m-1) a} \\\\\n& \\Leftrightarrow m(n-1)-(n-1)(m-1) a-m(n-2) a+(m-1)(n-2) a^2 \\\\\n& =n(m-1)-(n-1)(m-1) a-n(m-2) a+(n-1)(m-2) a^2 \\\\\n& \\Leftrightarrow n-m+2(m-n) a+(n-m) a^2=0 \\\\\n& \\Leftrightarrow(n-m)\\left(1-2 a+a^2\\right)=0 \\\\\n& \\Leftrightarrow a=1 (\\text { 因为 } n \\neq m) .\n\\end{aligned}\n$$\n因为 $\\mathbf{N}^*$ 是无限集, 所以 $S$ 或者为单元素集 $\\{1\\}$ (当且仅当 $a=1$ ), 或者为无限集.", + "remark": "", + "figures": [] +} \ No newline at end of file diff --git a/processed_dataset/proof/0005.json b/processed_dataset/proof/0005.json new file mode 100644 index 0000000000000000000000000000000000000000..abad4d160d11ccdb46962157a93c362106996ee6 --- /dev/null +++ b/processed_dataset/proof/0005.json @@ -0,0 +1,8 @@ +{ + "source_file": "./raw_volume-zh/volume1/chapter1.tex", + "problem_type": "proof", + "problem": "例8. 用 $\\sigma(S)$ 表示非空的整数集合 $S$ 的所有元素的和.\n设 $A=\\left\\{a_1\\right.$, $\\left.a_2, \\cdots, a_{11}\\right\\}$ 是正整数的集合, 且 $a_1k$ 时,考虑集合\n$$\nA=\\left\\{1,2, \\cdots, k, 2 k, 3 k, \\cdots, k^2, k^2+k, k^2+b\\right\\} \\varsubsetneqq\\{1,2, \\cdots, n\\},\n$$\n同样有\n$$\n|A|=2 k+1 \\leqslant 2[\\sqrt{n}]+1 \\text {, }\n$$\n且 $\\{|x-y| \\mid x, y \\in A, x \\neq y\\}=\\left\\{1,2, \\cdots, k^2+b-1\\right\\}$.\n综上知, 原命题成立.", + "remark": "", + "figures": [] +} \ No newline at end of file diff --git a/processed_dataset/proof/0008.json b/processed_dataset/proof/0008.json new file mode 100644 index 0000000000000000000000000000000000000000..9e571a1bd1259789fb4c8b4ce75c65cb703a08dd --- /dev/null +++ b/processed_dataset/proof/0008.json @@ -0,0 +1,8 @@ +{ + "source_file": "./raw_volume-zh/volume1/chapter2.tex", + "problem_type": "proof", + "problem": "例6. 设 $n \\in \\mathbf{N}$, 且 $n \\geqslant 15, A 、 B$ 都是 $\\{1,2, \\cdots, n\\}$ 的真子集, $A \\cap B=\\varnothing$, 且 $\\{1,2, \\cdots, n\\}=A \\cup B$. 证明: $A$ 或者 $B$ 中必有两个不同数的和为完全平方数.", + "solution": "证明:由题设, $\\{1,2, \\cdots, n\\}$ 的任何元素必属于且只属于它的真子集 $A$ 、$B$ 之一.\n假设结论不真, 则存在如题设的 $\\{1,2, \\cdots, n\\}$ 的真子集 $A 、B$, 使得无论是 $A$ 还是 $B$ 中的任何两个不同的数的和都不是完全平方数.\n不妨设 $1 \\in A$, 则 $3 \\notin A$. 否则 $1+3=2^2$, 与假设矛盾, 所以 $3 \\in B$. 同样, $6 \\notin B$, 所以 $6 \\in A$. 这时 $10 \\notin A$, 即 $10 \\in B$. 因 $n \\geqslant 15$, 而 15 或者在 $A$ 中, 或者在 $B$ 中,但当 $15 \\in A$ 时, 因 $1 \\in A, 1+15=4^2$, 矛盾; 当 $15 \\in B$ 时, 因 $10 \\in B, 10+15=5^2$, 仍然矛盾.\n因此假设不真, 即 $A$ 或者 $B$ 中必有两个不同数的和为完全平方数.\n说明由 $A 、 B$ 地位对称, 在上面的解法中我们采用了“不妨设 $1 \\in A$ ”这种技巧,有效简化了解题过程.\n例6 实际上给出了一个关于集合的方程组:\n$$\n\\left\\{\\begin{array}{l}\nA \\cup B=\\{1,2, \\cdots, n\\}, \\\\\nA \\cap B=\\varnothing .\n\\end{array}\\right.\n$$\n如果交换 $A 、 B$ 算两组解 (有序解), 那么这个方程组有多少组有序解呢?\n设 $U=\\{1,2, \\cdots, n\\}$, 由 $A \\cup B=U, A \\cap B=\\varnothing$,知 $A$ 与 $B$ 互补, 对于 $A \\subseteq U$, 可取 $B=\\complement_U A$. 故上述集合方程的有序解的个数为 $2^n$.", + "remark": "", + "figures": [] +} \ No newline at end of file diff --git a/processed_dataset/proof/0009.json b/processed_dataset/proof/0009.json new file mode 100644 index 0000000000000000000000000000000000000000..35c935d7332a85c134b91b153f40a6d969d44b5f --- /dev/null +++ b/processed_dataset/proof/0009.json @@ -0,0 +1,8 @@ +{ + "source_file": "./raw_volume-zh/volume1/chapter2.tex", + "problem_type": "proof", + "problem": "例8. 设集合 $S$ 含有 $n$ 个元素, $A_1, A_2, \\cdots, A_k$ 是 $S$ 的不同子集, 它们两两的交集非空,而 $S$ 的其他子集不能与 $A_1, A_2, \\cdots, A_k$ 都相交.\n求证: $k=2^{n-1}$.", + "solution": "分析: $S$ 有 $2^n$ 个子集, 将两个互为补集的子集作为一组, 则可将 $2^n$ 个子集分成 $2^{n-1}$ 个组, 记为 $\\left\\{A_i^{\\prime}, B_i^{\\prime}\\right\\}, i=1,2, \\cdots, 2^{n-1}$, 显然 $A_i$ 只能选取每组中的一个子集.\n证明: 设 $a \\in S$. 因为 $|S|=n$, 故 $S$ 的子集中含 $a$ 的子集有 $2^{n-1}$ 个.\n显然它们两两的交非空.\n所以, $k \\geqslant 2^{n-1}$.\n又可将 $S$ 的 $2^n$ 个子集分成 $2^{n-1}$ 组, 每组有两个集合, 它们互为补集.\n若 $k>2^{n-1}$, 则必有两个集合 $A_i 、 A_j(i \\neq j)$ 来自上述同一组, 但 $A_i \\cap A_j=\\varnothing$,与题意不符.\n所以, $k=2^{n-1}$.", + "remark": "", + "figures": [] +} \ No newline at end of file diff --git a/processed_dataset/proof/0010.json b/processed_dataset/proof/0010.json new file mode 100644 index 0000000000000000000000000000000000000000..325ba5fdcfe1eee80be1a31c1001af11dd961711 --- /dev/null +++ b/processed_dataset/proof/0010.json @@ -0,0 +1,8 @@ +{ + "source_file": "./raw_volume-zh/volume1/chapter2.tex", + "problem_type": "proof", + "problem": "例9. 有 1987 个集合,每个集合有 45 个元素,任意两个集合的并集有 89 个元素, 问此 1987 个集合的并集有多少个元素?", + "solution": "分析: 由每个集合有 45 个元素, 且任意两个集合的并集有 89 个元素知, 任意两个集合有且只有一个公共元素.\n解显然可以由题设找到这样的 1987 个集合, 它们都含有一个公共元素 $a$,而且每两个集合不含 $a$ 以外的公共元素.\n下面,我们来排除其他可能性.\n由任意两个集合的并集有 89 个元素可知, 1987 个集合中的任意两个集合有且只有一个公共元素, 则容易证明这 1987 个集合中必有一个集合 $A$ 中的元素 $a$ 出现在 $A$ 以外的 45 个集合中, 设为 $A_1, A_2, \\cdots, A_{45}$, 其余的设为 $A_{46}, A_{47}, \\cdots, A_{1986}$.\n设 $B$ 为 $A_{46}, \\cdots, A_{1986}$ 中的任一个集合, 且 $a \\notin B$, 由题设 $B$ 和 $A, A_1$, $A_2, \\cdots, A_{45}$ 都有一个公共元素, 且此 46 个元素各不相同, 故 $B$ 中有 46 个元素,与题设矛盾.\n所以这 1987 个集合中均含有 $a$.\n故所求结果为 $1987 \\times 44+1=87429$, 即这 1987 个集合的并集有 87429 个元素.\n说明: 在这里我们先设计一种符合题设的特殊情形, 然后再排除其他可能的情形, 从而达到解题目的.\n这是一种“先猜后证”的解题策略.", + "remark": "", + "figures": [] +} \ No newline at end of file diff --git a/processed_dataset/proof/0011.json b/processed_dataset/proof/0011.json new file mode 100644 index 0000000000000000000000000000000000000000..796f3f8714e99be0aad55eeb24b19dc03e983971 --- /dev/null +++ b/processed_dataset/proof/0011.json @@ -0,0 +1,8 @@ +{ + "source_file": "./raw_volume-zh/volume1/chapter2.tex", + "problem_type": "proof", + "problem": "例10. 设 $A$ 是集合 $S=\\{1,2, \\cdots, 1000000\\}$ 的一个恰有 101 个元素的子集.\n证明: 在 $S$ 中存在数 $t_1, t_2, \\cdots, t_{100}$, 使得集合\n$$\nA_j=\\left\\{x+t_j \\mid x \\in A\\right\\}, j=1,2, \\cdots, 100\n$$\n中, 每两个的交集为空集.", + "solution": "分析: 先弄清楚在什么情况下 $A_i \\cap A_j \\neq \\varnothing$. 设 $a \\in A_i \\cap A_j$, 则 $a=x+ t_i==y+t_j, x, y \\in A$. 于是 $t_i-t_j=y-x$. 这说明选取 $t_1, t_2, \\cdots, t_{100}$ 时, 只要保证其中任意两个之差不等于 $A$ 中任二元素之差即可.\n证明: 考虑集合 $D=\\{x-y \\mid x, y \\in A\\}$, 则\n$$\n|D| \\leqslant 101 \\times 100+1=10101 \\text {. }\n$$\n若 $A_i \\cap A_j \\neq \\varnothing$, 设 $a \\in A_i \\cap A_j$, 则 $a=x+t_i, a=y+t_j$, 其中 $x, y \\in A$, 则 $t_i-t_j=y-x \\in D$.\n若 $t_i-t_j \\in D$, 即存在 $x, y \\in A$, 使得 $t_i-t_j=y-x$, 从而 $x+t_i=y+t_j$, 即 $A_i \\cap A_j \\neq \\varnothing$.\n所以, $A_i \\cap A_j \\neq \\varnothing$ 的充要条件是 $t_i-t_j \\in D$. 于是, 我们只需在集 $S$ 中取出 100 个元素, 使得其中任意两个的差都不属于 $D$.\n下面用递推方法来取出这 100 个元素.\n先在 $S$ 中任取一个元素 $t_1$, 再从 $S$ 中取一个 $t_2$, 使得 $t_1 \\notin t_2+D=\\{t_2+ x \\mid x \\in D\\}$, 这是因为取定 $t_1$ 后, 至多有 10101 个 $S$ 中的元素不能作为 $t_2$, 从而在 $S$ 中存在这样的 $t_2$, 若已有 $k(\\leqslant 99)$ 个 $S$ 中的元素 $t_1, t_2, \\cdots, t_k$ 满足要求, 再取 $t_{k+1}$, 使得 $t_1, \\cdots, t_k$ 都不属于 $t_{k+1}+D=\\left\\{t_{k+1}+x \\mid x \\in D\\right\\}$. 这是因为 $t_1, t_2, \\cdots, t_k$ 取定后, 至多有 $10101 k \\leqslant 999999$ 个 $S$ 中的数不能作为 $t_{k+1}$, 故在 $S$ 中存在满足条件的 $t_{k+1}$. 所以, 在 $S$ 中存在 $t_1, t_2, \\cdots, t_{100}$, 其中任意两个的差都不属于 $D$.\n综上所述,命题得证.\n说明条件 $|S|=10^{\\varepsilon}$ 可以改小一些.\n一般地, 我们有如下更强的结论: 若 $A$ 是 $S=\\{1,2, \\cdots, n\\}$ 的一个 $k$ 元子集, $m$ 为正整数, 且 $m$ 满足条件 $n>(m-1) \\cdot\\left(C_k^2+1\\right)$, 则存在 $S$ 中的元素 $t_1, \\cdots, t_m$, 使得 $A_j=\\left\\{x+t_j \\mid x \\in A \\right\\}, j=1, \\cdots, m$ 中任意两个的交集为空集.", + "remark": "", + "figures": [] +} \ No newline at end of file diff --git a/processed_dataset/proof/0012.json b/processed_dataset/proof/0012.json new file mode 100644 index 0000000000000000000000000000000000000000..470578b26ae7ade7dd647fc5ce52e8d166b8d8e0 --- /dev/null +++ b/processed_dataset/proof/0012.json @@ -0,0 +1,8 @@ +{ + "source_file": "./raw_volume-zh/volume1/chapter3.tex", + "problem_type": "proof", + "problem": "例4. 设 $a_1, a_2, \\cdots, a_n$ 为 $1,2, \\cdots, n$ 的一个排列, $f_k=\\mid\\{a_i \\mid a_ik\\}\\left|, g_k=\\right|\\left\\{a_i \\mid a_i>a_k, ik\\right\\}$, 对 $A_k$ 换一种写法: $A_k=\\left\\{\\left(a_i, a_k\\right) \\mid a_ik\\right\\}$, 显然是合理的.\n易知 $k \\neq k^{\\prime}$ 时, $A_k \\cap A_k{ }^{\\prime}=\\varnothing$. 所以, $\\sum_{k=1}^n f_k=\\left|A_1\\right|+\\left|A_2\\right|+\\cdots+\\left|A_n\\right|=\\left|A_1 \\cup A_2 \\cup \\cdots \\cup A_n\\right|=\\left|\\left\\{\\left(a_i, a_j\\right) \\mid a_ij\\right\\}\\right|$.\n证明考虑集合 $A=\\left\\{\\left(a_i, a_j\\right) \\mid a_ij\\right\\}$ 的元素的数目 $|A|$. 一方面, 固定 $a_j$ 时, $a_i$ 的个数为 $f_j$. 所以\n$$\n|A|=\\sum_{j=1}^n f_j .\n$$\n另一方面, 固定 $a_i$ 时, $a_j$ 的个数为 $g_i$, 所以\n$$\n|A|=\\sum_{i=1}^n g_i\n$$\n所以, $\\sum_{k=1}^n g_k=\\sum_{k=1}^n f_k$.\n说明在这里, 我们没有直接证明 $\\sum_{k=1}^n g_k=\\sum_{k=1}^n f_k$, 而是引人一个中间量 $|A|=\\left|\\left\\{\\left(a_i, a_j\\right) \\mid a_ij\\right\\}\\right|$ 来过渡.", + "remark": "", + "figures": [] +} \ No newline at end of file diff --git a/processed_dataset/proof/0013.json b/processed_dataset/proof/0013.json new file mode 100644 index 0000000000000000000000000000000000000000..4bd6d56f67d2b0cdda8494c160709b405c0fbc4f --- /dev/null +++ b/processed_dataset/proof/0013.json @@ -0,0 +1,8 @@ +{ + "source_file": "./raw_volume-zh/volume1/chapter3.tex", + "problem_type": "proof", + "problem": "例5. 设 $p \\geqslant 5$ 是一个素数, $S=\\{1,2, \\cdots, p-1\\}, A=\\{a \\mid a \\in S, a^{p-1} \\not \\equiv 1\\left(\\bmod p^2\\right) \\}$. 证明 : $|A| \\geqslant \\frac{p-1}{2}$.", + "solution": "分析:如果 $1 \\leqslant a \\leqslant p-1$, 显然 $1 \\leqslant p-a \\leqslant p-1$. 将 $a$ 与 $p-a$ 配对, 如果 $a^{p-1}$ 与 $(p-a)^{p-1}$ 模 $p^2$ 不同余, 则结论成立.\n证明设 $a \\in S$, 则 $p-a \\in S$. 由二项式定理,有\n$$\n(p-a)^{p-1}-a^{p-1} \\equiv-(p-1) a^{p-2} \\cdot p \\not \\equiv 0\\left(\\bmod p^2\\right) .\n$$\n于是, $a$ 和 $p-a$ 中至少有一个在 $A$ 中, 从而有\n$$\n|A| \\geqslant \\frac{p-1}{2} \\text {. }\n$$", + "remark": "", + "figures": [] +} \ No newline at end of file diff --git a/processed_dataset/proof/0014.json b/processed_dataset/proof/0014.json new file mode 100644 index 0000000000000000000000000000000000000000..afe0ffb3d4066b1cfe3a8d8ef901858a12d5262b --- /dev/null +++ b/processed_dataset/proof/0014.json @@ -0,0 +1,8 @@ +{ + "source_file": "./raw_volume-zh/volume1/chapter3.tex", + "problem_type": "proof", + "problem": "例6. $A_1, A_2, \\cdots, A_{30}$ 是集合 $\\{1,2, \\cdots, 2003\\}$ 的子集,且 $\\left|A_i\\right| \\geqslant 660$, $i=1,2, \\cdots, 30$. 证明: 存在 $i \\neq j, i, j \\in\\{1,2, \\cdots, 30\\}$, 使得 $\\left|A_i \\cap A_j\\right| \\geqslant$ 203.", + "solution": "证明:不妨设每一个 $A_i$ 的元素都为 660 个(否则去掉一些元素). 作一个集合、元素的关系表: 表中每一行(除最上面的一行外)分别表示 30 个集合 $A_1, A_2, \\cdots, A_{30}$, 表的 $n$ 列 (最左面一列除外) 分别表示 2003 个元素 1 , $2, \\cdots, 2003$. 若 $i \\in A_j(i=1,2, \\cdots, 2003,1 \\leqslant j \\leqslant 30)$, 则在 $i$ 所在的列与 $A_j$ 所在行的交叉处写上 1 , 若 $i \\notin A_j$, 则写上 0 .\n\\begin{tabular}{c|ccccc} \n& 1 & 2 & 3 & $\\cdots$ & 2003 \\\\\n\\hline$A_1$ & $\\times$ & $\\times$ & $\\times$ & $\\cdots$ & $\\times$ \\\\\n$A_2$ & $\\times$ & $\\times$ & $\\times$ & $\\cdots$ & $\\times$ \\\\\n$\\cdots$ & $\\times$ & $\\times$ & $\\times$ & $\\cdots$ & $\\times$ \\\\\n$A_{30}$ & $\\times$ & $\\times$ & $\\times$ & $\\cdots$ & $\\times$\n\\end{tabular}\n表中每一行有 660 个 1 , 因此共有 $30 \\times 660$ 个 1 . 设第 $j$ 列有 $m_j$ 个 1 $(j=1,2, \\cdots, 2003)$, 则\n$$\n\\sum_{j=1}^{2003} m_j=30 \\times 660 .\n$$\n由于每个元素 $j$ 属于 $\\mathrm{C}_{m_j}^2$ 个交集 $A_s \\cap A_t$, 因此\n$$\n\\sum_{j=1}^{2003} \\mathrm{C}_{m_j}^2=\\sum_{1 \\leqslant s202, \\\\\n& \\left|A_i \\cap A_j\\right| \\geqslant 203 .\n\\end{aligned}\n$$\n从而说明本题中所作的表,称为元素、集合从属关系表.\n它在讨论涉及多个集合的问题时非常有用.", + "remark": "", + "figures": [] +} \ No newline at end of file diff --git a/processed_dataset/proof/0015.json b/processed_dataset/proof/0015.json new file mode 100644 index 0000000000000000000000000000000000000000..fa0c35836029b978d350457161caccd33d1fb083 --- /dev/null +++ b/processed_dataset/proof/0015.json @@ -0,0 +1,8 @@ +{ + "source_file": "./raw_volume-zh/volume1/chapter3.tex", + "problem_type": "proof", + "problem": "例7. 设 $n, k \\in \\mathbf{N}^*$, 且 $k \\leqslant n$. 并设 $S$ 是含有 $n$ 个互异实数的集合, $T=\\left\\{a \\mid a=x_1+x_2+\\cdots+x_k, x_i \\in S, x_i \\neq x_j(i \\neq j), 1 \\leqslant i, j \\leqslant k\\right\\}$. 求证: $|T| \\geqslant k(n-k)+1$.", + "solution": "分析:设 $S_n=\\left\\{s_1, s_2, \\cdots, s_{n-1}, s_n\\right\\}$, 且 $s_1c$ 的数对 (b,c) (共 190对), 考虑它们的差 $b-c$, 由于至多有 99 个不同的差 (这里用到反证法假设), 故必有至少 91 个数对 $(b, c)$, 使得存在 $b^{\\prime}, c^{\\prime} \\in S$, 满足 $b^{\\prime}2 k$, 得 $n-k>k$. 于是我们有\n$$\n\\begin{aligned}\n|M| & =-\\sum_{i=1}^n\\left|B_i\\right| \\geqslant k \\cdot k+(n-k) \\cdot(n-k) \\\\\n& =k^2+(n-k)^2 \\\\\n& \\geqslant \\frac{1}{2}(k+(n-k))^2=\\frac{n^2}{2} .\n\\end{aligned}\n$$\n综上所述,命题成立.\n说明本例的解答应用了最小数原理.\n关于最小数原理的应用, 我们将在后面作专门的介绍.", + "remark": "", + "figures": [] +} \ No newline at end of file diff --git a/processed_dataset/proof/0020.json b/processed_dataset/proof/0020.json new file mode 100644 index 0000000000000000000000000000000000000000..dc4a26e9e3035ee739a3a7296d5a85a1cbfb8dd9 --- /dev/null +++ b/processed_dataset/proof/0020.json @@ -0,0 +1,8 @@ +{ + "source_file": "./raw_volume-zh/volume1/chapter4.tex", + "problem_type": "proof", + "problem": "例7. 设自然数集分划成 $r$ 个互不相交的子集: $\\mathbf{N}=A_1 \\cup A_2 \\cup \\cdots \\cup A_r$. 求证其中必有某个子集 $A$, 它具有如下性质 $P$ : 存在 $m \\in \\mathbf{N}$, 使对任何正整数 $k$, 都能找到 $a_1, a_2, \\cdots, a_k \\in A$, 满足\n$$\n1 \\leqslant a_{j+1}-a_j \\leqslant m, j=1,2, \\cdots, k-1 .\n$$", + "solution": "分析:显然具有性质 $P$ 的子集 $A$, 不可能是 $\\mathbf{N}$ 的 $r$-分划中的有限集.\n不妨设 $\\mathbf{N}$ 的 $r$-分划中的无限集为 $A_1, A_2, \\cdots, A_{r^{\\prime}}$, 令 $B=A_2 \\cup A_3 \\cup \\cdots \\cup A_{r^{\\prime}}$. 设 $b$ 是集合 $A_{r^{\\prime}+1} \\cup \\cdots \\cup A_{r-1} \\cup A_r$ 中的最大自然数, 则 $b$ 以后的自然数都在 $N^{\\prime}= A_1 \\cup B$ 中, 即 $N^{\\prime}$ 中存在任意有限长度的相继自然数段.\n只需证明: 若 $A_1$ 不具有性质 $P$, 则 $B$ 必具有性质 $P$.\n证明先证下面的引理:\n引理设 $\\mathbf{N}=A_1 \\cup A_2 \\cup \\cdots \\cup A_r$, 且 $A_1, A_2, \\cdots, A_r$ 两两不相交.\n若 $A_i \\cup A_{i+1} \\cup \\cdots \\cup A_r$ 包含任意有限长度的相继自然数段.\n而 $A_i$ 不具有性质 $P$, 则 $A_{i+1} \\cup \\cdots \\cup A_r$ 中必定含有任意有限长度的相继自然数段.\n引理的证明若 $A_i$ 不具有性质 $P$, 则对于任给的 $m \\in \\mathbf{N}$, 存在 $k(m) \\in \\mathbf{N}$, 使得对于 $A_i$ 的任何 $k(m)$ 个数 $a_11$, 因 $2 \\in B$, 故 $n \\in A$. 这时, 对 $A$ 中任一元素 $m2^h+1$, 则 $2^{h+1}+2-n0$ 且 $(u, v, w)=1$.\n令\n$$\n\\begin{aligned}\n& A=\\left\\{\\left(\\frac{u}{w}, \\frac{v}{w}\\right) \\mid 2 \\nmid u\\right\\}, \\\\\n& B=\\left\\{\\left(\\frac{u}{w}, \\frac{v}{w}\\right)|2| u, 2 \\nmid v\\right\\}, \\\\\n& C=\\left\\{\\left(\\frac{u}{w}, \\frac{v}{w}\\right)|2| u, 2 \\mid v\\right\\} .\n\\end{aligned}\n$$\n让我们来验证这 3 个集合满足条件(i)和(ii).\n设平面上的直线方程为\n$$\na x+b y+c=0 .\n$$\n如果其上有两个不同的有理点 $\\left(x_1, y_1\\right)$ 和 $\\left(x_2, y_2\\right)$, 则有\n$$\n\\left\\{\\begin{array}{l}\na x_1+b y_1+c=0, \\\\\na x_2+b y_2+c=0 .\n\\end{array}\\right.\n$$\n如果 $c=0$, 则可取 $a 、 b$ 为有理数.\n如果 $c \\neq 0$, 不妨设 $c=1$, 于是, 从上面的联立方程中可解得 $a$ 和 $b$ 的值, 当然都是有理数.\n再通分即知, 可以使 $a 、 b 、 c$ 都是整数且满足 $(a, b, c)=1$.\n设有理点 $\\left(\\frac{u}{w}, \\frac{v}{w}\\right)$ 在直线 $a x+b y+c=0$ 上, 于是, 有\n$$\nL: a u+b v+c w=0 .\n$$\n(1) 先证集合 $A 、 B 、 C$ 满足条件(ii). 分三种情形.\n(a) $2 \\nmid c$. 若 $2|u, 2| v$, 则由 (1) 知 $2 \\mid c w$, 从而 $2 \\mid w$, 此与 $(u, v, w)=$ 1 矛盾.\n所以,集合 $C$ 中的点都不能在直线 $L$ 上.\n(b) $2 \\mid c, 2 \\nmid b$. 若 $2 \\nmid v$, 则 $2 \\nmid a u$, 从而 $2 \\nmid u$. 因此, 集合 $B$ 中的点都不能在直线 $L$ 上.\n(c) $2|c, 2| b$. 由(1)得 $2 \\mid a u$. 又因 $(a, b, c)=1$, 故 $2 \\nmid a$. 所以 $2 \\mid u$. 这表明集合 $A$ 中的点都不在直线 $L$ 上.\n综上可知, $A 、 B 、 C$ 这 3 个集合满足条件(ii).\n(2) 再证满足条件 (i).\n设 $D$ 是以有理点 $\\left(\\frac{u_0}{w_0}, \\frac{w_0}{w_0}\\right)$ 为圆心, 以 $r$ 为半径的圆.\n取正整数 $k$, 使得\n$$\n2^k>\\max \\left\\{w_0, \\frac{1}{r}\\left(\\left|u_0\\right|+\\left|v_0\\right|+1\\right)\\right\\} .\n$$\n于是易验证, 下列 3 个有理点\n$$\n\\begin{gathered}\n\\left(\\frac{u_0 2^k+1}{w_0 2^k}, \\frac{v_0 2^k}{w_0 2^k}\\right) \\in A,\\left(\\frac{u_0 2^k}{w_0 2^k}, \\frac{v_0 2^k+1}{w_0 2^k}\\right) \\in B, \\\\\n\\left(\\frac{u_0 2^k}{w_0\\left(2^k+1\\right)}, \\frac{v_0 2^k}{w_0\\left(2^k+1\\right)}\\right) \\in C\n\\end{gathered}\n$$\n都在 $\\odot D$ 的内部.\n注意, 在上述 3 点中, $u_0 、 v_0 、 w_0$ 不一定互质.\n但由于 $2^k> w_0$, 故约分之后不改变分子的奇偶性.\n这表明条件(i)成立.\n最后,我们来看一个非常特殊的集合分划的例子.", + "remark": "", + "figures": [] +} \ No newline at end of file diff --git a/processed_dataset/proof/0023.json b/processed_dataset/proof/0023.json new file mode 100644 index 0000000000000000000000000000000000000000..974b969d540899d74aa899bb4fe20032322bbf7d --- /dev/null +++ b/processed_dataset/proof/0023.json @@ -0,0 +1,8 @@ +{ + "source_file": "./raw_volume-zh/volume1/chapter5.tex", + "problem_type": "proof", + "problem": "例1. 试证: 任一有限集的全部子集可以排定次序, 使得任何相邻的两个子集都相差一个元素.", + "solution": "分析:不妨设有限集 $A=\\{1,2,3, \\cdots, n\\}$. 先来看一些简单情形:\n当 $n=1$ 时,显然可以排成: $\\varnothing,\\{1\\}$;\n当 $n=2$ 时,共有 $2^2=4$ 个子集,可排成: $\\varnothing,\\{1\\},\\{1,2\\},\\{2\\}$;\n当 $n=3$ 时,共有 $2^3=8$ 个子集,可排成: $\\varnothing,\\{1\\},\\{1,2\\},\\{2\\},\\{2,3\\},\\{1,2,3\\},\\{1,3\\},\\{3\\}$.\n显然符合条件的排序方式不是惟一的.\n请注意 $n=3$ 时的上述排法: 所有子集可分为两组, 前 4 个子集都不含元素 3 ; 后 4 个均含元素 3, 且去掉 3 后恰是前 4 个子集排列的逆序.\n事实上, $n=2$ 时也如此.\n这说明我们可以考虑用数学归纳法来证明.\n证明设有限集为 $M_n=\\left\\{w_1, w_2, \\cdots, w_n\\right\\}$, 我们对 $n$ 进行归纳.\n当 $n=1$ 时, $M_1=\\left\\{w_1\\right\\}$,将它的两个子集排列成 $\\varnothing,\\left\\{w_1\\right\\}$ 即可.\n假设当 $n=k$ 时,命题成立.\n当 $n=k+1$ 时,\n$$\nM_{k+1}=\\left\\{w_1, w_2, \\cdots, w_k, w_{k+1}\\right\\},\n$$\n它是由集合 $M_k=\\left\\{w_1, w_2, \\cdots, w_k\\right\\}$ 添加元素 $w_{k+1}$ 而形成的.\n$M_k$ 的子集个数为 $2^k$. 由归纳假设知, 可将 $M_k$ 的全体子集排成满足题设要求的一列, 不妨设\n$$\nA_1, A_2, A_3, \\cdots, A_{2^k}\\left(A_i \\subseteq M_k, i=1,2,3, \\cdots, 2^k\\right)\n$$\n就是这样的一个排列.\n我们来看排列\n$$\nA_1, A_2, A_3, \\cdots, A_{2^k}, A_{2^k} \\bigcup\\left\\{w_{k+1}\\right\\}, A_{2^k-1} \\bigcup\\left\\{w_{k+1}\\right\\}, \\cdots, A_1 \\bigcup\\left\\{w_{k+1}\\right\\},\n$$\n它恰好由 $M_{k+1}$ 的 $2^{k+1}$ 个不同子集排成, 且任意两个相邻集合的元素都仅相差 1 个.\n可见当 $n=k+1$ 时, 命题也成立.\n所以,对任意的 $n \\in \\mathbf{N}^*$, 所述命题成立.\n说明一个复杂的问题,也许一时找不到解题的突破口, 这时可考虑“以退求进”的策略.\n先解决一些简单的或特殊的情形, 从中发现规律和方法, 从而找到解决一般问题的办法.\n这也就是从特殊到一般的思维方法.", + "remark": "", + "figures": [] +} \ No newline at end of file diff --git a/processed_dataset/proof/0024.json b/processed_dataset/proof/0024.json new file mode 100644 index 0000000000000000000000000000000000000000..5498d06330859eec1c90bae0dc445f85d58d0914 --- /dev/null +++ b/processed_dataset/proof/0024.json @@ -0,0 +1,8 @@ +{ + "source_file": "./raw_volume-zh/volume1/chapter5.tex", + "problem_type": "proof", + "problem": "例2. 在某次竞选中各政党作出 $n$ 种不同的诺言 $(n>0)$, 有些政党可以作某些相同的诺言.\n现知其中每两个政党都至少作了一个相同的诺言, 但没有两个政党的诺言完全相同.\n求证: 政党个数 $\\leqslant 2^{n-1}$.", + "solution": "证明:设有 $m$ 个政党.\n以 $A$ 记所有诺言的集合, $A_i$ 记第 $i$ 个政党的诺言的集合 $(i=1,2, \\cdots, m)$. 由题设知\n$$\n|A|=n, A_i \\cap A_j \\neq \\varnothing, A_i \\neq A_j, 1 \\leqslant iC_2>\\cdots>C_n, \\text { 且 } \\\\\nD_k=\\sum_{i=1}^k a_i-\\left(1+2+\\cdots+2^{k-1}\\right)=\\sum_{i=1}^k a_i-\\left(2^k-1\\right) \\geqslant 0 .\n\\end{gathered}\n$$\n于是我们有\n$$\n\\begin{aligned}\n& 1+\\frac{1}{2}+\\cdots+\\frac{1}{2^{n-1}}-\\left(\\frac{1}{a_1}+\\frac{1}{a_2}+\\cdots+\\frac{1}{a_n}\\right) \\\\\n= & \\sum_{i=1}^n C_i d_1 \\\\\n= & C_1 D_1+C_2\\left(D_2-D_1\\right)+\\cdots+C_n\\left(D_n-D_{n-1}\\right) \\\\\n= & \\left(C_1-C_2\\right) D_1+\\left(C_2-C_3\\right) D_2+\\cdots+\\left(C_{n-1}-C_n\\right) D_{n-1}+C_n D_n \\\\\n\\geqslant & 0,\n\\end{aligned}\n$$\n故(2)式得证.\n注意到, 当 $S=\\left\\{1,2,2^2, \\cdots, 2^{n-1}\\right\\}$ 时, 题设条件成立.\n此时有\n$$\n\\frac{1}{a_1}+\\frac{1}{a_2}+\\cdots+\\frac{1}{a_n}=1+\\frac{1}{2}+\\cdots+\\frac{1}{2^{n-1}}=2-\\frac{1}{2^{n-1}} \\text {. }\n$$\n因此, 所求的最大值是 $2-\\frac{1}{2^{n-1}}$.", + "remark": "", + "figures": [] +} \ No newline at end of file diff --git a/processed_dataset/proof/0026.json b/processed_dataset/proof/0026.json new file mode 100644 index 0000000000000000000000000000000000000000..8451f49443dbb09684c7a1fdbe6e1bf4d99ac465 --- /dev/null +++ b/processed_dataset/proof/0026.json @@ -0,0 +1,8 @@ +{ + "source_file": "./raw_volume-zh/volume1/chapter5.tex", + "problem_type": "proof", + "problem": "例5. 对于整数 $n(n \\geqslant 2)$, 如果存在集合 $\\{1,2, \\cdots, n\\}$ 的子集族 $A_1$, $A_2, \\cdots, A_n$ 满足;\n(a) $i \\notin A_i, i=1,2, \\cdots, n$;\n(b) 若 $i \\neq j, i, j \\in\\{1,2, \\cdots, n\\}$, 则 $i \\in A_j$, 当且仅当 $j \\notin A_i$;\n(c) 任意 $i, j \\in\\{1,2, \\cdots, n\\}, A_i \\cap A_j \\neq \\varnothing$.\n则称 $n$ 是 “好数”.\n证明: (1) 7 是好数;\n(2)当且仅当 $n \\geqslant 7$ 时, $n$ 是好数.", + "solution": "分析:对于 $n=7$, 可以作出满足条件的子集族来验证; 当 $n \\geqslant 7$ 时, 可考虑用数学归纳法证明.\n证明 (1) 当 $n=7$ 时, 取\n$$\n\\begin{aligned}\n& A_1=\\{2,3,4\\}, A_2=\\{3,5,6\\}, A_3=\\{4,5,7\\}, \\\\\n& A_4=\\{2,6,7\\}, A_5=\\{1,4,6\\}, A_6=\\{1,3,7\\}, \\\\\n& A_7=\\{1,2,5\\}\n\\end{aligned}\n$$\n即可.\n(2) 先证当 $n \\geqslant 7$ 时, $n$ 是好数.\n对 $n$ 进行归纳.\n由 (1) 知, 当 $n=7$ 时, 结论成立.\n假设 $n(n \\geqslant 7)$ 是好数, 则存在子集族 $A_1, A_2, \\cdots, A_n$ 满足条件.\n对于 $n+$ 1 , 取子集族 $B_1=A_1, B_2=A_2, \\cdots, B_n=A_n, B_{n+1}=\\{1,2, \\cdots, n\\}$. 由归纳假设易知, 它们也是满足条件的.\n下面证明每一个好数 $n$ 都至少为 7 .\n如果 $A_1, A_2, \\cdots, A_n$ 是一个 $n$ 为好数的集合的子集族,那么, 每一个 $A_i$ 至少有三个元素.\n事实上,若 $A_i \\subset\\{j, k\\}$, 则\n$$\nA_i \\cap A_j=\\{k\\}, A_i \\cap A_k=\\{j\\} .\n$$\n所以, $k \\in A_j, j \\in A_k$. 矛盾.\n考虑一个由元素 $0 、 1$ 构成的 $n \\times n$ 阶正方形表格,当且仅当 $j \\in A_i$ 其第 $i$ 行第 $j$ 列的元素为 1 . 表中对角线上的元素为 0 , 对于余下的元素,因为 $i \\neq j$, 当且仅当 $a_{j i}=1$ 时 $a_{i j}=0$, 所以 0 的个数等于 1 的个数.\n因此, 表中元素的和为 $\\frac{n^2-n}{2}$. 又每行元素的和大于等于 3 , 所以 $n^2-n \\geqslant 6 n$, 故 $n \\geqslant 7$.", + "remark": "", + "figures": [] +} \ No newline at end of file diff --git a/processed_dataset/proof/0027.json b/processed_dataset/proof/0027.json new file mode 100644 index 0000000000000000000000000000000000000000..393de4e6673a245737dc2a81090c9b143c13bc27 --- /dev/null +++ b/processed_dataset/proof/0027.json @@ -0,0 +1,10 @@ +{ + "source_file": "./raw_volume-zh/volume1/chapter5.tex", + "problem_type": "proof", + "problem": "例6. 集合 $X=\\{1,2, \\cdots, 6 k\\}, k \\in \\mathbf{N}^*$. 试作出 $X$ 的三元子集族 $\\mathscr{A}$, 满足:\n(1) $X$ 的任一二元子集至少被族 $\\mathscr{A}$ 中的一个三元子集包含;\n(2) $|\\mathscr{A}|=6 k^2$.", + "solution": "解:先证明下面的引理:\n引理对 $n \\in \\mathbf{N}^*$, 集合 $X_1=\\{1,2, \\cdots, 2 n\\}$ 的全部二元子集可分成 $2 n-1$ 组, 且每组是 $X_1$ 的一个分划.\n引理的证明: 如图(),将 $1,2, \\cdots, 2 n-1$ 这 $2 n-1$ 个数按顺时针方向放到一个正 $2 n-1$ 边形的顶点上,数 $2 n$ 放在外接圆圆心.\n连结 $2 n$ 与 1 , 作 $n-1$ 条以 $2 n-1$ 边形顶点为端点且垂直于 1 与 $2 n$ 连线的线段,便得到 $X_1$ 的 $n$ 个二元子集构成 $X_1$ 的一个分划.\n将 $2 n$ 与 1 的连线依次顺时针旋转 $\\frac{2 \\pi}{2 n-1}, \\frac{4 \\pi}{2 n-1}, \\cdots, \\frac{(4 n-4) \\pi}{2 n-1}$, 作出相应的图及\n$X_1$ 的 $n$ 个二元子集.\n这样, $X_1$ 的全部 $\\mathrm{C}_{2 n}^2=n(2 n-1)$ 个二元子集被分成 $2 n-1$ 组, 且每组 $n$ 个集合构成 $X_1$ 的一个分划.\n下面来作满足题设的子集族:\n$$\n\\text { 令 } A=\\{1,2, \\cdots, 2 k\\}, B=\\{2 k+1,2 k+2, \\cdots, 4 k\\}, C=\\{4 k+1 ,4 k+2, \\cdots, 6 k\\}\n$$. \n由引理, $A$ 的全部二元子集可分成 $2 k-1$ 组, 每组是 $A$ 的一个分划.\n将其中一组重复一次, 得到 $A$ 的 $2 k$ 个分划, 让其中每个分划与 $B$ 的一个元素搭配作出 $k$ 个 $X$ 的三元子集.\n类似地,作出 $B$ 的 $2 k$ 个二元子集构成的分划, 包含 $B$ 的全部二元子集, 让其中每个分划与 $C$ 的一个元素搭配作出 $k$ 个 $X$ 的三元子集; 作出 $C$ 的 $2 k$ 个二元子集构成的分划, 包含 $C$ 的全部二元子集, 让其中每个分划与 $A$ 的一个元素搭配作出 $k$ 个 $X$ 的三元子集.\n上面得到的 $k \\times 2 k \\times 3=6 k^2$ 个 $X$ 的三元子集组成的族 $\\mathscr{A}$ 满足题设要求.\n说明 $X$ 的二元子集有 $\\mathrm{C}_{6 k}^2=3 k(6 k-1)=18 k^2-3 k$ 个.\n而所作的三元子集族中的每个集合 (子集族的元素) 都包含 3 个二元子集, 子集族共可生成二元子集 $3 \\times 6 k^2=18 k^2$ 个.\n这说明有 $3 k$ 个(次)二元子集在子集族中被重复生成.\n那么, 满足条件 (1) 的 $|\\mathcal{A}|$ 的最小值是 $6 k^2$ 吗?\n三、有关子集族的最值问题有关集合子集族的最值主要有三类:(1)求子集族阶的最值; (2) 求子集族中的集合阶的最值; (3) 求符合特定条件的集合元素的最值.", + "remark": "", + "figures": [ + "./images/volume1/figures/fig-c5e6.png" + ] +} \ No newline at end of file diff --git a/processed_dataset/proof/0028.json b/processed_dataset/proof/0028.json new file mode 100644 index 0000000000000000000000000000000000000000..77fe642d259e965a4c1ae4ad25f3e75fa51bfde7 --- /dev/null +++ b/processed_dataset/proof/0028.json @@ -0,0 +1,8 @@ +{ + "source_file": "./raw_volume-zh/volume1/chapter5.tex", + "problem_type": "proof", + "problem": "例7. 集合 $A=\\{0,1,2, \\cdots, 9\\},\\left\\{B_1, B_2, \\cdots, B_k\\right\\}$ 是 $A$ 的一族非空子集, 当 $i \\neq j$ 时, $B_i \\cap B_j$ 至多有两个元素.\n求 $k$ 的最大值.", + "solution": "分析:集合 $A$ 的一元、二元、三元子集显然符合要求.\n而 $A$ 的任一多于三元的子集 $B^{\\prime}$ 必包含了.\n$A$ 的三元子集, 故 $B^{\\prime}$ 与其包含的三元子集不能同在题中的子集族内.\n解首先至多含 3 个元素的 $A$ 的非空子集有\n$$\n\\mathrm{C}_{10}^1+\\mathrm{C}_{10}^2+\\mathrm{C}_{10}^3=10+\\frac{10 \\times 9}{2}+\\frac{10 \\times 9 \\times 8}{6}=175 \\text { (个). }\n$$\n这些集合的交集至多有两个元素, 否则两集合相等, 矛盾.\n因此 $k_{\\max } \\geqslant 175$.\n下面证明 $k_{\\max } \\leqslant 175$.\n设 $\\mathscr{b}$ 为满足题设的子集族.\n若 $B \\in \\mathscr{C}$, 且 $|B| \\geqslant 4$, 设 $b \\in B$, 则 $B$ 与 $B- \\{b\\}$ 不能同时含于 $\\mathscr{C}$, 以 $B-\\{b\\}$ 代 $B$, 则 $\\mathscr{C}$ 中元素数目不变.\n仿此对 $\\mathscr{C}$ 中所有元素数目多于 4 的集合 $B$ 作相应替代, 替代后子集族 $\\mathscr{C}$ 中的每个集合都是元素数目不多于 3 的非空集合.\n故 $k_{\\max } \\leqslant 175$.\n所以, $k$ 的最大值为 175 .\n说明上述解答采用了“两边夹”的策略: 先得出 $k$ 的最大值不小于 175 , 然后指出 $k$ 不大于 175 , 从而得出 $k_{\\max }=175$.", + "remark": "", + "figures": [] +} \ No newline at end of file diff --git a/processed_dataset/proof/0029.json b/processed_dataset/proof/0029.json new file mode 100644 index 0000000000000000000000000000000000000000..289cd067629e9a33aa29ebb607775360886ae2f1 --- /dev/null +++ b/processed_dataset/proof/0029.json @@ -0,0 +1,8 @@ +{ + "source_file": "./raw_volume-zh/volume1/chapter5.tex", + "problem_type": "proof", + "problem": "例9. 设 $n$ 为正整数, 在数集\n$$\n\\{-n,-n+1,-n+2, \\cdots,-1,0,1, \\cdots, n-1, n\\}\n$$\n中最多选取多少个数, 可使任意三个数的和均不为 0 (三个数可以相同)?", + "solution": "分析:显然, 当选取的数的绝对值充分大时, 可使任意三个数的和均不为 0 .\n解设从题中数集中最多选取 $k$ 个数, 可使任意三个数的和均不为 0 . 考察子集\n$$\n\\left\\{-n, \\cdots,-\\left[\\frac{n}{2}\\right]-1,\\left[\\frac{n}{2}\\right]+1, \\cdots, n\\right\\},\n$$\n其中 $[x]$ 表示不超 $x$ 的最大整数.\n知当 $n$ 为偶数时, $k \\geqslant n$; 当 $n$ 为奇数时, $k \\geqslant n+1$.\n设 $A=\\left\\{a_1, a_2, \\cdots, a_m\\right\\}, B=\\left\\{b_1, b_2, \\cdots, b_l\\right\\}$ 都是元素为整数的非空集合.\n定义集合\n$$\nA+B=\\{a+b \\mid a \\in A, b \\in B\\},\n$$\n可以证明 $A+B$ 至少有 $m+l-1$ 个元素.\n事实上, 不妨设 $a_11997$, 所以 $a<999$. 考虑集合 $A$ 的这样的元素 $b: 2 b \\in X_0$, $3 b \\notin X_0$. 易知 $b=666+i, i=0,1, \\cdots, 332$. 由 $B_i=\\{666+i\\} \\cup X_0 \\backslash \\{2(666+i)\\}, i=0,1, \\cdots, 332,\\left|B_i\\right|=999$, 知 $a \\leqslant 665$.\n解我们证明 $\\max a=665$.\n先证 $a \\leqslant 665$. 显然 $A$ 的 999 元子集 $X_0=\\{999,1000,1001, \\cdots, 1997\\}$ 中不存在 $x, y \\in X_0$, 使得 $x1997$, 即比 $X_0$ 的最大元素还大.\n这样, $a$ 就不能为 $999,1000,1001, \\cdots, 1997$ 中的任一个数.\n构造集合\n$$\nB_i=\\{666+i\\} \\bigcup X_0 \\backslash\\{2(666+i)\\}, i=0,1, \\cdots, 332 .\n$$\n对 $B_i$ 来说, $(666+i) \\times 3 \\geqslant 1998$, 而 $(666+i) \\times 2 \\notin B_i$, 故 $666+i$ 除本身外其他倍数都不在 $B_i$ 中.\n上面已证 $X_0$ 的任一非本身的倍数都不在 $X_0$ 中; 而 $666+i<999(i=0,1,2, \\cdots, 332)$, 故 $X_0$ 中任 $\\cdots$ 元素的倍数不可能为 $666+i(i=0,1, \\cdots, 332)$. 这样 $B_i$ 中仍不存在两元素满足 $xs,(r, s)=1$. 令集合\n$$\nN_\\alpha=\\{[n \\alpha] \\mid n=1,2, \\cdots\\} .\n$$\n求证:对任何 $m \\in N_\\alpha, r \\nmid m+1$.", + "solution": "分析:$n \\alpha=n \\cdot \\frac{r}{s}$. 当 $s=1$ 时, 结论显然成立.\n当 $s>1$ 时, 若 $1 \\leqslant n \\leqslant s-1$, 由 $\\frac{r}{s}>1$ 知, $1 \\leqslant n \\alpha \\leqslant r-\\frac{r}{s}1$, 结论显然成立.\n(2) 若 $s>1$, 因 $\\frac{r}{s}>1$, 故\n$$\n1 \\leqslant\\left[\\frac{r}{s}\\right]<\\left[\\frac{2 r}{s}\\right]<\\cdots<\\left[\\frac{(s-1) r}{s}\\right]=r+\\left[-\\frac{r}{s}\\right], 有 $0 \\leqslant[k \\alpha]1\n$$\n的数, 且 $\\alpha_i \\neq a_1, a_2, \\cdots, a_{2008}$. 由于\n$$\n\\alpha_i \\cdot A=\\alpha_i a_1 a_2 \\cdots a_{2008}=\\frac{p_i}{q_i} \\cdot \\frac{p}{q}\n$$\n为整数, 所以\n$$\nq_i \\mid p .\n$$\n由于 $p$ 只有有限个因子, 故有无数个分母为 $q_i^{\\prime}$ 的既约分数属于 $M$. 这些分数中的任意 2009 个的乘积都不是整数.\n这与题设矛盾.\n这说明 $M$ 中包含了无限多个整数, 记这些整数的集合为 $M^{\\prime}$.\n假设有 $\\frac{a}{b} \\in M,(a, b)=1, b>1$.\n设 $p$ 为 $b$ 的一个质因子.\n由于 $\\frac{a}{b}$ 与 $M^{\\prime}$ 中任意 2008 个整数的乘积为整数, 故 $p$ 为 $M^{\\prime}$ 中无数多个整数的质因子.\n而 $M^{\\prime}$ 中任意 2009 个含有因数 $p$ 的数的乘积可被 $p^{2009}$ 整除.\n这又与题设矛盾.\n这就证明了 $M$ 的元素均为整数.\n而这样的整数集是存在的, 如全部质数的集合.", + "remark": "", + "figures": [] +} \ No newline at end of file diff --git a/processed_dataset/proof/0035.json b/processed_dataset/proof/0035.json new file mode 100644 index 0000000000000000000000000000000000000000..fad32836a8980eb3886c571fef80aff90d2903a0 --- /dev/null +++ b/processed_dataset/proof/0035.json @@ -0,0 +1,8 @@ +{ + "source_file": "./raw_volume-zh/volume1/chapter6.tex", + "problem_type": "proof", + "problem": "例5. 三维空间中所有整点 (3 个坐标都为整数的点) 的集合记为 $T$. 两个整点 $(x, y, z)$ 和 $(u, v, w)$ 当且仅当 $|x-u|+|y-v|+|z-w|=1$ 时称为相邻.\n求证: 存在 $T$ 的一个子集 $S$, 使对每个 $P \\in T$, 点 $P$ 以及 $P$ 的所有邻点中恰有一点属于 $S$.", + "solution": "分析:设 $(u, v, w) \\in T$, 它的 6 个邻点分别为 $(u \\pm 1, v, w),(u, v \\pm 1$, $w),(u, v, w \\pm 1)$. 若函数 $f(x, y, z)$ 在以上 7 点的函数值为整数, 且除以 7 的余数都不相同,则原题获证.\n事实上, 取 $f=x+2 y+3 z$ 即可.\n证明显然, 两个整点相邻, 当且仅当两点的各 3 个坐标中的两对分别相等,而第 3 个坐标相差 1 .\n令 $\\quad S=\\{(x, y, z)\\mid \\, 7 | x+2 y+3 z\\}$,\n则 $S$ 满足题中要求.\n事实上, 对于任何 $(u, v, w) \\in T$, 它有 6 个邻点 $(u \\pm 1, v, w),(u, v \\pm 1, w),(u, v, w \\pm 1)$. 这 7 点所对应的 7 个整数\n$$\nu+2 v+3 w+j, j=-3,-2,-1,0,1,2,3\n$$\n中, 恰有一个是 7 的倍数, 从而相应的整点属于 $S$, 即 $S$ 满足题中要求.", + "remark": "", + "figures": [] +} \ No newline at end of file diff --git a/processed_dataset/proof/0036.json b/processed_dataset/proof/0036.json new file mode 100644 index 0000000000000000000000000000000000000000..a15d96e88e0abae72c414012eadcc518b03c867f --- /dev/null +++ b/processed_dataset/proof/0036.json @@ -0,0 +1,8 @@ +{ + "source_file": "./raw_volume-zh/volume1/chapter6.tex", + "problem_type": "proof", + "problem": "例6. 设 $A \\subset \\mathrm{N}^*$ 是无限集, $A$ 中每个数 $a$ 是至多 1990 个质数的乘积.\n证明: 必有.\n$A$ 的无限子集 $B$, 使得 $B$ 中任何两个不同数的最大公约数都相同.", + "solution": "分析:如果 $A$ 中含有无限多个两两互质的整数, 则结论显然成立.\n否则, 存在质数 $p_1$ 为 $A$ 的无限多个数的因数, 故 $A_1=\\left\\{\\frac{a}{p_1} \\mid \\frac{a}{p_1} \\in \\mathbf{Z}, a \\in A\\right\\}$ 为无限集.\n若 $A_1$ 中含有无限多个两两互质的整数, 则结论亦成立.\n否则, 继续上面的步骤.\n证明如果 $A$ 中含有无限多个两两互质的正整数, 将它们全部选出作成子集 $B$, 则结论成立.\n若存在质数 $p_1$ 为 $A$ 中无限多个数的因数,则集合\n$$\nA_1=\\left\\{\\frac{a}{p_1} \\mid \\frac{a}{p_1} \\in \\mathbf{Z}, a \\in A\\right\\}\n$$\n为无限集.\n依此类推 (用 $A_1$ 代替 $A$ ). 由于 $A$ 中每个数的质因数个数 $\\leqslant 1990$, 所以必有无限集\n$$\nA_k=\\left\\{\\frac{a}{p_1 p_2 \\cdots p_k} \\mid \\frac{a}{p_1 p_2 \\cdots p_k} \\in \\mathbf{Z}, \\frac{a}{p_1 p_2 \\cdots p_{k-1}} \\in A_{k-1}\\right\\},\n$$\n每个质数 $p_i$ 都仅是 $A_k$ 中有限多个数的因数.\n任取 $a_1 \\in A_k$. 在取定 $a_1, a_2, \\cdots, a_n$ 两两互质后, 由于每个质数都仅是 $A_k$ 中有限多个数的因数, 在 $A_k$ 中存在 $a_{n+1}$, 它与 $a_1, a_2, \\cdots, a_n$ 均互质.\n这样就得到 $A_k$ 的一个无穷子集 $B_k, B_k$ 中的元素两两互质.\n将 $B_k$ 中每个元素乘以 $p_1 p_2 \\cdots p_k$, 得到 $A$ 的无穷子集, 其中每两个数的最大公约数均为 $p_1 p_2 \\cdots p_k$.", + "remark": "", + "figures": [] +} \ No newline at end of file diff --git a/processed_dataset/proof/0037.json b/processed_dataset/proof/0037.json new file mode 100644 index 0000000000000000000000000000000000000000..cfab1a490e3a4411586c6adf3071a6064d3a6c64 --- /dev/null +++ b/processed_dataset/proof/0037.json @@ -0,0 +1,8 @@ +{ + "source_file": "./raw_volume-zh/volume1/chapter6.tex", + "problem_type": "proof", + "problem": "例7. 记 $\\mathbf{Q}$ 为有理数集合, $\\mathbf{Q}$ 的非空子集 $S$ 具有以下性质:\n(1) $0 \\notin S$;\n(2) 若 $s_1 \\in S, s_2 \\in S$, 则 $s_1 / s_2 \\in S$;\n(3) 存在一非零有理数 $q, q \\notin S$, 且每一个不在 $S$ 中的非零有理数都可写成 $q s$ 的形式,其中 $s \\in S$.\n证明: 若 $x \\in S$, 则存在 $y, z \\in S$, 使 $x=y+z$.", + "solution": "分析:设 $\\alpha, \\beta \\in \\mathbf{Q}$, 且 $\\alpha+\\beta=1$, 则\n$$\nx=x(\\alpha+\\beta)=x \\alpha+x \\beta .\n$$\n我们希望出现: $x \\alpha \\in S$ 且 $x \\beta \\in S$. 由(3)似乎应该有 $\\alpha, \\beta \\in S$. 于是我们要解决两个问题: (1) 怎样的 $\\alpha$ 、必定属于 $S$; (2) 如 $x_1 \\in S, x_2 \\in S$, 则 $x_1 x_2 \\in S$.\n证明假设 $s \\in S$. 令 $s_1=s_2 \\in S$, 则 $s_1 / s_2=1 \\in S$. 令 $s_1=1, s_2=s$, 则 $1 / s \\in S$.\n若 $t \\in S$, 令 $s_1=t, s_2=1 / s$, 则 $s_1 / s_2=t /(1 / s)=s t \\in S$ (这样 $s$ 就是乘法意义下的解).\n假设 $u$ 是一个非零有理数, 若 $u \\notin S$, 则 $u=q s$, 其中 $s \\in S$, 于是我们有 $u^2=q^2 s^2$.\n若 $q^2 \\notin S$, 则可设 $q^2=q t(t \\in S)$, 则 $q=t \\in S$, 矛盾.\n所以 $q^2 \\in S$, $u^2 \\in S$.\n假如 $x \\in S$, 则由 $(3 / 5)^2 、(4 / 5)^2$ 为平方数可知,\n$$\nx(3 / 5)^2 \\in S, x(4 / 5)^2 \\in S .\n$$\n又 $x=x(3 / 5)^2+x(4 / 5)^2$, 取 $y=x(3 / 5)^2, z=x(4 / 5)^2$, 则命题得证.", + "remark": "", + "figures": [] +} \ No newline at end of file diff --git a/processed_dataset/proof/0038.json b/processed_dataset/proof/0038.json new file mode 100644 index 0000000000000000000000000000000000000000..16395278bdec94c768b9495c1b13141bd4481e32 --- /dev/null +++ b/processed_dataset/proof/0038.json @@ -0,0 +1,8 @@ +{ + "source_file": "./raw_volume-zh/volume1/chapter6.tex", + "problem_type": "proof", + "problem": "例8. 证明: 对任意的 $n \\in \\mathbf{N}, n \\geqslant 2$, 都存在 $n$ 个互不相等的自然数组成的集合 $M$, 使得对任意的 $a \\in M$ 和 $b \\in M$, 都有 $(a-b) \\mid(a+b)$.", + "solution": "分析:设 $a_1j(i, j=1,2, \\cdots, n)$, 则\n$$\n\\begin{aligned}\nA & =\\frac{\\left(a_{k} !+a_i\\right)+\\left(a_{k} !+a_j\\right)}{\\left(a_{k} !+a_i\\right)-\\left(a_{k} !+a_j\\right)} \\\\\n& =\\frac{2\\left(a_{k} !\\right)+a_i+a_j}{a_i-a_j} .\n\\end{aligned}\n$$\n因为 $\\left(a_i-a_j\\right) \\mid\\left(a_i+a_j\\right)$ (归纳假设), $\\left(a_i-a_j\\right) \\mid 2\\left(a_k\\right.$ !), 所以 $A \\in \\mathbf{N}^*$.\n说明对上面的分析稍作整理即为本例的证明.\n略.", + "remark": "", + "figures": [] +} \ No newline at end of file diff --git a/processed_dataset/proof/0039.json b/processed_dataset/proof/0039.json new file mode 100644 index 0000000000000000000000000000000000000000..5e6d076f240e1fb2af1e08f2c4b45658a07d545d --- /dev/null +++ b/processed_dataset/proof/0039.json @@ -0,0 +1,8 @@ +{ + "source_file": "./raw_volume-zh/volume1/chapter6.tex", + "problem_type": "proof", + "problem": "例9. 平面上整点的集合 $M=\\{(x, y) \\mid x, y \\in \\mathbf{Z}$, 且 $1 \\leqslant x \\leqslant 12,1 \\leqslant y \\leqslant 13\\}$. 证明: 不少于 49 个点的 $M$ 的每一个子集, 必包含一个矩形的 4 个顶点, 且此矩形的边平行于坐标轴.", + "solution": "分析:设 $S$ 为 $M$ 的任一个 49 元子集.\n其中纵坐标相同的点的横坐标的集合为:\n$$\nX_i=\\{x \\mid(x, i) \\in S\\}, i=1,2, \\cdots, 13 .\n$$\n若存在关于整点横坐标的二元集 $(r, s)$ 同时是 $X_i 、 X_j \\quad(i \\neq j)$ 的子集, 则原题得证.\n证明设 $S$ 为 $M$ 的任一个 49 元子集.\n令\n$$\nX_i=\\{x \\mid(x, i) \\in S\\}, i=1,2, \\cdots, 13,\n$$\n则 $\\left|X_i\\right|=x_i, \\sum_{i=1}^{13} x_i=49,0 \\leqslant x_i \\leqslant 12$. 记\n$$\nP_i=\\{\\{r, s\\} \\mid r \\neq s,(r, i),(s, i) \\in S\\}, i=1,2, \\cdots, 13 .\n$$\n显然, 全体 $P_i$ 中只有 $\\mathrm{C}_{12}^2=66$ 种不同的二元集.\n又 $\\sum_{i=1}^{13}\\left|P_i\\right|=\\sum_{i=1}^{13} \\mathrm{C}_{x_i}^2$, 考虑其最小值.\n利用局部调整: 当 $x_1+x_2=c$ 时,\n$$\n\\mathrm{C}_{x_1}^2+\\mathrm{C}_{x_2}^2=\\frac{c^2-c}{2}-x_1 x_2 \\geqslant \\frac{c^2-c}{2}-\\frac{c^2}{4},\n$$\n$x_1=\\left[\\frac{c}{2}\\right], x_2=c-\\left[\\frac{c}{2}\\right]$ 时, $\\mathrm{C}_{x_1}^2+\\mathrm{C}_{x_2}^2$ 取得最小值.\n由此知, $\\sum_{i=1}^{13} \\mathrm{C}_{x_i}^2$ 取得最小值必须是将 $49=\\sum_{i=1}^{13} x_i$ 尽可能地平均到 $\\left\\{x_i\\right\\}$ 中, 即 $\\left\\{x_i\\right\\}$ 中有 $j$ 个 $\\left[\\frac{49}{13}\\right]=3$, $(13-j)$ 个 $\\left[\\frac{49}{13}\\right]+1=4$, 从而得 $j=3$.\n所以\n$$\n\\left(\\sum_{i=1}^{13} \\mathrm{C}_{x_i}^2\\right)_{\\min }=3 \\mathrm{C}_3^2+10 \\mathrm{C}_4^2=69\n$$\n从而, 有\n$$\n\\sum_{i=1}^{13}\\left|P_i\\right|=\\sum_{i=1}^{13} \\mathrm{C}_{x_i}^2 \\geqslant 69>66\n$$\n由此推知存在 $i \\neq j$, 使得 $(r, s) \\in P_i,(r, s) \\in P_j$.\n故有 $(r, i),(s, i),(r, j),(s, j) \\in S$, 结论成立.", + "remark": "", + "figures": [] +} \ No newline at end of file diff --git a/processed_dataset/proof/0040.json b/processed_dataset/proof/0040.json new file mode 100644 index 0000000000000000000000000000000000000000..76cf6d9f818de6d9adc1ce5990f06d38c05071a1 --- /dev/null +++ b/processed_dataset/proof/0040.json @@ -0,0 +1,8 @@ +{ + "source_file": "./raw_volume-zh/volume1/chapter6.tex", + "problem_type": "proof", + "problem": "例10. 设 $S=\\{1,2, \\cdots, 17\\}$, 而 $\\left\\{a_1, a_2, \\cdots, a_8\\right\\}$ 为 $S$ 的一个 8 元子集.\n求证:\n(1) 存在 $k \\in \\mathbf{N}^*$, 使得方程 $a_i-a_j=k$ 至少有 3 组不同的解;\n(2) 对于 $S$ 的 7 元子集 $\\left\\{a_1, a_2, \\cdots, a_7\\right\\}$,(1) 中的结论不再总是成立.", + "solution": "分析:(1) 不妨设 $a_1)), 且 $A B=A C$, 则取底边中点 $D$ 和底边另一点 $E$, 连结顶点和底边上这两个点, 把三角形分为三部分, 易知其中 $\\triangle A E C$ 为钝角三角形, 且能按照图()拼成矩形.\n若 $\\triangle A B C$ 为非等腰三角形(如图()), 不妨设 $\\angle A$ 为其最大的角.\n作 $A D \\perp B C$ 于点 $D$, 在线段 $B D$ 上取点 $M$, 使 $M D=D C$. 设 $B M 、 A B$ 的中点分别为 $E 、 F$, 连结 $E F$. 则 $\\triangle B E F 、 \\triangle A D C$ 、四边形 $A D E F$ 可按照图()拼成矩形, 且易知 $\\triangle B E F$ 为钝角三角形.\n在解有关整数的问题时,常常利用剩余类来分类.", + "remark": "", + "figures": [ + "./images/volume1/figures/fig-c7e5-1.png", + "./images/volume1/figures/fig-c7e5-2.png", + "./images/volume1/figures/fig-c7e5-3.png", + "./images/volume1/figures/fig-c7e5-4.png" + ] +} \ No newline at end of file diff --git a/processed_dataset/proof/0042.json b/processed_dataset/proof/0042.json new file mode 100644 index 0000000000000000000000000000000000000000..4b4f2b4086708da35e00fcc990cfdb6637bce1ba --- /dev/null +++ b/processed_dataset/proof/0042.json @@ -0,0 +1,8 @@ +{ + "source_file": "./raw_volume-zh/volume1/chapter7.tex", + "problem_type": "proof", + "problem": "例9. 设 $S$ 为集合 $\\{1,2, \\cdots, 50\\}$ 的具有下列性质的子集, $S$ 中任意两个不同的元素之和不被 7 整除.\n则 $S$ 中的元素最多可能有几个?", + "solution": "解:将 $\\{1,2, \\cdots, 50\\}$ 按照模 7 分成 7 类:\n$$\n\\begin{aligned}\n& K_1=\\{1,8,15,22,29,36,43,50\\}, \\\\\n& K_2=\\{2,9,16,23,30,37,44\\}, \\\\\n& K_3=\\{3,10,17,24,31,38,45\\}, \\\\\n& K_4=\\{4,11,18,25,32,39,46\\}, \\\\\n& K_5=\\{5,12,19,26,33,40,47\\}, \\\\\n& K_6=\\{6,13,20,27,34,41,48\\}, \\\\\n& K_0=\\{7,14,21,28,35,42,49\\} .\n\\end{aligned}\n$$\n下面证明 $S=K_1 \\cup K_2 \\cup K_3 \\cup\\{7\\}$ 为满足要求的元素最多的集合.\n首先, 对 $a, b \\in S, a \\neq b$, 有 3 种可能:\n(1) $a, b \\in K_i(1 \\leqslant i \\leqslant 3)$, 则\n$$\na+b \\equiv 2 i(\\bmod 7),\n$$\n有 $a+b$ 不能被 7 整除.\n(2) $a \\in K_i, b \\in K_j(1 \\leqslant i \\neq j \\leqslant 3)$, 则\n$$\na+b \\equiv i+j(\\bmod 7),\n$$\n有 $a+b$ 不能被 7 整除.\n(3) $a \\in K_i, b=7(1 \\leqslant i \\leqslant 3)$, 则\n$$\na+b=i(\\bmod 7) \\text {, }\n$$\n有 $a+b$ 不能被 7 整除.\n综上知, $S$ 中任两个元素之和不能被 7 整除.\n其次证明, 若给 $S$ 添加一个元素 $c$, 则必存在 $S$ 中的一个元素与 $c$ 之和, 能被 7 整除.\n添加的 $c$ 有 4 种可能:\n(1) $c \\in K_4$, 则 $c$ 与 $K_3$ 中的元素之和能被 7 整除.\n(2) $c \\in K_5$, 则 $c$ 与 $K_2$ 中的元素之和能被 7 整除.\n(3) $c \\in K_6$, 则 $c$ 与 $K_1$ 中的元素之和能被 7 整除.\n(4) $c \\in K_0$, 则 $c$ 与 7 之和能被 7 整除.\n综上知, $S$ 中的元素不能再增添.\n所以 $S$ 中元素数目的最大值为\n$$\n|S|=\\left|K_1\\right|+\\left|K_2\\right|+\\left|K_3\\right|+1=23 .\n$$\n说明这里首先按模 7 的剩余类对集合 $\\{1,2, \\cdots, 50\\}$ 的元素分类是自然的.\n后面的解答中又进行了两次分类, 但这两个分类的理由已经蕴涵在最初的分类之中了.", + "remark": "", + "figures": [] +} \ No newline at end of file diff --git a/processed_dataset/proof/0043.json b/processed_dataset/proof/0043.json new file mode 100644 index 0000000000000000000000000000000000000000..a02e0d0c7323812df5721e175a6cdaa0341a0a80 --- /dev/null +++ b/processed_dataset/proof/0043.json @@ -0,0 +1,8 @@ +{ + "source_file": "./raw_volume-zh/volume1/chapter7.tex", + "problem_type": "proof", + "problem": "例10. 设 $n 、 m 、 k$ 都是自然数, 且 $m \\geqslant n$. 证明: 如果\n$$\n1+2+\\cdots+n=m k,\n$$\n则可将数 $1,2, \\cdots, n$ 分成 $k$ 组, 使每一组数的和都等于 $m$.", + "solution": "证明:对 $n$ 进行归纳.\n当 $n=1$ 时,结论显然成立.\n假设对一切小于 $n$ 的自然数结论成立, 我们来考察集合 $S_n=\\{1$, $2, \\cdots, n\\}$ 的情形.\n如果 $m=n$, 那么 $\\frac{1}{2}(n+1)=k$ 为整数, 于是可按如下方式分组:\n$$\n\\{n\\},\\{1, n-1\\},\\{2, n-2\\}, \\cdots,\\left\\{-\\frac{1}{2}(n-1), \\frac{1}{2}(n+1)\\right\\} .\n$$\n如果 $m=n+1$, 那么 $n=2 k$ 为偶数, 则分组方式具有形式:\n$$\n\\{1, n\\},\\{2, n-1\\}, \\cdots,\\left\\{\\frac{n}{2}, \\frac{n}{2}+1\\right\\} .\n$$\n对其余情形再分三种情况讨论:\n情况 1: $n+10$. 我们从 $S_n$ 中分出 $S_{n-2 k}$, 后者中的数字之和为\n$$\n\\frac{1}{2}(n-2 k)(n-2 k+1)=\\frac{1}{2} n(n+1)-k(2 n+1)+2 k^2,\n$$\n它可被 $k$ 整除,且所得之商不小于 $n-2 k$. 这是因为\n$$\n\\frac{(n-2 k)(n-2 k+1)}{2(n-2 k)}=\\frac{1}{2}(n-2 k+1) \\geqslant k .\n$$\n于是由归纳假设知可将 $S_{n-2 k}$ 中的数字分为 $k$ 组, 使各组之和相等.\n再将剩下的 $2 k$ 个数字两两配对, 使各对数字之和相等: $\\{n-2 k+1, n\\},\\{n- 2 k+2, n-1\\}, \\cdots$ 然后再将这 $k$ 对数字分别并人前面所分出的 $k$ 组数字, 即可得到合乎需要的 $k$ 组数字.\n综上, 对 $S_n$ 结论成立.\n说明上述解答是在用数学归纳法证明的过程中采用分类法: 在归纳证明的第二步中, 我们对 $m$ 的取值范围分了 5 类来讨论.", + "remark": "", + "figures": [] +} \ No newline at end of file diff --git a/processed_dataset/proof/0044.json b/processed_dataset/proof/0044.json new file mode 100644 index 0000000000000000000000000000000000000000..67754ffd658663efe9637b44118dfd6e6e29da95 --- /dev/null +++ b/processed_dataset/proof/0044.json @@ -0,0 +1,8 @@ +{ + "source_file": "./raw_volume-zh/volume1/chapter8.tex", + "problem_type": "proof", + "problem": "例1. 已知 $S_1 、 S_2 、 S_3$ 为非空整数集合, 且对于 $1 、 2 、 3$ 的任意一个排列 $i 、 j 、 k$, 若 $x \\in S_i, y \\in S_j$, 则 $x-y \\in S_k$.\n(1) 证明: $S_1 、 S_2 、 S_3$ 三个集合中至少有两个相等.\n(2) 这三个集合中是否可能有两个集合无公共元素?", + "solution": "证明:(1)由已知,若 $x \\in S_i, y \\in S_j$, 则\n$$\ny-x \\in S_k,(y-x)-y=-x \\in S_i,\n$$\n所以每个集合中均有非负元素.\n当三个集合中的元素都为零时, 命题显然成立.\n否则, 设 $S_1 、 S_2 、 S_3$ 中的最小正元素为 $a$, 不妨设 $a \\in S_1$. 设 $b$ 为 $S_2 、 S_3$ 中最小的非负元素, 不妨设 $b \\in S_2$, 则 $b-a \\in S_3$.\n若 $b>0$, 则 $0 \\leqslant b-a\\sqrt{2 n} .\n\\end{gathered}\n$$\n因为 $a \\in N$, 所以 $a \\geqslant[\\sqrt{2 n}]$.", + "remark": "", + "figures": [] +} \ No newline at end of file diff --git a/processed_dataset/proof/0046.json b/processed_dataset/proof/0046.json new file mode 100644 index 0000000000000000000000000000000000000000..edd46893952e875eb4357858dc84d82fd2d5a78b --- /dev/null +++ b/processed_dataset/proof/0046.json @@ -0,0 +1,8 @@ +{ + "source_file": "./raw_volume-zh/volume1/chapter8.tex", + "problem_type": "proof", + "problem": "例3. 某地区网球俱乐部的 20 名成员举行 14 场单打比赛, 每人至少上场一次.\n求证: 必有六场比赛, 其 12 个参赛者各不相同.", + "solution": "证明:记参加第 $j$ 场比赛的选手为 $\\left(a_j, b_j\\right)$, 并记\n$$\nS=\\left\\{\\left(a_j, b_j\\right) \\mid j=1,2, \\cdots, 14\\right\\} .\n$$\n设 $M$ 为 $S$ 的一个子集.\n如果 $M$ 中所含选手对中出现的选手互不相同, 则称 $M$ 为 $S$ 的一个“好”子集.\n显然, 这样的“好”子集只有有限个, 其中必有一个元素最多的, 设这个元素最多的“好”子集为 $M_0$, 它的元素个数为 $r$, 显然只需证明 $r \\geqslant 6$.\n如果 $r \\leqslant 5$, 由于 $M_0$ 是元素个数最多的“好” 子集, 所以在 $M_0$ 中未出现过的 $20-2r$ 名选手之间互相没有比赛, 否则与 $M_0$ 的最大性矛盾.\n这就意味着, 这 $20-2 r$ 名选手所参加的比赛一定是同前 $2 r$ 名选手进行的.\n由于每名选手至少参加一场比赛, 所以除了 $M_0$ 中的 $r$ 场比赛之外, 至少还要进行 $20-2 r$ 场比赛.\n因此, 总比赛场数至少为\n$$\nr+20-2 r=20-r \\geqslant 15,\n$$\n与总比赛场次为 14 场矛盾.\n于是 $r \\geqslant 6$. 问题得证.", + "remark": "", + "figures": [] +} \ No newline at end of file diff --git a/processed_dataset/proof/0047.json b/processed_dataset/proof/0047.json new file mode 100644 index 0000000000000000000000000000000000000000..6332051536ac16131e830e6f45cc1ba2ca56cab5 --- /dev/null +++ b/processed_dataset/proof/0047.json @@ -0,0 +1,8 @@ +{ + "source_file": "./raw_volume-zh/volume1/chapter8.tex", + "problem_type": "proof", + "problem": "例4. 已知 $x_1, x_2, \\cdots, x_n$ 是实数, $a_1, a_2, \\cdots, a_n$ 和 $b_1, b_2, \\cdots, b_n$ 均是正整数, 令\n$$\n\\begin{aligned}\n& a=\\frac{a_1 x_1+a_2 x_2+\\cdots+a_n x_n}{a_1+a_2+\\cdots+a_n}, \\\\\n& b=\\frac{b_1 x_1+b_2 x_2+\\cdots+b_n x_n}{b_1+b_2+\\cdots+b_n} .\n\\end{aligned}\n$$\n求证: 在 $x_1, x_2, \\cdots, x_n$ 中必存在两个数 $x_i 、 x_j$, 使 $|a-b| \\leqslant \\mid a - x_i|\\leqslant| x_j-x_i \\mid$ 成立.", + "solution": "分析:要证明存在 $x_i$ 使 $|a-b| \\leqslant\\left|a-x_i\\right|$ 成立, 自然要在 $\\left|a-x_1\\right|$, $\\left|a-x_2\\right|, \\cdots,\\left|a-x_n\\right|$ 中取最大者来做 $\\left|a-x_i\\right|$. 同样的, 对于存在 $x_i 、 x_j$ 使 $\\left|a-x_i\\right| \\leqslant\\left|x_j-x_i\\right|$ 的证明, $\\left|x_j-x_i\\right|$ 应取 $\\left|x_1-x_i\\right|,\\left|x_2-x_i\\right|, \\cdots, \\left|x_n-x_i\\right|$ 中最大者.\n证明\n$$\n\\begin{aligned}\n& |a-b| \\\\\n= & \\left|a-\\frac{b_1 x_1+b_2 x_2+\\cdots+b_n x_n}{b_1+b_2+\\cdots+b_n}\\right| \\\\\n= & \\frac{\\left|b_1\\left(a-x_1\\right)+b_2\\left(a-x_2\\right)+\\cdots+b_n\\left(a-x_n\\right)\\right|}{b_1+b_2+\\cdots+b_n} \\\\\n\\leqslant & \\frac{b_1\\left|a-x_1\\right|+b_2\\left|a-x_2\\right|+\\cdots+b_n\\left|a-x_n\\right|}{b_1+b_2+\\cdots+b_n} .\n\\end{aligned}\n$$\n在 $\\left|a-x_1\\right|,\\left|a-x_2\\right|, \\cdots,\\left|a-x_n\\right|$ 中必有一个最大者, 设为 $\\left|a-x_i\\right|$.\n则有\n$$\n\\begin{aligned}\n|a-b| & \\leqslant \\frac{b_1\\left|a-x_i\\right|+b_2\\left|a-x_i\\right|+\\cdots+b_n\\left|a-x_i\\right|}{b_1+b_2+\\cdots+b_n} \\\\\n& =\\frac{\\left(b_1+b_2+\\cdots+b_n\\right)\\left|a-x_i\\right|}{b_1+b_2+\\cdots+b_n} \\\\\n& =\\left|a-x_i\\right| .\n\\end{aligned}\n$$\n下面再计算 $\\left|a-x_i\\right|$.\n$$\n\\begin{aligned}\n\\left|a-x_i\\right| & =\\left|\\frac{a_1 x_1+a_2 x_2+\\cdots+a_n x_n}{a_1+a_2+\\cdots+a_n}-x_i\\right| \\\\\n& =\\frac{\\left|a_1\\left(x_1-x_i\\right)+a_2\\left(x_2-x_i\\right)+\\cdots+a_n\\left(x_n-x_i\\right)\\right|}{a_1+a_2+\\cdots+a_n} \\\\\n& \\leqslant \\frac{a_1\\left|x_1-x_i\\right|+a_2\\left|x_2-x_i\\right|+\\cdots+a_n \\mid x_n-x_i \\mid}{a_1+a_2+\\cdots+a_n} .\n\\end{aligned}\n$$\n在 $\\left|x_1-x_i\\right|,\\left|x_2-x_i\\right|, \\cdots,\\left|x_n-x_i\\right|$ 中必有最大者, 设为 $\\left|x_j-x_i\\right|$.\n则\n$$\n\\begin{aligned}\n\\left|a-x_i\\right| & \\leqslant \\frac{a_1\\left|x_j-x_i\\right|+a_2\\left|x_j-x_i\\right|+\\cdots+a_n\\left|x_j-x_i\\right|}{a_1+a_2+\\cdots+a_n} \\\\\n& =\\frac{\\left(a_1+a_2+\\cdots+a_n\\right)\\left|x_j-x_i\\right|}{a_1+a_2+\\cdots+a_n} \\\\\n& =\\left|x_j-x_i\\right| .\n\\end{aligned}\n$$\n于是, 存在 $x_i 、 x_j$, 使\n$$\n|a-b| \\leqslant\\left|a-x_i\\right| \\leqslant\\left|x_j-x_i\\right|\n$$\n成立.", + "remark": "", + "figures": [] +} \ No newline at end of file diff --git a/processed_dataset/proof/0048.json b/processed_dataset/proof/0048.json new file mode 100644 index 0000000000000000000000000000000000000000..9069233226f21170e8cc3ab9234870201975ce11 --- /dev/null +++ b/processed_dataset/proof/0048.json @@ -0,0 +1,8 @@ +{ + "source_file": "./raw_volume-zh/volume1/chapter8.tex", + "problem_type": "proof", + "problem": "例5. 求方程\n$$\nx^4+4 y^4=2\\left(z^4+4 u^4\\right)\n$$\n的整数解.", + "solution": "分析:本例可以运用无穷递降法来解.\n设 $(x, y, z, u)$ 是方程的一组解, 且其中 $x$ 是所有解中取最小正整数者, 我们就让 “无穷递降” 的过程从此开始, 看看后面会出现什么情况.\n解显然, 方程(1)有解\n$$\nx=y=z=u=0 .\n$$\n我们证明这是方程(1)的惟一一组整数解.\n若 ( $x, y, z, u)$ 是方程(1)的解, 则 ( $|x|, y, z, u)$ 必是方程 (1) 的解.\n故不妨设 ( $x, y, z, u)$ 是方程 (1) 的所有解中 $x$ 取最小正整数者.\n易知, $x$ 为偶数.\n设 $x=2 x_1, x_1 \\in \\mathbf{N}^*$, 则有\n$$\n\\begin{aligned}\n16 x_1^4+4 y^4 & =2\\left(z^4+4 u^4\\right), \\\\\n8 x_1^4+2 y^4 & =z^4+4 u^4 .\n\\end{aligned}\n$$\n因而 $z$ 是偶数.\n设 $z=2 z_1, z_1 \\in \\mathbf{Z}$, 则有\n$$\n\\begin{gathered}\n8 x_1^4+2 y^4=16 z_1^4+4 u^4, \\\\\n4 x_1^4+y^4=8 z_1^4+2 u^4 .\n\\end{gathered}\n$$\n因而 $y$ 是偶数.\n设 $y=2 y_1, y_1 \\in \\mathbf{Z}$, 则有\n$$\n\\begin{gathered}\n4 x_1^4+16 y_1^4=8 z_1^4+2 u^4, \\\\\n2 x_1^4+8 y_1^4=4 z_1^4+u^4 .\n\\end{gathered}\n$$\n因而 $u$ 是偶数.\n设 $u=2 u_1, u_1 \\in \\mathbf{Z}$, 则有\n$$\n\\begin{aligned}\n2 x_1^4+8 y_1^4 & =4 z_1^4+16 u_1^4, \\\\\nx_1^4+4 y_1^4 & =2\\left(z_1^4+4 u_1^4\\right) .\n\\end{aligned}\n$$\n由(2)知, $\\left(x_1, y_1, z_1, u_1\\right)$ 也是方程 (1) 的解.\n但 $00, b>0$ 知\n$$\n-t c b-t \\geqslant 0 \\text {. }\n$$\n由 $c$ 是(2)的根得\n$$\nc^2-t c b+b^2-t=0,\n$$\n于是 $c^2+b^2=t c b+t \\leqslant 0$. 出现矛盾.\n因而 $c>0$. 由(4)知\n$$\n0b$.\n设 $s$ 与 $t$ 是满足下列条件的整数:\n$$\n\\left\\{\\begin{array}{l}\na=b s-t \\\\\ns \\geqslant 2,0 \\leqslant t0, t^2+1>0,\n$$\n于是 (2) 式大于 0 , 即\n$$\n\\frac{b^2 s^2-2 b s t+t^2+b^2}{b^2 s-b t+1}>s-1 .\n$$\n同理\n$$\n\\frac{b^2 s^2-2 b s t+t^2+b^2}{b^2 s-b t+1}b>t$, 所以 $t=0$ 时, $s=b^2$ 为平方数; 若 $t \\neq 0$, 可仿此继续下去, 经过有限步之后, 总可以使最小的数变为 0 , 所以 $s$ 是平方数, 即 $\\frac{a^2+b^2}{a b+1}$ 是某个正整数的平方.", + "remark": "", + "figures": [] +} \ No newline at end of file diff --git a/processed_dataset/proof/0050.json b/processed_dataset/proof/0050.json new file mode 100644 index 0000000000000000000000000000000000000000..5e9daaeb7ad4981568a2ee578bcaeb9be6ac29f0 --- /dev/null +++ b/processed_dataset/proof/0050.json @@ -0,0 +1,11 @@ +{ + "source_file": "./raw_volume-zh/volume1/chapter8.tex", + "problem_type": "proof", + "problem": "例7. 在平面上有 $n$ 个 $(n \\geqslant 2)$ 不全共线的点.\n试证: 一定存在一条直线恰好通过这 $n$ 个点中的两个点.", + "solution": "分析:假设结论不成立.\n不妨设其中三点 $A$ 、 $B 、 C$ 都在直线 $l$ 上, 且 $B$ 在 $A 、 C$ 之间, $D$ 为 $l$ 外一点, 如图(),作 $D P_1 \\perp A C$. 不妨设 $A 、 B$ 在 $P_1$ 的同侧, 再作 $B P_2 \\perp A D$. 易知 $D P_1>B P_2$. 如直线 $A D$ 上还有第三点 $E$, 不妨设 $D 、 E$ 在 $P_2$ 的同侧, 且 $D P_2>E P_2$, 作 $E P_3 \\perp B D$, 则 $B P_2> E P_3$. 由假设,这个过程可以无限地进行下去, 而且每次得到的 “点到直线的距离” 都比前一次小.\n另一方面, 过 $n$ 个点的每两点作一条直线 (可能有三点共线), 然后由 $n$ 个点中每一点作到这些直线的距离, 显然这样的距离只有有限个.\n于是出现矛盾.\n至此, 我们实际上已找到了本例的一种证明方法.\n下面我们用最小数原理来改写上面的过程.\n证明由 $n$ 个点中每两点作一条直线 (可能出现三点共线), 考虑 $n$ 个点中每一点到这些直线的距离所成之集, 这样的距离只有有限个, 其中必有一个最小者.\n不妨设点 $P$ 到直线 $l$ 的距离最短.\n下面证明: $l$ 上仅有已知点中的两个点.\n若 $l$ 上有已知 $n$ 个点中的三个点, 过点 $P$ 作 $P F \\perp l$ 于 $F$, 则必有两点在点 $F$ 的同侧,如图(),设点 $X$ 、 点 $Y$ 在点 $F$ 的同侧 (如图 8-2), 且 $Y F>X F$. 设过点 $P$ 与点 $Y$ 的直线为 $m$, 这时点 $X$ 到 $m$ 的距离 $X Z$ 小于点 $P$ 到 $l$ 的距离 $P F$, 与假设 $P F$ 最小矛盾.\n所以, 直线 $l$ 上仅有已知点中的两个点.\n$l$ 即为所求.", + "remark": "", + "figures": [ + "./images/volume1/figures/fig-c8e7-1.png", + "./images/volume1/figures/fig-c8e7-2.png" + ] +} \ No newline at end of file diff --git a/processed_dataset/proof/0051.json b/processed_dataset/proof/0051.json new file mode 100644 index 0000000000000000000000000000000000000000..044cc126b624cf2ec434a945eb19e053916c6026 --- /dev/null +++ b/processed_dataset/proof/0051.json @@ -0,0 +1,8 @@ +{ + "source_file": "./raw_volume-zh/volume1/chapter8.tex", + "problem_type": "proof", + "problem": "例8. 在某个星系的每一个星球上, 都有一位天文学家在观测最近的星球.\n若每两个星球间的距离都不相等, 证明: 当星球的个数为奇数时, 一定有一个星球任何人都看不到.", + "solution": "证明:设有 $n$ 个星球 (同时也表示 $n$ 个天文学家) $A_1, A_2, \\cdots, A_n, n$ 为奇数.\n这些星球两两之间的距离所成的集合是有限集, 故必有最小值, 不妨设 $A_1 A_2$ 最小.\n除 $A_1 、 A_2$ 外还有 $n-2$ 个星球和 $n-2$ 位天文学家.\n假若他们当中至少有一位看见已选出的星球.\n例如 $A_3$ 看见 $A_2$, 如果谁也看不见 $A_3$, 则结论成立; 否则还有一位天文学家如 $A_4$ 可看见 $A_3$. 如果谁也看不见 $A_4$, 结论同样成立; 否则还有一位天文学家如 $A_5$ 可看见 $A_4$. 仿此下去.\n由于上述过程中前面星球上的天文学家看不见后面的行星, 而 $n$ 是一个有限数,必然有最后一颗星球任何人都看不到.\n如果其他天文学家都看不到 $A_1 、 A_2$, 则再从 $n-2$ 颗星球中选择距离最近的两个.\n依此类推.\n因为 $n$ 是奇数, 所以最后存在一颗星球, 任何人都看不到它.", + "remark": "", + "figures": [] +} \ No newline at end of file diff --git a/processed_dataset/proof/0052.json b/processed_dataset/proof/0052.json new file mode 100644 index 0000000000000000000000000000000000000000..f815fea660dd94ccb1b5ac1fd3c78e84ca7b0d4a --- /dev/null +++ b/processed_dataset/proof/0052.json @@ -0,0 +1,8 @@ +{ + "source_file": "./raw_volume-zh/volume1/chapter8.tex", + "problem_type": "proof", + "problem": "例9. 平面上已给出 997 个点, 将连结每两点的线段的中点染成红色.\n证明至少有 1991 个红点.\n能否找到恰有 1991 个红点的点集?", + "solution": "证明:由 997 个点连结每两点的线段只有有限条, 所以必有一条最长者.\n设 $A B$ 为诸线段中的最长者.\n$A$ 与其他 996 个点连结的线段的中点均在以 $A$ 为圆心, $\\frac{1}{2} A B$ 为半径的圆的内部或圆周上.\n$B$ 与其他 996 个点连结的线段的中点均在以 $B$ 为圆心, $\\frac{1}{2} A B$ 为半径的圆的内部或圆周上.\n所以至少有\n$$\n2 \\times 996-1=1991\n$$\n个中点, 即有 1991 个红点.\n下面我们构造恰有 1991 个红点的 997 个点的点集:\n在 $x$ 轴上取 997 个点,坐标分别为 $1,2, \\cdots, 997$, 则区间 $(1,997)$ 内分母为 1 或 2 的有理点就是全部的红点, 个数恰为 1991 个.", + "remark": "", + "figures": [] +} \ No newline at end of file diff --git a/processed_dataset/proof/0053.json b/processed_dataset/proof/0053.json new file mode 100644 index 0000000000000000000000000000000000000000..bb5d270807bf407414846ed8090e0590f47f6649 --- /dev/null +++ b/processed_dataset/proof/0053.json @@ -0,0 +1,8 @@ +{ + "source_file": "./raw_volume-zh/volume1/chapter8.tex", + "problem_type": "proof", + "problem": "例10. 若干名儿童围成一圈, 他们手中都拿有一些糖块.\n规定进行如下传递, 每次传递的方法是: 如果某人手中糖块数是奇数, 则他可再领取一块, 然后每人都把手中糖块的一半传给右边的小朋友.\n求证: 一定可以经过若干次传递,使得所有儿童手中的糖块数都相同.", + "solution": "分析:由题设知, 在每次传递前, 每个儿童手中都有偶数块糖, 其中必有最多者和最少者.\n证明不妨设某次传递前手中糖块数最多的人有 $2 m$ 块, 最少的有 $2 n$ 块, $m>n$. 进行一次传递后, 结果是\n(1) 传递后每人手中的糖块数仍在 $2 n$ 与 $2 m$ 之间;\n(2) 原来手中糖块数超过 $2 n$ 块的,传递后仍然超过 $2 n$ 块;\n(3) 至少有一名原来糖块数为 $2 n$ 的孩子,传递后糖块数超过了 $2 n$.\n事实上, 圈子中至少有一名拿 $2 n$ 块糖的孩子的左邻手中糖块数为 $2 h> 2 n$. 传递之后, 原拿 $2 n$ 块糖的孩子手中的糖块数变为 $n+h>2 n$.\n由于每传递一次,拿 $2 n$ 块糖的孩子数至少减少 1 , 故若干次后, 将使所有孩子手中的糖块数都大于 $2 n$. 当他们都通过领取而使自己手中糖块数为偶数时,孩子手中糖块数的最小值至少上升了 2 .\n由于孩子手中糖块数的最大值在传递过程中不增, 而经过若干次传递之后最小值至少上升 2 , 故知经过多次传递后总可以使最大值与最小值相等, 即所有孩子手中的糖块数都相同.", + "remark": "", + "figures": [] +} \ No newline at end of file diff --git a/processed_dataset/proof/0054.json b/processed_dataset/proof/0054.json new file mode 100644 index 0000000000000000000000000000000000000000..e5567596f90a14fe7a8d26f16df53d426479104f --- /dev/null +++ b/processed_dataset/proof/0054.json @@ -0,0 +1,8 @@ +{ + "source_file": "./raw_volume-zh/volume1/chapter8.tex", + "problem_type": "proof", + "problem": "例11. 在 $n$ 名选手参加的循环赛中, 每两人比赛一场 (无平局). 试证下列两种情形恰有一种发生:\n(1) 可将所有选手分成两个非空集合,使得一个集合中的任何一名选手都战胜另一个集合中的所有选手;\n(2) 可将 $n$ 名选手从 1 到 $n$ 编号, 使得第 $i$ 名选手战胜第 $i+1$ 名选手, $i=1,2, \\cdots, n$, 其中将 $n+1$ 理解为 1 .", + "solution": "证明:显然, (1) 和 (2) 不能同时出现, 以下证明 (1) 和 (2) 至少有一种出现.\n设选手 $A$ 胜场最多.\n若 $A$ 战胜其他所有选手, 则 (1) 成立, 否则必有选手 $C$ 胜 $A$. 因 $A$ 胜场最多, 故必有负于 $A$ 的选手 $B$ 战胜 $C$, 于是得到一个选手圈 $\\{A, B, C\\}: A$ 胜 $B, B$ 胜 $C, C$ 胜 $A$.\n设这样的圈中含选手数最多的其中之一为 $\\left\\{A_1, A_2, \\cdots, A_m\\right\\}$, 其中 $A_1$ 胜 $A_2, A_2$ 胜 $A_3, \\cdots, A_{m-1}$ 胜 $A_m, A_m$ 胜 $A_1$. 若 $m=n$, 则 (2) 成立.\n以下设 $m3)$ 个点组成的点集, 其中任三点不共线, 又设自然数 $k$ 满足不等式 $\\frac{n}{2}\\frac{n}{2}>\\frac{3}{2}$, 所以 $k \\geqslant 2$, 即每个点都至少与 $Z$ 中 2 个点有线段相连.\n不妨设 $A B$ 为 $Z$ 中点连成的线段.\n令\n$$\n\\begin{aligned}\n& M=\\{P \\mid P \\in Z, P \\text { 与 } A \\text { 有线段相连 }\\}-\\{B\\}, \\\\\n& N=\\{P \\mid P \\in Z, P \\text { 与 } B \\text { 有线段相连 }\\}-\\{A\\} .\n\\end{aligned}\n$$\n由于 $Z$ 中任一点至少引出 $k$ 条线段, 所以有 $|M| \\geqslant k-1,|N| \\geqslant k-1$. 又由于 $M \\cup N$ 中不含 $A 、 B$, 所以有 $|M \\cup N| \\leqslant n-2$. 因此\n$$\n\\begin{aligned}\n|M \\cap N| & =|M|+|N|-|M \\cup N| \\\\\n& \\geqslant(k-1)+(k-1)-(n-2) \\\\\n& =2 k-2-(n-2)\n\\end{aligned}\n$$\n$$\n>(n-2)-(n-2)=0 .\n$$\n所以 $M \\cap N \\neq \\varnothing$, 即存在点 $C \\in M$, 且 $C \\in N(C \\neq A, C \\neq B)$. 显然线段 $A B 、 A C 、 B C$ 构成三角形的三边.", + "remark": "", + "figures": [] +} \ No newline at end of file diff --git a/processed_dataset/proof/0056.json b/processed_dataset/proof/0056.json new file mode 100644 index 0000000000000000000000000000000000000000..e350e38213b52fe35e86936656c674a3a0c0b4c8 --- /dev/null +++ b/processed_dataset/proof/0056.json @@ -0,0 +1,8 @@ +{ + "source_file": "./raw_volume-zh/volume1/chapter9.tex", + "problem_type": "proof", + "problem": "例7. 若 $A_1 \\cup A_2 \\cup \\cdots \\cup A_m=\\left\\{a_1, a_2, \\cdots, a_n\\right\\}$, 且 $A_1, A_2, \\cdots, A_m$ 均为非空集合, 则集合 $A_1, A_2, \\cdots, A_m$ 的组数为\n$$\ng(m, n)=\\sum_{k=0}^{m-1}(-1)^k \\mathrm{C}_m^k\\left(2^{m-k}-1\\right)^n .\n$$", + "solution": "证明:对于 $A_1 \\cup A_2 \\cdots \\cup A_m=\\left\\{a_1, a_2, \\cdots, a_n\\right\\}$, 如果对任意正整数 $k$ (其中 $1 \\leqslant k \\leqslant m-1$ ), 在 $A_1, A_2, \\cdots, A_m$ 中至少有 $k$ 个集合为空集, 先确定出 $k$ 个空集, 确定的方式有 $\\mathrm{C}_m^k$ 种.\n对每一种方式确定出的 $k$ 个空集, 都有剩下的 $m-k$ 个集合.\n不妨设它们为 $A_1^{\\prime}, A_2^{\\prime}, \\cdots, A_{m-k}^{\\prime}$, 它们的并集仍是 $\\left\\{a_1, a_2, \\cdots, a_n\\right\\}$.\n仿第 2 节例 6 , 知集合 $A_1^{\\prime}, A_2^{\\prime}, \\cdots, A_{m-k}^{\\prime}$ 的组数为 $\\left(2^{m-k}-1\\right)^n$.\n即有: 在 $A_1, A_2, \\cdots, A_m$ 中至少有 $k$ 个空集时, $A_1, A_2, \\cdots, A_m$ 的组数是 $\\mathrm{C}_m^k\\left(2^{m-k}-1\\right)^n$. 记\n$$\n\\mathrm{C}_m^k\\left(2^{m-k}-1\\right)^n=h(m, n, k) .\n$$\n若 $A_1, A_2, \\cdots, A_m$ 均为非空集合, 且\n$$\nA_1 \\cup A_2 \\cup \\cdots \\cup A_m=\\left\\{a_1, a_2, \\cdots, a_n\\right\\},\n$$\n则由容斥原理知集合 $A_1, A_2, \\cdots, A_m$ 的组数是\n$$\n\\left(2^m-1\\right)^n+\\sum_{k=1}^{m-1}(-1)^k h(m, n, k) .\n$$\n也就是 $g(m, n)=\\left(2^m-1\\right)^n+\\sum_{k=1}^{m-1}(-1)^k \\mathrm{C}_m^k\\left(2^{m-k}-1\\right)^n$\n$$\n=\\sum_{k=0}^{m-1}(-1)^k \\mathrm{C}_m^k\\left(2^{m-k}-1\\right)^n .\n$$", + "remark": "", + "figures": [] +} \ No newline at end of file diff --git a/processed_dataset/proof/0057.json b/processed_dataset/proof/0057.json new file mode 100644 index 0000000000000000000000000000000000000000..0883c201ec7a7241f698fecac7a7fde2d8cd1418 --- /dev/null +++ b/processed_dataset/proof/0057.json @@ -0,0 +1,8 @@ +{ + "source_file": "./raw_volume-zh/volume1/chapter9.tex", + "problem_type": "proof", + "problem": "例9. 设 $p_i(i=1,2, \\cdots, m)$ 为正整数 $n$ 的全部质因数.\n求证:\n$$\n\\varphi(n)=n \\prod_{i=1}^m\\left(1-\\frac{1}{p_i}\\right) .\n$$", + "solution": "证明:记 $S=\\{1,2, \\cdots, n\\}$, 并设\n$$\nA_i=\\left\\{a\\left|a \\in S, p_i\\right| a\\right\\}, i=1,2, \\cdots, m .\n$$\n则 $\\varphi(n)=\\left|\\bigcap_{i=1}^m \\complement_S A_i\\right|$. 注意到\n$$\n\\begin{aligned}\n& \\left|A_i\\right|=\\left[\\frac{n}{p_i}\\right],\\left|A_i \\cap A_j\\right|=\\left[\\frac{n}{p_i p_j}\\right], \\cdots, \\\\\n& \\left|A_1 \\cap A_2 \\cap \\cdots \\cap A_m\\right|=\\left[\\frac{n}{p_1 p_2 \\cdots p_m}\\right],\n\\end{aligned}\n$$\n而 $p_i$ 为 $n$ 的不同的质因数, 上面各式中 [] 都可去掉, 由筛法公式得\n$$\n\\begin{aligned}\n\\varphi(n) & =|S|-\\sum_{i=1}^m\\left[\\frac{n}{p_i}\\right]+\\sum_{1 \\leqslant i0$, 则 $0 \\leqslant b- a0$, 所以 $(x, y) \\notin A$. 这与 $(x, y) \\in A \\cap B$ 矛盾.", + "remark": "", + "figures": [] +} \ No newline at end of file diff --git a/processed_dataset/proof/0066.json b/processed_dataset/proof/0066.json new file mode 100644 index 0000000000000000000000000000000000000000..c403246369b9d4cc57a1d223ead39b3522f92460 --- /dev/null +++ b/processed_dataset/proof/0066.json @@ -0,0 +1,8 @@ +{ + "source_file": "./raw_volume-zh/volume1/exercise2.tex", + "problem_type": "proof", + "problem": "问题15 设 $Z$ 表示所有整数的集合.\n对于固定的 $A, B, C \\in \\mathbf{Z}$, 令\n$$\n\\begin{aligned}\n& M_1=\\left\\{x^2+A x+B \\mid x \\in \\mathbf{Z}\\right\\}, \\\\\n& M_2=\\left\\{2 x^2+2 x+C \\mid x \\in \\mathbf{Z}\\right\\},\n\\end{aligned}\n$$\n求证: 对任何 $A, B \\in \\mathbf{Z}$, 都可选取 $C \\in \\mathbf{Z}$, 使得集合 $M_1$ 与 $M_2$ 互不相交.", + "solution": "如果 $A$ 为奇数, 则有 $x(x+A)+B \\equiv B(\\bmod 2)$, 这表明 $M_1$ 中的所有数都与 $B$ 奇偶性相同.\n对于 $M_2$ 中的数, 有 $2 x(x+1)+C \\equiv C(\\bmod 2)$. 可见, 为使 $M_1 \\cap M_2=\\varnothing$, 只须取 $C=B+1$ 即可.\n如果 $A$ 为偶数, 则有 $2 x(x+1)+C \\equiv C(\\bmod 4)$. 又因 $\\left(x+\\frac{A}{2}\\right)^2$ 作为完全平方数模 4 时只能为 0 或 1 , 故由 $x^2+A x+B=\\left(x+\\frac{A}{2}\\right)^2-\\left(\\frac{A}{2}\\right)^2+B$ 知 $M_1$ 中元素模 4 时只能与 $B 、 B+1$ 或 $B+3$ 同余.\n因而, 当取 $C=B+2$ 时, $M_1 \\cap M_2=\\varnothing$.", + "remark": "", + "figures": [] +} \ No newline at end of file diff --git a/processed_dataset/proof/0067.json b/processed_dataset/proof/0067.json new file mode 100644 index 0000000000000000000000000000000000000000..41a069233252f5d32b9a1253fd1b45894bdc00ff --- /dev/null +++ b/processed_dataset/proof/0067.json @@ -0,0 +1,8 @@ +{ + "source_file": "./raw_volume-zh/volume1/exercise2.tex", + "problem_type": "proof", + "problem": "问题16 设集合 $S_n=\\{1,2, \\cdots, n\\}$, 若 $Z$ 是 $S_n$ 的子集, 把 $Z$ 中的所有数的和称为 $Z$ 的“容量” (规定空集的容量为 0 ). 若 $Z$ 的容量为奇 (偶) 数, 则称 $Z$ 为 $S_n$ 的奇 (偶)子集.\n(1) 求证: $S_n$ 的奇子集与偶子集个数相等;\n(2)求证: 当 $n \\geqslant 3$ 时, $S_n$ 的所有奇子集的容量之和与所有偶子集的容量之和相等;\n(3)当 $n \\geqslant 3$ 时, 求 $S_n$ 的所有奇子集的容量之和.", + "solution": "设 $S$ 为 $S_n$ 的奇子集, 令 $T=\\left\\{\\begin{array}{l}S \\cup\\{1\\}, \\text { 若 } 1 \\notin S, \\\\ S \\backslash\\{1\\} \\text {, 若 } 1 \\in S .\\end{array}\\right.$ 则 $T$ 是偶子集, $S \\rightarrow T$ 是奇子集到偶子集的一一对应, 而且对每个偶子集 $T$, 恰有一个奇子集 $S=\\left\\{\\begin{array}{l}T \\cup\\{1\\}, \\text { 若 } 1 \\notin T, \\\\ T \\backslash\\{1\\}, \\text { 若 } 1 \\in T .\\end{array}\\right.$ 与之对应, 所以(1) 的结论成立.\n对任一 $i(1 \\leqslant i \\leqslant n)$, 含 $i$ 的子集共 $2^{n-1}$ 个, 用上面的对应方法可知当 $i \\neq 1$ 时, 这 $2^{n-1}$ 个集中有一半是奇子集.\n当 $i=1$ 时, 由于 $n \\geqslant 3$, 将上边的 1 换成 3 , 同样可得其中有一半是奇子集.\n于是在计算奇子集容量之和时, 元素 $i$ 的贡献是 $2^{n-2} \\cdot i$. 奇子集容量之和是 $\\sum_{i=1}^n 2^{n-2} i=n(n+1) \\cdot 2^{n-3}$. 由上可知, 这也是偶子集的容量之和, 两者相等.", + "remark": "", + "figures": [] +} \ No newline at end of file diff --git a/processed_dataset/proof/0068.json b/processed_dataset/proof/0068.json new file mode 100644 index 0000000000000000000000000000000000000000..4f4a0c89a8ca5e272d41469dbe9d53d49d0011d3 --- /dev/null +++ b/processed_dataset/proof/0068.json @@ -0,0 +1,8 @@ +{ + "source_file": "./raw_volume-zh/volume1/exercise2.tex", + "problem_type": "proof", + "problem": "问题17 已知一族集合 $A_1, A_2, \\cdots, A_n$ 具有性质:\n(1) 每个 $A_i$ 含 30 个元素;\n(2) 对每一对 $i 、 j, 1 \\leqslant i\\mathrm{C}_{20}^9$. 由此可知, “好集”是存在的.", + "remark": "", + "figures": [] +} \ No newline at end of file diff --git a/processed_dataset/proof/0070.json b/processed_dataset/proof/0070.json new file mode 100644 index 0000000000000000000000000000000000000000..b0bda665f45f62f75511a10cad178ae500752d04 --- /dev/null +++ b/processed_dataset/proof/0070.json @@ -0,0 +1,8 @@ +{ + "source_file": "./raw_volume-zh/volume1/exercise2.tex", + "problem_type": "proof", + "problem": "问题19 设 $k \\geqslant 6$ 为自然数, $n=2 k-1$. $T$ 为所有 $n$ 元数组 $\\left(x_1, x_2, \\cdots, x_n\\right)$ 的集合, 其中 $x_i \\in\\{0,1\\}, i=1,2, \\cdots, n$. 对于 $x=\\left(x_1, x_2, \\cdots, x_n\\right), y= \\left(y_1, y_2, \\cdots, y_n\\right) \\in T$, 定义\n$$\nd(x, y)=\\sum_{i=1}^n\\left|x_i-y_i\\right| .\n$$\n特别地有 $d(x, x)=0$. 设有一个由 $T$ 的 $2^k$ 个元素组成的子集 $S$, 使对任何 $x \\in T$, 都存在惟一的 $y \\in S$, 使得 $d(x, y) \\leqslant 3$. 求证: $n=23$.", + "solution": "由于 $d(x, x)=0<3$, 所以 $S$ 中任何两个元素的距离都大于 3 . 因而当将 $S$ 中的元素 $x$ 的 $n$ 个分量改变 1 个、2 个或 3 个 (1 变为 0 或 0 变为 1 ) 时, 所得的元素都在 $T-S$ 中, 且 $S$ 中的不同元素按上述办法所得的元素也互不相同.\n按假设知, 对每个 $x \\in T$, 都有惟一的 $y \\in S$, 使得 $d(x, y) \\leqslant 3$, 所以 $T- S$ 中的每个元素都可由 $S$ 中的元素按上述办法生成.\n从而有 $2^n=2^k\\left(\\mathrm{C}_n^0+\\mathrm{C}_n^1+\\right. \\left.\\mathrm{C}_n^2+\\mathrm{C}_n^3\\right)$. 由于 $n=2 k-1$, 故有 $3 \\cdot 2^{k-2}=k\\left(2 k^2-3 k+4\\right)$. (1) 若 $3 \\times k$, 则 $k= 2^m$. 由于 $k \\geqslant 6$, 所以 $m \\geqslant 3$. 于是 $2 k^2-3 k+4$ 是 4 的倍数但不是 8 的倍数.\n从而由 (1) 可得 $2 k^2-3 k+4=12$, 这个方程无解, 所以 $k$ 为 3 的倍数.\n记 $k= 3 h=3 \\cdot 2^q, q \\geqslant 1$, 于是 (1) 式化为 $2^{3 h-2}=h\\left(18 h^2-9 h+4\\right)$. (2) 当 $q \\geqslant 3$ 时, $18 h^2-9 h+4$ 是 4 的倍数不是 8 的倍数.\n由 (2) 知 $18 h^2-9 h+4=4$, 无解.\n从而 $q=1$ 或 $2, h=2$ 或 4 . 代入 (2) 式知 $h=2$ 不是根而 $h=4$ 是根.\n所以得到 $n=2 k-1==6 h-1=23$.", + "remark": "", + "figures": [] +} \ No newline at end of file diff --git a/processed_dataset/proof/0071.json b/processed_dataset/proof/0071.json new file mode 100644 index 0000000000000000000000000000000000000000..b3cc16f2f47af7b3e7d3cce0124b55007023843f --- /dev/null +++ b/processed_dataset/proof/0071.json @@ -0,0 +1,8 @@ +{ + "source_file": "./raw_volume-zh/volume1/exercise2.tex", + "problem_type": "proof", + "problem": "问题20 设 $n \\in \\mathbf{N}^*$, 而 $A_1, A_2, \\cdots, A_{2 n+1}$ 是集合 $B$ 的一族子集且满足条件:\n(1) 每个 $A_i$ 中恰含有 $2 n$ 个元素;\n(2) $A_i \\cap A_j(1 \\leqslant i0$. 由条件 $|x-y| \\geqslant \\frac{x y}{25}$ 易知, $A$ 中至多有 1 个元素不小于 25 , 从而有 $x_{n-1} \\leqslant 24$. 由 $d_i=\\mid x_{i+1}- x_i \\mid \\geqslant \\frac{x_{i+1} \\cdot x_i}{25}=\\frac{\\left(x_i+d_i\\right) x_i}{25}$, 解得 $d_i \\geqslant \\frac{x_i^2}{25-x_i}$. 若 $x_5 \\geqslant 5$, 则 $d_5 \\geqslant \\frac{25}{20}>1$, 故有 $x_6 \\geqslant 7$. 从而 $d_6>2, x_7 \\geqslant 10 ; d_7>6, x_8 \\geqslant 17 ; d_8>36, x_9 \\geqslant 54>25$. 可见, $A$ 中至多有 9 个元素.\n另一方面, 容易验证集合 $\\{1,2,3,4,5,7,10$, 17,543 满足题中要求.\n从而知集合 $A$ 中最多有 9 个元素.", + "remark": "", + "figures": [] +} \ No newline at end of file diff --git a/processed_dataset/proof/0073.json b/processed_dataset/proof/0073.json new file mode 100644 index 0000000000000000000000000000000000000000..33032e104f652503200f228c039b213c2495b08d --- /dev/null +++ b/processed_dataset/proof/0073.json @@ -0,0 +1,8 @@ +{ + "source_file": "./raw_volume-zh/volume1/exercise3.tex", + "problem_type": "proof", + "problem": "问题10 对于集合 $S=\\left\\{\\left(a_1, a_2, \\cdots, a_5\\right) \\mid a_i=0\\right.$ 或 $\\left.1, i=1,2, \\cdots, 5\\right\\}$ 中的任意两个元素 $A=\\left(a_1, a_2, \\cdots, a_5\\right)$ 和 $B=\\left(b_1, b_2, \\cdots, b_5\\right)$, 定义它们的距离为: $d(A, B)=\\left|a_1-b_1\\right|+\\cdots+\\left|a_5-b_5\\right|$. 取 $S$ 的一个子集 $T$, 使 $T$ 中任意两个元素之间的距离都大于 2 . 问子集 $T$ 中最多含多少个元素? 证明你的结论.", + "solution": "假设有一个 5 个元素的子集也符合条件,则这 5 个元素中至少有 3 个的第一位数码相同.\n不妨设 $A 、 B 、 C$ 这三个元素的第一位数码相同.\n同样, 在 $A 、 B 、 C$ 中, 第二、三、四、五个数码上, 每一位都至少有两个元素的对应数码相同.\n但 $A 、 B 、 C$ 三元素两两分组只有 3 组, 故至少有两个元素, 它们除第一数码相同外, 至少还有两位数码相同, 不妨设 $A$ 与 $B$, 则 $A 、 B$ 的距离不大于 2 , 矛盾.\n故 $T$ 的元素不多于 4 个.\n可令 $T=\\{(1,1,1,1,1),(0,0,0,1$, $1),(1,1,0,0,0),(0,0,1,0,0)\\}$, 则不难验证结论成立.\n所以 $|T|_{\\max }=4$.", + "remark": "", + "figures": [] +} \ No newline at end of file diff --git a/processed_dataset/proof/0074.json b/processed_dataset/proof/0074.json new file mode 100644 index 0000000000000000000000000000000000000000..18edeef8b8a86d576f3bdb71fd945b0575f71d5d --- /dev/null +++ b/processed_dataset/proof/0074.json @@ -0,0 +1,8 @@ +{ + "source_file": "./raw_volume-zh/volume1/exercise3.tex", + "problem_type": "proof", + "problem": "问题12 我们称一个正整数的集合 $A$ 是“一致”的,是指: 删除 $A$ 中任何一个元素之后, 剩余的元素可以分成两个不相交的子集, 而且这两个子集的元素之和相等.\n求最小的正整数 $n(n>1)$, 使得可以找到一个具有 $n$ 个元素的 “一致”集合 $A$.", + "solution": "设 $A=\\left\\{a_1, a_2, \\cdots, a_n\\right\\}$, 又设 $M$ 是 $A$ 中各元素之和.\n根据题中的条件可以断定, 对任何 $i=1,2, \\cdots, n, M-a_i$ 是偶数.\n如果 $M$ 是偶数, 则 $A$ 中每个元素也都是偶数, 即 $a_i=2 b_i$, 而集合 $\\left\\{b_1, b_2, \\cdots, b_n\\right\\}$ 仍然是“一致” 的.\n假定 $M$ 是奇数,故对于 $i=1,2, \\cdots, n, a_i$ 也都是奇数.\n由于 $a_1+a_2+\\cdots+ a_n=M, n$ 也是奇数.\n$n=7$ 时, 容易验证集合 $A=\\{1,3,5,7,9,11,13\\}$ 是一致的.\n假定 $n \\leqslant 5$, 对于 $n=1,3$ 的情形是显然的.\n设 $n=5$. 将元素按升序排列, 即 $a_11$, 使在集合 $\\{1,2, \\cdots, h(r)\\}$ 分成 $r$ 组的任何分划中,都存在整数 $a \\geqslant 0,1 \\leqslant x \\leqslant y$, 使数 $a+x, a+y, a+x+y$ 含于分划的同一组中.", + "solution": "考察将 $\\{1,2, \\cdots, 2 r\\}$ 分成 $r$ 组的任一分划.\n在 $r, r+1, \\cdots, 2 r$ 这 $r+1$ 个数中, 必有两个数 $u$ 和 $v$ 属于同一组, 不妨设 $u0(i=1, \\cdots$, $n)$. 令 $S_1=\\left\\{f(A) \\mid A=\\left\\{u_1\\right\\}\\right\\} . S_k=\\left\\{f(A) \\mid u_1+u_2+\\cdots+u_{k-1}u_{1}+u_{2}+\\cdots+u_{k-1}$ 则$\\frac{最大数}{最小数}={\\frac{u_{1}+u_{2}+\\cdots+u_{k-1}+u_{k}}{u_{k}}}<2$ .", + "remark": "", + "figures": [] +} \ No newline at end of file diff --git a/processed_dataset/proof/0084.json b/processed_dataset/proof/0084.json new file mode 100644 index 0000000000000000000000000000000000000000..4dbcd1e1fc3d596889d4dba10dd82998cfceb04b --- /dev/null +++ b/processed_dataset/proof/0084.json @@ -0,0 +1,8 @@ +{ + "source_file": "./raw_volume-zh/volume1/exercise4.tex", + "problem_type": "proof", + "problem": "问题15 给定集合 $S=\\left\\{z_1, z_2, \\cdots, z_{1993}\\right\\}$, 其中 $z_1, z_2, \\cdots, z_{1993}$ 都是非零复数 (可看作平面上的非零向量). 求证: 可以把 $S$ 中的元素分成若干组, 使得\n(1) $S$ 中的每个元素属于且仅属于其中一组;\n(2) 每组中的任一复数与该组中所有复数之和的夹角不超过 $90^{\\circ}$;\n(3) 将任意两组中的所有复数分别求和, 所得的两个和数之间的夹角大于 $90^{\\circ}$.", + "solution": "考虑集合 $S$ 的所有子集并计算每个子集中所有复数的和的模.\n因这样得到的模数只有有限多个, 故其中必有最大数.\n将模取最大值的子集之一记为 $A$. 如果 $S-A \\neq \\varnothing$, 再将 $S-A$ 的所有子集中能使其中所有复数之和的模达到最大的一个子集取为 $B$. 如果 $S-(A \\cup B) \\neq \\varnothing$, 则令 $C=S-(A \\cup B)$. 这样选取的至多 3 个子集便满足题中要求.\n将 $A 、 B 、 C$ 中所有元素之和分别记为 $a 、 b 、 c$.\n(i) 对任意 $z \\in A$, 如果 $z$ 与 $a$ 的夹角为钝角, 则 $-z$ 与 $a$ 的夹角为锐角.\n于是有 $|a+(-z)|>|a|$, 即子集 $A-\\{z\\}$ 中所有元素之和的模大于 $a$ 的模, 此与 $|a|$ 的最大性矛盾.\n这就证明了 $A$ 中任一元素与 $a$ 的夹角都不超过 $90^{\\circ}$. 同理, $B$ 中任一元素与 $b$ 的夹角也不超过 $90^{\\circ}$.\n(ii) 对任意 $\\xi \\in S-A, \\xi$ 与 $a$ 的夹角都是钝角.\n否则又导致 $|a+\\xi|>|a|$, 矛盾.\n同理, $C$ 中任一元素 $\\eta$ 与 $b$ 的夹角都是钝角.\n由此可见, $B$ 中所有元素均与 $a$ 成钝角, 从而其和 $b$ 与 $a$ 夹钝角.\n同理, $c$ 与 $a, c$ 与 $b$ 都夹钝角, 即 (3) 成立.\n(iii) 若存在 $\\xi \\in C$, 使 $\\xi$ 与 $c$ 夹针角, 则由 (ii) 知, 4 个数 $a 、 b 、 c 、 \\xi$ 两两之间都夹钝角, 此不可能.\n所以, $C$ 中任一元素与 $c$ 的夹角都不超过 $90^{\\circ}$.", + "remark": "", + "figures": [] +} \ No newline at end of file diff --git a/processed_dataset/proof/0085.json b/processed_dataset/proof/0085.json new file mode 100644 index 0000000000000000000000000000000000000000..eb134ca87bb3eecd743664a0985e80541083811f --- /dev/null +++ b/processed_dataset/proof/0085.json @@ -0,0 +1,8 @@ +{ + "source_file": "./raw_volume-zh/volume1/exercise4.tex", + "problem_type": "proof", + "problem": "问题16 设 $r 、 s 、 n$ 都是正整数, 并且 $r+s=n$. 求证: 集合\n$$\n\\begin{aligned}\n& A=\\left\\{\\left[\\frac{n}{r}\\right],\\left[\\frac{2 n}{r}\\right], \\cdots,\\left[\\frac{(r-1) n}{r}\\right]\\right\\}, \\\\\n& B=\\left\\{\\left[\\frac{n}{s}\\right],\\left[\\frac{2 n}{s}\\right], \\cdots,\\left[\\frac{(s-1)}{s} \\underline{n}\\right]\\right\\}\n\\end{aligned}\n$$\n构成 $N=\\{1,2, \\cdots, n-2\\}$ 的分划的充要条件是 $r$ 和 $s$ 都与 $n$ 互质, 其中 $[x]$ 表示不超过实数 $x$ 的最大整数.", + "solution": "因 $|A|=r-1,|B|=s-1,|N|=n-2$, 故 $A$ 与 $B$ 构成 $N$ 的一个分划等价于 $A \\cap B=\\varnothing$.\n必要性.\n若 $r 、 n$ 之一与 $n$ 有公因数 $d$, 则另一个也与 $n$ 有公因数 $d$. 设 $r= r^{\\prime} d, s=s^{\\prime} d$, 于是 $\\left[\\frac{r^{\\prime} n}{r}\\right]=\\left[\\frac{s^{\\prime} n}{s}\\right], A \\cap B \\neq \\varnothing$, 矛盾.\n充分性.\n假设 $A \\cap B \\neq \\varnothing$, 则存在整数 $a 、 b$, 使 $\\left[\\frac{a n}{r}\\right]=\\left[\\frac{b n}{s}\\right]=p, p< \\frac{a n}{r}b)$ 的和 $a+b$ 与差 $a-b$ 中至少有一个属于该集合.\n证明: 若将该集合中的数按递增的顺序排列, 则相邻两个数的差相同.", + "solution": "将 2003 个正数按递增顺序排列, 并记 $A=\\left\\{a_1, a_2, \\cdots, a_{2003}\\right\\}$. 因为 $\\left(a_{2003}+a_i\\right) \\notin A$, 所以 $\\left(a_{2003}-a_i\\right) \\in A, i=1,2, \\cdots, 2002$. 即 $a_{2003}-a_{2002}< a_{2003}-a_{2001}<\\cdotsA_{10}$, 则 $A_1>1$.", + "solution": "只需证明: 如果 $A_1 \\leqslant 1$, 那么对一切 $1 \\leqslant n \\leqslant 29$, 都有 $A_{n+1}0$, 由此即得所证.", + "remark": "", + "figures": [] +} \ No newline at end of file diff --git a/processed_dataset/proof/0094.json b/processed_dataset/proof/0094.json new file mode 100644 index 0000000000000000000000000000000000000000..ab598233b754a5427b9abecb3041bcbb08b0c61d --- /dev/null +++ b/processed_dataset/proof/0094.json @@ -0,0 +1,8 @@ +{ + "source_file": "./raw_volume-zh/volume1/exercise6.tex", + "problem_type": "proof", + "problem": "问题7 $S$ 为 $m$ 个无序正整数对 $(a, b)(1 \\leqslant a497$. 这样一来, 当将平行四边形 $P^{\\prime}$ 被网络所分成的诸块都平移到基本区域 $F$ 中时, 必有两点重叠.\n设这两点是 $D_1^{\\prime}\\left(x_1, y_1\\right)$ 和 $D_2^{\\prime}\\left(x_2, y_2\\right)$. 由于平移是沿网格线移动的, 所以点 $M\\left(x_1-x_2, y_1-y_2\\right) \\in L$.\n另一方面, 因为 $D_1^{\\prime}, D_2^{\\prime} \\in P^{\\prime}$, 所以 $D_1\\left(2 x_1, 2 y_1\\right), D_2\\left(2 x_2, 2 y_2\\right) \\in P$. 又因 $P$ 是以原点为中心的平行四边形, 所以 $D_3\\left(-2 x_2,-2 y_2\\right) \\in P$. 从而线段 $D_1 D_3$ 的中点 $M \\in P$, 且 $M \\neq(0,0)$. 这就证明了平行四边形 $P$ 中至少含有 $L$ 中的两个点.", + "remark": "", + "figures": [] +} \ No newline at end of file diff --git a/processed_dataset/proof/0096.json b/processed_dataset/proof/0096.json new file mode 100644 index 0000000000000000000000000000000000000000..f3d0121c2733a3267564b6592214f4a55d9e40c8 --- /dev/null +++ b/processed_dataset/proof/0096.json @@ -0,0 +1,8 @@ +{ + "source_file": "./raw_volume-zh/volume1/exercise6.tex", + "problem_type": "proof", + "problem": "问题9 证明: 在集合 $\\left\\{1,2,3, \\cdots, \\frac{3^n+1}{2}\\right\\}\\left(n \\in \\mathbf{N}^*\\right)$ 中可取出 $2^n$ 个数, 其中无三个数成等差数列.", + "solution": "用数学归纳法.\n当 $n=k+1$ 时, 可分别从 $A_1=\\left\\{1,2, \\cdots, \\frac{3^k+1}{2}\\right\\}$ 及 $A_2=\\left\\{3^k+1,3^k+2, \\cdots, 3^k+\\frac{3^k+1}{2}\\right\\}$ 中各取 $2^k$ 个数满足条件.", + "remark": "", + "figures": [] +} \ No newline at end of file diff --git a/processed_dataset/proof/0097.json b/processed_dataset/proof/0097.json new file mode 100644 index 0000000000000000000000000000000000000000..095da717d7f98462f3e6d961f46f41d9d4bb2585 --- /dev/null +++ b/processed_dataset/proof/0097.json @@ -0,0 +1,8 @@ +{ + "source_file": "./raw_volume-zh/volume1/exercise6.tex", + "problem_type": "proof", + "problem": "问题10 设 $a_j 、 b_j 、 c_j$ 为整数, 这里 $1 \\leqslant j \\leqslant N$, 且对任意的 $j$, 数 $a_j 、 b_j 、 c_j$ 中至少有一个为奇数.\n证明: 存在一组数 $r 、 s 、 t$, 使得集合 $ \\{r a_j+s b_j+t c_j \\mid 1 \\leqslant j \\leqslant N\\}$ 中, 至少有 $\\frac{4 N}{7}$ 个数为奇数.", + "solution": "考虑不全为零的 7 个数组 $(x, y, z)$, 其中 $x, y, z \\in\\{0,1\\}$. 容易证明: 若 $a_j 、 b_j 、 c_j$ 不全为偶数, 则集合 $A_j=\\{x a_j+y b_j+z c_j \\mid x, y, z \\in\\{0,1\\}\\}$ 中恰有 4 个为偶数, 也恰有 4 个为奇数, 这里 $1 \\leqslant j \\leqslant N$. 当然, 在 $x=y= z=0$ 时, $x a_j+y b_j+z c_j$ 为偶数.\n由此可知 $ \\{x a_j+y b_j+z c_j \\mid x, y, z \\in\\{0,1\\}, x 、 y, z$ 不全为零, $1 \\leqslant j \\leqslant N\\}$ 中, 恰有 $4 N$ 个数为奇数.\n于是, 由抽庶原则, 可知存在一组数 $(x, y, z), x, y, z \\in\\{0,1\\}, x 、 y 、 z$ 不全为零, 使得 $\\left\\{x a_j+y b_j+z c_j \\mid 1 \\leqslant j \\leqslant N\\right\\}$ 中至少有 $\\frac{4 N}{7}$ 个数为奇数.", + "remark": "", + "figures": [] +} \ No newline at end of file diff --git a/processed_dataset/proof/0098.json b/processed_dataset/proof/0098.json new file mode 100644 index 0000000000000000000000000000000000000000..99a063856d36a23e9488adf4c94d26602247ef97 --- /dev/null +++ b/processed_dataset/proof/0098.json @@ -0,0 +1,8 @@ +{ + "source_file": "./raw_volume-zh/volume1/exercise6.tex", + "problem_type": "proof", + "problem": "问题11 平面上不含零向量的集合 $A$, 若其至少有三个元素, 且对任意 $u \\in A$, 存在 $v, w \\in A$, 使 $v \\neq w, u=v+w$, 则称 $A$ 具有性质 $S$. 证明:\n(1) 对任意 $n \\geqslant 6$, 存在具有性质 $S$ 的向量集;\n(2) 具有性质 $S$ 的有限向量集合都至少有 6 个元素.", + "solution": "(1) 对 $n(n \\geqslant 6)$ 进行归纳.\n当 $n=6$ 时, 考虑 $\\triangle A B C$ 及 $A B 、 B C 、 C A$ 、 $B A 、 C B 、 A C$. 对于具有性质 $S$ 的 $n$ 元集合 $A$, 设其非零向量为 $\\boldsymbol{v}_1, \\boldsymbol{v}_2, \\cdots, v_n$. 设 $v_i 、 v_j$ 是 $A$ 的两个不同向量, $v_i$ 与 $v_j$ 的夹角是 $A$ 中各向量之间的最小角.\n则 $\\left(\\boldsymbol{v}_i+\\boldsymbol{v}_j\\right) \\notin A$, 否则与最小性矛盾.\n因此, $A \\cup\\left\\{\\boldsymbol{v}_i+\\boldsymbol{v}_j\\right\\}$ 有 $(n+1)$ 个元素, 且满足性质 $S$.\n(2) 考虑一个均由 $O$ 为始点的具有性质 $S$ 的向量集合 $A= \\{\\boldsymbol{O} \\boldsymbol{X}_1,\\boldsymbol{O} \\boldsymbol{X}_2, \\cdots, \\boldsymbol{O X _ { n }} \\}$, 若 $\\boldsymbol{u}$ 与 $\\boldsymbol{v}$ 不平行, 且使得 $\\boldsymbol{u}$ 或 $\\boldsymbol{v}$ 平行于 $A$ 中的一个向量或 $\\boldsymbol{X}_i \\boldsymbol{X}_j (i \\neq j)$ 中的一个向量.\n记 $\\boldsymbol{O} \\boldsymbol{X}_i=a_i \\boldsymbol{u}+b_i \\boldsymbol{v}$, 对向量 $\\boldsymbol{O} \\boldsymbol{X}_i$ 分解, $i=1,2, \\cdots, n$. 实数集合 $M=\\left\\{a_1, a_2, \\cdots, a_n\\right\\}$ 具有类似于 $S$ 的性质.\n设 $M$ 中的最大数为 $a$. 显然, $a>0$, 存在 $b 、 c>0$, 使得 $a=b+c, b \\neq c$. 否则, $a$ 不是 $M$ 中的最大元素.\n同理, 对于 $M$ 中的最小元素 $a^{\\prime}$, 存在 $b^{\\prime}, c^{\\prime} \\in M$, 且 $b^{\\prime} 、 c^{\\prime}<0, b^{\\prime} \\neq c^{\\prime}$, 使得 $a^{\\prime}=b^{\\prime}+c^{\\prime}$. 由此得出 $M$ 中的 6 个不同元素.", + "remark": "", + "figures": [] +} \ No newline at end of file diff --git a/processed_dataset/proof/0099.json b/processed_dataset/proof/0099.json new file mode 100644 index 0000000000000000000000000000000000000000..3546687bfbe3b0e55396acc2f628ed3f1ebe29e5 --- /dev/null +++ b/processed_dataset/proof/0099.json @@ -0,0 +1,12 @@ +{ + "source_file": "./raw_volume-zh/volume1/exercise6.tex", + "problem_type": "proof", + "problem": "问题12 平面上的点集 $H$ 称为是好的, 如果 $H$ 中任意 3 个点都存在一条对称轴, 使得这 3 个点关于这条对称轴对称.\n证明:\n(1) 一个好的集合不一定是轴对称的;\n(2) 如果一个好的集合中恰有 2003 个点,则这 2003 个点在一条直线上.", + "solution": "(1) 如图(), $\\triangle A B C 、 \\triangle A D C 、 \\triangle B C D$ 均为等腰三角形, $A 、 B 、 D$ 也共线.\n所以, 任意三个点皆有一条对称轴.\n故它是一个好的集合.\n但是 $A 、 B$ 、 $C 、 D$ 不是轴对称的.\n(2) 反证法.\n假设结论不成立.\n于是, 不可能有集合中的 6 个点共线.\n否则, 在这条直线外必有 1 个属于集合的点 $K$, 过点 $K$ 作此直线的垂线, 则此直线上必有至少 3 个点在这条垂线的同侧, 记为 $A 、 B 、 C$ (如图()). 因为 $\\angle K C B>\\frac{\\pi}{2}$, 所以, $B K>B C$. 由于 $K 、 C 、 B$ 有对称轴, 则 $B C=C K$. 同理, $A C=C K$,矛盾.\n故不可能有集合中的 6 个点共线.\n不妨设 $A 、 B$ 为这个集合中距离最短的两个点(如图()). 则其余 2001 个点有以下 4 种情况:(i) 在线段 $A B$ 中垂线上; (ii) 在 $A B$ 所在直线上; (iii) 在以 $A$ 为圆心、 $A B$ 长为半径的圆上; (iv) 在以 $B$ 为圆心、 $A B$ 长为半径的圆上.\n由前面的证明可知, (i)、(ii) 两种情况点的总数不超过 10 个.\n又因为 $A B$ 的距离最小, 所以 (iii)、(iv)两种情况点的总数不超过 10 个.\n故 $10+10+2<2003$. 矛盾.", + "remark": "", + "figures": [ + "./images/volume1/figures/fig-c6p12-1.png", + "./images/volume1/figures/fig-c6p12-2.png", + "./images/volume1/figures/fig-c6p12-3.png" + ] +} \ No newline at end of file diff --git a/processed_dataset/proof/0100.json b/processed_dataset/proof/0100.json new file mode 100644 index 0000000000000000000000000000000000000000..3846609c3fc016b4e726533b46ec2a8968ddcade --- /dev/null +++ b/processed_dataset/proof/0100.json @@ -0,0 +1,8 @@ +{ + "source_file": "./raw_volume-zh/volume1/exercise6.tex", + "problem_type": "proof", + "problem": "问题13 一个正整数的集合 $C$ 称为 “好集”, 是指对任何整数 $k$, 都存在着 $a, b \\in C$, $a \\neq b$, 使得数 $a+k$ 与 $b+k$ 不是互质的数.\n证明: 如果一个好集 $C$ 的元素之和为 2003 , 则存在一个 $c \\in C$, 使得集合 $C \\backslash\\{c\\}$ 仍是一个“好集”.", + "solution": "设 $p_1, p_2, \\cdots, p_n$ 是 $C$ 中两个数的差的所有可能的质因子.\n假定对每个 $p_i$, 都存在一个剩余 $\\alpha_i$, 使得 $C$ 中至多有一个数关于模 $p_i$ 与 $\\alpha_i$ 同余.\n利用中国剩余定理 (即孙子定理) 可得, 存在一个整数 $k$, 满足 $k \\equiv p_i-\\alpha_i(\\bmod p_i ), i=1,2, \\cdots, n$. 利用题中的条件可得, 存在某个 $j$ 和某个 $a, b \\in C$, 使得 $p_j$ 整除 $a+k$ 与 $b+k$. 于是, $a$ 和 $b$ 关于模 $p_j$ 与 $\\alpha_j$ 同余.\n这与 $\\alpha_j$ 的假定矛盾.\n由此可以断定关于模 $p$ 的每个剩余, 在 $C$ 的数的剩余中至少出现两次.\n假定每个剩余都恰好出现两次, 则 $C$ 中元素的和等于 $p r+2(0+1+\\cdots+ p-1)=p(r+p-1), r \\geqslant 1$, 这与 2003 是质数矛盾.\n因此, 一定存在某个剩余, 它至少出现三次.\n将具有这种性质的 $C$ 中的元素删除一个, 就得到了一一个新的“好集”.", + "remark": "", + "figures": [] +} \ No newline at end of file diff --git a/processed_dataset/proof/0101.json b/processed_dataset/proof/0101.json new file mode 100644 index 0000000000000000000000000000000000000000..673069aaef0cc6aab260eab1d2cac8096f43cfcc --- /dev/null +++ b/processed_dataset/proof/0101.json @@ -0,0 +1,8 @@ +{ + "source_file": "./raw_volume-zh/volume1/exercise6.tex", + "problem_type": "proof", + "problem": "问题14 试证: 存在一个具有如下性质的正整数的集合 $A$, 使对任何由无限多个素数组成的集合 $S$, 都存在自然数 $k \\geqslant 2, m \\in A$ 及 $n \\notin A, m$ 和 $n$ 均为 $S$ 中 $k$ 个不同元素的乘积.", + "solution": "设 $q_1, q_2, \\cdots, q_j, \\cdots$ 是全体素数从小到大排成的数列, 即有 $q_1=2, q_2=3, q_3=5, q_4=7, q_5=11, q_6=13, \\cdots$. 令 $A_1=\\left\\{2 q_i \\mid i=2,3, \\cdots\\right\\}$, $A_2=\\left\\{3 q_i q_j \\mid i) 所示的两图形之一,两图形显然都不是双邻集.\n4 个整点 $(p, q) 、(p+1, q) 、(p+1, q-1) 、(p, q-1)$ 恰组成一个双邻集 (边长为 1 的正方形), 则 $n=4$.\n注意到 10 个整点 $(p, q) 、(p+1, q) 、(p+2, q) 、(p+3, q) 、(p, q- 1) 、(p+3, q-1) 、(p, q-2) 、(p+1, q-2) 、(p+2, q-2) 、(p+3, q-2)$ 也组成一个双邻集 (长为 3 宽为 2 的矩形边上的 10 个整点).\n因此, 当 $n=4 k(k \\in \\mathbf{N})$ 时, 取 $k$ 个 4 整点组成的双邻集,每两个双邻集的距离 (一个相邻集中任一点到另一双邻集中任一点距离的最小值) 大于 1 , 将这 $k$ 个 4 整点双邻集合并为一个集合, 这个集合当然是恰含 $4 k$ 个整点的双邻集.\n当 $n=4 k+2(k \\geqslant 2)$ 时, 由于 $n=4(k-2)+10$, 取 $k-2$ 个 4 整点组成的双邻集, 取一个 10 整点组成的双邻集, 每两个双邻集的距离大于 1 . 将这 $k-2$ 个 4 整点双邻集与一个 10 整点双邻集合并为一个集合,这个集合当然是恰含 $4 k+2(k \\geqslant 2)$ 个整点的双邻集.", + "remark": "", + "figures": [ + "./images/volume1/figures/fig-c6p18.png" + ] +} \ No newline at end of file diff --git a/processed_dataset/proof/0105.json b/processed_dataset/proof/0105.json new file mode 100644 index 0000000000000000000000000000000000000000..0c6633891ac2ebe7c4e6b79c0230c71347b23280 --- /dev/null +++ b/processed_dataset/proof/0105.json @@ -0,0 +1,8 @@ +{ + "source_file": "./raw_volume-zh/volume1/exercise7.tex", + "problem_type": "proof", + "problem": "问题5 证明: 三角形三条高线的中点共线的充分必要条件是这个三角形是直角三角形.", + "solution": "在 $\\triangle A B C$ 中, 三条高线的中点 $A^{\\prime} 、 B^{\\prime} 、 C^{\\prime}$ 分别在中位线 $\\triangle M N P$ 三边所在直线上.\n若 $\\triangle A B C$ 是锐角三角形, 则点 $A^{\\prime} 、 B^{\\prime} 、 C^{\\prime}$ 在 $\\triangle M N P$ 的三边上, 故此三点不共线; 若 $\\triangle A B C$ 是钝角三角形, 则点 $A^{\\prime} 、 B^{\\prime} 、 C^{\\prime}$ 中有两点在 $\\triangle M N P$ 的两条边的延长线上, 另一点在第三条边上, 故此三点不共线; 若 $\\triangle A B C$ 是直角三角形, 则点 $A^{\\prime} 、 B^{\\prime} 、 C^{\\prime}$ 在直角所对的中位线上.", + "remark": "", + "figures": [] +} \ No newline at end of file diff --git a/processed_dataset/proof/0106.json b/processed_dataset/proof/0106.json new file mode 100644 index 0000000000000000000000000000000000000000..92b1ed051191a8a78d4e55fb3b4a07331ec415aa --- /dev/null +++ b/processed_dataset/proof/0106.json @@ -0,0 +1,8 @@ +{ + "source_file": "./raw_volume-zh/volume1/exercise7.tex", + "problem_type": "proof", + "problem": "问题6 正整数 $x 、 y$ 满足 $xP^3$ 成立, 这说明 $P$ 可取 $\\geqslant 2$ 的任意整数.\n当 $P<0$ 时, 由 $\\left|x^3-y\\right|=y-x^32 P_k$, 证明: $n$ 能整除 $(n-k)$ !.", + "solution": "因为 $n$ 是合数,故设 $n=a b(2 \\leqslant a \\leqslant b)$.\n若 $a \\geqslant 3$, (i) $a \\neq b$, 则 $n>2 p_k \\geqslant \\frac{3 k}{2}, b \\leqslant \\frac{n}{3}$. 从而, $k<\\frac{2 n}{3}$. 故 $n-k> \\frac{n}{3} \\geqslant b>a$. 所以, $n \\mid(n-k) !$. (ii) $a=b$, 则 $n=a^2, n-k>\\frac{n}{3}=\\frac{a^2}{3}$. 因为 $k \\geqslant 14$, 则 $p_k \\geqslant 13, n>26, a \\geqslant 6$. 从而, $\\frac{a^2}{3} \\geqslant 2 a$. 故 $n-k>2 a$. 所以, $n \\mid(n-k)$ !. 若 $a=2$, 因为 $n>26$, 假设 $b$ 不为质数, 则 $b=b_1 b_2\\left(b_1 \\leqslant b_2\\right)$. 因为 $b>13$, 则 $b_2 \\geqslant 4$. 于是, $a b_1 \\geqslant 4$ 归人 $a \\geqslant 3$ 的情况.\n不妨设 $b$ 为质数, 则 $b= \\frac{n}{2}>p_k$. 因为 $p_k$ 是小于 $k$ 的最大质数, 则 $b>k$. 从而, $n-k=2 b-k>b$. 所以, $n \\mid(n-k) !$.", + "remark": "", + "figures": [] +} \ No newline at end of file diff --git a/processed_dataset/proof/0109.json b/processed_dataset/proof/0109.json new file mode 100644 index 0000000000000000000000000000000000000000..1f1c71925c4f7babca17c88f0052b2530d956c23 --- /dev/null +++ b/processed_dataset/proof/0109.json @@ -0,0 +1,8 @@ +{ + "source_file": "./raw_volume-zh/volume1/exercise7.tex", + "problem_type": "proof", + "problem": "问题11 设正整数 $n$ 具有如下性质: 在从 $\\{1,2, \\cdots, 1988\\}$ 中任取的 $n$ 个数中, 总有 29 个数组成一个等差数列.\n求证: $n>1788$.", + "solution": "首先从 $\\{1,2, \\cdots, 1988\\}$ 中删除 29 的所有倍数, 共 68 个.\n将余下的 1920 个数分成 69 个集合: $A_k=\\{1+29 k, 2+29 k, \\cdots, 28+29 k\\}, k=0,1,2, \\cdots, 67, A_{69}=\\{1973,1974, \\cdots, 1988\\}$. 由于 29 为素数,故当 $(d, 29)=1 $时,公差为 $d$ 的等差数列 $a, a+d, \\cdots, a+28 d$ 中的 29 个数模 29 互不同余, 其中必有 29 的倍数.\n由于这样的数已被删除, 故在剩下的数中不存在与 29 互素的公差 $d$ 的 29 项等差数列.\n下面再考察以 29 的倍数即以 29 或 58 为公差的 29 项等差数列的情形.\n删去集合 $A_{28} 、 A_{57}$ 中的所有数,共删去 56 个数.\n由于公差为 29 的等差数列的 29 项必分别属于 $A_0, A_1, \\cdots, A_{68}$ 中相继的 29 个集合,公差为 58 的等差数列的 29 项则分别属于 $A_0, A_1, \\cdots, A_{68}$ 中相间的 29 个集合, 故两者均必有某项属于 $A_{28}$ 或 $A_{57}$. 从而在删除 $A_{28} 、 A_{57}$ 的所有数之后, 即不存在任何 29 项的等差数列.\n易见, 两次共删除了 $68+2 \\times 28=124<200$, 所以余下的数多于 1788 . 这就证明了只有 $n>1788$, 才可能具备题中所述的性质.", + "remark": "", + "figures": [] +} \ No newline at end of file diff --git a/processed_dataset/proof/0110.json b/processed_dataset/proof/0110.json new file mode 100644 index 0000000000000000000000000000000000000000..b777c34cd9bc18bae62698ff86fdb16e29ce7be0 --- /dev/null +++ b/processed_dataset/proof/0110.json @@ -0,0 +1,8 @@ +{ + "source_file": "./raw_volume-zh/volume1/exercise7.tex", + "problem_type": "proof", + "problem": "问题12 某国学生参加城市联赛, 试卷由 6 题组成, 每题恰有 1000 个人做出来, 若找不到两个人, 使任何一题至少被两个人中的一个答出, 试求参加比赛的人数的最小值.", + "solution": "(1) 首先证明 2000 个人参加比赛是可以的, 定义三元数组 $(i, j, k)$ 表示答对第 $i, j, k$ 题 $(1 \\leqslant i, j, k \\leqslant 6)$. 考虑 10 个三元数组 $(1,2,3),(3,5,6),(1,2,5),(3,4,5),(1,3,4),(2,4,6),(2,3,6),(1,4,6),(1,5,6),(2,4,5)$ 满足: (1) 任两个数恰出现 5 次; (2) 每个数恰出现 5 次.\n将每个三元数组对应于 200 个人的答题情况, 则可知满足题目所有条件恰有 2000 人.\n(2) 证明不能少于 2000 人.\n设答对题最多的人为 $A$, 设 $A$ 答对 $k$ 题.\n(1) $k=6$, 则 $A$ 全部答对, 与条件矛盾!(2) $k=5$, 不妨设 $A$ 答对 $1,2,3,4,5$, 则由题知存在 $B$ 答对第 6 题, 则 $A$ 与 $B$ 答对所有题, 矛盾! (3) $k=4$, 不妨设 $A$ 答对 $1,2,3,4$, 则不存在 $B$ 既答对 5, 又答对 6 , 又因为答对 5,6 的共 2000 人, 再加上 $A$, 至少有 2001 人! (4) $k=3$, 则每人至多答对 3 题, 而每题有 1000 人答对, 所以至少有 $\\frac{6 \\times 1000}{3}=2000$ (人). 所以 2000 人为所求最小值.", + "remark": "", + "figures": [] +} \ No newline at end of file diff --git a/processed_dataset/proof/0111.json b/processed_dataset/proof/0111.json new file mode 100644 index 0000000000000000000000000000000000000000..8f0c45d7a5ffef20443a89269bbe93e7e1a9e3cf --- /dev/null +++ b/processed_dataset/proof/0111.json @@ -0,0 +1,8 @@ +{ + "source_file": "./raw_volume-zh/volume1/exercise7.tex", + "problem_type": "proof", + "problem": "问题13 将一些相同的正 $n$ 边形餐门纸放在桌子上, 允许任两张餐巾纸有可能有部分重叠.\n设任两张餐门纸可经过平移将一张移到和另一张重叠.\n当 $n=6$ 时, 是否总可以在桌上钉一些钉子, 使得每张餐巾纸恰好被钉了一次?", + "solution": "回答是可以做到的.\n由 $n=6$, 餐巾纸为正六边形,边按逆时针方向定向.\n由于任两张餐巾纸可经过平移而重叠.\n所以任两餐巾纸的六条边按相同定向互相平行.\n因此可以在平面上作出大小和餐巾纸一样的正六边形网格, 网格中每个正六边形和任一餐巾纸的六条边按相同定向互相平行.\n所有餐巾纸的中心可构成两个集合,一个集合由这样的中心构成, 这些中心都在网络线上, 另一个集合由不在网格线上的中心构成.\n前者记作 $N$, 后者记作 $M$.\n(i)设 $N=\\varnothing$, 即所有餐巾纸的中心都不落在网格线上.\n我们在网格的每个正六边形的中心上钉钉子.\n这些钉子要么不钉在任一餐巾纸上, 要么只钉上一次.\n因为若有一张餐巾纸上被钉了两个钉子, 那么餐巾纸的中心落在网格的两个不同的正六边形内,这是不可能的.\n(ii)设 $N \\neq \\varnothing$. 由于餐巾纸只有有限张, 所以 $M$ 为有限集.\n因此记 $d>0$, $d$ 为 $M$ 的中心和网格线的最近距离.\n我们将网格平移小于 $d$ 的距离.\n于是, $M$ 中的点仍不在网格线上.\n由于 $N$ 也为有限集, 所以我们可以选取这种平移的方向, 使得 $N$ 中的点也都不在新的网格线上.\n于是对新的网格线, 化为情形 (i). 这证明了命题成立.", + "remark": "", + "figures": [] +} \ No newline at end of file diff --git a/processed_dataset/proof/0112.json b/processed_dataset/proof/0112.json new file mode 100644 index 0000000000000000000000000000000000000000..b3a4e8111067ddf11a4dfbfb8c7d6ce2ed63d5d5 --- /dev/null +++ b/processed_dataset/proof/0112.json @@ -0,0 +1,8 @@ +{ + "source_file": "./raw_volume-zh/volume1/exercise7.tex", + "problem_type": "proof", + "problem": "问题14 平面上按如下方式给出一个螺旋放置的正方形系列: 最初是两个 $1 \\times 1$ 正方形, 这两个正方形有一条坚直的公共边, 并排水平放置; 第三个为 $2 \\times 2$ 正方形, 紧贴着放置在前两个正方形上方, 有一条边为前两个正方形各一条边之并; 第四个为 $3 \\times 3$ 正方形, 紧贴着放置在第一个和第三个正方形的左方, 有一条边为那两个正方形各一条边之并; 第五个为 $5 \\times 5$ 正方形, 紧贴着放在第一、第二和第四个正方形的下方, 有一条边为那三个正方形各一条边之并; 第六个为 $8 \\times 8$ 正方形, 紧贴着放置在第二、第三、 第五个正方形的右方……每一个新的正方形与已拼成的矩形有一条公共边,该公共边上含有上一个正方形的一条边.\n试证: 除了第一个正方形以外, 所有这些正方形的中心统统都在两条固定直线上.", + "solution": "设所作的正方形依次为 $S_0, S_1, S_2, \\cdots$, 这些正方形的中心依次为 $\\left(x_0, y_0\\right),\\left(x_1, y_1\\right),\\left(x_2, y_2\\right), \\cdots$, 为确定起见, 设最初 4 个正方形的中心为 $\\left(\\frac{1}{2}, \\frac{1}{2}\\right),\\left(\\frac{3}{2}, \\frac{1}{2}\\right),(1,2),\\left(-\\frac{3}{2}, \\frac{3}{2}\\right)$. 于是 $\\frac{y_2-y_0}{x_2-x_0}=3, \\frac{y_3-y_1}{x_3-x_1}= -\\frac{1}{3}$. 正方形 $S_0, S_1, S_2, \\cdots$ 的边长正好是菲波那契数列 $f_0, f_1, f_2, \\cdots$. 注意到 $f_n=f_{n-1}+f_{n-2}=2 f_{n-2}+f_{n-3}=3 f_{n-3}+2 f_{n-4}$. 以下将分情形利用正方形的边长计算 $x_n-x_{n-4}$ 和 $y_n-y_{n-4}$, 从而算出 $k_n=\\frac{y_n-y_{n-4}}{x_n-x_{n-4}}$.\n情形 $1 \\quad n \\equiv 0(\\bmod 4) \\cdot x_n-x_{n-4}=-\\frac{1}{2}\\left(f_{n-3}+f_{n-4}\\right), y_n-y_{n-1}= -\\frac{3}{2}\\left(f_{n-3}+f_{n-4}\\right), k_n=3$.\n情形 $2 \\quad n \\equiv 1(\\bmod 4): x_n-x_{n-4}=\\frac{3}{2}\\left(f_{n-3}+f_{n-4}\\right), y_n-y_{n-4}= -\\frac{1}{2}\\left(f_{n-3}+f_{n-4}\\right), k_n=-\\frac{1}{3}$.\n情形 $3 n \\equiv 2(\\bmod 4) . x_n-x_{n-4}=\\frac{1}{2}\\left(f_{n-3}+f_{n-4}\\right), y_n-y_{n-1}=\\frac{3}{2}\\left(f_{n-3}+ f_{n-4}\\right), k_n=3$.\n情形 $4 \\quad n \\equiv 3(\\bmod 4) . x_n-x_{n-4}=-\\frac{3}{2}\\left(f_{n-3}+f_{n-4}\\right), y_n-y_{n-4}= \\frac{1}{2}\\left(f_{n-3}+f_{n-4}\\right), k_n=-\\frac{1}{3}$.\n统观各种情形, 可以判定: 对于偶数 $n$ 有 $k_n=3$, 所有偶数编号正方形的中心全在过 $\\left(x_0, y_0\\right)$ 点且斜率为 3 的直线上; 对于奇数 $n$ 有 $k_n=-\\frac{1}{3}$, 所有奇数编号正方形的中心全在过 $\\left(x_1, y_1\\right)$ 点且斜率为 $-\\frac{1}{3}$ 的直线上.", + "remark": "", + "figures": [] +} \ No newline at end of file diff --git a/processed_dataset/proof/0113.json b/processed_dataset/proof/0113.json new file mode 100644 index 0000000000000000000000000000000000000000..f9b61bd87701f4723bd6b73b194827e30f87e0f6 --- /dev/null +++ b/processed_dataset/proof/0113.json @@ -0,0 +1,8 @@ +{ + "source_file": "./raw_volume-zh/volume1/exercise7.tex", + "problem_type": "proof", + "problem": "问题15 桌子上放着两堆重量和相等的硬币,第一堆硬币的个数是 $n$, 第二堆硬币的个数是 $m, S=\\min \\{n, m\\}$. 对于任意的不大于 $S$ 的自然数 $k$, 按硬币重量自大至小的顺序, 第一堆前 $k$ 个较重的硬币的重量和都不大于第二堆中前 $k$ 个较重的硬币的重量和.\n证明: 对于任意正数 $x$, 如果把两堆中每一个重量不小于 $x$ 的硬币的重量都按 $x$ 计算, 那么, 这样算出来的第一堆硬币的重量和都不小于第二堆硬币的重量和.", + "solution": "把第一堆 $n$ 枚硬币的重量依次表示为 $x_1 \\geqslant x_2 \\geqslant \\cdots \\geqslant x_n$, 把第二堆 $m$ 枚硬币的重量依次表示为 $y_1 \\geqslant y_2 \\geqslant \\cdots \\geqslant y_m$. 又设 $x_1 \\geqslant \\cdots \\geqslant x_s \\geqslant x \\geqslant x_{s+1} \\geqslant \\cdots \\geqslant x_n, y_1 \\geqslant \\cdots \\geqslant y_t \\geqslant x \\geqslant y_{t+1} \\geqslant \\cdots \\geqslant y_m$. (如果没有不轻于 $x$ 的硬币, 则结论显然成立.\n) 这样, 要证明的是: $x_s+x_{s+1}+\\cdots+x_n \\geqslant x_t+ y_{t+1}+\\cdots+y_m$. 设 $x_1+x_2+\\cdots+x_n=y_1+y_2+\\cdots+y_m=A$, 即证 $x_s+[A- \\left(x_1+\\cdots+x_s\\right) ] \\geqslant x_t+\\left[A-\\left(y_1+\\cdots+y_t\\right)\\right]$, 即证 $x_1+\\cdots+x_s+x(t-s) \\leqslant y_1+\\cdots+y_t$. 下面分两种情况:\n若 $t \\geqslant s$, 则 $x_1+\\cdots+x_s+x(t-s) \\leqslant\\left(y_1+\\cdots+y_s\\right)+\\left(y_{s+1}+\\cdots+y_t\\right)$. (因为 $x_1+\\cdots+x_s \\leqslant y_1+\\cdots+y_s$ 可由已知推出, 而且 $y_{s+1} \\geqslant x, \\cdots, y_t \\geqslant x$.)\n若 $ty$ ), $k>1, n$ 是自然数.\n显然, $x 、 y$ 中的任何一个都不能被 3 整除.\n如果 $k$ 是偶数, 则 $x^k$ 和 $y^k$ 被 3 除的余数都是 1 . 这样, $x^k$ 与 $y^k$ 的和除以 3 的余数是 2 , 而不是 3 的整数次幂.\n于是, 推出矛盾,所以 $k$ 不是偶数.\n如果 $k$ 是奇数且 $k>1$, 则 $3^n=(x+y)\\left(x^{k-1}-\\cdots+y^{k-1}\\right)$. 这样, $x+y= 3^m, m \\geqslant 1$.\n以下证明: $n \\geqslant 2 m$. 因为 $k$ 可被 3 整除, 取 $x_1=x^{\\frac{k}{3}}, y_1=y^{\\frac{k}{3}}$ 代入后, 可以认为 $k=3$. 这样, $x^3+y^3=3^n, x+y=3^m$. 要证明 $n \\geqslant 2 m$, 只要证明 $x^3+ y^3 \\geqslant(x+y)^2$, 即证明 $x^2-x y+y^2 \\geqslant x+y$. 由于 $x \\geqslant y+1$, 则 $x^2-x= x(x-1) \\geqslant x y$. $\\left(x^2-x-x y\\right)+\\left(y^2-y\\right) \\geqslant 0$. 不等式 $n \\geqslant 2 m$ 得证.\n由恒等式 $(x+y)^3-\\left(x^3+y^3\\right)=3 x y(x+y)$ 推出: $(*) 3^{2 m-1}-3^{n-m-1}= x y$, 而 $2 m-1 \\geqslant 1$, 且 $(* *) n-m-1 \\geqslant n-2 m \\geqslant 0$. 因此, 如果 (**) 中至少有一个不等号是严格不等号, 那么 $(*)$ 式中的左端可被 3 整除, 但右端不能被 3 整除, 推出矛盾.\n如果 $n-m-1=n-2 m=0$, 那么, $m=1, n=2$ 且 $3^2=2^3+1^3$. 故 $n=2$.", + "remark": "", + "figures": [] +} \ No newline at end of file diff --git a/processed_dataset/proof/0115.json b/processed_dataset/proof/0115.json new file mode 100644 index 0000000000000000000000000000000000000000..43a6ec8994569d2adcc26e553fbb8447c8752e37 --- /dev/null +++ b/processed_dataset/proof/0115.json @@ -0,0 +1,8 @@ +{ + "source_file": "./raw_volume-zh/volume1/exercise7.tex", + "problem_type": "proof", + "problem": "问题17 证明: 可以用 4 种颜色对正整数 $1,2, \\cdots, 2000$ 染色, 使它不含有由 7 个同色数组成的等差数列.", + "solution": "问题等价于把集合 $S=\\{1,2, \\cdots, 2000\\}$ 分拆成 4 个非空子集 $M_1 、 M_2 、 M_3 、 M_4$, 使得 $M_i \\cap M_j=\\varnothing(i \\neq j), M_1 \\cup M_2 \\cup M_3 \\cup M_4=S$.\n因为 $6 \\times 7^3>2000$, 所以 $S$ 中的每个数都可以表示成至多 4 位的 7 进制数 $(a b c d)_7$, 这里 $a, b, c, d \\in\\{0,1,2, \\cdots, 6\\}$. 设 $A_i= \\{(a b c d)_7 \\mid(a b c d)_7 \\in S, b \\neq i, c \\neq i, d \\neq i\\}, i=1,2,3,4$. 对任意 $x \\in S$, 由于每个 7 进制正整数末 3 位数上至少有 $1,2,3,4$ 中的一个数字末出现, 例如 $x$ 的末 3 位数中末出现 4 , 则 $x \\in A_4$, 所以, $A_1 \\cup A_2 \\cup A_3 \\cup A_4=S$.\n下证: 集合 $A_i(i=1,2,3,4)$ 中不含由 7 项构成的等差数列.\n反设某个$A_i$ 中含有由 7 项构成的等差数列: $a, a+d, \\cdots, a+6 d$. 若 $7 \\nmid d$, 则上述 7 个数模 7 两两不同余 (即构成一个模 7 的完系), 从而, 这 7 个数中必有一个, 它除以 7 的余数为 $i$, 即它的 7 进制表示中的末位数为 $i$, 矛盾.\n若 $7 \\mid d, 7^2 \\nmid d$, 仿上可得到这个等差数列中必有一项, 它的 7 进制表示中从右数的第二位数字为 $i$, 矛盾.\n若 $7^2 \\mid d, 7^3 \\nmid d$, 同理可得这个等差数列中必有一项, 它的 7 进制表示中从右数的第三位数字为 $i$, 矛盾.\n若 $7^3 \\mid d$, 则 $6 d \\geqslant 6 \\times 7^3>2000$, 矛盾.\n最后, 令 $M_1=A_1, M_2=A_2 \\cap \\bar{A}_1, M_3=A_3 \\cap \\bar{A}_1 \\cap \\bar{A}_2, M_4=A_4 \\cap \\bar{A}_1 \\cap \\bar{A}_2 \\cap \\bar{A}_3$, 得到符合要求的分拆.", + "remark": "", + "figures": [] +} \ No newline at end of file diff --git a/processed_dataset/proof/0116.json b/processed_dataset/proof/0116.json new file mode 100644 index 0000000000000000000000000000000000000000..d71d223be5adfcecc9954b2f0bb976caa40282f3 --- /dev/null +++ b/processed_dataset/proof/0116.json @@ -0,0 +1,8 @@ +{ + "source_file": "./raw_volume-zh/volume1/exercise7.tex", + "problem_type": "proof", + "problem": "问题18 一个正整数无穷等差数列, 包含一项是整数的平方, 另一项是整数的立方.\n证明: 此数列含有一项是整数的六次幂.", + "solution": "设数列 $\\{a+i h: i=0,1,2, \\cdots\\}$ 含 $x^2 、 y^3$ 项, $x 、 y$ 是整数.\n对公差 $h$ 用数学归纳法.\n$h=1$, 显然成立.\n对某个固定的 $h>1$, 假设其公差小于 $h$ 且满足题设条件的等差数列都成立.\n现考察在 $h$ 时的情形.\n令 $a 、 h$ 的最大公约数为 $d=(a, h), h=d e$. 分两种情况:\n情形 $1 \\quad(d, e)=1$. 易知 $x^2 \\equiv a \\equiv y^3(\\bmod h)$, 因而有 $x^2 \\equiv a \\equiv y^3(\\bmod e) . e$ 与 $a$ 互素, 故 $e$ 与 $x$ 和 $y$ 也互素.\n所以, 有整数 $t$, 使得 $t y \\equiv x(\\bmod e)$. 因此, $(t y)^6 \\equiv x^6(\\bmod e)$, 即 $t^6 a^2 \\equiv a^3(\\bmod e)$. 因 $(e, a)=1$, 故两端可除以 $a^2$ 有 $t^6 \\equiv a(\\bmod e)$. 又 $(d, e)=1$, 则对某个整数 $k$, 有 $t+k e \\equiv 0(\\bmod d)$. 于是, $(t+k e)^6 \\equiv 0 \\equiv a(\\bmod d)$. 因 $t^6 \\equiv a(\\bmod e)$, 由二项式公式, 可得 $(t+k e)^6 \\equiv a(\\bmod e)$. 又 $(d, e)=1, h=d e$, 由以上两同余式, 有 $(t+k e)^6 \\equiv a (\\bmod h)$. 显然, $k$ 可取任意大的整数, 故上式说明数列 $\\{a+i h \\mid i=0,1,2, \\cdots\\}$ 含一个整数的六次幂项.\n情形 $2(d, e)>1$. 令素数 $p, p|d, p| e$, 并设 $p^\\alpha$ 是整除 $a$ 的 $p$ 的最高次幂, $p^\\beta$ 是整除 $h$ 的 $p$ 的最高次幂.\n因 $h=d e,(e, a)=1$, 有 $\\beta>\\alpha \\geqslant 1$. 因而对 $\\{a+i h \\mid i=0,1, \\cdots\\}$ 中每一项, 能整除它的最高次幕是 $p^\\alpha$. 因 $x^2 、 y^3$ 是数列的两个项, $\\alpha$ 必被 2 和 3 整除, 故 $\\alpha=6 r$. 因此, $\\alpha \\geqslant 6$. 整数数列 $\\{p^{-6}(a+ i h) \\mid i=0,1,2, \\cdots\\}$ 的公差 $\\frac{h}{p^6}1$ 为自然数, 试证不能在 $k \\times k$ 的方格表中填人数 $1,2, \\cdots, k^2$, 使得每行和每列数之和都是 2 的方幕.", + "solution": "若不然, 设将 $1,2, \\cdots, k^2$ 填人表格后最小的行和是 $2^a$, 则有 $2^a \\geqslant 1+2+\\cdots+k=\\frac{1}{2} k(k+1)$. 因为表中所有数之和为 $1+2+\\cdots+k^2= \\frac{1}{2} k^2\\left(k^2+1\\right)$, 故应有 $2^a \\mid \\frac{1}{2} k^2\\left(k^2+1\\right)$. 当 $k$ 为奇数时, $\\frac{1}{2} k^2\\left(k^2+1\\right)$ 亦为奇数, 当然不能被 $2^a$ 整除; 当 $k$ 为偶数时, $k^2+1$ 为奇数, 于是应有 $2^a \\mid \\frac{1}{2} k^2$. 但这时又有 $\\frac{1}{2} k^2<\\frac{1}{2} k(k+1) \\leqslant 2^a$, 矛盾.", + "remark": "", + "figures": [] +} \ No newline at end of file diff --git a/processed_dataset/proof/0119.json b/processed_dataset/proof/0119.json new file mode 100644 index 0000000000000000000000000000000000000000..5d8926c0d8eac1222d7466692fe1e1e389e3bad7 --- /dev/null +++ b/processed_dataset/proof/0119.json @@ -0,0 +1,8 @@ +{ + "source_file": "./raw_volume-zh/volume1/exercise8.tex", + "problem_type": "proof", + "problem": "问题3 设 $S$ 是一个非空点集, 它的所有点都是整点.\n此外, 还给定一组有限多个有整数坐标的非零向量组.\n已知当将向量组中的所有向量的起点都放在 $S$ 中的任一点时, 它们的终点中属于 $S$ 的比不属于 $S$ 的多.\n求证: $S$ 必为无穷点集.", + "solution": "设 $S$ 为有限点集, 于是其中必有两点 $A$ 和 $B$, 使 $A$ 的纵坐标是所有点的纵坐标中最大的, 且在纵坐标同为最大的所有点中, $A$ 的横坐标最大; $B$ 的纵坐标是所有点的纵坐标中最小的, 且在纵坐标同为最小的所有点中, $B$ 的横坐标最小.\n首先把给定的所有向量的起点都放在点 $A$, 按已知, 满足 $y>0$ 和 $y=0 、 x>0$ 的向量少于半数.\n然后再把所有向量都放在点 $B$, 又知 $y<0$ 和 $y=0 、 x<0$ 的向量也少于半数,矛盾.", + "remark": "", + "figures": [] +} \ No newline at end of file diff --git a/processed_dataset/proof/0120.json b/processed_dataset/proof/0120.json new file mode 100644 index 0000000000000000000000000000000000000000..7105c7d3bb15ba38998191eedef751d94d96492e --- /dev/null +++ b/processed_dataset/proof/0120.json @@ -0,0 +1,8 @@ +{ + "source_file": "./raw_volume-zh/volume1/exercise8.tex", + "problem_type": "proof", + "problem": "问题4 一次 10 名选手参加的循环赛中无平局, 胜者得 1 分,负者得 0 分.\n证明: 各选手得分的平方和不超过 285 .", + "solution": "由于得分的情况仅有有限多种, 其中必有一种的平方和取最大值.\n这时各选手的得分 $p_1, p_2, \\cdots, p_{10}$ 必互不相同, 因为若 $p_i=p_j$, 则改变选手 $i$ 与 $j$ 之间的胜负, 即用 $p_i-1 、 p_j+1$ 来代替 $p_i 、 p_j$ 时, 由于 $\\left(p_i-1\\right)^2+ \\left(p_j+1\\right)^2-\\left(p_i^2+p_j^2\\right)=2>0$, 而平方和中其他项不变, 故平方和严格增大, 这与平方和已取得最大值矛盾.\n于是, 在 $p_i=i-1(i=1,2, \\cdots, 10)$ 时, $\\sum_{i=1}^{10} p_i^2$ 最大, 这时的值 $\\sum_{i=0}^9 i^2=285$.", + "remark": "", + "figures": [] +} \ No newline at end of file diff --git a/processed_dataset/proof/0121.json b/processed_dataset/proof/0121.json new file mode 100644 index 0000000000000000000000000000000000000000..e2e98981d91e2f5ce24e657fa79dab8bdcf308fd --- /dev/null +++ b/processed_dataset/proof/0121.json @@ -0,0 +1,8 @@ +{ + "source_file": "./raw_volume-zh/volume1/exercise8.tex", + "problem_type": "proof", + "problem": "问题5 设 $n$ 为大于 1 的整数, 全部正因数为 $d_1, d_2, \\cdots, d_k$, 其中 $1=d_12, D>d_{k-1} d_k=\\frac{n^2}{p}$. 如果 $D \\mid n^2$, 则 $\\frac{n^2}{D}$ 为 $n^2$ 的因子, 但 $1<\\frac{n^2}{D}\\frac{1+b}{2}$, 由于 $\\frac{a}{2} \\leqslant \\frac{1+b}{2}$ 且 $x_t=b \\leqslant \\frac{1+b}{2}$, 所以有以下两种情况:\n(i) $x_1=b, x_2>\\frac{1+b}{2}, \\cdots, x_n>\\frac{1+b}{2}$. 令 $x_m=\\min \\left\\{x_2, \\cdots, x_n\\right\\}$, 其中 $2 \\leqslant m \\leqslant n$, 显然 $x_{m-1}\\left(1-x_m\\right) \\geqslant x_1\\left(1-x_n\\right) \\geqslant \\frac{1}{4} x_1\\left(1-x_n\\right)$.\n(ii) 存在 $2 \\leqslant i \\leqslant n-1$, 使得 $x_i>\\frac{1+b}{2}, x_{i+1} \\leqslant \\frac{1+b}{2}$, 于是$x_i\\left(1-x_{i+1}\\right) \\geqslant \\frac{1+b}{2}\\left(1-\\frac{1+b}{2}\\right) \\geqslant \\frac{a}{4}(1-b) \\geqslant \\frac{1}{4} x_1\\left(1-x_n\\right)$.", + "remark": "", + "figures": [] +} \ No newline at end of file diff --git a/processed_dataset/proof/0123.json b/processed_dataset/proof/0123.json new file mode 100644 index 0000000000000000000000000000000000000000..5ea75322d9a96f5614cd7b43b8d52483d5d89452 --- /dev/null +++ b/processed_dataset/proof/0123.json @@ -0,0 +1,8 @@ +{ + "source_file": "./raw_volume-zh/volume1/exercise8.tex", + "problem_type": "proof", + "problem": "问题7 已知集合 $M$ 的元素都是整数,既有正整数又有负整数,且当 $a, b \\in M$ 时, $2 a$ 和 $a+b$ 也属于 $M$. 求证: 当 $a, b \\in M$ 时, $a-b \\in M$.", + "solution": "首先, 用归纳法容易证明, 若 $c \\in M$, 则对任意 $n \\in Z$, 都有 $n c \\in M$.\n设 $a>0$ 是集合 $M$ 中的最小正整数, $b<0$ 是 $M$ 中的最大负整数, 即绝对值最小的负整数.\n按已知, $a+b \\in M$, 且满足不等式 $b)). 由于 $R_1 B_2+R_2 B_1<\\left(R_1 P+P B_2\\right)+\\left(R_2 P+P B_1\\right)= R_1 B_1+R_2 B_2$, 所以, 当我们将 $R_1$ 与 $B_2$ 配对, $R_2$ 与 $B_1$ 配对, 其他的保持不变时, $n$ 条线段的长度和就减少了, 矛盾.\n因此, 这时候 $n$ 条线段是互不相交的.", + "remark": "", + "figures": [ + "./images/volume1/figures/fig-c8p11.png" + ] +} \ No newline at end of file diff --git a/processed_dataset/proof/0127.json b/processed_dataset/proof/0127.json new file mode 100644 index 0000000000000000000000000000000000000000..b1a40b39e7c641a3a25bc6e2d9291924aeafaf47 --- /dev/null +++ b/processed_dataset/proof/0127.json @@ -0,0 +1,10 @@ +{ + "source_file": "./raw_volume-zh/volume1/exercise8.tex", + "problem_type": "proof", + "problem": "问题12 平面上有 $n$ 个点,其中任意三点不共线,且任意三点构成的三角形的面积都小于 1 . 证明: 存在一个面积小于 4 的三角形包含这 $n$ 个点.", + "solution": "取 $n$ 个点中任意三点作一个三角形, 三角形的个数是有限的, 每一个三角形都有一个面积, 取其中面积最大的一个记为 $\\triangle A_1 A_2 A_3$. 由于每个三角形的面积都小于 1 , 所以 $S_{\\triangle A_1 A_2 A_3}<1$. 过顶点 $A_1 、 A_2 、 A_3$ 分别作对边的平行线, 得到一个 $\\triangle A B C$, 如图()所示.\n显然 $S_{\\triangle A B C}=4 S_{\\triangle A_1 A_2 A_3}<4$.\n下面证明 $\\triangle A B C$ 包含了这 $n$ 个点.\n用反证法.\n设 $\\triangle A B C$ 外还有这 $n$ 个点中的一点, 设为 $A_4$, 则$S_{\\triangle A_4 A_3 A_1}>S_{\\triangle A_2 A_3 A_1}$, 这与$\\triangle A_1 A_2 A_3$最大矛盾.\n于是 $\\triangle A B C$ 即为所求.", + "remark": "", + "figures": [ + "./images/volume1/figures/fig-c8p12.png" + ] +} \ No newline at end of file diff --git a/processed_dataset/proof/0128.json b/processed_dataset/proof/0128.json new file mode 100644 index 0000000000000000000000000000000000000000..98687c7552bef12bb56148c3cbf461cd607614c0 --- /dev/null +++ b/processed_dataset/proof/0128.json @@ -0,0 +1,8 @@ +{ + "source_file": "./raw_volume-zh/volume1/exercise8.tex", + "problem_type": "proof", + "problem": "问题14 设有 $n$ 个人 $A_1, A_2, \\cdots, A_n$, 其中有些人相互认识.\n证明: 可用适当方式把他们分成两组,使每人都至少有一半熟人不跟他在同一组.", + "solution": "设 $A_i(i=1,2, \\cdots, n)$ 有 $c_i$ 个熟人, 其中有 $d_i$ 个不与 $A_i$ 同组.\n这里$d_i$ 是随分组变化而变化的.\n本题相当于证明: 存在一个适当的分组法, 使得对一切 $i=1,2, \\cdots, n$, 有 $d_i \\geqslant \\frac{1}{2} c_i$.\n由于总人数只有有限多个, 分组方法也只有有限多种, 从而和 $d_1+ d_2+\\cdots+d_n$ 也只有有限多个不同的值.\n于是, 必存在某种分组法, 使上面的和取得最大值, 记这个最大值为 $d$. 下面证明: 使 $d_1+d_2+\\cdots+d_n$ 最大的分组方法符合要求.\n否则, 对这种分组法存在某个人, 不妨设为甲组的 $A_1$, 他在乙组的熟人数 $d_1<\\frac{1}{2} c_1$. 于是, $A_1$ 在甲组中的熟人数为 $c_1-d_1$. 现把 $A_1$ 从甲组调人乙组, 其余的人不动.\n对这个重新分组, $d_2, d_3, \\cdots, d_n$ 都末变, 这时, $A_1$ 在甲组的熟人数 $c_1-d_1$ 变为与他不同组的熟人数, 从而 $d_1$ 变为 $c_1-d_1$. 这时有 $\\left(c_1-d_1\\right)+d_2+\\cdots+d_n=c_1-d_1+d-d_1=d+\\left(c_1-2 d_1\\right)>d$, 这与 $d$ 是最大值矛盾.", + "remark": "", + "figures": [] +} \ No newline at end of file diff --git a/processed_dataset/proof/0129.json b/processed_dataset/proof/0129.json new file mode 100644 index 0000000000000000000000000000000000000000..42d20c4998a8b1c2ff4377a7707f74cd36b6c451 --- /dev/null +++ b/processed_dataset/proof/0129.json @@ -0,0 +1,8 @@ +{ + "source_file": "./raw_volume-zh/volume1/exercise8.tex", + "problem_type": "proof", + "problem": "问题15 平面上有若干个圆,它们所盖住的面积为 1 . 证明: 一定可以从这些圆中去掉一部分圆,使得余下的圆互不相交,且它们所覆盖的面积不小于 $\\frac{1}{9}$.", + "solution": "显然应尽可能地保留些大圆而去掉小圆, 为此, 将这些圆适当 “排序”. 设这若干个圆中最大的一个是 $\\odot O_1$, 其半径为 $r_1$, 则与 $\\odot O_1$ 相交的所有圆必落在以 $O_1$ 为圆心, $3 r_1$ 为半径的圆内.\n因此, $\\odot O_1$ 的面积不小于这组圆所覆盖面积的 $\\frac{1}{9}$. 去掉与 $\\odot O_1$ 相交的所有圆, 余下的圆与 $\\odot O_1$ 不相交, 再设这些圆中除 $\\odot O_1$ 外最大的一个是 $\\odot O_2$, 仿上讨论知 $\\odot O_2$ 的面积不小于所有与 $\\odot O_2$ 相交的一组圆所覆盖面积的 $\\frac{1}{9}$. 去掉与 $\\odot O_2$ 相交的所有圆, …... 如此继续, 直到 $\\odot O_1, \\odot O_2, \\cdots, \\odot O_n$ 它们彼此都不相交, 且面积都不小于与自己相交的那一组圆面积的 $\\frac{1}{9}$ (它们中的某几个也可能一开始就不与任何一个圆相交而被保留下来). 所以, 它们所覆盖的面积不小于总覆盖面积的 $\\frac{1}{9}$.", + "remark": "", + "figures": [] +} \ No newline at end of file diff --git a/processed_dataset/proof/0130.json b/processed_dataset/proof/0130.json new file mode 100644 index 0000000000000000000000000000000000000000..ec93165eed37ceb51389866c0526b5e57219a6a7 --- /dev/null +++ b/processed_dataset/proof/0130.json @@ -0,0 +1,8 @@ +{ + "source_file": "./raw_volume-zh/volume1/exercise8.tex", + "problem_type": "proof", + "problem": "问题16 证明: 不存在整数 $x 、 y 、 z$, 满足\n$$\n2 x^4+2 x^2 y^2+y^4=z^2, x \\neq 0 .\n$$", + "solution": "因为 $x \\neq 0$, 显然有 $y \\neq 0$. 不失一般性, 假定 $x 、 y$ 是题设方程的整数解, 且满足 $x>0, y>0$, 及 $(x, y)=1$, 我们还可以进一步假定 $x$ 是满足上述条件的最小的整数解.\n由于 $z^2 \\equiv 0,1,4(\\bmod 8)$, 可知 $x$ 是偶数, 而 $y$ 是奇数.\n注意到 $x^4+ \\left(x^2+y^2\\right)^2=z^2$, 及 $\\left(x^2, x^2+y^2\\right)=1$, 故存在一个奇整数 $p$ 和偶整数 $q$, 使得 $x^2=2 p q, x^2+y^2=p^2-q^2$ 及 $(p, q)=1$. 由此易证, 存在一个整数 $a$ 与奇数 $b$, 使得 $p=b^2, q=2 a^2$. 故 $x=2 a b, y^2=b^4-4 a^4-4 a^2 b^2$.\n注意到 $\\left(\\frac{2 a^2+b^2+y}{2}\\right)^2+\\left(\\frac{2 a^2+b^2-y}{2}\\right)^2=b^4$ 及 $\\left(\\frac{2 a^2+b^2+y}{2} \\frac{2 a^2+b^2-y}{2}\\right)=1$. 故存在整数 $s 、 t$, 其中 $s>t,(s, t)=1$, 使得 $\\frac{2 a^2+b^2+y}{2}= 2 s t, \\frac{2 a^2+b^2-y}{2}=s^2-t^2$, 或 $\\frac{2 a^2+b^2+y}{2}=s^2-t^2, \\frac{2 a^2+b^2-y}{2}=2 s t$, 及 $b^2=s^2+t^2$. 易知 $a^2=(s-t) t$.\n由于 $(a, b)=1,(s, t)=1$, 故存在正整数 $m 、 n((m, n)=1)$, 使得 $(s-t)=m^2, t=n^2$. 因此, $b^2=n^4+\\left(n^2+m^2\\right)^2$. 而 $x=2 a b>t=n^2 \\geqslant n$, 这与 $x$ 是最小解的假定矛盾.", + "remark": "", + "figures": [] +} \ No newline at end of file diff --git a/processed_dataset/proof/0131.json b/processed_dataset/proof/0131.json new file mode 100644 index 0000000000000000000000000000000000000000..9596a1667823579d979d794e5ff1440ee8cd3811 --- /dev/null +++ b/processed_dataset/proof/0131.json @@ -0,0 +1,8 @@ +{ + "source_file": "./raw_volume-zh/volume1/exercise8.tex", + "problem_type": "proof", + "problem": "问题17 设 $x_1, x_2, \\cdots, x_n$ 都是非负实数, $a$ 是它们中的最小值, 记 $x_{n+1}= x_1$. 求证:\n$$\n\\sum_{j=1}^n \\frac{1+x_j}{1+x_{j+1}} \\leqslant n+\\frac{1}{(1+a)^2} \\sum_{j=1}^n\\left(x_j-a\\right)^2 .\n$$\n其中等号成立当且仅当 $x_1=x_2=\\cdots=x_n$.", + "solution": "用归纳法.\n当 $n=1$ 时要证的不等式显然成立.\n设当 $n=k$ 时结论成立.\n当 $n=k+1$ 时, 由轮换对称性, 不妨设 $x_{k+1}$ 最大.\n于是由归纳假设可得\n$$\n\\sum_{j=1}^{k-1} \\frac{1+x_j}{1+x_{j+1}}+\\frac{1+x_k}{1+x_1} \\leqslant k+\\frac{1}{(1+a)^2} \\sum_{j=1}^k\\left(x_j-a\\right)^2 .\n$$\n由(1)可知, 为证原不等式, 只需证\n$$\n\\frac{1+x_k}{1+x_{k+1}}+\\frac{1+x_{k+1}}{1+x_1}-\\frac{1+x_k}{1+x_1} \\leqslant 1+\\frac{1}{(1+a)^2}\\left(x_{k+1}-a\\right)^2 .\n$$\n即\n$$\n\\frac{\\left(x_{k+1}-x_k\\right)\\left(x_{k+1}-x_1\\right)}{\\left(1+x_{k+1}\\right)\\left(1+x_1\\right)} \\leqslant \\frac{1}{(1+a)^2}\\left(x_{k+1}-a\\right)^2 .\n$$\n由于 $a=\\min \\left\\{x_1, x_2, \\cdots, x_{k+1}\\right\\}, x_{k+1}=\\max \\left\\{x_1, x_2, \\cdots, x_{k+1}\\right\\}$, 显然(2) 成立.\n这就证明了当 $n=k+1$ 时要证的不等式成立.\n而且为使 $n=k+1$ 时要证的不等式中的等号成立, 当且仅当(1)和(2)中的等号成立.\n由归纳假设 (1)中等号成立的充要条件是 $a=x_1=x_2=\\cdots=x_k$. 从而(2)中等号成立的充要条件是 $x_{k+1}=a$. 故当 $n=k+1$ 时, 原不等式中等号成立当且仅当 $x_1= x_2=\\cdots=x_{k+1}$.", + "remark": "", + "figures": [] +} \ No newline at end of file diff --git a/processed_dataset/proof/0132.json b/processed_dataset/proof/0132.json new file mode 100644 index 0000000000000000000000000000000000000000..b02987f720cd3f73ab74a25b71d8815ccf61a2dc --- /dev/null +++ b/processed_dataset/proof/0132.json @@ -0,0 +1,8 @@ +{ + "source_file": "./raw_volume-zh/volume1/exercise8.tex", + "problem_type": "proof", + "problem": "问题18 设 $S=\\{-(2 n-1),-(2 n-2), \\cdots,-1,0,1,2, \\cdots, 2 n-1\\}$, 求证: $S$ 的任一个 $2 n+1$ 元的子集中必有 3 个数之和为零.", + "solution": "若不然, 则存在 $S$ 的一个 $2 n+1$ 元的子集 $H$, 其中任何 3 个数之和都不为零.\n(1) 首先证明, $0 \\notin H$. 如果 $0 \\in H$, 则其余的 $2 n$ 个整数分属于如下 $2 n-1$ 个数对: $(-i, i), i=1,2, \\cdots, 2 n-1$. 由抽屈原理知其中必有一对的两个数都属于 $H$. 二者加上 0,3 数之和为零, 矛盾.\n(2) 设 $H$ 中绝对值最小的元素为 $d$, 不妨设 $d>0$. 令 $H^{+}=\\{x \\mid x \\in H, x>d\\}, H^{-}=\\{x \\mid x \\in H, x<-d\\}, H^{+-}=\\left\\{d-x \\mid x \\in H^{+}\\right\\}, H^{-+}= \\left\\{-d-x \\mid x \\in H^{-}\\right\\}$. 显然, 这些集都不是空集且由反证假设知 $H^{+} \\cap H^{-+}= \\varnothing$. (i)\n若 $-d \\notin H$, 则由 (i) 有 $2 n-1 \\geqslant\\left|H^{+} \\cup H^{-+}\\right|=\\left|H^{+}\\right|+\\left|H^{-+}\\right|=2 n$, 矛盾.\n故必有 $-d \\in H$. 于是 $H^{-} \\cap H^{+-}=\\varnothing$ 及 $\\left|H^{+}\\right|+\\left|H^{-+}\\right|=\\left|H^{+-}\\right|+ \\left|H^{-}\\right|=2 n-1$. 故有 $H^{+} \\cup H^{-+}=\\{1,2, \\cdots, 2 n-1\\}, H^{-} \\cup H^{+-}=\\{-1,-2, \\cdots,-(2 n-1)\\}$. (ii)\n将 $H^{+} 、 H^{-}$中各数之和分别记为 $h^{+}$和 $h^{-}$, 则 $H^{+-}$和 $H^{-+}$中各数之和分别为 $\\left|H^{+}\\right| \\cdot d-h^{+}$和 $-\\left|H^{-}\\right| \\cdot d-h^{-}$. 于是由 (ii) 便得 $h^{+}+h^{-}+\\left|H^{+}\\right| \\cdot d-h^{+}-\\left|H^{-}\\right| \\cdot d-h^{-}=0,\\left(\\left|H^{+}\\right|-\\left|H^{-}\\right|\\right) d=0$. 因 $\\left|H^{+}\\right|+\\left|H^{-}\\right|=2 n-1$, 故 $\\left|H^{+}\\right|-\\left|H^{-}\\right|$为奇数, 故得 $d=0$, 矛盾.", + "remark": "", + "figures": [] +} \ No newline at end of file diff --git a/processed_dataset/proof/0133.json b/processed_dataset/proof/0133.json new file mode 100644 index 0000000000000000000000000000000000000000..517ef7540ea7bfa4a0d867f04cf1b9be1edfbd43 --- /dev/null +++ b/processed_dataset/proof/0133.json @@ -0,0 +1,8 @@ +{ + "source_file": "./raw_volume-zh/volume1/exercise8.tex", + "problem_type": "proof", + "problem": "问题19 某市有 $n$ 所中学, 第 $i$ 所中学派出 $c_i$ 名学生到体育馆观看球赛.\n已知 $0 \\leqslant c_i \\leqslant 39, i=1,2, \\cdots, n, c_1+c_2+\\cdots+c_n=1990$, 看台的每一横排有 199 个座位.\n要求同一学校的学生必须坐在同一横排, 问体育馆最少要安排多少横排才能保证全部学生都能按要求人座?", + "solution": "由于 $c_i \\leqslant 39$, 故每一横排至少可坐 160 人.\n于是只要有 13 排, 至少可坐 $160 \\times 13=2080$ 人, 当然能坐下全部 1990 名学生.\n下面证明只要安排 12 个横排就够了.\n由于 $c_1, c_2, \\cdots, c_n$ 只有有限多个, 故它们的不超过 199 的有限和也只有有限多个.\n选取其中最接近 199 的有限和, 记为 $c_{i_1}+c_{i_2}+\\cdots+c_{i_k}$, 将这 $k$ 个学校的学生安排在第一排就坐.\n然后再对其余的诸 $c_i$ 人进行同样的讨论并选取不超过 199 且最接近 199 的有限和, 并把相应的学校的学生排在第二排.\n依此类推,一直排到第十排并记第 $i$ 排的空位数为 $x_i, i=1,2, \\cdots, 12$.\n如果 $x_{10} \\geqslant 33$, 则余下的未就坐的学校的学生数 $c_i$ 全都不小于 34 . 若余下的学校数不多于 4 个, 则只要 11 排就够了.\n若余下的学校数不少于 5 个, 则可任取 5 个学校的学生安排在第 11 排.\n这时有 $x_{11} \\leqslant 291$, 使得 $\\frac{2^n+1}{n^2}$ 是整数.", + "solution": "显然数 $n$ 为正奇数, 于是我们只需考虑 $n \\geqslant 3$ 且 $n$ 为奇数的情况.\n设 $p$ 是 $n$ 的最小素因数, 则 $p \\geqslant 3, p \\mid 2^n+1$. 令 $i$ 是使 $p \\mid 2^i+1$ 成立的最小正整数, 我们将证明 $1 \\leqslant i0$, 可令 $i=r_1+b, 1 \\leqslant bk+m-\\frac{k}{2} \\geqslant m+1$. 若 $\\beta \\geqslant m+2$, 则 $3^{m+2} \\mid 3^k \\mathrm{C}_n^k$. 注意到 $m \\geqslant 2$, 所以有 $2 m \\geqslant m+2$. 于是由 (i) 知, $3^{m+2} \\mid 3 n$. 进而 $3^{m+1} \\mid n$. 这与 $(c, 3)=1$ 矛盾.\n从而证明了 $m=1$, 即 $n=3 c$, $(c, 3)=1$.\n设 $c>1$, 而 $q$ 是 $c$ 的最小素因数, 显然有 $q \\geqslant 5$, 且 $q \\mid 2^n$. 类似地, 令 $j$ 为使 $q \\mid 2^j+1$ 成立的最小正整数, 则必有 $1 \\leqslant j0$ 知, 有数学家不妨设为 $v_3$ 与 $v_1, v_2$ 都合作过.\n又 $\\left|A_1 \\cap A_2 \\cap A_3\\right|=\\left|A_1 \\cap A_2\\right|+\\left|A_3\\right|-\\left|\\left(A_1 \\cap A_2\\right) \\cup A_3\\right| \\geqslant 3 \\times 1327- 2 \\times 1990=1$, 故存在数学家, 不妨设为 $v_4 \\in A_1 \\cap A_2 \\cap A_3$, 即 $v_4$ 与 $v_1 、 v_2$ 、 $v_3$ 都合作过.\n从而有数学家 $v_1 、 v_2 、 v_3 、 v_4$ 两两合作过.", + "remark": "", + "figures": [] +} \ No newline at end of file diff --git a/processed_dataset/proof/0136.json b/processed_dataset/proof/0136.json new file mode 100644 index 0000000000000000000000000000000000000000..2f2566995231e9e89d11adb342e95803e3301141 --- /dev/null +++ b/processed_dataset/proof/0136.json @@ -0,0 +1,8 @@ +{ + "source_file": "./raw_volume-zh/volume1/exercise9.tex", + "problem_type": "proof", + "problem": "问题14 证明: 任意 28 个介于 104 与 208 之间 (包括 104 和 208) 的不同的正整数,其中必有两个数不互质.", + "solution": "设 $I=\\{x \\mid 104 \\leqslant x \\leqslant 208, x \\in \\mathbf{N}\\}$, 并记 $E_k=\\{x \\mid x \\equiv 0(\\bmod k)$, $x \\in I\\}, k=2,3,5,7$. 由容斥原理得 $\\left|E_2 \\cup E_3 \\cup E_5 \\cup E_7\\right|=(53+35+ 21+15)-(17+10+7+7+5+3)+(3+2+1+1)-0=82$. 这说明 $[104$, $208]$ 中不能被 $2,3,5,7$ 任何一个整除的整数共有 $(208-103)-82=23$ 个.\n于是任意取 104 和 208 之间的 28 个数, 至少有 5 个数属于 $E_2 \\cup E_3 \\cup E_5 \\cup E_7$. 根据抽席原则知, 必有两个数属于某个 $E_k(k=2,3,5,7)$, 故结论成立.", + "remark": "", + "figures": [] +} \ No newline at end of file diff --git a/processed_dataset/proof/0137.json b/processed_dataset/proof/0137.json new file mode 100644 index 0000000000000000000000000000000000000000..2966955a462cdb8d68ad70b65a7588e601c9ba8c --- /dev/null +++ b/processed_dataset/proof/0137.json @@ -0,0 +1,8 @@ +{ + "source_file": "./raw_volume-zh/volume1/exercise9.tex", + "problem_type": "proof", + "problem": "问题16 空间中有 $2 m$ 个点, $m \\geqslant 2$, 其中任意四点不共面.\n证明: 如果这 $2 m$ 个点之间至少连有 $m^2+1$ 条线段,则所连的线段中至少有三条, 它们围成一个三角形.", + "solution": "对 $m$ 用归纳法.\n当 $m=2$ 时, 容易验证.\n假设对 $m$ 结论成立.\n对 $m+$ 1 , 空间有 $2 m+2$ 个点, $(m+1)^2+1$ 条连线.\n这 $2 m+2$ 个点中必有两点 $a$ 与 $b$, 它们之间有线段相连.\n其余 $2 m$ 个点的集合记为 $X$. 如果 $X$ 中 $2 m$ 个点之间至少连有 $m^2+1$ 条线段, 则结论成立.\n因此, 设 $X$ 中 $2 m$ 个点之间至多连有 $m^2$ 条线段.\n于是 $X$ 中的点与点 $a$ 或点 $b$ 所连线段至少有 $(m+1)^2+1-m^2-1= 2 m+1$ 条.\n记 $X$ 中与点 $a$ 有线段相连的点的集合为 $A$, 与点 $b$ 有线段相连的点的集合为 $B$. 则 $|A \\cup B| \\leqslant|X| \\leqslant 2 m$, 而且 $|A|+|B| \\geqslant 2 m+1$. 因此有 $2 m \\geqslant|A \\cup B|=|A|+|B|-|A \\cap B| \\geqslant 2 m+1-|A \\cap B|$, 由此得 $|A \\cap B| \\geqslant 1$. 于是必有点 $C \\in A \\cap B$. 点 $a$ 与 $b 、 c$ 之间两两均有线段相连.", + "remark": "", + "figures": [] +} \ No newline at end of file diff --git a/processed_dataset/proof/0138.json b/processed_dataset/proof/0138.json new file mode 100644 index 0000000000000000000000000000000000000000..843cf6d274d053176b7137b50fa455adecbc6f9a --- /dev/null +++ b/processed_dataset/proof/0138.json @@ -0,0 +1,8 @@ +{ + "source_file": "./raw_volume-zh/volume1/exercise9.tex", + "problem_type": "proof", + "problem": "问题18 由复数构成的有限集合 $A$ 满足: 对任意正整数 $n$, 若 $z \\in A$, 则 $z^n \\in A$. 证明:\n(1) $\\sum_{z \\in A} z$ 是整数;\n(2) 对任意整数 $k$, 可以找到一个集合 $A$, 使得 $A$ 满足条件且 $\\sum_{z \\in A} z=k$.", + "solution": "(1) 定义有限集合 $X$ 的元素和为 $S(X)$. 设 $0 \\neq z \\in A$. 因为 $A$ 是有限集合,故存在正整数 $m\\left|\\bigcap_{i=1}^4 A_i\\right|=3$, 同理, $\\left|A_1 \\cap A_3 \\cap A_4\\right|>$ 3, $\\left|A_1 \\cap A_2 \\cap A_4\\right|>3$, 所以 $\\left|A_1 \\cap A_2 \\cap A_3\\right|+\\left|A_1 \\cap A_3 \\cap A_4\\right|+\\mid A_1 \\cap A_2 \\cap A_4|-| \\bigcap_{i=1}^4 A_i \\mid>6$. 注意到 $\\left|A_1 \\cap \\bar{A}_2 \\cap \\bar{A}_3 \\cap \\bar{A}_4\\right|=\\mid A_1 \\cap \\left(\\overline{A_2 \\cup A_3 \\cup A_4}\\right)|=| A_1 \\cup A_2 \\cup A_3 \\cup A_4|-| A_2 \\cup A_3 \\cup A_4 \\mid= \\left(\\sum_{i=1}^4\\left|A_i\\right|-\\sum_{1 \\leqslant i235-59-29-15+6=138$, 可见 $\\left|A_1 \\cap \\bar{A}_2 \\cap \\bar{A}_3 \\cap \\bar{A}_4\\right| \\geqslant 139=3 \\times 46+1>3 \\times 46$. 故由抽屉原理知, 存在一个国家, 该国派出的选手中至少有 4 人做对了且只做对了第一题.", + "remark": "", + "figures": [] +} \ No newline at end of file diff --git a/processed_dataset/proof/0140.json b/processed_dataset/proof/0140.json new file mode 100644 index 0000000000000000000000000000000000000000..4aaf577e981abaf09203eece2ddbfdb36e01e9e6 --- /dev/null +++ b/processed_dataset/proof/0140.json @@ -0,0 +1,8 @@ +{ + "source_file": "./raw_volume-zh/volume10/chapter1.tex", + "problem_type": "proof", + "problem": "例1. 证明: $1 \\underbrace{0 \\cdots 0}_{200 \\text { 个0 }} 1$ 被 1001 整除.", + "solution": "证明:我们有\n$$\n\\begin{aligned}\n\\underbrace{10 \\cdots 01}_{200 \\text { 个 } 0} & =10^{201}+1=\\left(10^3\\right)^{67}+1 \\\\\n& =\\left(10^3+1\\right)\\left[\\left(10^3\\right)^{66}-\\left(10^3\\right)^{65}+\\cdots-10^3+1\\right],\n\\end{aligned}\n$$\n所以, $10^3+1(=1001)$ 整除 $1 \\underbrace{0 \\cdots 0}_{200 \\text { 个 } 0} 1$.", + "remark": "", + "figures": [] +} \ No newline at end of file diff --git a/processed_dataset/proof/0141.json b/processed_dataset/proof/0141.json new file mode 100644 index 0000000000000000000000000000000000000000..6c2fd7e51f06c184cfbc5311f7887f14b6106a26 --- /dev/null +++ b/processed_dataset/proof/0141.json @@ -0,0 +1,8 @@ +{ + "source_file": "./raw_volume-zh/volume10/chapter1.tex", + "problem_type": "proof", + "problem": "例2. 设 $m>n \\geqslant 0$, 证明 : $\\left(2^{2^n}+1\\right) \\mid\\left(2^{2^m}-1\\right)$.", + "solution": "证明:取 $x=2^{2^{n+1}}, y=1$, 并以 $2^{m-n-1}$ 代替那里的 $n$, 得出\n$$\n2^{2^m}-1=\\left(2^{2^{n+1}}-1\\right)\\left[\\left(2^{2^{n+1}}\\right)^{2^{m-n-1}-1}+\\cdots+2^{2^{n+1}}+1\\right],\n$$\n故\n$$\n\\left(2^{2^{n+1}}-1\\right) \\mid\\left(2^{2^m}-1\\right) \\text {. }\n$$\n又\n$$\n\\begin{gathered}\n2^{2^{n+1}}-1=\\left(2^{2^n}-1\\right)\\left(2^{2^n}+1\\right), \\\\\n\\left(2^{2^n}+1\\right) \\mid\\left(2^{2^{n+1}}-1\\right) .\n\\end{gathered}\n$$\n从而于是由整除性质知 $\\left(2^{2^n}+1\\right) \\mid\\left(2^{2^m}-1\\right)$.", + "remark": "注:整除问题中,有时直接证明 $b \\mid a$ 不易人手,我们可以尝试着选择适当的\"中间量\" $c$, 来证明 $b \\mid c$ 及 $c \\mid a$, 由此及整除性质(1), 便导出了结论.", + "figures": [] +} \ No newline at end of file diff --git a/processed_dataset/proof/0142.json b/processed_dataset/proof/0142.json new file mode 100644 index 0000000000000000000000000000000000000000..2b4f997e1aab0d52b3dc0e5bfc13f431b241174b --- /dev/null +++ b/processed_dataset/proof/0142.json @@ -0,0 +1,8 @@ +{ + "source_file": "./raw_volume-zh/volume10/chapter1.tex", + "problem_type": "proof", + "problem": "例3. 对正整数 $n$, 记 $S(n)$ 为 $n$ 的十进制表示中数码之和.\n证明: $9 \\mid n$ 的充分必要条件是 $9 \\mid S(n)$.", + "solution": "证明:设 $n=a_k \\times 10^k+\\cdots+a_1 \\times 10+a_0$ (这里 $0 \\leqslant a_i \\leqslant 9$, 且 $a_k \\neq 0$ ), 则 $S(n)=a_0+a_1+\\cdots+a_k$. 我们有\n$$\nn-S(n)=a_k\\left(10^k-1\\right)+\\cdots+a_1(10-1) . \\label{eq1}\n$$\n对 $1 \\leqslant i \\leqslant k$, 知 $9 \\mid\\left(10^i-1\\right)$, 故\\ref{eq1}式右端 $k$ 个加项中的每个都是 9 的倍数, 从而由整除性质 (2) 知, 它们的和也被 9 整除, 即 $9 \\mid(n- S(n)$ ). 由此易推出结论的两个方面.", + "remark": "注1 整除性质 (2) 提供了证明 $b \\mid\\left(a_1+a_2+\\cdots+a_n\\right)$ 的一种基本的想法,我们可尝试着证明更强的 (也往往是更易于证明的)命题:\n$b$ 整除每个 $a_i(i=1,2, \\cdots, n)$.\n这一更强的命题当然并非一定成立, 即使在它不成立时, 上述想法仍有一种常常奏效的变通: 将和 $a_1+a_2+\\cdots+a_n$ 适当地分组成为 $c_1+c_2+\\cdots+c_k$,而 $b \\mid c_i(i=1,2, \\cdots, k)$. 读者将看到, 为了证明 $b \\mid a$, 我们有时针对具体问题将 $a$ 表示为适当数之和,以应用上述想法论证.\n注2 例 3 的证明, 实际上给出了更强的结论: 对任意正整数 $n$, 数 $n$ 与 $S(n)$ 之差总是 9 的倍数.\n由此易知, $n$ 与 $S(n)$ 被 9 除得的余数相同 (这可表述为 $n$ 与 $S(n)$ 模 9 同余, 请看第 6 单元).\n注3 有些情形,我们能够由正整数十进制表示中的数码 (字) 的性质, 推断这整数能否被另一个整数整除, 这样的结论, 常称为 \"整除的数字特征\". 被 $2 、 5 、 10$ 整除的数的数字特征是显而易见的.", + "figures": [] +} \ No newline at end of file diff --git a/processed_dataset/proof/0143.json b/processed_dataset/proof/0143.json new file mode 100644 index 0000000000000000000000000000000000000000..b544f34ba0b95bd319cfb52dd1955b4762067f53 --- /dev/null +++ b/processed_dataset/proof/0143.json @@ -0,0 +1,8 @@ +{ + "source_file": "./raw_volume-zh/volume10/chapter1.tex", + "problem_type": "proof", + "problem": "例4. 设 $k \\geqslant 1$ 是一个奇数, 证明: 对任意正整数 $n$, 数 $1^k+2^k+\\cdots+n^k$ 不能被 $n+2$ 整除.", + "solution": "证明:$n=1$ 时结论显然成立.\n设 $n \\geqslant 2$, 记所说的和为 $A$, 则\n$$\n2 A=2+\\left(2^k+n^k\\right)+\\left(3^k+(n-1)^k\\right)+\\cdots+\\left(n^k+2^k\\right) .\n$$\n因 $k$ 是正奇数, 故由分解式 (6) 可知, 对每个 $i \\geqslant 2$, 数 $i^k+(n+2-i)^k$ 被 $i+(n+2-i)=n+2$ 整除, 故 $2 A$ 被 $n+2$ 除得的余数是 2 , 从而 $A$ 不可能被 $n+2$ 整除(注意 $n+2>2$ ).", + "remark": "注:论证中我们应用了\"配对法\", 这是代数中变形和式的一种常用手法.", + "figures": [] +} \ No newline at end of file diff --git a/processed_dataset/proof/0144.json b/processed_dataset/proof/0144.json new file mode 100644 index 0000000000000000000000000000000000000000..98576b25ed18445368c029be23a9dbe42fd85e33 --- /dev/null +++ b/processed_dataset/proof/0144.json @@ -0,0 +1,8 @@ +{ + "source_file": "./raw_volume-zh/volume10/chapter1.tex", + "problem_type": "proof", + "problem": "例5. 设 $m 、 n$ 为正整数, $m>2$, 证明: $\\left(2^m-1\\right) \\nmid\\left(2^n+1\\right)$.", + "solution": "证明:首先, 当 $n \\leqslant m$ 时, 易知结论成立.\n事实上, $m=n$ 时, 结论平凡; $n2$, 并参看整除性质 (3)).\n最后, $n>m$ 的情形可化为上述特殊情形: 由带余除法, $n=m q+r, 0 \\leqslant r0$. 由于\n$$\n2^n+1=\\left(2^{m q}-1\\right) 2^r+2^r+1,\n$$\n由分解式 (5) 知 $\\left(2^m-1\\right) \\mid\\left(2^{m q}-1\\right)$; 而 $0 \\leqslant rm$ 时也有 $\\left(2^m-1\\right) \\nmid\\left(2^n+1\\right)$. 这就证明了本题结论.\n我们顺便提一下,例 5 中的条件 $m>2$ 是必要的.\n因为当 $m=2$ 时, $2^m- 1=3$, 而由 (6) 知, 对于所有奇数 $n \\geqslant 1$, 数 $2^n+1$ 均被 3 整除.", + "remark": "", + "figures": [] +} \ No newline at end of file diff --git a/processed_dataset/proof/0145.json b/processed_dataset/proof/0145.json new file mode 100644 index 0000000000000000000000000000000000000000..3b2d3ee92049e5e731ed4f94f8a63b8f0c99f316 --- /dev/null +++ b/processed_dataset/proof/0145.json @@ -0,0 +1,8 @@ +{ + "source_file": "./raw_volume-zh/volume10/chapter1.tex", + "problem_type": "proof", + "problem": "例6. 任给 $n>1$, 证明: 有正整数 $a$, 使得 $a^a+1, a^{a^a}+1, \\cdots$ 中所有数均被 $n$ 整除.", + "solution": "解:我们注意, 若 $a$ 是奇数, 则 $a^a, a^{a^a}, \\cdots$ 均是奇数, 从而由 (6) 知, $a^a+ 1, a^{a^a}+1=a^{\\left(a^a\\right)}+1, \\cdots$ 均有因子 $a+1$. 因此取 $a=2 n-1$ 则符合问题中的要求.", + "remark": "", + "figures": [] +} \ No newline at end of file diff --git a/processed_dataset/proof/0146.json b/processed_dataset/proof/0146.json new file mode 100644 index 0000000000000000000000000000000000000000..c60871d645357ef78fa0a9bea479bc7b0009759a --- /dev/null +++ b/processed_dataset/proof/0146.json @@ -0,0 +1,8 @@ +{ + "source_file": "./raw_volume-zh/volume10/chapter1.tex", + "problem_type": "proof", + "problem": "例7. 任给 $n \\geqslant 2$, 证明: 存在 $n$ 个互不相同的正整数,其中任意两个的和, 整除这 $n$ 个数的积.", + "solution": "解:我们任取 $n$ 个互不相同的正整数 $a_1, \\cdots, a_n$, 并选取一个 (正整数) 参数 $K$, 希望 $K a_1, \\cdots, K a_n$ 的积 $K^n a_1 \\cdots a_n$ 被任意两项的和 $K a_i+K a_j$ 整除 ( $1 \\leqslant i, j \\leqslant n, i \\neq j)$. 由于 $n \\geqslant 2$, 显然, 取\n$$\nK=\\prod_{1 \\leqslant in^n-1$, 即为\n$$\np^{a n}>n \\log _b n .\n$$\n在 $n$ 很大时, 上式左边为 $n$ 的 (底大于 1 的)指数函数, 当然大于右边的一次函数与对数函数的积.\n(若记 $p^a=1+\\beta, \\beta>1$, 并用二项式定理将 $(1+\\beta)^n$ 展开, 则所说的事情便一目了然.)", + "solution": "证明:可设 $u>1$. 结论等价于证明方程\n$$\nn !=u^r\\left(u^s-1\\right)\n$$\n至多只有有限组正整数解 $(n, r, s)$.\n首先注意, 给定 $n$, 方程 式\\ref{eq1}显然至多有有限组解 $(r, s)$. 下面证明, 当 $n$ 充分大时,方程式\\ref{eq1}无解,由此便证明了上述的结论.\n取定一个素数 $p \\nmid u$. 可假定 式\\ref{eq1} 有解 $n>p$ (否则已无需证明), 并设 $p^\\alpha \\| n !$, 则有\n$$\n\\alpha=\\sum_{l=1}^{\\infty}\\left[\\frac{n}{p^l}\\right] \\geqslant\\left[\\frac{n}{p}\\right]>a n, \\label{eq2}\n$$\n其中 $a$ 是一个仅与 $p$ 有关的 (正) 常数.\n设 $u$ 模 $p$ 的阶为 $d$ 以及 $p^{k_0} \\|\\left(u^d-1\\right)$, 则由第 8 单元例 5 知, 当 $\\alpha>k_0$ 时, $u$ 模 $p^\\alpha$ 的阶为 $d p^{\\alpha-k_0}$. 因 $u 、 p$ 均为固定的数, 故 $k_0 、 d$ 也均为固定的数.\n若 式\\ref{eq1} 对充分大的 $n$ 有解, 则由 式\\ref{eq2} 知 $\\alpha>k_0$. 而由式\\ref{eq1}得\n$$\nu^s \\equiv 1\\left(\\bmod p^\\alpha\\right) \\text {, }\n$$\n故由阶的性质推出 $d p^{\\alpha-k_0} \\mid s$; 特别地, $s \\geqslant d p^{\\alpha-k_0}$. 因此,\n$$\nu^s-1 \\geqslant u^{d p^{\\alpha-k_0}}-1>u^{d p^{a n-k_0}}-1 .\\label{eq3}\n$$\n但当 $n$ 充分大时, 易知上式右边 $\\geqslant n^n-1$. 故由 式\\ref{eq3} 推出 $u^s-1>n$ !, 更有 $u^r\\left(u^s-1\\right)>n$ !, 因此当 $n$ 充分大时, 式\\ref{eq1}无正整数解 $(r, s)$. 这就完成了证明.", + "remark": "", + "figures": [] +} \ No newline at end of file diff --git a/processed_dataset/proof/0148.json b/processed_dataset/proof/0148.json new file mode 100644 index 0000000000000000000000000000000000000000..aba9f22e669a52006264cb3d219ae097c63fe60b --- /dev/null +++ b/processed_dataset/proof/0148.json @@ -0,0 +1,8 @@ +{ + "source_file": "./raw_volume-zh/volume10/chapter10.tex", + "problem_type": "proof", + "problem": "例2. 求所有整数 $n>1$, 使得 $\\frac{2^n+1}{n^2}$ 是整数.", + "solution": "解:容易猜想, $n=3$ 是唯一符合要求的解.\n下面证明事实确实如此.\n证明需分几步进行.\n第一个要点是考虑 $n$ 的最小素因子 $p$, 并由 $n \\mid\\left(2^n+1\\right)$ 导出 $p=3$, 见第 8 单元例 1 . 因此我们可设\n$$\nn=3^m c, m \\geqslant 1,3 \\nmid c . \\label{eq1}\n$$\n第二步, 证明 $m=1$. 由 $n^2 \\mid\\left(2^n+1\\right)$ 得 $2^{3^m c} \\equiv-1\\left(\\bmod 3^{2 m}\\right)$, 故\n$$\n2^{2 \\times 3^m c} \\equiv 1\\left(\\bmod 3^{2 m}\\right) . \\label{eq2}\n$$\n若 $m \\geqslant 2$, 则由第 8 单元例 5 可知, 2 模 $3^{2 m}$ 的阶是 $2 \\times 3^{2 m-1}$, 故由 式\\ref{eq2} 推出, $2 \\times 3^{2 m-1} \\mid 2 \\times 3^m c$, 即 $3^{m-1} \\mid c$, 从而 $3 \\mid c$ (因 $m \\geqslant 2$ ), 这与式\\ref{eq1}中 $3 \\nmid c$ 矛盾.\n故必须有 $m=1$.\n第三步, 证明(1)中的 $c=1$. 这可与上述第一步, 即第 8 单元中例 1 类似地进行:\n若 $c>1$, 设 $q$ 是 $c$ 的最小素因子, 则有\n$$\n2^{3 c} \\equiv-1(\\bmod q) . \\label{eq3}\n$$\n设 $r$ 是 2 模 $q$ 的阶, 由式\\ref{eq3}得 $2^{6 c} \\equiv 1(\\bmod q)$, 又 $2^{q-1} \\equiv 1(\\bmod q)$, 故 $r \\mid 6 c$ 及 $r \\mid(q-1)$, 从而 $r \\mid(6 c, q-1)$. 由 $q$ 的选取知 $(q-1, c)=1$, 所以 $r \\mid 6$, 再由 $2^r \\equiv 1(\\bmod q)$, 推知 $q=3$ 或 7. 易知 $q=3$ 为不可能; 而由 式\\ref{eq3} 知 $q=7$ 也不可能.\n所以必有 $c=1$. 因此 $n=3$.\n请注意, 若先证明式\\ref{eq1}中的 $c=1$ 将不易奏效.\n这里的论证次序颇为重要.\n此外,第二步中 $m=1$ 也可通过比较素数幕来证明:\n由二项式定理得\n$$\n2^n+1=(3-1)^n+1=3 n+\\sum_{k=2}^n(-1)^k \\mathrm{C}_n^k 3^k . \\label{eq4}\n$$\n设 $3^\\alpha \\| k !$, 则\n$$\n\\alpha=\\sum_{l=1}^{\\infty}\\left[\\frac{k}{3^l}\\right]<\\sum_{l=1}^{\\infty} \\frac{k}{3^l}=\\frac{k}{2}\n$$\n于是由 $\\mathrm{C}_n^k 3^k=\\frac{n(n-1) \\cdots(n-k+1)}{k !} 3^k$ 可见, $\\mathrm{C}_n^k 3^k$ 被 $3^\\beta$ 整除, 而 $\\beta$ 满足 (注意 $k \\geqslant 2$ )\n$$\n\\beta \\geqslant k+m-\\alpha>k+m-\\frac{k}{2} \\geqslant m+1,\n$$\n故 $\\beta \\geqslant m+2$, 从而 \\ref{eq4} 式右边的和被 $3^{m+2}$ 整除.\n若 $m>1$, 则 $2 m \\geqslant m+2$, 故由 $3^{2 m} \\mid\\left(2^n+1\\right)$ 及 式\\ref{eq4} 推出 $3^{m+2} \\mid 3 n$, 即 $3^{m+1} \\mid n$, 这与 式\\ref{eq1} 矛盾, 因此必有 $m=1$. 下面的例 3 也可用比较素数幕的方法解决.", + "remark": "", + "figures": [] +} \ No newline at end of file diff --git a/processed_dataset/proof/0149.json b/processed_dataset/proof/0149.json new file mode 100644 index 0000000000000000000000000000000000000000..35f6b49ee6d4d602206a98aa36457ceed507290d --- /dev/null +++ b/processed_dataset/proof/0149.json @@ -0,0 +1,8 @@ +{ + "source_file": "./raw_volume-zh/volume10/chapter10.tex", + "problem_type": "proof", + "problem": "例3. 证明: 对每个 $n>1$, 方程\n$$\n\\frac{x^n}{n !}+\\frac{x^{n-1}}{(n-1) !}+\\cdots+\\frac{x^2}{2 !}+\\frac{x}{1 !}+1=0\n$$\n没有有理数根.", + "solution": "证明:设 $a$ 是所说的方程的一个有理根, 则易知\n$$\na^n+\\frac{n !}{(n-1) !} a^{n-1}+\\cdots+\\frac{n !}{k !} a^k+\\cdots+\\frac{n !}{1 !} a+n !=0, \\label{eq1}\n$$\n于是 $a$ 是一个首项系数为 1 的整系数多项式的有理根, 故 $a$ 必是一个整数 .\n因 $n>1$, 故 $n$ 有素因子 $p$ (这一基本的事实已使用过多次). 由于 $n \\mid \\frac{n !}{k !}(k= 0,1, \\cdots, n-1)$, 故由 式\\ref{eq1} 推出 $p \\mid a^n$, 从而素数 $p$ 整除 $a$. 现在比较\\ref{eq1}式左边各项中含 $p$ 的方幂.\n因为 $p$ 在 $k$ ! 中出现的次数为\n$$\n\\sum_{l=1}^{\\infty}\\left[\\frac{k}{p^l}\\right]<\\sum_{l=1}^{\\infty} \\frac{k}{p^l}1, x_1, \\cdots, x_n$ 是 $n$ 个实数, 它们的积记为 $A$. 若对 $i=1$, $n$, 数 $A-x_i$ 都是奇整数.\n证明: 每一个 $x_i$ 都是无理数.", + "solution": "证明:反证法,若有一个 $i$ 使得 $x_i$ 为有理数,则因 $A-x_i$ 为奇整数,所以 $A$ 必是一个有理数.\n记 $A-x_i=a_i(i=1, \\cdots, n)$. 则由 $x_1 \\cdots x_n=A$, 得出\n$$\n\\left(A-a_1\\right) \\cdots\\left(A-a_n\\right)=A . \\label{eq1}\n$$\n由于 $a_i$ 均是(奇)整数, 从而 $A$ 满足了一个首项系数为 1 的整系数方程,故有理数 $A$ 必是一个整数.\n但另一方面, 无论 $A$ 是奇数或偶数,易知 \\ref{eq1}式左、右两边的奇偶性都不同, 从而式\\ref{eq1}决不能成立, 矛盾! 故每个 $x_i$ 都是无理数.", + "remark": "", + "figures": [] +} \ No newline at end of file diff --git a/processed_dataset/proof/0151.json b/processed_dataset/proof/0151.json new file mode 100644 index 0000000000000000000000000000000000000000..635352770b8ee3e65ebc43b5a971151b67879676 --- /dev/null +++ b/processed_dataset/proof/0151.json @@ -0,0 +1,8 @@ +{ + "source_file": "./raw_volume-zh/volume10/chapter10.tex", + "problem_type": "proof", + "problem": "例5. 设 $a, b, c$ 为整数, $f(x)=x^3+a x^2+b x+c$. 证明: 有无穷多个正整数 $n$, 使得 $f(n)$ 不是完全平方数.", + "solution": "证明:我们证明,对任意正整数 $n \\equiv 1(\\bmod 4)$, 四个整数 $f(n), f(n+$ 1), $f(n+2), f(n+3)$ 中至少有一个不是完全平方, 由此便证明了问题中的结论.\n易知\n$$\n\\begin{aligned}\n& f(n) \\equiv 1+a+b+c(\\bmod 4), \\\\\n& f(n+1) \\equiv 2 b+c(\\bmod 4), \\\\\n& f(n+2) \\equiv-1+a-b+c(\\bmod 4), \\\\\n& f(n+3) \\equiv c(\\bmod 4) .\n\\end{aligned}\n$$\n消去 $a 、 c$ 得\n$$\nf(n+1)-f(n+3) \\equiv 2 b, f(n)-f(n+2) \\equiv 2 b+2(\\bmod 4) .\n$$\n因此,或者 $f(n+1)-f(n+3) \\equiv 2(\\bmod 4)$, 或者 $f(n)-f(n+2) \\equiv 2(\\bmod 4)$. 因为完全平方数模 4 同余于 0 或 1 , 故或者 $f(n+1)$ 与 $f(n+3)$ 中至少有一个非平方数, 或者 $f(n)$ 与 $f(n+2)$ 中至少有一个非平方数, 从而 $f(n), f(n+1)$, $f(n+2), f(n+3)$ 中至少有一个不是完全平方.", + "remark": "", + "figures": [] +} \ No newline at end of file diff --git a/processed_dataset/proof/0152.json b/processed_dataset/proof/0152.json new file mode 100644 index 0000000000000000000000000000000000000000..f5170b490dc8b9cd39a913c43d256f3c7c8f2bc7 --- /dev/null +++ b/processed_dataset/proof/0152.json @@ -0,0 +1,8 @@ +{ + "source_file": "./raw_volume-zh/volume10/chapter10.tex", + "problem_type": "proof", + "problem": "例6. 设 $p(x)$ 是一个整系数多项式,对任意 $n \\geqslant 1$ 有 $p(n)>n$. 定义 $x_1= 1, x_2=p\\left(x_1\\right), \\cdots, x_n=p\\left(x_{n-1}\\right)(n \\geqslant 2)$. 若对于任意正整数 $N$, 数列 $\\left\\{x_n\\right\\}(n \\geqslant 1)$ 中均有被 $N$ 整除的项.\n证明 $p(x)=x+1$.", + "solution": "证明:我们分两步进行.\n首先证明, 对任一个固定的 $m>1$, 数列 $\\left\\{x_n\\right\\}$ 模 $x_m-1$ 是周期数列.\n显然 $x_m \\equiv 1=x_1\\left(\\bmod x_m-1\\right)$. 因 $p(x)$ 是整系数多项式, 故对任意整数 $u 、 v(u \\neq v)$ 有 $(u-v) \\mid(p(u)-p(v))$, 即\n$$\np(u) \\equiv p(v)(\\bmod u-v),\n$$\n在上式中取 $u=x_m, v=x_1=1$, 得 $x_{m+1} \\equiv x_2\\left(\\bmod x_m-1\\right)$. 依此类推, $x_{m+2} \\equiv x_3, x_{m+3} \\equiv x_4, \\cdots\\left(\\bmod x_m-1\\right)$, 可知 $\\left\\{x_n\\right\\}$ 模 $x_m-1$ 是周期数列 $x_1, \\cdots, x_{m-1}$, $x_1, \\cdots, x_{m-1}, \\cdots$.\n第二步,我们证明\n$$\nx_m-1=x_{m-1} . \\label{eq1}\n$$\n由已知条件, 对于数 $N=x_m-1$, 存在 $x_k$ 使得 $\\left(x_m-1\\right) \\mid x_k$, 而由上一段的结论, 可设 $1 \\leqslant k \\leqslant m-1$, 此外, $p\\left(x_{m-1}\\right)>x_{m-1}$, 故 $x_m-1 \\geqslant x_{m-1}$, 所以 $k$ 必须为 $m-1$, 即 $\\left(x_m-1\\right) \\mid x_{m-1}$, 于是, $x_{m-1} \\geqslant x_m-1$, 综合起来即知 \\ref{eq1}式成立.\n因为式\\ref{eq1}即是 $p\\left(x_{m-1}\\right)-1=x_{m-1}$. 由于 $m$ 是任意大于 1 的整数,这意味着 $p(x)=x+1$ 有无穷多个不同的根, 故 $p(x)$ 必须恒等于多项式 $x+1$. 这就证明了本题的结论.\n由一个(整系数) 多项式的算术 (数论) 性质, 推断其代数性质, 是数论中非常有趣的一个课题, 例 6 正是这样的一个简单例子, 下面的例 7 也是具有这种精神的问题.", + "remark": "", + "figures": [] +} \ No newline at end of file diff --git a/processed_dataset/proof/0153.json b/processed_dataset/proof/0153.json new file mode 100644 index 0000000000000000000000000000000000000000..a10a12ad9fa84901ce4069caf059ea48f3c127af --- /dev/null +++ b/processed_dataset/proof/0153.json @@ -0,0 +1,8 @@ +{ + "source_file": "./raw_volume-zh/volume10/chapter10.tex", + "problem_type": "proof", + "problem": "例7. 设 $f(x)$ 是一个实系数的二次多项式, 若对所有正整数 $n, f(n)$ 均是整数的平方.\n证明, $f(x)$ 是一次整系数多项式的平方.", + "solution": "证明:本题并不容易,但有几种完全不同的解法.\n这里介绍的方法基于数列的极限知识, 较为简单.\n设 $f(x)=a x^2+b x+c, a_n=f(n)(n \\geqslant 1)$, 则易知\n$$\n\\begin{aligned}\n\\sqrt{a_n}-\\sqrt{a_{n-1}} & =\\frac{a_n-a_{n-1}}{\\sqrt{a_n}+\\sqrt{a_{n-1}}} \\\\\n& =\\frac{2 a n-a+b}{\\sqrt{a n^2+b n+c}+\\sqrt{a n^2+(-2 a+b) n+a-b+c}} \\\\\n& =\\frac{2 a+\\frac{b-a}{n}}{\\sqrt{a+\\frac{b}{n}+\\frac{c}{n^2}}+\\sqrt{a+\\frac{b-2 a}{n}+\\frac{a-b+c}{n^2}}} .\n\\end{aligned}\n$$\n因此当 $n \\rightarrow \\infty$ 时, $\\sqrt{a_n}-\\sqrt{a_{n-1}}$ 有极限, 且极限值为 $\\frac{2 a}{\\sqrt{a}+\\sqrt{a}}=\\sqrt{a}$. 但已知 $\\sqrt{a_n}$ 都是整数, 故 $\\left\\{\\sqrt{a_n}-\\sqrt{a_{n-1}}\\right\\}(n \\geqslant 2)$ 是一个整数数列, 因此其极限值 $\\sqrt{a}$ 必是一个整数, 且 $n$ 充分大后, 所有项 $\\sqrt{a_n}-\\sqrt{a_{n-1}}$ 都等于极限 $\\sqrt{a}$, 即有一个 (固定的) 正整数 $k$, 使得\n$$\n\\sqrt{a_n}-\\sqrt{a_{n-1}}=\\sqrt{a} \\text {, 对 } n \\geqslant k+1 \\text {. }\n$$\n现在设 $m$ 是大于 $k$ 的任一个整数, 将上式对 $n=k+1, \\cdots, m$ 求和, 得出 $\\sqrt{a_m}= \\sqrt{a_k}+(m-k) \\sqrt{a}$, 即\n$$\na_m=\\left(m \\sqrt{a}+\\sqrt{a_k}-k \\sqrt{a}\\right)^2 . \\label{eq1}\n$$\n记 $\\alpha=\\sqrt{a}, \\beta=\\sqrt{a_k}-k \\sqrt{a}$, 则 $\\alpha 、 \\beta$ 都是与 $m$ 无关的固定整数,于是, 式\\ref{eq1}表明, 所有大于 $k$ 的整数 $m$ 都是多项式\n$$\nf(x)-(\\alpha x+\\beta)^2\n$$\n的根, 从而这多项式必是零多项式,即 $f(x)=(\\alpha x+\\beta)^2$.", + "remark": "", + "figures": [] +} \ No newline at end of file diff --git a/processed_dataset/proof/0154.json b/processed_dataset/proof/0154.json new file mode 100644 index 0000000000000000000000000000000000000000..88128568729025b5d901f26ee2f74ce72e5ecef4 --- /dev/null +++ b/processed_dataset/proof/0154.json @@ -0,0 +1,8 @@ +{ + "source_file": "./raw_volume-zh/volume10/chapter10.tex", + "problem_type": "proof", + "problem": "例8. 设 $n>1, n$ 个正整数的和为 $2 n$. 证明, 在其中一定可以选出某些数,使它们的和等于 $n$, 除非所给的数满足下面的条件之一:\n(1) 有一个数是 $n+1$, 其余的都是 1 ;\n(2) 在 $n$ 为奇数时, 所有数都等于 2 .", + "solution": "证明:. 设所给的正整数为 $01$, 则有\n$$\n\\dot{a_2}+\\cdots+a_k+a_n \\equiv 0(\\bmod n) .\n$$\n而上式左边显然是小于 $a_1+\\cdots+a_n=2 n$ 的正整数, 故\n$$\na_2+\\cdots+a_k+a_n=n .\n$$\n(iv) 设 $a_1-a_n \\equiv 0(\\bmod n)$. 我们已证明 $a_n \\leqslant n+1$ (见 \\ref{eq2} 式): 若 $a_n= n+1$, 则 $n-1$ 个正整数 $a_1, \\cdots, a_{n-1}$ 的和等于 $2 n-a_n=n-1$, 从而它们都等于 1 ,这正是问题中排除的情形 (1).\n设 $a_n \\leqslant n$, 则 $0 \\leqslant a_n-a_1 \\leqslant n-1$, 结合 $a_n-a_1 \\equiv 0(\\bmod n)$, 推出 $a_n=\\cdots=a_2=a_1=2$. 当 $n$ 为奇数时, 这是问题中排除的情形 (2). 若 $n$ 为偶数, 则任取 $\\frac{n}{2}$ 个 $a_i$ 的和便等于 $n$.\n大意可概述如下: 由于所有给定数的和为 $2 n$, 因此, 只要能证明有若干个(不是全部) 数的和是 $n$ 的倍数, 则这个和必然恰等于 $n$, 而后一问题正是同余的用武之地.", + "remark": "", + "figures": [] +} \ No newline at end of file diff --git a/processed_dataset/proof/0155.json b/processed_dataset/proof/0155.json new file mode 100644 index 0000000000000000000000000000000000000000..d366831a1d5ac376b4bef517e3866c55c7319188 --- /dev/null +++ b/processed_dataset/proof/0155.json @@ -0,0 +1,8 @@ +{ + "source_file": "./raw_volume-zh/volume10/chapter10.tex", + "problem_type": "proof", + "problem": "例9. 设 $p$ 为素数, 给定 $p+1$ 个不同的正整数.\n证明, 可以从中取出这样一对数,使得将两者中较大的数除以两者的最大公约数后, 所得的商不小于 $p+1$.", + "solution": "证明:将所给的 $p+1$ 个数都除以它们的最大公约数, 显然不影响本题的结论, 因此我们可设这 $p+1$ 个数互素.\n特别地, 其中必有一个数不被 $p$ 整除.\n记这 $p+1$ 个数是\n$$\nx_1, \\cdots, x_k, x_{k+1}=p^{l_{k+1}} y_{k+1}, \\cdots, x_{p+1}=p^{l_{p+1}} y_{p+1}, \\label{eq1}\n$$\n这里, $\\dot{x}_1, \\cdots, x_k$ 互不相等且均和 $p$ 互素 $(k \\geqslant 1), l_{k+1}, \\cdots, l_{p+1}$ 是正整数, $y_{k+1}, \\cdots, y_{p+1}$ 都是不被 $p$ 整除的正整数.\n在 $p+1$ 个数\n$$\nx_1, \\cdots, x_k, y_{k+1}, \\cdots, y_{p+1}\n$$\n中, 必有两个模 $p$ 同余, 我们区分三种情况讨论.\n(1) 式\\ref{eq1} 中的数至少有三个相等.\n此时结论容易证明.\n因为若 $y_r=y_s=y_t$,\n则 $p^{l_r}, p^{l_s}, p^{l_t}$ 互不相等, 其中最大的数至少是最小者的 $p^2$ 倍, 无妨设 $p^{l_r} \\geqslant p^2 \\cdot p^{l_t}$, 则 $x_r$ 与 $x_t$ 符合要求; 若 $y_r=y_s=x_t(1 \\leqslant t \\leqslant k)$, 无妨设 $l_r>l_s$, 则 $l_r \\geqslant 2$,于是 $x_r$ 与 $x_t$ 符合要求.\n(2) 式\\ref{eq1}中的数有两对相等.\n若 $y_i=y_j, y_r=y_s$, 则当 $\\left|l_i-l_j\\right| \\geqslant 2$ 或 $\\left|l_r-l_s\\right| \\geqslant 2$ 时, 同上可知结论成立; 当 $\\left|l_i-l_j\\right| \\leqslant 1$ 且 $\\left|l_r-l_s\\right| \\leqslant 1$ 时, 可改记 $x_i, x_j, x_r, x_s$ 为 $a, a p, b, b p$, 且 $ap\n$$\n故整数 $\\frac{b p}{(a, b p)} \\geqslant p+1$.\n若 $x_i=y_r, x_j=y_s(1 \\leqslant i, j \\leqslant k)$, 同样可证明结论.\n(3) 式\\ref{eq1} 中的数恰有两个相等.\n这只能是 $y_r=y_s$, 或 $x_i=y_r(1 \\leqslant i \\leqslant k)$. 这时可在式\\ref{eq1}中删去 $y_r$, 则剩下的 $p$ 个数互不相等, 但仍有两个模 $p$ 同余.\n现在又有三种可能:\n(i) 设 $y_r \\equiv y_s(\\bmod p)$. 无妨设 $y_r>y_s$. 若 $l_r>l_s$, 结论显然成立.\n若 $l_r \\leqslant l_s$, 记 $y_r=y_s+n$, 则 $n>0$, 且 $p \\mid n$. 设 $\\left(y_r, y_s\\right)=d$, 则 $p \\nmid d$, 于是 $\\left(x_r, x_s\\right)= p^{l_r} d$, 我们有 (注意 $d|n, p| n$, 以及 $p \\nmid d$ )\n$$\n\\frac{x_r}{\\left(x_r, x_s\\right)}=\\frac{y_r}{d}=\\frac{y_s}{d}+\\frac{n}{d} \\geqslant 1+p .\n$$\n所以, $x_r$ 与 $x_s$ 中的较大者除以它们的最大公约数后, 得出的商至少是 $p+1$.\n(ii) 设 $x_r \\equiv x_s(\\bmod p)(1 \\leqslant rx_r$, 则结论显然成立.\n若 $y_s< x_r$, 设 $x_r=y_s+n$, 则 $n>0$, 且 $p \\mid n$. 设 $\\left(x_r, y_s\\right)=d$, 则 $p \\nmid d$, 于是 $\\left(x_r, x_s\\right)= \\left(x_r, p^{l_s} y_s\\right)=d$, 因此\n$$\n\\frac{x_r}{\\left(x_r, x_s\\right)}=\\frac{y_s}{d}+\\frac{n}{d} \\geqslant 1+p,\n$$\n从而 $x_r$ 与 $x_s$ 中较大的数除以它们的最大公约数后, 得出的商不小于 $p+1$. 这就完成了问题的证明.\n我们注意,若例 9 中的 $p+1$ 个整数换为 $p$ 个整数,则结论不必正确.\n例如, $p$ 个数 $1,2, \\cdots, p$ 中显然没有符合要求的两个数.", + "remark": "", + "figures": [] +} \ No newline at end of file diff --git a/processed_dataset/proof/0156.json b/processed_dataset/proof/0156.json new file mode 100644 index 0000000000000000000000000000000000000000..18ed15d56292f985424ce81eafc68b994c52e674 --- /dev/null +++ b/processed_dataset/proof/0156.json @@ -0,0 +1,8 @@ +{ + "source_file": "./raw_volume-zh/volume10/chapter10.tex", + "problem_type": "proof", + "problem": "例10. 设 $S$ 是 $\\left\\{1,2, \\cdots, 2^m n\\right\\}$ 的一个子集, $S$ 的元素个数 $|S| \\geqslant\\left(2^m- 1 \\right) n+1$. 证明, $S$ 中有 $m+1$ 个不同的数 $a_0, \\cdots, a_m$, 使得 $a_{i-1} \\mid a_i(i=1, \\cdots$, m).", + "solution": "证明:每个正整数 $a$ 可唯一地表示为 $2^u k$ 的形式,其中 $u \\geqslant 0, k$ 为奇数, 我们称 $k$ 为 $a$ 的奇数部分, 并且若 $a$ 的奇数部分不超过 $n$, 则称 $n$ 为好数.\n这里的证明, 基于 $S$ 中好数个数的下界估计.\n为此, 我们首先计数在区间 $\\left(n, 2^m n\\right]$ 中有多少个好数.\n设区间 $[1, n]$ 中共有 $t$ 个奇数 $\\left(t\\right.$ 实际上等于 $\\left[\\frac{n+1}{2}\\right]$, 但我们并不需要这一点). 设 $k$ 是任意一个这样的奇数, 则满足 $n<2^u k \\leqslant 2^m n$ 的整数 $u$ 恰有 $m$ 个.\n这只要注意, 设整数 $v$ 满足 $2^{v-1} \\leqslant \\frac{n}{k}<2^v$, 则 $2^v k, 2^{v+1} k, \\cdots, 2^{v-1+m} k$ 是全部符合要求的数, 即在区间 $\\left(n, 2^m n\\right]$ 中奇数部分为 $k$ 的数共有 $m$ 个, 故其中恰有 $m t$ 个好数.\n因此这区间中非好数有 $2^m n-n-m t$ 个, 从而 $S$ 中好数的个数 $\\geqslant|S|-\\left(2^m n-n-m t\\right)=m t+1$ 个.\n设 $k_1, \\cdots, k_t$ 是 $[1, n]$ 中的全部奇数, 并设 $S$ 中恰有 $x_i$ 个数以 $k_i$ 为奇数部分 $(k=1, \\cdots, t)$, 则由上一段的结论, $S$ 中好数的个数为\n$$\nx_1+\\cdots+x_t \\geqslant m t+1,\n$$\n从而必有一个 $x_i(1 \\leqslant i \\leqslant t)$, 使得 $x_i \\geqslant m+1$, 即 $S$ 中至少有 $m+1$ 个整数具有相同的奇数部分 $k_i$, 这些数从小到大排列为 $a_0, a_1, \\cdots, a_m$, 即为符合要求的 $m$ 个数, 证毕.", + "remark": "注:1 当 $m=1$ 时, 本题化为了一个熟知的结果, 这里的证明即是此结果 (通常的)证明的推广.\n本题还有其他的解法, 例如, 对 $m$ 归纳或对 $n$ 归纳, 有兴趣的读者可自己试试.\n注:2 集合 $S=\\left\\{n+1, \\cdots, 2^m n\\right\\}$ 表明, 若例 10 中的 $S$ 满足 $|S|= \\left(2^m-1\\right) n$, 则结论不必正确.\n因若有 $a_0, \\cdots, a_m$ 符合要求, 则 $a_m \\geqslant 2^m a_0$, 从而现在有 $a_m \\geqslant 2^m(n+1)$, 这不可能.", + "figures": [] +} \ No newline at end of file diff --git a/processed_dataset/proof/0157.json b/processed_dataset/proof/0157.json new file mode 100644 index 0000000000000000000000000000000000000000..1273c61484210887c962737f25426ae57f58b041 --- /dev/null +++ b/processed_dataset/proof/0157.json @@ -0,0 +1,8 @@ +{ + "source_file": "./raw_volume-zh/volume10/chapter10.tex", + "problem_type": "proof", + "problem": "例11. 设 $A$ 是正整数的 $n$ 元集合 $(n \\geqslant 2)$. 证明, $A$ 有一个子集 $B$, 满足 $|B|>\\frac{n}{3}$, 且对任意 $x, y \\in B$, 有 $x+y \\notin B$.", + "solution": "证明:记 $A$ 中的数为 $a_1, \\cdots, a_n$. 由习题 3 的第 2 题知, 模 3 为 -1 的素数有无穷多个, 故可取一个这样的素数 $p>a_i(1 \\leqslant i \\leqslant n)$, 设 $p=3 k-1$. 考虑下面 ( $p$ 行 $n$ 列的) $p n$ 个数\n$$\n\\begin{aligned}\n& a_1, a_2, \\cdots, a_n ; \\\\\n& 2 a_1, 2 a_2, \\cdots, 2 a_n ; \\\\\n& \\cdots . . \\\\\n& p a_1, p a_2, \\cdots, p a_n .\n\\end{aligned} \\label{eq1}\n$$\n由于 $p>a_i$, 故 $\\left(p, a_i\\right)=1$. 因此 式\\ref{eq1} 中每一列数均构成模 $p$ 的一个完系, 从而对每个 $j(0 \\leqslant j\\frac{n}{3},\n$$\n即有一个 $l(1 \\leqslant l \\leqslant p)$, 使 $l a_1, l a_2, \\cdots, l a_n$ 中模 $p$ 为 $k, k+1, \\cdots, 2 k-1$ 之一的个数大于 $\\frac{n}{3}$. 我们取\n$$\nB=\\{a \\in A \\mid l a \\text { 模 } p \\text { 为 } k, k+1, \\cdots, 2 k-1 \\text { 之一 }\\},\n$$\n则 $B$ 符合要求: 因为对任意 $x, y \\in B$, 易知 $l(x+y)(=l x+l y)$ 模 $p$ 的余数或 $\\geqslant 2 k$, 或 $\\leqslant k-1$, 从而 $x+y \\notin B$.", + "remark": "", + "figures": [] +} \ No newline at end of file diff --git a/processed_dataset/proof/0158.json b/processed_dataset/proof/0158.json new file mode 100644 index 0000000000000000000000000000000000000000..288a413d9cd8918146de3712c90288a75e58c1f7 --- /dev/null +++ b/processed_dataset/proof/0158.json @@ -0,0 +1,8 @@ +{ + "source_file": "./raw_volume-zh/volume10/chapter10.tex", + "problem_type": "proof", + "problem": "例12. 给定 $n \\geqslant 2$. 证明, 存在 $n$ 个互不相同的正整数具有下述性质:\n(1) 这些数两两互素;\n(2) 这些数中任意 $k$ 个 $(2 \\leqslant k \\leqslant n)$ 数的和都是合数.", + "solution": "证明:.\n$n=2$ 时结论显然成立.\n设已有 $n$ 个正整数 $a_1, \\cdots, a_n$ 符合要求, 下面基于此造出 $n+1$ 个符合要求的数.\n由于素数有无穷多个,故可取 $2^n-1$ 个互不相同且均与 $a_1 a_2 \\cdots a_n$ 互素的素数 $p_i\\left(1 \\leqslant i \\leqslant 2^n-1\\right)$. 将由 $a_{11}, \\cdots, a_n$ 中任取 $k$ 个 $(1 \\leqslant k \\leqslant n)$ 所作成的 $2^n-$ 1 个和记为 $S_j\\left(1 \\leqslant j \\leqslant 2^n-1\\right)$, 其中 $k=1$ 时的和就是数 $a_i(1 \\leqslant i \\leqslant n)$.\n因为 $\\left(p_i, a_1 \\cdots a_n\\right)=1$, 故有 $b_i$ 使得 $a_1 \\cdots a_n \\cdot b_i \\equiv 1\\left(\\bmod p_i\\right)\\left(1 \\leqslant i \\leqslant 2^n-\\right.$ 1 ). 由中国剩余定理, 同余式组\n$$\nx \\equiv-b_i-b_i S_i\\left(\\bmod p_i\\right), 1 \\leqslant i \\leqslant 2^n-1 . \\label{eq1}\n$$\n有无穷多个正整数解 $x$. 我们取定一个解 $x_0>p_i\\left(1 \\leqslant i \\leqslant 2^n-1\\right)$, 并将式\\ref{eq1}中同余式两边同乘 $a_1 \\cdots a_n$, 得到\n$$\na_1 \\cdots a_n x_0+1+S_i \\equiv 0\\left(\\bmod p_i\\right), 1 \\leqslant i \\leqslant 2^n-1 . \\label{eq2}\n$$\n令 $a_{n+1}=a_1 \\cdots a_n x_0+1$, 则 $a_1, \\cdots, a_n, a_{n+1}$ 这 $n+1$ 个数符合要求: 因为 $x_0> p_i$, 故 $a_{n+1}+S_i>p_i$; 而 式\\ref{eq2} 意味着 $a_{n+1}+S_i$ 有约数 $p_i$, 故对任意 $i, a_{n+1}+S_i$ 是合数.\n而由 $a_{n+1}$ 的构作, 它当然与每个 $a_i$ 互素 $(1 \\leqslant i \\leqslant n)$. 这就完成了归纳构造.\n上述解法的精神是, 若已有了 $a_1, \\cdots, a_n$, 我们希望可以取参量 $x$ 的一个值, 使得数 $a_1 \\cdots a_n x+1$ 能够作为 $a_{n+1}$. 构作这种形式的数的主要益处在于, 所要求的 $\\left(a_{n+1}, a_i\\right)=1(1 \\leqslant i \\leqslant n)$ 自动成立.\n符合问题中要求的事物往往不止一个, 我们可以选择某些具有特别性质的事物来尝试, 即使之满足适当的充分条件, 以保证适合问题中的部分要求, 这种以退求进、舍多取少的手法在构造论证中应用极多.\n本题也可采用下面 (更为直接的)构造法: 取 $a_i=i \\cdot n !+1$, 则 $a_1, \\cdots, a_n$ 符合要求.\n这是因为:\n首先, 对 $i \\neq j$ 有 $\\left(a_i, a_j\\right)=1$. 这是因为若设 $\\left(a_i, a_j\\right)=d$, 则 $j a_i-i a_j$ 是 $d$ 的倍数, 即 $d \\mid(i-j)$. 但 $1 \\leqslant|i-j|1$, 由 $\\tau(n)$ 的计算公式可知, 问题等价于证明, 存在正整数 $\\alpha, \\beta, \\cdots, \\gamma$, 使得\n$$\n\\frac{(2 \\alpha+1)}{\\alpha+1} \\cdot \\frac{(2 \\beta+1)}{\\beta+1} \\cdot \\cdots \\cdot \\frac{(2 \\gamma+1)}{\\gamma+1}=k . \\label{eq2}\n$$\n现假设小于 $k$ 的奇数均符合要求, 对于奇数 $k$, 可设 $k=2^l m-1$, 这里 $l \\geqslant 1, m$ 为奇数.\n由 $k>1$ 易知 $mn$. 论证的关键是利用 $F_n \\mid\\left(F_m-2\\right)$ (见第 1 单元例 $2)$, 即有一个整数 $x$, 使得\n$$\nF_m+x F_n=2 .\n$$\n设 $d=\\left(F_m, F_n\\right)$, 则由上式推出 $d \\mid 2$, 所以 $d=1$ 或 2 . 但 $F_n$ 显然是奇数,故必须 $d=1$.", + "remark": "注:$F_k(k \\geqslant 0)$ 称为费马 (Fermat) 数.\n例 3 表明,费马数两两互素, 这是费马数的一个有趣的基本性质.", + "figures": [] +} \ No newline at end of file diff --git a/processed_dataset/proof/0163.json b/processed_dataset/proof/0163.json new file mode 100644 index 0000000000000000000000000000000000000000..34aa623b93bcc0aca5bf02b39e91f6c0e298fa19 --- /dev/null +++ b/processed_dataset/proof/0163.json @@ -0,0 +1,8 @@ +{ + "source_file": "./raw_volume-zh/volume10/chapter2.tex", + "problem_type": "proof", + "problem": "例4. 设 $a>1, m, n>0$, 证明:\n$$\n\\left(a^m-1, a^n-1\\right)=a^{(m, n)}-1 .\n$$", + "solution": "证明:设 $D=\\left(a^m-1, a^n-1\\right)$. 我们通过证明 $\\left(a^{(m, n)}-1\\right) \\mid D$ 及 $D \\mid \\left(a^{(m, n)}-1\\right)$ 来导出 $D==a^{(m, n)}-1$, 这是数论中证明两数相等的常用手法.\n因为 $(m, n)|m,(m, n)| n$, 由第 1 单元中分解公式 (5) 即知 $\\left(a^{(m, n)}-1\\right) \\mid \\left(a^m-1\\right)$, 以及 $\\left(a^{(m, n)}-1\\right) \\mid\\left(a^n-1\\right)$. 故由本单元的性质 (3) 可知, $a^{(m, n)}-1$ 整除 $\\left(a^m-1, a^n-1\\right)$, 即 $\\left(a^{(m, n)}-1\\right) \\mid D$.\n为了证明 $D \\mid\\left(a^{(m, n)}-1\\right)$, 我们设 $d=(m, n)$. 因 $m, n>0$, 故可选择 $u$, $v>0$, 使得 (参见本单元 (1) 中的注释)\n$$\nm u-n v=d . \\label{eq1}\n$$\n因为 $D \\mid\\left(a^m-1\\right)$, 故更有 $D \\mid\\left(a^{m u}-1\\right)$. 同样, $D \\mid\\left(a^{n u}-1\\right)$. 故 $D \\mid\\left(a^{m u}-\\right. \\left.a^{n o}\\right)$, 从而由 式\\ref{eq1}, 得\n$$\nD \\mid a^{n v}\\left(a^d-1\\right) . \\label{eq2}\n$$\n此外, 因 $a>1$, 及 $D \\mid\\left(a^m-1\\right)$, 故 $(D, a)=1$, 进而 $\\left(D, a^{n v}\\right)=1$. 于是, 从 式\\ref{eq2} 及性质 (7) 导出 $D \\mid\\left(a^d-1\\right)$, 即 $D \\mid\\left(a^{(m, n)}-1\\right)$.\n综合已证得的两方面的结果, 可知 $D=a^{(m, n)}-1$.", + "remark": "", + "figures": [] +} \ No newline at end of file diff --git a/processed_dataset/proof/0164.json b/processed_dataset/proof/0164.json new file mode 100644 index 0000000000000000000000000000000000000000..3cff7467046a3648ca421014ea07986a24be6a9f --- /dev/null +++ b/processed_dataset/proof/0164.json @@ -0,0 +1,8 @@ +{ + "source_file": "./raw_volume-zh/volume10/chapter2.tex", + "problem_type": "proof", + "problem": "例5. 设 $m, n>0, m n \\mid\\left(m^2+n^2\\right)$, 则 $m=n$.", + "solution": "证明:设 $(m, n)=d$, 则 $m=m_1 d, n=n_1 d$, 其中 $\\left(m_1, n_1\\right)=1$.\n于是, 已知条件化为 $m_1 n_1 \\mid\\left(m_1^2+n_1^2\\right)$, 故更有 $m_1 \\mid\\left(m_1^2+n_1^2\\right)$, 从而 $m_1 \\mid n_1^2$.\n但 $\\left(m_1, n_1\\right)=1$, 故 $\\left(m_1, n_1^2\\right)=1$. 结合 $m_1 \\mid n_1^2$, 可知必须 $m_1=1$. 同理 $n_1=$ 1 , 因此 $m=n$.", + "remark": "注1 对两个给定的不全为零的整数, 我们常借助它们的最大公约数, 并应用性质(5), 产生两个互素的整数, 以利用互素的性质作进一步论证 (参见性质 (6)、(7)). 就本题而言, 由于 $m n$ 为二次式, $m^2+n^2$ 为二次齐次式, 上述手续的功效,实质上是将问题化归成 $m 、 n$ 互素这种特殊情形.\n注2 在某些问题中,已知的条件 (或已证得的结论) $c \\mid a$ 并不适用,我们可试着选取 $c$ 的一个适当的约数 $b$, 并从 $c \\mid a$ 过渡到 (较弱的结论) $b \\mid a$, 以期望后者提供适宜于进一步论证的信息.\n本题中,我们便是由 $m_1 n_1 \\mid\\left(m_1^2+n_1^2\\right)$ 产生了 $m_1 \\mid n_1^2$, 进而导出 $m_1=1$.", + "figures": [] +} \ No newline at end of file diff --git a/processed_dataset/proof/0165.json b/processed_dataset/proof/0165.json new file mode 100644 index 0000000000000000000000000000000000000000..1448432c55531dd6a76e1b1d6a5185b841c80459 --- /dev/null +++ b/processed_dataset/proof/0165.json @@ -0,0 +1,8 @@ +{ + "source_file": "./raw_volume-zh/volume10/chapter2.tex", + "problem_type": "proof", + "problem": "例6. 设正整数 $a 、 b 、 c$ 的最大公约数为 1 , 并且\n$$\n\\frac{a b}{a-b}=c \\text {. }\n$$\n证明: $a-b$ 是一个完全平方数.", + "solution": "证明:设 $(a, b)=d$, 则 $a=d a_1, b=d b_1$, 其中 $\\left(a_1, b_1\\right)=1$. 由于 $(a$, $b, c)=1$, 故有 $(d, c)=1$.\n现在, 问题中的等式可化为\n$$\nd a_1 b_1=c a_1-c b_1, \\label{eq1}\n$$\n由此可见 $a_1$ 整除 $c b_1$. 因 $\\left(a_1, b_1\\right)=1$, 故 $a_1 \\mid c$. 同样得 $b_1 \\mid c$. 再由 $\\left(a_1, b_1\\right)=$ 1 便推出 $a_1 b_1 \\mid c$.\n设 $c=a_1 b_1 k$, 其中 $k$ 是-一个正整数.\n一方面,显然 $k$ 整除 $c$; 另一方面,结合 \\ref{eq1} 式得 $d=k\\left(a_1-b_1\\right)$, 故 $k \\mid d$. 从而 $k \\mid(c, d)$ . 但 $(c, d)=1$, 故 $k=1$.\n因此 $d=a_1-b_1$. 故 $a-b=d\\left(a_1-b_1\\right)=d^2$. 这就证明了 $a-b$ 是一个完全平方数.", + "remark": "", + "figures": [] +} \ No newline at end of file diff --git a/processed_dataset/proof/0166.json b/processed_dataset/proof/0166.json new file mode 100644 index 0000000000000000000000000000000000000000..87913b315ff9d03908d63bf7664d5545e9abfa9c --- /dev/null +++ b/processed_dataset/proof/0166.json @@ -0,0 +1,8 @@ +{ + "source_file": "./raw_volume-zh/volume10/chapter2.tex", + "problem_type": "proof", + "problem": "例7. 设 $k$ 为正奇数,证明: $1+2+\\cdots+n$ 整除 $1^k+2^k+\\cdots+n^k$.", + "solution": "证明:因为 $1+2+\\cdots+n=\\frac{n(n+1)}{2}$, 故问题等价于证明: $n(n+1)$ 整除 $2\\left(1^k+2^k+\\cdots+n^k\\right)$. 因 $n$ 与 $n+1$ 互素, 所以这又等价于证明\n$$\nn \\mid 2\\left(1^k+2^k+\\cdots+n^k\\right)\n$$\n及\n$$\n(n+1) \\mid 2\\left(1^k+2^k+\\cdots+n^k\\right) .\n$$\n事实上,由于 $k$ 为奇数,故由第 1 单元中分解公式 (6), 可知\n$$\n\\begin{aligned}\n& 2\\left(1^k+2^k+\\cdots+n^k\\right) \\\\\n= & {\\left[1^k+(n-1)^k\\right]+\\left[2^k+(n-2)^k\\right]+\\cdots+\\left[(n-1)^k+1^k\\right]+2 n^k }\n\\end{aligned}\n$$\n是 $n$ 的倍数.\n同理,\n$$\n2\\left(1^k+2^k+\\cdots+n^k\\right)=\\left[1^k+n^k\\right]+\\left[2^k+(n-1)^k\\right]+\\cdots+\\left[n^k+1^k\\right]\n$$\n是 $n+1$ 的倍数.", + "remark": "注:整除问题中, 有时直接证明 $b \\mid a$ 不易人手.\n若 $b$ 可分解为 $b=b_1 b_2$, 其中 $\\left(b_1, b_2\\right)=1$, 则我们可将原命题 $b \\mid a$ 分解为等价的两个命题 $b_1 \\mid a$ 及 $b_2 \\mid a$, 后者可能更容易导出来.\n例 7 应用了这一基本手法, 例 6 中证明 $a_1 b_1 \\mid c$ 也是这样做的.\n更一般地, 为了证明 $b \\mid a$, 可将 $b$ 分解为若干个两两互素的整数 $b_1$, $b_2, \\cdots, b_n$ 之积, 而证明等价的 $b_i \\mid a(i=1,2, \\cdots, n)$ (参见性质 (11), 并可比较第 1 单元例 3 的注 1 中说的想法).", + "figures": [] +} \ No newline at end of file diff --git a/processed_dataset/proof/0167.json b/processed_dataset/proof/0167.json new file mode 100644 index 0000000000000000000000000000000000000000..186b215402908f841c86e77f66ff0755a46be350 --- /dev/null +++ b/processed_dataset/proof/0167.json @@ -0,0 +1,8 @@ +{ + "source_file": "./raw_volume-zh/volume10/chapter3.tex", + "problem_type": "proof", + "problem": "例1. 证明: 无穷数列 $10001,100010001, \\cdots$ 中没有素数.", + "solution": "证明:记 $a_n=\\underbrace{10001 \\cdots 10001}_{n \\uparrow 1}(n \\geqslant 2)$, 则\n$$\na_n=1+10^4+10^8+\\cdots+10^{4(n-1)}=\\frac{10^{4 n}-1}{10^4-1} .\n$$\n为了将上式右端的数分解为两个 (大于 1 的)整数之积, 我们区分两种情形:\n$n$ 为偶数.\n设 $n=2 k$, 则\n$$\na_{2 k}=\\frac{10^{8 k}-1}{10^4-1}=\\frac{10^{8 k}-1}{10^8-1} \\cdot \\frac{10^8-1}{10^4-1} .\n$$\n易知, $\\frac{10^8-1}{10^4-1}$ 是大于 1 的整数, 而对 $k \\geqslant 2, \\frac{10^{8 k}-1}{10^8-1}$ 也是大于 1 的整数.\n故 $a_{2 k}(k=2,3, \\cdots)$ 都是合数.\n又 $a_2=10001=13 \\times 137$ 是合数.\n$n$ 为奇数.\n设 $n=2 k+1$, 则\n$$\na_{2 k+1}=\\frac{10^{4(2 k+1)}-1}{10^4-1}=\\frac{10^{2(2 k+1)}-1}{10^2-1} \\cdot \\frac{10^{2(2 k+1)}+1}{10^2+1}\n$$\n是两个大于 1 的整数之积, 故 $a_{2 k+1}$ 也均是合数.\n因此, 所有 $a_n$ 是合数.", + "remark": "", + "figures": [] +} \ No newline at end of file diff --git a/processed_dataset/proof/0168.json b/processed_dataset/proof/0168.json new file mode 100644 index 0000000000000000000000000000000000000000..41c1d8a868008868c03e03ddfbb572ab1aa2a12b --- /dev/null +++ b/processed_dataset/proof/0168.json @@ -0,0 +1,8 @@ +{ + "source_file": "./raw_volume-zh/volume10/chapter3.tex", + "problem_type": "proof", + "problem": "例2. 证明: 对任意整数 $n>1$, 数 $n^4+4^n$ 不是素数.", + "solution": "证明:若 $n$ 为偶数, 则 $n^4+4^n$ 大于 2 且均被 2 整除, 因此都不是素数.\n但对奇数 $n$,易知 $n^4+4^n$ 没有一个(大于 1 的) 固定的约数,我们采用不同的处理:\n设奇数 $n=2 k+1, k \\geqslant 1$, 则\n$$\n\\begin{aligned}\nn^4+4^n & =n^4+4 \\cdot 4^{2 k}=n^4+4 \\cdot\\left(2^k\\right)^4 \\\\\n& =n^4+4 n^2 \\cdot\\left(2^k\\right)^2+4 \\cdot\\left(2^k\\right)^4-4 n^2 \\cdot\\left(2^k\\right)^2 \\\\\n& =\\left(n^2+2 \\cdot 2^{2 k}\\right)^2-\\left(2 \\cdot n \\cdot 2^k\\right)^2 \\\\\n& =\\left(n^2+2^{k+1} n+2^{2 k+1}\\right)\\left(n^2-2^{k+1} n+2^{2 k+1}\\right) .\n\\end{aligned}\n$$\n上式右边第一个因数显然不为 1 , 而后一个因数为 $\\left(n-2^k\\right)^2+2^{2 k}$ 也不是 1 (因 $k \\geqslant 1$ ), 故 $n^4+4^n$ 对 $n>1$ 都是合数.\n这一解法的关键, 是在 $n$ 为奇数时, 将 $4^n$ 看作单项式 $4 y^4$, 以利用代数式的分解\n$$\nx^4+4 y^4=\\left(x^2+2 y^2+2 x y\\right)\\left(x^2+2 y^2-2 x y\\right),\n$$\n产生数的适用的分解.", + "remark": "", + "figures": [] +} \ No newline at end of file diff --git a/processed_dataset/proof/0169.json b/processed_dataset/proof/0169.json new file mode 100644 index 0000000000000000000000000000000000000000..76a6dfc3ee5424b34cb6c1e99d541f2e97518060 --- /dev/null +++ b/processed_dataset/proof/0169.json @@ -0,0 +1,8 @@ +{ + "source_file": "./raw_volume-zh/volume10/chapter3.tex", + "problem_type": "proof", + "problem": "例3. 设正整数 $a 、 b 、 c 、 d$ 满足 $a b=c d$, 证明: $a+b+c+d$ 不是素数.", + "solution": "证明:一本题不宜用代数式的分解来产生所需的分解.\n我们的第一种解法是应用数的分解,指出 $a+b+c+d$ 的一个真因子.\n由 $a b=c d$, 可设 $\\frac{a}{c}=\\frac{d}{b}=\\frac{m}{n}$, 其中 $m$ 和 $n$ 是互素的正整数.\n由 $\\frac{a}{c}=\\frac{m}{n}$ 意味着有理数 $\\frac{a}{c}$ 的分子、分母约去了某个正整数 $u$ 后, 得到既约分数 $\\frac{m}{n}$, 因此\n$$\na=m u, c=n u . \\label{eq1}\n$$\n同理,有正整数 $v$,使得\n$$\nb=n v, d=m v . \\label{eq2}\n$$\n因此, $a+b+c+d=(m+n)(u+v)$ 是两个大于 1 的整数之积, 从而不是素数.", + "remark": "", + "figures": [] +} \ No newline at end of file diff --git a/processed_dataset/proof/0170.json b/processed_dataset/proof/0170.json new file mode 100644 index 0000000000000000000000000000000000000000..ff71e11a13b85b62ab9a998c2d665cc3bdf6e694 --- /dev/null +++ b/processed_dataset/proof/0170.json @@ -0,0 +1,8 @@ +{ + "source_file": "./raw_volume-zh/volume10/chapter3.tex", + "problem_type": "proof", + "problem": "例3. 设正整数 $a 、 b 、 c 、 d$ 满足 $a b=c d$, 证明: $a+b+c+d$ 不是素数.", + "solution": "证明二由 $a b=c d$, 得 $b=\\frac{c d}{a}$. 因此\n$$\na+b+c+d=a+\\frac{c d}{a}+c+d=\\frac{(a+c)(a+d)}{a} .\n$$\n因 $a+b+c+d$ 是整数,故 $\\frac{(a+c)(a+d)}{a}$ 也是整数.\n若它是一个素数,设为 $p$, 则由\n$$\n(a+c)(a+d)=a p \\label{eq3}\n$$\n可见, $p$ 整除 $(a+c)(a+d)$, 从而素数 $p$ 整除 $a+c$ 或 $a+d$. 不妨设 $p \\mid(a+c)$, 则 $a+c \\geqslant p$, 结合 式\\ref{eq3} 推出 $a+d \\leqslant a$, 而这不可能(因 $d \\geqslant 1$ ).", + "remark": "", + "figures": [] +} \ No newline at end of file diff --git a/processed_dataset/proof/0171.json b/processed_dataset/proof/0171.json new file mode 100644 index 0000000000000000000000000000000000000000..678b65140999c5d273b7afe98f50a647bdef8a2e --- /dev/null +++ b/processed_dataset/proof/0171.json @@ -0,0 +1,8 @@ +{ + "source_file": "./raw_volume-zh/volume10/chapter3.tex", + "problem_type": "proof", + "problem": "例4. 证明: 若整数 $a 、 b$ 满足 $2 a^2+a=3 b^2+b$, 则 $a-b$ 和 $2 a+2 b+1$ 都是完全平方数.", + "solution": "证明:已知关系式即为\n$$\n(a-b)(2 a+2 b+1)=b^2 . \\label{eq1}\n$$\n论证的第一个要点是证明整数 $a-b$ 与 $2 a+2 b+1$ 互素.\n记 $d=(a-b$, $2 a+2 b+1)$. 若 $d>1$, 则 $d$ 有素因子 $p$, 从而由 式\\ref{eq1} 知 $p \\mid b^2$. 因 $p$ 是素数, 故 $p \\mid b$. 结合 $p \\mid(a-b)$ 知 $p \\mid a$. 再由 $p \\mid(2 a+2 b+1)$ 导出 $p \\mid 1$, 这不可能, 故 $d=1$. 因此, 由于式\\ref{eq1}的右端为 $b^2$, 是一个完全平方数, 故 $|a-b|$ 与 $\\mid 2 a+2 b+ 1 \\mid$ 均是完全平方数 (参见第 2 单元的 (8)).\n现在证明 $a-b \\geqslant 0$, 从而由 式\\ref{eq1} 知 $2 a+2 b+1 \\geqslant 0$, 于是 $a-b$ 及 $2 a+2 b+$ 1 均是完全平方.\n假设有整数 $a, b$ 满足问题中的等式,但 $a-b<0$. 因已证明 $|a-b|$ 是一个完全平方数, 故有 $b-a=r^2$, 这里 $r>0$; 结合 式\\ref{eq1}推出 $r \\mid b$, 再由 $b-a=r^2$ 知 $r \\mid a$. 设 $b=b_1 r, a=a_1 r$, 代入问题中的等式可得到(注意 $r>0$ 及 $b_1= a_1+r$)\n$$\na_1^2+6 a_1 r+3 r^2+1=0 . \\label{eq2}\n$$\n为了证明上式不可能成立,可采用下面的办法:\n将 式\\ref{eq2} 看作是关于 $a_1$ 的二次方程, 由求根公式解得\n$$\na_1=-3 r \\pm \\sqrt{6 r^2-1} .\n$$\n因 $a_1$ 为整数,故由上式知 $6 r^2-1$ 为完全平方数.\n但易知一个完全平方数被 3 除得的余数只能为 0 或 1 ; 而 $6 r^2-1$ 被 3 除得的余数为 2 , 产生矛盾.\n或者更直接地: 由于 $a_1^2$ 被 3 除得的余数为 0 或 1 , 故 式\\ref{eq2} 左边被 3 除得的余数是 1 或 2 ; 但 式\\ref{eq2}的右边为 0 ,被 3 整除.\n矛盾.\n即 式\\ref{eq2}对任何整数 $a_1$ 及 $r$ 均不成立, 从而必须有 $a-b \\geqslant 0$, 这就证明了本题的结论.", + "remark": "注1 许多数论问题需证明一个正整数为 1 (例如, 证明整数的最大公约数是 1), 本单元的 (1) 给出了整数是否为 1 的一个数论刻画.\n由此, 我们常假设所说的数有一个素因子, 利用素数的锐利性质 (3) 作进一步论证, 以导出矛盾.\n注2 上述证明(2)不成立的论证, 实质上应用了同余(比较余数) 的想法, 这是证明两个整数不等的一种基本的手法.", + "figures": [] +} \ No newline at end of file diff --git a/processed_dataset/proof/0172.json b/processed_dataset/proof/0172.json new file mode 100644 index 0000000000000000000000000000000000000000..03cd9a27812091e90d0d355f69db648f8416cbbf --- /dev/null +++ b/processed_dataset/proof/0172.json @@ -0,0 +1,8 @@ +{ + "source_file": "./raw_volume-zh/volume10/chapter3.tex", + "problem_type": "proof", + "problem": "例5. 设 $n 、 a 、 b$ 是整数, $n>0$ 且 $a \\neq b$. 证明: 若 $n \\mid\\left(a^n-b^n\\right)$, 则 $n \\mid \\frac{a^n-b^n}{a-b}$.", + "solution": "证明:设 $p$ 是一个素数, 且 $p^\\alpha \\| n$. 我们来证明 $p^\\alpha \\mid \\frac{a^n-b^n}{a-b}$, 由此即导出本题的结论(参见下面的注).\n记 $t=a-b$, 若 $p \\nmid t$, 则 $\\left(p^\\alpha, t\\right)=1$. 因 $n \\mid\\left(a^n-b^n\\right)$, 故 $p^\\alpha \\mid\\left(a^n-b^n\\right)$. 又 $a^n-b^n=t \\cdot \\frac{a^n-b^n}{t}$, 于是 $p^\\alpha \\mid \\frac{a^n-b^n}{t}$.\n若 $p \\mid t$, 用二项式定理, 得\n$$\n\\frac{a^n-b^n}{t}=\\frac{(b+t)^n-b^n}{t}=\\sum_{i=1}^n \\mathrm{C}_n^i b^{n-i} t^{i-1} . \\label{eq1}\n$$\n设 $p^\\beta \\| i(i \\geqslant 1)$, 则 $2 \\beta \\leqslant p^\\beta \\leqslant i$, 由此易知 $\\beta \\leqslant i-1$. 因此 $\\mathrm{C}_n^i t^{i-1}=\\frac{n}{i} \\mathrm{C}_{n-1}^{i-1} t^{i-1}$ 中所含的 $p$ 的幕次至少是 $\\alpha-\\beta+(i-1) \\geqslant \\alpha$, 故 式\\ref{eq1} 右边和中每一项均被 $p^\\alpha$ 整除, 故 $p^\\alpha \\mid \\frac{a^n-b^n}{t}$, 即 $p^\\alpha \\mid \\frac{a^n-b^n}{a-b}$.", + "remark": "注:为了证明 $b \\mid a$, 可将 $b$ 作标准分解 $b=p_1^{\\alpha_1} p_2^{\\alpha_2} \\cdots p_k^{\\alpha_k}$, 进而将问题分解为证明 $p_i^{\\alpha_i} \\mid a(i=1,2, \\cdots, k)$ , 这样做的益处在于能够应用素数的锐利性质, 例 5 的论证清楚地表现了这一点.", + "figures": [] +} \ No newline at end of file diff --git a/processed_dataset/proof/0173.json b/processed_dataset/proof/0173.json new file mode 100644 index 0000000000000000000000000000000000000000..5b7ab8a118e044c59de8ee1a8345010f3010b12d --- /dev/null +++ b/processed_dataset/proof/0173.json @@ -0,0 +1,8 @@ +{ + "source_file": "./raw_volume-zh/volume10/chapter3.tex", + "problem_type": "proof", + "problem": "例6. 设 $m 、 n$ 是非负整数,证明: $\\frac{(2 m) !(2 n) !}{m ! n !(m+n) !}$ 是一个整数.", + "solution": "证明:我们只需证明, 对每个素数 $p$, 分母 $m ! n !(m+n) !$ 的标准分解中 $p$ 的幕次, 不超过分子 $(2 m) !(2 n)$ ! 中 $p$ 的幕次.\n由 (7) 中的公式可知, 这等价于证明\n$$\n\\sum_{l=1}^{\\infty}\\left(\\left[\\frac{2 m}{p^l}\\right]+\\left[\\frac{2 n}{p^l}\\right]\\right) \\geqslant \\sum_{l=1}^{\\infty}\\left(\\left[\\frac{m}{p^l}\\right]+\\left[\\frac{n}{p^l}\\right]+\\left[\\frac{m+n}{p^l}\\right]\\right) . \\label{eq1}\n$$\n事实上,我们能够证明下述更强的结果:\n对任意实数 $x 、 y$, 有\n$$\n[2 x]+[2 y] \\geqslant[x]+[y]+[x+y] . \\label{eq2}\n$$\n为了证明式\\ref{eq2}, 我们注意, 对任意整数 $k$ 及任意实数 $\\alpha$, 有 $[k+\\alpha]=[\\alpha]+k$. 由此易知, 若 $x$ 或 $y$ 改变一个整数量, 则不等式\\ref{eq2}两边改变一个相同的量.\n因此只要对 $0 \\leqslant x<1,0 \\leqslant y<1$ 的情形证明式\\ref{eq2}, 于是问题化为证明不等式\n$$\n[2 x]+[2 y] \\geqslant[x+y] \\text {. }\n$$\n注意现在 $0 \\leqslant[x+y] \\leqslant 1$. 若 $[x+y]=0$, 则结论显然成立.\n若 $[x+y]=$ 1 , 则 $x+y \\geqslant 1$, 从而 $x 、 y$ 中至少有一个大于或等于 $\\frac{1}{2}$, 不妨设 $x \\geqslant \\frac{1}{2}$, 因此\n$[2 x]+[2 y] \\geqslant[2 x]=1$, 这就证明了式\\ref{eq2}, 从而更有式\\ref{eq1}成立, 这就证明了本题的结论.", + "remark": "", + "figures": [] +} \ No newline at end of file diff --git a/processed_dataset/proof/0174.json b/processed_dataset/proof/0174.json new file mode 100644 index 0000000000000000000000000000000000000000..b415a7a0091059ea97aa91262e7d437193fffe32 --- /dev/null +++ b/processed_dataset/proof/0174.json @@ -0,0 +1,8 @@ +{ + "source_file": "./raw_volume-zh/volume10/chapter3.tex", + "problem_type": "proof", + "problem": "例7. 设 $m 、 n$ 是互素的正整数, 证明: $m ! n ! \\mid(m+n-1) !$.", + "solution": "证法一, 我们证明, 对每个素数 $p$, 有\n$$\n\\sum_{l=1}^{\\infty}\\left[\\frac{m+n-1}{p^l}\\right] \\geqslant \\sum_{l=1}^{\\infty}\\left(\\left[\\frac{m}{p^l}\\right]+\\left[\\frac{n}{p^l}\\right]\\right) . \\label{eq1}\n$$\n为此,我们(与上例相同地)希望证明\"单项不等式\":\n$$\n\\left[\\frac{m+n-1}{p^l}\\right] \\geqslant\\left[\\frac{m}{p^l}\\right]+\\left[\\frac{n}{p^l}\\right]. \\label{eq2}\n$$\n对任意素数 $p$ 及任意正整数 $l$ 成立, 从而式\\ref{eq1}得证.\n然而, 现在的情形下, 我们不能指望建立像例 6 中式\\ref{eq2}那样的对所有实数成立的结果来导出式\\ref{eq2}, 我们需要利用所说整数的特别性质:\n由带余除法, $m=p^l q_1+r_1, n=p^l q_2+r_2$, 这里 $0 \\leqslant r_1 、 r_21, b>1$. 由 $(a-1)+b=n, a+(b-1)=n$, 及归纳假设可见\n$$\n(a-1) ! b !|(a+b-1) !, a !(b-1) !|(a+b-1) !, \\label{eq3}\n$$\n我们又有\n$$\n(a+b) !=(a+b-1) ! \\cdot(a+b)=(a+b-1) ! \\cdot a+(a+b-1) ! \\cdot b . \\label{eq4}\n$$\n由式\\ref{eq3}易知 $a ! b !=a \\cdot(a-1) ! b !$ 整除 $(a+b-1) ! \\cdot a$. 同样 $a ! b$ ! 整除 $(a+b- 1) ! \\cdot b$, 故 $a ! b !$ 整除式\\ref{eq4}的右端, 从而 $a ! b ! \\mid(a+b)$ !, 即 $a+b=n+1$ 时结论也成立, 这就完成了归纳证明.", + "figures": [] +} \ No newline at end of file diff --git a/processed_dataset/proof/0176.json b/processed_dataset/proof/0176.json new file mode 100644 index 0000000000000000000000000000000000000000..ce41ecea14bf0f2569bd26c9ee8b51e0dd7492a5 --- /dev/null +++ b/processed_dataset/proof/0176.json @@ -0,0 +1,8 @@ +{ + "source_file": "./raw_volume-zh/volume10/chapter4.tex", + "problem_type": "proof", + "problem": "例3. 证明: 两个连续正整数之积不能是完全平方,也不能是完全立方.", + "solution": "证明:反证法,我们假设有正整数 $x, y$,使得\n$$\nx(x+1)=y^2 .\n$$\n将方程两边乘以 4 ,变形为 $(2 x+1)^2=4 y^2+1$, 这可分解为\n$$\n(2 x+1+2 y)(2 x+1-2 y)==1 .\n$$\n因左边两个因数都是正整数,故有\n$$\n\\left\\{\\begin{array}{l}\n2 x+1+2 y=1 \\\\\n2 x+1-2 y=1\n\\end{array}\\right.\n$$\n解得 $x=y=0$, 矛盾.\n这就证明了问题中的第一个断言.\n然而, 对于方程\n$$\nx(x+1)=y^3,\n$$\n上面的分解方法不易奏效.\n我们采用另一种 (基于数的性质的) 分解: 设所说的方程有正整数解 $x 、 y$,则由于 $x$ 和 $x+1$ 互素, 而它们的积是一个完全立方,故 $x$ 和 $x+1$ 都是正整数的立方, 即\n$$\nx=u^3, x+1=v^3, y=u v,\n$$\n$u, v$ 都是正整数,由此产生 $v^3-u^3=1$, 故\n$$\n(v-u)\\left(v^2+u v+u^2\\right)=1,\n$$\n这显然不可能.\n不难看到, 用类似的论证, 可证明连续两个正整数之积不会是整数的 $k$ 次幂 (这里 $k \\geqslant 2$ ).\n判明一个乘积中的各个因数互素往往非常重要,下面的例 4, 例 5 均是如此.", + "remark": "", + "figures": [] +} \ No newline at end of file diff --git a/processed_dataset/proof/0177.json b/processed_dataset/proof/0177.json new file mode 100644 index 0000000000000000000000000000000000000000..8c663c449ca59364e1be6768fa450a142d3c97df --- /dev/null +++ b/processed_dataset/proof/0177.json @@ -0,0 +1,8 @@ +{ + "source_file": "./raw_volume-zh/volume10/chapter4.tex", + "problem_type": "proof", + "problem": "例4. 证明: 方程\n$$\ny+y^2=x+x^2+x^3\n$$\n没有 $x \\neq 0$ 的整数解.", + "solution": "证明:设方程有 $x \\neq 0$ 的整数解, 将它分解为\n$$\n(y-x)(y+x+1)=x^3 . \\label{eq1}\n$$\n我们先证明 $(y-x, y+x+1)=1$. 若这不正确, 则有一个素数 $p$ 为 $y- x$ 与 $y+x+1$ 的一个公约数.\n由式\\ref{eq1}知 $p \\mid x^3$, 故素数 $p$ 整除 $x$, 结合 $p \\mid(y-x)$ 知 $p \\mid y$, 但 $p \\mid(x+y+1)$, 从而 $p \\mid 1$, 这不可能, 故式\\ref{eq1}的左边两因数互素.\n因式\\ref{eq1}的右边是一个完全立方, 从而有整数 $a 、 b$, 使得\n$$\ny-x=a^3, y+x+1=b^3, x=a b .\n$$\n消去 $x, y$ 得到\n$$\nb^3-a^3=2 a b+1 . \\label{eq2}\n$$\n现在证明方程式\\ref{eq2}无整数解, 由此便导出了矛盾.\n我们将式\\ref{eq2}分解为\n$$\n(b-a)\\left(b^2+a b+a^2\\right)=2 a b+1 . \\label{eq3}\n$$\n注意 $x=a b$ 而 $x \\neq 0$, 故 $a b \\neq 0$. 若 $a b>0$, 则由式\\ref{eq3}易知 $b-a>0$, 因 $a 、 b$ 为整数,故 $b-a \\geqslant 1$, 于是(3)的左边 $\\geqslant b^2+a b+a^2>3 a b>$ 右边; 若 $a b<0$, 则 $|b-a| \\geqslant 2$, 故式\\ref{eq3}的左边的绝对值 $\\geqslant 2\\left(a^2+b^2-|a b|\\right)>2|a b|$, 而式\\ref{eq3}的右边的绝对值 $<2|a b|$, 因此式\\ref{eq3}不能成立, 这就证明了问题中的方程没有 $x \\neq 0$ 的整数解.\n方程式\\ref{eq3}无解的论证, 采用了不等式估计 (左边的绝对值总大于右边的绝对值), 这就是所谓的估计法.\n(数论中的) 估计法往往需着眼于整数, 利用整数的各种性质产生适用的不等式.\n例如, 上述论证应用了整数的最基本的性质: 若整数 $x>0$, 则 $x \\geqslant 1$.", + "remark": "", + "figures": [] +} \ No newline at end of file diff --git a/processed_dataset/proof/0178.json b/processed_dataset/proof/0178.json new file mode 100644 index 0000000000000000000000000000000000000000..93760e9939046f1ec54641091c6d1ab0090f3032 --- /dev/null +++ b/processed_dataset/proof/0178.json @@ -0,0 +1,8 @@ +{ + "source_file": "./raw_volume-zh/volume10/chapter4.tex", + "problem_type": "proof", + "problem": "例5. 设 $k$ 是给定的正整数, $k \\geqslant 2$, 证明: 连续三个正整数的积不能是整数的 $k$ 次幂.", + "solution": "证明:假设有正整数 $x \\geqslant 2$ 及 $y$, 使得\n$$\n(x-1) x(x+1)=y^k . \\label{eq1}\n$$\n请注意上面左端的三个因数 $x-1 、 x 、 x+1$ 并非总两两互素, 因此不能由式\\ref{eq1}推出它们都是 $k$ 次方幕.\n克服这个困难的一种方法是将式\\ref{eq1}变形为\n$$\n\\left(x^2-1\\right) x=y^k . \\label{eq2}\n$$\n因 $x$ 和 $x^2-1$ 互素,故由式\\ref{eq2}推出,有正整数 $a 、 b$,使得\n$$\nx=a^k, x^2-1=b^k, a b=y,\n$$\n由此我们有\n$$\n\\begin{aligned}\n1 & =a^{2 k}-b^k=\\left(a^2\\right)^k-b^k \\\\\n& =\\left(a^2-b\\right)\\left(a^{2 k-2}+a^{2 k-4} b+\\cdots+a^2 b^{k-2}+b^{k-1}\\right),\n\\end{aligned}\n$$\n由于 $x \\geqslant 2$, 故 $a \\geqslant 2$, 又 $k \\geqslant 2$, 故上式后一个因数必大于 1 , 导出矛盾.", + "remark": "", + "figures": [] +} \ No newline at end of file diff --git a/processed_dataset/proof/0179.json b/processed_dataset/proof/0179.json new file mode 100644 index 0000000000000000000000000000000000000000..c680d69fb4aa651078e087c2def1e20bbe3e074c --- /dev/null +++ b/processed_dataset/proof/0179.json @@ -0,0 +1,8 @@ +{ + "source_file": "./raw_volume-zh/volume10/chapter4.tex", + "problem_type": "proof", + "problem": "例7. 设正整数 $x 、 y 、 z$ 满足 $2 x^x=y^y+z^z$, 则 $x=y=z$.", + "solution": "证明:首先, 将 $(x+1)^{x+1}$ 展开即知\n$$\n(x+1)^{x+1}>x^{x+1}+(x+1) x^x>2 x^x, \\label{eq1}\n$$\n由此可知 $y 、 z$ 必须均 $\\leqslant x$ : 因若 $y 、 z$ 中有大于 $x$ 的, 无妨设 $y>x$, 因 $y 、 x$ 为整数,故 $y \\geqslant x+1$, 从而\n$$\ny^y+z^z>y^y \\geqslant(x+1)^y \\geqslant(x+1)^{x+1}>2 x^x \\text { (应用 式\\ref{eq1}), }\n$$\n产生矛盾.\n因此 $y \\leqslant x, z \\leqslant x$, 故\n$$\ny^y+z^z \\leqslant x^x+x^x=2 x^x,\n$$\n结合原方程知, 必须有 $y=x$, 且 $z=x$, 故 $x=y=z$. 证毕.", + "remark": "", + "figures": [] +} \ No newline at end of file diff --git a/processed_dataset/proof/0180.json b/processed_dataset/proof/0180.json new file mode 100644 index 0000000000000000000000000000000000000000..c0e7ffcae41b50fe573d42dd340c59ca0829d71c --- /dev/null +++ b/processed_dataset/proof/0180.json @@ -0,0 +1,8 @@ +{ + "source_file": "./raw_volume-zh/volume10/chapter5.tex", + "problem_type": "proof", + "problem": "例1. 设 $m \\geqslant n \\geqslant 1$, 证明: $\\frac{(m, n)}{m} \\mathrm{C}_m^n$ 是整数.", + "solution": "证明:因 $\\frac{x}{m} \\mathrm{C}_m^n$ 在 $x=m$ 时为 $\\mathrm{C}_m^n$, 是一个整数; 在 $x=n$ 时, 它是 $\\frac{n}{m} \\frac{m}{n} \\mathrm{C}_{m-1}^{n-1}=\\mathrm{C}_{m-1}^{n-1}$, 也是整数.\n又由裴蜀等式知, 存在整数 $u 、 v$, 使得\n$$\n(m, n)=m u+n v,\n$$\n故 $\\frac{(m, n)}{m} \\mathrm{C}_m^n=u \\mathrm{C}_m^n+v \\frac{n}{m} \\mathrm{C}_m^n$ 是整数.", + "remark": "注:由例 1 推出, 若 $m 、 n$ 为互素的正整数,则 $m \\mid \\mathrm{C}_m^n$. 这一结论也可如下证明: 因 $\\mathrm{C}_m^n=\\frac{m}{n} \\mathrm{C}_{m-1}^{n-1}$, 故 $n \\mathrm{C}_m^n=m \\mathrm{C}_{m-1}^{n-1}$. 由于 $\\mathrm{C}_{m-1}^{n-1}$ 为整数, 故 $m \\mid n \\mathrm{C}_m^n$, 但 $(m$, $n)=1$, 从而 $m \\mid \\mathrm{C}_m^n$.\n特别地, 设 $p$ 是一个素数, 由于每个 $k=1, \\cdots, p-1$ 均与 $p$ 互素, 故我们有 $p \\mid \\mathrm{C}_p^k$, 对 $k=1, \\cdots, p-1$ 成立, 这一结论, 用处很多.", + "figures": [] +} \ No newline at end of file diff --git a/processed_dataset/proof/0181.json b/processed_dataset/proof/0181.json new file mode 100644 index 0000000000000000000000000000000000000000..1c94ecc6052fb6c53d36c7a0962f122039c9ecdc --- /dev/null +++ b/processed_dataset/proof/0181.json @@ -0,0 +1,8 @@ +{ + "source_file": "./raw_volume-zh/volume10/chapter5.tex", + "problem_type": "proof", + "problem": "例2. 设 $a 、 b$ 是两个不同的正整数, $a b(a+b)$ 是 $a^2+a b+b^2$ 的倍数.\n证明: $|a-b|>\\sqrt[3]{a b}$.", + "solution": "证明:由于 $a b(a+b)$ 被 $a^2+a b+b^2$ 整除,我们首先用 $a^2+a b+b^2$ 除 $a b(a+b)$, 得\n$$\na b(a+b)=\\left(a^2+a b+b^2\\right) a-a^3,\n$$\n故 $\\left(a^2+a b+b^2\\right) \\mid a^3$. 同样 $\\left(a^2+a b+b^2\\right) \\mid b^3$, 即 $a^2+a b+b^2$ 是 $a^3$ 与 $b^3$ 的一个公约数,故 $\\left(a^2+a b+b^2\\right) \\mid\\left(a^3, b^3\\right)$. (见第 2 单元中的 (3). ) 又 $\\left(a^3, b^3\\right)=(a$ , $b)^3$ (见下面的注), 从而\n$$\n\\left(a^2+a b+b^2\\right) \\mid(a, b)^3 . \\label{eq1}\n$$\n记 $d=(a, b), a=a_1 d, b=b_1 d$, 则式\\ref{eq1}成为 $\\left(a_1^2+a_1 b_1+b_1^2\\right) \\mid d$. 从而 $d \\geqslant a_1^2+a_1 b_1+b_1^2$, 更有 $d>a_1 b_1$. 因 $a \\neq b$, 故整数 $a_1 \\neq b_1$, 因此 $\\left|a_1-b_1\\right| \\geqslant$ 1 , 进而我们得出\n$$\n|a-b|^3=d^3\\left|a_1-b_1\\right|^3 \\geqslant d^3>d^2 a_1 b_1=a b,\n$$\n即 $|a-b|>\\sqrt[3]{a b}$.", + "remark": "注:对任意整数 $k \\geqslant 1$, 有 $\\left(a^k, b^k\\right)=(a, b)^k$. 这可如下证明: 当 $(a, b) \\doteq$ 1 时, 则 $\\left(a^k, b^k\\right)=1=(a, b)^k$ (见第 2 单元 $(6)$ ). 当 $(a, b)=d>1$ 时, 则有 $\\left(\\frac{a}{d}, \\frac{b}{d}\\right)=1$, 从而由上述结果知, $\\left(\\left(\\frac{a}{d}\\right)^k,\\left(\\frac{b}{d}\\right)^k\\right)=1$, 故 $d^k= d^k\\left(\\frac{a^k}{d^k}, \\frac{b^k}{d^k}\\right)=\\left(\\frac{a^k}{d^k} \\cdot d^k, \\frac{b^k}{d^k} \\cdot d^k\\right)=\\left(a^k, b^k\\right)$, 从而结论得证.\n这一论证, 是将一般情形的问题, 化为特殊情形来解决的一个简单例子.\n本题的证明, 先由整数的整除等性质导出整除关系式\\ref{eq1}, 再由此过渡到不等式, 这是处理涉及整数的不等式问题以及用估计法解决数论问题的一种基本手法,下面两个例子均是这样做的.", + "figures": [] +} \ No newline at end of file diff --git a/processed_dataset/proof/0182.json b/processed_dataset/proof/0182.json new file mode 100644 index 0000000000000000000000000000000000000000..b7af77c0e22e337697aa8540fd56d27f4a1e3c99 --- /dev/null +++ b/processed_dataset/proof/0182.json @@ -0,0 +1,8 @@ +{ + "source_file": "./raw_volume-zh/volume10/chapter5.tex", + "problem_type": "proof", + "problem": "例3. 在两个相邻的完全平方数 $n^2$ 与 $(n+1)^2$ 之间任取若干个不同整数, 证明它们中两两乘积互不相同.", + "solution": "证明:设整数 $a 、 b 、 c 、 d$ 满足 $n^2a$ 及 $c>a$, 得出 $q>p$ 及 $v>u$. 因 $p 、 q 、 u 、 v$ 都是整数, 故 $q \\geqslant p+1, v \\geqslant u+1$. 因此我们得出 (注意 $a=p u>n^2$ )\n$$\n\\begin{aligned}\n& d=q v \\geqslant(p+1)(u+1)=p u+(p+u)+1 \\\\\n& >n^2+2 \\sqrt{p u}+1>n^2+2 n+1=(n+1)^2,\n\\end{aligned}\n$$\n矛盾.", + "remark": "", + "figures": [] +} \ No newline at end of file diff --git a/processed_dataset/proof/0183.json b/processed_dataset/proof/0183.json new file mode 100644 index 0000000000000000000000000000000000000000..ef4c1a7b012dc4a50afc669e7b5f0084ef3921c2 --- /dev/null +++ b/processed_dataset/proof/0183.json @@ -0,0 +1,8 @@ +{ + "source_file": "./raw_volume-zh/volume10/chapter5.tex", + "problem_type": "proof", + "problem": "例6. 求出具有下述性质的正整数 $n$ : 它被 $\\leqslant \\sqrt{n}$ 的所有正整数整除.", + "solution": "解:法一我们首先证明, 每个正整数 $n$ 可唯一地表示为形式\n$$\nn=q^2+r, 0 \\leqslant r \\leqslant 2 q. \\label{eq1}\n$$\n这是因为任意正整数 $n$ 必介于两个相邻的平方数之间, 即有正整数 $q$, 使得 $q^2 \\leqslant n<(q+1)^2$. 令 $r=n-q^2$, 则 $r \\geqslant 0$, 又 $r<(q+1)^2-q^2=2 q+1$, 故整数 $r \\leqslant 2 q$, 从而 $n$ 有形如式\\ref{eq1}的表示.\n另一方面, 若 $n$ 可表示为式\\ref{eq1}的形式, 则易知 $q^2 \\leqslant n<(q+1)^2$, 故 $q= [\\sqrt{n}]$, 由此即知 $q$ 被 $n$ 唯一确定, 相应的 $r$ 因此也被确定.\n利用式\\ref{eq1}便不难解决例 6. 因已知 $q=[\\sqrt{n}]$ 整除 $n$, 结合式\\ref{eq1}知 $q \\mid r$, 故 $r=0$ 、 $q$ 或 $2 q$, 即 $n$ 具有形式\n$$\nn=q^2, q^2+q, q^2+2 q .\n$$\n$n=1,2,3$ 显然合要求.\n设 $n>3$, 则 $q=[\\sqrt{n}] \\geqslant 2$, 故由已知条件知\n$(q-1) \\mid n$. 若 $n=q^2$, 由\n$$\nq^2=q(q-1)+q \\quad \\text { 及 } \\quad(q-1, q)=1\n$$\n可见,必须 $q-1=1$, 即 $q=2$, 所以 $n=4$.\n同样, 若 $n=q^2+q$, 则 $q=2,3$, 从而 $n=6,12$; 若 $n=q^2+2 q$, 则 $q=$ 2 或 4 , 相应地 $n=8,24$. 因此, $n$ 只可能是 $1,2,3,4,6,8,12,24$, 经检验它们均符合要求.", + "remark": "", + "figures": [] +} \ No newline at end of file diff --git a/processed_dataset/proof/0184.json b/processed_dataset/proof/0184.json new file mode 100644 index 0000000000000000000000000000000000000000..880e1abc13df5c72563bc4daa72c24cf9ec2cf7b --- /dev/null +++ b/processed_dataset/proof/0184.json @@ -0,0 +1,8 @@ +{ + "source_file": "./raw_volume-zh/volume10/chapter5.tex", + "problem_type": "proof", + "problem": "例6. 求出具有下述性质的正整数 $n$ : 它被 $\\leqslant \\sqrt{n}$ 的所有正整数整除.", + "solution": "解法二设 $q=[\\sqrt{n}]$, 我们证明 $q \\geqslant 6$ 时没有符合要求的 $n$. 反证法, 假设有这样的 $n$, 我们将利用 $q 、 q-1 、 q-2$ 均整除 $n$ 来产生矛盾.\n因为 $q$ 与 $q-2$ 整除 $n$, 故 $[q, q-2] \\mid n$, 即 $\\frac{q(q-2)}{(q, q-2)} \\mid n$ (见第 2 单元 (10)). 又 $q-1 \\mid n$, 故 $q-1$ 与 $\\frac{q(q-2)}{(q, q-2)}$ 的最小公倍数 $D$ 整除 $n$. 但 $q-1$ 与 $q$ 及 $q-2$ 均互素, 故 $q-1$ 与 $q(q-2)$ 互素, 从而 $D=(q-1) \\cdot \\frac{q(q-2)}{(q, q-2)}$. 因此\n$$\n\\frac{q(q-1)(q-2)}{(q, q-2)} \\leqslant n\n$$\n但显然 $(q, q-2) \\leqslant 2$, 故\n$$\nq(q-1)(q-2) \\leqslant 2 n .\n$$\n注意 $\\sqrt{n}0$, 矛盾.\n因此 $q \\geqslant 6$ 时无解.\n而当 $q \\leqslant 5$ 时易通过逐一检验求出所有符合要求的 $n: 1,2$, $3,4,6,8,12$ 及 24 .", + "remark": "", + "figures": [] +} \ No newline at end of file diff --git a/processed_dataset/proof/0185.json b/processed_dataset/proof/0185.json new file mode 100644 index 0000000000000000000000000000000000000000..5a9c7bfa2e971f9b44ce3e8fd9270759d69f137b --- /dev/null +++ b/processed_dataset/proof/0185.json @@ -0,0 +1,8 @@ +{ + "source_file": "./raw_volume-zh/volume10/chapter5.tex", + "problem_type": "proof", + "problem": "例7. 证明: 从 $1,2, \\cdots, 100$ 中任意取出 51 个数,其中必有两个数互素.", + "solution": "证明:问题点破了极为简单: 我们从 $1,2, \\cdots, 100$ 中依次取相邻的两个数,配成下面 50 个数对\n$$\n\\{1,2\\},\\{3,4\\}, \\cdots,\\{99,100\\},\n$$\n则任意取出的 51 个数必然包含了上述数对中的某一对, 因这两数相邻, 它们当然互素.", + "remark": "", + "figures": [] +} \ No newline at end of file diff --git a/processed_dataset/proof/0186.json b/processed_dataset/proof/0186.json new file mode 100644 index 0000000000000000000000000000000000000000..dd38adf8c9e4d84c2a71d42d401bb84041d926d6 --- /dev/null +++ b/processed_dataset/proof/0186.json @@ -0,0 +1,8 @@ +{ + "source_file": "./raw_volume-zh/volume10/chapter5.tex", + "problem_type": "proof", + "problem": "例8. 证明: 存在连续 1000 个正整数,其中恰有 10 个素数.", + "solution": "证明:这一证明的基础是习题 3 第 1 题, 由这结论可知, 存在连续 1000 个正整数\n$$\na, a+1, \\cdots, a+999, \\label{eq1}\n$$\n其中每个数都不是素数.\n现将式\\ref{eq1}中的数施行如下操作: 删去式\\ref{eq1}中最右边的 $a+999$, 而在最左边添上 $a-1$. 显然,所得的数列\n$$\na-1, a, \\cdots, a+998\n$$\n中至多有一个素数.\n重复这一手续, 直至达到 $1,2, \\cdots, 1000$ 后停止.\n我们注意, 一次操作后所得的 (连续 1000 个) 正整数中的素数个数, 与操作前的 1000 个正整数中的素数个数相比, 或相等, 或增、减 1 . 而最终得到的数 $1,2, \\cdots$, 1000 中, 显然有多于 10 个素数, 因此, 上述操作过程中, 必有一次所产生的 1000 个连续整数中恰包含 10 个素数.", + "remark": "例 7 和例 8 都是所谓的\"存在性问题\",即证明存在\"某事物\"具有\"某种性质\". 这里的论证并未实际地构造出符合要求的事物, 而是用逻辑的力量表明了它们的存在.\n例 7 应用了众所周知的 \"抽庶原理\", 例 8 则应用了一述的原则, 这有时被称作\"离散的零点定理\":\n设 $f(n)$ 为一个定义在 (正) 整数集上的函数, 取值也为整数.\n若对所有 $n$ 有 $|f(n)-f(n+1)| \\leqslant 1$, 并且存在整数 $a$ 及 $b$, 使得 $f(a) f(b)<0$, 则在数 $a 、 b$ 之间必有一整数 $c$, 使 $f(c)=0$. (例 8 中, 我们可取 $g(n)$ 为从 $n$ 开始的连续 1000 个正整数中素数的个数,而取 $f(n)=g(n)-10$. )\n处理存在性问题的另一种有效的方法是所谓的构造法, 即实际地造出符合要求的事物.\n构造法是一种重要的数学方法, 灵活多样.\n数论中有许多问题可以(甚至必须)用构造法来论证.\n我们举几个这样的例子.", + "figures": [] +} \ No newline at end of file diff --git a/processed_dataset/proof/0187.json b/processed_dataset/proof/0187.json new file mode 100644 index 0000000000000000000000000000000000000000..1ef024a87dff2cf6cc6eaa1c7f216f5f4c54d991 --- /dev/null +++ b/processed_dataset/proof/0187.json @@ -0,0 +1,8 @@ +{ + "source_file": "./raw_volume-zh/volume10/chapter5.tex", + "problem_type": "proof", + "problem": "例9. 若一个正整数的标准分解中, 每个素约数的幂次都大于 1 , 则称它为幂数.\n证明: 存在无穷多个互不相同的正整数, 它们及它们中任意多个不同数的和都不是幂数.", + "solution": "证明:设 $2=p_11)$ 都被 3 整除.", + "remark": "", + "figures": [] +} \ No newline at end of file diff --git a/processed_dataset/proof/0190.json b/processed_dataset/proof/0190.json new file mode 100644 index 0000000000000000000000000000000000000000..6468578575024d0bce2ea34b48a29fb95406ec26 --- /dev/null +++ b/processed_dataset/proof/0190.json @@ -0,0 +1,8 @@ +{ + "source_file": "./raw_volume-zh/volume10/chapter5.tex", + "problem_type": "proof", + "problem": "例11. 证明: 有无穷多个正整数 $n$, 满足 $n \\mid\\left(2^n+2\\right)$.", + "solution": "证明:我们仍采用归纳构造法, 其中的关键一着是加强归纳假设.\n面证明: 若 $n$ 满足\n$$\n2|n, n|\\left(2^n+2\\right),(n-1) \\mid\\left(2^n+1\\right), \\label{eq1}\n$$\n则对于 $m=2^n+2$, 有\n$$\n2|m, m|\\left(2^m+2\\right),(m-1) \\mid\\left(2^m+1\\right) . \\label{eq2}\n$$\n事实上, 由于 $2^n+2=2\\left(2^{n-1}+1\\right)$ 是奇数的 2 倍及 $2 \\mid n$, 故 $2^n+2=n k$ 中的整数 $k$ 是一个奇数, 所以\n$$\n2^m+1=2^{n k}+1=\\left(2^n\\right)^k+1\n$$\n是 $2^n+1=m-1$ 的倍数.\n同样, 从 $2^n+1=(n-1) l$ 知 $l$ 为奇数, 故\n$$\n2^m+2=2\\left(2^{m-1}+1\\right)=2\\left(\\left(2^{n-1}\\right)^l+1\\right)\n$$\n为 $2\\left(2^{n-1}+1\\right)=2^n+2=m$ 的倍数.\n又 $m=2^n+2$ 显然为偶数, 故上述的断言得到了证明.\n现在, 由于 $n=2$ 满足 式\\ref{eq1}, 于是用 式\\ref{eq2} 便递推地构造出无穷多个符合要求的数: $2,6,66, \\cdots$.\n我们注意, 式\\ref{eq1}中的 $2 \\mid n$ 是必要的, 即满足本题要求的数都是偶数.\n因为若有奇数 $n>1$, 适合 $n \\mid\\left(2^n+2\\right)$, 则 $n \\mid\\left(2^{n-1}+1\\right)$, 这将与第 8 单元例 3 的结论相违.", + "remark": "", + "figures": [] +} \ No newline at end of file diff --git a/processed_dataset/proof/0191.json b/processed_dataset/proof/0191.json new file mode 100644 index 0000000000000000000000000000000000000000..17ab877fc1cba6c12e7824602b0179402d9d73bf --- /dev/null +++ b/processed_dataset/proof/0191.json @@ -0,0 +1,8 @@ +{ + "source_file": "./raw_volume-zh/volume10/chapter6.tex", + "problem_type": "proof", + "problem": "例1. 设 $a 、 b 、 c 、 d$ 为正整数,证明: $a^{4 b+d}-a^{4 c+d}$ 被 240 整除.", + "solution": "证明:由于 $240=2^4 \\times 3 \\times 5$, 我们将分别证明 $a^{4 b+d}-a^{4 c+d}$ 被 3、5、16 整除,由此便证得了结论 (参见第 3 单元例 5 的注).\n首先证明 $3 \\mid\\left(a^{4 b+d}-a^{4 c+d}\\right)$. 由(12)中的结果 $a^2 \\equiv 0,1(\\bmod 3)$, 可知 $a^{4 b} \\equiv a^{4 c} \\equiv 0,1(\\bmod 3)$. 从而\n$$\na^{4 b+d}-a^{4 c+d}=a^d\\left(a^{4 b}-a^{4 c}\\right) \\equiv 0(\\bmod 3) .\n$$\n类似地, 由 $a^2 \\equiv 0, \\pm 1(\\bmod 5)$, 可知 $a^4 \\equiv 0,1(\\bmod 5)$, 从而 $a^{4 b} \\equiv a^{4 c} \\equiv 0,1(\\bmod 5)$. 于是 $a^{4 b+d}-a^{4 c+d} \\equiv 0(\\bmod 5)$.\n最后, 由 $a^4 \\equiv 0,1(\\bmod 16)$, 可知 $a^{4 b} \\equiv a^{4 c} \\equiv 0,1(\\bmod 16)$, 故 $a^{4 b+d}- a^{4 c+d} \\equiv 0(\\bmod 16)$. 这就证明了我们的结论.", + "remark": "", + "figures": [] +} \ No newline at end of file diff --git a/processed_dataset/proof/0192.json b/processed_dataset/proof/0192.json new file mode 100644 index 0000000000000000000000000000000000000000..f8a17a9df1df98edac90a963c3b1f286a9f99e74 --- /dev/null +++ b/processed_dataset/proof/0192.json @@ -0,0 +1,8 @@ +{ + "source_file": "./raw_volume-zh/volume10/chapter6.tex", + "problem_type": "proof", + "problem": "例2. 设整数 $a 、 b 、 c$ 满足 $a+b+c=0$, 记 $d=a^{1999}+b^{1999}+c^{1999}$. 证明: $|d|$ 不是素数.", + "solution": "证明:本题有好几种解法,这里我们采用同余来证明: $|d|$ 有一个非平凡的固定约数.\n首先, 对任意整数 $u$, 数 $u^{1999}$ 与 $u$ 的奇偶性相同, 即 $u^{1999} \\equiv u(\\bmod 2)$, 故 $d \\equiv a+b+c \\equiv 0(\\bmod 2)$, 即 $2 \\mid d$.\n此外, 对任意整数 $u$, 易于验证(区分 $3 \\mid u$ 及 $3 \\nmid u$ )\n$$\nu^3 \\equiv u(\\bmod 3) \\text {. } \\label{eq1}\n$$\n由此推出\n$$\n\\begin{aligned}\nu^{1999}=u \\cdot u^{1998} & \\equiv u \\cdot u^{666} \\equiv u \\cdot u^{222} \\equiv u^{75} \\\\\n& \\equiv u^{25} \\equiv u^9 \\equiv u^3 \\equiv u(\\bmod 3) .\n\\end{aligned}\n$$\n因此 $d \\equiv a+b+c \\equiv 0(\\bmod 3)$. 故 $6 \\mid d$, 从而 $d$ 不是素数.", + "remark": "注:解答中的同余式(1)是著名的费马小定理的特殊情形, 请参见下一单元.", + "figures": [] +} \ No newline at end of file diff --git a/processed_dataset/proof/0193.json b/processed_dataset/proof/0193.json new file mode 100644 index 0000000000000000000000000000000000000000..66bf68e8df1f2d4e84a185a9ba33b9a3ff807e46 --- /dev/null +++ b/processed_dataset/proof/0193.json @@ -0,0 +1,8 @@ +{ + "source_file": "./raw_volume-zh/volume10/chapter6.tex", + "problem_type": "proof", + "problem": "例3. 设整数 $x 、 y 、 z$ 满足\n$$\n(x-y)(y-z)(z-x)=x+y+z, \\label{eq1}\n$$", + "solution": "证明: $x+y+z$ 被 27 整除.\n证明,我们将由 式\\ref{eq1} 推出, $x 、 y 、 z$ 必须两两模 3 同余, 从而 $27 \\mid(x-y) (y-z)(z-x)$, 故由式\\ref{eq1} 知 $27 \\mid(x+y+z)$.\n反证法, 首先设 $x 、 y 、 z$ 中恰有两个数模 3 同余, 无妨设 $x \\equiv y(\\bmod 3)$, 但 $x \\neq z(\\bmod 3)$. 此时 $3 \\mid(x-y)$, 而 $3 \\nmid(x+y+z)$, 于是 式\\ref{eq1} 的左边 $\\equiv 0(\\bmod 3)$, 但右边 $\\neq \\equiv 0(\\bmod 3)$, 矛盾.\n故这种情形不会出现.\n其次设 $x 、 y 、 z$ 模 3 的余数互不相同, 此时易知 $3 \\mid(x+y+z)$, 但 $3 \\nmid(x-y)(y-z)(z-x)$, 从而 式\\ref{eq1} 两边模 3 的余数不同, 矛盾.\n即这种情形也不能出现.\n因此, 我们前述的断言正确, 即证明了本题的结论.", + "remark": "注:解法体现了应用同余处理数论问题的一个基本原则: 若整数 $A=0$, 则 $A$ 被任何正整数 $n(n>1)$ 除得的余数必然是 0 . 因此, 若能找到某一个 $n>1$, 使 $A$ 模 $n$ 不为 0 , 则整数 $A$ 决不能是 0 . 我们常基于这一原则, 用同余导出某种必要条件,或产生结果(如例 3), 或为进一步论证作准备, 本书的后面还有许多这样的例子.", + "figures": [] +} \ No newline at end of file diff --git a/processed_dataset/proof/0194.json b/processed_dataset/proof/0194.json new file mode 100644 index 0000000000000000000000000000000000000000..f3e2bcf820ff20207c88f7505d922925aa697889 --- /dev/null +++ b/processed_dataset/proof/0194.json @@ -0,0 +1,8 @@ +{ + "source_file": "./raw_volume-zh/volume10/chapter6.tex", + "problem_type": "proof", + "problem": "例4. 设 $n>1$, 证明: $\\underbrace{11 \\cdots 1}_{n \\text { 个1 } 1}$ 不是完全平方数.", + "solution": "证明:反证法,设有某个 $n>1$ 及整数 $x$,使得\n$$\n\\underbrace{11 \\cdots 1}_{n \\text { 个1 }}=x^2 . \\label{eq1}\n$$\n由式\\ref{eq1}可知 $x$ 是奇数 (实际上是将式\\ref{eq1}模 2 , 注意 $x^2 \\equiv x(\\bmod 2)$ ). 进一步, 因 $2 \\nmid x$, 故 $x^2 \\equiv 1(\\bmod 4)$. 但\n$$\n\\underbrace{11 \\cdots 1}_{n \\uparrow 1}-1=\\underbrace{11 \\cdots 10}_{n-1 \\uparrow 1}\n$$\n只能被 2 整除, 而不被 4 整除, 即式\\ref{eq1}的左边非 $(\\bmod 4)$,矛盾!\n用同余处理问题,关键在于选择模.\n但究竟怎样选择, 却并无简单的规则可循, 得视具体问题而定.\n在例 4 中, 我们先将式\\ref{eq1}模 2 , 虽不能解决问题, 但基于此得出的信息进一步模 4 , 则导出了矛盾.", + "remark": "", + "figures": [] +} \ No newline at end of file diff --git a/processed_dataset/proof/0195.json b/processed_dataset/proof/0195.json new file mode 100644 index 0000000000000000000000000000000000000000..bcfed0fec8bdaaf439ef8e5f783a026011135460 --- /dev/null +++ b/processed_dataset/proof/0195.json @@ -0,0 +1,8 @@ +{ + "source_file": "./raw_volume-zh/volume10/chapter6.tex", + "problem_type": "proof", + "problem": "例5. 用数码 $1 、 2 、 3 、 4 、 5 、 6 、 7$ 作七位数, 每个数码恰用一次.\n证明: 这些七位数中没有一个是另一个的倍数.", + "solution": "证明:假设有这样两个七位数 $a, b(a \\neq b)$ 使得\n$$\na=b c, \\label{eq1}\n$$\n其中 $c$ 为大于 1 的整数.\n由于 $a 、 b$ 的数码之和均是 $1+2+3+4+5+6+ 7 \\equiv 1(\\bmod 9)$, 故 $a \\equiv b \\equiv 1(\\bmod 9)$ . 现在将式\\ref{eq1}模 9 , 得出 $c \\equiv 1(\\bmod 9)$. 但 $c>1$, 故 $c \\geqslant 10$, 这样, $a \\geqslant 10 b>10^7$, 与 $a$ 是七位数矛盾.", + "remark": "", + "figures": [] +} \ No newline at end of file diff --git a/processed_dataset/proof/0196.json b/processed_dataset/proof/0196.json new file mode 100644 index 0000000000000000000000000000000000000000..e9c5cb3dc8617ccc5ee8536d68c210f3ec13be6e --- /dev/null +++ b/processed_dataset/proof/0196.json @@ -0,0 +1,8 @@ +{ + "source_file": "./raw_volume-zh/volume10/chapter6.tex", + "problem_type": "proof", + "problem": "例6. 数列 $\\left\\{x_n\\right\\}$ 为 $1,3,5,11, \\cdots$ 满足递推关系\n$$\nx_{n+1}=x_n+2 x_{n-1}, n \\geqslant 2 . \\label{eq1}\n$$\n数列 $\\left\\{y_n\\right\\}$ 为 $7,17,55,161, \\cdots$ 满足递推关系\n$$\ny_{n+1}=2 y_n+3 y_{n-1}, n \\geqslant 2 . \\label{eq2}\n$$", + "solution": "证明: 这两个数列没有相同的项.\n证明考虑以 8 为模.\n首先证明,数列 $\\left\\{x_n\\right\\}$ 模 8 后是一个周期数列\n$$\n1,3,5,3,5, \\cdots \\text {. } \\label{eq3}\n$$\n因为 $x_2 \\equiv 3, x_3 \\equiv 5(\\bmod 8)$. 若已有\n$$\nx_{n-1} \\equiv 3, x_n \\equiv 5(\\bmod 8) \\text {, }\n$$\n则由递推公式\\ref{eq1}, 得\n$$\n\\begin{aligned}\n& x_{n+1}=x_n+2 x_{n-1} \\equiv 5+2 \\times 3 \\equiv 3(\\bmod 8), \\\\\n& x_{n+2}=x_{n+1}+2 x_n \\equiv 3+2 \\times 5 \\equiv 5(\\bmod 8),\n\\end{aligned}\n$$\n这就归纳证明了我们的断言.\n同样由式\\ref{eq2}可证明,数列 $\\left\\{y_n\\right\\}$ 模 8 后成为周期数列\n$$\n7,1,7,1,7,1, \\cdots . \\label{eq4}\n$$\n由式\\ref{eq3}、\\ref{eq4}可见,两个数列 $x_2, x_3, \\cdots$ 与 $y_1, y_2, \\cdots$ 模 8 后无相同项, 故这两个数列无相同项.\n又因为 $\\left\\{y_n\\right\\}$ 是递增的, 所以 $y_1, y_2, \\cdots$ 决不会等于 $x_1=1$, 这就证明了 $\\left\\{x_n\\right\\}$ 与 $\\left\\{y_n\\right\\}$ 无相同项.", + "remark": "注1 易知 $\\left\\{x_n\\right\\}$ 与 $\\left\\{y_n\\right\\}$ 模 3 后分别成周期数列:\n$$\n1,0,2,2,0,1,1,0,2,2, \\cdots \\text {; 及 } 1,2,1,2, \\cdots\n$$\n两者有无穷多对项相等,因此模 3 不能解决问题.\n同样可知, 模 4 也不能解决问题.\n注2 线性递推数列式\\ref{eq1}和\\ref{eq2}模 8 后成为周期数列, 这一点并非偶然.\n实际上,给定 $m>1$, 若 $\\left\\{x_n\\right\\}(n \\geqslant 1)$ 是由递推公式\n$$\nx_{n+k}=f\\left(x_{n+k-1}, \\cdots, x_{n+1}, x_n\\right)\n$$\n确定的整数数列, 其中 $f$ 是 $k$ 元整系数多项式, 初值 $x_1, x_2, \\cdots, x_k$ 为给定整数, 则 $\\left\\{x_n\\right\\}$ 模 $m$ 后终将成为周期数列.\n为证明这一结论, 我们用 $\\bar{x}_i$ 表示 $x_i$ 被 $m$ 除得的余数 $\\left(0 \\leqslant \\bar{x}_i0$, 则 $(2 p)^{2 m}-(2 p-1)^n \\geqslant 4 p-1$, 且在 $m=1, n=2$ 时取得等号, 这就证明了我们的结论.", + "remark": "", + "figures": [] +} \ No newline at end of file diff --git a/processed_dataset/proof/0198.json b/processed_dataset/proof/0198.json new file mode 100644 index 0000000000000000000000000000000000000000..d0d0735d451eab5618ceb8491701615c003c65c5 --- /dev/null +++ b/processed_dataset/proof/0198.json @@ -0,0 +1,8 @@ +{ + "source_file": "./raw_volume-zh/volume10/chapter6.tex", + "problem_type": "proof", + "problem": "例8. 连结正 $n$ 边形的顶点, 得到一个闭的 $n$ 一折线.\n证明: 若 $n$ 为偶数, 则在连线中有两条平行线; 若 $n$ 为奇数,连线中不可能恰有两条平行线.", + "solution": "证明:这是一个不宜用几何方法解决的几何问题, 它与模 $n$ 的完全剩余系有关.\n依逆时针顺序将顶点标上数 $0,1, \\cdots, n-1$. 设问题中的闭折线为 $a_0 \\rightarrow a_1 \\rightarrow \\cdots \\rightarrow a_{n-1} \\rightarrow a_n=a_0$, 这里 $a_0, a_1, \\cdots, a_{n-1}$ 是 $0,1, \\cdots, n-1$ 的一个排列.\n首先, 由诸 $a_i$ 是正 $n$ 边形的顶点易知\n$$\n\\begin{aligned}\n& a_i a_{i+1} / / a_j a_{j+1} \\Leftrightarrow \\widetilde{a_{i+1} a_j}=\\widehat{a_{j+1} a_i} \\\\\n& \\Leftrightarrow a_i+a_{i+1} \\equiv a_j+a_{j+1}(\\bmod n) .\n\\end{aligned}\n$$\n当 $n$ 为偶数时, $2 \\nmid(n-1)$, 故模 $n$ 的任一完系之和 $\\equiv 0+1+\\cdots+(n-1)= \\frac{n(n-1)}{2} \\not \\equiv 0(\\bmod n)$.\n但另一方面,我们总有\n$$\n\\begin{aligned}\n\\sum_{i=0}^{n-1}\\left(a_i+a_{i+1}\\right) & =\\sum_{i=0}^{n-1} a_i+\\sum_{i=0}^{n-1} a_{i+1}=2 \\sum_{i=0}^{n-1} a_i=2 \\times \\frac{n(n-1)}{2} \\\\\n& =n(n-1) \\equiv 0(\\bmod n) . \\label{eq1}\n\\end{aligned}\n$$\n所以 $a_i+a_{i+1}(i=0,1, \\cdots, n-1)$ 不能构成模 $n$ 的完全剩余系, 即必有 $i \\neq j(0 \\leqslant i, j \\leqslant n-1)$, 使得\n$$\na_i+a_{i+1} \\equiv a_j+a_{j+1}(\\bmod n),\n$$\n因而必有一对边 $a_i a_{i+1} / / a_j a_{j+1}$.\n当 $n$ 为奇数时, 若恰有一对边 $a_i a_{i+1} / / a_j a_{j+1}$, 则 $n$ 个数 $a_0+a_1, a_1+ a_2, \\cdots, a_{n-1}+a_0$ 之中恰有一个剩余类 $r$ 出现两次, 从而也恰缺少一个剩余类 $s$, 于是 (这时 $2 \\mid(n-1)$ )\n$$\n\\begin{aligned}\n\\sum_{i=0}^{n-1}\\left(a_i+a_{i+1}\\right) & \\equiv 0+1+\\cdots+(n-1)+r-s=\\frac{n(n-1)}{2}+r-s \\\\\n& \\equiv r-s(\\bmod n) .\n\\end{aligned}\n$$\n结合式\\ref{eq1}得 $r \\equiv s(\\bmod n)$, 矛盾! 这表明在 $n$ 为奇数时, 不可能恰有一对边平行.", + "remark": "", + "figures": [] +} \ No newline at end of file diff --git a/processed_dataset/proof/0199.json b/processed_dataset/proof/0199.json new file mode 100644 index 0000000000000000000000000000000000000000..bb51a574a4cb2dfa0581a3571f9cc58d46e0ce48 --- /dev/null +++ b/processed_dataset/proof/0199.json @@ -0,0 +1,8 @@ +{ + "source_file": "./raw_volume-zh/volume10/chapter6.tex", + "problem_type": "proof", + "problem": "例9. 设 $n>3$ 是奇数,证明: 将 $n$ 元集合 $S=\\{0,1, \\cdots, n-1\\}$ 任意去掉一个元素后, 总可以将剩下的元素分成两组, 每组 $\\frac{n-1}{2}$ 个数, 使两组的和模 $n$ 同余.", + "solution": "证明:论证的一个关键是, 对任意 $x \\in S, x \\neq 0$, 集合 $S \\backslash\\{x\\}$ 可以从 $T= \\{1,2, \\cdots, n-1\\}$ 作变换\n$$\nT+x(\\bmod n)=\\{a+x(\\bmod n), a \\in T\\}\n$$\n得到.\n这就将问题化归为证明其特殊情形: $T=S \\backslash\\{0\\}$ 可以分成两组, 每组 $\\frac{n-1}{2}$ 个数,使两组的和模 $n$ 同余.\n我们区分两种情况.\n当 $n=4 k+1(k \\geqslant 1)$ 时, 注意 $2 k$ 个数对\n$$\n\\{1,4 k\\},\\{2,4 k-1\\}, \\cdots,\\{2 k, 2 k+1\\}\n$$\n中, 每对的和模 $n$ 均为 0 , 于是任取 $k$ 个数对作成一集, 剩下的 $k$ 对数作另一集便符合要求.\n若 $n=4 k+3(k \\geqslant 1)$, 我们先取 $1 、 2 、 4 k$ 于一集, $3 、 4 k+1 、 4 k+2$ 于另一集, 然后将剩下的 $2 k-2$ 个数对\n$$\n\\{4,4 k-1\\}, \\cdots,\\{2 k+1,2 k+2\\}\n$$\n各取 $k-1$ 对分置上述两集即可.", + "remark": "", + "figures": [] +} \ No newline at end of file diff --git a/processed_dataset/proof/0200.json b/processed_dataset/proof/0200.json new file mode 100644 index 0000000000000000000000000000000000000000..f4b586c9b18e4eb05a6fabb1f66810699a4a1391 --- /dev/null +++ b/processed_dataset/proof/0200.json @@ -0,0 +1,8 @@ +{ + "source_file": "./raw_volume-zh/volume10/chapter6.tex", + "problem_type": "proof", + "problem": "例10. 证明: 对任意整数 $n \\geqslant 4$, 存在一个 $n$ 次多项式\n$$\nf(x)=x^n+a_{n-1} x^{n-1}+\\cdots+a_1 x+a_0,\n$$\n具有下述性质:\n(1) $a_0, a_1, \\cdots, a_{n-1}$ 均为正整数;\n(2) 对任意正整数 $m$,及任意 $k(k \\geqslant 2)$ 个互不相同的正整数 $r_1, \\cdots, r_k$,均有\n$$\nf(m) \\neq f\\left(r_1\\right) f\\left(r_2\\right) \\cdots f\\left(r_k\\right) .\n$$", + "solution": "证明:本题的基本精神是要求两个整数不能相等, 同余对此正能派上用场(参见例 3 下面的注).\n我们希望作出一个 (首项系数为 1 的) 正整数系数的 $n$ 次多项式, 使得对任意整数 $a$, 均有 $f(a) \\equiv 2(\\bmod 4)$, 由此即知, 对任意 $k(k \\geqslant 2)$ 个整数 $r_1, \\cdots$,\n$r_k$, 有 $f\\left(r_1\\right) \\cdots f\\left(r_k\\right) \\equiv 0(\\bmod 4)$, 但 $f(m) \\equiv 2(\\bmod 4)$, 因此, 对任意整数 $m$, 数 $f(m)$ 与 $f\\left(r_1\\right) \\cdots f\\left(r_k\\right)$ 模 4 不相等, 从而它们决不能相等.\n我们取\n$$\nf(x)=(x+1)(x+2) \\cdots(x+n)+2 . \\label{eq1}\n$$\n将式\\ref{eq1}的右边展开即知 $f(x)$ 是一个 $n$ 次的首项系数为 1 的正整数系数的多项式.\n另一方面, 对任意整数 $a$, 由于 $n \\geqslant 4$, 故连续 $n$ 个整数 $a+1, \\cdots, a+n$ 中必有一个为 4 的倍数, 因此 $4 \\mid \\cdot(a+1) \\cdots(a+n)$, 故由式\\ref{eq1}知 $f(a) \\equiv 2(\\bmod 4)$. 这表明多项式\\ref{eq1}符合问题的要求.\n构作 $f(x)$ 的方式很多, 下面是一个稍有些不同的方法:\n我们注意, 当 $n \\geqslant 4$ 为偶数时, 则对任意整数 $a$ 有 $4 \\mid a^n-a^2$. 这是因为, 若 $a$ 为偶数,则 $4 \\mid a^2$,故 4 整除 $a^2\\left(a^{n-2}-1\\right)=a^n-a^2$;若 $a$ 为奇数,则因 $n-$ 2 为偶数,故 $a^{n-2}$ 是奇数的平方, 从而 $4 \\mid a^{n-2}-1$, 故 $4 \\mid a^2\\left(a^{n-2}-1\\right)$.\n同样不难证明,当 $n \\geqslant 5$ 为奇数时, $a^n-a^3=a^3\\left(a^{n-3}-1\\right)$ 被 4 整除.\n因此, 对偶数 $n \\geqslant 4$, 取\n$$\n\\begin{aligned}\nf(x) & =x^n+4\\left(x^{n-1}+\\cdots+x^3\\right)+3 x^2+4 x+2 \\label{eq2}\\\\\n& =x^n-x^2+4\\left(x^{n-1}+\\cdots+x\\right)+2 ;\\label{eq3}\n\\end{aligned}\n$$\n对奇数 $n \\geqslant 5$, 取\n$$\n\\begin{aligned}\nf(x) & =x^n+4\\left(x^{n-1}+\\cdots+x^4\\right)+3 x^3+4 x^2+4 x+2 \\label{eq4} \\\\\n& =x^n-x^3+4\\left(x^{n-1}+\\cdots+x\\right)+2 . \\label{eq5}\n\\end{aligned}\n$$\n则由式\\ref{eq2}、\\ref{eq4}可见, $f(x)$ 是 $n$ 次的首项系数为 1 的正整数系数多项式; 而由式\\ref{eq3}、\\ref{eq5}及前面说的结果知, 对任意整数 $a$, 有 $f(a) \\equiv 2(\\bmod 4)$. 因此多项式\\ref{eq2}或\\ref{eq4} 符合要求.\n证毕.\n请注意, 若不要求所说的多项式的首项系数为 1 , 则问题极为平凡.\n例如, 可取 $f(x)=4\\left(x^n+x^{n-1}+\\cdots+x\\right)+2$.", + "remark": "", + "figures": [] +} \ No newline at end of file diff --git a/processed_dataset/proof/0201.json b/processed_dataset/proof/0201.json new file mode 100644 index 0000000000000000000000000000000000000000..55dc220917d914d13602ef1eb9add59610c436cf --- /dev/null +++ b/processed_dataset/proof/0201.json @@ -0,0 +1,8 @@ +{ + "source_file": "./raw_volume-zh/volume10/chapter6.tex", + "problem_type": "proof", + "problem": "例11. 设 $k 、 l$ 是两个给定的正整数.\n证明, 有无穷多个正整数 $m$, 使得 $\\mathrm{C}_m^k$ 与 $l$ 互素.", + "solution": "证法一我们需证明, 有无穷多个 $m$, 使得对于 $l$ 的任一个素因子 $p$, 有 $p \\nmid \\mathrm{C}_m^k$. 注意\n$$\nk ! \\mathrm{C}_m^k=m(m-1) \\cdots(m-(k-1)) . \\label{eq1}\n$$\n对于任意一个素数 $p \\mid l$, 设 $p^\\alpha \\| k$ !, 即 $p^\\alpha \\mid k$ !, 但 $p^{\\alpha+1} \\nmid k$ !, 这里 $\\alpha \\geqslant 0$. 我们取 (无穷多个) $m$, 使得(1)的右边 $\\neq \\equiv 0\\left(\\bmod p^{\\alpha+1}\\right)$. 这样的 $m$ 可以取为\n$$\nm \\equiv k\\left(\\bmod p^{\\alpha+1}\\right) . \\label{eq2}\n$$\n对满足式\\ref{eq2}的 $m$, 式\\ref{eq1} 的右边 (在模 $p^{\\alpha+1}$ 意义下) 被简化为: $\\equiv k(k-1) \\cdots 1=k$ ! $\\left(\\bmod p^{\\alpha+1}\\right)$, 即有\n$$\nk ! \\mathrm{C}_m^k \\equiv k !\\left(\\bmod p^{\\alpha+1}\\right) . \\label{eq3}\n$$\n因 $p^{\\alpha+1} \\nmid k !$, 故由上式知 $p \\nmid \\mathrm{C}_m^k$.\n现在设 $p_1, \\cdots, p_t$ 是 $l$ 的全部的不同素因子, 并设 $p_i^{a i} \\| k !$, 由上面的结果知, 若 $m \\equiv k\\left(\\bmod p_i^{\\alpha_i+1}\\right)(i=1, \\cdots, t)$, 即\n$$\nm \\equiv k\\left(\\bmod p_1^{\\alpha_1+1} \\cdots p_t^{\\alpha_t+1}\\right),\n$$\n则 $\\mathrm{C}_m^k$ 与 $p_1, \\cdots, p_t$ 均互素, 从而与 $l$ 互素.\n满足式\\ref{eq3}的正整数 $m$ 当然有无穷多个.\n证毕.\n注意, 由 $p_i$ 及 $p_i^{\\alpha_i}$ 的定义可见, $p_1^{\\alpha_1+1} \\cdots p_t^{\\alpha_t+1} \\mid l \\cdot k$ !, 因此, 若 $m$ 满足 $m \\equiv k(\\bmod l \\cdot k !)$, 则更满足式\\ref{eq3}, 故也可取 $m$ 为显式依赖于给定整数 $k, l$ 的正整数: $m \\equiv k(\\bmod l \\cdot k !)$.", + "remark": "", + "figures": [] +} \ No newline at end of file diff --git a/processed_dataset/proof/0202.json b/processed_dataset/proof/0202.json new file mode 100644 index 0000000000000000000000000000000000000000..c6f6d567ed3322fabffafe4be376606775ced9ad --- /dev/null +++ b/processed_dataset/proof/0202.json @@ -0,0 +1,8 @@ +{ + "source_file": "./raw_volume-zh/volume10/chapter6.tex", + "problem_type": "proof", + "problem": "例11. 设 $k 、 l$ 是两个给定的正整数.\n证明, 有无穷多个正整数 $m$, 使得 $\\mathrm{C}_m^k$ 与 $l$ 互素.", + "solution": "证法二这一解法无需借助同余.\n将 $\\mathrm{C}_m^k$ 表示为\n$$\n\\begin{aligned}\n\\mathrm{C}_m^k & =\\frac{m(m-1) \\cdots(m-k+1)}{k !} \\\\\n& =\\frac{m}{1} \\cdot \\frac{(m-1)}{2} \\cdot \\cdots \\cdot \\frac{(m-(k-2))}{k-1} \\cdot \\frac{(m-(k-1))}{k} \\\\\n& =\\left(\\frac{m+1}{1}-1\\right)\\left(\\frac{m+1}{2}-1\\right) \\cdot \\cdots \\cdot\\left(\\frac{m+1}{k-1}-1\\right)\\left(\\frac{m+1}{k}-1\\right) .\n\\end{aligned}\n$$\n我们希望取正整数 $m$, 使得对任意 $i=1,2, \\cdots, k$, 数 $\\frac{m+1}{i}$ 为 $l$ 的倍数, 从而每个 $\\frac{m+1}{i}-1$ 均与 $l$ 互素, 故它们的积与 $l$ 互素, 即 $\\left(\\mathrm{C}_m^k, l\\right)=1$. 显然 $m= -1+x l \\cdot k$ ! 符合这样的要求, $x=1,2, \\cdots$, 这当然有无穷多个.", + "remark": "", + "figures": [] +} \ No newline at end of file diff --git a/processed_dataset/proof/0203.json b/processed_dataset/proof/0203.json new file mode 100644 index 0000000000000000000000000000000000000000..c9f8fe8ccab70b871f511d45625fd6021b8e8a32 --- /dev/null +++ b/processed_dataset/proof/0203.json @@ -0,0 +1,8 @@ +{ + "source_file": "./raw_volume-zh/volume10/chapter7.tex", + "problem_type": "proof", + "problem": "例1. 设 $p$ 是给定的素数.\n证明: 数列 $\\left\\{2^n-n\\right\\}(n \\geqslant 1)$ 中有无穷多个项被整除.", + "solution": "证明:$p=2$ 时结论显然成立.\n设 $p>2$, 则由费马小定理得 $2^{p-1} \\equiv 1(\\bmod p)$, 从而对任意正整数 $m$ 有\n$$\n2^{m(p-1)} \\equiv 1(\\bmod p) . \\label{eq1}\n$$\n我们取 $m \\equiv-1(\\bmod p)$, 则由 式\\ref{eq1}, 得\n$$\n2^{m(p-1)}-m(p-1) \\equiv 1+m \\equiv 0(\\bmod p) .\n$$\n因此, 若 $n=(k p-1)(p-1)$, 则 $2^n-n$ 被 $p$ 整除 ( $k$ 为任意正整数), 故数列中有无穷多项被 $p$ 整除.", + "remark": "", + "figures": [] +} \ No newline at end of file diff --git a/processed_dataset/proof/0204.json b/processed_dataset/proof/0204.json new file mode 100644 index 0000000000000000000000000000000000000000..c821d1240251acabb6095fa5fc673cbbceec0b76 --- /dev/null +++ b/processed_dataset/proof/0204.json @@ -0,0 +1,8 @@ +{ + "source_file": "./raw_volume-zh/volume10/chapter7.tex", + "problem_type": "proof", + "problem": "例2. 证明: 数列 $1,31,331,3331, \\cdots$ 中有无穷多个合数.", + "solution": "证明:因 31 是素数, 由费马小定理知, $10^{30} \\equiv 1(\\bmod 31)$, 故对任意正整数 $k$, 有 $10^{30 k} \\equiv 1(\\bmod 31)$, 从而\n$$\n\\frac{.1}{3}\\left(10^{30 k}-1\\right) \\equiv 0(\\bmod 31) .\n$$\n这表明, $30 k$ 个 3 组成的数被 31 整除,这数乘以 100 后再加上 31 ,也被 31 整除, 即数列中第 $30 k+2$ 项被 31 整除, 故它不是素数, 从而上述的数列中有天穷多个合数.", + "remark": "", + "figures": [] +} \ No newline at end of file diff --git a/processed_dataset/proof/0205.json b/processed_dataset/proof/0205.json new file mode 100644 index 0000000000000000000000000000000000000000..aadc7f4a9d7bcbab72627fd160a3f4ac12ad9808 --- /dev/null +++ b/processed_dataset/proof/0205.json @@ -0,0 +1,8 @@ +{ + "source_file": "./raw_volume-zh/volume10/chapter7.tex", + "problem_type": "proof", + "problem": "例3. 证明: 对任意给定的正整数 $n$,均有连续 $n$ 个正整数,其中每一个者有大于 1 的平方因子.", + "solution": "证明:由于素数有无穷多个, 我们可取出 $n$ 个互不相同的素数 $p_1 p_2, \\cdots, p_n$, 而考虑同余式组\n$$\nx \\equiv-i\\left(\\bmod p_i^2\\right), i=1,2, \\cdots, n . \\label{eq1}\n$$\n因 $p_1^2, p_2^2, \\cdots, p_n^2$ 显然两两互素,故由中国剩余定理知, 上述同余式组有正整数解, 于是, 连续 $n$ 个数 $x+1, x+2, \\cdots, x+n$ 分别被平方数 $p_1^2 p_2^2, \\cdots, p_n^2$ 整除.\n若不直接使用素数, 也可采用下面的变异的方法.\n由于费马数 $F_k=2^{2^k} 1(k \\geqslant 0)$ 两两互素, 故将 式\\ref{eq1} 中的 $p_i^2$ 换为 $F_i^2(i=1,2, \\cdots, n)$ 后, 相应的同余式组也有解, 同样导出证明.", + "remark": "注:例 3 的解法表现了中国剩余定理的一个基本功效,它常常能将\"找建续 $n$ 个整数具有某种性质\"的问题,化归为 \"找 $n$ 个两两互素的数具有某种忙质\", 后者往往易于解决.", + "figures": [] +} \ No newline at end of file diff --git a/processed_dataset/proof/0206.json b/processed_dataset/proof/0206.json new file mode 100644 index 0000000000000000000000000000000000000000..b116d76a8249e8aff6115498edf3354b484f0905 --- /dev/null +++ b/processed_dataset/proof/0206.json @@ -0,0 +1,8 @@ +{ + "source_file": "./raw_volume-zh/volume10/chapter7.tex", + "problem_type": "proof", + "problem": "例4. (1)证明: 对任意正整数 $n$, 存在连续 $n$ 个正整数,其中每一个都是幂数;\n(2)证明,存在无穷多个互不相同的正整数,它们及它们中任意多个不厓数的和均不是幂数.\n(幕数的定义请见第 5 单元例 9. )", + "solution": "证明:(1) 我们证明, 存在连续 $n$ 个正整数,其中每一个数都至少有一个素因子, 在这个数的标准分解中仅出现一次, 从而这个数不是幂数.\n由于素数有无穷多个,故可取 $n$ 个互不相同的素数 $p_1, \\cdots, p_n$. 考虑同余式组\n$$\nx \\equiv-i+p_i\\left(\\bmod p_i^2\\right), i=1,2, \\cdots, n . \\label{eq1}\n$$\n因 $p_1^2, p_2^2, \\cdots, p_n^2$ 两两互素, 故由中国剩余定理知, 上述同余式组有正整数解 $x$. 对 $1 \\leqslant i \\leqslant n$, 因 $x+i \\equiv p_i\\left(\\bmod p_i^2\\right)$, 故 $p_i \\mid(x+i)$; 但由 式\\ref{eq1} 可知 $p_i^2 \\nmid(x+ i)$, 即 $p_i$ 在 $x+i$ 的标准分解中恰出现一次,故 $x+1, x+2, \\cdots, x+n$ 都不是幂数.\n(2) 我们归纳构作一个由非幂数的正整数组成的 (严格增的)无穷数歹 $a_1, a_2, \\cdots, a_n, \\cdots$, 使得对每个 $n$, 数 $a_1, \\cdots, a_n$ 中任意多个的和均不是幂数由此即证明了 (2) 中的结论.\n首先, $a_1$ 可取为任一个非幂数的数,例如取 $a_1=2$. 设 $a_1, \\cdots, a_n$ 已确定, 我们证明, 可选择 $a_{n+1}$ 不是幂数, $a_{n+1}>a_n$, 且 $a_{n+1}$ 与 $a_1, \\cdots, a_n$ 中任意多个数的和均不是幂数.\n设 $s_1, \\cdots, s_m$ 是由 $a_1, \\cdots, a_n$ 产生的所有不同项的和, 这里 $m=2^n-1$. 由于素数有无穷多个, 故可取 $m+1$ 个不同素数 $p, p_1, \\cdots, p_m$, 考虑同余式组\n$$\nx \\equiv p\\left(\\bmod p^2\\right), x \\equiv-s_i+p_i\\left(\\bmod p_i^2\\right), i=1, \\cdots, m . \\label{eq2}\n$$\n因 $p^2, p_1^2, \\cdots, p_m^2$ 两两互素, 故同余式组\\ref{eq2}有无穷个正整数解 $x$. 任取一个大于 $a_n$ 的解, 记为 $a_{n+1}$. 则由 $a_{n+1} \\equiv p\\left(\\bmod p^2\\right)$ 知, $a_{n+1}$ 被 $p$ 整除, 但 $p^2 \\nmid a_{n+1}$, 故 $a_{n+1}$ 不是幕数.\n又 $a_{n+1} \\equiv-s_i+p_i\\left(\\bmod p_i^2\\right)$ 表明, $a_{n+1}+s_i$ 被 $p_i$ 整除但不被 $p_i^2$ 整除, 从而对每个 $i=1, \\cdots, m$, 数 $a_{n+1}+s_i$ 均不是幕数.\n由此就递推地构作了一个符合前述要求的无穷数列, $a_1, a_2, \\cdots$. 证毕.", + "remark": "注:本题 (2) 的另一种解法见第 5 单元例 9 , 那儿构作的数列中, 每一项均整除其后一项.\n作为对比, 我们注意, 将 (2) 的上述解法稍作修改, 则可使得我们构作的数列中的项两两互素.\n事实上, 归纳假设 $a_1, \\cdots, a_n$ 已两两互素, 设 $q_1, \\cdots, q_t$ 是这些数中出现的所有不同的素因子.\n现在取素数 $p, p_1, \\cdots, p_m$ 互不相同, 且与 $q_1, \\cdots, q_t$ 也不相同,在同余式组\\ref{eq2}中增加一个限制\n$$\nx \\equiv 1\\left(\\bmod q_1 \\cdots q_t\\right) . \\label{eq3}\n$$\n由于 $p^2, p_1^2, \\cdots, p_m^2, q_1 \\cdots q_t$ 两两互素, 故同余式组\\ref{eq2}增添\\ref{eq3}后有解, 并且由 式\\ref{eq3}知,任一个解 $x$ 与 $q_1 \\cdots q_t$ 互素, 从而与 $a_1, \\cdots, a_n$ 均互素.", + "figures": [] +} \ No newline at end of file diff --git a/processed_dataset/proof/0207.json b/processed_dataset/proof/0207.json new file mode 100644 index 0000000000000000000000000000000000000000..86546c3db0b18968be9498be1d1b1be442c90085 --- /dev/null +++ b/processed_dataset/proof/0207.json @@ -0,0 +1,8 @@ +{ + "source_file": "./raw_volume-zh/volume10/chapter7.tex", + "problem_type": "proof", + "problem": "例5. 给定正整数 $n$,设 $f(n)$ 是使 $\\sum_{k=1}^{f(n)} k$ 能被 $n$ 整除的最小正整数.\n证明: 当且仅当 $n$ 为 2 的幂时有 $f(n)=2 n-1$.", + "solution": "证明:问题的前一半甚为容易.\n如果 $n=2^m$, 则一方面\n$$\n\\sum_{k=1}^{2 n-1} k=\\frac{(2 n-1) \\times 2 n}{2}=\\left(2^{m+1}-1\\right) \\cdot 2^m\n$$\n被 $2^m=n$ 整除.\n另一方面,若 $r \\leqslant 2 n-2$, 则\n$$\n\\sum_{k=1}^r k=\\frac{r(r+1)}{2}\n$$\n不被 $2^m$ 整除, 这是因为 $r$ 和 $r+1$ 中有一个是奇数, 而另一个不超过 $(2 n- 2)+1=2^{m+1}-1$, 因而不被 $2^{m+1}$ 整除.\n综合上述两个方面, 即知 $f\\left(2^m\\right)=2^{m+1}-1$.\n现在设 $n$ 不是 2 的方幂,即 $n=2^m a$, 其中 $m \\geqslant 0, a>1$ 为奇数.\n我们证明, 存在正整数 $r<2 n-1$, 使得 $2^{m+1} \\mid r$, 且 $a \\mid(r+1)$, 于是\n$$\n\\sum_{k=1}^r k=\\frac{r(r+1)}{2}\n$$\n被 $2^m a==n$ 整除, 因而 $f(n)<2 n-1$.\n为证明.\n面的断言, 我们考虑\n$$\nx \\equiv 0\\left(\\bmod 2^{m+1}\\right), x \\equiv-1(\\bmod a) . \\label{eq1}\n$$\n因为 $\\left(2^{m+1}, a\\right)=1$,故由中国剩余定理知, 同余式组 式\\ref{eq1} 必有解 $x_0$, 并且其全部解为 $x \\equiv x_0\\left(\\bmod 2^{m+1} a\\right)$, 即 $x \\equiv x_0(\\bmod 2 n)$. 因此可确定一个 $r$ 满足 式\\ref{eq1} 且 $01, n \\mid\\left(2^n+1\\right)$, 证明 $3 \\mid n$.", + "solution": "证明:显然 $n$ 是奇数.\n设 $p$ 是 $n$ 的最小素因子,我们证明 $p=3$, 从而 31 $n$. 设 2 模 $p$ 的阶是 $r$. 由 $2^n \\equiv-1(\\bmod n)$ 知\n$$\n2^{2 n} \\equiv 1(\\bmod p) . \\label{eq1}\n$$\n又因 $p \\geqslant 3$, 故费马小定理给出\n$$\n2^{p-1} \\equiv 1(\\bmod p) . \\label{eq2}\n$$\n由 式\\ref{eq1}、\\ref{eq2}及阶的性质推出 $r \\mid 2 n$ 及 $r \\mid(p-1)$, 故 $r \\mid(2 n, p-1)$. 不难证明 $( 2 n, p-1)=2$. 这是因为, 由 $2 \\nmid n$ 知 $2 \\mid(2 n, p-1)$, 但 $2^2 \\nmid(2 n, p-1)$; 另一方面, 若有奇素数 $q \\mid(2 n, p-1)$, 则 $q \\mid(p-1)$, 及 $q \\mid n$, 但前者表明 $q1$, 证明 $n \\nmid\\left(2^n-1\\right)$.", + "solution": "证明:一反证法, 设有一个 $n>1$, 使 $n \\mid\\left(2^n-1\\right)$. 对 $n$ 的任一个素因子 $p$, 有 $p \\geqslant 3$. 设 2 模 $p$ 的阶为 $r$, 则显然 $r>1$. 由 $2^n \\equiv 1(\\bmod n)$ 推出\n$$\n2^n \\equiv 1(\\bmod p) . \\label{eq1}\n$$\n又由费马小定理得\n$$\n2^{p-1} \\equiv 1(\\bmod p) . \\label{eq2}\n$$\n因此 $r \\mid n$ 及 $r \\mid(p-1)$, 从而 $r \\mid(n, p-1)$. 现在我们特别地取 $p$ 为 $n$ 的最小素因子, 则必有 $(n, p-1)=1$. 因为否则就有素数 $q \\mid(n, p-1)$, 故 $q \\mid (p-1)$, 及 $q \\mid n$, 但前者意味着 $q1$ 等价于 $n$ 有一个素因子, 因此, 从 $2^n \\equiv 1(\\bmod n)$ 过渡到同余式 \\ref{eq1}, 虽然减弱了反证法假设, 但仍刻画了 $n>1$.\n模一个素数的同余, 往往有一些更适用的性质 (或结果), 就本题而言, 这样做的益处在于此时有同余式\\ref{eq2}. 例 1 及下面的例 3 均如此.\n注:2 同余式\\ref{eq1}和\\ref{eq2}对于 $n$ 的任一素因子 $p$ 均成立.\n因此, 在证法一的开始阶段, 我们将 $p$ 视为一待定参量, 导出 $r \\mid(p-1, n)$, 便提供了选择 $p$ 以产生矛盾的机会.\n保留参量, 使我们的处理留有选择的余地, 保持了某种灵活性, 这是一种非常基本的手法.\n注:3 由第 5 单元例 10 可知, 满足例 1 条件的 $n$ 有无穷多个, 这与例 2 的结论完全相反.\n读者可查看一下, 是论证中的哪些差异, 使得导出的结果如此的不同.\n注:4 顺便提一下, 不利用阶也能解决例 2. 设 $p$ 是 $n$ 的最小素因子, 则 $(p-1, n)=1$. 而由 式\\ref{eq1}, \\ref{eq1} 知 $p \\mid\\left(2^{p-1}-1,2^n-1\\right)$, 故由第 2 单元例 4 推出\n$p \\mid\\left(2^{(p-1, n)}-1\\right)$, 从而 $p \\mid 1$,矛盾!", + "figures": [] +} \ No newline at end of file diff --git a/processed_dataset/proof/0211.json b/processed_dataset/proof/0211.json new file mode 100644 index 0000000000000000000000000000000000000000..96077d6ca616cc4903b73a1f4b118af8d628dd86 --- /dev/null +++ b/processed_dataset/proof/0211.json @@ -0,0 +1,8 @@ +{ + "source_file": "./raw_volume-zh/volume10/chapter8.tex", + "problem_type": "proof", + "problem": "例2. 设 $n>1$, 证明 $n \\nmid\\left(2^n-1\\right)$.", + "solution": "证明二这一解法不必考虑 $n$ 的素因子.\n设有 $n>1$, 使 $n \\mid\\left(2^n-1\\right)$, 则 $n$ 为奇数, 设 $r$ 是 2 模 $n$ 的阶, 则由 $2^n \\equiv 1(\\bmod n)$ 知 $r \\mid n$. 又 $2^r \\equiv 1(\\bmod n)$, 故更有 $2^r \\equiv 1(\\bmod r)$, 即\n$$\nr \\mid\\left(2^r-1\\right) . \\label{eq3}\n$$\n因阶 $r$ 满足 $1 \\leqslant rr>r_1>r_2>\\cdots>1$, 这显然不可能.\n这一证明,也可采用下面更为简单的表述: 取 $n>1$ 是最小的使 $n \\mid\\left(2^n-1\\right)$ 的整数, 上面论证产生了一个整数 $r$, 使得 $r \\mid\\left(2^r-1\\right)$ 且 $11,2 \\nmid n$, 则对任意整数 $m>0$, 有 $n \\nmid\\left(m^{n-1}+1\\right)$.", + "solution": "证明:假设有大于 1 的奇数 $n$, 满足 $n \\mid\\left(m^{n-1}+1\\right)$, 则 $(m, n)=1$. 设 $p$ 是 $n$ 的任一个素约数, $r$ 是 $m$ 模 $p$ 的阶 (注意 $p \\nmid m$ ). 又设 $n-1=2^k t, k \\geqslant 1,2 \\nmid t$. 那么就有\n$$\nm^{2^k t} \\equiv-1(\\bmod p), \\label{eq1}\n$$\n从而 $m^{2^{k+1} t} \\equiv 1(\\bmod p)$,故 $r \\mid 2^{k+1} t$.\n关键的一点是证明 $2^{k+1} \\mid r$. 假设这结论不对, 那么 $r=2^s r_1$, 其中 $0 \\leqslant s \\leqslant k, r_1 \\mid t$. 则由 $m^r \\equiv 1(\\bmod p)$ 推出 $m^{2^k t} \\equiv 1(\\bmod p)$, 结合 式\\ref{eq1} 得 $p=2$, 矛盾! 故 $2^{k+1} \\mid r$.\n现在由 $(p, m)=1$, 得出 $m^{p-1} \\equiv 1(\\bmod p)$, 从而 $r \\mid(p-1)$, 故 $2^{k+1} \\mid (p-1)$, 即 $p \\equiv 1\\left(\\bmod 2^{k+1}\\right)$. 由于 $p$ 是 $n$ 的任一素因子, 将 $n$ 作标准分解, 即知 $n \\equiv 1\\left(\\bmod 2^{k+1}\\right)$, 即 $2^{k+1} \\mid(n-1)$, 但这与前面所设的 $2^k \\|(n-1)$ 相违.", + "remark": "", + "figures": [] +} \ No newline at end of file diff --git a/processed_dataset/proof/0213.json b/processed_dataset/proof/0213.json new file mode 100644 index 0000000000000000000000000000000000000000..0dbc77be4330fffb76e6db3e67263efa69ea1806 --- /dev/null +++ b/processed_dataset/proof/0213.json @@ -0,0 +1,8 @@ +{ + "source_file": "./raw_volume-zh/volume10/chapter8.tex", + "problem_type": "proof", + "problem": "例4. 设 $p$ 是一个奇素数.", + "solution": "证明: $\\frac{p^{2 p}+1}{p^2+1}$ 的任一正约数均 $\\equiv 1(\\bmod 4 p)$. 证明我们只要证明 $\\frac{p^{2 p}+1}{p^2+1}$ 的任一个素约数 $q$ 满足 $q \\equiv 1(\\bmod 4 p)$ 即可.\n首先注意\n$$\n\\frac{p^{2 p}+1}{p^2+1}=p^{2(p-1)}-p^{2(p-2)}+\\cdots-p^2+1 . \\label{eq1}\n$$\n故 $q \\neq p$. 设 $r$ 是 $p$ 模 $q$ 的阶, 因\n$$\np^{2 p} \\equiv-1(\\bmod q), \\label{eq2}\n$$\n故 $p^{4 p} \\equiv 1(\\bmod q)$, 所以 $r \\mid 4 p$. 于是 $r=1,2,4, p, 2 p$ 或 $4 p$.\n若 $r=1,2, p, 2 p$, 将导出 $p^{2 p} \\equiv 1(\\bmod q)$, 结合 式\\ref{eq2} 得到 $q=2$, 这不可能; 若 $r=4$, 则因 $q$ 是素数, 我们推出 $q \\mid\\left(p^2-1\\right)$ 或 $q \\mid\\left(p^2+1\\right)$. 前者已证明为不可能.\n若后者成立, 即 $p^2 \\equiv-1(\\bmod q)$. 我们将 \\ref{eq1} 式模 $q$, 其左边模 $q$ 当然为 0 , 而右边 $\\equiv(-1)^{p-1}-(-1)^{p-2}+\\cdots-(-1)+1 \\equiv p(\\bmod q)$. 因此 $p= q$, 这不可能,故 $r \\neq 4$. 因此只能 $r=4 p$.\n最后, 因 $(p, q)=1$, 故由费马小定理得 $p^{q-1} \\equiv 1(\\bmod q)$, 于是 $r \\mid(q-- 1)$, 即 $4 p \\mid(q-1)$, 因此 $q \\equiv 1(\\bmod 4 p)$.\n上面的解法中, 关键是确定 $p$ 模 $q$ 的阶.", + "remark": "", + "figures": [] +} \ No newline at end of file diff --git a/processed_dataset/proof/0214.json b/processed_dataset/proof/0214.json new file mode 100644 index 0000000000000000000000000000000000000000..d5f6f50acaddd3ec80623195e87720e4b0705693 --- /dev/null +++ b/processed_dataset/proof/0214.json @@ -0,0 +1,8 @@ +{ + "source_file": "./raw_volume-zh/volume10/chapter8.tex", + "problem_type": "proof", + "problem": "例5. (1)设 $p$ 是奇素数, $a \\neq \\pm 1, p \\nmid a$. 设 $r$ 是 $a$ 模 $p$ 的阶, $k_0$ 满足 $p^{k_0} \\|\\left(a^r-1\\right)$. 记 $r_k$ 是 $a$ 模 $p^k$ 的阶, 则有\n$$\nr_k=\\left\\{\\begin{array}{l}\nr, \\text { 若 } k=1, \\cdots, k_0, \\\\\nr p^{k-k_0}, \\text { 若 } k>k_0 .\n\\end{array}\\right.\n$$\n(2)设 $a$ 是奇数, $a \\equiv 1(\\bmod 4), a \\neq 1, k_0$ 满足 $2^{k_0} \\|(a-1)$. 记 $l_k$ 是 $a$ 模 $2^k$ 的阶, 则有\n$$\nl_k=\\left\\{\\begin{array}{l}\n1, \\text { 若 } k=1, \\cdots, k_0, \\\\\n2^{k-k_0}, \\text { 若 } k>k_0 .\n\\end{array}\\right.\n$$\n(3)设 $a$ 是奇数, $a \\equiv-1(\\bmod 4), a \\neq-1, k_0$ 满足 $2^{k_0} \\|(a+1)$. 记 $l_k$ 是 $a$ 模 $2^k$ 的阶, 则有\n$$\nl_k=\\left\\{\\begin{array}{l}\n1, \\text { 若 } k=1, \\\\\n2, \\text { 若 } k=2, \\cdots, k_0+1, \\\\\n2^{k-k_0}, \\text { 若 } k>k_0+1 .\n\\end{array}\\right.\n$$", + "solution": "证明:(1) 当 $1 \\leqslant k \\leqslant k_0$ 时, 由 $a^{r_k} \\equiv 1\\left(\\bmod p^k\\right)$ 推出 $a^r \\equiv 1(\\bmod p)$, 故由 $r$ 的定义知 $r \\mid r_k$. 另一方面, 由 $a^r \\equiv 1\\left(\\bmod p^{k_0}\\right)$ 可得 $a^r \\equiv 1\\left(\\bmod p^k\\right)$, 故由 $r_k$ 的定义推出 $r_k \\mid r$, 从而 $r_k=r\\left(k=1, \\cdots, k_0\\right)$.\n现在设 $k>k_0$. 我们先证明, 对每个 $i=0,1, \\cdots$, 有 $p^{k_0+i} \\|\\left(a^{r p^i}-1\\right)$,\n即有\n$$\na^{r p^i}==1+p^{k_0+i} u_i,\\left(u_i, p\\right)=1 . \\label{eq1}\n$$\n这可用归纳法来证明: 当 $i=0$ 时, 由 $k_0$ 的定义知 式\\ref{eq1} 成立.\n设 式\\ref{eq1} 对 $i \\geqslant 0$ 时已成立, 则由二项式定理易知\n$$\n\\begin{aligned}\na^{r p^{i+1}} & =\\left(1+p^{k_0+i} u_i\\right)^p=1+p^{k_0+i+1} u_i+\\mathrm{C}_p^2 p^{2 k_0+2 i} u_i^2+\\cdots \\\\\n& =1+p^{k_0+i+1}\\left(u_i+\\mathrm{C}_p^2 p^{k_0+i-1} u_i^2+\\cdots\\right) \\\\\n& =1+p^{k_0+i+1} u_{i+1}\n\\end{aligned}\n$$\n易知 $p \\nmid u_{i+1}$ (注意, 我们这里需要 $p \\geqslant 3$ ), 于是 式\\ref{eq1} 对所有 $i \\geqslant 0$ 都成立:\n利用式\\ref{eq1}, 我们对 $k \\geqslant k_0$ 归纳证明 $r_k=r p^{k-k_0}$. 当 $k=k_0$ 时, 前面已证明了结论成立.\n若 $k>k_0$, 设已有 $r_{k-1}=r p^{k-k_0-1}$. 一方面, 在 式\\ref{eq1} 中取 $i=k-k_0$ 可知 $a^{r p^{k-k_0}} \\equiv 1\\left(\\bmod p^k\\right)$,故 $r_k \\mid r p^{k-k_0}$. 另一方面, 由 $a^{r_k} \\equiv 1\\left(\\bmod p^k\\right)$ 可推出 $a^{r_k} \\equiv 1\\left(\\bmod p^{k-1}\\right)$, 故 $r_{k-1} \\mid r_k$, 因此 $r_k=r p^{k-k_0}$ 或 $r p^{k-k_0-1}$. 但在 (1) 中取 $i=k- k_0-1$, 可知 $a^{r p^{k-k_0-1}} \\not \\equiv 1\\left(\\bmod p^k\\right)$, 故必须 $r_k=r p^{k-k_0}$.\n(2)当 $1 \\leqslant k \\leqslant k_0$ 时,结论显然成立.\n当 $k>k_0$ 时,注意 $a \\equiv 1(\\bmod 4)$, $a \\neq 1$ 意味着 $k_0 \\geqslant 2$, 由此极易用归纳法对 $i=0,1, \\cdots$ 证明\n$$\na^{2^i}=1+2^{k_0+i} u_i, 2 \\nmid u_i . \\label{eq2}\n$$\n由式\\ref{eq2}则不难与 (1)中相同的论证推出, $l_k=2^{k-k_0}\\left(k \\geqslant k_0\\right)$.\n(3) 由 $a \\equiv-1(\\bmod 4)$, 易证明 $k=1,2, \\cdots, k_0+1$ 时的结论.\n又用归纳法不难得知, 对 $i=1,2, \\cdots$, 有\n$$\na^{2^i}=1+2^{k_0+i} u_i, 2 \\nmid u_i . \\label{eq3}\n$$\n由此可与 (1) 中论证相同地得到 $l_k=2^{k-k_0}$ (对 $k \\geqslant k_0+1$ ).", + "remark": "注1 设 $a$ 和 $n>0$ 为给定的互素的整数, 且均不是 \\pm 1 , 并设 $n$ 的标准分解为 $n=2^\\alpha p_1^{\\alpha_1} \\cdots p_k^{\\alpha_2}$ ( $p_i$ 是奇素数, $\\alpha \\geqslant 0$ ). 若已求得 $a$ 模 $p_i$ 的阶, 则由例 5 可确定 $a$ 模 $p_i^\\alpha$ 的阶, 也可求得 $a$ 模 $2^\\alpha$ 的阶.\n进而, 由习题 8 第 2 题的结果, 可求得 $a$ 模 $n$ 的阶.\n因此, 为确定 $a$ 模一个整数 $n$ 的阶, 最终均化归为求 $a$ 模一个奇素数 $p$ 的阶.\n后者一般而言, 是一个极其困难的问题, 但对于较小的 $a$ 和 $p$, 可以通过手算求得结果.\n注2 设 $p$ 是奇素数, $a \\neq \\pm 1, p \\nmid a, r$ 是 $a$ 模 $p$ 的阶, $k_0$ 满足 $p^{k_0} \\|\\left(a^r-1\\right)$ , 则由例 5 中 式\\ref{eq1} 的证明可见, 对任意与 $p$ 互素的正整数 $m$, 有\n$$\na^{m r p^i}=1+p^{k_0+i} u_i^{\\prime},\\left(u_i^{\\prime}, p\\right)=1, i=0,1, \\cdots .\n$$\n由此并注意 $p$ 必与 $r$ 互素, 我们易得:\n(1)设正整数 $n$ 满足 $r \\mid n, p^l \\| n$, 则 $p^l \\| \\frac{a^n-1}{a^r-1}$.\n此外,设 $a$ 为奇数, $a \\neq \\pm 1, k_0$ 满足 $2^{k_0} \\|\\left(a^2-1\\right), m$ 为任意正奇数,则有\n$$\na^{2^i m}=1+2^{k_0+i-1} u_i^{\\prime}, 2 \\nmid u_i^{\\prime}, i=1,2, \\cdots .\n$$\n由此即知:\n(2) 设 $n$ 为正整数, $2^l \\| n$. 若 $l \\geqslant 1$, 则 $2^{l-1} \\| \\frac{a^n-1}{a^2-1}$.\n注3 设 $p$ 是奇素数, $a, b$ 为整数, $p \\nmid a b$. 则必有正整数 $r$, 使得\n$$\na^r \\equiv b^r(\\bmod p) . \\label{eq4}\n$$\n这是因为有 $b_1$ 使 $b b_1 \\equiv 1(\\bmod p)$, 又有正整数 $r$ 满足 $\\left(a b_1\\right)^r \\equiv 1(\\bmod p)$, 由此导出 式\\ref{eq4}. 此外, 易知使 式\\ref{eq4}成立的最小正整数 $r$ 与 $a b_1$ 模 $p$ 的阶相等.\n由此推出, 若正整数 $n$ 满足 $a^n \\equiv b^n(\\bmod p)$, 则 $r \\mid n$\n注2 中的 (1) 与 (2) 有下面的推广, 其证明则完全类似.\n(1) 设 $a \\neq \\pm b, n$ 为正整数, 若 $r \\mid n, p^l \\| n$, 则 $p^l \\| \\frac{a^n-b^n}{a^r-b^r}$.\n(2)设 $a 、 b$ 为奇数, $a \\neq \\pm b, n$ 为正整数, $2^l \\| n$. 若 $l \\geqslant 1$, 则有 $2^{l-1} \\| \\frac{a^n-b^n}{a^2-b^2}$", + "figures": [] +} \ No newline at end of file diff --git a/processed_dataset/proof/0215.json b/processed_dataset/proof/0215.json new file mode 100644 index 0000000000000000000000000000000000000000..8e7c460e9feec11e659b1d8baebf1ecff8ff491c --- /dev/null +++ b/processed_dataset/proof/0215.json @@ -0,0 +1,8 @@ +{ + "source_file": "./raw_volume-zh/volume10/chapter8.tex", + "problem_type": "proof", + "problem": "例6. 设 $a$ 和 $n$ 为整数,均不为 \\pm 1 , 且 $(a, n)=1$. 证明: 至多有有限个 $k$, 使得 $n^k \\mid\\left(a^k-1\\right)$.", + "solution": "证明:因 $n \\neq \\pm 1$, 故 $n$ 有素因子.\n首先设 $n$ 有奇素数因子 $p$, 则 $p \\nmid a$. 设 $a$ 模 $p$ 的阶为 $r$, 因 $a \\neq \\pm 1$, 故有正整数 $k_0$ 使得 $p^{k_0} \\|\\left(a^r-1\\right)$.\n若有无穷多个 $k$ 使得 $n^k \\mid\\left(a^k-1\\right)$, 从而有无穷多个 $k>k_0$ 满足\n$$\na^k \\equiv 1\\left(\\bmod p^k\\right) . \\label{eq1}\n$$\n但由例 5 得知, $a$ 模 $p^k$ 的阶是 $r p^{k-k_0}$, 故由 式\\ref{eq1}知 $r p^{k-k_0} \\mid k$, 从而 $k \\geqslant r p^{k-k_0} \\geqslant 3^{k-k_0}$, 这样的 $k$ 显然只有有限多个, 产生矛盾.\n若 $n$ 没有奇素数因子, 则 $n$ 是 2 的方幕.\n首先注意, 若奇数 $k$ 使得 $n^k \\mid\\left(a^k-1\\right)$, 则\n$$\na^k-1=(a-1)\\left(a^{k-1}+\\cdots+a+1\\right) . \\label{eq2}\n$$\n被 $2^k$ 整除.\n但式\\ref{eq2}中后一个因数是奇数个奇数之和, 故是奇数, 从而 $2^k \\mid(a-1)$. 因 $a \\neq 1$, 这样的 $k$ 至多有有限多个.\n设有无穷多个偶数 $k=2 l$ 使得 $n^k \\mid\\left(a^k-1\\right)$, 则\n$$\n\\left(a^2\\right)^\\iota \\equiv 1\\left(\\bmod 2^\\iota\\right) . \\label{eq3}\n$$\n定义 $k_0$ 满足 $2^{k_0} \\|\\left(a^2-1\\right)$, 则 $k_0 \\geqslant 3$. 由例 5 知, 当 $l>k_0$ 时, $a^2$ 模 $2^l$ 的阶为 $2^{l-k_0}$, 故在 $l>k_0$ 时, 由式\\ref{eq3}推出 $2^{l-k_0} \\mid l$, 从而 $l \\geqslant 2^{l-k_0}$, 但这样的 $l$ 至多有有限多个,矛盾!", + "remark": "", + "figures": [] +} \ No newline at end of file diff --git a/processed_dataset/proof/0216.json b/processed_dataset/proof/0216.json new file mode 100644 index 0000000000000000000000000000000000000000..0ca72981f0ef280c88ee5de67d07d4c635bac462 --- /dev/null +++ b/processed_dataset/proof/0216.json @@ -0,0 +1,8 @@ +{ + "source_file": "./raw_volume-zh/volume10/chapter9.tex", + "problem_type": "proof", + "problem": "例1. 若 $n \\equiv 4(\\bmod 9)$, 证明不定方程\n$$\nx^3+y^3+z^3=n . \\label{eq1}\n$$\n没有整数解 $(x, y, z)$.", + "solution": "证明:若方程式\\ref{eq1}有整数解, 则式\\ref{eq1}模 9 也有整数解.\n熟知, 一完全立方模 9 同余于 $0,1,-1$ 之一,因而\n$$\nx^3+y^3+z^3 \\equiv 0,1,2,3,6,7,8(\\bmod 9) .\n$$\n但 $n \\equiv 4(\\bmod 9)$, 所以 式\\ref{eq1}模 9 无解, 这与前面所说的相违, 故方程 式\\ref{eq1}无整数解.\n用同余处理不定方程, 核心在于选择适当的模.", + "remark": "", + "figures": [] +} \ No newline at end of file diff --git a/processed_dataset/proof/0217.json b/processed_dataset/proof/0217.json new file mode 100644 index 0000000000000000000000000000000000000000..e90d2e33a1a6dc5711012777bfeb73032dd8d587 --- /dev/null +++ b/processed_dataset/proof/0217.json @@ -0,0 +1,8 @@ +{ + "source_file": "./raw_volume-zh/volume10/chapter9.tex", + "problem_type": "proof", + "problem": "例2. 确定方程\n$$\nx_1^4+x_2^4+\\cdots+x_{14}^4=1599\n$$\n的全部非负整数解 $\\left(x_1, \\cdots, x_{14}\\right)$ (不计解的排列次序).", + "solution": "解:模 16 就能够证明方程无整数解, 因为整数的四次幂模 16 同余于 0 或 1 , 故 $x_1^4+x_2^4+\\cdots+x_{14}^4$ 模 16 的所有可能值是 $0,1,2, \\cdots, 14$, 唯独不能取 15 . 但 $1599 \\equiv 15(\\bmod 16)$, 因此方程无解,证毕.\n之所以选择 16 , 是因为方程左边有 14 项, 剩余类的个数 $\\geqslant 15$ 才比较有希望导出矛盾 (这里我们采用同余来证明方程无整数解). 而 $15=3 \\times 5$, 根据中国剩余定理,模 15 相当于模 3 与模 5 的作用,不能解决问题.", + "remark": "", + "figures": [] +} \ No newline at end of file diff --git a/processed_dataset/proof/0218.json b/processed_dataset/proof/0218.json new file mode 100644 index 0000000000000000000000000000000000000000..6f79d0d4618330a0848ad2860204edd2cd355ca4 --- /dev/null +++ b/processed_dataset/proof/0218.json @@ -0,0 +1,8 @@ +{ + "source_file": "./raw_volume-zh/volume10/chapter9.tex", + "problem_type": "proof", + "problem": "例3. 证明: 下列数不能表示为若干个连续整数的立方和.\n(1) $385^{97}$;\n(2) $366^{17}$.", + "solution": "证明:利用\n$$\n1^3+2^3+\\cdots+k^3=\\left(\\frac{k(k+1)}{2}\\right)^2\n$$\n易知, 连续若干个整数的立方和可表示为形式\n$$\n\\left(\\frac{m(m+1)}{2}\\right)^2-\\left(\\frac{n(n+1)}{2}\\right)^2, \\label{eq1}\n$$\n$m, n$ 为整数.\n我们要证明,对于 (1)、(2) 中的整数, 不存在 $m, n$, 使之可表示为式\\ref{eq1}的形式.\n用分解方法虽也能解决问题, 但相当麻烦; 用同余论证, 则相当直接.\n首先, 按整数 $x$ 模 9 分类并逐一检验, 不难得知, $\\left(\\frac{x(x+1)}{2}\\right)^2$ 模 9 同余于 0 及 -1 , 因此,形如式\\ref{eq1}的数模 9 只能是 $0,1,-1$. 另一方面, 由欧拉定理知\n$$\n385^{97} \\equiv 385 \\times\\left(385^{16}\\right)^6 \\equiv 385 \\equiv 7(\\bmod 9),\n$$\n这就证明了 $385^{97}$ 不能表示为式\\ref{eq1}的形式.\n然而, 因 $366^{17} \\equiv 0(\\bmod 9)$, 故对于数 $366^{17}$, 模 9 不能解决问题.\n我们这次模 7. 易于验证, 对整数 $x$, 数 $\\left(\\frac{x(x+1)}{2}\\right)^2$ 模 7 同余于 0,1 , -1 . 故形如(1)的数模 7 只能是 $0, \\pm 1, \\pm 2$. 但\n$$\n366^{17} \\equiv 2^{17} \\equiv 2 \\times 2^4 \\equiv 4(\\bmod 7) .\n$$\n因此我们的断言成立.\n有整数解的方程, 仅用同余通常不易解决问题, 而需将同余与其他方法 (估计、分解等)结合使用.\n我们举几个这样的例子.", + "remark": "", + "figures": [] +} \ No newline at end of file diff --git a/processed_dataset/proof/0219.json b/processed_dataset/proof/0219.json new file mode 100644 index 0000000000000000000000000000000000000000..2501faa55b1c405b8bec5d23766094a4927843bf --- /dev/null +++ b/processed_dataset/proof/0219.json @@ -0,0 +1,8 @@ +{ + "source_file": "./raw_volume-zh/volume10/chapter9.tex", + "problem_type": "proof", + "problem": "例4. 求所有这样的 2 的幕, 将其(十进制表示中的)首位删去后, 剩下的数仍是一个 2 的幂.", + "solution": "解:问题即要求出方程\n$$\n2^n=2^k+a \\times 10^m . \\label{eq1}\n$$\n的全部正整数解 $(n, k, m, a)$, 其中 $a=1,2, \\cdots, 9$. 将式\\ref{eq1}变形为\n$$\n2^k\\left(2^{n-k}-1\\right)=a \\times 10^m . \\label{eq2}\n$$\n首先证明 $m=1$. 因为若 $m>1$, 则 \\ref{eq2} 式右边被 $5^2$ 整除, 从而 $5^2 \\mid\\left(2^{n-k}-1\\right)$. 又易知, 2 模 $5^2$ 的阶是 20 (这只需注意所说的阶整除 $\\varphi(25)=20$, 及 $2^{10} \\equiv-1(\\bmod 25))$, 因此,20 整除 $n-k$, 从而 $2^{20}-1$ 整除 式\\ref{eq2} 的左边, 但 $2^{20}-1= \\left(2^5\\right)^4-1$ 有因子 $2^5-1=31$, 而 31 不整除 式\\ref{eq1} 的右边,矛盾!因此 $m=1$.\n现在只需在为二位数的 2 的幕中, 检验符合要求的解, 易知这只有 32 和 64 .", + "remark": "", + "figures": [] +} \ No newline at end of file diff --git a/processed_dataset/proof/0220.json b/processed_dataset/proof/0220.json new file mode 100644 index 0000000000000000000000000000000000000000..16d04947c7b9fdb053e75e08f4297a447b55ea69 --- /dev/null +++ b/processed_dataset/proof/0220.json @@ -0,0 +1,8 @@ +{ + "source_file": "./raw_volume-zh/volume10/chapter9.tex", + "problem_type": "proof", + "problem": "例5. 求所有正整数 $x>1, y>1, z>1$, 使得\n$$\n1 !+2 !+\\cdots+x !=y^z . \\label{eq1}\n$$", + "solution": "解:关键一步是证明当 $x \\geqslant 8$ 时必有 $z=2$. 因为式\\ref{eq1}的左边被 3 整除, 故 $3 \\mid y^z$, 从而 $3 \\mid y$,于是式\\ref{eq1}的右边被 $3^z$ 整除.\n另一方面,\n$$\n1 !+2 !+\\cdots+8 !=46233\n$$\n被 $3^2$ 整除, 但不被 $3^3$ 整除; 而对 $n \\geqslant 9$ 有 $3^3 \\mid n !$. 所以, 当 $x \\geqslant 8$ 时, 式\\ref{eq1} 的左边被 $3^2$ 整除而不能被 $3^3$ 整除, 从而 (1) 的右边也如此,即必须 $z=2$.\n现在进一步证明,当 $x \\geqslant 8$ 时方程 式\\ref{eq1} 无解.\n模 5 : 当 $x \\geqslant 8$ 时, 式\\ref{eq1} 的左边 $\\equiv 1 !+2 !+3 !+4 ! \\equiv 3(\\bmod 5)$; 又已证明了此时有 $z=2$, 故 式\\ref{eq2} 的右边 $z^2 \\equiv 0$, 土 $1(\\bmod 5)$, 从而上述断言成立.\n最后, 当 $x<8$ 时, 不难通过检验求得 式\\ref{eq1} 的解是 $x=y=3, z=2$.\n通过比较某个素数在一个等式两边出现的幂次, 以导出结果, 同余的这种变形手法被称为比较素数幂法, 下面的例 6 也应用了这一方法.", + "remark": "", + "figures": [] +} \ No newline at end of file diff --git a/processed_dataset/proof/0221.json b/processed_dataset/proof/0221.json new file mode 100644 index 0000000000000000000000000000000000000000..e0e0f83a7a9e820546a8e57d0ca4c96f8f830a53 --- /dev/null +++ b/processed_dataset/proof/0221.json @@ -0,0 +1,8 @@ +{ + "source_file": "./raw_volume-zh/volume10/chapter9.tex", + "problem_type": "proof", + "problem": "例6. 证明,不定方程\n$$\n(x+2)^{2 m}=x^n+2 . \\label{eq1}\n$$\n没有正整数解.", + "solution": "证明:为了后面的论证, 我们先从方程式\\ref{eq1}导出一些简单的结论.\n显然 $n>1$. 此外, $x$ 必是奇数, 否则将 式\\ref{eq1} 模 4 则产生矛盾.\n进一步, $n$ 也是奇数, 因为若 $2 \\mid n$, 则 $x^n$ 为一个奇数的平方, 从而 式\\ref{eq1} 的右边 $\\equiv 1+2= 3(\\bmod 4)$, 但其左边 $\\equiv 1(\\bmod 4)$, 这不可能.\n故 $2 \\nmid n$.\n设 $x+1=2^\\alpha x_1$, 其中 $x_1$ 为奇数, $\\alpha>0$ (因 $x$ 为奇数). 将方程式\\ref{eq1}改写为\n$$\n(x+2)^{2 m}-1=x^n+1 . \\label{eq2}\n$$\n式\\ref{eq2}的左边有因子 $(x+2)^2-1=\\left(2^\\alpha x_1+1\\right)^2-1=2^{\\alpha+1}\\left(2^{\\alpha-1} x_1^2+x_1\\right)$, 故 $2^{\\alpha+1}$ 整除 式\\ref{eq2} 的左边.\n但另一方面, 由于 $n-1>0$ 为偶数, 用二项式定理易得\n$$\nx^n+1=x\\left(2^\\alpha x_1-1\\right)^{n-1}+1 \\equiv x \\cdot 1+1=2^\\alpha x_1\\left(\\bmod 2^{\\alpha+1}\\right) .\n$$\n因 $2 \\nmid x_1$, 故式\\ref{eq2}的右边 $x^n+1 \\not \\equiv 0\\left(\\bmod 2^{\\alpha+1}\\right)$, 矛盾!", + "remark": "", + "figures": [] +} \ No newline at end of file diff --git a/processed_dataset/proof/0222.json b/processed_dataset/proof/0222.json new file mode 100644 index 0000000000000000000000000000000000000000..416122e5b537c77f996306a947053b7825df12d6 --- /dev/null +++ b/processed_dataset/proof/0222.json @@ -0,0 +1,8 @@ +{ + "source_file": "./raw_volume-zh/volume10/chapter9.tex", + "problem_type": "proof", + "problem": "例7. 证明: 不定方程\n$$\n8^x+15^y=17^z . \\label{eq1}\n$$\n的全部正整数解是 $x=y=z=2$.", + "solution": "证明:我们先用同余证明, $y$ 和 $z$ 都是偶数.\n方程式\\ref{eq1}模 4 , 得到\n$$\n(-1)^y \\equiv 1(\\bmod 4),\n$$\n从而 $y$ 是偶数.\n将方程(1)模 16 ,得到\n$$\n8^x+(-1)^y \\equiv 1(\\bmod 16),\n$$\n即 $8^x \\equiv 0(\\bmod 16)$, 故 $x \\geqslant 2$.\n注意 $17^2 \\equiv 1,15^2 \\equiv 1(\\bmod 32)$, 故若 $z$ 是奇数, 则由 $2 \\mid y$ 及 $x \\geqslant 2$, 可从式\\ref{eq1}得出\n$$\n1 \\equiv 17(\\bmod 32),\n$$\n这不可能.\n所以 $z$ 必为一个偶数.\n设 $y=2 y_1, z=2 z_1$, 则方程(1)可分解为\n$$\n\\left(17^{z_1}-15^{y_1}\\right)\\left(17^{z_1}+15^{y_1}\\right)=8^x . \\label{eq2}\n$$\n易知式\\ref{eq2}中左边两个因数的最大公约数为 2 , 而式\\ref{eq2}的右边是 2 的幂, 故必须有\n$$\n\\left\\{\\begin{array}{l}\n17^{x_1}-15^{y_1}=2 . \\label{eq3}\\\\\n17^{x_1}+15^{y_1}=2^{3 x-1} . \\label{eq4}\n\\end{array}\\right.\n$$\n将式\\ref{eq3}模 32 可知, $z_1$ 与 $y_1$ 必须都是奇数 (否则, 式\\ref{eq3}的左边 $\\equiv 0,-14,16(\\bmod 32)$). 将式\\ref{eq3}、\\ref{eq4}相加, 得\n$$\n17^{z_1}=1+2^{3 x-2} . \\label{eq5}\n$$\n若 $x \\geqslant 3$, 则 式\\ref{eq5} 的右边 $\\equiv 1(\\bmod 32)$; 而因 $z_1$ 为奇数, 故左边 $\\equiv 17(\\bmod 32)$, 这不可能, 故必有 $x=2$. 由此及 式\\ref{eq5} 得 $z_1=1$, 即 $z=2$, 进而易知 $y_1=1$, 即 $y=2$. 因此 $x=y=z=2$.\n这一解法, 是同余结合分解方法的典型的例子.\n用同余导出 $y 、 z$ 均是偶数,正是为后面的分解方程作准备.", + "remark": "", + "figures": [] +} \ No newline at end of file diff --git a/processed_dataset/proof/0223.json b/processed_dataset/proof/0223.json new file mode 100644 index 0000000000000000000000000000000000000000..5f771793665d47662b210daa29cabd4a02d2c02d --- /dev/null +++ b/processed_dataset/proof/0223.json @@ -0,0 +1,8 @@ +{ + "source_file": "./raw_volume-zh/volume10/chapter9.tex", + "problem_type": "proof", + "problem": "例8. 证明: 不定方程\n$$\n(x+1)^y-x^z=1, x, y, z>1 . \\label{eq1}\n$$\n仅有一组正整数解 $x=2, y=2$ 及 $z=3$.", + "solution": "分析:这里介绍两种解法.\n第一种解法基于同余结合分解手法, 相当简单, 第二种解法采用比较素数幂方法, 虽然较为麻烦, 却具有一些代表性.\n证明一首先, 将方程式\\ref{eq1}模 $x+1$, 得\n$$\n-(-1)^z \\equiv 1(\\bmod x+1),\n$$\n故 $z$ 是奇数.\n将式\\ref{eq1}分解为\n$$\n(x+1)^{y-1}=x^{z-1}-x^{z-2}+\\cdots-x+1,\n$$\n由此易知 $x$ 是偶数.\n因为若 $x$ 为奇数, 则上式右边为奇数 $(z)$ 个奇数之和, 故是奇数, 而左边是偶数, 产生矛盾.\n同样, 将式\\ref{eq1}变形为\n$$\n(x+1)^{y-1}+(x+1)^{y-2}+\\cdots+(x+1)+1=x^{z-1},\n$$\n可见 $y$ 也是偶数.\n现在设 $x=2 x_1, y=2 y_1$, 则式\\ref{eq1}可分解为\n$$\n\\left((x+1)^{y_1}-1\\right)\\left((x+1)^{y_1}+1\\right)=x^z . \\label{eq2}\n$$\n因 $x$ 是偶数, 故 $(x+1)^{y_1}-1$ 与 $(x+1)^{y_1}+1$ 的最大公约数是 2 , 又显然有 $x \\mid(x+1)^{y_1}-1$. 由这些及式\\ref{eq2}推出, 必须\n$$\n(x+1)^{y_1}-1=2 x_1^z,(x+1)^{y_1}+1=2^{z-1} .\n$$\n因此 $2^{z-1}>2 x_1^z$, 故 $x_1=1$, 即 $x=2$, 从而易得 $y=2$ 及 $z=3$.", + "remark": "", + "figures": [] +} \ No newline at end of file diff --git a/processed_dataset/proof/0224.json b/processed_dataset/proof/0224.json new file mode 100644 index 0000000000000000000000000000000000000000..8689591f0da8165c122cafd2a8c18a3da3a3efbc --- /dev/null +++ b/processed_dataset/proof/0224.json @@ -0,0 +1,8 @@ +{ + "source_file": "./raw_volume-zh/volume10/chapter9.tex", + "problem_type": "proof", + "problem": "例8. 证明: 不定方程\n$$\n(x+1)^y-x^z=1, x, y, z>1 . \\label{eq1}\n$$\n仅有一组正整数解 $x=2, y=2$ 及 $z=3$.", + "solution": "证明二这一证明分两步进行.\n首先证明 $x$ 没有奇素数因子.\n采用反证法, 设有一个奇素数 $p$, 使 $p \\mid x$, 设 $x=p^a x_1$, 其中 $a \\geqslant 1, p \\nmid x_1$. 由二项式定理,可将式\\ref{eq1}变形为\n$$\nx y+\\sum_{i=2}^y \\mathrm{C}_y^i x^i=x^z . \\label{eq3}\n$$\n由此可见 $x^2 \\mid x y$, 即 $x \\mid y$, 从而 $p \\mid y$. 设 $y=p^b y_1, p \\nmid y_1$, 则 $b \\geqslant a$. 我们将通过比较\\ref{eq3}式两边所含 $p$ 的幂次来导出矛盾.\n对 $2 \\leqslant i \\leqslant y$, 设 $p^c \\| i$, 则在\n$$\n\\mathrm{C}_y^i x^i=\\frac{y}{i} \\mathrm{C}_{y-1}^{i-1} x^i=\\frac{p^b y_1}{i} \\mathrm{C}_{y-1}^{i-1}\\left(p^a x_1\\right)^i\n$$\n中, $p$ 的幂次至少是 $d=b+a i-c$. 若 $c=0$, 则 $d>a+b$; 若 $c>0$, 则由 $p \\geqslant$ 3 得 $p^c>c+1$, 又 $p^c \\mid i$, 故 $p^c \\leqslant i$. 因此, $i>c+1$, 从而\n$$\nd>b+a+c(a-1) \\geqslant a+b .\n$$\n故我们总有 $d \\geqslant a+b+1$, 于是 $p^{a+b+1} \\mid \\mathrm{C}_y^i x^i(2 \\leqslant i \\leqslant y)$, 进而有\n$$\np^{a+b+1} \\mid \\sum_{i=2}^y \\mathrm{C}_y^i x^i .\n$$\n又 $p^{a+b} \\| x y$, 因此\\ref{eq3}式左边含 $p$ 的幂次为 $a+b$.\n另一方面, 由于 $p^a \\| x$, 故 $p^{a z} \\| x^z$, 即 \\ref{eq3} 式右边含 $p$ 的幂次为 $a z$. 但由原方程式\\ref{eq1}可见 $z>y$, 又 $p^b \\mid y$, 故 $y \\geqslant p^b$, 从而\n$$\na z>a y \\geqslant a p^b \\geqslant a(b+1) \\geqslant a+b .\n$$\n因此\\ref{eq3}式左、右两边含 $p$ 的幕次不等, 这不可能.\n所以 $x$ 不含奇素数因子, 即 $x$ 为 2 的幕.\n设 $x=2^k(k \\geqslant 1)$. 由前面证明过的 $x \\mid y$, 可知 $y$ 是偶数,设 $y=2 y_1$. 方程式\\ref{eq1}可分解为\n$$\n\\left(\\left(2^k+1\\right)^{y_1}-1\\right)\\left(\\left(2^k+1\\right)^{y_1}+1\\right)=2^{k z},\n$$\n因上式左边两个因数的最大公约数为 2 , 而右边是 2 的幂, 故必须\n$$\n\\left(2^k+1\\right)^{y_1}-1=2,\\left(2^k+1\\right)^{y_1}+1=2^{k z-1} .\n$$\n因此 $k=y_1=1$, 即 $x=y=2$, 故 $z=3$.", + "remark": "", + "figures": [] +} \ No newline at end of file diff --git a/processed_dataset/proof/0225.json b/processed_dataset/proof/0225.json new file mode 100644 index 0000000000000000000000000000000000000000..0d2af16af81cd2e58145a9a3036d5dde8f6f4747 --- /dev/null +++ b/processed_dataset/proof/0225.json @@ -0,0 +1,8 @@ +{ + "source_file": "./raw_volume-zh/volume10/exercise1.tex", + "problem_type": "proof", + "problem": "问题3. 设正整数 $n$ 的十进制表示为 $n=\\overline{a_k \\cdots a_1 a_0}\\left(0 \\leqslant a_i \\leqslant 9, a_k \\neq 0\\right)$, 记 $T(n)=a_0-a_1+\\cdots+(-1)^k a_k$ (由 $n$ 的个位起始的数码的正、负交错和). 证明 $n-T(n)$ 被 11 整除.\n由此得出被 11 整除的数的数字特征: 11 整除 $n$ 的充分必要条件是 11 整除 $T(n)$.", + "solution": "我们有\n$$\nn-T(n)=\\left(a_0-a_0\\right)+\\left(10 a_1+a_1\\right)+\\cdots+\\left(a_k \\times 10^k-(-1)^k a_k\\right) .\n$$\n易知对 $i=0,1, \\cdots, k$, 数 $a_i \\times 10^i-(-1)^i a_i$ 被 11 整除(按 $i$ 为偶、奇数分别用分解式 (5)、(6)). 因此 $n-T(n)$ 被 11 整除, 故问题中两方面的结论均成立.", + "remark": "", + "figures": [] +} \ No newline at end of file diff --git a/processed_dataset/proof/0226.json b/processed_dataset/proof/0226.json new file mode 100644 index 0000000000000000000000000000000000000000..5544a8a78b966392a0c8ecb20194b0c561e2411b --- /dev/null +++ b/processed_dataset/proof/0226.json @@ -0,0 +1,8 @@ +{ + "source_file": "./raw_volume-zh/volume10/exercise1.tex", + "problem_type": "proof", + "problem": "问题4. 设 $n$ 个整数具有下述性质: 其中任意 $n-1$ 个数之积与剩下那个数的差都能被 $n$ 整除.\n证明: 这 $n$ 个数的平方和也能被 $n$ 整除.", + "solution": "设 $a_1, \\cdots, a_n$ 是具有所说性质的整数, $A$ 是它们的积, 对于 $1 \\leqslant i \\leqslant n$, 数 $n$ 整除 $\\frac{A}{a_i}-a_i$, 因而能整除\n$$\na_i\\left(\\frac{A}{a_i}-a_i\\right)=A-a_i^2 .\n$$\n故 $n$ 整除这些数的和 $\\left(A-a_1^2\\right)+\\cdots+\\left(A-a_n^2\\right)=n A-\\left(a_1^2+\\cdots+a_n^2\\right)$. 从而 $n$ 整除 $a_1^2+\\cdots+a_n^2$.", + "remark": "", + "figures": [] +} \ No newline at end of file diff --git a/processed_dataset/proof/0227.json b/processed_dataset/proof/0227.json new file mode 100644 index 0000000000000000000000000000000000000000..f987a4fc5873b11b7dab3a670787f8cff8472294 --- /dev/null +++ b/processed_dataset/proof/0227.json @@ -0,0 +1,8 @@ +{ + "source_file": "./raw_volume-zh/volume10/exercise1.tex", + "problem_type": "proof", + "problem": "问题5. 设整数 $a 、 b 、 c 、 d$ 满足 $a d-b c>1$, 证明: $a 、 b 、 c 、 d$ 中至少有一个数不被 $a d-b c$ 整除.", + "solution": "若 $a 、 b 、 c 、 d$ 都被 $a d-b c$ 整除, 则 $(a d-b c)^2$ 整除 $a d$ 及 $b c$, 故整除 $a d-b c$, 由此得知 $|a d-b c|=1$, 这与已知 $a d-b c>1$ 矛盾.", + "remark": "", + "figures": [] +} \ No newline at end of file diff --git a/processed_dataset/proof/0228.json b/processed_dataset/proof/0228.json new file mode 100644 index 0000000000000000000000000000000000000000..cf9749a98f32170935fbad457108d0ad61dcabca --- /dev/null +++ b/processed_dataset/proof/0228.json @@ -0,0 +1,8 @@ +{ + "source_file": "./raw_volume-zh/volume10/exercise10.tex", + "problem_type": "proof", + "problem": "问题1. 证明,对任意整数 $a \\geqslant 3$, 有无穷多个正整数 $n$, 使得 $a^n-1$ 被 $n$ 整除.", + "solution": "因 $a \\geqslant 3$, 故 $a-1$ 有素因子 $p$. 由费马小定理知, $a^p \\equiv a \\equiv 1(\\bmod p)$. 用归纳法易证, $n=p^k(k=1,2, \\cdots)$ 均符合要求.", + "remark": "", + "figures": [] +} \ No newline at end of file diff --git a/processed_dataset/proof/0229.json b/processed_dataset/proof/0229.json new file mode 100644 index 0000000000000000000000000000000000000000..5593430cd2511ae7ec43ba19240f8ce70ff46e5c --- /dev/null +++ b/processed_dataset/proof/0229.json @@ -0,0 +1,8 @@ +{ + "source_file": "./raw_volume-zh/volume10/exercise10.tex", + "problem_type": "proof", + "problem": "问题2. 设 $n_1, \\cdots, n_k$ 为正整数, 具有下面的性质:\n$$\nn_1\\left|\\left(2^{n_2}-1\\right), n_2\\right|\\left(2^{n_3}-1\\right), \\cdots, n_k \\mid\\left(2^{n_1}-1\\right) .\n$$\n证明: $n_1=\\cdots=n_k=1$.", + "solution": "已知条件可重述为\n$$\n2^{n_2} \\equiv 1\\left(\\bmod n_1\\right), 2^{n_3} \\equiv 1\\left(\\bmod n_2\\right), \\cdots, 2^{n_1} \\equiv 1\\left(\\bmod n_k\\right) .\n$$\n设 $D=\\left[n_1, \\cdots, n_k\\right]$. 则由上式得出\n$$\n2^D \\equiv 1\\left(\\bmod n_i\\right)(i=1, \\cdots, k),\n$$\n从而 $2^D \\equiv 1(\\bmod D)$, 故由第 8 单元中例 2 知 $D=1$, 所以 $n_1=n_2=\\cdots=n_k=1$.", + "remark": "", + "figures": [] +} \ No newline at end of file diff --git a/processed_dataset/proof/0230.json b/processed_dataset/proof/0230.json new file mode 100644 index 0000000000000000000000000000000000000000..2a628bba9c7f64b4270dbbfcc670b3b838380da9 --- /dev/null +++ b/processed_dataset/proof/0230.json @@ -0,0 +1,8 @@ +{ + "source_file": "./raw_volume-zh/volume10/exercise10.tex", + "problem_type": "proof", + "problem": "问题3. 设正整数 $a 、 b$ 满足 $a^2 b \\mid\\left(a^3+b^3\\right)$, 证明 $a=b$.", + "solution": "设 $a^3+b^3=m a^2 b$, 则 $\\left(\\frac{a}{b}\\right)^3-m\\left(\\frac{a}{b}\\right)^2+1=0$, 即有理数 $\\frac{a}{b}$ 是首项系数为 1 的整系数方程\n$$\nx^3-m x^2+1=0 . \\label{eq1}\n$$\n的一个根, 故 $\\frac{a}{b}$ 必是整数 . 另一方面, 方程式\\ref{eq1}的任一整数根必然整除常数项 1 , 从而只能是 \\pm 1 ; 又 $a, b$ 为正数, 故 $\\frac{a}{b}=1$, 即 $a=b$.", + "remark": "", + "figures": [] +} \ No newline at end of file diff --git a/processed_dataset/proof/0231.json b/processed_dataset/proof/0231.json new file mode 100644 index 0000000000000000000000000000000000000000..943c1822495e06d97bb8b341db51c99913fc9d21 --- /dev/null +++ b/processed_dataset/proof/0231.json @@ -0,0 +1,8 @@ +{ + "source_file": "./raw_volume-zh/volume10/exercise10.tex", + "problem_type": "proof", + "problem": "问题4. 证明: 不定方程 $x^n+1=y^{n+1}$ 没有正整数解 $(x, y, n)$, 其中 $(x, n+1)=1$, $n>1$.", + "solution": "显然 $y>1$. 原方程可分解为\n$$\n(y-1)\\left(y^n+y^{n-1}+\\cdots+y+1\\right)=x^n . \\label{eq1}\n$$\n关键是证明, $y-1$ 与 $y^n+y^{n-1}+\\cdots+y+1$ 互素.\n若它们的最大公约数 $d>$ 1 , 则 $d$ 有素因子 $p$. 由 $y \\equiv 1(\\bmod p)$ 知, $y^i \\equiv 1(\\bmod p)$, 从而有\n$$\ny^n+y^{n-1}+\\cdots+y+1 \\equiv n+1(\\bmod p),\n$$\n于是 $p \\mid(n+1)$; 但由 式\\ref{eq1} 又推出 $p \\mid x^n$, 从而素数 $p \\mid x$, 这与 $(x, n+1)=1$ 相违, 故 $d=1$. 现在由式\\ref{eq1}推出, 存在正整数 $a, b$, 使得\n$$\ny-1=a^n, y^n+y^{n-1}+\\cdots+y+1=b^n . \\label{eq2}\n$$\n但 $y^n1$, 则 $z$ 有素因子.\n若 $z$ 有奇素数因子 $p$, 我们设 $r$ 是使 $x^r \\equiv y^r(\\bmod p)$ 成立的最小正整数.\n由 式\\ref{eq1} 知 $x^n \\equiv y^n(\\bmod p)$, 故 $r \\mid n$. 设 $p^\\alpha\\left\\|n, p^\\beta\\right\\|\\left(x^r-y^r\\right)$ (注意, 因 $a \\neq b$, 故 $x \\neq y$ ), 则 知 $p^{\\alpha+\\beta} \\|\\left(x^n-y^n\\right)$, 但 式\\ref{eq1} 意味着 $p^n \\mid\\left(x^n-y^n\\right)$, 因此 $p^n \\leqslant p^{\\alpha+\\beta}$, 故 $n \\leqslant \\alpha+\\beta$, 又 $p^\\alpha \\leqslant n$, 故 $\\alpha \\leqslant \\log _p n$, 从而\n$$\nn \\leqslant \\log _p n+\\beta,\n$$\n这在 $n$ 充分大时不能成立 (注意 $\\beta$ 是一个固定的数), 因此 式\\ref{eq1}不可能对无穷多个 $n$ 成立, 矛盾.\n若 $z$ 没有奇素数, 则 $z$ 是 2 的幂.\n由 式\\ref{eq1}及 $(x, y, z)=1$ 知, $x, y$ 都是奇数.\n当 $n$ 为奇数时, 由\n$$\nx^n-y^n=(x-y)\\left(x^{n-1}+x^{n-2} y+\\cdots+x y^{n-2}+y^{n-1}\\right),\n$$\n并注意到上式右边第二个因子是奇数, 从而 $2^n \\mid\\left(x^n-y^n\\right)$ 意味着 $2^n \\mid(x-y)$, 因 $x \\neq y$, 故这样的 $n$ 只有有限多个.\n当 $n$ 为偶数时, 设 $2^s \\|\\left(x^2-y^2\\right)$, 若 $2^\\alpha \\| n$, 则 $2^{\\alpha+5-1} \\|\\left(x^n-y^n\\right)$. 结合 式\\ref{eq1}得 $n \\leqslant \\alpha+s-$ 1. 因 $\\alpha \\leqslant \\log _2 n$, 故\n$$\nn \\leqslant \\log _2 n+s-1,\n$$\n这对充分大的偶数 $n$ 不能成立,矛盾.", + "remark": "", + "figures": [] +} \ No newline at end of file diff --git a/processed_dataset/proof/0233.json b/processed_dataset/proof/0233.json new file mode 100644 index 0000000000000000000000000000000000000000..d023265e1b00f721832bee72455629452b4639de --- /dev/null +++ b/processed_dataset/proof/0233.json @@ -0,0 +1,8 @@ +{ + "source_file": "./raw_volume-zh/volume10/exercise10.tex", + "problem_type": "proof", + "problem": "问题6. 设 $n \\geqslant 4$ 是整数, $a_1, \\cdots, a_n$ 是小于 $2 n$ 的互不相同的正整数.\n证明: 从这些数中可取出若干个,使它们的和被 $2 n$ 整除.", + "solution": "若每个 $a_i$ 都不等于 $n$, 则结论易证.\n因为 $2 n$ 个数\n$$\na_1, a_2, \\cdots, a_n, 2 n-a_1, 2 n-a_2, \\cdots, 2 n-a_n .\n$$\n都是正整数, 且小于 $2 n$, 故其中必有两个相等, 即有 $i, j$ 使 $a_i=2 n-a_j$. 因 $i= j$ 意味着 $a_i=n$, 这与假设不符, 故 $i \\neq j$, 从而 $a_i+a_j=2 n$, 可被 $2 n$ 整除.\n现在无妨设 $a_n=n$. 考虑 $n-1(\\geqslant 3)$ 个整数 $a_1, a_2, \\cdots, a_{n-1}$, 这其中必有两个数的差不被 $n$ 整除, 因为,若所有的 $\\mathrm{C}_{n-1}^2$ 个两数之差都被 $n$ 整除, 则因 $\\mathrm{C}_{n-1}^2 \\geqslant 3$, 故有三个数 $a_i1$, 都存在连续 $n$ 个合数.", + "solution": "易于验证, $(n+1) !+2,(n+1) !+3, \\cdots,(n+1) !+(n+1)$ 是 $n$ 个连续的合数.", + "remark": "", + "figures": [] +} \ No newline at end of file diff --git a/processed_dataset/proof/0240.json b/processed_dataset/proof/0240.json new file mode 100644 index 0000000000000000000000000000000000000000..7e0c1398ac5a7dac8711f48b1cd004e5858f3d5d --- /dev/null +++ b/processed_dataset/proof/0240.json @@ -0,0 +1,8 @@ +{ + "source_file": "./raw_volume-zh/volume10/exercise3.tex", + "problem_type": "proof", + "problem": "问题2. 证明: 形如 $4 k-1$ 的素数有无穷多个,形如 $6 k-1$ 的素数也有无穷多个 $(k$ 为正整数).", + "solution": "可将欧几里得证明素数有无穷多个的方法稍作修改来论证: 假设 $4 k-$ 1 形式的素数只有有限多个, 设它们的全部为 $p_1, \\cdots, p_m$. 考虑数 $N= 4 p_1 \\cdots p_m-1$. 显然 $m>1$, 故 $N>1$, 故 $N$ 有素因子.\n进一步, 因两个 $4 k+1$ 形式的数之积仍具有形式 $4 k+1$, 而 $N$ 有形式 $4 k-1$, 故 $N$ 必有一个 $4 k-1$ 形式的素因子 $p$, 由前面的假设知, $p$ 应同于 $p_1, \\cdots, p_m$ 之一, 进而 $N-4 p_1 \\cdots p_m$ 被 $p$ 整除, 即 $p \\mid 1$, 矛盾.\n同样可证明,形如 $6 k-1$ 的素数有无穷多个.", + "remark": "", + "figures": [] +} \ No newline at end of file diff --git a/processed_dataset/proof/0241.json b/processed_dataset/proof/0241.json new file mode 100644 index 0000000000000000000000000000000000000000..e135ee01828c6eecdc1c40970968e3e6053f77ac --- /dev/null +++ b/processed_dataset/proof/0241.json @@ -0,0 +1,8 @@ +{ + "source_file": "./raw_volume-zh/volume10/exercise3.tex", + "problem_type": "proof", + "problem": "问题3. 证明: 有无穷多个奇数 $m$, 使 $8^m+9 m^2$ 是合数.", + "solution": "取 $m=9 k^3(k=1,3, \\cdots)$, 则 $8^m+9 m^2=\\left(2^m\\right)^3+\\left(9 k^2\\right)^3$. 易知它有真因子 $2^m+9 k^2$.", + "remark": "", + "figures": [] +} \ No newline at end of file diff --git a/processed_dataset/proof/0242.json b/processed_dataset/proof/0242.json new file mode 100644 index 0000000000000000000000000000000000000000..0594aff08dc2c9d586fd664024e820ec58aa8702 --- /dev/null +++ b/processed_dataset/proof/0242.json @@ -0,0 +1,8 @@ +{ + "source_file": "./raw_volume-zh/volume10/exercise3.tex", + "problem_type": "proof", + "problem": "问题4. 设整数 $a 、 b 、 c 、 d$ 满足 $a>b>c>d>0$, 且\n$$\na^2+a c-c^2=b^2+b d-d^2,\n$$\n证明: $a b+c d$ 不是素数.", + "solution": "反证法, 设有满足题设的一组 $a 、 b 、 c 、 d$, 使得 $a b+c d$ 为素数, 记之为 $p$, 将 $a=\\frac{p-c d}{b}$ 代入给出的等式, 得到\n$$\np(p-2 c d+b c)=\\left(b^2+c^2\\right)\\left(b^2+b d-d^2\\right) .\n$$\n因 $p$ 是素数,故 $p$ 整除 $b^2+c^2$, 或者 $p$ 整除 $b^2+b d-d^2$.\n若 $p \\mid\\left(b^2+c^2\\right)$, 则由 $04$, 故 $a>2$, 因而 $a^2>2 a$, 即 $m>2 a$. 所以 $a$ 与 $2 a$ 是数列 $1,2, \\cdots, m-1$ 中两个不同的项, 因此 $(m-1)$ ! 被 $a \\cdot 2 a= 2 m$ 整除.", + "remark": "", + "figures": [] +} \ No newline at end of file diff --git a/processed_dataset/proof/0245.json b/processed_dataset/proof/0245.json new file mode 100644 index 0000000000000000000000000000000000000000..8f2e8d4af51b205a212657c259112b78ee9edd06 --- /dev/null +++ b/processed_dataset/proof/0245.json @@ -0,0 +1,8 @@ +{ + "source_file": "./raw_volume-zh/volume10/exercise5.tex", + "problem_type": "proof", + "problem": "问题2. 证明: 正整数 $n$ 可以表示为连续若干个(至少两个) 正整数之和的充分必要条件是, $n$ 不是 2 的方幕.", + "solution": "设 $n=x+(x+1)+\\cdots+(x+k-1), x$ 为正整数, $k \\geqslant 2$. 即\n$$\n(2 x+k-1) k=2 n .\n$$\n若 $n$ 为 2 的方幂, 则 $k$ 与 $2 x-1+k$ 都是 2 的方幂, 但 $2 x-1$ 为奇数, 故必须 $k=1$, 这与题设不合.\n反过来, 若 $n$ 不是 2 的方幕, 设 $n=2^{m-1}(2 t+1), m \\geqslant 1, t \\geqslant 1$. 当 $t \\geqslant 2^{m-1}$ 时, 可取 $k=2^m, x=t+1-2^{m-1}$; 当 $t<2^{m-1}$ 时, 可取 $k=2 t+1, x= 2^{m-1}-t$. 则 $k$ 与 $x$ 都是正整数且 $k \\geqslant 2$.", + "remark": "", + "figures": [] +} \ No newline at end of file diff --git a/processed_dataset/proof/0246.json b/processed_dataset/proof/0246.json new file mode 100644 index 0000000000000000000000000000000000000000..20281fb4fb205aed5f86466288674f25bbe6c234 --- /dev/null +++ b/processed_dataset/proof/0246.json @@ -0,0 +1,8 @@ +{ + "source_file": "./raw_volume-zh/volume10/exercise5.tex", + "problem_type": "proof", + "problem": "问题3. 证明: 任意正整数 $n$ 可表示为 $a-b$ 的形式,其中 $a 、 b$ 为正整数,且 $a 、 b$ 的不同素因子的个数相同.", + "solution": "当 $n$ 为偶数时, 可取 $a=2 n, b=n$. 若 $n$ 为奇数, 设 $p$ 是不整除 $n$ 的最小奇素数,则 $p-1$ 或者没有奇素数因子 (即是 2 的幂), 或者其奇素数因子都整除 $n$. 因此 $a=p n, b=(p-1) n$ 的不同素因子的个数都等于 $n$ 的不同素因子个数加上 1 .", + "remark": "", + "figures": [] +} \ No newline at end of file diff --git a/processed_dataset/proof/0247.json b/processed_dataset/proof/0247.json new file mode 100644 index 0000000000000000000000000000000000000000..c72713a4582d76d2d9c02ae71c1d941d6db0867a --- /dev/null +++ b/processed_dataset/proof/0247.json @@ -0,0 +1,8 @@ +{ + "source_file": "./raw_volume-zh/volume10/exercise5.tex", + "problem_type": "proof", + "problem": "问题4. 任意给定整数 $n \\geqslant 3$, 证明, 存在一个由正整数组成的 $n$ 项的等差数列(公差不为 0$)$,其中任意两项互素.", + "solution": "数列 $\\{k \\cdot n !+1\\}(k=1, \\cdots, n)$ 符合要求.\n假设有 $s 、 t(1 \\leqslant s3$, 故此时 $N$ 不是素数.\n现在设 $3 \\nmid n$. 注意 $10^6 \\equiv 1(\\bmod 7)$, 我们因此将 $n$ 模 6 分类, 来讨论 $A_n$ 模 7 的值 $(n \\equiv 0,3(\\bmod 6)$ 的情形已不必考虑). 易于得知, 对 $l \\geqslant 0$,\n$$\n\\begin{aligned}\nA_{6 l+1} & =\\frac{1}{9} \\times\\left(10^{6 l+1}-1\\right)=\\frac{1}{9} \\times\\left(10^{6 l}-1\\right) \\times 10+\\frac{1}{9} \\times(10-1) \\\\\n& \\equiv 1(\\bmod 7), \\\\\nA_{6 l+2} & \\equiv 4, A_{6 l+4} \\equiv 5, A_{6 l+5} \\equiv 2(\\bmod 7) .\n\\end{aligned}\n$$\n此外, $10^0, 10^2, 10^4, 10^5$ 模 7 依次同余于 $1,2,4,5$. 因而当 $n>6$ 时, 按 $n \\equiv 1,2,4,5(\\bmod 6)$, 分别取 $k=0,4,5,2$, 即知\n$$\nN=A_n+6 \\times 10^k \\equiv A_n-10^k \\equiv 0(\\bmod 7),\n$$\n故 $N$ 不是素数, 从而大于 5 的 $n$ 均不合要求.\n在 $n \\leqslant 5$ 时, 不难验证只有 $n=$ 1,2 合要求.", + "remark": "", + "figures": [] +} \ No newline at end of file diff --git a/processed_dataset/proof/0251.json b/processed_dataset/proof/0251.json new file mode 100644 index 0000000000000000000000000000000000000000..e171926c855a8be293ff02ceb13376ea0447ae15 --- /dev/null +++ b/processed_dataset/proof/0251.json @@ -0,0 +1,8 @@ +{ + "source_file": "./raw_volume-zh/volume10/exercise6.tex", + "problem_type": "proof", + "problem": "问题3. 设 $p$ 是素数, $a \\geqslant 2, m \\geqslant 1, a^m \\equiv 1(\\bmod p), a^{p-1} \\equiv 1\\left(\\bmod p^2\\right)$. 证明: $a^m \\equiv 1\\left(\\bmod p^2\\right)$.", + "solution": "由 $a^m \\equiv 1(\\bmod p)$ 得 $a^m=1+p x$. 因此\n$$\na^{p m}=(1+p x)^p=1+p^2 x+C_p^2 p^2 x^2+\\cdots \\equiv 1\\left(\\bmod p^2\\right) . \\label{eq1}\n$$\n又 $a^{p-1} \\equiv 1\\left(\\bmod p^2\\right)$, 故 $a^{(p-1) m} \\equiv 1\\left(\\bmod p^2\\right)$, 从而 $a^{p m} \\equiv a^m\\left(\\bmod p^2\\right)$. 结合 式\\ref{eq1} 知 $a^m \\equiv 1\\left(\\bmod p^2\\right)$.", + "remark": "", + "figures": [] +} \ No newline at end of file diff --git a/processed_dataset/proof/0252.json b/processed_dataset/proof/0252.json new file mode 100644 index 0000000000000000000000000000000000000000..a5e9c080069f1a856e1db63b3b926caae464ace5 --- /dev/null +++ b/processed_dataset/proof/0252.json @@ -0,0 +1,8 @@ +{ + "source_file": "./raw_volume-zh/volume10/exercise6.tex", + "problem_type": "proof", + "problem": "问题4. 设 $m$ 是给定的正整数, 证明: 由\n$$\nx_1=x_2=1, x_{k+2}=x_{k+1}+x_k(k=1,2, \\cdots)\n$$\n定义的数列 $\\left\\{x_n\\right\\}$ 的前 $m^2$ 个项中, 必有一项被 $m$ 整除.", + "solution": "无妨设 $m>1$. 我们用 $\\bar{x}_k$ 表示 $x_k$ 被 $m$ 除得的余数.\n考虑有序数对\n$$\n\\left\\langle\\bar{x}_1, \\bar{x}_2\\right\\rangle,\\left\\langle\\bar{x}_2, \\bar{x}_3\\right\\rangle, \\cdots,\\left\\langle\\bar{x}_n, \\bar{x}_{n+1}\\right\\rangle, \\cdots . \\label{eq1}\n$$\n因为被 $m$ 除得的余数共组成 $m^2$ 个互不相等的有序数对, 故在序列 式\\ref{eq1}中取出前 $m^2+1$ 个数对, 其中必有两个相同.\n设 $\\left\\langle\\bar{x}_i, \\bar{x}_{i+1}\\right\\rangle$ 是下标最小的与某一个 $\\left\\langle\\bar{x}_j, \\bar{x}_{j+1}\\right\\rangle$ 相等的数对 $\\left(j \\leqslant m^2+1\\right)$, 我们证明 $i$ 必然是 1 , 否则从\n$$\nx_{i-1}=x_{i+1}-x_i, x_{j-1}=x_{j+1}-x_j\n$$\n推出 $x_{i-1} \\equiv x_{j-1}(\\bmod m)$, 故 $\\left\\langle\\bar{x}_{i-1}, \\bar{x}_i\\right\\rangle=\\left\\langle\\bar{x}_{j-1}, \\bar{x}_j\\right\\rangle$, 这与 $i$ 的最小性矛盾, 所以 $i=1$. 现在由 $\\left\\langle\\bar{x}_j, \\bar{x}_{j+1}\\right\\rangle=\\left\\langle\\bar{x}_1, \\bar{x}_2\\right\\rangle=\\langle 1,1\\rangle$. 可知 $x_{j-1} \\equiv x_{j+1}-x_j \\equiv 1- 1 \\equiv 0(\\bmod m)$, 即 $m \\mid x_{j-1}\\left(13$, 故由费马小定理得 $p \\mid\\left(2^{p-1}-1\\right)$. 结合 式\\ref{eq1} 推出 $2 p \\mid(n-1)$, 从而 $\\left(2^{2 p}-1\\right) \\mid\\left(2^{n-1}-1\\right)$. 再由条件知 $n \\mid\\left(2^{2 p}-1\\right)$, 所以 $n \\mid\\left(2^{n-1}-1\\right)$.", + "remark": "", + "figures": [] +} \ No newline at end of file diff --git a/processed_dataset/proof/0254.json b/processed_dataset/proof/0254.json new file mode 100644 index 0000000000000000000000000000000000000000..6da94138a0f5851ccd3e04617e7864b8ad80141d --- /dev/null +++ b/processed_dataset/proof/0254.json @@ -0,0 +1,8 @@ +{ + "source_file": "./raw_volume-zh/volume10/exercise7.tex", + "problem_type": "proof", + "problem": "问题2. 设 $m \\geqslant 2, a_1, a_2, \\cdots, a_m$ 是给定的正整数.\n证明: 有无穷多个正整数 $n$, 使得数 $a_1 \\cdot 1^n+a_2 \\cdot 2^n+\\cdots+a_m \\cdot m^n$ 都是合数.", + "solution": "显然 $a_1+2 a_2+\\cdots+m a_m>1$, 故有素数 $p$ 整除 $a_1+2 a_2+\\cdots+m a_m$. 取 $n=k(p-1)+1, k=1,2, \\cdots$, 则对 $1 \\leqslant i \\leqslant m$, 若 $p \\nmid i$, 由费马小定理知\n$$\ni^n=i \\cdot\\left(i^k\\right)^{p-1} \\equiv i(\\bmod p) .\n$$\n若 $p \\mid i$, 上式显然也成立.\n因此\n$$\na_1 \\cdot 1^n+a_2 \\cdot 2^n+\\cdots+a_m \\cdot m^n \\equiv a_1+2 a_2+\\cdots+m a_m \\equiv 0(\\bmod p),\n$$\n又 $a_1 \\cdot 1^n+a_2 \\cdot 2^n+\\cdots+a_m \\cdot m^n$ 显然大于 $p$, 故它是一个合数.\n因此上述选取的 $n$ 符合要求, 这显然有无穷多个.", + "remark": "", + "figures": [] +} \ No newline at end of file diff --git a/processed_dataset/proof/0255.json b/processed_dataset/proof/0255.json new file mode 100644 index 0000000000000000000000000000000000000000..15e7f709cca72e600c363db3b062be84aed280de --- /dev/null +++ b/processed_dataset/proof/0255.json @@ -0,0 +1,8 @@ +{ + "source_file": "./raw_volume-zh/volume10/exercise7.tex", + "problem_type": "proof", + "problem": "问题3. 设 $m 、 n$ 为正整数,且 $m \\geqslant n$, 具有性质: 等式\n$$\n(11 k-1, m)=(11 k-1, n)\n$$\n对所有正整数 $k$ 成立.\n证明: $m=11^r n, r$ 是一个非负整数.", + "solution": "设 $m=11^i u, n=11^j v$, 其中 $i, j$ 为非负整数, $u, v$ 为不被 11 整除的正整数.\n我们证明必有 $u=v$, 由此即知 $m=11^{i j} n$. 若 $u \\neq v$, 无妨设 $u>v$. 因 $(u, 11)=1$, 故由中国剩余定理,有正整数 $x$,使得\n$$\nx \\equiv 0(\\bmod u), x \\equiv-1(\\bmod 11), \\label{eq1}\n$$\n即 $x=11 k-1$ ( $k$ 为某个正整数). 由 式\\ref{eq1} 易知 $(11 k-1, m)=\\left(x, 11^i u\\right)=u$, 但 $(11 k-1, n)=\\left(x, 11^j v\\right) \\leqslant v0$, 都存在正整数 $n$, 使得 $2^k \\mid\\left(3^n+5\\right)$.", + "solution": "采用归纳法.\n$k=1,2$ 时结论显然成立.\n设对 $k \\geqslant 3$ 有 $n_0$ 使得 $2^k \\left(3^{n_0}+5\\right)$, 设 $3^{n_0}=2^k u-5$. 若 $u$ 是偶数,则 $2^{k+1} \\mid\\left(3^{n_0}+5\\right)$. 以下设 $u$ 是奇数.\n论证的关键是注意, 对 $k \\geqslant 3$ 有\n$$\n3^{2^{k-2}}=1+2^k v, v \\text { 是奇数.\n}\n$$\n现在我们有\n$$\n\\begin{aligned}\n3^{n_0+2^{k-2}} & =3^{n_0} \\cdot 3^{2^{k^{-2}}}=\\left(-5+2^k u\\right)\\left(1+2^k v\\right) \\\\\n& =-5+\\left(u-5 v+2^k u v\\right) \\cdot 2^k .\n\\end{aligned}\n$$\n上式括号内的数是偶数,故 $2^{k+1}$ 整除 $3^{n_0+2^{k-2}}+5$. 这就完成了归纳证明.", + "remark": "", + "figures": [] +} \ No newline at end of file diff --git a/processed_dataset/proof/0258.json b/processed_dataset/proof/0258.json new file mode 100644 index 0000000000000000000000000000000000000000..3c843b5b71c685792bdba7015fd07d4f75ec0346 --- /dev/null +++ b/processed_dataset/proof/0258.json @@ -0,0 +1,8 @@ +{ + "source_file": "./raw_volume-zh/volume10/exercise8.tex", + "problem_type": "proof", + "problem": "问题4. 证明, 若整数 $n>1$, 则 $n \\nmid 3^n-2^n$.", + "solution": "反证法,设有 $n>1$, 使 $n \\mid 3^n-2^n$. 设 $p$ 是 $n$ 的最小素因子, 则 $3^n \\equiv 2^n(\\bmod p)$, 从而 $p \\geqslant 5$. 故有整数 $a$, 使得 $2 a \\equiv 1(\\bmod p)$. 因此有\n$$\n(3 a)^n \\equiv 1(\\bmod p) .\n$$\n设 $d$ 是 $3 a$ 模 $p$ 的阶.\n由上式知 $d \\mid n$. 又费马小定理给出 $(3 a)_{-3}^{p-1} \\equiv 1(\\bmod p)$ ,\n故 $d \\mid p-1$. 若 $d>1$, 则 $d$ 有素因子 $q$, 而由 $d \\mid n$ 知 $q \\mid n$; 由 $d \\mid p-1$ 知 $q< p$, 这与 $p$ 的选取相违,故 $d=1$. 从而 $3 a \\equiv 1(\\bmod p)$, 结合 $2 a \\equiv 1(\\bmod p)$ 可知 $a \\equiv 1(\\bmod p)$, 进而 $2 a \\equiv 2(\\bmod p)$, 产生矛盾.", + "remark": "", + "figures": [] +} \ No newline at end of file diff --git a/processed_dataset/proof/0259.json b/processed_dataset/proof/0259.json new file mode 100644 index 0000000000000000000000000000000000000000..45c25152a1354484648f85a3b075c932e33db088 --- /dev/null +++ b/processed_dataset/proof/0259.json @@ -0,0 +1,8 @@ +{ + "source_file": "./raw_volume-zh/volume10/exercise9.tex", + "problem_type": "proof", + "problem": "问题1. 证明不定方程\n$$\nx^2+3 x y-2 y^2=122\n$$\n没有整数解.", + "solution": "将方程配方成\n$$\n(2 x+3 y)^2=17 y^2+4 \\times 122,\n$$\n模 17 得 $(2 x+3 y)^2 \\equiv 12(\\bmod 17)$. 但易于验证,一个整数的平方模 17 只可能取 $0 、 1 、 2 、 4 、 8 、 9 、 13 、 15 、 16$ 之一, 不能为 12 . 因此原方程无整数解.", + "remark": "", + "figures": [] +} \ No newline at end of file diff --git a/processed_dataset/proof/0260.json b/processed_dataset/proof/0260.json new file mode 100644 index 0000000000000000000000000000000000000000..42ccb838a7fff1aa13e5416c2ddbb5501a78b72c --- /dev/null +++ b/processed_dataset/proof/0260.json @@ -0,0 +1,8 @@ +{ + "source_file": "./raw_volume-zh/volume10/exercise9.tex", + "problem_type": "proof", + "problem": "问题2. 求所有正整数 $m 、 n$, 使得 $\\left|12^m-5^n\\right|=7$.", + "solution": "模 4 即知方程\n$$\n12^m-5^n=-7\n$$\n无正整数解.\n方程\n$$\n12^m-5^n=7 . \\label{eq1}\n$$\n显然有解 $m=n=1$. 下面证明当 $m>1$ 时它无正整数解.\n将 式\\ref{eq1}模 3 得 $-(-1)^n \\equiv 1(\\bmod 3)$, 故 $n$ 为奇数, 因此 $5^n \\equiv 5(\\bmod 8)$. 又 $m \\geqslant 2$, 故 $8 \\mid 12^m$.\n将 式\\ref{eq1} 模 8 得出 $-5 \\equiv 7(\\bmod 8)$, 这不可能.\n所以 $m=1$, 从而 $n=1$.", + "remark": "", + "figures": [] +} \ No newline at end of file diff --git a/processed_dataset/proof/0261.json b/processed_dataset/proof/0261.json new file mode 100644 index 0000000000000000000000000000000000000000..02e68adb19ca75ca9e202c118e05e40142cc22e7 --- /dev/null +++ b/processed_dataset/proof/0261.json @@ -0,0 +1,8 @@ +{ + "source_file": "./raw_volume-zh/volume10/exercise9.tex", + "problem_type": "proof", + "problem": "问题4. 证明: 不定方程\n$$\n5^x-3^y=2\n$$\n仅有正整数解 $x=y=1$.", + "solution": "方程显然有解 $x=y=1$. 将方程模 4 易知 $y$ 为奇数.\n若 $y>1$, 将方程模 9 得\n$$\n5^x \\equiv 2(\\bmod 9) . \\label{eq1}\n$$\n不难求得对 $x=1,2, \\cdots, 5^x$ 模 9 周期地为 $5,7,8,4,2,1$. 故由 式\\ref{eq1} 知 $x$ 必有形式 $6 k+5$. 再将原方程模 7 ,易验证,对奇数 $y$, 有\n$$\n3^y \\equiv 3,5,6(\\bmod 7) \\text {. }\n$$\n而 $x=6 k+5$ 时, 由费马小定理知 $5^6 \\equiv 1(\\bmod 7)$, 故\n$$\n5^x=5^{6 k+5} \\equiv 5^5 \\equiv 3(\\bmod 7),\n$$\n从而原方程两边模 7 不等, 因此它没有 $y>1$ 的解, 故仅有的正整数解为 $y= 1, x=1$.", + "remark": "", + "figures": [] +} \ No newline at end of file diff --git a/processed_dataset/proof/0262.json b/processed_dataset/proof/0262.json new file mode 100644 index 0000000000000000000000000000000000000000..a3e1d28d37ddcb507d842f6b5cf495e64a32683e --- /dev/null +++ b/processed_dataset/proof/0262.json @@ -0,0 +1,8 @@ +{ + "source_file": "./raw_volume-zh/volume10/exercise9.tex", + "problem_type": "proof", + "problem": "问题5. 证明 $x^3+y^4=7$ 没有整数解.", + "solution": "易于验证, $x^3 \\equiv 0,1,5,8,12(\\bmod 13) ; y^4 \\equiv 0,1,3,9(\\bmod 13)$.\n由这些易知 $x^3+y^4 \\neq 7(\\bmod 13)$, 故方程无整数解.", + "remark": "", + "figures": [] +} \ No newline at end of file diff --git a/processed_dataset/proof/0263.json b/processed_dataset/proof/0263.json new file mode 100644 index 0000000000000000000000000000000000000000..437cb98ce652362759962d19436b6c16bcce3846 --- /dev/null +++ b/processed_dataset/proof/0263.json @@ -0,0 +1,8 @@ +{ + "source_file": "./raw_volume-zh/volume11/chapter1.tex", + "problem_type": "proof", + "problem": "例11. 设 $\\varphi$ 是集合 $X=\\{1,2, \\cdots, n\\}$ 上的置换, 将 $X$ 上没有不动点的置换个数记为 $f_n$, 恰有一个不动点的置换个数记为 $g_n$, 证明: $\\left|f_n-g_n\\right|=1$.", + "solution": "证明:设 $g_{n_i}(i=1,2, \\cdots, n)$ 表示 $X$ 上恰有唯一不动点 $i$ 的置换个数.\n于是\n$$\ng_n=g_{n_1}+g_{n_2}+\\cdots+g_{n_n} .\n$$\n由上述推论, 有\n$$\nf_n=D_n, g_{n_i}=D_{n-1}(i=1,2, \\cdots, n),\n$$\n故\n$$\ng_n=n D_{n-1}\n$$\n所以\n$$\n\\begin{aligned}\n\\left|f_n-g_n\\right|= & \\left|D_n-n D_{n-1}\\right| \\\\\n= & \\mid n !\\left(1-\\frac{1}{1 !}+\\frac{1}{2 !}-\\cdots+\\frac{(-1)^{n-1}}{(n-1) !}+\\frac{(-1)^n}{n !}\\right)- \\\\\n& n \\cdot(n-1) !\\left(1-\\frac{1}{1 !}+\\frac{1}{2 !}-\\cdots+\\frac{(-1)^{n-1}}{(n-1) !}\\right) \\mid \\\\\n= & \\left|n ! \\cdot \\frac{(-1)^n}{n !}\\right| \\\\\n= & 1 .\n\\end{aligned}\n$$", + "remark": "", + "figures": [] +} \ No newline at end of file diff --git a/processed_dataset/proof/0264.json b/processed_dataset/proof/0264.json new file mode 100644 index 0000000000000000000000000000000000000000..72d44d2af983c9cfc0ba57ce82a9f5748e7d8e2d --- /dev/null +++ b/processed_dataset/proof/0264.json @@ -0,0 +1,8 @@ +{ + "source_file": "./raw_volume-zh/volume11/chapter10.tex", + "problem_type": "proof", + "problem": "例1. 在直角坐标平面内给定凸五边形 $A B C D E$, 它的顶点都是整点 (横坐标及纵坐标都为整数的点). 证明:其对角线相交成的凸五边形 $A_1 B_1 C_1 D_1 E_1$ (图 10-1) 的内部或周界上至少有一个整点.", + "solution": "证明:为了说话简便, 将\"在内部或在边界上\"统称为\"在……中\", 用反证法.\n假设结论不成立, 考虑不满足题目结论且具有最小面积 $S$ 的凸五边形 $A B C D E$\n(因整点五边形的面积的 2 倍是正整数,故其中必存在面积最小的) 并称凸五边形 $A_1 B_1 C_1 D_1 E_1$ 为 $A B C D E$ 的内五边形, 不妨设 $\\triangle A B C, \\triangle B C D, \\triangle C D E$, $\\triangle D E A, \\triangle E A B$ 中以 $\\triangle A B C$ 的面积为最小.\n为了导致矛盾, 我们只需证明以 $A B 、 B C$ 为邻边的平行四边形的另一个顶点 $O$ (必为整点) 在内五边形中, 为此, 我们先证明 $\\triangle A C_1 D_1$ 中的整点除 $A$ 外, 只可能在线段 $C_1 D_1$ 上.\n事实上若除了 $A$ 或 $C_1 D_1$ 上的点外, $\\triangle A C_1 D_1$ 中还有一点 $K$ 为整点, 则凸五边形 $K B C D E$ 的面积小于凸五边形 $A B C D E$ 的面积, 且 $K B C D E$ 的内五边形包含在 $A_1 B_1 C_1 D_1 E_1$ 中, 故 $K B C D E$ 的内五边形中也没有整点,这与 $S$ 的最小性假设矛盾.\n其次, 因 $S_{\\triangle A B C} \\leqslant S_{\\triangle B C D}$, 故 $D$ 到 $B C$ 的距离 $\\geqslant A$ 到 $B C$ 的距离, 又 $S_{\\triangle A B C} \\leqslant S_{\\triangle A B E}$, 故 $E$ 到 $A B$ 的距离 $\\geqslant C$ 到 $A B$ 的距离, 所以, 以 $A B 、 B C$ 为邻边的平行四边形的另一顶点 $O$ 必在 $\\triangle A C B_1$ 内, 且由 $A, B, C$ 为整点知 $O$ 为整点, 且 $O \\neq A, O \\neq C$. \n若 $O$ 在 $\\triangle A C_1 D_1$ 中则由前面证明知 $O$ 只能在 $C_1 D_1$ 上.\n同理,若 $O$ 在 $\\triangle C A_1 E_1$ 中, 则 $O$ 只能在 $A_1 E_1$ 上, 即 $O$ 只能在内五边形\n$A_1 B_1 C_1 D_1 E_1$ 中, 这与 $A_1 B_1 C_1 D_1 E_1$ 中无整点的假设矛盾.\n于是命题结论成立.", + "remark": "注:用反证法证明涉及整点多边形的结论时, 常常要取满足假设条件而且具有最小面积的整点多边形作为证明的出发点.", + "figures": [] +} \ No newline at end of file diff --git a/processed_dataset/proof/0265.json b/processed_dataset/proof/0265.json new file mode 100644 index 0000000000000000000000000000000000000000..f738a496c55535d756acd8e8e5fae9dc518f32a0 --- /dev/null +++ b/processed_dataset/proof/0265.json @@ -0,0 +1,8 @@ +{ + "source_file": "./raw_volume-zh/volume11/chapter10.tex", + "problem_type": "proof", + "problem": "例2. 将 100 个互异实数分别放置在圆周上的不同地方.\n求证:一定存在相邻的 4 个数使得两端的两数之和大于中间的两数之和.", + "solution": "证明:假设结论不成立, 设圆周上 100 个实数为 $a_1, a_2, \\cdots, a_{100}$ 且 $a_{n+100}=a_n$, 则\n$$\na_n+a_{n+3} \\leqslant a_{n+1}+a_{n+2},\n$$\n即\n$$\na_{n+3}-a_{n+2} \\leqslant a_{n+1}-a_n, n=1,2, \\cdots, 100 .\n$$\n由此可得\n$$\na_{100}-a_{99} \\leqslant a_{98}-a_{97} \\leqslant \\cdots \\leqslant a_2-a_1 \\leqslant a_{100}-a_{99} .\n$$\n因此 $a_{2 k}-a_{2 k-1}=m(m$ 为常数, $k=1,2, \\cdots, 50)$. 同理可得 $a_{2 k+1}-a_{2 k}=l(l$ 为常数, $k=1,2, \\cdots, 50)$. 将所有这些等式相加得 $0=50 m+50 l$, 于是 $m= -l$. 这推出\n$$\na_3-a_2=l=-m=a_1-a_2,\n$$\n即 $a_1=a_3$, 矛盾.\n故题中结论成立.", + "remark": "", + "figures": [] +} \ No newline at end of file diff --git a/processed_dataset/proof/0266.json b/processed_dataset/proof/0266.json new file mode 100644 index 0000000000000000000000000000000000000000..06da9d5879c813aa828b8eacf8cd1f248c153865 --- /dev/null +++ b/processed_dataset/proof/0266.json @@ -0,0 +1,8 @@ +{ + "source_file": "./raw_volume-zh/volume11/chapter10.tex", + "problem_type": "proof", + "problem": "例3. 证明: 在任意 $n(\\geqslant 4)$ 个人中都存在 2 人 $A$ 和 $B$ 使得其余 $n-2$ 人中至少有 $\\left[\\frac{n}{2}\\right]-1$ 人满足: 他们中每个人或者同 $A, B$ 都互相认识或者同 $A$, $B$ 都互相不认识.", + "solution": "证法一用反证法.\n假设结论不成立, 那么对 $n$ 个人中任意 2 个人 $A$ 和 $B$, 在其余 $n-2$ 人中同时与 $A, B$ 互相认识以及同时与 $A, B$ 互相不认识的人一共至多只有 $\\left[\\frac{n}{2}\\right]-2$ 个.\n再设在其余 $n-2$ 人中恰与 $A, B$ 中一人互相认识的有 $k$ 人.\n则\n$$\nn-2-k \\leqslant\\left[\\frac{n}{2}\\right]-2,\n$$\n所以\n$$\nk \\geqslant n-\\left[\\frac{n}{2}\\right] \\geqslant n-\\frac{n}{2}=\\frac{n}{2},\n$$\n即对任意两人 $A, B$, 恰与 $A, B$ 中一个人互相认识的人至少有 $\\frac{n}{2}$ 个.\n若 $C$ 恰与 $A, B$ 中一个人互相认识, 则将 $(A, B, C)$ 组成一个三元组.\n设这种三元组的个数为 $S$.\n因为对任意两人 $A, B$,含 $A, B$ 的三元组至少有 $\\frac{n}{2}$ 个, 而 $(A, B)$ 对有 $\\mathrm{C}_n^2$ 种取法,故\n$$\nS \\geqslant \\frac{n}{2} C_n^2=\\frac{n^2(n-1)}{4} . \\label{eq1}\n$$\n另一方面, 对 $n$ 个人中任何 1 人 $C$, 设其余 $n-1$ 个人中有 $h$ 个同 $C$ 互相认识, $n-1-h$ 个人同 $C$ 互相不认识.\n故含 $C$ 的三元组 $(A, B, C)$ 的个数为 $h(n-1-h) \\leqslant\\left[\\frac{h+(n-1-h)}{2}\\right]^2=\\frac{(n-1)^2}{4}$. 而 $C$ 有 $n$ 种不同的取法,故\n$$\nS \\leqslant \\frac{n(n-1)^2}{4} . \\label{eq2}\n$$\n式\\ref{eq2}与\\ref{eq1}矛盾.\n于是题中结论成立.", + "remark": "", + "figures": [] +} \ No newline at end of file diff --git a/processed_dataset/proof/0267.json b/processed_dataset/proof/0267.json new file mode 100644 index 0000000000000000000000000000000000000000..4581987401484a981802ef3d746914cae4b49c69 --- /dev/null +++ b/processed_dataset/proof/0267.json @@ -0,0 +1,8 @@ +{ + "source_file": "./raw_volume-zh/volume11/chapter10.tex", + "problem_type": "proof", + "problem": "例3. 证明: 在任意 $n(\\geqslant 4)$ 个人中都存在 2 人 $A$ 和 $B$ 使得其余 $n-2$ 人中至少有 $\\left[\\frac{n}{2}\\right]-1$ 人满足: 他们中每个人或者同 $A, B$ 都互相认识或者同 $A$, $B$ 都互相不认识.", + "solution": "证法二用平面内 $n$ 个点 $A_1, A_2, \\cdots, A_n$ 表示 $n$ 个人(其中任意三点不共线). 若两人互相认识 (互相不认识), 则对应两点的连线染红 (蓝)色.\n如果某点 $C$ 与另两点 $A, B$ 的连线同色, 那么称 $\\angle A C B$ 为从 $C$ 点引出的同色角, 也叫做点 $C$ 对 $(A, B)$ 所张的同色角,简称同色角.\n因此原题结论等价于证明: 存在两点 $A, B$, 使其余 $n-2$ 点中至少有 $\\left[\\frac{n}{2}\\right]-1$ 个点对 $(A, B)$ 张有同色角.\n如果结论不成立, 那么对任意两点 $A, B$, 对 $(A, B)$ 张有同色角的点至多有 $\\left[\\frac{n}{2}\\right]-2$ 个.\n又 $(A, B)$ 有 $C_n^2$ 种不同取法, 故图中同色角的个数 $S$ 满足\n$$\nS \\leqslant\\left(\\left[\\frac{n}{2}\\right]-2\\right) \\mathrm{C}_n^2 \\leqslant\\left(\\frac{n}{2}-2\\right) \\cdot \\frac{n(n-1)}{2}=\\frac{n(n-1)(n-4)}{4} . \\label{eq3}\n$$\n另一方面, 设从 $A_i$ 出发有 $x_i$ 条红线, $n-1-x_i$ 条蓝线, 那么从 $A_i$ 引出的同色角的个数为\n$$\n\\begin{aligned}\n\\mathrm{C}_{x_i}^2+\\mathrm{C}_{n-1-x_i}^2 & =\\frac{1}{2} x_i\\left(x_i-1\\right)+\\frac{1}{2}\\left(n-1-x_i\\right)\\left(n-2-x_i\\right) \\\\\n& =x_i^2-(n-1) x_i+\\frac{(n-1)(n-2)}{2}\n\\end{aligned}\n$$\n$$\n=\\left(x_i-\\frac{n-1}{2}\\right)^2+\\frac{(n-1)(n-3)}{4} \\geqslant \\frac{(n-1)(n-3)}{4} .\n$$\n所以图中同色角的个数 $S$ 满足\n$$\nS=\\sum_{i=1}^n\\left(\\mathrm{C}_{x_i}^2+\\mathrm{C}_{n-1-x_i}^2\\right) \\geqslant \\frac{n(n-1)(n-3)}{4}, \\label{eq4}\n$$\n式\\ref{eq4}与\\ref{eq3}矛盾,故题中结论成立.", + "remark": "", + "figures": [] +} \ No newline at end of file diff --git a/processed_dataset/proof/0268.json b/processed_dataset/proof/0268.json new file mode 100644 index 0000000000000000000000000000000000000000..a995decf2b610375a701572978ba3eeb988795e8 --- /dev/null +++ b/processed_dataset/proof/0268.json @@ -0,0 +1,8 @@ +{ + "source_file": "./raw_volume-zh/volume11/chapter10.tex", + "problem_type": "proof", + "problem": "例4. 设 $A_1, A_2, \\cdots, A_{2010}$ 是圆周上依次排列的 2010 个点, 最初 $A_1$ 上标的数为 $0, A_2, A_3, \\cdots, A_{2010}$ 上标的数为 1 . 允许进行如下操作: 任取一点 $A_j$, 若 $A_j$ 上所标的数为 1 , 则同时将 $A_{j-1}, A_j, A_{j+1}$ 上标的数 $a, b, c$ 分别改为 $1-a, 1-b, 1-c$. (这时 $A_0=A_{2010}, A_{2011}=A_1$ ) 问能否经过有限次这样的操作将所有点上标的数都变为 0 ?", + "solution": "解:不能.\n现用反证法证明如下: 若可以经过有限次 ( $m$ 次) 操作, 使所有点上标的数都变为 0 , 则由于 $(1-a)+(1-b)+(1-c)=3-(a+b+c)$ 与 $a+b+c$ 的奇偶性相反.\n故每经过一次操作, 标的 2010 个数之和的奇偶性都要改变一次,而最初 2010 个数之和为奇数 2009 ,所以 $m$ 是奇数.\n其次, 设以 $A_j$ 为出发点的操作次数为 $x_j(1 \\leqslant j \\leqslant 2010)$, 在 $A_j$ 上标的数改变的次数为 $y_j$, 则 $\\sum_{j=1}^{2010} x_j=m$, 并且 $y_1$ 为偶数, 当 $j \\in\\{2,3, \\cdots, 2010\\}$ 时 $y_j$ 为奇数.\n此外还有 $y_j=x_{j-1}+x_j+x_{j+1}(1 \\leqslant j \\leqslant 2010)$, 这里 $x_0=x_{2010}$, $x_{2011}=x_1$, 所以应有 $m=y_2+y_5+y_8+\\cdots+y_{2009}$. 而 $y_2, y_5, \\cdots, y_{2009}$ 一共是 670 个奇数, 它们之和 $m$ 应为偶数, 这与 $m$ 为奇数矛盾, 故不可能经过有限次操作使所有点上标的数都变为 0 .", + "remark": "注:由上述证明即知,把 2010 改为 $6 n\\left(n \\in \\mathbf{N}_{+}\\right)$, 其结论仍成立.", + "figures": [] +} \ No newline at end of file diff --git a/processed_dataset/proof/0269.json b/processed_dataset/proof/0269.json new file mode 100644 index 0000000000000000000000000000000000000000..123952adec99eab4c5bde534c36735978ee7a898 --- /dev/null +++ b/processed_dataset/proof/0269.json @@ -0,0 +1,8 @@ +{ + "source_file": "./raw_volume-zh/volume11/chapter10.tex", + "problem_type": "proof", + "problem": "例5. 能否将全体整数分拆为 3 个不相交的集合, 使得对任意整数 $n$, 数 $n, n-50, n+2011$ 分别属于所分成的 3 个不同的子集?", + "solution": "解:不可能, 现用反证法证明如下, 假设存在符合题目要求的分拆方法.\n我们用记号 $m \\sim k$ 表示 $m$ 与 $k$ 属于同一子集.\n若 3 个数 $p, q, r$ 分别属于 3 个不同的子集则记为 $(p, q, r) \\in \\mathcal{M}$. 于是, 邑 $(n, n-50, n+2011) \\in \\mathscr{M}$ 出发, 分别将 $n$ 用 $n-50$ 和 $n+2011$ 代替得\n$$\n\\begin{aligned}\n& (n-50, n-100, n+1961) \\in \\mathscr{M}, \\\\\n& (n+2011, n+1961, n+2 \\cdot 2011) \\in \\mathcal{M} .\n\\end{aligned}\n$$\n于是 $n+1961 \\not \\sim n-50, n+1961 \\not \\chi n+2011$, 故只有 $n+1961 \\sim n$. 故由 $(n-50, n-100, n+1961) \\in \\mathcal{M}$ 得 $(n-50, n-100, n) \\in \\mathcal{M}$. 再将其中 $n$\n用 $n-50$ 代替得 $(n-100, n-150, n-50) \\in \\mathcal{M}$, 于是由 $n \\Varangle n-50$ 和 $n \\nsucc n-100$ 得 $n \\sim n-150$. 再从 $n \\sim n+1961$ 和 $n \\sim n-150$ 出发可推得\n$$\n\\begin{aligned}\n& 0 \\sim 1961 \\sim 2 \\times 1961 \\sim \\cdots \\sim 50 \\times 1961 \\\\\n= & 654 \\cdot 150-50 \\sim 653 \\cdot 150-50 \\sim \\cdots \\sim 150-50 \\sim-50 .\n\\end{aligned}\n$$\n这与 $n \\Varangle n-50$ 从而 $0 \\nsucc-50$ 矛盾.\n故满足题目要求的分拆子集方法是不存在的.", + "remark": "", + "figures": [] +} \ No newline at end of file diff --git a/processed_dataset/proof/0270.json b/processed_dataset/proof/0270.json new file mode 100644 index 0000000000000000000000000000000000000000..a5d655dd1d73995f32bb8735683cff7c586c6ce6 --- /dev/null +++ b/processed_dataset/proof/0270.json @@ -0,0 +1,8 @@ +{ + "source_file": "./raw_volume-zh/volume11/chapter10.tex", + "problem_type": "proof", + "problem": "例6. 在 $2 \\times n$ 方格表的每个 $1 \\times 1$ 小方格内写上一个正实数,使得每列中两个数之和等于 1 . 证明: 可以从每列中删去一个数, 使得每行中剩下的各数之和都不超过 $\\frac{n+1}{4}$.", + "solution": "证明:假设第一行中的 $n$ 个数从左到右依次为 $a_1, a_2, \\cdots, a_n$, 必要时交换列的位置可使得 $a_1 \\leqslant a_2 \\leqslant \\cdots \\leqslant a_n$. 此时第二行中所写的数依次为 $b_1= 1-a_1, b_2=1-a_2, \\cdots, b_n=1-a_n$, 于是 $b_1 \\geqslant b_2 \\geqslant \\cdots \\geqslant b_n$. 如果 $a_1+a_2+\\cdots+ a_n \\leqslant \\frac{n+1}{4}$, 那么就删去第二行中所有各数即可达到目的.\n否则存在使 $a_1+ a_2+\\cdots+a_k>\\frac{n+1}{4}$ 成立的最小正整数 $k$, 这时我们只要删去 $b_1, b_2, \\cdots, b_{k-1}$ 及 $a_k, a_{k+1}, \\cdots, a_n$, 则由 $k$ 的取法有 $a_1+a_2+\\cdots+a_{k-1} \\leqslant \\frac{n+1}{4}$, 并且由于\n$$\na_k \\geqslant \\frac{a_1+a_2+\\cdots+a_k}{k}>\\frac{n+1}{4 k},\n$$\n所以\n$$\n\\begin{gathered}\nb_k+b_{k+1}+\\cdots+b_n \\leqslant(n+1-k) b_k=(n+1-k)\\left(1-a_k\\right) \\\\\n<(n+1-k)\\left(1-\\frac{n+1}{4 k}\\right)=\\frac{5}{4}(n+1)-\\left[\\frac{(n+1)^2+(2 k)^2}{4 k}\\right] \\\\\n\\leqslant \\frac{5}{4}(n+1)-\\frac{2(n+1)(2 k)}{4 k}=\\frac{n+1}{4},\n\\end{gathered}\n$$\n从而证明了题中结论成立.", + "remark": "", + "figures": [] +} \ No newline at end of file diff --git a/processed_dataset/proof/0271.json b/processed_dataset/proof/0271.json new file mode 100644 index 0000000000000000000000000000000000000000..aa65a16eca49da49397643ce97854321160d8b34 --- /dev/null +++ b/processed_dataset/proof/0271.json @@ -0,0 +1,8 @@ +{ + "source_file": "./raw_volume-zh/volume11/chapter10.tex", + "problem_type": "proof", + "problem": "例7. 有男女青年共 1000 人围成一个圆圈.\n如果有一个女孩 $G$, 她沿任意方向依次数到任何人 (包括她自己以及最后数到的人),女孩的数目总是大于男孩的数目, 那么称 $G$ 在一个好位置上, 试问: 要保证在任何情形下都至少有一个女孩在好位置上,女孩的人数的最小值是多少?", + "solution": "解:因为如果一个女孩的某一侧是一个男孩, 那么这个女孩一定不在好位置上.\n所以只要将圆圈上 1000 个位置分成 334 组, 前 333 组都是 2 个女孩中间站 1 个男孩, 最后一组只 1 个男孩, 于是一共有 666 个女孩和 334 个男孩,但没有一个女孩站在好位置上.\n故所求女孩人数的最小值不小于 667 .\n下面我们证明: 如果 1000 个男女青年中至少有 667 个女孩,那么其中必有 1 个女孩站在好位置上.\n实际上, 我们可以证明下列一般性结论: 如果 $3 n+1\\left(n \\in \\mathbf{N}_{+}\\right)$个男女青年站在一个圆圈上, 并且其中至少有 $2 n+1$ 个女孩, 那么必有一个女孩站在好位置上.\n当 $n=1$ 时,一共有 $3 \\times 1+1=4$ 个男女青年,其中至少有 $2 \\times 1+1=3$ 个女孩, 从而至多有 1 个男孩,站成一个圆圈时, 必有一个女孩 $G$, 她的两侧都为女孩,故 $G$ 在好位置上.\n设 $n=k$ 时结论成立, 则当 $n=k+1$ 时,一共有 $3(k+1)+1=3 k+4$ 个男女青年.\n其中女孩至少有 $2(k+1)+1=2 k+3$ 个.\n任取一个男孩 $A$, 并在 $A$ 的两侧各找一个到 $A$ 的距离最近的女孩 $B$ 和 $C$. 先证 $A 、 B 、 C$ 离开圆圈, 则圆圈上共有 $3 k+1$ 个男女青年.\n其中女孩至少有 $2 k+1$ 个.\n由归纳假设知其中必有 1 个女孩 $G$ 站在好位置上, 并且由好位置的定义知 $G$ 的两侧皆为女孩, 然后让 $A, B, C$ 站回去, 则由 $B 、 C$ 的取法知 $G$ 不在 $A, B, C$ 所在的圆弧上, 从 $G$ 出发沿任意方向计算男、女孩总人数时, 必然是先数到 $B$ (或 $C$ ) 后才数到 $A$ (如果要数到 $A$ 的话). 由归纳假设知, 数到的女孩总人数必定多于男孩的总人数, 即 $G$ 仍在好位置上, 于是 $n=k+1$ 时结论成立.\n故一般性命题得证.\n特别 $n=333$ 时结论成立.\n由此可知原题中所求女孩人数的最小值为 667 .", + "remark": "", + "figures": [] +} \ No newline at end of file diff --git a/processed_dataset/proof/0272.json b/processed_dataset/proof/0272.json new file mode 100644 index 0000000000000000000000000000000000000000..c920654a94da03b8a831bb442eab51b4ebe64d51 --- /dev/null +++ b/processed_dataset/proof/0272.json @@ -0,0 +1,8 @@ +{ + "source_file": "./raw_volume-zh/volume11/chapter11.tex", + "problem_type": "proof", + "problem": "例1. 有 $n \\times n(n \\geqslant 4)$ 的一张空白方格表, 在它的每一个方格内任意填人 +1 或 -1 两个数中的一个.\n现将表内 $n$ 个两两既不同(横)行又不同(竖)列的方格中的数的乘积称为一个基本项.\n试证: 按上述所填成的每一方格表,它的全部基本项的和 $S$ 总能被 4 整除.", + "solution": "证明:显然不论用怎样的填法, 所填成的表格总有 $n$ ! 个基本项.\n设第 $i$ 行第 $j$ 列的方格内填的数是 $a_{i j}(1 \\leqslant i, j \\leqslant n, n \\geqslant 4)$. 当把方格表中某一个 $a_{i j}$ 改变符号时 (即把 +1 换成 -1 或将 -1 换成 +1 ), 注意到 $a_{i j}$ 出现在和式 $S$ 的 $(n-1)$ ! 个基本项中, 故 $S$ 中将有 $(n-1)$ ! 个基本项变号, 设其中有 $h$ 项由 -1 变为 +1 , 有 $(n-1) !-h$ 项由 +1 变成 -1 . 所有基本项之和由 $S$ 变为 $S^{\\prime}$ 则 $S^{\\prime}-S=2 h-2[(n-1) !-h]=4 h-2 \\cdot(n-1) !$, 因为 $n \\geqslant 4$, 所以 $S^{\\prime}-S$ 是 4 的倍数, 即当某个数 $a_{i j}$ 改变符号时, 引起 $S$ 的改变值一定是 4 的倍数.\n若一张方格表内所有 $a_{i j}$ 全为 +1 , 则全部基本项之和 $S=n !(n \\geqslant 4)$ 显然能够被 4 整除.\n若 $a_{i j}$ 不全为 +1 时,则可将其中 -1 逐个改成 +1 , 每次均使基本项之和的改变值能被 4 整除, 于是经过有限步可使表中每个数都为 +1 , 并且最后得到的表格中所有基本项之和是 4 的倍数,所以一开始时,表中所有基本项之和也是 4 的倍数.", + "remark": "", + "figures": [] +} \ No newline at end of file diff --git a/processed_dataset/proof/0273.json b/processed_dataset/proof/0273.json new file mode 100644 index 0000000000000000000000000000000000000000..1fa5662127a803e0cc2b5f57f0274ac0ef21fba1 --- /dev/null +++ b/processed_dataset/proof/0273.json @@ -0,0 +1,8 @@ +{ + "source_file": "./raw_volume-zh/volume11/chapter11.tex", + "problem_type": "proof", + "problem": "例3. 14 人进行一种日本棋循环赛, 每人都与另外 13 人对弯.\n在比赛中没有平局, 求 \"三角联\" 个数的最大值 (这里 \"三角联\"指 3 人间的比赛每人皆一胜一负).", + "solution": "解:用 $A_1, A_2, \\cdots, A_{14}$ 表示 14 个人,并设 $A_i$ 胜了 $a_i$ 局 $(i=1,2, \\cdots$, 14). 若 3 人不形成\"三角联\",则一定是其中 1 人胜了其他两人,故不构成\"三角联\"的数目为 $\\sum_{i=1}^{14} \\mathrm{C}_{a_i}^2$ (约定 $\\mathrm{C}_0^2=\\mathrm{C}_1^2=0$ ), 于是 \"三角联\" 的总数为 $S \\mathrm{C}_{14}^3-\\sum_{t=1}^{14} \\mathrm{C}_{a_i}^2$, 要 $S$ 最大, 必须且只须 $\\sum_{t=1}^{14} \\mathrm{C}_{a_t}^2$ 最小, 下面我们证明当 $\\sum_{i=1}^{14} \\mathrm{C}_{a_i}^2$ 取最小值时必有 $\\left|a_i-a_j\\right| \\leqslant 1(1 \\leqslant ib>c \\geqslant 0$ 的情形, 又分为下列两种情形.\n(1)如果 $a=c+2$, 则 $b=c+1$, 因 $a+b+c=3 c+3 \\geqslant 4$, 所以 $c \\geqslant 1$. 于是可按下列步骤操作, 经过有限步将所有糖块集中到一个盘子中:\n$$\n\\begin{aligned}\n\\quad(c+2, c+1, c, 0) \\rightarrow(c+1, c, c, 2) \\rightarrow(c, c+2, c, 1) \\rightarrow(c-1, c+2, \\\\\nc+2,0) \\rightarrow \\cdots+(3 c+3,0,0,0) .\n\\end{aligned}\n$$\n(2) 如果 $a>c+2$ 时, 那么先作如下一次操作: $(a, b, c) \\rightarrow(a-1$, $b-1, c+2)$ 因 $a>b>c$ 及 $a>c+2$, 所以 $a-1>b-1 \\geqslant c, a-1 \\geqslant (c+3)-1 \\geqslant c+2$, 故经过调整后, 三盘中所放糖块量的最大数减少 1 , 而最小数不减少, 故经过有限步调整可归结为有两盘糖块数相等或前述情形 (1). 于是由前面证明可知经过有限步操作可将糖块集中到一个盘子中.", + "remark": "", + "figures": [] +} \ No newline at end of file diff --git a/processed_dataset/proof/0275.json b/processed_dataset/proof/0275.json new file mode 100644 index 0000000000000000000000000000000000000000..1e279171d7c987b6f279e4fc3919985a5d179c4d --- /dev/null +++ b/processed_dataset/proof/0275.json @@ -0,0 +1,8 @@ +{ + "source_file": "./raw_volume-zh/volume11/chapter11.tex", + "problem_type": "proof", + "problem": "例5. $n(\\geqslant 4)$ 个盘子里放有总数不少于 4 的糖块, 从任意选出的两个盘子里各取一块糖放人另一个盘子中称为一次操作, 问能否经过有限次操作, 把所有的糖块集中到一个盘子里去? 证明你的结论.", + "solution": "解法二设共有 $m$ 块糖 $(m \\geqslant 4)$. 我们对 $m$ 用数学归纳法.\n(1) $m=4$ 时, 4 块糖至多放在 4 个盘子中, 其分布情况 (不考虑顺序) 只有以下 4 种: $1^{\\circ}(1,1,1,1), 2^{\\circ}(1,1,2,0), 3^{\\circ}(1,3,0,0), 4^{\\circ}(2,2,0,0)$. 每种情形按下列步骤进行操作, 都可以经过有限步将所有糖块集中到一个盘子中:\n$$\n\\begin{aligned}\n& 1^{\\circ}(1,1,1,1) \\rightarrow(1,3,0,0) \\rightarrow(0,2,2,0) \\rightarrow(2,1,1,0) \\rightarrow(4,0,0,0) . \\\\\n& 2^{\\circ}(1,1,2,0) \\rightarrow(0,0,4,0) . \\\\\n& 3^{\\circ} \\text { 同 } 1^{\\circ} \\text { 中第 } 2 \\text { 步以后操作.\n} \\\\\n& 4^{\\circ}(2,2,0,0) \\rightarrow(1,1,2,0) \\rightarrow(0,0,4,0) .\n\\end{aligned}\n$$\n$3^{\\circ}$ 同 $1^{\\circ}$ 中第 2 步以后操作.\n(2)设 $m=k$ 时结论成立,考虑 $m=k+1$ 的情形,这时我们只考虑其中 $k$ 块糖, 由归纳假设, 总可以经过有限步操作将这 $k$ 块糖集中到一个盘子中.\n如果第 $k+1$ 块糖也在这个盘子中, 那么结论已成立, 否则只会出现这 $k$ 块糖在一个盘子中, 而第 $k+1$ 块糖在另一个盘子中的情形, 这时, 可按下列步骤, 经过有限步操作将 $k+1$ 块糖集中到一个盘子中.\n$$\n\\begin{aligned}\n& \\quad(k, 1,0,0) \\rightarrow(k-1,0,2,0) \\rightarrow(k-2,2,1,0) \\rightarrow(k-3,2,0,2) \\\\\n& \\rightarrow(k-1,1,0,1) \\rightarrow(k+1,0,0,0) .\n\\end{aligned}\n$$\n于是 $m=k+1$ 时结论成立, 这就证明了总可以经过有限步操作将所有糖块集中到一个盘子中.", + "remark": "", + "figures": [] +} \ No newline at end of file diff --git a/processed_dataset/proof/0276.json b/processed_dataset/proof/0276.json new file mode 100644 index 0000000000000000000000000000000000000000..83ef8479b139fd8581da4660ab71b500fc5970b3 --- /dev/null +++ b/processed_dataset/proof/0276.json @@ -0,0 +1,8 @@ +{ + "source_file": "./raw_volume-zh/volume11/chapter12.tex", + "problem_type": "proof", + "problem": "例1. 在平面直角坐标系中是否存在无穷多个圆组成的集合 $M_0$, 满足\n(1) $M_0$ 中任意两个圆至多有一个公共点;\n(2) $x$ 轴上每一个有理点都在 $M_0$ 中某个圆上.", + "solution": "解:对 $x$ 轴上任意有理点 $(r, 0)\\left(r=\\frac{p}{q}, p 、 q\\right.$ 为互素整数, $\\left.q>0\\right)$, 在 $x$ 轴上方作一个半径为 $R_r>0$ 的圆与 $x$ 轴切于点 $(r, 0)$, 并记这个圆为 $C_r\\left(R_r\\right)$, 所有这些圆组成的集合记为 $M_0$, 显然 $M_0$ 满足条件 (2), 为了使 $M_0$ 满足条件 (1), 我们来分析 $R_r$ 的值应该是多少.\n在 $M_0$ 中任取两个圆 $C_{r_i}\\left(R_{r_i}\\right)(i=1,2)$, 因为两个圆都在 $x$ 轴上方并且都与 $x$ 轴相切, 所以它们相交的充要条件是圆心距小于两圆半径之和, 即\n$$\n\\sqrt{\\left(r_2-r_1\\right)^2+\\left(R_{r_2}-R_{r_1}\\right)^2}0, i= 1,2)$, 得\n$$\n\\left(p_1 q_2-p_2 q_1\\right)^2<4 q_1^2 q_2^2 R_{r_1} R_{r_2} .\n$$\n若取 $R_{r_i}=\\frac{1}{k q_{r_i}^2}(k \\geqslant 2)$, 则上式化为\n$$\n\\left(p_1 q_2-p_2 q_1\\right)^2<\\frac{4}{k^2} \\leqslant 1\n$$\n当 $r_1 \\neq r_2$ 时, 上式左端为正整数, 矛盾.\n可见, 只要取 $R_r=\\frac{1}{k q^2}(k \\geqslant 2)$, 所作圆集合 $M_0$ 就满足题目的所有条件.\n于是我们证明了存在无穷多个圆组成的集合\n$$\nM_0=\\left\\{C_r\\left(R_r\\right) \\mid r=\\frac{p}{q}, R_r=\\frac{1}{k q^2}(k \\geqslant 2), p, q \\text { 为互素整数, } q>0\\right\\} .\n$$\n满足题目条件 (1)、(2), 显然这样的 $M_0$ 有无穷多个.", + "remark": "", + "figures": [] +} \ No newline at end of file diff --git a/processed_dataset/proof/0277.json b/processed_dataset/proof/0277.json new file mode 100644 index 0000000000000000000000000000000000000000..e081ef70c4c95e6e7ab53b0c45d49ffeb9b2b613 --- /dev/null +++ b/processed_dataset/proof/0277.json @@ -0,0 +1,8 @@ +{ + "source_file": "./raw_volume-zh/volume11/chapter12.tex", + "problem_type": "proof", + "problem": "例2. 求证能否将正整数集合 $\\mathbf{N}_{+}$分划为两个不相交的集合 $A$ 和 $B$, 满足:\n(1) $A$ 中任意三个数不成等差数列;\n(2) 不能由 $B$ 中元素组成一个非常数的无穷等差数列.", + "solution": "分析一设 $A=\\left\\{a_1, a_2, a_3, \\cdots\\right\\}\\left(a_1c$, 故只要 $a_{i+1} \\geqslant 2 a_i(i=1,2,3, \\cdots)$, 则 $A$ 中任意三个数不成等差数列, 要构造这样的 $A$ 是不困难的.\n为了使 $B=\\mathbf{N}_{+} \\backslash A$ 满足条件 (2), 只要满足对任意 $a, d \\in \\mathbf{N}_{+}$, 等差数列 $\\{a+n d\\}(n=0,1,2, \\cdots)$ 中至少有一项属于 $A$.\n解法一将首项为 $a$, 公差为 $d$ 的无穷等差数列用 $(a, d)$ 表示.\n易将所有正整数无穷等差数列 (非常数列) \"排序\" 如下: $(1,1),(1,2),(2,1)$, $(1,3),(2,2),(3,1) \\cdots$, 排序的规律是:先看 $a+d$ 的大小,小者排前, $a+d$ 相等的, $a$ 较小的排前, 按下列方式构造数列 $a_1, a_2, a_3, \\cdots$. 设 $a_1=1$, 如 $a_1$, $a_2, \\cdots, a_n$ 已取出,则在第 $n+1$ 个等差数列中取大于 $2 a_n$ 的某一项为 $a_{n+1}$.\n令 $A=\\left\\{a_1, a_2, \\cdots, a_n, \\cdots\\right\\}$, 则因 $a_{n+1}>2 a_n(n=1,2, \\cdots)$, 故 $A$ 中任何三个数不成等差数列, 再令 $B=\\mathbf{N}_{+} \\backslash A$, 则因为任何正整数的非常数列的无穷等差数列都有一项属于 $A$, 故 $B$ 中没有非常数列的无穷等差数列.\n于是存在满足题目条件的集合 $A$ 和 $B$.", + "remark": "", + "figures": [] +} \ No newline at end of file diff --git a/processed_dataset/proof/0278.json b/processed_dataset/proof/0278.json new file mode 100644 index 0000000000000000000000000000000000000000..ad960f350da275cd7a21584b97d5e46b096f1809 --- /dev/null +++ b/processed_dataset/proof/0278.json @@ -0,0 +1,8 @@ +{ + "source_file": "./raw_volume-zh/volume11/chapter12.tex", + "problem_type": "proof", + "problem": "例2. 求证能否将正整数集合 $\\mathbf{N}_{+}$分划为两个不相交的集合 $A$ 和 $B$, 满足:\n(1) $A$ 中任意三个数不成等差数列;\n(2) 不能由 $B$ 中元素组成一个非常数的无穷等差数列.", + "solution": "分析二同分析一知, 易构造满足条件 (1) 的 $A$, 为了使 $B=\\mathbf{N}_{+} \\backslash A$ 满足条件 (2), 只须满足对任意 $a, d \\in \\mathbf{N}_{+}$, 无穷等差数列 $\\{a+r d\\}(r=0,1,2$, …) 中至少有一项属于 $A$, 这只要 $A$ 中每一个数可写成 $a_i=b_i+c_i$ 的形式, 使得对任意 $a, d \\in \\mathbf{N}_{+}, b_i$ 可无穷多次取到 $a$ 的值, 且存在某个等于 $a$ 的 $b_i$, 它所对应的 $c_i$ 是任意 $d$ 的倍数.\n这只要 $A$ 中含有无穷多个形如 $a+m$ ! 的数即可.\n于是, 存在正整数 $m \\geqslant d$, 取 $r_0=\\frac{m !}{d}$, 则 $a+m !=a+r_0 d$ 为无穷等差数列 $\\{a+r d\\}$ 中一项.\n解法二令 $A=A_1 \\cup A_2 \\cup \\cdots \\cup A_n \\cup \\cdots$, 其中 $A_1=\\{1 !+1\\}, A_2= \\{2 !+1,3 !+2\\}, A_3=\\{4 !+1,5 !+2,6 !+3\\}, \\cdots, A_n=\\{m !+1, m !+ 2, \\cdots,(m+n-1) !+n\\}$ (其中 $\\left.m=\\frac{1}{2} n(n-1)+1\\right), \\cdots, B=\\mathbf{N}_{+} \\backslash A$, 则将 $A$ 中数从小到大排列时, 从第二项起, 每一项大于前面一项的 2 倍, 故 $A$ 中任意三个数不成等差数列, 其次 $B$ 中没有非常数列的无穷等差数列.\n事实上,若 $\\{a+n d\\}, n=0,1,2, \\cdots$ 是 $B$ 中一个非常数列的无穷等差数列, 这里 $a, d \\in \\mathbf{N}_{+}$, 由 $A$ 的构造知, 存在无穷多个 $m \\in \\mathbf{N}_{+}$使 $m !+a \\in A$, 故存在 $m \\geqslant d$, 使 $m !+a \\in A$. 令 $n_0=\\frac{m !}{d}$, 则 $m !+a=a+n_0 d \\in A$, 这与对任意 $n \\in \\mathbf{N}_{+}, a+n d \\in B$ 矛盾.\n可见存在满足条件 (1), (2) 的集合 $A$ 与 $B$.", + "remark": "", + "figures": [] +} \ No newline at end of file diff --git a/processed_dataset/proof/0279.json b/processed_dataset/proof/0279.json new file mode 100644 index 0000000000000000000000000000000000000000..9e9ff413c01a00907437e6b657084e9aa1cfdc3f --- /dev/null +++ b/processed_dataset/proof/0279.json @@ -0,0 +1,13 @@ +{ + "source_file": "./raw_volume-zh/volume11/chapter12.tex", + "problem_type": "proof", + "problem": "例3. 是否存在平面内一个有限点集, 使得对于其中每个点, 点集中恰有三个距离它最近的点.", + "solution": "分析:如图(), 由两个有公共底边的正三角形组成的图形共有 4 个顶点, 其中有 2 个点恰有三个距离它最近的点, 但另 2 个点却只有两个距离它最近的点, 不满足要求.\n其次,如图() 考虑用 $m$ 条线段将 $m$ 个这样的图形连起来看能否组成一个满足条件的图形.\n图 () 中为了保证每个点恰有三个距离它最近的点, 只要 $90^{\\circ}<\\alpha \\leqslant 120^{\\circ}$, 并且由凸多边形内角和公式有\n$$\nm \\cdot 120^{\\circ}+2 m\\left(\\alpha+60^{\\circ}\\right)=(3 m-2) \\cdot 180^{\\circ},\n$$\n即 $\\alpha=150^{\\circ}-\\frac{180^{\\circ}}{m}$, 代入 $90^{\\circ}<\\alpha \\leqslant 120^{\\circ}$ 解得 $3) 所示的三个点集都具有题目中要求的性质(图中已将每点与它距离最近的三个点相连)", + "remark": "注:本例中我们用部件组成法得到了满足题目条件的点集, 而且不止一个, 显然若干个这样点集的并集 (只要每两个这样点集之间点的最小距离大于原点集中任意两点之间距离的最小值) 也满足题目的所有条件, 而在解答中常常将组成的设计及探索过程省略, 只写出了构造的结果.", + "figures": [ + "./images/volume11/figures/fig-c12i1.png", + "./images/volume11/figures/fig-c12i2.png", + "./images/volume11/figures/fig-c12i2.png", + "./images/volume11/figures/fig-c12i3.png" + ] +} \ No newline at end of file diff --git a/processed_dataset/proof/0280.json b/processed_dataset/proof/0280.json new file mode 100644 index 0000000000000000000000000000000000000000..43e78d09f15fc6ce57785d955b87c2300bdca613 --- /dev/null +++ b/processed_dataset/proof/0280.json @@ -0,0 +1,10 @@ +{ + "source_file": "./raw_volume-zh/volume11/chapter12.tex", + "problem_type": "proof", + "problem": "例4. 平面内任给 2000 个点,证明: 可以用一些圆形纸片盖住这 2000 个点, 且满足:\n(1)这些圆形纸片直径之和不超过 2000;\n(2)任意两张圆心纸片的距离大于 1 .", + "solution": "证明:首先证明满足条件 (1) 的纸片存在.\n事实上, 取 2000 张直径为 1 的纸片, 使每张纸片的中心恰在给出的一个点上, 于是这 2000 张纸片盖住了 2000 个已知点,且这些纸片直径之和为 2000 .\n其次, 若这些纸片中有两张有公共点 (如图() 中圆 $O_1$ 与圆 $O_2$ ), 则进行一次调整.\n如图可用一张直径较大的图形纸片 $O_3$ 代替圆 $O_1$ 和圆 $O_2$, 满足 $O_3$ 在直线 $O_1 O_2$ 上且圆 $O_1$ 和圆 $O_2$ 都内切于圆 $O_3$, 显然圆 $O_3$ 的直径不大于圆 $O_1$ 和圆 $O_2$ 的直径之和, 并且圆 $O_3$ 所包含的已知点到圆 $\\mathrm{O}_3$ 周界的距离不小于 $\\frac{1}{2}$. 如果还有两张纸片有公共点, 可以继续进行这样的调整, 于是经过有限步可用有限张直径之和不大于 2000 且两两无公共点的圆形纸片盖住已知的2000个点, 并且每个已知点到覆盖它的纸片周界的距离不小于 $\\frac{1}{2}$. 设这些圆纸片每两张之间的距离的最小值为 $d$, 则 $d>0$.\n最后, 若 $d>1$, 则结论成立, 若 $01$, 于是题目结论得证.", + "remark": "", + "figures": [ + "./images/volume11/figures/fig-c12i4.png" + ] +} \ No newline at end of file diff --git a/processed_dataset/proof/0281.json b/processed_dataset/proof/0281.json new file mode 100644 index 0000000000000000000000000000000000000000..efcf534cf22718af1ae081e85882e1c909fae716 --- /dev/null +++ b/processed_dataset/proof/0281.json @@ -0,0 +1,8 @@ +{ + "source_file": "./raw_volume-zh/volume11/chapter12.tex", + "problem_type": "proof", + "problem": "例5. 是否存在一个无穷正整数列 $a_11$. 故必有两个小正方形的面积是重叠的, 若对任意 $1 \\leqslant i9 \\times \\frac{1}{5}-\\mathrm{C}_9^2 \\times \\frac{1}{45}=1$. 这与 $A_1, A_2, \\cdots, A_9$ 都在面积为 1 的图形内, 应有 $\\left|A_1 \\cup A_2 \\cup \\cdots \\cup A_9\\right| \\leqslant 1$ 矛盾.\n故存在 $1 \\leqslant iS$, 则存在两个平面图形 $A_i$ 与 $A_j(1 \\leqslant i< j \\leqslant n$ ) 有公共内点;\n(2) 若 $S_1+S_2+\\cdots+S_n\\left|A_i\\right|$ 时, 有 $x_1=x_1{ }^{\\prime}, x_2=x_2{ }^{\\prime}$, $\\cdots, x_{\\left|A_i\\right|}=x^{\\prime}{ }_{\\left|A_i\\right|}$, 即 $A_i \\subset A_j$, 这都与 $A_1, A_2, \\cdots, A_m$ 两两互不包含的假设矛盾,故\n$$\n\\sum_{i=1}^m\\left|A_i\\right| !\\left(n-\\left|A_i\\right|\\right) ! \\leqslant n !\n$$\n即 $\\sum_{i=1}^m \\frac{1}{\\mathrm{C}_n^{\\left|A_i\\right|}} \\leqslant 1$.\n(2) 由 (1)及柯西不等式得\n$$\n\\sum_{i=1}^m \\mathrm{C}_n^{\\left|A_i\\right|} \\geqslant\\left(\\sum_{i=1}^m \\frac{1}{\\mathrm{C}_n^{\\left|A_i\\right|}}\\right)\\left(\\sum_{i=1}^m \\mathrm{C}_n^{\\left|A_i\\right|}\\right) \\geqslant m^2 .\n$$", + "remark": "注:本题来源于下列著名的 Sperner 定理:\nSperner 定理设 $A$ 为 $n$ 元集, $A_1, A_2, \\cdots, A_m$ 为 $A$ 的子集且两两互不包含, 则 $m$ 的最大值为 $\\mathrm{C}_n^{\\left[\\frac{n}{2}\\right]}$.\n证明由上例(1)有 $\\sum_{i=1}^m \\frac{1}{\\mathrm{C}_n^{\\left|A_i\\right|}} \\leqslant 1$, 并且 $\\mathrm{C}_n^0, \\mathrm{C}_n^1, \\mathrm{C}_n^2, \\cdots, \\mathrm{C}_n^n$ 中以 $\\mathrm{C}_n^{\\left[\\frac{n}{2}\\right]}$ 为最大.\n所以 $\\frac{m}{\\mathrm{C}_n^{\\left[\\frac{n}{2}\\right]}} \\leqslant \\sum_{i=1}^m \\frac{1}{\\mathrm{C}_n^{\\left|A_i\\right|}} \\leqslant 1$, 即 $m \\leqslant \\mathrm{C}_n^{\\left[\\frac{n}{2}\\right]}$. 另一方面 $A$ 的 $\\mathrm{C}_n^{\\left[\\frac{n}{2}\\right]}$ 个 $\\left[\\frac{n}{2}\\right]$ 元子集两两互不包含,故 $m$ 的最大值为 $\\mathrm{C}_n^{\\left[\\frac{n}{2}\\right]}$.", + "figures": [] +} \ No newline at end of file diff --git a/processed_dataset/proof/0290.json b/processed_dataset/proof/0290.json new file mode 100644 index 0000000000000000000000000000000000000000..7fcbebddd9ce8136d2ebe05f2b216e037df5e190 --- /dev/null +++ b/processed_dataset/proof/0290.json @@ -0,0 +1,8 @@ +{ + "source_file": "./raw_volume-zh/volume11/chapter14.tex", + "problem_type": "proof", + "problem": "例6. 在某次竞赛中共有 $a$ 个参赛选手和 $b$ 个裁判, 其中 $b \\geqslant 3$ 为奇数.\n设每一位裁判对每一位参赛选手的判决方式只有 \"通过\"或\"不通过\". 已知任意两个裁判至多对 $k$ 个参赛选手有相同的判决.\n证明: $\\frac{k}{a} \\geqslant \\frac{b-1}{2 b}$.", + "solution": "证明:设 $a$ 个参赛选手为 $A_1, A_2, \\cdots, A_a, b$ 个裁判为 $B_1, B_2, \\cdots, B_b$. 若两个裁判 $B_i, B_j(i \\neq j)$ 对选手 $A_m$ 的判决相同, 则将 $\\left(B_i, B_j, A_m\\right)$ 组成三元组, 这种三元组的个数记为 $S$. 一方面, 由已知条件知对任意一对裁判 $B_i$, $B_j(i \\neq j)$, 至多存在 $k$ 个选手 $A_j$ 组成 $k$ 个 \"三元组\" $\\left(B_i, B_j, A_m\\right)$, 而 $B_i, B_j$ 有 $\\mathrm{C}_b^2$ 种取法, 所以\n$$\nS \\leqslant k \\mathrm{C}_b^2 . \\label{eq1}\n$$\n另一方面, 假设对选手 $A_m$ 有 $r_m$ 个裁判对 $A_m$ 的判决是 \"通过\", $t_m$ 个裁判对 $A_k$ 的判决是 \"不通过\", 于是 $r_m+t_m=b$ 且含 $A_m$ 的三元组恰有 $\\mathrm{C}_{r_m}^2+\\mathrm{C}_{t_m}^2$ 个, 故\n$$\nS=\\sum_{m=1}^a\\left(\\mathrm{C}_{r_m}^2+\\mathrm{C}_{t_m}^2\\right)\n$$\n而\n$$\n\\begin{aligned}\n\\mathrm{C}_{r_m}^2+\\mathrm{C}_{t_m}^2 & =\\frac{1}{2}\\left(r_m^2-r_m+t_m^2-t_m\\right) \\\\\n& =\\frac{1}{2}\\left[\\left(r_m+t_m\\right)^2-\\left(r_m+t_m\\right)-2 r_m t_m\\right] \\\\\n& =\\frac{1}{2}\\left(b^2-b-2 r_m t_m\\right) .\n\\end{aligned}\n$$\n因 $b$ 为奇数, 且 $r_m+t_m=b$, 所以 $r_m t_m \\leqslant \\frac{(b-1)(b+1)}{4}=\\frac{b^2-1}{4}$, 故\n$$\n\\mathrm{C}_{r_m}^2+\\mathrm{C}_{t_m}^2 \\geqslant \\frac{1}{2}\\left[b^2-b-\\frac{2}{4}\\left(b^2-1\\right)\\right]=\\frac{1}{4}(b-1)^2 .\n$$\n所以\n$$\nS \\geqslant a \\cdot \\frac{1}{4}(b-1)^2 . \\label{eq2}\n$$\n由式\\ref{eq1}及\\ref{eq2}得 $k \\cdot \\mathrm{C}_b^2 \\geqslant a \\cdot \\frac{1}{4}(b-1)^2$, 即 $\\frac{k}{a} \\geqslant \\frac{b-1}{2 b}$.", + "remark": "", + "figures": [] +} \ No newline at end of file diff --git a/processed_dataset/proof/0291.json b/processed_dataset/proof/0291.json new file mode 100644 index 0000000000000000000000000000000000000000..bd470ad3c85c7c6930950ded39c32ab27c0aa9a1 --- /dev/null +++ b/processed_dataset/proof/0291.json @@ -0,0 +1,8 @@ +{ + "source_file": "./raw_volume-zh/volume11/chapter14.tex", + "problem_type": "proof", + "problem": "例7. $S$ 是由同一条直线上 $6 n$ 个点构成的一个集合.\n随机地选择其中 $4 n$ 个点染成蓝色, 其余 $2 n$ 个点染成绿色.\n证明: 存在一条线段 $l$, 使 $l$ 上包含 $S$ 中 $3 n$ 个点, 其中 $2 n$ 个点为蓝色, $n$ 个点为绿色.", + "solution": "解:将直线上的点依次记为 $x_1, x_2, \\cdots, x_{6 n}$. 定义函数 $f(i)(i=1$, $2, \\cdots, 3 n+1)$ 表示 $\\left\\{x_i, x_{i+1}, \\cdots, x_{3 n-1+i}\\right\\}$ 中蓝色点的个数.\n于是, 我们只需证明存在 $j \\in\\{1,2, \\cdots, 3 n+1\\}$ 使 $f(j)=2 n$.\n事实上,一方面, 我们考虑 $f(i)$ 与 $f(i+1)$ 的关系.\n(1) 当 $x_{i+3 n}$ 与 $x_i$ 同色时, $f(i)=f(i+1)$;\n(2) 当 $x_{i+3 n}$ 为蓝色, $x_i$ 为绿色时, $f(i+1)=f(i)+1$;\n(3) 当 $x_{i+3 n}$ 为绿色, $x_i$ 为蓝色时, $f(i+1)=f(i)-1$.\n故总有 $|f(i+1)-f(i)| \\leqslant 1$.\n另一方面, 所有蓝点个数为 $f(1)+f(3 n+1)=4 n$. 当 $f(1)=2 n$ 时, 结论成立; 当 $f(1)<2 n$ 时, $f(3 n+1)>2 n$; 当 $f(1)>2 n$ 时, $f(3 n+1)<2 n$, 又已证 $|f(i+1)-f(i)| \\leqslant 1(i \\in\\{1,2, \\cdots, 3 n+1\\})$, 故必存在 $j \\in\\{1,2, \\cdots, 3 n+1\\}$ 使得 $f(j)=2 n$. 于是 $\\left\\{x_j, x_{j+1}, \\cdots, x_{j+3 n-1}\\right\\}$ 这 $3 n$ 个点中恰有 $2 n$ 个蓝点和 $n$ 个绿点.", + "remark": "注:本题的证明应用了下列显然成立的离散介值原理.\n离散介值原理设由整数 $f(1), f(2), \\cdots, f(m)$ ( $m$ 为正整数) 组成的数列及整数 $A$ 满足:\n(1) $|f(i+1)-f(i)| \\leqslant 1(i=1,2, \\cdots, m-1)$;\n(2) $f(1) \\leqslant A \\leqslant f(m)$ 或 $f(1) \\geqslant A \\geqslant f(m)$.\n那么,存在整数 $j \\in\\{1,2, \\cdots, m\\}$ 使 $f(j)=A$.\n证明只证 $f(1) \\leqslant A \\leqslant f(m)$ 的情形.\n如果 $f(1)=A$ 或 $f(m)=A$, 则结论成立.\n下设 $f(1)A>f(b))$, 则存在 $c \\in(a, b)$ 使 $f(c)=A$.\n这个原理的证明要用到大学《数学分析》课程中的实数基本定理, 但许多国家的竞赛题中已默许使用这一原理.", + "figures": [] +} \ No newline at end of file diff --git a/processed_dataset/proof/0292.json b/processed_dataset/proof/0292.json new file mode 100644 index 0000000000000000000000000000000000000000..c89dc5279677ec2186a3ec48255f444ca449a76b --- /dev/null +++ b/processed_dataset/proof/0292.json @@ -0,0 +1,8 @@ +{ + "source_file": "./raw_volume-zh/volume11/chapter15.tex", + "problem_type": "proof", + "problem": "例2. 将边长为正整数 $m, n$ 的矩形划分为若干个边长均为正整数的正方形, 每个正方形的边均平行矩形的相应边.\n试求这些正方形边长之和的最小值.", + "solution": "分析:不妨设 $m \\geqslant n$, 我们构造一种特殊的划分情形, 首先从较长边 (边长等于 $m$ 的边) 上尽可能划出边长等于较短边的正方形, 剩下一个 $n \\times r_1(0< r_1m+n-(m, n) .\n$$\n若 $a_1=n$, 则从 $m \\times n$ 矩形中划分出一个边长为 $a_1=n$ 的正方形后, 剩余部分组成一个 $(m-n) \\times n$ 的矩形, 且它被分成 $p-1$ 个边长分别为 $a_2, a_3, \\cdots, a_p$ 的正方形, 由归纳假设有\n$$\na_2+a_3+\\cdots+a_p \\geqslant(m-n)+n-(m-n, n)=m-(m, n),\n$$\n从而 $b_{m, n}=a_1+a_2+\\cdots+a_p \\geqslant m+n-(m, n)$, 所以 $f(m, n)=\\min b_{m, n} \\geqslant m+n-(m, n)$.\n综上得所求正方形边长之和的最小值为 $f(m, n)=m+n-(m, n)$.", + "remark": "注:本题也可从具体尺寸的一些矩形出发作一些特殊的划分猜出最小 值, 那么 $f(m, n) \\leqslant m+n-(m, n)$ 也可用数学归纳法去证明.", + "figures": [] +} \ No newline at end of file diff --git a/processed_dataset/proof/0293.json b/processed_dataset/proof/0293.json new file mode 100644 index 0000000000000000000000000000000000000000..e6e69545881db8cd2a07f10d90aed01c1b96f649 --- /dev/null +++ b/processed_dataset/proof/0293.json @@ -0,0 +1,10 @@ +{ + "source_file": "./raw_volume-zh/volume11/chapter15.tex", + "problem_type": "proof", + "problem": "例5. 设 $A$ 是有限集, 且 $|A| \\geqslant 2, A_1, A_2, \\cdots, A_n$ 是 $A$ 的子集且满足下述条件:\n(1) $\\left|A_1\\right|=\\left|A_2\\right|=\\cdots=\\left|A_n\\right|=k, k>\\frac{|A|}{2}$;\n(2) 对任意 $a, b \\in A$, 存在 3 个集合 $A_r, A_s, A_t(1 \\leqslant rk$, 所以 $m \\leqslant \\frac{k}{3}$ (舍去, 因为 $m \\geqslant k$ ) 或者\n$$\nm \\geqslant \\frac{1}{3}\\left(k^2-k+3\\right),\n$$\n由已知 $m \\leqslant 2 k-1$, 故\n$$\n2 k-1 \\geqslant m \\geqslant \\frac{1}{3}\\left(k^2-k+3\\right), \\label{eq5}\n$$\n由此解出 $1 \\leqslant k \\leqslant 6$.\n由 (i), (ii), 我们得到 $1 \\leqslant k \\leqslant 6$, 故所求 $k$ 的最大值不大于 6 , 并且由不等式 \\ref{eq5} 知 $k=6$ 当且仅当 $m=11$, 再由 式\\ref{eq4} 知 $k=6$ 且 $m=11$ 当且仅当 $n=11$.\n其次, 当 $k=6, m=11, n=11$ 时,存在满足条件(1),(2), (3) 的实例如下:令\n$$\nA_t=\\{t, t+1, t+2, t+6, t+8, t+9\\}, t=1,2, \\cdots, 11 .\n$$\n并且约定当 $j \\equiv i(\\bmod 11)$ 时, $j$ 与 $i$ 表示同一个元素.\n于是对任意 $i, j(1 \\leqslant i< j \\leqslant 11$ ), 含 $i$ 的集合有且只有以下 6 个:\n$$\n\\begin{aligned}\n& A_i=\\{i, i+1, i+2, i+6, i+8, i+9\\} ; \\\\\n& A_{i+2}=\\{i, i+2, i+3, i+4, i+8, i+10\\} ; \\\\\n& A_{i+3}=\\{i, i+1, i+3, i+4, i+5, i+9\\} ; \\\\\n& A_{i+5}=\\{i, i+2, i+3, i+5, i+6, i+7\\} ; \\\\\n& A_{i+9}=\\{i, i+4, i+6, i+7, i+9, i+10\\} ; \\\\\n& A_{i+10}=\\{i, i+1, i+5, i+7, i+8, i+10\\} .\n\\end{aligned}\n$$\n上述 6 个集合中除了 $i$ 出现 6 次以外, 其他 $i+1$, $i+2, i+3, i+4, i+5, i+6, i+7, i+8, i+ 9, i+10$ 都恰出现 3 次.\n故对任意 $i, j \\in\\{1,2,3$, $\\cdots, 11\\}, i \\neq j$, 有且只有 3 个集合包含 $i$ 与 $j$.\n假设将周长为 11 的圆周分为 11 等分, 将 11 个等分点按顺时针方向标记为 $1,2, \\cdots, 11$. 假设这个圆周上按顺时针方向从点 $i$ 到点 $j$ 的距离为 $d_{i j}$, 于是, 对于圆周上的点集 $A_i$ 可得下表,如图() \n\\begin{tabular}{|c|c|c|c|c|c|c|}\n\\hline$j$ & $i$ & $i+1$ & $i+2$ & $i+6$ & $i+8$ & $i+9$ \\\\\n\\hline$i$ & & & & & \\\\\n\\hline$i j$ & 11 & 1 & 2 & 6 & 8 & 9 \\\\\n\\hline$i+1$ & 10 & 11 & 1 & 5 & 7 & 8 \\\\\n\\hline$i+2$ & 9 & 10 & 11 & 4 & 6 & 7 \\\\\n\\hline$i+6$ & 5 & 6 & 7 & 11 & 2 & 3 \\\\\n\\hline$i+8$ & 3 & 4 & 5 & 9 & 11 & 1 \\\\\n\\hline$i+9$ & 2 & 3 & 4 & 8 & 10 & 11 \\\\\n\\hline\n\\end{tabular}\n由这个表可知在 $A_i=\\{i, i+1, i+2, i+6, i+8, i+9\\}$ 中, 按顺时针方向距离为 $1,2, \\cdots, 10$ 的点各有 3 对, 因为在圆周上将点集 $A_i$ 按顺时针方向旋转长为 $j-i$ 的距离后便得到点集 $A_j$, 故旋转后, $A_i$ 中的且只有 3 个点到达的位置正是 $A_j$ 中的三个点, 从而 $\\left|A_i \\cap A_j\\right|=3(1 \\leqslant i0$ (因为 $\\left(x_j-1\\right)-x_i \\geqslant 1$ ), 这与 $|S|$ 最小矛盾.\n又 $x_1+x_2+\\cdots+x_n=n m$, 所以 $x_1=x_2=\\cdots=x_n=m$ 时, $S$ 取最小值.\n故 $|S| \\geqslant n \\mathrm{C}_m^3$, 当且仅当 $x_1=x_2=\\cdots=x_n=m$ 时, 等号成立,所以\n$$\nN_n=\\mathrm{C}_n^4-|S|-|T| \\leqslant \\mathrm{C}_n^4-n \\mathrm{C}_m^3-0=\\frac{1}{48} n(n-3)\\left(n^2+6 n-31\\right) .\n$$\n(2)下面的设计方案表明 $N_n=\\mathrm{C}_n^4-n \\mathrm{C}_m^3$ 是可以成立的.\n首先将编号为 $1,2, \\cdots, n$ 的空间站依顺时针方向排在一个圆周上的 $n$ 个点 $A_1, A_2, \\cdots, A_n$ 处, 圆周上相邻两空间站的通道为双向通行主干道, 这样一共设置了 $n$ 条双向通行的主干道:\n$$\nA_1 A_2, A_2 A_3, \\cdots, A_{n-1} A_n, A_n A_1 .\n$$\n对任意 $i, j \\in\\{1,2,3, \\cdots, n\\}, i \\neq j$, 沿顺时针方向若从 $A_i$ 到 $A_j$ 的弧经过奇数个空间站, 那么规定 $A_i$ 与 $A_j$ 之间的通道是从 $A_i$ 到 $A_j$ 的严格单向通行道: $A_i \\rightarrow A_j$, 因为 $n$ 为奇数, 从 $A_i$ 到 $A_j$ 的顺时针方向的弧与从 $A_j$ 到 $A_i$ 的顺时针方向的弧当中恰有一个经过奇数空间站, 故上述规定不会导致矛盾.\n按此规定, 从每个 $A_i$ 出发的严格单向通行道的数目都为 $m=\\frac{1}{2}(n-3)$, 所以 $|S|=n \\mathrm{C}_m^3$. 下面证明: 此方案中必有 $|T|=0$.\n如果四站组中有两个空间站之间的通道是双向主干道, 那么易知这个四站组是互通四站组.\n因此, 如果四站组 $A 、 B 、 C 、 D$ 不是互通的, 那么它们中任何两站的通道都是严格单向通行道.\n设 $A$ 与 $B, B$ 与 $C, C$ 与 $D, D$ 与 $A$ 之间的空间站的个数分别为 $a, b, c, d$,于是 $a+b+c+d=n-4$ 为奇数, 从而 $a, b, c, d$ 中奇数个数是 1 和 3 .\n(i) 若 $a$ 为奇数, $b, c, d$ 为偶数, 则 $A \\rightarrow B \\rightarrow D \\rightarrow C \\rightarrow A$. 即 $A 、 B 、 C 、 D$ 为互通四站组, 如图() .\n(ii) 若 $a$ 为偶数, $b, c, d$ 为奇数, 则从 $B$ 到 $A 、 C 、 D$ 的通道都是从 $B$ 出发的严格单向通道, 这种非互通四站组属于 $S$ 类, 如图() .\n由以上讨论知此方案中 $|T|=0$, 从而 $|S|=\\mathrm{C}_n^4-n \\mathrm{C}_m^3$.\n综上可得, 互通四站组个数的最大值为 $\\mathrm{C}_n^4-n \\mathrm{C}_m^3=\\frac{1}{48} n(n-3)\\left(n^2+6 n-31\\right)$, 特别 $n=99$ 时, 本题所求互通四站组个数的最大值为 $\\mathrm{C}_{99}^4-99 \\mathrm{C}_{48}^3=2052072$.", + "remark": "", + "figures": [ + "./images/volume11/figures/fig-c15i4.png", + "./images/volume11/figures/fig-c15i5.png" + ] +} \ No newline at end of file diff --git a/processed_dataset/proof/0295.json b/processed_dataset/proof/0295.json new file mode 100644 index 0000000000000000000000000000000000000000..a2310d53ef85e0332453b6adb380bbc039500e89 --- /dev/null +++ b/processed_dataset/proof/0295.json @@ -0,0 +1,8 @@ +{ + "source_file": "./raw_volume-zh/volume11/chapter15.tex", + "problem_type": "proof", + "problem": "例8. 对于整数 $n \\geqslant 4$, 求出最小正整数 $f(n)$, 使得对任何正整数 $m$, 集合 $\\{m, m+1, \\cdots, m+n-1\\}$ 的任意 $f(n)$ 元子集中, 均有至少 3 个两两互素的元素.", + "solution": "解:设 $Z=\\{2,3, \\cdots, n+1\\}, T_n=A \\cup B$, 其中 $A$ 和 $B$ 分别为 $Z$ 中 2 的倍数和 3 的倍数组成的子集, 则\n$$\n\\begin{aligned}\n\\left|T_n\\right| & =|A \\cup B|=|A|+|B|-|A \\cap B| \\\\\n& =\\left[\\frac{n+1}{2}\\right]+\\left[\\frac{n+1}{3}\\right]-\\left[\\frac{n+1}{6}\\right] .\n\\end{aligned}\n$$\n且从 $T_n$ 中任取 3 个数, 其中必有 2 个数是 2 的倍数或 3 的倍数, 它们不互素, 所以\n$$\nf(n) \\geqslant\\left|T_n\\right|+1=\\left[\\frac{n+1}{2}\\right]+\\left[\\frac{n+1}{3}\\right]-\\left[\\frac{n+1}{6}\\right]+1, \\label{eq1}\n$$\n于是 $f(4) \\geqslant 4, f(5) \\geqslant 5, f(6) \\geqslant 5, f(7) \\geqslant 6, f(8) \\geqslant 7, f(9) \\geqslant 8$, 又显然 $f(n) \\leqslant n$, 故 $f(4)=4, f(5)=5$. 下证 $f(6)=5$.\n设 $x_1, x_2, \\cdots, x_5$ 为 $\\{m, m+1, \\cdots, m+5\\}$ 中任意 5 个数, 因为 $\\{m, m+ 1, \\cdots, m+5\\}$ 由 3 个奇数和 3 个偶数组成, 故 $x_1, x_2, \\cdots, x_5$ 中至少有 2 个奇数且至多有 3 个奇数.\n若 $x_1, x_2, \\cdots, x_5$ 中有 3 个奇数, 则必是 3 个连续的奇数, 它们两两互素; 若 $x_1, x_2, \\cdots, x_5$ 中有 3 个偶数, 不妨设 $x_1, x_2, x_3$ 为偶数, $x_4, x_5$ 为奇数, 则当 $1 \\leqslant i), 存在一个 2 色完全图 $K_5$ (其中实线表红色, 虚线表蓝色), 其中不存在同色三角形.\n一般地,使 $m$ 色完全图 $K_n$ 中存在同色三角形的最小正整数 $n$ 叫做 Ramsey 数, 记为 $R_m$. 上面结论实际上证明了 $R_2=6$. 寻求 Ramsey 数是一个很困难的问题, 目前仅找出了为数不多的几个 Ramsey 数: $R_2=6, R_3==17$, $R_4=65, \\cdots$. 详细情形读者可参看有关组合数学和图论的书籍.\n如果用点表示人, 并且两人互相认识时, 则对应点的连线染红色, 否则染蓝色.\n那么上述 Ramsey 定理便成为下述 1947 年的匈牙利数学竞赛试题:\n试证任何 6 个人中必有 3 个人互相认识或互相不认识.", + "remark": "", + "figures": [ + "./images/volume11/figures/fig-c2i1.png" + ] +} \ No newline at end of file diff --git a/processed_dataset/proof/0297.json b/processed_dataset/proof/0297.json new file mode 100644 index 0000000000000000000000000000000000000000..5c78b85161a0d97598f2c5d0f98c616bbb8cb6d4 --- /dev/null +++ b/processed_dataset/proof/0297.json @@ -0,0 +1,8 @@ +{ + "source_file": "./raw_volume-zh/volume11/chapter2.tex", + "problem_type": "proof", + "problem": "例3. 设 $\\alpha$ 是正实数, $n$ 为正整数,求证: 存在正整数 $p, q$ 使\n$$\n\\left|\\alpha-\\frac{q}{p}\\right| \\leqslant \\frac{1}{n p} \\text {. }\n$$", + "solution": "证明:先证下列命题:\n任给 $x_0, x_1, \\cdots, x_n \\in[0,1)$, 一定存在 $i, j(0 \\leqslant i0)$ 不被 $n$ 整除.\n考虑下列 $n$ 个数:\n$$\na_1, a_2, a_1+a_2, a_1+a_2+a_3, \\cdots, a_1+a_2+\\cdots+a_{n-1}\n$$\n(i)若这 $n$ 个数关于模 $n$ 的余数两两不同,则其中必有一个被 $n$ 整除.\n令此数为 $k n$ ( $k$ 为正整数). 若 $k$ 为偶数, 则结论成立.\n若 $k$ 为奇数, 则将此数加上\n$a_n=n$, 知结论也成立;\n(ii) 若这 $n$ 个数中有两个关于模 $n$ 同余,则它们之差被 $n$ 整除,但 $a_2-a_1$ 不被 $n$ 整除,故这个差必是 $a_1, a_2, \\cdots, a_{n-1}$ 中若干个数 (至少一个数)之和, 于是同(i)可证结论成立.", + "remark": "注:本题中 $n \\geqslant 4$ 是必要的, $n=3$ 时, 对集合 $\\{1,3,4\\}$, 题目结论不成立.", + "figures": [] +} \ No newline at end of file diff --git a/processed_dataset/proof/0299.json b/processed_dataset/proof/0299.json new file mode 100644 index 0000000000000000000000000000000000000000..fa38020067341f6093c8868bf7156f0511803f60 --- /dev/null +++ b/processed_dataset/proof/0299.json @@ -0,0 +1,8 @@ +{ + "source_file": "./raw_volume-zh/volume11/chapter2.tex", + "problem_type": "proof", + "problem": "例6. 49 个学生解 3 个问题,每个题的得分是 0 到 7 分的整数.\n求证: 存在两个学生 $A$ 和 $B$, 对每个问题, $A$ 的得分不少于 $B$.", + "solution": "证明:若有两个学生的第 $1 、 2$ 题的得分相同,设其中学生 $A$ 的第 3 题的得分不低于另一名学生 $B$, 于是, 对每一个问题, $A$ 的得分不低于 $B$, 结论成立.\n下设任意两名学生第 1,2 题的得分至少有一个不相同, 将每个学生用平面内的一个整点 $(i, j)$ 表示, 其中 $i, j$ 分别表示该学生在第 1,2 题的得分 $(0 \\leqslant i, j \\leqslant 7)$. 于是 49 个学生对应的整点互不相同.\n记\n$$\n\\begin{aligned}\n& M_1=\\{(i, j) \\mid i, j \\text { 为整数, } 0 \\leqslant i \\leqslant 7, j=0 \\text { 或 } i=7,1 \\leqslant j \\leqslant 7\\} ; \\\\\n& M_2=\\{(i, j) \\mid i, j \\text { 为整数, } 0 \\leqslant i \\leqslant 6, j=1 \\text { 或 } i=6,2 \\leqslant j \\leqslant 7\\} ; \\\\\n& M_3=\\{(i, j) \\mid i, j \\text { 为整数, } 0 \\leqslant i \\leqslant 5, j=2 \\text { 或 } i=5,3 \\leqslant j \\leqslant 7\\} ; \\\\\n& M_4=\\{(i, j) \\mid i, j \\text { 为整数, } 0 \\leqslant i \\leqslant 4, j=3 \\text { 或 } i=4,4 \\leqslant j \\leqslant 7\\} ; \\\\\n& M_5=\\{(i, j) \\mid i, j \\text { 为整数, } i=2,3,4 \\leqslant j \\leqslant 7\\} ; \\\\\n& M_6=\\{(i, j) \\mid i, j \\text { 为整数, } i=0,1,4 \\leqslant j \\leqslant 7\\} .\n\\end{aligned}\n$$\n因为 49 个学生对应的 49 个不同的整点属于上述 6 个集合, 故由抽屉原理知至少有 $\\left[\\frac{49-1}{6}\\right]+1=9$ 个整点属于同一个集合, 由于 $\\left|M_5\\right|=\\left|M_6\\right|=8$, 故这个集合只能是前 4 个集合中的一个,记这个集合为 $M$. 这 9 个整点对应的 9 个学生的第 3 题得分只有 $0,1,2, \\cdots, 7$ 这 8 种可能,再由抽屉原理知其中必有两个学生的第 3 题得分相同, 于是, 由 $M_1, M_2, M_3, M_4$ 的构造知, 这两个学生中必有一个学生 (记为 $A$ ), 他的第 $1 、 2$ 题的得分都不低于另一个学生 (记为 $B$ ), 故对每一个问题 $A$ 的得分不低于 $B$,结论得证.", + "remark": "注:(1) 本题中若将 49 个学生改为 48 个学生,则不保证原题结论成立: 我们用 $(a, b, c)$ 表示一个学生第 $1,2,3$ 题的得分分别为 $a, b, c$, 假设 48 个学生的得分如下:\n$$\n\\begin{aligned}\n& (3,7,0),(4,6,0),(5,5,0),(6,4,0),(7,3,0) ; \\\\\n& (2,7,1),(3,6,1),(4,5,1),(5,4,1),(6,3,1),(7,2,1) ; \\\\\n& (1,7,2),(2,6,2),(3,5,2),(4,4,2),(5,3,2),(6,2,2),(7,1,2)\n\\end{aligned}\n$$\n$$\n\\begin{aligned}\n& \\quad(0,7,3),(1,6,3),(2,5,3),(3,4,3),(4,3,3),(5,2,3),(6, \\\\\n& 1,3),(7,0,3) ; \\\\\n& \\quad(0,6,4),(1,5,4),(2,4,4),(3,3,4),(4,2,4),(5,1,4),(6,0,4) ; \\\\\n& \\quad(0,5,5),(1,4,5),(2,3,5),(3,2,5),(4,1,5),(5,0,5) ; \\\\\n& \\quad(0,4,6),(1,3,6),(2,2,6),(3,1,6),(4,0,6) ; \\\\\n& \\quad(0,3,7),(1,2,7),(2,1,7),(3,0,7) .\n\\end{aligned}\n$$\n则其中不存在两名学生 $A$ 和 $B$ 使得对每一个问题 $A$ 的得分却不低于 $B$.\n(2) 例 6 及注 (1) 中结论可等价地写成下列形式: 设 $D$ 为 $2^7 \\cdot 3^7 \\cdot 5^7$ 的所有不同正因数组成的集合, $S \\subseteq D$, 且 $S$ 内存在两个数 $A$ 和 $B$ 使得 $B$ 整除 $A$,那么 $S$ 内所含元素个数的最小值等于 49 .\n上述问题的一个自然推广是下列问题:\n问题 1 设 $n$ 为正整数, $D_n$ 为 $2^n 3^n 5^n$ 的所有不同正因数组成的集合, $S \\subseteq D_n$, 且 $S$ 中任意一数不整除 $S$ 中另一数.\n求 $|S|$ 的最大值.\n问题 1 的答数为 $\\left[\\frac{3(n+1)^2+1}{4}\\right]$. 换言之, 如果 $S$ 中存在 2 个数 $A$ 和 $B$, 使得 $B$ 整除 $A$, 那么 $|S|$ 的最小值为 $\\left[\\frac{3(n+1)^2+1}{4}\\right]+1$.", + "figures": [] +} \ No newline at end of file diff --git a/processed_dataset/proof/0300.json b/processed_dataset/proof/0300.json new file mode 100644 index 0000000000000000000000000000000000000000..3ba5f8c99c1be52e6ed6362274bbe9b964883f4e --- /dev/null +++ b/processed_dataset/proof/0300.json @@ -0,0 +1,8 @@ +{ + "source_file": "./raw_volume-zh/volume11/chapter2.tex", + "problem_type": "proof", + "problem": "例8. 平面内任给 $n(\\geqslant 4)$ 个点, 其中任意 4 点不共面, 若这些点之间连有 $\\left[\\frac{n^2}{4}\\right]+1$ 条线段,则存在两个有公共边的三角形.", + "solution": "证明:$n=4$ 时,一共有 4 个点 $A, B, C, D$, 它们之间连有 $\\left[\\frac{4^2}{4}\\right]+1=5$ 条线段,故其中只有 $\\mathrm{C}_4^2-5=1$ 对点之间没有连线,不妨设 $C$ 与 $D$ 没有连线, 这时存在两个有公共边的三角形: $\\triangle A B C$ 和 $\\triangle A B D$, 结论成立.\n假设 $n=k(\\geqslant 4)$ 时,结论成立, 即若 $k$ 个点之间连有 $\\left[\\frac{k^2}{4}\\right]+1$ 条线时, 则存在两个有公共边的三角形, 那么 $n=k+1$ 时, 假设 $k+1$ 个点之间连有 $\\left[\\frac{(k+1)^2}{4}\\right]+1$ 条线, 于是, 从各点出发的线段数之和为 $2\\left[\\frac{(k+1)^2}{4}\\right]+2$, 故由第二抽屉原理知, 其中必有一点 $A$, 从它出发的线段数至多为 $\\left[\\frac{1}{k+1}\\left(2\\left[\\frac{(k+1)^2}{4}\\right]+2\\right)\\right]$, 去掉这一点 $A$ 以及从 $A$ 出发的线段, 则还剩 $k$ 个点, 它们之间的连线数至少为\n$$\nN=\\left[\\frac{(k+1)^2}{4}\\right]+1-\\left[\\frac{1}{k+1}\\left(2\\left[\\frac{(k+1)^2}{4}\\right]+2\\right)\\right],\n$$\n而当 $k=2 m(m \\geqslant 2)$ 为偶数时, $N=m(m+1)+1-\\left[m+\\frac{m+2}{2 m+1}\\right]= m(m+1)+1-m=m^2+1=\\left[\\frac{k^2}{4}\\right]+1$, 当 $k=2 m-1(m \\geqslant 3)$ 为奇数时, $N= m^2+1-\\left[m+\\frac{1}{m}\\right]=m(m-1)+1=\\left[\\frac{k^2}{4}\\right]+1$, 即 $k$ 个点之间至少连有 $\\left[\\frac{k^2}{4}\\right]+1$ 条线段,故由归纳假设知存在两个有公共边的三角形, 于是原题结论得证.", + "remark": "注:当 $n \\geqslant 4$ 为偶数时, 本题为第二届国家集训队选拔考试试题.", + "figures": [] +} \ No newline at end of file diff --git a/processed_dataset/proof/0301.json b/processed_dataset/proof/0301.json new file mode 100644 index 0000000000000000000000000000000000000000..fedb3aa6592a0a1f8e16e60ea58a9057d2b43305 --- /dev/null +++ b/processed_dataset/proof/0301.json @@ -0,0 +1,8 @@ +{ + "source_file": "./raw_volume-zh/volume11/chapter2.tex", + "problem_type": "proof", + "problem": "例9. 将 10 个数 $1,2,3,4,5,6,7,8,9,10$ 按任意顺序排列成一个圆圈,证明: 其中必有连续相邻的 3 个数之和不小于 18 .", + "solution": "证明:设 10 个数在圆周上依次为 $a_1, a_2, \\cdots, a_{10}$, 则不妨设其中 $a_1=1$, 于是\n$$\n\\begin{aligned}\n& \\frac{1}{3}\\left[\\left(a_2+a_3+a_4\\right)+\\left(a_5+a_6+a_7\\right)+\\left(a_8+a_9+a_{10}\\right)\\right] \\\\\n= & \\frac{1}{3}(2+3+\\cdots+10)=\\frac{1}{3} \\times \\frac{1}{2} \\times 9 \\times 12=18,\n\\end{aligned}\n$$\n故 $a_2+a_3+a_4, a_5+a_6+a_7, a_8+a_9+a_{10}$ 中必有一个不小于 18 .", + "remark": "", + "figures": [] +} \ No newline at end of file diff --git a/processed_dataset/proof/0302.json b/processed_dataset/proof/0302.json new file mode 100644 index 0000000000000000000000000000000000000000..24cfaac29494a34aa30dd782919ebc938e565868 --- /dev/null +++ b/processed_dataset/proof/0302.json @@ -0,0 +1,8 @@ +{ + "source_file": "./raw_volume-zh/volume11/chapter2.tex", + "problem_type": "proof", + "problem": "例10. 平面内有 $n(\\geqslant 4)$ 个不同的点, 每两点间连一线段, 已知这些线段中恰有 $n+1$ 条长度等于 $d$ 的线段.\n证明: 其中必有一点, 从它出发的线段中至少有 3 条长度等于 $d$ 的线段.", + "solution": "证明:设 $n$ 个点为 $P_1, P_2, \\cdots, P_n$, 从 $P_i$ 出发的线段中恰有 $d_i$ 条长度为 $d$ 的线段 $(i=1,2, \\cdots, n)$, 于是\n$$\n\\begin{aligned}\n& d_1+d_2+\\cdots+d_n=2(n+1), \\\\\n& \\frac{1}{n}\\left(d_1+d_2+\\cdots+d_n\\right)=\\frac{2(n+1)}{n}>2,\n\\end{aligned}\n$$\n故其中必有某个 $d_i \\geqslant 3$, 即从 $P_i$ 出发至少有 3 条长度等于 $d$ 的线段.", + "remark": "", + "figures": [] +} \ No newline at end of file diff --git a/processed_dataset/proof/0303.json b/processed_dataset/proof/0303.json new file mode 100644 index 0000000000000000000000000000000000000000..91b41b1d7068744b145964eff15e7942d3232702 --- /dev/null +++ b/processed_dataset/proof/0303.json @@ -0,0 +1,8 @@ +{ + "source_file": "./raw_volume-zh/volume11/chapter2.tex", + "problem_type": "proof", + "problem": "例11. 已知 $f(z)=c_0 z^n+c_1 z^{n-1}+\\cdots+c_{n-1} z+c_n$ 是一个 $n$ 次复系数多项式, 求证: 一定存在复数 $z_0:\\left|z_0\\right| \\leqslant 1$, 使得 $\\left|f\\left(z_0\\right)\\right| \\geqslant\\left|c_0\\right|+\\left|c_n\\right|$.", + "solution": "证明一,设 $\\omega=\\cos \\frac{2 \\pi}{n}+\\mathrm{i} \\sin \\frac{2 \\pi}{n}, \\omega_k=\\omega^k=\\cos \\frac{2 k \\pi}{n}+\\mathrm{i} \\sin \\frac{2 k \\pi}{n}(k=0$, $1,2, \\cdots, n-1, n), \\alpha=\\cos \\theta+i \\sin \\theta$ ( $\\theta$ 待定), 于是 $\\omega^0=\\omega^n=1, \\omega^j \\neq 1(j=1,2, \\cdots, n-1)$, 从而有\n$$\n\\begin{aligned}\n& \\sum_{k=0}^{n-1} \\omega_k^j=n(j=0 \\text { 或 } n), \\\\\n& \\sum_{k=0}^{n-1} \\omega_k^j=\\sum_{k=0}^{n-1} \\omega^{j k}=\\frac{1-\\left(\\omega^j\\right)^n}{1-\\omega^j}=\\frac{1-1}{1-\\omega^j}=0(j=1,2, \\cdots, n-1),\n\\end{aligned}\n$$\n所以\n$$\n\\begin{aligned}\n\\sum_{k=0}^{n-1} f\\left(\\alpha \\omega_k\\right) & =c_0 \\alpha^n \\sum_{k=0}^{n-1} \\omega_k^n+c_1 \\alpha^{n-1} \\sum_{k=0}^{n-1} \\omega_k^{n-1}+\\cdots+c_{n-1} \\alpha \\sum_{k=0}^{n-1} \\omega_k+c_n \\cdot n \\\\\n& =n\\left(c_0 \\alpha^n+c_n\\right) .\n\\end{aligned}\n$$\n取 $\\alpha$ 的幅角 $\\theta$ 使 $c_0 \\alpha^n$ 与 $c_n$ 的幅角主值相等(事实上, 设 $c_0, c_n$ 的幅角主值分别为 $\\theta_0, \\theta_n$, 取 $\\theta=\\frac{1}{n}\\left(\\theta_n-\\theta_0\\right)$ 即可), 那么\n$$\n\\begin{aligned}\n\\sum_{k=0}^{n-1}\\left|f\\left(\\alpha \\omega_k\\right)\\right| & \\geqslant\\left|\\sum_{k=0}^{n-1} f\\left(\\alpha \\omega_k\\right)\\right|=n\\left|c_0 \\alpha^n+c_n\\right| \\\\\n& =n\\left(\\left|c_0\\right| \\cdot|\\alpha|^n+\\left|c_n\\right|\\right)=n\\left(\\left|c_n\\right|+\\left|c_0\\right|\\right) .\n\\end{aligned}\n$$\n由平均值原理知, 存在 $k_0$, 使\n$$\n\\left|f\\left(\\alpha \\omega_{k_0}\\right)\\right| \\geqslant \\frac{1}{n} \\sum_{k=0}^{n-1}\\left|f\\left(\\alpha \\omega_k\\right)\\right|=\\left|c_n\\right|+\\left|c_0\\right|,\n$$\n取 $z_0=\\alpha \\omega_{k_0}$, 则 $\\left|z_0\\right|=1$, 并且 $\\left|f\\left(z_0\\right)\\right| \\geqslant\\left|c_n\\right|+\\left|c_0\\right|$, 证毕.", + "remark": "", + "figures": [] +} \ No newline at end of file diff --git a/processed_dataset/proof/0304.json b/processed_dataset/proof/0304.json new file mode 100644 index 0000000000000000000000000000000000000000..8b661a4d2d496dd4262d914542891813e76cb59d --- /dev/null +++ b/processed_dataset/proof/0304.json @@ -0,0 +1,8 @@ +{ + "source_file": "./raw_volume-zh/volume11/chapter2.tex", + "problem_type": "proof", + "problem": "例11. 已知 $f(z)=c_0 z^n+c_1 z^{n-1}+\\cdots+c_{n-1} z+c_n$ 是一个 $n$ 次复系数多项式, 求证: 一定存在复数 $z_0:\\left|z_0\\right| \\leqslant 1$, 使得 $\\left|f\\left(z_0\\right)\\right| \\geqslant\\left|c_0\\right|+\\left|c_n\\right|$.", + "solution": "证明二取复数 $u$ 使 $u$ 与 $c_n$ 有相同的幅角并且 $|u|=\\left|c_0\\right|+\\left|c_n\\right|$, 构造多项式\n$$\ng(z)=f(z)-u=c_0 z^n+c_1 z^{n-1}+\\cdots+c_{n-1} z+c_n-u .\n$$\n设 $g(z)$ 的 $n$ 个复根为 $z_1, z_2, \\cdots, z_n$, 则\n$$\n\\begin{aligned}\n& \\left|z_1 z_2 \\cdots z_n\\right|=\\left|\\frac{c_n-u}{c_0}\\right|=\\frac{|| c_n|-| u||}{\\left|c_0\\right|}=\\frac{\\left|c_0\\right|}{\\left|c_0\\right|}=1 . \\\\\n& \\sqrt[n]{\\left|z_1\\right| \\cdot\\left|z_2\\right| \\cdots\\left|z_n\\right|}=1 .\n\\end{aligned}\n$$\n故由平均值原理知, 存在 $k_0\\left(1 \\leqslant k_0 \\leqslant n\\right)$ 使\n$$\n\\left|z_{k_0}\\right| \\leqslant 1 \\text {. }\n$$\n记 $z_0=z_{k_0}$, 则 $\\left|z_0\\right| \\leqslant 1$, 且\n$$\ng\\left(z_0\\right)=f\\left(z_0\\right)-u=0 .\n$$\n所以, $\\left|f\\left(z_0\\right)\\right|=|u|=\\left|c_0\\right|+\\left|c_n\\right|$, 证毕.", + "remark": "", + "figures": [] +} \ No newline at end of file diff --git a/processed_dataset/proof/0305.json b/processed_dataset/proof/0305.json new file mode 100644 index 0000000000000000000000000000000000000000..f40f92e2d1eb2ee974b31daeedf32a76aa1a3ed6 --- /dev/null +++ b/processed_dataset/proof/0305.json @@ -0,0 +1,8 @@ +{ + "source_file": "./raw_volume-zh/volume11/chapter3.tex", + "problem_type": "proof", + "problem": "例2. 证明: 对一切正整数 $n$, 有\n$$\n\\sum_{i=0}^n \\mathrm{C}_{2 n+1}^{2 i} \\mathrm{C}_{2 i}^i 2^{2 n-2 i+1}=\\mathrm{C}_{4 n+2}^{2 n+1} \\text {. }\n$$", + "solution": "证明:一方面 $(1+x)^{4 n+2}=\\sum_{i=0}^{4 n+2} \\mathrm{C}_{4 n+2}^i x^i$ 中 $x^{2 n+1}$ 的系数为 $\\mathrm{C}_{4 n+2}^{2 n+1}$, 另一方面\n$$\n\\begin{aligned}\n(1+x)^{4 n+2} & =\\left[\\left(1+x^2\\right)+2 x\\right]^{2 n+1} \\\\\n& =\\sum_{k=0}^{2 n+1} \\mathrm{C}_{2 n+1}^k\\left(1+x^2\\right)^k(2 x)^{2 n+1-k} \\\\\n& =\\sum_{k=0}^{2 n+1} \\mathrm{C}_{2 n+1}^k 2^{2 n+1-k} \\cdot x^{2 n+1-k}\\left(\\sum_{j=0}^k \\mathrm{C}_k^j x^{2 j}\\right)\n\\end{aligned}\n$$\n中 $x^{2 n+1}$ 的系数为 $\\sum_{i=0}^n \\mathrm{C}_{2 n+1}^{2 i} 2^{2 n+1-2 i} \\cdot \\mathrm{C}_{2 i}^i$, 所以\n$$\n\\sum_{i=0}^n \\mathrm{C}_{2 n+1}^{2 i} \\mathrm{C}_{2 i}^i 2^{2 n-2 i+1}=\\mathrm{C}_{4 n+2}^{2 n+1} .\n$$", + "remark": "", + "figures": [] +} \ No newline at end of file diff --git a/processed_dataset/proof/0306.json b/processed_dataset/proof/0306.json new file mode 100644 index 0000000000000000000000000000000000000000..7147f8da48573dd364333744f007237aa60ec996 --- /dev/null +++ b/processed_dataset/proof/0306.json @@ -0,0 +1,8 @@ +{ + "source_file": "./raw_volume-zh/volume11/chapter3.tex", + "problem_type": "proof", + "problem": "例3. 证明: $\\sum_{k=0}^{\\left[\\frac{n-1}{2}\\right]}(-1)^k \\mathrm{C}_{n+1}^k \\mathrm{C}_{2 n-2 k-1}^n=\\frac{1}{2} n(n+1)(n \\geqslant 1)$.", + "solution": "证明:一方面 $(1+x)^{n+1}=\\sum_{k=0}^{n+1} \\mathrm{C}_{n+1}^k x^k$ 中 $x^{n-1}$ 的系数为 $\\mathrm{C}_{n+1}^{n-1}=\\mathrm{C}_{n+1}^2= \\frac{1}{2} n(n+1)$, 另一方面, $(1+x)^{n+1}=\\frac{\\left(1-x^2\\right)^{n+1}}{(1-x)^{n+1}}=\\left(\\sum_{k=0}^n(-1)^k \\mathrm{C}_{n+1}^k x^{2 k}\\right)\\left(\\sum_{j=0}^{\\infty} \\mathrm{C}_{n+j}^n x^j\\right)$ 中 $x^{n-1}$ 的系数为 $\\sum_{k=0}^{\\left[\\frac{n-1}{2}\\right]}(-1)^k \\mathrm{C}_{n+1}^k \\mathrm{C}_{n+(n-1-2 k)}^n=\\sum_{k=0}^{\\left[\\frac{n-1}{2}\\right]}(-1)^k \\mathrm{C}_{n+1}^k \\mathrm{C}_{2 n-2 k-1}^n$, 所以 $\\sum_{k=0}^{\\left[\\frac{n-1}{2}\\right]}(-1)^k \\mathrm{C}_{n+1}^k \\mathrm{C}_{2 n-2 k-1}^n=\\frac{1}{2} n(n+1)$.", + "remark": "注:当恒等式中流动标号只出现在组合数 $\\mathrm{C}_m^n$ 的下位 $m$ 时, 可考虑用公式 $\\frac{1}{(1-x)^{n+1}}=\\sum_{j=0}^{\\infty} \\mathrm{C}_{n+j}^n x^j$.", + "figures": [] +} \ No newline at end of file diff --git a/processed_dataset/proof/0307.json b/processed_dataset/proof/0307.json new file mode 100644 index 0000000000000000000000000000000000000000..5f3918b70822faa960485427e2e58776a7c223ba --- /dev/null +++ b/processed_dataset/proof/0307.json @@ -0,0 +1,8 @@ +{ + "source_file": "./raw_volume-zh/volume11/chapter4.tex", + "problem_type": "proof", + "problem": "例3. 设 $\\frac{1}{2}\\left(1+\\frac{1}{2}\\right)+\\left(1+\\frac{1}{2^2}\\right)+\\cdots+\\left(1+\\frac{1}{2^n}\\right) \\\\\n& =n+1-\\frac{1}{2^n}>n+1-\\frac{1}{2}=n+\\frac{1}{2} .\n\\end{aligned}\n$$\n即\n$$\nn+\\frac{1}{2}<\\frac{1}{a_1}+\\frac{1}{a_2}+\\cdots+\\frac{1}{a_n}0$. 令 $a_n=\\sin \\lambda_n\\left(0<\\lambda_n<\\frac{\\pi}{2}\\right.$, $n \\geqslant 0)$, 则\n$$\n\\sin \\lambda_{n+1}=\\frac{\\sqrt{2}}{2} \\sqrt{1-\\sqrt{1-\\sin ^2 \\lambda_n}}=\\frac{\\sqrt{2}}{2} \\sqrt{1-\\cos \\lambda_n}=\\sin \\frac{\\lambda_n}{2} .\n$$\n从而有 $\\lambda_{n+1}=\\frac{1}{2} \\lambda_n(n \\geqslant 0)$, 又 $\\lambda_0=\\arcsin a_0=\\arcsin \\frac{\\sqrt{2}}{2}=\\frac{\\pi}{4}$, 所以\n$$\n\\lambda_n=\\frac{\\pi}{4}\\left(\\frac{1}{2}\\right)^n=\\frac{\\pi}{2^{n+2}}, a_n=\\sin \\lambda_n=\\sin \\frac{\\pi}{2^{n+2}}(n \\geqslant 0) .\n$$\n类似地, 令 $b_n=\\tan \\delta_n$, 可求得 $b_n=\\tan \\frac{\\pi}{2^{n+2}}(n \\geqslant 0)$. 由于当 $0\\frac{1}{2 \\times 133} \\times 2 \\times 6 \\times 133 \\times(2 \\times 6 \\times 133-133) \\\\\n& =\\frac{133 \\times 132}{2}=\\mathrm{C}_{133}^2 .\n\\end{aligned}\n$$\n而 133 个点一共只能形成 $\\mathrm{C}_{133}^2$ 个点对.\n故上述左端点对的计数有重复, 即至少存在一个点对 $(B, D)$ 属于两个不同的点 $A$ 和 $C$, 可见 $B$ 和 $D$ 既与 $A$ 连有线段, 又与 $C$ 连有线段.\n于是 $A, B, C, D$ 对应的正整数 $a, b, c, d$ 满足 $a$ 与 $b$, $b$ 与 $c, c$ 与 $d, d$ 与 $a$ 都互素,命题得证.", + "remark": "", + "figures": [] +} \ No newline at end of file diff --git a/processed_dataset/proof/0317.json b/processed_dataset/proof/0317.json new file mode 100644 index 0000000000000000000000000000000000000000..1e8ad09f9766af8eb00595b728179bd05c44a493 --- /dev/null +++ b/processed_dataset/proof/0317.json @@ -0,0 +1,8 @@ +{ + "source_file": "./raw_volume-zh/volume11/chapter6.tex", + "problem_type": "proof", + "problem": "例3. 设有 11 个集合 $M_1, M_2, \\cdots, M_{11}$, 满足\n(i) $\\left|M_i\\right|=5, i=1,2, \\cdots, 11$;\n(ii) $\\left|M_i \\cap M_j\\right| \\neq 0,1 \\leqslant i, j \\leqslant 11$.\n记 $T=M_1 \\cup M_2 \\cup \\cdots \\cup M_{11}$, 并对任意 $x \\in T$, 令\n$$\n\\begin{aligned}\n& n(x)=\\left|\\left\\{M_i \\mid x \\in M_i, 1 \\leqslant i \\leqslant 11\\right\\}\\right|, \\\\\n& n=\\max \\{n(x) \\mid x \\in T\\},\n\\end{aligned}\n$$\n求 $n$ 的可能值中的最小值.\n说明 $\\left|M_i\\right|$ 表示集合 $M_i$ 中元素的个数,即 $\\left|M_i\\right|=\\operatorname{Card} M_i . n(x)$ 表示包含元素 $x \\in T$ 的集合 $M_i$ 的个数,而 $n$ 是所有这些 $n(x)$ 的最大值.", + "solution": "解:设 $T=\\left\\{x_1, x_2, \\cdots, x_m\\right\\}$, 作 $m \\times 11$ 数表, 其中第 $i$ 行第 $j$ 列处的数为\n$$\na_{i j}=\\left\\{\\begin{array}{ll}\n1 & \\left(\\text { 若 } x_i \\in M_j\\right), \\\\\n0 & \\left(\\text { 若 } x_i \\notin M_j\\right)\n\\end{array}(i=1,2, \\cdots, m, j=1,2, \\cdots, 11) .\\right.\n$$\n于是 $n\\left(x_i\\right)=\\sum_{j=1}^{11} a_{i j}$ 表示 $x_i$ 属于 $M_1, M_2, \\cdots, M_{11}$ 中 $n\\left(x_i\\right)$ 个集合,而 $\\left|M_j\\right|=\\sum_{i=1}^m a_{i j}$ 表示 $M_j$ 中元素的个数, 从而由已知条件(i)有\n$$\n\\begin{aligned}\n\\sum_{i=1}^m n\\left(x_i\\right) & =\\sum_{i=1}^m \\sum_{j=1}^{11} a_{i j}=\\sum_{j=1}^{11} \\sum_{i=1}^m a_{i j} \\\\\n& =\\sum_{j=1}^{11}\\left|M_j\\right|=11 \\times 5=55 . \\label{eq1}\n\\end{aligned}\n$$\n若集合 $M_j, M_k(j \\neq k)$ 都包含元素 $x_i$, 则将 $M_j$ 与 $M_k$ 配成一对, 并称 $\\left(M_j, M_k\\right)$ 是属于元素 $x_i$ 的集合对, 于是分别属于 $x_1, x_2, \\cdots, x_m$ 的集合对的个数的总和为\n$$\n\\begin{aligned}\n\\sum_{i=1}^m \\mathrm{C}_{n\\left(x_i\\right)}^2 & =\\frac{1}{2} \\sum_{i=1}^m n\\left(x_i\\right)\\left(n\\left(x_i\\right)-1\\right) \\\\\n& \\leqslant \\frac{1}{2}(n-1) \\sum_{i=1}^m n\\left(x_i\\right), \\label{eq2}\n\\end{aligned}\n$$\n另一方面, 由已知条件 (ii) 知, 对每对集合 $\\left(M_i, M_j\\right)(ix$. 可见, 在波比写完后, 每个对子中的最大数总在 $A \\cup B$ 内, 从而每行中的最大数总在 $A \\cup B$ 内.\n于是, 当所有数写完后, 第一行中的最大数在 $A$ 内, 第 6 行的最大数在 $B$ 内.\n因为不可能在 $A \\cup B$ 内画一条线从 $A$ 达到 $B$, 所以波比必获胜.", + "remark": "", + "figures": [] +} \ No newline at end of file diff --git a/processed_dataset/proof/0319.json b/processed_dataset/proof/0319.json new file mode 100644 index 0000000000000000000000000000000000000000..f644203366cc415cb9bda588cd3ccd28eee7a158 --- /dev/null +++ b/processed_dataset/proof/0319.json @@ -0,0 +1,8 @@ +{ + "source_file": "./raw_volume-zh/volume11/chapter6.tex", + "problem_type": "proof", + "problem": "例6. 甲乙两人进行如下游戏, 甲先开始, 两人轮流从 $1,2,3, \\cdots, 100,101$,\n中每次任意勾去 9 个数, 经过 11 次操作后, 还剩两个数, 这时余下两个数之差即为甲的得分, 试证不论乙怎么做, 甲至少可得 55 分.", + "solution": "证明:甲第一次勾掉 $47,48,49, \\cdots, 55$ 这 9 个数,将剩下的数两两配对: $\\{i, 55+i\\}(i=1,2, \\cdots, 46\\}$, 同一对中两数之差为 55 . 在每次乙勾掉 9 个数之后, 甲的策略是使得甲勾掉的 9 个数与乙勾掉的 9 个数恰好组成上述 46 对数中的 9 对.\n这样一来, 最后余下的两个数必须是上述 46 对数中的一对,这两个数之差必为 55 , 可见甲可保证自己得 55 分.", + "remark": "", + "figures": [] +} \ No newline at end of file diff --git a/processed_dataset/proof/0320.json b/processed_dataset/proof/0320.json new file mode 100644 index 0000000000000000000000000000000000000000..cd978ff437756becbae2c97a8e08bb93bf20c416 --- /dev/null +++ b/processed_dataset/proof/0320.json @@ -0,0 +1,11 @@ +{ + "source_file": "./raw_volume-zh/volume11/chapter6.tex", + "problem_type": "proof", + "problem": "例7. 在一个无限大的方格纸上, 甲、乙两人轮流在空格上放棋子, 每次放一枚, 甲放黑棋子,乙放白棋子.\n如果在某一行、或某一列、或一条对角线上出现 11 枚连续摆放着的黑棋子, 则先放棋子的甲获胜, 证明: 乙总能阻止甲获胜.", + "solution": "证明:如图() 所示: 将一个 $4 \\times 4$ 正方形和一个与其相邻的 $2 \\times 2$ 正方形中 20 个方格内周期地填上 $0,1,2,3,4$ 这 5 个数字.\n我们把图中如图() 所示的 4 种已填数字的 2 个小方格组成的图形称之为一块多米诺骨牌.\n易见, 无论是在横行、坚列, 还是平行于对角线的直线上, 任何连续 11 个方格都含有一块多米诺骨牌.\n因此, 一旦甲在某块多米诺骨牌的一个方格内放上黑棋子, 则乙在另一个方格内放上白棋子, 这样一来, 甲就无法取胜了 (若甲在填数字 0 的方格上放黑棋子,乙就在其有公共边的另一个填数字 0 的方格上放白棋子).\n\\begin{tabular}{|l|l|l|l|l|l|l|l|l|l|l|l|l|l|}\n\\hline 1 & 4 & 3 & 3 & 2 & 2 & 4 & 1 & 0 & 0 & 1 & 4 & 3 & 3 \\\\\n\\hline 1 & 4 & 2 & 2 & 3 & 3 & 4 & 1 & 0 & 0 & 1 & 4 & 2 & 2 \\\\\n\\hline 2 & 2 & 4 & 1 & 0 & 0 & 1 & 4 & 3 & 3 & 2 & 2 & 4 & 1 \\\\\n\\hline 3 & 3 & 4 & 1 & 0 & 0 & 1 & 4 & 2 & 2 & 3 & 3 & 4 & 1 \\\\\n\\hline 0 & 0 & 1 & 4 & 3 & 3 & 2 & 2 & 4 & 1 & 0 & 0 & 1 & 4 \\\\\n\\hline 0 & 0 & 1 & 4 & 2 & 2 & 3 & 3 & 4 & 1 & 0 & 0 & 1 & 4 \\\\\n\\hline 3 & 3 & 2 & 2 & 4 & 1 & 0 & 0 & 1 & 4 & 3 & 3 & 2 & 2 \\\\\n\\hline 2 & 2 & 3 & 3 & 4 & 1 & 0 & 0 & 1 & 4 & 2 & 2 & 3 & 3 \\\\\n\\hline 4 & 1 & 0 & 0 & 1 & 4 & 3 & 3 & 2 & 2 & 4 & 1 & 0 & 0 \\\\\n\\hline 4 & 1 & 0 & 0 & 1 & 4 & 2 & 2 & 3 & 3 & 4 & 1 & 0 & 0 \\\\\n\\hline 1 & 4 & 3 & 3 & 2 & 2 & 4 & 1 & 0 & 0 & 1 & 4 & 3 & 3 \\\\\n\\hline 1 & 4 & 2 & 2 & 3 & 3 & 4 & 1 & 0 & 0 & 1 & 4 & 2 & 2 \\\\\n\\hline\n\\end{tabular}", + "remark": "", + "figures": [ + "./images/volume11/figures/fig-c6i1.png", + "./images/volume11/figures/fig-c6i2.png" + ] +} \ No newline at end of file diff --git a/processed_dataset/proof/0321.json b/processed_dataset/proof/0321.json new file mode 100644 index 0000000000000000000000000000000000000000..525eaffafeda213a4e78a376907f355f12bf5361 --- /dev/null +++ b/processed_dataset/proof/0321.json @@ -0,0 +1,8 @@ +{ + "source_file": "./raw_volume-zh/volume11/chapter6.tex", + "problem_type": "proof", + "problem": "例8. 把 $n$ 个物体排成一行, 如果这些物体的某个子集合中任何两个元素均不相邻, 则称这个子集是不亲切的, 证明: 其中含有 $k$ 个元素的不亲切子集的个数是 $\\mathrm{C}_{n-k+1}^k$.", + "solution": "证明:我们将排成一行的物体记为 $a_1, a_2, \\cdots, a_n$, 且设 $\\left\\{a_{i_1}, a_{i_2}, \\cdots\\right.$, $a_{i_k}$ 是它的一个含 $k$ 个元素的不亲切子集 $\\left(1 \\leqslant i_1|Y|$, 所以 $f$ 不是单射, 故存在 $\\{a, b\\},\\{c, d\\} \\in X,\\{a, b\\} \\neq\\{c, d\\}$ 使 $m_{a b}=f(\\{a, b\\})= f(\\{c, d\\})=m_{c d}$, 从而\n$$\na b c d=K_{a b}^2 m_{a b} \\cdot K_{c d}^2 m_{c d}=\\left(K_{a b} K_{c d} m_{a b}\\right)^2\n$$\n为完全平方数, 如果 $a, b, c, d$ 互不相等, 那么结论成立.\n否则 $\\{a, b\\}$ 中恰有一个数与 $\\{c, d\\}$ 中一个数相等.\n不妨设 $a \\neq c, b=d$. 于是由 $a b c d=a c b^2$ 是完全平方数知 $a c$ 是完全平方数, 从 $M$ 中去掉 $a, c$ 两个数, 还剩 46 个数, 因为 $\\mathrm{C}_{46}^2=1035>1024=|Y|$, 故同理可证 $X \\backslash\\{\\{a, c\\}\\}$ 中存在两个不同的二元子集 $\\left\\{a^{\\prime}, b^{\\prime}\\right\\},\\left\\{c^{\\prime}, d^{\\prime}\\right\\}$ 使 $a^{\\prime} b^{\\prime} c^{\\prime} d^{\\prime}$ 为完全平方数.\n如果 $a^{\\prime}, b^{\\prime}, c^{\\prime}, d^{\\prime}$ 互不相同, 那么结论成立, 否则同理可以不妨设 $a^{\\prime} \\neq c^{\\prime}, b^{\\prime}=d^{\\prime}$ 且 $a^{\\prime} c^{\\prime}$ 为完全平方数, 于是存在两个没有公共元的二元子集 $\\{a, c\\},\\left\\{a^{\\prime}, c^{\\prime}\\right\\}$, 使 $a c a^{\\prime} c^{\\prime}$ 为完全平方数.\n证毕.\n除了上述一对一的映射方法外, 有时还要用到一个对多个的对应方法.", + "remark": "", + "figures": [] +} \ No newline at end of file diff --git a/processed_dataset/proof/0325.json b/processed_dataset/proof/0325.json new file mode 100644 index 0000000000000000000000000000000000000000..3b4798df91f42678cf46fdf0fca58f7d9a9706b9 --- /dev/null +++ b/processed_dataset/proof/0325.json @@ -0,0 +1,8 @@ +{ + "source_file": "./raw_volume-zh/volume11/chapter7.tex", + "problem_type": "proof", + "problem": "例3. 有 $n$ 个人,已知他们任意 2 人至多通话一次,他们任意 $n-2$ 个人之间通话的总次数相等, 都等于 $3^k$ ( $k$ 为正整数). 求 $n$ 的所有可能值.", + "solution": "解:设 $n$ 个人之间通话的总次数为 $m$, 因为 $n$ 个人可形成 $\\mathrm{C}_n^{n-2}$ 个 $n-2$ 人组, 而每 $n-2$ 人之间通话的总次数都为 $3^k$, 故所有 $n-2$ 组中通话次数的总和为 $\\mathrm{C}_n^{n-2} \\cdot 3^k$.\n另一方面, 上述计数中, 每一对通话的人, 属于 $\\mathrm{C}_{n-2}^{n-4}$ 个 $n-2$ 组, 故每 2 人间的一次通话重复计算了 $\\mathrm{C}_{n-2}^{n-4}$ 次,所以\n$$\nm=\\frac{\\mathrm{C}_n^{n-2} \\cdot 3^k}{\\mathrm{C}_{n-2}^{n-4}}=\\frac{\\mathrm{C}_n^2 \\cdot 3^k}{\\mathrm{C}_{n-2}^2}=\\frac{n(n-1) 3^k}{(n-2)(n-3)} .\n$$\n(1) 若 3 不整除 $n$, 即 $(3, n)=1$, 则 $(n-3, n)=1,\\left(n-3,3^k\\right)=1$, 又 $(n-2, n-1)=1$, 所以 $n-3 \\mid n-1$, 即 $\\frac{n-1}{n-3}=1+\\frac{2}{n-3}$ 为正整数, 所以 $n-3 \\mid 2, n-3 \\leqslant 2, n \\leqslant 5$. 又 $\\mathrm{C}_{n-2}^2 \\geqslant 3^k \\geqslant 3$, 所以 $n \\geqslant 5$, 故 $n=5$.\n(2) 若 3 整除 $n$, 则 $3 \\mid n-3,3 \\times n-2$, 即 $(3, n-2)=1$, 又 $(n-2$, $n-1)=1$, 所以 $n-2 \\mid n$, 即 $\\frac{n}{n-2}=1+\\frac{2}{n-2}$ 为正整数, 故 $n-2 \\mid 2$, 由此得 $n-2 \\leqslant 2, n \\leqslant 4$, 这与 (1)中已证 $n \\geqslant 5$ 矛盾.\n由 (1), (2) 知 $n$ 只可能为 5 , 另一方面, 若有 $n=5$ 个人, 其中每 2 人通一次电话, 则任意 $n-2=3$ 人之间通电话的次数都为 $\\mathrm{C}_3^2=3^1$ (这里 $k=1$ 为正整数) 满足题目要求, 故所求正整数只有一个 $n=5$.", + "remark": "", + "figures": [] +} \ No newline at end of file diff --git a/processed_dataset/proof/0326.json b/processed_dataset/proof/0326.json new file mode 100644 index 0000000000000000000000000000000000000000..e3b10ddcf0efb10e98f40ffac78be95ca366fd15 --- /dev/null +++ b/processed_dataset/proof/0326.json @@ -0,0 +1,8 @@ +{ + "source_file": "./raw_volume-zh/volume11/chapter7.tex", + "problem_type": "proof", + "problem": "例4. 设 $2 \\leqslant r \\leqslant \\frac{n}{2}, \\mathscr{A}$ 为 $Z=\\{1,2, \\cdots, n\\}$ 的一些 $r$ 元子集所成的集族.\n如果 $\\mathscr{A}$ 中的每两个元素 ( $Z$ 的 $r$ 元子集) 的交非空, 那么 $|\\mathscr{A}| \\leqslant \\mathrm{C}_{n-1}^{r-1}$, 当 $\\mathscr{A}=\\{A \\mid A$ 为 $Z$ 的 $r$ 元子集且含有 $Z$ 的一个固定元素 $x\\}$ 时等号成立 (ErdösKo-Rado 定理)", + "solution": "证明:把 $Z$ 的元素任意排在一个圆周上有 $(n-1)$ ! 种排列方法, 对于每种排列方法 $\\pi_j(j=1,2, \\cdots,(n-1) !)$, 如果 $A_i$ 中的元素在 $\\pi_j$ 中相连没有间断, 则将 $\\left(A_i, \\pi_j\\right)$ 配成一对, 这种对子的集合记为 $S$. 一方面, 因为 $\\mathscr{A}$ 中每两个元素 ( $Z$ 的 $r$ 元子集) 的交非空, 且 $2 \\leqslant r \\leqslant \\frac{n}{2}$, 故对每个排列 $\\pi_j$, 至多有 $r$ 个集合 $A_i \\in \\mathscr{A}$ 使得每个 $A_i$ 中的元素在 $\\pi_j$ 中是相连没有间断的, 即含 $\\pi_j$ 的对子 $\\left(A_i, \\pi_j\\right)$ 至多有 $r$ 个, 而 $\\pi_j$ 有 $(n-1)$ ! 个, 所以\n$$\n|S| \\leqslant r \\cdot(n-1) ! . \\label{eq1}\n$$\n另一方面, 对 $\\mathscr{A}$ 中每个 $A_i$, 使 $A_i$ 中元素在圆周上相连不间断的圆排列有 $r !(n-r) !$ 个,而 $\\mathscr{A}$ 中元素 $A_i$ 共有 $|\\mathscr{A}|$ 个,故\n$$\n|S|=|\\mathscr{A}| \\cdot r ! \\cdot(n-r) !, \\label{eq2}\n$$\n由式\\ref{eq1}及\\ref{eq2}得\n$$\n|\\mathscr{A}| \\cdot r ! \\cdot(n-r) ! \\leqslant r(n-1) ! .\n$$\n由此可得 $|\\mathscr{A}| \\leqslant \\frac{(n-1) !}{(r-1) !(n-r) !}=\\mathrm{C}_{n-1}^{r-1}$.\n其次, 当 $\\mathscr{A}==\\{A \\mid A$ 为 $Z$ 的 $r$ 元子集且含 $Z$ 的一个固定元素 $x\\}$ 时, $\\mathscr{A}$ 中的 $A$ 恰有 $\\mathrm{C}_{n-1}^{r-1}$ 个, 这时 $|\\mathscr{A}|=\\mathrm{C}_{n-1}^{r-1}$ 成立.", + "remark": "注:进一步可以证明, 本题中等号成立的条件不仅是充分的, 也是必要的.", + "figures": [] +} \ No newline at end of file diff --git a/processed_dataset/proof/0327.json b/processed_dataset/proof/0327.json new file mode 100644 index 0000000000000000000000000000000000000000..3829078864ac4109be7eeee9a0a91d50bfdb5ba2 --- /dev/null +++ b/processed_dataset/proof/0327.json @@ -0,0 +1,8 @@ +{ + "source_file": "./raw_volume-zh/volume11/chapter7.tex", + "problem_type": "proof", + "problem": "例5. 已知集合 $M=\\left\\{x_1, x_2, \\cdots, x_{4 n+3}\\right\\}$, 它的 $4 n+3$ 个子集 $A_1, A_2$, $\\cdots, A_{4 n+3}$ 具有以下性质:\n(1) $M$ 中每 $n+1$ 个元素恰属于唯一一个子集 $A_j(1 \\leqslant j \\leqslant 4 n+3)$;\n(2) $\\left|A_i\\right| \\geqslant 2 n+1(i=1,2, \\cdots, 4 n+3)$.\n证明: 任意两个子集 $A_i$ 与 $A_j(1 \\leqslant i0, \\\\\n& y-x=\\left(a_n-2\\right)\\left(2 a_n-1\\right)^2+1>0,\n\\end{aligned}\n$$\n即 $z>y>x$, 又 $x+y=a_n\\left(2 a_n-1\\right)^2+1>a_n\\left(2 a_n-1\\right)^2-1=z$, 故以 $x$, $y, z$ 为边长可构成三角形, 且这个三角形的面积为\n$$\nS=a_n\\left(2 a_n-2\\right)\\left(2 a_n-1\\right)^2 .\n$$\n它是一个完全平方数, 这就证明了存在无穷多个满足题目条件的三角形.", + "remark": "注:从例 7 和例 8 可以看出, 在组合数论和组合几何中, 常常要证明存在无穷多个数组或几何图形满足一定的条件 (这些条件通常与正整数有关). 为此, 我们可先从一些特殊情形或简单情形做起, 进行探索与归纳, 构造对应的递推数列来完成证明.", + "figures": [] +} \ No newline at end of file diff --git a/processed_dataset/proof/0337.json b/processed_dataset/proof/0337.json new file mode 100644 index 0000000000000000000000000000000000000000..8245d5e3aa5a19e893b9ba1dcdfd72ece4cb3198 --- /dev/null +++ b/processed_dataset/proof/0337.json @@ -0,0 +1,10 @@ +{ + "source_file": "./raw_volume-zh/volume11/chapter9.tex", + "problem_type": "proof", + "problem": "例6. 把正六边形分成 24 个全等的小正三角形,在 19 个交点处任意写上互不相等的实数.\n证明: 24 个小正三角形中至少存在 7 个三角形,其顶点上所写的 3 个实数可按顺时针方向排成递减数列.", + "solution": "解:将 24 个三角形分为两类: 如图() 所示, 第I类三角形的三个顶点上所写的 3 个实数可按顺时针方向排成递减数列, 其余三角形为第 II 类, 并设第 I 类正三角形有 $N$ 个, 从而第 II 类正三角形有 $24-N$ 个, 我们将小正三角形每条边按照从大数对应顶点到小数对应顶点的方向画一个箭头 (表示递减!)并且每条边沿箭头方向在其右侧赋值 1 ,左侧赋值 -1 (右侧表明箭头方向为顺时针的, 左侧表明箭头方向为逆时针的), 于是第 I 类三角形内所赋 3 个数值之和为 $1+1+(-1)=1$, 第 II 类三角形内所赋 3 个数值之和为 $(-1)+(-1)+1=-1$ 于是, 24 个小正 $(a>b>c)$ 三角形内所赋数值的总和为\n$$\nN+(-1)(24-N)=2 N-24 .\n$$\n另一方面, 位于正六边形内的小三角形的每边两侧所赋的值之和为 $1+ (-1)=0$, 而位于正六边形的边上的 12 条小正三角形的边中至少有一条边在正六边形内侧赋的值为 1 , 故正六边形内赋的值的总和至少为 $11 \\times(-1)+ 1=-10$.\n综合上述两方面, 得 $2 N-24 \\geqslant-10$, 解得 $N \\geqslant 7$. 即符合题目要求的小正三角形至少有 7 个.\n应用上述证题方法不难将上述问题作如下推广:将一个凸(或凹)的多边形完全剖分成三角形区域,要求每一个三角形的顶点不在另一个三角形的边的内部,假设一共剖分为 $n$ 个三角形, 其中有一边位于多边形边界位置的三角形有 $m$ 个 $(n-m$ 为正偶数). 如果在每--个三角形的顶点处任意写上一个互不相等的实数,证明其中至少有 $\\frac{1}{2}(n-m)$ +1 个三角形,其顶点处所写的 3 个实数可按顺时针方向排成递减数列.", + "remark": "", + "figures": [ + "./images/volume11/figures/fig-c9i10.png" + ] +} \ No newline at end of file diff --git a/processed_dataset/proof/0338.json b/processed_dataset/proof/0338.json new file mode 100644 index 0000000000000000000000000000000000000000..a136e6d555e8d31c9956b236ee3120e64f47c725 --- /dev/null +++ b/processed_dataset/proof/0338.json @@ -0,0 +1,8 @@ +{ + "source_file": "./raw_volume-zh/volume11/exercise10.tex", + "problem_type": "proof", + "problem": "问题1. 某个委员会有 $n(\\geqslant 5)$ 个成员, 并且有 $n+1$ 个三人委员会, 其中没有两个三人委员会有完全相同的成员.\n证明: 存在两个三人委员会恰好有一个成员相同.", + "solution": "用反证法.\n假设任何两个三人委员会或者有两个成员相同,或者没有成员相同.\n如果委员会 $A$ 和 $B$ 有公共成员, 那么它们恰有两个公共成员 $a$ 和 $b$. 如果委员会 $B$ 又和 $C$ 有两个公共成员, 那么 $a$ 和 $b$ 中至少有一个属于 $C$, 从而 $C$ 与 $A$ 也应有两个公共成员.\n于是, 我们可将所有三人委员会进行分类: 使同一类的两个委员会都恰有两个公共成员,不同类的委员会没有公共成员.\n下面证明: 同一类中委员会的个数 $k$ 不大于这类中委员的人数 $h$, 显然 $h \\geqslant 3$. 当 $h=3$ 时 $k=1$; 当 $h \\geqslant 4$ 时 $k \\geqslant 2$, 设 $\\{x, a, b\\},\\{y, a, b\\}$ 是其中两个委员会, 则其他同类的委员会只能是 $\\{x, y, a\\},\\{x, y, b\\}$ 或 $\\{a, b, z\\}$ 的形式, 这里 $z$ 最多有 $h-4$ 种选择.\n所以 $k \\leqslant 4+(h-4)=h$. 于是委员会的总数 $n+1 \\leqslant$ 总人数 $n$, 矛盾.\n这表明, 至少有两个委员会恰有一个公共成员.", + "remark": "", + "figures": [] +} \ No newline at end of file diff --git a/processed_dataset/proof/0339.json b/processed_dataset/proof/0339.json new file mode 100644 index 0000000000000000000000000000000000000000..5bb141823782779d0cb6e37ff86f3cd2852e3661 --- /dev/null +++ b/processed_dataset/proof/0339.json @@ -0,0 +1,8 @@ +{ + "source_file": "./raw_volume-zh/volume11/exercise10.tex", + "problem_type": "proof", + "problem": "问题3. 设 $n$ 为奇数且大于 1 , 又 $k_1, k_2, \\cdots, k_n$ 是给定的整数, 对 $1,2, \\cdots, n$ 的 $n !$ 个排列中每一个排列 $a=\\left(a_1, a_2, \\cdots, a_n\\right)$, 记 $S(a)==\\sum_{i=1}^n k_i a_i$. 证明: 存在两个排列 $b$ 和 $c, b \\neq c$, 使 $S(b)-S(c)$ 被 $n !$ 整除.", + "solution": "用反证法.\n假设对任意排列 $b$ 和 $c, b \\neq c$ 都有 $S(b)-S(c) \\not \\equiv O(\\bmod n !) \\cdots$ (1), \n于是由(1)知当 $a$ 遍历 $1,2, \\cdots, n$ 的 $n !$ 个排列时, $S(a)$ 遍历模 $n$ ! 的一个完全剩余系.\n记 $1,2, \\cdots, n$ 的所有排列集合为 $\\sigma$,于是 $\\sum_{a \\in \\sigma} S(a) \\equiv \\sum_{k=1}^{n !} k=\\frac{1}{2}(n !)(n !+1)(\\bmod n !)$. \n又 $n>1$ 为奇数, 故 $\\sum_{a \\in \\sigma} S(a) \\equiv \\frac{1}{2} n ! \\not \\equiv 0(\\bmod n !) \\cdots$ (2). \n另一方面 $\\sum_{a \\in \\sigma} S(a)=\\sum_{a \\in \\sigma} \\sum_{i=1}^n k_i a_i=\\sum_{i=1}^n k_i\\left(\\sum_{a \\in \\sigma} a_i\\right)= \\frac{(n !)(n+1)}{2} \\sum_{i=1}^n k_i \\equiv 0(\\bmod n !)$; 这与(2)矛盾.\n故命题结论得证.", + "remark": "", + "figures": [] +} \ No newline at end of file diff --git a/processed_dataset/proof/0340.json b/processed_dataset/proof/0340.json new file mode 100644 index 0000000000000000000000000000000000000000..1a58b8ff488a62068ffcf25b15e13a157f2da67e --- /dev/null +++ b/processed_dataset/proof/0340.json @@ -0,0 +1,10 @@ +{ + "source_file": "./raw_volume-zh/volume11/exercise10.tex", + "problem_type": "proof", + "problem": "问题4. 某次聚会共 17 人, 其中每个人都恰好认识另外 4 人, 求证: 存在两人, 他们彼此不认识且没有共同认识的人.", + "solution": "如图(), 用 17 个点 $A_1, A_2, \\cdots, A_{17}$ 表示 17 个人,若两人互相认识, 则对应点连一线段, 否则不连线段.\n若两人互相认识 (即对应点间连有线段) 或两人有共同的熟人 (即对应两点与同一点连有线段, 这时称这两点张有角) 则称这两人对应的两点是关联的.\n问题归结为存在不关联的两点.\n用反证法.\n假设图中所有的点都是两两关联的,即任意两点或连有线段或张有角.\n已知图中共连有 $\\frac{1}{2} \\times 17 \\times 4=34$ 条线段,所张的角有 $17 C_4^2=102$ 个, 且 $34+102= 136=\\mathrm{C}_{17}^2$ 恰等于两点组数.\n因此, 任意两点或连有线段或张有唯一一个角, 二者必居之一.\n可见图中既不存在三角形 (否则连有线段的两点又张有角) 也不存在四边形 (否则有两点张有两个角). 考察其中任意一点 $X$, 它与 4 点 $A 、 B 、 C 、 D$ 连有线段.\n由上述讨论知 $A 、 B 、 C 、 D$ 两两之间没有连线.\n从 $A$ 、 $B 、 C 、 D$ 出发, 除 $X$ 外各自都与 3 个点连有线段 (如图所示). 这样一共有 17 个点,连有 16 条线段.\n从 $A 、 B 、 C 、 D$ 所连出的 4 个 3 点组, 同组内任何两点不连线, 每点都只能与其他组内的点连有线段, 每连一条线段, 使得到一个含点 $X$ 的 5 点圈 (如图中 $X A A_2 B_1 B X$ 和 $X B B_3 C_3 C X$ 等). 由于还要连 $34- 16=18$ 条线, 故包含点 $X$ 的 5 点圈有 18 个, 由于 $X$ 是任意的, 因此, 每一点都包含在 18 个 5 点圈内, 故图中的 5 点圈应有 $\\frac{17 \\times 18}{5}$ 个, 但 $\\frac{17 \\times 18}{5}$ 不是整数, 矛盾! 因此, 图中的点不可能是两两关联的, 故一定存在不关联的两点, 即存在两人彼此不认识且没有共同认识的人.", + "remark": "", + "figures": [ + "./images/volume11/figures/fig-c10a4.png" + ] +} \ No newline at end of file diff --git a/processed_dataset/proof/0341.json b/processed_dataset/proof/0341.json new file mode 100644 index 0000000000000000000000000000000000000000..15238e272aa88b398ff3ed7cb279182d4df56c5e --- /dev/null +++ b/processed_dataset/proof/0341.json @@ -0,0 +1,8 @@ +{ + "source_file": "./raw_volume-zh/volume11/exercise10.tex", + "problem_type": "proof", + "problem": "问题5. 设 $n$ 为正整数, $M$ 是具有下列性质的 $n^2+1$ 个正整数构成的集合: $M$ 中任意 $n+1$ 个数中必有 2 个数, 使得其中一个数是另一个数的倍数, 证明: $M$ 中存在 $n+1$ 个数 $a_1, a_2, \\cdots, a_{n+1}$, 使得对任意 $i=1,2, \\cdots, n$, 都有 $a_i$ 整除 $a_{i+1}$.", + "solution": "对任意 $k$ 个正整数 $x_1, x_2, \\cdots, x_k$, 若对 $i=1,2, \\cdots, k-1$ 均有 $x_i$ 整除 $x_{i+1}$, 则称 $\\left(x_1, x_2, \\cdots, x_k\\right)$ 为一条长为 $k$ 的链, 且称 $x_1$ 为该链的首元.\n对 $M$ 中每个元 $x_i\\left(1 \\leqslant i \\leqslant n^2+1\\right)$, 考虑取自 $M$ 的以 $x_i$ 为首元的链中最长的链, 并记这个最长的链的链长为 $l_i\\left(i=1,2, \\cdots, n^2+1\\right)$. 下面我们只需证明: $l_1, l_2$, $\\cdots, l_{n^2+1}$ 中至少有一个数不小于 $n+1$. 若对任意 $1 \\leqslant i \\leqslant n^2+1$ 均有 $l_i \\leqslant n$, 则由抽庶原理 $l_1, l_2, \\cdots, l_n{ }^2+1$ 中至少有 $\\left[\\frac{\\left(n^2+1\\right)-1}{n}\\right]+1=n+1$ 个相等.\n不妨设 $l_1=l_2=\\cdots=l_{n+1}=r$, 而由 $M$ 的性质知 $x_1, x_2, \\cdots, x_{n+1}$ 中必有一个数是另一个数的倍数.\n不妨设 $x_1$ 整除 $x_2$. 于是将 $x_1$ 置于以 $x_2$ 为首元的那条最长的链的前面, 我们得到一条长为 $l_2+1=r+1$ 且以 $x_1$ 为首元的链, 这与以 $x_1$ 为首元的最长链的长为 $l_1=r$ 矛盾.\n故 $l_1, l_2, \\cdots, l_{n^2+1}$ 中必有一个数不小于 $n+1$. 不妨设 $l_1 \\geqslant n+1$, 即存在 $M$ 中 $n+1$ 个数 $a_1, a_2, \\cdots, a_{n+1}$ 使 $a_i$ 整除 $a_{i+1}(i=1,2, \\cdots, n)$.", + "remark": "", + "figures": [] +} \ No newline at end of file diff --git a/processed_dataset/proof/0342.json b/processed_dataset/proof/0342.json new file mode 100644 index 0000000000000000000000000000000000000000..db60569c61d22544ef092b676bcab2e8eff2dccd --- /dev/null +++ b/processed_dataset/proof/0342.json @@ -0,0 +1,8 @@ +{ + "source_file": "./raw_volume-zh/volume11/exercise10.tex", + "problem_type": "proof", + "problem": "问题6. 某地区网球俱乐部的 20 名成员进行了 14 场单打比赛, 每人至少上场 1 次.\n求证: 必有 6 场比赛, 上场的 12 名队员互不相同.", + "solution": "记参加第 $i$ 场比赛的选手为 $\\left(a_i, b_i\\right)(i=1,2, \\cdots, 14)$, 并记 $M= \\left\\{\\left(a_i, b_i\\right) \\mid i=1,2, \\cdots, 14\\right\\}$. 我们称 $M$ 的一个子集为好子集,如果该子集所含选手对中出现的所有选手互不相同.\n显然好子集是存在的(例如只含一对选手的子集) 且个数有限.\n故存在一个含选手对最多的好子集 $M_0$, 设 $M_0$ 中含有 $r$ 对选手, 只要证 $r \\geqslant 6$, 反设 $r \\leqslant 5$. 于是 $M_0$ 中没有出现过的 $20-2 r$ 名选手之间没有进行比赛 (因为若有一对选手 $a$ 和 $b$ 比了赛, 则 $M_0 \\cup\\{(a, b)\\}$ 仍然是好子集, 这与 $M_0$ 是含选手对最多的,假设矛盾). 这表明这 $20-2 r$ 名选手参加的比赛一定是同 $M_0$ 中的 $2 r$ 名选手进行的.\n又已知每名选手至少参加一场比赛, 故这 20-2r 名选手至少参加了 $20-2 r$ 场比赛, 加上 $M_0$ 中的 $r$ 场比赛, 故总的比赛场次至少为 $(20-2 r)+r=20-r \\geqslant 15$, 这与一共只有 14 场比赛矛盾, 所以 $r \\geqslant 6$. 即至少有 6 场比赛, 参赛的 12 名选手互不相同.", + "remark": "", + "figures": [] +} \ No newline at end of file diff --git a/processed_dataset/proof/0343.json b/processed_dataset/proof/0343.json new file mode 100644 index 0000000000000000000000000000000000000000..a2b3ef0b7561b5c10c0b88de5cc71d2e1091067a --- /dev/null +++ b/processed_dataset/proof/0343.json @@ -0,0 +1,8 @@ +{ + "source_file": "./raw_volume-zh/volume11/exercise11.tex", + "problem_type": "proof", + "problem": "问题1. 在线段 $A B$ 上关于它的中点 $M$ 对称地放置 $2 n$ 个点.\n任意将这 $2 n$ 个点中的 $n$ 个染成红点, 另 $n$ 个染成蓝点.\n证明: 所有红点到 $A$ 的距离之和等于所有蓝点到 $B$ 的距离之和.", + "solution": "记所有红 (蓝)点到 $A(B)$ 的距离之和为 $S_{\\text {红 }}\\left(S_{\\text {蓝 }}\\right.$ ). 首先考察一个极端情形: $n$ 个蓝点均在中点 $M$ 的左边, $n$ 个红点均在中点 $M$ 的右边.\n此时显然有个蓝点 $D$. 我们取左边任一红点 $C$ 改为蓝点, 右边任意蓝点 $D$ 改为红点, 其余点的颜色不变, 则涂红色的点到 $A$ 的距离的总和 $S_{\\text {红 }}{ }^{\\prime}=S_{\\text {红 }}+C D$, 涂蓝色的点到 $B$ 的距离的总和 $S_{\\text {蓝 }}{ }^{\\prime}=S_{\\text {蓝 }}+C D$, 从而 $S_{\\text {红 }}{ }^{\\prime}-S_{\\text {蓝 }}{ }^{\\prime}=S_{\\text {红 }}-S_{\\text {蓝是常量.\n}}$. 于是, 经过有限次调整, 可将 $2 n$ 个点调整到前述极端情形, 因而结论成立.", + "remark": "", + "figures": [] +} \ No newline at end of file diff --git a/processed_dataset/proof/0344.json b/processed_dataset/proof/0344.json new file mode 100644 index 0000000000000000000000000000000000000000..ef44bca97e2e246388cf8418bc7778ffa4279906 --- /dev/null +++ b/processed_dataset/proof/0344.json @@ -0,0 +1,8 @@ +{ + "source_file": "./raw_volume-zh/volume11/exercise11.tex", + "problem_type": "proof", + "problem": "问题2. 空间有 1989 个点, 其中任何三点不共线, 把它们分成点数各不相同的 30 组,在任何三个不同的组中各取一点为顶点作三角形, 问要使三角形的总数最大,各组的点数应是多少?", + "solution": "设 30 组的点数为 $n_1, n_2, \\cdots, n_{30}$, 则 $n_1, n_2, \\cdots, n_{30}$ 各不相同, 且 $n_1+ n_2+\\cdots+n_{30}=1989$, 三角形总数为 $S=\\sum_{1 \\leqslant i0$, 即 $S^{\\prime}>S$. 这与 $S$ 的最大性假设矛盾; (2) 使 $n_{k+1}-n_k=2$ 的 $k$ 至多只有一个.\n事实上, 若存在 $k_1), 考虑一般情形: 设 $a_1, a_2, \\cdots, a_{2 n}$ 是 $1,2, \\cdots, 2 n$ 的任意排列, 求 $S_n= a_1 a_2+a_3 a_4+\\cdots+a_{2 n-1} a_{2 n}$ 的最大值.\n对 $n$ 用归纳法, $n=1$ 时, $S_1=1 \\times 2$, $n=2$ 时, $S_2 \\leqslant \\max \\{1 \\times 2+3 \\times 4,1 \\times 3+2 \\times 4,1 \\times 4+2 \\times 3\\}=1 \\times 2+ 3 \\times 4$. 设 $S_k \\leqslant 1 \\times 2+3 \\times 4+\\cdots+(2 k-1)(2 k)$, 那么 $n=k+1$ 时, 设 $S_{k+1}=a_1 a_2+a_3 a_4+\\cdots+a_{2 k+1} a_{2 k+2}$, 分两种情形: (1)若 $2 k+1,2 k+2$ 出现在 $S_{k+1}$ 中同一项中, 不妨设 $a_{2 k+1} a_{2 k+2}=(2 k+1)(2 k+2)$, 则由归纳假设有 $S_{k+1}=a_1 a_2+\\cdots+a_{2 k-1} a_{2 k}+(2 k+1)(2 k+2) \\leqslant 1 \\times 2+3 \\times 4+\\cdots+(2 k-$ 1) $(2 k)+(2 k+1)(2 k+2)$; (2) 若 $2 k+1$ 与 $2 k+2$ 出现在 $S_{k+1}$ 的两个不同的项中, 不妨设 $a_{2 k-1}=2 k+1, a_{2 k+1}=2 k+2$, 注意到 $\\left[a_{2 k} a_{2 k+2}+(2 k+1)(2 k+\\right. 2)]-\\left[(2 k+1) a_{2 k}+(2 k+2) a_{2 k+2}\\right]=\\left[a_{2 k}-(2 k+2)\\right]\\left[a_{2 k+2}-(2 k+1)\\right]>$ 0 , 所以 $S_{k+1}2 a_n$ 为奇数,\n$b_i(i=2,3, \\cdots, m-1)$ 为偶数且 $b_{i+1}>b_i \\cdot a_n(i=2,3, \\cdots, m-2)$. 记 $\\sum_{i=1}^{m-1} b_i^2=2 S+1$, 再取 $b_m=S$, 则 $b_1^2+b_2^2+\\cdots+b_m^2=(S+1)^2$. 不难验证, 上述填法符合题目要求.\n\\begin{tabular}{|c|c|c|c|c|c|c|}\n\\hline$a_1^2 b_1^2$ & $a_2^2 b_1^2$ & $\\cdots$ & $a_j^2 b_1^2$ & $\\cdots$ & $a_{n-1}^2 b_1^2$ & $a_n^2 b_1^2$ \\\\\n\\hline$a_1^2 b_2^2$ & $a_2^2 b_2^2$ & $\\cdots$ & $a_j^2 b_2^2$ & $\\cdots$ & $a_{n-1}^2 b_2^2$ & $a_n^2 b_2^2$ \\\\\n\\hline$\\cdots$ & $\\cdots$ & $\\cdots$ & $\\cdots$ & $\\cdots$ & $\\cdots$ & $\\cdots$ \\\\\n\\hline$a_1^2 b_i^2$ & $a_2^2 b_i^2$ & $\\cdots$ & $a_j^2 b_i^2$ & $\\cdots$ & $a_{n-1}^2 b_i^2$ & $a_n^2 b_i^2$ \\\\\n\\hline$\\cdots$ & $\\cdots$ & $\\cdots$ & $\\cdots$ & $\\cdots$ & $\\cdots$ & $\\cdots$ \\\\\n\\hline$a_1^2 b_{m-1}^2$ & $a_2^2 b_{m-1}^2$ & $\\cdots$ & $a_j^2 b_{m-1}^2$ & $\\cdots$ & $a_{n-1}^2 b_{m-1}^2$ & $a_n^2 b_{m-1}^2$ \\\\\n\\hline$a_1^2 b_m^2$ & $a_2^2 b_m^2$ & $\\cdots$ & $a_j^2 b_m^2$ & $\\cdots$ & $a_{n-1}^2 b_m^2$ & $a_n^2 b_m^2$ \\\\\n\\hline\n\\end{tabular}", + "remark": "", + "figures": [] +} \ No newline at end of file diff --git a/processed_dataset/proof/0349.json b/processed_dataset/proof/0349.json new file mode 100644 index 0000000000000000000000000000000000000000..eb042c596433345e02a2d3bd0921a5e95704a086 --- /dev/null +++ b/processed_dataset/proof/0349.json @@ -0,0 +1,8 @@ +{ + "source_file": "./raw_volume-zh/volume11/exercise12.tex", + "problem_type": "proof", + "problem": "问题2. 求证: 对任意正整数 $k$, 存在一个由有理数组成的等差数列 $\\frac{a_1}{b_1}, \\frac{a_2}{b_2}, \\cdots$, $\\frac{a_k}{b_k}$, 其中 $a_i$ 与 $b_i$ 是互素正整数 $(i=1,2, \\cdots, k)$, 且 $a_1, b_1, a_2, b_2, \\cdots$, $a_k, b_k$ 互不相同.", + "solution": "取 $a_i=\\frac{(k !)^2+i}{i}, b_i=\\frac{k !}{i}(i=1,2, \\cdots, k)$, 则 $\\frac{a_1}{b_1}=\\frac{(k !)^2+1}{k !}$, $\\frac{a_2}{b_2}=\\frac{(k !)^2+2}{k !}, \\cdots, \\frac{a_k}{b_k}=\\frac{(k !)^2+k}{k !}$ 是公差为 $\\frac{1}{k !}$ 的等差数列.\n因为 $\\left((k !)^2+\\right. i, k !)=i$, 故 $\\left(a_i, b_i\\right)=\\left(\\frac{(k !)^2+i}{i}, \\frac{k !}{i}\\right)=1(i=1,2, \\cdots, k)$. 又 $a_i> b_i(i=1,2, \\cdots, k)$ 且对任意 $1 \\leqslant ia_j=\\frac{(k !)^2+j}{j} >b_i=\\frac{k !}{i}>b_j=\\frac{k !}{j}$, 此时, 数列 $\\frac{a_1}{b_1}, \\frac{a_2}{b_2}, \\cdots, \\frac{a_k}{b_k}$ 满足题中条件.", + "remark": "", + "figures": [] +} \ No newline at end of file diff --git a/processed_dataset/proof/0350.json b/processed_dataset/proof/0350.json new file mode 100644 index 0000000000000000000000000000000000000000..c17c6ed75512dec452fe4601626d4153eab4c54c --- /dev/null +++ b/processed_dataset/proof/0350.json @@ -0,0 +1,8 @@ +{ + "source_file": "./raw_volume-zh/volume11/exercise12.tex", + "problem_type": "proof", + "problem": "问题5. 证明: 全体正整数可分成 100 个非空子集, 使得任何 3 个满足关系式 $a+ 99 b=c$ 的正整数 $a 、 b 、 c$, 都可以从中找到两个数属于同一个子集.", + "solution": "构造 100 个子集如下:\n$A_i=\\left\\{k \\mid k \\equiv i-1(\\bmod 99)\\right.$ 且 $k \\in \\mathbf{N}_{+}$为偶数 $\\}, i=1,2, \\cdots, 99$.\n$A_{100}=\\left\\{k ! k \\in \\mathbf{N}_{+}\\right.$为奇数 $\\}$.\n注意到当正整数 $a 、 b 、 c$ 满足 $a+99 b=c$ 时, $a 、 b 、 c$ 中奇数的个数或为 0 或为 2 且 $a \\equiv c(\\bmod 99)$. 若 $a 、 b 、 c$ 中有 2 个数为奇数时, 那么它们都属于 $A_{100}$ 结论成立.\n若 $a 、 b 、 c$ 中奇数个数为 0 , 即 $a 、 b 、 c$ 全为偶数时, 又因为 $a \\equiv c(\\bmod 99)$, 故 $a$ 和 $c$ 属于同一子集 $A_{i_0}\\left(1 \\leqslant i_0 \\leqslant 99\\right)$.", + "remark": "", + "figures": [] +} \ No newline at end of file diff --git a/processed_dataset/proof/0351.json b/processed_dataset/proof/0351.json new file mode 100644 index 0000000000000000000000000000000000000000..04981b74eb45f88aab8ea5ac42f3931562d8c573 --- /dev/null +++ b/processed_dataset/proof/0351.json @@ -0,0 +1,8 @@ +{ + "source_file": "./raw_volume-zh/volume11/exercise12.tex", + "problem_type": "proof", + "problem": "问题6. 证明: 存在无穷多个正整数 $n$, 使得集合 $S_n=\\{1,2, \\cdots, 3 n\\}$ 可划分为三个不相交的集合: $A=\\left\\{a_1, a_2, \\cdots, a_n\\right\\}, B=\\left\\{b_1, b_2, \\cdots, b_n\\right\\}, C= \\left\\{c_1, c_2, \\cdots, c_n\\right\\}$ 满足对任意 $i=1,2, \\cdots, n$ 都有 $a_i+b_i=c_i$.", + "solution": "(证明一) $n=1$ 时,取 $A=\\{1\\}, B=\\{2\\}, C=\\{3\\}$ 知结论成立.\n设对 $n \\in \\mathbf{N}_{+}$, 结论成立.\n即 $S_n=\\{1,2, \\cdots, 3 n\\}$ 可剖分为三个不相交的子集: $A= \\left\\{a_1, a_2, \\cdots, a_n\\right\\}, B=\\left\\{b_1, b_2, \\cdots, b_n\\right\\}, C=\\left\\{c_1, c_2, \\cdots, c_n\\right\\}$ 满足 $a_i+b_i= c_i(i=1,2, \\cdots, n)$. 那么对 $4 n \\in \\mathbf{N}_{+}$, 将集合 $S_{4 n}=\\{1,2,3, \\cdots, 12 n\\}$ 剖分为三个集合: $A^{\\prime}=\\left\\{a_1{ }^{\\prime}, a_2{ }^{\\prime}, \\cdots, a_{4 n}{ }^{\\prime}\\right\\}, B^{\\prime}=\\left\\{b_1{ }^{\\prime}, b_2{ }^{\\prime}, \\cdots, b_{4 n}{ }^{\\prime}\\right\\}, C^{\\prime}=\\left\\{c_1{ }^{\\prime}\\right.$, $\\left.c_2{ }^{\\prime}, \\cdots, c_{4 n}{ }^{\\prime}\\right\\}$. 其中 $a_i{ }^{\\prime}=2 i-1(i=1,2, \\cdots, 3 n), a_{3 n+k}{ }^{\\prime}=2 a_k(k=1,2$, $\\cdots, n), b_i{ }^{\\prime}=9 n+1-i(i=1,2, \\cdots, 3 n), b_{3 n+k}{ }^{\\prime}=2 b_k(k=1,2, \\cdots, n)$, $c_i{ }^{\\prime}=9 n+i(i=1,2, \\cdots, 3 n), c_{3 n+k}{ }^{\\prime}=2 c_k(k=1,2, \\cdots, n)$, 则对任意 $i(1 \\leqslant i \\leqslant 4 n)$ 都有 $a_i{ }^{\\prime}+b_i{ }^{\\prime}=c_i{ }^{\\prime}$. 因此, 存在无穷多个 $n=1,4,4^2, 4^3, \\cdots$ 满足题目要求.\n%%%%\n(证明二) $n=1$ 时结论成立.\n设对 $n \\in \\mathbf{N}_{+}$, 结论成立.\n即 $S_n=\\{1,2,3$ , $\\cdots, 3 n\\}$ 可剖分为不相交的三个子集 $A=\\left\\{a_1, a_2, \\cdots, a_n\\right\\}, B=\\left\\{b_1, b_2, \\cdots\\right.$,\n$\\left.b_n\\right\\}, C=\\left\\{c_1, c_2, \\cdots, c_n\\right\\}$, 使 $a_i+b_i=c_i(i=1,2, \\cdots, n)$. 于是对 $3 n+1$, $S_{3 n+1}=\\{1,2, \\cdots, 3(3 n+1)\\}$ 可剖分为三个不相交的子集 $A^{\\prime}=\\left\\{a_1{ }^{\\prime}, a_2{ }^{\\prime}\\right.$, $\\left.\\cdots, a_{3 n+1}{ }^{\\prime}\\right\\}, B^{\\prime}=\\left\\{b_1{ }^{\\prime}, b_2{ }^{\\prime}, \\cdots, b_{3 n+1}{ }^{\\prime}\\right\\}, C^{\\prime}=\\left\\{c_1{ }^{\\prime}, c_2{ }^{\\prime}, \\cdots, c_{3 n+1}{ }^{\\prime}\\right\\}$, 其中 $a_i{ }^{\\prime}=3 a_i-1, b_i{ }^{\\prime}=3 b_i, c_i{ }^{\\prime}=3 c_i-1(i=1,2, \\cdots, n), a_{n+i}{ }^{\\prime}=3 a_i, b_{n+i}{ }^{\\prime}= 3 b_i+1, c_{n+i}{ }^{\\prime}=3 c_i+1(i=1,2, \\cdots, n), a_{2 n+i}{ }^{\\prime}=3 a_i+1, b_{2 n+i}{ }^{\\prime}=3 b_i-1$, $c_{2 n+i}{ }^{\\prime}=3 c_i(i=1,2, \\cdots, n), a_{3 n+1}{ }^{\\prime}=1, b_{3 n+1}{ }^{\\prime}=9 n+2, c_{3 n+1}{ }^{\\prime}=9 n+3$. 则 $a_i{ }^{\\prime}+b_i{ }^{\\prime}=c_i{ }^{\\prime}(i=1,2, \\cdots, 3 n+1)$, 这就证明了存在无穷多个 $n=1,4,13$, $40,121, \\cdots$, 即 $n=\\frac{1}{2}\\left(3^k-1\\right)(k=1,2, \\cdots)$ 满足题目要求.", + "remark": "", + "figures": [] +} \ No newline at end of file diff --git a/processed_dataset/proof/0352.json b/processed_dataset/proof/0352.json new file mode 100644 index 0000000000000000000000000000000000000000..eaedc174defb4428c9fc331be44ac0e002b5baec --- /dev/null +++ b/processed_dataset/proof/0352.json @@ -0,0 +1,8 @@ +{ + "source_file": "./raw_volume-zh/volume11/exercise12.tex", + "problem_type": "proof", + "problem": "问题7. 证明: 除了有限个正整数外,其他任何正整数 $n$ 都可表示成为 2004 个不同的正整数之和: $n=a_1+a_2+\\cdots+a_{2004}$ 满足 $1 \\leqslant a_12 t_k+1$, 则 $t>t_k$, 由归纳假设 $t$ 可表成 $k$ 个不同正整数之和: $t=b_1+b_2+\\cdots+b_k$ 满足 $1 \\leqslant b_1), 以 $4 \\times 4 \\times 4$ 大正方体的底面为基准面, 将大正方体划分为平行基准面的 4 层 (每层为 $4 \\times 4 \\times 1$ 的长方体) 依次用 $1,2,3,4$ 给各层编号.\n将各红色正方体投影到基准面上, 并在基准面的投影方格内填上该红色正方体所在的层号, 这样得到一个 $4 \\times 4$ 的方格表.\n依题意, 每格内恰填 $\\{1,2,3,4\\}$ 中一个数, 并且填同样数的方格既不在同一行又不在同一列, 这样的 $4 \\times 4$ 的方格表被称为一个 4 阶拉丁方, 反之每一个 4 阶拉丁方唯一决定了一种符合条件的涂色法.\n因此, 题目转化为确定: 总共有多少个不同的 4 阶拉丁方? 用 $a_{i j}(i$, $j==1,2,3,4)$ 表示 $4 \\times 4$ 方格表内第 $i$ 行第 $j$ 列处小方格内填人的数.\n对 ( 1 , $2,3,4)$ 的任意排列 $(a, b, c, d)$, 先考虑 $a_{11}=a, a_{12}=a_{21}=b, a_{13}=a_{31}= c, a_{14}=a_{41}=d$ (如图)时拉丁方的个数.\n(1) $a_{22}=a$ 时, $a_{23}$ 与 $a_{32}$ 只可能为 $d, a_{24}$ 与 $a_{42}$ 只可能为 $c$. 然后 $a_{33}$ 可以等于 $a$ 或 $b$, 当 $a_{33}$ 选定后, $a_{34}, a_{43}, a_{44}$ 的值是唯一的.\n因此, 这种情形下对应的拉丁方恰有 2 个; (2) $a_{22}=c$ 时, 只有 $a_{24}= a_{42}=a, a_{23}=a_{32}=d, a_{34}=a_{43}=b, a_{44}=c, a_{33}=a$, 这种情形下对应的拉丁方只有 1 个; (3) $a_{22}=d$ 时同(2)可证这时拉丁方只有 1 个, 故如图所示的拉丁方恰有 4 个.\n因 $(a, b, c, d)$ 这种排列共 $4 !=24$ 个, 当 $(a, b, c, d)$ 确定后, 如图所示拉丁方通过交换第 $2,3,4$ 行一共可形成 3 ! 个不同的拉丁方.\n所以 4 阶拉丁方的总数为 4 ! $\\cdot 3$ ! ・ $4=576$ 个.\n故题目所述 16 个红色正方体的不同取法共有 576 种.", + "remark": "注:设 $n$ 阶拉丁方的个数为 $L_n$, 则 $L_n=n !((n-1) !) l_n$, 现已有的结论为 $l_1=l_2=l_3=1, l_4=4, l_5=56, l_6=9408, l_7=16942080, \\cdots$. 当 $n \\geqslant 5$ 时, 要分很多情形才能算出 $l_n$, 不可能在短时间内做到.\n因此, 作为试题选择 $n=4$ 是恰当的.", + "figures": [ + "./images/volume11/figures/fig-c13a6.png" + ] +} \ No newline at end of file diff --git a/processed_dataset/proof/0355.json b/processed_dataset/proof/0355.json new file mode 100644 index 0000000000000000000000000000000000000000..ca101ca3dff25c879872f5f0bc80a49ed72df925 --- /dev/null +++ b/processed_dataset/proof/0355.json @@ -0,0 +1,8 @@ +{ + "source_file": "./raw_volume-zh/volume11/exercise14.tex", + "problem_type": "proof", + "problem": "问题1. 求一切正整数 $n \\geqslant 5$, 使存在一种染色方法至多用 6 种颜色给 $n$ 边形的顶点染色 (每个顶点染一种颜色), 满足任意连续 5 个顶点互不同色.", + "solution": "设 6 种颜色是 $a, b, c, d, e, f$, 并记 $S_1=(a, b, c, d, e), S_2= (a, b, c, d, e, f)$ 为两个序列.\n若 $n$ 可表示成为 $5 x+6 y(x, y$ 为非负整数) 的形式, 且 $n \\geqslant 5$, 则我们将 $n$ 边形的 $n$ 个顶点按 $x$ 个 $S_1$ 序列, $y$ 个 $S_2$ 序列着色, 就满足了题目要求.\n令 $y=0,1,2,3,4$ 就得到 $n$ 可以等于形如 $5 x, 5 x+ 6,5 x+12,5 x+18,5 x+24$ ( $x$ 为非负整数) 的正整数.\n而大于 4 不具有上述形式的数只有 $7,8,9,13,14$ 和 19 等 6 个数.\n下面我们证明这 6 个数都不满足要求.\n假设 $n$ 边形存在满足题目要求的染色方法, 那么存在正整数 $k$ 使 $6 k500$, 导致与已知条件矛盾.\n若 $a \\frac{334+668-1}{2}=500$, 也导致与已知条件矛盾,故题中结论成立.", + "remark": "", + "figures": [] +} \ No newline at end of file diff --git a/processed_dataset/proof/0358.json b/processed_dataset/proof/0358.json new file mode 100644 index 0000000000000000000000000000000000000000..e0c29f8f0d6cf24db22ef92f9c25e1562d66cfca --- /dev/null +++ b/processed_dataset/proof/0358.json @@ -0,0 +1,8 @@ +{ + "source_file": "./raw_volume-zh/volume11/exercise14.tex", + "problem_type": "proof", + "problem": "问题4. 在一所学校中有 $n$ 名男生和 $n$ 名女生 $(n>2000)$. 规定每名学生参加社团的数目不能超过 100 个.\n已知任意两名异性学生至少参加了一个共同的社团, 证明: 存在一个社团至少有 11 名男生和 11 名女生.", + "solution": "假设每个社团要么最多有 10 名男生, 要么最多有 10 名女生.\n下面用算二次方法导出矛盾.\n设 $m$ 是形如 $(b, g, c)$ 的三元组的数目, 其中 $b$ 表示一名男生, $g$ 表示一名女生, $c$ 表示一个他们都参加的社团.\n由于任意两名异性学生至少参加了一个共同的社团, 故 $m \\geqslant n^2$. 另一方面, 设 $X$ 是最多有 10 名男生的社团的集合, $Y$ 是最少有 11 名男生的社团的集合(这样的社团最多有 10 名女生). 于是, 对 $c \\in X$, 三元组 $(b, g, c)$ 的数目不超过 $n \\times 10 \\times 100= 1000 n$, 其中, 对于 $b$ 最多有 10 种选择, 对于 $g$ 有 $n$ 种选择, 对于 $c$ 最多有 100 种选择 (每名学生最多参加 100 个社团). 用同样的方法可得对于 $c \\in Y$, 三元组 $(b, g, c)$ 至多有 $1000 n$ 个, 故 $m \\leqslant 1000 n+1000 n=2000 n$, 从而 $n^2 \\leqslant 2000 n$, $n \\leqslant 2000$, 这与已知条件 $n>2000$ 矛盾, 这就证明了题中结论成立.", + "remark": "", + "figures": [] +} \ No newline at end of file diff --git a/processed_dataset/proof/0359.json b/processed_dataset/proof/0359.json new file mode 100644 index 0000000000000000000000000000000000000000..802a6862aeaeb67de20e66db3b179aef2267578e --- /dev/null +++ b/processed_dataset/proof/0359.json @@ -0,0 +1,8 @@ +{ + "source_file": "./raw_volume-zh/volume11/exercise14.tex", + "problem_type": "proof", + "problem": "问题5. 一次高难度数学竞赛试题由初试、复试两部分组成,共 28 个题目,每名竞赛者恰好解出其中 7 道题目, 每对试题恰有两人解出.\n证明: 必有一名参赛者,他至少解出了 4 道初试题或没有解出初试题.", + "solution": "设共有 $n$ 名参赛者, $m$ 道初试题, 将每个人与他解出的二道题组成一个 \"三元组\", 这种三元组的集合为 $S$, 则由已知条件可得 $|S|=n \\cdot \\mathrm{C}_7^2=2$ ・ $\\mathrm{C}_{28}^2$, 于是 $n=36$. \n其次任取一道题目 $A$, 假设它被 $r$ 个人 $a_1, a_2, \\cdots, a_r$ 解出这 $r$ 个人每人还解出了其他 6 道试题, 于是 $S$ 中包含 $A$ 的三元组有 $6 r$ 个.\n另一方面将 $A$ 与其他 27 题中每个题配对, 每对题恰有 2 人解出 (因这两人解出了题目 $A$,故他们必是 $a_1, a_2, \\cdots, a_r$ 中两人) 从而可形成 $2 \\times 27$ 个含 $A$ 的三元组, 所以 $6 r=2 \\times 27$, 即 $r=9$, 也就是说每道题恰有 9 人解出.\n如果结论不成立, 那么, 每人解出的初试题目只可能为 $1,2,3$. 设解出 $1,2,3$ 道初试题的人数分别为 $x, y, z$, 则 $x+y+z=36 \\cdots$ (1). \n将每个人与他解出的一道初试题配对, 这种对子个数为 $x+2 y+3 z$, 又为 $9 m$ (因每道题恰有 9 人解出), 所以 $x+2 y+3 z=9 m \\cdots$ (2). \n又通过计算 $S$ 中恰含 2 道初试题的 \"三元组\"可得 $\\mathrm{C}_2^2 y+ \\mathrm{C}_3^2 z=2 \\mathrm{C}_m^2$, 即 $y+3 z=m(m-1)$ (3). \n由(1), (2), (3)可解出 $x=m^2-19 m+$ 108, $y=-2 m^2+29 m-108, z=m^2-10 m+36$, 其中 $y=-2\\left(m-\\frac{29}{4}\\right)^2- \\frac{23}{8}<0$, 这与 $y$ 为非负整数矛盾.\n于是题目结论成立.", + "remark": "", + "figures": [] +} \ No newline at end of file diff --git a/processed_dataset/proof/0360.json b/processed_dataset/proof/0360.json new file mode 100644 index 0000000000000000000000000000000000000000..a7abcf786803b2f9e732323f9faa11de0ed8efce --- /dev/null +++ b/processed_dataset/proof/0360.json @@ -0,0 +1,8 @@ +{ + "source_file": "./raw_volume-zh/volume11/exercise14.tex", + "problem_type": "proof", + "problem": "问题6. 设空间给定 $n$ 个点,其中任何 4 点不共面, 它们之间连有 $q$ 条线段,则这些线段至少构成 $\\frac{4 q}{3 n}\\left(q-\\frac{n^2}{4}\\right)$ 个不同的三角形.", + "solution": "设图中 $n$ 个点为 $A_1, A_2, \\cdots, A_n$, 若 $A_i$ 与 $A_j$ 连有线段, 则将该线段染红色, 若 $A_i$ 与 $A_j$ 没有连线, 则将 $A_i$ 与 $A_j$ 连一条蓝色线段, 得到一个 2 色完全图 $K_n$. 由已知条件知图 $K_n$ 有 $q$ 条红边.\n要证明图中至少有 $\\frac{4 q}{3 n}\\left(q-\\frac{n^2}{4}\\right)$ 个红色三角形 (三边为红色的三角形). \n设从 $A_i$ 出发有 $d_i$ 条红线, $n-1-d_i$ 条蓝线 $(i=1,2, \\cdots, n)$ 于是 $\\sum_{i=1}^n d_i=2 q \\cdots$ (1). \n设图中有 $\\alpha$ 个红色三角形, $\\beta$ 个两边红一边蓝的三角形, $\\gamma$ 个二边蓝一边红的三角形, 并称从一点出发的两条红线组成的角叫做红色角, 从一点出发的一条红线和一条蓝线组成的角叫做异色角, 于是红色角的个数为 $3 \\alpha+\\beta=\\sum_{i=1}^n \\mathrm{C}_{d_i}^2 \\cdots$ (2), \n异色角的个数为 $2(\\beta+\\gamma)=\\sum_{i=1}^n d_i\\left(n-1-d_i\\right) \\cdots$ (3). \n由(3)得 $\\beta \\leqslant \\beta+\\gamma=\\frac{1}{2} \\sum_{i=1}^n d_i(n- 1-d_i$ ), \n代入 (2) 并利用柯西不等式及 (1) 得 $\\alpha=\\frac{1}{3}\\left[\\sum_{i=1}^n \\mathrm{C}_{d_i}^2-\\beta\\right] \\geqslant \\frac{1}{3}\\left[\\sum_{i=1}^n \\frac{1}{2} d_i\\left(d_i-1\\right)-\\frac{1}{2} \\sum_{i=1}^n d_i\\left(n-1-d_i\\right)\\right]=\\frac{1}{3}\\left(\\sum_{i=1}^n d_i^2-\\frac{n}{2} \\sum_{i=1}^n d_i\\right) \\geqslant \\frac{1}{3}\\left[\\frac{1}{n}\\left(\\sum_{i=1}^n d_i\\right)^2-\\frac{n}{2} \\sum_{i=1}^n d_i\\right]=\\frac{1}{3}\\left[\\frac{1}{n}(2 q)^2-\\frac{n}{2}(2 q)\\right]=\\frac{4 q}{3 n}\\left(q-\\frac{n^2}{4}\\right)$, 于是原命题得证.", + "remark": "", + "figures": [] +} \ No newline at end of file diff --git a/processed_dataset/proof/0361.json b/processed_dataset/proof/0361.json new file mode 100644 index 0000000000000000000000000000000000000000..0f2282c67cb3d94e85bc880b92d25632244bb891 --- /dev/null +++ b/processed_dataset/proof/0361.json @@ -0,0 +1,8 @@ +{ + "source_file": "./raw_volume-zh/volume11/exercise14.tex", + "problem_type": "proof", + "problem": "问题7. 由 $n$ 个点和这些点之间的 $l$ 条线组成一个空间图形,其中 $n=q^2+q+1$, $l \\geqslant \\frac{1}{2} q(q+1)^2+1, q \\geqslant 2, q \\in \\mathbf{N}_{+}$. 已知图形中任意四点不共面, 每点至少连一条线段,且存在一点至少连有 $q+2$ 条线段.\n证明: 图中必存在一个空间四边形 (即由四点 $A 、 B 、 C 、 D$ 和四条线段 $A B 、 B C 、 C D 、 D A$ 组成的图形).", + "solution": "设 $n$ 个点的集合为 $V=\\left\\{A_0, A_1, \\cdots, A_{n-1}\\right\\}$, 记 $A_i$ 的所有邻点(即与 $A_i$ 连有线段的点) 的集合为 $B_i, B_i$ 中点的个数为 $\\left|B_i\\right|=b_i$. 鼠然 $\\sum_{i=0}^{n-1} b_i=2 l$, 且 $b_i \\leqslant n-1(i=0,1,2, \\cdots, n-1)$. \n(1) 若存在 $b_i=n-1$, 不妨设 $b_0= n-1$, 于是 $B_0$ 中 $n-1$ 个点之间的连线数 $l-b_0 \\geqslant \\frac{1}{2} q(q+1)^2+1-(n-1)=\\frac{1}{2}(q+1)(n-1)+1-(n-1) \\geqslant \\frac{3}{2}(n-1)+1-(n-1)=\\frac{n-1}{2}+1 \\geqslant\\left[\\frac{n-1}{2}\\right]+1$. 故 $B_0$ 中必存在一点 $A_i$ 与 $B_0$ 中另两点 $A_j$ 与 $A_k$ 都连有线段.\n于是存在四边形 $A_0 A_j A_i A_k$, 结论成立.\n因此, 下面是讨论 $b_i \\leqslant n-2(i= 0,1,2, \\cdots, n-1)$ 的情形.\n(2) 不妨设 $q+2 \\leqslant b_0 \\leqslant n-2$ 及 $b_i \\leqslant n- 2(i=1,2, \\cdots, n-1)$. 若图中不存在四边形, 则当 $i \\neq j$ 时, $\\left|B_i \\cap B_j\\right| \\leqslant 1(0 \\leqslant i0 \\cdots \\label{eq3}$ . \n因 $\\left(n-b_0\\right)(q+1),\\left(n-b_0-1\\right) q$ 皆为正数.\n因式\\ref{eq2},\\ref{eq3}联合起来与式\\ref{eq1}矛盾.\n故原命题成立.", + "remark": "", + "figures": [] +} \ No newline at end of file diff --git a/processed_dataset/proof/0362.json b/processed_dataset/proof/0362.json new file mode 100644 index 0000000000000000000000000000000000000000..84a8078e02eafe82b511033060c3454ccab4e291 --- /dev/null +++ b/processed_dataset/proof/0362.json @@ -0,0 +1,8 @@ +{ + "source_file": "./raw_volume-zh/volume11/exercise14.tex", + "problem_type": "proof", + "problem": "问题8. 凸 $n$ 边形 $p$ 中每条边和每条对角线被染为 $n$ 种颜色中的一种.\n问: 对怎样的 $n$, 存在一种染色方式, 使得对于这 $n$ 种颜色中的任何三种不同颜色, 都能找到一个三角形,其顶点为多边形 $p$ 的顶点,且它的三边被染为这三种颜色?", + "solution": "当 $n(\\geqslant 3)$ 为奇数时, 存在符合要求的染法; 当 $n$ 为偶数时,不存在所述的染法.\n因为每 3 个顶点确定一个三角形, 一共确定 $\\mathrm{C}_n^3$ 个三角形, 而 $n$ 种颜色的三三搭配也刚好有 $\\mathrm{C}_n^3$ 种.\n所以,本题只要证明不同的三角形对应于不同的颜色组合, 即形成一一对应.\n以下将多边形的边和对角线都称为线段.\n对于每一种颜色, 其他 $n-1$ 种颜色形成 $\\mathrm{C}_{n-1}^2$ 种不同的搭配, 每种颜色的线段 (边或对角线) 都应出现在 $\\mathrm{C}_{n-1}^2$ 个三角形中, 而每一条线段都是 $n-2$ 个三角形的公共边, 因此, 在满足要求的染法中, 每种颜色的线段都应有 $\\frac{C_{n-1}^2}{n-2}=\\frac{n-1}{2}$ (条).\n当 $n$ 为偶数时, $\\frac{n-1}{2}$ 不是整数,因此不可能存在满足条件的染法.\n下面设 $n=2 m+1$ 为奇数, 我们给出一种染法, 并证明它满足题中条件.\n将凸 $n$ 边形的顶点依次记为 $A_1, A_2, \\cdots, A_{2 m+1}$, 对于整数 $i \\notin\\{1,2, \\cdots, 2 m+1\\}$, 若 $i \\equiv j(\\bmod 2 m+1)$ 且 $j \\in\\{1,2, \\cdots, 2 m+1\\}$, 则认为 $A_i$ 就是 $A_j$. 再将 $2 m+1$ 种颜色分别记为颜色 $1,2, \\cdots, 2 m+1$. \n现将边 $A_i A_{i+1}$ 染颜色 $i (i=1,2,3, \\cdots, 2 m+1)$, 再对每个 $i$, 将对角线 $A_{i-k} A_{i+1+k}(k=1,2, \\cdots$, $m-1$ ) 也染颜色 $i$, 于是每种颜色的线段 (边或对角线) 都恰有 $m$ 条.\n值得注意的是: 在规定的染色方法下, 当且仅当 $i_1+j_1 \\equiv i_2+j_2(\\bmod 2 m+$ 1) $\\cdots \\label{eq1}$ 时, 线段 $A_{i_1} A_{j_1}$ 与 $A_{i_2} A_{j_2}$ 同色.\n([注] 如果 $n$ 为奇数且凸 $n$ 边形为正 $n$ 边形, 那么当且仅当 $A_{i_1} A_{j_1} / / A_{i_2} A_{j_2}$ 时, $A_{i_1} A_{j_1}$ 与 $A_{i_2} A_{j_2}$ 同色.\n) 因此, 对任何 $i \\not j j(\\bmod 2 m+1), k \\not \\equiv 0(\\bmod 2 m+1)$, 线段 $A_i A_j$ 不与 $A_{i+k} A_{j+k}$ 同色, 即如果 $i_1-j_1 \\equiv i_2-j_2(\\bmod 2 m+1) \\cdots \\label{eq2}$ , \n那么线段 $A_{i_1} A_{j_1}$ 必不与 $A_{i_2} A_{j_2}$ 同色.\n任取两个三角形: $\\triangle A_{i_1} A_{j_1} A_{k_1}$ 和 $\\triangle A_{i_2} A_{j_2} A_{k_2}$, 如果它们中至多只有一条对应线段同色, 当然它们不会含有相同的颜色组合.\n如果它们有 2 条对应线段同色,我们证明: 它们的第 3 条线段必不同色.\n为确定起见, 不妨设 $A_{i_1} A_{j_1}$ 与 $A_{i_2} A_{j_2}$ 同色.\n下面分两种情形讨论: \n(1) 若 $A_{j_1} A_{k_1}$ 与 $A_{j_2} A_{k_2}$ 同色, 则由式 \\ref{eq1} 知 $i_1+j_1 \\equiv i_2+j_2(\\bmod 2 m+1), j_1+k_1 \\equiv j_2+k_2 (\\bmod 2 m+1)$, 两式相减得 $i_1-k_1 \\equiv i_2-k_2(\\bmod 2 m+1)$, \n故由式 \\ref{eq2} 知 $A_{k_1} A_{i_1}$ 与 $A_{k_2} A_{i_2}$ 不同色; \n(2) 若 $A_{i_1} A_{k_1}$ 与 $A_{i_2} A_{k_2}$ 同色, 则由式 \\ref{eq1} 得 $i_1+j_1 \\equiv i_2+j_2(\\bmod 2 m+1), i_1+k_1 \\equiv i_2+k_2(\\bmod 2 m+1)$, 两式相减得 $j_1-k_1 \\equiv j_2-k_2(\\bmod 2 m+1)$, \n故由式 \\ref{eq2} 知 $A_{j_1} A_{k_1}$ 与 $A_{j_2} A_{k_2}$ 不同色.\n总之, $\\triangle A_{i_1} A_{j_1} A_{k_1}$ 与 $\\triangle A_{i_2} A_{j_2} A_{k_2}$ 对应不同的颜色组合.\n这就证明了当且仅当 $n$ 为奇数时, 存在满足题目要求的染色方法.", + "remark": "", + "figures": [] +} \ No newline at end of file diff --git a/processed_dataset/proof/0363.json b/processed_dataset/proof/0363.json new file mode 100644 index 0000000000000000000000000000000000000000..60191dbc125f1f720bfd83ccfd686240a3ddb3be --- /dev/null +++ b/processed_dataset/proof/0363.json @@ -0,0 +1,8 @@ +{ + "source_file": "./raw_volume-zh/volume11/exercise15.tex", + "problem_type": "proof", + "problem": "问题2. 设 $M$ 是有限数集, 若已知 $M$ 的任何三个元素中总存在两个数, 它们的和属于 $M$,试问 $M$ 中最多有多少个数?", + "solution": "首先易验证 $M=\\{-3,-2,-1,0,1,2,3\\}$ 中任取 3 个数, 其中必有两个数之和属于 $M$. 另一方面, 设 $M=\\left\\{a_1, a_2, \\cdots, a_m\\right\\}, a_1>a_2>\\cdots> a_m$, 且 $m \\geqslant 8$. 因为每个数乘 -1 不会改变 $M$ 是否满足题目条件的性质, 故可设 $a_4>0$. 于是 $a_1+a_2>a_1+a_3>a_1+a_4>a_1$, 从而 $a_1+a_2, a_1+a_3, a_1+a_4$ 都不属于 $M$, 并且 $a_2+a_3$ 与 $a_2+a_4$ 不可能都属于 $M$ (因 $a_2+a_3>a_2, a_2+ a_4>a_2$ 且 $a_2+a_3 \\neq a_2+a_4$, 而 $M$ 中只有一个数 $\\left.a_1>a_2\\right)$, 这样 $\\left(a_1, a_2, a_3\\right)$ 或 $\\left(a_1, a_2, a_4\\right)$ 至少有一组中任何两个数之和不属于 $M$. 即 $m \\geqslant 8$ 时, $M$ 不满足题目要求.\n综上可知 $M$ 中最多有 7 个数.", + "remark": "", + "figures": [] +} \ No newline at end of file diff --git a/processed_dataset/proof/0364.json b/processed_dataset/proof/0364.json new file mode 100644 index 0000000000000000000000000000000000000000..79e2a69c775099392c0677b12a48d65b9cd22c83 --- /dev/null +++ b/processed_dataset/proof/0364.json @@ -0,0 +1,8 @@ +{ + "source_file": "./raw_volume-zh/volume11/exercise15.tex", + "problem_type": "proof", + "problem": "问题6. 设 $Z$ 是一个 56 元集合,求最小正整数 $n$,使得 $Z$ 的任意 15 个子集, 只要他们中任何 7 个的并的元素个数都不小于 $n$, 则这 15 个子集中一定存在 3 个,它们的交非空.", + "solution": "设 $X=\\{1,2,3, \\cdots, 56\\}$, 令 $A_i=\\{i, i+7, i+14, i+21, i+28$, $i+35, i+42, i+49\\}(i=1,2,3, \\cdots, 7), B_j=\\{j, j+8, j+16, j+24$, $j+32, j+40, j+48\\}(i=1,2,3, \\cdots, 8)$. 显然 $\\left|A_i\\right|=8(1 \\leqslant i \\leqslant 7)$, $A_i \\cap A_j=\\varnothing(1 \\leqslant i) 所示方法染色, 而不同的 $P_i$ 与 $P_j$ 所连线段用如图() 所示方法染色, 其中 $a, b, c, d$ 表示 4 种不同颜色.\n这样染好色的 $G_{83}^*$ 显然不包含三边颜色相同的三角形.\n对 $G_{82}^*$ 可同染 $G_{83}^*$ 的方法去染色.\n而对 $G_i^*(1 \\leqslant i \\leqslant 81)$, 可先增加一点并与原 24 个点都连一线段, 然后按染 $G_{83}^*$ 的方法染好色后, 再去掉该点及从该点连出的线段,这样染好色的 $G_i^*$ 显然不含三边颜色相同的三角形.\n综上便知结论成立.", + "remark": "", + "figures": [ + "./images/volume11/figures/fig-c15a7-1.png", + "./images/volume11/figures/fig-c15a7-2.png" + ] +} \ No newline at end of file diff --git a/processed_dataset/proof/0366.json b/processed_dataset/proof/0366.json new file mode 100644 index 0000000000000000000000000000000000000000..4b3fc45d8fc884a2aa33457bff5603ff10fdb597 --- /dev/null +++ b/processed_dataset/proof/0366.json @@ -0,0 +1,10 @@ +{ + "source_file": "./raw_volume-zh/volume11/exercise15.tex", + "problem_type": "proof", + "problem": "问题8. 桌上放着 2000 张互不重叠的相同的圆纸片, 某些圆纸片互相外切.\n问最少要给这些纸片染上多少种颜色,才能使互相外切的圆纸片的颜色互不相同.", + "solution": "如图(), 若仅用 3 色,假设 11 张圆形纸片中有 6 张圆纸片的颜色如图标号, 则 $A, B, C$ 三圆纸片只能染 1 色或 3 色.\n并且 $A$ 和 $C$ 同色,但 $A 、 C$ 与 $B$ 不同色,于是 $D$ 与 $A 、 B$ 不同色, 故 $D$ 染 2 色.\n同理 $E$ 与 $B 、 C$ 不同色, 从而 $E$ 也染 2 色,这与 $D$ 和 $E$ 互相外切不同色矛盾.\n其次,我们证明染上 4 色就够了.\n我们对 $n$ 张圆纸片来证明, $n \\leqslant 4$ 时显然成立.\n设对 $k \\geqslant 4$ 张圆纸片, 染上 4 色就够了.\n对 $k+1$ 张圆纸片, 则这 $k+1$ 张圆纸片中心的凸包为凸多边形, 取某一个顶点 $A$, 则 $A$ 是某圆纸片的中心, 易证该纸片最多与 3 张圆形纸片相切, 去掉 $A$ 纸片, 余下 $k$ 张纸片, 由归纳假设知可染上 4 色使任何两张相切的纸片不同色, 放回 $A$ 纸片, 必可将它染上同它相切的 3 张纸片不同色的第 4 种颜色.\n于是对 $k+1$, 结论也成立.\n故对一切 $n \\in \\mathbf{N}_{+}$, 染上 4 色就够了, 特别对 $n=2000$ 张纸片, 染上 4 色就够了.\n综上可知, 最少要染上 4 种颜色.", + "remark": "", + "figures": [ + "./images/volume11/figures/fig-c15a8.png" + ] +} \ No newline at end of file diff --git a/processed_dataset/proof/0367.json b/processed_dataset/proof/0367.json new file mode 100644 index 0000000000000000000000000000000000000000..4132dac8221abd88175b3796604bdd54090469ee --- /dev/null +++ b/processed_dataset/proof/0367.json @@ -0,0 +1,8 @@ +{ + "source_file": "./raw_volume-zh/volume11/exercise2.tex", + "problem_type": "proof", + "problem": "问题1. 有 17 位科学家,其中每位科学家都同其他所有人通信,他们在通信时只讨论了三个题目,且每两位科学家之间只讨论一个题目, 证明至少有三位科学家,他们互相之间讨论的是同一个题目.", + "solution": "在圆周上任取 17 个点代表 17 位科学家,互相通信讨论的 3 个题目分别用对应两点连线染成红、蓝、黄 3 种颜色来表示, 因为从其中一点 $A_1$ 出发的 16 条线段被染成了 3 种颜色,由抽屈原理知其中至少有 $\\left[\\frac{16-1}{3}\\right]+1=6$ 条同色, 不妨设 $A_1 A_2, A_1 A_3, A_1 A_4, A_1 A_5, A_1 A_6, A_1 A_7$ 同为红色.\n若 $A_2$, $A_3, \\cdots, A_7$ 这 6 点中有 2 点 $A_i, A_j(2 \\leqslant i), 将 $\\triangle A B C$ 分成一个平行四边形 $B D E F$ 和两个三角形: $\\triangle C D E, \\triangle A F E$, (其中 $D 、 E 、 F$ 分别为三边 $B C 、 A C 、 A B$ 的中点) 由抽屈原理知至少在 $\\left[\\frac{7-1}{3}\\right]+1=3$ 个点落在同一个图形内, 以这 3 点为顶点的三角形面积不大于 $\\frac{1}{4}$.", + "remark": "", + "figures": [ + "./images/volume11/figures/fig-c2a3.png" + ] +} \ No newline at end of file diff --git a/processed_dataset/proof/0369.json b/processed_dataset/proof/0369.json new file mode 100644 index 0000000000000000000000000000000000000000..b6d53fde64490d68a4ebfc759c6c19f0336cf881 --- /dev/null +++ b/processed_dataset/proof/0369.json @@ -0,0 +1,8 @@ +{ + "source_file": "./raw_volume-zh/volume11/exercise2.tex", + "problem_type": "proof", + "problem": "问题5. 设 $S=\\{1,2,3, \\cdots, 2000\\}, M$ 是 $S$ 的一个子集且 $M$ 中任意两数之差都不等于 5 或 8 , 问 $M$ 中最多有多少个元素?", + "solution": "首先证明 $S$ 中任意 13 个连续正整数中最多有 6 个属于 $M$. 以 $T= \\{1,2, \\cdots, 13\\}$ 为例进行证明.\n考虑 $T$ 的下列 13 个子集: $\\{1,6\\},\\{2,7\\}$, $\\{3,8\\},\\{4,9\\},\\{5,10\\},\\{6,11\\},\\{7,12\\},\\{8,13\\},\\{1,9\\},\\{2,10\\}$, $\\{3,11\\},\\{4,12\\},\\{5,13\\}$ 且 $T$ 中每个数恰属于 2 个子集.\n任取 $T$ 中 7 个元素, 它们属于上述 13 个子集中 14 个子集, 由抽屉原理知其中必有 2 个元素属于同一个子集,它们之差等于 5 或 8 . 因此 $T$ 中任何 7 个元素都不能同时属于 $M$. 另一方面 $T^{\\prime}=\\{1,2,4,5,8,11\\}$ 中任何两个数之差不等于 5 或 8 , 它们可以同时属于 $M$, 故 $T$ 中最多有 6 个数属于 $M$. 因为 $2000=13 \\times 154-2$, 故 $S$ 中最多有 $154 \\times 6=924$ 个数属于 $M$. 又因为集合 $\\{1,2,4,5$, $8,11,14,15,17,18,21,24\\}$ 中任何两个数之差不等于 5 或 8 , 故集合 $\\{13 n+k \\mid k=1,2,4,5,8,11, n=0,1,2, \\cdots, 153\\}$ 中任何两个数之差不等于 5 或 8 ,并且它有 924 个数.\n综上可知 $M$ 内最多有 924 个数.", + "remark": "", + "figures": [] +} \ No newline at end of file diff --git a/processed_dataset/proof/0370.json b/processed_dataset/proof/0370.json new file mode 100644 index 0000000000000000000000000000000000000000..298f5863b8aec9e972d61b72ca559221f8396f26 --- /dev/null +++ b/processed_dataset/proof/0370.json @@ -0,0 +1,8 @@ +{ + "source_file": "./raw_volume-zh/volume11/exercise2.tex", + "problem_type": "proof", + "problem": "问题7. 10 人到书客去买书,已知:\n(1) 每人都买了 3 种书;\n(2)任何两人买的书中,都至少有一种相同.\n问购买人数最多的一种书最少有几人购买? 说明理由.", + "solution": "设 $A$ 是 10 人之一, 由已知 $A$ 共买了 3 种书, 且其余 9 人所买的书中都至少有一种与 $A$ 买的书相同, 于是由抽屉原理知, 9 人中至少有 $\\left[\\frac{9-1}{3}\\right]+ 1=3$ 人, 加上 $A$ 共 4 人买了同一种书, 因而所求最小值不小于 4 . 若购买人数最多的一种书共有 4 人购买, 则可以证明每种书恰有 4 人购买.\n设 10 人共买下了 $n$ 种不同的书,则有 $4 n=30$, 但因 4 不整除 30 , 此不可能,故知所求的最小值不小于 5 . 另一方面, 我们用数字表示书种的号码, 并使 10 人分别买书如下: $\\{1,2,3\\},\\{1,2,3\\},\\{1,4,5\\},\\{1,6,7\\},\\{2,4,6\\},\\{2,4,6\\},\\{2$, $5,7\\},\\{3,4,7\\},\\{3,5,6\\},\\{3,5,6\\}$. 容易验证,他们买的书满足题中要求且购买人数最多的一种书有 5 人购买, 故知所求最小值等于 5 .", + "remark": "", + "figures": [] +} \ No newline at end of file diff --git a/processed_dataset/proof/0371.json b/processed_dataset/proof/0371.json new file mode 100644 index 0000000000000000000000000000000000000000..9a00ce6da1a52b2262c1c0a955906cca54879f04 --- /dev/null +++ b/processed_dataset/proof/0371.json @@ -0,0 +1,8 @@ +{ + "source_file": "./raw_volume-zh/volume11/exercise2.tex", + "problem_type": "proof", + "problem": "问题8. 的面平面上每个点都以红、蓝色之一着色.\n证明: (1) 对任意实数 $a$, 存在边长为 $a, \\sqrt{3} a, 2 a$ 且三个顶点同色的直角三角形; (2) 存在两个相似三角形, 它们的相似比为 1995 , 并且每个三角形的三个顶点同色.", + "solution": "(1) 首先证明: 对任意实数 $a$, 存在距离等于 $2 a$ 且同色的两点.\n事实上, 取一个红点 $A$, 以 $A$ 为中心, $2 a$ 为半径作圆.\n若该圆上有一红点 $B$, 则结论成立, 否则该圆上所有的点全为蓝点.\n于是该圆的内接正六边形的一边的两个端点的距离等于 $2 a$ 且全为蓝点,结论也成立.\n其次设 $A B=2 a$ 且 $A$ 与 $B$ 同色,不妨设 $A$ 与 $B$ 同为红色, 以 $A B$ 为直径作圆, 并且设该圆的六等分点依次为 $A, C, D$, $B, E, F$. 若 $C 、 D 、 E 、 F$ 中有一点为红点, 例如 $C$ 为红点, 则直角 $\\triangle A B C$ 的 3 个顶点同为红色且 $B C=a, C A=\\sqrt{3} a, A B=2 a$. 结论得证, 否则直角 $\\triangle C D E$ 的 3 个顶点同为蓝色且 $C D=a, D E=\\sqrt{3} a, E C=2 a$, 结论也得证.\n(2) (证法一) 在 (1) 中分别取 $a=1, a=1995$ 所得两个直角三角形即为所求.\n(证法二) 以任意一点 $O$ 为圆心, 分别以 1 和 1995 为半径作两个同心圆.\n在内圆上任取 9 个点, 由抽庶原理知其中至少有 $\\left[\\frac{9-1}{2}\\right]+1=5$ 个点同色, 不妨设内圆上的 5 个点 $A_1, A_2, A_3, A_4, A_5$ 同色,作射线 $O A_i$ 交外圆于 $B_i(i=1,2,3,4,5)$, 再由抽庶原理知 $B_1, B_2, B_3, B_4, B_5$ 中至少有 3 个点 $B_i, B_j, B_k(1 \\leqslant i3$, 则 $\\frac{1}{8} \\sum_{i=1}^8 x_i>3$, 矛盾.\n所以 $x_1=x_2=\\cdots=x_8=3$, 即每个面均为三角形.", + "remark": "", + "figures": [] +} \ No newline at end of file diff --git a/processed_dataset/proof/0373.json b/processed_dataset/proof/0373.json new file mode 100644 index 0000000000000000000000000000000000000000..da0d35b3098d6b01bc3dd9a77d662b3dd4dad6ef --- /dev/null +++ b/processed_dataset/proof/0373.json @@ -0,0 +1,8 @@ +{ + "source_file": "./raw_volume-zh/volume11/exercise2.tex", + "problem_type": "proof", + "problem": "问题10. 平面有限点集叫做稳定的, 如果它内部任意两点间的距离是确定的.\n给定含 $n \\geqslant 4$ 个点的平面点集 $M_n$, 其中任意 3 点不共线, 若 $M_n$ 中有 $\\frac{1}{2} n(n-$ 3 ) +4 对点之间的距离是确定的.\n证明: $M_n$ 是稳定的.", + "solution": "对 $n$ 用归纳法.\n$n=4$ 时, 已知这 4 点中有 $\\frac{1}{2} \\times 4 \\times(4-3)+4=$ 6 对点之间的距离是确定的,但 4 个点一共只能形成 $\\mathrm{C}_4^2=6$ 对点, 所以这 4 点组成的点集 $M_4$ 是稳定的.\n假设 $n=k(k \\geqslant 4)$ 时结论成立, 即当 $M_k$ 中已有 $\\frac{1}{2} k(k-3)+4$ 对点间的距离是确定的时, $M_k$ 是稳定的.\n那么 $n=k+1$ 时, 假设 $M_{k+1}$ 中已有 $\\frac{1}{2}(k+1)(k-2)+4$ 对点之间的距离是确定的.\n那么从这 $k+1$ 个点出发有确定距离的点对数之和为 $(k+1)(k-2)+8$, 由平均值原理知, 其中必有一个点 $A$, 从它出发的有确定距离的点对数 $l \\leqslant \\frac{1}{k+1}\\{(k+1)(k-2)+8\\}=k-1+\\frac{7-k}{k+1}$, 而 $\\frac{7-k}{k+1}<1$, 所以 $l \\leqslant k-1$. 去掉点 $A$,于是还剩下 $k$ 个点, 这 $k$ 个点中至少有 $\\frac{1}{2}(k+1)(k-2)+4- (k-1)=\\frac{1}{2} k(k-3)+4$ 对点之间的距离是确定的.\n由归纳假设知, 这 $k$ 个点所组成的点集是稳定的且 $A$ 至少与这 $k$ 个点中 $\\frac{1}{2}(k+1)(k-2)+4- \\mathrm{C}_k^2=3$ 个点之间的距离是确定的.\n不妨设这 3 个点为 $B 、 C 、 D$, 且 $A B=x$, $A C=y, A D=z$ ( $x, y, z$ 是确定的正数). 这时点 $A$ 也唯一确定, 否则存在另一点 $A^{\\prime} \\neq A$, 使 $A^{\\prime} B=x, A^{\\prime} C=y, A^{\\prime} D=z$, 于是 $B 、 C 、 D$ 都在线段 $A A^{\\prime}$ 的垂直平分线上, 这与已知条件无 3 点共线矛盾, 所以 $M_{k+1}$ 也是稳定的, 这就完成了归纳证明.", + "remark": "", + "figures": [] +} \ No newline at end of file diff --git a/processed_dataset/proof/0374.json b/processed_dataset/proof/0374.json new file mode 100644 index 0000000000000000000000000000000000000000..6b7040d26cc64c6fcb9da94ce0a0e70b48761357 --- /dev/null +++ b/processed_dataset/proof/0374.json @@ -0,0 +1,8 @@ +{ + "source_file": "./raw_volume-zh/volume11/exercise3.tex", + "problem_type": "proof", + "problem": "问题2. 证明下列恒等式:\n(1) $\\sum_{k=1}^n \\mathrm{C}_n^k \\mathrm{C}_n^{n+1-k}=\\mathrm{C}_{2 n}^{n+1}$;\n(2) $\\sum_{k=0}^{\\left[\\frac{n}{2}\\right]}(-1)^k \\mathrm{C}_{n+1}^k \\mathrm{C}_{2 n-2 k}^n=n+1$;\n(3) $\\sum_{k=0}^n(-1)^{n-k} 2^{2 k} \\mathrm{C}_{n+k+1}^{2 k+1}=n+1$;\n(4) $\\sum_{k=p}^n(-1)^k \\mathrm{C}_n^k \\mathrm{C}_k^p=(-1)^n \\delta_{p n}$, 其中 $\\delta_{i j}=\\left\\{\\begin{array}{l}1, i=j, \\\\ 0, i \\neq j .\\end{array}\\right.$", + "solution": "(1) 一方面 $(1+x)^{2 n}=\\sum_{k=0}^{2 n} \\mathrm{C}_{2 n}^k x^k$ 中 $x^{n+1}$ 的系数为 $\\mathrm{C}_{2 n}^{n+1}$, 另一方面 $(1+x)^{2 n}=(1+x)^n(1+x)^n=\\left(\\sum_{i=0}^n \\mathrm{C}_n^i x^i\\right)\\left(\\sum_{j=0}^n \\mathrm{C}_n^j x^j\\right)$ 中 $x^{n+1}$ 的系数为 $\\sum_{k=1}^n \\mathrm{C}_n^k \\mathrm{C}_n^{n+1-k}$, 所以 $\\sum_{k=1}^n \\mathrm{C}_n^k \\mathrm{C}_n^{n+1-k}=\\mathrm{C}_{2 n}^{n+1}$.\n(2) 一方面 $(1+x)^{n+1}=\\sum_{k=0}^{n+1} \\mathrm{C}_{n+1}^k x^k$ 中 $x^n$ 的系数为 $\\mathrm{C}_{n+1}^n=n+1$, 另一方面 $(1+x)^{n+1}=\\frac{\\left(1-x^2\\right)^{n+1}}{(1-x)^{n+1}}=\\left(\\sum_{k=0}^{n+1}(-1)^k C_{n+1}^k x^{2 k}\\right) \\cdot\\left(\\sum_{j=0}^{\\infty} C_{n+j}^n x^j\\right)$ 中 $x^n$ 的系数为 $\\sum_{k=0}^{\\left[\\frac{n}{2}\\right]}(-1)^k \\mathrm{C}_{n+1}^k \\cdot \\mathrm{C}_{n+(n-2 k)}^n=\\sum_{k=0}^{\\left[\\frac{n}{2}\\right]}(-1)^k \\mathrm{C}_{n+1}^k \\mathrm{C}_{2 n-2 k}^n$, 所以 $\\sum_{k=0}^{\\left[\\frac{n}{2}\\right]}(-1)^k \\mathrm{C}_{n+1}^k \\mathrm{C}_{2 n-2 k}^n=n+1$.\n(3)注意到 $\\mathrm{C}_{n+k+1}^{2 k+1}=\\mathrm{C}_{n+k+1}^{n-k}$ 及 $(2-x)^{n+k+1}=\\sum_{i=0}^{n+k+1}(-1)^i \\mathrm{C}_{n+k+1}^i \\cdot 2^{n+k+1-i} x^i$ 中 $x^{n-k}$ 的系数为 $(-1)^{n-k} \\mathrm{C}_{n+k+1}^{n-k} \\cdot 2^{n+k+1-(n-k)}=(-1)^{n-k} 2^{2 k+1} \\mathrm{C}_{n+k+1}^{2 k+1}$, 所以 $f(x)= \\sum_{k=0}^n(-x+2)^{n+k+1} \\cdot x^k$ 中 $x^n$ 的系数为 $A_n=\\sum_{k=0}^n(-1)^{n-k} 2^{2 k+1} \\mathrm{C}_{n+k+1}^{2 k+1}$, 另一方面, $f(x)=(2-x)^{n+1} \\sum_{k=0}^n\\left(2 x-x^2\\right)^k=(2-x)^{n+1} \\cdot \\frac{1-\\left(2 x-x^2\\right)^{n+1}}{1-\\left(2 x-x^2\\right)}=(2- x)^{n+1}(1-x)^{-2}-x^{n+1}(2-x)^{2 n+2}(1-x)^{-2}$ 中含 $x^n$ 的项仅在 $(2-x)^{n+1}(1-x)^{-2}$ 中, 而 $(2-x)^{n+1}(1-x)^{-2}=[1+(1-x)]^{n+1}(1-x)^{-2}=\\mathrm{C}_{n+1}^0(1-x)^{n-1}+ \\mathrm{C}_{n+1}^1(1-x)^{n-2}+\\mathrm{C}_{n+1}^2(1-x)^{n-3}+\\cdots+\\mathrm{C}_{n+1}^n(1-x)^{-1}+(1-x)^{-2}$ 中, 仅最后两项含有 $x^n$, 且这两项中 $x^n$ 的系数之和为 $\\mathrm{C}_{n+1}^n+\\mathrm{C}_{n+1}^1=2(n+1)$, 故 $A_n=2(n+1)$, 即 $\\sum_{k=0}^n(-1)^{n-k} 2^{2 k+1} C_{n+k+1}^{2 k+1}=2(n+1)$, 所以 $\\sum_{k=0}^n(-1)^{n-k} 2^{2 k} C_{n+k+1}^{2 k+1}=n+1$.\n(4)注意到当 $k \\geqslant p$ 时 $(1+x)^k=\\sum_{i=0}^k \\mathrm{C}_k^i x^i$ 中 $x^p$ 的系数为 $\\mathrm{C}_k$; 当 $0 \\leqslant ka_n>$ 0 . \n从而 $a_{n+1}-a_n>0$, 故 $a_{n+1}=7 a_n-a_{n-1} \\cdots$ (3). \n又 $a_0=1, a_1=\\frac{7+\\sqrt{45-36}}{2} =5$ 为正整数且 $a_{n+1}>a_n$. \n用数学归纳法及 (3) 式便知对一切 $n \\in \\mathbf{N}, a_n$ 为正整数; \n第(2)问: 由 (1) 得 $a_n a_{n+1}-1=\\left(\\frac{a_{n+1}+a_n}{3}\\right)^2$. \n并且由 $a_0+a_1=6$ 是 3 的倍数, 以及 $a_{n+1}+a_n=9 a_n-\\left(a_n+a_{n-1}\\right)$ \n用数学归纳法可知对一切 $n \\in \\mathbf{N}, a_{n+1}+ a_n$ 是 3 的倍数,故对一切 $n \\in \\mathbf{N}, a_n a_{n+1}-1$ 是完全平方数.", + "remark": "", + "figures": [] +} \ No newline at end of file diff --git a/processed_dataset/proof/0377.json b/processed_dataset/proof/0377.json new file mode 100644 index 0000000000000000000000000000000000000000..3b3a3b2633c2f29c8c192c3b31a7539a4db6962e --- /dev/null +++ b/processed_dataset/proof/0377.json @@ -0,0 +1,8 @@ +{ + "source_file": "./raw_volume-zh/volume11/exercise4.tex", + "problem_type": "proof", + "problem": "问题9. 证明: 对任何非负整数, $\\left[(1+\\sqrt{3})^{2 n+1}\\right]$ 能被 $2^{n+1}$ 整除.\n这里 $[x]$ 表示不超过 $x$ 的最大整数", + "solution": "设 $a_n=(1+\\sqrt{3})^{2 n+1}+(1-\\sqrt{3})^{2 n+1}=(1+\\sqrt{3})(4+2 \\sqrt{3})^n+(1-\\sqrt{3}) (4-2 \\sqrt{3})^n, b_n=\\frac{a_n}{2^{n+1}}=\\frac{1+\\sqrt{3}}{2}(2+\\sqrt{3})^n+\\frac{1-\\sqrt{3}}{2}(2-\\sqrt{3})^n \\quad n=0,1, \\cdots$. 于是数列 $\\left\\{b_n\\right\\}$ 对应的特征根为 $2 \\pm \\sqrt{3}$, 特征方程为 $[r-(2+\\sqrt{3})][r-(2- \\sqrt{3})]=0$, 即 $r^2-4 r+1=0$, 所以 $b_n$ 满足递推关系 $b_n=4 b_{n-1}-b_{n-2}(n \\geqslant 2)$, 因 $b_0=\\frac{1+\\sqrt{3}}{2}+\\frac{1-\\sqrt{3}}{2}=1$ 和 $b_1=\\frac{1+\\sqrt{3}}{2}(2+\\sqrt{3})+\\frac{1-\\sqrt{3}}{2}(2-\\sqrt{3})=5$ 为整数.\n设 $b_{n-2}, b_{n-1}$ 为整数.\n则 $b_n$ 为整数, 故对一切非负整数 $n, b_n=\\frac{a_n}{2^{n+1}}$ 为整数, 即 $a_n$ 被 $2^{n+1}$ 整除, 而 $0<-(1-\\sqrt{3})^{2 n+1}<1$, 故 $a_n=(1+\\sqrt{3})^{2 n+1}+(1- \\sqrt{3})^{2 n+1}=\\left[(1+\\sqrt{3})^{2 n+1}\\right]$, 所以 $\\left[(1+\\sqrt{3})^{2 n+1}\\right]$ 被 $2^{n+1}$ 整除.", + "remark": "", + "figures": [] +} \ No newline at end of file diff --git a/processed_dataset/proof/0378.json b/processed_dataset/proof/0378.json new file mode 100644 index 0000000000000000000000000000000000000000..77fa40584efcd4d6ab1225b1581b386b4a36795e --- /dev/null +++ b/processed_dataset/proof/0378.json @@ -0,0 +1,8 @@ +{ + "source_file": "./raw_volume-zh/volume11/exercise4.tex", + "problem_type": "proof", + "problem": "问题10. 证明: 对任何非负整数 $n, \\sum_{k=0}^n \\mathrm{C}_{2 n+1}^{2 k+1} 2^{3 k}$ 不能被 35 整除.", + "solution": "令 $x_n=\\sum_{k=0}^n \\mathrm{C}_{2 n+1}^{2 k+1} 2^{3 k}=\\frac{1}{\\sqrt{8}} \\sum_{k=0}^n \\mathrm{C}_{2 n+1}^{2 k+1}(\\sqrt{8})^{2 k+1}, y_n=\\frac{1}{\\sqrt{8}} \\cdot \\sum_{k=0}^n \\mathrm{C}_{2 n+1}^{2 k}(\\sqrt{8})^{2 k}$, 于是 $x_n+y_n=\\frac{1}{2 \\sqrt{2}}(\\sqrt{8}+1)^{2 n+1}, x_n-y_n=\\frac{1}{2 \\sqrt{2}}(\\sqrt{8}-1)^{2 n+1}$, 所以 $x_n= \\frac{1}{4 \\sqrt{2}}\\left[(\\sqrt{8}+1)^{2 n+1}+(\\sqrt{8}-1)^{2 n+1}\\right]=\\frac{1}{4 \\sqrt{2}}\\left[(\\sqrt{8}+1)(9+4 \\sqrt{2})^n+(\\sqrt{8}-1)\\right. \\left.(9-4 \\sqrt{2})^n\\right]$, 可见数列 $\\left\\{x_n\\right\\}$ 的特征根为 $r_{1,2}==9 \\pm 4 \\sqrt{2}$, 由 $r_1+r_2=18$, $r_1 r_2=49$ 知 $\\left\\{x_n\\right\\}$ 对应的特征方程为 $r^2-18 r+49=0$, 所以 $x_n$ 满足递推关系 $x_n=18 x_{n-1}-49 x_{n-2}$, 从而 $2 x_{n-1}=36 x_{n-2}-98 x_{n-3}$, 两式相减整理得 $x_n= 20 x_{n-1}-85 x_{n-2}+98 x_{n-3} \\equiv 3 x_{n-3}(\\bmod 5)$. 可见 5 整除 $x_n$ 的充要条件是 5 整除 $x_{n-3}$. 但 $x_0=1, x_1=\\mathrm{C}_3^1 2^0+\\mathrm{C}_3^3 2^3=11$ 和 $x_2=\\mathrm{C}_5^1 2^0+\\mathrm{C}_5^3 \\cdot 2^3+\\mathrm{C}_5^5 2^6=149$ 都不被 5 整除.\n故对一切非负整数 $n, x_n=\\sum_{k=0}^n \\mathrm{C}_{2 n+1}^{2 k+1} 2^{3 k}$ 不被 5 整除, 又 $x_n= 18 x_{n-1}-49 x_{n-2} \\equiv 4 x_{n-1}(\\bmod 7)$, 可见 $7\\left|x_n \\Leftrightarrow 7\\right| x_{n-1}$, 但 $x_0=1$, 7 $x_0$, 所以 $7 \\times x_n$. 又 $(5,7)=1$, 故对一切非负整数 $n, x_n=\\sum_{k=0}^n \\mathrm{C}_{2 n+1}^{2 k+1} 2^{3 k}$ 不被 35 整除.", + "remark": "", + "figures": [] +} \ No newline at end of file diff --git a/processed_dataset/proof/0379.json b/processed_dataset/proof/0379.json new file mode 100644 index 0000000000000000000000000000000000000000..bbdff9bb2eee3306b59c23f578765ab9b0d5b4d6 --- /dev/null +++ b/processed_dataset/proof/0379.json @@ -0,0 +1,8 @@ +{ + "source_file": "./raw_volume-zh/volume11/exercise5.tex", + "problem_type": "proof", + "problem": "问题4. 有三条长度分别为1,2,3的线段,现将长为3的线段任意分成 $n$ 段, $n \\geqslant 2$, 证明: 在所得的 $n+2$ 条线段中必有三段可组成三角形.", + "solution": "设分成的 $n$ 条线段的长度分别是 $a_1, a_2, \\cdots, a_n$. 若有某个 $a_k>1$, 则 $1,2, a_k$ 可组成三角形,故不妨设 $a_1 \\leqslant a_2 \\leqslant \\cdots \\leqslant a_n \\leqslant 1$. 若 $a_{n-1}>\\frac{1}{2}$, 则 $a_{n-1}+a_n>1$, 从而 $1, a_{n-1}, a_n$ 可组成三角形; 若 $a_{n-1} \\leqslant \\frac{1}{2}$, 且存在 $k$ 使 $a_k+ a_{k+1}>a_{k+2}$, 则 $a_k, a_{k+1}, a_{k+2}$ 可组成三角形; 若 $a_{n-1} \\leqslant \\frac{1}{2}$ 且对任何 $k$ 有 $a_k+ a_{k+1} \\leqslant a_{k+2}$, 则 $a_k \\leqslant \\frac{1}{2} a_{k+2}$, 从 $a_{n-1} \\leqslant \\frac{1}{2}$ 出发推出, 当 $n$ 为奇数时, $a_1+a_2+ \\cdots+a_n \\leqslant 2\\left(a_2+a_4+\\cdots+a_{n-1}\\right)+a_n \\leqslant 2\\left(\\frac{1}{2}+\\frac{1}{4}+\\cdots+\\frac{1}{2^{\\frac{n}{2}}}\\right)+1<3$. 当 $n$ 为偶数时, $a_1+a_2+\\cdots+a_n \\leqslant 2\\left(a_1+a_3+\\cdots+a_{n-1}\\right)+1 \\leqslant 2\\left(\\frac{1}{2}+\\frac{1}{4}+\\cdots+\\frac{1}{2^{\\frac{n}{2}}}\\right)+1<3$. 矛盾.\n故命题得证.", + "remark": "", + "figures": [] +} \ No newline at end of file diff --git a/processed_dataset/proof/0380.json b/processed_dataset/proof/0380.json new file mode 100644 index 0000000000000000000000000000000000000000..93846520c3c56f8414bc2d7fda66644b2058bc22 --- /dev/null +++ b/processed_dataset/proof/0380.json @@ -0,0 +1,8 @@ +{ + "source_file": "./raw_volume-zh/volume11/exercise5.tex", + "problem_type": "proof", + "problem": "问题6. 在平面直角坐标系上有 9 个整点 $A_i\\left(x_i, y_i\\right)\\left(x_i, y_i \\in \\mathbf{Z}, i=1,2,3\\right.$, $\\cdots, 9)$, 其中任意三点不共线, 求证: 必存在一个 $\\triangle A_i A_j A_k(1 \\leqslant i)所示 7 点间连有 9 条线段可保证条件 (1) 被满足.\n面证明任何 8 条连线都不满足条件(1), 即存在三个点,它们之间无连线.\n因从各点出发的线段数之和为 $2 \\times 8=16$,故由第二抽屉原理 (或平均值原理) 知有一点 $A$,从它出发至多有 $\\left[\\frac{16}{7}\\right]=2$ 条线段.\n(i) 若从 $A$ 出发至多有 1 条连线,则至少有 5 点与 $A$ 没有连线, 这 5 点间至少有 $C_5^2-8=2$ 对点没有连线, 设 $C$ 与\n$D$ 没有连线, 于是 $A 、 C 、 D 3$ 点间无连线; (ii) 若从 $A$ 出发有两条连线 $A B$ 和 $A C$, 考察其余 4 点 $D 、 E 、 F 、 G$ 之间的连线情况.\n(a) 若有 6 条连线, 则 $B 、 C$ 之间以及 $B 、 C$ 与后 4 点的任何一点间没有连线, 从而 $B 、 C 、 D$ 之间无线相连.\n(b) 若至多有 5 条连线, 则至少有 $\\mathrm{C}_4^2-5=1$ 对点之间没有连线, 设 $D$ 与 $E$ 没有连线, 于是 $A 、 D 、 E$ 之间无线相连.\n综上可知, 满足要求的连线法最少要 9 条线.", + "remark": "", + "figures": [ + "./images/volume11/figures/fig-c5a8.png" + ] +} \ No newline at end of file diff --git a/processed_dataset/proof/0382.json b/processed_dataset/proof/0382.json new file mode 100644 index 0000000000000000000000000000000000000000..9f1104f46cbda49032318032a64a715d5783dd28 --- /dev/null +++ b/processed_dataset/proof/0382.json @@ -0,0 +1,10 @@ +{ + "source_file": "./raw_volume-zh/volume11/exercise5.tex", + "problem_type": "proof", + "problem": "问题9. 设凸六边形 $A_1 A_2 A_3 A_4 A_5 A_6$ 的面积为 $S$. 证明 : 以其中 3 点为顶点的所有三角形中必有一个三角形的面积不大于 $\\frac{1}{6} S$.", + "solution": "如图()所示设 3 条对角线 $A_1 A_4$ 与 $A_2 A_5, A_2 A_5$ 与 $A_3 A_6, A_3 A_6$ 与 $A_1 A_4$ 的交点分别为 $M_1, M_2, M_3$. 连接 $A_1 M_2, A_3 M_1, A_5 M_3$. 我们首先证明 6 个三角形: $\\triangle A_1 A_2 M_2, \\triangle A_2 A_3 M_1, \\triangle A_3 A_4 M_1, \\triangle A_4 A_5 M_3$, $\\triangle A_5 A_6 M_3, \\triangle A_6 A_1 M_2$ 中必有一个的面积 $\\leqslant \\frac{1}{6} S$. 事 $S$. 由平均值原理知其中必有一个三角形的面积 $\\leqslant \\frac{1}{6} S$, 不妨设 $S_{\\triangle A_1 A_2 M_2} \\leqslant \\frac{1}{6} S$. 其次我们证明题中结论成立.\n(a) 若 $A_1 A_2 / / A_3 A_6$, 则 $S_{\\triangle A_1 A_2 A_6}=S_{\\triangle A_1 A_2 M_2} \\leqslant\\frac{1}{6} S$. (b)若 $A_1 A_2 \\not X A_3 A_6$, 则 $\\min \\left\\{S_{\\triangle A_1 A_2 A_6}, S_{\\triangle A_1 A_2 A_3}\\right\\} \\frac{5}{6}(n-5)$, 结论成立.\n下设有一条线路 $l$ 上只有两个车站 $A$ 和 $B$, 因为过每个车站至多只有 3 条线路, 故过 $A, B$ 的线路除 $l$ 外, 至多还有 4 条, 我们称过 $A$, $B$ 的线路为红线 (至多 5 条), 其余至少有 $n-5$ 条线称为蓝线, 并将红线上的站称为红站, 个数为 $r$; 其余站称为蓝站, 其个数为 $b$,于是 $m=r+b$. 现在考察所有 (站, 蓝线) 的对数.\n一方面, 每条蓝线至少过 2 个站, 故所设对数 $\\geqslant 2(n-$ 5). 另一方面, 任意一个红站至多在两条蓝线上.\n故 (红站, 蓝线) 的对数 $\\leqslant 2 r$; 而每个蓝站至多有 3 条蓝线经过, 故 (蓝站, 蓝线) 的对数 $\\leqslant 3 b$. 因此 $2(n- 5) \\leqslant$ (站, 蓝线) 的对数 $=$ (红站, 蓝线) 的对数 + (蓝站, 蓝线) 的对数 $\\leqslant 2 r+$ 3b. 此外, 每条蓝线, 可经过一条线路与红线 $l$ 相连, 因此, 每条蓝线上至少有 1 个红站.\n故 (红站, 蓝线) 的对数 $\\geqslant n-5$. 于是, $n-5 \\leqslant 2 r$. 因此, 站的总数 $m= r+b=\\frac{1}{3}(2 r+3 b)+\\frac{1}{6} \\cdot 2 r \\geqslant \\frac{1}{3} \\cdot 2(n-5)+\\frac{1}{6} \\cdot(n-5)=\\frac{5}{6}(n-5)$.", + "remark": "", + "figures": [] +} \ No newline at end of file diff --git a/processed_dataset/proof/0385.json b/processed_dataset/proof/0385.json new file mode 100644 index 0000000000000000000000000000000000000000..5287fcbda1b7a8c196a0be195486205037edba8a --- /dev/null +++ b/processed_dataset/proof/0385.json @@ -0,0 +1,8 @@ +{ + "source_file": "./raw_volume-zh/volume11/exercise6.tex", + "problem_type": "proof", + "problem": "问题8. 在一个车厢中, 任何 $m(\\geqslant 3)$ 个旅客都有唯一的公共朋友 (当甲是乙的朋友时, 乙也是甲的朋友, 任何人不是自己的朋友). 问这个车厢中有朋友最多的人有多少个朋友?", + "solution": "设有朋友最多的人是 $A$, 他有 $k$ 个朋友 $B_1, B_2, \\cdots, B_k$, 记 $S=\\left\\{B_1\\right.$, $\\left.B_2, \\cdots, B_k\\right\\}$, 显然 $k \\geqslant m$. 若 $k>m$, 设 $\\left\\{B_{i_i}, B_{i_2}, \\cdots, B_{i_{m-1}}\\right\\}$ 是 $S$ 的任意 $m-1$ 元子集.\n于是 $A, B_{i_1}, \\cdots, B_{i_{m-1}}$ 有唯一的公共朋友 $C_i$. 因 $C_i$ 是 $A$ 的朋友,故 $C_i \\in S$. 我们令 $\\left\\{B_{i_1}, B_{i_2}, \\cdots, B_{i_{m-1}}\\right\\}$ 与 $C_i$ 对应,构成从 $S$ 的所有 $m-1$ 元子集构成的集族 $\\mathscr{B}$ 到 $S$ 的一个映射.\n下面证明 $f$ 为单射,若不然,则存在 $S$ 的两个不同的 $m-1$ 元子集 $\\left\\{B_{i_1}, B_{i_2}, \\cdots, B_{i_{m-2}}\\right\\}$ 和 $\\left\\{B_{j_1}, B_{j_2}, \\cdots, B_{j_{m-1}}\\right\\}$ 它们对应的元素 $C_i$ 与 $C_j$ 相同.\n(记为 $C$ ), 因 $C \\in S$, 故 $C \\neq A$, 于是 $\\left\\{B_{i_1}, \\cdots\\right.$, $\\left.B_{i_{m-1}}\\right\\} \\cup\\left\\{B_{j_1}, \\cdots, B_{j_{m-1}}\\right\\}$ 中至少有 $m$ 位旅客, 但他们却有两个公共朋友 $A$ 和 $C$, 这与已知矛盾.\n故 $f$ 是单射.\n所以 $|\\mathscr{B}| \\leqslant|S|$ 即 $\\mathrm{C}_k^{m-1} \\leqslant k$, 但已知 $m \\geqslant 3$ 且 $k>m$, 即 $k-2 \\geqslant m-1 \\geqslant 2$. 故 $\\mathrm{C}_k^{m-1} \\geqslant \\mathrm{C}_k^{k-2}=\\mathrm{C}_k^2>k$, 这与 $\\mathrm{C}_k^{m-1} \\leqslant k$ 矛盾.\n于是得到 $k=m$, 即有朋友最多的人有 $m$ 个朋友.", + "remark": "", + "figures": [] +} \ No newline at end of file diff --git a/processed_dataset/proof/0386.json b/processed_dataset/proof/0386.json new file mode 100644 index 0000000000000000000000000000000000000000..0f52120eac21bf999a51147a41da1f4e063fe456 --- /dev/null +++ b/processed_dataset/proof/0386.json @@ -0,0 +1,8 @@ +{ + "source_file": "./raw_volume-zh/volume11/exercise6.tex", + "problem_type": "proof", + "problem": "问题9. 设 $A_i$ 是有限集合, $i=1,2, \\cdots, n$. 若 $\\sum_{1 \\leqslant in m=|Y|$, 所以 $f$ 不是单射, 于是存在 $a_i, a_j \\in X(i \\neq j)$ 使 $\\left(x_i, y_i\\right)=f(i)=f(j)=\\left(x_j, y_j\\right)$, 但若 $a_ia_j$ 则应有 $y_i \\geqslant y_j+1$ 也矛盾, 于是命题得证.\n(注: 本题结论也可用抽屉原理去证明,这里就省略了)", + "remark": "", + "figures": [] +} \ No newline at end of file diff --git a/processed_dataset/proof/0388.json b/processed_dataset/proof/0388.json new file mode 100644 index 0000000000000000000000000000000000000000..784fba3814f5edf5b09a1cf7aea6f28ee50bafef --- /dev/null +++ b/processed_dataset/proof/0388.json @@ -0,0 +1,8 @@ +{ + "source_file": "./raw_volume-zh/volume11/exercise6.tex", + "problem_type": "proof", + "problem": "问题11. 由 $a, b$ 两个字母排成的长为 15 的序列中, 恰好出现 \" $a a$ \" 5 次, \" $a b$ \", \" $b a \", ~ \" b b \"$ 各 3 次的序列有多少个?", + "solution": "将连续出现的几个 $a$ 合并成一个 $(a)$, 连续出现的几个 $b$ 合并为一个 (b) , 于是符合条件的序列只可能缩写为下列两种形式: ( I ) (a) (b) (a) (b) (a) $(b)(a) ;($ II $)(b)(a)(b)(a)(b)(a)(b)$, 并设符合条件的序列中 ( I )、( II ) 类序列集合分别为 $B_1, B_2$. ( I )、(II) 类序列都保证出现 3 个\" $a b$ \"和 3 个\" $b a$ \", 根据条件 \"有 5 个 $a a, 3$ 个 $b b$ \"知, 集合 $B_1$ 与下述放法的全体 $E_1$ 之间可建立一一对应: 5 个 $a$ 分放进 (I) 中 4 个 $(a)$ 的括号内, 3 个 $b$ 分放进 (I) 中 3 个 (b) 的括号内.\n由第一讲中可重复的组合数公式得 $\\left|E_1\\right|=\\mathrm{C}_{4+5-1}^5 \\cdot \\mathrm{C}_{3+3-1}^3= \\mathrm{C}_8^5 \\mathrm{C}_5^3=560$, 所以 $\\left|B_1\\right|=\\left|E_1\\right|=560$, 同理可得 $\\left|B_2\\right|=\\left|E_2\\right|=\\mathrm{C}_{3+5-1}^5 \\mathrm{C}_{4+3-1}^3= \\mathrm{C}_7^5 \\mathrm{C}_6^3=420$. 于是满足题目条件的序列共有 $560+420=980$ 个.", + "remark": "", + "figures": [] +} \ No newline at end of file diff --git a/processed_dataset/proof/0389.json b/processed_dataset/proof/0389.json new file mode 100644 index 0000000000000000000000000000000000000000..e988cd74c9ab852ec51c907fb79f67fa7874e57c --- /dev/null +++ b/processed_dataset/proof/0389.json @@ -0,0 +1,8 @@ +{ + "source_file": "./raw_volume-zh/volume11/exercise6.tex", + "problem_type": "proof", + "problem": "问题13. $n$ 项的 0,1 序列 $\\left(x_1, x_2, \\cdots, x_n\\right)$ 称为长为 $n$ 的二元序列.\n$a_n$ 为无连续三项成 $0,1,0$ 的长为 $n$ 的二元序列的个数, $b_n$ 为无连续四项成 $0,0,1,1$ 或 $1,1,0,0$ 的长为 $n$ 的二元序列的个数.\n证明: 对每一个正整数 $n$, $b_{n+1}=2 a_n$.", + "solution": "对任意一个长为 $n+1$ 二元序列 $y=\\left(y_1, y_2, \\cdots, y_{n+1}\\right)$ 令它与一个长为 $n$ 的二元序列 $x=\\left(x_1, x_2, \\cdots, x_n\\right)$ 对应, 其中 $x_i \\equiv y_i+y_{i+1}(\\bmod 2)$, $i=1,2, \\cdots, n \\cdots$ (1). \n显然这样的 $x$ 由 $y$ 唯一确定.\n反过来, 对任意一个长为 $n$ 的二元序列 $x=\\left(x_1, x_2, \\cdots, x_n\\right)$ 及 $y_1=0$ 或 1 , 有一个长为 $n+1$ 的二元序列 $y=\\left(y_1, y_2, \\cdots, y_{n+1}\\right)$, 其中 $y_{i+1} \\equiv y_i+x_i(\\bmod 2), i=1,2, \\cdots$, $n \\cdots$ (2). \n由于 $y_j+y_{j+1}=y_j+y_j+x_j \\equiv x_j(\\bmod 2)$, 所以由(2)定义的对应恰好是(1)的对应的逆对应.\n在由(1)定义的对应中连续 4 项 0011 或 1100 产生连续 3 项 010, 反之由(2)定义的对应,连续 3 项 010 产生连续 4 项 0011 或 1100 . 于是 $a_n$ 个所述长为 $n$ 无连续 3 项成 010 的二元序列, 每个恰好与 2 个无连续 4 项成 0011 或 1100 的所述长为 $n+1$ 的二元序列对应, 从而 $b_{n+1}=2 a_n$.", + "remark": "", + "figures": [] +} \ No newline at end of file diff --git a/processed_dataset/proof/0390.json b/processed_dataset/proof/0390.json new file mode 100644 index 0000000000000000000000000000000000000000..1d69a0d07a7bec83b4410a69919963c501576157 --- /dev/null +++ b/processed_dataset/proof/0390.json @@ -0,0 +1,8 @@ +{ + "source_file": "./raw_volume-zh/volume11/exercise7.tex", + "problem_type": "proof", + "problem": "问题2. 一所学校有 $b$ 个老师和 $c$ 个学生且满足: (1) 每个老师恰教 $k$ 个学生; (2)对任意两个不同的学生恰有 $h$ 个老师同时教他们.\n求证: $\\frac{b}{h}= \\frac{c(c-1)}{k(k-1)}$.", + "solution": "如果老师 $t_r$ 同时教两名学生 $s_i$ 与 $s_j$, 那么将 $t_r$ 与 $s_i, s_j$ 组成三元组 $\\left(t_{r j}, s_i, s_j\\right)$, 并设这样的三元组共有 $M$ 个.\n一方面, 对任意一位老师 $t_r$, 他恰教了 $k$ 位学生, 可形成 $\\mathrm{C}_k^2$ 个含 $t_r$ 的三元组, 而 $t_r$ 有 $b$ 种取法, 所以 $M=b C_k^2$, 另一方面, 对任意两名学生 $s_i, s_j$, 恰有 $h$ 位老师教他们, 可形成 $h$ 个含 $s_i, s_j$ 的三元组, 而 $s_i, s_j$ 有 $\\mathrm{C}_c^2$ 种取法, 所以 $M=h \\mathrm{C}_c^2$, 综上可得 $b \\mathrm{C}_k^2=h \\mathrm{C}_c^2$, 所以 $\\frac{b}{h}=\\frac{\\mathrm{C}_c^2}{\\mathrm{C}_k^2}=\\frac{\\frac{1}{2} c(c-1)}{\\frac{1}{2} k(k-1)}=\\frac{c(c-1)}{k(k-1)}$.", + "remark": "", + "figures": [] +} \ No newline at end of file diff --git a/processed_dataset/proof/0391.json b/processed_dataset/proof/0391.json new file mode 100644 index 0000000000000000000000000000000000000000..d9ae32b62586eddfc33f3d686b366c9f68e60a6e --- /dev/null +++ b/processed_dataset/proof/0391.json @@ -0,0 +1,8 @@ +{ + "source_file": "./raw_volume-zh/volume11/exercise7.tex", + "problem_type": "proof", + "problem": "问题3. 设 $A_i \\subseteq M=\\{1,2, \\cdots, 2010\\}$ 且 $\\left|A_i\\right| \\geqslant 335(i=1,2, \\cdots, 30)$, 证明: 存在 $A_i, A_j(1 \\leqslant i1)$ 天, 共发出 $m$ 个奖牌, 第 1 天发出 1 个加上余下奖牌的 $\\frac{1}{7}$, 第 2 天发出 2 个加上余下奖牌的 $\\frac{1}{7}$, 如此继续下去, 最后第 $n$ 天刚好发出 $n$ 个奖牌恰无剩余.\n问运动会共开了几天? 共发出多少个奖牌?", + "solution": "设第 $k$ 天共发出 $a_k$ 个奖牌.\n则 $a_1=1+\\frac{1}{7}(m-1)=\\frac{1}{7}(m+6), a_k=k+ \\frac{1}{7}\\left(m-a_1-a_2-\\cdots-a_{k-1}-k\\right), a_{k+1}=k+1+\\frac{1}{7}\\left[m-a_1-a_2-\\cdots-a_k-(k+1)\\right]$, 两式相减得 $a_{k+1}-a_k=1+\\frac{1}{7}\\left(-a_k-1\\right)$, 即 $a_{k+1}-6=\\frac{6}{7}\\left(a_k-6\\right)$, 所以 $a_k-6= \\left(a_1-6\\right)\\left(\\frac{6}{7}\\right)^{k-1}=\\frac{1}{7}\\left(\\frac{6}{7}\\right)^{k-1}(m-36)$, 于是 $m=a_1+a_2+\\cdots+a_n=\\frac{1}{7}(m-$ 36) $\\left[1+\\frac{6}{7}+\\left(\\frac{6}{7}\\right)^2+\\cdots+\\left(\\frac{6}{7}\\right)^{n-1}\\right]+6 n=(m-36)\\left[1-\\left(\\frac{6}{7}\\right)^n\\right]+6 n$, 解出 $m$ 得 $m=\\frac{7^n}{6^{n-1}}(n-6)+36$. 因 $7^n$ 与 $6^{n-1}$ 互素, 且 $m, n$ 为正整数, 所以 $6^{n-1} \\mid n-6$. 又 $n>1$ 时易证 $6^{n-1}>|n-6|$. 故只能 $n=6$, 从而 $m=36$. 即运动会共开了 6 天, 一共发出 36 个奖牌.", + "remark": "", + "figures": [] +} \ No newline at end of file diff --git a/processed_dataset/proof/0397.json b/processed_dataset/proof/0397.json new file mode 100644 index 0000000000000000000000000000000000000000..ec7404d261887c77f0bf5d7e8867e355e55e3b63 --- /dev/null +++ b/processed_dataset/proof/0397.json @@ -0,0 +1,8 @@ +{ + "source_file": "./raw_volume-zh/volume11/exercise8.tex", + "problem_type": "proof", + "problem": "问题5. 一副纸牌共 52 张, 其中 \"方块\"、\"梅花\"、\"红心\"、\"黑桃\"每种花色的牌各 13 张, 标号依次是 $2,3, \\cdots, 10, \\mathrm{~J}, \\mathrm{Q}, \\mathrm{K}, \\mathrm{A}$, 其中相同花色、相邻标号的两张牌称为 \"同花顺牌\",并且 $A$ 与 2 也算顺牌 (即 $A$ 可当成 1 使用). 试确定, 从这副牌中取出 13 张牌, 使每种标号的牌都出现, 并且不含\"同花顺牌\"的取牌方法数.", + "solution": "将一个圆盘分成 13 个相等的扇形, 每个扇形依次代表标号为 2 , $3, \\cdots, 10, \\mathrm{~J}, \\mathrm{Q}, \\mathrm{K}, \\mathrm{A}$ 的一张牌.\n而\"方块\"、\"梅花\"、\"红心\"、\"黑桃\"分别用 4 种不同颜色表示.\n于是原问题等价于下列问题中 $n=13, m=4$ 的情形: 把圆分成 $n(\\geqslant 2)$ 个扇形,设用 $m(\\geqslant 2)$ 种颜色给这些扇形染色, 每个扇形恰染一种颜色, 并且要求相邻的扇形的颜色互不相同, 设共有 $a_n(m)$ 种不同的染色方法,求 $a_n(m)$. 解答如下: \n(1) 求初始值, $n=2$ 时,给 $S_1$ 染色有 $m$ 种方法,继而给 $S_2$ 染色只有 $m-1$ 种方法 (因 $S_1$ 与 $S_2$ 不同色), 所以 $a_2(m)=m(m-1$ ). \n(2) 求递推关系,因 $S_1$ 有 $m$ 种染色方法, $S_2$ 有 $m-1$ 种染色方法, $\\cdots, S_{n-1}$ 有 $m-1$ 种染色方法, $S_n$ 仍有 $m-1$ 种染色方法 (不保证 $S_n$ 与 $S_1$ 不同色), 这样共有 $m(m-1)^{n-1}$ 种染色方法, 但这 $m(m-1)^{n-1}$ 种染色方法可分为两类: 一类是 $S_n$ 与 $S_1$ 不同色, 此时的染色方法有 $a_n(m)$ 种, 另一类是 $S_n$ 与 $S_1$ 同色.\n则将 $S_n$ 与 $S_1$ 合并成一个扇形, 并注意此时 $S_{n-1}$ 与 $S_1$ 不同色, 故这时的染色方法有 $a_{n-1}(m)$ 种, 由加法原理得 $a_n(m)+a_{n-1}(m)=m(m-1)^{n-1}(n \\geqslant 2) \\cdots, \\label{eq1}$ .\n(3) 求 $a_n(m)$, 令 $b_n(m)=\\frac{a_n(m)}{(m-1)^n}$, 则由式\\ref{eq1}可得 $b_n(m)+\\frac{1}{m-1} b_{n-1}(m)= \\frac{m}{m-1}$, 即 $b_n(m)-1=-\\frac{1}{m-1}\\left(b_{n-1}(m)-1\\right)$, 所以 $b_n(m)-1=\\left(b_2(m)-1\\right)\\cdot\\left(-\\frac{1}{m-1}\\right)^{n-2}=\\left[\\frac{a_2(m)}{(m-1)^2}-1\\right]\\left(-\\frac{1}{m-1}\\right)^{n-2}=\\left[\\frac{m(m-1)}{(m-1)^2}-1\\right]\\left(-\\frac{1}{m-1}\\right)^{n-2} =(-1)^n \\frac{1}{(m-1)^{n-1}}$ 所以 $a_n(m)=(m-1)^n b_n(m)=(m-1)^n+(-1)^n(m-1)$. 即共有 $(m-1)^n+(-1)^n(m-1)$ 种不同的染色方法.\n于是原问题中所求取牌方法数为 $a_{13}(4)=3^{13}-3$.", + "remark": "", + "figures": [] +} \ No newline at end of file diff --git a/processed_dataset/proof/0398.json b/processed_dataset/proof/0398.json new file mode 100644 index 0000000000000000000000000000000000000000..eb95180175afd51ab4015828254ce65443d618ac --- /dev/null +++ b/processed_dataset/proof/0398.json @@ -0,0 +1,8 @@ +{ + "source_file": "./raw_volume-zh/volume11/exercise8.tex", + "problem_type": "proof", + "problem": "问题7. 是否存在无穷多组不同的正整数 $a, b$ 使 $a^2+b^2+1$ 被 $a b$ 整除?", + "solution": "类似于例 7 可以先用数学归纳法证明下列命题: 设 $a_1=1, a_2=2$, $a_{n+2}=3 a_{n+1}-a_n\\left(n \\in \\mathbf{N}_{+}\\right)$, 则 $3 a_n a_{n+1}=a_n^2+a_{n+1}^2+1\\left(n \\in \\mathbf{N}_{+}\\right)$(证明留给读者自己完成)然后令 $a=a_n, b=a_{n+1}\\left(n \\in \\mathbf{N}_{+}\\right)$便知存在无穷多对不同的正整数 $a, b$ 满足 $a^2+b^2+1$ 被 $a b$ 整除.", + "remark": "", + "figures": [] +} \ No newline at end of file diff --git a/processed_dataset/proof/0399.json b/processed_dataset/proof/0399.json new file mode 100644 index 0000000000000000000000000000000000000000..c355d6d30c77caf211cc9ad9ee7583fec1f68db3 --- /dev/null +++ b/processed_dataset/proof/0399.json @@ -0,0 +1,8 @@ +{ + "source_file": "./raw_volume-zh/volume11/exercise8.tex", + "problem_type": "proof", + "problem": "问题8. 是否存在无穷多对不同的三数组 $(a, b, c)$, 满足: $a, b, c$ 是成等差数列的正整数 $(a), 因为 $11 \\times 12$ 的矩形中有 $11 \\times 12=132$ 个 $1 \\times 1$ 的方格, 且 $126=7 \\times 18<132<7 \\times 19=133$, 所以,如果用 19 张 $1 \\times$ 6 或 $1 \\times 7$ 的矩形能够完全覆盖 $11 \\times 12$ 矩形,则要用 18 张 $1 \\times 7$ 矩形和 1 张 $1 \\times 6$ 矩形.\n如图对 $11 \\times 12$ 的矩形用黑白两种颜色染色, 使得任意两个黑色格都无法被 1 张 $1 \\times 7$ 的矩形同时覆盖.\n图中共有 20 个黑格, 18 张 $1 \\times 7$ 的矩形只能盖住其中 18 个黑格, 还剩下两个黑格, 那张 $1 \\times 6$ 的矩形无法把他们都盖住.\n这就证明了用 19 张 $1 \\times 6$ 或 1 $\\times 7$ 的矩形不可能完全覆盖 $11 \\times 12$ 的矩形.", + "remark": "", + "figures": [ + "./images/volume11/figures/fig-c9a1.png" + ] +} \ No newline at end of file diff --git a/processed_dataset/proof/0402.json b/processed_dataset/proof/0402.json new file mode 100644 index 0000000000000000000000000000000000000000..7a5920a45baf036069354128229b621c62a4882e --- /dev/null +++ b/processed_dataset/proof/0402.json @@ -0,0 +1,13 @@ +{ + "source_file": "./raw_volume-zh/volume11/exercise9.tex", + "problem_type": "proof", + "problem": "问题2. (1)如果 $m\\times n$ 棋盘能用形如图()或图()的 $L$ 形骨牌完全覆盖,证明$m n$是8的倍数\n(2) 对怎样的正整数 $m, n, m \\times n$ 棋盘能用 (1) 中规定的 $L$ 形骨牌完全覆盖.", + "solution": "(1) 设 $m \\times n$ 棋盘能用若干张 $\\mathrm{L}$ 型骨牌完全覆盖, 则显然有 $m \\geqslant 2$, $n \\geqslant 2$. 又因每张 $\\mathrm{L}$ 型骨牌含 4 个方格, 所以 $m n$ 是 4 的倍数.\n不妨设 $m$ 为偶数,将 $m \\times n$ 棋盘奇数行中每行 $n$ 个小方格都染成黑色, 偶数行中每行 $n$ 个小方格染成白色.\n于是每张 L 型骨牌或者盖住 3 个白格和一个黑格或者盖住 3 个黑格和一个白格.\n总之盖住的黑格是奇数个, 白格也是奇数个, 如果 $m n$ 不是 8 的倍数, 那么 $m=4(2 k+1)$, 若 $m \\times n$ 棋盘可用 $\\mathrm{L}$ 型骨牌完全覆盖, 则必须用 $2 k+1$ 张 $\\mathrm{L}$ 型骨牌, 奇数个奇数之和仍为奇数, 这表明 $m \\times n$ 棋盘有奇数个黑格和奇数个白格.\n但棋盘上黑格与白格各占一半都为 $2(2 k+1)$ 格, 矛盾.\n因此, $m n$ 是 8 的倍数.\n(2)由(1)知 $m \\times n$ 矩形可用 $\\mathrm{L}$ 型骨牌完全覆盖的必要条件是 $m \\geqslant 2$, $n \\geqslant 2$, 且 $m n$ 是 8 的倍数, 下面我们证明这一条件也是充分的.\n分两种情形: 情形 $1: m$ 与 $n$ 均为偶数.\n因 $m n$ 是 8 的倍数, 所以 $m, n$ 中必有一个是 4 的倍数.\n不妨设 $m=2 m_1, n=4 n_1$. 于是 $m \\times n$ 棋盘可分成 $m_1 n_1$ 个 $2 \\times 4$ 的棋盘, 而每个 $2 \\times 4$ 的棋盘可用 2 张 L 型骨牌覆盖 (如图()), 故 $m \\times n$ 棋盘可用 $2 m_1 n_1$ 张 $\\mathrm{L}$ 型骨牌完全覆盖.\n情形2: $m, n$ 中有一个为奇数, 另一个为 8 的倍数.\n不妨设 $m=2 m_1+1, n=8 n_1$, 于是 $m \\times n$ 棋盘可分成两部分, 一个是 $2\\left(m_1-1\\right) \\times 8 n_1$ 棋盘, 另一个是 $3 \\times 8 n_1$ 棋盘, 其中第一部分可分成 $\\left(m_1-1\\right) \\times 2 n_1$ 个 $2 \\times 4$ 的棋盘, 从而可用 $4\\left(m_1-1\\right) n_1$ 块 L 型骨牌覆盖.\n第二部可分成 $n_1$ 个 $3 \\times 8$ 的棋盘, 而 $3 \\times 8$ 的棋盘可用 6 张 L 型骨牌覆盖(如图()), 从而第 2 部分可用 $6 n_1$ 张 $\\mathrm{L}$ 型骨牌覆盖.\n因此总个 $m \\times n$ 棋盘可用 $\\mathrm{L}$ 型骨牌完全覆盖.", + "remark": "", + "figures": [ + "./images/volume11/figures/fig-c9p2-1.png", + "./images/volume11/figures/fig-c9p2-2.png", + "./images/volume11/figures/fig-c9a2-1.png", + "./images/volume11/figures/fig-c9a2-2.png" + ] +} \ No newline at end of file diff --git a/processed_dataset/proof/0403.json b/processed_dataset/proof/0403.json new file mode 100644 index 0000000000000000000000000000000000000000..c530184fad337bdbb2fe0a64ae6d78efa31500c7 --- /dev/null +++ b/processed_dataset/proof/0403.json @@ -0,0 +1,10 @@ +{ + "source_file": "./raw_volume-zh/volume11/exercise9.tex", + "problem_type": "proof", + "problem": "问题3. 求最小正整数 $n$, 使得任意 $n$ 个人中都存在 5 个人可分成两个恰有一个公共成员的 3 人组, 并且每个 3 人组内的 3 个人都互相认识或者都互相不认识.", + "solution": "如图(), 用 $n$ 点表示 $n$ 个人, 如果两人互相认识, 那么对应点连一条红线, 否则连一条蓝线, 问题化为: 求最小正整数 $n$, 使得 2 色完全图 $K_n$ 中存在两个恰有一个公共顶点的同色三角形.\n如图 2 色完全图 $K_8$ 中 (图中画的实线表红线, 蓝线没有画出) 没有蓝色三角形, 共有 8 个红色三角形, 且任何两个红色三角形或者有公共边或者无公共顶点.\n故所求最小正整数 $n \\geqslant 9$. 设 2 色 $K_9$ 的 9 个顶点为 $A_1, A_2, \\cdots, A_9$, 从 $A_i$ 出发有 $x_i$ 条红边和 $8-x_i$ 条蓝边 $(i=1,2, \\cdots, 9)$. 我们称从一点出发的一条红边和一条蓝边组成的角为异色角, 于是以 $A_i$ 为顶点的异色角有 $x_i\\left(8-x_i\\right)$ 个.\n我们称既有红边又有蓝边的三角形为异色二角形, 因每个异色三角形中有 2 个异色角,故 2 色 $K_9$ 中异色三角形的个数为 $\\frac{1}{2} \\sum_{i=1}^9 x_i\\left(8-x_i\\right) \\leqslant \\frac{1}{2} \\sum_{i=1}^9\\left(\\frac{x_i+8-x_i}{2}\\right)^2=\\frac{1}{2} \\times 9 \\times16=72$, 从而 2 色 $K_9$ 中同色三角形至少有 $\\mathrm{C}_9^3-72=84-72=12$ 个.\n12 个三角形共 36 个顶点,但一共只有 9 点, 故由抽庶原理知其中必有 $\\left[\\frac{36-1}{9}\\right]+1=4$ 个三角形有公共顶点, 若这 4 个三角形中有 2 个恰有 1 个公共顶点, 则结论成立, 否则这 4 个三角形有 1 条公共边, 从而这 4 个同色三角形的边都同色, 不妨设 $\\triangle A_1 A_2 A_3, \\triangle A_1 A_2 A_4, \\triangle A_1 A_2 A_5, \\triangle A_1 A_2 A_6$ 都是红色三角形.\n考察以 $A_3, A_4$, $A_5, A_6$ 为顶点的 2 色 $K_4$, 若其中有 1 条红边, 不妨设为 $A_3 A_4$, 则 $\\triangle A_1 A_3 A_4$ 和 $\\triangle A_1 A_2 A_5$ 是两个恰有 1 个公共顶点 $A_1$ 的红色三角形.\n结论成立; 若其中全是蓝色边, 则红色 $\\triangle A_1 A_2 A_3$ 与蓝色 $\\triangle A_3 A_4 A_5$ 是两个恰有一个公共顶点的同色三角形.\n这就证明了 2 色 $K_9$ 中必存在两个恰有一个公共顶点的同色三角形.\n综上知所求最小正整数 $n=9$.", + "remark": "", + "figures": [ + "./images/volume11/figures/fig-c9a3.png" + ] +} \ No newline at end of file diff --git a/processed_dataset/proof/0404.json b/processed_dataset/proof/0404.json new file mode 100644 index 0000000000000000000000000000000000000000..ad9c10fe96a3ca01bb0e77ed7c7512d26b23efb2 --- /dev/null +++ b/processed_dataset/proof/0404.json @@ -0,0 +1,10 @@ +{ + "source_file": "./raw_volume-zh/volume11/exercise9.tex", + "problem_type": "proof", + "problem": "问题4. 致求最小正整数 $n$, 使得任意 $n$ 个人中都存在 4 个人可分成两个恰有 2 个公共成员的 3 人组, 并且每组内的 3 个人都互相认识或互相都不认识.", + "solution": "如图(), 本题等价于求最小正整数 $n$,使得 2 色 $K_n$ 中必存在两个恰有一条公共边的同色三角形.\n如图 $K_9$ 中 (为了清楚起见, 我们把红线 (实线)、蓝线 (虚线) 分别画在两个图中), 有 6 个红色三角形和 6 个蓝色三角形.\n图中 18 条红边和 18 条蓝边恰好每边都是一个同色三角形的一条边, 任何两个同色三角形没有公共边.\n可见, 所求最小正整数 $n \\geqslant 10$, 同第 3 题方法可证 2 色 $K_{10}$ 中至少有 20 个同色三角形.\n每个三角形 3 条边,共 60 条边, 而 $K_{10}$ 中一共只有 $\\mathrm{C}_{10}^3=45$ 条边, 故至少有 $\\left[\\frac{60-1}{45}\\right]+1=2$ 个同色三角形恰有 1 条公共边.\n综上可知, 所求最小正整数 $n=10$.", + "remark": "", + "figures": [ + "./images/volume11/figures/fig-c9a4.png" + ] +} \ No newline at end of file diff --git a/processed_dataset/proof/0405.json b/processed_dataset/proof/0405.json new file mode 100644 index 0000000000000000000000000000000000000000..71598a90d8a945cd1303f333a6d6ba81fb77b535 --- /dev/null +++ b/processed_dataset/proof/0405.json @@ -0,0 +1,8 @@ +{ + "source_file": "./raw_volume-zh/volume11/exercise9.tex", + "problem_type": "proof", + "problem": "问题5. 9 名科学家在国际会议上相遇, 他们中任何 3 人中至少有 2 人能讲同一种语言, 而且每人最多能讲 3 种语言, 证明必存在 3 位科学家,他们能讲同一种语言.", + "solution": "用 9 个点 $A_1, A_2, \\cdots, A_9$ 表示 9 名科学家,若两人能同时讲第 $i$ 种语言.\n则对应点的连线染第 $i$ 种颜色 $(i=1,2, \\cdots)$, 否则对应点的连线不染色.\n(1)若任意两点的连线都染了某种颜色.\n则因为从每点出发的线段至多只有 3 种颜色, 故由抽庶原理知从一点 $A_1$ 出发至少有 $\\left[\\frac{8-1}{3}\\right]+ 1=3$ 条线段同色,不妨设 $A_1 A_2, A_1 A_3, A_1 A_4$ 同色.\n于是 $A_1, A_2, A_3$ 能讲同一种语言.\n(2) 存在两点 $A_1$ 和 $A_2$, 它们之间的连线没有染色.\n由已知条件知任何 3 点中必有 2 点间的连线染了某种颜色.\n因此对其余 7 点中任一点 $A_i(i=3,4$, $5,6,7,8,9) A_i A_1$ 与 $A_i A_2$ 这两条线中至少有一条染了颜色.\n由抽庶原理, 知 $A_1$ 和 $A_2$ 中必有一点, 例如 $A_1$, 从它出发的线段中至少有 $\\left[\\frac{7-1}{2}\\right]+1=4$ 条染了色,但这 4 条线段至多只能染 3 种颜色, 故其中必有 $\\left[\\frac{4-1}{3}\\right]+1=2$ 条同色.\n不妨设 $A_1 A_3, A_1 A_4$ 同色, 于是 $A_1, A_3, A_4$ 能讲同一种语言.", + "remark": "", + "figures": [] +} \ No newline at end of file diff --git a/processed_dataset/proof/0406.json b/processed_dataset/proof/0406.json new file mode 100644 index 0000000000000000000000000000000000000000..a7f91d1b48f2b2ccb4465c33b258787d3eb32907 --- /dev/null +++ b/processed_dataset/proof/0406.json @@ -0,0 +1,8 @@ +{ + "source_file": "./raw_volume-zh/volume11/exercise9.tex", + "problem_type": "proof", + "problem": "问题6. 设有两个完全相同的齿轮 $A$ 和 $B, B$ 被放在一个水平面上, $A$ 放在 $B$ 的上面,并使二者重合 (从而两轮在水平面上的投影完全重合)然后任意敲掉 4 对重合的齿.\n如果两轮原各有 14 个齿, 问能否将 $A$ 轮绕两轮的公共轴旋转到一个适当的位置使两轮在水平面上的投影是一个完整的齿轮的投影?\n如果两轮原各有 13 个齿, 又是怎样呢? 证明你的结论.", + "solution": "两轮各有 14 个齿时, 设 $A(B)$ 轮按顺时针方向每个齿对应一个数 $a_i\\left(b_i\\right), i=1,2, \\cdots, 14$. 对于敲掉的齿, 令 $a_i\\left(b_i\\right)=1$, 对于没有敲掉的齿, 令 $a_i\\left(b_i\\right)=0$. 旋转 $A$ 轮, 使 $A$ 轮的第 1 齿转到与 $B$ 轮的第 $i$ 齿重合, 作和 $S_i=a_1 b_i+a_2 b_{i+1}+\\cdots+a_{14} b_{i+13}\\left(i=1,2, \\cdots, 14, b_{j+14}=b_j\\right)$. 于是 $S_i=0$ 表明两轮的投影是一个完整的齿轮的投影.\n若对任意 $i(1 \\leqslant i \\leqslant 14)$ 有 $S_i \\neq 0$, 则 $S_1=4, S_i \\geqslant 1(2 \\leqslant i \\leqslant 14)$, 于是 $\\sum_{i=1}^{14} S_i \\geqslant 17$. 而 $\\sum_{i=1}^{14} S_i= \\left(\\sum_{i=1}^{14} a_i\\right)\\left(\\sum_{i=1}^{14} b_i\\right)=4 \\times 4=16$, 矛盾.\n故必存在 $i_0\\left(2 \\leqslant i_0 \\leqslant 14\\right)$ 使 $S_{i_0}=0$, 即 $A$ 轮的第 1 齿转到与 $B$ 轮的第 $i_0$ 齿重合时,两轮的投影是一个完整的齿轮的投影.\n两轮原各有 13 齿时,结论不再成立.\n事实上,依次编号后, 假设敲掉的是第 $1,2,5,7$ 对重合的齿.\n并设 $A$ 轮第 $i$ 齿转到 $B$ 轮第 $j$ 齿需要转动的齿数为 $C_{i j}(i, j=1,2,5,7)$, 则可得到下表.\n从表中可以看出, 无论 $A$ 轮转动多少个齿 (从 1 个到 13 个), 都会出现 $A$ 轮被敲掉的齿与 $B$ 轮某个被敲掉的齿重合的情况.\n从而两轮在水平面上的投影不可能是一个完整的齿轮的投影.\n\\begin{tabular}{|c|c|c|c|c|}\n\\hline$C_{i j}$ & 1 & 2 & 5 & 7 \\\\\n\\hline 1 & 13 & 1 & 4 & 6 \\\\\n\\hline 2 & 12 & 13 & 3 & 5 \\\\\n\\hline 5 & 9 & 10 & 13 & 2 \\\\\n\\hline 7 & 7 & 8 & 11 & 13 \\\\\n\\hline\n\\end{tabular}", + "remark": "", + "figures": [] +} \ No newline at end of file diff --git a/processed_dataset/proof/0407.json b/processed_dataset/proof/0407.json new file mode 100644 index 0000000000000000000000000000000000000000..dd66e59f5cc40ccb29433aae5c32625235c374a7 --- /dev/null +++ b/processed_dataset/proof/0407.json @@ -0,0 +1,10 @@ +{ + "source_file": "./raw_volume-zh/volume12/chapter1.tex", + "problem_type": "proof", + "problem": "例1. 某聚会有 605 个人参加, 已知每个人至少和其余的一个人握过手.\n证明: 必有一个人至少和其中的两个人握过手.", + "solution": "证明:将 605 个人用 605 个点 $v_1, v_2, \\cdots, v_{605}$ 表示, 如果其中两个人握过手, 就在相应的顶点之间连一条边.\n本例要证明: 必有一个人至少和其中的两个人握过手.\n倘若不然,则每个人至多和其中一个人握过手,再从题设的每个人至少和其中一个人握过手, 于是便有,每个人恰与其他一个人握过手.\n这样就得出,图 $G$ 恰由若干个两点间连一条边的图形构成(如图()).\n设图 $G$ 有 $r$ 条边, 则 $G$ 便有 $2 r$ (偶数) 个顶点, 这与 $G$ 的顶点数为 605 (奇数)矛盾.", + "remark": "", + "figures": [ + "./images/volume12/figures/fig-c1i5.png" + ] +} \ No newline at end of file diff --git a/processed_dataset/proof/0408.json b/processed_dataset/proof/0408.json new file mode 100644 index 0000000000000000000000000000000000000000..cfdac3cb303ccdd78d450644e67d7194e5456b2e --- /dev/null +++ b/processed_dataset/proof/0408.json @@ -0,0 +1,11 @@ +{ + "source_file": "./raw_volume-zh/volume12/chapter1.tex", + "problem_type": "proof", + "problem": "例4. 九名数学家在一次国际数学会议上相遇, 发现他们中的任意三个人中, 至少有两个人可以用同一种语言对话.\n如果每个数学家至多可说三种语言, 证明至少有三名数学家可以用同一种语言对话.", + "solution": "证明:,如图()用九个点 $v_1, v_2, \\cdots, v_9$ 表示这九名数学家,如果某两个数学家能用第 $i$ 种语言对话, 则在它们相应的顶点之间连一条边并涂以相应的第 $i$种颜色, 这样就得到了一个有九个顶点的简单图 $G$, 它的边涂上了颜色, 每三点之间至少有一条边, 每个顶点引出的边至多有三种不同的颜色.\n要证明的是: 图 $G$ 中存在三个点, 它们两两相邻, 且这三条边具有相同的颜色 (这种三角形称为同色三角形).\n如果边 $\\left(v_i, v_j\\right) 、\\left(v_i, v_k\\right)$ 具有相同的第 $i$ 种颜色, 则按边涂色的意义, 点 $v_j$ 与 $v_k$ 也相邻, 且边 $\\left(v_j, v_k\\right)$ 也具有第 $i$ 种颜色.\n所以对顶点 $v_1$ 来说, 有两种情形:\n(1) 点 $v_1$ 与点 $v_2, \\cdots, v_9$ 都相邻, 根据抽㞕原理知, 至少有两条边, 不妨设为 $\\left(v_1, v_2\\right) 、\\left(v_1, v_3\\right)$, 具有相同的颜色, 从而 $\\triangle v_1 v_2 v_3$ 是同色三角形.\n(2) 点 $v_1$ 与点 $v_2, \\cdots, v_9$ 中的至少一个点不相邻, 不妨设点 $v_1$ 与点 $v_2$\n不相邻.\n由于每三点之间至少有一条边, 所以从 $v_3$, $v_4, \\cdots, v_9$ 发出的, 另一个端点是 $v_1$ 或 $v_2$ 的边至少有 7 条, 由此可知, 点 $v_3, v_4, \\cdots, v_9$ 中至少有 4 个点与点 $v_1$ 或 $v_2$ 相邻, 不妨设点 $v_3, v_4, v_5, v_6$ 与点 $v_1$ 相邻, 如图 $1-12$ 所示.\n于是边 $\\left(v_1, v_3\\right) 、\\left(v_1, v_4\\right) 、\\left(v_1, v_5\\right) 、\\left(v_1\\right.$, $\\left.v_6\\right)$ 中必定有两条具有相同的颜色, 设 $\\left(v_1, v_3\\right) 、\\left(v_1\\right.$, $v_4$ ) 同色,则 $\\triangle v_1 v_3 v_4$ 是同色三角形.\n$v_2 \\circ$", + "remark": "注:: 若把题中的九改成八, 命题就不成立了.\n如图()给出的是一个反例.\n$v_1, v_2, \\cdots, v_8$ 表示 8 个顶点, $1,2, \\cdots, 12$ 表示 12 种颜色, 则图中无同色三角形.", + "figures": [ + "./images/volume12/figures/fig-c1i12.png", + "./images/volume12/figures/fig-c1i13.png" + ] +} \ No newline at end of file diff --git a/processed_dataset/proof/0409.json b/processed_dataset/proof/0409.json new file mode 100644 index 0000000000000000000000000000000000000000..2e8eb77fa65e1f335e0e57d86bf12ef3dde53697 --- /dev/null +++ b/processed_dataset/proof/0409.json @@ -0,0 +1,10 @@ +{ + "source_file": "./raw_volume-zh/volume12/chapter1.tex", + "problem_type": "proof", + "problem": "例5. 有 $n$ 名选手 $A_1, A_2, \\cdots, A_n$ 参加数学竞赛, 其中有些选手是互相认识的, 而且任何两个不相识的选手都恰好有两个共同的熟人.\n若已知选手 $A_1$ 与 $A_2$ 互相认识,但他俩没有共同的熟人,证明他俩的熟人一样多.", + "solution": "证明:用 $n$ 个点 $v_1, v_2, \\cdots, v_n$ 表示这 $n$ 名选手 $A_1, A_2, \\cdots, A_n$, 如果两个选手互相认识, 那么就在相应的两个点之间连一条边, 这样就得到一个简单图 $G$. 图 $G$ 中的顶点满足: 任意两个不相邻的顶点都恰好有两个共同相邻的顶点.\n要证明的是相邻的两个顶点 $v_1$ 与 $v_2$ 各自引出的边的条数一样多.\n如果记与 $v_1$ 相邻的顶点集合为 $N\\left(v_1\\right)$, 与 $v_2$ 相邻的顶点集合为 $N\\left(v_2\\right)$. 若在 $N\\left(v_1\\right)$ 中除 $v_2$ 外还有点 $v_i$, 则 $v_i \\notin N\\left(v_2\\right)$. 否则 $A_1$ 与 $A_2$ 有共同熟人 $A_i$. 于是 $v_2$ 与 $v_i$ 除 $v_1$ 外还应有一个与它们共同相邻的点 $v_j$, 则 $v_j \\in N\\left(v_2\\right)$. 如图() 所示.\n对于 $N\\left(v_1\\right)$ 中不同于 $v_2$ 的点 $v_i 、 v_k$, 它们不可能与 $N\\left(v_2\\right)$ 中除 $v_1$ 外的一个点 $v_j$ 都相邻 (否则, 两个不相邻的顶点 $v_1$, $v_j$ 有三个共同相邻的顶点 $v_2, v_i, v_k$ ). 因而, 对于 $N\\left(v_1\\right)$ 中不同于 $v_i$ 的 $v_k$, 必在 $N\\left(v_2\\right)$ 中存在不同于 $v_j$ 的相邻点 $v_l$. 由此可得 $N\\left(v_1\\right)$ 中的顶点个数不大于 $N\\left(v_2\\right)$ 中的顶点个数.\n同样地, $N\\left(v_2\\right)$ 中的顶点个数不大于 $N\\left(v_1\\right)$ 中的顶点个数.\n于是, 从点 $v_1$ 与 $v_2$ 引出的边的条数是相等的.", + "remark": "", + "figures": [ + "./images/volume12/figures/fig-c1i14.png" + ] +} \ No newline at end of file diff --git a/processed_dataset/proof/0410.json b/processed_dataset/proof/0410.json new file mode 100644 index 0000000000000000000000000000000000000000..a86e71a1e2f006ec0dfd0f0749b77c8aa08b4d38 --- /dev/null +++ b/processed_dataset/proof/0410.json @@ -0,0 +1,8 @@ +{ + "source_file": "./raw_volume-zh/volume12/chapter2.tex", + "problem_type": "proof", + "problem": "例1. 是否存在这样的多面体,它有奇数个面,每个面有奇数条棱?", + "solution": "解:不存在这样的多面体.\n事实上, 如果这样的多面体存在, 那么用顶点表示这个多面体的面, 并且仅当 $v_i, v_j$ 所代表的两个面有公共棱时, 在图 $G$ 相应的两顶点之间连一条边, 依题意 $d(v)$ 是奇数, 于是奇数个奇数也是奇数.\n与定理相违.\n证毕.", + "remark": "", + "figures": [] +} \ No newline at end of file diff --git a/processed_dataset/proof/0411.json b/processed_dataset/proof/0411.json new file mode 100644 index 0000000000000000000000000000000000000000..ef091837761cce3040c7dcc6ba52d707eb957b9a --- /dev/null +++ b/processed_dataset/proof/0411.json @@ -0,0 +1,11 @@ +{ + "source_file": "./raw_volume-zh/volume12/chapter2.tex", + "problem_type": "proof", + "problem": "例2. 如图所示, 大三角形的三个顶点分别涂以 $A 、 B 、 C$ 三种颜色.\n在大三角形内取若干个点, 将它分为若干个小三角形, 每两个小三角形或者有一条公共边, 或者有一个公共点, 或者完全没有公共点.\n将每个小三角形的顶点也分别涂以 $A 、 B 、 C$ 三种颜色之一, 证明不管怎样涂色, 都有一个小三角形, 它的三个顶点的颜色全不相同.", + "solution": "证明:在大三角形外及小三角形内部各取一点作顶点, 当两个面有一条公共边 $A B$ 时, 就在相应的两个顶点之间连一条边, 得图 $G^{\\prime}$, 如图() 所示.\n一个具有颜色 $A 、 B 、 C$ 顶点的小三角形对应于 $G^{\\prime}$ 中的度为 1 的顶点.\n其余的小三角形均对应于 $G$ 中度为 0 或 2 的顶点.\n由于大三角形外部的一个顶点 $u$ 的度是 1 , 且奇顶点的个数为偶数, 所以 $G^{\\prime}$ 中除了 $u$ 外, 至少还有一个奇顶点 $v$. 这就是说如图() 中至少有一个小三角形, 它的三个顶点分别为 $A 、 B 、 C$ 三种颜色.", + "remark": "注:: 本题常见的做法是赋值法, 结合奇偶分析来解决.\n这里的解法有图论的独到之处,非常简洁.", + "figures": [ + "./images/volume12/figures/fig-c2i3.png", + "./images/volume12/figures/fig-c2i3.png" + ] +} \ No newline at end of file diff --git a/processed_dataset/proof/0412.json b/processed_dataset/proof/0412.json new file mode 100644 index 0000000000000000000000000000000000000000..1303ed4a89ca1aa648f5e8c1db133d924832f73c --- /dev/null +++ b/processed_dataset/proof/0412.json @@ -0,0 +1,8 @@ +{ + "source_file": "./raw_volume-zh/volume12/chapter2.tex", + "problem_type": "proof", + "problem": "例3. 某地区网球俱乐部的 20 名成员举行 14 场单打比赛, 每人至少上场一次.\n证明: 必有六场比赛, 其中 12 个参赛者各不相同.", + "solution": "证明:用 20 个顶点 $v_1, v_2, \\cdots, v_{20}$ 代表 20 名成员,两名选手比赛过, 则在相应的顶点之间连一条边, 得图 $G$.\n图 $G$ 中有 14 条边, 设各顶点的度为 $d_i, i=1,2, \\cdots, 20$. 由题意知 $d_i \\geqslant$ 1. 根据定理一\n$$\nd_1+d_2+\\cdots+d_{20}=2 \\times 14=28 .\n$$\n在每个顶点 $v_i$ 处抹去 $d_i-1$ 条边, 由于一条边可能同时被其两端点抹去, 所以抹去的边数不超过\n$$\n\\left(d_1-1\\right)+\\left(d_2-1\\right)+\\cdots+\\left(d_{20}-1\\right)=28-20=8 .\n$$\n故抹去了这些边后所得的图 $G^{\\prime}$ 中至少还有 $14-8=6$ 条边, 并且 $G^{\\prime}$ 中每个顶点的度至多是 1 . 从而这 6 条边所相邻的 12 个顶点是各不相同的.\n即这 6 条边所对应的 6 场比赛的参赛者各不相同.", + "remark": "", + "figures": [] +} \ No newline at end of file diff --git a/processed_dataset/proof/0413.json b/processed_dataset/proof/0413.json new file mode 100644 index 0000000000000000000000000000000000000000..7f555d58708632fbe6a10a93cf7299626aa7940b --- /dev/null +++ b/processed_dataset/proof/0413.json @@ -0,0 +1,8 @@ +{ + "source_file": "./raw_volume-zh/volume12/chapter2.tex", + "problem_type": "proof", + "problem": "例4. 设 $S=\\left\\{x_1, x_2, \\cdots, x_n\\right\\}$ 是平面上的点集, 其中任意两点之间的距离至少是 1 , 证明: 最多有 $3 n$ 对点,每对点的距离恰好是 1 .", + "solution": "证明:取这 $n$ 个点作为顶点,两顶点相邻当且仅当两点之间的距离为 1 , 得一个图 $G . G$ 中的边数记为 $e$.\n显然图 $G$ 中和顶点 $x_i$ 相邻的点是在以 $x_i$ 为圆心, 半径为 1 的圆周上.\n由于集 $S$ 中任意两点之间的距离 $\\geqslant 1$, 故圆周上至多含有 $S$ 中的 6 个点, 所以 $d\\left(x_i\\right) \\leqslant 6$.\n对图 $G$ 用定理一, 有\n$$\n\\begin{gathered}\nd\\left(x_1\\right)+d\\left(x_2\\right)+\\cdots+d\\left(x_n\\right)=2 e, \\\\\n6 n \\geqslant 2 e,\n\\end{gathered}\n$$\n即 $e \\leqslant 3 n$. 就是说图 $G$ 中的边数 $e$ 不超过 $3 n$. 所以这 $n$ 个点中至多有 $3 n$ 对点,每对点的距离恰好是 1 .", + "remark": "", + "figures": [] +} \ No newline at end of file diff --git a/processed_dataset/proof/0414.json b/processed_dataset/proof/0414.json new file mode 100644 index 0000000000000000000000000000000000000000..01444603c98897325931b42b394fc9f59373eb42 --- /dev/null +++ b/processed_dataset/proof/0414.json @@ -0,0 +1,12 @@ +{ + "source_file": "./raw_volume-zh/volume12/chapter2.tex", + "problem_type": "proof", + "problem": "例7. 某俱乐部共有 99 名成员, 每一个成员都声称只愿意和自己认识的人一起打桥牌.\n已知每个成员都至少认识 67 名成员.\n证明一定有 4 名成员, 他们可以在一起打桥牌.", + "solution": "证法一,如图(), 作一个图 $G$ : 用 99 个点表示 99 名成员,如果两名成员相互认识, 就在相应的两个顶点之间连一条边.\n已知条件是: 对任意顶点 $v, d(v) \\geqslant$ 67. 欲证 $G$ 中含有一个 4 阶完全图 $K_4$.\n在 $G$ 中任取一个顶点 $u$, 由于 $d(u) \\geqslant$ 67 , 所以存在顶点 $v$, 使得与 $v$ 相邻且与 $u$ 不相邻的顶点至多为 $(99-1-67=) 31$ 个.\n同样,与 $v$ 不相邻且与 $u$ 相邻的顶点也至多 31 个.\n于是图 $G$ 中至少有 $(99-31- 31-2=) 35$ 个顶点和 $u, v$ 均相邻.\n如图() 所示, 设顶点 $x$ 和顶点 $u, v$ 均相邻.\n由于 $d(x) \\geqslant 67$, 并且 $G$ 中至多只有 $(31+31+2=) 64$ 个不同时和 $u, v$ 均相邻的顶点, 因此顶点 $x$ 至少还和一个与 $u, v$ 均相邻的顶点 $y$ 相邻.\n从而 $u, v, x, y$ 是 4 个两两相邻的顶点.\n于是命题得证.\n证法二用顶点表示成员, 如果两个人不认识就在相应的顶点之间连一条边, 得图 $G^{\\prime}$. 由于每个人认识的人数不少于 67 , 所以对每个顶点 $v$, 都有 $d(v) \\leqslant 99-1-67=31$. 要证明的是: $G^{\\prime}$ 中存在四个点, 两两之间不相邻.\n对于顶点 $u$, 取一个不与它相邻的顶点 $v$, 则剩下的 97 个顶点中与 $u$ 或 $v$ 相邻的顶点个数不超过\n$$\nd(u)+d(v) \\leqslant 31+31=62,\n$$\n因而存在与 $u, v$ 均不相邻的顶点 $x$, 与顶点 $u, v, x$ 中至少有一个相邻的顶点个数不超过\n$$\nd(u)+d(v)+d(x) \\leqslant 3 \\times 31=93,\n$$\n所以在剩下的 96 个点中, 必有一个点 $y$ 与 $u, v, x$ 均不相邻, 于是 $u, v, x, y$ 所代表的 4 个人是互相认识的, 他们可以在一起打桥牌.", + "remark": "注:1: 若将题中的 67 人改为 66 人,则不一定能找出 4 个互相认识的人来.\n反例如图()所示.\n将顶点集 $V$ 分成三个子集 $\\left\\{v_1\\right.$, $\\left.v_2, \\cdots, v_{33}\\right\\},\\left\\{v_{34}, v_{35}, \\cdots, v_{66}\\right\\},\\left\\{v_{67}\\right.$, $\\left.v_{68}, \\cdots, v_{99}\\right\\}$. 同一个子集中任意两顶点均不相邻, 不同子集中的任意两点均相邻.\n显然每个顶点的度都是 66 , 任意 4 点中, 至少有 2 点属于同一子集, 从而它们不相邻.\n也就是说图中不存在两两相邻的 4 顶点.\n注:2: 本题可推广为:\n俱乐部有 $n(n \\geqslant 4)$ 人, 其中每人都至少认识其中的 $\\left[\\frac{2 n}{3}\\right]+1$ 个人, 则在这 $n$ 个人中必定可以找到 4 个人, 他们是两两认识的.\n注:3: 如果 $G$ 是 $n$ 阶简单图, 从完全图 $K_n$ 中把属于 $G$ 的边全部去掉后, 得到的图称为 $G$ 的补图, 通常记为 $\\bar{G}$. 证法一中的图 $G$ 与证法二中的图 $G^{\\prime}$, 互为补图.", + "figures": [ + "./images/volume12/figures/fig-c2i4.png", + "./images/volume12/figures/fig-c2i4.png", + "./images/volume12/figures/fig-c2i5.png" + ] +} \ No newline at end of file diff --git a/processed_dataset/proof/0415.json b/processed_dataset/proof/0415.json new file mode 100644 index 0000000000000000000000000000000000000000..2eef3926506a53c83b546b29012aba75ae588743 --- /dev/null +++ b/processed_dataset/proof/0415.json @@ -0,0 +1,8 @@ +{ + "source_file": "./raw_volume-zh/volume12/chapter2.tex", + "problem_type": "proof", + "problem": "例8. 设在平面上有 $n$ 个给定的点.\n求证其中距离为 1 的点的对数不超过 $\\frac{n}{4}+\\frac{\\sqrt{2}}{2} n^{\\frac{3}{2}}$.", + "solution": "证把 $n$ 个点视为图 $G$ 的顶点, 记 $V=\\left\\{v_1, v_2, \\cdots, v_n\\right\\}$ 为图 $G$ 的顶点集, 在距离为 1 的两点之间连一边, 则由定理一,\n$$\n2 e=d\\left(v_1\\right)+d\\left(v_2\\right)+\\cdots+d\\left(v_n\\right) .\n$$\n用 $C_i$ 表示以 $v_i$ 为圆心、半径为 1 的圆, 这 $n$ 个圆两两交点总数不超过 $2 \\mathrm{C}_n^2=n(n-1)$ 个.\n另一方面, 若 $v_k, v_j$ 与 $v_i$ 相邻, 则 $v_i \\in C_k \\cap C_j$, 因此, $v_i$ 作为 $C_1, C_2, \\cdots$, $C_n$ 中两圆的交点恰好被计数了 $\\mathrm{C}_{d\\left(v_i\\right)}^2$ 次, 故\n$$\n\\mathrm{C}_{d\\left(v_1\\right)}^2+\\mathrm{C}_{d\\left(v_2\\right)}^2+\\cdots+\\mathrm{C}_{d\\left(v_n\\right)}^2 \\leqslant 2 \\mathrm{C}_n^2=n(n-1) . \\label{eq1}\n$$\n由 Cauchy 不等式,有\n$$\n\\mathrm{C}_{d\\left(v_1\\right)}^2+\\mathrm{C}_{d\\left(v_2\\right)}^2+\\cdots+\\mathrm{C}_{d\\left(v_n\\right)}^2 \\geqslant \\frac{2}{n} e^2-e . \\label{eq2}\n$$\n由 式\\ref{eq1}, \\ref{eq2}式得\n$$\n\\frac{2}{n} e^2-e \\leqslant n(n-1)\n$$\n即\n$$\n2 e^2-n e-n^2(n-1) \\leqslant 0 .\n$$\n解得\n$$\ne \\leqslant \\frac{n}{4}+\\frac{\\sqrt{2}}{2} n^{\\frac{3}{2}}\n$$", + "remark": "", + "figures": [] +} \ No newline at end of file diff --git a/processed_dataset/proof/0416.json b/processed_dataset/proof/0416.json new file mode 100644 index 0000000000000000000000000000000000000000..6631a74e77354189dbe75dd05ffd7809e77b6bd9 --- /dev/null +++ b/processed_dataset/proof/0416.json @@ -0,0 +1,8 @@ +{ + "source_file": "./raw_volume-zh/volume12/chapter3.tex", + "problem_type": "proof", + "problem": "例1. 设图 $G$ 有 20 个顶点, 101 条边.\n证明 $G$ 中一定有两个具有公共边的三角形.", + "solution": "证明:可将 20 改为更一般的自然数 $2 n(n \\geqslant 2)$, 用数学归纳法证明: 图 $G$ 有 $2 n(n \\geqslant 2)$ 个顶点, $n^2+1$ 条边, 则 $G$ 中一定有两个具有公共边的三角形.\n当 $n=2$ 时, $G$ 有 4 个顶点, 5 条边, 作完全图 $K_4, K_4$ 有 $\\mathrm{C}_4^2=6$ 条边, 容易验证不论在 $K_4$ 中去掉哪条边, 总有两个具有公共边的三角形, 即命题在 $n=2$ 时成立.\n假设命题在 $n=k(k \\geqslant 2)$ 时成立.\n设 $G$ 有 $2(k+1)$ 个顶点 $v_1, v_2, \\cdots$, $v_{2 k+2},(k+1)^2+1=k^2+2 k+2$ 条边.\n因为\n$$\n\\left[\\frac{(2 k+2)^2}{4}\\right]=\\left[k^2+2 k+1\\right]) 所示的是 $T_3(5), e_3(5)=8 . e_m(n)$ 的计算公式如下,证明留作习题.\n$$\ne_m(n)=\\mathrm{C}_{n-k}^2+(m-1) \\mathrm{C}_{k+1}^2 \\text {, 其中 } k=\\left[\\frac{n}{m}\\right] .\n$$\n若 $G=\\left(V_1, V_2, \\cdots, V_m ; E\\right)$ 是任一 $n$ 阶 $m$ 部图, 令 $p_i=\\left|V_i\\right|\\left(\\sum_{i=1}^m p_i=n\\right)$, 可以验证 $G$ 的边数 $\\leqslant e_m(n)$, 并且当等号成立时必有 $G$ 与 $T_m(n)$ 同构 . 换句话说, $T_m(n)$ 是包含边数最多的 $n$ 阶 $m$ 部图, 并且是唯一的这样的图.\n显然任意一个 $m$ 部图不含 $K_{m+1}$. 托兰进一步证明了 $T_m(n)$ 是边数最多的、不含 $K_{m+1}$ 的 $n$ 阶图, 并且是唯一的这样的图.\n定理二设 $n$ 阶图 $G$ 不含 $K_{m+1}$, 则 $G$ 的边数 $e(G) \\leqslant e_m(n)$; 当且仅当 $G$ 和 $T_m(n)$ 同构时等号成立.\n这便是托兰定理, 证明这里略去.\n有兴趣的读者可参阅 $\\mathrm{J} \\cdot \\mathrm{A}$ - 邦迪和 $\\mathrm{U} \\cdot \\mathrm{S} \\cdot \\mathrm{R} \\cdot$ 默蒂著的《图论及其应用》.", + "figures": [ + "./images/volume12/figures/fig-c3i3.png" + ] +} \ No newline at end of file diff --git a/processed_dataset/proof/0418.json b/processed_dataset/proof/0418.json new file mode 100644 index 0000000000000000000000000000000000000000..520fd5fe72126d78374a5cb313e7a4ec6da0d1cc --- /dev/null +++ b/processed_dataset/proof/0418.json @@ -0,0 +1,13 @@ +{ + "source_file": "./raw_volume-zh/volume12/chapter3.tex", + "problem_type": "proof", + "problem": "例3. 设 $A_1, A_2, A_3, A_4, A_5, A_6$ 是平面上的 6 点, 其中任意三点不共线.\n(i) 如果这些点之间任意连接 13 条线段, 证明: 必存在 4 点, 它们每两点之间都有线段连接.\n(ii)如果这些点之间只有 12 条线段, 请你画一个图形, 说明(i) 的结论不成立(不必用文字说明).\n(iii) 结论 (i) 能否加强为: 必存在 4 个 4 阶完全图, 给出反例或证明.", + "solution": "解:(i) 把题目转化成图论语言就是: 图 $G$ 有 6 个顶点, 13 条边, 证明 $G$ 中含有 $K_4$.\n容易算得 $e_4(6)=12<13$, 根据定理二, $G$ 中必含有 $K_4$.\n(ii) 构造完全 3 部图 $K_{2,2,2}$, 如图() 所示.\n因从 $K_{2,2,2}$ 中任取 4 点, 总有两点属于同一部分, 而这两点是不相邻的, 因此任取 4 点均不构成 $K_4$.", + "remark": "注:: 对于 (i), 不用定理二当然也能证明, 而且方法很多,这里仅举两种.\n(1) 因为 6 个点的度数之和 $=2 \\times 13=26$, 所以这 6 个点中至少有两个点的度数为 5 (否则, 度数之和 $\\leqslant 5+5 \\times 4=25<26$ ), 不妨设 $d\\left(A_1\\right)= d\\left(A_2\\right)=5$. 与 $A_1$ 或 $A_2$ 关联的边共 9 条, 如图() 所示.\n于是在 $A_3, A_4$, $A_5, A_6$ 之间还有 $13-9=4$ 条边.\n这 4 条边的任一条的两个端点与 $A_1, A_2$ 这 4 点构成 $K_4$.\n(2) 因为 6 阶完全图有 15 条边, 所以图 $G$ 就是在 $K_6$ 中去掉两条边.\n分两种情况讨论:\n(1) 两边有公共点, 如图(), 则四点组 $A_2, A_4, A_5, A_6$ 组成 $K_4$;\n(2) 两边无公共点, 如图(), 则四点组 $A_1, A_3, A_5, A_6$ 组成 $K_4$.\n(iii)按上述两种情况来讨论: 情况(1)中有 6 个 $K_4$ 组 $\\left(A_1, A_4, A_5, A_6\\right)$, $\\left(A_2, A_4, A_5, A_6\\right),\\left(A_3, A_4, A_5, A_6\\right),\\left(A_2, A_3, A_4, A_5\\right),\\left(A_1, A_3, A_4\\right.$,\n$\\left.A_6\\right),\\left(A_1, A_3, A_5, A_6\\right)$.\n情况(2)中有 4 个 $K_4$ 组 $\\left(A_1, A_3, A_5, A_6\\right),\\left(A_1, A_4, A_5, A_6\\right),\\left(A_1\\right.$, $\\left.A_3, A_4, A_5\\right),\\left(A_1, A_2, A_4, A_5\\right)$.\n所以,必存在 4 个 4 阶完全图.", + "figures": [ + "./images/volume12/figures/fig-c3i4.png", + "./images/volume12/figures/fig-c3i5.png", + "./images/volume12/figures/fig-c3i6.png", + "./images/volume12/figures/fig-c3i7.png" + ] +} \ No newline at end of file diff --git a/processed_dataset/proof/0419.json b/processed_dataset/proof/0419.json new file mode 100644 index 0000000000000000000000000000000000000000..da85bbd00c1b3136b13f0a3a0470eca2fe0a32f0 --- /dev/null +++ b/processed_dataset/proof/0419.json @@ -0,0 +1,10 @@ +{ + "source_file": "./raw_volume-zh/volume12/chapter3.tex", + "problem_type": "proof", + "problem": "例4. 在有 8 个顶点的简单图中, 没有四边形(即由四点 $A, B, C, D$ 和四条边 $A B, B C, C D, D A$ 组成的图) 的图的边数的最大值是多少?", + "solution": "解:边数的最大值为 11 .\n首先, 如图() 所示, 图中有 8个顶点和 11 条边,但其中没有四边形.\n下面我们证明: 若一个简单图有 12 条边, 则其中一定含有四边形.\n首先指出两个明显的事实:\n(a) 设 $A \\neq B$ 是两个顶点.\n如果点 $A$ 与点 $C_1, \\cdots, C_k$ 均有边相连, $B$ 至少与 $\\left\\{C_1, \\cdots, C_k\\right\\}$ 中的两点分别有边相连, 则图中必有四边形.\n(b) 如果 4 点之间连有 5 条边, 则图中必有四边形.\n设有 8 个顶点, 12 条边的图中没有四边形, 其中点 $A$ 是引出边最多的顶点之一.\n(1) 设 $A$ 共引出 $s \\geqslant 5$ 条边, 与 $A$ 有边相连的顶点的集合为 $S$, 除点 $A$ 及 $S$ 中的点之外的所有顶点的集合记为 $T$. 于是由 (a) 和 (b) 知, $S$ 中点之间连线数不超过 $\\left[\\frac{s}{2}\\right], T$ 中点之间连线数至多为 $\\mathrm{C}_{|T|}^2, S$ 与 $T$ 之间连线数至多为 $|T|$. 因而, 图中连线总数至多为\n$$\ns+\\left[\\frac{s}{2}\\right]+|T|+\\mathrm{C}_{|T|}^2=7+\\left[\\frac{s}{2}\\right]+\\mathrm{C}_{|T|}^2 .\n$$\n当 $s \\geqslant 5$ 时,边数小于 12 ,矛盾.\n(2) 点 $A$ 恰引出 4 条边: $A A_j(j=1,2,3,4)$. 设另外 3 点是 $B_1, B_2$, $B_3$. 于是 $\\left\\{A_1, A_2, A_3, A_4\\right\\}$ 之间至多两条边, $\\left\\{B_1, B_2, B_3\\right\\}$ 之间至多 3 条边, 这两个点集之间至多 3 条边.\n因为图中共有 12 条边, 故知 3 类边数恰分别为 $2,3,3$. 不妨设第 3 组的 3 条线为 $A_j B_j(j=1,2,3)$. 因为第 1 组有两条边且二者没有公共端点, 故 $\\left\\{A_1, A_2, A_3\\right\\}$ 之间有一条边, 不妨设为 $A_1 A_2$, 于是 $A_1 A_2 B_2 B_1$ 为四边形,矛盾.\n(3) 点 $A$ 恰引出 3 条线, 从而每点都引出 3 条线.\n设点 $A$ 和 $B$ 之间没有连线, 两点各引出的 3 条线分别为 $A A_j, B B_j(j=1,2,3)$. 于是由 (a) 知\n$\\left\\{A_1, A_2, A_3\\right\\}$ 与 $\\left\\{B_1, B_2, B_3\\right\\}$ 至多有 1 个公共点.\n如果二者没有公共点, 则它们的各 3 点间都至多有一条边, 两个三点集之间至多有 3 条连线.\n从而图中连线总数至多为 11 ,矛盾.\n如果两个三点集之间恰有 1 个公共点, 则考察第 8 点 $C$. 由抽屉原理知, 它引出的 3 条线中必有两条引向同一个三点集, 这导致四边形,矛盾.\n综上,我们证明了在有 8 个顶点和 12 条边的图中必有四边形.\n从而, 所求的边数的最大值为 11 .", + "remark": "", + "figures": [ + "./images/volume12/figures/fig-c3i8.png" + ] +} \ No newline at end of file diff --git a/processed_dataset/proof/0420.json b/processed_dataset/proof/0420.json new file mode 100644 index 0000000000000000000000000000000000000000..53a542ef4cb6345aaea26fa5967156ebebb5940c --- /dev/null +++ b/processed_dataset/proof/0420.json @@ -0,0 +1,8 @@ +{ + "source_file": "./raw_volume-zh/volume12/chapter3.tex", + "problem_type": "proof", + "problem": "例5. 证明:设 $G$ 是 $n$ 阶简单图, $G$ 中不含四边形, 则其边数\n$$\ne \\leqslant \\frac{1}{4} n(1+\\sqrt{4 n-3}) .\n$$", + "solution": "证设 $V=\\left\\{v_1, v_2, \\cdots, v_n\\right\\}$ 是图 $G$ 的顶点集,对于任意的顶点 $v_i \\in V$, 与 $v_i$ 相邻的顶点对 $\\{x, y\\}$ 有 $\\mathrm{C}_{d\\left(v_i\\right)}^2$ 个.\n由于图 $G$ 中没有四边形, 所以, 当 $v_i$ 在 $V$ 中变化时, 所有的顶点对 $\\{x, y\\}$ 都是互不相同的, 否则, 点对 $\\{x, y\\}$ 分别在 $\\mathrm{C}_{d\\left(v_i\\right)}^2$ 和 $\\mathrm{C}_{d\\left(v_j\\right)}^2$ 中被计数, 那么 $v_i, x, v_j, y$ 就组成一个四边形.\n所以\n$$\n\\sum_{i=1}^n \\mathrm{C}_{d\\left(v_i\\right)}^2 \\leqslant \\mathrm{C}_n^2\n$$\n由柯西不等式,有\n$$\n\\begin{aligned}\n\\sum_{i=1}^n \\mathrm{C}_{d\\left(v_i\\right)}^2 & =\\frac{1}{2} \\sum_{i=1}^n d^2\\left(v_i\\right)-e \\\\\n& \\geqslant \\frac{1}{2} \\cdot \\frac{1}{n}\\left(\\sum_{i=1}^n d\\left(v_i\\right)\\right)^2-e \\\\\n& =\\frac{2}{n} e^2-e,\n\\end{aligned}\n$$\n所以\n$$\n\\begin{gathered}\n\\frac{2}{n} e^2-e \\leqslant \\mathrm{C}_n^2, \\\\\ne^2-\\frac{n}{2} e-\\frac{1}{4} n^2(n-1) \\leqslant 0, \\\\\ne \\leqslant \\frac{n}{4}(1+\\sqrt{4 n-3}) .\n\\end{gathered}\n$$\n解得说明 本题得到了一个 $n$ 阶简单图不含四边形, 其边数的一个上界.\n但这还不是最大值, 对于一般的 $n$, 边数的最大值还有待于进一步的研究.", + "remark": "", + "figures": [] +} \ No newline at end of file diff --git a/processed_dataset/proof/0421.json b/processed_dataset/proof/0421.json new file mode 100644 index 0000000000000000000000000000000000000000..359cd22d1ff32476da80ca88c6dad7cac376f780 --- /dev/null +++ b/processed_dataset/proof/0421.json @@ -0,0 +1,11 @@ +{ + "source_file": "./raw_volume-zh/volume12/chapter3.tex", + "problem_type": "proof", + "problem": "例6. 由 $n$ 个点和这些点之间的 $l$ 条线段组成一个空间图形, 其中 $n= q^2+q+1, l \\geqslant \\frac{1}{2} q(q+1)^2+1, q \\geqslant 2, q \\in \\mathbf{N}$.\n已知此图中任意四点不共面, 每点至少有一条连线段, 存在一点至少有 $q+2$ 条连线段.\n证明: 图中必存在一个空间四边形(即由四点 $A, B, C, D$ 和四条连线段 $A B, B C, C D, D A$ 组成的图形).", + "solution": "证本题条件中任意四点不共面实际是为了保证无三点共线, 故从图论角度看, 只需证明图中存在四边形即可.\n解答本题可以援引例 5 思路, 但直接应用不可行.\n考虑将与 $d\\left(v_1\\right) \\geqslant q+2$ 的点 $v_1$ 相连的 $d\\left(v_1\\right)$ 个点去掉, 剩下的点对有 $\\mathrm{C}_{n-d\\left(v_1\\right)}^2$ 个.\n如例 5 , 没有四边形时 $\\mathrm{C}_{n-d(v)}^2 \\geqslant \\sum_{i=2}^n \\mathrm{C}_{d\\left(v_i\\right)-1}^2$.\n同样 $\\sum_{i=2}^n\\left(d\\left(v_i\\right)-1\\right)=2 l-n+1-d(v)$.\n结合 Cauchy 不等式得\n$$\n\\begin{aligned}\n& {\\left[n-d\\left(v_1\\right)\\right]\\left[n-d\\left(v_1\\right)-1\\right] } \\\\\n2 & \\frac{1}{2}\\left\\{\\sum_{i=2}^n\\left[n-d\\left(v_i\\right)\\right]^2-\\sum_{i=2}^n\\left[n-d\\left(v_i\\right)\\right]\\right\\} \\\\\n\\geqslant & \\frac{1}{2}\\left\\{\\frac{1}{n-1}\\left[2 l-n+1-d\\left(v_1\\right)\\right]^2-\\left[2 l-n+1-d\\left(v_i\\right)\\right]\\right\\}\n\\end{aligned}\n$$\n即\n$$\n\\begin{aligned}\n& (n-1)\\left[n-d\\left(v_1\\right)\\right]\\left[n-d\\left(v_1\\right)-1\\right] \\\\\n\\geqslant & {\\left[2 l-n+1-d\\left(v_1\\right)\\right]\\left[2 l-2 n+2-d\\left(v_1\\right)\\right] } \\\\\n\\geqslant & {\\left[q^3+q^2-d\\left(v_1\\right)+2\\right]\\left[q^3-q+2-d\\left(v_1\\right)\\right] } \\\\\n= & {\\left[n q-q+2-d\\left(v_1\\right)\\right]\\left[n q-q-n+3-d\\left(v_1\\right)\\right] }\n\\end{aligned}\n$$\n这与 $(q+1)\\left[n-d\\left(v_1\\right)\\right]) 所示, $S$ 的直径为 1 . 易知点对 $\\left(x_1, x_3\\right),\\left(x_2, x_4\\right),\\left(x_3, x_5\\right),\\left(x_4, x_6\\right)$, $\\left(x_5, x_1\\right),\\left(x_6, x_2\\right)$ 的距离为 $\\frac{\\sqrt{3}}{2}$. 所以在这个直径为 1 的点集中存在 9 个点对, 其距离大于 $\\frac{\\sqrt{2}}{2}$.\n但是 9 并非是在 6 个点上所能做到的最好答案.\n如果按图() 所示来安排这 6 个点的话(其中点 $x_1, x_3, x_5$ 构成边长为 1 的正三角形, 点 $x_2, x_4$, $x_6$ 构成边长为 0.8 , 中心与 $\\triangle x_1 x_3 x_5$ 重合, 边分别与 $\\triangle x_1 x_3 x_5$ 平行的正三角形), 则除了点对 $\\left(x_1, x_2\\right),\\left(x_3, x_4\\right),\\left(x_5, x_6\\right)$ 外, 其余的点对的距离均大于 $\\frac{\\sqrt{2}}{2}$. 因此我们有 $\\mathrm{C}_6^2-3=12$ 个点对, 其距离大于 $\\frac{\\sqrt{2}}{2}$. 事实上, 这是我们所能做到的最好答案.", + "remark": "", + "figures": [ + "./images/volume12/figures/fig-c3i9.png", + "./images/volume12/figures/fig-c3i10.png" + ] +} \ No newline at end of file diff --git a/processed_dataset/proof/0422.json b/processed_dataset/proof/0422.json new file mode 100644 index 0000000000000000000000000000000000000000..64e8af5d80c5ee7afeca59b310944ef40948e626 --- /dev/null +++ b/processed_dataset/proof/0422.json @@ -0,0 +1,8 @@ +{ + "source_file": "./raw_volume-zh/volume12/chapter4.tex", + "problem_type": "proof", + "problem": "例1. 若 $T$ 是树, 则\n(i) $T$ 是连通图, 但 $T$ 中去掉任一条边后, 所得的图 $G$ 不连通.\n(ii) $T$ 无圈, 但添加任何一条边后, 得到的图 $G$ 便包含一个且仅包含一个圈.\n反之, 若图 $T$ 满足(i) 或(ii), 则 $T$ 是树.", + "solution": "证明:(i) 若图 $G$ 是连通的,则 $G$ 仍然是树, 所以 $G$ 有 $n-1$ 条边, 与 $T$ 的边数相等,矛盾.\n(ii) 若 $G$ 没有圈, 则 $G$ 仍是树, 故 $G$ 有 $n-1$ 条边, 与 $T$ 的边数相同, 这是不可能的,所以 $G$ 含有圈.\n显然 $G$ 仅含一个圈.\n本题刻画了树的一个特征, 在点数给定的所有图中, 树是边数最少的连通图, 也是边数最多的无圈图.\n由此可见, 在任意一个图 $G$ 中, 若 $en-1$, 则 $G$ 必有圈.\n树的另一个特征也是很有用的.", + "remark": "", + "figures": [] +} \ No newline at end of file diff --git a/processed_dataset/proof/0423.json b/processed_dataset/proof/0423.json new file mode 100644 index 0000000000000000000000000000000000000000..17a43c39f1f12d77f845a97c942d73e375204bb8 --- /dev/null +++ b/processed_dataset/proof/0423.json @@ -0,0 +1,8 @@ +{ + "source_file": "./raw_volume-zh/volume12/chapter4.tex", + "problem_type": "proof", + "problem": "例2. 设 $T$ 是树, 则 $T$ 中任何两个顶点之间恰有一条链; 反之, 若图 $T$ 中,任何两点之间恰有一条链,则 $T$ 必是树.", + "solution": "证明:若 $T$ 是树, 由于 $T$ 是连通的,所以 $T$ 中任意两个顶点之间至少有一条链,又因为 $T$ 无圈,任意两点之间必只能有一条链.\n反之, 若 $T$ 中任何两点之间恰有一条链, 则 $T$ 显然是连通的, 同时 $T$ 也必定无圈.\n否则, 圈上的任意两点之间就至少有两条链, 这与假设矛盾.", + "remark": "", + "figures": [] +} \ No newline at end of file diff --git a/processed_dataset/proof/0424.json b/processed_dataset/proof/0424.json new file mode 100644 index 0000000000000000000000000000000000000000..dafeebaf5b7cda2947246d8d8e2ebbe4b65e77d1 --- /dev/null +++ b/processed_dataset/proof/0424.json @@ -0,0 +1,8 @@ +{ + "source_file": "./raw_volume-zh/volume12/chapter4.tex", + "problem_type": "proof", + "problem": "例3. $n$ 个城市, 每个城市都可以通过一些中转城市与另一个城市通话, 证明至少有 $n-1$ 条直通的电话线路, 每条连结两个城市.", + "solution": "证明:作图 $G$ : 用 $n$ 个顶点表示 $n$ 个城市, 若两个城市之间有直通电话, 则在相应的顶点之间连一条边.\n由题意知, 图 $G$ 是连通图.\n故 $G$ 的边数一定 $\\geqslant n-1$, 从而至少有 $n-1$ 条直通的电话线路连结两个城市.\n本题也可以这样考虑: 若得到的连通图 $G$ 有圈, 我们就去掉这个圈的一条边, 得图 $G_1$, 此时 $G_1$ 比 $G$ 少了一条边, 但仍然是连通的: 如果 $G_1$ 还有圈, 再去掉圈的一条边得连通图 $G_2 \\cdots \\cdots$...这样继续下去, 直到图中没有圈, 这个图当然就是树.\n它有 $n-1$ 条边.\n故图 $G$ 至少有 $n-1$ 条边.\n上面所得到的树称为原来连通图 $G$ 的生成树.\n在生成树上添上若干条边, 就可以得到(生成) 原来的图.", + "remark": "", + "figures": [] +} \ No newline at end of file diff --git a/processed_dataset/proof/0425.json b/processed_dataset/proof/0425.json new file mode 100644 index 0000000000000000000000000000000000000000..4f3d9fb77232d5e80f3dabc501347dfedff33093 --- /dev/null +++ b/processed_dataset/proof/0425.json @@ -0,0 +1,10 @@ +{ + "source_file": "./raw_volume-zh/volume12/chapter4.tex", + "problem_type": "proof", + "problem": "例5. 某地区网球俱乐部的 20 名成员已举行 14 场单打比赛,每人至少上场一次.\n求证: 其中必有 6 场比赛, 12 个参赛者各不相同.", + "solution": "证本题在第二节已经讲过,这里我们再从树的角度给出另一种证明.\n用 20 个点代表 20 名参赛者.\n若两人比一场则在他们之间连边.\n这样共有 14 条边, 每点至少连出 1 条边, 所证结论相当于: 可以找出 6 条边两两不相邻.\n设图中共有 $n$ 个连通分支, 其中第 $i$ 个分支点数为 $v_i$, 边数为 $e_i$, 显然 $e_i \\geqslant v_i-1$, 故 $\\sum_{i=1}^n e_i \\geqslant \\sum_{i=1}^n\\left(v_i-1\\right)=\\sum_{i=1}^n v_i-n$. 而 $\\sum_{i=1}^n e_i=14, \\sum_{i=1}^n v_i=20$, 故 $14 \\geqslant 20-n, n \\geqslant 20-14=16$. 由于每点至少连出一条边, 因此不可能存在一个连通分支, 它仅含一个孤立点.\n故从每个连通分支中各取一条边, 即可保证两两不相邻,边数至少为 6 . 原命题得证.\n如图() 可知, 顶点数为 20 ,边数为 14 ,任选 7 条边,则必有两边在同一连通分支内, 它们必定相邻.\n故 6 为最佳结果.", + "remark": "", + "figures": [ + "./images/volume12/figures/fig-c4i6.png" + ] +} \ No newline at end of file diff --git a/processed_dataset/proof/0426.json b/processed_dataset/proof/0426.json new file mode 100644 index 0000000000000000000000000000000000000000..cc202d4f109e8bdd5e71f4f54b269a8ac2cf9d5d --- /dev/null +++ b/processed_dataset/proof/0426.json @@ -0,0 +1,8 @@ +{ + "source_file": "./raw_volume-zh/volume12/chapter4.tex", + "problem_type": "proof", + "problem": "例6. 以一些圆(圆面)覆盖平面上给定的 $2 n$ 个点.\n证明: 若每个圆至少覆盖 $n+1$ 个点, 则任意两个点能由平面上的一条折线所连结, 而这条线段整个地被一些圆所覆盖.", + "solution": "证明:以这 $2 n$ 个点为顶点, 若存在一个圆, 它覆盖着两个点, 则在这两顶点之间连一条边, 得到一个图 $G$. 由题意知, $G$ 中每个顶点的度不小于 $n$. 每条边之所以能画出, 就表明它能整个地被一个圆所覆盖, 于是我们只需证明 $G$ 是连通图.\n若 $G$ 不是连通图, 则存在一个连通分支 $G_1$, 至多含有 $n$ 个顶点, 这样对\n$G_1$ 中每一个顶点 $v$, 都有 $d(v) \\leqslant n-1$, 与题意矛盾, 从而 $G$ 是连通图.", + "remark": "", + "figures": [] +} \ No newline at end of file diff --git a/processed_dataset/proof/0427.json b/processed_dataset/proof/0427.json new file mode 100644 index 0000000000000000000000000000000000000000..74f3771ebd54f4f5a57ce2721689eb35ebce9e03 --- /dev/null +++ b/processed_dataset/proof/0427.json @@ -0,0 +1,8 @@ +{ + "source_file": "./raw_volume-zh/volume12/chapter4.tex", + "problem_type": "proof", + "problem": "例7. $n(n>3)$ 名乒乓球选手单打比赛若干场后, 任意两个选手已赛过的对手恰好都不完全相同.\n证明: 总可以从中去掉一名选手, 而使余下的选手中, 任意两个选手已赛过的对手仍然都不完全相同.", + "solution": "证明:用 $n$ 个顶点 $v_1, v_2, \\cdots, v_n$ 表示这 $n$ 名选手, 如果命题不成立, 即每一个选手都是不可去选手.\n对选手 $v_k(1 \\leqslant k \\leqslant n)$, 因为他不是可去选手, 所以去掉 $v_k$ 后, 总可以找到一对选手 $v_i$ 与 $v_j$, 他们所赛过的选手相同 (若有不止一对这样的选手, 则任取其中的一对), 这说明 $v_i$ 和 $v_j$ 赛过的选手仅差 $v_k$.不妨设 $v_i$ 与 $v_k$ 赛过, 而 $v_j$ 与 $v_k$ 末赛过, 在这样的一对点 $v_i$ 与 $v_j$ 之间连一条边, 并标上数字 $k$. 这样就得到一个有 $n$ 个顶点, $n$ 条边的图, 并且这 $n$ 条边上标有 $n$ 个互不相同的数.\n由于 $n$ 个顶点 $n$ 条边的图一定有圈, 设 $v_{i_1} v_{i_2} \\cdots v_{i_k}$ 为一个圈, 沿着这个圈前进时, 每通过一条边就相当于比赛选手增加或者减少一个人, 并且增加或减少的人是互不相同的.\n由于沿着圈前进一周后仍回到 $v_{i_1}$, 即与 $v_{i_1}$ 比赛过的选手再增加或者减少不同的选手, 最后的结果仍与 $v_{i_1}$ 原来赛过的选手相同,产生矛盾.\n因此,在 $n$ 个选手中至少有一个可去选手.", + "remark": "", + "figures": [] +} \ No newline at end of file diff --git a/processed_dataset/proof/0428.json b/processed_dataset/proof/0428.json new file mode 100644 index 0000000000000000000000000000000000000000..6bb55f3bce6d4a53b88b6a31c6f7d7067f7512e3 --- /dev/null +++ b/processed_dataset/proof/0428.json @@ -0,0 +1,8 @@ +{ + "source_file": "./raw_volume-zh/volume12/chapter4.tex", + "problem_type": "proof", + "problem": "例8. 在一次演讲中, 有五名数学家每人均打两次盹, 并且每两人均有同时在打盹的时刻.\n证明:一定有三个人, 他们有同时打盹的时刻.", + "solution": "证明:作图 $G$ : 用 $v_1, v_2, \\cdots, v_{10}$ 这 10 个顶点表示这五位数学家的十次盹, 当且仅当第 $i$ 次盹与第 $j$ 次盹有共同时刻时, 在 $v_i$ 与 $v_j$ 之间连一一条边.\n由题意, 每两个数学家均有同时在打盹的时刻, 从而图 $G$ 中的边数至少是 $\\mathrm{C}_5^2=10$ 条.\n而图 $G$ 的顶点数为 10 , 故 $G$ 中必有圈.\n设这个圈为 $v_{i_1} v_{i_2} \\cdots v_{i_k} v_{i_1}$, 且 $v_{i_1}$ 是圈中最先结束的一个盹, 那么当 $v_{i_1}$ 刚结束时, $v_{i_2}$ 及 $v_{i_k}$ 还在进行, 这就证明了有三位数学家有同时打盹的时刻.", + "remark": "", + "figures": [] +} \ No newline at end of file diff --git a/processed_dataset/proof/0429.json b/processed_dataset/proof/0429.json new file mode 100644 index 0000000000000000000000000000000000000000..843c7dacd9da35b89d4e33536352262be4d266a9 --- /dev/null +++ b/processed_dataset/proof/0429.json @@ -0,0 +1,8 @@ +{ + "source_file": "./raw_volume-zh/volume12/chapter4.tex", + "problem_type": "proof", + "problem": "例9. 某居民区内有 1990 个居民, 每天他们之中每个人都把昨天听到的消息告诉给他所有的熟人,而且任何消息都能逐渐地被全区居民所知道.\n证明: 可以指定 180 个居民, 使得同时向他们报导某一消息,那么至多经过 10 天,这一消息便为全区居民所知道.", + "solution": "证明:用点表示这些居民,两个顶点相邻就表示相应的居民是熟人, 这样就得到了一个有 1990 个顶点的图 $G$.\n由题意知, 图 $G$ 是连通的.\n不妨设这个图是树 $T_{1990}$ (否则用这个图的生成树来代替它), 在树 $T_{1990}$ 中, 取一条最长的链, 设为\n$$\nv_1^{(1)} v_2^{(1)} v_3^{(1)} \\cdots v_{11}^{(1)} \\cdots v_n^{(1)} \\text {. }\n$$\n取 $v_{11}^{(1)}$ 作为一个居民代表, 并将边 $\\left(v_{11}^{(1)}, v_{12}^{(1)}\\right)$ 去掉.\n这时 $T_{1990}$ 被分成两棵树, 前一棵树中, 每个顶点 $v$ 到 $v_{11}^{(1)}$ 的距离不大于 10 (否则在树 $T_{1990}$ 中, $v$ 到 $v_n^{(1)}$ 是一条比 $v_1^{(1)}$ 到 $v_n^{(1)}$ 更长的链). 于是代表 $v_{11}^{(1)}$ 所知道的消息, 前一棵树的顶点所表示的人在 10 天之内都能知道.\n对后一棵树, 也有一条最长的链, 设为\n$$\nv_1^{(2)} v_2^{(2)} v_3^{(2)} \\cdots v_{11}^{(2)} \\cdots v_m^{(2)} \\text {. }\n$$\n这里 $m \\leqslant 1990-11=1979$. 同样地, 取 $v_{11}^{(2)}$ 作为一个居民代表, 并去掉边 $\\left(v_{11}^{(2)}, v_{12}^{(2)}\\right)$, 将这棵树再分为两棵树.\n这样继续下去, 当选好 $v_{11}^{(i)}(i \\leqslant 179)$ 时, 剩下的树的顶点数 $\\leqslant 11$, 这时代表总数为 $i+1 \\leqslant 180$, 命题成立.\n否则陆续得出代表\n$$\nv_{11}^{(1)}, v_{11}^{(2)}, \\cdots, v_{11}^{(179)} \\text {. }\n$$\n每个代表都可以把一个消息在 10 天之内告知他那个居民区中的居民.\n最后剩下一棵树,至多有\n$$\n1990-11 \\times 179=21\n$$\n个顶点.\n设\n$$\nv_1 v_2 \\cdots v_k\n$$\n是它的一条最长链, 若 $k \\geqslant 11$, 则取 $v_{11}$ 作为第 180 个居民代表 $v_{11}^{(180)}$, 若 $k<$ 11 , 则取 $v_1$ 作为第 180 个居民代表 $v_{11}^{(180)}$. 这样选出的 180 个居民代表\n$$\nv_{11}^{(1)}, v_{11}^{(2)}, \\cdots, v_{11}^{(179)}, v_{11}^{(180)}\n$$\n是满足题目要求的 180 个居民.", + "remark": "", + "figures": [] +} \ No newline at end of file diff --git a/processed_dataset/proof/0430.json b/processed_dataset/proof/0430.json new file mode 100644 index 0000000000000000000000000000000000000000..e1dc94c39a7cc604ff93353f057539a64c04a713 --- /dev/null +++ b/processed_dataset/proof/0430.json @@ -0,0 +1,11 @@ +{ + "source_file": "./raw_volume-zh/volume12/chapter5.tex", + "problem_type": "proof", + "problem": "例5.. 凸 $n$ 边形及 $n-3$ 条在形内不相交的对角线组成的图形称为一个剖分图.\n求证: 当且仅当 $3 \\mid n$ 时,存在一个剖分图是可以一笔画的圈(即可以从一个顶点出发,经过图中各线段恰一次,最后回到出发点).", + "solution": "证明:先用数学归纳法证明充分性.\n$n=3$ 时,命题显然成立.\n设对任何凸 $3 k$ 边形,存在一个剖分图是可以一笔画的圈.\n对一个凸 $3(k+1)=3 k+3$ 边形 $A_1 A_2 A_3 \\cdots A_{3 k+3}$, 连接 $A_4 A_{3 k+3}$. 由于 $A_4 A_5 \\cdots A_{3 k+3}$ 是凸 $3 k$ 边形, 根据归纳假设, $A_4 A_5 \\cdots A_{3 k+3}$ 存在一个剖分图是一个可以一笔画的圈, 作此剖分图,并连接 $A_2 A_4, A_2 A_{3 k+3}$,于是便得到一个凸 $3 k+3$ 边形 $A_1 A_2 A_3 \\cdots A_{3 k+3}$ 的剖分图.\n因 $A_4 A_5 \\cdots A_{3 k+3}$ 的剖分图是一个圈, 故从 $A_{3 k+3}$ 出发, 经过这个剖分图中的每一条边恰一次后可回到 $A_{3 k+3}$, 再经 $A_{3 k+3} A_1$,\n$A_1 A_2, A_2 A_3, A_3 A_4, A_4 A_2, A_2 A_{3 k+3}$ 后, 又回到 $A_{3 k+3}$, 这就证明了凸 $3 k+3$ 边形 $A_1 A_2 A_3 \\cdots A_{3 k+3}$ 也存在一个剖分图是一个可以一笔画的圈.\n于是充分性得证.\n再证必要性.\n因为一个凸 $n$ 边形存在剖分图是可以一笔画的圈, 则它的每个顶点都是偶顶点.\n显然凸四边形和凸五边形不存在每个顶点都是偶顶点的剖分图.\n从而当 $3 \\leqslant n<6$ 时, 如果凸 $n$ 边形存在每个顶点都是偶顶点的剖分图, 则 $n=3$.\n设当 $3 \\leqslant n<3 k(k>2)$ 时, 如果凸 $n$ 边形存在每个顶点都是偶顶点的剖分图, 则 $3 \\mid n$. 现考虑 $3 k \\leqslant n<3(k+1)$ 的情况.\n设凸 $n$ 边形 $A_1 A_2 \\cdots A_n$ 有一个每个顶点都是偶顶点的剖分图.\n易知任意凸 $n(n>3)$ 边形的任何剖分, 都把此凸 $n$ 边形分割成没有公共内部的 $n-2$ 个三角形, 而且这些三角形中至少有两个以这个凸 $n$ 边形两条相邻边为两边.\n因此不妨设 $A_1 A_3$ 是凸 $n$ 边形 $A_1 A_2 \\cdots A_n$ 剖分图中的一条对角线(如图() 所示). 于是 $A_1 A_3$ 还是剖分图中另一个 $\\triangle A_1 A_3 A_i$ 的一边.\n由假设 $A_1 A_2 \\cdots A_n$ 的剖分图使得每个顶点都是偶顶点, 故 $i \\neq 4$, 否则 $A_3$ 是奇顶点.\n同样 $i \\neq n$, 否则 $A_1$ 为奇顶点.\n因此 $4)). 用 $m$ 表示白色三角形的边数,显然 $3 \\mid m$, 每个白三角形的边同时也是黑三角形的边, 而多边形的所有边是黑三角形的边, 故黑三角形的边数为 $m+n$. 由 $3 \\mid m+n$, 便得 $3 \\mid n$.", + "remark": "", + "figures": [ + "./images/volume12/figures/fig-c5i11.png", + "./images/volume12/figures/fig-c5i12.png" + ] +} \ No newline at end of file diff --git a/processed_dataset/proof/0431.json b/processed_dataset/proof/0431.json new file mode 100644 index 0000000000000000000000000000000000000000..3b1cf426d9af8475fa6dcf529e9ee82bc8f3983b --- /dev/null +++ b/processed_dataset/proof/0431.json @@ -0,0 +1,8 @@ +{ + "source_file": "./raw_volume-zh/volume12/chapter5.tex", + "problem_type": "proof", + "problem": "例6. 设 $n>3$, 考虑在同一圆周上的 $2 n-1$ 个互不相同的点所成的集合 $E$. 将 $E$ 中一部分点染成黑色, 其余的点不染色.\n如果至少有一对黑点, 以它们为端点的两条弧中有一条的内部(不包含端点) 恰含 $E$ 中 $n$ 个点, 则称这种染色方式为\"好的\". 如果将 $E$ 中 $k$ 个点染黑的每一种染色方式都是好的,求 $k$ 的最小值.", + "solution": "解:将 $E$ 中的点按逆时针方向依次用顶点 $v_1, v_2, \\cdots, v_{2 n-1}$ 表示, 并在顶点 $v_i$ 与 $v_{i+(n+1)}$ 之间连一条边, $i=1,2, \\cdots, 2 n-1$. (我们约定 $v_{j+(2 n-1) k}= v_j, k \\in \\mathbf{Z}$ ). 这样便得一个图 $G, G$ 中每个顶点的度为 2 (即与两个点相邻), 并且 $v_i$ 与 $v_{i+3}$ 与同一个点相邻.\n由于 $G$ 中的每个点都是偶顶点, 所以 $G$ 是由一个或几个圈所组成的.\n(i) 当 $3 \\mid(2 n-1)$ 时, 图 $G$ 由三个圈组成,每个圈的顶点集为\n$$\n\\begin{aligned}\n& \\left\\{v_i \\mid i=3 k, k=1,2, \\cdots, \\frac{2 n-1}{3}\\right\\}, \\\\\n& \\left\\{v_i \\mid i=3 k+1, k=0,1, \\cdots, \\frac{2 n-4}{3}\\right\\}, \\\\\n& \\left\\{v_i \\mid i=3 k+2, k=0,1, \\cdots, \\frac{2 n-4}{3}\\right\\} .\n\\end{aligned}\n$$\n由于每个圈上的顶点数都是 $\\frac{2 n-1}{3}$, 故每个圈上至多可以取出 $\\frac{1}{2}\\left(\\frac{2 n-1}{3}-1\\right)=\\frac{n-2}{3}$ 个点,两两互不相邻 (注意 $\\frac{2 n-1}{3}$ 是奇数). 总共可以取出 $n-2$ 个点互不相邻.\n由抽屉原则, 至少要染黑 $n-1$ 个点, 才能保证至少有一对黑点相邻.\n(ii) 当 $3 \\nmid(2 n-1)$ 时, $v_1, v_2, \\cdots, v_{2 n-1}$ 中的每一个点都可以表为 $v_{3 k}$ 的形式, 因此图 $G$ 是一个长为 $(2 n-1)$ 的圈.\n在这圈上可以取出 $n-1$ 个互不相邻的点, 而且至多可以取出 $n-1$ 个互不相邻的点.\n因而至少要染黑 $n$ 个点, 才能保证至少有一对黑点相邻.\n综上所述, 当 $3 \\nmid(2 n-1)$ 时, $k$ 的最小值是 $n$, 当 $3 \\mid(2 n-1)$ 时, $k$ 的最小值为 $n-1$.", + "remark": "", + "figures": [] +} \ No newline at end of file diff --git a/processed_dataset/proof/0432.json b/processed_dataset/proof/0432.json new file mode 100644 index 0000000000000000000000000000000000000000..e915dfa408a50cc331ad229002107b34c0bd523a --- /dev/null +++ b/processed_dataset/proof/0432.json @@ -0,0 +1,12 @@ +{ + "source_file": "./raw_volume-zh/volume12/chapter6.tex", + "problem_type": "proof", + "problem": "例3. 判断如图() 所示的图 $G$ 有无哈密顿链或哈密顿圈?", + "solution": "解:我们将图中某个顶点标上 $A$, 例如把点 $a$ 标上 $A$, 所有与 $a$ 相邻的点均标上 $B$, 连续不断地用 $A$ 标记所有与已标上 $B$ 相邻的点, 用 $B$ 标记所有与已标上 $A$ 的相邻的点.\n直到图中所有点标记完毕.\n如图() 所示.\n如果这个图 $G$ 中有一条哈密顿链, 那么它必定交替通过点 $A$ 和点 $B$, 因而点 $A$ 的数目与点 $B$ 的数目相等或相差 1. 但是图() 中点 $A$ 有 9 个, 点 $B$ 有 7 个, 两者相差为 2 ,因此不可能有一条哈密顿链.\n一般地, 对于一个偶图 $G=\\left(V_1, V_2, E\\right)$, 有一个简单的方法可以断定它没有哈密顿链或哈密顿圈.\n定理一在偶图 $G=\\left(V_1, V_2, E\\right)$ 中, 如果 $\\left|V_1\\right| \\neq\\left|V_2\\right|$, 那么 $G$ 一定无哈密顿圈.\n如果 $\\left|V_1\\right|$ 与 $\\left|V_2\\right|$ 的差大于 1 , 那么 $G$ 一定无哈密顿链.\n可以采用同例 3 完全相同的方法证明.", + "remark": "", + "figures": [ + "./images/volume12/figures/fig-c6i4.png", + "./images/volume12/figures/fig-c6i5.png", + "./images/volume12/figures/fig-c6i5.png" + ] +} \ No newline at end of file diff --git a/processed_dataset/proof/0433.json b/processed_dataset/proof/0433.json new file mode 100644 index 0000000000000000000000000000000000000000..d69a67370bad9a906d4fd5f106ef41cd7586fa9a --- /dev/null +++ b/processed_dataset/proof/0433.json @@ -0,0 +1,10 @@ +{ + "source_file": "./raw_volume-zh/volume12/chapter6.tex", + "problem_type": "proof", + "problem": "例5. $n$ 个人参加一次会议, 在会议期间, 每天都要在一张圆桌上共进晚餐.\n如果要求每次晚餐就座时, 每个人相邻就座者都不相同, 问这样的晚餐最多能进行多少次?", + "solution": "证明:用 $n$ 个点表示 $n$ 个人,作完全图 $K_n$,如图() 所示,则 $K_n$ 中的一个哈密顿圈就是一次晚餐的就座方法.\n可见, 晚餐最多能进行的次数就是 $K_n$ 中无公共边的哈密顿圈的个数.\n$K_n$ 中有 $\\frac{1}{2} n(n-1)$ 条边, 每个哈密顿圈有 $n$ 条边, 因此, 边不相重的哈密顿圈最多有 $\\left[\\frac{n-1}{2}\\right]$ 个.\n当 $n=2 k+1$ 时, 将顶点 $0,1,2, \\cdots, 2 k$ 排列如图.\n先取一个哈密顿圈 $(0,1,2,2 k, 3,2 k-1,4, \\cdots$, $k+3, k, k+2, k+1,0)$, 然后绕 0 点依次顺时针旋转 $\\frac{\\pi}{k}, \\frac{2 \\pi}{k}, \\cdots,(k-1) \\frac{\\pi}{k}$, 共产生 $k=\\left[\\frac{n-1}{2}\\right]$ 个无公共边的哈密顿圈, 如果 $n=2 k+2$, 那么每次在中间添加一个顶点 $v$, 同样有 $k$ 个哈密顿圈.\n由定理三可以推得下面的定理四.\n这是 1952 年数学家 Dirac 给出的.\n定理四 $G$ 是 $n(n \\geqslant 3)$ 阶简单图, 如果每个顶点 $v$ 的度 $d(v) \\geqslant \\frac{n}{2}$, 则图 $G$ 一定存在哈密顿圈.", + "remark": "", + "figures": [ + "./images/volume12/figures/fig-c6i12.png" + ] +} \ No newline at end of file diff --git a/processed_dataset/proof/0434.json b/processed_dataset/proof/0434.json new file mode 100644 index 0000000000000000000000000000000000000000..c16286c9ba9c17d43f281a41191e8725384f8210 --- /dev/null +++ b/processed_dataset/proof/0434.json @@ -0,0 +1,8 @@ +{ + "source_file": "./raw_volume-zh/volume12/chapter6.tex", + "problem_type": "proof", + "problem": "例6. 在 7 天内安排 7 门课的考试, 要使得同一位教师所教的两门课考试不排在接连的两天里.\n如果每一位教师教的考试课最多 4 门, 证明这种安排是可能的.", + "solution": "证明:设 $G$ 是有 7 个顶点的图, 每个顶点对应一门考试课, 如果两个顶点对应的考试课是由不同教师担任的, 则在这两个顶点之间连一条边, 因为这个教师所教的课不超过 4 门, 故每个顶点的度数至少是 3. 任意两个顶点的度数之和至少是 6 , 根据定理二, $G$ 有一条哈密顿链.\n由于这条链上任何一条边相邻的两顶点对应的两门课不是同一位教师教的, 所以, 可以按照这条哈密顿链上的顶点顺序安排这 7 门课的考试.", + "remark": "", + "figures": [] +} \ No newline at end of file diff --git a/processed_dataset/proof/0435.json b/processed_dataset/proof/0435.json new file mode 100644 index 0000000000000000000000000000000000000000..d4331b0353c4c13f397d287adfd71663162828c4 --- /dev/null +++ b/processed_dataset/proof/0435.json @@ -0,0 +1,8 @@ +{ + "source_file": "./raw_volume-zh/volume12/chapter6.tex", + "problem_type": "proof", + "problem": "例7. 某工厂生产由 6 种不同颜色的纱织成的双色布.\n已知花布品种中, 每种颜色至少分别和其他 3 种不同的颜色搭配.\n试证可以挑出 3 种双色布, 它们恰好含有 6 种不同的颜色.", + "solution": "证明:用 6 个顶点表示 6 种颜色的纱,若两种颜色的纱能搭配织成一种双色布, 就在相应的顶点之间连一条边, 这样就得到一个图 $G$. 已知条件是, 每种颜色的纱至少和其他三种颜色的纱搭配, 也即对任意顶点 $v_i, d\\left(v_i\\right) \\geqslant 3$. 欲证的是,图 $G$ 中存在三条边, 其中任意两条边都没有公共端点.\n因为对图 $G$ 中任一顶点 $v_i, d\\left(v_i\\right) \\geqslant 3$, 根据定理四, $G$ 有哈密顿圈, 记为 $v_1 v_2 v_3 v_4 v_5 v_6 v_1$, 则边 $\\left(v_1, v_2\\right),\\left(v_3, v_4\\right),\\left(v_5, v_6\\right)$ 就是三条两两没有公共端点的边.\n我们往往用一个图满足充分条件来肯定这个图是哈密顿图,而用一个图不满足必要条件来否定这个图是哈密顿图.\n下面给出一个图是哈密顿图的必要条件.\n定理五如果图 $G$ 有哈密顿图, 从 $G$ 中去掉若干个点 $v_1, v_2, \\cdots, v_k$ 及与它们关联的边得到图 $G^{\\prime}$, 那么 $G^{\\prime}$ 的连通分支不超过 $k$ 个.\n证明设 $c$ 是图 $G$ 中的哈密顿圈, 将 $k$ 个顶点以及与它们关联的边去掉后, $c$ 最多分为 $k$ 段, 因此 $G^{\\prime}$ 的连通分支至多为 $k$ 个.", + "remark": "", + "figures": [] +} \ No newline at end of file diff --git a/processed_dataset/proof/0436.json b/processed_dataset/proof/0436.json new file mode 100644 index 0000000000000000000000000000000000000000..5153c11632c0862e77ee402512b478808add4b3b --- /dev/null +++ b/processed_dataset/proof/0436.json @@ -0,0 +1,12 @@ +{ + "source_file": "./raw_volume-zh/volume12/chapter6.tex", + "problem_type": "proof", + "problem": "例8. 证明如图() 没有哈密顿圈.", + "solution": "证明:如图() 中将顶点 $v_1, v_2$ 以及与它们关联的边去掉, 得到的 $G^{\\prime}$ 有三个连通分支, 不满足定理五的必要条件,所以图 () 没有哈密顿圈.\n最后再举一个例子作为本节的结束.", + "remark": "", + "figures": [ + "./images/volume12/figures/fig-c6i13.png", + "./images/volume12/figures/fig-c6i13.png", + "./images/volume12/figures/fig-c6i13.png" + ] +} \ No newline at end of file diff --git a/processed_dataset/proof/0437.json b/processed_dataset/proof/0437.json new file mode 100644 index 0000000000000000000000000000000000000000..5c6fd235d0251e17236a3346344e8c4dfea999d7 --- /dev/null +++ b/processed_dataset/proof/0437.json @@ -0,0 +1,8 @@ +{ + "source_file": "./raw_volume-zh/volume12/chapter6.tex", + "problem_type": "proof", + "problem": "例9. 若 $A_0 A_1 A_2 \\cdots A_{2 n-1}$ 为一个正 $2 n$ 边形,连接它的所有对角线, 这样得到一个图 $G$. 证明: 图 $G$ 的每个哈密顿圈都必定包含两条边, 它们在图中是平行线.", + "solution": "证明:设 $A_i A_j$ 与 $A_k A_l$ 平行, 由于 $A_i$ 与 $A_l$ 之间和 $A_j$ 与 $A_k$ 之间顶点数相同, 所以 $i-l=k-j$, 于是得到 $A_i A_j$ 与 $A_k A_l$ 平行的充要条件是\n$$\ni+j \\equiv k+l(\\bmod 2 n) \\text {. }\n$$\n设 $A_{i_0} A_{i_1} \\cdots A_{i_{2 n-1}}$ 是一个哈密顿圈 $\\left(i_0, i_1, \\cdots, i_{2 n-1}\\right.$ 是 $0,1, \\cdots, 2 n-1$ 的一个排列), 而其中任意的两条边在图中不是平行线.\n因此 $i_0+i_1, i_1+i_2, i_2+ i_3, \\cdots, i_{2 n-1}+i_0$ 这 $2 n$ 个数中的任意两个关于模 $2 n$ 都不同余, 即上述这 $2 n$ 个数是模 $2 n$ 的一组完全剩余系.\n于是\n$$\n\\begin{aligned}\n& \\left(i_0+i_1\\right)+\\left(i_1+i_2\\right)+\\cdots+\\left(i_{2 n-1}+i_0\\right) . \\\\\n\\equiv & 0+1+2+\\cdots+2 n-1=2 n^2-n \\\\\n\\equiv & n(\\bmod 2 n) .\n\\end{aligned}\n$$\n另一方面, 有\n$$\n\\begin{aligned}\n& \\left(i_0+i_1\\right)+\\left(i_1+i_2\\right)+\\cdots+\\left(i_{2 n-1}+i_0\\right) \\\\\n= & 2\\left(i_0+i_1+i_2+\\cdots+i_{2 n-1}\\right) \\\\\n= & 2(0+1+2+\\cdots+2 n-1) \\\\\n= & 2 n(2 n-1) \\\\\n\\equiv & 0(\\bmod 2 n) .\n\\end{aligned}\n$$\n上面得出了两个矛盾的结果, 从而命题得证.", + "remark": "", + "figures": [] +} \ No newline at end of file diff --git a/processed_dataset/proof/0438.json b/processed_dataset/proof/0438.json new file mode 100644 index 0000000000000000000000000000000000000000..52c850d10bfe1d31d511eb341d7ffb61ad40c6cb --- /dev/null +++ b/processed_dataset/proof/0438.json @@ -0,0 +1,8 @@ +{ + "source_file": "./raw_volume-zh/volume12/chapter7.tex", + "problem_type": "proof", + "problem": "例1. 证明完全图 $K_5$ 不是平面图.", + "solution": "证明:因为 $v=5, e==10$ 不满足 $e \\leqslant 3 v-6$, 所以 $K_5$ 不是平面图.", + "remark": "", + "figures": [] +} \ No newline at end of file diff --git a/processed_dataset/proof/0439.json b/processed_dataset/proof/0439.json new file mode 100644 index 0000000000000000000000000000000000000000..bbd3c183548b8919f06665d1283999c20aa27f37 --- /dev/null +++ b/processed_dataset/proof/0439.json @@ -0,0 +1,8 @@ +{ + "source_file": "./raw_volume-zh/volume12/chapter7.tex", + "problem_type": "proof", + "problem": "例2. 证明 $K_{3,3}$ 图不是平面图.", + "solution": "证明:如果 $K_{3,3}$ 是平面图, 因为在 $K_{3,3}$ 中任取 3 个顶点, 其中必有两个顶点不相邻,故每一个面都至少有 4 条边围成, 由\n$$\n4 f \\leqslant 2 e, f \\leqslant \\frac{e}{2} .\n$$\n代入欧拉公式\n$$\n2=v-e+f \\leqslant v-e+\\frac{e}{2},\n$$\n即\n$$\ne \\leqslant 2 v-4 \\text {. }\n$$\n在 $K_{3,3}$ 中, $v=6, e=9$, 且 $9>2 \\times 6-4$, 矛盾.\n故 $K_{3,3}$ 不是平面图.", + "remark": "", + "figures": [] +} \ No newline at end of file diff --git a/processed_dataset/proof/0440.json b/processed_dataset/proof/0440.json new file mode 100644 index 0000000000000000000000000000000000000000..cc29f3ddfdf0439ff3a0aa68e5c130fcc683b60e --- /dev/null +++ b/processed_dataset/proof/0440.json @@ -0,0 +1,10 @@ +{ + "source_file": "./raw_volume-zh/volume12/chapter7.tex", + "problem_type": "proof", + "problem": "例4. 正多面体有几种? 它们的棱数、顶点数和面数各是多少? 每个顶点连接几条棱?", + "solution": "证明:因为正多面体每个顶点至少有三个面拼在一起, 所以当正多边形的每个内角大于等于 $120^{\\circ}$ 时, 显然形不成正多面体的顶点.\n所以只能考虑以正五边形、正方形和正三角形为基础搭成什么样的正多面体了.\n(1) 以正五边形为面的多面体.\n因为正五边形内角为 $\\frac{3}{5} \\pi$, 而 $\\frac{3}{5} \\pi \\times 4>2 \\pi$, 所以以正五边形为面的正多面体每个顶点皆 3 次, 于是 $3 v=2 e, \\frac{5 f}{2}=e$, 代入欧拉公式得\n$$\n\\frac{2}{3} e-e+\\frac{2}{5} e=2,\n$$\n解得\n$$\ne=30, v=20, f=12 .\n$$\n即以正五边形为面的正多面体只有一种, 它是 20 个顶点、30 条棱且每个顶点处有 3 条棱的正十二面体.\n(2)对以正方形、正三角形为面的正多面体, 请读者自行证明,共有如下图()所示 4 种.\n从而正多面体只有 5 种.", + "remark": "", + "figures": [ + "./images/volume12/figures/fig-c7i7.png" + ] +} \ No newline at end of file diff --git a/processed_dataset/proof/0441.json b/processed_dataset/proof/0441.json new file mode 100644 index 0000000000000000000000000000000000000000..6c5bd10b01e2152ca84e80214293d757814684b7 --- /dev/null +++ b/processed_dataset/proof/0441.json @@ -0,0 +1,8 @@ +{ + "source_file": "./raw_volume-zh/volume12/chapter7.tex", + "problem_type": "proof", + "problem": "例5. 如果一个正方形被划分为 $n$ 个凸多边形, 当 $n$ 为定值时, 求这些凸多边形边数的最大值.", + "solution": "证明:由欧拉公式知一个凸多边形被划分为 $n$ 个多边形, 则 $v-e+n=$ 1(因为 $f=n+1$ ).\n由于一个正方形被划分为 $n$ 个凸多边形,因此这些多边形的每个顶点,如果它不是正方形的顶点, 必是至少 3 个凸多边形的顶点.\n用 $A 、 B 、 C 、 D$ 分别表示正方形的顶点,用 $v$ 表示除 $A 、 B 、 C 、 D$ 外的任一顶点,则\n$$\nd(v) \\leqslant 3(d(v)-2) .\n$$\n由上式对除 $A 、 B 、 C 、 D$ 外的所有点求和, 得\n$$\n\\begin{aligned}\n& 2 e-(d(A)+d(B)+d(C)+d(D)) \\\\\n\\leqslant & 3(2 e-(d(A)+d(B)+d(C)+d(D))-6(v-4)),\n\\end{aligned}\n$$\n于是\n$$\n4 e \\geqslant 2(d(A)+d(B)+d(C)+d(D))+6(v-4),\n$$\n由于 $d(A) \\geqslant 2, d(B) \\geqslant 2, d(C) \\geqslant 2, d(D) \\geqslant 2$, 所以\n$$\n2 e \\geqslant 8+3(v-4) \\text {. }\n$$\n由\n$$\nv-e+n=1,\n$$\n得\n$$\n3(e+1)=3 v+3 n \\leqslant 2 e+4+3 n,\n$$\n即\n$$\ne \\leqslant 3 n+1 \\text {. }\n$$\n过正方形的一边相继作 $n-1$ 条邻边的平行线,将这个正方形分为 $n$ 个矩形, 总边数为 $4+3(n-1)=3 n+1$.\n综上,所求边的最大值为 $3 n+1$.", + "remark": "", + "figures": [] +} \ No newline at end of file diff --git a/processed_dataset/proof/0442.json b/processed_dataset/proof/0442.json new file mode 100644 index 0000000000000000000000000000000000000000..620dbfd0fc0c99b5cada6165777fa80b7c781b8e --- /dev/null +++ b/processed_dataset/proof/0442.json @@ -0,0 +1,10 @@ +{ + "source_file": "./raw_volume-zh/volume12/chapter7.tex", + "problem_type": "proof", + "problem": "例6. 证明戈林伯格图 (如图())无哈密顿圈.", + "solution": "证明:设这个图含哈密顿圈.\n因为它只有 5 边形、8 边形和 9 边形, 根据定理四有\n$$\n3\\left(f_5^{\\prime}-f_5^{\\prime \\prime}\\right)+6\\left(f_8^{\\prime}-f_8^{\\prime \\prime}\\right)+7\\left(f_9^{\\prime}-f_9^{\\prime \\prime}\\right)=0 .\n$$\n由此得 $7\\left(f_9^{\\prime}-f_9^{\\prime \\prime}\\right) \\equiv 0(\\bmod 3)$.\n这与 $f_9^{\\prime}+f_9^{\\prime \\prime}=1$ 矛盾.\n故戈林伯格图无哈密顿圈.", + "remark": "", + "figures": [ + "./images/volume12/figures/fig-c7i8.png" + ] +} \ No newline at end of file diff --git a/processed_dataset/proof/0443.json b/processed_dataset/proof/0443.json new file mode 100644 index 0000000000000000000000000000000000000000..5d7f0f4fc00546ac71d2345af5fd5e4623c8dfa7 --- /dev/null +++ b/processed_dataset/proof/0443.json @@ -0,0 +1,12 @@ +{ + "source_file": "./raw_volume-zh/volume12/chapter8.tex", + "problem_type": "proof", + "problem": "例1. 证明: 在任何六个人中, 总可以找到三个相互认识的人或三个相互不认识的人.", + "solution": "我们用六个顶点表示六个人, 如果某两个人互相认识, 就在相应的两点间连一条边并涂上红色, 某两个人互相不认识, 就在相应的两点间连一条边并涂上蓝色.\n要证明的结论就是这个涂了色的 $K_6$ 中一定有一个各边同色的三角形.\n无独有偶, 这个变形就是 1953 年美国普特南数学竞赛试题: 空间中的六个点, 任意三点不共线, 任意四点不共面, 成对地连接它们得到十五条线段.\n用红色或蓝色染这些线段 (一条线段只染一种颜色), 求证: 无论如何染色, 总存在同色三角形.\n证明:设 $A_1, A_2, \\cdots, A_6$ 是所给的六点.\n考虑由 $A_1$ 出发的 5 条线段 $A_1 A_2, A_1 A_3, \\cdots, A_1 A_6$. 因这 5 条线段只有红、蓝两种颜色, 因此至少有 3 条染成同一种颜色.\n不妨设这 3 条线段就是 $A_1 A_2, A_1 A_3, A_1 A_4$, 且它们都染成红色 (实线表示红色,虚线表示蓝色). 若 $\\triangle A_2 A_3 A_4$ 三边都是蓝色 (如图()), 它即为同色三角形.\n若 $\\triangle A_2 A_3 A_4$ 至少有一条边, 例如 $A_2 A_3$ 为红色 (如图()), 则 $\\triangle A_1 A_2 A_3$ 是同色三角形.\n总之, 无论是哪种情况, 都有同色三角形.\n从本例我们还易知, 当 $n \\geqslant 6$ 时, 给完全图 $K_n$ 的所有边染两种颜色的某一种 (以后简称两色完全图 $K_n$ ), 则总存在同色三角形.\n如图() 所示是一个染两色的完全图 $K_5$, 其中没有同色三角形.\n之, 不难证明: 没有同色三角形的两色完全图 $K_5$, 必由两个不同色的五边形所组成.\n换言之, 在两色完全图 $K_5$ 中, 若既没有蓝色三角形, 又没有蓝色五边形, 则必有红色三角形.\n综上所述, 可得如下的结论.\n定理一两色完全图 $K_n$ 必存在同色三角形的最小 $n$ 是 6 .", + "remark": "", + "figures": [ + "./images/volume12/figures/fig-c8i1.png", + "./images/volume12/figures/fig-c8i2.png", + "./images/volume12/figures/fig-c8i3.png" + ] +} \ No newline at end of file diff --git a/processed_dataset/proof/0444.json b/processed_dataset/proof/0444.json new file mode 100644 index 0000000000000000000000000000000000000000..d544521c075f4b4184a239ae286c59b02ac88ff2 --- /dev/null +++ b/processed_dataset/proof/0444.json @@ -0,0 +1,8 @@ +{ + "source_file": "./raw_volume-zh/volume12/chapter8.tex", + "problem_type": "proof", + "problem": "例2. 证明不能对 $K_{10}$ 的边用四种颜色进行染色, 使得它的任何的 $K_4$ 子图的边包含所有四种颜色.", + "solution": "证明:我们用反证法.\n假设存在一种满足题目要求的染色方式.\n若有一点连出 4 条同色的边, 不妨设 $A B 、 A C 、 A D 、 A E$ 全是蓝色的.\n$B 、 C 、 D 、 E$ 之间一定有一条边是蓝色的, 不妨设为 $B C$, 则 $A 、 B 、 C 、 D$ 之间有 4 条蓝边,剩下两条边要染上三种颜色, 矛盾.\n因此, $A$ 至多连出 3 条同色的边, 而且一定有一种颜色恰好染了三条边.\n不妨设 $A B 、 A C 、 A D$ 全是蓝色的.\n$A$ 、 $B 、 C 、 D$ 之间有 6 条边, 所以其余的 3 条边一定各染一种颜色, $B C 、 B D 、 C D$ 之中无蓝色边.\n考虑余下的 6 个点, 由定理一, 其中必有蓝色三角形或者有某个三角形无蓝色边.\n若有三点 $E 、 F 、 G$ 之间无蓝边, 则 $A 、 E 、 F 、 G$ 之间无蓝边, 矛盾.\n因此不妨假设三角形 $E F G$ 为蓝色三角形.\n因为 $B 、 C 、 D 、 E$ 之间有蓝边, 故只能是 $B E 、 C E 、 D E$ 之一为蓝色, 假设 $B E$ 是蓝色的, 则 $B 、 E 、 F 、 G$ 之间有四条蓝色边,同上矛盾.\n综上命题成立.", + "remark": "", + "figures": [] +} \ No newline at end of file diff --git a/processed_dataset/proof/0445.json b/processed_dataset/proof/0445.json new file mode 100644 index 0000000000000000000000000000000000000000..03a368b8e8e340cda2f87c518d3344c833d10a62 --- /dev/null +++ b/processed_dataset/proof/0445.json @@ -0,0 +1,10 @@ +{ + "source_file": "./raw_volume-zh/volume12/chapter8.tex", + "problem_type": "proof", + "problem": "例3. 给定空间中 9 个点, 其中任意四个点都不共面.\n在每一对点之间都连着一条线段.\n试求出最小的 $n$ 值, 使得将其中任意 $n$ 条线段中的每一条任意地染为红、蓝两色之一, 在这 $n$ 条线段的集合中都必然包含有一个各边同色的三角形.", + "solution": "解:题中\"任意四点不共面\"只是为了保证 9 点中无 3 点共线, 因此本题仍是一个平面上的图形问题.\n于是问题归结为: 平面上有 9 个点, 无三点共线, 每两点连边共有 36 条边.\n问至少取多少条边, 才能保证把这些边任意染成红蓝色时,一定出现同色三角形.\n构造一个 9 阶 32 边的双色图 $G$. 顶点 $v_1$ 与 $v_2, v_3, v_3, v_9 4$ 点的连线染红色 (实线), $v_1$ 与 $v_4, v_5, v_6, v_7 4$ 点的连线染蓝色 (虚线). 把 $v_1$ 以外的 8 点分成 4 组, I : $\\left(v_2, v_3\\right), \\mathbb{I I}:\\left(v_4, v_5\\right)$, III: $\\left(v_6, v_7\\right) ; \\mathrm{IV}:\\left(v_8, v_9\\right)$, 称 I 与 II, $\\mathrm{II}$ 与 III, III 与 IV 为相邻的组.\n除 $v_1$ 外的两点, 属同一组的不连线, 分属两相邻组的连实线 (红色), 分属两不相邻组的连虚线 (蓝色), 如图(), $G$ 有 $\\mathrm{C}_9^2-4=32$ 条边, 其中染红色 (实线)与蓝色(虚线)的各 16 条, 易知 $G$ 中不含同色三角形.\n$$\nn \\geqslant 33 \\text {. }\n$$\n下面我们再证明 $n \\leqslant 33$.\n设给定的 9 个点中有 33 条边染了色, 故此时有 3 边不染色, 不妨设为 $e_1$ 、 $e_2 、 e_3$. 在 $e_1 、 e_2 、 e_3$ 中各取一个端点 $v_1 、 v_2 、 v_3$, 从 $K_9$ 中删除这 3 个点, 其余的 6 点构成 $K_6$, 于是用红蓝两色染色必出现同色三角形.\n所以, $n=33$.\n为了推广定理一的结果, 我们先只增加染色的数目.\n用 $k$ 种颜色 $c_1, c_2, \\cdots, c_k$ 去染完全图 $K_n$ 的边, 每条边只染其中一种颜色, 这样得到的完全图 $K_n$ 简称为 $k$ 色完全图 $K_n$. 可以想象, 当阶数 $n$ 充分大时, $k$ 色完全图 $K_n$ 中就必然会出现同色三角形, 使得每一个 $k$ 色完全图 $K_n$ 都含同色三角形的最小 $n$ 记为 $r_k$, 于是定理一即是 $r_2=6$. 至于 $r_1=3$ 是不言自明的.", + "remark": "", + "figures": [ + "./images/volume12/figures/fig-c8i4.png" + ] +} \ No newline at end of file diff --git a/processed_dataset/proof/0446.json b/processed_dataset/proof/0446.json new file mode 100644 index 0000000000000000000000000000000000000000..0b6f1538190e75ba2c94a803bd2e7a54569d2d0d --- /dev/null +++ b/processed_dataset/proof/0446.json @@ -0,0 +1,10 @@ +{ + "source_file": "./raw_volume-zh/volume12/chapter8.tex", + "problem_type": "proof", + "problem": "例4. 证明: 在任何九个人中, 总可以找到三个人相互认识或四个人相互不认识.", + "solution": "证明:我们用 9 个点 $A_1, A_2, \\cdots, A_9$ 表示 9 个人, 每两点连一条边.\n约定: 若 $A_i 、 A_j$ 两人相互认识, 则线段 $A_i A_i$ 染红色, 否则线段 $A_i A_j$ 染蓝色.\n要证明的是: 在上述两色完全图 $K_9$ 中, 必存在红色 $K_3$, 或者存在蓝色 $K_4$.\n若有一个顶点出发的红边数 $\\geqslant 4$, 设为 $A_1 A_2 、 A_1 A_3 、 A_1 A_4 、 A_1 A_5$. 若 $A_2 、 A_3 、 A_4 、 A_5$ 中有两个点的连线是红色的, 例如 $A_2 A_3$, 则 $\\triangle A_1 A_2 A_3$ 为红色三角形.\n若 $A_2 、 A_3 、 A_4 、 A_5$ 中无两点连线是红色的, 则 $A_2 、 A_3 、 A_4 、 A_5$ 为顶点的子图是蓝色 $K_4$, 命题得证.\n若每个顶点出发的红边数 $<4$, 则每个顶点出发的蓝边数都 $\\geqslant 5$. 考虑 $A_1$, $A_2, \\cdots, A_9$ 为顶点连同所有蓝边构成的图, 奇顶点的个数是偶数, 所以必有一个顶点.\n例如 $A_1$ 是偶顶点, 即 $A_1$ 出发的蓝边数是偶数,所以 $A_1$ 出发的蓝边数 $\\geqslant 6$. 设 $A_1 A_2, A_1 A_3, \\cdots, A_1 A_7$ 为蓝边, 考虑六个顶点 $A_2, A_3, \\cdots, A_7$, 它们每两点都有红边或蓝边连接.\n按定理一, 或存在红色三角形, 或存在蓝色三角形.\n在前一种情况下, 命题已经得证.\n对后一种情况, 不妨设 $\\triangle A_2 A_3 A_4$ 为蓝色三角形, 则以 $A_1 、 A_2 、 A_3 、 A_4$ 为顶点的完全图 $K_4$ 是蓝色的.\n命题也成立.\n再考虑一个两色完全图 $K_8$, 如图() 所示.\n我们用实线表示染红色的边, 用虚线表示染蓝色的边.\n可见存在一种染法, 使 $K_8$ 无红色 $K_3$, 也无蓝色 $K_4$, 说明 $r(3,4)>8$.\n综上所述, 即得 $r(3,4)=9$.", + "remark": "", + "figures": [ + "./images/volume12/figures/fig-c8i6.png" + ] +} \ No newline at end of file diff --git a/processed_dataset/proof/0447.json b/processed_dataset/proof/0447.json new file mode 100644 index 0000000000000000000000000000000000000000..3b587d541a2e5f5032a1d552ae16260778567f27 --- /dev/null +++ b/processed_dataset/proof/0447.json @@ -0,0 +1,8 @@ +{ + "source_file": "./raw_volume-zh/volume12/chapter8.tex", + "problem_type": "proof", + "problem": "例5. 把数 $1 、 2 、 3 、 4 、 5$ 任意分成两组.\n证明: 总可以在某一组中找到这样两个数,它们之差与这一组中的某一个数相同.", + "solution": "证明:设把数 $1 、 2 、 3 、 4 、 5$ 任意分成了 $A 、 B$ 两组.\n取六个点, 并依次编号为 $1 、 2 、 3 、 4 、 5 、 6$. 其中任意两点 $i>j$, 都有 $1 \\leqslant i-j \\leqslant 5$. 两个点 $i>j$, 如果 $i-j$ 分在 $A$ 组, 则把 $i j$ 边染成红色; 如果 $i-j$ 分在 $B$ 组, 则把 $i j$ 边染成蓝色.\n于是, 得到一个 2 色 6 阶完全图 $K_6$. 由本节例 1 知, 这个 $K_6$ 中有一个单色三角形, 设为 $\\triangle i j k$, 并且 $i>j>k$. 这表明 $a=i-k, b=i-j, c=j-k$\n这三个数分在同一组中, 并且有\n$$\na-b=(i-k)-(i-j)=j-k=c .\n$$\n题中结论得证.", + "remark": "注:: 在此例中可能有 $b=c$, 此时有 $a=2 b$. 因此,此题可改写为: 把数 1 、 $2 、 3 、 4 、 5$ 任意分成两组, 证明: 总可以在某一组中找到这样一个数, 它是同一组中某个数的 2 倍,或者是同一组中某两个数之和.\n单色三角形的一种变形是所谓的异色三角形, 即三边颜色互不相同的三角形.\n题是匈牙利数学奥林匹克试题.", + "figures": [] +} \ No newline at end of file diff --git a/processed_dataset/proof/0448.json b/processed_dataset/proof/0448.json new file mode 100644 index 0000000000000000000000000000000000000000..1c8f9272b53fd72cda0c25dfafba273112aa298f --- /dev/null +++ b/processed_dataset/proof/0448.json @@ -0,0 +1,8 @@ +{ + "source_file": "./raw_volume-zh/volume12/chapter8.tex", + "problem_type": "proof", + "problem": "例6. 某俱乐部有 $3 n+1$ 人,每两人可一起玩下面三种游戏中的某一种: 象棋、围棋、跳棋.\n已知每个人都与 $n$ 个人下象棋, 与 $n$ 个人下围棋, 与 $n$ 个人下跳棋.\n证明: 这 $3 n+1$ 个人中必有这样三个人, 他们之间有下象棋的, 有下围棋的, 有下跳棋的.", + "solution": "证明:$3 n+1$ 个人用 $3 n+1$ 个点表示,两人之间下象棋、下围棋、下跳棋, 则对应的两点分别用红线、蓝线、黑线连接.\n于是, 得一个 3 色完全图 $K_{3 n+1}$. 本题即是证明: 在这个 3 色完全图 $K_{3 n+1}$ 中, 必存在一个三边不同色的异色三角形.\n由一顶点引出的两条边如果不同色, 则称此两条边的夹角为异色角.\n- 个三角形是异色三角形当且仅当它的三个内角都是异色角.\n每一个顶点引出的 $3 n$ 条边, 其中红边、蓝边、黑边各有 $n$ 条, 因此, 由任一顶点引出的异色角有 $\\mathrm{C}_3^2 n^2=3 n^2$ 个, 从而这个 3 色完全图 $K_{3 n+1}$ 中共有 $3 n^2(3 n+1)$ 个异色角.\n另一方面, 完全图 $K_{3 n+1}$ 中共有 $\\mathrm{C}_{3 n+1}^3=\\frac{1}{2} n(3 n+1)(3 n-1)$ 个三角形.\n把这些三角形看作\"抽庶\", 异色角看作\"球\". 因为 $3 n^2(3 n+-1)>n(3 n+1)(3 n-1)$, 于是, 3 色完全图 $K_{3 n+1}$ 中异色角的数目大于三角形个数的两倍, 由抽屉原理, 必有某个三角形,它有三个异色角,这个三角形即是异色三角形.\n数学竞赛中经常出现与拉姆赛问题类同的试题, 我们再举几例作为本节的结束.", + "remark": "", + "figures": [] +} \ No newline at end of file diff --git a/processed_dataset/proof/0449.json b/processed_dataset/proof/0449.json new file mode 100644 index 0000000000000000000000000000000000000000..04f8516655ac08f35d847537a625470ea93cd868 --- /dev/null +++ b/processed_dataset/proof/0449.json @@ -0,0 +1,8 @@ +{ + "source_file": "./raw_volume-zh/volume12/chapter8.tex", + "problem_type": "proof", + "problem": "例7. 大厅中会聚了 100 个客人,他们中每人至少认识 67 人, 证明: 在这些客人中一定可以找到 4 人,他们之中任何两人都彼此相识.", + "solution": "证明:用 $A_1, A_2, \\cdots, A_{100}$ 这 100 个顶点表示客人.\n连接每两点并染以红、蓝两色, 当且仅当 $A_i$ 与 $A_j$ 互相认识时, 它们之间的边是红色.\n本题用图论的语言就是: 红蓝 2 色完全图 $K_{100}$ 中, 如果每个顶点出发的红边至少有 67 条, 则 $K_{100}$ 中含有一个红色完全子图 $K_4$.\n任取一顶点 $A_1$, 自它出发的红边至少有 67 条, 故必有一条红边 $A_1 A_2$, 因自 $A_2$ 出发的红边也至少有 67 条,故自 $A_1 、 A_2$ 出发的蓝边至多有 $32 \\times 2=64$ 条, 它们涉及 66 个顶点, 因而必有一点, 例如 $A_3$, 使 $A_1 A_3 、 A_2 A_3$ 都是红边.\n自 $A_1 、 A_2 、 A_3$ 出发的蓝边至多有 $32 \\times 3=96$ 条, 涉及 99 个点, 故必还有一点, 记为 $A_4$, 使 $A_1 A_4 、 A_2 A_4 、 A_3 A_4$ 都是红边.\n于是以 $A_1 、 A_2 、 A_3 、 A_4$ 为顶点的完全子图 $K_4$ 是红色的.", + "remark": "", + "figures": [] +} \ No newline at end of file diff --git a/processed_dataset/proof/0450.json b/processed_dataset/proof/0450.json new file mode 100644 index 0000000000000000000000000000000000000000..942183afb216f89866668fd96d62fd5548c5c46b --- /dev/null +++ b/processed_dataset/proof/0450.json @@ -0,0 +1,8 @@ +{ + "source_file": "./raw_volume-zh/volume12/chapter8.tex", + "problem_type": "proof", + "problem": "例8. 一棱柱以五边形 $A_1 A_2 A_3 A_4 A_5$ 和 $B_1 B_2 B_3 B_4 B_5$ 为上、下底,这两个多边形的每一条边及每一条线段 $A_i B_j(i, j=1,2, \\cdots, 5)$ 均涂上红色或蓝色, 每一个以棱柱顶点为顶点的, 以已涂色的线段为边的三角形都不是同色三角形.\n求证:上、下底的 10 条边颜色一定相同.", + "solution": "证明:先证上底 5 条边同色.\n若不然, 则五边形 $A_1 A_2 A_3 A_4 A_5$ 至少有两条边不同色, 从而总有两条相邻边,譬如 $A_1 A_2$ 与 $A_1 A_5$ 不同色.\n不妨设 $A_1 A_2$ 为红边, $A_1 A_5$ 为蓝边.\n由 $A_1$ 到 $B_1 、 B_2 、 B_3 、 B_4 、 B_5$ 的 5 条边中至少有 3 条边同色, 不妨设 $A_1 B_i 、 A_1 B_j 、 A_1 B_k\\left(i, j, k\\right.$ 互不相等) 为红边.\n因为 $\\triangle A_1 B_i B_j$ 不同色, 所以 $B_i B_j$ 为蓝边, 同理 $A_2 B_i$ 也为蓝边, 进而推知 $A_2 B_j$ 为红边.\n于是 $\\triangle A_1 A_2 B_j$ 为红色三角形,这与题设矛盾.\n同理可证下底五条边同色.\n若上下底面的边不同色, 不妨设 $A_1 A_2 A_3 A_4 A_5$ 为红色, $B_1 B_2 B_3 B_4 B_5$ 为蓝色,并不妨设 $A_1 B_1$ 为蓝边, 于是由每个三角形不同色的假定, 可知 $A_1 B_5$ 、 $A_1 B_2$ 均为红线段,进而推出 $A_2 B_2 、 A_5 B_5$ 为蓝线段.\n同理可知 $A_5 B_1 、 A_5 B_4$ 、 $A_2 B_1 、 A_2 B_3$ 等均为红线段, $A_3 B_3 、 A_4 B_4$ 为蓝线段.\n于是 $A_4 B_1$ 和 $A_4 B_2$ 为蓝线段,这样得到蓝色三角形 $A_4 B_1 B_2$, 矛盾.\n所以棱柱上、下底面的 10 条边同色.", + "remark": "", + "figures": [] +} \ No newline at end of file diff --git a/processed_dataset/proof/0451.json b/processed_dataset/proof/0451.json new file mode 100644 index 0000000000000000000000000000000000000000..224577d53e385c47090dff919b6c9476d007f976 --- /dev/null +++ b/processed_dataset/proof/0451.json @@ -0,0 +1,10 @@ +{ + "source_file": "./raw_volume-zh/volume12/chapter8.tex", + "problem_type": "proof", + "problem": "例9. 10 个地区之间有甲、乙两个国际航空公司服务,在任意两个地区之间都有且仅有由其中一个公司单独经营的直达航线 (可往返). 证明:两公司中必有某公司, 可以提供两条不经过同一地区的环游旅行线, 而且每条旅行线经过奇数个地区.", + "solution": "证明:10 个地区用 10 个点 $u_1, u_2, \\cdots, u_{10}$ 表示, $u_i$ 与 $u_j$ 两地区的航线若由甲公司经营, 则 $u_i u_j$ 边染红色 (图中用实线表示); 若由乙公司经营, 则 $u_i u_j$ 边染蓝色 (图中用虚线表示). 于是得到一个 2 色 10 阶完全图 $K_{10}$. 为证明结论, 即要证明这个 2 色 10 阶完全图 $K_{10}$ 中, 存在两个没有公共顶点、边数都是奇数并且同色的单色三角形或单色多边形.\n2 色 10 阶完全图 $K_{10}$ 中必有单色三角形,设 $\\triangle u_8 u_9 u_{10}$ 为单色三角形.\n由例 1 又知, 以 $u_1, u_2, \\cdots, u_7$ 为顶点的三角形中必有单色三角形, 设 $\\triangle u_5 u_6 u_7$ 为单色三角形.\n如果 $\\triangle u_5 u_6 u_7$ 与 $\\triangle u_8 u_9 u_{10}$ 同色, 则已无需再证.\n下面设 $\\triangle u_5 u_6 u_7$ 为红边三角形, 而 $\\triangle u_8 u_9 u_{10}$ 为蓝边三角形.\n在 $\\left\\{u_5, u_6, u_7\\right\\}$ 与 $\\left\\{u_8, u_9, u_{10}\\right\\}$ 两个顶点集之间共有 $3 \\times 3=9$ 条边,由抽庶原理知,其中必有 5 条同色,不妨为红色.\n这 5 条边由 $\\left\\{u_8, u_9, u_{10}\\right\\}$ 引出, 故必有某一顶点, 由该点引出 2 条红边, 设为 $u_8 u_6 、 u_8 u_7$, 如图() 所示.\n于是, 又有一个红边三角形 $u_6 u_7 u_8$.\n考虑由 $u_1 、 u_2 、 u_3 、 u_4 、 u_5$ 为顶点的 2 色 5 阶完全图 $K_5$. 如果这个 $K_5$ 中有单色三角形, 则不论这个单色三角形是红色边或者是蓝色边的, 再加上红边三角形 $u_6 u_7 u_8$ 或蓝边三角形 $u_8 u_9 u_{10}$, 在 2 色完全图 $K_{10}$ 中都有两个没有公共顶点、同色的单色三角形.\n否则, 这个 2 色 5 阶完全图 $K_5$ 中没有单色三角形, 于是易知 $K_5$ 中有两个单色五边形, 一个红色边五边形, 另一个蓝色边五边形.\n因此结论成立.", + "remark": "注:: 上题中若把 10 个地区改为 9 个地区,则结论不真.\n例子如下: 把 9 个地区分为 3 组, 即 $\\left\\{u_1, u_2, u_3, u_4, u_5\\right\\}=A,\\left\\{u_6, u_7, u_8\\right\\}=B,\\left\\{u_9\\right\\}, A$ 中的 5 个地区之间的航线由甲公司经营, $B$ 中的 3 个地区之间的航线由乙公司经营, $A$ 与 $B$ 之间、 $u_9$ 与 $A$ 之间的航线由乙公司经营, $u_9$ 与 $B$ 之间的航线由甲公司经营.", + "figures": [ + "./images/volume12/figures/fig-c8i8.png" + ] +} \ No newline at end of file diff --git a/processed_dataset/proof/0452.json b/processed_dataset/proof/0452.json new file mode 100644 index 0000000000000000000000000000000000000000..ff8be1b83133a707097725e0ef8b4193b1ebc7fc --- /dev/null +++ b/processed_dataset/proof/0452.json @@ -0,0 +1,8 @@ +{ + "source_file": "./raw_volume-zh/volume12/chapter9.tex", + "problem_type": "proof", + "problem": "例1. $n$ 个参赛者 $P_1, P_2, \\cdots, P_n(n>1)$ 进行循环赛, 每个参赛者同其他 $(n-1)$ 个参赛者都进行一局比赛.\n假设比赛结果没有平局出现, $w_r$ 和 $l_r$ 分别表示参赛者 $P_r$ 胜与负的局数,求证:\n$$\nw_1^2+w_2^2+\\cdots+w_n^2=l_1^2+l_2^2+\\cdots+l_n^2 .\n$$", + "solution": "证明:作一竞赛图 $\\bar{K}_n$, 每个顶点 $v_r$ 对应于参赛者 $P_r$, 如果参赛者 $P_i$ 胜了 $P_j$, 则在顶点 $v_i, v_j$ 之间作弧 $\\left(v_i, v_j\\right)$. 于是 $w_r$ 和 $l_r$ 分别是顶点 $v_r$ 的出度和人度.\n根据定理一,\n$$\nw_1+w_2+\\cdots+w_n=l_1+l_2+\\cdots+l_n .\n$$\n若注意到 $w_i+l_i=n-1(1 \\leqslant i \\leqslant n)$, 可得\n$$\n\\begin{aligned}\n& w_1^2+w_2^2+\\cdots+w_n^2-\\left(l_1^2+l_2^2+\\cdots+l_n^2\\right) \\\\\n= & \\left(w_1^2-l_1^2\\right)+\\left(w_2^2-l_2^2\\right)+\\cdots+\\left(w_n^2-l_n^2\\right) \\\\\n= & \\left(w_1+l_1\\right)\\left(w_1-l_1\\right)+\\left(w_2+l_2\\right)\\left(w_2-l_2\\right) \\\\\n& +\\cdots+\\left(w_n+l_n\\right)\\left(w_n-l_n\\right) \\\\\n= & (n-1)\\left[\\left(w_1+w_2+\\cdots+w_n\\right)-\\left(l_1+l_2+\\cdots+l_n\\right)\\right]=0 .\n\\end{aligned}\n$$\n从而有\n$$\nw_1^2+w_2^2+\\cdots+w_n^2=l_1^2+l_2^2+\\cdots+l_n^2 .\n$$\n在有向图 $D=(v, u)$ 中,一个由不同的弧组成的序列 $u_1, u_2, \\cdots, u_n$, 若其中 $u_i$ 的起点为 $v_i$, 终点为 $v_{i+1}(i=1,2, \\cdots, n)$, 称这个序列为从 $v_1$ 到 $v_{n+1}$ 的有向路 (简称路), $n$ 称为这个路的长.\n$v_1$ 为路的起点, $v_{n+1}$ 为路的终点.\n如果 $v_1=v_{n+1}$, 则称这个路为回路.", + "remark": "", + "figures": [] +} \ No newline at end of file diff --git a/processed_dataset/proof/0453.json b/processed_dataset/proof/0453.json new file mode 100644 index 0000000000000000000000000000000000000000..262236c856b22be404e606afa3b02014b38ab0ef --- /dev/null +++ b/processed_dataset/proof/0453.json @@ -0,0 +1,8 @@ +{ + "source_file": "./raw_volume-zh/volume12/chapter9.tex", + "problem_type": "proof", + "problem": "例2. $\\mathrm{MO}$ 太空城由 99 个空间站组成,任两个空间站之间有管形通道相联.\n规定其中 99 条通道为双向通行的主干道, 其余通道严格单向通行.\n如果某四个空间站可以通过它们之间的通道从其中任一站到达另外任一站, 则称这四个站的集合为一个互通四站组.\n试为 $\\mathrm{MO}$ 太空城设计一个方案, 使得互通四站组的数目最大 (请具体算出该最大数,并证明你的结论).", + "solution": "证明:将不能互通的四站组称之为坏四站组, 于是坏四站组有 3 种可能情形 :\n(1) 站 $A$ 引出的 3 条通道 $A B 、 A C 、 A D$ 全都离开 $A$;\n(2) 站 $A$ 引出的 3 条通道全都走进 $A$;\n(3) 站 $A$ 与 $B 、 C$ 与 $D$ 之间都是双向通道,但通道 $A C 、 A D$ 都离开 $A$, $B C 、 B D$ 都离开 $B$.\n将第(1)种的所有坏四站组的集合记为 $S$, 其余坏四站组的集合记为 $T$. 我们来计算 $|S|$.\n因为太空城共有 $\\mathrm{C}_{99}^2-99=99 \\times 48$ 条单行通道, 设第 $i$ 站走出的通道数为 $S_i$, 于是\n$$\n\\sum_{i=1}^{99} S_i=99 \\times 48\n$$\n这时从站 $A_i$ 引出 3 条通道的 (1) 类坏四站组有 $\\mathrm{C}_{S_i}^3$ 个.\n从而\n$$\n|S|=\\sum_{i=1}^{99} \\mathrm{C}_{S_i}^3 \\geqslant 99 \\times \\mathrm{C}_{48}^3 .\n$$\n上面的不等号是因为 $\\mathrm{C}_x^3=\\frac{1}{6} x(x-1)(x-2)$ 在 $x \\geqslant 3$ 时是凸函数, 又因为所有四站组总数为 $\\mathrm{C}_{99}^4$, 所以互通四站组的个数不多于\n$$\n\\mathrm{C}_{99}^4-|S| \\leqslant \\mathrm{C}_{99}^4-99 \\mathrm{C}_{48}^3 \\text {. }\n$$\n下面构造一个例子使互通四站组恰为 $C_{99}^4-99 C_{48}^3$ 个.\n为此, 必须使每个站 $A_i$ 走出的通道数都是 48 , 从而走人的通道数也是 48 , 且每站恰有两条双向通道, 同时保证只有 $S$ 类坏四站组没有 $T$ 类坏四站组.\n将 99 个空间站写在一个圆内接正 99 边形的 99 个顶点上.\n规定正 99 边形的最长对角线为双向通道, 于是每点都恰有两条双向通道.\n对于站 $A_i$, 按顺时针方向接下去的 48 个站与 $A_i$ 的单向通道都从 $A_i$ 走出; 按逆时针方向接下去的 48 个站都是走向 $A_i$ 的.\n下面我们验证, 在这个规定下只有 $S$ 类坏四站组而没有 $T$ 类坏四站组.\n设 $\\{A, B, C, D\\}$ 是任一四站组.\n(i)若四站之间有两条双向通道,则显然是连通的;\n(ii) 若 4 站之间有唯一的双向通道 $A C$,则 $B$ 和 $D$ 分别与 $A 、 C$ 形成一个环路, 从而也是互通的.\n(iii) 故坏四站组之间没有双向通道, 只能是 (1)(2)两种情形之一.\n若为 (2), 不妨设 $A$ 的 3 条通道全是走进 $A$ 的, 于是 $B 、 C 、 D$ 都在 $A$ 算起逆时针方向的 48 个站内, 设其 $D$ 离 $A$ 最远, 从而 $A D 、 B D 、 C D$ 都走出 $D$. 这表明坏四站组都是 $S$ 类的.\n综上可知,互通四站组最多 $\\mathrm{C}_{99}^4-99 \\mathrm{C}_{48}^3$ 组.", + "remark": "", + "figures": [] +} \ No newline at end of file diff --git a/processed_dataset/proof/0454.json b/processed_dataset/proof/0454.json new file mode 100644 index 0000000000000000000000000000000000000000..a7b6061a845f0701834bc012f2c43348267adab2 --- /dev/null +++ b/processed_dataset/proof/0454.json @@ -0,0 +1,8 @@ +{ + "source_file": "./raw_volume-zh/volume12/chapter9.tex", + "problem_type": "proof", + "problem": "例4. $n(n \\geqslant 3)$ 个人参加单循环比赛, 通过比赛确定优秀选手.\n选手 $A$ 被确定为优秀选手的条件是: 对任何其他选手 $B$, 或 $A$ 胜 $B$, 或存在选手 $C, C$ 胜 $B, A$ 胜 $C$. 如果按上述规则确定的优秀选手只有一名, 试证这名选手战胜所有其他选手.", + "solution": "证明:将 $n$ 个选手对应 $n$ 个点, 若选手 $v_i$ 胜 $v_j$, 则作一条从 $v_i$ 到 $v_j$ 的弧, 得到一个竞赛图 $\\bar{K}_n$. 不妨设 $v_1$ 是 $\\bar{K}_n$ 中出度最大的点, 由定理二知, $v_1$ 就是优秀选手.\n要证明的是顶点 $v_1$ 到别的顶点有长为 1 的路 (即弧), 也就是 $v_1$ 的人度 $d^{-}\\left(v_1\\right)=0$.\n假设命题结论不真, 记以 $v_1$ 为终点的弧的起点集合为 $N^{-}\\left(v_1\\right)=\\left\\{v_{i_1}\\right.$, $\\left.v_{i_2}, \\cdots, v_{i_r}\\right\\}, r \\geqslant 1$. 考虑由顶点 $v_{i_1}, v_{i_2}, \\cdots, v_{i_r}$ 组成的竞赛图 $\\bar{K}_r$, 取 $v_{i_1}$ 是 $\\bar{K}_r$ 中的出度最大的点.\n据定理二, $v_{i_1}$ 到 $v_{i_2}, \\cdots, v_{i_r}$ 各点有不大于 2 的路.\n又由于 $v_1$ 有到除 $v_{i_1}, \\cdots, v_{i_r}$ 外的所有其他顶点的弧, 故 $v_{i_1}$ 到除 $\\left\\{v_{i_1}, \\cdots, v_{i_r}\\right\\}$ 外的点也有不大于 2 的路, 因而, 在竞赛图 $\\bar{K}_n$ 中 $v_{i_1}$ 到其他各点均有不大于 2 的路, 于是 $v_{i_1}$ 也是优秀选手, 这与 $v_1$ 是唯一的优秀选手矛盾, 从而 $N^{-}\\left(v_1\\right)= \\varnothing$, 即 $d^{-}\\left(v_1\\right)=0$. 命题得证.", + "remark": "注:: 本题说明了竞赛图 $\\bar{K}_n$ 的一个性质: 若 $\\bar{K}_n$ 中出度最大的点唯一, 则这点的出度为 $n-1$.", + "figures": [] +} \ No newline at end of file diff --git a/processed_dataset/proof/0455.json b/processed_dataset/proof/0455.json new file mode 100644 index 0000000000000000000000000000000000000000..0c8a2064eee9f145b085e2c9a8a27519229b4557 --- /dev/null +++ b/processed_dataset/proof/0455.json @@ -0,0 +1,8 @@ +{ + "source_file": "./raw_volume-zh/volume12/chapter9.tex", + "problem_type": "proof", + "problem": "例5. 在象棋赛中, 每两名选手都要赛一场, 证明: 我们可以给所有参赛选手编号, 使得无论哪一一个选手都没有输给紧接在他后面编号的那个选手.", + "solution": "证明:设有 $n$ 名选手, 用 $n$ 个顶点 $v_1, v_2, \\cdots, v_n$ 表示这 $n$ 名选手, 当选手 $v_i$ 没有输给 $v_j$ 时, 从 $v_i$ 向 $v_j$ 引一条弧 $\\left(v_i, v_j\\right)$, 这样就得到了一个竞赛图 $\\bar{K}_n$. 由定理三, $\\bar{K}_n$ 中有哈密顿路, 则按此路上选手出现的顺序为他们编号即可.", + "remark": "", + "figures": [] +} \ No newline at end of file diff --git a/processed_dataset/proof/0456.json b/processed_dataset/proof/0456.json new file mode 100644 index 0000000000000000000000000000000000000000..1e38c3be78e2dfafb6c0ddcc3a3a30c6259ef6c2 --- /dev/null +++ b/processed_dataset/proof/0456.json @@ -0,0 +1,8 @@ +{ + "source_file": "./raw_volume-zh/volume12/exercise1.tex", + "problem_type": "proof", + "problem": "问题2. 设图 $G=(V,\\,E)$ 是简单图, $\\vert V\\vert=n,\\vert E\\vert=e,$ ,证明 $e\\leq{\\frac{n(n-1)}{2}}.$", + "solution": "简单图每条边与两个不同顶点关联,且每两个顶点间至多有一条边连接, 现有 $n$ 个顶点, 故边数至多为 $\\mathrm{C}_n^2=\\frac{n(n-1)}{2}$, 由于简单图不一定是完全图, 所以 $e \\leqslant \\frac{n(n-1)}{2}$.", + "remark": "", + "figures": [] +} \ No newline at end of file diff --git a/processed_dataset/proof/0457.json b/processed_dataset/proof/0457.json new file mode 100644 index 0000000000000000000000000000000000000000..03cb81c5a07c1798657213964e1fd93d14ba198a --- /dev/null +++ b/processed_dataset/proof/0457.json @@ -0,0 +1,8 @@ +{ + "source_file": "./raw_volume-zh/volume12/exercise1.tex", + "problem_type": "proof", + "problem": "问题5. 一次会议有 $n$ 名教授 $A_1, A_2, \\cdots, A_n$ 参加, 证明可以将这 $n$ 个人分为两组, 使得每一个人 $A_i$ 在另一组中认识的人数 $d_i$ 不少于他在同一组中认识的人数 $d_i^{\\prime}(i=1,2, \\cdots, n)$.", + "solution": "用 $n$ 个点 $v_1, v_2, \\cdots, v_n$ 表示这 $n$ 名教授, 并在相互认识的人之间连一条边, 将这 $n$ 个点任意分成两组, 只有有限多种分法.\n考虑在两组之间的连线条数 $S$, 其中必存在一种分法, 使 $S$ 达到最大值.\n此时定有 $d_i \\geqslant d_i^{\\prime}$ ( $i= 1,2, \\cdots, n)$. 若不然, 设对 $v_1$, 有 $d_10$. 这与 $S$ 已达到最大值矛盾.", + "remark": "", + "figures": [] +} \ No newline at end of file diff --git a/processed_dataset/proof/0458.json b/processed_dataset/proof/0458.json new file mode 100644 index 0000000000000000000000000000000000000000..0c266b99424d919bb71ae57689df1b53550e31d9 --- /dev/null +++ b/processed_dataset/proof/0458.json @@ -0,0 +1,8 @@ +{ + "source_file": "./raw_volume-zh/volume12/exercise1.tex", + "problem_type": "proof", + "problem": "问题6. 18 个队进行比赛, 每一轮中每个队与另一个队比赛一场, 并且在其他轮比赛中这两个已赛过的队彼此不再比赛, 现在比赛已进行完 8 轮.\n证明一定有三个队在前 8 轮比赛中, 彼此之间尚未比赛过.", + "solution": "设 $A$ 队经过 8 轮之后与 8 个队赛过, 而与其他 9 个队没有赛过, 若这 9 个队在前 8 轮中相互之间都赛过, 由于每队只赛了 8 场, 所以这 9 个队与其他各队都没有赛过.\n但这 9 个队中第一轮比赛只能赛 4 场, 所以必有一个队要与其他队比赛, 矛盾.\n所以在这 9 个队中必有两个队 $B, C$, 它们之间没有赛过, 这样 $A, B, C$ 三队之间便彼此没有赛过了.", + "remark": "", + "figures": [] +} \ No newline at end of file diff --git a/processed_dataset/proof/0459.json b/processed_dataset/proof/0459.json new file mode 100644 index 0000000000000000000000000000000000000000..1bcc45beb2ea8585c5f01b4c638705fb453ba352 --- /dev/null +++ b/processed_dataset/proof/0459.json @@ -0,0 +1,8 @@ +{ + "source_file": "./raw_volume-zh/volume12/exercise1.tex", + "problem_type": "proof", + "problem": "问题7. 某次会议有 $n$ 名代表出席, 已知任意的四名代表中都有一个人与其余的三个人握过手, 证明任意的四名代表中必有一个人与其余的 $n-1$ 名代表都握过手.", + "solution": "$n$ 名代表用 $n$ 个点表示, 如果两名代表没有握过手, 就在相应的顶点之间连一条边, 得图 $G$. 如果 $G$ 中任意 4 点 $v_1, v_2, v_3, v_4$ 中, 每一点都有与之相邻的点, 分别设为 $v_1^{\\prime}, v_2^{\\prime}, v_3^{\\prime}, v_4^{\\prime}$. 由已知条件知, $v_1, v_2, v_3, v_4$ 中有一点, 不妨设为 $v_1$, 与其余三点 $v_2, v_3, v_4$ 均不相邻.\n所以 $v_1^{\\prime} \\neq v_2, v_3, v_4$. 如果 $v_2^{\\prime} \\neq v_1^{\\prime}$, 则在 $v_1, v_2, v_1^{\\prime}, v_2^{\\prime}$ 这 4 个点中, 没有一个点与其余的三点均不相邻, 所以 $v_2^{\\prime}=v_1^{\\prime}$. 同理 $v_3^{\\prime}=v_1^{\\prime}$. 于是在 $v_1, v_2, v_3, v_1^{\\prime}$ 这 4 个点中, 又没有一个点与其余的三个点均不相邻.\n所以在任意 4 个顶点中必有一个点与其余的 $n-1$ 个点均不相邻.", + "remark": "", + "figures": [] +} \ No newline at end of file diff --git a/processed_dataset/proof/0460.json b/processed_dataset/proof/0460.json new file mode 100644 index 0000000000000000000000000000000000000000..f86282aae25fa875f9100ee39b1d8ace705f4dd9 --- /dev/null +++ b/processed_dataset/proof/0460.json @@ -0,0 +1,10 @@ +{ + "source_file": "./raw_volume-zh/volume12/exercise1.tex", + "problem_type": "proof", + "problem": "问题8. 有三所中学, 每所有学生 $n$ 名.\n每名学生都认识其他两所中学的 $n+1$ 名学生.\n证明: 从每所中学可以选出一名学生, 使选出来的 3 名学生互相认识.", + "solution": "如图(), 用 $3 n$ 个顶点表示这些学生,三所中学的学生组成的三个顶点集合分别记为 $X, Y$ 和 $Z$. 若 $u$ 和 $v$ 是不同学校的学生, 而且是互相认识的, 则在 $u$ 与 $v$ 之间连一条边, 这样便得图 $G$. 设 $x \\in X, Y$ 和 $Z$ 中和 $x$ 相邻的顶点数记作 $k$ 和 $l$, 则 $k+l=n+1$. $k$ 与 $l$ 中大的记作 $m(x)$, 让 $x$ 跑遍 $X, m(x)$ 的最大值记作 $m_X, m_Y$ 与 $m_Z$ 作同样理解.\n数 $m_X, m_Y, m_Z$ 中的最大值记作 $m$, 不妨设 $m=m_X$, 并且 $x_0 \\in X$,\n使得 $Y$ 中和 $x_0$ 相邻的顶点集合 $Y_1$ 中顶点数 $\\left|Y_1\\right|=m$. 于是 $Z$ 中与 $x_0$ 相邻的顶点数为 $n+1-m \\geqslant 1$. 设 $z_0 \\in Z$ 与 $x_0$ 相邻.\n如果有 $y_0 \\in Y_1$ 与 $z_0$ 相邻, 则 $\\triangle x_0 y_0 z_0$ 是 $G$ 中的一个三角形.\n若 $Y_1$ 中每一个 $y$ 与 $z_0$ 都不相邻, 则 $Y$ 中与 $z_0$ 相邻的顶点数 $\\leqslant n-m$. 因此 $z_0$ 与 $X$ 中相邻的顶点数 $\\geqslant n+1-(n-m)= m+1$,与 $m$ 的最大性矛盾.\n于是证得 $G$ 中必有 $\\triangle x_0 y_0 z_0$.", + "remark": "", + "figures": [ + "./images/volume12/figures/fig-c1a8.png" + ] +} \ No newline at end of file diff --git a/processed_dataset/proof/0461.json b/processed_dataset/proof/0461.json new file mode 100644 index 0000000000000000000000000000000000000000..a739eb0944349cfddbc6487242a60c6abaae639b --- /dev/null +++ b/processed_dataset/proof/0461.json @@ -0,0 +1,8 @@ +{ + "source_file": "./raw_volume-zh/volume12/exercise1.tex", + "problem_type": "proof", + "problem": "问题9. 一个很大的棋盘上有 $2 n$ 个红色的方格, 对任何两个红色方格可从其中一个出发, 每步横或坚走到相邻的红色方格而到另一个方格中.\n证明: 所有的红色方格可以分为 $n$ 个长方形.", + "solution": "当 $n=1$ 时,有 2 个红格相邻, 显然为一个矩形.\n设当 $n \\leqslant k$ 时成立, 即可以将 $2 k$ 个连通方格分成 $k$ 个矩形.\n当 $n=k+1$ 时,\n(i) 对于 $2 k+2$ 个方格中, 若去掉一对相邻的红格后有一个图仍为连通图, 则由归纳假设结论成立.\n(ii) 若去掉一对相邻的红格被分成若干个连通图, 而每个图的红格个数为偶数时, 由归纳假设结论成立.\n(iii) 若去掉任何一对相邻的红格被分成若干个连通图, 而其中存在连通图的红格个数为奇数时:\n当 $n=2$ 时,有\"T\"形图 $1 \\times 3$ 和 $1 \\times 1$ 的两个矩形满足要求.\n当 $n \\neq 2$ 时,观察所有方格中左上角的\"T\"形图,去掉这两个方格至多形成两个连通图.\n若去掉左上角的两个后所形成的两个连通图的红格的个数均为奇数,则去掉 $1 \\times 3$ 和 $1 \\times 1$ 的两个矩形后, 易知仍为两个连通图, 而红格的个数为偶数.\n综上所述, $n=k+1$ 也成立,故结论成立.", + "remark": "", + "figures": [] +} \ No newline at end of file diff --git a/processed_dataset/proof/0462.json b/processed_dataset/proof/0462.json new file mode 100644 index 0000000000000000000000000000000000000000..68219ba1f35488dc5331be2c057fa08eb3c71722 --- /dev/null +++ b/processed_dataset/proof/0462.json @@ -0,0 +1,8 @@ +{ + "source_file": "./raw_volume-zh/volume12/exercise1.tex", + "problem_type": "proof", + "problem": "问题10. 某参观团有 2000 个人, 其中任意 4 个人中一定有某一个人认识其他三人.\n问: 认识该参观团所有成员的人数最少是多少?", + "solution": "如果 2000 个成员都彼此认识, 则认识参观团所有成员的人数为 2000. 因此,不妨设有某两个成员 $u$ 与 $v$ 互不认识.\n下面分三个步骤.\n(i)除 $u, v$ 外的任意两个成员必彼此认识.\n设 $a, b$ 是另外两个成员, 由题设,在 $a, b, u, v$ 这 4 个成员中, 必有一人认识其余三人, 这个人只能是 $a$ 或 $b$,这表明 $a, b$ 互相认识.\n(ii)如果 $u, v$ 两人都认识其他的 1998 个人中的每一个,则该参观团有 1998 个人认识所有其他成员.\n设 $a$ 是除 $u, v$ 外的任意一个成员, 由假设知 $a$ 认识 $u, v$. 设 $b$ 是另一成员, 由前所证 $a$ 与 $b$ 必彼此认识.\n由 $b$ 的任意性知 $a$ 认识参观团的所有其他成员; 又由 $a$ 的任意性知该参观团除 $u, v$ 外的 1998 个人认识所有其他成员.\n(iii)如果 $u, v$ 中某一个不全认识其他 1998 个成员, 则该参观团有 1997 个人认识所有其他成员, 不妨设除 $v$ 外, $u$ 不认识另一个成员 $w$. 设 $a$ 是 $u, v$, $w$ 外的 1997 个成员中的任意一个,由题设,在 $a, u, v, w$ 中认识另外三人的人只能是 $a$, 这表明 $u, v, w$ 这三个人的每一个都认识该参观团中的其余 1997 人.\n综上,认识该参观团所有成员的人数最少是 1997 个.", + "remark": "", + "figures": [] +} \ No newline at end of file diff --git a/processed_dataset/proof/0463.json b/processed_dataset/proof/0463.json new file mode 100644 index 0000000000000000000000000000000000000000..71019c214088a5df48444694891571b63fcca149 --- /dev/null +++ b/processed_dataset/proof/0463.json @@ -0,0 +1,8 @@ +{ + "source_file": "./raw_volume-zh/volume12/exercise1.tex", + "problem_type": "proof", + "problem": "问题11. 在一个车相里, 任何 $m(m \\geqslant 3)$ 个旅客都有唯一的公共朋友 (当甲是乙的朋友时, 乙也是甲的朋友.\n任何人不作为他自己的朋友). 问: 在这个车厢里,有多少人?", + "solution": "根据条件,每个人都有朋友.\n如果 $k(k \\leqslant m)$ 个人彼此是朋友, 由于他们有一个公共的朋友, 所以 $k+1$ 个人彼此是朋友.\n依此类推, 导出有 $m+1$ 个人 $A_1, A_2, \\cdots, A_{m+1}$ 彼此是朋友.\n下面证车厢中除 $A_1, A_2, \\cdots, A_{m+1}$ 外,别无他人.\n若 $B$ 是这 $m+1$ 个人以外的人, 并且 $B$ 至少与 $A_1, A_2, \\cdots, A_{m+1}$ 中两个人是朋友.\n设 $B$ 与 $A_1 、 A_2$ 是朋友, 则 $B, A_3, A_4, \\cdots, A_{m+1}$ 这 $m$ 个人有两个公共的朋友 $A_1 、 A_2$, 与已知矛盾.\n因此 $A_1, A_2, \\cdots, A_{m+1}$ 之外的人 $B$ 至多与 $A_1, A_2, \\cdots, A_{m+1}$ 中一个人是朋友.\n故不妨设除 $A_1$ 外, $A_2, A_3, \\cdots, A_{m+1}$ 都不是 $B$ 的朋友.\n于是 $m$ 个旅客 $B$, $A_1, A_2, \\cdots, A_{m-1}$ 的公共朋友 $C$, 当然不是 $A_2, A_3, \\cdots, A_{m+1}$, 也不是 $A_1$. 由于\n$m \\geqslant 3, C$ 与 $A_1, A_2, \\cdots, A_{m-1}$ 中 $m-1 \\geqslant 2$ 个人是朋友.\n这与上面已证 $C$ 至多与 $A_1, \\cdots, A_{m+1}$ 中一个人是朋友矛盾.\n于是车相中只有 $A_1, A_2, \\cdots, A_{m+1}$ 这 $m+1$ 个人, 每个人的朋友恰好是 $m$ 个.", + "remark": "", + "figures": [] +} \ No newline at end of file diff --git a/processed_dataset/proof/0464.json b/processed_dataset/proof/0464.json new file mode 100644 index 0000000000000000000000000000000000000000..2ad3c02a0ebbc1b8fc5228713631a46c83462cbc --- /dev/null +++ b/processed_dataset/proof/0464.json @@ -0,0 +1,8 @@ +{ + "source_file": "./raw_volume-zh/volume12/exercise1.tex", + "problem_type": "proof", + "problem": "问题12. 平面上给定五点 $A 、 B 、 C 、 D 、 E$, 其中任三点不共线, 试证: 任意用线段连接某些点 (这些线段称为边), 若所得图形中不出现以这五点中任三点为顶点的三角形, 则此图不可能有 7 条或更多的边.", + "solution": "$K_5$ 有 $\\mathrm{C}_5^2=10$ 条边, $\\mathrm{C}_5^3=10$ 个三角形,与每条边有关的三角形恰有 3 个.\n若图中有不少于 7 条边, 则即从 $K_5$ 中至多去掉 3 边, 于是至多去掉 $3 \\times 3=9$ 个三角形,所以仍有 1 个三角形,与题设矛盾.\n所以此图不可能有 7 条或更多的边.", + "remark": "", + "figures": [] +} \ No newline at end of file diff --git a/processed_dataset/proof/0465.json b/processed_dataset/proof/0465.json new file mode 100644 index 0000000000000000000000000000000000000000..abf7ccfdeee30feb93e612329a27a9fd63303804 --- /dev/null +++ b/processed_dataset/proof/0465.json @@ -0,0 +1,8 @@ +{ + "source_file": "./raw_volume-zh/volume12/exercise2.tex", + "problem_type": "proof", + "problem": "问题1. 设图 $G=(V, E),|V|=n,|E|=e$, 证明 $: \\delta \\leqslant \\frac{2 e}{n} \\leqslant \\Delta$.", + "solution": "因为 $\\sum_{i=1}^n d\\left(v_i\\right)=2 e, n \\delta \\leqslant 2 e \\leqslant n \\Delta$, 所以 $\\delta \\leqslant \\frac{2 e}{n} \\leqslant \\Delta$.", + "remark": "", + "figures": [] +} \ No newline at end of file diff --git a/processed_dataset/proof/0466.json b/processed_dataset/proof/0466.json new file mode 100644 index 0000000000000000000000000000000000000000..70ba38e2d4d2de612368145842cc8ff539806c66 --- /dev/null +++ b/processed_dataset/proof/0466.json @@ -0,0 +1,8 @@ +{ + "source_file": "./raw_volume-zh/volume12/exercise2.tex", + "problem_type": "proof", + "problem": "问题2. 证明任何一群人 (多于两个人)中,至少有两个人,他们的朋友数目相同.", + "solution": "设任意给定的一群人有 $n(n>2)$ 个.\n用顶点表示这 $n$ 个人.\n当且仅当顶点 $u, v$ 表示的两个人是朋友时令 $u, v$ 相邻, 得到 $n$ 个顶点的简单图 $G$.\n对 $G$ 中任意 $x$, 由于它至多只能和其他 $n^{--}$个顶点相邻, 所以顶点 $x$ 的度 $d(x)$ 满足 $0 \\leqslant d(x) \\leqslant n-1$, 即图 $G$ 的顶点度只能是 $n-1$ 个非负数 $0,1, \\cdots, n-1$ 中的一个.\n如果图 $G$ 的顶点的度都不相同, 则图 $G$ 具有 0 度顶点 $u$ 和 $n-1$ 度顶点 $v . n-1$ 度顶点和 $G$ 中其他顶点都相邻, 特别地和顶点 $u$ 相邻.\n但 0 度顶点 $u$ 和 $G$ 中任何顶点都不相邻, 矛盾.\n这就证明了 $G$ 中必定有两个顶点, 它们的度相同.\n也就是说, 这群人中必有两个人, 他们的朋友一样多.", + "remark": "", + "figures": [] +} \ No newline at end of file diff --git a/processed_dataset/proof/0467.json b/processed_dataset/proof/0467.json new file mode 100644 index 0000000000000000000000000000000000000000..0a65c38185f9db1d4b10033621111d5d6ea53d7e --- /dev/null +++ b/processed_dataset/proof/0467.json @@ -0,0 +1,8 @@ +{ + "source_file": "./raw_volume-zh/volume12/exercise2.tex", + "problem_type": "proof", + "problem": "问题3. $n$ 名选手进行对抗赛, 每名选手至少赛一场, 每场两名选手参加, 已赛完 $n+1$ 场.\n证明:至少有一名选手赛过三次.", + "solution": "把选手看成顶点.\n当且仅当 $v_i, v_j$ 所代表的两名选手比赛过时, 令 $v_i$, $v_j$ 相邻, 得到含 $n$ 个顶点的简单图 $G$. 由于总共赛过 $n+1$ 场, 所以图 $G$ 的边数是 $n+1$.\n由定理, $d\\left(v_1\\right)+d\\left(v_2\\right)+\\cdots+d\\left(v_n\\right)=2(n+1)$,\n如果图 $G$ 中所有顶点的度都不超过 2 , 则由上式得到\n$$\n2(n+1)=d\\left(v_1\\right)+d\\left(v_2\\right)+\\cdots+d\\left(v_n\\right) \\leqslant 2 n,\n$$\n这不可能.\n因此图 $G$ 中至少有一个顶点 $x$, 它的度至少是 3. 于是, 顶点 $x$ 所表示的选手至少赛过三次.", + "remark": "", + "figures": [] +} \ No newline at end of file diff --git a/processed_dataset/proof/0468.json b/processed_dataset/proof/0468.json new file mode 100644 index 0000000000000000000000000000000000000000..fe65f3c08f9c121a7653ca09bd85636a395019be --- /dev/null +++ b/processed_dataset/proof/0468.json @@ -0,0 +1,10 @@ +{ + "source_file": "./raw_volume-zh/volume12/exercise2.tex", + "problem_type": "proof", + "problem": "问题4. 有一个参观团,其中任意四个成员中总有一名成员原先见过其他三名成员.\n证明: 在任意四名成员中, 总有一名成员原先见过所有成员.", + "solution": "如图(), 用图论语言表示即: 图 $G$ 的任意四点中至少有一个顶点和其他三个顶点相邻.\n证明图 $G$ 任意四个顶点中至少一个顶点和 $G$ 中其他所有顶点都相邻.\n反证法.\n如果命题不成立, 则 $G$ 中有四个点 $x$, $y, z, w$, 它们和图 $G$ 中的其他所有顶点不都相邻.\n于是存在四个顶点 $x^{\\prime}, y^{\\prime}, z^{\\prime}, w^{\\prime}$, 它们依次与 $x, y, z, w$ 都相邻.\n不妨设这个点为 $x$. 如图.\n所以 $x^{\\prime}$ 不是 $y, z, w$ 中的一个, 且 $y^{\\prime}$ 与 $x$ 是两个不同的顶点.\n如果 $y^{\\prime}$ 与 $x^{\\prime}$ 不同,则 $x, y, x^{\\prime}, y^{\\prime}$ 中没有一个顶点和其他三个顶点都相邻, 和已知矛盾.\n所以 $y^{\\prime}$ 和 $x^{\\prime}$ 重合.\n同理可证, $z^{\\prime}$ 和 $x^{\\prime}$ 重合.\n于是 $x^{\\prime}$ 和 $y z w$ 都不相邻, 和已知矛盾.\n这就证明了图 $G$ 中任意四个顶点中至少有一个顶点和 $G$ 的其他所有顶点都相邻.", + "remark": "注:本题和例题 4 关系密切, 两个命题实际是等价的.", + "figures": [ + "./images/volume12/figures/fig-c2a4.png" + ] +} \ No newline at end of file diff --git a/processed_dataset/proof/0469.json b/processed_dataset/proof/0469.json new file mode 100644 index 0000000000000000000000000000000000000000..04f59f30a790f0c2f37ce9b7d48a26c781ee4d7f --- /dev/null +++ b/processed_dataset/proof/0469.json @@ -0,0 +1,8 @@ +{ + "source_file": "./raw_volume-zh/volume12/exercise2.tex", + "problem_type": "proof", + "problem": "问题6. 参加某次学术讨论会共有 123 个人, 已知每个人至少和 5 位与会者讨论过问题.\n证明: 至少有一个人和 5 位以上的与会者讨论过问题.", + "solution": "作一个图 $G: 123$ 个顶点 $v_1, v_2, \\cdots, v_{123}$ 代表 123 个人, 如果两人讨论过问题, 则相应的顶点相邻.\n于是图 $G$ 中的每一个顶点的度 $\\geqslant 5$.\n如果 $G$ 中没有一个顶点的度大于 5 , 那么 $G$ 中的每个顶点的度就都等于 5 , 这就得出 $G$ 中的奇顶点个数是奇数, 这是不可能的.\n从而图 $G$ 中至少有一个顶点的度大于 5 .", + "remark": "", + "figures": [] +} \ No newline at end of file diff --git a/processed_dataset/proof/0470.json b/processed_dataset/proof/0470.json new file mode 100644 index 0000000000000000000000000000000000000000..7927ee90659b1ae2f896a9fe9770362a1972db6d --- /dev/null +++ b/processed_dataset/proof/0470.json @@ -0,0 +1,8 @@ +{ + "source_file": "./raw_volume-zh/volume12/exercise2.tex", + "problem_type": "proof", + "problem": "问题7. 在一次会议中,已知每个议员至多与三人不相识, 证明: 一定可以把所有议员分为两组,使每一组中, 每个议员至多与一人不相识.\n个人中总可以找出 4 个人来, 这 4 个人可以围着圆桌坐下, 使得每个人旁边都是认识的人 $(n \\geqslant 2)$.", + "solution": "作图 $G: n$ 个点表示 $n$ 名议员, 若两人不认识, 则连一条边在图 $G$ 中, 于是对任意的点 $v_i$, 均有 $d\\left(v_i\\right) \\leqslant 3$. 现把图 $G$ 任意分成两部分 $G_1$ 和 $G_2$. 在同一部分中的两点, 若原来有边相连, 则这条边仍留在这部分中.\n在不同部分中的两点, 若原来有边相连, 则这条边就不再存在了.\n把这些去掉的边放在一起构成集 $E$. 在两部分中, 如果有一点的度大于 1 , 不妨设在 $G_1$ 中有点 $v_1$, $d\\left(v_1\\right) \\geqslant 2$, 把这个点移到 $G_2$ 中去.\n此时 $G_1$ 中至少要消失两条边; 由于 $d\\left(v_1\\right) \\leqslant$ 3 , 因而 $G_2$ 中至多增加一条边, 于是 $E$ 中至少增加一条边.\n反复进行这一步骤.\n这样 $E$ 中的边不断增加, 但总边数是有限的, 所以到某一步, 这两部分中再也没有度大于 1 的点.\n于是命题得证.", + "remark": "", + "figures": [] +} \ No newline at end of file diff --git a/processed_dataset/proof/0471.json b/processed_dataset/proof/0471.json new file mode 100644 index 0000000000000000000000000000000000000000..b5fb8a7eec03764f95c9963e5cf987b0d8e7caad --- /dev/null +++ b/processed_dataset/proof/0471.json @@ -0,0 +1,8 @@ +{ + "source_file": "./raw_volume-zh/volume12/exercise2.tex", + "problem_type": "proof", + "problem": "问题8. 有$2n$个人在一起聚会;其中每个人至少同其中的$n$个人认识,证明这 $2n$个人中总可以找出4个人来,这4个人可以围着圆桌坐下,使得每个人旁边都是认识的人 $(n\\geq2)$", + "solution": "问题可转化为: 在一个 $2 n$ 阶图 $G$ 中, 每个顶点的度 $\\geqslant n$, 证明 $G$ 中有一个四边形.\n若 $G=K_{2 n}$, 结论显然.\n当 $G \\neq K_{2 n}$ 时, 则存在点 $v_1, v_2$ 不相邻, 由于 $d\\left(v_1\\right)+d\\left(v_2\\right) \\geqslant 2 n$, 根据抽屉原理, 其余的 $2 n-2$ 个点中必有两个点, 设为 $v_3, v_4$, 与 $v_1, v_2$ 都相邻, 于是这 4 点就构成一个四边形.", + "remark": "", + "figures": [] +} \ No newline at end of file diff --git a/processed_dataset/proof/0472.json b/processed_dataset/proof/0472.json new file mode 100644 index 0000000000000000000000000000000000000000..1879e6c57b98382a98d481800d71747c7f511a1d --- /dev/null +++ b/processed_dataset/proof/0472.json @@ -0,0 +1,8 @@ +{ + "source_file": "./raw_volume-zh/volume12/exercise2.tex", + "problem_type": "proof", + "problem": "问题9. 已知 9 个人 $v_1, v_2, \\cdots, v_9$ 中 $v_1$ 和 2 个人握过手, $v_2 、 v_3$ 各和 4 个人握过手, $v_4 、 v_5 、 v_6 、 v_7$ 各和 5 个人握过手, $v_8 、 v_9$ 各和 6 个人握过手, 证明这 9 个人中一定可以找出 3 个人,他们互相握过手.", + "solution": "作图 $G$ : 用 9 个点表示 9 个人,两顶点相邻当且仅当这两人握过手.\n因为 $d\\left(v_9\\right)=6$, 所以存在 $v_k \\neq v_1, v_2, v_3$ 与 $v_9$ 相邻.\n显然 $d\\left(v_k\\right) \\geqslant 5$. 在与 $v_9$ 相邻的其余五个点中一定有一个点 $v_h$ 与 $v_k$ 相邻 (否则 $d\\left(v_k\\right) \\leqslant 9-5-1=3$ ), 于是 $v_9, v_k, v_h$ 即为所求.", + "remark": "", + "figures": [] +} \ No newline at end of file diff --git a/processed_dataset/proof/0473.json b/processed_dataset/proof/0473.json new file mode 100644 index 0000000000000000000000000000000000000000..3eace952e0f37038c29e02b9964ac54da3eeb785 --- /dev/null +++ b/processed_dataset/proof/0473.json @@ -0,0 +1,8 @@ +{ + "source_file": "./raw_volume-zh/volume12/exercise2.tex", + "problem_type": "proof", + "problem": "问题10. 一个旅行团中共14人;在山上休息时,他们想打桥牌,而其中每个人都曾和其中的 5 个人合作过.\n现规定只有 4 个人中任两个人都未合作过, 才能在一起打一局牌.\n这样, 打了三局就没法再打下去了.\n这时, 来了另一位旅游者, 他当然没有与该旅行团中的任何人合作过.\n如果他也参加打牌, 证明一定可以再打一局桥牌.", + "solution": "用 14 个点 $v_1, v_2, \\cdots, v_{14}$ 表示 14 个人, 两顶点 $v_i$ 与 $v_j$ 相邻当且仅当这两人未合作过, 得图 $G$. 在 $G$ 中每个顶点的度都是 8 . 打三局要去掉 6 条边, 因此至少有 2 个顶点的度数保持为 8 , 设其中之一为 $v_1$. 在与 $v_1$ 相邻的 8\n个顶点中至少有一个 $v_2$ 的度数不小于 7 , 可以知道 $v_2$ 与和 $v_1$ 相邻的其余 7 个点之一设为 $v_3$ 相邻.\n这样, $v_1, v_2, v_3$ 与新来的 $v$ 组成 $K_4$.", + "remark": "", + "figures": [] +} \ No newline at end of file diff --git a/processed_dataset/proof/0474.json b/processed_dataset/proof/0474.json new file mode 100644 index 0000000000000000000000000000000000000000..df43c0a3892bf4d363d5f303ce03c6dbe61d6a5b --- /dev/null +++ b/processed_dataset/proof/0474.json @@ -0,0 +1,8 @@ +{ + "source_file": "./raw_volume-zh/volume12/exercise2.tex", + "problem_type": "proof", + "problem": "问题11. 对于平面上任意 $n$ 个点构成的点集 $P$, 如果其中任意两点之间的距离均已确定, 那么就称这个点集是\"稳定的\". 求证:在 $n(n \\geqslant 4)$ 个点的平面点集 $P$ 中, 无三点共线, 且其中的 $\\frac{1}{2} n(n-3)+4$ 个两点之间的距离已被确定,那么点集 $P$ 就是\"稳定的\".", + "solution": "先约定,确定距离的两点用边相连.\n我们用数学归纳法来证明本题.\n当 $n=4$ 时, $\\frac{1}{2} n(n-3)+4=6$, 四点之间只有 $\\left(\\mathrm{C}_4^2=\\right) 6$ 个距离, 它们均已确定,故命题成立.\n设 $n=k(k \\geqslant 4)$ 时命题成立.\n当 $n=k+1$ 时,点集共连了\n$$\n\\frac{1}{2}(k+1)(k-2)+4\n$$\n条边.\n设 $A_{k+1}$ 是这个点集中\"度\" (即自该点出发的边数) 最小的点, 则其度\n$$\n\\begin{aligned}\nd\\left(A_{k+1}\\right) & \\leqslant \\frac{2\\left[\\frac{1}{2}(k+1)(k-2)+4\\right]}{k+1} \\\\\n& =k-2+\\frac{8}{k+1} \\\\\n& \\leqslant k-2+\\frac{8}{5}$ 1. 考虑完全 $m$ 部图 $G^{\\prime}$, 它的 $m$ 部分顶点数分别为 $n_1, n_2, \\cdots, n_i-1, \\cdots, n_j+ 1, \\cdots, n_m$. 由于 $e(G)=\\frac{1}{2} \\sum_{k=1}^m\\left(n-n_k\\right) n_k, e\\left(G^{\\prime}\\right)=\\frac{1}{2} \\sum_{\\substack{k=1 \\\\ k \\neq i, j}}^m\\left(n-n_k\\right) n_k+\\frac{1}{2}(n- \\left.n_i+1\\right)\\left(n_i-1\\right)+\\frac{1}{2}\\left(n-n_j-1\\right)\\left(n_j+1\\right)=e(G)+\\left(n_i-n_j\\right)-1>e(G)$. 若 $G^{\\prime}$ 与 $T_m(n)$ 同构,结论成立, 否则重复上述过程直至所得的图与 $T_m(n)$ 同构为止.", + "remark": "", + "figures": [] +} \ No newline at end of file diff --git a/processed_dataset/proof/0478.json b/processed_dataset/proof/0478.json new file mode 100644 index 0000000000000000000000000000000000000000..30bedf2d8041aac0a75c1c8b80dfabda4e384b9b --- /dev/null +++ b/processed_dataset/proof/0478.json @@ -0,0 +1,8 @@ +{ + "source_file": "./raw_volume-zh/volume12/exercise3.tex", + "problem_type": "proof", + "problem": "问题5. $X 、 Y$ 两国留学生各 $n(n>2)$ 人, 每个 $X$ 国学生都与一些(不是所有) $Y$国学生跳过舞, 每个 $Y$ 国学生至少与一个 $X$ 国学生跳过舞, 证明一定可以找到两个 $X$ 国学生 $x, x^{\\prime}$ 及两个 $Y$ 国学生 $y, y^{\\prime}$, 使得 $x$ 与 $y, x^{\\prime}$ 与 $y^{\\prime}$跳过舞, 而 $x$ 与 $y^{\\prime}, x^{\\prime}$ 与 $y$ 没有跳过舞.", + "solution": "作偶图 $G=(X, Y ; E), X$ 的每个顶点表示一个 $X$ 国学生, $Y$ 的每一个顶点表示一个 $Y$ 国学生.\n如果一个 $X$ 国学生与一个 $Y$ 国学生跳过舞, 就在相应的两个点之间连一条边.\n设 $x$ 是集 $X$ 中度最大的点, 因 $d(x)\\frac{(k-1)(n-k)+k}{k^2} \\cdot \\mathrm{C}_n^{k-1}$, 则必存在一个 $X$ 的 $k+1$ 元子集,它的所有 $k$ 元子集都是红 $k$ 子集.", + "solution": "假设没有所求的 $k+1$ 元集, 我们证明此时\n$$\nm \\leqslant \\frac{(k-1)(n-k)+k}{k^2} \\cdot \\mathrm{C}_n^{k-1} .\n$$\n记所有红 $k$ 子集构成子集族 $S$,所有 $k-1$ 元子集构成的子集族为 $\\beta$. 对于任一个 $k-1$ 元子集 $B$, 记 $\\alpha(B)$ 为包含 $B$ 的红 $k$ 子集个数.\n对于任一个 $A \\in S, A$ 包含有 $k$ 个 $k-1$ 元子集.\n对于 $X \\backslash A$ 的任一元素 $x$, $x$ 与 $A$ 中 $k$ 个 $k-1$ 元集的至多 $k-1$ 个构成红 $k$ 子集(不然存在 $k+1$ 元集, 其所有 $k$ 元子集都是红 $k$ 子集). 因此 $\\sum_{\\substack{B \\subset A \\\\|B|=k-1}} \\alpha(B) \\leqslant(n-k)(k-1)+k$.\n故\n$$\n\\begin{gathered}\nm[(n-k)(k-1)+k] \\geqslant \\sum_{A \\in S} \\sum_{\\substack{B C A \\\\\n|B|=k-1}} \\alpha(B) \\\\\n=\\sum_{B \\in \\beta}(\\alpha(B))^2 \\geqslant \\frac{1}{|\\beta|}\\left(\\sum_{B \\in \\beta} \\alpha(B)\\right)^2 \\geqslant \\frac{1}{\\mathrm{C}_n^{k-1}}(\\mathrm{~km})^2 . \\\\\nm \\leqslant \\frac{[(n-k)(k-1)+k] \\cdot \\mathrm{C}_n^{k-1}}{k^2} .\n\\end{gathered}\n$$\n故\n$$\nm \\leqslant \\frac{[(n-k)(k-1)+k] \\cdot \\mathrm{C}_n^{k-1}}{k^2} .\n$$\n因此当 $m>\\frac{(n-k)(k-1)+k}{k^2} \\cdot \\mathrm{C}_n^{k-1}$ 时, 必存在一个 $X$ 的 $k+1$ 元子集, 它的所有 $k$ 元子集都是红 $k$ 子集.", + "remark": "", + "figures": [] +} \ No newline at end of file diff --git a/processed_dataset/proof/0480.json b/processed_dataset/proof/0480.json new file mode 100644 index 0000000000000000000000000000000000000000..3fd777b47d100940eb8b1564307064feb4fbdac4 --- /dev/null +++ b/processed_dataset/proof/0480.json @@ -0,0 +1,8 @@ +{ + "source_file": "./raw_volume-zh/volume12/exercise3.tex", + "problem_type": "proof", + "problem": "问题7. 记 $K_{3,3}$ 为图, 求证: 一个有 10 个顶点 40 条边的图, 必含有一个 $K_{3,3}$.", + "solution": "因为 $\\mathrm{C}_{10}^2=45$, 所以一个 10 阶完全图中含边 45 条.\n因此题中图为 10 阶完全图中去掉 5 边后所得, 把这 5 边称为 \"去掉边\", 记 10 个顶点为 $A_1 A_2, \\cdots, A_{10}$.\n不妨设 $A_1 A_2$ 为 \"去掉边\", 则把点 $A_1$ 及其连边去掉, 于是余下的 9 点图中至多有 4 条\"去掉边\".\n不妨设 $A_2 A_3$ 为一条\"去掉边\", 把 $A_2$ 及其连边去掉 (如果已不含 \"去掉边\", 则任去一点,下同), 于是余下的 8 阶图中至多有 3 条\"去掉边\".\n不妨设 $A_3 A_4$ 为一条 \"去掉边\", 把 $A_3$ 及其连边去掉, 于是余下的 7 阶图中至多有 2 条\"去掉边\".\n不妨设 $A_4 A_5$ 为一条 \"去掉边\", 把 $A_4$ 及其连边去掉, 于是余下的 6 阶图中至多有 1 条\"去掉边\".\n即此图为 6 阶完全图或从 6 阶完全图中去掉一边后所得的图, 不论如何必含有二部完全图 $K_{3,3}$.\n本题的推广: 若 $n$ 个顶点, $m$ 边图不含 $K_{r, r}$, 求证: $m), \n先证明: $m=\\left[\\frac{n^2}{4}\\right]+1$ 时, 存在图 $G$, 在 $G$ 中没有两个三角形, 它们恰有一个公共顶点.\n考虑 2 部图: 顶点集为 $M_1 、 M_2$, 在 $M_1$ 中有 $\\left[\\frac{n+1}{2}\\right]$ 个顶点, $M_2$ 中有 $\\left[\\frac{n}{2}\\right]$ 个顶点, 将 $M_1$ 中每个点与 $M_2$ 中的每个点连边, 且 $M_1$ 中某两个点连边, 其余顶点之间不连边, 则此图中有 $\\left[\\frac{n}{2}\\right]\\left[\\frac{n+1}{2}\\right]+1=\\left[\\frac{n^2}{4}\\right]+1$ 条边, 但该图中没有两个三角形恰有一个公共顶点.\n再证明: $m=\\left[\\frac{n^2}{4}\\right]+2$, 任意一个有 $m$ 条边的 $n$ 阶简单图 $G_n$ 中, 存在两个恰.\n有一个公共顶点的三角形.\n当 $n=5$ 时, $G_5$ 的边数 $=8$, 即完全图 $K_5$ 去掉 2 条边, 由于 $K_5$ 中每个顶点的度为 4 , 故去掉两条边后, 仍有一个顶点的度为 4 , 不妨设 $d(a)=4$, 于是 2 条边是由剩下的 $K_4$ 中去掉的, 这时剩下的 4 个点 $b 、 c 、 d 、 e$ 形成的三组对边中, 必有一组对边 (不妨设为 $b c 、 d e$ ) 同时存在, 所以, $a 、 b 、 c 、 d 、 e$ 构成满足条件的 5 个点.\n设命题对 $n-1(n \\geqslant 6)$ 成立, 考虑 $n$ 的情形.\n(1) 若 $G_n$ 中有一个点的度 $\\leqslant\\left[\\frac{n}{2}\\right]$, 则删去此点及其引出的边后, 剩下 $n-$ 1 个点之间所引的边数不小于\n$$\n\\begin{aligned}\n{\\left[\\frac{n^2}{4}\\right]+2-\\left[\\frac{n}{2}\\right] } & =2+\\left[\\frac{n}{2}\\right]\\left(\\left[\\frac{n+1}{2}\\right]-1\\right) \\\\\n& =2+\\left[\\frac{n}{2}\\right]\\left[\\frac{n-1}{2}\\right] \\\\\n& =2+\\left[\\frac{(n-1)^2}{4}\\right],\n\\end{aligned}\n$$\n利用归纳假设可知命题成立.\n(2) 若 $G_n$ 中每个点的度均 $\\geqslant\\left[\\frac{n}{2}\\right]+1$. 则当 $n=2 k$ 时,\n$$\ne \\geqslant k(k+1)>k^2+2=\\left[\\frac{n^2}{4}\\right]+2,\n$$\n而当 $n=2 k+1$ 时,\n$$\n\\begin{aligned}\ne & \\geqslant \\frac{1}{2}(2 k+1)(k+1) \\\\\n& =k(k+1)+\\frac{k+1}{2} \\\\\n& \\geqslant\\left[\\frac{n^2}{4}\\right]+2,\n\\end{aligned}\n$$\n其中等号仅当 $k=3$, 即 $n=7$ 时取到.\n如果 $e>\\left[\\frac{n^2}{4}\\right]+2$, 则从 $G_n$ 中去掉一条边, 重复上述讨论,除 $n=7$ 的情形, 其余情况必可转为情形一.\n所以, 只需讨论 $n=7$, 每个点的度均为 4 的情形.\n在这种情形中, 考虑与 $a$ 相邻的 4 个顶点 $b 、 c 、 d$ 、 $e$, 在 $b 、 c 、 d 、 e$ 组成的子图中, 每个点的度不小于 1 . 不妨设 $b 、 c$ 之间连有边, 这时若 $d 、 e$ 之间连有边, 则命题获证,故 $d 、 e$ 不连边.\n此时, 若 $d 、 e$ 分别与 $b 、 c$ (或 $c 、 b$ ) 连有边, 命题也获证, 从而 $d 、 e$ 同时与 $b$ (或与 $c$ ) 相邻, 不妨设 $d 、 e$ 与 $b$ 均相邻, 而 $c 、 d 、 e$ 之间两两不连边.\n考虑另外两个点 $f 、 g$, 由于 $c 、 d 、 e$ 的度都是 4 , 故 $c 、 d 、 e$ 都与点 $f 、 g$ 同时相邻, 而 $f 、 g$ 的度也都为 4 , 故 $f 、 g$ 相邻 (因为 $f 、 g$ 与 $a 、 b$ 都不相邻), 这样, 我们得到下边的图,此时 $e a 、 e b 、, e f 、 e g 、 a b 、 f g$ 满足题中的要求.\n综上可知, $m=\\left[\\frac{n^2}{4}\\right]+2$.", + "remark": "", + "figures": [ + "./images/volume12/figures/fig-c3a10.png" + ] +} \ No newline at end of file diff --git a/processed_dataset/proof/0484.json b/processed_dataset/proof/0484.json new file mode 100644 index 0000000000000000000000000000000000000000..e9a8dd1b5916ae8c5cd272b53c7fdf62fdf758f3 --- /dev/null +++ b/processed_dataset/proof/0484.json @@ -0,0 +1,8 @@ +{ + "source_file": "./raw_volume-zh/volume12/exercise4.tex", + "problem_type": "proof", + "problem": "问题6. 设 $d_1, d_2, \\cdots, d_n$ 是 $n$ 个正整数, $n \\geqslant 2$, 已知 $\\sum_{i=1}^n d_i=2 n-2$. 证明存在一棵顶点度数分别为 $d_1, d_2, \\cdots, d_n$ 的树.", + "solution": "$d_1, d_2, \\cdots, d_n$ 中至少有两个为 1 (否则, $\\sum_{i=1}^n d_i \\geqslant 2 n-1$ ). 对顶点数 $n$ 用数学归纳法.\n$n=2$ 时显然成立.\n设结论在 $n=k$ 时成立.\n当 $n=k+1$ 时在 $d_1, d_2, \\cdots, d_k, d_{k+1}$ 中存在为 1 的数, 不妨设 $d_{k+1}=1$. 易知这 $k+1$ 个数中存在大于等于 2 的数, 设为 $d_k$. 考虑 $d_1, d_2, \\cdots, d_{k-1},\\left(d_k-1\\right)$ 这 $k$ 个数, $d_1+\\cdots+d_{k-1}+\\left(d_k-1\\right)=2(k+1)-2-1-1=2 k-2$, 由归纳假设知, 存在树 $T^{\\prime}$, 其顶点为 $v_1, \\cdots, v_k, \\sum_{i=1}^k k\\left(v_i\\right)=d_1+\\cdots+d_{k-1}+\\left(d_k-1\\right)=2 k-$ 2. 从 $T^{\\prime}$ 中顶点 $v_k$ 引出一条边与 $v_{k+1}$ 相邻, 得树 $T$, 则 $\\sum_{i=1}^{k+1} d\\left(v_i\\right)=2 k-2+1+ 1=2(k+1)-2$. 所以 $T$ 即为所求.", + "remark": "", + "figures": [] +} \ No newline at end of file diff --git a/processed_dataset/proof/0485.json b/processed_dataset/proof/0485.json new file mode 100644 index 0000000000000000000000000000000000000000..255bd4a3e906c742cb4ced130da059deaceb61f8 --- /dev/null +++ b/processed_dataset/proof/0485.json @@ -0,0 +1,8 @@ +{ + "source_file": "./raw_volume-zh/volume12/exercise4.tex", + "problem_type": "proof", + "problem": "问题8. 一个 $n$ 行 $n$ 列的数表 (矩阵), 每两行都不完全相同.\n证明一定存在某一列, 去掉这一列后, 每两行仍然不完全相同.", + "solution": "本题参见本节例 7 .", + "remark": "", + "figures": [] +} \ No newline at end of file diff --git a/processed_dataset/proof/0486.json b/processed_dataset/proof/0486.json new file mode 100644 index 0000000000000000000000000000000000000000..de14ed561176c02ea91f2337a7f5768a99ed046f --- /dev/null +++ b/processed_dataset/proof/0486.json @@ -0,0 +1,10 @@ +{ + "source_file": "./raw_volume-zh/volume12/exercise4.tex", + "problem_type": "proof", + "problem": "问题9. 记图 $G$ 的所有顶点集合为 $V$, 所有边的集合为 $E$. 求证: 若 $|E| \\geqslant|V|+$ 4 , 则 $G$ 中必有两个无公共边的圈.\n(Pösá 定理)", + "solution": "假设命题不成立, 则必然存在着反例.\n我们考察其中 $|E|+|V|$ 最小的一个反例.\n在这个反例中, 一定有 $|E|=|V|+4$ (不然可以把多的边去掉, 这时所得的图仍是一个反例, 而 $|E|+|V|$ 变小, 矛盾!), 则 $|E|>|V|$. 图中必存在一个圈, 则最短圈长至少为 5 . (不然最短圈长不大于 4 , 则把这个圈去掉之后, 仍将有 $|E| \\geqslant|V|$, 从而图中仍存在圈.\n而这个圈与前一个圈无公共边, 矛盾!) 另外, 图中每一个顶点的度数至少为 3 (不然, 若某点度数为 2 , 则把该点去掉, 它连出的两条边连成一条边, 仍有 $|E|=|V|+4$, 而 $|E|+ |V|$ 变小, 矛盾! 若某点度数为 1 , 则把该点及其连出的边去掉, 仍有 $|E|= |V|+4$, 而 $|E|+|V|$ 变小,矛盾!若存在孤立点, 则把孤立点去掉, $|E|>|V|+4$, 而 $|E|+|V|$ 变小,矛盾!)\n取一个最短圈 $C_0$, 其长度至少为 5 , 则圈上至少有 5 个点.\n对于 $C_0$ 上各点, 每一点至少与圈外连出一点, 且各连出的点互不相同 (否则将出现长度小于 5 的圈), 这样易知 $|V| \\geqslant 2 \\times 5=10$. 另一方面, $2|E|=\\sum_{v \\in V} d(V) \\geqslant \\sum_{v \\in V} 3= 3|V|$, 而 $|E|=|V|+4$, 故 $2|V|+8 \\geqslant 3|V|,|V| \\leqslant 8$, 矛盾!\n因此反例不存在, 原命题得证.", + "remark": "注:: 此题的解法就是所谓的\"极端原理\". 要证明某个命题成立, 用反证法, 反设其不成立, 考察其中某个变量 $V \\in N$, 从 $V$ 的最小反例中推出矛盾,多了 $V$ 最小这个条件使证明难度降低了.\n题目的结论是最佳的, 当 $|E|=|V|+3$ 时, 可举出反例如图()所示.", + "figures": [ + "./images/volume12/figures/fig-c4a9.png" + ] +} \ No newline at end of file diff --git a/processed_dataset/proof/0487.json b/processed_dataset/proof/0487.json new file mode 100644 index 0000000000000000000000000000000000000000..b7fc572f8654f67d4052e863b2100f63476d942c --- /dev/null +++ b/processed_dataset/proof/0487.json @@ -0,0 +1,8 @@ +{ + "source_file": "./raw_volume-zh/volume12/exercise4.tex", + "problem_type": "proof", + "problem": "问题10. 某天晚上21个人之间通了电话,有人发现这 21 人共通话102次,且每两人至多通话一次.\n他还发现, 存在 $m$ 个人,第 1 个人与第 2 个人通了话, 第 2 个人与第 3 个人通了话, ......., 第 $m-1$ 个人与第 $m$ 个人通了话, 第 $m$ 个人又与第 1 个人通了话,他不肯透露 $m$ 的具体值, 只说 $m$ 是奇数.\n求证 21 个人中必存在 3 人,他们两两通了话.", + "solution": "用 21 个点表示 21 个人,两点之间有 1 条连线当且仅当这两个点代表的人通了电话.\n由已知, 存在 1 个长度为 $m$ 的奇圈 (长度为奇数的圈称为奇圈). 设图中长度最短的奇圈为 $C$, 长度为 $2 k+1$.\n若 $k=1$, 则 $C$ 为三角形, 其代表的 3 人两两通了电话.\n若 $k>1$. 设 $C$ 为 $v_1 v_2 \\cdots v_{2 k+1} v_1$, 则 $v_i, v_j$ 之间没有连线 $(1 \\leqslant i, j \\leqslant 2 k+ 1, i-j \\neq \\pm 1(\\bmod 2 k+1)$. 否则, 设 $v_i, v_j$ 相连, 则圈 $v_1 v_2 \\cdots v_i v_j \\cdots v_{2 k+1} v_1$ 与圈 $v_i v_{i+1} \\cdots v_j v_i$ 长度之和为 $2 k+3$, 故其中必有一个长度小于 $2 k+1$ 的奇数, 这与 $C$ 最短矛盾.\n若除 $v_1, v_2, \\cdots, v_{2 k+1}$ 之外的 $21-(2 k+1)=20-2 k$ 个点无三角形, 由托兰定理, 它们至多连了 $(10-k)^2$ 条边.\n又其中任一点不与 $C$ 的相邻两点连 (否则便有三角形), 所以它至多与 $C$ 中 $k$ 个点相连, 故总的边数\n$$\n\\begin{aligned}\n& 2 k+1+k(20-2 k)+(10-k)^2 \\\\\n= & 100+2 k+1-k^2 \\\\\n= & 102-(k-1)^2 \\\\\n\\leqslant & 102-(2-1)^2=101,\n\\end{aligned}\n$$\n矛盾!\n故图中必有三角形存在, 即存在 3 个人, 他们两两通了电话.", + "remark": "", + "figures": [] +} \ No newline at end of file diff --git a/processed_dataset/proof/0488.json b/processed_dataset/proof/0488.json new file mode 100644 index 0000000000000000000000000000000000000000..d00e7f6f8ec78f828e133f6a2c8465c9271fd763 --- /dev/null +++ b/processed_dataset/proof/0488.json @@ -0,0 +1,8 @@ +{ + "source_file": "./raw_volume-zh/volume12/exercise4.tex", + "problem_type": "proof", + "problem": "问题11. 某国有若干个城市, 某些城市之间有道路相连, 由每个城市连出 3 条道路.\n证明: 存在一个由道路形成的圈,它的长度不能被 3 整除.", + "solution": "假定存在这样的图, 它的每个顶点的度数都大于 2 , 但该图中的任何一个圈的长度都可被 3 整除.\n我们来考察具有这种性质的顶点数目最小的图 G. 显然, 该图中存在着长度最小的圈 $Z$, 该圈上的任意两个不相邻的顶点之间没有边相连, 又因每一顶点的度数都大于 2 , 所以圈 $Z$ 上的每个顶点都有一边与圈外顶点相连, 设圈 $Z$ 依次经过顶点 $A_1, A_2, \\cdots, A_{3 k}$. 假定存在连接顶点 $A_m$ 和 $A_n$ 的不包含圈 $Z$ 上的边的路径 $S$. 我们来分别考察由路径 $S$ 和 $Z$ 的 \"两半\"所组成的圈 $Z_1$ 和 $Z_2$. 由于这两个圈的长度都可被 3 整除, 不难推知路径 $S$ 的长度可被 3 整除.\n特别地, 对题目中所给出的图, 可知它的任何一个不在 $Z$ 上的顶点 $X$, 都不可能有边与 $Z$ 的两个不同顶点分别相连.\n即, 由圈 $Z$ 上的顶点所连出的不在圈上的边, 应分别连向各不相同的顶点.\n我们来作另外一个图 $G_1$, 把图 $G$ 中圈 $Z$ 上的所有顶点 $A_1, A_2, \\cdots, A_{3 k}$ 合并为一个顶点 $A$, 保留所有不在圈 $Z$ 上的顶点及它们之间所连的边, 且分别用边将 $A$ 同原来与 $Z$ 上的顶点有边相连的顶点逐一相连, 易知 $A$ 的度数 $\\geqslant 3 k$. 于是, 图 $G_1$ 中的顶点数目少于图 $G$, 而每个顶点的度数仍都大于 2. 于是, 按照前面所证的结论, 图 $G_1$ 中的任何一个圈的长度都可被 3 整除.\n这样一来, 我们便得出了矛盾: 因为如前所言, 图 $G$ 是具有这种性质的顶点数目最小的图.\n这样一来, 在任何所有顶点的度数都大于 2 的图中, 必定存在长度不能被 3 整除的圈, 接下来只需把这一断言应用于我们的题目, 并以城市作为顶点, 以道路作为边即可.", + "remark": "", + "figures": [] +} \ No newline at end of file diff --git a/processed_dataset/proof/0489.json b/processed_dataset/proof/0489.json new file mode 100644 index 0000000000000000000000000000000000000000..a2e811f22287ba57e803b4a65d2223d1317b50bf --- /dev/null +++ b/processed_dataset/proof/0489.json @@ -0,0 +1,8 @@ +{ + "source_file": "./raw_volume-zh/volume12/exercise4.tex", + "problem_type": "proof", + "problem": "问题12. 一条河的两岸有一些城市, 城市的总数不少于 3 个.\n城市由一些航线连接着, 每条航线将位于两岸的一对城市联系在一起, 每个城市恰好与另一边的 $k$ 个城市连接.\n人们可以在任何两座城市之间往来.\n证明: 如果航线中有一条被取消,人们还是可以在任何两座城市之间往来.", + "solution": "不妨称河的两岸分别为北岸与南岸.\n北岸的 $n$ 个城市用点 $x_1$, $x_2, \\cdots, x_n$ 表示, 其全体记为 $X=\\left\\{x_1, x_2, \\cdots, x_n\\right\\}$; 南岸的 $m$ 个城市用点 $y_1, y_2, \\cdots, y_m$ 表示, 其全体记为 $Y=\\left\\{y_1, y_2, \\cdots, y_m\\right\\}$. 如果北岸的城市 $x_i$ 与南岸的城市 $y_j$ 之间有航线, 则连接成为边 $\\left(x_i, y_i\\right)$, 所有的边组成的集合记为 $E$. 这就得到了一个由顶点集 $X 、 Y$ 与边集 $E$ 构成的图, 称为 2 部分图, 也称为偶图, 记为 $G=(X, Y ; E)$. 题中的后两个条件即是: 由任一顶点引出的边都是 $k$ 条; 图 $G$ 是连通的, 即任意两个顶点之间都有由若干条边连接而成的路.\n题目的结论是: 从 $E$ 中删去任意一条边 $e$, 图 $G^{\\prime}=(X, Y ; E-e)$ 仍然是连通的.\n因为每个顶点恰与 $k$ 条边相关联, 所以有\n$$\n|X| k=|E|=|Y| k,\n$$\n其中 $|X|,|E|,|Y|$ 表示集合 $X, E, Y$ 中元素的个数.\n于是有 $|X|=|Y|$, 即 $n=m$. 又因 $|X|+|Y| \\geqslant 3$, 所以 $|X|=|Y| \\geqslant 2$.\n现去掉 $G$ 的一条边, 得到的图为 $G^{\\prime}$. 若 $G^{\\prime}$ 不连通, 则 $G^{\\prime}$ 由两个连通部分 $G_1, G_2$ 构成.\n设\n$$\n\\begin{aligned}\n& X=X_1 \\cup X_2, X_1 \\cap X_2=\\varnothing \\\\\n& Y=Y_1 \\cup Y_2, Y_1 \\cap Y_2=\\varnothing . \\\\\n& G_1=\\left(X_1, Y_1 ; E_1\\right), G_2=\\left(X_2, Y_2 ; E_2\\right) .\n\\end{aligned}\n$$\n去掉的一条边是连接 $X_1$ 与 $Y_2$ 的顶点, 则\n$$\n\\begin{aligned}\n& \\left|X_1\\right| k-1=\\left|E_1\\right|=\\left|Y_1\\right| k, \\\\\n& \\left|X_2\\right| k=\\left|E_2\\right|=\\left|Y_2\\right| k-1,\n\\end{aligned}\n$$\n从而 $\\left(\\left|X_1\\right|-\\left|Y_1\\right|\\right) k=1$, 得 $k=1$.\n又 $G$ 连通, 则 $|X|=1$,与 $|X| \\geqslant 2$ 矛盾.\n故 $G^{\\prime}$ 连通, 从而结论成立.", + "remark": "", + "figures": [] +} \ No newline at end of file diff --git a/processed_dataset/proof/0490.json b/processed_dataset/proof/0490.json new file mode 100644 index 0000000000000000000000000000000000000000..03b8536f1a77fa3df734c54b3caf63b3a03c919d --- /dev/null +++ b/processed_dataset/proof/0490.json @@ -0,0 +1,8 @@ +{ + "source_file": "./raw_volume-zh/volume12/exercise5.tex", + "problem_type": "proof", + "problem": "问题5. 如果在一次会议上, 每个人都至少与 $\\delta \\geqslant 2$ 个人交换过意见, 证明一定可以找到 $k$ 个人 $v_1, v_2, \\cdots, v_k$, 使得 $v_1$ 与 $v_2$ 交换过意见, $v_2$ 和 $v_3$ 交换过意见, $\\cdots, v_{k-1}$ 与 $v_k$ 交换过意见, $v_k$ 与 $v_1$ 交换过意见.\n其中 $k$ 为大于 $\\delta$ 的某个整数.", + "solution": "作图 $G$ : 顶点表示人, 两人交换过意见就在相应的顶点之间连一条边.\n在 $G$ 中取一条最长的链 $\\mu$, 设 $\\mu$ 的一个端点为 $v_1$, 则与 $v_1$ 相邻的 $\\delta$ 个点 $v_2, \\cdots$, $v_{\\delta+1}$ 均在链 $\\mu$ 上, 否则 $\\mu$ 还可延长.\n沿着链 $\\mu$ 走过 $v_2, v_3, \\cdots, v_{\\delta+1}$, 然后再回到 $v_1$, 这就是一个长度大于 $\\delta$ 的圈.", + "remark": "", + "figures": [] +} \ No newline at end of file diff --git a/processed_dataset/proof/0491.json b/processed_dataset/proof/0491.json new file mode 100644 index 0000000000000000000000000000000000000000..09227e9c64e7abd1b78472d1be09b7803b94b5c3 --- /dev/null +++ b/processed_dataset/proof/0491.json @@ -0,0 +1,11 @@ +{ + "source_file": "./raw_volume-zh/volume12/exercise5.tex", + "problem_type": "proof", + "problem": "问题6. 如图()所示, 图 $G$ 有 4 个顶点, 6 条边, 它们都在同一平面上, 这个平面被 6 条边分成 4 个区域 I, II, III, IV, 称这些区域为面.\n设有两个点 $Q_1, Q_2$ 在这些面中, 证明平面上不存在一条连接 $Q_1$ 与 $Q_2$ 的线 $\\mu$ 同时满足: (1) $\\mu$ 截每条边 $e_i$ 恰好一次 $(i=1,2, \\cdots, 6)$; (2) $\\mu$ 不过任一顶点 $v_j(j=1,2$, $3,4)$.", + "solution": "如图(), 在每个面中取一点 $v_j^{\\prime}(j=1,2,3,4)$, 如果两个面有公共边, 则在所取的两个点之间连一条边, 这样得到的图 $G^*$ 称为 $G$ 的对偶图.\n在 $G$ 中, 从一个面穿过某条边 $e_i$ 到另一个面, 就相当于在 $G^*$ 中从一个顶点沿一条边到另一个顶点.\n因此, 若 $G$ 中有满足条件 (1)、(2) 的折线 $\\mu$ 存在, 那么 $G^*$ 就是一条链 $\\left(Q_1, Q_2\\right.$ 不在同一面内) 或一个圈 $\\left(Q_1, Q_2\\right.$ 在同一面内), 即 $G^*$ 可以一笔画成.\n但 $G^*$ 的 4 个顶点全是奇顶点, 至少需两笔才能画成.", + "remark": "", + "figures": [ + "./images/volume12/figures/fig-c5p6.png", + "./images/volume12/figures/fig-c5a6.png" + ] +} \ No newline at end of file diff --git a/processed_dataset/proof/0492.json b/processed_dataset/proof/0492.json new file mode 100644 index 0000000000000000000000000000000000000000..4b065029f26e72116bd7caa357ce262cd0a802c3 --- /dev/null +++ b/processed_dataset/proof/0492.json @@ -0,0 +1,8 @@ +{ + "source_file": "./raw_volume-zh/volume12/exercise6.tex", + "problem_type": "proof", + "problem": "问题2. 证明: 对于正四面体、正六面体、正八面体及正二十面体所表示的图是哈密顿图.", + "solution": "请读者自己在图上找.", + "remark": "", + "figures": [] +} \ No newline at end of file diff --git a/processed_dataset/proof/0493.json b/processed_dataset/proof/0493.json new file mode 100644 index 0000000000000000000000000000000000000000..f72cda6b44be03c7c6cefebf001eab912fbf2b33 --- /dev/null +++ b/processed_dataset/proof/0493.json @@ -0,0 +1,15 @@ +{ + "source_file": "./raw_volume-zh/volume12/exercise6.tex", + "problem_type": "proof", + "problem": "问题9. 一个有限集合的全部子集, 可以如此排列, 使任何相邻的两个子集恰相差一个元素.", + "solution": "设集合 $A$ 有 $n$ 个元素, 把每个元素编号, 设 $A=\\{1,2,3, \\cdots, n\\}$, 我们用一个长度为 $n$ 的由 0 与 1 构成的序列来表达每个子集, 规则是 $A$ 的元素 $i$ 在该子集中, 则在序列的第 $i$ 位上写 1 , 否则写 0 . 例如空集 $\\varnothing=0,0,0, \\cdots, 0$. $\\{1\\}=1,0,0, \\cdots, 0,\\{n\\}=0,0, \\cdots, 1,\\{2,3\\}=0,1,1,0, \\cdots, 0$, 则 $A$ 的全部子集共有 $2^n$ 个.\n以这 $2^n$ 个子集对应的序列为顶点, 仅当两序列只一个同位数码相异时, 在此二顶点间连一边, 得一个图 $G$, 例如 $n=1$ 时为图() 所示的单位线段, $n=2$ 时, $G$ 为图() 所示的正方形, 图() 是两个图() 如下作成的: 在一个的 0,1 码前方都加上 0 , 变成 00,01 , 在另一个的 0,1 码前都加上 1 , 变成 10,11 , 再把一个放在另一个上方, 连上两个\"坚边\" 作成的一个正方形.\n复制两个图 2 , 把一个放在另一个的上方, 把上方的各顶点标志码前方都加一个 0 , 把下方各顶点标志码前方都加一个 1 , 再用 4 条坚线连接上下相对的顶点构成 $n=3$ 时的图 $G, G$ 是一个立方体.\n如果 $n=k$ 的图 $G$ 已作好, 则把 $n=k$ 的图及其复制品分别放在上方和下方各一个,再把上方的图的各顶点标码前方都加上一个 0 , 把下方的图的各顶点标码前方都加上一个 1 , 把上、下两方对应的顶点连一条竖直的边, 则得 $n=k+1$ 的图 $G$. 图() 是 $n=3$ 的情形, 图() 是 $n=4$ 的情形, $n= k$ 的图 $G$ 称为 $n$ 维立方体图.\n用数学归纳法容易证明 $n$ 维立方体图有哈密顿圈 $(n \\geqslant 2)$. 对于 $n=1$, 显然成立, 因为这时 1 维立方体图是 $K_2$, 是一条哈密顿链.\n对于 $n=2$, 是四边形, 显然是哈密顿圈, 对于 $n=k$, 若是哈密顿图, 考虑 $n=k+1$, 把 $G$ 中上方和下方的 $n==k$ 时的哈密顿圈上各删去一条对应边, 再把这两条对应边的对应端点间的两条边选来与上下方的哈密顿链并\n成一个 $n=k+1$ 时的哈密顿圈, 如图 4 中的粗线所示.\n把 $G$ 中的顶点按哈密顿圈上的顺序放在一个圆周上, 从任一顶点出发, 沿逆时针 (或顺时针) 为序, 则把全部子集排了序, 使得相邻子集恰相差一个元素.", + "remark": "", + "figures": [ + "./images/volume12/figures/fig-c6a9-1.png", + "./images/volume12/figures/fig-c6a9-2.png", + "./images/volume12/figures/fig-c6a9-2.png", + "./images/volume12/figures/fig-c6a9-1.png", + "./images/volume12/figures/fig-c6a9-3.png", + "./images/volume12/figures/fig-c6a9-4.png" + ] +} \ No newline at end of file diff --git a/processed_dataset/proof/0494.json b/processed_dataset/proof/0494.json new file mode 100644 index 0000000000000000000000000000000000000000..98fb400bfef9aaa75fc209626994032e7424e2fe --- /dev/null +++ b/processed_dataset/proof/0494.json @@ -0,0 +1,12 @@ +{ + "source_file": "./raw_volume-zh/volume12/exercise6.tex", + "problem_type": "proof", + "problem": "问题10. 平面上 $n$ 个点和若干条边所成的图不是哈密顿图, 但若任意去掉一点及与之相连的边, 则剩下的图为哈密顿图.\n求 $n$ 的最小值.", + "solution": "首先, 每个点的度至少为 3 , 不然存在一点 $A$ 仅连出至多两边, 则把其中一边去掉后, 剩下的 $A$ 点必不在某个圈上, 这与条件不符.\n因此 $n \\geqslant 3$.\n不难证得 $n \\neq 4,5,6$.\n若 $n=7$, 则去掉其中度数最大的点 (显然该点度数至少为 3 ), 得到一个长度为 6 的圈.\n由于与该点相邻的点在圈上必不相邻 (否则将出现长度为 7 的圈), 于是被去掉的点至多能与圈上三个互不相邻的点相连, 因此该点度数至多为 3 , 从而可知图中各点度数均恰为 3 , 而 $3 \\times 7=21$ 是奇数, 而实际上各点度数之和应为偶数,矛盾!\n若 $n=8$, 则去掉度数最大的点后, 得到一个长度为 7 的圈.\n被去掉的点至多能与圈上三个互不相邻的点相连, 因此它的度数至多为 3 . 由此可知各点度数均为 3. 如图(), $A, C, F, O$ 的度数已经为 3, 不能再连出任何边.\n而 $B, D, E$, $G$ 每点还要各连出一条边.\n若 $B$ 与 $G$ 相连, 则 $D$ 与 $E$ 相连 (连有两条边), 不可能.\n若 $B$ 与 $D$ 相连, 则 $E$ 与 $G$ 相连, 而此时图中存在长度为 8 的圈, 矛盾! 若 $B$ 与 $E$ 相连, 则 $D$ 与 $G$ 相连, 而此时图中也存在长度为 8 的圈, 矛盾!\n若 $n=9$, 由于 $3 \\times 9=27$ 不是偶数,因此不可能每点都是 3 度,故至少有一点, 度数至少为 4 . 我们把度数最大的点去掉, 得到一个长度为 8 的圈, 因此被去掉的点至多能与圈上 4 个互不相邻的点相连, 因此图中的点度数最大为 4 , 最小为 3. 如图(), 则点 $B$ 至少要再连出一条边.\n显然 $B$ 不能与 $A, C$ 再连边.\n若 $B$ 与 $D$ 连边, 则图中存在长度为 9 的圈, 同理 $B$ 不能与 $H$ 相连.\n若 $B$ 与 $F$ 相连, 则图中也将存在长度为 9 的圈, 矛盾! 故 $B$ 只可能与 $E$ 或 $G$ 连边.\n由对称性可设 $B$ 与 $E$ 相连.\n仿上讨论可知 $H$ 与 $C$ 相连 (若 $H$ 与 $E$ 相连, 则 $E$ 的度数已达到 5 , 矛盾!), $F$ 与 $A$ 相连, $D$ 与 $G$ 相连.\n此时图中任何两点不可能再连边, 则去掉点 $A$ 后, 图中应存在长度为 8 的圈.\n而事实上, 若图中存在长度为 8 的圈, 则 $B E, B C, H G, H C, F E, F G$ 必在圈上, 而该 6 边已构成一个长度为 6 的圈, 矛盾!\n由以上讨论可知, 满足要求的 $n$ 值至少为 10 .\n$n=10$ 的例子如图(), 称 Peterson 图.", + "remark": "", + "figures": [ + "./images/volume12/figures/fig-c6a10-1.png", + "./images/volume12/figures/fig-c6a10-2.png", + "./images/volume12/figures/fig-c6a10-3.png" + ] +} \ No newline at end of file diff --git a/processed_dataset/proof/0495.json b/processed_dataset/proof/0495.json new file mode 100644 index 0000000000000000000000000000000000000000..570d68402eed58632a86b063fa8f6d446c5246e2 --- /dev/null +++ b/processed_dataset/proof/0495.json @@ -0,0 +1,8 @@ +{ + "source_file": "./raw_volume-zh/volume12/exercise6.tex", + "problem_type": "proof", + "problem": "问题11. 求证:围着圆桌至少坐着五个人,那么一定可以调整他们的座位, 使得每人两侧出现新的邻座.", + "solution": "若恰为五人, 设原来的座次是 $A B C D E A$ : 调整成 $A D B E C A$ 即可.\n若超过五人, 以人为顶点, 仅当两人原来不是邻座时, 在此二顶点间连一边, 得图 $G$. 由于每个顶点的度数都是 $|V(G)|-3$, 于是任意两个顶点度数之和为 $2 n-6, n$ 是顶点数.\n又 $n>5$, 故 $2 n-6 \\geqslant n$, 由定理三, $G$ 中有哈密顿圈, 按圈上的次序请各人人席即可.", + "remark": "", + "figures": [] +} \ No newline at end of file diff --git a/processed_dataset/proof/0496.json b/processed_dataset/proof/0496.json new file mode 100644 index 0000000000000000000000000000000000000000..88cd18c2ad94155677648a4380dba4cc50955cca --- /dev/null +++ b/processed_dataset/proof/0496.json @@ -0,0 +1,8 @@ +{ + "source_file": "./raw_volume-zh/volume12/exercise7.tex", + "problem_type": "proof", + "problem": "问题1. 求证:设 $G$ 是简单平面图, 则它一定有一个度数 $\\leqslant 5$ 的顶点.", + "solution": "设 $G$ 是连通的, 否则考虑它的每一个连通分支.\n若每个顶点的度 $\\geqslant$ 6 , 则 $6 v \\leqslant 2 e$, 即 $v \\leqslant \\frac{e}{3}$. 又因为 $f \\leqslant \\frac{2 e}{3}$, 故 $2==v-e+f \\leqslant \\frac{e}{3}-e+\\frac{2 e}{3}=$ 0 , 矛盾.", + "remark": "", + "figures": [] +} \ No newline at end of file diff --git a/processed_dataset/proof/0497.json b/processed_dataset/proof/0497.json new file mode 100644 index 0000000000000000000000000000000000000000..4b85b847bd38faae9a640cbede8ede9e5d050bba --- /dev/null +++ b/processed_dataset/proof/0497.json @@ -0,0 +1,8 @@ +{ + "source_file": "./raw_volume-zh/volume12/exercise7.tex", + "problem_type": "proof", + "problem": "问题2. 证明: 小于 30 条边的简单平面图有一个顶点度数 $\\leqslant 4$.", + "solution": "设每个点的度数 $>4$, 则 $2 e=\\sum_{i=1}^v d\\left(v_i\\right) \\geqslant 5 v$, 即 $v \\leqslant \\frac{2}{5} e$, 由于 $e \\leqslant 3 v-6$, 代入得 $e \\leqslant \\frac{6}{5} e-6$, 即有 $e \\geqslant 30$, 矛盾.", + "remark": "", + "figures": [] +} \ No newline at end of file diff --git a/processed_dataset/proof/0498.json b/processed_dataset/proof/0498.json new file mode 100644 index 0000000000000000000000000000000000000000..031ccef86bb1e8336f56cf2f47f54955e6aa3f22 --- /dev/null +++ b/processed_dataset/proof/0498.json @@ -0,0 +1,8 @@ +{ + "source_file": "./raw_volume-zh/volume12/exercise7.tex", + "problem_type": "proof", + "problem": "问题3. 证明: 在 6 个顶点 12 条边的连通简单平面图中, 每个面用 3 条边围成.", + "solution": "由欧拉公式, $f=2+e-v=8$. 因为平均每个面有边 $\\frac{2 e}{f}=3$ 条, 由于每个面至少有三条边, 所以每个面恰有三条边.", + "remark": "", + "figures": [] +} \ No newline at end of file diff --git a/processed_dataset/proof/0499.json b/processed_dataset/proof/0499.json new file mode 100644 index 0000000000000000000000000000000000000000..1dbdf0c158139fe38560835254df29bbf2bbf043 --- /dev/null +++ b/processed_dataset/proof/0499.json @@ -0,0 +1,8 @@ +{ + "source_file": "./raw_volume-zh/volume12/exercise7.tex", + "problem_type": "proof", + "problem": "问题4. 设 $G$ 是有.\n11 个或更多顶点的图,证明 $G$ 或 $\\bar{G}$ 是非平面图.", + "solution": "设 $G$ 和 $\\bar{G}$ 都是平面图, 图 $G$ 和 $\\bar{G}$ 的顶点数是 $v$, 边数分别为 $e$ 和 $e^{\\prime}$, 则 $e+e^{\\prime}=\\frac{1}{2} v(v-1)$. 由不等式 $e \\leqslant 3 v-6, e^{\\prime} \\leqslant 3 v-6$, 相加得 $\\frac{1}{2} v(v-1)= e+e^{\\prime} \\leqslant 6 v-12, v^2-13 v+24 \\leqslant 0, v \\leqslant 11$, 与题设矛盾.", + "remark": "", + "figures": [] +} \ No newline at end of file diff --git a/processed_dataset/proof/0500.json b/processed_dataset/proof/0500.json new file mode 100644 index 0000000000000000000000000000000000000000..62dc987cf972c6928ab16b3a6a69cf443a8a2a0d --- /dev/null +++ b/processed_dataset/proof/0500.json @@ -0,0 +1,8 @@ +{ + "source_file": "./raw_volume-zh/volume12/exercise7.tex", + "problem_type": "proof", + "problem": "问题6. 证明: 除四面体外, 不存在这样一个凸多面体, 它的每一个顶点和所有其余的顶点之间都有棱相连接.", + "solution": "$n$ 个顶点的凸多面体有 $\\mathrm{C}_n^2$ 条棱, 每个面至少有 3 条棱, 故多面体的面数不大于 $\\frac{2}{3} \\mathrm{C}_n^2$, 由欧拉公式得: $n+\\frac{2}{3} \\mathrm{C}_n^2 \\geqslant \\mathrm{C}_n^2+2$, 化简得 $n^2-7 n+12 \\leqslant 0, n$ 只能取 3 或 4 ,命题得证.", + "remark": "", + "figures": [] +} \ No newline at end of file diff --git a/processed_dataset/proof/0501.json b/processed_dataset/proof/0501.json new file mode 100644 index 0000000000000000000000000000000000000000..a335c0e2157024750dd0b66b09ef91e4d060dd25 --- /dev/null +++ b/processed_dataset/proof/0501.json @@ -0,0 +1,8 @@ +{ + "source_file": "./raw_volume-zh/volume12/exercise7.tex", + "problem_type": "proof", + "problem": "问题7. 有 $n$ 个车站组成的公路网, 每个站至少有 6 条公路引出, 求证必有两条公路在平面上相交.", + "solution": "设 $G$ 是连通的, 否则考虑它的每一个连通分支.\n若每个顶点的度 $\\geqslant$ 6 , 则 $6 v \\leqslant 2 e$, 即 $v \\leqslant \\frac{e}{3}$. 又因为 $f \\leqslant \\frac{2 e}{3}$, 故 $2==v-e+f \\leqslant \\frac{e}{3}-e+\\frac{2 e}{3}=$ 0 , 矛盾.", + "remark": "", + "figures": [] +} \ No newline at end of file diff --git a/processed_dataset/proof/0502.json b/processed_dataset/proof/0502.json new file mode 100644 index 0000000000000000000000000000000000000000..38b1854ebc22b4afebb73724216e510a43ff6417 --- /dev/null +++ b/processed_dataset/proof/0502.json @@ -0,0 +1,8 @@ +{ + "source_file": "./raw_volume-zh/volume12/exercise7.tex", + "problem_type": "proof", + "problem": "问题9. 一个凸多面体有 $10 n$ 个面.\n求证: 有 $n$ 个面边数相同.", + "solution": "设这个凸多面体有 $x$ 个顶点,且设 $10 n$ 个面上分别有 $C_1, C_2, \\cdots, C_{10 n}$ 个顶点和 $a_1, a_2, \\cdots, a_{10 n}$ 条边,故凸多面体棱的数目为 $\\frac{1}{2} \\sum_{i=1}^{10 n} a_i$. 由欧拉定理得 $10 n+x=\\frac{1}{2} \\sum_{i=1}^{10 n} a_i+2$. 又 $x \\leqslant \\frac{1}{3} \\sum_{i=1}^{10 n} a_i$, 所以 $\\frac{1}{2} \\sum_{i=1}^{10 n} a_i+2-10 n \\leqslant \\frac{1}{3} \\sum_{i=1}^{10 n} a_i$, 即 $\\sum_{i=1}^{10 n} a_i \\leqslant 60 n-12$.\n若 $10 n$ 个面中不存在 $n$ 个面边数相同, 则 $\\sum_{i=1}^{10 n} a_i \\geqslant(3+4+\\cdots+12) (n-1)+13 \\times 10=75 n+55>60 n-12$, 矛盾, 所以至少有 $n$ 个面边数相同.", + "remark": "", + "figures": [] +} \ No newline at end of file diff --git a/processed_dataset/proof/0503.json b/processed_dataset/proof/0503.json new file mode 100644 index 0000000000000000000000000000000000000000..997cfec0acd3da4a569fb594b68913feb7b12deb --- /dev/null +++ b/processed_dataset/proof/0503.json @@ -0,0 +1,10 @@ +{ + "source_file": "./raw_volume-zh/volume12/exercise7.tex", + "problem_type": "proof", + "problem": "问题10. 证明如图()无哈密顿圈.", + "solution": "图中的面只有 2 边形及 6 边形两种, 分别有 3 个.\n若此图有哈密顿圈, 根据定理四, $4\\left(f_6^{\\prime}-f_6^{\\prime \\prime}\\right)=0$, 即 $f_6^{\\prime}=f_6^{\\prime \\prime}$, 但 $f_6^{\\prime}+f_6^{\\prime \\prime}=3$, 这不可能.", + "remark": "", + "figures": [ + "./images/volume12/figures/fig-c7p10.png" + ] +} \ No newline at end of file diff --git a/processed_dataset/proof/0504.json b/processed_dataset/proof/0504.json new file mode 100644 index 0000000000000000000000000000000000000000..c164c4cad75e3fc0b7ffeaddd4e94ae042d1ca06 --- /dev/null +++ b/processed_dataset/proof/0504.json @@ -0,0 +1,10 @@ +{ + "source_file": "./raw_volume-zh/volume12/exercise7.tex", + "problem_type": "proof", + "problem": "问题11. 如图(), 有哈密顿圈, 证明任一哈密顿圈如果有边 $e$, 那么这个圈一定不含边 $e^{\\prime}$.", + "solution": "由定理四, $2\\left(f_4^{\\prime}-f_4^{\\prime \\prime}\\right)+3\\left(f_5^{\\prime}-f_5^{\\prime \\prime}\\right)=0$. 所以 $f_4^{\\prime}-f_4^{\\prime \\prime}$ 是 3 的倍数, 即 5 个四边形中有 4 个在圈外,一个在圈内或 4 个在圈内, 一个在圈外.\n如果这个哈密顿圈既过 $e$, 又过 $e^{\\prime}$, 则 $e$ 的两侧的两个四边形分别在哈密顿圈的内部与外部, $e^{\\prime}$ 的两侧的两个四边形也分别在圈的内部与外部, 从而圈内、圈外至少各有两个四边形,矛盾.", + "remark": "", + "figures": [ + "./images/volume12/figures/fig-c7p11.png" + ] +} \ No newline at end of file diff --git a/processed_dataset/proof/0505.json b/processed_dataset/proof/0505.json new file mode 100644 index 0000000000000000000000000000000000000000..0dbafd5bc8feeea60fe2ea342565557afa99ba7f --- /dev/null +++ b/processed_dataset/proof/0505.json @@ -0,0 +1,10 @@ +{ + "source_file": "./raw_volume-zh/volume12/exercise7.tex", + "problem_type": "proof", + "problem": "问题12. 设 $S=\\left\\{x_1, x_2, \\cdots, x_n\\right\\}(n \\geqslant 3)$ 是平面上的一个点集, 它的任意两点间的距离至少为 1 . 证明最多有 $3 n-6$ 个点对, 它们之间的距离为 1 .", + "solution": "如图(), 作图 $G=(V, E)$, 其中 $V=\\left\\{x_1, x_2, \\cdots, x_n\\right\\}, G$ 中两顶点 $x_i, x_j$ 相邻的充要条件是 $d\\left(x_i, x_j\\right)=1$. 设 $G$ 中存在不相同的两边 $A B 、 C D$ 交于 $O$ 点, 如图所示.\n因为 $d(A, B)=1, d(C, D)=1$, 不失一般性,设 $d(O, A) \\leqslant \\frac{1}{2}, d(O, C) \\leqslant \\frac{1}{2}, A B$ 与 $C D$ 间夹角为 $\\theta$, 则 $d(A, C)= \\left\\{d^2(O, A)+d^2(O, C)-2 d(O, A) \\times d(O, C) \\cos \\theta\\right\\}^{\\frac{1}{2}}$,\n按上述条件仅当 $\\theta=\\pi$ 且 $d(O, A)=\\frac{1}{2}, d(O, C)=\\frac{1}{2}$ 时, $d(A, C)=1$. 但这时 $A$ 和 $D$ 重合, $B$ 和 $C$ 重合.\n即 $A B$ 与 $D C$ 为同一边,这与假设是两条不相同的边矛盾.\n除此情况外, 还有 $d(A, C)<1$, 这与 $S$ 中任意两点距离不小于 1 的假设条件矛盾.\n综上所述, $G$ 为平面图, 于是 $G$ 的边数 $e \\leqslant 3 n-6$.", + "remark": "", + "figures": [ + "./images/volume12/figures/fig-c7a12.png" + ] +} \ No newline at end of file diff --git a/processed_dataset/proof/0506.json b/processed_dataset/proof/0506.json new file mode 100644 index 0000000000000000000000000000000000000000..48bffb44f38834fd840954be6cf4b2dec0defba5 --- /dev/null +++ b/processed_dataset/proof/0506.json @@ -0,0 +1,10 @@ +{ + "source_file": "./raw_volume-zh/volume12/exercise8.tex", + "problem_type": "proof", + "problem": "问题1. 设 $A_1, A_2, A_3, A_4, A_5, A_6$ 是平面上的 6 点, 其中任意三点不共线.\n如果这些点之间任意连接 13 条线段,证明: 必存在 4 点, 它们每两点之间都有线段连接.", + "solution": "如图(), 将已连结的 13 条线段全染成红色, 还未连上的两条用蓝线连上 (因为所有两点连一线段时应该共有 15 条). 于是必有一个同色三角形,现在的蓝色线只有两条, 所以同色三角形必为红色的.\n不妨设 $\\triangle A_1 A_2 A_3$ 是红色的.\n从 $A_4 、 A_5 、 A_6$ 引向 $\\triangle A_1 A_2 A_3$ 顶点各有 3 条, 这 9 条线段中最多只有 2 条蓝色, 起码有 7 条是红色的, 因此, 或者是 $A_4$, 或者是 $A_5$, 或者是 $A_6$, 引向\n$\\triangle A_1 A_2 A_3$ 顶点的线段全是红色.\n比如说, $A_4 A_1 、 A_4 A_2 、 A_4 A_3$ 全是红色, 那么 4 点 $A_1 、 A_2 、 A_3 、 A_4$ 的每 2 点连线全是红色的, 命题得证.", + "remark": "", + "figures": [ + "./images/volume12/figures/fig-c8a1.png" + ] +} \ No newline at end of file diff --git a/processed_dataset/proof/0507.json b/processed_dataset/proof/0507.json new file mode 100644 index 0000000000000000000000000000000000000000..4e9d2197da632afa2932ab164656bb6fa49eaa05 --- /dev/null +++ b/processed_dataset/proof/0507.json @@ -0,0 +1,10 @@ +{ + "source_file": "./raw_volume-zh/volume12/exercise8.tex", + "problem_type": "proof", + "problem": "问题2. 空间八个点两两连线, 并二染色其边, 证明 : 必存在三条无公共端点的同色线段.", + "solution": "如图(), 若不存在三条无公共端点的同色线段, 如图, 不妨设 $A_1 A_2$ 为红色.\n由假设 $A_3 A_4, A_5 A_8, A_6 A_7$ 三线段不可能都是蓝色, 不妨设 $A_3 A_4$ 为红色.\n由于 $A_1 A_2$, $A_3 A_4$ 为红色,得 $A_5, A_6, A_7, A_8$ 四点两两连线皆蓝色,同样可得 $A_1, A_2, A_3, A_4$ 四点两两连线皆红色.\n不妨设 $A_1 A_6$ 为蓝色, 则 $A_3 A_8$ 必为红色, 此时不论对 $A_4 A_5$ 染什么颜色,均与假设矛盾.", + "remark": "", + "figures": [ + "./images/volume12/figures/fig-c8a2.png" + ] +} \ No newline at end of file diff --git a/processed_dataset/proof/0508.json b/processed_dataset/proof/0508.json new file mode 100644 index 0000000000000000000000000000000000000000..cd7296b6e021bd51191ae217c2fe3a17f84fa465 --- /dev/null +++ b/processed_dataset/proof/0508.json @@ -0,0 +1,8 @@ +{ + "source_file": "./raw_volume-zh/volume12/exercise8.tex", + "problem_type": "proof", + "problem": "问题3. 平面上有六点, 任何三点都是不等边三角形的顶点.\n证明 : 这些三角形中有一个的最短边同时是另一个三角形的最长边.", + "solution": "把每个三角形的最短边染成红色, 剩下的边染成蓝色.\n由于 $r_2=6$, 必出现同色三角形, 而这个同色三角形必定是红色的,所以它的最长边也是另一个三角形的最短边.", + "remark": "", + "figures": [] +} \ No newline at end of file diff --git a/processed_dataset/proof/0509.json b/processed_dataset/proof/0509.json new file mode 100644 index 0000000000000000000000000000000000000000..ebaf394d460034cbc4a566794c040aeb556e8a76 --- /dev/null +++ b/processed_dataset/proof/0509.json @@ -0,0 +1,8 @@ +{ + "source_file": "./raw_volume-zh/volume12/exercise8.tex", + "problem_type": "proof", + "problem": "问题4. 空间六条直线, 其中每三条直线都不共面.\n证明必存在三条直线, 满足下列条件之一: (i)两两异面; (ii) 互相平行; (iii)交于同一点.", + "solution": "用六个点表示六条直线, 若两直线异面, 则对应两点连线染红色; 若两直线共面, 则对应两点连线染蓝色.\n得两色完全图 $K_6$. 故其中必有同色三角形.\n若是红色三角形, 则三顶点对应的三直线两两异面, 若为蓝色三角形, 则三顶点对应的三直线两两共面, 由于题设三直线不共于一面, 故这三直线所在的三个平面两两相交于这三条直线, 由此得这三直线互相平行或交于一点.", + "remark": "", + "figures": [] +} \ No newline at end of file diff --git a/processed_dataset/proof/0510.json b/processed_dataset/proof/0510.json new file mode 100644 index 0000000000000000000000000000000000000000..d9804dc7f6b1af4eadb6cc751b6ff63ccf5f6074 --- /dev/null +++ b/processed_dataset/proof/0510.json @@ -0,0 +1,10 @@ +{ + "source_file": "./raw_volume-zh/volume12/exercise8.tex", + "problem_type": "proof", + "problem": "问题5. 设 $n$ 个新生中, 任意 3 个人中有两个人互相认识, 任意四个人中有两个人互不认识.\n试求 $n$ 的最大值.", + "solution": "所求 $n$ 的最大值为 8 .\n当 $n=8$ 时, 如图()所示的例子满足要求, 其中 $A_1$, $A_2, \\cdots, A_8$ 表示 8 个学生, $A_i$ 与 $A_j$ 连线表示 $A_i$ 与 $A_j$ 认识,否则不认识.\n下设 $n$ 个学生满足题设要求,我们来证明 $n \\leqslant 8$. 为此, 我们先来证明如下两种情况不可能出现:\n(1) 若某人 $A$ 至少认识 6 个人, 设为 $B_1, \\cdots, B_6$, 由\nRamsey 定理, 这 6 个人中存在 3 个互不相识 (这与已知任 3 个人中有 2 个相识矛盾); 或存在 3 个人互相认识, 这时 $A$ 与这 3 个人共 4 人两两互相认识, 亦与已知矛盾.\n(2) 若某人 $A$ 至多认识 $n-5$ 个人,则剩下至少 4 个人均与 $A$ 不相识, 从而这 4 个人两两相识,矛盾.\n其次, 当 $n \\geqslant 10$ 时,(1) 与 (2) 必有一种情况出现, 故此时 $n$ 不满足要求; 当 $n=9$ 时, 要使 (1) 与 (2) 均不出现, 则此时每个人恰好认识其他 5 个人, 于是这时 9 个人产生的朋友对 (相互认识的对子) 的数目为 $\\frac{9 \\times 5}{2} \\notin \\mathbf{N}^*$, 矛盾!", + "remark": "", + "figures": [ + "./images/volume12/figures/fig-c8a5.png" + ] +} \ No newline at end of file diff --git a/processed_dataset/proof/0511.json b/processed_dataset/proof/0511.json new file mode 100644 index 0000000000000000000000000000000000000000..55a46fe3f1ca91b51a967aff49b676764ed07bf5 --- /dev/null +++ b/processed_dataset/proof/0511.json @@ -0,0 +1,8 @@ +{ + "source_file": "./raw_volume-zh/volume12/exercise8.tex", + "problem_type": "proof", + "problem": "问题6. 把连接圆周上 9 个不同点的 36 条边染成红色或蓝色.\n假定由 9 个点中每 3 个点所确定的三角形都含有红边, 证明有 4 个点, 其中每两点连的都是红边.", + "solution": "取两色完全图 $K_9$ 的顶点 $A$, 若 $A$ 连有 4 条蓝边 $A A_1, A A_2, A A_3$, $A A_4$, 则 $K_9$ 中以 $A_1, A_2, A_3, A_4$ 为顶点的完全子图 $K_4$ 不含蓝边.\n如果 $A$ 连有 6 条红边 $A A_1, A A_2, \\cdots, A A_6$, 则 $K_9$ 中以 $A_1, \\cdots, A_6$ 为顶点的完全子图含有同色三角形 $\\triangle A_i A_j A_k(1 \\leqslant i, j, k \\leqslant 6)$. 由于 $K_9$ 不含蓝色三角形, 故 $\\triangle A_i A_j A_k$ 是红色三角形.\n则 $A, A_i, A_j, A_k$ 为顶点的完全子图 $K_4$ 是红色的.\n如果 $K_9$ 中每一顶点都恰好连有 5 条红边, 则 $K_9$ 中红边数为 $\\frac{5 \\times 9}{2}$, 不可能.", + "remark": "", + "figures": [] +} \ No newline at end of file diff --git a/processed_dataset/proof/0512.json b/processed_dataset/proof/0512.json new file mode 100644 index 0000000000000000000000000000000000000000..8087ce6042d792b9433b90a4cb193bbb5eaf792a --- /dev/null +++ b/processed_dataset/proof/0512.json @@ -0,0 +1,8 @@ +{ + "source_file": "./raw_volume-zh/volume12/exercise8.tex", + "problem_type": "proof", + "problem": "问题7. 证明: 在任何 19 个人中, 总有 3 个人互相认识或者 6 个人互相不认识.", + "solution": "由定理三, $r(3,6) \\leqslant r(3,5)+r(2,6)-1=14+6-1=19$.", + "remark": "", + "figures": [] +} \ No newline at end of file diff --git a/processed_dataset/proof/0513.json b/processed_dataset/proof/0513.json new file mode 100644 index 0000000000000000000000000000000000000000..19511515549de1204f94884d098e85fc68dd81cb --- /dev/null +++ b/processed_dataset/proof/0513.json @@ -0,0 +1,8 @@ +{ + "source_file": "./raw_volume-zh/volume12/exercise8.tex", + "problem_type": "proof", + "problem": "问题8. 证明: 在任何 18 个人中, 总有 4 个人相互认识, 或者相互不认识.", + "solution": "由定理三, $r(4,4) \\leqslant r(4,3)+r(3,4)=9+9=18$.", + "remark": "", + "figures": [] +} \ No newline at end of file diff --git a/processed_dataset/proof/0514.json b/processed_dataset/proof/0514.json new file mode 100644 index 0000000000000000000000000000000000000000..dc563a7fc0b1bf4ba970ef4f067f3309b7a78e93 --- /dev/null +++ b/processed_dataset/proof/0514.json @@ -0,0 +1,8 @@ +{ + "source_file": "./raw_volume-zh/volume12/exercise8.tex", + "problem_type": "proof", + "problem": "问题9.求证: 将自然数 $1,2, \\cdots, N$ 分到 $n$ 个类中, 则在 $N$ 充分大时,一定有一个类同时含有数 $x 、 y$ 及这两个数的差 $|x-y|$. (许尔(Schur) 定理)", + "solution": "考虑 $n$ 色完全图 $K_{r_n}$, 染色方法是当且仅当 $|x-y|$ 在第 $i$ 类时,将 $(x$, $y$ ) 染上第 $i$ 种颜色.\n由定理二, $K_{r_n}$ 中一定有一个同色的三角形.\n设这三角形的三边均为第 $j$ 种颜色, 则在 $1,2, \\cdots, r_n$ 中有三个自然数 $a>b>c$, 使 $x= a-c, y=a-b, z=b-c=x-y$ 均在第 $j$ 类中.", + "remark": "", + "figures": [] +} \ No newline at end of file diff --git a/processed_dataset/proof/0515.json b/processed_dataset/proof/0515.json new file mode 100644 index 0000000000000000000000000000000000000000..80b182dfb57b0e24a1acd11c8c644bb7cefd4d2c --- /dev/null +++ b/processed_dataset/proof/0515.json @@ -0,0 +1,8 @@ +{ + "source_file": "./raw_volume-zh/volume12/exercise8.tex", + "problem_type": "proof", + "problem": "问题10. 证明: 在两色完全图 $K_7$ 中, 必有两个无公共边的同色三角形.", + "solution": "在 $A_1, A_2, \\cdots, A_7$ 中, 前 6 个点构成的三角形必有两个是同色的.\n它们如果没有公共边, 则命题已成立.\n如果有公共边, 不妨设为 $\\triangle A_1 A_2 A_3$ 、 $\\triangle A_1 A_2 A_4$, 现除去 $A_1$ 点, 加进 $A_7$ 点, 则又存在两个同色三角形, 在这两个同色三角形中必有一个不同于 $\\triangle A_2 A_3 A_4$, 这个三角形与 $\\triangle A_1 A_2 A_3$ 或 $\\triangle A_1 A_2 A_4$ 中的一个无公共边.", + "remark": "", + "figures": [] +} \ No newline at end of file diff --git a/processed_dataset/proof/0516.json b/processed_dataset/proof/0516.json new file mode 100644 index 0000000000000000000000000000000000000000..d74e96324904a23c8073efa8d58970096401b65a --- /dev/null +++ b/processed_dataset/proof/0516.json @@ -0,0 +1,8 @@ +{ + "source_file": "./raw_volume-zh/volume12/exercise8.tex", + "problem_type": "proof", + "problem": "问题11. 求最小正整数 $n$, 使得在任意给定的 $n$ 个无理数中, 总存在这样的三个无理数,其中任意两个数之和仍是无理数.\n但没有公共边的两个单色三角形.", + "solution": "取由四个无理数组成的集合 $\\{\\sqrt{2},-\\sqrt{2}, \\sqrt{3},-\\sqrt{3}\\}$. 从这个集合中任意取出三个数, 必然是: 或者 $\\sqrt{2},-\\sqrt{2}$ 都被取出, 此时 $\\sqrt{2}+(-\\sqrt{2})=0$ 是有理数; 或者 $\\sqrt{3},-\\sqrt{3}$ 都被取出, 此时 $\\sqrt{3}+(-\\sqrt{3})=0$ 是有理数.\n可见当 $n=4$ 时题中结论不成立.\n因此, 满足题中要求的 $n$, 必有 $n \\geqslant 5$. 下面证明: 任意给定五个无理数, 总可以从中找到这样的三个, 其中任意两个无理数之和仍是无理数.\n设 $\\{x, y, z, u, v\\}$ 是任意给定的由五个无理数组成的集合.\n把这五个数当成五个点, 如果两个数之和为无理数, 则对应的两点之间联结一条红色边;\n如果两个数之和为有理数,则对应的两点之间联结一条蓝色边.\n于是得一个 2 色 5 阶完全图 $K_5$. 先证明这个 2 色 5 阶完全图 $K_5$ 中不含蓝边三角形.\n否则, 设有蓝边三角形 $x y z$, 即 $x+y, y+z, z+x$ 都是有理数,则\n$$\nx=\\frac{1}{2}[(x+y)+(z+x)-(y+z)]\n$$\n也应是有理数,与 $x$ 是无理数矛盾.\n再证明这个 2 色完全图 $K_5$ 中不含蓝边五边形.\n否则, 设有蓝边五边形 $x y z u v$, 即 $x+y, y+z, z+u, u+v, v+x$ 都是有理数, 则\n$$\nx=\\frac{1}{2}[(x+y)+(z+u)+(v+x)-(y+z)-(u+v)]\n$$\n也应是有理数,与 $x$ 是无理数矛盾.\n这个 $K_5$ 中既不含蓝边五边形, 也不含蓝边三角形, 故由图 8-3 后的 * 得知, 必含红边三角形.\n设 $\\triangle x y z$ 是红边三角形,则有 $x+y, y+z, z+x$ 都是无理数.", + "remark": "", + "figures": [] +} \ No newline at end of file diff --git a/processed_dataset/proof/0517.json b/processed_dataset/proof/0517.json new file mode 100644 index 0000000000000000000000000000000000000000..6e8b5b31a9f1ea7c2ec16eea26ac657cbd2e4efe --- /dev/null +++ b/processed_dataset/proof/0517.json @@ -0,0 +1,11 @@ +{ + "source_file": "./raw_volume-zh/volume12/exercise8.tex", + "problem_type": "proof", + "problem": "问题12. 求最小正整数 $n$, 使当以任意方式将 $\\mathrm{K}_{n}$ 二染色时,总存在具有相同颜色但没有公共边的两个单色三角形", + "solution": "如图(), 对 $K_7$ 二染色, 红色边以实线、蓝色边以虚线表示.\n有 4 个红色三角形: $\\triangle A_1 A_4 A_6 、 \\triangle A_2 A_4 A_6 、 \\triangle A_3 A_4 A_6 、 \\triangle A_7 A_4 A_6$, 有 4 个蓝色三角形: $\\triangle A_1 A_2 A_3 、 \\triangle A_2 A_3 A_7 、 \\triangle A_1 A_3 A_7 、 \\triangle A_1 A_2 A_7$, 易见任何两个相同颜色的单色三角形都有一条公共边,所以 $n \\geqslant 8$.\n下面证明 $n=8$ 时命题一定成立.\n用反证法证之.\n先证明一个引理: 倘若命题不成立, 则必存在一个红色三角形和一个蓝色三角形, 二者恰有一个公共点.\n首先, 二染色 $K_8$ 必存在一个单色三角形.\n不妨设其为蓝色 $\\triangle A_1 A_2 A_3$. 这时 $A_3 A_4 A_5 A_6 A_7 A_8$ 中必存在一个单色三角形, 显然不能是蓝色三角形.\n若这个红色三角形含顶点 $A_3$, 则引理成立.\n不然设 $\\triangle A_4 A_5 A_6$ 为红色三角形, 则 $\\triangle A_1 A_2 A_3$ 与 $\\triangle A_4 A_5 A_6$ 间有 9 条连线,至少有 5 条是同色的.\n不妨设为红色, 则 $A_1, A_2, A_3$ 发出至少 5 条红边, 其中有一点至少发出 2 条红边, 由该点形.\n成一个红色三角形且与 $\\triangle A_1 A_2 A_3$ 有一个公共点.\n故引理必成立.\n下面证本命题: 倘若命题不成立, 由引理可设 $\\triangle A_1 A_2 A_3$ 为蓝色三角形, $\\triangle A_3 A_4 A_5$ 为红色三角形.\n考虑 $A_1 A_4 A_6 A_7 A_8$ 间的连线,显然其内不能再有单色三角形,故该 $K_5$ 由一个蓝五圈和一个红五圈组成.\n见如图(), 红色边用实线,蓝色边用虚线,不妨设 $A_1 A_4 A_6 A_7 A_8$ 为蓝五圈, $A_1 A_7 A_4 A_8 A_6$ 为红五圈,下面讨论 $A_3 A_7$ 的颜色.\n若 $A_3 A_7$ 为蓝色, 则 $A_3 A_8 、 A_3 A_6$ 必为红色 (不然 $\\triangle A_3 A_7 A_8$ 或 $A_3 A_6 A_7$ 为蓝色, 与 $\\triangle A_1 A_2 A_3$ 发生矛盾). 而此时 $\\triangle A_3 A_6 A_8$ 为红色, 又与 $\\triangle A_3 A_4 A_5$ 发生矛盾!\n若 $A_3 A_7$ 为红色.\n再讨论 $A_3 A_8$ 的颜色.\n若 $A_3 A_8$ 为蓝色, 则 $A_2 A_4$ 必须为红色, $A_2 A_8$ 必须为蓝色, $A_2 A_7$ 必须为红色.\n故 $\\triangle A_2 A_4 A_7$ 为红色三角形, 与 $\\triangle A_3 A_4 A_5$ 发生矛盾!\n若 $A_3 A_8$ 也是红色, 则 $A_5 A_7$ 必须是蓝色, $A_5 A_8$ 必须是蓝色.\n故 $\\triangle A_5 A_7 A_8$ 是蓝色, 与 $\\triangle A_1 A_2 A_3$ 发生矛盾!\n综上所述, $n=8$ 时命题必成立,故所求最小自然数是 8 .", + "remark": "", + "figures": [ + "./images/volume12/figures/fig-c8a12-1.png", + "./images/volume12/figures/fig-c8a12-2.png" + ] +} \ No newline at end of file diff --git a/processed_dataset/proof/0518.json b/processed_dataset/proof/0518.json new file mode 100644 index 0000000000000000000000000000000000000000..b92f88388625f25dbecefe4e32975c9abe93cdea --- /dev/null +++ b/processed_dataset/proof/0518.json @@ -0,0 +1,8 @@ +{ + "source_file": "./raw_volume-zh/volume12/exercise8.tex", + "problem_type": "proof", + "problem": "问题13. 在一个足球联赛里有 20 支足球队.\n第一轮它们分成 10 对互相比赛, 第二轮也分成 10 对互相比赛 (注: 每支球队两轮比赛的对手不一定不同). 求证: 在第三轮开赛之前,一定可以找出 10 支球队, 它们两两没有赛过.", + "solution": "用 20 个点代表 20 支球队.\n在第一轮互相比赛过的球队之间连红线, 在第二轮互相比赛过的球队之间连蓝线, 则每一个点都连出一红一蓝两条边, 从而整个图必然由若干个偶圈组成.\n在每个偶圈中可以选出半数顶点, 任两个顶点不相邻,则共选出了 10 支球队, 两两未赛过.\n原命题得证.", + "remark": "", + "figures": [] +} \ No newline at end of file diff --git a/processed_dataset/proof/0519.json b/processed_dataset/proof/0519.json new file mode 100644 index 0000000000000000000000000000000000000000..afc47f991ef48eaa6150bc30d1d248bf6d1c2086 --- /dev/null +++ b/processed_dataset/proof/0519.json @@ -0,0 +1,11 @@ +{ + "source_file": "./raw_volume-zh/volume12/exercise9.tex", + "problem_type": "proof", + "problem": "问题1. $n(>4)$ 个城市, 每两个城市之间有一条直达道路.\n证明: 可将这些道路改为单行道, 使得从任意城市可能到达任一其他城市, 中间至多经过 1 个城市.", + "solution": "(1) 当 $n=5$ 时,图()为所求.\n当 $n=6$ 时,图()为所求.\n(2)假设 $n=k$ 时,存在满足要求的有向图.\n当 $n=k+2$ 时, 先在顶点 $V_1, V_2, \\cdots, V_k$ 间作出满足要求的 $k$ 阶有向图.\n对于另两个顶点 $V_{k+1}, V_{k+2}$, 令 $V_1, V_2, \\cdots, V_k$ 均指向 $V_{k+1} ; V_{k+2}$ 指向 $V_1, V_2, \\cdots, V_k$, 再令 $V_{k+1}$ 指向 $V_{k+2}$, 则 $V_{k+1}$ 通过 $V_{k+2}$ 到达 $V_1, V_2, \\cdots$ (显然 $V_{k+2}$ 可以直达 $V_1, V_2, \\cdots, V_k$ ), $V_1, V_2, \\cdots, V_k$ 可通过 $V_{k+1}$ 到达 $V_{k+2}$ (显然 $V_1, V_2, \\cdots, V_k$ 可以直达 $\\left.V_{k+1}\\right)$. 故该 $k+2$ 阶图仍满足要求.\n由(1)、(2) 知对任意 $4160 .\n$$\n下证 $S>161$.\n因为 $a_1 b_1 c_1, a_2 b_2 c_2, \\cdots, a_6 b_6 c_6$ 这 6 个数的几何平均为 $12 \\sqrt{5}$, 而 $26< 12 \\sqrt{5}<27$, 所以 $a_1 b_1 c_1, a_2 b_2 c_2, \\cdots, a_6 b_6 c_6$ 中必有一个数不小于 27 , 也必有一个数不大于 26 , 而 26 不是 $1,2,3,4,5,6$ 中某三个 (可以重复) 的积, 所以必有一个数不大于 25 .\n不妨设 $a_1 b_1 c_1 \\geqslant 27, a_2 b_2 c_2 \\leqslant 25$,于是\n$$\n\\begin{aligned}\nS= & \\left(\\sqrt{a_1 b_1 c_1}-\\sqrt{a_2 b_2 c_2}\\right)^2+2 \\sqrt{a_1 b_1 c_1 a_2 b_2 c_2}+\\left(a_3 b_3 c_3+a_4 b_4 c_4\\right)+\\left(a_5 b_5 c_5+\\right. \\\\\n& \\left.a_6 b_6 c_6\\right) \\\\\n\\geqslant & (\\sqrt{27}-\\sqrt{25})^2+2 \\sqrt{a_1 b_1 c_1 a_2 b_2 c_2}+2 \\sqrt{a_3 b_3 c_3 a_4 b_4 c_4}+2 \\sqrt{a_5 b_5 c_5 a_6 b_6 c_6} \\\\\n\\geqslant & (3 \\sqrt{3}-5)^2+2 \\cdot 3 \\sqrt[6]{\\prod_{i=1}^6 a_i b_i c_i} \\\\\n= & (3 \\sqrt{3}-5)^2+72 \\sqrt{5}>161,\n\\end{aligned}\n$$\n所以 $S \\geqslant 162$.\n又当 $a_1, a_2, \\cdots, a_6 ; b_1, b_2, \\cdots, b_6$ 和 $c_1, c_2, \\cdots, c_6$ 都分别为 $1,2,3$ , $4,5,6 ; 5,4,3,6,1,2 ; 5,4,3,1,6,2$ 时, 有 $S=1 \\times 5 \\times 5+2 \\times 4 \\times 4+3 \\times 3 \\times 3+4 \\times 6 \\times 1+5 \\times 1 \\times 6+6 \\times 2 \\times 2=162$, 所以, $S$ 的最小值为 162.", + "remark": "", + "figures": [] +} \ No newline at end of file diff --git a/processed_dataset/proof/0527.json b/processed_dataset/proof/0527.json new file mode 100644 index 0000000000000000000000000000000000000000..d5dca9c48742ca2356d9a0b4f2344bb5111d8625 --- /dev/null +++ b/processed_dataset/proof/0527.json @@ -0,0 +1,8 @@ +{ + "source_file": "./raw_volume-zh/volume13/chapter1.tex", + "problem_type": "proof", + "problem": "例6. 对正整数 $M$, 如果存在整数 $a 、 b 、 c 、 d$, 使得 $M \\leqslant a2$, 即 $m \\geqslant 3$.\n这个估计虽不是最优的, 但由此可找到得分更高的人.\n因为 $m \\geqslant 3$ 表明: 至少有 3 个人与 $A$ 打成平局.\n设其中的一个人为 $B$. 那么, $B$ 至少得 $(2 n+2)+1=2 n+3$ 分.\n于是, $2 n+3<2 n+m, m>3$, 即 $m \\geqslant 4$.\n下证 $n>0$. 实际上, 若 $n=0$, 即 $A$ 未胜一场, 则 $A$ 的得分 $S(A) \\leqslant r-1$. 但 $r$ 个人的得分总和为 $2 \\mathrm{C}_r^2=r(r-1)$, 每个人平均可得分 $r-1$. 但 $A$ 的得分最多,所以, $S(A)=r-1$. 这样, 每个人都得 $r-1$ 分, 矛盾.\n综上, $r \\geqslant(m+n)+1 \\geqslant 6$.\n最后, $r=6$ 是可能的,各人得分如下表所示:\n\\begin{tabular}{|c|c|c|c|c|c|c|c|}\n\\hline & $A$ & $B$ & $C$ & $D$ & $E$ & $F$ & 总分 \\\\\n\\hline$A$ & & 1 & 1 & 1 & 1 & 2 & 6 \\\\\n\\hline$B$ & 1 & & 2 & 0 & 0 & 2 & 5 \\\\\n\\hline$C$ & 1 & 0 & & 0 & 2 & 2 & 5 \\\\\n\\hline$D$ & 1 & 2 & 2 & & 0 & 0 & 5 \\\\\n\\hline$E$ & 1 & 2 & 0 & 2 & & 0 & 5 \\\\\n\\hline$F$ & 0 & 0 & 0 & 2 & 2 & & 4 \\\\\n\\hline\n\\end{tabular}\n故 $r$ 的最小值是 6 .", + "remark": "", + "figures": [] +} \ No newline at end of file diff --git a/processed_dataset/proof/0530.json b/processed_dataset/proof/0530.json new file mode 100644 index 0000000000000000000000000000000000000000..f471429172f1b31cdd5d4e64846a4a58cf1eee4c --- /dev/null +++ b/processed_dataset/proof/0530.json @@ -0,0 +1,8 @@ +{ + "source_file": "./raw_volume-zh/volume13/chapter10.tex", + "problem_type": "proof", + "problem": "例4. 有 1000 张编号为 $000,001, \\cdots, 999$ 的证件和 100 个编号为 00 , $01, \\cdots, 99$ 的盒子.\n若盒子的号码可以由证件的号码划掉一个数字而得到, 则该证件可以放人该盒子中.\n若选择 $k$ 个盒子可以装下所有证件, 求 $k$ 的最小值.", + "solution": "分析:解找一个充分条件,使选定若干个盒子能装下所有的证件.\n考察所有含有数字 $a 、 b 、 c(a 、 b 、 c$ 可能相同)为编号的证件, 要装下这些证件, 则一定有含有其中某两个数 (比如 $a 、 b$ ) 为编号的盒子.\n为了保证数字的不同排列顺序为编号的证件都能装下,一个充分条件是, 编号为 $\\overline{a b} 、 \\overline{b a}$ 的盒子都被选.\n也就是说,要使得\"任何三个数中都有两个数被选取, 且这两个数的任何顺序都被选取\". 由此联想到抽屉原理: 3 个数归人 2 个集合, 必有一个集合中有两个数.\n于是, 将 $0,1,2, \\cdots, 9$ 划分为两个子集 $A 、 B$, 只要同一集合中任何 2 数组都被选取, 且每个 2 数组的任何顺序(即所有可重复元素的 2 元排列)都被选取 (保证不管什么顺序都可放下), 则这些编号合乎要求.\n设 $|A|=k,|B|=10-k$, 则 $A 、 B$ 中的元素可重复的 2 元排列分别有 $k^2 、(10-k)^2$ 个, 所以这样的 2 元排列共有 $k^2+(10-k)^2=2(k-5)^2+ 50 \\geqslant 50$ 个.\n取 $k=5$, 即 $|A|=|B|=5$, 比如, $A=\\{0,1,2,3,4\\}, B=\\{5,6,7$, $8,9\\}$. 则 $A 、 B$ 中的元素可重复的 2 元排列各有 25 个,相应的 50 个编号合乎要求.\n下面证明 $k \\geqslant 50$.\n设选用的 $k$ 个盒子中, 以 9 为首位的编号最少, 设有 $m$ (参数) 个, 记为 $\\overline{9 a_i} (i=1,2, \\cdots, m)$. 令 $A=\\left\\{a_1, a_2, \\cdots, a_m\\right\\}$, 任取不属于 $A$ 的两个数 $a 、 b$, 考察编号为 $\\overline{9 a b}$ 的证件.\n因为 $a$ 不属于 $A$, 所以没有编号为 $\\overline{9 a}$ 的盒子.\n同样, 没有编号为 $\\overline{9 b}$ 的盒子.\n于是必选编号为 $\\overline{a b}$ 的盒子, 注意到 $a 、 b$ 均有 $10-m$ 个取值, 于是, 这样的盒子应有 $(10-m)^2$ 个, 且这些盒子都不以 $a_1, a_2, \\cdots, a_m$ 为首位.\n又由 \"最少性\", 以 $a_1, a_2, \\cdots, a_m$ 为首位的盒子至少都有 $m$ 个, 所以这样的盒子至少有 $m^2$ 个.\n于是, $k \\geqslant m^2+(10-m)^2 \\geqslant \\frac{1}{2}[m+(10-m)]^2$. $=50$.\n综上所述, $k$ 的最小值为 50 .", + "remark": "", + "figures": [] +} \ No newline at end of file diff --git a/processed_dataset/proof/0531.json b/processed_dataset/proof/0531.json new file mode 100644 index 0000000000000000000000000000000000000000..e1e4c115b2fc2da836132ef2d84bb7f2093d24dc --- /dev/null +++ b/processed_dataset/proof/0531.json @@ -0,0 +1,8 @@ +{ + "source_file": "./raw_volume-zh/volume13/chapter10.tex", + "problem_type": "proof", + "problem": "例6. 对于整数 $n \\geqslant 4$, 求出最小的整数 $f(n)$, 使得对于任何正整数 $m$, 集合 $\\{m, m+1, \\cdots, m+n-1\\}$ 的任一个 $f(n)$ 元子集中, 均有至少 3 个两两互素的元素.", + "solution": "解: $n \\geqslant 4$ 时, 对集合 $M=\\{m, m+1, m+2, \\cdots, m+n-1\\}$, 若 $m$ 为奇数,则 $m, m+1, m+2$ 两两互质; 若 $m$ 为偶数,则 $m+1, m+2, m+3$ 两两互质, 于是 $M$ 的所有 $n$ 元子集中都至少有 3 个两两互质的数, 所以 $f(n)$ 存在, 且 $f(n) \\leqslant n$.\n设 $T_n=\\{t \\mid t \\leqslant n+1$, 且 $2 \\mid t$ 或 $3 \\mid t\\}$, 则 $T_n$ 为 $\\{2,3, \\cdots, n+1\\}$ 的子集, 但 $T_n$ 中任何 3 个数都不两两互质,所以 $f(n) \\geqslant\\left|T_n\\right|+1$.\n由容斥原理, $\\left|T_n\\right|=\\left[\\frac{n+1}{2}\\right]+\\left[\\frac{n+1}{3}\\right]-\\left[\\frac{n+1}{6}\\right]+1$, 所以 $f(n) \\geqslant \\left[\\frac{n+1}{2}\\right]+\\left[\\frac{n+1}{3}\\right]-\\left[\\frac{n+1}{6}\\right]+1$.\n此外, 注意到 $\\{m, m+1, m+2, \\cdots, m+n\\}=\\{m, m+1, m+2, \\cdots$, $m+n-1\\} \\cup\\{m+n\\}$, 所以 $f(n+1) \\leqslant f(n)+1$.\n所以 $f(4) \\geqslant 4, f(5) \\geqslant 5, f(6) \\geqslant 5, f(7) \\geqslant 6, f(8) \\geqslant 7, f(9) \\geqslant 8$.\n下面证明 $f(6)=5$.\n设 $x_1, x_2, x_3, x_4, x_5 \\in\\{m, m+1, m+2, \\cdots, m+5\\}$, 并设 $x_i(1 \\leqslant i \\leqslant 5)$ 中有 $k$ 个为奇数.\n因为 $\\{m, m+1, m+2, \\cdots, m+5\\}$ 中最多 3 个偶数, 从而 $x_1, x_2, x_3$, $x_4, x_5$ 中最多 3 个偶数, 所以 $k \\geqslant 2$.\n因为 $\\{m, m+1, m+2, \\cdots, m+5\\}$ 中最多 3 个奇数, 从而 $x_1, x_2, x_3$, $x_4, x_5$ 中最多 3 个奇数, 所以 $k \\leqslant 3$.\n若 $k=3$, 则 3 个奇数两两互质;\n若 $k=2$, 则不妨设 $x_1, x_2$ 为奇数, $x_3, x_4, x_5$ 为偶数, 当 $3 \\leqslant i4$.\n设 $M$ 是有点最多的圆, 由 $r>4$ 知 $M$ 上至少有 5 个点 $A 、 B 、 C 、 D 、 E$. 下面证明:其他点 (最多一个点除外)都在此圆周上. (*)\n实际上, 反设有两个点 $P 、 Q$ 不在圆 $M$ 上, 那么 $P 、 Q 、 A 、 B 、 C$ 这 5 点中有 4 个点共圆.\n但 $P 、 Q$ 都不在圆 $A B C$ 上, 只能是 $P 、 Q$ 与 $A 、 B 、 C$ 中的某两个点共圆, 不妨设 $P 、 Q 、 A 、 B$ 共圆 $M_1$.\n同样考察 5 点 $P 、 Q 、 C 、 D 、 E$, 必有 $P 、 Q$ 与 $C 、 D 、 E$ 中的某两个点共圆, 不妨设 $P 、 Q 、 C 、 D$ 共圆 $M_2$. 再考察 5 点 $P 、 Q 、 A 、 C 、 E$, 必有 $P 、 Q$ 与 $A 、 C 、 E$ 中的某两个点共圆.\n(i) 若 $P 、 Q 、 A 、 C$ 共圆 $M_3$, 则 $M_3$ 与 $M_1$ 重合, 所以 $P Q A B C$ 共圆, $P$ 、 $Q$ 在圆 $M$ 上,矛盾.\n(ii) 若 $P 、 Q 、 A 、 E$ 共圆 $M_3$, 则 $M_3$ 与 $M_1$ 重合, 所以 $P Q A B E$ 共圆, $P$ 、 $Q$ 在圆 $M$ 上,矛盾.\n(iii)若 $P 、 Q 、 C 、 E$ 共圆 $M_3$, 则 $M_3$ 与 $M_2$ 重合, 所以 $P Q C D E$ 共圆, $P$ 、 $Q$ 在圆 $M$ 上,矛盾.\n综上所述, 结论 (*) 成立, 即 $r \\geqslant 9$. 最后, $r=9$ 是可能的, 即 9 个点共圆, 另一个点在圆外显然合乎条件.\n综上所述, $r$ 的最小值为 9 .", + "remark": "注: 本题有相当的难度, 但当年的得分率却出乎意料的高.\n其原因是很容易猜出答案, 使思维有明确的方向.\n只要否定了 $r=4$, 即可发现 $r=9$, 而构造则是相当容易的.\n我们还可进一步考虑: 将题中的 10 个点推广到 $n$ 个点,结论如何?", + "figures": [] +} \ No newline at end of file diff --git a/processed_dataset/proof/0535.json b/processed_dataset/proof/0535.json new file mode 100644 index 0000000000000000000000000000000000000000..5008b8c06cb56c8072dfbfdd91041c4cb15c83e5 --- /dev/null +++ b/processed_dataset/proof/0535.json @@ -0,0 +1,8 @@ +{ + "source_file": "./raw_volume-zh/volume13/chapter11.tex", + "problem_type": "proof", + "problem": "例6. 有 16 名学生参加考试, 考题都是选择题, 每题有 4 个选择支, 考完后发现: 任何两人至多有一道题答案相同, 问最多有几道考题?", + "solution": "分析:解设共有 $n$ 道试题, 我们证明 $n_{\\text {max }}=5$.\n对每一个题, 16 个学生的答案构成一个长为 16 的只含有 $1 、 2 、 3 、 4$ (答案代号) 的数列, 将 $n$ 个题对应的数列排成一个 $n \\times 16$ 的数表.\n注意到条件\"任何两人至多有一道题答案相同\", 可计算每一道题 (每一行中)出现的相同答案构成的对子.\n为叙述问题方便, 对每一行, 如果两个答案代号相同, 则将这两个数字用一条线段连接, 称之为同色线段.\n设某一行有 $x$ 个 $1 、 y$ 个 $2 、 z$ 个 $3 、 t$ 个 4 , 其中 $x+y+z+t=16$, 则该行共有 $\\mathrm{C}_x^2+\\mathrm{C}_y^2+ \\mathrm{C}_z^2+\\mathrm{C}_t^2 \\geqslant \\mathrm{C}_4^2+\\mathrm{C}_4^2+\\mathrm{C}_4^2+\\mathrm{C}_4^2=24$ (条) 同色线段, 于是 $n$ 行至少有 $24 n$ 条同色线段.\n依题意, 任何两条同色线段在第 $n$ 行上的投影互不相同, 于是 $24 n \\leqslant \\mathrm{C}_{16}^2=120$,所以 $n \\leqslant 5$.\n若 $n=5$, 则上述所有不等式都成立等号, 于是, 合乎条件的数表应满足以下两点:\n(1) 每行 4 个 $1 、 4$ 个 $2 、 4$ 个 $3 、 4$ 个 4 ;\n(2) 不同的同色线段在第 $n$ 行上的投影不重合.\n不妨设第一行为 1111222233334444 (看成 4 组), 则其他行的每一个组都是 1234 的一个排列 (因为同色线段不重合).一种自然的排列是 1234 , 我们可在表中尽可能多地填人 1234 , 可发现第 2 行相应的组中都可填 1234 , 而第一组 (前 4 列) 除第一行外都可填 1234. 如此下去, 即可得到如下合乎条件的数表,它表明 $n=5$ 是可能的.\n\\begin{tabular}{|c|c|c|c|c|c|c|c|c|c|c|c|c|c|c|c|c|}\n\\hline 学生 & 1 & 2 & 3 & 4 & 5 & 6 & 7 & 8 & 9 & 10 & 11 & 12 & 13 & 14 & 15 & 16 \\\\\n\\hline 1 & 1 & 1 & 1 & 1 & 2 & 2 & 2 & 2 & 3 & 3 & 3 & 3 & 4 & 4 & 4 & 4 \\\\\n\\hline 2 & 1 & 2 & 3 & 4 & 1 & 2 & 3 & 4 & 1 & 2 & 3 & 4 & 1 & 2 & 3 & 4 \\\\\n\\hline 3 & 1 & 2 & 3 & 4 & 4 & 3 & 2 & 1 & 3 & 4 & 1 & 2 & 2 & 1 & 4 & 3 \\\\\n\\hline 4 & 1 & 2 & 3 & 4 & 2 & 1 & 4 & 3 & 4 & 3 & 2 & 1 & 3 & 4 & 1 & 2 \\\\\n\\hline 5 & 1 & 2 & 3 & 4 & 3 & 4 & 1 & 2 & 2 & 1 & 4 & 3 & 4 & 3 & 2 & 1 \\\\\n\\hline\n\\end{tabular}", + "remark": "注:: 本题原来的解答是采用整体估计处理的 ,这里采用算两次的技巧,不仅思路自然流畅,更重要的是为后面的构造指明了方向.", + "figures": [] +} \ No newline at end of file diff --git a/processed_dataset/proof/0536.json b/processed_dataset/proof/0536.json new file mode 100644 index 0000000000000000000000000000000000000000..f504db70a8f1f0c57dcbc67c0ca73aa9bb95071f --- /dev/null +++ b/processed_dataset/proof/0536.json @@ -0,0 +1,10 @@ +{ + "source_file": "./raw_volume-zh/volume13/chapter11.tex", + "problem_type": "proof", + "problem": "例7. $n$ 个人在某个节日期间互通电话问候,已知其中每个人至多打通了 3 个朋友家的电话; 任何 2 个人之间至多进行 1 次通话; 且任何 3 个人中至少有 2 人,其中一人打通了另一个人家里的电话, 求 $n$ 的最大值.", + "solution": "解:们需要如下的引理.\n引理: $n$ 阶简单图 $G$ 中不存在 $K_3$, 则 $\\| G|| \\leqslant\\left[\\frac{n^2}{4}\\right]$.\n引理的证明: 设 $A$ 是各顶点中度最大的顶点,设与 $A$ 相邻的点的集合为 $M=\\left\\{A_1\\right.$, $\\left.A_2, \\cdots, A_r\\right\\}$, 与 $A$ 不相邻的点的集合为 $N=\\left\\{B_1, B_2, \\cdots, B_s\\right\\}(r+s+1=n)$. 由于 $G$ 中无三角形, 从而 $G$ 在 $M$ 中没有边, 从而 $G$ 的其他边都在 $N$ 中或 $M 、 N$ 之间, 这样的边都是由顶点 $B_1, \\cdots, B_s$ 引出的 (如图()).\n于是, ||$G|| \\leqslant \\mathrm{d}(A)+\\mathrm{d}\\left(B_1\\right)+\\mathrm{d}\\left(B_2\\right)+\\cdots+\\mathrm{d}\\left(B_s\\right) \\leqslant r+r+\\cdots+ r=(s+1) r \\leqslant\\left(\\frac{s+r+1}{2}\\right)^2=\\frac{n^2}{4}$,\n又 $\\|G\\| \\in \\mathbf{Z}$, 所以 $\\|G\\| \\leqslant\\left[\\frac{n^2}{4}\\right]$.\n解答原题: 用 $n$ 个点表示 $n$ 个人, 如果一个人 $A$ 打通了另一个人 $B$ 家里的电话,则连一条从 $A$ 到 $B$ 的有向边, 得到一个简单的有向图 $G$.\n一方面, $\\bar{G}$ 中无三角形, 由引理有, || $\\bar{G}|| \\leqslant\\left[\\frac{n^2}{4}\\right]$, 所以 $\\| G||=\\mathrm{C}_n^2-$ || $\\bar{G}|| \\geqslant \\mathrm{C}_n^2-\\left[\\frac{n^2}{4}\\right]=\\left[\\frac{(n-1)^2}{4}\\right]$; 另一方面, ||$G||=\\sum_{i=1}^n \\mathrm{~d}^{+}\\left(x_i\\right) \\leqslant \\sum_{i=1}^n 3=3 n$, 所以\n$$\n\\left[\\frac{(n-1)^2}{4}\\right] \\leqslant 3 n . \\label{eq1}\n$$\n当 $n$ 为奇数时, 式\\ref{eq1} 变为 $\\frac{(n-1)^2}{4} \\leqslant 3 n$, 解得 $n \\leqslant 13$; 当 $n$ 为偶数时, 式\\ref{eq1} 变为 $\\frac{n^2-2 n}{4} \\leqslant 3 n$, 解得 $n \\leqslant 14$.\n综上所述, $n \\leqslant 14$.\n最后, $n=14$ 是可能的.\n构造两个 $K_7$, 对其中每个七边形 $A_1 A_2 \\cdots A_7$, 令 $A_i$ 指向 $A_{i+1}, A_{i+2}, A_{i+3}\\left(i=1,2, \\cdots, 7, A_{i+7}=A_i\\right)$, 则构图合乎条件.\n首先, 每个点都恰引出 3 条有向出边, 从而每个人至多打通了 3 个朋友家的电话;\n其次, 对任何 3 个点, 由抽庶原理, 必有两个点 $A_i 、 A_j(i3$, 则 $A_j$ 打通了 $A_i$ 家中的电话.", + "remark": "", + "figures": [ + "./images/volume13/figures/fig-c11i1.png" + ] +} \ No newline at end of file diff --git a/processed_dataset/proof/0537.json b/processed_dataset/proof/0537.json new file mode 100644 index 0000000000000000000000000000000000000000..f152f9f04c7f5b681cf692caba03698ceeeee923 --- /dev/null +++ b/processed_dataset/proof/0537.json @@ -0,0 +1,8 @@ +{ + "source_file": "./raw_volume-zh/volume13/chapter11.tex", + "problem_type": "proof", + "problem": "例9. 设 $|X|=56$, 对 $X$ 的任意 15 个子集, 只要它们中任何 7 个的并不少于 $n$ 个元素, 则这 15 个子集中一定存在其交非空的 3 个集合, 求 $n$ 的最小值.", + "solution": "解:$n_{\\min }=41$.\n首先证明 $n=41$ 合乎条件.\n用反证法: 假设存在 $X$ 的 15 个子集,它们中任何 7 个的并不少于 41 个元素, 而任何 3 个的交都为空集, 则每个元素至多属于 2 个子集, 不妨设每个元素恰属于 2 个子集 (否则在一些子集中添加一些元素, 上述条件仍然成立), 由抽屉原理, 必有一个子集, 设为 $A$, 至少含有 $\\left[\\frac{56 \\times 2}{15}\\right]+1=8$ 个元素, 又设其他 14 个子集为 $A_1$, $A_2, \\cdots, A_{14}$.\n考察不含 $A$ 的任何 7 个子集, 都对应 $X$ 中的 41 个元素, 所有不含 $A$ 的 7 -子集组一共至少对应 $41 \\mathrm{C}_{14}^7$ 个元素.\n另一方面, 对于元素 $a$, 若 $a \\notin A$, 则 $A_1, A_2, \\cdots, A_{14}$ 中有 2 个含有 $a$, 于是 $a$ 被计算 $\\left(\\mathrm{C}_{14}^7-\\mathrm{C}_{12}^7\\right)$ 次; 若 $a \\in A$, 则 $A_1, A_2, \\cdots, A_{14}$ 中有 1 个含有 $a$,于是 $a$ 被计算 $\\left(\\mathrm{C}_{14}^7-\\mathrm{C}_{13}^7\\right)$ 次, 于是, $41 \\mathrm{C}_{14}^7 \\leqslant(56-|A|)\\left(\\mathrm{C}_{14}^7-\\mathrm{C}_{12}^7\\right)+ |A| \\cdot\\left(\\mathrm{C}_{14}^7-\\mathrm{C}_{13}^7\\right)=56\\left(\\mathrm{C}_{14}^7-\\mathrm{C}_{12}^7\\right)-|A|\\left(\\mathrm{C}_{13}^7-\\mathrm{C}_{12}^7\\right) \\leqslant 56\\left(\\mathrm{C}_{14}^7-\\mathrm{C}_{12}^7\\right)- 8\\left(\\mathrm{C}_{13}^7-\\mathrm{C}_{12}^7\\right)$, 即 $48 \\mathrm{C}_{12}^7+8 \\mathrm{C}_{13}^7 \\leqslant 15 \\mathrm{C}_{14}^7$, 化简得, $3 \\times 48+4 \\times 13 \\leqslant 15 \\times 13$, 即 $196 \\leqslant 195$,矛盾.\n其次证明 $n \\geqslant 41$, 用反证法.\n假定 $n \\leqslant 40$, 设 $X=\\{1,2, \\cdots, 56\\}$, 令 $A_i=\\{x \\in X \\mid x \\equiv i(\\bmod 7)\\} (i=1,2, \\cdots, 7), B_j=\\{x \\in X \\mid x \\equiv j(\\bmod 8)\\}(j=1,2, \\cdots, 8)$, 显然, $\\left|A_i\\right|=8,\\left|A_i \\cap A_j\\right|=0(1 \\leqslant i3$ 时,我们希望 1 尽可能多.\n能否都为 1 ? 通过尝试, 发现至少有一个 -1 . 实际上, 考察紧靠外围一周 -1 的格, 其中至少有一个格填 -1 . 否则, 考察表中 $a 、 b 、 c 、 d$ 四格, 设其中最后一个填数的格是 $d$. 而 $a 、 b 、 c$ 三格都已填 1 ,则 $d$ 不论何时填数,都只能填 -1 ,矛盾.\n这样, $f(n) \\leqslant(n-2)^2-1$.\n其次, 由行积, $a$ 处可填 1 , 由列积, $b$ 处可填 1 , 再由行积, $e 、 c$ 处可填 1 , 又由列积, $f$ 处可填 1 , 这样, $d$ 处填 -1 . 对其他各格, 第二行和倒数第二行, 都由行积可填 1 , 其余的格都由列积可填 1 . 此时, 共有 $(n-2)^2-1$ 个 1 . 所以,\n$f(n)=(n-2)^2-1$.\n下面证明: $n>3$ 时, $g(n)=n-2$.\n若 $g(n)=r \\leqslant n-3$, 则表中 $r$ 个 1 至多占住 $n-3$ 行和 $n-3$ 列, 于是在中间的 $n-2$ 行和 $n-2$ 列中, 必有一行和一列都为 -1 , 但这是不可能的, 因为它们的交叉处无法填人 -1 , 矛盾.\n所以, $g(n) \\geqslant n-2$.\n另一方面, 将第 2 列 $n-2$ 个格都填 1 (按行填), 再对每一行, 从右至左按行可填 -1 , 此时表中共有 $n-2$ 个 1 . 故 $g(n)=n-2$.\n综上所述,\n$$\nf(n)= \\begin{cases}1 & (n=3) ; \\\\ (n-2)^2-1 & (n>3) .\\end{cases}\n$$\n$g(n)=n-2$.", + "remark": "", + "figures": [] +} \ No newline at end of file diff --git a/processed_dataset/proof/0539.json b/processed_dataset/proof/0539.json new file mode 100644 index 0000000000000000000000000000000000000000..2c8abb44279427685e70069e06666d00c7ce7dbb --- /dev/null +++ b/processed_dataset/proof/0539.json @@ -0,0 +1,8 @@ +{ + "source_file": "./raw_volume-zh/volume13/chapter12.tex", + "problem_type": "proof", + "problem": "例3. 彩票上依次排列着 50 个空格, 每个参加者都在彩票上填人 1 至 50 的整数 (每个数在同一张彩票中恰出现一次), 主持人亦填一张作底.\n如果某人所填的数列有一个位置与底票上对应位置上填的数相同, 则可中彩.\n试问: 一个参加者至少要填多少张彩票, 才能保证自己一定中彩?", + "solution": "分析:解若适当填 $k$ 张彩票可以中彩, 则称 $k$ 是中彩的.\n将所填的 $k$ 张彩票排成 $k \\times 50$ 的数表:\n\\begin{tabular}{|c|c|c|c|c|}\n\\hline$a_1$, & $a_2$ & $a_3$ & $\\cdots$, & $a_{50}$ \\\\\n\\hline$b_1$, & $b_2$ & $b_3$ & $\\cdots$, & $b_{50}$ \\\\\n\\hline$\\ldots$ & $\\ldots$ & $\\ldots$ & $\\ldots$ & $\\ldots$ \\\\\n\\hline$c_1$ & $c_2$ & $c_3$ & $\\cdots$, & $c_{50}$ \\\\\n\\hline$x_1$, & $x_2$ & $x_3$ & $\\cdots$, & $x_{50}$ \\\\\n\\hline\n\\end{tabular}\n所谓 $k$ 是中彩的,即不论 $x_1, x_2, x_3, \\cdots, x_{50}$ 如何填, 数表中至少有一个列, 此列中至少有一个数与此列中的 $x$ 相等.\n我们称这样的数为好数.\n这样, $k$ 是中彩的, 等价于存在 $k \\times 50$ 数表, 使表中至少有一个好数.\n显然, $k=50$ 是好的.\n实际上, $50 \\times 50$ 数表有 50 行、50 列, 每行是 1,2 , $3, \\cdots, 50$ 的一个排列, 表中共有 50 个 1 , 我们可以将 50 个 1 占住 50 列, 即每列一个 1 . 这样, 不论 $x_1, x_2, x_3, \\cdots, x_{50}$ 如何填, 其中的 1 必与表中的某个 1 同列.\n进一步, $k=49$ 是好的.\n可这样类似构造如下表:\n$\\begin{array}{ccccccc:c} & 1 & 2 & 3 & 4 & \\cdots & 49 & 50 \\\\ & 49 & 1 & 2 & 3 & \\cdots & 48 & 50 \\\\ & 48 & 49 & 1 & 2 & \\cdots & 47 & 50 \\\\ \\cdots & \\cdots & \\cdots & \\cdots & \\cdots & \\cdots & \\cdots \\\\ & 2 & 3 & 4 & 5 & \\cdots & 1 & 50\\end{array}$\n因为底票中的数码 $1,2, \\cdots, 49$ 不能都填在最后一个位置上, 即必有一个数码填在前面 49 个位置中, 它必与某一行的相应数码相同.\n由此我们发现, 可让每个数 $i$ 都占住每一列, 这样, 在此表的下方的任何一列填人 $1,2, \\cdots$, 50 中的任何一个数, 都必与该列中的一个数相等.\n由此可见, 只要构造如下的数表:\n$\\begin{array}{ccccccc}1, & 2, & 3, & 4, & \\cdots, & a-1, & a \\\\ 2, & 3, & 4, & 5, & \\cdots, & a, & 1 \\\\ 3, & 4, & 5, & 6, & \\ldots, & 1, & 2 \\\\ \\ldots & \\cdots & \\ldots & \\ldots & \\cdots, & \\cdots, & \\cdots \\\\ a, & 1, & 2, & 3, & \\cdots, & a-2, & a-1\\end{array}$\n而且底票上的数 $1,2, \\cdots, a$ 中至少有一个填人前 $a$ 列, 则前 $k$ 列中必有一个好数.\n要使底票上的数 $1,2, \\cdots, a$ 中至少有一个填入前 $a$. 列, 只须 $1,2, \\cdots$, $a$ 这 $a$ 个数不能都填在后 $50-a$ 列, 即 $50-a25$. 所以, $a \\geqslant 26$. 由此可见, $k=26$ 是中彩的.\n下面证明 $k=25$ 不是中彩的.\n实际上, 考察 $25 \\times 50$ 数表, 我们证明可以适当地填一张底票 $P: x_1, x_2, x_3, \\cdots, x_{50}$, 使表中没有一个好数.\n先填 $P$ 中的数 1 . 由于表中共有 25 个 1 , 有 50 列, 必有一个列中没有 1 , 在此列中填 1 即可.\n按此方法再依次填 $2,3, \\cdots, a-1$. 直至 $a$ 不能按上述方法填入.\n此时, 必有 $a \\geqslant$ 26. 否则, $a<26, P$ 中至多填了 $a-1 \\leqslant 24$ 个数, 占了 24 个 $P$ 中的格, 但表中只有 25 个 $a$, 占了 25 个格, 共占了 $24+25$ 个格, $P$ 中至少还有一个格可填 $a$, 矛盾.\n此外, 由于表中只有 25 个 $a$, 占了 25 个列, $P$ 中至少还有 25 个格可填 $a$. 但 $a$ 不能填人, 意味着这 25 个格被先填入的 $1,2,3, \\cdots, a-1$ 中的 25 个数 (记为 $x_1, x_2, x_3, \\cdots, x_{25}$ ) 占住.\n考察 $P$ 中任一个空格 $D$, 由于 $D$ 所在的列只有 25 个数, 于是 $D$ 中至多有 25 个数不能填人.\n这样, $a, x_1, x_2, x_3, \\cdots, x_{25}$ 中至少有一个数可填人 $D$ 中, 但 $a$ 不能填人 $D$, 所以 $x_1, x_2, x_3, \\cdots, x_{25}$ 中必有一个数 $x_i$ 可填人 $D$. 于是, 将 $x_i$ 填人 $D$, 并将 $a$ 填入 $x_i$ 原来所在的位置, 则底票上又新填人了一个数.\n如此下去, 可将底票的空格填满, 使其中任何一个数都不与它所在列中任何一个数相同.\n综上所述, $k$ 的最小值为 26 .", + "remark": "", + "figures": [] +} \ No newline at end of file diff --git a/processed_dataset/proof/0540.json b/processed_dataset/proof/0540.json new file mode 100644 index 0000000000000000000000000000000000000000..00259d332e85b015f64a5b8d0ac802db8d73bff1 --- /dev/null +++ b/processed_dataset/proof/0540.json @@ -0,0 +1,8 @@ +{ + "source_file": "./raw_volume-zh/volume13/chapter12.tex", + "problem_type": "proof", + "problem": "例5. 若自然数 $n$ 满足这样的条件: 存在由 $n$ 个实数组成的数列, 使得任何连续 17 个项之和为正, 而任何连续 10 个项之和为负, 求 $n$ 的最大值.", + "solution": "解:$n$ 的最大值为 25 .\n先证明 $n \\leqslant 25$, 用反证法.\n假设可以写出这样的 $n$ 个实数: $a_1, a_2, \\cdots, a_n$, 且 $n \\geqslant 26$, 则有如下性质:\n(1) 任意连续 7 个项之和为正.\n实际上, 考察连续 7 个项之和 $a_{i+1}+a_{i+2}+\\cdots+a_{i+7}(i=0,1,2, \\cdots, n- 7)$, 如果 $i \\geqslant 10$, 则因为 $\\left(a_{i-9}+a_{i-8}+\\cdots+a_i\\right)+\\left(a_{i+1}+a_{i+2}+\\cdots+a_{i+7}\\right)>0$, 而 $a_{i-9}+a_{i-8}+\\cdots+a_i<0$, 所以 $a_{i+1}+a_{i+2}+\\cdots+a_{i+7}>0$. 若 $i \\leqslant 9$, 则 $i+ 17 \\leqslant 9+17=26 \\leqslant n$. 因为 $\\left(a_{i+1}+a_{i+2}+\\cdots+a_{i+7}\\right)+\\left(a_{i+8}+a_{i+9}+\\cdots+a_{i+17}\\right)>$ 0 , 而 $a_{i+8}+a_{i+9}+\\cdots+a_{i+17}<0$, 所以 $a_{i+1}+a_{i+2}+\\cdots+a_{i+7}>0$.\n(2) 任意连续 3 个项之和为负.\n实际上, 考察连续 3 个项之和 $a_{i+1}+a_{i+2}+a_{i+3}(i=0,1,2, \\cdots, n-3)$, 如果 $i \\geqslant 7$, 则因为 $\\left(a_{i-6}+a_{i-5}+\\cdots+a_i\\right)+\\left(a_{i+1}+a_{i+2}+a_{i+3}\\right)<0$, 而 $a_{i-6}+ a_{i-5}+\\cdots+a_i>0$, 所以 $a_{i+1}+a_{i+2}+a_{i+3}<0$. 若 $i \\leqslant 6$, 则 $i+10 \\leqslant 6+10= 16 \\leqslant n$. 因为 $\\left(a_{i+1}+a_{i+2}+a_{i+3}\\right)+\\left(a_{i+4}+a_{i+5}+\\cdots+a_{i+10}\\right)<0$, 而 $a_{i+4}+ a_{i+5}+\\cdots+a_{i+10}>0$, 所以 $a_{i+1}+a_{i+2}+a_{i+3}<0$.\n(3) 任意连续 4 个项之和为正.\n实际上,考察连续 4 个项之和 $a_{i+1}+a_{i+2}+a_{i+3}+a_{i+4}(i=0,1,2, \\cdots$, $n-4)$, 如果 $i \\geqslant 3$, 则因为 $\\left(a_{i-2}+a_{i-1}+a_i\\right)+\\left(a_{i+1}+a_{i+2}+a_{i+3}+a_{i+4}\\right)>0$, 而 $a_{i-2}+a_{i-1}+a_i<0$, 所以 $a_{i+1}+a_{i+2}+a_{i+3}+a_{i+4}>0$. 若 $i \\leqslant 2$, 则 $i+7 \\leqslant 2+7=9 \\leqslant n$. 因为 $\\left(a_{i+1}+a_{i+2}+a_{i+3}+a_{i+4}\\right)+\\left(a_{i+5}+a_{i+6}+a_{i+7}\\right)>0$, 而 $a_{i+5}+a_{i+6}+a_{i+7}<0$, 所以 $a_{i+1}+a_{i+2}+a_{i+3}+a_{i+4}>0$.\n(4)每个项都为正.\n实际上, 考察任意一个项 $a_i(i=1,2, \\cdots, n)$, 如果 $i \\geqslant 4$, 则因为 $\\left(a_{i-3}+\\right. \\left.a_{i-2}+a_{i-1}\\right)+a_i>0$, 而 $a_{i-3}+a_{i-2}+a_{i-1}<0$, 所以 $a_i>0$. 若 $i \\leqslant 3$, 则 $i+ 3 \\leqslant 3+3=6 \\leqslant n$. 因为 $a_i+\\left(a_{i+1}+a_{i+2}+a_{i+3}\\right)>0$, 而 $a_{i+1}+a_{i+2}+a_{i+3}<$ 0 , 所以 $a_i>0$.\n显然 (4)与\"任何连续 10 个项之和为负\"矛盾, 所以 $n \\leqslant 25$.\n此外, $n=25$ 时, 我们可以构造出这样的 25 个实数.\n设 25 个合乎条件的实数为 $a_1, a_2, \\cdots, a_{25}$, 我们先研究它应满足的若干必要条件.\n从前面的论证可知, 对 $i=0,1,2, \\cdots, 25-7$, 除 $i=9$ 外,任意连续 7 个项之和 $a_{i+1}+a_{i+2}+\\cdots+a_{i+7}$ 为正, 于是除 $i=6 、 16$ 外, 任意连续 3 个项之和 $a_{i+1}+a_{i+2}+a_{i+3}$ 为负, 为了使构造简单, 我们可综合考虑对称构造、周期构造和待定参数构造技巧, 设 25 个实数为\n$$\na, a, b, a, a, b, c, c, c, b, a, a, d, a, a, b, c, c, c, b, a, a, b, a, a,\n$$\n其中 $a 、 b 、 c 、 d$ 为待定参数,满足 $a+a+b<0$, 取 $a=1, b=-3$ 实验,则数列变为\n$$\n1,1,-3,1,1,-3, c, c, c,-3,1,1, d, 1,1,-3, c, c, c,-3,\n$$\n$1,1,-3,1,1$.\n为了使任何连续 10 个项之和为负, 要求 $d+(1+1-3)+3 c+(-3+ 1+1)<0$, 即 $d+3 c<2$;\n为了使任何连续 17 个项之和为正, 要求 $(1-3)+3 c+(-3+1+1)+d+ (1+1-3)+3 c+(-3+1)>0$, 即 $d+6 c>6$. 所以 $6-6 c\\frac{4}{3}$, 取 $c=1.5$, 则 $-30$, 取 $a=8, b=-19$ 即可.\n于是, 得到的另一个合乎条件的数列为:\n$$\n8,8,-19,8,8,-19,8,8,8,-19,8,8,-19,8,8,-19,8,8 \\text {, }\n$$\n$8,-19,8,8,-19,8,8$.", + "figures": [] +} \ No newline at end of file diff --git a/processed_dataset/proof/0541.json b/processed_dataset/proof/0541.json new file mode 100644 index 0000000000000000000000000000000000000000..84cc94f2edcdeafccccf2ba2b625b9b60c31134a --- /dev/null +++ b/processed_dataset/proof/0541.json @@ -0,0 +1,8 @@ +{ + "source_file": "./raw_volume-zh/volume13/chapter13.tex", + "problem_type": "proof", + "problem": "例1. 某市有 $n$ 所中学, 第 $i$ 所中学派出 $c_i$ 名学生 $\\left(1 \\leqslant c_i \\leqslant 39\\right)$ 到体育馆看球赛, 其中 $\\sum_{i=1}^n c_i=1990$. 看台上每排有 199 个座位, 要求同一学校的学生坐在同一排.\n问: 最少要安排多少个排, 才能使所有学生一定能够坐下?", + "solution": "分析:解先考虑最坏的情况是什么.\n所谓情况最坏, 是指每个横排空下来的位置最多.\n显然, 如果各校的人数有多有少, 是比较好安排的, 因为剩下的空位可以让人数少的学校的学生坐.\n于是, 较坏的情况是每所学校派出的人数较多而又比较\"整齐\". 于是, 可先设想所有学校派出的人数相等.\n假定每所学校都派出 $r$ 人, 我们考察 $r$ 为何值时, 会使横排空余的位置最多.\n列表估计如下:\n\\begin{tabular}{|c|c|c|c|c|c|c|c|c|c|c|c|c|}\n\\hline 各学校所派人数 $r$ & 39 & 38 & 37 & 36 & 35 & 34 & 33 & 32 & 31 & 30 & 29 & $r \\leqslant 28$ \\\\\n\\hline 每排可安排学校个数 & 5 & 5 & 5 & 5 & 5 & 5 & 6 & 6 & 6 & 6 & 6 & $\\cdots$ \\\\\n\\hline 每排空余位子数 $t$ & 4 & 9 & 14 & 19 & 24 & 29 & 1 & 7 & 13 & 19 & 25 & $t \\leqslant 28$ \\\\\n\\hline\n\\end{tabular}\n由上表可知, 当每所学校派出的人数都为 34 时, 每个横排的空位最多, 为 29.\n注意到 $1990=34 \\times 58+18$. 于是, 先考察有 58 所学校各派 34 人, 另一所学校派 18 人的情形, 看需要多少个横排.\n此时, 每个横排最多坐 6 所学校, 而且坐 6 所学校的排只能有一排, 即坐派了 18 人的学校的那一排.\n于是, 所需要的排数至少是 $1+\\left[\\frac{58-6}{5}\\right]+1=12$.\n最后, 我们证明,对任何情形, 12 排是足够的.\n最自然的一个排法如下:\n先排第一排,使第一排的空位数 $x_1$ 最小.\n再排第二排: 又使第二排的空位数 $x_2$ 最小.\n显然 $x_1 \\leqslant x_2$. 否则, $x_1>x_2$, 将第 2 排的学生坐到第 1 排, 可使第 1 排的空位数减少, 这与 $x_1$ 的最小性矛盾.\n如此下去, 直至排第 11 排,使第 11 排的空位数 $x_{11}$ 最小.\n我们证明, 按这样的排法, 必定可将剩下的所有学生都排在第 12 排.\n实际上, 设排完 12 排以后还有 $x$ 人没有排下, 则 $x>x_{12}$. 否则, 将此 $x$ 人坐在第 12 排即可.\n所以, $x=1990-\\sum_{i=1}^{12}\\left(199-x_i\\right) \\geqslant x_{12}$, 由此得 $x_1+x_2+\\cdots+ x_{11}>398$, 所以, $398) ). 此时,第一、第二列不能再有棋.\n比如第一列还有一只棋则棋 $a$ 可去.\n第二列有一只棋, 则棋 $b$ 可去.\n这样剩下的 $2 n-3$ 只都在后 $n-2$ 列, 由抽庶原理,必有某个列有两只棋.\n不妨设第 3 列的第 $i$ 格和第 $j$ 格各有一只棋 $c 、 d$, 其中 $i 、 j$ 中至少有一个不为 1 . 不妨设 $i \\neq 1$, 则 $c$ 所在的行不能再有棋, 否则棋 $c$ 可去.\n这样, $a$ 所在的列只有一只棋, $c$ 所在的行只有一只棋.\n去掉这行和这列,对剩下的棋盘使用归纳假设,命题 (*) 获证.\n最后, 如图() 可知, $r=2 n-2$ 是可能的.\n故 $r_n=2 n-2$.\n证法 2: 为了找到可去棋, 先考虑棋在什么条件下可去.\n如果 $A$ 是可去棋,则 $A$ 所在的行至少 2 只棋, $A$ 所在的列也至少 2 只棋.\n于是, 可取定至少有 2 只棋的行, 再从中找有 2 只棋的列.\n设第 $i$ 行的棋子数为 $a_i(i=1,2, \\cdots, n)$, 不妨设 $a_1 \\leqslant a_2 \\leqslant \\cdots \\leqslant a_n$, 则 $a_1+a_2+\\cdots+a_n=2 n-1$. 因为每行至少一只棋, 所以 $a_1=1$.\n假定 $a_1=a_2=\\cdots=a_i=1(1 \\leqslant i \\leqslant n-1), 2 \\leqslant a_{i+1} \\leqslant a_{i+2} \\leqslant \\cdots \\leqslant a_n$, 则\n$$\n\\begin{aligned}\na_{i+1}+a_{i+2}+\\cdots+a_n & =2 n-1-\\left(a_1+a_2+\\cdots+a_i\\right) \\\\\n& =2 n-1-i \\\\\n& =n+(n-1-i) \\geqslant n .\n\\end{aligned}\n$$\n前 $i$ 行的 $i$ 只棋最多占住 $i$ 个列(如图() 的阴影方格所示), 不妨设这 $i$ 只棋都在前 $k(k \\leqslant i)$ 列中.\n如果后 $n-i$ 行中有一只棋 $A$ 在前 $k$ 列中,由于 $A$ 所在的行至少有 2 只棋, 所以 $A$ 可去.\n如果后 $n-i$ 行的棋都在后 $n-k$ 列中, 但 $n-k) 所示的放法可以猜想: $r_2=2(2 n-1)$, 即 $3 \\times(2 n-1)$ 棋盘最多可以放 $4 n-2$ 只棋.\n由于 $3 \\times(2 n-1)$ 棋盘要去掉两行才能化为 $1 \\times(2 n-1)$ 棋盘.\n那么去掉的两行中至多有多少只棋呢? 为此, 我们要研究一下 $2 \\times(2 n-1)$ 棋盘, 而这是我们已经跳过了的情形.\n这种情形对于研究 $3 \\times(2 n-1)$ 棋盘也许有帮助.\n因此,我们回头看看 $2 \\times(2 n-1)$ 棋盘.\n如图(), 在 $2 \\times(2 n-1)$ 棋盘中, 有 $r \\leqslant 2 n$, 且等号只能以如图() 的方式唯一实现.\n实际上,第一列至多有两只棋, 而后 $2 n-2$ 列可以划分为 $n-1$ 个 $2 \\times 2$ 棋盘, 每个 $2 \\times 2$ 棋盘至多可以放 2 只棋.\n所以, $r \\leqslant 2+2(n-1)=2 n$. 若 $r=2 n$, 则第一列必有两只棋, 且每个 $2 \\times 2$ 棋盘中都恰有 2 只棋.\n于是, 第一列有两只棋, 则第二列中无棋, 于是第三列中有两只棋.\n如此下去, 所有奇数列中都有两只棋, 而所有偶数列中都没有棋.\n即等号以唯一的方式出现.\n现在考虑 $3 \\times(2 n-1)$ 棋盘.\n我们要证明 $r \\leqslant 2(2 n-1)$. 很自然地, 应将 $3 \\times (2 n-1)$ 棋盘化为一个 $2 \\times(2 n-1)$ 棋盘(简称为 $A$ 盘) 和一个 $1 \\times(2 n-1)$ 棋盘 (简称为 $B$ 盘) 处理.\n由前面的讨论可知, $r_A \\leqslant 2 n, r_B \\leqslant 2 n-1$. 所以\n$$\nr=r_A+r_B \\leqslant 4 n-1 . \\label{eq1}\n$$\n若 式\\ref{eq1} 成立等号, 则 $r_A=2 n, r_B=2 n-1$. 此时, 棋盘中棋的放置如图() 所示,但其中有一个 $2 \\times 2$ 正方形放了 3 只棋.\n矛盾.\n所以, $r \\leqslant 4 n-2$.\n以上证明存在一个漏洞: 要使棋盘中有 $2 \\times 2$ 正方形, 必须 $n>1$, 即棋盘中至少要有两列.\n因此,要优化假设: $m \\leqslant n$. 此时, 必有 $n \\geqslant m>1$.\n一般地, 对 $(2 m-1) \\times(2 n-1)$ 棋盘, 若 $m \\leqslant n$, 我们证明: $r \\leqslant m(2 n-1)$. \n对 $m$ 归纳.\n当 $m=1$ 时, 结论显然成立, 设结论对于小于 $m$ 的自然数成立, 考察 $(2 m-1) \\times(2 n-1)$ 棋盘, 我们将之划分为一个 $A$ 盘: $2 \\times(2 n-1)$ 棋盘和一个 $B$ 盘: $(2 m-3) \\times(2 n-1)$ 棋盘.\n由前面的讨论和归纳假设可知 $r_A \\leqslant 2 n, r_B \\leqslant(m-1)(2 n-1)$, 所以\n$$\nr=r_A+r_B \\leqslant 2 n+(m-1)(2 n-1)=m(2 n-1)+1 . \\label{eq2}\n$$\n若 \\ref{eq2} 式成立等号,则 $r_A=2 n, r_B=(m-1)(2 n-1)$. 由 $r_A=2 n$ 知,整个 $(2 m-1) \\times(2 n-1)$ 棋盘的第一行中恰有 $n$ 只棋, 于是, 将后 $2 m-2$ 行划分为\n$m-1$ 个 $2 \\times(2 n-1)$ 棋盘, 每个 $2 \\times(2 n-1)$ 棋盘中不多于 $2 n$ 只棋, 于是, 棋盘中的棋子的个数 $r \\leqslant n+2 n(m-1)=2 m n-n \\leqslant 2 m n-m=m(2 n-1)$, 与 $r=m \\cdot(2 n-1)+1$ 矛盾.\n所以, \\ref{eq2} 式不成立等号.\n即 $r \\leqslant m(2 n-1)$. 命题获证.\n最后, 将棋盘的奇数行的每个格都放一只棋, 有 $r=m(2 n-1)$, 所以 $r_m=m(2 n-1)$. 特别地, 令 $m=10 、 n=45$, 有 $19 \\times 89$ 棋盘中至多可放 890 只棋.", + "remark": "", + "figures": [ + "./images/volume13/figures/fig-c13i4.png", + "./images/volume13/figures/fig-c13i5.png", + "./images/volume13/figures/fig-c13i5.png", + "./images/volume13/figures/fig-c13i6.png" + ] +} \ No newline at end of file diff --git a/processed_dataset/proof/0544.json b/processed_dataset/proof/0544.json new file mode 100644 index 0000000000000000000000000000000000000000..7585f40c4643fabc35939bd19d5e1958b0b0ea0e --- /dev/null +++ b/processed_dataset/proof/0544.json @@ -0,0 +1,11 @@ +{ + "source_file": "./raw_volume-zh/volume13/chapter13.tex", + "problem_type": "proof", + "problem": "例4. 一个 $9 \\times 9$ 的棋盘的方格被染成黑白两种颜色, 使得与每个白格相邻的格中黑格的数目多于白格的数目, 与每个黑格相邻的格中白格的数目多于黑格的数目 (至少有一条公共边的两格称为相邻). 求所有这样的染色方式中,黑、白格数目之差的最大值.", + "solution": "分析:解要使染色满足条件, 则每个方格至多有一个邻格的颜色与其相同, 因而不能出现如下一些特殊情形: (1) $3-\\mathrm{L}$ 型的 3 个方格同色.\n(2) $1 \\times 3$ 的矩形的 3 个方格同色.\n如果棋盘中任何 2 个相邻方格的颜色不同, 则黑、白格数目之差不大于 1 .\n如果棋盘中存在 2 个相邻方格 $A 、 B$ 的颜色相同, 不妨设 $A 、 B$ 在同一行 (如图() 所示). 考察与这行相邻的行, 由于棋盘中没有同色的 3-L 型, 所以此行中与 $A 、 B$ 相邻的两个方格与 $A 、 B$ 异色.\n如此下去可知, $A 、 B$ 所在的两列中, 同行的两个方格都同色, 同列的方格颜色黑白相间 (相邻两格异色).下面证明,棋盘的任何一列中都没有两个相邻的格同色.\n否则, 设 $P 、 Q$ 是某列中相邻的方格, $P 、 Q$ 同色, 同上可证, $P 、 Q$ 所在的两行中, 同列的两个方格都同色、同行的方格颜色黑白相间.\n此时, 考察 $A 、 B$ 两列与 $P 、 Q$ 两行交叉的 4 个格, 由于同行同色且同列同色,所以 4 个格同色,矛盾.\n所以整个棋盘的每一列的方格的颜色都是黑白相间.\n去掉第一行, 则剩下的棋盘的每一列中黑、白格数目相等, 从而剩下的棋盘中所有方格黑、白格数目相等.\n而在第一行中, 由于没有同色的 $1 \\times 3$ 矩形, 所以每 3 个格中黑白格数目之差不大于 1 , 于是第一行中黑白格数目之差不大于 3 .\n所以,对任何合乎条件的染色,棋盘中黑、白格数目之差不大于 3 . 又如图() 的染色符合要求, 此时黑白格个数之差为 3 .\n综上所述,所求的最大值为 3 .", + "remark": "", + "figures": [ + "./images/volume13/figures/fig-c13i7.png", + "./images/volume13/figures/fig-c13i8.png" + ] +} \ No newline at end of file diff --git a/processed_dataset/proof/0545.json b/processed_dataset/proof/0545.json new file mode 100644 index 0000000000000000000000000000000000000000..abd7a494505e00af6461037e9f7360264ce99e4c --- /dev/null +++ b/processed_dataset/proof/0545.json @@ -0,0 +1,8 @@ +{ + "source_file": "./raw_volume-zh/volume13/chapter13.tex", + "problem_type": "proof", + "problem": "例5. 设正整数 $n \\geqslant 3, a_1, a_2, \\cdots, a_n$ 是任意 $n$ 个互异的实数, 其和为正数.\n如果它们的一个排列 $b_1, b_2, \\cdots, b_n$ 满足:对任意的 $k=1,2, \\cdots, n$, 均有 $b_1+b_2+\\cdots+b_k>0$, 则称这个排列是好的.\n求好的排列个数的最小值.", + "solution": "分析:解考察最坏情形: 对 $k=1,2, \\cdots, n, b_1+b_2+\\cdots+b_k>0$ 都很难满足, 这只需 $a_1, a_2, \\cdots, a_n$ 中的负数尽可能多.\n取 $a_2, \\cdots, a_n$ 均为负数, 而 $a_1=-a_2-\\cdots-a_n+1$. 此时任何一个好的排列 $\\left(b_1, b_2, \\cdots, b_n\\right)$, 均有 $b_1= a_1$, 而 $b_2, \\cdots, b_n$ 可以是 $a_2, \\cdots, a_n$ 任意排列, 故此时有 $(n-1)$ ! 个好的排列.\n下面证明至少有.\n$(n-1)$ ! 个好的排列.\n注意到 $(n-1)$ ! 是将 $a_1, a_2, \\cdots, a_n$ 排在圆周上的不同圆排列的个数, 我们先证明每一个圆排列对应一个好排列.\n为方便, 称好排列的首项为好数.\n我们只需证明每个圆排列中必存在一个好数.\n方法 1: 对 $n$ 归纳.\n当 $n=1$ 时,结论显然成立.\n假设对一切 $n0$, 所以至少有一个正数.\n将每一个正数和按逆时钟顺序在它之后的下一个正数之间的所有数编为一组, 每组至少有一个数, 且至少有一组有至少两个数 (由于不是所有数都为正), 故至多有 $k-1$ 组.\n对每组数求和, 得到少于 $k$ 个和.\n将这些和按它们所在组的顺序写在圆周上, 由于这些和的总和为正, 由归纳假设知, 这些和中存在一个和为好数.\n考虑这个和所在的组中的那个正数, 则这个数是整个圆排列中的好数.\n由归纳原理,结论成立.\n方法 2: 利用极端性原理.\n对任何一个圆排列 $\\left(b_1, b_2, \\cdots, b_n\\right)$, 考察所有以 $b_i$ 为首项的部分和: $b_i+b_{i+1}+\\cdots+b_{i+t}$, 其中大于 $n$ 的下标取模 $n$ 的余数.\n对所有 $i=1,2, \\cdots, n$ 和所有 $t=0,1,2, \\cdots, n-1$, 必存在一个最小的部分和 $b_i+b_{i+1}+\\cdots+b_{i+t}$. 因为至少存在一个非正数, 所以 $b_i+b_{i+1}+\\cdots+ b_{i+t} \\leqslant 0$. 在所有这样的最小和中又设项数 $t+1$ 最大的一个为 $b_i+b_{i+1}+\\cdots+ b_{i+t}$, 我们证明 $b_{i+t+1}$ 是好数.\n实际上, 若存在正整数 $k$, 使 $b_{i+t+1}+b_{i+t+2}+\\cdots+b_{i+t+k} \\leqslant 0$, 则 $\\left(b_i+\\right. \\left.b_{i+1}+\\cdots+b_{i+t}\\right)+\\left(b_{i+t+1}+b_{i+t+2}+\\cdots+b_{i+t+k}\\right) \\leqslant b_i+b_{i+1}+\\cdots+b_{i+\\iota}$, 这与和 $b_i+b_{i+1}+\\cdots+b_{i+t}$ 最小且项数最多矛盾.\n由于共有 $(n-1)$ ! 个圆排列, 而每个圆排列至少对应一个好排列, 且不同的圆排列对应的好排列是不同的,故至少有 $(n-1)$ ! 个好的排列.\n综上所述, 所求的最小值为 $(n-1) !$.", + "remark": "", + "figures": [] +} \ No newline at end of file diff --git a/processed_dataset/proof/0546.json b/processed_dataset/proof/0546.json new file mode 100644 index 0000000000000000000000000000000000000000..204db2f6ca064b14fb48df3b1f5a2853aa39ce72 --- /dev/null +++ b/processed_dataset/proof/0546.json @@ -0,0 +1,8 @@ +{ + "source_file": "./raw_volume-zh/volume13/chapter13.tex", + "problem_type": "proof", + "problem": "例6. 岛上住着 $n$ 个本地人, 他们中每两个人要么是朋友, 要么是敌人.\n一天, 首领要求每位居民 (包括首领自己) 按以下原则自己做一条石头项链: 每两个朋友间, 他们的项链上至少有一块石头相同; 每两个敌人间, 他们的项链上没有相同的石头 (一条项链上可以无石头). 求证: 要完成首领的命令, 需要 $\\left[\\frac{n^2}{4}\\right]$ 种不同的石头; 而石头种数少于 $\\left[\\frac{n^2}{4}\\right]$ 时,此命令可能无法实现.", + "solution": "分析:解当 $n=1$ 时,结论显然成立.\n设 $n>1$, 记需要的不同石头种数的最小值为 $S_n$. 当 $n=2$ 时, 设两个人为 $A 、 B$, 如果 $A 、 B$ 是敌人, 则 $S_2=0$. 如果 $A 、 B$ 是朋友, 则 $S_2=1$. 结论成立.\n当 $n=3$ 时, 设三个人为 $A 、 B 、 C$, 如果 $A 、 B 、 C$ 两两是敌人, 则 $S_3=0$. 如果 $A 、 B 、 C$ 两两是朋友, 则 $S_3=1$. 如果 $A 、 B 、 C$ 中有一个二人组是朋友, 另两个二人组是敌人, 则两个为朋友的二人组需要 1 种石头, 此时 $S_3=1$. 如果 $A 、 B 、 C$ 中有一个二人组是敌人, 另两个二人组是朋友, 则两个为朋友的二人组需要 2 种不同的石头.\n否则, 3 人拥有同一种石头,但其中有两个人是敌人,矛盾, 此时 $S_3=2$.\n由前面的一些特例, 我们发现一个有用的规律: 如果 3 个人中有一个二人组是敌人, 另两个二人组是朋友, 则两个为朋友的二人组需要 2 种不同的石头.\n再考虑 $n=4$ 的情形, 设四个人为 $A 、 B 、 C 、 D$, 如果为朋友的二人组不多于 4 , 则 $S_4 \\leqslant 4$. 如果为朋友的二人组为 5 , 另一个二人组为敌人, 不妨设 $A 、 B$ 为敌人,则 $A C D$ 是朋友三角形,设他们拥有同种的石头 $1 . B C D$ 是朋友三角形, 设他们拥有同种的石头 2. 此时 $A 、 B 、 C 、 D$ 的项链分别为 $\\{1\\},\\{2\\},\\{1,2\\}$, $\\{1,2\\}$, 合乎条件.\n此时 $S_4=2$. 如果 $A 、 B 、 C 、 D$ 两两都是朋友, 则 $S_4 \\stackrel{?}{=} 1$.\n现在考察何时有 $S_4=4$. 此时, 显然有 4 个为朋友的二人组, 另两个二人组为敌人.\n如果为敌人的两个二人组有公共的人, 不妨设 $A 、 B$ 为敌人且 $A 、 C$ 为敌人.\n因为 $B C D$ 是朋友三角形, 设他们拥有共同的石头 1 , 再注意到 $A 、 D$ 是朋友, 设他们拥有共同的石头 2. 此时 $A 、 B 、 C 、 D$ 的项链分别为 $\\{2\\},\\{1\\}$, $\\{1\\},\\{1,2\\}$, 合乎条件, 此时 $S_4=2$. 如果为敌人的两个二人组没有公共的人, 不妨设 $A 、 B$ 为敌人且 $C 、 D$ 为敌人.\n此时, $A 、 B 、 C 、 D$ 被分为 2 组, 每组 2 人,\n同一组的 2 个人是敌人, 而任何不同组的 2 个人都是朋友.\n此时, 每个为朋友的二人组对应一种石头, 我们证明: 4 个为朋友的二人组对应的石头互不相同.\n实际上, 如果某 2 个为朋友的二人组对应相同的石头, 而这 2 个为朋友的二人组至少包含 3 个不同的人, 他们拥有公共的石头.\n但将 3 人归人前述的两组,必有 2 人在同一组,他们应该是敌人,矛盾.\n于是 $S_4=4$.\n有上述一些特例不难发现一般情况下的构造方法.\n当 $n$ 为奇数时, 设 $n=2 k+1$, 将 $n$ 个人分成两组,一组 $k$ 人, 另一组 $k+1$ 人, 令同一组的任何 2 个人都是敌人, 而任何不同组的任何 2 个人都是朋友.\n此时, 共有 $k(k+1)$ 个为朋友的二人组, 每个为朋友的二人组对应一块石头, 我们证明: $k(k+1)$ 个为朋友的二人组对应的石头互不相同.\n实际上, 如果某 2 个为朋友的二人组对应相同的石头, 而这 2 个为朋友的二人组至少包含 3 个不同的人, 他们拥有公共的石头.\n但将 3 人归人前述的两组, 必有 2 人在同一组, 他们应该是敌人,矛盾.\n于是, 此时至少需要 $k(k+1)=\\left[\\frac{n^2}{4}\\right]$ 块石头.\n当 $n$ 为偶数时, 设 $n=2 k$, 类似地, 将 $n$ 个人分成两组, 每组 $k$ 人, 则至少需要 $k^2=\\left[\\frac{n^2}{4}\\right]$ 块石头.\n下面证明, $S=\\left[\\frac{n^2}{4}\\right]$ 时, 可按要求构造项链.\n对 $n$ 归纳.\n假定 $n=k$ 时结论成立, 考虑 $n=k+1$ 的情形, 我们来分析增量 $\\Delta=S_{k+1}-S_k=\\left[\\frac{(k+1)^2}{4}\\right]-\\left[\\frac{k^2}{4}\\right]$. 为了便于计算 $\\Delta$, 应讨论 $k$ 的奇偶情况.\n当 $k$ 为奇数时, 设 $k=2 r+1$, 此时, $\\Delta=\\left[\\frac{(k+1)^2}{4}\\right]-\\left[\\frac{k^2}{4}\\right]=(r+ 1)^2-\\left(r^2+r\\right)=r+1=\\frac{k+1}{2}$. 当 $k$ 为偶数时, 设 $k=2 r$, 此时, $\\Delta= \\left[\\frac{(k+1)^2}{4}\\right]-\\left[\\frac{k^2}{4}\\right]=\\left(r^2+r\\right)-r^2=r=\\frac{k}{2}$. 由此可见, 由 $k$ 到 $k+1$, 增加的石头种数为 $\\frac{k+1}{2}$ (当 $k$ 为奇数时) 或 $\\frac{k}{2}$ (当 $k$ 为偶数时). 由此想到将原来 $k$ 个人分成 $\\frac{k+1}{2}$ (当 $k$ 为奇数时) 或 $\\frac{k}{2}$ (当 $k$ 为偶数时) 组,每组不多于 2 人 ( $k$ 为奇数时恰有一组为 1 人, 其余各组都是 2 人, 而 $k$ 为偶数时, 每组都是 2 人). 希望新增加 1 人 $P$ 后, $P$ 与每一组至多需要一块新的石头.\n这一要求能否实现? 如果某组中的 2 人与 $P$ 都是敌人, 则无需增加新石头.\n如果该组中的 2 人与 $P$ 一是朋友一是敌人, 则将为朋友的 2 人各增加一块相同新石头即可.\n但如果该组中的 2 人之间是敌人, 而他们与 $P$ 都是朋友呢, 此时每人需要增加一块不同的新石头, 需要 2 块新石头.\n由此可见, 与 $P$ 都是朋友且互为敌人的 2 人不能在同一组, 但这样的分组也未必能实现, 因为 $P$ 的敌人个数也许比朋友个数多.\n现在, 换一个角度思考, 如果固定某两个为朋友的人 $A 、 B$ (假定这两人是新增加的), 则原来的每个人与 $A 、 B$ 之间至多增加一块石头, 因此采用 $k$ 到 $k+2$ 的归纳方式即可完成证明.\n假定 $k$ 个人时结论成立, 考虑 $k+2$ 个人的情形.\n如果 $k+2$ 人中没有朋友, 则结论显然成立 (无需石头). 此外, 设 $A 、 B$ 是朋友, 则由归纳假设, 另 $k$ 个人之间至多需要 $\\left[\\frac{k^2}{4}\\right]$ 种石头.\n考察这 $k$ 个人中任意一个人 $P, P$ 与 $A 、 B$ 构成一个 3 人组, 我们证明此 3 人组只需增加一种新石头.\n实际上, 如果 $A 、 B$ 与 $P$ 都是敌人, 则无需增加新石头.\n如果 $A 、 B$ 与 $P$ 一是朋友一是敌人, 则将为朋友的 2 人各增加一块新石头即可.\n如果 $A 、 B$ 与 $P$ 都是朋友, 则每人增加一块新石头即可.\n由 $P$ 的任意性可知, $k$ 个人至多增加 $k$ 种新石头.\n又 $A 、 B$ 之间至多需要一种新石头, 所以 $k+2$ 人至多需要 $\\left[\\frac{k^2}{4}\\right]+ k+1=\\left[\\frac{(k+2)^2}{4}\\right]$ 种石头.\n综上所述, 命题获证.", + "remark": "", + "figures": [] +} \ No newline at end of file diff --git a/processed_dataset/proof/0547.json b/processed_dataset/proof/0547.json new file mode 100644 index 0000000000000000000000000000000000000000..1c90293ea73dced7da55229c12c639973bfa4b25 --- /dev/null +++ b/processed_dataset/proof/0547.json @@ -0,0 +1,8 @@ +{ + "source_file": "./raw_volume-zh/volume13/chapter13.tex", + "problem_type": "proof", + "problem": "例7. 设 $m 、 n$ 为正整数, $m<2001, n<2002$. 有 $2001 \\times 2002$ 个不同的实数, 将这些数填人 $2001 \\times 2002$ 棋盘的方格, 使得每个方格内恰有一个数.\n如果某个方格内的数小于其所在列的至少 $m$ 个数, 也小于其所在行的至少 $n$ 个数, 则将此方格称为 \"坏格\". 对所有填数方法, 求坏格个数 $S$ 的最小值.", + "solution": "分析:解考察一种特殊情形: 将 $1,2,3, \\cdots, 2001 \\times 2002$ 按自然顺序填人 $2001 \\times 2002$ 的棋盘的方格 (如下表), 此时坏格个数 $S=(2001-$ m) $(2002-n)$.\n\\begin{tabular}{|c|c|c|c|}\n\\hline 1 & 2 & $\\cdots$ & 2002 \\\\\n\\hline 2003 & 2004 & $\\cdots$ & 4004 \\\\\n\\hline$\\cdots$ & $\\cdots$ & $\\cdots$ & $\\cdots$ \\\\\n\\hline $2000 \\times 2002+1$ & $2000 \\times 2002+2$ & $\\cdots$ & $2001 \\times 2002$ \\\\\n\\hline\n\\end{tabular}\n我们猜想, $S$ 的最小值为 $(2001-m)(2002-n)$. 一般地, 对 $p \\times q(my$, 矛盾.\n引理获证.\n下面证明: 对任意 $p \\times q(ma$, 于是 $X \\backslash\\left\\{a_i, a_{i+1}, \\cdots, a_n\\right\\}$ 的子集的和也跑遍了 $1,2, \\cdots, a$, 这与 $n$ 的最小性矛盾.\n对给定的 $a$, 设 $2^r \\leqslant a<2^{r+1}$, 若 $n \\leqslant r$, 则因为 $a_i \\leqslant 2^{i-1}(i=1,2, \\cdots, n)$, 所以对 $X$ 的任何子集 $A$, 有 $S(A) \\leqslant S(X)=a_1+a_2+\\cdots+a_n \\leqslant 2^0+2^1+ 2^2+\\cdots+2^{n-1}=2^n-1 \\leqslant 2^r-1<2^r \\leqslant a$, 所以不存在 $X$ 的子集 $A$, 使 $S(A)=a$,矛盾,所以 $n \\geqslant r+1$.\n当 $n=r+1$ 时,令 $a_i=2^{i-1}(i=1,2, \\cdots, r), a_{r+1}=a+1-2^r$,下证 $X=\\left\\{a_1, a_2, a_3, \\cdots, a_{r+1}\\right\\}$ 满足条件.\n实际上, 由二进制可知, $\\left\\{a_1, a_2, a_3, \\cdots, a_r\\right\\}$ 的子集的和跑遍了 $1,2, \\cdots$, $2^r-1, X$ 的含有 $a_{r+1}$ 的子集的和跑遍了 $a_{r+1}, a_{r+1}+1, a_{r+1}+2, \\cdots, a_{r+1}+ 2^r-1=a$, 又 $2^r \\leqslant a \\leqslant 2^{r+1}$, 有 $a_{r+1}=a+1-2^r<2^{r+1}+1-2^r=2^r+1$, 所以 $a_{r+1} \\leqslant 2^r$,于是 $X$ 的子集的和跑遍了 $1,2, \\cdots, a$.\n综上所述, $n$ 的最小值为 $r+1$, 其中 $r=\\left[\\log _2 a\\right]$.", + "remark": "", + "figures": [] +} \ No newline at end of file diff --git a/processed_dataset/proof/0549.json b/processed_dataset/proof/0549.json new file mode 100644 index 0000000000000000000000000000000000000000..a17dc15fcb301664606f93e74d460b9d30fa6139 --- /dev/null +++ b/processed_dataset/proof/0549.json @@ -0,0 +1,8 @@ +{ + "source_file": "./raw_volume-zh/volume13/chapter2.tex", + "problem_type": "proof", + "problem": "例3. 给定正整数 $k$ 及正数 $a$, 又 $k_1+k_2+\\cdots+k_r=k$ ( $k_i$ 为正整数, $1 \\leqslant r \\leqslant k$ ), 求 $F=a^{k_1}+a^{k_2}+\\cdots+a^{k_r}$ 的最大值.", + "solution": "分析:解本题的实质是将 $k$ 分解为若干个正整数 $k_i$, 使 $a^{k_1}+a^{k_2}+\\cdots+ a^{k_r}$ 的值最大.\n但其分解出的正整数的个数不确定, 因而应分两步走(求累次最值). 先固定 $r$, 假定 $k$ 分解为 $r$ 个正整数 $k_i(i=1,2, \\cdots, r)$, 求 $a^{k_1}+a^{k_2}+\\cdots +a^{k_r}$ 的最大值 $f(r)$. 然后再解冻变量 $r$, 求 $f(r)$ 的最大值.\n先走第一步.\n取 $k=6, r=3$, 则 $k_1+k_2+k_3=6, F=a^{k_1}+a^{k_2}+a^{k_3}$.\n(1) 若 $\\left(k_1, k_2, k_3\\right)=(2,2,2)$, 则 $F_1=a^2+a^2+a^2=3 a^2$;\n(2) 若 $\\left(k_1, k_2, k_3\\right)=(1,2,3)$, 则 $F_2=a+a^2+a^3$;\n(3) 若 $\\left(k_1, k_2, k_3\\right)=(1,1,4)$, 则 $F_3=a+a+a^4=2 a+a^4$.\n$$\n\\begin{aligned}\nF_2-F_1 & =a-2 a^2+a^3=a\\left(1-2 a+a^2\\right)=a(1-a)^2 \\geqslant 0, \\\\\nF_3-F_2 & =a+a^4-a^2-a^3=a\\left(1+a^3-a^2-a\\right) \\\\\n& =a\\left(1-a^2\\right)(1-a) \\geqslant 0,\n\\end{aligned}\n$$\n所以 $F_3$ 最大.\n一般地, 不难想到, 当指数 $k_1, k_2, \\cdots, k_r$ 尽量集中到某一个指数时, $F$ 的值最大.\n即 $F$ 的极值点为 $(1,1, \\cdots, k-r+1)$. 我们先证明下面的引理: 设 $a>0, x, y \\in \\mathbf{N}^*$, 则 $a^x+a^y \\leqslant a^{x+y-1}+a$.\n实际上, $a^{x+y-1}+a-a^x-a^y=a\\left[a^{x-1}-1\\right]\\left[a^{y-1}-1\\right] \\geqslant 0$.\n反复利用引理, 得\n$$\n\\begin{aligned}\nF & =a^{k_1}+a^{k_2}+\\cdots+a^{k_r} \\\\\n& \\leqslant a+a^{k_1+k_2-1}+a^{k_3}+\\cdots+a^{k_r} \\\\\n& \\leqslant a+a+a^{k_1+k_2+k_3-2}+a^{k_4}+\\cdots+a^{k_r} \\\\\n& \\leqslant \\cdots \\leqslant a+a+\\cdots+a+a^{k_1+k_2+\\cdots+k_r-(r-1)} \\\\\n& =(r-1) a+a^{k-r+1} .\n\\end{aligned}\n$$\n下面再对 $1 \\leqslant r \\leqslant k$; 求 $f(r)=(r-1) a+a^{k-r+1}$ 的最大值.\n令 $f(x)=a(x-1)+a^{k-x+1}$, 则因 $a(x-1) 、 a^{k-x+1}$ 都是凸函数, 所以 $f(x)$ 是凸函数.\n于是 $f(r) \\leqslant \\max \\{f(1), f(k)\\}=\\max \\left\\{a^k, k a\\right\\}$.\n综上所述, $F$ 的最大值为 $\\max \\left\\{a^k, k a\\right\\}$.", + "remark": "", + "figures": [] +} \ No newline at end of file diff --git a/processed_dataset/proof/0550.json b/processed_dataset/proof/0550.json new file mode 100644 index 0000000000000000000000000000000000000000..9bea5c8dd6344b5258051eff4a676f26eef3bdce --- /dev/null +++ b/processed_dataset/proof/0550.json @@ -0,0 +1,8 @@ +{ + "source_file": "./raw_volume-zh/volume13/chapter2.tex", + "problem_type": "proof", + "problem": "例4. 圆内接四边形 $A B C D$ 的四条边长 $A B, B C, C D, D A$ 的长均为正整数, $D A=2005, \\angle A B C=\\angle A D C=90^{\\circ}$, 且 $\\max \\{A B, B C, C D\\}<2005$, 求四边形 $A B C D$ 的周长的最大值和最小值.", + "solution": "解: $A B=a, B C=b, C D=c$, 则 $a^2+b^2=A C^2=c^2+2005^2$, 所以 $2005^2-a^2=b^2-c^2=(b+c)(b-c)$, 其中 $a, b, c \\in\\{1,2, \\cdots, 2004\\}$.\n不妨设 $a \\geqslant b$, 先固定 $a$, 令 $a_1=2005-a$, 则\n$$\n(b+c)(b-c)=2005^2-a^2=a_1\\left(4010-a_1\\right) . \\label{eq1}\n$$\n由 $a^2+b^2>2005^2$, 得 $a>\\frac{2005}{\\sqrt{2}}>1411$, 所以 $1 \\leqslant a_1<2005-1411=594$.\n所以, 由 式\\ref{eq1} 有 $b+c>\\sqrt{(b+c)(b-c)}=\\sqrt{a_1\\left(4010-a_1\\right)}$.\n当 $a_1=1$ 时, $a=2004,(b+c)(b-c)=4009=19 \\times 211$, 所以, $b+c \\geqslant 211, a+b+c \\geqslant 2004+211=2215>2155$;\n当 $a_1=2$ 时, $a=2003,(b+c)(b-c)=2^4 \\times 3 \\times 167$, 又 $b+c$ 与 $b-c$ 同奇偶, 所以, $b+c \\geqslant 2 \\times 167, a+b+c \\geqslant 2003+2 \\times 167>2155$;\n当 $a_1=3$ 时, $a=2002,(b+c)(b-c)=3 \\times 4007$, 所以, $b+c \\geqslant 4007$, $a+b+c \\geqslant 2002+4007>2155$\n当 $a_1=4$ 时, $a=2003 ,(b+c)(b-c)=2^3 \\times 2003$, 所以, $b+c \\geqslant 2 \\times 2003, a+b+c \\geqslant 2001+2 \\times 2003>2155$;\n当 $a_1=5$ 时, $a=2000,(b+c)(b-c)=3^2 \\times 5^2 \\times 89$, 所以, $b+c \\geqslant 225$, $a+b+c \\geqslant 2000+225>2155$;\n当 $a_1=6$ 时, $a=1999,(b+c)(b-c)=6 \\times 4004=156 \\times 154$, 所以, $b+c \\geqslant 156, a+b+c \\geqslant 1999+156=2155$;\n当 $a_1 \\geqslant 7$ 时, 因为 $b+c>\\sqrt{a_1\\left(4010-a_1\\right)}$, 所以 $a+b+c> \\sqrt{a_1\\left(4010-a_1\\right)}+2005-a_1$, 但 $7 \\leqslant a_1<594$, 我们可证明 $\\sqrt{a_1\\left(4010-a_1\\right)} +2005-a_1>2155$.\n实际上, $\\sqrt{a_1\\left(4010-a_1\\right)}+2005-a_1>2155 \\Leftrightarrow \\sqrt{a_1\\left(4010-a_1\\right)}> 150+a_1 \\Leftrightarrow-a_1^2+4010 a_1>a_1^2+300 a_1+150^2 \\Leftrightarrow a_1^2-1855 a_1+11250<0$, 由二次函数性质可知, 此不等式在 $7 \\leqslant a_1<594$ 时成立.\n综上所述, 恒有 $a+b+c \\geqslant 2155$, 于是 $A B+B C+C D+D A \\geqslant 2155+ 2005=4160$.\n当 $A B=1999, B C=155, C D=1$ 时等号成立,所以四边形 $A B C D$ 的周长的最小值为 4160 .\n下面求四边形 $A B C D$ 的周长的最大值:\n因为 $a \\geqslant b, c<2005$, 所以 $b+cn_t n_{t+1}$, 所以\n$S^{\\prime}>S$, 矛盾.\n所以 $n_{t+1}-n_t=1$ 或 2 .\n(2) 至少有一个 $t(1 \\leqslant t \\leqslant 29)$, 使 $n_{t+1}-n_t=2$.\n实际上, 若对所有 $t$, 都有 $n_{t+1}-n_t \\neq 2$, 而由 (1), 有 $n_{t+1}-n_t \\leqslant 2$, 所以 $n_{t+1}-n_t=1$, 即 $n_1, n_2, \\cdots, n_{30}$ 是 30 个连续正整数, 它们的和为 15 的倍数.\n但 $\\sum_{t=1}^{30} n_t=1989$ 不是 15 的倍数,矛盾.\n(3) 最多有一个 $t(1 \\leqslant t \\leqslant 29)$, 使 $n_{t+1}-n_t=2$.\n实际上, 若有 $s 、 t(1 \\leqslant sn_s n_{t+1}$, 所以 $S^{\\prime}>S$, 矛盾.\n由 (2) 和 (3) 可知, 恰有一个 $t(1 \\leqslant t \\leqslant 29)$, 使 $n_{t+1}-n_t=2$.\n最后证明, 同时满足 (1) (2) 和 (3) 的数组: $n_1, \\cdots, n_{30}$ 是唯一的.\n实际上, 不妨设 30 个数为: $n_1, n_1+1, n_1+2, \\cdots, n_1+t-1, n_1+t+1$, $n_1+t+2, \\cdots, n_1+30$, 那么 $n_1+\\left(n_1+1\\right)+\\left(n_1+2\\right)+\\cdots+\\left(n_1+t-\\right.$ 1) $+\\left(n_1+t+1\\right)+\\left(n_1+t+2\\right)+\\cdots+\\left(n_1+30\\right)=1989$.\n所以 $\\left(n_1+t\\right)+\\left(n_1+1\\right)+\\left(n_1+2\\right)+\\cdots+\\left(n_1+t-1\\right)+\\left(n_1+t+\\right.$ 1) $+\\left(n_1+t+2\\right)+\\cdots+\\left(n_1+30\\right)=1989+t$, 即 $1989+t=30 n_1+(1+ 2+\\cdots+30)=30 n_1+15 \\times 31$.\n所以 $1974+t=30 n_1+15 \\times 30,30|1974+t, 30| 24+t$, 但 $1 \\leqslant t \\leqslant 29$, 所以 $t=6$.\n综上所述, 所求的各组的点数为 $51,52, \\cdots, 56,58,59, \\cdots, 81$.", + "remark": "", + "figures": [] +} \ No newline at end of file diff --git a/processed_dataset/proof/0553.json b/processed_dataset/proof/0553.json new file mode 100644 index 0000000000000000000000000000000000000000..d35430bbf32d4ec973af52daf20838cfb033a3d8 --- /dev/null +++ b/processed_dataset/proof/0553.json @@ -0,0 +1,8 @@ +{ + "source_file": "./raw_volume-zh/volume13/chapter3.tex", + "problem_type": "proof", + "problem": "例4. IMO 太空站由 99 个空间站组成,任两个空间站之间有管形通道相连.\n规定其中 99 条通道为双向通行的主干道, 其余通道严格单向通行.\n如果某 4 个空间站可以通过它们之间的通道从其中任一站到达另外任一站, 就称这些站组成的集合为一个互通四站组.\n试求互通四站组个数的最大值, 并证明你的结论.", + "solution": "分析:解由正面求 \"四通组\" 的个数是比较困难的, 因为其条件较为苛刻.\n而满足非互通的四站组的条件相对较易.\n比如, 想象一个空间站 \"无回路\" 即可.\n由此想到考察从一点引出的所有单向通道, 每 3 条这样的通道对应一个非互通的四站组.\n当然,还可能有其他的非互通的四站组, 但那些非互通的四站组并非必定存在, 也就是说, 也许可以选择一个方案, 使那些非互通的四站组的个数为零, 从而可以略去这些非互通的四站组的个数估计.\n假设 99 个空间站为 $A_1, A_2, \\cdots, A_{99}$, 由 $A_i$ 出发的单向通道条数、指向 $A_i$ 的单向通道条数、通过 $A_i$ 的主干道条数分别为 $w_i 、 l_i 、 k_i$. 由条件, 有 $w_i+l_i+k_i=98$, 且 $k_1+\\cdots+k_{99}=198, w_1+\\cdots+w_{99}=l_1+\\cdots+l_{99}$, 于是\n$$\n\\begin{aligned}\n\\sum_{i=1}^{99} w_i & =\\sum_{i=1}^{99} l_i=\\frac{1}{2} \\sum_{i=1}^{99}\\left(w_i+l_i\\right) \\\\\n& =\\frac{1}{2} \\sum_{i=1}^{99}\\left(w_i+l_i+k_i\\right)-\\frac{1}{2} \\sum_{i=1}^{99} k_i=4752 .\n\\end{aligned}\n$$\n因为 $A_i$ 引出 $w_i$ 条单向通道, 其中任何 3 条组成一个输出型三面角, 这个三面角的四个顶点是一个不互通四站组,而且同一个不互通的四站组不可能包含两个输出三面角, 所以非互通四站组的数目 $S \\geqslant \\sum_{i=1}^{99} \\mathrm{C}_{w_i}^3$.\n下面求 $\\sum_{i=1}^{99} \\mathrm{C}_{w_i}^3$ 的最小值.\n方法 1: 首先, 由于满足 $w_1+\\cdots+w_{99}=4752$ 的数组 $\\left(w_1, \\cdots, w_{99}\\right)$ 只有有限个, 从而最小值一定存在.\n其次, 从直观猜测, 当 $w_1=\\cdots=w_{99}=48$ 时, $\\sum_{i=1}^{99} \\mathrm{C}_{w_i}^3$ 最小.\n反设结论不然, 则必有一个 $i$, 使 $w_i<48$, 也必有一个 $j$, 使 $w_j>$ 48. 将 $w_i$ 改为 $w_i+1, w_j$ 改为 $w_j^{--1}$, 得到一个新的数组, 由于\n$$\n\\begin{aligned}\n\\mathrm{C}_{w_i}^3+\\mathrm{C}_{w_j}^3-\\mathrm{C}_{w_i+1}^3-\\mathrm{C}_{w_j-1}^3 & =\\frac{1}{2}\\left(w_j-1\\right)\\left(w_j-2\\right)-\\frac{1}{2} w_i\\left(w_i-1\\right) \\\\\n& >0\\left(\\text { 因为 } w_j-1>w_i\\right),\n\\end{aligned}\n$$\n所以新数组对应的和式比原数组小, 矛盾.\n于是 $\\sum_{i=1}^{99} \\mathrm{C}_{w_i}^3$ 的最小值为 $99 \\mathrm{C}_{48}^3$, 从而互通的四站组不多于 $C_{99}^4-99 C_{48}^3=2052072$.\n方法 2: 由幕平均不等式, 有\n$$\n\\begin{gathered}\n\\sum_{i=1}^{99} w_i^3 \\geqslant \\frac{1}{\\sqrt{99}}\\left(\\sum_{i=1}^{99} w_i^2\\right)^{\\frac{3}{2}}, \\\\\n\\sum_{i=1}^{99} \\mathrm{C}_{w_i}^3=\\frac{1}{6} \\sum_{i=1}^{99} w_i^3-\\frac{1}{2} \\sum_{i=1}^{99} w_i^2+\\frac{1}{3} \\sum_{i=1}^{99} w_i \\\\\n\\geqslant \\frac{1}{6 \\sqrt{99}}\\left(\\sum_{i=1}^{99} w_i^2\\right)^{\\frac{3}{2}}-\\frac{1}{2} \\sum_{i=1}^{99} w_i^2+\\frac{1}{3} \\times 4752 .\n\\end{gathered}\n$$\n注意到\n$$\n\\frac{1}{6 \\sqrt{99}} x^{\\frac{3}{2}}-\\frac{1}{2} x=\\frac{1}{6} x\\left(\\sqrt{\\frac{x}{99}}-3\\right)\n$$\n严格递增, 且\n$$\n\\sum_{i=1}^{99} w_i^2 \\geqslant \\frac{1}{99}\\left(\\sum_{i=1}^{99} w_i\\right)^2=228096,\n$$\n故\n$$\n\\begin{aligned}\n\\sum_{i=1}^{99} \\mathrm{C}_{w_i}^3 & \\geqslant \\frac{1}{6 \\sqrt{99}} \\times 228096^{\\frac{3}{2}}-\\frac{1}{2} \\times 228096+\\frac{1}{3} \\times 4752 \\\\\n& =1712304,\n\\end{aligned}\n$$\n从而互通四站组的数目不多于 $\\mathrm{C}_{99}^4-1712304=2052072$.\n下面证明等号可以成立.\n先将所有通道设成单向,方向如下规定: 对 $i< j$, 若 $i, j$ 同奇偶, 则由 $A_i$ 指向 $A_j$, 否则 $A_j$ 指向 $A_i$. 此时每个点恰发出 49 条单向通道.\n现将 99 条通道 $A_i A_{i+1}\\left(i=1,2, \\cdots, 99, A_{100}=A_1\\right)$ 改为双向通道, 则每个点发出和进人的单向通道恰有一条被改为双向通道, 故 $w_i=l_i=48(i= 1,2, \\cdots, 99)$. 故有中心的四站组个数为 $99 \\mathrm{C}_{48}^3=1712304$.\n下面证明每一非互通四站组都有一个中心 (向其他三点引出单向通道的点). 事实上, 设 $\\left(A_i, A_j, A_k, A_t\\right.$ ) 是任意一个无中心的非互通四站组 (以下仅用 $i$ 代表 $A_i$ ), 若其中有一条双向通道 $i j$, 则 $i k$ 和 $j k$ 不可能都由 $k$ 发出, 或都指向 $k$ (若 $j=i+1$ 或 $j=i-1$, 则 $k$ 要么比 $i 、 j$ 都大, 要么都小, 而 $i 、 j$ 不同奇偶; 若 $\\{i, j\\}=\\{1,99\\}$, 则 $i, j$ 同奇偶, 但 $k$ 比一个大, 比另一个小), 对 $t$\n也一样, 故 $i j k t$ 是互通的,矛盾.\n故其中无双向通道.\n由于 $i j k t$ 不互通, 故其中有一个站(设为 $i$ ), 它和其余三个站之间的通道都是单向的, 而且都指向 $i$. 故 $j 、 k 、 t$ 只有两种情况: 要么比 $i$ 小且与 $i$ 同奇偶 (称为 $I$ 型), 要么比 $i$ 大且奇偶性不同 (称为 II 型). 如果 $j 、 k 、 t$ 都为 I 型, 则 $j 、 k 、 t$ 同奇偶, 故其中最小的是 ( $i j k t)$ 的一个中心, 矛盾.\n如果 $j 、 k 、 t$ 都为 II 型, 则 $j 、 k 、 t$ 中最小的是 $(i j k t)$ 的一个中心, 矛盾.\n如果 $j 、 k 、 t$ 中有两个, 比如 $j 、 k$, 为 I 型, 则 $t$ 比 $j$ 、 $k$ 都大且不同奇偶, 故 $t$ 是 ( $i j k t$ ) 的中心, 矛盾.\n如果 $j 、 k 、 t$ 中有一个, 比如 $j$, 为 I 型, 则 $k 、 t$ 比 $i 、 j$ 大且不同奇偶, 故 $k 、 t$ 中必有一个为中心,矛盾.\n这就证明了每个非互通四站组必有中心.\n于是, 非互通四站组共有 1712304 个, 从而互通四站组有 2052072 个.\n综上所述,所求最大值为 2052072 .", + "remark": "", + "figures": [] +} \ No newline at end of file diff --git a/processed_dataset/proof/0554.json b/processed_dataset/proof/0554.json new file mode 100644 index 0000000000000000000000000000000000000000..5a16403833e9291a0abebd5a983b2f1aa32a24e1 --- /dev/null +++ b/processed_dataset/proof/0554.json @@ -0,0 +1,8 @@ +{ + "source_file": "./raw_volume-zh/volume13/chapter3.tex", + "problem_type": "proof", + "problem": "例5. 将 2006 表示成 5 个正整数 $x_1, x_2, x_3, x_4, x_5$ 之和, 记 $S= \\sum_{1 \\leqslant ix_1 x_2$.\n将 $S$ 改写成\n$$\nS=\\sum_{1 \\leqslant i0$, 这与 $S$ 在 $x_1, x_2, x_3, x_4, x_5$ 时取到最大值矛盾, 所以必有 $\\left|x_i-x_j\\right| \\leqslant 1(1 \\leqslant i, j \\leqslant 5)$. 因此当 $x_1=402, x_2= x_3=x_4=x_5=401$ 时 $S$ 取到最大值.\n(2) 当 $x_1+x_2+x_3+x_4+x_5=2006$ 且 $\\left|x_i-x_j\\right| \\leqslant 2(1 \\leqslant i, j \\leqslant 5)$ 时, 只有如下这三种情形满足要求:\n(I) 402, 402, 402, 400, 400;\n(II) 402, 402, 401, 401, 400;\n(III) 402, 401,401,401,401.\n而后两种情形是在第一种组情形下作调整: $x_i^{\\prime}=x_i-1, x_j^{\\prime}=x_j+1$ 而得到的, 根据 (1) 的证明可知, 每调整一次, 和式 $S=\\sum_{1 \\leqslant i\\left[\\frac{n}{2}\\right] \\text { 时 }\\right) \\text {. }\n$$\n我们证明 $F(x)\\left[\\frac{n}{2}\\right]$, 记 $d=x_{i+1}-x$, 则将 $x$ 调整到 $x^{\\prime}=x_{i+1}$ 后, $\\left|x-x_1\\right|$, $\\left|x-x_2\\right|, \\cdots,\\left|x-x_i\\right|$ 都增加 $d$, 一共增加 $i$ 个 $d$. 而 $\\left|x-x_{i+1}\\right|,\\left|x-x_{i+2}\\right|$, $\\cdots,\\left|x-x_n\\right|$ 都减少 $d$, 一共减少 $n-i$ 个 $d$. 于是调整后, $F$ 增加了 $(2 i-n)$ 个 $d$, 即 $F^{\\prime}(x)-F(x)=(2 i-n) d$. 因为 $i>\\left[\\frac{n}{2}\\right]$, 又 $i 、 n$ 都是正整数, 所以当 $n$ 为偶数时, $i>\\frac{n}{2}$, 当 $n$ 为奇数时, $i \\geqslant \\frac{n+1}{2}>\\frac{n}{2}$. 所以恒有 $2 i-n>0, F^{\\prime}(x)>F(x)$, 于是 $x$ 不是最大值点,矛盾.\n(2) 若存在 $i(i=1,2, \\cdots, n)$, 使得 $x=x_i$, 则令\n$$\n\\left.x^{\\prime}=x_{i-1} \\text { (当 } i \\leqslant\\left[\\frac{n}{2}\\right] \\text { 时), 及 } x^{\\prime}=x_{i+1} \\text { (当 } i>\\left[\\frac{n}{2}\\right] \\text { 时 }\\right) \\text {. }\n$$\n我们证明 $F(x)\\left[\\frac{n}{2}\\right]$, 记 $d=x_{i+1}-x$, 则将 $x$ 调整到 $x^{\\prime}=x_{i+1}$ 后, $\\left|x-x_1\\right|$, $\\left|x-x_2\\right|, \\cdots,\\left|x-x_i\\right|$ 都增加 $d$,一共增加 $i$ 个 $d$. 而 $\\left|x-x_{i+1}\\right|,\\left|x-x_{i+2}\\right|$, $\\cdots,\\left|x-x_n\\right|$ 都减少 $d$, 一共减少 $n-i$ 个 $d$. 于是调整后, $F$ 增加了 $(2 i-n)$ 个 $d$, 即 $F^{\\prime}(x)-F(x)=(2 i-n) d$. 因为 $i>\\left[\\frac{n}{2}\\right]$, 又 $i 、 n$ 都是正整数,所以当 $n$ 为偶数时, $i>\\frac{n}{2}$, 当 $n$ 为奇数时, $i \\geqslant \\frac{n+1}{2}>\\frac{n}{2}$. 所以恒有 $2 i-n>0$, $F^{\\prime}(x)>F(x)$, 于是 $x$ 不是最大值点,矛盾.\n综上所述, $F$ 取最大值时, 必有 $x \\in\\{0,1\\}$.\n下面解答原题.\n由于 $F$ 在闭域上连续, 所以必定存在最大值.\n固定 $x_2, x_3, \\cdots, x_n$, 则 $F\\left(x_1\\right)$ 是关于 $x_1$ 的函数:\n$$\nF\\left(x_1\\right)=\\left|x_1-x_2\\right|+\\left|x_1-x_3\\right|+\\cdots+\\left|x_1-x_n\\right|+\\sum_{2 \\leqslant i\\frac{n-1}{25}$. 所以 $n-1<\\frac{25}{a_1} \\leqslant 25, n \\leqslant 25$. 但此估计过宽, 我们采用\"起点后移\" 的办法对之进行改进.\n令 $i=2,3, \\cdots, n-1$, 并将得到的各式累加, 得 $\\frac{1}{a_2}-\\frac{1}{a_n} \\geqslant \\frac{n-2}{25}$. 所以 $\\frac{1}{a_2}>\\frac{n-2}{25}$, 所以 $2 \\leqslant a_2<\\frac{25}{n-2}, n<\\frac{29}{2}, n \\leqslant 14$. 经试验, 还要继续改进估计.\n类似地, 有 $3 \\leqslant a_3<\\frac{25}{n-3}, n \\leqslant 11 ; 4 \\leqslant a_4<\\frac{25}{n-4}, n \\leqslant 10 ; 5 \\leqslant a_5< \\frac{25}{n-5}, n \\leqslant 9 ; 6 \\leqslant a_6<\\frac{25}{n-6}, n \\leqslant 10$. 由 $n$ 的范围的变化趋势, 可猜想 $n \\leqslant$ 9 是最好的估计.\n下面证明存在合乎条件的 9 元子集 $A$. 首先, 当 $x y \\leqslant 25$ 时, 条件 $\\mid x- y \\mid \\geqslant \\frac{x y}{25}$ 显然满足.\n从而可取 $1,2,3,4,5 \\in A$, 此时, $6 \\notin A$. 否则 $|6-5|< \\frac{30}{25}$, 不合条件.\n如此下去, 发现可取 $7,10,17,54 \\in A$. 所以, $A=\\{1,2,3,4$, $5,7,10,17,54\\}$ 为所求.\n综上所述, $|A|$ 的最大值为 9 .", + "remark": "", + "figures": [] +} \ No newline at end of file diff --git a/processed_dataset/proof/0559.json b/processed_dataset/proof/0559.json new file mode 100644 index 0000000000000000000000000000000000000000..ae0ddd184927c8f4a5d1138f1341e0e4fc06179b --- /dev/null +++ b/processed_dataset/proof/0559.json @@ -0,0 +1,8 @@ +{ + "source_file": "./raw_volume-zh/volume13/chapter7.tex", + "problem_type": "proof", + "problem": "例3. 设 $A \\subseteq\\{0,1,2, \\cdots, 29\\}$, 满足: 对任何整数 $k$ 及 $A$ 中任意数 $a$ 、 $b$ ( $a 、 b$ 可以相同), $a+b+30 k$ 均不是两个相邻整数之积.\n试求出所有元素个数最多的 $A$.", + "solution": "解:求的 $A=\\{3 t+2 \\mid 0 \\leqslant t \\leqslant 9, t \\in \\mathbf{Z}\\}$.\n设 $A$ 满足题中条件且 $|A|$ 最大.\n因为对两个相邻整数 $a 、 a+1$, 有 $a(a-1) \\equiv 0,2,6,12,20,26(\\bmod 30)$. 于是对任一 $a \\in A$, 取 $b=a, k=$ 0 , 可知 $2 a \\neq 0,2,6,12,20,26(\\bmod 30)$, 即 $a \\neq 0,1,3,6,10,13,15$, $16,18,21,25,28(\\bmod 30)$. 因此, $A \\subseteq M=\\{2,4,5,7,8,9,11,12,14$, $17,19,20,22,23,24,26,27 , 29\\}$, 而 $M$ 可分拆成下列 10 个子集的并: $A_1=\\{2,4\\}, A_2=\\{5,7\\}, A_3=\\{8,12\\}, A_4=\\{11,9\\}, A_5=\\{14,22\\}$, $A_6=\\{17,19\\}, A_7=\\{20\\}, A_8=\\{23,27\\}, A_9=\\{26,24\\}, A_{10}=\\{29\\}$. 其中每一个子集 $A_i$ 至多包含 $A$ 中一个元素, 故 $|A| \\leqslant 10$.\n若 $|A|=10$, 则每个子集 $A_i$ 恰好包含 $A$ 中一个元素, 于是, $20 \\in A$, $29 \\in A$. 由 $20 \\in A$ 知 $12 \\notin A, 22 \\notin A$, 从而 $8 \\in A, 14 \\in A$, 这样 $4 \\notin A$, $24 \\notin A$. 因此 $2 \\in A, 26 \\in A$. 由 $29 \\in A$ 知 $7 \\notin A, 27 \\notin A$, 从而 $5 \\in A$, $23 \\in A$, 这样 $9 \\notin A, 19 \\notin A$, 因此 $11 \\in A, 17 \\in A$.\n综上所述,所求的集合 $A=\\{2,5,8,11,14,17,20,23,26,29\\}$, 经验证, $A$ 满足要求.", + "remark": "", + "figures": [] +} \ No newline at end of file diff --git a/processed_dataset/proof/0560.json b/processed_dataset/proof/0560.json new file mode 100644 index 0000000000000000000000000000000000000000..bc681994985f2160aba183d019ee94d625e1871c --- /dev/null +++ b/processed_dataset/proof/0560.json @@ -0,0 +1,8 @@ +{ + "source_file": "./raw_volume-zh/volume13/chapter7.tex", + "problem_type": "proof", + "problem": "例4. 设 $A$ 是 $X=\\{1,2,3, \\cdots, 1989\\}$ 的子集, 对任何 $x, y \\in A$,有 $|x-y| \\neq 4$ 和 7. 求 $|A|$ 的最大值.", + "solution": "分析: 设$A$ 是 $X=\\{1,2,3, \\cdots, 1989\\}$ 的子集,若对任何 $x, y \\in A$,有 $|x-y| \\neq 4$ 和 7 , 则称 $A$ 是好子集.\n显然,好子集具有平移不变性, 即 $A$ 是好子集,则对任何 $a, A+a$ 也是好子集.\n所以可进行均匀划块估计.\n设 $X=P_1 \\cup P_2 \\cup \\cdots \\cup P_k$, 注意题目的目标是 $|A| \\leqslant r, r$ 越小, 估计越精确.\n因此应使 \"好\" 元素 (属于 $A$ ) 在 $P_i$ 中所占的比例尽可能小.\n设 $P= \\{1,2, \\cdots, t\\}$, 列表观察:\n\\begin{tabular}{c|ccccccccccccc}\n$t=|P|$ & 1 & 2 & 3 & 4 & 5 & 6 & 7 & 8 & 9 & 10 & 11 & 12 & 13 \\\\\n\\hline\n$|A \\cap P|$ & 1 & 2 & 3 & 4 & 4 & 4 & 4 & 4 & 5 & 5 & 5 & 6 & 7 \\\\\n\\hline\n$\\frac{|A \\cap P|}{|P|}$ & 1 & 1 & 1 & 1 & $\\frac{4}{5}$ & $\\frac{2}{3}$ & $\\frac{4}{7}$ & $\\frac{1}{2}$ & $\\frac{5}{9}$ & $\\frac{1}{2}$ & $\\frac{5}{11}$ & $\\frac{1}{2}$ & $\\frac{7}{13}$\n\\end{tabular}\n其中以 $P=\\{1,2, \\cdots, 11\\}$ 时得到的比值 $\\frac{5}{11}$ 最小, 猜想以 $\\{1,2, \\cdots, 11\\}$ 为一个子集的划分是最佳的.\n解设 $A$ 是合乎题意的子集, 对 $P=\\{1,2, \\cdots, 11\\}$, 我们证明 $\\mid A \\cap P \\mid \\leqslant 5$.\n实际上, 将 $P$ 划分为 6 个子集 $\\{1,5\\} 、\\{2,9\\} 、\\{3,7\\} 、\\{4,8\\} 、\\{6,10\\}$ 、 $\\{11\\}$, 对所划分的每个子集, $A$ 最多含有它的一个元素, 所以 $|A \\cap P| \\leqslant 6$. 若 $|A \\cap P|=6$, 则 $A$ 在每个划分的子集中都至少含有一个元素, 于是 $11 \\in A, \\Rightarrow 4 \\notin A, \\Rightarrow 8 \\in A, \\Rightarrow 1 \\notin A, \\Rightarrow 5 \\in A, \\Rightarrow 9 \\notin A, \\Rightarrow 2 \\in A, \\Rightarrow 6 \\notin A, \\Rightarrow 10 \\in A, \\Rightarrow 3 \\notin A, \\Rightarrow 7 \\in A$, 但 $11-7=4$,矛盾.\n所以 $|A \\cap P| \\leqslant 5$.\n令 $P_k=\\{11 k+1,11 k+2, \\cdots, 11 k+11\\}(k=0,1,2, \\cdots, 179)$, $P_{180}=\\{1981,1982, \\cdots, 1989\\}$. 则同样可知, $A$ 至多含有 $P_k(k=0,1$, $2, \\cdots, 180)$ 中的 5 个元素, 所以 $|A| \\leqslant 5 \\times 181=905$.\n最后, 令 $A_k=\\{11 k+1,11 k+3,11 k+4,11 k+6,11 k+9\\}(k=0,1$, $2, \\cdots, 180), A=A_0 \\cup A_1 \\cup \\cdots \\cup A_{180}$, 则 $A$ 合乎题意, 此时 $|A|=905$, 故 $|A|$ 的最大值为 905 .", + "remark": "", + "figures": [] +} \ No newline at end of file diff --git a/processed_dataset/proof/0561.json b/processed_dataset/proof/0561.json new file mode 100644 index 0000000000000000000000000000000000000000..aac24157903d4ab3cc663c3549ad1892aad4a056 --- /dev/null +++ b/processed_dataset/proof/0561.json @@ -0,0 +1,8 @@ +{ + "source_file": "./raw_volume-zh/volume13/chapter7.tex", + "problem_type": "proof", + "problem": "例5. 设 $p$ 为给定的正整数, $A$ 是 $X=\\left\\{1,2,3,4, \\cdots, 2^p\\right\\}$ 的子集, 且具有性质: 对任何 $x \\in A$, 有 $2 x \\notin A$. 求 $|A|$ 的最大值.", + "solution": "分析:解将 $X$ 划块, 对 $p$ 归纳.\n当 $p=1$ 时, $X=\\{1,2\\}$, 取 $A=\\{1\\}$,于是 $f(1)=1$;\n当 $p=2$ 时, $X=\\{1,2,3,4\\}$, 将 $X$ 划分为 3 个子集 $\\{1,2\\} 、\\{3\\} 、\\{4\\}$, 则 $A$ 至多含每个子集中的一个数, 于是 $|A| \\leqslant 3$. 取 $A=\\{1,3,4\\}$, 于是 $f(2)=3$;\n当 $p=3$ 时, $X=\\{1,2, \\cdots, 8\\}$. 将 $X$ 划分为 5 个子集:\\{1,2\\}、\\{3,6\\}、 $\\{4,8\\} 、\\{5\\} 、\\{7\\}$, 则 $A$ 至多含有每个子集中的一个数, 于是 $|A| \\leqslant 5$. 取 $A=\\{1,5,6,7,8\\}$, 则 $f(3)=5$.\n一般地, 当 $X_p=\\left\\{1,2,3, \\cdots, 2^p\\right\\}$ 时, 可进行划块估计.\n注意到 $2^{p-1}+1$, $2^{p-1}+2, \\cdots, 2^p$ 都可属于 $A$, 于是想到划块: $X=\\left\\{1,2,3, \\cdots, 2^{p-1}\\right\\} \\cup\\left\\{2^{p-1}+\\right. \\left.1,2^{p-1}+2, \\cdots, 2^p\\right\\}=X_{p-1} \\cup M$, 其中 $X_{p-1}=\\left\\{1,2,3, \\cdots, 2^{p-1}\\right\\}, M=\\left\\{2^{p-1}+\\right. \\left.1,2^{p-1}+2, \\cdots, 2^p\\right\\}$. 这样, 问题在于 $X_{p-1}=\\left\\{1,2,3, \\cdots, 2^{p-1}\\right\\}$ 中至多有多少个属于 $A$, 这是否为原问题在 $p-1$ 的情形? 问题没有这么简单! 试想: $M$ 中的数 $2^{p-1}+1,2^{p-1}+2, \\cdots, 2^p$ 都属于 $A$ 时, $X_{p-1}$ 中有很多数不能属于 $A$, 比如: $2^{p-2}+1,2^{p-2}+2, \\cdots, 2^{p-1}$ 都不属于 $A$; 但未必 $2^{p-1}+2,2^{p-1}+4, \\cdots, 2^p$ 都属于 $A$. 因此, 还要作更细的划块: $M$ 中的部分数 $2^{p-1}+2,2^{p-1}+4, \\cdots, 2^p$ 与 $X_{p-1}$ 中的有关数 ( 2 倍关系) 搭配构造集合: $\\left\\{2^{p-1}+2,2^{p-2}+1\\right\\},\\left\\{2^{p-1}+4\\right.$, $\\left.2^{p^{-2}}+2\\right\\}, \\cdots,\\left\\{2^p, 2^{p-1}\\right\\}$, 由此得到 $X_p$ 的一个划分:\n$$\nX_{p-2}=\\left\\{1,2,3, \\cdots, 2^{p-2}\\right\\}, M_t=\\left\\{2^{p-1}+2 t, 2^{p-2}+t\\right\\} \\quad(t=1,2, \\cdots,\n$$\n$\\left.2^{p-2}\\right), M_0=\\left\\{2^{p-1}+1,2^{p-1}+3,2^{p-1}+5, \\cdots, 2^{p-1}+2^{p-1}-1\\right\\}$.\n因为 $A$ 至多含 $M_t\\left(t=1,2, \\cdots, 2^{p-2}\\right)$ 中一个元素, 至多含 $X_{p-2}$ 中 $f(p-2)$ 个元素, 至多含 $M_0$. 中 $2^{p-2}$ 个元素, 于是 $f(p) \\leqslant f(p-2)+2^{p-2}+ 2^{p-2}=f(p-2)+2^{p-1}$.\n下面考虑, 能否构造集合 $A$, 证明 $f(p) \\geqslant f(p-2)+2^{p-1}$.\n设 $X=\\left\\{1,2,3, \\cdots, 2^{p-2}\\right\\}$ 的合乎题意的最大子集为 $A_1$, 令 $A_2= \\left\\{2^{p-1}+1,2^{p-1}+2, \\cdots, 2^p\\right\\}$, 则对任何 $x \\in A_1$, 有 $2 x \\leqslant 2 \\cdot 2^{p-2}=2^{p-1}< 2^{p-1}+1 \\notin A_2$, 于是, $A=A_1 \\cup A_2$ 是合乎题意的子集, 故 $f(p) \\geqslant|A|= f(p-2)+2^{p-1}$.\n综上所述, $f(p)=f(p-2)+2^{p-1}$.\n下面用两种方案解此递归关系.\n方案 1: 迭代 ( $p$ 个等式相加), 得 $f(p-1)+f(p)=f(1)+f(2)+ 2^2+2^3+\\cdots+2^{p-1}=1+\\left(2^0+2^1\\right)+2^2+2^3+\\cdots+2^{p-1}=2^p$.\n再迭代 (第 $p-1$ 个等式减第 $p-2$ 个等式, 加第 $p-3$ 个等式等等), 得\n$$\nf(p)+(-1)^p f(1)=2^p-2^{p-1}+\\cdots+(-1)^p \\cdot 2^2,\n$$\n所以\n$$\nf(p)=2^p-2^{p-1}+\\cdots+(-1)^p \\cdot 2^2+(-1)^{p+1},\n$$\n注意到 $(-1)^{p+1} \\cdot 2^1+(-1)^{p+2} \\cdot 2^0=(-1)^{p+1}(2-1)=(-1)^{p+1}$,\n所以 $f(p)=2^p-2^{p-1}+\\cdots+(-1)^p \\cdot 2^2+(-1)^{p+1} \\cdot 2^1+(-1)^{p+2} \\cdot 2^0$\n$$\n=\\frac{2^p\\left[1-\\left(-\\frac{1}{2}\\right)^{p+1}\\right]}{1+\\frac{1}{2}}=\\frac{2^{p+1}+(-1)^p}{3} .\n$$\n方案 2: 分类求解.\n当 $p$. 为奇数时, $f(p)=f(p-2)+2^{p-1}=f(p-4)+2^{p-3}+2^{p-1}= f(1)+2^2+2^4+\\cdots+2^{p-1}=2^0+2^2+2^4+\\cdots+2^{p-1}=\\frac{2^{p+1}-1}{3}$;\n当 $p$ 为偶数时, $f(p)=f(p-2)+2^{p-1}=f(p-4)+2^{p-3}+2^{p-1}= f(2)+2^3+2^5+\\cdots+2^{p-1}=1+2^1+2^3+2^5+\\cdots+2^{p-1}=\\frac{2^{p+1}+1}{3}$.", + "remark": "", + "figures": [] +} \ No newline at end of file diff --git a/processed_dataset/proof/0562.json b/processed_dataset/proof/0562.json new file mode 100644 index 0000000000000000000000000000000000000000..bc1e2b2c39ac305b52e1feedea447b499b70b648 --- /dev/null +++ b/processed_dataset/proof/0562.json @@ -0,0 +1,8 @@ +{ + "source_file": "./raw_volume-zh/volume13/chapter7.tex", + "problem_type": "proof", + "problem": "例6. 设 $X=\\{1,2, \\cdots, 2001\\}$, 求最小的正整数 $m$, 使其适合要求: 对 $X$ 的任何一个 $m$ 元子集 $W$, 都存在 $u, v \\in W$ ( $u 、 v$ 可以相同),使得 $u+v$ 是 2 的方幂.", + "solution": "分析:解为叙述问题方便, 如果 $u+v$ 是 2 的方幂, 则称 $u 、 v$ 是一个对子.\n我们从反面考虑, 如果 $X$ 的子集 $W$ 不含对子, 则 $W$ 最多有多少个元素? 显然, 我们如果能将 $X$ 划分成若干块, 使每一块中任何 2 个数是对子, 则 $W$ 只能含每一块中的一个元素.\n于是, 令 $A_i=\\{1024-i, 1024+i\\}{ }^{\\prime \\prime}(i=1,2, \\cdots$, $977), B_j=\\{32-j, 32+j\\}(j=1,2, \\cdots, 14), C=\\{15,17\\}, D_k=\\{8- k, 8+k\\}(k=1,2, \\cdots, 6), E=\\{1,8,16,32,1024\\}$ .假定 $W$ 不含有对子, 则 $W$ 不能含有 $E$ 中的元素, 且最多只能含有各 $A_i 、 B_j 、 D_k$ 与 $C$ 中的一个元素, 于是, $|W| \\leqslant 977+14+6+1=998$. 这表明, 当 $|W| \\geqslant 999$ 时, $W$ 中必有对子, 也就是说, $m=999$ 合乎条件.\n其次, 若 $|W|=998$ 且 $W$ 不含有对子, 则 $W$ 恰含各 $A_i 、 B_j 、 D_k$ 与 $C$ 中的一个元素, 令 $W=\\{1025,1026, \\cdots$, $2001\\} \\cup\\{33,34, \\cdots, 46\\} \\cup\\{17\\} \\cup\\{9,10,11,12,13,14\\}$, 容易验证 $W$ 中没有对子.\n于是, 当 $m<999$ 时,取 $W$ 的一个 $m$ 元子集,则该子集中没有对子.\n综上所述, $m$ 的最小值为 999 .", + "remark": "", + "figures": [] +} \ No newline at end of file diff --git a/processed_dataset/proof/0563.json b/processed_dataset/proof/0563.json new file mode 100644 index 0000000000000000000000000000000000000000..f0d82fbddf29a14de2c0b7a21c31323276e60a89 --- /dev/null +++ b/processed_dataset/proof/0563.json @@ -0,0 +1,15 @@ +{ + "source_file": "./raw_volume-zh/volume13/chapter7.tex", + "problem_type": "proof", + "problem": "例7. 设 $n$ 是一个固定的正偶数,考虑一个 $n \\times n$ 的正方形棋盘,如果两个方格至少有一条公共边, 则称它们是相邻的.\n现在, 将棋盘上 $N$ 个方格作上标记, 使得棋盘上任何一个方格(作上标记的和未作上标记的) 都与至少一个作上标记的方格相邻.\n试确定 $N$ 的最小值.", + "solution": "解: $n \\times n$ 棋盘按如下方式染色: 如果 $n$ 不被 4 整除,则如图()染色,\n否则如图() 染色.\n考虑所有黑色方格, 若 $n=4 k$, 如图() 染色后, 共有 $4 \\times 3+4 \\times 7+\\cdots+4 \\times(4 k-1)=2 k(4 k+2)$ 个黑色方格; 若 $n=4 k+2$, 染色后, 共有 $4 \\times 1+4 \\times 5+\\cdots+4 \\times(4 k+1)=2(k+1)(4 k+2)$ 个黑色方格.\n不论哪种情形, 黑色方格都是 $\\frac{1}{2} n(n+2)$ 个, 而其中任意三个都不能与同一个方格相邻.\n而由条件, 它们中任意一个应与某个作了标记的方格相邻, 于是 $N \\geqslant \\frac{1}{4} n(n+2)$.\n另一方面, 我们证明, 可适当标记 $N=\\frac{1}{4} n(n+2)$ 个格, 使之合乎题目要求.\n事实上, 如图(), 我们将棋盘的 \"第一层边框\" 的 4(n-1) 个方格从左上角开始, 按顺时针方向依次编号为 $1,2, \\cdots, 4 n-4$, 将编号被 4 除余 $1 、 2$ 的方格作标记(图() 中阴影部分).\n去掉棋盘的外围两层边框, 对剩下的棋盘仍从左上角开始, 沿顺时针方向进行类似编号, 又将编号被 4 除余 $1 、 2$ 的方格作上标记, 如此下去, 直到此步骤不能再进行为止.\n这样, 我们恰对一半的黑色方格作了标记, 故共有 $\\frac{1}{4} n(n+2)$ 个方格作了标记.\n下面证明这种标记方法符合要求.\n事实上,如图() 不难看出任意两个标记方格不会有一个公共的\"邻格\", 假设这些标记的方格为 $A_1, \\cdots, A_N$, 其中 $N=\\frac{1}{4} n(n+2)$, 与 $A_i$ 相邻的方格集合为 $M_i$, 则 $M_i(i=1,2, \\cdots, N)$ 两两不交.\n且对位于棋盘角上的格 $A_i,\\left|M_i\\right|=2$ (共有 2 个这样的格); 对位于棋盘边上的格 $A_i,\\left|M_i\\right|=3$ (共有 $2 n-4$ 个这样的格); 对位于非棋盘边界上的格 $A_i,\\left|M_i\\right|=4$ (共有 $\\frac{n^2-6 n+8}{4}$ 个这样的格). 于是 $\\left|M_1 \\cup M_2 \\cup \\cdots \\cup M_N\\right|=\\frac{n^2-6 n+8}{4} \\times4+(2 n-4) \\times 3+2 \\times 2=n^2$, 故 $M_1 \\cup M_2 \\cup \\cdots \\cup M_N$ 包含了所有的方格, 即每个方格都与某一个标记方格相邻.\n综上所述, $N_{\\min }=\\frac{1}{4} n(n+2)$.", + "remark": "", + "figures": [ + "./images/volume13/figures/fig-c7i1.png", + "./images/volume13/figures/fig-c7i2.png", + "./images/volume13/figures/fig-c7i2.png", + "./images/volume13/figures/fig-c7i3.png", + "./images/volume13/figures/fig-c7i3.png", + "./images/volume13/figures/fig-c7i3.png" + ] +} \ No newline at end of file diff --git a/processed_dataset/proof/0564.json b/processed_dataset/proof/0564.json new file mode 100644 index 0000000000000000000000000000000000000000..3acb5b2f9746c7094d84607f036389e31689c681 --- /dev/null +++ b/processed_dataset/proof/0564.json @@ -0,0 +1,11 @@ +{ + "source_file": "./raw_volume-zh/volume13/chapter7.tex", + "problem_type": "proof", + "problem": "例8. 在 $x O y$ 平面上有 2002 个点, 它们组成一个点集 $S$, 已知 $S$ 中任意两点的连线都不与坐标轴平行.\n对 $S$ 中的任意两个点 $P 、 Q$, 考虑以 $P Q$ 为对角线, 其边平行于坐标轴的矩形 $M_{P Q}$, 用 $W_{P Q}$ 表示 $S$ 在矩形 $M_{P Q}$ 内 (不含 $P$ 、 $Q)$ 的点的个数.\n当命题: \" $S$ 中的点无论在坐标平面上如何分布, 在 $S$ 中至少有一对点 $P$ 、 $Q$, 满足 $W_{P Q} \\geqslant N$ \" 为真时, 求 $N$ 的最大值.", + "solution": "解:$N_{\\max }=400$. 先证明必有 $P 、 Q$ 使得 $W_{P Q} \\geqslant 400$. 事实上, 设 $A$ 是 $S$ 中纵坐标最大的点, $B$ 是 $S$ 中纵坐标最小的点, $C$ 是 $S$ 中横坐标最大的点, $D$ 是 $S$ 中横坐标最小的点.\n如果 $A 、 B 、 C 、 D$ 中有两点重合, 结论显然成立 (此时不妨设 $A=C$, 则 $M_{A B} 、 M_{B D} 、 M_{A D}$ 覆盖了 $S$, 从而 $\\max \\left\\{W_{A B}, W_{B D}, W_{A D}\\right\\} \\geqslant \\left.\\frac{2002-3}{3}>400\\right)$. 如果 $A 、 B 、 C 、 D$ 两两不重合, 则它们的分布如图() 所示.\n对于情形(1)(4), $M_{A C} 、 M_{B C} 、 M_{A D} 、 M_{B D}$ 覆盖了 $S$, 从而 $\\max \\left\\{W_{A C}, W_{B D}\\right.$, $\\left.W_{A D}, W_{B C}\\right\\} \\geqslant \\frac{2002-4}{4}>400$.\n对于情形(2)(3), $M_{A C} 、 M_{B C} 、 M_{A D} 、 M_{B D} 、 M_{A B}$ 覆盖了 $S$, 从而 $\\max \\left\\{W_{A C}\\right.$, $\\left.W_{B D}, W_{A D}, W_{B C}, W_{A B}\\right\\} \\geqslant \\frac{2002-4}{5}>399$, 从而 $\\max \\left\\{W_{A C}, W_{B D}, W_{A D}\\right.$, $\\left.W_{B C}, W_{A B}\\right\\} \\geqslant 400$. 于是必有 $P 、 Q$ 使得 $W_{P Q} \\geqslant 400$.\n下面证明存在这样的 $S$, 使得对所有 $P 、 Q \\in S$ 都有 $W_{P Q} \\leqslant 400$. 事实上, 如图() 所示, 将 $S$ 中的 2002 个点分成 5组 $E$ 、 $F 、 G 、 H 、 I$, 其中 $H 、 F$ 中各有 401 个点, 而 $E$ 、 $I 、 G$ 中各有 400 个点, 每组中的点都位于相应方格的对角线上.\n显然, 对任何一个矩形 $M_{P Q} (P 、 Q \\in S)$, 它至多含有一个组中的点.\n如果它包含的点属于组 $I$, 则 $W_{P Q} \\leqslant 400$. 如果矩形 $M_{P Q}$ 包含了 $I$ 外的其他组中的点, 则点 $P 、 Q$ 中至少有一个属于这个组, 于是 $W_{P Q} \\leqslant 401-1=400$.\n综上所述, $N_{\\text {max }}=400$.", + "remark": "", + "figures": [ + "./images/volume13/figures/fig-c7i4.png", + "./images/volume13/figures/fig-c7i5.png" + ] +} \ No newline at end of file diff --git a/processed_dataset/proof/0565.json b/processed_dataset/proof/0565.json new file mode 100644 index 0000000000000000000000000000000000000000..d647eb0adf8612aa71f73512acb24d92ebbe6340 --- /dev/null +++ b/processed_dataset/proof/0565.json @@ -0,0 +1,8 @@ +{ + "source_file": "./raw_volume-zh/volume13/chapter8.tex", + "problem_type": "proof", + "problem": "例1. 如果一个集合不包含满足 $x+y=z$ 的三个数 $x 、 y 、 z$, 则称之为单纯的.\n设 $M=\\{1,2, \\cdots, 2 n+1\\}, A$ 是 $M$ 的单纯子集, 求 $|A|$ 的最大值.", + "solution": "分析:虑到奇数十奇数 $\\neq$ 奇数, 于是很容易发现 $A=\\{1,3,5, \\cdots$, $2 n+1\\}$ 是合乎要求的集合, 此时 $|A|=n+1$. 我们猜想 $|A|$ 的最大值为 $n+$ 1 . 这就要证明: 若 $|A| \\geqslant n+2$, 则 $A$ 中必存在 3 个数 $x 、 y 、 z$, 使 $x+y=z$. 注意到 $x+y=z$, 等价于 $x=z-y$, 由此想到以 $A$ 中的元素作差构造 \"新元素\" 以利用抽庶原理.\n解取 $A=\\{1,3,5, \\cdots, 2 n+1\\}$, 则 $A$ 是单纯的, 此时 $|A|=n+1$. 下面证明: 对任何单纯子集 $A$, 有 $|A| \\leqslant n+1$. 用反证法.\n假设 $|A|>n+1$, 则 $A$ 中至少有 $n+2$ 个元素,设为: $a_1n$ 个正偶数.\n由抽庶原理, 必有两个元素相等, 且只能是某个 $b_i$ 与某个 $a_j-a_1$ 相等, 于是 $a_1+b_i=a_j$, 所以 $A$ 不是单纯的,矛盾.\n方法 2: 考察 $2 n+2$ 个元素: $a_21$, $(p q, b) \\geqslant q>1$. 又 $a 、 b$ 互质保证了 $p q$ 异于 $a 、 b$, 从而取 $c=p q$ 即可.\n当 $(a$, $b)=e>1$ 时, 取 $e$ 的最小素因子 $p$ 和不整除 $[a, b]$ 的最小素数 $q$. 显然 $p \\neq q$, 且 $p, q \\in\\{2,3,5,7,11\\}$, 于是 $p q \\in S$, 且 $(p q, a) \\geqslant(p, a)=p>1$, $(p q, b) \\geqslant(p, b)=p>1$. 又 $q$ 不整除 $[a, b]$ 保证了 $p q$ 异于 $a 、 b$, 从而取 $d=p q$ 即可.\n下面证明, 对任何满足条件的集合 $S$, 有 $|S| \\leqslant 72$.\n显然, $1 \\notin S$; 对于任意两个大于 10 的质数 $p 、 q$, 因为与 $p 、 q$ 都不互质的数最小是 $p q$, 而 $p q>100$, 所以根据条件 (3), 在 $[10,100]$ 中的 21 个质数 11 , $13, \\cdots, 89,97$ 最多有一个在 $S$ 中.\n记除 1 与这 21 个质数外的其余 78 个不超过 100 的正整数的集合为 $T$, 我们证明 $T$ 中至少有 7 个数不在 $S$ 中, 从而 $|S| \\leqslant 78-7+1=72$.\n实际上, 当有某个大于 10 的质数 $p \\in S$ 时, $S$ 中各数的最小素因子只可能是 $2 、 3 、 5 、 7$ 和 $p$. 结合条件 (2), 知: (i) 若 $7 p \\in S$, 因 $2 \\times 3 \\times 5 、 2^2 \\times 3 \\times 5$ 、 $2 \\times 3^2 \\times 5$ 与 $7 p$ 包括了所有的最小素因子, 所以由条件(2), $2 \\times 3 \\times 5 、 2^2 \\times 3 \\times 5 、 2 \\times 3^2 \\times 5 \\notin S$. 若 $7 p \\notin S$, 而 $2 \\times 7 p>100$, 且 $p \\in S$, 所以由条件(3) 知, $7 \\times 1 、 7 \\times 7 、 7 \\times 11 、 7 \\times 13 \\notin S$. (ii) 若 $5 p \\in S$, 则 $2 \\times 3 \\times 7 、 2^2 \\times 3 \\times 7 \\notin S$. 若 $5 p \\notin S$, 则 $5 \\times 1 、 5 \\times 5 \\notin S$. (iii) $2 \\times 5 \\times 7$ 与 $3 p$ 不同属于 $S$. (iv) $2 \\times 3 p$ 与 $5 \\times 7$ 不同属于 $S$. (v) 若 $5 p 、 7 p \\notin S$, 则 $5 \\times 7 \\notin S$. 于是, 当 $p=11$ 或 13 时,由 (i)、(ii)、(iii)、(iv) 可分别得出至少有 $3 、 2 、 1 、 1$ 个中的数不属于 $S$, 合计有 7 个.\n当 $p=17$ 或 19 时,由(i)、(ii)、(iii) 可分别得出至少有 $4 、 2 、 1$ 个 $T$ 中的数不属于 $S$, 合计有 7 个.\n当 $p>20$ 时, 由(i)、(ii)、(iii) 可分别得出至少有 $4 、 2 、 1$ 个 $T$ 中的数不属于 $S$, 合计有 7 个.\n所以结论成立.\n当大于 10 的质数 $p$ 都不属于 $S$ 时, $S$ 中各数的最小素因子只可能是 2 、 $3 、 5 、 7$. 此时, 7 个 2 元集合 $\\{3,2 \\times 5 \\times 7\\} 、\\{5,2 \\times 3 \\times 7\\} 、\\{7,2 \\times 3 \\times 5\\}$ 、 $\\{2 \\times 3,5 \\times 7\\} 、\\{2 \\times 5,3 \\times 7\\} 、\\{2 \\times 7,3 \\times 5\\} 、\\left\\{2^2 \\times 7,3^2 \\times 5\\right\\}$ 中的任何一个都至少有一个数不在 $S$ 中, 结论成立.\n综上所述, $|S|_{\\text {max }}=72$.", + "remark": "", + "figures": [] +} \ No newline at end of file diff --git a/processed_dataset/proof/0569.json b/processed_dataset/proof/0569.json new file mode 100644 index 0000000000000000000000000000000000000000..df383f3ea9d7ff7ace70861558ff33ae34221b95 --- /dev/null +++ b/processed_dataset/proof/0569.json @@ -0,0 +1,8 @@ +{ + "source_file": "./raw_volume-zh/volume13/chapter8.tex", + "problem_type": "proof", + "problem": "例7. 设 $a_i 、 b_i(i=1,2, \\cdots, n)$ 是有理数, 使得对任意的实数 $x$ 都有 $x^2+x+4=\\sum_{i=1}^n\\left(a_i x+b_i\\right)^2$, 求 $n$ 的最小可能值.", + "solution": "解:易发现 $n=5$ 是可以的, 实际上 $x^2+x+4=\\left(x+\\frac{1}{2}\\right)^2+\\left(\\frac{3}{2}\\right)^2+ 1^2+\\left(\\frac{1}{2}\\right)^2+\\left(\\frac{1}{2}\\right)^2$.\n由此可猜想 $n$ 的最小可能值为 5 , 这只需证明 $n \\neq 4$.\n用反证法, 假设 $x^2+x+4=\\sum_{i=1}^4\\left(a_i x+b_i\\right)^2, a_i, b_i \\in \\mathbf{Q}$, 则 $\\sum_{i=1}^4 a_i^2=1$, $\\sum_{i=1}^4 a_i b_i=\\frac{1}{2}, \\sum_{i=1}^4 b_i^2=4$.\n于是, $\\frac{15}{4}=\\left(\\sum_{i=1}^4 a_i^2\\right)\\left(\\sum_{i=1}^4 b_i^2\\right)-\\left(\\sum_{i=1}^4 a_i b_i\\right)^2$\n$$\n\\begin{aligned}\n= & \\left(-a_1 b_2+a_2 b_1-a_3 b_4+a_4 b_3\\right)^2+\\left(-a_1 b_3+a_3 b_1-\\right. \\\\\n& \\left.a_4 b_2+a_2 b_4\\right)^2+\\left(-a_1 b_4+a_4 b_1-a_2 b_3+a_3 b_2\\right)^2 .\n\\end{aligned}\n$$\n上式两边乘以 4 , 表明方程 $a^2+b^2+c^2=15 d^2=-d^2(\\bmod 8)$. 有解.\n不妨设 $a 、 b 、 c 、 d$ 中至少有一个奇数(否则方程两边约去公因数), 但 $a^2$, $b^2, c^2, d^2 \\equiv 0,1,4(\\bmod 8)$, 所以上式无解,矛盾.\n另解: $n=5$ 时, 有 $x^2+x+4=\\left(\\frac{1}{2} x+1\\right)^2+\\left(\\frac{1}{2} x+1\\right)^2+\\left(\\frac{1}{2} x-1\\right)^2+ \\left(\\frac{1}{2} x\\right)^2+(1)^2$.\n下证 $n \\neq 4$.\n如果 $x^2+x+4=\\sum_{i=1}^4\\left(a_i x+b_i\\right)^2$, 我们可以设 $a_i=\\frac{x_i}{2 m}, b_i=\\frac{y_i}{k}\\left(m k \\neq 0, m, k, x_i, y_i \\in \\mathbf{Z}\\right)$, \n则比较系数可以得到 $\\left\\{\\begin{array}{l}x_1^2+x_2^2+x_3^2+x_4^2=4 m^2, \\label{eq1}\\\\ y_1^2+y_2^2+y_3^2+y_4^2=4 n^2, \\label{eq2}\\\\ x_1 y_1+x_2 y_2+x_3 y_3+x_4 y_4=m n. \\label{eq3}\\end{array}\\right.$\n不妨设 $\\left(m 、 k 、 x_i 、 y_i\\right)$ 是满足 式\\ref{eq1},\\ref{eq2},式\\ref{eq3} 的使 $|m k|$ 非 0 且最小的一组.\n由 式\\ref{eq1} 知 $x_1^2+x_2^2+x_3^2+x_4^2 \\equiv 0(\\bmod 4)$, 由 $x^2 \\equiv 0$ 或 $1(\\bmod 4)$ 知诸 $x_i$ 同奇偶, 同理诸 $y_i$ 也同奇偶.\n因此 $x_i y_i(i=1 、 2 、 3 、 4)$ 同奇偶, 由 式\\ref{eq3} 知 $m m$ 为偶, 不妨设 $m$ 为偶, 则由 式\\ref{eq1} 知 $x_1^2+x_2^2+x_3^2+x_4^2 \\equiv 0(\\bmod 8)$.\n若诸 $x_i$ 同为奇, 则 $x_i^2 \\equiv 1(\\bmod 8)$, 故 $x_1^2+x_2^2+x_3^2+x_4^2 \\equiv 0(\\bmod 8)$, 矛盾.\n因此诸 $x_i$ 同偶, 故 $\\left(\\frac{m}{2} 、 k 、 \\frac{x_i}{2} 、 y_i\\right)$ 是满足 式\\ref{eq1},\\ref{eq2},式\\ref{eq3} 的使 $|m k|$ 非 0 且更小的一组解,矛盾.", + "remark": "", + "figures": [] +} \ No newline at end of file diff --git a/processed_dataset/proof/0570.json b/processed_dataset/proof/0570.json new file mode 100644 index 0000000000000000000000000000000000000000..7e4f196b4e9ec7658f758e24b7d3c14fdbab436c --- /dev/null +++ b/processed_dataset/proof/0570.json @@ -0,0 +1,8 @@ +{ + "source_file": "./raw_volume-zh/volume13/chapter8.tex", + "problem_type": "proof", + "problem": "例8. 若一个集合含有偶数个元素, 则称之为偶集.\n设 $M=\\{1,2, \\cdots$, $2011\\}$, 如果存在 $M$ 的 $k$ 个偶子集: $A_1, A_2, \\cdots, A_k$, 使对任何 $1 \\leqslant i200 .\n\\end{aligned}\n$$", + "remark": "", + "figures": [] +} \ No newline at end of file diff --git a/processed_dataset/proof/0573.json b/processed_dataset/proof/0573.json new file mode 100644 index 0000000000000000000000000000000000000000..a94579b948ef8e3706f04d5e6d77de67c2fc784d --- /dev/null +++ b/processed_dataset/proof/0573.json @@ -0,0 +1,8 @@ +{ + "source_file": "./raw_volume-zh/volume13/chapter9.tex", + "problem_type": "proof", + "problem": "例4. 一群童子军, 年龄是 7 到 13 的整数, 来自 11 个国家.\n求证: 至少有 5 个孩子, 对其中的任何一个孩子, 在童子军中与其同年龄的人多于同国籍的人.", + "solution": "证明:虑加权的元素关系表:\n\\begin{tabular}{|c|c|c|c|c|}\n\\hline & $A_1$ & $A_2$ & $\\cdots$ & $A_{11}$ \\\\\n\\hline 7 & $a_{7,1}$ & $a_{7,2}$ & $\\cdots$ & $a_{7,11}$ \\\\\n\\hline 8 & $a_{8,1}$ & $a_{8,2}$ & $\\cdots$ & $a_{8,11}$ \\\\\n\\hline$\\vdots$ & $\\vdots$ & $\\vdots$ & $\\vdots$ & $\\vdots$ \\\\\n\\hline 13 & $a_{13,1}$ & $a_{13,2}$ & $\\cdots$ & $a_{13,11}$ \\\\\n\\hline\n\\end{tabular}\n其中 $A_j$ 是来自第 $j$ 个国家的人的集合, $a_{i j}$ 是第 $j$ 国中年龄为 $i$ 岁的人数.\n令第 $i$ 行的和为 $r_i$, 第 $j$ 列的和为 $t_j$, 则\n$$\n\\begin{aligned}\n\\sum_{i=7}^{13} \\sum_{j=1}^{11} a_{i j}\\left(\\frac{1}{t_j}-\\frac{1}{r_i}\\right) & =\\sum_{i=7}^{13} \\sum_{j=1}^{11} \\frac{a_{i j}}{t_j}-\\sum_{i=7}^{13} \\sum_{j=1}^{11} \\frac{a_{i j}}{r_i} \\\\\n& =\\sum_{j=1}^{11} \\frac{1}{t_j} \\sum_{i=7}^{13} a_{i j}-\\sum_{i=7}^{13} \\frac{1}{r_i} \\sum_{j=1}^{11} a_{i j} \\\\\n& =\\sum_{j=1}^{11}\\left(\\frac{1}{t_j} \\cdot t_j\\right)-\\sum_{i=7}^{13}\\left(\\frac{1}{r_i} \\cdot r_i\\right) \\\\\n& =\\sum_{j=1}^{11} 1-\\sum_{i=7}^{13} 1=4 .\n\\end{aligned}\n$$\n由于 $\\frac{1}{t_j}-\\frac{1}{r_i}<1$, 将上式中 $a_{i j}\\left(\\frac{1}{t_j}-\\frac{1}{r_i}\\right)$ 看作是 $a_{i j}$ 个 \" $\\frac{1}{t_j}-\\frac{1}{r_i}$ \" 的和, 那么, 上式中至少有 5 个这样的 $\\frac{1}{t_j}-\\frac{1}{r_i}$ 为正, 从而至少有 5 个孩子合乎要求.", + "remark": "", + "figures": [] +} \ No newline at end of file diff --git a/processed_dataset/proof/0574.json b/processed_dataset/proof/0574.json new file mode 100644 index 0000000000000000000000000000000000000000..20c2ee981ff7d1ec8ca4174791c4eec1626ebf24 --- /dev/null +++ b/processed_dataset/proof/0574.json @@ -0,0 +1,8 @@ +{ + "source_file": "./raw_volume-zh/volume13/chapter9.tex", + "problem_type": "proof", + "problem": "例5. 设 $A=\\{1,2,3,4,5,6\\}, B=\\{7,8,9, \\cdots, n\\}$. 在 $A$ 中取 3 个数, 在 $B$ 中取 2 个数,组成含有 5 个元素的集合 $A_i(i=1,2, \\cdots, 20)$, 使得 $\\left|A_i \\cap A_j\\right| \\leqslant 2,1 \\leqslant i4)$ 次.\n考察含 $b$ 的 $k$ 个子集, 它们共含有 $A$ 中的 $3 k>12$ 个元素.\n于是, 由抽庶原理, $A$ 中至少有一个元素, 设为 $a$, 在这 $k$ 个子集中出现 3 次.\n设这 3 个同时含有 $a 、 b$ 的子集合为 $P 、 Q 、 R$, 则 $A \\backslash\\{a\\}$ 中的 5 个元素在 $P 、 Q 、R$ 中共出现 $2 \\times 3=6$ 次.\n于是必有一个元素 $c$ 出现 2 次,这样便得到 2 个同时含有 $a 、 b 、 c$ 的子集,与条件 $\\left|A_i \\cap A_j\\right| \\leqslant 2$ 矛盾.\n由上, $B$ 中每个元素在各个子集 $A_i(i=1,2, \\cdots, 20)$ 中至多出现 4 次, 而 $B$ 中元素在各个子集 $A_i(i=1,2, \\cdots, 20)$ 中出现的总次数是 $2 \\times 20=40$, 于是 $|B| \\geqslant \\frac{40}{4}=10$, 所以 $n \\geqslant 10+6=16$.\n最后, 当 $n=16$ 时,存在合乎题目条件的 20 个集合: $\\{1,2,3,7,8\\}$ 、 $\\{1,2,4,12,14\\} 、\\{1,2,5,15,16\\} 、\\{1,2,6,9,10\\} 、\\{1,3,4,10,11\\}$ 、 $\\{1,3,5,13,14\\} 、\\{1,3,6,12,15\\} 、\\{1,4,5,7,9\\} 、\\{1,4,6,13,16\\}$ 、 $\\{1,5,6,8,11\\} 、\\{2,3,4,13,15\\} 、\\{2,3,5,9,11\\} 、\\{2,3,6,14,16\\}$ 、 $\\{2,4,5,8,10\\} 、\\{2,4,6,7,11\\} 、\\{2,5,6,12,13\\} 、\\{3,4,5,12,16\\}$ 、 $\\{3,4,6,8,9\\} 、\\{3,5,6,7,10\\} 、\\{4,5,6,14,15\\}$.\n综上所述, $n$ 的最小值是 16 .", + "remark": "", + "figures": [] +} \ No newline at end of file diff --git a/processed_dataset/proof/0575.json b/processed_dataset/proof/0575.json new file mode 100644 index 0000000000000000000000000000000000000000..5b664ed016ec1ff5db10031f061aa7d1f61a427e --- /dev/null +++ b/processed_dataset/proof/0575.json @@ -0,0 +1,8 @@ +{ + "source_file": "./raw_volume-zh/volume13/chapter9.tex", + "problem_type": "proof", + "problem": "例6. 设 $n$ 是给定的正整数, $6 \\mid n$. 在 $n \\times n$ 方格棋盘中, 每个方格都填上一个正整数,第 $i$ 行的方格填人的数从左至右依次为 $(i-1) n+1,(i-1) n+ 2, \\cdots,(i-1) n+n$. 今任取 2 个相邻 (具有公共边) 的方格, 将其中一个数加 1 , 另一个数加 2 , 称之为一次操作.\n问: 至少要经过多少次操作, 才能使棋盘中的数变得都相等?", + "solution": "解:棋盘中各数的和记为 $S$.\n显然, 当棋盘中的数都相等时, 每个数至少是 $n^2$, 于是棋盘中各数的和 $S^{\\prime} \\geqslant n^2 \\cdot n^2=n^4$, 又每次操作使 $S$ 增加 3, 而最初棋盘中 $S_0=1+2+3+\\cdots +n^2=\\frac{n^2\\left(n^2+1\\right)}{2}$, 于是操作到各数相等时 $S$ 至少增加 $n^4-\\frac{n^2\\left(n^2+1\\right)}{2}= \\frac{n^2\\left(n^2-1\\right)}{2}$, 所以操作次数不少于 $\\frac{1}{3} \\cdot \\frac{n^2\\left(n^2-1\\right)}{2}=\\frac{n^2\\left(n^2-1\\right)}{6}$.\n下面证明: 可适当操作 $\\frac{n^2\\left(n^2-1\\right)}{6}$ 次,使棋盘中的数变得都相等.\n这等价于证明: 可适当操作若干次,使棋盘中的数都变成 $n^2$.\n将棋盘每一行从左至右每 3 个连续格一组, 分成 $\\frac{n}{3}$ 组, 依次记为第一组, 第二组, $\\cdots$, 第 $\\frac{n}{3}$ 组.\n首先注意到任何连续 3 个格, 都可适当操作 2 次, 使每个数都增加 2 , 我们称这两个操作合成一个大操作 $A:(a, b, c) \\rightarrow(a+2, b+1, c) \\rightarrow(a+2$, $b+2, c+2)$. 现在, 将某行的每一组都进行 $\\frac{n}{2}$ 次大操作 $A$, 则该行的数都增加\n$n$,于是, 对每一行, 都可进行若干次操作, 使其变得与第 $n$ 行完全相同.\n再注意到连续 2 个格, 可适当操作两次, 使其中的数都增加 3 , 我们称这两个操作合成一个大操作 $B:(a, b) \\rightarrow(a+2, b+1) \\rightarrow(a+3, b+3)$. 现在, 假定棋盘的每一行都已与第 $n$ 行完全相同.\n再考察第 $i 、 i+1$ 这 2 行的第 $j$ 组, 对该组同一列的 2 个格都进行一次大操作 $B$, 则这 2 行第 $j$ 组中的数都增加 3 , 而同一行中不同组对应的数相差 3 的倍数,于是, 我们可对这 2 行每一组都进行类似的操作若干次, 使这 2 行的所有组都变得与第 $\\frac{n}{3}$ 组完全相同, 为 $\\left(n^2-\\right. \\left.2, n^2-1, n^2\\right)$, 它经过一次操作可变为 $\\left(n^2, n^2, n^2\\right)$, 于是这 2 行都可变成 $n^2$.\n每连续 2 行都进行这样的操作, 则所有行都变成 $n^2$.\n综上所述, 操作的最小次数为 $\\frac{n^2\\left(n^2-1\\right)}{6}$.", + "remark": "", + "figures": [] +} \ No newline at end of file diff --git a/processed_dataset/proof/0576.json b/processed_dataset/proof/0576.json new file mode 100644 index 0000000000000000000000000000000000000000..5534e8d5caa95de4682068f5799554c27cb4e56c --- /dev/null +++ b/processed_dataset/proof/0576.json @@ -0,0 +1,8 @@ +{ + "source_file": "./raw_volume-zh/volume13/chapter9.tex", + "problem_type": "proof", + "problem": "例8. 给定正整数 $n(n \\geqslant 2)$, 求最大的 $\\lambda$, 使得: 若有 $n$ 个袋子,每一个袋子中都是一些重量为 2 的整数次幕克的小球, 且各个袋子中的小球的总重量都相等 (同一个袋子中可以有相等重量的小球), 则必有某一重量的小球的总个数至少为 $\\lambda$.", + "solution": "解:不妨设最重的小球的重量为 1 , 设每个袋子中小球的总重量为 $G$, 则 $G \\geqslant 1$.\n先证明: 当 $\\lambda=\\left[\\frac{n}{2}\\right]+1$ 合乎题意, 即必有某一重量的小球的总个数至少为 $\\left[\\frac{n}{2}\\right]+1$.\n反设任一个重量的小球的总个数都不大于 $\\left[\\frac{n}{2}\\right]$, 考察这 $n$ 个袋子中所有小球的总重量, 有 $n \\leqslant n G<\\left[\\frac{n}{2}\\right] \\cdot\\left(1+2^{-1}+2^{-2}+\\cdots\\right)=2\\left[\\frac{n}{2}\\right] \\leqslant 2 \\cdot \\frac{n}{2}=n$, 矛盾!\n其次证明: $\\lambda \\leqslant\\left[\\frac{n}{2}\\right]+1$. 取充分大的正整数 $s$, 使得 $2-2^{-s} \\geqslant \\frac{2 n}{n+1}$, 由于 $\\left[\\frac{n}{2}\\right]+1 \\geqslant \\frac{n+1}{2}$, 所以 $2-2^{-s} \\geqslant \\frac{2 n}{n+1} \\geqslant \\frac{n}{\\left[\\frac{n}{2}\\right]+1}$, 从而 $\\left(\\left[\\frac{n}{2}\\right]+1\\right)\\left(1+2^{-1}+\\cdots+\\right. \\left.2^{-s}\\right) \\geqslant n \\cdot 1$.\n所以可在 $\\underbrace{1,1, \\cdots, 1}_{\\left[\\frac{n}{2}\\right]+1}, \\underbrace{2^{-1}, 2^{-1}, \\cdots, 2^{-1}}_{\\left[\\frac{n}{2}\\right]+1}, \\cdots, \\underbrace{2^{-s}, 2^{-s}, \\cdots, 2^{-s}}_{\\left[\\frac{n}{2}\\right]+1}$ 中从前至后取出和为 1 的连续若干项, 且至少可取 $n$ 次, 所以 $\\lambda \\leqslant\\left[\\frac{n}{2}\\right]+1$.\n综上可知, $\\lambda_{\\text {max }}=\\left[\\frac{n}{2}\\right]+1$.", + "remark": "", + "figures": [] +} \ No newline at end of file diff --git a/processed_dataset/proof/0577.json b/processed_dataset/proof/0577.json new file mode 100644 index 0000000000000000000000000000000000000000..74c792ad633f71e0944ce032e6f35b527f3dc00f --- /dev/null +++ b/processed_dataset/proof/0577.json @@ -0,0 +1,8 @@ +{ + "source_file": "./raw_volume-zh/volume13/exercise1.tex", + "problem_type": "proof", + "problem": "问题5. 设 $x_1, x_2, \\cdots, x_{1990}$ 是 $1,2, \\cdots, 1990$ 的一个排列, 求 $F=|\\cdots| \\mid x_1- x_2\\left|-x_3\\right|-\\cdots\\left|-x_{1990}\\right|$ 的最大值.", + "solution": "$F_2=|1-2|=1 \\leqslant 2, F_3=|| 1-2|-3|=2 \\leqslant 3, F_4=|| \\mid 1- 2|-3|-4 \\mid=4 \\leqslant 4$. 一般地, 猜想 $F_n \\leqslant n$. 下用数学归纳法证明.\n首先注意到 $x 、 y>0$ 时, $|x-y| \\leqslant \\max \\{x, y\\}$. 所以 $\\left|x_1-x_2\\right| \\leqslant \\max \\left\\{x_1, x_2\\right\\}$, ||$x_1-x_2\\left|-x_3\\right| \\leqslant \\max \\left\\{\\max \\left\\{x_1, x_2\\right\\}, x_3\\right\\}=\\max \\left\\{x_1, x_2, x_3\\right\\}$. 设 $|\\cdots|\\left|x_1-x_2\\right|-x_3|-\\cdots|-x_k \\mid \\leqslant \\max \\left\\{x_1, x_2, x_3, \\cdots, x_k\\right\\}$, 则 $|\\cdots| \\mid x_1- x_2\\left|-x_3\\right|-\\cdots\\left|-x_{k+1}\\right| \\leqslant \\max \\left\\{\\max \\left\\{x_1, x_2, x_3, \\cdots, x_k\\right\\}, x_{k+1}\\right\\}=\\max \\left\\{x_1, x_2, x_3, \\cdots, x_k, x_{k+1}\\right\\} \\label{eq1}$, 所以 $F=||\\left|x_1-x_2\\right|-x_3|-\\cdots|-x_{1990} \\mid \\leqslant \\max \\left\\{x_1, x_2, x_3, \\cdots, x_{1990}\\right\\}=1990$. 由初值可知, 上述不等式 \\ref{eq1} 的等号不一定成立.\n实际上, $k=1990$ 时不成立等号.\n因为去掉绝对值符号和改变项的正负符号, 代数式 $F$ 的值的奇偶性不变, 所以 $F \\equiv x_1+x_2+x_3+\\cdots+x_{1990}=1+ 2+\\cdots+1990 \\equiv 1(\\bmod 2) .1990 \\equiv 0(\\bmod 2)$, 所以 $F \\leqslant 1989$. 下面构造 $x_1$, $x_2, x_3, \\cdots, x_{1990}$, 使 $F=1989$. 我们的策略是使 \"和\"中尽可能产生 0 . 注意到要使差最大,必须是最大的减最小的,即 $1990-1=1989$. 于是希望其他 1988 个数相互抵消.\n考察 4 个连续自然数 $n+1 、 n+2 、 n+3 、 n+4$, 我们有 || $\\mid n+ 3-(n+1)|-(n+4)|-(n+2) \\mid=0$. 将 $x_1, x_2, x_3, \\cdots, x_{1988}$ 按相连 4 个数一组, 分为 497 组, 第 $k$ 组为: $\\left(x_{4 k+1}, x_{4 k+2}, x_{4 k+3}, x_{4 k+4}\\right)(k=0,1$, $2, \\cdots, 496)$. 令 $\\left(x_{4 k+1}, x_{4 k+2}, x_{4 k+3}, x_{4 k+4}\\right)=(4 k+2,4 k+4,4 k+5,4 k+ 3)$, 则 ||$\\left|x_{4 k+1}-x_{4 k+2}\\right|-x_{4 k+3}\\left|-x_{4 k+4}\\right|=|||(4 k+2)-(4 k+4)|-(4 k+ 5)|-(4 k+3)|=0$. 于是, 再令 $x_{1989}=1990, x_{1990}=1$, 则 $F=\\mid 1990- 1 \\mid=1989$.", + "remark": "", + "figures": [] +} \ No newline at end of file diff --git a/processed_dataset/proof/0578.json b/processed_dataset/proof/0578.json new file mode 100644 index 0000000000000000000000000000000000000000..b6676bdba4823b9b1c8665108ea73fea86dc6f8c --- /dev/null +++ b/processed_dataset/proof/0578.json @@ -0,0 +1,8 @@ +{ + "source_file": "./raw_volume-zh/volume13/exercise10.tex", + "problem_type": "proof", + "problem": "问题4. 对每个正整数 $n$, 用 $s(n)$ 表示满足下列条件的最大整数: 对任何正整数 $k \\leqslant s(n), n^2$ 可以表成 $k$ 个正整数的平方和.\n(1) 求证: $s(n) \\leqslant n^2- 14(n \\geqslant 4)$. (2) 找出一个 $n$, 使 $s(n)=n^2-14$. (3) 求证: 存在无数个 $n$, 使 $s(n)=n^2-14$.", + "solution": "(1) 反设 $n^2=a_1^2+a_2^2+\\cdots+a_k^2$, 其中 $k=n^2-13$, 不妨设 $a_1 \\leqslant a_2 \\leqslant a_3 \\leqslant \\cdots \\leqslant a_k$, 则 $a_k^2=n^2-\\left(a_1^2+a_2^2+\\cdots+a_{k-1}^2\\right) \\leqslant n^2-(k-1)=n^2-\\left(n^2-\\right. 14)=14$. 所以 $a_k \\leqslant 3$. 不妨设 $a_1, a_2, \\cdots, a_k$ 中有 $i$ 个为 $1 、 j$ 个为 $2 、 t$ 个为 3 , 其中 $i+j+t=k=n^2-13$, 那么 $n^2=\\left(a_1^2+a_2^2+\\cdots+a_k^2\\right)=i+4 j+9 t= \\left(n^2-13\\right)+3 j+8 t$, 所以 $3 j+8 t=13,8 t=13-3 j \\leqslant 13$, 所以 $t \\leqslant 1$. 当 $t=$ 0 时, $3 j=13$, 矛盾.\n当 $t=1$ 时, $3 j=5$, 亦矛盾.\n所以, 当 $s(n) \\geqslant n^2-13$ 时, 都存在 $k=n^2-13 \\leqslant s(n)$, 使 $n^2$ 不能表成 $k$ 个正整数的平方和.\n所以 $s(n) \\leqslant n^2-14$. \n(2) 若 $s(n)=n^2-14$, 则对任何自然数 $k \\leqslant n^2-14$, 正整数 $n$ 都可表成 $k$ 个正整数的平方和.\n考察其中的任意一个正整数 $k \\leqslant n^2-14$, 令 $k=n^2- r\\left(14 \\leqslant r \\leqslant n^2-1\\right)$, 我们要找到一个 $n$, 使 $n^2$ 存在相应分拆: $n^2=a_1^2+ a_2^2+\\cdots+a_k^2$ (其中 $a_1 \\leqslant a_2 \\leqslant a_3 \\leqslant \\cdots \\leqslant a_k$ ). 引人参数: 不妨设 $a_1, a_2, \\cdots, a_k$ 中有 $i$ 个为 $1, j$ 个为 $2 、 t$ 个为 3 , 其中 $i+j+t=k=n^2-r$. 我们记此 $k-$ 分拆为 $k(i, j, t)$, 那么, $n^2=\\left(a_1^2+a_2^2+\\cdots+a_k^2\\right)=i+4 j+9 t=\\left(n^2-r\\right)+ 3 j+8 t$, 所以 $3 j+8 t=r . \\label{eq1}$. 使 $n$ 合乎要求的一个必要条件是, 对所有 $14 \\leqslant r \\leqslant n^2-1$, 方程 式\\ref{eq1} 有非负整数解.\n实际上, 若 $r \\equiv 1(\\bmod 3)$, 令 $r=3 r_1+ 1\\left(r_1 \\geqslant 5\\right)$, 此时, $(j, t)=\\left(r_1-3,1\\right)$ 是 式\\ref{eq1} 的解.\n若 $r \\equiv 2(\\bmod 3)$, 令 $r=3 r_1+ 2\\left(r_1 \\geqslant 4\\right)$, 此时, $(j, t)=\\left(r_1-2,1\\right)$ 是 式\\ref{eq1} 的解.\n若 $r \\equiv 0(\\bmod 3)$, 令 $r=3 r_1 \\left(r_1 \\geqslant 5\\right)$, 此时, $(j, t)=\\left(r_1, 0\\right)$ 是 式\\ref{eq1} 的解.\n注意到 $k=n^2-r$, 所以 式\\ref{eq1} 等价于 $3 j+8 t=n^2-k .\\label{eq2}$ . 于是, $n^2$ 存在 $k(i, j, t)$ 分拆, 则 $j 、 t 、 k$ 满足 式\\ref{eq2}. 注意 式\\ref{eq2} 中不含对 $i$ 的要求, 所以 $k 、 i 、 j 、 t$ 还要满足: $i=k-(j+t) \\geqslant 0$, 即 $k \\geqslant j+t$. 于是, 对任何 $n$, 只要分拆的项数 $k$ 不小于 $j+t$, 其中 $j 、 t$ 满足 式\\ref{eq2}, 则 $n$ 存在相应的 $k$ 一分拆 $k(i, j, t)$. 注意到 $\\left(n^2-k\\right)=3 j+8 t \\geqslant 3(j+t)$, 从而使 $k \\geqslant j+t$ 的一个充分条件是 $k \\geqslant \\frac{n^2-k}{3}$, 即 $k \\geqslant \\frac{n^2}{4}$. 于是, 当 $k \\geqslant \\frac{n^2}{4}$ 时, 对所有 $n$, 都存在相应的 $k(i, j, t)$ 分拆.\n若分拆的项数 $k<\\frac{n^2}{4}$, 则 $n$ 不一定存在 $k (i, j, t)$ 分拆.\n此时应立足于找其他形式的分拆.\n若 $k<\\frac{n^2}{4}$, 则当 $n$ 较小时, 分拆的项数的可能取值较少.\n于是, 可从较小的自然数 $n$ 开始一一验证.\n为了便于利用勾股数, 可取 $3 、 4 、 5 、 6 、 8 、 10 、 12$, 这些都难于分解.\n取 $n=13$, 有 $13^2=12^2+5^2=12^2+\\left(3^2+4^2\\right)=8^2+8^2+5^2+4^2$. 又 $8^2=4^2+4^2+4^2+ 4^2, 4^2=2^2+2^2+2^2+2^2, 2^2=1^2+1^2+1^2+1^2$, 所以 $13^2$ 可以进行 7,10 , $13, \\cdots, 43$ 分拆.\n又 $5^2=4^2+3^2$, 于是 $13^2$ 又可进行 $5,8,11, \\cdots, 44$ 分拆.\n再由 $12^2=6^2+6^2+6^2+6^2, 6^2=3^2+3^2+3^2+3^2, 4^2=2^2+2^2+2^2+2^2$, $2^2=1^2+1^2+1^2+1^2$, 可知, $13^2$ 可以进行 $3,6,9, \\cdots, 33$ 分拆.\n最后, $13^2= 3^2+3^2+\\cdots+3^2+4^2, 3^2=2^2+2^2+1^2$, 于是选择其中 9 个或 12 个 $3^2$ 拆开, 则 $13^2$ 又可进行 $18+2 \\times 9=36 、 18+2 \\times 12=42$ 分拆.\n于是, $13^2$ 可进行 1 , $2, \\cdots, 44$ 分拆.\n而对 $k \\geqslant 45$, 有 $k \\geqslant \\frac{13^2}{4}$, 所以 $13^2$ 可以进行 $k(i, j, t)$ 分拆.\n故 $s(13)=13^2-14$. \n(3) 我们证明当 $n=2^m \\times 13$ 时, $s(n)=n^2-14$. 实际上, $n^2=\\left(2^m \\times 13\\right)^2=4^4\\left(2^{m-t} \\times 13\\right)^2(0 \\leqslant t \\leqslant m)$. 因为 $13^2$ 可以进行 $1,2,3, \\cdots$, 分拆, 所以 $n^2$ 可以进行 $1,2,3, \\cdots, 4^m \\times 155$ 分拆.\n但 $4^m \\times 155> \\frac{\\left(2^m \\times 13\\right)^2}{4}=\\frac{n^2}{4}$, 由前面的讨论, $n^2$ 可以进行 $1,2,3, \\cdots, n^2-14$ 分拆, 故 $s\\left(2^m \\times 13\\right)=\\left(2^m \\times 13\\right)^2-14$.", + "remark": "", + "figures": [] +} \ No newline at end of file diff --git a/processed_dataset/proof/0579.json b/processed_dataset/proof/0579.json new file mode 100644 index 0000000000000000000000000000000000000000..11c2b7f0709c7e9a9f0035773cc19c86d00e1f59 --- /dev/null +++ b/processed_dataset/proof/0579.json @@ -0,0 +1,8 @@ +{ + "source_file": "./raw_volume-zh/volume13/exercise10.tex", + "problem_type": "proof", + "problem": "问题5. 实数 $a_1, a_2, \\cdots, a_n(n>3)$ 满足: $a_1+a_2+\\cdots+a_n \\geqslant n$, 且 $a_1^2+a_2^2+\\cdots+ a_n^2 \\geqslant n^2$. 求 $\\max \\left\\{a_1, a_2, \\cdots, a_n\\right\\}$ 的最小值.", + "solution": "取 $n=4, a_1=a_2=a_3=a_4=2$, 则 $a_1+a_2+a_3+a_4=8 \\geqslant 4, a_1^2+ a_2^2+a_3^2+a_n^2=16 \\geqslant 4^2$. 所以 $a_1 、 a_2 、 a_3 、 a_4$ 合乎题目条件, 此时 $\\max \\left\\{a_1\\right.$, $\\left.a_2, \\cdots, a_n\\right\\}=2$. 下面证明, 对任何满足 $a_1+a_2+\\cdots+a_n \\geqslant n$, 且 $a_1^2+a_2^2+\\cdots+ a_n^2 \\geqslant n^2$ 的实数 $a_1, a_2, \\cdots, a_n(n>3)$, 有 $\\max \\left\\{a_1, a_2, \\cdots, a_n\\right\\} \\geqslant 2$. 假设 $\\max \\left\\{a_1, a_2, \\cdots, a_n\\right\\}<2$, 并设 $a_1, a_2, \\cdots, a_n$ 中有 $i$ 个非负数, 记为 $x_1$, $x_2, \\cdots, x_i$, 有 $j$ 个负数, 记为 $-y_1,-y_2, \\cdots,-y_j$, 其中 $y_1, y_2, \\cdots, y_j>0$, $i>0, j \\geqslant 0, i+j=n$. 那么, $\\max \\left\\{x_1, x_2, \\cdots, x_i\\right\\}<2$. 因为 $x_1+x_2+\\cdots+ x_i+\\left[\\left(-y_1\\right)+\\left(-y_2\\right)+\\cdots+\\left(-y_j\\right)\\right] \\geqslant n$, 所以 $x_1+x_2+\\cdots+x_i \\geqslant n+ y_1+y_2+\\cdots+y_j$. 又 $\\max \\left\\{x_1, x_2, \\cdots, x_i\\right\\}<2, y_1, y_2, \\cdots, y_j>0$, 于是 $2 i= 2+2+\\cdots+2>x_1+x_2+\\cdots+x_i \\geqslant n+y_1+y_2+\\cdots+y_j=i+j+y_1+ y_2+\\cdots+y_j$. 移项得 $i-j>y_1+y_2+\\cdots+y_j$. 因为 $x_1^2+x_2^2+\\cdots+x_i^2+\\left(-y_1\\right)^2+ \\left(-y_2\\right)^2+\\cdots+\\left(-y_j\\right)^2 \\geqslant n^2$, 所以 $x_1^2+x_2^2+\\cdots+x_i^2 \\geqslant n^2-\\left(y_1^2+y_2^2+\\cdots+\\right. \\left.y_j^2\\right) \\geqslant n^2-\\left(y_1+y_2+\\cdots+y_j\\right)^2>n^2-(i-j)^2=(i+j)^2-(i-j)^2=4 i j$. 又 $i>0$, 所以 $j<1$, 即 $j=0$, 故 $a_1, a_2, \\cdots, a_n$ 都是非负数, 所以 $0 \\leqslant a_i< 2(i=1,2, \\cdots, n)$, 所以 $4 n>a_1^2+a_2^2+\\cdots+a_n^2 \\geqslant n^2$, 解得 $n<4$, 与条件 $n>3$ 矛盾.\n综上所述, $\\max \\left\\{a_1, a_2, \\cdots, a_n\\right\\}=2$.", + "remark": "", + "figures": [] +} \ No newline at end of file diff --git a/processed_dataset/proof/0580.json b/processed_dataset/proof/0580.json new file mode 100644 index 0000000000000000000000000000000000000000..e0c8d537daf07b1800ea3db756a0d31cd0ea6bcd --- /dev/null +++ b/processed_dataset/proof/0580.json @@ -0,0 +1,8 @@ +{ + "source_file": "./raw_volume-zh/volume13/exercise11.tex", + "problem_type": "proof", + "problem": "问题1. 1650 个学生排成 22 行、75 列.\n已知其中任意两列处于同一行的两个人中, 性别相同的学生都不超过 11 对.\n证明: 男生的个数不超过 928.", + "solution": "设第 $i$ 行的男生数为 $a_i$, 则女生数为 $75-a_i$. 依题意, 可知, $\\sum_{i=1}^{22}\\left(\\mathrm{C}_{a_i}^2+\\right. \\left.\\mathrm{C}_{75-a_i}^2\\right) \\leqslant 11 \\times \\mathrm{C}_{75}^2$. 这是因为任意给定的两列处于同一行的两个人中, 性别相同的学生不超过 11 对, 故所有性别相同的两人对的个数不大于 $11 \\times \\mathrm{C}_{75}^2$. 于是, 我们有 $\\sum_{i=1}^{22}\\left(a_i^2-75 a_i\\right) \\leqslant-30525$, 即 $\\sum_{i=1}^{22}\\left(2 a_i-75\\right)^2 \\leqslant 1650$. 利用柯西不等式, 可知 $\\left[\\sum_{i=1}^{22}\\left(2 a_i-75\\right)\\right]^2 \\leqslant 22 \\sum_{i=1}^{22}\\left(2 a_i-75\\right)^2 \\leqslant 36300$, 因此, $\\sum_{i=1}^{22}\\left(2 a_i-75\\right)<$ 191 , 从而 $\\sum_{i=1}^{22} a_i<\\frac{191+1650}{2}<921$. 所以,男生的个数不超过 928 .", + "remark": "", + "figures": [] +} \ No newline at end of file diff --git a/processed_dataset/proof/0581.json b/processed_dataset/proof/0581.json new file mode 100644 index 0000000000000000000000000000000000000000..a27ce44c71d2dce2b4956cc4e7751773365d13a3 --- /dev/null +++ b/processed_dataset/proof/0581.json @@ -0,0 +1,8 @@ +{ + "source_file": "./raw_volume-zh/volume13/exercise11.tex", + "problem_type": "proof", + "problem": "问题2. 一次会议有 $12 k$ 个人参加, 每人恰与其中 $3 k+6$ 个人打过招呼.\n对任意两人, 与他们打过招呼的人数都相等, 问: 此次会议有多少个人参加?", + "solution": "用点表示人, 对打过招呼的两个人, 对应的两个点连线, 得到一个简单图.\n计算此图中角的个数 $S$. 一方面, 对 $G$ 计算, 每个点引出 $3 k+6$ 条边, 有 $\\mathrm{C}_{3 k+6}^2$ 个角.\n$12 k$ 个点, 共有 $12 k \\mathrm{C}_{3 k+6}^2$ 个角.\n又每个角都有顶点, 且不同的顶点对应的角不同, 于是 $S=12 k \\mathrm{C}_{3 k+6}^2$. 另一方面, 从 \" 2 人组\"出发, 因为对任意两人,与他们打过招呼的人数都相等.\n设这样的人数为 $t$, 则每个 2 人组都得到 $t$ 个角.\n又每个角都对应一个 2 人组, 且不同的 2 人组对应的角不同, 于是 $S= t \\mathrm{C}_{12 k}^2$. 所以 $12 k \\mathrm{C}_{3 k+6}^2=t \\mathrm{C}_{12 k}^2$, 解得 $t=\\frac{(3 k+6)(3 k+5)}{12 k-1}$. 所以 $16 t= \\frac{(12 k-1+25)(12 k-1+21)}{12 k-1}=(12 k-1)+25+21+\\frac{25 \\times 21}{12 k-1}$. 显然,(3, $12 k-1)=1$, 所以 $12 k-1 \\mid 25 \\times 7$. 注意到 $12 k-1$ 模 4 余 3 , 所以 $12 k-1=7$, $5 \\times 7,5^2 \\times 7$, 其中只有 $12 k-1=5 \\times 7$ 有整数解 $k=3 、 t=6$, 所以会议的人数只可能是 36 . 下面证明: 36 人的会议是可能的.\n即存在 36 阶图 $G, G$ 中每个点的度是 15 , 并且对每一对点, 同时与它们相连的点都有 6 个.\n先作 6 个完全图 $K_6$. 每个完全图的顶点都用 $1 、 2 、 3 、 4 、 5 、 6$ 编号.\n现在将这 6 个完全图的有关点用边联接, 构成图 $G . G$ 中的顶点记为 $(i, j)$, 它表示第 $i$ 个完全图中的第 $j$ 个顶点.\n注意到每个点已与所在完全图中的 5 个点相连, 现将每个点再与纵坐标相同的点相连, 则每个点又连了 5 条边.\n再将坐标差相同的点相连, 即 $i-j=i^{\\prime}-j^{\\prime}$, 则 $(i, j)$ 与 $\\left(i^{\\prime}, j^{\\prime}\\right)$ 相连.\n这样, 每个点又引出了 5 条边.\n所以每个点都连了 15 条边且每个点向它不在的完全图都有 2 点相连, 其中一个点与它的纵坐标相同, 另一个点与它的坐标差相同.\n对任意两个点, 若某个点与它们都相连, 则称这两个点对了一个角.\n下面证明 $G$ 中的任何两个点都对了 6 个角.\n实际上, 考察任意的两个点 $(i, j) 、\\left(i^{\\prime}, j^{\\prime}\\right)$, 若 $i=i^{\\prime}$, 即这两个点在同一个完全图中, 于是它们在该完全图中对了 4 个角.\n此外, 恰有两个点: $\\left(i^{\\prime}-\\right. \\left.j^{\\prime}+j, j\\right)$ 和 $\\left(i-j+j^{\\prime}, j^{\\prime}\\right)$ 这两个点与它们相连, 所以它们共对了 6 个角.\n若 $i \\neq i^{\\prime}$, 且 $i-j=i^{\\prime}-j^{\\prime}$, 则 $\\left(i, j^{\\prime}\\right)$ 和 $\\left(i^{\\prime}, j\\right)$ 与它们都相连.\n此外, 对 $i, i^{\\prime}$ 以外的 4 个 $i^{\\prime \\prime}$, 点 $\\left(i^{\\prime \\prime}, i^{\\prime \\prime}-i+j\\right)$ 与它们都相连, 其他点都不同时与它们相连.\n所以它们对了 6 个角.\n若 $i \\neq i^{\\prime}$, 且 $i-j \\neq i^{\\prime}-j^{\\prime}$, 则 $\\left(i, j^{\\prime}\\right) 、\\left(i^{\\prime}, j\\right) 、\\left(i, i-i^{\\prime}+\\right. \\left.j^{\\prime}\\right)$ 和 $\\left(i^{\\prime}, i^{\\prime}-i+j\\right)$ 与它们都相连.\n此外, 还有两点 $\\left(i^{\\prime}-j^{\\prime}+j, j\\right) 、(i-j+ \\left.j^{\\prime}, j^{\\prime}\\right)$ 与它们都相连, 其他点都不同时与它们相连.\n所以它们对了 6 个角.\n综上述,参加会议的人数为 36 .", + "remark": "", + "figures": [] +} \ No newline at end of file diff --git a/processed_dataset/proof/0582.json b/processed_dataset/proof/0582.json new file mode 100644 index 0000000000000000000000000000000000000000..bdac3e78e5e1a505212dbef8ead736783f159e01 --- /dev/null +++ b/processed_dataset/proof/0582.json @@ -0,0 +1,8 @@ +{ + "source_file": "./raw_volume-zh/volume13/exercise11.tex", + "problem_type": "proof", + "problem": "问题5. 若干学生参加考试, 共 4 个选择题, 每题有 3 个选择支.\n已知: 任何 3 个考生都有一个题, 他们的答案各不相同.\n求考生人数的最大值.", + "solution": "设有 $n$ 个考生.\n易知, 每个考生的答卷都是一个长为 4 的序列, 此序列由 $A 、 B 、 C$ 三个字母组成.\n从而本题等价于对 $n \\times 4$ 方格棋盘的格 3 -染色,使 \"任何三行都有某列的 3 个格两两异色\". 设所有列中的异色格对的总数为 $S$. 一方面,对任何一列, 设该列中有 $a$ 个格为 $A$ 色, $b$ 个格为 $B$ 色, $c$ 个格为 $C$ 色, 则该列中的异色对有 $a b+b c+c a$ 个.\n注意到 $3(a b+b c+c a) \\leqslant a^2+b^2+c^2+2(a b+b c+c a)=(a+b+c)^2=n^2$, 所以, 4 列中至多有 $\\frac{4 n^2}{3}$ 个异色对.\n即 $S \\leqslant \\frac{4 n^2}{3}$. 另一方面, 任何 3 行都有一列包含 3 色, 此 3 色构成 3 个异色对, 于是有 $3 \\mathrm{C}_n^3$ 个异色对.\n但同一个异色对可能出现在 $n^{--}$个不同的 $3-$ 行组中, 这样, $S \\geqslant 3 \\cdot \\frac{\\mathrm{C}_n^3}{n-2}$. 所以, $3 \\cdot \\frac{\\mathrm{C}_n^3}{n-2} \\leqslant S \\leqslant \\frac{4 n^2}{3}$. 但此不等式恒成立, 不能求出 $n$ 的范围.\n读者可以考虑, 这一估计是否可以改进? 从另一角度考虑\"任何三行都有某列的 3 个格两两异色\"的反面: 存在三行, 这三行的所有列的 3 个格都只有两色.\n我们用反证法找每列只有两色的那些列.\n通过尝试, 发现 $n<10$. 否则, 取其中任意 10 行得到 $10 \\times 4$ 方格表 $M$. 考察 $M$ 的第一列格, 必有一种颜色至多出现 3 次.\n于是, 至少有 7 行, 这 7 行的首列只有两色.\n考察这 7 行的第二列, 必有一种颜色至多出现 2 次.\n从而在上述 7 行中至少有 5 行, 这 5 行的首列、第二列都只有两色.\n再考察这 5 行的第三列, 必有一颜色至多出现 1 次.\n从而在上述 5 行中至少有 4 行, 这 4 行的首列、第二列、第三列都只有两色.\n再考察这 4 行的最后一列, 必有一颜色至多出现 1 次, 从而在上述 4 行中至少有 3 行, 这 3 行的每一列都只有两色.\n设这 3 行为 $A_1 、 A_2 、 A_3$, 则 $A_1$ 、 $A_2 、 A_3$ 的任何一列都有两个格同色,矛盾.\n当 $n=9$ 时, 9 个人对每道题选择的答案代号分别为 $(1,2,1,2) 、(2,3,2,2) 、(3,1,3,2) 、(1,1,2,1)$ 、 $(2,2,3,1) 、(3,3,1,1) 、(1,3,3,3) 、(2,1,1,3) 、(3,2,2,3)$, 所以 $n=9$ 是可能的, 故 $n$ 的最大值为 9 .", + "remark": "", + "figures": [] +} \ No newline at end of file diff --git a/processed_dataset/proof/0583.json b/processed_dataset/proof/0583.json new file mode 100644 index 0000000000000000000000000000000000000000..18943e023668bb38f14b635e4f8fb2da5dad4d48 --- /dev/null +++ b/processed_dataset/proof/0583.json @@ -0,0 +1,8 @@ +{ + "source_file": "./raw_volume-zh/volume13/exercise11.tex", + "problem_type": "proof", + "problem": "问题6. 给定 25 人, 其中每 5 人可以组成一个委员会, 且每两个委员会至多有一个公共成员.\n求证: 委员会的个数不多于 30 .", + "solution": "将每两个委员至多有一个公共成员理解为 $\\left|A_i \\cap A_j\\right| \\leqslant 1$, 则应记 $A_i$ 为第 $i$ 个委员会的人的集合.\n再令 $X$ 为给定的 25 个人的集合, $F=\\left\\{A_1\\right.$, $\\left.A_2, \\cdots, A_k\\right\\}$. 我们来计算所有二人组的总数 $S$. 一方面, $\\left|A_i\\right|=5$, 从而每个 $A_i$ 中有 $\\mathrm{C}_5^2=10$ 个二人组, 于是, $F$ 中的 $k$ 个集合可产生 $10 k$ 个二人组.\n由于 $\\left|A_i \\cap B_j\\right| \\leqslant 1$, 这 $10 k$ 个二人组互异.\n所以, $S \\geqslant 10 k$. 另一方面, 设 $|X|=25$, 则 $X$ 中的二人组的总数为 $S=\\mathrm{C}_{25}^2$. 所以, $\\mathrm{C}_{25}^2=S \\geqslant 10 k$, 所以, $k \\leqslant 30$. 命题获证.", + "remark": "", + "figures": [] +} \ No newline at end of file diff --git a/processed_dataset/proof/0584.json b/processed_dataset/proof/0584.json new file mode 100644 index 0000000000000000000000000000000000000000..556e184c71d1edda1c13e12ea42644834dc422c0 --- /dev/null +++ b/processed_dataset/proof/0584.json @@ -0,0 +1,8 @@ +{ + "source_file": "./raw_volume-zh/volume13/exercise12.tex", + "problem_type": "proof", + "problem": "问题1. 设 $A$ 是集合 $\\{1,2,3, \\cdots, 16\\}$ 的一个 $k$ 元子集, 且 $A$ 的任何两个子集的元素之和不相等.\n而对于集合 $\\{1,2,3, \\cdots, 16\\}$ 的包含集合 $A$ 的任意 $k+1$ 元子集 $B$, 则存在 $B$ 的两个子集,它们的元素之和相等.\n(1) 证明: $k \\leqslant 5$; (2) 求集合 $A$ 的元素之和的最大值与最小值.", + "solution": "(1) 若 $k \\geqslant 7$, 则 $A$ 的非空子集有 $2^k-1$ 个, 而其中每个子集元素和不超过 $17 k$, 但 $2^k-1>17 k$, 必有两个子集的和相等, 矛盾.\n若 $k=6$, 考虑 $A$ 的一、二、三、四元子集, 共有 $\\mathrm{C}_6^1+\\mathrm{C}_6^2+\\mathrm{C}_6^3+\\mathrm{C}_6^4=56$ 个不同的子集, 其元素和都在区间 $[1,57]$ 内 (因为任意一个这样的和 $\\leqslant 16+15+14+13=58$, 且由 $13+16=15+14$ 知, $13 、 14 、 15 、 16$ 不都属于 $A$ ). 若 $1 \\in A$, 则由 $1+15=16$ 知, $15 、 16$ 不同时属于 $A$. 由 $1+13=14$ 知, $13 、 14$ 不同时属于 $A$. 由 $1+ 11=12$ 知, $11 、 12$ 不同时属于 $A$. 所以此时最大的和不大于 $16+14+12+ 10=52$, 而 $56>52$, 必有两个子集的和相等, 矛盾.\n若 $2 \\in A$, 则由 $2+14=$ 16 知, $14 、 16$ 不同时属于 $A$. 由 $2+13=15$ 知, $13 、 15$ 不同时属于 $A$. 由 $2+ 10=12$ 知, 10、12 不同时属于 $A$. 所以此时最大的和不大于 $16+15+12+ 9=52$, 而 $56>52$, 必有两个子集的和相等,矛盾.\n若 1 和 2 都不属于 $A$,则最小的和不小于 3 . 于是, 其和都属于区间 $[3,57]$, 最多有 55 个不同的和.\n而 $56>55$, 必有两个子集的和相等, 矛盾.\n综上所述, $k \\leqslant 5$. (2) 设 $A$ 的元素和为 $S$. 若 $S<16$, 考察包含 $A$ 的 $k+1$ 元子集 $B=A \\cup\\{16\\}$. 由于 $A$ 的任意两个子集元素之和不等,且 $B$ 的任意一个包含 16 的子集元素和比 $B$ 的任意一个不包含 16 的子集元素和大, 从而 $B$ 的任意两个子集元素之和不相等, 与条件矛盾.\n从而 $S \\geqslant 16$. 又 $A=\\{1,2,4,9\\}$ 满足要求, 此时 $S(A)=16$, 从而 $S$ 最小值为 16 . 若 $k \\leqslant 4$, 则 $S \\leqslant 16+15+14+13=58<66$; 若 $k=5$, 且 16 、 15 不全属于 $A$, 则 $S \\leqslant 16+14+13+12+11=66$; 若 $k=5$, 且 $16 、 15$ 都属于 $A$, 则 $(14,13) 、(12,11) 、(10,9)$ 每一组中的两个数都不能全属于 $A$, 故 $S \\leqslant 16+15+14+12+10=67$, 且等号不成立, 否则 $14 、 12 、 10 、 16 \\in A$, 但 $16+10=12+14$, 矛盾.\n于是 $S \\leqslant 66$. 又 $A=\\{16,15,14,12,9\\}$ 满足要求, 此时 $S(A)=66$. 从而 $S$ 最大值为 66 .", + "remark": "", + "figures": [] +} \ No newline at end of file diff --git a/processed_dataset/proof/0585.json b/processed_dataset/proof/0585.json new file mode 100644 index 0000000000000000000000000000000000000000..55a3f78e3d21a3df08f5eab63791fbacf97b77ee --- /dev/null +++ b/processed_dataset/proof/0585.json @@ -0,0 +1,8 @@ +{ + "source_file": "./raw_volume-zh/volume13/exercise12.tex", + "problem_type": "proof", + "problem": "问题2. 议会中有 2000 名议员, 他们决定审核财政预算, 共有 200 项开支.\n各名议员都准备一份预算草案, 列出各项支出的数额.\n各位列出的数额的总数都不超过 $S$. 议会审议时, 均将之确定为至少 $k$ 名议员所同意的数目 (即 $k$ 名议员列出的数额均不低于通过的数目). 试问: 至少应将 $k$ 确定为多少, 才能保证通过的总额不超过 $S$ ?", + "solution": "显然, 每个议员所填的各项开支是一个长为 200 的数列, 2000 个议员所填的数列构成一个 $2000 \\times 200$ 的数表.\n此数表的各行的和都不大于 $S$. 现在要在数表的最下面填一行数, 使每列中所填的数至少小于该列中 $k$ 个数.\n求最小的 $k$, 使最后所填的一行数的和不大于 $S$. 若数 $k$ 合乎上述要求, 则称 $k$ 是好的.\n$k=2000$ 显然是好的.\n进一步, $k=1999$ 也是好的.\n实际上, $k=1999$ 时, 每一列只有一个数小于 $x_j$, 我们称这样的数为坏数.\n于是, 200 个列至多有 200 个坏数.\n但数表有 2000 行, 于是, 至少有一个行没有坏数.\n这表明: 填此行数的议员同意选出的每一项开支.\n如此下去 (不断缩小包围圈), 不难发现, $k=1991$ 是好的.\n实际上, 当 $k=1991$ 时, 每列至多有 9 个坏数,200 列至多有 1800 个坏数.\n于是, 至少有一个行中没有坏数.\n下面证明: $k=1990$ 不是好的.\n实际上, 将数表的每 10 行分为一个组, 第 $i$ 组中, 每行的数都填 $\\frac{S}{199}$, $\\frac{S}{199}, \\cdots, \\frac{S}{199}, 0, \\frac{S}{199}, \\cdots, \\frac{S}{199}$. 其中只有第 $i$ 列的数为 0 , 其余各数都为 $\\frac{S}{199}$. 这样, 数表中的每一行都有 199 个 $\\frac{S}{199}$,一个 0 . 于是, 每行的和为 $S$. 每列中有 10 个 0,1990 个 $\\frac{S}{199}$. 从而对任何 $j$, 令 $x_j=\\frac{S}{199}$. 此时通过的支出总额为$200 \\cdot \\frac{S}{199}>S$,矛盾.\n综上所述, $k$ 的最小值为 1991 .", + "remark": "", + "figures": [] +} \ No newline at end of file diff --git a/processed_dataset/proof/0586.json b/processed_dataset/proof/0586.json new file mode 100644 index 0000000000000000000000000000000000000000..487ad2b966251c78dbc61a7532ab3c10f0e7dcc1 --- /dev/null +++ b/processed_dataset/proof/0586.json @@ -0,0 +1,8 @@ +{ + "source_file": "./raw_volume-zh/volume13/exercise12.tex", + "problem_type": "proof", + "problem": "问题3. 有 $n(n \\geqslant 5)$ 支球队参加足球联赛, 每两支球队都恰比赛一场.\n规定胜一场得 3 分, 平一场得 1 分, 负一场得 0 分.\n联赛结束后, 有一些球队可能会被取消比赛资格, 因此他们的比赛结果也会被取消.\n剩下的球队中, 如果一个队的积分多于其他任何球队, 则该队成为这次联赛的冠军 (如果只有一支球队没有被取消资格, 则他就是冠军队). 记为了让第 $i$ 支球队获得冠军而需要取消比赛资格的球队数的最小值为 $f_i$, 求 $F=f_1+\\cdots+f_n$ 的最大值与最小值.", + "solution": "如果将除 $i$ 队外的其他所有队取消资格, 则 $i$ 队自然是冠军, 所以 $f_i \\leqslant n-1$. 于是 $F \\leqslant n(n-1)$. 又当所有队打平时, 对一切 $i=1,2, \\cdots, n$, 有 $f_i=n-1$. 此时 $F=n(n-1)$. 故 $F_{\\text {max }}=n(n-1)$. 当 $n \\geqslant 5$ 时, 我们先证明 $F \\geqslant n . F \\geqslant n$ 的一个充分条件是 $f_i \\geqslant 1$, 即没有球队的积分多于其他所有球队.\n此外, 假定对 $i=1,2, \\cdots, n-1$, 第 $n$ 队的积分 $S_n$ 多于第 $i$ 队的积分 $S_i$,则第 $i$ 队要成为冠军, 至少要去掉一支球队, 即 $f_i \\geqslant 1(iS_i+2$,去掉第 $a_i$ 队后第 $n$ 队的积分至少是 $S_n-3$, 而第 $i$ 队的积分不增, 而 $S_n-3 \\geqslant S_i$, 与第 $i$ 队要为冠军矛盾.\n(2) 当 $a_i \\neq n$ 时, 第 $a_i$ 队输给第 $n$ 队.\n否则, 去掉 $a_i$ 后第 $n$ 队的积分至少是 $S_n-1$, 而第 $i$ 队的积分不增.\n但 $S_n>S_i$, 所以 $S_n- 1 \\geqslant S_i$, 与第 $i$ 队要为冠军矛盾.\n(3) 当 $i \\neq j$ 时, $a_i \\neq a_j$. 否则, 去掉 $a_i$ 后有 2 个冠军队第 $i$ 队和第 $j$ 队, 矛盾.\n由 (2) (3) 可知, 第 $a_1, a_2, \\cdots, a_{n-1}$ 中不是第 $n$ 队的队都输给第 $n$ 队.\n于是至少有 $n-2$ 个队输给第 $n$ 队, 所以 $S_n \\geqslant 3(n-2)$. \n于是, $S=\\sum_{i=1}^n S_i=\\sum_{i=1}^{n-1} S_i+S_n \\geqslant \\sum_{i=1}^{n-1}\\left(S_n-2\\right)+S_n=(n-1)\\left(S_n-2\\right)+ S_n=n S_n-2(n-1) \\geqslant n \\cdot 3(n-2)-2(n-1)=3 n^2-8 n+2$. 另一方面,每场比赛对 $S$ 的贡献至多是 3 分, 于是 $S \\leqslant 3 C_n^2=3 \\cdot \\frac{n(n-1)}{2}$. 所以 $3 \\cdot \\frac{n(n-1)}{2} \\geqslant 3 n^2-8 n+2$, 解得 $n<5$, 矛盾.\n最后, 当 $A_1$ 胜 $A_2, A_2$ 胜 $A_3 \\cdots \\cdots A_n$胜 $A_1$, 且其余比赛全部平局时, 可以验证所有 $f_i=1$, 此时 $F=n$. 从而 $n \\geqslant 5$时 $F_{\\min }=n$.", + "remark": "", + "figures": [] +} \ No newline at end of file diff --git a/processed_dataset/proof/0587.json b/processed_dataset/proof/0587.json new file mode 100644 index 0000000000000000000000000000000000000000..481f5d43a2b360b27d8d31ee983ab6e1ec16cd68 --- /dev/null +++ b/processed_dataset/proof/0587.json @@ -0,0 +1,8 @@ +{ + "source_file": "./raw_volume-zh/volume13/exercise13.tex", + "problem_type": "proof", + "problem": "问题2. 设 $a_1, a_2, \\cdots, a_k$ 是以不超过 $n$ 的正整数为项的有限数列, 其中任何一个项的两个相邻项都不同, 且不存在任何四个指标 $p1$, 则令 $A=\\left\\{a_1, a_2, \\cdots, a_v\\right\\}, B=\\left\\{a_{v+1}\\right.$, $\\left.a_{v+2}, \\cdots, a_{k-1}\\right\\}$, 则 $A 、 B$ 中分别没有数相同的连续两项, 且 $B$ 的项都在 $\\{1$, $2, \\cdots, n\\} \\backslash\\{t\\}$ 中, $A$ 的项都在 $\\{1,2, \\cdots, n\\} \\backslash\\{B\\}$ 中, 从而都可利用归纳假设.\n设 $A$ 中互异的项的个数为 $p, B$ 中互异的项的个数为 $q$, 则 $p+q \\leqslant n$. 这样分别对 $A 、 B$ 使用归纳假设, 有 $v \\leqslant 2 p-1, k-v-1 \\leqslant 2 q-1$. 所以 $k \\leqslant 2 q+ v \\leqslant 2 q+2 p-1=2(p+q)-1 \\leqslant 2 n-1$. 故 $k$ 的最大值为 $4 n-2$.", + "remark": "", + "figures": [] +} \ No newline at end of file diff --git a/processed_dataset/proof/0588.json b/processed_dataset/proof/0588.json new file mode 100644 index 0000000000000000000000000000000000000000..d72ab2260fcb28f3b8c6f87b881a41c8eafb76f7 --- /dev/null +++ b/processed_dataset/proof/0588.json @@ -0,0 +1,8 @@ +{ + "source_file": "./raw_volume-zh/volume13/exercise13.tex", + "problem_type": "proof", + "problem": "问题3. 设有 $2^n$ 个由数字 $0 、 1$ 组成的有限数列, 其中任何一个数列都不是另一个数列的前段.\n求所有数列的长度和 $S$ 的最小值.", + "solution": "一个数列可以看成是一个排列.\n对两个不同的排列, 其中一个不是另一个的前段的一个充分条件是: 它们的长度相等.\n假设它们的长度都是 $r$, 那么互异的排列有 $2^r$ 个.\n但题给的数列有 $2^n$ 个, 所以 $r=n$. 即长为 $n$ 的互异的排列有 $2^n$ 个,它们中任何一个不是另一个的前段, 此时 $S=n \\times 2^n$. 下面证明 $S \\geqslant n \\times 2^n$. 我们只须把任意一个合乎条件的排列集中的每一个排列都操作到长度不小于 $n$. 我们称长度为 $n$ 的排列为标准排列, 长度小 (大) 于 $n$ 的排列为短 (长) 排列.\n如果合乎条件的排列集中存在短排列, 则必存在长排列.\n否则, 每个短排列至少可以扩充为 2 个互异的标准排列, 使得标准排列个数超过 $2^n$, 矛盾.\n任取一个短排列 $A$, 必存在长排列 $B$. 去掉 $A, B$, 加人两个排列 : $A \\cup \\{0\\}, A \\cup\\{1\\}$, 得到的排列集仍合乎条件.\n因为 $|A|n$, 于是操作后长度和 $S$ 的增量为 $|A|+1+|A|+1-|A|-|B|=2+|A|-|B| \\leqslant 2+(n-1)-(n+1)=0$, 即 $S$ 不增.\n如此下去, 直至排列中不存在短排列, 必有 $S \\geqslant n \\times 2^n$.", + "remark": "", + "figures": [] +} \ No newline at end of file diff --git a/processed_dataset/proof/0589.json b/processed_dataset/proof/0589.json new file mode 100644 index 0000000000000000000000000000000000000000..8ed44f89f8bb46b72d2ea5022a5366aed07047db --- /dev/null +++ b/processed_dataset/proof/0589.json @@ -0,0 +1,8 @@ +{ + "source_file": "./raw_volume-zh/volume13/exercise13.tex", + "problem_type": "proof", + "problem": "问题4. 在 $m \\times n(m>1, n>1)$ 棋盘上放有 $r$ 只棋, 每个格最多一只棋.\n若 $r$ 只棋具有如下的性质 $p$ : 每行每列至少有一只棋.\n但去掉其中任何一只棋, 则它们便不再具有上述的性质 $p$. 求 $r$ 的最大值 $r(m, n)$.", + "solution": "$r(m, n)=m+n-2$. 一方面, 由构图可知, $r=m+n-2$ 是可能的(第一行与第一列各格除第一格外都放棋).下面证明: $r2$ 且 $n>2$, 则由于棋盘上至少放有 $m+ n-1>m$ 只棋, 必有一行至少含有两只棋.\n不妨设第一行的棋最多, 共有 $t$ 只棋, 设为 $a_{11}, a_{12}, \\cdots, a_{1 t}(t>1)$. 则这些棋所在的列中不能再有其他的棋, 比如, 若 $a_{11}$ 所在的列中还有一只棋, 则 $a_{11}$ 可去.\n于是, 去掉第一行和前 $t$ 列, 剩下 $(m-1) \\times(n-t)$ 棋盘, 此棋盘中至少有 $m+n-1-t=(m-1)+(n-t)$ 只棋.\n注意到 $(m-1)+(n-t)=k-t-11, n>1)$ 棋盘 $C$ 中, 每格填一个数, 使对任何正整数 $p 、 q$ 及任何 $p \\times q$ 矩形, 相对顶点两格所填的数的和相等.\n若对适当的 $r$ 个格填数后, 余下各格所填的数被唯一确定, 求 $r$ 的最小值.", + "solution": "填好数表的第一行和第一列后, 数表被唯一确定, 此时, 数表只填了 $m+n-1$ 个数.\n即 $r=m+n-1$ 时, 存在相应的填法.\n下面证明, 对所有合乎条件的填法, 有 $r \\geqslant m+n-1$. 用反证法.\n即 $r \\leqslant m+n-2$ 时, 不论怎样填表, 数表都不唯一确定.\n对 $m+n$ 归纳.\n当 $m+n=4$ 时, $m=n=2,2 \\times 2$ 数表中填 2 个数, 数表不唯一确定, 结论成立.\n设结论对 $m+n=k$ 成立.\n考察 $m+n= k+1>4$ 时的情形.\n数表填人 $r \\leqslant m+n-2=k-1$ 个数, 我们要证明此数表不唯一确定.\n为了利用假设, 应去掉数表的一行或一列.\n而且去掉的行或列应具有这样的性质: (1) 去掉这行后, 数表至少还有两行 (否则不能利用归纳假设). 为此, 不妨设 $m \\leqslant n$, 则 $n>2$. 于是可去掉一个列.\n(2) 去掉这列后, $(m-1) \\times n$ 数表中的数不多于 $(m-1)+n-1=k-2$ 个数, 即去掉的列中至少有一个数.\n(3) 加上去掉的这个列, 数表仍不唯一确定, 这只须此列中的数不多于一个 (一个充分条件). 实际上, 若有一个数 $a$, 在子表中不被唯一确定, 而被后加上的一列中的两个数 $b 、 c$ 及与 $a$ 同列的一个数 $d$ 唯一确定, 但这列中只有一个数,则不妨设 $c$ 是新填人的, 则 $c$ 只能由另一列中的两个数 $e$ 、 $f$ 确定.\n于是, $a$ 可由子表中的 $e 、 f 、 d$ 唯一确定.\n矛盾.\n现在要找到合乎上述两个条件的一个列.\n即此列中恰有一个数.\n注意到 $m \\leqslant n$, 所以, 棋盘中的数的个数 $r \\leqslant k-1=m+n-2 \\leqslant 2 n-2<2 n$. 所以, 至少有一列不多于一个数.\n若此列中没有数, 则此列不唯一确定, 结论成立.\n若此列中恰有一个数 $a$, 则去掉此列, 其余的数必可唯一确定, 否则假设 $b$ 不唯一确定, 则 $b$ 要由 $a$ 确定, 从而 $a, b$ 是同一矩形的两个顶点,且此矩形中除 $b$ 以外的 3 个数都已知, 这 3 个数中除 $a$ 外必有一个与 $a$ 同列, 矛盾.\n于是, $(m-1) \\times n$ 数表对应的最小值 $\\leqslant r-1 \\leqslant k-2=m+n-30$, 所以 $x_1^{\\prime} x_2 \\cdots x_9 x_{10}^{\\prime}>x_1 x_2 \\cdots x_{10}$, 矛盾.\n由此可见, $x_1, x_2, \\cdots, x_{10}$ 均不小于 198. 若 $x_1, x_2, \\cdots, x_{10}$ 中有大于 199 者, 设为 $x_1$, 那么其中必有小于 199 者, 设为 $x_{10}$. 因为 $x_{10}$ 不小于 198 , 所以 $x_{10}=$ 198. 令 $x_1^{\\prime}=x_1-1, x_{10}^{\\prime}=x_{10}+1$, 则得到一种新的分拆 $\\left(x_1^{\\prime}, x_2, \\cdots, x_9\\right.$, $\\left.x_{10}^{\\prime}\\right)$, 相应的积为 $x_1^{\\prime} x_2 \\cdots x_9 x_{10}^{\\prime}$. 但 $x_1^{\\prime} x_{10}^{\\prime}-x_1 x_{10}=\\left(x_1-1\\right)\\left(x_{10}+1\\right)- x_1 x_{10}=x_1 x_{10}+x_1-x_{10}-1-x_1 x_{10}=x_1-x_{10}-1=x_1-198-1=x_1- 199>0$, 所以 $x_1^{\\prime} x_2 \\cdots x_9 x_{10}^{\\prime}>x_1 x_2 \\cdots x_{10}$, 矛盾.\n所以 $x_1, x_2, \\cdots, x_{10}$ 均只能是 199 或 198 , 从而分拆: $199+199+\\cdots+199+198$ 对应的积为 $199^9 \\times 198$ 为最大.", + "remark": "", + "figures": [] +} \ No newline at end of file diff --git a/processed_dataset/proof/0594.json b/processed_dataset/proof/0594.json new file mode 100644 index 0000000000000000000000000000000000000000..c89d92500bb4adceb2aff01c8d04cfa696c96ba8 --- /dev/null +++ b/processed_dataset/proof/0594.json @@ -0,0 +1,8 @@ +{ + "source_file": "./raw_volume-zh/volume13/exercise3.tex", + "problem_type": "proof", + "problem": "问题4. 给定实数 $P_1 \\leqslant P_2 \\leqslant P_3 \\leqslant \\cdots \\leqslant P_n$, 求出实数 $x_1 \\geqslant x_2 \\geqslant \\cdots \\geqslant x_n$, 使 $d=\\left(P_1-x_1\\right)^2+\\left(P_2-x_2\\right)^2+\\cdots+\\left(P_n-x_n\\right)^2$ 最小.", + "solution": "由直观猜想最值点 $\\left(x_1, x_2, \\cdots, x_n\\right)$ 是均匀的, 即 $x_1=x_2=\\cdots= x_n=x$ (待定). 此时 $d=\\sum_{i=1}^n\\left(P_i-x_i\\right)^2=\\sum_{i=1}^n\\left(P_i-x\\right)^2=n x^2-2 \\sum_{i=1}^n\\left(P_i\\right) x+ \\sum_{i=1}^n P_i^2$. 此二次函数在 $x=\\sum_{i=1}^n \\frac{P_i}{n}=P$ 时达到最小.\n于是, 我们猜想 $d$ 在 $x_1= x_2=\\cdots=x_n=P$ 时达到最小.\n我们只须证明: 给定实数 $P_1 \\leqslant P_2 \\leqslant P_3 \\cdots \\leqslant P_n$, 对任何实数 $x_1 \\geqslant x_2 \\geqslant \\cdots \\geqslant x_n, \\sum_{i=1}^n\\left(P_i-x_i\\right)^2 \\geqslant \\sum_{i=1}^n\\left(P_i-\\right. P)^2\\left(\\right.$ 其中 $\\left.P=\\sum_{i=1}^n \\frac{P_i}{n}\\right)$. 记上式左边与右边的差为 $H$, 则 $H=\\sum_{i=1}^n x_i^2- 2 \\sum_{i=1}^n P_i x_i+2 P \\sum_{i=1}^n P_i-n P^2=\\sum_{i=1}^n x_i^2-2 \\sum_{i=1}^n P_i x_i+n P^2 \\geqslant$ (切比雪夫不等式) $\\sum_{i=1}^n x_i^2-2 \\cdot \\frac{1}{n} \\sum_{i=1}^n P_i \\sum_{i=1}^n x_i+n P^2=\\sum_{i=1}^n x_i^2-2 P \\sum_{i=1}^n x_i+n P^2=\\sum_{i=1}^n\\left(x_i-\\right. P)^2 \\geqslant 0$.", + "remark": "", + "figures": [] +} \ No newline at end of file diff --git a/processed_dataset/proof/0595.json b/processed_dataset/proof/0595.json new file mode 100644 index 0000000000000000000000000000000000000000..43e8b5814e7de6c77cd8b7467447dd0f26535716 --- /dev/null +++ b/processed_dataset/proof/0595.json @@ -0,0 +1,8 @@ +{ + "source_file": "./raw_volume-zh/volume13/exercise3.tex", + "problem_type": "proof", + "problem": "问题5. 给定平面上的点集 $P=\\left\\{p_1, p_2, \\cdots, p_{1994}\\right\\}, P$ 中任何三个点不共线.\n将 $P$ 中的点分为 83 组, 每组至少 3 个点.\n将同一组中的点两两连线, 不同的组中的点不连线, 得到一个图 $G, G$ 中的三角形的个数记为 $m(G)$.\n(1) 求 $m(G)$ 的最小值.\n(2)设使 $m(G)$ 达到最小的图为 $G^{\\prime}$. 求证: 可以将 $G^{\\prime}$ 中的点 4-染色, 使 $G^{\\prime}$ 中不含同色三角形.", + "solution": "(1) 因为分组方法是有限的, 必存在一种分组方法, 使得三角形个数最少.\n注意到 $1994=83 \\times 24+2=81 \\times 24+2 \\times 25$, 于是, 将 1994 个点分为 83 组,其中 81 组中各有 24 个点, 2 组中各有 25 个点, 我们证明这样的分组才使三角形的个数最少, 即 $(m(G))_{\\text {min }}=81 \\mathrm{C}_{24}^3+2 \\mathrm{C}_{25}^3=168544$. 否则, 将 ( $i$, $j)(i \\geqslant j+2), S=\\mathrm{C}_i^3+\\mathrm{C}_j^3$, 调整为 $(i-1, j+1), S^{\\prime}=\\mathrm{C}_{i-1}^3+\\mathrm{C}_{j+1}^3$, 有 $S- S^{\\prime}=\\mathrm{C}_i^3+\\mathrm{C}_j^3-\\left(\\mathrm{C}_{i-1}^3+\\mathrm{C}_{j+1}^3\\right)=\\mathrm{C}_{i-1}^2-\\mathrm{C}_j^2>0$. 按此方法调整一次, 三角形个数减少.\n(2) $G^{\\prime}$ 由若干个独立的连通图组成, 因而只须考虑 $\\left|G_1\\right|=25$ 和 $\\left|G_2\\right|=24$ 的两个图 $G_1$ 和.\n$G_2$ 的染色.\n进一步可知, 只须考虑泈 $G_1$ 的染色.\n实际上, 对 $\\left|G_2\\right|=24$, 在 $G_1$ 的染色的基础上去掉其中一个点及其关联的边即可.\n将 25 个点分为 5 组, 每组 5 个点.\n每一组中的 5 点之间的边 $12 、 23$ 、 $34 、 45 、 51$ 用第一种颜色染; 边 $13 、 35 、 52 、 24 、 41$ 用第二种颜色染.\n再将染色后的五点组看作一个\"大点\", 有 5 个 \"大点\". 对此 5 个大点之间的边再按上述方法用另外两种颜色染色, 从而 4 色可完成染色.", + "remark": "", + "figures": [] +} \ No newline at end of file diff --git a/processed_dataset/proof/0596.json b/processed_dataset/proof/0596.json new file mode 100644 index 0000000000000000000000000000000000000000..106e1efc1fae3d4b9d7bf2294ed7f5fae90d7166 --- /dev/null +++ b/processed_dataset/proof/0596.json @@ -0,0 +1,8 @@ +{ + "source_file": "./raw_volume-zh/volume13/exercise3.tex", + "problem_type": "proof", + "problem": "问题6. 有 14 人进行一种日本棋循环赛, 每个人都与另外 13 个人比赛一局, 在比赛中无\"平局\". 如果三个人之间的比赛结果是每个人都胜一局负一局, 则称这 3 人是一个 \"三联角\", 求 \"三联角\" 个数的最大值.", + "solution": "解: 1: 设 14 个人为 $A_1, A_2, \\cdots, A_{14}$, 他们胜的场数分别为 $w_1,w_2, \\cdots, w_{14}$, 则 $\\sum_{i=1}^{14} w_i=\\mathrm{C}_{14}^2=91$. 如果某三个人不组成\"三联角\", 那么这三个人中一定有一个人胜了其余两个人.\n而 $A_i$ 胜另两人的三人组有 $\\mathrm{C}_{w_i}^2$ 个, 从而非三联角的三人组总数为 $\\sum_{i=1}^{14} \\mathrm{C}_{w_i}^2$ (其中规定 $\\mathrm{C}_0^2=\\mathrm{C}_1^2=0$ ). 所以三联角的三人组总数 $S=\\mathrm{C}_{14}^3-\\sum_{i=1}^{14} \\mathrm{C}_{w_i}^2$. 下面求 $S^{\\prime}=\\sum_{i=1}^{14} \\mathrm{C}_{w_i}^2$ 的最小值.\n首先, 比赛结果只有有限种, 从而最小值一定存在.\n其次, 我们证明: 当 $\\sum_{i=1}^{14} \\mathrm{C}_{w_i}^2$ 达到最小时, 对任何 $1 \\leqslant i0$, 于是 $\\sum_{i=1}^{14} \\mathrm{C}_{w_i}^2$ 不是最小的,矛盾.\n注意到 $91=14 \\times 6+7$, 所以当 $\\left\\{w_1, w_2, \\cdots, w_{14}\\right\\}=\\{6$, $6,6,6,6,6,6,7,7,7,7,7,7,7\\}$ 时, $\\sum_{i=1}^{14} \\mathrm{C}_{w_i}^2$ 达到最小值 $7 \\mathrm{C}_6^2+7 \\mathrm{C}_7^2=$ 252 , 于是 $S=\\mathrm{C}_{14}^3-\\sum_{i=1}^{14} \\mathrm{C}_{w_i}^2 \\leqslant \\mathrm{C}_{14}^3-252=112$. 解法 2 : 同解法 1 , 得非三联角的三人组总数为 $\\sum_{i=1}^{14} \\mathrm{C}_{w_i}^2$. 又设 $A_i$ 输的场数为 $l_i$, 同样可知, 非三联角的三人组总数为 $\\sum_{i=1}^{14} \\mathrm{C}_{l_i}^2$. 于是非三联角的三人组总数为 $\\frac{1}{2} \\sum_{i=1}^{14}\\left(\\mathrm{C}_{l_i}^2+\\mathrm{C}_{w_i}^2\\right)$. 由于 $w_i+ l_i=13$, 所以 $w_i^2+l_i^2=\\frac{1}{2}\\left[13^2+\\left(w_i-l_i\\right)^2\\right] \\geqslant 85$, 从而 $\\mathrm{C}_{w_i}^2+\\mathrm{C}_{l_i}^2=\\frac{w_i^2+l_i^2}{2} -\\frac{13}{2} \\geqslant 36$,于是 $\\frac{1}{2} \\sum_{i=1}^{14}\\left(\\mathrm{C}_{l_i}^2+\\mathrm{C}_{w_i}^2\\right) \\geqslant \\frac{1}{2} \\sum_{i=1}^{14} 36=252$. 故三联角数目 $S \\leqslant \\mathrm{C}_{14}^3 -252=112$. 另一方面, 对任意 $1 \\leqslant i2, \\lambda$ 是一个给定的常数.\n确定函数: $F=x_1^2+x_2^2+\\cdots+ x_n^2+\\lambda x_1 x_2 \\cdots x_n$ 的最大值与最小值, 这里 $x_1, x_2, \\cdots, x_n$ 是非负实数, 且 $x_1+x_2+\\cdots+x_n=1$.", + "solution": "因为 $F$ 在闭域 $0 \\leqslant x_i \\leqslant 1$ 上连续, 所以 $F$ 必有最大值和最小值.\n不妨设 $\\left(x_1, x_2, \\cdots, x_n\\right)$ 是 $F$ 的最值点, 我们证明, 对任何 $i \\neq j, x_i x_j=0$, 或 $x_i=x_j$. 实际上, 由对称性, 我们只须考察 $x_1 、 x_2$. 固定 $x_3, x_4, \\cdots, x_n$, 则 $x_1+x_2=1-\\left(x_3+x_4+\\cdots+x_n\\right)=c($ 常数 $) . F=\\left(x_1+x_2\\right)^2+x_3^2+\\cdots+ x_n^2-2 x_1 x_2+\\lambda x_1 x_2 \\cdots x_n=\\left(x_1+x_2\\right)^2+x_3^2+\\cdots+x_n^2+\\left(\\lambda x_3 x_4 \\cdots x_n-2\\right) \\cdot x_1\\left(c-x_1\\right)$. 令 $f\\left(x_1\\right)=x_1\\left(c-x_1\\right)$, 因为 $0 \\leqslant x_1 \\leqslant c$, 由二次函数的性质, 当 $f\\left(x_1\\right)$ 达到最值时, $x_1 \\in\\left\\{0, c, \\frac{c}{2}\\right\\}$. 注意此时 $x_1+x_2=c$, 于是 $x_1 、 x_2$ 之间有以下关系: (1) $x_1=0, x_2=c$. (2) $x_1=c, x_2=0$. (3) $x_1=x_2=\\frac{c}{2}$. 于是, 要么 $x_1 x_2=0$, 要么 $x_1=x_2$. 由上面讨论可知, 若 $\\left(x_1, x_2, \\cdots, x_n\\right)$ 是 $F$ 的最值点, 则 $x_1, x_2, \\cdots, x_n$ 中的非零数都相等.\n不妨设 $x_1=x_2=\\cdots=x_k \\neq 0$, $x_{k+1}+x_{k+2}+\\cdots+x_n=0$, 有以下情况: (1) 若 $k=n$, 那么 $F$ 在 $x_1=x_2=\\cdots= x_n=\\frac{1}{n}$ 达到最值, 此时 $F=\\sum_{i=1}^n \\frac{1}{n^2}+\\lambda \\prod_{i=1}^n \\frac{1}{n}=\\frac{\\lambda+n^{n-1}}{n^n}$. (2) 若 $k0$. 等号在 $x_1+x_2+x_3=\\frac{1}{2}$, 且 $x_2 x_3=x_1\\left(x_2+x_3\\right)=0$, 即 $x_1=\\frac{1}{2}, x_2=x_3=0$ 时成立.\n由上可以猜想, 一般情况下的极值点为:\n$\\left(\\frac{1}{2}, 0,0, \\cdots, 0\\right)$. 采用磨光变换.\n先证明引理: 若 $0 \\leqslant x, y \\leqslant 1$, 则 $(1- x)(1-y) \\geqslant 1-x-y$. 此式左边直接展开即证.\n此磨光工具相当于 $(x, y) \\rightarrow (x+y, 0)$. 设 $n \\geqslant 2$, 对自变量组 $\\left(x_1, x_2, \\cdots, x_n\\right)$, 不妨设 $x_1 \\geqslant x_2 \\geqslant \\cdots \\geqslant x_n$, 则 $F=\\left(1-x_1\\right)\\left(1-x_2\\right) \\cdots\\left(1-x_n\\right) \\geqslant\\left(1-x_1\\right)\\left(1-x_2\\right) \\cdots\\left(1-x_{n-2}\\right)(1- \\left.x_{n-1}-x_n\\right) \\geqslant\\left(1-x_1\\right)\\left(1-x_2\\right) \\cdots\\left(1-x_{n-3}\\right)\\left(1-x_{n-2}-x_{n-1}-x_n\\right) \\geqslant \\cdots \\geqslant 1- x_1-x_2-\\cdots-x_n \\geqslant \\frac{1}{2}$. 取等号时变量组变为 $\\left(\\frac{1}{2}, 0,0, \\cdots, 0\\right)$, 此时, $F$ 达到最小值 $\\frac{1}{2}$.", + "remark": "", + "figures": [] +} \ No newline at end of file diff --git a/processed_dataset/proof/0599.json b/processed_dataset/proof/0599.json new file mode 100644 index 0000000000000000000000000000000000000000..79b6e9371628b1a239e2371052f28d369bf60852 --- /dev/null +++ b/processed_dataset/proof/0599.json @@ -0,0 +1,8 @@ +{ + "source_file": "./raw_volume-zh/volume13/exercise5.tex", + "problem_type": "proof", + "problem": "问题3. 设 $x_i \\geqslant 0(1 \\leqslant i \\leqslant n), \\sum_{i=1}^n x_i=\\pi, n \\geqslant 2$. 求 $F=\\sum_{i=1}^n \\sin ^2 x_i$ 的最大值.", + "solution": "当 $n=2$ 时, $x_1+x_2=\\pi, F=\\sin ^2 x_1+\\sin ^2 x_2=2 \\sin ^2 x_1 \\leqslant 2$. 其中等式在 $x_1=x_2=\\frac{\\pi}{2}$ 时成立.\n当 $n \\geqslant 3$ 时, 设 $x_3, x_4, \\cdots, x_n$ 为常数, 则 $x_1+x_2$ 亦为常数.\n考察 $\\mathrm{A}=\\sin ^2 x_1+\\sin ^2 x_2, 2-2 \\mathrm{~A}=1-2 \\sin ^2 x_1+1-2 \\sin ^2 x_2= \\cos 2 x_1+\\cos 2 x_2=2 \\cos \\left(x_1+x_2\\right) \\cos \\left(x_1-x_2\\right)$. 为了找到磨光工具, 我们考察 $\\cos \\left(x_1-x_2\\right)$ 的极值以及 $\\cos \\left(x_1+x_2\\right)$ 的符号.\n注意到 $x_1+x_2 \\leqslant \\frac{\\pi}{2}$ 时, $\\cos \\left(x_1+x_2\\right) \\geqslant 0, x_1+x_2>\\frac{\\pi}{2}$ 时, $\\cos \\left(x_1+x_2\\right)<0$. 所以当 $x_1+x_2 \\leqslant \\frac{\\pi}{2}$ 时, $\\left|x_1-x_2\\right|$ 越大, $A$ 越大.\n此时的磨光工具为 $\\left(x_1, x_2\\right) \\rightarrow\\left(x_1+x_2, 0\\right)$; 当 $x_1+ x_2>\\frac{\\pi}{2}$ 时, $\\left|x_1-x_2\\right|$ 越小, $A$ 越大.\n此时需要 $x_1=x_2$. 磨光工具为 $\\left(x_1, x_2\\right) \\rightarrow \\left(\\frac{x_1+x_2}{2}, \\frac{x_1+x_2}{2}\\right)$. 为了保证存在 $x_1 、 x_2$, 使 $x_1+x_2 \\leqslant \\frac{\\pi}{2}$, 一个充分条件是 $n \\geqslant 4$. 于是, $n \\geqslant 4$ 时, 可利用如下的磨光工具.\n引理: 若 $0 \\leqslant x_1 、 x_2 \\leqslant \\frac{\\pi}{2}$, 且 $x_1+x_2 \\leqslant \\frac{\\pi}{2}$, 则 $\\sin ^2 x_1+\\sin ^2 x_2 \\leqslant \\sin ^2\\left(x_1+x_2\\right)$. 实际上, 由 $0 \\leqslant x_1 、 x_2 \\leqslant \\frac{\\pi}{2}$ , $x_1+x_2 \\leqslant \\frac{\\pi}{2}$ 知, $\\left|x_1-x_2\\right| \\leqslant\\left|x_1+x_2\\right| \\leqslant \\frac{\\pi}{2}$. 所以 $\\cos \\left(x_1-x_2\\right) \\geqslant \\cos \\left(x_1+\\right. x_2$ ), 所以 $2-2\\left(\\sin ^2 x_1+\\sin ^2 x_2\\right)=\\cos 2 x_1+\\cos 2 x_2=2 \\cos \\left(x_1+x_2\\right) \\cos \\left(x_1-\\right. \\left.x_2\\right) \\geqslant 2 \\cos \\left(x_1+x_2\\right) \\cos \\left(x_1+x_2\\right)=2 \\cos ^2\\left(x_1+x_2\\right)=2\\left[1-\\sin ^2\\left(x_1+x_2\\right)\\right]= 2-2 \\sin ^2\\left(x_1+x_2\\right)$, 移项, 引理即证.\n下面分情况讨论.\n当 $n=3$ 时, 若三个角为 $\\left(\\frac{\\pi}{2}, \\frac{\\pi}{2}, 0\\right)$, 则调整为 $\\left(\\frac{\\pi}{2}, \\frac{\\pi}{4}, \\frac{\\pi}{4}\\right), F$ 的值由 2 增大到 $1+\\sqrt{2}$, 所以不妨设\n$x_1 \\leqslant x_2 \\leqslant x_3$, 且 $\\left(x_1, x_2, x_3\\right) \\neq\\left(0, \\frac{\\pi}{2}, \\frac{\\pi}{2}\\right)$. 则 $x_2<\\frac{\\pi}{2}, x_1+x_3>\\frac{\\pi}{2}$, $x_1 \\leqslant \\frac{\\pi}{3} \\leqslant x_3$. 将 $\\left(x_1, x_2, x_3\\right)$ 磨光到 $\\left(\\frac{\\pi}{3}, x_2, x_1+x_3-\\frac{\\pi}{3}\\right)$, 由以上叙述可知, $F$ 增大.\n再作一次磨光变换, 便得到 $\\left(\\frac{\\pi}{3}, \\frac{\\pi}{3}, \\frac{\\pi}{3}\\right)$, 所以, $F \\leqslant \\frac{9}{4}$. 当 $n \\geqslant 4$ 时, 不妨设 $x_1 \\geqslant x_2 \\geqslant \\cdots \\geqslant x_{n-1} \\geqslant x_n$, 则必有两个角: $x_{n-1}+x_n \\leqslant \\frac{\\pi}{2}$. 由引理, $F=\\sin ^2 x_1+\\cdots+\\sin ^2 x_{n-1}+\\sin ^2 x_n \\geqslant \\sin ^2 x_1+\\sin ^2 x_2+\\cdots+\\sin ^2\\left(x_{n-1}+\\right. x_n$ ) $=\\sin ^2 x_1^{\\prime}+\\cdots+\\sin ^2 x_{n-2}^{\\prime}+\\sin ^2 x_{n-1}^{\\prime}$ (其中 $x_1^{\\prime}, x_2^{\\prime}, \\cdots, x_{n-2}^{\\prime}, x_{n-1}^{\\prime}$ 是 $x_1$, $x_2, \\cdots, x_{n-2}, x_{n-1}+x_n$ 由大到小排列). 如果 $n-1 \\geqslant 4$, 则必有两个角: $x_{n-2}^{\\prime}+ x_{n-1}^{\\prime} \\leqslant \\frac{\\pi}{2}$. 再继续利用引理进行上述变换, 如此至多进行 $n-3$ 次变换, 可将变量组变为 $\\left(x_1^{\\prime}, x_2^{\\prime}, x_3^{\\prime}, 0,0, \\cdots, 0\\right)$. 再利用 $n=3$ 时的结果, 可知, $F \\leqslant \\frac{9}{4}$, 等号在 $x_1=x_2=x_3=\\frac{\\pi}{3}, x_4=x_5=\\cdots=x_n=0$ 时成立.\n故当 $n=2$ 时, $F$ 的最大值为 2 ; 当 $n>2$ 时, $F$ 的最大值为 $\\frac{9}{4}$.", + "remark": "", + "figures": [] +} \ No newline at end of file diff --git a/processed_dataset/proof/0600.json b/processed_dataset/proof/0600.json new file mode 100644 index 0000000000000000000000000000000000000000..fe0bf5a67aba0f3964a298ea0757d2ea8d629a61 --- /dev/null +++ b/processed_dataset/proof/0600.json @@ -0,0 +1,8 @@ +{ + "source_file": "./raw_volume-zh/volume13/exercise6.tex", + "problem_type": "proof", + "problem": "问题2. 的在 $n \\times n$ 棋盘 $C$ 中, 两个具有公共顶点的格称为是相连的.\n将 $1,2,3, \\cdots$, $n^2$ 分别填人各格中, 每格填一个数.\n若任何相连的两个格的数至多相差 $g$, 则称 $g$ 为一个 $C$-间隙.\n求出最小的 $C$-间隙 $C_g$.", + "solution": "$C_{\\dot{g}}=n+1$. 首先, 第 $i$ 行依次填 $(i-1) n+1,(i-1) n+2, \\cdots,(i-1) n+ n$. 此时, 数表的 $C$-间隙为 $n+1$. 对任何一个数表, 设 $g$ 是它的 $C$-间隙.\n即对任何两个相连的数 $x 、 y$, 有 $|x-y| \\leqslant g$. 我们证明: $g \\geqslant n+1$. 将 1 和 $n^2$ 所在的格用一条链连接, 此链 (包括 1 和 $n^2$ 所在的格) 至多有 $n$ 个格.\n不妨设共有 $m$ 个格 $(m \\leqslant n)$, 这 $m$ 个格中的数依次为 $a_1=1, a_2, a_3, \\cdots, a_m=n^2$. 考察各相连两数之差, 有 $\\left|a_2-a_1\\right|+\\left|a_3-a_2\\right|+\\cdots+\\left|a_m-a_{m-1}\\right| \\geqslant\\left(a_2-a_1\\right)+ \\left(a_3-a_2\\right)+\\cdots+\\left(a_m-a_{m-1}\\right)=a_m-a_1=n^2-1$. 于是, 必有一个 $i$, 使 $\\mid a_i- a_{i-1} \\mid \\geqslant \\frac{n^2-1}{m-1} \\geqslant \\frac{n^2-1}{n-1}=n+1$.", + "remark": "", + "figures": [] +} \ No newline at end of file diff --git a/processed_dataset/proof/0601.json b/processed_dataset/proof/0601.json new file mode 100644 index 0000000000000000000000000000000000000000..666c883e0785c45417863886f2a98bdb6a8dd9e3 --- /dev/null +++ b/processed_dataset/proof/0601.json @@ -0,0 +1,8 @@ +{ + "source_file": "./raw_volume-zh/volume13/exercise6.tex", + "problem_type": "proof", + "problem": "问题4. 设集合 $M=\\{1,2, \\cdots, 10\\}$ 的五元子集 $A_1, A_2, \\cdots, A_k$ 满足条件: $M$ 中的任意两个元素最多在两个子集 $A_i$ 与 $A_j(i \\neq j)$ 内出现, 求 $k$ 的最大值.", + "solution": "记 $i(i=1,2, \\cdots, 10)$ 在 $A_1, A_2, \\cdots, A_k$ 中出现的次数为 $d(i)$. 首先证明 $d(i) \\leqslant 4(i=1,2, \\cdots, 10)$. 事实上,对 $i \\in M, i$ 与 $M$ 中另 9 个元素中的某个元素 $j(j \\neq i)$ 组成二元组 $(i, j)$ 在所有满足题设条件的五元子集 $A_1,A_2, \\cdots, A_k$ 中最多出现两次, 因此 $i$ 参与组成的 9 个二元组 $(i, j)$ 最多出现 $2 \\times 9=18$ 次.\n由于 $\\left|A_j\\right|=5(j=1,2, \\cdots, k)$, 而每个含 $i$ 的子集恰有 4 个二元组 $(i, j)$, 因此 $4 \\cdot d(i) \\leqslant 18$, 故 $d(i) \\leqslant 4$. 其次, $k$ 个 5 元子集共有 $5 k$ 个元素, 而每个 $M$ 的元素在 $A_1, \\cdots, A_k$ 中出现的次数为 $d(i)$, 从而 $5 k= d(1)+\\cdots+d(10) \\leqslant 4 \\times 10$, 所以 $k \\leqslant 8$. 最后, 当 $k=8$ 时, 下述 8 个 5 元数集满足要求: $A_1=\\{1,2,3,4,5\\} 、 A_2=\\{1,6,7,8,9\\} 、 A_3=\\{1,3,5,6$, $8\\} 、 A_4=\\{1,2,4,7,9\\} 、 A_5=\\{2,3,6,7,10\\} 、 A_6=\\{3,4,7,8,10\\}$ 、 $A_7=\\{4,5,8,9,10\\} 、 A_8=\\{2,5,6,9,10\\}$, 故 $k_{\\max }=8$.", + "remark": "", + "figures": [] +} \ No newline at end of file diff --git a/processed_dataset/proof/0602.json b/processed_dataset/proof/0602.json new file mode 100644 index 0000000000000000000000000000000000000000..d2b73ad26dd2ae9fe925c54aece0605c9b7d6302 --- /dev/null +++ b/processed_dataset/proof/0602.json @@ -0,0 +1,8 @@ +{ + "source_file": "./raw_volume-zh/volume13/exercise7.tex", + "problem_type": "proof", + "problem": "问题6. 自然数 $k$ 满足如下性质: 在 $1,2, \\cdots, 1988$ 中, 可取出 $k$ 个不同的数, 使其中任何两个数的和不被这两个数的差整除.\n求 $k$ 的最大值.", + "solution": "$k$ 的最大值为 663. 首先证明 $k \\leqslant 663$. 我们注意如下的事实: 当 $x- y=1$ 或 2 时,有 $x-y \\mid x+y$. 由此可知,在任何连续 3 个自然数中任取两个数 $x 、 y$, 必有 $x-y \\mid x+y . k>663$ 时, 令 $A_i=\\{3 i-2,3 i-1,3 i\\}(i=1$, $2, \\cdots, 662), A_{663}=\\{1987,1988\\}$, 将取出的 $k \\geqslant 664$ 个数归人上述 663 个集合, 至少有一个集合含有其中的两个数 $x 、 y$, 此时具然有 $x-y \\mid x+y$, 矛盾.\n其次, 当 $k=663$ 时, 令 $A=\\{1,4,7, \\cdots, 1987\\}$, 对 $A$ 中的任何两个数 $a_i 、 a_j$, 有 $a_i-a_j=(3 i-2)-(3 j-2)=3(i-j), a_i+a_j=(3 i-2)+(3 j-2)= 3(i+j-1)-1$, 所以 $a_i-a_j$ 不整除 $a_i+a_j$.", + "remark": "", + "figures": [] +} \ No newline at end of file diff --git a/processed_dataset/proof/0603.json b/processed_dataset/proof/0603.json new file mode 100644 index 0000000000000000000000000000000000000000..c6db3e0a8d701a5267925b4d3822d43d509518ba --- /dev/null +++ b/processed_dataset/proof/0603.json @@ -0,0 +1,8 @@ +{ + "source_file": "./raw_volume-zh/volume13/exercise8.tex", + "problem_type": "proof", + "problem": "问题1. 正整数 $n$ 满足如下性质: 在 $1,2, \\cdots, 100$ 中任取 $n$ 个不同的奇数, 必有两个的和为 102 . 求 $n$ 的最小值.", + "solution": "$n$ 的最小值为 27 . 首先证明 $n \\geqslant 27$. 若 $n \\leqslant 26$, 则令 $A=\\{1,3,5, \\cdots$, $2 n-1\\}$, 则 $|A|=n$, 但 $A$ 中任何两个数的和: $(2 i-1)+(2 j-1)=2(i+j)- 2 \\leqslant 2(26+25)-2=100<102$ (其中 $02$, 则 $\\left|A_i \\cap A_j\\right|=3$, 所以 $A_i=A_j$, 矛盾.\n于是, $|F|_{\\text {max }} \\geqslant \\mathrm{C}_{10}^1+\\mathrm{C}_{10}^2+\\mathrm{C}_{10}^3=175$. 上述构造的 $F$ 是饱和的, 即不能再放进任何一个集合.\n但这并不能说明 $|F|$ 最大.\n下面证明 $|F| \\leqslant 175$. 反设 $|F|>175$, 则必有 $F$ 中的一个集合, 设为 $A$, 使 $|A|>3$. 取 $A$ 的任意一个三元子集 $A^{\\prime}$, 由于 $\\left|A \\cap A^{\\prime}\\right|=3>2, A$ 在 $F$ 中, 所以 $A^{\\prime}$ 不在 $F$ 中.\n将 $F$ 中的 $A$ 换作 $A^{\\prime}$, 得到 $F^{\\prime}$, 则 $F^{\\prime}$ 也合乎条件.\n如此下去, 可将 $F$ 中的所有元素个数多于 3 的集合都换作 3 元集合,所得到的 $F^*$ 仍合乎条件.\n但此时 $|F|= \\left|F^*\\right| \\leqslant 175$,矛盾.", + "remark": "", + "figures": [] +} \ No newline at end of file diff --git a/processed_dataset/proof/0605.json b/processed_dataset/proof/0605.json new file mode 100644 index 0000000000000000000000000000000000000000..e51c3b85dc8405da1a79abefb21ed5f076a6fa7d --- /dev/null +++ b/processed_dataset/proof/0605.json @@ -0,0 +1,8 @@ +{ + "source_file": "./raw_volume-zh/volume13/exercise8.tex", + "problem_type": "proof", + "problem": "问题4. 设 $A_i=\\{i, i+1, i+2, \\cdots, i+59\\}(i=1,2, \\cdots, 11), A_{11+j}=\\{11+j$, $12+j, \\cdots, 70,1,2, \\cdots, j\\}(j=1,2, \\cdots, 59)$. 在这 70 个集合中存在 $k$ 个集合,其中任 7 个集合的交非空.\n求 $k$ 的最大值.", + "solution": "令 $A_i=\\{i, i+1, i+2, \\cdots, i+59\\}(i=1,2, \\cdots, 70)$, 其集合中的元素按模 70 理解, 即 $x>70$ 时, 将 $x$ 换作 $x-70$. 显然, $60 \\in A_1, A_2, \\cdots, A_{60}$, 所以 $k_{\\text {max }} \\geqslant 60$. 我们猜想, $k_{\\text {max }}=60$. 这只须证明任何满足条件的 $k$, 有 $k \\leqslant 60$. 用反证法.\n若 $k \\geqslant 61$, 我们证明取出的任何 $k$ 个集合,都能找到 7 个集合的交为空集.\n要找到 7 个集合 $A_{i_1}, A_{i_2}, \\cdots, A_{i_7}$, 其交为空集, 只须它们的补集的并为全集, 即 $\\bar{A}_{i_1} \\cup \\bar{A}_{i_2} \\cup \\cdots \\cup \\bar{A}_{i_7}=I$, 注意到 $\\bar{A}_{i_1}, \\bar{A}_{i_2}, \\cdots, \\bar{A}_{i_7}$ 都是 10 元集, 而 $I$ 有 70 个元素, 要使 $\\bar{A}_{i_1}, \\bar{A}_{i_2}, \\cdots, \\bar{A}_{i_7}$ 包含所有元素, 则它们应在 $A_1, A_2, \\cdots, A_{70}$ 中均匀分布, 即 $A_{i_1}, A_{i_2}, \\cdots, A_{i_7}$ 在 $A_1, A_2, \\cdots, A_{70}$ 中均匀分布, 所以要找的 7 个集合具有形式: $A_i, A_{10+i}, \\cdots, A_{60+i}$. 现在的问题是, 这样的 7 个集合是否都被取出.\n注意到这 7 个集合的共同特征是下标的个位数都是 $i$. 因为取出了 $k \\geqslant 61$ 个集合后, 至多剩下 9 个集合, 它们不能同时含有下标的个位数为 $0,1,2, \\cdots, 9$ 这 10 种可能, 即存在 $0 \\leqslant i \\leqslant 9$, 使 $i$ 不是剩下的 9 . 个集合的下标的个位数.\n也即集合 $A_i, A_{10+i}, \\cdots, A_{60+i}$ 都是取出的集合.\n $A_i \\cap A_{10+i} \\cap \\cdots \\cap A_{60+i}=\\Phi$,矛盾.", + "remark": "", + "figures": [] +} \ No newline at end of file diff --git a/processed_dataset/proof/0606.json b/processed_dataset/proof/0606.json new file mode 100644 index 0000000000000000000000000000000000000000..860c02485f215aa7693903a8d703bc4356451c89 --- /dev/null +++ b/processed_dataset/proof/0606.json @@ -0,0 +1,8 @@ +{ + "source_file": "./raw_volume-zh/volume13/exercise8.tex", + "problem_type": "proof", + "problem": "问题5. 设 $S$ 为集合 $\\{1,2, \\cdots, 108\\}$ 的一个非空子集, 满足: (i) 对 $S$ 中任意的数 $a 、 b$, 总存在 $S$ 中数 $c$,使得 $(a, c)=(b, c)=1$; (ii) 对 $S$ 中任意的数 $a$ 、 $b$, 总存在 $S$ 中数 $c^{\\prime}$, 使得 $\\left(a, c^{\\prime}\\right)>1,\\left(b, c^{\\prime}\\right)>1$. 求 $S$ 中元素个数的最大可能值.", + "solution": "$S$ 中元素个数的最大可能值为 76. 设 $|S| \\geqslant 3, p_1^{\\alpha_1} p_2^{\\alpha_2} p_3^{\\alpha_3} \\in S, p_1 、 p_2$ 、 $p_3$ 为三个不同的素数, $p_11,\\left(c_3, c_2\\right)>1$. 由 $\\left(c_1, c_2\\right)=1$ 知 $c_1 、 c_2$ 的最小素因子之积 $\\leqslant c_3 \\leqslant 108$. 从而 $q \\mid c_1$. 由 $\\left(c_2, c_1\\right)=1,\\left(c_2, p_1^{\\alpha_1} p_2^{\\alpha_2} p_3^{\\alpha_3}\\right)=1,\\left\\{p_1, p_2, p_3, q\\right\\}= \\{2,3,5,7\\}, c_2 \\leqslant 108$ 知 $c_2$ 为大于 10 的素数.\n由 $\\left(c_3, c_2\\right)>1$ 知 $c_2 \\mid c_3$. 又 $1<\\left(c_1, \\frac{c_3}{c_2}\\right)<10,\\left(p_1^{\\alpha_1} p_2^{\\alpha_2} p_3^{\\alpha_3}, c_1\\right)=1$, 故 $\\left(c_1, \\frac{c_3}{c_2}\\right)=q^\\alpha$ (1). 由 (i) 知存在 $c_4 \\in S$, 使 $\\left(c_4, p_1^{\\alpha_1} p_2^{\\alpha_2} p_3^{\\alpha_3}\\right)=1,\\left(c_4, c_3\\right)=1$. 因此 (由 (1) $)\\left(c_4, p_1 p_2 p_3 q\\right)=1$, 即 $\\left(c_4, 2 \\times 3 \\times 5 \\times 7\\right)=1$. 从而 $c_4$ 为大于 10 的素数.\n由 (ii) 知存在 $c_5 \\in S$, 使 $\\left(c_5, c_2\\right)>1,\\left(c_5, c_4\\right)>1$. 所以 $c_2\\left|c_5 、 c_4\\right| c_5$. 又 $c_2 \\mid c_3 、\\left(c_4, c_3\\right)=1$ 知 $\\left(c_2\\right.$, $\\left.c_4\\right)=1$. 所以 $c_2 c_4 \\mid c_5$. 而 $c_2 c_4 \\geqslant 11 \\times 13>108$, 矛盾.\n取 $S_1=\\{1,2, \\cdots$, $108\\} \\backslash\\left(\\{1\\right.$ 及大于 11 的素数 $\\} \\bigcup\\left\\{2 \\times 3 \\times 11,2 \\times 3 \\times 5,2^2 \\times 3 \\times 5,2 \\times 3^2 \\times\\right. \\left.\\left.5,2 \\times 3 \\times 7,2^2 \\times 3 \\times 7,2 \\times 5 \\times 7,3 \\times 5 \\times 7\\right\\}\\right)$. 则 $\\left|S_1\\right|=76$. 下面证明 $S_1$ 满足 (i)、(ii). 若 $p_1^{\\alpha_1} p_2^{\\alpha_2} p_3^{\\alpha_3} \\in S_1, p_11$, $\\left(2 q_1, b\\right)>1$; 若 $b \\neq 3 q_1$, 则 $3 q_1 \\in S_1,\\left(3 q_1, a\\right)>1$, $\\left(3 q_1, b\\right)>1$. (2) $a= 2 \\times 3 \\times 17, b \\neq a$, 则 (1) 可证.\n(3) $a=b$, 由于 5、7、11 中至少有一个不整除 $a$ , 故 (i) 成立.\n若 $a$ 为合数, 取 $a$ 的最小素因子 $p$, 则 $p \\in S_1,(p, a)>1$; 若 $a$ 为素数,则 $a \\leqslant 11,2 a \\in S_1,(2 a, a)>1$. (4) $a 、 b$ 为 $S_1$ 中两个不同的数, $a 、 b$ 均至多含两个不同素因子, $a1,\\left(b, r_1\\right)>1$; 若 $r_1= r_2=a$, 则取 $u=2$ 或 3 , 使 $b \\neq u a$. 则 $u a \\in S,(u a, a)>1$, $(u a, b)>1$; 若\n$r_1 r_2 \\neq a, r_1 r_2 \\neq b$, 则 $r_1 r_2 \\in S_1,\\left(r_1 r_2, a\\right)>1,\\left(r_1 r_2, b\\right)>1$; 若 $r_1 r_2=a$, 则 $r_11$, $\\left(u r_2, b\\right)>1$; 若 $r_1 r_2=b$, 则取 $v=2 、 3 、 5$, 使 $a \\neq v r_1, b \\neq v r_1$, 则 $v r_1 \\in S_1$, $\\left(v r_1, a\\right)>1,\\left(v r_1, b\\right)>1$. 已证 $S_i$ 满足 (i) (ii). 另一方面已可见 $2 \\times 3 \\times 5$ 、 $2^2 \\times 3 \\times 5 、 2 \\times 3^2 \\times 5 、 2 \\times 3 \\times 7 、 2^2 \\times 3 \\times 7 、 2 \\times 5 \\times 7 、 3 \\times 5 \\times 7$ 均不属于 $S$. 现证: $2 \\times 3 \\times 11 、 2 \\times 3 \\times 13 、 5 \\times 7$ 不同时属于 $S$. 反设此三数均属于 $S$. 由 (i) 知存在 $d_1, d_2 \\in S$, 使 $\\left(2 \\times 3 \\times 11, d_1\\right)=1,\\left(5 \\times 7, d_1\\right)=1,(2 \\times 3 \\times 13$, $\\left.d_2\\right)=1,\\left(5 \\times 7, d_2\\right)=1$. 因此 $d_1 、 d_2$ 均为大于 10 的素数.\n由 (ii) 知 $d_1= d_2 \\geqslant 17$. 由(ii) 知存在 $d_3 \\in S$, 使 $\\left(7, d_3\\right)>1,\\left(d_2, d_3\\right)>1$. 因此, $7 d_2 \\mid d_3$. 而 $7 d_2 \\geqslant 7 \\times 17=119$,矛盾.\n另一方面, 大于 10 的素数中至多有一个属于 $S$, $1 \\notin S$, 这样 $|S| \\leqslant 108-7-1-23-1=76$.", + "remark": "", + "figures": [] +} \ No newline at end of file diff --git a/processed_dataset/proof/0607.json b/processed_dataset/proof/0607.json new file mode 100644 index 0000000000000000000000000000000000000000..f4537630636f3bb18f555a53c86925e18cb1adf1 --- /dev/null +++ b/processed_dataset/proof/0607.json @@ -0,0 +1,8 @@ +{ + "source_file": "./raw_volume-zh/volume13/exercise8.tex", + "problem_type": "proof", + "problem": "问题6. 求具有如下性质的最小正整数 $n$ : 将正 $n$ 边形的每一个顶点任意染上红, 黄, 蓝三种颜色之一, 那么这 $n$ 个顶点中一定存在四个同色点, 它们是一个等腰梯形的顶点.", + "solution": "先对 $n \\leqslant 16$ 构造出不满足题目要求的染色方法.\n用 $A_1, A_2, \\cdots, A_n$ 表示正 $n$ 边形的顶点 (按顺时针方向), $M_1 、 M_2 、 M_3$ 分别表示三种颜色的顶点集.\n当 $n=16$ 时, 令 $M_1=\\left\\{A_5, A_8, A_{13}, A_{14}, A_{16}\\right\\}, M_2=\\left\\{A_3, A_6, A_7\\right.$, $\\left.A_{11}, A_{15}\\right\\}, M_3=\\left\\{A_1, A_2, A_4, A_9, A_{10}, A_{12}\\right\\}$. 对于 $M_1$, 点 $A_{14}$ 到另 4 个顶点的距离互不相同, 而另 4 个点刚好是一个矩形的顶点.\n类似于 $M_1$, 可验证 $M_2$ 中不存在 4 个顶点是某个等腰梯形的顶点.\n对于 $M_3$, 其中 6 个顶点刚好是 3 条直径的顶点, 所以任意 4 个顶点要么是某个矩形的 4 个顶点, 要么是某个不等边 4 边形的 4 个顶点.\n当 $n=15$ 时, 令 $M_1=\\left\\{A_1, A_2, A_3, A_5, A_8\\right\\}, M_2=\\left\\{A_6, A_9, A_{13}, A_{14}\\right.$ , $\\left.A_{15}\\right\\}, M_3=\\left\\{A_4, A_7, A_{10}, A_{11}, A_{12}\\right\\}$, 每个 $M_i$. 中均无 4 点是等腰梯形的顶点.\n当 $n=14$ 时, 令 $M_1=\\left\\{A_1, A_3, A_8, A_{10}, A_{14}\\right\\}, M_2=\\left\\{A_4, A_6, A_7\\right.$, $\\left.A_{11}, A_{12}\\right\\}, M_3=\\left\\{A_2, A_6, A_9, A_{13}\\right\\}$, 每个 $M_i$ 中均无 4 点是等腰梯形的顶点.\n当 $n=13$ 时, 令 $M_1=\\left\\{A_5, A_6, A_7, A_{10}\\right\\}, M_2=\\left\\{A_1, A_8, A_{11}, A_{12}\\right\\}$, $M_3=\\left\\{A_2, A_3, A_4, A_9, A_{13}\\right\\}$, 每个 $M_i$ 中均无 4 点是等腰梯形的顶点.\n在上述情形中去掉顶点 $A_{13}$, 染色方式不变, 即得到 $n=12$ 的染色方法; 然后再去掉顶点 $A_{12}$, 即得到 $n=11$ 的染色方法; 继续去掉顶点 $A_{11}$, 得到 $n=$ 10 的染色方法.\n当 $n \\leqslant 9$ 时, 可以使每种颜色的顶点个数小于 4 , 从而无 4 个同色顶点是某个等腰梯形的顶点.\n由此可见, $n \\leqslant 16$ 不具备题目要求的性质.\n下面证明 $n=17$ 时,结论成立.\n反证法.\n反设存在一种将正 17 边形的顶点三染色的方法, 使得不存在 4 个同色顶点是某个等腰梯形的顶点.\n由于 $\\left[\\frac{17-1}{3}\\right]+1=6$, 故必存在某 6 个顶点染同一种颜色, 不妨设为黄色.\n将这 6 个点两两连线, 可以得到 $\\mathrm{C}_6^2=15$ 条线段.\n由于这些线段的长度只有 $\\left[\\frac{17}{2}\\right]=8$ 种可能, 于是必出现如下的两种情况之一:\n(1) 有某 3 条线段长度相同.\n注意到 $3 \\nmid 17$, 不可能出现这 3 条线段两两有公共顶点的情况, 所以存在.\n两条线段, 顶点互不相同.\n这两条线段的 4 个顶点即满足题目要求,矛盾.\n(2) 有 7 对长度相等的线段.\n由假设,每对长度相等的线段必有公共的黄色顶点, 否则能找到满足题目要求的 4 个黄色顶点.\n再根据抽屉原理, 必有两对线段的公共顶点是同一个黄色点.\n这 4 条线段的另 4 个顶点必然是某个等腰梯形的顶点,矛盾.\n所以, $n=17$ 时,结论成立.\n综上所述,所求的 $n$ 的最小值为 17 .", + "remark": "", + "figures": [] +} \ No newline at end of file diff --git a/processed_dataset/proof/0608.json b/processed_dataset/proof/0608.json new file mode 100644 index 0000000000000000000000000000000000000000..7ed80705795968bcabc6c17e4d5a843524e786d9 --- /dev/null +++ b/processed_dataset/proof/0608.json @@ -0,0 +1,8 @@ +{ + "source_file": "./raw_volume-zh/volume13/exercise9.tex", + "problem_type": "proof", + "problem": "问题1. 设 $A_1, A_2, \\cdots, A_{29}$ 是 29 个不同的正整数集合.\n对 $1 \\leqslant i200$.", + "solution": "先改造条件.\n令 $A_i^{\\prime}=\\left\\{A_i\\right.$ 中不大于 1988 的数 $\\}=A_i \\cap\\{1,2, \\cdots$, $1988\\}$, 则 $N_i(1988)=\\left|A_i^{\\prime}\\right|, N_{i j}(1988)=\\left|A_i^{\\prime} \\cap A_j^{\\prime}\\right|$ : 因此本题实质上是要证明存在 $i 、 j(1 \\leqslant i200$. 由题意, 对任何 $i$, $\\left|A_i^{\\prime}\\right|=N_i(1988) \\geqslant \\frac{1988}{\\mathrm{e}}>731$, 所以, $\\left|A_i^{\\prime}\\right| \\geqslant 732$. 令 $X=\\{1,2,3, \\cdots$, $1988\\}$, 则 $A_1^{\\prime}{ }_1, A^{\\prime}{ }_2, \\cdots, A^{\\prime}{ }_{1988}$ 是 $X$ 的子集.\n不妨设 $\\left|A_i^{\\prime}\\right|=732$, 否则, 去掉 $A_i^{\\prime}$ 中的一些元素.\n考察集合元素关系表, 设第 $i$ 行 $m_i$ 个 1 . 计算各元素在子集中出现的总次数, 有 $\\sum_{i=1}^{1988} m_i=S=\\sum_{i=1}^{29}\\left|A_i^{\\prime}\\right|=732 \\times 29$. 再计算各元素在集合对的交集中出现的总次数, 有 $\\sum_{i=1}^{1988} C_{m_i}^2=T=\\sum_{1 \\leqslant i253$. 即必有一个 $A_i^{\\prime} \\cap A_j^{\\prime}$, 使 $\\left|A_i^{\\prime} \\cap A_j^{\\prime}\\right|>200$.", + "remark": "", + "figures": [] +} \ No newline at end of file diff --git a/processed_dataset/proof/0609.json b/processed_dataset/proof/0609.json new file mode 100644 index 0000000000000000000000000000000000000000..c1ab71a26fdda0b298f318f6274ef4e42db6dd7b --- /dev/null +++ b/processed_dataset/proof/0609.json @@ -0,0 +1,8 @@ +{ + "source_file": "./raw_volume-zh/volume13/exercise9.tex", + "problem_type": "proof", + "problem": "问题3. 设 $X$ 是有限集, $A_1, A_2, \\cdots, A_m$ 是 $X$ 的子集, 且 $\\left|A_i\\right|=r(1 \\leqslant i \\leqslant m)$. 若对任何 $i \\neq j$, 有 $\\left|A_i \\cap A_j\\right| \\leqslant k$. 求证: $|X| \\geqslant \\frac{m r^2}{r+(m-1) k}$.", + "solution": "解: 1: 考察 $m$ 个集合与 $n$ 个元素的关系表 $B(n, m)$. 计算表中 1 的个数.\n设第 $i$ 行有 $t_i$ 个 $1(i=1,2, \\cdots, n)$, 那么, $\\sum_{i=1}^n t_i=S=\\sum_{j=1}^m\\left|A_j\\right|=r m$. 再计算各元素在集合对的交集中出现的总次数, 有 $\\sum_{i=1}^n \\mathrm{C}_{t_i}^2=\\mathrm{T}= \\sum_{1 \\leqslant i1$. 当 $n>1$ 时, 对某道题 $A$, 若有 5 人选择了同一答案, 那么, 这 5 人在 $A$ 以外的任何一道题 $B$ 中所选的答案互不相同.\n但题 $B$ 只有 4 个选择支, 矛盾.\n所以, 任何一道题至多只有 4 人选择同一选择支.\n另一方面, 对于题目 $A, 16$ 个人的答案分布到 4 个支中, 又每个支至多 4 个人选择, 从而每个选择支都恰有 4 个人选择.\n这样, 对每个人 $x$, 第 $i(i=1,2, \\cdots, n)$ 道题恰有 3 人与其同答案, 得到一个 3 人组 $A_i$. 从整体上考察 $A_1, A_2, \\cdots, A_n$, 若有两个 3 人组 $A_i 、 A_j(i)所示的 11 条直飞航线合乎要求.\n所以,所有这些航空公司一共至少提供 11 条直飞航线.", + "remark": "", + "figures": [ + "./images/volume13/figures/fig-c9a6.png" + ] +} \ No newline at end of file diff --git a/processed_dataset/proof/0614.json b/processed_dataset/proof/0614.json new file mode 100644 index 0000000000000000000000000000000000000000..8b0142dbb83d0c47c3f8f5bf8aff2b7e7036751a --- /dev/null +++ b/processed_dataset/proof/0614.json @@ -0,0 +1,8 @@ +{ + "source_file": "./raw_volume-zh/volume14/chapter1.tex", + "problem_type": "proof", + "problem": "例4. 求出所有满足下列条件的正整数数列 $x_1, x_2, \\cdots, x_n, \\cdots$ :\n(1) 对每个 $n, x_n \\leqslant n \\sqrt{n}$;\n(2) 对任意不同的正整数 $m 、 n,(m-n) \\mid\\left(x_m-x_n\\right)$.", + "solution": "证明: 由(1)知, $x_1 \\leqslant 1, x_2 \\leqslant 2 \\sqrt{2}$, 所以 $x_1=1, x_2=1$ 或 2 .\n[1] 若 $x_2=1$, 由 (2) 知, $n-1\\left|x_n-x_1, n-2\\right| x_n-x_2$, 即\n$$\nn-1\\left|x_n-1, n-2\\right| x_n-1,\n$$\n由于 $(n-1, n-2)=1$, 所以 $(n-1)(n-2) \\mid x_n-1$.\n若 $x_n \\neq 1$, 则\n$$\nx_n \\geqslant(n-1)(n-2)+1=n^2-3 n+3 .\n$$\n当 $n \\geqslant 9$ 时, $x_n \\geqslant n^2-3 n+3>n(n-3)>n \\sqrt{n}$, 这与 (1) 矛盾, 从而当 $n \\geqslant 9$ 时, $x_n=1$.\n于是对每个 $n(\\geqslant 9), n-i \\mid 1-x_i(i=3,4, \\cdots, 8)$; 所以 $x_3= x_4=\\cdots=x_8=1$.\n因此,数列 $1,1,1, \\cdots$ 是满足题意的一个数列.\n[2] 若 $x_2=2$, 令 $x_n^{\\prime}=x_n-(n-1), n=1,2, \\cdots$. 那么数列 $\\left\\{x_n^{\\prime}\\right\\}$ 是满足题设条件 (1), (2) 的.\n事实上, 对每个 $n, x_n^{\\prime}=x_n-(n-1) \\leqslant n \\sqrt{n}-(n-1) \\leqslant n \\sqrt{n}$, 对任意不同的正整数 $m 、 n, x_n^{\\prime}-x_n^{\\prime}=x_m-(m-1)-x_n+(n-1)=\\left(x_m-x_n\\right)- (m-n)$, 从而 $m-n \\mid x_m^{\\prime}-x_n^{\\prime}$.\n由于 $x_1^{\\prime}=1, x_2^{\\prime}=1$, 由[1]知, $\\left\\{x_n^{\\prime}\\right\\}$ 是常数列 $1,1,1 \\cdots$. 所以, $x_n=n$.\n综上所述, 满足题设的正整数数列有 2 个, 它们是\n$$\n\\begin{aligned}\n& 1,1,1, \\cdots, 1, \\cdots, \\\\\n& 1,2,3, \\cdots, n, \\cdots .\n\\end{aligned}\n$$", + "remark": "注:本题的第(1)种情形是容易解的.\n对于第(2)种情形, 我们通过一个代换, 把它化归为第(1)种情形, 也就是我们曾解决的一个问题, 进而求得解答, 这是一种常用的手法.", + "figures": [] +} \ No newline at end of file diff --git a/processed_dataset/proof/0615.json b/processed_dataset/proof/0615.json new file mode 100644 index 0000000000000000000000000000000000000000..1c4e613f6bebdbc8b473e2903e2225c111851fd8 --- /dev/null +++ b/processed_dataset/proof/0615.json @@ -0,0 +1,8 @@ +{ + "source_file": "./raw_volume-zh/volume14/chapter1.tex", + "problem_type": "proof", + "problem": "例5. 设 $n(>4)$ 是给定的整数, $x_1, x_2, \\cdots, x_n \\in[0,1]$, 求证:\n$$\n2\\left(x_1^3+x_2^3+\\cdots+x_n^3\\right)-\\left(x_1^2 x_2+x_2^2 x_3+\\cdots x_n^2 x_1\\right) \\leqslant n .\n$$", + "solution": "证明:我们先证明一个简单的命题: 若 $x, y \\in[0,1]$, 则 $x^3+y^3 \\leqslant x^2 y+1$.\n事实上,当 $x \\leqslant y$ 时, $x^3 \\leqslant x^2 y, y^3 \\leqslant 1$, 所以 $x^3+y^3 \\leqslant x^2 y+1$;\n当 $x>y$ 时, $x^3 \\leqslant 1, y^3 \\leqslant x^2 y$, 所以 $x^3+y^3 \\leqslant x^2 y+1$.\n于是\n$$\n\\begin{aligned}\n& x_1^3+x_2^3 \\leqslant x_1^2 x_2+1, \\\\\n& x_2^3+x_3^3 \\leqslant x_2^2 x_3+1, \\\\\n& \\cdots \\cdots \\\\\n& x_n^3+x_1^3 \\leqslant x_n^2 x_1+1 .\n\\end{aligned}\n$$\n把上面这 $n$ 个不等式相加, 便得到要证明的命题.", + "remark": "注:化归,有时是将一个问题转化为与它等价的问题, 有时, 新的问题与原来的问题并不等价, 但是, 从新的问题可以很容易得到原问题的解.\n这种不等价的化归并不鲜见, 在不等式的证明中常常用到.", + "figures": [] +} \ No newline at end of file diff --git a/processed_dataset/proof/0616.json b/processed_dataset/proof/0616.json new file mode 100644 index 0000000000000000000000000000000000000000..8fd3cbc5a248675c4026f5240830f931f5843b10 --- /dev/null +++ b/processed_dataset/proof/0616.json @@ -0,0 +1,8 @@ +{ + "source_file": "./raw_volume-zh/volume14/chapter1.tex", + "problem_type": "proof", + "problem": "例6. 设 $a 、 b 、 c$ 为三角形三边长, 证明如下一组结论:\n(1) $(a+b)(b+c)(c+a) \\geqslant 8 a b c$;\n(2) $(a+b-c)(b+c-a)(c+a-b) \\leqslant a b c$;\n(3) $(a+b)(b+c)(c+a)(a+b-c)(b+c-a)(c+a-b) \\leqslant 8 a^2 b^2 c^2$.", + "solution": "解:1) 结论是显然的, 事实上, 根据基本不等式有\n$$\n(a+b)(b+c)(c+a) \\geqslant 2 \\sqrt{a b} \\cdot 2 \\sqrt{b c} \\cdot 2 \\sqrt{c a}=8 a b c .\n$$\n(2) 由已知条件,可令 $\\left\\{\\begin{array}{c}{{a=y+z,}}\\\\ {{b=z+x,}}\\\\ {{c=x+y,}}\\end{array}\\right.\\label{eq1}$ 则$x,y,z>0$.\n要证的式子转化为 $2 z \\cdot 2 x \\cdot 2 y \\leqslant(y+z)(z+x)(x+y)$, 这正是 (1) 的结论.\n(3) 沿用 (2) 中的符号, 则要证的式子转化为\n$$\nx y z(2 x+y+z)(x+2 y+z)(x+y+2 z) \\leqslant(x+y)^2(y+z)^2(z+x)^2 \\label{eq2}\n$$\n由于\n$$\n\\begin{aligned}\nx y(x+y+2 z)^2 & =x y(x+y)^2+4 x y z(x+y+z) \\\\\n& \\leqslant x y(x+y)^2+(x+y)^2 \\cdot z(x+y+z) \\\\\n& =(x+y)^2(x y+z(x+y+z)) \\\\\n& =(x+y)^2(x+z)(y+z),\n\\end{aligned}\n$$\n同理有\n$$\n\\begin{aligned}\n& y z(2 x+y+z)^2 \\leqslant(x+y)(z+x)(y+z)^2, \\\\\n& z x(x+2 y+z)^2 \\leqslant(x+y)(z+x)^2(y+z),\n\\end{aligned}\n$$\n以上三式相乘, 开方即得(2), 故原不等式成立.", + "remark": "注一在证明与三角形边长 $a 、 b 、 c$ 有关的不等式时, 有隐含约束条件 $\\left\\{\\begin{array}{l}a+b>c, \\\\ b+c>a,\\\\ c+a>b .\\end{array}\\right.$ 这些条件有时不便使用, 因而可通过代换 式\\ref{eq1}, 将原问题化归为关于正数 $x 、 y 、 z$ 的代数不等式问题, 这是一种将条件规范化的转化命题的技巧.\n注二第(2)问是不等式证明中的一个典型例子.\n也可以这样证明:\n$$\n\\begin{gathered}\n(a+b-c)(b+c-a)=b^2-(a-c)^2 \\leqslant b^2, \\\\\n(b+c-a)(c+a-b) \\leqslant c^2, \\\\\n(c+a-b)(a+b-c) \\leqslant a^2,\n\\end{gathered}\n$$\n同理得\n$$\n\\begin{aligned}\n& (b+c-a)(c+a-b) \\leqslant c^2 \\\\\n& (c+a-b)(a+b-c) \\leqslant a^2\n\\end{aligned}\n$$\n三式相乘并开方即可.\n值得一提的是第 (3) 问中对 式\\ref{eq2} 的证明恰好借鉴了这种轮换相乘的手法,另一个趣向则是由 (1)、(3) 可以证明 式\\ref{eq2} .\n注三本题中,每一小问的条件均可削弱为 $a 、 b 、 c>0$ (此时,第 (1) 问证明过程不需修改, 读者不妨对 (2)、(3) 的证明过程进行适当补充, 使得对 $a$ 、 $b 、 c>0$ 的一般情形成立).", + "figures": [] +} \ No newline at end of file diff --git a/processed_dataset/proof/0617.json b/processed_dataset/proof/0617.json new file mode 100644 index 0000000000000000000000000000000000000000..05fc5b2a365eddf5e687fb74a9c6ad0882c3c762 --- /dev/null +++ b/processed_dataset/proof/0617.json @@ -0,0 +1,8 @@ +{ + "source_file": "./raw_volume-zh/volume14/chapter1.tex", + "problem_type": "proof", + "problem": "例7. 设 $01)$, 进一步将上述不等式转化为 $2 \\ln t1)$, 因此当 $t>1$ 时 $F(t)$ 单调递减, 故 $F(t)1)$, 即 $2 \\ln t1)$, 从而原不等式成立.", + "remark": "在证明原不等式时, 先通过等价变形和换元, 化归为关于 $t$ 的一个不等式, 同时起到了减少字母和简化约束条件的作用; 此后构造函数 $F(t)=2 \\ln t-\\left(t-\\frac{1}{t}\\right)$, 将问题进一步化归为对函数单调性的讨论, 而 $F(t)$ 的单调性又通过求导得以判断.\n题的求解很简短, 化归思想却蕴含在多个步骤中.\n正如美籍匈牙利数学家 $\\mathrm{G}$ - 波利亚所说, \"不断地变换你的问题\", \"我们必须一再地变换它,重新叙述它, 变换它, 直到最后成功地找到某些有用的东西为止\". 由此可见, 问题转化的思想在数学解题中的重要性.", + "figures": [] +} \ No newline at end of file diff --git a/processed_dataset/proof/0618.json b/processed_dataset/proof/0618.json new file mode 100644 index 0000000000000000000000000000000000000000..467a52adceda942a72ed48a2a4a765a3eed428e2 --- /dev/null +++ b/processed_dataset/proof/0618.json @@ -0,0 +1,8 @@ +{ + "source_file": "./raw_volume-zh/volume14/chapter1.tex", + "problem_type": "proof", + "problem": "例9. 记 $F=\\max _{1 \\leqslant x \\leqslant 3}\\left|x^3-a x^2-b x-c\\right|$. 当 $a 、 b 、 c$ 取遍所有实数时, 求 $F$ 的最小值 .", + "solution": "解:令 $f(x)=(x+2)^3-a(x+2)^2-b(x+2)-c$, 原问题可转化为求 $\\max _{-1 \\leqslant x \\leqslant 1}|f(x)|$ 的最小值, 其中\n$$\n\\begin{aligned}\nf(x) & =x^3+(6-a) x^2+(12-4 a-b) x+(8-4 a-2 b-c) \\\\\n& =x^3+a_1 x^2+b_1 x+c_1 .\n\\end{aligned}\n$$\n将 $6-a 、 12-4 a-b 、 8-4 a-2 b-c$ 分别简记为 $a_1 、 b_1 、 c_1$, 易见 $a 、 b 、 c$ 取遍所有实数当且仅当 $a_1 、 b_1 、 c_1$ 取遍所有实数.\n先证明 $F=\\max _{-1 \\leqslant x \\leqslant 1}|f(x)| \\geqslant \\frac{1}{4}$ : 在 $f(x)$ 表达式中分别取 $x= \\pm 1, \\pm \\frac{1}{2}$, 可得\n$$\n\\begin{gathered}\nf(1)=1+a_1+b_1+c_1 \\leqslant F, \\label{eq1}\\\\\nf(-1)=-1+a_1-b_1+c_1 \\leqslant F, \\label{eq2}\\\\\nf\\left(\\frac{1}{2}\\right)=\\frac{1}{8}+\\frac{a_1}{4}+\\frac{b_1}{2}+c_1 \\leqslant F, \\label{eq3} \\\\\nf\\left(-\\frac{1}{2}\\right)=-\\frac{1}{8}+\\frac{a_1}{4}-\\frac{b_1}{2}+c_1 \\leqslant F, \\label{eq4}\n\\end{gathered}\n$$\n由式\\ref{eq1}, \\ref{eq2}得\n$$\nf(1)-f(-1)=2+2 b_1,\n$$\n由式\\ref{eq3}, \\ref{eq4}得\n$$\nf\\left(\\frac{1}{2}\\right)-f\\left(-\\frac{1}{2}\\right)=\\frac{1}{4}+b_1,\n$$\n所以\n$$\nf(1)-f(-1)-2 f\\left(\\frac{1}{2}\\right)+2 f\\left(-\\frac{1}{2}\\right)\n$$\n$$\n=2+2 b_1-2\\left(\\frac{1}{4}+b_1\\right)=\\frac{3}{2},\n$$\n又\n$$\nf(1)-f(-1)-2 f\\left(\\frac{1}{2}\\right)+2 f\\left(-\\frac{1}{2}\\right) \\leqslant 6 F,\n$$\n因此 $F \\geqslant \\frac{1}{4}$.\n另一方面, 假定 $F=\\frac{1}{4}$, 从上式看出必有\n$$\nf(1)=f\\left(-\\frac{1}{2}\\right)=\\frac{1}{4}, f(-1)=f\\left(\\frac{1}{2}\\right)=-\\frac{1}{4},\n$$\n可确定 $a_1=c_1=0, b_1=-\\frac{3}{4}$, 故而使 $F=\\frac{1}{4}$ 的函数 $f(x)$ 是存在的: $f(x)= x^3-\\frac{3}{4} x$, 因此 $F$ 的最小值为 $\\frac{1}{4}$.", + "remark": "注:相应地, 原问题中使 $F$ 取到最小值的数组 $(a, b, c)= \\left(6,-\\frac{45}{4}, \\frac{13}{2}\\right)$.\n若记 $x^3-a x^2-b x-c=F(x)$, 那么本题开始所做的工作实质上是将 \"求 $\\max _{1 \\leqslant x \\leqslant 3}|F(x)|$ 最小值\" 的问题化归为 \"求 $\\max _{-1 \\leqslant x \\leqslant 1}|F(x+2)|$ 最小值\", 即 \"求 $\\max _{-1 \\leqslant x \\leqslant 1}|f(x)|$ 最小值\" 的问题.\n这样的转化虽不是必须的, 然而, 一旦化归为关于原点对称的区间 $[-1,1]$ 考虑问题, 经过适当取点得到 (1) 至 (4) 式后, 较易观察出如何对 $F$ 进行估计, 运算量也控制在较小的程度 (读者不妨尝试写出不做代换但实质相同的解法, 并比较一下运算量和直观性).", + "figures": [] +} \ No newline at end of file diff --git a/processed_dataset/proof/0619.json b/processed_dataset/proof/0619.json new file mode 100644 index 0000000000000000000000000000000000000000..4244c38c229bd58555974c5a4d77a21c165187b5 --- /dev/null +++ b/processed_dataset/proof/0619.json @@ -0,0 +1,8 @@ +{ + "source_file": "./raw_volume-zh/volume14/chapter10.tex", + "problem_type": "proof", + "problem": "例1. 已知正整数 $p, q, r, a$ 满足 $p q=r a^2$,其中 $r$ 是素数, $p, q$ 互素.\n证明: $p, q$ 中有一个是完全平方数.", + "solution": "证明: $p=p_1^{\\alpha_1} p_2^{\\alpha_2} \\cdots p_k^{\\alpha_k}, q=q_1^{\\beta_1} q_2^{\\beta_2} \\cdots q_l^{\\beta_l}, a=a_1^{\\gamma_1} a_2^{\\gamma_2} \\cdots a_m^{\\gamma_m}$ 为 $p, q, a$ 的标准素因数分解,则有\n$$\np_1^{\\alpha_1} \\cdot p_2^{\\alpha_2} \\cdots p_k^{\\alpha_k} \\cdot q_1^{\\beta_1} \\cdot q_2^{\\beta_2} \\cdots q_l^{\\beta_l}=r a_1^{2 \\gamma_1} a_2^{2 \\gamma_2} \\cdots a_m^{2 \\gamma_m} .\n$$\n由于 $p, q$ 互素且 $r$ 是素数,故 $p, q$ 中不被 $r$ 整除的那个数每个素因子都具有偶数次幂, 它一定是完全平方数.", + "remark": "注:算术基本定理指的是: 任意一个大于 1 的整数 $n$ 有唯一的素因数分解\n$$\nn=p_1^\\alpha p_2^\\alpha \\cdots p_k^{\\alpha_k},\n$$\n其中 $p_1r_{n-1}>\\cdots>r_1 \\geqslant 0, r_i \\in \\mathbf{Z}$.\n于是\n$$\n\\begin{aligned}\nf(m) & =\\left[\\frac{m}{2}\\right]+\\left[\\frac{m}{2^2}\\right]+\\left[\\frac{m}{2^3}\\right]+\\cdots=\\left[\\underline{\\sum} \\frac{2^{r_i}}{2}\\right]+\\left[\\frac{\\sum 2^{r_i}}{2^2}\\right]+\\left[\\frac{\\sum 2^{r_i}}{2^3}\\right]+\\cdots \\\\\n& =\\sum 2^{r_i-1}+\\sum 2^{r_i-2}+\\sum 2^{r_i-3}+\\cdots,\n\\end{aligned}\n$$\n其中和号只对非负指数的项求和.\n进一步有\n$$\nf(m)=\\sum\\left(2^{r_i-1}+2^{r_i-2}+\\cdots+1\\right)=\\sum\\left(2^{r_i}-1\\right)=m-n .\n$$\n所以 $m-f(m)=n$, 即 $m-f(m)$ 等于 $m$ 的二进制表示下非零数字的个数.\n由于存在无穷个正整数 $m$, 使得它们二进制表示中恰有 1000 个非零数字, 从而命题得证.", + "remark": "注:由 $f(m)$ 的定义提示我们把 $m$ 表为二进制的形式, 这样也便于最终描述满足条件的 $m$ 的性质.\n用本题的方法可以得到一个推广的结论:\n对正整数 $m$, 定义 $f_p(m)$ 为 $m$ ! 中素因数 $p$ 的个数 (即满足 $p^k \\mid m$ ! 的最大整数 $k)$, 则 $m$ 在 $p$ 进制表示下的数码之和等于 $\\frac{m-f_p(m)}{p-1}$.", + "figures": [] +} \ No newline at end of file diff --git a/processed_dataset/proof/0623.json b/processed_dataset/proof/0623.json new file mode 100644 index 0000000000000000000000000000000000000000..b9517a380aab7019bfbcc84321defb0c872b208d --- /dev/null +++ b/processed_dataset/proof/0623.json @@ -0,0 +1,8 @@ +{ + "source_file": "./raw_volume-zh/volume14/chapter10.tex", + "problem_type": "proof", + "problem": "例5. 设 $f(n)$ 是 $\\mathbf{N}^*$ 到 $\\mathbf{N}^*$ 的函数, $f(1)=1$, 且对任意 $n \\in \\mathbf{N}^*, \\varepsilon \\in\\{0,1\\}$ 有\n$$\nf(2 n+\\varepsilon)=3 f(n)+\\varepsilon .\n$$\n求函数 $f(n)$ 的值域.", + "solution": "解:计算 $f(n)$ 的一些具体的数值:\n$\\begin{array}{cccccccc}n & 1 & 2 & 3 & 4 & 5 & 6 & \\cdots \\\\ f(n) & 1 & 3 & 4 & 9 & 10 & 12 & \\cdots\\end{array}$\n如果将 $n$ 与 $f(n)$ 分别用二进制和三进制来表示, 重新填写表格, 即为\n$\\begin{array}{cccccccc}n & (1)_2 & (10)_2 & (11)_2 & (100)_2 & (101)_2 & (110)_2 & \\cdots \\\\ f(n) & (1)_3 & (10)_3 & (11)_3 & (100)_3 & (101)_3 & (110)_3 & \\cdots\\end{array}$\n因此猜测 : 对任意正整数 $n=\\left(\\overline{\\left(a_k a_{k-1} \\cdots a_1\\right.}\\right)_2$, 有 $f(n)=\\left(\\overline{\\left(a_k a_{k-1} \\cdots a_1\\right.}\\right)_3$.\n下面对 $n=\\left(\\overline{a_k a_{k-1} \\cdots a_1}\\right)_2$ 的位数 $k$ 用数学归纳法证明上述结论.\n当 $k=1$ 时显然成立.\n设上述结论在 $k$ 位数时成立, 考虑任意一个 $k+1$ 位数 $n_1= \\left(\\overline{a_k a_{k-1} \\cdots a_0}\\right)_2$.\n在 $f(2 n+\\varepsilon)=3 f(n)+\\varepsilon$ 中令 $n=\\left(\\overline{a_k a_{k-1} \\cdots a_1}\\right)_2, \\varepsilon=a_0 \\in\\{0,1\\}$, 由于此时\n$$\n2 n+\\varepsilon=2 \\cdot\\left(\\overline{a_k a_{k-1} \\cdots a_1}\\right)_2+a_0=\\left(\\overline{a_k a_{k-1} \\cdots a_1 a_0}\\right)_2=n_1,\n$$\n故\n$$\nf\\left(n_1\\right)=3 f(n)+a_0=3 \\cdot\\left(\\overline{\\left(a_k a_{k-1} \\cdots a_1\\right.}\\right)_3+a_0=\\left(\\overline{a_k a_{k-1} \\cdots a_1 a_0}\\right)_3,\n$$\n可见 $k+1$ 的情形也成立.\n由数学归纳法可知结论成立.\n因此函数 $f(n)$ 的值域是三进制表示中只含数码 0,1 的一切正整数集合, 即\n$$\n\\left\\{3^{r_1}+3^{r_2}+\\cdots+3^{r_s} \\mid s \\in \\mathbf{N}^*, r_1, r_2, \\cdots, r_s \\in \\mathbf{N}, r_1>r_2>\\cdots>r_s\\right\\} .\n$$", + "remark": "注:本题先进行探究, 计算了 $f(n)$ 的一些具体的值.\n一旦将 $n$ 与 $f(n)$ 的十进制表示替换为适当的进位制 (二进制和三进制), 取值规律就在新的记号下显现无疑,在书写上也带来便利.\n此后的证明是很容易的.\n在下述试题中, 恰好可以引用本题的结论:\n函数 $f: \\mathbf{N}^* \\rightarrow \\mathbf{N}^*$ 适合条件 $f(1)=1$, 且对任何 $n \\in \\mathbf{N}^*$ 有\n$$\n3 f(n) f(2 n+1)=f(2 n)(1+3 f(n)), f(2 n)<6 f(n) .\n$$\n试求方程 $f(k)+f(l)=293, k),不妨设 $\\triangle A B C$ 外接圆为复平面上的单位圆 ( $O$ 为圆心), 且射线 $O M$ 方向为实轴正向.\n由已知条件易得 $D$ 在 $O M$ 延长线上, 且 $O M$ ・ $O D=O A^2=1$.\n不妨设 $A, B$ 分别对应复数 $z, \\bar{z}$, 则点 $M$ 对应的复数为 $\\operatorname{Re} z$, 点 $D$ 对应的复数为 $\\frac{1}{\\operatorname{Re} z}$. 又设点 $C$ 对应复数 $c$.\n显然 $\\angle A C M$ 与 $\\angle B C D$ 均为锐角, 故只需证明 $H=\\frac{c-z}{c-\\frac{1}{\\operatorname{Re} z}}: \\frac{c-\\operatorname{Re} z}{c-\\bar{z}} \\in \\mathbf{R}$, 即证 $\\bar{H}=H$.\n记 $(c-z)(c-\\bar{z})=P,\\left(c-\\frac{1}{\\operatorname{Re} z}\\right)(c-\\operatorname{Re} z)=Q$, 则 $H=\\frac{P}{Q}$.\n注意到 $z \\cdot \\bar{z}=c \\cdot \\bar{c}=1$, 则\n$$\n\\begin{gathered}\n\\bar{P}=(\\bar{c}-\\bar{z})(\\bar{c}-z)=\\left(\\frac{1}{c}-\\frac{1}{z}\\right)\\left(\\frac{1}{c}-\\frac{1}{\\bar{z}}\\right)=\\frac{1}{c^2}(z-c)(\\bar{z}-c)=\\frac{P}{c^2}, \\\\\n\\bar{Q}=\\left(\\frac{1}{c}-\\frac{1}{\\operatorname{Re} z}\\right)\\left(\\frac{1}{c}-\\operatorname{Re} z\\right)=\\frac{1}{c^2 \\operatorname{Re} z}(\\operatorname{Re} z-c)(1-c \\operatorname{Re} z) \\\\\n=\\frac{1}{c^2 \\operatorname{Re} z} \\cdot Q \\operatorname{Re} z=\\frac{Q}{c^2},\n\\end{gathered}\n$$\n故 $\\bar{H}=\\frac{\\bar{P}}{\\bar{Q}}=\\frac{P}{Q}=H$. 从而 $\\angle A C M=\\angle B C D$.", + "remark": "注:本题中 $A, B$ 两点地位对称, 而 $D, M$ 两点与 $A, B$ 关系密切, 因此在不失一般性的前提下, 将 $A, B$ 分别对应单位圆上的复数 $z, \\bar{z}$, 并将需证的角相等的结论转化为\"一个复数式取实数值\"这样一类表述.\n复数法是一种代数方法,然而复数的乘除在模、辐角等方面又具有清晰的几何意义, 这是复数法的优点所在.\n复数的几种表示形式 (代数、三角、指数形式等)在解题中也可适当选用, 它们揭示了代数、三角、几何等知识的联系.\n凡与旋转、位似有关的问题, 常常可以利用复数法求解.", + "figures": [ + "./images/volume14/figures/fig-c10i1.png" + ] +} \ No newline at end of file diff --git a/processed_dataset/proof/0626.json b/processed_dataset/proof/0626.json new file mode 100644 index 0000000000000000000000000000000000000000..e011c51921bee304c808e809cdc8138c59a79673 --- /dev/null +++ b/processed_dataset/proof/0626.json @@ -0,0 +1,8 @@ +{ + "source_file": "./raw_volume-zh/volume14/chapter10.tex", + "problem_type": "proof", + "problem": "例9. 从左到右编号为 $B_1, B_2, \\cdots, B_n$ 的 $n$ 个盒子共装有 $n$ 个小球,每次可以选择一个盒子 $B_k$, 进行如下操作: (1) 若 $k=1$ 且 $B_1$ 中至少有 1 个小球, 则可从 $B_1$ 中移 1 个小球至 $B_2$ 中; (2) 若 $k=n$ 且 $B_n$ 中至少有 1 个小球,则可从 $B_n$ 中移 1 个小球至 $B_{n-1}$ 中; (3) 若 $2 \\leqslant k \\leqslant n-1$ 且 $B_k$ 中至少有 2 个小球, 则可从 $B_k$ 中分别移 1 个小球至 $B_{k+1}$ 和 $B_{k-1}$ 中.\n求证 : 无论初始时这些小球如何放置, 总能经过有限次操作使得每个盒子中恰有 1 个小球.", + "solution": "证明:对于任意两个向量 $\\vec{x}=\\left(x_1, x_2, \\cdots, x_n\\right)$ 和 $\\vec{y}=\\left(y_1, y_2, \\cdots\\right.$, $\\left.y_n\\right)$, 若存在 $1 \\leqslant k \\leqslant n$ 使得 $x_1=y_1, \\cdots, x_{k-1}=y_{k-1}, x_k>y_k$, 则记 $\\vec{x}>\\vec{y}$. 用一非负整数向量 $\\vec{x}=\\left(x_1, x_2, \\cdots, x_n\\right)$ 表示各盒子中的小球数目.\n经过一次对 $B_k$ 的操作后, 各盒子中的小球数目从 $\\vec{x}$ 变为 $\\vec{x}+\\alpha_k$, 其中 $\\alpha_1=(-1,1$, $0, \\cdots, 0), \\alpha_k=(\\underbrace{0, \\cdots, 0}_{k-2 \\uparrow}, 1,-2,1,0, \\cdots, 0)(2 \\leqslant k \\leqslant n-1), \\alpha_n= (0, \\cdots, 0,1,-1)$. 当 $k \\geqslant 2$ 时, 总有 $\\vec{x}+\\alpha_k>\\vec{x}$. 因此, 对于任意初始状态, 总可以通过一系列对 $B_2, \\cdots, B_n$ 的操作 (只要 $k \\geqslant 2$ 且 $B_k$ 中至少有两个小球, 就对 $B_k$ 施行操作), 使得操作后的小球数目 $\\vec{y}=\\left(y_1, y_2, \\cdots, y_n\\right)$ 满足 $y_k \\leqslant 1, \\forall k \\geqslant 2$. 若 $y_2=\\cdots=y_n=1$, 则已经满足题目要求; 否则有 $y_1 \\geqslant 2$. 设 $i$ 是满足 $y_i=0$ 的最小整数, 通过一系列对 $B_1, \\cdots, B_{i-1}$ 的操作, 可以使得小球数目变为 $\\left(y_1-1,1, \\cdots, 1, y_{i+1}, \\cdots, y_n\\right)$. 具体操作如下:\n$$\n\\begin{aligned}\n& \\left(y_1, 1, \\cdots, 1,0, y_{i+1}, \\cdots, y_n\\right) \\stackrel{B_1, B_2, \\cdots, B_{i-1}}{\\longrightarrow}\\left(y_1, 1, \\cdots, 1,0,1, y_{i+1}, \\cdots,\\right. \\\\\n& \\left.y_n\\right) \\stackrel{B_1, B_2, \\cdots, B_{i-2}}{\\longrightarrow}\\left(y_1, 1, \\cdots, 1,0,1,1, y_{i+1}, \\cdots, y_n\\right) \\rightarrow \\cdots \\rightarrow\\left(y_1, 0,\\right. \\\\\n& \\left.1, \\cdots, 1, y_{i+1}, \\cdots, y_n\\right) \\stackrel{B_1}{\\longrightarrow}\\left(y_1-1,1, \\cdots, 1, y_{i+1}, \\cdots, y_n\\right) .\n\\end{aligned}\n$$\n重复以上操作,最终可使小球数目满足题目要求.", + "remark": "注:本题采用 $n$ 维向量的记号, 使表达准确且紧凑.", + "figures": [] +} \ No newline at end of file diff --git a/processed_dataset/proof/0627.json b/processed_dataset/proof/0627.json new file mode 100644 index 0000000000000000000000000000000000000000..59a9dfcdc14d56af32de4ac8d9a8686713d80ddf --- /dev/null +++ b/processed_dataset/proof/0627.json @@ -0,0 +1,8 @@ +{ + "source_file": "./raw_volume-zh/volume14/chapter11.tex", + "problem_type": "proof", + "problem": "例2. 圆 $O_1$ 与圆 $O_2$ 交于 $A 、 B$ 两点.\n过点 $A$ 作直线 $C D \\perp A B$, 交圆 $O_1$ 于 $C$, 交圆 $O_2$ 于 $D$. 过 $C 、 D$ 分别作 $C D$ 垂线 $M C 、 M D$, 使 $M C=M B, N D= N B$. 连接 $M N$, 过点 $B$ 作 $M N$ 的平行线, 交圆 $O_1$ 于 $P$, 交圆 $O_2$ 于 $Q$. 求证: $B P=B Q$.", + "solution": "证明:面给出一个解析几何证法.\n设 $O_1 O_2$ 交 $A B$ 于原点 $O$, 不妨设 $A(0,-1), B(0,1), O_1\\left(x_1, 0\\right)$, $\\mathrm{O}_2\\left(x_2, 0\\right)$.\n圆 $O_1$ 方程为 $\\left(x-x_1\\right)^2+y^2=x_1^2+1$, 即\n$$\nx^2-2 x_1 x+y^2-1=0 . \\label{eq1}\n$$\n同理, 圆 $\\mathrm{O}_2$ 方程为:\n$$\nx^2-2 x_2 x+y^2-1=0 . \\label{eq2}\n$$\n由已知条件及抛物线定义得: $M 、 N$ 在以直线 $C D$ (即直线 $y=-1$ ) 为准线, 以 $B$ 为焦点的抛物线 $x^2=2 p y$ 上.\n其中, $p$ 为 $B$ 到 $C D$ 的距离, 故 $p=2$. 所以\n$$\nx^2=4 y . \\label{eq3}\n$$\n因为 $C D \\perp A B$, 所以 $B C$ 为圆 $O_1$ 的直径, $B D$ 为圆 $O_2$ 的直径, 易知 $C 、 D$ 的坐标分别为 $\\left(2 x_1,-1\\right),\\left(2 x_2,-1\\right)$.\n又 $M C \\perp C D, N D \\perp C D$, 所以 $x_M=x_C=2 x_1, x_N=x_D=2 x_2$. 考虑到 $M 、 N$ 分别满足 \\ref{eq3} 式, 有: $y_M=\\frac{1}{4} x_M^2=x_1^2, y_N=\\frac{1}{4} x_N^2=x_2^2$. 注意 $x_1 \\neq x_2$, 于是 $M N$ 的斜率为 $k=\\frac{y_M-y_N}{x_M-x_N}=\\frac{x_1^2-x_2^2}{2 x_1-2 x_2}=\\frac{x_1+x_2}{2}$, 又 $P Q$ 过点 $B$ 且与 $M N$ 平行, 故其方程应为:\n$$\ny=\\frac{x_1+x_2}{2} x+1 . \\label{eq4}\n$$\n联立 式\\ref{eq1}、\\ref{eq4}, 舍去根 $x=0$, 得: $x_P==\\frac{x_1-x_2}{1+\\left(\\frac{x_1+x_2}{2}\\right)^2}$. 同理得: $x_Q=$\n$$\n\\frac{x_2-x_1}{1+\\left(\\frac{x_1+x_2}{2}\\right)^2}=-x_P .\n$$\n故 $B P 、 B Q$ 在 $x$ 轴上的投影长相等.\n故 $B P=B Q$.", + "remark": "注:本题运用解析法证明, 即: 选择适当的坐标系, 把几何问题转化为代数问题, 经过计算和逻辑推理, 得到有关的代数结论, 再还原成题目所需证明的几何结论.\n在对问题的处理上, 例 1 是 \"以形助数\", 本例是 \"由数解形\", 这是相互对应的两种取向.\n用解析法证明平面几何问题具有模型化、规律性强的特点, 其案例屡见不鲜, 这里不作赘述.", + "figures": [] +} \ No newline at end of file diff --git a/processed_dataset/proof/0628.json b/processed_dataset/proof/0628.json new file mode 100644 index 0000000000000000000000000000000000000000..48ec7068a6b4200f74465d804b055f775f40a170 --- /dev/null +++ b/processed_dataset/proof/0628.json @@ -0,0 +1,10 @@ +{ + "source_file": "./raw_volume-zh/volume14/chapter11.tex", + "problem_type": "proof", + "problem": "例3. 已知正数 $a 、 b 、 c 、 A 、 B 、 C$ 满足 $a+A=b+B=c+C=k$. 求证:\n$$\na B+b C+c A) ), 在边上取 $L 、 M 、 N$, 根据已知条件, 使\n$$\n\\begin{aligned}\n& Q L=A, L R=a, R M=B, M P=b, P N=C, \\\\\n& N Q=c . \\text { 则 }\n\\end{aligned}\n$$\n$$\n\\begin{aligned}\n& S_{\\triangle L R M}=\\frac{1}{2} a B \\sin 60^{\\circ}, S_{\\triangle M P N}=\\frac{1}{2} b C \\sin 60^{\\circ}, \\\\\n& S_{\\triangle N Q L}=\\frac{1}{2} c A \\sin 60^{\\circ}, S_{\\triangle P Q R}=\\frac{1}{2} k^2 \\sin 60^{\\circ} .\n\\end{aligned}\n$$\n由图显然有\n$$\nS_{\\triangle L R M}+S_{\\triangle M P N}+S_{\\triangle N Q L}):\n请读者自己验证我们前面的论述.", + "remark": "注:本问题构造折线图形, 把所有条件都适当地转述为折线的特征, 相当直观, 在此基础上一看图形便知结论.\n另一个问题与本题有相似之处:\n一个环形轨道上有 $n$ 个加油站, 所有加油站的油量总和正好够车跑一圈.\n证明, 总能找到其中一个加油站, 使得初始时油箱为空的汽车从这里出发, 能够顺利环行一圈回到起点.", + "figures": [ + "./images/volume14/figures/fig-c11i4.png" + ] +} \ No newline at end of file diff --git a/processed_dataset/proof/0632.json b/processed_dataset/proof/0632.json new file mode 100644 index 0000000000000000000000000000000000000000..22c1568c30a0c2cf7cbb2f3084dc2344d676f84f --- /dev/null +++ b/processed_dataset/proof/0632.json @@ -0,0 +1,8 @@ +{ + "source_file": "./raw_volume-zh/volume14/chapter11.tex", + "problem_type": "proof", + "problem": "例6. 求证: 对任意八个实数 $a_1, a_2, \\cdots, a_8$ 下列 6 个数\n$$\na_1 a_3+a_2 a_4, a_1 a_5+a_2 a_6, a_1 a_7+a_2 a_8, a_3 a_5+a_4 a_6, a_3 a_7+a_4 a_8, a_5 a_7+a_6 a_8\n$$\n中至少有一个是非负的, 并举例说明这 6 个数可能恰有一个是非负的.", + "solution": "证明:取平面直角坐标系内的点 $A\\left(a_1, a_2\\right), B\\left(a_3, a_4\\right), C\\left(a_5, a_6\\right)$, $D\\left(a_7, a_8\\right)$, 设 $O$ 为坐标原点.\n若 $A, B, C, D$ 中有某个点与 $O$ 重合, 例如 $A$ 与 $O$ 重合, 则 $a_1 a_3+ a_2 a_4=0$ 为非负数.\n以下假定 $A, B, C, D$ 中任意一点不与 $O$ 重合, 不妨设射线 $O A, O B$, $O C, O D$ 依逆时针顺序排列, 根据抽屉原理, 它们的夹角 $\\angle A O B, \\angle B O C$, $\\angle C O D, \\angle D O A$ 中必有一个不超过 $90^{\\circ}$, 不妨设 $0^{\\circ} \\leqslant \\angle A O B \\leqslant 90^{\\circ}$, 则 $a_1 a_3+ a_2 a_4=\\overrightarrow{O A} \\cdot \\overrightarrow{O B} \\geqslant 0$.\n另外, 取 $a_1=3, a_2=1, a_3=-1, a_4=2, a_5=-1, a_6=-2, a_7=3$, $a_8=-1$, 其中仅有 $a_1 a_7+a_2 a_8=8>0$, 则表明这 6 个数中可以恰有一个是非负的.", + "remark": "注:我们考虑形如 $a c+b d$ 的式子的几何意义: $a c+b d$ 可以看成平面直角坐标系中向量 $(a, b),(c, d)$ 的数量积, 而数量积的符号恰能反映两个向量的夹角是否大于 $90^{\\circ}$. 有了这样的认识, 本题便能构造几何模型证明结论.\n在最后举例时, 也不必盲目尝试, 而只需使射线 $O A, O B, O C, O D$ 中有且仅有一对夹角不超过 $90^{\\circ}$, 即可找到相应的 $a_1, a_2, \\cdots, a_8$ 的值.", + "figures": [] +} \ No newline at end of file diff --git a/processed_dataset/proof/0633.json b/processed_dataset/proof/0633.json new file mode 100644 index 0000000000000000000000000000000000000000..cd0a14556e1991bc33b2ae8718c7b459fbfb3b6a --- /dev/null +++ b/processed_dataset/proof/0633.json @@ -0,0 +1,8 @@ +{ + "source_file": "./raw_volume-zh/volume14/chapter11.tex", + "problem_type": "proof", + "problem": "例7. 从空间一点最多可以引出多少条射线, 使得其中每两条射线夹角均为钝角?", + "solution": "解:先, 若由正四面体的中心向它的 4 个顶点各引一条射线, 这 4 条射线两两之间的夹角均为钝角, 满足条件.\n其次证明 5 条射线无法满足题目要求.\n假设从点 $O$ 引出的 5 条射线 $O A_1, O A_2, \\cdots, O A_5$ 两两夹角均为钝角.\n我们用空间向量的坐标来表示射线方向, 不妨设 $\\overrightarrow{O A_5}=(0,0,-1)$, 而\n$$\n\\overrightarrow{O A_i}=\\left(x_i, y_i, z_i\\right)(i=1,2,3,4) .\n$$\n由 $\\overrightarrow{O A_i} \\cdot \\overrightarrow{O A_5}=-z_i$ 可知 $z_i>0(i=1,2,3,4)$.\n记 $A_i$ 在 $x O y$ 平面内的射影为 $P_i$.\n根据抽屈原理, 在 $\\overrightarrow{O P}_i=\\left(x_i, y_i, 0\\right)(i=1,2,3,4)$ 中必有两个向量夹角不大于 $90^{\\circ}$, 不妨设它们是 $\\overrightarrow{O P_1}$ 与 $\\overrightarrow{O P_2}$, 则 $\\overrightarrow{O P_1} \\cdot \\overrightarrow{O P_2}=x_1 x_2+y_1 y_2 \\geqslant 0$, 再结合 $z_1, z_2>0$ 可知 $\\overrightarrow{O A_1} \\cdot \\overrightarrow{O A_2}=x_1 x_2+y_1 y_2+z_1 z_2>0$, 即 $\\overrightarrow{O A_1}$ 与 $\\overrightarrow{O A_2}$ 夹角为锐角, 与条件矛盾!\n综上所述, 自空间一点最多只能引出 4 条射线, 使其中每两条射线夹角均为钝角.", + "remark": "注:数量关系是对几何表象的一种内在支撑.\n本题中, 从直观上并不容易说清楚为何 \"5 条射线无法满足两两之间夹角为钝角\", 于是引人空间向量, 转而考虑代数问题.\n此后, 为了表明平面中四个投影向量 $\\overrightarrow{O P}_i(i=1,2,3,4)$ 必有两个的数量积非负, 又回归几何关系, 用与前一题类似的方式进行处理.\n本题可谓先 \"由形及数\", 再\"由数及形\", 相互转化, 补充互助.", + "figures": [] +} \ No newline at end of file diff --git a/processed_dataset/proof/0634.json b/processed_dataset/proof/0634.json new file mode 100644 index 0000000000000000000000000000000000000000..d348095643e6012c846be3fe8690a4b8163d8908 --- /dev/null +++ b/processed_dataset/proof/0634.json @@ -0,0 +1,8 @@ +{ + "source_file": "./raw_volume-zh/volume14/chapter11.tex", + "problem_type": "proof", + "problem": "例8. 设 $a, b, c, d$ 为整数, $a>b>c>d>0$, 且\n$$\na c+b d=(b+d+a-c)(b+d-a+c) .\n$$\n证明: $a b+c d$ 不是素数.", + "solution": "证明: 由 $a c+b d=(b+d+a-c)(b+d-a+c)=(b+d)^2-(a-c)^2$ 可知 $a^2-a c+c^2=b^2+b d+d^2$. 因此可构造凸四边形 $A B C D$, 使得 $A B=a$, $A D=c, C B=d, C D=b$, 且 $\\angle B A D=60^{\\circ}, \\angle B C D=120^{\\circ}$, 此时 $A, B, C$, $D$ 四点共圆, 且\n$$\nB D^2=a^2-a c+c^2 . \\label{eq1}\n$$\n设 $\\angle A B C=\\alpha$, 则 $\\angle A D C=180^{\\circ}-\\alpha$. 对 $\\triangle A B C$ 与 $\\triangle A D C$ 分别用余弦定理可得\n$$\nA C^2=a^2+d^2-2 a d \\cos \\alpha=b^2+c^2-2 b c \\cos \\left(180^{\\circ}-\\alpha\\right)=b^2+c^2+2 b c \\cos \\alpha .\n$$\n解出 $2 \\cos \\alpha=\\frac{a^2+d^2-b^2-c^2}{a d+b c}$, 所以\n$$\n\\begin{aligned}\nA C^2 & =a^2+d^2-a d \\cdot \\frac{a^2+d^2-b^2-c^2}{a d+b c} \\\\\n& =\\frac{\\left(a^2+d^2\\right)(a d+b c)-a d\\left(a^2+d^2-b^2-c^2\\right)}{a d+b c} \\\\\n& =\\frac{\\left(a^2+d^2\\right) b c+a d\\left(b^2+c^2\\right)}{a d+b c}=\\frac{(a b+c d)(a c+b d)}{a d+b c} . \\label{eq2}\n\\end{aligned}\n$$\n圆内接四边形 $A B C D$ 中, 由托勒密定理得 $A C \\cdot B D=a \\cdot b+c \\cdot d$, 平方得\n$$\nA C^2 \\cdot B D^2=(a b+c d)^2 . \\label{eq3}\n$$\n将 式\\ref{eq1} 和 \\ref{eq2} 代入 式\\ref{eq3} , 整理得 $\\left(a^2-a c+c^2\\right)(a c+b d)=\\left(a b^{\\prime}+c d\\right)(a d+b c)$, 所以\n$$\na c+b d \\mid(a b+c d)(a d+b c) . \\label{eq4}\n$$\n因 $a>b>c>d$, 所以 $a b+c d>a c+b d>a d+b c$, 假如 $a b+c d$ 是素数, 则 $a c+b d$ 与 $a b+c d$ 互素, 结合 式\\ref{eq4} 得 $a c+b d \\mid a d+b c$, 这又与 $a c+b d> a d+b c$ 矛盾! 所以 $a b+c d$ 不是素数.", + "remark": "注:以上证法中, 根据条件的特点巧妙地构造出圆内接四边形 $A B C D$, \"以形助数\"实施证明.\n进一步,解题过程中发现有下面的结论成立:\n若正数 $a, b, c, d$ 满足 $a^2-a c+c^2=b^2+b d+d^2=M$, 则\n$$\n\\frac{(a b+c d)(a d+b c)}{a c+b d}=M\n$$\n这是个纯代数的结论, 能否重新给出一个代数的证明?\n其实不难发现, 只要 $a c+b d \\neq 0$, 就有\n$$\n\\begin{aligned}\n\\frac{(a b+c d)(a d+b c)}{a c+b d} & =\\frac{\\left(a^2+c^2\\right) b d+\\left(b^2+d^2\\right) a c}{a c+b d} \\\\\n& =\\frac{\\left(a^2-a c+c^2\\right) b d+\\left(b^2+b d+d^2\\right) a c}{a c+b d} \\\\\n& =\\frac{M b d+M a c}{a c+b d}=M .\n\\end{aligned}\n$$\n用这样一段文字代替上述证明(4)式之前的部分, 则使证明显著简化.\n然而, 在证明原题时,若要凭空联想到这个结论, 诚然有一定困难.", + "figures": [] +} \ No newline at end of file diff --git a/processed_dataset/proof/0635.json b/processed_dataset/proof/0635.json new file mode 100644 index 0000000000000000000000000000000000000000..00cf32eb551362dd3eff5f902e1c81b031359895 --- /dev/null +++ b/processed_dataset/proof/0635.json @@ -0,0 +1,8 @@ +{ + "source_file": "./raw_volume-zh/volume14/chapter12.tex", + "problem_type": "proof", + "problem": "例5. 对 $n \\in \\mathbf{N}^*$, 记不大于 $n$ 且与 $n$ 互素的所有正整数乘积为 $\\pi(n)$, 求证:\n$$\n\\pi(n) \\equiv \\pm 1(\\bmod n) .\n$$", + "solution": "证明:当 $n=1,2$ 时,命题显然成立.\n以下设 $n \\geqslant 3$.\n设不大于 $n$ 且与 $n$ 互素的正整数构成集合 $X_n$.\n引理: 对任意 $t \\in X_n$, 必有唯一的 $t^{\\prime} \\in X_n$, 使 $t^{\\prime} \\cdot t \\equiv 1(\\bmod n)$.\n证明: 由于 $t, n$ 互素, 则存在整数 $p, q$ 使 $p t+q n=1$, 且此时 $p, n$ 互素.\n取 $t^{\\prime} \\in X_n$ 使 $t^{\\prime} \\equiv p(\\bmod n)$, 可见 $t^{\\prime} \\cdot t \\equiv 1(\\bmod n)$. 又假如 $t_1^{\\prime} \\cdot t \\equiv t_2^{\\prime} \\cdot t \\equiv 1(\\bmod n)$, 则 $t_1^{\\prime} \\equiv t_1^{\\prime} \\cdot t \\cdot t_2^{\\prime} \\equiv t_2^{\\prime}(\\bmod n)$. 因此这样的 $t^{\\prime} \\in X_n$ 是唯一的.\n(1) 当 $t \\neq t^{\\prime}$ 时, 将 $t$ 与 $t^{\\prime}$ 归为一组.\n设这样的组共有 $i$ 组, 其中每组两数之积 $t^{\\prime} \\cdot t \\equiv 1(\\bmod n)$.\n(2) 当 $t=t^{\\prime}$ 时, 有 $t^2 \\equiv 1(\\bmod n)$, 则 $(n--t)^2 \\equiv n^2-2 t n+t^2 \\equiv 1(\\bmod n$ ), 故有 $n-t=(n-t)^{\\prime}$. 将 $t$ 与 $n-t$ 归为一组 (其中 $t \\neq n-t$, 这是因为当 $n$ 为奇数时显然 $t \\neq n-t$, 而当 $n$ 为偶数时 $t$ 不能取 $\\frac{n}{2}$ ). 设这样的组共有 $j$ 组, 其中每组两数之积 $t(n-t) \\equiv-t^2 \\equiv-1(\\bmod n)$.\n由 (1)、(2)可知, $X_n$ 中所有元素经过配对后求积得\n$$\n\\pi(n) \\equiv 1^i \\cdot(-1)^j \\equiv \\pm 1(\\bmod n) .\n$$\n综上, 对一切正整数 $n(n=1,2$ 或 $n \\geqslant 3), \\pi(n) \\equiv \\pm 1(\\bmod n)$.", + "remark": "注:本例采用的实质上是 \"数论倒数\" 及 \"模 $n$ 的相反数\" 的配对方法.\n下面再对本题结果作深人一些的讨论.\n对 $t \\in X_n$, 定义 $t$ 关于 $n$ 的阶是 $r_n(t)=r$, 其中 $r$ 是使 $t^r \\equiv 1(\\bmod n)$ 成立的最小正整数.\n注意到 $r_n(t)=1$ 当且仅当 $t=1, r_n(t)=2$ 当且仅当 $t=t^{\\prime} \\neq$ 1 , 本题的证明揭示了这样的事实: 对 $n \\geqslant 3$, 当 $X_n$ 中 2 阶元个数模 4 余 1 时, $\\pi(n) \\equiv-1(\\bmod n)$; 当 $X_n$ 中 2 阶元个数模 4 余 3 时, $\\pi(n) \\equiv 1(\\bmod n)$. 下面是两个推论:\n推论 1 (Wilson 定理): 若 $p$ 为素数,则 $(p-1) ! \\equiv-1(\\bmod p)$.\n事实上, 对 $p=2$ 可直接验证; 对任何奇素数 $p, p-1$ 是 $X_p$ 中唯一的 2 阶元,因此 $\\pi(p)=(p-1) ! \\equiv-1(\\bmod p)$.\n推论 2 若 $n$ 为偶数,则 $\\pi(2 n) \\equiv 1(\\bmod n)$.\n事实上, 对 $i \\in \\mathbf{Z}, i$ 与 $2 n$ 互素当且仅当 $i$ 与 $n$ 互素, 且 $n+i \\equiv i(\\bmod n)$, 故有\n$$\n\\pi(2 n) \\equiv \\pi^2(n) \\equiv 1(\\bmod n) .\n$$\n一个比推论 2 稍强些的结果是: 若 $4 \\mid n, n \\geqslant 8$, 则 $\\pi(n) \\equiv 1(\\bmod n)$. 这是由于对每个不超过 2 阶的元 $t \\in X_n, 1 \\leqslant t<\\frac{n}{4}$, 四个数 $t, \\frac{n}{2}-t, \\frac{n}{2}+t$, $n-t$ 两两不同, 可归为一大组, 且它们的乘积在模 $n$ 的意义下等于 1 . 这也是配对思想的应用.\n下面两个例题进一步体现了解题中建立对应的技巧.", + "figures": [] +} \ No newline at end of file diff --git a/processed_dataset/proof/0636.json b/processed_dataset/proof/0636.json new file mode 100644 index 0000000000000000000000000000000000000000..15a998144baf79424d729900584964b216d35344 --- /dev/null +++ b/processed_dataset/proof/0636.json @@ -0,0 +1,8 @@ +{ + "source_file": "./raw_volume-zh/volume14/chapter12.tex", + "problem_type": "proof", + "problem": "例7. 若数列 $\\left\\{a_n\\right\\}$ 满足: 对任意 $n \\in \\mathbf{N}^*$, 有 $\\sum_{d \\mid n} a_d=2^n$, 证明: $n \\mid a_n$.", + "solution": "证明:由条件易知 $a_n$ 的值唯一确定.\n定义一个 $0-1$ 序列是 \"长为 $k$ 的循环序列\", 若它的项数为 $k$, 且可由其前 $d$ 项重复 $\\frac{k}{d}$ 次写出来,其中 $d$ 为某个小于 $k$ 且整除 $k$ 的正整数, 同时, 将 $\\frac{k}{d}$ 的最大可能值称为该 0-1 序列的循环次数.\n其余的 $0-1$ 序列称为 \"不循环序列\", 并将循环次数定义为 1 .\n考虑由长为 $n$ 的 $0-1$ 序列组成的集合 $S$ 的元素个数.\n一方面, $S$ 的元素个数显然为 $2^n$.\n另一方面, 对 $S$ 中所有序列按循环次数分类计数.\n显然每个长为 $n$ 的 $0-$ 1 序列有确定的循环次数 $c$, 且 $c$ 必须是 $n$ 的正约数, 对该正整数 $c$, 记 $d=\\frac{n}{c}$. 对每个长为 $n$ 且循环次数是 $c$ 的 $0-1$ 序列, 其前 $d$ 项恰好对应一个长为 $d$ 的 \"不循环序列\"; 反之亦然.\n因此, 若将长为 $m$ 的 \"不循环序列\" 的个数记为 $b_m$, 则 $b_d$ 等于长为 $n$ 且循环次数是 $c$ 的 $0-1$ 序列的个数.\n对所有整除 $n$ 的正整数 $d$ 求和即得 $\\sum_{d|n} b_d=2^n$, 特别地 $b_1=2=a_1$, 根据 $\\left\\{a_n\\right\\}$ 与 $\\left\\{b_n\\right\\}$ 同样的递推关系可知,对一切正整数 $n$,均有 $b_n=a_n$.\n注意到每个长为 $n$ 的\"不循环序列\" 在允许轮换的情况下恰好对应 $n$ 个两两不同的\"不循环序列\", 因此 $n \\mid b_n$, 从而 $n \\mid a_n$.", + "remark": "注:本题中引人长为 $m$ 的 \"不循环序列\" 的个数 $b_m$, 并证明 $b_n=a_n$ 及 $n \\mid b_n$, 十分巧妙.\n证明中有两个关键之处涉及对应的思想方法: 一是将长为 $n$ 且循环次数是 $c$ 的 $0-1$ 序列与长为 $d=\\frac{n}{c}$ 的 \"不循环序列\"一一对应, 得以给出 $\\left\\{b_n\\right\\}$ 的递推公式,最终证得 $b_n=a_n$; 二是将每个长为 $n$ 的 \"不循环序列\" 与通过轮换得到的一组\"不循环序列\" 作\" 1 对 $n$ \" 的对应, 由此说明了 $n \\mid b_n$.", + "figures": [] +} \ No newline at end of file diff --git a/processed_dataset/proof/0637.json b/processed_dataset/proof/0637.json new file mode 100644 index 0000000000000000000000000000000000000000..100083e07aee33356532675b1b218d28b4c2e8e7 --- /dev/null +++ b/processed_dataset/proof/0637.json @@ -0,0 +1,8 @@ +{ + "source_file": "./raw_volume-zh/volume14/chapter12.tex", + "problem_type": "proof", + "problem": "例8. 设 $a_n$ 为下述正整数 $N$ 的个数: $N$ 的各位数字之和为 $n$, 且每位数字只能取 1,3 或 4. 求证: 对 $n \\in \\mathbf{N}^*, a_{2 n}$ 是完全平方数.", + "solution": "证明:记 $A$ 为数码仅有 $1,3,4$ 的数的全体, $A_n=\\{N \\in A \\mid N$ 的各位数码之和为 $n\\}$, 则 $\\left|A_n\\right|=a_n$. 欲证 $a_{2 n}$ 是完全平方数.\n再记 $B$ 为数码仅有 1,2 的数的全体, $B_n=\\{N \\in B \\mid N$ 的各位数码之和为 $n\\}$, 令 $\\left|B_n\\right|=b_n$. 下证 $a_{2 n}=b_n^2$.\n作映射 $f: B \\rightarrow \\mathbf{N}^*$, 对 $N \\in B, f(N)$ 是由 $N$ 按如下法则得到的一个数: 把 $N$ 的数码从左向右看, 凡见到 2 , 把它与后面的一个数相加, 用和代替, 再继续看下去, 直到不能做为止 (例如, $f(1221212)=14$ 132, $f(21121221)= 31341)$. 易知 $f$ 是单射,于是\n$$\nf\\left(B_{2 n}\\right)=A_{2 n} \\cup A_{2 n-2}^{\\prime},\n$$\n其中 $A_{2 n-2}^{\\prime}=\\left\\{10 k+2 \\mid k \\in A_{2 n-2}\\right\\}$. 所以\n$$\nb_{2 n}=a_{2 n}+a_{2 n-2} \\text {. }\n$$\n但 $b_{2 n}=b_n^2+b_{n-1}^2$ (这是因为 $B_{2 n}$ 中的数或是由两个 $B_n$ 中的数拼接, 或是由两个 $B_{n-1}$ 中的数中间放 2 拼接而成), 所以\n$$\na_{2 n}+a_{2 n-2}=b_n^2+b_{n-1}^2, n \\geqslant 2 .\n$$\n因 $a_2=b_1^2=1$, 由上式便知, 对一切 $n \\in N, a_{2 n}=b_n^2$, 即 $a_{2 n}$ 是完全平方数.", + "remark": "", + "figures": [] +} \ No newline at end of file diff --git a/processed_dataset/proof/0638.json b/processed_dataset/proof/0638.json new file mode 100644 index 0000000000000000000000000000000000000000..25f04d8351cb641a1edd4ab1aec11839553fd302 --- /dev/null +++ b/processed_dataset/proof/0638.json @@ -0,0 +1,8 @@ +{ + "source_file": "./raw_volume-zh/volume14/chapter13.tex", + "problem_type": "proof", + "problem": "例3. 在 $2 \\times n$ 的方格表的 $2 n$ 个方格中, 每格用黑、白两色之一染色.\n若要求任何两个有公共边的方格不都染黑色,求不同的染色方案的数目.", + "solution": "解:递推方法.\n设满足条件的染色方案共有 $a_n$ 种, 其中在第 $n$ 列中均染白色的染色方案共 $b_n$ 种, 因而第 $n$ 列染不同色的染色方案数为 $a_n-b_n$.\n当 $n \\geqslant 2$ 时,方格表的前 $n-1$ 列一定是 $2 \\times(n-1)$ 方格表的一种满足要求的染色方案.\n当第 $n-1$ 列均染白色时, 第 $n$ 列 (从上到下,下同) 对应有\"白白\"、\"黑白\"、\"白黑\" 3 种染色方法; 当第 $n-1$ 列染不同色时, 不妨设为 \"黑白\", 则第 $n$ 列对应有\"白白\"、\"白黑\"2 种染色方法.\n因此可建立递推关系:\n$$\na_n=3 b_{n-1}+2\\left(a_{n-1}-b_{n-1}\\right)=2 a_{n-1}+b_{n-1} .\n$$\n另一方面, $2 \\times(n-1)$ 方格表的每一种染色方案与 $2 \\times n$ 方格表每一种第 $n$ 列染\"白白\"的染色方案一一对应,故 $b_n=a_{n-1}$.\n因此 $n \\geqslant 3$ 时,有 $a_n=2 a_{n-1}+a_{n-2}$, 又验证知 $a_1=3, a_2=7$, 故解得\n$$\na_n=\\frac{1}{2}\\left((1+\\sqrt{2})^{n+1}+(1-\\sqrt{2})^{n+1}\\right) .\n$$", + "remark": "注一:递推关系具有形式多样性.\n比较多的问题中所利用的是单递推关系, 但也有一些是利用多元递推关系.\n本题中 $\\left\\{a_n\\right\\}$ 的递推关系并不明显, 因而将染色方案分为\"第 $n$ 列中均染白色\"与 \"第 $n$ 列染不同色\"两种类型,引人辅助量 $b_n$ 参与递推关系的建立, 最后消去 $b_n$, 即得到 $\\left\\{a_n\\right\\}$ 的递推关系.\n注二:与之等价的问题有:\n设数列 $\\left\\{a_n\\right\\},\\left\\{b_n\\right\\}$ 每项均为 0 或 1 , 且满足\n$$\na_k a_{k+1}=b_k b_{k+1}=a_k b_k=0\\left(k \\in \\mathbf{N}^*\\right) .\n$$\n对给定正整数 $n$, 求 $2 n$ 个数 $a_1, a_2, \\cdots, a_n, b_1, b_2, \\cdots, b_n$ 的不同的取值方法总数.", + "figures": [] +} \ No newline at end of file diff --git a/processed_dataset/proof/0639.json b/processed_dataset/proof/0639.json new file mode 100644 index 0000000000000000000000000000000000000000..a72d1685b9995ae1f298f0a32d674a87e5798195 --- /dev/null +++ b/processed_dataset/proof/0639.json @@ -0,0 +1,8 @@ +{ + "source_file": "./raw_volume-zh/volume14/chapter13.tex", + "problem_type": "proof", + "problem": "例6. 设非负实数数列 $\\left\\{a_n\\right\\},\\left\\{b_n\\right\\}$ 满足: 对任意 $i, j \\in \\mathbf{N}^*,|i-j| \\in \\{2011,2012\\}$, 都有 $a_i+b_j \\leqslant a_i b_j$. 求证: $\\left\\{a_n\\right\\}$ 与 $\\left\\{b_n\\right\\}$ 中的所有项不是全大于 1 , 就是全等于 0 .", + "solution": "证明:果对任意 $n \\in \\mathbf{N}^*$, 有 $a_n, b_n>1$, 则命题已成立.\n以下考虑 $\\left\\{a_n\\right\\}$, $\\left\\{b_n\\right\\}$ 中存在某项 (不妨设为 $a_k$ ) 小于 1 的情况.\n因不等式 $a_i+b_j \\leqslant a_i b_j$ 等价于 $\\left(1-a_i\\right)\\left(1-b_j\\right) \\geqslant 1$, 若 $0 \\leqslant a_i<1$, 则\n$$\n1 \\geqslant 1-b_j \\geqslant \\frac{1}{1-a_i} \\geqslant 1 \\text {, }\n$$\n可见不等号均为等号, 所以 $a_i=b_j=0$.\n由于 $0 \\leqslant a_k<1$, 故令 $(i, j)=(k, k+2011),(k, k+2012)$, 得\n$$\na_k=b_{k+2011}=b_{k+2012}=0 .\n$$\n再令 $(i, j)=(k-1, k+2011),(k+1, k+2012)$ (若已有 $k=1$ 则省略前者), 得\n$$\na_{k-1}=a_{k+1}=0 \\text {. }\n$$\n以此类推得\n$$\na_{k-2}=a_{k-3}=\\cdots=a_1=0, a_{k+2}=a_{k+3}=\\cdots=0 \\text {, }\n$$\n即 $\\left\\{a_n\\right\\}$ 中每项都是 0 . 又 $\\left\\{b_n\\right\\}$ 中已有一项是 0 , 同样递推得 $\\left\\{b_n\\right\\}$ 中每项都是 0 . 命题成立.", + "remark": "注:本题的大致求解思路是: 若存在某项小于 1 , 先确定它等于 0 , 再充分利用条件证明它的前后相邻项都等于 0 , 再往正向和逆向递推, 就说明了所有项都是 0 . 对另一个数列同理可验证所有项为 0 .\n在同样的求解思路下,本题可将条件中的\"2011,2012\"推广至更一般的情形: 当正整数 $p, q$ 互素,且不均为奇数时,命题仍能成立.", + "figures": [] +} \ No newline at end of file diff --git a/processed_dataset/proof/0640.json b/processed_dataset/proof/0640.json new file mode 100644 index 0000000000000000000000000000000000000000..b8dc9b03449e9b23710373371f7828e80ad724bf --- /dev/null +++ b/processed_dataset/proof/0640.json @@ -0,0 +1,8 @@ +{ + "source_file": "./raw_volume-zh/volume14/chapter13.tex", + "problem_type": "proof", + "problem": "例7. 设 $n$ 为大于 1 的奇数, $\\alpha$ 是 $P(x)=(x-1)^n-x^2$ 的零点, 证明 $\\alpha> 2+\\frac{1}{n}$.", + "solution": "证明: $\\alpha<1$, 由于 $n$ 为奇数,则 $P(\\alpha)=(\\alpha-1)^n-\\alpha^2<0$,矛盾.\n所以 $\\alpha \\geqslant 1$.\n若 $1 \\leqslant \\alpha<2$, 则 $(\\alpha-1)^n<1 \\leqslant \\alpha^2$, 故 $P(\\alpha)<0$, 矛盾.\n所以 $\\alpha \\geqslant 2$.\n若 $2 \\leqslant \\alpha \\leqslant 2+\\frac{1}{n}$, 则 $(\\alpha-1)^n \\leqslant\\left(1+\\frac{1}{n}\\right)^n<\\mathrm{e}<4 \\leqslant \\alpha^2$, 故 $P(\\alpha)<0$, 矛盾.\n所以 $\\alpha>2+\\frac{1}{n}$.", + "remark": "注:本题是个关于多项式零点分布的问题, 但若简单用导数研究 $P(x)$ 的单调性并不足以解决问题, 因此我们通过三个步骤来递推讨论零点的范围: 先选取第一个范围 $\\alpha<1$, 此时非常容易证明 $\\alpha$ 不是零点, 这样就将所需考虑的范围缩小为 $\\alpha \\geqslant 1$, 故而为第二步讨论创设了有利条件 $\\alpha^2 \\geqslant 1$; 对第二个范围 $1 \\leqslant \\alpha<2$ 的选取也是根据同样的策略,这样为第三步创设了条件 $\\alpha^2 \\geqslant 4$; 到第三步时再发起总攻, 把问题完全解决.\n这种步步为营、层层推进的做法有利于降低推理难度,有时候反而能更快且干净地解决问题.\n在解决某些数学问题时,我们还可采取 \"递归\"的思想方法, 粗略地讲, 其中包括\"回溯\"和\"递推\"两个过程.\n例如对某种规模为 $n$ 的问题, 将其降解成若干个规模小于 $n$ 的问题,依次降解直到问题规模可求; 再求出低阶规模的解, 逐次代入高阶问题中, 直至求出规模为 $n$ 的问题的解.", + "figures": [] +} \ No newline at end of file diff --git a/processed_dataset/proof/0641.json b/processed_dataset/proof/0641.json new file mode 100644 index 0000000000000000000000000000000000000000..28134d34767fc88eb098c6c223977ab6f7fd6c06 --- /dev/null +++ b/processed_dataset/proof/0641.json @@ -0,0 +1,8 @@ +{ + "source_file": "./raw_volume-zh/volume14/chapter13.tex", + "problem_type": "proof", + "problem": "例8. 一开始桌上放着 3 堆火柴, 其中一堆的根数是另两堆之和.\n两人依次轮流做取火柴游戏: 游戏者每次任意取走其中一堆,并把余下两堆中的任意一堆分成非空的两堆.\n谁无法这样做, 就算输了.\n证明或否定: 先取火柴的一方有必胜策略,并说明理由.", + "solution": "解:若 3 堆火柴根数分别为 $2^{n_1} a_1, 2^{n_2} a_2, 2^{n_3} a_3$ (其中 $n_1, n_2, n_3 \\in \\mathbf{N}$, $a_1, a_2, a_3$ 为奇数), 那么当 $n_1, n_2, n_3$ 不全相等时, 定义这个状态为 $W$; 当 $n_1=n_2=n_3$ 时, 定义这个状态为 $L$.\n首先说明: 在状态 $W$ 下总可以进行游戏的下一步, 并且能通过适当的策略得到某个状态 $L$. 事实上, 不妨设 $n_1 \\leqslant n_2 \\leqslant n_3$, 根据状态 $W$ 的定义, 有 $n_1< n_3$, 注意此时第三堆火柴多于一根, 必能分堆, 并且可这样操作: 取走第二堆火柴, 再将第三堆分成根数为 $2^{n_1}, 2^{n_1}\\left(2^{n_3-n_1} a_3-1\\right)$ 的两堆, 那么三堆火柴根数可以写成 $2^{n_1} a_1, 2^{n_1}, 2^{n_1}\\left(2^{n_3-n_1} a_3-1\\right)$, 其中 $a_1, 1,2^{n_3-n_1} a_3-1$ 均为奇数, 这是一个状态 $L$.\n其次说明, 状态 $L$ 下若能进行操作, 则操作后只能得到状态 $W$. 事实上, 在状态 $L$ 下三堆火柴根数可写成 $2^n a_1, 2^n a_2, 2^n a_3\\left(n \\in \\mathbf{N}, a_1, a_2, a_3\\right.$ 为奇数) 的形式.\n假设操作后仍得到一个状态 $L$, 不妨设操作中第三堆火柴未动, 那么新的三堆火柴根数必是 $2^n b_1, 2^n b_2, 2^n a_3$ ( $b_1, b_2, a_3$ 为奇数) 的形式, 其中 $b_1+b_2$ 等于 $a_1$ 或 $a_2$, 但这与 $a_1, a_2, b_1, b_2$ 为奇数相矛盾.\n故状态 $L$ 操作后必变为状态 $W$.\n最后说明游戏的初始状态为 $W$. 事实上, 假设此时 $2^{n_1} a_1+2^{n_2} a_2=2^{n_3} a_3$ 且为状态 $L$, 即 $n_1=n_2=n_3=n$, 则 $a_1+a_2=a_3$, 与 $a_1, a_2, a_3$ 为奇数矛盾.\n考虑到每步操作都将使火柴数减少, 故游戏必在有限步内结束, 那么综上可知, 本题的结论是肯定的, 即先取火柴的一方有必胜策略.", + "remark": "注:数学竞赛中常常出现这样的双人博弯问题: 博弯是有限的、零和的, 对局双方依照规则轮流进行操作, 直至判定胜负.\n当双方均采取最佳方案时, 须确定必胜的一方 (有时候是要确定不败的一方). 此类博恋问题的一般解决步骤为:\n(1) 手推小数据,对\"胜状态\" (即可以采取适当操作使最终获胜的局面)和 \"负状态\" (即无论进行何种操作, 对方总能适当操作取胜的局面)作出合理猜测;\n(2)验证从每个胜状态确实可以适当操作得到一个负状态;\n(3)验证从每个负状态无论进行何种操作均只能得到胜状态.\n这样就能确定每种初始状态下的胜负结果.\n本题改编自 1994 年俄罗斯数学奥林匹克试题.\n还可以进一步考虑这样的问题: 如果初始状态是 4 堆火柴, 其中一堆的根数是另外某两堆之和, 那么按照司样规则进行游戏, 先取火柴的一方是否仍有必胜策略.", + "figures": [] +} \ No newline at end of file diff --git a/processed_dataset/proof/0642.json b/processed_dataset/proof/0642.json new file mode 100644 index 0000000000000000000000000000000000000000..8b29363a2048c5130ad3f9c99e67f2e11a8f5239 --- /dev/null +++ b/processed_dataset/proof/0642.json @@ -0,0 +1,8 @@ +{ + "source_file": "./raw_volume-zh/volume14/chapter14.tex", + "problem_type": "proof", + "problem": "例1. 证明: 任意 6 人中, 或者有 3 人互相认识, 或者有 3 人互不相识.", + "solution": "证明: 6 个人视为 6 个点 $A_1, A_2, \\cdots, A_6$, 两人相识就在对应的顶点间连一条红边, 否则就连一条蓝边.\n这样构成一个图.\n问题转化为证明: 该图中必有同色三角形.\n考虑 $A_1$ 引出的 5 条边 $A_1 A_2, A_1 A_3, \\cdots, A_1 A_6$, 根据抽屉原理, 必有 3 条边染有同一种颜色.\n不妨设 $A_1 A_2, A_1 A_3, A_1 A_4$ 都染了红色.\n若 $\\triangle A_2 A_3 A_4$ 三边均为蓝色, 则结论已成立; 若 $\\triangle A_2 A_3 A_4$ 三边中有一条红色, 不妨设 $A_2 A_3$ 为红色,则 $\\triangle A_1 A_2 A_3$ 为红色三角形, 结论仍成立.", + "remark": "注:这是染色问题中一个经典例子, 它存在各种各样的变形.\n通过对边进行染色,我们把问题转化为证明图论中一个著名的拉姆赛 (Ramsey) 型命题: 2 色完全图 $K_6$ 中必存在同色三角形.\n此外,每条边既已染色,我们还可以从\"同色角\"(即由两条同色边组成的角) 的计数出发, 给出另一种证明如下:\n设同色三角形个数为 $n$. 考虑图 $K_6$ 中的同色角个数 $S$.\n一方面,每个同色三角形有 3 个同色角,非同色三角形有一个同色角, 所以 $S=3 n+\\left(\\mathrm{C}_6^3-n\\right)=2 n+20$; 另一方面, 如果一个顶点引出 $k$ 条红边, 则它引出 $5-k$ 条蓝边, 故以该点为顶点的同色角个数为 $\\mathrm{C}_k^2+\\mathrm{C}_{5-k}^2$, 其最小值为 4 ,\n从而 $S \\geqslant 6 \\times 4=24$. 综合两方面可知 $n \\geqslant 2$, 即有更强的结论成立: 2 色完全图 $K_6$ 中至少有两个同色三角形.\n以上的两种证明方法均可以视作证明图的染色问题的典型方法.", + "figures": [] +} \ No newline at end of file diff --git a/processed_dataset/proof/0643.json b/processed_dataset/proof/0643.json new file mode 100644 index 0000000000000000000000000000000000000000..c846ac1d85aa84c54acf9771b2eded1335a71e18 --- /dev/null +++ b/processed_dataset/proof/0643.json @@ -0,0 +1,11 @@ +{ + "source_file": "./raw_volume-zh/volume14/chapter14.tex", + "problem_type": "proof", + "problem": "例3. 有一张 $4 \\times 8$ 的方格棋盘.\n求证: …只 \"马\"不能从一格出发, 遍历这张棋盘的每一方格恰好一次,最后回到原出发点.", + "solution": "证明:假设存在一条满足题目所述条件的\"马\" 的路径, 那么不妨将左上角 $A$ 格定为它的出发点, 最后它又回到 $A$ 格.\n先将棋盘按黑白相间方式染色(如图(), $A$ 在白格). 每步\"马\"一定从一种颜色的格子跳人另一种颜色的格子, 因此 \"马\" 奇数步走遍一切黑格, 偶数步走遍一切白格.\n另一方面, 若将第 $1 、 4$ 行染为白色, 第 $2 、 3$ 行染为黑色 (如图()), 由于\"马\"从白格只能跳人黑格, 因此为了遍历每一格后恰好回到出发点, \"马\" 每次在黑格时也必跳向白格, 故\"马\" 奇数步走遍一切黑格, 偶数步走遍一切白格.\n显然两种染色意义下的黑格全体不同, 故假设不成立.\n从而命题得证.", + "remark": "注:本例使用了两种染色方法.\n第一种染色方法与国际象棋棋盘相似, 这种黑白相间的染色方法往往称作 \"自然染色\", 在解题中常与配对、奇偶性等有内在关系, 可谓一种常规的染色方法; 后一种染色方法则体现了求解具体问题的灵活性, 可谓非常规方法.\n在本题中, 我们充分考虑棋盘与棋子的特性, 将常规方法与非常规方法配套使用, 每种方法各揭示了棋子遍历棋盘过程中的一种不变规律,再相互对比导致矛盾,使命题获证.", + "figures": [ + "./images/volume14/figures/fig-c14i2.png", + "./images/volume14/figures/fig-c14i3.png" + ] +} \ No newline at end of file diff --git a/processed_dataset/proof/0644.json b/processed_dataset/proof/0644.json new file mode 100644 index 0000000000000000000000000000000000000000..88f664aff5795f2387f8272f088c2e994b9b0545 --- /dev/null +++ b/processed_dataset/proof/0644.json @@ -0,0 +1,10 @@ +{ + "source_file": "./raw_volume-zh/volume14/chapter14.tex", + "problem_type": "proof", + "problem": "例4. 甲、乙两人在一张无限大的方格棋盘上一人一步轮流下棋, 甲先走, 乙后走, 每步棋可将一枚棋子放人任意一个尚未棋子的方格中.\n谁先在棋盘上横着或坚着连出 5 枚自己所放的棋子, 判谁获胜.\n问: 甲是否有必胜策略?", + "solution": "解:我们证明乙存在一种使甲无法获胜的策略, 因而甲没有必胜策略.\n如图(),将棋盘按 $2 \\times 2$ 为一\"大格\"划分, 然后将这些大格黑白相间染色.\n当甲在黑格中放一枚棋子时,乙就在横向相邻的黑格中也放一枚棋子; 当甲在白格中放一枚棋子时,乙就在纵向相邻的白格中也放一枚棋子.\n在这种策略下, 显然当甲每步放棋子后, 乙所需放棋子的位置必然空着, 故乙的策略可以一直执行下去.\n考虑到任何横向连出的 5 枚棋子中必有两枚在相邻黑格中, 任何纵向连出的 5 枚棋子中必有两枚在相邻白格中, 但这两种情形都被乙破坏, 故甲无法获胜.", + "remark": "注:本题类似于\"五子棋\"游戏, 只是斜向连出 5 枚棋子不算获胜, 根据下五子棋的经验, 不妨大胆预测甲没有必胜策略.\n余下的问题是乙究竟怎样阻止甲获胜.\n不妨考虑采用配对思想, 例如将棋盘自然染色, 并把相邻的黑格与白格两两配对, 甲每下一子, 乙就在旁边配对的位置放一子进行 \"防守\". 此时, 为了兼顾横向和纵向的防守, 须适当调整配对的秩序, 为此引人 \"大格\" 的概念, 对大格进行自然染色, 在黑格与白格中分别进行\"横向配对\"与 \"纵向配对\",在此基础上便能构造出一种使乙不败的策略.", + "figures": [ + "./images/volume14/figures/fig-c14i4.png" + ] +} \ No newline at end of file diff --git a/processed_dataset/proof/0645.json b/processed_dataset/proof/0645.json new file mode 100644 index 0000000000000000000000000000000000000000..6ee2dfbda0610073c2f6c987376141f019ea2831 --- /dev/null +++ b/processed_dataset/proof/0645.json @@ -0,0 +1,18 @@ +{ + "source_file": "./raw_volume-zh/volume14/chapter14.tex", + "problem_type": "proof", + "problem": "例5. 在一个 $10 \\times 10$ 的方格表中有一个由 $4 n$ 个 $1 \\times 1$ 的小方格组成的图形, 它既可被 $n$ 个\"图()\"型的图形覆盖, 也可被 $n$ 个\"图()\"或\"图()\"型(可以旋转) 的图形覆盖.\n求正整数 $n$ 的最小值.", + "solution": "解:首先论证 $n$ 是偶数.\n用如图() 所示方法将平面网格染色.\n记图()的图形为A形, 图()和图()可以旋转的图形为 $B$ 形.\n无论 $A$ 形覆盖哪 4 个方格, 其中黑格数必是偶数,而对于 $B$ 形则是奇数.\n如果 $n$ 是奇数, $n$ 个 $A$ 形所覆盖的黑方格数必是偶数; 而 $n$ 个 $B$ 形所覆盖的黑方格数必是奇数, 矛盾.\n所以 $n$ 必是偶数.\n如果 $n=2$, 由 2 个 $A$ 形拼成的图形只有如图() 所示的两种情形,但是它们都不能由 2 个 $B$ 形拼成.\n所以, $n \\geqslant 4$. 如图() 是 $n=4$ 时的拼法.", + "remark": "注:本题采用的染色方法与上一题相同, 但从不同的角度考察染色的特性, 因而解决了一个全然不同的问题.", + "figures": [ + "./images/volume14/figures/fig-c14e1.png", + "./images/volume14/figures/fig-c14e2.png", + "./images/volume14/figures/fig-c14e3.png", + "./images/volume14/figures/fig-c14i5.png", + "./images/volume14/figures/fig-c14e1.png", + "./images/volume14/figures/fig-c14e2.png", + "./images/volume14/figures/fig-c14e3.png", + "./images/volume14/figures/fig-c14i6.png", + "./images/volume14/figures/fig-c14i7.png" + ] +} \ No newline at end of file diff --git a/processed_dataset/proof/0646.json b/processed_dataset/proof/0646.json new file mode 100644 index 0000000000000000000000000000000000000000..e8905de16ae5d7ddf4f3e674461d3bca5e04dde7 --- /dev/null +++ b/processed_dataset/proof/0646.json @@ -0,0 +1,8 @@ +{ + "source_file": "./raw_volume-zh/volume14/chapter14.tex", + "problem_type": "proof", + "problem": "例7. $15 \\times 15$ 的方格表中有一条非自交闭折线,该折线由若干条连接相邻小方格 (两个有公共边的小方格称为相邻小方格) 的中心的线段组成, 且它关于方格表的某条对角线对称.\n证明:这条闭折线的长度不大于 200 .", + "solution": "解:显然,折线与对角线相交.\n令 $A$ 是一个这样的交点.\n我们沿着折线运动, 设 $B$ 是第一个再次与对角线相交的点.\n由对称性, 如果我们沿着折线按另一方向运动, $B$ 仍然是第一个与对角线的交点.\n这样折线在 $A$ 与 $B$ 之间已经封闭起来.\n这表明折线与该对角线有且只有两个交点.\n现在将方格表中的小方格用黑白相间的方式染色,使得对角线上的小方格全为黑色.\n注意到沿折线运动时, 黑白格交替经过.\n因此, 经过的黑白格数目相等.\n表中黑格比白格多一个.\n由于对角线上都是黑格, 折线与其中的 13 个不交,故折线至少与 12 个白格不交.\n由此,折线的长度不超过 $15^2-13-12=$ 200.", + "remark": "", + "figures": [] +} \ No newline at end of file diff --git a/processed_dataset/proof/0647.json b/processed_dataset/proof/0647.json new file mode 100644 index 0000000000000000000000000000000000000000..dff9162535cc55512168fd296a8989736b6f523e --- /dev/null +++ b/processed_dataset/proof/0647.json @@ -0,0 +1,8 @@ +{ + "source_file": "./raw_volume-zh/volume14/chapter14.tex", + "problem_type": "proof", + "problem": "例8. 已知圆周上有 $3 k\\left(k \\in \\mathbf{N}^*\\right)$ 个分点, 它们把圆周分成 $3 k$ 段弧, 其中长度为 $1 、 2 、 3$ 的弧各有 $k$ 条.\n求证: 这 $3 k$ 个分点中必有两点为对径点 (即这两点的连线为圆的直径).", + "solution": "证明:将给定的 $3 k$ 个分点都染为红色,再将所有长度为 2 的弧段的中点和长度为 3 的弧段的三等分点都染成蓝色.\n显然蓝点也是 $3 k$ 个.\n于是, 问题转化为证明: 有一对红点为对径点.\n假设结论不成立, 则红点的对径点都是蓝点.\n又由于红点与蓝点一样多, 故蓝点的对径点必为红点.\n在 $3 k$ 段弧中, 任取一段长度为 2 的弧 $\\overparen{A C}$, 则 $A, C$ 为红点, $\\overparen{A C}$ 的中点 $B^{\\prime}$ 是蓝点, 因此 $B^{\\prime}$ 的对径点 $B$ 是红点.\n考察长度为 $3 k-1$ 的弧 $\\overparen{A B}$, 设其上长度为 $i$ 的弧段数目是 $n_i(i=1,2$, 3 , 则有 $n_1+2 n_2+3 n_3=3 k-1$. 因为长度为 1 的弧段的两个端点都是红点, 所以它们的对径点都是蓝点.\n这两个相邻的蓝点恰好对应了弧 $\\overparen{B C}$ 上一条包含它们的长度为 3 的弧, 并且这是个一一对应.\n从而弧 $\\overparen{B C}$ 上长为 3 的弧段数等于 $n_1$. 另一方面, 弧 $\\overparen{A B}$ 和弧 $\\overparen{B C}$ 上一共有 $k$ 段长度为 3 的弧, 所以 $n_3+n_1=k$. 从而\n$$\n2 n_2+2 n_3=\\left(n_1+2 n_2+3 n_3\\right)-\\left(n_3+n_1\\right)=(3 k-1)-k=2 k-1 .\n$$\n上式左端为偶数而右端为奇数, 矛盾.", + "remark": "注:本题借助点的染色使条件和问题转化, 有利于我们观察、分析对象间的关系, 使证明过程直观具体.\n在证明中还涉及了对应原理和奇偶性原理.", + "figures": [] +} \ No newline at end of file diff --git a/processed_dataset/proof/0648.json b/processed_dataset/proof/0648.json new file mode 100644 index 0000000000000000000000000000000000000000..8c9536a3c5d43afc8f1839f2bbcf0ae7c4d222d4 --- /dev/null +++ b/processed_dataset/proof/0648.json @@ -0,0 +1,8 @@ +{ + "source_file": "./raw_volume-zh/volume14/chapter15.tex", + "problem_type": "proof", + "problem": "例1. 已知 $n$ 个点 $A_1, A_2, \\cdots, A_n$ 顺次排在一条直线上, 每个点染上红色或蓝色之一.\n如果线段 $A_i A_{i+1}(1 \\leqslant i \\leqslant n-1)$ 的两端颜色不同, 就称它为标准线段.\n已知 $A_1$ 与 $A_n$ 的颜色不同, 证明: 在 $A_i A_{i+1}(i=1,2, \\cdots, n-1)$ 中,标准线段的条数为奇数.", + "solution": "证明: $A_1, A_2, \\cdots, A_n$ 中的每一个点 $A_i$ 赋值 $a_i$ : 若 $A_i$ 为红色, 则 $a_i=1$; 若 $A_i$ 为蓝色, 则 $a_i=-1$.\n设这 $n-1$ 条线段中有 $m$ 条是标准线段,那么\n$$\n\\left(a_1 a_2\\right)\\left(a_2 a_3\\right) \\cdots\\left(a_{n-1} a_n\\right)=(-1)^m .\n$$\n另一方面, 有\n$$\n\\left(a_1 a_2\\right)\\left(a_2 a_3\\right) \\cdots\\left(a_{n-1} a_n\\right)=a_1 a_2^2 a_3^2 \\cdots a_{n-1}^2 a_n=a_1 a_n=-1 .\n$$\n所以, $(-1)^m=-1$, 故 $m$ 是奇数, 即标准线段的条数为奇数.", + "remark": "注:本题解法颇多.\n上述解法中, 我们通过赋值使问题数值化, 并通过数的运算性质简洁而直接地解答了问题.\n另一种典型的赋值方法可表述为: 对各点 $A_i$ 中的红点赋值 $a_i=0$, 蓝点赋值 $a_i=1$, 并考察所有线段 $A_i A_{i+1}$ 对应的数值 $a_i+a_{i+1}$ 之和的奇偶性.\n这两种赋值方法在相差一个运算级别的意义下是一样的.", + "figures": [] +} \ No newline at end of file diff --git a/processed_dataset/proof/0649.json b/processed_dataset/proof/0649.json new file mode 100644 index 0000000000000000000000000000000000000000..c7f15b0751ae92dc25457080939da25af76cdb5b --- /dev/null +++ b/processed_dataset/proof/0649.json @@ -0,0 +1,8 @@ +{ + "source_file": "./raw_volume-zh/volume14/chapter15.tex", + "problem_type": "proof", + "problem": "例2. 男女生共 $n$ 人围坐一圆桌, 规定相邻座为同性时两人中间插一枝红花, 异性时两人中间插一枝蓝花.\n结果发现所插红花与蓝花数目一样.\n证明: $n$ 一定是 4 的倍数.", + "solution": "证明:由于红花和蓝花数目一样, 故 $n$ 必为偶数, 设 $n=2 m$.\n取定一人开始, 将 $n$ 个人依次记为 $1,2, \\cdots, n$. 给第 $i(1 \\leqslant i \\leqslant n)$ 个人赋值 $x_i$, 其中对男生令 $x_i=1$, 对女生令 $x_i=-1$.\n根据条件可知, 当 $x_i x_{i+1}=1$ 时,在 $i, i+1$ 两人之间插红花; 当 $x_i x_{i+1}=-1$ 时, 在 $i, i+1$ 两人之间插蓝花 (约定第 $n+1$ 个人就是第 1 个人, $x_{n+1}=x_1$ ).\n转化为证明这样的结论: 若在 $x_i x_{i+1}(1 \\leqslant i \\leqslant n)$ 这 $n=2 m$ 个值中, 有 $m$ 个 $1, m$ 个 -1 , 则 $m$ 为偶数.\n事实上, 此时有 $(-1)^m=\\prod_{i=1}^n x_i x_{i+1}=\\left(x_1 x_2 \\cdots x_n\\right)^2=1$, 故 $m$ 为偶数.\n从而 $n=2 m$ 一定是 4 的倍数.\n证毕.", + "remark": "注:本题的做法与上一题有所相似.\n虽说本题完全可以不用赋值法来分析, 但上述讨论实际上建立了一个组合命题与一个数论命题之间的等价性 (该数论命题为: 若 $x_i \\in\\{1,-1\\}, 1 \\leqslant i \\leqslant n$, 且 $\\sum_{i=1}^n x_i x_{i+1}=0$, 则 $4 \\mid n$ ), 从中可以看出赋值法的转化功能.", + "figures": [] +} \ No newline at end of file diff --git a/processed_dataset/proof/0650.json b/processed_dataset/proof/0650.json new file mode 100644 index 0000000000000000000000000000000000000000..1174b72633c705cf273001a362d4869a0a8dbe0e --- /dev/null +++ b/processed_dataset/proof/0650.json @@ -0,0 +1,10 @@ +{ + "source_file": "./raw_volume-zh/volume14/chapter15.tex", + "problem_type": "proof", + "problem": "例5. 如图() 是一个向右和向下无限的表格.\n一开始在左上角 $A$ 格内放一枚棋子, 此后每一步下棋规则如下: 若某格 $P$ 放有棋子, 且它的右边相邻格 $Q$ 和下边相邻格 $R$ 都没有棋子, 则可将 $P$ 中的棋子去掉, 在 $Q 、 R$ 两格中各放一枚棋子.\n证明: 无论经过多少步,左上角 $3 \\times 3$ 表格中总存在棋子.", + "solution": "证明:们对第 $i$ 行第 $j$ 列的格子赋值 $\\left(\\frac{1}{2}\\right)^{i+j}, i, j \\in \\mathbf{N}^*$. 由于\n$$\n\\left(\\frac{1}{2}\\right)^{i+(j+1)}+\\left(\\frac{1}{2}\\right)^{(i+1)+j}=\\left(\\frac{1}{2}\\right)^{i+j},\n$$\n故每步下棋不改变所有棋子所在格的赋值之和, 记这个和为 $S$, 其中初始情况下的 $S=\\frac{1}{4}$.\n假设若干步后,左上角 $3 \\times 3$ 表格中不存在棋子, 那么此时\n$$\n\\begin{aligned}\nS & \\leqslant \\sum_{i=1}^{\\infty} \\sum_{j=1}^{\\infty}\\left(\\frac{1}{2}\\right)^{i+j}-\\sum_{i=1}^3 \\sum_{j=1}^3\\left(\\frac{1}{2}\\right)^{i+j} \\\\\n& =\\sum_{i=1}^{\\infty}\\left(\\frac{1}{2}\\right)^i \\cdot \\sum_{j=1}^{\\infty}\\left(\\frac{1}{2}\\right)^j-\\sum_{i=1}^3\\left(\\frac{1}{2}\\right)^i \\cdot \\sum_{j=1}^3\\left(\\frac{1}{2}\\right)^j \\\\\n& =1-\\left(\\frac{7}{8}\\right)^2=\\frac{15}{64}<\\frac{1}{4},\n\\end{aligned}\n$$\n矛盾.\n故无论经过多少步,左上角 $3 \\times 3$ 表格中总存在棋子.", + "remark": "注:本题中通过等比赋值的方式给每个格子一个具体的数值, 这种赋值方法以及 \" $\\frac{1}{2}$ \" 这个比值的选用是根据下棋规则而度身定制的, 从而在下棋过程中, 所有棋子所在格的数值之和是一个不变量.\n在赋值法解题时常会遇到这样的情况: 需要先明确赋值的目标, 再具体问题具体分析 (试比较本节习题 4).", + "figures": [ + "./images/volume14/figures/fig-c15i5.png" + ] +} \ No newline at end of file diff --git a/processed_dataset/proof/0651.json b/processed_dataset/proof/0651.json new file mode 100644 index 0000000000000000000000000000000000000000..878b5d870a02ff915b87786236d9a28d2c55482c --- /dev/null +++ b/processed_dataset/proof/0651.json @@ -0,0 +1,8 @@ +{ + "source_file": "./raw_volume-zh/volume14/chapter15.tex", + "problem_type": "proof", + "problem": "例6. $k$ 个开关顺次排成一行, 分别指向上、下、左、右四个方向.\n若其中出现三个连续的开关,它们的方向各不相同,则将它们同时调整为第四个方向.\n证明: 这个操作不能无限次进行下去.", + "solution": "证明:将这 $k$ 个开关依次按 $1,2, \\cdots, k$ 进行编号.\n我们按下述方法对第 $n(1 \\leqslant n \\leqslant k)$ 个开关赋值 $f(n)$ :\n$n=1$ 时, $f(n)=1$;\n$n \\geqslant 2$ 时, 若第 $n-1$ 个开关和第 $n$ 个开关方向相同, 则 $f(n)=n$; 若不然, 则 $f(n)=1$.\n令 $h=f(1) f(2) \\cdots f(k)$, 显然 $h>0$. 可以证明, 随着操作次数的增加, $h$ 是递增的.\n事实上, 对任意的 $n(1 \\leqslant n \\leqslant k-2)$, 假定本次操作只对第 $n, n+1, n+2$ 个开关进行调整, 那么在这次操作之前, $f(n) \\leqslant n, f(n+1)=f(n+2)=1$, $f(n+3) \\leqslant n+3$, 所以\n$$\nf(n) f(n+1) f(n+2) f(n+3) \\leqslant n(n+3) .\n$$\n进行操作后, 得到 $f^{\\prime}(n) \\geqslant 1, f^{\\prime}(n+1)=n+1, f^{\\prime}(n+2)=n+2$, $f^{\\prime}(n+3) \\geqslant 1$.\n所以 $f^{\\prime}(n) f^{\\prime}(n+1) f^{\\prime}(n+2) f^{\\prime}(n+3) \\geqslant(n+1)(n+2)>f(n) f(n+$ 1) $f(n+2) f(n+3)$.\n而操作前后第 $i(1 \\leqslant i \\leqslant n-1$ 或 $n+4 \\leqslant i \\leqslant k)$ 个开关的值不变, 即 $f^{\\prime}(i)=f(i)$.\n于是 $h^{\\prime}=f^{\\prime}(1) f^{\\prime}(2) \\cdots f^{\\prime}(k)>f(1) f(2) \\cdots f(k)=h$.\n注意到 $h$ 有界 $(h \\leqslant 1 \\times 2 \\times \\cdots \\times k)$, 所以 $h$ 只能增长有限次.\n所以这个操作只能进行有限次.", + "remark": "注:本题中,如果仅简单地对 \"方向相同的相邻开关对\"进行计数, 则无法说明其恒增性.\n我们进行适当赋值, 构造出 \"高度函数\" $h$, 它是一个恒增量, 这就解决了困难.\n我们还可以定义其他的高度函数 $h$. 例如: 对第 $n$ 个开关赋值 $f(n)$ : 若第 $n$ 个开关和第 $n+1$ 个开关方向相反, 则 $f(n)=\\sqrt{n}$; 若不然, 则 $f(n)=0$. 定义高度函数 $h=\\sum_{n=1}^k f(n)$. 可以证明随着操作次数的增加, $h$ 是递减的.\n而 $h>0$, 所以 $h$ 只能减少有限次, 即得证.\n读者可以尝试用\"凹函数\" 来构造更多的高度函数 $h$. 关键是要定义一个正值函数 $f(n)$, 且对于所有的 $1 \\leqslant n \\leqslant k$, 满足 $f(n) \\cdot f(n+3)0$, 即 $x_A>y_A$, 从而命题成立.", + "remark": "注:注意到每个球都是球堆中的\"一份子\",本题中的赋值具有某种权重的意义: 若球堆中的球数为 $a$, 则其所占份额为 $\\frac{1}{a}$. 于是, 先后两次分堆时球堆堆数之差 $k$, 表现为所有球的份额值总和的先后两次之差, 而 \"原来所在球堆中的球数大于后来所在球堆中的球数\" 可理解为 \"后来球堆中所占的份额大于在原来球堆中所占的份额\". 在这样的理解下, 就化为一个代数问题了.", + "figures": [] +} \ No newline at end of file diff --git a/processed_dataset/proof/0654.json b/processed_dataset/proof/0654.json new file mode 100644 index 0000000000000000000000000000000000000000..259f71e3f5c13adc16375f5c5f9a07ec9deb2643 --- /dev/null +++ b/processed_dataset/proof/0654.json @@ -0,0 +1,8 @@ +{ + "source_file": "./raw_volume-zh/volume14/chapter16.tex", + "problem_type": "proof", + "problem": "例1. 证明: $\\frac{n(n+1)(n+2)}{6}=1 \\cdot n+2 \\cdot(n-1)+3 \\cdot(n-2)+\\cdots+ n \\cdot 1$.", + "solution": "证明:虑在 $n+2$ 个数 $1,2, \\cdots, n+2$ 中任取 3 个的取法总数 $S$.\n一方面,显然有 $S=\\mathrm{C}_{n+2}^3=\\frac{n(n+1)(n+2)}{6}$.\n另一方面, 在 $1,2, \\cdots, n+2$ 中, 为取三个数 $a, b, c(1 \\leqslant ax_1 x_2$. 将 $S$ 改写成\n$$\nS=\\sum_{1 \\leqslant i0$. 这与 $S$ 在 $x_1, x_2, x_3, x_4, x_5$ 时取到最大值矛盾.\n所以必有 $\\left|x_i-x_j\\right| \\leqslant 1(1 \\leqslant i, j \\leqslant 5)$. 因此当 $x_1=402, x_2= x_3=x_4=x_5=401$ 取到最大值.\n(2) 当 $x_1+x_2+x_3+x_4+x_5=2006$ 且 $\\left|x_i-x_j\\right| \\leqslant 2$ 时,只有\n(I ) 402, 402, 402, 400, 400;\n( II ) 402, 402, 401, 401, 400;\n(III) $402 , 401 , 401 , 401 , 401$ 三种情形满足要求.\n而后面两种情形是在第一组情形下作 $x_i^{\\prime}=x_i-1, x_j^{\\prime}=x_j+1$ 调整下得到的.\n根据上一小题的证明可以知道, 每调整一次, 和式 $S=\\sum_{1 \\leqslant i0$, 方程 $a x^2+b y^2+c z^2=$ 1 必有一组有理数解 $\\left(x_1, y_1, z_1\\right)$, 使得 $x_1^2+y_1^2+z_1^2>N$.", + "solution": "证明:已知得 $a x_0^2+b y_0^2+c z_0^2=0$, 不妨设其中 $x_0 \\neq 0$, 令\n$$\nx=k+t x_0, y=t y_0, z=t z_0,\n$$\n其中 $k, t$ 为待定有理数,则\n$$\n\\begin{aligned}\na x^2+b y^2+c z^2 & =a\\left(k+t x_0\\right)^2+b t^2 y_0^2+c t^2 z_0^2 \\\\\n& =a k^2+2 a k x_0 t+t^2\\left(a x_0^2+b y_0^2+c z_0^2\\right) \\\\\n& =a k^2+2 a k x_0 t,\n\\end{aligned}\n$$\n取 $t$ 使 $a k^2+2 a k x_0 t=1$, 即 $t=\\frac{1-a k^2}{2 a k x_0}$, 可得\n$$\n\\left\\{\\begin{array}{l}\nx_1=k+t x_0=k+\\frac{1-a k^2}{2 a k}, \\\\\ny_1=t y_0=\\frac{1-a k^2}{2 a k} \\cdot \\frac{y_0}{x_0}, \\\\\nz_1=t z_0=\\frac{1-a k^2}{2 a k} \\cdot \\frac{z_0}{x_0},\n\\end{array}\\right.\n$$\n是方程 $a x^2+b y^2+c z^2=1$ 的一组有理数解 (其中 $k$ 仍为待定参数, 但 $k \\neq 0$ ).\n由于 $x_1=k+\\frac{1-a k^2}{2 a k}=\\frac{k}{2}+\\frac{1}{2 a k}$, 取 $k>\\max \\left\\{2 \\sqrt{n}+1, \\frac{1}{|a|}\\right\\}$ 且 $k$ 为有理数, 则\n$$\n\\frac{k}{2}>\\sqrt{N}+\\frac{1}{2},\\left|\\frac{1}{2 a k}\\right|<\\frac{1}{2|a|} \\cdot|a|=\\frac{1}{2},\n$$\n从而 $x_1=\\frac{k}{2}+\\frac{1}{2 a k}>\\sqrt{N}+\\frac{1}{2}-\\frac{1}{2}=\\sqrt{N}$, 故对这样一组有理数解 $\\left(x_1, y_1\\right.$, $z_1$ ), 有 $x_1^2+y_1^2+z_1^2>N$.", + "remark": "注:对于本题, 想到如何构造出一组有理数解是很关键的, 整个解题过程也是先找到构造有理数解 $\\left(x_1, y_1, z_1\\right)$ 的一种策略, 再对构造过程进行调整, 使限制条件 $x_1^2+y_1^2+z_1^2>N$ 也得以满足.\n引入待定参数 $k$ 的目的正是为了留出调整余地, 使最终 $x_1$ 的选取仍具有足够的自由度, 从而 $x_1^2+y_1^2+z_1^2$ 能取到任意大的值.\n事实上, 本题是作者由一道陈题添加限制条件 $x_1^2+y_1^2+ z_1^2>N$ 所得, 上述解法中, 若只留有待定参数 $t$, 而平凡地取 $k=1$, 即给出原题的一种证明.", + "figures": [] +} \ No newline at end of file diff --git a/processed_dataset/proof/0662.json b/processed_dataset/proof/0662.json new file mode 100644 index 0000000000000000000000000000000000000000..c9ceb31b22ece9df5590745028bf8b79613a456c --- /dev/null +++ b/processed_dataset/proof/0662.json @@ -0,0 +1,8 @@ +{ + "source_file": "./raw_volume-zh/volume14/chapter17.tex", + "problem_type": "proof", + "problem": "例9. 设 $A_1, A_2, \\cdots, A_n$ 是集合 $X$ 的 $n$ 个非空子集, 且对任意 $i, j \\in \\{1,2, \\cdots, n\\}, A_i \\cap A_j$ 不是单元集, 求证: 可把集合 $X$ 的元素分成两类, 使每个子集 $A_i(i=1,2, \\cdots, n)$ 的元素不全在同一类中.", + "solution": "证明:每个子集 $A_i$ 非空, 且 $A_i=A_i \\cap A_i$ 不是单元集, 故 $A_i$ 至少含两个元素.\n在集合 $X$ 的某种分类下, 若 $A_i$ 的元素只出现在一类中, 则称 $A_i$ 为单类集, 若 $A_i$ 的元素出现在两类中, 则称 $A_i$ 为双类集.\n记 $t$ 为 $A_i(i=1,2, \\cdots, n)$ 中双类集的数目,若 $t$ 能取到 $n$ 则命题得证.\n因为 $A_1$ 至少含两个元素, 故先将集合 $X$ 的元素分成两类且使 $A_1$ 为双类集, 此时 $t \\geqslant 1$.\n下面证明,对使 $t4 a(a+b+c) \\text {. }\n$$", + "solution": "分析:证明 $(b-c)^2>4 a(a+b+c)$, 即证明 $(b-c)^2-4 a(a+b+-c)>$ 0 , 联想到一元二次方程根的判别式, 进而构造符合条件的二次函数, 通过对函数图象和性质的研究, 使得问题得以解决.\n解若 $a=0$, 则 $c(b+c)<0$, 从而 $b \\neq c$ (否则, $\\left.2 b^2<0\\right)$, 于是 $(b-c)^2>$ 0 , 命题成立.\n若 $a \\neq 0$, 设二次函数 $y=a x^2+(b-c) x+(a+b+c)$.\n令 $x_1=0$, 得函数值 $y_1=a+b+c$, 令 $x_2=-1$, 得函数值 $y_2=2(a+c)$.\n因为 $(a+c)(a+b+c)<0$, 所以 $y_1 y_2<0$, 这说明二次函数 $y=a x^2+ (b-c) x+(a+b+c)$ 上两点 $\\left(x_1, y_1\\right)$ 和 $\\left(x_2, y_2\\right)$ 分别在 $x$ 轴的两侧, 由此可见抛物线与 $x$ 轴有两个不同的交点, 即方程 $a x^2+(b-c) x+(a+b+c)=0$ 有两个不相等的实数根.\n因此 $\\Delta=(b-c)^2-4 a(a+b+c)>0$, 即\n$$\n(b-c)^2>4 a(a+b+c) .\n$$", + "remark": "", + "figures": [] +} \ No newline at end of file diff --git a/processed_dataset/proof/0665.json b/processed_dataset/proof/0665.json new file mode 100644 index 0000000000000000000000000000000000000000..9da21f5cf46e9ee8e6f30d3db3af510a09ef4ca7 --- /dev/null +++ b/processed_dataset/proof/0665.json @@ -0,0 +1,8 @@ +{ + "source_file": "./raw_volume-zh/volume14/chapter18.tex", + "problem_type": "proof", + "problem": "例3. 设 $a, b, c$ 是绝对值小于 1 的实数, 证明: $a b+b c+c a+1>0$.", + "solution": "证明:造函数 $f(x)=(b+c) x+b c+1$, 它的图象是一条直线, 若能证明函数值 $f(-1), f(1)$ 都大于 0 , 则以点 $(-1, f(-1))$ 和 $(1, f(1))$ 为端点的线段上的每一点的函数值都大于 0 , 即对满足 $-10$, 从而命题得证.\n因为\n$$\n\\begin{gathered}\nf(-1)=-(b+c)+b c+1=(b-1)(c-1)>0, \\\\\nf(1)=(b+c)+b c+1=(b+1)(c+1)>0,\n\\end{gathered}\n$$\n所以\n$$\nf(a)=a(b+c)+b c+1>0 .\n$$", + "remark": "", + "figures": [] +} \ No newline at end of file diff --git a/processed_dataset/proof/0666.json b/processed_dataset/proof/0666.json new file mode 100644 index 0000000000000000000000000000000000000000..10cd5c2cfa057b8bc3a80d180768756186a60112 --- /dev/null +++ b/processed_dataset/proof/0666.json @@ -0,0 +1,10 @@ +{ + "source_file": "./raw_volume-zh/volume14/chapter18.tex", + "problem_type": "proof", + "problem": "例4. 设 $a \\geqslant c, b \\geqslant c, c>0$, 证明不等式\n$$\n\\sqrt{c(a-c)}+\\sqrt{c(b-c)} \\leqslant \\sqrt{a b} .\n$$", + "solution": "证明: $a=c$ 或 $b=c$ 时, 不等式显然成立.\n当 $a \\neq c$ 且 $b \\neq c$ 时, 讨论如下: 两个正数 $x 、 y$ 的乘积 $x y$ 可以看成边长为 $x$ 和 $y$ 的矩形的面积, 也可以看成直角边为 $x$ 和 $y$ 的直角三角形面积的两倍, 于是构造图形,如图().\n$$\n\\begin{gathered}\nA B=D E=\\sqrt{c}, B C=\\sqrt{a-c}, \\\\\nC D=\\sqrt{b-c}, \\\\\n\\angle A B C=\\angle C D E=\\frac{\\pi}{2} .\n\\end{gathered}\n$$\n设 $\\angle A C E=\\alpha$, 则\n$$\n\\begin{aligned}\nS_{A B D E} & =\\sqrt{c}(\\sqrt{a-c}+\\sqrt{b-c})=S_{\\triangle A B C}+S_{\\triangle C D E}+S_{\\triangle A C E} \\\\\n& =\\frac{1}{2} \\sqrt{c} \\sqrt{a-c}+\\frac{1}{2} \\sqrt{c} \\sqrt{b-c}+\\frac{1}{2} \\sqrt{a} \\sqrt{b} \\sin \\alpha \\\\\n& \\leqslant \\frac{1}{2} \\sqrt{c}(\\sqrt{a-c}+\\sqrt{b-c})+\\frac{1}{2} \\sqrt{a b},\n\\end{aligned}\n$$\n所以\n$$\n\\sqrt{c(a-c)}+\\sqrt{c(b-c)} \\leqslant \\sqrt{a b},\n$$\n于是命题得证.", + "remark": "", + "figures": [ + "./images/volume14/figures/fig-c18i1.png" + ] +} \ No newline at end of file diff --git a/processed_dataset/proof/0667.json b/processed_dataset/proof/0667.json new file mode 100644 index 0000000000000000000000000000000000000000..15ddfaeb7c37f4c889e2b71288432e0a459322ec --- /dev/null +++ b/processed_dataset/proof/0667.json @@ -0,0 +1,10 @@ +{ + "source_file": "./raw_volume-zh/volume14/chapter18.tex", + "problem_type": "proof", + "problem": "例5. 设实数 $x, y, z$ 满足 $0\\sin 2 x+\\sin 2 y+\\sin 2 z .\n$$", + "solution": "证明:直角坐标平面上以原点为圆心作单位 圆.\n考虑第一象限, 在单位圆上取点 $A_1, A_2, A_3$, 使得 $\\angle A_1 O x=x, \\angle A_2 O x=y, \\angle A_3 O x=z($ 如图() 所示).\n由于三个矩形面积之和 $S_1+S_2+S_3$ 小于 $\\frac{1}{4}$ 单位圆的面积, 此即\n$$\n\\sin x(\\cos x-\\cos y)+\\sin y(\\cos y-\\cos z)+\\sin z \\cos z<\\frac{\\pi}{4},\n$$\n整理后便得\n$$\n\\frac{\\pi}{2}+2 \\sin x \\cos y+2 \\sin y \\cos z>\\sin 2 x+\\sin 2 y+\\sin 2 z .\n$$", + "remark": "注:例 4 和例 5 条件中的数量关系能以某种方式与几何图形建立联系或具有明显的几何意义, 从而构造图形, 将题设条件及数量关系直接在图形中得到实现,然后在所构造的图形中寻求所证的结论.", + "figures": [ + "./images/volume14/figures/fig-c18i2.png" + ] +} \ No newline at end of file diff --git a/processed_dataset/proof/0668.json b/processed_dataset/proof/0668.json new file mode 100644 index 0000000000000000000000000000000000000000..9647008c1365ebca35b7972c303830ad21d53b24 --- /dev/null +++ b/processed_dataset/proof/0668.json @@ -0,0 +1,8 @@ +{ + "source_file": "./raw_volume-zh/volume14/chapter18.tex", + "problem_type": "proof", + "problem": "例8. 正整数 $a_1, a_2, \\cdots, a_{2006}$ (可以有相同的)使得 $\\frac{a_1}{a_2}, \\frac{a_2}{a_3}, \\cdots, \\frac{a_{2005}}{a_{2006}}$ 两不相等.\n问: $a_1, a_2, \\cdots, a_{2006}$ 中最少有多少个不同的数?", + "solution": "解: $a_1, a_2, \\cdots, a_{2006}$ 中出现 $n$ 个互不相同的数, 那么这些正整数两两间所产生的不同比值不多于 $n(n-1)+1$ 个(其中, 相等的两数产生比值 1), 为保证条件满足,必有 $n(n-1)+1 \\geqslant 2005$, 可知 $n>45$.\n下面构造一个例子, 说明 $n=46$ 可以取到.\n设 $p_1, p_2, \\cdots, p_{46}$ 为 46 个互不相同的素数,构造 $a_1, a_2, \\cdots, a_{2006}$ 如下:\n$$\n\\begin{aligned}\n& p_1, p_1, \\\\\n& p_2, p_1, \\\\\n& p_3, p_2, p_3, p_1, \\\\\n& p_4, p_3, p_4, p_2, p_4, p_1, \\\\\n& \\cdots \\ldots \\\\\n& p_k, p_{k-1}, p_k, p_{k-2}, p_k, \\cdots, p_k, p_2, p_k, p_1, \\\\\n& \\cdots \\cdots \\\\\n& p_{45}, p_{44}, p_{45}, p_{43}, p_{45}, \\cdots, p_{45}, p_2, p_{45}, p_1, \\\\\n& p_{46}, p_{45}, p_{46}, p_{44}, p_{46}, \\cdots, p_{46}, p_{34},\n\\end{aligned}\n$$\n这 2006 个正整数满足要求.\n所以 $a_1, a_2, \\cdots, a_{2006}$ 中最少有 46 个互不相同的数.", + "remark": "注:本题是一个典型的离散最值问题, 解答分为两个部分, 一是估计出 $n$ 的范围 $n>45$, 二是通过构造具体例子表明范围中最极端的值 $n=46$ 可以实现.\n数学竞赛中的最值问题覆盖代数、几何、组合等方方面面, 其中有些变量的上、下界并不能用经典的求最值方法求得, 此时, 构造一个例子或一个反例本身就解决了题目的一半, 这样的情况在组合最值问题及离散最值问题中是极为多见的.\n甚至有时候一个本质的例子对解决另一半问题还具有启发性.\n构造法的价值由此可见一斑.", + "figures": [] +} \ No newline at end of file diff --git a/processed_dataset/proof/0669.json b/processed_dataset/proof/0669.json new file mode 100644 index 0000000000000000000000000000000000000000..62a29ae75d8518456101faa59be519ad7f496d15 --- /dev/null +++ b/processed_dataset/proof/0669.json @@ -0,0 +1,8 @@ +{ + "source_file": "./raw_volume-zh/volume14/chapter18.tex", + "problem_type": "proof", + "problem": "例9. 求证: 对每个正整数 $m$, 平面内存在一个有限非空点集 $S$, 具有如下性质: 对于任意一点 $A \\in S$, 在 $S$ 中与点 $A$ 距离为 1 的点恰有 $m$ 个.", + "solution": "证明: $m$ 用数学归纳法.\n当 $m=1$ 时,取长为 1 的线段的两个端点构成点集 $S$ 即可.\n假设 $m=k$ 时命题成立, 即存在点集 $S_k$, 对任意 $A \\in S_k$, 恰有 $S_k$ 中 $k$ 个点到点 $A$ 距离为 1 .\n以 $S_k$ 中的每个点为圆心作半径为 1 的圆, 这些圆两两之间的交点是有限个, 设它们构成集合 $T_k$, 那么 $S_k \\cup T_k$ 中任意两点的连线的方向只有有限个.\n任取一个方向 $\\vec{d}$ 不属于这有限个方向, 将 $S_k$ 沿 $\\vec{d}$ 平移一个单位得到点集 $S_k^{\\prime}$.\n由 $\\vec{d}$ 的取法不难验证: 一方面 $S_k \\cap S_k^{\\prime}=\\varnothing$; 另一方面, 两点 $A \\in S_k$ 和 $A^{\\prime} \\in S_k^{\\prime}$ 之间距离为 1 当且仅当 $A^{\\prime}$ 是由 $A$ 平移所得.\n当 $m=k+1$ 时, 令 $S_{k+1}=S_k \\cup S_k^{\\prime}$. 对任意 $A \\in S_{k+1}$, 不失一般性, 设 $A \\in S_k$, 根据归纳假设, 恰有 $S_k$ 中的 $k$ 个点与它距离为 1 , 又 $S_k^{\\prime}$ 中恰有一点与它距离为 1 , 故 $S_{k+1}$ 中恰好有 $k+1$ 个点与 $A$ 的距离为 1 . 因此 $m=k+1$ 时命题成立.\n由数学归纳法知, 对任意正整数 $m$, 平面内存在满足题意的点集.", + "remark": "注:本题用的是归纳构造方法.\n由于 $m$ 不是具体数值, 而且点集不易直接构造, 那么先把最简单的 $m=1$ 的情形构造出来, 再通过适当平移点集后取并集(有点像把自己和自己的影子合起来)的手法实现归纳过渡.\n一般地, 对一个与正整数 $n$ 有关的命题, 可以从 $n$ 等于某个 $n_0$ 开始, 先构造出满足题意的对象 $P\\left(n_0\\right)$, 然后假设 $P(n)$ 已经构造出来, 证明 $P(n+1)$ 也能被构造出来, 当然在具体问题中, 这个证明可以是构造性的, 也可以是非构造性的.", + "figures": [] +} \ No newline at end of file diff --git a/processed_dataset/proof/0670.json b/processed_dataset/proof/0670.json new file mode 100644 index 0000000000000000000000000000000000000000..822b44120b980a81ec252459941db72e978b941f --- /dev/null +++ b/processed_dataset/proof/0670.json @@ -0,0 +1,8 @@ +{ + "source_file": "./raw_volume-zh/volume14/chapter18.tex", + "problem_type": "proof", + "problem": "例10. 对任何正整数 $n$, 证明恒等式: $\\sum_{k=0}^n 2^k \\mathrm{C}_n^k \\mathrm{C}_{n-k}^{\\left[\\frac{n-k}{2}\\right]}=\\mathrm{C}_{2 n+1}^n$.", + "solution": "证法一(构造函数方法)\n考虑函数 $f(x)=(1+x)^{2 n+1}$. 一方面,需证等式右边显然是 $f(x)$ 的 $n$ 次项系数.\n另一方面,\n$$\n\\begin{aligned}\nf(x) & =(1+x)\\left(1+2 x+x^2\\right)^n=(1+x) \\sum_{k=0}^n \\mathrm{C}_n^k\\left(1+x^2\\right)^{n-k}(2 x)^k \\\\\n& =\\sum_{k=0}^n 2^k \\mathrm{C}_n^k(1+x)\\left(1+x^2\\right)^{n-k} x^k,\n\\end{aligned}\n$$\n当 $n-k$ 为偶数时, $(1+x)\\left(1+x^2\\right)^{n-k}$ 中 $x^{n-k}$ 的系数为 $\\mathrm{C}_{n-k}^{\\frac{n-k}{2}}$;\n当 $n-k$ 为奇数时, $(1+x)\\left(1+x^2\\right)^{n-k}$ 中 $x^{n-k}$ 的系数为 $\\mathrm{C}_{n-k}^{\\frac{n-k-1}{2}}$.\n所以对 $k=0,1, \\cdots, n$, 在 $2^k \\mathrm{C}_n^k(1+x)\\left(1+x^2\\right)^{n-k} x^k$ 中 $x^n$ 的系数总是 $2^k \\mathrm{C}_n^k \\mathrm{C}_{n-k}^{\\left[\\frac{n-k}{2}\\right]}$.\n从而 $f(x)$ 的 $n$ 次项系数为 $\\sum_{k=0}^n 2^k \\mathrm{C}_n^k \\mathrm{C}_{n-k}^{\\left[\\frac{n-k}{2}\\right]}$.\n综合两方面可得: $\\sum_{k=0}^n 2^k \\mathrm{C}_n^k \\mathrm{C}_{n-k}^{\\left[\\frac{n-k}{2}\\right]}=\\mathrm{C}_{2 n+1}^n$.", + "remark": "", + "figures": [] +} \ No newline at end of file diff --git a/processed_dataset/proof/0671.json b/processed_dataset/proof/0671.json new file mode 100644 index 0000000000000000000000000000000000000000..432400df4fb67c7682f4322a63403c2ec19885d0 --- /dev/null +++ b/processed_dataset/proof/0671.json @@ -0,0 +1,8 @@ +{ + "source_file": "./raw_volume-zh/volume14/chapter18.tex", + "problem_type": "proof", + "problem": "例10. 对任何正整数 $n$, 证明恒等式: $\\sum_{k=0}^n 2^k \\mathrm{C}_n^k \\mathrm{C}_{n-k}^{\\left[\\frac{n-k}{2}\\right]}=\\mathrm{C}_{2 n+1}^n$.", + "solution": "证法二(构造计数模型方法)\n设有 $n$ 对夫妻, 一个导游, 共 $2 n+1$ 人到甲乙两地游玩, 其中 $n$ 人去甲地, $n+1$ 人去乙地.\n一方面, 分配方案显然为 $\\mathrm{C}_{2 n+1}^n$ 种.\n另一方面, 我们将所有分配方案按 $n$ 对夫妻中恰有 $k(k=0,1, \\cdots, n)$ 对分开游玩进行分类计数.\n对给定的 $k(0 \\leqslant k \\leqslant n)$, 先从 $n$ 对夫妻中选取 $k$ 对分开游玩的, 其中每对中谁去甲地各有 2 种可能,所以安排这 $2 k$ 个人的方案有 $2^k \\mathrm{C}_n^k$ 种.\n对其余 $n-k$ 对夫妻和一个导游, 若 $n-k$ 为偶数, 则必有 $\\frac{n-k}{2}$ 对夫妻去甲地; 若 $n-k$ 为奇数, 则必有 $\\frac{n-k-1}{2}$ 对夫妻外加导游去甲地, 选择方案总是 $\\mathrm{C}_{n^{n-k}}^{\\left[\\frac{n-k}{2}\\right]}$ 种.\n由乘法原理可知: $n$ 对夫妻中恰有 $k$ 对分开游玩的情况总数为 $2^k \\mathrm{C}_n^k \\mathrm{C}_{n-k}^{\\left[\\frac{n-k}{2}\\right]}$. 对 $k=0,1, \\cdots, n$ 求和即知 $\\sum_{k=0}^n 2^k \\mathrm{C}_n^k \\mathrm{C}_{n-k}^{\\left[\\frac{n-k}{2}\\right]}=\\mathrm{C}_{2 n+1}^n$.", + "remark": "注:构造函数与构造模型是数学中经常使用的方法, 前者主要是通过构造函数,把原问题的求解转化为对所作函数性质的研究; 后者则是将原问题中的条件、数量关系在某个模型上实现并得到一种解释, 从而转化为对模型上相应问题的考察, 其中, 所构造的计数模型常常取材于一些通俗易懂的虚构情景( 例如与日常生活有关).\n运用构造法解题, 能使代数、三角、几何等各种知识相互渗透, 有利于提高分析问题和解决问题的能力.", + "figures": [] +} \ No newline at end of file diff --git a/processed_dataset/proof/0672.json b/processed_dataset/proof/0672.json new file mode 100644 index 0000000000000000000000000000000000000000..50957e0f0d813827de1526e196db0b4dddb550f0 --- /dev/null +++ b/processed_dataset/proof/0672.json @@ -0,0 +1,8 @@ +{ + "source_file": "./raw_volume-zh/volume14/chapter19.tex", + "problem_type": "proof", + "problem": "例1. 设 $a_1, a_2, \\cdots, a_9$ 是非零实数.\n证明: 行列式\n$$\n\\left|\\begin{array}{lll}\na_1 & a_2 & a_3 \\\\\na_4 & a_5 & a_6 \\\\\na_7 & a_8 & a_9\n\\end{array}\\right|=a_1 a_5 a_9+a_2 a_6 a_7+a_3 a_4 a_8-a_1 a_6 a_8-a_2 a_4 a_9-a_3 a_5 a_7\n$$\n这六项中, 至少有一项是负数,且至少有一项是正数.", + "solution": "证明:六项 $a_1 a_5 a_9, a_2 a_6 a_7, a_3 a_4 a_8,-a_1 a_6 a_8,-a_2 a_4 a_9,-a_3 a_5 a_7$ 的乘积为\n$$\n-\\left(a_1 a_2 \\cdots a_9\\right)^2<0 .\n$$\n所以其中必有奇数个负项.\n从而结论成立.", + "remark": "注:在例 1 中, 关键的一点就是: 无论 $a_1, a_2, \\cdots, a_9$ 中有多少个正数,多少个负数,所给的六项的乘积总是负数.\n这就是一个不变量.", + "figures": [] +} \ No newline at end of file diff --git a/processed_dataset/proof/0673.json b/processed_dataset/proof/0673.json new file mode 100644 index 0000000000000000000000000000000000000000..6cacae1da282a06bb840f9a8fec972f9b8fae93b --- /dev/null +++ b/processed_dataset/proof/0673.json @@ -0,0 +1,8 @@ +{ + "source_file": "./raw_volume-zh/volume14/chapter19.tex", + "problem_type": "proof", + "problem": "例3. 若干名立场不坚定的人对某一问题有两种相反的意见.\n他们在辩论桌上围坐一圈, 按逆时针次序轮流发言表明立场.\n如果一个人发现他的朋友中大多数与他意见相反, 他就改变立场, 转而发言支持相反的意见, 否则他就表明原来的立场.\n求证: 经过若干轮发言之后, 这些人均不再改变立场.\n(注:朋友关系是相互的,且不改变.)", + "solution": "证明:察所有意见相反的朋友对数目.\n每当一名立场不坚定的人改变立场时, 他的意见便与大多数朋友相同, 因此意见相反的朋友对数目严格减小.\n由于开始时意见相反的朋友对的数目是有限的, 所以不可能有某个人无限次改变立场, 从而若干轮发言之后, 这些人均不再改变立场.\n在例 3 中,我们所考虑的是一种广义的\"不变量\"\n\"恒增(减) 量\". 在不变量方法中,所谓\"不变量\"并不仅限于恒定的量, 更广泛地说, 也可以指变化着的量, 只要这种\"变化\" 具有某种\"不变的规律\". 本题中, \"意见相反的朋友对数目\"这个量具有单调性, 就是一个\"不变量\".\n利用\"恒增(减)量\",乃至一般地,利用某种不变的规律解题是十分常见的, 同时也是技巧性较高的一个方面.", + "remark": "", + "figures": [] +} \ No newline at end of file diff --git a/processed_dataset/proof/0674.json b/processed_dataset/proof/0674.json new file mode 100644 index 0000000000000000000000000000000000000000..6b2ba3120bc44b8c6c8771108fc920532361af8d --- /dev/null +++ b/processed_dataset/proof/0674.json @@ -0,0 +1,8 @@ +{ + "source_file": "./raw_volume-zh/volume14/chapter19.tex", + "problem_type": "proof", + "problem": "例4. 设直线上一开始从左到右依次写有 $1,2, \\cdots, 2011$ 这 2011 个数.\n可以对相邻位置的三个数 $a, b, c$ 进行这样的操作: 把 $(a, b, c)$ 换成 $(b, c, a)$. 证明: 无论怎样操作, 不可能使直线上的数从左到右依次是 $2011,2010, \\cdots, 2,1$.", + "solution": "证明: $(1,2, \\cdots, 2011)$ 的每一个排列 $P=\\left(a_1, a_2, \\cdots, a_{2011}\\right)$, 称 $\\left(a_i\\right.$, $\\left.a_j\\right)$ 是 $P$ 的一个 \"逆序对\", 如果有 $ia_j$. 将 $P$ 的逆序对个数称为逆序数, 记作 $I(P)$.\n根据该定义,若在排列 $P$ 中选取相邻三个数 $a, b, c$ 进行操作, 得到另一个排列 $P^{\\prime}$, 则必有 $I\\left(P^{\\prime}\\right) \\equiv I(P)(\\bmod 2)$. 事实上, 对\n$$\nP=(\\cdots, a, b, c, \\cdots), P^{\\prime}=(\\cdots, b, c, a, \\cdots),\n$$\n若 $(a, b),(a, c)$ 中有 $k$ 个是 $P$ 的逆序对, 则 $(b, a),(c, a)$ 中有 $2-k$ 个是 $P^{\\prime}$ 的逆序对.\n其余的数对是 $P$ 中的逆序对当且仅当是 $P^{\\prime}$ 中的逆序对, 因此\n$$\nI\\left(P^{\\prime}\\right)-I(P)=k-(2-k) \\equiv 0(\\bmod 2) .\n$$\n由于对 $P_0=(1,2, \\cdots, 2011)$ 与 $P_1=(2011,2010, \\cdots, 1)$, 分别有\n$$\nI\\left(P_0\\right)=0, I\\left(P_1\\right)=\\frac{2011 \\times 2010}{2} \\equiv 1(\\bmod 2),\n$$\n所以不能通过操作使 $P_0$ 变为 $P_1$. 证毕.", + "remark": "注:\"逆序对\" 和 \"逆序数\" 是排列问题中常会涉及的概念, 考虑逆序数的变化规律 (如单调性、奇偶性、增量具有何种上界等) 往往成为解题的关键.\n本题中我们用到的是逆序数的奇偶性这个不变量.\n关于逆序数,一个基本且重要的结论是: 若 $n$ 个实数 $a_1, a_2, \\cdots, a_n$ 两两不等,则任取其中两个数对换位置后, 排列的逆序数奇偶性改变.\n特别地, 只需注意依次作 $(a, b) \\rightarrow(b, a)$ 和 $(a, c) \\rightarrow(c, a)$ 这两个对换恰好可将 $(a, b, c)$ 换成 $(b, c, a)$, 本题中逆序数的奇偶不变性就能轻易获得.", + "figures": [] +} \ No newline at end of file diff --git a/processed_dataset/proof/0675.json b/processed_dataset/proof/0675.json new file mode 100644 index 0000000000000000000000000000000000000000..bf43f655832b9a0ec3f3cfd87a70f22617b5edb8 --- /dev/null +++ b/processed_dataset/proof/0675.json @@ -0,0 +1,8 @@ +{ + "source_file": "./raw_volume-zh/volume14/chapter19.tex", + "problem_type": "proof", + "problem": "例5. 数列 $\\left\\{a_n\\right\\}$ 和 $\\left\\{b_n\\right\\}$ 满足 $a_1=1, b_1=2$,\n$$\na_{n+1}=\\frac{1+a_n+a_n b_n}{b_n}, b_{n+1}=\\frac{1+b_n+a_n b_n}{a_n},\n$$\n求证: $a_{2008}<5$.", + "solution": "证明:已知得\n$$\n1+a_{n+1}=\\frac{\\left(1+a_n\\right)\\left(1+b_n\\right)}{b_n}, 1+b_{n+1}=\\frac{\\left(1+a_n\\right)\\left(1+b_n\\right)}{a_n},\n$$\n且根据递推关系显然有 $a_n, b_n>0$, 故当 $n \\in \\mathbf{N}^*$ 时,\n$$\n\\begin{aligned}\n& \\frac{1}{1+a_{n+1}}-\\frac{1}{1+b_{n+1}} \\\\\n= & \\frac{b_n-a_n}{\\left(1+a_n\\right)\\left(1+b_n\\right)} \\\\\n= & \\frac{\\left(1+b_n\\right)-\\left(1+a_n\\right)}{\\left(1+a_n\\right)\\left(1+b_n\\right)} \\\\\n= & \\frac{1}{1+a_n}-\\frac{1}{1+b_n},\n\\end{aligned}\n$$\n这表明 $\\frac{1}{1+a_n}-\\frac{1}{1+b_n}$ 是不变量.\n以下有\n$$\n\\frac{1}{1+a_{2008}}>\\frac{1}{1+a_{2008}}-\\frac{1}{1+b_{2008}}=\\frac{1}{1+a_1}-\\frac{1}{1+b_1}=\\frac{1}{2}-\\frac{1}{3}=\\frac{1}{6},\n$$\n所以 $a_{2008}<5$.", + "remark": "注:具体问题中, 数量关系及变化形式可能多种多样, 要从中寻找出微妙的不变性并加以利用, 须对问题进行透彻的分析, 同时也离不开一定的解题经验.\n例如在本题中, 有一定解题经验的读者首先会想到在递推式两边加 1 , 而此后拟定计划时就有几个选择, 要做进一步的盘算.\n如能注意到取倒数并作差后所出现的裂项的结构特征, 便能发现不变量 $\\frac{1}{1+a_n}-\\frac{1}{1+b_n}$. 一旦抓住这个不变量并加以利用, 问题就迎刃而解了.", + "figures": [] +} \ No newline at end of file diff --git a/processed_dataset/proof/0676.json b/processed_dataset/proof/0676.json new file mode 100644 index 0000000000000000000000000000000000000000..dcbf60db39227dc9a4e5d79685990eff17e0498f --- /dev/null +++ b/processed_dataset/proof/0676.json @@ -0,0 +1,8 @@ +{ + "source_file": "./raw_volume-zh/volume14/chapter19.tex", + "problem_type": "proof", + "problem": "例7. 已知数列\n$$\n1,0,1,0,1,0,3, \\cdots\n$$\n中,每一项等于它前面 6 项之和的末位数字.\n证明: 在这个数列中不存在连续的 6 项依次是 $0,1,0,1,0,1$.", + "solution": "证明:这个数列中每连续的 6 项 $x, y, z, u, v, w$ 对应于数\n$$\n2 x+4 y+6 z+8 u+10 v+12 w\n$$\n的个位数字.\n例如对于开始的六项 $1,0,1,0,1,0$, 对应的数是\n$$\n2 \\times 1+4 \\times 0+6 \\times 1+8 \\times 0+10 \\times 1+12 \\times 0=18\n$$\n的个位数字 8 .\n如果 $x, y, z, u, v, w, r$ 是顺次的 7 项, 那么 $y, z, u, v, w, r$ 所对应的数 $B$ 与 $x, y, z, u, v, w$ 所对应的数 $A$ 应满足:\n$$\n\\begin{aligned}\nB-A & =(2 y+4 z+6 u+8 v+10 w+12 r)-(2 x+4 y+6 z+8 u+10 v+12 w) \\\\\n& =12 r-2(x+y+z+u+v+w) \\equiv 12 r-2 r \\equiv 0(\\bmod 10) .\n\\end{aligned}\n$$\n所以, 在每一项换成它后面一项时, 连续 6 项所对应的数保持不变, 即对于这个数列, 每连续 6 项所对应的数永远是 8 .\n假如 $0,1,0,1,0,1$ 能作为这个数列中的连续 6 项, 这时对应的数为\n$$\n2 \\times 0+1 \\times 1+6 \\times 0+8 \\times 1+10 \\times 0+12 \\times 1=24\n$$\n的个位数字 4 , 不等于 8 . 所以数列中不存在连续 6 项依次是 $0,1,0,1,0,1$.", + "remark": "注:本题递推阶数较高, 不宜从数列的周期性着手, 又若从模周期性 (例如奇偶性)考虑, 并不能推出矛盾.\n因此不妨设出一个具有 $a x+b y+c z+d u+ e v+f w$ 形式的不变量, 其中 $a$ 至 $f$ 为待定的整数.\n注意到若 $r \\equiv x+y+z+u+v+w(\\bmod 10)$, 则有\n$$\n\\begin{gathered}\n(a y+b z+c u+d v+e w+f r)-(a x+b y+c z+d u+e v+f w) \\\\\n\\equiv(f-a) x+(a+f-b) y+(b+f-c) z \\\\\n+(c+f-d) u+(d+f-e) v+e w(\\bmod 10),\n\\end{gathered}\n$$\n因此,适当地取整数 $a$ 至 $f$, 使 $x, y, z, u, v, w$ 前的系数均被 10 整除, 即可构造出这样的不变量.\n本题便是通过这样的不变量证明了结论.", + "figures": [] +} \ No newline at end of file diff --git a/processed_dataset/proof/0677.json b/processed_dataset/proof/0677.json new file mode 100644 index 0000000000000000000000000000000000000000..df0f16abb9a5594362c6e7c06db90f4c72e271eb --- /dev/null +++ b/processed_dataset/proof/0677.json @@ -0,0 +1,10 @@ +{ + "source_file": "./raw_volume-zh/volume14/chapter19.tex", + "problem_type": "proof", + "problem": "例8. 如图() 所示, 圆形的水池被分割为 $2 n(n \\geqslant 5)$个 \"格子\". 我们把有公共隔墙 (公共边或公共弧) 的\"格子\"称为相邻的, 从而每个\"格子\"都有三个邻格.\n水池中一共跳人了 $4 n+1$ 只青蛙,青蛙难于安静共处,只要某个\"格子\"中有不少于 3 只青蛙,那么迟早一定会有其中 3 只分别同时跳往三个不同邻格.\n证明: 只要经过一段时间之后, 青蛙便会在水池中大致分布均匀.\n所谓大致分布均匀, 就是任取其中一个\"格子\", 或者它里面有青蛙, 或者它的三个邻格里都有青蛙.", + "solution": "证明:们把一个格子中出现一次 3 只青蛙同时分别跳向三个邻格的事件称为该格子发生一次 \"爆发\". 而把一个格子或者是它里面有青蛙, 或者是它的三个相邻的格子里面都有青蛙, 称为该格子处于\"平衡状态\".\n容易看出,一个格子一旦有青蛙跳入, 那么它就一直处于 \"平衡状态\". 事实上,只要不 \"爆发\", 那么该格子中的青蛙不会动, 它当然处于 \"平衡状态\"; 而如果发生 \"爆发\", 那么它的三个邻格中就都有青蛙, 并且只要三个邻格都不\"爆发\", 那么它就一直处于\"平衡状态\"; 而不论哪个邻格发生\"爆发\", 都会有青蛙跳到它里面, 它里面就一定有青蛙, 所以它一直处于\"平衡状态\".\n这样一来, 为证明题中断言, 我们就只要证明: 任何一个格子都迟早会有青蛙跳入.\n任取一个格子,把它称为 $A$ 格, 把它所在的扇形称为 1 号扇形, 把该扇形中的另一个格子称为 $B$ 格, 我们要证明 $A$ 格迟早会有青蛙跳入.\n按顺时针方向依次将其余扇形接着编为 2 至 $n$ 号.\n首先证明 1 号扇形迟早会有青蛙跳人.\n假设 1 号扇形中永无青蛙到来,那么就不会有青蛙越过 1 号扇形与 $n$ 号扇形之间的隔墙.\n我们来考察青蛙所在的扇形编号的平方和.\n由于没有青蛙进人 1 号扇形 (尤其没有青蛙越过 1 号扇形与 $n$ 号扇形之间的隔墙), 所以只能是有 3 只青蛙由某个 $k(3 \\leqslant k \\leqslant n-1)$ 号扇形分别跳入 $k-1$, $k$ 和 $k+1$ 号扇形各一只,因此平方和的变化量为\n$$\n(k-1)^2+k^2+(k+1)^2-3 k^2=2,\n$$\n即增加 2. 一方面, 由于青蛙的跳动不会停止 (因为总有一个格子里有不少于 3 只青蛙), 所以平方和的增加趋势不会停止; 但是另一方面, 青蛙所在扇形编号的平方和不可能永无止境地增加下去 (不会大于 $(4 n+1) n^2$ ), 由此产生矛盾, 所以迟早会有青蛙越过 1 号扇形与 $n$ 号扇形之间的隔墙,进人 1 号扇形.\n我们再来证明 1 号扇形迟早会有 3 只青蛙跳人.\n如果 1 号扇形中至多有两只青蛙跳人, 那么它们都不会跳走, 并且自始至终上述平方和至多有两次变小 (只能在两只青蛙越过 1 号扇形与 $n$ 号扇形之间的隔墙时变小), 以后便一直持续不断地上升, 从而又重蹈刚才的矛盾.\n所以 1 号扇形迟早会有 3 只青蛙跳人.\n如果这 3 只青蛙中有位于 $A$ 格的, 那么 $A$ 格中已经有青蛙跳人; 如果这 3 只青蛙全都位于 $B$ 格, 那么 $B$ 格迟早会发生 \"爆发\", 从而有青蛙跳人 $A$ 格.", + "remark": "注:除了不变量方法, 本例中同时运用了化归、抽屉原理(用于保证青蛙的跳动不会停止)等解题思想方法.", + "figures": [ + "./images/volume14/figures/fig-c19i1.png" + ] +} \ No newline at end of file diff --git a/processed_dataset/proof/0678.json b/processed_dataset/proof/0678.json new file mode 100644 index 0000000000000000000000000000000000000000..a3687ede2586e1f36e620745e60eecdfe473cce8 --- /dev/null +++ b/processed_dataset/proof/0678.json @@ -0,0 +1,8 @@ +{ + "source_file": "./raw_volume-zh/volume14/chapter2.tex", + "problem_type": "proof", + "problem": "例1. 九名数学家在一次国际数学会议上相遇, 发现他们中的任意三个人中, 至少有两个人可以用同一种语言对话.\n如果每个数学家至多可说三种语言, 证明这些数学家中, 至少有三人可以用同一种语言对话.", + "solution": "证明:假定不存在三人能说同一种语言, 那么每种语言最多只有两人能说, 于是每个人用一种语言最多只能与另一个人对话.\n设这九名数学家是 $A_1, A_2, \\cdots, A_9$, 由于 $A_1$ 最多能说三种语言, 因此至少将与另外五个人, 不妨设是 $A_2, A_3, A_4, A_5, A_6$ 不能对话, 又因为 $A_2$ 也最多能说三种语言, 因而他至少与 $A_3, A_4, A_5, A_6$ 中的一个人不能对话, 不妨设是 $A_3$, 于是 $A_1, A_2, A_3$ 三个人互相之间都不能对话, 这与题设矛盾.\n所以,原结论正确.", + "remark": "注:本例结论中\"至少三人\"的反面是\"至多两人\". 反设是反证法的基础, 为了正确地作出反设, 熟练掌握一些常用的互为否定的表述形式是有必要的,例如: 是/不是; 都是/不都是; 存在/不存在; 大于/不大于; 至少有一个/ 一个也没有; 至多有一个/至少有两个; 存在一个……满足……/对任意都不满足……等等.\n还有一些表面上为肯定, 实质上为否定的表述也应注意到, 例如: \"互素\"即\"不存在大于 1 的公因数\"; \"无理数\"即\"不能表示为两个互素的整数之商的数\"等等.", + "figures": [] +} \ No newline at end of file diff --git a/processed_dataset/proof/0679.json b/processed_dataset/proof/0679.json new file mode 100644 index 0000000000000000000000000000000000000000..83391e53e3f89a45f701deb3a7fc4302dc77cec6 --- /dev/null +++ b/processed_dataset/proof/0679.json @@ -0,0 +1,8 @@ +{ + "source_file": "./raw_volume-zh/volume14/chapter2.tex", + "problem_type": "proof", + "problem": "例2. 证明: 对任意三角形, 一定存在它的两条边长 $a, b$, 满足\n$$\n1 \\leqslant \\frac{a}{b}<\\frac{1+\\sqrt{5}}{2}\n$$", + "solution": "证明:若结论不成立, 则对于 $\\triangle A B C$ 的三边长 $a, b, c$, 不妨设 $a \\geqslant b \\geqslant c$, 于是\n$$\n\\frac{a}{b} \\geqslant \\frac{1+\\sqrt{5}}{2}, \\frac{b}{c} \\geqslant \\frac{1+\\sqrt{5}}{2},\n$$\n即 $\\frac{b}{a} \\leqslant \\frac{\\sqrt{5}-1}{2}, \\frac{c}{b} \\leqslant \\frac{\\sqrt{5}-1}{2}$.\n从而\n$$\n\\frac{b+c}{a}=\\frac{b}{a}\\left(1+\\frac{c}{b}\\right) \\leqslant \\frac{\\sqrt{5}-1}{2}\\left(1+\\frac{\\sqrt{5}-1}{2}\\right)=1,\n$$\n但这与 $\\triangle A B C$ 中 $b+c>a$ 矛盾! 从而命题得证.", + "remark": "", + "figures": [] +} \ No newline at end of file diff --git a/processed_dataset/proof/0680.json b/processed_dataset/proof/0680.json new file mode 100644 index 0000000000000000000000000000000000000000..e4e463006e0ff384af2381ea8c32094073d64bfb --- /dev/null +++ b/processed_dataset/proof/0680.json @@ -0,0 +1,8 @@ +{ + "source_file": "./raw_volume-zh/volume14/chapter2.tex", + "problem_type": "proof", + "problem": "例3. 已知 5 根细棍中任意 3 根都可以首尾相接得到一个三角形.\n证明: 这些三角形中必存在锐角三角形.", + "solution": "证明:反证法, 假设 $a, b, c, d, e(0e$;\n由 $a 、 b 、 c$ 为针角或直角三角形的三边长知: $a^2+b^2 \\leqslant c^2$;\n由 $c 、 d 、 e$ 为钝角或直角三角形的三边长知: $c^2+d^2 \\leqslant e^2$, 于是\n$$\nc^2+d^2 \\leqslant e^2<(a+b)^2 \\leqslant 2\\left(a^2+b^2\\right) \\leqslant 2 c^2 \\leqslant c^2+d^2,\n$$\n矛盾!\n故假设不成立, 从而命题得证.", + "remark": "注:本例中可供利用的信息甚少, 若能通过反证法的假设获得较多可供利用的信息,则有利于打开局面.\n上述解法在反证法假设之下, 再辅以对 5 根细棍的长度排序, 这样就创设了大量明确的条件, 容易从中大做文章, 直到矛盾产生为止 (上述解答已经过 \"修枝剪叶\", 将不需要用来导出矛盾的条件全都去除了, 故而显得相对精简).", + "figures": [] +} \ No newline at end of file diff --git a/processed_dataset/proof/0681.json b/processed_dataset/proof/0681.json new file mode 100644 index 0000000000000000000000000000000000000000..4423a90224eb8954883e6b115a35403c68d579bc --- /dev/null +++ b/processed_dataset/proof/0681.json @@ -0,0 +1,8 @@ +{ + "source_file": "./raw_volume-zh/volume14/chapter2.tex", + "problem_type": "proof", + "problem": "例4. 证明: $\\triangle A B C$ 的六条内角三等分线中, 不存在某三条共点.", + "solution": "证明:反证法, 假设有三条内角三等分线共点,则必然分别引自 $A 、 B$ 、 $C$, 设它们共点于 $P$, 则\n$$\n\\frac{\\sin \\angle B A P}{\\sin \\angle A B P} \\cdot \\frac{\\sin \\angle C B P}{\\sin \\angle B C P} \\cdot \\frac{\\sin \\angle A C P}{\\sin \\angle C A P}=\\frac{B P}{A P} \\cdot \\frac{C P}{B P} \\cdot \\frac{A P}{C P}=1 . \\label{eq1}\n$$\n设 $A=3 \\alpha, B=3 \\beta, C=3 \\gamma$, 则 $0<\\alpha, \\beta, \\gamma<\\frac{\\pi}{3}, \\alpha+\\beta+\\gamma=\\frac{\\pi}{3}$.\n(1)当 $\\angle B A P=\\alpha, \\angle C B P=\\beta, \\angle A C P=\\gamma$ 时, 式\\ref{eq1}即为\n$$\n\\frac{\\sin \\alpha}{\\sin 2 \\beta} \\cdot \\frac{\\sin \\beta}{\\sin 2 \\gamma} \\cdot \\frac{\\sin \\gamma}{\\sin 2 \\alpha}=1 \\text {, }\n$$\n化简得\n$$\n\\cos \\alpha \\cos \\beta \\cos \\gamma=\\frac{1}{8},\n$$\n但 $\\cos \\alpha \\cos \\beta \\cos \\gamma>\\left(\\cos \\frac{\\pi}{3}\\right)^3=\\frac{1}{8}$, 矛盾!\n(2) 当 $\\angle B A P=\\alpha, \\angle C B P=\\beta, \\angle A C P=\\gamma$ 中恰有两个成立时, 不妨设 $\\angle B A P=\\alpha, \\angle C B P=\\beta, \\angle A C P=2 \\gamma$, 则(1)成为\n$$\n\\frac{\\sin \\alpha}{\\sin 2 \\beta} \\cdot \\frac{\\sin \\beta}{\\sin \\gamma} \\cdot \\frac{\\sin 2 \\gamma}{\\sin 2 \\alpha}=1,\n$$\n化简得\n$$\n\\cos \\gamma=2 \\cos \\alpha \\cos \\beta,\n$$\n但\n$$\n\\begin{aligned}\n1 & >\\cos \\gamma=2 \\cos \\alpha \\cos \\beta=\\cos (\\alpha-\\beta)+\\cos (\\alpha+\\beta) \\\\\n& >2 \\cos (\\alpha+\\beta)>2 \\cos \\frac{\\pi}{3}=1,\n\\end{aligned}\n$$\n矛盾!\n(3) 当 $\\angle B A P=\\alpha, \\angle C B P=\\beta, \\angle A C P=\\gamma$ 中至多有一个成立时, 转而考察 $\\angle A B P, \\angle B C P, \\angle C A P$, 与 (1)、(2)类似, 可推得矛盾!\n综上, 假设不成立, 即不存在三条内角三等分线共点.", + "remark": "注:本例中\"不存在某三条共点\" 是一种否定判断, 作为其反面的肯定判断更为具体、明确, 故采用反证法.\n注意本题中结论的反面情形不止一种类型,但可以通过分类讨论逐一排除.", + "figures": [] +} \ No newline at end of file diff --git a/processed_dataset/proof/0682.json b/processed_dataset/proof/0682.json new file mode 100644 index 0000000000000000000000000000000000000000..0601e01eb80b0c860d1a3a5192050394d71e00a3 --- /dev/null +++ b/processed_dataset/proof/0682.json @@ -0,0 +1,8 @@ +{ + "source_file": "./raw_volume-zh/volume14/chapter2.tex", + "problem_type": "proof", + "problem": "例6. 一个棱柱以五边形 $A_1 A_2 A_3 A_4 A_5$ 与 $B_1 B_2 B_3 B_4 B_5$ 为上、下底, 这两个五边形的每一条边及每条线段 $A_i B_j(i, j=1,2,3,4,5)$ 均染上红色或蓝色.\n每一个以棱柱顶点为顶点, 以已染色的线段为边的三角形均有两条边颜色不同.\n求证:上、下底的 10 条边颜色一定相同.", + "solution": "证明:证明上底的 5 条边颜色相同.\n用反证法.\n若不然, 不妨设 $A_1 A_2$ 为红色, $A_1 A_5$ 为蓝色.\n在 $A_1 B_j(j=1,2,3,4,5)$ 这 5 条线段中,至少有三条颜色相同,且这三条线段在下底面上的端点必有两个是相邻的, 不妨设 $A_1 B_1, A_1 B_2$ 均为红色, 其中 $B_1, B_2$ 相邻.\n考虑 $\\triangle A_1 A_2 B_1, \\triangle A_1 A_2 B_2, \\triangle A_1 B_1 B_2$, 由于 $A_1 A_2, A_1 B_1, A_1 B_2$ 为红色, 根据题意可得 $A_2 B_1, A_2 B_2, B_1 B_2$ 必为蓝色, 但此时 $\\triangle A_2 B_1 B_2$ 三边都是蓝色,与已知矛盾! 这就证明了上底的 5 条边颜色相同.\n同理可得下底的 5 条边颜色相同.\n故余下只要证明上、下底的颜色一样.\n仍用反证法.\n若不然,不妨假设 $A_1 A_2 A_3 A_4 A_5$ 各边为红色, $B_1 B_2 B_3 B_4 B_5$ 各边为蓝色.\n前面已证明在 $A_1 B_j(j=1,2,3,4,5)$ 这 5 条线段中, 必有两条同色线段在下底面上的端点相邻, 不妨设 $A_1 B_1, A_1 B_2$ 同色, 其中由于 $B_1 B_2$ 为蓝边, 故 $A_1 B_1, A_1 B_2$ 均为红色, 和前面完全一样导致矛盾.\n所以上、下底的 10 条边颜色完全相同.", + "remark": "", + "figures": [] +} \ No newline at end of file diff --git a/processed_dataset/proof/0683.json b/processed_dataset/proof/0683.json new file mode 100644 index 0000000000000000000000000000000000000000..1835910f6342a39b56467749a64af26499187951 --- /dev/null +++ b/processed_dataset/proof/0683.json @@ -0,0 +1,8 @@ +{ + "source_file": "./raw_volume-zh/volume14/chapter2.tex", + "problem_type": "proof", + "problem": "例7. 设 $a_0, a_1, a_2, \\cdots$ 为任意无穷正实数数列.\n求证: 不等式 $1+a_n> \\sqrt[n]{2} a_{n-1}$ 对无穷多个正整数 $n$ 成立.", + "solution": "证明:设 $1+a_n>\\sqrt[n]{2} a_{n-1}$ 仅对有限个正整数成立.\n设这些正整数中最大的一个为 $M$, 则对任意的正整数 $n>M$, 上述不等式均不成立, 即有\n$$\n1+a_n \\leqslant \\sqrt[n]{2} a_{n-1}(n>M), \n$$\n也就是\n$$\na_n \\leqslant \\sqrt[n]{2} a_{n-1}-1(n>M) .\n\\end{aligned}\n$$\n由伯努利不等式:\n$$\n\\begin{gathered}\n\\sqrt[n]{2}=(1+1)^{\\frac{1}{n}}<1+\\frac{1}{n}=\\frac{n+1}{n}(n \\geqslant 2), \\\\\na_n<\\frac{n+1}{n} a_{n-1}-1(n>M) .\n\\end{gathered}\n$$\n可得\n$$\na_n<\\frac{n+1}{n} a_{n-1}-1(n>M) .\n$$\n下面对一切非负整数 $n$ 用数学归纳法证明:\n$$\na_{M+n} \\leqslant(M+n+1)\\left(\\frac{a_M}{M+1}-\\frac{1}{M+2}-\\cdots-\\frac{1}{M+n+1}\\right) \\label{eq1}.\n$$\n当 $n=0$ 时, 不等式两边都等于 $a_M$, 成立.\n假设当 $n=k(k \\in \\mathbf{N})$ 时成立, 即有\n$$\na_{M+k} \\leqslant(M+k+1)\\left(\\frac{a_M}{M+1}-\\frac{1}{M+2}-\\cdots-\\frac{1}{M+k+1}\\right) .\n$$\n于是得\n$$\n\\begin{aligned}\na_{M+k+1} \\leqslant & \\frac{M+k+2}{M+k+1} a_{M+k}-1 \\\\\n\\leqslant & \\frac{M+k+2}{M+k+1}(M+k+1)\\left(\\frac{a_M}{M+1}-\\frac{1}{M+2}-\\cdots-\\frac{1}{M+k+1}\\right)-1 \\\\\n= & (M+k+2)\\left(\\frac{a_M}{M+1}-\\frac{1}{M+2}-\\cdots-\\frac{1}{M+k+1}\\right)-1 \\\\\n= & (M+k+2)\\left(\\frac{a_M}{M+1}-\\frac{1}{M+2}-\\cdots-\\frac{1}{M+k+1}-\\right. \\\\\n& \\left.\\frac{1}{M+k+2}\\right) .\n\\end{aligned}\n$$\n故由归纳假设知,在 $n=k+1$ 时, 原不等式成立.\n易知\n$$\n\\lim _{n \\rightarrow \\infty}\\left(\\frac{1}{M+2}+\\frac{1}{M+3}+\\cdots+\\frac{1}{n}\\right)=+\\infty,\n$$\n故存在正整数 $N_0 \\geqslant M+2$, 满足 $\\frac{1}{M+2}+\\frac{1}{M+3}+\\cdots+\\frac{1}{N_0}>\\frac{a_M}{M+1}$.\n在式\\ref{eq1}中取 $n=N_0-M-1$, 得 $a_{N_0-1}<0$, 矛盾.\n故原命题得证.", + "remark": "", + "figures": [] +} \ No newline at end of file diff --git a/processed_dataset/proof/0684.json b/processed_dataset/proof/0684.json new file mode 100644 index 0000000000000000000000000000000000000000..c3d93d0f4ca47572f5a6bd38d7663d28c967bd02 --- /dev/null +++ b/processed_dataset/proof/0684.json @@ -0,0 +1,11 @@ +{ + "source_file": "./raw_volume-zh/volume14/chapter2.tex", + "problem_type": "proof", + "problem": "例8. 如图(),锐角三角形 $A B C$ 的外心为 $O, K$ 是边 $B C$ 上一点 (不是边 $B C$ 的中点), $D$ 是线段 $A K$ 延长线上一点, 直线 $B D$ 与 $A C$ 交于点 $N$, 直线 $C D$ 与 $A B$ 交于点 $M$. \n求证: 若 $O K \\perp M N$, 则 $A, B, D, C$ 四点共圆.", + "solution": "证明:用反证法.\n若 $A, B, D, C$ 不共圆, 设三角形 $A B C$ 的外接圆与射线 $A D$ 交于点 $E$,连接 $B E$ 并延长交直线 $A N$ 于点 $Q$, 连接 $C E$ 并延长交直线 $A M$ 于点 $P$, 连接 $P Q$.\n因为 $P K^2=P$ 的幂 (关于 $\\odot O$)+$K$ 的幂 (关于 $\\odot O$ ) (见注一), 所以\n$$\nP K^2=\\left(P O^2-r^2\\right)+\\left(K O^2-r^2\\right),\n$$\n同理\n$$\nQ K^2=\\left(Q O^2-r^2\\right)+\\left(K O^2-r^2\\right),\n$$\n所以\n$$\nP O^2-P K^2=Q O^2-Q K^2,\n$$\n故\n$$\nO K \\perp P Q .\n$$\n由题设, $O K \\perp M N$, 所以 $P Q / / M N$, 于是\n$$\n\\frac{A Q}{Q N}=\\frac{A P}{P M} \\label{eq1}\n$$\n由梅内劳斯 (Menelaus)定理,得\n$$\n\\begin{aligned}\n& \\frac{N B}{B D} \\cdot \\frac{D E}{E A} \\cdot \\frac{A Q}{Q N}=1, \\label{eq2} \\\\\n& \\frac{M C}{C D} \\cdot \\frac{D E}{E A} \\cdot \\frac{A P}{P M}=1 . \\label{eq3}\n\\end{aligned}\n$$\n由式\\ref{eq1}, \\ref{eq2}, 式\\ref{eq3},可得\n$$\n\\frac{N B}{B D}=\\frac{M C}{C D}\n$$\n所以 $\\frac{N D}{B D}=\\frac{M D}{D C}$, 故 $\\triangle D M N \\backsim \\triangle D C B$, 于是 $\\angle D M N=\\angle D C B$, 所以 $B C / / M N$, 故 $O K \\perp B C$, 即 $K$ 为 $B C$ 的中点,矛盾! 从而 $A, B, D, C$ 四点共圆.", + "remark": "注一 \" $P K^2=P$ 的幂 (关于 $\\left.\\odot O\\right)+K$ 的幂 (关于 $\\odot O)$ )\"的证明: 延长 $P K$ 至点 $F$( 如图()), 使得\n$$\nP K \\cdot K F=A K \\cdot K E, \\label{eq4}\n$$\n则 $P, E, F, A$ 四点共圆,故\n$$\n\\angle P F E=\\angle P A E=\\angle B C E,\n$$\n从而 $E, C, F, K$ 四点共圆,于是\n$$\nP K \\cdot P F=P E \\cdot P C, \\label{eq5}\n$$\n式\\ref{eq5}-\\ref{eq4}, 得\n$$\n\\begin{aligned}\nP K^2 & =P E \\cdot P C-A K \\cdot K E \\\\\n& =P \\text { 的幂 }(\\text { 关于 } \\odot O)+K \\text { 的幂 }(\\text { 关于 } \\odot O) .\n\\end{aligned}\n$$\n注二若点 $E$ 在线段 $A D$ 的延长线上,完全类似.\n注三本例有深刻的射影几何背景及较高的难度.\n该结论虽有直接证法, 但其逆命题是一个已有结果, 相对而言 \"对结论反面的否定\"易于\"对结论本身的直接证明\",故而会想到采用反证法.", + "figures": [ + "./images/volume14/figures/fig-c2i1.png", + "./images/volume14/figures/fig-c2i2.png" + ] +} \ No newline at end of file diff --git a/processed_dataset/proof/0685.json b/processed_dataset/proof/0685.json new file mode 100644 index 0000000000000000000000000000000000000000..01fc86071b6b8339de5ec73898e49c298b9e0d50 --- /dev/null +++ b/processed_dataset/proof/0685.json @@ -0,0 +1,8 @@ +{ + "source_file": "./raw_volume-zh/volume14/chapter2.tex", + "problem_type": "proof", + "problem": "例9. 证明: 方程 $2 x^3+5 x-2=0$ 恰有一个实数根 $r$, 且存在唯一的严格递增正整数数列 $\\left\\{a_n\\right\\}$, 使得 $\\frac{2}{5}=r^{a_1}+r^{a_2}+r^{a_3}+\\cdots$.", + "solution": "证明: $f(x)=2 x^3+5 x-2$, 则 $f^{\\prime}(x)=6 x^2+5>0$, 所以 $f(x)$ 是严格递增的.\n又 $f(0)=-2<0, f\\left(\\frac{1}{2}\\right)=\\frac{3}{4}>0$, 故 $f(x)$ 有唯一实数根 $r \\in \\left(0, \\frac{1}{2}\\right)$. 所以\n$$\n\\begin{aligned}\n& 2 r^3+5 r-2=0, \\\\\n& \\frac{2}{5}=\\frac{r}{1-r^3}=r+r^4+r^7+r^{10}+\\cdots .\n\\end{aligned}\n$$\n故数列 $a_n=3 n-2(n=1,2, \\cdots)$ 是满足题设要求的数列.\n若存在两个不同的正整数数列 $a_1) 作正方体 $A B C D- E F G H$, 记 $E H$ 中点为 $M$, 易验证直线 $A B, B F, F H$, $H M, M A$ 满足题意.\n综上, $n$ 的最大可能值为 5 .", + "remark": "注:本题中,若只从空间形态来考虑问题将不胜其烦, 而一旦转化成图论语言, 回到了拉姆赛(Ramsey)问题这一图论中的重要背景(参考第 14 节例 1), 解题目标就变得十分明确了.", + "figures": [ + "./images/volume14/figures/fig-c20i1.png" + ] +} \ No newline at end of file diff --git a/processed_dataset/proof/0690.json b/processed_dataset/proof/0690.json new file mode 100644 index 0000000000000000000000000000000000000000..84099fd62d938cd222f41c564d5eac9bc3de1232 --- /dev/null +++ b/processed_dataset/proof/0690.json @@ -0,0 +1,8 @@ +{ + "source_file": "./raw_volume-zh/volume14/chapter20.tex", + "problem_type": "proof", + "problem": "例4. 已知 $\\triangle A B C$ 内有 $m$ 个点, 连同 $A, B, C$ 三点一共 $m+3$ 个点.\n以这些点为顶点将 $\\triangle A B C$ 分成若干个互不重叠的小三角形.\n将 $A, B, C$ 三点分别染成红色、黄色、蓝色.\n而三角形内的 $m$ 个点, 每个点任意染成红色、黄色、 蓝色三色之一.\n问: 三个顶点颜色都不同的小三角形的个数是奇数还是偶数?", + "solution": "证明:这样的方法构造一个图 $G$ : 在 $\\triangle A B C$ 外及每个小三角形内各取一点代表它们所在的区域,这些点构成图 $G$ 的顶点集合; 当两个顶点 $u, v$ 所在区域有一条公共边, 且公共边端点是红、黄两种颜色时, 在 $u, v$ 间连一条边.\n一个具有颜色红、黄、蓝顶点的小三角形对应于图 $G$ 中度为 1 的顶点, 其余小三角形均对应 $G$ 中度为 0 或 2 的顶点, 而 $\\triangle A B C$ 外部顶点 $u$ 的度是 1 . 由于图 $G$ 中奇顶点的个数必为偶数, 所以除了 $u$ 之外奇顶点的个数为奇数, 因此三个顶点颜色都不同的小三角形的个数是奇数.", + "remark": "注:很多问题转化为图论问题后, 能以图论中比较完整的理论体系作依托来解题 (前面的例 3 就是这方面的典型). 本题从奇偶分析人手, 运用了有限图中 \"奇顶点的个数为偶数\" 这个熟知的结论, 简单明了地解决了问题.", + "figures": [] +} \ No newline at end of file diff --git a/processed_dataset/proof/0691.json b/processed_dataset/proof/0691.json new file mode 100644 index 0000000000000000000000000000000000000000..262f6faec154717d0187e586a327385f06f8cf64 --- /dev/null +++ b/processed_dataset/proof/0691.json @@ -0,0 +1,8 @@ +{ + "source_file": "./raw_volume-zh/volume14/chapter20.tex", + "problem_type": "proof", + "problem": "例5. 在某学校里有 $b$ 名教师和 $c$ 名学生.\n假设满足下列条件:\n(1) 每名教师恰好教 $k$ 名学生;\n(2) 对任意两名学生,恰好有 $h$ 名教师教他们两人.\n证明: $\\frac{b}{h}=\\frac{c(c-1)}{k(k-1)}$.", + "solution": "证明:构造一个图 $G$ : $G$ 含有两组顶点 $A, B$, 其中每个老师对应 $A$ 中的一个顶点, 每个 \"学生对\"对应 $B$ 中的一个顶点, 若某位老师正好教某一学生对, 则在 $A, B$ 相应顶点之间连一条边.\n由 (1) 得, $A$ 中每个顶点与 $B$ 中 $\\mathrm{C}_k^2$ 个顶点相邻, 故 $G$ 共有 $b \\cdot \\mathrm{C}_k^2$ 条边.\n由 (2) 得, $B$ 中每个顶点与 $A$ 中 $h$ 个顶点相邻, 故 $G$ 共有 $\\mathrm{C}_c^2 \\cdot h$ 条边.\n从而 $b \\cdot \\mathrm{C}_k^2=\\mathrm{C}_c^2 \\cdot h$, 即\n$$\n\\frac{b}{h}=\\frac{c}{k}(c-1)\n$$", + "remark": "注:本题运用模型化思想, 构造图的模型来阐述问题.\n上述图 $G$ 的两组顶点中, 每组内不存在相邻顶点, 这样的图称为 2 部图或偶图.\n从每组顶点出发, 对 2 部图的边数从两方面计数,即得到我们所需要的恒等式.", + "figures": [] +} \ No newline at end of file diff --git a/processed_dataset/proof/0692.json b/processed_dataset/proof/0692.json new file mode 100644 index 0000000000000000000000000000000000000000..c69f02b3d0b58535a4e6971c4bc28fef75a048ce --- /dev/null +++ b/processed_dataset/proof/0692.json @@ -0,0 +1,8 @@ +{ + "source_file": "./raw_volume-zh/volume14/chapter20.tex", + "problem_type": "proof", + "problem": "例6. 34 对选手参加双人舞比赛, 赛前, 某些选手互相握手.\n同一对的两人不握手.\n后来, 某男选手问其他 67 名参赛选手他们与人握手的次数, 得到的答案都不相同.\n问: 该男选手的搭档女选手和多少人握过手?", + "solution": "解: 68 个顶点顶点表示 68 个参赛选手.\n对于顶点 $u, v$, 当且仅当 $u, v$ 所表示的两名选手握过手时, 令它们相邻, 于是得到一个 68 个顶点的简单图 $G$.\n由于同一对的两名选手不握手, 所以对任意顶点 $u, d(u) \\leqslant 66$.\n设某男选手为 $x$. 图 $G$ 中除顶点 $x$ 外尚有 67 个点, 由题意, 它们的度各不相同, 因此必有一个点 $v$ 满足 $d(v)=0$, 即 $G$ 中没有一个人和 $v$ 握过手.\n所以 $d(w)=66$ 的那名选手 $w$ 只能与 $v$ 来自同一对.\n从图 $G$ 中去掉 $v$ 和 $w$, 得到含 66 个顶点的图 $G_1$. 则 $x$ 是 $G_1$ 中的顶点, 并且除 $x$ 之外, 其他顶点的度也都不相同.\n因此和前述证明相同, $G_1$ 含有度分别为 0 和 64 的顶点 $p$ 和 $q$, 它们在原来图 $G$ 中的度分别为 1 和 65. 如此继续,\n可证得对 $0 \\leqslant j \\leqslant 33$, 图 $G$ 中含有顶点 $x_j, y_j$, 它们的度分别为 $j$ 和 $66-j$, 而且所代表的选手来自同一对, 特别地, 最后有 $x_{33}=x$, 所以 $d\\left(x_{33}\\right)=33$. 因此该男选手的搭档女选手握手次数为 33 .", + "remark": "注:将有限图 $G$ 的顶点编号, 按度的非降次序 $\\left(d_1 \\leqslant d_2 \\leqslant \\cdots \\leqslant d_n\\right)$ 排列, 所得到的 $\\left(d_1, d_2, \\cdots, d_n\\right)$ 称为 $G$ 的度序列.\n度序列是图的基本特征之一.\n本题中, 我们从度序列考虑问题, 这是解题的一种重要方法.", + "figures": [] +} \ No newline at end of file diff --git a/processed_dataset/proof/0693.json b/processed_dataset/proof/0693.json new file mode 100644 index 0000000000000000000000000000000000000000..f3038a613bec48ba5a875963ea8b81f9d3d466f0 --- /dev/null +++ b/processed_dataset/proof/0693.json @@ -0,0 +1,10 @@ +{ + "source_file": "./raw_volume-zh/volume14/chapter20.tex", + "problem_type": "proof", + "problem": "例7. 求满足如下条件的最小正整数 $n$, 在圆 $O$ 的圆周上任取 $n$ 个点 $A_1$, $A_2, \\cdots, A_n$, 则在 $\\mathrm{C}_n^2$ 个角 $\\angle A_i O A_j(1 \\leqslant i), 设 $A B$ 是圆 $O$ 的直径, 在点 $A$ 和 $B$ 的附近分别取 45 个点, 此时, 只有 $2 \\mathrm{C}_{45}^2=45 \\times 44=1980$ 个角不超过 $120^{\\circ}$, 所以, $n=90$ 不满足题意.\n当 $n=91$ 时,下面证明至少有 2010 个角不超过 $120^{\\circ}$.\n把圆周上的 91 个点 $A_1, A_2, \\cdots, A_{91}$ 看作一个图的 91 个顶点 $v_1$, $v_2, \\cdots, v_{91}$, 若 $\\angle A_i O A_j>120^{\\circ}$, 则在它们对应的顶点 $v_i, v_j$ 之间连一条边, 这样就得到一个图 $G$.\n设图 $G$ 中有 $e$ 条边, 易知, 图中没有三角形.\n若 $e=0$, 则有 $\\mathrm{C}_{91}^2=4095>2010$. 个角不超过 $120^{\\circ}$, 命题得证.\n若 $e \\geqslant 1$, 不妨设顶点 $v_1, v_2$ 之间有边相连, 因为图中没有三角形, 所以, 对于顶点 $v_i(i=3,4, \\cdots, 91)$, 它至多与 $v_1, v_2$ 中的一个有边相连, 所以\n$$\nd\\left(v_1\\right)+d\\left(v_2\\right) \\leqslant 89+2=91,\n$$\n其中 $d(v)$ 表示顶点 $v$ 的度, 即顶点 $v$ 处引出的边数.\n因为 $d\\left(v_1\\right)+d\\left(v_2\\right)+\\cdots+d\\left(v_{91}\\right)=2 e$, 而对于图 $G$ 中的每一条边的两个顶点 $v_i, v_j$, 都有\n$$\nd\\left(v_i\\right)+d\\left(v_j\\right) \\leqslant 91,\n$$\n于是, 上式对每一条边求和可得\n$$\n\\left(d\\left(v_1\\right)\\right)^2+\\left(d\\left(v_2\\right)\\right)^2+\\cdots+\\left(d\\left(v_{91}\\right)\\right)^2 \\leqslant 91 e,\n$$\n由柯西不等式得\n$$\n\\begin{aligned}\n& 91\\left[\\left(d\\left(v_1\\right)\\right)^2+\\left(d\\left(v_2\\right)\\right)^2+\\cdots+\\left(d\\left(v_{91}\\right)\\right)^2\\right] \\\\\n\\geqslant & {\\left[d\\left(v_1\\right)+d\\left(v_2\\right)+\\cdots+d\\left(v_{91}\\right)\\right]^2=4 e^2, }\n\\end{aligned}\n$$\n所以 $\\frac{4 e^2}{91} \\leqslant\\left(d\\left(v_1\\right)\\right)^2+\\left(d\\left(v_2\\right)\\right)^2+\\cdots+\\left(d\\left(v_{91}\\right)\\right)^2 \\leqslant 91 e$, 故\n$$\ne \\leqslant \\frac{91^2}{4}<2071\n$$\n所以, 91 个顶点中, 至少有 $\\mathrm{C}_{91}^2-2071=2024>2010$ 个点对, 它们之间没有边相连, 从而, 它们对应的顶点所对应的角不超过 $120^{\\circ}$.\n综上所述, $n$ 但最小值为 91 .", + "remark": "", + "figures": [ + "./images/volume14/figures/fig-c20i2.png" + ] +} \ No newline at end of file diff --git a/processed_dataset/proof/0694.json b/processed_dataset/proof/0694.json new file mode 100644 index 0000000000000000000000000000000000000000..37592a497c05a262f5fbce81e4775769742055cf --- /dev/null +++ b/processed_dataset/proof/0694.json @@ -0,0 +1,8 @@ +{ + "source_file": "./raw_volume-zh/volume14/chapter20.tex", + "problem_type": "proof", + "problem": "例8. 设 $F$ 是一个由整数组成的有限集,满足:\n(1) 对任意 $x \\in F$, 存在 $y, z \\in F$ (可以相同), 使得 $x=y+z$;\n(2) 存在 $n \\in \\mathbf{N}^*$, 使得对任何正整数 $k(1 \\leqslant k \\leqslant n)$ 及任意 $x_1, x_2, \\cdots$, $x_k \\in F$ (可以相同), 都有 $\\sum_{i=1}^k x_i \\neq 0$.\n求证: $F$ 至少含有 $2 n+2$ 个元素.", + "solution": "证明:然 $0 \\notin F$, 且 $F$ 中所有元素不全同号(否则,绝对值最小的完素 $x$ 无法表示成 $y+z(y, z \\in F)$ 的形式).\n设 $x_1, x_2, \\cdots, x_m$ 是 $F$ 中所有正的元素.\n我们证明 $m \\geqslant n+1$.\n作含 $m$ 个顶点 $v_1, v_2, \\cdots, v_m$ 的有向图 $G$ 如下: 对每个 $i \\in\\{1,2, \\cdots$, $m\\}$, 由于存在 $y, z \\in F$, 使 $x_i=y+z>0$, 不妨设 $y>0$, 则 $y$ 必等于某个 $x_j(j \\neq i)$, 如此就在 $G$ 中作一条有向边 $v_i \\rightarrow v_j$.\n由 $G$ 的作法可知每个顶点 $v_i$ 的出度为 1 , 因此 $G$ 中必存在圈, 不妨设为\n$$\nv_{i_1} \\rightarrow v_{i_2} \\rightarrow \\cdots \\rightarrow v_{i_k} \\rightarrow v_{i_1}(k \\leqslant m) .\n$$\n这表明存在 $z_1, z_2, \\cdots, z_k \\in F$, 使得\n$$\n\\left\\{\\begin{array}{l}\nx_{i_1}=x_{i_2}+z_1, \\\\\nx_{i_2}=x_{i_3}+z_2, \\\\\n\\cdots \\\\\nx_{i_k}=x_{i_1}+z_k,\n\\end{array}\\right.\n$$\n求和得\n$$\nz_1+z_2+\\cdots+z_k=0 .\n$$\n为不与条件 (2) 矛盾, 必有 $k>n$, 从而 $m \\geqslant k \\geqslant n+1$.\n同理知 $F$ 中负的元素也至少有 $n+1$ 个, 由此可知 $F$ 至少含有 $2 n+2$ 个元素.", + "remark": "注:本题巧妙构造有向图来解题.\n从本题的论证中可以发现, 即使对 $F$ 为有限实数集合的情况, 结论仍是成立的.\n读者可进一步考虑 $F$ 能否恰含有 $2 n+2$ 个元素.", + "figures": [] +} \ No newline at end of file diff --git a/processed_dataset/proof/0695.json b/processed_dataset/proof/0695.json new file mode 100644 index 0000000000000000000000000000000000000000..c934cd4204cccc444dea97ad335b263d9c08d828 --- /dev/null +++ b/processed_dataset/proof/0695.json @@ -0,0 +1,8 @@ +{ + "source_file": "./raw_volume-zh/volume14/chapter3.tex", + "problem_type": "proof", + "problem": "例1. 设 $\\left\\{x_n\\right\\}$ 是一实数列, 且对任一非负整数 $n$, 满足\n$$\nx_0^3+x_1^3+\\cdots+x_n^3=\\left(x_0+x_1+\\cdots+x_n\\right)^2 .\n$$\n求证: 对所有非负整数 $n$, 存在整数 $m$, 使得 $x_0+x_1+\\cdots+x_n= \\frac{m(m+1)}{2}$.", + "solution": "证明:我们用数学归纳法来证明结论.\n当 $n=0$ 时, 由题设, $x_0^3=x_0^2$, 得 $x_0=0$ 或 $1$. 这时可取 $m=0$ 或 $1$ , 从而当 $n=0$ 时结论成立.\n设 $n=k$ 时结论成立, 即当 $x_0^3+x_1^3+\\cdots+x_k^3=\\left(x_0+x_1+\\cdots+x_k\\right)^2$ 时, 存在整数 $m$, 使得 $x_0+x_1+\\cdots+x_k=\\frac{m(m+1)}{2}$.\n为方便书写, 记 $\\frac{m(m+1)}{2}=c$, 则 $x_0^3+x_1^3+\\cdots+x_k^3=c^2$.\n对于 $x_{k+1}$, 如果 $x_0^3-+x_1^3+\\cdots+x_k^3+x_{k+1}^3=\\left(x_0+x_1+\\cdots+x_k+x_{k+1}\\right)^2$, 那么 $c^2+x_{k+1}^3=\\left(c+x_{k+1}\\right)^2$, 所以 $x_{k+1}\\left(x_{k+1}^2-x_{k+1}-m(m+1)\\right)=0$, 解得 $x_{k+1}=0,-m, m+1$.\n当 $x_{k+1}=0$ 时, $x_0+x_1+\\cdots+x_k+x_{k+1}=\\frac{m(m+1)}{2}$;\n当 $x_{k+1}=-m$ 时, $x_0+x_1+\\cdots+x_k+x_{k+1}=\\frac{m(m+1)}{2}-m=\\frac{m(m-1)}{2}$;\n当 $x_{k+1}=m+1$ 时, $x_0+x_1+\\cdots+x_k+x_{k+1}=\\frac{m(m+1)}{2}+m+1=$\n$$\n\\frac{(m+1)(m+2)}{2}\n$$\n即结论在 $n=k+1$ 时也成立, 从而对一切非负整数 $n$, 结论成立.", + "remark": "注:运用数学归纳法时, 证明的核心与难点多数情况下在于如何实现归纳过渡.\n此时应设法使 $P(k)$ 与 $P(k+1)$ 的关系充分显露出来, 根据题目的特点, 或是从 $P(k)$ 出发, 过渡到 $P(k+1)$, 或是先从 $P(k+1)$ 人手, 从中分离出 $P(k)$ 的形式, 某些时候还需对一些代数式和命题进行反复变形转化.\n总之应当创设条件,使归纳假设得以充分利用.", + "figures": [] +} \ No newline at end of file diff --git a/processed_dataset/proof/0696.json b/processed_dataset/proof/0696.json new file mode 100644 index 0000000000000000000000000000000000000000..fb0ba72449b3e32154ee90a9330154a040ac6eed --- /dev/null +++ b/processed_dataset/proof/0696.json @@ -0,0 +1,8 @@ +{ + "source_file": "./raw_volume-zh/volume14/chapter3.tex", + "problem_type": "proof", + "problem": "例2. 设有 $2^n$ 个球分成了许多堆, 我们可以任意选取甲、乙两堆按如下规则挪动: 若甲堆中的球数 $p$ 不小于乙堆中的球数 $q$, 则从甲堆中拿出 $q$ 球放人乙堆, 这算是挪动一次.\n证明可以经过有限次挪动把所有球并成一堆.", + "solution": "证明:对 $n$ 用数学归纳法.\n当 $n=1$ 时, 只有两个球, 至多挪动一次即可,结论成立.\n设 $n=k$ 时结论成立.\n在 $n=k+1$ 的情形下, 首先注意到总球数为偶数, 所以球数为奇数的堆必有偶数个, 先将它们两两配对, 并在每对的两堆球之间进行一次挪动, 其中对每对球堆, 设球数分别为奇数 $p$ 和 $q(p \\geqslant q)$, 那么挪动后两堆球的个数分别为偶数 $p-q, 2 q$. 因此一系列挪动后可使所有这些堆中的球数都变为偶数 (球数变为 0 个的球堆就消失了). 这时将每堆中的球两两捆绑起来视为一个\"球\", 于是总 \"球\" 数变为 $2^k$, 由归纳假设知可以在有限步挪动后全部并为一堆,所以 $n=k+1$ 的情形结论也成立.\n由数学归纳法得: 对一切正整数 $n$ 结论成立.", + "remark": "注:本题中 $P(k)$ 与 $P(k+1)$ 的关系是球数的 2 倍关系.\n如果我们能在 $n =k+1$ 的情形下找到某些球堆, 它们的总球数为 $2^k$, 那么剩下的球堆的总球数也是 $2^k$, 归纳过渡自然能实现, 遗憾的是这种有利情况并不必然发生.\n如何利用好归纳假设实现过渡呢? 上述解法中, 我们是先通过操作调整使所有球堆都含有偶数个球, 然后只需将 $2^{k+1}$ 个球两两捆绑看成一些新的\"球\", 这样 \"球\" 数就回到了 $2^k$ 个, 从而找到了 $P(k)$ 和 $P(k+1)$ 的过渡.\n特别注意本题中命题 $P(k)$ 所涉及的对象是球, 而归纳假设则是作用在新的载体一一捆绑以后的 $2^k$ 个\"球\"上.", + "figures": [] +} \ No newline at end of file diff --git a/processed_dataset/proof/0697.json b/processed_dataset/proof/0697.json new file mode 100644 index 0000000000000000000000000000000000000000..4ef9d9c2b1fb29ba27e8d301ebf47f0917f11708 --- /dev/null +++ b/processed_dataset/proof/0697.json @@ -0,0 +1,8 @@ +{ + "source_file": "./raw_volume-zh/volume14/chapter3.tex", + "problem_type": "proof", + "problem": "例3. 求证:第 $n$ 个素数 (将素数从小到大编上序号, 2 算作第一个素数) $p_n$ 小于 $2^{2^n}$.", + "solution": "解: $n=1$ 时, $p_1=2<2^{2^1}$, 结论成立.\n设当 $n \\leqslant k$ 时, 结论成立, 即 $p_i<2^{2^i}(i=1,2, \\cdots, k)$, 于是将这 $k$ 个不等式两边分别相乘, 得\n$$\np_1 p_2 \\cdots p_k<2^{2^1+2^2+\\cdots+2^k},\n$$\n所以 $p_1 p_2 \\cdots p_k+1 \\leqslant 2^{2^1+2^2+\\cdots+2^k}=2^{2^{k+1}-2}<2^{2^{k+1}}$.\n因为 $p_1, p_2, \\cdots, p_k$ 都不能整除 $p_i p_2 \\cdots p_k+1$, 所以 $p_1 p_2 \\cdots p_k+1$ 的素因数 $q$ 不可能是 $p_1, p_2, \\cdots, p_k$, 而只能大于或等于 $p_{k+1}$.\n于是, 我们有 $p_{k+1} \\leqslant q \\leqslant p_1 p_2 \\cdots p_k+1<2^{2^{k+1}}$.\n这就是说, 当 $n=k+1$ 时, 结论也成立.\n根据数学归纳原理, 对于任意正整数 $n$, 都有 $p_n<2^{2^n}$.", + "remark": "注:在运用数学归纳法证明本题的过程中, 我们将 \"假设 $n=k$ 时结论成立\" 改为更有力的\"假设 $n \\leqslant k$ 时结论成立\". 在这样的变通之下, 很容易对第 $k+1$ 个素数给出所需的上界估计.", + "figures": [] +} \ No newline at end of file diff --git a/processed_dataset/proof/0698.json b/processed_dataset/proof/0698.json new file mode 100644 index 0000000000000000000000000000000000000000..419fceacbb86db4ce7e760880f205ca85f233baf --- /dev/null +++ b/processed_dataset/proof/0698.json @@ -0,0 +1,8 @@ +{ + "source_file": "./raw_volume-zh/volume14/chapter3.tex", + "problem_type": "proof", + "problem": "例4. 设 $01$.", + "solution": "证法 1 我们用数学归纳法证明结论.\n当 $n=1,2$ 时, $x_1=1+a>1, x_2=\\frac{1}{1+a}+a=\\frac{1+a+a^2}{1+a}>1$, 命题成立.\n设 $n=k$ 时命题成立, 则 $n=k+2$ 时, 根据归纳假设和递推关系得\n$$\n0\\frac{1}{1+a}+a=\\frac{1+a+a^2}{1+a}>1,\n$$\n所以 $n=k+2$ 时命题成立.\n因此对任意 $n \\in \\mathbf{N}^*$, 有 $x_n>1$.", + "remark": "", + "figures": [] +} \ No newline at end of file diff --git a/processed_dataset/proof/0699.json b/processed_dataset/proof/0699.json new file mode 100644 index 0000000000000000000000000000000000000000..cce5e1dbbf2259d857601206a4b79555cfc0db3d --- /dev/null +++ b/processed_dataset/proof/0699.json @@ -0,0 +1,8 @@ +{ + "source_file": "./raw_volume-zh/volume14/chapter3.tex", + "problem_type": "proof", + "problem": "例4. 设 $01$.", + "solution": "证法 2 我们主动加强命题, 证明 $11$ 成立.", + "remark": "注:本题若直接用 $x_k>1$ 来证明 $x_{k+1}>1$ 是不可能的.\n上述两个证明展现了归纳法的两种典型的变通方式.\n第一个证明是设置跨度 2 来证明一般的 $n$ 成立, 注意此时我们要验证的起点也应变为两个.\n另有些问题虽可以用归纳法的基本形式给出证明,但若将设置适当跨度可使论证显著简化, 此时只要确保起点都能逐一验证即可.\n第二个证明是主动给 $x_k$ 设置一个上限, 根据递推关系, 下一项 $x_{k+1}$ 就自动得到一个下限.\n第一步的验证固然要增多一点工作量, 但实施归纳时, 从归纳假设中获得的信息更强,这点有利于结论的证明.\n应当注意的是, 在加强命题之后, 应对 $n=k+1$ 的情况也推出加强后的结论, 例如本题中不应当仅推出 $x_{k+1}>1$ 完事, 而应推出 $12\\left(\\frac{a_2}{2}+\\frac{a_3}{3}+\\cdots+\\frac{a_n}{n}\\right) .\n$$", + "solution": "证明:把命题加强为当 $n \\geqslant 2$ 时, 有\n$$\na_n^2>2\\left(\\frac{a_2}{2}+\\frac{a_3}{3}+\\cdots+\\frac{a_n}{n}\\right)+\\frac{1}{n} .\n$$\n当 $n=2$ 时, 左边 $=a_2^2=\\left(1+\\frac{1}{2}\\right)^2=\\frac{9}{4}$, 右边 $=a_2+\\frac{1}{2}=2$, 命题成立.\n假设命题在 $n=k$ 时成立, 即\n$$\na_k^2>2\\left(\\frac{a_2}{2}+\\frac{a_3}{3}+\\cdots+\\frac{a_k}{k}\\right)+\\frac{1}{k} .\n$$\n当 $n=k+1$ 时,有\n$$\n\\begin{aligned}\na_{k+1}^2 & =\\left(a_k+\\frac{1}{k+1}\\right)^2=a_k^2+\\frac{2 a_k}{k+1}+\\frac{1}{(k+1)^2} \\\\\n& >2\\left(\\frac{a_2}{2}+\\frac{a_3}{3}+\\cdots+\\frac{a_k}{k}\\right)+\\frac{1}{k}+\\frac{2}{k+1}\\left(a_{k+1}-\\frac{1}{k+1}\\right)+\\frac{1}{(k+1)^2} \\\\\n& =2\\left(\\frac{a_2}{2}+\\cdots+\\frac{a_k}{k}+\\frac{a_{k+1}}{k+1}\\right)+\\frac{1}{k}-\\frac{1}{(k+1)^2} \\\\\n& >2\\left(\\frac{a_2}{2}+\\cdots+\\frac{a_k}{k}+\\frac{a_{k+1}}{k+1}\\right)+\\frac{1}{k+1},\n\\end{aligned}\n$$\n即 $n=k+1$ 时命题也成立.\n所以, 由数学归纳法知, 加强的命题成立, 从而原命题成立.", + "remark": "", + "figures": [] +} \ No newline at end of file diff --git a/processed_dataset/proof/0701.json b/processed_dataset/proof/0701.json new file mode 100644 index 0000000000000000000000000000000000000000..edb08ca670f7eab26b31a5146191d6586f034a5e --- /dev/null +++ b/processed_dataset/proof/0701.json @@ -0,0 +1,8 @@ +{ + "source_file": "./raw_volume-zh/volume14/chapter3.tex", + "problem_type": "proof", + "problem": "例6. 有 2005 个青年围坐在一个大圆桌旁, 其中男孩不多于 668 人.\n如果一个女孩 $G$ 从两个方向之一数到任何一个人 (从自己的下一个人开始数起), 女孩数目总是大于男孩数目, 称 $G$ 是在一个 \"好位置\"上.\n证明: 无论怎样安排座位, 总存在一个女孩在一个好位置上.", + "solution": "证明:我们用数学归纳法证明一个更强的且具有一般形式的命题:\n若 $n$ 个男孩与 $2 n+1$ 个女孩围坐在一个大圆桌旁,则一定存在一个女孩, 从她开始逆时针数到任何一个人时,女孩总比男孩多.\n将圆桌旁的座位按顺时针编号为 1 至 $3 n+1$.\n当 $n=1$ 时,不妨设 $1 、 2 、 3$ 号为女孩, 4 号为男孩,则 3 号女孩满足命题.\n假设当 $n=k$ 时命题成立.\n当 $n=k+1$ 时, 由于女孩多于男孩, 必有两个女孩相邻.\n以这两个女孩为起点逆时针数到第一个男孩, 不妨设在 1 号位置, 则 $2 、 3$ 号都是女孩.\n当所有人坐好后, 撤掉 $1 、 2 、 3$ 号三人,由归纳假设知剩下的人中必有一个女孩,不妨记为 $A$,她处在 $m$ 号位置, 从她开始逆时针数,女孩总比男孩多.\n再让 $1 、 2 、 3$ 号三人坐回原处.\n此时, 根据 $A$ 的取法, $A$ 从 $m-1$ 号数到 4 号时知女孩一直比男孩多; 当数到 $3 、 2 、 1$ 号时, 由于先数到两个女孩, 因此仍是女孩比男孩多; 再数下去时,根据 $A$ 的取法,仍保持女孩比男孩多.\n所以 $n=k+1$ 时命题成立.\n故加强命题成立.\n在原问题中取 $n=668$ 即得证(若男孩人数少于 668 , 则表明即便将某些女孩当做男孩计人,仍能满足命题,因此结论必定成立).", + "remark": "注:求解本题时, 先观察出已知条件中的数字 2005 与 668 实际上是 $3 n+1$ 和 $n$ 的内在关系, 不妨将问题一般化, 便于抓住本质予以证明.\n在试图归纳证明时, 又发现, 当考虑 $n=k+1$ 的情形时, 若撤掉相邻 3 人后只是利用 $n=k$ 时\"存在一个女孩, 她可以从两个方向之一数到任何一个人,女孩总比男孩多\"的归纳假设,那么归纳过渡会发生困难.\n因此, 为了获得更强的归纳假设, 我们大胆加强命题,证明\"存在一个女孩, 当她逆时针数时,女孩总比男孩多\",并且这种性质具有继承性, 从而问题解决.\n本题可谓加强命题并辅以结论一般化的典型.\n有时候, 我们把一个命题的结论加强或者一般化, 表面上看起来命题由于结论加强了, 问题更加难于处理, 但是, 恰恰相反, 解起来反而比特殊的、具体的情况轻松容易.\n这在用数学归纳法时尤其常见.\n苏联数学家辛钦曾说过: \"在数学归纳法的证明中, 假设命题当 $n-1$ 时成立, 再来证明它当 $n$ 时也成立, 因此, 命题越强, 在 $n-1$ 的情况下所给的条件也越多, 而对数 $n$, 要证明的东西也越多, 但是在许多问题中, 条件较多显得更为重要.", + "figures": [] +} \ No newline at end of file diff --git a/processed_dataset/proof/0702.json b/processed_dataset/proof/0702.json new file mode 100644 index 0000000000000000000000000000000000000000..0662f82aaf5014d88c937bd5c434d055c688d31d --- /dev/null +++ b/processed_dataset/proof/0702.json @@ -0,0 +1,8 @@ +{ + "source_file": "./raw_volume-zh/volume14/chapter3.tex", + "problem_type": "proof", + "problem": "例7. 证明: 对一切正整数 $n$, 不定方程 $x^2+y^2=z^n$ 都有正整数解.", + "solution": "证明:当 $n=1$ 时,取 $x=y=1, z=2$; 当 $n=2$ 时,取 $x=3, y=4$, $z=5$, 即可使它们满足方程, 故知命题在 $n=1$ 和 2 时成立.\n假定当 $n=k$ 时, $x=x_0, y=y_0, z=z_0$ 是一组正整数解;那么当 $n= k+2$ 时, 只要取 $x=x_0 z_0, y=y_0 z_0, z=z_0$, 就有\n$$\n\\left(x_0 z_0\\right)^2+\\left(y_0 z_0\\right)^2=z_0^2\\left(x_0^2+y_0^2\\right)=z_0^{k+2},\n$$\n知它们恰为方程的一组正整数解.\n所以当 $n=k+2$ 时,命题也成立.\n由于我们采用了两个起点, 所以可以用跨度 2 跳跃.\n这表明对一切正整数 $n$, 不定方程都有正整数解,证毕.", + "remark": "", + "figures": [] +} \ No newline at end of file diff --git a/processed_dataset/proof/0703.json b/processed_dataset/proof/0703.json new file mode 100644 index 0000000000000000000000000000000000000000..b0ef1476a0a655b7ee7e56865b8a069132f1cff0 --- /dev/null +++ b/processed_dataset/proof/0703.json @@ -0,0 +1,8 @@ +{ + "source_file": "./raw_volume-zh/volume14/chapter3.tex", + "problem_type": "proof", + "problem": "例8. 对怎样的正整数 $n$, 集合 $\\{1,2, \\cdots, n\\}$ 可以分成 5 个互不相交的子集, 每个子集的元素和相等.", + "solution": "解:先找一个必要条件:如果 $\\{1,2, \\cdots, n\\}$ 能分成 5 个互不相交的子集, 各个子集的元素和相等,那么\n$$\n1+2+\\cdots+n=\\frac{1}{2} n(n+1)\n$$\n能被 5 整除.\n所以 $n=5 k$ 或 $n=5 k-1$,其中 $k \\in \\mathbf{N}^*$.\n显然, $k=1$ 时, 上述条件不是充分的.\n下用数学归纳法证明 $k \\geqslant 2$ 时, 条件是充分的.\n当 $k=2$, 即 $n=9,10$ 时, 我们把集合 $\\{1,2, \\cdots, 9\\}$ 和 $\\{1,2, \\cdots, 10\\}$ 作如下分拆:\n$$\n\\begin{aligned}\n& \\{1,8\\},\\{2,7\\},\\{3,6\\},\\{4,5\\},\\{9\\} ; \\\\\n& \\{1,10\\},\\{2,9\\},\\{3,8\\},\\{4,7\\},\\{5,6\\} ;\n\\end{aligned}\n$$\n当 $k=3$ 时, 即 $n=14,15$ 时,有\n$$\n\\begin{aligned}\n& \\{1,2,3,4,5,6\\},\\{7,14\\},\\{8,13\\},\\{9,12\\},\\{10,11\\} ; \\\\\n& \\{1,2,3,5,6,7\\},\\{4,8,12\\},\\{9,15\\},\\{10,14\\},\\{11,13\\} .\n\\end{aligned}\n$$\n因为若集合 $\\{1,2, \\cdots, n\\}$ 能分成 5 个互不相交的子集, 并且它们的元素和相等, 那么 $\\{1,2, \\cdots, n, n+1, \\cdots, n+10\\}$ 也能分成 5 个元素和相等但互不相交的子集.\n事实上, 如果\n$$\n\\{1,2, \\cdots, n\\}=A_1 \\cup A_2 \\cup A_3 \\cup A_4 \\cup A_5,\n$$\n令 $B_1=A_1 \\cup\\{n+1, n+10\\}, B_2=A_2 \\bigcup\\{n+2, n+9\\}, B_3=A_3 \\bigcup\\{n+ 3, n+8\\}, B_4=A_4 \\cup\\{n+4, n+7\\}, B_5=A_5 \\bigcup\\{n+5, n+6\\}$, 那么\n$\\{1,2, \\cdots, n, n+1, \\cdots, n+10\\}=B_1 \\cup B_2 \\cup B_3 \\cup B_4 \\cup B_5$,\n并且 $B_i \\cap B_j=\\varnothing, 1 \\leqslant i$ j) 属于同一个 $B_k$, 从而 $0)). 由于\n$$\n\\left(a_1+a_2+a_3\\right)+\\left(a_4+a_5+a_6\\right)+\\left(a_7+a_8+a_9\\right)=2+3+\\cdots+10=54,\n$$\n所以 $a_1+a_2+a_3, a_4+a_5+a_6, a_7+a_8+a_9$ 这三个数中一定有一个数不小于 $\\frac{54}{3}=18$.", + "remark": "", + "figures": [ + "./images/volume14/figures/fig-c4i1.png" + ] +} \ No newline at end of file diff --git a/processed_dataset/proof/0712.json b/processed_dataset/proof/0712.json new file mode 100644 index 0000000000000000000000000000000000000000..669e8184bf4a0daef14e51ec6a064dffd8288b0c --- /dev/null +++ b/processed_dataset/proof/0712.json @@ -0,0 +1,10 @@ +{ + "source_file": "./raw_volume-zh/volume14/chapter4.tex", + "problem_type": "proof", + "problem": "例2-2. 把 $1,2, \\cdots, 10$ 按任意次序排成一个圆圈.\n证明:一定可以找到三个相邻的数,它们的和不大于 15 .", + "solution": "证明:设这 10 个数在圆周上排列为 $10, b_1, b_2 \\cdots, b_9$ (如图()). 由于\n$$\n\\left(b_1+b_2+b_3\\right)+\\left(b_4+b_5+b_6\\right)+\\left(b_7+b_8+b_9\\right)=1+2+\\cdots+9=45 \\text {, }\n$$\n所以, $b_1+b_2+b_3, b_4+b_5+b_6, b_7+b_8+b_9$ 这三个数中一定有一个数不大于 $\\frac{45}{3}=15$.", + "remark": "注:本例运用的是与抽屉原理本质相同的\"平均数原理\".", + "figures": [ + "./images/volume14/figures/fig-c4i2.png" + ] +} \ No newline at end of file diff --git a/processed_dataset/proof/0713.json b/processed_dataset/proof/0713.json new file mode 100644 index 0000000000000000000000000000000000000000..941c5e30bdc870d07983990a9f883ceab9ed45f9 --- /dev/null +++ b/processed_dataset/proof/0713.json @@ -0,0 +1,8 @@ +{ + "source_file": "./raw_volume-zh/volume14/chapter4.tex", + "problem_type": "proof", + "problem": "例3. 设 $m$ 为给定正整数.\n证明: 若集合 $A_1, A_2, \\cdots, A_m$ 两两交集为空, 且满足 $\\bigcup_{i=1}^m A_i=\\mathbf{N}^*$, 则必存在一个集合 $A_i(1 \\leqslant i \\leqslant m)$, 它含有任意正整数的倍数.", + "solution": "证明:我们考虑无穷数列 $\\{n !\\}$, 其中 $n ! \\in \\mathbf{N}^*$, 即 $n ! \\in \\bigcup_{i=1}^m A_i$. 根据抽屉原理, 在有限个集合 $A_1, A_2, \\cdots, A_m$ 中, 必存在一个集合, 不妨设为 $A_1$, 它含有上述数列中的无穷项.\n此时, 对任意正整数 $N$, 必存在一个正整数 $N_1 \\geqslant N$, 使 $N_{1} ! \\in A_1$, 因而集合 $A_1$ 含有 $N$ 的倍数 $N_1$ !. 由 $N$ 的任意性知结论成立.", + "remark": "注:本例所使用的是着眼于无限形式的抽屉原理 3. 注意题目中的条件 \"两两交集为空\"可以去掉而不影响结果.", + "figures": [] +} \ No newline at end of file diff --git a/processed_dataset/proof/0714.json b/processed_dataset/proof/0714.json new file mode 100644 index 0000000000000000000000000000000000000000..33573d46e17067a66bc91159388167a2480cc8a5 --- /dev/null +++ b/processed_dataset/proof/0714.json @@ -0,0 +1,8 @@ +{ + "source_file": "./raw_volume-zh/volume14/chapter4.tex", + "problem_type": "proof", + "problem": "例4. 从 $1,2,3, \\cdots, 16$ 这 16 个数中, 最多能选出多少个数, 使得被选出的数中, 任意三个数都不是两两互素的.", + "solution": "解:首先, 取出 $1,2, \\cdots, 16$ 中所有 2 或 3 的倍数, 有\n$$\n2,3,4,6,8,9,10,12,14,15,16 .\n$$\n这 11 个数要么是 2 的倍数, 要么是 3 的倍数.\n由抽屉原理知, 这 11 个数中的任意三个数,都必有两个数同为 2 或 3 的倍数, 它们的最大公约数大于 1 , 也就是说这三个数是不两两互素的.\n所以, 从 $1,2, \\cdots, 16$ 中可以选出 11\n个数满足要求.\n下面证明从 $1,2, \\cdots, 16$ 中任取 12 (或 12 以上) 个数, 其中一定有 3 个数两两互素.\n事实上, 令数组 $A=\\{1,2,3,5,7,11,13\\}$. 数组 $A$ 中有 7 个数, 而且这 7 个数是两两互素的.\n从 $1,2, \\cdots, 16$ 中任取 12 个数, 由于 $A$ 以外只有 9 个数, 故 $A$ 中至少有 3 个数被选出, 这三个数是两两互素的.\n综上可知,最多可以选出 11 个满足题设的数.", + "remark": "", + "figures": [] +} \ No newline at end of file diff --git a/processed_dataset/proof/0715.json b/processed_dataset/proof/0715.json new file mode 100644 index 0000000000000000000000000000000000000000..a56cf82a3f30256a66f6e266867b9812579cbe5f --- /dev/null +++ b/processed_dataset/proof/0715.json @@ -0,0 +1,8 @@ +{ + "source_file": "./raw_volume-zh/volume14/chapter4.tex", + "problem_type": "proof", + "problem": "例5. 设 $T$ 是由 $60^{100}$ 的所有正因数组成的集合.\n$S$ 是 $T$ 的一个子集, 其中没有一个数是另一个数的倍数.\n求 $S$ 的元素个数的最大值.", + "solution": "解:约定下文中的 $a, b, c$ 都是非负整数.\n因为 $60=2^2 \\cdot 3 \\cdot 5$, 所以\n$$\n\\begin{gathered}\nT=\\left\\{2^a 3^b 5^c \\mid 0 \\leqslant a \\leqslant 200,0 \\leqslant b, c \\leqslant 100\\right\\} . \\\\\nS=\\left\\{2^{200-b-c} 3^b 5^c \\mid 0 \\leqslant b, c \\leqslant 100\\right\\},\n\\end{gathered}\n$$\n对任意 $0 \\leqslant b, c \\leqslant 100$, 有 $0 \\leqslant 200-b-c \\leqslant 200$, 所以 $S$ 是 $T$ 的一个子集且含 $101^2$ 个元素.\n下面我们证明 $S$ 中没有一个数是另一个数的倍数, 并且元素个数超过 $101^2$ 的子集都不满足这个条件.\n假设 $2^{200-b-c} 3^b 5^c$ 是 $2^{200-i-j} 3^i 5^j$ 的倍数, 且 $(b, c) \\neq(i, j)$, 则\n$$\n200-b-c \\geqslant 200-i-j, b \\geqslant i, c \\geqslant j,\n$$\n第一个不等式表明 $b+c \\leqslant i+j$, 与后两个不等式联立得 $b=i, c=j$. 矛盾.\n所以 $S$ 中没有一个元素是另一个的倍数.\n设 $U$ 是 $T$ 的一个超过 $101^2$ 个元素的子集.\n因为只有 $101^2$ 对互异的 $(b$, $c$ ), 由抽屉原理, $U$ 中必有两个元素 $u_1=2^{a_1} 3^{b_1} 5^{c_1}, u_2=2^{a_2} 3^{b_2} 5^{c_2}$, 其中 $b_1= b_2, c_1=c_2$, 而 $a_1 \\neq a_2$. 若 $a_1>a_2$, 则 $u_1$ 是 $u_2$ 的倍数; 若 $a_1a_{k_2}>\\cdots>a_{k_{n+1}} . \\label{eq3}\n$$\n否则, 设 $k_i\\frac{1}{2} \\cdot(2 n) !,\n$$\n即 $A$ 中元素数超过总排列数的一半.\n故具有性质 $P$ 的排列比不具有性质 $P$ 的排列的个数多.", + "remark": "注:在找到集合 $A_k$ 后, 根据 \\ref{eq1} 式可以对 $|A|$ 作出下界估计.\n如果完整地应用容斥原理的等式,我们则可进一步将 $|A|$ 表示成关于 $n$ 的求和式\n$$\n|A|=\\sum_{k=1}^n(-1)^{k-1} 2^k C_n^k(2 n-k) ! .\n$$", + "figures": [] +} \ No newline at end of file diff --git a/processed_dataset/proof/0723.json b/processed_dataset/proof/0723.json new file mode 100644 index 0000000000000000000000000000000000000000..68f785d8a762099d280203d6bf19fb11c4fcb9e7 --- /dev/null +++ b/processed_dataset/proof/0723.json @@ -0,0 +1,8 @@ +{ + "source_file": "./raw_volume-zh/volume14/chapter5.tex", + "problem_type": "proof", + "problem": "例7. 证明: 对 $n$ 个有限集 $A_1, A_2, \\cdots, A_n$, 有\n$$\n\\begin{gathered}\n\\left|A_1 \\cap A_2 \\cap \\cdots \\cap A_n\\right|=\\sum_{i=1}^n\\left|A_i\\right|-\\sum_{1 \\leqslant i\\frac{n-2}{n-1}\\left|A_{i+1}\\right|, i=1,2, \\cdots, n\\left(A_{n+1}=A_1\\right) .\n$$\n证明: $\\bigcap_{i=1}^n A_i \\neq \\varnothing$.", + "solution": "证明:对 $k=2,3, \\cdots, n$, 令 $B_k=\\bigcap_{i=1}^k A_i$. 下面证明: 当 $k=2,3, \\cdots, n$ 时, 有\n$$\n\\left|B_k\\right|>\\frac{n-2}{n-1}\\left|A_k\\right|-\\frac{1}{n-1} \\sum_{i=2}^{k-1}\\left|A_i\\right| \\label{eq1}\n$$\n用数学归纳法.\n当 $k=2$ 时, $\\left|B_2\\right|=\\left|A_1 \\cap A_2\\right|>\\frac{n-2}{n-1}\\left|A_2\\right|$, (1)成立.\n设 式\\ref{eq1} 已对 $k$ 成立, 并且 $2 \\leqslant k\\frac{n-2}{n-1}\\left|A_{k+1}\\right|$, 并且 $B_k \\cap\\left(A_k \\cap A_{k+1}\\right)=B_{k+1}$.\n综合以上各方面可得\n$$\n\\left|A_k\\right|>\\frac{n-2}{n-1}\\left|A_k\\right|-\\frac{1}{n-1} \\sum_{i=2}^{k-1}\\left|A_i\\right|+\\frac{n-2}{n-1}\\left|A_{k+1}\\right|-\\left|B_{k+1}\\right|,\n$$\n整理即得 式\\ref{eq1} 对 $k+1$ 成立.\n由数学归纳法可知, 特别地, 式\\ref{eq1} 对 $n$ 成立, 即\n$$\n\\left|B_n\\right|>\\frac{n-2}{n-1}\\left|A_n\\right|-\\frac{1}{n-1} \\sum_{i=2}^{n-1}\\left|A_i\\right| .\n$$\n不失一般性, 不妨一开始就设 $\\left|A_n\\right|=\\max _{1 \\leqslant i \\leqslant n}\\left|A_i\\right|$, 则由上式得 $\\left|B_n\\right|>0$, 从而\n$$\n\\bigcap_{i=1}^n A_i=B_n \\neq \\varnothing .\n$$", + "remark": "注:本例反复运用容斥原理在 $n=2$ 的情形, 配合数学归纳法证明(1)式.\n考虑到已知条件具有轮换结构, 故可得到 $n$ 个形如(1)式的不等关系.\n因此, 无论利用平均数原理还是上述证明中的极端原理,均可证得结论.", + "figures": [] +} \ No newline at end of file diff --git a/processed_dataset/proof/0725.json b/processed_dataset/proof/0725.json new file mode 100644 index 0000000000000000000000000000000000000000..2d1e5952d06e56f3c24e65f3beae46054dcc9bda --- /dev/null +++ b/processed_dataset/proof/0725.json @@ -0,0 +1,8 @@ +{ + "source_file": "./raw_volume-zh/volume14/chapter6.tex", + "problem_type": "proof", + "problem": "例1. 已知 $2 n+1\\left(n \\in \\mathbf{N}^*\\right)$ 个星球两两之间距离不相等.\n每个星球上的人都观察离该星球最近的一个星球.\n求证: 必存在一个星球未被观察.", + "solution": "证明:我们用数学归纳法证明结论.\n当 $n=1$ 时,显然距离最近的一对星球相互观察, 另一个星球未被观察, 命题成立.\n对 $n(n \\geqslant 2)$ 的情况, 因为 $2 n+1$ 个星球两两之间距离是有限个数且两两不等, 根据事实 (1), 必有唯一的一对星球距离最近, 它们必相互观察.\n现在去掉这两个星球, 根据归纳假设, 必有一个星球没被观察到, 因此对一般的 $n$ 命题也成立.", + "remark": "注:在用数学归纳法证明中, 需将 $n$ 的情况转化为小于 $n$ 的情况.\n不难发现只要两个星球距离很近, 以至于它们不去观察别的星球, 那么就能将这两个星球去掉.\n根据事实 (1), 有限个距离中必有最小值, 而距离的最小性足以保证上述目的可以达到.\n一般地, 在对元素个数 $n$ 施行数学归纳法证明时, 可以考虑具有某种极端性质的元素, 试图将其去掉, 从而将 $n$ 的情况转化为小于 $n$ 的情况.", + "figures": [] +} \ No newline at end of file diff --git a/processed_dataset/proof/0726.json b/processed_dataset/proof/0726.json new file mode 100644 index 0000000000000000000000000000000000000000..d8a64514c263b8bed945c92d98775aa40b645e07 --- /dev/null +++ b/processed_dataset/proof/0726.json @@ -0,0 +1,8 @@ +{ + "source_file": "./raw_volume-zh/volume14/chapter6.tex", + "problem_type": "proof", + "problem": "例2. 平面上有 $n(n \\geqslant 5)$ 个点.\n将它们用红、蓝两色染色.\n设任何 3 个同色点不共线.\n求证: 存在一个三角形使得\n(1)它的三个顶点染有相同颜色;\n(2)这三角形至少有一条边上不包含另一种颜色的点.", + "solution": "证明:于点数 $n \\geqslant 5$, 且所有的点只染两种颜色, 所以, 至少有三点同色.\n因此,存在三个顶点同色的三角形.\n我们在这些顶点同色的三角形中取一个面积最小的三角形.\n如果这个三角形的每一条边上都有一个另一种颜色的点, 那么我们就找到了另一个三个顶点同色的三角形, 而且, 这个三角形具有更小的面积, 这是不可能的.\n因此题设的三角形一定存在.", + "remark": "注:本题证明中使用了极端原理.\n我们考虑的是面积, 是一种很好的想法,值得我们仔细体会.", + "figures": [] +} \ No newline at end of file diff --git a/processed_dataset/proof/0727.json b/processed_dataset/proof/0727.json new file mode 100644 index 0000000000000000000000000000000000000000..c268929b22baa578a4e43ee3389f138a93175143 --- /dev/null +++ b/processed_dataset/proof/0727.json @@ -0,0 +1,8 @@ +{ + "source_file": "./raw_volume-zh/volume14/chapter6.tex", + "problem_type": "proof", + "problem": "例3. 证明不定方程 $x^3+2 y^3=4 z^3$ 没有正整数解 $(x, y, z)$.", + "solution": "证明:假设方程有正整数解 $\\left(x_1, y_1, z_1\\right)$. 由于\n$$\nx_1^3+2 y_1^3=4 z_1^3,\n$$\n所以 $x_1^3$ 是偶数,故 $x_1$ 是偶数.\n设 $x_1=2 x_2$, 则\n$$\n\\begin{gathered}\n8 x_2^3+2 y_1^3=4 z_1^3, \\\\\n4 x_2^3+y_1^3=2 z_1^3,\n\\end{gathered}\n$$\n即故 $y_1$ 是偶数.\n设 $y_1=2 y_2$, 则\n$$\n\\begin{aligned}\n& 4 x_2^3+8 y_1^3=2 z_1^3, \\\\\n& 2 x_2^3+4 y_1^3=z_1^3,\n\\end{aligned}\n$$\n故 $z_1$ 是偶数.\n设 $z_1=2 z_2$, 则\n$$\n2 x_2^3+4 y_1^3=8 z_2^3 .\n$$\n所以 $\\left(x_2, y_2, z_2\\right)$ 也是方程的一组正整数解, 且 $x_2A B, A D+D B>A B,\n$$\n两式相加得\n$$\nA C+C B+A D+D B>2 A B,\n$$\n重新组合得\n$$\n(A C+A D)+(B C+B D)>2 A B,\n$$\n所以 $A C+A D>A B$ 与 $B C+B D>A B$ 中必有一项成立, 不妨设 $A C+ A D>A B$, 由于已设 $A B$ 是最长的棱, 故从顶点 $A$ 引出的三条棱可构成三角形的三边.", + "remark": "注:一般而言, 若要考虑从某个顶点 (比如点 $A$ ) 所引出的三条棱可否构成三角形的三边, 需考察三个条件, 即 $A B, A C, A D$ 是否满足任意两数之和大于第三者.\n然而, 一旦已知 $A B$ 是最长的棱, 就只需要集中精力考察 $A C+ A D>A B$ 这一个条件了.\n本题中, 极端性假设所带来的效果正是使目标更为集中,最终证明了满足题意的顶点必能在 $A$ 与 $B$ 中选出.", + "figures": [] +} \ No newline at end of file diff --git a/processed_dataset/proof/0730.json b/processed_dataset/proof/0730.json new file mode 100644 index 0000000000000000000000000000000000000000..35ac65bd854f6204e88f6d06877d6e5849f35337 --- /dev/null +++ b/processed_dataset/proof/0730.json @@ -0,0 +1,8 @@ +{ + "source_file": "./raw_volume-zh/volume14/chapter6.tex", + "problem_type": "proof", + "problem": "例6. 设集合 $A_1, A_2, \\cdots, A_n$ 与集合 $B_1, B_2, \\cdots, B_n$ 是集合 $M$ 的两个分划, 且满足对任意两个交集为空集的集合 $A_i, B_j(1 \\leqslant i \\leqslant n, 1 \\leqslant j \\leqslant n)$, 都有 $\\left|A_i \\cup B_j\\right| \\geqslant n$ (其中 $|X|$ 表示集合 $X$ 的元素个数). 求证: $|M| \\geqslant \\frac{n^2}{2}$.", + "solution": "证明:不妨设 $A_i, B_j(1 \\leqslant i \\leqslant n, 1 \\leqslant j \\leqslant n)$ 中元素个数最少的为 $A_1$, $\\left|A_1\\right|=p$.\n若 $p \\geqslant \\frac{n}{2}$, 则\n$$\n|M|=\\left|A_1\\right|+\\left|A_2\\right|+\\cdots+\\left|A_n\\right| \\geqslant n p \\geqslant \\frac{n^2}{2} .\n$$\n若 $p<\\frac{n}{2}$, 则将 $B_1, B_2, \\cdots, B_n$ 分为两类: 第一类是与 $A_1$ 的交非空的, 不妨设为 $B_1, B_2, \\cdots, B_q$, 它们中每个集合的元素个数至少为 $p$; 第二类是与 $A_1$ 的交集为空集的, 为 $B_{q+1}, B_{q+2}, \\cdots, B_n$, 它们中每个集合与 $A_1$ 的元素个数之和不小于 $n$, 故每个集合所含元素至少为 $n-p$ 个.\n因此\n$$\n\\begin{aligned}\n|M| & =\\left|B_1\\right|+\\left|B_2\\right|+\\cdots+\\left|B_n\\right| \\geqslant p q+(n-p)(n-q) \\\\\n& =\\frac{n^2}{2}+\\frac{1}{2}(n-2 p)(n-2 q) .\n\\end{aligned}\n$$\n由于 $\\left|A_1\\right|=p$, 且 $B_1, B_2, \\cdots, B_q$ 两两交集为空集, 故 $q \\leqslant p<\\frac{n}{2}$, 因此\n$$\n(n-2 p)(n-2 q)>0,\n$$\n故 $|M|>\\frac{n^2}{2}$.\n综上可知 $|M| \\geqslant \\frac{n^2}{2}$.", + "remark": "注一本题中所考虑的极端情形是 \"所有 $A_i, B_j$ 中元素个数最少的集合\". 由于本题所要估计的是 $|M|$ 的下界, 而 \" $A_1$ 元素最少\" 实则导致如下几点 \"益处\":\n(1) 与 $A_1$ 不相交的集合 $B_j$ 较多;\n(2) 每个与 $A_1$ 不相交的集合 $B_j$ 满足 $\\left|A_i \\cup B_j\\right| \\geqslant n$, 势必含有较多的元素;\n(3) 与 $A_1$ 相交的那些集合 $B_j$ 不会很多;\n(4) 每个与 $A_1$ 相交的集合 $B_j$ 所含元素个数不至于太小 (不能小于 $\\left|A_1\\right|$ ). 因此,始终围绕元素最少的集合 $A_1$ 进行讨论便在情理之中了.\n解题中应充分挖掘极端元素的价值, 而在寻找极端元素时也应注意角度的选取.\n注二:某题如下:\n设\n$$\n\\left(\\begin{array}{cccc}\na_{11} & a_{12} & \\cdots & a_{1 n} \\\\\na_{21} & a_{22} & \\cdots & a_{2 n} \\\\\n\\cdots & \\cdots & \\cdots & \\cdots \\\\\na_{n 1} & a_{n 2} & \\cdots & a_{n n}\n\\end{array}\\right)\n$$\n是一个由非负整数组成的方阵.\n已知如果 $a_{i j}=0$, 那么\n$$\na_{i 1}+a_{i 2}+\\cdots+a_{i n}+a_{1 j}+a_{2 j}+\\cdots+a_{n j} \\geqslant n,\n$$\n证明 : 方阵中的所有元素之和不小于 $\\frac{n^2}{2}$.\n这是本例的另一种表达形式.", + "figures": [] +} \ No newline at end of file diff --git a/processed_dataset/proof/0731.json b/processed_dataset/proof/0731.json new file mode 100644 index 0000000000000000000000000000000000000000..bb075beab7cf4755b873959a5739df9661035265 --- /dev/null +++ b/processed_dataset/proof/0731.json @@ -0,0 +1,10 @@ +{ + "source_file": "./raw_volume-zh/volume14/chapter6.tex", + "problem_type": "proof", + "problem": "例7. 设有 $n(\\geqslant 7)$ 个圆, 其中任意 3 个圆都不两两相交 (包括相切), 求证:一定可以找到一个圆, 它至多只能与 5 个圆相交.", + "solution": "证明:如图(),设 $n$ 个圆的圆心分别为 $O_1, O_2, \\cdots, O_n$, 取 $n$ 个圆中半径最小的圆, 设为 $\\odot O_1$.\n若 $\\odot O_1$ 与 6 个 (或多于 6 个圆) 圆 $O_2, O_3, \\cdots, O_7$ 相交, 连接 $O_1 O_2$,\n$O_1 O_3, \\cdots, O_1 O_7$, 则 $\\angle O_2 O_1 O_3, \\angle O_3 O_1 O_4$, $\\angle O_4 O_1 O_5, \\angle O_5 O_1 O_6, \\angle O_6 O_1 O_7, \\angle O_7 O_1 O_2$ 中必有一个角不超过 $\\frac{360^{\\circ}}{6}=60^{\\circ}$, 不妨设 $\\angle O_2 O_1 O_3 \\leqslant 60^{\\circ}$.\n连接 $\\mathrm{O}_2 \\mathrm{O}_3$, 令 $\\odot O_1, \\odot O_2, \\odot O_3$ 的半径是 $r_1$, $r_2, r_3$, 则 $r_1 \\leqslant r_2, r_1 \\leqslant r_3$.\n由 $\\odot O_1$ 与 $\\odot O_2$ 相交, $\\odot O_1$ 与 $\\odot O_3$ 相交, 得 $r_1+ r_2 \\geqslant O_1 O_2, r_1+r_3 \\geqslant O_1 O_3$, 则\n$$\n\\begin{aligned}\n& O_1 O_2 \\leqslant r_1+r_2 \\leqslant r_2+r_3, \\\\\n& O_1 O_3 \\leqslant r_1+r_3 \\leqslant r_2+r_3 .\n\\end{aligned}\n$$\n而 $\\angle O_2 O_1 O_3 \\leqslant 60^{\\circ}$, 那么其余两个角中必有一个角 $\\geqslant 60^{\\circ}$, 令 $\\angle O_3 \\geqslant 60^{\\circ}$, 就有 $O_1 O_2 \\geqslant O_2 O_3$, 所以有\n$$\n\\mathrm{O}_2 \\mathrm{O}_3 \\leqslant \\dot{r}_1+r_2 \\leqslant r_2+r_3,\n$$\n从而知 $\\odot \\mathrm{O}_2$ 与 $\\odot \\mathrm{O}_3$ 必相交, 由此推得 $\\odot O_1, \\odot \\mathrm{O}_2, \\odot \\mathrm{O}_3$ 两两相交, 这与题中的条件是矛盾的.\n所以一定存在一个圆至多只能与 5 个圆相交.", + "remark": "", + "figures": [ + "./images/volume14/figures/fig-c6i1.png" + ] +} \ No newline at end of file diff --git a/processed_dataset/proof/0732.json b/processed_dataset/proof/0732.json new file mode 100644 index 0000000000000000000000000000000000000000..0e91b0b7aceef09c83848acd88eb0bb21016510c --- /dev/null +++ b/processed_dataset/proof/0732.json @@ -0,0 +1,10 @@ +{ + "source_file": "./raw_volume-zh/volume14/chapter6.tex", + "problem_type": "proof", + "problem": "例9. 给定平面上不全在一条直线上的 $n$ 个点, 则必有一条直线恰好通过这 $n$ 个点中的两个点.", + "solution": "证明:因为平面的 $n$ 个点不全在同一条直线上, 于是这 $n$ 个点中任意两点所确定的直线外, 都有这 $n$ 点中的其他点.\n对于每条直线, 求出不在这条直线上的点到这条直线的距离.\n这些距离的个数可能很多, 但总是有限多个.\n因此, 根据事实 (1), 其中必有一个最小的, 不妨设为 $d_0$.\n不妨设点 $P$ 到点 $A$ 和点 $B$ 所确定的直线的距离是最小距离 $d_0$. 我们证明直线 $A B$ 是通过 $n$ 个点中的恰好两个点的直线.\n下面我们用反证法.\n设 $P H=d_0$. 如果在直线 $A B$ 上还有 $n$ 点中的点 $C$, 则 $A 、 B 、 C$ 至少有两个点落在垂足 $H$ 的同侧, 不妨设这两个点是 $B$ 和 $C$, 它们在直线 $A B$ 上点 $H$ 的同侧, 如图(). 作直线 $P B$, 则直线 $P B$ 也是通过 $n$ 点组中两个点的一条直线.\n我们过点 $C$ 作直线 $P B$ 的垂线 $C D$, 垂足为 $D$, 则有\n$$\nd_0 \\geqslant H E \\geqslant C D \\text {, }\n$$\n这与 $P H$ 的最小性矛盾, 所以直线 $A B$ 上不能有 $n$ 点组中的其他点.\n直线 $A B$ 就是我们要找的只通过两点的直线.", + "remark": "注:这个问题是英国数学家西尔维斯特提出的,故被称为西尔维斯特问题.\n该问题看似简单, 但西尔维斯特生前却没能解决它, 尔后不少数学家也曾试图给以证明,也没能成功.\n这种状况持续了 50 年之久.\n本题证明的关键在于考察极端.\n通常在解题过程中, 一些极端性质是在操作调整的探索过程中发现并予以利用的, 例如西尔维斯特问题中, 一旦发现距离可以产生更小的距离, 问题就解决了.\n另有一个与西尔维斯特问题对偶的命题: 在平面上给定 $n$ 条两两不平行的直线, 若对于他们中任何两条直线的交点, 都有这 $n$ 条直线中的另一条过这个点,则这 $n$ 条直线共点.\n证明过程相仿, 留给读者练习.", + "figures": [ + "./images/volume14/figures/fig-c6i2.png" + ] +} \ No newline at end of file diff --git a/processed_dataset/proof/0733.json b/processed_dataset/proof/0733.json new file mode 100644 index 0000000000000000000000000000000000000000..5c6630594c4b712394498ce8427ce0d3dd286b7c --- /dev/null +++ b/processed_dataset/proof/0733.json @@ -0,0 +1,8 @@ +{ + "source_file": "./raw_volume-zh/volume14/chapter7.tex", + "problem_type": "proof", + "problem": "例1. 设 $a_1, a_2, \\cdots, a_n \\in \\mathbf{Z}$, 其中 $n$ 为正奇数, $\\left(b_1, b_2, \\cdots, b_n\\right)$ 是 $\\left(a_1\\right.$, $\\left.a_2, \\cdots, a_n\\right)$ 的任-一排列.\n证明: $\\prod_{i=1}^n\\left(a_i+\\left|b_i\\right|\\right)$ 与 $\\prod_{i=1}^n\\left(a_i-\\left|b_i\\right|\\right)$ 均为偶数.", + "solution": "证明:由于对整数 $x$, 总有 $|x| \\equiv x \\equiv-x(\\bmod 2)$, 故由已知得\n$$\n\\sum_{i=1}^n\\left(a_i+\\left|b_i\\right|\\right) \\equiv \\sum_{i=1}^n\\left(a_i-b_i\\right) \\equiv \\sum_{i=1}^n a_i-\\sum_{i=1}^n b_i \\equiv \\equiv(\\bmod 2),\n$$\n又上式左边是奇数项 ( $n$ 项) 求和, 因此必有某个 $a_i+\\left|b_i\\right|(1 \\leqslant i \\leqslant n)$ 为偶数,从而 $\\prod_{i=1}^n\\left(a_i+\\left|b_i\\right|\\right)$ 为偶数.\n同理可证得 $\\prod_{i=1}^n\\left(a_i-\\left|b_i\\right|\\right)$ 为偶数.\n证明: $\\prod_{i=1}^n\\left(a_i+\\left|b_i\\right|\\right)$ 与 $\\prod_{i=1}^n\\left(a_i-\\left|b_i\\right|\\right)$ 均为偶数.", + "remark": "注:本题的各种特殊形式在竞赛题中并不少见.\n纵观整个解题过程似乎平淡无奇, 却已用到了前述多条奇偶性质.\n这些性质虽然简单明显, 却常常是我们平时解题的出发点.\n尤其突出的一点是, 当把奇偶性看成模 2 的一种分类时, 整数取绝对值相当于作恒等运算, 加减实质上是同一种运算, 乘法则等同于某种意义下的逻辑运算, 这些事实给奇偶分析提供了极大的便利.", + "figures": [] +} \ No newline at end of file diff --git a/processed_dataset/proof/0734.json b/processed_dataset/proof/0734.json new file mode 100644 index 0000000000000000000000000000000000000000..9f536984b9a3b9dd2eb1fb141fb5cfeaf81be6cb --- /dev/null +++ b/processed_dataset/proof/0734.json @@ -0,0 +1,8 @@ +{ + "source_file": "./raw_volume-zh/volume14/chapter7.tex", + "problem_type": "proof", + "problem": "例2. 设 $a_1, a_2, \\cdots, a_n(n \\geqslant 4)$ 中的每一个值或等于 1 , 或等于 -1 , 且\n$$\na_1 a_2 a_3 a_4+a_2 a_3 a_4 a_5+\\cdots+a_{n-1} a_n a_1 a_2+a_n a_1 a_2 a_3=0 .\n$$\n求证: $4 \\mid n$.", + "solution": "证明:不妨记 $a_{n+i}=a_i(i=1,2,3)$, 已知条件可以写为\n$$\n\\sum_{k=1}^n a_k a_{k+1} a_{k+2} a_{k+3}=0 . \\label{eq1}\n$$\n由已知得, 对 $k=1,2, \\cdots, n$, 有 $a_k a_{k+1} a_{k+2} a_{k+3} \\in\\{1,-1\\}$, 因此在 式\\ref{eq1} 左边的和式中取 1 的项与取 -1 的项一样多, 不妨记有 $m$ 项为 $1, m$ 项为 -1 , 故 $n$ 为偶数,且 $n=2 m$.\n进一步考虑到 $\\prod_{k=1}^n a_k a_{k+1} a_{k+2} a_{k+3}=\\left(a_1 a_2 \\cdots a_n\\right)^4=1$ 恒成立, 而该式左边的值为\n$$\n1^m \\cdot(-1)^m=(-1)^m,\n$$\n因此 $m$ 为偶数,故 $n=2 m$ 为 4 的倍数.", + "remark": "注:本题的证明分两步,各自体现了奇偶分析的想法:第一步是考虑到 $n$个值为奇数 $( \\pm 1)$ 的项相加等于偶数 $(0)$, 先推出 $n$ 为偶数; 第二步是用到积为正数的若干个数字中负数必有偶数个, 从而得出 $m=\\frac{n}{2}$ 仍是偶数.", + "figures": [] +} \ No newline at end of file diff --git a/processed_dataset/proof/0735.json b/processed_dataset/proof/0735.json new file mode 100644 index 0000000000000000000000000000000000000000..79b83503e9d2bd4c94317e1443cd03bf2d4e148f --- /dev/null +++ b/processed_dataset/proof/0735.json @@ -0,0 +1,8 @@ +{ + "source_file": "./raw_volume-zh/volume14/chapter7.tex", + "problem_type": "proof", + "problem": "例3. 设 $a, b$ 是正整数,且满足关系式\n$$\n(11111+a)(11111-b)=123456789 .\n$$\n求证: $a-b$ 是 4 的倍数.", + "solution": "证明:由已知条件可得 $11111+a$ 与 $11111-b$ 均为奇数,所以 $a, b$ 均为偶数, 又由已知条件得\n$$\n11111(a-b)=a b+2468,\n$$\n因为 $a b$ 是 4 的倍数, $2468=4 \\times 617$ 也是 4 的倍数, 所以 $11111 \\times(a-b)$ 是 4 的倍数,故 $a--b$ 是 4 的倍数.", + "remark": "注:本题通过奇偶性分析避免了过多的情况枚举.\n另外,如从模 4 同余的角度考虑, 可进一步减少运算量.", + "figures": [] +} \ No newline at end of file diff --git a/processed_dataset/proof/0736.json b/processed_dataset/proof/0736.json new file mode 100644 index 0000000000000000000000000000000000000000..97ff5d30e46304614e381b8e0cba275d5d57d6b5 --- /dev/null +++ b/processed_dataset/proof/0736.json @@ -0,0 +1,10 @@ +{ + "source_file": "./raw_volume-zh/volume14/chapter7.tex", + "problem_type": "proof", + "problem": "例4. 从集合 $\\{0,1,2, \\cdots, 13,14\\}$ 中选出 10 个不同的数填人图 () 中圆圈内,使每两个用线相连的圆圈中的数所成差的绝对值各不相同, 能否做到这一点? 证明你的结论.", + "solution": "解:结论是否定的.\n若不然,那么所说的差的绝对值共有 14 个, 它们互不相同, 并且均不大于 14 , 不小于 1 , 因此它们只能是 $1,2,3, \\cdots, 14$, 从而它们的和.\n$$\nS=1+2+\\cdots+14=7 \\times 15=105\n$$\n是一个奇数.\n另一方面, 每个圆圈与偶数个 (2 个或 4 个) 圆圈相连, 设填人的数为 $a$, 那么 $a$ 在 $S$ 中出现偶数次 (2 次或 4 次). 偶数个 $a$ 用加、减号相连, 运算结果必为偶数.\n因此, $S$ 是 10 个偶数的和, 从而 $S$ 是偶数.\n从上面可知, $S$ 既是奇数又要是偶数, 矛盾!", + "remark": "", + "figures": [ + "./images/volume14/figures/fig-c7i1.png" + ] +} \ No newline at end of file diff --git a/processed_dataset/proof/0737.json b/processed_dataset/proof/0737.json new file mode 100644 index 0000000000000000000000000000000000000000..56ff1d146fbb345b7671894daa0037136c2bc29f --- /dev/null +++ b/processed_dataset/proof/0737.json @@ -0,0 +1,8 @@ +{ + "source_file": "./raw_volume-zh/volume14/chapter7.tex", + "problem_type": "proof", + "problem": "例5. 已知直角坐标平面内有 $n$ 个整点, 满足任意三点不共线, 且所构成的三角形面积不是整数.\n求 $n$ 的最大可能值.", + "solution": "解:将平面上的整点分成如下四个集合:\n$$\n\\begin{gathered}\nM_1=\\{(x, y) \\mid x \\equiv y \\equiv 0(\\bmod 2)\\}, \\\\\nM_2=\\{(x, y) \\mid x \\equiv 1(\\bmod 2), y \\equiv 0(\\bmod 2)\\}, \\\\\nM_3=\\{(x, y) \\mid x \\equiv 0(\\bmod 2), y \\equiv 1(\\bmod 2)\\}, \\\\\nM_4=\\{(x, y) \\mid x \\equiv y \\equiv 1(\\bmod 2)\\} .\n\\end{gathered}\n$$\n假如 $n \\geqslant 5$, 由抽屉原理可知, 必有两点 $A\\left(x_1, y_1\\right), B\\left(x_2, y_2\\right)$ 属于上述同一集合, 则 $A B$ 中点 $M\\left(\\frac{x_1+x_2}{2}, \\frac{y_1+y_2}{2}\\right)$ 为整点.\n任取 $n$ 个点中的另一点 $C$, 注意整点三角形面积的 2 倍必为整数, 则 $S_{\\triangle A B C}=2 S_{\\triangle A M C} \\in \\mathbf{Z}$, 矛盾.\n因此 $n \\leqslant 4$.\n另一方面, 取 $(0,0),(1,0),(0,1),(1,1)$ 这四个点, 易验证满足题意.\n综上, $n$ 的最大可能值为 4 .", + "remark": "注:本例通过坐标的奇偶性来制造抽屉,这是研究整点问题的一种常用的技巧.\n读者可以进一步思考如下两个稍难的问题:\n(1)已知直角坐标平面内有 $n$ 个整点, 满足任意三点不共线, 且所构成的三角形面积为奇数.\n求 $n$ 的最大可能值.\n(2)已知直角坐标平面内有 $n$ 个整点, 满足任意三点不共线, 且所构成的三角形面积不是偶数.\n求 $n$ 的最大可能值.", + "figures": [] +} \ No newline at end of file diff --git a/processed_dataset/proof/0738.json b/processed_dataset/proof/0738.json new file mode 100644 index 0000000000000000000000000000000000000000..18c783830a5d8baba8f01a73bd479ea075c70c05 --- /dev/null +++ b/processed_dataset/proof/0738.json @@ -0,0 +1,8 @@ +{ + "source_file": "./raw_volume-zh/volume14/chapter7.tex", + "problem_type": "proof", + "problem": "例7. 设 $a, b, c, d$ 为奇数, $0c-b>0$, 所以\n$$\n(a+d)^2=(d-a)^2+4 a d>(c-b)^2+4 b c=(b+c)^2,\n$$\n故 $k>m$. 又\n$$\na\\left(2^k-a\\right)=a d=b c=b\\left(2^m-b\\right),\n$$\n从而 $2^m b-2^k a=b^2-a^2$, 即\n$$\n2^m\\left(b-2^{k-m} a\\right)=(b+a)(b-a), \\label{eq1}\n$$\n注意 $a, b$ 为奇数,故 $b+a$ 与 $b-a$ 均为偶数,且它们的差为 $2 a \\equiv 2(\\bmod 4)$,故其中必有一个只能被 2 整除而不能被 4 整除, 再注意 $b-2^{k-m} a$ 为奇数, 故只有两类情形:\n$$\n\\left\\{\\begin{array} { l } \n{ b + a = 2 ^ { m - 1 } u , } \\\\\n{ b - a = 2 v }\n\\end{array} \\text { 或 } \\left\\{\\begin{array}{l}\nb+a=2 v, \\\\\nb-a=2^{m-1} u,\n\\end{array}, u, v \\in \\mathbf{N}^*\\right.\\right. \\label{eq2}\n$$\n在其中任一情形下均有 $u v=b-2^{k-m} a), $\\frac{B F}{F C}=\\frac{S_{\\triangle A B F}}{S_{\\triangle A F C}}=\\frac{S_{\\triangle A B F}}{S_{\\triangle A B D}} \\cdot \\frac{S_{\\triangle A B D}}{S_{\\triangle A F C}}$, 显然 $\\angle B A F=\\angle A D B, \\angle F A C=\\angle A B D$, 所以\n$$\n\\begin{aligned}\n& \\frac{S_{\\triangle A B F}}{S_{\\triangle A B D}}=\\frac{A B \\cdot A F}{A D \\cdot B D}, \\\\\n& \\frac{S_{\\triangle A B D}}{S_{\\triangle A F C}}=\\frac{A B \\cdot B D}{A F \\cdot A C},\n\\end{aligned}\n$$\n因此 $\\frac{B F}{F C}=\\frac{A B}{A D} \\cdot \\frac{A B}{A C}=2$, 即 $B F=2 F C$.", + "remark": "", + "figures": [ + "./images/volume14/figures/fig-c8i2.png" + ] +} \ No newline at end of file diff --git a/processed_dataset/proof/0740.json b/processed_dataset/proof/0740.json new file mode 100644 index 0000000000000000000000000000000000000000..83e86601b15faf854b7aa83a9d76e562ba38905e --- /dev/null +++ b/processed_dataset/proof/0740.json @@ -0,0 +1,10 @@ +{ + "source_file": "./raw_volume-zh/volume14/chapter8.tex", + "problem_type": "proof", + "problem": "例2. 如图(), 在凸四边形 $A B C D$ 的边 $A B$ 和 $B C$ 上取点 $E 、 F$, 使得线段 $D E$ 、 $D F$ 分对角线 $A C$ 为三等分.\n已知 $S_{\\triangle A D E}=S_{\\triangle C D F}=\\frac{1}{4} S_{A B C D}$, 证明: $A B C D$ 是平行四边形.", + "solution": "证明:设 $D E 、 D F$ 分别与 $A C$ 交于 $P 、 Q$. 连接 $B D$, 交 $A C$ 与 $M$.\n由 $A P=Q C$ 得 $S_{\\triangle A D P}=S_{\\triangle C D Q}$, 又 $S_{\\triangle A D E}=S_{\\triangle C D F}$, 所以 $S_{\\triangle A E P}=S_{\\triangle C F Q}$. 故 $E 、 F$ 到 $A C$ 的距离相等, 因此 $E F / / A C$.\n设 $\\frac{A B}{A E}=\\frac{C B}{C F}=k$, 则\n$$\n\\frac{S_{\\triangle A D B}}{S_{\\triangle A D E}}=\\frac{A B}{A E}=k, \\frac{S_{\\triangle C D B}}{S_{\\triangle C D F}}=\\frac{C B}{C F}=k,\n$$\n所以\n$$\n\\begin{aligned}\nS_{A B C D} & =S_{\\triangle A D B}+S_{\\triangle C D B}=k\\left(S_{\\triangle A D E}+S_{\\triangle C D F}\\right) \\\\\n& =k \\cdot \\frac{1}{2} S_{A B C D},\n\\end{aligned}\n$$\n即 $k=2$. 因而 $\\frac{A Q}{A P}=\\frac{A B}{A E}=2$, 所以 $B Q / / E P$. 同理有 $B P / / F Q$. 因此 $B P D Q$ 为平行四边形, 故 $B M=M D, P M=M Q$, 又 $A P=Q C$, 所以 $A M=M C$, 即 $A C 、 B D$ 互相平分, 故而 $A B C D$ 是平行四边形.", + "remark": "注:本题中一些重要的平行关系都是通过面积关系导出的: 先是通过面积的运算得到 $E F / / A C$, 再是通过图形关系列出 $S_{A B C D}$ 满足的面积等式, 为证明 $B Q / / E P$ 与 $B P / / F Q$ 起到桥梁作用.\n面积法的特点是把各已知量和未知量用面积公式联系起来, 使几何元素之间的关系变成数量关系, 通过运算达到求证的结果, 很多场合下这可以降低分析问题或添置补助线的难度, 使证明简洁明快.", + "figures": [ + "./images/volume14/figures/fig-c8i1.png" + ] +} \ No newline at end of file diff --git a/processed_dataset/proof/0741.json b/processed_dataset/proof/0741.json new file mode 100644 index 0000000000000000000000000000000000000000..a16b2c2f07e1b25b8fb909d6d7b340240fbb7c67 --- /dev/null +++ b/processed_dataset/proof/0741.json @@ -0,0 +1,10 @@ +{ + "source_file": "./raw_volume-zh/volume14/chapter8.tex", + "problem_type": "proof", + "problem": "例3. 设 $\\triangle A B C$ 中, 顶点 $A, B, C$ 的对边分别为 $a, b, c$, 内心 $I$ 到顶点 $A, B, C$ 的距离分别为 $m, n, l$. 求证:\n$$\na l^2+b m^2+c n^2=a b c .\n$$", + "solution": "证明:如图(), 设 $\\triangle A B C$ 内切圆与三边 $B C, C A, A B$ 分别相切于 $D, E, F$.\n由于 $\\angle A F I=\\angle A E I=90^{\\circ}$, 故四边形 $A E I F$ 为圆内接四边形, 且 $A I$ 为该圆的直径.\n又显然有 $A I \\perp E F$, 故由四边形面积公式可得\n$$\n\\begin{aligned}\nS_{A E I F} & =\\frac{1}{2} A I \\cdot E F=\\frac{1}{2} A I \\cdot A I \\sin A \\\\\n& =\\frac{1}{2} l^2 \\cdot \\frac{a}{2 R}=\\frac{a l^2}{4 R},\n\\end{aligned}\n$$\n其中第 2、第 3 个等号分别是对 $\\triangle A E F$ 与 $\\triangle A B C$ 用了正弦定理, $R$ 为 $\\triangle A B C$ 的外接圆半径.\n同理可得 $S_{B F I D}=\\frac{b m^2}{4 R}, S_{C D I E}=\\frac{c n^2}{4 R}$.\n所以 $S_{\\triangle A B C}=S_{A E I F}+S_{B F I D}+S_{C D I E}=\\frac{a l^2+b m^2+c n^2}{4 R}$.\n但另一方面, $S_{\\triangle A B C}=\\frac{a b c}{4 R}$, 从而 $a l^2+b m^2+c n^2=a b c$.", + "remark": "注:用两种不同的方法计算同一个面积, 得到的结果应当相等, 这是面积法的一种基本思想 (参看第 16 节\"算两次\"). 本题中正是将 $\\triangle A B C$ 分割为 3 个四边形, 建立了面积等式.\n考虑到这 3 个四边形都有外接圆, 且对角线相互垂直, 因此在用已知量表示它们面积时没有实质的困难, 而引人 $\\triangle A B C$ 的外接圆半径 $R$ 又可以消去角的正弦, 起到过渡作用.\n三角形与四边形的面积公式揭示了边角等基本元素之间的内在关系,而三角形的面积又常常能和内心、外心等 (及有关量) 相联系, 这是用面积证题时值得注意的一点.\n例如就本题图形出发, 读者不妨证一下另一个有趣的结论: $\\frac{S_{\\triangle A B C}}{S_{\\triangle D E F}}=\\frac{2 R}{r}$, 其中 $R, r$ 分别为 $\\triangle A B C$ 外接圆半径与内切圆半径.", + "figures": [ + "./images/volume14/figures/fig-c8i3.png" + ] +} \ No newline at end of file diff --git a/processed_dataset/proof/0742.json b/processed_dataset/proof/0742.json new file mode 100644 index 0000000000000000000000000000000000000000..340b0842fd07a0630e7d05f831965ea735a4d6f3 --- /dev/null +++ b/processed_dataset/proof/0742.json @@ -0,0 +1,10 @@ +{ + "source_file": "./raw_volume-zh/volume14/chapter8.tex", + "problem_type": "proof", + "problem": "例4. 已知圆内接六边形 $A B C D E F$ 中, $A B \\cdot C D \\cdot E F=B C \\cdot D E \\cdot F A$, 证明 $A D, B E, C F$ 三线共点.", + "solution": "证明: 如图(), 连接 $A C, C E, E A$. 记 $A C$ 交 $B E$ 于点 $P, C E$ 交 $A D$ 于点 $Q, E A$ 交 $C E$ 于点 $R$.\n在圆内接四边形 $A B C E$ 中, $\\angle B A E$ 与 $\\angle B C E$ 互补, 故 $\\sin \\angle B A E=\\sin \\angle B C E$. 从而由共边定理和三角形面积公式可得:\n$$\n\\frac{A P}{P C}=\\frac{S_{\\triangle B A E}}{S_{\\triangle B C E}}=\\frac{A B \\cdot A E}{B C \\cdot C E} .\n$$\n同理可得\n$$\n\\begin{aligned}\n& \\frac{C Q}{Q E}=\\frac{S_{\\triangle C A D}}{S_{\\triangle E A D}}=\\frac{A C \\cdot C D}{A E \\cdot D E}, \\\\\n& \\frac{E R}{R A}=\\frac{S_{\\triangle E C F}}{S_{\\triangle A C F}}=\\frac{C E \\cdot E F}{A C \\cdot A F} .\n\\end{aligned}\n$$\n以上三式相乘可得\n$$\n\\begin{aligned}\n\\frac{A P}{P C} \\cdot \\frac{C Q}{Q E} \\cdot \\frac{E R}{R A} & =\\frac{A B \\cdot A E}{B C} \\cdot \\frac{A C \\cdot C E}{A E \\cdot D E} \\cdot \\frac{C E \\cdot E F}{A C \\cdot A F} \\\\\n& =\\frac{A B \\cdot C D \\cdot E F}{B C \\cdot D E \\cdot F A}=1,\n\\end{aligned}\n$$\n又 $A D, B E, C F$ 不平行,故由 Ceva 定理的逆定理可得 $A D, B E, C F$ 三线共点.", + "remark": "注:本题通过两种不同的方式表示 $\\frac{S_{\\triangle B A E}}{S_{\\triangle B C E}}$ 等三个面积比例式, 建立了线段的比例关系,最后用 Ceva 定理的逆定理证明三线共点.\nCeva 定理有几种等价形式,使用时可选择适合的角度,例如本题用角元形式的 Ceva 定理书写会更为简洁, 但其本质是一样的.", + "figures": [ + "./images/volume14/figures/fig-c8i4.png" + ] +} \ No newline at end of file diff --git a/processed_dataset/proof/0743.json b/processed_dataset/proof/0743.json new file mode 100644 index 0000000000000000000000000000000000000000..7faf9071880679137aa8d765571800f1cf2d3e79 --- /dev/null +++ b/processed_dataset/proof/0743.json @@ -0,0 +1,11 @@ +{ + "source_file": "./raw_volume-zh/volume14/chapter8.tex", + "problem_type": "proof", + "problem": "例5. 如图(), 延长凸四边形 $A B C D$ 的边 $A B 、 D C$ 交于点 $E$, 延长边 $A D 、 B C$ 交于点 $F$. 求证: $A C 、 B D 、 E F$ 的中点 $M 、 N 、 L$ 这三点共线 (这条线称为完全四边形 $A B C D E F$ 的\"牛顿线\").", + "solution": "证明:连接 $M B 、 M D 、 M E 、 M F 、 N E 、 N F 、 M N$.\n由 $M 、 N$ 分别是 $A C 、 B D$ 的中点可得:\n$$\n\\begin{aligned}\nS_{\\triangle M D E} & =S_{\\triangle M N E}+S_{\\triangle M D N}+S_{\\triangle E D N} \\\\\n& =S_{\\triangle M N E}+S_{\\triangle M B N}+S_{\\triangle E B N}=2 S_{\\triangle M N E}+S_{\\triangle M B E},\n\\end{aligned}\n$$\n故\n$$\n\\begin{aligned}\nS_{\\triangle M N E} & =\\frac{S_{\\triangle M D E}-S_{\\triangle M B E}}{2} \\\\\n& =\\frac{S_{\\triangle A D E}-S_{\\triangle C B E}}{4}=-\\frac{1}{4} S_{A B C D} .\n\\end{aligned}\n$$\n同理可得\n$$\nS_{\\triangle M N F}=\\frac{S_{\\triangle M B F}-S_{\\triangle M D F}}{2}=\\frac{S_{\\triangle A B F}-S_{\\triangle C D F}}{4}=\\frac{1}{4} S_{A B C D} .\n$$\n从而 $S_{\\triangle M N E}=S_{\\triangle M N F}$.\n由于 $E 、 F$ 在直线 $M N$ 异侧 (在已知的图形关系下, 有直线 $M N$ 与线段 $E F$ 相交), 故直线 $M N$ 平分线段 $E F$, 即 $M 、 N 、 L$ 三点共线.", + "remark": "注:本题中,我们先充分运用 \" $M 、 N$ 为 $A C 、 B D$ 中点\"的条件进行面积转换.\n最后, 我们利用 $S_{\\triangle M N E}= S_{\\triangle M N F}$ 证得另一个中点 $L$ 在直线 $M N$ 上, 事实上, 这是基于以下定理:\n定理设点 $P$ 为 $\\triangle A B C$ 所在平面上一点, 直线 $C P$ 与 $A B$ (或其延长线) 相交于点 $D$, 如图(), 则 $\\frac{S_{\\triangle A P C}}{S_{\\triangle B P C}}=\\frac{A D}{B D}$.\n因此,若 $P$ 在 $\\triangle A B C$ 内, $D$ 在线段 $A B$ 上,且 $\\frac{S_{\\triangle A P C}}{S_{\\triangle B P C}}=\\frac{A D}{B D}$, 则 $C 、 P 、 D$ 三点共线.\n根据这个结论不难完成本题最后的证明步骤.\n可见, 面积方法是证明三点共线的方法之一.\n在组合几何方面, 也常常要从面积人手考虑一些问题.\n下例是一个覆盖问题,证明中用到了面积重叠原理:\n将 $n$ 个面积为 $S_i(1 \\leqslant i \\leqslant n)$ 的区域放人一个面积为 $S_0$ 的区域 $C$, 若这 $n$ 个区域的面积总和大于 $k S_0$, 则 $C$ 中必有一点被其中至少 $k+1$ 个区域覆盖; 若面积总和小于 $k S_0$, 则 $C$ 中必有一点被其中至多 $k-1$ 个区域覆盖.", + "figures": [ + "./images/volume14/figures/fig-c8i5.png", + "./images/volume14/figures/fig-c8i6.png" + ] +} \ No newline at end of file diff --git a/processed_dataset/proof/0744.json b/processed_dataset/proof/0744.json new file mode 100644 index 0000000000000000000000000000000000000000..398e53399221900fe568146f22aa364cf60dea31 --- /dev/null +++ b/processed_dataset/proof/0744.json @@ -0,0 +1,8 @@ +{ + "source_file": "./raw_volume-zh/volume14/chapter8.tex", + "problem_type": "proof", + "problem": "例6. 在半径为 16 的圆中有 650 个红点, 证明: 可作一个内半径为 2 、外半径为 3 的圆环 $C$, 使 $C$ 内 (不含边界) 至少含有 10 个红点.", + "solution": "证明:以 650 个红点中的每一点为中心, 作内半径为 $2+\\varepsilon$, 外半径为 $3- \\varepsilon$ 的圆环 $C_i(1 \\leqslant i \\leqslant 650)$, 每个 $C_i$ 的面积为\n$$\nS_i=\\pi\\left((3-\\varepsilon)^2-(2+\\varepsilon)^2\\right)=5 \\pi(1-2 \\varepsilon),\n$$\n其中 $0<\\varepsilon<\\frac{1}{2}$.\n显然这些圆环都在一个半径为 19 的圆 $C_0$ 内, 其中 $C_0$ 的面积 $S_0=361 \\pi$.\n取定 $\\varepsilon=0.0001$, 则\n$$\n\\sum_{i=1}^{650} S_i=3250 \\pi \\times 0.9998>3249 \\pi=9 S_0,\n$$\n故由面积重叠原理可知, $C_0$ 中必有一点被不少于 10 个圆环 $C_i$ 覆盖.\n以这一点为中心, 作一个内半径为 2 、外半径为 3 的圆环 $C$, 其内部必含有 10 个 $C_i$ 的中心, 故圆环 $C$ 满足题意.", + "remark": "注:面积重叠原理可谓几何上的\"抽屉原理\". 在考虑一些覆盖、嵌人、重叠问题时, 常通过面积来证明一些存在性的结论(有时需辅以膨胀、收缩的技巧).", + "figures": [] +} \ No newline at end of file diff --git a/processed_dataset/proof/0745.json b/processed_dataset/proof/0745.json new file mode 100644 index 0000000000000000000000000000000000000000..ce4ee537ebff2c14500e41d0663d3c36740cb317 --- /dev/null +++ b/processed_dataset/proof/0745.json @@ -0,0 +1,8 @@ +{ + "source_file": "./raw_volume-zh/volume14/chapter8.tex", + "problem_type": "proof", + "problem": "例7. 已知圆 $O$ 在平面直角坐标系中, 半径为 $R$, 圆周上整点个数为 $n(n \\geqslant 3)$. 证明: $n<2 \\pi \\cdot \\sqrt[3]{R^2}$.", + "solution": "证明:设圆周上的所有整点按逆时针排列为 $A_1, A_2, \\cdots, A_n(n \\geqslant 3)$.\n约定 $A_{n+1}=A_1, A_{n+2}=A_2$. 对 $i=1,2, \\cdots, n$, 由于 $\\triangle A_i A_{i+1} A_{i+2}$ 为整点三角形,其面积 $S_{\\triangle A_i A_{i+1} A_{i+2}} \\geqslant \\frac{1}{2}$.\n另一方面, 设 $\\angle A_i O A_{i+1}=\\theta_i, i=1,2, \\cdots, n$, 则\n$$\n\\begin{aligned}\n& \\left|A_i A_{i+1}\\right|=2 R \\sin \\frac{\\theta_i}{2},\\left|A_{i+1} A_{i+2}\\right|=2 R \\sin \\frac{\\theta_{i+1}}{2}, \\\\\n& \\angle A_i A_{i+1} A_{i+2}=\\frac{\\pi-\\theta_i}{2}+\\frac{\\pi-\\theta_{i+1}}{2}=\\pi-\\frac{\\theta_i+\\theta_{i+1}}{2},\n\\end{aligned}\n$$\n故\n$$\n\\begin{aligned}\nS_{\\triangle A_i A_{i+1} A_{i+2}} & =\\frac{1}{2} \\cdot\\left|A_i A_{i+1}\\right| \\cdot\\left|A_{i+1} A_{i+2}\\right| \\cdot \\sin \\angle A_i A_{i+1} A_{i+2} \\\\\n& =\\frac{1}{2} \\cdot 2 R \\sin \\frac{\\theta_i}{2} \\cdot 2 R \\sin \\frac{\\theta_{i+1}}{2} \\cdot \\sin \\frac{\\theta_i+\\theta_{i+1}}{2} \\\\\n& <2 R^2 \\cdot \\frac{\\theta_i}{2} \\cdot \\frac{\\theta_{i+1}}{2} \\cdot \\frac{\\theta_i+\\theta_{i+1}}{2} .\n\\end{aligned}\n$$\n所以\n$$\n\\frac{1}{2}<\\frac{R^2}{4} \\cdot \\theta_i \\theta_{i+1}\\left(\\theta_i+\\theta_{i+1}\\right) \\leqslant \\frac{R^2}{16}\\left(\\theta_i+\\theta_{i+1}\\right)^3,\n$$\n从而\n$$\n\\theta_i+\\theta_{i+1}>\\frac{2}{\\sqrt[3]{R^2}} \\label{eq1}\n$$\n注意到 $\\sum_{i=1}^n \\theta_i=2 \\pi$, 故 式\\ref{eq1} 中令 $i=1,2, \\cdots, n$, 并将 $n$ 个不等式相加得\n$$\n4 \\pi>n \\cdot \\frac{2}{\\sqrt[3]{R^2}}\n$$\n即 $n<2 \\pi \\cdot \\sqrt[3]{R^2}$.", + "remark": "注:由于圆 $O$ 的周长 $2 \\pi R$ 是 $R$ 的一阶量, 本例的结论 $n<2 \\pi \\cdot \\sqrt[3]{R^2}$, 实际上说明了随着 $R$ 的增大, 圆周上的整点分布大致会越来越稀疏, 也因如此, 若能找到一个量来刻画这种特征, 将有助于解决问题.\n上述解法中所找的量是\"整点三角形的面积\", 它确实可以用来刻画这种稀疏性: 在半径很大的圆周上,假设依次有三个整点 $A, B, C$ 且它们十分临近,则 $\\triangle A B C$ 的面积必然小于 $\\frac{1}{2}$, 与 $\\triangle A B C$ 为整点三角形矛盾.\n利用这一点, 我们可以估计每相邻两个圆心角之和 $\\theta_i+\\theta_{i+1}$ 的下界, 最终得到整点个数 $n$ 的上界.\n一般的整点问题中有不少与数论有关,但本例中圆 $O$ 的圆心位置和半径并未给出有效的信息,这里,整点三角形面积的\"离散性\"起了关键作用.\n请读者思考如何把结论推广到椭圆的情形.", + "figures": [] +} \ No newline at end of file diff --git a/processed_dataset/proof/0746.json b/processed_dataset/proof/0746.json new file mode 100644 index 0000000000000000000000000000000000000000..f8470c34289824e83bca258117f514522e73e8c8 --- /dev/null +++ b/processed_dataset/proof/0746.json @@ -0,0 +1,8 @@ +{ + "source_file": "./raw_volume-zh/volume14/chapter9.tex", + "problem_type": "proof", + "problem": "例2. 已知: 三元集 $A=\\{a b, 2 b, 3 c\\}, B=\\left\\{a, 2 b^2, 3 b c\\right\\}$, 其中 $a 、 b 、 c$ 是非零实数,使得 $A=B$. 求 $b$ 的所有可能值.", + "solution": "解:一设 $A=B$, 由已知条件, 对集合 $B$ 中 $a$ 的取值分三类情况讨论.\n若 $a=a b$, 则两边同除以非零常数 $a$, 知: $b=1$.\n若 $a=2 b$, 则 $a b=2 b^2, A$ 和 $B$ 中分别剩下的还有 $3 c$ 和 $3 b c$, 它们相等.\n考虑到 $c$ 是非零常数,有: $b=1$.\n若 $a=3 c$, 则 $a b=3 b c, A$ 和 $B$ 中分别剩下的还有 $2 b$ 和 $2 b^2$, 它们相等.\n考虑到 $b$ 是非零常数,仍有: $b=1$.\n最后检验一下: 当 $b=1$ 时, $A=\\{a, 2,3 c\\}=B$, 只要适当取非零的 $a 、 c$, 使得 $a 、 2 、 3 c$ 两两不等, 即可保证 $A 、 B$ 为满足条件的三元集.\n故 $b=1$.", + "remark": "", + "figures": [] +} \ No newline at end of file diff --git a/processed_dataset/proof/0747.json b/processed_dataset/proof/0747.json new file mode 100644 index 0000000000000000000000000000000000000000..87baa2f1b696e1f39e2bc3f4e9def870c8910c88 --- /dev/null +++ b/processed_dataset/proof/0747.json @@ -0,0 +1,8 @@ +{ + "source_file": "./raw_volume-zh/volume14/chapter9.tex", + "problem_type": "proof", + "problem": "例3. 设集合 $M=\\{1,2, \\cdots, 19\\}, A=\\left\\{a_1, a_2, \\cdots, a_k\\right\\} \\subseteq M$. 求最小的正整数 $k$, 使得对任意 $b \\in M$, 存在 $a_i, a_j \\in A$, 满足 $a_i=b$ 或 $a_i \\pm a_j=b\\left(a_i\\right.$, $a_j$ 可以相同).", + "solution": "解:照题意, $A$ 中元素至多给出 $k+k+2 \\mathrm{C}_k^2=k(k+1)$ 种可能的运算结果, 故 $k(k+1) \\geqslant 19$, 即 $k \\geqslant 4$.\n对 $k=4$, 假定有 $A=\\left\\{a_1, a_2, a_3, a_4\\right\\}$ 满足题意, 不妨设 $a_1a>e$, 证明: $a^b>b^a$.", + "solution": "取对数后只须证 $b \\ln a>a \\ln b$, 只须证 $f(x)=\\frac{\\ln x}{x}(x>\\mathrm{e})$ 单调减.\n此时显然 $f^{\\prime}(x)=\\frac{1-\\ln x}{x^2}<0$, 故结论成立.", + "remark": "注:本题用\"作商法\"也可证明,而\"作差法\" 将使问题大大复杂化.", + "figures": [] +} \ No newline at end of file diff --git a/processed_dataset/proof/0752.json b/processed_dataset/proof/0752.json new file mode 100644 index 0000000000000000000000000000000000000000..a0b25cf8e31c896903cea8cb768a6c401b389afc --- /dev/null +++ b/processed_dataset/proof/0752.json @@ -0,0 +1,8 @@ +{ + "source_file": "./raw_volume-zh/volume14/exercise1.tex", + "problem_type": "proof", + "problem": "问题4. 设实数 $x 、 y 、 z$ 大于或等于 1 , 求证:\n$$\n\\left(x^2-2 x+2\\right)\\left(y^2-2 y+2\\right)\\left(z^2-2 z+2\\right) \\leqslant(x y z)^2-2 x y z+2 .\n$$", + "solution": "注意到 $x \\geqslant 1, y \\geqslant 1$, 所以\n$$\n\\begin{aligned}\n& \\left(x^2-2 x+2\\right)\\left(y^2-2 y+2\\right)-\\left((x y)^2-2 x y+2\\right) \\\\\n= & (-2 y+2) x^2+\\left(6 y-2 y^2-4\\right) x+\\left(2 y^2-4 y+2\\right)\n\\end{aligned}\n$$\n$$\n\\begin{aligned}\n& =-2(y-1)\\left(x^2+(y-2) x+1-y\\right) \\\\\n& =-2(y-1)(x-1)(x+y-1) \\leqslant 0,\n\\end{aligned}\n$$\n所以\n$$\n\\left(x^2-2 x+2\\right)\\left(y^2-2 y+2\\right) \\leqslant(x y)^2-2 x y+2 .\n$$\n同理, 因为 $x y \\geqslant 1, z \\geqslant 1$, 所以\n$$\n\\left((x y)^2-2 x y+2\\right)\\left(z^2-2 z+2\\right) \\leqslant(x y z)^2-2 x y z+2 .\n$$\n从而命题得证.", + "remark": "", + "figures": [] +} \ No newline at end of file diff --git a/processed_dataset/proof/0753.json b/processed_dataset/proof/0753.json new file mode 100644 index 0000000000000000000000000000000000000000..6600b31a3f5bc1e888c8f9fdf991d4631693a1c8 --- /dev/null +++ b/processed_dataset/proof/0753.json @@ -0,0 +1,8 @@ +{ + "source_file": "./raw_volume-zh/volume14/exercise1.tex", + "problem_type": "proof", + "problem": "问题5. 已知 $n$ 个实数 $x_1, x_2, \\cdots, x_n$ 的算术平均值为 $a$, 证明:\n$$\n\\sum_{k=1}^n\\left(x_k-a\\right)^2 \\leqslant \\frac{1}{2}\\left(\\sum_{k=1}^n\\left|x_k-a\\right|\\right)^2 .\n$$", + "solution": "当 $a=0$ 时,有\n$$\n0=\\left(x_1+x_2+\\cdots+x_n\\right)^2=\\sum_{k=1}^n x_k^2+2 \\sum_{1 \\leqslant i),这里, 我们用解析法给出一个证明.\n以直线 $B C$ 为 $x$ 轴, 线段 $B C$ 的垂直平分线为 $y$ 轴建立直角坐标系, 设三角形 $A B C$ 的三个顶点的坐标分别为 $A(a, b), B(-r, 0), C(r, 0)$, 其中 $r$ 是圆 $O$ 的半径, 如图所示.\n于是, 圆 $O$ 的方程为 $x^2+y^2=r^2$.\n设点 $P 、 Q$ 的坐标分别为 $P\\left(x_1, y_1\\right) 、 Q\\left(x_2, y_2\\right)$, 则切线 $A P 、 B P$ 的方程为 $x x_1+y y_1=r^2$ 与 $x x_2+y y_2=r^2$, 因为这两条切线都过 $A$ 点, 所以 $a x_1+ b y_1=r^2, a x_2+b y_2=r^2$, 即 $P, Q$ 都在直线 $a x+b y=r^2$ 上, 此即直线 $P Q$ 的方程.\n设 $B F$ 与 $C E$ 是三角形 $A B C$ 的两条高, 因为直线 $A C$ 的斜率为 $\\frac{b}{a-r}$, 所以直线 $B F$ 的斜率为 $\\frac{r-a}{b}$, 故直线 $B F$ 的方程为 $y=\\frac{r-a}{b}(x+r)$, 即\n$$\n(a-r) x+b y=r^2-a r ; \\label{eq1}\n$$\n同理可得, 直线 $C E$ 的方程为\n$$\n(a+r) x+b y=r^2+a r . \\label{eq2}\n$$\n由 式\\ref{eq1} + \\ref{eq2}, 得\n$$\na x+b y=r^2, \\label{eq3}\n$$\n即点 $H$ 的坐标满足方程.\n而 式\\ref{eq3} 就是直线 $P Q$ 的方程, 所以, $P, H, Q$ 三点共线.", + "remark": "", + "figures": [ + "./images/volume14/figures/fig-c10a8.png" + ] +} \ No newline at end of file diff --git a/processed_dataset/proof/0759.json b/processed_dataset/proof/0759.json new file mode 100644 index 0000000000000000000000000000000000000000..005003922c309f06ae13d4a5e66c5ec6cbd34c3f --- /dev/null +++ b/processed_dataset/proof/0759.json @@ -0,0 +1,8 @@ +{ + "source_file": "./raw_volume-zh/volume14/exercise10.tex", + "problem_type": "proof", + "problem": "问题9. 对于正整数 $n$, 令 $f_n=\\left[2^n \\cdot \\sqrt{2008}\\right]+\\left[2^n \\cdot \\sqrt{2009}\\right]$. 求证: 数列 $f_1$, $f_2, \\cdots$ 中有无穷多个奇数和无穷多个偶数 ( [ $\\left.x\\right]$ 表示不超过 $x$ 的最大整数).", + "solution": "记 $x-[x]=\\{x\\}$, 设 $a, b$ 分别为 $\\{\\sqrt{2008}\\}$ 与 $\\{\\sqrt{2009}\\}$ 在二进制下的表示.\n由于 $\\sqrt{2008}$ 和 $\\sqrt{2009}$ 是无理数,故 $a, b$ 是不循环小数,而 $f_n$ 的奇偶性取决于 $a, b$ 小数点后第 $n$ 位是否相同.\n假设 $f_1, f_2, \\cdots$ 中只有有限个奇数, 则存在某个 $n \\in \\mathbf{N}^*$, 使 $f_n, f_{n+1}, \\cdots$ 均为偶数, 即小数点后第 $n$ 位起 $a, b$ 是一样的,故 $a-b$ 实际上是有限小数,但\n$a-b$ 显然是无理数, 矛盾! 再假设 $f_1, f_2, \\cdots$ 中只有有限个偶数, 则存在某个 $n \\in \\mathbf{N}^*$, 使 $f_n, f_{n+1}, \\cdots$ 均为奇数, 即 $a, b$ 在小数点后第 $n$ 位起的每个数位上都不一样,于是 $a+b$ 在该数位上等于 1 , 这样 $a+b$ 是有限小数, 但 $a+b$ 显然是无理数, 矛盾!\n从而原命题成立.", + "remark": "", + "figures": [] +} \ No newline at end of file diff --git a/processed_dataset/proof/0760.json b/processed_dataset/proof/0760.json new file mode 100644 index 0000000000000000000000000000000000000000..0a265c3d502bb78a10247623c44408b259687681 --- /dev/null +++ b/processed_dataset/proof/0760.json @@ -0,0 +1,10 @@ +{ + "source_file": "./raw_volume-zh/volume14/exercise11.tex", + "problem_type": "proof", + "problem": "问题1. 证明: 对任意实数 $x$, 均有\n$$\n\\left|\\sqrt{x^2+x+1}-\\sqrt{x^2-x+1}\\right|<1 .\n$$", + "solution": "因为 $\\left|\\sqrt{x^2+x+1}-\\sqrt{x^2-x+1}\\right|=$\n$$\n\\left|\\sqrt{\\left(x+\\frac{1}{2}\\right)^2+\\left(\\frac{\\sqrt{3}}{2}\\right)^2}-\\sqrt{\\left(x-\\frac{1}{2}\\right)^2+\\left(\\frac{\\sqrt{3}}{2}\\right)^2}\\right|\n$$\n这可以看作直角坐标系中点 $P\\left(x, \\frac{\\sqrt{3}}{2}\\right)$ 到点 $A\\left(-\\frac{1}{2}, 0\\right)$ 与点 $B\\left(\\frac{1}{2}, 0\\right)$ 的距离的差, 如图()所示.\n根据三角形两边之差小于第三边及 $|A B|=1$, 得\n$$\n\\left|\\sqrt{x^2+x+1}-\\sqrt{x^2-x+1}\\right|<1 .\n$$", + "remark": "", + "figures": [ + "./images/volume14/figures/fig-c11a1.png" + ] +} \ No newline at end of file diff --git a/processed_dataset/proof/0761.json b/processed_dataset/proof/0761.json new file mode 100644 index 0000000000000000000000000000000000000000..63c621841947094a300a7889284992f907c3bbba --- /dev/null +++ b/processed_dataset/proof/0761.json @@ -0,0 +1,8 @@ +{ + "source_file": "./raw_volume-zh/volume14/exercise11.tex", + "problem_type": "proof", + "problem": "问题4. 已知函数 $f(x)=|\\sin x|$ 的图像与直线 $y=k x(k>0)$ 有且仅有三个交点, 交点的横坐标的最大值为 $\\alpha$, 求证: $\\frac{\\cos \\alpha}{\\sin \\alpha+\\sin 3 \\alpha}=\\frac{1+\\alpha^2}{4 \\alpha}$.", + "solution": "$f(x)$ 的图像与直线 $y=k x(k>0)$ 在 $(-\\infty, 0)$ 上无交点, 在 $[0, \\pi]$ 上恰有两个交点, 因此在 $\\left(\\pi, \\frac{3 \\pi}{2}\\right)$ 内必然相切, 根据题意, 切点的坐标可写为 $(\\alpha$, $-\\sin \\alpha)$. 当 $x \\in\\left(\\pi, \\frac{3 \\pi}{2}\\right)$ 时, 由于 $f^{\\prime}(x)=(-\\sin x)^{\\prime}=-\\cos x$, 故 $-\\cos \\alpha= \\frac{-\\sin \\alpha}{\\alpha}$, 即 $\\alpha=\\tan \\alpha$. 因此\n$$\n\\frac{\\cos \\alpha}{\\sin \\alpha+\\sin 3 \\alpha}=\\frac{\\cos \\alpha}{2 \\sin 2 \\alpha \\cos \\alpha}=\\frac{1}{2 \\sin 2 \\alpha}=\\frac{\\cos ^2 \\alpha+\\sin ^2 \\alpha}{4 \\sin \\alpha \\cos \\alpha}=\\frac{1+\\tan ^2 \\alpha}{4 \\tan \\alpha}=\\frac{1+\\alpha^2}{4 \\alpha} \\text {. }\n$$", + "remark": "", + "figures": [] +} \ No newline at end of file diff --git a/processed_dataset/proof/0762.json b/processed_dataset/proof/0762.json new file mode 100644 index 0000000000000000000000000000000000000000..7b0d5d2b7af3d0589d030f887d0f4f0a12211b8e --- /dev/null +++ b/processed_dataset/proof/0762.json @@ -0,0 +1,8 @@ +{ + "source_file": "./raw_volume-zh/volume14/exercise11.tex", + "problem_type": "proof", + "problem": "问题7. 在空间直角坐标系内, 点 $A, B_1, B_2, \\cdots, B_6$ 与原点 $O$ 这八个点中, 任意三点不共线, 点 $A$ 的坐标 $(a, b, c)$ 满足 $a+b+c=0$, 点 $B_1, B_2, \\cdots, B_6$ 的坐标依次为 $\\left(x_1, x_2, x_3\\right),\\left(x_1, x_3, x_2\\right),\\left(x_2, x_1, x_3\\right),\\left(x_2, x_3, x_1\\right)$, $\\left(x_3, x_1, x_2\\right),\\left(x_3, x_2, x_1\\right)$. 证明: $\\angle A O B_i(i=1,2, \\cdots, 6)$ 中锐角至少有两个.", + "solution": "不妨设 $x_1>x_2>x_3, a \\geqslant b \\geqslant c$ (此处两个等号不同时取到), 由切比雪夫不等式得, $\\overrightarrow{O A} \\cdot \\overrightarrow{O B_1}=x_1 a+x_2 b+x_3 c>\\frac{1}{3}(a+b+c)\\left(x_1+x_2+x_3\\right)=0$. 所以 $\\cos \\angle A O B_1>0$, 又 $O, A, B_1$ 不共线,故 $\\angle A O B_1$ 为锐角.\n另一方面, 由切比雪夫不等式得,\n$$\nx_3 a+x_2 b+x_1 c<\\frac{1}{3}(a+b+c)\\left(x_3+x_2+x_1\\right)=0,\n$$\n故\n$$\n\\begin{aligned}\n& \\max \\left\\{\\overrightarrow{O A} \\cdot \\overrightarrow{O B_2}, \\overrightarrow{O A} \\cdot \\overrightarrow{O B_3}\\right\\}=\\max \\left\\{x_1 a+x_3 b+x_2 c, x_2 a+x_1 b+x_3 c\\right\\} \\\\\n\\geqslant & \\frac{1}{2}\\left(\\left(x_1 a+x_3 b+x_2 c\\right)+\\left(x_2 a+x_1 b+x_3 c\\right)\\right)=-\\frac{1}{2}\\left(x_3 a+x_2 b+x_1 c\\right)>0 .\n\\end{aligned}\n$$\n故 $\\angle A O B_2$ 与 $\\angle A O B_3$ 中至少有一个锐角.\n综上, $\\angle A O B_i(i=1,2, \\cdots, 6)$ 中锐角至少有两个.", + "remark": "", + "figures": [] +} \ No newline at end of file diff --git a/processed_dataset/proof/0763.json b/processed_dataset/proof/0763.json new file mode 100644 index 0000000000000000000000000000000000000000..7879eb8e49e21b16382d793e300fb56fddb61f52 --- /dev/null +++ b/processed_dataset/proof/0763.json @@ -0,0 +1,8 @@ +{ + "source_file": "./raw_volume-zh/volume14/exercise11.tex", + "problem_type": "proof", + "problem": "问题8. 设 $\\triangle A B C$ 中 $B C=a, C A=b, A B=c, P$ 是 $\\triangle A B C$ 内一点.\n(1) 求证: $a \\cdot P B \\cdot P C+b \\cdot P C \\cdot P A+c \\cdot P A \\cdot P B \\geqslant a b c$;\n(2) 求证: $a \\cdot P A^2+b \\cdot P B^2+c \\cdot P C^2 \\geqslant a b c$.", + "solution": "设 $P, A, B, C$ 在复平面中分别对应复数 $z_0, z_1, z_2, z_3$. 令\n$$\n\\begin{aligned}\n& f(z)=\\frac{\\left(z-z_2\\right)\\left(z-z_3\\right)}{\\left(z_1-z_2\\right)\\left(z_1-z_3\\right)}+\\frac{\\left(z-z_3\\right)\\left(z-z_1\\right)}{\\left(z_2-z_3\\right)\\left(z_2-z_1\\right)}+\\frac{\\left(z-z_1\\right)\\left(z-z_2\\right)}{\\left(z_3-z_1\\right)\\left(z_3-z_2\\right)}, \\\\\n& g(z)=\\frac{\\left(z-z_1\\right)^2}{\\left(z_1-z_2\\right)\\left(z_1-z_3\\right)}+\\frac{\\left(z-z_2\\right)^2}{\\left(z_2-z_3\\right)\\left(z_2-z_1\\right)}+\\frac{\\left(z-z_3\\right)^2}{\\left(z_3-z_1\\right)\\left(z_3-z_2\\right)} .\n\\end{aligned}\n$$\n易验证 $f\\left(z_i\\right)=g\\left(z_i\\right)=1, i=1,2,3$, 即 $f(z)-1=0$ 与 $g(z)-1=$ 0 有至少三个复根, 但 $f(z)-1$ 与 $g(z)-1$ 都是低于 3 次的多项式, 从而 $f(z) \\equiv 1, g(z) \\equiv 1$.\n由于\n$$\n\\begin{aligned}\n1=f\\left(z_0\\right) \\leqslant & \\frac{\\left|z_0-z_2\\right| \\cdot\\left|z_0-z_3\\right|}{\\left|z_1-z_2\\right| \\cdot\\left|z_1-z_3\\right|}+\\frac{\\left|z_0-z_3\\right| \\cdot\\left|z_0-z_1\\right|}{\\left|z_2-z_3\\right| \\cdot\\left|z_2-z_1\\right|}+ \\\\\n& \\frac{\\left|z_0-z_1\\right| \\cdot\\left|z_0-z_2\\right|}{\\left|z_3-z_1\\right| \\cdot\\left|z_3-z_2\\right|} \\\\\n= & \\frac{P B \\cdot P C}{c b}+\\frac{P C \\cdot P A}{a c}+\\frac{P A \\cdot P B}{b a},\n\\end{aligned}\n$$\n故 $a \\cdot P B \\cdot P C+b \\cdot P C \\cdot P A+c \\cdot P A \\cdot P B \\geqslant a b c,(1)$ 成立;\n类似地,由于\n$$\n\\begin{aligned}\n1=g\\left(z_0\\right) \\leqslant & \\frac{\\left|z_0-z_1\\right|^2}{\\left|z_1-z_2\\right| \\cdot\\left|z_1-z_3\\right|}+\\frac{\\left|z_0-z_2\\right|^2}{\\left|z_2-z_3\\right| \\cdot\\left|z_2-z_1\\right|}+ \\\\\n& \\frac{\\left|z_0-z_3\\right|^2}{\\left|z_3-z_1\\right| \\cdot\\left|z_3-z_2\\right|}\n\\end{aligned}\n$$\n$$\n=\\frac{P A^2}{c b}+\\frac{P B^2}{a c}+\\frac{P C^2}{b a}\n$$\n故 $a \\cdot P A^2+b \\cdot P B^2+c \\cdot P C^2 \\geqslant a b c$, (2) 成立.", + "remark": "", + "figures": [] +} \ No newline at end of file diff --git a/processed_dataset/proof/0764.json b/processed_dataset/proof/0764.json new file mode 100644 index 0000000000000000000000000000000000000000..ef53291d1da6387a94c52c47a51c393eda47241f --- /dev/null +++ b/processed_dataset/proof/0764.json @@ -0,0 +1,8 @@ +{ + "source_file": "./raw_volume-zh/volume14/exercise12.tex", + "problem_type": "proof", + "problem": "问题6. 设 $n$ 为大于 2 的正整数,证明:在 $1,2, \\cdots, n$ 中,与 $n$ 互素的数的立方和能被 $n$ 整除.", + "solution": "设 $a), 黑、白染色,则黑三角形共有\n$$\n1+2+3+\\cdots+n=\\frac{1}{2} n(n+1) \\text { 个, }\n$$\n白三角形共有\n$$\n1+2+3+\\cdots+(n-1)=\\frac{1}{2} n(n-1) \\text { 个.\n}\n$$\n由于要求\"号码相邻的三角形有相邻边\", 且有相邻号码的两个三角形染有不同的颜色, 因此标上号码的黑三角形至多比标上号码的白三角形多 1 个, 所以 $m \\leqslant 2 \\times \\frac{1}{2} n(n-1)+1:=n^2-n+1$.", + "remark": "", + "figures": [ + "./images/volume14/figures/fig-c14a3.png" + ] +} \ No newline at end of file diff --git a/processed_dataset/proof/0772.json b/processed_dataset/proof/0772.json new file mode 100644 index 0000000000000000000000000000000000000000..d81f9386d3450b07ef3d31907ad58e956c6077dd --- /dev/null +++ b/processed_dataset/proof/0772.json @@ -0,0 +1,10 @@ +{ + "source_file": "./raw_volume-zh/volume14/exercise14.tex", + "problem_type": "proof", + "problem": "问题4. 用不相交的对角线把凸 $n$ 边形划分成三角形,并且在多边形的每个顶点处汇集奇数个三角形.\n证明: $3 \\mid n$.", + "solution": "用黑白两种颜色给这些三角形染色, 使得任何有公共边的两个三角形的颜色不同.\n由于已知 $n$ 边形的每个顶点都是奇数个三角形的顶点,所以这 $n$ 边形的每条边都属于同一颜色的三角形 (不妨染为黑色, 如图()所示), 可知黑色三角形比白色三角形多 $n$ 个角.\n假设 $n$ 边形被分成 $x$ 个黑色三角形及 $y$ 个白色三角形, 则它们的数目之差为 $n=3 x-3 y=3(x-y)$, 为 3 的整数倍, 命题得证.", + "remark": "", + "figures": [ + "./images/volume14/figures/fig-c14a4.png" + ] +} \ No newline at end of file diff --git a/processed_dataset/proof/0773.json b/processed_dataset/proof/0773.json new file mode 100644 index 0000000000000000000000000000000000000000..48de80d2e830f248882984eee3283081bc9401dc --- /dev/null +++ b/processed_dataset/proof/0773.json @@ -0,0 +1,10 @@ +{ + "source_file": "./raw_volume-zh/volume14/exercise14.tex", + "problem_type": "proof", + "problem": "问题5. 有 9 个人,其中任意 3 个人中总有 2 个互相认识.\n证明: 必存在 4 人, 他们相互之间都认识.", + "solution": "将人对应成点, 两人间的关系对应成两点连线的颜色, 两人不相识对应为红色, 两人相识对应为蓝色, 于是原题变成如下染色问题:\n二染色 $K_9$, 设不存在红色三角形,证明必存在一个各边为蓝色的 $K_4$.\n考虑一点 $A$ 引出的 8 条线段,分以下 3 种情况:\n(1) 有 4 条 (或 4 条以上) 红色线段, 如图(). 现考虑 $B_1, B_2, B_3, B_4$ 间所连线段的颜色.\n由题意, 不存在红色三角形, 因此 $B_1, B_2, B_3, B_4$ 组成蓝色的 $K_4$, 原命题成立.\n(2). 有 6 条(或 6 条以上)蓝色线段.\n现考虑这 6 条蓝色线段除 $A$ 点外的 6 个端点.\n熟知它们组成的 $K_6$ 中必存在单色三角形, 由于不存在红色三角形, 因此必是蓝色三角形,此蓝色三角形加上 $A$ 点组成蓝色的 $K_4$.\n(3)恰有 3 条红色线段、 5 条蓝色线段,此时另外 8 个点中必存在一点,它引出的红色线段不是 3 条, 于是可用 (1) 或 (2) 去证明.\n否则, 每个点引出的红色线段都恰好 3 条,则红色线段总数为 $\\frac{3 \\times 9}{2}=13.5$, 这是不可能的.", + "remark": "", + "figures": [ + "./images/volume14/figures/fig-c14a5.png" + ] +} \ No newline at end of file diff --git a/processed_dataset/proof/0774.json b/processed_dataset/proof/0774.json new file mode 100644 index 0000000000000000000000000000000000000000..9536de3409bb831e1f51f6b9ed44179b79fb6ce6 --- /dev/null +++ b/processed_dataset/proof/0774.json @@ -0,0 +1,8 @@ +{ + "source_file": "./raw_volume-zh/volume14/exercise14.tex", + "problem_type": "proof", + "problem": "问题6. 平面上有 6 个点, 每三点的两两连线都组成一个不等边的三角形.\n求证: 一定可以找到两对三角形, 使每对三角形的公共边既是其中一个三角形的最长边, 又是另一个三角形的最短边.", + "solution": "记 6 个点为 $A_1, A_2, \\cdots, A_6$. 将 $A_i A_j(1 \\leqslant i0$, 且当 $a_{i j}>0$ 时, $r_i=c_j$.\n对 $1 \\leqslant i \\leqslant m, 1 \\leqslant j \\leqslant n$, 无论 $a_{i j}>0$ 还是 $a_{i j}=0$, 总有 $\\frac{a_{i j}}{r_i}=\\frac{a_{i j}}{c_j}$.\n用两种方法计算 $S=\\sum_{i=1}^m \\sum_{j=1}^n \\frac{a_{i j}}{r_i}$.\n一方面,有\n$$\nS=\\sum_{i=1}^m \\frac{1}{r_i} \\cdot \\sum_{j=1}^n a_{i j}=\\sum_{i=1}^m \\frac{1}{r_i} \\cdot r_i=m ;\n$$\n另一方面,有\n$$\nS=\\sum_{j=1}^n \\sum_{i=1}^m \\frac{a_{i j}}{c_j}=\\sum_{j=1}^n \\frac{1}{c_j} \\cdot \\sum_{i=1}^m a_{i j}=\\sum_{j=1}^n \\frac{1}{c_j} \\cdot c_j=n .\n$$\n所以 $m=n$.", + "remark": "", + "figures": [] +} \ No newline at end of file diff --git a/processed_dataset/proof/0784.json b/processed_dataset/proof/0784.json new file mode 100644 index 0000000000000000000000000000000000000000..a72c84a886b9506dd686a0944f22121dd7d4f5ae --- /dev/null +++ b/processed_dataset/proof/0784.json @@ -0,0 +1,8 @@ +{ + "source_file": "./raw_volume-zh/volume14/exercise16.tex", + "problem_type": "proof", + "problem": "问题6. 设 $x_1, x_2, \\cdots, x_n, y_1, y_2, \\cdots, y_n \\in \\mathbf{R}$, 矩阵 $A=\\left(a_{i j}\\right)_{1 \\leqslant i, j \\leqslant n}$ 满足\n$$\na_{i j}= \\begin{cases}1, & x_i+y_j \\geqslant 0, \\\\ 0, & x_i+y_j<0 .\\end{cases}\n$$\n证明: 若 $n \\times n$ 矩阵 $B$ 的元素为 0 或 1 , 使得 $B$ 的每一行和每一列的元素之和与 $A$ 的对应的和相等, 则 $B=A$.", + "solution": "设 $B=\\left(b_{i j}\\right)_{1 \\leqslant i, j \\leqslant n}$. 定义 $S=\\sum_{1 \\leqslant i, j \\leqslant n}\\left(x_i+y_j\\right)\\left(a_{i j}-b_{i j}\\right)$.\n一方面,有\n$$\nS=\\sum_{i=1}^n x_i\\left(\\sum_{j=1}^n a_{i j}-\\sum_{j=1}^n b_{i j}\\right)+\\sum_{j=1}^n y_j\\left(\\sum_{i=1}^n a_{i j}-\\sum_{i=1}^n b_{i j}\\right)=0 .\n$$\n另一方面, 对任意 $i, j\\left(1 \\leqslant i, j \\leqslant n\\right.$ ), 都有 $\\left(x_i+y_j\\right)\\left(a_{i j}-b_{i j}\\right) \\geqslant 0$ (事实上, 若 $x_i+y_j \\geqslant 0$, 则 $a_{i j}-b_{i j}=1-b_{i j} \\geqslant 0$; 若 $x_i+y_j<0$, 则 $a_{i j}- \\left.b_{i j}=-b_{i j} \\leqslant 0\\right)$.\n从而这些不等式只能同时取等号, 即 $\\left(x_i+y_j\\right)\\left(a_{i j}-b_{i j}\\right)=0$ 对所有 $i, j$ 成立.\n特别地, 若 $a_{i j}=0$, 则 $x_i+y_j<0$, 只能 $a_{i j}-b_{i j}=0$, 即 $b_{i j}=0$, 因此对所有 $i, j$, 总有 $a_{i j} \\geqslant b_{i j}$. 但 $B$ 的每一行元素之和与 $A$ 对应行的元素之和相等, 从而只能 $a_{i j}=b_{i j}$ 对所有 $i, j$ 成立, 即有 $B=A$.", + "remark": "", + "figures": [] +} \ No newline at end of file diff --git a/processed_dataset/proof/0785.json b/processed_dataset/proof/0785.json new file mode 100644 index 0000000000000000000000000000000000000000..144f739e90d3fcf0199beae4bb09875b9dfb1765 --- /dev/null +++ b/processed_dataset/proof/0785.json @@ -0,0 +1,8 @@ +{ + "source_file": "./raw_volume-zh/volume14/exercise17.tex", + "problem_type": "proof", + "problem": "问题4. 设 $\\pi(n)$ 表示不大于 $n$ 的素数的个数.\n求证: 对任意正整数 $n$ 和非负整数 $k \\leqslant \\pi(n)$, 总存在 $n$ 个连续正整数, 其中恰含有 $k$ 个素数.", + "solution": "对正整数 $m$, 定义 $f(m)$ 为 $m, m+1, \\cdots, m+n-1$ 这 $n$ 个连续正整数中素数的个数.\n显然 $f(1)=\\pi(n)$.\n取 $(n+1) !+2,(n+1) !+3, \\cdots,(n+1) !+(n+1)$ 这 $n$ 个连续正整数, 它们都是合数, 即 $f((n+1) !+2)=0$. 所以\n$$\nf((n+1) !+2) \\leqslant k \\leqslant \\pi(n)=f(1) . \\label{eq1}\n$$\n另一方面,考虑 $f(m+1)$ 与 $f(m)$ 的关系:\n当 $m, m+n$ 都为素数或都为合数时,有 $f(m+1)=f(m)$;\n当 $m$ 为素数且 $m+n$ 为合数时,有 $f(m+1)=f(m)-1$;\n当 $m$ 为合数且 $m+n$ 为素数时,有 $f(m+1)=f(m)+1$.\n总之 $|f(m+1)-f(m)| \\leqslant 1$, 结合 \\ref{eq1} 式知必有某个 $i \\in\\{1,2, \\cdots,(n+1) !+2\\}$ 使得 $f(i)=k$, 即连续 $n$ 个正整数 $i, i+1, \\cdots, i+n-1$ 中恰含有 $k$ 个素数.", + "remark": "", + "figures": [] +} \ No newline at end of file diff --git a/processed_dataset/proof/0786.json b/processed_dataset/proof/0786.json new file mode 100644 index 0000000000000000000000000000000000000000..768ff8441cd63ff58799f2f7b020f1482744529c --- /dev/null +++ b/processed_dataset/proof/0786.json @@ -0,0 +1,8 @@ +{ + "source_file": "./raw_volume-zh/volume14/exercise17.tex", + "problem_type": "proof", + "problem": "问题5. 已知条件组 $\\left\\{\\begin{array}{l}x_1+x_2+\\cdots+x_n=m, \\\\ x_1^2+x_2^2+\\cdots+x_n^2-\\frac{m^2}{n}<2 .\\end{array}\\right.$\n(1) 求所有正整数 $n$, 使条件组对一切正整数 $m$ 都有整数解 $\\left(x_1, x_2, \\cdots\\right.$, $\\left.x_n\\right)$;\n(2) 求一切正整数组 $(m, n)$, 使得条件组存在整数解 $\\left(x_1, x_2, \\cdots, x_n\\right)$.", + "solution": "引理: 令 $S=\\sum_{i=1}^n x_i^2-\\frac{m^2}{n}$. 当 $S$ 取最小值时, 对任意 $i, j$, 都有 $\\left|x_i-x_j\\right| \\leqslant 1$.\n证明: 假定存在情形 $x_1 \\geqslant x_2+2$. 令 $x_1^{\\prime}=x_1-1, x_2^{\\prime}=x_2+1, x_3^{\\prime}=x_3$,\n$\\cdots, x_n^{\\prime}=x_n, m^{\\prime}=\\sum_{i=1}^n x_i^{\\prime}, S^{\\prime}=\\sum_{i=1}^n x_i^{\\prime 2}-\\frac{m^{\\prime 2}}{n}$, 则 $m^{\\prime}=m, S^{\\prime}-S=\\sum_{i=1}^n\\left(x_i^{\\prime 2}-\\right. \\left.x_i^2\\right)=x_1^{\\prime 2}-x_1^2+x_2^{\\prime 2}-x_2^2=\\left(x_1-1\\right)^2-x_1^2+\\left(x_2+1\\right)^2-x_2^2=2-2 x_1+ 2 x_2<0$. 故在 $m$ 不变的前提下, 通过调整可使 $S$ 取到更小的 $S^{\\prime}$, 又这样的调整只能做有限次, 故 $S$ 可取最小值, 且此时任何 $x_i, x_j$ 之差不超过 1 . 引理证毕.\n下面对 (1)、(2)一并解决.\n设 $m=n k-t, k \\in \\mathbf{N}^*, t \\in\\{0,1, \\cdots, n-1\\}$.\n设 $x_1=x_2=\\cdots=x_t=k-1, x_{t+1}=x_{t+2}=\\cdots=x_n=k$, 它们的和恰为 $m$, 且由引理知, 已使 $S$ 尽可能小 (由于其他形式的一切解 $\\left(x_1, x_2, \\cdots, x_n\\right)$ 都对应一组这样的调整后的解, 所以此设法不失一般性). 此时\n$$\n\\begin{gathered}\nS==\\sum_{i=1}^n x_i^2-\\frac{m^2}{n}=\\sum_{i=1}^n x_i^2-\\frac{1}{n} \\sum_{i=1}^n x_i^2-\\frac{2}{n} \\sum_{1 \\leqslant i0$.\n当 $n \\leqslant 7$ 时, 对一切 $t$, 都成立 $f(t)=\\left(t-\\frac{n}{2}\\right)^2+\\frac{n(8-n)}{4}>0$, 即对一切正整数 $m$, 条件组都存在整数解.\n当 $n \\geqslant 8$ 时, $f(4)=16-2 n \\leqslant 0$, 故对 $m=n k-4$, 无整数解满足.\n这样,第 (1) 问的结果为 $n=1,2, \\cdots, 7$.\n此外, 对 $n \\geqslant 8$ 寻求别的正整数组 $(m, n)$.\n经算得: $f(0)=2 n>0, f(1)=f(n-1)=n+1>0, f(2)=f(n-$ 2) $=4>0$.\n当 $n=8$ 时, $f(3)=f(5)=1>0$, 故可取 $t=0,1,2,3,5,6,7$.\n当 $n \\geqslant 9$ 时, $f(3)=f(n-3)=9-n \\leqslant 0$, 由二次函数性质知, 当 $t \\in(3$, $n-3)$ 时, $f(t)<\\max \\{f(3), f(n-3)\\} \\leqslant 0$, 故恰可取 $t=0,1,2, n-2$, $n-1$. 结合所设的 $m=n k-t, k=1,2, \\cdots, t \\in\\{0,1, \\cdots, n-1\\}$ 可知, 第 (2) 问的一切正整数组 $(m, n)$ 有如下 3 类:\n(1) $(a, 1),(a, 2), \\cdots,(a, 7)$, 其中 $a$ 为任意正整数;\n(2) $(b k-t, b)$, 其中 $b \\geqslant 8, k$ 为任意正整数, $t=0,1,2, b-2, b-1$;\n(3) $(8 k-3,8),(8 k-5,8)$, 其中 $k$ 为任意正整数.\n而 \" $x_1=x_2=\\cdots=x_t=k-1, x_{t+1}=x_{t+2}=\\cdots=x_n=k$ \" 保证了每组 $(m, n)$ 都对应了一组具体的解 $\\left(x_1, x_2, \\cdots, x_n\\right)$.", + "remark": "", + "figures": [] +} \ No newline at end of file diff --git a/processed_dataset/proof/0787.json b/processed_dataset/proof/0787.json new file mode 100644 index 0000000000000000000000000000000000000000..1d3ab3b1c552c73d116ca56e7b7fb5f41c14a0ed --- /dev/null +++ b/processed_dataset/proof/0787.json @@ -0,0 +1,8 @@ +{ + "source_file": "./raw_volume-zh/volume14/exercise17.tex", + "problem_type": "proof", + "problem": "问题7. 设正实数 $a, b, c, d$ 满足 $a b c d=1$, 求证:\n$$\n\\frac{1}{a}+\\frac{1}{b}+\\frac{1}{c}+\\frac{1}{d}+\\frac{9}{a+b+c+d} \\geqslant \\frac{25}{4} .\n$$", + "solution": "不妨设 $a \\leqslant b \\leqslant c \\leqslant d$, 并记\n$$\nf(a, b, c, d)=\\frac{1}{a}+\\frac{1}{b}+\\frac{1}{c}+\\frac{1}{d}+\\frac{9}{a+b+c+d} .\n$$\n先往证: $\\quad f(a, b, c, d) \\geqslant f(\\sqrt{a c}, b, \\sqrt{a c}, d)$. \\label{eq1}\n事实上,上式等价于\n$$\n\\begin{aligned}\n& \\frac{1}{a}+\\frac{1}{c}+\\frac{9}{a+b+c+d} \\geqslant \\frac{1}{\\sqrt{a c}}+\\frac{1}{\\sqrt{a c}}+\\frac{9}{2 \\sqrt{a c}+b+d} \\\\\n\\Leftrightarrow & \\frac{(\\sqrt{a}-\\sqrt{c})^2}{a c} \\geqslant \\frac{9(\\sqrt{a}-\\sqrt{c})^2}{(a+b+c+d)(2 \\sqrt{a c}+b+d)} \\\\\n\\Leftarrow & \\left.(a+b+c+d)(2 \\sqrt{a c}+b+d) \\geqslant 9 a c \\text { (因为 }(\\sqrt{a}-\\sqrt{c})^2 \\geqslant 0\\right) \\\\\n\\Leftarrow & \\left(a+c+\\frac{2}{\\sqrt{a c}}\\right)\\left(2 \\sqrt{a c}+\\frac{2}{\\sqrt{a c}}\\right) \\geqslant 9 a c \\text { (因为 } b+d \\geqslant 2 \\sqrt{b d}=\\frac{2}{\\sqrt{a c}} \\text { ) } \\label{eq2}\n\\end{aligned}\n$$\n而 $1=a b c d \\geqslant a \\cdot a \\cdot c \\cdot c \\Rightarrow a c \\leqslant 1 \\Rightarrow \\frac{2}{\\sqrt{a c}} \\geqslant 2 \\sqrt{a c}$. 且 $a+c \\geqslant 2 \\sqrt{a c}$, 故\n式\\ref{eq2} 左边 $\\geqslant\\left(2 \\sqrt{a c}+\\frac{2}{\\sqrt{a c}}\\right)\\left(2 \\sqrt{a c}+\\frac{2}{\\sqrt{a c}}\\right) \\geqslant 4 \\sqrt{a c} \\cdot 4 \\sqrt{a c}= 16 a c>9 a c=$ 式\\ref{eq2} 右边.\n所以 式\\ref{eq1} 成立.\n式\\ref{eq1} 说明, $f(a, b, c, d)$ (其中 $a \\leqslant b \\leqslant c \\leqslant d$ ) 的最小值 (或极小值) 总是在 $a=c$, 即 $a=b=c$ 时取得.\n欲得到该四元函数的下界, 我们就可不妨设 $(a, b$, $c, d)=\\left(\\frac{1}{t}, \\frac{1}{t}, \\frac{1}{t}, t^3\\right)$, 这里 $t \\geqslant 1$; 这也说明了只需证明对 $\\forall t \\geqslant 1$, 总有\n$$\nf\\left(\\frac{1}{t}, \\frac{1}{t}, \\frac{1}{t}, t^3\\right) \\geqslant \\frac{25}{4}, \\label{eq3}\n$$\n就证明了原不等式成立.\n$$\n\\begin{aligned}\n& f\\left(\\frac{1}{t}, \\frac{1}{t}, \\frac{1}{t}, t^3\\right) \\geqslant \\frac{25}{4} \\\\\n& \\Leftrightarrow 3 t+\\frac{1}{t^3}+\\frac{9}{t^3+\\frac{3}{t}} \\geqslant \\frac{25}{4} \\\\\n& \\Leftrightarrow 12 t^8-25 t^7+76 t^4-75 t^3+12 \\geqslant 0 \\\\\n& \\Leftrightarrow(t-1)^2\\left(12 t^6-t^5-14 t^4-27 t^3+36 t^2+24 t+12\\right) \\geqslant 0 \\\\\n& \\Leftrightarrow 12 t^6-t^5-14 t^4-27 t^3+36 t^2+24 t+12 \\geqslant 0 \\\\\n& \\Leftrightarrow(t-1)\\left(12 t^5+11 t^4-3 t^3-30 t^2+6 t+30\\right)+42 \\geqslant 0 . \\label{eq4} \\\\ \n& \\text { 而 } t \\geqslant 1,12 t^5+6 t \\geqslant 2 \\sqrt{12 t^5 \\cdot 6 t}=12 \\sqrt{2} t^3>3 t^3, \\\\\n& \\qquad 11 t^4+30 \\geqslant 2 \\sqrt{11 t^4 \\cdot 30}=2 \\sqrt{33} \\overline{0} t^2>30 t^2 .\n\\end{aligned}\n$$\n而 $t \\geqslant 1,12 t^5+6 t \\geqslant 2 \\sqrt{12 t^5 \\cdot 6 t}=12 \\sqrt{2} t^3>3 t^3$,\n$$\n11 t^4+30 \\geqslant 2 \\sqrt{11 t^4 \\cdot 30}=2 \\sqrt{33} \\overline{0} t^2>30 t^2 .\n$$\n故 $(t-1)\\left(12 t^5+11 t^4-3 t^3-30 t^2+6 t+30\\right)+42>0$, 式\\ref{eq4} 成立.\n至此, 式\\ref{eq3} 成立, 原不等式得证.", + "remark": "", + "figures": [] +} \ No newline at end of file diff --git a/processed_dataset/proof/0788.json b/processed_dataset/proof/0788.json new file mode 100644 index 0000000000000000000000000000000000000000..6011db2d1cbf2d7a953df5609ec676bac37cffcf --- /dev/null +++ b/processed_dataset/proof/0788.json @@ -0,0 +1,8 @@ +{ + "source_file": "./raw_volume-zh/volume14/exercise18.tex", + "problem_type": "proof", + "problem": "问题1. 试构造恒等式证明下述不定方程均有无穷多组正整数解:\n(1) $x^2+y^2+1=z^2$;\n(2) $2 x^2+2 y^2+1=z^2$;\n(3) $x^2+y^2+1=2 z^2$.", + "solution": "(1) $\\left(2 k^2\\right)^2+(2 k)^2+1=\\left(2 k^2+1\\right)^2$;\n(2) $2\\left(k^2+k\\right)^2+2\\left(k^2-k\\right)^2+1=\\left(2 k^2+1\\right)^2$;\n(3) $\\left(k^2\\right)^2+\\left((k+1)^2\\right)^2+1=2\\left(k^2+k+1\\right)^2$.", + "remark": "", + "figures": [] +} \ No newline at end of file diff --git a/processed_dataset/proof/0789.json b/processed_dataset/proof/0789.json new file mode 100644 index 0000000000000000000000000000000000000000..18559839fd4e7e658ab6ec5235855aaf06f7793f --- /dev/null +++ b/processed_dataset/proof/0789.json @@ -0,0 +1,8 @@ +{ + "source_file": "./raw_volume-zh/volume14/exercise18.tex", + "problem_type": "proof", + "problem": "问题9. 证明: 存在无穷多组正整数 $(m, n)$, 使得 $\\frac{n+1}{m}+\\frac{m+1}{n}$ 是一个整数.", + "solution": "首先, $m=1, n=2$ 使得 $\\frac{n+1}{m}+\\frac{m+-1}{n}$ 是一个整数.\n设正整数 $m, n(mn$,于是\n$$\nt n=\\frac{n(n+1)}{m}+m+1=s+\\frac{n(n+1)}{s}+1,\n$$\n所以\n$$\nt=\\frac{s+1}{n}+\\frac{n+1}{s} .\n$$\n即若 $(m, n)$ 是满足题意的数对, 则 $(n, s)$ 也是满足题意的数对.\n且 $s> n>m$.\n故存在无穷多组正整数 $(m, n)$, 使得 $\\frac{n+1}{m}+\\frac{m+1}{n}$ 是一个整数.", + "remark": "注:试比较第 6 节极端原理中的例 8 .", + "figures": [] +} \ No newline at end of file diff --git a/processed_dataset/proof/0790.json b/processed_dataset/proof/0790.json new file mode 100644 index 0000000000000000000000000000000000000000..561068de25ea3fa197d53f08cf0f06d8c5fe2c62 --- /dev/null +++ b/processed_dataset/proof/0790.json @@ -0,0 +1,8 @@ +{ + "source_file": "./raw_volume-zh/volume14/exercise18.tex", + "problem_type": "proof", + "problem": "问题11. 给定整数 $n \\geqslant 3$. 用 $f(x)$ 表示有限数集 $X$ 中元素的算术平均.\n(1) 证明: 存在 $n$ 元正整数集 $S_1$, 满足对任意两个不同的非空集 $A, B \\subseteq S_1, f(A)$ 和 $f(B)$ 是两个不相等的正整数;\n(2) 若集合 $S_1$ 满足 (1) 中的条件, 证明: 对给定正整数 $K>\\max _{A_1 \\subseteq S_1} f\\left(A_1\\right)$ 及任意 $x \\in \\mathbf{N}^*$, 集合 $S_2=\\left\\{K ! x \\alpha+1 \\mid \\alpha \\in S_1\\right\\}$ 满足:对任意两个不同的非空集 $A, B \\subseteq S_2, f(A)$ 与 $f(B)$ 是两个互素的整数;\n(3)证明: 在 (2) 的基础上,可适当选择一个正整数 $x$, 使 $S_2$ 进一步满足: 对每个非空集 $A \\subseteq S_2, f(A)$ 为合数.", + "solution": "(1) 取定整数 $q>n$, 取 $S_1=\\left\\{n ! q, n ! q^2, \\cdots, n ! q^n\\right\\}$, 则对任意两个不同的非空集 $A, B \\subseteq S_1, f(A)$ 和 $f(B)$ 显然是正整数.\n假设此时 $f(A)= f(B)$, 那么\n$$\n|B| \\sum_{n ! q^i \\in A} q^i=|A| \\sum_{n ! q^j \\in B} q^j .\n$$\n因为 $q>n \\geqslant \\max \\{|A|,|B|\\}$, 故上述等式中的正整数 $|B| \\sum_{n ! q^i \\in A} q^i$ 与 $|A| \\sum_{n ! q^i \\in B} q^j$ 的 $q$ 进制表示分别是 $\\sum_{n ! q^i \\in A}|B| q^i$ 与 $\\sum_{n ! q^j \\in B}|A| q^j$, 从而它们的形式应当完全相同,由此可得 $A=B$,矛盾!\n所以对 $S_1$ 的任意两个不同的非空子集 $A, B, f(A)$ 和 $f(B)$ 是两个不相等的正整数.\n(2)由 $S_2$ 的定义易知,对任意两个不同的非空集 $A, B \\subseteq S_2$, 有两个非空集 $A_1, B_1 \\subseteq S_1$ 满足 $\\left|A_1\\right|=|A|,\\left|B_1\\right|=|B|$, 且\n$$\nf(A)=K ! x f\\left(A_1\\right)+1, f(B)=K ! x f\\left(B_1\\right)+1 . \\label{eq1}\n$$\n显然 $f(A)$ 与 $f(B)$ 都是正整数.\n若正整数 $d$ 是 $f(A)$ 与 $f(B)$ 的一个公约数, 则 $d \\mid f(A) f\\left(B_1\\right)- f(B) f\\left(A_1\\right)$, 故由 式\\ref{eq1} 可知 $d \\mid f\\left(A_1\\right)-f\\left(B_1\\right)$, 但由 $K$ 的选取及 $S_1$ 的构作可知, $\\left|f\\left(A_1\\right)-f\\left(B_1\\right)\\right|$ 是小于 $K$ 的非零整数, 故为 $K$ ! 的约数, 从而 $d \\mid K !$ ! 再结合 $d \\mid f(A)$ 及 式\\ref{eq1} 可知 $d=1$, 因此 $f(A)$ 与 $f(B)$ 互素.\n(3)显然可选择 $2^n-1$ 个大于 $K$ 且互不相同的素数 $p_1, p_2, \\cdots, p_{2^n-1}$. 将 $S_1$ 中一切非空子集的元素平均值分别记为 $\\alpha_1, \\alpha_2, \\cdots, \\alpha_2{ }^n-1$, 则\n$$\n\\left(p_i, K ! \\alpha_i\\right)=1,1 \\leqslant i \\leqslant 2^n-1,\n$$\n且\n$$\n\\left(p_i^2, p_j^2\\right)=1,1 \\leqslant i), 若在某一时刻与未染色的 $P$ 格相邻的四格 $A, B, C, D$ 中已有 $k$ 个为黑色,那么添上 $P$ 后黑色区域少去 $k$ 条边界, 同时增加 $4-k$ 条边界, 但 $P$ 能染色的前提是 $k \\geqslant 2$, 从而 $4-k \\leqslant k$,这样 $P$ 格染色后黑色区域周长之和不增.\n假设最终所有方格都染黑, 那么黑色区域周长应为 $4 n> 4(n-1)$,矛盾!所以必定无法染黑所有的方格.", + "remark": "", + "figures": [ + "./images/volume14/figures/fig-c19a6.png" + ] +} \ No newline at end of file diff --git a/processed_dataset/proof/0793.json b/processed_dataset/proof/0793.json new file mode 100644 index 0000000000000000000000000000000000000000..a8fc73ccb0fdcf1d79cec36a007c388074283b57 --- /dev/null +++ b/processed_dataset/proof/0793.json @@ -0,0 +1,8 @@ +{ + "source_file": "./raw_volume-zh/volume14/exercise19.tex", + "problem_type": "proof", + "problem": "问题7. 在一块已写有一个正整数的黑板上进行如下操作: 若 $x$ 已写在黑板上, 则可以在黑板上写上数 $2 x+1$ 或 $\\frac{x}{x+2}$. 已知在某个时刻黑板上写有数 2008. 证明: 黑板上原有的数是 2008.", + "solution": "显然黑板上只能出现正有理数.\n设某一时刻黑板上有有理数 $x=\\frac{p}{q}$, $(p, q)=1$, 则由 $x$ 可得到 $2 x+1=\\frac{2 p+q}{q}$ 或 $\\frac{x}{x+2}=\\frac{p}{p+2 q}$.\n注意到\n$$\n(2 p+q, q)=(2 p, q) \\leqslant 2(p, q)=2,(p, p+2 q)=(p, 2 q) \\leqslant 2(p, q)=2,\n$$\n而\n$$\n(2 p+q)+q=p+(p+2 q)=2(p+q),\n$$\n故由 $x$ 得到的最简分数的分子与分母之和为 $p+q$ 或 $2(p+q)$, 即分子分母之和或者不变或者变为原来的两倍.\n由于后来出现的 2008 的分子分母之和为 2009 是一个奇数,故从初始数到 2008 的变化过程中没有出现过加倍的情况.\n因此初始数的分子分母之和就是 2009. 再考虑到初始数是一个正整数, 故它为 2008 .", + "remark": "", + "figures": [] +} \ No newline at end of file diff --git a/processed_dataset/proof/0794.json b/processed_dataset/proof/0794.json new file mode 100644 index 0000000000000000000000000000000000000000..1b3f1d0069b90db2c73e8abae0a9f13f131aea69 --- /dev/null +++ b/processed_dataset/proof/0794.json @@ -0,0 +1,8 @@ +{ + "source_file": "./raw_volume-zh/volume14/exercise19.tex", + "problem_type": "proof", + "problem": "问题8. 桌上有 2009 枚硬币, 将其一面染上白色, 另一面染上黑色.\n起初, 将所有硬币排成一排,其中一枚硬币黑面朝上,其余 2008 枚硬币均白面朝上.\n按如下规则操作: 每一次操作中, 选择一枚黑面朝上的硬币, 并翻转与其相邻的两枚硬币; 若所选的黑面朝上的硬币在这一排硬币的两端,则只需翻转与端点相邻的那一枚硬币.\n请找出初始状态时那枚黑面朝上的硬币的所有可能位置, 使得经过若干次操作后, 所有的硬币均黑面朝上.", + "solution": "将给定的这一排硬币从左至右依次编号为 $1,2, \\cdots, 2009$.\n在任何一步操作之前, 设所有黑面朝上的硬币所在位置从左至右分别对应编号 $a_1, a_2, \\cdots, a_k$ (其中 $k$ 亦可能随操作而变化).\n构造一个量 $S=\\sum_{i=1}^k(-1)^{i+1} a_i$.\n设此时对编号 $a_j$ 的硬币进行操作.\n考虑操作后相应于 $S$ 的量 $S^{\\prime}$.\n(1)硬币 $a_j$ 不是第一枚或最后一枚, 即 $a_j \\neq 1,2009$. 我们来验证 $S^{\\prime}=$ S:\n若与 $a_j$ 相邻的两枚硬币同色面朝上, 可不妨假定操作前第 $a_j-1, a_j+1$ 枚硬币均白面朝上,则操作后它们黑面朝上,其余硬币无变化, 故\n$$\nS^{\\prime}-S=\\left((-1)^{j+1}\\left(a_j-1\\right)+(-1)^{j+2} a_j+(-1)^{j+3}\\left(a_j+1\\right)\\right)-(-1)^{j+1} a_j=0 .\n$$\n若第 $a_j-1, a_j+1$ 枚硬币异色面朝上,可不妨假定操作前第 $a_j-1$ 枚硬币白面朝上,第 $a_j+1$ 枚硬币黑面朝上 (即 $a_{j+1}=a_j+1$ ), 类似地有\n$$\nS^{\\prime}-S=\\left((-1)^{j+1} a_j+(-1)^{j+2}\\left(a_j+1\\right)\\right)-\\left((-1)^{j+1}\\left(a_j-1\\right)+(-1)^{j+2} a_j\\right)=0 .\n$$\n(2)硬币 $a_j$ 是最后一枚, 即 $j=k, a_k=2009$.\n我们引进编号为 $a_{k+1}=2010$ 的\"虚拟硬币\",操作前它黑面朝上.\n记\n$$\nS_1=\\sum_{i=1}^{k+1}(-1)^{i+1} a_i=S+(-1)^{k+2} 2010 .\n$$\n由前一种情形的讨论知 $S_1^{\\prime}=S_1$, 而操作后第 2010 枚硬币白面朝上,所以\n$$\nS^{\\prime}=S_1^{\\prime}=S_1=S+(-1)^{k+2} 2010 .\n$$\n(3)硬币 $a_j$ 就是第一枚,即 $j=1, a_1=1$.\n若操作前 $a_2=2$, 则\n$$\nS^{\\prime}=1-\\sum_{i=3}^k(-1)^{i+1} a_i=-a_1+a_2-\\sum_{i=3}^k(-1)^{i+1} a_i=-S ;\n$$\n若操作前 $a_2>2$, 则\n$$\nS^{\\prime}=1-2+\\sum_{i=3}^{k+1}(-1)^{i+1} a_{i-1}=-a_1+\\sum_{i=2}^k(-1)^{i+2} a_i=-S .\n$$\n综合 (1)、(2)、(3) 可知, 操作前后 $S, S^{\\prime}$ 总满足 $S^{\\prime} \\equiv \\pm S(\\bmod 2010)$. 若要使所有硬币均黑面朝上, 则最终的 $S=\\sum_{i=1}^{2009}(-1)^{i+1} i=1005$, 故仅有一种可能的初始状态, 即起初第 1005 个位置上的硬币黑面朝上.\n最后构造一种操作方式使上述初始状态最终达到所有硬币黑面朝上的状态.\n记第 1005 个位置为 $m$. 假如黑面朝上的硬币刚好出现在 $m-k$ 到 $m+k$ 位置上 $(k \\in \\mathbf{N})$, 若 $k=2 l(l \\in \\mathbf{N})$, 则依次对如下位置的硬币进行操作:\n$$\nm-2 l, m+2 l, m-(2 l-2), m+(2 l-2), \\cdots, m-2, m+2, m ;\n$$\n若 $k=2 l+1(l \\in \\mathbf{N})$, 则依次对如下位置的硬币进行操作 :\n$$\nm-(2 l-1), m+(2 l-1), m-(2 l-3), m+(2 l-3), \\cdots, m-1, m+1 \\text {. }\n$$\n两种情况下均可使黑面朝上的硬币刚好出现在 $m-k-1$ 到 $m+k+1$ 位置上.\n由于初始状态就是 $k=0$ 的情形, 因此根据数学归纳法可知 $k=1004$ 的情形可以通过适当操作而得到, 此时所有硬币均黑面朝上.\n从而满足题意的位置就是第 1005 个位置.", + "remark": "", + "figures": [] +} \ No newline at end of file diff --git a/processed_dataset/proof/0795.json b/processed_dataset/proof/0795.json new file mode 100644 index 0000000000000000000000000000000000000000..795cd941e03df5fc274561cd54a8a97d8513294d --- /dev/null +++ b/processed_dataset/proof/0795.json @@ -0,0 +1,8 @@ +{ + "source_file": "./raw_volume-zh/volume14/exercise2.tex", + "problem_type": "proof", + "problem": "问题1. 证明: 对任何实数 $x, y, z$,下述三个不等式不可能同时成立:\n$$\n|x|<|y-z|,|y|<|z-x|,|z|<|x-y| \\text {. }\n$$", + "solution": "用反证法.\n假设 $|x|<|y-z|,|y|<|z-x|,|z|<|x-y|$ 同时成立, 不妨设其中 $x \\geqslant y \\geqslant z$, 则有 $|x|+|z|<|y-z|+|x-y|= (y-z)+(x-y)=x-z$, 矛盾! 故假设不成立.\n证毕.", + "remark": "", + "figures": [] +} \ No newline at end of file diff --git a/processed_dataset/proof/0796.json b/processed_dataset/proof/0796.json new file mode 100644 index 0000000000000000000000000000000000000000..a78f29a25f7175eea07ae596801aebf7b033ac25 --- /dev/null +++ b/processed_dataset/proof/0796.json @@ -0,0 +1,8 @@ +{ + "source_file": "./raw_volume-zh/volume14/exercise2.tex", + "problem_type": "proof", + "problem": "问题2. 已知点 $E, F, G, H$ 分别在单位正方形 $A B C D$ 的四条边上, 求证: 在四边形 $E F G H$ 中至少有一条边的长度不小于 $\\frac{\\sqrt{2}}{2}$.", + "solution": "假设四边形 $E F G H$ 每条边的长度都小于 $\\frac{\\sqrt{2}}{2}$, 则\n$$\nE F^2+F G^2<\\left(\\frac{\\sqrt{2}}{2}\\right)^2+\\left(\\frac{\\sqrt{2}}{2}\\right)^2=1 .\n$$\n又四边形 $E F G H$ 的四个内角中, 至少有一个内角不大于 $90^{\\circ}$ (否则, 四边形内角和将大于 $360^{\\circ}$ ), 因此, 不妨设 $\\angle E F G \\leqslant 90^{\\circ}$, 则 $E G^2 \\leqslant E F^2+E G^2$, 所以 $E G^2<1, E G<1$. 但在正方形 $A B C D$ 中, $A B / / C D$, 且 $A B$ 与 $C D$ 间距离为 1 , 所以 $E G \\geqslant 1$, 与 $E G<1$ 矛盾.", + "remark": "", + "figures": [] +} \ No newline at end of file diff --git a/processed_dataset/proof/0797.json b/processed_dataset/proof/0797.json new file mode 100644 index 0000000000000000000000000000000000000000..eb56ef1cd49f1a89a0de7040c3306d4a13fd7ad7 --- /dev/null +++ b/processed_dataset/proof/0797.json @@ -0,0 +1,8 @@ +{ + "source_file": "./raw_volume-zh/volume14/exercise2.tex", + "problem_type": "proof", + "problem": "问题4. 已知 $a, b, c$ 是实数, 且 $a>2000$, 证明: 至多存在两个整数 $x$, 使得\n$$\n\\left|a x^2+b x+c\\right| \\leqslant 1000 .\n$$", + "solution": "用反证法.\n假设存在三个不同的正整数 $x_1, x_2, x_3$, 使得\n$$\n\\left|a x_i^2+b x_i+c\\right| \\leqslant 1000,\n$$\n令 $f(x)=a x^2+b x+c$, 则 $x_1, x_2, x_3$ 中至少有两个在对称轴 $x=-\\frac{b}{2 a}$ 的一侧 (包括对称轴上), 不妨设 $x_1>x_2 \\geqslant-\\frac{b}{2 a}$, 则 $2 a x+b \\geqslant 0$. 因为\n$$\na x_1^2+b x_1+c \\leqslant 1000,-a x_2^2-b x_2-c \\leqslant 1000,\n$$\n所以\n$$\n\\left(x_1-x_2\\right)\\left[a\\left(x_1+x_2\\right)+b\\right] \\leqslant 2000 .\n$$\n又 $x_1, x_2$ 是整数, 所以 $x_1 \\geqslant x_2+1$, 于是\n$$\na\\left(x_1+x_2\\right)+b \\geqslant a\\left(x_2+1+x_2\\right)+b=a+2 a x_2+b \\geqslant a>2000,\n$$\n故\n$$\n\\left(x_1-x_2\\right)\\left[a\\left(x_1+x_2\\right)+b\\right]>2000 \\text {. }\n$$\n(1)与(2)矛盾, 从而命题得证.", + "remark": "", + "figures": [] +} \ No newline at end of file diff --git a/processed_dataset/proof/0798.json b/processed_dataset/proof/0798.json new file mode 100644 index 0000000000000000000000000000000000000000..cecb6850099fdf02f03bef39d1f00acac8331416 --- /dev/null +++ b/processed_dataset/proof/0798.json @@ -0,0 +1,8 @@ +{ + "source_file": "./raw_volume-zh/volume14/exercise2.tex", + "problem_type": "proof", + "problem": "问题5. 是否存在三边长都为整数的三角形, 满足以下条件: 最短边长为 2007 , 且最大的角等于最小角的两倍?", + "solution": "不存在这样的三角形,证明如下:\n不妨设 $\\angle A \\leqslant \\angle B \\leqslant \\angle C$, 则 $C=2 A$ 且 $a=2007$. 作 $\\angle A C B$ 的内角平分线 $C D$, 则 $\\angle B C D=\\angle A$, 从而 $\\triangle C D B \\backsim \\triangle A C B$. 所以,\n$$\n\\frac{C B}{A B}=\\frac{B D}{B C}=\\frac{C D}{A C}=\\frac{B D+C D}{B C+A C}=\\frac{B D+A D}{B C+A C}=\\frac{A B}{B C+A C} .\n$$\n即 $c^2=a(a+b)=2007(2007+b)$, 这里 $2007 \\leqslant b \\leqslant c<2007+b$.\n由 $a, b, c$ 都是正整数知 $2007 \\mid c^2$, 故 $3|c, 223| c$, 可设 $c=669 m$, 则 $223 m^2=2007+b$, 即 $b=223 m^2-2007$, 结合 $2007 \\leqslant b$, 可得 $m \\geqslant 5$.\n另一方面, 由于 $c \\geqslant b$, 得 $669 m \\geqslant 223 m^2-2007$, 这要求 $m<5$,矛盾! 因此, 满足条件的三角形不存在.", + "remark": "", + "figures": [] +} \ No newline at end of file diff --git a/processed_dataset/proof/0799.json b/processed_dataset/proof/0799.json new file mode 100644 index 0000000000000000000000000000000000000000..8dd68a3d916e16f6c0e2344962738084d4ce9785 --- /dev/null +++ b/processed_dataset/proof/0799.json @@ -0,0 +1,8 @@ +{ + "source_file": "./raw_volume-zh/volume14/exercise2.tex", + "problem_type": "proof", + "problem": "问题6. 知集合 $A$ 由全体正整数的倒数组成.\n是否存在无限个 $A$ 中的数 $a_1$, $a_2, a_3, \\cdots$ (不必不同), 使得对任何 $i, j \\in \\mathbf{N}^*$, 有 $\\frac{a_i}{i}+\\frac{a_j}{j} \\in A$ ? 证明你的结论.", + "solution": "结论是否定的.\n下用反证法证明之:\n假设存在无限个 $A$ 中的数 $a_1, a_2, a_3, \\cdots$, 满足对任何 $i, j \\in \\mathbf{N}^*$, 有 $\\frac{a_i}{i}+\\frac{a_j}{j} \\in A$, 显然 $a_1 \\neq 1$ (否则, 若 $a_1=1$, 令 $i=j=1$, 则推出 $2 \\in A$, 矛盾).\n于是, 设 $a_1=\\frac{1}{m}, m \\geqslant 2, m \\in \\mathbf{N}^*$.\n此时, 令 $i=1, j=m^2$, 则\n$$\n\\frac{a_i}{i}+\\frac{a_j}{j}=\\frac{1}{m}+\\frac{a_m^2}{m^2} \\leqslant \\frac{1}{m}+\\frac{1}{m^2}<\\frac{1}{m-1},\n$$\n故 $\\frac{1}{m}<\\frac{a_i}{i}+\\frac{a_j}{j}<\\frac{1}{m-1}$, 与 $\\frac{a_i}{i}+\\frac{a_j}{j} \\in A$ 矛盾!\n所以不存在满足条件的无限个数.", + "remark": "", + "figures": [] +} \ No newline at end of file diff --git a/processed_dataset/proof/0800.json b/processed_dataset/proof/0800.json new file mode 100644 index 0000000000000000000000000000000000000000..58b4429aaee6dc9f1767f1ee47d31006d859a45f --- /dev/null +++ b/processed_dataset/proof/0800.json @@ -0,0 +1,8 @@ +{ + "source_file": "./raw_volume-zh/volume14/exercise2.tex", + "problem_type": "proof", + "problem": "问题7. 在 $n$ 个元素组成的集合中取 $n+1$ 个两两不同的 3 元子集.\n证明: 其中必有两个子集, 它们恰有一个公共元.", + "solution": "假设结论不真.\n则对所给的 3 元子集中的任意两个 $A, B$, 它们要么不交, 要么恰有两个公共元, 如果是后一种情况, 则记 $A \\sim B$.\n可以证明: 对三个子集 $A, B, C$, 若 $A \\sim B, B \\sim C$, 则 $A \\sim C$. 事实上, 设 $A=\\{a, b, c\\}, B=\\{a, b, d\\}$, 因为 $C$ 与 $B$ 有两个公共元素, 故 $C \\cap\\{a$, $b\\} \\neq \\varnothing$, 即 $C$ 与 $A$ 相交, 有 $A \\sim C$. 于是所有给定的 3 元子集可以分为若干类,使同一类中任意两个子集恰有两个公共元, 而不同类的两个子集不交.\n对每个类, 考虑在它的所有子集中出现的不同元素的总个数, 有三种情形:\n(1)总共出现 3 个元素;\n(2) 总共出现 4 个元素;\n(3)总共出现不少于 5 个元素.\n在情形 (1)下, 该类恰由一个 3 元子集组成.\n在情形 (2)下, 类中至多只有 4 个不同可能的子集.\n考虑情形 (3) : 设 $A=\\{a, b, c\\}, B=\\{a, b, d\\}$ 是类中的两个子集, 则类中还有某个集合 $C$, 它含有除 $a, b, c, d$ 外的另一元素 $e$, 结合 $A \\sim C, B \\sim C$ 可知 $C=\\{a, b, e\\}$. 对类中任意别的子集 $D$, 由 $A \\sim D, B \\sim D, C \\sim D$ 可知 $\\{a, b\\} \\subseteq D$. 于是类中子集个数比类中元素个数少 2(因为除 $a, b$ 外每个元素恰好对应它所属的子集).\n以上每种情形中, 每类的子集个数不超过该类中所出现的元素个数, 但题中所给的子集总数大于元素总数,矛盾! 故命题得证.", + "remark": "", + "figures": [] +} \ No newline at end of file diff --git a/processed_dataset/proof/0801.json b/processed_dataset/proof/0801.json new file mode 100644 index 0000000000000000000000000000000000000000..1de649a88004945e752fe8e2834a53de4c86fa71 --- /dev/null +++ b/processed_dataset/proof/0801.json @@ -0,0 +1,8 @@ +{ + "source_file": "./raw_volume-zh/volume14/exercise2.tex", + "problem_type": "proof", + "problem": "问题8. 设 $a_1, a_2, \\cdots$ 为全体正整数的一个排列.\n证明: 存在无穷多个正整数 $i$, 使得 $\\left(a_i, a_{i+1}\\right) \\leqslant \\frac{3}{4} i$.", + "solution": "假设结论不成立, 则存在 $i_0$, 当 $i>i_0$ 时, $\\left(a_i, a_{i+1}\\right)>\\frac{3}{4} i$.\n取定一个正整数 $M\\left(M>i_0\\right)$. 当 $i \\geqslant 4 M$ 时, 有 $\\left(a_i, a_{i+1}\\right)>\\frac{3}{4} i \\geqslant 3 M$. 从而, $a_i \\geqslant\\left(a_i, a_{i+1}\\right)>3 M$.\n由于 $a_1, a_2, \\cdots$ 是正整数的一个排列, 则 $\\{1,2, \\cdots, 3 M\\} \\subseteq\\left\\{a_1, a_2, \\cdots\\right.$, $\\left.a_{4 M-1}\\right\\}$, 故\n$$\n\\left|\\{1,2, \\cdots, 3 M\\} \\cap\\left\\{a_{2 M}, a_{2 M+1}, \\cdots, a_{4 M-1}\\right\\}\\right| \\geqslant 3 M-(2 M-1)=M+1 .\n$$\n由抽屉原理知, 存在 $j_0\\left(2 M \\leqslant j_0<4 M-1\\right)$, 使得 $a_{j_0}, a_{j_0+1} \\leqslant 3 M$. 故\n$$\n\\left(a_{j_0}, a_{j_0+1}\\right) \\leqslant \\frac{1}{2} \\max \\left\\{a_{j_0}, a_{j_0+1}\\right\\} \\leqslant \\frac{3 M}{2}=\\frac{3}{4} \\times 2 M \\leqslant \\frac{3}{4} j_0,\n$$\n矛盾.\n所以,存在无穷多个 $i$, 使得 $\\left(a_i, a_{i+1}\\right) \\leqslant \\frac{3}{4} i$.", + "remark": "", + "figures": [] +} \ No newline at end of file diff --git a/processed_dataset/proof/0802.json b/processed_dataset/proof/0802.json new file mode 100644 index 0000000000000000000000000000000000000000..c8eda71cd4d9a6df3a21c7a34da2a6c3a3e29f2e --- /dev/null +++ b/processed_dataset/proof/0802.json @@ -0,0 +1,8 @@ +{ + "source_file": "./raw_volume-zh/volume14/exercise20.tex", + "problem_type": "proof", + "problem": "问题1. 证明: 任何一群人中, 至少有两个人, 它们的朋友数目相同.", + "solution": "设任意给定的一群人有 $n$ 个.\n用顶点表示这 $n$ 个人.\n当且仅当顶点 $u$, $v$ 表示的两个人是朋友时令 $u, v$ 相邻,得到 $n$ 个顶点的简单图 $G$.\n$G$ 中任意顶点 $x$ 的度 $d(x)$ 满足 $0 \\leqslant d(x) \\leqslant n-1$. 如果图 $G$ 的顶点的度都不相同, 则图 $G$ 具有 0 度顶点 $u$ 和 $n-1$ 度顶点 $v . n-1$ 度顶点和 $G$ 中其他顶点都相邻, 特别地和 0 度顶点 $u$ 相邻, 矛盾.\n这就证明了 $G$ 中必定有两个顶点, 它们的度相同.\n也就是说, 这群人中必有两个人, 他们的朋友一样多.", + "remark": "", + "figures": [] +} \ No newline at end of file diff --git a/processed_dataset/proof/0803.json b/processed_dataset/proof/0803.json new file mode 100644 index 0000000000000000000000000000000000000000..ce35a2ee316dfa86dfa368917ddd7d8e8999b955 --- /dev/null +++ b/processed_dataset/proof/0803.json @@ -0,0 +1,8 @@ +{ + "source_file": "./raw_volume-zh/volume14/exercise20.tex", + "problem_type": "proof", + "problem": "问题2. 㩊是否存在这样的多面体, 它有奇数个面, 每个面有奇数条棱?", + "solution": "不存在这样的多面体.\n事实上, 如果这样的多面体存在, 那么用顶点表示这个多面体的面, 并且仅当 $v_i, v_j$. 所代表的两个面有公共棱时, 在图 $G$ 相应的两顶点之间连一条边, 依题意, 每个 $d(v)$ 是奇数, 于是奇数个顶点度数之和也是奇数,这与定理相违.\n证毕.", + "remark": "", + "figures": [] +} \ No newline at end of file diff --git a/processed_dataset/proof/0804.json b/processed_dataset/proof/0804.json new file mode 100644 index 0000000000000000000000000000000000000000..01e0da5a4bd738d797ef21c5db20a3470de4828b --- /dev/null +++ b/processed_dataset/proof/0804.json @@ -0,0 +1,8 @@ +{ + "source_file": "./raw_volume-zh/volume14/exercise20.tex", + "problem_type": "proof", + "problem": "问题4. 证明: 在任何 5 个无理数中, 总可以选出 3 个数, 使它们两两之和为无理数.", + "solution": "将 5 个数看成 5 个顶点, 当且仅当两数之和为有理数时, 令相应的两个顶点相邻, 得一个简单图 $G$, 只需证明 $G$ 中存在 3 个顶点两两不相邻即可.\n首先说明图中不存在三角形 (即长为 3 的圈). 事实上, 若顶点 $x, y, z$ 两两相邻, 那么 $x+y, y+z, z+x$ 都是有理数, 从而 $x, y, z$ 都是有理数, 与已知矛盾.\n同理可知图中无五边形 (即长为 5 的圈).\n若某个顶点 $x$ 与至少三个顶点 $y, z, u$ 相邻, 则 $y, z, u$ 两两不相邻 (否则将产生以 $x$ 为一个顶点的三角形), 这三个顶点即为所求.\n若某个顶点 $x$ 与至多一个顶点 $v$ 相邻, 那么由于其余三个顶点 $y, z, u$ 不构成三角形, 所以必有两点, 例如 $y, z$ 不相邻, 则 $x, y, z$ 即为所求.\n假如以上两种情况都不满足, 那么每个顶点恰好是两条边的端点, 由一笔画理论易知这个图是一个长为 5 的圈, 矛盾! 故这种情况不会发生.\n综上所述,必有三个顶点两两不相邻,故命题得证.", + "remark": "注:实际上, 这里相当于证明了: 对 5 阶完全图 $K_5$ 所有的边染 $A, B$ 两种颜色之一, 若不存在颜色 $A$ 的五边形, 则必存在同色三角形.", + "figures": [] +} \ No newline at end of file diff --git a/processed_dataset/proof/0805.json b/processed_dataset/proof/0805.json new file mode 100644 index 0000000000000000000000000000000000000000..c9e72085f492c1ee124d282292620f3ab66db0d9 --- /dev/null +++ b/processed_dataset/proof/0805.json @@ -0,0 +1,8 @@ +{ + "source_file": "./raw_volume-zh/volume14/exercise20.tex", + "problem_type": "proof", + "problem": "问题7. 某地区网球俱乐部的 20 名成员举行 14 场单打比赛,每人至少上场一次.\n证明: 必有 6 场比赛,其中 12 个参赛者各不相同.", + "solution": "用 20 个顶点 $v_1, v_2, \\cdots, v_{20}$ 代表 20 名成员, 两名选手比赛过, 则在相应顶点间连一条边, 得图 $G$.\n设各顶点度数为 $d_i, i=1,2, \\cdots, 20$, 由题意得 $d_i \\geqslant 1$, 又图 $G$ 中有 14 条边, 因此有 $d_1+d_2+\\cdots+d_{20}=2 \\times 14=28$.\n在每个顶点 $v_i$ 处抹去 $d_i-1$ 条边, 至多抹去\n$$\n\\left(d_1-1\\right)+\\left(d_2-1\\right)+\\cdots+\\left(d_{20}-1\\right)=8\n$$\n条边.\n此时所得的图 $G^{\\prime}$ 中至少还有 $14-8=6$ 条边, 并且 $G^{\\prime}$ 中每个顶点的度数至多是 1 . 因此这 6 条边的 12 个端点各不相同, 从而相应的 6 场比赛的 12 名参赛者各不相同.", + "remark": "", + "figures": [] +} \ No newline at end of file diff --git a/processed_dataset/proof/0806.json b/processed_dataset/proof/0806.json new file mode 100644 index 0000000000000000000000000000000000000000..2b6e4bfc9c221e716d97fb0fa6321207da3881aa --- /dev/null +++ b/processed_dataset/proof/0806.json @@ -0,0 +1,8 @@ +{ + "source_file": "./raw_volume-zh/volume14/exercise20.tex", + "problem_type": "proof", + "problem": "问题8. $n$ 项正整数列 $x_1, x_2, \\cdots, x_n$ 的各项之和为 2009 , 如果这 $n$ 个数既可分为和相等的 41 个组, 又可分为和相等的 49 个组, 求 $n$ 的最小值.", + "solution": "设数列的 $n$ 项可分成每组之和为 49 的 41 个组 $u_1, u_2, \\cdots, u_{41}$, 又可分成每组之和为 41 的 49 个组 $v_1, v_2, \\cdots, v_{49}$. 把 $u_i(1 \\leqslant i \\leqslant 41)$ 和 $v_j(1 \\leqslant j \\leqslant 49)$ 看成顶点, 如果存在 $x_k(1 \\leqslant k \\leqslant n)$ 同时属于 $u_i$ 和 $v_j$, 就在 $u_i, v_j$ 间连一条边, 得简单图 $G$. 下面证明 $G$ 是连通图.\n考虑 $G$ 的最大连通分支 $G^{\\prime}$. 设 $G^{\\prime}$ 含有 $a$ 个顶点 $u_{i_s}, s=1,2, \\cdots, a$ 和 $b$ 个顶点 $v_{j_t}, t=1,2, \\cdots, b$, 则所有出现在某个 $u_{i_s}(1 \\leqslant s \\leqslant a)$ 中的项 $x_k$ 必同时出现在某个 $v_{j_t}(1 \\leqslant t \\leqslant b)$ 中 (否则 $G$ 中将有比 $G^{\\prime}$ 更大的连通分支), 反之亦然.\n这些数的总和一方面等于 $49 a$,一方面又等于 $41 b$, 故 $49 a=41 b$, 所以 $a+b \\geqslant 41+49=90$, 只能 $G^{\\prime}=G$. 这样 $G$ 至少有 89 条边, 又数列的项数不少于 $G$ 的边数, 故 $n \\geqslant 89$.\n另一方面, 当 $n=89$ 时, 若数列满足\n$$\nx_1=x_2=\\cdots=x_{41}=41, x_{42}=x_{43}=\\cdots=x_{81}=8, x_{82}=x_{83}=\\cdots=x_{89}=1 \\text {, }\n$$\n则容易验证满足题意.\n因此 $n$ 的最小值为 89 .", + "remark": "", + "figures": [] +} \ No newline at end of file diff --git a/processed_dataset/proof/0807.json b/processed_dataset/proof/0807.json new file mode 100644 index 0000000000000000000000000000000000000000..8eb8eb34bee2d0a472c336a03343ea905dbd0e1c --- /dev/null +++ b/processed_dataset/proof/0807.json @@ -0,0 +1,8 @@ +{ + "source_file": "./raw_volume-zh/volume14/exercise20.tex", + "problem_type": "proof", + "problem": "问题9. 今有 10 个互不相同的非零数, 它们之中任意两个数的和与积中, 至少有一个是有理数.\n证明: 每个数的平方都是有理数.", + "solution": "首先注意到, 只要存在某个 $x \\in \\mathbf{Q}$, 则对其余任意数 $t$, 由 $x+t$ 与 $x t$ 之一为有理数即可推知 $t \\in \\mathbf{Q}$ (注意这些数均非零), 从而 10 个数均为有理数.\n作一个 2 色完全图 $K_{10}$, 在它的 10 个顶点上分别放上这 10 个数.\n如果某两数的和为有理数, 就在相应的顶点间连一条蓝边, 否则, 这两数的积必为有理数,那么连一条红边.\n由 Ramsey 定理得, 该图中必有同色三角形.\n(1)若存在蓝色三角形,则表明存在 3 个数 $x, y, z$ 两两之和为有理数, 故 $x, y, z \\in \\mathbf{Q}$, 从而 10 个数均为有理数.\n(2)若存在红色三角形,则表明存在 3 个数 $x, y, z$ 两两之积为有理数, 因而 $x^2=\\frac{x y \\cdot x z}{y z} \\in \\mathbf{Q}$. 设 $x=m \\sqrt{a}$, 其中 $a \\in \\mathbf{Q}, m= \\pm 1$. 由于 $x y= m \\sqrt{a} y=b \\in \\mathbf{Q}$, 所以 $y=\\frac{b \\sqrt{a}}{m a}=c \\sqrt{a}$, 其中 $c \\in \\mathbf{Q}, c \\neq m$. 对其余任意数 $t$, 如果 $x t$ 或 $y t$ 为有理数, 那么经过类似的讨论, 可知 $t=d \\sqrt{a}$, 其中 $d \\in \\mathbf{Q}$, 因而 $t^2 \\in \\mathbf{Q}$. 反之, 如果 $x t, y t \\notin \\mathbf{Q}$, 则 $x+t, y+t \\in \\mathbf{Q}$, 但 $(x+t)-(y+t)= (m-c) \\sqrt{a} \\notin \\mathbf{Q}$,矛盾.\n综上, 我们证明了或者每个数都是有理数, 或者每个数的平方都是有理数, 这正是所要证明的.", + "remark": "", + "figures": [] +} \ No newline at end of file diff --git a/processed_dataset/proof/0808.json b/processed_dataset/proof/0808.json new file mode 100644 index 0000000000000000000000000000000000000000..e9d6de66f1c6248a2db7c4d8e909a98dcc1b0cf4 --- /dev/null +++ b/processed_dataset/proof/0808.json @@ -0,0 +1,8 @@ +{ + "source_file": "./raw_volume-zh/volume14/exercise20.tex", + "problem_type": "proof", + "problem": "问题10. 已知 $X$ 为 $n$ 元集合.\n设 $X$ 的每个有序元素对 $(x, y)(x, y \\in X)$ 对应一个数 $f(x, y)=0$ 或 1 , 且对一切 $x, y \\in X, x \\neq y$, 恒有 $f(x, y) \\neq f(y$, $x$ ). 求证:下述两种情况恰有一种出现:\n(1) $X$ 是两个不交的非空集 $A, B$ 之并, 且对一切 $x \\in A, y \\in B$, 有 $f(x$, $y)=1$;\n(2) $X$ 的元素可排成序列 $x_1, x_2, \\cdots, x_n$, 使得对 $i=1,2, \\cdots, n$, 有 $f\\left(x_i, x_{i+1}\\right)=1$, 其中 $x_{n+1}=x_1$.", + "solution": "将 $X$ 的 $n$ 个元素视作顶点, 对任意 $x, y \\in X, x \\neq y$, 若 $f(x, y)=$ 1 , 则作一条有向边 $x \\rightarrow y$, 得有向图 $G$. 由已知条件得, 图 $G$ 的任意两个顶点 $x, y$ 之间恰有一条有向边相连 (这样的图 $G$ 是 $n$ 阶有向完全图 $\\overline{K_n}$, 称作\"竞赛图\").\n情况 (1)表明不存在从顶点 $y \\in B$ 到顶点 $x \\in A$ 的有向边,情况(2)表明可以沿有向边遍历 $X$ 各点恰好一次, 即 $G$ 存在有向哈密顿圈.\n显然两者不能同时成立.\n假设上述情况 (1)不成立, 我们证明情况 (2)必成立.\n任取一点 $x \\in X$, 记 $P=\\{a \\in X \\mid x \\rightarrow a\\}, Q=\\{a \\in X \\mid a \\rightarrow x\\}$. 因情况 (1) 不成立, 故 $P, Q$ 均为非空集, 且存在 $y \\in P, z \\in Q$, 使 $y \\rightarrow z$, 这表明图 $G$ 中存在圈 $x \\rightarrow y \\rightarrow z \\rightarrow x$.\n设 $x_1 \\rightarrow x_2 \\rightarrow \\cdots \\rightarrow x_k \\rightarrow x_1$ 是图 $G$ 中的圈.\n下面证明, 若 $k0$, 且 $\\sum_{i=1}^n a_i^3=\\left(\\sum_{i=1}^n a_i\\right)^2$, 求证: $a_n=n$.", + "solution": "对 $n$ 用数学归纳法.\n当 $n=1$ 时, $a_1^3=a_1^2$, 故 $a_1=1$. 即 $n=1$ 时命题成立.\n假设 $n \\leqslant k$ 时命题成立, 即 $a_1=1, a_2=2, \\cdots, a_k=k$, 则由题设\n$$\n1^3+2^3+\\cdots+k^3+a_{k+1}^3=\\left(1+2+\\cdots+k+a_{k+1}\\right)^2,\n$$\n所以\n$$\na_{k+1}\\left(a_{k+1}^2-a_{k+1}-k(k+1)\\right)=0,\n$$\n考虑到 $a_{k+1}>0$, 从而 $a_{k+1}=k+1$, 故命题对 $n=k+1$ 也成立.", + "remark": "", + "figures": [] +} \ No newline at end of file diff --git a/processed_dataset/proof/0810.json b/processed_dataset/proof/0810.json new file mode 100644 index 0000000000000000000000000000000000000000..58e987a272accafea9b15dbebb2784a9517e0760 --- /dev/null +++ b/processed_dataset/proof/0810.json @@ -0,0 +1,8 @@ +{ + "source_file": "./raw_volume-zh/volume14/exercise3.tex", + "problem_type": "proof", + "problem": "问题2. 数列 $\\left\\{a_n\\right\\}$ 定义如下: $a_1=3, a_{n+1}=3^{a_n}, n=1,2, \\cdots$, 数列 $\\left\\{b_n\\right\\}$ 定义如下: $b_1=8, b_{n+1}=8^{b_n}, n=1,2, \\cdots$. 求证: 对一切正整数 $n$, 有 $a_{n+1}>b_n$.", + "solution": "下面证明加强的命题: $a_{n+1}>3 b_n$.\n当 $n=1$ 时, $a_2=3^3=27>24=3 b_1$, 即 $n=1$ 时命题成立.\n假设 $n=k$ 时命题成立, 即 $a_{k+1}>3 b_k$, 则\n$$\na_{k+2}=3^{a_{k+1}}>3^{3 b_k}=27^{b_k}>2^{b_k} \\cdot 8^{b_k} \\geqslant 2 b_{k+1},\n$$\n故由数学归纳法知, 加强的命题成立.", + "remark": "", + "figures": [] +} \ No newline at end of file diff --git a/processed_dataset/proof/0811.json b/processed_dataset/proof/0811.json new file mode 100644 index 0000000000000000000000000000000000000000..d602fbc3a3da39bd60fa3e4f11c900af93804c9f --- /dev/null +++ b/processed_dataset/proof/0811.json @@ -0,0 +1,10 @@ +{ + "source_file": "./raw_volume-zh/volume14/exercise3.tex", + "problem_type": "proof", + "problem": "问题3. 设 $n$ 为不小于 6 的整数,证明: 可将一个正方形分成 $n$ 个较小的正方形.", + "solution": "因为一个正方形可以等分为 4 个小正方形, 因此要将小正方形的数目增加 3 个是容易做到的,所以我们采用步长为 3 .\n当 $n=6,7,8$ 时, 可按图 () 所示方式进行分割, 所以知命题成立.\n假设对某个 $n=k \\geqslant 6$, 已将正方形分为 $k$ 个小正方形, 那么只要在将其中一个小正方形等分为 4 个更小的正方形, 即可得到 $n=k+3$ 个小正方形.\n所以知命题对一切整数 $n \\geqslant 6$ 都成立.", + "remark": "", + "figures": [ + "./images/volume14/figures/fig-c3a3.png" + ] +} \ No newline at end of file diff --git a/processed_dataset/proof/0812.json b/processed_dataset/proof/0812.json new file mode 100644 index 0000000000000000000000000000000000000000..a9b25d738863830299f940ce84270d6b85e2a192 --- /dev/null +++ b/processed_dataset/proof/0812.json @@ -0,0 +1,8 @@ +{ + "source_file": "./raw_volume-zh/volume14/exercise3.tex", + "problem_type": "proof", + "problem": "问题4. 设 $M=2^{n_1}+2^{n_2}+\\cdots+2^{n_s}, n_1, n_2, \\cdots, n_s$ 是互不相同的正整数,求证:\n$$\n2^{\\frac{n_1}{2}}+2^{\\frac{n_2}{2}}+\\cdots+2^{\\frac{n_s}{2}}<(1+\\sqrt{2}) \\sqrt{M} .\n$$", + "solution": "对 $s$ 用数学归纳法.\n(1) 当 $s=1$ 时,结论显然成立.\n(2)假设结论对 $s=k$ 时成立, 则 $s=k+1$ 时, 不妨设 $n_1>n_2>\\cdots> n_k>n_{k+1}$. 对 $n_2, n_3, \\cdots, n_{k+1}$, 由归纳假设可知\n$$\n2^{\\frac{n_2}{2}}+2^{\\frac{n_3}{2}}+\\cdots+2^{\\frac{n_{k+1}}{2}}<(1+\\sqrt{2}) \\sqrt{M-2^{n_1}},\n$$\n则\n$$\n2^{\\frac{n_1}{2}}+2^{\\frac{n_2}{2}}+\\cdots+2^{\\frac{n_{k+1}}{2}}<(1+\\sqrt{2}) \\sqrt{M-2^{n_1}}+2^{\\frac{n_1}{2}} .\n$$\n以下只需证明\n$$\n(1+\\sqrt{2}) \\sqrt{M-2^{n_1}}+2^{\\frac{n_1}{2}}<(1+\\sqrt{2}) \\cdot \\sqrt{M} .\n$$\n由于 $M=2^{n_1}+2^{n_2}+\\cdots+2^{n_{k+1}} \\leqslant 2^{n_1}+2^{n_1-1}+\\cdots+2^{n_1-k}<2 \\times 2^{n_1}, M- 2^{n_1}<2^{n_1}$, 故 $\\sqrt{M-2^{n_1}}+\\sqrt{M}<(1+\\sqrt{2}) \\cdot 2^{\\frac{n_1}{2}}$, 两边乘以 $\\sqrt{M}-\\sqrt{M-2^{n_1}}$ 得\n$$\nM-\\left(M-2^{n_1}\\right)<(1+\\sqrt{2}) \\cdot 2^{\\frac{n_1}{2}}\\left(\\sqrt{M}-\\sqrt{M-2^{n_1}}\\right),\n$$\n即 $2^{\\frac{n_1}{2}}<(1+\\sqrt{2})\\left(\\sqrt{M}-\\sqrt{M-2^{n_1}}\\right)$, 此即(1). 从而 $s=k+1$ 时结论成立.\n因此, 由数学归纳法知, 命题对任意正整数 $s$ 均成立.", + "remark": "", + "figures": [] +} \ No newline at end of file diff --git a/processed_dataset/proof/0813.json b/processed_dataset/proof/0813.json new file mode 100644 index 0000000000000000000000000000000000000000..30b954d56778bedeb8694838d8d8d9c6ad894a7a --- /dev/null +++ b/processed_dataset/proof/0813.json @@ -0,0 +1,8 @@ +{ + "source_file": "./raw_volume-zh/volume14/exercise3.tex", + "problem_type": "proof", + "problem": "问题5. 已知数列 $\\left\\{a_n\\right\\}$ 中每项都是正整数且逐项递增, $a_2=2$. 若对任意 $m, n \\in\\mathbf{N}^*$, 有 $a_{m n}=a_m a_n$, 证明: $a_n=n$.", + "solution": "用反向数学归纳法.\n先证明对任意 $n=2^k\\left(k \\in \\mathbf{N}^*\\right)$, 有 $a_n=n$.\n$n=2$ 时结论显然成立.\n假设 $n=2^k$ 时成立, 即 $a_{2^k}=2^k$, 那么在已知条件中令 $m=2, n=2^k$ 得: $a_{2^{k+1}}=a_2 a_{2^k}=2 \\times 2^k=2^{k+1}$. 故 $n=2^{k+1}$ 时也成立.\n所以 $a_{2^k}=2^k$ 对任意正整数 $k$ 成立.\n再证明对任意正整数 $n \\geqslant 2$, 若 $a_n=n$, 则当 $1 \\leqslant k2)$, 则 $1+2 \\cdot a \\in A, 1+2 \\cdot(1+2 a)=3+4 a \\in A$,\n$$\n1+(1+2 a) \\cdot(3+4 a)=4+10 a+8 a^2 \\in A,\n$$\n若 $a$ 是偶数,则 $4 \\mid\\left(4+10 a+8 a^2\\right)$, 于是 $4 \\in A$; 若 $a$ 是奇数,则把 $4+10 a+ 8 a^2$ 作为 $a$, 重复上面的过程可得 $4 \\in A$.\n又 $1+2 \\times 4=9 \\in A$, 所以 $3 \\in A, 1+2 \\times 3=7 \\in A, 1+2 \\times 7=15 \\in A$, 所以 $5 \\in A$.\n所以, 1, 2, 3, 4, 5 都是集合 $A$ 的元素.\n假设 $1,2, \\cdots, n \\in A(n \\geqslant 5)$, 下证 $n+1 \\in A$.\n如果 $n+1=2 k+1$ 为奇数, 那么 $3 \\leqslant k2$.\n不妨设物理是一门选修课且某对室友一个选修了物理, 一个没选修物理.\n我们将所有选修物理的学生集合记为 $A$, 没选修物理的学生集合记为 $B$, 则 $|A|=|B|=2^{n-1}$. 我们将 $A$ 和 $B$ 中的所有同学都各自安排到另外有 $2^{n-2}$ 间宿舍的学校, 原来的室友还是室友, 剩下的 $A$ 中任意两两配对成为室友 (称之为新室友). 由归纳假设, $A$ 中学生可以排成一圈 $K$ 满足要求.\n设 $\\left(x_1, x_2\\right), \\cdots$, $\\left(x_{2 k-1}, x_{2 k}\\right)$ 是 $K$ 上按顺时针方向排列的所有新室友对.\n令 $x_i^{\\prime}$ 表示 $x_i$ 原来的室友, 显然 $x_i^{\\prime} \\in B$. 在 $B$ 中令 $\\left(x_2^{\\prime}, x_3^{\\prime}\\right), \\cdots,\\left(x_{2 k-2}^{\\prime}, x_{2 k-1}^{\\prime}\\right),\\left(x_{2 k}^{\\prime}, x_1^{\\prime}\\right)$ 构成新的室友对集, 与 $B$ 中原来的室友对集共同构成了 $B$ 中的室友对集.\n对 $B$ 应用归纳假设, 可将 $B$ 中同学排成一圈 $K^{\\prime}$. 我们将 $K$ 中 $x_{2 i}$ 到 $x_{2 i+1}$ 之间的同学保持顺序地插人到 $K^{\\prime}$ 中 $x_{2 i}^{\\prime}, x_{2 i+1}^{\\prime}$ 之间, 得到新的大圈满足要求.", + "remark": "", + "figures": [] +} \ No newline at end of file diff --git a/processed_dataset/proof/0819.json b/processed_dataset/proof/0819.json new file mode 100644 index 0000000000000000000000000000000000000000..289905b97b83a3605011f38bfddd782cfc52d8e9 --- /dev/null +++ b/processed_dataset/proof/0819.json @@ -0,0 +1,8 @@ +{ + "source_file": "./raw_volume-zh/volume14/exercise4.tex", + "problem_type": "proof", + "problem": "问题1. 从 $1,2, \\cdots, 2 n$ 这 $2 n$ 个正整数中任取 $n+1$ 个数, 证明其中一定存在两个数是互素的.", + "solution": "把 $1,2, \\cdots, 2 n$ 这 $2 n$ 个正整数分成如下 $n$ 组:\n$$\n\\{1,2\\},\\{3,4\\}, \\cdots,\\{2 n-1,2 n\\} .\n$$\n从这 $n$ 组中任取 $n+1$ 个数, 由抽屉原理知, 其中一定有两个数取自同一组, 同一组中的两个数是相邻的正整数, 从而它们是互素的.", + "remark": "", + "figures": [] +} \ No newline at end of file diff --git a/processed_dataset/proof/0820.json b/processed_dataset/proof/0820.json new file mode 100644 index 0000000000000000000000000000000000000000..3ce46bc3916bafddef9bd2ad3c536b936a95fa8d --- /dev/null +++ b/processed_dataset/proof/0820.json @@ -0,0 +1,8 @@ +{ + "source_file": "./raw_volume-zh/volume14/exercise4.tex", + "problem_type": "proof", + "problem": "问题2. 在 $1,2,3, \\cdots, 100$ 这 100 个正整数中任取 11 个数,证明其中一定有两个数的比值不超过 $\\frac{3}{2}$.", + "solution": "把 $1,2, \\cdots, 100$ 这 100 个数分成如下 10 组 (10 个抽屉), 每一组中的最大数与最小数之比不超过 $\\frac{3}{2}$ :\n$$\n\\begin{aligned}\n& \\{1\\},\\{2,3\\},\\{4,5,6\\},\\{7,8,9,10\\},\\{11,12, \\cdots, 16\\},\\{17,18, \\cdots, 25\\}, \\\\\n& \\{26,27, \\cdots, 39\\},\\{40,41, \\cdots, 60\\},\\{61,62, \\cdots, 91\\},\\{92,93, \\cdots, 100\\} .\n\\end{aligned}\n$$\n从 $1,2, \\cdots, 100$ 中任取 11 个数, 即是从上面 10 组中任取 11 个数, 由抽庶原理知, 其中一定有两个数取自同一组, 这两个数的比值不超过 $\\frac{3}{2}$.", + "remark": "", + "figures": [] +} \ No newline at end of file diff --git a/processed_dataset/proof/0821.json b/processed_dataset/proof/0821.json new file mode 100644 index 0000000000000000000000000000000000000000..0c989f30b3fb1a2facc71b03ccbd265fb99333c1 --- /dev/null +++ b/processed_dataset/proof/0821.json @@ -0,0 +1,8 @@ +{ + "source_file": "./raw_volume-zh/volume14/exercise4.tex", + "problem_type": "proof", + "problem": "问题3. 任给 7 个实数, 求证: 其中必有两个数 $x, y$, 满足 $0 \\leqslant \\frac{x-y}{1+x y}<\\frac{\\sqrt{3}}{3}$.", + "solution": "不妨设这 7 个实数分别为 $\\tan \\alpha_i, i=1,2, \\cdots, 7$, 其中\n$$\n-\\frac{\\pi}{2}<\\alpha_1 \\leqslant \\alpha_2 \\leqslant \\cdots \\leqslant \\alpha_7<\\frac{\\pi}{2} .\n$$\n将区间 $\\left[\\alpha_1, \\alpha_7\\right.$ 平均分成 6 个区间, 根据抽屉原理知, 在 $\\alpha_i(1 \\leqslant i \\leqslant 7)$ 中必有两个角 $\\theta_1, \\theta_2\\left(\\theta_1 \\leqslant \\theta_2\\right)$ 同属于一个区间, 则 $0 \\leqslant \\theta_2-\\theta_1 \\leqslant \\frac{\\alpha_7-\\alpha_1}{6}<\\frac{\\pi}{6}$.\n取 $x=\\tan \\theta_2, y=\\tan \\theta_1$, 则 $x, y$ 满足\n$$\n0 \\leqslant \\tan \\left(\\theta_2-\\theta_1\\right)=\\frac{\\tan \\theta_2-\\tan \\theta_1}{1+\\tan \\theta_2 \\tan \\theta_1}=\\frac{x-y}{1+x y}<\\tan \\frac{\\pi}{6}=\\frac{\\sqrt{3}}{3} .\n$$", + "remark": "", + "figures": [] +} \ No newline at end of file diff --git a/processed_dataset/proof/0822.json b/processed_dataset/proof/0822.json new file mode 100644 index 0000000000000000000000000000000000000000..520a034d42b04beadb6792b0a760d813d833eb65 --- /dev/null +++ b/processed_dataset/proof/0822.json @@ -0,0 +1,10 @@ +{ + "source_file": "./raw_volume-zh/volume14/exercise4.tex", + "problem_type": "proof", + "problem": "问题4. 在 $3 \\times 4$ 的长方形中, 任意放置 6 个点, 证明:一定可以找到两个点, 它们的距离不大于 $\\sqrt{5}$.", + "solution": "如图()所示, 把 $3 \\times 4$ 的矩形分成如下 5 个部分, 由勾股定理可以算得每个部分的任两点之间的距离不大于 $\\sqrt{5}$. 从而命题得证.", + "remark": "", + "figures": [ + "./images/volume14/figures/fig-c4a4.png" + ] +} \ No newline at end of file diff --git a/processed_dataset/proof/0823.json b/processed_dataset/proof/0823.json new file mode 100644 index 0000000000000000000000000000000000000000..e6f758ffee81516ea5dfbc8d3fb610fd4a24b7c4 --- /dev/null +++ b/processed_dataset/proof/0823.json @@ -0,0 +1,8 @@ +{ + "source_file": "./raw_volume-zh/volume14/exercise4.tex", + "problem_type": "proof", + "problem": "问题7. 在 $50 \\times 50$ 表格中, 每格写有一个正整数,使整个表格中 $1,2, \\cdots, 50$ 各出现 50 次.\n证明: 存在某行或某列至少包含 8 个不同数.", + "solution": "记 $M=\\{1,2, \\cdots, 50\\}$.\n引理: 每个 $i \\in M$ 至少出现在 15 个行与列中.\n证明: 设 $i \\in M$ 出现在 $x$ 个行中, 由抽屉原理, 有某一行出现了至少 $\\left\\lceil\\frac{50}{x}\\right\\rceil$ 次, 从而含 $i$ 的行数与列数之和 $S_i \\geqslant x+\\frac{50}{x} \\geqslant 2 \\sqrt{50}>14$, 故引理成立.\n以下对每个 $i \\in M$ 标出有它出现的一切行与列, 当 $i$ 取遍 $M$ 中元素时, 至少标出了 $50 \\times 15=750$ 个行与列, 但表格只有 $50+50=100$ 个行与列, 从而根据抽㞕原理, 必有某个行或列被标了不少于 8 次, 即它包含了至少 8 个不同的数.", + "remark": "", + "figures": [] +} \ No newline at end of file diff --git a/processed_dataset/proof/0824.json b/processed_dataset/proof/0824.json new file mode 100644 index 0000000000000000000000000000000000000000..93234be5d7c09cda1bc7908ab65f2892cc067582 --- /dev/null +++ b/processed_dataset/proof/0824.json @@ -0,0 +1,8 @@ +{ + "source_file": "./raw_volume-zh/volume14/exercise4.tex", + "problem_type": "proof", + "problem": "问题8. 整设正整数构成的数列 $\\left\\{a_n\\right\\}$ 使得\n$$\na_{10 k-9}+a_{10 k-8}+\\cdots+a_{10 k} \\leqslant 19\n$$\n对一切 $k \\in \\mathbf{N}^*$ 恒成立.\n记该数列若干连续项的和 $\\sum_{p=i+1}^j a_p$ 为 $S(i, j)$, 其中 $i, j \\in \\mathbf{N}^*$ 且 $i0$, 则称为一组点.\n将半圆分为 $N^2+1$ 段弧 $L_i\\left(1 \\leqslant i \\leqslant N^2+1\\right)$. 令 $a$ 充分小, 则可在每段弧 $L_i$ 上取一组点, 由抽屉原理, 这组点中必有两点同色, 且可用数组 $\\left(c_i, l_i\\right)$ 表示, 其中, $c_i$ 代表这两点的 $N$ 种颜色之一, $l_i$ 代表这两点对应的 $N$ 种可能的弧长之一.\n不同的数组 $\\left(c_i, l_i\\right)$ 只有 $N^2$ 种, 由抽屉原理, 一定存在 $i \\neq j$, 使 $\\left(c_i, l_i\\right)=\\left(c_j, l_j\\right)$, 于是这两组中存在四点同色,且构成等腰梯形.", + "remark": "", + "figures": [] +} \ No newline at end of file diff --git a/processed_dataset/proof/0827.json b/processed_dataset/proof/0827.json new file mode 100644 index 0000000000000000000000000000000000000000..9ae27affbe9804ff0efd0b9767b819b75ac7c3c0 --- /dev/null +++ b/processed_dataset/proof/0827.json @@ -0,0 +1,8 @@ +{ + "source_file": "./raw_volume-zh/volume14/exercise5.tex", + "problem_type": "proof", + "problem": "问题5. 在面积为 1001 的区域中,有 2001 个面积为 1 的区域 $S_i(1 \\leqslant i \\leqslant 2001)$. 求证,必有两个区域 $S_i, S_j(1 \\leqslant i1$, 且满足 $\\frac{1}{a}+\\frac{1}{b}+\\frac{1}{c}>1$. 定义集合 $S(x)=\\{[k x] \\mid k \\in \\mathbf{N}^*$ \\}.证明: 在 $S(a), S(b), S(c)$ 中必有两个的交集是无限集.", + "solution": "记 $I_n=\\{1,2, \\cdots, n\\}$. 我们将 $I_n$ 看成全集, 对集合\n$$\nA_n=S(a) \\cap I_n, B_n=S(b) \\cap I_n, C_n=S(c) \\bigcap I_n\n$$\n用容厉原理, 可得\n$$\n\\begin{aligned}\n\\left|A_n \\cup B_n \\cup C_n\\right|= & \\left|A_n\\right|+\\left|B_n\\right|+\\left|C_n\\right|-\\left|A_n \\cap B_n\\right|- \\\\\n& \\left|B_n \\cap C_n\\right|-\\left|C_n \\cap A_n\\right|+\\left|A_n \\cap B_n \\cap C_n\\right| .\n\\end{aligned}\n$$\n上式中有 $\\left|A_n \\cup B_n \\cup C_n\\right| \\leqslant n,\\left|A_n \\cap B_n \\cap C_n\\right| \\geqslant 0$, 又注意到\n$$\n\\left|A_n\\right| \\geqslant\\left[\\frac{n}{a}\\right]>\\frac{n}{a}-1,\\left|B_n\\right| \\geqslant\\left[\\frac{n}{b}\\right]>\\frac{n}{b}-1,\\left|C_n\\right| \\geqslant\\left[\\frac{n}{c}\\right]>\\frac{n}{c}-1,\n$$\n所以\n$$\nn>\\left(\\frac{n}{a}+\\frac{n}{b}+\\frac{n}{c}-3\\right)--\\left|A_n \\cap B_n\\right|-\\left|B_n \\cap C_n\\right|-\\left|C_n \\cap A_n\\right| .\n$$\n由此可知\n$$\n\\left|A_n \\cap B_n\\right|+\\left|B_n \\cap C_n\\right|+\\left|C_n \\cap A_n\\right|>\\left(\\frac{1}{a}+\\frac{1}{b}+\\frac{1}{c}-1\\right) n-3 .\n$$\n由于 $\\frac{1}{a}+\\frac{1}{b}+\\frac{1}{c}>1$, 故当 $n \\rightarrow \\infty$ 时上式右端可以任意大, 但对任意 $n \\in \\mathbf{N}^*$, 总有 $A_n \\subseteq S(a), B_n \\subseteq S(b), C_n \\subseteq S(c)$, 所以 $S(a), S(b), S(c)$ 中必有两个的交集为无限集.", + "remark": "", + "figures": [] +} \ No newline at end of file diff --git a/processed_dataset/proof/0830.json b/processed_dataset/proof/0830.json new file mode 100644 index 0000000000000000000000000000000000000000..6ff5d878391d1b2d7c743c27b047155d1cc51f8c --- /dev/null +++ b/processed_dataset/proof/0830.json @@ -0,0 +1,8 @@ +{ + "source_file": "./raw_volume-zh/volume14/exercise6.tex", + "problem_type": "proof", + "problem": "问题1. 在一次乒乓球循环赛中, $n(n \\geqslant 3)$ 名选手中没有全胜的.\n证明:一定可以从中找出三名选手 $A, B, C$, 有 $A$ 胜 $B, B$ 胜 $C, C$ 胜 $A$.", + "solution": "取 $n$ 个选手中所赢局数最多的一位选手作 $A$ (取极端!), 由于 $A$ 未全胜, 所以存在选手 $C$ 胜 $A$. 考虑 $A$ 击败的选手全体, 知其中必有选手 $B$ 胜 $C$ (否则, $A$ 的败将也都是 $C$ 的败将, 从而 $C$ 赢的局数超过了 $A$ ). 于是所述的 $A, B, C$ 即为所求.", + "remark": "", + "figures": [] +} \ No newline at end of file diff --git a/processed_dataset/proof/0831.json b/processed_dataset/proof/0831.json new file mode 100644 index 0000000000000000000000000000000000000000..99f5f9a03d303262444789f0578808f6dc837853 --- /dev/null +++ b/processed_dataset/proof/0831.json @@ -0,0 +1,10 @@ +{ + "source_file": "./raw_volume-zh/volume14/exercise6.tex", + "problem_type": "proof", + "problem": "问题2. 平面上给定 100 个点, 已知其中任意两点的距离不超过 1 , 且任意三点所成的三角形是钝角三角形.\n证明: 这 100 个点被一个半径为 $\\frac{1}{2}$-的圆覆盖.", + "solution": "在这 100 个点中, 每两点有一个距离, 这有限个数 $\\left(\\frac{100 \\times 99}{2} 个 \\right)$ 中, 一定有一个最大数, 不妨设 $A, B$ 两点间的距离最大, 以线段 $A B$ 为直径作一个圆, 由题设, $A B \\leqslant 1$, 所以, 此圆的半径 $\\leqslant \\frac{1}{2}$.\n在其余的 98 个点中任取一点 $C$, 因为 $A B \\geqslant A C$, $A B \\geqslant B C$, 又由题设知 $\\triangle A B C$ 是钝角三角形, 如图()所示, 所以 $\\angle C$ 一定是钝角, 于是点 $C$ 位于以 $A B$ 为直径的圆内, 也就是说这 100 个点都被这个圆所覆盖.", + "remark": "", + "figures": [ + "./images/volume14/figures/fig-c6a2.png" + ] +} \ No newline at end of file diff --git a/processed_dataset/proof/0832.json b/processed_dataset/proof/0832.json new file mode 100644 index 0000000000000000000000000000000000000000..f16eaa513f8472c33c4892755a06dc7e0a482e21 --- /dev/null +++ b/processed_dataset/proof/0832.json @@ -0,0 +1,8 @@ +{ + "source_file": "./raw_volume-zh/volume14/exercise6.tex", + "problem_type": "proof", + "problem": "问题3. 空间中给出了 8 个点,其中任意 4 个点都不在同一平面上,在它们之间连以 17 条线段.\n证明:这些线段至少形成了一个三角形.", + "solution": "每一个点都连出了若干条线段 (至多为 7 条), 不妨设连出线段数目最多的点为 $A$, 它共连出了 $n$ 条线段.\n如果所有 17 条线段都没有形成三角形, 那么与 $A$ 相连的 $n$ 个点之间彼此都没有线段相连, 而其余的 $(7-n)$ 个点中, 每一点所连出的线段条数不多于 $n$ 条,因此,线段的总数目不超过\n$$\n\\begin{aligned}\nn+(7-n) n & =-n^2+8 n \\\\\n& =-(n-4)^2+16 \\leqslant 16,\n\\end{aligned}\n$$\n这与已知的有 17 条线段矛盾.\n从而命题成立.", + "remark": "注:其实本题的结论可加强为 \"三角形的数目不少于 4 个\", 这个问题较难, 留给有兴趣的读者思考.", + "figures": [] +} \ No newline at end of file diff --git a/processed_dataset/proof/0833.json b/processed_dataset/proof/0833.json new file mode 100644 index 0000000000000000000000000000000000000000..16492eaaf36baecc10b604e90068c11ccdfc4143 --- /dev/null +++ b/processed_dataset/proof/0833.json @@ -0,0 +1,8 @@ +{ + "source_file": "./raw_volume-zh/volume14/exercise6.tex", + "problem_type": "proof", + "problem": "问题4. 若干个人聚会, 其中某些人彼此认识, 已知如果某两人在聚会者中有相同数目的熟人,那么他俩便没有共同的熟人.\n证明: 若聚会者中有人至少有 2012 个熟人,则必然也有人恰好有 2012 个熟人.", + "solution": "我们考虑 (聚会者中) 熟人最多的某个人(如果这样的人不止一个, 那么任取其中一个), 记为 $A$, 设 $A$ 共认识 $n$ 个人, 这些熟人依次记为 $B_1$, $B_2, \\cdots, B_n$.\n由于 $B_1, B_2, \\cdots, B_n$ 中任意两个人 $B_i$ 与 $B_j$ 都认识 $A$, 即是他俩的共同熟人, 因此由题设推出了 $B_i$ 与 $B_j$ 的熟人数目不等.\n此外, $B_1, B_2, \\cdots, B_n$ 的熟人数目均不会超过 $n$ (这里用到了 $n$ 的 \"最大性\"!), 于是他们的熟人数目恰好是\n$$\n1,2, \\cdots, n \\text {. }\n$$\n现在已知有人至少认识 2012 人, 这意味着 $n \\geqslant 2012$, 所以数 2012 在上述数列中出现, 于是 $B_1, B_2, \\cdots, B_n$ 中恰好有人有 2012 个熟人.", + "remark": "", + "figures": [] +} \ No newline at end of file diff --git a/processed_dataset/proof/0834.json b/processed_dataset/proof/0834.json new file mode 100644 index 0000000000000000000000000000000000000000..b29a90e4e5674b7b3d3b44105a8109e16e60994d --- /dev/null +++ b/processed_dataset/proof/0834.json @@ -0,0 +1,8 @@ +{ + "source_file": "./raw_volume-zh/volume14/exercise6.tex", + "problem_type": "proof", + "problem": "问题5. 由凸多边形内任意一点向它的各边引垂线, 证明: 至少有一个垂足在多边形的某条边内 (不含端点).", + "solution": "由于凸多边形内任意一点 $P$ 向各边所引的垂线只有有限条, 故必存在一条垂线段是其中最短的, 设为 $P Q, P Q$ 与边 $A B$ 垂直于 $Q$. 我们证明 $Q$ 必在线段 $A B$ 内.\n假设 $Q$ 不在线段 $A B$ 内, 由多边形的凸性知 $Q$ 在线段 $A B$ 端点或在多边形外,无论哪种情况,线段 $P Q$ 必与多边形的某条不是 $A B$ 的边有公共点.\n不妨设 $P Q$ 与边 $M N$ 有公共点 $R$, 此时 $P R$ 与 $M N$ 不垂直, 故 $P$ 到直线 $M N$ 的距离 $d